Location via proxy:   [ UP ]  
[Report a bug]   [Manage cookies]                

NCLEX Real Questions

Download as pdf or txt
Download as pdf or txt
You are on page 1of 1193

1 A client exposed to Mycobacterium tuberculosis Correct answer: 1 To prevent active tuberculosis after exposure, the client is initiated on a single

initiated on a single agent regimen, The critical words in the stem of the question are exposed and chemoprophylaxis.
starts on chemoprophylaxis. The nurse provides what usually isoniazid (INH). For newly diagnosed active disease (option 2), a combination of Differentiate exposure from infection as the key concept being tested. Recall that if active
instruction to the client? antitubercular agents is used for at least the first several weeks: isoniazid (INH), rifampin infection requires multi‐drug therapy, exposure can be managed with a single agent alone.
(Rifadin), and pyrazinamide (Tebrazid). The combination therapy lessens the risk of drug
resistance (option 3). Except for streptomycin, which is for IM use, the antitubercular agents
are administered orally (option 4).
1.‐ “You will take a single drug such as isoniazid (INH) by mouth every day for 6 to 12 months.”
2.‐ “You will be on at least two drugs effective against the tubercle bacillus for three months.”
3.‐ “You will be on combination therapy in order to prevent development of drug resistance.”
4.‐ “You will need to learn to give yourself subcutaneous injections.”

2 The nurse delegates an unlicensed assistive person Correct answer: 2 The nurse ensures that the UAP understands the importance of reporting immediately any The core issue of the question is the appropriate procedure for the nurse to use when
(UAP) to assist a client with a clean urinary difficulties during the procedure such as bleeding. This provides for safe and effective care. delegating care to a UAP. Eliminate option 4 first because it is the role of the nurse.
catheterization procedure. The client had formerly Option 1 is incorrect because the client cannot do the procedure because of arthritis. Option 3 Eliminate options 1 and 3 next, because they are not indicated or unnecessary, respectively.
been able to do the procedure but because of arthritis, is unnecessary if the UAP is qualified to do the procedure. Option 4 is a function of the nurse,
he has been unable to perform the catheterization. not the UAP.
Although the UAP has done this procedure before,
which of the following must the nurse emphasize to
the UAP?
1.‐ Let the client do most of the procedure and report the expected output.
2.‐ Report immediately any unusual observations, such as bleeding.
3.‐ Complete in proper order the steps of the procedure.
4.‐ Perform health teaching while performing the procedure.

3 The client is in the operating room for a surgical Correct answer: 4 Only option 4 relates to the client’s physiological integrity. Options 1 and 2 pertain to the The core issue of the question is knowledge of physiological assessment priorities in the
procedure. The nurse in the operating room is psychological aspects of client care, while option 3 relates to the safety in the environment. perioperative client. Fluid loss directly relates to cardiovascular status, which is one of the
monitoring the physiological integrity of the client. ABCs (airway, breathing, and circulation). Use nursing knowledge and the process of
Which of the following activities is most appropriate? elimination to make a selection.

1.‐ Determine client satisfaction with care received.


2.‐ Assess client’s emotional status.
3.‐ Monitor asepsis in the environment.
4.‐ Calculate fluid loss and its effects.

4 The clinic nurse is conducting health screenings. Correct answer: 1, 2, 4 Obesity, hypertension, and smoking are modifiable risk factors for stroke. The core issue of the question is knowledge of risk factors for stroke. Recall that these are
Which of the following client assessment findings Hypercholesterolemia (cholesterol level greater than 200 mg) would also be a risk factor, but similar to the risk factors for cardiac disease to help make your selections.
indicates that client teaching is needed about the risk this client’s level is less than 200 mg/dL. Eating a diet containing fiber helps keep cholesterol
for stroke? Select all that apply. levels low and is not a risk factor for stroke.
1.‐ Weight 205 lbs and height 5 feet 4 inches
2.‐ Blood pressure 164/92 mmHg
3.‐ Eats bran for breakfast daily
4.‐ Smokes 1/2 pack cigarettes per day
5.‐ Serum cholesterol level is 172 mg/dL

5 Which of the following actions would the nurse take Correct answer: 1, 2, 5 Options 1, 2, and 5 are core principles of medical asepsis. Option 3 violates principles of Use knowledge of medical versus surgical asepsis as essential core concepts. Eliminate
to maintain medical asepsis when caring for a client medical asepsis. Option 4 uses principles of surgical asepsis. Option 6 is unrelated to the needs options that utilize surgical asepsis or are unrelated to the needs of the client.
with diabetes mellitus on the medical nursing unit who of this client.
requires irrigation of a leg ulcer and insulin injections?
Select all that apply.
1.‐ Wash hands before and after client care.
2.‐ Wear personal protective equipment during the dressing change.
3.‐ Recap a needle after administering insulin.
4.‐ Change the dressing for a diabetic ulcer using sterile gloves.
5.‐ Wipe the rubber stopper on the insulin vial before withdrawing dose.

6 Laboratory test results indicate a client is in the nadir Correct answer: 2 Red blood cells, white blood cells, and platelet counts may be decreased during the nadir The core issue of the question is the ability to determine which drugs could increase the
period that follows administration of a chemotherapy period following administration of chemotherapy that has hematological toxicity. Medications risk of bleeding when a client’s blood counts may be low. Use the process of elimination
drug. Which drug should the nurse avoid administering that inhibit platelet aggregation should be avoided during the nadir period following and knowledge of drug actions and adverse effects to make a selection.
to this client at this time? antineoplastic therapy. Aspirin, ibuprofen, and indomethacin are examples of some of these
agents. Tylenol is the drug of choice for mild pain and fever. Benadryl is often used for sinus
drainage or as an antihistamine and Robitussin is used to manage cough.

1.‐ Acetaminophen (Tylenol)


2.‐ Ibuprofen (Motrin)
3.‐ Diphenhydramine (Benadryl)
4.‐ Guanefesin (Robitussin)

7 The newborn nursery has recently formed a unit Correct answer: 4 A situational leader recognizes that leadership style depends on the readiness and willingness The core issue of the question is knowledge of various leadership styles. Use this
policy and procedure committee. The nurse, while of the group or the individuals to perform the assigned tasks. The democratic or participative knowledge and the process of elimination to make a selection.
attending and participating in the meetings, leader offers suggestions, asks questions, and guides the group toward achieving the group
determines that which nurse exemplifies a situational goals. The laissez‐faire leader recognizes the group’s need for autonomy and abdicates
leader? responsibility. A bureaucratic leader relies on the organization’s rules, policies, and procedures
to direct the group’s work.
1.‐ The nurse who offers suggestions, asks questions, and guides the group toward achieving group goals.
2.‐ The nurse who recognizes the group’s need for autonomy and abdicates responsibility.
3.‐ The nurse who relies on the organization’s rules, policies, and procedures to direct the group’s work.
4.‐ The nurse who recognizes that leadership style depends on the readiness and willingness of the group or the individuals to perform the assigned tasks.

8 The nurse places highest priority on taking which of Correct answer: 1 Hand hygiene is a core principle of standard precautions. Using gloves is appropriate when Use the process of elimination based on nursing knowledge of standard precautions.
the following actions to reduce the spread of there is a risk of exposure to blood, body fluids, secretions, and excretions. However, Elements of transmission‐based precautions are not initiated with all clients.
microorganisms when caring for a client at risk for handwashing should be done after removal of gloves. Not all clients require transmission‐
infection? based precautions (option 3) or a private room (option 4).
1.‐ Wash hands before and after client care.
2.‐ Use clean gloves when implementing client care.
3.‐ Institute transmission‐based precautions.
4.‐ Place the client in a private room.

9 The nurse would report to the physician which of the Correct answer: 1, 4 The white blood cell count is elevated (normal 5,000–10,000/mm<sup>3</sup>), The core issue of the question is the ability to discriminate between normal and abnormal
following abnormal laboratory values for a 58‐year‐old as is the BUN (0.8–22 mg/dL). These changes would be expected with infection (noted by laboratory values. Note the critical symptoms fever and diarrhea, which could lead you to
client newly admitted to the nursing unit with fever fever) and possibly accompanying dehydration from diarrhea. The sodium (135–145 mEq/L), select elevated white count for infection and elevated BUN with fluid loss from diarrhea.
and diarrhea? Select all that apply. potassium (3.5–5.1 mEq/L), and serum creatinine (0.8–1.6 mg/dL) are all within normal limits.

1.‐ White blood cell count 12,260/mm3


2.‐ Sodium 142 mEq/L
3.‐ Potassium 3.9 mEq/L
4.‐ Blood urea nitrogen 38 mg/dL
5.‐ Serum creatinine 0.9 mg/dL

10 The mental health nurse working with children Correct answer: 3 According to Erikson’s stages of development, a 10‐year‐old child is experiencing industry vs. The core issue of the question is the ability to anticipate levels of growth and development
anticipates that unrealistic expectations or a sense of inferiority. Shame (option 1), guilt (option 2), and role confusion (option 4) occur at other in a 10‐year‐old child. Use knowledge of Erikson’s theory to make a selection.
failure to meet standards would cause a 10‐year‐old developmental levels.
child to develop a sense of which of the following?

1.‐ Shame
2.‐ Guilt
3.‐ Inferiority
4.‐ Role confusion

11 A postoperative client who has an order for 5,000 Correct answer: 4 Low‐dose heparin therapy is indicated in many postoperative clients to prevent the The critical words in the stem of the question are best answer the question. This tells you
units of heparin SubQ for three doses wants to know development of thromboembolic episodes. It is not used in every postoperative situation that the correct answer is one that responds to the client’s concern, rather than just
why this drug is being ordered. What information (option 1), but it is usually used for clients who have orthopedic surgery or are anticipated to reciting a fact about the medication. Use nursing knowledge and the process of elimination
would the nurse provide to the client to best answer be immobilized for a time following surgery. Short‐term therapy is not given to maintain to answer the question.
the question? adequate blood clotting levels (option 2) but merely to intervene as a preventative measure.
While the statement that heparin is given SC into the abdomen and is not usually painful is
factual, it is not the reason for the medication being given to the client (option 3).

1.‐ “Heparin is used as a common medication in many clients who have surgery.”
2.‐ “Heparin is essential during the postoperative period to maintain adequate blood clotting levels.”
3.‐ “The injections will be given in the abdomen and are not usually associated with discomfort.”
4.‐ “Heparin is being used to prevent blood clots from forming as a result of surgery or decreased mobility.”

12 The Emergency Department has recently experienced Correct answer: 1 Shared leadership recognizes that there are many leaders within a group so the leader The core issue of the question is knowledge of various leadership styles. Use this
a significant increase in client visits. The year‐to‐date encourages the formation of self‐directed work teams. In transformational leadership, the knowledge and the process of elimination to make a selection.
census reveals a 20% increase in admission from the leader encourages risk taking such as trying out nursing approaches that are evidence‐based or
same period last year. In an effort to reduce staff research‐based. A transactional leader uses incentives to promote productivity such as giving
stress and burnout by empowering the staff, the nurse rewards for excellent performance. A democratic leader provides constructive criticism and
manager uses which of the following approaches to facilitates the group to meet their goals.
demonstrate shared leadership?

1.‐ Encourages the formation of self‐directed work teams.


2.‐ Encourages the group to try out nursing approaches that are evidence‐based.
3.‐ Suggests that staff who have demonstrated charting excellence be given opportunities for professional development activities.
4.‐ Provides constructive criticism and facilitates the group to meet their goals.

13 A 56‐year‐old client reports to the nurse that his sleep Correct answer: 2 Middle‐aged adults have a decrease in deep sleep, stage IV NREM. Option 1 is an expected The core issue of the question is knowledge of age‐related changes in sleep pattern. Use
patterns are different than when he was younger. The pattern in older adults; option 3 is expected in young adults, and option 4 is expected in this knowledge and the process of elimination to make a selection.
nurse anticipates that this client is likely to be neonates.
experiencing which normal developmental pattern?

1.‐ 6 to 8 hours of sleep per night with about 20 to 25% of rapid eye movement (REM) sleep and a marked decrease in Stage IV non‐REM (NREM) sleep.
2.‐ 6 to 8 hours of sleep per night with about 20% REM sleep and a decrease in Stage IV NREM sleep.
3.‐ Erratic sleep because of work schedule with about 30% of REM sleep and no marked decrease in Stage IV NREM sleep.
4.‐ Light sleep with equal amounts of REM sleep and NREM sleep.

14 The nurse concludes that teaching has been effective Correct answer: 3 Crowning is the point in time when the perineum is thin and stretching around the fetal head The critical word in the stem of the question is crowning. Use knowledge of what occurs
when the laboring client’s partner shouts, “She’s both between and during contractions. Delivery is imminent when crowning occurs. Crowning during crowning and the process of elimination to make a selection. Visualize the word
crowning!” as: occurs later than the first sight of the infant’s head. A head that recedes upward between crown and select the answer that matches the part of the head that a crown would sit on.
contractions is not crowning. The mouth and nose cannot be suctioned during crowning
because they are not accessible, nor is it timely.

1.‐ The nurse first starts to see a little of the baby’s head.
2.‐ The baby’s head recedes upward between pushing contractions.
3.‐ The perineum is thin and stretching around the occiput.
4.‐ The mouth and nose are being suctioned.
15 A client questions the surgical nurse about the Correct answer: 4 Option 4 gives the client an opportunity to explain to the nurse the reason for asking the The core issue of the question is knowledge of communication techniques that are
personnel in the operating room. Which of the question. This helps the nurse understand the client’s frame of reference and allows the nurse effective when working with a client who will undergo surgery. Use knowledge of
following initial responses by a nurse to the client’s to best address the client’s concern. Options 1 and 3 offer false reassurance and can give the communication theory and the process of elimination to make a selection.
concern is most therapeutic? impression that the nurse did not listen to or address the client’s concerns. Option 2 is a close‐
ended question and may not help the nurse explore the client’s concerns.

1.‐ “The nurses are well‐qualified for the job they do.”
2.‐ “Have you had a bad experience in the OR?”
3.‐ “You’re concerned about the personnel, but you have no need to worry.”
4.‐ “Can you tell me about why you are interested in the personnel?”

16 After three defibrillation attempts, the client Correct answer: 3 Lidocaine is the primary medication used to treat ventricular dysrhythmias. Lidocaine The core issue of the question is knowledge that Lidocaine is an antidysrhythmic that
continues to be in a pulseless ventricular tachycardia. suppresses automaticity in the HIS‐Purkinje system by elevating electrical stimulation should reduce the irritability of the ventricle, thus making it more amenable to shock
A lidocaine bolus of 100 mg IV is administered. The threshold of the ventricle during diastole, thus decreasing ventricular irritability. Ventricular therapy. The reduction in ventricular irritability could manifest as a conversion to a
nurse would expect to see which of the following as a fibrillation (option 1) is a worsening dysrhythmia. Slowing the heart rate (option 2) without supraventricular rhythm.
therapeutic response to lidocaine? converting the rhythm to an atrial or sinus rhythm is not therapeutic. An increase in level of
consciousness (option 4) would only occur once the ventricular rhythm is terminated.

1.‐ Conversion from a ventricular tachycardia to a ventricular fibrillation


2.‐ Slowing of heart rate to 80 beats per minute
3.‐ A reduction in ventricular irritability
4.‐ An increase in the level of consciousness

17 The nurse is assigned to a client diagnosed with head Correct answer: 4 The UAP is qualified to complete simple procedures, such as bathing a client and changing The core issue of the question is an appropriate activity to delegate to an unlicensed
and neck cancer who is receiving enteral feedings via bed linens. While the UAP could possibly administer mouth care to this client, the nurse must assistant. Keep in mind that any activity that involves assessment is retained by the RN, so
gastrostomy tube. When the nurse is called away to assess the oral cavity (option 2) and should be the one to assess tube feeding residual (option eliminate options 1 and 2. Choose option 4 over 3 because it is procedural in nature.
care for another client, which task for this client could 1). UAPs are not trained in therapeutic communication skills and techniques (option 3).
most appropriately be delegated to the unlicensed
assistive person (UAP)?

1.‐ Determining the amount of residual for the tube feeding


2.‐ Giving mouth care and assessing the oral cavity
3.‐ Exploring how the client is currently coping with the diagnosis
4.‐ Administering a bath and changing bed linens

18 A client with metabolic acidosis is admitted. Which of Correct answer: 3 A client in metabolic acidosis may also be hyperkalemic. As the hydrogen ions shift from the Note the critical word acidosis in the question. Use this to eliminate options 1 and 2
the following laboratory values would the nurse expect ECF to the ICF, potassium enters the ECF, leading to an increased serum potassium. pH values because the pH is not low in either option. Focus on the critical word metabolic to pick the
to find in this client? of < 7.35 are associated with acidosis (option 2). Options 3 and 4 have option that contains a cation with the highest value since hydrogen ions can enter the cell,
K<sup>+</sup> levels above 5.5 mEq/ L that are associated with acidosis, but which in this case is option 3.
option 3 contains the higher value. Option 1 has a normal pH and serum potassium level.

1.‐ pH 7.40; serum potassium 3.8 mEq/L


2.‐ pH 7.36; serum potassium 3.1 mEq/ L
3.‐ pH 7.2; serum potassium 6.2 mEq/ L
4.‐ pH 7.0; serum potassium 5.5 mEq/ L
19 A client has a BUN of 68 mg/dL and a creatinine level Correct answer: 4 Potassium (KCL) is contraindicated in clients with renal dysfunctions. It can not be filtered out Protein creates more potassium in the body and the lab shows that the kidneys are not
of 6.0 mg/dL. The IV fluid is 5% dextrose in 0.9% if there is decreased renal filtration. With increased damage in tissues additional potassium is filtering as they should. Additional potassium from protein metabolism may cause death.
sodium chloride with 40 mEq KCL @ 100 mL/hour. released, causing an even greater level of potassium that can be life‐threatening. Encouraging Activities, such as ambulation, will not change the BUN or creatinine since they reflect
Which action would be most appropriate for the nurse protein, ambulation, and taking vital signs do not safeguard the client from the danger of this filtration of the renal system and not the rate of circulation of the blood. Taking the vital
to take? potential electrolyte imbalance. signs every hour only tells you information about the circulatory status and does not
explain or improve the renal functions. Action needs to be taken immediately to
discontinue the IV with the potassium to minimize the buildup of potassium to toxic levels
that could be life‐threatening.

1.‐ Encourage more protein in the diet.


2.‐ Ambulate the client more to increase circulation.
3.‐ Take vital signs every hour.
4.‐ Question the use of potassium in the IV fluids.

20 The nurse concludes that a child is in Piaget’s Correct answer: 1 In Piaget’s theory on development the conservation is a hallmark sign in the concrete The core issue of the question is knowledge of characteristics of various cognitive
concrete operations stage after observing which of the operational stage. Options 2, 3, and 4 are not characteristic of this stage. developmental levels according to Piaget. Use this knowledge and the process of
following traits in the child? elimination to make a selection.
1.‐ Conservation.
2.‐ Egocentrism.
3.‐ Animism.
4.‐ Preconventional thought.

21 A 60‐year‐old client has been prescribed rabeprazole Correct answer: 3 Omeprazole, pantoprazole, and rabeprazole must be swallowed whole. Lansoprazole and The core issue of the question is knowledge of which medications used for GERD can be
(Aciphex) for symptoms of gastroesophageal reflux esomeprazole capsules may be opened and sprinkled on applesauce or dissolved in 40 mL of opened because they come in capsule form. Use knowledge of pharmacology to answer this
disease (GERD). He has trouble swallowing pills. What juice. question, which tests specific nursing knowledge of drug forms.
alternate medication should the nurse plan to request
for this client?

1.‐ Omeprazole (Prilosec)


2.‐ Pantoprazole (Protonix)
3.‐ Lansoprazole (Prevacid)
4.‐ There is no substitute for Aciphex

22 At the start of the shift there were only three Correct answer: 4 It is an RN’s responsibility to do assessments, analyze the data, plan and implement care and Recognize that assessment and client education are part of the professional scope of
newborns in the nursery, so staffing consisted of one teaching, and evaluate the outcomes. A second RN needs to be assigned to the nursery to practice. The correct answer would be the option that safely retains these functions for the
RN and one LPN. Within two hours, three more safely manage the care of the Level I newborns. RN given the change in unit census.
newborns were admitted to the nursery, one requiring
Level II care, and the parents of two newborns needed
discharge teaching so they could go home. The RN was
needed full time in the Level II nursery as the newborn
was stabilized. What staffing is needed to provide
appropriate care in this situation?

1.‐ The LPN can complete the admission assessments and discharge teaching for the five Level 1 newborns.
2.‐ An UAP from the postpartum unit can be reassigned to the nursery to do the discharge teaching.
3.‐ The RN can complete the admission assessments while continuing to stabilize the Level II newborn.
4.‐ Another RN needs to be assigned to the nursery to implement the admission assessments and discharge teaching.

23 The nurse is working with a client suffering from Correct answer: 3 Anxiety or anger increases peristalsis leading to subsequent diarrhea. Excessive intake of The core issue of the question is knowledge of ordinary factors that can contribute to
chronic diarrhea. In teaching ways to reduce diarrhea, cheese or eggs, ignoring the urge to defecate, and lack of exercise can lead to the development diarrhea. Evaluate each of the options in turn and determine whether it is likely to
the nurse would encourage the client to avoid which of of constipation. aggravate diarrhea. Note that anxiety and anger stimulate the sympathetic nervous system,
the following that contribute to the development of which then increases peristalsis; this will help you to choose correctly.
diarrhea?
1.‐ Excessive intake of cheese and eggs
2.‐ Habitually ignoring the urge to defecate
3.‐ Anxiety or anger
4.‐ Lack of exercise

24 The fetal head is determined to be presenting in a Correct answer: 2 The normal attitude of the fetal head is one of moderate flexion. Changes in fetal attitude, The core issue of the question is the significance of moderate flexion of the fetal head.
position of complete extension. After learning of this, particularly the position of the head, present larger diameters to the maternal pelvis, which Recognize that changes in the position of the fetal head affect delivery to choose the
the nurse anticipates which of the following? contributes to a prolonged and difficult labor and increases the likelihood of cesarean delivery. correct option.

1.‐ Precipitous labor and delivery


2.‐ Prolonged labor and possible cesarean delivery
3.‐ Normal labor and spontaneous vaginal delivery
4.‐ Forceps‐assisted vaginal delivery

25 The nurse notices that an elderly nursing home Correct answer: 3 Mental status changes and concentrated urine are common signs of dehydration in the Note the critical words in the question are not eating or drinking and deficit. With this in
resident has not been eating or drinking as much as elderly. Tenting and dry, flaky skin are consistent changes seen with normal aging. Hand veins mind, look for a physical assessment finding that is consistent with dehydration. Eliminate
usual. Which assessment finding would best indicate that fill within 3 to 5 seconds and clear lungs sounds with unlabored breathing are normal options 1 and 2 first because of the words clear and dry respectively. Choose option 3 over
the presence of fluid volume deficit? findings. 4 recalling that neurological symptoms are often present with altered fluid balance because
sodium imbalance may occur simultaneously.

1.‐ Clear lung fields with unlabored respirations


2.‐ Tenting and dry, flaky skin
3.‐ Increased drowsiness, mild confusion, and concentrated urine
4.‐ Hand veins that fill within 3 to 5 seconds of being lowered below the heart

26 Following a liver transplant the client is taking Correct answer: 4 Liver function includes the regulation of blood clotting and corticosteroids can impair wound The core issue of the question is knowledge that the liver is a vascular organ and that
prednisone among other medications to prevent organ healing and irritate the GI tract. Thus, the client should be instructed to report signs and some medications used to suppress the immune system to prevent rejection, such as
rejection. The nurse should instruct the client to make symptoms of bleeding. Option 1 is a side effect of corticosteroids but is not the priority from a corticosteroids, can lead to bleeding.
it a priority to report which of the following signs and physiological basis. Options 2 and 3 do not reflect the associated risk of bleeding with
symptoms to the health care provider? corticosteroid medications.

1.‐ Moon face


2.‐ Diminished pigmentation
3.‐ Dysphagia
4.‐ Bleeding

27 The nurse would place highest priority on which of Correct answer: 1 Frequent coughing and deep breathing is an easy maneuver that has great benefit to optimize Note the client in the question has newly arrived to the nursing unit following surgery. The
the following nursing interventions when planning to ventilation in the postoperative client. Good pain management facilitates effective coughing critical words “nursing interventions” help you to eliminate options 3 and 4, which require a
prevent atelectasis in the newly admitted and deep breathing. Getting the client out of bed and administering oxygen and medical order. Choose option 1 over 2 because of the word “hourly” and because there is
postoperative client? bronchodilators are all appropriate interventions for preventing or treating atelectasis, but not enough information in the stem to determine whether the client can safely get out of
clearly the best option is to prevent its occurrence by simple maneuvers such as coughing and bed at this time.
deep breathing.
1.‐ Hourly coughing and deep breathing
2.‐ Assisting the client out of bed
3.‐ Administration of bronchodilators
4.‐ Supplemental oxygen

28 A 14‐year‐old client has been diagnosed with bipolar Correct answer: 4 Children with bipolar disorders are often misdiagnosed as having conduct disorder or ADHD. The core issue of the question is knowledge of how bipolar disorders may present in a
disorder. The nurse would expect to see which of the Intense mood swings (option 1), inflated self‐esteem (option 2), and spending sprees (option 3) child that is in early adolescence. Use nursing knowledge and the process of elimination to
following problems? occur more often in adults. make a selection.
1.‐ Intense mood swings lasting only 1 to 2 hours
2.‐ Inflated self‐esteem
3.‐ Spending sprees
4.‐ Fire‐setting and gang behavior

29 A client’s hemoglobin level is 14 grams/dL. Which Correct answer: 2 The laboratory value given is within normal limits (12–16.5 grams/dL). All the other The core issue of the question is knowledge of normal and abnormal hematological
interpretation of the laboratory value by the nurse is statements are inaccurate. The client is not malnourished (option 1), at nutritional risk (option laboratory values. Use specific nursing knowledge and the process of elimination to make a
most accurate? 3), and does not have polycythemia (high level) as indicated by option 4. selection. Note that options 1 and 3 are somewhat similar so you may eliminate both of
those initially.
1.‐ Client has a low value and is malnourished.
2.‐ Client has a normal laboratory value and has no nutritional risk.
3.‐ Client has a low to normal value indicative of a nutritional risk.
4.‐ Client has an elevated value indicative of polycythemia.

30 Which of the following care measures should the Correct answer: 1 Crusting of dried exudate is common with bacterial conjunctivitis and it is important for the Note the critical word conjunctivitis in the stem of the question. Recall that this infection is
nurse include in the discussion when teaching home child’s vision and safety that the crusts are removed. Warm, moist wipes aid in comfort and highly contagious. Then determine the correct option by associating the word disposable in
care measures to the parents of a child who has they need to be disposable to reduce the risk of transmitting the infection to others in the the correct option with the concept of infection in the stem of the question.
bilateral bacterial conjunctivitis? home. Oral antihistamines, ophthalmic corticosteroids, and topical anesthetics are not
indicated in the management of bacterial conjunctivitis.
1.‐ Use of warm, moist, disposable compresses to remove crusting
2.‐ Use of oral antihistamine medication to relieve eye itching
3.‐ Use of ophthalmic corticosteroids to decrease inflammatory response
4.‐ Use of topical anesthetics applied to relieve discomfort

31 After a client has experienced a seizure, what is the Correct answer: 2 After the seizure, the client will be postictal, which is a deep sleeping state. She/he could The core issue of the question is knowledge of a position that will reduce the risk of
most appropriate position in which the nurse should aspirate secretions unless side‐lying to promote drainage from the upper airway. Positioning aspiration following seizure activity. Use nursing knowledge and the process of elimination
place the client? the client on the back (option 1) increases risk of aspiration. Positioning the client on the to make a selection. Recall that the side‐lying position is commonly used in any situation in
abdomen (option 3) or upright in chair (option 4) is unrealistic given the client’s postictal state. which aspiration is a risk.

1.‐ On back with head raised 15 degrees


2.‐ On the side
3.‐ On abdomen
4.‐ Upright in chair

32 After correctly positioning a client for a wound Correct answer: 1 A client fall is a potential medical emergency; however, the nurse’s responsibility is ensuring Options 1 and 4 are incorrect, sterile equipment is considered contaminated if left
dressing change, the nurse sets up a sterile field, the safety of the client being attended to. Option 2 ignores the safety of the potentially injured unattended and therefore must be thrown away. Option 2 is incorrect; the nurse needs to
placing the wound supplies in the field. The nurse client. Option 3 wastes supplies. Option 4 could lead to a contaminated sterile field. prioritize care appropriately. Thus the nurse needs to respond to the client who fell rather
hears a page to respond to another client who has than continue with the wound dressing change.
fallen in the hallway. Which of the following is the
most appropriate nursing action for the nurse to take?

1.‐ Ensure the client’s safety, cover the field with a sterile towel, and respond to the other client.
2.‐ Continue quickly with the procedure, and then assist the other client, checking back with the first client as soon as possible.
3.‐ Ensure the client’s safety, discard the sterile equipment, and respond to the other client.
4.‐ Explain the situation to the client needing wound dressing change, leave the sterile supplies in place, and attend to the other client.

33 A client recently diagnosed with type 1 diabetes Correct answer: 4 The American Diabetes Association Exchange Lists divide food into groups with similar First recall the basic food groups that are part of the American Diabetes Association
mellitus is learning to use the American Diabetes content (milk, vegetables, fruit, starch/bread, meat, and fat). All foods within a list are similar Exchange Lists. Then compare each food choice identified with the list. Eliminate options 2
Association exchange lists. The nurse determines that in calories, protein, fat, and carbohydrates if eaten in a certain size portion. Foods may be and 3 first as vegetables and fruits, then pick option 4 over 1 because it is a starch/bread.
the teaching has been effective if the client chooses exchanged within the same list. Rice and bread are starches, egg is meat, tomato is vegetable,
which of the following as an appropriate exchange for and orange is fruit.
white rice?
1.‐ Egg
2.‐ Tomato
3.‐ Orange
4.‐ Bread

34 The nurse is teaching a group of adults about health Correct answer: 1, 3 Genetic screening can identify markers for several types of cancer. One method to remind Elimination of number 4 and looking suspiciously at the phrase most tumors will help to
screenings for cancer. The nurse would include in the men to perform self‐checks for cancer is to mark a calendar to monthly check for changes. Self discriminate between the options. When in doubt, identify alternatives with most or all in
discussion which of the following points? Select all that exams as well as regular medical tests and exams uncover tumors. After a total mastectomy, the answer as false.
apply. women do not need mammograms. Skin cancer risk increases with age.

1.‐ Genetic screening is helpful in identification of cancer risks.


2.‐ Annual medical exams uncover most tumors.
3.‐ Men need to perform breast and testicle exams monthly.
4.‐ Annual mammograms are recommended after a total mastectomy.
5.‐ Inspection of the skin for cancer becomes less important as one ages.

35 During a coffee break, the nurse notices two Correct answer: 2 The nurse should speak privately to the coworkers about their behavior and the impact on Options 1, 3, and 4 are incorrect. To effectively manage conflict between staff members,
coworkers arguing about how to handle a difficult the nurse overhearing them. It does not help the climate of the unit to let it pass (option 1). address the conflict within an appropriate timeframe; do not let it pass unattended. Do not
client. Their voices are raised and body postures are The nurse is not in a position to confront and reprimand coworkers (option 3). Option 4 is openly and publicly reprimand staff in front of other staff members or clients. Finally,
tense and defensive. Which would be the most somewhat plausible but option 2 personalizes the discussion between the nurse and the address staff members privately but keep in mind what behavior is acceptable on the unit.
appropriate approach for the nurse to use to address coworkers, and thus is best to diffuse the situation.
this conflict between staff members?

1.‐ Let it pass because the coworkers probably did not intend to be critical.
2.‐ Speak privately to the coworkers, telling them about personal reactions to this public encounter.
3.‐ Confront and reprimand the coworkers publicly.
4.‐ Inform each coworker privately that it would be most helpful not to display this behavior again.

36 The school nurse is assessing a muscular 17‐year‐old Correct answer: 2 The student’s age, along with symptoms of hair loss and edema indicate that this is not a The core issue of the question is knowledge of adverse effects of steroid use. Use this
female who is coming to the high school health service stage of puberty. The symptoms are not indicated in abuse of barbiturates or marijuana use. information and the process of elimination to make a selection.
for complaints of edema, voice changes, and hair loss. By the process of elimination, the correct answer is option 2.
The nurse’s primary analysis based on the subjective
and objective data is that the student:

1.‐ Is going through a stage of puberty.


2.‐ May be using steroids.
3.‐ May be abusing barbiturates.
4.‐ Is using marijuana regularly.

37 A 4‐year‐old child has been exposed to chickenpox. Correct answer: 1 The prodomal period refers to the period of time between the initial symptoms and the The critical words in the stem of the question are need for additional information. This
After the nurse has provided information about presence of the full‐blown disease. The rash would not be apparent during this time. All the tells you that the correct option is an incorrect statement. Use knowledge of this
chickenpox, the nurse asks the mother to repeat the other statements are correct. communicable viral infection and the process of elimination to make a selection.
information. Which statement by the mother indicates
a need for additional information?

1.‐ “During the prodomal period, my child will have pox all over his body.”
2.‐ “Chickenpox is a viral infection that can be spread to other children.”
3.‐ “I should monitor my child for Reye syndrome, which is a complication of chickenpox.”
4.‐ “My child should not visit my pregnant sister at this time.”
38 The school health nurse is interested in promoting Correct answer: 2 Adolescents tend to feel that they are invulnerable and that if anything bad will happen, it will Focus on the developmental level of the client. To answer this question correctly, it is
safety in the high school population. In planning safety affect others but not themselves. They also tend to feel immortal, as it is difficult for them to necessary to understand growth and development and apply this knowledge to the needs
education for this age group and their parents, the comprehend their own death. Option 1 is a factor related to the adult, option 3 is related to of the adolescent for safety.
nurse would recognize that which of the following is a school‐age children, and option 4 is related to the elderly.
developmental risk factor for adolescents?

1.‐ Substance abuse as a lifestyle means of dealing with stress


2.‐ Feelings of immortality related to perception of being invulnerable to risks that affect others
3.‐ Sports‐related injuries that are usually related to not obeying rules and/or intense competition
4.‐ Polypharmacy, which results in mixing of multiple medications

39 When giving directions to a 24‐year‐old female with Correct answer: 3 A full bladder is necessary to bounce the sound waves off to compare other tissues or Fluids are needed to fill the bladder and are not withheld prior to testing. Bowel structures
possible appendicitis who is about to undergo a pelvic structures are being assessed. If done during pregnancy, the fetus must be over 26 weeks to do not interfere with the assessment of structures and an enema is not required.
sonogram, which statement should the nurse make to not have the restriction for the full bladder, since the amniotic fluid would be used at that Medications do not impact on sound waves and holding medications is not necessary for
the client? point. It would not be helpful to be NPO, because this would deprive the client of fluids. any reason.
Enemas and refraining from medications are unnecessary.
1.‐ “Drink nothing for several hours prior to the exam.”
2.‐ “You will be given an enema to cleanse the bowel.”
3.‐ “Drink plenty of liquids so you will have a full bladder.”
4.‐ Do not take any medications prior to the exam.”

40 The nurse is conducting an initial interview with a 10‐ Correct answer: 2 Children at 10 years of age are egocentric and concerned with themselves. Asking about The core issue of the question is knowledge of communication strategies that are likely to
year‐old boy who has been brought to the mental interests and hobbies is likely to foster establishment of rapport. Focusing on behavioral be effective in developing a therapeutic relationship. Focus on the age of the child and
health clinic by his parents. The nurse can establish symptoms (option 1) could lead to an adversarial relationship. Children often are cognitive developmental level to make a selection.
rapport and credibility with the child by asking the uncomfortable talking about friends and family (option 3) until they get to know a person
child about his: better. Most children are unconcerned about past medical problems (option 4); they are
focused on the here‐and‐now.
1.‐ Behavioral symptoms.
2.‐ Interests and hobbies.
3.‐ Relationships with friends and family members.
4.‐ Medical problems in the past.

41 The nurse is providing medication instructions to a Correct answer: 3 Spironolactone is a potassium‐sparing diuretic used to treat hypertension. Gynecomastia is The core issue of the question is knowledge of adverse drug effects of spironolactone. Use
client. The nurse informs the client that persistent one of its adverse reactions. Adverse reactions usually disappear after the drug is specific drug knowledge and the process of elimination to make a selection.
gynecomastia can result from taking which of the discontinued; however, gynecomastia may persist after discontinuing spironolactone.
following newly prescribed diuretics?

1.‐ Hydrochlorothiazide (HCTZ)


2.‐ Furosemide (Lasix)
3.‐ Spironolactone (Aldactone)
4.‐ Indapamide (Lozol)

42 As the nursing unit representative member serving on Correct answer: 3 Client and family satisfaction surveys are a formal set of activities that can be used to remedy Note the critical word services in the stem of the question. With this in mind, the correct
the hospital quality management committee, the deficiencies identified in the quality of direct patient care, administrative, and support services. option is one that gathers data from the recipients of services. Options 1, 2, and 4 are not
nurse has been asked to evaluate the quality of nursing Incident reports (option 1) serve as an indicator of risk. Documentation of time and activities quality service measures.
services on the unit. What would be an appropriate related to direct care may be done as part of time and motion studies. Acuity relates to the
quality improvement activity for the nurse to ask team need for nursing staff on the unit.
members to participate in?

1.‐ Tracking the number of accidents or incidents on the unit


2.‐ Documenting nursing time and activities spent on direct client care
3.‐ Administering a client and family satisfaction survey
4.‐ Assessing clients and report acuity to shift managers daily
43 When a female client preparing for surgery suddenly Correct answer: 3 Option 3 is best because it represents a communication with the client and is open‐ended. The core issue of the question is the ability of the nurse to care for the emotional needs of
bursts into tears, the preoperative holding unit nurse Options 1 and 2 are not the most appropriate initial approaches since the client is not a perioperative client. Since this is potentially an anxiety‐producing time for clients, choose
should take which of the following actions? encouraged to share her concerns, although later on in the interaction these may be the option in which the nurse provides a therapeutic response to the client.
appropriate. Option 4 ignores the client and does not address the client’s concerns.

1.‐ Pull the curtain closed and leave the area to provide privacy.
2.‐ Be silent as a sign of compassion.
3.‐ Ask the client to share what she is feeling.
4.‐ Continue with the physical preparation of the client.

44 After reviewing the client’s health history, the nurse Correct answer: 3 Cigarette smoking is the leading cause of lung cancer. Smokeless tobacco is more often Eliminate option 1 first because it is a health problem, not a risk factor. From there,
concludes that which of the following is the most associated with oral cancer. Air pollution may also be a contributing factor to development of choose cigarette smoking over the other options because it is highly associated with lung
significant factor related to the development of lung cancer. History of asthma is not associated with greater risk of lung cancer. cancer.
bronchogenic carcinoma for this client?

1.‐ Asthma
2.‐ Smokeless tobacco
3.‐ Cigarette smoking
4.‐ Air pollution

45 The nurse is setting up the breakfast tray for a client Correct answer: 3 Foods that reduce lower esophageal sphincter (LES) pressure will increase reflux symptoms. The core issue of the question is knowing that certain types of foods lower LES pressure,
with gastroesophageal reflux disease (GERD) and These include coffee, fatty foods, alcohol, and chocolate. All the other items can be given to and then being able to take it a step further and identify what types of foods those are.
notices one food that the client should not eat. Which the client. Eliminate each option systematically by reasoning that any foods high in fat (such as the
food should the nurse remove from the meal tray? cream in the coffee) can have this effect.

1.‐ Poached egg


2.‐ Dry toast
3.‐ Coffee with cream
4.‐ Skim milk

46 The nurse is assessing a 30‐year‐old client with a prior Correct answer: 2 Theophylline is a xanthine that causes bronchial dilation due to smooth muscle relaxation. The core issue of the question is knowledge that adverse effects of xanthine medication
history of smoking who takes theophylline (Theo‐Dur) Increased levels of theophylline occur with liver disease and congestive heart failure. Option 3 such as theophylline are increased in liver disease. Use specific knowledge of drug adverse
for chronic obstructive pulmonary disease. Additional is incorrect because the client is young and therefore the age is insignificant. The smoking effects and the process of elimination to make a selection.
diagnoses include liver disease and congestive heart history (option 1) is not an issue; in fact, smokers metabolize theophylline more quickly and
failure. The client is experiencing tremors, dizziness, may need increased doses. There is no data about the client’s weight (option 4) in the stem.
tachycardia, and nausea. The nurse explains to the
client that these symptoms may be the result of:

1.‐ A history of smoking cigarettes.


2.‐ Liver disease.
3.‐ The client’s age.
4.‐ The client’s weight.

47 The nurse has admitted to the surgical unit a client Correct answer: 1, 2, 4, The LPN/LVN is trained to collect data that is then reported to the registered nurse (RN). Recall that procedures and simple data collection can be delegated to the LPN/LVN. With
who just underwent open reduction and internal 5 However, assessment remains the responsibility of the RN. For these reasons, the LPN/LVN can this in mind, eliminate each of the incorrect options systematically.
fixation of a severely fractured right radius and ulna. be expected to take vital signs, report drainage, administer medication, and elevate the casted
Which nursing care activities would be appropriate for limb. The RN should retain the responsibility for assessing neurovascular status to the casted
the nurse to delegate to the Licensed extremity in the immediate postoperative period.
Practical/Vocational Nurse (LPN/LVN)? Select all that
apply.
1.‐ Measure vital signs every 30 minutes.
2.‐ Report drainage on the cast if it appears.
3.‐ Assess neurovascular status of the fingers of the casted arm hourly.
4.‐ Elevate the casted arm above heart level.
5.‐ Administer the prescribed intramuscular analgesic as ordered.

48 Which of the following actions would the nurse Correct answer: 1 Tuberculosis is a respiratory infection, transmitted via airborne droplet nuclei less than 5 Specific knowledge of the mode of transmission of Mycobacterium tuberculosis and the
institute that is specific to the care of the assigned microns in size. types of transmission‐based precautions is needed to select the correct answer. Eliminate 2
client who has tuberculosis? and 3 as tuberculosis is transmitted via air currents. Choose option 1 over option 4 because
tuberculosis is transmitted via airborne droplet nuclei less than 5 microns in size.

1.‐ Wearing a particulate respirator mask when taking vital signs.


2.‐ Instructing the client to cover the mouth with the sheet from the stretcher when transported to other hospital departments.
3.‐ Wearing sterile gloves when collecting a sputum specimen.
4.‐ Keeping the client’s door open to promote ventilation.

49 A client has a potassium level of 6.8 mEq/L. Which Correct answer: 1 The potassium level is abnormally high (normal 3.5–5.1 mEq/L). Since potassium is an The core issue of the question is accurate interpretation of the potassium level and its
sign or symptom would the nurse expect to find when intracellular ion, higher levels will alter the electrical pattern of the EKG. “Peaking of a T wave” significance. From there, associate the symptoms of hyperkalemia to make a selection.
assessing this client? is an indication that potassium is too high. With hyperkalemia (higher than normal potassium
levels), muscle weakness, flaccidity of muscles, diarrhea, abdominal cramping, cerebral
irritability/restlessness are present. Therefore, bowel sounds would be hyperactive and not
silent, such as with an ileus. Muscles are weak and flaccid, not in a cramping state. Cerebral
functions are stimulated and somnolence (sleeping, sluggishness) is not present.

1.‐ Peaking of T wave on the telemetry monitor


2.‐ The absence of bowel sounds, such as in an ileus
3.‐ Muscle cramping of the lower extremities
4.‐ Somnolence with early changes

50 The nurse is preparing to take a client to the Correct answer: 3 The client should be NPO before the procedure in order to be given anesthesia for the The core issue of the question is knowledge that ECT requires anesthesia, which leads to
electroconvulsive therapy (ECT) treatment suite. The procedure (options 3 and 4). The client, not the husband, should sign the consent form (option loss of airway protective reflexes. Use this knowledge to reason that the client must be NPO
nurse must ensure that which of the following 1). The client should be wearing loose‐fitting clothing (option 2). to prevent the risk of aspiration during the procedure.
pretreatment processes has been completed?

1.‐ The client’s husband has signed the consent form.


2.‐ The client is wearing snug‐fitting clothing.
3.‐ The client is NPO.
4.‐ The client has been given ample liquids before the procedure.

51 To minimize the pain related to intramuscular Correct answer: 4 Administering very thick preparations such as penicillin G with benzathine (Bicillin LA) can be The core issue of the question is knowledge of proper administration technique for thick
injection of 2 mL of penicillin G benzathine (Bicillin LA) painful. To lessen the pain, intramuscular injection into a larger gluteal muscle should be liquid parenteral medications. Use knowledge of intramuscular injection techniques and
in an adult client, the nurse would take which of the administered over 12 to 15 seconds to separate the muscle fibers more gradually. Cold knowledge of drug absorption principles to make a selection.
following actions? compresses to the injection site would delay absorption of the drug (option 1). Aspiration for
blood return with all IM injections is necessary for safety since muscles contain larger blood
vessels (option 3). Injection into the deltoid may also result in prolonged discomfort resulting
in limited motion of the upper extremities (option 2). Rotating sites, light massage, and warm
compress to site may also be employed to limit discomfort.

1.‐ Apply cold compress to site after injection.


2.‐ Divide the dose and inject half into each deltoid.
3.‐ Limit prolonging the time taken to administer the drug by not aspirating.
4.‐ Administer the drug deep IM slowly into a large muscle such as the gluteus.
52 The nurse is assigned to the care of an obese client Correct answer: 4 Teaching and assessment are within the domain of the registered nurse (RN) and cannot be The core issue of the question is knowledge of the appropriate tasks to delegate to a UAP.
who has gastroesophageal reflux disease (GERD). delegated to a UAP. The UAP is also not trained in therapeutic communication or counseling Recalling that teaching, counseling, and assessment remain the RN’s responsibility assists in
Which of the following activities could the nurse techniques. These ancillary caregivers can complete tasks under the supervision and direction eliminating each of the incorrect options.
appropriately delegate to the unlicensed assistant of the nurse, and report simple data when asked to do so. With this in mind, the only activity
person (UAP)? that can be delegated is the simple direction to the client to remain upright after eating.

1.‐ Teach the client about the need for weight loss.
2.‐ Explore any concerns about the prescribed regimen for managing GERD.
3.‐ Explain why it is important to eat several small meals per day.
4.‐ Instruct the client to remain upright for at least 2 hours after eating.

53 The nurse is admitting a client with thermal burns to Correct answer: 1 Clients should remain NPO upon admission to the clinical setting with a major burn. Initial The core issue of the question is knowledge that the client who has experienced burn
both arms and anterior trunk. The client asks for a fluid replacement is started via the parenteral route. NPO status is maintained because the injury is under severe physiological stress, and as such, blood flow is directed away from the
drink of water. What is the most appropriate response client may be in shock with blood flow directed away from the digestive organs to more vital digestive tract. Focus on the need to stabilize the client physiologically and provide fluids by
for the nurse to make? tissues. In addition it is possible that the client suffered burn injuries that could cause internal the IV route to help you choose correctly.
damage to body structures, and aspiration is also a risk initially. Options 2, 3, and 4 are
incorrect—fluids and food via the mouth would be restricted at this time.

1.‐ “I’m sorry, you cannot drink anything right now; let me moisten your mouth instead.”
2.‐ “I can only give you juice to drink, not water.”
3.‐ “I’ll get you a drink as soon as I’m finished.”
4.‐ “Would you also like me to order you a meal tray?”

54 A mother brings a 3‐year‐old child to the clinic for a Correct answer: 4 Every time a child enters the healthcare system, the immunization status should be checked. The critical word in the stem of the question is priority. This tells you that more than one
well‐child checkup. The child has not been to the clinic Some children have uncertain history of immunization because of parental noncompliance or option is likely to be a correct nursing action, but that one is more important than the
since 6 months of age. The nurse determines that special circumstances such as being refugees. Once immunization status has been determined, others. Note the age of the child to help you choose immunizations as the priority,
which of the following is the priority of care for this the nurse can go on to assess growth and development and hearing, and to teach the parents especially noting that the child has not received healthcare for 2.5 years, during a time
child? about dental care as necessary. when vaccinations should be kept up to date.

1.‐ Assess growth and development.


2.‐ Begin dental care.
3.‐ Complete hearing screening.
4.‐ Update vaccinations.

55 A client who has pancreatitis is experiencing pain. Correct answer: 4 The pain in pancreatitis is usually aggravated by lying in a recumbent position, but improved The core issue of the question is knowledge of proper positioning techniques to reduce the
After administering an analgesic, the nurse should by sitting up and leaning forward or in the fetal position with the knees pulled up to the chest. pain of inflammation that can be aggravated by movement. Use the process of elimination
place the client in which of the following positions to This position reduces pressure caused by contact of the inflamed pancreas with the posterior to select the position in which the pancreas is not as likely to be compressed against other
promote comfort? abdominal wall. body structures.
1.‐ Supine
2.‐ Prone
3.‐ Left lateral decubitus
4.‐ Sitting up and leaning forward

56 The nurse would be most careful to assess for Correct answer: 1 Although many chemotherapy agents can cause stomatitis, the antimetabolites are The core issue of the question is knowledge of which antineoplastic agents cause
stomatitis in a client receiving which of the following commonly known for causing this side effect. Fluorouracil is the only drug listed in this class. stomatitis as an adverse effect. Use nursing knowledge and the process of elimination to
chemotherapeutic agents? Cisplatin is an alkylating agent; bleomycin is an antitumor antibiotic; and vincristine is a plant answer the question.
(vinca) alkaloid.
1.‐ Fluorouracil (5‐FU)
2.‐ Cisplatin (Platinol)
3.‐ Bleomycin (Blenoxane)
4.‐ Vincristine (Oncovin)
57 The nurse will be working with an unlicensed assistive Correct answer: 1 Safe and effective delegation is based on knowledge of the laws governing nursing practice Option 2 is incorrect; it is not necessary to provide written directions when delegating
person (UAP) for the work shift. Prior to delegating and knowledge about job duties and responsibilities. Nurses must understand the tasks to UAPs as long as verbal directions are clear and expectations are understood.
care to the UAP, the nurse places high priority on competencies and training of unlicensed assistive personnel. Option 3 is incorrect; your responsibility is not preparing the supplies for a delegated task
which of the following? but rather to ensure the delegated task is completed safely and correctly. Option 4 is
incorrect; it is not necessary to inform the client about the tasks or assignments delegated
to non‐staff members. It is however, the responsibility of the staff member to inform the
client prior to the assigned task what will be accomplished.

1.‐ Determining that the UAP is competent to perform the required task
2.‐ Providing written directions to the UAP
3.‐ Making sure all the necessary supplies are available at the client’s bedside
4.‐ Informing clients that an unlicensed staff member will be assigned to them

58 The nurse believes a client has slight one‐sided Correct answer: 1 This assessment may be done to detect small changes in muscle strength that might not Specific knowledge of physical assessment techniques is needed to answer the question.
weakness and further tests the client’s muscle otherwise be noted. Pronator drift occurs when a client cannot maintain the hands in a Note the association between the terms supinated in the question and pronator in the
strength. The nurse asks the client to hold the arms up supinated position with the arms extended and eyes closed. Nystagmus is the presence of fine, correct answer, in response to the client’s change in hand position.
with hands supinated, as if holding a tray, and then involuntary eye movements. Hyperreflexia is an excessive reflex action. Ataxia is a disturbance
asks the client to close the eyes. The client’s right hand in gait.
moves downward slightly and turns. The nurse
documents and reports that the client has which of the
following findings on assessment?

1.‐ Pronator drift


2.‐ Nystagmus
3.‐ Hyperreflexia
4.‐ Ataxia

59 When a client has arterial blood gases drawn from Correct answer: 3 Packing the sample in ice will minimize the changes in gas levels during the transportation of The wording of the question tells you that the correct answer is also a true statement of
the radial artery, the nurse should plan to do which of the specimen to the lab. The arterial site should be held for 5 minutes or longer if the client is fact. Eliminate option 1 first as being factually incorrect. Next, eliminate option 2 because
the following? receiving anticoagulant therapy. The blood is drawn originally in a heparinized syringe and the syringe is heparinized and the blood is not transferred to a test tube. Finally, eliminate
does not need to be transferred to one. A second specimen is not necessary. option 4 because it is unnecessary.

1.‐ Hold the site for up to 1 minute.


2.‐ Transfer the blood sample to a heparinized test tube.
3.‐ Pack the sample in ice for transporting to the laboratory.
4.‐ Obtain a second specimen after 10 minutes for comparison.

60 The nurse knows that a client in the long‐term care Correct answer: 2 For clients with dysthymia, a major concern is social isolation. Option 1 is contraindicated, as The core issue of the question is knowledge of strategies to reduce the risk of isolation in a
unit suffers from dysthymia. The most important is option 3. If the client has a poor appetite, assigning 2 liters of liquid intake (option 4) is not client with dysthymia. Use nursing knowledge and the process of elimination to make a
nursing intervention to include in the nursing care plan therapeutic, nor is planning three regular meals per day (option 3). selection.
is:
1.‐ Provide at least 2 hours of quiet time every morning for the client.
2.‐ Encourage the client to eat in the main dining room with other clients.
3.‐ Include at least three regular meals per day and no snacks.
4.‐ Include at least 2 liters of clear liquids per day in the diet regime.
61 A client who is receiving intravenous heparin by Correct answer: 3 The effectiveness of a heparin protocol is monitored by trending APTT results to achieve a The core issue of the question is recognition that this is a critically high value for the APTT
protocol orders has an activated partial therapeutic level. An APTT of 140 is above the therapeutic level of anticoagulation and and that the action that will maintain client safety is to turn off the heparin for a period of
thromboplastin time (APTT) level of 140 seconds therefore the infusion should be stopped per protocol, and resumed at a decreased dose in time. Use the process of elimination and knowledge of the effects of heparin on APTT times
(control time is 36 seconds). What is the priority action one hour’s time with a repeat APTT ordered in 2–3 hours per protocol. The dose should not be to answer the question.
that the nurse should institute? increased, as this would cause serious consequence to the client. Stopping the medication for
a total 6 hours would undermine the anticoagulation control that the physician is trying to
achieve. Ordering another APTT and continuing to run the infusion could also cause serious
consequences to the client.

1.‐ Increase the heparin dose as the APTT level is not therapeutic. Obtain a repeat APTT in 6 hours.
2.‐ Stop the heparin therapy for 6 hours, then restart the therapy at the same unit dose and obtain a repeat APTT in 6 hours.
3.‐ Stop the heparin therapy for 1 hour. Decrease the
rate of infusion per protocol and restart the
medication in 1 hour. Obtain a repeat APTT in 2 to 3
hours from the restart of the infusion.
4.‐ Obtain an additional APTT in 1 hour and continue to monitor the client.

62 The nurse has admitted to the intermediate care unit Correct answer: 1, 3 The UAP can perform tasks or nursing care activities under the direct supervision of the The core issue of the question is the ability to discriminate between what the RN may
a client who sustained a spinal cord injury at T1 in a registered nurse (RN). The nurse retains responsibility for assessment (options 2 and 5) and delegate and what he or she may not. Evaluate each option and either choose it because it
motor vehicle accident. Which of the following nursing teaching (option 4). is a simple procedure or task, or choose not to select it because it involves assessment or
care activities can the nurse delegate to the unlicensed teaching.
assistive person (UAP) when working with this client?
Select all that apply.

1.‐ Measure oxygen saturation level every hour.


2.‐ Listen to breath sounds.
3.‐ Provide mouth care.
4.‐ Teach use of incentive spirometer.
5.‐ Assess for Homan’s sign while bathing client.

63 The nurse has been instructed to have a surgical Correct answer: 4 The client’s right to withdraw consent is necessary to be part of the consent and it means that The core issue of the question is knowledge of the nurse’s role in obtaining informed
consent form signed by a client who will be undergoing coercion was not utilized in obtaining the signature. It is the physician’s responsibility, not the consent. Keep in mind that the nurse reinforces explanations already given by the physician
a surgical procedure. What is the most essential nurse’s, to explain the diagnosis (option 1) and the need for the surgical procedure (option 2). and use the process of elimination to make a selection.
information to include in the discussion prior to the Cost (option 2) is not an important aspect for informed consent. The technical aspects of the
client signing the permission? procedure are not needed by the client, although an overview of the procedure should be
included (option 3), but again this is the role of the physician. All preparation for the procedure
should include information about what the client will see, feel, and hear.

1.‐ The client’s diagnosis


2.‐ Treatment proposed and the cost
3.‐ The technical aspects of the procedure
4.‐ Right to withdraw consent

64 The pregnant client is 7 centimeters, 100% effaced, Correct answer: 1 Presentation refers to the part of the fetus that is coming through the cervix and birth canal Associate the word face in the question with the word face in the correct response. The
and at a +1 station. The fetus is in a face presentation. first. Thus a face presentation occurs when the face is coming through first. word presentation helps you to choose option 1 over option 2, which also contains the
The nurse concludes that teaching has been effective word face, but in an inappropriate context to this question.
when the client’s husband states:

1.‐ “Our baby will come out face first.”


2.‐ “Our baby will come out facing one hip.”
3.‐ “Our baby will come out buttocks first.”
4.‐ “Our baby will come out with the back of the head first.”
65 A client is scheduled to have a transverse colostomy Correct answer: The correct area is the proximal stoma, not the distal one that is nearer to the distal colon To answer this question correctly, recall the names of the anatomic portions of bowel. It
performed. While doing client teaching, the nurse and rectum. Coming from the small bowel in the center of the diagram, the stomas represent, will also help you to choose correctly if you recall that the prefix trans means across. This
points to which stoma on the diagram to show the in anatomical order, an ileostomy, cecostomy, ascending colostomy, transverse colostomy, might help you select the stoma that is halfway across the abdomen.
client the location of the stoma? Select the correct descending colostomy, and sigmoidoscopy.
stoma.

66 A child diagnosed with deficiency of growth hormone Correct answer: 2 Children with growth hormone deficiency are smaller than their peers and frequently The core issue of the question is knowledge that deficiency of growth hormone leads to
who needs replacement drug therapy comes to the experience problems with self‐esteem and body image. Option 1 would be the opposite short stature and often disturbed body image in the child. Use nursing knowledge and the
clinic for treatment. Which one of the following problem of what the client is experiencing. The nursing diagnoses in options 3 and 4 are process of elimination to make a selection.
nursing diagnoses would be most appropriate for this unrelated to the client in this question.
client?
1.‐ Imbalanced nutrition: More than body requirements
2.‐ Disturbed body image
3.‐ Diversional activity deficit
4.‐ Decreased cardiac output

67 The nursing unit is understaffed and a nurse from the Correct answer: 3, 4 The intermediate care surgical nurse should be most comfortable assuming the care of Note the critical word surgical in the description of the work setting of the float nurse.
surgical intermediate care unit has been floated to the surgical clients. Heart failure, diabetes, and thyrotoxicosis are medical problems, and the client With this in mind, choose the two clients that have procedures that are surgical in nature.
unit for the day shift. Which of the following two with diabetes will also require extensive teaching. The client with nephrolithiasis may also
clients should the nurse assign to this RN float nurse? require teaching about the procedure, but since the client will undergo moderate sedation, the
Select all that apply. nurse would be completing typical preoperative care.

1.‐ A client newly admitted with exacerbation of heart failure


2.‐ A client newly diagnosed with type 2 diabetes mellitus
3.‐ A client who underwent emergency appendectomy during the night
4.‐ A client with nephrolithiasis scheduled for lithotripsy later in the morning
5.‐ A client admitted with thyrotoxicosis

68 The nurse would conclude that hypomagnesemia has Correct answer: 2 Effects of hypomagnesemia are mainly due to increased neuromuscular responses. Paralysis, Recall that options that have similarities are not likely to be correct. Examine the options
not resolved if which of the following neuromuscular flaccidity, and decreased reflexes may be present with hypermagnesemia. from the viewpoint of neurological stimulation. Eliminate each of the incorrect responses
signs is still present after treatment? because they reflect abnormally low activity of the nervous system.

1.‐ Paralysis
2.‐ Tetany
3.‐ Flaccidity
4.‐ Decreased reflexes

69 A client presents to the Emergency Department with Correct answer: 2 Troponin is a sensitive test that indicates damage to the myocardial cells. A CK‐MM Specific knowledge is needed to answer this question. Recall that troponin is a newer
a complaint of chest pain. Which serum laboratory test isoenzyme elevation would indicate skeletal muscle damage. The LDH<sup>4</sup> enzyme that can be measured very early during myocardial damage and is an indicator of
does the nurse check off on the laboratory slip as part isoenzyme is utilized to determine hepatic function and amylase is a digestive enzyme. myocardial damage and thus myocardial infarction.
of a protocol order to rule out an acute myocardial
infarction?
1.‐ LDH4
2.‐ Troponin
3.‐ Amylase
4.‐ CK‐MM

70 The nurse is planning for a multidisciplinary team Correct answer: 2 The client’s level of risk for self‐harm is a major concern. The client may need a private room Critical words in the stem of the question are safety and bipolar disorder. Use nursing
meeting concerning a client with bipolar disorder. In (option 1) and restricted visitors (option 3) if in a manic state. The client should not be knowledge to associate depression as part of bipolar disorder with the threat to safety with
discussing the client’s safety needs, the nurse would be overstimulated (option 4). suicide as a form of self‐harm. This will lead you to the correct answer.
sure to include:
1.‐ Placement of the client in a four‐bed room.
2.‐ The client’s risk level for self‐harm.
3.‐ Unrestricted visitors.
4.‐ The need of the client to participate daily in many concentrated activities.

71 A nurse is teaching a female client newly diagnosed Correct answer: 4 Ciprofloxacin is not recommended for Helicobacter pylori infection during pregnancy. The The core issue of the question is knowledge of the pregnancy categories of the specific
with Helicobacter pylori infection. The nurse other medications can be used after consulting with the physician. drugs listed. Use the process of elimination to make a selection, realizing that specific drug
anticipates that which of the following medications will knowledge is needed to answer the question.
not be used after learning the client is pregnant?

1.‐ Metronizadole
2.‐ Amoxicillin
3.‐ Clarithromycin
4.‐ Ciprofloxacin

72 The nurse admitting a client with a history of Correct answer: 2 Trigeminal neuralgia is manifested by spasms of pain that begin suddenly and last anywhere Note the critical word neuralgia in the question, which tells you the pain is of nervous
trigeminal neuralgia (tic Douloureux) would question from seconds to minutes. Clients often describe the pain as stabbing or similar to an electric system origin. Recalling that this type of pain is usually sharp, stabbing, and possibly burning
the client about which of the following manifestations? shock. It is accompanied by spasms of facial muscles, which cause closure of the eye and/or may help you to eliminate some incorrect options. Distinguish between spasm associated
twitching of parts of the face or mouth. with this disorder and paralysis (an opposite finding) to discriminate between options 2 and
4.
1.‐ Facial droop accompanied by numbness and tingling
2.‐ Stabbing pain that occurs with twitching of part of the face
3.‐ Aching pain and ptosis of the eyelid
4.‐ Burning pain and intermittent facial paralysis

73 Which of the following would be an appropriate Correct answer: 1 Abuse of laxatives and diuretics is a frequent purging behavior for bulimic clients. Options 2 The critical word in the question is bulimia. Recall that this disorder has the classic
intervention for the nurse to include in a plan of care and 3 pertain to anorexia nervosa clients. In regard to option 4, food should never be used as a features of binging and purging to guide you to the correct answer, which in this question is
for a client with clinical diagnosis of bulimia? reward. one that signifies agents that help one to purge.

1.‐ Assess for laxative and diuretic possession.


2.‐ Supervise mealtimes to ensure eating.
3.‐ Observe for ritualistic eating patterns.
4.‐ Reward nonpurging behavior with a favorite snack.

74 A client has a strong family tendency toward Correct answer: 3 Lifestyle modifications and recognition of risk factors are important parts of prevention of The core issue of the question is lifestyle management to reduce the risk of developing
hypertension. He denies that he will get hypertension long‐term complications. Family history is a very strong risk factor but encouraging the client hypertension. Select the option that focuses on prevention while addressing the continued
because he watches what he eats, gets plenty of to maintain his current lifestyle and following up with health screening would be the best plan risk that the client faces.
exercise, and keeps his weight within normal range. of action. False reassurance that he will never be hypertensive and prophylactic
When implementing the plan of care, the nurse would antihypertensive medications are inappropriate.
do which of the following?

1.‐ Praise the client and reassure him that these actions will prevent him from becoming hypertensive.
2.‐ Emphasize that no matter what he does, the client will eventually develop hypertension because of his family history.
3.‐ Recognize the client’s efforts towards a healthy lifestyle and emphasis that early detection is essential to prevent complications.
4.‐ Recommend that the client request antihypertensive medications prophylactically because of his family history.

75 A parent asks the nurse what to do with rough edges Correct answer: 4 When a cast is dry, edges that are not smooth or covered by a piece of stockinette should be The wording of the question indicates that the correct response is a true statement.
of her child’s cast, which are beginning to cause covered to prevent skin irritation. This can be done by petaling the cast edges with strips of Eliminate options 1 and 3 first as least plausible after visualizing these options, then discard
excoriation on the child’s skin. Which of the following adhesive tape, beginning each strip on the inside of the cast, and folding over the edge to the option 2 as unrealistic, since the procedure would be completed at the time of application.
responses by the nurse describes the appropriate outside of the cast.
action to take?
1.‐ “Perform good skin care to the skin around the cast edges, with a protective barrier like Vaseline.”
2.‐ “Call the physician to have the rough edges of the cast cut away.”
3.‐ “Tape a soft towel to the edge of the cast to provide some protection from the rough edges.”
4.‐ “Petal the cast edges with strips of adhesive tape, placing the tape from just inside the cast over the edge to outside the cast.”

76 A 3‐month‐old infant is diagnosed with leukemia. Correct answer: 3 Immunizations should be withheld during leukemia exacerbations because the immune The core issue of the question is knowledge that leukemia adversely affects the immune
Which of the following does the nurse anticipate will system is compromised and the client cannot manage an appropriate response to the system. With this in mind, the nurse needs to be mindful that immunizations will need to be
be part of the plan of care for this infant? immunization. There is no need to place the client in isolation without added evidence of withheld during an exacerbation. Use nursing knowledge and the process of elimination to
immunosuppression (option 1). Options 2 and 4 are irrelevant to the issue of the question. make a selection.

1.‐ The baby will be placed in isolation.


2.‐ Leukemia is familial and other children should be assessed.
3.‐ All immunizations will be withheld during exacerbations.
4.‐ The baby will be NPO during chemotherapy.

77 The registered nurse (RN) is assigned to the Correct answer: 1 The RN is responsible for delegating tasks appropriately and is responsible for the actions of Note the critical word beginning to describe the student nurse. With this in mind, select
postpartum unit. Which task could the RN safely unlicensed personnel. Ambulating a postoperative client is the only task from those listed that the delegation assignment that is simple and procedural in nature, and does not require
delegate to a beginning student nurse? the RN could delegate to a novice student. The other tasks require higher level assessment and assessment, teaching, or advanced knowledge in nursing.
critical thinking skills and should be performed by the RN.

1.‐ Ambulate a client who delivered by cesarean 2 days ago.


2.‐ Complete the admission assessment on a newly delivered client.
3.‐ Call the physician to report a low hemoglobin level.
4.‐ Verify a unit of blood prior to transfusion.

78 A client presents to the Emergency Department with Correct answer: 2 The primary organ in the right upper quadrant of the abdominal cavity is the liver. Because of First analyze the client’s vital signs to determine that the client’s status is consistent with a
a stab wound to the right upper abdominal quadrant. the early shock symptoms, which are presented, it would be expected that this organ has shock state. Then determine which organs are located in the right upper quadrant.
The client’s vital signs are BP 85/60, pulse 125, and possibly been lacerated, causing extensive uncontrolled internal bleeding. The other organ Associate the liver, which is a vascular organ, and the location to determine the correct
respiratory rate of 28 breaths/minute. The nurse systems would not be located in this area. option.
should immediately suspect damage to what organ?

1.‐ Stomach
2.‐ Liver
3.‐ Large intestine
4.‐ Kidney

79 The client is scheduled for a barium enema and is Correct answer: 2 The client will, in most cases, return to the unit with barium still present in the bowel. The Note the critical words best response in the stem of the question. This tells you that the
expressing concern that the barium will not be physician will order laxatives or enemas if the client is potentially not able to expel the barium correct response is a true statement of fact. Recall that this test can cause constipation
evacuated and a bowel obstruction will occur. What on his or her own. The nurse should encourage the client to increase fluid intake if possible as from residual barium to aid in selecting the correct option.
would be the best response for the nurse to make to well. This is a common concern for many clients undergoing this procedure, and their feelings
the client? should not be ignored or belittled.
1.‐ “Don’t worry. The physicians will make sure that all of the barium is out of your bowel before you return to the unit.”
2.‐ “You will be given extra fluids, laxatives, and an enema if you have not expelled the barium within 24 hours.”
3.‐ “The barium they are using will not cause an obstruction.”
4.‐ “Should I have the test rescheduled for when you are less concerned about it?”

80 The nurse is conducting an educational group on an Correct answer: 1 The only respectful therapeutic response here is option 1. The others are contraindicated for The core issue of the question is knowledge of group process and conduct of a group
inpatient unit. One of the clients has not spoken during any group process. Everyone does not need to participate in every session (option 2). It is meeting. Use knowledge of this treatment modality and the process of elimination to make
the group. An effective therapeutic response by the inappropriate to focus the group’s attention on one individual because of level of participation a selection.
nurse would include: (option 3). The client should be allowed to remain part of the group until the client is ready to
participate (option 4).
1.‐ Allowing the client to remain present but nonparticipative.
2.‐ Explaining to the client that everyone in the group needs to participate.
3.‐ Asking the rest of the group members how they feel about this member not sharing.
4.‐ Stopping the group and asking the client to leave.

81 A client is experiencing seizure activity. The nurse Correct answer: 3 Phenytoin is a first‐line anticonvulsant medication that is used to control seizure activity. The core issue of the question is knowledge of medications that are effective against
should prepare to administer which of the following Selegilene (option 1) is used to treat Parkinson’s disease. Diclofenac (option 2) is an NSAID, seizure activity. Use specific drug knowledge and the process of elimination to make a
medications according to protocol? while sumatriptan (option 4) is used to treat headaches. selection.
1.‐ Selegilene (Eldepryl)
2.‐ Diclofenac sodium (Voltaren)
3.‐ Phenytoin (Dilantin)
4.‐ Sumatriptan (Imitrex)

82 As part of the ongoing assessment of a client who has Correct answer: 1 After burn injuries, an elevated potassium level (normal 3.5–5.1 mEq) is expected because of First visualize what happens when cells are destroyed—intracellular contents are released
an electrical burn, a complete blood count (CBC), cellular tissue damage with release of intracellular potassium into the bloodstream. The into the circulation. Secondly, with burn injury fluid is lost through the burn surface and can
electrolyte panel, and renal panel were ordered. The hematocrit will be elevated (not decreased as in option 4) due to hemoconcentration, and the lead to hemoconcentration. With this in mind, eliminate each option except potassium,
nurse would expect to find which of the following white blood cell count will be elevated as part of the inflammatory response to injury. which increases for both of the reasons just stated.
results?
1.‐ Potassium level of 5.9 mEq/L
2.‐ Potassium level of 2.8 mEq/L
3.‐ Hematocrit of 28 mg/dL
4.‐ White blood cell count of 4,000/mm3

83 A client with congestive heart failure (CHF) has been Correct answer: 2 In a 2‐gram sodium diet, foods high in sodium content should be eliminated. It is not enough The critical words in the question are low‐sodium. With this in mind, eliminate options 3
advised to follow a low‐sodium diet. Which statement to stop adding salt or to go only by taste; clients should also be taught to read food labels for and 4 first because they are the least restrictive. Then eliminate option 1 because it is less
by the client indicates to the nurse that diet teaching hidden sodium content. Added salt while cooking is allowed in a 4‐gram sodium diet, not a 2‐ comprehensive than option 2 and because option 2 addresses other sources of hidden
has been effective? gram sodium diet. sodium.
1.‐ “If I stop adding table salt, I shouldn’t have any problems.”
2.‐ “I need to avoid eating processed foods and canned meats and vegetables.”
3.‐ “I can still use a small amount of table salt in cooking.”
4.‐ “I only have to worry about salty‐tasting foods like potato chips.”

84 After delivery, a Chinese client states she needs to Correct answer: 2 Chinese clients may perceive an imbalance in the hot and cold forces in the body after Use principles of culturally competent care to answer this question. If using a multicultural
restore the balance between hot and cold forces in her delivery. They will avoid sources of cold, such as wind, cold beverages, and water (even if perspective rather than one centered in a Western health care approach, you will be able
body and refuses to bathe. The most appropriate warmed) to regain a sense of balance between these extremes. A client’s culture plays a very to eliminate each incorrect response easily.
nursing intervention is to: important part in who they are, and nurses should respect the client’s wishes as long as it will
not result in harm to the client or others.
1.‐ Show her a videotape on postpartum self‐care.
2.‐ Recognize her cultural beliefs and respect her wishes.
3.‐ Discuss postpartum complications related to poor personal hygiene.
4.‐ Request a psychiatric consult for this client.

85 While talking with a client the nurse notes that the Correct answer: 3 When a client’s level of anxiety markedly increases the nurse can relieve the anxiety by The core issue of the question is the ability to recognize escalating anxiety in a client and
client rapidly becomes more uncomfortable and altering the focus of the discussion. Asking the client more details or abruptly stopping the determining the best means to effectively reduce it. Use knowledge of therapeutic
anxious. What action should the nurse take? interview will probably increase the client’s anxiety level. Asking the client to relax may or may measures for anxious clients and the process of elimination to make a selection.
not be effective in reducing the client’s anxiety.

1.‐ Ask specific, focused questions to elicit detailed information about the source of the client’s stress.
2.‐ Encourage the client to try to relax by using guided imagery or other means preferred by the client.
3.‐ Refocus the conversation on a less threatening topic.
4.‐ Stop the interview at this time.
86 The nurse is preparing a client for discharge who will Correct answer: 4 Option 4 is correct because the client is honest, has an understanding of how to take the The core issue of the question is which statement indicates correct understanding of
be taking lithium carbonate. Which of the following medication and what the side effects are, and knows that the side effect will subside lithium as a medication. Use specific drug knowledge and the process of elimination to
statements indicates that the client is feeling eventually. Options 1 and 2 indicate that the client is feeling forced to take the medication but make a selection.
comfortable with being discharged on an antimanic has no desire or understanding of the benefits of the daily routine and dosages. Option 3
medication? indicates that the client has memorized the actions but does not understand the benefits or
side effects of the medications.
1.‐ “I don’t want to take the medicine you will give me, but you said I have to.”
2.‐ “I know that if I take my lithium every day I won’t have to come to the hospital again.”

3.‐ “I have a hard time taking this medicine and I don’t


like the shaking, but I will take it every day with meals,
and have my blood tests done, and come back to the
clinic next month for my check‐up like you said.”

4.‐ “Even though I don’t like taking medicine, I will take


lithium daily with meals and have my blood tests on
the dates I marked on my calendar. I should be able to
do my normal things every day, and in a few weeks I
won’t feel shaky anymore.”

87 A client has just finished a dose of intravesicular Correct answer: 2 For 6 hours following intravesicular chemotherapy, the toilet should be disinfected after each The core issue of the question is how to prevent unintentional exposure of other people to
chemotherapy as treatment for bladder cancer. When use. This will help ensure that the biohazard of excreted chemotherapy drug is contained. The biohazardous chemicals in the client’s urine following intravesicular chemotherapy. With
giving instructions to the unlicensed assistive toilet may also be double‐flushed. Options 1 and 3 are insufficient, while option 4 is this principle in mind, eliminate options 1 and 3 first because they are ordinary measures
personnel (UAP) who will give routine care to this unnecessary and does not address the biohazardous aspect of chemicals remaining in the that do not provide additional protection. Eliminate option 4 next because of the word
client, what statement should the nurse make? toilet. sterile. Clean gloves are needed only.

1.‐ “Be sure the client flushes the toilet after each use.”
2.‐ “Cleanse the toilet with bleach after each use for the next 6 hours.”
3.‐ “Ask the client to wipe the toilet seat with tissue after each bathroom use.”
4.‐ “Assist the client to the bathroom and wear sterile gloves for pericare.”

88 The nurse is caring for a young child who has mitt Correct answer: 1 It is important that circulation is checked regularly. Typically the restraints are removed, one Focus on the word priority in the stem of the question. Recalling that many aspects of
restraints. Which of the following priority actions at a time, every 2 hours to evaluate skin condition and circulation. Although options 3 and 4 restraint care are important, use the ABCs (airway, breathing, and circulation) to focus on
needs to be done regularly to ensure that the child’s are correct, they are not the best response as they do not have to be checked as regularly as the correct option—which addresses the child’s circulation to the restrained limb.
needs are met? the circulation and skin condition. Option 2 applies to an elbow restraint.

1.‐ Check adequacy of circulation and skin condition.


2.‐ Check that the tongue blades in pockets are intact and ends are covered or padded.
3.‐ Ensure that the straps are tied to nonmovable parts of the crib.
4.‐ Check that the call bell is pinned to the child’s gown.

89 The client is to undergo an extensive process of Correct answer: 1 Emergency airway and resuscitation equipment should be readily accessible whenever allergy Note the critical word priority in the stem of the question. This tells you that the correct
allergy testing as an outpatient. The nurse would testing is administered because of the potential for hypersensitivity response and anaphylactic answer is the most important option and that more than one may be technically correct.
complete which of the following as a priority reaction. Because of the potential for a serious reaction, the client will be asked to wait in the Recall that allergic reaction is a risk with skin testing to guide you to the correct answer.
intervention during the initial testing? office for a period of time so he or she can be monitored for any untoward responses. Visibility
of the tested areas is important but not immediately essential. The room should be set up
prior to the arrival of the client but it is not a priority.

1.‐ Have emergency equipment available in the event of an anaphylactic reaction.


2.‐ Instruct the client to wear a t‐shirt to assure easy access to the skin testing sites.
3.‐ Give discharge instructions prior to testing since the client will be able to go home immediately after the testing.
4.‐ Set up the room before the client enters the examination area.
90 A client who has been experiencing panic attacks asks Correct answer: 3 Symptoms associated with a number of medical conditions are very similar to the symptoms The core issue of the question is knowledge that physiological symptoms need to be ruled
why the physician has ordered several laboratory tests. associated with panic attacks. When a medical condition is present, it should be identified and out as having a medical basis before they can be attributed strictly to psychological origins.
The nurse’s answer should incorporate which of the treated. The other options are inaccurate responses to the client’s question. Use this information and the process of elimination to choose correctly.
following pieces of information?

1.‐ Laboratory tests can differentiate between true anxiety and the anxiety associated with depression.
2.‐ Laboratory tests can determine the specific cause of the panic attacks.
3.‐ Physiologic symptoms associated with panic disorders often mimic medical disorders.
4.‐ Symptoms of panic disorders are usually related to hypochondriasis.

91 A female client has been taking norethindrone Correct answer: 2 Norethindrone (Micronor) contains only progestin and no estrogen. Because estrogen may The core issue of the question is which oral contraceptive is safe to use while
(Micronor) oral contraceptive pills. Which of the decrease lactation, progestin‐only pills are commonly used in lactating women. The other breastfeeding. Use knowledge of the estrogen component of norethindrone and the
following items is most likely to be found in her health options do not address the issue of contraception during lactation. process of elimination to make a selection.
history?
1.‐ Superficial phlebitis
2.‐ Currently breastfeeding
3.‐ Dysmenorrhea
4.‐ Menarche at age 18

92 The nurse in the emergency department is caring for Correct answer: 1 It is essential that the client’s spinal cord be immobilized to prevent further injury and loss of Focus on the critical words most important. Whenever a client has suffered a traumatic
a client who has fallen 20 feet from a roof. While function. Assessing for lacerations, exposure of the client, and performing a full mental status injury, the nurse must first address the ABCs and then address neurological status and
performing the primary assessment, the most exam are all part of the secondary assessment. needs. With this in mind, select option 1 over 4 as the priority because it safeguards the
important nursing intervention will be which of the client.
following?
1.‐ Maintain cervical spine precautions.
2.‐ Assess for facial lacerations.
3.‐ Remove clothing.
4.‐ Perform a mental status exam.

93 Which breakfast option indicates to the nurse that Correct answer: 3 The American Heart Association recommends a diet with reduced saturated fats and The wording of the question tells you that the correct answer to the question is the one
the client with coronary artery disease requires further cholesterol for clients with coronary artery disease. Canned peaches are high in concentrated that contains incorrect items. Correlate the words coronary artery disease with fat‐
diet instruction? sugars, which increase triglyceride levels. Egg yolks are high in cholesterol and whole milk is containing foods to begin the elimination process. Choose option 3 because it contains eggs
high in saturated fats. The other options reflect appropriate food selections that are low in and whole milk, two sources of fat and cholesterol.
saturated fat and cholesterol content.
1.‐ Orange juice, shredded wheat, skim milk, toast with jelly
2.‐ Grapefruit juice, oatmeal, 1% milk, bagel with jelly
3.‐ Canned peaches, egg omelet, whole milk, fruited yogurt
4.‐ Applesauce, bagel with margarine, egg‐white omelet, skim milk

94 The nurse would encourage the new mother to use Correct answer: 3 The football, or clutch, position provides the mother with more control of the newborn’s Visualize each of the options and systematically eliminate those that do not promote
which breast‐feeding position to enable the mother to head and full view of face. The lying‐down position is usually done in bed (option 1). The cradle visualization of the face while maintaining control of the head.
have optimal control of the newborn’s head while position often causes the newborn’s head to wobble around on the mother’s arm (option 2).
giving the mother a full view of the infant’s cheeks and Options 1, 2, and 4 do not allow full view of the infant’s face.
jaw?
1.‐ Lying‐down position
2.‐ Cradle position
3.‐ Clutch (football) position
4.‐ Across‐the‐lap position
95 The nurse observes a sinus rhythm pattern on the Correct answer: 1 PEA is associated with what appears to be a normal electrical conduction pattern but there is Associate the words unable to palpate in the stem of the question with the word pulseless
cardiac monitor of a client admitted with diarrhea and no mechanical pumping of the myocardium. Ventricular fibrillation, ventricular tachycardia, in the correct option. Otherwise, it is necessary to understand the pathophysiology involved
vomiting. On physical assessment, the nurse is unable and asystole will not demonstrate an effective electrical conduction pattern on the cardiac in this question.
to palpate a central pulse. The nurse would suspect monitor.
that the client is demonstrating which of the
following?
1.‐ Pulseless electrical activity (PEA)
2.‐ Ventricular fibrillation
3.‐ Asystole
4.‐ Ventricular tachycardia

96 While teaching a client about the proper Correct answer: 3 To promote absorption, the client should not blink for 30 seconds after the administration of The core issue of the question is knowledge of proper administration technique for
administration of dipivefrine (Propine), the nurse dipivefrine. Options 1, 2, and 4 are incorrect for the administration of dipiveprine. dipivefrine. Use specific drug knowledge and the process of elimination to make a selection.
would provide which of the following instructions?

1.‐ Gently squeeze eyes closed for 30 seconds immediately after instillation of medication.
2.‐ Close, but do not squeeze, eyes immediately after instillation of medication.
3.‐ Do not blink for 30 seconds after instillation of medication.
4.‐ Close the eyes for 1 full minute after instillation of medication.

97 The nursing unit is short‐staffed for the shift and a Correct answer: 1 Pediatric clients can be diagnosed with diabetes and the float nurse should be familiar with Review the diagnoses of each of the possible clients and choose the one that the pediatric
registered nurse (RN) from the pediatric unit has been this health problem and could do client teaching. The nurse is not as likely to have recent nurse is most likely to have experience working with.
floated to the nursing unit. Which of the following experience in working with clients with Guillain‐Barré syndrome or who have had prostate
clients should the nurse assign to the float nurse? gland surgery. The client with dementia who is being transferred will require transfer
paperwork to be completed, and the pediatric nurse may not be as familiar with these types of
forms because of the pediatric population usually worked with.

1.‐ A 32‐year‐old client newly diagnosed with diabetes who needs dietary and medication teaching
2.‐ A 56‐year‐old client newly admitted with Guillain‐Barré syndrome who has severe leg weakness
3.‐ An 86‐year‐old client with dementia who will be transferred to a skilled nursing facility during the shift
4.‐ A 59‐year‐old client who will be returning from surgery following transurethral resection of the prostate

98 A client has experienced a near‐drowning event in salt Correct answer: 3 Pulmonary edema occurs as a result of fluid shifts caused by the ingestion of the hypertonic Note the critical words salt water and consider concepts and dynamics of fluid movement
water. The nurse anticipates that one of the salt water. The result is fluid collecting in the interstitial spaces causing pulmonary edema. in the body. Because of the hypertonic water entering the client’s lungs, envision that the
complications this client may experience is: Hypoxia, hypovolemia, and acidosis occur as a result of near‐drowning incidents. client’s own body fluid would move into the alveoli to equalize the tonicity.

1.‐ Heart block.


2.‐ Renal failure.
3.‐ Pulmonary edema.
4.‐ Respiratory alkalosis.

99 The nurse has just read the results of a client’s Correct answer: 1 A positive TB test means that the organism is present in the body in either an active or a Note the presence of the critical word best. This tells you that the correct answer is a true
tuberculin (TB) test at a health fair. An induration is dormant state. It should not be ignored nor should further testing be deferred for several statement of fact. Use knowledge of this test and the process of elimination to make a
apparent. The client asks what this means. The nurse’s months. The client can expect to be scheduled for sputum tests for the presence of the bacillus selection.
best response would be: and a chest x‐ray to determine the presence of lesions or active disease. Medications and
isolation are not instituted until a probable or definitive diagnosis has been made.

1.‐ “A positive test means that you have been exposed to the TB organism. It does not mean that you currently have active bacteria. Further testing will be needed.”
2.‐ “A positive TB test means that you currently have active TB, and you will need to be isolated immediately.”
3.‐ “Many false positives occur. You can expect to be retested in 6 months.”
4.‐ “A positive TB test means that you are currently infectious and will need to be started on medication immediately.”
100 An anxious client begins to yell and interrupt other Correct answer: 1 Speaking slowly and softly reduces stress‐related emotions. Instructing the clients to ignore The core issue of the question is knowledge of therapeutic communication techniques
clients. The client’s speech is rapid and pressured. the behavior will not assist them in reducing anxiety. A client experiencing severe or panic with a client whose anxiety is escalating. Select the option that is most likely to have a
What action should the nurse take? anxiety will be unable to focus on identifying behaviors of anxiety. Reminding a client of the calming effect on the client from a behavioral perspective.
need to use good manners when talking with other clients ignores the client’s anxiety and may
only increase the symptoms of anxiety.
1.‐ Ask the client to speak more slowly and softly.
2.‐ Instruct the other clients to ignore this client’s behavior.
3.‐ Point out to the client that the behavior is a sign of anxiety.
4.‐ Remind the client of the need to use good manners when talking with other people.

101 The nurse suspects that hepatotoxicity is developing Correct answer: 2 Jaundice in the dark‐skinned client can best be observed by assessing the hard palate. The core issue of the question is how to assess for jaundice in a client with dark skin. Keep
in a dark‐skinned client who is on an antibiotic. In what Normally fat may be deposited in the layer beneath the conjunctivae that can reflect as a in mind that the oral cavity is a good choice to help guide you to a correct response.
area of the body should the nurse assess for jaundice? yellowish hue of the conjunctivae and the adjacent sclera in contrast to the dark periorbital
skin. In these clients, palms and soles may appear jaundiced, but calluses on the surface of
their skin can also make the skin appear yellow.
1.‐ Palms of the hands or soles of the feet
2.‐ Hard palate of oral cavity
3.‐ Sclera
4.‐ Conjunctivae

102 The nurse on the oncology unit has received intershift Correct answer: 2, 1, 4, The nurse should assess first the client who has the low platelet count (normal Remember that physiological needs take priority over psychosocial and learning needs.
reports on 4 clients. In what order should the nurse 3 150,000–450,000/mm<sup>3</sup>), and then the client who has the borderline Choose the client with the most serious physiological need first (which is the client with the
assess these clients? Place in order of priority by low WBC count, because these represent greater and then lesser threat to physiological status. most abnormal labs) followed by the other client with a physiological concern. Then use
clicking and dragging the options below to move them From there, the nurse should answer the questions for the client going for chemotherapy, and time as a means of setting priorities for the remaining clients, since the client who is in
up or down. finally see the client who is upset so that the nurse can plan to spend time with this client. psychological distress would benefit from greater interaction time with the nurse.

1.‐ Client receiving radiation therapy who has a white blood cell (WBC) count of 4,500/mm3
2.‐ Client receiving chemotherapy who has a platelet count of 50,000/mm3
3.‐ Client who is crying because she has newly learned that her cancer has metastasized
4.‐ Client who has questions about upcoming chemotherapy

103 Which nutritional measure would help a client with Correct answer: 3 A client with GERD should limit (or possibly eliminate) the intake of coffee because this can Recall that coffee, chocolate, and fatty foods lower LES pressure and therefore increase
gastroesophageal reflux disease (GERD) to minimize relax LES pressure and lead to symptoms. The other options would not be warranted because the risk of reflux. Knowing that these types of food choices need to be limited helps guide
the risk of symptoms? all would contribute to the development of symptoms: large meals, spicy foods (extra garlic), you to select option 3.
and peppermint (which would relax LES pressure).
1.‐ Eating 3 large meals a day with no snacks
2.‐ Using a lot of garlic to season food rather than salt
3.‐ Limiting intake of coffee drinks to 2 or fewer cups a day
4.‐ Using peppermint candies to take away the bitter taste in the mouth

104 A client who is 20 weeks gestation is concerned about Correct answer: 3 The child should be included in planning for the new baby. Children may feel threatened by a Use knowledge of growth and development principles and communication skills to make a
how to tell her 3‐year‐old son about her pregnancy. new sibling and so may need extra time and attention. Parents should avoid putting too much selection. The correct answer is the option that includes the needs of the child as a client as
Which of the following would be the best statement responsibility on the child. well as the parents.
when counseling this client?
1.‐ “If he is not pleased with the news of a new baby, you should tell him that you are disappointed in him.”
2.‐ “Tell him that he is going to have a lot of responsibilities in helping care for the baby.”
3.‐ “Try to provide extra attention to him and include him in plans for the baby.”
4.‐ “Tell him that he will have to stay with his grandparents when the baby is born because you will be busy with the baby.”
105 A nurse is caring for a client with pneumonia. ABG Correct answer: 3 The pH is elevated, HCO<sub>3</sub><sup>‐</sup> is elevated, and Note the critical word pneumonia in the question. With this in mind, reason that the
results are pH 7.49, PaCO<sub>2</sub> 32 PaCO<sub>2</sub> is low. This indicates that there is a mixed respiratory and disorder is likely to be respiratory in origin, which allows you to eliminate options 1 and 2.
mmHg, HCO<sub>3</sub><sup>‐ metabolic alkalosis. Clients with pneumonia are prone to develop respiratory alkalosis. Option
</sup> 28 mEq/L, PaO<sub>2</sub> 1 is incorrect because the HCO<sub>3</sub><sup>‐</sup> level alone
89 mmHg. This nurse analyzes these results as: would be decreased. Options 2 and 4 are incorrect because the ABG values do not reflect these
conditions.

1.‐ Metabolic acidosis, uncompensated


2.‐ Metabolic alkalosis, uncompensated
3.‐ Mixed respiratory and metabolic alkalosis, compensated
4.‐ Respiratory acidosis, uncompensated

106 A client experiences severe nausea for up to 2 weeks Correct answer: 3 A client at risk for nausea should not lie down for at least 30 minutes after meals to avoid The core issue of the question is knowledge of factors that will relieve or aggravate nausea
following her chemotherapy treatment. Which aspiration. The physician should be notified of excessive weight loss (option 1). Foods and caused by cancer chemotherapeutic agents. Use knowledge of the effect of gravity upon
statement indicates a need for further instruction on beverages are better tolerated when they are neither hot nor cold (option 2). Option 4 is a digestion as well as general measures of managing nausea to make a selection.
management of nausea? good client action if other measures fail (option 4).
1.‐ “I need to call my doctor if I lose more than 10 percent of my body weight.”
2.‐ “I should try to eat bland, chilled foods, and drink liquids separate from my meals.”
3.‐ “I need to lie down for an hour after each meal.”
4.‐ “I should call the doctor if my nausea doesn’t go away, to see if a different anti‐emetic could provide better relief.”

107 While assessing the chest tube drainage system of a Correct answer: 3 The movement of the fluid, also referred to as tidaling, in the water indicates normal lung To answer this question it is necessary to have a basic understanding of chest tube
client, the nurse observes a slight rise and fall in the expansion. The physician should not be called unless the movement ceases. Coughing will function. Beyond that, note the critical word slight in the stem of the question, which helps
water level in the water seal. The nurse should take increase the movement and repositioning the chest tube will have no effect on the oscillation. to eliminate option 1. Eliminate option 4 because it is not within the scope of nursing
which of the following actions? practice. Choose option 3 over 2 because there is no reason to ask the client to cough.

1.‐ Notify the physician immediately.


2.‐ Have the client cough.
3.‐ Continue to monitor the system.
4.‐ Reposition the chest tube.

108 During which of the following procedures should the Correct answer: 2 According to standard precautions, the caregiver should wear goggles when contamination The core issue of the question is knowledge of when to use various personal protective
labor and delivery nurse wear protective goggles in from splashing is possible, as when the membranes are artificially ruptured (amniotomy). The equipment items. Recall that amniotomy refers to rupture of the amniotic membrane and
addition to gloves? other options place the nurse at risk for contamination from skin contact, necessitating the use then reason that this could involve splash and require the use of goggles.
of gloves.
1.‐ Changing a soaked disposable bed pad
2.‐ Assisting during an amniotomy
3.‐ Starting an intravenous line
4.‐ Washing dirty instruments

109 A client with cancer has a calcium level of 11.8 mg/dL. Correct answer: 4 The normal calcium level is 9.0–11.0 mg/dL, making this client hypercalcemic. Muscle The core issue of the question is knowledge of electrolyte imbalance (hypercalcemia in
Which of the following symptoms would indicate a weakness is a key feature of hypercalcemia due to alterations in excitable membranes. This this case) and the associated manifestations. Recall that calcium plays a key role in nervous
need for the nurse to call the physician for treatment occurs as a complication in some clients with cancer. Peaked T waves, muscle spasm, and system function to help guide you to the correct option.
orders? increased gastric motility are signs of hyperkalemia.
1.‐ Increased gastric motility
2.‐ Peaked T waves on 12‐lead ECG
3.‐ Muscle spasms
4.‐ Muscle weakness
110 When evaluating the effectiveness of nursing care Correct answer: 2 Anxiety can be a healthy protective response to an actual threat. Defense mechanisms are The core issue of the question is knowledge that anxiety can exist to a greater or lesser
plans used for an anxious client, it is important to unconscious psychological responses designed to diminish or delay anxiety. Anxiety, at times, state at any given time, and that some anxiety may be helpful as it increases alertness and
validate that the client understands that: cannot be avoided and is a healthy adaptive reaction when it alerts the person to impending performance. Use this background knowledge to select the correct option.
threats.
1.‐ Defense mechanisms should not be used.
2.‐ Some anxiety can be helpful.
3.‐ He should strive to never experience anxiety.
4.‐ He should try to avoid the fight or flight response.

111 In assessing a hospitalized client 1 hour after Correct answer: 2 Apresoline is a vasodilator and if the client becomes dehydrated, hypotension will result. In The core issue of the question is knowledge of factors that will compound or worsen a low
receiving hydralazine (Apresoline) 20 mg PO, the nurse other words, during dehydration both preload and afterload are reduced, causing the tank to blood pressure in a client taking an antihypertensive medication. Recall that factors that
notes that the BP is 68/42. The client has been taking get larger with less volume. The normal dose of hydralazine is 5 to 25 mg PO. Serum potassium cause vasodilation or reduce the circulating volume (such as dehydration) can cause a drop
this medication for several years at home without is high but unrelated to apresoline. The increased heart rate is a reflexive response to the low in blood pressure. Use the process of elimination to systematically discard options that do
difficulty. Which of the following factors most likely cardiac output to compensate with decreased preload and afterload. not have this causative influence.
contributed to this episode of hypotension?

1.‐ Dose is excessive for this medication.


2.‐ Total intake for the previous 24 hours is 1,000 mL.
3.‐ Serum potassium is 5.8 mEq/L.
4.‐ Heart rate is 145 beats per minute.

112 A client with a history of heart failure suddenly Correct answer: 4 In a client whose condition is deteriorating, the RN should delegate the task that is most Use knowledge of the principles of delegation. Eliminate the options that address IV and IV
exhibits shortness of breath, a respiratory rate of 30, procedural in nature (in this case the urinary catheter). The LPN is able to collect data to report medication, because these should be retained by the RN. Choose the catheter over vital
crackles auscultated bilaterally, and frothy sputum. to the RN, but in a client whose acuity is changing, it is better for the RN to make the signs because the RN would need to interpret the significance of the vital signs, not merely
After telephoning the physician for medical orders, assessments (option 2). The RN should also insert the IV line and immediately administer the IV measure them.
which action should the nurse delegate to the Licensed medication.
Practical/Vocational Nurse (LPN/LVN)?

1.‐ Start an intravenous line and cap it with a saline lock.


2.‐ Monitor vital signs every 15 minutes.
3.‐ Administer morphine sulfate 2 mg IV push immediately.
4.‐ Insert a urinary catheter.

113 An 86‐year‐old client will be undergoing a surgical Correct answer: 1 Older clients need time to digest the information and ask questions. Option 2 is incorrect The core issue of the question is the need for the older adult undergoing surgery to have
procedure. Which of the following changes would the because most older clients are able to make decisions for themselves. Option 3 can be sufficient time to process information. Choose the option that takes into consideration age‐
nurse make in the informed consent process for this considered coercion, while option 4 can be appropriate but is not the best option since clients related changes of older adults.
elderly client? need more than reading material for an informed consent.
1.‐ Providing adequate time for the client to process the information
2.‐ Encouraging the family members to make the decision for the client
3.‐ Encouraging the client to sign immediately before the client forgets the purpose of the surgery
4.‐ Providing the client with reading material about the surgery and the postoperative instructions

114 The labor and delivery nurse would make it a priority Correct answer: 4 To begin life, the infant must make the adaptations to establish respirations and circulation. Use the ABCs—airway, breathing, and circulation—as the strategy for answering this
to assess which of the following two newborn body These two changes are crucial to life. All other body systems become established over a longer question.
systems immediately after birth? period of time (options 1, 2, 3).
1.‐ Gastrointestinal and hepatic
2.‐ Urinary and hematologic
3.‐ Neurologic and temperature control
4.‐ Respiratory and cardiovascular
115 The nurse is caring for the client who is recovering Correct answer: 2 The eggs provide 24 grams of protein and the whole milk adds calories. The other options are First recall that clients with burn injury need to take in foods that are high in protein and
from partial thickness burns. Which of the following lower in protein and calories. A client recovering from burns requires a high‐protein, high‐ calories. With this in mind, compare each option against this need to eliminate each of the
breakfast options indicates client understanding of the calorie diet. Option 1 does not reflect an adequate protein source. Option 3 reflects an incorrect options systematically.
recommended diet? increased carbohydrate source and bacon is considered a fat, not protein. Option 4 does not
reflect a high‐protein, high‐calorie meal but rather a low‐calorie meal selection with a greater
carbohydrate content.
1.‐ Two slices of toast with butter, orange juice, skim milk
2.‐ Two poached eggs, hash brown potatoes, whole milk
3.‐ Three pancakes with syrup, two slices of bacon, apple juice
4.‐ One cup of oatmeal with skim milk, 1/2 grapefruit, coffee

116 An adult client with diabetes insipidus who has been Correct answer: 1 Signs of overdosage of desmopressin, an antidiuretic hormone, include blood pressure and The core issue of the question is knowledge that fluid retention is an adverse drug effect
taking desmopressin (DDAVP) intranasally comes to pulse elevation, mental status changes, and water and sodium retention. Because the and that this client is showing signs of excessive drug therapy. Use drug knowledge and the
the clinic for a regularly scheduled appointment. The medication therapy needs to be interrupted, the nurse should notify the physician. Option 2 process of elimination to answer the question.
nurse assesses the client’s mental status and notes would place the client at risk because of lack of timely treatment. Options 3 and 4 would not
some disorientation and behavioral changes. address the current complication.
Significant pedal edema is also present. What should
be the nurse’s next action?

1.‐ Check vital signs and notify the physician.


2.‐ Have the client return in the morning for reevaluation.
3.‐ Instruct the client to limit salt intake for a few days.
4.‐ Suggest that the client change the route of administration to subcutaneous injections.

117 The nurse is assigned to the care of a client receiving Correct answer: 2, 5 Simple activities and nursing procedures can be delegated to the UAP. For this client, this Recall that the RN does not delegate assessment, teaching, and counseling and evaluate
radiation therapy for cancer. Which of the following would include ambulation and documentation of intake and output. The RN retains each of the options in relation to these guidelines.
activities needed in the care of a client receiving responsibility for assessment, teaching, and counseling the client. For this reason, the nurse
external beam radiation therapy could be safely should not delegate assessment of the skin at the treatment site, patterns of fatigue, or how
delegated to an unlicensed assistive person (UAP) the client is coping with the diagnosis and treatment.
working on the nursing unit? Select all that apply.

1.‐ Observe the skin site following a treatment session.


2.‐ Document intake from the meal trays.
3.‐ Assess variations in level of fatigue during the shift.
4.‐ Explore how the client is coping with treatment.
5.‐ Assist the client to ambulate in the hall.

118 A 76‐year‐old woman visits the ambulatory clinic with Correct answer: 4 Visual difficulty caused by distortions and impairment of central vision is common with Specific knowledge of the various visual disorders is needed to answer the question.
reports of having difficulty reading and doing macular degeneration. Peripheral vision in most cases is normal. The symptoms are not Eliminate options 2 and 4 first because of the client’s description. Then choose correctly
needlework because of visual distortions with blurring characteristic of glaucoma (loss of peripheral vision), cataracts (gradual deterioration of vision from the remaining two options because of the nature of the disorder.
of images directly in the line of vision. The peripheral with opacity of lens), or detached retina (sudden change in vision with a sense of a curtain
vision assessment by the nurse yields normal findings. falling over the field of vision).
The nurse suspects that this client is experiencing
which of the following visual problems?

1.‐ Glaucoma
2.‐ Detached retina
3.‐ Cataracts
4.‐ Macular degeneration
119 A female client states that she will not undergo any Correct answer: 1 An ultrasound is the only noninvasive procedure listed. The others require swallowing (option The core issue of the question is knowledge of noninvasive diagnostic tests for the
invasive testing for her “stomach pain.” The nurse 2) or injecting (option 4) contrast, or insertion of an endoscope (option 3). gastrointestinal system. Eliminate each of the incorrect options because of the words or
explains that which of the following tests could be suffixes swallow in option 2, ‐oscopy in option 3, and contrast in option 4. These all imply
completed to assess the abdomen and still meet the that the test will be intrusive to the body.
client’s wishes?
1.‐ Abdominal ultrasound
2.‐ Barium swallow
3.‐ Colonoscopy
4.‐ CT scan with contrast

120 Certain that her stomach pain is a symptom of Correct answer: 2 When a client with a somatization disorder does not receive symptom relief, anxiety The core issue of the question is the ability to determine that the basis for the client’s
cancer, a female client with somatization disorder increases (as evidenced by her current symptoms). Although the client may experience pain, agitation is anxiety. The critical words in the stem of the question are somatization
exhibits pressured, rapid speech; elevated pulse and hopelessness, and disturbed body image, the major issue is anxiety. disorder. Review this topic area if this question was difficult.
blood pressure; palpitations; and preoccupation with
her pain, despite negative results from a gastroscopy.
The nurse formulates which of the following as the
priority nursing diagnosis?

1.‐ Pain
2.‐ Anxiety
3.‐ Hopelessness
4.‐ Disturbed body image

121 A client is taking an over‐the‐counter preparation Correct answer: 1 Bismuth‐containing preparations, such as Pepto‐Bismol, can cause all the listed side effects, The critical word in the stem of the question is unique. With this in mind, use the process
containing bismuth subsalicylate (Pepto‐Bismol) for but transient darkening of the tongue and stool is a specific side effect to bismuth. of elimination and knowledge of drug components to determine which side effect is caused
diarrhea. Which of the following side effects would a by bismuth. As an alternative strategy, select option 1 because it is the only one that is
nurse monitor for that is unique to the bismuth located in the very upper GI tract.
portion of this drug?

1.‐ Darkening of the tongue


2.‐ Dyspepsia
3.‐ Abdominal pain
4.‐ Diarrhea

122 The nurse is taking a nursing history from the mother Correct answer: 1 Acute episodes are characterized by bulky, frothy stools and steatorrhea because of The core issue of the question is the manifestations of celiac disease that occur because of
of a child being admitted with flare‐up of celiac malabsorption, anorexia, and irritability. The client would not exhibit increased appetite the underlying pathophysiology. Recall that this disorder is characterized by malabsorption
disease. What piece of information would the nurse (option 2), cheerful behavior (option 3), or soft, formed stools (option 4). of key nutrients to help eliminate incorrect options.
expect the mother to report?
1.‐ Steatorrhea
2.‐ Increased appetite
3.‐ Cheerful behavior
4.‐ Soft, formed stools

123 A nurse is discussing the home maintenance regimen Correct answer: 1 Regular exercise can help to normalize bowel function. Cigarette smoking and gum chewing Use knowledge of healthy lifestyle habits that stimulate normal bowel function as a means
with a client who has irritable bowel syndrome. Which increase swallowed air; fresh vegetables are gas‐producing. of answering this question. Eliminate options 2 and 3 first as least helpful in health
of the following statements indicates client promotion. Choose option 1 over 4 because excessive fresh fruits and vegetables could
understanding? aggravate irritable bowel syndrome.
1.‐ “I’ll take a walk after dinner each evening.”
2.‐ “I’ll have a cigarette after meals to relax.”
3.‐ “I’ll chew gum between meals to curb my appetite.”
4.‐ “I’ll eat a lot of fresh vegetables and fruits.”
124 A primigravida client of 16 weeks gestation states Correct answer: 4 The embryo’s muscles spontaneously contract beginning at 7 weeks. The mother perceives The core issue of the question is knowledge of fetal growth and development. An easy way
that she has not yet felt fetal movement. The nurse’s sensations of movement of the fetus from 16 to 20 weeks gestation. A primigravida usually to remember this information is to equate 4 weeks to be one month and then remember
best response is: perceives movement closer to 20 weeks. movements are felt at 4 to 5 months (16 to 20 weeks).
1.‐ “Your fetus will move any day now. Call me in a week if you don’t feel it.”
2.‐ “Your fetus will begin moving at about 20 weeks gestation.”
3.‐ “You should have been feeling the movement already.”
4.‐ “Your fetus has been moving for the past 9 weeks without you feeling it. You will feel it within a month.”

125 The mother of an infant who underwent surgery to Correct answer: 1 A double‐diapering technique will help to protect a urinary stent following repair of The core issue of the question is the rationale for a specific diapering technique following
repair hypospadias asks the nurse why the infant is hypospadias or epispadias. The inner diaper collects the infant’s stool, while the outer one surgery to correct hypospadias. Eliminate option 2 first as least realistic and choose the
diapered as shown. The nurse would respond that this collects urine. correct option after determining which option best reflects safety considerations and
method of diapering will help to: protection of the surgical area.

1.‐ Protect the urinary stent that has been put in place.
2.‐ Adequately measure the urinary output.
3.‐ Provide for maximum absorption of urine.
4.‐ Provide optimal protection of perineal skin from infected urine.

126 Following the administration of a Correct answer: 1 An inspiratory stridor is indicative of a hypersensitivity reaction to the DPT immunization and The core issue of the question is recognition that stridor following immunization is a sign
diphtheria/pertussis/tetanus (DPT) immunization the epinephrine should be administered to counteract the symptoms of the allergic response. of hypersensitivity to the drug. With this in mind, use the process of elimination to select
nurse notes that the infant has inspiratory stridor. The Options 2 and 3 are irrelevant, and option 4 places the infant at risk for injury or death. option 1 as the answer, since epinephrine is the drug treatment of choice.
nurse should take which of the following actions?

1.‐ Administer epinephrine as per protocol orders.


2.‐ Evaluate for pulmonary edema.
3.‐ Inspect for periorbital edema.
4.‐ Assess the baby again in 15 minutes.

127 The nurse is talking with the unlicensed assistive Correct answer: 4 To coach is to give direction and suggestions for improvement. Option 4 illustrates this The critical word in the stem of the question is coach. Use the ordinary definition of the
person (UAP) about time management skills and concept. Option 1 is threatening rather than coaching. Option 2 is a criticism without a word and choose the option that gives suggestions or advice to improve performance.
techniques. Which of the following statements would suggestion for improvement. Option 3 is helpful as a statement of positive reinforcement but
the nurse make if intending to act as a coach? does not specifically give direction for future actions.

1.‐ “You must get the vital signs taken on time or you will be disciplined.”
2.‐ “You never report morning blood glucose levels on time.”
3.‐ “Your timely response to clients’ call lights is exemplary.”
4.‐ “It may be helpful if you bring in linens to the client rooms when you restock the gloves.”

128 A nurse is explaining to a woman considering Correct answer: 3 The nurse would determine that the client understood the information if the client stated Knowledge about the transmission of rubella and the elements of each type of
pregnancy how rubella is transmitted. The nurse rubella is transmitted by the droplet route. Clients with rubella are placed in droplet transmission‐based precautions is required. Select an option based on nursing knowledge.
determines that the teaching session had the desired precautions, as the causative agent is transmitted by particle droplets larger than 5 microns.
outcome if the client states that rubella is transmitted The other responses are factually incorrect.
by:
1.‐ The airborne route.
2.‐ Contaminated food.
3.‐ The droplet route.
4.‐ Direct contact.

129 A female client has been successfully resuscitated Correct answer: 2 A pH of 7.6 indicates an alkalotic state. The administration of bicarbonate would be the best First recall that a pH of 7.6 is alkalotic to eliminate options 1 and 3. Next eliminate option
after cardiac arrest. Her arterial blood gas reveals a pH answer. Anaerobic metabolism and the production of lactic acid lead to an acidotic state, 4 because the result is not normal. Alternatively associate the high pH with the drug sodium
of 7.6. The nurse attributes this result to which of the explaining why blood gases drawn during a code usually show acidosis. This pH is not within bicarbonate, which raises pH.
following? normal limits.
1.‐ Anaerobic metabolism, which caused lactic acid production
2.‐ Excess sodium bicarbonate, which was administered during the resuscitation
3.‐ Repeat blood gases, which are performed during a code, frequently show acidosis
4.‐ Normal blood gas results

130 The nurse would anticipate finding which of the Correct answer: 2 Characteristics of a client with pain disorder include believing there is a physical cause for The critical words in the stem of the question are anticipate and pain disorder. With this in
following client characteristics when working with a distress when there is no organic basis, the need to use analgesics or drugs to reduce pain, and mind, determine that the core issue of the question is characteristics that are compatible
client who has a pain disorder? impaired role performance. with this disorder. Use nursing knowledge and the process of elimination to make a
selection.
1.‐ A preference to handle pain without medication
2.‐ A lack of understanding of the relationship between pain and stress
3.‐ Adequate role performance
4.‐ Structural damage at the site of pain

131 Which of the following should be the highest priority Correct answer: 1 The client must understand the medication information as a priority item. Option 2 is a false The critical words in the stem of the question are highest priority. This tells you that more
of the education plan for a client being treated with statement. Effective medication dosing should control seizure activity (option 4). Teaching that than one option may be factually correct and that you must choose the most important
medication therapy for a generalized seizure disorder? urine may turn pink to brown may be included if appropriate, but is not the highest priority. item. Recall that insufficient drug therapy may lead to seizure recurrence to help you select
appropriately.
1.‐ Take medication even if there is no seizure activity.
2.‐ Physical dependency may result from extended use of medications.
3.‐ Urine may turn pink to brown but is not harmful.
4.‐ Therapeutic effects of medications may not be seen for 2 to 3 weeks.

132 The pediatric nurse needs to rearrange room Correct answer: 2, 5 The child with the low white blood cell count (normal Examine the clients in the question and determine similarities and differences among
assignments of clients to accommodate three 5,000–10,000/mm<sup>3</sup>) and the child receiving chemotherapy are at risk them. The two that are the most similar and that have the most similar needs from an
additional clients who will be admitted during the day. for infection and could be cohorted together because they should both be on neutropenic infection control perspective are the ones who should be placed together.
Which two of the following clients would be best for precautions. The child who underwent appendectomy should be separated from the children
the nurse to place together in the same room? Select with viral encephalitis and scarlet fever. The children with infections should not be cohorted
all that apply. because one is viral (encephalitis) and one is bacterial (scarlet fever) in origin.

1.‐ An 8‐year‐old who has encephalitis


2.‐ A 10‐year old who has a white blood cell count of 2,800/mm3
3.‐ A 12‐year‐old who had an appendectomy
4.‐ An 11‐year‐old with scarlet fever
5.‐ A 9‐year‐old receiving chemotherapy for cancer

133 A 32‐year‐old female client who is HIV‐positive is Correct answer: 4 A client who is HIV‐positive (regardless of sex) is likely to lose weight due to repeated cycle of Analyze each of the options and choose the one that has the most direct and positive
receiving treatment at an outpatient clinic. The nurse wasting and malnutrition. The client, who may be unable to merely increase caloric intake, impact on weight gain. Using this strategy, you can systematically eliminate each of the
reviewing the dietary assessment record notes that should be instructed in dietary techniques that maximize quality of intake. Option 1 is incorrect options.
the client has been skipping meals and progressively incorrect—even though a food diary would provide pertinent information, the response allows
losing weight. What dietary interventions would be for a delay in treatment that could result in further weight loss for the client. The priority is to
best for the nurse to suggest to promote weight gain? intervene early on to prevent the onset of wasting. Option 2 is incorrect because it provides
the client with a false belief that fluid retention changes associated with the menstrual cycle
may have an impact on nutritional status. Option 3 is incorrect—even though increased salt in
the diet can lead to fluid retention and weight, it does not address the underlying issue of
nutritional balance.

1.‐ Have the client keep a food diary and submit it at the next visit so that more information can be obtained regarding food preferences and usual dietary pattern.
2.‐ Tell the client that her weight may fluctuate in response to her menstrual cycle so there is no need to worry for now.
3.‐ Tell the client that additional salt in the diet will help to increase weight.
4.‐ Tell the client that the use of nutrient‐dense food and fortified protein shakes will help promote weight gain.
134 The nurse would assess a 76‐year‐old client for which Correct answer: 4 Polypharmacy is using multiple doctors and multiple pharmacies to get the health care Although taking medications on one’s own in combination with prescriptive meds can lead
common problems that most increases the risk for needed often from a variety of specialists. The overall problem is that different doctors may to problems, a greater problem is the polypharmacy issue. Sharing meds is also done in
major complications of heart and lung disease? not know what other doctors had ordered. Some drugs may interact with others and others some adults when they want to assist another by offering them a medication that helped in
may be the same drug in a different form. Overdosing and interactions become more common their case. Financial issues may come into play as adults share meds, but this is also a
with this problem. smaller issue than polypharmacy. Taking the medications as ordered will not increase the
risk of complications; it should reduce that risk.

1.‐ Taking over‐the‐counter meds with prescription meds


2.‐ Sharing medications with family and friends
3.‐ Following directions exactly and taking medications on a regular basis
4.‐ Polypharmacy resulting from visits to multiple doctors

135 A client with acute respiratory distress syndrome Correct answer: 4 Placing the client with ARDS in a prone position allows for expansion of the posterior chest The core issue of the question is an intervention that may increase oxygenation in a client
(ARDS) shows no improvement despite increases in the wall, which may be effective in enhancing oxygenation. Transfusing red blood cells or albumin with ARDS. Note the critical words nursing intervention to eliminate options 1 and 2, which
concentration of oxygen administered. What does not increase oxygenation in ARDS. Option 3 should have been done as an initial measure. require a physician’s order. Choose option 4 over 3 because it allows for expansion of the
intervention should the nurse attempt which may back side of the client’s lungs, and redistribution of blood flow using gravity.
improve ventilation‐perfusion matching?

1.‐ Transfusion of packed red blood cells


2.‐ Infusion of albumin
3.‐ Positioning supine with head elevated 30 to 45 degrees
4.‐ Prone positioning

136 The nurse is giving general information about Correct answer: 3 Because ACE inhibitors can cause fetal harm or death, they should be discontinued as soon as The core issue of the question is knowledge that ACE inhibitors need to be avoided during
antihypertensive medications to a young female client pregnancy is detected. Their effect on breastfeeding infants is unknown. The effect of other pregnancy because they are harmful to the fetus. Use knowledge of drug therapy and the
with a history of hypertension. The nurse includes that medications is unknown during pregnancy. process of elimination to make a selection.
which of the following types of antihypertensives
should not be used if the client becomes pregnant?

1.‐ Vasodilators
2.‐ Diuretics
3.‐ Angiotensin converting enzyme (ACE) inhibitors
4.‐ Calcium channel blockers

137 A nurse from the pediatric intensive care unit has Correct answer: 2 The pediatric intensive care nurse is more likely to have experience working with heart Note the critical words pediatric intensive care in the stem of the question. Then
floated to the cardiovascular intermediate care unit failure, since children can experience heart failure secondary to cardiac defects. Myocardial determine which client has the health problem that could also be experienced in the
for the shift. Which of the following clients would the infarction, stent placement, and coronary atherectomy are problems and procedures done for pediatric population.
nurse assign to the float nurse for the shift? adult clients.

1.‐ A client who experienced myocardial infarction 36 hours ago


2.‐ A client in heart failure receiving digoxin (Lanoxin) and bumetanide (Bumex)
3.‐ A client who just underwent coronary atherectomy
4.‐ A client scheduled for cardiac stent placement later in the day

138 A client is admitted to the pre‐surgical area before Correct answer: 1 The priority nursing intervention is one that maintains contact of the retina with the choroid The critical words in the question are actions and first. This indicates that more than one
undergoing surgery to repair a detached retina. The by positioning the client so the detached area falls against the choroid. It is unnecessary to option may be correct but that one is more important than the others. Use knowledge of
admitting nurse would take which of the following darken the client’s immediate environment. A preoperative medication may be ordered, but pathophysiology to make the correct selection.
actions first? has lesser priority than maintaining proper position of the head to protect the eye. Both eyes,
not just the affected eye, are patched to minimize eye movement.

1.‐ Position the client properly.


2.‐ Darken the bedside area of the client.
3.‐ Administer the prescribed preoperative analgesic.
4.‐ Cover the affected eye.

139 A client has been admitted to the nursing unit with a Correct answer: 3 Loss of potassium caused by vomiting and diarrhea, in addition to lack of replacement intake, Critical words in the question are vomiting and diarrhea. With this in mind, recall that
three‐day history of severe nausea and vomiting with will lead to a risk for hypokalemia (normal range is 3.5–5.1 mEq/L). Calcium levels (normal potassium may be lost from the GI tract. Eliminate option 2 first because it is within normal
diarrhea. The client is experiencing fatigue, anorexia, 9–11 mg/dL) are not affected by vomiting and diarrhea and the sodium level (normal 135–145 range, and then eliminate the calcium levels as less relevant to the question than
and muscle weakness. Based on this history, which mEq/L) will be elevated with the loss of potassium. potassium.
laboratory findings should the nurse expect to find?

1.‐ Calcium 11.6 mg/dL


2.‐ Sodium 144 mEq/L
3.‐ Potassium 2.9 mEq/L
4.‐ Calcium 7.4 mEq/L

140 The nurse would select which of the following as a Correct answer: 1 The client who has many physical complaints with no organic basis is not conscious of A key phrase is many physical complaints and a critical word is priority. With these in
priority nursing diagnosis for a client who has many conflicts and stressors, and is, therefore, unable to use other means to cope with anxiety. mind, use the process of elimination to select the nursing diagnosis that is compatible with
physical complaints that are not supported by There is no evidence of impaired adjustment or verbal communication. Nothing in the stem of the client information as stated. It is important not to read into the question.
diagnostic test evidence? the question specifically states that the client is in pain.
1.‐ Ineffective individual coping
2.‐ Impaired adjustment
3.‐ Impaired verbal communication
4.‐ Pain

141 A client presents to the Emergency Department with Correct answer: 2 Epinephrine is a beta‐adrenergic agent that has beta 1 adrenergic action, causing increased The core issue of the question is knowledge of the timeframe for the onset of action with
inspiratory and expiratory wheezes and intercostal heart rate and increased force of myocardial contraction. The results of subcutaneous epinephrine. Use specific drug knowledge and the process of elimination to make a
retractions. A diagnosis of acute bronchospasm epinephrine should be seen in 5 minutes. The effects may last up to 4 hours. The other options selection.
secondary to acute bronchitis is made. Epinephrine are incorrect.
(Bronkaid) is ordered to be given subcutaneously. The
nurse would anticipate seeing the intended effect of
the medication in:

1.‐ 1 minute.
2.‐ 5 minutes.
3.‐ 10 minutes.
4.‐ 15 minutes.

142 During a scheduled exam the client’s glycosylated Correct answer: 1 Glycosylated hemoglobin is elevated due to long‐term hyperglycemia. Values greater than 8 Recall that this test is a general indicator of diabetic control over several weeks. With this
hemoglobin was found to be 9%. The client has had percent indicate consistently poor control of blood glucose and the need to assess the client’s in mind, eliminate options 2 and 3 first. Choose option 1 over 4 because it relates to long‐
diabetes mellitus for 3 years. The nurse should do dietary pattern for the past several months in relation to the treatment plan. The other term control, not immediate control.
which of the following? options do not apply.
1.‐ Explore the client’s general dietary pattern for the past 4 months.
2.‐ Assess for signs of infection and client’s intake for the past 24 hours.
3.‐ Review the client’s understanding of diabetic foot care.
4.‐ Immediately give sliding scale insulin medication.

143 A client is admitted to the hospital with a primary Correct answer: 4 Sjögren’s syndrome is an autoimmune disease that destroys exocrine glands in the body, and To answer this question correctly, it is necessary to know the underlying pathophysiology
diagnosis of hip fracture and a secondary diagnosis of leads to a generalized “dryness” of body systems. The restriction of fluids is a concern because of Sjögren’s syndrome. From there, analyze each of the options that could exacerbate or
Sjögren’s syndrome. Which one of the following orders the use of fluids helps to keep the oral cavity moist. There is no information to suggest that the worsen the underlying disease state.
would be of most concern with regard to the client has a need for fluid restriction due to other disease processes so this order should be
nutritional status of the client? clarified. All of the other options are reasonable for this client.

1.‐ NPO after midnight for surgery with a 7:30 a.m. case
2.‐ IV of lactated Ringer’s at 125 mL/hr
3.‐ Maintain diet as tolerated
4.‐ Restrict oral fluids to 1,000 mL/day

144 The nurse notes on the antepartal history that the Correct answer: 2 An android pelvic structure is narrow in both the anterior‐posterior diameter and the lateral First determine the significance of the critical word android in the stem of the question.
client has an android pelvis. The nurse plans to assess diameter, and can cause a prolonged labor with a large fetus or a malpositioned fetus. Eliminate options 3 and 4 first because they relate least to risks during labor caused by
this client carefully because of the increased risk of bone structure. Choose option 2 over 1 because the prefix andr‐ refers to males and from
which of the following? there determine that it indicates a narrower pelvis.
1.‐ Occiput posterior position
2.‐ Prolonged labor
3.‐ Precipitous delivery
4.‐ Developing postpartum complications

145 The nurse would utilize which of the following Correct answer: 1 Physical exercise, within the client’s ability level, reduces muscle tension and pain. Note that critical words in the question are chronic pain and cope. This indicates that the
interventions when caring for a client with chronic pain Additionally, exercise creates a feeling of greater self‐efficacy. Verbal expression of conflicts correct answer is an activity that will help the client tolerate the pain to a greater extent.
disorder to help that client cope with the disorder? and minimal use of analgesics are also indicated. Complete bedrest would not be indicated Eliminate options 3 and 4 first as most extreme, and then choose option 1 over 2 because
unless required by incapacitating conditions, but there is no evidence that this is the case in of the physiological benefits.
this question.
1.‐ A program of physical exercise
2.‐ Music therapy for expression
3.‐ Patient‐controlled analgesia pump
4.‐ Complete bedrest

146 A client receiving hydroxyamphetamine (Paredrine) Correct answer: 4 Confusion and increased heart rate are signs of toxicity or adverse side effects of The core issue of the question is knowledge of adverse drug effects. Use specific drug
for open‐angle glaucoma demonstrates an hydroxyamphetamine. Stinging, headache, and brow ache are usual side effects of knowledge and the process of elimination to make a selection.
understanding of the medication’s serious side effects hydroxyamphetamine.
when informing the health care provider of which of
the following symptoms?

1.‐ Stinging on instillation


2.‐ Occasional headache
3.‐ Occasional brow ache
4.‐ Confusion

147 The nurse is seeking employment in a hospital that Correct answer: 3 Shared governance is based on the philosophy that nursing practice is best determined by The critical words in the question are shared governance. Choose the option that gives the
uses a shared governance model. The nurse should nurses. Option 1 represents standard nursing practice. Option 2 is unrelated to governance. best evidence of some kind of sharing.
accept a job offer in the hospital that has which of the Option 4 represents leadership input into decision‐making for the organization.
following attributes?
1.‐ Staff nurses delegate activities to certified nursing assistants (CNAs).
2.‐ A unit manager seeks advice from her supervisor.
3.‐ Staff nurses and CNAs make their own schedules.
4.‐ Procedure manuals are written by a committee of nurse managers.

148 The nurse observes an unlicensed assistive person Correct answer: 4 The employee should limit the amount of time in the client’s room to minimize exposure. In The wording of the question indicates that something was done incorrectly. Use the
(UAP) in the room of a client with severe acute option 1, the employee is wearing the correct combination of personal protective equipment. process of elimination after noting that options 1, 2, and 3 are correct actions. Only option
respiratory syndrome (SARS). Which of the following In option 3, the employee has followed the correct procedure for exiting the client’s room. 4 identifies an incorrect action.
actions by the UAP indicates intervention and further Equipment required for the care of the isolation client should remain in the client’s room to
teaching by the nurse is needed? limit exposure to other clients on the nursing unit.

1.‐ The UAP wears a protective gown, gloves, N95 respirator, and eye protection when entering the room.
2.‐ The UAP does not remove the stethoscope, blood pressure cuff, and thermometer being kept in the room.
3.‐ The UAP removes all personal protective equipment and washes the hands right before leaving the client’s room.
4.‐ The UAP visits with the client for 25 minutes.

149 The nurse examines the white blood cell (WBC) Correct answer: 3 Eosinophils are responsible for responding to allergic reactions. Neutrophils and monocytes The core issue of the question is knowledge of the various components of the WBC
differential for a client who experienced a severe are primary responders to infection and tissue injury and inflammation. Lymphocytes assist in differential and their significance. Specific knowledge is needed to answer the question so
allergic reaction. The nurse anticipates that which of immune responses. take time to review if you have the need.
the following values will be elevated?

1.‐ Neutrophils
2.‐ Monocytes
3.‐ Eosinophils
4.‐ Lymphocytes

150 The partner of a client who has dissociative identity Correct answer: 1 All family members are affected by dissociative identity disorder. Children must also find ways The core issue of the question is an understanding of the purposes and benefits of family
disorder with several alters is puzzled about why the to understand and deal with what is occurring to a parent, rather than denying what is obvious therapy. Use knowledge of family dynamics to choose the correct answer.
children are included in family therapy. Which of the or proceeding on incorrect assumptions that are not challenged by accurate information.
following would be the best explanation for the nurse
to offer?
1.‐ “Children need to have their experiences confirmed—and to learn to deal with the different personalities.”
2.‐ “There is probably a mistake in the referral; your partner is the one who has the problem.”
3.‐ “You and your partner should be seen, but it could be traumatizing to the children.”
4.‐ “It would be best to ask the children if they would like to participate, and bring them if they want.”

151 The nurse assesses the results of a vancomycin Correct answer: 4 A serum specimen for peak level is drawn 15 to 30 minutes after IV administration to test for The core issue of the question is the ability to draw correct conclusions about the
(Vancocin) blood level drawn just prior to the next toxicity. Trough drug levels are drawn just prior to administration of the next IV dose to significance of serum drug level results. Focus on the words just prior to in the stem of the
scheduled intravenous (IV) dose. The nurse would measure whether satisfactory therapeutic levels are being maintained. If the peak is too high, question, which tells you that it is the trough level that is being described. With this in mind,
collaborate with the prescriber after drawing which of toxicity can occur and the dose needs to be reduced and/or the frequency of administration use the process of elimination to find the conclusion that is true of a need to collaborate
the following correct conclusions about the result? extended. If the trough is too low, then the dosage and/or frequency of administration need to about the trough level.
be increased.
1.‐ There is a high serum level, indicating the peak level is too high.
2.‐ This test measures the highest therapeutic concentration and it is low.
3.‐ Toxicity is evident, suggesting the drug’s half‐life is too short with the frequency prescribed.
4.‐ The drug level is low, indicating the drug dosage and/or frequency should be increased.

152 In a child with acute renal failure, the nurse would Correct answer: 1 Potatoes, tomatoes, and oranges have a high level of potassium content. The others have The core issue of the question is knowledge of foods that are high in potassium. Eliminate
help to prevent hyperkalemia by limiting which of the lesser amounts of potassium in them, when considering the groupings of foods in each option. options 3 and 4 first because of the carbonated beverages and sugars, respectively. Choose
following foods in the diet? option 1 over 2 because these foods have a greater potassium content.

1.‐ Potatoes, tomatoes, and oranges.


2.‐ Grains, cheese, and citrus fruits.
3.‐ Cereals, processed sugars, and wheat.
4.‐ Rice, leafy green vegetables, and carbonated beverages.

153 A 28‐year‐old female client has recently been Correct answer: 4 A client who receives a diagnosis of SLE will be profoundly affected by the chronic nature of The core issue of the question is what are initial priorities of care when a client is
diagnosed with systemic lupus erythematosus (SLE). this autoimmune disease process. The establishment of a healthcare team using a diagnosed with a chronic illness in which the client’s condition is expected to worsen over
Which of the following would be most helpful for the multidisciplinary approach will help the client to identify and realize individual goals. Even time. With this in mind, choose the option that calls together the interdisciplinary team so
overall management of care? though the initiation of advance directives is important, it is not the priority concern at this that the client has the fullest range of resources at hand.
point in time—there is no information provided to suggest that the client requires immediate
activation of advance directives. Even though it is important to discuss the progressive effects
of the disease, the priority is to establish a multidisciplinary team to assist the client. Option 3
is incorrect—telling the client to limit her work pattern may not be financially feasible or
physically indicated at this time.
1.‐ Have the client institute advance directives immediately.
2.‐ Discuss with the client lifestyle modifications that will be needed as the disease progresses.
3.‐ Ascertain information about the client’s working environment and suggest limiting work schedule to minimize potential stress.
4.‐ Establish the multidisciplinary healthcare team to help client identify goals.

154 The nurse is leading a support group for adult Correct answer: 2 Allowing independence as long as possible gives dignity and self‐worth to clients. Option 1 is Completely taking over all aspects of an adult’s life does not give value or worth to those
children of aging parents who have come to live in not helpful because it does not foster independence within the scope of remaining abilities. adults, especially if done too prematurely. Allowing them to do whatever they want to do
their home because of deteriorating health. Which of Option 3 could result in harm to the parents. Option 4 could be degrading and does not foster may not be safe for them and harm may be done despite saving some self‐esteem. Financial
the following principles does the nurse encourage the maintaining independence within limits of current ability. issues are the most worrisome issues that must be dealt with, and taking them over also
group members to follow to promote quality of life for removes the independence of the client. A plan of care needs to be clarified when the adult
all concerned? is thinking clearly and can delegate or make an advanced directive.

1.‐ Do as much as possible for aging parents to prevent problems from occurring.
2.‐ Allow independence in those things that are safe or with minimal risk of harm.
3.‐ Let the parents do whatever they want as a means to maintain their self‐esteem.
4.‐ Take over responsibility for making important decisions to avoid major financial losses.

155 A client of 26 weeks gestation experiences a partial Correct answer: 1 One of the major functions of the placenta is provision of nutrients to the fetus across the To answer this question correctly, it is necessary to recall the function of the placenta to
placenta abruptio. She asks, “Will this harm my baby?” placenta membrane. An interference with the placenta circulation, such as abruptio placentae, deliver oxygen and nutrients to the fetus. Focus on the critical word partial to aid in
The nurse responds that this may: impairs this ability. Another important function is removing metabolic waste from the fetus. selecting the correct option.
While this takes place metabolically the fetus produces and excretes urine independently of
the placenta. Hydrops is gross fetal edema related to hemolytic action, not placenta
dysfunction. Anomalies usually occur in the first trimester when organogenesis occurs.

1.‐ Decrease the amount of nutrients the fetus receives.


2.‐ Cause a buildup of urine in the fetus, causing kidney damage.
3.‐ Cause the fetus to develop hydrops.
4.‐ Cause a fetal anomaly.

156 The nurse writes on the worksheet for the shift to Correct answer: 3 Clients who are taking cholestyramine (which is a bile resin) should be monitored for fat‐ The core issue of the question is knowledge that cholestyramine places the client at risk
assess a client taking cholestyramine (Questran) for soluble vitamin deficiencies (Vitamins A, D, E, and K), as the gastrointestinal side effects of the for deficiency of fat‐soluble vitamins. Use the process of elimination and reason that this
signs of possible deficiency of which vitamins? medication can lead to reduced absorption. Niacin, thiamine, folic acid, cyanocobalamin, and answer is correct because the action of cholestyramine is to bind onto cholesterol (fat) and
Vitamin C (options 1, 2, and 4) are all examples of water‐soluble vitamins. prevent its absorption into the GI tract.

1.‐ Niacin and thiamine


2.‐ Folic acid and Vitamin C
3.‐ Vitamins A and D
4.‐ Thiamine and cyanocobalamin

157 Case management has become an important nursing Correct answer: 3 Clients with less complex and more common diagnoses are selected for case management. Focus on the critical words case management. Use the common definition of this method
care model in the 21st century. Which of the following The clients in the remaining options have problems that are more likely to have variation in to eliminate each option systematically.
clients would most likely be selected for case their conditions (options 1 and 2) or have a less common diagnosis (option 4).
management?
1.‐ A 21‐year‐old male with a gunshot wound in the ER
2.‐ A 32‐year‐old male with a fractured pelvis
3.‐ A 75‐year‐old female awaiting a hip replacement
4.‐ A 41‐year‐old male coming in for outpatient tonsillectomy
158 While making rounds, the nurse observes a client Correct answer: 2 The client in the photograph is receiving oxygen through a Venturi mask. Oxygen Specific knowledge is needed to answer the question. Reflect on the various modes of
receiving oxygen by this mode. The nurse concludes administered by a Venturi mask can be regulated to deliver between 24% and 50%, which is a oxygen delivery and note that this type of device can be regulated easily because it is a
the client is using this mode of oxygen therapy because benefit for clients who require higher oxygen supplement without mechanical ventilation. The mask and because the percentage of oxygen can be manipulated easily.
of which primary benefit? Venturi mask does not prevent rebreathing of carbon dioxide, as does a non‐rebreather mask.
Oxygen concentration of 100% would be administered to COPD clients only in rare
circumstances via mechanical ventilation.
1.‐ The ability to prevent rebreathing of exhaled carbon dioxide.
2.‐ Oxygen concentration can be regulated.
3.‐ Constant humidity can be administered to liquefy pulmonary secretions.
4.‐ The ability to deliver up to 100% oxygen concentration for clients with COPD.

159 A client diagnosed with hypochondria states an Correct answer: 1 MRI is the only diagnostic examination listed that does not possibly require the ingestion or The core issue of the question is knowledge of which tests do and do not require use of
allergy to the dyes used in diagnostic tests and “all” administration of contrast or radioactive material. Options 2 and 4 involve the use of contrast some form of contrast media. With this in mind, eliminate each of the incorrect options
radioisotopes. The nurse explains that the client could dyes or agents, while option 3 uses a radioisotope. using basic knowledge of diagnostic tests. Take time to review this material if you had
undergo which diagnostic procedure without risk of difficulty selecting.
possible allergic response?
1.‐ Magnetic resonance imaging (MRI)
2.‐ Myelogram
3.‐ Ventilation/perfusion (VQ) scan
4.‐ Computed tomography (CT) scan with contrast

160 A client had assumed a new identity and gained Correct answer: 4 Fugue states are characterized by wandering or moving away from one’s familiar place with The core issue of the question is the ability to correctly interpret a client’s behavior as
employment when he was found 400 miles away from an amnesia for the complete past, including self. The person often assumes a new identity for characteristic of a fugue state. Use knowledge of characteristics of this mental health
his home. The mental health nurse interprets that this the duration of the fugue. Amnesia is simply a loss of memory owing to brain damage or to disorder and the process of elimination to make a selection.
client’s behavior is characteristic of: severe emotional trauma. Akathisia is an abnormal condition characterized by restlessness and
agitation. Confabulation is replacement of gaps in memory with imaginary information.

1.‐ Amnesia.
2.‐ Akathisia.
3.‐ Confabulation.
4.‐ Fugue state.

161 The client is receiving a loading dose of lidocaine Correct answer: 1 Lidocaine is given via IV push in doses of 1 to 1.5 mg/kg. The initial loading dose (bolus) is The core issue of the question is knowledge of therapeutic protocols for intravenous
(Xylocaine) 100 mg IV for treatment of ventricular intended to achieve adequate blood levels to suppress ventricular dysrhythmias and is antidysrhythmic medications such as Lidocaine. Use drug knowledge and the process of
tachycardia. The nurse prepares to take which action followed by an infusion of 1 to 4 mg/min via infusion pump. The initial bolus lasts elimination to answer the question.
next? approximately 10 minutes so the infusion must not be delayed. The dose may be repeated 1
time under certain conditions, but the total dose should not exceed 3mg/kg. Oral therapy and
pacemaker insertion are not indicated at this time.
1.‐ Start a continuous IV infusion at 1 to 4 mg/minute.
2.‐ Repeat the dose every 10 minutes for 1 hour or PRN.
3.‐ Begin oral procainamide (Pronestyl) therapy.
4.‐ Prepare for pacemaker insertion to override the dysrhythmia.

162 The nurse on a cardiac medical unit has an unlicensed Correct answer: 2 The nurse should delegate the activity that is procedural in nature, which is within the scope Keep in mind the principles of delegation. Recall that the nurse does not delegate
assistive person (UAP) assigned to the nursing team. of training of the UAP. The nurse does not delegate teaching (options 1 and 3) or interventions assessment, teaching, or medication administration to a UAP.
The nurse would delegate which of the following client for chest pain (option 4).
care activities to the UAP?

1.‐ Assist the client to choose low‐fat and low‐sodium food selections from the dietary menu.
2.‐ Measure client’s pulse, blood pressure, and oxygen saturation after ambulation.
3.‐ Explain the need to alternate activity periods with rest.
4.‐ Help the client use nitroglycerin left at the bedside if chest pain occurs.
163 A client recovering from Guillain‐Barré syndrome is Correct answer: 2 A client who is recovering from Guillain‐Barré syndrome will need a diet that promotes The key words in the stem of the question are general rehabilitative care. This tells you
admitted to the rehabilitation unit for general positive nitrogen balance in order to counteract the effects of long periods of immobility on that the client has no specific deficits that would affect nutritional status. With this in mind,
rehabilitative care. The nurse anticipates that which of the body. Option 1 is incorrect—there is no evidence to support that the client is experiencing choose the option that promotes the best nutrition for this client. Avoid reading into the
the following methods will most likely be used to malabsorption at this time. Option 3 is incorrect because there is no clinical reason to limit question.
provide nutritional support for the client during this fresh fruit. Even though the client may experience difficulty in chewing and swallowing, this is
time? usually in the acute phase of the disease process. There is nothing to suggest that the client is
experiencing problems in this area or is at risk for aspiration (option 4).

1.‐ Using a gastrostomy tube for feedings due to high incidence of malabsorption
2.‐ Maintaining oral intake with adequate calories to maintain positive nitrogen balance
3.‐ Limiting fresh fruit in the diet
4.‐ Using thickened liquids to prevent aspiration

164 Based on the highest risks during this period of life, Correct answer: 4 A healthy 30‐year‐old has the greatest risks of safety related to lifestyle behaviors: multiple Cancers of the breast, uterus, lung, or prostate are not the mindset of a 30‐year‐old. This
what would be the focus of the nurse who is setting up sexual partners, “on the edge” lifestyle (thrill seekers), haphazard dietary intake, speeding, not problem is the center of thinking for the older adult. Bone density testing for osteoporosis
a health promotion booth for healthy adults in their sleeping enough. is often not recommended nor tested for the female in her thirties. Most women will test
thirties? for this near menopause.
1.‐ Screenings for breast, cervical, uterine, and prostate cancers
2.‐ Chest x‐rays for lung cancer
3.‐ Bone density test for osteoporosis
4.‐ Safety education for accident prevention

165 A client admitted with exacerbation of chronic Correct answer: 1 All of these nursing diagnoses are appropriate for the client with COPD; however, the primary Compare the data in the question and use that as a means of selecting the priority nursing
obstructive pulmonary disease (COPD) has a alteration is related to impaired gas exchange because of the abnormal blood gas results. The diagnosis.
respiratory rate of 18, a dry cough, and arterial blood breathing pattern is satisfactory because the rate is within normal limits, and there is no data
gases that reveal a pH of 7.29, to support activity intolerance, although it is plausible. The client is at risk for infection but
CO<sub>2</sub> of 50 mmHg, and actual problems take priority over potential ones.
O<sub>2</sub> of 72 mmHg. The nurse
identifies which nursing diagnosis as the priority?

1.‐ Impaired gas exchange


2.‐ Activity intolerance
3.‐ Risk for infection related to impaired respiratory defenses
4.‐ Ineffective breathing pattern

166 The nurse is caring for a client who has just been Correct answer: 4 Graves’ disease is caused by elevated levels of thyroid hormone. Clients experience The core issue of the question is that Graves’ disease is characterized by excessive function
diagnosed with Graves’ disease. Client education tachycardia, nervousness, insomnia, increased heat production, and weight loss. Medication of the thyroid gland. From there, you need to determine which medication will reduce the
regarding medication therapy needs to include which therapy with an agent such as propylthiouracil will help control the disorder. Option 1 is function of the thyroid. Eliminate options 1 and 3 as irrelevant first, then choose option 4
of the following? irrelevant, while option 2 is indicated for hypothyroidism. A client with this disorder does not over 2 by its action.
need insulin, because the pancreas is not affected by Graves’ disease (option 3).

1.‐ Atropine
2.‐ Thyroxine
3.‐ Insulin
4.‐ Propylthiouracil (PTU)

167 The nurse has delegated to an unlicensed assistive Correct answer: 2 A client with right hemisphere stroke has left‐sided paralysis or paresis and may have Recall that the manifestations of stroke appear on the opposite side of the body from the
person (UAP) the care of a client who had a right unilateral neglect. The UAP should keep all items on the right side so that the client is aware lesion. Use this principle to eliminate the incorrect responses after eliminating actions that
hemisphere thrombotic stroke with hemiplegia. The they exist in the environment. are carried out correctly.
nurse would give further direction to the UAP after
noting that the UAP did which of the following?
1.‐ Provided passive range of motion exercises to the affected arm and leg
2.‐ Placed a chair for a visitor to the left of the bed
3.‐ Placed the overbed table to the right side of the bed
4.‐ Sat the client up slowly

168 The nurse determines that a client who has an Correct answer: 2 Transmission‐based precautions are required for all these organisms. Only penicillin‐resistant Knowledge of droplet precautions is necessary to answer the question. Penicillin‐resistant
infection with which of the following antibiotic‐ Streptococcus pneumoniae is transmitted via respiratory droplets. The organisms specified in Streptococcus pneumoniae suggests a microorganism that causes a type of pneumonia.
resistant microorganisms requires transmission‐based options 1, 3, and 4 are transmitted by direct contact. Clients with pneumonia have increased respiratory secretions and coughing. Using a
droplet precautions? process of elimination, choose the microorganism that sounds as though it would cause a
respiratory infection—option 2.
1.‐ Methicillin‐resistant <i>Staphylococcus aureus</i>
2.‐ Penicillin‐resistant <i>Streptococcus pneumoniae</i>
3.‐ Vancomycin‐resistant <i>enterococci</i>
4.‐ Vancomycin‐intermediate‐resistant <i>Staphylococcus</i>

169 The nurse is reviewing the results of a male client’s Correct answer: 1 HDL is felt to be a beneficial lipoprotein because of its protective function against coronary The core issue of the question is knowledge of which lipid levels should be raised and
recent lipid profile. The nurse notes that the client is artery disease. An elevation in this level is healthy and indicates compliance with diet and lowered to achieve cardiovascular health. Recall that HDL has the letter H and associate this
experiencing beneficial effects of a heart healthy diet exercise recommendations. LDL and HDL are fractions of the total cholesterol level. with the word healthy to make the positive association between these.
and exercise after noting elevations in which Triglycerides and LDL have proven to be major contributors to and predictors of coronary
laboratory value? artery disease, making elevations in all three remaining options threats to cardiovascular
health in the future.
1.‐ High‐density lipoprotein (HDL)
2.‐ Low‐density lipoprotein (LDL)
3.‐ Total cholesterol
4.‐ Triglycerides

170 The nurse concludes that client education about Correct answer: 3 When the client realizes the connection between stress, anxiety, and dissociation, he The core issue of the question is the ability to recognize triggers to a dissociative state.
dissociation is effective when the dissociative client becomes able to modify his stressors or his response to them, thus preventing the dissociative Recall that stress and anxiety can trigger this state to make the appropriate selection.
states: process. The other responses in options 1, 2, and 4 do not reflect this concept.

1.‐ “When I want to get out of a situation, I choose to space out.”


2.‐ “When I have to cope with problems, I imagine I am somewhere else.”
3.‐ “When I’m under stress, I have a tendency to dissociate.”
4.‐ “When I think about my life, I pretend I am someone else.”

171 A client is advised to take an antiemetic to prevent Correct answer: 3 Anticipatory prevention of nausea with antiemetics is effective if medication is taken 30 to 60 The core issue of the question is knowledge of how soon to take medication prior to
nausea and vomiting. The nurse explains that minutes before any activity causing nausea. The other options indicate incorrect timeframes. activities that cause nausea. Recall that many oral drugs act in 30 to 60 minutes to help you
anticipatory dosing should be done how long prior to make a selection.
activities that generally cause nausea?
1.‐ 24 hours
2.‐ 3 to 5 hours
3.‐ 1/2 to 1 hour
4.‐ 12 hours

172 The nurse working on a neuroscience unit has just Correct answer: 3 The client who had a hemorrhagic stroke and has a headache could be about to have another When trying to decide priorities among clients who are acutely ill, it may help to analyze
received an intershift report. Which of the following bleed. Headache is a classic sign with intracranial bleed, and a second bleed carries a higher the complications each is at risk for or the consequences that could result from the current
assigned clients should the nurse assess first? mortality rate than the first. The other clients have less severe needs that can be attended to condition or complaint. The client with the most serious issue or who could experience the
after the client who is at risk for a fatal complication is seen. most severe complication is the one that takes priority.
1.‐ A client with Parkinson’s disease who is crying because she cannot get up easily
2.‐ A client with multiple sclerosis who is having noticeable leg spasms
3.‐ A client who had a hemorrhagic stroke and is complaining of headache
4.‐ A client scheduled for a craniotomy in 4 hours
173 Which of the following statements made by a client Correct answer: 1 Contaminated foods are not a source of HIV/AIDS infection. While contaminated foods may The critical word in the stem of the question is clarify. This tells you that the correct
regarding human immunodeficiency virus (HIV) and cause GI symptoms and food poisoning due to various etiologic agents, they do not cause the answer is an incorrect statement on the part of the client. Use nursing knowledge and the
acquired immunodeficiency syndrome (AIDS) would transmission of this disease. The nurse should clarify this statement by the client in order to process of elimination to make a selection.
the nurse seek to further clarify? provide accurate information. All of the other client statements reflect information that is
appropriate for the management of client with HIV/AIDS.

1.‐ “I can get the disease from eating contaminated food products.”
2.‐ “Blood tests will tell me if I have a nutritional anemia.”
3.‐ “Maintaining adequate fluid and fiber intake will help me.”
4.‐ “If I feel sick to my stomach, I should not drink liquids.”

174 The registered nurse is assigned to the postpartum Correct answer: 1 The RN is responsible for delegating tasks appropriately and is responsible for the actions of Use principles of delegation and select the care activity that is the simplest and requires
unit. Which task could the RN safely delegate to an unlicensed employees. Ambulating a postoperative client is the only task that the RN could the least amount of high‐level judgment, especially since the level of the student is not
unlicensed assistive person (UAP)? delegate from those listed. The other tasks require higher level assessment and critical thinking identified.
skills and should be performed by the RN.
1.‐ Ambulate a client who had a vaginal delivery yesterday.
2.‐ Complete the admission assessment on a newly delivered client.
3.‐ Call the physician to report a low hemoglobin level.
4.‐ Verify a unit of blood prior to transfusion.

175 The client has undergone hypophysectomy using a Correct answer: 2 The presence of a halo effect indicates cerebrospinal fluid (CSF). Glucose present in the nasal To answer this question correctly, analyze the significance of the findings. Eliminate each
transphenoidal approach. You change the mustache drainage also suggests that the drainage is CSF. A persistent headache indicates a CSF leak. The of the incorrect responses systematically after noting that a risk after this type of surgery is
dressing, noting clear exudate with a pale yellow physician needs to be informed of these assessment findings and the client must be CSF leak.
colored ring at the edge of the drainage on the maintained on bedrest to stop the leak. A spinal tap may be done to decrease CSF pressure.
dressing. You should do which of the following next? Option 1 is incorrect because it does nothing for the client. Options 3 and 4 do not address the
real problem, a probable CSF leak.
1.‐ Document this as serous drainage and continue to monitor the client.
2.‐ Assess for headache and check the glucose level in the drainage.
3.‐ Apply an ice pack to the nasal bridge and a large, fluffy dressing.
4.‐ Lower the head of the bed to decrease the gravity pressure on the wound.

176 The client is experiencing severe itching with a skin Correct answer: 3 Hydroxyzine hydrochloride is an antihistamine that is a competitive inhibitor of the The core issue of the question is knowledge of which drug relieves itching. Use specific
disorder. Which of the following drugs, if ordered, H&lt;sub&gt;1&lt;/sub&gt; receptor. It is used to treat various reactions that are mediated by drug knowledge and the process of elimination to make a selection.
would the nurse administer as an appropriate oral histamine. It will decrease the pruritus produced by the release of histamine. Cimetidine is an
preparation to decrease the itching? H&lt;sub&gt;2&lt;/sub&gt; histamine antagonist and these agents are not effective against
hypersensitivity reactions. Lorazepam is a short‐acting benzodiazepine that is indicated for
anxiety. Bupivacaine is a local anesthetic for nerve blocks.

1.‐ Cimetidine (Tagamet)


2.‐ Lorazepam (Ativan)
3.‐ Hydroxyzine (Atarax)
4.‐ Bupivacaine (Sensorcaine)

177 A client who is legally blind has been admitted to the Correct answer: 1, 4 The UAP can perform procedures and nursing care activities. Client care that requires Recall that UAPs are trained and educated to perform simple care procedures. Use this
nursing unit. Which of the following activities should assessment (options 2, 3, and 5) are not within the scope of the functions of the UAP. framework to eliminate each of the incorrect options systematically.
the nurse delegate to the unlicensed assistive person
(UAP)? Select all that apply.

1.‐ Assist the client with meals.


2.‐ Complete the admission interview form.
3.‐ Assess the impact of the lost vision on the client’s daily life.
4.‐ Assist the client to ambulate in the hall.
5.‐ Ask the client what community services are being utilized.

178 A client who has a history of Graves’ disease Correct answer: 4 With exophthalmos, the eyelids may not cover and protect the cornea of the eye. Thus, eye The core issue of the question is which need of the client with Graves’ disease can be met
accompanied by exophthalmos is arriving from protection from the sheets or preventing the hands from accidentally touching the eyes is by the UAP. The need of the client with respect to eye safety can be met using ordinary
surgery. Based on the observations as you note the needed while the client is in bed. With Graves’ disease, clients usually experience heat nursing procedures, so this is the task that may be delegated to the UAP with education.
photo, what should you educate the unlicensed intolerance, thus less covering and a cool room are preferred (option 1). Hyperglycemia is not
assistive person (UAP) to do? usually associated with Graves’ disease. The head of the bed should be elevated 30 degrees to
minimize eye pressure (option 3).

1.‐ Keep the client’s room warm to promote comfort.


2.‐ Obtain fingerstick blood glucose every 2 hours twice.
3.‐ Keep the head of the bed flat for 4 hours.
4.‐ Provide eye protection measures for the client.

179 A client with bone cancer receiving chemotherapy has Correct answer: 3 Most chemotherapeutic agents cause some degree of bone marrow suppression. This results The critical word in the question is priority. With this in mind, you need to determine
developed bone marrow suppression. Which in a decrease in leukocyte and erythrocyte counts, both components of a hematology testing. which lab value has greatest importance in terms of monitoring. The core issue of the
laboratory report is of highest priority for the nurse to Calcium, phosphorus, and serum PSA levels are not specifically affected by bone marrow question is bone marrow suppression, which could affect production of red blood cells,
monitor at this time? suppression. The calcium level could change because of the underlying bone cancer, and this in white blood cells, and platelets. Choose the option that best correlates with this risk.
turn could affect phosphorus, but this is not the focus of the question.

1.‐ Calcium
2.‐ Phosphorus
3.‐ White blood cell (WBC) count
4.‐ Serum prostate‐specific antigen (PSA)

180 Which of the following behaviors would the nurse Correct answer: 2 All of the options are dissociative responses. However, only localized amnesia is the inability Focus on the critical words expected and localized amnesia. These words indicate that the
conclude is expected in a client who suffers from to recall events in a circumscribed time period. correct option is one that is consistent with what is assessed in this state. Focus on the
localized amnesia? word localized in the question and the time‐bound nature of option 2 to choose correctly.

1.‐ Wandering about in his neighborhood using a new name.


2.‐ Forgetting about what happened during an assault.
3.‐ Awareness of only a few of many alters.
4.‐ Feelings of separation from his body.

181 The client will be discharged to home tomorrow on Correct answer: 3 Option 3 is correct because it acknowledges the client’s feelings and addresses his concerns The best answer to communication questions is to choose the response that addresses the
an antidepressant medication that will be taken once while still allowing him to make decisions for his present and future. Options 1 and 2 disregard client’s issue or concern. Use the process of elimination and this principle of
daily in the morning. He asks, “Do I have to take and negate the client’s feelings. Option 4 acknowledges his concern but takes away his communication to make a selection.
medicine every day? How will I be able to sleep when I decision‐making options by having someone else (the nurse) make a plan for his daily activities,
go home? Do you think I’ll be able to work, too, even rather than have him participate and make decisions for himself with help.
though I have been in the hospital this long?” The
nurse’s best response is:

1.‐ “The best approach is to take it one step at a time, so that everything will work out.”
2.‐ “I understand you’re worried, but you and your wife will decide tomorrow when you get home.”
3.‐ “You seem to be worried about when you get home and how you will function. Would you like to sit and discuss a plan for your daily activities?”
4.‐ “I’ll do my best to set up a plan for discharge that you can take home with you and refer to later.”
182 The client is admitted with all of the following orders Correct answer: 1 Fluid and electrolyte replacement is the highest priority. Hyperglycemia is treated with To answer this question correctly, it is necessary to understand the underlying
to treat diabetic ketoacidosis (DKA) with severe regular insulin rather than NPH insulin (option 3). Concurrent administration of IV regular pathophysiology. Determining that dehydration is a key issue will help you to focus on
metabolic acidosis. Which order would the nurse insulin would also be done as a priority. The items in the other options can be done after rehydration. Attend to regular care measures and monitoring after acute manifestations
determine to be the first priority in managing this definitive treatment for dehydration is done. have been addressed.
client?
1.‐ Start IV fluid infusion for rehydration.
2.‐ Insert an indwelling urinary catheter.
3.‐ Administer NPH insulin.
4.‐ Initiate continuous pulse oximetry.

183 The nurse would include which of the following Correct answer: 3 Malnutrition is seen as a consequence of the HIV/AIDS virus because the disease process has The wording of the question tells you that the correct answer is a true statement of fact.
statements when discussing nutritional status with a a progressive effect on client’s nutritional status. Option 1 is incorrect—even clients who are Use nursing knowledge and the process of elimination to make a selection.
client who is infected with human immunodeficiency asymptomatic may already have nutrient deficiencies and could be experiencing subclinical
virus (HIV) and is progressing toward acquired signs of malnutrition. Option 2 is incorrect because wasting syndrome occurs early in the
immunodeficiency syndrome (AIDS)? disease process; current clinical research states that the maintenance and preservation of
nutritional status is a priority in the clinical management of this condition. Option 4 is
incorrect—clients can experience vitamin and mineral deficiencies early on during the disease
process.
1.‐ Clients who are asymptomatic have adequate nutritional stores of nutrients.
2.‐ The HIV wasting syndrome is seen in the latter stages of the disease process.
3.‐ Malnutrition is seen as a consequence of the immune disease.
4.‐ Vitamin and mineral deficiencies occur in the latter stages of the disease process.

184 The nurse is taking the health history of a 77‐year‐old Correct answer: 3 Bladder and sphincter weakness are normal with the aging process. Decreased tolerance to Understanding the expected changes at the various age brackets will allow you to
man. Which of the following symptoms reported by spicy foods also is reflected by decreased acidity and motility of the digestive processes that anticipate what is within the normal range of changes and what is not.
the client would the nurse consider to be an abnormal are common in the aging process. Circulatory instability can occur when getting up too quickly
finding? since the vasoconstriction process of the legs can be slower as one ages. Also, dehydration can
lead to slight dizziness when moving about. Increasing the process of isolation from others is
not a healthy adaptation although it is common when one spouse dies that the other seems
totally lost since most events include whole couples rather than newly singled again
individuals.
1.‐ Delay of urination, hesitation, and decreased flow of urine stream
2.‐ Increased tolerance to spicy foods
3.‐ Increased isolating behaviors after his wife’s death
4.‐ Slight dizziness when getting up too quickly after lying down for a while

185 In planning care for a client with an axis II personality Correct answer: 1 Clients who are diagnosed with a personality disorder most frequently perceive their The core issue of the question is the ability to discriminate among various types of mental
disorder, the nurse anticipates that the client will differ personality patterns as ego‐syntonic or a natural part of themselves rather than as ego‐ health disorders using DSM‐IV criteria. Use this knowledge and the process of elimination
from clients with axis I disorders in that the client will: dystonic (option 3). This is one reason it is difficult to motivate individuals with personality to make a selection.
disorders to try to change their maladaptive behavioral patterns. Individuals with personality
disorders display problems living rather than clinical symptoms. Personality disorders are
associated with concomitant disorders including substance abuse.

1.‐ Tend to experience symptoms as ego‐syntonic.


2.‐ Usually display clinical symptoms.
3.‐ Tend to experience symptoms as ego‐dystonic.
4.‐ Seldom experience addictive behaviors.
186 The home health care nurse is visiting an elderly Correct answer: 2 Calcium channel blockers should be administered with a high‐fat meal; grapefruit should be The core issue of the question is knowledge that grapefruit juice affects the availability of
client who is taking a prescribed calcium channel avoided before and after dosing due to its ability to alter drug effects. The foods listed in the some drugs, such as calcium channel blockers, because of their action on enzyme systems.
blocker. In conducting dietary teaching, the nurse other options will not have a dose‐altering effect. Use this knowledge and the process of elimination to make a selection.
instructs the client that what food is contraindicated to
take with a calcium channel blocker?

1.‐ Oranges
2.‐ Grapefruit
3.‐ Bananas
4.‐ Grapes

187 The client is admitted with thyroid storm. Assessment Correct answer: 4 Tachycardia, hypertension, and tachypnea increase stroke volume and tissue demand for Recall that physiological needs take priority over psychosocial needs. Also remember that
reveals: BP 188/102, HR 132 regular, RR 28 full depth oxygen, leading to increased cardiac workload and possible heart failure. If fluid volume deficit the ABCs (airway, breathing, and circulation) are of highest priority in many cases.
and symmetrical, no urine output since admission to is present, there is an additional risk for decreased cardiac output. There is insufficient data to
the Emergency Department 3 hours ago, alert, and determine fluid volume status. The tachypnea is a symptom of the increased metabolic rate.
anxious. Which of the following would be the high
priority nursing diagnosis for this client?

1.‐ Deficient fluid volume related to decreased absorption as evidenced by no urine output since admission
2.‐ Anxiety related to fear as evidenced by client’s appearance
3.‐ Ineffective breathing pattern related to increased metabolism as evidenced by RR 28
4.‐ Decreased cardiac output related to increased ventricular workload as evidenced by adverse vital signs

188 The nurse must assess the temperature and blood Correct answer: 1 Equipment for client care is dedicated to the client on contact precautions and kept in the The key word appropriate suggests there is only one correct answer. Look for the nursing
pressure of a client on contact precautions for wound client’s room. Any other action does not uphold principles of infection control. action that would limit the spread of pathogenic microorganisms.
infection every shift. Which is the appropriate nursing
action to minimize the spread of microorganisms?

1.‐ Keep the equipment in the client’s room.


2.‐ Store the equipment in the soiled utility room between uses.
3.‐ Cleanse the equipment after each use.
4.‐ No special action is required with the equipment.

189 A client presents to the clinic with a chief complaint Correct answer: 3 Clients with gout will usually have elevated serum uric acid levels. Laboratory findings as well The core issue of the question is knowledge of diagnostic testing for gout. Recall that the
of a swollen and painful great toe. He states that his as physical assessment will confirm the diagnosis. The joint of the great toe is usually involved word gout contains the letter u to associate this with measurement of uric acid, which
brother has it, and he has the same symptoms. The in initial attacks of acute gouty arthritis as seen in the accompanying figure. There are many begins with u.
physician suspects gout. What specific laboratory test other factors that will affect the results of hematocrit, serum calcium, and sodium levels.
would the nurse expect to be ordered for this client? Erythrocyte sedimentation rate (ESR or sed rate) and white blood cell (WBC) counts will also be
elevated in cases of gout.
1.‐ Calcium
2.‐ Hematocrit
3.‐ Uric acid
4.‐ Sodium

190 A female client has been diagnosed with a dependent Correct answer: 4 It is difficult for individuals diagnosed with dependent personality disorder to make decisions The critical word in the stem of the question is dependent. Focus on this word and look for
personality disorder. Which statement is likely to be on their own (options 1 and 3); rather, they try to get others to make decisions for them. This an association between that word and the nature of the statement in each option. The
her response to the nurse’s suggestion that she characteristic is reflected in DSM‐IV diagnostic criteria. They would be disinclined to make option that most closely simulates a response that relies on another is the correct answer
complete her morning care? critical remarks (option 2) related to their need for support from others. to the question.

1.‐ “I’ll have no problem in deciding what to wear.”


2.‐ “I think you should wear more makeup.”
3.‐ “I think this outfit looks good on me.”
4.‐ “What do you think I should wear?”
191 The nurse should question an order for which beta Correct answer: 3 Terbutaline, pirbuterol, and metaproterenol are all beta 2 stimulants. Isoproterenol The core issue of the question is knowledge that isoproterenol is contraindicated because
agonist used to treat respiratory disease in a client stimulates beta 1 and beta 2 receptors and therefore is contraindicated and should not be it is a cardiac stimulant. Use specific drug knowledge and the process of elimination to
with a history of atrial fibrillation accompanied by used with clients with tachydysrhythmias. make a selection.
intermittent heart rates of 100/minute or greater?

1.‐ Terbutaline (Brethine)


2.‐ Pirbuterol (Maxair)
3.‐ Isoproterenol (Isuprel)
4.‐ Metaproterenol (Alupent)

192 A client with osteoporosis who has experienced Correct answer: 1, 3 The nurse can delegate procedures to the UAP and retains responsibility for the outcomes of Use the principles of delegation to answer the question. Eliminate those options that
fractures in the past is now admitted for dizziness and those tasks that are delegated. Clearing the room of unnecessary objects and remaining with represent assessment, teaching, or interdisciplinary communication or collaboration.
shortness of breath and has been determined to be at the client during ambulation are among those that can be delegated. The nurse needs to retain
risk for falls. Which nursing intervention to assist this responsibility for assessment (option 2), teaching (option 5), and collaborating with the
client can the nurse delegate to the unlicensed interdisciplinary team (option 4).
assistive person (UAP)? Select all that apply.

1.‐ Clear the room of unnecessary objects.


2.‐ Inquire if the dizziness has led to any recent falls.
3.‐ Remain with the client during ambulation.
4.‐ Ask physical therapy to evaluate the client for a walker.
5.‐ Advise the client about the benefits of calcium in the diet.

193 A client is receiving radiation to the head and neck Correct answer: 4 Dry mouth can be a common complaint of clients undergoing radiation therapy. Using sugar‐ The core issue of the question is determining a strategy to relieve dry mouth for a client
area for treatment of cancer. What interventions free candies or gum will help to stimulate the flow of saliva and ease the discomfort that the with cancer that will not contribute to anorexia. Use general principles of nutrition and
would you use to help the client’s complaint of a dry client is experiencing without contributing to dental caries or lack of appetite from sugar knowledge of the disease process to make a selection.
mouth? intake. Option 1 is incorrect—eating meals prior to radiation therapy may lead to increased
nausea because the client would be lying down after eating the meal. It has no effect on
complaints of a dry mouth. Option 2 is incorrect—eating larger portions of food will not help
to ease complaints of a dry mouth. Furthermore, the client may not be able to increase the
size of meals due to side effects experienced as a result of radiation therapy. Option 3 is
incorrect—the use of mouthwash can further cause the mouth to be dry and intensify the
client’s symptoms.

1.‐ Have client eat prior to radiation therapy.


2.‐ Encourage the client to eat larger portions of food.
3.‐ Advise the client to use mouthwash.
4.‐ Suggest the use of sugar‐free candies.

194 The client experienced an 18‐hour labor with a Correct answer: 3 Although this client is not demonstrating positive signs of bonding at this time, it is important Compare the nursing diagnoses with the information in the stem of the question.
second stage that lasted 2 hours. When the nurse to look at her history before concluding that she is not bonding well with her infant. This client Eliminate each incorrect option based on lack of supporting data in the question.
brings the infant into the room 1 hour after delivery, just experienced a long labor and the influence of fatigue on the attachment process should be
the client tells the nurse to leave the infant in the crib considered. It is important to continue to assess infant bonding with this client throughout her
and shows no interest in holding the newborn. The hospitalization to reach a nursing judgment based on evidence.
nurse should record which of the following nursing
diagnoses in the chart?

1.‐ Ineffective individual coping related to assuming parental role


2.‐ Powerlessness related to loss of individual choices
3.‐ Fatigue related to prolonged labor
4.‐ Anxiety related to feelings of incompetence in parenting role
195 The nurse would expect to find a diminished Correct answer: 1 Carbon dioxide is eliminated from the body as exhaled gas. The faster the rate of breathing, Note the stem of the question contains the words diminished
pCO&lt;sub&gt;2&lt;/sub&gt; level in the assigned the greater the quantity of carbon dioxide eliminated. pCO&lt;sub&gt;2&lt;/sub&gt;, which indicates that the client is blowing off excessive
client who has which of the following physical CO&lt;sub&gt;2&lt;/sub&gt;. Use knowledge of respiratory disorders to select the option
assessment findings? that is consistent with excessive respiration, which is option 1.
1.‐ Hyperventilation
2.‐ Hypoventilation
3.‐ Prolonged expiration
4.‐ Stridor

196 A client is scheduled for an ophthalmic examination. Correct answer: 3 Ophthalmic epinephrine is used to produce mydriasis for ocular examination. Dilation of pupil The core issue of the question is knowledge that angle‐closure glaucoma is a
Before administering the prescribed epinephrine further constricts ocular fluid outflow, possibly causing an acute attack of glaucoma in a client contraindication to use of epinephrine for mydriasis during an ocular examination. Use
solution, the nurse would assess for which of the with narrow‐angle glaucoma. Systemic absorption also causes hypertension and tachycardia. specific drug knowledge and the process of elimination to make a selection.
following conditions? Brow ache is a typical side effect of adrenergic agonists such as epinephrine (option 4).

1.‐ Hypotension
2.‐ Wide‐angle glaucoma
3.‐ Angle‐closure glaucoma
4.‐ Brow ache

197 The nurse is working on an orthopedic unit. After Correct answer: 4 The client that is the most stable and with the fewest needs that the nurse must attend to is Recall the principles of delegation and that clients who need assessment or teaching need
receiving intershift report, which client should the the client who is 6 days postoperative and awaiting placement in a rehabilitation facility. The to remain under the direct responsibility of the nurse.
nurse assign to the unlicensed assistive person (UAP)? nurse could attend to this client’s discharge paperwork later in the shift. The nurse needs to
assess the pain and neurovascular status of the client in option 1, since the client could be
experiencing a complication of an overly tight cast. The nurse also needs to assess the client
with the new spinal fracture. The nurse would need to teach and counsel the client who has
phantom limb sensation.
1.‐ A client with a newly applied cast who has increasing pain despite medication
2.‐ A client with osteoporosis admitted 2 hours ago who fell and fractured the vertebra at L1
3.‐ A client with a below‐knee amputation who is anxious because “the leg feels like it’s still there”
4.‐ A client who had surgical repair of a fractured left hip 6 days ago and will be discharged to a rehabilitation facility near the end of the day

198 An adult client arrives to the Emergency Department Correct answer: 1, 3, 5 Knowledge of the cardiovascular disease risk factors and associated symptoms can assist in The core issue of the question is knowledge of risk factors of cardiac disease leading to
with complaints of chest pain and shortness of breath. determining the origin of chest pain and direct the nurse to prioritize and implement chest pain. Eliminate option 2 as unrelated because of the critical word recent, recalling
The nurse concludes that which of the following appropriate care. Diabetes, smoking, and hypertension are known modifiable and non‐ that chest pain from cardiac origin is not related to travel. Eliminate option 4 next because
points, if present in the client’s history, would indicate modifiable risk factors to cardiac disease. Chest pain that occurs during activity may indicate cardiac pain does not correlate with the respiratory cycle.
that this pain may be related to cardiac disease? Select cardiac ischemia due to the increased oxygen demand. Associated symptoms of nausea and
all that apply. diaphoresis are known warning signs of cardiac ischemia. Chest pain that increases with
breathing, especially taking a deep breath, is most likely pleuritic pain and travel out of the
country is an unrelated factor.
1.‐ History of diabetes and smoking
2.‐ Recent travel out of the country
3.‐ The pain increases with activity
4.‐ The pain is reproducible when taking a deep breath
5.‐ The client is experiencing sweating and nausea when the pain is severe

199 A client asks the nurse to repeat what the physician Correct answer: 1 The definition of moderate sedation is that there is a minimal depression of the level of The core issue of the question is knowledge of moderate sedation and communication
explained about the anesthesia planned for an consciousness in which the client is able to maintain a patent airway and respond techniques that explain this clearly and accurately. Use knowledge of key features of
upcoming procedure. The nurse understands the appropriately to verbal and physical stimuli. The pain threshold is increased so that the client moderate sedation and the process of elimination to make a selection.
procedure will be performed under moderate can tolerate pain (option 2). Amnesia is induced partially with conscious sedation (option 3).
sedation. Which statement should the nurse make to Option 4 is false because the client is awake.
the client concerning moderate sedation?
1.‐ “You will be able to breathe and respond appropriately to physical stimuli and words that are spoken.”
2.‐ “Your pain threshold will be decreased so you can tolerate the pain.”
3.‐ “You will have a patent airway and will be able to remember and comprehend what is happening.”
4.‐ “You will not be awake but you will still feel slight pain during the surgical intervention.”

200 A client is diagnosed with paranoid personality Correct answer: 3 These characteristics are reflected in DSM‐IV diagnostic criteria for paranoid personality The core issue of the question is knowledge of characteristics of paranoid personality
disorder. Which of the following assessment data does disorder. They must be considered in planning and implementing care. Delusions and disorder. Use nursing knowledge and the process of elimination to make a selection. Note
the nurse conclude are consistent with this diagnosis? hallucinations are consistent with schizophrenia or other psychotic disorders. Options 2 and 4 the word paranoid in the stem and secretiveness in the correct option to help make an
describe behavior traits but they are not consistent with paranoid personality disorder. association between the two.

1.‐ Delusions and hallucinations


2.‐ Passivity and compliance with rules
3.‐ Jealousy and secretiveness
4.‐ Respect for authority

201 The nurse is administering nitrogen mustard Correct answer: 2 The question indicates that extravasation may be occurring. Prompt nursing action in general The core issue of the question is knowledge that nitrogen mustard is an antineoplastic
(Mustargen) and notes swelling at the intravenous (IV) will minimize tissue damage; therefore nursing actions should be initially directed towards the agent and that these drugs may be vesicants. From there, you need to determine what
site. The nurse should take which of the following suspicious site. The drug administration should be stopped, since failure to do so will further action will reduce the risk of further damage, which is stopping the drug and trying to
actions initially? disperse drug into the tissue. Clients can experience extravasation without pain, but not aspirate it out of tissue.
without swelling. Flushing the line with saline or dextrose is not advised, since there may still
be vesicant drug remaining in the tubing.

1.‐ Continue with infusion after trying to aspirate for a blood return.
2.‐ Stop administration and attempt to aspirate.
3.‐ Flush the line with saline.
4.‐ Obtain a new site for drug administration.

202 The nurse working on adult medical‐surgical unit Correct answer: 3 The nurse should delegate the care of the 81‐year‐old client with heart failure and Recall the principles of delegation. The nurse should not delegate the care of clients who
would assign which of the following clients to the emphysema. This client was admitted 3 days ago and has a stable medical status. The nurse require assessment due to changes in acuity or status and clients who require teaching.
licensed practical/vocational nurse (LPN/LVN) under would want to assess the client recently admitted with exacerbation of COPD and the 25‐year‐ Stable clients may be delegated to the LPN/LVN under the RN’s supervision.
the supervision of the RN? old with a concussion less than 24 hours ago. The newly diagnosed diabetic client would
require teaching that should not be delegated to the LPN/LVN.
1.‐ A 62‐year‐old client admitted 8 hours ago with exacerbation of COPD
2.‐ A 25‐year‐old client with a concussion from an auto accident the prior evening
3.‐ An 81‐year‐old client with chronic heart failure and emphysema admitted 3 days ago
4.‐ A 54‐year old client newly diagnosed with diabetes mellitus who will be discharged today

203 A client has been referred for dietary teaching Correct answer: 4 Nutritional goals for a client with hepatitis are aimed at providing a diet that is high in calories The critical word in the stem of the question is hepatitis. From this point, analyze that the
regarding the management of hepatitis. The nurse (3,000–4,000 kcal) and high in quality protein (1.5–2.0 g/kg). The diet should also be adequate client recovering from hepatitis needs a high‐calorie, high‐protein diet for healing to make
would base development of nutritional goals on which in carbohydrates to spare protein and fat, provide concentrated calories, and improve the the correct selection.
of the following pieces of information? taste of food. Option 1 is incorrect—the nutritional management of hepatitis is the same for all
types. Option 2 is incorrect—there is no clinical indication to place the client on tube feedings
given the information that is provided. If the gut works, then the usual clinical model is to use
it. Option 3 is incorrect because dietary fat should not be limited unless the client is
experiencing problems with malabsorption (steatorrhea) and there is no evidence to support
this.

1.‐ The type of hepatitis that the client has, as this will affect the treatment
2.‐ The need for tube feedings to allow the liver to rest and regenerate
3.‐ That dietary fats should be limited
4.‐ That the diet should be high in calories and high in protein
204 The nurse is caring for a 15‐year‐old primipara who Correct answer: 3 Although all of the options may be appropriate, demonstrating newborn care will allow the Recall principles of teaching and learning, and recall that active participation leads to most
delivered yesterday. The nurse identifies the following client to ask questions and gain confidence as she cares for her baby. Having her return the effective learning outcomes.
nursing diagnosis for this client: risk for altered demonstration will allow the nurse to evaluate the teaching.
parenting related to knowledge deficit in newborn
care. Which is the most appropriate intervention when
planning this client’s discharge teaching?

1.‐ Have the client watch a video on newborn care.


2.‐ Give her information about a support group for adolescent mothers.
3.‐ Demonstrate how to care for the newborn and have the client return the demonstration.
4.‐ Give the client printed instructions on newborn care.

205 To decrease skin irritation in children with the Correct answer: 4 The illustration shows the typical appearance of skin that has eczema. Use of a mild soap such To answer this question correctly, it is necessary to be familiar with the skin disorder in
condition illustrated, the nurse instructs the parents as Dove&lt;sup&gt;®&lt;/sup&gt; or Tone&lt;sup&gt;®&lt;/sup&gt; prevents the skin from the picture. Beyond that, eliminate the incorrect options because of the words hot and
do which of the following? excessive dryness. Hot water is drying to the skin so should be avoided. Fabric softeners and daily in option 1, liberally and entire in option 2, and all in option 3. Although option 4
many lotions contain perfumes that are irritating to the skin so should also be avoided. contains the word only, note that it is tempered when combined into the phrase only as
needed.
1.‐ Take hot baths (not showers) daily
2.‐ Liberally apply a lotion of choice over entire body
3.‐ Use fabric softener for all clothes
4.‐ Use mild soap only as needed

206 A client who is taking warfarin (Coumadin) therapy Correct answer: 3 Clients who are taking Coumadin should be alerted to the potential for drug interactions The core issue of the question is knowledge that drugs containing aspirin can have an
comes to the office for a follow‐up visit and states that when they are on long‐term anticoagulation therapy. Aspirin can potentiate the effect of interactive effect with warfarin, which increases the risk of bleeding. With this in mind, use
he has taken propoxyphene with aspirin (Darvon Coumadin and interfere with the ability to maintain a therapeutic level. The use of Darvon, the process of elimination to select the option that results in stopping pain therapy with an
Compound 65) for aches and pains related to an old although previously prescribed, is not in the best interest of the client at this time due to aspirin‐containing drug.
back injury. How should the ambulatory care nurse Coumadin therapy. Telling the client to keep taking Darvon would lead to drug interactions
respond to this information? (option 2). While a further assessment of the client’s back pain may be necessary (option 1), it
is not the primary action that the nurse should be addressing at this time. Option 4 is a false
statement, because the two drugs together could enhance bleeding.

1.‐ Ask the client how long his back been hurting him and assess the need for a referral for pain management.
2.‐ Tell the client that it is important to prevent the pain cycle from starting and to continue to take Darvon as ordered by the physician.
3.‐ Advise the client that Darvon contains aspirin, which may interfere with Coumadin therapy, and consult with the physician for an alternate pain medication.
4.‐ Instruct the client that continued daily use of Darvon would help to relieve back pain, but would require an increase in the dose of the Coumadin.

207 Which of the following is an assessment finding with Correct answer: 3 All symptoms listed are clinical manifestations of developmental dysplasia of the hip, Specific knowledge about this disorder is needed to answer the question. Take time to
developmental dysplasia of the hip in a 5‐year‐old although the only one that would be found in a 5‐year‐old would be the telescoping of the review this disorder if you had difficulty with this question.
child? femoral head into the pelvis. Other clinical signs in an older child would be lordosis and a
waddling gait with a marked limp. A positive Ortolani‐Barlow maneuver is found in the infant
younger than 2 to 3 months of age. Limited abduction is the sign most often used for an infant
older than 3 months, along with asymmetry of thigh and gluteal folds.

1.‐ Asymmetry of gluteal and thigh fat folds


2.‐ Positive Ortolani‐Barlow maneuver
3.‐ Telescoping of the femoral head into the pelvis
4.‐ Limited abduction of the affected hip
208 The nurse would intervene after noting another Correct answer: 3 The postoperative care of the child undergoing repair of clubfoot would not include The core issue of the question is knowledge that a client who underwent repair of clubfoot
nursing staff member take which of the following administering pain medication immediately when due and covering the cast with blankets. will have a cast in place. After determining this, evaluate each of the options in terms of
actions in the care of a child who has had surgery for Medication for pain should be administered as needed, and the cast should not be covered their appropriateness as part of management of the client in a cast.
clubfoot? with blankets because this will interfere with the cast drying and could enhance swelling if
excessive heat is retained under the blanket. Use of ice bags, elevation, diversional activities,
and assessment of neurovascular status, swelling, and drainage or bleeding are all appropriate
interventions.
1.‐ Applying ice bags to the foot and keeping the ankle and foot elevated on a pillow
2.‐ Checking for drainage or bleeding and observing for swelling around cast edges
3.‐ Administering pain medication immediately when it is due and covering the cast with blankets
4.‐ Performing neurovascular status checks every 2 hours and providing diversional activities

209 An adolescent is undergoing a spinal fusion for Correct answer: 3 All of the information above is needed by the adolescent undergoing a spinal fusion but the The core issue of the question is knowledge of which information is within the domain of
scoliosis. Which of the following would not need to be physician, not the nurse, should explain the actual procedure. The nurse should focus on the nursing practice and which information needs to be given by the physician to obtain
included in the preoperative teaching completed by care of this child following surgery, the exercises for breathing, turning, moving extremities, informed consent. Choose the option that is not included by selecting the option that is
the nurse? the tubes that will be placed—the nasogastric tube, urinary catheter, and intravenous lines. within the surgeon’s scope of practice.
Ways that pain will be dealt with should also be explained in the preoperative period.

1.‐ Deep‐breathing and coughing exercises, use of incentive spirometry


2.‐ Use of postoperative pain medications
3.‐ The procedure for the spinal fusion and bone grafting
4.‐ Placement of a urinary catheter to drain urine after surgery

210 The nurse interprets that which of the following Correct answer: 1 Individuals diagnosed with paranoid personality disorder frequently are critical or The core issue of the question is knowledge of the behavioral characteristics of a client
statements made by a coworker is a typical staff argumentative to maintain a safe distance between themselves and others related to their with paranoid personality disorder. Reflect on the common meaning of the word paranoid
response when working with a client diagnosed with a inability to trust others. Nursing staff may need to remind themselves that criticism of nursing and evaluate each option for consistency to make an appropriate selection.
paranoid personality disorder? care may be a manifestation of a personality disorder. The other statements listed do not
reflect behavior that is typical of a client with this disorder.
1.‐ “He constantly criticizes his care. I’m so frustrated.”
2.‐ “He is so pleasant but so shy.”
3.‐ “He has a wonderful sense of humor but he doesn’t let it show often.”
4.‐ “I am pleased he was so helpful with his roommate. He can be so irritable at times.”

211 A client has been admitted to the hospital with chest Correct answer: 3 The standard protocol is to administer up to three doses of NTG 5 minutes apart as long as The core issue of the question is knowledge that nitroglycerine can be repeated up to 3
pain. The pain has not been relieved after one dose of the vital signs remain stable. After three doses, the physician should be called if pain is doses as long as the pain continues and the blood pressure is stable. Use the process of
nitroglycerine (NTG) sublingually. Upon monitoring the unrelieved. An electrocardiogram (ECG) may be ordered, but not an EEG (to measure brain elimination and safe drug action to answer the question.
vital signs (VS), the nurse notices that the blood waves). Using NTG paste, a longer acting form of the medication, is not appropriate at this
pressure has dropped to 126/84 from 130/90. Which time.
of the following actions should the nurse take next?

1.‐ Notify the physician.


2.‐ Obtain an electroencephalogram (EEG).
3.‐ Give another dose of nitroglycerine.
4.‐ Add a dose of nitroglycerine paste.

212 The nurse working on an adult medical‐surgical unit Correct answer: 2 The nurse should delegate the care of the 78‐year‐old client with diabetes and osteoarthritis. Recall the principles of delegation. The nurse should not delegate the care of clients who
would assign which of the following clients to the This client has a stable medical status. The nurse would want to assess the client recently require assessment due to changes in acuity or status, and clients who require teaching.
licensed practical/vocational nurse (LPN/LVN) under admitted following nephrectomy and the 32‐year‐old who fractured the pelvis. Since pelvic Stable clients may be delegated to the LPN/LVN under the RN’s supervision.
the supervision of the RN? exenteration is done to treat cancer, the nurse would want to assess this client and also
address this client’s psychosocial needs in coping with the diagnosis and surgery.

1.‐ A 45‐year‐old client admitted yesterday after a nephrectomy


2.‐ A 78‐year‐old client with diabetes mellitus and osteoarthritis
3.‐ A 32‐year‐old client with a fractured pelvis from an auto accident 3 days ago
4.‐ A 62‐year old client who underwent pelvic exenteration receiving medication via patient‐controlled analgesia

213 The nurse anticipates which of the following Correct answer: 3 The development of ascites (third spacing) is a common complication of cirrhosis. With the The core issue of the question is the ability to correlate collection of ascetic fluid in the
regarding sodium restriction for a client diagnosed development of ascites, sodium restriction is instituted. Depending on the extent and response abdomen with an aggravating factor, sodium. Use this information and the process of
with ascites secondary to cirrhosis? to clinical treatment, the restrictions may be 500 to 1,000 mg per day if the client does not elimination to choose correctly.
respond to customary diuretic therapy. Option 1 is incorrect—sodium is necessary for all
individuals and the development of hyponatremia carries its own metabolic consequences.
Option 2 is incorrect—even though paracentesis may sometimes be indicated, it is not the
primary solution to the problem. It is important to look at the underlying fluid and electrolyte
disturbances and correct them in order to prevent the recurring problem of ascites. While low‐
salt diets are often unpalatable, there is nothing to suggest that the client would be
noncompliant with sodium restriction therapy. In addition, other seasonings can be used to
provide taste to the client’s diet.

1.‐ Sodium restriction is critical in managing the occurrence of ascites; therefore, the client should not receive any sodium.
2.‐ There is no need to restrict sodium as paracentesis can be used to remove excess fluid.
3.‐ If the client experiences resistance to diuretic therapy, there may need to be more stringent sodium restrictions.
4.‐ Diets with sodium restriction are unpalatable; therefore, the client will likely be noncompliant with therapy.

214 A mother calls a clinic nurse to state that a letter had Correct answer: 3 Pediculosis capitis is head lice. The nits (eggs) are usually found at the nape of the neck or Note that the word capitis refers to the head to eliminate options 2 and 4. Discriminate
come home with her child from school stating she behind the ears. Head lice do not move away from the scalp to lay eggs; therefore, other between the other two options by selecting the one where the nits would be harder to
should examine her child for the nits from pediculosis choices are not appropriate. detect and to remove.
capitis. She asks where she should look for these nits.
The nurse would tell the mother to examine:

1.‐ The forehead and scalp.


2.‐ In the webs of the fingers.
3.‐ The hair shafts at the nape of the neck.
4.‐ In the folds of elbows.

215 Which of the following instructions would be Correct answer: 2 Activity restrictions should be followed for 6 to 8 months following a spinal fusion. Lying, To answer this question correctly, it is important to understand the disorder and the
appropriate for the nurse to include in the discharge standing, sitting, walking, normal stair climbing, and gentle swimming are generally allowed limitations in the postoperative period. Eliminate option 1 because the restriction is so
teaching of an adolescent following a spinal fusion? following spinal fusion. Bending and twisting at the waist is not recommended, along with extreme. Next, evaluate each of the options and choose the one that protects the spine
lifting more than 10 pounds, household chores such as vacuuming, mowing the lawn, physical immediately after discharge.
education classes, and any sports besides walking.

1.‐ No contact sports will be allowed again.


2.‐ The adolescent should not bend at the waist.
3.‐ Walking is limited to only one half mile per day.
4.‐ The adolescent should not climb stairs.

216 The physician has prescribed Vitamin D for a client. Correct answer: 2 Vitamin D regulates calcium and phosphorus levels by increasing blood levels, increasing The core issue of the question is the purpose and intended effect of vitamin D. Use basic
The client asks the nurse what the medication is for. intestinal absorption and mobilization from bone, and reducing renal excretion of both knowledge of nutrition and vitamin therapy and the process of elimination to make a
Which of the following is the best response by the elements. The statements in the other options are the opposites of the actions of Vitamin D. selection.
nurse?
1.‐ “Vitamin D decreases intestinal absorption of calcium and phosphorus and decreases their mobilization from bone.”
2.‐ “Vitamin D helps regulate calcium and phosphorus balance.”
3.‐ “Vitamin D helps the kidneys rid the body of excess calcium and phosphorus.”
4.‐ “Vitamin D decreases blood levels of calcium and phosphorus.”
217 The nurse working on an adult medical‐surgical unit Correct answer: 4 The nurse should delegate the care of the 53‐year‐old client with hypertension and chronic Recall the principles of delegation. The nurse should not delegate the care of clients who
would assign which of the following clients to the renal insufficiency. This client has a stable medical status. The nurse would want to assess the require assessment due to changes in acuity or status, and clients who require teaching.
licensed practical/vocational nurse (LPN/LVN) under client recently admitted following adrenalectomy and the 24‐year‐old who has hemophilia and Stable clients may be delegated to the LPN/LVN under the RN’s supervision.
the supervision of the RN? fractured a leg the previous day. The nurse would want to provide teaching to the client being
discharged to home following arthroscopy and pain management, limitations in activity, and
follow‐up care.
1.‐ A 45‐year‐old client who underwent bilateral adrenalectomy the previous day
2.‐ A 66‐year‐old client being discharged to home following arthroscopy
3.‐ A 24‐year old with hemophilia who fractured a leg after falling from a horse yesterday
4.‐ A 53‐year‐old client with hypertension and chronic renal insufficiency

218 A client with acquired immunodeficiency syndrome Correct answer: 1 Standard precautions are used with all clients, regardless of the medical diagnosis. Clients Use the process of elimination based on nursing knowledge of standard precautions and
(AIDS) who has Pneumocystis carinii is being admitted with AIDS or Pneumocystis carinii pneumonia are not contagious and do not require the route of transmission for AIDS.
to the nursing unit. The nurse should institute which of transmission‐based precautions.
the following?
1.‐ Standard precautions
2.‐ Airborne precautions
3.‐ Droplet precautions
4.‐ Contact precautions

219 A family member is sitting at bedside and observes Correct answer: 2 Wound infection is decreased by skin preparation when debris and transient microbes from The core issues of the question are recognition of the family question as relating to
the nurse admitting the client from the postanesthesia the skin are removed. The other possibilities are all incorrect since skin preparation will not surgical skin preparation and knowledge of the purposes and expected results of that prep.
care unit (PACU) following a surgical procedure. The prevent complications such as positioning injury or pressure ulcers. Dermatitis does not result Use nursing knowledge and the process of elimination to make a selection.
family member asks why the area around the surgical if surgical skin preparation is omitted.
wound is orange. Which of the following statements
about surgical skin preparation is the best response to
the family member?

1.‐ “It reduces the risk of all postoperative complications.”


2.‐ “It reduces the risk of postoperative wound infection.”
3.‐ “It lessens the chance for decreased tissue perfusion.”
4.‐ “It decreases the possibility for dermatitis.”

220 The nurse is planning care for a client recently Correct answer: 2 It would be counterproductive to confront and challenge a client’s paranoid ideation until The core issue of the question is knowledge of therapeutic communication techniques
admitted with paranoid ideation. The nurse trust has been developed. A consistent program schedule will cut down on the number of with a client who is paranoid. Note the word counterproductive in the stem and the word
determines that it would be counterproductive to do surprises for the client and help develop trust in the staff (option 1). Orienting the client to the challenge in the correct option. It will help you to choose correctly if you can make an
which of the following when working with this client? unit and introducing him to the staff will enable the client to start developing therapeutic association between these words.
relationships (option 3). Communicating clear expectations will prevent the client from being
confused (option 4).

1.‐ Ensure that a consistent program schedule be followed.


2.‐ Confront and challenge inaccuracies in the client’s ideation.
3.‐ Orient the client to the unit and introduce him to the other staff.
4.‐ Establish clearly defined expectations of the client.

221 Following the administration of a measles‐mumps‐ Correct answer: 1 The nurse should assess for signs and symptoms of hypersensitivity reaction following the The core issue of the question is knowledge that the MMR vaccine may cause allergic
rubella (MMR) vaccine, the nurse should make a administration of all vaccines. Wheezing is a sign of hypersensitivity reaction and warrants reaction in clients who have hypersensitivity to egg yolks. Use specific drug knowledge and
priority assessment for which of the following client immediate further assessment and emergency action to prevent possible death. Local the process of elimination to make a selection.
manifestations? discomfort (option 1) may be expected and is treated if necessary with acetaminophen.
Anxiety and vomiting (options 3 and 4) are not associated with administration.

1.‐ Wheezing
2.‐ Pain
3.‐ Anxiety
4.‐ Vomiting

222 The nurse working on an adult medical‐surgical unit Correct answer: 3 The nurse should delegate the care of the 59‐year‐old client with hypertension and Paget’s Recall the principles of delegation. The nurse should not delegate the care of clients who
would assign which of the following clients to the disease. This client has a stable medical status. The nurse would want to assess the client require assessment due to changes in acuity or status, and clients who require teaching.
licensed practical/vocational nurse (LPN/LVN) under awaiting surgery and complete preoperative care and the preoperative checklist. The nurse Stable clients may be delegated to the LPN/LVN under the RN’s supervision.
the supervision of the RN? would also want to assess and care for the client in sickle cell crisis because of the acuity of the
client’s condition. The nurse would want to collaborate and communicate with the pulmonary
rehabilitation specialist to formulate the ongoing plan of care.

1.‐ A 46‐year‐old client with COPD who will be seen by the pulmonary rehabilitation specialist in 2 hours
2.‐ A 26‐year‐old client who is in sickle cell crisis
3.‐ A 59‐year‐old client with Paget’s disease who also has hypertension
4.‐ A 75‐year‐old client who fractured a hip and is awaiting surgical repair scheduled for mid‐morning

223 When assessing the genitourinary system of a 75‐year‐ Correct answer: 2 Benign prostatic hyperplasia (BPH) is the most common disorder of the aging male client. The critical words in the stem of the question are older men. Recall that the prostate gland
old male client, the nurse questions the client about Testicular cancer is the most common cancer in men between the ages of 15 and 35. Testicular undergoes changes in later life to help select the correct option.
symptoms of which of the following conditions that is torsion occurs at any age and gonorrhea is highest in occurrence during the sexually active
common in older men? years. Women 15 to 19 years old and men 20 to 24 years old have the highest rate.

1.‐ Testicular cancer


2.‐ Benign prostatic hyperplasia
3.‐ Testicular torsion
4.‐ Gonorrhea

224 The nurse is teaching a class on newborn care to a Correct answer: 4 At birth, the infant’s skin is thin with little subcutaneous fat. In addition, the infant has a Specific information on the functions of the skin is needed to answer the question. Look at
group of expectant parents. In explaining why the greater proportion of body surface area relative to the amount of water present in the skin. the critical word newborn and think about the characteristics of newborn skin to help make
parents need to protect the infant from heat loss, the Lanugo is shed within a few weeks of birth and has no relationship to heat loss. Sebaceous a selection.
nurse should discuss that the characteristic of the glands and apocrine glands are immature in the infant but are not related to heat loss or
infant’s skin that is responsible for heat loss is: temperature regulation.

1.‐ Lanugo.
2.‐ Nonfunctioning sebaceous glands.
3.‐ Nonfunctioning apocrine glands.
4.‐ Thinner skin.

225 Which of the following statements indicates that a Correct answer: 3 PMS occurs only during the luteal phase of the menstrual cycle (7 to 10 days before The critical word in the question is understands. This tells you that the correct option is
client understands appropriate information about menstrual flow begins). Increasing sexual activity doesn’t prevent PMS, and caffeine can one that contains a true statement. Use knowledge of PMS and that caffeine aggravates it
premenstrual syndrome (PMS)? worsen the symptoms. to successfully eliminate incorrect options.

1.‐ “I have PMS all month long.”


2.‐ “My husbands says if we had sex more often it would help my PMS.”
3.‐ “PMS starts about 10 days before my period.”
4.‐ “I should drink more coffee when I have PMS.”

226 Which of the following symptoms would the nurse Correct answer: 4 Tonic‐clonic seizures are the most common generalized seizures. Periods of inattention and The core issue is knowledge of the characteristics of the most common seizure disorder.
assess for in a client with the most common daydreaming characterize an absence seizure. Sudden loss of muscle tone and falling Use the process of elimination and specific nursing knowledge to answer the question.
generalized seizure disorder? characterize an atomic seizure. Repetitive small muscle group activity characterizes a partial
seizure.
1.‐ Periods of inattention and daydreaming
2.‐ Sudden loss of muscle tone and falling
3.‐ Repetitive small muscle group activity
4.‐ Tonic and clonic activity of the extremities

227 The nurse must assess the temperature and blood Correct answer: 1 Equipment for client care is dedicated to the client on contact precautions and kept in the The key word appropriate suggests there is only one correct answer. Look for the nursing
pressure of a client on contact precautions every shift. client’s room. action that would limit the spread of pathogenic microorganisms.
Which is the appropriate nursing action to minimize
the spread of microorganisms?

1.‐ Keep the equipment in the client’s room.


2.‐ Store the equipment in the soiled utility room between uses.
3.‐ Cleanse the equipment after each use.
4.‐ No special action is required with the equipment.

228 The nurse would implement which of the following as Correct answer: 4 Pulses are assessed frequently to ensure adequate circulation is present and an occlusion or Note the critical words first postoperative day. This tells you that the client condition
the most important measure on the surgical unit on leakage of the graft has not occurred. Pulses should be marked preoperatively so the nurse has could change and that diligent assessment and ongoing monitoring is required. Use
the first postoperative day following surgical repair of a comparison point postoperatively. Pulses may be absent for a short‐term postoperatively knowledge of the surgical procedure and routine postoperative care to make a selection.
an abdominal aneurysm? due to vasospasm or hypothermia. Anticoagulant therapy is not indicated. Trendelenburg
position could reduce blood flow to the affected lower extremities. Elastic stockings may or
may not be ordered because they could interfere with neurovascular assessment of the lower
extremities; however, pneumatic boots would help to prevent deep vein thrombosis and allow
visualization of lower extremities.

1.‐ Administer anticoagulant therapy.


2.‐ Position the legs in Trendelenburg position.
3.‐ Apply elastic stockings to both legs.
4.‐ Palpate peripheral pulses every 2 to 4 hours.

229 The nurse is preparing a client for surgery. Prior to Correct answer: 1 Abrasions, pustules, or other skin conditions have to be assessed and documented because The core issue of the question is knowledge that broken areas of skin are at risk for
completing the skin preparation, the nurse assesses these may interfere with wound healing. Hair growth—lack of it or presence of lanugo or fine infection and need to be reported to the surgeon. Knowing that this is an important item
the surgical site for which of the following? hair—will not interfere with the skin preparation. Pulsation is not always visible or available to helps to prioritize this as the item to assess.
assess depending upon the part of the body being operated on.

1.‐ Presence of pustules or abrasions


2.‐ Absence of hair growth
3.‐ Presence of lanugo
4.‐ Absence of pulsation

230 A client with schizophrenia is admitted to the Correct answer: 4 Option 4 provides the client with the choice of how he would like to take the medication, The core issue of the question is knowledge of therapeutic communication techniques
psychiatric unit. As the nurse approaches the client while being firm that he must take it; the choice gives the client a sense of control and helps to when working with a client who has schizophrenia. Use nursing knowledge of this disorder
with medication, he refuses it, accusing the nurse of reduce the power struggle. Simply telling the client that the medication is not poison (option 1) and the process of elimination to make a selection.
trying to poison him. The nurse’s best response would would do little to persuade him to adhere. Option 2 provides no choice and implies
be to tell him that: punishment. The client must take the medication; therefore, option 3 would be inappropriate.

1.‐ “It is not poison, and you must take the medication.”
2.‐ “If you won’t take this, I will give you an injection.”
3.‐ “I’m sorry you think that this medication is poison. You don’t have to take it right now if you don’t want to.”
4.‐ “You may decide if you want to take the medication by mouth or injection, but this medication will help you.”

231 The nurse is making a plan of care for a client who is Correct answer: 3 Dry mouth occurs from the anticholinergic effects seen with fluphenazine. Options 1 and 2 The core issue of the question is knowledge of drug adverse effects and how to prevent
prescribed fluphenazine (Prolixin) 1 mg daily at are incorrect because orthostatic hypotension is not a major side effect of fluphenazine. them. Recall that anticholinergic effects are of concern with this medication and use the
bedtime. The nurse will include which of the following Confusion (option 4) is not a side effect of this agent. process of elimination to make a selection.
to monitor for side effects of the medication?
1.‐ Remind him frequently to rise slowly when getting out of bed or from a chair.
2.‐ Assess for dizziness or lightheadedness frequently during the day.
3.‐ Make sugarless hard candy, gum, and water available during the day.
4.‐ Monitor for confusion frequently.

232 The nurse working on an adult medical unit would Correct answer: 4 The nurse should delegate the care of the 79‐year‐old who has chronic bronchitis and early Recall the principles of delegation. The nurse should not delegate the care of clients who
assign which of the following clients to the licensed Alzheimer’s disease. This client has a stable medical status. The nurse would want to assess the require assessment due to changes in acuity or status, and clients who require teaching.
practical/vocational nurse (LPN/LVN) under the client undergoing cardiac catheterization and complete the preparation activities for the Stable clients may be delegated to the LPN/LVN under the RN’s supervision.
supervision of the RN? procedure. The nurse would also want to assess the client with low back pain. The nurse would
want to assess and monitor the client who had chest pain the previous evening because of the
nature of the problem.
1.‐ A 46‐year‐old client who will undergo cardiac catheterization later in the morning
2.‐ A 54‐year‐old client who has osteoarthritis and low back pain
3.‐ A 62‐year‐old client admitted the previous evening with chest pain
4.‐ A 79‐year‐old client who has chronic bronchitis and early Alzheimer’s disease

233 A client with venous stasis ulcers is being treated with Correct answer: 1 Elevation of the extremities promotes venous return. Pulses are assessed to ensure adequate The core issue of the question is knowledge of the type of condition that requires the use
an Unna boot. The nurse should include which of the circulation. Option 3 is unnecessary because the Unna boot is treating the ulcer and is changed of an Unna boot and then determining an intervention that meets the same need. To
following additional interventions in the plan of care? every 1 to 2 weeks. answer correctly, you need to determine that the underlying problem is venous in nature
and that leg elevation aids in relieving symptoms of venous disease.

1.‐ Elevating legs and assessment of peripheral pulses


2.‐ Keeping legs dependent for pain relief and improved circulation
3.‐ Wet to dry dressings to ulcer twice daily
4.‐ Elevating legs and standing as much as possible

234 A priority nursing action needed following a full body Correct answer: 4 Anxiety reduction is needed when the client is waiting for the outcome of tests to assist them The purpose of the test is to identify a possible problem and the client’s greatest fear is
scan would include which of the following actions by in processing their feelings and exploring their options based upon the results of the test. that the test will validate that something is wrong. Therefore, communication is the first
the nurse? priority in care of a post‐test client. Pain is a possibility of a problem since the test does
require the client to lie still for a while. But fear will intensify pain even more if not
addressed first. Only minimal radiation exposure does occur during a scan and not enough
to cause any radiation sickness (nausea). Bleeding is not a possible outcome from the scan
since the procedure is not invasive.

1.‐ Pain assessment due to discomfort of the actual procedure


2.‐ Vital signs to assess for possible bleeding
3.‐ Prophylactic antiemetic due to radiation exposure causing nausea
4.‐ Therapeutic communication to reduce possible anxiety caused by outcomes

235 The nurse is teaching a client and family about health Correct answer: 2 COPD places a client at risk to develop malnutrition due to reduction in muscle mass and fat The wording of the question tells you the correct answer is a true statement of fact. Use
maintenance with chronic obstructive pulmonary reserves. Option 1 is incorrect because COPD clients are more likely to suffer from respiratory general knowledge of nutritional concepts and COPD to eliminate each of the incorrect
disease (COPD). The nurse explains to the family that infections due to altered immune response (decreased cell‐mediated immunity, altered options.
nutrition is important in managing this condition using immunoglobulin production, and impaired cellular resistance). Options 3 and 4 are incorrect
which rationale? because COPD clients usually present with weight loss and are hypermetabolic (require
additional calories due to increased energy requirements as a result of increased work of
breathing).
1.‐ COPD clients have an adequate immune response.
2.‐ COPD clients are at increased risk of suffering from malnutrition.
3.‐ COPD clients are likely to experience weight gain due to fluid retention.
4.‐ Decreased energy requirements lead to weight gain.
236 The nurse provides discharge instructions to the Correct answer: 3 In hypertensive urgencies, clients present with a systolic BP greater than 240 mmHg and The core issue of the question is knowledge of the parameters of hypertensive crisis. Use
client taking an antihypertensive medication. The diastolic BP greater than 120 mmHg. In hypertensive emergencies, the client’s diastolic BP is this knowledge and the process of elimination to make a selection.
nurse should include in the teaching plan that a greater than 130 mmHg.
hypertensive crisis will exist if the diastolic blood
pressure (BP) is greater than which of the following?

1.‐ 100 mmHg


2.‐ 120 mmHg
3.‐ 130 mmHg
4.‐ 140 mmHg

237 Which of these statements if made by a client Correct answer: 1 Atherosclerosis indicates the need to adopt a low‐fat diet. Both butter and margarine have 4 The critical words in the question are further discussion. With this in mind, evaluate each
receiving dietary instruction for atherosclerosis would grams of fat per serving, making the client’s statement incorrect and in need of further of the options in terms of how they relate to a low‐fat diet.
indicate a need for further discussion? clarification. The responses in the other options are correct.
1.‐ “Margarine has less fat than butter, so I will no longer use butter.”
2.‐ “I will steam, bake, or broil my foods.”
3.‐ “American cheese has 76 percent fat calories.”
4.‐ “I will increase my consumption of fruits and vegetables.”

238 The nurse would choose to use medical aseptic Correct answer: 3 Medical asepsis requires clean, not sterile, technique. Only option 3 requires medical aseptic Knowledge of medical versus surgical asepsis is essential. Look for similarities in the
technique when collecting which of the following technique. Collecting a wound culture (option 1), suctioning a tracheostomy (option 2), and choices. Options 1, 2, and 4 require sterile technique. Option 3 is the only choice that
specimens? catheterizing the client (option 4) all require the nurse to use sterile asepsis. requires medical aseptic technique.

1.‐ C & S from an abdominal wound


2.‐ Sputum specimen via a tracheostomy
3.‐ Stool specimen for ova and parasites
4.‐ Urine specimen via straight cath

239 Following placement of a central venous line, which Correct answer: 3 Increased heart rate and/or respiratory rate within minutes to several hours following central Consider an acute complication of central line insertion, which would include
information should the nurse report immediately to venous line insertion are symptoms of a pneumothorax caused by puncture of the pleura. The pneumothorax. Then consider how pneumothorax would manifest in the client to make a
the physician? client will require a chest x‐ray to determine if a pneumothorax is present. If the client does selection. Pain (option 1) might be expected to some degree. Fever (option 2) could occur
have a pneumothorax, placement of a chest tube is likely. Pain at the central line insertion site, with infection but this could not happen that quickly. Eliminate option 4 because of the
fever, and diminished breath sounds in lung bases will require intervention, but the etiology of qualifier bases with lung sounds, which indicates atelectasis, not pneumothorax.
these symptoms is not likely to be potentially life threatening as is the development of a
pneumothorax.

1.‐ Pain at the insertion site


2.‐ Fever
3.‐ Increased heart rate and respiratory rate
4.‐ Diminished breath sounds in lung bases

240 A client diagnosed with schizophrenia has improved Correct answer: 2 Ideas of reference or misinterpretation occurs when the client believes that an incident has a The core issue of the question is the ability to draw correct conclusions from the behavior
and is playing a card game with peers. The group personal reference to one’s self when, in fact, it is not at all related. A hallucination is the of a client with schizophrenia. Use knowledge of the features of this diagnosis and the
begins laughing at a joke told to them. The client jumps occurrence of a sight, sound, touch, smell, or taste without any external stimulus to the process of elimination to make a selection.
up and shouts, “You are all making fun of me.” The corresponding sensory organ; they are real to the person (option 1). Delusions are false beliefs
nurse concludes that the client is displaying: that cannot be changed by logical reasoning or evidence (option 3). Loose association is a
vague, unfocused, illogical flow or stream of thought (option 4).

1.‐ Hallucinations.
2.‐ Ideas of reference.
3.‐ Delusions.
4.‐ Loose association.
241 A 70‐year‐old client with chronic obstructive Correct answer: 3 With increased age, there is an increased sensitivity to xanthines. Also, there could be other The core issue of the question is knowledge that the drug level is high and knowledge of
pulmonary disease (COPD) is taking theophylline (Theo‐ disease processes that may lead to this elevated value. The dose of theophylline should be what factors can increase the drug level. Use concepts of the effects of age on
Dur). A blood level is drawn and the result is 25 mg/dL. decreased to get the blood level to the 10 to 20 mg/dL range. Theophylline doses should be pharmacokinetics and the process of elimination to make a selection.
What explanation by the nurse helps the client based on lean body weight to prevent entering the medication into the adipose tissue.
understand this lab result?
1.‐ “Your dose of theophylline needs to be increased.”
2.‐ “Your blood level is low because the dose was based on total body weight instead of lean body weight.”
3.‐ “The lab value could be high because of your age. We may have to decrease the dosage of your medication.”
4.‐ “I am sure that lab value is incorrect. Theophylline levels are never that high.”

242 The mother of a 2‐month‐old receiving immunizations Correct answer: 3 Immunizations interrupt the chain of infection by generating immunity in a susceptible host Knowledge of the chain of infection is required. Immunizations change the immunity
for the first time will also be a beginning nursing by introducing a weakened or killed antigen into the body. Immunizations do not affect the status of the person receiving them. Option 3 is the only choice where that is possible.
student when the next semester starts. When the portal of entry, portal of exit, or the mode of transmission of a pathogenic organism.
mother asks the nurse to relate the immunizations to
what she learned in the microbiology class, the nurse
states that administering childhood immunizations
interrupts the chain of infection at what link?

1.‐ Mode of transmission


2.‐ Portal of entry
3.‐ Susceptible host
4.‐ Portal of exit

243 A 9‐year‐old child is being treated with methimazole Correct answer: 2 The thyroidectomy is the third alternative treatment used when medication and iodine‐based Determine the core issue of the question, which is a disadvantage of performing a
for Graves’ disease. She has not responded to the drug radiation therapy are unsuccessful. There is a great concern of causing hypothyroidism in the thyroidectomy in a child. Use knowledge about hyperthyroid state and age‐related concepts
therapy as quickly as the physician expected so a client. The other statements are not reflective of the underlying concern with performing a to eliminate each of the incorrect responses.
thyroidectomy is being considered. The child’s mother thyroidectomy in a child.
asks the nurse, “Why would the physician seem
hesitant to encourage the surgery?” Which response
by the nurse is best?

1.‐ “The surgery will leave a scar on the child’s neck and will cause problems with her self‐esteem.”
2.‐ “Removal of the thyroid gland may result in permanent hypothyroidism, which will require lifelong hormone replacement therapy.”
3.‐ “The convalescent time for this surgery is 6 months.”
4.‐ “Removal of the thyroid gland causes a change in thermoregulation.”

244 The nurse doing health promotion in an ambulatory Correct answer: 2 Uterine prolapse is caused by weakened pelvic muscles, which can be strengthened by Kegel The core issue of the question is knowledge that Kegel exercises can strengthen the pelvic
women’s health clinic would plan to teach Kegel exercises. The other conditions are not treated with Kegel exercises. floor. Evaluate each of the options to determine which condition could be improved by the
exercises to a woman with which of the following use of these exercises.
conditions?
1.‐ Menopause
2.‐ Uterine prolapse
3.‐ Urinary tract infection
4.‐ Premenstrual syndrome

245 A client with a diagnosis of paranoid schizophrenia Correct answer: 2 With this client, being a danger only to himself (option 1) isn’t enough, he may not be a The core issue of the question is safety of all possible clients in the question. Note the
who threatened his parents with a knife was placed on danger to himself but he still may want to harm his parents (others). Although the goal is for association between the word knife in the stem and the word danger to narrow the
a 48‐hour hold by the courts and the psychiatrist. The the client to continue to take his medication (option 3) and remain in treatment (option 4), possibilities to options 1 and 2. Choose option 2 over 1 because it is the most
nurse explains to the family that once the 48‐hour hold safety is a priority. Depending on state law, the length of hold may be either 48 or 72 hours. comprehensive option.
is expired, the psychiatrist and court must determine if
the client is:
1.‐ A danger to himself.
2.‐ A danger to himself and others.
3.‐ Agreeable to take his medications.
4.‐ Willing to remain in outpatient treatment.

246 Which of the following statements made by a client Correct answer: 1 Clients receiving ophthalmic corticosteroids have an increased risk of infection. Contact The core issue of the question is knowledge that corticosteroids increase risk of infection
receiving ophthalmic corticosteroids indicates a need lenses should not be used during ophthalmic corticosteroid therapy. Options 2, 3, and 4 and how to reduce this risk when taking ophthalmic corticosteroids. Use nursing knowledge
for further teaching? indicate an appropriate understanding of ophthalmic corticosteroid therapy. and the process of elimination to make a selection.

1.‐ “I remove my contact lenses before instilling the medication, then put them back in after 30 minutes.”
2.‐ “I am not wearing my contact lenses for the duration of the corticosteroid treatment.”
3.‐ “I will take my medication for the length of time prescribed by my physician.”
4.‐ “I will return to my physician to have my eyes examined after my treatment is completed.”

247 A nurse is teaching a new group of hospital teen Correct answer: 4 Sneezing and coughing are examples of modes of transmission, whereby droplet nuclei can be Look for commonalities among the options in order to eliminate choices. Options 1, 2, and
volunteers about the chain of infection. Which of the transmitted directly to a susceptible host. 3 are inanimate objects that serve as vehicles to transmit infectious microorganisms.
following items would the nurse include as an example Choose option 4, as direct transmission of microorganisms occurs.
of how an infection would spread through droplets?

1.‐ Nonsterile surgical instruments


2.‐ Soiled linens
3.‐ Contaminated dressings
4.‐ Sneezing and coughing

248 A 12‐year‐old boy has signs of precocious puberty. He Correct answer: 3 The premature secretion of testosterone promotes the closure of the epiphyseal growth To answer this question, it is necessary to have an understanding of the underlying
is 5 feet 7 inches tall, has a deep voice, and has started plates. Many of these children appear very tall around sixth grade, but their friends eventually pathophysiology. Take time to review this information if you have the need.
to shave his facial hair. His friends are envious of his catch up and surpass them in linear growth.
tall stature and basketball skills. The boy comments on
the fact that he expects to be over 6 feet tall and
become a professional basketball player. What will the
nurse use as information in explaining that the client
will probably not reach that height?

1.‐ Neither of the child’s parents is 6 feet tall.


2.‐ The child doesn’t eat enough nutritious food.
3.‐ The early presence of sex hormones stimulates closure of the epiphyseal growth plates, resulting in short stature in the future.
4.‐ Few children attain heights above 6 feet and become professional basketball stars.

249 The client was taught calf‐pumping exercises prior to Correct answer: 1 Calf pumping exercises involve contracting and then relaxing the leg muscles in an alternating The core issue of the question is knowledge of correct implementation of leg exercises in
surgery to decrease the possibility of thrombophlebitis fashion. Options 2, 3, and 4 do not exercise the calf muscles, including the gastrocnemius the perioperative period. Use this knowledge and the process of elimination to make a
developing postoperatively. The nurse observes the muscles. selection.
client performing the procedure and notes that the
client correctly understands the technique when the
client is observed doing which of the following?

1.‐ Alternately contracting and relaxing the leg muscles


2.‐ Alternately flexing and extending the knees
3.‐ Raising and lowering the legs
4.‐ Alternately dorsiflexing and plantar flexing the feet
250 The nurse determines that the highest priority action Correct answer: 4 When intervening in delirium, highest priority is given to nursing interventions that will The core issue of the question is knowledge that a state of delirium is characterized by
when caring for a client who has alcohol‐withdrawal maintain life. Fluid and electrolyte loss caused by nausea and vomiting can be a life‐ some type of metabolic imbalance. Recall that questions that address priorities of care in
delirium would be: threatening condition during alcohol withdrawal, requiring replacement by intravenous unstable clients often focus on physiological needs first. With this in mind, eliminate
therapy. options 1 and 3 first. Choose option 4 over 2 because it assists in correcting the client’s
internal metabolic state, and thus meets a physiological need. Option 2 addresses a safety
need.
1.‐ Reality orientation.
2.‐ Restraint application.
3.‐ Referral to Alcoholics Anonymous.
4.‐ Replacement of fluids and electrolytes.

251 The nurse working the evening shift on an adult Correct answer: 3 The nurse should delegate the care of the 62‐year‐old who had surgery 3 days ago to repair Recall the principles of delegation. The nurse should not delegate the care of clients who
surgical unit would assign which of the following the fractured femur. This client has a stable medical status. The nurse would want to assess require assessment due to changes in acuity or status, and clients who require teaching.
clients to the licensed practical/vocational nurse the client returning to the unit following gastric surgery and the client who had wisdom teeth Stable clients may be delegated to the LPN/LVN under the RN’s supervision.
(LPN/LVN) under the supervision of the RN? extraction. The nurse would want to complete discharge teaching with the client who had the
cholecystectomy.
1.‐ A 24‐year old client who underwent extraction of four wisdom teeth earlier in the day
2.‐ A 54‐year‐old client who had a laparoscopic cholecystectomy the previous day and will be discharged
3.‐ A 62‐year‐old client who underwent open reduction, internal fixation of a fractured femur 3 days ago
4.‐ A 48‐year‐old client returning from PACU following partial gastrectomy

252 The nurse should implement contact precautions with Correct answer: 3 A wound infection can be spread by direct contact with the wound. Scarlet fever, pertussis, Look for commonalities among the options in order to eliminate choices. Options 1, 2, and
the client with which of the following health and rubella involve the spread of infection by respiratory particle droplets larger than 5 4 are contagious infections characterized by coughing. Choose option 3, as direct
problems? microns. transmission of microorganisms occurs by direct contact with the client.
1.‐ Scarlet fever.
2.‐ Pertussis.
3.‐ A wound infection.
4.‐ Rubella.

253 The nurse is caring for a 68‐year‐old male diagnosed Correct answer: 4 The statements in the first three options correctly describe signs of BPH. Option 4 indicates The critical words in the stem of the question are need for further teaching. This tells you
with benign prostatic hyperplasia (BPH). Which of the the need for further teaching because the client should increase his fluid intake (unless that the correct answer is an incorrect statement made by the client. Use the process of
following statements by the client indicates the need contraindicated) to prevent urinary tract infections and lessen dysuria. elimination and knowledge of the disorder to make a selection.
for further teaching?
1.‐ “The enlarged prostate gland causes me to get up three times every night to urinate.”
2.‐ “The enlarged prostate gland may produce blood in my urine.”
3.‐ “I can get urinary tract infections because of the enlarged prostate gland.”
4.‐ “I should cut down on the fluids I drink so I won’t have to urinate so often.”

254 Parents of a 10‐year‐old boy with mild cerebral palsy Correct answer: 2 While work or industry is the primary developmental task of children this age, emphasis Consider the word mild in the stem of the question and choose the response that is the
ask the nurse about having their son join a Boy Scout should not be placed exclusively on school. Recreational activities are an integral part of broadest and most encompassing.
troop that meets after school. The boy attends a growing up, and all efforts should be made to provide access to such programs. Scouting
regular grade school class. The nurse considers which programs provide recognition of individual successes and strengths and can do much to
of the following when formulating a response? enhance a child’s self‐esteem.

1.‐ The rigors of most scout events would be physically beyond this child’s capability.
2.‐ Scouting can provide children of all abilities with opportunities for recreation and socialization.
3.‐ It would be embarrassing for the child to be different from the other boys and lower his self‐esteem.
4.‐ It is more important that the child conserve his energy for doing schoolwork.
255 The nurse should assess carefully a 79‐year‐old client Correct answer: 1 Elderly clients have slower metabolism and elimination of drugs, causing an increased The core issue of the question is knowledge of adverse effects of the drug haloperidol. Use
who has been frequently sedated with haloperidol susceptibility to side effects. Extrapyramidal side effects are most common with haloperidol, a specific nursing knowledge and the process of elimination to make a selection.
(Haldol) for signs of which of the following? high‐potency antipsychotic. Frequent sedation of this elderly client with haloperidol can lead
to the development of tardive dyskinesia, and requires careful monitoring by the nurse.

1.‐ Tardive dyskinesia


2.‐ Fecal impaction
3.‐ Respiratory depression
4.‐ Restlessness

256 A new staff nurse wants to clarify her responsibilities Correct answer: 4 The state nurse practice act defines the scope of nursing practice in each state. Although Use knowledge of the source of various regulations to answer the question. If needed,
regarding delegation. To which of the following there are general principles that apply to all, each state retains the right to formulate its own review concepts related to legal governance of nursing practice.
documents would the nurse mentor refer this nurse? regulations about nursing practice, including delegation. The ANA standards of practice apply
to care given to clients. Job descriptions and policy manuals are agency‐specific and do not
address the state regulations directly.
1.‐ Policy manual
2.‐ Job description
3.‐ ANA standards of care
4.‐ State nurse practice act

257 The nurse is preparing to enter the room of a client Correct answer: 4, 3, 1, The gown is applied first, as it takes the most time to don. The mask is donned next, followed Rationalize the ordering based on nursing knowledge of standard precautions and surgical
with pneumonia caused by penicillin‐resistant 2 by eye protection. These items can be more securely applied with ungloved hands. Gloves are asepsis.
Streptococcus pneumoniae (PRSP). The client has a donned last, so the gloves can be pulled up to cover the cuffs of the gown.
tracheostomy and requires suctioning. Put the
following personal protective equipment in order of
donning. Click and drag the options below to move
them up or down.
1.‐ Eye protection
2.‐ Gloves
3.‐ Mask
4.‐ Gown

258 A school‐age child has recently been diagnosed as Correct answer: 2 The teacher is most aware of the varied reactions of the classmates and together the parents The focus of the question is on how to maintain the client’s self‐esteem and the location
having a seizure disorder. The parents express a and teacher can plan strategies to promote acceptance of this child. A Medic Alert bracelet is of the concern centers around being at school. With this in mind, select the option that
concern about what will happen if the child has a appropriate but will not improve self‐esteem. A psychiatrist might be consulted if the child directly addresses the concern.
seizure at school. The parents are afraid other children shows symptoms of altered self‐esteem, but this is not required now.
will make fun of their child. Which of the following
responses by the nurse would be most helpful?

1.‐ The child should always wear a Medic Alert bracelet.


2.‐ The parents should talk with the teacher about how to handle the situation.
3.‐ The child should learn about the pathophysiology of seizures so his self‐esteem will not be affected.
4.‐ The parents should make an appointment with a psychiatrist to talk about their concerns.

259 The nurse is preparing to administer a purified Correct answer: 2 The client who has had a positive PPD test in the past should be evaluated with a chest x‐ray, The core issue of the question is knowledge of proper procedure and concerns when
protein derivative (PPD) tuberculin skin test to a client. which is the screening test of choice in this case. The arm should be cleansed with alcohol and administering a PPD test. Recall that this is a skin test for tuberculosis to help you recall that
Before administration, the nurse should take which of allowed to air dry prior to the administration of the test. The test is usually read in 48 to 72 assessment of a client’s past reaction is a key first action.
the following actions? hours and the client may wash the area as usual.
1.‐ Cleanse the area thoroughly with soap and water.
2.‐ Ensure that the client has not had a positive test result in the past.
3.‐ Determine whether the client can return to the office in 24 hours for the test to be read.
4.‐ Instruct the client not to wash the area for 48 hours.
260 The nurse determines that an appropriate outcome Correct answer: 3 Initially, the delirious client is dazed, drowsy, and perceptions will be disturbed, making it The core issue of the question is understanding of the condition of delirium. The critical
criterion for the initial nursing care of a client with difficult for the client to sustain attention to any mental task. Delirium is characterized by word in the stem of the question is initial. With this in mind, choose the option that shows
acute delirium would be which of the following? alternating periods of confusion with lucidity; therefore, option 3 is an appropriate initial the beginnings of return of neurological status to normal.
outcome criterion. Options 1, 2, and 4 are appropriate outcome criteria once the client has
been stabilized.
1.‐ The client will verbalize dependence on drugs.
2.‐ The client will demonstrate adaptive coping strategies for dealing with stress.
3.‐ The client will be oriented to person, place, and time during lucid periods.
4.‐ The client will explore reasons for addictive behaviors.

261 A child is admitted to the nursing unit with acute Correct answer: 3 Dehydration results in hypovolemia, which can precipitate acute renal failure in infants and Consider the various etiologies of acute renal failure. Recall that the kidneys need a
renal failure (ARF). When reviewing the nursing children. The other responses are incorrect because they don’t directly impact renal perfusion. minimum glomerular filtration rate to function properly. Use the concept to choose
history, the nurse notes a history of all of the following correctly.
health problems. The nurse concludes that which item
in the child’s history most likely precipitated the onset
of ARF?
1.‐ Chickenpox
2.‐ Influenza
3.‐ Dehydration
4.‐ Hypervolemia

262 The nurse concludes client teaching about infection Correct answer: 1 A client with tuberculosis must wear a particulate respiratory mask if transportation to Knowledge of how tuberculosis is transmitted is essential. Eliminate options 3 and 4
control measures has been effective when a client with another hospital department is unavoidable. This is an element of airborne precautions because they do not address transmission via the respiratory tract. Select option 1 over 2 as
tuberculosis states, necessary to limit the transmission of the microorganism. Tuberculosis is not transmitted via clients would not wear gloves to protect themselves from their own infections.
eating utensils (option 3) or urine (option 4). Removal and disposal of respiratory secretions is
important but does not require the client to wear gloves.

1.‐ “I need to wear a particulate respiratory mask when I go to x‐ray.”


2.‐ “Nurse, can you give me some gloves so I can blow my nose?”
3.‐ “I will need to use paper plates and cups to eat.”
4.‐ “I will flush the toilet twice after urinating.”

263 An 18‐year‐old client is seen in the Emergency Correct answer: 4 Severe scrotal pain, nausea, and absent cremasteric reflex are characteristic of testicular Note the critical word sudden in the stem of the question. Eliminate options 1 and 3,
Department with sudden onset of severe scrotal pain, torsion. Severe pain and an absent cremasteric reflex are not typical symptoms of the which are not as sudden in onset. Choose between Options 2 and 4, noting that the word
nausea, and an absent cremasteric reflex. The nurse disorders listed in the other options. torsion indicates twisting, which is compatible with the symptoms described by the client.
should suspect which of the following conditions?

1.‐ Hydrocele
2.‐ Prostatitis
3.‐ Varicocele
4.‐ Testicular torsion

264 Evidence that the outcome of “restore tissue Correct answer: 4 A goal of venous ulcer care is for the client to experience no signs of inflammation or Focus on the term skin integrity to compare the options. Eliminate options 2 and 3 first
integrity” has been met in a client with a venous stasis infection. This is the goal directly related to tissue integrity. The other options are good because they are not associated with skin integrity. Choose option 4 over option 1 because
ulcer includes: outcomes but do not relate directly to the question as stated. it is a more global or encompassing item, which is typical when determining an outcome.

1.‐ Absence of bleeding.


2.‐ No reports of pain.
3.‐ Increased activity tolerance.
4.‐ No signs of inflammation or infection.
265 A 58‐year‐old client reports to the nurse during a Correct answer: 2 The onset of dementia symptoms for this client was at or before 58 years of age. When The core issue of the question is the association of the age of 58 with the appropriate type
health history that the physician recently diagnosed a Alzheimer’s disease occurs in people under the age of 65, it is called presenile dementia. of dementia. Use nursing knowledge of the types of dementia and the process of
type of dementia. The nurse checks in the medical elimination to make a selection.
record for documentation related to which of the
following most likely disorders?
1.‐ Senile dementia
2.‐ Presenile dementia
3.‐ Pseudodementia
4.‐ Vascular dementia

266 A client is brought to the emergency department Correct answer: 2 The jaw thrust maneuver is used whenever head or cervical spine injury is suspected to avoid Note key information in the stem, which indicates the client has suffered a traumatic
awake and alert following a fall from a ladder from a causing further physiological damage. The head‐tilt‐chin‐lift method (option 1) is the standard injury and is therefore at risk of cervical spine injury. Next use knowledge of basic CPR
height of 15 feet. While the nurse is conducting an method for opening the airway when there is no suspected cervical spine injury. The tongue‐ procedures to select the option for opening the airway in a client with suspected head or
initial assessment, the client becomes unresponsive jaw lift (option 3) aids in visualizing foreign bodies in the airway. The client does not need neck injury.
and stops breathing. Which method should the nurse emergency intubation (option 4).
use to open the airway?

1.‐ Head‐tilt chin‐lift


2.‐ Jaw thrust
3.‐ Tongue‐jaw lift
4.‐ None; client needs emergency intubation

267 The nurse has begun CPR on a 5‐year‐old child. The Correct answer: 3 The proper ventilation rate for a child or infant is 12 to 20 breaths per minute, which is the Use the process of elimination and knowledge of basic CPR procedures to make the proper
nurse times the rate of ventilation to achieve how same as delivering one breath every 3 to 5 seconds. Ventilation rates of 8 (option 1) or 10 selection. Remember that compressions and ventilation rates need to be higher in children
many breaths per minute? (option 2) do not provide sufficient oxygenation for the child during cardiopulmonary arrest. A than in adults to help you choose correctly.
rate of 30 breaths/min (option 4) is excessive and could be harmful.

1.‐ 8
2.‐ 10
3.‐ 20
4.‐ 30

268 A nurse has begun to resuscitate a 10‐month‐old Correct answer: 1 The brachial artery is the correct location for determining whether an infant under one year Eliminate options 2 and 4 first because they are not used in CPR. Choose option 1
infant. After delivering breaths, the nurse next checks of age has a pulse. The radial artery would not generate enough pulsation in an infant to be (brachial) over option 3 (carotid) using knowledge of infant anatomy and accessibility of the
the pulse at which of the following locations? reliable (option 2) and is also more difficult to palpate. The carotid pulse is not as easily site.
located in an infant with a small neck and neck folds (option 3), while the temporal pulse is not
used in CPR for an individual of any age.
1.‐ Brachial
2.‐ Radial
3.‐ Carotid
4.‐ Temporal

269 The nurse on a surgical nursing unit has just called a Correct answer: 3 In an adult, the sternum should be depressed during CPR to a depth of 1.5 to 2 inches. The Use knowledge of basic CPR procedures to answer the question. Eliminate options 1 and 2
code blue using the telephone in the room of an head‐tilt‐chin‐lift method of opening the airway is used for the client who has no head or neck because they indicate incorrect procedure. Eliminate option 4 because the time frame is
unresponsive client who had abdominal surgery. injury (option 1). The nurse should deliver two breaths to initiate ventilation (option 2). The excessively long.
Which of the following actions would be appropriate nurse should reevaluate the client’s status after approximately 1 minute (option 4).
during initiation of CPR?

1.‐ Open the airway using the jaw thrust method.


2.‐ Deliver one deep breath before checking for a pulse.
3.‐ Depress the sternum 1.5 to 2 inches during cardiac compressions.
4.‐ Reevaluate status every 2 to 3 minutes until the code team arrives.
270 The nurse who is doing the documentation during a Correct answer: 3 On an adult client, chest compressions should be done to a depth of 1.5 to 2 inches to be Use the process of elimination and knowledge of basic CPR procedures to answer the
code blue on an adult client observes an unlicensed effective. Options 2 and 4 are excessively deep and could lead to injury, while option 4 is not question. Recall that compressions are never deeper than 2 inches to eliminate options 1
assistive personnel (UAP) doing CPR. The nurse deep enough to provide effective circulation. and 2. Recall that there is only a half inch variability in compression depth to choose option
interprets that the UAP is performing CPR correctly 3 over 4.
after noting that the UAP is depressing the sternum
how many inches?
1.‐ 2.5 to 3.5
2.‐ 2 to 2.5
3.‐ 1.5 to 2
4.‐ 1 to 2

271 The nurse is performing CPR on a 10‐month‐old Correct answer: 3 The rate of compressions for an infant during CPR is at least 100 per minute. Options 1 and 2 Use knowledge of basic CPR procedures to answer the question. Eliminate options 1 and 2
infant. The nurse times the rate of compressions to are higher than the minimum number of compressions per minute, while option 4 does not because they are too excessive for any client. Note that the client in the question is an
achieve a total number of approximately how many deliver a sufficient number of compressions per minute. infant to choose option 3 over 4.
compressions per minute?
1.‐ 180
2.‐ 120
3.‐ 100
4.‐ 80

272 A nurse witnesses an adult male collapse at the Correct answer: 4 The client should not be lying in water or other liquid, which could lead to burns or to First eliminate option 1 because hair interferes with good skin contact of any type of
airport and an automated external defibrillator (AED) defibrillating another individual who comes in contact with the liquid during AED shock electrode. Next eliminate option 3 because brain death can occur within 4 to 6 minutes if
is brought to the scene. The nurse should do which of delivery. The electrodes should not be placed on hairy areas, or the site should be shaved CPR is not initiated. Use general principles of electrical safety to choose option 4 over
the following in utilizing the device? (option 1). All people should stand clear of the individual during an AED shock to avoid being option 2.
defibrillated themselves (option 2). CPR is initiated after 1 minute or whenever the series of
shocks is terminated, as indicated by client condition. However, 5 minutes is too excessive and
could lead to permanent brain damage if the client survives (option 3).

1.‐ Press the electrodes down firmly, because the client has a hairy chest.
2.‐ Instruct another person at the scene to keep the airway open during delivery of the electric shock.
3.‐ Initiate CPR after 5 minutes if the AED has not restored a perfusing cardiac rhythm.
4.‐ Quickly wipe up the spilled coffee under the victim’s chest before using the AED.

273 A nurse is eating in a restaurant when a woman who Correct answer: 1 In a pregnant client, the Heimlich maneuver is performed in a manner that avoids causing Note key information in the question that the client is pregnant. Then use knowledge of
is 8 months pregnant at the next table begins to choke. injury to the fetus. For this reason, the hand placement is at the midsternum rather than at the basic CPR procedures to answer the question. Eliminate options 3 and 4 first because they
Which of the following hand placements should the abdomen (options 3 and 4). The lower sternum (option 1) should be avoided to prevent involve the abdomen, and eliminate option 2 next as possibly unsafe.
nurse use to perform the Heimlich maneuver? accidental fracture of the xiphoid process, which could lead to internal injury.

1.‐ Midsternum
2.‐ Lower sternum
3.‐ Midway between umbilicus and xiphoid process
4.‐ Midway between umbilicus and symphysis pubis

274 The long‐term care nurse has been called to the aid of Correct answer: 2 There is a specific sequence of actions that is performed as part of basic life support when a Remember the ABCs of life support to answer this question. Choose the option that
a resident who has become unconscious after choking client is choking. After positioning the client on the back, the nurse would observe the oral attempts to clear the airway before taking any other actions.
in the dining room. After positioning the client on the cavity to detect any foreign body that may be removed immediately. Next, the nurse would
back, which of the following actions should the nurse open the airway and attempt to ventilate (option 1). If unsuccessful, this process would be
take next? repeated. Finally the nurse would perform abdominal thrusts (option 3). Chest thrusts (option
4) are performed in the adult only for pregnant or obese clients.

1.‐ Attempt to ventilate the client


2.‐ Observe the oral cavity; carry out a finger sweep of the mouth
3.‐ Perform five abdominal thrusts
4.‐ Perform five chest thrusts

275 A nurse enters an adult client’s room and says, “Good Correct answer: 3, 1, 2, The first action of the nurse is to establish unresponsiveness. This can be done by shaking the Specific knowledge of the sequence of events is needed to answer the question. Using the
morning!” while doing initial shift rounds after 4 shoulder and asking if the client is okay. The subsequent actions of the nurse would be to call ABCs (airway, breathing, and circulation) will be of assistance once unresponsiveness has
receiving report. The client does not respond. Put the for help (option 1), open the airway (option 2), and ventilate the client (option 4). been determined.
nurse’s actions in order of priority. Click and drag the
options below to move them up or down.

1.‐ Call for someone to announce a code blue.


2.‐ Open the airway.
3.‐ Shake the client’s shoulder and ask, “Are you okay?”
4.‐ Take the manual resuscitation bag from the head of the bed and give two breaths.

276 An adult client arrives to the Emergency Department Correct answer: 1, 3, 4, An awareness of the risk factors for cardiovascular disease and associated symptoms can The core issue of the question is knowledge of risk factors of cardiac disease leading to
with complaints of chest pain, and shortness of breath. 5 assist the nurse in analyzing the origin of chest pain and prioritizing and implementing chest pain. Eliminate option 2 as unrelated because cardiac pain does not correlate with
The nurse concludes that which of the following appropriate care. Diabetes (option 1), smoking (option 4), and hypertension are known the respiratory cycle.
points, if present in the client’s history, would indicate modifiable and non‐modifiable risk factors. Chest pain that occurs during activity (option 3)
that this pain may be related to cardiac disease? may indicate cardiac ischemia due to the increased oxygen demand. The associated symptom
Select all that apply. of diaphoresis is a known warning sign of cardiac ischemia. Chest pain that worsens with a
deep breath, is most likely pleuritic pain and suggests a diagnosis of pleurisy and the nurse
should next listen for a pleural friction rub.

1.‐ History of diabetes mellitus


2.‐ Pain worsens with a deep breath
3.‐ Pain increases with activity
4.‐ Client smokes a pack of cigarettes per day
5.‐ Client is diaphoretic during pain episode

277 A nurse who is walking through a parking lot at the Correct answer: 3, 2, 4, Current BLS guidelines include establishing responsiveness as the first step to avoid Never initiate CPR on someone until trying to establish responsiveness and determining
mall notices an adult male who calls out for help 1, 5 performing CPR unnecessarily. With the use of AEDs and the benefit of early defibrillation, that the client is not breathing, or is pulseless. Time is critical, and clearly clients in this
before collapsing on the ground. What should the requesting to get the AED or defibrillator equipment is initiated early in the BLS sequence. critical state will need a higher level of care, warranting early notification of emergency
nurse do to perform basic life support correctly? Place Opening the airway, rescue breaths, and assessing circulation follow the recommended personnel, early defibrillation, and maintaining the ABCs of cardiopulmonary support.
the following steps in the correct sequence by clicking sequence.
and dragging the options below to move them up or
down.
1.‐ Perform rescue breathing.
2.‐ Activate the emergency medical system (EMS).
3.‐ Determine unresponsiveness.
4.‐ Position the client, open his airway, and assess for breathing.
5.‐ Check for pulse.

278 A nurse who normally works on an adult medical unit Correct answer: 2 The rate of rescue breathing for a child is 20 breaths/minute, compared with 12 A child’s respiratory rate is faster than an adult’s, which makes this option the best choice.
has floated to the pediatric unit for the day. One of the breaths/minute for an adult. A blind finger sweep should not be performed in infants and children, since the foreign
seriously ill children codes shortly after report. The body could be pushed back into the airway. The airway always must be established, and
nurse responds to the code, incorporating the rescue breathing before compressions. The BLS guidelines recommend two rescue breaths,
understanding that the procedure for initiating basic not one.
life support, unlike for an adult, is:

1.‐ Perform a finger sweep to check for choking.


2.‐ Open the client’s airway, and perform rescue breathing at approximately 20 breaths/minute.
3.‐ Once you establish unresponsiveness, begin chest compressions.
4.‐ Call for help after giving one rescue breath.
279 The nurse is called to assist with the resuscitation of a Correct answer: 2 The jaw thrust maneuver is used when there is suspicion of a neck injury. Diving into shallow The critical concept in this question is the history of diving into shallow water. Also, the
16‐year‐old who dove into the shallow end of a water is a known cause of spinal cord injuries. Despite the unconsciousness of the client and client is an unconscious accident victim. Neck and spinal cord injury should be assumed in
swimming pool. He is unconscious, and currently is the inability to assess for spinal injury, a neck injury is to be suspected in this client. In all other this circumstance, thus warranting the use of the jaw‐thrust method for opening the
lying on his back out of the water near the pool. What scenarios, the head‐tilt chin‐lift is appropriate to open the airway. airway.
method would the nurse use to open the airway?

1.‐ Head‐tilt chin‐lift


2.‐ Jaw thrust
3.‐ Tongue/jaw lift
4.‐ Insert an airway from the emergency kit

280 The nurse responds to a code of an adult client on the Correct answer: 1 The bent arms will displace the downward force and make the chest compressions less Eliminate option 3, since it is an adult client, and 2 and 4 because the positions do not
hospital unit. In the room are two students performing effective. Bouncing movements decrease effectiveness of resuscitation, and most likely will follow the BLS guidelines for correct positioning, and could cause injury to the client.
cardiopulmonary resuscitation (CPR). The nurse cause injuries. Using the heel of one hand is appropriate for CPR in the child.
notices the person performing compressions kneeling
in a straddled position over the client. To which
position would the nurse direct the student to change
in order to improve the depression of the sternum?

1.‐ Positioned at the side of the client, leaning directly over the victim, with arms straight, using a smooth, straight downward thrust
2.‐ Positioned with arms slightly bent, using a quick bouncing movement
3.‐ Using the heel of one hand to depress the sternum
4.‐ Positioned leaning slightly backwards, with arms straight, using a smooth, straight downward thrust

281 While performing CPR using a bag‐valve mask on an Correct answer: 1, 5 Proper positioning of the airway is essential for rescue breathing, and might not be Remember to have an open airway and a good seal of the mask to assure proper rescue
adult client, the nurse observes that the chest is not established with the first attempt. BLS guidelines recommend a second attempt to open the breathing. Increasing rate, force, and adding oxygen will be ineffective if the airway is not
rising and falling with ventilations. What interventions airway. A good seal over the mouth and nose is necessary to ensure ventilations are being open and there is not a good seal.
could be used to improve ventilations? Select all that delivered to the client. These two interventions must be established before any other steps are
apply. taken to ventilate the client.
1.‐ Reposition airway, and try again to ventilate.
2.‐ Increase the rate of ventilations.
3.‐ Increase the oxygen flow into the bag.
4.‐ Forcefully collapse the bag to ventilate.
5.‐ Ensure the mask covers the nose and mouth with a good seal.

282 While recording interventions during a code for an Correct answer: 2 The rate of rescue breathing for an adult client is 12/minute, or 1 every 5 seconds. Use knowledge of BLS guidelines to identify that option 1 is too slow, 3 is the rate for
adult client, the nurse notices that the unlicensed children, and that the selection in the stem is still too fast for an adult.
assistive person (UAP) is ventilating at a rate of 1
breath for every 2 seconds. What rate for ventilations
would the nurse direct the UAP to perform?

1.‐ 1 every 10 seconds


2.‐ 1 every 5 seconds
3.‐ 1 every 3 seconds
4.‐ No change in frequency is warranted
283 The nurse has been performing CPR on an adult Correct answer: 4 Once breathing and circulation return, the recommended position if no injury is the Eliminate all the other options. While a nasogastric tube might resolve the problem with
client. The nurse notes that after a series of cycles of “recovery” position. It has been found to be optimal to keep the airway open, and also will aspiration from emesis, it will not help open the airway. In addition, keeping the client
compressions and ventilations have been completed, reduce the risk of aspiration if the client has an emesis. flat—even with the head of the bed elevated—will not open the airway, because the jaw
the client still is unresponsive, but a pulse is present and tongue will tend to fall back. Do not continue rescue breathing if spontaneous
and breathing has returned. What intervention would respirations are adequate.
the nurse perform to maintain an open airway?

1.‐ Keep the client flat on his back, and elevate the head of the bed 30 degrees.
2.‐ Place the client on oxygen by face mask.
3.‐ Continue rescue breathing until the client no longer tolerates the procedure.
4.‐ Place the client on his side, moving his head and torso simultaneously without twisting.

284 The nurse is performing basic life support on an infant Correct answer: 2 BLS guidelines indicate that when not confident that signs of circulation are present, if you Waiting for one minute would lose time to restore oxygenation and circulation, and BLS
who is under 1 year of age. The nurse thought the feel no pulse, or if you feel a pulse rate less than 60/minute with poor signs of perfusion, you guidelines direct you to not take more than 10 seconds to check for signs of circulation.
brachial pulse was present, but is unsure. What action begin chest compressions. Knowledge of CPR for infants would tell you that a carotid would not be a good place to find
would the nurse take next? a pulse, and in infants, palpating the apical impulse does not assure that circulation is
present.
1.‐ Count the pulse you feel for one minute to verify.
2.‐ Begin chest compressions.
3.‐ Palpate the apical impulse.
4.‐ Palpate the carotid pulse.

285 The nurse in the pediatric unit walks into a 5‐year‐old Correct answer: 1, 3 Since the child was eating, you must determine if foreign body airway occlusion (FBAO) has Look at the information given in the stem of the question. The child has just eaten, which
child’s room and notes the child is on the floor, occurred. This can be accomplished by the tongue/jaw lift and visualization of the mouth. would indicate that FBAO might have occurred. If so, recall knowledge of BLS guidelines for
unconscious and cyanotic. The lunch tray is near the Opening the airway and attempting rescue breathing would aid in identifying if FBAO is children indicating not to do a blind finger sweep. This could push the object further into
bed, with half of the food consumed. What present. Blind finger sweeps are not done. The nurse would not check a brachial pulse on a 5‐ the airway. For children, palpating a carotid pulse is recommended.
interventions would the nurse perform? Select all that year‐old, and an external defibrillator is not the core issue when there is FBAO.
apply.
1.‐ Open mouth, using tongue/jaw lift, to visualize any foreign object.
2.‐ Perform a blind finger sweep.
3.‐ Open the airway, and attempt rescue breathing.
4.‐ Palpate for a brachial pulse, and begin compressions if not present.
5.‐ Call for an external defibrillator.

286 While performing cardiopulmonary resuscitation Correct answer: 4 The most common cause of sudden cardiac arrest is an abnormal heart rhythm called Recall knowledge of causes of cardiac arrest and the BLS guidelines to aid in answering this
(CPR) on an adult visitor in a shopping mall, the mall ventricular fibrillation. Therefore, delivering a shock via the AED can restore normal cardiac question. Early defibrillation is the key in sudden cardiac arrest, and waiting is not
security guard arrives with an automatic external rhythm. An AED warrants use immediately when it becomes available. warranted, which would eliminate options 1 and 3. Rescue breathing is not done during the
defibrillator (AED) device. What actions would the analysis and shock, because of the danger of shock to the rescuer.
nurse at the scene take at this time?

1.‐ Continue CPR for one minute, then apply the AED.
2.‐ Provide rescue breathing during the AED analysis and shock.
3.‐ Wait until EMS is available to apply cardiac monitoring before using the AED.
4.‐ Stop CPR, apply the AED, and follow the instructions provided.

287 A school nurse is performing cardiopulmonary Correct answer: 1 The child is old enough to apply the adult guidelines, which are 1 1/2–2 inches for effective Knowing the age parameters for the techniques in BLS will aid in answering this question.
resuscitation (CPR) on a 9‐year‐old child. Rescue chest compressions. This child is old enough to use the adult guidelines. The other options are either incorrect in
breathing has been initiated. The child has no pulse. depth for compressions or used for smaller children and infants.
The nurse would begin chest compressions using which
of the following methods?

1.‐ Both hands interlocked, compressing the sternum 1 1/2–2 inches


2.‐ The heel of one hand to the depth of 1–1 1/2 inches
3.‐ The middle and ring fingers at a depth of 1/2–1 inch
4.‐ The heel of one hand 1 1/2–2 inches

288 The nurse follows the instructions of the AED for Correct answer: 1 BLS guidelines for AED use is to perform three analyses and, if no shock is indicated and there Knowledge of the AED will help to eliminate the incorrect options. Do not remove the AED
three analyses, with no shock indicated. The adult is still no sign of circulation, perform CPR for one minute before checking circulation and pads, because analyzation of the rhythm is needed again. Also, correct procedure is to
client remains unconscious and pulseless, and is not cycling the AED to analyze the rhythm again. perform only three analyses, then resume CPR for one minute, so the third and the fourth
breathing. What would be the nurse’s next choice are incorrect.
intervention?
1.‐ Leave the AED pads on and perform CPR for one minute, then re‐analyze.
2.‐ Remove the AED pads and continue CPR.
3.‐ Recycle the AED to analyze again for three more intervals.
4.‐ Perform two rescue breaths and then recycle the AED to analyze the rhythm.

289 A nurse dining at a restaurant is summoned to assist Correct answer: 1 An alternate position with the obese client is using the chest thrusts while the individual is in Don’t lose precious time by waiting until a person becomes unconscious to intervene,
with a choking adult who is markedly obese. The a standing or sitting position. especially when alternate methods can be used. Asking the client to lie down is not in the
individual is conscious. While attempting to perform recommended guidelines, and a finger sweep is used only when the client becomes
the Heimlich maneuver, the nurse cannot encircle the unconscious.
arms around the abdomen to be effective. Which
option should the nurse choose to modify the
technique?
1.‐ Attempt chest thrusts with the client in either a standing or sitting position.
2.‐ Wait until the client becomes unconscious, lower him to the floor, and perform abdominal thrusts.
3.‐ Attempt a finger sweep to remove the foreign body.
4.‐ Ask the client to lie down to perform chest thrusts.

290 The nurse working in a food manufacturing plant is Correct answer: 2, 1, 3, Most electric shock injuries in adults occur at work, as is possible in this scenario. Removing In the stem of the question, identify the risk of electrical shock opposed by the client’s
summoned to help with a worker who has collapsed. 4 the electrical source that this client is on or near and providing scene safety prior to being near wires. Scene safety measures to prevent injury to the rescuer would direct you
Upon arrival, the nurse finds an adult male lying on the approaching the victim are the first steps in this sequence. Then follow the BLS guidelines for to select turning off the power before approaching the client to implement CPR.
floor near wires from a bottle‐packing machine. The EMS activation, CPR, and AED use.
individual is not moving, and does not respond to his
name being called. The plant noise is loud, and it is
possible he does not hear his name. Order the
sequence of steps the nurse would follow to assist in
helping this individual. Click and drag the options
below to move them up or down.

1.‐ Approach the client and determine unconsciousness.


2.‐ Direct an employee to turn off the electrical power to the surrounding machines.
3.‐ Activate the emergency medical system (EMS), and summon the automatic external defibrillator (AED).
4.‐ Begin the ABCs of cardiopulmonary resuscitation (CPR).

291 The nurse is monitoring clients in the dining room of a Correct answer: 2 Individuals with partial obstruction can still breathe and cough. They are to be encouraged to Knowledge of BLS guidelines for FBAO will help in determining the answer. Eliminate the
rehabilitation unit. An adult stroke client begins to continue this. FBAO interventions are to be used only with severe or complete airway other options, because there is no indication that there is complete airway obstruction.
cough after eating a piece of meat. What intervention obstruction.
would the nurse use to help this client?

1.‐ Perform the Heimlich maneuver with the client in a sitting position.
2.‐ Encourage the client to continue to cough to clear the airway.
3.‐ Wait until the client becomes unconscious to perform the Heimlich maneuver.
4.‐ Attempt a finger sweep to help remove the piece of meat.
292 The nurse has conducted client teaching with the Correct answer: 3 The prodromal period is the time between the initial symptoms and the presence of the full‐ The core issue of the question is knowledge of client teaching points related to
mother of a 4‐year‐old child who has been exposed to blown disease. The rash would not be apparent during this time. All the other statements are chickenpox, particularly related to the timing of symptoms. Use nursing knowledge and the
chickenpox. In evaluating the effectiveness of the correct. process of elimination to make a selection.
instruction, the nurse determines that the mother
needs additional information after the mother makes
which statement?

1.‐ “I should monitor my child for Reye syndrome, which is a complication of chickenpox.”
2.‐ “My child should not visit my pregnant sister at this time.”
3.‐ “During the prodomal period, my child will have pox all over his body.”
4.‐ “Chickenpox is a viral infection that can be spread to other children.”

293 A mother overhears two nurses discussing the Correct answer: 1 The incubation period is the time between exposure and outbreak of the disease. It is often a The core issue of the question is knowledge of the significance of the prodromal period in
incubation period for a measles outbreak. The mother period when the child can be contagious without others being aware of the possible exposure. a communicable disease. Use nursing knowledge and the process of elimination to make a
asks the nurses why it is important to know the selection.
incubation period. The nurse’s reply would include
which of the following statements about the
incubation period?
1.‐ It describes a period when the child might be contagious.
2.‐ It determines the severity of the infection.
3.‐ It varies depending on the age of the child.
4.‐ It is a time when medications can prevent the development of symptoms.

294 A 9‐year‐old child is at the 98th percentile for weight Correct answer: 2 The NCHS growth charts use the 5th and 95th percentiles as criteria for determining those The critical words are "9‐year‐old child" and "98th percentile for weight and 40th
and at the 40th percentile for height. The school nurse children who fall outside the normal limits for growth. Children whose height and weight are percentile for height." Knowledge of the growth charts and normal growth is needed to
determines that this child is: above the 95th percentile are considered overweight or large for stature. Prepubescent answer the question correctly.
growth spurts are between ages 10 and 12 for girls and 12 and 14 for boys. This is not a normal
proportion for height and weight for this 9‐year‐old.
1.‐ Underweight or small in stature.
2.‐ Overweight or large in stature.
3.‐ Experiencing a prepubescent growth spurt.
4.‐ Normal for size.

295 A 2‐year‐old child in the hospital for a fractured Correct answer: 1 Caladryl will reduce itching and discomfort and therefore diminish scratching and skin The core issue of the question is knowledge of various products used in the care of
femur breaks out with chickenpox. Which nursing breakdown. Acetylsalicylic acid should not be given to young children with a viral disease children and which one will reduce the likelihood of itching or pruritus with skin lesions. Use
intervention will best prevent secondary skin because of the relationship to Reye syndrome. Immunoglobin will not decrease skin eruptions. nursing knowledge and the process of elimination to make a selection.
infections? Nubaine is a narcotic analgesic.
1.‐ Caladryl lotion to lesions
2.‐ Acetylsalicylic acid
3.‐ Immune globulin for the first 3 days
4.‐ Nubaine every 4 hours as needed for pain

296 A child is being treated at home for chickenpox. The Correct answer: 1 Tepid baths allow heat to be removed from the body. Aspirins are avoided because of the risk The core issue of the question is an effective measure to prevent febrile seizures as a
home‐health nurse is visiting and notes an elevated of Reye syndrome. The child should wear only light clothing to allow heat to escape. Antibiotics complication of fever in a child. Use nursing knowledge and the process of elimination to
temperature. To prevent a common complication of an are not usually ordered for this viral infection. make a selection.
elevated temperature, the nurse recommends which
of the following?
1.‐ Tepid sponge baths
2.‐ Aspirin as needed for fever control
3.‐ Keep child well covered to prevent chilling
4.‐ Antibiotics as ordered
297 A child has been diagnosed with mumps, and the Correct answer: 4 Mumps is a viral infection and thus antibiotics will not be effective. The other statements are The core issue of the question is knowledge of supportive measures for a child with
mother has been given instructions on caring for the true. Acetaminophen, fluids, and soft foods are helpful, and the mother should watch for mumps. Use nursing knowledge and the process of elimination to make a selection.
child during the acute period. Which statement by the vomiting and headache.
mother indicates a need for additional education?

1.‐ “I can give my child acetaminophen for fever.”


2.‐ “My child will be more comfortable if I give him fluids and soft foods.”
3.‐ “I should watch my child for headache and vomiting.”
4.‐ “I will give my child antibiotics every four hours around the clock.”

298 A 2‐year‐old child with rubeola (measles) is brought Correct answer: 3 The child has a stuffy nose, which can impair air exchange. Nursing care involves use of a cool‐ The core issue of the question is the ability to set appropriate priorities of care for a child
to the hospital with a rash covering the entire body, mist vaporizer and gentle suctioning of the nose. The rash does not cause skin impairment. A 2‐ with a communicable disease. Use nursing knowledge and the process of elimination to
photophobia, and stuffy nose that interferes with year‐old will not have a disturbed body image. Disturbed sleep pattern would have less priority make a selection.
breathing. The nurse utilizes which of the following than gas exchange if this problem developed.
nursing diagnoses as a priority for care when
administering care to this child?

1.‐ Impaired skin integrity


2.‐ Disturbed body image
3.‐ Risk for impaired gas exchange
4.‐ Risk for disturbed sleep pattern

299 A child is exposed to a playmate who contracted Correct answer: 2 The upper respiratory symptoms may be early prodromal symptoms of chickenpox. The The core issue of the question is knowledge of the incubation period for chickenpox. Use
chickenpox. Two days later, the child is admitted to the incubation period of chickenpox is 10 to 21 days. The other responses are either too short nursing knowledge and the process of elimination to make a selection.
hospital for another problem, and the parents inform (option 1) or too long (options 3 and 4).
the nurse of the exposure on admission. How long
after the exposure should the child be watched for
signs of upper respiratory illness?

1.‐ 5 to 10 days
2.‐ 10 to 21 days
3.‐ 21 to 25 days
4.‐ One month

300 The home‐health nurse sees a child with mumps. The Correct answer: 3 Cool fluids will help decrease the swelling of the glands around the mouth and neck. Acidic The core issue of the question is knowledge of foods and beverages that will be helpful to
mother says that the child is not eating well and asks foods are too irritating and difficult to swallow. Warm, chopped foods may be difficult to the child with mumps. Use principles of diet therapy that utilize cool, soft, and nonirritating
for suggestions. The nurse most appropriately suggests swallow (option 1), and spices are also likely to be irritating (option 2). The child should be food items to make a selection.
which of the following? given small, frequent meals with soft foods rather than a regular diet (option 4).

1.‐ Provide warm, chopped foods.


2.‐ Provide cool table foods with spices.
3.‐ Provide cool fluids with minimum of acids.
4.‐ Provide a regular diet tray at frequent intervals.

301 The mother of a 3‐year‐old child with measles calls Correct answer: 3 Soothing the skin with an oatmeal‐based substance will decrease the itching and redness. The core issue of the question is an effective measure to treat itching caused by a
the nurse at the clinic and asks what she can do to help Overdressing the child will increase perspiration and thereby increase the itching. Although communicable disease such as measles. Use nursing knowledge and the process of
decrease the redness and itching. The nurse responds drinking adequate fluids is helpful, it does not directly affect the itching. elimination to make a selection.
that which of the following actions is likely to be
helpful?
1.‐ Overdress the child and cause him to perspire.
2.‐ Keep the child out of drafts.
3.‐ Bathe the child in an oatmeal (Aveeno) bath.
4.‐ Provide adequate oral fluids.
302 The clinic nurse is working with a toddler who has Correct answer: 1 The route of transmission of roseola is unknown. It is not known to be transmitted by the The core issue of the question is knowledge of transmission of roseola. The wording of the
been diagnosed with roseola (exanthema subitum) respiratory tract (option 2), contact with contaminated articles (option 3), or body secretions question tells you the correct answer is also a true statement. Use nursing knowledge and
after being seen for fever and a skin rash. The nurse such as urine or stool (option 4). the process of elimination to make a selection.
makes which response to the mother who asks how to
reduce the risk of infecting other children at home?

1.‐ “There is no way to reduce risk because the route of transmission is unknown.”
2.‐ “Do not allow the child to cough or sneeze in the presence of others whenever possible.”
3.‐ “Use disposable dishes and eating utensils, and dispose of them in a separate trash bag.”
4.‐ “Select one bathroom to be used exclusively by the toddler until the rash clears.”

303 A college student was hospitalized following onset of Correct answer: 2 Pertussis is most infectious early in the course of the disease, so it is not necessary for the The core issue of the question is knowledge of care to a client recovering from pertussis.
a severe case of pertussis. In preparing for discharge, client to self‐isolate following discharge from the hospital. Coughing bouts may still be Note that the client is nearing discharge and is not in an acute state to choose the item that
the nurse would correct which client statement that triggered by irritants, so these should be avoided. Frequent handwashing and increased fluid does not need to continue. The wording of the question tells you the correct answer is an
indicates a misconception about postdischarge care? intake are generally helpful measures that should also be continued in the home setting. incorrect client statement. Use nursing knowledge and the process of elimination to make a
selection.
1.‐ “Irritants that I breathe, such as smoke or dust, could make me have coughing spells again.”
2.‐ “I will try to avoid being around people for a full week after going home so I don’t spread this to others.”
3.‐ “I will be very careful to wash my hands often.”
4.‐ “It will still be important to try to drink a lot of fluids when I go home.”

304 A child who may have scarlet fever is being evaluated Correct answer: 3 Koplik's spots are seen with roseola, not scarlet fever. Reddened edematous pharynx, red The core issue of the question is the ability to discriminate between clinical findings
in the urgent care clinic. The nurse concludes that the strawberry tongue, and rash in the axillae and groin are findings consistent with scarlet fever. associated with scarlet fever and roseola. The wording of the question tells you the correct
client’s presentation is not consistent with scarlet answer is an incorrect client statement. Use nursing knowledge and the process of
fever after noting which of the following during elimination to make a selection.
assessment?
1.‐ Rash in the axillae and groin
2.‐ Pharyngeal redness and swelling
3.‐ Koplik's spots in the oral mucosa
4.‐ Red strawberry tongue

305 The nurse is assessing a child in the outpatient clinic Correct answer: 4 Fifth disease is characterized by flulike symptoms such as fever, malaise, nausea, and The core issue of the question is the ability to discriminate the classic sign of Fifth disease
who has fever, lethargy, nausea, and vomiting. The vomiting, and by the characteristic “slapped cheeks” appearance. This finding is not from other childhood communicable diseases. Use nursing knowledge and the process of
nurse notes that the child’s cheeks have the characteristic of chickenpox, measles, or diphtheria. elimination to make a selection.
appearance of being wind‐burned or slapped. The
nurse suspects which of the following childhood
communicable diseases?
1.‐ Chickenpox
2.‐ Measles
3.‐ Diphtheria
4.‐ Fifth disease

306 The spouse of a postal worker who contracted Correct answer: 4 Anthrax is caused by a bacterium and is therefore amenable to treatment with antibiotics. The core issue of the question is knowledge of available treatment methods for anthrax.
cutaneous anthrax asks the nurse whether this Antivirals and immune globulin play no role in treating this disease, and the statements in Use nursing knowledge and the process of elimination to make a selection.
communicable disease can be treated. Which of the options 1 and 2 are incorrect because they indicate no treatment is available.
following responses by the nurse is most appropriate?

1.‐ “No, there is only supportive care available for the itching associated with skin lesions.”
2.‐ “No, although we will be ready to provide aggressive respiratory support measures if needed.”
3.‐ “Yes, the infection can be treated with antiviral agents and immune globulin.”
4.‐ “Yes, the infection can be treated with antibiotics such as ciprofloxacin or erythromycin.”
307 The nurse is providing health teaching to a group of Correct answer: 6 The incubation period for infectious mononucleosis is up to 6 weeks (with a minimum of 4 The core issue of the question is knowledge of the incubation period for infectious
high school students regarding infectious weeks). This has important implications for the nurse and the client, since the source of the mononucleosis. Specific knowledge is needed to answer this type of question. Note that the
mononucleosis. When discussing the incubation period exposure may be difficult to determine after several weeks. question asks for the number of weeks, which suggests that the number to be typed in is
as part of disease transmission, the nurse explains that not excessively large.
the incubation period for this infection is ____ weeks.
Write in a numerical answer.

308 The child is receiving an intravenous antibiotic that Correct answer: 4 The nurse’s responsibility involves early recognition of side effects from a drug. Therefore, Recognize that ototoxicity is damage to the ears and look for options that refer to ears
has a known side effect of ototoxicity. The nurse the nurse would monitor the child for symptoms of ototoxicity. Diluting the dose and slowing and ear function.
administering the dose should: the infusion would not diminish the total dose of drug and would not prevent ototoxicity. The
nurse cannot decrease a dose independently. Reducing the dose will decrease the blood levels
and may lead to bacterial resistance.

1.‐ Dilute the dose as much as possible.


2.‐ Infuse the dose over one hour.
3.‐ Decrease the dose by 50%.
4.‐ Monitor the child for ringing in the ears and dizziness.

309 A child is being treated at home for chickenpox. The Correct answer: 1, 5 Tepid baths allow heat to be removed from the body. Aspirins are avoided because of the risk Knowledge of the nursing management of a child with fever will help to choose the correct
home‐health nurse is visiting and notes an elevated of Reye Syndrome. The child should have only light clothing to allow heat to escape. Antibiotics answer.
temperature. To prevent a common complication of an are not usually ordered for this viral infection.
elevated temperature, the nurse recommends which
of the following? Select all that apply.

1.‐ Antipyretics such as acetaminophen and ibuprofen


2.‐ Aspirin as needed for fever control
3.‐ Keep child well covered to prevent chilling
4.‐ Antibiotics as ordered
5.‐ Remove unnecessary clothing

310 A child who enters the nursing clinic has a diagnosis Correct answer: 1 Immunoglobulin titers are low in children with SCID, with or without an infection. The low The critical concept is the severe combined immunodeficiency. Select the option that
of rule out severe combined immunodeficiency disease titer levels are what prevent the child from fighting an infection. The other options do not contains the laboratory value that would be expected in this client.
(SCID). The child is being seen in the clinic because of a address this concept.
possible infection. In evaluating the lab reports, the
nurse would expect to find:

1.‐ Low immunoglobulin titers.


2.‐ Elevated white blood cell counts.
3.‐ Elevated eosinophil counts.
4.‐ Positive RAST tests.

311 The nurse is working in a clinic that provides free Correct answer: 2 Epinephrine 1:1000 is the drug of choice for an acute anaphylaxis reaction. A child may have Knowledge of the emergency management of anaphylaxis will aid in choosing the correct
immunizations to clients with financial hardship. In the allergies yet unknown at the time of immunizations. Albuterol is a bronchodilator that opens answer. Learn to automatically pair anaphylaxis with epinephrine.
morning before seeing any of the day’s clients, the the airway but epinephrine is the drug of choice during anaphylaxis. Toxoids and
nurse checks that which of the following supplies is immunoglobulins are other classes of drugs that affect the immune system but they do not
available should a client experience an anaphylactic treat anaphylaxis.
allergic reaction to a vaccine?

1.‐ Albuterol (Proventil)


2.‐ Epinephrine
3.‐ Immunoglobulin
4.‐ Toxoid

312 The child has been diagnosed with mumps. The home‐ Correct answer: 4 Children with mumps are uncomfortable but rarely very ill. Give non‐aspirin analgesics and Critical words are “indicates a need for additional teaching.” Look for an answer that is
health nurse has given the mother instructions on antipyretics to control fever and pain. Swallowing and chewing may be painful so give fluids incorrect. Knowledge of the nursing management of the child with mumps will aid in
caring for the child during the acute period. Which and soft foods. Be alert to signs of complications such as headache, stiff neck, vomiting, and choosing the correct answer.
statement by the mother indicates a need for photophobia which may indicate meningeal irritation. Antibiotics are not prescribed.
additional education?
1.‐ "I can give my child acetaminophen for fever."
2.‐ "My child will be more comfortable if I give him fluids and soft foods."
3.‐ "I should watch my child for headache and vomiting."
4.‐ "I will give my child antibiotics every four hours around the clock."

313 A 2‐year‐old child is seen in the pediatric clinic with Correct answer: 3 Koplik’s spots are unique lesions found in the mouth of the individual with rubeola. The rash characteristics are not enough to distinguish rubeola from other diseases. Select
rubeola (measles). The mother asks how the doctor the one unique feature that is seen in no other disease.
can be sure it is rubeola and not some other disease.
The nurse explains that the characteristic lesion of
rubeola is:
1.‐ The distribution of the rash.
2.‐ The appearance of the rash.
3.‐ Koplik’s spots.
4.‐ The low grade fever.

314 A young infant is admitted to the pediatric unit with a Correct answer: 4 Altered temperature, jaundice, and respiratory distress are all symptoms of sepsis in infants. Recognize that all responses are correct for this child. Consider which diagnosis would
diagnosis of sepsis. The nurse is completing a nursing Respiratory function is the highest priority. have the highest priority.
assessment. The priority assessment for this infant
would be which of the following?

1.‐ Skin integrity


2.‐ Temperature
3.‐ Jaundice
4.‐ Respiratory function

315 In caring for an adolescent with suspected narcotic Correct answer: 1, 2, 5 Typical symptoms of narcotic abuse include constricted pupils, euphoria, and respiratory The symptoms that the nurse will assess for are the same that would be seen in the client
abuse, the nurse would monitor the adolescent for depression. Options 3 and 4 do not apply. receiving a narcotic for a therapeutic purpose.
which of the following? Select all that apply.

1.‐ Constricted pupils


2.‐ Euphoria
3.‐ Hyperactivity
4.‐ Aggressive behavior
5.‐ Respiratory depression

316 The nurse in the hospital would suspect severe Correct answer: 1 The first infection often seen in these children is oral candidiasis (thrush). That along with the Two of the options refer to a newborn. Eliminate those options because maternal
combined immunodeficiency disorder (SCID) when low white blood cell count would a be a warning symptom. A 2‐year‐old is unlikely to have antibodies acquired intrauterinely will protect these infants.
which child is admitted to the unit? survived this long undiagnosed.
1.‐ A 2‐month‐old with thrush and low white blood cell counts
2.‐ A 2‐year‐old with history of recent repeated infections
3.‐ A newborn with positive TORCH titer
4.‐ A newborn admitted with positive ELISA test
317 A 5‐year‐old child is brought into the clinic after Correct answer: 3 Maintaining an open airway is always the highest priority. With anaphylactic shock, the Note the critical word “action” in the stem of the question. This suggests the correct
experiencing an insect sting. The child appears to be airway may constrict, mucous membranes swell, and air trapping occurs. answer is a nursing intervention rather than an assessment. Airway always has the highest
going into anaphylactic shock. The nursing action with priority.
the highest priority is:
1.‐ Assessment of urinary output
2.‐ Application of cold, wet compresses
3.‐ Maintaining an open airway
4.‐ Encouraging fluid intake

318 The mother of a newborn is concerned her infant has Correct answer: 1 Babies are born with nonspecific immunity. Active immunity and specific immune response Consider which type of immunity a young infant would have. Immunizations can be
been exposed to a communicable disease. She states are developed over time with exposure to an organism. Immunizations are inactivated eliminated because a newborn will not have had any.
the doctor says the baby is protected because of some substances or weakened organisms given to promote the development of immunity.
kind of immunity, but is unsure of the type. The nurse
would identify the type of immunity as:

1.‐ Nonspecific immunity.


2.‐ Active immunity.
3.‐ Specific immune response.
4.‐ Immunizations.

319 A 12‐year‐old boy is hospitalized and diagnosed with Correct answer: 2 The family will need to know how to protect themselves from the virus. Handwashing is the Knowledge of the emotional support and care of the child with HIV will help to choose the
the recent development of human immunodeficiency first line of protection. However, family coping skills will best be enhanced by the nurse correct answer.
virus (HIV) infection secondary to factor transfusions demonstrating acceptance of the child. This along with child and family education will help the
for hemophilia. The family of the child is very family deal with this disease.
concerned about the risk this child presents to the rest
of the family and questions whether the child should
remain in the home. The nursing activity that will best
promote family coping would be:

1.‐ Explain to the family that the infection cannot be spread by casual contact.
2.‐ Demonstrate positive acceptance of the child with each contact.
3.‐ Explain that prophylactic drugs will prevent the virus from spreading.
4.‐ Show the family how to wash their hands properly.

320 A newborn has health problems immediately after Correct answer: 4 TORCH is an acronym for toxoplasmosis; other, which includes hepatitis and syphilis; rubella; Since all the letters in TORCH are capitalized that should be a hint that this is an acronym.
birth. The mother tells the nurse that the doctor cytomegalovirus; and herpes simplex. These infections are caused by bacteria, viruses, and
suspected a TORCH infection, but she does not know other organisms.
what causes this. The nurse would respond that a
TORCH is:
1.‐ A viral disease like meningitis.
2.‐ A bacterial infection that causes mental retardation.
3.‐ A blood borne pathogen that is contracted during delivery.
4.‐ An acronym for a group of infections that can harm the fetus.

321 The school nurse is teaching teenagers about how the Correct answer: 3 Transmission is by blood or body fluid contact. It is not an inherited disorder. Equipment is Since HIV is a blood borne pathogen, look for the option that would have blood exposure.
human immunodeficiency virus (HIV) is spread. The not shared in the nursery, but the virus can be spread only if the contact involves blood or
students state they know about blood transfusions, IV body fluid.
drug use, and sexual contact, but question how babies
can be born with HIV. The nurse explains that babies
can receive the HIV virus:

1.‐ On the genes from the father.


2.‐ On the chromosome from either parent.
3.‐ During delivery from maternal contact.
4.‐ In the nursery from shared equipment.

322 A child is to receive "allergy shots." The mother tells Correct answer: 4 Hyposensitization injections carry the risk of allergic reaction including anaphylaxis. They The core concept in this question is maintaining a safe environment for the child.
the office nurse that she is a nurse and asks why she should only be given in a controlled environment with emergency drugs and equipment on
can’t give her child his shots at home because she was hand.
trained to give insulin injections to the family pet. The
office nurse should respond:

1.‐ "That sounds like a good idea, then your child will not be exposed to other children in the office waiting room."
2.‐ "Let me check your injection technique this time, and next time you can give it at home."
3.‐ "No, your insurance company will not pay for allergy shots in the home."
4.‐ "Allergy shots should be given in a controlled environment so you will need to bring your child in for his shots."

323 A child is exposed to a playmate that contracted Correct answer: 2 The upper respiratory symptoms may be early prodromal symptoms of chickenpox. The Recognize that upper respiratory symptoms relate to the onset of chickenpox, and
chickenpox. Two days later the child is admitted to the incubation period of chickenpox is 14 to 21 days. consider the prodromal period.
hospital for another problem. The nurse is informed of
the exposure on admission. How long after the
exposure should the child be watched for signs of
upper respiratory illness?
1.‐ Five to 10 days
2.‐ 14 to 21 days
3.‐ 21 to 25 days
4.‐ One month

324 The nurse is providing care to a toddler who has Correct answer: 2 Option 1 is incorrect as gastric lavage would be performed before the administration of Consider the purpose of the lavage and activated charcoal to deternine the correct order
ingested an unknown amount of his grandfather's activated charcoal. The activated charcoal will aid in the absoption and removal of any of administration.
medication which is described as "a white pill." The medication left after gastric lavage is completed. The term lavage indicates wash the stomach,
physician has ordered the gastric lavage and removing the saline after administration.
administration of activated charcoal. What action
should the nurse take?
1.‐ Question the order because gastric lavage and activated charcoal are not to be used together.
2.‐ Perform the gastric lavage and then administer the activated charcoal.
3.‐ Administer the activated charcoal, then perform gastric lavage.
4.‐ Perform gastric lavage, leaving the saline solution in the stomach, and then administer the activated charcoal.

325 Parents report that their small child stiffens when Correct answer: 1 Although a thorough examination and assessment is necessary to diagnose autism, it can be Recall that lack of interaction with others is a primary feature of autism.
being held and does not smile or make eye contact suspected based on the information provided. These symptoms are not normally associated
with them. Based on this initial information, the nurse with the other disorders listed.
suspects that the child may have which disorder?

1.‐ Autism
2.‐ Attention deficit hyperactivity disorder
3.‐ Mental retardation
4.‐ Down syndrome

326 The nurse is providing information about reducing Correct answer: 3 Both live and silk flowers will have increased dust levels associated with them. The other Eliminate those options which would reduce dust in the environment leaving only the
exposure to antigens in a client who is allergic to dust. activities are appropriate. incorrect response.
When the nurse visits the home, she evaluates the
learning of the family. Which finding in the client’s
home indicates a need for additional teaching?
1.‐ Hardwood floors with attractive throw rugs decorate the family room
2.‐ The adult family members have switched to the nicotine patch as they attempt to stop smoking
3.‐ The room is decorated with hanging live plants and arrangements of silk flowers
4.‐ The family reports they take turns vacuuming the entire house each day

327 A teenager has been diagnosed with asthma, and Correct answer: 2 Cromolyn sodium is an aerosol taken daily to prevent an attack. All of the other answers are Options 1 and 4 discuss the drug relieving an immediate attack and can be eliminated.
cromolyn sodium (Intal) has been ordered for this incorrect. Knowledge of the drug will help you to choose from the remaining options.
child. Which statement by the child indicates a correct
understanding of this drug?
1.‐ "I should take this drug at the first sign of an asthma attack."
2.‐ "This drug won’t stop an asthma attack but may prevent a future attack."
3.‐ "Cromolyn sodium is a form of corticosteroid."
4.‐ "After taking one dose, I should wait 15 minutes and then take a second dose if my wheezing hasn’t stopped."

328 A client has an opportunistic respiratory infection. Correct answer: 3 An opportunistic infection is one in which an individual develops a disease from an organism Note the use of the word compromised. Select the option that matches the definition of
Which of the following is most likely correct? that does not cause disease in healthy individuals. This occurs with compromised immunity. opportunistic infection.

1.‐ The client has consumed contaminated food or water.


2.‐ The client has encountered an extremely virulent microorganism.
3.‐ The client's immune system is compromised.
4.‐ The client has likely become infected in a healthcare facility.

329 A client presents in the emergency department with Correct answer: 2 Antibiotics may affect the outcome of the culture. Fever will continue to be present until the This item requires priority setting. All of the options may be pertinent, but identification of
fever of 102 degrees F, malaise, and a productive bacteria are eliminated, making obtaining a culture a priority. the first step is important.
cough. Which of the following should be done first?

1.‐ Administer the prescribed antibiotic


2.‐ Obtain a sputum culture
3.‐ Administer acetaminophen to lower fever
4.‐ Teach client the importance of handwashing

330 The drug of choice to treat a "walking" or Correct answer: 2 The erythromycin products are the best for treating mycoplasmal pneumonia or walking Select the option that identifies a drug commonly used for pneumonias.
mycoplasmal pneumonia is: pneumonia. Vermox is used for helminthic infections; Aralen hydrochloride is used for
protozoal infections; and Chloromycetin is used for spirochetal infections.
1.‐ Mebendazole (Vermox).
2.‐ Erythromycin (E‐mycin).
3.‐ Chloroquine (Aralen hydrochloride).
4.‐ Chloramphenicol (Chloromycetin).

331 Which of the following refers to the ability of bacteria Correct answer: 2 A pathogen is any organism capable of causing disease. Pathogenicity refers to the ability of Note the use of the word “pathologic” in the question. This will lead to selection of the
to produce pathologic changes or disease in the host? the organism to cause pathologic changes. correct response.

1.‐ Virulence
2.‐ Pathogenicity
3.‐ Toxogenicity
4.‐ Latency

332 Which of the following is true concerning human Correct answer: 1 The virus makes a DNA copy of its own RNA using the reverse transcriptase enzyme and the Identify the response that addresses the pathophysiology of HIV.
immunodeficiency virus (HIV)? DNA copy is inserted into the genetic material of the infected cell.
1.‐ HIV infection involves CD4 receptor protein on the surface of helper T‐cells.
2.‐ The presence of circulating antibodies that neutralize HIV is evidence that the individual has immunity to HIV.
3.‐ HIV replication occurs extracellularly.
4.‐ DNA replication is similar to that of other viruses.

333 Which of the following viruses is most likely to be Correct answer: 1 Influenza virus is transmitted through respiratory droplets. Herpes virus is transmitted by The key element in the question is the term ‘casual contact.’ This leads to the correct
acquired through casual contact with an infected direct contact and HIV through blood and body fluids. Cytomegalovirus is an opportunistic response of influenza.
individual? infection.
1.‐ Influenza virus
2.‐ Herpes virus
3.‐ Cytomegalovirus (CMV)
4.‐ Deficiency virus (HIV)

334 A female prostitute enters the clinic for treatment of Correct answer: 2 Epidemiological studies indicate chlamydia as the most prevalent sexually transmitted disease This question requires application of factual knowledge about the disease chlamydia.
a sexually transmitted disease. Given that this disease in the United States.
is the most prevalent in the United States, the nurse
can anticipate that the woman has which of the
following?
1.‐ Herpes
2.‐ Chlamydia
3.‐ Gonorrhea
4.‐ Syphilis

335 Endotoxins differ from exotoxins in that exotoxins Correct answer: 3 Endotoxins are often not destroyed even by autoclaving. Options 1, 2, and 4 are descriptions This is a question that requires application of facts about endotoxins and exotoxins.
have which of the following characteristics? of endotoxins.

1.‐ Are composed of lipopolysaccharides


2.‐ Are found only in Gram‐negative bacteria
3.‐ Are easily destroyed by heat
4.‐ Are typically not very toxic

336 A client exhibiting symptoms of a rickettsia infection Correct answer: 3 Rickettsia are parasites of ticks, fleas, and lice. Influenza is an example of transmission by This is a question that requires application of knowledge about the transmission of
probably acquired it through: respiratory droplets, encephalitis is transmitted by mosquitos; lice and scabies are transmitted rickettsia.
by direct contact.
1.‐ Respiratory droplets.
2.‐ Mosquitos.
3.‐ Bites or feces of ticks, lice, or fleas.
4.‐ Direct skin contact.

337 Nurses should understand the chain of infection Correct answer: 2 Infection occurs in a predictable sequence requiring virulence, movement from a reservoir, Note that the question stem is asking about the ‘chain of infection’ which most closely
because it refers to: and entry into a susceptible host. relates to option 2, referring to transmission of disease.
1.‐ The linkages between various forms of microorganisms.
2.‐ The sequence required for transmission of disease.
3.‐ The clustering of bacteria in a specific pattern.
4.‐ Increasing virulence patterns among species of microorganisms.

338 An infection characterized by bacterial resistance that Correct answer: 1 The capsule contributes to the invasiveness of pathogenic bacteria. Encapsulated bacteria are This item requires application of knowledge of bacterial resistance. Option 4 can be
develops may be linked to the fact that the bacterial protected from phagocytosis unless coated with anticapsular antibody. eliminated because of use of the absolute ‘always’.
capsule:
1.‐ Contributes to the invasiveness of pathogenic bacteria.
2.‐ Plays a major role in adherence of bacteria.
3.‐ Does not protect the organism from phagocytosis.
4.‐ Is always composed of glutamic acid.
339 Which of the following client laboratory test results Correct answer: 4 Eosinophilia is present with allergies and infestation with parasites. Neutrophils are elevated White blood cells would be elevated, eliminating options 2 and 3.
would likely be elevated with nematode infestation? with acute infections and bacterial organisms. Options 2 and 3 are irrelevant.

1.‐ Neutrophils
2.‐ Liver enzymes
3.‐ Red blood cells
4.‐ Eosinophils

340 A nurse researcher is interested in the epidemiology Correct answer: 4 Epidemiology is the study of how various states of health are distributed in the population. Select the option with the word ‘population.’
of HIV. This means that the researcher is concerned
about which of the following?

1.‐ The causation of the disease.


2.‐ How the disease is transmitted.
3.‐ The most effective treatment regimens.
4.‐ The distribution of the disease in a given population.

341 A young male college student came to the clinic after Correct answer: 3 Herpes is a virus and is spread through direct contact. An antifungal would not be useful; Because the priority with this condition is prevention of transmission select the option
contracting genital herpes. Which of the following bedrest and temperature measurement are usually not necessary. that would accomplish this.
interventions would be most appropriate?

1.‐ Encourage him to maintain bedrest for several days


2.‐ Monitor temperature every 4 hours
3.‐ Instruct him to avoid sexual contact during acute phases of illness
4.‐ Encourage him to use antifungal agents regularly

342 Which of the following drugs would be most effective Correct answer: 3 Acyclovir is the antiviral drug of choice for treating herpesvirus. Penicillin products are used This item requires knowledge of the preferred treatment for herpes. Options 1 and 2 can
in treating genital herpes? for a wide variety of bacterial infections. Rifadin is used for TB and Virazole is an antiviral be eliminated as they are not used to treat viral disease.
agent.
1.‐ Penicillin (Bicillin)
2.‐ Rifampin (Rifadin)
3.‐ Acyclovir (Zovirax)
4.‐ Ribavirin (Virazole)

343 A young adult male relates to the nurse that he has Correct answer: 2 Neutrophil counts are often decreased in viral infections and elevated in bacterial infections. This item requires application of factual knowledge that viral infection often lowers
recently experienced signs and symptoms of infection. Neutropenia can occur because of chemotherapy and immunosuppression. With recovery, his neutrophil count.
His neutrophil count is lower than normal. The nurse neutrophil count should be returning to normal.
concludes that most likely he:
1.‐ Has a bacterial infection.
2.‐ Has a viral infection.
3.‐ Has an immune deficiency disorder.
4.‐ Is recovering from the illness.

344 A young girl presents with fever and abdominal Correct answer: 3 Abdominal distention is caused from infestation of worms. Blood in sputum often results Select the response which best corresponds with the symptoms.
distention. Her mother states that she has also from migration of worms through alveoli. Mycoplasma pneumonia has similar side effects as
"coughed up blood" in recent days. Which of the bacterial pneumonia (cough, fatigue, rales, and temperature). Spirochetes cause fever, neck
following is compatible with these symptoms? stiffness, and lymphadenopathy; rickettsial infections cause headaches, nausea, vomiting, and
muscle aches.
1.‐ Mycoplasma pneumonia
2.‐ Rickettsial infection
3.‐ Infection with nematodes
4.‐ Infection with spirochetes

345 Which of the following is correct concerning prion Correct answer: 2 Prions are associated with degenerative encephalopathies. While similar to viruses, they lack This item is a knowledge question, requiring that the student recall a fact.
disease? nucleic acid and lesions are usually limited to a single organ.
1.‐ Prions are opportunistic organisms frequently seen in clients with HIV.
2.‐ Prions have been linked to chronic degenerative disorders of the central nervous system such as Creutzfeldt‐Jakob disease.
3.‐ Lesions are usually distributed throughout the body.
4.‐ Prions are similar to viruses in their nucleic acid structure.

346 A client has a WBC of 15,000, of which 60 percent are Correct answer: 3 With bacterial infection there is an increased need for neutrophils. When the percentage of Note that the WBC remains the same, making options 1 and 3 incorrect while bands and
segmented neutrophils and 3 percent are bands. An immature neutrophils (bands) increases at a greater rate than mature neutrophils (segs), it is segs are increased, making option 4 incorrect.
antibiotic is prescribed. Three days later the WBC an indication that the infection is severe or prolonged. This is often referred to as a shift to the
remains at 15,000; 62 percent segs and 10 percent left.
bands. This most likely indicates:
1.‐ The infection is resolving.
2.‐ The client is immunocompromised.
3.‐ The infection is severe or prolonged and not responding to antimicrobial agents.
4.‐ There is a shift to the right in the differential.

347 Which of the following is incorrect concerning Lyme Correct answer: 1 Lyme disease is a spirochetal infection. Examples of rickettsial infections are Typhus and Note the word ‘incorrect’ in the question stem is asking for identification of the only
disease? Rocky Mountain Spotted Fever. option that is not related to Lyme disease.
1.‐ The disease is caused by rickettsial pathogens.
2.‐ Humoral and cell‐mediated responses by the body will not generally be sufficient to eliminate the disease.
3.‐ The disease is transmitted through ticks.
4.‐ If untreated it may lead to complications of arthritis and destruction of joints.

348 Which of the following substances is useful in Correct answer: 3 Cytokines serve as mediators of inflammation, while leukocidin, adherins, and coagulase This item requires application of factual knowledge.
inhibiting viral and bacterial growth in the body? enhance bacterial resistance to body defenses.
1.‐ Leukocidin
2.‐ Coagulase
3.‐ Cytokines
4.‐ Adherins

349 One structural difference in the cell wall of fungi that Correct answer: 2 Most antifungal agents act by inhibiting biosynthesis of ergosterol. Select the only response that speaks to the composition of the cell wall.
makes them susceptible to antifungal agents is that:

1.‐ Spore formation weakens the cell wall.


2.‐ The cell wall is composed of ergosterol rather than cholesterol.
3.‐ All fungi are composed of single cells.
4.‐ Most fungi do not have a cell wall.

350 Which of the following has been associated with Correct answer: 1 A surface peptide found in chlamydia is similar to one in heart myosin and may trigger T‐cells This item requires application of learned information about chlamydia.
chlamydial infection? to attack both chlamydia and the heart.
1.‐ Myocarditis
2.‐ Rheumatic fever
3.‐ Arthritis
4.‐ Liver abscess
351 The drug of choice for systemic fungal infections is: Correct answer: 4 Fungizone and Mycostatin are both antifungal agents. Mycostatin is most commonly used for Select the antifungal agent that can be given intravenously.
topical application, while Fungizone is used systemically.
1.‐ Nystatin (Mycostatin).
2.‐ Penicillin (Penn‐VK).
3.‐ Isoniazid (INH).
4.‐ Amphotericin B (Fungizone).

352 The drugs of choice for treating chlamydia include: Correct answer: 2 Chlamydia is a bacteria and responds to bacteriostatic agents that interfere with protein Options 1 and 3 should be immediately eliminated, as they are not effective in treatment
synthesis. of bacterial infections such as chlamydia. Select the drug that is the drug of choice.

1.‐ Antifungal agents.


2.‐ Macrolides such as erythromycin (E‐Mycin).
3.‐ Interferon (Roferon‐A).
4.‐ Vancomycin (Vancocin).

353 Which of the following is generally associated with Correct answer: 4 Neuropathies are usually associated with spirochetal infections along with lymphadenopathy, This item requires application of learned factual information.
spirochetal infections? fever, and stiff neck. Skin rash is associated with rickettsial infections; an unproductive cough
and fever could be many infections including protozoal. Toxic shock is usually associated with
staphylococcal infections.
1.‐ Skin rash
2.‐ Unproductive cough and fever
3.‐ Toxic shock
4.‐ Neuropathies

354 Pneumocystis carinii pneumonia is caused by: Correct answer: 3 Pneumocystis pneumonia is a protozoal infection that often affects immunocompromised Eliminate options 1, 2, and 4 as this is a protozoal infection.
clients with human immunodeficiency (HIV). It is characterized by a dry, unproductive cough
and results from aggregation of parasites and cellular debris.

1.‐ Viral infection.


2.‐ Bacterial infection.
3.‐ Protozoal infection.
4.‐ Rickettsial infection.

355 Which of the following is correct concerning Hepatitis Correct answer: 1 Administration of immune globulins can provide passive short‐term immunity to the disease if Recognize that options 2, 3, and 4 can be eliminated as they describe hepatitis B.
A? administered within 7 days of exposure. Hepatitis A is transmitted through food feces and is
usually not chronic.
1.‐ Treatment with immune globulins can prevent disease if exposed.
2.‐ It is transmitted through blood and body fluids.
3.‐ Jaundice is rare in this form of hepatitis.
4.‐ It usually becomes a chronic disease condition.

356 The Epstein‐Barr virus is most often associated with: Correct answer: 2 Epstein‐Barr virus is the causative agent for mononucleosis. The question is asking about a virus. Options 3 and 4 are bacterial and can be eliminated.

1.‐ Hepatitis B.
2.‐ Mononucleosis.
3.‐ Streptococcal infections.
4.‐ Tuberculosis.

357 Which of the following is correct concerning varicella Correct answer: 3 Varicella zoster results from reactivation of a latent virus in sensory cells of the dorsal root The key characteristic of varicella zoster is pain along a dermatome.
zoster? ganglion. Activation tends to follow the nerve path.
1.‐ It is caused by the influenza virus.
2.‐ It is usually asymptomatic.
3.‐ Painful lesions appear to follow the path of a dermatome.
4.‐ Interferon stimulates viral replication.

358 Which of the following viral infections has been Correct answer: 4 Influenza has been associated with Guillian‐Barré syndrome, which causes progressive Eliminate options 1, 2, and 3 which have no known association with Guillian‐Barré.
associated with Guillian‐Barré syndrome? paralysis.
1.‐ Human immunodeficiency virus (HIV)
2.‐ Epstein‐Barr
3.‐ Cytomegalovirus (CMV)
4.‐ Influenza

359 Which of the following is true of botulism? Correct answer: 2 The bacteria produces a neurotoxin that blocks release of acetylcholine at the neuromuscular The key term in the correct response is ‘neurotoxin.’ The neurotoxin produces the
junction. characteristic symptom of stiff jaw.
1.‐ It is caused by a staphylococcus bacteria.
2.‐ Neurotoxins produced by the bacteria cause paralysis.
3.‐ It is a resident bacterium in the intestinal flora.
4.‐ It produces a toxin that leads to necrosis of epithelial cells.

360 Which of the following is used to treat Klebsiella Correct answer: 1 Klebsiella is a Gram‐negative rod. Aminoglycosides are used effectively against Gram‐negative Eliminate option 3 as Flagyl is an anti‐fungal. Recall that penicillins and sulfonamides are
pneumoniae? bacteria by binding to ribosome and preventing protein synthesis. not used to treat Klebsiella.
1.‐ Aminoglycosides
2.‐ Penicillins
3.‐ Metronidazole (Flagyl)
4.‐ Sulfonamides

361 Which of the following is true of Helicobacter pylori Correct answer: 2 The enzyme urease produced by the bacteria raises the pH of the stomach, allowing the Note the term ‘erosion of gastric mucosa’ in option 2. Recall that H. Pylori is often
bacteria? bacteria to survive. Urea in the stomach is converted to ammonia, which is cytotoxic to gastric associated with ulcers of the GI tract.
mucosa.
1.‐ It protects the stomach from harmful acids.
2.‐ It causes erosion of gastric mucosa.
3.‐ It is an acid‐fast bacillus.
4.‐ It is transmitted through contaminated water.

362 Which of the following is associated with Correct answer: 4 Scarlet fever, rheumatic fever, and glomerulonephritis can all result from streptococcal Use process of elimination to eliminate those options not associated with streptococcal
streptococcal infection? bacteria. Tuberculosis is caused by M. tuberculosis; PID by Neisseria gonorrhoeae; and toxic infection.
shock by staphylococcus.
1.‐ Tuberculosis
2.‐ Pelvic inflammatory disease
3.‐ Toxic shock syndrome
4.‐ Arthritis

363 Which of the following is true of purified protein Correct answer: 3 While the PPD skin test can detect previous exposure to the tuberculosis bacillus, it does not This requires application of knowledge that PPD is used to screen for TB exposure.
derivative (PPD) and tuberculosis? necessarily establish the presence of active infection. Eliminate the options that do not address detection of exposure.
1.‐ It is used to detect current infection.
2.‐ It is used to prevent tuberculosis.
3.‐ It is used to detect previous exposure of mycobacterial infection.
4.‐ It is used to neutralize toxins produced by the tuberculosis bacillus.
364 One of the primary concerns with treatment for Correct answer: 1 Treatment for tuberculosis often requires months of antimicrobial therapy and compliance Eliminate options 2 and 4 as they are not true. Select option 1 as the best option as
tuberculosis is: must be encouraged. compliance with long‐term treatment is often a concern.
1.‐ Compliance with drug therapy.
2.‐ The lack of antibacterial agents that are effective against tuberculosis.
3.‐ Bone marrow suppression caused by drug therapy.
4.‐ Hospitalization and isolation for long periods of time.

365 The tetracyclines are identical in their overall Correct answer: 4 Tetracyclines and chloramphenicol are identical in mechanism of action and organisms Select the option most like tetracyclines in action.
mechanism of action and are often used similarly to: against which they are effective.

1.‐ Sulfonamides.
2.‐ Penicillins.
3.‐ Isoniazid (INH).
4.‐ Chloramphenicol (Chloromycetin).

366 The most serious disadvantage of clinical use of Correct answer: 4 Chloramphenicol is reserved for serious infection because it may cause bone marrow Look for bone marrow suppression when side effects or disadvantages of chloramphenicol
chloramphenicol (Chloromycetin) is the: suppression. are requested.
1.‐ Nausea and vomiting it causes.
2.‐ Slow and incomplete absorption from the gastrointestinal tract.
3.‐ Ineffectiveness when given by mouth.
4.‐ Possibility of bone marrow suppression.

367 The mother of a 12‐year‐old who has been diagnosed Correct answer: 2 Mycoplasma pneumonia is an atypical form of pneumonia, occurs often in children, and is Recall that ‘walking pneumonia’ is the common term for mycoplasma pneumonia.
with mycoplasma pneumonia wants to know what transmitted by droplets. Signs and symptoms are similar to bacterial pneumonia and the
illness she has. The best explanation would be that: virulence is no worse.

1.‐ It is more severe than pneumonia caused by other bacteria.


2.‐ It is described as walking pneumonia.
3.‐ It is characterized by copious sputum production.
4.‐ It is more difficult to treat.

368 A middle‐aged woman presents with signs and Correct answer: 3 The proximity of the anus to the urethra in female clients increases the risk for infection from Recall that the most common causative agent of UTI in females is E. coli because of the
symptoms of urinary tract infection. The most likely bacteria normally found in the colon, such as E. coli. Prion disease is similar to viruses. proximity of the urethra to the rectum, where E. coli is normally found.
causative agent is: Staphylococcus is responsible for many infections such as sepsis, cellulites, and toxic shock
syndrome; Treponema causes spirochetal infections.
1.‐ <i>Staphylococcus aureus</i>.
2.‐ <i>Treponema pallidum</i>.
3.‐ <i>Escherichia coli</i>.
4.‐ <i>Prions</i>.

369 Helicobacter pylori is able to survive in the acidic Correct answer: 1 Urease breaks down urea in the stomach, producing ammonia that increases the pH to allow Select the option that would influence pH in the stomach.
environment of the stomach because of the enzyme: survival of Helicobacter pylori.

1.‐ Urease.
2.‐ Protease.
3.‐ Lipase.
4.‐ Alkaline phosphatase.
370 A client with an infection caused by amebiasis will Correct answer: 4 Amebic infection can be carried via the blood to other organs, with the liver as the most Recall that the liver is structurally close to the GI tract.
most likely have gastrointestinal symptoms. Another common site, causing liver abscess.
site that could be affected is the:
1.‐ Lung.
2.‐ Brain.
3.‐ Kidney.
4.‐ Liver.

371 Which of the following properties would allow Correct answer: 4 Both forms of hepatitis may result in jaundice, fever, and elevated liver enzymes, but the Use the process of elimination to eliminate options 1, 2, and 3 as they occur with both
distinction between Hepatitis A and Hepatitis B incubation period for Hepatitis B is greater than that for Hepatitis A. Hepatitis A and B.
infection?
1.‐ Jaundice
2.‐ Fever
3.‐ Alanine aminotransferase (ALT) level
4.‐ Incubation period

372 A woman with recurring gonorrhea or chlamydia Correct answer: 2 Both gonorrhea and chlamydia may be asymptomatic, with the bacteria invading Consider that both of these are STDs and affect the organs of reproduction.
should be concerned about which of the following? reproductive organs prior to detection.

1.‐ Urethritis
2.‐ Pelvic inflammatory disease
3.‐ Pyelonephritis
4.‐ Arthritis

373 A child with suspected toxoplasmosis is experiencing Correct answer: 3 Toxoplasmosis is spread through contact with cat feces. Ticks carry the threat of Lyme disease Recall that toxoplasmosis is associated with contact with cat feces. Select the only option
signs and symptoms of fatigue, fever, and or rickettsial infections. Protozoal infections can be found all over the world. that refers to cats.
lymphadenitis. Which of the following would be most
important in obtaining a nursing history?

1.‐ Determine any recent contact with ticks or fleas


2.‐ Assess for prior streptococcal infections
3.‐ Inquire about the presence of cats in the home
4.‐ Inquire about recent trips out of the country

374 Which of the following clients would most likely Correct answer: 3 M. avium‐intracellulare is a mycobacterial infection (an opportunistic infection) that has been Select the option that reflects the immunocompromised individual.
acquire Mycobacterium avium‐intracellulare? identified in those who are HIV‐positive or have AIDS.
1.‐ Child
2.‐ Homeless adult
3.‐ HIV‐positive individual
4.‐ Teenager

375 A client has recently experienced fever, chills, Correct answer: 2 The convalescent stage occurs when the infection is contained and symptoms are diminished. Use the definition of the term ‘convalescent’ (getting better) to identify it as the correct
headache, and myalgia from influenza. The symptoms The acute stage is when all symptoms are present; prodromal is the presence of initial response.
have diminished; however, fatigue is still present. This symptoms; resolution is elimination of an organism.
is an example of what stage of the infectious process?

1.‐ Resolution
2.‐ Convalescent
3.‐ Acute
4.‐ Prodromal
376 Mycoplasmal infections are usually resistant to Correct answer: 1 Mycoplasma have no cell wall membrane and therefore are not sensitive to penicillin, which This item requires knowledge of mycoplasma and penicillins. Eliminate options 2, 3, and 4
antibiotics such as penicillin because: works by interfering with cell wall synthesis. as being incorrect.
1.‐ Mycoplasma have no cell wall membrane.
2.‐ Most mycoplasma have become methicillin‐resistant.
3.‐ Mycoplasma does not replicate using DNA.
4.‐ Penicillin cannot penetrate the mycoplasma capsule.

377 Which of the following is correct concerning viral Correct answer: 2 The viral replication cycle can range from minutes to days. Some viruses remain latent for Eliminate options 1, 3, and 4 as being untrue regarding virus.
infections? long periods of time without replicating.
1.‐ Most viral infections have specific antiviral agents that kill the virus.
2.‐ Viruses may remain in a latent, non‐replicating form for months or even years
3.‐ Lymphocytes are usually decreased
4.‐ Viruses have strands of both DNA and RNA

378 Which one of the following suggestions by the nurse Correct answer: 2 Zinc deficiency is associated with taste changes; therefore, supplementation may benefit a The core issue of the question is knowledge of measures to minimize taste alterations in a
would be most helpful to a human immunodeficiency client experiencing altered taste perception. Drinking salty broth and fluids will not help with client with HIV infection. Use nursing knowledge and the process of elimination to make a
virus (HIV) positive client who has altered taste taste changes but may help restore electrolyte balance in clients experiencing diarrhea. Dairy selection.
perception? products, fish, and poultry are better food choices than meat when taste is altered.
Substitution of plastic utensils for metal ones is suggested to decrease possibility of taste
perception of “metal.”
1.‐ Drink plenty of salty broths and other fluids to stimulate taste buds.
2.‐ Try zinc supplementation to improve taste perception.
3.‐ Increase intake of meat to at least one serving per day.
4.‐ Avoid using plastic eating utensils.

379 Which of the following suggestions would the nurse Correct answer: 3 Small, frequent meals help lessen nausea because they require less work of digestion and do The core issue of the question is the ability to provide teaching to minimize nausea in a
give to a client with human immunodeficiency virus not overwhelm the client with food odors from a lengthy meal. High‐fat foods are more client with HIV. Use nursing knowledge and the process of elimination to make a selection.
(HIV) infection to best alleviate nausea? difficult to digest and may distend the stomach. Lying down after eating can encourage reflux.
Drinking liquids can give a sensation of fullness. High‐fat foods, reclining after meals, and
drinking large quantities of liquid all increase the risk of nausea and vomiting.

1.‐ Drink liquids with meals.


2.‐ Eat high‐fat foods.
3.‐ Eat small, frequent meals.
4.‐ Lie down after eating.

380 To enhance meeting the psychosocial needs of a Correct answer: 4 It is important to assess the psychosocial needs of a client on transmission‐based precautions The critical word in the question is psychosocial. With this word in mind, focus on the
client on transmission‐based precautions, the nurse and to intervene to provide sensory stimulation for the client. Isolation procedures can cause intervention that best meets nonphysical needs of the client. Use nursing knowledge and
should place highest priority on which of the clients to become depressed and withdrawn and to sleep excessively. Although it is important the process of elimination to make a selection.
following? to maintain isolation precautions as ordered, attention must be given to include the client’s
psychosocial needs as part of the plan of care. Limiting contact time may be indicated for
infection control, but it does not provide psychosocial support.

1.‐ Letting the client sleep to build up stamina


2.‐ Maintaining strict precautions when entering and leaving the room so that the client feels he or she is getting the best care
3.‐ Providing client care within a limited time frame to maintain isolation and keep client safe
4.‐ Providing the client with diversional activities to enhance sensory input
381 A client diagnosed with scleroderma is complaining of Correct answer: 2 Clients who have scleroderma usually have Raynaud’s phenomenon. Raynaud’s can be The core issue of the question is recognition of Raynaud’s syndrome and the ability to
painful fingers that change colors (pale to red) when triggered by temperature changes, and prolonged water contact may cause activation. Use of select an appropriate intervention for that problem. Use nursing knowledge and the
washing dishes. Which suggestion by the nurse might gloves when washing dishes may prevent temperature changes yet still allow the client to process of elimination to make a selection.
help the client with this symptom? participate in ADLs. Hotter water may increase the risk of scalding and so is not suggested.
Physical therapy and H&lt;sub&gt;2&lt;/sub&gt; receptor blockers are indicated for treatment
of esophageal problems associated with scleroderma.

1.‐ Increase the water temperature.


2.‐ Use gloves during dishwashing.
3.‐ Start physical therapy to increase blood flow to the hands.
4.‐ Take over‐the‐counter H2 receptor antagonist medications.

382 The white blood cell (WBC) count of a client with Correct answer: 4 All identified nursing diagnoses are of concern for a client with SLE. However, the results of The core issue of the question is the ability to analyze WBC differential count data to
systemic lupus erythematosus (SLE) shows a shift to the laboratory test demonstrate an increased risk for infection that is due to the disease determine risk of infection. Use nursing knowledge and the process of elimination to make
the left. Which nursing diagnosis reflects the highest process and/or possible treatment measures such as steroids and immunosuppressive agents. a selection.
priority for this client? A shift to the left in a WBC differential indicates an increased number of immature cells,
suggesting infection.
1.‐ Ineffective health maintenance
2.‐ Impaired skin integrity
3.‐ Ineffective individual coping
4.‐ Ineffective protection

383 A client is to start taking prednisone for the Correct answer: 2 Steroid therapy is usually done as part of a tapered‐dose treatment plan. It is important to The core issue of the question is knowledge of client teaching related to steroid therapy.
treatment of rheumatoid arthritis (RA). Which client take this medication at the same time each day and to become aware of tapered‐dose effect. Use nursing knowledge and the process of elimination to make a selection.
statement indicates that medication teaching was Steroids are usually taken with foods to minimize GI upset. Steroids cause fluid retention, and
successful? therefore sodium intake may be restricted. Steroids also increase blood glucose, so insulin
therapy dosages may have to be adjusted.

1.‐ “I will take the medication on an empty stomach to maximize absorption.”


2.‐ “I will take the specific dose ordered at the same time every day.”
3.‐ “I will not have to limit my sodium intake.”
4.‐ “I will not have to adjust my insulin regimen.”

384 The nurse assesses the client with rheumatoid Correct answer: 1 Swan‐neck deformity occurs at the proximal interphalangeal (PIP) joint and ulnar deviation The core issue of the question is identification of signs and symptoms of RA. Use nursing
arthritis for which of the following characteristic joint occurs as a result of joint destruction with disease progression. Heberden’s and Bouchard’s knowledge and the process of elimination to make a selection.
changes? nodes are commonly found in clients with osteoarthritis. Tophi (firm, moveable nodules) are
associated with gout. Charcot’s joint is considered a neuropathic disorder that falls under the
broader category of rheumatism. It is not specific to RA and is more likely to be seen as a
complication in clients with diabetes.
1.‐ Swan‐neck deformity and ulnar deviation
2.‐ Heberden’s and Bouchard’s nodes
3.‐ Tophi
4.‐ Charcot’s joints

385 In establishing a plan of care to manage pain for a Correct answer: 3 Heat and cold applications can provide analgesia and relieve muscle spasms. The individual The core issue of the question is knowledge of measures that relieve the symptoms of RA.
client with rheumatoid arthritis, what intervention client will have to determine whether heat, cold, or alternation of both is most effective. Pain Use nursing knowledge and the process of elimination to make a selection.
would the nurse use to increase the client’s mobility? medication should be taken on a regular schedule if the client has chronic pain so that the pain
threshold can be raised and pain relief maintained at a constant level. Exercising in the
presence of pain may only further exacerbate pain. Flexing of muscle groups is not related to
effective pain control.
1.‐ Have the client work through pain by continuing exercise in order to establish endurance.
2.‐ Have the client use pain medication only when pain is present.
3.‐ Teach the client that both heat and cold applications may help to relieve pain.
4.‐ Teach the client to flex muscle groups when pain is felt in an extremity.

386 Which of the following information will the nurse use Correct answer: 3 Methotrexate treatment takes several weeks to effect relief. Once relief is obtained, the dose The core issue of the question is knowledge of management principles for RA. Use nursing
when explaining therapeutic measures to a client is adjusted to achieve maximum response at the lowest dose. If the drug is discontinued, then knowledge and the process of elimination to make a selection.
taking methotrexate (Rheumatrex) for rheumatoid symptoms of the disease do return.
arthritis?
1.‐ Relief of symptoms will be assessed for within 1 week of starting medication.
2.‐ Fluids should be restricted to prevent possible edema formation.
3.‐ Drug doses will be adjusted for optimum effect at lowest dose once relief has been established.
4.‐ Six months of therapy will be adequate to stop the disease process from progressing.

387 The nurse looks for results of which laboratory Correct answer: 2 CD&lt;sub&gt;4&lt;/sub&gt; cells are indicative of a client’s HIV status. As the disease The core issue of the question is knowledge of which laboratory measure will provide
measurement that provides a reliable indicator of progresses, the T‐helper cells decrease in number and lose their ability to function effectively, information about the status of the immune system of a client with HIV. Use nursing
lymphocyte status in a client with HIV infection? leading to an overaggressive immune response. B lymphocytes indicate the status of humoral knowledge and the process of elimination to make a selection.
immunity and are not directly associated with HIV infection. NK cells and T‐cytotoxic cells are
not directly related to HIV infection and as such are not considered to be reliable indicators of
HIV status.
1.‐ B lymphocytes
2.‐ T‐helper cells (CD4)
3.‐ Natural killer cells (NK)
4.‐ T‐cytotoxic cells

388 The nurse who is providing care to a group of clients Correct answer: 4 Transfusion and Goodpasture’s are examples of type II cytotoxic hypersensitivity reactions The core issue of the question is the ability to associate various types of hypersensitivity
concludes that the client with which of the following and are involved with the activation of complement. Lupus is an example of a type III reactions with their etiologies. Use nursing knowledge and the process of elimination to
problems exhibits a type III immune‐ hypersensitivity reaction, which involves IgG and IgM with the activation of complement. make a selection.
complex–mediated hypersensitivity reaction?

1.‐ Transfusion reaction


2.‐ Goodpasture’s syndrome
3.‐ Transplant rejection
4.‐ Systemic lupus erythematosus

389 A male client who has acquired immunodeficiency Correct answer: 2 While Megace is used as a palliative treatment for clients with advanced cancers, this is not The core issue of the question is the purpose of oral progesterone in a client with AIDS.
syndrome (AIDS) asks why oral progesterone (Megace) the rationale for its use with AIDS. In AIDS clients, it provides appetite enhancement. Side Use nursing knowledge about anorexia as a symptom of AIDS and the process of elimination
is being prescribed for treatment. What is the nurse’s effects of Megace can include nausea and constipation. to make a selection.
best response?

1.‐ “Megace is used to treat the nausea associated with this infection.”
2.‐ “Megace is used as an appetite stimulant to boost nutritional support.”
3.‐ “Megace provides symptomatic relief of constipation.”
4.‐ “Megace is used as an antineoplastic agent for palliative treatment.”

390 The nurse would assess for which of the following Correct answer: 1 Hyponatremia is a common finding in clients with AIDS. The incidence of opportunistic The core issue of the question is identification of an electrolyte disturbance that is more
electrolyte imbalances as a common finding in a client infections may contribute to this decrease in sodium. Hypernatremia, hyperkalemia, and common to clients with AIDS. Use nursing knowledge and the process of elimination to
with AIDS? hypocalcemia are not usually seen in clients who have AIDS. make a selection.
1.‐ Hyponatremia
2.‐ Hypernatremia
3.‐ Hyperkalemia
4.‐ Hypocalcemia
391 Which of the following assessments by the nurse Correct answer: 2 Prolonged morning stiffness is associated with RA. Occasional use of NSAIDs is not by itself a The core issue of the question is the ability to identify symptoms that are possibly
warrants further investigation to determine if the direct link to the development of RA. Complaints of pain with movement are more likely to be associated with RA. Use nursing knowledge and the process of elimination to make a
client has rheumatoid arthritis (RA)? associated with degenerative joint disease (osteoarthritis). selection.

1.‐ Negative family history


2.‐ Complaints of prolonged morning stiffness lasting for 1 hour
3.‐ Occasional use of NSAIDs for aches and pains
4.‐ Complaints of pain with movement

392 The nurse teaches a client that which of the following Correct answer: 1 Pregnancy can be associated with an exacerbation because of increased estrogen levels. The core issue of the question is risk factors and triggers for SLE. Use nursing knowledge
factors might increase risk of developing an Hypotension, fever, and GI upset do not exacerbate SLE. and the process of elimination to make a selection.
exacerbation of systemic lupus erythematosus (SLE)?

1.‐ Pregnancy
2.‐ Hypotension
3.‐ Fever
4.‐ GI upset

393 A client will undergo scratch tests for allergies. In Correct answer: 3 A scratch test tests many allergens at once. It is of low sensitivity, but many allergens can be The core issue of the question is identification of appropriate concepts to teach a client
teaching the client about the planned tests, the nurse tested at once, and the results can be obtained in 30 minutes. about scratch tests for allergies. Use nursing knowledge and the process of elimination to
should include which of the following information? make a selection.

1.‐ This test allows us to rule out one or two specific antigens.
2.‐ The scratch test is the most sensitive allergy test.
3.‐ Results can be obtained in 30 minutes.
4.‐ The scratch test involves drawing a small amount of blood from the client.

394 The nurse would expect which of the following Correct answer: 4 Diseases with HLA associations have poorly understood etiologies, are usually chronic or The core issue of the question is knowledge of diseases associated with the HLA antigen.
findings in a client with an immunologic disorder subacute in nature, and have limited effect on reproductive capacity. Use nursing knowledge and the process of elimination to make a selection.
associated with an HLA antigen?
1.‐ Acute course
2.‐ Frequent effects on reproductive capacity
3.‐ Genetic determination
4.‐ Chronic and possibly subacute course

395 A client presents with dyspnea, pruritis, and localized Correct answer: 3 The priority intervention is to maintain a patent airway in a potential anaphylactic reaction. Remember in emergency or near‐emergency situations to use the ABCs (airway, breathing,
swelling of the forearm after being stung by a bee. Therefore, the nurse should assess for swelling of the tongue and stridor, which could indicate and circulation) to plan priorities of care. Use the process of elimination to make a
What is the priority intervention? impending respiratory obstruction. The other interventions are supportive measures that can selection.
be used during an allergic response.
1.‐ Remove the stinger from the client’s arm
2.‐ Keep the client warm with soft blankets
3.‐ Check the tongue for swelling and listen for stridor
4.‐ Place client in the Trendelenburg position

396 Medication instruction for the client with rheumatoid Correct answer: 1 Gold salts may cause anaphylaxis. Sulfasalazine may cause nausea and vomiting, but fluids The core issue of the question is knowledge of appropriate client teaching related to
arthritis (RA) should include which the following should be encouraged (option 2). Acetaminophen does not provide the same anti‐ medications used to treat rheumatoid arthritis. Use nursing knowledge and the process of
teaching points? inflammatory effects as ASA and NSAIDs (option 3). Penicillamine cannot be used during elimination to make a selection.
pregnancy (option 4).
1.‐ Injection of gold salts requires monitoring for anaphylactic reactions every half‐hour.
2.‐ Treatment with sulfasalazine requires fluid restriction to avoid nausea and vomiting.
3.‐ NSAIDs, acetaminophen, and aspirin may be used interchangeably to decrease inflammation associated with RA.
4.‐ Penicillamine may be safely used during pregnancy.
397 The nurse writing a care plan determines that which Correct answer: 1 Skin manifestations are a common finding in clients with scleroderma and therefore require The core issue of the question is knowledge that scleroderma is primarily a skin disorder in
of the following is a priority nursing diagnosis early in preventative and supportive nursing care as the priority. As the disease progresses, many cases and that therefore the primary nursing diagnosis needs to address loss of skin
the care of a client with scleroderma? dermatologic effects may lead to disturbances in body image. In addition, with disease as a protective barrier. Use nursing knowledge and the process of elimination to make a
progression, there may be an impact on respiratory and musculoskeletal function, leading to selection.
activity intolerance. Similarly, hopelessness can develop with new and worsening symptoms.
Therefore, the nursing diagnoses in options 2, 3, and 4 are of lesser priority in the early phase
of the disease process.
1.‐ Impaired skin integrity
2.‐ Disturbed body image
3.‐ Activity intolerance
4.‐ Hopelessness

398 An infant is admitted to the pediatric unit with a Correct answer: 4 Altered temperature, jaundice, and respiratory distress are all symptoms of sepsis in infants. Use the ABCs and the process of elimination to make a selection. Airway and breathing
diagnosis of sepsis. The nurse is completing a nursing Respiratory function is the highest priority because without an adequate airway and breathing, typically take priority in situations of high acuity, such as sepsis.
assessment. The priority assessment for this infant the client cannot maintain life.
would be
1.‐ Skin integrity.
2.‐ Temperature.
3.‐ Jaundice.
4.‐ Respiratory function.

399 The nurse is caring for a pediatric client with acquired Correct answer: 1 Body fluid‐contaminated liquids may contain the human immunodeficiency virus (HIV) and The core issue of the question is the ability to identify a breach in standard precautions.
immunodeficiency syndrome (AIDS). Which activity by can be absorbed through the eye mucosa. The other activities do not expose the nurse to Use nursing knowledge about transmission of HIV via body fluids and the process of
the nurse should be reported to the employee health blood and/or body fluids of the client and therefore pose no risk of contracting HIV. elimination to make a selection.
department as an exposure for the nurse?

1.‐ While flushing out the used bedpan, fluid splashes in the nurse’s eyes.
2.‐ The nurse does not wear a mask while in the client’s room.
3.‐ During the bath, the nurse removes gloves when giving a backrub on intact skin.
4.‐ The nurse is stabbed with a sterile syringe to be used to draw up the client’s medications.

400 The pediatric nurse would suspect severe combined Correct answer: 1 The first infection often seen in these children is oral candidiasis (thrush). That symptom, The core issue of the question is the ability to identify signs and symptoms of SCID. Use
immunodeficiency disorder (SCID) when which of the along with the low WBC count, would be a warning symptom of SCID. A 2‐year‐old is unlikely nursing knowledge and the process of elimination to make a selection.
following children is admitted to the hospital nursing to have survived this long undiagnosed. ELISA tests evaluate HIV infection, and a TORCH titer is
unit? unrelated. A newborn is too young for symptoms to have manifested.

1.‐ A 2‐month‐old with thrush and low white blood cell counts
2.‐ A 2‐year‐old with history of recent repeated infections
3.‐ A newborn with positive TORCH titer
4.‐ A newborn admitted with positive ELISA test

401 A 5‐year‐old child is brought into the clinic after being Correct answer: 3 Maintaining an open airway is always the highest priority. With anaphylactic shock, the Use the ABCs—airway, breathing, and circulation to answer questions related to
stung by an insect. The child appears to be going into airway may constrict, mucous membranes swell, and air trapping occurs. The second priority anaphylaxis. Airway is always the first priority in life‐threatening situations.
anaphylactic shock. Which of the following nursing would be airway access, followed by renal assessment, and finally site care.
actions is of highest priority?
1.‐ Assess urinary output to determine renal perfusion
2.‐ Apply cold, wet compresses to the site
3.‐ Position the child’s head to maintain an open airway
4.‐ Establish intravenous access for medication delivery
402 A 12‐year‐old boy is hospitalized and diagnosed with Correct answer: 2 The family has stated multiple concerns, and demonstrating acceptance of the child is the The core issue of the question is the best action of the nurse to model acceptance of the
the recent development of human immunodeficiency best way to foster acceptance of the child and development of further coping skills. Prevention child and lead to enhanced coping skills by the family. Select the option that is the most
virus (HIV) infection secondary to factor transfusions of transmission, handwashing, and drug therapy are all important, but none of these global in nature because the family has multiple concerns, and use the process of
for hemophilia. The family is very concerned about individually targets the global concerns of the family. elimination to make a selection.
their ability to manage his care, risk of infection to
family members, and whether the child should remain
in the home. Which action by the nurse will best
promote family coping at this time?

1.‐ Explain to the family that the infection cannot be spread by casual contact.
2.‐ Demonstrate positive acceptance of the child with each contact.
3.‐ Explain that prophylactic drugs will prevent the virus from spreading.
4.‐ Show the family how to wash their hands properly.

403 A client must undergo skin testing for allergies. The Correct answer: 3 The client needs to discontinue use of antihistamines for 72 hours (3 days) prior to allergy The core issue of the question is knowledge of the time frame that antihistamine drugs
nurse determines during client history that the client testing to avoid false negative readings. need to be withheld so as not to interfere with the results of allergy testing. Use specific
takes an antihistamine to control symptoms. The nurse nursing knowledge to determine the correct answer.
explains that the client must discontinue use of the
antihistamine for _____ days before the skin testing in
order to avoid false negative results. Write in a
numerical answer.

404 A mother brings her child to the clinic complaining of Correct answer: 2 Koplik's spots are associated with measles (rubeola) and appear on the buccal mucosa two Knowledge of the presenting signs and symptoms and clinical manifestations of measles
malaise and low‐grade temperature. In reviewing the days before and after the onset of the rash. will aid in choosing the correct answer.
child's medical history, the nurse notes the child is
behind on immunizations. When the nurse assesses
the mouth of this child, Koplik's spots, reddish spots,
are seen on the buccal mucosa. The nurse suspects:

1.‐ Mumps.
2.‐ Measles (Rubeola).
3.‐ Chickenpox.
4.‐ Rubella.

405 A child is being seen in ambulatory clinic for vague Correct answer: 4 An elevated basophil count is associated with a chronic infection, inflammatory reactions, or Knowledge of the laboratory values and possible findings will help to choose the correct
symptoms. A CBC with differential is drawn. The nurse stress. It is not associated specifically with allergy, viral infection, or bacterial infection. answer.
notes that the basophil count on the CBC is elevated.
The nurse concludes that the problem is probably:

1.‐ Allergic in nature.


2.‐ Viral‐based.
3.‐ Bacterial in nature.
4.‐ A chronic condition.

406 A 4‐year‐old child is having scratch tests for allergies. Correct answer: 3 A scratch test tests many allergens at once. It is of low sensitivity, but many allergens can be Note that the term “scratch” test should give some information necessary to respond to
In teaching the family about the planned tests, the tested at once and the results can be obtained in 30 minutes. this question.
nurse should include the information that:

1.‐ This test allows us to rule out one or two specific antigens.
2.‐ The scratch test is the most sensitive allergy test.
3.‐ Results can be obtained in 30 minutes.
4.‐ The scratch test involves drawing a small amount of blood from the client.

407 A 2‐year‐old child hospitalized for a fractured femur Correct answer: 1 Caladryl will reduce itching and discomfort and therefore diminish scratching and skin Determine which nursing activities will help the affected child, eliminating options which
breaks out with chickenpox. The physician has written breakdown. Acetylsalicylic acid should not be given to young children with a viral disease are not beneficial to the child. Then determine which activity will decrease scratching.
the following orders: Which intervention will best because of the relationship to Reye's syndrome. Immunizing the sibling and isolation will have
prevent secondary skin infections? no effect on skin eruptions.

1.‐ Apply Caladryl lotion to lesions prn


2.‐ Acetaminophen (Tylenol) 320 mg orally every six hours prn
3.‐ Immunize all siblings
4.‐ Institute airborne and contact isolation

408 A mother known to be infected with the human Correct answer: 1 The child will need to be tested at approximate three‐month intervals until the child is 18 Be aware that tests for HIV are repeated over time.
immunodeficiency virus (HIV) gives birth to a healthy‐ months to 2 years. CD4+ counts are used to assess a young child's immune status and risk for
appearing male infant. Which plan is best to follow up disease progressions. The p24 antigen test needs to be repeated if positive. The ELISA is used
on the infant's HIV status? with children over 18 months.
1.‐ CD4+ counts every three months until 2‐years‐old
2.‐ p24 antigen test one time
3.‐ White blood cell counts every four weeks
4.‐ ELISA at 2 months of age.

409 A toddler is being discharged from the hospital Correct answer: 4, 5 Carpet, bedding, fabrics, pets, dust, and cigarette smoking can cause allergic reactions. Option Remember that anything that would hold dust should be removed from the child’s room.
diagnosed with allergies. The child is on corticosteroids 1 can be eliminated as the use of sterile water is unnecessary.
and prophylactic antibiotics. The nurse will discuss
environmental control of the home and include which
of the following suggestions: (Select all that apply.)

1.‐ Cleaning the baby's room with sterile water.


2.‐ Covering the wood floor with carpeting for easier cleaning.
3.‐ Not allowing the dog outside because it will pick up pollens in its coat.
4.‐ Insisting that no one smoke in the house.
5.‐ Storing the out of season clothes in another room.

410 A young infant is diagnosed with severe combined Correct answer: 1 Even with aggressive treatment, prognosis is poor. Current developments in bone marrow Determine which statement is incorrect information about SCID.
immunodeficiency disorder (SCID). The nurse has transplantation are hopeful. Because of possible genetic involvement, parents may feel some
taught the mother about the disease. The statement guilt.
by the mother that indicates a lack of understanding is:

1.‐ "My child will grow out of this."


2.‐ "Bone marrow transplantation may be possible."
3.‐ "The prognosis for this disease is not good."
4.‐ "My child contracted the disease because of me."

411 A child is being treated for human immunodeficiency Correct answer: 3, 4, 5 The child's immunizations should be kept up‐to‐date. Live vaccines should be avoided for the Consider all activities which will protect the child and others.
virus (HIV) infection. In planning health care for this child and family. The family will need information on how to protect themselves and how to
child, the nurse would share information related to administer the prophylactic drugs as well as their side effects. The child can safely attend
which of the following? Select all that apply. school with proper education of the school personnel.

1.‐ Keeping immunizations current


2.‐ Preventing the spread of the organism
3.‐ Encouraging the family to provide home‐schooling
4.‐ Encouraging the child to participate in activities with other children
5.‐ Handwashing technique for the entire family

412 A 13‐year‐old child is scheduled for a bone marrow Correct answer: 1 Bone marrow aspirations are usually performed under local anesthesia unless the child is too Eliminate all responses that indicate correct understanding of a bone marrow aspiration,
aspiration. The nurse has explained the procedure to small to cooperate to hold still. The other statements are correct and do not require further then consider each of the options left.
the patient and his mother. Which statement by the follow‐up or teaching.
mother indicates a need for additional teaching?

1.‐ "How long will it take my child to wake up from the anesthesia?"
2.‐ "I can't believe they will take the sample out of his hip."
3.‐ "He will need to be watched for bleeding and infection after the procedure."
4.‐ "The doctors are going to use this test to find out why he can't fight infections."

413 An infant born to a mother known to be infected with Correct answer: 1 Emotional support for families of HIV+ clients can be challenging. Families who have already Knowledge of the need for psychosocial support for families with children who are
human immunodeficiency virus (HIV) has also been dealt with the problems associated with the disease process are most likely to be receptive to diagnosed with chronic conditions will aid in choosing the correct answer.
diagnosed as HIV positive. While assessing the the discussion and able to offer emotional support.
psychosocial support for the family, the nurse should
ask:
1.‐ "Does the family belong to a support group?"
2.‐ "Are there neighbors nearby?"
3.‐ "What type of insurance does the family have?"
4.‐ "Does mother have a car?"

414 The nurse has explained allergy‐proofing the home to Correct answer: 4, 5 Cloth items hold in dust. Only essential items should be stored in the child’s bedroom, and Consider what objects would hold dust and eliminate them from the environment.
the mother of a child with dust allergies. Which those should be in drawers or closets. Stuffed animals retain dust and should be removed from
statement by the mother indicates a clear the bedroom. Cotton curtains would be preferred over blinds because cotton curtains can be
understanding of appropriate allergy‐proofing? (Select washed frequently. Both the mattress and the bed should be enclosed in special plastic covers
all that apply.) to eliminate a source of dust.
1.‐ “I’m going to replace the cotton curtains on the window with blinds.”
2.‐ “The only toys allowed in his bedroom are his stuffed toys.”
3.‐ “I should store his out‐of‐season clothes in his bedroom.”
4.‐ “The mattress and box springs both need to be enclosed in a thick plastic cover.”
5.‐ “I will try to clean and vacuum the bedroom frequently to limit dust collection.”

415 A child is in the clinic for a prick test. Because of the Correct answer: 1 Prick tests determine allergens. Should the child have an allergy, epinephrine might be Eliminate Naloxone immediately after associating it with narcotic overdose. The other
risk of anaphylaxis, the nurse has which of the needed to counteract anaphylaxis. Corticosteroids such as prednisone are helpful in minimizing three drugs are related to allergies, but the correct answer is one that will work quickly and
following medications available for emergency allergic response, but would not be effective in the management of anaphylaxis. In addition, have a systemic response rather than a local one.
treatment? pretreatment with prednisone would make test results invalid. Naloxone reverses the effects
of opioid analgesics, and cromolyn sodium is useful in managing asthma.

1.‐ Epinephrine (Adrenalin)


2.‐ Prednisone (Deltasone)
3.‐ Naloxone (Narcan)
4.‐ Cromolyn sodium (Intal)

416 A mother brings a 3‐year‐old child to the clinic for a Correct answer: 3 Every time a child enters the healthcare system, the immunization status should be checked. The key word in this stem is the priority nursing action. While all these activities must be
well‐child checkup. The child has not been to the clinic Some children have uncertain history of immunization due to parental noncompliance or completed, the most important is to update immunizations.
since 6 months of age. The nurse determines that special circumstances, such as being refugees.
which of the following is the priority care for this child?

1.‐ Assess growth and development.


2.‐ Begin dental care.
3.‐ Update vaccinations.
4.‐ Complete hearing screening.

417 The mother of a 1‐year‐old child says that breast‐ Correct answer: 1 Infants receive passive immunity, which lasts 3 to 4 months, through the placenta or Option 4 can be eliminated because the nurse will never give the immunizations without
feeding her infant is sufficient to provide immunity. breastmilk. Active immunity lasts long term and is acquired by exposure to disease or maternal permission. Asking about the diet will not affect the need for immunization,
She does not want to sign the permit for immunizations. leaving only two choices to choose between.
immunizations. What is the nurse's best approach in
working with this client?
1.‐ Discuss active and passive immunity.
2.‐ Tell her immunizations are legally mandatory.
3.‐ Ask about the mother’s diet.
4.‐ Give the immunization without her permission.

418 The nurse is caring for several children in a hospital Correct answer: 1 The immunocompromised child would be the one at greatest risk for acquiring an infectious The ability to fight infection is related to the immune system. Select the child with a
unit where there has been a recent outbreak of organism. The other children would be at less risk for acquiring the gastrointestinal infection. disease of the immune system.
bacterial diarrhea. None of these children were
admitted for diarrhea, but the nurse is aware that they
may be exposed. After assessing the client population
on the unit, the nurse determines that the child most
susceptible to developing diarrhea would be the:

1.‐ Toddler with severe combined immunodeficiency disease.


2.‐ Preschooler in traction for a fractured femur.
3.‐ School‐age child with eczema.
4.‐ Teenager with frequent stools secondary to malabsorption syndrome.

419 A child is admitted to the hospital with an allergic Correct answer: 4 Eosinophils are the type of white blood cell that is associated with allergic reactions. The RBCs and the hemoglobin level are not related to the immune system, so these can be
reaction. The physician orders a complete blood count Hemoglobin is present in red blood cells (RBCs), and RBCs carry oxygen to tissues. Leukocytes eliminated immediately. Choose between the remaining two to select the type of WBC that
(CBC) with differential. The nurse would expect to see fight infection. is associated with allergies.
an elevation in the level of:
1.‐ Red blood cells (RBCs).
2.‐ Hemoglobin.
3.‐ Leukocytes.
4.‐ Eosinophils.

420 An infant is being discharged from the infant and Correct answer: 1, 3, 5 TORCH is the acronym for a set of microbes that includes toxoplasmosis; other (including Recall what the acronym TORCH stands for in order to answer the question correctly.
children’s unit with a positive TORCH titer. Parents syphilis and hepatitis); rubella; cytomegalovirus; and herpes simplex. If an infant has one of the
should be informed that: (Select all that apply.) viruses, the virus could be shed for up to 1 year. The baby may be asymptomatic at birth, but
the disease may show up later. The disease is congenital—present at birth but not genetic.
Since the baby may shed the virus, which in turn would affect an embryo, a pregnant woman
should avoid contact with the baby. The earlier in pregnancy the embryo is exposed, the
greater the risk of fetal loss or damage.

1.‐ The child may shed the virus for a year.


2.‐ TORCH is a genetic disorder.
3.‐ Pregnant women should avoid contact with the baby.
4.‐ Since the infant is asymptomatic at birth, there will be no residual effect.
5.‐ The earlier in pregnancy that TORCH occurred, the greater the risk to the embryo.
421 An infant with acquired immunodeficiency syndrome Correct answer: 2 The HIV virus is spread by contact with blood and body fluids. Clean gloves should be worn Recall that the virus responsible for AIDS is a bloodborne pathogen. The correct answer is
(AIDS) will be attending daycare. The daycare workers when changing the diapers as bare hands would expose the workers to body fluids. It is not the one that represents standard precautions, which are sufficient to prevent the
are concerned about spreading the human necessary to store the infant’s items separately from those of others, since the virus is not acquisition of a bloodborne pathogen.
immunodeficiency virus (HIV). The public health nurse transmitted on objects. It is also not necessary, and in fact is excessive, to wear isolation
is explaining to the workers the precautions they gowns, and it is unnecessary to minimize contact when the infant has a fever.
should take. The nurse would include in this discussion
which of the following precautions that need to be
taken?
1.‐ Storing all of this infant’s supplies separately from those of the other children
2.‐ Wearing gloves when changing the child’s diapers
3.‐ Always wearing gloves and isolation gowns when handling the infant
4.‐ Minimizing contact with the infant when it is febrile

422 A mother overhears two nurses discussing a measles Correct answer: 1 The incubation period is the time between exposure and outbreak of the disease. It is often a To answer this question correctly, it is necessary to understand the concept of the
outbreak. The nurses are talking about the incubation period when the child can be contagious without others being aware of the possible exposure. incubation period.
period. The mother asks the nurses why it is important
to know the incubation period for a childhood disease.
The nurse would include which information about the
incubation period of a disease in the reply?

1.‐ Describes a period when the child might be contagious


2.‐ Determines the severity of the infection
3.‐ Varies depending on the age of the child
4.‐ Is a period of time when medications can prevent the development of symptoms

423 An infant with numerous congenital defects and a Correct answer: 3 With the birth of a less‐than‐expected infant, the parents may have difficulty accepting the Options 1 and 4 can be eliminated since there is no evidence of either in the stem of the
diagnosis of rule out TORCH syndrome is admitted child. In addition, the anticipated longer hospitalization and separation from the parents inhibit question. Choose between options 2 and 3 by choosing the option that includes all of the
from the birth hospital directly to the pediatric bonding, which could lead to altered attachment. clients in the question.
hospital. The father tells the pediatric nurse that he
and his wife had planned a beautiful birth experience
and can’t believe what’s happened. The nurse would
formulate which of the following nursing diagnoses as
a priority for this family at this time?

1.‐ Risk for caregiver role strain


2.‐ Situational low self‐esteem
3.‐ Risk for impaired parent/infant attachment
4.‐ Parental role conflict

424 A 14‐year‐old child is receiving intravenous antibiotics Correct answer: 3 One of the most common side effects of gentamycin is nephrotoxicity. The nurse can monitor Gentamycin is a member of the aminoglycoside group of antibiotics, all of which are
for an infection. The physician has ordered gentamycin kidney function by monitoring intake and output. nephrotoxic and ototoxic.
(Garamycin). Because of the side effects of this drug,
the nurse would monitor:

1.‐ Temperature.
2.‐ Blood pressure.
3.‐ Intake and output.
4.‐ Breath sounds.
425 A 3‐year‐old child is admitted to the hospital to rule Correct answer: 3 White blood cells are one component of the general nonspecific immune response. They are Recall that a WBC count can determine infections. The WBC with differential can also
out an infection. Which diagnostic test does the nurse among the first responders stimulated by a pathogenic organism. A white cell differential can determine if an allergic reaction has occurred.
anticipate being ordered that is likely to differentiate often determine if the illness is of bacterial, viral, or allergic origin.
an infection from an allergic response?

1.‐ Hemoglobin and hematocrit


2.‐ Red blood cell count
3.‐ White blood cell differential
4.‐ Platelet agglutinization

426 A 2‐year‐old child has eczema that causes extreme Correct answer: 1 Because of the itching, the child will be scratching. Intense scratching can break the skin, and The diet is a problem for the child due to wheat allergy, but an appropriate diagnosis
itching. Treatment has not been able to control the the child might develop a bacterial infection secondary to the skin trauma. Imbalanced related to diet is risk for infection. The fact that treatment has not controlled the symptoms
rash. It has been determined that the primary allergen nutrition, more than body requirements, does not clearly state the problem with the food does not indicate noncompliance.
is wheat. An appropriate nursing diagnosis would be: allergies, nor does ineffective infant feeding behavior. There is no evidence of noncompliance,
and infant feeding would not be a diagnosis for a 2‐year‐old.

1.‐ Risk for infection.


2.‐ Imbalanced nutrition, more than body requirements.
3.‐ Ineffective infant feeding behavior.
4.‐ Noncompliance.

427 A child’s mother tells the nurse that her child has Correct answer: 4 Oral polio virus vaccine contains a live virus, which could cause an infection in a child who is The word “live” in the option should be a clue to the right response.
been on steroids for several months. Which of the immune‐depressed as a result of taking steroids.
following vaccines is contraindicated?
1.‐ Tetanus toxoid
2.‐ Recombinant hepatitis B vaccine
3.‐ Poliovirus vaccine inactivated
4.‐ Poliovirus vaccine live oral trivalent

428 A child with severe combined immunodeficiency Correct answer: 2 Care of the immunocompromised child focuses on preventing infection. The nursing Recognize that a child with an immunodeficiency will not be able to fight infections, so
disorder (SCID) is being discharged from the hospital to implementations related to reaching this goal might include limiting contact with a large prevention is important.
home. Client teaching is important to reach client number of people, but that would not be the goal of the nursing care plan.
goals. The nursing care goal for the client before and
after discharge would be that the child:

1.‐ Remains well oxygenated.


2.‐ Remains free of infection.
3.‐ Maintains hydration.
4.‐ Avoids contact with other people.

429 A child is being worked up for allergies. The mother Correct answer: 4 Allergies are confirmed by a RAST test. RAST is a radioallergosorbent test that detects IgE There are several testing methods for allergies. Eliminate symptoms of the disease as that
asks how the diagnosis will be made. The nurse antibodies that are part of the allergic response. Urticaria is itching and is symptomatic of is not a diagnostic test. The learner should be aware that eosinophil levels would be
explains that diagnosis of allergies is based on: allergies and other diseases, and an increase in eosinophils is diagnostic of allergies. elevated. IgG is a normal immunoglobulin found in the blood.

1.‐ Medical history of urticaria alone.


2.‐ IgG levels.
3.‐ Decreased eosinophils count.
4.‐ RAST test.
430 An infant is born with microcephaly. Part of the Correct answer: 3 The acronym TORCH stands for toxoplasmosis, other (syphilis, hepatitis), rubella, Recall the meaning of the acronym TORCH to answer this question correctly.
infant’s assessment includes a TORCH test. In providing cytomegalovirus, and herpes simplex virus. It is a study of common viruses that cause
client education, the nurse explains to the mother that significant fetal damage.
the TORCH test will assess for:
1.‐ Presence of the TORCH virus.
2.‐ Complications of pregnancy.
3.‐ Presence of one or more specific viruses.
4.‐ Evidence of thalidomide poisoning.

431 A 2‐month‐old infant has been admitted with a Correct answer: 1 Neonates with sepsis may display either hypothermia or hyperthermia, but hypothermia is Recall that neonates are as likely to demonstrate hypothermia as hyperthermia.
diagnosis of sepsis. The nurse would monitor the child more common. The other symptoms are not associated with sepsis.
for evidence of:
1.‐ Hypothermia.
2.‐ Rash.
3.‐ Sunken fontanels.
4.‐ Glucosuria.

432 A 12‐year‐old child with positive human Correct answer: 4 Families need to know that casual contact cannot spread HIV. However, basic infection The spread of infection would be a concern for this child whether at home or at the
immunodeficiency virus (HIV) antibodies is going home control practices must be maintained to prevent exposure through body fluids. Growth and hospital.
from the hospital. Which of the following would be the development milestones and immunization schedules are routine elements of teaching, and
most important home‐going instructions? are therefore not as high of a priority for this client as infection control. Lab studies and results
are ongoing and are therefore also of lesser priority.

1.‐ Growth and developmental milestones


2.‐ Immunization schedules
3.‐ Lab studies and results
4.‐ Prevention of the spread of HIV

433 A 4‐year‐old child has been exposed to chickenpox. Correct answer: 1 The prodromal period refers to the period of time between the initial symptoms and the Understanding the meaning of the term “prodomal” will guide the learner to the right
After the nurse has provided information about presence of the full‐blown disease. The rash would not be apparent during this time. All the response.
chickenpox, the nurse asks the mother to repeat the other statements are correct.
information. The mother’s statement that indicates a
need for additional information is:

1.‐ “During the prodromal period, my child will have pox all over his body.”
2.‐ “Chickenpox is a viral infection that can be spread to other children.”
3.‐ “I should monitor my child for Reye Syndrome, which is a complication of chickenpox.”
4.‐ “My child should not visit my pregnant sister at this time.”

434 A child who contracts chicken pox at age 5 has Correct answer: 1 Active acquired immunity occurs when the body produces antibodies or develops immune Use the rule of opposites to answer this question.
developed which type of immunity? lymphocytes against specific antigens (chickenpox). Breastfeeding a child would offer passively
acquired immunity; immune globulins offer passively acquired artificial immunity;
immunizations offer actively acquired artificial immunity.
1.‐ Active acquired, natural
2.‐ Passive acquired, natural
3.‐ Passive acquired, artificial
4.‐ Active acquired, artificial

435 A person who is HIV‐positive starts to exhibit signs of Correct answer: 4 A client with AIDS will usually have a low CD4 count and a high viral load. What is desired is to This question requires knowledge about viral loads and CD4 levels.
AIDS. The indication that the client has seroconverted have a high CD4 count and a low viral load (which should normally be zero). The white blood
would be partially diagnosed by which of the count will usually show neutropenia.
following?
1.‐ Low viral load
2.‐ High CD4 count
3.‐ High white blood count (WBC)
4.‐ High viral load

436 A sexually active teenager with flu‐like symptoms is Correct answer: 2 The ELISA test may be negative upon initial testing and positive at the time of seroconversion, This question requires knowledge about the ELISA tests.
given an ELISA test that returns negative. The physician which takes 6 to 12 weeks after infection. This time period when the antibodies are negative is
informs her that another ELISA test will be conducted called the seroconversion window and virally infected individuals may have negative antibody
in several weeks. The client wants to know why. The tests.
best explanation is that:

1.‐ The first test may be inaccurate.


2.‐ The antibodies do not always show up initially.
3.‐ The test is sensitive and can give false positives.
4.‐ It is standard practice.

437 A client experiences an anaphylactic reaction after Correct answer: 2 Type I hypersensitivity reactions are caused by widespread antigen‐antibody reactions such as This question requires knowledge about hypersensitivity responses.
taking an antibiotic for the first time. The results of this anaphylaxis. These responses are usually immediate and lead to an antigen‐antibody complex
Type I hypersensitivity response are caused by: that causes the release of histamine. Option 4 is an explanation of what occurs with a blood
transfusion reaction. Option 3 is an explanation of a Type IV delayed hypersensitivity. Option 1
is false.
1.‐ A histamine precursor causing anaphylaxis.
2.‐ Antigen‐IgE‐mast cell interaction.
3.‐ Cell‐mediated response.
4.‐ Massive numbers of destroyed red blood cells.

438 A client who has recently been diagnosed with Correct answer: 1 Recognition of self as foreign is the definition of any autoimmune disease. Further This question requires knowledge about diabetes.
diabetes mellitus (DM) Type 1 asks the nurse how she explanation may be needed to explain that the immune system usually recognizes self and
developed this because no one in her family is a identifies what is foreign, targets foreign cells, and destroys them.
diabetic. The nurse’s best response is, “DM is an
autoimmune disease characterized by:”
1.‐ “Failure of the immune system to recognize self.”
2.‐ “Exacerbations and remissions.”
3.‐ “Accelerated production of killer T‐cells.”
4.‐ “Immunosuppression and altered cortisol levels.”

439 A mother of twins calls the office and speaks to the Correct answer: 3 You should have recognized this as serum sickness, a reaction a week after ingestion of a This question requires knowledge about hypersensitivity reactions.
nurse concerning a rash that has developed on both drug. Serum sickness is a type III hypersensitivity reaction where formation of IgG or IgM
children since taking an antibiotic prescribed 5 days antibody‐antigen complexes occur in the blood.
ago. The nurse knows that this is most likely a:

1.‐ Type I hypersensitivity reaction.


2.‐ Type II hypersensitivity reaction.
3.‐ Type III hypersensitivity reaction.
4.‐ Type IV hypersensitivity reaction.

440 In working with clients with HIV, the nurse knows that Correct answer: 4 Symptoms of HIV infection are vague and nonspecific. Characteristic manifestations of HIV This question requires knowledge about HIV/AIDS.
the illness is more difficult to manage once AIDS has disease resulting from opportunistic infections and neoplasm make treatment difficult.
been diagnosed. Which of the following best Invasion may be from sexual contact as well as blood contact. HIV is not always predictable
characterizes HIV disease? because the virus can lie dormant for many years. There are really no carrier states in HIV.

1.‐ Individuals who test positive are carriers and considered contagious.
2.‐ Clinical manifestations have a characteristic and predictable sequence.
3.‐ The HIV virus invades cells primarily via the bloodstream.
4.‐ Symptoms result from opportunistic pathology.

441 Which of the following individuals is at highest risk of Correct answer: 3 Only fluids containing blood or blood cells have been identified as a mode of transmission for This question requires knowledge about blood borne pathogens.
contact with HIV? An individual who: HIV. Collecting blood, especially in a mobile unit (where the population is more diverse) is a
risk for any healthcare worker. Appropriate gloving is essential. Counseling may require touch,
which is not a form of transmission; perspiration has not been identified as a form of contact;
and the ELISA test requires contact with saliva.

1.‐ Counsels HIV victims and their families.


2.‐ Works with athletes who perspire a lot.
3.‐ Collects blood donations via a mobile blood unit.
4.‐ Performs physicals involving an ELISA test for insurance companies.

442 A client is brought to the Emergency Department Correct answer: 1 Because laryngeal spasms and bronchial constriction can occur with anaphylaxis, assessing Recall the ABCs (airway, breathing, and circulation). Airway is always a priority.
after taking a dose of penicillin. Which of the following the client's airway is top priority. The nurse should maintain and establish a patent airway first.
diagnoses is the highest priority in this client who is Remember the ABCs (airway, breathing, and circulation), cardiac output would come next
demonstrating anaphylaxis? followed by risk for injury and finally anxiety.

1.‐ Ineffective airway clearance


2.‐ Decreased cardiac output
3.‐ Risk for injury
4.‐ Anxiety

443 In addition to a viral load of 25,000, which of the Correct answer: 3 A client with AIDS will have exacerbations and remissions with opportunistic infections, Use the process of elimination to answer this question.
following would indicate that the medications being therefore symptoms may vary. With a diagnosis of AIDS, an ELISA test would remain positive
taken by a client with AIDS are working? for antibodies. WBC of 1,700 shows neutropenia which does not indicate improvement. The
CD4 cell count between 200 and 500 is in the "suppressed immune state" but certainly above
the 200 mark that is indicative of severe depression of the immune system.

1.‐ Rare occurrence of symptoms


2.‐ Negative ELISA test
3.‐ CD4 cell count of 490
4.‐ WBC of 1,700 mm

444 A child with asthma caused by allergies would be Correct answer: 1 Eosinophils are usually elevated in an allergic response. The WBC in option 2 is barely above This question requires knowledge about the CBC.
expected to have which of the following findings on a normal. The monocytes are normal in option 3 and the elevated neutrophils indicate an acute
complete blood count (CBC) report? infection (option 4).
1.‐ Eosinophils 21.9
2.‐ WBC 10.9
3.‐ Monocytes 4.0
4.‐ Neutrophils 85.7

445 The mother of a child with swollen lymph nodes is Correct answer: 2 The mother is already alarmed enough, and the nurse needs to be careful with wording of the Use the process of elimination to discover the correct answer.
extremely panic‐stricken that the swelling means response. Option 2 is correct and is not alarming so that the mother may be able to focus on a
cancer. The nurse could calm the mother by stating different perspective besides cancer.
that:
1.‐ "The finding is very alarming and could be serious, but you must remember it may be insignificant also."
2.‐ "The lymph nodes are the organs that filter foreign products and may only be swollen because of an infection."
3.‐ "The lymph nodes will swell quite often and we may not ever know it."
4.‐ "The lymph nodes are the major organ indicating a problem with the immune system."
446 A client reports to the clinic complaining of itching Correct answer: 4 This type of contact dermatitis is commonly a delayed reaction and a type IV hypersensitivity. This question requires knowledge about hypersensitivity reactions.
and weeping along the back of her legs. Upon This reaction is cell‐mediated rather than antibody‐mediated and delayed 24 to 48 hours.
inspection, wheals are evident that appear to be
poison ivy. After talking to the client, it is learned that
she broke out a day after sitting on the car seat in
shorts. She sat on the same seat as her husband, who
had been working in a field of grass all day. This type of
reaction is a:
1.‐ Type I hypersensitivity.
2.‐ Type II hypersensitivity.
3.‐ Type III hypersensitivity.
4.‐ Type IV hypersensitivity.

447 A client with A‐negative blood can receive which type Correct answer: 2 Remember the Rh must also match besides the type of blood (A in this case). Rh matching is Use the process of elimination to determine the correct answer.
of blood transfusion in order to avoid any allergic not just for mothers and infants to prevent erythroblastosis fetalis.
reaction?
1.‐ A+
2.‐ A‐
3.‐ O+
4.‐ AB+

448 In assessing a client with a suspected latex allergy, the Correct answer: 3 Clients with a history of allergies to fruit such as bananas or kiwi tend to have latex allergies. Use the process of elimination to determine the correct answer.
nurse should ask which of the following? The degree of moistness of the skin might need to be assessed but will not determine a latex
allergy. Although drug allergies should be asked, this information does not help in determining
a latex allergy. Option 4 is also important information for an assessment, but the focus of the
question for a latex allergy would be if there were any problems after the surgery similar to
the one being exhibited now.
1.‐ "Are your hands usually moist or dry?"
2.‐ "What drug allergies do you have?"
3.‐ "Are you allergic to bananas or kiwi fruit?"
4.‐ "What types of surgeries have you had?"

449 Which of the following symptoms would be expected Correct answer: 4 A barking cough, wheezing, and stridor are clinical manifestations of the bronchoconstriction This question requires knowledge about anaphylactic reactions.
in an anaphylactic reaction? and edema that accompanies anaphylaxis. The blood pressure is usually low (hypotension) and
the pulse fast (tachycardia).
1.‐ Hypertension
2.‐ Bradycardia
3.‐ Rales
4.‐ Stridor

450 The pathophysiology behind the destructive power of Correct answer: 2 The T‐helper cells are the primary target for the parasite to infect in order to replicate. The This question requires knowledge about blood borne transmissions.
AIDS is that HIV kills the: virus destroys the T‐cells and along with this destruction, memory cells can also be destroyed,
hence opportunistic infections are more prevalent.
1.‐ B cells that produce antibodies.
2.‐ T‐helper cells.
3.‐ White blood cells.
4.‐ Platelets.
451 Of the following people who is at increased risk of Correct answer: 2 The police officer and nurse on the telemetry unit should be using standard precautions This question requires knowledge about HIV.
acquiring HIV? including gloves anytime body secretions are encountered. Although either of these may
encounter blood accidentally, the percentage is low. A school nurse should not be coming into
contact with body secretions that would increase the risk factor. A sexually active teenager,
especially if the act is unprotected, is at highest risk.

1.‐ A police officer who works the streets and responds to emergencies
2.‐ A sexually active teenager
3.‐ A school nurse who exams children
4.‐ A nurse working on a telemetry unit

452 A client with recent HIV seroconversion and early Correct answer: 2 The stages of HIV are varied, but most clients begin with flu‐like symptoms that occur days to This question requires knowledge about HIV.
infectious disease asks the nurse what to expect in weeks after contracting the virus. Following this is a long asymptomatic period; however the
terms of disease progression. The nurse tells the client virus is still present. It is unclear why or when a client moves from being asymptomatic to
that although the disease can vary greatly among AIDS.
individuals, the usual pattern of progression includes:

1.‐ About a 10‐year period of vague and nonspecific symptoms before the onset of AIDS.
2.‐ Eight to 12 years of chronic flu‐like symptoms before the development of serious opportunistic infections or tumors.
3.‐ A 10‐year period of normal to slightly decreased T cell counts followed by about 2 years of symptoms before AIDS occurs.
4.‐ An 8‐ to 10‐year period of asymptomatic infection in which the virus is in remission before it becomes active and causes opportunistic diseases.

453 The second child (who is Rh+) of an Rh‐ mother may Correct answer: 4 To answer this question correctly, you must understand that transfusion reactions (in this This question requires knowledge about Rh factor.
develop problems because of: case from mother to child since the Rh was incompatible) is a type II hypersensitivity reaction.
The maternal antibodies that were developed with a first child who may have been Rh+ are
passed to the infant and cause hemolysis of fetal red blood cells (not white blood cells). The
child may suffer from anemia (option 3) but this is not the primary cause of the problem.

1.‐ Antigens passed to the child by the mother.


2.‐ Hemolysis of white blood cells in the child.
3.‐ Low hemoglobin level.
4.‐ Antibodies passed to the child by the mother.

454 The parents of a child who is in surgery having a Correct answer: 2 The spleen is vital in storing blood and in the breakdown of red blood cells, but it is not This question requires knowledge about splenectomy.
splenectomy ask the nurse how their child can live essential for life. The liver and bone marrow assume its function when it is removed, and most
without a spleen. The nurse's best response is: clients do well even though it is removed.

1.‐ "Somehow the body in its miraculous form takes over."


2.‐ "The liver and bone marrow assume its function."
3.‐ "The spleen slowly begins to atrophy anyway and serves little purpose after adulthood."
4.‐ "It is a vital organ, but most people do fine when it is removed."

455 A common clinical manifestation of a transfusion Correct answer: 2 Common manifestations are fever, chills, low back pain, hypotension, tachycardia, nausea, This question requires knowledge about hypersensitivity reactions.
reaction (type II hypersensitivity reaction) is: and vomiting. Urticaria and red‐colored urine are often seen.

1.‐ Diarrhea.
2.‐ Fever.
3.‐ Hypertension.
4.‐ Bradycardia.
456 In assessing the lung sounds of a client in anaphylaxis, Correct answer: 3 Edema and bronchoconstriction are the clinical manifestations involving the respiratory This question requires knowledge about anaphylaxis.
the nurse would expect to hear which of the following system in anaphylaxis and usually produce diminished lung sounds, wheezing and stridor
sounds? (which does not require a stethoscope).
1.‐ Gurgles
2.‐ Coarse sounds
3.‐ Wheezing
4.‐ Friction rub

457 A positive tuberculosis (TB) skin test would be Correct answer: 2 TB skin tests are read 72 hours after administration and a true positive reading should show This question requires knowledge about TB skin test.
manifested in which of the following? redness and be raised (greater than 5 mm). Clients may react within several hours to 24 hours
of receiving the injection and then show a negative finding at 72 hours. A TB skin reaction at 72
hours is a type IV delayed hypersensitivity reaction and can indicate exposure or active
disease.
1.‐ Red area 3‐cm wide within 24 hours
2.‐ Red, raised area at least 5 mm, 72 hours later
3.‐ Redness and urticaria, 72 hours later
4.‐ Raised area, 24 hours later

458 A client, who received a skin graft 2 months ago Correct answer: 1 Acute tissue rejection is common and usually occurs between 4 days and 3 months after This question requires knowledge about acute and chronic reactions.
because of extensive burns, reports to the clinic with transplant. The manifestations are caused by the inflammatory process.
complaints of redness, swelling, fever, and tenderness
over the graft site. This client is exhibiting:

1.‐ Acute tissue rejection.


2.‐ Hyperactive tissue rejection.
3.‐ Chronic tissue rejection.
4.‐ Graft‐versus‐host disease.

459 A client receiving a unit of packed red blood cells Correct answer: 3 The key word in this stem is 'first.' All of the options are correct, and the nurse should This question requires knowledge about the first aid needed.
(RBCs) begins to complain of chills, temperature is perform all of them, but in the proper sequence. The transfusion should be stopped because of
101.4 degrees F, pulse is 185, and blood pressure is the signs of a transfusion reaction. The physician then needs to be called at the same time the
80/50. The nurse should do which of the following vital signs and client are monitored every 5 minutes. Sending the bag to the laboratory is the
first? last step.
1.‐ Call the physician
2.‐ Send the blood bag to the laboratory
3.‐ Stop the transfusion and flush the line
4.‐ Continue to record the VS and monitor the client every 5 minutes

460 A client with AIDS who is at high risk and having Correct answer: 1 When the viral load (number of circulating HIV particles per milliliter) is high and the CD4 Recognize the inverse relationships in each option.
multiple opportunistic diseases would probably have a count is low, the client is most at risk. This would explain why the opportunistic infections are
low CD4 count and which of the following? recurring, the immune system is extremely compromised. A zero viral load is expected in
normal individuals. A moderate or low viral load may be seen in clients who have received
medication for AIDS.
1.‐ High viral load (greater than 100,000)
2.‐ Low viral load (less than 10,000)
3.‐ Moderate viral load (10,000 to 100,000)
4.‐ Zero viral load

461 The difficulty in assessing clients who are HIV‐positive Correct answer: 1 Although options 2, 3, and 4 are sometimes the case, the flu‐like symptoms are rather vague. Use the process of elimination to determine the correct answer.
after exposure, but who have a negative ELISA test, is Most individuals do not rush to a physician with flu‐like symptoms unless they are not getting
that the symptoms are: any better.
1.‐ Flu‐like and vague.
2.‐ Specific and similar to tuberculosis.
3.‐ Often ignored.
4.‐ Attributed to other illnesses.

462 The wife of a client diagnosed as HIV‐positive states Correct answer: 4 Sexual activity if one partner is positive for HIV can be resumed as long as protection is always This question requires knowledge about HIV states.
she will never be able to have sexual intercourse again. used. Option 1 is inappropriate and option 2 is incorrect. Option 3 may need to occur but
The nurse should respond with: seems to be an answer that avoids the client's concern at this time.

1.‐ "It probably would be best not to engage in sexual activity."


2.‐ "It shouldn't be a problem as long as oral or anal sex is avoided."
3.‐ "Perhaps counseling for you and your husband would help."
4.‐ "Sexual activity can be resumed, but you must always have protected sex."

463 Education in the community about HIV should consist Correct answer: 2 Consenting sexual partners should be tested to determine that each is HIV‐negative if This question requires knowledge about HIV.
of which of the following? unprotected sex is preferred. No known proof exists that saliva is a route for transmission.
HIV1 is the most common form in the United States, HIV2 is in Africa, and AIDS is still a major
threat to certain populations in the United States.
1.‐ It is not as big of a threat as it once was.
2.‐ Unprotected sex should never be engaged in unless both partners are known to be HIV‐negative.
3.‐ HIV can be transmitted by the oral (saliva) route.
4.‐ HIV2 virus is the most common form in the United States.

464 Which of the following is responsible for direct Correct answer: 2 Killer T cells bind with cell surface antigen or virus‐infected or foreign cells. Killer T cells This question requires knowledge about direct antigen attacks.
antigen attack and destruction? destroy the antigen by combining with it and then either destroying its cell membrane or
releasing cytotoxic substances into the cell.
1.‐ Helper T cells
2.‐ Killer T cells
3.‐ Suppressor B cells
4.‐ Memory B cells

465 A nurse accidentally gets stuck with a needle from a Correct answer: 3 After exposure to a known antigen such as hepatitis, temporary immunity is recommended in Use the rule of opposites to answer this question.
known hepatitis B client. The protocol would be for her the form of immune globulins. If the nurse had received the hepatitis B vaccine (Heptavax), he
to receive a hepatitis B immune globulin to offer: or she should have artificial active immunity. Remember, natural immunity comes in the form
of antibodies from having the disease or from mother's who breastfeed.

1.‐ Artificial active immunity.


2.‐ Natural acquired immunity.
3.‐ Artificial passive immunity.
4.‐ Natural passive immunity.

466 The immune complexes in type III allergic reactions Correct answer: 3 An Arthus reaction is a type III hypersensitivity reaction that causes acute, localized edema This question requires knowledge about Arthus reactions.
cause a localized reaction of tissue necrosis. This is and tissue inflammation (usually of the skin). It usually occurs at the site of an injection of an
called: antigen in a client previously sensitized.
1.‐ Gangrene.
2.‐ Inflammation.
3.‐ Arthus.
4.‐ Infarction.

467 An example of a type I hypersensitivity immune Correct answer: 1 Hay fever is an atopic type I reaction that is local instead of systemic. Transplant rejection is a Use the process of elimination to correctly answer this question.
response includes which of the following? type IV; transfusion reaction is a type II and serum sickness is a type III.
1.‐ Hay fever
2.‐ Transplant rejection
3.‐ Transfusion reaction
4.‐ Serum sickness

468 The most common transplanted organ affected by Correct answer: 4 Graft‐versus‐host disease is most common with bone marrow transplants. When This question requires knowledge about GvHD.
graft‐versus‐host disease (GvHD) is which of the immunocompetent graft cells recognize host tissue as foreign, a cell‐mediated immune
following? response occurs.
1.‐ Kidney
2.‐ Liver
3.‐ Heart
4.‐ Bone Marrow

469 The reason the AIDS virus is so devastating to the Correct answer: 3 CD4 or T helper cells are those that play a key role in controlling the immune response by This question requires knowledge about the AIDS virus.
immune system is that it attacks: stimulating proliferation of other T cells, amplifying the cytotoxic activity of killer T cells,
activating B cells to proliferate and differentiate, and interact directly with B cells to promote
antibody production.
1.‐ Neutrophils.
2.‐ B lymphocytes.
3.‐ CD4 lymphocytes.
4.‐ CD8 lymphocytes.

470 The "seroconversion window" of HIV infection refers Correct answer: 3 In category A of the CDC classification, individuals who have been infected may not This question requires knowledge about the seroconversion in HIV.
to that time which: demonstrate antibodies on an ELISA test or Western blot. This time period when antibodies
are negative, but infection has occurred, is called the "seroconversion window." The danger
here is that the individual does not know he or she is HIV‐positive and may infect others.

1.‐ The individual has been infected and the acute mononucleosis symptoms appear.
2.‐ The individual has been infected and initial symptoms appear.
3.‐ The individual has been infected and antibody levels are detectable.
4.‐ The individual has been infected and opportunistic diseases occur.

471 An individual who is HIV‐positive and who develops Correct answer: 3 AIDS dementia complex involves cognitive, behavioral, and motor deficits and is a common This question requires knowledge about AIDS dementia.
memory loss, difficulty concentrating, euphoria, and central nervous system complication of untreated HIV. Along with the above symptoms,
lethargy is developing which of the following? apathy, confusion, hallucinations, personality changes, unsteady gait, leg tremors, impaired
handwriting, and mental slowing will occur.
1.‐ Alzheimer's disease
2.‐ Senile dementia
3.‐ AIDS dementia complex
4.‐ Wasting syndrome

472 The nurse encourages a new mother to breastfeed Correct answer: 3 Antibodies that the mother has will be passed on to the infant. This form of immunity is This question requires knowledge about passive versus active immunities.
her infant, even for a short time, because colostrums natural versus artificial. Remember the difference between passive (temporary immunity) and
in breast milk will provide the infant with: active (long‐term). Words such as indefinite (option 1) and all (option 2) should be red flags
that these are incorrect.
1.‐ Indefinite active immunity to childhood illnesses.
2.‐ Passive immunity to all childhood illnesses for several months.
3.‐ Passive immunity to diseases to which the mother has immunity.
4.‐ Active immunity for several years to diseases to which the mother has immunity.
473 A client with seasonal allergies has just recovered Correct answer: 1 Allergic rhinitis is a type I or IgE‐mediated hypersensitivity where an allergen interacts with This question requires knowledge about immunity.
from a severe case of rhinitis and an increased IgE that is bound to mast cells and basophils. A radio allergy sorbent test (RAST) will determine
incidence of asthma attacks. The nurse would expect the presence of IgE.
to see an increase in which of the following?

1.‐ IgE
2.‐ IgG
3.‐ IgA
4.‐ IgM

474 Initial evidence that should indicate to the nurse that Correct answer: 3 Any local reaction (type I hypersensitivity) to an injected allergen should place the nurse or Use intuition to determine the initial response.
a client may experience a systemic anaphylactic health care provider on guard for a possible anaphylactic reaction. The client should be closely
reaction to an injected allergen is the development of: monitored. The itching and edema are common local reactions. If itching occurs all over,
especially on the palms and scalp, a systemic effect is likely. If dyspnea occurs, a systemic
anaphylactic reaction is occurring and precaution should be taken to manage the airway. A
wheal and flare reaction usually doesn't occur for several hours or days and is a local reaction,
sometimes expected.
1.‐ Dyspnea.
2.‐ Dilation of the pupils.
3.‐ Itching and edema at the injection site.
4.‐ A wheal and flare reaction at the injection site.

475 A client develops severe angioedema involving her Correct answer: 1 Reactions such as these may be genetic and knowing whether other members of a family This question requires knowledge about family history of allergies.
face, hands, and feet with burning and stinging of the have similar reactions is useful in determining a cause. The use of OTC medications and home
lesions after consuming her 9:00 A.M. medications. A medications may be helpful, but if this reaction has not occurred before, it is less likely to be
significant risk factor for allergies that the nurse should from those sources. Option 3 would not offer any assistance at this time.
question the client about is:

1.‐ A family history of allergies.


2.‐ The use of OTC medications.
3.‐ Recurrent infections or illnesses.
4.‐ Home medications she is taking.

476 After being bitten by an unknown insect, a client Correct answer: 2 Airway is always first (ABCs) when determining priority in an emergency situation such as this. Remember the ABCs (airway, breathing, and circulation). Airway is a key priority.
allergic to wasp stings is brought to a clinic by a All of the other options are accurate and should be implemented, but with the symptom of
coworker. Upon arrival, the client is anxious and difficulty breathing, laryngeal edema is a priority concern.
having difficulty breathing. The first action by the nurse
is to:
1.‐ Administer oxygen.
2.‐ Maintain the client's airway.
3.‐ Remove the stinger from the site.
4.‐ Place the client in a recumbent position with legs elevated.

477 The nurse discusses the prevention and management Correct answer: 4 Although a change in occupation may be wise, the beekeeper can practice cautious steps and Use the process of elimination to determine the correct answer.
of allergic reactions with a beekeeper who has preventative measures to protect self. A Medic‐alert bracelet is highly suggestive and
developed a hypersensitivity to bee stings. The nurse epinephrine (Epi‐Pen) should be with him always. Use of corticosteroids as a maintenance dose
identifies a need for additional teaching when the is not recommended because of the vast majority of side effects.
client states:
1.‐ "I need to think about a change in my occupation."
2.‐ "I should wear a Medic‐alert bracelet indicating my allergy to bee stings."
3.‐ "I will learn to administer epinephrine so that I will be prepared if I am stung again."
4.‐ "I will need to take maintenance doses of corticosteroids to prevent reactions to further stings."
478 Over‐the‐counter (OTC) pseudoephedrine tablets and Correct answer: 2 Decongestant nasal sprays have a rebound effect, which causes congestion and swelling of Use knowledge of allergy medications to answer this question.
nasal sprays are used by a client to control symptoms the mucous membranes with long‐term use. Although temporary relief may be obtained,
of seasonal rhinitis. In teaching the client about the continued chronic use of the sprays may be needed due to this engorgement of the vessels and
use of OTC allergy medications, the nurse advises the increased congestion. This client may need to consult an allergist or physician. This question
client that: draws on your knowledge of pharmacology. As you study and you find a question such as this
that you miss, go back and look up the classification of this drug and agents such as
oxymetazoline (Afrin) or phenylephrine (Neo‐Synephrine) and reread about it.

1.‐ Seasonal allergies should be treated only with prescription drugs.


2.‐ Overmedication with decongestant nasal sprays may increase her nasal congestion and swelling.
3.‐ Pseudoephedrine may cause drowsiness, and she should not drive or use machinery while taking it.
4.‐ She should take the pseudoephedrine continuously to prevent allergy symptoms from developing.

479 The client who received a bone marrow transplant for Correct answer: 1 This is an example of graft‐versus‐host disease (GvHD), which is a complication of bone This question requires knowledge about leukemia
treatment of leukemia develops a skin rash 10 days marrow transplants. When immunocompetent graft cells recognize host tissue as foreign, a
after the transplant. The nurse recognizes that this cell‐mediated immune response is initiated. The skin, liver, and gastrointestinal tract are often
reaction indicates that: targets.
1.‐ The skin is a target organ for destruction by T‐cells.
2.‐ The client's circulating antibodies are rejecting the donor bone marrow.
3.‐ A delayed hypersensitivity reaction is occurring with necrosis of donor tissue.
4.‐ The client's epithelial T‐cells are destroying new white cells produced by the donor marrow.

480 A client who tested positive for HIV 3 years ago is Correct answer: 4 This client has a Category C2, which is an AIDS‐indicator condition (pneumonia) and a CD4 This question requires knowledge about HIV.
admitted to the hospital with Pneumocystis carinii count between 200 and 499. Category A: PGL is persistent generalized lymphadenopathy and
pneumonia and a CD4 count of 200 would not be accompanied by the pneumonia. Category B has several conditions that may
mm&lt;sub&gt;3&lt;/sub&gt;. Based on diagnostic occur but Pneumocystis carinii pneumonia is not one of these.
criteria established by the Centers for Disease Control
and Prevention, the client is diagnosed as having:

1.‐ Category A: Persistent generalized lymphadenopathy (PGL).


2.‐ Category B.
3.‐ Latent chronic disease.
4.‐ Category C: AIDS.

481 A client receives a skin prick test for determination of Correct answer: 3 Wheals, erythema, and itching are common after a skin prick test, which is conducted by This question requires knowledge about allergy testing.
allergies. Besides the wheals and erythema present, placing a drop of a specific allergen to the skin and pricking the skin at the site of the drop. A
another symptom that will most likely be exhibited by response should occur in 15 to 20 minutes. Dyspnea would indicate an anaphylactic reaction
the client with a positive test would be: and usually hypotension occurs. A rash usually doesn't occur over the entire body.

1.‐ Dyspnea.
2.‐ Hypertension.
3.‐ Itching.
4.‐ Rash over entire body.

482 Which of the following atypical findings would the Correct answer: 3 Mental status changes ranging from restlessness to confusion is one of the most frequent Recall that changes in behavior are early signs of change in status.
nurse look for in the older adult client who presents "atypical" signs of infection in older adults. Fever, erythema, edema, and leukocytosis may be
with an infection? present in varying degrees; however, these presentations are considered typical responses.
Coexisting chronic conditions along with the use of prescribed medications may cause typical
responses to be minimized or absent altogether in the elderly client.

1.‐ Fever
2.‐ Erythema and edema
3.‐ Behavioral changes and confusion
4.‐ Leukocytosis

483 A client has an unexplained weight loss of more than Correct answer: 4 Any client who presents with unexplained weight loss and persistent nonspecific complaints This question requires knowledge about the importance of HIV screening.
10 percent of ideal body weight (IBW) and voices of fatigue and nausea should be evaluated with regard to HIV status. Testing measures are not
nonspecific complaints of fatigue and nausea over the always conclusive and it is not apparent from the client's statement exactly what specific tests
last 6‐month period. The nurse should place the were administered. Low‐grade fever does not correlate directly with the presence of HIV.
highest priority on further assessing the client when Vitamin supplements could be considered to be supportive and protective. A history of blood
the client makes which of the following statements? transfusion may prove to warrant further assessment but it is not the highest priority at the
present time.
1.‐ "I have had a low‐grade fever for the past week."
2.‐ "I had a blood transfusion several years ago after having surgery."
3.‐ "I have been taking vitamin supplements on a daily basis for the last 2 weeks."
4.‐ "I have been tested for HIV in the past, but the results were negative."

484 While obtaining a review of systems the client Correct answer: 1 Type 1 hypersensitivity involves humorally mediated antigen‐antibody reactions. Food This question requires knowledge about hypersensitivity reactions.
informs you that he is "highly allergic" to many food allergies and medications can provide a localized as well as systemic response. Clients who
items and medications. You conclude that which have a history of multiple allergies usually have high IgE levels that are a characteristic
hypersensitivity reaction would be responsible for this measure of this type of reaction. The other hypersensitivity reactions do not apply to this
type of clinical presentation? characteristic presentation.
1.‐ Type 1, IgE mediated hypersensitivity
2.‐ Type 2, cytotoxic hypersensitivity
3.‐ Type 3, immune complex‐mediated hypersensitivity
4.‐ Type 4, delayed hypersensitivity

485 A client who receives a positive antinuclear antibody Correct answer: 2 Antinuclear antibodies indicate the presence of an autoimmune disorder. They are not This question requires knowledge about ANA testing.
(ANA) test result with a titer level &gt; 1:40 does not considered specific for systemic lupus, because many other autoimmune disorders have
understand what the test result means and asks the significant numbers of these antibodies. This reported titer is suggestive of the presence of
nurse for an explanation. Which of the following ANA antibodies, and therefore it is an abnormal response.
responses to the client would be most appropriate in
this situation?
1.‐ "The test result is normal."
2.‐ "The test indicates that you may have an autoimmune disorder, and this result should be discussed in more detail with your physician."
3.‐ "You should have the test repeated to verify its specificity for autoimmune disorders."
4.‐ "Your test result is specific for the detection of systemic lupus, and this should be discussed further with your physician.”

486 Which one of the following measures would be Correct answer: 4 Clients with a past medical history of anaphylaxis should have epinephrine readily available This question requires knowledge about anaphylaxis.
beneficial in helping a client with a past history of for emergencies because it is the drug of choice for treatment. Tylenol and ASA will not
anaphylaxis to develop a plan for handling possible mediate the chemical response to prevent anaphylaxis. Benadryl, although an antihistamine,
allergic reactions? may not be effective enough to prevent a full‐blown anaphylactic response.

1.‐ Have acetaminophen (Tylenol) readily available.


2.‐ Have acetylsalicylic acid (ASA) readily available.
3.‐ Have diphenhydramine (Benadryl) readily available.
4.‐ Have an EpiPen (Epinephrine) readily available.

487 Hydroxychloroquine (Plaquenil) is prescribed for a Correct answer: 2 Plaquenil is an antimalarial agent used in the treatment of rheumatoid arthritis. This This question requires knowledge about medication side effects.
client for the treatment of rheumatoid arthritis. The medication can cause retinal toxicity, and therefore the client should be closely monitored for
nurse would include which one of the following this possibility with specified visual exams. Gastric irritation, fluid retention, pulse elevations,
measures as part of client teaching with regard to this and drowsiness are not routinely seen with this type of medication.
medication?
1.‐ Take this medication on an empty stomach to minimize gastric irritation.
2.‐ Have an initial baseline eye exam performed and adhere to follow‐up exam schedule to monitor for potential ocular changes.
3.‐ Monitor weight and vital signs as the medication can cause fluid retention and pulse elevations.
4.‐ Be aware that medication can cause drowsiness and do not take it if planning to drive a car.

488 The nurse has conducted discharge teaching for a Correct answer: 4 Anticholinesterase medications are aimed at symptom management. These medications Recall that an empty stomach is often optimal to enhance medication absorption.
client diagnosed with myasthenia gravis. The nurse should be taken prior to eating to help the client chew and swallow and to minimize gastric
evaluates that the client understood the instructions upset. Taking this medication at night may not provide symptom relief and since absorption is
given with regard to the administration of variable, the client may not be assured of receiving the correct dose. The medication does not
anticholinesterase medication if the client takes the have to be taken with milk in order to minimize gastric upset. Taking the medication on a full
medication: stomach (which would constitute after eating) would not allow for the primary effect of aiding
with swallowing and chewing that is needed in clients who have this disease process.

1.‐ On a full stomach.


2.‐ Only at night.
3.‐ With 8 ounces of milk.
4.‐ 30 minutes prior to meals.

489 The nurse would assess for which one of the following Correct answer: 1 Raynaud's phenomenon is one of the most common findings associated with systemic This question requires knowledge about systemic sclerosis.
findings that is consistent with clinical manifestations sclerosis. Conjunctivitis, photophobia, and splenomegaly can all be seen in clients who
of systemic sclerosis? experience the effects of systemic lupus erythematosus.
1.‐ Raynaud's phenomenon
2.‐ Conjunctivitis
3.‐ Photophobia
4.‐ Splenomegaly

490 Which one of the following statements indicates the Correct answer: 3 The client's understanding is demonstrated by acknowledging the fact that sun exposure Use the process of elimination to answer this question.
client's understanding of measures used in the should be limited to times other than 10:00 A.M. to 3:00 P.M. (when the sun is at its highest
treatment of systemic lupus erythematosus? intensity). Tanning bed exposure can be considered to be an ultraviolet light trigger and could
exacerbate dermatologic presentations. Initial use of SPF 15 sunscreen (or higher value) is
indicated, as is the reapplication of sunscreen during exposure periods. Clients should avoid
exposure to potential infection.
1.‐ "I will be able to continue with my tanning bed appointments."
2.‐ "I can go visit this weekend with my grandmother who has been ill with a cold."
3.‐ "I can go for a walk on the beach after 3:00 P.M."
4.‐ "I have to apply SPF 10 sunscreen when I go to the beach."

491 Regardless of the type of isolation precautions that a Correct answer: 2 Regardless of isolation precautions, the basic action by the nurse to prevent infection is hand Recall that hand washing is generally a high priority.
client has been assigned, which of the following washing. All of the other options should also be followed but hand washing establishes the first
actions by the nurse should be given the highest line of defense and is therefore of highest importance.
priority in terms of infection control?
1.‐ Using strict aseptic technique
2.‐ Washing of hands before and after giving client care
3.‐ Checking sterile supplies for expiration date
4.‐ Changing intravenous tubing according to hospital policy

492 An HIV‐positive client now presents with a Correct answer: 4 CDC case definition of AIDS for adults states that the two factors described in the question This question requires knowledge about HIV clients.
CD&lt;sub&gt;4&lt;/sub&gt; count of &amp;lt; 200/ul are diagnostic of progression to AIDS. Seroconversion and positive HIV status has already
and invasive cervical cancer. How would the nurse occurred. The latent period is considered to be one in which the individual is asymptomatic.
evaluate these findings in terms of current CDC
definitions?
1.‐ The client has seroconverted.
2.‐ The client is HIV‐positive.
3.‐ The client is in the latent period of the disease process.
4.‐ The client has acquired immunodeficiency syndrome.

493 A client who has been diagnosed with an Correct answer: 3 A client diagnosed with an autoimmune disease is faced with a lifetime of chronic illness and This question requires knowledge about therapeutic communication.
autoimmune disorder questions the nurse as to what yet may not appear acutely ill because of the episodic nature of remissions and exacerbations.
impact this may have on activities of daily living in the The nurse promotes a therapeutic relationship by allowing the client to ventilate feelings. It is
years to come. The best explanation that you, as the inappropriate to minimize any changes that a client may experience that are unnoticeable to
nurse, can give is: others as they may be quite unsettling to the individual. It is not the role of the nurse to
speculate how a disease process will progress. Suggesting that the client use any "available
remedy" may lead the client to potential harm or medical quackery.

1.‐ "The changes will be subtle at first so it won't be noticeable to others."


2.‐ "It is hard to predict what the disease process has in store for any one individual."
3.‐ "I can hear the concern in your voice. Perhaps we can talk for awhile and discuss some of your concerns."
4.‐ "I would suggest the use of any available remedy that might give you some comfort."

494 Which of the following nursing diagnoses has the Correct answer: 2 Pain and pain control are the most important elements of care for a client who has Recall that pain management is a high priority.
highest priority for a client who has rheumatoid rheumatoid arthritis. Interventions aimed at pain management will allow the client to function
arthritis? at a more optimal level. While the other diagnoses are important, pain management remains
the critical factor.
1.‐ Fatigue
2.‐ Pain
3.‐ Ineffective role performance
4.‐ Disturbed body image

495 A client who has been diagnosed with scleroderma is Correct answer: 4 Raynaud's phenomenon is a common presentation in clients who have scleroderma. It is This question requires knowledge about scleroderma.
complaining of pain in his fingertips and pallor characterized as a vasospastic disease of the periphery that causes color changes ranging from
followed by blanching of the extremities and redness. pallor to reactive hyperemia. Joint swelling, effusion, and symmetric polyarthritis can be seen
The nurse communicates in intershift report that the in other autoimmune processes such as systemic lupus erythematosus and rheumatoid
client reports symptoms of which of the following arthritis.
disorders?

1.‐ Joint swelling and effusion


2.‐ Symmetric polyarthritis
3.‐ Swan‐neck deformity
4.‐ Raynaud's phenomenon

496 A client who is HIV‐positive and is taking antiretroviral Correct answer: 2 One of the most critical problems with regard to antiretroviral therapy is the emergence of This question requires knowledge about HIV and antiretroviral medications.
medications asks why he was told that a change in antiretroviral resistance as the HIV virus continues to mutate. Combination therapies have
medication might be expected during the course of been proven to be more effective in treating disease progression. Antiretroviral therapies, in
treatment. The best explanation to give the client is: proper dosage, do not cause specific organ toxicity although they can cause myelosuppression.

1.‐ "Antiretroviral medication regimens must be changed to prevent expected toxicity to major organs."
2.‐ "Antiretroviral resistance is a major challenge to long‐term management of HIV infection, and drug therapy may change based on research results."
3.‐ "Monotherapy is recommended for the treatment of HIV and must be adjusted."
4.‐ "Your treatment regimen will remain in place and is unlikely to change."
497 A client receives a polio vaccine during a clinic visit. Correct answer: 2 Vaccines are administered to the client to promote the development of specific antibodies to Use the rule of opposites to answer this question.
The nurse explains that this will provide what type of afford protection. This is an example of active artificial immunity. Active natural immunity
immunity to the client? implies the development of antibodies in response to a client who had an actual active
infection. Passive natural immunity implies the maternal and or placental transfer of
antibodies. Passive artificial immunity implies the specific injection of an immune serum.

1.‐ Active natural immunity


2.‐ Active artificial immunity
3.‐ Passive natural immunity
4.‐ Passive artificial immunity

498 A client who is diagnosed with myasthenia gravis Correct answer: 4 The client should be monitored for myasthenic crisis, which is often a result of missed or Consider the respiratory depression in MG as a priority.
(MG) had not been compliant with his medication under medication. The other options (gastrointestinal symptoms, vertigo, and bradycardia) are
regimen and has missed several doses of associated with cholinergic crisis. Cholinergic crisis is usually the result of overmedication. Both
pyridostigmine (Mestinon). For which complication complications are viewed as acute in nature and may require airway assistance. The nurse
would the nurse monitor? must be acutely aware of the potential for clients with MG to have these types of
complications.
1.‐ Gastrointestinal symptoms
2.‐ Vertigo
3.‐ Bradycardia
4.‐ Respiratory distress

499 A client with tuberculosis is being admitted to the Correct answer: 3 Airborne precautions should be instituted for all clients being admitted with a diagnosis of This question requires knowledge about isolation procedures.
medical‐surgical unit. Which type of precautions tuberculosis. Specific CDC guidelines may also be instituted to prevent TB transmission in
should the nurse institute to protect the client and healthcare facilities. Standard precautions should be maintained for all clients in the hospital
staff from possible exposure? setting. Contact and droplet precautions do not apply to this disease process.

1.‐ Standard precautions


2.‐ Contact precautions
3.‐ Airborne precautions
4.‐ Droplet precautions

500 You have been asked to perform a home assessment Correct answer: 3 Scattered area rugs are a potential safety hazard for an individual who has longstanding RA Use the process of elimination to answer this question.
on a client who has longstanding rheumatoid arthritis. because of possible joint deformities and contractures that could increase risk of falls. All of
Which one of the following findings should receive the the other assessment findings are considered to be supportive of this client with RA because
highest priority for follow‐up teaching? they enhance mobility, safety, and medication compliance.

1.‐ The client lives in an apartment building that has an elevator.


2.‐ The client has an installed handrail support in the bathroom.
3.‐ The client has area rugs scattered throughout the apartment.
4.‐ The client keeps her medications in a plastic case on the kitchen counter.

501 The nurse is performing an abdominal examination Correct answer: 3 The spleen is not usually palpated in an individual with normal immune function. Splenic This question requires knowledge about the physiological function of the spleen.
and is able to palpate the spleen. What information enlargement (splenomegaly) is associated with a deviation from normal and bears further
does this alert the nurse to suspect? investigation. Deep palpation is not indicated when splenic congestion is noted as it may cause
the spleen to rupture. Dehydration and allergic reaction are not consistent with enlargement
of the spleen.
1.‐ That the client has an allergic reaction.
2.‐ That the client is dehydrated.
3.‐ That the client may be at risk for immune dysfunction.
4.‐ That deep palpation is warranted to verify the size of the organ.
502 A client is breastfeeding her infant and asks the nurse Correct answer: 4 While breastfeeding does convey passive acquired immunity to the infant, it by no means Use the process of elimination and use care when answering questions which require
whether or not this method of feeding will protect the offers complete protection against any and all infectious processes. Immunizations with absolutes.
infant against infection. Based on the understanding of antigens (vaccines) will provide the infant with active artificial immunity and is more long
immunity, how would the nurse answer this question? lasting. In addition, the infant can and will receive "booster" doses to maintain immunity
status. The length of time or duration of breast‐feeding is not the sole determinant of an
infant’s passive immunity. The richest source of immunoglobulins is actually provided to the
infant during the transfer of colostrum, which is the precursor to actual breast milk.

1.‐ "Breastfeeding will protect your infant against all infection."


2.‐ "In order to protect your infant against any infection, it is necessary that your infant be given the complete schedule of immunizations."
3.‐ "Breastfeeding must be done for a minimum of 9 months in order to convey natural immunity."
4.‐ “Breastfeeding provides a short‐lived type of immunity, and your infant must be protected by other methods as well.”

503 The nurse working with a client who has an Correct answer: 4 The thymus gland is located in the superior mediastinum. In the child, the gland is usually This question requires knowledge about the thymus gland.
autoimmune disease explains that which of the quite large. As the individual ages, the gland shrinks in size. Initially, the thymus gland is
following occurs to the size of the thymus gland with responsible for T‐lymphocyte differentiation and maturation. The gland becomes a source of
increasing age? connective tissue, lymphocytes, and fibers in the older client.

1.‐ It remains the same.


2.‐ It enlarges due to fat deposition.
3.‐ It decreases once the individual reaches adolescence.
4.‐ It decreases with age.

504 The nurse conducting an in‐service presentation Correct answer: 2 The MPS helps to trigger or promote an immune response by capturing, processing, and This question requires knowledge about the listed processes.
about immune disorders to coworkers explains that presenting the antigen to the lymphocyte. The MPS contains monocytes and macrophages that
which of the following is one of the primary actions of participate in forming a bound antigen complex and presenting it to circulating lymphocytes to
the mononuclear phagocyte system (MPS)? elicit an immune response. While the MPS is a critical factor in the immune response, it does
not complete the immune response but rather serves as a pathway for the response to start
and progress. The MPS operates using the process of phagocytosis, whereby it engulfs the
antigen.
1.‐ It completes the immune response.
2.‐ It presents the antigens to the lymphocytes for recognition and processing.
3.‐ It operates utilizing the process of diffusion.
4.‐ It consists of erythrocytes, monocytes, and phagocytes.

505 The nurse looks for results of which of the following Correct answer: 2 CD&lt;sub&gt;4&lt;/sub&gt; cells are indicative of a client's HIV status. As the disease This question requires knowledge about HIV.
laboratory measurements of lymphocytes to provide a progresses, the T‐helper cells decrease in number and lose their ability to function effectively
reliable indicator of the immune response in a client leading to an overaggressive immune response. B‐lymphocytes indicate the status of humoral
with HIV infection? immunity and are not directly associated with HIV infection. NK cells and T‐cytotoxic cells are
not directly related to HIV infection and as such are not considered to be reliable indicators of
HIV status.
1.‐ B‐lymphocytes
2.‐ T‐helper cells (CD4)
3.‐ Natural killer cells (NK)
4.‐ T‐cytotoxic cells

506 The nurse is providing care to a group of clients. The Correct answer: 4 Transfusion and Goodpasture's are examples of Type II cytotoxic hypersensitivity reactions This question requires knowledge about immune responses.
client with which of the following problems exhibits a and are involved with the activation of complement. Lupus is an example of a Type III
Type III immune‐complex mediated hypersensitivity hypersensitivity reaction which involves IgG and IgM with the activation of complement.
reaction?
1.‐ Transfusion reaction
2.‐ Goodpasture’s syndrome
3.‐ Transplant rejection
4.‐ Systemic lupus erythematosus

507 The client complains of loss of warmth in the foot Correct answer: 1 The assessment of the circulation or potential disruption of the circulation would be a priority Remeber the ABC rule: airway, breathing, and circulation to select the highest priority for
after a fall from the bed. The assessment technique in assessment of this client. Palpation to assess disruption of sensorimotor or bone integrity this client.
the nurse should perform following inspection of the would follow.
foot would be to:
1.‐ Palpate the pedal pulses.
2.‐ Palpate for sensation of touch.
3.‐ Palpate for any bony deformities.
4.‐ Percuss the bony prominence.

508 A client presents to the clinic with a chief complaint Correct answer: 3 Clients with gout will usually have elevated serum uric acid levels. Laboratory findings as well This question requires knowledge about gout.
of a swollen and painful great toe. The client states as physical assessment will confirm the diagnosis. The joint of the great toe is usually involved
that the clien'ts brother has it, and has the same in initial attacks of acute gouty arthritis. There are many other factors that will affect the
symptoms. The physician suspects gout. What specific results of hematocrit, serum calcium, and sodium levels. Erythrocyte sedimentation rate (ESR
laboratory test would the nurse expect to be ordered or sed rate) and white blood cell (WBC) counts will also be elevated in cases of gout.
for this client?
1.‐ Calcium
2.‐ Hematocrit
3.‐ Uric acid
4.‐ Sodium

509 A client with acquired immunodeficiency syndrome Correct answer: 3 With a diagnosis of neutropenia, the primary concern is to protect the client who is This question requires knowledge about neutropenia.
(AIDS) is being admitted to the hospital with a immunosuppressed from developing further infections. The immunosuppressed client should
diagnosis of neutropenia. The nurse determines that not be exposed to potential infection in a semi‐private room (option 1). Use of Standard
which of the following room assignments would be Precautions will prevent transmission of the virus (option 3). Contact precautions are
most appropriate for this client? unnecessary (option 4).

1.‐ The client should be placed in a semi‐private room with the implementation of Standard Precautions.
2.‐ The client should be placed in strict isolation to prevent the transmission of the virus.
3.‐ The client should be placed on reverse isolation to protect the client from the possibility of acquiring further infection.
4.‐ The client should be placed on contact precautions.

510 A client with rheumatoid arthritis is seeking diet Correct answer: 4 Celery juice, honey, and fresh fruit are considered to be "non‐effective" dietary foods for This question requires knowledge about the listed foods and rheumatoid arthritis
modifications to ease symptoms of this disease. Which clients with arthritis. Salmon is high in omega‐3 fatty acids. Omega‐3 fatty acids have proven to symptoms.
of the following food items would be beneficial in be of benefit in clients with heart disease and rheumatoid arthritis by reducing inflammation.
easing symptoms of rheumatoid arthritis?

1.‐ Celery juice


2.‐ Honey
3.‐ Fresh fruit
4.‐ Salmon

511 A client diagnosed with human immunodeficiency Correct answer: 3 Client identification of help from family support indicates that coping strategies have been Use the process of elimination to determine the correct answer to this question.
virus (HIV) 2 years ago comes to the clinic for a instituted and have allowed the client to adapt to the disease process. Refusal to discuss other
scheduled check‐up. Which of the following client matters does not reflect successful coping strategies or open communication. Scheduling
behaviors would indicate that the client is coping well requests relates to personal preference and time management, not coping.
with this disease process?

1.‐ The client focuses discussion on medication scheduling.


2.‐ The client requests that future visits be scheduled later in the afternoon as this time is not convenient.
3.‐ The client relates how helpful family support has been during this time frame.
4.‐ The client relates that reading no longer provides relaxation.

512 A 4‐year‐old child is receiving postoperative care for Correct answer: 1 There is great potential for alteration in bowel function (adynamic ileus) because of surgery The core issue of the question is knowledge that Wilms' tumor affects the kidney, and
surgical resection of a Wilms' tumor. In addition to and radiation to the abdominal area and the use of chemotherapeutic agents. This is an therefore principles related to care following abdominal surgery apply to this client. Use
urinary functioning, one of the most important intrarenal tumor, so neurological status and bone pain are not related manifestations. Activity nursing knowledge and the process of elimination to make a selection.
postoperative assessments is level would not be a specific assessment to make with this diagnosis.

1.‐ Bowel function.


2.‐ Neurological status.
3.‐ Presence of bone pain.
4.‐ Activity level.

513 A child with a brain tumor has shown symptoms of Correct answer: 2 Diabetes insipidus presents with symptoms of increased urinary output and very dilute urine. The core issue of the question is knowledge of diabetes insipidus and methods to assess
diabetes insipidus. A nursing assessment to monitor Urinary specific gravity will measure the concentration of the urine. Blood glucose and BUN are the status of this complication. Use nursing knowledge and the process of elimination to
this condition would include unrelated to the issue of the question. ACTH levels are not routinely monitored in any client. make a selection.

1.‐ Blood glucose levels.


2.‐ Urine specific gravity.
3.‐ Adrenocorticotropic hormone (ACTH) levels.
4.‐ Blood urea nitrogen (BUN) levels.

514 The nurse determines that the client with Ewing’s Correct answer: 1 Bone marrow suppression occurs with radiation therapy, which can lead to risk of infection The core issue of the question is knowledge of bone marrow suppression in a client
sarcoma understands instruction related to radiation when white blood cells are affected, bleeding when platelets are affected, and anemia when receiving radiation therapy. Use basic nursing knowledge and the process of elimination to
therapy when the client reports that side effects red blood cells are affected. Constipation and hemorrhagic cystitis occur after chemotherapy. make a selection.
include which of the following? If appetite is affected, it decreases rather than increases.
1.‐ An increased risk of infection
2.‐ Constipation
3.‐ An increased appetite
4.‐ Hemorrhagic cystitis

515 A 6‐month‐old infant is being treated for Correct answer: 1 Because infant kidneys do not concentrate urine as well as the kidneys of adults, urine The core issue of the question is the most reliable method of determining fluid balance in
neuroblastoma. Because of the chemotherapy, the volume and specific gravity may not indicate fluid volume as accurately as will daily weight. an infant that is not feeding well because of neuroblastoma. Use nursing knowledge of fluid
infant feeds poorly and vomits frequently. The nurse Weight loss can be directly tied to fluid loss. Hemoglobin and hematocrit could rise and fall balance measurement and the process of elimination to make a selection.
would use which of the following assessments to best because of hemodilution or hemoconcentration, depending on fluid status, but these levels
determine the child’s fluid status? would be indirect indicators with large changes in fluid status and therefore not specific fluid
balance measurements.
1.‐ Daily weight
2.‐ Urinary output
3.‐ Specific gravity of urine
4.‐ Hemoglobin and hematocrit

516 A child is being treated for acute lymphocytic Correct answer: 3 In leukemia, the WBCs that are present are immature and incapable of fighting infection. The core issue of the question is knowledge of an indicator of infection in a client who is
leukemia (ALL). The laboratory report shows a white Increases or decreases in the number of WBCs can be related to the disease process and immunosuppressed from leukemia. Recall that temperature and WBC counts are frequently
blood cell (WBC) count of treatment and not related to infection. The only value that indicates the child is infection‐free used as indicators of infection. Recall that in leukemia the WBCs are abnormal to choose
7,000/mm&lt;sup&gt;3&lt;/sup&gt;. The nursing care is the temperature. The use of proper handwashing technique is a measure or intervention the option related to temperature.
plan lists risk for infection as a priority nursing used to meet a goal but is not a goal itself.
diagnosis, and measures are being taken to reduce the
child’s exposure to infection. The nurse determines
that the plan has been successful when the

1.‐ Child’s WBC count goes up.


2.‐ Child’s WBC count goes down.
3.‐ Child’s temperature remains within normal range.
4.‐ Parents demonstrate good handwashing technique.

517 A child with neuroblastoma will be started on total Correct answer: 1 Only regular insulin is administered in solutions administered by the IV route. Monitoring The core issue of the question is knowledge of total parenteral nutrition as a means of
parenteral nutrition (TPN) because of cancer cachexia. blood glucose and I&amp;O is appropriate. The child is usually anorexic but will be allowed to providing nutritional support to a client with cancer. Use nursing knowledge about the uses
The nurse would question which of the following newly eat any food that appeals to him or her. of various preparations of insulin and the process of elimination to make a selection.
written physician orders?
1.‐ Add 10 units NPH insulin to the TPN solution
2.‐ Monitor blood glucose level every four hours
3.‐ Daily intake and output
4.‐ Regular diet

518 A child has been diagnosed with a brain tumor, but Correct answer: 3 When a client is lying flat, the blood flow to the brain is greater, increasing the intracranial Recall principles of gravity to answer this question. When a client lies flat, blood can
surgery cannot be scheduled for several days. The pressure. If the client sleeps in a semi‐Fowler’s position, less pressure will develop, which in accumulate to a greater extent in the cranium, resulting in vasodilation, increased pressure,
mother asks what she can do to ease her child’s turn should ease headaches. Excess liquids and blowing the nose could aggravate headache. and worsening headache. Placing the client’s head in an elevated position allows gravity to
headaches. The nurse suggests that the mother: Discouraging bowel movements will reduce straining but is not a helpful measure from a drain blood to the heart and thereby keeps intracranial pressure rises to a minimum.
gastrointestinal perspective.
1.‐ Help the child to drink plenty of liquids.
2.‐ Discourage the child from having a bowel movement.
3.‐ Encourage the child to sleep in a semi‐Fowler’s position.
4.‐ Encourage the child to blow the nose when headaches become severe.

519 A child is being treated with corticosteroids for acute Correct answer: 3 A person taking steroids may have increased blood pressure, increased appetite, and weight The core issue of the question is knowledge of adverse effects of combination agents,
lymphocytic leukemia (ALL). On a follow‐up visit, the gain. Alopecia is related to chemotherapeutic agents that may be used to treat the leukemia. specifically steroids in this case, needed to treat cancer in a child. Use nursing knowledge
pediatric home health nurse assesses for side effects and the process of elimination to make a selection.
of steroid use, including:
1.‐ Decreased blood pressure.
2.‐ Alopecia.
3.‐ Weight gain.
4.‐ Anorexia.

520 A client with squamous cell carcinoma of the lung Correct answer: 3 While all of the above are potential risks to clients with cancer depending on site, edema of The core issue of the question is knowledge of various oncological emergencies. Use
comes to the emergency department reporting the face and arms results from obstruction of blood flow, which is indicative of superior vena nursing knowledge about which body systems are affected by each and then use the
shortness of breath and respiratory difficulty. On cava syndrome. Spinal cord compression would give rise to neurological symptoms. SIADH process of elimination to make a selection.
assessment, the nurse notes generalized cyanosis and would result in general fluid overload, and sepsis would be noted by signs of infection.
edema of the face and arms. Based on this assessment,
the nurse suspects the client is probably experiencing
which of the following?

1.‐ Spinal cord compression


2.‐ Syndrome of inappropriate antidiuretic hormone (SIADH)
3.‐ Superior vena cava syndrome
4.‐ Sepsis

521 The nurse reads in the medical record that a client’s Correct answer: 2 T2 indicates a measurable tumor, N0 indicates no regional node involvement, and M0 The core issue of the question is knowledge of staging for solid tumors. Recall that for an
tumor is at stage T2, N0, M0. The nurse concludes that indicates no evidence of distant metastasis. Options 1, 3, and 4 are either partially or totally option to be correct, all of the parts of the option must be correct. Note the numeric zeros
this staging indicates which of the following about the incorrect. by the N and the M to choose the option that does not contain metastasis or lymph node
client’s status? involvement.
1.‐ There is an advanced tumor with metastasis.
2.‐ The client has a measurable tumor with no indication of metastasis or involvement of nodes.
3.‐ There is an advanced tumor with indication of involvement of lymph nodes but no indication of metastasis.
4.‐ The client has an advanced tumor with indication of metastasis but no indication of involvement of lymph nodes.
522 A client with lung cancer is admitted to the oncology Correct answer: 2 Radiation is palliative treatment for spinal cord compression to reduce the tumor size and The core issue of the question is the rationale for using radiation therapy in a client with
clinic to receive radiation therapy for treatment of relieve compression. Options 1, 3, and 4 are incorrect statements. spinal cord compression secondary to cancer. Recall that radiation therapy is often used as
spinal cord compression. The client’s spouse asks why a supplement to shrink tumors to aid in making a selection.
radiation is being done. The nurse’s response would
include that radiation therapy is used:

1.‐ To eradicate the tumor.


2.‐ To reduce size of the tumor.
3.‐ In the treatment of all oncological emergencies.
4.‐ Instead of chemotherapy to treat the lung cancer.

523 A client with esophageal cancer arrives in the Correct answer: 2 Oxygen and IV access are immediate interventions for the client with cardiac tamponade. The stem of the question indicates that the client has an ineffective airway (shortness of
emergency department with shortness of breath, Vasopressor agents will be administered to manage hypotension (option 1); a breath and cyanosis). Look for the option that first addresses airway (oxygen) as the correct
tachycardia, hypotension, and cyanosis. The physician pericardiocentesis is performed, not a thoracentesis (option 3); and radiation therapy is not answer.
determines the client is experiencing cardiac indicated for cardiac tamponade (option 4).
tamponade. Which of the following interventions
would the nurse expect to include in this client’s care?

1.‐ Administer a vasodilator agent intravenously.


2.‐ Initiate oxygen and insert an intravenous catheter for IV access.
3.‐ Prepare to assist physician with a thoracentesis.
4.‐ Prepare the client for radiation therapy.

524 The nurse is discussing risk factors associated with Correct answer: 2 Prostate cancer has surpassed lung cancer in order of occurrence; colorectal cancer is the Specific knowledge of the risks associated with various cancers in men is needed to answer
cancer to a male client in the clinic. The client asks third‐most common cancer. Options 1, 3, and 4 are incorrect. the question. Use nursing knowledge related to epidemiology of cancer and the process of
which cancers have the highest incidence in men. In elimination to make a selection.
order of occurrence, the nurse would explain that the
client has greatest risk for which of the following
cancers based on gender?

1.‐ Lung, prostate, and colorectal cancers


2.‐ Prostate, lung, and colorectal cancers
3.‐ Colorectal, lung, and prostate cancers
4.‐ Prostate, colorectal, and lung cancers

525 A new nurse on the unit is admitting a severely Correct answer: 3 Because of the immunosuppression, the client is at severe risk of infection. Precautionary The core issue of the question is knowledge of room accommodations required by a client
leukopenic client who is receiving radiation therapy. measures such as a private room and protective isolation must be instituted to protect the who needs neutropenic precautions. Recall that infection is the risk and use process of
The preceptor determines that the new nurse client from sources of infection. The client with pneumonia (option 1) poses a risk of infection, elimination to make a selection.
understands precautionary measures necessary for contact isolation (option 2) is not necessary, and option 4 does not provide the client with the
this client when he or she admits this client to which of necessary isolation precautions.
the following rooms?
1.‐ A semiprivate room with a client who has pneumonia
2.‐ A private room with contact isolation
3.‐ A private room with protective isolation
4.‐ A private room with no isolation precautions

526 After completing a health risk assessment on a client, Correct answer: 4 Smoking and drinking large quantities of alcohol daily increase the risk of oral and esophageal The core issue of the question is a risk factor for laryngeal cancer. Note the word smoking
you determine health teaching and education is cancers. Options 1 and 2 are risk factors of development of other types of cancers. Option 3 is in one of the options and associate it with the word laryngeal (referring to airway) to make
necessary because of an increased risk for laryngeal unrelated. a word connection between the stem and the correct option.
cancer caused by which of the following risk factors?
1.‐ Past infection with Epstein‐Barr virus
2.‐ Past exposure to asbestos
3.‐ Smoking marijuana for recreational drug use
4.‐ Smoking cigarettes and consuming large quantities of alcohol daily

527 A client newly diagnosed with breast cancer is Correct answer: 3 The lymph node biopsy is performed to assess any metastasis from the primary site of cancer, The core issue of the question is knowledge that a biopsy procedure is used to diagnose a
scheduled for a lymph node biopsy and asks the nurse and a common metastatic site for breast cancer is regional lymph nodes. Options 1, 2, and 4 primary tumor or to evaluate lymph node involvement or metastasis. Use nursing
why it is necessary when a diagnosis of cancer has are incorrect statements. knowledge and the process of elimination to make a selection.
already been made. The nurse’s response is based on
which of the following?
1.‐ A lymph node biopsy is necessary to determine what types of cancer cells are present.
2.‐ A lymph node biopsy is performed on all females with cancer.
3.‐ A lymph node biopsy is performed to determine if cancer has metastasized.
4.‐ A lymph node biopsy is performed to determine what type of chemotherapy is indicated.

528 A 65‐year‐old postmenopausal client tells the nurse Correct answer: 2 The nurse should be concerned because painless bleeding not related to the menstrual cycle The core issue of the question is the significance of painless vaginal bleeding in a client
that she has recently experienced painless vaginal is often the only symptom of uterine cancer. Postmenopausal bleeding is not normal, with or who is postmenopausal. Use nursing knowledge and the process of elimination to make a
bleeding. The nurse should: without pain. Anemia is not an immediate concern. Pain is often considered to be a late sign selection.
related to the diagnosis of cancer.
1.‐ Not be concerned because postmenopausal bleeding is normal.
2.‐ Be concerned because painless uterine bleeding is a sign of uterine cancer.
3.‐ Be concerned because the client may develop anemia.
4.‐ Not be concerned because the client does not complain of pain.

529 A client is brought to the surgical unit after a Correct answer: 2 A very dark red output character following prostatectomy may indicate venous bleeding or The core issues of the question are interpreting the significance of dark red urine flow
suprapubic prostatectomy. He has a three‐way Foley inadequate dilution of the urine. The Foley catheter is at risk for occlusion. Increasing the following prostatectomy and the ability to choose a corrective action. Interpret that the
catheter. The nurse notices a very dark red output via irrigation flow will prevent the formation of blood clots and occlusion of the catheter. If the dark color is due to clots and then select the answer on the basis of which intervention will
the catheter. What is the appropriate nursing urine does not clear, then it would be appropriate to notify the physician. Although reviewing reduce clot formation.
intervention? the latest hemoglobin and hematocrit may be appropriate, it is not the most pressing
intervention the nurse must do following prostate surgery.

1.‐ Report the finding immediately to the physician.


2.‐ Increase the irrigation flow rate.
3.‐ Check the latest hemoglobin and hematocrit count.
4.‐ Chart the observation in the medical record.

530 The nurse who is screening female clients for cancer Correct answer: 3 Ovarian cancer generally causes no warning signs or symptoms in the early stages, which is The core issue of the question is knowledge of early signs of ovarian cancer. Use nursing
anticipates which of the following in relation to the why screening is important. Painful urination, pelvic pain radiating to the thighs, and low back knowledge and the process of elimination to make a selection.
early signs of ovarian cancer? pain are not associated with this health problem.
1.‐ Painful urination is a common complaint.
2.‐ Pelvic pain radiating to thighs may occur.
3.‐ Usually no symptoms are seen.
4.‐ Low back pain is a common complaint.

531 A client has stage II ovarian cancer documented as a Correct answer: 2 Option 2 describes stage II ovarian cancer. Option 1 describes stage I, option 3 describes stage The core issue of the question is knowledge of the staging system for ovarian cancer. Use
diagnosis on the medical record. The nurse plans care III, and option 4 describes stage IV. nursing knowledge and the process of elimination to make a selection.
based on which characteristic of this tumor at this
stage?
1.‐ Tumor growth is limited to the ovaries.
2.‐ Tumor growth involves one or both ovaries, with pelvic extension.
3.‐ Tumor growth involves the ovaries and peritoneum, with positive lymph nodes.
4.‐ Tumor growth involves distant metastasis.
532 A client is scheduled for a radical mastectomy. When Correct answer: 2 Option 2 describes a radical mastectomy. Option 1 describes a modified radical mastectomy; The core issue of the question is the ability to discriminate among various types of
reinforcing the surgeon’s explanation of the option 3 is a simple mastectomy; and option 4 is a lumpectomy. mastectomy procedures. Use nursing knowledge and the process of elimination to make a
procedure, the nurse would include that the surgery selection.
involves which of the following?
1.‐ Removal of breast tissue and lymph nodes under the arm
2.‐ Removal of the entire breast, underlying chest muscles, and lymph nodes
3.‐ Removal of the complete breast only
4.‐ Removal of the tumor and surrounding tissues

533 A client returns to the medical unit following Correct answer: 1 Clients with three‐way Foley catheters usually complain of sensations of having to void The core issue of the question is knowledge of the significance of bladder spasms following
transurethral resection of the prostate (TURP). The despite the presence of the catheter. This urge to void is caused by the pressure exerted by the prostate surgery. Recall that spasms are not expected findings to help narrow the plausible
client states to the nurse that he wants the three‐way balloon in the internal sphincter of the bladder and the wide diameter of the catheter that is options. Then use nursing knowledge and the process of elimination to make a selection.
Foley catheter to be removed because it is causing him used for the purpose of irrigation. Antispasmodics may be prescribed for the client with a
to have bladder spasms. Which of the following is the three‐way irrigation catheter. A TURP involves the insertion of a resectoscope via the urethra.
best response by the nurse? The complaint of having the urge to void is common with clients undergoing bladder irrigation.
Local reactions to the catheter usually do not include bladder spasms.

1.‐ “Spasms are painful but expected because of the wide diameter of the catheter and the retention balloon in the bladder.”
2.‐ “This must be a complication because the Foley catheter is supposed to evacuate clots that cause the spasms.”
3.‐ “The spasm is an unexpected finding because the procedure does not invade the urethra.”
4.‐ “Foley catheters in general have the tendency to cause bladder spasms because of the silicone used in the catheter.”

534 The client who underwent prostate surgery is Correct answer: 4 The healing period after prostate surgery is 4 to 8 weeks, and the client should avoid The core issue of the question is knowledge of postdischarge care to a client following
approaching the time of discharge from the hospital. strenuous activity during this period. Blood in the urine is fairly common after surgery. prostate surgery. Use nursing knowledge and the process of elimination to make a
Which of the following instructions should the nurse Continued increased fluid intake will help the urine to remain dilute and reduce the risk of clot selection.
provide to this client as part of discharge teaching? formation. The client should not drive for 2 weeks, except for short rides.

1.‐ “Reduce your fluid intake so you won’t need to void as often.”
2.‐ “Call the doctor immediately if you notice blood in your urine.”
3.‐ “You may drive yourself home.”
4.‐ “Avoid strenuous activity and heavy lifting for 4 to 8 weeks.”

535 The nurse would write which of the following Correct answer: 4 The arm should be elevated above heart level following mastectomy to reduce the risk of The core issue of the question is knowledge that edema is a risk following mastectomy and
interventions in the care plan of a client following edema after lymph node removal on the affected side. Warm, moist compresses could of nursing measures that can reduce this risk. Use nursing knowledge and the process of
mastectomy for breast cancer? enhance edema formation, and IV lines should not be used on the affected side at any location elimination to make a selection.
(lab draws and injections and blood pressure readings should also be avoided). Gentle, simple
range of motion exercises can be started immediately after surgery.

1.‐ Use warm, moist compresses on affected arm for comfort.


2.‐ Start intravenous lines on the affected side only in the antecubital area.
3.‐ Wait for the third postoperative day to begin any arm exercises.
4.‐ Keep the affected arm elevated above heart level.

536 A client has a continuous bladder irrigation running Correct answer: 700 The total infused is 600 + 1500 = 2,100 mL. The total drained was 800 + 1050 + 950 = 2,800 The principles for intake and output calculation are the same as for any other client. Tally
after prostatectomy. During the shift, 600 mL of one mL. Subtract 2,100 from 2,800 to obtain 700 mL, the true urine output for the shift. first the intake, then the output, and subtract the difference to determine how much of the
bag of irrigant has infused and 1,500 mL of the next output is actually because of urinary drainage. Use knowledge of basic nursing procedures
has also infused. Upon draining the urine bag three to calculate the answer.
times during the shift, the nurse measures the volumes
to be 800 mL, 1,050 mL, and 950 mL. The client’s true
urine output is _____ mL. Write in a numerical answer.
537 A child is admitted to the hospital with a diagnosis of Correct answer: 2 The low red blood cell count will limit the ability of the blood to carry enough oxygen to meet Option 3 is not related to blood values and can be eliminated. Option 4 is not a reflection
leukemia. Presenting lab values show low numbers of tissue needs, making risk for impaired gas exchange the correct diagnosis. There is no of the lab values. Choose between options 1 and 2. Realize leukemia would cause the high
platelets and red blood cells. A very high white blood indication that the child is at risk for injury. The high WBC count does not indicate an infection white count and not infection. In addition, infection is a medical diagnosis, not a nursing
cell (WBC) count is also noted. The nurse formulates is present but is an indication of the disease process. Since the WBCs are immature, they diagnosis.
which of the following as an appropriate nursing would be unable to fight an infection appropriately. The deficiencies are not related to
diagnosis? inadequate nutritional intake. The volume of blood is adequate; rather, it is the cell count that
is abnormal.
1.‐ Risk for injury
2.‐ Risk for impaired gas exchange
3.‐ Imbalanced nutrition: Less than body requirements
4.‐ Deficient fluid volume

538 A child with leukemia develops oral stomatitis Correct answer: 1 Stomatitis may cause the child to refuse fluids and foods. Hydration status would be an Oral stomatitis is mouth ulcers which are very painful. The child will refuse liquids. The
secondary to chemotherapy treatments. Nursing appropriate assessment to monitor the child's condition. Vitamin C intake would be important only other possible option is handwashing and that is not an assessment.
assessments related to this condition should focus on: in healing the mucous membranes but not as important as hydration. The condition of the
teeth and handwashing techniques are not involved in stomatitis.

1.‐ Hydration status.


2.‐ Vitamin C intake.
3.‐ Condition of teeth.
4.‐ Handwashing techniques.

539 A nurse is administering a chemotherapy drug to a Correct answer: 1 The chemotherapy agent poses a risk to all individuals including the child who was to have Option 4 can be eliminated as all of the other options indicate concern over cleanup.
patient via a central line when the intravenous bag received the drug. Removal of individuals from the area reduces the risk of inadvertent Option 3 can be eliminated as that would expose the water system to the chemotherapy.
slips and breaks open, spilling the chemotherapy agent exposure. Care is taken to avoid inhalation of the fumes, but the procedure would involve
on the tile floor. The nurse should: activities to avoid aeration of the chemical not speed in cleanup. Disposable cleanup materials
are included in a spill kit.
1.‐ Remove all people from the room until the spill has been cleaned up.
2.‐ Clean up the spill quickly to avoid inhaling fumes from the chemotherapy agent.
3.‐ Use only material that can be washed to clean up the spill.
4.‐ Call housekeeping immediately then proceed with routine nursing care.

540 The mother of a child receiving chemotherapy asks Correct answer: 1 Using multiple chemotherapeutic agents with different modes of action allows for the Knowledge of the rationale underlying chemotherapy induction will help to choose the
the nurse why they are giving the child more than one greatest amount of cell destruction. Using multiple drugs does not prevent renal damage, correct answer. Recognize that options 2, 3, and 4 are not reasonable.
drug at a time. The mother states this is why her child reduce nausea, or allow for efficient use of nursing time.
is so sick and it would be better to give the series one
drug at a time. The nurse explains to the mother that
using protocols of combination drugs:

1.‐ Allows for a better kill of cancer cells.


2.‐ Prevents renal damage.
3.‐ Helps the child to get over the nausea faster.
4.‐ Is a more efficient use of the nurse's time.

541 The surgeon has instructed the nurse to discuss the Correct answer: 4 The access device illustrated is a tunneled central venous access device. The tunneled device Recognize this is a tunneled catheter as the “tunnel” can be seen on the client’s skin.
care of the venous access device (shown) to the can last for years and once implanted, does not require puncturing the skin for access.
parents of a child who will be receiving chemotherapy
for at least two years. The nurse provides the parents
information related to a:
1.‐ Peripheral catheter.
2.‐ Peripheral subcutaneous port.
3.‐ Subcutaneous infusion port.
4.‐ Tunneled catheter.

542 A child with neoplastic disease is in terminal Correct answer: 3 There is no maximum dose of morphine. Increasing the dose in small increments is Recognize that because of tolerance, dosing of narcotics will need to be increased
condition. The child is receiving a large dose of appropriate to control the pain. Addiction is not an issue as the child is terminal. Stopping the periodically.
morphine by continuous infusion for pain control. infusion will put the child into withdrawal, which could be fatal. The physician is not misusing a
After the mother and nurse both reported to the narcotic.
physician that the child continues to be in severe pain,
the physician has ordered a small increase in the
morphine dosage. The nurse should:

1.‐ Refuse to administer the drug because the child may develop tolerance to the dose.
2.‐ Stop the infusion until another physician can check the order.
3.‐ Increase the dose as prescribed.
4.‐ Maintain the drug at the previous level and report the doctor for drug misuse.

543 A 3‐year‐old girl is undergoing radiation therapy for a Correct answer: 1, 5 Because of the danger of radiation, the child will be alone in the therapy room. Staff or Considering the child’s age, option 4 is inappropriate. In radiation therapy, nurses and
neoplasm. She states she is afraid of the large parents cannot stay with the child. A 3‐year‐old is not old enough to use mental imaging when parents should not be exposed to the radiation.
machines in the radiation department. To calm the frightened. A favorite stuffed toy can provide comfort as well as the calming voice of the
child during the procedure, the nurse could do which parent over the intercom system in the room.
of the following? Select all that apply.

1.‐ Allow the child to keep a favorite stuffed animal with her during the therapy
2.‐ Stay with her during the therapy
3.‐ Allow a parent to stay with her during therapy
4.‐ Encourage her to think about a favorite place during the therapy
5.‐ Allow the parents to talk with the child over the intercom system

544 In planning care for a 4‐year‐old with anemia Correct answer: 3 Assessing this child will give baseline data to plan and evaluate care. Activity intolerance will The core concept is anemia and nursing interventions. Think of energy conservation when
secondary to chemotherapy and radiation therapy, the be likely, so rest is important. This child is also too young to make choices about planning a stem describes anemia.
nurse would include: schedules and is likely to choose favorite foods that do not meet his nutritional needs. The
caregiver should be educated in planning activities and making food choices that include soft,
non‐spicy foods that are high in iron and protein.

1.‐ Frequent play periods.


2.‐ A diet focused on food preferences.
3.‐ Educating parents about the child's nutritional and activity needs.
4.‐ Allowing the child to plan daily activities.

545 Appropriate interventions for a 2‐year‐old with Correct answer: 1 Using antiemetics regularly will help to manage nausea and vomiting. Foods that have strong Knowledge of the administration of antiemetic drugs to lessen nausea from chemotherapy
nausea and vomiting related to chemotherapy would odors will increase nausea and vomiting. Only small amounts of fluids should be offered. will aid in choosing the correct answer. Option 4 is unrelated to nausea. Options 2 and 3 are
include: During periods of illness, children often regress to a "safer" period and regressive behaviors inappropriate activities in the care of nausea.
should be allowed if they comfort the child.
1.‐ Giving antiemetics at the start of treatment and on a fixed schedule rather than on demand.
2.‐ Offering fluids in large amounts to combat dehydration.
3.‐ Using foods with strong odors to stimulate appetite.
4.‐ Assessing ability to drink independently.

546 Nursing considerations for hemorrhagic cystitis, Correct answer: 2, 5 One of the side effects of cyclophosphamide is hemorrhagic cystitis not anaphylaxis. Consider which option will lessen cellular exposure to the chemical.
which may occur with the use of cyclophosphamide Appropriate interventions include using Mesna to counteract the irritating nature of Cytoxan,
(Cytoxan), include: (Select all that apply.) forcing fluids, and having the client empty the bladder frequently.
1.‐ The use of antibiotics.
2.‐ Emptying the bladder frequently.
3.‐ Restricting fluids.
4.‐ Planning for anaphylaxis.
5.‐ Encouraging increased fluid intake.

547 A 6‐year‐old child is being admitted for surgical Correct answer: 2 Vomiting is a symptom of increased intracranial pressure. Bulging fontanels would not be Consider two items in answering the question: normal growth and development of the
removal of a brain tumor. The nurse anticipates that present in a school‐age child. Drainage from the ear or nose might indicate a basilar skull school‐age child, and typical symptoms of a brain tumor.
which of the following nursing assessment data will be fracture, not a brain tumor. Some brain tumors display the symptom of diabetes insipidus, not
present during the preoperative period? diabetes mellitus, thus the symptom would be dilute urine rather than elevated blood glucose.

1.‐ Bulging fontanels


2.‐ Vomiting
3.‐ Drainage from the ear or nose
4.‐ Elevated blood glucose levels

548 A child is receiving chemotherapy to induce remission Correct answer: 2, 5 Nausea and vomiting, anorexia, mouth sores, constipation, and pain are early and common The core concept is early side effects of chemotherapy. Recall that chemotherapy drugs
in acute leukemia. When considering common side side effects of chemotherapy. Bone marrow suppression reaches its peak 7 to 10 days after kill rapidly growing and dividing cells to help select the options that pertain to rapidly
effects of chemotherapy, the nurse would write which induction. Sleep disturbance may be related but is not directly caused by chemotherapy. growing cells that line the gastrointestinal tract.
of the following as appropriate nursing diagnoses early Peripheral tissue perfusion is not related to the question.
in the course of therapy? (Select all that apply.)

1.‐ Disturbed sleep pattern


2.‐ Impaired oral mucous membranes
3.‐ Risk for infection
4.‐ Risk for impaired tissue perfusion: peripheral
5.‐ Imbalanced nutrition: less than body requirements related to nausea and vomiting

549 A client is to begin radiation therapy after the Correct answer: 3 Self‐care during external radiation therapy includes loose‐fitting clothes, gentle washing with The critical words in the question are “lack of understanding.” Eliminate those responses
removal of Wilms’ tumor. The parent statement that mild soap, avoiding sun exposure, and avoiding scratching and other irritation. Any lubricant that would be appropriate skin care for a radiation patient.
indicates to the nurse a lack of understanding of must be water‐soluble, not oil‐based such as petrolatum jelly.
related skin care would be “We will:

1.‐ Use loose‐fitting clothes on our child.”


2.‐ Protect our child from sun exposure.”
3.‐ Keep the area moist with petrolatum jelly.”
4.‐ Prevent our child from scratching the site.”

550 An adolescent receiving cyclophosphamide (Cytoxan) Correct answer: 4 Hematuria is an adverse effect of the commonly used cancer medication cyclophosphamide Options 2 and 3 are normal activities for all patients receiving chemotherapy. Determine
for acute lymphocytic leukemia (ALL) asks the nurse to (Cytoxan) and should be reported. Fluids are usually encouraged prior to administration, and the correct response from options 1 and 4 considering that hematuria is not a normal
come quickly to evaluate “blood in my urine.” The the bladder is emptied frequently to prevent hematuria. Measuring intake and output should finding.
most important action by the nurse would be to: be done routinely on all clients and is not specific to managing this complication.

1.‐ Explain this is normal for these drugs.


2.‐ Measure intake and output.
3.‐ Force fluids to improve the hematuria.
4.‐ Recognize that this is untoward and report the event.
551 A pediatric client who is known to have cancer is Correct answer: 3 A private room assignment is indicated for children with chemotherapy‐related neutropenia. This oral infection is spread by direct contact, so the infection itself would not prevent the
being admitted for mild neutropenia and a severe oral Careful handwashing is also an essential element to reduce the risk of infection. Because the child from sharing a room. However, the neutropenia would require the client not to share
monilial infection. The nurse should assign the child to neutropenia is mild at this time, the client does not require neutropenic precautions and does a room. Note the critical word “mild” to determine that neither neutropenic precautions
which room? not require full protective isolation. However, neutropenic precautions could be instituted nor protective isolation are needed at this time.
later if the client’s neutrophil count continues to decline.

1.‐ A semi‐private room with a medical patient


2.‐ A semi‐private room with a surgical patient
3.‐ A private room without further precautions
4.‐ A private room with protective isolation

552 The nurse is assigned to the postoperative care of a Correct answer: 4 Nursing care must be supportive of body image adjustment. The child would be encouraged To answer correctly, consider appropriate stump care and eliminate those options as
child with a below‐the‐knee amputation for osteogenic to sit in a chair and ambulate on crutches while waiting for the permanent prosthesis. The incorrect, leaving only the psychological response.
sarcoma. The nurse plans to include which of the stump dressing is a continuous ace bandage, which supports the stump shape in preparation
following in nursing care of the child? for the prosthesis.

1.‐ Maintain bedrest until able to use permanent prosthesis.


2.‐ Keep stump elevated continuously until prosthesis is applied.
3.‐ Apply a dressing to the stump that allows continuous visualization of the distal stump.
4.‐ Encourage early visits from friends.

553 The nursing diagnosis for a child undergoing Correct answer: 2 The client’s goal should be stated in terms of behaviors of the child that demonstrate the Consider which response best answers the problem of imbalanced nutrition. The correct
chemotherapy for leukemia is “Imbalanced nutrition: problem is solved. Option 1 is a nursing action, not a goal. Absence of nausea does not option would likely be addressing caloric intake or maintaining/gaining weight.
less than body requirements related to nausea and guarantee adequate intake. Equal intake and output indicates fluid balance but does not
anorexia.” The nurse would formulate which of the indicate adequate nutrition. Only the caloric intake adequately addresses the outcome needed
following as an appropriate goal for this client? by this client.

1.‐ Administer antiemetics PRN.


2.‐ The child’s caloric intake will be within normal range.
3.‐ The child does not complain of nausea.
4.‐ Intake and output are approximately equal.

554 A child is to receive chemotherapy with a vesicant Correct answer: 2 A vesicant drug can cause significant tissue damage if the IV line infiltrates. By checking for The core concept in this question is the vesicant property of the drug and safe
drug by the intravenous (IV) route. The nurse can blood return throughout the administration, the nurse can stop the infusion at any time a administration. Eliminate options 1 and 4 because they are excessive or extreme. Choose
ensure safe administration of this drug by doing which blood return does not occur. A positive pressure infusion pump, maintaining the infusion site option 2 over 3 because it directly addresses the issue of proper IV catheter placement in
of the following? below the level of the heart, or rapid drug delivery does not guarantee that the infusion does the vein.
not extravasate.
1.‐ Administering the drug using a positive pressure infusion pump
2.‐ Checking for blood return before, during, and after administration of the drug
3.‐ Maintaining the infusion site below the level of the heart
4.‐ Delivering the infusion as rapidly as possible

555 A child with leukemia has developed pancytopenia. Correct answer: 2, 4 Studies have shown that simply rinsing the mouth with water decreases the onset of Consider the pain of stomatitis to determine the appropriate activities. Choose the options
The nurse would institute which measures designed to stomatitis in chemotherapy patients. Alcohol‐based mouthwash would be avoided as it is that relieve discomfort or prevent irritation as the correct care measures.
reduce stomatitis in this child during the course of drying to the oral mucous membranes. A stiff toothbrush may cause the gums to bleed. Should
chemotherapy? (Select all that apply.) oral lesions be present, acidic foods and liquids will increase discomfort. Drinking through a
straw and pain management will provide comfort for the child, as will using swabs for mouth
care.
1.‐ Using alcohol‐based mouthwash to reduce oral organisms
2.‐ Administering pain medications as ordered
3.‐ Increasing intake of citrus juices, such as orange juice, that contain Vitamin C
4.‐ Rinsing the mouth several times a day with plain water
5.‐ Brushing the teeth twice a day with a firm‐bristled toothbrush
556 During rounds, the interdisciplinary team is discussing Correct answer: 4 The stages of grief and bereavement include denial, anger, bargaining, depression, and Most of the options include information not provided in the stem. Only option 4 is
a child with leukemia who has just been diagnosed as acceptance. The anger expressed may often be displaced and directed toward persons who complete with the stem information.
terminal. The nurses describe the mother’s behavior as have a role in the loss. Nurses and other healthcare personnel must be aware of this in order
angry, claiming the nurses are not providing care for to help the family cope with the impending loss.
her child. The team leader then focuses on which of
the following, which is most likely the cause of the
mother’s anger?

1.‐ Poor care on the part of the nurses


2.‐ Lack of attention for the mother’s needs
3.‐ Overwhelming guilt for having caused the leukemia
4.‐ A stage of bereavement over the anticipated loss of the child

557 An 18‐month‐old client is brought in for a well‐child Correct answer: 1, 3, 4 This is the usual presentation of Wilms’ tumor (nephroblastoma), and palpating the area may Consider what conditions are common in the abdomen of toddlers. All normal
visit. The parent reports feeling a lump to the right of cause the tumor to spread. Since Wilms’ tumor is a cancer of the kidney, it is important to assessments would be performed as well as assessments related to the possible condition.
the “bellybutton” during bathing. Initial assessments assess growth and development; kidney function; and blood pressure, which may be elevated Knowledge of Wilms’ tumor and the contraindication for palpating the abdomen will also
should include: (Select all that apply.) due to increased renin production. There is no evidence of abuse or rationale for performing a help to identify the correct answer.
neuro check.
1.‐ Measuring weight and height.
2.‐ Palpation of the area.
3.‐ Routine urine testing.
4.‐ Vital signs.
5.‐ Questioning the parents about abuse.

558 The parent of a child with neuroblastoma verbalizes Correct answer: 2 This tumor occurs in 1 in 10,000 live births. It arises out of embryonic neural crest cells and, Look for a therapeutic response that addresses the parent’s comment. Eliminate the
regret at not coming in earlier for the child’s therefore, is usually found in the adrenals or retroperitoneal sympathetic chain. Symptoms are answers that are not therapeutic and only convey information. Knowledge of
complaints. An appropriate response is: vague and depend on location. neuroblastoma and therapeutic communication will aid in choosing the correct answer.

1.‐ “This tumor may be diagnosed early because of obvious symptoms.”


2.‐ “This is a silent tumor, which is difficult to diagnose early.”
3.‐ “This is a very common brain tumor in children.”
4.‐ “I know you feel guilty about not being more observant, but you shouldn’t blame yourself.”

559 A 4‐year‐old is diagnosed with acute lymphocytic Correct answer: 4 Acute lymphocytic leukemia (ALL) is staged at diagnosis to determine treatment. The goal is The core concepts to correlate are leukemia tumor staging. Knowledge of ALL and the use
leukemia (ALL). Following teaching about the testing remission, which is usually accomplished using chemotherapy. of tests to diagnose the disease will aid in choosing the correct answer.
and therapy, the nurse evaluates the family’s
understanding of the problem. The statement by the
family that indicates appropriate knowledge would be:
“Tests will:

1.‐ Determine the extent of the tumor process and support the need for palliation.”
2.‐ Help determine if treatment is needed.”
3.‐ Determine if surgery is needed.”
4.‐ Determine the extent of the malignant process and stage the leukemia.”

560 The school health nurse has seen a child several times Correct answer: 3 The most common reported symptoms of brain tumors in children are headache, especially Remember that the earliest symptoms of brain tumors in school‐age children are often
with the same complaints. The school health nurse upon awakening, and vomiting that is unrelated to eating. Both are related to increased due to the inability of the skull to expand and select symptoms accordingly.
would suspect a brain tumor after noting the presence intracranial pressure. Irritability and ataxia may also be present; however, presenting
of which of the following symptoms that is compatible symptoms are often vague. Fever is not a symptom of a brain tumor. Papilledema may be
with this health problem? noted, but red reflex is not indicative of brain tumors.
1.‐ Ataxia and irritability
2.‐ Papilledema and positive red reflex
3.‐ Early morning headache and vomiting
4.‐ Fever and seizures

561 In Chapter 12, Pretest, question 5 should read: "A 17‐ Correct answer: 2 Bone tumors usually occur in otherwise healthy children. Given the interruption of normalcy The critical words in the stem of the question are “age‐related.” Body image is a concern
year‐old is being admitted for an amputation related and the developmental tasks of the adolescent, body image disturbance can occur when a limb for all adolescents even without physical problems. Knowledge of the clinical management
to a bone tumor. The nurse is developing a nursing is lost. of a bone tumor will also aid in identifying the correct answer.
care plan and determines the most appropriate age‐
related diagnosis is:
1.‐ Risk for disuse syndrome.
2.‐ Disturbed body image.
3.‐ Self‐care deficit.
4.‐ Activity intolerance.

562 A child has been treated with chemotherapy for Correct answer: 2 Neutropenia is a reduced white blood count, which increases the risk for infection. Only live Knowledge of the side effects of chemotherapy and how to manage the resultant
cancer. The nurse anticipates that neutropenia is an vaccines are contraindicated in children who are immunocompromised. Contact sports would neutropenia aids in answering the question correctly.
expected consequence and teaches the parents to: be a problem with thrombocytopenia and spicy foods would increase discomfort if an
alteration in mucous membranes occurred.
1.‐ Avoid contact sports.
2.‐ Avoid crowded spaces.
3.‐ Avoid spicy foods.
4.‐ Avoid all immunizations.

563 A child diagnosed with Ewing’s sarcoma is being Correct answer: 4, 5 Thrombocytopenia refers to a decrease in platelets. Preventing falls and bruises would be The learner will need to understand which blood cell is deficient in thrombocytopenia.
treated with chemotherapy. The results of a complete appropriate for an individual with platelet deficiencies. Fresh flowers may contain molds and
blood count (CBC) indicate severe thrombocytopenia. fungus that can lead to infection and would be a concern for a child with neutropenia.
Nursing interventions related to this finding would Providing foods high in iron would be appropriate to restore red blood cells. Limiting contact
include: (Select all that apply.) with the child could affect his or her body image and self‐esteem. Contact is acceptable as long
as the individual is not infectious.
1.‐ Encouraging foods high in iron.
2.‐ Limiting physical contact with the child.
3.‐ Removing fresh flowers from the child’s room.
4.‐ Clearing the floor of the child’s room to prevent falls and bruises.
5.‐ Minimizing needle sticks and intrusive procedures.

564 The parents of a child with neutropenia secondary to Correct answer: 3 Healthy children are often a source of infectious organisms. Children in hospitals may carry a Knowledge of neutropenia and the care of the child will aid in choosing the answer that
chemotherapy have been taught protective isolation number of infectious organisms. Hospitalized neutropenic children should be protected from indicates a need for further teaching of the parents. Consider the activity that would have
behaviors. Nursing observations that indicate a need exposure to other children whenever possible. Toys from home would not carry a high risk. the highest exposure to organisms.
for further education is when the parents: Handwashing before contact with the child is the important intervention. Limiting physical
contact with peers would decrease exposure to infectious organisms. Telephone contacts
allow for the peer support the child needs.

1.‐ Bring the child toys from home.


2.‐ Encourage friends to visit by phone rather than in person.
3.‐ Pull the child in a wagon around the nursing unit for entertainment.
4.‐ Wash their hands before entering the child’s room but not upon exiting the room.

565 Following diagnosis of Wilms’ tumor, the child Correct answer: 4 All of these assessments look at possible postoperative complications. Since the child is left Although respiratory is often the response in prioritizing assessments, in this specific case,
undergoes removal of the affected kidney. In the with only one kidney, failure of that kidney due to inadequate blood flow, infection, or any the learner should recognize the importance of the remaining kidney.
postoperative period, priority nursing assessments other cause could be fatal.
should focus on:
1.‐ The incision.
2.‐ Lung sounds.
3.‐ Temperature.
4.‐ Kidney function.

566 A child will be undergoing chemotherapy. The nurse Correct answer: 2 Preparation helps individuals handle stressful situations. If the child had not been prepared Knowledge of the affect of hair loss on the child and the need to prepare the child prior to
discusses the issue of hair loss with the child and for hair loss, it could be more anxiety‐provoking for the child. Hair loss cannot be prevented. this stressful event to facilitate coping will help to identify the correct answer.
family before chemotherapy begins. Later the family
asks the nurse why this information was given to the
child at this time. The nurse’s response will include the
information that:

1.‐ Hair loss is a symptom of toxic blood levels of chemotherapy, so the child should be watching for this phenomenon.
2.‐ The presence or absence of hair is related to body image. Strategies for handling hair loss should precede the event.
3.‐ It is the nurse’s legal responsibility to discuss this issue with the child.
4.‐ Hair loss can be prevented with appropriate nursing interventions.

567 Which of the following assessment parameters in the Correct answer: 4 As the immature white blood cells crowd the marrow, the ability for the marrow to perform This question requires knowledge of bleeding as a key complication.
client with leukemia is most important? the proliferation of red blood cells and platelets is also inhibited leading to a potential for
infection, occult blood, and hemorrhage. While the client may also experience constipation,
nausea, vomiting, anxiety, or depression, the potential for hemorrhage is most important.

1.‐ Anxiety and depression


2.‐ Constipation
3.‐ Nausea and vomiting
4.‐ Melena and petechiae

568 A client with hypercalcemia caused by malignant Correct answer: 3 Hypercalcemia produces a generalized slowing of functions through neuromuscular This question requires knowledge of malignant melanoma.
myeloma might exhibit which of the following? depression, i.e., constipation, increased urination, hyporeflexia, and confusion.

1.‐ Diarrhea
2.‐ Hyperreflexia
3.‐ Confusion
4.‐ Oliguria

569 An important consideration when administering Correct answer: 1 Gallium nitrate can impair renal function as evidenced by elevated urea and creatinine levels. This question requires knowledge of renal clearance.
gallium nitrate for a client with multiple myeloma is to: Hypotension, extravasation, and hyperglycemia are not side effects of gallium nitrate.

1.‐ Evaluate renal function studies.


2.‐ Monitor blood pressure.
3.‐ Use extravasation precautions.
4.‐ Assess for signs of hyperglycemia.

570 A client with cancer presents with a calcium of 12.5 Correct answer: 3 Although all the interventions help to decrease serum calcium, ambulating the client is the This question requires knowledge of methods to decrease calcium.
mg/dL. She is alert and oriented with good urine action that will most likely encourage calcium to return to the bone.
output. The most appropriate intervention to aid in
decreasing the serum calcium concentration is to:

1.‐ Perform active range of motion exercises.


2.‐ Order a footboard.
3.‐ Ambulate the client.
4.‐ Assist her to stand at the bedside.
571 A client with suspected Hodgkin's disease wants to Correct answer: 2 A lymph node biopsy is the only definitive means of establishing the diagnosis of Hodgkin's Recall that the biopsy is the definitive diagnostic tool.
know why a lymph node biopsy is necessary. The best disease. The presence of the Reed‐Sternberg cell is ascertained through this biopsy. Options 1,
response by the nurse is that the biopsy is: 3, and 4 are not necessarily true.

1.‐ The easiest test to confirm the diagnosis.


2.‐ The most definitive test to confirm the diagnosis.
3.‐ Less likely to cause major complications.
4.‐ The quickest way to find the correct answer.

572 A client presents with acute myelogenous leukemia Correct answer: 1 Preventing contact with contagious visitors decreases the client's risk of infection. The use of This question requires knowledge of neutropenic precautions.
(AML). Her white blood cell (WBC) count is 1,000/ aspirin or aspirin products as well as injections should be avoided in clients with
mm&lt;sub&gt;3&lt;/sub&gt;, hemoglobin is 8.0 g/dL, thrombocytopenia. The client's temperature should be monitored every 4 hours for signs and
hematocrit is 20 percent, and platelet count is 80,000/ symptoms of infection, not necessarily the respirations.
mm&lt;sub&gt;3&lt;/sub&gt;. Which of the following
interventions is appropriate?

1.‐ Prevent contact with visitors or personnel with colds


2.‐ Administer pain medicine with aspirin or aspirin‐containing products
3.‐ Keep the client pain‐free with intamuscular pain medications
4.‐ Monitor the client's respirations every 4 hours

573 A client admitted with newly diagnosed sickle cell Correct answer: 3 Vasospasm creates the log jam, which impedes blood flow that leads to clots, infarction, and This question requires knowledge of sickle cell crisis.
crisis asks the nurse why there is so much pain. The pain. The repeated sickling and unsickling causes the weakened cell membrane and hemolysis.
best explanation is which of the following? A clot can form (option 2) but is usually not in the deep vessels, but in the microcirculation.
Option 4 is incorrect because vasodilation does not occur.

1.‐ "Hemolyzed cells weaken the cell membrane."


2.‐ "Clotting of blood occurs in deep vessels."
3.‐ "Blood flow is hindered by vasospasm."
4.‐ "A log jam effect occurs due to vasodilation."

574 A client with hemophilia A asks the nurse whether the Correct answer: 3 Since hemophilia A is carried on the X‐chromosome, all of the female children will be carriers. This question requires knowledge of the transmission of hemophilia A.
disease can be passed on to his children. The nurse’s The disease will not affect a male child, unless he marries a woman who is also a carrier.
best response would be:
1.‐ All your male children will have the disease.
2.‐ Only your female children will have the disease.
3.‐ Your female children will be carriers of the disease.
4.‐ None of your children will be carriers of the disease.

575 The nurse doing health promotion in a client with Correct answer: 2 During decreased oxygen tension in the plasma, the hemoglobin S causes the red blood cells This question requires knowledge of impact of hypoxia on sickled cells.
sickle cell disease teaches the client to avoid situations to elongate, become rigid, and assume a crescent, sickled shape causing the cells to clump
that lead to sickle cell crisis, which are primarily together, obstruct capillary blood flow causing ischemia and possible tissue infarction.
instances when there is an inadequate amount of:

1.‐ Iron.
2.‐ Oxygen.
3.‐ Hemoglobin.
4.‐ Red blood cells.
576 For a client with iron‐deficiency anemia, adequate Correct answer: 4 Adequate iron stores are required to allow the oxygen molecule to attach to the red blood This question requires knowledge of the pathophysiology of anemia.
iron is a crucial part of hemoglobin because: cell. Small hemoglobin molecules insufficient in iron and oxygen are released into the
circulation resulting in the signs and symptoms of fatigue and shortness of breath.

1.‐ It cements the hemoglobin chain together.


2.‐ It facilitates the release of oxygen to the tissues.
3.‐ The heme molecule attaches to it to make a chain.
4.‐ The oxygen molecule attaches to it.

577 When severe neutropenia is present, the primary Correct answer: 2 In severe neutropenia, there is a decrease of mature white blood cells, which significantly This question requires knowledge of temperature as an early sign of infection.
symptom of infection for the nurse to assess may be: decreases the inflammatory response. Therefore, fever may be the first sign of infection in
these clients.
1.‐ Pus formation.
2.‐ Temp 101 degrees F.
3.‐ +2/4 edema.
4.‐ Local erythema.

578 The presence of Auer rods in the cytoplasm is Correct answer: 2 Auer rods may be present in the cytoplasm of the myeloblasts in AML. A standard diagnostic This question requires knowledge of the Auer rods.
primarily found in which type of leukemia? criterion for AML is that over 30 percent of hematopoietic cells must be myeloblasts.

1.‐ Acute lymphocytic leukemia (ALL)


2.‐ Acute myelogenous leukemia (AML)
3.‐ Chronic lymphocytic leukemia (CLL)
4.‐ Chronic myelogenous leukemia (CML)

579 The nurse prepares the client with suspected Correct answer: 2 Gastric secretion analysis in the client with pernicious anemia reveals an absence of free This question requires knowledge of pernicious anemia.
pernicious anemia for gastric analysis. This disorder hydrochloric acid in a pH maintained at 3.5.
would be confirmed by a subsequent laboratory report
that indicates which of the following?

1.‐ A high bile concentration


2.‐ An absence of hydrochloric acid
3.‐ A low bicarbonate concentration
4.‐ The presence of immature red blood cells

580 In a client with chronic myelogenous leukemia (CML), Correct answer: 1 Leukostasis occurs as the leukemic blast cells accumulate and invade the vessel walls, causing This question requires knowledge of leukemia.
the nurse should monitor for high white blood cell rupture and bleeding. Patients with extremely high circulating blasts (WBC &gt;
counts caused by the complications associated with 50,000/mm&lt;sub&gt;3&lt;/sub&gt;) are at increased risk for leukostasis.
leukostasis. The most common and most lethal
complication of leukostasis is:
1.‐ Intracerebral hemorrhage.
2.‐ Cerebellar toxicity.
3.‐ Blast crisis.
4.‐ Disseminated intravascular coagulation.

581 The nurse conducting health teaching for a client with Correct answer: 2 Tissue requires a certain degree of oxygenation to prevent feeling fatigued. When inadequate This question requires knowledge of the role of hypoxia with iron deficiency anemia.
iron‐deficiency anemia explains that activity oxygenation is available due to the pathophysiology of the disease, a supply and demand
intolerance is specifically related to: imbalance occurs. Iron‐deficiency anemia results in less iron available and less oxygen to
adhere to the hemoglobin molecule.
1.‐ Intravascular fluid volume loss.
2.‐ Oxygen supply and demand imbalance.
3.‐ Decreased systemic blood flow.
4.‐ Inadequate secondary defenses.

582 The nurse monitors a client with Hodgkin's disease for Correct answer: 4 Relentless proliferation of lymphocytes invade and compromise the function of various This question focuses on safety related to Hodgkin’s disease.
which of the following complications that could be organs, especially the bone marrow. Most clients with Hodgkin's disease exhibit signs of
fatal? immune deficiency early in the disease. Immunosuppressive therapy makes the client even
more susceptible to infection and hemorrhage because of a compromised bone marrow
function.
1.‐ Spinal cord compression
2.‐ Superior vena cava obstruction
3.‐ Kidney and/or liver failure
4.‐ Infection and/or hemorrhage

583 Multiple myeloma is characterized by which of the Correct answer: 4 Leukocyte proliferation is characteristic of lymphomas. Multiple myeloma is a neoplastic This question requires knowledge of multiple myeloma.
following? proliferation of the plasma cells.
1.‐ Increase in fibrin split products or fibrin degradation products
2.‐ Lymphocyte depletion
3.‐ Leukocyte proliferation
4.‐ Homogeneous immunoglobulin in the urine or serum

584 The nurse teaches a client that which of the following Correct answer: 4 Antacids coat the stomach and intestinal lining thereby inhibiting the absorption of iron Recall that antacids hamper the absorption of many medications by making the stomach
medications will decrease absorption of an iron supplements. Vitamin C increases absorption. Birth control pills and aspirin products have no alkaline. Note that this is a frequent NCLEX&lt;sup&gt;®&lt;/sup&gt; question.
supplement if taken simultaneously? effect on absorption.
1.‐ Birth control pills
2.‐ Ascorbic acid (Vitamin C)
3.‐ Acetylsalicylic acid (aspirin)
4.‐ Aluminum hydroxide (Mylanta)

585 Which of the following nursing diagnoses is most Correct answer: 2 The greatest risk affecting a client with hemophilia is the potential for hemorrhage. Although Consider the disease and the risk when establishing priority nursing diagnoses.
appropriate for the client with hemophilia? activity intolerance may also be a problem, in these clients it is caused by the injury potential,
not a decrease in functioning hemoglobin. Option 1 is appropriate for a client with pernicious
anemia and option 3 for a client with leukemia.

1.‐ Alteration in nutrition: less than body requirements related to a decreased absorption of Vitamin B12
2.‐ Risk for injury: hemorrhage related to the absence of factor VIII in circulatory system
3.‐ Risk for infection related to a decreased number of circulating neutrophils
4.‐ Activity intolerance related to decreased levels of oxygen in the circulation

586 A client has just been diagnosed with Hodgkin's Correct answer: 3 After a lymphangiogram, veins of the lower extremities, dorsal skin of the feet, and urine may This question requires knowledge of Hodgkin’s disease.
disease. Several hours after she returns to the room have a blue‐green discoloration from dye excretion for 2 to 5 days. The best response to the
after having a lymphangiogram, she tells the nurse her client's question is one that provides accurate information as well as reassuring the client.
urine is blue. The nurse's best response is:

1.‐ "That's OK. It will go away on its own."


2.‐ "I'll call the doctor, and he can explain the cause of the urine discoloration."
3.‐ "This is a predictable response to the test and shows you are adequately excreting the dye."
4.‐ "This is an expected side effect of the test. It will go away by itself in a few hours."

587 The nurse assesses for petechiae and ecchymosis as Correct answer: 4 Thrombocytopenia is a decrease in circulating platelets leading to a prolonged bleeding time Consider the condition most prone to bleeding.
the most common signs in a client diagnosed with and disruption of the primary homeostatic plug. While a Vitamin K deficiency can lead to a
which of the following disorders? prolonged prothrombin time, the patient does not exhibit petechiae and ecchymosis.
1.‐ Iron‐deficiency anemia
2.‐ Pernicious anemia
3.‐ Vitamin K deficiency
4.‐ Thrombocytopenia

588 A client is prescribed to take ferrous sulfate for iron‐ Correct answer: 1 Iron supplements are better absorbed if taken before meals; however, the side effect of This question requires knowledge of ferrous sulfate.
deficiency anemia and is experiencing nausea with the nausea may be experienced if taken on an empty stomach. Taking with meals may hinder
medications. The client should be instructed by the absorption, so orange juice or other foods containing Vitamin C may be taken with the
nurse to: medicine to help with absorption.
1.‐ Take the medication with meals.
2.‐ Take the medication before meals.
3.‐ Take the medication after meals.
4.‐ Take the medication between meals.

589 The nursing care plan for a client diagnosed with Correct answer: 3 ITP is an autoimmune disorder in which the body destroys platelets. In order to decrease the This question requires knowledge of ITP.
idiopathic thrombocytopenia purpura (ITP) takes into immune response, corticosteroids are usually administered. Platelet transfusions may be given
account which of the following therapies for this in acute bleeding, however the body will actively destroy these as well.
disorder?
1.‐ Alkylating chemotherapy
2.‐ Platelet transfusions
3.‐ Corticosteroid therapy
4.‐ Bone marrow transplant

590 The nurse providing health teaching to a client Correct answer: 1 Although COPD can be an underlying etiology for the formation of secondary polycythemia; it This question requires knowledge of compensatory mechanisms for hypoxia.
explains that the underlying cause of secondary is the chronic tissue hypoxia from the COPD that is the underlying cause of the condition. Not
polycythemia is which of the following? all patients with COPD develop the condition.
1.‐ Chronic tissue hypoxia
2.‐ A myeloproliferative disorder
3.‐ Depletion of erythropoiesis
4.‐ Chronic obstructive pulmonary disease (COPD)

591 A 67‐year‐old female client refuses to be screened for Correct answer: 2 Approximately 67% of all cancer occurs in people over age 65, necessitating early screening This question requires knowledge of the cancer screening methods.
cancer, stating, "I am too old to get cancer and if I and detection. The incidence of cancer increases with age, making it a significant factor in the
don't have it by now, I will never get it." Which of the development of cancer.
following would be the basis of your response to her?

1.‐ Unless symptomatic, screening the client for cancer is not necessary.
2.‐ Screening the elderly for cancer is essential.
3.‐ The incidence of cancer decreases with advancing age.
4.‐ Age is not a significant factor in the development of cancer.

592 A client just diagnosed with a benign neoplasm asks Correct answer: 1 Benign neoplasms are localized, encapsulated growths. They are not malignant, and they do This question requires knowledge of benign processes.
you if he is going to die. On which of the following not metastasize. They are harmful only if they interfere with vital functions such as circulation.
items of information will you base your response? Malignant neoplasms have a high mortality rate unless therapeutic interventions are
performed. The client's question is a normal, expected response.

1.‐ Benign neoplasms are localized, encapsulated growths that are easily removed and rarely cause death.
2.‐ Benign neoplasms are rapidly growing tumors that metastasize to other areas of the body.
3.‐ Benign neoplasms are associated with a high mortality rate.
4.‐ The client is experiencing an unusually high level of anxiety.
593 A client receiving intravenous chemotherapy is Correct answer: 3 Administering antiemetics before chemotherapy helps reduce the severity of nausea. Waiting This question requires knowledge of basic care concepts.
experiencing nausea. Which of the following would be until the client is experiencing nausea demonstrates lack of planning. Cool foods and liquids
the best intervention to lessen the severity of nausea? are better tolerated and are less irritating than warm foods and liquids. Small, frequent meals
are more easily tolerated and may reduce the incidence of nausea and vomiting.

1.‐ Administer antiemetics when client complains of nausea.


2.‐ Offer warm liquids during chemotherapy.
3.‐ Administer antiemetics before chemotherapy.
4.‐ Encourage the client to eat a full meal before receiving chemotherapy.

594 When teaching safety precautions to the client with Correct answer: 2 The client is a risk to others as long as the radiation implant is present. Therefore, certain This question requires knowledge of the means to protect the client and others.
an internal radiation implant, you would include which precautions to protect others must be taken. The client should have a private room, and
of the following in explanations to the client? visitors should maintain a distance of 6 feet and limit visits to 10 to 30 minutes. The client may
not need isolation for the entire period of hospitalization; rather just for the time the implant
is in place.
1.‐ No precautions are necessary for internal radiation implants.
2.‐ The client poses a risk of radiation exposure to others.
3.‐ The client must remain in solitary isolation for the entire hospitalization.
4.‐ Visitors should maintain a distance of 3 feet from the client at all times.

595 A nurse is educating a client who will likely experience Correct answer: 1 Washing the hair daily will promote further hair loss. Hair washing should be limited to 2 to 3 Use the process of elimination to determine the correct answer, in this case the incorrect
alopecia (hair loss) as a result of the current times per week. Options 2, 3, and 4 are correct actions taken by the client. option requiring further instruction.
chemotherapy treatment. Further instructions are
necessary when the client states which of the
following?
1.‐ "I will wash my hair every day."
2.‐ "I will pat my hair dry and avoid the use of hairdryers."
3.‐ "My hair will begin to grow back after the chemotherapy is completed."
4.‐ "I will choose a wig or hairpiece before the loss of hair occurs."

596 A client receiving chemotherapy is experiencing a low Correct answer: 3 The client with a low WBC count is at high risk for infection. The grandchild recently exposed Use the process of elimination to determine which poses the greatest risk.
white blood cell (WBC) count. The nurse should teach to varicella could be contagious at this point. The nephew with HIV, unless currently infected
the client to avoid contact with which of the following with another communicable disease, does not pose a risk. There is no indication that the
family members? husband has tuberculosis. The pregnant daughter does not pose a risk.

1.‐ 34‐year‐old nephew with HIV infection


2.‐ 68‐year‐old husband with a history of exposure to tuberculosis as a youth
3.‐ 9‐year‐old grandchild with a recent exposure to chicken‐pox (varicella)
4.‐ 31‐year‐old daughter who is 4 months pregnant

597 The nurse is counseling a 22‐year‐old female in the Correct answer: 1 The American Cancer Society recommends a breast examination by a healthcare provider This question requires knowledge of the correct screening procedures.
health clinic. The nurse determines that she every 3 years for ages 20 to 39, then yearly from age 40 and older. Breast self‐examinations
understands the teaching instructions for the early should be performed monthly. Mammograms are recommended yearly beginning at age 40.
detection and screening of breast cancer when she
makes which of the following statements?

1.‐ "I should have a breast examination by a healthcare provider every 3 years."
2.‐ "I should have a breast examination by a healthcare provider yearly."
3.‐ "I should perform a breast self‐examination every 3 months."
4.‐ "I should have a mammogram performed yearly."
598 A client is to receive intravenous chemotherapy via a Correct answer: 2 Extravasation of the chemotherapeutic agent, especially if the agent is a vesicant, is a major This question requires knowledge of the function of platelets.
peripherally inserted central catheter (PICC). The nurse complication of intravenous administration of chemotherapy. Never test vein patency with the
should plan to take which of the following essential medication. Making the client comfortable is important, but assuring vein patency is the
actions before beginning the administration? highest priority. There is no indication to administer acetominophen.

1.‐ Make the client as comfortable as possible.


2.‐ Ensure patency of vein.
3.‐ Flush the catheter with the medication to test patency of the vein.
4.‐ Administer Tylenol (acetominophen) prophylactically.

599 A client with a low platelet count demonstrates Correct answer: 1 The client with a low platelet count (thrombocytopenia) is at risk for bleeding. Aspirin further Recognize that this question asks about basic hygiene concepts.
understanding of instructions to avoid potential interferes with platelet functioning. Monitoring for fever (option 2) is necessary for low WBC
complications by doing which of the following? count, and managing fatigue (option 3) is necessary for anemia. Flossing is contraindicated in
the client with low platelet count (option 4).
1.‐ Avoiding aspirin and aspirin (salicylate)‐containing products
2.‐ Monitoring for fever every 4 hours
3.‐ Allowing for rest periods to avoid fatigue
4.‐ Brushing and flossing teeth daily to prevent infection

600 The nurse is evaluating the nutritional status of a Correct answer: 4 Damage to the mucous membranes, especially oral mucous membranes (stomatitis), leads to This question requires knowledge of the listed processes.
client who is receiving chemotherapy. On assessment, painful ulcerations of the mouth, interfering with the client's desire to eat. The mucous
which finding could potentially affect the client's membranes and mouth may be dry (xerostomia) as a side effect of the chemotherapy. Pale
nutritional intake? skin may be a sign of anemia, and ecchymosis may be indicative of a low platelet count.

1.‐ Pale and moist mucous membranes


2.‐ Pale skin
3.‐ Ecchymotic areas on forearms
4.‐ Ulcerations of oral mucosa

601 A client undergoing radiation therapy has a severely Correct answer: 1 The immunosuppressed client is at high risk for infection. A private room, maintaining aseptic This question requires knowledge of the function of WBCs.
depressed white blood cell (WBC) count. A priority technique, and limiting visitors will reduce exposure and risk. Fresh fruits and vegetables may
nursing intervention would include which of the harbor bacteria; serve cooked foods only. The client with a decreased platelet count should be
following? counseled to avoid using razors.
1.‐ Place client in a private room and maintain strict aseptic technique with all procedures.
2.‐ Encourage client to include fresh fruits and vegetables in the diet.
3.‐ Educate client to avoid shaving with a razor.
4.‐ Encourage frequent visitors to reduce client's feelings of isolation.

602 The assessment nurse is working in a local clinic. Correct answer: 1 Unexplained, rapid weight loss may be the first symptom associated with cancer, and Recognize that this question focuses on the significance of unintended weight loss.
Based on the history provided by the clients, which immediate evaluation is required. Options 2, 3, and 4 are risk factors associated with cancer,
one of the following requires an immediate referral for and education and screening are important to reduce the risk of cancer.
screening and evaluation?
1.‐ The client who reports an unintended weight loss of 25 pounds over the past 3 months
2.‐ The client who smokes 2 packages of cigarettes a day
3.‐ The client who reports a history of long‐term sun exposure
4.‐ The client who consumes a diet high in fats and low in fiber

603 The nurse is counseling a client on risk factors Correct answer: 4 Options 1, 2, and 3 are noncontrollable or nonmodifiable risk factors. Diet is the only listed Use the process of elimination to identify modifiable risk factors.
associated with cancer. For which of the following will risk factor that is controllable. Assisting the client to develop a diet plan low in fat and high in
the nurse assist the client in developing a plan to fiber will help reduce the risk of some types of cancer.
reduce the risk of cancer?
1.‐ Heredity
2.‐ Gender
3.‐ Age
4.‐ Diet

604 The nurse is making a home visit to a client receiving Correct answer: 3 Lotion, deodorant, and powders should not be applied to the radiation site during the Use the process of elimination to determine the correct answer.
external radiation therapy on an outpatient basis. treatment period to avoid further irritation to the skin. Options 1, 2, and 4 are correct actions.
Further teaching is necessary when the nurse observes
the client doing which of the following?

1.‐ Washing the radiation site with plain water and patting the skin dry
2.‐ Protecting the skin with soft, loose clothing
3.‐ Applying lotion to the irritated skin
4.‐ Inspecting the skin for damage

605 A hospitalized client with an internal radiation Correct answer: 2 Long‐handle forceps should be used to pick up the implant. Lead containers are necessary to This question requires knowledge of the handling of radiation implants.
implant calls the nurse to the room to report the prevent exposure to radiation. Direct handling of the implant causes exposure to radiation; no
implant is dislodged and is lying in the bed. The nurse’s one should directly touch the implant. Gloves and biohazard bags do not offer protection from
actions would include which of the following? radiation. Infection control personnel have no role in the disposal of the implant, which should
be returned to the radiation therapy department after properly being placed in the lead
container.
1.‐ Apply gloves and place the implant in a biohazard bag
2.‐ Use long‐handle forceps to pick up the implant and place it into a lead container
3.‐ Have the client pick up the implant and place it into a lead container
4.‐ Notify infection control personnel to dispose of the implant

606 A client with leukemia is undergoing the "conditioning Correct answer: 4 The conditioning phase depresses bone marrow function, and infection is the major cause of Recognize that this question asks about priorities related to infection, important with the
phase" for a bone marrow transplant (BMT). The death for clients with leukemia. Options 1, 2, and 3 are appropriate diagnoses for clients neutropenic client.
priority nursing diagnosis for this client is which of the receiving chemotherapy and radiation, but the risk for infection is the highest priority during
following? this phase.
1.‐ Fatigue related to anemia
2.‐ Imbalanced nutrition: less than body requirements
3.‐ Altered mucous membranes
4.‐ Risk for infection

607 The nurse has counseled a male 52‐year‐old client Correct answer: 1 The American Cancer Society recommends a digital rectal examination and PSA yearly for This question requires knowledge of the screening tests for cancer.
about early detection and screening for prostate males beginning at age 50. Options 2 and 3 are only partly correct, and option 4 is incorrect.
cancer. The nurse evaluates that the client has
understood instructions when he states which of the
following?
1.‐ "I should have a digital rectal examination and prostate‐specific antigen (PSA) test done yearly."
2.‐ "I should have a prostate‐specific antigen (PSA) test done yearly."
3.‐ "I should have a digital rectal examination done yearly."
4.‐ "I don't need a screening unless I develop symptoms."

608 A client who had a mastectomy yesterday refuses to Correct answer: 2 Denial is a protective mechanism, and during this time, the client needs a supportive Recall that allowing for client ventilation of feelings is an important strategy.
look at the incision. The nurse can best assist the client environment. Allowing the client to express feelings will enable an effective adaptation to this
to cope with the disturbed body image by doing which change. Option 1 is not therapeutic. Wound care must be done in order to prevent
of the following? complications (option 3), and the client is obviously not psychologically ready to participate in
self‐care (option 4).
1.‐ Tell the client that eventually everyone accepts the loss of a body part.
2.‐ Encourage the client to express feelings about loss of body part.
3.‐ Delay wound care until client is prepared to look at the wound.
4.‐ Have the client assist you with the dressing change.

609 In assessing a client receiving chemotherapy, which of Correct answer: 2 Options 1, 3, and 4 are common side effects of chemotherapy. Even though they do require Use the process of elimination to differentiate common and uncommon side effects.
the following would require further evaluation? intervention, ecchymotic areas may be a sign of decreased platelet count, making the risk of
hemorrhage the priority.
1.‐ Dry mucous membranes
2.‐ Large areas of ecchymosis in various sites on body
3.‐ Complaints of fatigue
4.‐ Hair loss on scalp

610 A client receiving external radiation expresses Correct answer: 2 External radiation poses no risk of radiation exposure to contacts, even during intimate This question requires knowledge of the care of a client having radiation therapy.
concern to the nurse about physical intimacy with physical contact. Clients are encouraged to maintain their usual activities, as long as they are
spouse. When offering sexual counseling to the client tolerated (option 3). There is no increase in risk of infection to the client with cancer during
and spouse, the nurse tells them which of the intimate physical contact unless that person has a current infection (then contact should be
following about intimate physical contact? avoided until infection is treated).
1.‐ It should be avoided during treatment to avoid radiation exposure to the spouse.
2.‐ It is safe during treatment; there is no risk of radiation exposure to the spouse.
3.‐ It should be avoided during treatment to conserve energy.
4.‐ It increases the risk of infection to the client with cancer.

611 A client with acute leukemia is admitted for a bone Correct answer: 2 Bone marrow is usually harvested from the iliac crest (option 1), frozen, then stored until This question requires knowledge of the care of the client having a bone marrow
marrow transplant. The nurse concludes that the client treatment. The marrow is administered intravenously through a central line. Options 3 and 4 transplant.
understands teaching based on which of the following are incorrect.
statements?
1.‐ "The bone marrow will be transplanted into my iliac crest."
2.‐ "The bone marrow is given to me through an intravenous line."
3.‐ "The bone marrow will be transplanted into my sternum."
4.‐ "The bone marrow is injected into my muscle."

612 The nurse is teaching a male client with cancer about Correct answer: 3 Even though anemia (option 1) and hemorrhage (option 2) also result from bone marrow Use the process of elimination to determine the greatest risk.
potential complications associated with bone marrow suppression, immunosuppression leading to an inability to fight infection is the priority to
suppression as a side effect of the treatment plan. The prevent complications potentially leading to death. Weight loss (option 4) may occur as a
nurse determines that the client understands the risk result of anorexia and can be managed.
of complications when he states that the most life‐
threatening complication is which of the following?

1.‐ Anemia
2.‐ Hemorrhage
3.‐ Infection
4.‐ Weight loss

613 A female client comes to the clinic for a cancer Correct answer: 1 For cancers in females, by order of occurrence, breast cancer ranks first, followed by lung and This question requires knowledge of the most common cancers.
screening and preventive education. When educating colorectal cancers. Options 2, 3, and 4 are incorrect.
the client, the nurse would include information about
which of the following cancers found in females, in
order of occurrence?
1.‐ Breast, lung, and colorectal cancers
2.‐ Lung, breast, and colorectal cancers
3.‐ Breast, cervical, and lung cancers
4.‐ Lung, breast, and cervical cancers
614 A client receiving chemotherapy is experiencing mild Correct answer: 1 Stomatitis is inflammation of the oral mucosa, a common side effect of chemotherapy. Use the process of elimination to outline mouth care.
stomatitis. Based on an understanding of this Management includes teaching client to use a soft toothbrush, rinse mouth with plain water or
complication, the nurse will teach the client which of saline, and to avoid irritants such as mouthwashes, peroxide, and hot liquids (options 2, 3, and
the following measures? 4).
1.‐ Brush teeth with a soft toothbrush before and after meals
2.‐ Rinse mouth with mouthwash before and after meals
3.‐ Rinse mouth with warm hydrogen peroxide twice daily
4.‐ Drink hot liquids regularly

615 The nurse is caring for a client receiving Correct answer: 2 Profound malnutrition can result from loss of appetite related to nausea and vomiting, and This question requires knowledge of the priorities associated with chemotherapy.
chemotherapy who is experiencing severe nausea, the concurrent stress of the body fighting cancer. Options 1, 3, and 4 do not completely
vomiting, and anorexia, and has not eaten in 2 days. address the issue of altered nutrition.
Which of the following is the priority nursing diagnosis
for this client?
1.‐ Self‐care deficit: feeding
2.‐ Imbalanced nutrition: less than body requirements
3.‐ Knowledge deficit: adequate nutrition
4.‐ Discomfort related to nausea and vomiting

616 The nurse is counseling a group of college students Correct answer: 3 Dietary habits that reduce the risk of cancer include consuming cruciferous vegetables such as This question requires knowledge of the foods thought to play a role in cancer prevention.
about healthy lifestyle choices to reduce the risk of cauliflower, broccoli, and cabbage (option 3) and high‐fiber foods (option 4), avoiding nitrates
cancer. The students demonstrate understanding of in prepared meats (option 1), and limiting intake of red meats. Pineapple and other tropical
instruction related to nutritional practices when they fruits offer no special protection against cancer (option 2).
identify that eating which of the following foods may
reduce the risk of cancer?

1.‐ Lean ham


2.‐ Pineapple and other tropical fruits
3.‐ Broccoli and cauliflower
4.‐ Low‐fiber foods

617 The nurse is caring for a client hospitalized to receive Correct answer: 2 The client receiving a bone marrow transplant must first undergo an immunosuppressive Use the process of elimination to determine the correct answer.
a bone marrow transplant. When establishing the plan phase before receiving donor marrow. This places the client at extreme high risk for infection.
of care, the nurse determines that the client is at Options 1, 3, and 4 are potential complications but are not immediately life‐threatening.
highest risk for which of the following?

1.‐ Anorexia
2.‐ Infection
3.‐ Nausea and vomiting
4.‐ Weight loss

618 An elderly male client undergoing treatment for Correct answer: 3 In the elderly and immunocompromised client, even a slight elevation of temperature may Use the process of elimination to determine the correct answer.
cancer is being cared for at home by the family. After indicate an infection and must be investigated and treated immediately. Options 1, 2, and 4
teaching potential complications associated with are appropriate actions to be taken by the family.
chemotherapy and radiation, the nurse concludes the
family requires further education when one of them
makes which of the following statements?

1.‐ "We will call for the ambulance if we can't wake him up."
2.‐ "We will not allow sick people around him."
3.‐ "We will notify the doctor if his temperature rises over 102 degrees."
4.‐ "We will offer foods he likes and avoid undercooked meats and raw vegetables."
619 The nurse working in the outpatient oncology clinic Correct answer: 2 A white coating on the tongue may indicate thrush (Candida). Infections should be identified Use the process of elimination to differentiate normal versus abnormal results.
determines that the client receiving radiation therapy and treated immediately to avoid complications. The client may experience a very dry mouth
to the neck has understood the teaching instructions (xerostomia), but it is not a serious complication (option 1). Dental floss should be avoided if
about follow‐up care when the client calls to report the client has thrombocytopenia secondary to bone marrow suppression (option 3).
which of the following? Mouthwash should be avoided to reduce irritation to the mouth (option 4); however, options 3
and 4 do not relate to follow‐up care.

1.‐ A very dry mouth


2.‐ A white coating on the tongue
3.‐ Gums that bleed slightly after using dental floss
4.‐ Mouth burning after using mouthwash

620 The nurse is conducting an in‐service to a group of Correct answer: 4 Superior vena cava syndrome is usually caused by the growth of a lung or mediastinal tumor, This question requires knowledge of superior vena cava syndrome.
nurses about oncologic emergencies in clients with not by metastasis (option 1). The tumor obstructs the flow of blood to the right atrium, leading
cancer. Using the diagram shown, the nurse would to facial and arm edema. Superior vena cava syndrome occurs as a late‐stage manifestation
explain which of the following about the complication (option 3), and option 4 is incorrect.
of superior vena cava syndrome?
1.‐ Lung cancers usually metastasize to the superior vena cava, obstructing blood flow, leading to the early symptoms of facial and arm edema.
2.‐ Superior vena cava syndrome will eventually occur in all clients with cancer.
3.‐ Superior vena cava syndrome occurs early in the development of cancer.
4.‐ A lung tumor can compress the superior vena cava, obstructing blood flow, leading to the early symptoms of facial and arm edema.

621 The nurse is preparing a care plan for a client with Correct answer: 2 Clients with polycythemia experience satiety and fullness resulting from hepatomegaly and The core issue of the question is knowledge of an appropriate diet for a client with
polycythemia vera on ways to maintain nutrition. The splenomegaly. Frequent, small meals will help maintain adequate nutrition. Foods rich in iron polycythemia. Use nursing knowledge and the process of elimination to make a selection.
nurse should include which of the following in the are not appropriate because there is an increase in erythrocytes in this condition. Spicy foods
plan? will increase the gastrointestinal symptoms, which also include dyspepsia and increased gastric
secretions.

1.‐ Increase intake of foods high in iron.


2.‐ Encourage small, frequent meals rather than three big meals.
3.‐ Increase the amount of red meats and organ meats in the diet.
4.‐ Encourage the use of hot spices in foods to stimulate appetite.

622 A client with thrombocytopenia presents to the Correct answer: 2 Clients with thrombocytopenia have decreased platelet counts below 150,000/uL. The usual The core issue of the question is the laboratory test results that best indicate a disorder of
primary care center. During assessment, the nurse presenting manifestation of this condition is the appearance of petechiae, purpura, and abnormal clotting ability. Use nursing knowledge and the process of elimination to make a
notices petechiae. The nurse interprets that which ecchymosis. The other laboratory values will not explain the petechiae or support the presence selection.
laboratory result best supports the presence of a of a clotting disorder.
disorder of hemostasis?
1.‐ Decreased erythrocyte count
2.‐ A platelet count below 150,000/uL
3.‐ An elevated lymphocyte count
4.‐ A hemoglobin value of 14 or more

623 A nurse is admitting a client with a diagnosis of Correct answer: 2 Clients with aplastic anemia usually experience pancytopenia (decreased erythrocytes, The core issue of the question is knowledge of the effects of aplastic anemia on the
aplastic anemia. Which of the following is the best leukocytes, and platelets). The client with this type of hypoplastic anemia should therefore immune system, which then requires special intervention to prevent infection. Use nursing
room for the nurse to assign this client? have a room where reverse isolation can be instituted. The client with aplastic anemia is knowledge and the process of elimination to make a selection.
susceptible to infection as well as hemorrhage. Respiratory isolation requiring negative airflow
(option 2) is not necessary in the care of clients with aplastic anemia.

1.‐ A semiprivate room with a client whose diagnosis is urosepsis.


2.‐ A regular private room at the end of the hall.
3.‐ A private isolation room equipped with a negative airflow.
4.‐ A semiprivate room with a client whose diagnosis is thrombophlebitis.

624 The nurse is reviewing laboratory results of a client Correct answer: 4 In DIC, there is abnormal initiation and formation of blood clots. As clots are formed and then The core issue of the question is knowledge of trends in changes of laboratory data in DIC.
suspected of having disseminated intravascular begin to dissolve, more end products of fibrinogen and fibrin are also formed. These are called Use nursing knowledge and the process of elimination to make a selection.
coagulopathy (DIC). The nurse looks to the results of fibrin degradation products or fibrin split products. Although the PT and PTT are prolonged and
which test as the more specific marker for DIC? the platelet count is reduced in DIC, they could also be a result of other coagulation
disturbances. Only the increase in FDP would occur because of the widespread accelerated
clotting present in DIC.
1.‐ Partial thromboplastin time (PTT)
2.‐ Prothrombin time (PT)
3.‐ Platelet count
4.‐ Fibrin degradation products (FDP)

625 The husband of a client with disseminated Correct answer: 3 Initially, there is an enhanced coagulation mechanism with resulting increase in fibrin and The core issue of the question is the possible role of heparin in the management of DIC.
intravascular coagulopathy (DIC) approaches the nurse platelet deposition in arterioles and capillaries in DIC, resulting in thrombosis. Although it Use nursing knowledge and the process of elimination to make a selection.
and expresses his concern that his wife might be remains controversial in DIC, the use of heparin is aimed at preventing the formation of
getting the wrong medication after he was told that additional thrombotic clots that further complicate the bleeding disorder.
the client was receiving heparin. What is the nurse’s
best response?
1.‐ “I understand your concern, but the doctors know what they are doing.”
2.‐ “Let me make sure that I have not misread the physician’s order.”
3.‐ “The drug is being used to stop the abnormal clotting in capillaries and arterioles.”
4.‐ “Please ask the physician why this medication is being given.”

626 The nurse is administering oral care on a client with Correct answer: 3 Clients with DIC should be protected from injury that will result in bleeding. An oral swab is The core issue of the question is appropriate methods of providing mouth care to a client
disseminated intravascular coagulopathy (DIC). Which least likely to cause tissue injury to the oral cavity during the performance of oral care. with stomatitis. Use nursing knowledge and the process of elimination to make a selection.
of the following is the most appropriate for this client? Mouthwashes containing alcohol should be avoided because they may cause discomfort and
because they tend to dry the mucous membranes. Toothbrushes may be used only if they are
soft‐bristled, but a swab or toothette is the best option.

1.‐ Limit flossing to once a day.


2.‐ Use an alcohol‐based mouthwash to prevent infection.
3.‐ Use swabs to administer oral care.
4.‐ Encourage tooth brushing at least once a shift.

627 A client with stomatitis and on neutropenic Correct answer: 3 Hydrogen peroxide is not a good choice of mouthwash solution in clients with stomatitis The core issue of the question is a mouth care product that would be irritating to a client
precautions is ordered to have mouthwashes every 2 because it tends to dry the oral mucosa and further aggravate the discomfort. The other three with stomatitis. Use nursing knowledge and the process of elimination to make a selection.
hours. Which of the following choices of mouthwash options are acceptable mouthwash solutions; diphenhydramine (Benadryl) or Maalox may also
solutions should the nurse question if ordered by the be used.
physician?
1.‐ Viscous lidocaine (Xylocaine)
2.‐ Normal saline solution
3.‐ Hydrogen peroxide
4.‐ Diluted baking soda

628 A client with acute myelogenous leukemia (AML) is Correct answer: 2 Harvested bone marrow is infused into the recipient intravenously. The transplantation is The core issue of the question is knowledge of bone marrow transplantation as a
scheduled for a bone marrow transplant (BMT). In usually preceded by chemotherapy and radiation therapy. During this period and up to when treatment method. Use nursing knowledge and the process of elimination to make a
teaching the client’s family about BMT, which of the the client’s response to the transplantation has been successful, nursing interventions should selection.
following statements by the nurse is best? focus on prevention of infection.

1.‐ “The client will be in the operating room with the donor so that immediate transplantation can occur.”
2.‐ “The specially prepared marrow is infused intravenously to the client.”
3.‐ “The client will be brought to the radiology department to transplant the marrow.”
4.‐ “A large bore needle will be inserted into the client’s bone marrow where the donor marrow will be infused.”

629 A client has undergone a lymph node biopsy. The Correct answer: 1 Histological isolation of Reed‐Sternberg cells in lymph node biopsy examination is a diagnostic The core issue of the question is knowledge of characteristic findings in the diagnosis of
nurse anticipates that the report will reveal which of feature of Hodgkin’s lymphoma. Philadelphia chromosome is attributed to chronic lymphoma. Use nursing knowledge and the process of elimination to make a selection.
the following if the client has Hodgkin’s lymphoma? myelogenous leukemia. Viruses are much smaller than can be visualized with cytology.

1.‐ Reed‐Sternberg cells


2.‐ Philadelphia chromosome
3.‐ Epstein‐Barr virus
4.‐ Herpes simplex virus

630 During physical examination, the nurse finds a Correct answer: 2 A nontender and moveable cervical node may suggest the presence of malignancy and even The core issue of the question is the ability to interpret assessment data related to lymph
nontender, moveable cervical node on a client. The lymphoma. Palpable nodes do not confirm the diagnosis of a malignancy. Biopsy and nodes. Use nursing knowledge and the process of elimination to make a selection.
nurse makes which interpretation of this finding? histological examination will aid in interpreting the significance of enlarged nodes.

1.‐ Normal, since the node is moveable


2.‐ Abnormal and may suggest the presence of a malignancy
3.‐ Normal, since the node is nontender
4.‐ Abnormal and a positive indicator of a malignancy

631 A client with thrombocytopenia has an order for Correct answer: 2 Clients with thrombocytopenia are at risk for altered cerebral perfusion from bleeding. Since The core issue of the question is knowledge that a low platelet increases risk of bleeding,
neurological checks every hour. The nurse explains to a a neurologic assessment can assist in determining the presence of occult bleeding in the which includes the risk of intracranial bleeding. Use this nursing knowledge and the process
curious nursing assistant that the reason for frequent cerebrovascular system, it is a necessary nursing intervention to include in the care of these of elimination to make a selection.
neurological assessment is which of the following? clients.

1.‐ To determine if the coagulopathy is related to a neurological disorder


2.‐ To monitor for signs of intracranial bleeding
3.‐ To evaluate the effectiveness of pharmacologic interventions
4.‐ To correlate increasing platelet counts with the neurological status

632 During assessment, the nurse notices a systolic Correct answer: 1 In anemia, there is a decrease in the viscosity of blood as a result of a decrease in the number The core issue of the question is knowledge of how pathophysiology relates to assessment
murmur on a client with anemia. The nurse interprets of red blood cells. The increase in cardiac output and flow are compensatory mechanisms data in a client with anemia. Use nursing knowledge and the process of elimination to make
that this finding correlates with which of the because of the decrease in the quantity of hemoglobin in circulating blood. a selection.
following?
1.‐ Increased quantity and speed of low‐viscosity blood through valves
2.‐ Structural abnormality of heart valves from the anemia
3.‐ High viscosity of blood circulating through the cardiac structures
4.‐ Decreased blood flow through the vascular system

633 A client with anemia has a nursing diagnosis of Correct answer: 1 Activity intolerance in clients with anemia results from the imbalance between oxygen The core issue of the question is knowledge that anemia causes fatigue and that measures
activity intolerance. Which of the following nursing demand and supply. Activities should be planned to intersperse activity with periods of rest to to prevent fatigue need to be incorporated in the plan of care. Use this knowledge and the
interventions should the nurse implement? decrease hypoxemic episodes and to decrease tissue demand for oxygen. All the other options process of elimination to make a selection.
are appropriate interventions for a client with anemia, but they do not relate to the nursing
diagnosis of activity intolerance.
1.‐ Space interventions and encourage rest periods.
2.‐ Teach client the basics of good nutrition.
3.‐ Promote active or passive range of motion activities
4.‐ Teach client to change position slowly to prevent dizziness
634 The nurse is teaching a client with hemophilia A about Correct answer: 2 Clients with hemophilia should be taught to participate in noncontact sports and to avoid any The core issue of the question is an appropriate element of client teaching with
home management. Which of the following should the activities that increase the risk of tissue injury and bleeding. Clients with hemophilia should hemophilia. Use concepts related to prevention of bleeding and the process of elimination
nurse include in the teaching plan? never use aspirin because of the risk of bleeding. Joint pain may be caused by hemarthrosis to make a selection.
(bleeding in the joints), a situation in which the client should be taught to seek medical care
immediately. Iron‐rich foods are not appropriate in clients with this condition unless there is
an accompanying anemia.
1.‐ Increase iron‐rich foods in the diet
2.‐ Avoid contact sports
3.‐ Use aspirin when severe pain occurs
4.‐ Minimize joint pain by walking and weight‐bearing

635 The nurse is obtaining a health history on a client who Correct answer: 2 Aplastic anemia may be congenital or acquired, but most cases do not have an identifiable The core issue of the question is knowledge of the possible etiologies of aplastic anemia.
is admitted with a diagnosis of “rule out aplastic etiology. It is known that aplastic anemia may follow exposure to chemicals (e.g., Benzene, Use knowledge about the possible causes of this disorder and the process of elimination to
anemia.” Considering the diagnosis, which of the DDT) or drugs (chloramphenicol, sulfonamides). It is therefore important that the nurse obtain make a selection.
following data is most important for the nurse to elicit exposure history on this client.
during the interview?
1.‐ Recent travel outside the country
2.‐ Exposure to chemicals and drugs
3.‐ History of blood transfusion
4.‐ Medication allergies

636 The nurse is teaching family members about Correct answer: 4 A client with neutropenia has a compromised immune system and is predisposed to The core issue of the question is knowledge of the components of neutropenic
precautions to take in visiting a client who has infections. Fresh fruits and flowers in the client’s room are not allowed because they tend to precautions. The wording of the question tells you the correct answer is an incorrect
neutropenia. Which of the following instructions would harbor bacteria. All the other options are reasonable instructions to be given to visitors as well statement. Use nursing knowledge and the process of elimination to make a selection.
not be included by the nurse in the discussion? as health care personnel who come in contact with the client.

1.‐ People who have colds or infectious diseases should not visit.
2.‐ Visitors must wash their hands before and after a visit.
3.‐ Face masks should be worn by all those who come in contact with the client.
4.‐ Fresh flowers will help to provide a cheerful environment.

637 A client has a platelet count of Correct answer: 1 A platelet count below 20,000 indicates that the client is at risk for bleeding and necessitates The core issue of the question is appropriate interpretation of a low platelet count and
18,000/mm&lt;sup&gt;3&lt;/sup&gt;. What the avoidance of activities and interventions that increase this risk. Nursing interventions such interpreting the appropriate intervention to protect the client from bleeding. Use nursing
intervention must the nurse include in the plan of as the use of intramuscular injections, rectal temperatures, and shaving with a razor are knowledge and the process of elimination to make a selection.
care? activities that predispose the client to further injury. Reverse isolation is not appropriate for
this client unless there is accompanying evidence of neutropenia.

1.‐ Institute bleeding precautions.


2.‐ Institute reverse isolation.
3.‐ Schedule medications by intramuscular route when able.
4.‐ Obtain temperatures rectally.

638 A nurse is assisting a physician with a bone marrow Correct answer: 1 Application of direct pressure and pressure dressing should follow the withdrawal of the The core issue of the question is knowledge of specific care following bone marrow
aspiration on a client with anemia. After the aspiration needle after a bone marrow aspiration. If the client has thrombocytopenia, pressure aspiration that will prevent complications of the procedure. Use nursing knowledge and the
procedure, the nurse should take which of the should be applied on the site for at least 3 to 5 minutes or until hemostasis has been achieved. process of elimination to make a selection.
following actions? The other options are not appropriate following a bone marrow aspiration. Continued
observation of the site should be made to assure that there is no bleeding.

1.‐ Apply pressure on the site to stop bleeding.


2.‐ Massage the area to decrease pain.
3.‐ Apply heat to the area to diminish the discomfort.
4.‐ Cover the area with a light dressing.
639 The white blood cell (WBC) differential on a client Correct answer: 2 A shift to the left indicates an increase in immature neutrophils or bands. An increase in the The core issue of the question is the ability to make an accurate interpretation of findings
indicates a shift to the left. The nurse makes which of number of bands indicates an increase in the production of granulocytes, which could be a on a laboratory report of WBC count and morphology. Use nursing knowledge and the
the following accurate interpretations of this report? compensatory mechanism in response to infection. process of elimination to make a selection.

1.‐ There is an increase in the number of segmented neutrophils.


2.‐ There is an increase in the number of bands released into the circulation.
3.‐ The number of lymphocytes increased in number.
4.‐ The number of lymphocytes exceeds the total WBC count.

640 A client with iron‐deficiency anemia is scheduled for a Correct answer: 3 The morphologic characteristics of RBCs in iron‐deficiency anemia is microcytic and The core issue of the question is the pathophysiological changes of RBCs in specific
complete blood count. The nurse anticipates that the hypochromic. Vitamin B&lt;sub&gt;12&lt;/sub&gt; anemia produces a macrocytic and anemias. Use nursing knowledge and the process of elimination to make a selection.
report will show which characteristics of the red blood normochromic morphology. Aplastic anemia, hemolysis, and acute blood loss will reveal RBCs
cells (RBCs)? with normocytic and normochromic characteristics.
1.‐ Normocytic, normochromic
2.‐ Macrocytic, normochromic
3.‐ Microcytic, hypochromic
4.‐ Normocytic, hyperchromic

641 The nurse in the hematology clinic is reviewing Correct answer: 2 Reticulocytes are immature RBCs. An increase in the number of reticulocytes indicates the The core issue of the question is the ability to evaluate outcomes of care for a client with
laboratory findings for a 2‐year‐old being treated for body is producing new RBCs. Iron intake does not indicate an improvement in anemia status, anemia. Use nursing knowledge and the process of elimination to make a selection.
anemia. Which finding is the best indication that the and the child with anemia is not cyanotic but pale. An increase in activity is hard to measure
treatment is successful? subjectively and would be a late finding.
1.‐ The child is no longer cyanotic.
2.‐ The reticulocyte count is rising.
3.‐ The child is more active.
4.‐ Stools are black, indicating iron intake.

642 A pregnant woman tells the nurse that she has a Correct answer: 3 Sickle cell is inherited as an autosomal recessive disorder. Both parents must carry the The core issue of the question is the ability to teach a client about genetics as they relate
family history of sickle cell anemia and is afraid her defective gene. The other statements are factually incorrect. to sickle cell disease. Use nursing knowledge and the process of elimination to make a
baby will be born with the disease. The nurse would selection.
provide which information during a discussion with this
client?
1.‐ Sickle cell anemia is a male disease and would be passed on through the man’s family.
2.‐ Genetic testing will be needed to determine if her fetus is affected.
3.‐ Both mother and father must carry the defective gene for the child to have sickle cell anemia.
4.‐ The child only needs one parent to be a carrier in order for the child to be affected.

643 A young child who was admitted to the hospital with Correct answer: 1 Hemophilia is characterized by a deficiency in one or more clotting factors, while ITP is a The core issue of the question is an understanding of the differences between hemophilia
a bleeding disorder has been diagnosed with idiopathic platelet disorder. Because the child with ITP is not deficient in clotting factors, this treatment and bleeding disorders caused by platelet problems. Use nursing knowledge and the
thrombocytopenic purpura (ITP). The mother of the would not be beneficial. process of elimination to make a selection.
child says to the nurse, “I have a friend who has a son
with hemophilia. When he bleeds, they give him a
‘factor,’ which they keep in their home refrigerator.
Can we just give my child this factor?” Which of the
following is the best response by the nurse?

1.‐ “Your friend’s child has a natural deficiency in clotting factors; your child does not.”
2.‐ “Factor has a lot of negative side effects, and the doctors would rather not use it on your child.”
3.‐ “The amount of factor that would be required to treat your child would be excessive.”
4.‐ “That treatment may be tried later if your child does not respond to steroids.”
644 The nurse has admitted a child newly diagnosed with Correct answer: 2 Clients with anemia will experience activity intolerance with even the simplest activities of The core issue of the question is knowledge of typical pathophysiology and client
anemia of unknown origin. Which of the following daily living. There is no vaso‐occlusion or abnormal platelet count with anemia. There may be assessments in anemia and using this information to identify the most important nursing
nursing diagnoses is most appropriate? insufficient cardiac output, but it will not be related to platelet count. There is no information diagnosis. Use nursing knowledge about anemia and the process of elimination to make a
in the question to indicate that the anemia is secondary to poor diet. selection.

1.‐ Insufficient cardiac output related to abnormal platelet count


2.‐ Activity intolerance related to generalized weakness and fatigue
3.‐ Imbalanced nutrition: less than body requirements
4.‐ Risk for pain related to vaso‐occlusion

645 The nurse is caring for a child with beta‐thalassemia Correct answer: 3 Frequent blood transfusion will lead to an overload of iron in the body. This iron is stored in The core issue of the question is identification of a complication of chronic blood
who has received many blood transfusions. The nurse tissues and organs and is called hemosiderosis. Blood transfusions do not lower the white transfusion therapy. Note the critical word 'priority' in the question, which tells you the
assesses for which of the following as a priority at this count or cause petechiae or hemoglobin in the bile. correct option is the condition of most importance at the current time. Use nursing
time? knowledge of thalassemia and the process of elimination to make a selection.

1.‐ Neutropenia
2.‐ Petechiae
3.‐ Hemosiderosis
4.‐ Hemoglobin S formation

646 A client with Vitamin B&lt;sub&gt;12&lt;/sub&gt; Correct answer: 2, 4, 5 Clients with nutritional anemias require dietary sources of folic acid, such as green, leafy The core issue of the question is knowledge of foods that are rich in Vitamin
deficiency needs to increase dietary intake of foods vegetables; fish; citrus fruits; yeast; dried beans; grains; nuts; and liver. Apples and carrots are B&lt;sub&gt;12&lt;/sub&gt;. Use nursing knowledge and the process of elimination to make
that are good sources of this vitamin. The nurse not as rich in folic acid as the other food sources listed. your selections.
recommends that the client increase intake of which of
the following foods? Select all that apply.

1.‐ Apples
2.‐ Spinach
3.‐ Carrots
4.‐ Oranges
5.‐ Liver

647 An 8‐year‐old is being admitted in vaso‐occlusive Correct answer: 3 A vaso‐occlusive crisis is a very painful experience and proactive assessment and pain control First eliminate activities not related to sickle cell then determine priority of those
crisis. When creating the care plan, to which of the are imperative. Although oxygen will help in pain control by preventing more sickling, high remaining options.
following actions should the nurse give priority? concentrations are not needed. Acid‐base balance is not routinely disrupted in a vaso‐occlusive
crisis. Factor VIII replacement therapy is utilized with hemophilia.

1.‐ Administering high concentration of oxygen to provide adequate oxygenation


2.‐ Evaluating the acid‐base status and administering sodium bicarbonate as necessary
3.‐ Assessing pain and administering pain medication as necessary
4.‐ Replacing Factor VIII

648 The nurse is completing discharge teaching for the Correct answer: 3 Bed rest without immobilizing and elevating the affected area is inappropriate to stop the Two of the responses include elevating the body part. That should be an indication that
parents of a child newly diagnosed with hemophilia B. bleeding. If coagulants are needed, they would be applied topically, not orally. Gentian violet one of those responses is the correct answer. Knowledge of the care of bleeding episodes
Which measures should the nurse include when will have no effect upon the bleeding. Warm compresses will increase the blood flow, making will help to eliminate the incorrect options.
discussing the control of minor bleeding episodes? it harder to stop the bleeding.

1.‐ Bed rest, oral coagulants, and cold compresses


2.‐ Gentian violet, ice packs, and constant pressure to the affected areas for one hour
3.‐ Immobilize and elevate the affected area, apply constant pressure to the areas for at least 15 minutes
4.‐ Elevate the affected area, alternating warm and cold compresses every 20 minutes
649 The elementary school nurse is called to the gym Correct answer: 2 As the child's safety is the first concern, walking to the office would not be safe at this time. Knowledge of the care of the child with thalassemia to prevent injury will aid in choosing
when a child with a history of thalassemia complains of Sitting immediately will decrease the chance of falling. The urgent safety requirements take the correct answer. Consider which activity provides the highest level of safe practice.
dizziness during physical education class. The priority precedence over physical assessment. The information given does not indicate the need for
action of the nurse should be to: ammonia at this time to prevent fainting.

1.‐ Assist the child to the nurse's office to lie down until the dizziness passes.
2.‐ Assist the child to sit in the gym until the dizziness passes.
3.‐ Utilize an ammonia ampoule to prevent loss of consciousness.
4.‐ Assess the child's blood pressure and pulse.

650 The nurse in the pediatrician's office receives a call Correct answer: 2 A change in the stools to a black, tarry color is an expected side effect of the medication and The core concept in this stem is the addition of an oral iron preparation. Consider
from the mother of a child who was started on iron a sign that the medication is working properly. It is not a symptom of the anemia, a sign that common side effects of this drug in making a selection.
supplements approximately two weeks ago. The the dose is high, or that the child is experiencing bleeding.
mother is panicked because this morning her
daughter's stools were a black, tarry color. The nursing
response would be:
1.‐ "This is an expected symptom of the iron deficiency anemia."
2.‐ "This is a normal sign that the iron preparation is working properly."
3.‐ "This is a sign that the dose of iron is more than required."
4.‐ "This is a sign that the child is experiencing bleeding and needs to come to the office immediately."

651 A child with sickle cell anemia is admitted to the Correct answer: 3 Children with sickle cell anemia develop sickling of the red cells when exposed to low oxygen Compare each of the responses with what is known about sickle cell anemia. The fact that
hospital. The nurse anticipates that laboratory tension; this means that the cells become crescent‐shaped. Polycythemia is not a finding with it is an anemia eliminates polycythemia.
evaluation of the client's red blood cells would reveal: sickle cell anemia (option 1). Hematopoiesis is the formation of new cells, which occurs at a
rapid rate in children with sickle cell anemia due to the rapid destruction of RBCs; however,
this process is not visible under the laboratory microscope. Children with sickle cell anemia
have adequate iron stores so the cells are not pale in color (option 4).

1.‐ Polycythemia.
2.‐ Hematopoiesis.
3.‐ Crescent‐shaped red blood cells.
4.‐ Hypochromatic red blood cells.

652 During a routine visit to the hematology clinic, the Correct answer: 3 All of the other activities hold a risk of bleeding, whether it is from physical contact or stress Consider which of the activities provides the least risk of physical contact.
parents of a 10‐year‐old with hemophilia voice their on joints and muscles. Swimming is considered ideal for clients with hemophilia as it provides
concerns as to their son's physical activity in relation to necessary activity with minimal risk for bleeding and injuries.
the potential for injury and bleeding. Physical activities
the nurse can suggest include:

1.‐ Basketball.
2.‐ Baseball.
3.‐ Swimming.
4.‐ Gymnastics.

653 An 11‐month‐old girl who has iron‐deficiency anemia Correct answer: 2 Children with iron‐deficiency anemia are more susceptible to infection related to microcytosis Option 4 can be eliminated as it does not relate to iron deficiency anemia. Then choose if
is hospitalized for a respiratory infection. Her mother and limited bone marrow function. The information in the other options is incorrect and does there is a relationship between infections and anemia in a child with both conditions.
voices confusion as to the connection between her not address the connection between the conditions of anemia and infection.
daughter's anemia and her infection. The nurse would
explain that children with iron‐deficiency anemia are:

1.‐ Equally susceptible to infection as other children.


2.‐ More susceptible to infection than other children.
3.‐ Less susceptible to infection than other children.
4.‐ At risk for respiratory infections because of the inability to produce leukocytes.

654 A 16‐month‐old toddler is admitted to the hospital for Correct answer: 3 A child with hemoglobin and hematocrit levels this low would already have a heart that is Monitoring vital signs and allergic reactions are common nursing activities during a blood
severe anemia secondary to insufficient iron intake. under stress. A sudden increase in blood volume can cause congestive heart failure. All of transfusion. Recognize that the blood values listed are very low and consider which of the
The child's hemoglobin is 8 grams/dL and hematocrit is these assessments are appropriate, but pulse rate is the priority assessment. vital signs would take precedence in this situation.
23%. A blood transfusion is ordered. During the
transfusion, the nurse’s priority in assessment would
be:
1.‐ Temperature.
2.‐ Respirations.
3.‐ Pulse rate.
4.‐ Observation for rash.

655 A 2‐year‐old has just been diagnosed with sickle cell Correct answer: 3 The child with sickle cell anemia does not need more iron supplements than the regular child. Knowledge of the management of sickle cell disease will help to choose the correct
anemia. The nurse has explained the diagnosis to the The cause of the child's anemia is fragile red blood cells, which are broken down more rapidly answers. The wording of the question indicates the need for an option that would be
family as well as provided information about the than the normal cell. Children with sickle cell anemia must guard against low oxygen tension in inappropriate for a child with sickle cell anemia.
treatment plan. The nurse will anticipate the need for the air. For that reason, they should not fly in unpressurized planes. Because infections
additional teaching when the mother makes which increase the basal metabolic rate (BMR) and oxygen requirements, infections often precipitate
statement? a crisis. During a sickling crisis, the child will need hydration therapy and pain management to
break the sickling cycle.

1.‐ "My husband loves to fly his small plane. I guess we'll have to take a commercial plane for our trips from now on."
2.‐ "If my child gets the flu bug, she might develop a sickling crisis."
3.‐ "My child will need extra iron tablets because of her anemia."
4.‐ "During a sickling crisis, my child will probably be hospitalized for pain control and hydration therapy."

656 The nurse is caring for a child in the ICU being treated Correct answer: 4 The PTT is used to monitor heparin therapy, as it is an indicator of the clot formation Heparin therapy reduces blood clotting. Consider which lab results relate to blood clotting.
for disseminated intravascular coagulopathy (DIC). pathways. Bilirubin levels are elevated with liver disease or excessive RBC damage. A client
Which of the following laboratory tests will take with DIC will likely need to have platelets and hemoglobin and hematocrit monitored, but they
priority for the nurse when monitoring the efficacy of are not indicators of effectiveness of heparin therapy.
the heparin therapy?

1.‐ Hemoglobin and hematocrit


2.‐ Platelet count
3.‐ Direct and indirect bilirubin levels
4.‐ Partial prothrombin time (PTT)

657 A 2‐year‐old with hemophilia is being discharged, and Correct answer: 4 It is not possible for parents of a hemophiliac to prevent a bleeding episode, no matter how Consider which statement would be inappropriate for a child with hemophilia.
the nurse is completing discharge teaching with his careful they are. The nurse should reinforce this information along with methods for
parents. Which of the following statements by the decreasing the chance of an injury that will lead to a bleeding episode. The other statements
parents indicates they require further teaching all indicate an appropriate understanding of hemophilia.
regarding hemophilia?

1.‐ “It is good to know that his sister will not get hemophilia also.”
2.‐ “If our son has a temperature, we will not give aspirin or ibuprofen, only acetaminophen.”
3.‐ “We will get a Medic‐Alert™ bracelet for our son as soon as we get home.”
4.‐ “We will be sure to watch our son very closely to make sure he does not have another episode of bleeding.”
658 The parents of a child with sickle cell anemia are Correct answer: 2 Sickle cell anemia is an autosomal recessive condition. Therefore, if both parents have the Since neither parent has the disease, they are both carriers, meaning that they each have
asking for information about future pregnancies. trait, each pregnancy carries a 25% (1 in 4) risk that the child will have the disease. only 1 gene with the disorder. Affected children must have both genes affected. There are
Neither parent has sickle cell anemia. The nurse would four combinations of the parents’ genes.
provide them with the information that any future
pregnancies will have a:
1.‐ 1 in 4 chance of producing a child with sickle cell trait.
2.‐ 1 in 4 chance of producing a child with sickle cell anemia.
3.‐ 1 in 2 chance of producing a child with neither sickle cell disease or trait.
4.‐ 1 in 2 chance of producing a child with sickle cell anemia.

659 The nurse is working with the family of an 8‐month‐ Correct answer: 3 Many infants with nutritional anemia rely primarily on the milk/formula for dietary intake and Nutritional anemia means a diet with inadequate iron. Milk is a poor source of iron.
old infant who has severe nutritional anemia. In refuse solid foods. When the milk/formula is limited, the child will be more willing to take solid
providing dietary recommendations, the nurse should foods. Cow’s milk is a poor source of iron. Peanuts and unsweetened chocolates are sources of
instruct the family to: iron but are not appropriate for this child’s diet.

1.‐ Switch the baby to cow’s milk.


2.‐ Delay the introduction of table food in the diet.
3.‐ Restrict the amount of milk or formula in the baby’s diet to 1 quart per day.
4.‐ Provide dietary iron sources such as peanuts and unsweetened chocolates.

660 A child is being admitted to the unit with thalassemia Correct answer: 2 Blood transfusions are utilized in order to maintain normal hemoglobin (HGB) levels. There is Because of fragile blood cells, the child will be anemic and require blood. Use this
major. In preparing client assignments, the charge an excess of iron secondary to repeated transfusions, and, thus, iron supplements will not be information to select the correct response.
nurse wants to assign a nurse to this child who can: necessary. The other therapies are inappropriate for the child with thalassemia major.

1.‐ Teach dietary sources of iron.


2.‐ Administer blood transfusions.
3.‐ Work with a dying child.
4.‐ Monitor the child for bleeding tendencies.

661 The nurse is caring for a child who is being treated for Correct answer: 3 Appropriate oxygenation is not possible when there is significant loss of blood volume. Use Maslow’s hierarchy of needs to answer the question. The physiological intervention
extensive bleeding in the Emergency Department. The Replacing the blood volume is critical to saving the child’s life, and it is imperative that would be the first concern, as the child is actively bleeding.
source and extent of bleeding are being determined as replacement occurs prior to any of the listed nursing actions.
the nurse is trying to control the bleeding. The nurse
places highest priority on which of the following
activities?
1.‐ Obtain the client’s history.
2.‐ Talk with the family regarding the risk of HIV and hepatitis C with blood transfusions.
3.‐ Replace blood volume.
4.‐ Provide psychosocial support to the family.

662 The nurse is working with the family of a toddler who Correct answer: 4 Folic acid potentiates the removal of iron from ferritin, which makes it further available for Iron deficiency anemia refers to inadequate hemoglobin and RBCs. Look for nutritional
is being treated for iron‐deficiency anemia. In teaching heme production. The synthesis of albumin, blood proteins, fibrinogen, and hemoglobin is additions that will help develop RBCs.
dietary considerations, the nurse will instruct the dependent upon the presence of proteins. None of the others are involved in building red
family to add sources of iron and: blood cells (RBCs).
1.‐ Vitamin D and thiamine.
2.‐ Calcium and riboflavin.
3.‐ Carbohydrates and vitamins.
4.‐ Folic acid and proteins.
663 The elementary school nurse is assessing and giving Correct answer: 2 The nurse would elevate the leg above the level of the heart to reduce bleeding. Aspirin or Since hemarthrosis is bleeding into the joints, look for strategies to reduce bleeding.
initial care to a client with hemophilia who has aspirin‐like products such as ibuprofen interfere with the clotting mechanisms. During active
significant pain in his knee. The nurse suspects bleeds, the joint should be immobilized. Warm soaks would promote bleeding; ice packs
hemarthrosis. As the nurse waits for his caregiver to should be used instead.
arrive, the nurse would take which of the following
actions?
1.‐ Maintain joint mobility with passive range‐of‐motion exercises.
2.‐ Elevate the leg above his heart.
3.‐ Administer children’s aspirin or ibuprofen for pain.
4.‐ Apply warm soaks to reduce the swelling.

664 The nurse has admitted a 2‐year‐old in vaso‐occlusive Correct answer: 1 Such positioning indicates the likelihood of abdominal pain. Nausea or constipation does not The core concept is preferred positioning and why a child chooses a particular position.
crisis. As the nurse starts the initial assessment, the generally cause a child to self‐position as described. Fear related to the hospitalization would
child insists upon lying in bed, on her side, with her be common in a child this age. However, if this were the case, it is more likely the child would
knees flexed to the abdomen. The nurse decides to seek refuge in the arms of one of her parents.
further assess the child for the presence of which of
the following?

1.‐ Stomach pain


2.‐ Nausea
3.‐ Constipation
4.‐ Fear secondary to the impact of hospitalization.

665 The 10‐year‐old client in the Emergency Department Correct answer: 4 Such lab results indicate severe anemia. Fatigue results when the oxygen‐carrying capacity of First recognize that these lab values are very low. Therefore, the child’s oxygen‐carrying
has complete blood count (CBC) results that include RBCs is impaired and cellular hypoxia is present. Fatigue can be diminished and oxygen ability is reduced. Select the option that relates to oxygenation.
hemoglobin (Hgb) of 8 grams/dL and hematocrit (Hct) depletion limited when the client’s energy is conserved. There will be an increased oxygen
of 24%. The nurse determines that, based on requirement and increased fatigue with increased mobility. Increasing general hydration
laboratory results, which nursing action has the without transfusing RBCs will not positively affect the anemic state. Skin integrity is not a high
highest priority? priority at this point. Although improving nutrition is appropriate, the response would not be
immediate. The priority activity would be conserving energy and reducing cardiac stress.

1.‐ Assessing and promoting skin integrity


2.‐ Promoting hydration
3.‐ Promoting nutrition
4.‐ Conserving energy

666 The nurse is caring for a child diagnosed with Correct answer: 3 Chelation therapy works to rid the body of excess iron storage that results from the frequent First determine the pathology of thalassemia as well as its treatment. Then determine
thalassemia major who is receiving her first chelation transfusions required to maintain adequate hemoglobin. Chelation will have no effect upon which response relates to this knowledge.
therapy. The nurse reinforces teaching about chelation hypoxia or bleeding. Sickling of RBCs does not occur with thalassemia.
therapy with the parents by stating that it is done to:

1.‐ Decrease the risk of hypoxia.


2.‐ Decrease the risk of bleeding.
3.‐ Eliminate excess iron.
4.‐ Prevent further sickling of red blood cells (RBCs).

667 The nurse has completed some child and family Correct answer: 4 A child diagnosed with thalassemia who will receive multiple transfusions throughout life will Consider which statements are appropriate for a child with thalassemia and eliminate
education for a child diagnosed with thalassemia. The need chelation therapy for excessive iron stores. An iron supplement would be inappropriate those. That will leave only the response that indicates the need for more education.
medical plan of treatment includes blood transfusions in this child.
when the anemia reaches a severe point. Which
statement by the parents indicates a need for further
education?
1.‐ “Because of the anemia, my child will need extra rest periods.”
2.‐ “My child inherited this disorder from both of us.”
3.‐ “We should be alert to periods when our child seems paler than usual.”
4.‐ “My child needs an iron supplement.”

668 The nursing assistant is setting up a hospital room Correct answer: 1 Rectal temperatures can traumatize the fragile rectal mucosa, leading to bleeding, and should This child is at risk for bleeding. Consider equipment that might trigger a hemorrhage and
preparing to admit a child with disseminated be avoided. The vital signs will need to be measured on a regular basis. An intravenous start kit eliminate that from the environment.
intravascular coagulopathy (DIC). Which item would is appropriate as the child will need plasma and blood products. A bedpan will be needed if the
the nurse remove from the set‐up? child is on bed rest. Urinary catheters are avoided if possible.

1.‐ Rectal thermometer


2.‐ Bedpan
3.‐ Intravenous therapy start kit
4.‐ Sphygmomanometer

669 At a hemophilia camp, several children with injuries Correct answer: 3 All of the injuries require nursing care; however, the child with the head injury has a Consider which injury has the greatest opportunity for serious injury. Knowledge of the
arrive at the clinic at the same time. When prioritizing potentially life‐threatening injury. care of medical emergencies and prioritizing care of injuries will aid in choosing the correct
care for the children, the child who requires the most answer.
immediate care from the nurse is the child with:

1.‐ A swollen knee.


2.‐ Abrasions on both arms.
3.‐ A slight head injury.
4.‐ A puncture wound in the foot.

670 A 14‐year‐old boy with sickle cell anemia is admitted Correct answer: 1 RBCs sickle under conditions where low oxygen concentrations exist; therefore, administering Knowledge of the underlying rationale for the management of sickle cell disease will help
with severe pain in his abdomen and legs. He asks why oxygen will prevent additional sickling. The oxygen has no effect on the oxygen‐carrying to answer the question correctly.
the doctor ordered oxygen when he is not having any capacity of RBCs. It will not have an effect on development of respiratory complications. It will
problems breathing. The nurse will be most accurate in not decrease the potential for infection.
stating that the main therapeutic benefit of oxygen is
to:
1.‐ Prevent further sickling.
2.‐ Prevent respiratory complications.
3.‐ Increase the oxygen‐carrying capacity of red blood cells (RBCs).
4.‐ Decrease the potential for infection during the crisis.

671 The nurse is administering a liquid iron preparation to Correct answer: 3 Iron preparations should be taken through a straw in order to prevent staining the teeth. While option 4 might be true, the stem describes a difficult situation for the family.
a 3‐year‐old with iron deficiency anemia. It will be While it is best to give toddlers choices in the hospital setting, the other options are not Consider the difference the options are offering.
most appropriate to: appropriate as iron is best absorbed on an empty stomach.
1.‐ Mix the medication in the child’s milk and give it at lunch.
2.‐ Give the medication after lunch with a sweet dessert to disguise the taste.
3.‐ Give the medication in a small cup and allow the child to sip it through a straw.
4.‐ Allow the child to decide whether to take the medicine with breakfast or dinner.

672 The nurse is admitting a child newly diagnosed with Correct answer: 3 In an acute care setting such as a hospital and with a potentially life‐threatening disease such While option 4 is appropriate, the stem describes a difficult situation for the family. Option
disseminated intravascular coagulopathy (DIC). as DIC, the family members may need help with coping with the stress they are feeling. This 3 responds to this difficult situation.
Although the physician has explained the plan of care stress often interferes with communication. A patient response by the nurse with repetition of
to the family members, they continue to ask about information will allow the family to absorb the information. The other options are not helpful.
each nursing activity. The nurse notes that the family
seems unable to comprehend the answers. The nurse
would most appropriately:

1.‐ Notify the doctor because the family seems to have a comprehension problem.
2.‐ Ask the doctor to write down the information for the family.
3.‐ Recognize that the family is under stress and continue to answer their questions.
4.‐ Assume they are a family with English as a second language (ESL).

673 The nurse is administering factor VIII to a child with Correct answer: 2 Factor VIII concentrate is a blood product. Fluid volume overload is an unlikely concern, as Since factor is a blood product, it carries many of the same potential complications as
hemophilia. The nurse should observe for which the factor will be given in a comparatively small volume of fluid. There is no greater a chance most other blood products.
potential complication during the infusion? of emboli formation with administration of factor than with any other IV preparation. Concern
as to contracting AIDS from administration of a blood product is a long‐term concern related
to multiple administrations. It is not a concern during the actual administration of the factor.

1.‐ Fluid overload


2.‐ Transfusion reaction
3.‐ Emboli formation
4.‐ Contracting AIDS

674 The clinic nurse has organized a class for several Correct answer: 1, 2, 5 There are 3 common problems seen in sickle cell anemia. First there is the anemia crisis. This Consider the definition of the terms. That will eliminate two choices, polycythemia and
parents of children newly diagnosed with sickle cell is a continuous problem for the sickle cell patient. Sequestration crisis occurs primarily in hemochromatosis. Knowledge of the etiology and pathophysiology of sickle cell disease will
disease. The nurse explains that problems with the children under 6 and in older children and adults who have functioning spleens. The crisis is a help to aid in the final selection.
disease can include: (Select all that apply.) pooling of the blood causing circulatory collapse. The third crisis is the vaso‐occlusive crisis.

1.‐ Aplastic crisis.


2.‐ Sequestration of blood.
3.‐ Hemarthrosis.
4.‐ Polycythemia.
5.‐ Vaso‐occlusive crisis.

675 Which of the following statements should be included Correct answer: 3 Anemia does occur easily in infancy, and infants have limited stores of iron. The first solid Knowledge of infant iron needs and nutrition will help you to choose the correct answer.
when teaching the parents of a 7‐month‐old infant food offered to infants is often cereal, which is an excellent source of iron. All infants do not Options 2 and 3 are opposites, indicating one is probably the right answer.
about preventing anemia? require iron supplements; it is preferable that the iron comes from dietary intake.

1.‐ “Anemia is unusual in infancy as infants use fetal iron stores until 18 months of age.”
2.‐ “Cow’s milk is an excellent source of iron, and infants should be changed from formula to milk as soon as possible after 6 months of age.”
3.‐ “Milk is a poor source of iron, and infants should be given solid foods high in iron such as cereals, vegetables, and meats.”
4.‐ “Anemia can easily occur during infancy, and all infants should receive iron supplements.”

676 A child with an alteration in platelet function has Correct answer: 4 Alterations in platelet function necessitate treating a break in the skin’s integrity as you would Platelet dysfunction refers to clotting problems. Two options refer to measures to prevent
been receiving intravenous fluids for two days. The an arterial stick—apply pressure for 5 minutes or more. The goal of treatment is to apply bleeding; one of these is the correct response. Knowledge of the care of the client with
nurse is discontinuing the peripheral IV. The nurse pressure long enough that the defective clotting mechanism will have time to form a clot. Steri‐ decreased platelets and IV care will help to make a final selection.
should: strips would not close the wound adequately, and restricting arm movement will not assist in
the initial formation of a clot.
1.‐ Restrict movement of the arm for 12 hours.
2.‐ Obtain a culture of the tip of the IV catheter.
3.‐ Place steri‐strips over site and have child hold the arm above the level of the heart for 15 minutes.
4.‐ Apply direct pressure to the site for at least 5 minutes.

677 A client with anemia has a hemoglobin of 6.5 g/dL. Correct answer: 3 Cerebral tissue hypoxia is commonly associated with dizziness. The greatest potential risk to Recall that safety, here prevention of falls, is a key priority.
The client is experiencing symptoms of cerebral tissue the client with dizziness is injury, especially with changes in position. Planning for periods of
hypoxia. Which of the following nursing interventions rest and consuming energy are important with someone with anemia because of his or her
would be the most important in providing care? fatigue level, but most important is safety.

1.‐ Providing rest periods throughout the day


2.‐ Instituting energy conservation techniques
3.‐ Assisting in ambulation to the bathroom
4.‐ Checking temperature of water prior to bathing

678 The nurse observing manifestations of complications Correct answer: 1 Major complications of multiple myeloma include bone pain, hypercalcemia, renal failure, This question requires knowledge of the pathophysiology of myeloma, also notice, odd
in a client with multiple myeloma concludes that they anemia, and impaired immune responses that are a result of bone marrow involvement and man out, the correct answer is longer.
are caused by which of the following? the systemic effects of substances secreted by the malignant plasma cells.

1.‐ Systemic effects of substances secreted by malignant plasma cells


2.‐ Increased white blood cells and platelet production
3.‐ Increased red blood cells and immunoglobulin production
4.‐ Increased platelet production and bone marrow destruction

679 A client has a diagnosis of myelodysplastic syndrome. Correct answer: 2 Myelodysplastic syndromes often progress to acute myelogenous leukemia. They are often This question requires knowledge of myelodysplastic syndrome.
The nurse planning care keeps in mind that this refractory to treatment and are associated with a poor prognosis. They are not hereditary and
condition: are often referred to as pre‐leukemia.
1.‐ Is one of a group of hereditary bone marrow conditions.
2.‐ May progress to acute myelogenous leukemia.
3.‐ Is referred to as pre‐lymphoma.
4.‐ Responds well to treatment.

680 A 40‐year‐old client is referred to a hematologist with Correct answer: 4 Bone marrow biopsy and aspirate is the only definitive diagnosis of AML. The presence of Recall that the biopsy procedure is usually definitive for a malignant process.
a tentative diagnosis of acute myelogenous leukemia Auer rods is diagnostic for AML. The presence of leukemic cells in the spinal fluid is more
(AML). The client's only complaint is fatigue. Which of common in acute lymphocytic leukemia (ALL). Uric acid and lactic dehydrogenase levels may
the following diagnostic tests would the nurse expect be elevated in AML, but this is not diagnostic for the disease.
to be ordered first?

1.‐ Liver function studies


2.‐ Uric acid
3.‐ Lumbar puncture
4.‐ Bone marrow biopsy

681 Which of the following dietary recommendations Correct answer: 1 Option 1 contains foods high in protein, folic acid, iron, and Vitamin This question requires knowledge of high‐iron foods.
should the nurse make to increase the intake of B&lt;sub&gt;12&lt;/sub&gt; that are needed for erythropoiesis. Options 2 and 4 contain lesser
nutrients needed for erythropoiesis? amounts.

1.‐ Milk; eggs; liver; and green, leafy vegetables.


2.‐ Apples, peanuts, oats, and cottage cheese.
3.‐ Cantaloupe, lima beans, and sweet potatoes.
4.‐ Dry yeast, grapefruit, and tuna fish.

682 A client has an order for an iron preparation to be Correct answer: 2 IM administration is recommended over intravenous infusion because of the potential for This question requires knowledge of the basics of parenteral medications.
given by the parenteral route. The nurse plans to give anaphylaxis. The gluteal muscle is the best route for administration since the muscle is large
the medication by which of the following routes? and highly vascular. The Z‐track method is preferable to prevent tattooing of the skin and
tissue necrosis caused by infiltration into the subcutaneous tissue.

1.‐ Intermittent infusion


2.‐ Deep gluteal intramuscular (IM) injection, using the Z‐track method
3.‐ Intramuscular in the deltoid to promote medication dissipation through muscle contraction
4.‐ Subcutaneous injection with weekly site rotation
683 In the normal blood clotting cycle, the final formation Correct answer: 3 Platelet aggregation forms a platelet plug at the site of bleeding, but fibrin reinforces the This question requires knowledge of the clotting cycle.
of a clot will occur at which of the following times? platelet plug. The absence of clotting factors impairs the coagulation response and the
capacity to form a stable clot.
1.‐ During the platelet phase
2.‐ During the vascular phase
3.‐ When fibrin reinforces the platelet plug
4.‐ When the plasmin system produces fibrinolysis

684 The client is given radioactive Vitamin Correct answer: 3 Pernicious anemia is caused by the body's inability to absorb Vitamin This question requires knowledge of pernicious anemia.
B&lt;sub&gt;12&lt;/sub&gt; in water for a Schilling B&lt;sub&gt;12&lt;/sub&gt;. This is caused by a lack of intrinsic factor in the gastric juices. The
test. The nurse instructs the client that the primary Schilling test helps diagnose pernicious anemia by determining the client's ability to absorb
purpose of this test is to measure his body's ability to Vitamin B&lt;sub&gt;12&lt;/sub&gt;.
do which of the following?
1.‐ Store Vitamin B12
2.‐ Digest Vitamin B12
3.‐ Absorb Vitamin B12
4.‐ Produce Vitamin B12

685 A client with a history of sickle cell anemia begins to Correct answer: 1 Pain from sickle cell crisis is primarily related to obstructed capillary blood flow causing This question requires knowledge of the pathophysiology of sickle cell anemia.
complain of pain. The nurse expects that the client is ischemia and possible tissue infarction. While dehydration often causes increased viscosity, the
going into sickle cell crisis because of the pain, which primary cause of pain is vasoocclusion of the blood vessels from sickled red blood cells.
signals which of the following?

1.‐ Vasoocclusion by sickled cells


2.‐ Viscosity of the blood
3.‐ Spasms caused by thickened blood
4.‐ Cells in the marrow causing bone pain

686 The nurse diligently assesses a leukemic client with Correct answer: 3 The client with neutropenia is unable to mount an inflammatory response. Fever is usually This question requires knowledge of leukemia.
neutropenia for signs of infection, anticipating that the first sign of infection. Options 1, 2, and 4 are all true, but they explain why the neutropenic
they will be absent or muted because: client is at greater risk for infection.

1.‐ Most infections are caused by organisms that are part of the body's normal flora.
2.‐ The white blood cells (WBCs) drop rapidly and recovery time is slow.
3.‐ Neutrophils are necessary to produce an inflammatory response.
4.‐ The immunoglobulins are reduced.

687 A client with anemia due to chemotherapy has a Correct answer: 1 Cerebral tissue hypoxia is commonly associated with dizziness. Recognition of cerebral This question requires knowledge of the sequelae of cerebral hypoxia.
hemoglobin of 7.0 g/dL. Which of the following hypoxia is critical since the body will attempt to shunt oxygenated blood to vital organs.
complaints would be indicative of tissue hypoxia
related to anemia?
1.‐ Dizziness
2.‐ Fatigue relieved by rest
3.‐ Skin that is warm and dry to the touch
4.‐ Apathy

688 A client's medical record indicates that a client has a Correct answer: 3 Approximately 95 percent of clients with CML are Philadelphia chromosome‐positive. This This question requires knowledge of the listed disorders.
positive Philadelphia chromosome. The nurse plans represents a translocation of the long arms of chromosomes 9 and 22.
care for which of the following disorders?

1.‐ Acute myelocytic leukemia (AML)


2.‐ Hairy cell leukemia
3.‐ Chronic myelogenous leukemia (CML)
4.‐ Chronic lymphocytic leukemia (CLL)

689 A nurse caring for a client who has experienced a Correct answer: 1 The risk for hemorrhage is of greatest risk since a large‐bore needle is used to perform the Recall that the greatest risk of many invasive procedures is bleeding.
bone marrow biopsy and aspiration should assess for biopsy and aspiration. Many of these clients often have an altered clotting capability. While
which of the following as the most serious the risk of infection is also a consideration, the procedure is performed under sterile
complication? conditions and is less of a concern than hemorrhage.
1.‐ Hemorrhage
2.‐ Infection
3.‐ Shock
4.‐ Splintering of bone marrow fragments

690 For the client diagnosed with iron‐deficiency anemia, Correct answer: 3 Organ meats such as liver are a good source of iron as well as green, leafy vegetables and egg This question requires knowledge of high‐iron foods.
the nurse should recommend an increased intake of yolks. Whole grain breads also contain iron, however not in as high a quantity as organ meats.
which of the following foods?
1.‐ Fresh citrus fruits
2.‐ Milk and cheese
3.‐ Organ meats
4.‐ Whole grain breads

691 The nurse would interpret that a client is most Correct answer: 3 Options 1, 2, and 4 relate to the white blood cells. A platelet count below 20,000 increases This question requires knowledge of the function of platelets.
severely at risk for bleeding when which of the the client's risk for severe bleeding because of reduced platelets to assist in the clotting
following is noted on laboratory test results? cascade to form a clot.
1.‐ Neutrophils are 50 percent
2.‐ Lymphocytes are 30 percent
3.‐ Platelets are less than 20,000
4.‐ Basophils are 0

692 The common feature of leukemia is which of the Correct answer: 2 Leukemia is a result of erratic production of white blood cells by the bone marrow, which This question requires knowledge of leukemia.
following? replace normal marrow components. It can arise from both a lymphatic and a myelocytic
etiology. White blood cells are often immature and incapable of performing their expected
function(s).
1.‐ A compensatory polycythemia stimulated by thrombocytopenia.
2.‐ An unregulated accumulation of white blood cells in the bone marrow.
3.‐ Increased blood viscosity resulting from an overproduction of white cells.
4.‐ Reduced plasma volume in response to a reduced production of cellular components.

693 The nurse would teach a client with sickle cell trait Correct answer: 4 Sickle cell trait is generally a mild condition that produces few if any manifestations. These This question requires knowledge of sickle cell trait.
that he or she: clients are considered carriers of the disease and require genetic counseling to determine
presence of the hemoglobin S. Certain stressors result in a sickle cell crisis.

1.‐ Should avoid fluid loss and dehydration.


2.‐ Can expect to experience hemolytic jaundice.
3.‐ Can expect to experience chronic anemia.
4.‐ Is protected from sickle cell crises under normal circumstances.

694 A client with pancytopenia enters the clinic with Correct answer: 1 All of the medications in the first option can affect platelet aggregation and should be avoided Recognize that aspirin should be considered with caution on
excess ecchymosis. The client should be cautioned to in a client with bleeding tendencies. A thorough review of all medications taken at home NCLEX&lt;sup&gt;®&lt;/sup&gt; questions.
avoid which of the following, since medications can should be done whenever clients are issued new medications.
alter platelet function?
1.‐ Acetylsalicylic acid (aspirin), digitalis (Lanoxin), and quinidine sulfate (Quinidex)
2.‐ Milk of magnesia, heparin, and quinidine sulfate (Quinidex)
3.‐ Senna (Senokot), furosemide (Lasix), and phenytoin (Dilantin)
4.‐ Acetaminophen (Tylenol), sulfonamides, and penicillins

695 The nurse would explain to a client newly diagnosed Correct answer: 2 Hemophilia is a group of hereditary clotting factor disorders characterized by prolonged This question requires knowledge of hemophilia.
with hemophilia that it is a hereditary bleeding coagulation time that results in prolonged and sometimes excessive bleeding. It is an X‐linked
disorder that: recessive characteristic transmitted by female carriers, displayed almost exclusively in males
often resulting in spontaneous bleeding into the joints resulting in hemoarthrosis with joint
deformity and potential disability. Option 3 is a specific form of hemophilia, von Willebrand's
Disease. Option 4 is pernicious anemia.

1.‐ Has a higher incidence in females.


2.‐ Is associated with joint bleeding, swelling, and damage.
3.‐ Is related to genetic deficiency of the von Willebrand factor.
4.‐ Can be caused by lack of Vitamin B12.

696 A client enters the clinic with complaints of a sore Correct answer: 1 Assessment findings in pernicious anemia include a smooth, red, beefy tongue; altered This question requires knowledge of the listed diagnoses.
mouth. Physical assessment reveals a beefy red sensations such as numbness or tingling in the extremities; and difficulty identifying one's
tongue. These symptoms are most suggestive of which position in space, which may progress to difficulty with balance and spinal cord damage.
of the following disorders?
1.‐ Pernicious anemia
2.‐ Hemolytic anemia
3.‐ Sickle cell anemia
4.‐ Polycythemia vera

697 In preparation for discharge, which of the following Correct answer: 4 Palpitation is a significant change in the condition of the client and may be indicative of This question requires knowledge of the function of hypoxia in leading to dysrhythmias.
statements would indicate that a client does not have progressing anemia. If palpitations occur, the client should report the symptom to the
a full understanding about her diagnosis of anemia? physician. Options 1, 2, and 3 are all positive responses to client teaching.

1.‐ "I should eat foods high in protein, iron, and Vitamin B12."
2.‐ "If I notice dizziness, I should avoid driving my car."
3.‐ "I may not be able to do all of my housework and go to work, too."
4.‐ "If I notice palpitations, I will lie down and rest awhile."

698 Pain in a client with multiple myeloma commonly Correct answer: 3 Lytic bone lesions are the most common cause of pain in multiple myeloma. Although the This question requires knowledge of multiple myeloma.
results from which of the following? marrow may be involved, this is not a common cause of pain. Neural infiltrations and intestinal
obstructions are not common in multiple myeloma.
1.‐ Intestinal obstruction
2.‐ Neural infiltration of plasma cells
3.‐ Lytic bone lesions
4.‐ Marrow infiltration

699 Which of the following characteristics is associated Correct answer: 2 Chronic leukemia progresses over a period of years rather than weeks. It occurs primarily Use the process of elimination to determine the answer most associated with chronic,
with chronic leukemia? between ages of 50 and 70. rather than acute, leukemia.
1.‐ Immature lymphocytes proliferate the marrow
2.‐ Gradual onset of the disease
3.‐ Rapid disease progression
4.‐ Primarily affects younger adults
700 Sickle‐shaped hemoglobin in sickle cell anemia may Correct answer: 4 Hemoglobin S in sickle cell anemia causes the red blood cells to elongate, become rigid, and This question requires knowledge of sickle cell disease.
cause which of the following that leads to assume a crescent sickle shape causing the cells to clump together, obstruct capillary blood
manifestations of the disease? flow in small vessels causing ischemia, decreased organ perfusion, and possible tissue
infarction.
1.‐ Cellular blockage in large vessels
2.‐ Increased organ perfusion
3.‐ Tissue ischemia and infarction
4.‐ Increased viscosity of the blood

701 The nurse discusses hypercalcemia and diuretic use Correct answer: 2 Increased calcium excretion in the urine from the diuretics decreases calcium levels. All other This question requires knowledge of the function of kidneys in excreting many body
with the client diagnosed with multiple myeloma. measures increase calcium. minerals and substances.
Which of the following mechanisms described by the
nurse explains the depletion of calcium from the body?

1.‐ Increased gastrointestinal absorption


2.‐ Increased urinary excretion
3.‐ Increased levels of parathyroid hormone
4.‐ Decreased levels of calcitonin

702 A client asks the nurse why Vitamin Correct answer: 3 Vitamin B&lt;sub&gt;12&lt;/sub&gt; deficiency anemia causes the production of abnormally This question requires knowledge of the pathophysiology of pernicious anemia.
B&lt;sub&gt;12&lt;/sub&gt; is important for red blood large red blood cells. This deficiency causes the red blood cell to be irregular and oval, rather
cell formation. The nurse responds with the knowledge than the biconcave shape of a normal red blood cell. This shape predisposes the cells to a
that Vitamin B&lt;sub&gt;12&lt;/sub&gt; deficiency shorter lifespan. In this type of anemia, there is an increase in the MCV (option 1) and a
causes which of the following changes in the red blood decrease in the hemoglobin (option 2). Option 4 is characteristic of iron deficiency anemia.
cell?

1.‐ Decreased mean corpuscular volume (MCV)


2.‐ Increased hemoglobin in the red blood cell
3.‐ Makes the cell irregular and oval‐shaped
4.‐ Makes the cell smaller in shape and deficient in hemoglobin

703 A nurse is discussing the role of hypoxia in red blood Correct answer: 2 Hypoxia stimulates the release of the hormone erythropoietin from the kidney and increases This question requires knowledge of the pathophysiology of hypoxia.
cell (RBC) production. Which of the following bone marrow production of RBCs. The hemoglobin does not increase in size with hypoxia.
statements is accurate? Reticulocytes mature in 24 to 48 hours, and their maturation is not influenced by hypoxia.

1.‐ Hypoxia stimulates the hemoglobin content of the RBC to increase.


2.‐ Hypoxia stimulates the release of erythropoietin in the kidneys.
3.‐ Reticulocytes become erythrocytes faster with hypoxia.
4.‐ RBC destruction is increased with hypoxia therefore stimulating RBC production.

704 A client with a hemolytic blood disorder presents to Correct answer: 1 Lysis of red blood cells cause retention of iron and other substances including bilirubin to This question requires knowledge of the pathophysiology associated with jaundice.
the primary care center with jaundice. The nurse accumulate in plasma. The accumulation of bilirubin causes jaundice. Although hepatitis
explains to the client that the jaundice is most likely infection may also be the reason for jaundice, the hemolytic anemia present most likely caused
caused by which of the following? the jaundice to occur.

1.‐ Increased bilirubin in plasma


2.‐ Increased haptoglobin in plasma
3.‐ Hepatitis infection
4.‐ Loss of plasma proteins
705 A nurse is evaluating the response of a patient with Correct answer: 4 The reticulocyte (immature RBC) count is an indicator that new red blood cells are being This question requires knowledge of bone marrow function.
anemia to therapy. Which of the following laboratory produced by the bone marrow. An increase in the reticulocyte count in an anemic client
tests would the nurse look to that best reflects bone indicates that the bone marrow is responding to the decrease in RBCs. The hematocrit count
marrow production of red blood cells? measures the percent of RBCs in the total blood volume. Hemoglobin is not directly linked to
bone marrow activity. Serum ferritin levels reflect available iron stores.

1.‐ Hematocrit
2.‐ Hemoglobin
3.‐ Serum ferritin
4.‐ Reticulocyte count

706 The nurse who is assessing a client with iron‐ Correct answer: 3 Iron‐deficiency anemia is manifested clinically by glossitis or inflammation of the tongue. This question requires knowledge of the definitions for the listed terms.
deficiency anemia notes that the tongue is inflamed. After pallor, this is the second most frequent manifestation of this type of anemia. Cheilitis or
The nurse documents this observation as: inflammation of the lips is another finding in this type of anemia. Achlorhydria, or the absence
of free hydrochloric acid is a manifestation of a depressed parietal cell function and is
associated with Vitamin B&lt;sub&gt;12&lt;/sub&gt; deficiency anemia. Cheilosis is cracking of
lips at the angles of the mouth.

1.‐ Cheilitis
2.‐ Achlorhydria
3.‐ Glossitis
4.‐ Cheilosis

707 The nurse is teaching a client about measures to Correct answer: 2 An acidic environment (such as in the presence of Vitamin C) enhances the absorption of iron. This question requires knowledge of orange juice’s effect on iron absorption.
increase the absorption of the prescribed oral iron Administering the medication with meals binds the iron with food and interferes with its
preparation. Which of the following instructions would absorption.
the nurse give to the client?
1.‐ Take the medicine with milk.
2.‐ Take the pill with a drink that contains Vitamin C.
3.‐ Take the iron with meals.
4.‐ Take the iron after meals.

708 Which of the following statements made by a client Correct answer: 1 The client on an oral iron preparation should be taught to expect stools to turn black because Use the process of elimination to determine the correct answer.
with iron‐deficiency anemia indicates the need for of the excessive iron that is eliminated. All the other choices should be included in the teaching
further teaching? plan. The health care practitioner may change the iron preparation prescribed to the client if
gastrointestinal symptoms become intolerable.

1.‐ "I should stop taking the medicine if my stools turn black."
2.‐ "I should dilute the liquid iron preparation and use a straw when taking it."
3.‐ "I can prevent the constipation by increasing the intake of fluids and fiber."
4.‐ "I should return to the clinic if my stomach upset worsens with this medication."

709 Which of the following food choices made by a client Correct answer: 4 Liver and muscle meats are excellent sources of iron. The other foods are also beneficial for This question requires knowledge of dietary sources of iron.
with anemia best indicates that the teaching regarding the dietary management of anemia, but option 4 is specifically an excellent source of iron.
selection of foods high in iron has been successful?

1.‐ Citrus fruits


2.‐ Green, leafy vegetables
3.‐ Eggs, milk, and milk products
4.‐ Liver and muscle meats
710 A nurse is preparing to administer an intramuscular Correct answer: 1 When administering an iron preparation intramuscularly, it should be given deep in the Recognize that this question reinforces basic psychomotor skills.
(IM) dose of iron to a client with anemia. Which of the muscle. The site should be in the upper outer quadrant of the buttocks utilizing the Z tract
following precautions should the nurse take? technique. No more than 2 mL of the solution should be administered and the length of the
needle should be 2 to 3 inches. The area should not be massaged after the injection.

1.‐ Administer the drug utilizing a Z tract technique.


2.‐ Use a 1‐inch, 19‐gauge needle.
3.‐ Administer the drug deep in the deltoid muscle.
4.‐ Massage the area vigorously after administering the iron.

711 The nurse would assess a client who has undergone a Correct answer: 2 Resection of the distal ileum results in the impaired absorption of Vitamin This question requires knowledge of pernicious anemia.
small bowel resection of the ileum for development of B&lt;sub&gt;12&lt;/sub&gt;. The other cause of Vitamin B&lt;sub&gt;12&lt;/sub&gt; deficiency
which type of anemia? is the loss of intrinsic factor‐secreting surfaces that are normally secreted by parietal cells.

1.‐ Iron‐deficiency anemia


2.‐ Vitamin B12 deficiency anemia
3.‐ Anemia of chronic disease
4.‐ Aplastic anemia

712 A client has an order for a test to determine if Correct answer: 2 Schilling test involves the administration of radioactive Vitamin B&lt;sub&gt;12&lt;/sub&gt;. This question requires knowledge of the diagnostic tests for pernicious anemia.
pernicious anemia is present. For which of the Increased absorption of Vitamin B&lt;sub&gt;12&lt;/sub&gt; when intrinsic factor is given
following tests should the nurse schedule the client? parenterally is indicative of pernicious anemia.

1.‐ Serum folate level


2.‐ Schilling test
3.‐ Serum iron and total iron‐binding capacity (TIBC)
4.‐ Bone marrow aspiration

713 The nurse is assessing a group of clients and identifies Correct answer: 2 Individuals who are chronically undernourished including the elderly, alcoholics, substance This question requires knowledge of risk factors.
which of the following as being at high risk for the abusers; and those with high metabolic requirements and on total parenteral nutrition are also
development of folic acid deficiency anemia? at risk for folic acid deficiency anemia. Alcoholics are particularly at risk because alcohol
interferes with folate metabolism.
1.‐ Obese individuals
2.‐ Alcoholics
3.‐ Young adults
4.‐ Athletes

714 Which of the following questions during the data‐ Correct answer: 3 The differentiating symptom of Vitamin B&lt;sub&gt;12&lt;/sub&gt; and folic acid deficiency This question requires knowledge of key assessments.
gathering phase is important for the nurse to ask a anemia is the absence of neurologic symptoms such as numbness and altered proprioception
client suspected of having a nutritional anemia? in folic acid deficiency anemia. The gastrointestinal symptoms of cheilosis, glossitis, and
diarrhea are present in both forms of nutritional anemia although usually more severe in folic
acid deficiency anemia.
1.‐ "Do you have a sore tongue?"
2.‐ "How has the consistency of your stools been?"
3.‐ "Do you experience any tingling or numbness?"
4.‐ "Have you had blood transfusions in the past?"

715 A couple seeks genetic counseling for sickle cell. Both Correct answer: 2 Sickle cell disease is an autosomal recessive genetic disorder where the individual is This question requires knowledge of genetic transmission.
have sickle cell traits. The nurse understands that the homozygous for the abnormal hemoglobin. If both parents have sickle cell traits, there is a 25
chances of the couple's offspring developing sickle cell percent chance that each pregnancy will produce a child with the disease.
disease with each pregnancy is:
1.‐ None of the offspring will develop sickle cell disease.
2.‐ Twenty‐five percent of their offspring will develop sickle cell disease.
3.‐ Fifty percent of their offspring will develop sickle cell disease.
4.‐ All their children will have sickle cell traits, but none will have the disease.

716 The nurse is preparing a teaching plan for a client Correct answer: 2 Clients with sickle cell disease have scarred spleen resulting in decreased ability to fight off Use the process of elimination to determine the correct answer.
with sickle cell disease about ways to prevent crisis infection. The individual with sickle cell disease must seek early treatment of infections.
episodes. Which of the following should be Pneumonia is one of the most common infections affecting individuals with sickle cell disease.
emphasized to prevent sickle cell crisis? Option 4 is inaccurate in that vigorous physical activity should be avoided.

1.‐ Eat nutritious foods that are high in iron.


2.‐ Seek treatment for infections as soon as possible.
3.‐ Take adequate amounts of supplemental vitamins and minerals.
4.‐ Avoid any type of physical activity.

717 Which of the following nursing diagnoses should Correct answer: 1 The client in sickle‐cell crisis will have pain related to ischemic tissue injury resulting from Use Maslow’s hierarchy of needs to reinforce the priority of comfort.
receive the highest priority in a client with sickle cell obstruction of blood flow. The other diagnoses, although important, are of lesser priority than
crisis? the nursing diagnosis of pain.
1.‐ Pain
2.‐ Self‐care deficit
3.‐ Activity intolerance
4.‐ Altered health maintenance

718 The nurse is reviewing laboratory results of a client. Correct answer: 2 Clients with sickle cell disease express 80 to 90 percent of HbS. Clients with sickle cell trait This question requires knowledge of the listed values.
Which of the following laboratory results indicate that usually express less than 40 percent of HbS. The hematocrit of clients with sickle cell disease is
a client has sickle cell disease? usually decreased between 20 and 30 percent.
1.‐ 30 percent HbS
2.‐ 90 percent HbS
3.‐ Hematocrit of 40 percent
4.‐ Hematocrit of 50 percent

719 Which of the following statements made by a client Correct answer: 1 Clients with sickle cell trait may also develop sickle cell crisis although their symptoms are Use the process of elimination to determine the correct answer.
with sickle cell trait indicates the need for further often milder since only about 30 percent of their hemoglobin is abnormal. The other options
teaching? are rational lifestyle adjustments the client makes in order to deal with the disease.

1.‐ "I don't have to worry about developing sickle cell crisis since I only have the trait."
2.‐ "I will need to seek genetic counseling before I get married and plan children."
3.‐ "I will need to plan my activities avoiding those that decrease my oxygen levels."
4.‐ "I need to avoid the use of recreational drugs and alcohol."

720 Which of the following nursing observations indicate Correct answer: 1 An observation for the client in sickle cell crisis that indicates a positive outcome includes Use the process of elimination to determine the correct answer. Recall that appropriate
that a positive outcome for a client with sickle cell stable vital signs, an oral intake of 3,000 mL/day, and verbalization of pain control. Maintaining urine output is 30 mL per hour.
crisis has been met? an adequate intake is essential to maintain blood flow, decrease pain, and prevent renal
damage.
1.‐ The client has an intake of 3,000 mL per day.
2.‐ The urinary output is 20 cc per hour.
3.‐ Client complains of persistent joint pain.
4.‐ The client has a temperature of 100 degrees F.
721 The nurse is preparing a teaching plan for a client Correct answer: 2 Clients with polycythemia experience satiety and fullness resulting from hepatomegaly and Use the process of elimination to determine the correct answer.
with polycythemia vera on ways to maintain nutrition. splenomegaly. Frequent small meal servings will help maintain adequate nutrition. Foods rich
The nurse should include in the plan which of the in iron are not appropriate since there is an increase in erythrocytes in this condition. Spicy
following? foods will increase the gastrointestinal symptoms, which also include dyspepsia and increased
gastric secretions.
1.‐ Increase intake of foods high in iron.
2.‐ Encourage small, frequent meals rather than three big meals.
3.‐ Increase the amount of red meats and organ meats in the diet.
4.‐ Encourage the use of hot spices in foods to stimulate appetite.

722 A client with thrombocytopenia presents to the Correct answer: 2 Clients with thrombocytopenia have decreased platelet counts below 150,000/uL. The usual This question requires memorization of selected laboratory values.
primary care center. During assessment, the nurse presenting manifestation of this condition is the appearance of petechiae, purpura, and
notices petechiae. The nurse anticipates that which of ecchymoses. The other laboratory values will not explain the petechiae or support the
the following laboratory results would support the presence of a hemostatic disorder.
presence of a hemostatic disorder?

1.‐ Decreased erythrocyte count


2.‐ A platelet count that is less than 150,000 uL
3.‐ An elevated lymphocyte count
4.‐ A hemoglobin value of 14 or more

723 A nurse is admitting a client with a diagnosis of Correct answer: 2 Clients with aplastic anemia usually experience pancytopenia (decreased erythrocytes, Recall that when a client is at risk for infection, answer with the option that provides the
aplastic anemia. Which of the following is the best leukocytes, and platelets). The client with this type of hypoplastic anemia should therefore greatest amount of privacy and protection.
room for the nurse to assign this client? have a room where reverse isolation can be instituted. The client with aplastic anemia is
susceptible to infection as well as hemorrhage. Respiratory isolation requiring negative airflow
(option 2) is not necessary in the care of clients with aplastic anemia.

1.‐ A semi‐private room with a client whose diagnosis is urosepsis.


2.‐ A regular private room at the end of the hall.
3.‐ A private isolation room equipped with a negative airflow.
4.‐ A semi‐private room with a client whose diagnosis is thrombophlebitis.

724 The nurse evaluates the laboratory results of a client Correct answer: 4 In DIC, there is abnormal initiation and formation of blood clots. As clots are formed, more This question requires knowledge of the listed lab tests.
suspected of having DIC (disseminated intravascular end products of fibrinogen and fibrin are also formed. These are called fibrin degradation
coagulopathy). Which of the following laboratory tests products or fibrin split products. Although the PT and PTT are prolonged and the platelet count
is a more specific marker for diagnosing DIC? is reduced in DIC, they could also be a result of other coagulation disturbances.

1.‐ Partial thromboplastin time (PTT)


2.‐ Prothrombin time (PT)
3.‐ Platelet count
4.‐ Fibrin‐split products (FSP) or fibrin degradation products (FDP)

725 The husband of a client with disseminated Correct answer: 3 Initially there is an enhanced coagulation mechanism with resulting increase in fibrin and This question reinforces sound teaching and communication principles.
intravascular coagulopathy (DIC) approaches the nurse platelet deposition in arterioles and capillaries in DIC. This results in thrombosis. Although it
and expresses his concern that his wife might be remains controversial in DIC, the use of heparin is aimed at preventing the formation of further
getting the wrong medication after he was told that thrombotic clots that further complicates the bleeding disorder.
the client was receiving heparin. The nurse's best
response is:
1.‐ "I understand your concern, but the doctors know what they are doing."
2.‐ "Let me make sure that I have not misread the physician's order."
3.‐ "The drug is used to interrupt coagulation in capillaries and arterioles."
4.‐ "Please ask the physician as to why this medication is being given."
726 A client is brought to the emergency department Correct answer: 4 Heat stroke is a life‐threatening situation, and interventions to cool the body must be Recognize that bringing the client's temperature down is the priority and should be
after being found with a body temperature of 106°F. accomplished quickly. Removing the clothing and cooling by evaporation is the most effective accomplished before completing the other options.
The client is unresponsive, hypotensive, and intervention to accomplish cooling quickly. A complete health assessment and documentation
tachypneic. A medical diagnosis of heat stroke is made. are important but after the cooling process has begun. Core body temperatures will be utilized
What would be the nurse's priority intervention in the to monitor effectiveness of treatment.
care of this client?

1.‐ Obtain a complete health assessment


2.‐ Obtain an oral temperature to monitor effectiveness of treatment
3.‐ Contact next of kin for any advanced directives
4.‐ Remove all clothing, wrap in wet sheets and cool with fans

727 A client is being weaned from a ventilator. Arterial Correct answer: 3 Evaluate the pH first to determine acidosis or alkalosis. Then evaluate This question requires knowledge of respiratory acidosis and ABGs.
blood gases drawn prior to extubation reveal: pH 7.32; PaCO&lt;sub&gt;2&lt;/sub&gt; as the metabolic component. The client's pH &amp;lt; 7.35 and
PaO&lt;sub&gt;2&lt;/sub&gt; 90 mmHg; PaCO&lt;sub&gt;2&lt;/sub&gt; &gt; 45mm Hg indicate a state of respiratory acidosis and
PaCO&lt;sub&gt;2&lt;/sub&gt; 56 mmHg; indicates that the client is not tolerating the weaning process. Metabolic alkalosis would be
HCO&lt;sub&gt;3&lt;/sub&gt;&lt;sup&gt;‐&lt;/sup&gt; indicated by a pH &gt; 7.45 and a HCO&lt;sub&gt;3&lt;/sub&gt;&lt;sup&gt;‐&lt;/sup&gt; &gt;
26 mEq/L. The nurse calls the physician with these 26 mEq/L. Respiratory alkalosis would be seen in a client with a pH &gt; 7.45 with a
results because they indicate that the client is in a PCO&lt;sub&gt;2&lt;/sub&gt; &amp;lt; 35 mmHg. Metabolic acidosis would be indicated in a
state of: client with a pH &amp;lt; 7.35 with a HCO&lt;sub&gt;3&lt;/sub&gt;&lt;sup&gt;‐&lt;/sup&gt;
&amp;lt; 21 mEq/L.

1.‐ Metabolic alkalosis.


2.‐ Respiratory alkalosis.
3.‐ Respiratory acidosis.
4.‐ Metabolic acidosis.

728 A client presents to the emergency department with Correct answer: 3 The primary assessment includes the assessment of airway, breathing, circulation, and Differentiate the primary and secondary assessment. Select the distractor that is
a stab wound to the neck. During the primary disability. Although a decrease in neuromuscular function and loss of rectal tone would specifically part of the primary assessment.
assessment of this client who sustained a traumatic indicate neurological damage, they would not be included in the primary assessment. Neck
injury, the nurse would assess for which of the range of motion would not be included until cervical spines have been cleared.
following?
1.‐ Neuromuscular function of the hand
2.‐ Rectal tone
3.‐ Excessive uncontrolled bleeding
4.‐ Neck range of motion

729 A client is scheduled for biopsy to rule out lymphoma. Correct answer: 1 Histological isolation of Reed‐Sternberg cells in lymph node biopsy examination is a diagnostic This question requires knowledge of lymphoma.
The nurse understands that a biopsy finding suggestive feature of Hodgkin's lymphoma. Philadelphia chromosome is attributed to chronic
of a lymphoma is the presence of: myelogenous leukemia.

1.‐ Reed‐Sternberg cells.


2.‐ Philadelphia chromosome.
3.‐ Epstein Barr virus.
4.‐ Herpes simplex virus.

730 The nurse prepares a client for insertion of a Correct answer: 3 A pulmonary artery catheter will provide information about the function of the left ventricle Eliminate options that are inconsistent with the direct actions of a pulmonary artery
pulmonary artery catheter. Preprocedural teaching for when the balloon is wedged. The pulmonary artery catheter does not directly determine the catheter.
this client will include which of the following cardiac output and cardiac index. An arterial line is used to directly monitor the client's arterial
statements? pressure.
1.‐ "The catheter will assist in directly monitoring your arterial pressure."
2.‐ "The catheter will provide information about your cardiac output."
3.‐ "The catheter will provide information about your left ventricular function."
4.‐ "The catheter will provide information about your cardiac index."

731 In order to communicate effectively with a client who Correct answer: 4 The best method to improve communication with the client is to eliminate background noises The core issue of the question is the appropriate strategy for communicating with a client
has sensorineural hearing loss caused by presbycusis, that could interfere with hearing. The client should be approached from the front so as not to who is hearing impaired. Recall that clients rely on visual cues and can benefit from reduced
the nurse should do which of the following to improve frighten him or her. The nurse should use normal pronunciation of words, speak in normal background noise to aid in answering the question.
communication with the client? tones, and refrain from shouting, which is demeaning and not helpful.

1.‐ Approach the client from behind.


2.‐ Use the mouth to exaggerate word pronunciation.
3.‐ Shout initially to get the client’s attention.
4.‐ Turn down background noise from radio or TV before speaking.

732 A client has completed a full course of antibiotics for Correct answer: 4 Ear pain is the most common symptom of otitis media that motivates clients to seek health The critical words in the question are most common, which tell you it is necessary to
acute otitis media. The nurse conducting a follow‐up care; secondary or associated symptoms include fever, nausea and vomiting, dizziness, and prioritize the options in terms of the frequency of their occurrence. Use nursing knowledge
assessment determines whether medication therapy hearing impairment. and the process of elimination to make this selection.
was effective by questioning the client about relief
from which of the following most common presenting
symptoms?

1.‐ Dizziness
2.‐ Impaired hearing
3.‐ Nausea and vomiting
4.‐ Ear pain

733 A client has undergone myringotomy. The nurse Correct answer: 2 Myringotomy is a surgical procedure that perforates the tympanic membrane to allow The core issue of the question is identification of activities that could be harmful to the
working in an ambulatory surgery center would drainage from the middle ear. Postoperatively, the client should avoid getting water into the client while healing is occurring after surgery. Recall that it is necessary to avoid getting the
instruct the client to avoid which of the following ear canal, which could potentially enter the middle ear. The other activities are not risks to the surgical area wet to make the appropriate selection.
activities while healing is occurring? client.
1.‐ Gardening
2.‐ Swimming
3.‐ Softball
4.‐ Bowling

734 A 68‐year‐old female client tells the ambulatory care Correct answer: 1 Presbycusis is the most common form of sensorineural hearing loss in older adults. Otalgia is The core issue of the question is the ability to identify age‐related changes in hearing in an
nurse during a routine visit that she has recently an earache; otitis externa is infection in the external auditory canal and can occur in clients of older adult. Use nursing knowledge and the process of elimination to make a selection.
noticed a decline in her ability to hear. The nurse any age. Meniere’s disease is an inner ear disorder that primarily affects middle‐aged adults.
documents this information on the client’s health
record, suspecting that this client most likely is
exhibiting which of the following?
1.‐ Presbycusis
2.‐ Otitis externa
3.‐ Otalgia
4.‐ Meniere’s disease

735 After a client has undergone outpatient surgery for a Correct answer: 2 The client should avoid lying on the operative side following eye surgery in order to minimize The core issue of the question is client teaching about safe and unsafe activities following
right eye cataract removal, the nurse teaches the edema and intraocular pressure. Options 3 and 4 pose no risk to the client. Option 1 is not a cataract surgery. Recall that it is important to avoid positions in which gravity can lead to
client to avoid which of the following when the client problem given the information in the question. Some clients with severe visual impairment or increased edema in order to make the correct selection.
gets home? other health problems may need assistance to move about in the environment.

1.‐ Walking about the house unassisted


2.‐ Lying on the right side
3.‐ Picking up objects that are at waist level
4.‐ Washing dishes in the sink

736 A client has hearing loss that is characterized by Correct answer: 1 When hearing loss is characterized by distortion of sounds, amplification of sound is of little The core issue of the question is the ability to correlate the types of hearing loss that can
distortion of sounds that are heard. The client help because it only increases the intensity of distorted sounds. The other options are be improved with the use of a hearing aid. To do this, reflect on the types of hearing loss
questions the nurse about the benefits of obtaining a incorrect. and the likely effect of a hearing aid for that condition. Use nursing knowledge and the
hearing aid. The nurse would include in a response that process of elimination to make a selection.
a hearing aid will have which of the following effects
for this client?
1.‐ It will intensify the already distorted sounds.
2.‐ It will improve the client’s ability to distinguish words from background noises.
3.‐ It will make sounds louder and clearer.
4.‐ It will have no effect on hearing.

737 A client reports ongoing problems with vertigo. The Correct answer: 4 Vertigo, tinnitus, hearing loss, and a sense of fullness in the ear are classic symptoms of The core issue of the question is identification of signs and symptoms of Meniere’s
nurse should question the client about which of the Meniere’s disease. Nystagmus also occurs with acute attacks. Headache, double vision, and disease. Recall that this is a disorder of the inner ear, thus making symptoms related to
following accompanying manifestations to determine pain are not part of this clinical picture. Purulent drainage suggests infection. balance as well as hearing important to identify. Use nursing knowledge and the process of
whether the client has developed Meniere’s disease? elimination to make a selection.

1.‐ Purulent discharge from the ear and pain


2.‐ Headache and double vision
3.‐ Nausea, vomiting, and headache
4.‐ Tinnitus, hearing loss, and a sense of fullness in the ear

738 The nurse prepares to initiate client teaching for Correct answer: 1 Antivertigo and antiemetic medications, such as meclizine, are used to control symptoms The core issue of the question is the ability to anticipate medications that will be effective
which of the following medications commonly used to associated with Meniere’s disease. Diuretics are used between acute attacks to reduce the in relieving the symptoms associated with Meniere’s disease. To answer correctly, it is
treat a client diagnosed with Meniere’s disease? volume of endolymph and prevent attacks. Glucocorticoids (options 2), beta‐blockers (option necessary to have a core body of knowledge related to pharmacology. Use nursing
4), and analgesics (option 3) are not part of an effective treatment plan. knowledge and the process of elimination to make a selection.

1.‐ Meclizine (Antivert)


2.‐ Dexamethasone (Decadron)
3.‐ Acetaminophen (Tylenol)
4.‐ Propranolol (Inderal)

739 A client with glaucoma has been prescribed Correct answer: 3 Pilocarpine is a miotic agent, which constricts the pupil and thereby stimulates the ciliary The core issue of the question is the ability to anticipate medications that will be effective
pilocarpine (Pilocar). The nurse explaining the use of muscles to pull on the trabecular meshwork surrounding the canal of Schlemm, which in relieving the symptoms associated with glaucoma. To answer correctly, it is necessary to
this medication would state that it is useful because it increases the flow of aqueous humor and decreases intraocular pressure. have a core body of knowledge related to pharmacology. Use nursing knowledge and the
works by: process of elimination to make a selection.

1.‐ Dilating the pupil.


2.‐ Constricting intraocular vessels.
3.‐ Constricting the pupil.
4.‐ Relaxing the ciliary muscles.

740 The initial nursing intervention for a client in the Correct answer: 4 The immediate priority for clients with chemical burns is flushing the affected eye with The critical word in the question is initial, which tells you that more than one or all options
emergency department who suffered a chemical burn copious amounts of normal saline or water. Evaluation of visual acuity is an appropriate could be correct and that it is necessary to prioritize the most important or immediate
to the eyes is to: intervention after flushing. Analgesics, with the exception of topical anesthesia, are not action needed. Whenever there is an injury involving chemicals, the priority action is to
indicated. Antibiotics may be administered after the initial actions have been taken. remove the offending substance.

1.‐ Administer analgesics as prescribed.


2.‐ Evaluate vision with and without prescription eyeglasses.
3.‐ Administer antibiotics as prescribed.
4.‐ Irrigate the eyes with 0.9% saline solution or water.

741 The nurse is caring for a client who is in the recovery Correct answer: 2 Clients with retinal detachment frequently report flashing lights and loss of vision, commonly The core issue of the question is knowledge of complications of cataract surgery. Use
area following cataract surgery. The nurse would ask described as a veil or curtain being drawn across the eye. Retinal detachment is not associated nursing knowledge and the process of elimination to make a selection.
the client about which of the following manifestations with increased lacrimation or tearing (option 1), eye pain (option 3), or change in ocular
that would indicate onset of retinal detachment as a movements (option 4).
postoperative complication?

1.‐ Increased lacrimation


2.‐ Flashing lights and loss of part of the visual field
3.‐ Sudden, severe eye pain
4.‐ Inability to move the eye and loss accommodation

742 A client who was diagnosed with chronic open‐angle Correct answer: 3 Medications that end in ‐olol are beta‐adrenergic blocking agents. When taken as ophthalmic The core issue of the question is the ability to identify adverse effects of medication used
glaucoma has been started on medication therapy with preparations, they can produce systemic effects such as bradycardia, hypotension, and to treat glaucoma. To answer correctly, it is necessary to have a core body of knowledge
timolol maleate (Timoptic). The nurse assesses for bronchospasm. Beta‐adrenergic blockers act as CNS depressants and may also be used to treat related to pharmacology. Use nursing knowledge and the process of elimination to make a
which of the following possible adverse systemic anxiety, but this does not relate to the issue of this question, which is glaucoma. selection.
responses to the drug?
1.‐ Tachycardia
2.‐ Anxiety
3.‐ Bradycardia
4.‐ Hypertension

743 The daughter of an elderly client diagnosed with dry Correct answer: 1 Atrophic or dry macular degeneration results from atrophy and degeneration of the outer The core issue of the question is the ability to discriminate correct information to be used
macular degeneration asks the nurse to explain this layer of the retina. In exudative or wet macular degeneration, blood leaks into the subretinal in teaching clients and/or families about disease processes. Thus, to answer this question, it
disorder. In formulating a response, the nurse would space and scar tissue gradually forms. The resulting loss of vision occurs rapidly and is more is necessary to understand the two types of macular degeneration and how they present in
include information that this condition is characterized profound. Exudative macular degeneration accounts for 90% of all cases of legal blindness. terms of symptoms. Use nursing knowledge and the process of elimination to make a
by which of the following? selection.

1.‐ Atrophy and degeneration of outer pigmented layer of the retina


2.‐ Scar formation between the retina and the choroid
3.‐ Rapid and severe loss of vision
4.‐ Separation of the retina from the choroids

744 The nurse is evaluating the effectiveness of Correct answer: 1 Scleral buckling is correctly described in option 1. It is used in conjunction with laser The core issue of the question is evaluation of a client’s understanding of a surgical
preoperative teaching done for a client who needs to photocoagulation or cryothermy to achieve the best results. Option 2 defines pneumatic procedure. To select correctly, it is necessary to be able to identify how the surgery will be
have repair of a detached retina. The nurse evaluates retinopexy. Options 3 and 4 are incorrect. performed. Use nursing knowledge and the process of elimination to make a selection.
that the client understands the procedure if the client
indicates that scleral buckling involves

1.‐ Using a piece of silicone to indent the sclera to increase contact between the retinal layers.
2.‐ Injecting air into the vitreous humor to push the detached retina against the choroid.
3.‐ Removing the torn segment of retina.
4.‐ Replacing the torn segment of the retina with donor retinal tissue.

745 The nurse would take which of the following actions Correct answer: 3 Prevention or reduction of intraocular pressure (that may accompany blunt trauma to the The core issue of the question is identification of a correct action in the treatment of eye
when the client first comes into the emergency eye) can be accomplished by the use of semi‐Fowler’s position and administration of a trauma. Recall that injuries are characterized by formation of edema at the site, and
department with blunt trauma to the eye? carbonic anhydrase inhibitor, such as acetazolamide (Diamox). Semi‐Fowler’s position also therefore early actions for any injury may involve proper positioning of the client to reduce
reduces edema formation at the site of injury when compared to lying flat. Constriction of the edema formation.
pupil with miotics is not indicated. Blunt trauma does not cause loss of intraocular contents;
and no foreign body is present.
1.‐ Irrigate the eye to remove foreign substances.
2.‐ Administer miotics.
3.‐ Place the client in semi‐Fowler’s position.
4.‐ Prevent loss of intraocular contents.

746 Which of the following statements indicates that the Correct answer: 1 The client should avoid activities that raise intraocular pressure, such as bending over. The The core issue of the question is the ability to evaluate client understanding of
client has understood home care instructions following client should sleep on the nonoperative side. Activities involving the eyes are done at the postoperative instructions following cataract surgery. Evaluate each of the options in terms
cataract surgery? advice of the surgeon. Typically, an eye shield is used at night, and dark protective glasses are of the truth of the statement, since the question contains the critical word understood.
worn during the day.
1.‐ “I should not bend over to pick up objects from the floor.”
2.‐ “I can sleep on whichever side I want as long as my head is raised.”
3.‐ “I may not watch television for 6 weeks.”
4.‐ “I should keep the protective eye shield in place 24 hours a day.”

747 The nurse is assessing a client’s hearing using the Correct answer: 4 To perform the Weber test, the nurse places a vibrating tuning fork on the midline vertex of The core issue of the question is the ability to correctly analyze results of physical
Weber test. The nurse documents the client’s result as the client’s head. The sound should normally be heard equally in both ears. Sound that assessment techniques. Use nursing knowledge and the process of elimination to make a
normal if the client reports which of the following after lateralizes to one side indicates either conductive hearing loss on that side or sensorineural selection.
placement of the vibrating tuning fork on the midline hearing loss on the opposite side.
vertex of his head?
1.‐ Sound lateralizes to the left ear.
2.‐ Sound lateralizes to the right ear.
3.‐ Sound is absent in both ears.
4.‐ Sound is heard equally in both ears.

748 The nurse has an order to do an otic irrigation to the Correct answer: 4 The client should lie on the affected side following the irrigation to allow gravity to further The core issue of the question is the ability to correctly perform the nursing procedure of
left ear of an assigned client. The nurse implements assist in draining the ear canal. The irrigant should be directed along the wall of the external eye irrigation. Use nursing knowledge and the process of elimination to make a selection.
this procedure correctly by doing which of the canal, not the center (which could damage the tympanic membrane). Usually, 50 to 70 mL of
following? solution are used, according to the size of the syringe used for the procedure. A single cotton
ball is placed loosely into the external meatus to absorb any remaining irrigant after the
procedure.
1.‐ Direct the stream into the center of the ear canal.
2.‐ Draw up 250 mL of solution.
3.‐ Pack the external ear tightly with cotton balls after instilling the irrigant.
4.‐ Help the client to lie on the left side after the irrigation is finished.

749 Which of the following would be least useful for the Correct answer: 1 Warm compresses, not cold, should be used as part of the management of conjunctivitis. The critical words in the stem of the question are least useful, indicating that the correct
nurse to use as part of the collaborative management Warm compresses help relieve discomfort and reduce inflammation by increasing circulation answer is an option that is either incorrect or lowest priority. Use nursing knowledge about
of a client who has conjunctivitis? to the area. The other options contain items that are part of the standard collaborative care of the client with conjunctivitis and the process of elimination to make a selection.
management for conjunctivitis. Dark sunglasses are helpful in reducing photophobia.

1.‐ Cold eye compresses


2.‐ Careful handwashing
3.‐ Antibiotic therapy
4.‐ Dark sunglasses

750 The nurse who is administering an ophthalmic Correct answer: 2 The nurse should apply pressure to the inner canthus (nasolacrimal duct) during and for at The core issue of the question is the ability to administer eye medication correctly. Use
medication to a client would do which of the following least 30 seconds following instillation, according to agency procedure. Doing so will help nursing knowledge and the process of elimination to make a selection.
as correct procedure? prevent systemic absorption of the medication. The medication should be dropped into the
lower conjunctival sac. The eye should not be rubbed after instillation of the medication. The
nurse should wait from 1 to 5 minutes between drops, depending on the medication and
manufacturer’s recommendations.
1.‐ Drop the medication onto the eyeball.
2.‐ Apply pressure to the inner canthus while administering the medication.
3.‐ Rub the eye with a cotton ball after instillation.
4.‐ Wait 10 seconds between drops.

751 The nurse has administered a dose of antibiotic Correct answer: 1 It is appropriate to comfort the child following a painful procedure. Option 1 provides support The core issue of the question is the best response to a 5‐year‐old client who is responding
intramuscularly to a 5‐year‐old client with tonsillitis. and comfort. By fulfilling the child’s request, the nurse allows the child to regain some control to a painful procedure such as an injection. Eliminate options 2 and 3 because these
The child cries for a Band‐Aid over the injection site. over the situation. It is not appropriate to argue with the child. communications do not meet the client’s immediate need for comfort following the
Which of the following is the best nursing action? injection, and option 4 is incorrect because the issue is a psychological/comfort issue, not a
question of stopping bleeding.
1.‐ Apply a Band‐Aid.
2.‐ Ask the child why he wants the Band‐Aid.
3.‐ Explain to the child that a Band‐Aid dressing is not necessary.
4.‐ Show the child that the site is not bleeding.

752 Which is the most appropriate nursing intervention Correct answer: 4 A febrile infant is at risk for fluid volume deficit resulting from larger‐than‐normal insensible The core issue of the question is an appropriate nursing intervention when a client has
when caring for an infant with acute otitis media and fluid losses and decreased fluid intake. It is contraindicated to sponge with cool water or add hyperthermia. Use nursing knowledge and the process of elimination to make a selection.
102.7 degrees F. fever? blankets. Intake of solid food is less important than preventing dehydration.

1.‐ Provide sponging with cool water to reduce fever.


2.‐ Encourage the baby’s intake of solids to maintain adequate caloric intake.
3.‐ Swaddle the baby in layers of blankets to promote comfort and prevent chills.
4.‐ Offer fluids frequently to prevent dehydration.

753 Which of the following would be the most Correct answer: 3 A symptom of pharyngitis is sore throat and difficult swallowing, which could lead to the The core issue of the question is the ability to determine priority concerns for a client with
appropriate nursing diagnosis for a child with refusal to drink. Thus, risk for deficient fluid volume is an appropriate diagnosis. Option 2 pharyngitis by selecting a nursing diagnosis. Whenever the airway is involved, first think
pharyngitis? would apply when the client cannot clear secretions from the respiratory tract, which is not about the ABCs and then think about hydration/food intake as the next priority of
applicable to this question. Options 1 and 4 are not pertinent to this health problem. physiological needs using Maslow’s hierarchy.

1.‐ Disturbed body image


2.‐ Risk for ineffective airway clearance
3.‐ Risk for deficient fluid volume
4.‐ Impaired growth and development

754 The nurse recommends a humidified atmosphere for Correct answer: 4 Humidifying the air can prevent dry mucous membranes and recurrence of epistaxis. Other The core issue of the question is an understanding of the rationale for nursing actions for a
a child with recurrent epistaxis. When questioned by options do not correctly identify the benefit of humidity for a client with recurrent nosebleed. client with recurrent epistaxis. Use nursing knowledge and the process of elimination to
the parent, the nurse explains that which of the make a selection.
following is a benefit of humidity for the child?

1.‐ Liquefies secretions


2.‐ Improves oxygenation
3.‐ Increases ventilation
4.‐ Prevents drying of mucous membranes

755 A 9‐month‐old infant has been diagnosed with Correct answer: 2 Children of this age cannot understand the necessity of cooperating with medication The core issue of the question is identification of correct procedure for administering an
conjunctivitis. An antibiotic ointment has been administration. Mummying the child reduces the risk of injury from the ointment tip and ophthalmic medication to a child. Use nursing knowledge of this basic procedure and the
prescribed. In teaching the mother to administer this promotes adequate dosing. Applying ointment to the eyes of a sleeping child would increase process of elimination to make a selection.
drug, the nurse would recommend which of the the child’s fears. The ointment is instilled in the lower conjunctival sac, not on the lids.
following?
1.‐ Wait until the child is asleep to instill the ointment.
2.‐ Mummy the child to prevent accidental injury.
3.‐ Place the ointment on a swab and spread across closed lids.
4.‐ Use sterile gauze to apply the ointment to the lids.
756 The nurse would provide which of the following Correct answer: 2, 3 Typical instructions after fluorescein angiography include increased fluid intake to aid in dye The core issue of the question is knowledge of postprocedure instructions to a client
instructions to the client after fluorescein excretion. The client should know that the dye causes temporary skin discoloration in the following fluorescein dye eye examination. Use nursing knowledge and general concepts of
angiography? Select all that apply. injected area and temporary green discoloration of urine that resolves when dye is fully procedures that utilize contrast dye to make your selections.
excreted. The client should avoid sunlight or other bright light sources for several days until
pupil dilation returns to normal. Although the client should rest after the procedure, it is not
necessary to lie down with eyes closed for 12 hours. Headache and blurred vision are not
expected.
1.‐ Encourage reduced fluid intake to limit intraocular pressure.
2.‐ The dye causes temporary green discoloration to urine.
3.‐ Avoid sunlight until pupil size returns to normal.
4.‐ Lie down with eyes closed for 12 hours postprocedure after returning home.
5.‐ Expect headache and blurred vision for approximately 24 hours after the procedure.

757 The nurse is teaching a mother to administer Correct answer: 2 Pulling the pinna down and back straightens the auditory canal of an infant, permitting the Although the stem of the question is quite long, the basic question is: “To open the ear
eardrops to her infant. After teaching is complete, the instillation of eardrops. Pulling the pinna up and back is the proper method for straightening canal of the infant, how do you move the pinna?”
nurse will know the mother understands when the the canal of an adult client. The other options are incorrect.
mother is observed administering ear drops by pulling
the pinna of the ear:

1.‐ Down and forward.


2.‐ Back and down.
3.‐ Up and forward.
4.‐ Up and back.

758 The nurse is assessing the ears of a 4‐year‐old client. Correct answer: 4 A normal tympanic membrane is pearly gray and mobile when a puff of air is applied to it Knowledge of normal assessment findings is the key to answering this question normally.
Upon examination, the tympanic membrane is mobile using an attachment on an otoscope. A red, bulging non‐mobile tympanic membrane would be
and is pearly gray in color. The nurse concludes: typical with otitis media, a common ear infection in young children.

1.‐ This finding would be normal for an adolescent but not a preschooler.
2.‐ There will be permanent hearing loss for this child.
3.‐ The condition of the tympanic membrane will resolve spontaneously in 4 to 5 days.
4.‐ This is the appearance of a normal tympanic membrane.

759 The school health nurse has seen three children who Correct answer: 3 The child is no longer considered contagious after completing 24 hours of antibiotic therapy. Option 2 does not really define a time that the student could return to school.
have symptoms compatible with bacterial Until 24 hours of antibiotic therapy, the child would be contagious. Conjunctivitis is not a systemic disease. Therefore, eliminate options 2 and 4. The core issue
conjunctivitis (pink eye). The nurse calls for the parents of the question is the ability to transmit infection to others; use this concept to choose
to bring the children home and writes in a note that between options 1 and 3.
they can safely return to school:
1.‐ When the eyes stop tearing.
2.‐ If complications or sequelae do not occur.
3.‐ After being seen by a physician and taking antibiotic therapy for 24 hours.
4.‐ If they do not have any systemic symptoms.

760 The nurse counsels the parents of an infant with Correct answer: 3, 5 Exposure to secondhand smoke increases incidence of otitis media so this should be avoided Consider which options will have an effect on the middle ear. Warm compresses cannot be
chronic otitis media. Which of the following should the to reduce the risk of future episodes of otitis media. Preventing the infant from falling asleep applied to the middle ear so that option is incorrect. From this point, recall risk factors for
nurse recommend to prevent future infections? Select with a pacifier will also help because saliva from sucking cannot accumulate and enter the otitis media to aid in choosing the correct options.
all that apply. Eustachian tube. Infants who feed in the supine position have an increased risk of otitis media.
Medications such as a nasal decongestant would have side effects and should be avoided
unless specifically needed. Warm compresses will not prevent future infection.

1.‐ The parents should routinely administer nasal decongestant drops to their infant.
2.‐ The parents should position the child supine for all feedings.
3.‐ The parents should avoid exposing their infant to tobacco smoke.
4.‐ The parents should apply warm compresses to the ear daily.
5.‐ The parent should not allow the baby to fall asleep with a pacifier.

761 A 4‐year‐old has been diagnosed with amblyopia. The Correct answer: 2, 3 Although strabismus is associated with a positive family history in many cases, the mode of Knowledge of amblyopia, its causes and treatment, will assist in correctly answering this
nurse who is providing the parents with information inheritance is unknown. The pathophysiology is misalignment of the eyes causing the brain to straight forward question.
about this diagnosis should include which of the stop receiving the signal of the affected eye. Without treatment, including patching, corrective
following items of information? Select all that apply. lenses, and muscular exercises, the damage will become permanent.

1.‐ The eye itself has lost the cells that send the visual data to the brain
2.‐ The child will need to wear a patch over his unaffected eye
3.‐ If not treated, the child may become permanently blind in the affected eye
4.‐ The brain shuts off the signal from both eyes because of misalignment of the eyes
5.‐ Amblyopia is diagnosed with the Snellen E chart

762 A child who is otherwise healthy is brought to the Correct answer: 4 The child should be positioned where pressure to the nostrils can be applied and upright to Three of the four options relate to position. Consider which position would be safe for the
emergency room with an episode of epistaxis. To stop avoid excessive amounts of blood swallowed. Hyperextending the head opens the airway and child with epistaxis.
the episode of epistaxis, the nurse would do which of increases the risk of aspiration. Positioning the client in Trendelenburg position also increases
the following? the risk for aspiration by allowing blood to accumulate in the nasopharynx.

1.‐ Position the child supine with head hyperextended


2.‐ Apply ice on the upper lip
3.‐ Position supine in Trendelenburg position
4.‐ Position upright with head tilted forward

763 The nurse working in a pediatric outpatient clinic Correct answer: 3 Early identification and treatment of visual impairment can prevent significant vision loss in The critical words in the question are “beginning at.” This tells you the correct option is
routinely performs assessments to detect visual children at risk. For this reason, screening should be started at birth. the one that indicates the initial start time for visual assessment.
impairment beginning at what time?
1.‐ In the preschool years
2.‐ At onset of puberty
3.‐ At birth
4.‐ During the kindergarten year

764 The nurse working in a pediatric clinic concludes that Correct answer: 2 Symptoms of decreased visual acuity are squinting to focus, excessive tearing of the eyes, and Language would not be affected by diminished vision. To distinguish among the other
which assessment finding in a child indicates a risk for rubbing of the eye. options, recall signs of visual impairment at various ages.
visual impairment?
1.‐ Delayed language development
2.‐ Excessive rubbing of the eyes
3.‐ Disinterest in watching television
4.‐ Bilateral symmetric corneal light reflex

765 To prevent amblyopia, the nurse would implement Correct answer: 3 To encourage the child to use the weaker, deviating eye and in an attempt to strengthen the Note the choices offered. One choice is patching the affected eye, another choice is
which of the following as part of therapeutic muscles of the affected eye, the unaffected eye is patched. patching the unaffected eye. More than likely, one of these choices is the right answer
management of a child with strabismus? because they are opposites.

1.‐ Patching of the affected eye


2.‐ Mydriatic eye drops
3.‐ Patching of the unaffected eye
4.‐ Antibiotic eye drops
766 The nurse is caring for a school age child with Correct answer: 1, 5 Strabismus (or cross‐eyes) affects the appearance and visual acuity of the child. This can Consider nursing diagnoses related to visual problems and physical appearance.
strabismus. In planning the nursing care, the nurse affect parental relationships. It is not a painful disease and there is no risk for infection. The
would identify which of the following as appropriate circulation to the eye is normal.
nursing diagnoses? Select all that apply.

1.‐ Sensory/perceptual alterations (visual)


2.‐ Risk for infection
3.‐ Body image disturbance
4.‐ Pain
5.‐ Risk for ineffective family coping related to visual impairment of child

767 A child is to receive eye ointment that has been Correct answer: The procedure for instilling eye ointment begins with washing hands and applying clean The core concept is safe administration of eye ointment. Recall that ointment needs to be
ordered to treat conjunctivitis. Indicate on the picture gloves. After ensuring the medication is at room temperature and removing any discharge administered in the lower conjunctival sac to follow correct administration procedure.
shown where the nurse should place the ointment. from the eye, the lower lid is pulled down to make a sac and the ointment is applied there. The
tip of the tube should never touch the eye surface, and the tip is held parallel to the eye so as
to prevent injury if the child should move suddenly.

768 The nurse is caring for a 6‐year‐old child who just Correct answer: 4 The nurse observes increased swallowing rather than decreased swallowing when there is The choices here have multiple responses; consider each option individually. The first
returned to the day‐surgery recovery area following a bleeding following tonsillectomy. The child may also spit out red blood from the mouth at this choice is either tachy‐ or bradycardia. Tachycardia is seen in early hemorrhage, eliminating
tonsillectomy, adenoidectomy, and bilateral time. Tachycardia and hypotension are late signs of significant blood loss, and these would be option 2. The second choice is hyper‐ or hypotension. The most common option is
myringotomy with insertion of tympanostomy tubes. consistent with active uncontrolled bleeding. The child would not exhibit hypertension or hypotension, eliminating option 1. Choose option 4 over option 3, recalling that there is
Which assessment data indicates to the nurse that the bradycardia. increased swallowing (early) or possible hemoptysis (late) when there is surgical bleeding.
child is experiencing active, uncontrolled bleeding at
the operative site?

1.‐ Tachycardia, hypertension, hemoptysis


2.‐ Bradycardia, hypotension, increased swallowing
3.‐ Tachycardia, hypotension, decreased swallowing
4.‐ Tachycardia, hypotension, increased swallowing

769 The nurse caring for a pediatric client following Correct answer: 1 Age and developmental level affect the pain response of a child. Infants are less able to Consider the two core concepts in this question—response to pain and children as they
tonsillectomy would keep in mind which of the communicate their feelings than an older child and usually demonstrate restlessness and differ from adults. Use knowledge of postoperative care and appropriate developmental
following nursing considerations when planning care crying behaviors. Adolescents are able to describe their pain sensations. Children do not considerations to choose the correct answer.
for this child? generally recover from painful procedures more quickly than adults (option 2). Children do not
have higher pain thresholds than adults (option 4) and do require opioid analgesics (option 3).

1.‐ A child’s behavioral response to pain is affected by age and developmental level.
2.‐ Recovery from a painful procedure occurs at a faster rate in children as compared to adults.
3.‐ Opioid analgesic use in children is dangerous because of increased risk of addiction and respiratory depression.
4.‐ The immaturity of the nervous system in young children provides them with an increased pain threshold.

770 The nurse is beginning an otoscopic examination of Correct answer: 4 Uncooperative pediatric clients may need to be gently restrained long enough to accomplish Critical words are “2‐year‐old” and “cries, kicks, and pulls away.” This child, at the age of 2,
the ear of a 2‐year‐old child. The child cries, kicks, and the assessment or procedure that is necessary. Parents may be able to assist with this is too young to understand. Reason that the answer is an intervention that allows for the
pulls away from the nurse. The nurse should take effectively. A 2‐year‐old will not listen to explanations (option 1) and is not likely to respond to exam to proceed.
which of the following actions? pleas for acting maturely (option 3). The exam should not be postponed until the next yearly
exam (option 2).
1.‐ Explain to the child why the ear must be examined.
2.‐ Postpone the examination until the next clinic visit in one year.
3.‐ State, “I thought you were going to be grown up for me today.”
4.‐ Get assistance to gently restrain the child to proceed with the exam.
771 During a day‐surgery hospitalization experience for Correct answer: 1 One of the greatest fears of preschoolers is fear of mutilation. Options 2 and 3 Knowledge of normal stress reaction to hospitalization for the developmental stage is
tonsillectomy for a 4‐year‐old child, the nurse (immobilization and premature death) are not developmentally appropriate concerns of a necessary to answer the question. Compare each response with the age of the child to
anticipates that the child will most likely be fearful of: preschooler. Unfamiliar caregivers (option 4) could be a concern for any child, but is less so determine which behavior is developmentally appropriate.
than the fear of mutilation for a child of this age.
1.‐ Intrusive procedures.
2.‐ Immobilization.
3.‐ Premature death.
4.‐ Unfamiliar caregivers.

772 The nurse is performing an assessment of a 14‐month‐ Correct answer: 1 The tympanic method is preferred. It is quick, accurate, and convenient. Oral temperature can Consider the age of the child and safety to eliminate the rectal and oral temperatures.
old toddler admitted to the day surgery unit for be obtained on a cooperative child age 3 and older. A rectal temperature is obtained as a last Then consider the need for accuracy to eliminate the forehead strip.
bilateral myringotomy and placement of resort, when other methods are not possible.
tympanostomy tubes. To take an accurate
temperature, the nurse should use:
1.‐ A tympanic thermometer with disposable speculum.
2.‐ An oral thermometer with disposable plastic sheath.
3.‐ A rectal thermometer with disposable plastic sheath.
4.‐ A temperature strip placed on the child’s forehead.

773 Which of the following care measures should the Correct answer: 1, 4, 5 Crusting of dried exudate is common with bacterial conjunctivitis. The parents will need to Critical words are “home‐care measures” and “bacterial conjunctivitis.” Knowledge of
nurse include when teaching home‐care of a child who know how to administer the eye drops or ointment. Washing the hands frequently will reduce conjunctivitis as a contagious disease and management will help to answer the question
has bilateral bacterial conjunctivitis? (Select all that the spread of the infection, which is hand‐to‐eye and spreads easily to other children. The use correctly. Consider both medical and nursing interventions as necessary to treat this
apply.) of antihistamines and topical anesthetics is not indicated in the management of bacterial infection.
conjunctivitis.
1.‐ Use of warm, moist disposable compresses to remove crusting
2.‐ Use of oral antihistamine medication to relieve eye itching
3.‐ Use of topical anesthetics applied to relieve discomfort
4.‐ Teach the parents to administer the antibiotic eye medication
5.‐ Teach the child to wash hands frequently throughout the day

774 The nurse should teach the parent of an infant Correct answer: 4 Increased fussiness and elevated temperature are expected symptoms of viral pharyngitis in The core concept is that the illness is viral. Viral infections are usually treated
diagnosed with viral nasopharyngitis to notify the infants. A cough may occur because of local irritation. Symptoms of ear infection can occur symptomatically, but complications should always be noted. Knowledge of the
health care provider if which of the following occurs? because of secondary infection and should be reported to the health care provider. complications of nasopharyngitis is then necessary to answer correctly.

1.‐ Increased fussiness


2.‐ Cough
3.‐ Temperature greater than 98.6 degrees F
4.‐ Signs of ear infection

775 Decongestant nasal drops are prescribed for an infant Correct answer: 1 Eliminating contact or sharing of items with the infected person can reduce the potential Critical words are “nasal drops are prescribed for an infant.” Specific knowledge of the
with nasopharyngitis. The nurse should include which spread of infection to other family members. Medication should be used as specifically correct method to administer nasal drops will help to answer this question correctly.
of the following in instructions given to the caregiver ordered (option 4). Medication should not be saved for use during future illness (option 2). The
who will be administering the drops? dropper should not be inserted “as far as possible” due to risk of injury to the infant.

1.‐ “Do not use the drops or dropper for any other family member.”
2.‐ “Save any remaining medication for the next time the child is congested.”
3.‐ “Administer the drops frequently, until the nasal congestion subsides.”
4.‐ “Insert the dropper tip as far into the infant’s nose as is possible.”
776 The nurse teaches the importance of finishing the full Correct answer: 4 Rheumatic fever can follow an infection of certain strains of group‐A beta‐hemolytic The core question is “what is a sequela of untreated streptococcus infection?” Knowledge
course of oral antibiotic therapy to the caregivers of a streptococci. Otitis media is an ear infection that can be caused by many organisms, but is not of antibiotic resistance and complications from strep help to answer the question correctly.
toddler who has streptococcal pharyngitis. The nurse the priority concern related to a strep infection. Diabetes insipidus and nephrotic syndrome
explains that a potential complication of inadequately are pituitary and renal disorders, respectively, but are not related to sequelae of strep
treated streptococcal infection is: infection.

1.‐ Otitis media.


2.‐ Diabetes insipidus.
3.‐ Nephrotic syndrome.
4.‐ Acute rheumatic fever.

777 The physician orders amoxicillin (Amoxil) 500 mg IVPB Correct answer: 1 Amoxicillin is given only by the oral route. Because of this, the nurse cannot give the dose and Critical words are “amoxicillin” and “IVPB.” The question identifies an error in ordering
q 8 hours for a pediatric client with tonsillitis. What is must question the order. There is no problem with the dosage or the frequency. amoxicillin. Focus on route, dose, and timing to determine the right answer.
the appropriate nursing action?

1.‐ Question the order because the route of administration is incorrect.


2.‐ Give the medication as ordered.
3.‐ Question the order because the dosage is too high.
4.‐ Question the order because the dosing frequency is incorrect.

778 The nurse administers cefprozil (Cefzil) as ordered to Correct answer: 3 Candida infections are a common side effect of antibiotic therapy due to alteration of the Key words are “cefprozil” and “patches of white on the child’s oral mucosa that cannot be
a 22‐month‐old client with bacterial pharyngitis. The normal bacterial flora by the antibiotic agent. The white patches in the mouth do not removed.” Knowledge of the side effects of antibiotic therapy is necessary to answer the
nurse notes patches of white on the child’s oral represent allergic reaction, herpes simplex infection, or mumps. question correctly. This question asks the learner to make the association between
mucosa that cannot be removed. Which condition antibiotics and the development of thrush infection due to loss of normal flora.
does the nurse suspect?
1.‐ Allergic reaction to the medication manifested by the development of stomatitis
2.‐ A herpes simplex virus infection
3.‐ Oral thrush caused by Candida albicans
4.‐ Mumps

779 A pediatric client has been diagnosed with otitis Correct answer: 2 The nurse must emphasize the importance of completing the full course of antibiotic therapy, Key words are “diagnosed with otitis media” and “highest priority.” Although all choices
media. The nurse should place highest priority on even though symptoms may have resolved before the antibiotic is finished. are knowledge that should be discussed with the parents, how to administer the
teaching the parent: medication and completing the medication regimen must be done immediately. Although
ear drops may be given, the antibiotics will be given systemically, making completing the
regimen the highest priority.
1.‐ How to administer ear drops.
2.‐ The importance of completing the full course of antibiotic therapy.
3.‐ About myringotomy and tympanostomy tube insertion.
4.‐ About eliminating environmental allergens.

780 Nursing care of the child who is postoperative for a Correct answer: 2, 5 The nurse must observe the post‐tonsillectomy client for signs of excessive bleeding or Critical words are “nursing care of a child who is postoperative for a tonsillectomy.” Two
tonsillectomy should include: (Select all that apply.) hemorrhage from the operative site. In the posterior pharynx, the bleeding can be concealed choices mention temperatures—warm compresses on neck area and warm liquids—and
by the child swallowing the blood. Applying heat to the neck, warm liquids, or giving a straw one must be wrong. Since warmth promotes vasodilatation, that would make that one
would be contraindicated, as this could cause bleeding. wrong. All correct activities relate to reducing the risk of hemorrhage.

1.‐ Apply warm, moist compresses to the neck area.


2.‐ Observe for excessive swallowing.
3.‐ Maintain the child in a supine position.
4.‐ Offer warm liquids with a straw for the child to sip.
5.‐ Report any trickle of bright red blood.
781 The nurse is caring for a child with a common cold Correct answer: 3 The common cold is a viral infection. It is self‐limiting, with symptoms lasting about 4 to 10 Key words are “child with a common cold” and “primary goal of nursing care.” Although
(nasopharyngitis). The primary goal of nursing care is days. Therefore, emphasis is on symptom management. Antibiotics are not indicated for a viral there is information about the child’s age in the question, nothing in the choices is
directed toward: infection. Nutrition and injury prevention are general concerns for a child, but do not relate to differentiated by age. Therefore, that can be ignored. The key concept is the medical
the question. diagnosis and determining the purpose of nursing interventions.
1.‐ Preventing injury.
2.‐ Promoting nutrition.
3.‐ Relieving symptoms.
4.‐ Administering antibiotics.

782 The nurse obtains a health history on a pediatric Correct answer: 3 Turning up the volume loudly is a behavioral indicator suggesting hearing impairment. The Critical words are “sign” and “possible hearing impairment.” Knowledge of hearing
client. A sign alerting the nurse to possible hearing other options are behaviors that are not consistent with or indicative of hearing impairment. impairment and behavioral indicators helps to answer the question correctly. The only
impairment in the child is: symptom that relates to hearing is turning volume up.
1.‐ Distractibility and short attention span.
2.‐ Disinterest in reading storybooks.
3.‐ Turning up the volume on the family television set.
4.‐ Temper tantrums.

783 The nurse is caring for a 1‐month‐old client who is Correct answer: 2 Development of parent–infant attachment is important in promoting developmental Critical words are “1‐month‐old client who is blind” and “teaching the parents to promote
blind, secondary to retinopathy of prematurity. The progress. Parents are encouraged to talk, sing, and interact with their baby to learn about their development.” Use knowledge of normal interventions for parents of one‐month‐olds to
nurse is teaching the parents about activities to infant’s response, and to provide appropriate stimulation at 1 month of age. The other options promote normal development. Consider activities that may be used on any infant to
promote their infant’s development. Which of the are incorrect statements to the parents. stimulate development. Then identify those that do not require vision.
following statements by the nurse is correct?
1.‐ “Infants with visual impairment respond to tactile stimuli rather than auditory stimuli.”
2.‐ “Talking, holding, and singing to your baby are appropriate activities at this age.”
3.‐ “You should expect your baby to smile in response to your voice by 4 months of age.”
4.‐ “Position the baby side‐lying in the crib at all times, and avoid loud noises, which could startle the infant.”

784 The nurse is assessing a child with conjunctivitis (pink Correct answer: 4 Purulent exudate and crusting are characteristics of conjunctivitis. Conjunctivitis associated Critical words are “child with conjunctivitis” and “most likely assess.” Note that in this
eye). Which of the following would the nurse most with foreign body can cause severe eye pain. Serous drainage and periorbital edema are not question, two choices describe discharge from the eye. That indicates that one of these
likely assess? associated with conjunctivitis. choices is probably the right answer.
1.‐ Serous drainage from the affected eye
2.‐ Severe eye pain
3.‐ Periorbital edema
4.‐ Crusting of eyelids and eyelashes

785 The nurse teaches a child with conjunctivitis Correct answer: 4 The infected area should be cleansed with a disposable tissue after a single use. Handwashing Critical words are “teaches a child with conjunctivitis measures to prevent the spread” and
measures to prevent the spread of infection. The nurse is important to prevent the spread of infection. Items that come in contact with the infected “concludes that further teaching is needed.” This means you need to identify an incorrect
concludes that further teaching is needed when the eye are considered contaminated. response from the child that would indicate he or she did not understand the teaching.
child states the following: Notice that two of the choices describe activities relating to wiping drainage from the eyes.
Because the question addresses preventing the spread of infection, disposing of the tissue
is the best choice.
1.‐ “I will wash my hands frequently.”
2.‐ “I will use a tissue to clean my eye and then throw the tissue away.”
3.‐ “I will use my own washcloth and towel, and not use my brother’s.”
4.‐ “I will carry a handkerchief with me so that I can wipe my eyes during the day.”

786 A 4‐month‐old infant has severe nasal congestion, Correct answer: 3 Saline nose drops will loosen secretions and crusting. The bulb syringe is necessary because Critical words are “infant has severe nasal congestion” and “best way to clear the nasal
nasal mucous drainage, and crusting in and around the infants cannot blow their own noses. The other options are incorrect nursing actions for this passages.” This question concerns a 4‐month‐old child. This would rule out option 4, as the
nares. What is the best way for the nurse to clear the purpose. child will not follow commands at this age. Of the 3 remaining choices, the only one that
infant’s nasal passages? removes drainage is option 3.
1.‐ Administer vasoconstrictive nose drops every 3 hours.
2.‐ Place the infant in a mist tent.
3.‐ Administer saline drops in the nose and suction with bulb syringe.
4.‐ Instruct the client to blow the nose and keep disposable tissues handy.

787 A client has just been diagnosed with closed‐angle Correct answer: 3 Narrow‐angle glaucoma develops abruptly and manifests with acute face and eye pain and is This question requires differentiation of open and closed angle glaucoma.
(narrow‐angle or acute angle‐closure) glaucoma. The a medical emergency. Halo vision, dull eye pain, and impaired night vision are symptoms
nurse assesses the client for which of the following commonly associated with open‐angle glaucoma.
most common presenting symptoms of this disorder?

1.‐ Halo vision


2.‐ Dull eye pain
3.‐ Severe eye and face pain
4.‐ Impaired night vision

788 The nurse notes a cloudy appearance to the lens of an Correct answer: 3 A cloudy‐appearing lens is symptomatic of cataract development. As the cataract matures, Recall that the opacities of cataracts decrease the red reflex.
80‐year‐old client's eye. Which of the following the red reflex is lost. A sense of a curtain falling over the visual field is associated with
additional assessment findings would help confirm the detached retina. Eye pain and double vision are not associated with cataracts.
diagnosis of cataracts?
1.‐ Sense of a curtain falling over the visual field
2.‐ Persistent, dull eye pain
3.‐ Loss of red reflex
4.‐ Double vision

789 The priority nursing measure for the client with a Correct answer: 2 The foreign body should not be removed or manipulated. It should be immobilized if possible This question requires the knowledge of first aid.
penetrating eye injury from a visible foreign body is to: and the eye covered to protect from further injury. A paper cup can be used in place of an eye
patch. Patching both eyes is an appropriate intervention to prevent ocular movement but
follows immobilization of the foreign body. Irrigation with water is an intervention for
chemical burns to the eyes. Carbonic anhydrase inhibitors are used to decrease intraocular
pressure following blunt trauma.
1.‐ Patch both eyes.
2.‐ Immobilize the foreign body and cover the eye.
3.‐ Irrigate the eye with copious amounts of water.
4.‐ Administer carbonic anhydrase inhibitors as prescribed.

790 A client is diagnosed with conductive hearing loss and Correct answer: 4 Conductive hearing loss results from changes that occur in the external or middle ear. Hearing This question requires knowledge of the differences between conductive and
asks how this occurred. The nurse should respond by aids, assistive listening devices (i.e., "pocket talkers"), and reconstructive surgeries can sensorineural hearing loss.
stating that conductive hearing loss: improve or correct hearing loss. Exposure to high levels of noise on an intermittent or constant
basis damages the hair cells of the Organ of Corti, resulting in sensorineural hearing loss.

1.‐ Has an unknown etiology.


2.‐ Occurs as a result of damage to the hair cells of the inner ear.
3.‐ Usually results from chronic exposure to loud noise.
4.‐ Occurs as a result of damage to the ear structures.

791 A client with sensorineural hearing loss should be Correct answer: 4 Weber and Rinne tests are used to differentiate conductive hearing loss from sensorineural. This question requires the knowledge of sensorioneural hearing loss.
assessed using which of the following? Tympanocentesis is the aspiration of fluid/pus from the middle ear to identify the causative
organism of acute otitis media. Transillumination of the sinuses is a diagnostic tool used to
assess for sinusitis. The diagnosis of Meniere's disease is confirmed by electronystagmography,
a series of tests to evaluate vestibular‐ocular reflexes.

1.‐ Tympanocentesis
2.‐ Transillumination of the sinuses
3.‐ Electronystagmography
4.‐ Weber and Rinne tests

792 The teaching plan for the parents of an infant with Correct answer: 2 A higher incidence of acute otitis media is noted in infants who are bottle‐fed in a horizontal Use process of elimination to determine the correct answer.
acute otitis media should include which of the position and who live in homes with smokers. The full 10‐ to 12‐day course of antibiotic
following? therapy must be administered. There is no relationship between the ingestion of fruit juices
and acute otitis media.
1.‐ Antibiotics can be discontinued when the infant is afebrile.
2.‐ When bottle‐feeding, the infant should be maintained in an upright position.
3.‐ Orange juice and other fruit juices should be eliminated from the diet.
4.‐ Cigarette smoke in the home is not a significant risk factor in acute otitis media.

793 The nurse developing a teaching plan for a client with Correct answer: 2 Atrophic macular degeneration causes loss of central vision. Magnification devices and This question requires knowledge of macular degeneration.
atrophic macular degeneration should include enhanced lighting help to promote safety. Peripheral vision remains intact. Although laser
information concerning: photocoagulation is effective for exudative macular degeneration, there is no treatment for
the atrophic form. Since macular degeneration is not an infectious process, antibiotic therapy
is not indicated.
1.‐ Surgical treatment options.
2.‐ The availability of aids to enhance vision and promote safety.
3.‐ The risks associated with the loss of peripheral vision.
4.‐ Antibiotic therapy.

794 Which of the following ophthalmic medications would Correct answer: 1 Pilocarpine (Pilocar) is a miotic and the most commonly prescribed drug for glaucoma. Recognize that three options are mydriatics; one is a miotic indicating the odd man out.
be indicated for the client with open‐angle glaucoma? Scopolamine (Hycoscine) and atropine are anticholinergics; epinephrine is an adrenergic
agonist. All three are mydriatrics. The pupil dilation caused by mydriatrics is contraindicated in
glaucoma.
1.‐ Pilocarpine (Pilocar)
2.‐ Scopolamine hydrobromide (Hyoscine)
3.‐ Atropine sulfate (Isopto Atropine)
4.‐ Epinephrine (Epitrate)

795 A 60‐year‐old male presents to the clinic complaining Correct answer: 3 These symptoms, along with dysphagia, foul‐smelling breath, and pain when drinking hot or This question requires the knowledge of the symptoms of cancer.
of hoarseness and a cough. His spouse states his voice acidic fluids, are common signs of laryngeal cancer. Chronic sinusitis can produce foul breath
has changed in the last few months. The nurse and pain or burning in the throat. GERD and CAD may produce epigastric and/or chest pain, but
interprets that the client's symptoms are consistent hoarseness and change of voice do not occur.
with which of the following disorders?

1.‐ Gastroesophageal reflux disease (GERD)


2.‐ Coronary artery disease (CAD)
3.‐ Laryngeal cancer
4.‐ Chronic sinusitis

796 The priority postoperative nursing action following Correct answer: 3 All of the nursing actions listed are appropriate for the client following nasal packing for Remember the ABCs (airway, breathing, circulation). Airway is always a priority.
nasal packing for epistaxis is: epistaxis; however, the risk of aspiration is high, and monitoring respiratory function essential.
Notice the question asks for the priority intervention.
1.‐ Providing frequent oral care.
2.‐ Ensuring adequate intake of oral fluids.
3.‐ Monitoring respiratory function and oxygen saturation.
4.‐ Administering analgesics as prescribed.
797 A client reports sudden onset of continuous eye pain Correct answer: 1 Closed or narrow‐angle glaucoma has an abrupt onset and is characterized by severe pain of This question requires knowledge that open angle glaucoma is painless and closed‐angle is
and impaired vision. Pupil dilation is noted. The nurse sudden onset. The pain usually lasts longer than 20 minutes with closed‐angle glaucoma. Eye painful.
concludes that this assessment data is consistent with pain that comes and goes quickly can be indicative of allergies. Open‐angle glaucoma occurs
which of the following disorders? gradually with no initial manifestations. Pain is not associated with cataracts or retinal
detachment.
1.‐ Closed‐angle (narrow‐angle) glaucoma
2.‐ Open‐angle glaucoma
3.‐ Cataracts
4.‐ Retinal detachment

798 A client with Meniere's disease would probably Correct answer: 2 Meniere's disease is associated with vertigo that may last for hours as well as fluctuating This question requires the knowledge of Meniere’s disease.
complain of which of the following? hearing loss, nausea, and vomiting. The disorder is unilateral, but because hearing is bilateral,
the client often does not realize the extent of the hearing loss. Option 3 is indicative of
swimmer's ear, and option 4 is indicative of acute otitis media.

1.‐ Bilateral hearing impairment


2.‐ Vertigo and nausea
3.‐ Pain when the tragus is touched
4.‐ Tenderness over the mastoid area

799 A male client has just had a cataract operation Correct answer: 4 Without a lens, the eye cannot accommodate. Since it is difficult to judge distance and climb This question requires knowledge of cataract surgery.
without a lens implant. In discharge teaching, the stairs when the eyes cannot accommodate, for safety reasons the client should have
nurse will instruct the client's wife to: assistance when climbing stairs.
1.‐ Prepare only soft foods for several days to prevent facial movement.
2.‐ Keep the eye dressing on for 10 days.
3.‐ Have her husband remain in bed for 3 days.
4.‐ Allow him to walk upstairs only with assistance.

800 An adult client presents to the Emergency Correct answer: 2 Retinal detachment is painless, but eventually floaters and visual loss will be manifested, This question requires knowledge of the early symptoms of retinal detachment.
Department after having a rock hit his eye while especially if hemorrhage has occurred. Subconjunctival hemorrhage is a manifestation of blunt
weeding along a ditch near a busy highway. The event trauma to the eye. Halo vision is characteristic of glaucoma.
occurred several hours earlier. The nurse assesses the
eye and finds redness and weeping, but the client
denies any pain. Which of the following early
symptoms would help confirm retinal detachment?

1.‐ Pain
2.‐ Floaters
3.‐ Subconjunctival hemorrhage
4.‐ Halo vision

801 A client is diagnosed with a cholesteatoma and asks Correct answer: 2 Cholesteatomas are benign, slow‐growing tumors of the middle ear that are filled with Use the process of elimination to select the correct answer.
the nurse to explain what this is. The best explanation epithelial cell debris. Untreated cholesteatomas may enlarge to fill the middle ear, destroy the
by the nurse would be, “Cholesteatoma is an:” ossicles, and cause profound hearing loss. Option 1 (endolymph) is indicative of Meniere's
disease. Options 3 and 4 are distracters.
1.‐ “Overaccummulation of fluid in the membranous labyrinth of the inner ear.”
2.‐ “Inflamed cyst or mass filled with epithelial cell debris resulting from chronic middle ear infection.”
3.‐ “Irritation to part of the uvea; the middle coat of the eyeball.”
4.‐ “Inflammation of the center of the retina.”
802 Which of the following clients is at highest risk for Correct answer: 2 Age‐related macular degeneration is the leading cause of loss of vision in clients over 50 years This question requires knowledge of the characteristics of aging.
macular degeneration? of age. Blunt trauma, exposure to toxins, and allergies are not known causes of macular
degeneration.
1.‐ A biochemist exposed to various toxins
2.‐ An elderly client
3.‐ A youth hit in the eye with a baseball
4.‐ A young adult with multiple allergies

803 A client presents to the clinic with complaints of Correct answer: 3 Transillumination of the sinuses in a non‐invasive technique used to detect fluid in the This question requires knowledge of the diagnosis of sinus disease.
sinusitis. The nurse anticipates that the most useful maxillary and frontal sinuses. The Weber test differentiates conductive hearing loss from
techniques for diagnosis would be: sensorineural hearing loss. Fluorescein staining helps identify corneal abrasions and caloric
testing is used to evaluate nystagmus in Meniere's disease.
1.‐ The Weber test.
2.‐ Fluorescein staining.
3.‐ Transillumination.
4.‐ Caloric ice water testing.

804 Which of the following is a priority nursing diagnosis Correct answer: 1 Since the amount of blood lost in a nosebleed can be frightening to clients, anxiety is a Recall priority establishment based on Maslow’s Hierarchy of needs.
for a client with epistaxis? priority nursing diagnosis. Blood draining into the nasopharynx poses a risk of aspiration. Risk
for infection and pain are appropriate nursing diagnoses related to nasal packing but are not
the priorities. Impaired verbal communication is unlikely.
1.‐ Anxiety with risk for aspiration
2.‐ Risk for infection
3.‐ Pain
4.‐ Impaired verbal communication

805 Which of the following clients is at highest risk for Correct answer: 3 The two major risk factors for laryngeal cancer are prolonged smoking along with This question requires knowledge of those factors that place the client at increased risk.
laryngeal cancer? A client who: concomitant use of alcohol. Although the majority of cases occur in men ages 50 to 75,
advancing age does not significantly increase risk. Injury to the larynx and chronic sinusitis are
not risk factors.
1.‐ Has an injury to the larynx.
2.‐ Is 85‐years‐old.
3.‐ Has chewed tobacco for 20 years.
4.‐ Suffers from chronic sinusitis.

806 Nursing interventions for a client with sensorineural Correct answer: 2 The client with sensorineural hearing loss experiences social isolation and depression and This question requires knowledge of differentiating types of hearing loss.
hearing loss include: may appear withdrawn. Amplification devices such as hearing aids are helpful for clients with
conductive hearing loss but only amplify noxious sounds for the client with sensorineural
hearing loss. Antibiotics are not helpful for sensorineural hearing loss, and tympanoplasty is
used to correct damage to structures in the middle ear.

1.‐ Providing information about the availability of hearing aids to amplify sound.
2.‐ Educating family members about social isolation and depression.
3.‐ Administering antibiotics as prescribed.
4.‐ Preparing the client for tympanoplasty.

807 In order to improve communication with the client Correct answer: 4 The best method to improve communication with the client is to face him directly when This question requires selecting the answer which shows the most respect for the client.
having a diagnosis of sensorineural hearing loss, the speaking. Hearing‐impaired clients often consciously or unconsciously lip‐read to enhance
nurse should instruct the client's family to: perception. The other options are ineffective and may frustrate or demean the client.

1.‐ Use one‐word answers.


2.‐ Exaggerate pronunciation of words.
3.‐ Speak loudly to get the client's attention.
4.‐ Face the client directly when speaking.

808 The nurse would reinforce explanations about which Correct answer: 2 Leukoplakia are white, patchy, precancerous lesions; erythroplakia are red, velvety, Recognize that biopsies are the most conclusive diagnostic test listed.
of the following diagnostic test is recommended for precancerous patches. Both can be found on the laryngeal mucosa. Biopsies of both types of
laryngeal cancer? lesions aid in diagnosing and staging laryngeal cancer. Gonioscopy and tonometry are tests
used for glaucoma. Caloric testing is associated with Meniere's disease and central nervous
system (CNS) disorders.
1.‐ Gonioscopy
2.‐ Biopsies of leukoplakia and erythroplakia
3.‐ Tonometry
4.‐ Caloric testing

809 A child is noted to have a very short stature, non‐ Correct answer: 1 These are the key assessment features in Turner's syndrome. If the child is diagnosed early in The question describes one of these genetic disorders. If guessing, eliminate the ones you
pitting lymphedema of the hands and feet, webbed age, proper treatment with growth hormone can be offered to the family. know are incorrect.
neck, and low posterior hairline. The nurse will expect
which of the following diagnoses?

1.‐ Turner's syndrome


2.‐ Down syndrome
3.‐ Marfan's syndrome
4.‐ Klinefelter's syndrome

810 The nurse working in an ambulatory surgery center Correct answer: 2 Tympanostomy (ventilation) tubes allow air into the middle ear. While in place, it is important This question uses the “odd man out” for the activities.
would instruct the parents of a pediatric client to to avoid getting water into the ear canal, which could potentially enter the middle ear. The
prohibit their child from participating in which of the other activities are not risks to the pediatric client with tympanostomy tubes.
following activities following insertion of
tympanostomy tubes?
1.‐ Playing outdoors
2.‐ Swim lessons
3.‐ Gymnastics
4.‐ T‐ball practice

811 A 75‐year‐old client reports to the nurse during the Correct answer: 3 Presbycusis is the most common form of sensorineural hearing loss in older adults. Meniere's This question requires knowledge of the normal changes of aging.
admission process that she was recently diagnosed disease is an inner ear disorder that affects primarily middle‐aged adults. Otalgia is an earache;
with age‐related hearing loss. The client is most likely otitis externa is infection in the external auditory canal and can occur in clients of any age.
describing:
1.‐ Otalgia.
2.‐ Meniere's disease.
3.‐ Presbycusis.
4.‐ Otitis externa.

812 After surgery for a cataract removal, the nurse Correct answer: 3 Bending over to pick up objects from the floor is contraindicated because it increases This question requires knowledge of the components that raise ocular pressure.
teaches the client about home care. Which of the intraocular pressure. Activities such as walking on level surfaces, lying on the nonoperative
following activities would be contraindicated? side, and performing simple isometric exercises are not harmful.
1.‐ Walking down the hall unassisted
2.‐ Lying in bed on the nonoperative side
3.‐ Bending over to pick up newspapers from the floor
4.‐ Performing simple isometric exercises
813 A client with sensorineural hearing loss asks about Correct answer: 3 Amplification is of no help with sensorineural hearing loss and serves only to increase the This question requires knowledge of conductive versus sensorineural hearing loss.
whether a hearing aid would be beneficial. The nurse intensity of distorted sounds. The other options are incorrect.
makes which of the following replies to this client?

1.‐ "It will make sounds louder and clearer."


2.‐ "It will have no effect on hearing."
3.‐ "It will intensify the already distorted sounds."
4.‐ "It will improve the client's ability to distinguish words from background noises."

814 A client diagnosed with Meniere's disease would Correct answer: 2 These are the classic symptoms of Meniere's disease. Nystagmus occurs with acute attacks. This question requires knowledge of Meniere’s disease.
complain of vertigo and which of the following? Headache, double vision, pain, and purulent drainage are not reported.

1.‐ Nausea, vomiting, and headache


2.‐ Tinnitus, hearing loss, and a sense of fullness in the ear
3.‐ Purulent discharge from the ear and pain
4.‐ Headache and double vision

815 The nurse anticipates that which of the following Correct answer: 1 Anticholinergics and antiemetics are used to control symptoms associated with Meniere's This question requires knowledge of Meniere’s disease.
treatment options would be included in the disease. Diuretics are used between acute attacks to reduce the volume of endolymph and
management of a client diagnosed with Meniere's prevent attacks. Glucocorticoids, beta blockers, and analgesics are not commonly used.
disease?
1.‐ Anticholinergics
2.‐ Glucocorticoids
3.‐ Analgesics
4.‐ Beta Blockers

816 The nurse explaining the use of medication therapy Correct answer: 3 Constricting the pupil stimulates the ciliary muscles to pull on the trabecular meshwork This question requires knowledge of the medications needed to treat glaucoma.
for a client diagnosed with glaucoma would state that surrounding the Canal of Schlemm. This increases the flow of aqueous humor and decreases
miotics are useful because they work by: intraocular pressure.

1.‐ Dilating the pupil.


2.‐ Constricting intraocular vessels.
3.‐ Constricting the pupil.
4.‐ Relaxing the ciliary muscles.

817 The initial nursing intervention for a client in the Correct answer: 4 The immediate priority for clients with chemical burns is flushing the affected eye with This question requires knowledge of first aid.
emergency department who suffered a chemical burn copious amounts of normal saline or water. Evaluation of visual acuity is an appropriate
to the eyes is to: intervention after flushing. Analgesics, with the exception of topical anesthesia, are not
indicated. Antibiotics may be administered after the initial actions have been taken.

1.‐ Administer analgesics as prescribed.


2.‐ Evaluate vision with and without prescription eyeglasses.
3.‐ Administer antibiotics as prescribed.
4.‐ Irrigate the eyes with normal saline or water.

818 The nurse would question the client about which of Correct answer: 4 Clients with retinal detachment frequently report flashing lights and loss of vision commonly This question requires knowledge of retinal detachment.
the following clinical manifestations that commonly described as a curtain being drawn across the eye. Retinal detachment is painless, does not
occurs in retinal detachment resulting from cataract cause increased lacrimation (unless associated with trauma), and does not affect ocular
extraction? movement.
1.‐ Sudden, severe eye pain
2.‐ Inability to move the eye and loss of accommodation
3.‐ Increased lacrimation
4.‐ Flashing lights and loss of part of the visual field

819 An elderly client is diagnosed with chronic open‐angle Correct answer: 1 Remember drugs that end in ‐olol are beta blockers. Beta blockers, when administered as Note that a frequently asked topic on NCLEX&lt;sup&gt;®&lt;/sup&gt; is that a heart rate
glaucoma and has been prescribed timolol maleate ophthalmic preparations, can produce systemic effects such as bradycardia, hypotension, and (HR) is always assessed before giving a beta blocker and that the drug should be held if HR is
(Timoptic). The nurse assesses for which of the bronchospasm. Beta blockers act as central nervous system (CNS) depressants and are used to less than 60/min.
following possible adverse systemic responses to the treat anxiety.
drug?
1.‐ Bradycardia
2.‐ Hypertension
3.‐ Tachycardia
4.‐ Anxiety

820 A client presents to the walk‐in clinic with fever, Correct answer: 2 Symptoms of acute sinusitis include facial pain, purulent nasal discharge, fever, and This question requires knowledge of the diagnosis of sinus illness.
complaints of right facial pain, and copious yellow‐ headache. Transillumination is a diagnostic tool used to detect fluid in the maxillary or frontal
green nasal discharge. There is no transillumination of sinuses. Fluid in the sinuses indicates infection. Transillumination is not used to diagnose the
the right maxillary sinus. The nurse anticipates that the other conditions.
most likely diagnosis is:

1.‐ Dental abscess.


2.‐ Acute sinusitis.
3.‐ Meningitis.
4.‐ Tonsillitis.

821 Prior to a total laryngectomy for laryngeal cancer, the Correct answer: 2 Although all of the options are appropriate, providing the client with an opportunity to This question requires knowledge of the value of ventilating concerns.
client appears withdrawn and depressed. He keeps the express feelings of anger or fear is the priority. Clients with laryngeal cancer requiring total
curtain drawn, refuses visitors, and states he wants to laryngectomy need permission to grieve for their anticipated losses before mobilizing coping
be left alone. Which of the following is the priority strategies to deal with the surgery and follow‐up care.
nursing intervention?

1.‐ Discussing options for postoperative speech rehabilitation.


2.‐ Providing an opportunity for the client to express feelings of anger or fear.
3.‐ Referring the client to counseling services.
4.‐ Identifying cancer support groups in the community.

822 A priority nursing intervention for the infant with cleft Correct answer: 1 The infant with cleft lip is unable to create an adequate seal for sucking. The child is at risk for Consider Maslow’s hierarchy when prioritizing.
lip is which of the following? inadequate nutritional intake as well as aspiration.
1.‐ Monitoring for adequate nutritional intake
2.‐ Teaching high‐risk newborn care
3.‐ Assessing for respiratory distress
4.‐ Preventing injury

823 The clinic nurse who is treating a client with anterior Correct answer: 3 The trauma resulting from nose picking is a common cause of anterior epistaxis because of This question requires knowledge of first aid.
epistaxis needs to assess for which of the following as damage to Kiesselbach's area, a highly vascular area in the anterior septum. Blood dyscrasias,
an etiology for the episode? hypertension, and diabetes are causes of posterior epistaxis.

1.‐ Blood dyscrasias


2.‐ Hypertension
3.‐ Nose picking
4.‐ Diabetes
824 The nurse is providing preoperative teaching for a Correct answer: 3 Options 1 and 2 are incorrect. Option 4 is the definition of pneumatic retinopexy. Scleral This question requires knowledge of the treatment for detached retina.
client who has a detached retina. The nurse reinforces buckling as described in option 3 is used in conjunction with laser photocoagulation or
that scleral buckling involves: cryothermy.
1.‐ Removing the torn segment of retina.
2.‐ Replacing the torn segment of the retina with retinal tissue from a donor.
3.‐ Utilizing a piece of silicone to indent the sclera and enhance contact between the retinal layers.
4.‐ Injecting air or gas into the vitreous humor to force the detached retina back against the choroid.

825 The priority intervention for the client with blunt Correct answer: 1 Blunt trauma does not cause loss of intraocular contents, and no foreign body is present. This question requires knowledge of measures to decrease edema.
trauma to the eye is to: Constriction of the pupil with miotics is not indicated. Intraocular pressure may increase as a
result of the trauma and is prevented by the use of carbonic anhydrase inhibitors.

1.‐ Decrease intraocular pressure.


2.‐ Prevent loss of intraocular contents.
3.‐ Irrigate the eye to remove foreign substances.
4.‐ Administer miotics.

826 Which of the following statements indicates that the Correct answer: 3 Proper positioning is important after eye surgery to avoid complications. The client should Use the process of elimination to find the correct answer.
client has understood the instructions to follow at avoid bending, straining, and strenuous activity in order to reduce intraocular pressure in the
home after cataract surgery? affected eye.
1.‐ "I may not watch television for 3 weeks."
2.‐ "I should keep the protective eye shield in place at all times."
3.‐ "I should not bend over to pick up objects from the floor."
4.‐ "I can sleep on whichever side I want."

827 The client is diagnosed as having presbycusis. The Correct answer: 2 Presbycusis is a term to describe degenerative changes from aging and is associated with loss This question requires knowledge of hearing disorders.
nurse explains to the client that this disorder is which of the hair cells in the Organ of Corti leading to sensorineural hearing loss. Air and bone
of the following type of hearing disorder? conduction are terms applying to conductive hearing loss. The term "central" is not commonly
used when referring to hearing loss.
1.‐ Bone conduction
2.‐ Sensorineural
3.‐ Conductive
4.‐ Central nervous system

828 The nurse has provided discharge teaching for the Correct answer: 2 Low‐sodium diets and diuretics help reduce the volume of endolymph and prevent This question requires knowledge of the dietary restrictions associated with Meniere’s
client with Meniere's disease. The nurse is able to recurrence. Salted cashews are the high‐sodium food in this list. disease.
evaluate the client's understanding of the teaching if
the client states that which of the following foods
should be avoided?
1.‐ Ice cream
2.‐ Salted cashews
3.‐ Baklava
4.‐ Fried okra

829 The clinic nurse should provide which of the following Correct answer: 1 Ninety percent of nosebleeds arise from the anterior portion of the nose known as This question requires knowledge of first aid.
as the initial treatment for epistaxis? Kiesselbach's area. Pinching the area for 5 to 10 minutes aids in hemostasis. Nasal packing and
cauterization are subsequent treatments if bleeding doesn't subside. The application of warm
compresses is contraindicated.
1.‐ Applying pressure on the anterior portion of the nose
2.‐ Application of warm compresses to the nose
3.‐ Insertion of nasal packing
4.‐ Cauterization of the bleeding vessel
830 A client with laryngeal cancer has just been told that Correct answer: 3 Laryngeal cancer spreads by direct invasion into surrounding tissues and by metastasis. The This question requires knowledge of a common origin of metastasis.
the cancer has metastasized. The nurse suspects that most common site of metastasis is the lungs. Metastasis to other sites such as the brain,
metastasis has occurred to which of the following breast, and uterus is rare.
most common areas for this type of cancer?

1.‐ Brain
2.‐ Breast
3.‐ Lungs
4.‐ Uterus

831 The nurse prepares to carry out which of the Correct answer: 1 Intranasal steroids are commonly used to treat sinusitis to reduce inflammation. Nasal Recall that a function of steroids is to decrease swelling.
following orders for a client with sinusitis as part of oxygen by cannula (which dries the mucous membranes), the use of diuretics, and restriction
standard medical management? of oral fluids are contraindicated.
1.‐ Intranasal steroids
2.‐ Administration of oxygen by nasal cannula
3.‐ Use of diuretics
4.‐ Restriction of oral fluids

832 Which of the following assessment findings would be Correct answer: 1 Open‐angle glaucoma is characterized by halo and blurred vision. The abrupt onset of severe Recall that halo vision is a classic sign of glaucoma.
noted in the client with open‐angle glaucoma? eye pain, a fixed, partially dilated pupil, and increases in intraocular pressure are indicative of
closed‐angle glaucoma. The normal intraocular pressure ranges from 12 to 20 mmHg. Floaters
in the visual field are found with detached retina.

1.‐ Halo vision


2.‐ Dilated, nonreactive pupil
3.‐ Floaters in the visual field
4.‐ Intraocular pressure of 15 mmHg

833 The pediatric health nurse explains to parents that Correct answer: 3 A child's Eustachian tubes are shorter and straighter than those of an adult. The other This question requires knowledge of ear physiology.
acute otitis media is more common in infants and responses are incorrect.
children than adults because a child's Eustachian
tubes:
1.‐ Are longer and curved.
2.‐ Are undeveloped.
3.‐ Are shorter and straighter.
4.‐ Have less cilia.

834 The nurse would carry out which of the following Correct answer: 2 The development of cataracts causes scattering of the light entering the eye, resulting in Consider the least invasive measure available.
interventions for a client newly diagnosed with glare. Sunglasses will reduce glare. Night vision declines in clients with cataracts and would be
cataracts? discouraged. Pilocarpine eye drops and carbonic anhydrase inhibitors are used to treat
glaucoma.
1.‐ Administer pilocarpine eye drops.
2.‐ Teach the client to wear sunglasses during the daytime.
3.‐ Encourage night driving.
4.‐ Administer carbonic anhydrase inhibitors such as acetazolamide (Diamox).

835 A client is admitted with a new diagnosis of retinal Correct answer: 4 The priority nursing intervention is directed toward maintaining contact of the retina with the Consider positioning a basic strategy.
detachment. The nurse should place the highest choroid and positioning the client so the area of detachment falls against the choroid. Both
priority on which of the following interventions? eyes are patched to limit ocular movement. Retinal detachment is generally painless.
Darkening the room is not necessary.
1.‐ Administering analgesics as prescribed
2.‐ Covering the affected eye
3.‐ Darkening the room
4.‐ Positioning the client properly

836 A 70‐year‐old client comes to the outpatient clinic Correct answer: 2 Increasing difficulty with central vision and distortion of vision in one eye are common This question requires knowledge of eye disorders.
complaining of increasing difficulty with "close work" manifestations of macular degeneration. Peripheral vision is usually not impaired. The
such as knitting. She indicates she does not have symptoms are not characteristic of glaucoma or cataracts. Subconjunctival hemorrhage occurs
difficulty seeing objects on either side but does state with ocular trauma.
that straight lines appear distorted or wavy. The nurse
suspects which of the following disorders that is
consistent with the client's reported symptoms?

1.‐ Cataracts
2.‐ Macular degeneration
3.‐ Glaucoma
4.‐ Subconjunctival hemorrhage

837 A client who has undergone a visual acuity test has Correct answer: 1 A result of 20/120 means that this client can read at a distance of 20 feet what another This question requires knowledge of visual screening.
results of 20/120 in the right eye. When explaining this individual with normal vision can read at a distance of 120 feet. This means that the client is
finding to the client, which of the following statements nearsighted in that eye. The other responses are incorrect.
would be most appropriate?

1.‐ The client is nearsighted.


2.‐ The client is farsighted.
3.‐ The client's vision is normal.
4.‐ The client is legally blind in that eye.

838 A client being prepared for an ocular examination has Correct answer: 4 Cyclopentolate is a mydriatic and a cycloplegic medication that is used to dilate the pupil and This question requires knowledge of the functions of myotics, mydriatics, and cycloplegics.
an order for a topical eye medication. The nurse paralyze the ciliary muscles before an eye exam. Carbachol and latanoprost are miotics that
prepares to administer which of the following constrict the pupil and are used to treat glaucoma. Glycerin is an osmotic diuretic used to treat
medications? acute angle‐closure glaucoma.
1.‐ Carbachol (Carboptic)
2.‐ Latanoprost (Xalatan)
3.‐ Glycerin (Ophthalgan)
4.‐ Cyclopentolate (Cyclogyl)

839 A client has just been diagnosed with glaucoma. The Correct answer: 3 It is important to share with the client that lifelong medication therapy is needed to preserve Use the process of elimination to choose the highest priority.
nurse should place highest priority on teaching the vision. The statement in option 2 is correct also but is not as critical as option 3, since the
client which of the following pieces of information? client has just been diagnosed. Options 1 and 4 are false statements.

1.‐ Fluid restriction is needed to reduce intraocular pressure.


2.‐ The disorder often has no symptoms.
3.‐ Adherence to medication therapy is essential to reduce risk of vision loss.
4.‐ The disorder is typically diagnosed after an episode of eye infection.

840 The nurse notes a slight cloudy appearance to the Correct answer: 2 A cloudy‐appearing lens is characteristic of cataract development. Early symptoms of cataract This question requires knowledge of cataract development.
lens of a 64‐year‐old client's eye. Which of the formation include blurred vision and a loss of ability to see colors. A sense of a curtain falling
following symptoms should the nurse question the across the field of vision characterizes detached retina. Eye pain and double vision are not
client about? symptoms associated with cataracts.
1.‐ Sense of a curtain falling over vision
2.‐ Blurring of vision
3.‐ Slight but constant eye pain
4.‐ Double vision

841 A client admitted to the hospital has a notation on Correct answer: 1 A client who is legally blind has either visual acuity no better than 20/200 in the better eye This question requires knowledge of the definition of legally blind.
the medical record that reads "legally blind." The nurse with optimal correction, or has a visual field of 20 degrees rather than 180 degrees.
interprets that the client's best corrected vision in the
better eye must be no better than which of the
following?
1.‐ 20/200
2.‐ 20/120
3.‐ 20/360
4.‐ 20/100

842 A client with a suspected impaction of cerumen has Correct answer: 4 It is essential to determine that the tympanic membrane is intact before completing an otic Use the highest priority to prevent further injury.
an order for an otic irrigation. The nurse should take irrigation. No more than 50 to 70 mL of solution should be drawn up at one time, and the fluid
which of the following essential actions before should be at body temperature. The client may be positioned wherever it is comfortable, and
beginning the irrigation? needs only a receptacle to hold the drainage, and a waterproof pad to protect clothing.

1.‐ Draw up no more than 120 mL of solution for use at one time
2.‐ Make sure the client is seated comfortably in the bathroom
3.‐ Make sure the irrigant is at room temperature
4.‐ Examine the tympanic membrane to be sure it is intact

843 A client has been treated for acute otitis media. The Correct answer: 3 Ear pain is a primary or classic symptom associated with otitis media. Secondary Recall pain as a classic symptom of infection.
nurse would evaluate whether the client obtained manifestations could include dizziness, vertigo, and diminished hearing in the affected ear.
relief from which of the following primary symptoms
associated with this disorder?
1.‐ Dizziness
2.‐ Headaches
3.‐ Ear pain
4.‐ Nausea

844 The nurse working in an ambulatory surgery center Correct answer: 2 Following ear surgery, clients should avoid activities that could result in increased pressure in Use the process of elimination to find the correct answer.
would plan to instruct the client that which of the the middle ear. These include blowing the nose, sneezing, coughing, or doing any activities that
following activities is acceptable following ear surgery? involve holding the breath or bearing down. Talking is an acceptable activity.

1.‐ Blowing the nose

2.‐ Talking
3.‐ Sneezing
4.‐ Doing push‐ups

845 The nurse is performing an otic examination on a Correct answer: 1 Otosclerosis is characterized by Schwartz's sign, a tympanic membrane that is reddish or This question requires knowledge of otosclerosis.
client with otosclerosis. Then nurse documents that pinkish‐orange because of increased vascularity. It would not be pearly white or pale (options
the tympanic membrane is: 2 and 4), nor would it have a bruised appearance (option 3).
1.‐ Reddish or pinkish‐orange.
2.‐ Light, pale pink.
3.‐ Ecchymotic.
4.‐ Pearly white.
846 The nurse has conducted discharge teaching for a Correct answer: 4 The client with Meniere's disease should limit intake of salty foods that could cause an This question requires knowledge of the diet related to Meniere’s disease.
client diagnosed with Meniere's disease. The nurse increase in endolymphatic fluid in the inner ear. The other foods listed pose no problem.
evaluates that the client understood the instructions
given if the client states to refrain from eating which of
the following favorite foods?

1.‐ Granola bars


2.‐ Baked eggplant
3.‐ Sherbet
4.‐ Salted pretzels

847 A client comes to the ambulatory clinic seeking care Correct answer: 3 The nurse should evaluate the client's vision first to provide a baseline, and then treat the Always remember the need to assess first.
with a complaint of "getting something in my eye." injury. Irrigation is often used to remove foreign bodies from the eye, followed by application
Which of the following actions should the nurse take of an eye patch.
first?
1.‐ Provide copious irrigation with normal saline
2.‐ Get a detailed health history
3.‐ Evaluate the client's vision
4.‐ Swab the cornea several times using a cotton‐tipped applicator

848 A client is being admitted to the post‐anesthesia Correct answer: 2 Following eye surgery, the head of bed should be elevated 30 to 45 degrees and the client When looking at positioning questions, it is important to consider positioning on the
recovery area following lens removal and replacement should lie on back or unaffected side to reduce intraocular pressure. Small pillows may be used unaffected side.
in the left eye for a cataract. The nurse places the at the sides of the head to immobilize the head when lying on the back.
client into which of the following most appropriate
positions?
1.‐ On the left side with the head of bed elevated 30 degrees
2.‐ On the right side with the head of bed elevated 30 degrees
3.‐ Supine with the head of bed flat
4.‐ Upright with the head and neck turned to the right

849 A client who has developed impaired vision because Correct answer: 1 Glaucoma is characterized by a gradual loss of vision that is irreversible, because of the Use the process of elimination to select the correct answer.
of previously undiagnosed glaucoma asks the nurse if effects of increased intraocular pressure on the optic neurons. Compliance with medication
the lost vision will return. Which of the following therapy is important to preserve the current level of vision, although vision that is lost cannot
replies by the nurse is most accurate? be regained.

1.‐ "Vision that is lost will not return, but compliance with therapy can help to maintain current vision."
2.‐ "It will take at least 2 to 3 months for vision to return to baseline."
3.‐ "It is difficult to answer that question with accuracy. Some clients experience return of vision, while others do not."
4.‐ "Vision will return to normal within a week or so if intraocular pressure is reduced quickly."

850 A client is admitted with a newly diagnosed detached Correct answer: 4 The client with a detached retina should have activity restricted with eyes patched to reduce Use the basic principle of decreasing stimuli as an intervention.
retina. The nurse should place highest priority on doing eye movement and prevent worsening of the detachment. The client may be prepared for
which of the following? surgery quickly, and thus may be placed on NPO status rather than clear liquids.

1.‐ Limiting visitors and providing clear liquids


2.‐ Allowing the client to get out of bed but keeping the room darkened
3.‐ Giving eye drops every hour and allowing bathroom privileges only
4.‐ Placing the client on bedrest and patching the eyes
851 The nurse who is planning care for a client who is Correct answer: 2 The nurse should orient the client to the room for safety, using both words and a physical Recall that safety is always a priority.
legally blind should do which of the following as most walking tour for best effect. Options 2 and 4 are helpful, but do not ensure client safety.
important to ensure the client's safety? Leaving doors partially closed (option 1) is hazardous because the client could inadvertently
walk into the door during ambulation. Pathways should be free of obstacles.

1.‐ Leave doors partially closed


2.‐ Orient the client verbally and physically to the room
3.‐ Provide radio and television for stimulation
4.‐ Describe the weather and community events for the client

852 The nurse is performing the Rinne test on a client Correct answer: 4 The nurse places the base of the tuning fork on the client's mastoid bone to perform the This question requires knowledge of screening assessment.
with a suspected hearing impairment. The nurse places Rinne test. When the sound is no longer heard, it is quickly repositioned in front of the client's
the base of the tuning fork in which of the following ear, and hearing is again assessed. The tuning fork may be placed at the top of the forehead or
locations? the vertex of the skull in the midline to perform the Weber test. The bridge of the nose is not
used as a reference point for assessing hearing.

1.‐ On the forehead in the midline


2.‐ On the tip of the nose
3.‐ On the vertex of the skull
4.‐ On the mastoid bone

853 The nurse is providing instructions to a client who has Correct answer: 3 The client should avoid the use of aerosol sprays, cosmetics, or other hair or facial products Use process of elimination to discern the correct answer.
been diagnosed with hearing impairment and has just near the hearing aid. The aid should not get excessively wet. The hearing aid should be turned
received a hearing aid. The nurse would include which off when not in use, and should be maintained on the lowest setting that is comfortable and
of the following statements in discussion with the effective.
client?
1.‐ Immerse the hearing aid daily in warm water for 15 minutes to clean it.
2.‐ Leave the hearing aid on at all times to keep the battery charged.
3.‐ Avoid use of aerosol sprays near the aid, because particles can clog the receiver.
4.‐ Adjust the volume control to the maximum setting for efficient use.

854 A nurse is providing care to a client who just Correct answer: 2 A stapedectomy is a common surgical procedure used to treat the hearing loss that is This question requires knowledge of the care for a post‐operative client.
underwent a stapedectomy. The nurse checks the associated with otosclerosis. It is not performed for the other conditions listed.
client's medical record, noting that this client had
which of the following diagnoses prior to surgery?

1.‐ Otitis externa


2.‐ Otosclerosis
3.‐ Meniere's disease
4.‐ Foreign body obstruction of the auditory canal

855 A 72‐year‐old client reports to the nurse during a Correct answer: 3 Presbycusis is an age‐related decline in hearing. Otosclerosis is a familial disorder This question requires knowledge of the definitions for the various disorders listed.
health history that the physician previously diagnosed characterized by hearing loss. Meniere's disease is a disorder of the inner ear that results in
an age‐related loss of hearing. The nurse documents in vertigo. Otalgia is an earache.
the medical record that the client reports which of the
following disorders?
1.‐ Otosclerosis
2.‐ Meniere's disease
3.‐ Presbycusis
4.‐ Otalgia
856 The nurse would prioritize that which of the following Correct answer: 1 Meniere's disease is characterized by bouts of vertigo, which place the client at risk for falls Use safety principles to establish priorities.
nursing diagnoses has highest priority for a client and injury. The client may have manifestations of the other nursing diagnoses as well, but the
experiencing an attack of Meniere's disease? highest priority is on preventing injury.

1.‐ Risk for injury


2.‐ Risk for disturbed sleep pattern
3.‐ Sensory perception: auditory
4.‐ Risk for ineffective individual coping

857 A client has a history of angle‐closure glaucoma. The Correct answer: 1 Angle‐closure glaucoma can manifest abruptly as acute onset of eye and facial pain. It is This question requires knowledge of closed‐ and open‐angle glaucoma.
nurse would assess the client for which of the considered a medical emergency because it signals a rapid rise in intraocular pressure. Halo
following symptoms to determine a recurrent episode vision and difficulty seeing at night are symptoms commonly associated with open‐angle
of this health problem? glaucoma. Itching of the eyes is an unrelated item.
1.‐ Severe eye pain
2.‐ Halo vision
3.‐ Difficulty seeing at night
4.‐ Itching of the eyes

858 The nurse is performing a routine eye assessment on Correct answer: 2 A Snellen eye chart, as shown in the photograph, is used to test distance vision. The client This question requires knowledge of screening procedures.
a client as shown in the accompanying photograph. reads aloud the smallest line of print that can be seen. Each line is coded to interpret the visual
The nurse documents results of this test as an acuity for that line. Normal vision is 20/20. A Rosenbaum chart is used to test near vision; an
indication of the client's: Ishihara chart is used to test color vision. The nurse performs peripheral vision testing by
facing the client and bringing an object into both of the client's visual fields from the side.

1.‐ Color vision.

2.‐ Distance vision.

3.‐ Near vision.


4.‐ Peripheral vision.

859 A 73‐year‐old client has a clouding of the lens of the Correct answer: 1 A clouding of the lens occurs with cataract development. The red reflex becomes absent or is This question requires knowledge of screening techniques.
right eye. The clinic nurse would assess for which of lost as the cataract gets worse or "matures." Double‐vision and intermittent aching eye pain
the following that supports a suspected finding of a do not occur. Option 3 describes findings with macular degeneration.
cataract?
1.‐ Absence of red reflex
2.‐ Double‐vision
3.‐ Blurring only in the center of the visual field
4.‐ Intermittent aching eye pain

860 A client is brought to the emergency department Correct answer: 4 The foreign body should not be removed or manipulated. It should be kept immobilized if Choose the intervention which requires the least invasive measures.
after a shard of glass penetrated the left eye in an possible with an item such as a plastic cup, which will not put pressure on the eye. Eye
industrial accident. The glass is presently visible, patching is desirable (option 3), but a flat patch could cause further eye injury. Eye irrigation is
immobile, and protruding from the eyeball. Which of done when the client suffers chemical burns to the eye (option 1). Assessment of vision (option
the following is the priority nursing action on 2) while the glass is still imbedded could cause further harm by encouraging eye movement.
admission?
1.‐ Irrigate the eye with copious amounts of sterile normal saline.
2.‐ Rapidly assess distance and near vision, and check the cardinal fields of gaze.
3.‐ Patch a flat sterile eye patch on both eyes.
4.‐ Use an item such as a plastic cup to cover and protect the eye until the physician examines the client.
861 A client is suspected of having presbycusis. Which of Correct answer: 2 Presbycusis is characterized by sensorineural hearing loss. Weber and Rinne tests are used to This question requires knowledge of hearing loss.
the following tests would provide additional data to differentiate conductive from sensorineural hearing loss. Caloric testing evaluates ocular‐
confirm this diagnosis? vestibular reflexes. Tympanocentesis is the aspiration of fluid/pus from the middle ear to
identify the causative organism of acute otitis media. CT scanning is not used.

1.‐ Caloric testing


2.‐ Weber and Rinne tests
3.‐ Tympanocentesis
4.‐ CT scan

862 The nurse should place highest priority on teaching a Correct answer: 4 Atrophic macular degeneration results in loss of central vision. Thus, the priority is to obtain Recall that safety is always a priority.
client newly diagnosed with atrophic macular magnification devices, other aids, and provide enhanced lighting to promote safety. Antiviral
degeneration about which of the following? medication is not a form of therapy. Surgical therapy is not an option for the atrophic (dry)
form of macular degeneration. Peripheral vision loss rarely occurs with macular degeneration.

1.‐ How to compensate for loss of peripheral vision


2.‐ Antiviral medication therapy
3.‐ Currently known surgical therapies
4.‐ Aids to enhance vision and promote safety

863 The nurse questions a client with open‐angle Correct answer: 3 Open‐angle glaucoma is characterized by halo and blurred vision. The presence of floaters or Recall that halos are indicative of glaucoma.
glaucoma about the presence of which of the following the sensations of a curtain or veil over the visual field are found with detached retina. A
symptoms? burning sensation in the eyes is not part of the clinical picture.
1.‐ Floaters in the visual field
2.‐ A veil or curtain over the visual field
3.‐ Halo vision
4.‐ Burning sensation in the eyes

864 A client is admitted to the pre‐surgical area before Correct answer: 1 The priority nursing intervention is one that maintains contact of the retina with the choroid Use Maslow’s hierarchy to establish priorities.
undergoing surgery to repair detached retina. The by positioning the client so the detached area falls against the choroid. It is unnecessary to
admitting nurse would take which of the following darken the client's immediate environment. A preoperative medication may be ordered, but
actions first? has lesser priority than maintaining proper position of the head to protect the eye. Both eyes,
not just the affected eye, are patched to minimize eye movement.

1.‐ Position the client properly


2.‐ Darken the bedside area of the client
3.‐ Administer the prescribed preoperative analgesic
4.‐ Cover the affected eye

865 A 76‐year‐old woman visits the ambulatory clinic with Correct answer: 4 Visual difficulty caused by distortions and impairment of central vision is common with This question requires knowledge of macular degeneration.
reports of having difficulty reading and doing macular degeneration. Peripheral vision in most cases is normal. The symptoms are not
needlework because of visual distortions. The characteristic of glaucoma, cataracts, or detached retina.
peripheral vision assessment by the nurse yields
normal findings. The nurse suspects that this client is
experiencing which of the following visual problems?

1.‐ Glaucoma
2.‐ Detached retina
3.‐ Cataracts
4.‐ Macular degeneration
866 Which of the following strategies by the nurse would Correct answer: 2 Communication with a hearing impaired client is enhanced by facing the client while speaking, Use the process of elimination to determine the correct answer.
be most useful in promoting communication with a because they often consciously or unconsciously lip‐read to better interpret the spoken word.
client with presbycusis? The other options are inappropriate and ineffective.
1.‐ Speak loudly to the client.
2.‐ Face the client when speaking.
3.‐ Use single‐syllable words whenever possible.
4.‐ Over‐pronounce words to augment hearing.

867 A nurse is teaching a group of young adults about skin Correct answer: 1 Tanning and sun exposure can increase susceptibility to skin cancers. This is a potentially The wording of the question tells you that the correct response is a false statement of fact.
lesions. Which of the following would be inappropriate harmful activity and should not be included in client teaching. The other items are important Use the process of elimination and nursing knowledge to make a selection.
to include in discussions with these clients? to discuss with clients who are trying to maintain healthy skin.

1.‐ The benefits of suntanning


2.‐ Importance of monthly skin self‐inspections
3.‐ Evaluation of skin lesions using the ABCD method
4.‐ Need for seeking professional advice regarding lesions

868 A client with psoriasis receives a prescription from Correct answer: 4 Methotrexate is used for severe and nonresponsive cases of psoriasis. It is not a first‐line The core issue of the question is knowledge of the sequence of treatments for psoriasis.
the primary physician. The nurse concludes that the form of therapy. Options 1, 2, and 3 are first‐line treatments for psoriasis. The wording of the question tells you that the correct response is also a true statement of
client is not receiving first‐line therapy after noting fact. Use the process of elimination and nursing knowledge to make a selection.
that the prescription is written for which of the
following?
1.‐ An emollient
2.‐ Coal tar application
3.‐ Topical corticosteroid
4.‐ Methotrexate

869 The nurse practitioner documents in a client record Correct answer: 1 Whiteheads are classified as closed comedones. Blackheads are open comedones. Options 3 The core issue of the question is knowledge of various skin eruptions. The wording of the
that a client has closed comedones. The nurse explains and 4 are irrelevant. question tells you that the correct response is also a true statement of fact. Use the process
to the client that this means the lesions are what type of elimination and nursing knowledge to make a selection.
of skin eruption?
1.‐ Whiteheads
2.‐ Blackheads
3.‐ Pustules
4.‐ Cysts

870 The nurse evaluates that a client understands the Correct answer: 3 No medication is indicated for seborrheic keratosis (options 1, 2, and 3). These lesions may be The core issue of the question is knowledge of treatments for seborrheic keratosis. The
therapeutic management of seborrheic keratosis if he treated with electrocautery or liquid nitrogen for removal. wording of the question tells you that the correct response is also a true statement of fact.
mentions which of the following? Use the process of elimination and nursing knowledge to make a selection.

1.‐ Antibiotics
2.‐ Antifungal creams
3.‐ Liquid nitrogen
4.‐ Corticosteroids

871 A client with contact dermatitis asks the nurse how Correct answer: 1 Contact dermatitis is an inflammatory response following prior sensitization to an antigen The core issue of the question is the ability to correctly describe contact dermatitis. The
he could have developed the condition. The nurse with production of a specific IgE antibody. Skin manifestations occur with subsequent wording of the question tells you the correct answer is also a true statement. Use the
includes in a response that contact dermatitis is exposures. The other options are false. process of elimination and nursing knowledge to make a selection.
caused by which of the following?
1.‐ Allergic reaction mediated by IgE
2.‐ Poor hygiene
3.‐ Reactivation of IgA antibodies
4.‐ Side effect of oral medication

872 When explaining the disorder to a client with tinea Correct answer: 2 Fungal infections such as tinea corporis may be transmitted by direct contact with animals The core issue of the question is knowledge of the characteristics of infection with tinea
corporis, the nurse should include which of the and other persons. Therefore, it is contagious from person to person. It does require corporis. The wording of the question tells you the correct statement is the correct answer.
following about this skin disorder? treatment, is not malignant, and is not treatable by sunlight. Use the process of elimination and nursing knowledge to make a selection.

1.‐ It requires no treatment.


2.‐ It can be passed human to human.
3.‐ It should be exposed to sunlight.
4.‐ It is a malignant skin condition.

873 A client asks the nurse about the meaning of the term Correct answer: 2 A full‐thickness burn involves all layers, including the epidermis and dermis, and may extend The core issue of the question is knowledge of the various depths of burn injury. The
“full‐thickness burn.” The nurse should respond that into the subcutaneous tissue and fat. The other options indicate varying depths of burn injury. wording of the question tells you the correct statement is the correct answer. Use the
burns classified as full‐thickness involve tissue process of elimination and nursing knowledge to make a selection.
destruction down to which of the following levels?

1.‐ Epidermis
2.‐ Subcutaneous tissue
3.‐ Bone
4.‐ Dermis

874 The client comes to the office for evaluation of a skin Correct answer: 1 The most important question for this office visit for evaluation of a skin rash would be to get The issue of the question is appropriate questions to ask when obtaining a nursing history
rash. What question would be included in obtaining a information about the chief complaint. In this case, it would be to investigate additional about a skin disorder. The wording of the question tells you the correct statement is the
history? information about the presenting rash. The other options are either unrelated or could be correct answer. Use the process of elimination and knowledge of health assessment to
asked at a later time. make a selection.
1.‐ “Have you ever had a skin rash like this before?”
2.‐ “Do you smoke?”
3.‐ “How long have you had that mole on your left arm?”
4.‐ “Do you have a family history of skin cancer?”

875 The nurse is assessing the client’s skin and wants to Correct answer: 2 When assessing petechiae, pressure applied to the site will not produce blanching of the skin. The issue of the question is knowledge of nursing assessment techniques for the skin. The
evaluate a site for petechiae. What technique can the For other lesions, blanching may occur. Options 1, 3, and 4 are false. wording of the question tells you the correct statement is the correct answer. Use the
nurse perform to help identify whether the site has process of elimination and nursing knowledge to make a selection.
petechiae?
1.‐ Rub the site and watch for bleeding.
2.‐ Apply pressure to the site to evaluate for blanching of the skin.
3.‐ Look for other lesions that are similar to that lesion.
4.‐ Apply a tourniquet in a limb and watch for the development of petechiae.

876 A client has been diagnosed with eczema. Which of Correct answer: 1 Skin care for eczema should include keeping the skin well hydrated and avoiding harsh soaps The core issue of the question is knowledge of appropriate care to the skin when the client
the following statements made by the client indicates (option 2). This can be done by using mild bath soaps and applying emollients immediately has eczema. The wording of the question tells you the correct statement is the correct
an understanding of the management of eczema? after bathing (option 4). Option 3 is false. answer. Use the process of elimination and nursing knowledge to make a selection.

1.‐ “I will avoid excessive use of soap and water and keep my skin well‐hydrated with emollients such as Eucerin.”
2.‐ “I will take daily baths and use strong antibacterial soaps to prevent skin infections.”
3.‐ “I will make sure to expose my skin to the sun at least 1 hour a day.”
4.‐ “I will wait 3 hours after bathing to apply lotion to my skin.”
877 The client diagnosed with psoriasis has been Correct answer: 4 Antihistamines are useful to help relieve itching. The other options do not explain the An understanding of the nature of the condition and the client’s symptoms are needed to
prescribed an antihistamine. The client asks the nurse rationale for the use of this type of medication with psoriasis. make the correct response. The wording of the question tells you the correct statement is
what this is used for because she does not have nasal the correct answer. Use the process of elimination and nursing knowledge to make a
allergies and congestion. What is the most appropriate selection.
response?
1.‐ “Antihistamines are not used for psoriasis and should not have been prescribed.”
2.‐ “The insert on the package will tell you what the medication is for.”
3.‐ “Let me ask the doctor because I’m not really sure.”
4.‐ “Antihistamines are used to help relieve itching associated with the lesions.”

878 Instructions for a client diagnosed with seborrheic Correct answer: 2 Coal tar shampoos are recommended for seborrheic dermatitis of the scalp. Over‐the‐ The core issue of the question is knowledge of care and treatment for seborrheic
dermatitis would include which of the following? counter shampoos may not control symptoms. Seborrheic dermatitis cannot be cured. dermatitis. The wording of the question tells you the correct statement is the correct
Symptoms can be controlled with proper treatment. answer. Use the process of elimination and nursing knowledge to make a selection.

1.‐ “Use any brand of shampoo because it does not make a difference in scalp therapy.”
2.‐ “Coal tar shampoos are recommended for seborrheic dermatitis of the scalp.”
3.‐ “Medicated shampoos are recommended because if not treated successfully, seborrheic dermatitis can cause permanent hair loss.”
4.‐ “Daily use of coal tar shampoos will cure seborrheic dermatitis.”

879 In teaching clients about measures to reduce the risk Correct answer: 2 The most common form of skin cancer is basal cell carcinoma, with approximately 400,000 The core issue of the question is knowledge of behaviors that can reduce the risk of
of developing basal cell carcinoma, the nurse should new cases per year. Protecting the skin with sunscreen SPF 15 or higher, along with avoiding developing basal cell carcinoma. The wording of the question tells you the correct
include which of the following? the sun during the peak hours of 10:00 a.m. to 2:00 p.m., is recommended to help prevent skin statement is the correct answer. Use the process of elimination and nursing knowledge to
cancer. make a selection.
1.‐ Limiting the use of tanning beds
2.‐ Using sunscreen with protection SPF of 15 or greater when exposed to the sun
3.‐ Only exposing the skin to the sun for 3 to 4 hours per day
4.‐ The role of diet in prevention of skin cancer

880 The client with cellulitis is being discharged from the Correct answer: 3 Infection may be manifested by fever, chills, erythema, tenderness, and drainage at the site, The core issue of the question is the risk of infection and the need to notify the health care
hospital. Discharge instructions for the client should especially if it is cloudy or serous. The physician must be notified if these symptoms occur. provider if signs of infection occur. The wording of the question tells you the correct
include which of the following statements? statement is the correct answer. Use the process of elimination and nursing knowledge to
make a selection.
1.‐ “If pustules develop, squeeze the lesion to remove the pus.”
2.‐ “If the lesion looks healed, stop taking the antibiotics so that you will not develop resistance to antibiotics.”
3.‐ “Monitor for signs of infection such as fever, chills, malaise, and redness or tenderness at the site.”
4.‐ “Drainage from the site is an expected finding and is no cause for concern.”

881 Which of the following is a priority nursing diagnosis Correct answer: 4 Clients with cellulitis experience pain at the local site. Controlling the pain is the priority Recall that pain relief is high priority for many clients and is included in the physiological
for a client experiencing the skin infection cellulitis? nursing diagnosis for this client. Option 1 may not apply unless the client is in pain. Options 2 needs on Maslow’s hierarchy. Use the process of elimination and nursing knowledge to
and 3 may or may not apply, but would have lower priority than the physiological need (option make a selection, considering that physiological needs take priority over psychosocial needs
4). in most cases.
1.‐ Sleep pattern disturbance related to skin infection
2.‐ Social isolation related to skin infection
3.‐ Powerlessness related to inability to control the infection
4.‐ Pain related to skin infection of cellulitis

882 The client receives a prescription to treat a skin Correct answer: 3 Current treatments for psoriasis include coal tar shampoo and topical steroids. Folliculitis, The core issue of the question is knowledge of the uses of medication therapy for
infection affecting the scalp and neck. The medications cellulitis, and furuncles are bacterial infections of the skin and would be treated with psoriasis. The wording of the question tells you the correct statement is the correct answer.
prescribed are coal tar shampoo and topical steroids. antimicrobial therapy. Use the process of elimination and nursing knowledge to make a selection.
The nurse concludes that which of the following is the
probable diagnosis of this client?
1.‐ Folliculitis
2.‐ Cellulitis
3.‐ Psoriasis
4.‐ Furuncles

883 A client has just been diagnosed with herpes virus Correct answer: 1 The initial outbreak of herpes is the most uncomfortable or painful. Recurrent episodes of The core issue of the question is knowledge of the characteristics and presentation of this
type 2. The nurse should share with the client which of herpes infection present with a prodrome of symptoms, such as tingling and burning. Herpes is type of viral infection. The wording of the question tells you the correct statement is the
the following pieces of information? a virus that may lie dormant for periods of time, and repeated episodes may occur during correct answer. Recall that the first outbreak is the most severe to make a selection.
periods of stress.

1.‐ “The initial outbreak of herpes is usually the most


uncomfortable. Recurrent infections usually present
with a prodrome of tingling and burning sensation
prior to the outbreak of the vesicle.”
2.‐ “Each outbreak of herpes is uncomfortable, with the amount of pain at the genital area worsening each time.”
3.‐ “You can only have one outbreak of herpes, so you will never experience another.”
4.‐ “You should never experience discomfort with herpes; therefore, if you have pain in the genital area, you probably have a sexually transmitted infection.”

884 A client presents for removal of a skin lesion after it is Correct answer: 4 To meet all four criteria for removal of a lesion, the lesion will be asymmetrical (A) with an The core issue of the question is knowledge of criteria that determine the need to remove
determined that he meets all four criteria for removal irregular border (B), have color change or more than one color (C), along with an increased a skin lesion. The wording of the question tells you the correct statement is the correct
according to the ABCD rule. The nurse interprets this diameter (D). answer. Use the process of elimination and nursing knowledge to make a selection.
to mean which of the following?

1.‐ The lesion is symmetrical, with a smooth border, a single color, and the diameter has stayed the same.
2.‐ The lesion is symmetrical with an irregular border, a single color, and the diameter has increased.
3.‐ The lesion is asymmetrical with a regular border, two colors, and the diameter is smaller.
4.‐ The lesion is asymmetrical with an irregular border, two colors, and has increased in diameter.

885 The nurse performing a skin examination of a client Correct answer: 2 Spider angiomas are red lesions with vessels radiating from the center. A venous star is a flat The core issue of the question is knowledge of various types of skin lesions. The wording of
notes the client has vascular skin lesions that are flat, blue lesion with radiating linear veins. Petechiae appear as red “freckles” or dots. A port wine the question tells you the correct statement is the correct answer. Use the process of
bright red in color, with tiny vessels that radiate out stain is a flat, irregular‐shaped lesion that does not have radiating vessels. elimination and nursing knowledge to make a selection.
from the center of the lesion. The nurse concludes that
the lesions are probably which of the following?

1.‐ Petechiae
2.‐ Spider angiomas
3.‐ Venous stars
4.‐ Port wine stains

886 A client presents on admission with pressure ulcers Correct answer: 4 Stage 4 ulcers result in full thickness skin loss with extensive damage to the muscle and bone. The core issue of the question is knowledge of various stages of ulcer development. The
extending into the bone. The nurse documents this wording of the question guides you to a decision. Use nursing knowledge to make a
ulcer at stage ____. Write in a numerical answer. selection.

887 A child was admitted to the emergency department Correct answer: 1 Coughing and wheezing may indicate that the child has inhaled smoke or toxic fumes. The core issue of the question is the ability to determine that the client’s airway could be
with a thermal burn to the right arm and leg. Which of Maintaining airway patency is the highest nursing priority in this situation. Skin color changes in jeopardy. Use the ABCs whenever a question deals with burn injury as a first method to
the following assessments requires immediate action? are expected. Thirst may be present but does not require immediate nursing action. determine priority setting.

1.‐ Coughing and wheezing


2.‐ Bright red skin with small blisters on the burn sites
3.‐ Thirst
4.‐ Singed hair
888 A school‐aged child develops eczema secondary to Correct answer: 2 Eczema is a chronic inflammatory skin disorder. School‐aged children are very aware of their The core issue of the question is recognition of key concerns of a child with eczema. The
food allergies. An appropriate nursing diagnosis for this own and others’ skin appearance. Children with eczema will feel different from other children, wording of the question tells you the correct statement is the correct answer. Use the
child would be which of the following? and this may affect their body image. Food allergies do not relate to decreased nutrition. process of elimination and nursing knowledge to make a selection.
Eczema does not affect the skin’s ability to maintain temperature, and it does not affect blood
flow to the area.
1.‐ Imbalanced nutrition: less than body requirements
2.‐ Disturbed body image
3.‐ Risk for ineffective thermoregulation
4.‐ Impaired tissue perfusion

889 An infant has a positive family history of allergies. To Correct answer: 4 Infants with eczema frequently have food sensitivities. Slow introduction of new foods allows The core issue of the question is appropriate client teaching to reduce risk of developing
reduce the risk of the infant developing eczema, the the parents to recognize food sensitivities and eliminate the offending item from the diet. The eczema. Recall the risk factors and use the process of elimination to make a selection.
nurse teaches the family to mother is taught to avoid scratchy clothing such as wool. Childhood immunizations would be
given as scheduled but do not reduce risk. Eczema is not an infectious disease. Avoiding
infectious personnel is appropriate for all children but does not prevent the development of
eczema.
1.‐ Avoid synthetic clothing—use natural fibers like cotton and wool.
2.‐ Keep up with the schedule of childhood immunizations.
3.‐ Avoid contact with infected personnel.
4.‐ Introduce only one new food a week so food allergies can be recognized and eliminated.

890 The parents of an 18‐month‐old with eczema are Correct answer: 3 The nurse should encourage parents to identify and discuss their feelings and concerns. The core issue of the question is the ability to use basic communication techniques in
concerned that a secondary infection that has Changing the topic or giving false reassurance is inappropriate. Merely not blaming parents responding to the concerns of a parent. Choose the option that directly addresses the
developed will permanently disfigure their child. How does not give them the opportunity to discuss what is important to them. client’s or family’s issues and concerns.
can the nurse best support the parents’ feelings?

1.‐ Divert the conversation to another topic.


2.‐ Let them know that they are not being blamed for their feelings.
3.‐ Encourage them to discuss their fears and concerns.
4.‐ Tell them not to worry because scarring is unlikely with eczema.

891 The child has just been admitted to the pediatric burn Correct answer: 3 Until a complete assessment and treatment plan are initiated, the child should be kept NPO. The core issue of the question is the need to avoid fluid intake during the acute phase of
unit. Currently, the child is being evaluated for burns to A complication of major burns is paralytic ileus, so until that has been ruled out, oral fluids burn injury when hemodynamics could be unstable. Use nursing knowledge to recall that
his chest and upper legs. He complains of thirst and should not be provided. fluid resuscitation needs to be done parenterally.
asks for a drink. What is the most appropriate nursing
action?
1.‐ Give a small glass of a clear liquid.
2.‐ Give a small glass of a full liquid.
3.‐ Keep the child NPO.
4.‐ Order a pediatric meal tray with extra liquids.

892 Hospitalization of a child has resulted in disturbance Correct answer: 3 Identification of nursing diagnoses that apply to the specific problem(s) of the child and family Concepts of hospitalization and family dynamics are needed to answer this question
of the dynamics of the family life. Which nursing is an essential step of the nursing process. Family‐centered care addresses the needs of the correctly.
diagnosis would be most appropriate for the nurse to family members, including the child's siblings. The primary goals are to maintain the
formulate? relationship with the child and siblings during the period of separation while hospitalized and
avoid boredom and distress for the hospitalized child.

1.‐ Diversional activity deficit related to separations from siblings and peers
2.‐ Disturbed sleep pattern related to unfamiliar surroundings
3.‐ Interrupted family processes related to hospitaliztion
4.‐ Ineffective individual coping related to procedures
893 The parent of a 6‐month‐old infant is concerned that Correct answer: 4 Fontanels are inspected and palpated for size, tenseness, and pulsation. The anterior fontanel Critical words are "6‐month‐old" and "concern that the anterior fontanel is still open." Use
the infant's anterior fontanel is still open. The nurse should be soft, flat, and pulsatile with the child in the sitting position. The anterior fontanel knowledge of normal closure of anterior fontanel to choose the correct answer.
would explain to the parent that further evaluation is should be completely closed by age 12 to 18 months. If the fontanel is found to be open after
needed if the anterior fontanel is still open after: 18 months, the child is referred for further evaluation.

1.‐ 6 months.
2.‐ 10 months.
3.‐ 12 months.
4.‐ 18 months.

894 The nurse determines that parents understood Correct answer: 4 Use of a mild soap such as Dove® or Tone® prevents the skin from excessive dryness in atopic Consider which activities would not be drying to the skin and which would not expose the
instructions about the care of the child with atopic dermatitis. Hot water is drying to the skin so should be avoided. Fabric softeners and many skin to perfumes.
dermatitis (eczema) after the parents state that the lotions contain perfumes that are irritating to the skin so should also be avoided.
child should:
1.‐ Take hot baths (not showers) daily.
2.‐ Liberally apply a lotion of choice over entire body.
3.‐ Use fabric softener for all clothes.
4.‐ Use mild soap only as needed.

895 A parent of a child with a full thickness burn asks why Correct answer: 2 Sulfamylon is a topical antibiotic that is used on burns to prevent bacteria from infecting the Knowledge of the actions for the medications used to treat burns will aid in choosing the
the nurse keeps spreading "that white cream" on the burn site. The other options are incorrect statements correct answer.
child's burns. The nurse explains that the cream is
mafenide (Sulfamylon), which is being applied to the
burned area because it is a:

1.‐ Lubricant that will keep the area well hydrated.


2.‐ Topical antibiotic that inhibits infection.
3.‐ Tissue hormone that stimulates new tissue growth.
4.‐ Steroid that reduces edema at the site.

896 A child has been diagnosed with scabies. The nurse Correct answer: 1, 5 The saliva, ova, and feces of the scabies mite triggers an antigenic response that causes Consider the symptoms of scabies to determine the correct response.
reinforces to the parents that, in addition to 5% intense pruritus. Pain is not present nor is scarring. Antibiotics would only be added to the
permethrin lotion, appropriate medical therapy for regimen if the itching leads to scratching and breaks in the skin with development of a
this child would include: (Select all that apply.) bacterial infection. Scabicides are used to kill the mites causing scabies.

1.‐ Antihistamines.
2.‐ Narcotic analgesics.
3.‐ Non‐steroidal anti‐inflammatory drugs (NSAIDs).
4.‐ Antibiotics.
5.‐ Scabicides

897 When assessing a teenager with atopic dermatitis Correct answer: 2 By the teenage years, eczema presents as large patches of thickened dried (lichenified) skin. The core concept in this stem is the age of the child in association with the disease
(eczema), the nurse would expect to find what type of Areas of excoriation, crusts, and papules characterize eczema in infants and younger children. process.
skin lesions? Bullae are not present in eczema.
1.‐ A reddened macular, papular rash
2.‐ Large lichenified plaques
3.‐ Vesicles and pustules
4.‐ Bullae and papules
898 A child with cellulitis of the leg is being treated at Correct answer: 3 The cause of cellulitis is a bacterial infection, often preceded by trauma. Colds and flu illness The wording of the question indicates that the correct answer is an incorrect statement on
home. Following patient teaching, which statement by are viral infections and are not related to cellulitis; thus further teaching is needed about the the part of the parents. Eliminate responses that contain correct information.
the mother indicates a need for additional education? cause. The child should complete all medication ordered and is not contagious so company is
allowed. If further swelling occurs, it could indicate the infection is not responding to the
antibiotics.
1.‐ "I will give my child antibiotics as ordered until he has completed the entire bottle of medication."
2.‐ "It's OK if he wants to have friends over to play board games."
3.‐ "From now on, I will seek medical treatment whenever my child gets a cold or the flu."
4.‐ If the leg swells further, I should call my doctor."

899 When telling parents how to apply topical steroids to Correct answer: 1 Topical steroids are readily absorbed through the skin; therefore, they should be applied Recall that corticosteroids would not be applied to normal skin so that option can be
treat eczema the nurse should explain that: sparingly over affected areas only. They are applied three to four times daily as ordered over eliminated. An antibiotic cream will be most effective on dry skin so that option can also be
clean dry skin. eliminated.
1.‐ It is applied in a thin layer over affected areas only.
2.‐ It is applied liberally to affected area and adjacent skin.
3.‐ It should only be applied at night.
4.‐ It is applied to damp skin only.

900 The nurse is admitting several children who have skin Correct answer: 3 Eczema is also called allergic dermatitis. It is not contagious. Scabies is caused by the mite and Determine the cause of each condition. Those that have an organism associated are
conditions to the hospital unit during the work shift. is highly contagious. Pediculosis capitis is an infestation with lice and can spread. Impetigo is an usually contagious.
When considering room assignments, the child who infection caused by staphylococci and streptococci and can also be spread on contact.
does not pose a risk for spread of his or her disease is
the child with:
1.‐ Scabies.
2.‐ Impetigo.
3.‐ Eczema.
4.‐ Pediculosis capitis.

901 The nurse is caring for several children with burn Correct answer: 3 Burns that are circumferential are always considered major because they can cause edema Of the four options, all are very similar except the option that describes the
injuries. The nurse determines that it is most that restricts blood flow to an extremity. Therefore, it is important to check pedal pulses in this circumferential burn.
important to check the pedal pulses in a child with: child. Pedal pulses should always be assessed, but the nurse would be more alert to the
possibility of impaired circulation in a child with a circumferential burn.

1.‐ A full‐thickness burn to the medial aspect of the left thigh.


2.‐ A full thickness burn to the posterior of the right leg.
3.‐ A circumferential burn to the upper half of the right lower leg.
4.‐ Partial and full thickness burns to the upper thigh of the left leg.

902 A 4‐year‐old child was just diagnosed with impetigo. Correct answer: 4 Handwashing is always the most important action that a nurse can take to prevent the spread Understanding the means of transmission is important in correctly responding to this
What is the most important action the nurse should of infection. Merely applying ointment or covering the site does not address the spread of question. Knowledge of impetigo and the prevention of the spread of infections will aid in
take to ensure that it does not spread? infection, nor does isolation of a child at home. The nurse would teach the family the choosing handwashing as the first line of defense against the spread of illness.
importance of good handwashing.
1.‐ Apply bacitracin.
2.‐ Keep it covered.
3.‐ Isolate the child at home.
4.‐ Teach and use good handwashing.
903 The nurse is providing home care instructions for a Correct answer: 2, 3, 4, Live nits can hatch up to 8 to 10 days later, so it is important to remove them from the Knowledge of the spread of lice and the home care necessary to prevent reinfestation is
family with a toddler diagnosed with lice. The nurse 5 environment. Soaking combs in a Lysol or anti‐lice shampoo mixture will kill lice or nits. Dry necessary to choose the correct answer. Identify those options that are absolutely incorrect
includes which of the following instructions in the cleaning is not necessary because home washing and drying on hot settings will be sufficient to first. Then consider the remaining options.
teaching plan? (Select all that apply.) kill lice and nits. Use of commercial sprays is not recommended. Each member of the family
should be assessed so those infested can be treated. Sharing of haircare material spreads lice
and should be avoided.
1.‐ Immerse combs and brushes in boiling water for 30 minutes to kill lice.
2.‐ Vacuum floor and furniture to remove hair that might have live nits.
3.‐ Have the mother use a bright light and magnifying glass to check the hair for lice.
4.‐ Launder the child’s bedding and clothing in hot water with detergent and dry in a hot dryer for 20 minutes.
5.‐ Teach children to not share combs, brushes, and hats.

904 The Emergency Department (ED) nurse hears a radio Correct answer: 2 The anticipated appearance of partial thickness burns is bright red skin with blisters of varying Organize the options from the least serious burn injury to the most serious. Since partial
transmission from an ambulance stating that a 10‐year‐ sizes. A superficial burn typically only has pink or red skin. A full thickness burn may be dark in thickness falls in the middle of the least serious to most serious, this should allow the
old boy is en route who sustained partial thickness color, from deep red to black. learner to select the right option.
burns to his right arm and abdomen after tossing
gasoline on a fire. On admission to the ED, the nurse
expects the appearance of the burn site to be:

1.‐ Smooth and bright red.


2.‐ Bright red with numerous blisters.
3.‐ White and waxy.
4.‐ Dark brown and firm.

905 Permethrin 5% (Elimite) is prescribed for a 10‐year‐ Correct answer: 4 Permethrin is applied to cool dry skin after a bath, but only from the neck down. The child Scabies primarily occur where skin is in contact with skin. Applying the medication to the
old child diagnosed with scabies. What instructions may dress after the lotion is applied. It should be washed off after 8 to 12 hours. A second head would be unnecessary. It is necessary to remember how long the medication must
should the nurse provide for the mother? application is often prescribed for 1 week later. remain in contact with the skin. Since the scabies burrow into the skin, it would make sense
that a longer contact time is needed for treatment.

1.‐ Apply the lotion liberally from head to toe.


2.‐ Wrap the child in a clean sheet after treatment.
3.‐ Leave the lotion on for 10 minutes, then rinse.
4.‐ Apply lotion only after the child has had a bath and dried thoroughly.

906 When assessing a child with a possible diagnosis of Correct answer: 1 A recent history of otitis media is often present in children with facial cellulitis. Sunburn Knowledge of cellulitis and the etiology and pathophysiology of cellulitis will aid in
facial cellulitis, the nurse will want to question the would present as more diffuse and widespread redness. An insect or animal bite can be a answering the question.
parent about a recent history of: cause of cellulitis, but in the case of cellulitis on the face the nurse would question a recent
history of an ear infection first if a bite was not obvious. Dental caries are unrelated.

1.‐ Otitis media.


2.‐ Cat scratch.
3.‐ Sunburn.
4.‐ Dental caries.

907 In teaching a group of school‐age children, a nurse Correct answer: 2 Lice can only be passed by direct contact because lice do not fly. The usual mode of Select the option that allows for direct contact. The incorrect options do not allow for
would explain that lice on a child can be most easily transmission is sharing of hats, combs, brushes, or hair ornaments. Being close to someone in direct contact of the infected individual or contact with the infected individual’s belongings.
spread by: a classroom, bus, or car does not presuppose direct contact with hair or nits that have been
shed on hair.
1.‐ Sitting close to someone who has lice.
2.‐ Sharing hats at recess.
3.‐ Riding in the same car.
4.‐ Sharing a seat on the same bus.
908 The nurse is developing a care plan for a 10‐year‐old Correct answer: 3 Keeping the skin well hydrated will prevent the need to scratch dry skin that can lead to Consider the common symptoms of the disease to determine typical client goals.
girl with eczema of the elbows, hands, and face. The excoriation and secondary infection. Eczema is not infectious, nor is it managed by dietary Determine that the correct answer must address the needs of the skin based on disease
nurse would formulate which of the following as an restrictions. Pruritus, not pain, is associated with eczema. pathophysiology.
appropriate client goal for this child?

1.‐ Pain will be managed.


2.‐ Infection will not spread.
3.‐ Skin will be well hydrated.
4.‐ Dietary restriction will be maintained.

909 A 5‐year‐old boy was brought to the Emergency Correct answer: 2 Because he was in close proximity to the fire and tried to put it out, he is at risk of having Recall that a patent airway is almost always the primary assessment. Assessing pain,
Department after being burned trying to put out a fire inhaled smoke and therefore having a compromised airway. Other physiological signs will be of psychosocial needs, or for infection occurs only after establishing airway patency.
in his closet, where he was playing with matches. The next highest priority, such as pain. Infection would be a third priority since it would not happen
priority nursing assessment for this child would be: immediately, and psychosocial concerns are addressed once physiological needs have been
met.
1.‐ Level of pain.
2.‐ Airway patency.
3.‐ Psychosocial needs.
4.‐ Signs of infection.

910 Intravenous (IV) morphine sulfate is ordered for a 13‐ Correct answer: 2 The predictable rate of absorption makes IV morphine useful in treating severe pain. As part The core concept is the purpose for giving morphine by the IV route. Eliminate options 3
year‐old girl hospitalized with major burns to 30% of of the physiological stress response, blood is shunted away from the gastrointestinal tract, and 4 because they are false. Choose option 2 over 1 because it directly addressed the
her body. A licensed practical nurse (LPN) asks the making oral absorption rates less predictable. The IV route will not prevent ileus and may purpose of this route.
registered nurse (RN) why the morphine is given by the actually have greater side effects because of rapid onset of action. The half life of the drug is
IV route when the child can talk and swallow. The RN not relevant to the question asked.
should explain to the LPN that, when given by the IV
route, morphine does which of the following?

1.‐ Has a longer half‐life


2.‐ Has a predictable absorption rate
3.‐ Prevents ileus
4.‐ Leads to fewer side effects

911 The nurse is providing a teaching session for parents Correct answer: 4 Permethrin is the over‐the‐counter treatment of choice for head lice. Other choices are Note that three of the medications are antibacterial. The one that is different is the only
about over‐the‐counter treatment for head lice. Which topical agents, but they would not be used for lice. Option 1 would be used for infection, while medication listed for lice.
of the following would the nurse mention as options 2 and 3 would be used to treat burns.
appropriate for treating this problem?

1.‐ Neosporin
2.‐ Mafenide (Sulfamylon)
3.‐ Silver sulfadiazine (Silvadene)
4.‐ Permethrin (Nix)

912 When bathing a 3‐year‐old with eczema, the nurse Correct answer: 3, 5 Hot water can exacerbate symptoms of eczema and increase pruritus. Tepid water feels more Options 1 and 2 include the word “hot,” which would be wrong, leaving only options 3 and
tells the mother to have the bathwater: (Select all that comfortable than cool water. Strong or harsh soaps and perfumed products could be irritating 4 to choose between.
apply.) to the skin and should not be used.
1.‐ As hot as the child can tolerate.
2.‐ Hot to the touch on the inner wrist.
3.‐ Tepid to the touch.
4.‐ Cool.
5.‐ Cool with soap bubbles added.
913 The nurse explains to the mother that a child who has Correct answer: 2 Impetigo remains contagious for 48 hours after antibiotics are begun. The presence or With many infectious diseases, the patient is not considered contagious after 48 hours of
begun treatment for impetigo with a topical antibiotic absence of crusts does not address the issue of contagion. antibiotics.
can return to daycare:
1.‐ Immediately.
2.‐ After 48 hours.
3.‐ As soon as crusts are evident.
4.‐ When crusts fall off.

914 When assessing a child’s hair and scalp, the nurse Correct answer: 3 The characteristic appearance of pediculosis capitis (lice) is nits that adhere to the hair shaft The critical words are "dandruff . . . does not flake off easily."
notices what looks like dandruff, but it does not flake about 1/4‐inch from the scalp. They cannot be easily brushed off as dandruff. Scabies, eczema,
off easily. The nurse suspects the child has: and impetigo do not typically appear on the scalp and present as skin lesions elsewhere on the
body.
1.‐ Scabies.
2.‐ Eczema.
3.‐ Pediculosis capitis.
4.‐ Impetigo.

915 A child has been admitted to the burn unit with a Correct answer: 4 The fluid shift that occurs in burns leads to edema, so the burned extremity should always be Options 2 and 4 are opposites. One of them is likely to be the correct answer. Use gravity
circumferential burn to the right leg. The nurse will elevated above the level of the heart. as the method of making a final selection.
position the client:
1.‐ Flat in bed.
2.‐ With the right leg dependent.
3.‐ On the left side.
4.‐ With the right leg elevated.

916 A 3‐year‐old child is suspected of having eczema. The Correct answer: 1 Eczema in a young child tends to be characterized by dry, scaly crusts that are well Eliminate any option that describes skin lesions that would not be dry and scaly.
nurse assesses for which of the following as a major circumscribed. Pruritus is always present.
symptom of eczema?
1.‐ Pruritus
2.‐ Pustules
3.‐ Vesicles
4.‐ Lichenification

917 A child has been diagnosed with eczema. While taking Correct answer: 3 About 60% of children with eczema have a family history of asthma or other allergy. Scabies is Since another name for eczema is atopic dermatitis, and the term atopy refers to allergies,
the nursing history, the nurse will assess for a family caused by contact with a mite; impetigo and cellulitis are bacterial infections. the nurse would look for this family history.
history of:
1.‐ Scabies.
2.‐ Cellulitis.
3.‐ Asthma.
4.‐ Impetigo.

918 When assessing a child with facial cellulitis, the nurse Correct answer: 2 Sinusitis frequently precedes periorbital cellulitis. Facial cellulitis may be preceded by otitis The suffix “‐itis” refers to inflammation, which is often related to infection. Look for an
will want to ask the parent about a recent history of: media. A dog bite could cause cellulitis anywhere. Sun exposure causes a thermal injury. option that suggests another infection in the general area.

1.‐ Nosebleeds.
2.‐ Sinusitis.
3.‐ Dog bite.
4.‐ Sun exposure.
919 A child will be treated for cellulitis of the left leg. The Correct answer: 1 The only way to eliminate the infectious agent is to complete the prescribed course of Any bacterial infectious disease requires completion of the antibiotic therapy.
nurse will include in the care plan the need for: antibiotics. Strict bed rest is not indicated, although the child initially may feel more
comfortable resting with the extremity elevated. Fluid intake has no effect on the course of
the infection, which is not contagious; therefore, visitors do not have to be limited.

1.‐ Continuing oral antibiotics until the prescription is completed.


2.‐ Strict bed rest with the left leg elevated.
3.‐ Increased fluid intake.
4.‐ Limiting visitors to prevent spreading infection.

920 There have been several cases of lice in the Correct answer: 4 Lice is spread by sharing combs and hats. Close contact is required as the lice do not jump or Option 1 can be eliminated as healthy children are not treated. Lice is spread by contact,
elementary school. The school health nurse is fly. so options 3 and 4 are the options to choose between.
providing information about lice prevention at the PTA
meeting. Correct information should include:

1.‐ All children in the school should be treated with the anti‐lice shampoo.
2.‐ Lice readily jumps from one head to another, so a large number of children will be affected.
3.‐ Lice can be spread by the family pet.
4.‐ Wearing each other’s hats will spread the lice infestation.

921 The nurse is working with a teenager diagnosed with Correct answer: 3 A teenager can and should be part of the treatment plan. If itching is avoided to prevent The key to this question is to increase treatment compliance. Learning that scarring can be
eczema. In order to increase treatment compliance, excoriation and secondary infection, scarring is unlikely. Improvement is often slow, and the prevented will encourage compliance.
the nurse will explain: problem may persist into adulthood. Food avoidance will not change the course of the disease.

1.‐ The appearance of the skin will improve in a few days.


2.‐ Avoiding foods with eggs and milk will speed healing.
3.‐ Scarring is not likely if the treatment plan is followed.
4.‐ This problem will not likely recur past adolescence.

922 The nurse is teaching self‐care to a client with Correct answer: 1 Emollients will ease the problem of dry skin that increases pruritus and causes the psoriasis to Recognize that the longest and correct answer mentions the scales found in psoriasis.
psoriasis. The nurse should encourage which of the be worse. Washing and drying the skin with rough linens or pressure may cause excoriation.
following for his scaled lesion? Constant occlusion may increase the effects of the medication and increase the risk of
infection.
1.‐ Emollients and moisturizers to soften the scales, and soft brushing of scales
2.‐ Importance of follow‐up appointments
3.‐ Use of a clean razor blade each time he shaves
4.‐ Keep occlusive dressings on the lesions 24 hours a day

923 The nurse teaches a client that the first step in self‐ Correct answer: 2 In order to plan the appropriate management of contact/irritant dermatitis, the cause of the Recall the logical first step; remove the cause.
management of contact/irritant dermatitis is to do inflammation should be identified. Removal of the cause may be all the treatment needed.
which of the following? Antihistamines and hydrocortisone creams are treatment options. Skin testing may be helpful
to determine the allergen if not evident in the history.

1.‐ Take antihistamines to control the itch.


2.‐ Identify and remove the causative agent.
3.‐ Use over‐the‐counter (OTC) hydrocortisone cream.
4.‐ Refer for allergy testing.

924 A mother asks how her 2nd‐grade child got head lice. Correct answer: 2 Lice are transmitted by direct contact with infested persons or by sharing hats, brushes, or Knowledge of the spread of head lice is needed for this question.
The nurse responds by telling the mother that lice: combs of infected persons. Classrooms are excellent areas for close contact, and children
often do not know when other classmates have lice. Options 1, 3, and 4 are incorrect.
1.‐ Only occurs in socioeconomically deprived people.
2.‐ Was probably spread by person‐to‐person contact in the classroom.
3.‐ Is an airborne infestation.
4.‐ Is due to improper washing of her hair.

925 The nurse conducting a health fair teaches attendees Correct answer: 1 The disease is ten times more common in fair‐skinned people who work indoors. This In this question look for opposites in the responses and make sure both components of
that which of the following groups of people are most population often experiences severe sunburns and blistering in childhood and tends to the question are correct.
at risk for developing malignant melanoma? vacation in areas of intense sun exposure. Remember, episodic intense sun exposure is more
damaging than constant exposure.
1.‐ Light‐skinned people who work indoors
2.‐ Dark‐skinned people who work indoors
3.‐ Light‐skinned people with regular sun exposure
4.‐ Dark‐skinned people with regular sun exposure

926 A client is admitted to the emergency department Correct answer: 4 There is no sensation of pain to light touch in full thickness burns because the pain and touch Remember full thickness burns are painless, except around the edges where partial
with 50 percent burns to the chest and arms. The skin receptors have been destroyed. There may not be pain with some partial thickness degree thickness burns exist.
is white, dry, and there is no pain. The nurse assesses burns, but the appearance described is characteristic of full thickness.
the type of burn the client has as which of the
following?
1.‐ Superficial thickness
2.‐ Superficial partial thickness
3.‐ Deep partial thickness
4.‐ Full thickness

927 A young boy is brought to the trauma unit with a Correct answer: 3 A burn involving the face, neck, or chest may cause airway closure because of the edema that Remember the ABCs (airway, breathing, circulation). Airway is a key priority in questions.
chemical burn to the face. Priority assessment would occurs within hours. Remember the ABCs: airway, breathing, and circulation. Airway always
include which of the following? comes first, even before pain. The nurse will also assess skin integrity (option 1), blood
pressure and pulse (option 2), and pain (option 4), but these are not the highest priority
assessments.
1.‐ Skin integrity
2.‐ Blood pressure and pulse
3.‐ Patency of airway
4.‐ Amount of pain

928 A 10‐year‐old female client complains of dandruff. On Correct answer: 4 Pediculosis capitis is head lice, and nits are cemented to the hair shaft. They are most This question requires knowledge of the appearance of head lice.
examination, the nurse notices the dandruff flakes commonly seen on hair on the back of the head near the nape of the neck. A papular
don't brush off the hair, and there is a papular rash on excoriation may be present at the nape of the neck secondary to scratching.
her neck. The nurse suspects which of the following
disorders?
1.‐ Tinea capitis
2.‐ Dandruff
3.‐ Seborrheic dermatitis
4.‐ Pediculosis capitis

929 The nurse is most concerned about a wasp sting for a Correct answer: 3 If the client previously had a reaction to a wasp sting, immediate treatment must be Remember the risk in allergies is with second and subsequent exposures.
client who: administered. If a reaction is anticipated, do not complete the exam or wait for symptoms to
develop, be proactive. Pain often causes the blood pressure to rise. The client who has never
been stung should be monitored closely.
1.‐ Has never been stung before.
2.‐ Has a history of fever or chills when bitten.
3.‐ Had hives and shortness of breath with the last sting.
4.‐ Had a rise in blood pressure to 140/90 when stung.
930 A client who has been on two antibiotics complains of Correct answer: 2 Candidiasis (oral thrush) often develops as a result of the overgrowth of bacteria after a client This question requires knowledge of secondary infections.
burning on the tongue and doesn't want to eat. has been on an antibiotic.
Inspection of the tongue reveals a white, milky plaque
that does not come off with rubbing. The nurse
suspects which condition?
1.‐ Impetigo
2.‐ Candidiasis
3.‐ Burns
4.‐ Herpes

931 The nurse would include which of the following pieces Correct answer: 1 Since a wart is a virus‐induced epidermal tumor, it may reappear at the original site or Use the process of elimination to select the correct and the longest answer.
of information in health teaching for a client with another body area despite the fact that the original wart was removed. Immunity may develop
warts? to further warts after 5 years.
1.‐ They are viral and may reappear at the same site or other areas of the skin.
2.‐ They will not go away without treatment.
3.‐ They cannot be transmitted.
4.‐ They only appear in childhood.

932 A 70‐year‐old female complains of burning and itching Correct answer: 2 Dermatomal pain, itching, or burning may be severe and often begins 4 to 5 days before Recall that herpes effects along nerve lines.
for several days in her right abdominal area. She thinks eruption occurs. The trunk is affected in the majority of cases. An insect bite usually manifests
she has been bitten by something because she can feel immediately, warts do not burn or itch, and scabies usually do not burn. Scabies form lines in
small bumps at the site. The nurse suspects which of the folds of the skin.
the following conditions based on this assessment
data?

1.‐ Insect bite


2.‐ Herpes zoster
3.‐ Scabies
4.‐ Warts

933 A 23‐year‐old male presents with a pustular Correct answer: 3 Folliculitis is an inflammation of the hair follicle caused by infection, chemical irritants or Use the principles of asepsis to answer this question.
inflammation of his neck. Which of the following would injury. It is most commonly caused by Staphylococcus aureus. Using a clean razor each time he
the nurse use as an important management strategy? shaves will decrease the risk of reinfection.

1.‐ Squeeze the area periodically


2.‐ Keep the area moist
3.‐ Changing razors each time he shaves
4.‐ Strict dietary changes

934 A client asks the nurse in the dermatologist's office Correct answer: 4 All three diagnoses are a bacterial infection of the skin arising from the hair follicle, where Notice that bacterial infection is repeated in two options so look at those closely for the
about what characteristics folliculitis, furunculosis, and bacteria can accumulate, grow, and cause a localized infection. Options 1 &amp; 3 are correct answer.
carbunculosis all have in common. The nurse replies incorrect.
that all three disorders are:

1.‐ Viral infections involving the epidermal layer of the skin.


2.‐ Fungal infections caused by poor hygiene.
3.‐ Bacterial infections that are highly contagious.
4.‐ Bacterial infections involving the hair follicles.
935 A 2‐year‐old child has been sick with a cold and Correct answer: 1 Impetigo is an infection of the skin typically beginning with a vesicle or pustule. The lesion This question requires knowledge of impetigo as a bacterial infection.
develops a honey‐colored lesion on her chin within the ruptures, leaving an open area that discharges a honey‐colored serous liquid that hardens into
past 3 days. The nurse determines that this clinical a crust. Impetigo spreads quickly if not treated.
picture is consistent with which of the following?

1.‐ Impetigo
2.‐ Scabies
3.‐ Herpes simplex
4.‐ Contact dermatitis

936 The nurse would include which of the following in the Correct answer: 4 Psoriasis is a chronic disease with factors such as stress precipitating an exacerbation. A Use the process of elimination to choose the correct answer.
nursing management of the client with psoriasis? healthy lifestyle is recommended and includes a well‐balanced diet, frequent exercise,
moderate alcohol intake, and avoidance of tobacco products. The disease cannot be
transferred to another person, keeping the skin moist relieves the itching, and medications can
be effective.
1.‐ Emphasize that the disease can be transmitted by skin‐to‐skin contact
2.‐ Emphasize the importance of keeping the areas dry
3.‐ Medications can be tried but may not be effective
4.‐ Educate the client on healthy lifestyle changes

937 A client presents to the clinic with thick, white, flaky Correct answer: 1 Normally, the keratinocyte migrates to the outer layer of the skin in 14 days. Psoriatic skin Use the process of elimination to choose the correct answer.
skin. The nurse knows that these symptoms occur cells complete this journey in 4 to 7 days producing an abnormal keratin that forms thick, flaky
when the cell cycle is shortened, such as occurs with scales at the surface of the skin. Options 2, 3 and 4 do not have white, flaky skin as a
which of the following diseases? characteristic.
1.‐ Psoriasis
2.‐ Herpes
3.‐ Impetigo
4.‐ Acne

938 The nurse working at a community health fair in the Correct answer: 3 Melanomas tend to have asymmetry (A), border irregularity (B), color variegation (C), and Recall the ABCD rule for melanomas.
booth for skin assessment should be teaching the diameter (D) greater than 6 mm. The ABCD rule can be applied to any skin condition, but
ABCD rule as a definitive measurement for which malignant melanoma is the most severe and has changes in all 4 of the rules.
diagnosis?
1.‐ Basal cell carcinoma
2.‐ Squamous cell carcinoma
3.‐ Malignant melanoma
4.‐ Acne

939 A man suffers from burns to the arms and chest when Correct answer: 2 Restrictive jewelry and clothing are removed immediately from the burn victim to prevent Recall knowledge of pressure areas.
a fire got out of control. The nurse removes jewelry circumferential constriction of the torso and extremities. Assessment of the fingers and
from the affected burn site in order to do which of the interference with treatment may require removal of jewelry, but the primary reason is option
following? 2.
1.‐ Be able to assess the fingers more accurately
2.‐ Prevent a tourniquet effect at the site
3.‐ Prevent infection
4.‐ Avoid interference with the first aid treatment of the burn

940 A 9‐year‐old child plays three baseball games on a Correct answer: 4 A superficial thickness burn, such as sunburn, involves only the epidermal layer of the skin. Recall that sun exposure increases the risk of sunburn.
hot, humid summer day. The child is at high risk for The color ranges from pink to bright red and small painful blisters may form.
dehydration and which of the following conditions?

1.‐ Skin cancer


2.‐ Full thickness burn
3.‐ Partial thickness burn
4.‐ Superficial thickness burn

941 When evaluating a client with a new skin lesion, it is Correct answer: 3 Tanning and sun exposure can increase susceptibility to skin cancer and not all individuals use Select the client at highest risk.
most important to ask the client about which of the a sunscreen or consider tanning as having serious consequences.
following?
1.‐ What foods they eat
2.‐ What cosmetics they use
3.‐ Sun tanning habits
4.‐ Allergies

942 The nurse teaches the client with urticaria about Correct answer: 1 Moisturizing the skin helps decrease dryness that can aggravate pruritus. Soaps and hot water Recall that dry skin leads to itching, requiring lotion.
controlling the pruritus. Which of the following are also drying to the skin. Tepid water and a mild soap should be used and it is not necessary
statements would the nurse include in discussion with to take a bath daily. Oral intake should be increased to bring extra moisture to the skin, and
the client? only those perfumes/lotions with alcohol should be avoided.

1.‐ Skin lubricants help maintain moisture


2.‐ The client should bathe frequently using soap and water
3.‐ Oral fluid intake should be maintained as usual
4.‐ All perfumes or lotions should be avoided

943 A male client complains of having red lesions on the Correct answer: 2 Staphylococcus aureus is the most common organism causing folliculitis, furuncles, and When given choices, be sure both answers are correct.
face after shaving and wants to know what it is. The carbuncles. The red lesion noticeable after shaving is common in inflammation of the hair
nurse identifies these as characteristic of: follicle.
1.‐ Carbunculosis caused by Streptococcus.
2.‐ Folliculitis caused by Staphylococcus aureus.
3.‐ Furunculosis caused by E. coli.
4.‐ Impetigo caused by Klebsiella.

944 In planning care for a female client with tinea capitis, Correct answer: 2 Hair loss occurring with tinea capitis is usually temporary but can be of extreme concern for a Knowledge of tinea capitis is required.
the nurse should explain that the client should expect female. The other responses are incorrect for this disorder.
which of the following?
1.‐ Characteristic, permanent circular, erythematous lesions
2.‐ The probability of temporary alopecia
3.‐ A chronic yellowing of the toenails
4.‐ Vesiculopustular areas on fingernails

945 In teaching the client about her topical medication for Correct answer: 4 Symptoms may resolve quickly after a few days, but the full course of prescribed medication Remember to ensure that a full course of antibiotics is taken.
candidiasis, the nurse should stress which of the for antifungals (7 to 10 days) should be used to prevent recurrence.
following?
1.‐ Stop the medication as soon as the symptoms disappear
2.‐ It is not necessary to keep the area dry while using the medication
3.‐ Apply a thick coating of medication to the rash if necessary.
4.‐ Use of topical medications should continue for the prescribed time.
946 The mother of a 5‐year‐old child with recurrent Tinea Correct answer: 1 The combination of moisture and the rubber boots contribute to fungal growth. Wearing Recall that moisture increases risk of infection.
pedis asks the nurse how the child got the condition. clean cotton socks daily and changing them frequently decreases the moist environment of the
After talking with the mother about the child's habits, feet. The other two options should not be problematic.
the nurse determines that which of the following
would be the most likely cause?

1.‐ Wearing rubber boots with moist feet several days in a row
2.‐ Wearing sandals every day to play
3.‐ Wearing cotton socks and tennis shoes to school
4.‐ Wearing house‐shoes around the house and going barefoot outside

947 A client goes to the dermatology clinic for the Correct answer: 4 Plantar warts occur at pressure parts on the soles of the feet, which prevent the wart from Recall that pain is a key symptom of plantar warts.
removal of a wart. The nurse documents which of the growing outward so they extend deeper and become painful.
following most classic distinguishing features of a
plantar wart?
1.‐ Flattened appearance
2.‐ Scaling
3.‐ Itching
4.‐ Pain

948 The nurse conducting discharge teaching for a client Correct answer: 3 Increased pain, fever, drainage, or spread of blisters can indicate a secondary infection. The This question requires knowledge of the symptoms of secondary infections.
with herpes zoster should include which of the disease is contagious to people who have not had chickenpox. Secondary occurrences are rare.
following in discussions with the client? Antiviral medications, antipruritics, and pain medications are usually prescribed.

1.‐ Plan to entertain guests as usual since the condition is not contagious.
2.‐ It is highly likely that there will be recurrence of the disease.
3.‐ Report any increased pain, fever, drainage, or spread of blisters to the healthcare provider.
4.‐ Use over‐the‐counter preparations, since prescription medications are not necessary.

949 A client is admitted with 25 percent total body Correct answer: 1 Fluid shifts from the intravascular compartment to the interstitial compartment due to Be sure both options are correct when choosing answers with more than one item
surface area (TBSA) burned. The rationale for damage to the cells' permeability, causing a drop in blood pressure and edema. Option 2 does mentioned.
administering intravenous fluids initially is which of the occur in the beginning, but is not the reason for administering fluids. Notice the question asks
following? about initially; options 3 and 4 would be appropriate rationales for decreasing fluid later on.

1.‐ The intravascular compartment becomes dehydrated.


2.‐ The interstitial compartment becomes overhydrated.
3.‐ The intravascular compartment becomes overhydrated.
4.‐ The interstitial compartment becomes dehydrated.

950 Which of the following clients would be most likely to Correct answer: 3 The major etiological factor in basal cell carcinoma is solar radiation. An outdoor construction Knowledge is needed of the causes of skin cancer.
develop basal cell carcinoma? worker is more prone to constant sun exposure and for long periods of time. Chemicals
(options 1 and 4) and radiation (option 2) are not causes.
1.‐ A chemistry teacher
2.‐ An x‐ray technician at a local hospital
3.‐ A construction worker on an oilrig in the ocean
4.‐ A janitor at the local high school

951 In teaching a client about sunscreens, the nurse Correct answer: 3 A waterproof or water‐resistant sunscreen with SPF 15 or more should be used before every Note that this question refers to the minimum level of protection.
advises using which of the following as the minimum exposure to the sun.
level of protection?
1.‐ Lotion with sun protection factor (SPF) of 30
2.‐ Oil with SPF of 4
3.‐ Lotion with SPF of 15
4.‐ Oil with SPF of 12

952 A client diagnosed with senile lentigo is concerned Correct answer: 1 Senile lentigo occurs on UV light‐exposed skin of older Caucasian adults and is benign. It is This question requires knowledge of senile lentigo.
about skin cancer. The best response by the nurse commonly called "liver spots." Option 2 is premalignant lentigo; option 3 is squamous cell
would be? carcinoma; and option 4 is commonly called benign lentigo.
1.‐ "These are known as liver spots and are benign."
2.‐ "This form of lentigo may become malignant in 10 to 15 years."
3.‐ "This is a slow‐growing, atypical type of skin cancer that can be removed."
4.‐ "These spots are benign and will fade in the winter."

953 A mother asks the nurse for advice concerning a Correct answer: 3 Shampooing alone is not enough to remove the eggs; manual removal with a comb is Use the process of elimination, noting that option 3 is a more inclusive answer than option
recurring case of head lice in her child. Upon necessary. Also, NIX does not need to be repeated in 7 to 10 days, it is a one‐time treatment. 1.
assessment the nurse discovers the mother washed The stuffed animals and pillows must be washed but do not need to be discarded.
the child's hair one time with NIX. Further instructions
would be to:
1.‐ Repeat the shampoo now and again in 7 to 10 days.
2.‐ Discard all her stuffed animals and pillows.
3.‐ Repeat the treatment and use a comb to manually remove the nits.
4.‐ Seek the advice of a physician.

954 A client presents to the clinic with herpes zoster. It is Correct answer: 3 Herpes zoster is believed to result from reactivation of a varicella virus that has remained in This question requires knowledge of the relationship of varicella to herpes zoster.
most important for the nurse to ask about which of the sensory dorsal ganglia after a childhood infection of chickenpox.
the following?
1.‐ Last tetanus booster immunization
2.‐ Home medications
3.‐ Prior varicella infection
4.‐ Previous surgeries

955 A 40‐year‐old male presents to the emergency Correct answer: 3 A client with previous reactions to insect bites may have a severe reaction to a large number Remember the ABCs (airway, breathing, circulation). Airway is the priority.
department with 20 to 30 hornet stings after hitting a of stings by hornets. Options 1, 2, and 4 are characteristic of insect bites and should be
nest while mowing the lawn. The wife states he is watched closely. Option 3 is extremely concerning and could mean an anaphylactic reaction
allergic to bee stings. Which of the following has begun.
symptoms warrants immediate attention?

1.‐ Erythema and edema at the sites


2.‐ Generalized itching and swelling
3.‐ Shortness of breath and blood pressure 90/60
4.‐ Nausea and vomiting

956 A client presents with a skin rash on her hands, Correct answer: 2 The lesions of scabies are characteristic: small red‐brown burrows sometimes covered with This question requires knowledge of scabies.
elbows, and axillary areas. On examination, the nurse vesicles. The collection of lesions appears as a rash. The other responses do not have these
assesses raised burrows with thread‐like ridges classic characteristics.
between the fingers, on the palms, antecubital spaces,
and axillary areas. The client probably has which of the
following disorders?
1.‐ Poison ivy
2.‐ Scabies
3.‐ Contact dermatitis
4.‐ Urticaria
957 A client visits the dermatology clinic at 1:00 P.M. with Correct answer: 2 True urticaria lesions do not last longer than 24 hours. If a lesion lasts longer than that time, This question requires knowledge of urticaria.
what appears to be urticaria lesions. Confirmation of other differential diagnoses must be investigated.
the diagnosis would be made based on information
that the lesions first started:

1.‐ Around 8:00 A.M. the day before.


2.‐ Around 6:00 P.M. the day before.
3.‐ Around noon two days ago.
4.‐ Around 10:00 P.M. 2 days ago.

958 Nursing management of a client with a superficial Correct answer: 4 Treatment of superficial thickness or first‐degree burns includes cleansing with a mild soap Choose the least invasive option.
thickness burn includes which of the following? such as Phisohex, topical anesthetics as needed, and no dressings.

1.‐ Hospitalization to burn unit


2.‐ Large bulky dressings
3.‐ Oxygen therapy
4.‐ Topical anesthetic creams or sprays several times a day

959 A 45‐year‐old male asks the nurse to assess several Correct answer: 3 Seborrheic keratosis appears as brown "stuck‐on" spots over the trunk, may bleed when This question requires knowledge of seborrheic keratosis.
spots that have suddenly appeared. They are large, irritated by clothing or picking, are usually benign, and occur in the middle aged.
smooth, velvety, and brown. The nurse determines
that these are characteristic of which of the following
conditions?
1.‐ Lentigo
2.‐ Squamous cell carcinoma
3.‐ Seborrheic keratosis
4.‐ Melanoma

960 The client with a skin rash has been prescribed an Correct answer: 1 Although option 4 is true, option 1 is the appropriate rationale for use of an antihistamine for This question requires knowledge of the action of antihistamines.
antihistamine and asks why it is needed. The nurse most skin rashes. Option 2 is not therapeutically stated and option 3 avoids the client’s
responds by using which of the following as the best question.
explanation?
1.‐ "Antihistamines are used to help control and relieve itching which in turn decreases scratching and prevents secondary infections."
2.‐ "The pharmacist should be able to explain why."
3.‐ "Antihistamines are normally not prescribed for skin rashes, but the physician must have felt it was necessary."
4.‐ "Antihistamines decrease the pain and foster sleep."

961 In the rehabilitation phase of care for a burn client, Correct answer: 3 The rehabilitative stage of burn injury is to return the client to the highest level of health Note that the word ‘rehabilitation’ in the question stem is key in answering this question.
the nurse would implement which of the following as a restoration, which includes physical therapy, occupational therapy, psychological, cultural and
priority intervention? spiritual counseling if needed. By the time the client is ready for rehabilitation, the concerns
for shock, electrolyte, and fluid imbalances should be lessened.

1.‐ Assessment for shock


2.‐ Monitoring electrolytes
3.‐ Beginning occupational and physical therapy
4.‐ Administration of intravenous fluids

962 A client who presents with a burn to the anterior Correct answer: 4 Remember that according to the Rule of Nines, the anterior chest is 18 percent and both This question requires knowledge of the rule of nines.
chest and both arms anterior and posterior is said to arms are 9 percent each, totaling 36 percent.
have burned what percentage of the body using the
Rule of Nines?
1.‐ 27 percent
2.‐ 45 percent
3.‐ 37 percent
4.‐ 36 percent

963 Client teaching concerning the causes of contact Correct answer: 3 Contact dermatitis is a type of dermatitis caused by a hypersensitivity response or chemical Note the key word ‘contact’ in the question stem.
dermatitis would include which of the following? irritation.
1.‐ Heat
2.‐ Poor hygiene
3.‐ External irritants
4.‐ An infection

964 A mother brings her three children to the clinic with a Correct answer: 1 Tinea corporals are a fungal infection of the body called ringworm. The most common lesions This question requires knowledge of tinea.
rash on each of their faces. On assessment, the nurse are large, circular patches with raised, red borders of vesicles, papules, or pustules.
notes the areas are circular patches with raised red
borders. The nurse concludes that this is compatible
with which of the following problems?

1.‐ Tinea corporals


2.‐ Tinea curries
3.‐ Scabies
4.‐ Impetigo

965 Discharge instructions for the client with cellulitis Correct answer: 4 Signs of infection may include fever, chills, erythema, tenderness, drainage, and malaise. A Remember the importance of assessment as a key intervention.
should include which of the following? healthcare provider should be notified.
1.‐ To add extra dressings if the site begins to drain
2.‐ To stop the antibiotic as soon as the area appears healed
3.‐ To squeeze pustules if they develop to prevent infection
4.‐ Monitor the site for redness, tenderness, or drainage

966 The client has been prescribed to use tar shampoo for Correct answer: 3 Coal tar shampoo is a treatment option for psoriasis of the scalp. Folliculitis and cellulitis are This question requires knowledge of the treatment of psoriasis.
a scalp infection. The nurse tells the client to return to bacterial infections requiring antibiotics. Pediculosis is head lice, which requires a shampoo
the ambulatory clinic if this treatment is not effective such as Nix or Kwell.
against the condition, which is probably caused by:

1.‐ Pediculosis.
2.‐ Folliculitis.
3.‐ Psoriasis.
4.‐ Cellulitis.

967 Which of the following nursing diagnoses is of highest Correct answer: 2 Clients with herpes zoster have impaired skin integrity and pruritis with scratching along with Note that in this question safety is a priority.
priority for a client with herpes zoster? possible excoriation, which causes a high risk for secondary bacterial infection. Altered
comfort would also be a diagnosis but is not one of the options.
1.‐ Ineffective coping
2.‐ Risk for infection
3.‐ Social isolation
4.‐ Self‐care deficit

968 When counseling clients regarding prevention of Correct answer: 3 Most burns that occur at home are caused by hot water or steam. Lowering the temperature This question requires knowledge of the common causes of burns.
common burns, the nurse should stress which of the setting of the hot water heater is a first‐line prevention measure, especially with children
following? present.
1.‐ Stop smoking
2.‐ Smoke detectors in the home
3.‐ Temperature setting of the water heater
4.‐ First aid measures

969 The client, a healthcare provider herself, presents Correct answer: 3 A contact dermatitis common to healthcare providers is latex glove allergies. All options could This question refers to the risk of the occupation.
with a rash to her hands. Assessment of which of the be correct but the addition in the stem of the question about the client's field of work should
following would be most helpful in differentiating the direct the learner to the correct option. Other common causes of contact dermatitis include
cause of the rash? chemicals, soaps and detergents, but most agencies use milder forms.

1.‐ Use of any new soaps, detergents, or chemicals


2.‐ History of skin rashes
3.‐ Type of gloves used and frequency
4.‐ Access to pets at home

970 In addition to impaired skin integrity, which of the Correct answer: 3 By altering the skin integument, all three diagnoses place the client at risk for infection by This question requires knowledge that the skin is the first line of defense.
following would be a priority nursing diagnosis for Staphylococcus, which is normally found on the skin. Psoriasis can also be triggered by a
clients with disorders such as burns, psoriasis, and respiratory infection, particularly pharyngitis caused by Streptococcus. Pain is usually not a
shingles? concern in psoriasis and will depend on the stage of a burn. Options 2 and 3 could be correct,
but notice the stem asked for the priority diagnosis.

1.‐ Pain
2.‐ Ineffective individual coping
3.‐ Risk for infection
4.‐ Knowledge deficit

971 During a teaching session about acne, a client asks Correct answer: 2 Dietary restrictions have not been found to be clinically relevant in the severity or cause of This question requires knowledge that diet is not associated with acne.
which foods should be avoided. The nurse's best reply acne. Stress should be placed however on healthy food preferences.
is:
1.‐ "Greasy, fried foods should be avoided."
2.‐ "All foods are acceptable."
3.‐ "Snack foods contribute tremendously to acne problems."
4.‐ "Dietary restrictions can help somewhat but shouldn't be the priority in treatment."

972 The nurse notes that a client has an elevated lesion Correct answer: 3 A bulla measures &gt; 0.5 cm. A papule is solid; a vesicle measures &amp;lt; 0.5 cm; and a This question requires knowledge of the definition of the terms in the answer choices.
that contains clear fluid and measures &gt;1 cm in pustule contains purulent exudates.
diameter. This finding is best documented by the nurse
as which of the following?

1.‐ Papule
2.‐ Vesicle
3.‐ Bulla
4.‐ Pustule

973 The nurse alerts the primary care practitioner about a Correct answer: 1 The mole in option 1 meets the criteria of the "ABCD" rule: the size has increased in diameter The key to this question is rapid growth.
mole. Which of these characteristics indicates the over two months, the mole has two colors, the center is black, and the border is irregular.
need for intervention?
1.‐ A dark brown, irregular‐bordered mole with a black‐appearing center that has grown approximately 2 cm in the past 2 months.
2.‐ A 2 cm waxy papule that has a "stuck on" appearance.
3.‐ A small, flat mole that has been on the client's back since birth.
4.‐ Multiple small, flat, nonpruritic moles that have been on the client's elbow for 10 years.
974 A client with herpes simplex virus 1 makes the remark Correct answer: 2 Herpes simplex virus 1 may reappear in times of reactivation. The infection is described as a This question requires knowledge of herpes simplex virus.
that she hopes she never gets another lesion on her lip vesicular lesion that occurs on the oral mucosa (lips, mouth), making option 3 incorrect.
"like this one." What is the nurse's best response? Option 4 is false because herpes lesions are painful and because the description is incorrect.

1.‐ "The chances of getting another lesion on your lip are very low."
2.‐ "Herpes simplex virus 1 can be reactivated at any time."
3.‐ "The lesions will also appear on other parts of the body."
4.‐ "A red, pinpoint painless rash will continue to develop for the next two weeks."

975 The nurse would include which of the following Correct answer: 3 The client with herpes zoster may experience impaired skin integrity and pruritis in which the This question stresses the importance of assessing for further infection.
priority interventions in the plan of care for a client client may frequently scratch the lesions, contributing to a secondary bacterial infection. Cool
diagnosed with herpes zoster? environments should be maintained because heat and scratching will make the pruritis worse
(option 4). Options 1 and 2 are irrelevant to the client's case.

1.‐ Monitor daily dietary intake and daily weights.


2.‐ Encourage daily activity routines to keep the client mobile.
3.‐ Monitor skin integrity for secondary bacterial infections.
4.‐ Maintain a warm environment to decrease the intensity of the pruritus.

976 The client telephones that she wants to come to the Correct answer: 3 The common wart, flat wart, and filiform wart are not painful, whereas the plantar wart is This question requires knowledge that plantar warts are painful.
office for evaluation of a painful wart. The nurse often painful.
anticipates that the client will present with which of
the following cutaneous lesions that is commonly
painful?
1.‐ Common wart
2.‐ Flat wart
3.‐ Plantar wart
4.‐ Filiform wart

977 The client presents with an increase in the number of Correct answer: 1 Vitiligo is a slowly progressive depigmentating condition of the skin caused by disappearance This question requires knowledge of vitiligo.
white patches across his chest and back. Multiple of melanocytes. Eczema is an inflammatory condition in which the skin appears erythemic, dry,
creams and lotions were not helpful. The nurse and thickened. Psoriasis is a chronic inflammatory condition in which lesions appear whitish
concludes that this client's clinical picture is consistent and scaly and commonly appear on the scalp, knees, and elbows. Contact dermatitis is an
with which of the following conditions? eruption of the skin related to contact with an irritating substance or allergen.

1.‐ Vitiligo
2.‐ Eczema
3.‐ Psoriasis
4.‐ Contact dermatitis

978 When counseling clients regarding first‐line burn Correct answer: 1 Most burns occur at home caused by hot water or steam. All other aspects are important to This question requires knowledge of the common causes of burns.
prevention, the nurse should plan to include which of general prevention but temperature setting of the water is a first‐line prevention.
the following items?
1.‐ Discuss the temperature setting of the water heater.
2.‐ Demonstrate the use of a fire extinguisher.
3.‐ Assist the planning of an escape route.
4.‐ Stress the need for smoke detectors.

979 When caring for a client with a burn in the emergent Correct answer: 4 In the emergent stage, the nurse assesses the cause and extent of the burn and determines Use the process of elimination to select the one correct answer.
stage, which of the following has lowest priority as first aid measures that were used. Gender is not a factor in burn assessment.
part of an accurate burn assessment?
1.‐ Where it occurred
2.‐ Cause of the burn
3.‐ First‐aid treatment given
4.‐ Gender

980 The client presents with a pruritic rash. Questions Correct answer: 1 The location of the rash helps identify the possible offending antigen. Age, gender, and recent Use the process of elimination to select the one correct answer.
about which of the following would help differentiate travel are less helpful in identifying the etiology of the pruritic lesion.
the cause?
1.‐ Location of the rash
2.‐ Age of the client
3.‐ Gender
4.‐ Recent travel

981 In obtaining a health history on a client with psoriasis, Correct answer: 2 Psoriaris can often be brought on by a respiratory infection, particularly streptococcal This question requires knowledge that strep infections may precipitate psoriasis.
which recent infection is significant? pharyngitis. The other responses are insignificant findings as they relate to psoriasis.

1.‐ Escherichia coli


2.‐ Streptococcus
3.‐ Staphylococcus
4.‐ Pneumonia

982 A client presents with silvery plaques on both elbows Correct answer: 3 Psoriasis is characterized by the presence of silvery plaques, particularly on the extensor Use the process of elimination to select the one correct answer.
that are not itchy, but bleed when the scales are prominences, that bleed when scales are removed. The other disorders listed are not
removed. The nurse concludes that the client most characterized in this way.
likely has which of the following conditions?

1.‐ Eczema
2.‐ Contact dermatitis
3.‐ Psoriasis
4.‐ Poison ivy

983 A client presents to the primary care clinic Correct answer: 1 Even though all these problems may cause itching, a classical symptom of scabies is pruritus This question requires knowledge of scabies.
complaining of frequent scratching and itching of the with worsening at night. The mites tend to have increased movement at night, which accounts
skin that is worse at night. The nurse should suspect for the worsening symptoms at that time.
which of the following skin disorders?
1.‐ Scabies
2.‐ Hives
3.‐ Fleas
4.‐ Drug reaction

984 The nurse determines that which of the following Correct answer: 3 Dietary restrictions were once believed to be necessary to decrease acne, but this has not Use the process of elimination to select the most feasible answer.
reported by a client with acne would not contribute to been clinically relevant or supported in research. Stress and the use of moisturizers and oil‐
the severity of the acne? based cosmetics do seem to affect the severity of the disorder.
1.‐ Stress
2.‐ Oil‐based cosmetics
3.‐ Diet
4.‐ Moisturizers
985 The nurse who is counseling a family about treatment Correct answer: 2 Lindane (Kwell) can cause neurotoxicity in young children and nursing/pregnant women. This This question requires knowledge of the adverse effects of the listed medications.
for scabies would tell the family to avoid which of the is not a concern with the other products listed.
following products, based on knowledge that the
family has young children and the woman is pregnant?

1.‐ Elimite
2.‐ Lindane (Kwell)
3.‐ Permethrin (Nix)
4.‐ Pyrethrin shampoo (Rid)

986 Which of the following would not be included in the Correct answer: 3 Plastic shoes or sandals increase moisture collection in the feet. This should be avoided as it Look for incorrect answers and use the process of elimination to select the one correct
nurse's instructions to a client to prevent a recurrence increases the risk of recurrence. The other options describe helpful measures to prevent answer.
of tinea pedis? recurrence of tinea pedis.
1.‐ Wear rubber sandals in communal showers
2.‐ Wear light cotton socks
3.‐ Wear plastic shoes or sandals
4.‐ Carefully dry feet and toes after bathing

987 Which of the following clients that the nurse is Correct answer: 3 The consistent use of condoms helps protect from spreading herpes virus type 2 to other This question requires knowledge that herpes is spread via sexual contact.
scheduled to see this morning at the ambulatory care partners. The other options do not represent circumstances that provide any protection
clinic is the least likely to be infected with herpes virus against sexually transmitted diseases.
type 2?
1.‐ A smoker
2.‐ A client who has multiple sexual partners
3.‐ A client who consistently uses condoms during intercourse
4.‐ A 16‐year‐old client who uses a diaphragm

988 The client asks the nurse if a lesion on her hand could Correct answer: 1 A wart is described as being a round, raised, firm lesion of the skin that may have ragged This question requires knowledge of the appearance of warts.
be a wart. The nurse would examine the area to borders. Warts do not contain fluid and generally have the color of normal flesh (options 3 and
determine the presence of which of the following 4). Generally only plantar warts on the feet are associated with pain.
characteristics?
1.‐ Firm, skin‐colored papule
2.‐ Vesicle that is painful
3.‐ Oval erythemic lesion
4.‐ White round vesicle

989 The nurse would plan to include which of the Correct answer: 1 To improve folliculitis, the use of antibacterial soap daily along with good hand washing will Remember that good handwashing is often a feasible answer.
following in the care of a client diagnosed with control and prevent spread of the infection. Isolation is not needed. The site should also be
folliculitis? allowed to air dry and should not be covered with a bandage.
1.‐ Warm compresses, good hand washing, and use of antibacterial soap
2.‐ Strict isolation of the site with sterile dressing changes four times a day
3.‐ Skin care is no different than routine skin care
4.‐ Keep skin site covered with clean bandage for 1 week

990 The nurse should plan to include which of the Correct answer: 2 Good hygiene is recommended to help to prevent spreading the infection to other family Beware of answer choices containing absolutes (never, only, etc.).
following statements in client teaching for impetigo? members. Option 1 is false. Impetigo is contagious and antibiotic therapy is the recommended
treatment (options 3 and 4).
1.‐ Poor hygiene is the only reason impetigo occurs.
2.‐ Good hand washing with antibacterial soap helps reduce spreading the infection to others.
3.‐ Impetigo is not contagious and cannot spread to other members of the family.
4.‐ Antibacterial medication is not useful in impetigo.
991 The nurse conducting health promotion about Correct answer: 4 The epidermis protects the tissues from damage and prevents fluid loss of the body. The This question requires knowledge of the function of skin.
maintaining healthy skin in the community teaches dermis regulates body temperature (options 2 and 3). Option 1 is false.
clients that the epidermis has which of the following
functions?
1.‐ To protect foreign objects from entering the eyes
2.‐ To regulate body heat by excretion of perspiration
3.‐ To produce androgens and regulate temperature
4.‐ To protect the tissues from physical damage and prevent water loss

992 A client visits the primary care clinic because of a Correct answer: 1 The fastest and most cost‐effective method to diagnose a fungal infection is using KOH This question requires knowledge of the medications for fungal infections.
rash. The nurse suspects the rash to be a fungal preparation to reveal more clearly the spores and hyphae of each fungus.
infection. The nurse anticipates using which of the
following tests as the most appropriate?
1.‐ KOH (potassium hydroxide)
2.‐ Culture and sensitivity
3.‐ Mineral oil scrape
4.‐ CBC

993 The nurse would document the presence of which of Correct answer: 2 A macule is a nonpalpable flat lesion. All the other lesions listed are elevated. If needed, refer This question requires knowledge of these types of lesions.
the following in the medical record after noting a back to Table 10‐1 (Primary Skin Lesions) in the textbook, for detailed descriptions.
lesion that is not raised?
1.‐ Papule
2.‐ Macule
3.‐ Pustule
4.‐ Vesicle

994 The nurse noting the presence of "satellite" lesions Correct answer: 1 Satellite lesions are maculopapular areas outside an area of original infection and are This question requires knowledge of the listed skin disorders.
would suspect that the client has which of the characteristic of candidiasis. Satellite lesions are not characteristic of the other conditions
following skin disorders? listed.
1.‐ Candidiasis
2.‐ Eczema
3.‐ Acne
4.‐ Psoriasis

995 The nurse should give which of the following nursing Correct answer: 1 Herpes zoster presents with vesicular lesions that could become infected if the skin is not Recall that safety is a high priority according to Maslow.
diagnoses highest priority for an elderly client monitored carefully. This is the priority diagnosis for the elderly client. There could be a
diagnosed with herpes zoster? possibility of injury and ineffective coping, however these are not the priority diagnoses. Risk
for fluid volume deficit is irrelevant to the situation described.

1.‐ Risk for infection


2.‐ Risk for injury
3.‐ Risk for ineffective coping
4.‐ Risk for fluid volume deficit

996 Which of the following would not be included in the Correct answer: 2 A systematic skin inspection should be done at least once a day with particular attention to This question calls for determination of the frequency of assessment.
care plan for a client with a pressure ulcer? the bony prominences. Weekly skin inspections are too infrequent to meet the needs of the
client and evaluate how the ulcer is healing. The other interventions listed are appropriate to
the care of a client with a pressure ulcer.
1.‐ Monitor nutritional intake and weight
2.‐ Perform weekly skin inspections
3.‐ Use pressure‐reduction aids
4.‐ Change client position at least every 2 hours

997 Which of the following rehabilitation measures would Correct answer: 2 Rehabilitation measures focus on the prevention of contractures and scars. The client is Note the word ‘rehabilitation’ in the question stem.
be done upon discharge of a client with partial‐ taught to continue ROM exercises to enhance mobility and to support the injured joints.
thickness burns? Options 1, 3, and 4 should have been completed prior to the rehabilitative phase of burn
management.
1.‐ Psychosocial evaluation
2.‐ Continuing range of motion (ROM) exercises
3.‐ Ongoing pain assessment and management
4.‐ Burn wound management

998 A client visits the primary care center because of Correct answer: 2 First‐line therapy consists of using topical retinoids or benzoyl peroxide. Antibiotics and This question calls for the least risky medications to be used first.
acne. In anticipating the pharmacologic management accutane are used for moderate and severe cases. Corticosteroids are not to be used on the
of this client, the nurse should anticipate which of the face because of absorption of the medication.
following as a first line medication therapy?

1.‐ Corticosteroids
2.‐ Topical retinoids
3.‐ Antibiotics
4.‐ Accutane

999 Which of the following lesions if found on a client Correct answer: 3 All other lesions are benign, but lentigo melana (pre‐melanoma) may develop into a true This question requires knowledge of the listed lesions.
poses concern for the nurse because of the possibility melanoma over time.
of progression to a melanoma over time?

1.‐ Senile lentigo


2.‐ Freckle
3.‐ Lentigo melana
4.‐ Macule

1000 Client education for repeated bee stings would not Correct answer: 2 Reexposure to a bee sting may precipitate a more severe reaction and require emergency Note that this question requires use of the process of elimination to find the wrong
include which of the following? care. It would be inappropriate to give the client false information that reactions will decrease. answer.
Options 1, 3, and 4 should be included in client teaching.
1.‐ First‐aid measures
2.‐ Assurance of decreased reactions with repeated exposure
3.‐ Use of insect repellants
4.‐ Method for removal of stinger

1001 Which electrolyte abnormalities would the nurse Correct answer: 1 Sodium levels decrease and potassium levels increase secondary to massive fluid shifts into Associate high potassium levels with cell destruction and make sure both items in the
expect to occur while working with a client who just the interstitium and release of potassium from cells that are destroyed. The other responses option are correct.
sustained partial‐ and full‐thickness burns? are incorrect.

1.‐ Decreased sodium and increased potassium


2.‐ Increased calcium and decreased potassium
3.‐ Decreased magnesium and increased sodium
4.‐ Increased sodium and decreased potassium
1002 The nurse provides teaching to a client after the Correct answer: 3 Dead skin and exudates often collect under the cast, and efforts to remove it should be done The core issue of the question is the knowledge of skin care following cast removal. Use
removal of a short leg cast. The nurse should include gradually. The client should be instructed to avoid any vigorous scrubbing of the skin to avoid nursing knowledge and the process of elimination to make a selection.
which of the following in discussions with the client? breaks, which increase the risk for infection. The use of undiluted peroxide is too harsh for the
skin. There is no reason why the leg cannot be touched after removal of the cast.

1.‐ Wash the skin with undiluted hydrogen peroxide.


2.‐ Vigorously scrub the legs to remove dead skin.
3.‐ Gently wash and lubricate the leg.
4.‐ Avoid touching the leg for 2 weeks.

1003 Which of the following nursing diagnoses would be Correct answer: 3 Impaired physical mobility is the appropriate priority nursing diagnosis for a client with The core issue of the question is the knowledge of priorities for the client with Paget’s
the priority for a client with Paget’s disease? Paget’s disease. The client needs to remain active to decrease the complications associated disease. Use nursing knowledge and the process of elimination to make a selection.
with immobility and to maintain the ability to perform self‐care activities. The other diagnoses,
although appropriate, are not the priority in clients with Paget’s disease.

1.‐ Risk for noncompliance


2.‐ Disturbed sleep pattern
3.‐ Impaired physical mobility
4.‐ Disturbed body image

1004 A client with a right arm cast for fractured humerus Correct answer: 1 This symptom suggests neurological injury caused by pressure on nerves and soft tissue The core issue of the question is the knowledge of priority assessments in a client with
states, “I haven’t been able to extend the fingers on because of swelling. Other symptoms of neurovascular compromise should be assessed and possible compartment syndrome. Use nursing knowledge and the process of elimination to
my right hand since this morning.” What action should reported to the physician. make a selection.
the nurse take next?
1.‐ Assess neurovascular status.
2.‐ Ask the client to massage the fingers.
3.‐ Encourage the client to take the prescribed analgesics as ordered.
4.‐ Elevate the right arm on a pillow to reduce edema.

1005 A client with an open fracture is at risk for developing Correct answer: 2 Elevated temperature is a classic symptom seen with this osteomyelitis as a systemic The core issue of the question is the knowledge of manifestations of osteomyelitis. Use
osteomyelitis. Which of the following classic symptoms response to the invading organism. Pain, swelling, and tenderness may also accompany the nursing knowledge and the process of elimination to make a selection.
would the nurse assess for to detect development of fever. Acute respiratory distress (option 3) is more suggestive of embolism but not infection.
this complication? The extremity does not shorten.

1.‐ Low bone density


2.‐ Elevated temperature
3.‐ Acute respiratory distress
4.‐ Shortening of the affected extremity

1006 An obese client with degenerative joint disease is Correct answer: 1 Aspirin therapy for this condition is continuous and is effective only after a therapeutic level is The core issue of the question is the knowledge of appropriate self‐management
being managed pharmacologically with aspirin therapy. reached. It should not be taken intermittently (option 1). The other options are correct techniques for degenerative joint disease. Use nursing knowledge and the process of
The nurse knows that additional client teaching is statements about self‐care measures when taking aspirin for degenerative joint disease. elimination to make a selection.
necessary when the client makes which of the
following statements?
1.‐ “I take my aspirin only when I have extreme pain and stiffness.”
2.‐ “I use heat sometimes to help decrease my pain and joint stiffness.”
3.‐ “I frequently examine my stools for bleeding.”
4.‐ “I started an exercise program to lose weight.”
1007 A client underwent a lumbar laminectomy today. Correct answer: 4 Immediately after surgery, the client will be inclined not to move because of pain and fear of The core issue of the question is the knowledge of priority nursing diagnoses following
Which nursing diagnosis has highest priority for this disturbing the operative site. Minimal scarring results from this surgery, so body image musculoskeletal surgery. Use nursing knowledge and the process of elimination to make a
client? disturbance is not likely to be appropriate (option 1). The psychosocial diagnoses in options 2 selection.
and 3 have less priority than option 4 because option 4 is a physiological concern.

1.‐ Disturbed body image disturbance


2.‐ Social isolation
3.‐ Ineffective role performance
4.‐ Impaired physical mobility

1008 A client had a left above‐the‐knee amputation today. Correct answer: 1 Elevating the limb on a pillow facilitates venous return, decreases swelling, and promotes The core issue of the question is the knowledge of postoperative stump care and
For the first 24 hours postoperatively, the nurse makes comfort. The stump dressing is usually a compression type to mold the stump and to decrease positioning. Use nursing knowledge and the process of elimination to make a selection.
it a priority to do which of the following to properly the edema associated with inflammation, so option 2 is an inappropriate intervention. The
manage the surgical site? other options are also inappropriate because option 3 increases risk of edema and option 4 is
done as ordered.
1.‐ Elevate the residual limb on a pillow.
2.‐ Loosen the stump dressing every 4 hours.
3.‐ Maintain the residual limb in a dependent position.
4.‐ Change dressings as often as needed.

1009 A client with a femoral fracture is in Buck’s traction. Correct answer: 3 Traction, to be effective, must have an opposing force (countertraction). The aim in traction is The core issue of the question is the knowledge of proper use of traction. Use nursing
While making rounds, the nurse notices that the to maintain a constant force to align the distal and proximal ends of a fractured bone. Options knowledge and the process of elimination to make a selection.
client’s foot is flush with the footboard of the bed. 1, 2, and 4 violate this principle of traction in the treatment of fractures. Centering the client in
Based on the nurse’s knowledge of the principles of bed maintains the line of pull and ensures that countertraction is maintained.
traction, an appropriate action is to do which of the
following?
1.‐ Wedge a pillow between the footboard and the client’s foot.
2.‐ Praise the patient for maintaining countertraction.
3.‐ Center the client on the bed.
4.‐ Ask the client to pull up in bed while holding the weights.

1010 A truck driver presents to the primary care provider Correct answer: 4 Prolonged sitting or standing aggravates back injury because of the additional stress placed The core issue of the question is the knowledge of risk factors and aggravating factors of
with complaints of persistent back pain. The nurse on the structures supporting the back. Lifting objects close to the body, shifting positions low back pain. Use nursing knowledge and the process of elimination to make a selection.
explains that which client activity documented during frequently, and providing back support are appropriate actions to maintain good body
the nursing history may contribute to further back mechanics.
injury?
1.‐ Lifting objects close to the body
2.‐ Shifting positions often when sitting for prolonged periods
3.‐ Providing back support with a pillow when sitting
4.‐ Prolonged standing or sitting

1011 A client underwent a lumbar laminectomy. Which of Correct answer: 3 The physician orders the client’s activity after a laminectomy. After a laminectomy procedure, The core issue of the question is the knowledge of activity levels after surgery that will not
the following activities would be best 4 hours a client should be assisted to logroll from side to side. The principle is to maintain the cause harm to the surgical area following laminectomy. Use nursing knowledge and the
postoperatively? alignment of the vertebral column at all times. Clients with lumbar laminectomy should be process of elimination to make a selection.
kept flat or with head of bed slightly elevated to minimize stress on the suture line. Using the
side‐rails to get out of bed causes shifting of the vertebral column. Sitting up in a chair or on
the side of the bed is usually done the evening of the surgery or the first day following surgery,
and it is for brief periods only.

1.‐ Sitting up in a chair to watch television


2.‐ Sitting at the side of the bed
3.‐ Lying in bed in good alignment with the head of bed flat
4.‐ Using the side‐rails for support to get out of bed
1012 The nurse provides teaching to a 50‐year‐old male Correct answer: 1 Smoking has been found to contribute to disc deterioration. Lack of exercise predisposes the The core issue of the question is the knowledge of factors that aggravate low back pain.
Caucasian client with chronic low back pain. The client muscles of the back to strain. The extra weight of obese individuals imposes more strain on the Use nursing knowledge and the process of elimination to make a selection.
weighs 200 pounds, works as a truck driver, sits for back and also interferes in maintaining good body mechanics in lifting. Occupations that
prolonged periods, and seldom participates in exercise require prolonged sitting or standing predispose those individuals to exacerbation of back pain.
activities. The client smokes one pack of cigarettes and Option 1 is the only answer that accurately reflects risk factors associated with chronic low
drinks six cans of beer per day. What risk factors back pain for the client described in the question.
should the nurse include in the discussion?

1.‐ Lack of exercise, obesity, sitting for long periods, smoking, sedentary occupation
2.‐ Degenerative disk disease, gender, race, smoking
3.‐ Degenerative disk disease, race, alcohol use, smoking, inactivity
4.‐ Age, obesity, lack of exercise, genetic factors

1013 The nurse is teaching a postmenopausal client about Correct answer: 2 A combination of calcium and Vitamin D is recommended for the prevention of osteoporosis. The core issue of the question is the knowledge of risk factors for and prevention of
the use of calcium to prevent the effects of Vitamin D increases the intestinal absorption of calcium and mobilizes calcium and phosphorus osteoporosis. Use nursing knowledge and the process of elimination to make a selection.
osteoporosis. The client asks: “Why do I have to take into the bone. Vitamin D alone does not prevent osteoporosis (option 2). Whereas some
Vitamin D with my calcium?” Which of the following is elderly might be deficient in Vitamin D, a postmenopausal state does not necessarily cause the
the nurse’s best response? deficiency (option 3). There are other interventions for the prevention of osteoporosis,
including lifestyle modifications (e.g., smoking cessation), which makes option 4 inaccurate.

1.‐ “Vitamin D prevents osteoporosis.”


2.‐ “Vitamin D increases intestinal absorption of calcium.”
3.‐ “You are most likely to be deficient in Vitamin D.”
4.‐ “Calcium and Vitamin D supplementation is the only way to prevent osteoporosis.”

1014 The nurse is caring for a client with a week‐old cast. Correct answer: 3 A complication of cast application is skin breakdown underneath the cast. If this occurs, The core issue of the question is the knowledge of various complications of casts. Use
The client asks why the nurse palpates the casted area infection can set in and can cause the area over the breakdown to be warmer than other nursing knowledge and the process of elimination to make a selection.
when doing the assessment. Which of the following is areas. A bad odor coming from the area may also be noted. Option 1 is inaccurate because
the most appropriate response by the nurse? generally plaster casts dry in 48 hours or less and fiberglass casts in 30 minutes to 1 hour. If a
cast is too tight, symptoms associated with neurovascular compromise will be noted, which
include pain, paresthesia, pallor, diminished pulse distal to the cast, and paralysis (option 4).

1.‐ “I am making sure that the cast has dried.”


2.‐ “I am evaluating the strength of the cast.”
3.‐ “I am feeling for hot spots that might indicate infection.”
4.‐ “I am making sure that the cast is not too tight.”

1015 A client is placed on continuous passive motion (CPM) Correct answer: 1 The client’s knee will externally rotate if there is insufficient space between the client’s hip The core issue of the question is the knowledge of appropriate assessment and care of the
machine postoperatively after a total knee and the machine. The knee should be upright, facing the ceiling, as the machine moves the leg client in CPM. Use nursing knowledge and the process of elimination to make a selection.
replacement. The nurse observes that the client’s knee back and forth.
is externally rotating during flexion. What should the
nurse do next?
1.‐ Move the client up in bed or move the CPM machine down toward the foot of the bed
2.‐ Support the client’s knee with sandbags to prevent external rotation
3.‐ Assist the client to sit up in bed in a 45‐degree position
4.‐ Do nothing; the client’s knee is properly aligned
1016 A client in skeletal traction for a right femur fracture Correct answer: 1 Pain and absent pulse in the affected extremity are urgent signs requiring immediate The core issue of the question is the knowledge of adverse neurovascular changes to a
is complaining of pain in the affected limb. The nurse intervention. Impairment of circulation in the affected limb initiates various pathophysiologic client in a cast. Recall principles of gravity and blood flow to aid in answering the question.
assesses that the right foot is pale without a pulse. processes, including destruction of nerves and tissues. If this state is uninterrupted, loss of the Use nursing knowledge and the process of elimination to make a selection.
What should the nurse do next? limb may occur. The nurse needs to ensure that the leg is not above heart level so no further
damage occurs. The physician needs to be notified immediately so medical interventions can
be instituted before irreversible tissue and nerve damage occurs.

1.‐ Ensure that the leg is not raised above heart level
2.‐ Administer analgesics as ordered
3.‐ Release the traction
4.‐ Document the observation and recheck the pulse in 5 minutes

1017 A nurse receives a client from the emergency Correct answer: 1 It is essential to monitor the condition of the skin under traction, as well as bony The critical word in the question is priority, which tells you that all or more than one
department in Buck’s traction following a fracture of prominences, because these areas are at risk for breakdown due to continuous friction and options are correct and that the most essential one is the correct answer. Use nursing
the right femur. The nurse documents which of the pressure from the skin traction device. Option 2 is incorrect because Buck’s traction is a type of knowledge about skin traction and the process of elimination to make a selection.
following as a priority in the client medical record? skintraction. Skeletal tractions use pins, wires, or tongs to aid in realignment. Option 3 is
appropriate, but the most essential assessment to be documented for a client with skin
traction is the condition of the skin underneath the straps.

1.‐ Status of skin underneath the traction and over bony prominences
2.‐ Type of pin, wire, or tongs used
3.‐ The effectiveness of pain medication given in the field
4.‐ Medications given in the emergency department

1018 The nurse planning for the care of a client admitted Correct answer: 2 Balanced suspension allows for ease with bedpan use and skin care without disturbing the The core issue of the question is the knowledge of Buck’s traction as a skin traction and
with balanced suspension traction explains to the line of traction. In this type of traction, the client’s injured extremity is lifted off the bed and a the need to assess the underlying skin. Use nursing knowledge and the process of
family that an advantage of balanced suspension is straight pull is accomplished by the application of several forces and several weights. Skin elimination to make a selection.
which of the following? breakdown is not eliminated with this type of traction because any immobile client can be at
risk.
1.‐ It eliminates the risk for skin breakdown.
2.‐ It allows the client to raise the buttocks off the bed for bedpan use and skin care.
3.‐ It is more effective in reducing hip contracture.
4.‐ It requires only one weight to maintain traction.

1019 A client is taking colchicine for gout. The client Correct answer: 2 Colchicine is used in treating the acute attack of gout. The symptoms described are signs of The core issue of the question is the knowledge of actions and adverse effects of
complains of weakness, abdominal pain, nausea, toxicity. The client should be instructed to stop the medication and be seen for follow‐up colchicines in the client with gout. Use nursing knowledge and the process of elimination to
vomiting, and diarrhea for the past 2 days. The nurse treatment. The expected effect of colchicine is to diminish the joint pain associated with the make a selection.
interprets these complaints indicating which of the acute attack.
following?
1.‐ Therapeutic effects of the medication
2.‐ Signs of toxicity
3.‐ Expected side effects
4.‐ An allergic response

1020 An 87‐year‐old client sustained a right hip fracture. Correct answer: 2 Age, site of the fracture, and blood supply to the affected area all affect the rate of bone The core issue of the question is knowledge of possible threats to bone healing in an
The client asks the nurse about the length of time healing. Younger and healthy individuals prior to the injury will have faster bone healing than identified client. Use nursing knowledge and the process of elimination to make a selection.
needed for the fracture to heal. The nurse’s response the elderly and those with chronic illnesses. Although physical therapy will assist in mobility, it
includes consideration of which client factor that does not directly enhance bone healing. The weight of the client, unless accompanied by
influences the rate of bone healing? malnutrition, does not have a direct bearing on bone healing.

1.‐ Frequency of physical therapy


2.‐ Age of the client
3.‐ Weight of the client
4.‐ Early ambulation

1021 A client is scheduled to have a closed reduction of a Correct answer: 3 In a closed reduction procedure, the physician applies traction and manipulates the bone until The core issue of the question is the knowledge of various approaches to correct bone
right ankle fracture. The nurse determines the client the broken ends are realigned. Open reduction is a realignment of bone with surgery (option fracture. Use nursing knowledge and the process of elimination to make a selection.
understands the procedure when the client states that 2), and internal fixation devices are surgically inserted during an open reduction to immobilize
the procedure involves which of the following? the fracture during the healing process (option 4). Endoscopy (option 1) is not a surgical
modality for reducing fractures.
1.‐ Applying an endoscopic procedure to realign the bones
2.‐ Realigning the bone using surgery
3.‐ Correcting the bone alignment using manual manipulation
4.‐ Inserting pins, rods, or other implantable devices

1022 A child is admitted to the hospital with a diagnosis of Correct answer: 1 The history of a child with osteomyelitis may include a recent upper respiratory infection The core issue of the question is the knowledge of risk factors for osteomyelitis. Use
osteomyelitis. Which of the following would the nurse (which may include an ear infection or sinus infection), skin infection, or blunt trauma to a nursing knowledge and the process of elimination to make a selection.
likely find when gathering the nursing history? bone. Gastroenteritis would not be found in the recent history of this child that would lead to
this illness. LCPD and CHD do not lead to osteomyelitis.
1.‐ History of an upper respiratory infection
2.‐ History of gastroenteritis
3.‐ History of Legg‐Calve‐Perthes disease
4.‐ History of congenital hip dysplasia

1023 Two hours after a child had a cast applied for a Correct answer: 3 A very swollen hand despite application of ice and elevation is a grave concern, especially The core issue of the question is recognition of a complication, compartment syndrome,
fractured radius, the nursing assessment reveals with the child complaining of numbness. Such swelling can lead to compartment syndrome, that can lead to neurological damage. The correct answer is the one that provides for
swelling in the hand, which is elevated higher than the which can lead to neurological damage. This is a medical emergency, and the physician should definitive treatment of the problem, which in this case is in the practice realm of the
heart. Ice has been applied continuously. The child be called immediately. The nurse can then provide diversional activities while waiting for physician.
does not complain of pain but does complain of definitive orders.
numbness and tingling. Which should the nurse do
first?
1.‐ Medicate for pain.
2.‐ Elevate the injured extremity even higher.
3.‐ Call the physician.
4.‐ Provide the child with diversional activities.

1024 The pediatric nurse interprets that which of the Correct answer: 3 The infant who is carried with the hips abducted is at decreased risk for developing The core issue of the question is recognition of which situation allows the infant to keep
following infants is the least likely to be diagnosed with developmental dysplasia of the hip. Options 1, 2, and 4 are all factors that would possibly the hips abducted. Evaluate each option according to this criteria to make a selection.
developmental dysplasia of the hip? increase the incidence of this defect.

1.‐ The infant with a family history of developmental dysplasia of the hip
2.‐ The infant who weighs over 10 pounds
3.‐ The infant carried on the mother’s hips
4.‐ The infant who had breech position while in the uterus

1025 Which of the following interventions would be Correct answer: 3 Pain must be managed properly in the child after spinal fusion in order for the client to The core issue of the question is the ability to prioritize nursing activities. While the ABCs
essential for the nurse to implement to promote a participate in respiratory exercises. Logrolling and repositioning, as well as coughing, deep‐ are quite important, the client cannot meet goals for the respiratory portion of ABCs unless
stable respiratory status in the adolescent who breathing, and use of incentive spirometry should be done every 2 hours around the clock with pain relief is achieved. With this in mind, choose option 3 as the correct answer.
recently had a spinal fusion for scoliosis? this postoperative client. Providing adequate pain relief will enable the client to carry out
these important activities.
1.‐ Logrolling and repositioning every 4 hours
2.‐ Coughing and deep‐breathing every 2 hours during the day
3.‐ Assessing pain status and ensuring adequate pain relief
4.‐ Encouraging use of incentive spirometry every 4 hours while awake
1026 An 8‐year‐old child presents to the emergency Correct answer: 4 The symptoms described are symptoms of osteomyelitis. This disease can result from a The issue of the question is the ability of the nurse to analyze assessment data and
department with complaints of his ankle hurting and penetrating wound, but it also may result from an infection elsewhere in the body that compare it to typical data of childhood musculoskeletal problems. Note that the
difficulty walking. The triage nurse notes the following traveled to the bone. Osteomyelitis may follow an upper respiratory infection, which is temperature is elevated to help choose the option related to infection.
assessments: pain, redness, and swelling of the ankle. common in school‐aged children.
The ankle has decreased mobility and range of motion.
The child has a temperature of 100.8 degrees F and a
heart rate of 140 beats per minute. The child does not
recollect any injury to the ankle. Which of the
following diagnoses would the triage nurse suspect?

1.‐ Legg‐Calve‐Perthes disease


2.‐ Slipped capital femoral epiphysis
3.‐ Fracture of the ankle
4.‐ Osteomyelitis

1027 The nurse is preparing to help a client get up from a Correct answer: 4, 1, 3, The proper procedure is as follows:<BR /> Visualize the procedure and think about principles of joint support and balance to
chair using crutches. Place in order the steps that the 2, 6, 5 complete the ordered steps.
nurse outlines to the client to do this procedure
correctly. Click and drag the options below to move
them up or down.
1.‐ Place unaffected leg slightly under or at the edge of the chair.
2.‐ Grasp the arm of the chair using the hand on the unaffected side.
3.‐ Grasp the crutches by the horizontal hand bars using the hand on the affected side.
4.‐ Move forward to the edge of the chair.
5.‐ Assume a tripod position.
6.‐ Push down on the crutches and the chair armrest while raising the body out of the chair.

1028 The mother of a newborn is upset that her baby has Correct answer: 1, 5 The exact cause of clubfoot is unknown, though several possible etiologies exist. Abnormal Knowledge of the etiology of clubfoot will help to determine the correct response.
congenital clubfoot. She asks the nurse what she did to intrauterine position may cause the deformity, along with neuromuscular or vascular Consider which response, in addition to being accurate, would be most comforting for the
cause her baby's deformity. Which of the following problems. A positive family history increases the chance of this deformity. mother.
answers is the most appropriate? Select all that apply.

1.‐ Abnormal uterine positioning could have caused this deformity


2.‐ A lack of good nutrition during pregnancy could have caused this defect
3.‐ Having the baby before the due date could have caused this problem
4.‐ There are no known etiologies of this defect
5.‐ Neuromuscular and vascular problems may have caused the problem

1029 Which of the following are appropriate nursing Correct answer: 1, 2, 4 The postoperative care of the child undergoing repair of clubfoot includes elevation, Knowledge of the postsurgical care of the infant with clubfoot will aid in choosing the
interventions to include in the initial postoperative application of ice, assessment for neurovascular status, bleeding and swelling, and pain. correct answer.
care of a child who has had surgery for clubfoot? Nasogastric intubation is usually not needed and warm blankets are not indicated.
Select all that apply.
1.‐ Apply ice bags to the foot; keep the ankle and foot elevated on a pillow
2.‐ Check for drainage or bleeding; observe for swelling around the cast edges
3.‐ Administer pain medications routinely and maintain nasogastric intubation
4.‐ Perform neurovascular status checks every 2 hours and provide diversional activities
5.‐ Cover the surgical extremity with warm blankets
1030 Which of the following does the nurse expect to find Correct answer: 3 All symptoms listed are clinical manifestations of developmental dysplasia of the hip, Knowledge of the assessment findings for a child with DDH will aid in choosing the correct
during assessment of a 5‐year‐old client who has although the only one that would be found in a 5‐year‐old would be the telescoping of the answer.
developmental dysplasia of the hip (DDH)? femoral head into the pelvis. Other clinical signs in an older child would be lordosis, and a
waddling gait with a marked limp. A positive Ortolani‐Barlow maneuver is found in the infant
younger than 2 to 3 months of age. Limited abduction is the sign most often used for an infant
older than three months, along with asymmetry of thigh and gluteal folds.

1.‐ Asymmetry of gluteal and thigh fat folds


2.‐ Positive Ortolani‐Barlow maneuver
3.‐ Telescoping of the femoral head into the pelvis
4.‐ Limited abduction of the affected hip

1031 A 14‐year‐old boy is diagnosed with slipped capital Correct answer: 2 The exact cause of SCFE is unknown. Predisposing factors include obesity, a growth spurt Knowledge of the possible etiology of SCFE will aid in choosing the correct answer.
femoral epiphysis (SCFE). He asks the nurse what resulting in a tall and thin stature, and endocrine disorders such as hypothyroidism and
caused this condition. Which of the following best hypogonadism. There may be a genetic predisposition to this disorder.
answers his question?
1.‐ SCFE is a result of an injury to the hip
2.‐ SCFE may be caused by an endocrine disorder
3.‐ SCFE may be caused by an abnormality of the muscles
4.‐ SCFE is caused by abnormal intrauterine position

1032 An adolescent is wearing a cast following a spinal Correct answer: 1, 2, 3 All are appropriate interventions for the child who has undergone a spinal fusion, although Knowledge of the interventions aimed at helping the adolescent in a cast with body image
fusion for scoliosis. The nurse would include which of only the first three are appropriate interventions directly aimed at the client experiencing an difficulties will help to choose the correct answers.
the following interventions to address the nursing altered body image. Teaching cast care is important, but would be appropriate under the
diagnosis of disturbed body image related to wearing a nursing diagnosis of knowledge deficit. The nurse would assist with coping, but this does not
cast after spinal fusion? Select all that apply. necessarily involve new hairstyle or clothes.

1.‐ Encourage independence in daily activities


2.‐ Encourage the adolescent to participate in community activities
3.‐ Provide contact with a peer who has undergone the same treatment
4.‐ Teach cast care as appropriate
5.‐ Suggest the client change hairstyle or buy new clothes as a coping mechanism

1033 Which of the following instructions would be Correct answer: 2 Activity restrictions should be followed for six to eight months following a spinal fusion. Lying, Knowledge of the postoperative care of the child with spinal fusion and the relevant
appropriate for the nurse to include in the discharge standing, sitting, walking, normal stair climbing, and gentle swimming are generally allowed discharge instructions will aid in choosing the correct answer.
teaching of an adolescent following a spinal fusion? following spinal fusion. Bending and twisting at the waist is not recommended, along with
lifting more than 10 pounds, household chores such as vacuuming, mowing the lawn, physical
education classes, and any sports besides walking.

1.‐ No sitting for longer than 15 minutes


2.‐ The adolescent should not bend at the waist
3.‐ Walking is limited to only one half mile per day
4.‐ The adolescent should not climb stairs

1034 A 15‐year‐old adolescent with osteogenesis Correct answer: 3 Exercise such as swimming is allowed for clients with osteogenesis imperfecta and will help Knowledge of the normal developmental needs and activities permitted for the adolescent
imperfecta (OI) who is hospitalized for a fractured improve muscle tone and prevent obesity. Bowling and wheelchair sports would not be with osteogenesis imperfecta will aid in choosing the correct answer.
femur says to the nurse, "I feel so unhealthy. I can't do allowed, as the weight involved with both could cause fractures of the upper extremities.
any activities at school or any sports, and I love
sports." Which of the following responses by the nurse
would provide the most accurate information to this
client?
1.‐ "It must be very hard not to be able to be involved in any sports; it's just too dangerous for you."
2.‐ "Have you thought about bowling? That is a sport that is allowed when you have this condition."
3.‐ "Swimming is an activity that you can participate in and will improve overall muscle tone."
4.‐ "How about trying wheelchair sports? Maybe you would enjoy wheelchair basketball."

1035 Which of the following assessment findings would the Correct answer: 1 A child with DMD would have enlargement of muscles as a result of fatty tissue infiltration. A Knowledge of the assessment findings for the school aged child with muscular dystrophy
nurse expect to find in the school‐age child with school‐aged child with DMD is generally still ambulatory. Muscles at this age are weak, not will aid in choosing the correct answer.
Duchenne's muscular dystrophy? paralyzed. A weak cough reflex would occur as the disease progresses.

1.‐ Enlargement of muscles


2.‐ Bedridden
3.‐ Weak cough reflex
4.‐ Paralysis of lower muscles

1036 The nurse concludes that a child in Bryant’s traction is Correct answer: 4 Bryant's traction is used specifically for children under 3 years of age and weighing less than Knowledge of Bryant’s traction will aid in choosing the correct answer.
in correct position after noting which of the following? 35 pounds who have developmental hip dysplasia or fractured femur. This bilateral traction is
applied to the child's legs, with the hips flexed at 90 degree angle, with knees extended and
buttocks slightly off the bed. The other distracters describe Russell, Buck, and Dunlop traction.

1.‐ The lower leg is suspended in a padded sling


2.‐ Leg is in extended position without hip flexion
3.‐ The arm is kept flexed and is suspended horizontally
4.‐ Hips are flexed at a 90 degree angle, with buttocks off the mattress

1037 A child with Legg‐Calve‐Perthes disease is undergoing Correct answer: 2 To promote healing of the affected hip in LCPD, the femoral head is contained in the hip Knowledge of the care of the child with LCPD will help to choose the correct answer.
non‐surgical treatment. Which of the following would socket until ossification is complete, which may take up to two years or more. This is
indicate to the nurse that the parents understand such accomplished by keeping the hips abducted by continual use of Petrie casting, or Toronto and
treatment? Scottish‐Rite braces. The child should be encouraged to attend school during this time.
Untreated LCPD clients may develop osteoarthritis and hip dysfunction. The other answers are
incorrect statements of fact.
1.‐ "My child will need to wear a brace while he is sleeping."
2.‐ "My child will need to wear a brace for two years or more."
3.‐ "My child will not be able to attend school until healing has occurred."
4.‐ "My child will develop hip dysfunction later in life."

1038 Parents of an unborn infant have just learned that, Correct answer: 1 The initial treatment for clubfoot begins immediately or shortly after birth and consists of Option 3 can be eliminated as it refers to the hip, not the foot. The other options are used
based on ultrasound, their infant has clubfoot. They weekly cast changes and manipulation. Surgery is completed only if nonsurgical intervention of in clubfoot, but the learner must choose the option that would be used immediately after
ask the nurse how clubfoot is treated. Which of the serial casting is not effective. A Denis Browne splint may be used to maintain correction once it birth.
following treatments should the nurse discuss with the is achieved. Abduction devices are used for hip conditions.
parents?
1.‐ Weekly cast changes with manipulation
2.‐ Probable surgery on the affected limb
3.‐ Abduction device to keep the hip in full abduction
4.‐ Use of a Denis Browne splint to achieve correction

1039 An infant is placed in a Pavlik harness for Correct answer: 1, 2, 3 Diapers should be placed underneath the straps of a Pavlik harness; a t‐shirt should be worn Knowledge of the care of the child in a Pavlik harness will aid in choosing the correct
developmental dysplasia of the hip. The nurse has under the straps of the harness. The harness should be worn for 23 hours a day. The child answer. The wording of the question guides you to eliminate responses that are correct
completed parent teaching, but the parents seem to quickly “catches up” once the device is no longer worn if developmental milestones are information.
be overwhelmed by the condition and make several delayed because of the abduction device. Babies should never be lifted by their legs when
statements indicating a lack of understanding. The changing diapers. Early treatment is usually successful without surgery. The treatment is not
statements that indicate more teaching is needed are: painful.
(Select all that apply.)

1.‐ “The straps of the harness should be placed next to the skin.”
2.‐ “The harness should be worn for 6 hours a day.”
3.‐ “It will take a long time for my child to walk and crawl.”
4.‐ “I should not lift the baby by his legs when changing his diaper.”
5.‐ “Because my child’s defect was caught early, treatment will not usually require surgery.”

1040 A 4‐year‐old child with osteogenesis imperfecta (OI) is Correct answer: 3 Because of their very fragile bones, children with OI experience countless fractures, and the Option 4 can be eliminated as the child is 4 years old and body image is not a great
admitted to the hospital unit for an unrelated prevention of injury takes highest priority in this child’s care. Pain would be important if a concern. Of the three remaining, choose the option that would be a concern throughout
condition. The nurse determines that which nursing fracture actually occurs, but the key is prevention of fractures, making risk for injury more the care of this child.
diagnosis has the highest priority for this child? appropriate. Skin integrity impairment would also not be a concern unless a fracture actually
occurred.
1.‐ Impaired skin integrity related to cast
2.‐ Pain related to fractures
3.‐ Risk for injury related to disease state
4.‐ Disturbed body image related to short stature

1041 A child is admitted to the hospital unit with a Correct answer: 1 Swelling and redness of involved joints is a symptom found in juvenile arthritis, not LCP Knowledge of the signs and symptoms of LCP disease will help to choose the correct
diagnosis of “rule out acute onset of Legg‐Calve‐ disease. Stiffness in the morning or after rest, an insidious limp after activities, and referred answer. Eliminate symptoms normally seen in LCP. That leaves only the correct response.
Perthes (LCP) disease.” The symptom that would not pain to the knee are all consistent with this diagnosis.
be associated with LCP is:
1.‐ Swelling and redness of the involved joint(s).
2.‐ Stiffness in the morning or after rest.
3.‐ Insidious limp after activities.
4.‐ Referred pain to the knee.

1042 A 12‐year‐old male is admitted to the adolescent unit Correct answer: 2 Once the diagnosis is made, the child should be non‐weight‐bearing on the affected hip, as Knowledge of the care of the client with slipped capital femoral epiphysis will help to
with a diagnosis of slipped capital femoral epiphysis. weight‐bearing can increase the amount of slippage. Wheelchair use should be avoided, as this answer the question correctly. After noting the critical word “not” in the question, select
Which of the following activities should not be allowed also may increase the amount of slippage. the option that places the affected joint at risk.
by the nurse prior to surgical correction?

1.‐ Ambulation with crutches; avoid bearing weight on the affected leg
2.‐ Sitting in a wheelchair
3.‐ Moving on a stretcher
4.‐ Maintaining bed rest

1043 An adolescent diagnosed with idiopathic structural Correct answer: 1 Back pain is not identified as a symptom of idiopathic structural scoliosis. Skirts that hang Determine which options are symptoms of scoliosis. Eliminate these, leaving only the
scoliosis describes all of the following symptoms. unevenly, unequal shoulder height, and uneven waist level are all positive symptoms of this correct answer.
Which one would the nurse conclude is not associated disorder.
with this diagnosis?
1.‐ Back pain
2.‐ Skirts that hang unevenly
3.‐ Unequal shoulder heights
4.‐ Uneven waist angles

1044 A 15‐year‐old who has a diagnosis of scoliosis is being Correct answer: 1 Adolescents are greatly concerned about their physical appearance as part of their growth Eliminate options 3 and 4 because they are so similar. Then, consider the developmental
seen in the outpatient clinic. The nurse is planning care and development. Unless there is a clear priority based on physiological need, attention to period of the child, which is key to determining the correct response. All adolescents worry
for this adolescent and develops the following nursing developmental concerns such as body image is important when caring for the adolescent about body image and being different. This child will appear different and may be encased
diagnoses. Which nursing diagnosis should take highest client. in a brace or cast.
priority?
1.‐ Disturbed body image related to treatment of scoliosis
2.‐ Diversional activity deficit related to treatment of scoliosis
3.‐ Anxiety related to outcome of treatment for scoliosis
4.‐ Fear related to treatment and unknown outcomes
1045 An adolescent is returning to the hospital unit after Correct answer: 2, 3, 5 There is some degree of paralytic ileus following a spinal fusion; therefore, nasogastric Look carefully at each option to make sure the option is totally correct. Eliminate those
surgical spinal fusion for scoliosis. The nurse would intubation is required along with frequent assessment of return of bowel function. The pain that are either incorrect or only partially correct.
include which of the following in the immediate experienced by this client is severe and requires intravenous medication, preferably with
postoperative care of this client? (Select all that apply.) patient‐controlled analgesia (PCA). Logrolling must be done every 2 hours, once allowed, to
prevent the accumulation of secretions in the lungs. Urinary retention is common, and an
indwelling catheter is used if present rather than repeated straight catheterization. Monitoring
the child’s respiratory status is crucial as is the use of an incentive spirometer.

1.‐ Oral analgesia for pain


2.‐ Logrolling every 2 hours
3.‐ Nasogastric intubation
4.‐ Straight catheterization every 4 hours
5.‐ Use of an incentive spirometer every two hours while awake

1046 A 3‐year‐old child is suspected of having Duchenne’s Correct answer: 3 The child with Duchenne’s muscular dystrophy (MD) has a history of meeting early Knowledge of Duchenne’s muscular dystrophy will aid in choosing the correct answer.
muscular dystrophy. Which of the following developmental milestones. Symptoms usually begin at around 3 years of age and include Eliminate option 4 as it is the opposite of the findings of Duchenne’s MD. Also, children
assessment findings by the nurse would support this difficulty climbing stairs, running, and pedaling. Duchenne’s MD is also called develop normally until there is onset of symptoms, so option 1 would be incorrect.
diagnosis? pseudohypertrophic MD as the muscles appear enlarged. The appearance of the hips is
normal.
1.‐ A history of delayed crawling
2.‐ Outward rotation of the hips
3.‐ Difficulty climbing stairs
4.‐ Wasted muscle appearance

1047 A child is admitted to the hospital with a diagnosis of Correct answer: 2 Serum laboratory studies in a child with osteomyelitis will reveal an increased WBC count, C‐ Option 1 can be eliminated because with infections the WBC is elevated. Two of the other
“rule out osteomyelitis.” Which of the following serum reactive protein, and sedimentation rate. The blood culture is usually positive. This disease tests have no relation to infections.
laboratory values noted by the nurse supports this process does not affect the HCT or BUN.
diagnosis?
1.‐ Decreased white blood cell (WBC) count
2.‐ Positive blood cultures
3.‐ Increased hematocrit (HCT)
4.‐ Increased BUN

1048 A 6‐year‐old child has a cast applied for a fractured Correct answer: 3 The sensation of numbness or tingling is a sign of neurovascular impairment. Neurovascular Determine which finding is abnormal. All others are expected findings.
radius. The nurse completes an orthopedic assessment impairment can lead to nerve ischemia and destruction, with possible permanent paralysis of
on this child. Which of the following symptoms the extremity. Any symptom of neurovascular impairment, such as paresthesia, lack of pulses,
requires immediate attention and should be reported edema that does not improve with elevation, pallor, and pain, needs immediate attention.
to the physician?

1.‐ Capillary refill of 4 seconds in the affected hand


2.‐ Edema in the affected fingers that improves with elevation
3.‐ Child describing feeling of the affected hand being “asleep”
4.‐ Skin surrounding the cast is warm

1049 Which of the following nursing care measures takes Correct answer: 2 The child with skeletal traction has a pin that passes through the skin into the end of a long The key concept is monitoring for complications of skeletal traction.
highest priority in caring for a child in skeletal traction? bone. This procedure provides an entrance for microorganisms. Frequent monitoring of the pin
site, pin care according to institutional policy, and frequent monitoring for signs of infection
take priority over the other nursing interventions listed.

1.‐ Assessing bowel sounds every shift


2.‐ Assessing temperature every 4 hours
3.‐ Providing adequate nutrition
4.‐ Providing age‐appropriate activities

1050 A nurse performs triage in a pediatric orthopedic Correct answer: 1 Slipped capital femoral epiphysis is a slipping of the femoral head that occurs most frequently First consider the age of the child most frequently seen with this condition. This will rule
clinic. Which of the following should the nurse before or during the rapid adolescent growth spurt. The onset of symptoms is gradual, and out one of the responses. Option 4 can be eliminated as the symptoms are not associated
recognize as a symptom of slipped capital femoral symptoms include limp, holding the leg in external rotation to relieve pain, restricted and with rotation.
epiphysis? painful internal rotation, and knee and hip pain.

1.‐ Pain in the hip of a preadolescent child


2.‐ Acute onset of knee pain
3.‐ Presence of a limp in a school‐age child
4.‐ Painful external rotation of the affected leg

1051 Which of the following statements made by the Correct answer: 1 The therapeutic management of the child with osteomyelitis includes limiting weight‐bearing Determine the right answer by eliminating any choice that is obviously correct
parent of a child being discharged with osteomyelitis on the affected part, immobilization, and administration of antibiotics. Antibiotic therapy may information.
requires further teaching by the nurse? continue intravenously for 3 to 6 weeks, and orally for another 2 weeks depending on duration
of symptoms, response to treatment, and sensitivity of the organism. Discharge teaching
needs to include follow‐up antibiotic care at home, care of the IV site, and continuing
antibiotic therapy even though it may seem as if all the symptoms are gone. Food sources such
as calcium and protein should be provided for bone healing.

1.‐ “I can stop the antibiotics when I see that my child is afebrile for one week”
2.‐ “We will make sure that our child’s diet has plenty of calcium and protein.”
3.‐ “I will look at the intravenous site for signs of infection a couple of times a day.”
4.‐ “My child won’t take physical education at school until allowed by the doctor.”

1052 A 5‐month‐old infant is being assessed for Correct answer: 3, 5 All four of the signs are assessment tests for developmental dysplasia of the hip. Ortolani and The core concept is the age of the child at the time of diagnosis. Wrong answers can be
developmental dysplasia of the hip. The nurse Barlow signs disappear after 2 to 3 months. Trendelenburg sign will be seen in the child who is eliminated based on age.
concludes that positive signs and symptoms that able to stand. Allis sign, shortening of the affected limb on the affected side, is a reliable test at
indicate this disorder include: (Select all that apply) 4 months of age. Asymmetric folds would be a positive sign at any age. The child is too young
to walk, so a limp would not be observed.

1.‐ Ortolani sign.


2.‐ Barlow sign.
3.‐ Allis sign.
4.‐ Trendelenburg sign.
5.‐ Asymmetric thigh and gluteal folds.

1053 A newborn is being admitted to the newborn nursery. Correct answer: 3 Clubfoot is apparent at birth, with the affected foot fixed in an abnormal position. The Eliminate option 4 first as clubfoot does not affect circulation. Basically options 1 and 2 are
The nurse would assess the infant for congenital affected foot is usually smaller, shorter, with an empty heel pad. The affected limb is usually the same, so they can be eliminated.
defects. In addition to the abnormal position of the shorter and has some calf muscle atrophy.
foot, the nurse would note which of the following if
clubfoot is present?
1.‐ Affected foot is larger and longer.
2.‐ Affected limb is longer.
3.‐ There is calf muscle atrophy of the affected limb.
4.‐ Affected foot is cooler.

1054 A child is admitted with osteogenesis imperfecta (OI). Correct answer: 3 Children with this disorder have normal calcium and phosphorus and abnormal precollagen Three of the tests listed are common tests. One is uncommon.
In reviewing laboratory findings, the nurse would type I. This prevents the formation of collagen, the major component of connective tissue. The
expect to find abnormal levels of: precollagen remains relatively unstable and unable to undergo final transformation into
collagen.
1.‐ Calcium.
2.‐ Phosphorus.
3.‐ Precollagen type I.
4.‐ Vitamin D.

1055 Which of the following statements made by a parent Correct answer: 4 Children with mild OI may be able to participate in sports, and many are able to participate in The goal of treatment for all children is to promote growth and development. The one
of a child with osteogenesis imperfecta (OI) needs swimming. There are no current medications that stop this disease process. There are a variety action that would limit growth and development would be home schooling.
clarification by the nurse? of surgical procedures that may be done to help strengthen the bones; one is the insertion of
intermedullary rods to provide for stability. The child with OI may participate in school, though
care needs to be provided to protect this child from injury.

1.‐ “My child may be able to participate in sports.”


2.‐ “There are no medications available to help this disease process.”
3.‐ “Surgery may be needed to place rods in the bone for stability.”
4.‐ “My child will need to be home schooled to protect him from injury.”

1056 The physician has written the following orders for a Correct answer: 4 Children with muscular dystrophy quickly suffer from complications of immobility. Therefore, The core concept is Duchenne muscular dystrophy. It is important with these children that
child with Duchenne muscular dystrophy hospitalized when hospitalized, these children should have physical therapy, range‐of‐motion exercises, and function be maintained, and bed rest will promote disability.
for a respiratory infection. The nurse should question bed‐to‐chair activity as soon as possible. Children with respiratory infections are treated with
the order for: vigorous antibiotic therapy, as well as postural drainage and cupping.

1.‐ Physical therapy.


2.‐ Antibiotic therapy.
3.‐ Passive range of motion exercises.
4.‐ Strict bed rest.

1057 A 14‐year‐old adolescent has just been fitted for a Correct answer: 3 The Milwaukee brace is worn for scoliosis, when the degree of curve is greater than 20 but The goal of therapy is to prevent progression of the scoliosis. To be successful in answering
Milwaukee brace. Which of the following should the less than 40 degrees. It is worn for 23 hours a day. Exercises to increase pelvic tilt, for lateral this question, the learner must understand the treatment plan.
nurse include in teaching about this brace? strengthening, and to correct lordosis should be done several times a day while in the brace.
The brace should be worn over a T‐shirt to minimize skin irritation. The adolescent may
experience muscle aches resulting from new alignment.

1.‐ The brace should be worn only when the adolescent is sleeping or in the recumbent position.
2.‐ The brace should be worn next to the skin.
3.‐ Exercises to increase pelvic tilt should be done several times per day while in the brace.
4.‐ The adolescent should experience no pain as a result of wearing this brace.

1058 The nurse has completed instructions on health Correct answer: 3 Aerobic exercises such as swimming help the client to maintain maximum range of motion Choose the odd option of the four, noting that swimming is the only non‐weight bearing
maintenance for a client diagnosed with osteoarthritis. (ROM) and mobility while minimizing strain on joints. Isotonic exercises such as tennis, jogging, exercise on the list.
The nurse verifies that the client understood the and volleyball place excessive strain on diseased joints.
instructions if the client states that participation in
which of the following sports would be beneficial?

1.‐ Tennis
2.‐ Jogging
3.‐ Swimming
4.‐ Volleyball
1059 A health history and physical assessment on a client Correct answer: 4 Swan neck deformities of the hand are classic deformities associated with rheumatoid This question requires differentiation of osteoarthritis and rheumatoid arthritis.
with rheumatoid arthritis (RA) may reveal which of the arthritis secondary to the presence of fibrous connective tissue within the joint space. Clients
following assessment data? with RA do experience morning stiffness, but it can last from 30 minutes up to several hours.
RA is characterized by symmetrical joint involvement, and Heberden's nodes are characteristic
of osteoarthritis.
1.‐ Heberden's nodes
2.‐ Morning stiffness no longer than 30 minutes
3.‐ Asymmetric joint swelling
4.‐ Swan neck deformities

1060 Which of the following over‐the‐counter (OTC) agents Correct answer: 3 Aspirin interferes with the action of uricosuric drugs. Acetaminophen, naproxen, or ibuprofen Be cautious with choosing ASA as a medication choice due to the high number of side and
should the client on uricosuric drugs be instructed to may be used effectively as an analgesic in the treatment of pain associated with acute gout, adverse effects associated with it, especially in the presence of a chronic illness.
avoid? and they do not interfere with the action of uricosuric drugs.

1.‐ Acetaminophen (Tylenol)


2.‐ Ibuprofen (Motrin)
3.‐ Aspirin (ASA)
4.‐ Naproxen (Naprosyn)

1061 Which of the following nursing interventions is Correct answer: 1 Options 2, 3, and 4 are appropriate nursing interventions when caring for a client diagnosed Remember not to use heat with infection; it will increase circulation and the dissemination
contraindicated in the care of a client with acute with osteomyelitis. The application of heat can increase edema and pain in the affected area of the infection.
osteomyelitis? and spread bacteria through vasodilatation.
1.‐ Apply heat compresses to the affected area
2.‐ Immobilize the affected area
3.‐ Administer narcotic analgesics for pain
4.‐ Administer OTC analgesics for pain

1062 Which of the following lab data would be most Correct answer: 4 Serum alkaline phosphatase is elevated because of increased activity of bone cells. This question requires knowledge of the diagnostics associated with the illness.
significant in the client with Paget's disease? Inflammation is in the bone and usually doesn't reveal an elevated serum WBC, ESR, or the
presence of Staphylococcus.
1.‐ Elevated white blood count (WBC)
2.‐ Elevated erythrocyte sedimentation rate (ESR)
3.‐ Positive tissue biopsy for <i>Staphylococcus</i>
4.‐ Elevated serum alkaline phosphatase

1063 Which of the following individuals is at greatest risk Correct answer: 4 Osteosarcomas are most commonly seen in males during optimal growth years. Middle‐aged Make sure when answering this question that the gender and age are both correct.
for developing an osteosarcoma? males (option 1), females age 50 to 60 (option 2), and females of childbearing age (option 3)
are less likely to develop osteosarcoma.
1.‐ Male, age 42
2.‐ Female, age 52
3.‐ Female, age 20
4.‐ Male, age 15

1064 A drug history is important in a client with systemic Correct answer: 1 Although the etiology of SLE is unknown, certain environmental factors have been associated Recall that valium and aspirin are common medications and that Imuran is an
lupus erythematosus (SLE) because the disease may be with the onset of symptoms. The administration of procainamide (Procan SR) and hydralazine immunosuppressant. Use the process of elimination to choose Procan.
associated with use of which of the following drugs? (Apresoline) have been associated with SLE symptoms, which usually subside after the drug is
discontinued.
1.‐ Procainamide (Procan SR)
2.‐ Acetylsalicylic acid (aspirin)
3.‐ Diazepam (Valium)
4.‐ Azathioprine (Imuran)
1065 To prevent occurrences of Raynaud's phenomenon, Correct answer: 1 Raynaud's disease is characterized by spasms of the blood vessels within the fingers of the Recall that vasoconstriction occurs in response to cold, which then exacerbates the client’s
what should the client diagnosed with systemic hands resulting in diminished circulation. Gloves protect the hands from cold temperatures pain.
Scleroderma be instructed to do? and provide warmth, which promotes blood flow to the affected areas. Raynaud's
phenomenon is in the CREST syndrome, a type of scleroderma.
1.‐ Wear gloves
2.‐ Perform range‐of‐motion exercises daily
3.‐ Limit sodium intake
4.‐ Avoid warm temperature

1066 Which of the following occurs during the remodeling Correct answer: 3 During earlier stages of bone healing, overproduction of callus enlarges the bone and acts as This question requires knowledge of the remodeling stage.
phase of bone healing? a splint. Callus is eventually replaced with mature bone during the ossification phase of bone
healing, and then the excess callus is resorbed during the remodeling phase to return the bone
to its original shape.
1.‐ Callus formation occurs.
2.‐ Callus is replaced with mature bone.
3.‐ Osteoclasts resorb excess callus to return the bone to its original shape.
4.‐ Proliferation of osteoblast and fibroblasts occurs within the hematoma at the fracture site.

1067 Which of the following statements applies to Correct answer: 1 Edema is expected immediately following a fracture, but because the fascia is non‐elastic, With compartment syndrome, think edema.
compartment syndrome? excessive swelling will lead to increased capillary pressure within the area resulting in nerve
and muscle damage if left untreated. Damage is irreversible if the capillary pressure reaches 30
mmhg. A pulse may still be present during early stages of compartment syndrome. One factor
that can differentiate pain associated with trauma from the fracture, and that from
compartment syndrome, is the ineffectiveness of analgesics when compartment syndrome
occurs.
1.‐ Increased pressure within the tissues results from excessive edema at the fracture site.
2.‐ Capillary pressures over 15 mmhg can result in permanent muscle and nerve damage.
3.‐ The presence of a peripheral pulse will rule out compartment syndrome.
4.‐ Pain associated with compartment syndrome will be relieved with use of analgesics.

1068 To determine if a client is experiencing compartment Correct answer: 3 Although assessing for edema, pulses, and the presence of drainage is important in the care Recall that pain is a key indicator with test questions; look at it carefully.
syndrome, which of the following is a priority area for of a client with a fracture, pain unrelieved by analgesics is the symptom most indicative of
nursing assessment? compartment syndrome.
1.‐ Assessing for edema at the fracture site
2.‐ Palpation of a pulse at the fracture site
3.‐ Performing a pain assessment
4.‐ Assessing for the presence of drainage on the cast

1069 Beta blockers are given to clients with scleroderma to Correct answer: 4 When clients with scleroderma develop Raynaud's phenomenon, which is characterized by Recall that beta‐blockers have cardiovascular actions; Raynaud’s is a cardiovascular
treat which of the following symptoms? vasospasms of the arteries and veins of the hands, beta blockers are the treatment of choice. phenomenon.
Bradycardia is not specific to scleroderma. Clients may experience pericarditis, but this is not
treated with beta blockers.
1.‐ Telangiectasis
2.‐ Pericarditis
3.‐ Bradycardia
4.‐ Raynaud's phenomenon

1070 Which of the following lab values is most significant Correct answer: 1 Hypercalcemia (option 1) can occur as a complication of Paget's disease secondary to With questions referring to bone diseases, such as Paget’s, think calcium.
when assessing complications associated with Paget's increased osteoclast activity. The other tests are not specific to Paget's disease although they
disease? are all abnormal values.
1.‐ Calcium level of 15 mg/Dl
2.‐ Positive RF factor.
3.‐ Blood urea nitrogen (BUN) of 140 mg/dl
4.‐ Eosinophil sedimentation rate (ESR) of 30 mm/hr

1071 Which of the following statements applies to Ewing's Correct answer: 2 Ewing's sarcoma is a primary bone tumor associated with rapid metastasis to the lung. It Use the process of elimination to find the one correct answer.
sarcoma? occurs most frequently in males during optimal growth periods.
1.‐ Occurs most frequently in men between the ages of 20 and 40 years of age
2.‐ Is a primary bone tumor characterized by rapid growth and lung metastasis
3.‐ Is a metatastic bone tumor that occurs secondary to colon cancer
4.‐ Usually originates in bone cartilage

1072 The pathophysiology underlying gouty arthritis may Correct answer: 2 The pathophysiology of gouty arthritis is related to overproduction or decreased excretion of This question requires knowledge that Gout is related to uric acid build‐up.
be attributed to: uric acid in the primary form. The other options are incorrect.
1.‐ Increased immune complexes within the joint cavity.
2.‐ An alteration in purine metabolism resulting in hyperuricemia.
3.‐ Excessive bone remodeling secondary to increased levels of phosphorus.
4.‐ The presence of fibrous adhesions within bony cartilage.

1073 A client is diagnosed with osteomyelitis involving the Correct answer: 2 Although all the items listed are important in the plan of care for the client diagnosed with Maslow's hierarchy may identify option 1 as highest priority, but infection is the cause of
foot. The highest priority during the implementation of osteomyelitis, maintaining aseptic technique and preventing the spread of infection is crucial the pain. Recall what causes the problem and deal with that first in a question like this one.
nursing care is: to resolving the disease process.
1.‐ Maintaining adequate pain control.
2.‐ Implementing aseptic technique.
3.‐ Promoting adequate nutrition.
4.‐ Splinting the foot to prevent foot drop.

1074 When counseling the parents of a child diagnosed Correct answer: 3 Symptoms of muscular dystrophy usually manifest themselves in the toddler years. The child This question requires knowledge of the prognosis of muscular dystrophy.
with muscular dystrophy, which of the following has a waddling gait and experiences frequent falls. There is no cure for the disease and muscles
statements most accurately describes the disease become progressively weak. Most children are wheelchair confined by the teen years. As the
progression? disease progresses, heart and lung muscle are affected, resulting in cardiac and pulmonary
failure. These complications frequently occur by the age of 20.

1.‐ "Prognosis is favorable with early detection."


2.‐ "With aggressive physical therapy and the use of a walker, your child will remain ambulatory through adulthood."
3.‐ "The muscles and the lungs may also become involved, and this can shorten your child's lifespan."
4.‐ "Muscle weakness is progressive and rapid, and your child will most likely be confined to a wheelchair by the age of 5."

1075 Which of the following statements best describes the Correct answer: 1 Articular cartilage is responsible for decreasing friction during joint movement and displacing This question requires knowledge that osteoarthritis is a degenerative disease.
underlying pathology of osteoarthritis (OA)? the force of the workload onto the subchondral bone. In OA, the composition of the articular
cartilage is changed because of a malfunction in the production in proteoglycans.
Consequently, the articular cartilage can no longer perform its original function.

1.‐ Changes in the composition of the articular cartilage contribute to increased friction during joint movement.
2.‐ Joint inflammation occurs secondary to the presence of immune complexes within the joint cavity.
3.‐ Excessive bone necrosis within the joint occurs secondary to increased osteoclastic activity.
4.‐ Bone damage occurs secondary to osteolysis and excessive bone remodeling.
1076 In the early treatment of rheumatoid arthritis (RA), Correct answer: 4 DMARDs are now being used earlier in the treatment regime for RA because evidence Notice that option 4 is the most inclusive answer as it includes both options 1 and 3.
which of the following drugs would the nurse expect to suggests that they may play a role in arresting the disease process. NSAIDs are used in
be included in the client's drug regime? combination with this drug classification for pain management. Systemic corticosteroids are
not used until NSAIDs are no longer effective because of the severe side effects associated
with their use.
1.‐ Non‐steroidal antiinflammatory drugs (NSAIDs)
2.‐ Systemic corticosteroids
3.‐ Disease modifying anti‐rheumatoidal drugs (DMARDs)
4.‐ NSAIDs and DMARDs

1077 Which of the following statements best describes the Correct answer: 2 Osteoporosis is characterized by excessive bone resorption that exceeds the body's ability to Use the process of elimination to find the one right answer.
pathophysiology underlying osteoporosis? The disease produce new bone. It is more prevalent in postmenopausal women with low levels of
process is related to: estrogen. A decrease in the number and activity of osteoblasts and an increase in the number
and activity of osteoclasts occurs.
1.‐ Increased amounts of estrogen in postmenopausal women.
2.‐ A decrease in osteoblasts and an increase in osteoclasts.
3.‐ Decrease in blood supply to the bone resulting in bony necrosis.
4.‐ Invasion of a pathogen leading to osteolysis of bone.

1078 After analyzing a urine report on a client with Correct answer: 2 Proteinuria in a client with SLE indicates renal involvement. The nurse should record I &amp; With proteinuria, think of diseases which may cause or be related to damage in the
systemic lupus erythematosus (SLE), the nurse O and analyze the results of additional labwork including blood urea nitrogen (BUN) and kidneys. This question asks about assessments which measure kidney function.
determines that the client has proteinuria. This finding creatinine (CR).
would lead the nurse to place the highest priority on
assessing:
1.‐ Blood pressure.
2.‐ Intake and output (I&O).
3.‐ Pulse.
4.‐ Temperature.

1079 Which of the following classifications of scleroderma Correct answer: 1 Scleroderma is not drug‐induced. Systemic sclerosis may result in changes to the esophagus, Notice that the word ‘systemic’ in option 1 provides a hint for the answer.
is associated with a poor prognosis, secondary to intestines, lungs, heart, and kidneys. Depending on which organs are involved, the condition
involvement of vital organs? may be life‐threatening. CREST syndrome and limited scleroderma are self‐limiting and
associated with a good prognosis.
1.‐ Systemic sclerosis
2.‐ CREST syndrome
3.‐ Limited scleroderma
4.‐ Drug Induced scleroderma

1080 Which of the following statements best describes the Correct answer: 4 Although the etiology of scleroderma is unknown, the disease process is related to This question requires knowledge of the pathophysiology of scleroderma.
pathophysiology related to scleroderma? overproduction of collagen leading to fibrosis and inflammation, which causes damage to the
affected area.
1.‐ Excessive ossification and thickening of collagen tissue causes thickening of the skin.
2.‐ Overproduction of osteoblasts contributes to bone deformities.
3.‐ Overactivity of osteoclasts contributes to erosion of the bone.
4.‐ Overproduction of collagen leads to fibrosis and inflammation of affected areas.

1081 Which of the following diagnostic tests is most Correct answer: 1 Serum ESRs and ANAs are elevated in scleroderma but are also indicative of other conditions. For this question, look for the most invasive and most conclusive diagnostic test.
conclusive in the diagnosis of systemic lupus A skin biopsy reveals collagen thickening and confirms the diagnosis. The rheumatoid factor is
erythematosus (SLE)? only significant for rheumatoid arthritis.
1.‐ Skin biopsy
2.‐ Eosinophil sedimentation rate (ESR)
3.‐ Anti‐nuclear antibodies (ANA)
4.‐ Rheumatoid factor

1082 Which of the following is the priority nursing Correct answer: 1 Immobilizing the fracture is the priority nursing intervention in the emergency management This question uses a basic principle of first aid.
intervention for a client immediately following a of a client with a fracture. After the area is immobilized, the nurse should follow through with
fracture? interventions outlined in options 2, 3, and 4.
1.‐ Immobilize the fracture
2.‐ Administer analgesics to control pain
3.‐ Apply ice to the fracture site
4.‐ Assess the radial pulse

1083 A client on bedrest for the past 48 hours secondary to Correct answer: 2 A fat emboli is a common complication following the fracture of a long bone. Symptoms of fat This question requires knowledge of the symptoms of fat emboli.
a fractured femur presents with tachycardia and a emboli include tachycardia, a petechial rash, and tachypnea.
petechial rash. The nurse attributes these symptoms to
a(n):
1.‐ Deep vein thrombosis (DVT).
2.‐ Fat Emboli.
3.‐ Hypovolemic shock.
4.‐ Allergic reaction to medication.

1084 Compartment syndrome occurs as a result of Correct answer: 3 Compartment syndrome occurs as a result of an excessive pressure within the joint cavity Notice that option 3 is the most comprehensive answer, containing both options 1 and 2.
obstruction in: which results in obstruction of both arterial and venous blood flow. Hence, the symptoms of
decreased or absent pulse, pallor, and decreased capillary refill.
1.‐ Arterial circulation.
2.‐ Venous circulation.
3.‐ Arterial and venous circulation.
4.‐ The lymphatic system.

1085 Which of the following teaching tips should the nurse Correct answer: 3 An adequate intake of calcium, either in the diet or through supplements and regular With prevention, think of the most basic means, in this case nutrition.
include when providing health teaching to clients participation in weight‐bearing activities are the most effective way to prevent osteoporosis.
regarding prevention of osteoporosis? Walking, jogging, and weight‐lifting are examples of weight bearing exercises; swimming is not.
Estrogen replacement is effective in preventing osteoporosis in postmenopausal women.

1.‐ Swimming is an effective weight‐bearing activity.


2.‐ Estrogen replacement is important for preventing osteoporosis in premenopausal women.
3.‐ Maintaining calcium intake between 1,200 to 1,500 mg/day is important.
4.‐ All clients over 40 years of age should undergo bone density studies annually.

1086 The first indication of bone loss in clients with Correct answer: 3 Although clients with osteoporosis are predisposed to pathological fractures, loss of height is Notice the hint in the question stem with the words ‘first indication.’ Recall that loss of
osteoporosis is: usually one of the first indicators of osteoporosis. height is one of the indications of bone loss.
1.‐ Crepitus with joint movement.
2.‐ Symmetrical joint deformities.
3.‐ Loss of height.
4.‐ Pathological fracture.

1087 Which of the following responses from a client Correct answer: 3 Application of heat prior to exercising may help improve joint function. Jogging is not Use the process of elimination to determine the most realistic option for this client.
recently diagnosed with osteoarthritis (OA) indicates a recommended because it puts excessive stress on joints. Clients with OA should maintain their
clear understanding of health maintenance teaching? weight to prevent excessive stress on joints. A firm mattress is recommended for support of
the lumbar spine.
1.‐ "I have no dietary restrictions and can eat whatever I want."
2.‐ "I can resume jogging 1 mile a day once I start on medication."
3.‐ "Heat applications before exercising may help improve my joint function."
4.‐ "I will get a softer mattress on my bed for extra support."

1088 A client is admitted to the emergency department Correct answer: 1 A client admitted to the emergency department and diagnosed with an acute episode of gout Recall that pain is an early sign and an indicator for nursing intervention.
and diagnosed with acute gouty arthritis. Which of the will be in severe pain. Although a serum uric acid level should be obtained, pain management
following is a priority nursing intervention? should be the first priority for the nurse. Clients prescribed uricosuric drugs for prevention of
further gouty attacks should be instructed to increase fluids to prevent kidney stones
associated with their use, however teaching should be conducted after the client is relieved of
pain.
1.‐ Providing pain management
2.‐ Collecting intake and output (I&O)
3.‐ Straining urine for kidney stones
4.‐ Obtaining a serum uric acid level

1089 Which of the following clients is at risk for developing Correct answer: 3 Hematogenus osteomyelitis originates as a blood borne infection and occurs most commonly Use the process of elimination to find the only option which indicates a hematogenic
hematogenus osteomyelitis? in children under 10 years of age with a recent history of a throat, ear, or skin infection. origin.
Options 1, 2, and 4 are examples of direct entry osteomyelitis.
1.‐ 17‐year‐old client with a gunshot wound to the femur
2.‐ 78‐year‐old client diagnosed with a fractured hip
3.‐ 8‐year‐old client with otitis media
4.‐ 20‐year‐old client, five days status post‐appendectomy

1090 The osteoblastic scelerotic phase of Paget's disease is Correct answer: 3 Paget's disease is characterized by two phases. The osteoblastic phase is characterized by This question requires knowledge of the progress of Paget’s disease.
characterized by: slowing down of bone resorption and enlarging of bones.
1.‐ Excessive bone remodeling.
2.‐ Replacement of normal bone marrow with vascular, fibrous, connective tissue.
3.‐ Slowing down of bone resorption and enlargement of bones.
4.‐ An increase in the activity of osteoclasts causing erosion of bone.

1091 Which of the following statements made by the Correct answer: 3 Muscular dystrophy is a genetic disorder of the recessive gene on the X‐chromosome (mother This question requires knowledge of sex‐linked genetics.
parents of a child diagnosed with muscular dystrophy to son). Option 1 would be true if all future pregnancies were males.
(MD) verifies to the nurse that they understand
teaching regarding the disease process?

1.‐ "Any child from future pregnancies is at risk for inheriting the disease."
2.‐ "The disease occurred because my child has a defective immune system."
3.‐ "The disease is genetically transmitted from mother to son."
4.‐ "The disease relates to a genetic disorder on the father's side."

1092 A client diagnosed with a metastatic bone tumor asks Correct answer: 3 The most common sites for metastatic bone tumors are the ribs, spine, and pelvis. This question requires knowledge of bone tumors.
the nurse where the tumor is. The nurse knows the
most likely place is in the:
1.‐ Tibia.
2.‐ Fibia.
3.‐ Spine.
4.‐ Skull.

1093 Osteosarcomas originate in: Correct answer: 1 The tissue of origin associated with osteosarcomas is the metaphyseal of long bones. The This question requires knowledge of osteosarcomas.
formation of chrondrosarcomas originate in cartilage, and Ewing's sarcoma originate in the
nerve tissue within the bone marrow.
1.‐ The metaphyseal of long bones.
2.‐ In the cartilage.
3.‐ In nerve tissue within the bone marrow.
4.‐ The lung.

1094 A client has been diagnosed with a metastatic bone Correct answer: 3 Gently supporting limbs and bony structures during transfers and position changes is the most For this question, use the most supportive measure which also increases client
tumor. Which of the following interventions would be effective way to prevent pathological fractures associated with bone tumors. Option 1 might independence.
most effective in preventing a pathological fracture? cause limited range‐of‐motion (ROM); options 2 and 3 aren't applicable.

1.‐ Restricting activities to complete bedrest.


2.‐ Instructing the client on pivot transfers.
3.‐ Gently supporting limbs during position changes.
4.‐ Providing a calcium supplement.

1095 Which of the following clients is at greatest risk for Correct answer: 3 Although the etiology of SLE is unknown, hormonal imbalances associated with pregnancy are For this question, ensure both the gender and age range are correct.
developing systemic lupus erythematosus (SLE)? thought to precipitate the onset of the disease. Women of childbearing years are at a greater
risk.
1.‐ A male, 25 years of age
2.‐ An elderly female, 72 years of age
3.‐ A college female, 24 years of age
4.‐ A Caucasian female, 42 years of age

1096 A client with osteoarthritis (OA) asked the nurse Correct answer: 4 The joints most often affected in osteoarthritis are the weight‐bearing joints; hips, knees, Use common experience to guide you in this question. Recall that many people with
which joints are most commonly effected? The best lumbar and cervical spine, and the phalangeal joints. arthritis are effected in their fingers.
answer is:
1.‐ Thoracic spine.
2.‐ Wrist.
3.‐ Elbow.
4.‐ Distal phalangeal joints.

1097 Which of the following musculoskeletal disorders is Correct answer: 2 During chronic osteomyelitis infection spreads through the bone, bone cells become necrotic This question requires knowledge of sequestra.
characterized by the formation of sequestra? and break off into segments called sequestra.

1.‐ Osteoarthritis
2.‐ Osteomyelitis
3.‐ Scleroderma
4.‐ Osteosarcoma

1098 Which of the following is a priority when providing Correct answer: 3 Although options 1, 2, and 4 are all important interventions in the health teaching of clients Notice the stem asks for the priority intervention. This also allows the client the greatest
health teaching to a client with rheumatoid arthritis with rheumatoid arthritis, instructing the client on energy conservation techniques and pacing level of independence.
(RA)? Instruct the client: activities early in the teaching plan will help to provide immediate symptom control.
Decreasing activity (option 2) may further limit range‐of‐motion.

1.‐ On weight reduction.


2.‐ To decrease activity to avoid joint pain.
3.‐ To pace activities to decrease joint stress.
4.‐ To contact the Arthritis Foundation for support services.

1099 A 40‐year‐old athletic male client has been diagnosed Correct answer: 3 In adult males, the most prevalent contributing factor in the development of osteoarthritis is Recall that a previous injury may be a site for arthritis.
with osteoarthritis of the knee. The nurse will most history of joint trauma. An athletic male may have had trauma to the knee from a sport many
likely attribute the etiology of the disease to the years earlier in high school.
client's:
1.‐ Age.
2.‐ Gender.
3.‐ Previous trauma.
4.‐ Current weight.

1100 Which of the following clinical manifestations is a Correct answer: 4 Tophi are hard movable nodules with irregular surfaces associated with chronic untreated This question requires knowledge of chronic gouty arthritis.
primary indicator of chronic gouty arthritis? gout. Acute pain and hyperuricemia are not exclusive to chronic gouty arthritis.

1.‐ Acute pain


2.‐ Hyperuricemia
3.‐ Swan neck deformities
4.‐ Tophi on the helix of the ear

1101 The primary organism responsible for osteomyelitis Correct answer: 1 The most common causative agent in clients with osteomyelitis is Staphylococcus aureus. The This question requires knowledge of osteomyelitis.
is: other organisms could contribute but are not usually the primary organism; look for key
words.
1.‐ <i>Staphylococcus aureus</i>.
2.‐ <i>Escherichia coli</i>.
3.‐ <i>Streptococcus</i>.
4.‐ <i>Pseudomonas</i>.

1102 Which of the following clinical manifestations Correct answer: 3 Increased head size, headaches, and hearing loss indicate that Paget's disease has progressed This question requires knowledge of Paget's disease, remembering that it is a bone
indicates the involvement of the cranium in a client and involves the cranium. disease.
with Paget's disease?
1.‐ Ptosis
2.‐ Alterations in vision
3.‐ Enlarged head size
4.‐ Difficulty chewing

1103 Which of the following is usually one of the first Correct answer: 4 Symptoms of muscular dystrophy present around 2 to 3 years of age. Children usually present Notice that this question asks for the first or earliest signs of having MD.
symptoms observed in children with muscular with a waddling gait, toe walking, and frequent falls. Kyphosis occurs with disease progression,
dystrophy (MD)? and recurrent URIs occur with involvement of lung tissue.
1.‐ Leg bowing
2.‐ Recurrent URI's
3.‐ Kyphosis
4.‐ Toe walking and frequent falls

1104 Which of the following statements made by a client Correct answer: 1 The etiology of fibromyalgia is unknown, but it is believed that it may be precipitated by This question reinforces the connection of stress with fibromyalgia.
with fibromyalgia indicates to the nurse that the client stress. The condition may be acute or chronic, non‐steroidal antiinflammatory drugs (NSAIDs)
has a clear understanding of the health teaching? will help to control joint and muscle pain associated with the condition but is not a cure.

1.‐ "I need to identify the stressors in my life and work on reducing them."
2.‐ "The drugs prescribed by the doctor will cure my condition within six months."
3.‐ "I will include high‐fiber foods in my diet."
4.‐ "My frequent headaches are unrelated to my condition."

1105 The nurse analyzes the results of x‐rays done on a Correct answer: 4 Options 1 and 3 are characteristic of malignant tumors. Soft tissue sarcomas are not detected Notice options 3 and 4 are opposites, meaning one of them is likely to be the correct
client with a benign bone tumor. Which of the on x‐rays; a computed tomography (CT) scan or magnetic resonance imagery (MRI) must be answer.
following results would the nurse expect to find? performed.
1.‐ Extension of tumor beyond the structure of the bone
2.‐ The presence of a soft tissue sarcoma
3.‐ Undefined tumor borders
4.‐ Well‐defined tumor margins

1106 The pathophysiology of systemic lupus erythematosus Correct answer: 1 SLE is an autoimmune disease in the body, characterized by production of autoantibodies This question requires knowledge of SLE and its causes.
(SLE) is primarily related to: against DNA secondary to hyperactivity of B‐cells.
1.‐ The production of autoantibodies against DNA.
2.‐ A mutated gene.
3.‐ Overproduction of collagen.
4.‐ Chronic inflammation secondary to a virus.

1107 Which of the following joint deformities is usually Correct answer: 3 Bouchard's nodes are located on the Proximal Interphalangeal (PIP) joints of clients with This question requires knowledge of the terminology and OA. Use the process of
evident in the hand of a client diagnosed with osteoarthritis. Swan neck deformities, ulnar drift, and boutaniere deformities are elimination to select the option associated with OA.
osteoarthritis (OA)? characteristic of rheumatoid arthritis (RA).
1.‐ Swan neck deformities
2.‐ Boutaniere deformities
3.‐ Bouchard's nodes
4.‐ Ulnar drift

1108 The primary care provider determines that a 55‐year‐ Correct answer: 3 Women of menopausal age are at risk for osteoporosis, and foods high in calcium should be This question requires knowledge of nutrition. Select the option with the highest amount
old female client is experiencing menopause and is encouraged. All the foods in option 3 are high in calcium. Chicken and eggs are high in protein; of calcium.
also at risk for osteoporosis. What foods other than wheat and corn are high in carbohydrates.
milk can the nurse suggest to this client to increase her
calcium intake?
1.‐ Seafood, wheat, corn, green vegetables
2.‐ Chicken, green vegetables, sardines, broccoli
3.‐ Green vegetables, sardines, salmon with the bone, broccoli
4.‐ Eggs, cheese, sardines, fish

1109 What risk factors identified by the nurse would put a Correct answer: 1 The factors presented in option 1 put the client at risk for osteoporosis. Smoking, alcohol Select the answer with the greatest number of risk factors.
client at risk for developing osteoporosis? intake, and dietary deficiency of calcium and Vitamin D are major factors in the development
of osteoporosis. Deficient protein and carbohydrate intake, obesity, depression, and history of
falls do not contribute to the development of osteoporosis.

1.‐ Menopause, stress, sedentary lifestyle, smoking, excessive alcohol intake, and diet deficient in calcium and Vitamin D
2.‐ Family history, age, history of falls, smoking, alcohol, and diet deficient in protein
3.‐ Diet deficient in protein and carbohydrates, smoking, excessive alcohol intake, stress, and sedentary lifestyle
4.‐ Inadequate sunlight exposure, obesity, depression, poor dietary intake of calcium, and excessive alcohol intake

1110 Alendronate (Fosamax) is ordered for a client with Correct answer: 4 Fosamax is the drug that prevents bone resorption. Calcitonin (Micalcin) increases bone mass This question requires knowledge of Fosamax.
osteoporosis. Which information should the nurse and is dispensed as a nasal spray; raloxifene (Evista) is a selective receptor modulator.
include in teaching the client about this drug?

1.‐ It is a selective estrogen receptor modulator.


2.‐ It increases bone mass.
3.‐ It may be obtained as a nasal spray.
4.‐ It prevents bone resorption and is taken orally.

1111 The nurse is preparing a client who sustained a hip Correct answer: 1 The client with hip surgery should avoid all activities that will cause hip adduction, internal Notice that this question asks for an incorrect answer. Use the process of elimination to
fracture for discharge. The nurse should teach the rotation, and flexion beyond 90 degrees. The focus of the teaching on clients with hip surgery find the one wrong answer.
client to avoid which of the following groups of is to avoid dislocation and the risk for further injury.
activities to prevent dislocation of the hip?
1.‐ Crossing legs, bending at hips, and sitting on low toilet seats
2.‐ Taking leisurely walks, low chair seats, and bending at hips
3.‐ Using reachers for applying shoes and socks, and sitting in chairs with arms
4.‐ All exercises, bedrest, and using raised toilet seats

1112 A client with a total hip replacement is concerned Correct answer: 1 Extremes of internal rotation, adduction, and 90‐degree flexion of the hip should be avoided 4 This question asks about the basic care after hip replacement. Recognize that option 1 is
about dislocation of the prosthesis. What can the to 6 weeks after surgery to prevent dislocation. Although use of elevated seats prevents excess the best answer, although several options are true.
nurse say to reassure this client? flexion of the hip, it alone does not suffice in preventing dislocation. Bending activities (such as
putting on shoes) place the client at risk for dislocation. Abduction pillows are used to prevent
external rotation and must be used postoperatively.

1.‐ "Avoid activities that cause adduction of the hip to prevent dislocation."
2.‐ "Use of elevated toilet seats alone will prevent dislocation."
3.‐ "Perform bending exercises as often as able to prevent dislocation."
4.‐ "Remove the foam abduction pillow as soon as possible postoperatively."

1113 A client has undergone a lumbar laminectomy and Correct answer: 3 Musculoskeletal injuries and subsequent treatment have the potential to cause Remember assessments are usually an essential action to be done first.
has just returned to the nursing unit. It is essential for complications. Bleeding and swelling from the surgery may cause compression of nerves that
the nurse to perform which of the following activities can lead to permanent neurological damage and paralysis. Frequent assessment of the
during this period? neurovascular status of the client is essential following laminectomy. Neurovascular
assessment includes assessing for pain, pulses, pallor, paresthesia, and paralysis. The physician
usually orders ambulation. Vital signs are not done every 30 minutes unless the client is in the
post‐anesthesia care unit. Although loss of bladder tone may indicate nerve damage, it may
also be a residual effect of the anesthesia. Assessing ability to void becomes of prime
importance if the client is due to void, usually 6 to 8 hours after last voiding.

1.‐ Early ambulation


2.‐ Vital signs checks every half‐hour
3.‐ Neurovascular checks
4.‐ Assessment of bladder function

1114 A client in traction slides down in the bed so that the Correct answer: 3 The pull of traction on the affected limb should never be disturbed to ensure healing and This question asks for the traction to be uninterrupted. Note that option 3 is best.
feet touch the foot of the bed. What should the nurse union of the bone in proper alignment. This intervention is an independent nursing activity and
do to ensure that the pull of traction remains does not require a physician's order. A change in weight is not indicated. Elevating the client's
uninterrupted? feet will not correct the situation.
1.‐ Release the weights, pull the client up in bed, and then reapply weights
2.‐ Ask the physician for a change in the amount of weight ordered
3.‐ Move the client up in bed without releasing the pull of traction on the extremity
4.‐ Elevate the client's feet on a pillow

1115 The nurse is caring for a client with skeletal traction. Correct answer: 1 A major complication of skeletal traction is infection. The nurse must provide pin site care Recognize that although the other options might be appropriate, option 1 is an essential
It is most important that the nurse monitor which of using aseptic technique to prevent infection. nursing intervention.
the following?
1.‐ The pin site for unusual redness, swelling, purulent drainage, and foul odor.
2.‐ The distance between the client's hip and the traction.
3.‐ The number of times the client exercises the affected limb.
4.‐ How the client is coping with immobilization.
1116 A client in skeletal traction complains of unrelieved Correct answer: 4 Unrelieved pain, diminished pulses, pallor, paresthesias, and pain on passive motion are all Notice that this situation calls for emergency care. The nurse has conducted the
pain at rest and paresthesia in the affected extremity. symptoms of compartment syndrome. This is a medical emergency because the pressure must appropriate assessments and basic comfort measures and it is time to call the physician.
The assessment by the nurse reveals diminished pulse, be relieved in the affected limb. Otherwise, the swelling in the closed compartment may lead
pallor, and increased pain on passive motion. What to further permanent complications, such as loss of the limb. Options 2 and 3, although
must the nurse do first? appropriate, are not the priority interventions in this case. The administration of oxygen is an
inappropriate initial action in this situation.

1.‐ Administer oxygen


2.‐ Encourage deep‐breathing and coughing exercises
3.‐ Administer pain medication as ordered
4.‐ Notify the physician immediately

1117 The nurse is caring for a client who had open Correct answer: 3 The elevated temperature, chills, malaise, and pain are all clinical manifestations of Recall that these are cardinal signs of infection.
reduction and internal fixation (ORIF) of the right osteomyelitis. Symptoms of fat embolism include acute respiratory distress. Symptoms of
femur 4 days ago. The client complains of intense pain, compartment syndrome include progressively worsening pain distal to the affected site
swelling, tenderness and warmth at the site, chills, unrelieved by analgesics. Malunion of the bone will not cause an elevated temperature.
malaise, and has a temperature of 102.2 degrees F (39
degrees C). This data indicates which of the following?

1.‐ Fat embolism


2.‐ Compartment syndrome
3.‐ Osteomyelitis
4.‐ Malunion of the bone

1118 The nurse prepares a client for an arthrogram with Correct answer: 2 An arthrogram involves injecting dye into a joint for diagnostic purposes. It is critical that the Recall that allergy assessments are integral prior to diagnostics.
contrast dye. What priority nursing assessment should nurse evaluate the client for history of allergic reaction to contrast dye before the procedure
be performed for this client? since this can lead to a life‐threatening response such as anaphylactic shock. The other options
are not priority assessments or are irrelevant.
1.‐ History of claustrophobia
2.‐ History of allergic reaction to contrast dye
3.‐ Vital signs
4.‐ Presence of metallic implants such as a pacemaker or aneurysm clips

1119 A retired 66‐year‐old female client is being evaluated Correct answer: 1 Low bone mass, structural deterioration of bone tissue leading to bone fragility, and This question requires knowledge of risk factors associated with osteoporosis.
for osteoporosis as part of a yearly physical. The client increased susceptibility to fractures is seen with osteoporosis. The client also has risk factors
tells the nurse that she is a smoker, watches television associated with osteoporosis: smoking, sedentary lifestyle, and being female and
for most of the day, and has been hospitalized with postmenopausal.
three different fractures within the last year. Based on
the information given by the client, the nurse suspects
which of the following?

1.‐ Low bone mass leading to increased bone fragility.


2.‐ Degeneration of the articular cartilage.
3.‐ Recurrent attacks of acute arthritis.
4.‐ Personality changes caused by the chronic nature of the illness.

1120 Following laminectomy surgery, the nurse should turn Correct answer: 3 After laminectomy it is critical that proper body alignment is maintained to prevent Recognize that logrolling is a key intervention with spinal surgery.
and reposition the client by doing which of the postoperative complications such as neurological damage. Logrolling technique ensures that
following? the client turns as a unit. All the other options put stress on the spine.
1.‐ Having the client use the side rails of the bed.
2.‐ Elevating the head of the bed 45 degrees, then turning the legs together towards the floor, bending at the waist.
3.‐ Logrolling the client as a unit, keeping the body in proper alignment.
4.‐ Turning the client's head and shoulders then hips.
1121 The nurse receives a client with a hip spica cast that is Correct answer: 3 Handling a cast that is not completely dry with the fingertips creates indentations in the cast. Recall basic principles of cast care.
not completely dry. When turning the client the nurse These indented areas are thinner and are prone to cracks when the cast is completely dry. A
uses the palms and not the fingertips. The nurse wet cast should be handled with the flat part of the hands and exposed to air to assist in
chooses this technique for which of the following drying.
purposes?
1.‐ To speed‐dry the cast.
2.‐ To decrease pain from moving.
3.‐ To prevent damage to the cast.
4.‐ To prevent swelling.

1122 The nurse is assigned to a 70‐pound client in skin Correct answer: 3 Countertraction will prevent the client from sliding to the foot of the bed. This can be This question assesses knowledge of traction and countertraction.
traction. The nurse plans care to maintain effective achieved with Trendelenburg position of the bed or raising the foot of the bed slightly if the
countertraction by doing which of the following? client's body weight is not sufficient. The other options do not add to countertraction.

1.‐ Elevating the head of the bed


2.‐ Adding weights to the existing traction
3.‐ Placing the bed in Trendelenburg position
4.‐ Keeping the bed flat

1123 A client underwent hip replacement yesterday. Which Correct answer: 4 Clients with joint replacement require aggressive physical therapy postoperatively to regain Remember physiological needs should be met first, then physical mobility is important
nursing diagnosis is of highest priority to be included in range of motion in the joint caused by pain and swelling. The other nursing diagnoses are not a after this surgery.
the client's plan of care? priority at this time.
1.‐ Self‐care deficit
2.‐ Chronic pain
3.‐ Disturbed body image
4.‐ Impaired physical mobility

1124 The nurse observes that a female client has Correct answer: 2 A classic sign of scoliosis is asymmetrical dress or skirt tail/hem caused by unevenness of the Recall the classic signs of scoliosis.
asymmetry of the shoulder, hips, and the tail/hem of affected shoulder and hip. The lateral curvature resulting from the spinal deformity causes the
her dress. The nurse suspects that the client may be asymmetry. The other options do not necessarily cause all the manifestations listed in the
presenting with which of the following disorders? question.

1.‐ Congenital hip dislocation


2.‐ Scoliosis
3.‐ A fractured tibia
4.‐ Degenerative disc disease

1125 The nurse is caring for a client in Russell's traction. Correct answer: 1 Weights help to keep the fractured extremity in proper alignment to facilitate healing and Notice that all the other options are not as essential as option 1, which is correct.
The nurse observes the client's son playing with the therefore should not be manipulated. Nursing interventions for clients on traction should
weights attached to the traction. The nurse takes include ensuring that the weights hang freely at all times to maintain the line of pull. Traction
immediate action for which of the following reasons? is not released to maintain a steady pull.

1.‐ Manipulation of the weights will affect healing of the client's fracture.
2.‐ Traction should only be released once a day.
3.‐ The spasms of the extremity might increase.
4.‐ The client's hip may dislocate.
1126 The nurse encourages a 68‐year‐old client to discuss Correct answer: 2 Estrogen therapy decreases bone demineralization preventing progression of osteoporosis. It This question requires knowledge of estrogen replacement therapy.
estrogen replacement therapy with the physician after also increases bone density in the spine and hip and therefore reduces the risk of fractures.
explaining that estrogen has which of the following The other options do not appropriately describe the action of estrogen in the preventive and
benefits? therapeutic management of osteoporosis.
1.‐ Enhances the storage of Vitamin D
2.‐ Helps prevent progression of osteoporosis
3.‐ Increases longevity in postmenopausal women
4.‐ Cures osteoporosis

1127 A client is ready for discharge from the hospital Correct answer: 1 A client who undergoes surgery to the hip must be careful to avoid flexing the joint to greater This requires knowledge of hip surgery and treatment.
following hip surgery. The nurse would ensure that than 90 degrees postoperatively to prevent dislocating the hip. There is no portable Buck’s
which of the following is available for the client at the traction, although traction may be used preoperatively to immobilize the limb. A soft cushion
time of discharge? is not essential. The client may need to use a walker for assistance, but does not need
crutches.
1.‐ Raised toilet seat
2.‐ Portable Buck’s traction
3.‐ Soft cushion to use on chairs
4.‐ Crutches

1128 A client presents to the Emergency Department with Correct answer: 2 Standard x‐rays will not provide the detail necessary to evaluate soft tissue or cartilage Recall that MRIs show all structures.
a shoulder injury sustained while playing ice hockey. damage; arthroscopy is used to examine the interior of a joint; a bone scan is used to diagnose
The nurse teaches the client about which of the bone malignancies.
following diagnostic tests that would best identify
abnormalities of the cartilage and soft tissue
surrounding the joint?
1.‐ Standard x‐ray
2.‐ Magnetic resonance imaging (MRI)
3.‐ Bone scan
4.‐ Arthroscopy

1129 A client presents to the Emergency Department with Correct answer: 3 A fractured extremity will be shorter than the unaffected extremity because of contraction of Note that options 3 and 4 are opposites, therefore one may be correct.
multiple injuries caused by a motor vehicle accident. the muscle, swelling at the site, and misalignment of the bone fragments. Pain with movement
The nurse suspects that the client may have a right and bruising are not caused solely by hip fracture, and thus they are not distinctive signs.
fractured hip after noting which of the following
distinctive signs?
1.‐ The client reports increased pain with movement of the limb.
2.‐ There is bruising over the right hip.
3.‐ The right leg is shorter than the left.
4.‐ The right leg is longer than the left.

1130 The nurse is providing care for a young athlete who Correct answer: 2 Routine stretching and warm‐up exercises are essential before strenuous physical activities to Recall basic principles of mobility.
presents with muscle strain of the back that occurred prepare the muscle to withstand the stress of the motion. The intake of supplemental
during a tennis match. Client teaching about electrolytes (such as what is contained in Gatorade) and high‐protein drinks do not prevent
preventing such injuries in the future should include strains from occurring. The use of a back brace impedes movement during an activity such as
which of the following? tennis and is not the best strategy to preventing future muscle strain.

1.‐ Drinking 2 liters of Gatorade before every sports activity


2.‐ Routine stretching and warm‐up exercises before every sports activity
3.‐ Wearing a back support brace
4.‐ Drinking a high‐protein shake before activity
1131 The nurse provides care to an elderly client in Correct answer: 2 Russell's traction can partially immobilize the extremity to reduce spasms. It is important that Recall that skin integrity is the body’s first line of defense.
Russell's skin traction prior to reduction of a left hip the nurse monitors the skin under the traction for breakdown, especially over bony
fracture. Priority nursing assessment should include prominences and other pressure areas. Option 1 is incorrect because this is a type of skin
observation of which of the following? traction, and therefore there is no insertion site. Assessment of the skin for dehydration does
not necessarily have to be done under the skin traction; rather it can be done in other areas.

1.‐ The pin sites for infection


2.‐ The client's skin for breakdown
3.‐ The left leg for decreasing length
4.‐ The client's skin for dehydration

1132 A client presents to the clinic with Paget's disease. Correct answer: 1 Skeletal pain is a classic symptom of Paget's disease. The location of the pain depends on the This question requires knowledge of the process of Paget's disease.
The client's chief complaint is skeletal pain. The nurse bone affected. Arthritis may result from damage to joint cartilage, but Paget's disease does not
interprets this symptom as which of the following? result from bone deformities or from poor calcium uptake. There is excessive bone resorption
followed by bone formation which leads to weakened bone, pain, deformity, and pathologic
fractures.
1.‐ A characteristic of the disease process
2.‐ An ominous sign that should be reported to the physician immediately
3.‐ Caused by bony deformities
4.‐ Caused by poor calcium uptake by the bones

1133 The nurse teaches a client with osteoarthritis about Correct answer: 1 Joint pain with movement seen with osteoarthritis is aggravated by continual activity. A Recall that rest periods are often key to disease management.
managing joint stiffness associated with the disease. planned rest period is a very important intervention for managing the discomfort. Clients with
The nurse knows that the teaching objective has been osteoarthritis should maintain a regularly scheduled exercise program to maintain joint
met when the client makes which of the following flexibility and mobility. The application of heat and cold on the affected joint is for temporary
statements? pain relief. A high‐calorie intake does not address the management of joint stiffness.

1.‐ "I will balance activities with rest periods throughout the day."
2.‐ "I will increase my calorie intake to supplement my energy needs."
3.‐ "I will apply a heating pad to the affected site when I feel pain."
4.‐ "I will apply a cold pack to the affected site when I feel pain."

1134 The nurse assists a client with osteoporosis to plan a Correct answer: 1 The combination of foods in option 1 has the highest amount of calcium. Each food in this Select the option with the highest amount of calcium.
meal high in calcium. Which meal would be the best meal is high in calcium overall.
choice for the client?
1.‐ Baked salmon with the bone, broccoli, 1 cup of fruit yogurt, 1 cup of skim milk
2.‐ Turkey sandwich with Swiss cheese and mayonnaise on wheat bread, 1 small peach, iced tea
3.‐ Egg sandwich on whole wheat bread, 1/2 cup cottage cheese, iced tea, 1 slice cheesecake
4.‐ Steamed chicken on brown rice, 1 cup skim milk, tossed salad, 1 slice yellow cake

1135 The nurse provides teaching to a 35‐year‐old client Correct answer: 2 Strengthening the back muscle is a critical intervention to preventing injury. This goal can be This question requires knowledge and management of chronic low back pain.
who is a construction worker about managing accomplished by regular exercise. Teaching a client the principles of good body mechanics,
symptoms associated with chronic low back pain. The maintenance of good posture, and the importance of adhering to a prescribed exercise routine
nurse determines that the teaching objective was met are important to strengthen back and abdominal muscles. The goal of managing back pain is to
when the client makes which of the following prevent its recurrence. The use of a back brace and sporadic exercise are not appropriate in
statements? decreasing the episodes of back injury.

1.‐ "I will wear a brace at work."


2.‐ "I plan to start a regular exercise program."
3.‐ "I will not carry objects more than 10 pounds."
4.‐ "I will try to exercise whenever I can."
1136 A client had an above‐the‐knee amputation yesterday Correct answer: 1 It is normal to continue to have sensation in the amputated limb site since nerve endings are Recall that pain needs to be managed from the client’s perspective.
because of advanced peripheral vascular disease. The still present. The client feels real pain, and interventions should be provided to relieve it. Fat
client is complaining of pain and itching in the embolism (option 3) is usually a complication of bone fracture and may have a clinical
amputated limb. The nurse should take which of the presentation of confusion. However, other signs accompany fat embolism.
following actions?

1.‐ Administer the prescribed analgesic for pain and explain to the client that this sensation is typical.
2.‐ Explain to the client that the limb is no longer present and the sensation will go away.
3.‐ Notify the physician immediately since this could be a sign of fat embolism.
4.‐ Obtain a psychiatric assessment for the client who may be hallucinating.

1137 The nurse assessing a client with osteoarthritis notes Correct answer: 3 Heberden's nodes are bony enlargements on the distal interphalangeal joints (DIP), and This question requires differentiation of osteoarthritis and rheumatoid arthritis.
finger deformities on the proximal and distal Bouchard nodes are bony enlargements on the proximal interphalangeal joints (PIP). The figure
interphalangeal joints. The nurse documents this shows the typical changes of the DIP and PIP. Ulnar deviation and Boutonniere and swan neck
common finding as which of the following in the deformities are typically associated with rheumatoid arthritis.
medical record?
1.‐ Interphalangeal drift and ulnar deviation
2.‐ Boutonniere deformity
3.‐ Heberden's and Bouchard nodes
4.‐ Swan neck deformity

1138 A client recently diagnosed with hypothyroidism Correct answer: 3 Clients with hypothyroidism need to take thyroid hormone replacement medication daily all The core issue of the question is knowledge of medications used to manage
demonstrates understanding of prescribed of their lives. After the client has reached normal serum T&lt;sub&gt;4&lt;/sub&gt; levels, the hypothyroidism. Use nursing knowledge and the process of elimination to make a selection.
levothyroxine (Synthroid) medication when she makes normal metabolic rate may help the client lose the weight gained during the hypothyroid state,
which of the following statements? but this is not the purpose of the replacement medication. Usually, the medication should be
taken on an empty stomach, 1 hour prior to a meal or 2 hours after a meal.

1.‐ “I should be able to become pregnant in a couple of months.”


2.‐ “This medication will help me lose all this excess weight.”
3.‐ “I should call the physician for nervousness, diarrhea, or increased pulse.”
4.‐ “This medication should be taken with food, preferably dairy products.”

1139 The client is post‐transsphenoidal hypophysectomy. Correct answer: 3 Bending the knees and squatting is preferred to bending at the waist to reach the floor as a The core issue of the question is knowledge of measures to prevent rises in intracranial
The client demonstrates understanding of education means of preventing rises in intracranial pressure (ICP) following pituitary surgery. Holding the pressure following pituitary surgery. Use nursing knowledge and the process of elimination
when he states, “I know I need to be careful not to breath as well as leaning over will increase ICP. Clients should be taught to avoid holding the to make a selection.
increase the pressure in my head by breath for any reason and to avoid leaning forward or bending at the waist to prevent an
increase in intracranial pressure. To tie shoes, the client should sit on the couch or bed, bend
the knee and place his or her foot on the couch or bed to reach the shoelaces; Alternatively,
the client can sit on the floor to tie shoes or can avoid shoes that tie until there is no risk for
increased ICP.
1.‐ Sitting in a soft chair and leaning over slowly to tie my shoes.”
2.‐ Holding my breath when I reach down to pick up something from the floor.”
3.‐ Bending my knees first before squatting down to reach something on the floor.”
4.‐ Holding my breath while I use mouthwash, then leaning my head down toward the sink to spit it out.”

1140 The client who is 80 hours post‐transphenoidal Correct answer: 2 The numbness of the upper lip and gum near the incision as well as a decreased sense of The core issue of the question is the ability to accurately interpret the significance of client
hypophysectomy reports numbness on the upper lip smell are normal and should resolve in 3 to 4 months. The movement of the small rugs findings after pituitary surgery and then determine the next action. Use nursing knowledge
and gum, a headache when reclining, and has a suggests an unsteady gait or foot drop; both this and the headache are signs of increased and the process of elimination to make a selection, recalling that further assessments are
tendency to kick around small rugs in the room when intracranial pressure. In‐depth assessments of neuromuscular function and incision site are often indicated when encountering abnormal data.
walking. The home health nurse should do which of the needed, and then the surgeon should be consulted immediately. The other responses are
following? either excessive or insufficient.
1.‐ Inform the client that these are normal responses and will disappear over 2 to 3 weeks.
2.‐ Assess neuromuscular function and incisional area and then report all findings to the surgeon.
3.‐ Immediately arrange for the client to be transported to the hospital for treatment of increased intracranial pressure.
4.‐ Assess vital signs, fluid volume status, bowel function and nutrition status.

1141 The client with diabetes mellitus requests a Correct answer: 2 Headache, restlessness, anxiety, sweating, and increased pulse are signs of hypoglycemia. The core issue of the question is recognition that the client is at risk for hypoglycemia and
medication for headache soon after returning from an Resolution of symptoms should occur after the client drinks the juice. The other options either the corrective actions that need to be taken. Use nursing knowledge and the process of
early morning x‐ray procedure. The nurse observes the delay treatment (options 1 and 3) or fail to recognize the real problem (option 4). elimination to make a selection.
client is upset about the headache, angry at missing
breakfast, and has moist hands. The nurse should do
which of the following as the priority action?

1.‐ Administer the medication for headache and arrange for a breakfast tray.
2.‐ Check the blood glucose level and be prepared to give 4 ounces of juice immediately.
3.‐ Acknowledge his dissatisfaction, offer to obtain a snack, and give the medication.
4.‐ Administer the headache medication and review the day’s lab test results.

1142 A 70‐year‐old client admitted a few hours ago with a Correct answer: 3 Increased preload caused by the intravenous infusion at 250 mL/hr may exceed the The core issue of the question is recognition that rapid infusion of fluid in a 70‐year‐old
blood glucose level of 750 mg/dL is being treated for myocardium’s workload capacity, leading to signs of decreased cardiac output and congestive client could lead to circulatory decompensation. Use nursing knowledge and the process of
hyperosmolar hyperglycemic nonketotic coma (HHNK) heart failure. The other options focus on inappropriate information. elimination to make a selection.
with intravenous regular insulin at 10 units/hour,
normal saline with 40 mEq of potassium per liter
infusing at 250 mL/hr, and oxygen at 2 L/min. The
client is oriented when stimulated, and FBG has
dropped to 400 mg/dL. The client starts demanding to
get out of bed and the nurse notes the skin feels cool
and moist. The nurse should do which of the
following?

1.‐ Interpret this as a sign of hypoglycemia and check his blood glucose.
2.‐ Recognize that client is feeling better and is seeking control of his situation.
3.‐ Auscultate breath sounds and assess oxygen saturation for signs of decreased cardiac output.
4.‐ Assess the client for bladder distention or signs of imbalanced body temperature.

1143 A client who underwent a colonoscopy after being Correct answer: 4 Decreased level of consciousness, weak hand grasp, and peripheral pulses with increased The core issue of the question is accurate interpretation of a change in client status. A
premedicated with midazolam (Versed), returns to the heart rate and decreased BP result from acidosis. These are signs of respiratory acidosis critical word in the stem of the question is midazolam. Recall the properties of this
nursing unit. The client is given morphine sulfate IV secondary to hypoventilation from the midazolam. For this reason, option 4 is the appropriate medication, and use nursing knowledge and the process of elimination to make a selection.
push after reporting abdominal pain associated with diagnostic reasoning process.
BP 140/80, pulse 78, RR 20. Twenty minutes later, the
client is lethargic, has weak hand grasps, weak
peripheral pulse of 88, BP 120/66, RR 14. The nurse
makes which interpretation of these assessment
findings?

1.‐ The client is resting with pain relieved.


2.‐ The client is now showing signs of dehydration because of the colon procedure preparation.
3.‐ The client is now fatigued because of anxiety, pain, and fear of outcome of the procedure.
4.‐ The client is experiencing impaired gas exchange because of hypoventilation.

1144 The client is scheduled for bilateral adrenalectomy Correct answer: 1 During the first 48 hours after adrenalectomy, clients are at risk for adrenal insufficiency and The core issue of the question is knowledge that clients are at risk for adrenal insufficiency
secondary to an adrenal cortex tumor. Which of the hypovolemic shock. The lack of cortisol production can cause fluid and electrolyte loss and following adrenalectomy and how to assess for its occurrence. Use nursing knowledge and
following is the nurse’s highest priority for this client in hypoglycemia. Elevated cortisol does place the client at risk for delayed wound healing and the process of elimination to make a selection.
the immediate postoperative period? infection, but adrenal insufficiency is more life threatening and more common in the first 2
days following surgery.
1.‐ Assess fluid and electrolyte balance, signs of hypoglycemia, and hypotension.
2.‐ Assess for signs of hypoxia, cardiac arrhythmias, and peripheral edema.
3.‐ Monitor the incision integrity, peripheral pulses, and magnesium level.
4.‐ Assess for hyperthermia, bed mobility, pupil reaction and eye movement.

1145 A female client has been taking propylthiouracil (PTU) Correct answer: 1 The client’s complaints of lack of energy and weight gain are consistent with hypothyroidism, The core issue of the question is the ability to correlate client reports with the underlying
for 5 months to treat hyperthyroidism. After falling which is diagnosed with a serum T&lt;sub&gt;4&lt;/sub&gt;. Considering the client’s diagnosis of hypothyroidism. Use nursing knowledge and the process of elimination to
and spraining her ankle, she is treated in the complaints of energy deficit, the recent fall causing the sprain, and information about the make a selection.
emergency department and is given crutch‐walking thyroid medication, the nurse is obligated to consult the physician for
instructions. She states that she will never have T&lt;sub&gt;4&lt;/sub&gt; evaluation to prevent further injury. Encouraging the client to rest
enough energy to get around on crutches and is and investigating the need for a walking splint are appropriate actions but not the first priority.
frustrated at the 10 pounds she gained this winter. The
nurse’s first action should be to

1.‐ Document the client’s complaints and consult the physician to order a serum T4.
2.‐ Discharge the client to home and encourage her to have a TSH level drawn.
3.‐ Encourage the client to rest at home until the sprain is healed, then increase activity.
4.‐ Investigate the availability of a walking splint instead of using the crutches.

1146 The client with diabetes mellitus (DM) is going home Correct answer: 2 Clients with either diabetic mellitus or other conditions that have arterial insufficiency as a The core issue of the question is interpretation of behaviors that pose risk for
following angioplasty. The nurse observes that the component of the disorder must constantly protect their feet from injury; assess the skin complications to the client with diabetes mellitus. Use nursing knowledge and the process
client walks to the restroom barefoot, although condition daily; prevent dry, cracked skin; and avoid crossing the legs in order to maintain of elimination to make a selection.
slippers are in reach. The priority nursing diagnosis for tissue perfusion and prevent infection. These clients have delayed wound healing and poor
this client is which of the following? sensation in their feet, increasing risk for injury and undetected injury with infection.

1.‐ Risk for injury related to potential for falls while walking barefoot
2.‐ Risk for infection related to impaired tissue perfusion and walking barefoot
3.‐ Deficient knowledge related to post angioplasty care
4.‐ Risk for impaired cerebral perfusion related to potential for hypoglycemia

1147 The client is 6 hours post‐thyroid surgery. The nursing Correct answer: 1 Usually, with thyroid surgery, there is minimal bleeding postoperatively. Blood on the gown The core issue of the question is possible threat to the airway and breathing with
assistant reports that the client is upset because there indicates excessive incisional bleeding. Breath sounds, including auscultating over the tracheal excessive bleeding following thyroid surgery. Use nursing knowledge, the ABCs, and the
is blood on the client’s gown. Which of the following is area, and respiratory effort should be assessed first to determine if edema is present in the process of elimination to make a selection.
the priority action of the nurse? tissues, thus compromising the airway. After thoroughly assessing the client and reinforcing or
changing the dressing per protocol, the nurse should inform the surgeon of the amount of
bleeding and all other assessment data. Options 2 and 3 do not protect the client from possible
harm. Option 4 ignores the client’s airway, a high priority following this surgery.

1.‐ Assess the client’s breath sounds and respiratory effort.


2.‐ State that it is normal to have some bleeding and ask the nurse aide to change the gown.
3.‐ Reassure the client that some bleeding is normal, and then assess the client’s level of pain.
4.‐ Reinforce the dressing, change the gown, and call the surgeon.

1148 A client who was admitted with hyperglycemic Correct answer: 3 HHNK is associated with hyperglycemic response to infection or other disease or illness, some The critical word in the stem of the question is prevent. With this in mind, look for the
hyperosmolar nonketotic coma (HHNK) asks how he medications, dehydration, stress‐induced hyperglycemia, or a combination of these factors. option that will reduce the likelihood of the client experiencing a recurrence. Use nursing
can prevent recurrence of this illness. The nurse would HHNK occurs in clients with type 2 diabetes mellitus, primarily the elderly, and thus insulin is knowledge and the process of elimination to make a selection.
instruct the client that which of the following are not part of the usual treatment plan (option 1). Option 2 is insufficient; 6 to 8 glasses of water
helpful prevention measures? are recommended for general health. Option 4 does not demonstrate an understanding of
prevention of HHNK.
1.‐ Use sliding‐scale insulin to cover periodic snack of candy.
2.‐ Maintain fluid balance by drinking 4 glasses of water daily.
3.‐ Detect and treat infection early, maintain hydration, and use stress‐management techniques.
4.‐ Consult primary care provider when fasting blood glucose is elevated.
1149 The client who has a long history of type 1 diabetes Correct answer: 4 Diabetic ketoacidosis can occur in diabetic clients with infection and is characterized by The core issue of the question is recognition that a diabetic client with an infection is at
mellitus is being treated for bronchitis and sinusitis. elevated blood glucose and ketonuria. Deep, rapid, unlabored respirations are called Kussmaul risk for diabetic ketoacidosis and knowing how to assess for this complication. Use nursing
The nurse observes deep, rapid, unlabored respirations. Kussmaul respirations, fruity odor, and dry skin are signs of hyperglycemia. knowledge and the process of elimination to make a selection.
respirations, fruity odor on the client’s clothes, and dry Option 2 represents the opposite problem, not the hyperglycemia being displayed. Options 1
skin. Which of the following actions should the nurse and 3 do not address hyperglycemia and ketoacidosis, which is the issue of the question.
take next?
1.‐ Assess breath sounds for additional signs of response to treatment of the infection.
2.‐ Assess blood glucose level for signs of hypoglycemia.
3.‐ Encourage the client to rest frequently and to drink 8 to 10 glasses of fluids daily.
4.‐ Assess blood glucose level for hyperglycemia and check urine for ketones.

1150 A female client newly diagnosed with hypothyroidism Correct answer: 3 Hypothyroidism is associated with fatigue, weight gain, and decreased activity tolerance. The core issue of the question is the etiology of the client’s symptoms and applying a
indicates that she no longer wants to participate in There is not enough data to conclude decreased cardiac output or sleep alterations. Client nursing diagnostic label to the problem. Use nursing knowledge and the process of
evening social activities stating, “There is too much stated she is able to socialize during the day at work. elimination to make a selection.
walking, and I prefer to go to bed early. I see enough of
my friends at work every day.” The nurse formulates
which of the following as a priority nursing diagnosis
for this client?

1.‐ Social isolation related to sleep rest needs as evidenced by desire to go to bed early
2.‐ Disturbed sleep pattern related to excessive work as evidenced by desire to go to bed early and avoid evening activities
3.‐ Fatigue related to reduced metabolic rate as evidenced by desire to avoid evening activities after work
4.‐ Decreased cardiac output related to weak myocardium as evidenced by desire to avoid walking

1151 A recently retired client who lives alone is admitted Correct answer: 4 Myxedema is characterized by severely decreased cardiac output, fluid and electrolyte The core issue of the question is assigning a priority to client needs during myxedema.
with myxedema coma. It is determined that imbalance, acidosis, decreased respiratory function, tongue edema, and hypothermia. Skin Recall that physiological needs take priority before psychosocial needs. Use nursing
myxedema occurred after she stopped her thyroid breakdown is a significant risk that needs to be managed concurrently with promotion of knowledge about the condition and the process of elimination to make a selection.
medication because she could not afford to buy the oxygenation, but airway and circulation have highest priority.
medication. Which of the following is the highest
priority of the nurse in this client’s plan of care?

1.‐ Assist the client to chair every 4 hours to promote oxygenation and prevent skin breakdown.
2.‐ Prevent injury related to mental confusion and elevated BP.
3.‐ Prevent skin breakdown and promote nutrition with low‐fiber foods.
4.‐ Monitor for signs of decreased cardiac output and airway obstruction.

1152 A client with hyperparathyroidism is admitted to the Correct answer: 4 Hyperparathyroidism causes hypercalcemia. Large doses of saline infusions concurrently with The core issue of the question is knowledge of methods used to manage hypercalcemia in
critical care unit with cardiac dysrhythmias, including Lasix will stimulate a decrease in serum calcium through renal excretion. In acute situations hyperparathyroidism. Use nursing knowledge and the process of elimination to make a
frequent premature atrial contractions, bursts of requiring rapid reduction, clients can be given IV calcitonin and phosphates. selection.
supraventricular tachycardia, and occasional
premature ventricular contractions. The client asks
why the cardiologist prescribed so much IV fluid and
then furosemide (Lasix). The best explanation by the
nurse is that these orders would

1.‐ Improve cardiac output.


2.‐ Eliminate metabolic wastes.
3.‐ Replace missing electrolytes.
4.‐ Promote excretion of calcium.
1153 A client with hypoparathyroidism is to be discharged Correct answer: 2 Clients with hypoparathyroidism have low serum calcium levels, paresthesia, mood disorders, The core issue of the question is health teaching that is appropriate for a client with
home after stabilization of fluid and electrolyte levels. muscle spasms, and hyperactive reflexes placing them at risk for falling. They must actively hypoparathyroidism. Use nursing knowledge about safety measures and altered calcium
Which of the following critical concepts does the nurse seek to increase their intake of calcium and Vitamin D to maintain therapeutic serum levels in levels and the process of elimination to make a selection.
teach the client prior to discharge? addition to taking their prescribed medication. The other options do not address safety as the
critically important need.
1.‐ Importance of keeping follow‐up appointments for lab and with primary health care provider
2.‐ Strategies to prevent falling and how to plan meals high in calcium
3.‐ Significant signs of hypoglycemia to monitor for and report
4.‐ Signs and symptoms of renal calculi and urinary tract infections to monitor for and report

1154 A client recently diagnosed with syndrome of Correct answer: 2 Clients with SIADH are usually on a strict fluid restriction to correct water overload; The core issue of the question is proper fluid use and management in a client with SIADH.
inappropriate antidiuretic hormone (SIADH) is therefore, all fluids (including the enteral feeding and the flush solution) should be considered Use nursing knowledge about fluid imbalance in this disorder and the process of elimination
receiving a continuous enteral nutrition via an enteral when planning the fluid restriction. Clients are also encouraged to drink fluids high in sodium, to make a selection.
feeding tube. Considering the impact of the disorder so clients being treated for SIADH should have their feeding tubes flushed with normal saline
on fluid balance, the nurse does which of the following and not water. To prevent electrolyte loss, all of the residual that is aspirated from a feeding
when working with the enteral feeding tube? tube should be returned to the client.

1.‐ Discard the 50 mL residual and replace it with 50 mL water.


2.‐ Flush the tube with 50 mL normal saline.
3.‐ Count the flush but not the feeding in planning the fluid limitation.
4.‐ Flush the tube with 50 mL water to maintain patency.

1155 A client with a history of Cushing’s syndrome is being Correct answer: 4 The BUN and sodium are elevated because of dehydration and deficient fluid volume, since The core issue of the question is the ability to determine priorities of care for a client with
admitted for acute management of multiple the creatinine is normal, thus supporting normal renal function. The potassium and chloride Cushing’s syndrome who experiences trauma. Use knowledge of pathophysiology and the
contusions and lacerations following a motor vehicle are at the higher end of the normal range, which also supports dehydration and fluid volume process of elimination to make a selection.
accident. Morning serum laboratory values are BUN 30 deficit. Clients with Cushing’s syndrome are at risk for infection because of an impaired
mg/dL, creatinine 1.0 mg/dL, sodium 148 mEq/L, immune function related to an elevated cortisol level.
potassium 4.8 mEq/L, chloride 108 mEq/L, and cortisol
29 mcg/dL. This client’s two high‐priority nursing
diagnoses are

1.‐ Impaired urinary elimination and risk for fluid volume excess.
2.‐ Risk for injury and risk for disuse syndrome.
3.‐ Ineffective airway clearance and ineffective health maintenance.
4.‐ Risk for infection and deficient fluid volume.

1156 A client underwent adrenal gland radiation therapy Correct answer: 1 Florinef and other adrenal replacement drugs cause sodium and fluid retention. Clients are at The core issue of the question is the ability to determine priority concerns for a client
for benign tumors. The client is being treated with risk for excess sodium and fluid retention leading to fluid volume excess. Risk for infection receiving mineralocorticoid and glucocorticoid therapy. Use nursing knowledge and the
fludrocortisone acetate (Florinef Acetate) for could apply but is not timely if the client has completed this course of therapy. Impaired gas process of elimination to make a selection.
mineralocorticoid and glucocorticoid replacement. The exchange may result from extensive fluid volume excess that can lead to ineffective breathing
high‐priority nursing diagnosis for this client is pattern. The highest priority is the risk for fluid volume excess.

1.‐ Risk for excess fluid volume.


2.‐ Risk for infection related to radiation damage.
3.‐ Risk for constipation.
4.‐ Ineffective breathing pattern.
1157 The client is being treated for Addison’s disease with Correct answer: 4 Glucocorticoid replacement medication can cause fluid and sodium retention, leading to The core issue of the question is knowledge of adverse effects of drug therapy following
glucocorticoid replacement medication. The nurse weight gain and fluid volume excess. These medications need to be increased during times of client teaching. Use nursing knowledge and the process of elimination to make a selection.
evaluates that the client understands medication stress and can impair the body’s ability to recover from an infection. Therefore, the physician
therapy when the client makes which of the following must be consulted for weight gain or signs of a cold or infection. These medications should be
statements? taken in the morning with food and will increase BP (thus are not safe for clients with
hypertension), and the medication will not affect cardiac rhythm.

1.‐ “I should take this medication every evening at bedtime.”


2.‐ “My irregular pulse should convert to a regular rate and rhythm.”
3.‐ “This medication will help me control my increased blood pressure.”
4.‐ “I should call my doctor if I gain 2 pounds, feel weak, or have a cold.”

1158 An 8‐month‐old infant born outside the United States Correct answer: 1 A low‐phenylalanine diet reduces the amount of toxic metabolites in the body, thus reducing The core issue of the question is management of PKU in a newly diagnosed infant. Use
is brought to the endocrine clinic with symptoms of a or preventing additional damage. There is no indication of a need to admit the child to a long‐ nursing knowledge and the process of elimination to make a selection.
musty body odor, seizures, and an eczema‐like rash. term care facility, and babies with PKU have normal life expectancy. No medications are
The infant is tested for phenylketonuria (PKU) and the currently being used to treat PKU.
diagnosis is positive. In planning nursing care for this
infant, the nurse will anticipate:

1.‐ The need for dietary information about a low‐phenylalanine diet.


2.‐ Admission to a long‐term care setting for handicapped infants.
3.‐ Preparing the family for the child’s early demise.
4.‐ Providing instruction on medication management of PKU.

1159 A 15‐year‐old weighing 250 pounds has started to Correct answer: 2 Some teens develop type 2 diabetes, especially those who are overweight. They might need The core issue of the question is correct information about medication therapy for
experience increased thirst, increased appetite, and to take an oral hypoglycemic with or without accompanying insulin. Insulin is not used for overweight adolescents with new onset diabetes. Use nursing knowledge and the process
frequent urination. When he is admitted to the those who won’t take oral medication. Sweets and complex carbohydrates still need to be of elimination to make a selection.
hospital, he is given oral medication after being restricted. Option 1 does not offer the information that the child needs about the treatment
diagnosed with diabetes mellitus. What information options.
should the nurse give the teenager about medication
therapy?
1.‐ “You might receive a pill now, but you’ll get insulin in the future if you don’t comply with diet and medication therapy.”
2.‐ “Overweight teenagers may develop type 2 diabetes, which can be treated with an oral medication. You may or may not need insulin in the future.”
3.‐ “Insulin is used when diabetics won’t take oral pills, so you can avoid these by taking your medication as ordered.”
4.‐ “Your diabetes is mild, so you don’t need to take medication for long. You will probably only need to restrict sweets.”

1160 A mother is quite concerned about her 7‐year‐old Correct answer: 2 The child should be seen by the physician because there might be secretion of sex hormones, The core issue of the question is the priority need of a client with suspected precocious
daughter after noticing some breast development and and precocious puberty may affect linear growth. Although she may be teased in school by the puberty. Use nursing knowledge and the process of elimination to make a selection.
the appearance of a small amount of pubic hair. The other children (option 3), the main reason for seeking treatment is health promotion. Options
mother asks the nurse if this is a cause for concern. 1 and 4 are incorrect statements.
What would be the best response by the nurse?

1.‐ “No. Some girls just develop earlier than boys.”


2.‐ “Yes. Your daughter may have precocious puberty. Let’s talk to the pediatrician because she may need referral to an endocrinologist.”
3.‐ “Yes. She probably doesn’t want the other children at school making fun of her.”
4.‐ “No. This early development may slow down when she reaches 9 years old.”
1161 The mother of a diabetic adolescent tells the nurse Correct answer: 1 Adolescents need to feel like part of their group, even if it means impairing their health. The core issue of the question is knowledge of age‐specific concerns of adolescents with
that her son likes to go out with his friends on Friday Displaying risk‐taking behaviors is not likely the primary motivation, but rather a secondary diabetes mellitus. Use nursing knowledge and the process of elimination to make a
nights and eat burgers and french fries. The adolescent event. Option 3 is true but is not likely to be the motivating factor. There is no information to selection.
knows that he is exceeding the allowable carbohydrate support a self‐destructive wish (option 2).
exchanges on the diabetic diet, but he does so anyway.
The nurse discusses with the mother that teens
sometimes take chances that place health at risk
because:
1.‐ They want to be just like their peers.
2.‐ They have a self‐destruction wish.
3.‐ They often like french fries and can’t eat them at home.
4.‐ They want to show risk‐taking behavior.

1162 A 13‐year‐old girl is being evaluated for delayed Correct answer: 1 A karyotype is simply a study of the chromosomes. A blood sample may be used to provide The core issue of the question is appropriate information about a karyotype test. Use
puberty. The client says, “The doctor said he was going the cells for analysis. Options 2 and 3 are incorrect. Option 4 provides no information at all for nursing knowledge and the process of elimination to make a selection.
to do a karyotype. Will that hurt?” Which of the the child and does not address the client’s concern.
following is the best response by the nurse?
1.‐ “A karyotype is just a microscopic picture of your chromosomes.”
2.‐ “The karyotype test is an evaluation of your luteinizing hormone levels.”
3.‐ “The doctor has ordered an x‐ray of your hand to determine your bone age.”
4.‐ “You don’t need to worry about that because I will be with you.”

1163 The nurse is assessing a client with a tentative Correct answer: 2, 3, 5 The client with hyperpituitarism will exhibit the following: tall stature if onset in childhood, The core issue of the question is knowledge of assessment findings with hyperpituitarism.
diagnosis of hyperpituitarism. What assessment large hands and feet with prominent jawbone, joint changes consistent with arthritis, deep Recall the functions of the pituitary gland and then correlate the functions with the logical
findings should the nurse observe for in this client? voice and possible dysphagia, hypertension, organomegaly, and skin changes leading to rough, signs of excess to make the appropriate selections.
Select all that apply. oily texture. The client would not have a soft voice or be short in stature.

1.‐ Short stature if onset is in childhood


2.‐ Large hands and feet with prominent jawbone
3.‐ Joint changes consistent with arthritis
4.‐ Soft, high‐pitched voice
5.‐ Hypertension

1164 The nurse is preparing an 8‐year‐old child for a Correct answer: 1 Visual aids such as dolls, puppets, and outlines of the body can be used to illustrate the cause Critical phrases are "preparing and 8‐year‐old child for a procedure" and "intervention."
procedure. What is the most appropriate nursing and treatment of the child's illness. Use of such equipment provides information to the school‐ Knowledge of coping mechanisms appropriate for the age of the child to deal with illness is
intervention considering the child's stage of growth age child to understand and cope with feelings about the procedure. Written pamphlets should needed.
and development? be given to the parents to review prior to the procedure. Children should be allowed to cry or
verbalize feelings without guilt as long as they hold still. Parents should be given a choice to
accompany their child during the procedure.

1.‐ Provide visual aids, such as dolls, puppets, and diagrams in the explanation
2.‐ Provide a written pamphlet for the child to review prior to the procedure
3.‐ Discourage any display of emotional outbursts
4.‐ Request that the parents wait outside while the nurse provides instructions to the child

1165 A school‐age child has a thyroidectomy performed for Correct answer: 1 A complication of a thyroidectomy is thyroid storm, which can be fatal. Other primary The key concept is the priority nursing activity. A crisis or other critical event would be the
treatment of hyperthyroidism. In planning the concerns would be observing for hemorrhaging, respiratory obstruction, and laryngeal nerve priority.
postoperative care, the priority nursing activity would damage. These would be the priority nursing activities.
be:
1.‐ Observing for signs of "thyroid storm."
2.‐ Beginning treatment with synthetic thyroid hormone.
3.‐ Reassuring the child that the scar will be barely noticeable.
4.‐ Teaching the child about the effects of hyperthyroidism.
1166 After providing a lecture on puberty for 5th‐ and 6th‐ Correct answer: 1, 3, 2, Puberty is a process that brings about the development of secondary sexual characteristics, Critical words are "girls" and "ordering the signs of puberty." Knowledge of puberty is
grade girls, the school nurse asks the group to place 4 which begin with the appearance of breast buds at 9 or 11 years followed by the growth of needed to answer the question correctly.
the secondary sexual characteristics in the order of pubic hair. Menarche follows approximately 1 year later. Following menarche, there is an
their appearance during puberty. The nurse concludes abrupt deceleration of linear growth.
the students learned the information presented if the
students responded with which order to the
characteristics listed? Click and drag the options below
to move them up or down.

1.‐ Appearance of breast buds


2.‐ Occurance of the first menarche
3.‐ Appearance of pubic hair
4.‐ Growth slows

1167 A pre‐adolescent girl is being followed in the Correct answer: 3 The child is expressing concern about her appearance as compared to others, making body The one option that directly relates to the child’s statement is body image disturbance.
endocrine clinic for precocious puberty. The child has image disturbance the best diagnosis. There is no evidence in the stem regarding social Knowledge of the body image changes in precocious puberty will aid in choosing the correct
already developed secondary sexual characteristics isolation or personal identity disturbance. The cause of the precocious position would have diagnosis for this child.
and is taller than her classmates. She complains that been discussed earlier.
because of her height she "stands out and everyone
stares at her." The nurse would formulate which of the
following as an appropriate nursing diagnosis for this
child?
1.‐ Deficient knowledge related to the cause of precocious puberty
2.‐ Social isolation related to height
3.‐ Body image disturbance related to differences from classmates
4.‐ Disturbed personal identity related to height

1168 A 5‐year‐old known diabetic has been admitted with Correct answer: 3 The only form of insulin given IV is regular. Physicians often order Knowledge of the clinical therapy for ketoacidosis will aid in determining the correct
ketoacidosis secondarily to an infection. Which one of D&lt;sub&gt;5&lt;/sub&gt;1/2NS with insulin added providing the child the glucose to meet answer. Recall that the only insulin given IV is regular.
the physician's orders should the nurse question? the body's needs while providing the insulin in the same infusion. Antibiotics would be
appropriate to treat the underlying infection. The blood glucose should be monitored on a
regular basis.
1.‐ IV D51/2NS at 80 mL/hr
2.‐ Add 100,000 units aqueous penicillin to the IV every 6 hours
3.‐ Add 4 units NPH insulin to the IV for every 100 mLs IV fluid
4.‐ Monitor blood glucose levels every 4 hours and prn

1169 A 14‐year‐old boy has just been diagnosed with type 1 Correct answer: 2 Lispro insulin peaks at one hour after administration. A food source should be available at the Consider the onset, peak, and duration of each time of insulin to determine the correct
diabetes mellitus and has been taught to draw up and bedside to prevent the possibility of hypoglycemia shortly after administration. All of the other answer.
administer his own insulin. The nurse arrives at the choices are incorrect.
bedside with the vials for the morning insulin dose:
NPH 15 units and Lispro 8 units; before breakfast
arrives. The client draws up and mixes the insulin in
one U‐100 syringe, but the nurse instructs him not to
inject it just yet for which of the following reasons?

1.‐ Adolescents should not administer insulin to themselves until they have practiced drawing up the medication at least 10 times.
2.‐ Lispro insulin is very fast‐acting insulin. The breakfast tray should be at the bedside before insulin administration.
3.‐ The child's mother should be present to witness the injection.
4.‐ These two forms of medication should never be mixed.
1170 The nurse is meeting with a child recently diagnosed Correct answer: 3, 5 The exchange diet plan is suggested for clients with diabetes mellitus. Even young children Options 1 and 3 are opposites, so consider these options first as possible correct choices.
with type I diabetes mellitus and his family to provide can learn to trade foods in the same exchange category to maintain control. All basic food
diabetic diet counseling. What information should be groups should be included. Diet sodas are allowed. A high fiber diet is recommended for
included in the discussion? Select all that apply. improved control of blood glucose.

1.‐ As long as the child consumes 1,200 calories a day, the food selection doesn't matter
2.‐ To stop eating snacks immediately; eat only three balanced meals a day
3.‐ Foods from all basic food groups are important, but don't overdo simple sugars and carbohydrates
4.‐ Sodas in any form must be avoided; the child should drink water and juices only
5.‐ A high fiber diet is recommended

1171 The nurse in the endocrine clinic is performing an Correct answer: 4, 5 Exophthalmos (bulging eyes) and an enlarged thyroid are evidence of hyperthyroidism. Other Knowledge of the signs and symptoms of hyperthyroidism will help to determine the
intake assessment on an adolescent. The adolescent symptoms would include weight loss, tremors, tachycardia, and elevated basal body correct answer. Eliminate those options that are known to be wrong. The remaining options
demonstrates all of the following symptoms. The nurse temperature. Some children may display behavior problems and have sleeping difficulties. The should be considered individually as to whether the physiology of hyperthyroidism would
would suspect hyperthyroidism based on the presence other symptoms are not associated with hyperthyroidism. cause the problem.
of which of the following? Select all that apply.

1.‐ Acne
2.‐ Dilated pupils
3.‐ Asymmetrical facies
4.‐ Bulging eyes
5.‐ Enlarged thyroid

1172 A woman who wants to get pregnant comes to the Correct answer: 2 Women with PKU should maintain good control prior to becoming pregnant. The fetus may No information is known about the father, so option 4 could not be correct. Options 1 and
clinic for counseling. Her health history includes the have complications if the mother's phenylalanine levels are high. She should also avoid diet 2 are opposites, so consider these statements to determine the correct answer.
information that she was diagnosed with drinks because of the aspartame, which is high in phenylalanine. PKU follows the autosomal
phenylketonuria (PKU) as an infant and remained on a recessive inheritance, and her baby could be affected if the father is a carrier.
low phenylalanine diet through the age of 10. The
nurse will include in the counseling session the
information that:
1.‐ Since she is over the age of 8, she doesn't have to worry about keeping her phenylalanine level low.
2.‐ She should be on a low phenylalanine diet for at least three months prior to becoming pregnant in order to prevent congenital defects in the fetus.
3.‐ Since her condition promotes excessive weight gain, she should stop drinking regular soda and drink diet drinks with aspartame instead.
4.‐ Since this is an autosomal recessive disorder, her infant will not be born with phenylketonuria.

1173 After completing family education for the parents of a Correct answer: 1 Symptoms of excessive fatigue may indicate inadequate medication. Symptoms of overdose Since hypothyroidism results from insufficient thyroid hormones, the medication will
recently diagnosed infant with hypothyroidism, the would include a rapid pulse rate, diarrhea, and weight loss. The other answers indicate correct supply the hormones. Insufficient dosage will result in hypothyroid symptoms, excessive
nurse evaluates the effectiveness of the teaching. The understanding of the management of hypothyroidism. dosage will cause hyperthyroid symptoms. Key words are “will need to provide additional
nurse will need to provide additional instructions if the instructions” which indicates that the parent’s response is incorrect. Therefore, choose the
parents state: answer that indicates a misunderstanding on the part of the parents.

1.‐ "If my child seems excessively tired, I know he will probably need a decrease in his medication."
2.‐ "My child will need to take this medication for the rest of his life."
3.‐ "I'm so glad that he was diagnosed soon after birth so that he will not develop mental retardation."
4.‐ "His tablets can be crushed and mixed with a small amount of his baby cereal."

1174 A 12‐year‐old boy was just diagnosed with Type 1 Correct answer: 2 Children with Type 1 diabetes mellitus must take insulin because they have a total absence of The core concept in this question is the difference between type 1 and type 2 diabetes.
diabetes mellitus. As the nurse teaches him about secretion of insulin from their pancreas. Type 2 diabetes mellitus, which does not make the
insulin injections, he asks why he can’t take the client dependent on insulin, may be associated with some insulin production so the client can
diabetic pills that his aunt takes. What would be the take the oral antidiabetic agents.
best response by the nurse?
1.‐ “You will be able to take the pills once you reach adult height.”
2.‐ “You have a different type of diabetes where the pill won’t work.”
3.‐ “We have to test you to see if you can take the diabetic pills.”
4.‐ “You might be able to switch between taking the pills and insulin.”

1175 An adolescent with diabetes has had several episodes Correct answer: 3 Checking the blood glucose at least twice a day prevents sustained levels of either high or low The more frequently blood glucose is checked, the better the control of diabetes.
demonstrating lack of diabetic control. The nurse is glucose readings. The glycosolated hemoglobin measures long‐term control and is a very Therefore, select the option that provides the most frequent check.
reviewing techniques for checking the control of important value.
diabetes. The nurse states to the adolescent, “The best
way to maintain control of your disease is to:

1.‐ Check your urine glucose three times a week.”


2.‐ Check the glycosolated hemoglobin every 3 months and then every 6 months when stable.”
3.‐ Check the blood glucose twice a day and the glycosolated hemoglobin every 3 months.”
4.‐ Not check anything as long as you feel well.”

1176 A mother attends the pediatric clinic with her 10‐year‐ Correct answer: 1 Long‐term effects of Type 1 DM include retinopathy, heart disease, renal failure, and Knowing that diabetes is a problem with carbohydrate metabolism and that exercise
old daughter who has diabetes mellitus (DM). After peripheral vascular disease. These complications can affect children and adults. The longer the affects this metabolism, options 2 and 4 can be eliminated.
completing the diabetic teaching, the nurse evaluates child lives with diabetes, the greater the likelihood of complications. Exercise increases the
the mother’s knowledge. Which statement by the utilization of glucose, thus an afternoon snack would be very important. Milkshakes would be
mother indicates a satisfactory understanding of concentrated carbohydrates that should be avoided.
diabetes?
1.‐ “I worry about my daughter maintaining control since children with diabetes have more complications than adults do.”
2.‐ “My daughter should drink vanilla milkshakes to maintain a high caloric intake.”
3.‐ “Complications from diabetes could include cataracts and kidney stones.”
4.‐ “My child won’t need a mid‐afternoon snack since she takes a gym class in the afternoon.”

1177 Considering a child’s developmental level in diabetic Correct answer: 4, 5 The toddler needs to feel some control. Cleaning off his fingers with alcohol, with supervision, Consider the growth and development of the toddler to determine which activity can
care is essential. The nurse should include which will allow some control. Another way to promote control would be for the toddler to choose safely be assumed by a toddler.
information in teaching the parents of a recently food selections from options offered. It is inappropriate to allow the toddler to assist with
diagnosed toddler with diabetes? Select all that apply. injections and it is unnecessary to test glucose every time the toddler goes out to play.

1.‐ Allow the toddler to assist with the daily insulin injections.
2.‐ Prepare meat, vegetables, and potatoes for each dinner. The toddler cannot be allowed many choices in food selection.
3.‐ Test the toddler’s blood glucose every time he goes outside to play.
4.‐ Allow the toddler to assist with cleaning off his fingers before blood glucose monitoring.
5.‐ Allow the toddler to choose food selections from options offered.

1178 A 2‐month‐old infant arrives at the pediatric clinic. Correct answer: 2, 5 Most babies with congenital hypothyroidism exhibit bradycardia, protruding tongue, and Open fontanels are expected in a 2‐month‐old infant, so that is not a symptom of
Upon assessment, the baby exhibits the following hypotonia. Open fontanels are normal for a 2‐month‐old infant. hypothyroidism. Hypertonia and tachycardia would both be symptoms of hyperthyroidism.
characteristics. Which characteristic does the nurse
relate to a diagnosis of congenital hypothyroidism?
(Select all that apply.)
1.‐ Open fontanels
2.‐ Protruding tongue
3.‐ Tachycardia
4.‐ Hypertonia
5.‐ Hypotonia
1179 A 10‐year‐old girl comes to the office of the school Correct answer: 1 Exercise makes the body more sensitive to insulin, thus metabolizing the glucose faster. While First decide if the symptoms the child is displaying are hypo‐ or hyperglycemia. Knowledge
nurse after recess. This is the child’s first day back in hospitalized, the child was less active. Now that the child has returned to normal activity, it is of the effect of exercise on glucose metabolism will aid in choosing the correct answer.
school after hospitalization, where she was diagnosed possible that the insulin dose is too high or more glucose is required in the diet. The other Since the symptoms are hypoglycemic, there is only one option appropriate.
with diabetes. The child reports she took the dose of options are inaccurate.
insulin as instructed and that it was the same as she
took while hospitalized. The nurse notices that she is
nervous with hand tremors present. She is pale,
sweaty, and complaining of sleepiness. The school
nurse would suspect:

1.‐ Exercise‐induced hypoglycemia.


2.‐ Hyperglycemia caused by increased intake at lunch.
3.‐ Ketoacidosis caused by an infection.
4.‐ The child is avoiding returning to class.

1180 After being diagnosed with Graves’ disease, a Correct answer: 3 Lethargy may indicate an overdose of the drug, causing the child to exhibit signs of Graves’ disease is hyperthyroidism. Recall that the drug is used to suppress the thyroid.
teenager begins taking propylthiouracil (PTU) for hypothyroidism. The other signs indicate signs of hyperthyroidism. Oversuppression would be a symptom of too much drug.
treatment of the disease. What symptom would
indicate to the nurse that the dose may be too high?

1.‐ Weight loss


2.‐ Polyphagia
3.‐ Lethargy
4.‐ Difficulty with schoolwork

1181 A 13‐year‐old boy is being evaluated for delayed Correct answer: 1 An adolescent client with delayed puberty may need to talk about issues of low self‐esteem. If Consider which response would be therapeutic to determine the correct answer.
puberty. He has had an examination with a pediatric he has a constitutional delay, puberty will usually follow with time. Hormone therapy is not
endocrinologist who states that the child has a given until after the age of 14.
constitutional delay. An appropriate communication to
the child by the nurse that reinforces the physician’s
explanation of the diagnosis would be:

1.‐ “All of your hormone levels are normal, so no medication is needed at this time. If you want to talk about it, I would be happy to discuss it with you.”
2.‐ “I am worried about your stature. I think you should get another opinion.”
3.‐ “Your father’s stature doesn’t matter. We just look at your height.”
4.‐ “If you want testosterone shots, I will arrange for them to be given.”

1182 A child demonstrated a sudden onset of Correct answer: 2 Propranalol, a beta‐adrenergic blocking agent, provides relief from adrenergic Consider which drug would promote symptoms opposite to excessive thyroid hormone.
thyrotoxicosis. The nurse anticipates that, besides hyperresponsiveness. It is usually needed for 2 to 3 weeks along with antithyroid hormone
antithyroid therapy, the child is likely also to receive therapy.
which of the following types of drugs?

1.‐ Antacid
2.‐ Beta‐adrenergic blocker
3.‐ Muscle relaxant
4.‐ Cardiac glycoside

1183 Four newborns have blood drawn for the Guthrie test Correct answer: 4 The screening is done only after an adequate amount of protein has been ingested. Breast Knowing that 24 hours of formula/breast milk are required to provide adequate test
for phenylketonuria. The nurse would question the milk and formula meet the requirements. The testing is usually done at 48 hours of age. results, eliminate any choice that would not provide this.
results of the baby:
1.‐ Whose test is performed at 48 hours of age.
2.‐ Who was breast‐fed for the 24 hours before the test.
3.‐ Who was fed glucose water followed by formula for 30 hours.
4.‐ Who was tested immediately after birth.

1184 A mother of a 4‐month‐old tells the nurse that her Correct answer: 2, 3, 4 Congenital hypothyroidism in infants is diagnosed due to hypotonicity and hypoactivity. The Think of the baby with hypothyroidism as lethargic and consider which symptoms might be
child has been diagnosed with hypothyroidism. The infants are often described as a “good baby” because they rarely cry. Prolonged jaundice, seen in a lethargic baby.
mother asks the nurse what symptoms led to the constipation, and umbilical hernia are common findings in hypothyroidism.
diagnosis. The nurse explains that which of the
following symptoms are consistent with this diagnosis?
(Select all that apply.)

1.‐ High‐pitched shrill cry


2.‐ Prolonged jaundice at birth
3.‐ Described as a “good baby”
4.‐ Constipation
5.‐ Tall for gestation age at birth

1185 An infant was born 24 hours ago. The nurse has been Correct answer: 2 Tests done 24 to 48 hours after delivery may be interpreted as high because of the rise in TSH The learner should be familiar with common screening blood tests. With that knowledge,
instructed to collect blood by heel stick for neonatal that occurs immediately after birth. options 3 and 4 can be eliminated. Then determine whether the test response would be
screening for congenital hypothyroidism before the high or low after birth.
baby is discharged. The nurse questions the order with
the pediatrician because 24 to 48 hours after birth is
not the optimal time to collect this specimen for what
reason?

1.‐ At 24 hours, the T4 level will be extremely low.


2.‐ There is an immediate rise in the TSH after birth.
3.‐ The baby needs to digest formula before a blood sample can be taken.
4.‐ A thyroid scan should be done first.

1186 The nurse is administering propylthiouracil (PTU) to a Correct answer: 3 Sore throat and enlarged cervical nodes are common side effects of the medication. A dosage Knowledge of the side effects of this drug is the core concept of this item.
12‐year‐old recently diagnosed with Graves’ disease. reduction or withdrawal of the drug should be considered.
The child has been receiving the drug 3 times a day for
3 weeks. She suddenly complains of a severe sore
throat. What would be the appropriate nursing action?

1.‐ Continue to give the medication or she will continue to exhibit signs of Graves’ disease.
2.‐ Offer lozenges for the relief of the sore throat.
3.‐ Withhold the dose and report this to the physician since a sore throat is a common side effect.
4.‐ Question whether she is complaining to avoid going to the school room in the hospital.

1187 A 10‐year‐old diabetic client tells the school nurse Correct answer: 2 If a child exhibits signs of hypoglycemia, a source of sugar like orange juice can elevate Since the child is showing signs of hypoglycemia, the nurse will recommend action to
that he has some early signs of hypoglycemia. The glucose levels and prevent further signs of hypoglycemia. A 10‐year‐old must remember to increase the blood glucose level.
nurse recommends to the child that the child: only take one serving and wait ten minutes for symptoms to be alleviated.

1.‐ Take an extra injection of regular insulin.


2.‐ Drink a glass of orange juice.
3.‐ Skip the next dose of insulin.
4.‐ Start exercising.
1188 The nurse is teaching a 15‐year‐old client about the Correct answer: 4 The peak action of NPH or Lente insulin is 6 to 12 hours after administration subcutaneously. Add 6 to 12 hours to 8 am. That computes to be 2 to 8 pm. Only one response falls during
different types of insulin. The client takes NPH insulin During peak times, the client may need a snack to offset potential hypoglycemia. that time frame.
at 8:00 a.m. The nurse interprets that the adolescent
understands this type of insulin when the child states
that the most likely time for an insulin reaction would
be:
1.‐ While working out at 9:00 am.
2.‐ While taking a test at 10:00 am.
3.‐ While eating lunch at noon.
4.‐ While golfing after school at 3 pm.

1189 A teenage mother arrives at the clinic with her new Correct answer: 3 Since hypothyroidism is a lifelong condition, the levothyroxine will need to be taken Options 1 and 3 discuss how to administer the medication in relation to the amount of
baby who has recently been diagnosed with congenital indefinitely. It is important that the infant takes the medication in a small amount of food or formula. One of these is probably the right response.
hypothyroidism. When instructing the mother about liquid and not placed in the bottle since he/she may not receive the full dose if the entire
administering levothyroxine medication, the nurse bottle is not consumed.
would include the information that she should:

1.‐ Crush the medication and place it in a full bottle of formula or juice to disguise the taste.
2.‐ Administer the medication every third day.
3.‐ Give the crushed medication in a syringe or in the nipple mixed with a small amount of formula.
4.‐ Understand that the medication will not be needed after age 5.

1190 A new mother of an infant diagnosed with Correct answer: 1 In each pregnancy, there is a 25% chance of the child having the disease, a 50% chance that The key concept is autosomal recessive inheritance.
phenylketonuria (PKU) meets with the nurse who the child will be a carrier of the gene, and a 25% chance that the child will be unaffected. PKU
informs her that PKU follows autosomal recessive affects both sexes equally.
inheritance. The mother states that this is a relief since
she now knows her next baby will not have the
disease. What additional information does the mother
need?
1.‐ With autosomal recessive inheritance, each baby has a 25 percent chance of having the disease.
2.‐ Only female babies will have PKU.
3.‐ The mother passes the gene only to male offspring.
4.‐ Since she already has one baby with the disease, the next one will probably be a carrier for the disease.

1191 A 4‐month‐old infant has been diagnosed with Correct answer: 3 Decreased levels of tyrosine cause a deficiency of the pigment melanin, causing most children The learner must know common symptoms of PKU and be able to link the symptoms to
phenylketonuria (PKU). The child has eczema and with PKU to have blond hair, blue eyes, and fair skin that is prone to eczema. the disease process.
sensitivity to sunlight. The mother asks the nurse why
her child’s skin is so sensitive. An appropriate
explanation by the nurse would be:
1.‐ "Some children just have sensitive skin. There is no reason to be excessively concerned."
2.‐ “Your child will outgrow his sensitivity when he is 5 years old. Just use sunscreen for now.”
3.‐ “Your child has a deficiency in melanin because of decreased tyrosine. You will always have to take special care of his skin.”
4.‐ “The phenylketones in your baby’s blood concentrate the sun’s rays, making burning more likely. Children with PKU can never be in the sun.”

1192 The nurse was working with a group of parents of Correct answer: 3 Foods with low phenylalanine levels include vegetables, fruits, juices, and some cereals and Eliminate any meal choice that has a lot of protein, especially meat and dairy products.
children with phenylketonuria. The nurse has breads. The amount of protein in the diet is restricted based on phenylalanine blood levels.
completed family teaching on the dietary restrictions.
The parents are given sample menus to choose a meal
for their child. Which menu choice indicates
understanding of the dietary instructions?

1.‐ A hamburger and a diet soda sweetened with aspartame


2.‐ Steak and mashed potatoes with orange juice
3.‐ A large bowl of dry cereal with strawberries and apple juice
4.‐ Milkshakes and grilled cheese sandwich

1193 Mothers in the waiting room of the endocrine clinic Correct answer: 1 Keeping the levels of phenylalanine at a low level in children with PKU and daily Review the disease process of PKU and hypothyroidism. There is a special diet in PKU, but
are discussing their children’s illnesses. The nurse administration of levothyroxine in children with congenital hypothyroidism will decrease the not in hypothyroidism. Hypothyroidism is treated with medication, but PKU is not. Both
determines that the mothers of children with incidence of mental retardation by allowing normal brain growth. diseases may lead to mental retardation if left untreated.
phenylketonuria (PKU) and congenital hypothyroidism
recognize a common goal in the early treatment of
their children when they state they are hoping to
avoid:
1.‐ mental retardation.
2.‐ fever.
3.‐ obesity.
4.‐ protein foods.

1194 The nurse is obtaining a health history on a 36‐year‐ Correct answer: 2 Hyperthyroidism is an excess of thyroid hormone (TH), which places the body in a This question is answered best by using the process of elimination. First consider the organ
old female who reports an increase in appetite, weight hypermetabolic state manifested by increases in appetite, body temperature, and oxygen function which is reflected in the symptoms, in this case metabolism. That leads
loss, intolerance to heat, and nervousness. On physical consumption. Hypothyroidism manifestations are the opposite of those seen in consideration of the thyroid function. Then consider whether the question refers to hyper‐
assessment, the client is noted to have thin hair and hyperthyroidism. The manifestations of parathyroidism are related to disturbances in calcium or hypo‐ functioning in order to select the correct answer.
moist skin. Based on this information, the nurse would levels.
suspect which of the following?

1.‐ Hypothyroidism
2.‐ Hyperthyroidism
3.‐ Hypoparathyroidism
4.‐ Hyperparathyroidism

1195 A client is returning from a subtotal thyroidectomy Correct answer: 1 Though fluid volume status, neurological status, and pain are all important assessments, the This question reinforces the priority of a patent airway. Since the thyroid gland is located
for the treatment of hyperthyroidism. The immediate immediate priority for postoperative thyroidectomy is airway management. Respiratory in the neck, next to the trachea and upper airway structures, post‐operative edema may
priority in assessing this client would include which of distress may result from hemorrhage, edema, laryngeal damage, or tetany. Assessment of lead to respiratory distress.
the following? respiratory status should include rate, depth, rhythm, and effort.

1.‐ Assess for respiratory distress.


2.‐ Assess fluid volume status.
3.‐ Assess neurological status.
4.‐ Assess for pain.

1196 A client with hypothyroidism is taking levothyroxine Correct answer: 3 For best absorption, thyroid medications should be taken 1 hour before meals or 2 hours Questions which ask about client teaching often ask to recount knowledge of material.
sodium (Synthroid), a thyroid replacement hormone. after meals. Lifelong treatment of hypothyroidism is necessary. Foods that inhibit thyroid Recall that Synthroid is best absorbed on an empty stomach, similar to some other
Which of the following statements made by the client hormone (TH) synthesis, such as cabbage, spinach, and carrots, should not be consumed in medications.
would indicate additional teaching is required? excessive amounts. Thyroid medications should be taken in the morning to reduce the
possibility of insomnia.
1.‐ "I know I will be on this medication for the rest of my life."
2.‐ "I don't eat excessive amounts of cabbage or spinach."
3.‐ "I take my Synthroid with food."
4.‐ "I take my Synthroid in the morning."

1197 In providing care for a client being admitted for Correct answer: 2 Hypercalcemia is the primary complication of hyperparathyroidism, and the manifestations of Note that this client is already diagnosed with hyperparathyroid disease so the question
hyperparathyroidism, the nurse anticipates the disorder are directly related to the effects of hypercalcemia. Administering large amounts asks about treatment. Option 2 is the best treatment choice listed.
implementing which of the following actions? of intravenous saline promotes renal excretion of calcium. Calcium gluconate would increase
serum calcium levels, and tetany is a symptom of hypocalcemia.
1.‐ Administering intravenous calcium gluconate
2.‐ Administering large amounts of intravenous saline
3.‐ Maintaining strict fluid restriction
4.‐ Monitoring for tetany

1198 The nurse evaluating a client receiving supplemental Correct answer: 3 The normal serum calcium level is 8.8 to 10 mg/dL. The therapeutic response of supplemental This question calls upon memorized normal ranges for calcium. Note that this is an
calcium treatment for hypoparathyroidism knows that calcium is demonstrated by normal calcium levels. element commonly asked about on NCLEX&lt;sup&gt;®&lt;/sup&gt;.
the client has achieved therapeutic effects of the
calcium supplement when which of the following
serum calcium levels is obtained?

1.‐ 5.6 mg/dL


2.‐ 12.0 mg/dL
3.‐ 9.0 mg/dL
4.‐ 7.0 mg/dL

1199 A client with Cushing's syndrome is admitted with the Correct answer: 2 Cushing's syndrome is manifested by sodium retention, which leads to edema and The symptom edema provides the hint for the issue of fluid volume excess. Recall that
symptoms of hypertension, fatigue, and edema. The hypertension. Fluid volume excess is the appropriate diagnosis. Treatment is aimed at physiological needs take priority.
priority nursing diagnosis for this client would be which restoring normal body fluid balance. Anxiety and knowledge deficit should be addressed
of the following? following fluid volume excess.
1.‐ Fluid volume deficit
2.‐ Fluid volume excess
3.‐ Anxiety related to lack of knowledge
4.‐ Knowledge deficit

1200 A client with Conn's syndrome (hyperaldosteronism) Correct answer: 4 Hypertension and hypokalemia are the most common signs and symptoms of The question calls for knowledge of the disease and of medications to deal with side
who will not be treated surgically is receiving hyperaldosteronism. Surgical removal of the adrenal gland(s) is the treatment of choice; effects from over functioning of the organs, in this case the adrenals.
spironolactone (Aldactone). The nurse explains the however, if that is not possible, the client is treated with Aldactone, a potassium‐sparing
purpose of this drug to the client as being which of the diuretic, to treat the hypertension and correct the hypokalemia.
following?

1.‐ To reverse the hyperaldosteronism


2.‐ To decrease the serum potassium level
3.‐ To promote fluid retention
4.‐ To treat hypertension and hypokalemia

1201 A client with Addison's disease is being discharged Correct answer: 3 Weight must be monitored daily; any increase indicates fluid retention and should be Recognize that questions asking for further instruction are looking for an incorrect answer,
home and will be taking hydrocortisone (Cortisol). The reported immediately. Corticosteroids are immunosuppressants; therefore, careful monitoring in other words something which is wrong requiring continued teaching.
client requires further instructions about this for infection is necessary. Additionally, an increase in the medication may be required for
medication when which of the following statements is stressors such as infection. A Medic‐Alert bracelet is recommended to inform healthcare
made? providers of Addison's disease and cortisol treatment. Safety measures are encouraged to
prevent injuries.
1.‐ "I will monitor closely for any signs of infection."
2.‐ "I will wear a Medic‐Alert bracelet indicating disease and treatment."
3.‐ "I will report any rapid weight gain or fluid in my legs if it persists for over 1 week."
4.‐ "I will take safety measures at home to prevent injuries."

1202 The nurse is establishing a plan of care for a client Correct answer: 3 SIADH results in fluid retention and hyponatremia. Correction is aimed at restoring fluid and Recall that too much ADH (antidiuretic hormone) causes fluid retention leading to fluid
newly admitted with syndrome of inappropriate electrolyte balance. Anxiety and risk for injury should be addressed following fluid volume volume excess.
antidiuretic hormone secretion (SIADH). The priority excess.
diagnosis for this client would be which of the
following?
1.‐ Fluid volume deficit
2.‐ Anxiety related to disease process
3.‐ Fluid volume excess
4.‐ Risk for injury

1203 The nurse is discussing the treatment regimen for a Correct answer: 1 Type 1, or insulin‐dependent diabetes, requires lifelong replacement of insulin, because no This question requires knowledge of Type 1 diabetes. Recognize that the other three
client newly diagnosed with Type 1 diabetes mellitus. insulin is produced from the beta cells of the pancreas. Options 2, 3, and 4 are incorrect for options are not true for Type 1 diabetes.
During the discussion of insulin administration, the Type 1 diabetes.
client asks, "Why can't I just take a pill like my friend
does?" Which of the following statements indicates
the client understands the nurse's explanation?

1.‐ "My body does not produce insulin, therefore I must receive the injections."
2.‐ "I will be on insulin for a short while, then I can take the pills."
3.‐ "The pills are not as effective as the insulin injections."
4.‐ "When my body starts making insulin again, I can stop taking the injections."

1204 A client with exophthalmos as a result of Graves' Correct answer: 3 Exophthalmos occurs as a result of accumulation of fat deposits and by‐products in the retro‐ This question requires knowledge of the composition of the exopthalmos, being fatty build
disease has expressed a desire for the medications to orbital tissues. Even with treatment of Graves' disease, these changes are not reversible. The ups which may not resolve.
"hurry up and work so that my eyes will go down." The client should receive instructions on proper eye care.
nurse's response to the client will be based on which
of the following?
1.‐ Reversal of exophthalmos occurs after a therapeutic level of the antithyroid medication is achieved.
2.‐ Reversal of exophthalmos occurs after treatment with ophthalmic medications.
3.‐ Changes in the eyes as a result of Graves' disease are not reversible, even after treatment of the disease.
4.‐ Exophthalmos as a result of Graves' disease is only a temporary symptom, and should resolve spontaneously.

1205 The nurse is providing care to a client with myxedema Correct answer: 3 Myxedema coma is a life‐threatening crisis manifested by hypothermia, hyponatremia, Recall that airway and other physiological needs have high priority.
coma. Priority nursing care would include which of the hypoglycemia, lactic acidosis, cardiovascular collapse, and coma. Maintaining airway and
following measures? circulation are the priority interventions.
1.‐ Decrease body temperature.
2.‐ Decrease heart rate.
3.‐ Maintain airway, fluid and electrolyte balance, and cardiovascular status.
4.‐ Decrease blood pressure.

1206 A client diagnosed with primary hyperparathyroidism Correct answer: 1 The treatment for primary hyperparathyroidism is a parathyroidectomy (surgical removal of In this question look for a correct answer to evaluate the effectiveness of teaching and
demonstrates that she understands the teaching plan parathyroid glands). Options 2, 3, and 4 are incorrect treatments for primary learning.
when the client makes which of the following hyperparathyroidism.
statements?
1.‐ "I know I must have surgery to remove my parathyroid gland."
2.‐ "I must take diuretics the rest of my life."
3.‐ "I must eat a diet low in potassium."
4.‐ "I must limit my daily fluid intake."

1207 The priority nursing diagnosis for a client with Correct answer: 2 Risk for injury related to hypocalcemia is the priority diagnosis as injury may occur as a result Remember safety is a priority, as is prevention of injury.
hypoparathyroidism would be which of the following? of low calcium levels and tetany. The client is at risk for fluid volume deficit, not excess, and
anxiety and knowledge deficit would not take priority over injury.

1.‐ Risk for fluid volume excess


2.‐ Risk for injury
3.‐ Anxiety related to lack of knowledge
4.‐ Knowledge deficit
1208 A client with Cushing's syndrome is receiving Correct answer: 3 The therapeutic effects of mitotane are the results of direct suppression of activity of the Recall that cytotoxic drugs cause cell death; death of the cells of the adrenal cortex will
mitotane (Lysodren), a cytotoxic antihormonal agent. adrenal cortex. Modrastane blocks the synthesis of glucocorticoids (option 1), octreotide cause a decrease in function.
When assessing the client's response to the suppresses ACTH (option 2), and radiation destroys the pituitary gland (option 4).
medication, the nurse would expect therapeutic
effects to be the result of which of the following?

1.‐ Blocking the utilization of glucocorticoids


2.‐ Suppression of adrenocorticotropic hormone (ACTH)
3.‐ Direct suppression of activity of the adrenal cortex
4.‐ Destruction of the pituitary gland

1209 The health education nurse has provided Correct answer: 1 Since the client will have a bilateral adrenalectomy, lifetime corticosteroid replacement is Recall that once an organ is removed, lifetime replacement is required to fill that function.
preoperative teaching for a client with Conn's necessary. After the adrenalectomy, the client's aldosterone levels should return to normal;
syndrome who is scheduled for a bilateral therefore, no dietary restrictions will be necessary.
adrenalectomy. The client understands the teaching
when which of the following statements is made?

1.‐ "I will need to be on lifetime replacement of glucocorticoids."


2.‐ "I will need to increase my salt intake."
3.‐ "I need to avoid salt the rest of my life."
4.‐ "I will need temporary replacements of glucocorticoids."

1210 A client with a history of Addison's disease is Correct answer: 1 Hydrocortisone is given to replace cortisol in the client with adrenal insufficiency. Abrupt In this question the medications provide replacement of organ function. Recognize that if
admitted to the unit with Addisonian crisis manifested withdrawal of the hormone can precipitate Addisonian crisis. Florinef is given to replace the medications are stopped, symptoms of the disease recur.
by severe hypotension and nonresponsiveness. Which mineralocorticoids. Hyponatremia is caused by aldosterone deficiency, which affects the renal
of the following statements provided by the client's tubules ability to conserve sodium; therefore adding salt to the diet is recommended. Insulin‐
spouse would the nurse suspect precipitated the dependent diabetes is a complication of Addison's disease; however, there is no indication that
crisis? diabetes precipitated the crisis.
1.‐ The client stopped taking the prescribed hydrocortisone several days ago.
2.‐ The client routinely adds salt to meals.
3.‐ The client is taking fludrocortisone (Florinef).
4.‐ The client is an insulin‐dependent diabetic.

1211 The nurse administering vasopressin (Pitressin) to a Correct answer: 2 Because of a deficiency in antidiuretic hormone, diabetes insipidus results in massive diuresis Recall that diabetes insipidus is a disease of not enough ADH, therefore there is an
client with diabetes insipidus anticipates implementing and dehydration. Vasopressin (antidiuretic hormone) is administered to promote fluid increased urine output. Assessing urine output is an important way to assess fluid status.
which of the following actions for this medication? retention and achieve fluid balance. Oral fluids are encouraged, and hypotonic fluids are
administered.
1.‐ Maintaining fluid restriction
2.‐ Monitoring urine output
3.‐ Administering intravenous hypertonic fluids
4.‐ Maintaining NPO status

1212 Priority nursing management of the client with Correct answer: 1 Hypertension with systolic blood pressures reaching up to 300 mmHg is possible with Recognize that safety is a high priority in this problem. Significant hypertension must be
pheochromocytoma would include monitoring which pheochromocytoma, making this disorder a life‐threatening event. Monitoring blood pressure treated.
of the following? is a priority. Urine output and neurological status would follow blood pressure, and there is no
indication to monitor glucose levels. Treatment of choice for this disorder is an adrenalectomy.

1.‐ Blood pressure


2.‐ Urine output
3.‐ Neurological status
4.‐ Serum glucose levels
1213 The nurse knows that a client newly diagnosed with Correct answer: 4 The client should inform the healthcare provider of illness, and then should follow "sick‐day The process of elimination is helpful with the three options that are correct. Choose the
insulin‐dependent diabetes (Type 1) will require rules" as prescribed by healthcare provider, which include taking insulin as prescribed, or incorrect answer.
further teaching when which of the following increasing insulin as prescribed, consuming extra fluids, resting, and monitoring glucose every
statements is made? 2 to 4 hours. Options 1, 2, and 3 are all correct responses by the client.

1.‐ "I will notify my healthcare provider if my glucose levels run higher or lower than the target range."
2.‐ "I will take my insulin as prescribed, and I will not miss a dose."
3.‐ "I will check my glucose level 30 minutes before I eat and at bedtime."
4.‐ “I will not take my insulin if I am sick and cannot eat.”

1214 While administering an iodine preparation Correct answer: 1 Iodine reduces the size and vascularity of the thyroid, reducing the risk of hemorrhage, which This question calls for knowledge of the relationship of iodine with thyroid function which
preoperatively to a client who is scheduled for a is a potential complication of thyroidectomy. Antithyroid medications, not iodine, are given to may be transferred to the need for iodine treatment.
thyroidectomy to treat hyperthyroidism, the client reduce hormone comprehension levels. The treatment for cancer of the thyroid is a total
asks the nurse to explain the purpose of the iodine. thyroidectomy.
The best explanation is based on which of the
following?
1.‐ Iodine decreases the vascularity and size of the thryoid gland, thereby reducing the risk of intraoperative and postoperative hemorrhage.
2.‐ Iodine reduces circulating hormone levels, thereby reducing the effects of elevated hormones.
3.‐ Iodine is recommended in the treatment of cancer of the thyroid.
4.‐ Iodine enlarges the thyroid gland and facilitates the surgeon's ability to locate the thyroid.

1215 The nurse knows that a client with hyperthyroidism Correct answer: 2 Hyperthyroidism causes a hypermetabolic state, resulting in increased body temperature This question asks for correlation of the symptoms of hypothyroid with the environmental
understands the discharge teaching instructions about causing the client to have heat intolerance, and sensitivity to noise and loud sounds. A cool, modifications which may accommodate to those.
the management of hyperthyroidism when the client quiet environment is recommended. Clients with hyperthyroidism require an increased, not a
repeats which of the following instructions? decreased, caloric consumption.

1.‐ "I will consume a low‐calorie diet."


2.‐ "I will keep the environment cool and quiet."
3.‐ "I will keep the environment warm."
4.‐ "I will maintain an environment with constant stimulation."

1216 Further assessment by the RN of postoperative Correct answer: 3 The thyroid gland is highly vascular, therefore there is a potential risk of postoperative This question requires use of the process of elimination to look for the incorrect option.
hemorrhaging in the thyroidectomy client is required hemorrhage. The client should be thoroughly assessed for hemorrhage behind the dressing, as
when the student nurse makes which of the following blood may drain and run back and under the client. Options 1, 2, and 4 are correct assessments
statements? for hemorrhage.
1.‐ "Blood pressure and pulse are equal to baseline measurements."
2.‐ "The client denies complaints of dressing tightness."
3.‐ "The dressing is dry and intact; therefore, no signs of bleeding are present."
4.‐ "No signs of respiratory distress are noted."

1217 While performing an assessment on a client with Correct answer: 1 Muscle twitching when pressure is applied in taking a BP is called a positive Trousseau's sign, This question uses opposites; ensure that both options in the answer are correct while
hypoparathyroidism, the nurse notes that the client is which indicates tetany in a client with hypoparathyroidism. using the process of elimination.
experiencing muscle twitching when the blood
pressure is taken. Based on this finding, the client is
most likely experiencing which of the following
imbalances?
1.‐ Hypocalcemia
2.‐ Hypercalcemia
3.‐ Hyperkalemia
4.‐ Hypokalemia
1218 Upon initial admission of a client diagnosed with Correct answer: 2 Metabolic acidosis is the alteration in acid‐base balance with DKA. This question draws on Reading of ABGs requires use of the process of elimination, assessing the pH, and then
diabetic ketoacidosis (DKA), the nurse notices a fruity your knowledge of ABG values. The results show uncompensated metabolic acidosis because looking at the CO&lt;sub&gt;2&lt;/sub&gt; and HCO&lt;sub&gt;3&lt;/sub&gt;&lt;sup&gt;‐
odor to the breath, Kussmaul's respirations, and the question asked about the initial admission. Partially compensated (option 1) or fully &lt;/sup&gt; to determine the correct one for metabolic acidosis.
lethargy. The glucose level is 700 mg/dL, positive compensated (option 3) values would be expected later. Option 4 indicates respiratory
ketones in the urine and the family member states acidosis.
they have not been able to afford the insulin. Which of
the following results would be expected on ABGs?

1.‐ pH 7.25; CO2 33 mmHg; HCO3‐ 22 mEq/L


2.‐ pH 7.25; CO2 36 mmHg; HCO3‐ 22 mEq/L
3.‐ pH 7.38; CO2 33mmHg; HCO3‐ 22 mEq/L
4.‐ pH 7.25; CO2 48mmHg; HCO3‐ 29 mEq/L

1219 A client with hypothyroidism is given dietary Correct answer: 2 Constipation is a potential complication of hypothyroidism as a result of decreased gastric This question provides opposites; determine if fluid restrictions or forcing fluids is correct.
instructions. After the instructions, the nurse motility. Instructing client to consume at least 2,000 mL of fluid a day, unless contraindicated,
determines that further instructions are needed when increase fiber intake, and maintain a well‐balanced diet will help promote bowel elimination.
the client makes which of the following statements? Options 1, 3, and 4 are appropriate responses.

1.‐ "I will increase fiber intake in my diet."


2.‐ "I will limit my fluid intake to 1,000 mL a day."
3.‐ "I will eat a well balanced diet."
4.‐ "I will increase my fluid intake to 2,000 mL a day."

1220 Which of the following laboratory values would Correct answer: 3 The normal values for T3 (triiodothyronine) are 80 to 200 ng/dL, and the normal values for T4 Recall knowledge of normal ranges of diagnostic tests.
indicate a therapeutic effect of levothyroxine sodium (thyronine) are 5 to 12 &amp; µg/dL. Both of these values are decreased in hypothyroidism.
(Synthroid) for a client with hypothyroidism? With treatment of thyroid medications resulting in therapeutic effects, the thyroid levels
should be normal. The other values are too low and do not indicate therapeutic effects have
been achieved.
1.‐ T3 50 ng/dL, T4 2 µg/dL
2.‐ T3 50 ng/dL , T4 10 µg/dL
3.‐ T3 100 ng/dL, T4 7 µg/dL
4.‐ T3 100 ng/dL, T4 2 µg/dL

1221 In performing an assessment on a client on the fourth Correct answer: 2 The most common cause of hypoparathyroidism and the resulting hypocalcemia is damage to This question asks for understanding of the signs of low serum calcium levels and the role
postoperative day following thyroidectomy, the nurse the parathyroid glands during a thyroidectomy. Hyperactive reflexes, tetany, positive calcium plays with parathyroid function.
notes hyperactive reflexes, tetany, and positive Chvostek's and Trousseau's signs are all manifestations of hypocalcemia.
Chvostek's and Trousseau's sign. The client is
experiencing which of the following disturbances?

1.‐ Hyperparathyroidism
2.‐ Hypoparathyroidism
3.‐ Hypothyroidism
4.‐ Hyperthyroidism

1222 A client is recently admitted with signs and symptoms Correct answer: 1 Increased serum cortisol levels are diagnostic for Cushing's syndrome. Options 2, 3, and 4 are Requires knowledge of the diagnostic values associated with Cushing’s Disease.
suggesting Cushing's syndrome. When evaluating the opposite of the expected abnormalities seen in Cushing's syndrome.
laboratory results, which of the following would
support the diagnosis of Cushing's syndrome?

1.‐ Increased serum cortisol levels


2.‐ Decreased serum sodium levels
3.‐ Increased serum potassium levels
4.‐ Decreased serum glucose level

1223 Which of the following would be a priority nursing Correct answer: 2 Hyperaldosteronism leads to sodium retention, which increases fluid volume and blood Requires knowledge of the function of the organ in fluid balance, such that a decrease in
problem for a client with Conn's syndrome pressure. Hypertension and hypokalemia are the most common signs of this disorder. function would lead to excess fluid.
(hyperaldosteronism)?
1.‐ Fluid volume deficit
2.‐ Fluid volume excess
3.‐ Hyperkalemia
4.‐ Hyponatremia

1224 The nurse is evaluating laboratory results on a client Correct answer: 4 Hyperaldosteronism leads to sodium retention (hypernatremia), potassium depletion Use the process of elimination to assist with this question in which all three items need to
admitted with complaints of hypertension, fatigue, and (hypokalemia), and, as a result, metabolic alkalosis. be correct for the option to be the correct choice.
headaches. Based on these symptoms, Conn's
syndrome (hyperaldosteronism) is suspected. Which of
the following laboratory values would support the
diagnosis of Conn's syndrome?

1.‐ Hyponatremia, hypokalemia, and metabolic acidosis


2.‐ Hypernatremia, hypokalemia, and metabolic acidosis
3.‐ Hyponatremia, hyperkalemia, and metabolic alkalosis
4.‐ Hypernatremia, hypokalemia, and metabolic alkalosis

1225 When providing discharge instructions to a client with Correct answer: 1 To decrease incidence of gastric ulcers, cortisol replacements (prednisone) should be taken Recall knowledge that prednisone increases the risk for gastrointestinal irritation and
Addison's disease, which of the following should the with food or milk. Clients should weigh themselves daily and report changes, and increase fluid strategies to alleviate this.
nurse include in the teaching? intake up to 3,000 ml a day unless contraindicated. Abruptly discontinuing cortisol
replacements can result in Addisonian crisis.
1.‐ Take prescribed prednisone (Deltasone) with food or milk
2.‐ Weigh self weekly and report changes
3.‐ Stop taking prednisone if stomach upset occurs
4.‐ Limit fluid intake to 1,000 ml in 24 hours

1226 A client with Addison's disease expresses concern Correct answer: 2 A deficiency in cortisol leads to fluid volume depletion and sodium depletion. Added salt is Use the process of elimination to assist with this question.
about adding extra salt to meals. The nurse should recommended to replace sodium loss. Options 1, 3, and 4 are incorrect.
explain that the added salt is necessary for which of
the following reasons?
1.‐ Excess production of aldosterone means sodium is lost in the urine, therefore additional salt is necessary for replacement.
2.‐ The lack of sufficient aldosterone means sodium is lost in the urine, therefore additional salt is necessary for replacement.
3.‐ The added salt will help prevent dehydration.
4.‐ The added salt will help prevent an Addisonian crisis.

1227 The plan of care for a client newly admitted with a Correct answer: 3 SIADH results in fluid retention and hyponatremia. Intravenous hypertonic saline solution is Recall knowledge that too much ADH causes fluid retention to be led to the nursing
diagnosis of syndrome of inappropriate antidiuretic administered in addition to diuretics. Maintaining oral fluid restriction is necessary to correct intervention of fluid restriction.
hormone secretion (SIADH) would include which of the fluid imbalance.
following for fluid management?

1.‐ Administration of intravenous infusion of hypotonic saline


2.‐ Encourage an increase in oral fluids
3.‐ Maintain oral fluid restriction
4.‐ Administration of antidiuretic medications
1228 A client with chronic syndrome of inappropriate Correct answer: 1 Chronic SIADH may be life‐threatening if a medical emergency arises and the client is unable Use the process of elimination to determine the correct answer which evaluates
antidiuretic hormone secretion (SIADH) indicates to inform healthcare providers. Chronic SIADH requires lifelong treatment. The client should understanding of teaching.
understanding of discharge teaching by making which weigh daily and report any changes, and there is no indication to consume more than the
of the following statements? normal requirements of fluids once a fluid balance has been obtained.

1.‐ "I will wear a Medic‐Alert bracelet at all times."


2.‐ "I will only be on this medication for a short time."
3.‐ "I will weigh myself every month and report any changes."
4.‐ "I will drink 3,000 ml of fluid a day."

1229 In establishing a plan of care for a client admitted Correct answer: 1 Excretion of massive volumes of urine leads to fluid volume deficit, which is the priority This question requires knowledge of the pathophysiology of diabetes insipidus.
with diabetes insipidus, the priority nursing diagnosis diagnosis. Care and treatment is aimed at restoring fluid and electrolyte balance. Risk for injury
for this client would be which of the following? and knowledge deficit are important diagnoses and should follow fluid volume deficit.

1.‐ Fluid volume deficit


2.‐ Fluid volume excess
3.‐ Risk for injury
4.‐ Knowledge deficit

1230 The nurse admitting a client with the diagnosis of Correct answer: 2 Hypertension and tachycardia are the most common manifestations of this life‐threatening This question calls for looking at sets of opposites and using the process of elimination to
pheochromocytoma knows that expected signs and disorder. Careful monitoring of blood pressure is required as blood pressure elevation can determine when both items are correct.
symptoms of pheochromocytoma would include which reach dangerously high levels.
of the following?
1.‐ Hypotension, bradycardia
2.‐ Hypertension, tachycardia
3.‐ Hypertension, bradycardia
4.‐ Hypotension, tachycardia

1231 A client with type II diabetes is admitted to the Correct answer: 2 Stressors such as illness or surgery increase blood glucose levels. Temporary requirements for Use the process of elimination to find the one correct answer.
hospital for pneumonia. During the hospitalization, exogenous insulin may be necessary to adequately control glucose levels. The client should be
insulin is administered to control elevated glucose informed of this temporary need. Options 1 and 3 are incorrect, and option 4 does not
levels. The client expresses concern that the diabetes recognize the client's need for information or address the psychosocial needs as evidenced by
is "worse," and that insulin will be required the expression of concern.
permanently. The best response to the client would be
which of the following?

1.‐ "Because you have not been compliant with your treatment regimen, you must now take insulin injections."
2.‐ "Stressors such as illness increase blood glucose levels. You will require insulin temporarily until your glucose becomes stabilized again."
3.‐ "Persons with type II diabetes eventually progress to type I diabetes."
4.‐ "You should not worry about the insulin now, just focus on getting better."

1232 A client with Type 1 diabetes mellitus requires further Correct answer: 2 Diabetic clients often learn how to compensate and even be noncompliant without affecting Recognize that questions which call for further instruction require looking for an incorrect
instructions when which of the following statements is their glucose levels excessively. A well‐balanced diet with three meals is usually required to answer.
made? maintain the glucose at the appropriate level, especially since insulin is probably given in the
morning. Options 1, 3, and 4 are all correct.
1.‐ "I will bring some fruit with me when I go swimming."
2.‐ "I will eat less for breakfast since I'm going to a big party tonight with lots of food."
3.‐ "I will check my glucose level before my insulin injection since I've been vomiting."
4.‐ "I will monitor my glucose levels as instructed."
1233 When implementing a teaching plan for a client newly Correct answer: 1 Signs of Hypoglycemia include shakiness, coldness, sweating, nervousness, and palpitations. A Recognize the symptoms and how to manage hypoglycemia.
diagnosed with Type 1 diabetes mellitus the nurse fast‐acting carbohydrate such as juice, crackers, or milk should be taken. If a glucometer is
knows that the client has understood the instructions readily available, the glucose should be confirmed. Glucose levels should be monitored as
when which of the following statements is made? instructed (option 3) and a glucose level of 100 indicates a therapeutic response to insulin
(option 4).
1.‐ "If I feel shaky, cold, and sweaty, I will take my fast‐acting carbohydrate immediately."
2.‐ "If I feel shaky, cold, and sweaty, I will take extra insulin immediately."
3.‐ "I will check my glucose only if I feel like it is low."
4.‐ "I will not take my insulin if my glucose is less than 100."

1234 The nurse is educating a client with hyperthyroidism Correct answer: 4 Since antithyroid drugs inhibit thyroid hormone (TH) production, they do not affect the Use the process of elimination to find the one correct answer.
about tapazole (Methimazole). The instructions to the hormones already formed. Therapeutic effects may not be noticed for several weeks, possibly
client would include which of the following? up to 12 weeks. The client should be informed of this and encouraged to continue medications
as prescribed. Weight gain, not loss, may be a sign of hypothyroidism, which is a potential
effect of treatment with antithyroid drugs. Side effects include rash, pruritis, and elevated
temperature.
1.‐ Generally, there are no specific side effects of this medication.
2.‐ Stop taking the medication and notify healthcare provider if symptoms have not improved within 2 weeks.
3.‐ Monitor and report any weight loss.
4.‐ Therapeutic effects of the medication may not be noticed for several weeks.

1235 A client who is post‐thyroidectomy shows evidence of Correct answer: 4 To decrease strain on the suture line, the client should support the head and neck with both This question calls for using the process of elimination to find a correct answer.
understanding the preoperative teaching instructions hands behind the neck while moving in bed, coughing, or any other activity that could increase
when the nurse observes the client demonstrating strain on sutures. The client should be encouraged to deep‐breathe and cough, and pain
which of the following actions? medication will promote comfort.

1.‐ Maintains a supine position at all times


2.‐ Avoids coughing
3.‐ Refuses pain medications
4.‐ Supports head and neck with both hands while moving in bed

1236 A client diagnosed with hypothyroidism is admitted to Correct answer: 2 Hypotension, bradycardia, and dysrhythmias result from decreased cardiac output related to In this question all four options are correct, so establish priority among the four to select
the unit. In planning care, the nurse's priority hypothyroidism. Options 1, 3, and 4 are all appropriate assessments for clients with the correct answer.
assessment would be which of the following? hypothyroidism; however, they should follow the cardiovascular assessment.

1.‐ Muscle weakness, fatigue, and lethargy


2.‐ Blood pressure, heart rate, and rhythm
3.‐ Baseline weight and bowel elimination pattern
4.‐ Neurological status and skin integrity

1237 In providing care to a client on the first postoperative Correct answer: 4 Maintaining integrity of suture line is essential following a parathyroidectomy. Nursing care is Use the process of elimination to determine the one correct answer.
day following a parathyroidectomy for the treatment essentially the same as for a thyroidectomy. The head should be supported with both hands
of hyperparathyroidism, the priority nursing care for behind neck when moving, sitting up in bed, or coughing. Restricting fluids is contraindicated,
this client would include which of the following as is calcium, since hypercalcemia is a complication of hyperparathyroidism. Antithyroid
actions? medications are given to treat hyperthyroidism.

1.‐ Restricting intravenous and oral fluids


2.‐ Administering antithyroid medications
3.‐ Administering intravenous calcium
4.‐ Assuring proper support of head
1238 In planning care for a client with hypoparathyroidism, Correct answer: 1 The primary complication of hypoparathyroidism is hypocalcemia, which is a result of failure Use the process of elimination to find the one correct answer.
the nurse should assess for which of the following to release parathyroid hormone. Manifestations are directly related to decreased calcium
potential complications? levels. Options 2, 3, and 4 are manifestations of hyperparathyroidism.

1.‐ Laryngeal spasm


2.‐ Hypercalcemia
3.‐ Hypohosphatemia
4.‐ Hypertension

1239 The nurse is providing care to a client on the second Correct answer: 3 Adrenalectomy results in adrenal insufficiency, causing fluid imbalance (loss) manifested by This question asks the function of the adrenals. Recall that when there is a loss of function
postoperative day following a bilateral adrenalectomy. hypotension. The lack of mineralcorticoids will result in hyponatemia and hyperkalemia there will be manifestations of the illness.
In planning care for this client, which of the following (options 1 and 2), and the lack of glucocorticoids will result in hyperglycemia (option 4).
imbalances may occur as a result of adrenalectomy?

1.‐ Hypernatremia
2.‐ Hypokalemia
3.‐ Hypotension
4.‐ Hyperglycemia

1240 A client is admitted with a diagnosis of Addisonian Correct answer: 1 Severe hypotension, circulatory collapse, shock, then coma are the complications of Use the process of elimination to find the one right answer and to rule out the opposites.
crisis. In planning the immediate care for this client, Addisonian crisis. Immediate intravenous replacement of glucocorticoids and fluids are
the nurse should anticipate which of the following indicated to prevent death. Options 2, 3, and 4 are recommended treatments for Cushing's
actions? syndrome, the opposite of Addison's disease.
1.‐ Administration of intravenous glucocorticoids and saline fluids
2.‐ Administration of mitotane (Lysodren)
3.‐ Preparation of client for adrenalectomy
4.‐ Administration of trilostane (Modrastane)

1241 A 27‐year‐old male sustains a head injury as a result Correct answer: 1 Syndrome of inappropriate antidiuretic hormone secretion (SIADH) results in fluid retention Use the process of elimination to find the one right answer in the treatment of SIADH.
of a fall from a roof. He began experiencing seizure and hyponatremia. Treatment is aimed at correcting fluid and electrolyte imbalance, which
activity and fluid retention and was diagnosed with includes intravenous administration of hypertonic saline solution and oral fluid restriction.
syndrome of inappropriate antidiuretic hormone Edema is usually not a complication because fluid is retained between the intracellular and
secretion (SIADH). His plan of care should include extracellular spaces. Lifetime treatment of this disorder is not necessary for an acute
which of the following? occurrence because the manifestations usually resolve within 3 days.

1.‐ Administering intravenous hypertonic saline solution


2.‐ Administering oral fluids up to 2,000 mL per day
3.‐ Educating the client about the need for lifetime replacement medications
4.‐ Preventing complications associated with edema

1242 Which of the following statements indicates that the Correct answer: 1 The primary side effect of insulin is hypoglycemia. The client should be aware of the signs and Use the process of elimination to find the one correct answer.
client performing insulin self‐administration symptoms, and should always have a source of sugar available. Options 2, 3, and 4 are
understands the basic principles? incorrect and these responses would require further instruction.
1.‐ "I will monitor myself for low glucose levels and keep a source of sugar available."
2.‐ "I will not take my insulin if my glucose level is normal."
3.‐ "I will not take my insulin if I skip a meal."
4.‐ "I will take less insulin if I have an illness."
1243 A client is admitted in diabetic ketoacidosis (DKA) and Correct answer: 2 The nurse should watch for hypoglycemia during the peak time of both the regular insulin and Recall knowledge of the peaks of both insulins, and then find the true answer for both
is given both regular and NPH insulin. After 2 days, the NPH, which differ. The peak for regular is 2 to 4 hours; NPH 8 to 12 hours. types of insulins.
glucose levels are maintained at 200 to 300 mg/dL.
NPH and regular insulin is administered every day at
7:30 A.M. The nurse knows to watch for a
hypoglycemic reaction between:

1.‐ 9:20 A.M. to 11:30 A.M.


2.‐ 9:30 A.M. to 7:30 P.M.
3.‐ 3:30 P.M. to 7:30 P.M.
4.‐ 9:30 A.M. to 3:00 P.M.

1244 A female client is being given 30 mCi sodium iodide‐ Correct answer: 2 Clients receiving doses of I‐131 that are greater than 30 mCi may not have visitors for 24 or Knowledge is needed of the precautions associated with radioactive isotopes.
131 (Iodotope) to treat Graves' disease. Before giving more hours based on radiation dose. For dose of 30 mCi or less, visitors must remain several
the client her first dose you should do which of the feet away from client and client may not hold/cuddle children or sleep in same room as
following? another person for 8 days (to protect them from radiation exposure). Clients who are allergic
to shellfish are also allergic to iodine but egg allergy is irrelevant (option 1). Options 3 and 4
are unrelated to this medication.
1.‐ Assess the client for hypersensitivity by asking if she is allergic to eggs.
2.‐ Instruct her that she must not sleep in the same room with another person for 8 days.
3.‐ Assess her temperature to use as a baseline to evaluate the medication effectiveness.
4.‐ Instruct the client not to drink the medication mixture with a straw in order to ensure she drinks the entire dose.

1245 A client is 12 hours post‐partial thyroidectomy. During Correct answer: 3 The parathyroid glands, located near the thyroid gland, may have been injured or accidentally This question requires knowledge of the function of proximity of the parathyroid and
the postoperative phase the nurse asks the client removed, resulting in hypocalcemia. Hypocalcemia is life‐threatening; thus it is important to thyroid glands.
about any numbness or tingling of the face, mouth, or identify early signs. Numbness and/or tingling of the mouth, face, or extremities are early
extremities for which of the following assessment symptoms of low serum calcium. Reduced thyroid hormone levels are expected results of
purposes? surgery (option 1). Option 2 should refer to the pituitary gland. Option 4 is possible, but could
be detected by hoarseness or weak voice.

1.‐ Early identification of low thyroid hormone


2.‐ Detection of thyroid‐induced hypoglycemia
3.‐ Early identification of hypocalcemia
4.‐ Detection of nerve damage related to surgery

1246 The client is diagnosed with an allergy to iodine. In Correct answer: 2 Iodine intake is needed for the thyroid gland to produce thyroid hormone. Insufficient iodine This question requires knowledge of the function of iodine as it impacts thyroid function.
addition to client education about avoiding foods with intake leads to low thyroid hormone production and symptoms of hypothyroidism, which
iodine, the client should also be taught to report which includes constipation, weight gain, and muscle stiffness, among others. The other options are
of the following symptoms associated with endocrine incorrect.
malfunction related to low iodine intake?

1.‐ Diarrhea, weight loss, blurred vision


2.‐ Constipation, weight gain, muscle stiffness
3.‐ Fatigue, dry skin, increased BP
4.‐ Anorexia, dyspnea, weight loss

1247 A client is 20 hours post‐colon resection with end‐to‐ Correct answer: 2 Clients with Addison disease should be assessed for signs of Addisonian crisis following a This question requires knowledge of the assessments specific to Addison’s Disease.
end anastomosis for ruptured diverticulum. You have stressful event such as surgery. Signs of Addisonian crisis include decreased urine output,
read in the medical record that he has an 8‐year decreased blood pressure, dry skin, and altered level of consciousness. Options 1, 3, and 4 do
history of Addison's disease. After noting new onset of not apply to the necessary priority assessments related to Addisonian crisis, although they are
lethargy with the current assessment, you would do good general post operative assessments.
which of the following next?
1.‐ Review his patient‐controlled analgesia (PCA) record for dose history
2.‐ Assess him for decreased urine output and blood pressure (BP)
3.‐ Check pupils for direct and consensual reaction
4.‐ Obtain a pulse oximeter to check his oxygen saturation level

1248 A client has new onset type I diabetes mellitus (DM). Correct answer: 1 Research by the National Institute of Health and the American Diabetes Association Recall knowledge of the long term complications of hyperglycemia.
He asks you why he needs to check his blood glucose demonstrates a strong correlation between chronic hyperglycemia and complications of
level so frequently. You explain that frequent coverage retinopathy, nephropathy, and neuropathy. Thus, there is damage to the eyes, kidneys, and
with insulin to keep his blood glucose level between 80 peripheral nerves, respectively. Lactic acidosis occurs with diabetic ketoacidosis (option 2).
and 155 mg/dL is important for which of the following Option 3 is a false rationale for the client in the question. Insulin is needed to carry glucose
reasons? across the cell membrane into the cell, not to be transported in the blood (option 4).

1.‐ Chronic elevated blood glucose levels damage cells and causes multiple organ damage.
2.‐ High glucose levels cause the body to use proteins for energy, causing lactic acidosis.
3.‐ Early identification of hypoglycemia before the onset of symptoms is easier to treat.
4.‐ Carbohydrates are constantly being converted to glucose and transported in the blood by insulin.

1249 You have been teaching the client with new onset of Correct answer: 4 In SIADH there is excess secretion of ADH that causes fluid retention, dilutes the plasma In this question, look closely at the opposites to make the correct selection.
syndrome of inappropriate antidiuretic hormone causing suppression of aldosterone, and increases renal excretion of sodium. Water then
(SIADH) about the disorder. Which of the following moves into the cells from the plasma and interstitial spaces causing cellular edema. The
statements by the client best indicates that he treatment is fluid restriction and hypertonic saline infusion. Options 1 and 3 are the opposite
correctly understands how to manage this disease? of standard treatment and are therefore incorrect. Option 2 is unrelated to this client.

1.‐ "I should limit my sodium intake to 2 grams daily."


2.‐ "I should report constipation or fatigue to the doctor."
3.‐ "I should drink at least 3,000cc or 10 glasses of water daily."
4.‐ "I should limit my fluid intake to approximately 800 cc or 4 glasses of water daily."

1250 The client is admitted with decreased level of Correct answer: 3 Diabetes insipidus (DI) can develop with head injury, tumors, and other conditions that cause This question requires knowledge of the relationship to lack of organ function.
consciousness secondary to a closed head injury increased intracranial pressure. Excessive urine output of 350 ml/hr or more is a classic early
resulting from a fall while roller‐skating. Urine output symptom of DI. The specific gravity provides valuable information about renal function and
is 500 ml from 6:00 A.M. to11:00 A.M., 1000 ml from response to ADH. Using critical inquiry to analyze the urine output, specific gravity and other
11:00 A.M. to 2:00 P.M. and 350 ml from 2:00 P.M. to characteristics of the urine, the nurse assesses for classic signs of DI that can occur following a
3:00 P.M. which of the following actions by the nurse is head injury. Options 1 and 4 are false. Option 2 would be insufficient fluid replacement.
appropriate?
1.‐ Realize that this is normal urine output and continue to monitor the client.
2.‐ Encourage the client to drink 8 to 10 glasses of fluid daily.
3.‐ Check the urine specific gravity and report any abnormality as well as the urine output.
4.‐ Decrease the IV rate from 100 ml/hr to 25 ml/hr suspecting fluid excess.

1251 The client with acromegaly secondary to excessive Correct answer: 4 Option 4 addresses the lifelong hormone replacement of thyroid, glucocorticoids, and In this question, determine first if the goals are met or not met, and then determine the
growth hormone (GH) states: "I'll be glad to have this gonadotropin needed when the entire pituitary gland is removed. Options 1 and 3 are rationale for this decision.
surgery; after the pituitary gland is removed I will be incorrect. Option 2 relates to the immediate postoperative time while the client's comments
cured–then no more lab tests and pills!" Which of the relate to long‐term outcome.
following statements should the nurse document to
evaluate the client's understanding of preoperative
teaching?
1.‐ Criteria met: Client correctly verbalized understanding of outcomes.
2.‐ Criteria not met: Client needs to know about routine post operative lab tests done on first postoperative day to evaluate response to surgery.
3.‐ Criteria not met: Client needs to know surgery will slow down the disease process but client will need regular blood tests and x‐rays for 1 year.
4.‐ Criteria not met: Client needs to know surgery will
stop excess production of GH and probably other
hormones, and thus will need daily replacement
medications for life.

1252 The client is admitted with metabolic acidosis Correct answer: 2 DKA is associated with excessive urine output, dehydration, and hypokalemia, placing the This question asks to set sound priorities based on the life‐threatening nature of the
secondary to diabetic ketoacidosis (DKA). client at risk for decreased cardiac output and cardiac dysrhythmias. Option 1 is false regarding complication.
Understanding metabolic acidosis, you should choose output and does not address the metabolic problem. Options 3 and 4 may apply to the client
which of the following as the priority nursing but are not the priority needs, in addition, option 3 will resolve as the DKA is treated.
diagnosis?
1.‐ Decreased urinary elimination related to reduced output and muscle function.
2.‐ Decreased cardiac output related to fluid and electrolyte imbalance.
3.‐ Ineffective breathing pattern related to hyperventilation.
4.‐ Anxiety related to fears of long‐term outcomes and discomfort.

1253 An elderly female client with dry flaky skin and Correct answer: 4 Daily total baths remove the protective sebum from the skin, placing the client at risk for This question requires going “back to the basics” associated with fundamental nursing
activity intolerance secondary to myxedema is altered skin integrity. Since the client's level of participation in the bath or other self‐care knowledge.
admitted to the progressive care unit to improve her activity is not presented, the other 3 options are inappropriate. In addition, options 2 and 3 are
activity tolerance and gain independence in self‐care. not specific and measurable enough to meet criteria for an outcome statement.
For hygiene care the nursing order is two total baths
per week on Saturday and Wednesday with partial
baths on the remaining days of the week. Which of the
following would be the desired outcome of this
intervention?
1.‐ Client is able to sleep through the night and stay awake most of the day
2.‐ Gradual increase in ambulation ability over the first month
3.‐ Increased energy by the end of the first week of paced rest and activity
4.‐ Intact elastic moist warm skin by the end of first week

1254 The client is 8 hours post partial thyroidectomy for Correct answer: 3 Early signs of edema of the larynx leading to airway obstruction are tight‐fitting dressing, In this question, recall the ABCs (airway, breathing, circulation) and focus on airway, a very
Graves' disease. What's the best documentation by the stridor, stertor, and weak or harsh voice. Numbness or tingling of the extremities, lips or high priority with all clients.
nurse of evaluation outcome criteria for the nursing mouth are signs of hypocalcemia that can lead to respiratory distress due to tetany. The data
diagnosis risk for Ineffective airway clearance? in the other options are important routine postoperative assessments, but they do not relate
to the client's airway.
1.‐ Dressing is clean dry and intact, pain minimal and controlled, alert and oriented.
2.‐ Vital signs stable; client supports neck with hand during change of position.
3.‐ No tracheal stridor, speaks clearly, and denies numbness or tingling.
4.‐ Balanced intake and output, vital signs stable, and alert and oriented.

1255 Which of the following evaluation data would best Correct answer: 1 HHNK results from hyperglycemia, causing excessive loss of water and retention of glucose This question reinforces the importance of client behavior in assessing client illness and
lead the nurse to conclude that the client with that leads to dehydration, hypernatremia and hypokalemia. Symptoms are dry, tenting skin, responses to treatment.
hyperglycemic hyperosmolar nonketotic coma (HHNK) dry mucous membranes, altered level of consciousness and hyperthermia. Ketones are not
has demonstrated improvement during the first 24 present in HHNK; thus, monitoring for ketones is inappropriate (option 3). Options 2 and 4 do
hours? not address the primary problems that occur with HHNK.

1.‐ Alert and oriented, balanced intake and output, moist mucous membranes
2.‐ Intake equals output, denies pain and shortness of breath
3.‐ Alert and oriented, blood and urine without ketones, no orthostatic BP
4.‐ Respirations easy and even, eats 50 to 75 percent of meals, vital signs stable
1256 The nurse is caring for a client with type 1 diabetes Correct answer: 1 The signs of hypoglycemia include hunger, shakiness, sweating, pale cool skin, and irritability. The correct answer is a symptom of hypoglycemia, the rest are indicative of
mellitus. In developing a teaching plan, which of the These signs may be manifestations of impaired cerebral function from the hypoglycemia. The hyperglycemia.
following signs and symptoms of hypoglycemia should other options are all signs of hyperglycemia.
be included?
1.‐ Shakiness
2.‐ Increased thirst
3.‐ Fever
4.‐ Fruity breath

1257 A client with diabetes is being tested for glycosylated Correct answer: 2 Glycosylated hemoglobin reflects the average blood glucose over the life of the RBC, usually 4 In this question, define glycosylated hemoglobin to then determine the answer.
hemoglobin. In explaining the purpose of the months. This test is not a ratio of hemoglobin to glucose content (option 3) nor is it helpful in
laboratory test the nurse explains that glycosylated diagnosing anemia (option 1). The time frame in option 4 is too long.
hemoglobin is used for which of the following
purposes?
1.‐ To check for anemia
2.‐ To determine the average blood glucose level for up to the previous 4 months
3.‐ To compare hemoglobin to glucose levels
4.‐ To calculate the amount of glucose in hemoglobin for the past 6 months

1258 A post‐surgical client is brought back to the nursing Correct answer: 4 The danger of hemorrhage is greatest during the first 24 hours following thyroid surgery. The Consider the client position, the vascularity of the thyroid gland, and the need for in depth
unit following a thyroidectomy. Which of the following tendency is for blood to flow down at the sides and posteriorly if hemorrhage occurs in the nursing assessments.
methods should the nurse use to assess for bleeding? area of the neck. Inspecting the front of the dressings for signs of hemorrhage may not reveal
bleeding (option 1). Changing dressings immediately after surgery is not appropriate (option 2).
A drop in hemoglobin may be a clue to bleeding but is not the best initial assessment action
(option 3).
1.‐ Inspect the front of the dressing for signs of hemorrhage.
2.‐ Change the dressing applied in the operating room.
3.‐ Check the latest hemoglobin to determine if there has been a drop in value.
4.‐ Palpate the back of the neck and shoulders for evidence of bleeding.

1259 A diabetic client with the flu asks why he should drink Correct answer: 1 Starvation‐induced ketosis can be prevented by drinking juices that equal the prescribed This question requires the knowledge of the development of DKA.
juices, check his finger stick glucose every 4 hours and carbohydrate meal pattern. Fluids are needed to prevent dehydration and hyperosmolality,
take insulin when he is not eating and is vomiting. which could result from large fluid losses from persistent vomiting. The liver breaks down fats
Which of the following would be the best explanation to form glucose for energy and ketones, leading to DKA. The other options do not address the
by the nurse? key issues of dehydration and hyperglycemia.
1.‐ He needs to prevent dehydration, excessive breakdown of fats for glucose, and monitor for hyperglycemia.
2.‐ He needs to check his blood glucose because vomiting could cause hypoglycemia and drinking fluids will prevent dehydration.
3.‐ His body uses protein for energy when he is sick, causing increased ketones and hypoglycemia.
4.‐ If he could substitute water for the juices to prevent dehydration, then he would not need to check his blood glucose levels so often.

1260 The client with diabetic ketoacidosis (DKA) is given Correct answer: 3 Level of consciousness responds quickly to early changes in pH and restoration of fluid and Note that this question highlights behavior changes and level of consciousness as early
intravenous normal saline infusion and regular insulin. electrolyte balance. Urine output decreases as hyperglycemia is resolved. The respiratory signs of condition changes.
In addition to hourly blood glucose monitoring, what buffer system takes a few hours to respond to change in ph. Dehydration is usually so severe
assessment data are early signs of clinical that several hours of rehydration are needed to reduce pulse (option 2) and resolve orthostatic
improvement? BP (option 1). Option 4 is inappropriate because eating a full meal is not an early sign of
improvement.
1.‐ Respiratory rate of 12 to 15 and normal BP in the standing position
2.‐ Temperature and pulse in normal range
3.‐ Improved level of consciousness and decreasing urine output
4.‐ Client eats a full meal and respiratory rate is normal
1261 The nurse is preparing to discharge a client newly Correct answer: 4 The candy bar and ice cream may have too much glucose and fat, potentially leading to This question asks the nurse to recommend both a nutritionally sound snack and one that
diagnosed with diabetes mellitus. The client states, "I hyperglycemia. In addition the fat may delay glucose absorption. Immediate absorption of raises the blood sugar rapidly.
should eat a candy bar or cup of ice cream every time I glucose is needed in hypoglycemia. The client should also check the blood glucose within 15
feel shaky, hungry, or nauseated." Which of the minutes of taking glucose because of signs of hypoglycemia.
following is the best response by the nurse?

1.‐ "Yes, a candy bar or cup of ice cream is needed to treat the hypoglycemia."
2.‐ "Yes, you should eat the snack, then have a meal as soon as possible."
3.‐ "No, you should quickly eat a meal; the candy will cause hyperglycemia."
4.‐ "No, these have too much sugar and fat, 5 Lifesavers candy or skim milk are better."

1262 The client had a bilateral adrenalectomy for Cushing's Correct answer: 4 Usually the cortex of the adrenal gland (not the medulla as in option 1) increases secretion of Recognize that this question has a focus on the risk of medications.
syndrome. He is being sent home with a new cortisol to stimulate the immune system in response to an infection. Thus the replacement
prescription for hydrocortisone. The best statement dose during illness may need to be adjusted once a client's adrenal glands are removed.
indicating understanding of the drug and associated Hydrocortisone can irritate gastric mucosa and so clients should not take gastric irritants such
risk is: as aspirin or nonsteroidal anti‐inflammatory drugs (NSAIDs). Dosage may be adjusted during
illness. Options 2 and 3 do not address the risk of the drug, which is the issue of the question.

1.‐ "I am taking this drug to replace the hormones usually secreted by the adrenal medulla."
2.‐ "I should take this pill every morning before breakfast."
3.‐ "This pill may cause weight gain, so I should exercise more and eat less."
4.‐ "I should call the doctor if I think I am starting a cold, and I should not take aspirin."

1263 An obese, elderly client is being given a high‐dose Correct answer: 1 The client is at risk for type 2 diabetes mellitus (DM). Polydipsia and polyuria are signs of Note that this client is on steroids and is obese, increasing the risk of secondary and Type 2
steroid protocol as emergency treatment of a spinal hyperglycemia, a symptom of DM. Steroids may also increase carbohydrate metabolism, diabetes.
cord injury. The client's family has requested frequent leading to hyperglycemia in clients with insufficient insulin. Thus the nurse should assess the
refills for his cup of chipped ice and his urine output is client's blood glucose. Since hyperglycemia places the client at risk for fluid volume deficit the
600 mL for the first 4 hours and 900 mL in the second 4 nurse should calculate the client's fluid balance. The nurse also needs to record the assessment
hours. The family states that the client was often data and report it to the physician. The client is not at risk for hypervolemia (options 2 and 3).
thirsty at home, even prior to the injury. The nurse Option 4 is not a priority.
should take which of the following most appropriate
actions?

1.‐ Obtain finger stick blood glucose and tabulate the client's 24 hour intake and output.
2.‐ Restrict the client's intake to prevent fluid volume excess and electrolyte imbalance.
3.‐ Obtain and monitor CBC, hemoglobin and hematocrit, and vital signs for hypervolemia.
4.‐ Evaluate the clients breathing pattern and understanding of current treatment plan.

1264 The client has undergone hypophysectomy using a Correct answer: 2 The presence of a halo effect indicates cerebrospinal fluid (CSF). Glucose present in the nasal This question requires use of the mnemonic that halo secretions lead to CSF.
transphenoidal approach. You change the mustache drainage also suggests that the drainage is CSF. A persistent headache indicates a CSF leak. The
dressing, noting clear exudate with a pale yellow physician needs to be informed of these assessment findings and the client must be
colored ring at the edge of the drainage on the maintained on bedrest to stop the leak. A spinal tap may be done to decrease CSF pressure.
dressing. You should do which of the following next? Option 1 is incorrect because it does nothing for the client. Options 3 and 4 do not address the
real problem, a probable CSF leak.
1.‐ Document this as serous drainage and continue to monitor the client.
2.‐ Assess for headache and check the glucose level in the drainage.
3.‐ Apply an ice pack to the nasal bridge and a large fluffy dressing.
4.‐ Lower the head of the bed to decrease the gravity pressure on the wound.
1265 A client who has a history of Graves' disease Correct answer: 4 With exophthalmos, the eyelids may not cover and protect the cornea of the eye. Thus, eye This question asks the nurse to define eye protrusion and the safe care with this symptom.
accompanied by exophthalmos is arriving from protection from the sheets or preventing the hands from accidentally touching the eyes is
surgery. Based on the observations as you note the needed while the client is in bed. With Graves' disease clients usually experience heat
photo, what should you educate the nurse assistant to intolerance, thus less covering and a cool room are preferred (option 1). Hyperglycemia is not
do? usually associated with Graves' disease. The head of the bed should be elevated 30&amp;deg;
to minimize eye pressure (option 3).

1.‐ Keep the client's room warm to promote comfort


2.‐ Obtain fingerstick blood glucose every 2 hours twice
3.‐ Keep the head of the bed flat for 4 hours
4.‐ Provide eye protection measures for the client

1266 The client is admitted with all of the following orders Correct answer: 1 Fluid and electrolyte replacement is the highest priority. Hyperglycemia is treated with The treatment requires IV access for the treatment with hydration and insulin.
to treat diabetic ketoacidosis (DKA) with severe regular insulin rather than an oral agent (option 3). Concurrent administration of IV regular
metabolic acidosis. Which order would the nurse insulin would also be done as a priority. The items in the other options can be done after
determine to be the first priority in managing this definitive treatment for dehydration is done.
client?
1.‐ Start IV fluid infusion for rehydration.
2.‐ Insert an indwelling urinary catheter.
3.‐ Give oral glucophage (Metformin).
4.‐ Initiate continuous pulse oximetry.

1267 A client is diagnosed with hyperglycemic Correct answer: 4 Glucagon is given IM or SC for low blood glucose associated with unconsciousness; usually the Recall treatment given for low blood sugar.
hyperosmolar nonketotic coma (HHNK) after being blood glucose is less than 20 mg/dL. Hypoglycemia associated with a change in level of
admitted with a blood glucose level of 720 mg/dL. The consciousness or seizure requires immediate interventions. Glucagon will not help urine
admitting orders include: normal saline infusion, output or blood pressure, and is not administered routinely when glucose levels fall to 150
insulin infusion, tylenol PRN and glucagon PRN. When mg/dL.
should the nurse prepare to give the glucagons?

1.‐ When the blood sugar reaches 150 mg/dL


2.‐ For urine output less than 60 mL/hr
3.‐ For symptomatic BP less than 100/60 mmHg
4.‐ For precipitous drop in blood glucose leading to unresponsiveness

1268 The client is being started on methylprednisolone Correct answer: 2 Clients taking methylprednisolone, a glucocorticoid, should be monitored for signs of This question asks the nurse to understand the connection of prednisone with
(Solu‐Medrol) following craniotomy for removal of a Cushing’s syndrome and hyperglycemia. Increased corticosteroid serum level can cause sodium hyperglycemia.
tumor. Understanding the potential side effects for retention, increased BP, edema, hypokalemia, weakness, and ecchymosis.
this class of medication, the nurse knows it is
important to monitor for which of the following?

1.‐ Rapid scar formation of the incision


2.‐ Polydipsia, polyuria, increased blood pressure (BP)
3.‐ Numbness and tingling of fingers
4.‐ Orthostatic hypotension, increased pulse

1269 The nurse is teaching a client about long‐term use of Correct answer: 3 Long‐term corticosteroid therapy can cause Cushing's syndrome. To prevent the osteoporosis Use the process of elimination to find the one correct answer.
therapeutic (additional) corticosteroid medication. associated with Cushing's syndrome, clients should eat diet high in calcium. Extra
Which of the following is the best statement that corticosteroids over the long term can cause weight gain and increased hair on the body.
indicates client understanding of the concepts taught? These clients are at risk for gastrointestinal bleeding and should avoid taking aspirin.

1.‐ "It is normal to have some hair loss."


2.‐ "I should eat more to prevent excessive weight loss."
3.‐ "I should eat 4 to 6 servings of food high in calcium."
4.‐ "I should take aspirin for flu or cold symptoms."

1270 The client is admitted with thyroid storm. Assessment Correct answer: 4 Tachycardia, hypertension, and tachypnea increase stroke volume and tissue demand for With this question consider the priority cardiac output plays in clients with hydration
reveals: BP188/102, HR 132 regular, RR 28 full depth oxygen, leading to increased cardiac workload and possible heart failure. If fluid volume deficit issues.
and symmetrical, no urine output since admission to is present, then there is an additional risk for decreased cardiac output. There is insufficient
the emergency department, alert, and anxious. Which data to determine fluid volume status. The tachypnea is a symptom of the increased metabolic
of the following would be the high priority nursing rate.
diagnosis for this client?

1.‐ Fluid volume deficit r/t decreased absorption as evidenced by no urine output since admission
2.‐ Anxiety r/t fear as evidenced by client’s appearance
3.‐ Ineffective breathing pattern r/t increased metabolism as evidenced by RR 28.
4.‐ Risk for decreased cardiac output r/t increased ventricular workload as evidenced by BP 188/102, HR 132, RR 28

1271 A client with diabetic ketoacidosis had repeat arterial Correct answer: 3 The pH indicates acidosis; the PCO&lt;sub&gt;2&lt;/sub&gt; is low, indicating increased Use the opposites to determine the pH, and then use the CO&lt;sub&gt;2&lt;/sub&gt; and
blood gases (ABGs) drawn 24 hours after initiation of a respiratory rate; and the low bicarbonate level is usually associated with metabolic acidosis. HCO&lt;sub&gt;3&lt;/sub&gt;&lt;sup&gt;‐&lt;/sup&gt; to determine the state of metabolic
Regular Insulin intravenous infusion. The results are: The respiratory system is attempting to compensate for the excess metabolic acids but there acidosis.
pH 7. 30, PaCO&lt;sub&gt;2&lt;/sub&gt; 29, continues to be an excess of them. The bicarbonate level is low because it is being depleted in
PaO&lt;sub&gt;2&lt;/sub&gt; 95, an attempt to buffer the metabolic acids.
O&lt;sub&gt;2&lt;/sub&gt; saturation 99%,
HCO&lt;sub&gt;3&lt;/sub&gt;&lt;sup&gt;‐&lt;/sup&gt;
20 mEq/L and Base excess ‐1. 2. The nurse would
interpret the results to be which of the following
imbalances?

1.‐ Respiratory acidosis


2.‐ Respiratory alkalosis
3.‐ Metabolic acidosis
4.‐ Metabolic alkalosis

1272 The nurse is instructing a client with diabetes mellitus Correct answer: 2 Clients with diabetes mellitus are susceptible to injuries because of the decreased sensation Use the process of elimination to find the one correct answer.
(DM) regarding foot care. The nurse should include associated with the effects of chronic hyperglycemia, compounded by the diabetes‐induced
which of the following? arteriosclerosis. The client should be taught to inspect the feet daily utilizing a mirror to
facilitate inspection of hard to see areas. Cotton socks are preferred as they absorb moisture
and allow the feet to dry. The use of prophylactic antibiotics is not appropriate, the moisture
could lead to skin maceration and breakdown.

1.‐ Inspect feet once a week


2.‐ Allow 1/2 to 3/4‐inch toe room when choosing shoes.
3.‐ Wear nylon socks.
4.‐ Apply antibiotic ointment sparingly between the toes to prevent infection.

1273 During a scheduled exam the client's glycosylated Correct answer: 1 Glycosylated hemoglobin is elevated due to long‐term hyperglycemia. Values greater than 8 Recall knowledge of the interventions associated with poor control, indicated by this
hemoglobin was found to be 9 percent. The client has percent indicate consistently poor control of blood glucose and the need to assess the client's HgbA1c.
had diabetes mellitus for 3 years. The nurse should do dietary pattern for the past several months in relation to the treatment plan. The other
which of the following? options do not apply.

1.‐ Explore the client's general dietary pattern for the past 4 months.
2.‐ Assess for signs of infection and client's intake for the past 24 hours.
3.‐ Review the client's understanding of diabetic foot care.
4.‐ Immediately give sliding scale insulin medication.
1274 A client has a total gastrectomy. The nurse explains to Correct answer: 3 The loss of parietal cells that secrete intrinsic factor results in Vitamin The core issue of the question is knowledge that gastric surgery results in loss of ability to
the client the need for long‐term injections of which of B&lt;sub&gt;12&lt;/sub&gt; (cyanocobalamin) deficiency postgastrectomy, because intrinsic produce intrinsic factor and subsequent Vitamin B&lt;sub&gt;12&lt;/sub&gt; deficiency.
the following vitamins? factor is needed for absorption of Vitamin B&lt;sub&gt;12&lt;/sub&gt;. For this reason, clients Use nursing knowledge and the process of elimination to make a selection.
require Vitamin B&lt;sub&gt;12&lt;/sub&gt; injections for life. The other options identify other
B‐complex vitamins.
1.‐ Thiamine
2.‐ Folic acid
3.‐ Cyanocobalamin
4.‐ Niacin

1275 A client with diverticular disease undergoes a Correct answer: 3 Bowel perforation is a possible result of colonoscopy if the colonoscope accidentally pierces The core issue of the question is assessment data that correlates with complications of
colonoscopy. When conducting an abdominal the bowel wall. Perforation could lead to symptoms of peritonitis, such as guarding and colonoscopy, such as peritonitis. Use nursing knowledge and the process of elimination to
assessment, the nurse looks for which of the following rebound tenderness. The other options are incorrect, because diarrhea (option 1), nausea and make a selection.
as a sign of possible complication of the procedure? vomiting as signs of obstruction (option 2), and redness and warmth of abdominal skin (option
4) are not of concern.
1.‐ Diarrhea
2.‐ Nausea and vomiting
3.‐ Guarding and rebound tenderness
4.‐ Redness and warmth of the abdominal skin

1276 The client who has ulcerative colitis is scheduled for Correct answer: 2 A client with an ileostomy has no control over bowel movements and must always wear a The core issue of the question is knowledge of stool characteristics and associated stoma
an ileostomy. When the client asks the nurse what to collection device. The drainage tends to be liquid but becomes pastelike with intake of specific appliance needs following ileostomy. Use nursing knowledge and the process of elimination
expect related to bowel function and care after foods. to make a selection.
surgery, what response should the nurse make?

1.‐ “You will be able to have some control over your bowel movements.”
2.‐ “The stoma will require that you wear a collection device all the time.”
3.‐ “After the stoma heals, you can irrigate your bowel so you will not have to wear a pouch.”
4.‐ “The drainage will gradually become semisolid and formed.”

1277 The nurse is conducting dietary teaching with a client Correct answer: 3 Dumping syndrome, in which gastric contents rapidly enter the bowel, can occur following The core issue of the question is knowledge of foods to avoid when the client has dumping
who has dumping syndrome. The nurse encourages the gastrectomy. Dietary fats and proteins are increased, and carbohydrates, especially simple syndrome. Use nursing knowledge and the process of elimination to make a selection.
client to avoid which of the foods that the client carbohydrates such as fruits, are reduced. This helps slow the GI transit time and reduce the GI
usually enjoys? cramping, diarrhea, and vasomotor symptoms associated with dumping syndrome.

1.‐ Eggs
2.‐ Cheese
3.‐ Fruit
4.‐ Pork

1278 A client is being evaluated for possible duodenal Correct answer: 1 The pain of a gastric ulcer is dull and aching, occurs after eating, and is not relieved by food as The core issue of the question is expected assessment findings in duodenal ulcer. Recall
ulcer. The nurse assesses the client for which of the is the pain from duodenal ulcer. The pancreatic juices that are high in bicarbonate are released the effect of pancreatic juices on the duodenal ulcer surface and use the process of
following manifestations that would support this with food intake and relieve duodenal ulcer pain when the client eats. Chronic aspirin use is elimination to make a selection.
diagnosis? irritating to the stomach (option 2). The manifestations in options 3 and 4 are unrelated.

1.‐ Epigastric pain relieved by food


2.‐ History of chronic aspirin use
3.‐ Distended abdomen
4.‐ Positive fluid wave
1279 The client returning from a colonoscopy has been Correct answer: 1 Steatorrhea is often present in the client with Crohn’s disease. Diarrhea is also key feature, The core issue of the question is identification of common symptoms of Crohn’s disease.
given a diagnosis of Crohn’s disease. The oncoming but unlike ulcerative colitis, the loose stool usually does not contain blood and is usually less Use nursing knowledge and the process of elimination to make a selection.
shift nurse expects to note which of the following frequent in number of episodes.
manifestations in the client?
1.‐ Steatorrhea
2.‐ Firm, rigid abdomen
3.‐ Constipation
4.‐ Enlarged hemorrhoids

1280 A client is scheduled for a fecal fat exam. In planning Correct answer: 4 It is suggested that adults consume at least 100 grams of fat per day for 3 days before the test The core issue of the question is the ability to provide correct information when teaching a
client education, the nurse includes that which dietary and throughout specimen collection. The other responses provide incorrect information. client about proper preparation for fecal fat examination. Use nursing knowledge and the
modification is necessary before the test? process of elimination to make a selection.

1.‐ Eat a fat‐free diet the day before the exam.


2.‐ Eat a high‐fat meal right before the exam.
3.‐ Eat a diet containing 35 grams of fat for 36 hours before the test.
4.‐ Eat at least 100 grams of fat for 3 days before and during the test.

1281 The client with diverticular disease is scheduled for a Correct answer: 1 Perforation of an obstructed diverticulum can cause abscess formation or generalized The core issue of the question is the ability to identify the occurrence of peritonitis as a
sigmoidoscopy. He suddenly complains of severe peritonitis. The manifestations of peritonitis are abdominal guarding and rigidity and pain. complication of diverticular disease and determine the appropriate course of action. Use
abdominal pain. On examination, the nurse notes a Sigmoidoscopy is contraindicated in cases of perforation. Because treatment of this nursing knowledge and the process of elimination to make a selection.
rigid abdomen with guarding. What action should the complication is beyond the scope of independent nursing practice, the physician must be
nurse take next? notified.
1.‐ Notify the physician.
2.‐ Place the client in a more comfortable position.
3.‐ Keep the client distracted until the procedure begins.
4.‐ Tell the client that the test will show what is causing his problem.

1282 The nurse is educating the client with Correct answer: 1 Lifestyle modifications can minimize symptoms of GERD. Anything that increases intra‐ The core issue of the question is ability to identify risk factors that aggravate symptoms of
gastroesophageal reflux disease (GERD) about ways to abdominal pressure should be avoided, such as lifting weights. Obesity also aggravates GERD. Use nursing knowledge and the process of elimination to make a selection.
minimize symptoms. Which information in the client’s symptoms, but a body mass index of 23 is normal. Being a vegetarian does not increase risk,
history should the nurse address as an indicator that and calcium carbonate tablets often aid in symptom relief.
needs to be changed?
1.‐ Lifting weights for exercise
2.‐ Being a vegetarian
3.‐ Having a body mass index of 23
4.‐ Taking calcium carbonate tablets

1283 The client with a duodenal ulcer asks the nurse why Correct answer: 2 H. pylori infection is a major cause of peptic ulcers. Treatment includes eradicating H. pylori The core issue of the question is knowledge of etiology of peptic ulcers, including duodenal
an antibiotic is part of the treatment regimen. Which with antibiotics. The other responses are incorrect. ulcers. Use nursing knowledge and the process of elimination to make a selection.
information should the nurse include in the response?

1.‐ Antibiotics decrease the likelihood of infection.


2.‐ Many doudenal ulcers are caused by the Helicobacter pylori organism.
3.‐ Antibiotics are used in an attempt to sterilize the stomach.
4.‐ Many people have Clostridium difficile, which can lead to ulcer formation.

1284 The nurse should evaluate results of which of the Correct answer: 1 Many clotting factors are produced in the liver, including fibrinogen (factor I), prothrombin The critical word in the question is safe. With this in mind, the correct answer is one that
following laboratory tests for a client who has cirrhosis (factor II), factor V, serum prothrombin conversion accelerator (factor VII), factor IX, and factor could detect a complication of cirrhosis. Use nursing knowledge and the process of
in order to plan for safe care? X. The client’s ability to form these factors may be impaired with cirrhosis, putting the client at elimination to make a selection.
risk for bleeding. The prothrombin time will evaluate blood clotting ability; the others will not.
1.‐ Prothrombin time
2.‐ Urinalysis
3.‐ Serum lipase
4.‐ Serum troponin

1285 The nurse is caring for a client with a history of Correct answer: 3 Manifestations of chronic pancreatitis include nausea, vomiting, weight loss, flatulence, The core issue of the question is the ability to identify assessment findings that are
alcoholism. Which of the following findings would constipation, and steatorrhea (fatty stools) that result from a decrease in pancreatic enzyme consistent with the development of chronic pancreatitis. Use nursing knowledge and the
indicate that the client has possibly developed chronic secretion. Weight gain (option 1) is the opposite of what occurs with this disorder, while process of elimination to make a selection.
pancreatitis? options 2 and 4 are unrelated.
1.‐ Steady weight gain
2.‐ Flank pain on left side only
3.‐ Fatty stools
4.‐ Excessive hunger

1286 The nurse caring for a client with hemolytic jaundice Correct answer: 1 Hemolytic jaundice is caused by excessive breakdown of red blood cells, and the amount of The core issue of the question is knowledge of clinical indicators of hemolytic jaundice.
anticipates which of the following findings on the bilirubin produced exceeds the ability of the liver to conjugate it, so there is an increase in Use nursing knowledge and the process of elimination to make a selection.
laboratory results? indirect bilirubin. Unconjugated bilirubin is insoluble in water and is not found in the urine.

1.‐ Elevated serum indirect bilirubin


2.‐ Decreased serum protein
3.‐ Elevated urine bilirubin
4.‐ Decreased urine pH

1287 A client was admitted to the hospital with Correct answer: 4 Nausea and RUQ pain occur in cystic duct disease, but obstruction of the common bile duct The core issue of the question is knowledge of clinical indicators of common bile duct
cholelithiasis the previous day. Which of the following results in reflux of bile into the liver, which produces jaundice. Alkaline phosphatase increases obstruction. Think about the pathophysiology of blocked bile drainage and use the process
new assessment findings indicates to the nurse that with biliary obstruction but cholesterol level does not increase. of elimination to make a selection.
the stone has probably obstructed the common bile
duct?
1.‐ Nausea
2.‐ Elevated cholesterol level
3.‐ Right upper quadrant (RUQ) pain
4.‐ Jaundice

1288 The nurse caring for a client with uncomplicated Correct answer: 2 Obstructive biliary disease causes a significant elevation in alkaline phosphatase. Obstruction Use nursing knowledge and the process of elimination to make a selection.
cholelithiasis anticipates that the client’s laboratory in the biliary tract causes an elevation in direct bilirubin, not indirect bilirubin (option 4).
test results will show an elevation in which of the Options 1 and 3 are unrelated.
following?
1.‐ Serum amylase
2.‐ Alkaline phosphatase
3.‐ Mean corpuscular hemoglobin concentration (MCHC)
4.‐ Indirect bilirubin

1289 In caring for the client 4 days post‐cholecystectomy, Correct answer: 3 The T‐tube may drain 500 mL in the first 24 hours and decreases steadily thereafter. If there is The core issue of the question is knowledge of appropriate nursing action following
the nurse notices that the drainage from the T‐tube is excessive drainage, the nurse should further assess the drainage to be able to describe it notation of excessive T‐tube drainage. Use nursing knowledge and the process of
600 mL in 24 hours. Which is the appropriate action by accurately and notify the physician immediately. Option 1 would be contraindicated; options 2 elimination to make a selection.
the nurse? and 4 are of no help.
1.‐ Clamp the tube q 2 hours for 30 minutes
2.‐ Place the patient in a supine position
3.‐ Assess drainage characteristics and notify the physician
4.‐ Encourage an increased fluid intake
1290 The post‐cholecystectomy client asks the nurse when Correct answer: 1 When T‐tube drainage subsides and stools return to a normal brown color, the tube can be The core issue of the question is the appropriate timeframe for use of a T‐tube following
the T‐tube will be removed. Which of the following clamped 1 to 2 hours before and after meals in preparation for tube removal. If the client gallbladder surgery. Use nursing knowledge and the process of elimination to make a
responses by the nurse would be appropriate? tolerates clamping, the tube will then be removed. selection.

1.‐ “When your stool returns to a normal brown color, the tube can be removed.”
2.‐ “The tube will be removed at the same time as your staples.”
3.‐ “When the tube stops draining, it will be removed.”
4.‐ “The tube is usually removed the day after surgery.”

1291 Which of the following assessments made by the Correct answer: 1 Obstruction to portal blood flow causes a rise in portal venous pressure resulting in The core issue of the question is knowledge of associated findings in a client with portal
nurse could indicate the development of portal splenomegaly, ascites, and dilation of collateral venous channels predominantly in the hypertension. Use knowledge of the pathophysiology of the condition and the process of
hypertension in a client with cirrhosis? paraumbilical and hemorrhoidal veins, the cardia of the stomach, and extending into the elimination to make a selection.
esophagus. Bleeding gums would indicate insufficient Vitamin K production in the liver. Muscle
wasting commonly accompanies the poor nutritional intake commonly seen in clients with
cirrhosis. Hypothermia is an unrelated finding.
1.‐ Hemorrhoids
2.‐ Bleeding gums
3.‐ Muscle wasting
4.‐ Hypothermia

1292 The nurse is caring for a client who has ascites, and Correct answer: 3 Spironolactone (Aldactone) is used in clients with ascites that show no improvement with The core issue of the question is knowledge of medication effects in a client with ascites.
the health care provider prescribes spironolactone bedrest and fluid restriction. It inhibits sodium reabsorption in the distal tubule and promotes Use nursing knowledge related to pharmacology and the process of elimination to make a
(Aldactone). The client asks why this drug is being potassium retention by inhibiting aldosterone. The other options do not address this rationale. selection.
used. Which is the best response by the nurse?

1.‐ “This drug will help increase the level of protein in your blood.”
2.‐ “The drug will cause an increase in the amount of the hormone aldosterone your body produces.”
3.‐ “This medication is a diuretic but does not make the kidneys excrete potassium.”
4.‐ “This will help you excrete larger amounts of ammonia.”

1293 When caring for a client who has cirrhosis, the nurse Correct answer: 4 Asterixis, also called liver flap, is the flapping tremor of the hands when the arms are The core issue of the question is knowledge of typical assessment findings in a client with
notices flapping tremors of the wrist and fingers. How extended. Option 1 reflects hypocalcemia. Option 2 refers to spiderlike abdominal veins that cirrhosis. Use nursing knowledge and the process of elimination to make a selection.
should the nurse chart this finding? are also commonly found in clients with cirrhosis who have portal hypertension as a
complication. Option 3 is a specific odor noted in liver failure.

1.‐ “Trousseau’s sign noted.”


2.‐ “Caput medusa noted.”
3.‐ “Fetor hepaticus noted.”
4.‐ “Asterixis noted.”

1294 A mother arrives at the pediatric clinic with her 6‐ Correct answer: 3 It is a common finding that when the infant with an umbilical hernia cries, the hernia The core issue of the question is knowledge of the consequences of umbilical hernia and
month‐old infant. While the nurse is assessing the protrudes. It is not going to rupture. The family is instructed not to apply tape, straps, or coins knowledge of therapeutic communication techniques. Use this knowledge and the process
child, the mother points to the umbilicus and says: to the umbilicus to reduce the hernia. of elimination to make a selection.
“What am I going to do about this? When he cries, it
looks like it’s going to burst.” Which of the following is
the best response by the nurse?
1.‐ “It’s best if you let him cry. Just let him do what he wants.”
2.‐ “It probably won’t rupture unless he gets excessively upset. I wouldn’t worry about it at this time.”
3.‐ “I know it looks frightening, but it really won’t burst.”
4.‐ “Put a binder around it, and that will keep it from bursting when he gets mad.”
1295 A 9‐year‐old male client with severe esophagitis is 12 Correct answer: 3 Pain management is a high priority following gastric surgery, and the nurse should use age‐ The core issue of the question is knowledge of appropriate interventions in the first 24
hours status/post‐Nissen fundoplication for appropriate tools to assess for pain, such as the Wong FACES rating scale. A gastrostomy tube hours following gastric surgery. Use knowledge that the gastric tube should not be
gastroesophageal reflux. To implement appropriate or nasogastric tube placed during surgery is kept in place to maintain gastric decompression. manipulated or used for feeding to eliminate some options. Use nursing knowledge of
nursing care, the nurse should do which of the The child is kept NPO until bowel function returns. The use of a pH probe to measure gastric routine postoperative care and the process of elimination to make a final selection.
following? acidity is not necessary.
1.‐ Encourage him to take small amounts of clear liquids every 4 hours.
2.‐ Administer NG or gastrostomy feedings every 4 hours.
3.‐ Ask him to choose a face on the Wong FACES pain rating scale.
4.‐ Insert a pH probe to monitor esophageal acidity.

1296 A 10‐month‐old female infant with biliary atresia is Correct answer: 4 Kasai procedure is palliative, and prognosis is best if performed before 10 weeks of age. Its The core issue of the question is knowledge of the typical success of surgery with Kasai
being discharged after a Kasai procedure. Which purpose is to achieve biliary drainage and avoid liver failure. A liver transplant is required in 80 procedure in an infant with biliary atresia. Use nursing knowledge and the process of
statement, if made by the parents, indicates that to 90% of cases. elimination to make a selection.
teaching with regard to prognosis has been
understood?
1.‐ “We are glad this problem was found so early; now everything will be fine.”
2.‐ “We will stop her liver medicine now that she is being discharged.”
3.‐ “We are happy to be able to stop that special formula and many of those vitamins.”
4.‐ “We know that even though surgery is over, she will likely need a liver transplant.”

1297 Which of the following diagnostic assessment Correct answer: 1 Measuring urine specific gravity provides data about the concentration of urine and provides The core issue of the question is dehydration and thus the correct option is one that
methods would the nurse expect to be ordered for a information regarding hydration. Urine specific gravity is elevated in dehydration and would be addresses fluid balance in the body in some way. Use nursing knowledge and the process of
child with dehydration as a result of vomiting and decreased with high fluid intake. The other tests listed are not indicated in the care of the elimination to make a selection.
diarrhea? dehydrated client.
1.‐ Serum sodium and serum osmolality
2.‐ Stool for ova and parasites
3.‐ Upper‐gastrointestinal series
4.‐ Seventy‐two‐hour fecal fat collection

1298 The nurse is caring for a child with a history of severe Correct answer: 3 In severe diarrhea, excess bicarbonate (base) is lost, which predisposes to metabolic acidosis. The core issue of the question is the ability to correlate acid‐base imbalance with a
diarrhea. Which of the following notations in the There is also carbohydrate malabsorption and depletion of glycogen stores, resulting in fat diagnosis of diarrhea. Recall that bicarbonate is a base and that the respiratory system is
medical record about acid‐base imbalance would the metabolism. Ketoacids are the by‐products of fat metabolism, which adds to the metabolic not directly involved to make a selection.
nurse expect to find? acidosis. It is not a respiratory problem.
1.‐ Respiratory acidosis
2.‐ Respiratory alkalosis
3.‐ Metabolic acidosis
4.‐ Metabolic alkalosis

1299 A nurse who floats to the infant and toddlers nursing Correct answer: 2 ESSR is the abbreviation for the four key steps in feeding the infant or child with cleft lip or The core issue of the question is knowledge of a feeding technique in cleft lip or palate
unit asks the pediatric nurse about the notation “ESSR” palate. These steps are to Enlarge nipple; Stimulate suck reflex; Swallow fluid; Rest after each that reduces the risk of aspiration. Use nursing knowledge and the process of elimination to
on the care plan of a client. The nurse explains that swallow. It does not refer to treatment of gastroesophageal reflux, pyloric stenosis, or make a selection.
this documentation refers to Hirschsprung’s disease.
1.‐ the feeding method for children with gastroesophageal reflux.
2.‐ the feeding method for children with cleft lip or palate.
3.‐ the procedure for repair of pyloric stenosis.
4.‐ the procedure for repair of Hirschsprung’s disease.

1300 A child with Hirschsprung’s disease is being Correct answer: 3 It is important that any signs of infection be reported at once. After Soave procedure, the The core issue of the question is knowledge of routine discharge teaching following an
discharged after Soave endorectal pull‐through colostomy is usually closed and normal bowel function is expected (options 1 and 4). No rectal abdominal surgical procedure. Use nursing knowledge and the process of elimination to
procedure for colostomy closure. Which of the irrigations are necessary (option 2). make a selection.
following items should the nurse include in the
discharge teaching plan?
1.‐ Stools may be infrequent and uncomfortable for the first few weeks.
2.‐ It will be necessary to perform weekly rectal irrigations for approximately 6 weeks.
3.‐ Report fever, increasing pain or discomfort, or redness of the incision to the surgeon.
4.‐ Stools will be fatty for a week or so and then gradually return to normal.

1301 The nurse is taking a history from the mother of a Correct answer: 1 Acute episodes of celiac disease are characterized by bulky, frothy stools, anorexia, and The core issue of the question is knowledge of assessment findings in a client with celiac
child being admitted with flare‐up of celiac disease. irritability. Pain does not occur in waves prior to mealtimes. disease. Use nursing knowledge and the process of elimination to make a selection.
What piece of information would the nurse expect the
mother to report?
1.‐ Stools that are fatty
2.‐ An increased appetite with no weight gain
3.‐ Episodes of abdominal pain that are wavelike just before meals
4.‐ Soft, formed stools

1302 The mother of a child undergoing an emergency Correct answer: 2 Parents often react to a child’s illness with feelings of guilt for not recognizing the severity of The core issue of the question is the ability to formulate a therapeutic response to a
appendectomy tells the nurse, “If I had brought him in the condition sooner. A response that provides emotional support and reduces parental parent who indicates distress about not seeking help earlier for an ill child. Use nursing
yesterday when he complained of an upset stomach, anxiety encourages parents to feel confident in their abilities as caregiver. The other responses knowledge of therapeutic communication skills and the process of elimination to make a
this wouldn’t have happened.” Which of the following ignore the parent’s feelings (option 1) or add to the parent’s guilt or stress (options 3 and 4). selection.
is the best response by the nurse?

1.‐ “It’s okay; you got him here just in time before it ruptured.”
2.‐ “It is often difficult to predict when a simple complaint will become more serious.”
3.‐ “Next time he seems sick, you should bring him in immediately.”
4.‐ “Sometimes parents can make a mistake without meaning to do so.”

1303 The nurse is teaching home feeding guidelines to the Correct answer: 3 Finger foods are helpful in encouraging children with failure to thrive to increase food intake. The core issue of the question is the intervention that will help to increase food intake in a
mother of a child with nonorganic failure to thrive. The parent should also be taught to encourage increased food intake and to make mealtimes child with nonorganic failure to thrive. Use nursing knowledge and the process of
Essential information for the nurse to include would be regular, nonstressful, but structured family events. elimination to make a selection.
the importance of
1.‐ restricting eating except at mealtimes.
2.‐ allowing the child to eat alone to minimize distraction.
3.‐ allowing the child to snack on finger foods, such as Cheerios, french fries, and bananas.
4.‐ a relaxed mealtime with few limits on behavior.

1304 The nurse is admitting a child with a diagnosis of “rule Correct answer: 1, 2, 4 Manifestations of appendicitis often include generalized abdominal pain progressively The core issue of the question is knowledge of manifestations that are consistent with
out appendicitis.” The nurse assesses this client for worsening and localizing in the right lower quadrant at McBurney’s point, nausea and appendicitis. Use knowledge that the affected area is the large intestine to eliminate
which of the following manifestations? Select all that vomiting, fever, chills, anorexia, diarrhea or acute constipation, and elevated WBC count: indigestion (stomach area, too vague) and fatty stools (small intestine absorption problem).
apply. 15,000 to 20,000 cells/mm&lt;sup&gt;3&lt;/sup&gt;. Fatty stools and indigestion are not part
of the clinical picture.
1.‐ Generalized abdominal pain
2.‐ Pain localizing in right lower quadrant
3.‐ Fatty stools
4.‐ Elevated white blood cell count
5.‐ Indigestion
1305 Whenever the parents of a 10‐month‐old leave their Correct answer: 2 Infants and toddlers between the ages of 6 months and 30 months experience separation Core concepts are the age of the child and recent hospitalization. Knowledge of coping
hospitalized child for short periods, the child begins to anxiety. There are three stages of separation anxiety. The child who demonstrates crying and mechanisms of infants and separation anxiety reaction are needed to answer the question
cry and scream. The nurse explains that this behavior rejecting anyone other than the parent is in protest, the first stage of separation anxiety. This correctly.
demonstrates that the child: behavior does not exhibit spoiling or any indication of discomfort. The second stage is depair.
The child expresses hopelessness, appears quiet, and is withdrawn. The third stage is
detachment. The child becomes interested in the environment, especially the cregivers. If the
parents return, the child ignores them.

1.‐ Needs to remain with the parents at all times.


2.‐ Is experiencing separation anxiety.
3.‐ Is experiencing discomfort.
4.‐ Is extremely spoiled.

1306 A 9‐year‐old with severe esophagitis is 12 hours Correct answer: 3 A gastrostomy tube or nasogastric tube placed during surgery is kept in place to maintain Options 1 and 2 focus on feeding. Recall that feedings cannot be restarted until bowel
status/post‐Nissen fundoplication for gastric decompression. The child is kept NPO until bowel function returns. Appropriate pain sounds have returned.
gastroesophageal reflux (GER). To implement management is an ethical nursing obligation; a pain rating scale recognizes the child's right to
appropriate nursing care, the nurse should do which of be in control.
the following?
1.‐ Encourage him to take small amounts of clear liquids every four hours
2.‐ Administer NG or gastrostomy feedings every four hours
3.‐ Ask him to rank his pain on a scale of zero to 10 when zero is no pain and 10 is the worst possible pain
4.‐ Insert a pH probe to monitor esophageal acidity

1307 A mother asks the pediatric nurse about what she Correct answer: 2 Introduction of solid food is recommended at age 4 to 6 months, when the gastrointestinal Critical words are "begin to feed" and "6‐month‐old." Knowledge of introduction of solid
should begin to feed her 6‐month‐old infant. The system has matured sufficiently to handle complex nutrients. The suck reflex and tongue‐ food is necessary to choose the correct answer.
correct response is: thrust reflex diminish at 4 months of age. Rice cereal is the first solid food because it is a rich
source of iron and rarely induces allergenci reactions. Fruits and vegetables, good sources of
vitamins and fiber, are introduced after cereal, one at a time to determine allergic reactions.
Egg whites are highly allergenic.

1.‐ Egg whites are the least allergenic food to be introduced to the baby's diet.
2.‐ Rice cereal is the first solid introduced that is least allergenic of the cereals.
3.‐ Formula is the only source of nutrition given for the first year.
4.‐ Fruits and vegetables are good sources of iron.

1308 A child has been admitted to the hospital with Correct answer: 1, 5 Urine specific gravity is a measurement of the concentration of urine and provides One of the options is a test that a nurse will not perform so that one can be eliminated.
dehydration. Which of the following measures would information regarding hydration. Urine specific gravity is elevated in dehydration. Careful Only one of the lab tests will give any information about the hydration status of the child.
the nurse expect to be included in the plan of care? measurement of intake and output, level of consciousness, skin turgor and mucous membrane
Select all that apply. moisture will also indicate the child’s status. Sending a stool for ova and parasites evaluation
could be indicated if this was suspected as a cause of diarrhea, but the stem of the question
does not state that the child has diarrhea. Upper GI series and stool evaluation for fecal fat
would help to diagnose GI disorders.

1.‐ Urine specific gravity


2.‐ Stool ova and parasites
3.‐ Upper‐gastrointestinal (GI) series
4.‐ 72‐hour fecal fat collection
5.‐ Intake and output

1309 The nurse is caring for a child with a history of severe Correct answer: 3 In severe diarrhea, excess bicarbonate is lost. There is also carbohydrate malabsorption and Recall that loss of bicarbonate would cause acidosis and the condition is metabolic in
diarrhea. Which of the following acid‐base depletion of glycogen stores, resulting in fat metabolism. Ketoacids are the byproducts of fat origin rather than a respiratory problem.
abnormalities would the nurse assess for as a possible metabolism. For both of these reasons, the client can develop acidosis. The nature of the
consequence of the diarrhea? problem is metabolic rather than respiratory.
1.‐ Respiratory acidosis
2.‐ Respiratory alkalosis
3.‐ Metabolic acidosis
4.‐ Metabolic alkalosis

1310 The nurse is reviewing nursing notes and sees a Correct answer: 2 ESSR feeding technique for cleft lip or palate: Enlarge nipple; Stimulate suck reflex; Swallow Knowledge of the clinical therapy for cleft lip and palate will aid in choosing the correct
notation of "ESSR" in the medical record of a child with fluid; Rest after each swallow. It has nothing to do with a suture maintenance tool, the surgical answer. Recall that feeding is a major concern for the child with cleft lip and palate.
a cleft lip and palate. The nurse interprets that the procedure, or a method of positioning the infant.
notation of "ESSR" is referring to:

1.‐ The suture maintenance tool.


2.‐ The feeding method.
3.‐ The procedure for repair.
4.‐ The positioning of the infant.

1311 A child with Hirschsprung disease is being discharged Correct answer: 3 It is important that any signs of infection be reported at once. After Soave procedure, the Remember that the stem states for colostomy closure, even if the medical name of the
after Soave endorectal pull‐through procedure for colostomy is usually closed and normal bowel function is expected. procedure is not recalled. This should help to eliminate most of the options.
colostomy closure. Which of these measures should
the nurse include in the home care plan?

1.‐ Refer the parents to an enterostomal therapist for ostomy care


2.‐ Teach parents how to perform weekly rectal irrigations
3.‐ Teach parents signs and symptoms of infection
4.‐ Teach parents PCA pain‐control methods

1312 A child is brought to the Emergency Department with Correct answer: 1 Corrosive agents cause the signs and symptoms listed. Indications of aspirin overdose are Aspirin and acetaminophen can easily be eliminated as they would not cause the swelling
excessive drooling, edema of lips and tongue, swollen nausea, vomiting, diaphoresis, and seizures. Hydrocarobons cause nausea, vomiting, cyanosis, of the mouth. It is then necessary to choose between the hydrocarbons and corrosive
mucous membranes, and is hypotensive and and altered sensorium, and acetaminophen casues nausea, vomiting, diaphoresis, and later, agent.
tachycardiac. Based on this initial assessment, the jaundice.
nurse suspects that the child has ingested which of the
following agents?
1.‐ Corrosive agent
2.‐ Aspirin
3.‐ Hydrocarbons
4.‐ Acetaminophen

1313 The mother of a child undergoing an emergency Correct answer: 2 Parents often react to a child's illness with feelings of guilt for not recognizing the severity of Consider which response will reduce parental feelings of guilt.
appendectomy tells the nurse "If I had brought him in the condition sooner. Emotional support and reduction of parental anxiety encourages parents
yesterday when he complained of an upset stomach, to feel confident in their abilities as caregiver.
this wouldn't have happened." The nurse's best
response is:

1.‐ "It's okay; you got him here just in time before it ruptured."
2.‐ "It is often difficult to predict when a simple complaint will become more serious."
3.‐ "Next time he seems sick, you should bring him in immediately."
4.‐ "Sometimes parents can make a mistake without meaning to do so."

1314 A child has been diagnosed with non‐organic failure Correct answer: 1, 4, 5 Non‐organic failure to thrive is not due to metabolic or organic problems or the absence of Option 2 can be eliminated as it is not associated with non‐organic failure to thrive. All
to thrive. In addition to poor weight gain, the nurse food availability. Children with this form of malnutrition often display other non‐specific other responses are correct.
would expect the child to exhibit which of the symptoms related to the emotional illness.
following? Select all that apply.
1.‐ Erratic sleep patterns
2.‐ Diarrhea
3.‐ Developmental delays
4.‐ Irritability and being difficult to soothe
5.‐ Food refusal

1315 A child who underwent cleft palate repair has just Correct answer: 3 Elbow restraints are used to keep hands away from the mouth after cleft palate surgery. This Consider what movements elbow restraints will allow the child to determine the right
returned from surgery with elbow restraints in place. precaution will be maintained at home until the palate is healed, usually 4 to 6 weeks. They are answer.
The parents question why their child must have the not used to protect the IV site, maintain NPO status, or maintain body alignment.
restraints. The nurse would give which of the following
as the best explanation to the parents?

1.‐ “This device is frequently used postoperatively to protect the IV site in small children.”
2.‐ “The restraints will help us maintain proper body alignment.”
3.‐ “Elbow restraints are used postoperatively to keep children’s hands away from the surgical site.”
4.‐ “The restraints help maintain the child’s NPO status.”

1316 The nurse is caring for an infant vomiting secondary Correct answer: 2 In pyloric stenosis, bile is unable to enter the stomach from the duodenum because the Consider the site of the pylorus to determine the correct answer.
to pyloric stenosis. The mother questions why the pylorus muscle is hypertrophied, which causes the obstruction.
vomitus of this child appears different from that of her
other children when they have the flu. The nurse
would explain that the emesis of an infant with pyloric
stenosis does not contain bile because:

1.‐ The GI system is still immature in newborns and infants.


2.‐ The obstruction is above the bile duct.
3.‐ The emesis is from passive regurgitation.
4.‐ The bile duct is obstructed.

1317 The nurse is teaching the parents of a child with celiac Correct answer: 2, 3, 4 Most children who remain on a gluten‐free diet remain healthy and free of symptoms and Children with celiac disease can eat corn and rice. All other grains need to be eliminated
disease about the dietary restrictions. The nurse would complications. Gluten is a protein found in wheat, barley, rye, and oats. For this reason, from the diet.
explain that the most appropriate diet for their child is appropriate foods need to be free of these grains.
a diet that is free of which of the following? (Select all
that apply.)
1.‐ Rice
2.‐ Wheat
3.‐ Oats
4.‐ Barley
5.‐ Corn

1318 A high school experiences an outbreak of hepatitis B. Correct answer: 4 HBV vaccine provides active immunity, and current recommendations include immunizations The student must know which forms of hepatitis are blood‐borne and which are not. The
In teaching the high school students about hepatitis B, for all newborns, as well as for several high‐risk groups. Hepatitis B is spread by blood and only form of hepatitis for which there is a vaccine is HBV.
the school nurse would explain: body fluids, including sexual contact, not the fecal‐oral route. The disease can exist in a carrier
state.
1.‐ Hepatitis B cannot exist in a carrier state.
2.‐ Hepatitis B is primarily transmitted through the fecal‐oral route.
3.‐ Immunity to all types of hepatitis will occur after this current attack.
4.‐ Hepatitis B can be prevented by receiving the HBV vaccine.
1319 A 4‐month‐old infant is admitted to the nursing unit Correct answer: 1, 2, 5 The nurse would expect an increased desire to drink fluids and a higher specific gravity caused Two of the options are age related and appropriate for the infant. Polyuria and weight gain
with moderate dehydration. Which of the following by the concentration of urine. The heart rate would be elevated, and the fontanels sunken. The would not be symptoms of dehydration.
symptoms does the nurse suspect led to the diagnosis degree of dehydration is based on the percent of weight loss, so a weight gain would not be
of moderate dehydration in this child? (Select all that likely. Diminished urine output with elevated specific gravity is an expected normal finding in
apply.) dehydration. Capillary refill is slowed, especially in children under 2 years of age.

1.‐ Elevated heart rate


2.‐ Urine specific gravity of 1.038
3.‐ Weight gain
4.‐ Polyuria
5.‐ Slow capillary refill

1320 While performing a newborn assessment, the nurse Correct answer: 1 Clinical findings will vary in infants born with congenital diaphragmatic hernias, but the first Note that all findings indicate the absence of normal findings on the left side of the chest.
notes the infant is having difficulty breathing, with indications are of respiratory distress. Further assessment will reveal bowel sounds
nasal flaring, cyanosis, retractions, and an absence of auscultated over the chest, cardiac sounds on the right of the chest, and a sunken abdomen
breath sounds on the left side. The nurse auscultates with a barrel‐shaped chest.
the apical pulse on the right side of the chest. The
nurse notifies the physician immediately because of
suspected:

1.‐ Diaphragmatic hernia.


2.‐ Pyloric stenosis.
3.‐ Cleft palate.
4.‐ Omphalocele.

1321 The nurse has taught dietary restrictions to a 7‐year‐ Correct answer: 4 Celiac disease is characterized by intolerance for gluten. Gluten is found in wheat, barley, rye, Determine which menu does not contain any rye, wheat, barley, or oats. Alternatively,
old child who has celiac disease. After teaching, the and oats. This includes bread, cake, doughnuts, cookies, and crackers, as well as processed select the menu that contains one or more items that are rice or corn.
child is allowed to choose a meal from the hospital foods that contain gluten as filler.
menu. The nurse evaluates that teaching was effective
when the child chooses which of the following?

1.‐ Beef and barley soup, rice cakes, and celery


2.‐ Ham and cheese sandwich with lettuce and tomato on rye toast
3.‐ Beef patty on a hamburger bun and home fries
4.‐ Baked chicken, green beans, and a slice of cornbread

1322 An infant returns from initial surgery for Correct answer: 2 The corrective surgery for Hirschsprung’s disease requires pulling the end of the normal The critical word in the question is “exclude.” With this in mind, choose the option that
Hirschsprung’s disease. All of the following are routine bowel through the muscular sleeve of the rectum. With this type of procedure, rectal represents an incorrect or unacceptable nursing action.
postoperative nursing interventions. Because of the temperatures and any invasive procedure would be avoided to allow proper healing to occur.
type of surgery this child had, the nurse would exclude
which of them?
1.‐ Maintaining the child NPO until bowel sounds return
2.‐ Monitoring rectal temperature every 4 hours
3.‐ Reuniting the parents with the child as soon as possible
4.‐ Assessing the surgical site every 2 hours

1323 A 3‐month‐old infant has gastroesophageal reflux Correct answer: 2 Infants with GER should be given small, frequent feedings. After a feeding the infant should be Consider which intervention decreases pressure on the abdomen.
(GER) but is thriving without other complications. The placed in a prone position with the head of the bed elevated. A harness can be used to help
mother wants to know what she can do differently to maintain this position. Infant seats should be avoided because of the increased intraabdominal
decrease the reflux. Which intervention should the pressure this position creates.
nurse suggest to minimize reflux?

1.‐ Discontinue breast‐feeding immediately.


2.‐ Increase frequency of feedings and keep them small.
3.‐ Place the baby in prone position with the head flat.
4.‐ Place the infant in a car seat after feeding.

1324 A 10‐year‐old boy has been admitted with a diagnosis Correct answer: 3 Signs and symptoms of a ruptured appendix include fever, sudden relief from abdominal pain, Read the question carefully and eliminate each of the incorrect options because there is
of “rule out appendicitis.” While the nurse is guarding, abdominal distention, rapid shallow breathing, pallor, chills, and irritability. no data in the stem of the question to support them. As an alternative, recall pain pattern
conducting a routine assessment, the boy states, “It in appendicitis before and after rupture to choose accurately.
doesn’t hurt anymore.” The nurse suspects that:

1.‐ The boy is afraid of going to surgery.


2.‐ The boy is having difficulty expressing his pain adequately.
3.‐ The appendix has ruptured.
4.‐ This is a method the boy uses to receive attention.

1325 An 18‐month‐old child with a history of cleft lip and Correct answer: 3 During the immediate postoperative period, protecting the operative site is a priority in the The core concepts for this question are cleft palate surgery and postoperative care. Since
palate has been admitted for palate surgery. The nurse nursing care of this child. A toothbrush should be a familiar object to an 18‐month‐old child. the stem states why use of toothbrush care is avoided, the learner must connect this to
would provide which explanation about why a Deciduous (primary) teeth are still present at this age and are replaced by permanent postoperative care.
toothbrush should not be used immediately after (secondary) teeth around 6 years of age. Oral care will be performed according to the
surgery? physicians’ orders but usually consists of cleansing the area with sterile water.

1.‐ The toothbrush would be frightening to the child.


2.‐ The child no longer has deciduous teeth.
3.‐ The suture line could be interrupted.
4.‐ The child will be NPO.

1326 The nurse instructs the parents about postoperative Correct answer: 3 The goal after pyloromyotomy is to slowly increase the volume of feeding while preventing Recognize the problem with pyloric stenosis is not related to vomiting but due to a tight
feeding following their infant’s pyloromyotomy. The vomiting. Bubbling is essential after feed. Rocking is avoided as this might increase vomiting. pyloric muscle that will be incised.
nurse evaluates that the parents understand the Antiemetics are not helpful as the vomiting is not associated with nausea.
instructions when the parents state they will:

1.‐ Avoid bubbling the baby after feeding to prevent vomiting.


2.‐ Rock the baby to sleep after feeding to keep the infant calm.
3.‐ Slowly increase the volume offered according to the physician’s orders.
4.‐ Maintain the infant on antiemetics to prevent vomiting.

1327 Immediately after the delivery of an infant with an Correct answer: 4 Omphaloceles are congenital malformations in which abdominal contents protrude through The stem of the question is seeking the first action of the nurse. Ordering blood is not a
omphalocele, the nurse would take which of the the umbilical cord. The protrusion is covered by a translucent sac; immediately after birth, the nursing function. The nurse would not insert an orogastric tube without a medical order.
following actions? sac requires priority attention. The sac is covered with sterile gauze soaked in normal saline Therefore, choose the most important action between the other two options.
solution to prevent drying and injury.
1.‐ Weigh the infant
2.‐ Insert an orogastric tube
3.‐ Call the blood bank for 2 units of blood
4.‐ Cover the sac with moistened sterile gauze

1328 While gathering admission data on a 16‐month‐old Correct answer: 1, 3, 5 Infants with Hirschsprung’s disease usually display failure to thrive, poor weight gain, and Consider symptoms of Hirschsprung’s disease without looking at the options. Then review
child, the nurse notes all the following abnormal delayed growth. Vomiting is usually bile stained. The child will demonstrate alternating the options to determine which options match those symptoms.
findings. Which finding is related to a diagnosis of constipation and diarrhea, but the stools are not bloody. Decreased urine output and
Hirschsprung’s disease? (Select all that apply.) intermittent sharp pain are nonspecific symptoms that can be associated with many different
diseases and disorders.
1.‐ Bile‐stained vomitus
2.‐ Decreased urine output
3.‐ Poor weight gain since birth
4.‐ Intermittent sharp pain
5.‐ Alternating constipation and diarrhea

1329 A 6‐week‐old infant is brought into the pediatrician’s Correct answer: 4 Small, frequent feedings followed by placing the infant at a 30‐ to 45‐degree angle has been Knowledge of the care of the infant with gastroesophageal reflux will aid in choosing the
office with a history of frequent vomiting after shown to be beneficial in treating gastroesophageal reflux. Diluting the formula would not be correct answer. Consider which activities will decrease vomiting.
feedings and failure to gain weight. The diagnosis of recommended because the infant needs the calories from the full‐strength formula. It may be
gastroesophageal reflux is made and discharge recommended to thicken the formula with rice cereal. It is recommended to burp frequently;
instructions are begun. While planning discharge to delay burping would only increase the occurrences of reflux. Gastroesophageal reflux is not
teaching on feeding techniques with the parents, the related to milk intolerance so changing the formula would not help the child.
nurse should include instructions to:

1.‐ Dilute the formula.


2.‐ Delay burping to prevent vomiting.
3.‐ Change from milk‐based formula to soy‐based formula.
4.‐ Position the infant at a 30‐ to 45‐degree angle after feedings.

1330 A 14‐year‐old boy is brought into the Emergency Correct answer: 2 An ice bag may help relieve his pain. A rectal tube is contraindicated because it stimulates Knowledge of the care of the child with appendicitis will aid in choosing the correct
Department with a diagnosis of rule out appendicitis. bowel motility, which would increase the pain. A heating pad is contraindicated because it answer. One choice is apply an ice bag, another choice is a heating pad. These are
He is complaining of right lower quadrant pain. The increases the flow of blood to the appendix and may lead to rupture. An antispasmodic agent opposites, so there is an increased likelihood that one is right and the other is wrong.
nurse’s most appropriate action to assist in managing would not be beneficial for the pain associated with appendicitis. Antispasmodic agents are
his pain would be to: typically used to inhibit smooth muscle contractions.

1.‐ Insert a rectal tube.


2.‐ Apply an ice bag.
3.‐ Apply a heating pad.
4.‐ Administer an intravenous antispasmodic agent.

1331 The nurse has completed discharge teaching on the Correct answer: 1 Discharge planning focuses on educating the parents in maintaining a gluten‐free diet for the Because celiac disease is a type of intolerance, the child will not outgrow the intolerance
dietary regimen of a child with celiac disease. The child. Dietary modifications are lifelong and should not be discontinued when the child is and must continue the diet throughout life.
nurse recognizes that client education has been symptom‐free. Symptoms will return if dietary restrictions are not maintained.
successful when the mother states that the child must
comply with the gluten‐free diet:
1.‐ Throughout life.
2.‐ Until the child has achieved all major developmental milestones.
3.‐ Only until all symptoms are resolved.
4.‐ Until the child has reached adolescence.

1332 An appropriate nursing assessment of an infant Correct answer: 4 Measuring the abdominal girth frequently aids in early detection of necrotizing enterocolitis, Because the disease is a gastrointestinal disease, locate symptoms that relate to the GI
suspected of having necrotizing enterocolitis would be: which, in turn, minimizes loss of bowel. Assessment of gastric pH is not done. Frequent system.
assessment of the neurologic status is not specific to this disease. Rectal temperatures are
contraindicated because of the increased risk of perforation.
1.‐ pH of the stomach contents.
2.‐ Neurological status every 2 hours.
3.‐ Rectal temperature every 2 hours.
4.‐ Abdominal girth every 4 hours.

1333 The nurse is developing a teaching plan for the Correct answer: 2 Hepatitis A is highly contagious and is transmitted primarily through the fecal‐oral route. The Consider how the virus spreads to determine the correct answer. Options 3 and 4 can be
parents of an infant diagnosed with hepatitis A. Which virus is transmitted by direct person‐to‐person contact or through ingestion of contaminated eliminated since they include a separate host.
of the following instructions would be included to food or water, especially shellfish growing in contaminated water. The remaining answers are
reduce the risk for transmission of this disease? related to other infectious diseases.

1.‐ Disinfect all clothing and eating utensils on a daily basis.


2.‐ Tell family members to wash their hands frequently.
3.‐ Spray the yard to eliminate infected insects.
4.‐ Vacuum the carpets and upholstery to rid the house of the infectious host.

1334 Which of the following signs would the nurse Correct answer: 3 Mucous membranes typically appear dry when moderate dehydration is observed. Other Option 1 is the only symptom that is age related and is not associated with a toddler. Two
recognize as an indication of moderate dehydration in typical findings associated with moderate dehydration include restlessness with periods of options are not associated with dehydration, leaving only one response as correct.
a preschooler? irritability (especially infants and young children), rapid pulse, poor skin turgor, delayed
capillary refill, and decreased urine output. Both anterior and posterior fontanels are closed on
a preschool‐age child. The skin is usually dry with decreased elasticity, not diaphoretic. Urine
specific gravity increases with decreased urine output associated with dehydration.

1.‐ Sunken fontanel


2.‐ Diaphoresis
3.‐ Dry mucous membranes
4.‐ Decreased urine specific gravity

1335 A client is being admitted to a hospital unit Correct answer: 1 Pain over McBurney's point, the point halfway between the umbilicus and the iliac crest, is Look for an option describing right sided abdominal pain.
complaining of severe pain in the lower abdomen and diagnostic for appendicitis. Assessment for rebound tenderness would also assist in the
is lying on the bed with his knees flexed. Admission diagnosis. Options 2 and 3 are common with ulcers; option 4 may suggest ulcerative colitis or
vital signs reveal an oral temperature of 101.2 degrees diverticulitis.
F. Which of the following would confirm a diagnosis of
appendicitis?

1.‐ The pain is localized at a position halfway between the umbilicus and the right iliac crest.
2.‐ The client describes the pain as occurring 2 hours after eating.
3.‐ The pain subsides after eating.
4.‐ The pain is in the left lower quadrant.

1336 An elderly client presents with fever, leukocytosis, left Correct answer: 2 Fever indicates an infection, ruling out options 3 and 4. Appendicitis typically causes pain in Select an ‘itis’ as this indicates infection. Appendix pain is right‐sided.
lower quadrant pain, and diarrhea alternating with the umbilical area or right lower quadrant and is not usually accompanied by diarrhea. Fever
constipation. The nurse concludes that these are and diarrhea accompany diverticulitis.
frequently seen in clients with:

1.‐ Appendicitis.
2.‐ Diverticulitis.
3.‐ Peptic ulcer disease.
4.‐ Irritable bowel syndrome.

1337 A client says to the nurse, "My doctor told me my Correct answer: 4 H. pylori causes release of toxins and enzymes that promote inflammation and ulceration. It is Option 4 names the commonly associated bacteria.
ulcer may have been caused by bacteria. I thought not spread from one person to another. Contributing factors are those that increase secretion
ulcers were caused by diet and too much stress." of acid and pepsin.
Which of the following responses by the nurse is the
best?
1.‐ "If it was caused by bacteria, you would have a fever as a result of the inflammatory process."
2.‐ "We know that ulcers are communicable. They can be spread easily. Be careful you don't spread it to your children."
3.‐ "Diet and stress have nothing to do with developing an ulcer."
4.‐ "Even though the bacteria <i>Helicobacter pylori</i> causes inflammation, other factors may cause increased acid in the stomach."

1338 In caring for a client with a hiatal hernia, which of the Correct answer: 1 Heavy lifting is one factor that leads to development of a hiatal hernia. Dietary factors involve Recall that heavy lifting is associated with hernia, even hiatal hernia.
following should be included in a teaching plan limiting fat intake or spicy foods, not restricting the client to soft foods. It is more prevalent in
regarding causes? individuals who are middle‐aged or older. Fair‐skinned individuals are not prone to this
condition.
1.‐ To avoid heavy lifting
2.‐ A dietary plan based on soft foods
3.‐ Its prevalence in young adults
4.‐ Its prevalence in fair‐skinned individuals

1339 An elderly male client is worried about bright red Correct answer: 2 Red blood in the stool is more characteristic of left‐sided cancer of the colon. If blood occurs Recall that the descending colon is nearer to the rectum, so the blood from this area
blood in his stool along with feeling tired and worn in the stool at all in right‐sided cancer of the colon or gastric ulcers, it will be black or tarry. would be bright red, or fresh.
out. The nurse determines that these symptoms are There is no blood in the stool of a client with gallbladder disease. Remember, bright red blood
characteristic of: can also occur with hemorrhoids, but this choice was not available.

1.‐ Ascending (right‐sided) colon cancer.


2.‐ Descending (left‐sided) colon cancer.
3.‐ Gallbladder disease.
4.‐ Gastric ulcers.

1340 A client states, "My doctor told me to quit taking Correct answer: 2 Aspirin is one of the nonsteroidal anti‐inflammatory drugs (NSAIDs). These drugs are Select the response that allows the exploration of an alternative medication.
aspirin since I've developed this ulcer. I have to take predisposing or contributing factors in the development of an ulcer, because of the effect on
aspirin to keep my arthritis from hurting. I don't know prostaglandins. Many of the medications used for arthritis may also irritate an ulcer; therefore,
what to do." Which response on the part of the nurse a physician should be consulted.
is best?
1.‐ "Let's worry about treating your ulcer‐‐your arthritis will have to wait."
2.‐ "Aspirin is one of the medications that makes an ulcer worse; another medicine can be ordered by the doctor for your arthritis."
3.‐ "Go ahead and take the aspirin if it helps, but watch closely for bleeding."
4.‐ "The doctor knows what is best for you, and you should follow those instructions."

1341 The nurse is caring for a female client during Correct answer: 4 These are all signs of perforation. If the client is going into shock, it is important to establish Select the option which includes the reporting of a significant change in the client’s
recuperation following development of a duodenal IV access before the veins collapse. The doctor will probably schedule emergency surgery. If a condition.
ulcer. The client suddenly experiences severe client has a possible perforation, she should be in low Fowler's position (option 1) to contain
abdominal pain, increased heart rate, increased the secretions in the abdomen. Walking (option 2) is not recommended, and food allergies
respiratory rate, and diaphoresis. On palpation, the (option 3) are not as likely to be the problem.
abdomen is rigid; bowel sounds are faint and
diminished. Which of the following nursing actions is
appropriate?
1.‐ Immediately place her in high Fowler's position to facilitate breathing.
2.‐ Help her walk to the bathroom to get rid of any flatus.
3.‐ Check to see if she has food allergies and see if she ate anything to which she might be allergic.
4.‐ Establish IV access and call the doctor to report the assessment data.

1342 A female client complains of a burning, cramping pain Correct answer: 1 This description of pain is consistent with ulcer pain. The pain is epigastric and is worse when Associate pain relieved by food with ulcers.
in the top part of the abdomen that becomes worse in the stomach is empty and is relieved by food. These symptoms are not common with
the middle of the afternoon and sometimes awakens cholelithiasis. Ordinary indigestion does not present with this clinical scenario.
her at night. She reports that eating something usually
helps the pain go away but that the pain is now
becoming more intense. Which of the following is the
best conclusion for the nurse to draw?

1.‐ These symptoms are consistent with an ulcer.


2.‐ The client is probably developing cholelithiasis.
3.‐ The client probably has indigestion and needs to watch what she eats.
4.‐ A snack before bed should be recommended.
1343 Which of the following clients would be most at risk Correct answer: 2 One form of intestinal obstruction is paralysis, caused by decreased movement of the Recall that immobility is associated with impaired bowel motility.
for an intestinal obstruction? intestinal contents by normal peristalsis. The client in option 1 is at high risk for Crohn's
disease and ulcerative colitis. Option 3 enhances the risk of cancer of the colon and diverticular
disease; option 4 is consistent with peritonitis.
1.‐ A Jewish client who smokes and consumes large amounts of caffeine
2.‐ An elderly client who is on bed rest because of postoperative abdominal surgery
3.‐ An individual eating a low‐fiber, high‐fat diet
4.‐ An adult diagnosed with cirrhosis of the liver

1344 A client is complaining of dyspepsia, frequent Correct answer: 4 Gastroesophageal reflux disease causes epigastric pain that is usually described as burning; it Frequent belching is the hint to identifying option 4 as correct.
belching, and increased salivation. The nurse suspects is accompanied by belching with a sour taste, pain after eating, increased salivation, and
which of the following? flatulence. The symptoms of a sliding hiatal hernia are similar to GERD, but not those of a
rolling hernia. Symptoms of PUD are more pronounced and reflective of a full or empty
stomach. Ulcerative colitis symptoms are related to pain and bowel movements.

1.‐ Peptic ulcer disease (PUD)


2.‐ Ulcerative colitis
3.‐ Rolling hiatal hernia
4.‐ Gastroesophageal reflux disease (GERD)

1345 Hepatic fat accumulation in a 55‐year‐old male is Correct answer: 4 A fatty liver is one of the main effects of alcohol consumption, known as Laennec's cirrhosis. Recall that Laennec’s is alcohol related cirrhosis.
usually a result of which type of cirrhosis? Other factors such as dietary intake of fat, body stores of fat, and hormonal status can also
contribute to fatty liver.
1.‐ Biliary
2.‐ Metabolic
3.‐ Postnecrotic
4.‐ Laennec's

1346 A concerned mother doesn't understand how her Correct answer: 2 Hepatitis A has an acute onset, and accounts for about 25 percent of hepatitis cases in the Omit options 3 and 4 as descriptive of hepatitis B.
child acquired hepatitis A, when he was perfectly United States. The usual incubation period is 15 to 40 days. The disease is spread where there
healthy up to a week ago. One characteristic of is fecal contamination of water supplies and from oral contamination (such as in day care).
hepatitis A that may help her to understand is that
hepatitis A has:
1.‐ An incubation period of 60 to 180 days.
2.‐ A fecal‐oral mode of transmission.
3.‐ A positive carrier state.
4.‐ A sexual mode of transmission.

1347 The physical assessment findings of spider angiomas, Correct answer: 3 Portal hypertension and liver cell failure contribute to the late manifestations of cirrhosis. Recall that change in mental status is associated with cirrhosis, but not the other options.
palmar erythema, peripheral edema, ascites, and Cholelithiasis and cholecystitis will be accompanied by pain, food intolerances and/or
change in mental status are consistent with which of vomiting. Pancreatitis presents with pain radiating to the back, mild cardiovascular changes,
the following disorders? and hypocalcemia.
1.‐ Cholelithiasis
2.‐ Cholecystitis
3.‐ Cirrhosis
4.‐ Pancreatitis

1348 A 45‐year‐old female hospitalized with acute Correct answer: 3 The onset of action for meperidine is 10 to 15 minutes and the onset for morphine is 20 to 60 Recognize that the sphincter of Oddi should be the hint, as this is the reason for morphine
pancreatitis has orders for meperidine (Demerol) 50 minutes. Both drugs are equal in the potential for addiction. Demerol is less sedating than being contraindicated.
mg IM every 4 hours as needed for pain. Demerol has morphine. The most important difference is that the meperidine causes fewer spasms of the
been ordered rather than morphine for this client sphincter of Oddi, which contributes to the goal of giving the pancreas a rest.
because it:
1.‐ Has a faster onset of action than morphine.
2.‐ Is less addictive than morphine.
3.‐ Causes fewer spasms of the sphincter of Oddi.
4.‐ Has fewer cognitive side effects.

1349 A 65‐year‐old female with a history of hepatic Correct answer: 4 The client is at increased risk for a return of the encephalopathy because of the diagnosis of Select the option that gives the client an accurate explanation and an option for other
encephalopathy is hospitalized for pneumonia and pneumonia and dehydration. She has volume depletion and the potential for electrolyte foods.
dehydration. When she complains to the nurse about imbalance, both of which can contribute to the development of encephalopathy. Dietary
the small portions of meat ordered by the dietitian, protein intake must be controlled (or eliminated) in order to minimize the ammonia levels in
the best response would be: the blood stream.
1.‐ "Ask your doctor about it in the morning."
2.‐ "I will call and order larger portions for you."
3.‐ "The amount of meat on your tray is dictated by certain blood test results."
4.‐ "Your protein is being limited, but you can have more food from another group."

1350 The physical assessment of a 55‐year‐old female with Correct answer: 3 In the cirrhotic liver, fibrous tissue develops among the parenchymal cells preventing the Recall that dullness on the dependent side indicated fluid or ascites.
end‐stage cirrhosis reveals a protuberant abdomen production of adequate plasma proteins. The consequence of low plasma proteins leads to a
with bulging flanks and dullness to the dependent side decrease in colloid osmotic pressure and generalized edema. When combined with high portal
while lying on the right. The appropriate terminology capillary pressures, large amounts of fluid and protein form in the abdominal cavity, which is
for documentation of this assessment is: called ascites. Gravity causes the fluid to sink and gas‐filled loops of the bowel rise creating the
shifting dullness and tympany during assessment.

1.‐ Fluid overload.


2.‐ Malnutrition.
3.‐ Ascites.
4.‐ Distention.

1351 Which of the following statements is true regarding Correct answer: 3 The cancer cells alter enzyme secretion and flow to the duodenum in addition to causing fat Select the option that reflects a change in nutrient absorption.
cancer of the pancreas? and protein malabsorption. These changes result in weight loss and nausea, which are
common signs and symptoms of cancer of the pancreas regardless of location.

1.‐ Clients with pancreatic cancer have often had a long history of painful sitting.
2.‐ Clients with pancreatic cancer will describe recent onset of black tarry stools.
3.‐ Clients with pancreatic cancer will describe a slow onset of anorexia and weight loss.
4.‐ Clients with pancreatic cancer will have no symptoms to report.

1352 When providing discharge teaching to the client with Correct answer: 1 When bile production is reduced, the body has reduced ability to absorb fat‐soluble vitamins. Recall that the liver plays an important role in clotting; the diseased liver cannot assist in
chronic cirrhosis, his wife asks the nurse to explain why Without adequate Vitamin K absorption, clotting factors II, VII, IX, and X are not produced in clotting.
there is so much emphasis on bleeding precautions. sufficient amounts.
Which of the following provides the most appropriate
response?
1.‐ "The liver affected by cirrhosis is unable to produce clotting factors."
2.‐ "The low protein diet will result in reduced clotting factors."
3.‐ "The increased production of bile decreases clotting factors."
4.‐ "The required medications reduce clotting factors."

1353 Which of the following statements is true regarding Correct answer: 3 The posticteric phase follows jaundice. Symptoms decrease and the serum enzymes begin to Recall that posticteric means after jaundice onset.
viral hepatitis infection? return to normal. Hepatitis B is transmitted by parenteral, sexual, or perinatal routes. Hepatitis
A is transmitted by the fecal‐oral route. Hepatitis D has a rapid onset.

1.‐ Hepatitis B is transmitted by the fecal‐oral route.


2.‐ Hepatitis A is a sexually transmitted disease.
3.‐ The posticteric phase follows jaundice and lasts several weeks.
4.‐ Hepatitis D has a slow onset.

1354 When explaining the rationale for the use of lactulose Correct answer: 2 Chronulac is a synthetic nonabsorbable disaccharide metabolized to organic acids by enteric Option 2 mentions the metabolism of ammonia, which is the purpose for the use of
(Chronulac) syrup to the client with chronic cirrhosis, bacteria and causes osmotic catharsis while reducing the growth of ammonia‐forming Chronulac.
the nurse would use which of the following bacteria. Chronulac also lowers the pH of the colon, which converts ammonia to a
statements? nonabsorbable form allowing expulsion through the laxative action of the drug. The dose
required is 15 to 30 mL orally every 4 to 6 hours and may be titrated to a lower dose if
diarrhea occurs.
1.‐ "Chronulac syrup reduces constipation, which is a frequent complaint with cirrhosis."
2.‐ "Chronulac syrup suppresses the metabolism of ammonia and aids in its elimination through feces."
3.‐ "Chronulac syrup helps to reverse cirrhosis of the liver."
4.‐ "Chronulac syrup can be taken intermittently to reduce side effects."

1355 A client presents to the clinic with "bad pain" in the Correct answer: 3 Lying on the side with legs flexed, pain over McBurney's point, and rebound tenderness are Recall that pain described as mid‐abdominal with rebound is often related to appendicitis.
middle of the abdomen, vomiting, and "not knowing characteristic symptoms of appendicitis. Vomiting frequently accompanies the pain. The client
what is wrong." Palpation reveals rebound tenderness definitely should not have an enema if appendicitis is suspected. If surgery is needed for
with increased pain halfway between the umbilicus appendicitis, the client needs to be NPO.
and the top of the pelvis. The client seems to have less
pain when lying on the left side and flexing the knees.
What is the best conclusion for the nurse to draw?

1.‐ To make sure the client does not have an impaction, an enema should be given.
2.‐ Since the client is vomiting, the problem is probably gastroenteritis.
3.‐ The client should be checked for possible appendicitis.
4.‐ Since the client has been vomiting, nourishment may help.

1356 A client with Zollinger‐Ellison syndrome thinks she has Correct answer: 2 Zollinger‐Ellison syndrome is a condition usually caused by a gastrin‐secreting tumor of the Omit options 3 and 4 as they are describing other conditions. The key is the mention of
a rare form of cancer. The nurse explains that this pancreas, stomach, or intestines that leads to the increased secretion of pepsin and hydrochloric acid.
syndrome is characterized by which of the following? hydrochloric acid. This often leads to peptic ulcer disease. Option 1 explains one of the
pathologic reasons for peptic ulcer disease; Option 3 explains a volvulus obstruction, and
another name for Crohn's disease is regional enteritis.
1.‐ A destruction of the mucus‐protecting cells of the stomach that could lead to an ulcer
2.‐ A condition that causes increased secretion of pepsin and hydrochloric acid
3.‐ A twisting of the bowel that leads to intestinal obstruction
4.‐ Crohn's disease, which is an inflammatory bowel disorder

1357 A client who is exhibiting borborygmi, cramping pain, Correct answer: 1 Dumping syndrome is the rapid influx of stomach contents into the duodenum or jejunum The question mentions that there was surgical treatment of PUD, which should omit
vomiting, and diarrhea has a diagnosis of peptic ulcer causing increased peristalsis and dilation of the intestines. Although this occurs primarily after options 2, 3, and 4.
disease (PUD) with recent surgical treatment. The a gastrectomy, the condition can cause an ulcer.
client is probably experiencing which of the following?

1.‐ Dumping syndrome


2.‐ Complications of the PUD
3.‐ Perforation of the stomach
4.‐ Peritonitis

1358 A client diagnosed with peptic ulcer disease wants to Correct answer: 3 The bacteria H. pylori has been discovered to be the leading cause of many ulcers and can be Option 3 mentions ‘H. pylori,’ the bacteria associated with ulcers.
know why he is being given antibiotics. The nurse's treated with success by antibiotics. Options 1, 2, and 4 are unrealistic answers for the action of
best answer would be: antibiotics.
1.‐ "Antibiotics help calm the stomach and decrease the symptoms."
2.‐ "Antibiotics decrease dumping syndrome, which can lead to PUD."
3.‐ "<i>H. pylori</i> is a bacterial cause for PUD and antibiotics will treat the cause."
4.‐ "The excess acid can be decreased when the stomach is sterile."

1359 Which of the following foods should be avoided in a Correct answer: 4 Caffeine stimulates the acid secretion and can interfere with the function of the lower Look for the caffeine containing options as it is thought to be a source of increased acidity.
client with peptic ulcer disease? esophageal sphincter. Chocolate contains caffeine and should be limited along with other
drinks and foods with caffeine. Spicy or hot foods, smoking, and alcohol should also be
avoided.
1.‐ Vegetables
2.‐ Meats
3.‐ Fruits
4.‐ Chocolate

1360 The nurse explains to a family that the main Correct answer: 3 The bowel wall becomes congested, thickens, and sometimes develops fistulas, which can The correct option has the word ‘malabsorption’ in it. This is the cause of weight loss.
physiologic reason for weight loss in a client with become infected. This leads to malabsorption and deficiency in absorption of folic acid,
Crohn's disease is which of the following? calcium, and Vitamin D. The anorexia can play a role in weight loss, but most clients eat and
cannot explain why they have weight loss.
1.‐ The symptoms of anorexia prevent the client from eating.
2.‐ The inflammation of the disease decreases the appetite.
3.‐ The thickening and congestion of the bowel wall results in malabsorption.
4.‐ The "skip lesions" interfere with food passage through the bowel.

1361 The nurse explains to a certified nursing assistant Correct answer: 1 Elderly clients often eat less food with less roughage and fiber and therefore do not obtain Omit options 2 and 3 as constipation is more commonly associated.
(CNA) who is also a nursing student that the reason the proper nutrients from their diet, which can aid in the development of the disease. Most
elderly clients are more prone to diverticulitis is often chronic constipation, not diarrhea, is a cause.
because of:
1.‐ Poor, deficient diet.
2.‐ Chronic diarrhea.
3.‐ Frequent laxative use.
4.‐ Sedentary lifestyle.

1362 A client who has Crohn's disease and is noncompliant Correct answer: 2 Clients with Crohn's disease are at risk of developing cancer of the GI tract. A noncompliant Eliminate options 3 and 4 as they are not associated with Crohn’s.
should be cautioned about the risk for which of the client increases that risk and should be educated that Crohn's can be successfully kept under
following? control.
1.‐ Perforation of the bowel
2.‐ Colorectal cancer
3.‐ PUD
4.‐ Ulcerative colitis

1363 Prior to giving an analgesic for pain to a postoperative Correct answer: 3 A client with any GI disorder, especially a peptic ulcer, should never receive any aspirin Recall that aspirin is contraindicated with ulcers.
client who has a history of peptic ulcer disease (PUD), product. Many pain medications contain aspirin and are combinations of an opioid analgesic
the nurse should check to see that the agent does not and a non‐opioid analgesic such as aspirin. The nurse administering the pain medication should
contain which of the following? know what ingredients are in it. Hydrocodone is a schedule III opioid analgesic in agents such
as Lortab.
1.‐ An opioid product
2.‐ Acetaminophen (Tylenol)
3.‐ Acetylsalicylic acid (aspirin)
4.‐ Hydrocodone
1364 A client is admitted to the unit with a large, distended Correct answer: 2 Inserting an NG tube will decompress the bowel, which will relieve the vomiting and pain and The item mentions feculent emesis. The NG would prevent aspiration and help to protect
bowel, acute tenderness upon palpation of the hopefully prevent the client from going into shock. This may be a measure to institute only the airway and lungs.
abdomen, fever, rigidity, and absent bowel sounds. until surgery can be performed. All of the interventions are appropriate, but vomiting fecal
After being on the unit, the client's level of matter can be dangerous (as well as unpleasant) because of the possibility of aspiration,
consciousness decreases, and he begins to have especially with a decreasing level of consciousness.
feculent vomit. The priority therapeutic intervention
would be to:
1.‐ Reduce the fever through antipyretics.
2.‐ Insert an NG tube to wall suction and monitor the output.
3.‐ Administer pain medications to relax the client.
4.‐ Listen to bowel sounds.

1365 Which of the following clients is more likely to Correct answer: 4 Causes of pancreatitis include alcohol abuse of excessive intake of liquor or wine for 6 years Obstruction of the duct allows the enzymes to act on the tissue of the pancreas causing
develop pancreatitis? or more; high triglyceride levels, and hypercalcemia. Stones lodged in the pancreatic duct can inflammation.
cause obstruction and lead to inflammation of the pancreas. Options 1, 2, 3 are worded
incorrectly.
1.‐ A 59‐year‐old male with a history of occasional alcohol use
2.‐ A client with renal problems and hypocalcemia
3.‐ A client recovering from a myocardial infarction with hypercholesterolemia
4.‐ A client with a stone lodged in the pancreatic duct

1366 Which of the following actions of pancreatic enzymes Correct answer: 2 When the pancreas is injured and/or has an impaired or disrupted function, the pancreatic The digestive action on the organ causes pain and inflammation.
can cause pancreatic damage? enzymes (phospholipaseA, lipase, and elastase) leak into the pancreatic tissue and initiate
autodigestion. Options 3 &amp; 4 can be causes of pancreatitis. Option 1 is incorrect to this
situation.
1.‐ Utilization by the intestine
2.‐ Autodigestion of the pancreas
3.‐ Reflux into the pancreas
4.‐ Clogging of the pancreatic duct

1367 Which of the following laboratory tests indicates a Correct answer: 1 In pancreatitis, the lipase, amylase, glucose and white blood count (WBC) are all elevated. The Recall that lipase is a pancreatic enzyme.
diagnosis of pancreatitis? calcium is low for 7 to 10 days and is a sign of severe pancreatitis. This question draws on your
knowledge of laboratory results and what is normal and abnormal. If this question was
problematic, review your normal lab values for each of these tests and as you study.

1.‐ Lipase 230 IU/L


2.‐ Calcium 6.0 mEq/L
3.‐ Blood glucose 65 mg/dL
4.‐ White blood cell count 5,000/mm3

1368 The client with pancreatitis may exhibit Cullen's sign Correct answer: 4 Bleeding is a complication of pancreatitis and is usually identified through a positive Turner's Omit option 2 as it is not descriptive of pancreatic pain.
on physical examination. Which of the following data sign (flank bruising) or Cullen's sign (umbilical bruising). Options 1 and 3 are correct in
best describes Cullen's sign? pancreatitis, but do not answer the question. Read the stem carefully when taking the test.
Option 2 is incorrect; remember the pain may be relieved by flexing the left leg or by walking.

1.‐ Jaundiced sclera


2.‐ Pain that occurs with movement
3.‐ Bluish discoloration of the left flank area
4.‐ Bluish discoloration of the periumbilical area
1369 Which of the following findings would strongly Correct answer: 2 Although option 4 is correct, it is not a strong indicator of cirrhosis. Pruritus can occur for Recall that hepatomegaly means enlarged liver, which is common in early cirrhosis.
indicate the possibility of cirrhosis? many reasons. Options 1 and 3 are incorrect, fluid accumulation is usually in the form of
ascites in the abdomen. Hepatomegaly is an enlarged liver, which is correct. The spleen may
also be enlarged.
1.‐ Dry skin
2.‐ Hepatomegaly
3.‐ Peripheral edema
4.‐ Pruritus

1370 A client with cirrhosis may have alterations in which Correct answer: 3 Clients with cirrhosis have used their clotting factors, and the liver is unable to provide Recall that the liver plays a role in clotting.
of the following laboratory values? enough clotting factors. A prothrombin time is an indication of the time needed for blood to
clot. If clotting factors aren't present, bleeding is more likely.
1.‐ Carbon dioxide level
2.‐ pH
3.‐ Prothrombin time (PT)
4.‐ White blood cell count (WBC)

1371 Which of the following clients is most likely to acquire Correct answer: 4 Although hepatitis is associated with cholestasis (option 2), the most likely candidate would Select the option that describes a viral infection.
hepatitis? be someone with a viral infection. A classic example is someone with varicella zoster. Options
1 and 3 are not related to hepatitis. Other causes include alcohol, toxins, and severe
hepatocellular damage.
1.‐ A child with a bacterial infection
2.‐ A client with dysfunction of the biliary system
3.‐ A client with metastasis of liver cancer
4.‐ An adult with varicella zoster

1372 In reviewing the possible causes of hepatitis A in a 22‐ Correct answer: 3 Hepatitis A is transmitted by fecal‐oral route. The virus is excreted in oropharyngeal Hepatitis A is associated with eating chilled contaminated foods.
year‐old male, which of the following would be the secretions (nose and throat) and transmitted by direct contact of person‐to‐person, or by fecal
most likely factor? contamination of food or water. A worker at the pub could have hepatitis A and transfer it to
the food that is being prepared. Options 1, 2, and 4 are classic of hepatitis B, C, and D.

1.‐ Contact with blood in his profession as a policeman


2.‐ Receiving a blood transfusion during surgery
3.‐ Eating shrimp at the local pub
4.‐ Admitting to being sexually active

1373 A client is admitted to the unit with complaints of Correct answer: 2 The symptoms in preicteric hepatitis are vague and more flu‐like as described above. The Realize that the question states that the client has the hepatitis B surface antigen.
malaise, nausea, vomiting, anorexia, and headaches. physician usually needs laboratory work to verify a diagnosis. In this case, the presence of the
The lab work shows abnormal electrolytes and antigen HBsAG concludes that the client has an active form of the disease since hepatitis B
elevated aspartate aminotransferase (AST), alanine surface antigen is present.
aminotransferase (ALT), and alkaline phosphatase
(ALP). Hepatitis B surface antigen (HbsAG) is also
present. The nurse would assume the client has:

1.‐ Hepatitis A.
2.‐ Hepatitis B.
3.‐ Cirrhosis.
4.‐ Pancreatitis.
1374 A client is admitted with possible liver cancer. Which Correct answer: 3 Although an abdominal ultrasound, x‐ray, and CT scan are useful in the diagnosis of cancer of Recognize that option 3 is the only option that is not a radiology study.
of the tests below would be the most confirming of the liver, the alpha‐fetoprotein serum markers are specific to detecting primary hepatocellular
this diagnosis? carcinoma.
1.‐ Abdominal ultrasound (US)
2.‐ Abdominal flat plate X‐ray
3.‐ Alpha‐fetoprotein markers
4.‐ Computed tomography (CT) scan

1375 A client says to the nurse "I have this pain from my Correct answer: 3 Eating at bedtime may cause increased secretion of pepsin and gastric acid, which will cause Recognize that option 3 gives the client a valid, concrete suggestion.
ulcer. When it comes on at night, I have difficulty pain later when the stomach is empty. Sleeping pills should be the last resort, and stress can
sleeping." Which is the best response for the nurse to aggravate the circumstance, but it would not be just at night.
make?
1.‐ "There really is nothing that can be done‐‐that is typical of ulcer pain."
2.‐ "Are you worrying a lot?"
3.‐ "Try limiting the food you eat at bedtime."
4.‐ "The best thing to do is to take a sleeping pill."

1376 A child with a confirmed diagnosis of appendicitis has Correct answer: 2 When the appendix ruptures, there is a decrease in pain because the appendix is no longer Recall that temporary relief of pain is common with rupture.
been scheduled for an emergency appendectomy. distended. The problem is worse, not better. There is no indication in the question that the
Suddenly, the child states his pain is much less. The child has had a change in level of consciousness.
best interpretation of this is that:

1.‐ He is tolerating the pain much better.


2.‐ There is a possibility the appendix has ruptured.
3.‐ His level of consciousness has decreased.
4.‐ Perhaps the problem has been resolved.

1377 Which of the following describes the best practice in Correct answer: 2 A high‐fiber diet increases stool bulk and decreases intraluminal pressure, decreasing Recall that fiber is essential with diverticulosis to prevent infection.
an attempt to control diverticulosis? development of diverticula. The other responses will not necessarily be effective or they are
incorrect.
1.‐ Daily intake of Vitamins A, C, and E
2.‐ High intake of dietary fiber
3.‐ High intake of carbohydrates
4.‐ Increasing intake of fluids

1378 The nurse would include which of the following in a Correct answer: 2 Clients with rectal and intestinal polyps have a higher incidence of colon and rectal cancer. Recall that polyps put the client at increased risk.
teaching plan for the prevention of colon cancer? Early diagnosis facilitates more effective treatment. Caloric and fat content should not be
increased.
1.‐ Increasing the fat content of the diet
2.‐ Colonoscopy if the client has a history of rectal polyps
3.‐ Avoidance of strenuous exercise and lifting
4.‐ Increased caloric content of the diet through carbohydrates

1379 A client with Crohn's disease is having continuous Correct answer: 4 The inflammatory process is the pathology in the development of Crohn's disease. Recall that Crohn’s disease is an inflammatory bowel disorder.
watery diarrhea. The nurse concludes that this Inflammation results in ulcers and fissures or fistulas as well as fibrosis. The other options are
symptom is caused by: not related to Crohn's disease.
1.‐ Multiple tumor growths within the wall of the small intestine.
2.‐ Allergic manifestations as a result of lactose intolerance.
3.‐ Excessive intake of fat.
4.‐ Inflammatory process in the bowel with ulcer and fissure formation.
1380 A client is admitted with complaints of being tired, is Correct answer: 2 Presence of a mass demands immediate attention. The other symptoms are characteristic of Recognize that a mass requires further investigation.
anorexic, and has lost weight. He denies pain at the cancer of the liver. These symptoms could represent stomach cancer also.
present time but states he "knows something is
wrong." Assessment reveals a mass in the right upper
quadrant that is tender on palpation. What conclusion
should the nurse draw from these findings?

1.‐ He is probably tired due to malnutrition and anemia.


2.‐ Referral needs to be made regarding the presence of the mass.
3.‐ A mass frequently is found in patients with hepatitis.
4.‐ He is probably dehydrated with electrolyte imbalances.

1381 In planning a health screening for cancer awareness, Correct answer: 1 Annual guaiac testing is recommended by the American Cancer Society as a means of Recall that occult blood testing is a common screening method.
the nurse would want to include which of the following detecting colon cancer. The other options are not.
to prevent rectal cancer?
1.‐ Annual guaiac testing for occult fecal blood for people over age 50
2.‐ Upper GI x‐rays annually
3.‐ Routine testing of blood cholesterol
4.‐ Stress test

1382 A client with a history of peptic ulcer disease is taking Correct answer: 1 Ranitidine (Zantac) is an H2‐receptor antagonist; the action is to block the secretion of HCl by The word ‘blocks’ in relation to hydrochloric acid is the key to the correct response.
ranitidine (Zantac). He questions the action of this depressing the histamine receptors. It does not coat the lining. The object is not to increase
drug. The best answer for the nurse to give is: gastric acid or make histamine receptors more sensitive.

1.‐ "It blocks the secretion of hydrochloric (HCl) acid in the stomach."
2.‐ "It coats the lining of the stomach."
3.‐ "The release of gastric acid is increased."
4.‐ "The histamine receptors become more sensitive and act to protect the stomach."

1383 As part of preparation for discharge for a client who Correct answer: 3 These symptoms are characteristic of dumping syndrome and occur because of hypertonic Recall that ulcer surgery may result in dumping syndrome.
has undergone a Billroth II surgical procedure for food entering the jejunum with no sphincter. The other options result in a different set of
peptic ulcer disease, the nurse teaches the client about symptoms.
the possibility of dizziness, paleness, sweating, and
feeling the heartbeat. The nurse explains that these
symptoms are an indication of:

1.‐ Recurrence of the ulcer.


2.‐ Anemia caused by blood loss during surgery.
3.‐ Dumping syndrome.
4.‐ Perforation.

1384 An elderly client has had several gastrointestinal Correct answer: 4 These studies require the client to take strong laxatives and enemas and to be NPO as This option relates to the preparation that is common for all of these examinations.
diagnostic procedures, including endoscopy and upper preparation for the procedure. The client may become dehydrated.
GI series. Following these procedures, the nurse should
assess the client for:

1.‐ Level of consciousness.


2.‐ Level of pain.
3.‐ Hypotension.
4.‐ Hydration and nutrition level.
1385 An elderly client expresses concern about the Correct answer: 1 Increasing fiber provides bulk and keeps the stools soft and easy to expel. Fluid intake affects The correct option identifies good health practices as the solution to constipation.
possibility of constipation, stating "I have not had a the consistency of the stool. An obstruction takes more than one day to form.
bowel movement today at all, and I don't want to get
an obstruction." Which response by the nurse is most
helpful to the client?
1.‐ "It is better to control your bowel habits with increased fiber and increased fluid intake."
2.‐ "Oh, don't worry about that!"
3.‐ "You probably need to take a laxative."
4.‐ "That's a part of the aging process; it is to be expected."

1386 A client is admitted with a history of chronic Crohn's Correct answer: 4 Colon cancer is more likely to develop in clients who have had inflammatory bowel disease. It Omit option 1 as this is a chronic disease. Omit option 2 as there are helpful treatments.
disease. The nurse plans care based on which of the is not infectious and there is no effective cure. Symptomatic control is possible. Omit option 3 as this is not a communicable disease.
following ideas?
1.‐ If the client adheres to the therapeutic regimen, there is a high chance of cure.
2.‐ There is nothing that can be done to alleviate the symptoms of this disease.
3.‐ This is an infectious process; therefore, communicable disease precautions must be taken.
4.‐ There is an increased risk of developing colon cancer with this disease.

1387 A client visiting an outpatient clinic mentions that she Correct answer: 4 Caffeine increases acidity in the stomach and can affect the lower esophageal sphincter, Select the option that does contribute to increased acid production.
has been having severe indigestion and burning in her which could account for the symptoms that are consistent with gastritis.
chest. The nurse takes a history and learns that she
does not smoke or consume alcohol. Another dietary
consideration to ask about initially would be:

1.‐ Vitamin intake.


2.‐ Herbal products.
3.‐ Amount of meat eaten per day.
4.‐ Amount of caffeine.

1388 A female client hospitalized for a broken pelvis from a Correct answer: 3 Stresses to the body such as burns, trauma, and surgery can cause stress ulcers even in Recall that stress can lead to increased gastric acidity.
motor vehicle accident is being sent home on an H2‐ individuals who did not have an ulcer before the event. Option 2 is not supported by the
antagonist and an antacid as part of her home scenario and options 1 and 4 are incorrect.
medications. When the client questions why, the nurse
explains that:
1.‐ The medications are a preventive measure only.
2.‐ She has GERD because of the other medications she is taking.
3.‐ The stress of the accident and injury caused a stress ulcer.
4.‐ She probably always had an ulcer and didn't know it.

1389 A 7‐year‐old boy is admitted to a children's hospital Correct answer: 2 A pH probe test is done to determine both the number of drops in pH of the stomach and the Pain in the center of the chest and low pH are the key to selecting GERD as the correct
because of frequent complaints of burning pain in the length of each of the drops. Even though the level did not remain low, 100 drops in 24 hours is response.
middle of his chest and frequent regurgitation. A pH excessive. Dyspepsia is a symptom (option 2). The scenario does not support options 1 and 4.
probe test is done and reveals over 100 drops in the
pH to between 2.0 and 3.5 within a 24‐hour period;
however, none of the decreases remained low for
longer than 2 to 3 seconds. The child most likely has:

1.‐ Curling's ulcer.


2.‐ GERD.
3.‐ Dyspepsia.
4.‐ PUD.
1390 The mother of a 15‐year‐old girl notices that she has Correct answer: 1 Crohn's disease has a characteristic skip lesion on visualization of the intestinal wall. Option 2 Omit options 3 and 4 as not associated with inflammatory diseases. Crypts, as in option 2
lost weight, complains of stomach pains, doesn't want occurs with ulcerative colitis; options 3 and 4 are not suggestive of Crohn's disease. would suggest ulcerative colitis.
to eat, and begins having frequent diarrhea. She is
tested for Crohn's disease. The nurse expects that
sigmoidoscopy will reveal which of the following?

1.‐ Skip lesions


2.‐ Lieberkuhn's crypts
3.‐ Ulcerated cavities
4.‐ Polyps

1391 The reason diverticular disease is more common in Correct answer: 4 Elderly people have a lack of adequate blood supply because of the aging process and often Reduction of blood supply and diet changes would be associated with aging.
the elderly is because of: consume less of the nutrients necessary in the diet to maintain adequate peristalsis and
normal bowel function. Options 1 and 2 can be general causes; option 3 does not apply.

1.‐ Obesity and high dietary fat content.


2.‐ More frequent problems with constipation.
3.‐ Difficulty evacuating all of the bowels because of poor sphincter control.
4.‐ Blood supply and diet.

1392 Of the following clients, the one who is at high risk for Correct answer: 3 Intestinal obstructions are mechanical (options 1, 2, 4) or paralytic, in which neurogenic or Make the association between paralytic ileus and surgery to identify option 4 as correct.
a functional paralytic intestinal obstruction is the one: muscular impairment hinders peristalsis. Early ambulation after surgery usually helps to
prevent this.
1.‐ With a tumor of the colon.
2.‐ Who develops adhesions after a bowel resection.
3.‐ Who develops a paralytic ileus after abdominal surgery.
4.‐ With a twisted colon.

1393 An example of appropriate teaching at a health fair in Correct answer: 1 Middle school and high school age students are high users of tobacco by smoking, dipping, Select the response that reflects a risk factor in this age group.
a middle or high school would be to emphasize which and chewing. Teenagers do not feel anything can harm them; therefore, they think they will
of the following? probably never have cancer. Options 2 and 3 are less likely to apply to their age and teenagers
are constantly altering their weight and appetite. Option 4 is false; lesions should be reported.

1.‐ Tobacco in any form increases risk of oral cancer.


2.‐ A stool specimen can be used to determine colorectal cancer.
3.‐ Any change in appetite or sudden weight loss could signal stomach cancer.
4.‐ White or red patches on the mouth do not need to be reported immediately.

1394 An elderly man presents to the outpatient clinic Correct answer: 1 Although the client's symptoms could suggest option 4, they are highly suggestive of Changes in the girth of stool are suggestive of colorectal cancer.
complaining of changes in bowel habits, increased colorectal cancer. Options 2 and 3 are incorrect.
diarrhea, thin stools, and some rectal bleeding. The
nurse would be most concerned about:

1.‐ Left‐sided colorectal cancer.


2.‐ Stomach cancer.
3.‐ Intestinal obstruction.
4.‐ Diverticulitis.
1395 The nurse caring for a client with acute pancreatitis Correct answer: 4 Biliary obstruction caused by a stone in the pancreatic duct is one cause of pancreatitis. This Associate jaundice with obstruction to the flow of bile.
observes that the client’s skin has become slightly type of Pancreatitis may result in mild jaundice depending on the degree of obstruction. It is
jaundiced. Which of the following is the best not an indication of a terminal stage. The decision to give oral feedings is based on degree of
conclusion for the nurse to make regarding the pain and presence of bowel sounds or laboratory data, which are an indication of
prognosis of this client? improvement of the client and moving from an acute to chronic stage.

1.‐ The client has been misdiagnosed. He probably has cirrhosis or hepatitis.
2.‐ This client is approaching the terminal stage of pancreatitis.
3.‐ This is an indication the pancreas is overworked and the client should not be allowed to eat.
4.‐ The client's pancreatitis has resulted from biliary obstruction.

1396 A client says to the nurse, "I had a hepatitis B Correct answer: 3 Hepatitis B vaccine provides protection against hepatitis B or possibly hepatitis D only. The immunization is for hepatitis B, not other strains of hepatitis.
immunization. Now I know I won't have to worry about Hepatitis A may also be contracted from contaminated food, water or direct contact.
contacting any hepatitis any more." Which is the best
interpretation for the nurse to make?
1.‐ The client is correct in his interpretation.
2.‐ The immunization does not provide post‐exposure protection against any form of hepatitis.
3.‐ The client does not understand that he can still contract hepatitis A.
4.‐ Hepatitis B is not contracted from contaminated needles.

1397 Diffuse fibrosis and conversion of normal liver tissue Correct answer: 2 The process of cirrhosis involves fibrotic changes in the liver in which fibrous bands form Recall that fibrous liver tissue is descriptive of late stage cirrhosis.
into abnormal nodules with fibrous bands describes nodules, which gives the liver a cobblestone appearance. Liver failure is a secondary condition.
the progression of which of the following hepatic The process in hepatitis is more necrosis, hyperplasia, and inflammation. Liver cancer is the
disorders? development of tumor cells.
1.‐ Liver failure
2.‐ Cirrhosis
3.‐ Hepatitis A
4.‐ Liver cancer

1398 Lab results for a client who has portal hypertension Correct answer: 1 In portal hypertension, decreased protein synthesis results in a decrease in albumin, which Albumin holds fluid in the vascular system. Low albumin level results in fluid in the
show a low level of serum albumin. Based on this causes edema and ascites. Hypoglycemia occurs as a result of increased insulin production. abdominal cavity, or ascites.
information, the nurse knows that he has a high Esophageal bleeding can occur in portal hypertension, which can result in hypovolemic shock,
chance of developing which of the following? but they are not a direct result of decreased serum albumin.

1.‐ Ascites
2.‐ Hypoglycemia
3.‐ Esophageal bleeding
4.‐ Hypovolemic shock

1399 At a health fair for adults in the community, the nurse Correct answer: 1 The incidence of cancer of the pancreas is twice as high in smokers as nonsmokers. Other risk Remember that smoking cessation is a good health promotion measure.
is asked by a client on means to prevent cancer of the factors are high‐fat diet and pancreatitis. Correlation has not been shown to obesity, fiber, or
pancreas. The nurse would suggest which of the alcohol.
following?
1.‐ Cessation of smoking
2.‐ Consuming a low‐fiber diet
3.‐ Reduction of weight
4.‐ Cessation of alcohol

1400 A client has a history of gallbladder disease and is Correct answer: 1 Bile reflux causes activation of pancreatic enzymes, which cause autodigestion of the Recall that biliary obstruction is often the cause of pancreatitis.
admitted for a diagnostic work‐up to rule out pancreatic tissue. There is a definite relationship between these two conditions. Pancreatitis is
pancreatitis. The nurse knows the relationship not always directly caused by alcohol intake.
between these two diseases is:
1.‐ When a stone obstructs the common bile duct, a reflux of bile may cause breakdown of pancreatic tissue.
2.‐ The symptoms are very similar, and it is necessary to differentiate between the two diseases.
3.‐ There is no relationship between gallbladder disease and pancreatitis.
4.‐ Pancreatitis is directly related to alcohol intake, and this is intensified by gallbladder disease.

1401 A client is admitted with recurrent abdominal pain, Correct answer: 4 Replacement of pancreatic tissue by fibrous changes causes exocrine and endocrine changes The inclusion of diabetes mellitus in option 4 is the key to the correct response.
steatorrhea, weight loss, and elevated serum amylase with loss of pancreatic enzymes for digestion and loss of function of islets of Langerhans,
and lipase with a history of chronic alcohol abuse. A which secrete insulin, resulting in diabetes mellitus. As a chronic condition, there is very
possible diagnosis is chronic pancreatitis. The nurse limited recovery. There is a direct correlation between alcohol abuse and chronic pancreatitis.
knows that chronic pancreatitis:

1.‐ Has very limited correlation to alcohol intake.


2.‐ Is easier to treat than acute pancreatitis with quick recovery and limited repercussions.
3.‐ Is an infectious process that occurs following appendicitis or other infectious conditions.
4.‐ Is a progressive disease, in which pancreas tissue is replaced by fibrous connective tissue and malabsorption occurs along with diabetes mellitus.

1402 Decreased excretion of bilirubin would probably be Correct answer: 3 Obstruction of the common bile duct interferes with movement of the bile so that it cannot Associate jaundice with biliary obstruction. Jaundice is due to increased bilirubin.
attributed to which of the following? be excreted.
1.‐ A blood transfusion reaction
2.‐ Hemolytic anemia
3.‐ An obstructed common bile duct
4.‐ An obstructed pancreatic duct

1403 In caring for a client with acute hepatitis B, the client Correct answer: 4 The client's ability to determine activities that would not cause excess fatigue allow for self‐ Select the open‐ended response that involves the client.
says, "I am so tired of this bed. I don't think I can stay direction and participation. Bedrest is encouraged and activities are progressed slowly;
here any longer!" The best response of the nurse is: however, strict bedrest is not common. Planned rest periods are highly recommended.

1.‐ "You must stay on strict bedrest, with slow progression to normal activities, for the liver to heal"
2.‐ "I know it gets boring staying in bed so long, but you must."
3.‐ "Don't you want to get well?"
4.‐ "What activity do you think you can do without getting too tired?"

1404 In caring for a client with liver failure, the nurse Correct answer: 4 Increased ammonia occurs because the liver is unable to convert ammonia to urea. Calcium is Remember that the liver is responsible for forming urea from ammonia.
knows that the pathophysiology of the disease causes probably decreased because of bedrest. Serum creatinine is probably increased because of
which of the following? impaired renal function; RBCs are probably decreased because of decreased production in the
bone marrow.
1.‐ Decreased serum creatinine
2.‐ Increased red blood cells (RBCs)
3.‐ Increased calcium
4.‐ Increased ammonia

1405 A newly admitted client with cirrhosis of the liver has Correct answer: 3 Low albumin in the blood causes a decrease in plasma colloidal osmotic pressure, causing Recall that albumin holds fluid in the vascular system.
a distended abdomen and the umbilicus is protruding. fluid to escape into the extravascular compartment.
The nurse knows the pathological basis for this is:

1.‐ Increased fluid intake resulting from excessive use of alcohol causing overhydration.
2.‐ Increased size of the liver results in abdominal distension.
3.‐ Hypoalbuminemia causes fluid to leave the vascular system and enter the peritoneal cavity.
4.‐ Shunting of the blood to the collateral circulation in the esophagus results in decreased blood volume and accumulation of fluid.
1406 A client with hepatic encephalopathy would expect to Correct answer: 4 In hepatic encephalopathy, the level of ammonia is increased with high levels of protein in the Recall that ammonia is nitrogenous. Nitrogen comes from protein.
have which of the following changes in the diet? intestine. Calories are needed to promote healing. Potassium levels are usually increased
because of impaired kidney function, therefore the intake should not be increased. Sodium is
restricted because of ascites.
1.‐ Increase sodium intake to draw fluid from the abdomen
2.‐ Restriction of calories to decrease energy used as a result of digestion
3.‐ Increase intake of potassium in order to promote cardiac activity
4.‐ Restriction of protein in order to decrease levels of ammonia

1407 A client with a diagnosis of cirrhosis of the liver and a Correct answer: 3 Portal hypertension develops as a result of development of fibrous bands, which develop Omit option 4 as this is the result of portal hypertension. Fibrosis around the portal vessels
history of alcohol abuse is admitted to the unit. The following necrosis and regeneration of lung tissue. The other conditions are due to other cause increased pressure.
nurse knows that portal hypertension is a possible changes in the liver as well as bleeding. Esophageal bleeding occurs as a result of portal
complication of this condition for which of the hypertension, not just the opposite.
following reasons?
1.‐ Hyponatremia and hypoproteinemia
2.‐ Ecchymosis, edema, and jaundice
3.‐ Fibrotic tissue from cell destruction
4.‐ Development of esophageal varices

1408 A male client comes to the ambulatory care center Correct answer: 4 Esophageal varices occur as a complication of portal hypertension and cirrhosis. It results in Recognize that the scenario states that the client has dried blood on his lips. This leads to
complaining of weakness and vomiting. He states he vomiting of blood. The angiomas are another indication of bleeding tendencies common with the selection of option 4, esophageal varices.
"has several drinks a day and has done this over the cirrhosis.
past 10 years." The client has spider angiomas on his
forearm and dried blood on his lips. What should the
nurse suspect?
1.‐ Hepatitis A
2.‐ Cerebral disorder
3.‐ Pancreatitis
4.‐ Esophageal varices

1409 A client asks, "What conditions lead to the Correct answer: 4 Obese, middle‐aged or older women are more likely to develop cholelithiasis. It is also seen in The common association of gallstones: Female, Fat, and Forty can assist in identification of
development of gallstones? My mother had a bad time Native Americans. the correct response.
with them." Which response is the best for the nurse
to make?
1.‐ "They occur more often in younger people."
2.‐ "They are limited to older malnourished men."
3.‐ "Gallstones are only seen in elderly Jewish people."
4.‐ "Overweight middle‐aged women are especially susceptible."

1410 In caring for a client recovering from hepatitis A, who Correct answer: 1 All of the other options contain fat. Fat generally is not appetizing to clients with hepatitis. If Look for the highest calorie, highest quality protein (eggs), and the lowest fat menu.
is no longer infectious and without any indication of there are no complications, it is better to give high‐calorie and high‐protein early in the
hepatic encephalopathy, which of the following meals morning before developing nausea.
would be most appropriate?
1.‐ Pancakes, poached eggs, orange juice, coffee (breakfast)
2.‐ Fried chicken, potatoes and gravy, green beans (noon)
3.‐ Enchiladas, tortillas, chips, salsa (evening)
4.‐ Salami sandwich, French fries, ketchup, coca cola (evening)
1411 The doctor has ordered oral administration of Correct answer: 4 Increased amount of gastrointestinal bleeding results in the formation of increased amount of Recall that Chronulac should be associated with removal of ammonia.
lactulose (Chronulac) for a client with esophageal ammonia because of intestinal bacteria metabolizing the blood cells. The lactulose creates an
bleeding from esophageal varices. The nurse knows acid environment, which causes the ammonia to leave the circulatory system and to be
that the reason for administration of this drug is to: expelled through the colon.

1.‐ Correct constipation.


2.‐ Stop bleeding from the varices.
3.‐ Act as a nutritional supplement.
4.‐ Promote excretion of ammonia.

1412 When admitting a client with a diagnosis of cirrhosis, Correct answer: 4 Medications such as acetaminophen (Tylenol) are highly metabolized by the liver and should Omit options 1 and 2 as these are commonly used to reduce ammonia. Demerol is the
the nurse assesses the medications ordered. She be avoided. Other such drugs are barbiturates and sedatives. Options 1 and 2 are ordered to better option for pain.
should question the administration of which of the decrease the ammonia level, and option 3 may be ordered for pain.
following?
1.‐ Lactulose (Chronulac)
2.‐ Neomycin (Mycifradin)
3.‐ Meperidine (Demerol)
4.‐ Acetaminophen (Tylenol)

1413 A client with cirrhosis of the liver and esophageal Correct answer: 2 Pain usually does not accompany ruptured varices. The increased venous pressure and gastric Eliminate options 1, 3, and 4 as commonly associated with ruptured varices.
varices suddenly begins vomiting copious amount of acid causes the rupture, which is usually followed by bleeding. Hypertension, melena, and high
dark‐colored blood. The sign/symptom that is least ammonia levels are all expected.
expected would be which of the following:

1.‐ Hypertension.
2.‐ Pain.
3.‐ Melena.
4.‐ High ammonia level.

1414 A client on the unit with hepatitis B suddenly Correct answer: 2 A condition developing 6 to 8 weeks after initial symptoms in a client with hepatitis is Omit option 1 as this is alcoholic cirrhosis, and option 3 as a complication of cirrhosis.
develops anorexia, vomiting, abdominal pain, fulminant hepatitis where there is necrosis and shrinking of the liver with possible liver Omit option 4 as cancer of the liver is a risk for later in life in a post‐hepatitis B client.
progressive jaundice, lethargy, and disorientation. The damage. The symptoms are classic and the condition often leads to coma, possibly death.
nurse knows that these indicate which of the
following?
1.‐ Laennec's cirrhosis
2.‐ Fulminant hepatitis
3.‐ Portal hypertension
4.‐ Cancer of the liver

1415 A client with a history of hiatal hernia states he has Correct answer: 1 Elevating the chest and head may reduce gastric reflux, which is causing the pain. The other Recall the most common recommendation for clients with hiatal hernia.
trouble sleeping because the pain is worse at night. positions do not accomplish this. Position change is preferable to medication for sleep.
Which response by the nurse is most appropriate?

1.‐ "Try sleeping with your upper body elevated."


2.‐ "What sleep medication do you take?"
3.‐ "Try laying flat or on your side."
4.‐ "Sleep with your feet elevated."
1416 A client is taking misoprostol (Cytotec) because of an Correct answer: 3 The action of this drug is to promote healing by promoting mucus and bicarbonate Protection of the gastric lining by increasing mucous production is a goal of ulcer
ulcer that developed while taking NSAIDs for arthritis. production. It promotes healing because it is a prostaglandin. NSAIDs decrease prostaglandins treatment.
The client questions the action of the drug. Which is and promote ulcer formation. It does not affect the proton pump, esophageal sphincter
the most appropriate explanation? pressure, or the speed of gastric emptying.

1.‐ This drug increases the speed of gastric emptying.


2.‐ It increases lower esophageal sphincter pressure.
3.‐ It promotes healing by promoting mucus production and bicarbonate secretions.
4.‐ It binds the enzyme that acts as a proton pump.

1417 Which of the following symptoms would lead the Correct answer: 4 Peritonitis follows gastric perforation with spilling of stomach contents into the peritoneal Rigid abdomen is a serious symptom and needs immediate intervention, just as peritonitis
nurse to suspect that a client has peritonitis? cavity. Increased white blood cell count, abdominal rigidity, and severe pain occur, does.
accompanied by fever. Dysuria is not seen. Although tarry stools may be seen with gastric
bleeding, it is not characteristic of peritonitis.
1.‐ Leukopenia
2.‐ Tarry stools
3.‐ Dysuria
4.‐ Abdominal rigidity

1418 A client with Crohn's disease has a low potassium Correct answer: 1 Potassium has a tendency to be irritating to the lining of the esophagus or stomach and may The strong taste and tendency to cause stomach upset should lead to the selection of
level from excessive diarrhea. Oral potassium chloride cause nausea and vomiting. Giving it with meals will decrease this tendency. Options 2 and 4 option 1.
(K‐lor) is prescribed. The nurse intends to give this would be irritating because the client has an empty stomach. It is appropriate to give this drug
drug: at the same time other medications are given.

1.‐ With food or after meals.


2.‐ 2 hours after meals.
3.‐ In conjunction with no other medications.
4.‐ Before breakfast and at bedtime.

1419 As opposed to ulcerative colitis, Crohn's disease is Correct answer: 3 Crohn's disease is characterized by lesions anywhere in the gastrointestinal system. The Omit options 2 and 4 as diarrhea and fluid and electrolyte disturbances are common issues
characterized by: diarrhea is more liquid and harder to control. The excessive diarrhea frequently causes fluid for the client with Crohn’s disease. Omit option 1 as Crohn’s lesions can occur anywhere in
and electrolyte imbalance. It is often accompanied by fistulas between the colon and other the GI tract.
organs or other segments of the bowel. The fibrotic changes cause the colon to be inflexible
and thick.
1.‐ Lesions that are limited to the lower colon and rectum.
2.‐ Diarrhea that is self‐limiting and easier to control.
3.‐ Fistulas and fibrotic changes that cause the bowel to become less flexible.
4.‐ Fewer problems with fluid and electrolyte imbalance.

1420 While recovering from a burn, a client suddenly Correct answer: 1 Curling's ulcers, which occur after a major burn, are characterized by multiple superficial Recall that stress ulcers are common post burn.
begins to vomit blood, but denies any pain. The nurse ulcers caused by ischemia from vasoconstriction. They are not caused by excessive gastric acid.
knows this is characteristic of: They are painless and may or may not involve massive bleeding. They are not related to H.
pylori.
1.‐ Multiple superficial ulcers that occur following a major trauma.
2.‐ Rupture of a blood vessel while coughing.
3.‐ Cancer of the stomach.
4.‐ Chronic gastritis caused by <i>H. pylori</i>.

1421 Using the diagram on common sites for peptic ulcer Correct answer: 2 An ulcer below the pyloric valve is a duodenal ulcer, and the pain is more common when the Remember that duodenal ulcers are relieved by food.
disease, a client with an ulcer below the pyloric valve stomach is empty; food helps stop the pain for awhile. Option 1 is common with gastric ulcers.
would complain of: Options 3 and 4 are not applicable to PUD.
1.‐ Pain immediately after a meal.
2.‐ Pain on an empty stomach.
3.‐ Pain all the time whether the stomach is full or empty.
4.‐ Pain that is worse when drinking milk or milk products.

1422 A client who had been admitted for shock develops Correct answer: 3 Severe stress conditions such as burns, hypoxia, and shock decrease mucus production, which Recall that mucous protects the stomach lining from gastric acid.
symptoms of peptic ulcer disease several days later. leaves the mucosal cells unprotected from the high acid environment of the stomach. Option 1
The reason this occured is most likely: and 2 are causes of PUD but not as a secondary condition of shock. Option 4 indicates the
pathophysiology of shock, not PUD.
1.‐ Excess acid production.
2.‐ Increased delivery of acid.
3.‐ Decreased mucus production.
4.‐ Decreased blood flow.

1423 In scheduling a b.i.d. dose of antacid for a client with Correct answer: 1 Antacids should be separated from other medications by at least 1 to 2 hours and after meals Select the option that would allow 1‐2 hours between meals and other medications and
a hiatal hernia (9:00 A.M. and 9:00 P.M.), the nurse by at least 1 hour. Therefore, if medications are scheduled for 9:00 A.M. and 9:00 P.M., the the antacid.
knows to schedule antacids at: antacid could be given an hour after (10:00 A.M./P.M.).
1.‐ 10:00 A.M. and 10:00 P.M.
2.‐ 11:00 A.M. and 11:00 P.M.
3.‐ 7:00 A.M. and 7:00 P.M.
4.‐ 12:00 noon and 12:00 midnight

1424 A client who has Crohn's disease complains of feeling Correct answer: 3 When diarrhea occurs for an extended time in a client with Crohn's disease, dehydration is a Recognize that the scenario describes classic symptoms of dehydration.
very tired, extremely thirsty, and has experienced complication. Signs and symptoms are excess thirst, fatigue, sunken eyeballs, and decreased
excessive diarrhea the last few days. The client has skin turgor. Although the client may be experiencing malabsorption (option 1) and electrolyte
sunken eyeballs and upon examination, poor skin imbalance (option 2), the scenario did not give symptoms consistent with these complications.
turgor. The nurse should recommend immediate
treatment for:
1.‐ Malabsorption syndrome.
2.‐ Electrolyte imbalance.
3.‐ Dehydration.
4.‐ Peritonitis.

1425 A client being treated for chronic cholecystitis should Correct answer: 2 These foods are all high in fat and are usually not tolerated with a client with cholecystitis. Recall that high fat foods aggravate gallbladder pain.
be given which of the following instructions? Clients should continue to ambulate and be as active as usual. Protein does not necessarily
have to be increased.
1.‐ Increase rest
2.‐ Avoid sausage, bacon, fried foods, and peanut butter
3.‐ Increase protein in diet
4.‐ Use anticholinergics as prescribed

1426 Which of the following laboratory values will the Correct answer: 1 The elevated amylase and lipase are key lab tests for pancreatitis. The glucose is elevated Omit options 2 and 4 as serum glucose would be elevated in pancreatic dysfunction. Omit
nurse interpret as confirming a client's diagnosis of because of the role of the pancreas in controlling glucose values. The calcium is decreased option 3 as pancreatic enzymes would be increased.
pancreatitis? because the calcium is deposited in the fatty necrotic tissue of the pancreas. In this type of
question, try to think of the rationale for the decrease or elevation, instead of memorizing.

1.‐ Elevated amylase, elevated lipase, elevated serum glucose and decreased serum calcium levels
2.‐ Elevated amylase, elevated lipase, decreased serum glucose, and decreased serum calcium levels
3.‐ Decreased amylase, decreased lipase, elevated serum glucose, and increased serum calcium levels
4.‐ Decreased amylase, decreased lipase, decreased serum glucose, and increased serum calcium levels.
1427 A client is admitted with cholelithiasis. The most Correct answer: 2 An ultrasound of the gallbladder will detect the presence of stones. A barium swallow, as well Select the option that is specific to the gallbladder.
common test to order for diagnosis of this disease is as endoscopy, is for upper gastrointestinal disorders. A CT scan is not usually the first choice if
which of the following? the stones are visible on the ultrasound.
1.‐ Abdominal computed tomography (CT) scans
2.‐ Ultrasound (US) of gallbladder
3.‐ Barium swallow
4.‐ Endoscopy

1428 A client with cirrhosis begins with a flapping tremor of Correct answer: 4 Hepatic encephalopathy is a complication of cirrhosis and is manifested by changes in Focus on the mental status changes. This will lead to the selection of option 4.
the hands whenever the arms are extended. Her consciousness, mentation, and motor function. Asterixis (or liver flap) is the flapping tremor of
orientation is decreased, she is having trouble the hands when extending the arms.
concentrating, and appears anxious. The client is
probably developing which of the following?

1.‐ Portal hypertension


2.‐ Esophageal varices
3.‐ Fulminant hepatitis
4.‐ Hepatic encephalopathy

1429 Which of the following measures should the nurse Correct answer: 1 The cause of bleeding in a client with esophageal varices is usually rupture, which is a medical Think of hemorrhage associated with esophageal varices.
focus on in the client with esophageal varices who is emergency. The hemorrhage that occurs is usually frank bleeding such as vomiting of copious
scheduled to undergo sclerotherapy? amounts of dark‐colored blood. The nurse should be able to recognize signs of hemorrhage
(tachycardia, hypotension, low platelets, and hematocrit and hemoglobin H &amp; H).

1.‐ Recognizing hemorrhage


2.‐ Controlling blood pressure
3.‐ Encouraging nutritional intake
4.‐ Teaching the client about varices

1430 In teaching a high school health class, the nurse Correct answer: 3 Alcoholism in high school age students is common, and this population usually feels Select the option that is a true risk for the population being addressed.
should emphasize the possibility of which of these? invulnerable to illnesses such as cirrhosis. Although biliary cirrhosis may occur because of drug
abuse, alcoholism is more prevalent in teenagers in the United States today.

1.‐ Biliary cirrhosis


2.‐ Cholecystitis
3.‐ Laennec's cirrhosis
4.‐ Cancer of the liver

1431 When admitting a client to the hospital with Correct answer: 3 The bilirubin will be elevated in cholelithiasis and cholecystitis. When the indirect bilirubin is Associate indirect with secondary cause of liver damage.
suspected acute cholecystitis, which of the following elevated, liver damage is suspected. The elevated direct bilirubin indicates involvement of the
would help the nurse to know if liver damage is biliary ducts. The amylase should be normal and the alkaline phosphatase confirms the
present? diagnosis.
1.‐ Elevated alkaline phosphatase
2.‐ Elevated direct bilirubin
3.‐ Elevated indirect bilirubin
4.‐ Normal serum amylase

1432 When a client who has a liver disorder is having an Correct answer: 1 Bleeding is a primary complication of a liver biopsy or invasive procedures involving the liver Recall that clotting is impaired with liver disease.
invasive procedure, the nurse helps assure safety by because the liver disorder has more than likely altered the clotting factors. In order to prevent
assessing the results of which of the following tests? a massive hemorrhage or complications, the coagulation studies should be assessed prior to
the procedure.
1.‐ Prothrombin time (PT) and Partial thromboplastin time (PTT)
2.‐ Liver enzyme levels
3.‐ Serum chemistries
4.‐ White blood cell count (WBC)

1433 When teaching preventive measures to a client who Correct answer: 2 High fat content in the diet as well as hyperlipidemia are risk factors for cholelithiasis. In Associate high fat intake with gall stones.
has a strong family history for cholelithiasis, which of addition, this client has a family history, which is another risk factor. Obesity is certainly a
the following guidelines are most important? concern, but the stem does not indicate that as a problem (don't read into the question).

1.‐ Eat a low‐protein diet


2.‐ Eat a low‐fat low‐cholesterol diet
3.‐ Limit exercise to 10 minutes a day
4.‐ Keep weight proportional to height

1434 Which of the following considerations is of the Correct answer: 4 In cirrhosis, the liver is usually not functioning properly and cannot metabolize medications as Remember that the liver plays a role in the metabolism of many common medication.
highest priority when preparing to administer a well as it normally would if healthy due to the scarring of the tissue. Certain medications are
medication to a client with cirrhosis? metabolized primarily by the liver, while other medications are metabolized by other organs.
Consideration should be made for each medicine ordered to avoid overburdening the liver.

1.‐ Frequency of the medication


2.‐ Purpose of the medication
3.‐ Necessity of the medication
4.‐ Metabolism of the medication

1435 A client asks the nurse how she can live without her Correct answer: 1 The liver produces between 700 and 1,000 mL of bile a day. The gallbladder stores and Recall that the role of the gallbladder is to store and concentrate, not to produce bile.
gallbladder. In order to respond to this client, the concentrates bile and then releases it when stimulated, but is not an essential structure.
nurse must have which understanding of the
hepatobiliary system?
1.‐ The liver produces about 1,000 mL of bile per day.
2.‐ The gallbladder makes about 90 mL of bile per day.
3.‐ The liver concentrates bile more than 10 times.
4.‐ The gallbladder dilutes and releases bile.

1436 The nurse reviews a client's laboratory tests and Correct answer: 3 Hyperbilirubinemia (total serum bilirubin greater than 2.5 mg/dL) manifests in jaundice, a Associate bilirubin with jaundice, which is often first noted in the sclera.
notices that the total serum bilirubin is 2.5 mg/dL. The yellow discoloration of the body tissues. Ascites (option 1) may accompany liver disease in
nurse should assess the client for which of these later stages, but there is no evidence in the question to indicate this. Options 2 and 4 are
clinical manifestations? unrelated to the question as stated.
1.‐ Ascites
2.‐ Diarrhea
3.‐ Scleral icterus
4.‐ Hypertension

1437 The client is diagnosed with obstructive jaundice. The Correct answer: 2 Clay colored stools indicate that no bile is reaching the intestine and suggests obstructive Recall that clay‐colored stool and dark urine are associated with jaundice.
nurse should ask the client about which of these jaundice. Options 1 and 3 are unrelated to the question. Option 4 could be present due to
manifestations? cardiovascular disease or as an indirect consequence of portal hypertension with impaired
venous return, but there is insufficient information in the question to support this option.

1.‐ Clear, pale urine


2.‐ Clay‐colored stools
3.‐ Lactose intolerance
4.‐ Ankle edema
1438 A client has jaundice. Which of the following comfort Correct answer: 3 Jaundice frequently causes pruritis. Comfort measures include keeping the air temperature Omit options 2 and 4 as both would increase the risk of drying of the skin.
measures would be appropriate for the nurse to cool (68 to 70 degrees F) and the humidity at 30 to 40 percent. Tepid baths (not hot) with
implement? colloidal agents decrease itching (option 2). Use of an emollient lotion is also helpful, but
anything drying should be avoided (option 4). Hot beverages (option 1) are of no benefit as a
comfort measure for pruritus due to jaundice.

1.‐ Offer hot beverages frequently


2.‐ Encourage taking a hot bath or shower
3.‐ Keep the air temperature at approximately 68 to 70 degrees F
4.‐ Suggest the use of alcohol‐based skin lotion

1439 The client is exposed to hepatitis A. When teaching Correct answer: 2 The incubation period for hepatitis A is 4 to 6 weeks in length with viral shedding highest 10 Select the only option that identifies a period of time before the symptoms develop.
this client about infection control, the nurse explains to 14 days before the onset of symptoms and during the first week of symptoms. The other
that the client is most infectious to others at which of options do not fall within this time frame.
these times?
1.‐ 7 days after exposure
2.‐ 10 days before the onset of symptoms
3.‐ 2 months after exposure
4.‐ 14 days after symptoms begin

1440 The client with cirrhosis of the liver asks the nurse Correct answer: 1 The liver is responsible for the production of albumin, which in turn is responsible for Select the option that describes the relationship between the liver and proteins.
why he has edema. The nurse would use which of the maintaining colloidal osmotic pressure. With less production of albumin, osmotic pressure
following statements to explain how edema results decreases and edema develops. Options 2, 3, and 4 are false statements that do not explain
from pathophysiologic changes in cirrhosis? the relationship between cirrhosis and edema.

1.‐ "The edema occurs because your liver produces fewer proteins that help draw fluid into the blood stream."
2.‐ "The high osmotic pressure of proteins in your blood pushes fluid into body tissues."
3.‐ "Because of the liver disease, the kidneys are able to filter less fluid, so the body cannot excrete it as urine very easily."
4.‐ "Your body is metabolizing sex hormones more quickly, leading to fluid retention."

1441 The client has just had a liver biopsy. Which of the Correct answer: 1 Complications of liver biopsy include hemorrhage or accidental penetration of biliary Recall that bleeding is the largest concern post liver biopsy. Vital sign changes are the only
following nursing actions would be the priority after canniculi. The nurse should assess for signs of hemorrhage (increased pulse, decreased blood way to detect this complication.
the biopsy? pressure) every 30 minutes for the first few hours and then hourly for 24 hours. The client
should be monitored for fever every 4 hours and remain on bedrest for 24 hours.

1.‐ Monitor pulse and blood pressure every 30 minutes until stable and then hourly for up to 24 hours.
2.‐ Ambulate every 4 hours for the first day as long as client can tolerate this.
3.‐ Measure urine specific gravity every 8 hours for the next 48 hours.
4.‐ Maintain NPO status for 24 hours post‐biopsy.

1442 Lactulose (Cephulac) is ordered for the client with Correct answer: 2 Lactulose (Cephulac) is a disaccharide laxative used to decrease the absorption of ammonia in Recall that Lactulose is used to assist in reduction of ammonia.
cirrhosis. Which of the following serum laboratory the intestines, thereby lowering the serum ammonia and resulting in improvement in hepatic
tests should the nurse monitor to determine if the encephalopathy.
drug is having the desired effect?

1.‐ Albumin
2.‐ Ammonia
3.‐ Sodium
4.‐ Lactate
1443 The client is admitted to the hospital for possible Correct answer: 4 Factors that increase the risk of gallstone formation include female gender, aging, use of oral Omit options 1 and 2 as having no relationship to the disease.
cholelithiasis. While taking the history, the nurse notes contraceptives, pregnancy, rapid weight loss, high cholesterol level, and diseases of the ileum.
that the client has which of the following risk factors
for the development of gallstones?

1.‐ Black race


2.‐ History of hypertension
3.‐ Age of 37 years
4.‐ Use of oral contraceptives

1444 The client is diagnosed with chronic pancreatitis, and Correct answer: 1 Pancrelipase (Lipancreatin) aids in the digestion of starches and fats and should be taken with Recall that the drug is used to replace the natural digestive enzymes from the pancreas.
pancrelipase (Lipancreatin) is prescribed. Which of the meals. It should not be crushed since hydrochloric acid destroys the drug, and it should not be
following instructions should the nurse give to this mixed with alkaline foods (milk, ice cream).
client about the administration of this medication?

1.‐ "Take the drug with meals."


2.‐ "Take the drug with a large glass of milk."
3.‐ "Take the drug between meals."
4.‐ "Take the drug after it is crushed and mixed with ice cream."

1445 Which of the following assessments is essential for Correct answer: 2 The posterior pharynx is anesthetized for easy passage of the endoscope into the esophagus. Any upper GI procedure using a scope requires the anesthesia of the gag reflex, which can
the nurse to make when caring for a client who has The return of the gag reflex indicates that normal function is returning and the client is able to lead to aspiration.
just had an esophagogastroduodenoscopy (EGD)? swallow.

1.‐ Auscultate bowel sounds


2.‐ Check gag reflex
3.‐ Monitor salivary pH
4.‐ Measure abdominal girth

1446 A client has a nasogastric (NG) tube in place for Correct answer: 3 Thick secretions and particulate matter may obstruct the tube, causing drainage to cease; the Select the option that assesses patency of the tube.
gastric decompression and complains of increasing client may experience nausea and vomiting. The tube should be gently flushed to ensure
nausea. Which action should the nurse take first? patency and rule out obstruction as the cause of the client's symptoms.

1.‐ Advance the tube 2 cm


2.‐ Place client in a recumbent position
3.‐ Instill 20 mL of saline
4.‐ Obtain abdominal x‐ray to assess placement

1447 A client with a subtotal gastrectomy is scheduled for Correct answer: 3 Dumping syndrome is the rapid dumping of food into the jejunum without proper mixing and Select the option that would slow the movement of food through the GI tract.
discharge. Which of these instructions should the digestion. Interventions that help to minimize dumping syndrome are lying down after eating,
nurse give the client to reduce the possibility of eating a diet high in fat and protein and low in carbohydrates, and no fluids with meals.
dumping syndrome?
1.‐ "Be sure to eat foods high in complex carbohydrates."
2.‐ "It is helpful to take a walk after eating."
3.‐ "Avoid drinking fluids with your meal."
4.‐ "Don't lie down for at least 2 hours after eating."

1448 A 32‐year‐old client is admitted to the hospital with a Correct answer: 2 BMI is an estimation of total body fat in relation to height and weight. An optimal BMI is 20 to Omit options 1 and 4 as they reflect extremes. Omit option 3 as there is no information to
body mass index (BMI) of 25. The nurse interprets this 25, increasing to 24 to 27 in the elderly. support this.
to mean the client:
1.‐ Is undernourished.
2.‐ Has an optimal amount of body fat.
3.‐ Is 10 percent overweight.
4.‐ Is morbidly obese.

1449 The nurse teaches the client with gastroesophageal Correct answer: 1 The client with GERD is encouraged to eat smaller, more frequent, low‐fat meals and to avoid Remember that tea contains caffeine.
reflux disease (GERD) about ways to minimize lying down after eating. Clients are instructed to not eat for at least 2 hours before bedtime
symptoms. Which of the following statements made by and avoid foods that decrease lower esophageal sphincter pressure such as anything
the client indicates that more teaching is needed? containing caffeine (coffee, tea, cola, chocolate).

1.‐ "I will be sure to drink tea instead of coffee."


2.‐ "I will take a walk after I eat."
3.‐ "I will try to eat smaller meals more frequently."
4.‐ "I will sleep with the head of the bed elevated about 12 inches.”

1450 The client with a gastric ulcer is admitted to the Correct answer: 1 Gastric ulcers are usually a result of a disruption of the protective mechanism of the gastric Recall that aspirin is often associated with ulcer formation.
hospital. The nurse should assess the client for intake epithelium. Substances that reduce prostaglandin secretion in the gastric mucosa (aspirin,
of which of these substances that increases the risk of NSAIDs, alcohol) are responsible for gastric ulcers. Although certain foods and fluids may
developing a gastric ulcer? aggravate an existing ulcer, they do not cause them.
1.‐ Aspirin
2.‐ Spicy foods
3.‐ Acetaminophen (Tylenol)
4.‐ Coffee

1451 The client with gastroesophageal reflux disease Correct answer: 4 Famotidine (Pepcid) is a histamine‐2 receptor antagonist and reduces the secretion of gastric Think of the pathophysiology of GERD, which is reflux of gastric acid into the esophagus.
(GERD) is prescribed famotidine (Pepcid). In order to acid. This class of drugs does not have a direct effect on reflux, LES tone, or GI motility. The drug is used to reduce acid.
provide effective teaching, the nurse must have which
of these understandings about the action of the drug?

1.‐ The drug improves gastric motility.


2.‐ The drug coats the distal portion of the esophagus.
3.‐ The drug increases LES tone.
4.‐ The drug decreases the secretion of gastric acid.

1452 The client is admitted to the hospital with ulcerative Correct answer: 1 Hemorrhage and bleeding is a common feature of ulcerative colitis, and over time this can Recall that bleeding is common with ulcerative colitis.
colitis. The nurse should assess the client for which lead to significant loss of RBCs. The client should be assessed for possible anemia.
complication of the disease?
1.‐ Anemia
2.‐ Steatorrhea
3.‐ Cholelithiasis
4.‐ Thrombocytopenia

1453 The nurse is developing a health promotion program Correct answer: 1 Diverticular disease is virtually unknown in cultures where highly refined foods are not Recall that the addition of dietary fiber is generally considered a good health promotion
for intestinal health. Which of the following pieces of available (e.g., Africa, Asia) and was unknown in the United States prior to 1900. The other measure.
information should the nurse include in the program? statements are false.

1.‐ The addition of dietary fiber can reduce the risk of diverticulosis.
2.‐ A diet high in fat increases the risk of developing Crohn's disease.
3.‐ Irritable bowel syndrome is caused by a deficiency in soluble fiber.
4.‐ Laxatives can improve motility and bowel health.
1454 A client is admitted to the hospital with a bowel Correct answer: 1 Early in a bowel obstruction, the bowel attempts to move the contents past the obstruction, Omit options 2 and 4 as they are descriptive of normal. Omit option 3 as a late sign.
obstruction. Which of these findings by the nurse and this is heard as high‐pitched tinkling bowel sounds. As the obstruction progresses, bowel
would indicate that the obstruction is in the early sounds will diminish and may finally become absent.
stages?
1.‐ High‐pitched, tinkling bowel sounds
2.‐ Low rumbling bowel sounds
3.‐ No bowel sounds auscultated
4.‐ Normal bowel sounds heard in all four quadrants

1455 A client with cirrhosis is admitted to the hospital. Correct answer: 1 In cirrhosis, the liver becomes fibrotic, which obstructs the venous blood flow through the Recall that ‘heme’ means blood. Portal hypertension causes leaking of the esophageal
Which of the following assessments made by the nurse liver. This increases the vascular pressure in the portal system, and causes congestion in the vessels.
would indicate the development of portal spleen and development of varicosities in the esophagus. Bleeding esophageal varices are a
hypertension? complication of portal hypertension and result in vomiting of blood and possible hemorrhage
and death.
1.‐ Hematemesis
2.‐ Asterixis
3.‐ Elevated blood pressure
4.‐ Confusion

1456 The nurse should teach the client with liver disease to Correct answer: 4 Any medication that is metabolized by the liver should be avoided, such as acetaminophen, Recall that Tylenol is metabolized by the liver and should be avoided.
avoid which of these over‐the‐counter medications sedatives, and barbiturates. Ranitidine is a histamine&lt;sub&gt;2&lt;/sub&gt; reception
after discharge? antagonist, psyllium is a laxative, and ascorbic acid is Vitamin C.

1.‐ Ranitidine (Zantac)


2.‐ Psyllium (Metamucil)
3.‐ Ascorbic acid (Vitamin C)
4.‐ Acetaminophen (Tylenol)

1457 The nurse is doing discharge teaching for a client who Correct answer: 3 A low‐sodium diet is recommended for clients that have cirrhosis and ascites. Potato chips Recall that potato chips would rarely be a recommended snack.
has cirrhosis and ascites. Which of the following foods are high in sodium. Cookies and hard candy are high in sugar, while bread is high in complex
used by the client as snacks should the nurse instruct carbohydrates.
the client to avoid?
1.‐ Whole wheat bread
2.‐ Cookies
3.‐ Potato chips
4.‐ Hard candy

1458 The client who has liver disease asks the nurse why he Correct answer: 1 The liver synthesizes clotting factors I, II, VII, IX, and X as well as prothrombin and fibrinogen. Recall the liver’s role in clotting to select option 1 as correct.
bruises so easily. Which of the following information These substances are needed for adequate clotting, so their reduction leads to increased risk
should the nurse include in the response? of bleeding. The other responses do not address this concern.

1.‐ "Your liver is unable to make the proteins that are needed to make clotting factors."
2.‐ "Your liver can no longer metabolize drugs and render them inactive."
3.‐ "Your liver is breaking down blood cells too rapidly."
4.‐ "Your liver can't store Vitamin C any longer."

1459 A client is seen in the clinic for a routine physical Correct answer: 2 HBsAg is hepatitis surface antigen and is usually present before symptoms manifest. It Recognize that this is the antigen, therefore it reflects active disease.
examination and the laboratory test results indicate an indicates acute disease. The other options are incorrect conclusions regarding this test result.
elevated HBsAg. In order to plan teaching for this
client, the nurse interprets this lab result to mean:

1.‐ The client has immunity to hepatitis B.


2.‐ The client has active hepatitis B.
3.‐ The client has resolving hepatitis B.
4.‐ The client has had the hepatitis B vaccine.

1460 The client who has esophageal varices is receiving a Correct answer: 1 Vasopressin causes vasoconstriction and may precipitate an acute anginal attack or Select the most serious vascular symptom.
vasopressin infusion. Which of these findings would myocardial infarction, especially in those with known cardiovascular disease. The other options
indicate a complication of this therapy? are unrelated to the question.

1.‐ Chest pain


2.‐ Tinnitus
3.‐ Flushed skin
4.‐ Polyuria

1461 The client who has cholelithiasis is scheduled for Correct answer: 1 After the extracorporeal shock wave lithotripsy, the nurse should monitor for biliary colic and Select the symptom that would be most like cholelithiasis symptoms.
extracorporeal shock wave lithotripsy. The nurse nausea. The colicky pain is caused by passage of stone fragments through the biliary tree into
should tell the client about which of these symptoms the small intestine. Headache, diarrhea, and hiccoughs are unrelated manifestations.
that may occur after this procedure?
1.‐ Colic‐type pain
2.‐ Headache
3.‐ Diarrhea
4.‐ Hiccups

1462 The client who has acute cholecystitis tells the nurse, Correct answer: 3 With the advent of laparoscopic surgical technique, the only absolute contraindication for Select the option that acknowledges the clients concern while offering an accurate
"I just want my gallbladder taken out now." Which of surgery is acute infection. The other options do not address this concern. explanation.
the following is the best response by the nurse?

1.‐ "I don't blame you, but they want your pain under control first."
2.‐ "Would you like me to ask if your physician will schedule surgery today?"
3.‐ "The symptoms are distressing, but the surgeon must wait until your gallbladder is less infected."
4.‐ "They will try to dissolve the stones before they do the surgery.”

1463 The client is admitted to the hospital with acute Correct answer: 2 Pancreatitis is associated with alcoholism in men and gallstones in women. The disorders in Associate acute pancreatitis with alcohol abuse.
pancreatitis. The nurse taking a history should question options 1 and 3 are not associated with increased risk of pancreatitis, while option 4 promotes
the client about which of these risks for developing health.
pancreatitis?
1.‐ Inflammatory bowel disease
2.‐ Alcoholism
3.‐ Diabetes mellitus
4.‐ High‐fiber diet

1464 The client with chronic pancreatitis is being Correct answer: 1 The client with chronic pancreatitis may require pancreatic enzyme supplements such as ‘Pancrease’ sounds like pancreas. Use this finding to select the correct response.
discharged. The nurse should anticipate teaching the pancrelipase (Lipancreatin). These will promote proper digestion of foods. The other
client about which of these medications? medications do not address this need.
1.‐ Pancrelipase (Pancrease)
2.‐ Morphine sulfate
3.‐ Biotin
4.‐ Lactulose (Cephulac)

1465 A client is to receive gavage feeding through an NG Correct answer: 2 Keeping the client in a high Fowler's position minimizes the risk of aspiration. The other Select the option that will help to prevent aspiration.
tube. Which of the following nursing actions should be options do not address this priority issue of care.
instituted to prevent complications?
1.‐ Flush with 20 mL of air
2.‐ Place client in high Fowler's position
3.‐ Advance tube 1 cm
4.‐ Plug the air vent during feeding

1466 A client is to have an intestinal tube placed to Correct answer: 3 Activity, including position changes and ambulation, stimulate intestinal peristalsis and assist Recognize that the intestinal tube needs to advance in order to be effective.
decompress the bowel. Which of the following in the forward movement of the tube.
explanations should the nurse give to the client about
what to expect?
1.‐ "You will need to remain on bed rest until the tube is removed."
2.‐ "While the tube is in place, you will need to lie on your right side."
3.‐ "Walking in the hall will help move the tube forward."
4.‐ "Keeping the bed flat should make you more comfortable."

1467 The nurse is caring for a client with a Sengstaken‐ Correct answer: 4 Scissors should be kept at the bedside of all clients with an esophagogastric tube and the tube Use the ABCs (airway, breathing, circulation) to select the best response. If the client is
Blakemore tube. Which of the following actions should should be cut if the client experiences respiratory compromise. Maintaining the client's airway having difficulty breathing it is likely due to the balloon obstructing the airway. Removal of
the nurse take first if the client suddenly experiences is the first priority of care. the tube is the best option.
difficulty breathing?
1.‐ Elevate the head of the bed
2.‐ Apply oxygen with a nasal cannula
3.‐ Listen to the client's lungs
4.‐ Cut and remove the tube

1468 The client returns to the nursing unit postoperatively Correct answer: 2 A healthy stoma is red to reddish‐pink, moist, and shiny. A stoma that appears dark red, Recognize that a bluish stoma indicates lack of blood flow.
after a colostomy. Which of the following assessments bluish, or black indicates ischemia or necrosis. This finding must be reported immediately
would require immediate action by the nurse? because the viability of the tissue is at risk. Options 3 and 4 are of no concern immediately
postop.
1.‐ Stoma is bright red.
2.‐ Stoma is bluish.
3.‐ Stoma is draining serous fluid.
4.‐ Stoma is draining no fluid.

1469 A client who had a Billroth I procedure is beginning to Correct answer: 2 Symptoms of dumping syndrome can occur within 5 minutes to 3 hours after eating and Recall that dumping syndrome causes diarrhea.
eat solid foods. The nurse should assess the client for include nausea, vomiting, tachycardia, diaphoresis, abdominal pain, diarrhea, syncope, and
the development of dumping syndrome. Which of the hyperactive bowel sounds.
following assessments would be indicative of dumping
syndrome? The presence of:

1.‐ Bradycardia.
2.‐ Diarrhea.
3.‐ Dyspnea.
4.‐ Coughing.

1470 A client is admitted to the hospital in a malnourished Correct answer: 1 Undernutrition affects many systems, causing decreases in metabolic function and cell‐ Recall that poor nutrition reduces ability to fight infection.
state. The nurse understands the client is at a high risk mediated and humoral immunity, thereby increasing the susceptibility to infection. The other
for which of the following conditions as a result of responses are incorrect.
decreased nutrition?
1.‐ Infection
2.‐ Diarrhea
3.‐ Fever
4.‐ Tumor formation
1471 The nurse is preparing a client with hiatal hernia for Correct answer: 4 Conservative treatment for hiatal hernia consists of lifestyle changes including remaining The most common instruction is for the client to remain upright after a meal.
discharge. Which of the following statements made by upright after eating; avoiding straining, tight clothing, and vigorous exercise; and eating small,
the client would indicate that teaching has been frequent, low‐fat meals.
effective?
1.‐ "I will join the gym and get in shape by lifting weights."
2.‐ "I know I need to eat a high‐fat diet to slow down my digestion."
3.‐ "I will join a support group."
4.‐ "I will take a walk after dinner each night."

1472 The nurse should question the client with Correct answer: 2 Many common substances contribute to decreased LES pressure including fatty foods, Recall that calcium channel blockers cause muscular relaxation.
gastroesophageal reflux disease (GERD) about the use caffeinated beverages, nicotine, beta‐adrenergic blocking agents, calcium channel blockers,
of which of these medications that decreases LES nitrates, theophylline, peppermint, alcohol, high levels of estrogen and progesterone, and
pressure? anticholinergic drugs.
1.‐ Antidepressants
2.‐ Calcium channel blockers
3.‐ Antiestrogen agents
4.‐ Alpha‐adrenergic blocking agents

1473 The client with irritable bowel syndrome (IBS) asks Correct answer: 3 There is no known cause of IBS, and diagnosis is made by excluding all the other diseases that Omit options 1 and 4 as there is no causative relationship between foods and IBS.
the nurse what causes the disease. Which of the cause the symptoms. There is no inflammation of the bowel. Some factors exacerbate the
following responses by the nurse would be most symptoms, including anxiety, fear, stress, depression, some foods and drugs, but these do not
appropriate? cause the disease.
1.‐ "This is an inflammation of the bowel caused by eating too much roughage."
2.‐ "IBS is caused by a stressful lifestyle."
3.‐ "The cause of this condition is unknown."
4.‐ "There is thinning of the intestinal mucosa caused by ingestion of gluten."

1474 A client with Crohn's disease (regional enteritis) who Correct answer: 2 Sulfasalazine is a GI anti‐inflammatory medication that exerts its action by decreasing Recall that anti‐inflammatory medications often work by influencing prostaglandin
is taking sulfasalazine (Azulfidine) asks the nurse why prostaglandin production in the bowel. It does not have the other effects listed. production.
this medication is necessary. Which information should
the nurse include in the response?

1.‐ The drug decreases abdominal cramping by slowing peristalsis.


2.‐ The drug decreases prostaglandin production in the bowel so it decreases inflammation.
3.‐ The drug inhibits neurotransmission of pain impulses.
4.‐ The drug stimulates the release of endorphins so pain is relieved.

1475 The nurse should evaluate results of which of the Correct answer: 1 Many clotting factors are produced in the liver including fibrinogen (factor I), prothrombin The item asks about safety. Recall that bleeding is a safety concern for clients with
following laboratory tests for a client with cirrhosis in (factor II), factor V, serum prothrombin conversion accelerator (factor VII), factor IX, and factor cirrhosis and PT results would help to address this.
order to plan for safe care? X. The prothrombin time will evaluate blood clotting ability while the others will not.

1.‐ Prothrombin time


2.‐ Urinalysis
3.‐ Serum lipase
4.‐ Troponin

1476 The nurse is caring for a client with a history of Correct answer: 3 Manifestations of chronic pancreatitis include nausea, vomiting, weight loss, flatulence, The pain of pancreatitis is not in the flank. Weight loss is associated with pancreatitis.
alcoholism. Which of the following findings would constipation, and steatorrhea that result from a decrease in pancreatic enzyme secretion. Hiccoughs are not related. Steatorrhea is due to the inability to breakdown fats.
indicate that the client has possibly developed chronic Weight gain (option 1) is the opposite of what occurs with this disorder, while options 2 and 4
pancreatitis? are unrelated.
1.‐ Weight gain
2.‐ Flank pain
3.‐ Steatorrhea
4.‐ Hiccoughs

1477 The nurse caring for a client with hemolytic jaundice Correct answer: 1 Hemolytic jaundice is caused by excessive breakdown of red blood cells and the amount of Recall that indirect means due to a secondary cause, in this case the inability of the liver to
would expect which of these findings on the laboratory bilirubin produced exceeds the ability of the liver to conjugate it, so there is an increase in metabolize the excessive RBC breakdown.
results? indirect bilirubin. Unconjugated bilirubin is insoluble in water and is not found in the urine.

1.‐ Elevated serum indirect Bilirubin


2.‐ Decreased serum protein
3.‐ Elevated urine Bilirubin
4.‐ Decreased urine pH

1478 The client has cholelithiasis. Which of the following Correct answer: 4 Nausea and RUQ pain occur in cystic duct disease, but obstruction of the common bile duct Recall that obstruction of bile flow causes jaundice.
assessment findings indicates to the nurse that the results in reflux of bile into the liver, which produces jaundice. Alkaline phosphatase increases
stone has probably obstructed the common bile duct? with biliary obstruction but not cholesterol.

1.‐ Nausea
2.‐ Hypercholesterolemia
3.‐ Right upper quadrant (RUQ) pain
4.‐ Jaundice

1479 The nurse caring for a client with uncomplicated Correct answer: 2 Obstructive biliary disease causes a significant elevation in alkaline phosphatase. Obstruction Omit options 1 and 3 as having no relationship to gall bladder disease.
cholelithiasis should expect an elevation in which of in the biliary tract causes an elevation in direct bilirubin, not indirect bilirubin (option 4).
these laboratory tests? Options 1 and 3 are unrelated.
1.‐ Serum amylase
2.‐ Alkaline phosphatase
3.‐ Mean corpuscular hemoglobin concentration (MCHC)
4.‐ Indirect Bilirubin

1480 In caring for the client 4 days post‐cholecystectomy, Correct answer: 3 The T‐tube may drain 500 mL in the first 24 hours and decreases steadily thereafter. If there is Omit options 2 and 4 as these actions would not influence drainage.
the nurse notices that the drainage from the T‐tube is excessive drainage, the physician should be notified immediately. Option 1 would be
600 mL in 24 hours. Which is the appropriate action by contraindicated while options 2 and 4 are of no help.
the nurse?
1.‐ Clamp the tube q 2 hours for 30 minutes
2.‐ Place the client in a supine position
3.‐ Notify the physician
4.‐ Encourage an increased fluid intake

1481 The post‐cholecystectomy client asks the nurse when Correct answer: 1 When T‐tube drainage subsides and stools return to a normal brown color, the tube can be Omit option 2 as the removal of staples is not the criteria for removal of the t‐tube.
the T‐tube will be removed. Which of the following clamped 1 to 2 hours before and after meals in preparation for tube removal. If the client Recognize that option 4 is incorrect as the first post operative day is too soon for tube
responses by the nurse would be appropriate? tolerates clamping, the tube will then be removed. removal.

1.‐ "When your stool returns to a normal brown color the tube can be removed."
2.‐ "The tube will be removed at the same time as your staples."
3.‐ "When the tube stops draining, it will be removed."
4.‐ "The tube is usually removed the day after surgery."
1482 Which of the following assessments made by the Correct answer: 1 Obstruction to portal blood flow causes a rise in portal venous pressure resulting in Look for the option that would reflect increased venous pressure.
nurse could indicate the development of portal splenomegaly, ascites, and dilation of collateral venous channels predominantly, in the
hypertension in a client with cirrhosis? paraumbilical and hemorrhoidal veins, the cardia of the stomach, and extending into the
esophagus.
1.‐ Hemorrhoids
2.‐ Bleeding gums
3.‐ Muscle wasting
4.‐ Hypothermia

1483 The nurse is caring for a client who has ascites and Correct answer: 3 Spironolactone (Aldactone) is used in clients with ascites that show no improvement with Recall that Aldactone is the most common potassium sparing diuretic.
the healthcare provider prescribes spironolactone bedrest and fluid restriction. It inhibits sodium reabsorption in the distal tubule and promotes
(Aldactone). The client asks why this drug is being potassium retention by inhibiting aldosterone.
used. Which is the best response by the nurse?

1.‐ "This will help increase the level of protein in your blood."
2.‐ "The drug will cause an increase in the amount of the hormone aldosterone your body produces."
3.‐ "This medication is a diuretic but does not make the kidneys excrete potassium."
4.‐ "This will help you excrete larger amounts of ammonia."

1484 When caring for a client that has cirrhosis, the nurse Correct answer: 4 Asterixis, also called liver flap, is the flapping tremor of the hands when the arms are Recall that asterixis is called liver flap.
notices flapping tremors of the wrist and fingers. How extended.
should the nurse chart this finding?
1.‐ Trousseau's sign noted
2.‐ Caput medusa noted
3.‐ Fetor hepaticus noted
4.‐ Asterixis noted

1485 A client has a total gastrectomy. The nurse should Correct answer: 2 The loss of parietal cells that secrete intrinsic factor results in Vitamin Recall that parietal cells in the stomach are needed for B&lt;sub&gt;12&lt;/sub&gt;
teach the client about long‐term treatment for which B&lt;sub&gt;12&lt;/sub&gt; deficiency post‐gastrectomy. For this reason, clients require absorption.
of these conditions? Vitamin B&lt;sub&gt;12&lt;/sub&gt; injections for life.
1.‐ Vitamin K deficiency
2.‐ Vitamin B12 deficiency
3.‐ Vitamin A deficiency
4.‐ Vitamin C deficiency

1486 A client with suspected duodenal ulcer has an upper Correct answer: 1 A side effect of Gastrografin is diarrhea. It does not cause the other signs and symptoms Select the gastrointestinal symptom listed.
GI series with meglumine diatrizoate (Gastrografin). listed.
The nurse should inform the client about which of the
following side effects of this contrast medium?

1.‐ Diarrhea
2.‐ Peritonitis
3.‐ Flushing
4.‐ Dysuria

1487 A client with diverticular disease undergoes a Correct answer: 3 Bowel perforation is a possible result of colonoscopy, if the colonoscope accidentally pierces Recall that perforation would always be a risk when an instrument is introduced into the
colonoscopy. The nurse should assess the client for the bowel wall. The other options are incorrect. bowel.
which of the following possible complications of the
procedure?
1.‐ Diarrhea
2.‐ Obstruction
3.‐ Bowel perforation
4.‐ Infection

1488 A nasogastric (NG) tube is ordered to be inserted in a Correct answer: 1 Use of the high Fowler's position utilizes gravity to protect against aspiration and is the Select the option that puts the client in the highest position.
client with a possible bowel obstruction. In position of choice for NG tube insertion. Option 2 provides minimal protection against
preparation for this procedure, the nurse places the aspiration, while options 3 and 4 provide none.
client in which position?
1.‐ High Fowler's
2.‐ Head of the bed elevated 20 degrees
3.‐ Lying on right side
4.‐ Flat

1489 After completing a gavage feeding using a bag and Correct answer: 3 The tube should be clamped to prevent air from entering the stomach; air causes cramping Omit option 1 as this would increase the chance for aspiration. Omit option 2 as water
tubing for a client receiving an enteral formula, the and bloating. Next, the tube should be flushed with water (30 ‐ 60 mL). The client should should be used to clear or flush the line.
nurse should take which of these actions first? remain in a high Fowler's position or an elevated side‐lying position for 30 to 60 minutes to
reduce the risk of aspiration.
1.‐ Position the client in the left lateral position
2.‐ Instill 30 mL of air to clear the line
3.‐ Clamp the tube
4.‐ Flush the tube with water

1490 Which of the following nursing actions should be a Correct answer: 1 Intestinal tubes are used to treat bowel obstruction, a symptom of which is a fecal taste in Select the option that names a structure that is part of the GI tract.
priority for a client with an intestinal tube? the mouth. Frequent mouth care including hard candy, ice chips, and throat lozenges is
essential to reduce the experience of fecal taste. The client is encouraged to move in bed and
ambulate to assist in the advancement of the tube.
1.‐ Mouth care
2.‐ Vital signs
3.‐ Skin care
4.‐ Elevate side rails

1491 The nurse is caring for a client who has severe Correct answer: 4 Excessive blood loss will result in the development of shock symptoms such as tachycardia, Consider the ABCs (airway, breathing, and circulation). After these have been addressed,
continuous bleeding from a self‐inflicted wrist cool, clammy skin, and changes in mental status, because hypovolemia leads to additional interventions can be implemented safely.
laceration. While applying direct pressure to the area vasoconstriction and shunting of blood to the central circulation. Applying ice to lower the
with a dry, sterile dressing, the nurse's next action body temperature would not be appropriate since the body temperature is usually decreased
would to be to do which of the following? with hypovolemia. Measures to decrease circulation to the affected extremity, such as
elevation of that extremity to heart level would be appropriate, although elevating it further
could lead to ischemia. A psychiatric evaluation would be appropriate after the client has
become hemodynamically stable.

1.‐ Call for a psychiatric evaluation since this was a self‐inflicted injury
2.‐ Apply ice to lower the body temperature to slow circulation
3.‐ Lower the extremity to below heart level
4.‐ Assess for signs of shock

1492 The nurse should encourage the client with dumping Correct answer: 3 Dumping syndrome can occur following gastrectomy, in which gastric contents rapidly enter Select the highest carbohydrate option as these foods are too easily digested.
syndrome to avoid which of these foods? the bowel. Dietary fats and proteins are increased, and carbohydrates, especially simple
carbohydrates such as fruits, are reduced. This helps slow the GI transit time and reduce the GI
cramping, diarrhea, and vasomotor symptoms.
1.‐ Eggs
2.‐ Cheese
3.‐ Fruit
4.‐ Pork
1493 A client is admitted to the hospital for morbid obesity Correct answer: 4 The individual is considered obese at 20 percent over ideal body weight and morbidly obese Select the option with the highest number as morbid obesity is considered severe.
and asks what that means. In order to answer the when over 100 percent above the ideal body weight.
client's question, the nurse should understand that
morbid obesity is determined when the client is:

1.‐ 20 percent over ideal body weight.


2.‐ 40 percent over ideal body weight.
3.‐ 60 percent over ideal body weight.
4.‐ 100 percent over ideal body weight.

1494 The nurse would expect which of these assessment Correct answer: 1 The pain of a gastric ulcer is dull and aching and occurs after eating and is not relieved by Recall that pain relieved by food intake is associated with ulcers.
findings in a client with duodenal ulcers? food as is the pain from duodenal ulcer. Chronic aspirin use is irritating to the stomach (option
2). The manifestations in options 3 and 4 are unrelated.
1.‐ Epigastric pain relieved by food
2.‐ History of chronic aspirin use
3.‐ Distended abdomen
4.‐ Positive fluid wave

1495 Which of the following statements made by a client Correct answer: 4 Many salt substitutes use potassium chloride. Potassium intake is carefully regulated in The core issue of the question is the ability to determine accurate statements about self‐
with chronic renal failure and who is on hemodialysis clients with renal failure, and the use of salt substitutes will worsen hyperkalemia. Increases in care of clients with renal failure. Specifically, clients need to restrict both sodium and
indicates the need for further teaching? weight (option 1) do need to be reported to the health care provider as a possible indication of potassium, and salt substitutes are high in potassium. Use nursing knowledge and the
fluid volume excess. The control of hypertension (option 2) is essential in the management of a process of elimination to make a selection.
client with renal failure. An AV fistula does need to be protected from injury that could be
caused by constricting clothing, venipunctures, and other items (option 3).

1.‐ “I will report any increase in my weight of 5 pounds in a 2‐day period.”


2.‐ “I take my prescribed antihypertensive drugs daily.”
3.‐ “I am careful to take precautions in the arm with the AV fistula.”
4.‐ “I comply with salt restrictions in my diet by using salt substitutes.”

1496 What type of renal failure would the nurse expect to Correct answer: 4 Nephrotoxic drugs, such as aminoglycoside antibiotics (tobramycin), can damage the The core issue of the question is the ability to associate causes of renal failure with their
see in a client who overdosed accidentally on nephrons and cause intrarenal (within the kidneys) failure. There is no condition called categories in specific client situations. Use nursing knowledge and the process of
tobramycin (Nebcin)? extrarenal failure. elimination to make a selection.
1.‐ Prerenal failure
2.‐ Postrenal failure
3.‐ Extrarenal failure
4.‐ Intrarenal failure

1497 A client with urinary tract infection (UTI) is prescribed Correct answer: 2 The drug makes the urine reddish orange in color, and the client should be advised that this The core issue of the question is knowledge of expected adverse effects of Pyridium. Use
phenazopyridine (Pyridium). Which of the following might stain the underwear and other clothing. The client should also be reassured that it nursing knowledge and the process of elimination to make a selection.
instructions would the nurse give the client? should not be confused with blood in the urine. The use of Pyridium in UTI is controversial
because it does not target the cause of the infection. However, it offers relief of UTI symptoms
such as pain, frequency, and urgency (option 1). Taking the drug after meals minimizes GI
symptoms associated with the use of this drug (option 3). Option 4 is incorrect because the
indiscriminate use of a urinary analgesic can mask symptoms and delay initiation of treatment.

1.‐ “This drug will take care of the infection causing your symptoms.”
2.‐ “Your urine may turn reddish orange and may cause staining of your clothes.”
3.‐ “Take the drug before meals to minimize GI symptoms.”
4.‐ “Always keep this drug and use it at the first symptom of a UTI.”
1498 A client with a urinary diversion device has the Correct answer: 3 Emptying the reservoir bag every 2 hours prevents overfilling and possible leakage of urine The core issue of the question is knowledge of appropriate care for a client with a urinary
nursing diagnosis risk for impaired skin integrity. Which into the skin surface. The urine collection device should be changed as needed to maintain diversion. Use nursing knowledge and the process of elimination to make a selection.
of the following interventions will the nurse use with integrity of the system. Self‐catheterization is not appropriate for this nursing diagnosis.
this client? Monitoring for foul‐smelling urine and monitoring for signs of infection are more appropriate
interventions for the diagnosis risk for infection.

1.‐ Change urine collection device every other day.


2.‐ Teach self‐catheterization technique.
3.‐ Empty the bag reservoir every 2 hours.
4.‐ Monitor for foul‐smelling urine.

1499 A client with renal calculi is advised to restrict calcium Correct answer: 3 Chocolate, smoked fish, milk products, beans, lentils, and dried fruits are high in calcium. In The core issue of the question is knowledge of foods that are high in calcium. Use nursing
in his diet. The nurse determines that the client calcium phosphate and calcium oxalate calculi, dietary management includes an acid‐ash diet knowledge and the process of elimination to make a selection.
understands the restrictions when he states he will and limiting foods high in calcium and oxalate.
avoid which of the following?
1.‐ Chicken, beef, and salmon
2.‐ Green vegetables, fruit, and legumes
3.‐ Chocolate, smoked fish, and low‐fat milk
4.‐ Eggs, meat, and poultry

1500 In conducting client teaching with a client who will Correct answer: 4 Peritonitis is a grave complication of peritoneal dialysis, caused by bacteria that may enter The core issue of the question is knowledge of the complications associated with
undergo peritoneal dialysis at home, the nurse through the catheter or dialysate solution. Hypotension is a common complication of peritoneal dialysis and their relative frequency. Use nursing knowledge and the process of
includes discussion of what common and significant hemodialysis but not peritoneal dialysis (option 2). Pulmonary embolism and dyspnea are not elimination to make a selection.
complication of peritoneal dialysis? common complications of peritoneal dialysis.
1.‐ Pulmonary embolism
2.‐ Hypotension
3.‐ Dyspnea
4.‐ Peritonitis

1501 The nurse is preparing to admit a client with urge Correct answer: 4 Urge incontinence is the unpredictable passage of urine soon after a strong urge to void is Use nursing knowledge and the process of elimination to make a selection.
incontinence. In writing the nursing care plan, the felt. Option 1 describes total incontinence, option 2 describes stress incontinence, and option
nurse writes interventions that target the client’s 3 describes urinary retention. The pathophysiology, contributing factors, therapeutic and
nursing interventions for the different types of incontinence differ.

1.‐ involuntary loss of urine without warning or stimulus.


2.‐ loss of urine when coughing or sneezing.
3.‐ inability to empty bladder.
4.‐ inability to stop urine flow long enough to reach the toilet.

1502 A male client who presents to the emergency Correct answer: 3 The symptoms are typical of acute glomerulonephritis. Hematuria and proteinuria are caused The core issue of the question is the ability to identify signs and symptoms of
department with coffee‐colored urine and edema by a damaged glomerular capillary membrane, which allows blood cells and proteins to escape glomerulonephritis and associate it with a common etiology. Use nursing knowledge and
states that he had a bad sore throat a few weeks ago. into the renal filtrate. A urinary tract infection usually manifests with signs of infection the process of elimination to make a selection.
His blood pressure is elevated, and urinalysis shows including fever, malodorous urine, frequency, and urgency. Clients with urinary calculi usually
blood and protein in the urine. The nurse interprets present with renal colic. Prostatitis, or inflammation of the prostate gland, also has presenting
that the client’s clinical picture is consistent with which symptoms similar to a urinary tract infection.
of the following?
1.‐ Urinary tract infection
2.‐ Urinary calculi
3.‐ Acute glomerulonephritis
4.‐ Acute prostatitis
1503 A client in the intensive care unit develops prerenal Correct answer: 3 Prerenal failure is caused by factors such as hypovolemia and decreased cardiac output that The core issue of the question is the ability to identify causes of prerenal failure. Use
failure following surgery. Which of the following affect renal blood flow and perfusion. Urethral obstruction (option 2) can cause postrenal nursing knowledge and the process of elimination to make a selection.
causes should the nurse suspect? failure. Vascular disease and glomerulonephritis may be factors in the development of
intrarenal failure.
1.‐ Vascular disease
2.‐ Urethral obstruction
3.‐ Hypovolemia
4.‐ Glomerulonephritis

1504 Which of the following discharge instructions would Correct answer: 2 To reduce the risk of nephrotoxicity, the client who receives aminoglycoside antibiotics The core issue of the question is the ability to correctly institute client teaching with
the nurse give to a client who will receive should report signs of edema or hypertension and maintain a fluid intake of 2,000 to 2,500 mL knowledge of nephrotoxicity as an adverse effect of aminoglycoside medications. Use
aminoglycoside antibiotics at home? per day. nursing knowledge and the process of elimination to make a selection.

1.‐ Limit fluid intake to 1200 mL daily


2.‐ Report sudden weight gain or puffy eyes
3.‐ Edema is a normal side effect of the medication
4.‐ Elevated blood pressure is an expected effect of the medication

1505 The nurse caring for a client undergoing a Correct answer: 4 Hypotension is the most common complication during hemodialysis and is related to several The core issue of the question is the ability to identify important complications associated
hemodialysis procedure places high priority on factors, including changes in serum osmolality and rapid removal of fluid from the with hemodialysis. Use nursing knowledge and the process of elimination to make a
evaluating the client frequently for what common intravascular compartment. Dialysis dementia is a progressive, long‐term complication. selection.
complication during the treatment? Infection and fever should be an ongoing assessment for a hemodialysis client. Hyperglycemia
could occur because of the composition of the dialysate, but it is not of great concern unless
the client has diabetes mellitus.
1.‐ Hyperglycemia
2.‐ Infection and fever
3.‐ Dialysis dementia
4.‐ Hypotension

1506 The nurse is explaining the process of peritoneal Correct answer: 3 The peritoneum acts as a semipermeable membrane, allowing substances to move from an The core issue of the question is the ability to relate accurately the key elements of
dialysis to a client who recently developed renal area of high concentration (the blood) to an area of lower concentration (the dialysate). peritoneal dialysis. Use nursing knowledge and the process of elimination to make a
failure. Which of the following statements would the Metabolic waste products and excess water can be eliminated through osmosis and diffusion selection.
nurse include in a discussion with the client? utilizing the peritoneum as the semipermeable membrane.

1.‐ “The solutes in the dialysate will enter the bloodstream through the peritoneum.”
2.‐ “The peritoneum is more permeable because of the presence of excess metabolites.”
3.‐ “The peritoneum acts as a semipermeable membrane through which wastes move by diffusion and osmosis.”
4.‐ “The metabolites will diffuse from the interstitial space to the bloodstream mainly through diffusion and ultrafiltration.”

1507 Which of the following laboratory data is the most Correct answer: 2 Creatinine is the metabolic end product of creatine phosphate and is excreted via the kidneys The critical words in the question are most accurate. This tells you that more than one
accurate indicator that a client with acute renal failure in relatively constant amounts. BUN, a measurement of the nitrogen portion of urea, is also response is technically correct, and you must prioritize to choose the option that best
has met the expected outcomes? excreted in urine and is a good indicator of renal function. However, conditions that increase answers the question. Use nursing knowledge and the process of elimination to make a
protein catabolism also cause a rise in BUN levels. Therefore, the serum creatinine levels are selection.
more appropriate to evaluate in determining the return of renal function. Neutrophils and
lymphocytes are not used to monitor the return of renal function.

1.‐ Decreasing blood urea nitrogen (BUN) levels


2.‐ Decreasing serum creatinine
3.‐ Decreasing neutrophil count
4.‐ Decreasing lymphocyte count
1508 Which of the following statements made by a client Correct answer: 3 Adult polycystic kidney disease is an autosomal‐dominant disorder, and the client should be The core issue of the question is knowledge that this disorder has a genetic basis. Use
with polycystic kidney disease indicates that the advised to have family members screened for the disease. The management of clients with nursing knowledge and the process of elimination to make a selection.
desired outcome has been met? polycystic kidney disease is mainly supportive. Eventually, clients with this disease require
dialysis or transplantation.
1.‐ “I know these drugs will make the cysts disappear.”
2.‐ “The development of renal failure with this disease is very rare.”
3.‐ “I will have my family seek genetic counseling and screening.”
4.‐ “I sure am glad that hemodialysis will shrink the cysts.”

1509 A client is scheduled for a partial nephrectomy. In Correct answer: 4 The upper abdominal incision site in clients with nephrectomy predisposes them to the The core issue of the question is the ability to correlate location of incision with risks for
teaching the client about postoperative care, the nurse development of respiratory complications, particularly atelectasis and pneumonia. The postoperative complications following nephrectomy. Use nursing knowledge and the
uses which rationale to explain why aggressive proximity of the incision to the muscles involved in breathing and coughing makes the client process of elimination to make a selection.
measures are needed to prevent atelectasis and breathe shallowly and avoid coughing because of the fear of pain. Adequate pain control is
pneumonia? necessary in the care of this client. The other options are not accurate statements.

1.‐ Nephrectomy involves paralyzing the intercostal muscles.


2.‐ Intraoperative surgical contamination of the pulmonary structures is unavoidable.
3.‐ The client must be maintained in a flat position for 24 hours.
4.‐ The surgery involves an upper abdominal incision.

1510 Which of the following statements made by a client Correct answer: 2 Clients with renal transplant need to be on long‐term immunosuppressive drugs. This The core issue of the question is the knowledge that clients who have had organ
who has received a renal transplant indicates that the predisposes them to infection. The client must verbalize factors that potentially expose him to transplant are greatly at risk for infection because of drug therapy needed to prevent organ
desired outcome of the discharge teaching plan has infection. Dietary restrictions must be discussed with the physician and the dietician. The client rejection. Use nursing knowledge and the process of elimination to make a selection.
been met? with renal transplant also needs to verbalize understanding of his medications to prevent
rejection, including the use of immunosuppressants. However, he must adhere to the dose
prescribed by the physician. The success of transplantation is not guaranteed.

1.‐ “I will double my prednisone dose if my urine output is less than 300 mL/day.”
2.‐ “I will need to avoid crowds and prevent infection.”
3.‐ “Now I can eat whatever I want as long as I watch how much salt I use.”
4.‐ “Since I have not yet rejected the transplant, I never have to worry about rejection anymore.”

1511 Which of the following statements by a female client Correct answer: 2 Maintaining an intake of 8 to 10 glasses of fluid daily will help prevent UTI. Cotton underpants The core issue of the question is knowledge of risk factors for UTIs that must be avoided
indicates that instruction in ways to prevent urinary are best, and nylon should be avoided because synthetic fibers dry and irritate the perineal by clients at risk. Use nursing knowledge and the process of elimination to make a selection.
tract infection (UTI) was understood? area. Irritation of the perineal area can promote the growth of bacteria. The client should not
delay voiding when the urge is felt. Emptying the bladder every 2 to 4 hours while awake is
recommended to prevent urinary stasis.
1.‐ “I should limit intake of water so I won’t need to urinate so often.”
2.‐ “I should drink 8 to 10 glasses of fluid per day.”
3.‐ “I should only wear nylon underpants.”
4.‐ “I should void every 6 hours while I am awake.”

1512 A client with chronic renal failure asks the nurse why Correct answer: 3 Anemia is common in clients with renal failure. Among the factors causing the anemia are The core issue of the question is the pathophysiology underlying renal failure and
he is anemic. Which of the following responses by the decreased production of erythropoietin by the kidneys and shortened RBC life. Erythropoietin associated changes. Use nursing knowledge and the process of elimination to make a
nurse is best? is involved in the stimulation of the bone marrow to produce RBCs. selection.

1.‐ “The increased metabolic waste products in your body depress the bone marrow.”
2.‐ “We will need to review your dietary intake of iron‐rich foods.”
3.‐ “There is a decreased production by the kidneys of the hormone erythropoietin.”
4.‐ “It is most likely that you have hereditary traits for the development of anemia.”
1513 A client with end‐stage renal failure is to be admitted Correct answer: 3 Clients with potassium levels of 6.5 and greater are predisposed to develop cardiac The core issue of the question is knowledge of the significance of a high serum potassium
to the hospital because of shortness of breath. The arrhythmias, muscle cramps, and gastrointestinal symptoms. The client should be admitted to level and the appropriate placement of the client to detect possible complications. Use
serum potassium level is 7.0 mEq/L. What appropriate a nursing unit with telemetry or cardiac monitoring capabilities because of the risk of nursing knowledge and the process of elimination to make a selection.
hospital unit should this client be admitted to? developing life‐threatening cardiac dysrhythmias. Typical ECG abnormalities associated with
hyperkalemia are prolonged PR interval; wide QRS; tall, tented T‐wave; and ST segment
depression. Major cardiac dysrhythmias common in clients with highly elevated potassium
levels include heart block, ventricular standstill, and ventricular fibrillation.

1.‐ A semiprivate room in a medical surgical unit


2.‐ A private room in a medical surgical unit
3.‐ A nursing unit with continuous electrocardiographic monitoring
4.‐ A nursing unit for ventilator‐assisted clients

1514 A client with chronic renal failure has fluid volume Correct answer: 3 Clients with renal failure retain sodium, and any decrease in the serum level will most likely The core issue of the question is the ability to make an accurate interpretation of
excess. The laboratory report indicates the sodium be caused by hemodilution from the excessive fluid retention. A sodium level of 112 mEq/L is laboratory data in a client with renal failure. Use nursing knowledge and the process of
level to be 120 mEq/L. The nurse interprets this as lower than normal. elimination to make a selection.
which of the following?
1.‐ An elevated sodium level that must be reported immediately to the physician
2.‐ An error in the laboratory analysis
3.‐ A possible hemodilution effect secondary to excessive water retention
4.‐ An expected electrolyte abnormality in clients with chronic renal failure

1515 A child has been admitted to the unit with nephrotic Correct answer: 2 Typical symptoms of nephrotic syndrome are clear, frothy urine that is diminished in volume. The core issue of the question is knowledge of the similarities and differences between
syndrome. In talking with the mother, she reports that AGN presents with smoky urine that is also diminished in volume. AGN is a postinfectious nephrotic syndrome and glomerulonephritis. Use nursing knowledge and the process of
a cousin had acute glomerulonephritis (AGN) last year. disease with no genetic basis. Antibiotics are not used in nephrotic syndrome. Oliguria is elimination to make a selection.
The mother asks how these two diseases compare, as usually defined as 0.5 to 1.0 mL/kg/hr.
they both affect the kidneys. The nurse’s response
would include the information that

1.‐ both diseases produce smoky colored urine.


2.‐ both diseases cause greatly reduced urine output.
3.‐ both diseases have a genetic basis.
4.‐ treatment for both involves antibiotic therapy.

1516 A child is being treated for nephrotic syndrome. The Correct answer: 4 Nephrotic syndrome involves the loss of protein in the urine. Gamma globulins, which help The core issue of the question is the ability to relate gamma globulin deficiency in
nurse has told the mother that it is important to keep the body fight infections, are proteins. Skin that is not clean and dry is more prone to nephrotic syndrome to situations that increase risk of infection, such as unclean or moist
the child’s skin clean and dry. The mother asks why. breakdown, which could lead to infection. The child is oliguric and therefore does not urinate skin. Use nursing knowledge and the process of elimination to make a selection.
The nurse’s response is based on the knowledge that frequently. The only restrictions on the child’s intake are fluid and perhaps sodium. There is no
electrolyte deficiency.
1.‐ the skin is fragile secondary to electrolyte deficiency.
2.‐ frequent urination may leave moisture on the skin that predisposes breakdown.
3.‐ dietary restrictions make fighting infection hard.
4.‐ the condition causes a reduction of gamma globulin in the body.

1517 In a child with acute renal failure, the nurse would Correct answer: 2 Potatoes, tomatoes, and oranges have a high level of potassium content. The others have less The core issue of the question is knowledge of foods that are high in potassium to avoid in
help to prevent hyperkalemia by limiting which of the potassium in them. the client with renal failure. Use nursing knowledge and the process of elimination to make
following foods in the child’s diet? a selection.
1.‐ Grains, cheese, and citrus fruits
2.‐ Potatoes, tomatoes, and oranges
3.‐ Cereals, processed sugars, and wheat
4.‐ Rice, leafy green vegetables, and carbonated beverages
1518 A child has been admitted with acute Correct answer: 1 An elevated ASO titer indicates a recent streptococcal infection, which is a precursor to AGN. The critical words in the question are most indicative. This tells you that all options are
glomerulonephritis (AGN). All of the following tests are The elevated ESR indicates inflammation in the body and is associated with many diseases. correct, and you must select the response that uniquely identifies glomerulonephritis as the
positive for AGN. The nurse concludes that which Hematuria is simply blood in the urine, which has many possible causes. Creatinine disorder. Use nursing knowledge and the process of elimination to make a selection.
laboratory test is most conclusive of this disease? concentrations reflect the functioning of the kidney.

1.‐ Elevated antistreptinolysin O (ASO) titers


2.‐ Elevated erythrocyte sedimentation rate (ESR)
3.‐ Presence of hematuria according to urinalysis
4.‐ Elevated creatinine concentrations

1519 The mother of a child at the renal clinic asks why a Correct answer: 1 Radiological evaluations done after a documented UTI in children reveal structural The core issue of the question is knowledge that UTIs are uncommon in children and could
radiological evaluation is performed on all children abnormalities in 1% to 2% of girls and 10% of boys. Radiological tests cannot confirm bacterial result from structural abnormalities that are yet undiagnosed. With this in mind, use the
who have had one documented UTI. The best colonies, determine the site of an old infection, or help predict whether infection will reoccur. process of elimination to make a selection from the available options.
explanation by the nurse will include the information
that the x‐ray
1.‐ rules out structural abnormalities.
2.‐ confirms the absence of bacterial colonies after antimicrobial therapy.
3.‐ determines which kidney was infected.
4.‐ determines the probability of the infection recurring.

1520 The nurse is caring for a client with poor urine output. Correct answer: 30 The minimal urine output by the kidneys per hour is 30 mL. It is prudent for the nurse to The core issue of the question is knowledge of minimal hourly urine output based on
The nurse would report to the health care provider if report a drop below this amount if it persists for 2 hours or longer so that corrective treatment normal kidney function. Use nursing knowledge to formulate an answer.
the client had a urine output less than _____ milliliters can be undertaken.
per hour for 2 consecutive hours. Write in a numerical
answer.

1521 A 4‐year‐old child has been diagnosed with renal Correct answer: 2 Dietary intake is often inadequate in children with renal failure related to anorexia and Recognize that a child in renal failure will have problems excreting wastes. The only option
failure. The nurse would ensure that the diet for this dietary restrictions. Calories and nutrition are needed to optimize growth and to prevent that does not provide excessive volume or ingredients is option 2.
child would contain: growth retardation. Depending on the degree of renal failure, sodium, potassium and
phosphorus may be restricted. Fluids are monitored closely for balance and may be restricted
if oliguria is present.
1.‐ Foods high in potassium and sodium.
2.‐ Adequate calories to optimize growth.
3.‐ Foods high in calcium content to promote bone growth.
4.‐ Increased fluid intake to flush the urinary system.

1522 The priority concern for the nurse in assessing a child Correct answer: 1 The kidney normally excretes potassium. Hyperkalemia occurs with decreased kidney function Consider which of the electrolyte imbalances occur with renal failure. In renal failure, the
with acute renal failure (ARF) should be to look for resulting in cardiac arrhythmias, which can be life‐threatening. normal imbalances are hyperkalemia, hyponatremia (due to excessive fluid retention),
which electrolyte imbalance? hypocalcemia, and hyperphosphatemia.
1.‐ Hyperkalemia
2.‐ Hypernatremia
3.‐ Hypercalcemia
4.‐ Hypophosphatemia

1523 The nurse would formulate which of the following as Correct answer: 3, 5 Peritoneal dialysis is an invasive procedure that places the child at risk for infection. Consider typical symptoms with renal failure. There would not be fluid volume deficit and
an appropriate nursing diagnosis for a child receiving Hypervolemia is secondary to poor kidney function and does not cause altered renal tissue renal tissue perfusion would not be related to hypervolemia. That leaves two choices to
peritoneal dialysis? Select all that apply. perfusion. The child is anorexic and the child is not at risk for fluid volume deficit. The child’s choose between. Use knowledge of the disease to make a final selection.
condition is chronic and routine health maintenance will need to be integrated with chronic
disease management.
1.‐ Deficient fluid volume related to sodium and water retention
2.‐ Imbalanced nutrition, greater than body requirements related to increased hunger
3.‐ Risk for infection related to invasive procedures and diminished immune functioning
4.‐ Impaired renal tissue perfusion related to hypervolemia
5.‐ Ineffective health maintenance related to chronic condition

1524 A child is admitted to the nursing unit with acute Correct answer: 3 Dehydration results in hypovolemia, which can precipitate acute renal failure in infants and Consider which of the diseases directly relates to the kidneys. Also note that the options
renal failure (ARF). When reviewing the nursing children. The other responses are incorrect because they don't directly impact renal perfusion. contain opposites which usually indicates one is the right answer.
history, the nurse notes a history of all of the following
diseases. The nurse concludes that which most likely
precipitated the onset of ARF?
1.‐ Chickenpox
2.‐ Influenza
3.‐ Dehydration
4.‐ Hypervolemia

1525 A child has been admitted in renal failure. The nurse Correct answer: 3, 5 Azotemia and oliguria are characteristics associated with renal failure in children. The BUN Renal failure prevents the elimination of waste products from the body so the nurse would
would expect to find which of the following laboratory would be elevated. Renal failure is characterized by inadequate glomerular filtration. expect excesses in most lab values.
values? Select all that apply.
1.‐ Decreased BUN
2.‐ Adequate glomerular filtration
3.‐ Oliguria

4.‐ Polyuria
5.‐ Azotemia

1526 The nurse is planning care for a child who has been Correct answer: 3 Although there is fluid retention as a result of oliguria, this does not hamper the renal tissue The core concept of nephrotic syndrome is protein loss leading to loss of osmotic
diagnosed with nephritic syndrome. Which of the perfusion. The weight gain is a result of fluid retention not overeating. The child will be pressure. Using this fact, eliminate the wrong options.
following would be the most appropriate nursing lethargic but not in acute pain.
diagnosis for this child?
1.‐ Ineffective tissue perfusion (renal) related to fluid retention.
2.‐ Imbalanced nutrition, more than body requirements related to excessive weight gain.
3.‐ Risk for impaired skin integrity related to edema and lowered resistance to infection.
4.‐ Pain related to decreased kidney function

1527 An infant has been admitted for treatment of Correct answer: 2 This condition is repaired surgically. Preoperative teaching can relieve parental anxiety Recall the medical treatment for hypospadias to help eliminate the incorrect options.
hypospadias. The nurse would include which of the regarding body image disturbances. The other options are unrelated to care needed for the Knowledge of the care of the child and parents regarding anxiety and body image
following in the plan for nursing management of the client with hypospadias. disturbances will also help in choosing the correct option.
child and family?
1.‐ Parent education regarding steroid therapy.
2.‐ Addressing parental anxiety related to functioning and appearance of the penis.
3.‐ Home health teaching of proper straight catheterization techniques.
4.‐ Monitoring for signs and symptoms of nephrotic syndrome.

1528 Shortly after birth, the newborn was found to have Correct answer: 3 The prepuce or foreskin may be needed in the reconstruction of the penis. The mother can The proper response is likely to be specific to the congenital defect so this helps to narrow
epispadias. Prior to delivery, the mother had signed sign permission for her son without needing the father’s permission as long as she is fully alert. the options to 3 and 4. Knowledge of the care of the child who will need a surgical repair for
permission for a circumcision. The obstetrician is The risk of infection is no greater for this child than for others. epispadias will help to choose the correct answer.
planning to perform the circumcision immediately
after delivery. The nurse explains to the mother that
the circumcision will not be done at this time because:

1.‐ The mother should not have signed consent before delivery.
2.‐ The father also needs to sign permission for surgery.
3.‐ The prepuce will be used for the surgical correction.
4.‐ The risk of infection is greater with a child with epispadias.

1529 A baby is born six weeks prematurely. On admission Correct answer: 2 Premature males are often born with undescended testicles. The testes normally descend Be aware that the testicles have nothing to do with urination so options 1 and 4 can be
to the nursery, the nurse is unable to locate any during the last few weeks of gestation or shortly after birth. This would not be a concern at eliminated. Knowledge of cryptorchidism and the usual management will aid in choosing
testicles in the scrotum. The nurse should: this time. If surgery should be needed, it will be done prior to age 2. Undescended testicles do the correct answer.
not affect urine formation.
1.‐ Immediately notify the physician as the child is at risk for renal failure.
2.‐ Note the findings in the child's record and take no further action at this time.
3.‐ Discuss with the father the need for surgical correction of cryptorchidism.
4.‐ Catheterize the child to determine if urine is present in the bladder.

1530 The mother of an infant who underwent surgery to Correct answer: 1 A double‐diapering technique will help to protect a urinary stent following repair of Knowledge of the rationale for the double diapering technique will aid in choosing the
repair hypospadias asks the nurse why the infant is hypospadias or epispadias. The inner diaper collects the infant's stool, while the outer one correct answer. Options 2 and 3 are virtually the same, leaving only options 1 and 4 to
double‐diapered. The nurse would respond that this collects urine. choose between.
method of diapering will help to:

1.‐ Protect the urinary stent that has been put in place.
2.‐ Adequately measure the urinary output.
3.‐ Provide for maximum absorption of urine.
4.‐ Provide optimal protection of perineal skin from infected urine.

1531 The nurse would place highest priority on which Correct answer: 3 Urinary tract infections are ascending in nature; an untreated UTI can lead to acute With any infectious disease, it is important that antibiotics be administered on schedule.
nursing activity in managing a young child diagnosed pyelonephritis with resulting kidney scarring and damage. Early diagnosis and prompt
with urinary tract infection (UTI)? antimicrobial therapy will prevent or minimize permanent renal damage.
1.‐ Provide adequate nutrition to prevent dehydration.
2.‐ Prevent enuresis.
3.‐ Administer ordered antibiotics on schedule.
4.‐ Restrict fluids to provide kidney rest.

1532 When reviewing a urinalysis report of a client with Correct answer: 3 Proteinuria (presence of protein in urine) is a prime manifestation of acute Eliminate those tests that are blood exams. That leaves proteinuria and specific gravity.
acute glomerulonephritis, the nurse expects to note glomerulonephritis. The other options are inconsistent with this diagnosis. Both findings would be abnormal, but specific gravity would not be specific for
which of the following? glomerulonephritis.
1.‐ Decreased creatinine clearance
2.‐ Decreased specific gravity
3.‐ Proteinuria
4.‐ Decreased erythrocyte sedimentation rate (ESR)

1533 While a child is receiving prednisone (Deltasone) for Correct answer: 1 Prednisone is a synthetic corticosteroid that depresses the immune response and increases The core concept is side effects of the drug.
treatment of nephrotic syndrome, the nurse susceptibility to infection. Steroids mask infection; therefore, the child must be assessed for
determines that it is important to assess the child for: subtle signs and symptoms of illness.

1.‐ Infection.
2.‐ Urinary retention.
3.‐ Easy bruising.
4.‐ Hypoglycemia.

1534 The nurse is admitting a 12‐year‐old girl to the Correct answer: 4 Specimens collected utilizing proper technique will minimize contamination of the urine Eliminate any response that would adversely affect the results, such as forcing fluids
hospital prior to surgery. The physician has ordered a sample ensuring accurate urinalysis results. It is unnecessary to force fluids prior to specimen before collection (dilutes the urine) and cleansing the specimen container with Betadine.
urinalysis. In order to obtain accurate urinalysis data, collection. The specimen container is not cleansed, although the urinary meatus is. The Recall that all specimens should be delivered to the lab as soon as possible—if the
the nurse should: specimen should be sent to the lab immediately after collection to prevent urine degradation. specimen needs to be cooled, it can be done in the lab.
1.‐ Encourage fluids to 1000 mL prior to specimen collection.
2.‐ Cleanse the specimen container with povidone‐iodine (Betadine) prior to collecting the specimen.
3.‐ Allow the urine to cool to room temperature before taking it to the lab.
4.‐ Provide client/parent education for specimen collection before the specimen is obtained.

1535 The parents of a child diagnosed with upper urinary Correct answer: 3 With infectious or inflammatory processes of the upper urinary tract, the kidneys’ ability to Two of the options deal with fluids. Determine if one of these is the right response. Then
tract infection (UTI) ask the nurse why the child needs filter and reabsorb salt and water is altered, resulting in edema. Weights can be an easy and review the other options to ensure the right response.
a daily weight. In formulating a response, the nurse effective measure to determine fluid loads.
includes that it is important because a daily weight
will:
1.‐ Determine if the child’s caloric intake is adequate.
2.‐ Indicate the need for dietary restrictions of sodium and potassium.
3.‐ Keep track of possible loss or gain of fluid retained in body tissues.
4.‐ Track the amount of fluid ingested orally each day.

1536 A child has been diagnosed with acute renal failure Correct answer: 2 Gentamicin is an aminoglycoside antibiotic that is nephrotoxic. Nephrotoxic drugs should be The core concept is which medication has side effects that would be detrimental to kidney
secondary to an infectious organism. The nurse would avoided in a child with acute renal failure. The other options do not represent drug groups that function.
question the medical order for: are particularly nephrotoxic.
1.‐ Aqueous penicillin.
2.‐ Gentamicin (Garamycin).
3.‐ Antihypertensives.
4.‐ Corticosteroids.

1537 The newborn has been diagnosed with Correct answer: 4 HCG is given to induce the descent of testes if testes have not descended during the first year The medication is not an antibiotic or an antipyretic, so options 1 and 2 can be eliminated.
cryptorchidism. The physician has ordered human of life. The other reasons listed are incorrect rationales. Medications do not prevent cancer. This leaves only the correct response.
chorionic gonadotropin (HCG) to be administered to
the baby. The mother asks the nurse why the baby is
receiving this drug. The nurse’s best explanation would
be that the drug will:

1.‐ Maintain an adequate temperature around the testes.


2.‐ Prevent infections in the undescended testes.
3.‐ Prevent the development of cancer.
4.‐ Promote descent of the testes.

1538 The nurse admits children with the following diseases Correct answer: 3, 5 Nephrotic syndrome is an inflammatory reaction in the kidneys. Urinary tract infections and Eliminate those conditions that are not associated with kidney disease.
to the unit. The nurse determines that the children obstructions are also associated with the development of acute renal failure. The other
with which diseases are at risk for the development of diseases pose minimal risk of developing acute renal failure.
acute renal failure (ARF)? (Select all that apply.)

1.‐ Leukemia
2.‐ Cryptorchidism
3.‐ Nephrotic syndrome
4.‐ Phenylketonuria
5.‐ Urinary tract infection

1539 A child has recurrent nephrotic syndrome. The Correct answer: 4 The parents must understand the need for compliance with medical orders to promote the Consider which response would be inappropriate for the child.
mother reports to the nurse that she is overwhelmed child’s health. Relaxation should be accomplished without harming the child.
with the care of her child. After the nurse discusses
options with the mother, which statement by the
mother indicates continued coping difficulties?
1.‐ “I joined a support group like you suggested. I hope it does some good.”
2.‐ “I’m going to ask my mother‐in‐law to come on a regular basis to allow me an afternoon out.”
3.‐ “My husband has agreed to help me manage my son’s medication.”
4.‐ “We’re going to skip his dietary restrictions one day a week to allow us both some relaxation.”

1540 A child returning to the unit after an intravenous Correct answer: 4 The additional fluids will increase urinary output, causing greater urine volume and more Consider the testing methods to determine which is the correct response. Knowledge that
pyelogram (IVP) has an order to drink extra fluids. frequent voiding, thus flushing the dye from the urinary system. The other options do not the test uses a dye to visualize the kidney’s collection system and that the dye needs to be
When the mother asks the purpose of these fluids, the describe the correct rationale for this intervention. excreted will help to choose the correct answer.
nurse responds that increased fluid intake will:

1.‐ Overhydrate the child.


2.‐ Increase serum creatinine levels.
3.‐ Make up for fluid losses from NPO status before tests.
4.‐ Flush any remaining dye from the urinary tract.

1541 A newborn is found to have exstrophy of the bladder. Correct answer: 2, 5 Epispadias and bilateral inguinal hernias are frequent anomalies associated with exstrophy of Knowledge of commonly associated defects with exstrophy of the bladder will help to
The nurse should evaluate the infant for: (Select all the bladder. The other conditions listed are not. choose the correct answers.
that apply.)
1.‐ Hypospadias.
2.‐ Epispadias.
3.‐ Cryptorchidism.
4.‐ Acute tubular necrosis.
5.‐ Bilateral inguinal hernias.

1542 A child has been admitted to the hospital with a Correct answer: 2 Edema is the major clinical symptom of nephrosis. The child may gain twice his or her normal Nephrosis is a urinary condition, so look first for the options related to urinary function.
diagnosis of “rule out nephrosis.” The nurse would weight in severe cases. That would eliminate the rash, leaving hematuria, dehydration, and edema. Body fluid is
assess the child for: involved in dehydration and edema, which are opposites. If the kidneys aren’t working
effectively, there will be problems eliminating fluid.
1.‐ Hematuria.
2.‐ Edema.
3.‐ Petechial rash.
4.‐ Dehydration.

1543 The nurse is caring for a toddler who is not toilet‐ Correct answer: 3 Diapers are weighed on a gram scale before using them and after removal (1 g = 1 mL). The Notice that options 2 and 3 are very similar. Determine which of the two would provide
trained. The doctor has ordered intake and output weight of the dry diaper is then subtracted from the weight of the wet diaper to determine the most accurate information.
measurement. The nurse will most accurately measure urine output.
the urine by:
1.‐ Estimating output as small, moderate, or large and recording on the child’s chart.
2.‐ Weighing each wet diaper and recording the weight of the diaper as the amount of urine output.
3.‐ Subtracting the weight of a dry diaper from a wet diaper and recording this amount.
4.‐ Determining urine output by the number of diaper changes in each 24‐hour period.

1544 The nurse is teaching the parents of a preschooler Correct answer: 1, 2, 4 Bubble baths are irritating to the meatus and increase the incidence of urinary tract The core concept in this item is that it is a little girl with a urinary tract infection. Girls are
information about urinary tract infections (UTIs) and infections. An acidic urine is desirable in preventing urinary tract infections, and the proper more at risk for UTIs from ascending organisms due to the shorter urethra.
means of reducing their recurrence. Statements from way to wipe is front to back.
the parents that indicate an understanding of ways to
prevent UTIs include: (Select all that apply.)

1.‐ “I should try to get her to drink a lot of water and juices.”
2.‐ "I will buy her underwear made with cotton."
3.‐ “Soaking in a bubble bath will reduce meatal irritation.”
4.‐ “If I notice her starting to wet the bed again, I need to have her checked for another urinary tract infection.”
5.‐ “I should avoid giving her cranberry juice as it has been shown to make the urine more acidic.”

1545 The nurse would include which of the following in the Correct answer: 2, 3, 4 Although children with acute glomerulonephritis may feel well, they are confined to bed until With glomerulonephritis, there is damage to the glomerular capillary causing the loss of
care of a child with acute glomerulonephritis? (Select hematuria resolves. This can lead to boredom, making it important for the nurse to provide red blood cells through the urine, decreased urine output, and hypertension. Consider any
all that apply.) activities that are fun for the child to help pass the time. interventions related to these symptoms.
1.‐ Careful handling of edematous extremities
2.‐ Observing the child for evidence of hypertension
3.‐ Providing fun activities for the child on bed rest
4.‐ Monitoring the urine for hematuria
5.‐ Encouraging fluid intake

1546 A urinalysis is ordered for a child with a throat culture Correct answer: 4 Urinalysis allows for early diagnosis and treatment of acute glomerulonephritis, which is a Remember to associate strep infections with the common complications of rheumatic
positive for group‐A beta‐hemolytic streptococcus serious complication that can follow group‐A beta‐hemolytic streptococcal infection. fever and glomerulonephritis.
(strep throat). The mother asks why this test is being
ordered. The nurse explains:

1.‐ The urinalysis will indicate whether an HIV infection is also present.
2.‐ Urinary tract infections are common with streptococcal infections and need to receive prompt treatment.
3.‐ Pyelonephritis is a potential complication of antibiotic therapy.
4.‐ Group‐A beta‐hemolytic streptococcus infections can be followed by the complication of acute glomerulonephritis.

1547 An appropriate nursing diagnosis for a toddler with Correct answer: 4 The open bladder allows bacteria to enter the urinary system, and urinary tract infections are Exstrophy of the bladder is a congenital defect where the bladder does not have an
unrepaired exstrophy of the bladder would be: common. At this age, sexual dysfunction would not be an appropriate diagnosis. The unformed anterior wall so cannot collect urine. With the absence of the abdominal skin, there is no
bladder does not hold urine, so urinary retention would not be an appropriate diagnosis. prevention of infections.
Disorganized behavior doesn’t apply.
1.‐ Disorganized infant behavior.
2.‐ Sexual dysfunction.
3.‐ Urinary retention.
4.‐ Risk for infection.

1548 A child has been admitted to the unit with acute Correct answer: 1 The ASO titer indicates a preceding infection with group A beta‐hemolytic streptococcus. The Knowing the cause of AGN, the learner will select the option that would test for the
glomerulonephritis. The test that would confirm this urinalysis would show hematuria, but this alone would not be diagnostic of acute causative agent.
diagnosis is: glomerulonephritis. Blood cultures may be negative as the infection preceded the illness by 1
to 3 weeks.
1.‐ Antistreptolysin‐O (ASO) titer.
2.‐ Urinalysis.
3.‐ Blood cultures.
4.‐ White blood cell (WBC) count.

1549 The doctor orders a clean‐catch urine specimen on an Correct answer: 1 Clean‐catch urine specimens are not reliable urine samples; therefore, catheterization is Determine whether the specimen should be sterile or clean, then select the option that
infant who is not toilet‐trained. The best means of necessary. The urine does need to be obtained at the time of voiding. would provide an uncontaminated specimen.
collecting this urine would be to:
1.‐ Perform a straight catheterization.
2.‐ Apply a urine collection bag.
3.‐ Use diaper analysis.
4.‐ Perform Foley catheterization.

1550 A 14‐year‐old is being treated for renal failure. The Correct answer: 3 With the inability to secrete urine, electrolytes will build up in the blood, including sodium In renal failure, the kidneys have difficulty excreting waste products. Therefore, the diet
nurse would ensure that the child follows a: and potassium. The child should be on a low‐sodium, low‐potassium diet with restricted fluids will reduce the amount of substances that are hard to clear.
and proteins.
1.‐ High‐sodium diet.
2.‐ High‐protein diet.
3.‐ Low‐sodium diet.
4.‐ Low‐fiber diet.

1551 A 25‐year‐old college student is diagnosed with an Correct answer: 2 Pyelonephritis is an upper urinary tract infection, involving the kidney tissue. Lower urinary Omit options 1, 2, and 4 as being lower urinary structures.
upper urinary tract infection. Which of the following tract infections include urethritis, prostatitis, and cystitis. The most common upper urinary
would be documented on the medical record? tract infection is pyelonephritis.

1.‐ Cystitis
2.‐ Pyelonephritis
3.‐ Urethritis
4.‐ Prostatitis

1552 The nurse would expect to see which pathogen on the Correct answer: 3 E. coli is the infective organism in over 90 percent of first‐time infections. The nurse should E. coli inhabits the bowel and due to the proximity with the GU tract is often the causative
urine culture and sensitivity (C &amp; S) of a client check that the organism is sensitive to the antibiotic or notify the healthcare professional. organism in UTI.
with a urinary tract infection?
1.‐ <i>Streptococcus</i>
2.‐ <i>Staphylococcus</i>
3.‐ <i>E. coli</i>
4.‐ <i>Klebsiella</i>

1553 A urinary tract infection would be considered Correct answer: 4 Due to the anatomic structure of the male urethra and bacteriostatic effect of prostatic fluid, Identify the male as the client in which a diagnosis of UTI is unusual.
complicated in which of the following clients? all urinary tract infections in the male client should be considered complicated.

1.‐ Teenager who has recently become sexually active


2.‐ A child, age 2
3.‐ An elderly, bedridden client
4.‐ A male client

1554 The client with cystitis has a routine urinalysis (UA) Correct answer: 3 Urine may have a foul odor and appear cloudy because of mucus and excess white cells Omit option 1 as the term for blood in the urine is hematuria. Casts and protein in the
done. Pyuria is noted on the report, which means the present, which is common in cystitis. Casts and proteins are never normal in urine. urine would be noted by their names.
urine has:
1.‐ Blood.
2.‐ Casts.
3.‐ Excess mucus and white blood cells.
4.‐ Protein.

1555 When assessing the client with urinary calculi, an Correct answer: 2 The greatest risk factor for stone formation is a prior personal or family history of urinary Recall that there is thought to be a genetic link to incidence of renal stones.
important subjective finding would be: calculi. The other answers are important information to know but do not contribute greatly as
risk factors.
1.‐ Allergies to sulfonamides.
2.‐ Father of client who has had urinary stones twice.
3.‐ Last urinary tract infection 2 months ago.
4.‐ Alcohol intake of two six‐packs of beer per day.

1556 The presence of red blood cell casts is most likely Correct answer: 4 Red blood cell casts are not present in the normal urinalysis but are present in glomerular Recognize that casts are abnormal and would not be present in infection or failure.
caused by which disorder? diseases.
1.‐ Urinary tract infection
2.‐ Pyelonephritis
3.‐ Renal failure
4.‐ Glomerulonephritis
1557 Which of the following is an iatrogenic cause of acute Correct answer: 3 Iatrogenic causes result from treatment from a physician or other care provider. Examples Recall that the term ‘iatrogenic’ is often used to describe untoward medication effects.
renal failure? include nephrotoxic medications, radiologic contrast dye, and shock after surgery.

1.‐ Alcohol
2.‐ Diet
3.‐ Nephrotoxic medications
4.‐ Exercise

1558 Glomerular filtration rate is less than 30 percent of Correct answer: 3 In end stage renal failure, 90 percent or more of the nephrons are destroyed; glomerular Recall that GFR reduces in renal failure.
normal in which of the following? filtration rate is &amp;lt; 20 percent normal with increased creatinine and BUN.

1.‐ Urinary tract infection


2.‐ Kidney cancer
3.‐ Acute renal failure
4.‐ Polycystic kidney disease

1559 What laboratory test is a common measure of renal Correct answer: 2 The blood urea nitrogen is primarily used as an indicator of kidney function because most Associate creatinine with renal function.
function? renal diseases interfere with its excretion and cause blood levels to rise. Creatinine is produced
in relatively constant amounts, according to the amount of muscle mass and is excreted
entirely by the kidneys making it a good indicator of renal function.

1.‐ Complete blood count


2.‐ Blood urea nitrogen/creatinine
3.‐ Glucose
4.‐ Alanini aminotransferase (ALT)

1560 In the second stage of chronic renal failure, what Correct answer: 4 In the second stage of chronic renal failure, renal insufficiency, there is at least 75 percent of Recall that renal failure is insidious, so that by the time it is discovered a great deal of
percent of renal tissue is destroyed? functional renal parenchyma destroyed. function is lost. Select the response that reflects the largest loss of function.

1.‐ 25%
2.‐ 30%
3.‐ 50%
4.‐ 75%

1561 The nurse would question a client with balanitis about Correct answer: 2 Balanitis, or inflammation of the foreskin and prepuce, would cause edema and pain of the Omit option 1 as this could not occur in a female. Use the process of elimination to work
complaints of which of the following? penile glans, leading to dysuria. Option 1 is inappropriate for balanitis; a urethral discharge through the other options.
(option 3) may occur in gonorrhea; back pain (option 4) could indicate many diseases, but not
balanitis.
1.‐ Vaginal discharge
2.‐ Pain with urination
3.‐ Spontaneous urethral discharge
4.‐ Back pain

1562 The nurse would assess the client experiencing Correct answer: 4 Prostatitis creates pain in the tissues surrounding the prostate gland. Option 1 indicates a Think of the location of the prostate and identify likely symptoms.
prostatitis for which of the following symptoms? gonococcal infection; option 2 herpes virus; and option 3 syphilis (secondary stage).

1.‐ Spontaneous penile discharge, dysuria, and pain with ejaculation


2.‐ Painful blisters and crater‐like lesions, enlarged groin nodes, fever
3.‐ Brownish rash on palms, painful crater‐like lesions, malaise, and fever
4.‐ Rectal pain, pain with erection, low abdominal pain, and low back pain
1563 A client reports to the clinic with complaints that a Correct answer: 1 Secondary syphilis begins with the healing of the chancre, and ends when the rash disappears, Recall that rash on the palms is the symptom associated with syphilis.
sexual partner from last year has been diagnosed with which can take up to 6 months from time of infection. The latent stage of syphilis then starts,
syphilis. The nurse would expect the client to have: which can last for years. VDRL and RPR would need to be positive for syphilis (option 2); option
3 could indicate TB or HIV; option 4 is signs and symptoms of nongonococcal infections in
females.
1.‐ Had a painless sore that healed and a rash on the palms of his hands.
2.‐ Negative VDRL, negative rapid plasma reagin (RPR), and positive fluorescent treponemal antibody absorption (FTA‐ABS).
3.‐ Night sweats, cough, low‐grade fever, and elevated white count.
4.‐ Vaginal discharge, dysuria, and pain with orgasm.

1564 The nurse concludes that teaching has been effective Correct answer: 3 Hypospadias repair is undertaken using the foreskin to create a channel through the penis to Recall that fertility is the major concern with hypospadias.
when the mother of an infant born with hypospadias the tip of the glans so that he will deposit his sperm near his partner's cervix. Although option
says, "Our son will:” 1 may also be an appropriate answer, it is not the best answer to demonstrate effective
teaching. Options 2 and 4 are incorrect.
1.‐ “Need surgical correction so that he looks like the other boys."
2.‐ “Require circumcision now to prevent complications later."
3.‐ “Have surgical correction so he will be fertile as an adult."
4.‐ “Likely have other reproductive tract anomalies we can't see."

1565 Which medication would the nurse expect the client Correct answer: 4 DUB most commonly results from a progesterone deficiency that causes a fragile Recognize that options 1 and 4 do not relate to the condition and can be eliminated.
with dysfunctional uterine bleeding (DUB) be endometrium that fails to mature from proliferative stage to secretory. This causes irregular
prescribed? menstrual bleeding. Treatment is aimed at correcting the cause, thus progesterone
supplementation is prescribed.
1.‐ Testosterone
2.‐ Estrogen
3.‐ Steroids
4.‐ Progesterone

1566 In preparing to discharge a client with gonococcal Correct answer: 1 Douching should be avoided in order to prevent bacteria present in the lower reproductive Recall that douching is a practice that is usually discouraged.
pelvic inflammatory disease (PID), the nurse would tract from being forced upwards into the uterus, potentially causing PID.
intervene if the client made which statement?

1.‐ "I should douche after every episode of intercourse."


2.‐ "Using condoms will decrease the risk of this happening again.”
3.‐ "My boyfriend and I should be monogamous."
4.‐ "The sexual position I use won't prevent this infection."

1567 The 23‐year‐old client has been diagnosed with Correct answer: 3 Chemotherapy and radiation used in the treatment of testicular cancer often cause a radically Focus on preservation of fertility.
testicular cancer. Which of the following should be decreased sperm count. If the client desires children, he should consider sperm banking prior
included in his teaching plan? to beginning treatment.
1.‐ Future fertility is not affected by treatment.
2.‐ Impotence often results from needed treatments.
3.‐ Sperm banking should be done prior to treatment.
4.‐ Sexual interest will increase as a result of treatment.

1568 The client with fibrocystic breast disease is most likely Correct answer: 4 Caffeine can precipitate or worsen fibrocystic breast disease. No other dietary factors have Caffeine content of foods is a concern in fibrocystic disease.
to report a diet high in which of the following? been identified.

1.‐ Bacon, sausage, ground meat


2.‐ Fresh fruits and grain cereal
3.‐ Cheese and milk
4.‐ Coffee and cola

1569 A male client has been diagnosed with Chlamydia Correct answer: 1 Doxycycline (Vibra‐tabs) is a commonly utilized treatment for chlamydia infections, and like Omit options 2 and 3 as having information that is inconsistent with sound health
trachomatis infection. The plan of care should include all antibiotics must be taken until the medication is gone. Use of condoms with every sexual practices. Completion of antimicrobial therapy is the focus of treatment.
which of the following? encounter decreases the transmission of sexually transmitted diseases. Sexual contacts should
be notified of the infection so that appropriate testing can be obtained. This is especially
important with Chlamydia because it is so often asymptomatic in women, and early detection
can prevent complications such as pelvic inflammatory disease. Testicular self‐exam is
screening for testicular cancer, not diagnosing.

1.‐ Instructions to take all of the doxycycline (Vibra‐tabs) that was ordered.
2.‐ Encouragement to use condoms with most episodes of intercourse.
3.‐ Obtaining the names of sexual contacts if client desires.
4.‐ Teaching of testicular self‐exam (TSE) for diagnosis.

1570 The client has a diagnosis of primary dysmenorrhea. Correct answer: 3 Primary dysmenorrhea begins at menarche and is usually a lifelong condition. Options 1, 2, Recall that the prefix 'dys' indicates pain.
Which of the following would likely appear in her and 4 can occur with secondary dysmenorrhea or endometriosis.
history?
1.‐ Heavy flow and clots for at least 3 months
2.‐ Irregular menses with breakthrough bleeding
3.‐ Painful periods since menarche
4.‐ No periods for the last 7 months

1571 A urinalysis is ordered for a client with urinary calculi. Correct answer: 4 Hematuria, either gross or microscopic, is generally present in clients with urinary calculi. Remember that mechanical irritation of the calculi causes bleeding.
Which of the following is usually positive on the Leukocytes and protein may be common with other urinary disorders.
urinalysis report?
1.‐ Leukocytes
2.‐ Nitrite
3.‐ Protein
4.‐ Blood

1572 A 40‐year‐old male is at high risk for which of the Correct answer: 3 Adult polycystic kidney disease is an autosomal dominant disorder. In children, it is caused by Omit options 1, 2, and 3 as not being genetically linked.
following, if his dad also had the problem? an autosomal recessive trait.
1.‐ Glomerulonephritis
2.‐ Renal failure
3.‐ Polycystic kidney disease
4.‐ Urinary tract infection

1573 The client has a urinalysis return showing a high white Correct answer: 1 Acute pyelonephritis is a bacterial infection of the kidney. Chronic pyelonephritis is associated Associate an increased white cell count with an acute infectious process.
blood count. Further tests pinpoint the origin in the with nonbacterial infections and noninfectious processes that may be metabolic, chemical or
kidney. The client most likely has: immunological.
1.‐ Acute pyelonephritis.
2.‐ Renal failure.
3.‐ Chronic pyelonephritis.
4.‐ Calculi.

1574 It is important to teach clients with chronic Correct answer: 1 Hypertension may develop as renal tissue is destroyed. Recall that kidney function plays a key role in the control of blood pressure.
pyelonephritis to monitor:
1.‐ Blood pressure.
2.‐ Temperature.
3.‐ Diet.
4.‐ Stress level.

1575 Acute glomerulonephritis is usually caused by which Correct answer: 1 Infection of the pharynx or skin with group‐A beta Hemolytic‐streptococcus is the common Associate glomerulonephritis with being preceded by strep throat.
pathogen? precipitating factor for acute glomerulonephritis.
1.‐ <i>Group‐A beta Hemolytic‐streptococcus</i>
2.‐ <i>Staphylococcus</i>
3.‐ <i>E. coli</i>
4.‐ <i>Proteus</i>

1576 Which of the following clients is at highest risk for a Correct answer: 1 Risk factors of urinary tract infections include female, older clients, urinary obstruction or Recall that the elderly are generally at higher risk for most infections.
urinary tract infection (UTI)? calculi, strictures, chronic disease, prostatic hypertrophy and prostatitis, diaphragm use,
instrumentation and impaired immune system. Although children and teenagers can contract a
UTI, incontinence and disease conditions in the elderly make them a higher risk population.

1.‐ Elderly
2.‐ Male adult
3.‐ Teenager
4.‐ Child

1577 In rapidly progressive glomerulonephritis, crescent‐ Correct answer: 1 Glomerular cells proliferate along with macrophages to form crescent‐shaped lesions Eliminate option 2 as not being a part of the kidney. Use the process of elimination to
shaped lesions form and obliterate: obliterating Bowman's space resulting in a rapid decline in glomerular filtration rate (GFR), work through the other answers.
which leads to many of the complications.
1.‐ Bowman's space.
2.‐ Urinary bladder.
3.‐ Renal tubules.
4.‐ Loop of Henle.

1578 Sally is diagnosed with an uncomplicated urinary tract Correct answer: 2 Sulfonamides are considered to be the drug of choice for acute, recurrent, or chronic urinary Look for the sulfa drug as the first line treatment of UTI.
infection. The first‐line therapy is: tract infections when there is no evidence of obstruction or bacteremia.

1.‐ Ciprofloxacin (Cipro).


2.‐ Trimethoprim‐sulfamethoxazole (Bactrim).
3.‐ Amoxicillin (Amoxil).
4.‐ Erythromycin (Erythrocin).

1579 Which vitamin can help to maintain acidic urine and Correct answer: 3 Vitamin C helps to maintain a pH of 5 or less thereby inhibiting bacterial growth. The other Recall that Vitamin C is associated with acidic food like oranges.
may help a client with recurrent urinary tract vitamins have no proven use in preventing urinary tract infections.
infections?
1.‐ E
2.‐ B
3.‐ C
4.‐ D

1580 Nursing management of the client with a urinary tract Correct answer: 3 Caffeine and alcohol can increase bladder spasms and mucosal irritation, thus increase the Eliminate options 1, 2, and 3 as they are contrary to the treatment plan for UTI of
infection should include: signs and symptoms of a urinary tract infection (UTI). Fluids should be taken, and douches will increasing fluid and taking prescription medications until the course of treatment is
not help a UTI. All antibiotics should be taken completely to prevent resistant strains of complete. Douching is rarely a recommended treatment.
organisms.
1.‐ Taking medication until feeling better.
2.‐ Restricting fluids.
3.‐ Decreasing caffeine drinks and alcohol.
4.‐ Douching daily.

1581 When triaging in the clinic, which client should be Correct answer: 1 The primary genital herpes infection involves systemic viremia, and encephalitis is a possible Select the option with a severe symptom such as a neurological symptom.
seen first? The client with: complication. Headache and stiff neck may indicate encephalitis, and require further
investigation.
1.‐ Genital herpes infection diagnosed yesterday, with a severe headache.
2.‐ Recurrent herpes infections for 3 years, with burning during urination.
3.‐ Chlamydia diagnosed yesterday, now with worsening pelvic pain.
4.‐ Secondary syphilis diagnosed last month, due for penicillin injection.

1582 Which of the following statements should be Correct answer: 4 Breast cancer detection begins with monthly self‐breast exams. Mammograms should be Eliminate options 1, 2, and 3 as being inconsistent with current recommendations
presented in a breast cancer detection program for the performed yearly after age 40. Birth control pills do not increase nor decrease breast cancer regarding breast cancer.
community? risk, but the longer a woman is on estrogen replacement therapy the greater her risk for
developing the disease.
1.‐ Mammograms should be started when women become age 50.
2.‐ Self‐breast exams should be performed weekly.
3.‐ Birth control pills may prevent breast cancer.
4.‐ Estrogen replacement may increase breast cancer risk.

1583 The client with benign prostatic hyperplasia has Correct answer: 1 Continuous bladder irrigation serves to flush out the blood that will be oozing from the raw Select the response that addresses clotting.
undergone transurethral resection of the prostate edges of the TURP site before the blood can clot. Clots in the bladder would obstruct the urine
(TURP) and is asking why he needs continuous bladder flow through the catheter.
irrigation (CBI). The nurse's best response would be:

1.‐ "The irrigation prevents blood from clotting and blocking the catheter."
2.‐ "Your bladder needs to be kept full to promote healing after this surgery."
3.‐ "The urine would be very concentrated without the irrigation."
4.‐ "The saline running through the bladder helps keep you hydrated."

1584 Teaching for the client with endometriosis Correct answer: 4 Lupron is a synthetic analog of luteinizing hormone‐releasing hormone, and acts as an Recall that hot flashes are associated with hormonal therapies.
undergoing leuprolide (Lupron) therapy is effective estrogen antagonist, causing the endometriosis deposits to shrink in size, thus decreasing the
when she states: pain and infertility associated with endometriosis. Hot flashes and night sweats are common
side effects while on the medication, and resolve upon discontinuation of the medication.

1.‐ "I may have cold hands and feet while taking this medication."
2.‐ "It's possible that I'll be heat intolerant while taking these shots."
3.‐ "My voice may become higher during this therapy."
4.‐ "Hot flashes are likely while I am taking these shots."

1585 Which client is at the highest risk for developing Correct answer: 3 Balanitis, inflammation of the foreskin, occurs due to poor hygiene, and occurs after the Recall that balanitis only occurs in uncircumcised clients.
balanitis? foreskin becomes retractable (at about age 3). Options 2 and 4 are incorrect because of the
circumcision.
1.‐ 1‐year‐old with intact foreskin
2.‐ Circumcised 40‐year‐old
3.‐ 12‐year‐old with intact foreskin
4.‐ Circumcised 6‐year‐old
1586 Which client is at lowest risk for developing breast Correct answer: 3 Early childbearing with breastfeeding for a total of 2 years or more decreases a woman's Eliminate the options that put an individual at higher risk, such as genetic link, prior cancer
cancer? lifetime risk of developing breast cancer. BRCA1 or BRCA2 gene mutations increase risk. diagnosis, and increased age.
Hodgkin's disease treatment usually involves chest radiation, which increases breast cancer
risk and breast cancer mortality. Aging is another factor: The older a woman becomes, the
more likely she is to develop breast cancer.
1.‐ Client with BRCA1 gene mutation.
2.‐ Client who had Hodgkin's disease
3.‐ First child at age 18, breastfed for 2 years
4.‐ Smoker, age 74

1587 Female clients with Chlamydia trachomatis infections Correct answer: 4 Chlamydia, although often silent and asymptomatic, will eventually present symptoms Recall that chlamydia does not have lesions or rash.
will most likely present with which signs and including new occurrence of dyspareunia, dysmenorrhea, and low abdominal and pelvic pain,
symptoms? with yellow or yellow‐green vaginal discharge.
1.‐ Painful perineal blisters and high fever of sudden onset
2.‐ Painless crater‐like lesion on the labia that lasts 6 weeks
3.‐ Rapidly progressing pruritic rash on labia and buttocks
4.‐ Yellow‐green vaginal discharge, dyspareunia, pelvic pain

1588 The client with a new diagnosis of genital herpes Correct answer: 3 HSV1 does not survive more than a few minutes on inanimate objects, and although it is often Recall that HSV1 is known as a sexually transmitted disease.
simplex virus 1 (HSV1) wants to know how she a genital tract infection it does occur orally. Sexual contact is the most likely method of
contracted the infection. The nurse's best answer is transmission, and persons with intact immune systems can easily be infected.
based on which of the following?
1.‐ Inanimate objects can harbor HSV1 for several hours.
2.‐ HSV1 is found only in the genital tract and not orally.
3.‐ Sexual contact is the most common mode of transmission.
4.‐ Immune system suppression is needed to contract the infection.

1589 The client with benign prostatic hyperplasia (BPH) Correct answer: 2 BPH clients may be asymptomatic until large alcohol intake, which relaxes the bladder Eliminate options 3 and 4 as not having a relationship to bladder function.
may experience a sudden onset of urinary retention sphincter, making it impossible to empty the bladder.
after doing which of the following?
1.‐ Drinking several cups of coffee in the morning
2.‐ Having a weekend of binge drinking
3.‐ Starting a multivitamin supplement
4.‐ Walking farther than he usually does

1590 The nurse would assess the client with multiple Correct answer: 1 Subserosal uterine myomas are located on the outer surface of the uterus and tend to cause Recognize that uterine enlargement is a key symptom.
subserosal uterine myomas for which of the following? fewer menstrual disorders than submucosal or intramural myomas. However, they do cause
mechanical pressure on the pelvic contents from their size and weight, including bladder
pressure that results in urinary frequency and urgency.
1.‐ Uterine enlargement, urinary frequency, and pelvic pressure
2.‐ Uterine enlargement, pain with urination, and vaginal discharge
3.‐ Uterine atrophy, heavy and painful menses, and decreased libido
4.‐ Uterine atrophy, pain with intercourse, and vaginal dryness

1591 An 80‐year‐old female is brought to the office by her Correct answer: 3 Older clients may not exhibit the classic symptoms of UTI but present with nonspecific Recall that confusion in an elderly client is often associated with UTI.
daughter with decreased appetite and confusion. The complaints such as nocturia, incontinence, confusion, behavior change, lethargy, anorexia, or
daughter has noticed these complaints for the past 3 just not feeling right. Urinalysis would be a first‐line diagnostic test.
days and wonders what could be the reason. The nurse
knows that these complaints may be symptoms of:

1.‐ Dementia.
2.‐ Cerebral vascular accident (CVA).
3.‐ Urinary tract infections (UTI).
4.‐ Constipation.

1592 Mr. Smith is admitted with hemorrhage caused by a Correct answer: 2 Pre‐renal causes of acute renal failure include those affecting renal blood flow and perfusion. Recall that pre‐renal indicates a cause that comes before the kidney in terms of perfusion
gastrointestinal bleed and begins to experience acute Hemorrhage, an extracellular fluid loss, can lead to renal ischemia due to decreased renal and blood flow.
renal failure (ARF). Which category of ARF is caused by perfusion, decreased glomerular filtration rate and azotemia.
a hemorrhage?
1.‐ Post‐renal
2.‐ Pre‐renal
3.‐ Intrarenal
4.‐ Inter‐renal

1593 Which of the clients below would most likely Correct answer: 3 The most common cause of urologic obstruction is urolithiasis, stones within the urinary Recall that colicky pain is common with stones which can obstruct the urinary tract.
experience a urinary obstruction? A client with: tract. The symptoms in option 3 are the typical signs and symptoms of stones. Option 1 is
associated with bladder cancer, option 2 with UTI, and option 3 with glomerulonephritis.

1.‐ Painless hematuria.


2.‐ Complaints of burning, frequency, and difficulty in urination.
3.‐ Complaints of severe colicky pain, nausea, pallor, and clammy skin.
4.‐ Brown‐tinged urine, edema, hypertension, and fatigue.

1594 A client has a routine physical and the urinalysis Correct answer: 1 Painless hematuria is the presenting sign in 75 percent of urinary tract tumors, along with Recognize that options 2, 3, and 4 are more common with infection and presence of
returns with microscopic hematuria. Which other flank pain and an abdominal mass. Hematuria may be gross or microscopic. bacteria in the urine.
symptoms would likely be present in a client with a
urinary tract tumor?
1.‐ Flank pain
2.‐ Burning on urination
3.‐ Infection
4.‐ Polyuria

1595 A Caucasian male is more prone to develop which of Correct answer: 2 Males are more likely to develop nephrolithiasis than females. Option 1 would be more Without any other information, recall that males more commonly suffer from kidney
the following? common in females, option 3 in African‐Americans, and option 4 does not apply. stones.

1.‐ Urinary tract infection


2.‐ Nephrolithiasis
3.‐ Acute renal failure
4.‐ Polycystic kidney disease

1596 A client experiencing urinary calculi asks the nurse Correct answer: 4 The majority of kidney stones are comprised of calcium oxalate or calcium phosphate. Select the answer that reflects the most common stone component, calcium.
what makes a stone in the body. The nurse's best Sturvite stones (15 to 20 percent), uric acid (5 to 10 percent) and cystine stones are
answer is that the majority of kidney stones consist of: uncommon.

1.‐ Uric acid.


2.‐ Cystine.
3.‐ Sturvite.
4.‐ Calcium.

1597 The client with a uric acid kidney stone requires which Correct answer: 1 A low‐purine diet is required for clients with uric acid stones. High‐purine foods such as Recall that purine metabolizes into uric acid
type of diet? sardines and organ meats are eliminated. Moderate‐level purine foods such as red and white
meats and some seafoods are limited. Proteins are restricted in renal failure.
1.‐ Low purine
2.‐ Low protein
3.‐ High protein
4.‐ High purine

1598 A client experiencing oliguria would probably have Correct answer: 2 Oliguria is a urine output of less than 400 mL/24 hrs. It may result in decreased glomerular Recall that oliguria is defined as urine output of less than 400 mL/24 hours.
which of the following urinary outputs in a 24‐hour filtration rate. Option 3 is of concern and should be monitored closely.
period:
1.‐ 750 mL/24 hrs.
2.‐ 400 mL/24 hrs.
3.‐ 600 mL/24 hrs.
4.‐ 1,000 mL/24 hrs.

1599 A disruption in the renin‐angiotensin system in the Correct answer: 4 The decreased glomerular filtration rate caused by the inflammation of the glomerular Recall that renin affects blood pressure.
client with glomerulonephritis can cause: membrane causes activation of the renin angiotensin‐aldosterone system and can lead to
hypertension. Hypertension can also be caused from fluid retention and disruption of the renin‐
angiotensin system, a key regulator of blood pressure. Option 1 is common with urinary tract
infections, option 2 with kidney stones, and option 3 with polycycstic kidney disease.

1.‐ A temperature of 101 degrees F, positive urine culture.


2.‐ Severe colicky pain with nausea.
3.‐ Proteinuria, polyuria, and/or nocturia.
4.‐ Blood pressure of 190/98.

1600 A client enters the clinic complaining of cocoa‐ or Correct answer: 1 The urine of acute glomerulonephritis is often cocoa‐ or coffee‐colored. Hematuria is Recall that coffee‐colored urine is associated with glomerulonephritis.
coffee‐colored urine. This would most likely indicate associated with the other disorders and may be gross or microscopic.
which disorder?
1.‐ Glomerulonephritis
2.‐ Kidney stone
3.‐ Urinary tract infection
4.‐ Bladder cancer

1601 A client is taking tobramycin sulfate (Tobrex) for a Correct answer: 4 This question draws on your knowledge of pharmacology as well as acute renal failure. Omit option 3 as this disorder has a genetic disorder. Associate drug reactions with an
severe infection and could possibly experience which Tobramycin can be nephrotoxic to the kidneys and a blood urea nitrogen/creatinine should be acute response.
of the following: monitored during administration, especially if high doses are given. Even in a client with
normal, healthy kidneys, the nephrotoxic effects can occur. This question draws on your
knowledge of pharmacology as well as ARF.

1.‐ Glomerulonephritis.
2.‐ Pyelonephritis.
3.‐ Polycystic kidney disease.
4.‐ Acute renal failure.

1602 What is the classic triad of symptoms indicating renal Correct answer: 2 The classic triad of symptoms, gross: hematuria, flank pain, and a palpable abdominal mass Eliminate options 1, 3, and 4 as they all focus on fever which is more common with
tumors? occur in only about 10 percent of people with renal cell carcinoma. Painless hematuria is the inflammatory response than malignancy.
most consistent symptom.
1.‐ Fever, weight loss, oliguria
2.‐ Gross hematuria, flank pain, a palpable abdominal mass
3.‐ Dysuria, pain, fever
4.‐ Fever, pain, dysuria
1603 Which diagnostic test usually provides the first Correct answer: 4 Renal ultrasound often provides the first diagnostic evidence of a kidney tumor. It is Eliminate options 1 and 3 as these are preliminary tests and will not provide specific
evidence of a kidney tumor? particularly beneficial in differentiating cystic kidney disease from renal neoplasms. CT is used enough information.
to provide information about the tumor.
1.‐ Intravenous pyelography (IVP)
2.‐ Computed tomography (CT)
3.‐ Kidney, ureter, bladder (KUB)
4.‐ Renal ultrasound

1604 The treatment of choice for nonmetastatic tumors of Correct answer: 3 Radical nephrectomy is the treatment of choice for tumors of the kidney. The adrenal gland, Recall that removal of the tumor is the first line of treatment.
the kidney is: perirenal fat, upper ureter, and fascia surrounding the kidney are removed. Radiation and
chemotherapy are treatment options for metastatic kidney tumors. Dialysis is used for end‐
stage renal disease.
1.‐ Radiation.
2.‐ Chemotherapy.
3.‐ Radical nephrectomy.
4.‐ Dialysis.

1605 Benign prostatic hypertrophy (BPH) is the most Correct answer: 4 Obstructive causes of acute renal failure are classified as post renal. BPH is the most frequent Think of the prostate in terms of location, below the kidneys.
common precipitating factor in which type of acute precipitating factor and a form of obstruction. Other causes may include renal and urinary
renal failure? calculi and tumors.
1.‐ Pre‐renal
2.‐ Intrarenal
3.‐ Inter‐renal
4.‐ Post‐renal

1606 Clients in the maintenance phase of acute renal Correct answer: 1 Salt and water retention lead to edema and puts the client at risk for congestive heart failure Inability of the kidney to remove fluids and wastes leads to the correct response.
failure are at risk for heart failure and pulmonary and pulmonary edema. The immune function is impaired, leading to infectious complications.
edema caused by: Increased urine volume occurs at the end of the maintenance phase, diuretic period.
Nephrotoxins are associated with the initiation phase of ischemia.

1.‐ Salt and water retention.


2.‐ Immune function.
3.‐ Increased urine volume.
4.‐ Nephrotoxins.

1607 The most common cause of chronic renal failure is: Correct answer: 4 Conditions causing chronic renal failure typically involve diffuse, bilateral disease of the Recall that diabetes is the leading reason for dialysis due to renal failure.
kidneys leading to progressive destruction and scarring of the nephron. Diabetic nephropathy
causes glomerulosclerosis and thickening of the glomerular basement membrane.

1.‐ Cystic kidney disease.


2.‐ Hypertension.
3.‐ Glomerulonephritis.
4.‐ Diabetic nephropathy.

1608 Which of the following would be expected in the Correct answer: 4 Cardiovascular disease is a common cause of death in clients with end‐stage renal disease Identify option 4 as the condition likely to cause death.
uremic stage of end‐stage renal disease and to resulting from accelerated atherosclerosis. Hypertension, hyperlipidemia, and glucose
contribute to death of the client? intolerance all contribute to the process. All other disorders may be present throughout the
uremic stage but are less likely to cause death.
1.‐ Respiratory infection
2.‐ GI disturbance
3.‐ Neurologic complication
4.‐ Cardiovascular disease
1609 Which of the following would be the most critical Correct answer: 2 Weight is the most critical index of fluid status. Although options 1 and 3 suggest fluid Eliminate options 1 and 4 as expected in most renal failure clients. Weight is a more
index of fluid excess in a client with chronic renal problems, weight is used as a measure for how much fluid is retained. If over hydrated, the significant indicator of fluid balance than edema.
failure? hematocrit would be low.
1.‐ Intake greater than output
2.‐ Weight gain of 6 pounds
3.‐ Edema +1
4.‐ Low hematocrit

1610 Which of the following serum creatinine levels is Correct answer: 3 Normal serum creatinine for an adult female is 0.5 to 1.1 mg/dL and 0.5 to 1.2 mg/dL for an Based on the question, select the highest value.
considered high and is expected in a client with chronic adult male. Levels greater than 4.0 mg/dL indicate serious impairment of renal function.
renal failure? Although options 2 and 4 are high, with chronic renal failure, levels greater than 4.0 should be
expected.
1.‐ 0.8 mg/dL
2.‐ 1.5 mg/dL
3.‐ 4.5 mg/dL
4.‐ 3.0 mg/dL

1611 Which of the client phone calls to the urology clinic Correct answer: 2 Urinary retention may result when a client with benign prostatic hypertrophy ingests large Recognize that inability to void for 24 hours is an emergent situation.
should be returned first? amounts of alcohol or takes a medication with B‐sympathomimetic side effects. The bladder
sphincter becomes relaxed and does not open, and the bladder is unable to generate enough
force to get urine past the enlarged prostate gland.
1.‐ 28‐year‐old man, with burning upon urination, greenish‐yellow penile discharge
2.‐ 68‐year‐old man, complaining of inability to void since taking allergy medication yesterday
3.‐ 45‐year‐old man, with weak stream of urine, worsening over last 6 months
4.‐ 52‐year‐old man, with foul‐smelling dark‐colored urine for past 2 days

1612 Which client should the home health nurse visit first? Correct answer: 3 A modified radical mastectomy requires Jackson‐Pratt drains, which would still be in place 6 Radical mastectomy is a major surgical procedure and in the first week post‐surgery the
days post‐operative, and is therefore at greater risk for infection than the other clients. In client would have significant care needs.
addition, this client faces significant psycho‐social issues related to the cancer diagnosis and
removal of the breast, which will affect her body image. Physiologic and psycho‐social
assessments should be performed.
1.‐ 79‐year‐old male, 2 weeks post‐transurethral resection of the prostate for prostate cancer
2.‐ 15‐year‐old pregnant female, 1‐week for post‐pelvic inflammatory disease treatment
3.‐ 37‐year‐old female, 6 days post‐modified radical mastectomy
4.‐ 23‐year‐old male, 4 days post‐circumcision for phimosis

1613 The client with secondary dysmenorrhea would be Correct answer: 4 Secondary dysmenorrhea develops after menses have become ovulatory and regular, Recall that the term secondary implies after a significant related illness.
most likely to have recently experienced: following menarche. Obstruction is the most common cause and can result from uterine
fibroids or scarring of the uterine cavity.
1.‐ Normal, spontaneous, vaginal delivery of her third child.
2.‐ Severe viral illness with high fever, body aches, and diarrhea.
3.‐ Infertility requiring diagnostic laparoscopic surgery.
4.‐ Chlamydia trachomatis pelvic inflammatory disease.

1614 To help the client with pedunculated uterine fibroids Correct answer: 3 Uterine fibroids or myomas are solid tissue tumors that are not precancerous. Pedunculated The word ‘pedunculated’ in the question should lead to the selection of option 3 that uses
understand her diagnosis, the nurse describes them as: fibroids are on a stalk or stem and can extend either into the uterine cavity or outward into the the word stem to describe the growths.
pelvic cavity.
1.‐ Fluid‐filled sacs on the outside wall of the uterus.
2.‐ Precancerous spots on the inside of the uterus.
3.‐ Solid growths on a stem that attach to the uterus wall.
4.‐ The same as what is shed each month with menstruation.

1615 The pregnant client with gonorrhea wants to know Correct answer: 3 Gonococcal opthalmia neonatorum is the eye infection in newborns caused by Neisseria Recall that transmission occurs by contact during the birth process. This leads to the
why she needs to be treated now before the baby is gonorrhoeae. This infection can cause blindness within a few hours after birth if not treated selection of option 3.
born. The nurse explains that treatment is necessary to with antibiotic eye medication.
avoid the fetus developing:

1.‐ Pneumonia that can cause death


2.‐ Seizures that can cause brain injury.
3.‐ Eye infection that can cause blindness.
4.‐ Heart failure that can create preterm labor.

1616 Which statement indicates that the teaching for the Correct answer: 1 Anti‐viral medications like acyclovir (Zovirax) and valcyclovir (Valtrex) are used to treat Notice that the use of the word ‘recurrence’ in option 1 is a hint to the correct response.
client with genital herpes simplex virus 1 (HSV1) primary infections and recurrences and will shorten the duration of the outbreak. As
infection has been effective? suppressive therapy, they help prevent recurrences. No cure exists for HSV1 infections; the
virus will live on the nerve root until the next outbreak.
1.‐ "When I feel a recurrence starting, I'll begin taking my medication."
2.‐ "When I am finished with this medication, I'll be cured of the infection."
3.‐ "The medication will help me feel better but doesn't affect the infection."
4.‐ "I'm guaranteed I'll never have another infection if I take my medication."

1617 The client with prostate cancer has had an abdominal‐ Correct answer: 1 Radiation treatments cause nausea and vomiting; fluid status should be assessed to Eliminate option 3 as being unrelated to the condition described. Eliminate option 4 as
perineal resection and is now undergoing radiation determine if the client is dehydrated. physical problems will take priority over psychosocial. Radiation would more likely cause
treatments. Which of the following is the most problems with hydration than temperature and blood pressure.
important action for the nurse to take?

1.‐ Pinch the skin on the back of the hand gently.


2.‐ Take the temperature and blood pressure.
3.‐ Check the urine for the presence of protein and glucose.
4.‐ Determine how the client is coping with cancer.

1618 The client with phimosis will experience difficulty Correct answer: 3 Phimosis is a tight foreskin that is unable to be retracted by an age when retraction should Eliminate options 2 and 4 as unrelated to the condition.
with: take place.
1.‐ Replacing the retracted foreskin.
2.‐ Complete bladder emptying.
3.‐ Retracting the foreskin.
4.‐ Maintaining an erection.

1619 The client is scheduled for a transurethral resection of Correct answer: 1 Benign prostatic hyperplasia (BPH) is seen in elderly men in all ethnic groups and races and is Eliminate option 2 as the condition is benign. Options 3 and 4 are incorrect since diet and
the prostate (TURP) to treat his benign prostatic thought to result from decreasing testosterone levels that begin in middle‐age. BPH is neither bowel activity are not contributory to the condition.
hypertrophy (BPH) and asks the nurse how his prostate cancerous nor precancerous.
became enlarged. The best response is:

1.‐ "Prostate enlargement happens to most men as they age and their hormones change."
2.‐ "Your prostate has become cancerous, which is really quite a rare occurrence."
3.‐ "Because of your diet, your prostate gland has quit working, and became larger."
4.‐ "Your chronic constipation put excessive force on the prostate, and it enlarged."

1620 The client has had a benign breast cystectomy. The Correct answer: 2 Caffeine and smoking both increase the incidence of benign cysts of the breast. Breast cysts Select the option that identifies a positive behavior change to reduce risk.
nurse knows that discharge teaching has been (fibrocystic breast disease) are neither a precursor to nor a risk factor for developing breast
effective when the client states: cancer. The small incision will usually not affect lactation.
1.‐ "I'll need to wear an underwire bra from now on."
2.‐ "I should quit smoking and drinking coffee."
3.‐ "My risk for cancer is much higher after this cyst."
4.‐ "I won't be able to breast feed if I have children."

1621 The client has undergone a lumpectomy for breast Correct answer: 1 Small tumors that are localized are often treated with lumpectomy. Recurrence rates and Eliminate options 2 and 4 as they are non‐therapeutic responses. Recognize that option 1
cancer and asks, "Why didn't the doctor take off my survival rates are not improved in these cases with more radical surgery. is the only true statement.
whole breast?" The nurse's best response is:

1.‐ "Some cancers don't have better outcomes with mastectomy."


2.‐ "Every surgeon I know only does mastectomies. I don't know."
3.‐ "Your cancer had progressed so far that a mastectomy wouldn't help."
4.‐ "Your doctor would only do what's best. Everything will be just fine."

1622 Which client is at greatest risk for developing Correct answer: 2 Cryptorchidism is the single greatest predictor of testicular cancer, even when the condition The word ‘uncorrected’ in option 2 helps in identification of that as the correct response.
testicular cancer? is corrected in early childhood. Varicocele does not increase the risk of the disease, nor does a
second‐degree relative with the disease. No activity has been associated with testicular cancer.

1.‐ Client who is a bicycle racer


2.‐ Client who has uncorrected cryptorchidism
3.‐ Client who has an uncle with the disease
4.‐ Client who underwent repair of varicocele

1623 Nursing management of the client with Neisseria Correct answer: 4 Condoms used consistently will decrease the transmission of sexually transmitted infections Recall that reduction in transmission is a major goal of care.
gonorrhoeae infection of the cervix includes: and protect sexual partners.

1.‐ Education on the use of birth control pills.


2.‐ Assessment of tampon or sanitary napkin use.
3.‐ Arranging a method of transportation to the hospital.
4.‐ Instructions on the correct use of condoms.

1624 Which client with a sexually transmitted infection is Correct answer: 3 Latent syphilis usually has no symptoms. The latent phase begins when the rash of secondary Note that the word ‘latent’ is the hint to identification of option 3 as correct.
most likely to have an asymptomatic infection? syphilis clears, and may last for decades without further symptoms.

1.‐ Female with cervical gonorrhea


2.‐ Male with urethral Chlamydia
3.‐ Female with latent syphilis
4.‐ Male with primary herpes simplex virus, type 1

1625 Which statement, if made by the client with prostate Correct answer: 2 Giving away personal possessions is a sign of depression and suicidal ideation. Although the Omit options 3 and 4 as being factual assessments of symptoms. Option 1 is eliminated
cancer, requires immediate intervention? diagnosis of cancer may result in depression, indications of suicidal tendencies must be based on the fact that it addresses a common outcome.
addressed. Impotence after prostatectomy is common, and sometimes responds to oral or
injectable medications. Chemotherapy often causes nausea, and medication to control the
nausea should be used. Normal incisional healing involves some itching, and because the
subcutaneous nerves are severed during surgery, numbness around the area may result.

1.‐ "I want the pill that will let me have sex again."
2.‐ "I've made out my will and given away my money."
3.‐ "I get nauseated for 2 days after each chemotherapy."
4.‐ "My incision is itchy, and the skin around it is numb."
1626 The nurse is planning a community presentation on Correct answer: 4 Nerve‐sparing surgical techniques can sometimes be utilized, but impotence and incontinence In a community presentation the common concern of impotence should be addressed.
prostate cancer. The presentation should include: are still common side effects. First‐degree relative with the disease is a strong risk factor for
developing prostate cancer. In elderly men the cancer is slow‐growing, but the younger the
man is at the time of diagnosis the more aggressive the cancer is likely to be.

1.‐ Having a first‐degree relative with the disease does not increase the risk for a patient.
2.‐ Prostate cancers are slow‐growing and rarely progress beyond the gland in men of any age.
3.‐ New surgical techniques do not injure surrounding nerves, so incontinence never occurs.
4.‐ Impotence is a common side effect of abdominal or perineal surgical treatment.

1627 The 34‐year‐old woman client presents with regular, Correct answer: 2 Uterine fibroids or myomas will cause heavy menses with large clots. The presence of Recall that ultrasound would be done before laparoscopy for diagnosis.
heavy menses with clots up to golf‐ball size. She has intramural or submucosal myomas can cause early pregnancy loss or infertility. Ultrasound
had three first‐trimester spontaneous abortions this examination will detect the presence of myomas.
year. The nurse would expect the physician to order
which of the following?

1.‐ Magnetic resonance imaging (MRI)


2.‐ Ultrasound
3.‐ Blood type and Rh
4.‐ Diagnostic laparoscopy

1628 The client has given birth to a stillborn infant in which Correct answer: 3 Congenital syphilis can occur when a mother is in any stage of syphilis infection during the Omit options 1, 2, and 4 as not related to the condition.
a diagnosis of congenital syphilis has been made. The pregnancy, including if she becomes infected during this pregnancy. All sexual partners should
plan of care would include: be treated. Penicillin is the antibiotic used for treating syphilis. Because syphilis is blood‐borne,
gowning and gloving when having contact with the dead infant's skin is all that would be
required. Diagnostic work‐up may include lumbar puncture.

1.‐ Asking if the client is allergic to erythromycin.


2.‐ Utilizing droplet precautions when handling the body.
3.‐ Asking the client for the names of all sexual contacts.
4.‐ Preparing the client for a magnetic resonance imaging (MRI) scan.

1629 The client with endometriosis is most likely to make Correct answer: 2 Endometriosis causes infertility both in the presence of blockage of the fallopian tubes from Recall that endometriosis is a common cause of infertility.
which statement? endometrial implants as well as without blockages for unknown reasons. Pregnancy is often
extremely difficult if not impossible unless medical or surgical intervention is initiated.

1.‐ "I've been alternating between diarrhea and constipation."


2.‐ "I haven't been able to get pregnant in 7 years of trying."
3.‐ "I get urinary tract infections about every other month."
4.‐ "I have green vaginal discharge and pain with intercourse."

1630 Which of the following complaints is most likely to be Correct answer: 4 Herpes simplex virus (HSV) infections of the genital tract are exquisitely painful, start as The description of the herpes lesions as painful is the key to identification of the correct
herpes simplex virus 1 (HSV1)? blisters that become crater‐like lesions, and are often accompanied by enlarged groin nodes. response.

1.‐ Yellow penile discharge


2.‐ Painless crater‐like lesion
3.‐ Dysuria of sudden onset
4.‐ Painful blisters and craters

1631 A client who lives in a heavily industrialized area has a Correct answer: 3 People living in heavily industrialized states experience higher rates of urinary tract cancers Associate industrial pollution with cancer risk.
higher incidence of which renal/urinary disorder? than those living in agricultural states. People living in northern regions have a higher risk than
those living in southern regions.
1.‐ Urinary tract infection
2.‐ Kidney stones
3.‐ Bladder carcinoma
4.‐ Renal failure

1632 If a urine specimen is needed for a client with signs Correct answer: 1 Midstream urine is considered less likely to be contaminated from the external genitalia. Recall that midstream collection is always preferable.
and symptoms of acute renal failure (ARF), the nurse Analysis for disorders as serious as ARF requires a good specimen if not a cath specimen.
should instruct the client to collect a urine specimen:

1.‐ Midstream.
2.‐ At the beginning of urination.
3.‐ At the end of urination.
4.‐ First thing in the morning.

1633 A urinalysis returns with positive proteins, positive Correct answer: 3 Cranberry juice helps the acidity of the urine to inhibit bacterial growth. The juice should not Recall that cranberry juice acidifies urine, thus altering pH.
glucose, specific gravity of 1.010, and pH of 4.5. The account for the presence or absence of glucose or protein. The specific gravity is affected by
nurse knows that cranberry juice can alter which part fluid balance.
of the urinalysis?
1.‐ Protein
2.‐ Specific gravity
3.‐ pH
4.‐ Glucose

1634 Phenazopyridine (Pyridium) is sometimes ordered for Correct answer: 2 Phenazopyridine (Pyridium) is a urinary analgesic that stains the urine/semen orange. Omit options 1, 3, and 4 as incorrect information. Recall that the drug is an analgesic.
the client with a urinary tract infection. Which of the Antibiotics can be used (option 1); options 3 and 4 are incorrect. Fluids should always be
following should be explained to a male client? encouraged.

1.‐ "This is an antibiotic that is commonly used."


2.‐ "This drug may discolor the urine and semen orange."
3.‐ "The drug will cure the urinary tract infection and takes the place of increasing fluids."
4.‐ "This drug is good, but very nephrotoxic."

1635 The nurse is preparing a teaching plan for the client Correct answer: 3 An increase in fluid intake of 2,500 to 3,000 mL/day is a prevention measure for further Remember that fluid restriction is sometimes necessary in CHF.
with a urinary stone. Fluid intake is an important urolithiasis. A client with CHF, however, will probably not tolerate this amount of fluid without
preventive measure and should be between 2,500‐ complications.
3,000 mL/day. Which of the following clients would
have to decrease this measure even though it is
recommended? A client with:

1.‐ Arthritis.
2.‐ Systemic lupus.
3.‐ Congestive heart failure (CHF).
4.‐ Diabetes.

1636 Which of the following diets should be ordered for Correct answer: 1 Restricting dietary protein early in chronic renal failure may slow the disease progression and Associate low protein and calories, primarily from carbohydrates, as the preferred diet in
clients with chronic renal failure? also reduce nausea and vomiting due to anorexia associated with uremia. Protein intake of 0.6 renal failure.
g/kg body weight or approximately 40 g/day is usually adequate. Carbohydrates should be high
to compensate for energy needs.
1.‐ Low protein, high carbohydrates
2.‐ High protein, high carbohydrates
3.‐ High protein, low carbohydrates
4.‐ Low protein, low carbohydrates
1637 The goal of treatment for the client with chronic renal Correct answer: 1 Early management of the client with chronic renal failure focuses on elimination factors that Recognize that option 1 identifies the most important goal, which is to preserve function
failure is to: may further the decrease of renal failure and measures to slow the progression of the disease as long as possible.
to end‐stage renal disease. If conservative treatment fails, dialysis or transplantation is the
treatment option.
1.‐ Maintain present renal function.
2.‐ Prepare the client for dialysis.
3.‐ Prepare for transplantation.
4.‐ Limit involvement and activity to decrease stress.

1638 Because most urinary stones consist of calcium, Correct answer: 3 Limiting Vitamin D inhibits absorption of calcium from the GI tract. Acid‐ash foods promote Recognize that milk and dairy products are high in calcium and often fortified with Vitamin
which of the following should be reduced in a client acidity of the urine, whereas alkaline‐ash foods promote calcium stones. D.
with stones?
1.‐ Low‐purine foods
2.‐ Bananas
3.‐ Vitamin D
4.‐ Acid‐ash foods

1639 Which test is the best measure of renal function? Correct answer: 2 Creatinine is solely indicative of renal function and represents damage to a large number of Creatinine should always be considered as the best measure of renal function.
nephrons. BUN can be affected by the amount of protein in the diet. Bilirubin indicates liver
problems and electrolytes can be altered for many reasons.
1.‐ Creatinine
2.‐ Bilirubin
3.‐ Blood urea nitrogen (BUN)
4.‐ Electrolytes

1640 A common factor that predisposes adolescents and Correct answer: 4 Vesicoureteral reflux is a condition in which urine moves from the bladder back toward the Eliminate options 1, 2, and 3 as not generally causes of infection.
children to developing pyelonephritis is: kidney. Diet, swimming, and circumcision should not cause an infection of the kidneys.

1.‐ Diet.
2.‐ Frequent swimming.
3.‐ Circumcision.
4.‐ Vesicoureteral reflux.

1641 When discharging a client from the hospital after a Correct answer: 4 The indwelling urinary catheter is left in place about 2 weeks after a TURP procedure. The Select the response that provides for sound basic advice in the post‐operative period.
transurethral resection of the prostate (TURP) or urine should progressively become clearer and less pink, and clots are to be reported to the
benign prostatic hyperplasia (BPH), which of the physician immediately. Low abdominal pain of new onset can be a symptom of cystitis, and
following is essential to teach the client? should be reported. Nonsteroidal anti‐inflammatory drugs are to be avoided because of the
potential for increased bleeding. Adequate fluid intake is important to keep the catheter
draining well, but caffeine and alcohol are both bladder irritants and may cause bladder spasm
as well as dehydration.

1.‐ The indwelling catheter will be removed by the physician in about 2 days.
2.‐ The color of the urine may become red and contain small clots.
3.‐ Pain in the lower abdomen is to be expected; take aspirin or ibuprofen (Advil).
4.‐ Drink plenty of fluids, especially water, and avoid caffeine and alcohol.

1642 Which client is at greatest risk for contracting Correct answer: 1 The highest incidence of syphilis infections is among black men in urban areas, particularly in Notice that option 1 states lack of condom use, so this can be identified as the correct
syphilis? the southeastern United States. This client has two risk factors, because syphilis is spread response.
through sexual contact as well as being blood borne.
1.‐ African‐American, urban‐dwelling intravenous drug‐user who does not use condoms
2.‐ Hemophiliac who has had multiple blood transfusions
3.‐ Caucasian suburban teen who has had 4 sexual partners
4.‐ Emergency department healthcare worker in rural setting

1643 The parent of a newborn with epispadias is asking Correct answer: 1 Epispadias is a rare defect that develops very early in fetal development. No known risk Select the response that does not place blame on the parent.
how this happened. The best answer would be: factors have been identified, and parents should be assured it wasn't caused by anything they
did or didn't do.
1.‐ "The defect happens early in fetal development."
2.‐ "The alcohol you drank caused the defect."
3.‐ "Because you smoked, the penis did not form correctly."
4.‐ "You had a deficiency of folic acid that caused this."

1644 The female client with Chlamydia trachomatis is at Correct answer: 2 Chlamydia trachomatis is the most common bacterial cause of pelvic inflammatory disease. Select the response that makes sense in terms of progression of the infection to adjacent
risk for which possible complication? The bacteria ascend from the cervix into the uterus and fallopian tubes, where infection structures.
worsens.
1.‐ Diarrhea with dehydration
2.‐ Pelvic inflammatory disease
3.‐ Rash on the palms and feet
4.‐ Encephalitis

1645 Which client is most likely to be diagnosed with Correct answer: 2 Testicular cancer occurs most often between the ages of 15 and 34, and presents with lump Select the response that describes pain not associated with strenuous activity.
testicular cancer? or thickening of one testicle, pain in the testicle, and lower abdomen, lower back, or rectum.

1.‐ 25‐year‐old with sudden onset of testicular pain that started when he was lifting weights
2.‐ 18‐year‐old with unilateral testicular pain and low abdominal pain that is worsening
3.‐ 40‐year‐old with increasing rectal pain, lower back pain, and low‐grade temperature
4.‐ 50‐year‐old with nocturia, hesitant and weak urinary flow whose father had lung cancer

1646 The client has been diagnosed with neurosyphilis. The Correct answer: 4 Neurosyphilis can occur during any stage of syphilis but most commonly occurs during the Omit options 1, 2, and 3 as syphilis is sexually transmitted.
nurse knows that teaching has been effective when the tertiary stage. Central nervous system involvement causing dementia, paralysis, gradual
client states: blindness, and numbness characterizes neurosyphilis. Syphilis is blood borne and sexually
transmitted, so all sexual contacts should be tested. Treatment is based on lumbar puncture
results and estimated length of time of the syphilis infection.

1.‐ "I must have gotten infected when I ate at a Caribbean restaurant last winter."
2.‐ "My wife doesn't have to worry about being tested because she's postmenopausal."
3.‐ "I'll need to make sure that my grandson doesn't drink from the same cup that I use."
4.‐ "The treatment for this will be ongoing and based on what my lab results show.

1647 The client who may have dysfunctional uterine Correct answer: 4 Bleeding calendars are utilized to quantify the amount and frequency of bleeding. The client Note that option 4 is more descriptive of the bleeding than the other options.
bleeding (DUB) is being instructed to complete a should document each day that she has bleeding, how much bleeding she has, the color of the
bleeding calendar. The client should include: flow, and when she has clots (including the size of the clots.) All information must be obtained
in order to make an accurate diagnosis.
1.‐ Days of bleeding and color of the flow.
2.‐ Amount of flow and temperature.
3.‐ Time bleeding started and clots.
4.‐ Amount of flow and color of blood.

1648 Teaching has been effective when the client Correct answer: 1 Endometriosis is small areas of growing endometrium in the pelvic and/or abdominal cavity, Notice that the phrase ‘outside the uterus’ in option 1 is the key to identifying this as the
diagnosed with endometriosis describes the condition which increase in size during the secretory phase of the menstrual cycle. Because there is correct option.
as: nowhere for the endometrium to be shed, scarring occurs.
1.‐ Collections of endometrium outside the uterus.
2.‐ Endometrium growing among the myometrium.
3.‐ Small, solid tumors within the uterine wall.
4.‐ Tiny pockets of blood throughout the pelvis.

1649 Which of the following would probably be ordered for Correct answer: 4 Primary dysmenorrhea results from the over‐production of prostaglandins by the Omit options 1 and 3 as narcotics are not generally necessary. Propanolol is a beta blocker
the client suffering from primary dysmenorrhea? myometrium. Nonsteroidal anti‐inflammatory (NSAIDs) medications (like Naprosyn) have anti‐ and would not be useful.
prostaglandin activity and thus decrease the dysmenorrhea.
1.‐ Meperidine hydrochloride (Demerol)
2.‐ Propanolol (Inderal)
3.‐ Acetaminophen and codeine phosphate (Tylenol #3)
4.‐ Naproxen (Naprosyn)

1650 The client with benign prostatic hyperplasia will be Correct answer: 4 Ibuprofen is a nonsteroidal anti‐inflammatory drug, and has anti‐platelet aggregation Look for medications that have an influence on bleeding. Recall that NSAIDs are
undergoing a transurethral resection of the prostate in properties. It must be discontinued 10 days prior to surgery to prevent excessive blood loss. discontinued due to bleeding.
2 weeks. Which medication does the client need to
discontinue prior to surgery?
1.‐ Phenytoin (Dilantin)
2.‐ Nystatin powder (Micostatin)
3.‐ Omeprazole (Prilosec)
4.‐ Ibuprofen (Motrin)

1651 The nurse has admitted a client with uremia. The Correct answer: 2 Uremia is a syndrome, or group of symptoms, associated with end‐stage renal disease. The Recognize that options 3 and 4 are treatable and not associated with uremia. Polycystic
nurse plans care for which of the following underlying normal function of the kidney is altered, resulting in various metabolic and systemic effects kidney disease is also not a uremic condition.
disorders? including fluid and electrolyte disturbances. Pyelonephritis (inflammation of the kidney and
renal pelvis) and cystitis (inflammation of the urinary bladder) do not lead to uremia. Polycystic
kidney disease is a hereditary disease characterized by kidney enlargement and cyst formation.

1.‐ Polycystic kidney disease


2.‐ End‐stage renal failure
3.‐ Pyelonephritis
4.‐ Cystitis

1652 A client receiving peritoneal dialysis (PD) has outflow Correct answer: 2 If outflow drainage is less than inflow, the nurse should change the client's position to shift Select a simple non‐invasive strategy for a first attempt.
that is less than the inflow for two consecutive abdominal fluid, and hopefully move the catheter into contact with the fluid in the abdomen.
exchanges. Which of the following actions would be Although vital signs are monitored, the blood pressure is not a concern at this time (option 1).
best for the nurse to take first? The catheter does not need to be irrigated (option 3). A direct nursing intervention is needed,
while continuing to monitor is an assessment and does not correct the current problem
(option 4).
1.‐ Check the client's blood pressure
2.‐ Change the client's position
3.‐ Irrigate the dialysis catheter
4.‐ Continue to monitor the third exchange

1653 The nurse is planning to teach the client with acute Correct answer: 2 A client with glomerulonephritis should eat a diet that is high in calories but low in protein to Recall that protein restriction is associated with kidney diseases.
glomerulonephritis about dietary restrictions. The inhibit protein catabolism, and allow the kidneys to rest by diet (since they have fewer
nurse should include in the plan to make which of the nitrogenous wastes to clear). It is important to protect the kidneys while they are recovering
following dietary changes? their function. The other responses are incorrect.
1.‐ Limit fluid intake to 500 mL per day
2.‐ Restrict protein intake by limiting meats and other high‐protein foods
3.‐ Increase intake of high‐fiber foods, such as bran cereal
4.‐ Increase intake of potassium‐rich foods such as bananas or cantaloupe
1654 A client develops a renal disorder after taking an Correct answer: 3 Acute renal failure is a condition that may be caused by nephrotoxic drugs such as Eliminate options 1 and 2 as they are not caused by nephrotoxic drugs. The situation
antibiotic that has nephrotoxicity as an adverse effect. aminoglycoside antibiotics. Acute renal failure has a rapid onset and is potentially reversible. described is acute, not chronic.
The nurse adds to the client's medical record a The condition usually responds to treatment if diagnosed early. Chronic renal failure develops
standardized care plan for which of the following insidiously and requires dialysis or transplantation.
disorders?
1.‐ Polycystic kidney disease
2.‐ Glomerulonephritis
3.‐ Acute renal failure
4.‐ Chronic renal failure

1655 A client with a chronic urinary tract infection is Correct answer: 3 Serum creatinine measures the amount of creatinine in the blood. Creatinine is the end Look for the option that would best assess renal function, creatinine.
scheduled for a number of laboratory tests. The nurse product of creatine phosphate, used in skeletal muscle contraction. Blood urea nitrogen (BUN),
would review results of which of the following tests to another common laboratory test, measures the nitrogen portion of urea and helps detect
best evaluate whether the kidneys are being adversely dehydration. These tests are often ordered together when assessing renal function.
affected?
1.‐ Serum potassium
2.‐ Urinalysis
3.‐ Serum creatinine
4.‐ Urine culture

1656 A female client with recurrent cystitis has been told Correct answer: 4 An acid‐ash diet lowers urine pH, which may reduce bacterial growth. An acid‐ash diet Recall that dairy foods tend to raise pH.
to follow an acid‐ash diet. The client demonstrates includes the following foods: meat, fish, shellfish, poultry, cheese, eggs, cranberries, prunes,
understanding of diet instruction if she states to avoid plums, corn, lentils, grains, and foods high in chlorine, phosphorus, and sulfur. Foods to be
which of the following foods? avoided include: milk and milk products; all vegetables except corn and lentils; all fruits except
cranberries, plums, and prunes; and foods containing high amounts of sodium, potassium,
calcium, and magnesium.
1.‐ Fish
2.‐ Corn
3.‐ Eggs
4.‐ Milk

1657 The nurse is teaching the client to perform peritoneal Correct answer: 2 Peritonitis is the major complication of PD. The nurse should use strict aseptic technique and The most common complication is peritonitis. Select the option that stresses asepsis.
dialysis (PD). The nurse reviews in detail which of the should teach the client to use it whenever accessing the catheter. The client does not need
following essential actions that will help to prevent the post‐void residuals (option 1). Heparin is added to dialysate as ordered, but it would be added
major complication of peritoneal dialysis? to each bag, not to one bag per day randomly; the catheter site dressing is changed daily
(options 3 and 4).
1.‐ Monitor the client's post‐void residuals
2.‐ Maintain strict aseptic technique during connection and disconnection
3.‐ Add heparin to the dialysate at least once per day
4.‐ Change catheter site dressing twice daily

1658 The nurse would assess a client with kidney stones for Correct answer: 1 Renal colic is an acute, severe pain in the flank and upper abdominal quadrant on the affected Recall that flank pain is the symptom associated with renal colic.
which of the following to best determine whether the side, generally associated with renal calculi that obstruct a ureter. Clients experiencing renal
client is developing renal colic? colic describe it as sudden in onset and may be accompanied by nausea, diaphoresis, and
vomiting.
1.‐ Flank pain
2.‐ Difficult urination
3.‐ Absence of urine
4.‐ Headache
1659 A client seen in the emergency department complains Correct answer: 2 Painful urination, frequency, and urgency are common signs of cystitis, or bladder infection. Omit options 3 and 4 as the symptoms are inconsistent with these conditions.
of painful urination, frequency, and urgency. Which of In addition, the urine may have a foul odor and appear cloudy. Bacteria, virus, parasites, or
the following conditions would the nurse suspect? fungi may cause the condition, with GI tract bacteria being the most common cause.

1.‐ Renal calculi


2.‐ Cystitis
3.‐ Glomerulonephritis
4.‐ Polycystic kidney disease

1660 Which of the following is the priority nursing Correct answer: 4 Pain is the most common sign of UTI and is usually the most distressing symptom for the Recall that pain is the most frequent complaint in UTI.
diagnosis for a client with urinary tract infection (UTI)? client. The pain may be caused by inability to void or by bladder spasms. The client may have
manifestations of the other nursing diagnoses as well, but pain is the highest priority.

1.‐ Anxiety
2.‐ Disturbed sleep pattern
3.‐ Disturbed body image
4.‐ Pain

1661 The nurse caring for a client with benign prostatic Correct answer: 3 Various relationships between BPH and diet, obesity, sexual activity, and racial origins have Options 1, 2, and 3 list specific causes. Note that option 4 is different and in this case
hyperplasia (BPH) explains that currently the cause of been explored; however, none of these provide insight into its etiology. correct.
this disorder is:
1.‐ Linked to sexual activity.
2.‐ Linked to diet.
3.‐ Unknown.
4.‐ Related to racial origins.

1662 A male client presents to the emergency room with Correct answer: 1 Priapism is considered a medical emergency, because continued erection may lead to tissue Note that the situation is set in an emergency room; therefore immediate medical
priapism, or sustained erection. The nurse understands fibrosis and impotence. Immediate treatment involves ice packs, not warm soaks. Options 2 attention is a logical response.
that this client needs which of the following? and 4 do not apply.

1.‐ Immediate medical attention


2.‐ A relaxing environment so his erection will recede
3.‐ Warm soaks to the penis
4.‐ An evaluation for sexual dysfunction

1663 The nurse is evaluating a client with erectile Correct answer: 1 Propranolol, a beta adrenergic blocker, and many other antihypertensive medications can Note that only option 1, the beta blocker, has neurovascular effects.
dysfunction. Which of the following medications contribute to erectile dysfunction. Other examples include clonidine (Cata pres) and benazepril
currently used by the client could be an underlying (Lotensin). The other medications listed in options 2, 3, and 4 aren't known to have this effect.
cause?
1.‐ Propranolol (Inderal)
2.‐ Acetylsalicylic acid (aspirin)
3.‐ Penicillin
4.‐ Furosemide (Lasix)

1664 A 68‐year‐old female client presents to the Correct answer: 2 Because painless vaginal bleeding is often the only symptom of cervical or uterine cancer, this Recognize that unusual bleeding is symptomatic of cancer and clients should be evaluated
gynecology clinic with complaints of painless vaginal client should be tested for cancer. Options 1 and 4 are not probable given the client's age. accordingly.
bleeding. The nurse should be certain the client is Hormonal imbalances (option 3) may cause bleeding but are less urgent than the threat of
tested for which of the following health problems? cancer.

1.‐ Ovarian cyst


2.‐ Cervical or uterine cancer
3.‐ Hormonal imbalances
4.‐ Endometriosis

1665 The nurse concludes that a client who undergoes Correct answer: 3 NPTR monitoring helps differentiate between psychogenic and organic causes of erectile Notice that the question uses the words ‘penile’ and ‘rigidity,’ leading to the correct
nocturnal penile tumescence and rigidity (NPTR) dysfunction. The other options are not assessed using NPTR monitoring. response.
monitoring is most likely being evaluated for which of
the following disorders?
1.‐ Prostate cancer
2.‐ Infertility
3.‐ Erectile dysfunction
4.‐ Phimosis

1666 When examining a female client, the nurse observes Correct answer: 4 Third‐degree uterine prolapse is visible outside the body as the uterus inverts the vaginal Omit option 3 as the scenario does not indicate infection. The uterus is above the vagina
tissue protruding from the vagina. The nurse checks canal. Rectocele is prolapse of the rectum. Cystocele is prolapse of the urethra. A vaginal and a prolapse would present as described.
the medical record for a documented history of which infection would not cause tissue protrusion from the vagina, although the vaginal tissues
of the following disorders? would be reddened and/or edematous.

1.‐ Rectocele
2.‐ Cystocele
3.‐ Vaginal infection
4.‐ Uterine prolapse

1667 A client has just had a Papanicolaou (Pap) test. The Correct answer: 3 The Pap smear test is used to screen women for cervical cancer, assess hormonal status, and Recall that a Pap test is helpful in diagnosis of cervical cancer. Options 1, 2 and 4 are not
nurse would write which of the following indications identify the presence of sexually transmitted diseases, such as HPV infection. Infertility and associated with cervical cancer.
for the test on the laboratory requisition? AIDS are not diagnosed with the Pap test. Sterility is a male reproductive problem.

1.‐ Infertility
2.‐ Sterility
3.‐ Human papilloma virus (HPV) infection
4.‐ Acquired immunodeficiency syndrome (AIDS)

1668 When evaluating a client for breast cancer, the nurse Correct answer: 4 Early menstruation, before the age of 12, is a risk factor for breast cancer. Use of foam Select the response that reflects that the client has had prolonged exposure to hormones,
recognizes that which of the following client‐related contraceptives is not a factor. Late menopause increases the risk for breast cancer, but not specifically estrogen and progesterone.
factors is a risk for developing this disease? early menopause. A first birth after the age of 30 is a risk factor, but first birth before age 20 is
not.
1.‐ Use of foam contraceptives
2.‐ Early menopause, before age 45
3.‐ First birth before age 20
4.‐ Early menarche, before age 12

1669 When teaching a female client to perform breast self‐ Correct answer: 2 Breast examinations should be done monthly, at the same time each month to aid in This is the best response as it describes the procedure being done on a consistent basis.
examination (BSE), the nurse should instruct her to remembering to do it regularly. A post‐menopausal woman would select the same date each
perform the exam: month, while premenopausal women should do BSE at completion of the menstrual cycle.
Breast examination during menstrual flow is not the best time, because of hormonal influences
on the breasts.
1.‐ During menstrual flow.
2.‐ At the same time each month.
3.‐ At a random time each month.
4.‐ Every 2 months.
1670 When caring for a client with syphilis, the nurse Correct answer: 2 Syphilis is transmitted from open lesions during any sexual contact: genital, oral‐genital, or Recall that syphilis is a sexually transmitted disease.
instructs the client that syphilis may be transmitted by anal‐genital. Kissing, sharing eating utensils, and shaking hands do not transmit the disease.
which of the following methods?
1.‐ Kissing
2.‐ Open lesions during any sexual contact
3.‐ Sharing eating utensils
4.‐ Shaking hands

1671 When caring for a client diagnosed with end‐stage Correct answer: 2 With end‐stage renal disease the kidneys have difficulty excreting protein and the build‐up of Recognize that a client in renal failure gets necessary calories from carbohydrates.
renal failure, which of the following diets should the toxins in the system causes systemic problems. Clients must usually restrict dietary protein
nurse recommend? while increasing carbohydrate intake to meet energy needs and prevent tissue breakdown.
Potassium and sodium are restricted in clients with end‐stage renal failure. Protein‐rich foods
are also high in phosphorus, which is restricted to avoid osteodystrophy. Magnesium is not
specifically restricted.
1.‐ Increased protein, decreased carbohydrates
2.‐ Restricted protein, increased carbohydrates
3.‐ Increased potassium and sodium
4.‐ Restricted phosphorus and magnesium

1672 The client diagnosed with cystitis will be given a Correct answer: 1 The pain experienced with cystitis usually resolves as antibiotic therapy becomes effective. Recall that Pyridium is the only commonly used urinary analgesic.
prescription for an antibiotic. The nurse explains to the However, clients may be treated for urinary tract pain with phenazopyridine, which is a urinary
client that which of the following medications that is analgesic. Bethanechol chloride is a cholinergic agent used with neurogenic bladder or urinary
combined with the antibiotic will reduce the symptom retention. Oxybutinin and propantheline bromide are antispasmodics used to treat bladder
of dysuria? spasm.
1.‐ Phenazopyridine (Pyridium)
2.‐ Bethanechol chloride (Urecholine)
3.‐ Oxybutinin chloride (Ditropan)
4.‐ Propantheline bromide (Pro‐Banthine)

1673 The client has developed urolithiasis, and it is Correct answer: 3 Clients who have urinary stones of the uric‐acid type should avoid foods containing high Associate organ meats with high purine content, and therefore contribute to excess uric
determined that the client has uric acid stones. The amounts of purines, including the following: organ meats (liver, brain, heart, kidney, and acid.
nurse instructs the client to limit which of the sweetbreads), herring, sardines, anchovies, meat extracts, consommés, and gravies. Foods that
following foods in the diet that was previously eaten are low in purines include all fruits, many vegetables, milk, cheese, eggs, refined cereals, sugars
regularly? and sweets, coffee, tea, chocolate, and carbonated beverages.

1.‐ Oranges
2.‐ Cheese
3.‐ Liver
4.‐ Eggs

1674 In gathering data on an elderly male client the nurse Correct answer: 1 Benign prostatic hyperplasia (BPH) is a common cause of urinary retention when the enlarged Notice that the question states that the client is elderly. BPH is common in elderly men.
suspects that the most likely cause of his urinary prostate gland obstructs urinary flow. The other answers may also cause retention, but are less
retention is: common than BPH.
1.‐ Benign prostatic hyperplasia (BPH).
2.‐ Urinary tract infection.
3.‐ Voluntary urinary retention.
4.‐ Anticholinergic medications.
1675 A client complains of inability to inhibit urine flow Correct answer: 3 This type of incontinence is called urge incontinence, caused by a hypertonic or overactive Notice that the word ‘urge’ helps to identify this as the correct response. When the urge
long enough to reach the toilet. The nurse documents detrusor muscle that leads to increased pressure within the bladder. Stress incontinence is loss to void occurs, urine flow tends to be immediate.
the presence of which type of urinary incontinence? of urine with abdominal pressure. Reflex incontinence refers to loss of urine at somewhat
predictable intervals when a specific bladder volume is reached. Functional incontinence is an
involuntary, unpredictable passage of urine.

1.‐ Stress
2.‐ Reflex
3.‐ Urge
4.‐ Functional

1676 Which of the following nursing actions is most Correct answer: 1 Clients with excess fluid volume need to have restrictions in sodium intake because of the Use the fact that sodium causes retention of water to select the correct answer.
appropriate when caring for a client with a nursing relationship of water and sodium. Elevated serum sodium will cause water to be retained. The
diagnosis of excess fluid volume? client should be instructed to avoid foods that are high in sodium such as cured meats,
preserved foods, and canned goods. In addition, developing a schedule for oral intake and
offering limited ice chips and frequent mouth care helps in the water restriction necessary for
these clients. The other options are not appropriate interventions for the nurse to implement
specifically for the nursing diagnosis of excess fluid volume.

1.‐ Teaching clients about sodium content of foods


2.‐ Administration of Vitamin D supplements
3.‐ Assessing and documenting client's energy level
4.‐ Observing for signs of hypocalcemia

1677 Which of the following types of liquid should the Correct answer: 4 Cranberry juice reduces bacteria by acidifying urine and making it more difficult for bacteria Recall common information to identify option 4 as correct.
nurse recommend for a client who has frequent to remain attached to the bladder wall. Citrus fruits should not be used because they make the
urinary tract infections? urine alkaline. Drinks containing caffeine, including sodas, may irritate the bladder and worsen
the urinary frequency.
1.‐ Soda drinks
2.‐ Caffeine drinks
3.‐ Citrus juices
4.‐ Cranberry juice

1678 A client has undergone creation of an ileal conduit. Correct answer: 4 Peristomal skin should be cleansed with each appliance change using a gentle soap and water, Recognize that option 4 describes good basic skin care, not necessarily specific to the
Which of the following instructions to the client about and then should be rinsed and dried thoroughly. The client should change the appliance early ostomate.
ostomy care would be appropriate to include in the in the morning, when urine production is slowest from lack of fluid intake during sleep. The
teaching plan? opening of the appliance should be cut no larger than 3 mm greater than the opening of the
stoma; an opening smaller than the stoma would prevent proper application. Fluids are
encouraged to dilute the urine and decrease the odor.

1.‐ Cut the faceplate of the appliance so that the opening is slightly smaller than the stoma.
2.‐ Plan to do appliance changes just before bedtime.
3.‐ Limit fluids to minimize odor from urine breakdown to ammonia.
4.‐ Cleanse the skin around the stoma using gentle soap and water, rinse and dry well.

1679 The nurse is caring for a client with a history of renal Correct answer: 1 Nephrotoxicity can be caused by aminoglycoside antibiotics. This type of drug accumulates in Associate this drug category with nephrotoxicity.
disease. The nurse most closely monitors the client for tubular cells, eventually killing them. Options 2 and 3 are ototoxic, while option 4 is avoided in
signs of nephrotoxicity if the client is ordered to renal disease.
receive which of the following medications?

1.‐ Aminoglycoside antibiotics


2.‐ Aspirin‐containing drugs
3.‐ Loop diuretics
4.‐ Potassium supplements

1680 A client underwent cystectomy for cancer of the Correct answer: 2 A Kock pouch is a continent internal ileal reservoir, eliminating the need for an external Recognize that option 2 is the only option specific to Kock pouch.
bladder and had a Kock pouch created for urinary pouch. The nurse needs to instruct the client about the technique for catheterizing the pouch
diversion. The home care nurse would follow up with to empty the urine. Antibiotics are not required unless an infection is present and dietary
the client about which of the following instructions for restrictions are unnecessary.
self‐care?
1.‐ Application and care of external pouch
2.‐ Technique for catheterizing the Kock pouch
3.‐ Proper administration of prophylactic antibiotics
4.‐ Foods that must be restricted in the diet

1681 A client asks the nurse what causes syphilis. The nurse Correct answer: 2 The spirochete Treponema pallidum causes syphilis. The other responses are incorrect. Note that the three incorrect responses all contain the word ‘infection.’
answers correctly that syphilis is caused by a:

1.‐ Bacterial infection.


2.‐ A spirochete.
3.‐ Fungal infection.
4.‐ Yeast infection.

1682 A client with syphilis exhibits flu‐like symptoms, a skin Correct answer: 2 The client's symptoms are consistent with secondary syphilis, which occurs 2 weeks to 6 Omit option 4 based on the word ‘latent,’ implying no symptoms. Omit option 3 as in this
rash on the palms of the hands, and hair loss. The months after the initial chancre disappears. Latent syphilis produces no symptoms, and tertiary stage symptoms are neurological. Primary infection is noted by only a chancre.
nurse develops a plan of care based on which stage of syphilis is the final stage of the illness. Tertiary syphilis is characterized by the development of
the disease? infiltrating tumors and involvement of the central nervous and cardiovascular systems.

1.‐ Primary syphilis


2.‐ Secondary syphilis
3.‐ Tertiary syphilis
4.‐ Latent syphilis

1683 Which of the following nursing diagnoses is the top Correct answer: 1 Herpetic lesions are very painful, so the first priority is to provide comfort measures for the Recognize that prioritization is the key to correctly answering this question. The most
priority during a client's first outbreak of genital client. The other diagnoses do not address the client's most immediate concern. immediate issue is pain.
herpes?
1.‐ Pain
2.‐ Sexual dysfunction
3.‐ Anxiety
4.‐ Risk for infection

1684 The nurse is teaching a group of men about testicular Correct answer: 1 A painless, hard nodule is the classic presenting symptom of testicular cancer. Testicular self‐ Omit options 3 and 4 as incorrect. Options 1 and 2 are opposite, therefore one is likely
cancer. Which of the following statements would the examination can help detect this sign. Testicular cancer is the most common cancer in men correct.
nurse include about this type of cancer? between the ages of 15 and 35, and is the third‐leading cause of cancer death in young men.

1.‐ It's usually painless, and therefore hard to detect.


2.‐ It causes severe pain in the early stages.
3.‐ Testicular self‐examination doesn't help detect it.
4.‐ It rarely affects young men.

1685 The nurse working in an urgent care center would Correct answer: 3 Testicular torsion, or twisting of the testes and spermatic cord, is a potential emergency, Note that options 1, 2, and 3 are similar in that they are all ‘cele’ conditions and generally
consider it a surgical emergency if a client presented because compromised blood flow to the testicle may lead to ischemia and necrosis. The other not urgent.
with which of the following conditions? conditions usually don't require emergency intervention.
1.‐ Hydrocele
2.‐ Spermatocele
3.‐ Testicular torsion
4.‐ Varicocele

1686 A client is admitted for treatment of hydrocele. The Correct answer: 1 Hydrocele is a fluid‐filled mass within the scrotum. Option 2 describes spermatocele. Option 3 Use the prefix ‘hydro’ in the question stem to identify the correct option.
nurse understands this results from which of the describes varicocele. Option 4 describes testicular torsion.
following?
1.‐ A fluid‐filled mass within the scrotum
2.‐ A mass containing dead spermatozoa
3.‐ Dilated veins within the spermatic cord
4.‐ Twisting of the testes and spermatic cord

1687 The nurse anticipates that a client being evaluated for Correct answer: 3 Sildenafil is an oral medication used to treat erectile dysfunction in men. The other Omit options 1, 2, and 3 as not specific to the condition being treated.
erectile dysfunction may receive a prescription for medications may worsen the client's condition.
which of the following medications?

1.‐ Propranolol (Inderal)


2.‐ Diazepam (Valium)
3.‐ Sildenafil (Viagra)
4.‐ Progesterone

1688 The nurse is teaching a male client about the use of Correct answer: 4 Clients who take nitrates may experience severe hypotension when using sildenafil. Taking Select the option that is a common concern with medications, combination with other
sildenafil (Viagra). Which of the following instructions the drug after a high‐fat meal will delay its onset. The drug should not be combined with other drugs.
is correct? treatments for erectile dysfunction, and should not be taken more often than once daily.

1.‐ "Take the drug after a high‐fat meal to avoid GI upset."


2.‐ "You may combine the drug with your vacuum constriction device."
3.‐ "It's acceptable to use the drug several times a day."
4.‐ "Don't use the drug if you take nitrate medications, such as nitroglycerin."

1689 The nurse is teaching about sexually transmitted Correct answer: 2 Chlamydial infection may be present for months or years without producing symptoms in Recall that a lack of symptoms is common with this infection and so is key in identifying
diseases. Which of the following is the greatest women. The disease may invade the uterus, resulting in devastating complications. It is caused the correct response.
concern with chlamydial infection that the nurse by Chlamydia trachomatis, a bacterium that behaves like a virus. Young women using oral
should emphasize? contraceptives have the highest risk.
1.‐ It causes severe pain and burning with urination.
2.‐ There is a lack of symptoms until infection invades the uterus.
3.‐ It is caused by yeast infection that spreads easily.
4.‐ Older women are at highest risk in contracting the disease.

1690 When teaching a male client about prostate cancer, Correct answer: 1 When diagnosed early and confined to the prostate gland, prostate cancer is curable and the There is no indication of client age or race in the question so omit options 3 and 4. Omit
the nurse explains that the survival rate for prostate 5‐year survival rate is 100 percent. Men under the age of 40 rarely have prostate cancer. option 2 as being incorrect information. Recall most cancers have a variable prognosis
cancer is: African‐American men have a higher incidence of prostate cancer and a higher mortality rate. based on factors such as age and point of diagnosis.

1.‐ Excellent when diagnosed early.


2.‐ Poor, no matter when diagnosed.
3.‐ Best for men under age 40.
4.‐ Best for African‐American men.
1691 The nurse plans for interventions for the nursing Correct answer: 4 Positioning the client in semi‐Fowler's position allows optimal lung expansion. Positioning the Select the response that includes more than one position. Recall that prone is generally
diagnosis "Ineffective breathing pattern" on a client client on the operative side helps to splint the incision and improves lung expansion. The not a useful client position.
who underwent a nephrectomy today. Which of the combination of these two positions provides the best positions to prevent respiratory
following is the best intervention for this diagnosis? depression in a post‐nephrectomy client.

1.‐ Alternating prone and semi‐Fowler's position


2.‐ Positioning the client on the operative side
3.‐ Positioning the client on the non‐operative side
4.‐ Alternating semi‐Fowlers and side‐lying on the operative side

1692 The nurse is caring for a client admitted to the Correct answer: 2 Dehydration and immobility predispose the client to the formation of renal calculi. Prolonged Notice that the scenario indicates the client was in a nursing home. Select the option that
hospital from a nursing home with suspected renal immobilization leads to loss of bone calcium and hypercalciuria, which predisposes an reflects common problems with nursing home clients.
calculi. In reviewing the client's history, which of the individual to the development of calcium oxalate stones. Adequate hydration prevents urinary
following conditions most likely contributed to the stasis and stone formation. Option 3 does not have a direct link to the development of renal
disorder? calculi. While infection of the urinary system can lead to development of renal stones, it is not
the most likely cause.
1.‐ Fever and malnutrition
2.‐ Dehydration and immobility
3.‐ Glycosuria and ketonuria
4.‐ Indwelling Foley catheter

1693 A client with renal failure has a creatinine level of 7 Correct answer: 2 A client with impaired renal function requires close monitoring of intake and output. The Select the option that will best measure elimination.
mg/dL. Which of the following nursing assessments is serum creatinine value of 7 (normal 0.8 to 1.2 mg/dL for males; 0.6 to 0.9 for females) provides
most important to evaluate? evidence of an estimated decrease in the glomerular filtration rate. Although peripheral
edema, weight increase, and skin changes may occur in renal impairment, assessment of
intake and output is vital in monitoring fluid balance.

1.‐ Peripheral edema


2.‐ Intake and output
3.‐ Weight
4.‐ Skin turgor

1694 Which of the following nursing actions is most Correct answer: 3 Forcing fluids helps prevent urinary retention and flushes bacteria from the bladder. Citrus Recall that UTI is best treated by flushing the system with oral fluids. Only specific juices
appropriate for a client with a severe urinary tract juices along with caffeine and alcohol should be avoided because they cause bladder irritation. (i.e., cranberry) are indicated.
infection? The use of a heating pad and sitz baths may ease the discomfort associated with the infection
but increasing the fluid intake is a more important action.

1.‐ Maintaining NPO status


2.‐ Encouraging citrus juices
3.‐ Forcing oral fluids
4.‐ Applying of heating pad

1695 The nurse is caring for a client with incontinence. Correct answer: 3 Urinary incontinence is not a normal part of aging. The causes of urinary incontinence include Select the response that is incorrect. In this case option 3 reflects that the client does not
Which of the following statements made by the client anything that may interfere with bladder or sphincter control. Option 1 refers to Kegel understand that the condition can be controlled.
indicates a need for further teaching? exercises, which can contribute to regaining continence. Voiding schedules and prompted
voiding help too in regaining continence. Application of pressure over the bladder area, also
known as the Crede maneuver, helps in successful urination.

1.‐ "I do pelvic exercises five times each day."


2.‐ "I go to the bathroom to try to urinate every 2 hours."
3.‐ "I am trying to accept this incontinence as a normal part of aging."
4.‐ "I apply pressure over my bladder to help me urinate successfully."
1696 The nurse is collecting a 24‐hour‐urine specimen from Correct answer: 2 The first specimen contains urine that was in the bladder before the test began, so it should Omit options 3 and 4 as they are inconsistent with the 24‐hour collection. Option 1 is
a client. After collecting the first specimen, the nurse be discarded. The test begins with the next voided specimen. incorrect as it would alter the timing of the collection.
should:
1.‐ Save it as part of the collection.
2.‐ Discard the specimen, and then begin collecting.
3.‐ Test the specimen, and then discard it.
4.‐ Collect 100 mL of the specimen, discard the rest.

1697 Which of the following nursing diagnoses is of highest Correct answer: 1 Infection is a significant risk for CAPD clients, because organisms can enter the body through Associate peritonitis as the highest risk in CAPD.
priority for a client receiving continuous ambulatory the peritoneal catheter and through the dialysate solutions. The other diagnoses are either
peritoneal dialysis (CAPD)? inappropriate or have lower priority because the problems are less threatening.

1.‐ Risk for infection


2.‐ Risk for bleeding
3.‐ Altered body image
4.‐ Risk for fluid volume deficit

1698 A client who receives a kidney transplant will need to Correct answer: 4 Corticosteroids, such as prednisone, help control rejection of the new kidney. Rejection is a Recall that prednisone is the only anti‐inflammatory or anti‐rejection drug listed.
take medications for life. The nurse places highest common cause of graft loss in individuals with renal transplants. The other medications are not
priority on client teaching for which of the following routinely ordered for the client.
medications?
1.‐ Penicillin G
2.‐ Heparin
3.‐ Gamma globulin
4.‐ Prednisone

1699 A client with chronic renal failure asks the nurse why Correct answer: 2 Erythropoietin, a hormone synthesized in the kidneys, stimulates the bone marrow to Choose the option that best refers to erythropoietin and the kidneys' function in blood
he has developed anemia. The most accurate response produce red blood cells. The production of this hormone is in response to low oxygen levels in formation.
by the nurse is: the kidney cells. When there is decrease in functioning renal mass, erythropoietin synthesis is
also decreased. With deficient erythropoietin, anemia develops in the client with renal disease.

1.‐ "The increase in nitrogen wastes in your blood destroys your bone marrow."
2.‐ "A hormone in your kidneys that stimulates your bone marrow is lacking."
3.‐ "You have lost some blood through your urine."
4.‐ "The low protein diet that you are on causes the anemia."

1700 A client is scheduled for urinary diversion surgery. Correct answer: 3 The figure shows that a portion of the ileum is formed into a tubular pouch with the open end Select the most common urinary diversion.
Based on this figure, nursing interventions will be brought to the surface of the skin and the ureters inserted into the pouch. An ileal conduit is
planned for which type of urinary diversion? the most common urinary diversion. A colon conduit (option 1) is similar to the ileal conduit
but utilizes a portion of the sigmoid colon rather than the small intestine. Option 2 (Indiana
continent reservoir), describes a surgical diversion whereby the reservoir is formed from the
colon and cecum. A Kock pouch (option 4) is similar to the ileal conduit but nipple valves are
formed, which prevents leakage and reflux.

1.‐ Colon conduit


2.‐ Indiana continent urinary reservoir
3.‐ Ileal conduit
4.‐ Kock pouch
1701 When teaching a male client who is being tested for Correct answer: 2 The seminiferous tubules within the testes produce sperm, while Leydig's cells (interstitial Recall that testosterone is a hormone manufactured in the testes, the only endocrine
fertility, the nurse explains that sperm and cells) within the testes manufacture testosterone. The other options are incorrect. gland listed.
testosterone are produced in the:
1.‐ Prostate gland.
2.‐ Testes.
3.‐ Seminal vesicles.
4.‐ Bulbourethral glands.

1702 A client is preparing to undergo a prostatectomy. Correct answer: 1 Because the statement in option 1 is incorrect, the client needs more teaching. He will be Options 1 and 2 express opposite concepts. One of the items in the pair is likely correct.
Which statement indicates that he needs more able to have intercourse and will remain fertile without his prostate gland. Semen volume will
teaching? decrease, because the prostate secretions make up one‐third of the volume.

1.‐ "I won't be able to have sex without my prostate gland."


2.‐ "The prostate gland isn't necessary in order to have intercourse."
3.‐ "I'll still be fertile without my prostate gland."
4.‐ "My semen volume will be less without the prostate gland."

1703 In developing a teaching plan for a group of high Correct answer: 4 The vagina, ovaries, fallopian tubes, and uterus are parts of the internal organs of the female Recognize that option 4 is the only option that names an internal structure.
school students about the female internal reproductive system. The other organs listed are external.
reproductive system, the nurse includes discussion of
the:
1.‐ Labia majora.
2.‐ Labia minora.
3.‐ Clitoris.
4.‐ Vagina.

1704 A client with symptoms of chlamydia needs to have a Correct answer: 3 The definitive diagnosis for chlamydia is tissue culture of the endocervix or the urethra. This is Omit options 1, 2, and 4 as being diagnostic for other STDs.
diagnostic test for the infection. Which of the following an expensive test and therefore a careful analysis of the history and physical assessment
is the appropriate test that the nurse should prepare findings are relied upon. VDRL, RPR, and FTA‐ABS are diagnostic tests for syphilis.
for?
1.‐ VDRL
2.‐ RPR
3.‐ Tissue culture
4.‐ FTA‐ABS

1705 Which of the following questions is most appropriate Correct answer: 1 Option 1 is the most sensitive way to inquire about erectile function. Interview questions are Select the option that is open and would most encourage the client to discuss sensitive
for the nurse to ask a male client when assessing the less threatening if they are asked in a way that gives the client the option to report behaviors issues.
reproductive system? and symptoms.
1.‐ "Have you noticed any changes in your erections?"
2.‐ "Do you have difficulty achieving an erection?"
3.‐ "Can you maintain an erection?"
4.‐ "Has your partner complained about your erections?"

1706 When assessing a 75‐year‐old male client, the nurse Correct answer: 2 Benign prostatic hyperplasia (BPH) is the most common disorder of the aging male client. Recognize that option 2 is the only condition that risk increases with age. The others are
questions the client about symptoms of which of the Testicular cancer is the most common cancer in men between the ages of 15 and 35. Testicular common in younger individuals.
following conditions that is common in older men? torsion occurs at any age and gonorrhea is highest in occurrence during the sexually active
years. Among women, 15‐ to 19‐year‐olds and men 20‐ to 24‐years‐old have the highest rate.

1.‐ Testicular cancer


2.‐ Benign prostatic hyperplasia
3.‐ Testicular torsion
4.‐ Gonorrhea
1707 The nurse is caring for a 68‐year‐old male diagnosed Correct answer: 4 The statements in the first three options correctly describe signs of BPH. Option 4 indicates Recall that reducing fluid intake is rarely advised.
with benign prostatic hyperplasia (BPH). Which of the the need for further teaching because the client should increase his fluid intake (unless
following statements by the client indicates the need contraindicated) to prevent urinary tract infections and lessen dysuria.
for further teaching?
1.‐ "The enlarged prostate gland causes me to get up three times every night to urinate."
2.‐ "The enlarged prostate gland may produce blood in my urine."
3.‐ "I can get urinary tract infections because of the enlarged prostate gland."
4.‐ "I should cut down on the fluids I drink so I won't have to urinate so often."

1708 A client with benign prostatic hyperplasia (BPH) is Correct answer: 2 A retropubic prostatectomy is used to remove massive prostatic tissue. It involves having a Omit options 1 and 4 as they are inconsistent with the information in the scenario, in that
scheduled tomorrow for surgery to resect prostatic low midline abdominal incision but the bladder is not incised. A suprapubic prostatectomy they would not require an abdominal incision.
tissue. The physician described the surgery with the involves an abdominal incision but cuts through the bladder to access the anterior aspect of
above diagram indicating an approach through an the prostate. TURP is a surgical approach that involves insertion of a resectoscope into the
abdominal incision. What surgical approach will the urethra. In the perineal resection approach, the incision is made between the anus and the
nurse plan for? scrotum. Perineal resection is used more commonly in cancer of the prostate.

1.‐ Transurethral resection of the prostate (TURP)


2.‐ Retropubic prostatectomy
3.‐ Suprapubic prostatectomy
4.‐ Perineal resection

1709 Which of the following instructions should the nurse Correct answer: 4 The healing period after prostate surgery is 4 to 8 weeks, and the client should avoid Identify the instruction that would be common in any post operative client.
provide for a client returning home after prostate strenuous activity during this period. Blood in the urine is fairly common after surgery. The
surgery? client should not drive for 2 weeks, except for short rides.
1.‐ "Reduce your fluid intake so you won't need to void as often."
2.‐ "Call the doctor immediately if you notice blood in your urine."
3.‐ "You may drive yourself home."
4.‐ "Avoid strenuous activity and heavy lifting for 4 to 8 weeks."

1710 A client tells the nurse he has been diagnosed with Correct answer: 4 With prostatodynia, the client experiences symptoms of prostatitis, but shows no signs of Recognize that options 1, 2, and 3 would generally occur together and be associated with
chronic, nonbacterial prostatitis (prostatodynia). The inflammation or infection. Positive cultures, chills and fever, and purulent secretions are infection.
nurse would expect to note which of the following symptoms of infection.
findings?
1.‐ Positive urine cultures
2.‐ Chills and fever
3.‐ Purulent prostatic secretions
4.‐ No evidence of inflammation in the prostate

1711 Which of the following assessment findings in a 35‐ Correct answer: 1 A positive Kernig’s sign is common in intracranial hematomas, which is described in option 1. The core issue of the question is knowledge of associated findings with intracranial
year‐old client with an intracranial hematoma should Option 2 is a negative Babinski; with a hematoma, the nurse should expect a positive Babinski hematoma. Use nursing knowledge and the process of elimination to make a selection.
concern the nurse? (dorsiflexion of the toes in an adult). Option 3 is common in many illnesses; option 4 is specific
to Parkinson’s disease.
1.‐ Hamstring pain when the hip and knee are flexed and then extended
2.‐ Curling of the toes when the bottom of the foot is stroked in upward motion
3.‐ Muscle aches and cramping, especially at night
4.‐ Cogwheel and lead pipe rigidity
1712 The nurse would prevent corneal abrasion in the Correct answer: 2 Corneal abrasion in the client with myasthenia gravis is caused by dryness of the cornea from Use nursing knowledge and the process of elimination to make a selection.
client with myasthenia gravis by performing which of inability to close the eyelids and blink. It can be prevented by application of artificial tears
the following nursing interventions? every 1 to 2 hours. The other options do not address this need.

1.‐ Doing a saline eye irrigation every shift


2.‐ Instilling artificial tears in the eyes every 1 to 2 hours
3.‐ Ensuring the client’s contact lenses are on while awake
4.‐ Providing sunglasses when client is outside

1713 The client with newly diagnosed Parkinson’s disease Correct answer: 1 The nurse should first encourage the client experiencing a loss to express his feelings. This The core issue of the question is a therapeutic communication. For communication
states, “I just don’t think I can handle having answer acknowledges the client’s feelings, is open‐ended, and promotes further discussion. questions, look first for the option that addresses the client’s feelings or concerns.
Parkinson’s disease.” What is the nurse’s best first Option 2 provides false reassurance. Options 3 and 4 do not address the client’s feelings as
response? shared with the nurse.
1.‐ “You sound overwhelmed. Can you tell me more?”
2.‐ “I am sure you can. A lot of other people do!”
3.‐ “What do you think will be the hardest thing to handle?”
4.‐ “The entire health care team will help you manage the disease.”

1714 The nurse caring for the client with myasthenia gravis Correct answer: 3 When the muscles involved in chewing and swallowing as well as the diaphragm and The core issue of the question is the ability to determine a priority complication of
would do which of the following as a priority to intercostal muscles are weak, the client may aspirate or experience poor gas exchange; both myasthenia gravis and to then select the intervention that reduces it. Use nursing
minimize the risk for complications of the disease? increase the risk for pneumonia. Options that protect the airway always have highest priority. knowledge and the process of elimination to make a selection.
The client is not at risk for hemorrhage (option 1) or pneumonia (option 2). Option 4 may be an
element of routine care.
1.‐ Inspect for hemorrhage.
2.‐ Assess for pneumonia.
3.‐ Offer to cut the client's food as needed.
4.‐ Provide the client with a bedside commode.

1715 A client calls the telephone triage nurse to report Correct answer: 2 A stiff sore neck is a sign of meningeal irritation and possible meningitis. The nurse may The core issue of the question is the ability to recognize clients at risk for or showing early
fever, nausea, chills, and malaise. The nurse instructs further inquire if flexion of the neck causes pain and the hip and knee to flex (Brudzinski’s sign) signs of meningitis. Use nursing knowledge and the process of elimination to make a
the client to come immediately to the emergency and how high the fever is. The other symptoms are typical of influenza. selection.
room when he relates he also has which of the
following?
1.‐ A bad headache
2.‐ A stiff sore neck
3.‐ A heart rate of 106
4.‐ A roommate with the same symptoms

1716 When assessing the client with meningitis, the nurse Correct answer: 2 The first signs of increased intracranial pressure are often subtle changes in level of The core issue of the question is the ability to discriminate early signs of rising intracranial
looks for which of the following as a frequent first sign consciousness. Other changes (including rising systolic BP, irregular respiratory rate, and pressure from later ones. Use nursing knowledge and the process of elimination to make a
of increased intracranial pressure? bounding pulse) come later as intracranial pressure rises further. selection.

1.‐ A rising systolic blood pressure


2.‐ Change in mood or attention level
3.‐ Irregular respiratory rate and depth
4.‐ A bounding radial pulse

1717 The nurse is instructing the client who has been in the Correct answer: 1 It is essential that the client recovering from bacterial meningitis take all of the prescribed The core issue of the question is the ability to prioritize completion of antibiotic therapy
hospital with bacterial meningitis and will be going antibiotic as directed. Failure to do so puts the client at risk for a relapse of symptoms and with bacterial meningitis as essential. Use nursing knowledge and the process of elimination
home soon. Which of the following will be of the contributes to development of bacterial resistance to antibiotics. Options 2, 3, and 4 are to make a selection.
highest priority? important aspects of self‐care during recuperation but are not as essential as the completion
of antimicrobial therapy.
1.‐ Take all of the antibiotics as directed until completely gone.
2.‐ Eat a high‐protein, high‐calorie diet
3.‐ Exercise daily, beginning with active ROM.
4.‐ Get at least 8 hours of sleep per night with frequent rest periods.

1718 Which of the following instructions would the nurse Correct answer: 4 Urinary retention in the client with multiple sclerosis is a sequela of impaired conduction of The core issue of the question is knowledge of appropriate methods to manage urinary
give to a client with multiple sclerosis who has urinary nerves innervating the bladder. Performing self‐catheterization will drain the bladder and help retention in a client with multiple sclerosis. Use nursing knowledge and the process of
retention? prevent urinary tract infection. The client with multiple sclerosis will be encouraged to elimination to make a selection.
increase fluid intake to prevent constipation. Because urinary retention is incomplete emptying
of the bladder, neither running water nor caffeinated beverages would be useful.

1.‐ “Run water whenever you experience difficulty initiating urination.”


2.‐ “Decrease your fluid intake to prevent urgency.”
3.‐ “Drink a caffeinated beverage to promote the ability to form urine.”
4.‐ “Catheterize your bladder according to the schedule we discussed.”

1719 The nurse advises the family of the client with Correct answer: 3 While options 1, 2, and 4 are all appropriate interventions for the client with Parkinson’s The critical word in the question is approach, which implies a manner of behaving rather
Parkinson’s disease that the best approach to helping disease, the essential approach to enhance and encourage self‐care abilities will be an than a specific or single action. Use nursing knowledge and the process of elimination to
the client maintain as much functional independence unhurried one that allows sufficient time for self‐expression and for the client to do as much as make a selection.
as possible will be to possible for himself or herself.
1.‐ Assist the client to take a warm bath every morning.
2.‐ Perform passive ROM three times a day.
3.‐ Display an unhurried manner that allows the client sufficient time to respond or act.
4.‐ Obtain assistive devices that will make activities of daily living (ADLs) easier.

1720 The nurse concludes that more teaching may be Correct answer: 2 Fluid restriction may be needed in the period immediately following a stroke, but this is not The core issue of the question is knowledge of fluid needs in a client post‐stroke who has
necessary when visiting a client post‐CVA during the necessary after discharge to home. Keeping urine dilute will help prevent urinary tract been discharged from the acute care setting. Use nursing knowledge of rehabilitation and
first home visit because infection. A fluid intake of 2000 cc per day will also improve bowel elimination. the process of elimination to make a selection.

1.‐ The commode is observed to be at the bedside.


2.‐ A fluid restriction chart is on the refrigerator.
3.‐ Metamucil is on the kitchen counter.
4.‐ Hand weights are next to the couch.

1721 The office nurse should encourage a client on the Correct answer: 1 What the client describes is a classic ascending progression of Guillain‐Barré syndrome. The Remember the ABCs and prioritize an answer that refers to a possible impaired airway.
phone to go directly to the hospital emergency room muscular weakness may ascend to include the diaphragm. Total respiratory paralysis can Use nursing knowledge and the process of elimination to make a selection.
based on which of the following statements? occur, requiring ventilatory support. The incorrect responses refer to chronic problems, not an
acute one.
1.‐ “My legs are weak and now I’m having trouble getting a good breath.”
2.‐ “My shaky hand is no better than last visit. In fact, I think it’s getting worse”
3.‐ “The double vision went away when I put my eye patch on.”
4.‐ “My headache doesn’t seem any better even though I gave up coffee.”

1722 A 76‐year‐old woman comes to the emergency room Correct answer: 3 The client is showing signs of rising intracranial pressure, and infusion of IV fluids leads to Use nursing knowledge and the process of elimination to make a selection.
by ambulance because of a possible stroke. Vital signs hypervolemia and worsens the intracranial pressure. Following Maslow’s hierarchy, choose
are pulse 90, blood pressure 150/100, respirations 20. physiological before psychological answers. A Dilantin level would not be relevant to CVA
Thirty minutes later, vital signs are pulse 78, blood status. The nurse would want to avoid adding to the client’s volume status by offering fluids,
pressure 170/90, respirations 24 and irregular. The and dehydration is not a concern at this time.
nurse should take which action at this time?

1.‐ Ask the woman to describe how she’s feeling.


2.‐ Check the client’s phenytoin (Dilantin) level.
3.‐ Get an order to decrease the rate of IV fluids.
4.‐ Offer the client clear liquids to prevent dehydration.

1723 A client seen in the neighborhood clinic complains of Correct answer: 4 A more detailed assessment is important in collecting data to meet client needs. A picture of The core issue of the question is selection of an appropriate communication that focuses
“eye problems” and noticed generalized weakness that multiple sclerosis may be unfolding. The nurse takes the time to be therapeutic without on assessment. Use nursing knowledge and the process of elimination to make a selection.
became markedly worse after visiting with a friend and providing false reassurance or limiting responses. Open‐ended, nonjudgmental responses are
using the friend’s hot tub. The client gives considerably ideal.
long, detailed answers during the history. The nurse’s
best response is:

1.‐ “Was the weather the same each time you used the hot tub?”
2.‐ “You’ll feel better after getting this all off your mind.”
3.‐ “Please be brief in your answers so I can get you through this.”
4.‐ “Can you tell me more about the eye problems?”

1724 An abnormal EEG indicates that a 2‐year‐old client Correct answer: 1 Also known as absence seizures, petit mal seizures may be no more observable than brief The core issue of the question is the ability to discriminate different types of seizures
has epilepsy, but the parents indicate that they have staring instances. The mother should be instructed to note and report any change in the child’s based on presentation (or lack of manifestations). Use nursing knowledge and the process
never observed a seizure. The pediatric nurse behavior, no matter how small. of elimination to make a selection.
concludes that the child may be experiencing seizures
of the following type?
1.‐ Petit mal
2.‐ Myoclonic
3.‐ Jacksonian
4.‐ Grand mal

1725 The nurse provides care for a 13‐year‐old who was Correct answer: 2 The nurse would want to assess the pin sites for redness, edema, and drainage and would The core issue of the question is knowledge of appropriate care for a client who is in a
placed in a halo brace within the last 24 hours because want to ensure that the vest fits snugly. Following the nursing process, an assessment answer halo vest. Use nursing knowledge and the process of elimination to make a selection.
of a spinal cord injury. Which of the following is the would precede an implementation answer (options 3 and 4).
first priority of the nurse?
1.‐ Loosen any connections on the vest to assess the skin
2.‐ Assess the pin sites
3.‐ Ask how the client is able to reposition self in bed
4.‐ Encourage active range of motion to lower extremities

1726 The rehabilitation nurse is admitting a client following Correct answer: 4 Hemisection of the anterior and posterior portions of the spinal cord results in loss of The core issue of the question is knowledge of characteristics of Brown‐Séquard syndrome
spinal cord injury. The nurse concludes that the client position sense (proprioception) on the same side of the body as the trauma, below the level of following spinal cord injury. Use nursing knowledge and the process of elimination to make
has developed Brown‐Séquard syndrome after injury. Option 3 is seen in anterior cord syndrome; option 1 is incorrect. a selection.
assessing which of the following in the client?

1.‐ Ipsilateral motor loss above the lesion


2.‐ Contralateral loss of proprioception
3.‐ Hyperanesthesia below the level of the lesion
4.‐ Ipsilateral motor loss below lesion

1727 While assessing airway and breathing, the client Correct answer: 4 The brain stem’s final effort to maintain cerebral perfusion is seen with an increased systolic The core issue of the question is the ability to recognize patterns of change in vital signs
presenting with increased intracranial pressure would blood pressure, bradycardia, and an irregular respiratory pattern know as Cushing’s response. that reflect increasing intracranial pressure. Use nursing knowledge and the process of
most likely exhibit which of the following vital signs? elimination to make a selection.

1.‐ BP 190/84, HR 150, and an irregular respiratory pattern


2.‐ BP 80/50, HR 50, and Kussmaul respirations
3.‐ BP 80/50, HR 150, and Cheyne‐Stokes respirations
4.‐ BP 190/84, HR 50, and an irregular respiratory pattern
1728 In providing for the safety of the client during a grand Correct answer: 3 The nurse’s priority is to protect the client from injury. To promote drainage, it is more The core issue of the question is priority concerns for a client experiencing a seizure. Use
mal seizure, the nurse performs which of the following effective to secure an airway by turning the client onto the side (option 1). Inserting a tongue Maslow’s hierarchy of needs, nursing knowledge, and the process of elimination to make a
interventions? blade can cause trauma (option 2); the tongue blade may move during the seizure and obstruct selection.
the airway. Oxygen should be available but does not have to be applied (option 4).

1.‐ Positions the client on his back


2.‐ Gently places a padded tongue blade between the teeth
3.‐ Protects the client from injury
4.‐ Applies oxygen immediately via mask

1729 The community health nurse interprets that clients Correct answer: 3 Mosquitoes, the vectors that transport encephalitis, are found in large numbers in swampy The core issue of the question is knowledge of risk factors for encephalitis. Use nursing
who live in a swampy bayou area in the southern areas. Meningitis can be attributed to overcrowded conditions; Parkinson’s has an uncertain knowledge and the process of elimination to make a selection.
United States might be at risk of contracting which of etiology; and risk factors for multiple sclerosis include genetics and family history.
the following?
1.‐ Meningitis
2.‐ Parkinson’s
3.‐ Encephalitis
4.‐ Multiple sclerosis

1730 The client recently diagnosed with Guillain‐Barré Correct answer: 2 Guillain‐Barré syndrome is an acute demyelinating disorder that less commonly may present The core issue of the question is the ability to explain pathophysiology underlying signs
syndrome is drooling and having difficulty swallowing with initial weakness in the cranial nerves that progresses downward. Impairment of cranial and symptoms of Guillain‐Barré syndrome. Use nursing knowledge and the process of
secretions. When the family asks why this occurs, the nerves IX and X will affect swallowing. elimination to make a selection.
nurse indicates that the cause is
1.‐ Obstructed blood flow to the midbrain.
2.‐ Demyelination of cranial nerves responsible for swallow and gag reflex.
3.‐ Enlargement of the parotid and salivary glands.
4.‐ Deficiency in the thiamine and pyridoxine in the central nervous system.

1731 A 1‐year‐old child has been diagnosed with cerebral Correct answer: 4 At this age, a 1‐year‐old is beginning speech. This child will have trouble developing language The core issue of the question is the ability to determine appropriate nursing diagnoses for
palsy. The child has the spastic form that affects all because of the spasticity. Urinary incontinence occurs in all 1‐year‐old children, as does a client with cerebral palsy while taking into consideration growth and development. Use
extremities. Which of the following nursing diagnoses feeding self‐care deficit. Thought processes are difficult to evaluate in a 1‐year‐old. nursing knowledge and the process of elimination to make a selection.
would be appropriate for a child at this age?

1.‐ Urinary incontinence


2.‐ Feeding self‐care deficit
3.‐ Impaired thought processes
4.‐ Impaired verbal communication

1732 The nurse observes a child starting to have a seizure. Correct answer: 2 Observation and documentation of seizure activity can provide valuable information to help The core issue of the question is the ability to provide safe care to a client experiencing a
After assessing the airway, which is the highest priority in diagnosis and treatment. Once a seizure is in process, it would be dangerous to attempt to seizure. Use nursing knowledge and the process of elimination to make a selection.
of the nurse? insert an airway. Administration of medication would require a physician’s order. Actually
restraining the extremities is more likely to inflict injury than prevent.

1.‐ Insert an artificial airway.


2.‐ Observe and record seizure activity.
3.‐ Administer diazepam (Valium).
4.‐ Restrain the extremities to protect the child from injury.
1733 The nurse has taught the parents of a 6‐year‐old child Correct answer: 4 In most children, by age 6, the cranial suture lines have fused and the fontanelles are closed, The core issue of the question is knowledge of early signs of rising intracranial pressure,
with a ventriculoperitoneal (VP) shunt to monitor the so the first three symptoms would not be common. An altered level of consciousness would be which is a sign of shunt malfunction. Use nursing knowledge and the process of elimination
child for shunt malfunction. The nurse determines the a symptom of shunt malfunction for the older child. to make a selection.
parents understand the instructions if they state they
will notify the physician if the child presents with:

1.‐ Bulging soft spot.


2.‐ Expanding head size.
3.‐ Sunset eyes.
4.‐ Altered level of consciousness.

1734 A child is admitted with a head injury after being in a Correct answer: 2 Drainage of cerebrospinal fluid (a clear fluid) from the ear is a symptom of basilar skull The core issue of the question is the ability to correctly interpret signs of head injury. Use
motor vehicle accident. The nurse notes a clear fracture. Children with linear skull fractures are often asymptomatic. Subdural and epidural nursing knowledge and the process of elimination to make a selection.
drainage from the left ear. The nurse would suspect: hematomas present with signs of increasing intracranial pressure.

1.‐ Linear skull fracture.


2.‐ Basilar skull fracture.
3.‐ Subdural hematoma.
4.‐ Epidural hematoma.

1735 The nurse has formulated a nursing diagnosis of Correct answer: 3 While families may need education about seatbelts and sources of support, it is not the The core issue of the question is the ability to determine priorities for the family of a
ineffective family processes related to hospitalization optimal time to implement such teaching at this point in the crisis. It is optimal to find out critically ill child. Select the option that will most closely address the family’s current issues
of a child with a potentially fatal condition for the what the family members' perceptions are of what is going on and what they feel their needs and concerns.
family of a child who sustained a brain injury during an are. The best way to determine this is to encourage them to ask questions and express their
automobile accident. Which of the following feelings. Timelines for visitation are appropriate but of less priority than option 3.
interventions would have the highest priority?

1.‐ Teach the family the importance of using seatbelts.


2.‐ Refer the family to support services in the community.
3.‐ Encourage family to ask questions and express feelings.
4.‐ Explain rules for visiting in the intensive care unit.

1736 A client arrives at the Emergency Department Correct answer: 1 In a grade 1 concussion, the client exhibits transient confusion with no loss of consciousness The core issue of the question is the ability to correctly interpret signs of head injury
following head injury and is diagnosed with a and a duration of abnormal mental status for less than 15 minutes. Grades 2 and 3 concussion related to concussion. Use knowledge of the pathophysiology of concussion to determine
concussion. The client exhibits transient confusion with consist of more severe neurological symptoms, with increasing levels of loss of consciousness an answer.
no loss of consciousness and a duration of abnormal and more significant abnormalities of mental status.
mental status for less than 15 minutes. The nurse
concludes that this client’s symptoms are compatible
with a concussion of _____ grade? Write in a
numerical answer.

1737 The Emergency Department nurse is educating a Correct answer: 1, 2, 4 Using warning stickers like Mr. Yuk to teach the child to avoid these items will help to keep Knowledge of the ways to avoid childhood poisoning will help to choose the correct
parent on safety measures for a child who has been substances from being ingested. Using childproof caps will also inhibit the child from opening answers.
treated for accidental ingestion of acetaminophen medicine containers. An important aspect of teaching is to inform parents to contact the
(Tylenol). The nurse concludes that the parent has Poison Control Center or 911 for instructions on the appropriateness of care following
understood the teaching instructions when the parent ingestion of substances. Activated charcoal, magnesium citrate, and magnesium sulfate are not
makes which statements? Select all that apply. used in the home setting for treatment of poisonings.

1.‐ “I will use warning stickers like Mr. Yuk on all medicine containers.”
2.‐ “I will buy products with childproof caps.”
3.‐ “I will keep magnesium citrate available.”
4.‐ “I will put the Poison Control Center phone number by every phone in the house.”
5.‐ “I will keep activated charcoal in the house and use it readily if needed.”
1738 The family who has a child with a chronic health Correct answer: 2 All families deal with stresses, and the family with a child with a chronic health problem is no Knowledge of the response of the parents to developmental crisis from inability of the
problem such as spina bifida experiences "chronic exception. Chronic sorrow is the emotional experience many families experience of grieving child to achieve traditional milestones will aid in determining the correct answers.
sorrow" throughout the child's life. The nurse can for the loss of the perfect child. This grief is intensified at times of developmental crisis and
anticipate that this will be more prevalent when: each time in the child's development at traditional milestones such as "first steps" when the
parent is reminded of what their child will not be able to do.

1.‐ The child is admitted to the hospital for a planned procedure.


2.‐ The child reaches the age of a "developmental milestone" that the child cannot attain.
3.‐ The child attains independence by attending school.
4.‐ A sibling is born without any health problems.

1739 The nurse discusses the risk of aspiration with the Correct answer: 3 Toddlers chew well, but may have difficulty swallowing large pieces of food. Young children Considering the normal activities of a child of this age will lead to the right answer.
parents of an 18‐month‐old. The nurse recommends cannot discard pits (such as from cherries). Foods like peanuts and hard candies are easily
the parents avoid giving their child: aspirated.
1.‐ Oranges, crackers, and applesauce.
2.‐ Apples, fruit juice, and raisins.
3.‐ Cherries, peanuts, and hard candy.
4.‐ Cheerios, toast, and bananas.

1740 A newborn has been admitted to the unit with a Correct answer: 3, 5 Prior to surgical repair of the meningocele, leaking cerebrospinal fluid usually reduces the Three of the options related to increased intracranial pressure. One option, the correct
myelomeningocele. The nurse would include which of intracranial pressure. The priority concern preoperatively is maintaining the meningocele sac option, deals with the spinal defect.
the following as priority elements of care during the and preventing infection. It is not a priority to measure head circumference daily, use semi‐
preoperative period? Select all that apply. Fowler’s position, or monitor for irritability and vomiting (signs of increased intracranial
pressure).
1.‐ Measuring the head circumference on a daily basis
2.‐ Preventing increased intracranial pressure by laying the baby in semi‐Fowler's position
3.‐ Positioning the infant on his abdomen to protect the spinal defect
4.‐ Monitoring the child for signs of irritability and vomiting
5.‐ Covering the sac with a sterile saline dressing to protect its integrity.

1741 The parents of an infant who has just had a Correct answer: 2 The prognosis for children receiving shunts depends on brain damage that has already Knowledge of the pathophysiology of hydrocephalus will eliminate options 3 and 4. While
ventriculoperitoneal (VP) shunt inserted for occurred. The shunt will need to be revised as the child grows. Usually an extended length of option 1 is basically correct, option 2 expands the knowledge in as much as the need for the
hydrocephalus are concerned about the infant's tubing is placed with ventriculoperitoneal shunts to allow for some growth but eventually a shunt is permanent, the device itself will be replaced on an ongoing basis as the child
prognosis and ongoing care. The nurse should include revision is necessary. Most brain damage is not reversible. grows.
which of the following in discussions with the parents?

1.‐ The prognosis is excellent and the shunt is permanent


2.‐ The shunt will need to be revised as the child gets older
3.‐ During the first year of life, any brain damage that has occurred is reversible
4.‐ Hydrocephalus is usually self‐limiting by 2 years of age and the shunt will then be removed

1742 The nurse is teaching a class on health Correct answer: 1 Research studies have shown a significant decrease in incidence of spina bifida in infants born Each of the options describe known causative factors associated with a variety of
promotion/disease prevention to women of to mothers who took folic acid supplements prior to pregnancy and during the first trimester. conditions. Recall that folic acid has been associated with neural tubes defects to make the
childbearing age. The nurse stresses that one action Spina bifida is not related to rubella or rubeola. No relationship has been seen between correct selection.
that has been shown to be effective in reducing the maternal age and the development of spina bifida, nor is there a genetic trait that can be
incidence of spina bifida is: linked to spina bifida.
1.‐ Taking folic acid supplements or using fortified enriched grain products during pregnancy.
2.‐ Being immunized for rubella and rubeola.
3.‐ Avoiding pregnancy after the age of 45.
4.‐ Not having children with a man who also carries the spina bifida genetic trait.
1743 A 7‐year‐old child has just been diagnosed with a Correct answer: 1, 5 Common side effects with antiepileptic medications include ataxia and rashes, which This question requires specific knowledge about carbamazepine. Recall that many
seizure disorder and the physician has prescribed disappear when dosage is adjusted. Some drugs such as phenobarbital can adversely affect antiepileptic drugs have a side effect of drowsiness and dizziness. Eliminate hives because it
carbamazepine (Tegretol) 500 mg/day. The nurse cognitive function, school performance, and behavior. Carbamazepine is considered relatively would be an indication of an allergic reaction.
should teach the parents about which common side free of the sedative‐like side effects but does have the side effects of blurred vision, diplopia,
effects of this medication? Select all that apply. drowsiness, vertigo, headache, and rarely a rash (Stevens‐Johnson syndrome).

1.‐ Dizziness and headache


2.‐ Hives and aching joints
3.‐ Diaphoresis and vomiting
4.‐ Blurred vision and papular skin rash
5.‐ Drowsiness and vertigo

1744 Following surgery for the insertion of a Correct answer: 4 A serious complication after the insertion of shunt would be infection, most likely meningitis. With two options being decreased or increased intracranial pressure, these should be
ventriculoperitoneal (VP) shunt for hydrocephalus, the It is not common to see decreased intracranial pressure, but too‐rapid decrease can result in a evaluated first for the right answer. However, neither fits the symptoms listed in the stem.
infant demonstrated irritability, high‐pitched cry, subdural hematoma. Symptoms of shunt malfunction would include all signs of increased The temperature elevation should have been a clue to point toward infection.
elevated pulse rate, and temperature of 40 degrees C. intracranial pressure. Symptoms of infection such as meningitis in the young infant include
(104 degrees F.). The nurse concludes that these fever, poor feeding, vomiting, marked irritability, restlessness, seizures, and a high‐pitched cry.
symptoms are consistent with which postoperative
complication?

1.‐ Shunt obstruction


2.‐ Increased intracranial pressure
3.‐ Decreased intracranial pressure
4.‐ Infection

1745 During a well‐child visit for an 8‐month‐old girl, her Correct answer: 4 The best answer is to do further assessments of the child's abilities. At eight months, most Responses which deny the parents’ concerns or tell them not to worry are usually not
parents express concern that their older child was infants can sit without support; however a remarkable piece of history for this child is her correct. In addition, for an option to be correct, every part of that option must be correct.
already sitting alone at this age. The child was born six prematurity. Up until 2 years of age, it is important to remember to adjust for the weeks
weeks premature but had no major difficulties during premature to have more realistic milestones for this individual child. Additional assessment of
the neonatal period. The best response of the nurse to motor skills is important to determine developmental progress while accounting for
the parents is: prematurity. Motor impairments associated with voluntary control are not usually apparent
until after 2 to 4 months at the earliest so that motor dysfunction (and subsequent diagnosis of
cerebral palsy) may not be confirmed until the second half of the first year. It is not unusual for
the disorder to be overlooked in mildly affected infants until they exhibit a delay in some
advanced motor skill such as walking.

1.‐ "Your observations are good. Your child is demonstrating a developmental delay and probably has cerebral palsy."
2.‐ "You shouldn't jump to conclusions. All children are individuals, and it is not fair to compare one child to another."
3.‐ "You have nothing to worry about. Your child's development is completely normal."
4.‐ "Can you tell me more about your child? Is she sitting alone and turning over?"

1746 A 15‐year‐old wrestler who suffered a concussion Correct answer: 2 Since it may take up to a month for the brain to heal after a concussion and subsequent Options 1, 3, and 4 contain false information. Since the nurse is evaluating the parental
after being thrown on his head during a match was injuries can compound the original injury, the child should not engage in contact sports for a understanding of discharge information, these must be incorrect options.
seen in the emergency room for assessment and month. Post‐concussion syndrome in adolescents includes headache, dizziness, irritability, and
observation. After providing the parent with discharge impaired concentration. It can be helpful for teachers to understand any possible changes in
instructions about post‐concussion syndrome, the behavior or school performance during this period. Vomiting should be reported and is a
nurse determines the parents have adequate serious symptom of increasing intracranial pressure; but it is not treated with antiemetic.
understanding of the information if they state to: While monitoring the child on a regular basis for 24 to 28 hours post injury is recommended,
having him bend his head to his chest (Brudzinski's sign) would be a good indicator of
meningitis but will not provide helpful information about complications of brain injury.

1.‐ Closely observe the child when he participates in a wrestling meet tomorrow.
2.‐ Plan to speak with his teachers about the injury.
3.‐ Call the primary health care provider for an antiemetic prescription if he has any vomiting.
4.‐ Check him every four hours during the night and have him bend his head to his chest to check for pain.

1747 A 3‐month‐old infant has been admitted with a Correct answer: 3 While all other choices are important to monitor, the priority in assessing any critically ill Utilize the ABCs: Airway is always the first physical priority.
diagnosis of encephalitis. The first nursing priority child follows the ABC rule—airway, breathing, and circulation.
would be to assess:
1.‐ Pupillary reaction.
2.‐ Level of consciousness.
3.‐ Ability to maintain airway.
4.‐ Blood glucose level.

1748 The nurse places a young child scheduled for a lumbar Correct answer: 3 This position opens the intervertebral spaces and allows easier access to the spinal canal. The Visualize the procedure and positioning. Compare each option to the information in the
puncture in a side‐lying position with the head flexed position does not decrease pain or help to restrain the child. All lumbar punctures are done stem looking for an option that fits with the question. To choose correctly, recall that the
and knees drawn up to the chest. The mother asks why below L4 (the level of the spinal nerves), so injury to the spinal cord is always avoided. spinal vertebrae need to be separated to allow easier access by the spinal needle.
the child has to be positioned this way. The nurse
explains the rationale for the positioning is that:

1.‐ Pain is decreased through this comfort measure.


2.‐ Injury to the spinal cord is prevented.
3.‐ Access to the spinal fluid is facilitated.
4.‐ Restraint is needed to prevent unnecessary movement.

1749 An 18‐month‐old child is observed having a seizure. Correct answer: 4 It is important to never forcibly restrain a child during a seizure or insert a padded tongue Padded tongue blades and restraints are not used during a seizure, leaving two options.
The nurse notes that the child’s jaws are clamped. The blade; both are more likely to add trauma than prevent it. Oxygen via mask is of little benefit. Although airway is always the priority response, providing oxygen through a mask will not
priority nursing responsibility at this time would be: Overall, the child must be protected from injury from the environment. maintain oxygenation because the child may not be breathing or may not have an open
airway. Activities toward maintaining an open airway would be more important. That leaves
protecting the child from the environment as the only viable option.

1.‐ Start oxygen via mask.


2.‐ Insert padded tongue blade.
3.‐ Restrain child to prevent injury to soft tissue.
4.‐ Protect the child from harm from the environment.

1750 A 3‐year‐old child is admitted to the hospital unit with Correct answer: 1, 3, 4 Viral meningitis does not require antibiotics. Treatment is aimed at reducing the symptoms. Viral infections do not require antibiotics. Consider the age of the child to determine if
a diagnosis of viral meningitis. The nurse should take The child should be allowed to assume a position of comfort; the room should be kept dim and head measurement will provide any important data. Eliminate any options that would
which of the following actions in the care of this child? stimulation reduced. Seizures can occur, although the disease is usually self‐limiting. increase stimulation.
(Select all that apply.) Measuring the head circumference is of no benefit because the sutures are fused.

1.‐ Allow the child to assume a position of comfort.


2.‐ Keep the lights bright to monitor skin color.
3.‐ Administer acetaminophen for pain.
4.‐ Monitor the child for seizures.
5.‐ Administer antibiotics.

1751 The nurse is providing client education for a family Correct answer: 3 This therapy involves an implanted pump that must be accessed through the skin to refill the To determine the correct answer, first determine which responses are accurate and then
whose child has cerebral palsy and is receiving pump. Parents are not taught to refill the pump. Baclofen does inhibit the neurotransmitter prioritize the information that the parents will need.
baclofen epidural therapy to control spasticity. Which GABA; however, this is not the essential data to be shared with the parents. Promising the
of the following is most important for the nurse to parents that the child will be able to run with normal gait offers false hopes. The implanted
include in the discussion? pump’s dosage cannot be changed without special equipment.

1.‐ The drug acts to inhibit the neurotransmitter GABA.


2.‐ The child should be able to run with normal gait after insertion of the pump.
3.‐ Parents must bring the child back to the clinic on a regular basis to have more medicine added to the pump.
4.‐ Parents can be taught to regulate the dosage on a sliding scale.

1752 A 10‐year‐old client presents with weakness in the Correct answer: 3 Guillain‐Barré syndrome is an ascending paralysis. While the child will have increasingly less Consider which option can have serious implications. Knowledge of Guillain‐Barré as an
legs and history of the flu. The medical diagnosis is muscle tone in the extremities, the hoarseness could indicate involvement in the muscles of ascending paralysis would indicate that hoarseness would be an impending sign of difficulty.
Guillain‐Barré syndrome. It would be imperative for respiration. Serious concern is raised when the respiratory muscles are affected. Sometimes
the nurse to inform the physician after observing mechanical ventilation is indicated. Tingling is a common sign of Guillain‐Barré and not related
which of the following? to respiratory distress.
1.‐ Weak muscle tone in feet
2.‐ Weak muscle tone in legs
3.‐ Increasing hoarseness
4.‐ Tingling in the hands

1753 The nurse is providing discharge instructions for a Correct answer: 3 Discharge instructions will include the necessity of waking the child to check neurological Consider care of the client with a head injury and determine which response is not in line
child who has suffered a head injury within the last status throughout the night. Vomiting could be a sign of increasing intracranial pressure and with the normal care.
four hours. The nurse determines there is a need for should be reported. Narcotics are not given after a head injury. Amnesia for the events
additional teaching when the mother states: surrounding the injury may be permanent. It is not a sign of increasing intracranial pressure.

1.‐ “I will call my doctor immediately if my child starts vomiting.”


2.‐ “I won’t give my child anything stronger than Tylenol for headache.”
3.‐ “My child should sleep for at least 8 hours without arousing after we get home.”
4.‐ “I recognize that continued amnesia about the injury is not uncommon.”

1754 A 2‐year‐old child is admitted to the neurosurgical Correct answer: 4 No eye opening, no verbal response, and no motor response are the lowest criteria on the Options 2 and 3 can be eliminated as these are normal findings. Consider options 1 and 4
unit following a head injury. The nurse is using the scale. Confusion is a criterion applicable only for the older child and adult but is comparable to and determine which response shows the least brain functioning.
Glasgow Coma Scale to measure neurological “irritable and cries” for the infant (which is a 4 out of 5 on the verbal response subscale). “Eyes
functioning. Which of the following assessment open only to pain” is the next to the lowest level on the eye‐opening category.
findings indicates the lowest level of functioning for
this child?
1.‐ Confusion
2.‐ Irritable and cries
3.‐ Eyes open only to pain
4.‐ No response to painful stimuli

1755 Upon performing a physical assessment of a 7‐month‐ Correct answer: 3 The Moro or startle, tongue extrusion, and tonic neck reflexes are all neonatal reflexes that Differentiate normal from abnormal findings in a 7‐month‐old infant.
old child, the nurse notes the following findings. The should have disappeared by this child’s age. Lack of head lag indicates good motor
nurse concludes that which finding is abnormal and development. A developmental delay or the presence of a neonatal reflex are some of the
could suggest cerebral palsy? earliest clues to cerebral palsy.
1.‐ No head lag when pulled to a sitting position
2.‐ No Moro or startle reflex
3.‐ Positive tonic neck reflex
4.‐ Absence of tongue extrusion

1756 A 4‐year‐old child is being evaluated for Correct answer: 4 All of the above are symptoms of increased ICP or hydrocephalus. Head enlargement and The key concept here is the age of the child. At four years old, the sutures and fontanels
hydrocephalus. The nurse notes which of the following bulging fontanels would not be seen in the child after closure of the sutures (12 to 18 months). have closed, eliminating two of the four options.
as an early sign of hydrocephalus in this child? Shrill, high‐pitched cry is a late‐stage symptom of children. Headache and vomiting on arising
would be an early symptom in an older child.

1.‐ Bulging fontanels


2.‐ Rapid enlargement of the head
3.‐ Shrill, high‐pitched cry
4.‐ Early morning headache

1757 A child with a history of a seizure was admitted 2 Correct answer: 2 Brudzinski’s sign indicates meningeal irritation. As the head and neck are flexed toward the Consider how Brudzinski’s sign is tested. Flexing the neck is not likely to affect things going
hours ago. The history reports fever, chills, and chest, the legs flex at both the hips and the knees in response. Brudzinski’s sign may be seen in on in the skull, so the most likely response is 2.
vomiting for the past 24 hours. In report, the nurse is the other options because of the meningeal irritation.
told that the child has a positive Brudzinski’s sign. The
nurse knows that this is most likely caused by:

1.‐ Increased intracranial pressure.


2.‐ Meningeal irritation.
3.‐ Encephalitis.
4.‐ Intraventricular hemorrhage.

1758 A nurse is assessing a new admission. The 6‐month‐ Correct answer: 1 Increased intracranial pressure in infants is characterized by lethargy, irritability, bradycardia, The bulging fontanels are a strong indication of increased intracranial pressure. The other
old infant displays irritability, bulging fontanels, and tachycardia, apnea, bulging fontanels, setting‐sun eyes, vomiting, and hypertension. options do not have this as a symptom.
setting‐sun eyes. The nurse would suspect: Myelomeningocele refers to a neural tube defect, which is obvious on the back. Skull fractures
indicate injury to the head and may be asymptomatic or may be accompanied by other
pathology that could lead to increased intracranial pressure. Hypertension does not display
symptoms of setting‐sun eyes.
1.‐ Increased intracranial pressure.
2.‐ Hypertension.
3.‐ Skull fracture.
4.‐ Myelomeningocele.

1759 An 8‐year‐old client with a ventriculoperitoneal shunt Correct answer: 3 The most common mechanisms for the development of hydrocephalus include decreased Knowledge of shunt function would indicate that a malfunctioning shunt would obstruct
was admitted for shunt malfunction. He presents with reabsorption (communicating hydrocephalus) and obstruction to the flow of CSF flow of CSF.
symptoms of increased intracranial pressure. The (noncommunicating). Obstruction may result from congenital anomalies, inflammation,
mechanism of the development of his symptoms is external blockage, and other causes.
most probably related to:
1.‐ Increased flow of cerebrospinal fluid.
2.‐ Increased reabsorption of cerebrospinal fluid.
3.‐ Obstructed flow of cerebrospinal fluid.
4.‐ Decreased production of cerebrospinal fluid.

1760 A child with a myelomeningocele is started on a Correct answer: 2 Most children with spina bifida cystica (myelomeningocele included) have the level of their Determine if both a lack of mobility and innervation exist. Since they both exist in the child
bowel management plan. The child’s mother questions defect at a point that does affect the innervation to both the colon and anal sphincter. The with this spinal defect, consider which would have the most effect on bowel function. Once
why this is being done. The nurse’s response will be result is constipation and incontinence. Any lack of mobility increases the risk for constipation, it is determined that the major problem is lack of innervation, the learner must choose
based on the understanding that lack of: and all children need a pattern of regular bowel movements. whether this is likely to cause diarrhea or incontinence.

1.‐ Innervation to the colon predisposes the child to diarrhea.


2.‐ Innervation to the anal sphincter predisposes the child to being incontinent.
3.‐ Mobility increases the gastric‐colic reflex.
4.‐ Mobility decreases the need for regular bowel movements.

1761 A child has just been diagnosed with bacterial Correct answer: 3 Clients are considered contagious until the causative organism is determined and antibiotic Three of the four choices relate to antibiotics, so the correct response is likely to be one of
meningitis. The parent asks the nurse how long the therapy has been initiated. Children are usually placed in respiratory or droplet isolation. them. Choose the time frame that is not immediate and not too lengthy.
child will be in isolation. The nurse’s reply will be based Twenty‐four hours of antibiotic therapy usually eliminates the necessity of isolation.
on a protocol that isolation continues until:

1.‐ The organism is located.


2.‐ The antibiotics are initiated.
3.‐ The antibiotics have been administered for 24 hours.
4.‐ Ten days of antibiotic therapy have been completed.

1762 The nurse observes a client with the neck and back Correct answer: 4 The child with meningitis will hyperextend the neck and head in an arching position referred The learner must be able to distinguish between the positioning seen in neurological
arched and extremities severely extended. The mother to as opisthotonic. The child does this to relieve discomfort from the meningeal irritation. defects.
asks why the child is doing that. The nurse explains Decerebrate posturing is a symptom of dysfunction at the level of the midbrain and is
that this posturing is called: characterized by rigid extension and pronation of arms and legs. Decorticate posturing is a
symptom of a dysfunction of the cerebral cortex and is characterized by adduction of the arms
at the shoulders, the arms flexed on the chest with hands in fists and wrists flexed, and lower
extremities extended and adducted. Jacksonian seizure is a simple motor seizure characterized
by clonic movements that begin in a foot, hand, or face and then spread to sometimes include
the entire body.

1.‐ Decerebrate.
2.‐ Decorticate.
3.‐ Jacksonian seizure.
4.‐ Opisthotonos.

1763 A child is being treated for increased intracranial Correct answer: 1, 2, 4 Turning the head to one side can occlude the flow of CSF, increasing the ICP. Oxygen can Eliminate any activities that would increase intracranial pressure such as emotional upset,
pressure (ICP). Appropriate actions to decrease serve as a vasodilator, decreasing the ICP. Keeping the head of the bed slightly elevated also vigorous exercise, and increased fluid intake.
intracranial pressure would include: (Select all that promotes flow of CSF. Diuretics are often part of the medical treatment to decrease ICP.
apply.) Vigorous range of motion and forcing oral fluids would not be appropriate.

1.‐ Keeping head of bed at a 30‐degree angle.


2.‐ Providing supplemental oxygen.
3.‐ Turning head to one side.
4.‐ Administering IV osmotic diuretics as ordered.
5.‐ Promoting fluid intake.

1764 A 10‐year‐old boy receives a blow to his head with a Correct answer: 1 Epidural hematomas are characterized by arterial bleeding. Onset of symptoms occurs within Recall that an epidural bleed is arterial, which would mean symptoms would appear
hard baseball and is admitted to the hospital for minutes to hours. Other types of bleeding are often venous, which have a slower onset of quickly.
observation. If the child were to develop an epidural symptoms.
hematoma, the child would most like display
symptoms:
1.‐ In the emergency room or soon after arriving on the unit.
2.‐ On the unit over the next few days.
3.‐ After discharge home.
4.‐ Over the next two months.

1765 A 15‐year‐old client is seen in the emergency Correct answer: 3 Obtunded indicates a diminished level of consciousness with limited response to the Recognize that this child is not fully conscious. At the same time, he is not in a stupor or
department following a head injury from football. environment. The child will fall asleep unless given verbal or tactile stimulation. Stupor is a semicomatose. Then choose between lethargic and obtunded based on knowledge of levels
During the first few hours after admission, he sleeps diminished level of consciousness with response only to vigorous stimulation. Semicomatose is of consciousness.
unless awakened, but he can be aroused easily and is when a child only responds to painful stimuli; lethargy is when a child sleeps if left undisturbed
oriented. In charting assessment findings, the nurse and has sluggish speech and movement.
would describe this level of consciousness as:

1.‐ Semicomatose.
2.‐ Lethargic.
3.‐ Obtunded.
4.‐ Stuporous.
1766 A young child has just been diagnosed with spastic Correct answer: 2 The most common form of cerebral palsy involves spasticity of muscles. Because of the The child with spasticity has hypertonic muscles, so option 3 can be eliminated. The spastic
cerebral palsy. The nurse is teaching the parents how excessive energy expended, these children often need more calories than other children their muscles use extra energy to stay in that hyperactive state.
to meet the dietary needs of their child. The nurse age and size. Feeding difficulties are often a component of cerebral palsy, but whether a child
would explain that children with cerebral palsy needs assistance with feedings is dependent upon the muscle groups affected.
frequently have special dietary needs or feeding
challenges because:
1.‐ The paralysis of their muscles decreases their caloric need.
2.‐ The spasticity of their muscles increases their caloric need.
3.‐ The hypotonic muscles make eating difficult.
4.‐ The child’s inactivity increases the risk of obesity.

1767 The priority in preparing the room for a client with a Correct answer: 3 Although a ventilator is not required for injury below C3, the innervation of intercostal Recall that protecting respiratory function is the most important issue for the spinal cord
C7 level spinal cord injury is having: muscles is affected. Hemorrhage and cord swelling extends the level of injury, making it likely injured client.
that this client will need a ventilator.
1.‐ The special kinetic bed.
2.‐ The halo brace device.
3.‐ A ventilator on stand‐by.
4.‐ A catheterization tray.

1768 Upon return of the closed‐head injury client from CT Correct answer: 2 Keeping the head of the bed elevated to 30 degrees promotes venous drainage, which is Recall that appropriate drainage is enhanced by gravity. Keeping the head of the bed
scan, the head of the bed is maintained at 30 degrees important in decreasing ICP. Alignment of the head prevents obstruction of the jugular veins. elevated will promote drainage.
and the client is positioned on his side with a towel roll Obstruction would impede venous drainage.
placed vertically under the pillow. This unique
positioning facilitates:
1.‐ Prevention of pulmonary embolism.
2.‐ Venous drainage from the brain.
3.‐ Airway management.
4.‐ Intracranial pressure (ICP) readings.

1769 After regaining consciousness, a client reports a Correct answer: 1 Momentary loss of consciousness followed by a lucid period and rapid deterioration is a Recall that head injury would be most closely related to epidural hematoma development.
tremendous headache as he was taken from the site of classic picture resulting from a torn cerebral artery, producing an epidural bleed.
a motorcycle crash by ambulance. The wife is
unprepared when arriving at the hospital to find he
had become comatose. The nurse explains the cause
as which of the following?
1.‐ An expanding epidural hematoma
2.‐ A reticular activating system concussion
3.‐ A diffuse axonal injury
4.‐ An expanding pericardial hematoma

1770 Following a grand mal seizure, the client is Correct answer: 4 The period after the clonic phase of a seizure is the postictal period. Typically, the client Recall that sleepiness following a seizure is expected. Select the option that takes this into
unconscious and unresponsive when the nurse tries to slowly regains consciousness, moving from a relaxed, quiet state to confusion or disorientation account.
awaken the client. The nurse takes which of the on awakening.
following actions?
1.‐ Calls a code
2.‐ Notifies the physician
3.‐ Reduces the Dilantin
4.‐ Allows gradual awakening
1771 The nurse assesses the client's understanding of Correct answer: 1 Protective pads/diapers should be used only after all other treatment modes have been tried. Select the option that refers to a situation that does not address expected outcome of
discharge needs and goals after experiencing a Early dependency on incontinence products may decrease motivation to seek evaluation and CVA.
cerebral vascular accident (CVA). Which of the treatment.
following statements indicates further information and
teaching will be necessary?
1.‐ "I'm getting a lifetime supply of adult diapers."
2.‐ "I worry about those scary transient ischemic attacks."
3.‐ "I've got to find a walking buddy."
4.‐ "I'm getting a rail installed in my tub."

1772 The major focus for nursing care of the client with Correct answer: 3 Clients with meningitis will be less able to protect themselves from both internal and external Omit options that would put added stress on the client, such as options 2 and 4. Focus of
meningitis will be to: injury. Providing cognitive stimulation and increasing cardiac output are contraindicated with care is on physiological protection.
meningitis. Enhancing coping skills may be a focus if the client has residual effects from
meningitis but it is not a major focus.
1.‐ Enhance coping skills.
2.‐ Provide cognitive stimulation.
3.‐ Assess risk for injury and prevent complications.
4.‐ Increase cardiac output.

1773 The client comes to the Emergency Department with Correct answer: 1 Pre‐syndrome to Guillain‐Barré syndrome is often a viral infection or immunization. Lower Select the response that addresses the cause of Guillain‐Barré, which is viral infection.
weakness that has been progressing upward in both extremity weakness or paralysis that progresses upward is classic in Guillain‐Barré. Fatigue is
legs for a couple of days. The nurse, suspecting Guillain‐ not usually seen, nor are tremors or seizures.
Barré syndrome, begins care by:

1.‐ Taking medical history, noting recent viral influenza.


2.‐ Giving the client orange juice for fatigue and low blood sugar.
3.‐ Instructing on tests for myasthenia gravis
4.‐ Evaluating for petit mal seizures.

1774 The teaching plan for a client with myasthenia gravis Correct answer: 3 The client should know the signs of crisis and should report them immediately. There is often Recall that the priority of care is to recognize and treat crisis.
should include which of the following as a priority more fatigue and weakness later in the day than in the morning, so the client should plan
instruction? important activities for early in the day. It may be easier to eat three small meals with snacks
because chewing may cause fatigue.
1.‐ Exercise to increase peripheral circulation
2.‐ Plan important activities for late afternoon
3.‐ Identify signs of an action during crisis
4.‐ Eat three well‐balanced meals a day

1775 The client with Parkinson's disease finds the resting Correct answer: 3 The resting or nonintentional tremor may be controlled with purposeful movement, such as Choose the option that addresses the non‐intentional nature of Parkinsonian tremor.
tremor he is experiencing in his right hand very holding an object. Deep breathing, a warm bath, and diazepam will promote relaxation but are
frustrating. The nurse advises him to: not specific interventions for the tremor.
1.‐ Practice deep breathing.
2.‐ Take a warm bath.
3.‐ Hold an object in that hand.
4.‐ Take diazepam (Valium) as needed.
1776 A client on the first day after abdominal surgery ranks Correct answer: 2 Always believe the client's report and ranking of pain. The client tolerated the last full dose of Omit options 1 and 4 as they do not address the client’s complaint of pain. Option 3 is not
his pain as 9 on a scale of 0 to 10, 0 being no pain and medication so he should be given a full dose now. The nurse would reassess the client's pain appropriate as the client’s self assessment of his pain level is high.
10 being the most pain possible. He is laughing and level within 30 minutes of administering pain medication depending on peak action time of the
talking with visitors at this time. The client has defined drug. The nurse would not need to take vital signs unless indicated.
a pain level of 5 as his comfort level goal. He has an
order for analgesics every 1 to 2 hours. His last dose
was the maximum ordered 2 hours ago. He had no
untoward effects. The nurse should do which of the
following?

1.‐ Record his pain level at 5


2.‐ Administer the maximum dose of the analgesic
3.‐ Administer half the maximum dose of the analgesic
4.‐ Take his vital signs in 30 minutes

1777 A young female with a diagnosis of multiple sclerosis Correct answer: 4 Stresses such as pregnancy can increase the chance of an exacerbation of MS. Signs of an Omit options 1, 2, and 3 as not indicative of MS exacerbation.
(MS) has just discovered she is also pregnant. She and exacerbation are spasticity, weakness, or visual changes. Option 1 indicates Guillain‐Barré
her husband have been trying to conceive for several syndrome; option 2 indicates increased intracranial pressure or hematoma; option 3 indicates
years and are very excited. What client education meningitis.
should be given to the client concerning her disease
and pregnancy?

1.‐ Immediately report weakness that progresses upward in the extremities.


2.‐ Report a change in level of consciousness, severe headache, or slow pulse.
3.‐ Recognize that a sore/stiff neck and pain when the neck is flexed are critical signs to report.
4.‐ Immediately report spasticity, weakness, or visual changes.

1778 A client has seizure activity that is continuous in Correct answer: 2 When seizure activity becomes continuous and repetitive, respirations are affected and the Recognize that the word “epilepticus” in option 2 is a hint.
nature. When the nurse leaves to obtain an oxygen set‐ progression of status epilepticus is life‐threatening. Nursing interventions address prevention
up, the seizures resume. The client's coloring is getting of hypoxia, acidosis, hypoglycemia, hyperthermia, and exhaustion.
worse. The nurse decides to enact standing orders to
treat:
1.‐ Anticonvulsant syndrome.
2.‐ Status epilepticus.
3.‐ Brain herniation.
4.‐ Syphilitic posturing.

1779 A client has recently completed chemotherapy and Correct answer: 3 Most chemotherapeutic agents cause some degree of bone marrow suppression. This results Identify what is produced by the bone marrow. Select the option that is consistent with
has developed bone marrow suppression. Which in a decrease in leukocyte and erythrocyte counts. Calcium, phosphorus, and serum PSA levels this.
laboratory report should the nurse monitor? are not specifically affected by bone marrow suppression.

1.‐ Calcium
2.‐ Phosphorus
3.‐ White blood cell (WBC) count
4.‐ Serum prostate‐specific antigen (PSA)

1780 After a magnetic resonance imaging (MRI) to rule out Correct answer: 3 The spinal needle is inserted into the area below the spinal cord, eliminating the likelihood of Look for the option that addresses the most common complication of this procedure,
an expanding brain lesion, the nurse prepares the paralysis. A misdirected needle may puncture a distended bladder. The client must maintain headache.
client for a lumbar puncture. Information for the client the knees to chest position until completion of the lumbar puncture. Because cerebrospinal
would include: fluid (CSF) has been removed during the lumbar puncture, time must be allowed for
production and replacement of the CSF.
1.‐ The risk of paralysis because the needle is inserted through the spinal cord.
2.‐ Maintaining a full bladder for better visualization.
3.‐ Keeping his head flat to avoid a headache.
4.‐ Directions to straighten the legs slowly.

1781 The nurse at the community clinic assesses a client Correct answer: 4 Spinal cord injury at or above the level of T6 can experience an exaggerated sympathetic Select the response that is related to the injury, but can occur after an extended period of
with complaints of a pounding headache. The client is response, seen only after recovery from spinal shock. If untreated, autonomic dysreflexia is time. Sinus infection is not related to the injury.
known to have a T4 level spinal cord injury that potentially fatal as bradycardia and severe hypertension progress.
occurred some time ago. The nurse should first assess
for:
1.‐ Sinus infection.
2.‐ Spinal shock.
3.‐ Upper motor neuron deficit.
4.‐ Autonomic dysreflexia.

1782 Corneal abrasion in the client with myasthenia gravis Correct answer: 2 Corneal abrasion in the client with myasthenia gravis is caused by dryness of the cornea from Recognize that keeping the eye moistened by frequently administering artificial tears is the
can be prevented by which of the following? inability to close the eyelids and blink. It can be prevented by application of artificial tears key to the correct response.
every 1 to 2 hours.
1.‐ Saline soaks applied every shift
2.‐ Instilling artificial tears in the eyes every 1 to 2 hours
3.‐ Ensuring the client's contact lenses are on while awake
4.‐ Providing sunglasses when client is outside

1783 To reduce risks for meningitis, the nurse's best advice Correct answer: 2 Meningitis bacteria or viruses often gain entry into the cerebrospinal fluid secondary to an Option 2 is the only one that addresses a specific method of preventing infection.
to an elderly client is: upper respiratory tract infection. Options 1, 3, and 4 are generally healthy practices for the
elderly client but not specific health promotion for prevention of meningitis.

1.‐ Stay mentally active and engaged.


2.‐ Get pneumococcal pneumonia and influenza vaccinations.
3.‐ Drink at least 3,000 mL fluids per day.
4.‐ Exercise 15 to 30 minutes most days of the week.

1784 The nurse is providing care for a client with Guillain‐ Correct answer: 3 Plasmapheresis is performed to remove autoantibodies that attack the myelin sheaths of Eliminate option 1 as it is an infectious process.
Barré syndrome who has plasmapheresis. Which other motor and sensory nerves in Guillain‐Barré and attack the acetylcholine receptors at the
neurological condition is sometimes treated with neuromuscular junction in myasthenia gravis. The other diseases have not been identified with
plasmapheresis? autoimmune disorder.

1.‐ Meningitis
2.‐ Parkinson's disease
3.‐ Myasthenia gravis
4.‐ Amyotrophic lateral sclerosis

1785 The nurse is caring for a client who is paralyzed and Correct answer: 4 PCV is used for those clients who are unable to initiate spontaneous inspiration. SIMV and Review the types of mechanical ventilation. Select the option that best represents the
requires mechanical ventilation. The client is CPAP are primarily weaning modalities. ACMV is used when the client has some inspiratory client who is unable to initiate a breath and trigger the ventilator.
unresponsive and has no spontaneous ventilatory effort.
effort at the present time. What type of ventilation
should the nurse expect the physician to order?

1.‐ Synchronized intermittent mandatory ventilation (SIMV)


2.‐ Continuous positive airway pressure (CPAP)
3.‐ Assist‐control mode ventilation (ACMV)
4.‐ Pressure‐control ventilation (PCV)
1786 The client's postoperative pain seems to be getting Correct answer: 4 The client's pain is affective as well as sensory. Grieving his wife's death is a normal response Recognize that the scenario refers to a recent loss.
worse instead of better. When the nurse asks the that does not necessarily require psychiatric consult. Options 1 and 2 address the sensory, not
client, "Why do you think it's getting worse?" the client the affective component of his pain.
replies, "My wife died last month. It's all I can think
about." The nurse must now consider:

1.‐ Calling the physician for an increased dosage of pain medication.


2.‐ Calling the physician for a sedative.
3.‐ Referring the client for a psychiatric consult.
4.‐ Developing interventions for grief and loss.

1787 When assessing the client with meningitis, the nurse Correct answer: 2 The first signs of increased intracranial pressure are often subtle changes in level of Recall that change in affect or mood should be associated with early symptoms of IICP.
knows that the first sign of increased intracranial consciousness. Other changes (including rising systolic BP, irregular respiratory rate, and
pressure is often: bounding pulse) come later as intracranial pressure rises more.
1.‐ A rising systolic blood pressure.
2.‐ Change in mood or attention level.
3.‐ Irregular respiratory rate and depth.
4.‐ A bounding radial pulse.

1788 The nurse is instructing the client who has been in the Correct answer: 1 It is essential that the client recovering from meningitis take all of the prescribed antibiotic as Option 1 discusses the appropriate use of antimicrobials, which is key in preventing
hospital with meningitis and will be going home soon. directed. Failure to do so puts the client at risk for a relapse of symptoms and contributes to treating this infection.
Which of the following instructions will be of the development of bacterial resistance to antibiotics. Options 2, 3, and 4 are important aspects of
highest priority? self‐care during recuperation but not as essential as the completion of antimicrobial therapy.

1.‐ Take all of the antibiotics as directed until completely gone.


2.‐ Eat a high‐protein, high‐calorie diet.
3.‐ Exercise daily beginning with active range‐of‐motion.
4.‐ Get at least eight hours of sleep per night with frequent rest periods.

1789 When assessing the client with chronic pain, the Correct answer: 2 Chronic pain is multidimensional, often without an identifiable cause and not responsive to Recognize that the question is asking about planning care. Reference to interruption of
nurse acquires essential data for planning care when conventional treatment. By asking how/if the pain interferes with the client's daily activities, daily life and activities is a hint to the correct response.
asking the client to quantify: the nurse will obtain information about the impact the pain has on the person's quality of life.

1.‐ The severity of the pain using a 1 to 10 scale.


2.‐ How the pain affects the client's daily life.
3.‐ What the client thinks is causing the pain.
4.‐ What the client does to relieve the pain.

1790 The nurse teaches the client with multiple sclerosis Correct answer: 4 Urinary retention in the client with multiple sclerosis is a sequelae of impaired conduction of Select the option that will directly assist the client in emptying the bladder.
who has urinary retention to: nerves innervating the bladder. The client with multiple sclerosis will be encouraged to
increase her fluid intake to prevent constipation. Urinary retention is incomplete emptying of
the bladder. Neither running water nor caffeinated beverages would be useful.

1.‐ Run water when experiencing difficulty initiating urination.


2.‐ Decrease fluid intake to prevent urgency.
3.‐ Drink a caffeinated beverage to promote diuresis.
4.‐ Perform self‐intermittent catheterization.

1791 While obtaining the health history from the elderly Correct answer: 4 The nurse interviewing the client will promote client independence, communication, and self‐ Select the response that directs the family member to allow the client to respond.
client with Parkinson's disease, the client's daughter esteem by talking directly to the client and patiently and carefully listening to him.
continually answers all questions directed to the client.
The nurse:
1.‐ Accepts this as helpful because the client's voice is low, monotone, and difficult to understand.
2.‐ Stops the interview and resumes it later when the daughter is not there.
3.‐ Allows the daughter to complete the interview form.
4.‐ Respectfully tells the daughter, "I need to hear what your father has to say."

1792 The nurse advises the family of the client with Correct answer: 3 While options 1, 2, and 4 are all appropriate interventions for the client with Parkinson's The question is asking about functional independence. Select the answer that encourages
Parkinson's disease that the best approach to helping disease, the essential approach to enhance and encourage self‐care abilities will be an the client to perform activities at his/her own pace.
the client maintain as much functional independence unhurried one that allows sufficient time for self‐expression and for the client to do as much as
as possible will be to: possible for himself.
1.‐ Assist the client to take a warm bath every morning.
2.‐ Perform passive range‐of‐motion three times a day.
3.‐ Maintain an unhurried manner that allows the client sufficient time.
4.‐ Obtain assistive devices that will make activities of daily living easier.

1793 The nurse receives report from the nurse caring for Correct answer: 2 A positive Babinski is an indicator of upper motor neuron disease of the pyramidal tract. The Recall that a positive Babinski in an adult is always a grave sign and should be reported
an elderly client following transfer to the unit with a physician must be contacted immediately when a client's reflex changes from a negative to a immediately.
closed head injury. Which statement, if made by the positive Babinski as this reflects increased intracranial pressure.
nurse, would indicate negligence?
1.‐ "The client was medicated with Tylenol for a headache an hour ago."
2.‐ "The client began to have a positive Babinski an hour ago."
3.‐ "The client had the head of the bed at 30 degrees the entire shift."
4.‐ "The client had a positive bulbocavernous reflex the entire shift."

1794 The sister of a 76‐year‐old client with increasing Correct answer: 3 Increasing intracranial pressure is aggravated by hypercarbia and suctioning should not be Omit options 1 and 4 as they refer to extra ”air” as opposed to extra oxygen. Omit option
intracranial pressure asks why the nurse uses the extra done for longer than 10 seconds. The other three options are not appropriate. 2 as bradykinesia is not the reason for increasing oxygen.
oxygen when suctioning. Which of the following
statements is most correct?
1.‐ "Your brother seems to pink‐up when we give him extra air."
2.‐ "The extra oxygen helps relieve the bradykinesia by decreasing the carbon dioxide."
3.‐ "The extra oxygen helps relieve the pressure in his brain by decreasing the carbon dioxide."
4.‐ "Your brother seems less congested when we give him extra air."

1795 During the insertion of a Swan‐Ganz catheter, the Correct answer: 3 The anatomical proximity of the apex of the lung and the subclavian vein increases the The principle being tested is that with any insertion of a central venous catheter there is a
client complains of shortness of breath and possibility of the development of a pneumothorax caused by accidental puncture of the lung. risk of collapsing the lung from a puncture during insertion. Select the option that is
assessment finds labored respirations, decreased The client was demonstrating classic signs of pneumothorax. Pulmonary embolism, myocardial consistent with this complication.
breath sounds on the side of the insertion, and infarction, and anxiety do not cause asymmetrical chest movement.
asymmetrical chest movement. The nurse should
suspect:
1.‐ Pulmonary embolism.
2.‐ Myocardial infarction.
3.‐ Pneumothorax.
4.‐ Anxiety.

1796 Given her deficits, which of the following is an Correct answer: 1 Mechanical ventilation would not be necessary with a thoracic injury at T8. Options 2, 3, and Option 1 is the only response that reflects a disability above the level of the injury.
unrealistic statement offered by a 16‐year‐old with a 4 are all applicable for a T8 injury.
T8 spinal cord injury concerning an expected outcome?

1.‐ "I know this means being on a ventilator the rest of my life."
2.‐ "I know this means my legs won't work like before."
3.‐ "I know this means using a catheter for a while."
4.‐ "I know this means an adaptation to my car before I can drive again."
1797 The office nurse should encourage a client on the Correct answer: 1 What the client describes is a classic ascending progression of Guillain‐Barré syndrome. The Note that there is a reference to difficulty breathing in option 1, which makes it the most
phone to go directly to the Emergency Department muscular weakness may ascend to include the diaphragm. Total respiratory paralysis can urgent.
based on which statement? occur, requiring ventilatory support.
1.‐ "My legs are weak, and now I'm having trouble getting a good breath."
2.‐ "My shaky hand is no better than last visit."
3.‐ "My diplopia changed when I put my eye patch on."
4.‐ "My headache doesn't seem any better even though I gave up coffee."

1798 A 76‐year‐old woman comes to the Emergency Correct answer: 3 A Dilantin level would not be relevant to a stroke status. The nurse would want to avoid Note that the change in vital signs, specifically blood pressure, indicate that fluid volume
Department by ambulance because of a possible adding to the client's volume status or contributing to increasing intracranial pressure (IICP) should be controlled.
stroke. Vital signs are pulse 90, blood pressure since a confirmed diagnosis has not been made.
150/100, and respirations 20. Thirty minutes later, vital
signs are pulse 78, blood pressure 170/90, respirations
24 and irregular. The nurse should first:

1.‐ Ask the woman to describe how she's feeling.


2.‐ Check the client's phenytoin (Dilantin) level.
3.‐ Decrease the rate of intravenous (IV) fluids.
4.‐ Offer the client clear liquids to prevent dehydration.

1799 A client seen in the neighborhood clinic complains of Correct answer: 4 A more detailed assessment is important in collecting data to meet client needs. The picture Select the response that focuses the conversation, yet encourages the client to give
"eye problems" and generalized weakness that of multiple sclerosis may be unfolding. The nurse takes the time to be therapeutic, without information on the important symptom.
became markedly worse after visiting with a friend and providing false reassurance or limiting responses. Open‐ended, nonjudgmental responses are
frequently using the hot tub. The client is going into ideal.
considerably long, detailed answers as you take the
history. Your response is:

1.‐ "This is important information but there's just not enough staffing now."
2.‐ "You'll feel better after getting this all off your mind."
3.‐ "Please be brief in your answers so I can get you through this."
4.‐ "Tell me more about the eye problems."

1800 The nurse has assessed a client and suspect that the Correct answer: 4 Tracheal and mediastinal shifting will occur to the uninjured side because of increased This question requires an understanding of the pathophysiology of a tension
client has developed a tension pneumothorax. Which intrathoracic pressure on the side of the injury (option 4). The client will exhibit hypotension pneumothorax. Apply this pathophysiology to each distractor to determine the correct
of the following assessment findings would support caused by the decreased cardiac output (option 2), respiratory acidosis caused by hypoxemia response.
this conclusion? (option 3), and tachypnea (option 1). Mediastinal shift is always a late sign and requires
immediate treatment to relieve the buildup of trapped air to prevent death.

1.‐ Decreased tidal volume and normal respiratory rate


2.‐ Hypertension
3.‐ Respiratory alkalosis and hypoxemia
4.‐ Mediastinal shift toward uninjured side

1801 The nurse provides care for a 13‐year‐old in a halo Correct answer: 2 After assessing the pin sites for redness, edema, and drainage, a sterile applicator dipped in Recognize that limiting risk of infection is the highest priority.
brace because of a spinal cord injury. Which of the hydrogen peroxide is used around each pin site. This may be followed by normal saline and a
following is the first priority? topical antibiotic per hospital policy.
1.‐ Tighten any loose connections on the vest.
2.‐ Assess the pin sites.
3.‐ Turn the client every hour.
4.‐ Provide range of motion to extremities.
1802 The rehabilitation nurse discussing the client's Correct answer: 4 Hemisection of the anterior and posterior portions of the spinal cord result in loss of position They key is identifying the response that refers to same side deficit, below the lesion.
transition to home may note which of the following sense (proprioception) on the same side of the body as the trauma, below the level of injury.
characteristics in a client with Brown‐Séquard's Option 3 is seen in anterior cord syndrome; option 1 is incorrect.
syndrome?
1.‐ Ipsilateral motor loss above the lesion
2.‐ Contralateral loss of proprioception
3.‐ Hyperaresthesia below the level of the lesion
4.‐ Ipsilateral motor loss below lesion

1803 While assessing airway and breathing, the client Correct answer: 4 The brain stem's final effort to maintain cerebral perfusion is seen with an increased systolic Recall that wide pulse pressure combined with bradycardia is a hallmark of Cushing’s
presenting with increasing intracranial pressure (IICP) blood pressure, bradycardia, and an irregular respiratory pattern know as Cushing's response. Triad.
Cushing's triad presents with:
1.‐ BP 190/84, HR 150, and an irregular respiratory pattern.
2.‐ BP 80/50, HR 50, and Kussmaul respirations.
3.‐ BP 80/50, HR 150, and Cheyne‐Stokes respirations.
4.‐ BP 190/84, HR 50, and an irregular respiratory pattern.

1804 In providing for the safety of the client during a grand Correct answer: 3 To promote drainage, it is more effective to secure an airway by turning the client on the Eliminate any option that calls for direct contact with the client during a seizure.
mal seizure, the nurse performs which of the side. Inserting a tongue blade can cause trauma; the tongue blade may move during the
following? seizure and obstruct the airway. Oxygen should be available but does not have to be applied.

1.‐ Positions the client on his back.


2.‐ Gently places a padded tongue blade between the teeth.
3.‐ Protects the client from injury with padded bedrails.
4.‐ Applies oxygen immediately per mask.

1805 The nurse has completed the health history for a new Correct answer: 1 Inspection is the first diagnostic technique utilized in a physical assessment . Percussion is Review the best order for how to perform a physical assessment.
client and is ready to begin a physical assessment. next followed by palpation and finally auscultation.
What assessment technique will the nurse describe to
the client that will be completed first?

1.‐ Inspection
2.‐ Percussion
3.‐ Palpation
4.‐ Auscultation

1806 While caring for a client undergoing initial warming Correct answer: 3 During re‐warming, the client is at high risk for development of tissue ischemia, seizures, and During re‐warming, electrolytes leave the cell and enter the serum. Select the distractor
for hypothermia, the most appropriate nursing cardiac arrhythmias. Infection, pain, and knowledge deficit about the plan of care and that represents the greatest risk to the client’s life.
diagnosis would be: prognosis are lower priorities.
1.‐ Deficient knowledge.
2.‐ Risk of infection.
3.‐ Risk for injury.
4.‐ Pain.

1807 The nurse caring for the client with myasthenia gravis Correct answer: 2 When the muscles involved in chewing and swallowing as well as the diaphragm and Recognize that the respiratory complications should take priority.
knows that awareness of possible complications intercostal muscles are weak, the client may aspirate or experience poor gas exchange; both
directs the care provided. Prioritizing, the nurse will increase the risk for pneumonia.
first:
1.‐ Inspect for hemorrhage.
2.‐ Assess for pneumonia.
3.‐ Offer to cut the client's food as needed.
4.‐ Provide the client with a bedside commode.
1808 The nurse knows that a major goal for nursing Correct answer: 4 Parkinson's disease is a progressive degenerative neurological disease. The goal of care is to Option 4 acknowledges the progressive nature of this disease.
management of the client with Parkinson's disease is: control symptoms so that the client can maintain as much independence and function as
possible. The disease does not go into remission. Skin care and weight gain may be
intermediate goals but are not the major goal.
1.‐ Skin care.
2.‐ Disease remission.
3.‐ Weight gain.
4.‐ Symptom management.

1809 The nurse enters the room and finds the client Correct answer: 1 Vital signs (VS) should be watched for 30 minutes to an hour after a grand mal seizure. The Assessment of vital functions is the priority post seizure.
posturing in bed and unconscious. The nurse should respiratory rate, pulse, and blood pressure may be decreased but should slowly return to
assess which of the following during the postictal normal. An aura would precede the seizure. Electrolytes and remembrance of the event are
phase? not critical during the postictal stage.
1.‐ Vital signs
2.‐ Aura
3.‐ Remembrance of the event
4.‐ Electrolytes

1810 Applying the gate control theory of pain, the nurse: Correct answer: 2 The gate control theory of pain postulates that large sensory fibers close the gates to pain in Recall that the gate control theory is associated with massage.
the dorsal form, blocking the transmission of small nociceptive fibers. Rhythmic breathing
promotes relaxation, affecting the behavioral components of pain. Guided imagery affects the
cognitive components of pain.
1.‐ Instructs the client to call for pain medication as soon as the pain begins.
2.‐ Gives the client a back massage.
3.‐ Teaches the client slow rhythmic breathing.
4.‐ Leads the client in guided imagery.

1811 A newly admitted trauma victim is flown in by Correct answer: 1 Changes in level of consciousness, confusion, restlessness, lethargy, disorientation to time, The risk is for IICP. Option 1 identifies the early symptom of this change.
helicopter after a fall from a second‐story window. The place, and then person are the most sensitive and earliest indicators of increased intracranial
MRI shows an intracranial hematoma. Concerned pressure (IICP) produced by an expanding lesion.
about expansion, the nurse will assess for:

1.‐ Level of consciousness.


2.‐ Pupil changes.
3.‐ Respiratory changes.
4.‐ Seizure activity.

1812 Which statement by the client with myasthenia gravis Correct answer: 3 Extremes of temperature may cause an exacerbation of myasthenia gravis. Alcohol and Options 1, 2, and 4 reflect positive adaptation to the changes caused by the disease
indicates a need for further instruction before quinine water may increase weakness. Medications should be taken on time to maintain blood process.
discharge? levels and thus muscle strength. The client should avoid energy activities that will cause
excessive fatigue.
1.‐ "I realize I need to be sure I take my medications at the scheduled times."
2.‐ "I won't be drinking anymore gin and tonics!"
3.‐ "I am looking forward to taking a really long, hot bubble bath when I get home."
4.‐ "I am not going to train for the triathlon this year."

1813 The client with Guillain‐Barré syndrome is Correct answer: 4 The nurse needs to further assess the client's respiratory status by first auscultating the lungs, Recognize that the assessment in option 4 matches the symptoms in the scenario.
complaining of shortness of breath and seems checking tidal volume and oxygen saturation, and suctioning the nasopharynx if needed.
confused. The nurse should: Confusion may be caused by cerebral hypoxia.
1.‐ Call for the code team.
2.‐ Suction the client's oropharynx.
3.‐ Perform a mental status exam.
4.‐ Auscultate breath sounds.

1814 A client presents with circumferential burns to the Correct answer: 3 Electrical burns cause massive soft tissue and muscle injury from the inside out. This question is testing knowledge of the mechanism of injury for an electrical burn and
chest and shortness of breath following an electrical Circumferential burns to the chest wall will decrease chest expansion and ventilation and will the implications for the client. Select the distractor that reflects the extent of injury and
burn injury. The nurse identifies that the priority compromise breathing. An ineffective breathing pattern is evident as a result of this injury. associated risk.
nursing diagnosis for this injury would be: There is potential for further tissue damage, decreased cardiac output, and fluid volume deficit
caused by hypoxia and edema formation for burn with third‐spacing of fluids. However,
breathing and airway are priorities in this case.
1.‐ Deficient fluid volume.
2.‐ Risk for injury.
3.‐ Ineffective breathing pattern.
4.‐ Decreased cardiac output.

1815 The nurse knows the client with a patient‐controlled Correct answer: 1 Only the client is allowed to administer the medication. The PCA pump has preset dose limits. Recognize that option 1 is consistent with patient controlled.
analgesia (PCA) pump and his family understand Both of these measures prevent overdose. The client is instructed to push the delivery button
instructions when they relate: when the pain begins. Persons using opioids for acute pain have a very low prevalence of
addiction. Less total medication is needed with use of a PCA.

1.‐ The client alone is allowed to administer the medication.


2.‐ They should call the nurse when the pain begins.
3.‐ Pushing the control button too often may lead to an overdose.
4.‐ The client who uses a PCA is more likely to become addicted.

1816 To treat a client with autonomic dysreflexia, the Correct answer: 4 This position promotes venous drainage from the head and decreases venous return, Identify the option that describes a position of comfort and that would promote drainage.
nurse positions the client: reducing blood pressure.
1.‐ Prone, head to the side, arms on pillows.
2.‐ In a fetal position with a blood pressure cuff on.
3.‐ Leaning forward over the bedside table.
4.‐ By elevating the head of bed and removing support hose.

1817 Which of the following nursing actions would be Correct answer: 4 Alcohol is a drying agent and should not be used when performing mouth care (lemon‐ Recall that alcohol is considered a drying agent and is always avoided with skin or oral
contraindicated when performing mouth care with an glycerin products should also be avoided). Use a small toothbrush to make cleaning easier. care.
unconscious client? Place the client on the right or left side and avoid the supine position to reduce the risk of
aspiration. Toothettes can be used on the gums, tongue, and mucous membranes to reduce
drying and subsequent breakdown.
1.‐ Give oral care using toothettes
2.‐ Brush the teeth with a small (child‐size) toothbrush
3.‐ Position the client to one side or the other
4.‐ Use an alcohol‐based product for better cleansing

1818 A nurse monitoring a client who has sustained a head Correct answer: 4 Vital signs changes are late indicators of rising intracranial pressure. Trends include an Look for the combined symptoms of bradycardia, increased temperature and increased
injury would determine that the intracranial pressure increase in temperature and blood pressure, and a decrease in pulse and respirations. The blood pressure indicating IICP.
(ICP) is rising if which of the following vital sign trends level of consciousness would also deteriorate before these manifestations arise.
is noted during the course of the work shift?

1.‐ Increased temperature, decreased pulse, increased respirations, decreased BP


2.‐ Decreased temperature, increased pulse, decreased respirations, increased BP
3.‐ Decreased temperature, increased pulse, increased respirations, decreased BP
4.‐ Increased temperature, decreased pulse, decreased respirations, increased BP
1819 The client has been intubated and placed on a Correct answer: 2 Hyperventilation to achieve a PaCO&lt;sub&gt;2&lt;/sub&gt; of 25 to 30 mm Hg causes Omit options 3 and 4 as both are too high to achieve the goal.
mechanical ventilator to reduce intracranial pressure cerebral vasoconstriction that will lead to reduced intracranial blood volume and reduced ICP.
(ICP) by decreasing the carbon dioxide level. Which of Option 1 is excessive; option 3 is normal, and option 4 indicates hypercarbia (excess carbon
the following carbon dioxide values would indicate dioxide).
that the optimal amount of hyperventilation has been
achieved?
1.‐ PaCO2 18 mm Hg
2.‐ PaCO2 29 mm Hg
3.‐ PaCO2 38 mm Hg
4.‐ PaCO2 46 mm Hg

1820 A client who experienced a thrombotic stroke and has Correct answer: 2 Hemiparesis is a one‐sided weakness that often occurs following stroke. The client will have Recognize the need to place the items within reach on the side with function.
residual hemiparesis of the right side is undergoing maximum return of function and the least amount of frustration with relearning new tasks
rehabilitation. The nurse caring for this client when objects are placed within easy reach on the unaffected side. This will also decrease the
reinforces occupational therapy recommendations by risk of client injury because the client will not have to reach for objects needed for self‐care.
placing items for personal hygiene: Unilateral neglect is not a problem when the client has right‐sided deficits, so objects do not
need to be placed on the affected side.

1.‐ On the overbed table on the right side.


2.‐ On the overbed table on the left side.
3.‐ One foot away from the bed on the right side.
4.‐ One foot away from the bed on the left side.

1821 A client with spinal cord injury is at risk for Correct answer: 3 Clients who have a spinal cord injury above the level of T7 are at risk for autonomic Identify the symptom that may be associated with hypertension that is the primary issue
experiencing autonomic dysreflexia. The nurse would dysreflexia, an exaggerated autonomic response to a noxious stimulus. This complication can with autonomic dysreflexia.
carefully monitor for which of the following be assessed by noting the presence of severe, throbbing headache, flushed face and neck,
manifestations? bradycardia, and severe hypertension that is sudden in onset. Other signs to assess for are
nausea, sweating, nasal stuffiness, and blurred vision.
1.‐ Tachycardia
2.‐ Hypotension
3.‐ Severe, throbbing headache
4.‐ Cyanosis of the head and neck

1822 A nurse is caring for a client who just experienced a Correct answer: 2 Documentation about seizure activity includes the time the seizure began, changes in pupil The item is asking about post seizure documentation. Identify the option that refers to
seizure. While doing follow‐up documentation, the size, eye deviation or nystagmus, body part(s) affected, utterance or sounds (epileptic cry), the post seizure function.
nurse would include which of the following items in type of movements and progression, client condition during the seizure, and post‐ictal status.
the nursing progress note? The other items listed are unnecessary.

1.‐ The amount of lighting in the room when the seizure began
2.‐ Utterance of sounds
3.‐ Amount of sleep the client had during the night prior to the seizure
4.‐ Food and fluid intake just before onset of the seizure

1823 The nurse has implemented a teaching plan for the Correct answer: 2 Clients with Parkinson's disease have bradykinesia (slow movements that are hard to initiate), Eliminate options 1, 3, and 4 as being contraindicated in caring for Parkinson’s clients.
client with Parkinson's disease. The nurse evaluates which can be offset to some degree by rocking back and forth to initiate movement. Activities
the teaching as effective if the client states to do which should be interspersed with rest periods throughout the day to minimize fatigue. Chairs should
of the following to help combat manifestations of the be high and firm rather than soft and deep. Velcro fasteners and slide buckles will be of most
disease? use to a client who is trying to maintain independence with dressing and grooming.

1.‐ Plan the most strenuous activities for the evening hours when bedtime is near
2.‐ Use a rocking motion to get up out of chairs
3.‐ Sit in a soft reclining chair to support joints while watching television
4.‐ Choose clothing with several snaps and buttons to increase the benefits of physical therapy

1824 A client who is in Stage II of Alzheimer's disease has Correct answer: 1 Nurses caring for clients who have Alzheimer's disease should ensure that these clients are Recall that wandering is a concern for clients with Alzheimer’s.
memory impairment. The nurse should plan to do wearing an identification bracelet so they do not become lost if they wander. It is unnecessary
which of the following at the beginning of the to assess LOC hourly, and restraints are also not indicated. It is not essential that the client be
upcoming work shift? placed in a quiet, calm environment; rather, they often prefer to be allowed to move about at
will.
1.‐ Check to ensure that the client is wearing an ID badge.
2.‐ Place the client in a quiet, calm environment.
3.‐ Write a note on the cover of the chart asking for a prn order for restraints.
4.‐ Instruct ancillary caregivers to assess the client's level of consciousness hourly.

1825 A client newly diagnosed with trigeminal neuralgia Correct answer: 1 The pain of trigeminal neuralgia is triggered by stimulation of the sensory fibers of the Recognize that the use of the word 'nerve' in option 1 is a helpful hint.
asks the nurse to explain why it hurts so much when an trigeminal nerve. Examples of pressure‐related triggering events include shaving,
episode occurs. The nurse would explain that the pain toothbrushing, washing the face, and eating or drinking. Examples of temperature‐related
of trigeminal neuralgia is the result of which of the triggers include environmental changes and hot or cold food and drink. The other options
following? listed do not initiate the pain of this disorder.
1.‐ Stimulation of the nerve by temperature or pressure
2.‐ Irritation due to cellular effects of hypoglycemia
3.‐ Release of epinephrine during the fight‐or‐flight response
4.‐ An immune system reaction to cold and influenza viruses

1826 A client is scheduled for an electroencephalogram Correct answer: 2 Antidepressants, tranquilizers, and anticonvulsants are generally withheld for 24 to 48 hours Because EEG looks at the electrical activity in the brain, alteration of the activity with
(EEG) early in the morning. The nurse working the before an EEG. The client does not have to be NPO, but should avoid stimulants such as coffee, anticonvulsants is contraindicated.
night shift prior to the procedure would write a note tea, cola, alcohol, and cigarettes. Pre‐procedure care for EEG involves teaching that there is no
to do which of the following per protocol order in the discomfort, and shampooing the hair.
early morning on the day of the test?

1.‐ Instruct the client to refrain from washing the hair


2.‐ Hold the daily dose of anticonvulsant
3.‐ Place the client on NPO status
4.‐ Reinforce client teaching that the test is only mildly uncomfortable

1827 An unconscious client who is not receiving mechanical Correct answer: 1 Oral or gastrointestinal secretions can enter the client's airway and cause aspiration. The Identify airway complications as most important.
ventilation and who does have enteral feeding infusing onset of adventitious breath sounds indicates this risk clearly. The other options are incorrect
has sudden onset of adventitious breath sounds. The because they do not relate to the client's airway.
nurse would first gather additional data to determine
the presence of which of the following nursing
diagnoses?

1.‐ Risk for aspiration


2.‐ Risk for fluid volume deficit
3.‐ Risk for imbalanced nutrition: less than body requirements
4.‐ Altered thought processes

1828 A client has undergone insertion of an intracranial Correct answer: 4 Normal ICP readings extend up to 10 mm Hg pressure (options 1, 2, and 3). Sustained In this case the incorrect options are rather close and so they should be eliminated.
pressure (ICP) monitoring device. The nurse would elevations above 15 mm Hg are of concern, as they are abnormally high. The client's
become most concerned if the ICP readings measured neurological status is probably deteriorating as well.
which of the following for a prolonged period of time?

1.‐ 3 mm Hg
2.‐ 7 mm Hg
3.‐ 10 mm Hg
4.‐ 22 mm Hg

1829 The nurse is admitting a client from the emergency Correct answer: 2 As outlined in the options, the Glasgow Coma Scale is divided into three subsets. Each subset Options 1 and 4 must be eliminated as they exceed the maximum value for the scale.
department following a fall that resulted in increased has a range of scores within it, and for the total scale the highest possible score is 15 while the Option 2 reflects the most improvement.
intracranial pressure (ICP). The nurse interprets that lowest is 3. The higher the score, the more optimal should be the recovery. Scores in the "best
the client's Glasgow Coma Scale score has improved eye opening response" category range from spontaneously (4), to speech (3), to pain (2), to no
the most after making which of the following latest response (1). Scores in the "best motor response" category range from obeys verbal
assessments? commands (6), localizes pain (5), flexion‐withdrawal (4), flexion‐abnormal (3), extension‐
abnormal (2), to no response (1). Scores in the "best verbal response" category range from
oriented (5), conversation‐confused (4), speech‐inappropriate (3), sounds‐incomprehensible
(2), to no response (1).

1.‐ Best eye opening response 5, best motor response 4, best verbal response 8
2.‐ Best eye opening response 4, best motor response 6, best verbal response 5
3.‐ Best eye opening response 6, best motor response 5, best verbal response 4
4.‐ Best eye opening response 3, best motor response 8, best verbal response 6

1830 The nurse planning care for a client who suffered a Correct answer: 2 A client who experienced a CVA may have involvement of the cranial nerve responsible for Recall that thin liquids are difficult to swallow. The other options assist with swallowing.
cerebrovascular accident (CVA) with residual dysphagia swallowing (XII), and generally undergoes a swallowing evaluation to determine whether a diet
would write on the care plan to avoid doing which of can be taken. The client with some residual dysphagia may be started on a diet once the gag
the following during meals? and swallow reflexes have returned. In this instance, liquids should be thickened to avoid
aspiration. The other options represent helpful actions for the client with dysphagia.

1.‐ Feed the client slowly


2.‐ Give the client thin liquids
3.‐ Give foods with the consistency of oatmeal
4.‐ Place food on the unaffected side of the mouth

1831 A client who experienced a spinal cord injury at the Correct answer: 3 Above the level of T6, clients with spinal cord injury are at risk for autonomic dysreflexia. It is Recall that autonomic dysreflexia is always associated with severe hypertension and
level of T5 rings the call bell for assistance. Upon a life‐threatening syndrome triggered by a noxious stimulus below the level of the injury. This bradycardia.
entering the room, the nurse finds the client to have a complication is characterized by severe, throbbing headache, flushing of the face and neck,
flushed head and neck, complaining of severe bradycardia, and sudden severe hypertension. A client may also exhibit nasal stuffiness,
headache, and being diaphoretic. The pulse is 47 and blurred vision, nausea and sweating.
BP is 220/114 mm Hg. The nurse concludes that
immediate treatment is needed for:

1.‐ Malignant hypertension.


2.‐ Pulmonary embolism.
3.‐ Autonomic dysreflexia.
4.‐ Spinal shock.

1832 A nurse is preparing to admit a client from the Correct answer: 1 It is highly controversial whether or not to use a bite stick when a client is experiencing Recall that putting anything in a client’s mouth during a seizure is contraindicated.
emergency department who experienced seizure seizure activity. The greatest risk is that teeth could be damaged if it is inserted during a
activity. The nurse would omit placing which of the seizure. The other pieces of equipment listed are useful in the care of the client and should be
following pieces of equipment in this client's room? made ready at the bedside.

1.‐ Padded tongue blade


2.‐ IV equipment
3.‐ Oxygen and suction equipment
4.‐ Padded bedrails
1833 The home care nurse is doing an admission Correct answer: 2 The Parkinsonian gait is characterized by short, shuffling, accelerating steps. The head leans Recall that shuffling is the term frequently used to describe the gait of a client with
assessment on a client discharged from the hospital forward, the hips and knees flexed, and the client has difficulty starting (bradykinesia) and Parkinson’s disease.
with a diagnosis of Parkinson's disease. When stopping. Options 1, 3, and 4 describe ataxic, dystrophic, and festinating gait, respectively.
assessing the client's neurological status, the nurse
would find the client's gait to be:
1.‐ Staggering and unsteady.
2.‐ Shuffling and propulsive.
3.‐ Waddling but broad‐based.
4.‐ Accelerating with walking on tips of toes.

1834 A client with Alzheimer's disease begins to speak to Correct answer: 4 Since long‐term memory is retained for a longer period of time than short‐term memory, Select the option that best allows the client to discuss the events that he remembers.
the nurse about life in the 1930s. Which of the clients with Alzheimer's disease will be able to recollect events from long ago. It is helpful to
following actions by the nurse is most appropriate? allow clients to reminisce. The other options represent non‐therapeutic techniques for this
client as described.
1.‐ Orient the client to time, place, and person.
2.‐ Distract the client by inviting him to watch television.
3.‐ Encourage the client to talk about recent events in the news.
4.‐ Listen to the client's anecdotes.

1835 The nurse reads in an admission note that the Correct answer: 3 The optic nerve, which governs vision, is cranial nerve II. For this client it would be most Identify that cranial nerve II is the optic nerve, effecting vision.
physical examination of a client revealed an helpful to clear the area of objects that may not be perceived by the client but that could lead
impairment of cranial nerve II. The nurse instructs to falls. The actions described in the other options are unnecessary.
ancillary caregivers to do which of the following when
caring for this client?
1.‐ Whisper to the client
2.‐ Serve food at room temperature
3.‐ Clear the client's path of obstacles
4.‐ Test the temperature of any running water

1836 The nurse is instructing the client with Bell's palsy on Correct answer: 4 Prednisone is often used to treat Bell's palsy. The drug is a steroid, which will reduce Associate reduction of edema with steroids.
information regarding medications that might reduce inflammation and edema and thereby allow the return of normal circulation in the area of the
nerve tissue edema. The nurse would explain the nerve. It can help preserve a significant amount of function, and is effective against pain, when
actions and side effects of which of the following given early in the course of treatment.
medications?
1.‐ Acetaminophen (Tylenol)
2.‐ Ibuprofen (Advil)
3.‐ Dexamethasone (Decadron)
4.‐ Prednisone (Deltasone)

1837 A client with a C4 fracture has been stabilized and Correct answer: 1 The client should move the entire torso to scan the visual field because the client cannot turn Safety measures due to inability to turn the head are key in correctly answering this
fitted for the device shown. Which of the following the head. The client should use straws to prevent spills with liquids. The client should avoid question.
discharge instructions would be appropriate for the bending because the device has a high center of gravity, which could lead to falls. The device is
nurse to share with the client? not removed at bedtime.

1.‐ Move the entire torso in order to be able to scan the visual field
2.‐ Avoid the use of straws until the device is removed
3.‐ Bend slowly to pick objects up from the floor
4.‐ Take the device off only at bedtime
1838 A client who experienced an intracerebral bleed is Correct answer: 1 The zero level for ICP monitoring is 1 inch above the ear, which is at the height of the Note the word “avoid” in the stem. Options 2, 3, and 4 are reflective of good technique
undergoing intracranial pressure (ICP) monitoring. The foramen of Munro. Each time an ICP reading is done, it should be done with the client's head in when using invasive monitoring.
nurse would be sure to avoid doing which of the the same position.
following while caring for this client?

1.‐ Level the transducer at the pinna of the ear


2.‐ Check all stopcocks and connections for leaks
3.‐ Utilize surgical aseptic technique when touching the system
4.‐ Monitor the insertion site for signs and symptoms of infection

1839 The nurse would conclude that a clear fluid leaking Correct answer: 4 Basilar skull fracture can lead to leakage of CSF from the ears or nose. CSF is noticeable in Associate CSF with glucose content.
from the nose following basilar skull fracture is that the drainage will separate into bloody or yellow concentric rings on the dressing material,
cerebrospinal fluid (CSF) after noting that the fluid: which is called Halo's sign. The fluid also will test positive for glucose.

1.‐ Has bright red blood in it and has a pH of 6.


2.‐ Is clear in appearance and tests negative for glucose.
3.‐ Has a foul odor and has a pH of 7.
4.‐ Separates into concentric rings and tests positive for glucose.

1840 A client who experienced a stroke now has a residual Correct answer: 3 Scanning the environment can help to overcome homonymous hemianopsia, loss of one half Note that the question asks about compensation. Scanning the visual field is a method of
deficit of homonymous hemianopsia. The nurse of the visual field. The other items listed will not be of help to the client who has this type of compensation.
explains that which of the following strategies will visual deficit.
assist the client to compensate for this complication?

1.‐ Keep all objects in the impaired field of vision to strengthen eye muscles
2.‐ Wear a patch on the affected eye
3.‐ Scan the visual field by turning the head
4.‐ Wear sunglasses during the day and early evening

1841 The client newly admitted with spinal shock following Correct answer: 4 During spinal shock, there is loss of voluntary control of skeletal muscles, autonomic reflexes Omit options 1 and 2 as they refer to spastic paralysis. Look for an option that combines
spinal cord injury would exhibit which of the following below the level of the injury. These lead to flaccid paralysis, loss of spinal reflex arcs, and flaccid paralysis and bowel and bladder retention.
manifestations during assessment? bowel and bladder retention. The other responses are either partially or totally incorrect.

1.‐ Spastic paralysis of the legs, bowel and bladder incontinence, hyperreflexia
2.‐ Spastic paralysis of the legs, bowel and bladder retention, hyperreflexia
3.‐ Flaccid paralysis of the legs, bowel and bladder incontinence, areflexia
4.‐ Flaccid paralysis of the legs, bowel and bladder retention, areflexia

1842 A client develops sudden seizure activity. The client's Correct answer: 2 Generalized seizures are seizures without a focal point of onset and that are bilaterally Recognize that the scenario is describing tonic‐clonic activity.
entire body first becomes very rigid, followed by symmetric. There are seven subtypes, including tonic‐clonic, tonic, clonic, absence, atonic,
alternating periods of muscle relaxation and myoclonic, and infantile spasms. The tonic‐clonic pattern is as described in the stem. Partial
contraction occurring in all four extremities. The nurse seizures begin locally, and are divided into three subtypes, including simple partial seizures
documents that the client is exhibiting: (without impaired LOC), complex partial seizures (with impaired LOC), and partial seizures
secondarily generalized.
1.‐ Partial seizures secondarily generalized.
2.‐ Generalized tonic‐clonic seizures.
3.‐ Simple partial seizures.
4.‐ Complex partial seizures.
1843 The nurse had been describing to a client the Correct answer: 4 Parkinson's disease is characterized by depletion of dopamine levels in the substantia nigra, Identify the option that refers to dopamine and basal ganglia as correct.
physiological basis for symptoms in Parkinson's leading to the onset of symptoms of Parkinson's disease. Option 1 describes Guillain Barré
disease. Which of the following explanations for syndrome. Option 2 is nonspecific. Option 3 describes myasthenia gravis.
symptoms would be most accurate for the nurse to
share?
1.‐ A viral infection triggers an autoimmune reaction in the nervous system.
2.‐ Peripheral nerve compression is responsible.
3.‐ Antibodies against acetylcholine receptors impair neuromuscular transmission.
4.‐ Dopamine levels decrease in the substantia nigra and basal ganglia.

1844 The family of a client with Alzheimer's disease asks Correct answer: 2 The plaque that characterizes Alzheimer's disease is a cluster of degenerating nerve Eliminate options 1 and 4 as not being associated with this disease. Note the word
about the pathologic changes that are occurring in the terminals, both dendritic and axonal, which contain amyloid protein. The other responses are “abnormal” in option 2 as a key to this being the correct response.
brain. The nurse would explain that this disease is incorrect statements.
caused by:
1.‐ Damage to the myelin sheath of neurons.
2.‐ Abnormal accumulation of proteins.
3.‐ Destruction of neurons.
4.‐ Increase in production of cerebrospinal fluid (CSF).

1845 The nurse admitting a client with a history of Correct answer: 2 Trigeminal neuralgia is manifested by spasms of pain that begin suddenly and last anywhere Note that pain and twitching as symptoms help to identify the correct response.
trigeminal neuralgia (tic Douloureux) would question from seconds to minutes. Clients often describe the pain as stabbing or similar to an electric
the client about which of the following manifestations? shock. It is accompanied by spasms of facial muscles, which cause closure of the eye and/or
twitching of parts of the face or mouth.
1.‐ Facial droop accompanied by numbness and tingling
2.‐ Stabbing pain that occurs with twitching of part of the face
3.‐ Aching pain and ptosis of the eyelid
4.‐ Burning pain and intermittent facial paralysis

1846 The nurse believes a client has slight one‐sided Correct answer: 1 This assessment may be done to detect small changes in muscle strength that might not Recall that the opposite of supinate is pronate.
weakness and decides to further test the client's otherwise be noted. Pronator drift occurs when a client cannot maintain the hands in a
muscle strength. The nurse asks the client to hold the supinated position with the arms extended and eyes closed. Nystagmus is the presence of fine,
arms up and supinated, as if holding a tray, and then involuntary eye movements. Hyperreflexia is an excessive reflex action. Ataxia is a disturbance
asks the client to close the eyes. The client's right hand in gait.
moves downward slightly and turns. The nurse
documents and reports that the client has:

1.‐ Pronator drift.


2.‐ Nystagmus.
3.‐ Hyperreflexia.
4.‐ Ataxia.

1847 The nurse has admitted a client to the emergency Correct answer: 2 A CK level above 150 with over 5% MB isoenzyme indicates myocardial damage from acute The core issue of the question is the ability to correlate indicators of myocardial damage
room with complaints of chest pain over the previous myocardial infarction. Elevated potassium is not indicative of myocardial infarction. Elevated with a client situation. Evaluate each option carefully, and use nursing knowledge and the
2 hours. There are no clear changes on the 12‐lead WBC is an indicator of many conditions, including MI. process of elimination to make a selection.
ECG. The nurse would expect which laboratory test to
provide confirmation of a myocardial infarction (MI)?

1.‐ Potassium of 5.2 mEq/L


2.‐ Creatinine kinase (CK) of 545 with MB of 4%
3.‐ CK of 320 with MB of 12%
4.‐ WBC of 11,400/mm3
1848 The nurse is preparing to discharge a client after Correct answer: 4 Nausea and anorexia are signs of digitalis toxicity. The other laboratory values would not The core issue of the question is the ability to correlate early signs of digoxin toxicity with
CABG surgery. The client is taking several new explain the client’s symptoms and therefore are not priorities to assess before telephoning the a need to check digoxin level in a client with cardiac disease. Evaluate each option carefully,
medications, including digoxin (Lanoxin), metoprolol physician. and use nursing knowledge and the process of elimination to make a selection.
(Lopressor), and furosemide (Lasix). The client
complains of nausea and anorexia. The nurse is
preparing to report this finding to the physician before
discharging the client. Which laboratory result will the
nurse check before calling the physician?

1.‐ Potassium level


2.‐ Sodium level
3.‐ PT/INR
4.‐ Digoxin level

1849 The registered nurse has finished reviewing the 7:00 Correct answer: 1 A stable client with complex dressings is an appropriate assignment for an LPN because the Evaluate each option carefully, and use nursing knowledge and the process of elimination
a.m. shift report on a telemetry unit. Which of the task is appropriate for an LPN. Initial assessment (new admission from the ED), the assessment to make a selection.
following clients would be the best for the RN to assign of a client before and after a complex procedure (PTCA), and discharge teaching are all
to the licensed practical nurse? responsibilities of the professional registered nurse and may not be delegated to the LPN.

1.‐ A 7‐day postoperative CABG client with an infection in the sternal surgical incision, requiring dressings and irrigation
2.‐ A client who has just arrived on the unit from the emergency room for observation to rule out a myocardial infarction
3.‐ A client who has had successful valve replacement therapy and will be discharged this morning
4.‐ A client who is scheduled for a percutaneous transluminal coronary angioplasty (PTCA) at 10:00 a.m.

1850 The nurse is caring for a client with a history of Correct answer: 2 The client’s heart rate is bradycardic, and metoprolol, a beta‐blocker, decreases the heart The core issue of the question is determining which medication is responsible for the
hypertension. The client is being treated with rate. Neither the captopril nor the hydrochlorothiazide lowers the heart rate, and either may adverse effect on client status and acting accordingly. Evaluate each option carefully, and
metoprolol (Lopressor), hydrochlorothiazide be safely administered to maintain control of the hypertension. When a dose of medication is use nursing knowledge and the process of elimination to make a selection.
(HydroDiuril), and captopril (Capoten). The client has a withheld, it is the responsibility of the nurse to notify the physician of the action and rationale.
blood pressure of 120/80 mmHg and a pulse rate of
48. Which of the following is the best action by the
nurse?
1.‐ Administer the metoprolol (Lopressor) and the hydrochlorothiazide (HydroDiuril), hold the captopril (Capoten), and notify the physician.
2.‐ Administer the captopril (Capoten) and the hydrochlorothiazide (HydroDiuril), hold the metoprolol (Lopressor), and notify the physician.
3.‐ Administer all the medications and notify the physician.
4.‐ Withhold all the medications and notify the physician.

1851 The nurse has finished reviewing the shift report on a Correct answer: 3 A client with endocarditis is at risk for thrombus formation, and chest pain and anxiety are The key to determining the answer to priority‐setting questions is to evaluate which client
cardiac unit. The nurse should plan to see which of the signs of pulmonary embolism (PE), which is a life‐threatening complication requiring is most unstable or has the greatest risk for developing a complication. Evaluate each
following assigned clients first? immediate attention. Dyspnea is a chronic symptom with hypertrophic cardiomyopathy, which option carefully using these methods, and use nursing knowledge and the process of
requires assessment; a temperature of 101 degrees F requires additional assessment, and a elimination to make a selection.
client who is ambulating for the first time will be assessed by the nurse. However, the client
who needs to be assessed for PE is the most emergent.

1.‐ A client with hypertrophic cardiomyopathy who is reporting dyspnea


2.‐ A client who had a cardiac catheterization and will be ambulating for the first time
3.‐ A client receiving antibiotics for bacterial endocarditis who is reporting anxiety and chest pain
4.‐ A client who is recovering from coronary artery bypass grafting (CABG) surgery with a temperature of 101 degrees F
1852 The nurse is discharging a client to home with a new Correct answer: 4 A serious complication of atrial fibrillation is pulmonary embolism. Chest pain and hemoptysis The core issue of the question is knowledge of signs and symptoms of complications to
diagnosis of atrial fibrillation. The nurse explains that are common symptoms of pulmonary embolism. Irregular pulse is expected with atrial report to the physician in the presence of atrial fibrillation. Evaluate each option carefully,
which of the following is the most important symptom fibrillation. Fatigue may accompany atrial fibrillation in some individuals. Fever is not and use nursing knowledge and the process of elimination to make a selection.
to report to the physician? associated with atrial fibrillation and is not necessarily included in discharge teaching.
However, it could be a sign of illness that could increase the workload of the heart, and
therefore it would be the second‐most important item to report if it occurred.

1.‐ Irregular pulse


2.‐ Fever
3.‐ Fatigue
4.‐ Hemoptysis

1853 The nurse is caring for a client with a history of renal Correct answer: 3 Renal failure is a common cause of hypocalcemia, and a value of 7.0 mg/dL is below the The core issue of the question is knowledge of normal and abnormal values that are
failure and a new myocardial infarction. The nurse who normal range of serum calcium. Options 1 and 2 are within the upper limits for potassium and important to report in a client with an acute cardiac problem and a history of renal failure.
is reviewing laboratory findings would call the doctor sodium, and option 4 is within the therapeutic range of digoxin. The best strategy in questions such as these is to pick the value with the most abnormal
to report which of the following results? number and/or one that relates to the underlying disorder(s).

1.‐ Potassium level of 5.0 mEq/L


2.‐ Sodium level of 145 mEq/L
3.‐ Calcium level of 7.0 mg/dL
4.‐ Digoxin/digitalis level of 0.8 ng/mL

1854 The nurse is caring for a client who had a permanent Correct answer: 4 The client is not allowed to ambulate for 24 hours to prevent dislodging of the electrodes. Evaluate each option carefully, and use nursing knowledge and the process of elimination
pacemaker inserted because of a complete heart Normal sinus rhythm, heart rate of 80, and a BP of 120 over 80 do not reflect pacemaker to make a selection.
block. The nurse determines that which of the function. Paced beats indicate that the pacemaker is functioning.
following client outcomes indicates a successful
procedure?
1.‐ Client ambulating in the hall within 4 hours of the procedure without dyspnea or chest pain
2.‐ Client’s ECG monitor demonstrates normal sinus rhythm
3.‐ Heart rate of 80 beats per minute, blood pressure 120 systolic, and 80 diastolic
4.‐ Client’s ECG monitor shows paced beats at the rate of 68 per minute

1855 The nurse is caring for a client with a diagnosis of Correct answer: 1 Symptomatic aortic stenosis has a poor prognosis without surgery. Restricting activity limits The core issue of the question is the level of activity that will minimize the client’s risk of
aortic stenosis. The client reports episodes of angina myocardial oxygen consumption. Since the incidence of sudden death is high in this population, complications or sudden death until surgery. Evaluate each option carefully, and use
and passing out recently at home. The client has it is prudent to decrease the strain on the heart while awaiting surgery. nursing knowledge and the process of elimination to make a selection.
surgery scheduled in 2 weeks. Which of the following
would be the nurse’s best explanation about activity at
this time?
1.‐ “It is best to avoid strenuous exercise, stairs, and lifting before your surgery.”
2.‐ “Take short walks three times daily to prepare for postoperative rehabilitation.”
3.‐ “There are no activity restrictions unless the angina reoccurs; then please call the office.”
4.‐ “Gradually increase activity before surgery to build stamina for the postoperative period.”

1856 The nurse is caring for a client who has just Correct answer: 3 The dye used in angiography is nephrotoxic, and a client should have adequate fluids after the The core issue of the question is knowledge of the correlation between lack of fluid intake
undergone cardiac angiography. The catheter insertion procedure to eliminate the dye. The client should lie with the affected leg extended for 6 to 8 and risk of kidney complications following angiography. Evaluate each option carefully, and
site is free from bleeding or signs of hematoma. The hours. Leg exercises are not recommended because exercise could disrupt the clot that formed use nursing knowledge and the process of elimination to make a selection.
vital signs and distal pulses remain in the client’s at the insertion site. Option 1 is incorrect because it gives false reassurance to a client who
normal range. The intravenous fluids were could be at risk if fluids are not taken in.
discontinued. The client is not hungry or thirsty and
refuses any food or fluids, asking to be left alone to
rest. Which of the following is the nurse’s best
response?
1.‐ “You are recovering well from the procedure and resting is a good idea.”
2.‐ “It is important for you to walk, so I will be back in 1 hour to walk with you.”
3.‐ “It is important to drink fluids after this procedure, to protect your kidney function. I will bring you a pitcher of water, and I encourage you to drink.”
4.‐ “You will need to do the leg exercises that you practiced before the procedure to keep good circulation to your legs. After your exercises, you can rest.”

1857 A client undergoes ligation of varicose veins. The Correct answer: 1 Compression stockings exert pressure on the veins of the lower extremities, promoting The core issue of the question is a measure that will improve tissue perfusion for a client
nurse includes in the plan of care which of the venous return back to the heart. Stockings are removed for at least an hour per day to allow following vein ligation. Using principles of blood flow, choose the option that will aid
following important interventions for the nursing for inspection and ensure blood flow through small, superficial vessels. Flexing the extremities circulation. Evaluate each option carefully, and use nursing knowledge and the process of
diagnosis of ineffective tissue perfusion? does not aid tissue perfusion, although it maintains joint range of motion. However, after this elimination to make a selection.
surgery clients are taught to either stand or lie down and avoid flexing at the hip and knee.
Numbness is a temporary or rarely permanent complication of surgery. Briskly scrubbing the
extremities will not aid tissue perfusion.

1.‐ Teach client to remove compression stockings for at least 1 hour per day.
2.‐ Teach client to flex lower extremities four times a day.
3.‐ Teach client that numbness is common after vein ligation.
4.‐ Encourage client to briskly scrub lower extremities to improve circulation.

1858 A client’s angiogram demonstrates the final stage of Correct answer: 1 The final stage of the atherosclerotic process is the development of atheromas, which are Note the critical words "final stage." Evaluate each option carefully, and use knowledge of
atherosclerosis. The nurse concludes that this client’s complex lesions consisting of lipids, fibrous tissue, collagen, calcium, cellular waste, and pathophysiology and the process of elimination to make a selection.
pathophysiology includes which of the following capillaries. The calcified lesions may rupture or ulcerate, stimulating thrombosis. The other
elements? options are not consistent with the ultimate or final changes in the atherosclerotic process.

1.‐ The presence of atheromas


2.‐ Fatty deposits in the intima
3.‐ Lipoprotein accumulation in the intima
4.‐ Inflammation of the arterial wall

1859 When assessing a client with peripheral arterial Correct answer: 3 Leg pain (also called intermittent claudication) is a primary manifestation of peripheral The critical words in the question are peripheral arterial disease, which direct you to look
disease, the nurse assesses the client for which of the arterial disease. Intermittent claudication is muscle pain caused by interruption in arterial flow, for manifestations that are abnormal and that are consistent with arterial but not venous
following signs and symptoms that would be resulting in tissue hypoxia. Peripheral edema and brownish discoloration to the skin on the leg disease. Evaluate each option carefully, and use nursing knowledge and the process of
consistent with tissue ischemia? would be consistent with venous disease, not arterial disease. Widened pulse pressure would elimination to make a selection.
be an unrelated finding.
1.‐ Peripheral edema
2.‐ Widened pulse pressure
3.‐ Leg pain while walking
4.‐ Brownish discoloration to the skin on the leg

1860 In providing community education on prevention of Correct answer: 4 Nicotine in cigarettes promotes vasoconstriction. The three most significant risk factors for Note the critical word “prevention” to focus on the option that contains information that
peripheral arterial disease, the nurse is careful to development of peripheral arterial disease are smoking, hyperlipidemia, and hypertension. The will affect the likelihood of whether the client will develop peripheral arterial disease.
include which of the following as a major risk factor? presence of dysrhythmias, low‐protein intake, and exposure to cool weather are not risk Evaluate each option carefully, using nursing knowledge and the process of elimination to
factors for the disease, although cool weather could worsen the symptoms when disease is make a selection.
already present.
1.‐ Dysrhythmias
2.‐ Low‐protein intake
3.‐ Exposure to cool weather
4.‐ Cigarette smoking
1861 When teaching a client with an aneurysm what signs Correct answer: 4 Aneurysms vary by size and location. Signs of rupture depend on the location of the With the critical words “signs” and “symptoms” in mind, choose the option that most
and symptoms may indicate impending rupture, the aneurysm. Dissection can occur anywhere but most often occurs in the ascending aorta where directly relates to the core issue of the question. Evaluate each option carefully, and choose
nurse considers which of the following? pressure is the highest. The medication the client is receiving is vague and is not directly option 4 as the only one that could affect the specific list of signs and symptoms that the
related. The blood pressure relates to whether the aneurysm may rupture, not to the nurse would teach related to aneurysm rupture.
associated signs and symptoms. The age and gender of the client are unrelated to the size and
symptoms of aneurysm rupture.
1.‐ Medication therapy the client is receiving
2.‐ Client’s usual blood pressure
3.‐ Age and gender of the client
4.‐ Size and location of the aneurysm

1862 An important outcome of care for a female client Correct answer: 2 An important outcome in care of the hypertensive client is the ability to identify and The core issue of the question is the ability to identify an indicator that is a positive effect
with hypertension has been met when the client is counteract personal risk factors that the client has the ability to change. Modifiable risk factors of care for the hypertensive client. Evaluate each option carefully, and use nursing
able to do which of the following? for hypertension include smoking, hypercholesterolemia, diabetes mellitus, sedentary lifestyle, knowledge and the process of elimination to make a selection.
obesity, stress, and alcohol use. Option 1 is not likely to be an issue. Option 3 may or may not
be sufficient. Option 4 is contraindicated.

1.‐ Return to her usual activities of daily living


2.‐ Identify actions to counteract two of her modifiable risk factors
3.‐ Lower her blood pressure by 10%
4.‐ Discontinue lifestyle modifications

1863 Which of the following suggestions should the nurse Correct answer: 1 The client should avoid long periods of standing or sitting to promote adequate blood flow. A critical word in the stem of the question is “arterial,” which tells you that the correct
include when conducting health teaching for clients The legs and feet should be below heart level to increase peripheral circulation. Regular answer is an option that is beneficial to the client with impaired circulation to the legs.
with arterial insufficiency? exercise enhances development of collateral circulation, increases vascular return, and is Choose option 1 over the others because it is a generally helpful measure to increase
recommended for clients with either arterial or venous insufficiency. Moist heat is helpful for circulation, while options 2 and 4 are helpful with venous problems. Option 3 does not help
venous problems. either arterial or venous circulatory problems.

1.‐ Avoid long periods of sitting and standing.


2.‐ Keep the legs and feet in a raised position.
3.‐ Decrease ambulation to decrease pain.
4.‐ Apply moist heat twice a day.

1864 A client with endocarditis develops sudden leg pain Correct answer: 2 The client is exhibiting symptoms of acute arterial occlusion. Without immediate The core issue of the question is recognizing the complication of acute arterial occlusion
with pallor, tingling, and a loss of peripheral pulses. intervention, ischemia and necrosis will result within hours. The nurse should first wrap the leg and then determining which action should be taken first. Choose an option that is client‐
The nurse’s initial action should be to: to maintain warmth and protect it from further injury, and should then quickly notify the focused rather than physician‐notification focused, if one is available. In this case, the nurse
physician. The leg should not be elevated above heart level because doing so would worsen can protect the client from further injury with option 2.
the tissue ischemia, and passive range of motion will also increase ischemia by increasing
tissue demand for oxygen.
1.‐ Elevate the leg above the level of the heart.
2.‐ Wrap the leg in a loose blanket.
3.‐ Notify the physician about the findings.
4.‐ Perform passive ROM exercise to stimulate circulation.

1865 In coordinating care for a client with venous stasis Correct answer: 4 The client with venous ulcers must keep the legs elevated above the level of the heart as The critical words in the stem of the question are “most important intervention,”
ulcers, the nurse explains to unlicensed assistive much as possible. Elevation of the extremities enhances venous return and improves indicating that more than one option, or all options, may be correct, but one is better than
personnel that which of the following is the most circulation, providing oxygen and nutrients to the lower extremities. The client with a leg ulcer the others. Look at the question carefully and note that the nurse is talking to an ancillary
important intervention in ulcer healing? should avoid exercise to prevent further damage to tissues at risk. Option 1 may or may not be caregiver. Consider that the correct option is one that is within the scope of practice of that
indicated. Asepsis is important, but no ulcer will heal unless the edema and stagnant tissue caregiver in making a selection.
metabolites can be reduced through leg elevation.

1.‐ Surgical debridement


2.‐ Meticulous cleaning of the ulcers to prevent infection
3.‐ Performance of leg exercises to increase collateral circulation
4.‐ Elevation of the extremities to increase venous return

1866 Which of the following clients is most at risk for Correct answer: 3 A major risk factor for formation of thrombophlebitis is oral contraceptive use in women who The critical words in the stem of the question are “most at risk,” telling you that the
developing a deep‐vein thrombosis? smoke. Being 1‐week postpartum does not place a client at risk since mobility is usually correct option is the one that contains the most severe or greatest number of risk factors
restored. Anticoagulant therapy is used to prevent development of thrombi. Laparoscopic for thrombophlebitis. With this in mind, evaluate each option and use the process of
surgical procedures are associated with more rapid recovery times with reduced immobility, elimination to make a selection.
keeping this client at lower risk than the client in option 3.

1.‐ A 30‐year‐old client who is 1 week postpartum


2.‐ A 63‐year‐old client post‐CVA on anticoagulant therapy
3.‐ A 40‐year‐old woman who smokes and uses oral contraceptives
4.‐ A 41‐year‐old female who underwent laparoscopic cholecystectomy

1867 The nurse is caring for a 2‐month‐old child with Correct answer: 3 The open ductus arteriosus will allow a small amount of mixing of oxygenated and Use Maslow’s hierarchy of needs to review each option and choose the one that most
transposition of the great vessels. Which of these unoxygenated blood. Stress will increase the cardiac workload and therefore is a priority for closely relates to the ABCs and thus cardiac workload. Use this knowledge and the process
interventions has highest priority? the nurse to avoid. Maintaining caloric intake and comfort are the next priorities using of elimination to make a selection.
Maslow’s hierarchy. Documenting vital signs is a routine activity and not a priority when
compared to actual care activities.
1.‐ Providing comfort for parents
2.‐ Maintaining proper caloric intake
3.‐ Reducing stressors for infant
4.‐ Documenting vital signs

1868 During the acute phase of rheumatic fever, which of Correct answer: 2 The main complication of rheumatic fever is carditis. The nurse must assess for early signs of The core issue of the question is the ability to set priorities for a client with rheumatic
the following is a priority action of the nurse? bacterial endocarditis. The client should be encouraged to rest during the acute phase, and fever. Omit option 1 because of the words 'at least,' knowing that rest is encouraged.
hydration needs may not be sufficiently met with sips of water. Narcotic analgesics may not be Likewise, eliminate option 3 because of the word 'sips.' Choose option 2 over 4 knowing
necessary, although NSAIDs are likely to be ordered. that NSAIDs are likely to be effective in managing pain and inflammation from rheumatic
fever.
1.‐ Encourage ambulation at least four times per day.
2.‐ Assess for early signs of endocarditis.
3.‐ Maintain hydration by encouraging sips of water.
4.‐ Manage pain with strong narcotic analgesics.

1869 A 6‐year‐old child has been diagnosed with Correct answer: 1 Decreased circulation to lower extremities would contribute to muscle fatigue and pain in the The core issue of the question is knowledge of signs of exercise intolerance in a 6‐year‐old
coarctation of the aorta. Lately, he has been legs. Many of the children returning from recess will have increased respiratory rate secondary client with a cyanotic heart defect. Use principles of gas exchange and knowledge of normal
complaining when he comes in from recess. The health to play activities. Blurred vision and bruises are not related to coarctation. and abnormal findings after exercise to make a selection.
nurse should question the child about which of the
following?
1.‐ Weakness and pain in legs
2.‐ Blurred vision
3.‐ Increased respiratory rate
4.‐ Bruises on shins

1870 A toddler with Kawasaki’s disease is going home on Correct answer: 1 Salicylates prevent platelet agglutination. Gastrointestinal bleeding is often a side effect of The core issue of the question is knowledge of adverse drug effects of salicylate therapy
salicylate (aspirin) therapy. Which is the priority aspirin therapy. It is not necessary to avoid other children. Tingling of extremities is not a for the child with Kawasaki’s disease. Use this knowledge and the process of elimination to
teaching at the time of discharge? concern, although ringing in the ears could be a sign of salicylate toxicity. A low‐calorie diet is make a selection.
not indicated.
1.‐ Monitor the child for gastrointestinal bleeding.
2.‐ Avoid contact with other children.
3.‐ Report complaints of tingling extremities.
4.‐ Maintain a low‐calorie diet.
1871 A toddler requires supplemental oxygen therapy for a Correct answer: 2 Allowing mobility is helpful to promote growth and development in the toddler. Strategies The core issue of the question is home care needs of a toddler receiving oxygen therapy.
cyanotic heart defect. In planning for home care, the should be discussed to promote mobility while maintaining the supplemental oxygen. Options Use principles of needs related to normal growth and development to help select the
nurse would discuss which of the following with the 1 and 4 are unnecessary. Signs of oxygen toxicity are not the priority based on the information correct option.
parents? in the question.
1.‐ The need to maintain the child on bedrest
2.‐ Means of promoting mobility while meeting the need for supplemental oxygen
3.‐ Symptoms of oxygen toxicity
4.‐ How to draw blood for blood gases

1872 The nurse would assess for which of the following Correct answer: 1, 2, 4 The client would exhibit pain, pallor of the affected skin, diminished or absent radial pulse, The core issue of the question is knowledge of assessment findings in arterial embolism.
manifestations in a client with suspected arterial parasthesias (altered local sensation), paralysis (weakness or inability to move extremity), and Visualize a clot in the local circulation and use that image to determine the effect of the
embolism to the left hand? Select all that apply. poikilothermia (cooler temperature). The client would not have a bounding radial pulse blockage on circulation to the affected area.
(opposite finding is true) or pitting edema, indicating a fluid volume excess or heart failure.

1.‐ Pain
2.‐ Pale skin
3.‐ Bounding radial pulse
4.‐ Parasthesias
5.‐ Pitting edema

1873 The nurse is caring for a child with Kawasaki’s disease. Correct answer: 3 Cardiac involvement is the most serious complication. The other signs and symptoms are The critical words in the question are “most serious complication.” This indicates the need
The nurse determines that which assessment data diagnostic indicators of Kawasaki's disease. to know what the most serious complication is and also how it is manifested. All symptoms
indicates the presence of the most serious are present in Kawasaki’s Disease but the one with the most potential for damage is
complication of Kawasaki’s disease? cardiac.
1.‐ Dermatitis of extremities
2.‐ Strawberry tongue, erythema of mouth
3.‐ Change in blood pressure, pulse, skin color
4.‐ Fever over five days, bilateral conjunctivitis

1874 A 6‐year‐old is admitted with suspected acyanotic Correct answer: 1 Older children with acyanotic congenital heart disease may be asymptomatic, or manifest Responding correctly to this question requires consideration of the facts about acyanotic
heart disease. After learning that the heart defect is a exercise intolerance, chest pain, arrhythmias, syncope, or sudden death. Option 2 is false, and heart disease and reducing psychological stress.
congenital disorder, the parents ask the nurse how options 3 and 4 are both false and place inappropriate blame on the parents.
they could have missed the problem all these years.
The nurse's response should include the information
that:
1.‐ Acyanotic heart disease may be asymptomatic.
2.‐ The child would only be cyanotic with great exertion.
3.‐ The parents should have recognized the symptoms associated with an acyanotic heart defect.
4.‐ The parents were probably ignoring the symptoms and hoping they would go away.

1875 The pediatric nurse is caring for two children with Correct answer: 2 Chronic hypoxemia in cyanotic heart disease leads to polycythemia, an above‐normal The difference between acyanotic and cyanotic heart disease is that in cyanotic heart
heart disease. The nurse concludes that which increase in the number of red cells in the blood. This change increases the amount of disease, the blood is partially unoxygenated. Consider how this will affect each of the
laboratory finding would be seen in the client with hemoglobin available to carry oxygen. The other answers do not differentiate between the option lab values over time.
cyanotic heart disease but not in the client with cyanotic and acyanotic forms of heart disease in the child.
acyanotic heart disease?
1.‐ Elevated pO2
2.‐ Elevated hemoglobin
3.‐ Decreased hematocrit
4.‐ Decreased pCO2
1876 The mother asks the purpose of aspirin therapy in a Correct answer: 1, 5 Aspirin is an antipyretic (option 5), an analgesic (option 1), and an anti‐inflammatory (option Consider the therapeutic actions of aspirin therapy and its use in rheumatic fever to aid in
child with rheumatic fever. The nurse explains that the 1). It does not prevent complications, hasten recovery, or relate to the development of chorea. choosing the correct answers.
purpose is to do which of the following? Select all that
apply.
1.‐ Provide comfort and reduce inflammation
2.‐ Prevent cardiac complications
3.‐ Hasten recovery
4.‐ Prevent the development of chorea
5.‐ Reduce fever

1877 The parents of a toddler who has undergone a Correct answer: 1 Infective endocarditis is the most common complication of the cardiac surgery. Children may The core concept is prophylactic antibiotic use. Of the options listed, infective endocarditis
successful repair of a ventricular septal defect (VSD) need prophylactic antibiotic therapy for specific conditions as recommended by the American is the primary bacterial condition.
question why the child is being sent home on Heart Association.
antibiotics when no infection is present. The nurse
explains that this is prophylactic to prevent the
complication of:
1.‐ Infective endocarditis.
2.‐ Pulmonary embolism.
3.‐ Cerebrovascular accident.
4.‐ Gastritis.

1878 In assessing children with congenital heart defects, Correct answer: 1, 3 Clubbing of the fingers and toes occurs in cyanotic heart defects, such as transposition of the Any defect that bypasses the lungs will lead to these symptoms. Those defects which
the pediatric acute care nurse would expect to see great vessels. recycle through the lungs will not.
clubbing of the fingers and toes in the child diagnosed
with: (Select all that apply.)
1.‐ Transposition of the great vessels.
2.‐ Atrial septal defect.
3.‐ Tetralogy of Fallot
4.‐ Patent ductus arteriosus.
5.‐ Coarctation of the aorta.

1879 A 7‐year‐old client is diagnosed with rheumatic fever. Correct answer: 1 Rheumatic fever is an inflammatory response of collagen tissue after experiencing a Knowledge that the agent involved in the pathophysiology is bacterial will eliminate option
The physician orders throat cultures of all family streptococcal infection. Some members of the family may be asymptomatic carriers. 4. Eliminate option 3 as vague and incorrect. Of the other options, option 1 is the best
members. When questioned by the parents about the answer because option 2 is not necessarily true.
rationale for this order, the nurse explains that:

1.‐ "Family members can carry streptococcus and be asymptomatic."


2.‐ "The child must have infected others."
3.‐ "Rheumatic fever is familial."
4.‐ "Family members can carry the virus for rheumatic fever."

1880 A 3‐month‐old was diagnosed with transposition of Correct answer: 3 At this stage of grieving, the mother needs someone to listen and validate that her feelings Notice that the correct response utilizes communication terminology. This is often a clue
the great vessels. The mother cannot stop sobbing and are respected. Information will not be heard or remembered. to the right answer.
tells the nurse she feels guilty about her child's
condition. The nurse's best response is to:

1.‐ Agree that a teratogenic stressor could cause this.


2.‐ Disagree with her feelings of guilt.
3.‐ Use therapeutic listening and support.
4.‐ Talk about the newest technology available for cures.
1881 The nurse explains to parents of an infant with an Correct answer: In the child with an atrial septal defect, blood follows the normal pathway from right atrium An atrial septal defect is an acyanotic heart defect. All acyanotic heart defects have either
atrial septal defect (ASD) that, in a healthy heart, the to right ventricle to left atrium. At that point, some of the blood will flow through the atrial no shunting or left to right shunting, so eliminate the left atrium and ventricle as the first
blood pathway is right atrium to right ventricle to left septal defect back to the right atrium. chamber that received abnormal blood flow. Use the word “atrial” to narrow the choice to
atrium to left ventricle. In explaining the altered the right atrium.
cardiac blood flow in ASD, the nurse points to what
area on the picture shown to indicate the first
chamber receiving blood via the abnormal pathway?
Click on the correct area on the image.

1882 A child has been diagnosed with tetralogy of Fallot Correct answer: 1 Prostaglandin E1 helps maintain a patent ductus arteriosus open and thereby allows for Consider the therapeutic effects of the drug to determine the correct answer.
and is taking prostaglandin E1 (Alprostadil). The child is mixing of blood. If the ductus arteriosus closes, cyanosis will increase. The other options
very cyanotic, weak, and has moist respirations. Which represent false statements.
evaluation would indicate a therapeutic response to
this drug?
1.‐ Cyanosis does not increase
2.‐ Blood pressure lowers
3.‐ Respirations increase
4.‐ Temperature drops

1883 The nurse is teaching a class at an outpatient cardiac Correct answer: The pressure is greatest in the left ventricle because that heart chamber must supply blood Recall that pressure gradients are higher in the ventricles than in the atria. Then recall that
clinic held for parents of children with congenital throughout the body. The pressure in the other chambers is lower. the left ventricle has the greatest muscle mass and must push blood throughout the body.
cardiac defects. Before explaining the movement of
blood through septal defects, the nurse tells the
parents about pressure gradients in the normal heart.
The nurse will indicate on the picture of the heart that
the pressure is greatest in which chamber?

1884 An infant who has a congenital heart defect comes Correct answer: 3 The hemoglobin molecule carries oxygen. The oxyhemoglobin gives the skin the pink color. In Recall that cyanosis is due to unoxygenated hemoglobin. Look at the responses to
into the clinic with irritability, pallor, and increased the absence of oxyhemoglobin, the skin color darkens. determine which responses would contribute to unoxygenated hemoglobin.
cyanosis that began quickly over the last 30 minutes.
As the nurse assesses the infant, the parent asks why
the child’s color is bluish. The best response by the
nurse is, “The baby’s skin color is:

1.‐ Caused by a left‐to‐right shunting of blood.”


2.‐ Associated with liver dysfunction secondary to congestive heart failure.”
3.‐ Related to hemoglobin level and oxygen saturation.”
4.‐ Due to poor iron levels in your child’s body.”

1885 A 14‐year‐old child is admitted with a diagnosis of Correct answer: 4 Jones Criteria is a protocol to assist in identifying rheumatic fever. It consists of major Recall that the Jones Criteria divides the symptoms of rheumatic fever into major
“rule out rheumatic fever.” Based on Jones Criteria, symptoms, minor symptoms, and supporting evidence. Erythema, polyarthritis, and elevated manifestations and minor manifestations based on the frequency seen in the disease. Note
the nurse assesses for: ASO titer are among the major and minor symptoms and supporting evidence. that the correct option contains the phrase “strep‐,” which is often associated with the
term “rheumatic fever.”
1.‐ Polyarthritis and dental caries.
2.‐ Fever, headache, and low red blood cell count.
3.‐ Chorea, muscle weakness, and decreased erythrocyte sedimentation rate.
4.‐ Erythema, polyarthritis, and elevated antistreptolysin‐O (ASO) titer.
1886 A toddler with Kawasaki’s disease is ordered to Correct answer: 2 Aspirin therapy is ordered 80 to 100 mg/kg/day until fever drops. Then aspirin is continued at The test item requires specific knowledge about the effects of ASA on fever and platelets.
receive aspirin therapy. The nurse anticipates that 10 mg/kg/day until platelet count drops. Aspirin is used as an anti‐pyretic and anti‐
medication therapy with aspirin will follow which agglutination drug.
principle?
1.‐ High doses of aspirin will be given while fever is high.
2.‐ Length of aspirin therapy is related to child’s response.
3.‐ Aspirin dose will be increased after fever is gone.
4.‐ Aspirin dosage is unrelated to platelet count.

1887 The nurse has taught the parents of a baby girl who Correct answer: 2 Furosemide (Lasix) is a diuretic given to the client with congestive heart failure to assist the Recall that Lasix is a diuretic that increases urine output. Next determine which option
has congestive heart failure (CHF) about drug therapy kidneys in reducing the fluid load in the body. An ineffective heartbeat tends to retain excess describes increased urine output.
with digoxin (Lanoxin) and furosemide (Lasix). The body water.
nurse concludes that the parents understand the
information presented when one of the parents says,
“The Lasix:
1.‐ Is given to the baby because she has a kidney defect.”
2.‐ Helps her heart by reducing the amount of water in her system.”
3.‐ Prevents the Lanoxin from becoming toxic.”
4.‐ Keeps the potassium levels in her body from getting too high.”

1888 A 2‐year‐old boy is being discharged home and will Correct answer: 1 Although a child requiring surgery for tetralogy of Fallot may have a need for additional During the toddler years the child begins to explore the environment. Considering the
have palliative surgery for tetralogy of Fallot at a later services, such as supplemental oxygen at home, the child should be able to play and move activity level of a 2‐year‐old as well as the child’s need for growth and development,
date. The mother wants to know about how much about in the environment to meet both physiological and developmental needs. eliminate options 2 and 3 as unrealistic. Next eliminate option 4 as being difficult to
physical activity she can allow for the child. The nurse’s accomplish and unnecessary.
best answer is:
1.‐ “Allow him to regulate his activity.”
2.‐ “Keep him on complete bed rest.”
3.‐ “Limit his activities to a few hours per day.”
4.‐ “Keep him from crying.”

1889 After a pediatric client has a cardiac catheterization, Correct answer: 4 Direct pressure on the wound site helps to form a clot and reduce bleeding. Hemorrhage can Options 3 and 4 are the best responses for interventions in the immediate
which intervention would the nurse consider to be of be life‐threatening in the immediate postprocedure period. Food intake is a lesser concern postcatheterization period. Of those two, the highest priority would be preventing bleeding.
highest priority during the immediate post‐procedure than maintaining hemostasis. Infection would not be apparent immediately following the
period? procedure. Signs of congestive heart failure could relate to the original disease process but are
not a priority at this time; physiological needs take current priority.

1.‐ Encourage intake of small amounts of fluid.


2.‐ Teach the parents signs of congestive heart failure.
3.‐ Monitor the site for signs of infection.
4.‐ Apply direct pressure to entry site for 15 minutes.

1890 A 6‐month‐old infant is receiving digoxin (Lanoxin) Correct answer: 1, 2 Furosemide is a diuretic, so measurements that most directly illustrate output and water loss Recall that furosemide promotes urine output, which indicates that intake and output
and furosemide (Lasix) for congestive heart failure. The would be evaluated. With this in mind, intake and output and daily weight would be key should be measured. In children, one of the best tools to evaluate fluid volume is daily
nurse who is evaluating the effectiveness of assessment parameters, as they typically and accurately reflect fluid balance. Hemoglobin level weight. Eliminate the other options because they are not directly linked to fluid status.
furosemide would monitor which of the following? measures the iron content of red blood cells. The pulse can be influenced by many variables.
(Select all that apply.) Partial pressure of oxygen is measured via arterial blood gases and is unrelated to the
question.
1.‐ Intake and output
2.‐ Daily weight
3.‐ Hemoglobin and hematocrit levels
4.‐ Pulse rate
5.‐ Partial pressure of oxygen
1891 A 2‐year‐old child has a known cardiac defect and is in Correct answer: 3 Signs of digoxin toxicity include bradycardia, cardiac dysrhythmias, nausea, vomiting, Knowledge of the toxic effects of digoxin is required to correctly answer this question.
congestive heart failure. Which assessment finding anorexia, dizziness, headache, weakness, and fatigue. Recall that the purpose of digoxin is to slow and strengthen the heart so that it can contract
indicates to the nurse a toxic dose of digoxin? more effectively. Reasoning that toxic effects of a drug are often related to excessive effect
of its original purpose, select the option that exhibits a slow heart rate.

1.‐ Tachycardia and dysrhythmia


2.‐ Headache and diarrhea
3.‐ Bradycardia and nausea and vomiting
4.‐ Tinnitus and nuchal rigidity

1892 The home health nurse is monitoring the status of a Correct answer: 1 Because of activity intolerance and respiratory distress, the child may be unable to take in Consider each nursing diagnosis as it would relate to the child with low oxygen levels
child with known cyanotic heart defect. In addition to enough nutrients to meet the body’s need for growth. The child is not at risk for seizures or secondary to a cyanotic heart defect. Eliminate option 4 as it is a psychosocial need, not a
monitoring cardiac function, the nurse monitors the pain because of this health problem. There is no information in the question to support the physiological one. Eliminate options 2 and 3 as unrelated.
child’s other body systems for problems secondary to diagnosis of diversional activity deficit.
the heart defect. Which of the following nursing
diagnoses should the nurse write on the client care
plan?
1.‐ Imbalanced nutrition, less than body requirements
2.‐ Risk for injury, seizures
3.‐ Pain
4.‐ Diversional activity deficit

1893 An infant is admitted with an acyanotic heart defect. Correct answer: 2 Children with acyanotic heart defects may have a murmur without other symptoms. Dyspnea Eliminate all normal findings in a patient with acyanotic heart disease. Weight gain could
Which assessment finding should be discussed with the and tachycardia are early signs of pulmonary edema, which may lead to congestive heart be a positive growth sign or a sign of impending heart failure. Without additional
physician? failure. information, that option is questionable. Dyspnea is a negative finding that should be
reported.
1.‐ Heart murmur
2.‐ Dyspnea
3.‐ Weight gain
4.‐ Eupnea

1894 The nurse is caring for an infant with a cyanotic heart Correct answer: 1, 4, 5 Pulmonary overload occurs prior to congestive heart failure. Crackles and frothy secretions The learner needs to remember that symptoms of congestive heart failure arise from
defect. Symptoms that would indicate risk for are signs of moist respirations, a symptom of pulmonary overload. Fluid volume excess, impaired cardiac output, pulmonary venous congestion, and systemic venous congestion.
congestive heart failure include: (Select all that apply.) secondary to ineffective cardiac function, leads to hepatomegaly and rapid weight gain.

1.‐ Respiratory crackles and frothy secretions.


2.‐ Increased blood pressure.
3.‐ Oxygen saturation increase.
4.‐ Hepatomegaly.
5.‐ Rapid weight gain.

1895 A child is admitted with a diagnosis of “rule out Correct answer: 1 ASO titers indicate history of streptococcal infection, which is a precursor to rheumatic fever. The ASO titer tests for antibodies produced by the body against an enzyme produced by
rheumatic fever.” All of the following symptoms are The other symptoms are not related to this diagnosis. The streptococcus may or may not be the streptococcal organism, streptolysin O. If the organism has not been present recently,
present in the child. Which finding supports the present at the time of diagnosis, so the blood culture could be negative or positive and the the test will be negative.
diagnosis of rheumatic fever? WBC count normal or elevated.
1.‐ Elevated antistreptolysin‐O (ASO)
2.‐ Elevated hematocrit
3.‐ Blood cultures negative
4.‐ White blood cell count within the normal range.
1896 A child is admitted with possible coarctation of the Correct answer: 4 Blood pressure will be elevated in upper extremities and reduced in lower extremities with Vital signs include blood pressure. One order says on admission and once a day. The other
aorta. The admitting nurse reviews the medical orders presence of coarctation of aorta. The constriction of the aorta may be progressive. Vital sign order says blood pressure every 4 hours. These orders are in conflict so one must be wrong.
for the child and should question which of the assessments provide data related to this progression and should be more frequent than once a
following orders? day.
1.‐ Regular diet
2.‐ BP of upper and lower extremities q 4 hours
3.‐ Intake and output
4.‐ Vital signs on admission, then daily.

1897 A child with tetralogy of Fallot becomes acutely ill Correct answer: 2 The knee‐chest position decreases venous return to the heart and thereby increases systemic The key concept is to “relieve cardiac load.” Neither suction nor oxygen will modify cardiac
with an increase in cyanosis, tachycardia, and vascular resistance, which leads to decreased cardiac output. load, so the correct response must have something to do with positioning. Which position
tachypnea. To relieve the cardiac load, the nurse will: will decrease the blood return to the heart?

1.‐ Place the child in Trendelenburg position.


2.‐ Place the child in knee‐chest position.
3.‐ Have oxygen equipment available.
4.‐ Have suction equipment available.

1898 A child with a cyanotic heart defect is being Correct answer: 3 Parents need to be prepared for emergencies. Crying for short periods is effective as deep Eliminate those responses that do not relate to the heart. Then prioritize the responses
discharged home to await surgical repair. In the breathing exercises. Increased intracranial pressure is not associated with cardiac failure. that remain.
discharge teaching, the nurse instructs the parents: Monitoring growth and development would not be the primary concern.

1.‐ To prevent the child from crying at all.


2.‐ To observe the child for signs of increased intracranial pressure.
3.‐ In cardio‐pulmonary resuscitation.
4.‐ To identify growth and development milestones.

1899 A child with rheumatic fever is admitted to the Correct answer: 2 Among the symptoms of rheumatic fever is migratory polyarthritis. The child will complain of Consider those responses that relate to rheumatic fever; eliminate options not related to
nursing unit. The nurse’s most important intervention aching joints. At the time of diagnosis, the child is not infectious. CPR is not a priority at this rheumatic fever. Then prioritize the remaining responses, remembering that the stem asks
is to: time because the child is hospitalized. for the most important intervention during hospitalization.

1.‐ Prevent spread of rheumatic fever.


2.‐ Provide comfort from arthralgia.
3.‐ Evaluate for nervous system complications.
4.‐ Teach parents about cardiopulmonary resuscitation (CPR).

1900 A child with Kawasaki’s disease is admitted to the Correct answer: 1 Aspirin is ordered as an antipyretic and anti‐clotting agent, while immunoglobulins decrease Consider activities appropriate for the treatment of a child with this condition. Then
pediatric unit. Since promotion of comfort is an fever and inflammation. Reducing symptoms of the disease will increase client comfort. The consider the options that would promote comfort.
appropriate nursing goal, the nurse: child’s lips are cracked, and soft foods and liquids are comforting. The child will be lethargic,
and passive range of motion exercises are utilized to facilitate joint movement.

1.‐ Administers aspirin and immunoglobulins as ordered.


2.‐ Splints extremities to prevent contractures.
3.‐ Keeps child NPO for the first 24 hours.
4.‐ Encourages a vigorous exercise program.

1901 A pediatric client is discharged after an acute phase of Correct answer: 2 The child needs to take prescribed antibiotics indefinitely to prevent future infection and Options 1 and 3 are opposites but in this case, neither one is accurate. Option 4 is not
rheumatic fever. The priority discharge instruction possible endocarditis from streptococcal infection. Complete bed rest is not required in the related to the condition. Therefore, by elimination, the correct answer must be 2.
given by the nurse is that the child: recovery period, but the child is maintained with limited activities. Complications of rheumatic
fever are cardiac, not CNS.
1.‐ Is to resume regular activities.
2.‐ Needs to take antibiotics as ordered.
3.‐ Needs to maintain complete bed rest.
4.‐ Will experience central nervous system (CNS) complications.

1902 The nurse is talking with the parents of an infant with Correct answer: The patent ductus is a fetal structure that lies between the aorta and pulmonary artery. In The patent ductus arteriosus lies between the aorta and pulmonary artery.
patent ductus arteriosus. The nurse uses the picture fetal life, the ductus allows blood to bypass the lungs. After birth, because of the change in
shown to illustrate where the defect occurs. Indicate pressures, oxygenated blood will return to the lungs by the ductus.
the spot representing patent ductus arteriosus.

1903 The client complains of chest pain after mowing the Correct answer: 3 Angina pectoris is the term for chest pain related to myocardial ischemia (not enough oxygen Associate the symptom of chest pain with lack of oxygen or ischemia.
lawn. This pain is most likely the result of which of the supply to the tissue for the demand). Any activity that increases the need for oxygen without
following? an adequate available supply can cause angina. Pain from a pulmonary embolus would be
abrupt in onset and not necessarily related to activity.
1.‐ Pericardial effusion of fluid
2.‐ Pulmonary edema
3.‐ Myocardial ischemia
4.‐ Pulmonary emboli

1904 The client who has peripheral edema during the day Correct answer: 2 Orthopnea is shortness of breath caused by the movement of fluid back into the vasculature Recall the definition of orthopnea (difficulty breathing at night when lying flat) to select
states he wakes up in bed at night with difficulty when the client lies down. He may have beginning signs of congestive heart failure and should the correct response.
breathing. Which of the following is he most likely be checked. Sleep apnea may cause orthopnea but not edema; angina doesn't necessarily
experiencing? cause edema either but should be accompanied by chest pain.

1.‐ Angina pectoris


2.‐ Orthopnea caused by recumbent position
3.‐ A sinus infection
4.‐ Sleep apnea

1905 The client is experiencing shortness of breath, Correct answer: 2 Left ventricular failure causes pulmonary congestion and increased pressure in the lungs, Eliminate options 1 and 4 because the symptoms described are not typical of these
productive cough, tachycardia, and orthopnea. The which leads to tachycardia. Remember Left and Lung—the two Ls go together; two of the diagnoses. Angina would be more likely with CAD.
nurse concludes that these are frequently noted as symptoms deal with respiratory symptoms and none of the other answers are related to a
signs and symptoms of: diagnosis affecting the lungs.
1.‐ Hypertension (HTN).
2.‐ Left ventricular failure.
3.‐ Coronary artery disease (CAD).
4.‐ Peripheral vascular disease.

1906 The client has S‐T segment depression on his 12‐lead Correct answer: 3 Depressed S‐T segments and inverted T waves represent myocardial ischemia. Injury usually Look for an option of ischemia with S‐T depression and inversion.
electrocardiogram (ECG). The nurse determines that has an S‐T segment elevation.
this would be indicative of:
1.‐ Necrosis.
2.‐ Injury.
3.‐ Ischemia.
4.‐ Nothing significant.

1907 The early stage of left ventricular failure would most Correct answer: 3 Left ventricular failure results in inability to empty the pulmonary vascular system leading to Recall that the blood entering the left side of the heart comes from the pulmonary system.
likely result in which of the following changes in the increased pulmonary pressures. Failure of the ventricle to pump efficiently would increase pulmonary pressure.
client?
1.‐ Right ventricular failure
2.‐ Diminished left atrial pressures
3.‐ Higher pulmonary pressures
4.‐ Low pulmonary pressure

1908 A client on a telemetry monitor has a heart rate of 54 Correct answer: 4 Bradycardia decreases the myocardium’s demand for oxygen by decreasing workload of the Use the definition of the term bradycardia to select the correct response.
bpm. The nurse knows that this rate would probably heart. Heart rates less than 60 bpm are considered bradycardia. However, if the heart rate is
not increase oxygen demand for the myocardium, but too low, blood supply is decreased and oxygen supply may be hindered.
the rate is indicative of:
1.‐ Tachycardia.
2.‐ Ventricular hypertrophy.
3.‐ Hypertension.
4.‐ Bradycardia.

1909 The client demonstrates significant Q waves on the Correct answer: 4 Infarction (heart attack) is the term for tissue that has been deprived of oxygen until the cells Omit options 1 and 3 as they are not generally associated with ECG changes. Eliminate
electrocardiogram (ECG). The nurse should notify the have died. Immediate attention should be given to the client who has just had a myocardial option 2 as ischemia is associates with S‐T changes.
physician because this is indicative of: infarction (MI), which is noted by Q waves on an ECG.
1.‐ Gangrene.
2.‐ Ischemia.
3.‐ Infection.
4.‐ Infarction.

1910 The client in the Emergency Department was Correct answer: 1 Occlusion of a coronary artery blocks the blood flow and prevents oxygen from getting to the The scenario states that the client had an infarction. Recognize that only option 1 refers to
diagnosed with acute myocardial infarction (MI). He myocardium. Option 2 indicates ischemia; option 3 could be bradycardia or a block; option 4 tissue death or infarction.
asks the nurse to explain what this is. The nurse should could be hypertrophy.
tell him that an MI usually results from which of the
following?
1.‐ Obstruction of a coronary artery with death of tissue distal to the blockage
2.‐ Spasm of a coronary artery causing temporary decreased blood supply
3.‐ A slow heart rate leading to decreased blood supply to myocardium
4.‐ Dilation of the ventricular wall causing decreased blood supply

1911 The nurse teaching the client about behavioral Correct answer: 1 Hyperlipidemia (option 2), cigarette smoking (option 3), and lifestyle (option 4) can be Identify the option that describes characteristics that cannot be changed.
changes, which can affect development of changed by modifying behaviors. These risk factors should be stressed and plans made for how
atherosclerosis, should discuss which of the following to change them.
as a nonmodifiable risk factor for atherosclerosis?

1.‐ Female over 55 years of age


2.‐ Hyperlipidemia
3.‐ Cigarette smoking
4.‐ Sedentary lifestyle and obesity

1912 The client is in the clinic for a follow‐up visit following Correct answer: 4 NTG can be taken as a preventive measure prior to activities that trigger angina. This is The question asks for a method of handling the precipitating event. Option 4 is the only
new onset of stable angina. The nurse should teach the especially helpful with sexual activity or work‐related activities that may need to be continued. option that provides a way of preventing the angina associated with exercise and stress.
client precipitating causes of angina such as exercise Modifying such activities may be necessary, but cardiac clients should not become restricted
and stress. Which of the following should also be by their condition and lead sedentary lifestyles.
taught as a way to handle these precipitating causes?

1.‐ Avoid these activities.


2.‐ Perform such activities anyway.
3.‐ Lead a sedentary lifestyle.
4.‐ Use a nitroglycerin (NTG) tablet before the activity.
1913 A client is at the local health clinic and the nurse takes Correct answer: 3 Systolic hypertension over 150 mmHg may occur after the use of over‐the‐counter (OTC) cold Recognize that the words “stenosis” and “increased ingestion of salt” are both associated
and records a systolic blood pressure of greater than remedies. A warning is placed on OTC medications, cautioning clients with hypertension to with increased blood pressure.
160 mmHg. The nurse tells the client that it could be consult a physician first. Renal diseases contribute to hypertension, and salt causes fluid
caused by: volume retention, which increases the blood pressure. Increases in the electrical activity of the
heart will cause a variety of arrhythmias, not systolic hypertension (option 1). The failure of
the elastic tissue is frequently seen in the skin of the elderly and side effects of
antihypertensives would most likely cause systolic hypotension (option 2). Anaphylactic shock
or an increased electrical activity of the heart would cause a state of systolic hypotension
(option 4).
1.‐ An increase in the electrical activity, causing hypertrophy of the left ventricle.
2.‐ Failure of the elastic tissue or the side effects of hypertension medications.
3.‐ Use of over‐the‐counter medications, renal stenosis, or increased ingestion of salt.
4.‐ Anaphylactic shock or increased electrical activity of the heart causing hypertrophy of the heart muscle.

1914 The nurse should assess the client's pulse and blood Correct answer: 2 When administering propranolol (Inderal), the client's apical pulse and blood pressure must Select the option in which both medications require the monitoring.
pressure as part of the administration of which of the be assessed. Propranolol is a beta blocker used to treat hypertension and tachycardia. The
following drugs? drug should not be given if the apical pulse is below 60 beats per minute (bpm), if there has
been a significant drop in the blood pressure, or if the systolic pressure is below 100 mm Hg.
Side effects include bradycardia, congestive heart failure, pulmonary edema, hypotension and
edema, depression, memory loss, insomnia, drowsiness, and dizziness. Clonidine (Catapres) is
an alpha blocker prescribed to control mild to moderate hypertension. Side effects are
drowsiness, nightmares, nervousness, depression, hypotension, and bradycardia.
Sulfinpyrazone (Anturane) is a medication used to manage long‐term gout, while calcitonin
(Calcimar) is used in the treatment of Paget's disease by decreasing the rate of bone
destruction (option 1). Glucagon (GlucaGen) is a specific medication for the management of
hypoglycemia when glucose is not appropriate (option 3). Hydroxyzine (Vistaril) is used in the
treatment of anxiety, pruritus caused by allergies, psychiatric and emotional emergencies,
nausea and vomiting (excluding the nausea and vomiting of pregnancy), as a preoperative and
postoperative sedation, and as a prepartum and postpartum adjunct therapy (option 4).

1.‐ Sulfinpyrazone (Anturane) and calcitonin (Calcimar)


2.‐ Propranolol (Inderal) and clonidine (Catapres)
3.‐ Glucagon (GlucaGen) and pyridostigmine bromide (Regonol)
4.‐ Hydroxyzine (Vistaril) and prazosin hydrochloride (Minipress)

1915 The nurse should discuss which of the following as Correct answer: 4 Family history and age cannot be modified as risk factors in any form of hypertension. Current Select the option in which both items in the pair are non‐modifiable.
non‐modifiable risk factors influencing hypertension? research suggests there are several genes influencing the development of hypertension.
Ethnicity is a non‐modifiable risk factor; however, stress can be modified if the interruption of
the stressor is undertaken (option 1). Obesity and substance abuse are risk factors that may be
managed through behavioral modification and support groups (option 2). Nutrition and
occupation are both modifiable with assistance (option 3).

1.‐ Ethnicity and stress


2.‐ Obesity and substance abuse
3.‐ Nutrition and occupation
4.‐ Family history and age

1916 In the process of a physical examination of an adult Correct answer: 2 Blood pressure readings from three different positions are helpful in ruling out the presence Select the option that would provide the most data. Orthostatic readings are performed in
client with a possible diagnosis of hypertension, the of hypertension. The difference between each of these readings should be less than 5 mmHg. If the lying, sitting, and standing positions.
nurse should gather data through the use of which the reading difference is higher, repeat readings should be within the follow‐up plan for this
method? client. Options 1, 3, and 4 will not provide enough data to determine if a problem exists.

1.‐ Blood pressure from one arm only


2.‐ Orthostatic blood pressure with 2 minutes between each reading
3.‐ Blood pressure from both arms taken 5 minutes apart
4.‐ Cuff and doppler blood pressure in both arms

1917 Priority teaching for a client with Buerger's disease Correct answer: 4 Smoking is the primary etiological factor identified with clients diagnosed with Buerger's Recall the association of smoking with Buerger’s disease to lead to selection of the correct
would include which of the following? disease. Emphasis should be placed on cessation of smoking, use of nicotine (Nicoderm) option.
patches, and support groups. Option 1 is usually required with clients with Raynaud's disease,
although clients with Buerger's should protect the extremities from cold injury as well. Clients
with Buerger's disease should wear comfortable shoes that will not cause blisters or sores, but
they do not necessarily have to be flat (option 2). Although pain is present, the use of opioids is
usually not indicated (option 3).

1.‐ Wear gloves if the extremities are cold and painful.


2.‐ Avoid wearing flat‐heeled shoes.
3.‐ Report severe pain that may require analgesics.
4.‐ Cessation of smoking

1918 One of the largest concerns with the hypertensive Correct answer: 3 Frequently, clients perceive and become helpless when confronted with multiple lifestyle Note that the words “lifestyle changes” in option 3 help to identify it as the correct option.
client is non‐adherence to the care regimen. changes. The healthcare team and the client need to determine the most significant lifestyle
Noncompliance is most likely influenced by which of modification needed and begin to work with this one. The other modifications are started as
the following factors? the client is able to incorporate them. The major focus is to establish and maintain a
partnership with this client. Many times the therapeutic action of the medication will not
cause the client to feel or perceive any difference in their well‐being (option 1); however,
client education can help to enlighten a client about the need for lifelong therapy. Option 4
may or may not be true for some clients, and option 2 is not a plausible response by
healthcare workers.
1.‐ Medications are not working as indicated by the client statement, "I don't feel any different."
2.‐ A health professional reportedly stating, "Take it easy and stop worrying about taking those medications."
3.‐ The many lifestyle changes needed in diet, exercise, and smoking patterns seem overwhelming.
4.‐ The client has a lack of support persons to assist the client with different therapies.

1919 Which of the following clients has the greatest risk of Correct answer: 2 Cardiac diseases such as congestive heart failure, myocardial infarction, and cardiomyopathy Recognize that only option 2 refers to a specific condition related to thromboembolism
developing a thromboembolism? are conditions that coexist with thromboembolism. Each of these conditions creates the formation.
possibility of thrombus occurring because of ineffective emptying of the heart during its
pumping action. Thromboembolism generally occurs in clients over the age of 40. Gender
usually does not play a role in embolism; a male who is Jewish and over the age of 40 is more
prone to Buerger's disease (option 3). Kidney disease has not been identified as a cause of
emboli (option 4).
1.‐ A 20‐year‐old client
2.‐ A client with a cardiac disease
3.‐ A female client who is Jewish
4.‐ A client with known kidney disease
1920 The nursing management for a client with Correct answer: 1 Part of the medical regime will include anticoagulant therapy. The rationale for this is to Omit options 2 and 4 as being vague and non‐specific. Omit option 3 as being a
thrombophlebitis would include: prevent the development or extension of thrombi by inhibiting the synthesis of the clotting prophylactic measure, not a treatment.
factors or through deactivation of the mechanism. The client's legs need to remain in an
elevated position for comfort and to facilitate venous circulation to prevent the development
of emboli and thrombi in the lower extremities (option 2). Low molecular weight heparin
(LMWH) is usually used as a preventative agent in clients prone to thrombophlebitis, not as a
treatment with a confirmed diagnosis (option 3). The treatment is usually to raise the foot of
the bed 6 inches off the floor (Trendelenburg's position). The knee level needs to remain in this
position 24 hours per day until the healthcare team considers the need for elevation of the
legs no longer exists (option 4). The head of the bed may be elevated for activities such as
eating and bathing.

1.‐ The use of anticoagulant therapy to inhibit the clotting factors.


2.‐ Keeping the client's legs in a position of comfort.
3.‐ Using low molecular weight heparin (LMWH) once a confirmed diagnosis exists.
4.‐ Elevating the head of the bed 6 inches on wooden blocks.

1921 An elderly gentleman enters the Emergency Correct answer: 2 The symptoms exhibited by the client are typical of an abdominal aortic aneurysm. The most The identification of an audible abdominal pulse should lead to the correct option.
Department with complaints of back pain and feeling significant sign is the audible pulse in the abdominal area. If hemorrhage were present, the
fatigued. Upon examination, his blood pressure is abdomen would be tender and firm. There isn't enough information to determine if the
200/110, pulse is 120, and hematocrit and hemoglobin hypertension is secondary or essential (option 1). There is no evidence of congestive heart
are both low. The nurse palpates the abdomen which failure (CHF) in the scenario (option 3). Signs of Buerger's disease involve the extremities
is soft, non‐tender, and ausculates an abdominal pulse. (option 4).
The most likely diagnosis is:

1.‐ Secondary hypertension.


2.‐ Aneurysm.
3.‐ Congestive heart failure (CHF).
4.‐ Buerger's disease.

1922 When caring for a client with Raynaud's disease, Correct answer: 1 The major task of the healthcare team is to medicate the client with drugs that produce Omit options 3 and 4 as this disease affects the hands and does not generally cause
which of the following outcomes concerning smooth muscle relaxation, which will decrease the vasospasm and increase the arterial flow to disability. Prevention of spasms is the priority of care.
medication regimen is of highest priority? the affected part. The drugs used are calcium antagonists. Frequently, the client will be
medicated during the cool to cold months when vasoconstriction is a physiological response to
the environmental temperature. Options 2 and 3 are a concern, but the highest priority is
reducing spasms. If the medicines work, pain will be reduced and blood flow maintained (thus
lesions prevented). Raynaud's disease does not usually cause major disabilities (option 4). A
client may develop gangrene of the skin of the tips of the digits, but these are in the upper
extremities (option 3).

1.‐ Relaxing smooth muscle to avoid vasospasms


2.‐ Controlling the pain once vasospasms occur
3.‐ Avoiding lesions on the feet
4.‐ Preventing major disabilities that may occur

1923 The client on a telemetry unit exhibits regular ECG Correct answer: 1 Depolarization of the myocardium results in contraction (systole) and that produces the Omit options 3 and 4 as having no pulse. Recall that contraction produces the pulse.
complexes of normal duration and a heart rate of 82 palpable pulse and the corresponding QRS complex on the electrocardiogram.
bpm. The nurse assesses the client and notes that for
each complex there is a corresponding palpable pulse.
The nurse knows that for each pulse to be palpable
what had to occur?

1.‐ Ventricular contraction


2.‐ Ventricular relaxation
3.‐ Fibrillation
4.‐ Cardiac standstill (asystole)

1924 The client arrives in the Emergency Department with Correct answer: 2 Stimulation of the SNS increases heart rate and respiratory rate. Remember "fight‐or‐flight" Eliminate options 3 and 4 as they are not part of the central nervous system (CNS).
a heart rate of 130 bpm. The client appears anxious syndrome is a response by the SNS that increases heart rate. Increased pulse and respiratory rate is a sympathetic response.
and is tachypneic. What part of the central nervous
system is most likely responsible for this increased
heart rate?
1.‐ Parasympathetic nervous system (PSNS)
2.‐ Sympathetic nervous system (SNS)
3.‐ Acetylcholine‐cholinesterase feedback
4.‐ Vagus nerve

1925 The client appears in the Emergency Department with Correct answer: 1 Right‐sided heart failure leads to backward venous congestion, resulting in jugular vein Focus on the distension of neck veins, which combined with the other symptoms is typical
distended neck veins, a large, round abdomen, distention, portal hypertension, and abdominal venous congestion resulting in ascites. with CHF.
palpable liver edges, and peripheral edema. These Remember Right means Rest of the body, whereas Left means Lung in identifying where fluids
signs are suggestive of which of the following? stagnate.

1.‐ Congestive heart failure (CHF)


2.‐ Acute liver dysfunction
3.‐ Chronic liver disease
4.‐ Restrictive lung disease

1926 A client with chest pain is given nitroglycerin (NTG) Correct answer: 4 The client does need another NTG if the chest pain is still present; however, a BP should be Note that the BP is mentioned in the scenario, giving a clue for the need to assess the BP
1/150 sublingual for complaint of angina pectoris. Prior assessed first. If the systolic is greater than 100, another NTG can be given. If the systolic is less before NTG is administered.
to the NTG, his blood pressure was 110/78. After 5 than 100, the physician should be consulted. Morphine sulfate is often given in this case.
minutes, he says the chest pain is better but not gone.
The nurse should first:
1.‐ Give another NTG.
2.‐ Check the pulse rate.
3.‐ Give morphine sulfate instead of NTG.
4.‐ Check the blood pressure (BP).

1927 A client with congestive heart failure has digoxin Correct answer: 4 The Lanoxin should be held for a pulse rate less than 60 bpm. Nurses cannot arbitrarily give The mention of a therapeutic level and a pulse in an acceptable range will help to identify
(Lanoxin) ordered every day. Prior to giving the half of a dose without a physician's order. Unless specific parameters are given concerning option 4 as the correct response.
medicine, the nurse checks the digoxin level that is pulse rate, most resources identify 60 as the reference pulse.
therapeutic and auscultates an apical pulse. The apical
pulse is 62 bpm for 1 minute. The nurse should:

1.‐ Hold the Lanoxin.


2.‐ Give half the dose now, wait an hour, and then give the other half.
3.‐ Call the physician.
4.‐ Give the Lanoxin as ordered.

1928 Which of the following clients is at high risk for Correct answer: 2 Family history is a nonmodifiable risk factor. Although diet and exercise should be Select the option that implies a genetic component.
developing coronary artery disease (CAD) that may not encouraged, this may not be sufficient to lower cholesterol and prevent CAD.
be responsive to diet and exercise alone? A client who:

1.‐ Has diabetes.


2.‐ Has a strong family history of cardiovascular disease.
3.‐ Is overweight and 55 years old.
4.‐ Had a stroke at age 40.
1929 The client presents to the Emergency Department Correct answer: 4 The inner layer of the heart is referred to as the endocardium. Use the prefix “sub” as the clue to the correct answer.
with substernal chest pain and is diagnosed with a
subendocardial infarction. The client asks the nurse
what that means. The nurse should tell him that the
damage is:
1.‐ On the anterior wall of the left ventricle.
2.‐ On the posterior wall of the left ventricle.
3.‐ Involving the full thickness of the wall.
4.‐ Involving the inner layer of the heart.

1930 The client is admitted to the coronary care unit with a Correct answer: 4 Left ventricular failure leads to pulmonary congestion. When the left side cannot pump the Eliminate options 1 and 3 as not being associated with crackles in the lung fields.
diagnosis of left‐sided heart failure. When listening to blood out adequately, congestion occurs in the lungs where the blood backs up from the left Pulmonary congestion is likely associated with left ventricular failure.
the lung sounds, the nurse hears crackles bilaterally. ventricle.
The nurse anticipates this because left ventricular
failure leads to:
1.‐ Increased coronary artery perfusion.
2.‐ Pulmonary emboli.
3.‐ Increased peripheral resistance.
4.‐ Pulmonary congestion.

1931 The client with a diagnosis of anterior myocardial Correct answer: 2 The ischemia that causes the MI can also cause the heart muscle to become irritable and Note that myocardial infarction is the only diagnosis mentioned in the stem. Tissue
infarction (MI) begins to show dysrhythmias on the irritated cells fire early, causing dysrhythmias. Although options 3 and 4 are true, nothing in ischemia is associated with MI.
monitor. Which of the following most likely the stem indicates that these are specific to this client. Acidosis is usually the shift with MI, if
predisposes to dysrhythmia development with a one occurs.
myocardial infarction?
1.‐ Respiratory alkalosis
2.‐ Tissue ischemia
3.‐ Hypokalemia
4.‐ Digitalis toxicity

1932 The client is being discharged from the hospital and Correct answer: 3 A low‐fat, low‐sodium diet aids in the reduction of cholesterol and/or triglycerides that could This is addressing discharge teaching which often focuses on positive life‐style changes.
needs discharge teaching. Some important measures have caused the MI. A client who has had an MI should not participate in heavy exercise; a
the nurse should teach the client to prevent moderate exercise program with daily walking would be sufficient. Anticoagulant therapy with
reinfarction following a myocardial infarction (MI) aspirin, not thrombolytics, may be recommended, and clients with an MI should stop smoking
would include: completely.
1.‐ Thrombolytic therapy.
2.‐ Heavy exercise such as high‐intensity aerobics.
3.‐ Low‐fat, low‐sodium diet.
4.‐ Reduction in cigarette smoking.
1933 In teaching a group of middle‐aged men concerned Correct answer: 1 The healthcare provider must be sure that the client is aware of the need to eat sufficient Select the option that teaches a positive lifestyle change that is reasonable for all
about hypertension, the nurse should emphasize which amounts of calcium, magnesium, and potassium. Foods high in magnesium are green, leafy members of the group.
of the following? vegetables; seafood; wheat bran; milk; legumes; bananas; oranges; grapefruit; and chocolate.
Foods high in calcium are milk, cottage cheese, cheese, yogurt, rhubarb, broccoli, collard
greens, spinach, tofu, canned sardines, and salmon. Potassium‐rich foods are fruits and fruit
juices, vegetables and vegetable juices, meats, and milk products. These food products are
best if they are fresh and not processed. The consumption of alcohol is limited to 1 ounce per
day; however, with some anti‐hypertensive medications, the recommendation is the client not
consume any alcohol (option 2). Jogging is a good activity in moderation, i.e., 1 mile per week
(option 3). Herbal therapies containing licorice cannot be safely used by a hypertensive client.
Licorice causes the blood pressure to rise (option 4).

1.‐ Adequate intake of dietary calcium, magnesium, and potassium


2.‐ Alcohol consumption should not exceed 5 ounces per day
3.‐ Jogging 3 miles per day
4.‐ Use of herbal therapy containing licorice

1934 Secondary hypertension (HTN) is an elevated blood Correct answer: 3 By definition, secondary hypertension has some underlying cause. Approximately 10 percent Eliminate option 1 due to the qualifier “short‐term,” and option 4 as it refers to a “large
pressure caused by a number of underlying of all pregnant women develop this condition. The criteria are that the systolic blood pressure population” which makes the statement untrue. Option 3 is the only true statement.
physiological processes. Which of the following rises 30 mmHg and the diastolic blood pressure rises 15 mmHg prior to the 20th week of
statements about secondary HTN is correct? gestation. Diagnostic tests are conducted to confirm the diagnosis and rule out polycythemia,
hyperaldosteronism, and pheochromocytoma. The systematic long‐term use of contraceptives
containing estrogen may contribute to secondary hypertension (option 1). Coarctation of the
aorta is rare; when it occurs, it interferes with the renal blood flow, which stimulates the renin‐
angiotension‐aldosterone system of the kidney (option 2). Endocrine disorders and
hypertension are rare and involve the adrenal medullary system (option 4).

1.‐ A nonsystematic, short‐term use of contraceptives containing estrogen can cause HTN.
2.‐ Coarctation of the aorta has little to no relationship with the renin‐angiotension‐aldosterone system of the kidney.
3.‐ Hypertension in pregnancy is a frequent cause for maternal and fetal morbidity and mortality.
4.‐ Hypertension with an endocrine disorder accounts for a large population of clients.

1935 The pathophysiology of hypertension is related to Correct answer: 2 One of the factors that regulates blood pressure is the amount of fluid volume within the Select the option that is clearly associated with hypertension, regulation of body fluids.
which of the following monitoring systems? body system. Excess concentration of sodium and water increases the blood pressure and the
pressure in the kidney filtration, resulting in diuresis. The baroreceptors respond to the activity
of the receptors as well as pressure and chemical composition within the vascular system.
Arterial receptors are also involved (option 1). The endocrine system is usually not involved
(option 3). Hypertension causes an increased production of sodium and water releasing
hormone (option 4).

1.‐ Baroreceptors only


2.‐ Regulation of the amount of body fluid volume
3.‐ Endocrine system
4.‐ Underproduction of sodium‐retaining substances

1936 Laboratory tests are usually ordered to complete the Correct answer: 4 These laboratory studies would be the most helpful and give an estimate of the degree of Select the option in which all items in the string would be included in the assessment of
assessment of the client with hypertension (HTN). vascular involvement as well as the degree of damage. An elevated cholesterol level would hypertension.
Which laboratory values are essential in the diagnosis? suggest HTN related to atherosclerosis. Creatinine is the most specific test of kidney function
(a cause of HTN) and is not affected by foods as is the blood urea nitrogen (BUN). A hematocrit
will be helpful in determining fluid problems, which could account for HTN also. Bone scan
(option 1), glucose tolerance (option 2), and prothrombin time (option 3) are not essential
tests for diagnosing HTN.
1.‐ Serum sodium, serum potassium, and a bone density scan.
2.‐ Hemoglobin, hematocrit, and a glucose tolerance test.
3.‐ Stress test, prothrombin time, and urine analysis.
4.‐ Serum cholesterol, complete blood count, and serum creatinine.

1937 The nurse would identify which of the following Correct answer: 2 An overweight client on bed rest from a hip surgery is at higher risk because of the two risk Recall that thrombophlebitis is a common postoperative complication, particularly with
clients to be at high risk for thrombophlebitis? factors (obesity and immobility). Even though the client will be ambulated and progressively overweight clients and with joint surgery.
increase weight‐bearing, the potential exists because of the immobility. HTN does not increase
the risk, nor does smoking (option 1). Raynaud's is not a factor (option 3); option 4 is vague
about the cardiac history and further information is needed.

1.‐ 25‐year‐old male who is a smoker with hypertension


2.‐ 67‐year‐old overweight female recovering from a hip replacement (1st day post‐op)
3.‐ 22‐year‐old female with a history of Raynaud's disease in the family
4.‐ 72‐year‐old male with a history of arthritis and bypass surgery

1938 A male client is seen in the clinic complaining of pain Correct answer: 4 These are signs and symptoms of PVD; the diagnosis is supported by the pallor noted when Omit option 3 as Raynaud’s affects the fingers. The color and tingling are the symptoms
when he tries to walk in order to exercise three times the feet are elevated for 30 minutes. The pulse is diminished because it is arterial occlusion that should lead to the correct response of PVD.
a week. Upon examination, his feet are noted to be and not venous. Pain and itching are usually felt with varicose veins (option 1).
dusky and purplish while dangling from the stretcher. Thrombophlebitis is associated with redness, warmth, and swelling of an extremity (option 2).
The pedal pulse is palpable but diminished, and he Raynaud's is more involved with the digits of both the hands and feet (option 3).
states his feet "tingle on occasion." The nurse has him
lie supine on the stretcher with the foot of the
stretcher elevated for about 30 minutes, after which
his feet show pallor. The client is probably suffering
from:

1.‐ Varicose veins.


2.‐ Thrombophlebitis.
3.‐ Raynaud's disease.
4.‐ Peripheral vascular disease (PVD).

1939 The client with intermittent claudication is at risk for Correct answer: 2 Whenever there is tissue breakdown associated with intermittent claudication, the client will Select the option that differs from the others, which all refer to some sort of exercise.
activity intolerance and possible tissue breakdown. If be confined to bed in order to be able to meet the oxygen requirements for the damaged
tissue breakdown occurs, the plan of care would tissues. Activity (options 1, 3, and 4) raises the amount of oxygen required to sustain both
consist of: healthy and diseased tissues to a point where deficits will occur and healing will be stalled. At
the time the client is to be ambulated, the shoe of choice is a supportive, comfortable shoe.

1.‐ Limited exercise.


2.‐ Bed rest.
3.‐ Range of motion exercises.
4.‐ Walking in comfortable light slippers.

1940 The most common cause of a thrombus is: Correct answer: 3 These are known as Virchow's triad and are the most commonly associated reasons for a Omit options 2 and 4 which list conditions that may increase risk, but not the common
blood clot. A thrombus usually involves the venous, not arterial, system (option 1). Situations cause. All items in the string must be correct. &lt;u&gt;Hypo&lt;/u&gt;coagulability would
that contribute to venous stasis are myocardial infarction and prolonged sitting; however, a not contribute to thrombus formation.
stroke is not classified in this manner (option 2). Injury may or may not cause thrombi, while
continued bed rest can contribute (option 4).
1.‐ Arterial stasis, hypocoagulability, and arterial wall injury.
2.‐ Myocardial infarction, stroke, and prolonged sitting.
3.‐ Venous stasis, hypercoagulability, and venous wall injury.
4.‐ Motor vehicle accident and prolonged bed rest.
1941 Assessment of a client with possible thrombophlebitis Correct answer: 2 Pain felt in the calf while pulling up on the toes is abnormal and indicates a positive test. If the Eliminate options 1 and 4, while identifying the options with Homan’s sign as the
to the left leg and a deep vein thrombosis is done by client feels nothing or just feels like the calf muscle is stretching, it is considered negative assessment for thrombophlebitis. Recall that pain indicates a positive response.
pulling up on the toes while gently holding down on (option 3). A tourniquet test (options 1 and 4) is used to measure for varicose veins.
the knee. The client complains of extreme pain in the
calf. This should be documented as:

1.‐ Positive tourniquet test.


2.‐ Positive Homan's sign.
3.‐ Negative Homan's sign.
4.‐ Negative tourniquet test.

1942 Effective teaching for a client with Raynaud's disease Correct answer: 3 A client with Raynaud's disease needs to be taught to protect the digits from extreme cold by Recognize that “gloves and socks” in option 3 lead to the correct answer as clients with
is evident in which of the following statements? “I using warm clothing, gloves, and socks. Use of gloves is essential any time the digits may be Raynaud’s are taught to protect distal extremities.
will:” cold (such as at night). Smoking should be stopped completely (option 1). Relaxation and stress
management are essential (option 2). Diet is not associated with Raynaud's disease (option 4).

1.‐ "Decrease my smoking to 6 cigarettes per day."


2.‐ "Try to learn to relax, but I can't promise anything."
3.‐ "Wear gloves and socks to bed at night."
4.‐ "Try to eat a healthier diet."

1943 The client admitted for an acute myocardial infarction Correct answer: 2 Decreased workload of the myocardium leads to decreased oxygen demand. Rest periods Focus on the term “workload” to assist in identification of the correct response. This is a
(MI) is getting ready for discharge. The client is anxious allow the demand to equal the supply. Although a slower heart rate can decrease workload goal of care in the post MI client.
to go home. He states, "I have so much work to do and oxygen demand, rest does not necessarily lower the heart rate. Option 3 is indicated for
around the house." The nurse instructs the client on hypertension, and option 4 is indicated for congestive heart failure.
the need for rest periods, especially if he begins to feel
chest discomfort. The rationale for rest in the
treatment of acute myocardial infarction is to:

1.‐ Decrease the heart rate.


2.‐ Reduce the workload of the heart.
3.‐ Decrease the peripheral vascular resistance.
4.‐ Improve the stroke volume.

1944 The client was admitted with right‐sided congestive Correct answer: 4 Marked rales, rhonchi, S&lt;sub&gt;3&lt;/sub&gt; heart sounds, and frothy sputum Recall that frothy, pink sputum is associated with pulmonary edema.
heart failure and treated with diuretics, and his edema (frequently pink from being blood‐tinged) are classic pulmonary edema characteristics.
cleared quickly on the second day. He was given a Corticosteroids are used in situations like this, but remember that a side effect is congestive
round of steroids to help because he had no appetite heart failure.
and was extremely fatigued. On day 3, he has a
relatively normal head‐to‐toe assessment at 8:00 A.M.
when the nurse makes rounds. Around 10:00 A.M., the
nurse aide reports the client is having trouble
breathing and his respirations are 42 and labored. The
nurse assesses his condition and finds that he has
marked rales and rhonchi; an
S&lt;sub&gt;3&lt;/sub&gt; heart sound; and frothy,
pink sputum. The nurse suspects:

1.‐ Return of right‐sided heart failure.


2.‐ Development of mild left‐sided heart failure.
3.‐ Tamponade.
4.‐ Pulmonary edema.
1945 The client presenting with angina pain in the Correct answer: 4 Angina pectoris is pain related to insufficient oxygen supply to meet the workload demands of Look for methods of relieving the pain identified in the options. Pain relief with rest is key
Emergency Department states, "I thought I was having the heart. If the workload demand is decreased (as in rest), the pain goes away. Option 1 may in differentiating MI from angina.
a heart attack." Which response by the nurse would be correct but is not usually the case; option 2 is incorrect. In option 3, a CPK‐MB level
provide the client with the most accurate information indicates the amount of muscle damage.
about the difference between the pain of angina and
that of myocardial infarction?

1.‐ "The pain associated with angina is much more intense than that of a heart attack."
2.‐ "The pain associated with a heart attack radiates up the jaw and that of angina radiates down the arm."
3.‐ "The intensity of pain with a heart attack indicates the amount of muscle damage."
4.‐ "The pain of angina is usually relieved by resting or lying down."

1946 The echocardiogram report on a client states that the Correct answer: 1 Increased pressure needed to push the blood through the stenotic valve causes the left Use knowledge of anatomy to note that the mitral valve is on the left side of the heart.
client has dilation and hypertrophy of the left atrium. ventricle to enlarge in order to become stronger and bigger to accommodate the increased
The nurse knows these are the initial compensatory blood volume and the higher pressure needed.
responses in:
1.‐ Mitral stenosis.
2.‐ Aortic stenosis.
3.‐ Tricuspid stenosis.
4.‐ Acute rheumatic fever.

1947 The client was admitted with a diagnosis of right Correct answer: 3 Rales and rhonchi are not characteristic of right heart failure (RHF). All the others are classic Recall that &lt;u&gt;left&lt;/u&gt; heart failure leads to &lt;u&gt;lung&lt;/u&gt;
ventricular failure. If the client's condition were to signs and symptoms. If the client entered with RHF and develops left heart failure (LHF), symptoms.
worsen and go into biventricular failure, which of pulmonary complications would develop.
these symptoms would the nurse expect to find?

1.‐ Decreased urine output


2.‐ Jugular neck vein distention
3.‐ Rales and rhonchi
4.‐ 3+ pitting edema in the legs

1948 The client is being discharged after a two‐day hospital Correct answer: 4 Angina is the result of decreased supply of oxygen to meet the demands of increased Recall that lack of oxygen is the cause of pain. Select the response that addresses
stay following admission for new‐onset angina. An workload of the heart. Rest decreases the workload and therefore the oxygen supply is metabolic needs.
angiogram revealed a 90% blockage, which was sufficient. This client could have had an anterior wall myocardial infarct of the left ventricle
successfully opened by percutaneous transluminal because of the location of the blockage in the picture. Since the left ventricle is the "power" of
coronary angioplasty (PTCA). The client asks the nurse the heart, an adequate supply of oxygenated blood flow is needed at all times.
why rest is important when the chest pain begins. The
nurse should tell the client that:

1.‐ The coronary blood vessels dilate at rest.


2.‐ Venous return to the heart is increased in the seated or supine position.
3.‐ Coronary blood flow is improved because of an increase in blood pressure.
4.‐ The coronary circulation adequately meets the metabolic needs at rest.

1949 A compensatory mechanism for congestive heart Correct answer: 3 The law states that the more stretch of the muscles the better the contraction force until the Eliminate options 1, 2, and 3 as not being directly related to CHF.
failure in the initial phase that improves cardiac output muscle is stretched out and can no longer contract sufficiently to move the blood through the
is: circulation. Initially, this improves cardiac output. Option 1 is helpful in hypotension; Option 2
explains contraction with each heartbeat; option 4 is the pathophysiology behind
hypertension.
1.‐ The renin‐angiotensin‐aldosterone system.
2.‐ An action potential.
3.‐ The Frank‐Starling's law.
4.‐ Increasing peripheral vascular resistance.

1950 In congestive heart failure (CHF), the goal is to Correct answer: 2 Afterload is the force or resistance that the left ventricle must pump against the impedance Omit options 1 and 4 as they do not relate to afterload. Omit option 3 as it talks about
decrease workload, increase contractility, and to the flow of the blood. By decreasing afterload in CHF, a pump that is already inefficient has force of contraction.
decrease afterload. Which of the following is the best to work less to be effective.
definition of afterload?
1.‐ The volume of blood remaining in the left ventricle after systole
2.‐ The impedance to ejection of blood from the left ventricle
3.‐ The amount of force of ventricular contraction
4.‐ The measurement of left ventricular end‐diastolic pressure

1951 The client is being discharged from the hospital and Correct answer: 4 Congestive heart failure is the inability of the heart to pump adequate blood to meet the Focus on the response that identifies “inefficient pump” as the cause of CHF.
asks the nurse to explain to him what congestive heart oxygen and nutritional needs of the tissue. Think of CHF as a "pooped‐out" pump.
failure (CHF) really is. The nurse's best response would
be that CHF is:
1.‐ Circulatory congestion resulting from extracellular fluid volume excess.
2.‐ Inadequate myocardial function due to damage of the cardiac muscle.
3.‐ Circulatory congestion resulting from heart failure and its compensatory mechanisms.
4.‐ Cardiac output insufficient to supply the metabolic needs of the peripheral tissues caused by an inefficient pump.

1952 A client is due to receive a calcium channel blocker Correct answer: 3 Both the beta blocker and the calcium channel blocker have the potential of lowering the BP Note the pulse rate below 60 bpm which in the case of cardiac drugs requires a call to the
and a beta blocker for angina. Vital signs are BP and pulse (sometimes significantly). Most textbooks still list 60 bpm as the baseline pulse. physician.
100/68, pulse 52 bpm, respirations 20. Which of the Giving the medicine as usual could lower the vital signs even further. Even if vital signs are
following is an appropriate action by the nurse? monitored, the drugs will be in the client's system and could result in harm. Holding the
medicine until rounds is not usually an acceptable practice. Unless specific parameters are
established on a unit, the nurse should cover herself with a written order for parameters.

1.‐ Give the medicine and check vital signs in 30 minutes.


2.‐ Hold the medicine until the physician makes rounds.
3.‐ Call the physician first.
4.‐ Give the medicine as usual.

1953 An understanding of the renin‐angiotension Correct answer: 1 Renin is released to conserve water, thus increasing the systemic blood pressure. Options 2, Recall that the renin‐angiotensin system produces vasoconstriction, thus correcting
mechanism in hypertension is important because renin 3, and 4 are incorrect because the mechanism results in decreased renal flow, decreased blood hypotension.
is released to: pressure, and decreased tubular sodium concentration.
1.‐ Correct hypotension.
2.‐ Increase renal flow.
3.‐ Raise blood pressure.
4.‐ Increase tubular sodium concentration.

1954 A thorough history concerning any blood pressure Correct answer: 3 Prior to surgery, the client should be questioned concerning any family history or prior The question is addressing hypertension in a pre‐surgical situation. Focus on the possible
problems before surgery is necessary in order to problems with hypertension (HTN) if anesthetics were used. Malignant HTN is a medical complication of malignant HTN.
determine: emergency that can occur after administration of anesthetics. Although problems with
hypotension (option 1) and the normal BP range (option 4) are important, anesthetics are
usually associated with concern for malignant HTN.
1.‐ Potential problems with hypotension.
2.‐ Need for antihypertensives because of pain.
3.‐ Possibility of malignant hypertension.
4.‐ Client’s normal blood pressure (BP) range.
1955 Early in the development of hypertension, there may Correct answer: 2 The disease of hypertension progresses slowly into the vessels, the heart, the kidney, and the Recognize that the word “intermittent” in the stem is a hint to the correct response of
be few, if any, pathological changes except brain. Essential hypertension is an elevated systemic arterial pressure (option 1). There is no Labile hypertension.
intermittent elevations of the systemic blood pressure. known cause. Normotension is a client with a normal blood pressure (option 3). Secondary
This is called: hypertension is an elevated blood pressure associated with several primary diseases (option
4).
1.‐ Essential hypertension.
2.‐ Labile hypertension.
3.‐ Normotension.
4.‐ Secondary hypertension.

1956 Frequently, the client with chronic, untreated Correct answer: 1 As hypertension progresses untreated, the fundi of the eye will demonstrate changes. These The key word of “changes” in option 1 aids in identifying it as the correct response.
hypertension will have an ophthalmoscopic include a) minimal vascular changes in the early stage; b) irregular appearance of arterioles; c)
examination. The findings will be reported as: changes progressing to attenuation of retinal vessels with retinal hemorrhage; and d)
attenuation of retinal vessels with disc swelling in the late stage. Microaneurysms are seen in
clients with diabetes mellitus (option 2). No red reflex is seen in clients with a completely
opaque lens (option 3). Cupping of the optic disc is seen in a client with glaucoma (option 4).

1.‐ Ocular changes in the fundi.


2.‐ Microaneurysms.
3.‐ No red reflex.
4.‐ Cupping of the optic disc.

1957 Renin converts angiotensin I to angiotensin II (a Correct answer: 4 ACE inhibitors block the conversion of angiotensin I to angiotensin II or inhibit the effect of Omit options 1, 2, and 3 as not influencing the renin‐angiotensin system.
potent vasoconstrictor) in order to increase the angiotensin II; therefore, the potent vasoconstrictor is not able to increase the blood pressure.
systemic blood pressure. This explains the Sympatholytics and beta blockers block the sympathetic nervous system, which stimulates
effectiveness of which group of antihypertensives? renin (options 1 and 2); diuretics affect the absorption of sodium and water (option 3).

1.‐ Beta blockers


2.‐ Sympatholytics
3.‐ Diuretics
4.‐ ACE inhibitors

1958 An aggressive teaching plan is devised with the client Correct answer: 4 Teaching about the antihypertensive drug management will include informing the client about The phrase “untoward reaction” leads to the identification of the correct option, manage
for the purpose of reducing the risk of untoward situations that might cause lightheadedness and fainting. These situations would be standing medication therapy. The other options would not reduce the risk of untoward reactions.
reactions of hypertension. The highest priority would motionless for a prolonged period of time, rising suddenly from a sitting position, or soaking in
be to: a hot bath. All of the answers are correct. However, if the drugs are properly administered,
compliance is maintained (options 1 and 2) and the BP could be monitored on a monthly basis
(option 3).
1.‐ Prevent poor compliance.
2.‐ Identify unawareness of noncompliance.
3.‐ Actively monitor the blood pressure (BP).
4.‐ Manage medication therapy.

1959 Collaborative care frequently includes modifications Correct answer: 4 MHR is determined by subtracting the client's age from 220. A target heart rate (THR) should The item asks for a maximum heart rate. Eliminate options 1 and 3 as too low for a
in lifestyle for the hypertensive client. One area for be estimated based on the client's condition. However, the MHR is the upper limit the client maximum. Heart rate over 200 is not recommended, so eliminate option 2.
modification is to increase the client's physical activity. can safely reach (if possible), but should not be exceeded. Options 1 and 3 are too low; Option
A 49‐year‐old client should be taught to exercise three 2 is too high for an MHR.
times a week for 20 to 30 minutes and not to exceed
his or her maximum heart rate (MHR) of:

1.‐ 110 beats per minute.


2.‐ 250 beats per minute.
3.‐ 152 beats per minute.
4.‐ 171 beats per minute.

1960 A client with varicose veins needs to consider the Correct answer: 4 The client with varicose veins must be assessed for the presence of constipation. If this is The question asks for the area that would be overlooked. Select the option not directly
possibility of risk reduction. What area would probably present, the client is encouraged to consume a high‐fiber diet. Constipation increases the intra‐ related to the risks for varicose veins.
be overlooked by a client and should be stressed in a abdominal pressure, thus promoting venous stasis in the lower extremities. Options 1, 2, and 3
teaching plan? are more commonly considered because these are likely problematic to the client already.
Clients should be taught to avoid standing or sitting in one spot for a prolonged time period.
Frequently, the healthcare provider will suggest wearing support hose, elastic stockings, or
wrapping the legs from toes to upper thigh with elastic bandage. Knee‐bending exercises are
also suggested (option 1). Walking for the client with varicose veins is appropriate; however,
the walking time should be about 1 mile per day, unless the client is already walking 3 miles
(option 2). Encouraging the obese client to investigate a weight loss program is satisfactory;
however, the key is to enroll and to become actively involved with the weight loss program
(option 3).

1.‐ Avoid standing or sitting in the same spot for more than 5 minutes at a time.
2.‐ Walk at least 3 miles per day.
3.‐ Encourage the obese client to investigate weight loss.
4.‐ Encourage the client to consume a high‐fiber diet.

1961 Which nursing diagnosis is the primary focus in Correct answer: 1 The client with Raynaud's disease will have altered peripheral tissue perfusion to the fingers Focus on the pathology of Raynaud’s, which is peripheral vasospasm. This will help to
planning the care for a client with Raynaud's disease? and toes caused by spasms within the arterioles. This phenomenon may occur unilaterally or identify the correct response.
bilaterally as a result of a number of diseases, such as, collagen vascular diseases, pulmonary
hypertension, and many others. Raynaud's disease does not usually impair physical mobility
(option 2). Anxiety is not a common finding; however, it may be present because Raynaud's
disease is considered a chronic illness (option 3). Skin integrity may be decreased because of a
diminished level of oxygen to the peripheral vessels. When this occurs, skin breakdown might
occur and the risk of infection would increase (option 4).

1.‐ Altered tissue perfusion: peripheral


2.‐ Impaired physical mobility
3.‐ Anxiety
4.‐ Risk for infection

1962 Which of the following nursing diagnoses is the Correct answer: 4 When assessing fluid volume, the nurse needs to include the amount of blood loss from the The question asks for the priority, usually indicating that all of the options are correct.
highest priority in a postoperative client after the client's system. Warning signs would be a decreasing blood pressure, restlessness, clammy Recall that the nature of the surgery would place option 4 at the highest priority due to
repair of an abdominal aneurysm? skin, pallor, decreasing levels of consciousness, thirst, oliguria less than 30‐50 mL/hr, and an blood and fluid loss.
increasing abdominal girth. Any changes indicating hypovolemia need to be reported to the
surgical team immediately. Although options 1, 2, and 3 may be appropriate, notice that the
question asks for the highest priority. Ischemia of the bowel may occur when the mesenteric
vessels have been clamped and an ischemic colitis may follow (option 1). Altered tissue
perfusion has probably occurred during the surgery. It is essential that pulses in each of the
extremities be determined for presence or absence and have the client report tingling or
numbness anywhere in the body (option 2). Impaired gas exchange may occur because of the
extent of the abdominal incision and the client's ability to cough and deep breathe (option 3).

1.‐ Potential for injury: ischemia of the bowel


2.‐ Altered tissue perfusion: peripheral
3.‐ Impaired gas exchange
4.‐ Fluid volume deficit
1963 Which of the following diagnosis is high priority for a Correct answer: 4 A client who is stable (without symptoms) and does not want to take medications or modify Focus on options 3 and 4 as they are stated as “risk for.” Recall that the priority for a client
client who is stable with a medical diagnoses of his or her lifestyle is likely to be noncompliant. Certain treatments for CHF are life‐long in with CHF is the risk for decreased cardiac output.
congestive heart failure (CHF), but is having trouble order to prevent major complications, such as keeping blood pressure under control,
modifying their lifestyle? modifying salt intake, exercising, maintaining a proper weight, and taking prescribed
medication. Emphasis should be placed on the need to continue care even though there are no
symptoms present.
1.‐ Altered tissue perfusion: systemic
2.‐ Altered nutrition: more than body requirements
3.‐ Risk for injury
4.‐ Risk for decreased cardiac output

1964 The client comes to the clinic for a routine check‐up. Correct answer: 4 Hypertension is one of many potentially modifiable risk factors. All the other noted factors Focus on the option that describes a factor that can be changed or controlled.
The nurse notes that the blood pressure (BP) is 180/96. are not modifiable by the client. Other modifiable risk factors are smoking, high cholesterol,
The client denies any history of hypertension. The obesity, stress, and leading a sedentary lifestyle.
nurse begins to educate the client on risk factors for
coronary artery disease (CAD). The client is a 50‐year‐
old male with a strong family history of CAD. The nurse
would include which of the following in his or her
teaching about modifiable risk factors?

1.‐ Age
2.‐ Gender
3.‐ Family history
4.‐ Hypertension

1965 The client presents with a cholesterol level of 325 Correct answer: 1 Cholesterol is the substance carried by lipids that deposits along the arterial walls causing The question is asking about the relationship of cholesterol to CAD. Select the option that
mg/dL. In teaching the client about risk factors for stiffening and narrowing of the vessel. These atherosclerotic plaques lead to coronary artery best describes the disease process. Omit option 3 as food is not the only source of
coronary artery disease (CAD), the nurse would best disease. The other answers are not applicable. cholesterol. Omit option 2 as being physiologically untrue.
describe cholesterol as:
1.‐ The substance that sticks to the inside of the blood vessels, decreasing blood flow
2.‐ The stuff that deposits in the veins and prevents absorption of nutrients.
3.‐ The food you eat that causes the blood vessels to get soft and pliable.
4.‐ The number one cause of heart failure.

1966 In evaluating the electrocardiogram (ECG) of the Correct answer: 2 Inverted T waves and a depressed S‐T segment are classic signs of ischemia. Elevated S‐T Recall the association of S‐T depression with ischemia to choose the correct answer.
client in the Emergency Department, the nurse notes segment means damage. LVH would be noted by changes on a 12‐lead EKG.
inverted T waves and S‐T segment depression,
indicating which of the following?
1.‐ Left ventricular hypertrophy (LVH)
2.‐ Ischemia of the myocardium
3.‐ Atrial wall infarction
4.‐ Right ventricular infarction

1967 A client on the telemetry unit with a diagnosis of Correct answer: 1 Atrial fibrillation is accompanied by an irregular rhythm. The heart rate can be slow, within Omit options 2 and 3 as calling the physician is not indicated by the scenario. Omit option
congestive heart failure and atrial fibrillation has an normal limits, or fast. The action of digoxin (Lanoxin) is to increase the force of contraction 4 as the client is already on a monitor.
order for digoxin (Lanoxin). The client's pulse is 112 (positive inotropic) and decrease the heart rate (negative chronotropic). These actions will
and irregular. The nurse should: help in both congestive heart failure and atrial fibrillation. There is no need to monitor the vital
signs every 15 minutes while on a telemetry monitor and in this situation.

1.‐ Give the dose as ordered.


2.‐ Hold the dose and call the physician.
3.‐ Call the physician immediately.
4.‐ Give the dose and monitor vital signs every 15 minutes.
1968 The nurse teaching on the management of risk factors Correct answer: 4 Normal cholesterol is less than 200 mg/dL. All the other options are nonmodifiable but are Identify the correct answer by selecting the only option that can be changed or controlled.
in coronary artery disease would include which of the risk factors nevertheless. Emphasis should always be placed on modifiable risks.
following as modifiable risk factors?
1.‐ Postmenopausal syndrome
2.‐ Gender
3.‐ Father died of acute myocardial infarction at age 63
4.‐ Cholesterol level of 290 mg/dL

1969 A client states that over the past several months he Correct answer: 1 Intermittent chest discomfort relieved by rest is most likely angina pectoris. The pain of a Make an association between angina and pain that is relieved by rest. Recall that heart
has begun experiencing some chest discomfort that myocardial infarction does not disappear until the damage has been done. Although the chest attack pain is not relieved by rest.
comes and goes. It usually stops when the client is pain can mimic gastrointestinal distress or other illnesses, a thorough exam of the client
resting. The client asks the nurse if he is experiencing a should be made.
heart attack. Which of the following is the nurse’s best
response?
1.‐ "Pain related to a heart attack would not stop when you rest."
2.‐ "You could be having a heart attack but it is not doing any damage to the muscle."
3.‐ "The discomfort you describe is not related to your heart."
4.‐ "The discomfort you are experiencing is related to stomach gas production and will go away."

1970 The client's electrocardiogram (ECG) shows significant Correct answer: 2 A silent myocardial infarction can occur with no noticeable chest discomfort, and Q waves will Focus on the nurse questioning the client regarding a history of a heart attack, in spite of
Q waves in two contiguous leads. The nurse asks the appear on the EKG. the absence of chest pain. This leads to the selection of the correct response, option 2.
client if he has ever had a heart attack. The client
states he has never experienced any significant
amount of chest pain. The best explanation for the Q
waves would be that:

1.‐ The ECG results are wrong due to misplacement of electrodes.


2.‐ Sometimes a person can have a heart attack without any pain.
3.‐ Q waves have no relevance to the presence of a heart attack.
4.‐ If the client never experienced chest pain, then he never had a heart attack.

1971 The client presents to the Emergency Department Correct answer: 3 Nitroglycerin is a vasodilator that dilates the coronary arteries and increases the blood flow Always associate pain with lack of oxygen to the tissues. Recall that NTG relieves pain by
with substernal crushing chest pain. The doctor orders to the myocardium, therefore relieving the pain. Morphine sulfate would be an example of enhancing the delivery of oxygenated blood.
nitroglycerin (NTG) sublingual to be given. The client option 2; options 1 and 4 are not related to angina.
asks the nurse what the NTG is for. The best
explanation is that NTG is a(n):

1.‐ Neuroblocking agent to reduce the pain.


2.‐ Opioid for pain relief.
3.‐ Vasodilator and works by providing more oxygenated blood flow to the heart.
4.‐ Antiarhythmic for the treatment of supraventricular tachyarhythmia.

1972 The nurse knows that in heart failure, decreased Correct answer: 1 Increased heart rate decreases diastolic filling time, compromises coronary artery perfusion, Eliminate options 2 and 3 as not being consistent with the question. Option 1 is correct as
cardiac output leads to the onset of certain and increases myocardial oxygen demand; the resulting ischemia leads to decreased cardiac adaptive or compensatory mechanisms will fail after extended use.
compensatory mechanisms. The effects of these output. The body's initial attempts are an effort to meet the supply and demand of the heart.
compensatory mechanisms may be to: Compensatory mechanisms, however, begin to work negatively later on.

1.‐ Hasten the deterioration of cardiac function.


2.‐ Decrease heart rate and decrease cardiac output.
3.‐ Increase sensitivity to the sympathetic nervous system (SNS).
4.‐ Promote vasodilatation by increasing alpha‐receptor sensitivity.
1973 The client is admitted to the hospital with a diagnosis Correct answer: 4 Shortness of breath, dyspnea on exertion, and crackles are classic signs and symptoms of left‐ Use the association of LEFT = Lung for heart failure symptoms.
of left‐sided heart failure. How should the nurse sided heart failure. The fluid backs up into the lungs from the left side. Right‐sided heart failure
differentiate left and right heart failure? The client leads to backflow of blood to the peripheral circulation causing edema in the extremities,
would: jugular vein distention, and possible ascites.

1.‐ Demonstrate distended neck veins in left heart failure.


2.‐ Present with crackles in his lung fields with right heart failure.
3.‐ Have peripheral edema associated with left heart failure.
4.‐ Present with dyspnea and crackles bilaterally with left heart failure.

1974 The client in the coronary care unit, status post‐ Correct answer: 2 Left heart failure produces increased pulmonary congestion and therefore elevated The key in the question is the presence of crackles, which would be associated with left
anterior myocardial infarction (MI), has been pulmonary artery pressures. A normal PA is 25/10 mmHg. Pink, frothy sputum and exaggerated failure.
diagnosed with heart failure. A pulmonary artery (PA) symptoms would be present for pulmonary edema to be diagnosed. Right‐sided heart failure
catheter has been placed. The PA pressure is 30/15 includes signs of peripheral edema.
mmHg and the client has mild crackles. The nurse
knows from this reading that the client is experiencing:

1.‐ Right heart failure.


2.‐ Left heart failure.
3.‐ Both right and left failure.
4.‐ Pulmonary edema.

1975 The client returns to the clinic for a follow‐up visit Correct answer: 3 PND is a result of the fluid shift at night in the supine position, from the interstitial to the Use the terminology in the question to identify the correct response; nocturnal means
post hospitalization for heart failure. On his discharge intravascular compartment causing increased workload to the heart. Because of the increased night, dyspnea means difficulty breathing.
papers, the term paroxysmal nocturnal dyspnea (PND) volume, pulmonary congestion occurs. The client awakes with severe dyspnea.
was listed. The client asks the nurse to explain this.
Which of the following is the best response?

1.‐ "PND is when you have to get up frequently at night to urinate."


2.‐ "PND is a term that doctors use to encourage clients into compliance with care instructions."
3.‐ "PND is a sudden onset of difficulty breathing during the night."
4.‐ “PND is the onset of a life‐threatening dysrhythmia, and you must call 911 immediately.”

1976 The client presents to the Emergency Department Correct answer: 2 Elevated heart rates lead to decreased ventricular filling time and weaker pulse. Arrhythmias Omit options 3 and 4 as being contradictory. Recall that a high pulse rate would generally
complaining of "palpitations." The nurse connects the such as atrial tachycardia may increase the heart rate to 180 to 200 bpm. Assess for signs of result in a weak and thready pulse.
client to the monitor. The heart rate is 180 bpm and adequate cardiac output.
blood pressure is 108/50. The nurse would expect his
pulse to be:
1.‐ Full and bounding.
2.‐ Weak and thready.
3.‐ Weak and bounding.
4.‐ Full and thready.

1977 The client admitted to the coronary care unit with a Correct answer: 1 Frequent monitoring of vital signs and hemodynamic monitoring allows for early detection of Select the response that is client‐focused to answer this question correctly.
diagnosis of heart failure asks why he must be changes in condition and early intervention. Clients with heart failure can rapidly deteriorate;
awakened so often and why his blood pressure is being usually vital signs are the first indicator.
taken so often. The nurse's best response would
include which of the following statements?

1.‐ "Assessment of subtle changes in vital signs may lead to early interventions."
2.‐ "The doctor ordered frequent assessments."
3.‐ "Charting of frequent blood pressure is necessary for joint commission documentation."
4.‐ "The nurse manager is coming in early to do chart assessments and we need all the blanks filled in."

1978 The client in the coronary care unit with a diagnosis Correct answer: 2 Atrial dysrhythmias are frequently seen in heart failure. Atrial fibrillation is irregular, has no Eliminate options 1 and 4 as absence of P waves eliminates normal sinus rhythm as an
of heart failure states his chest feels "funny." The discernable P waves, and is often accompanied by a fast ventricular response. option.
nurse assesses the client and monitor, noting an
irregular rhythm with a rate of 110. All the QRS
complexes look alike, but there are no discernable P
waves and the rhythm is irregular. The client is most
likely experiencing:
1.‐ Normal sinus rhythm with premature ventricular contractions.
2.‐ Atrial fibrillation.
3.‐ Accelerated junctional rhythm.
4.‐ Normal sinus rhythm with premature junctional contractions.

1979 The client comes in for a follow‐up visit after Correct answer: 1 Sinus arrhythmia is a normal variant related to increased intrathorasic pressure (vagus Focus on the statement in the scenario that defines the rhythm as a normal sinus rhythm.
discharge from the hospital with a diagnosis of acute stimulation) as seen with deep inspiration and expiration. It is a benign arrhythmia that
myocardial infarction. The client explains that when he requires no treatment.
walks, he sometimes notes that his pulse is slightly
irregular. The nurse assesses his pulse and finds that it
is slightly irregular. The monitor confirms a slightly
irregular rhythm with no ectopic beats and all the
characteristics of a normal sinus rhythm. The nurse
explains this to the client as a:

1.‐ Normal variation that occurs in some people and is frequently associated with deep breathing.
2.‐ Very serious condition that must be addressed by the physician.
3.‐ Condition that must be treated with medication.
4.‐ Condition that reflects a decreased oxygen supply to the myocardium.

1980 The client presents to the Emergency Department Correct answer: 2 Athletic syndrome is common with very active individuals that have increased cardiac Omit options 1 and 3. Recall that anxiety and pain would increase the heart rate.
complaining of knee pain. The client was running a strength and force of contraction. The heart rate may be as low as the 30s in extremely
race at his school and fell and twisted his knee. The athletic individuals, especially runners. If the client is asymptomatic, no treatment is necessary.
client tells the nurse that he has been running track for
4 years. On assessment, his pulse is 52. The nurse's
explanation for this sinus bradycardia is:

1.‐ Pain slows the heart rate.


2.‐ A slower pulse is normal in athletic persons.
3.‐ Anxiety causes a slower heart rate.
4.‐ A slower pulse could mean the client has other medical or heart conditions unknown to him.

1981 The client is brought to the Emergency Department Correct answer: 3 Junctional rhythm is a regular rhythm originating in the AV junction with a rate of 40 to 60 Eliminate options 1 and 4 as being inconsistent with the slow rate in the scenario.
by the emergency medical system with complaints of bpm, no discernable P wave or an inverted P wave before, during, or after the QRS complex.
near syncopal episode. After placing the client on the The QRS complex is less than 0.12 seconds in duration. The cause of the arrhythmia should be
monitor, the nurse notes a regular, slow rhythm‐‐a determined and treated or a pacemaker inserted.
rate of 42 bpm. There are no discernable P waves and
the QRS complexes are less than 0.12 seconds in
duration. The nurse would interpret the rhythm as:

1.‐ Atrial fibrillation with slow ventricular response.


2.‐ Idioventricular rhythm.
3.‐ Junctional rhythm.
4.‐ Sinus bradycardia.

1982 The client's 12‐lead electrocardiogram was just Correct answer: 4 Second‐degree heart block Type II (also referred to as Classical) has two or more P waves per Recognize that the difference in the atrial and ventricular rate helps to define this as a
placed on the chart. The nurse looks it over and notes QRS complex and has a constant PR interval. The ventricular rate is usually bradycardic. second‐degree block.
that the atrial rate is 82 and the ventricular rate is 40.
The PR interval is constant at 0.16 seconds, with two P
waves for every QRS complex. With the above
description, the nurse interprets the client's rhythm to
be:
1.‐ Third‐degree heart block.
2.‐ Second‐degree heart block Type I.
3.‐ First‐degree heart block.
4.‐ Second‐degree heart block Type II.

1983 Persistent severe hypertension is a medical Correct answer: 1 When assessing a client with severe hypertension, the client may be demonstrating Select the option that is likely to occur and have serious consequences, in this case,
emergency. The nursing assessment should include: hypertensive encephalopathy. If this occurs, the client will have a change in the level of altered level of consciousness.
consciousness ranging from confusion to coma and possible seizures. The client experiencing
severe hypertension may or may not demonstrate a headache (option 2). Unless pulmonary
edema is present, the breath sounds will be clear bilaterally (option 3). Temperature is less
critical than pulse (option 4).
1.‐ Level of consciousness.
2.‐ Presence of headache.
3.‐ Breath sounds.
4.‐ Temperature.

1984 The medical management of thrombophlebitis is Correct answer: 3 The PTT is used to monitor the level of heparin so that a therapeutic dose may be Omit options 1 and 4 as not being associated with anticoagulant therapy.
through the use of anticoagulant therapy. The nurse administered. GPT is a liver enzyme and will not be affected by heparin (option 1). The PT is
will determine the effectiveness of heparin therapy by used to monitor Coumadin therapy (option 2). The FBS is used to monitor the blood glucose
assessing which of the following? levels (option 4).

1.‐ Glutamate pyruvate transaminase (GPT)


2.‐ Prothrombin time (PT)
3.‐ Partial thromboplastin time (PTT)
4.‐ Fasting blood sugar (FBS)

1985 Frequently, sodium consumption and substance Correct answer: 4 When reviewing the risk factors that are listed, both sodium consumption and the use of Recognize that both sodium consumption and substance abuse are controllable or able to
abuse are linked to hypertension. These factors are substances may be modified if the client desires to participate in a lifestyle change. be changed, therefore modifiable.
classified as: Nonmodifiable risk factors are those that are not changeable, such as family history, gender,
and ethnicity (option 1). Personal choice risk factors is not a commonly used phrase (option 2).
Gender risk factors are a subgroup of the nonmodifiable risk factors (option 3).

1.‐ Nonmodifiable risk factors.


2.‐ Personal choice risk factors.
3.‐ Gender risk factors.
4.‐ Modifiable risk factors.

1986 Raynaud's disease is a disease of the small arteries Correct answer: 2 Triggers in Raynaud's are cold, caffeine, smoking, and stress. Raynaud's disease has the Associate Raynaud’s with spasm of the digital vessels. Recall that symptoms should be
and arterioles, which constrict in the presence of characteristic of the vessels that become vasospastic in nature, but inflammation does not associated with cold exposure.
stimuli. Which of the following is true? occur (option 1). Smoking is considered a trigger because of the nicotine contained in the
tobacco, but claudication is not usually found in clients with Raynaud's disease (option 3).
Smoking is the primary cause of Buerger's disease (option 4). Raynaud's is caused by genetics
or autoimmune disorders.
1.‐ Vasospasm occurs, causing inflammation of the veins.
2.‐ Stimuli include cold temperatures and caffeine.
3.‐ Smoking triggers an event and claudication of the leg occurs.
4.‐ Smoking is the primary cause.

1987 The primary nursing management of a medical client Correct answer: 3 The primary treatment of a client with thrombi is to monitor the anticoagulant therapy. The Focus on the aspect of therapy with the most risk associated, which is the anticoagulant
with thrombi includes which of the following? therapy will prevent the formation or extension of thrombi by inhibiting the clotting factors or therapy.
by quickening their inactivation. Options 1, 2, and 4 are correct, but are not the primary
treatment. Clients need an analgesic for their discomfort. Warm packs may also be used to
promote comfort (option 1). The client's bed is to be elevated 6 inches at the foot of the bed.
This forces (via gravity) the blood to return and not to remain stagnant (option 2). Pulmonary
emboli may occur and lung sounds should be auscultated (option 3).

1.‐ Monitor the client for the degree of discomfort.


2.‐ Elevate the legs at least to a 45‐degree angle.
3.‐ Monitor the anticoagulant therapy.
4.‐ Assess lung sounds.

1988 Community screening and self‐care are general public Correct answer: 1 Hypertension is a major public health concern in the areas of prevention and early detection Recognize that the question is asking about screening efforts, which are usually directed at
health initiatives. Which one of the following is of the of new cases across the age continuum. Monitoring the existent hypertensive clients is a the general public as in option 1.
highest priority? challenge and the focus is on the prevention of further complications of the disease and to
reduce the cardiovascular risks (option 2). The study of cardiovascular risks is a long‐term goal.
The population awareness and early detection are important within each community (option
3). The study of isolated systolic hypertension is narrowly focused and could be addressed in
other initiatives (option 4).

1.‐ Hypertension as a major concern of the general public


2.‐ Monitoring the existing hypertensive clients
3.‐ Studying cardiovascular risk factors
4.‐ Studying isolated systolic hypertension

1989 Dietary management and client teaching for a Correct answer: 3 The diet of the client with hypertension needs to include the essential amounts of calcium Eliminate foods known to be high sodium such as processed foods and nuts. Recognize
hypertensive client include which of the following food found in milk for neuromuscular irritability, transmission of nerve impulses, skeletal muscle that carbonated beverages have little or no nutritional value.
selections? contraction, and clotting. Magnesium in the form of oranges is needed as an important
intracellular enzyme system. Also, neither of these foods is high in sodium. Processed foods
are generally high in sodium and should be avoided by the hypertensive client (option 1).
Carbonated beverages contain either high levels of sodium or potassium (option 2). Nuts are
high in phosphorus and sodium. Phosphorus and calcium are in an inverse relationship to each
other (option 4).
1.‐ Processed foods
2.‐ Carbonated beverages
3.‐ Milk and oranges
4.‐ Nuts

1990 The elderly client with decreased total body water Correct answer: 1 Drug distribution is determined by the adequacy of the plasma volume, the extracellular fluid The item is asking about total body water. Select the only option that refers to drug
may react differently to antihypertensive drugs volume, and the serum protein levels. Many of the drugs are processed through the liver or distribution as related to fluid volume.
because: the kidney. Any impairment of either organ will potentially cause a problem of overdose
(option 2). With a decrease of the total body water, the peripheral resistance will be increased
to provide the body with an auto‐transfusion (option 3). The elderly client has a slowing of the
intestinal motility and absorption. This may adversely affect the therapeutic drug level (option
4).
1.‐ Drug distribution is changed by plasma volume, extracellular fluid volume, and serum protein levels.
2.‐ Liver and renal functions have little to no influence on the drug's distribution.
3.‐ Decreased peripheral resistance occurs when body fluid decreases.
4.‐ Intestinal absorption of the antihypertensive drug is accelerated.

1991 The priority intervention in the plan of care for a Correct answer: 2 When the client has a diagnosis of fluid volume excess, monitoring all fluids is important. Assume all of the items to be correct, so select the priority. Recall that intake and output
hypertensive client with excess fluid volume includes: Failure to monitor the client places him or her at risk for further complications, such as is the priority assessment as it can assist in the evaluation of the other options.
pulmonary edema and congestive heart failure. Options 1, 3, and 4 are correct but are not the
highest priority. Teaching the client about the treatment plan is important but is not the
highest priority (option 1). The client needs to know about a low‐sodium diet and appropriate
food selection (option 3). Assessing the client for signs of dependent edema is important, but
intake and output of all fluids is higher (option 4).

1.‐ Teaching the client the importance of following the treatment plan.
2.‐ Monitoring the intake and output of fluids carefully for each shift.
3.‐ Teaching the client about low‐sodium diets.
4.‐ Assessing the client for signs of dependent edema.

1992 A characteristic of essential hypertension is: Correct answer: 4 The kidneys will not excrete water and sodium unless there is an adequate pressure gradient. Focus on the characteristic of essential hypertension as increased fluid volume and
The mean arterial pressure rises between 40 and 60 percent in essential hypertension (option sodium.
1). Blood flow resistance rises between two‐ and four‐fold in essential hypertension (option 2).
The client with essential hypertension has a normal cardiac output (option 3).

1.‐ A 90 percent increase of the arterial pressure.


2.‐ The decrease in resistance of the blood flow through the kidneys.
3.‐ The rise in cardiac output.
4.‐ That the kidneys will not excrete water and sodium unless the arterial pressure is high.

1993 Raynaud's disease is characterized by: Correct answer: 1 Raynaud's disease occurs predominately in young women between the ages of 20 and 40. Recall the association of this condition with females of childbearing years.
Causes are usually unknown or genetic (option 2). Raynaud's disease is known as a progressive
disease, which becomes worse over time (option 3). Intermittent claudication is not associated
with Raynaud's disease (option 4).
1.‐ Onset between 20 and 40 years of age, predominately in women.
2.‐ A heavy smoking history.
3.‐ An intermittent course for the disease.
4.‐ Intermittent claudication.

1994 The nursing management of Raynaud's disease has as Correct answer: 3 The major therapeutic intervention that has the highest priority is to keep the client's hands Look for a response that identifies cold as the trigger for symptoms.
its highest priority: and feet warm as well as free from injury. The hands and feet must be kept from exposure to
cold temperature, which is a trigger stimulus. The client with Raynaud's disease does not have
intermittent claudication (option 1). Opioids are usually not needed in Raynaud's (option 2).
Stress management is considered important because anxiety may be a triggering stimuli
(option 4).
1.‐ Providing rest during the periods of intermittent claudication.
2.‐ Providing opioid analgesics for pain.
3.‐ Keeping the client's hands and feet warm and injury‐free.
4.‐ Teaching the client stress management.
1995 Buerger's disease is characterized as: Correct answer: 1 Buerger's disease is a vascular disease of the small‐ and medium‐sized peripheral arteries that Look for the word occlusive in the correct response to make a choice.
become inflamed, thrombotic, and spastic. This disease is primarily found in the legs and feet.
The exact cause of Buerger's disease is unknown; however, it is possible that it is an
autoimmune response. There is a high incidence of the disease in heavy smokers and it occurs
in men under the age of 40 (option 2). Buerger's disease has an intermittent course
characterized by exacerbations and remissions. Over time, the severity and duration of the
attacks become more severe (option 3). The hands of the client are rarely affected (option 4).

1.‐ An occlusive vascular disease with an autoimmune response.


2.‐ Occurring in men over 65 years of age.
3.‐ Progressively worsening over time.
4.‐ Affecting only the hands.

1996 Nursing management of a client with Buerger's Correct answer: 4 When planning for the nursing management of a client with Buerger's disease, managing the Look for the most serious of the options, which is altered tissue perfusion.
disease has as its highest priority: altered peripheral tissue perfusion is primary. Because the arterial circulation is thrombosed,
inflamed, and spastic, there is limited ability to carry oxygen and nutrients to the tissues and
return with the waste products of metabolism. All options are correct but the stem asks for
the highest priority. The risk of injury to the tissue may be due to tissue hypoxia. The client
must be encouraged to completely stop smoking (option 1). The client will probably be placed
on bed rest because of the impaired activity and immobility during the exacerbations of
symptoms (option 2). Pain is caused by the intermittent claudication. This, in turn, is caused by
the vasoconstriction and vasospasm of the arteries following inflammation of the arteries
(option 3).

1.‐ Prevention of injury.


2.‐ Limiting activity.
3.‐ Pain management.
4.‐ Managing altered tissue perfusion.

1997 A compensatory mechanism of the body in a client Correct answer: 4 As the muscle hypertrophies in order to contract forcefully enough to overcome the high Hypertrophy of the left ventricle is discussed in the scenario. Recall that this is associated
with essential hypertension is the development of pressure and peripheral vascular resistance, this mechanism eventually requires more with CHF.
hypertrophy in order to increase the workload of the oxygenated blood to the heart. The increased workload will cause the heart to "tire out" and
left ventricle. Over a long period, however, this congestive heart failure can occur. What was a compensatory mechanism of the heart
increases the risk of which of the following? becomes a complication. The stiffened muscle cannot produce the necessary cardiac output
and becomes congested. Options 1, 2, and 3 are not compensatory mechanisms.

1.‐ Pulmonary embolus


2.‐ Cardiac tamponade
3.‐ Myocardial infarction
4.‐ Congestive heart failure (CHF)

1998 The first priority of care in the management of a Correct answer: 1 Anticoagulant therapy is started early to prevent the extension of the thrombus or the Associate thrombophlebitis with anticoagulant therapy. This is a priority over monitoring
client with thrombophlebitis is: possible embolization of the thrombus. All options are correct but option 1 is the highest of the blood work. Monitoring the blood studies will aid in the medication management.
priority. Elevation of the client's legs is a comfort measure, and the elastic bandages will
provide support to the extremity. The client's heel must be included in the wrap. These
bandages are to be applied snuggly and rewrapped every 4 to 8 hours. They must be inspected
frequently as they may become dislodged (option 2). The use of warm packs and an analgesic
are appropriate but not as the first priority (option 3). The monitoring of the PTT or the INR
will be done every 4 hours (option 4).

1.‐ Anticoagulant therapy with heparin or coumadin (Warfarin).


2.‐ Elevation of the client's legs, which are supported by elastic bandages from toes to groin.
3.‐ Using warm packs and a sedative.
4.‐ Monitoring effect of anticoagulant therapy every 36 hours by using the partial thromboplastin time (PTT) or the International Normalized Ratio (INR).
1999 The first priority in assessing a client with Correct answer: 2 The nurse will assess the lower extremities for swelling. Frequently, measurement of the mid‐ Recall that inspection is the first step in assessment.
thrombophlebitis is: calf is obtained every eight hours. Homan's sign is considered somewhat unreliable. It is
reported that approximately 35 percent of clients with deep venous thrombosis will have a
positive Homan's sign (option 1). The extremity may be reddened in the area of the thrombosis
if a superficial vein is involved. The client may or may not report calf tenderness (option 3). A
pulse is usually palpable or audible by doppler unless an arterial clot is present (option 4).

1.‐ Homan's sign because it is reliable.


2.‐ Inspection of the lower extremities for swelling.
3.‐ Color change and tenderness.
4.‐ Palpation of a pulse.

2000 Education for a client with varicose veins should Correct answer: 1 Teach the client not to sit for prolonged periods of time (more than 30 minutes) or stand Recall that activity is a key element in treatment/control of varicose veins. Look for that
include which one of the following? (more than 5 minutes) without changing positions. Instruct the client not to cross the legs word to help identify option 1 as the correct answer.
when sitting and to elevate the legs if prolonged standing is needed. Medication is usually not
recommended with varicose veins (option 2). Protection from cold is needed in Raynaud's and
Buerger's diseases, not for varicose veins (option 3). The management of the client's pain may
be of concern but should be controlled by over‐the‐counter medications and regulating
activity. The pain is caused by the venous pooling of blood and the lack of nutrients and oxygen
to the tissues. If the pain is worsening, the client's condition needs further assessment and
possible modification of the treatment regimen (option 4).

1.‐ Activity tolerance


2.‐ Medication compliance
3.‐ Protecting extremities from cold
4.‐ Pain management

2001 The pathophysiology of varicose veins is explained by Correct answer: 2 The major pathophysiological factor in the development of varicose veins is the prolonged Recall that venous pressure is the key to identification of the correct option.
which of the following? increased pressure within the venous structure. The process is compounded by the venous
values becoming stretched and unable to close, creating a pooling of venous blood. Blood flow
is compromised, and is moving slower in the venous system of the lower extremities (option
1). Varicose veins involve the venous, not arterial, system (option 3). The intra‐abdominal
pressure is constant and compresses the venous system of the lower extremities (option 4).

1.‐ Chronic status of blood in the vessels


2.‐ Chronic increased intravenous pressure
3.‐ Increased arterial pressure
4.‐ Intermittent intra‐abdominal pressure on the veins

2002 A nurse is able to distinguish venous insufficiency Correct answer: 4 The pulses with a venous insufficiency are normal or decreased; however, the pulses for an Look for a reference to pulses to help identify the correct response, as the quality of the
from arterial insufficiency by noting: arterial ulcer are diminished or absent. Warmth usually indicates inflammation and possible peripheral pulse is a distinguishing factor in determining venous versus arterial disease.
thrombophlebitis (option 1). The skin appearance is brown with a venous stasis, and cyanotic
when placed in a dependent position (option 2). The level of pain the client is reporting is
usually a mild, achy pain with venous insufficiency (option 3).

1.‐ Warmth to the calf.


2.‐ Red skin appearance.
3.‐ Severe pain.
4.‐ Pulses.
2003 The nurse is monitoring a client who recently has Correct answer: 2 When cardiac tamponade occurs, the restriction reduces stroke volume, cardiac output, and The identification of a narrowing of pulse pressure is the key to identifying option 2 as
undergone pericardiocentesis. The nurse suspects blood pressure. The right atrium is restricted, causing JVD and increasing pressure during correct.
cardiac tamponade after observing which of the diastole. While the decreased stroke volume decreases the pressure during systole, the client
following? compensates for decreased stroke volume and cardiac output by increasing heart rate.
Because of decreased filling pressure, cardiac output drops, and blood pumped from the right
heart is reduced. Lung sounds are usually clear; heart sounds become more distant and
muffled because they are heard through the fluid collection in the pericardium.

1.‐ A rapid increase in blood pressure, and flushing


2.‐ Jugular vein distention (JVD) and narrowing pulse pressure
3.‐ Bradycardia and bilateral crackles
4.‐ Louder and harsher heart sounds

2004 A 54‐year‐old male client recently was diagnosed with Correct answer: 2 Once a client is diagnosed with SBE, he is at risk for repeated episodes. Taking prophylactic Recall that antibiotic prophylaxis is part of ongoing treatment, and should be included in
subacute bacterial endocarditis (SBE). The nurse antibiotics prior to dental care is an important activity to prevent further infections. There is teaching.
determines that the client understands the discharge no routine sodium restriction with SBE. Antibiotic treatment for SBE is given by the IV route for
teaching when he does which of the following? the entire course. Although stopping smoking will decrease his risk factor for coronary artery
disease, it does not affect the SBE.
1.‐ Asks for a referral to a dietician for a low‐sodium diet.
2.‐ Explains to his wife why he needs antibiotics before seeing the dentist.
3.‐ Asks when he can start to take his antibiotics in pill form.
4.‐ Explains his plans to quit smoking.

2005 The nurse on a cardiac unit is caring for a client Correct answer: 3 Pulmonary edema in a client with heart failure is the accumulation of fluid in the alveoli Recall that pink, frothy sputum often is associated with pulmonary edema.
admitted with an acute exacerbation of heart failure. characterized by increased rales; tachypnea; tachycardia; pink, frothy sputum; and decreased
The nurse concludes that the client is developing SO&lt;sub&gt;2&lt;/sub&gt; and PO&lt;sub&gt;2&lt;/sub&gt;. The client presents with acute
pulmonary edema after observing which change in the restlessness and anxiety. Urine output generally is decreased in heart failure clients; increased
client? urinary output usually is caused by diuretic therapy.

1.‐ Bradycardia
2.‐ Increased urination
3.‐ Cough with pink, frothy sputum
4.‐ Increased sleepiness

2006 A client is scheduled for a cardiac angiography. In Correct answer: 1 The dye typically used for cardiac angiography is iodine‐based. The client with known allergy Recall that seafood allergy is associated with problems with angiography dye. Look for an
reviewing the client's record, what significant finding to seafood is at risk for anaphylaxis, and requires alternate media; atrial fibrillation and chronic option that reflects this.
needs to be reported to the physician before the renal failure are not contraindications to cardiac angiography; 5.0 mEq/L is a normal value for
exam? potassium.
1.‐ The client reported an allergy to shrimp.
2.‐ The client's ECG shows atrial fibrillation.
3.‐ The potassium level is 5.0 mEq/L.
4.‐ The client has a history of chronic renal failure.

2007 The nurse is developing a plan for a client who is Correct answer: 3 Daily weight is the most sensitive indicator of changes in fluid status. It is more accurate for a Recall that the best monitoring of fluid status is weight. Identify the option that talks about
going home with a new diagnosis of heart failure. The client at home than is urine output. A fluid restriction may be recommended for a client with accurate measurement of daily weights.
nurse is teaching the client to monitor fluid status. The advanced heart failure, but it is not a method of monitoring fluid status. The client should
best instruction is to teach the client to do which of never adjust the dose of her medications.
the following?
1.‐ Restrict fluid intake to 800 mL per day.
2.‐ Increase the dose of diuretics if there is decreased urination.
3.‐ Record body weight every day before breakfast, and report a weight gain of 3 or more pounds in a week.
4.‐ Keep track of daily output, and call the doctor for if it is less than 1 L on any day.
2008 The nurse is caring for a client who has just had a Correct answer: 2 Bedrest is prescribed to allow the arterial puncture to seal and reduce the risk of bleeding. Identify a reference to bleeding as the reason for bedrest.
cardiac catheterization. The client insists on getting up Explaining the rationale to the client is the best way to facilitate the client's cooperation.
to go to the bathroom to urinate immediately when he Although the factual information in the other options might be true, they do not assist the
is brought back to his room. Which of the following client to understand the basis for care restrictions.
would be the nurse's best response?

1.‐ "You can't walk yet. You might be too weak after the procedure, and may fall."
2.‐ "If you bend your leg, you will risk bleeding from the insertion site. It is an artery, and it could lead to complications."
3.‐ "If you get out of bed, you could have an arrhythmia from the catheterization. Your heart has to rest after this procedure."
4.‐ "The doctor has ordered that you stay on bedrest for the next six hours. It is important that you follow these orders."

2009 A client is getting ready to go home after a Correct answer: 4 Metoprolol (Lopressor) is a beta blocker, and it slows heart rate; the main therapeutic effect Recall that the word ending “‐olol” helps to identify the medication as a beta blocker.
myocardial infarction (MI). The client is asking after an MI is to reduce cardiac workload. It does not dilate the coronary arteries, and it Option 4 describes the action of a beta blocker.
questions about his medications, and wants to know actually decreases contractility (strength of the heartbeat).
why metoprolol (Lopressor) was prescribed. The
nurse's best response would be which of the
following?
1.‐ "Your heart was beating too slowly, and Lopressor increases your heart rate."
2.‐ "Lopressor helps to increase the blood supply to the heart by dilating your coronary arteries."
3.‐ "This medication helps make your heart beat stronger to supply more blood to your body."
4.‐ "It slows your heart rate and decreases the amount of work the heart has to do, so it can heal."

2010 A client is taking digoxin (Lanoxin) and furosemide Correct answer: 1 A prudent diet would be high in potassium because digoxin and furosemide both can deplete Omit options 2, 3, and 4 as being too high in sodium content.
(Lasix) for heart failure. Which of the following would potassium. The diet needs to be low in sodium to prevent additional fluid overload with heart
be the best menu choices for this client? failure. Chicken, potato, and cantaloupe are all potassium‐rich foods, and options 2, 3, and 4
are higher in sodium.
1.‐ Chicken with baked potato and cantaloupe
2.‐ Eggs and ham
3.‐ Grilled cheese sandwich and French fried potatoes
4.‐ Pizza with pepperoni

2011 A nurse is preparing to admit a client with restrictive Correct answer: 4 Although some clients might have fear, hopelessness, or knowledge deficit related to their Identification of the option that would apply to all clients with this diagnosis will lead to
cardiomyopathy to the hospital for the management disease progression, most clients with cardiomyopathy are likely to have decreased cardiac selection of option 4 as correct.
of worsening heart failure. Which of the following output and corresponding activity intolerance. More data would be needed to determine
would be the most appropriate nursing diagnosis for whether the other nursing diagnoses apply.
this client?
1.‐ Fear related to new onset of symptoms
2.‐ Hopelessness related to lack of cure and debilitating symptoms
3.‐ Knowledge Deficit related to medication regime
4.‐ Activity Intolerance related to decreased cardiac output

2012 The nurse is preparing to utilize an external Correct answer: 4 A client who is in ventricular fibrillation requires immediate defibrillation; a client with atrial Recognize the need to omit all options that require the use of a defibrillator for either
pacemaker for a client with a dysrhythmia. The nurse fibrillation might require synchronized cardioversion; a client with ventricular tachycardia defibrillation or cardioversion; that would be options 1, 2, and 3.
knows that this pacemaker often is necessary when a could require defibrillation. The client with second‐degree heart block is the client in this group
client is in which of the following cardiac rhythms? most likely to need a pacemaker.

1.‐ Ventricular fibrillation


2.‐ Atrial fibrillation
3.‐ Ventricular tachycardia
4.‐ Second‐degree heart block
2013 A client with hypertension has a blood pressure of Correct answer: 4 Blood pressure should be consistently below 140/90. Lifestyle modification must be used in Look for the option that will continue the lifestyle modifications but add medications.
170/96 after six months of intensive exercise and diet all hypertensive clients, with or without medication therapy.
modifications. The nurse advises the client:

1.‐ To continue the current treatment plan, as his blood pressure is being adequately controlled.
2.‐ To discontinue the current treatment plan, as it has not been effective, and medications will be required.
3.‐ To increase his exercise twofold and continue dietary modifications to attempt to lower blood pressure further.
4.‐ That medication therapy likely will need to be started along with the exercise and diet program.

2014 Evidence that the outcome of increased arterial blood Correct answer: 1 Pain of arterial occlusive disease is related to interrupted blood flow, which causes tissue Carefully read the option, focusing on the qualifier paired with the symptom. Reduction of
supply to the extremity has been met in a client with hypoxia. An increase in blood supply, then, should reduce the client's ischemic pain. The other muscle pain is the only option that applies to increased blood supply.
peripheral arterial disease includes: options list additional manifestations of peripheral arterial disease.
1.‐ Reduced muscle pain.
2.‐ Reduced sensation to touch.
3.‐ Increased rubor.
4.‐ Decreased hair on the extremity.

2015 In teaching a hypertensive client about the side Correct answer: 3 Beta adrenergic blocking agents, such as propranolol, cause a decrease in heart rate and Recall that the word ending “‐olol” often is associated with beta‐blocking agents. Option 3
effects of propranolol (Inderal), the nurse plans to decreased contractility, which can result in bradycardia or heart failure. Constipation is a side is reflective the decrease in contractility associated with the use of these drugs.
include which side effect of this medication therapy? effect of therapy with some of the calcium channel blockers, while hypokalemia increases risk
of digitalis toxicity.
1.‐ Hypokalemia
2.‐ Constipation
3.‐ Heart failure
4.‐ Tachycardia

2016 A client is at high risk for developing deep vein Correct answer: 4 The classic manifestations of a deep vein thrombosis are calf or groin pain, which might or Look for the option that includes pain, recalling that it often is associated with DVT.
thrombosis. For which of the following manifestations might not be associated with leg swelling. The other options describe symptoms of arterial
does the nurse assess? disease.
1.‐ Absent pulse and pale extremity
2.‐ Ulcerated toes and rubor
3.‐ Cyanotic extremity and numbness
4.‐ Leg swelling and calf pain

2017 A client on furosemide (Lasix) therapy demonstrates Correct answer: 4 Orange juice is an excellent source of potassium. Coffee will adversely elevate blood pressure. Recall that oranges are high in potassium.
understanding of how to increase potassium in his diet Milk is high in sodium. Cranberry juice is not as high in potassium as is orange juice.
when he states he will add which of the following
beverages to his diet?
1.‐ Milk
2.‐ Cranberry juice
3.‐ Coffee
4.‐ Orange juice

2018 Which of the following findings in a client awaiting Correct answer: 1 The primary symptom of a dissecting aneurysm is sudden, severe pain. Abdominal dissections The words “severe pain” in the distractor help to identify a reason to contact a physician.
abdominal aortic aneurysm repair would you report commonly cause back pain. The other responses do not address this emergency.
immediately to the physician?
1.‐ Severe back pain
2.‐ Swelling of the arms and face
3.‐ Increased blue areas of the feet
4.‐ Hoarseness or difficulty swallowing
2019 Which of the following laboratory values is most Correct answer: 2 Heparin dose concentration and number of units per milliliter per hour are ordered to Always associate heparin therapy with monitoring of PTT.
important for the nurse to assess to monitor maintain a therapeutic PTT. The other responses are incorrect.
therapeutic levels of heparin therapy?
1.‐ Prothrombin time (PT)
2.‐ Partial thromboplastin time (PTT)
3.‐ Clotting time
4.‐ Bleeding time

2020 A client complains of pain and cramping after short Correct answer: 3 Intermittent claudication caused by muscle ischemia is a primary symptom of peripheral The description of the pain in the question will help to identify option 3, intermittent
periods of walking, symptoms that stop when he rests. arterial disease. Pain occurs with activity but is relieved with rest. The other options are not claudication, as correct.
The nurse concludes he is describing which of the associated with this disorder.
following symptoms of peripheral arterial disease?

1.‐ Arterial–venous shunting


2.‐ Phlebitis
3.‐ Intermittent claudication
4.‐ Raynaud's phenomenon

2021 Which nursing activity would be important to add to Correct answer: 1 Clients with orthostatic hypotension are at risk for dizziness and syncope if they arise quickly. Recall that controlling change of position is key for clients with orthostatic hypotension.
the plan of care for an older adult suspected of having Option 3 is a correct action, but does not relate directly to orthostatic hypotension. Blood
orthostatic hypotension? pressure should also be taken while the client is sitting and standing (option 2). Option 4 is
unrelated to the question.
1.‐ Teaching the client to get out of bed slowly
2.‐ Monitoring all blood pressure readings when the client is lying down
3.‐ Taking blood pressure readings in both arms
4.‐ Teaching the client about the use of sublingual nitroglycerin

2022 When educating the client with primary hypertension, Correct answer: 4 A common complication of hypertensive disease is target organ disease, including retinal Omit options 1 and 2 as being inappropriate methods of treating and monitoring
the nurse instructs the client to: damage to the eye. The appearance of the retina can provide important information about the hypertension.
severity of the hypertensive process.
1.‐ Take anti‐hypertensive medications when blood pressure is elevated.
2.‐ Monitor blood pressure annually.
3.‐ Avoid foods with concentrated sugars.
4.‐ Have regular eye exams.

2023 A client is being treated for new‐onset heart failure Correct answer: 2 A side effect of digoxin and furosemide is that they promote the excretion of potassium, and Focus on the combination of two drugs that contributes to hypokalemia.
with a sodium‐controlled diet, digoxin (Lanoxin), and a U wave is a sign of hypokalemia. The other options are incorrect.
furosemide (Lasix). The ECG monitor shows a new U
wave. Based on this new finding, the nurse determines
that it is important to check which of the following
laboratory test results?

1.‐ Sodium
2.‐ Potassium
3.‐ Calcium
4.‐ Magnesium

2024 The nurse is caring for a client who has a central Correct answer: 1 The transducer must be at the same level as the right atrium to obtain an accurate Select the option that stresses the importance of aligning the transducer with the right
venous pressure (CVP) monitor. The nurse prepares to measurement. It is the nurse's responsibility to level the CVP transducer to this point at regular atrium.
measure the CVP by placing the client in which intervals according to policy and before each measurement.
position?
1.‐ Supine, with the transducer at the level of the right atrium
2.‐ Sitting in the chair, with the transducer at the level of the left ventricle
3.‐ On the left side, with the head of the bed flat
4.‐ In any position, since it does not affect readings

2025 The nurse on a telemetry unit is caring for a client Correct answer: 2 Vitamin K is not associated with the incidence of PVCs. There is no evidence in the question Recall that caffeine and nicotine are known to increase myocardial irritability.
who has infrequent premature ventricular contractions that this client has a potassium deficiency, or is at risk for one. It is recommended that clients
(PVCs) noted on the continuous ECG monitor. In having PVCs eliminate caffeine and nicotine from their diets.
preparing discharge instructions, which of the
following statements is most appropriate to say to this
client?
1.‐ “Make sure that you have several servings of green, leafy vegetables daily to keep up your intake of vitamin K."
2.‐ "Avoid caffeine and nicotine, because they can contribute to your irregular heartbeats."
3.‐ "Make sure that you have foods rich in potassium, because decreased potassium could be the cause of your irregular heartbeats."
4.‐ "There are no dietary recommendations for your irregular heart rhythm."

2026 The nurse is caring for a client with atrial fibrillation. Correct answer: 3 Atrial fibrillation is characterized by irregularly irregular QRS complexes and rhythm. There is Omit options 1, 2, and 4, as they describe a regular rhythm, which is not descriptive of
The nurse is administering digoxin (Lanoxin), and is no underlying regular rhythm with atrial fibrillation. atrial fibrillation.
assessing the apical pulse. The nurse expects to hear
which of the following types of rhythms?

1.‐ A very rapid, regular rhythm


2.‐ A regular rhythm with intermittent irregular beats
3.‐ A rhythm that is irregularly irregular
4.‐ A regular rhythm with intermittent pauses

2027 The nurse in the emergency room is administering Correct answer: 1 Cerebrovascular accident (CVA) is a very serious complication of thrombolytic therapy. The Select the option that would detect a serious bleed to answer correctly.
tPA (Activase) for an acute myocardial infarction. In most important intervention to detect this complication is frequent assessment of neurological
order to avoid a serious complication of this status. Testing for occult blood is important with these clients to detect GI or urinary tract
treatment, the nurse plans to do which of the bleeding, a less serious complication. PTT monitoring does not detect a specific area of
following? bleeding. Teaching a client to use a soft toothbrush is important to prevent bleeding gums, a
minor complication of this therapy.

1.‐ Assess the client's neurological status frequently.


2.‐ Test stools, urine, and blood for occult blood.
3.‐ Monitor the client's PTT.
4.‐ Teach the client to use a soft toothbrush.

2028 The homecare nurse is caring for a client with Correct answer: 4 Prognosis often is poor with advanced cardiomyopathy, and little can be done to increase the Focus on the option that is open‐ended and allows expression of feelings on the part of
cardiomyopathy whose symptoms have become more client's activity level. The symptoms usually become worse as the disease progresses. the client.
severe over the last year. On the first visit, the client Irritability and withdrawal could be signs of feelings of inadequacy or despair. Validating the
reports extreme fatigue and dyspnea with any activity. difficulty of the client's experience is an intervention to create an environment of acceptance
The client is irritable and withdrawn. The best and empathy.
response by the nurse is which of the following?

1.‐ "Let's see what we can do to increase your energy."


2.‐ "Have you tried resting frequently?"
3.‐ "Sometimes these symptoms improve with time."
4.‐ "It must be difficult to experience these changes."
2029 The nurse is caring for a client with subacute bacterial Correct answer: 3 Sudden‐onset dyspnea, anxiety, and tachycardia are signs of pulmonary embolism, a serious The question is asking for a complication of SBE. Focus on the most severe set of
endocarditis (SBE). In assessing for complications of complication of SBE. Chills and fever could be symptoms of SBE; bleeding gums and occult symptoms that would be associated with SBE.
SBE, the nurse is alert for which symptom(s)? blood are not symptoms of a direct complication of SBE; the client with SBE usually has a
normal WBC.
1.‐ Chills and fever
2.‐ Bleeding gums or occult blood in stools
3.‐ Sudden onset of dyspnea, anxiety, and tachycardia
4.‐ Increased white blood cell (WBC) count

2030 The homecare nurse is visiting a client who has heart Correct answer: 3 In a client with heart failure, a weight gain of 3–5 pounds over a week is a significant indicator Look for a response that would indicate the nurse is assessing the use of diuretics.
failure. The client denies any changes in the way she of an increase in retained fluid. It is not appropriate to provide false reassurance to a client.
feels. The nurse notes that the client has gained 3 The fluid increase indicates that the therapeutic regime is not adequate for this client. It is
pounds in the last week, and the client is concerned important for the nurse to ascertain if the client has been taking her prescribed diuretics, and
about the weight gain. Which of the following would to consult with the primary care provider before the client's fluid overload becomes excessive.
be best for the nurse to say at this time? Diet alone is not adequate to treat this increase in fluids.

1.‐ "Let's go over your diet for the last week and see if we can plan menus for next week that are lower in fat."
2.‐ "I can't even tell that you gained the weight. Three pounds isn't really a problem."
3.‐ "Tell me what medications you took this week."
4.‐ "What did you eat differently this week?"

2031 The nurse is caring for a client who recently was Correct answer: 3 Pain usually is the first presenting sign of new or extended MI, which is a very serious Identify the option that addresses the priority of post‐MI care, pain.
admitted to the intermediate care unit with a complication for this client. Activity order for a client immediately post‐MI usually is bedrest or
myocardial infarction (MI). The nurse is most commode privileges. Although an important client outcome is to be free from life‐threatening
concerned about achieving which client outcome? dysrhythmias, clients frequently have benign dysrhythmias after an MI, and many are not in
normal sinus rhythm. Maintaining a balanced intake and output is important, but not as critical
as remaining pain‐free.
1.‐ The client will ambulate in the room without fatigue.
2.‐ The client will maintain a balanced intake and output.
3.‐ The client will be pain‐free.
4.‐ The client will have a normal sinus rhythm.

2032 The nurse is caring for a client with continuous ECG Correct answer: 4 The rhythm shown in the figure is ventricular fibrillation. This is a cardiac emergency, and The scenario indicates that the client is in an emergent situation. Options 1 and 2 are not
monitoring. The nurse observes that the client's immediate defibrillation is the recommended response. Administration of intravenous priorities of care at this time. Recognize that defibrillation is the only treatment.
rhythm has changed to the rhythm shown. After lidocaine is recommended for ventricular tachycardia. Checking vital signs and calling the
activating the emergency response system, the next physician are included once the emergency response system is activated.
best action by the nurse is to:
1.‐ Call the physician and report the change in rhythm.
2.‐ Check the client's blood pressure.
3.‐ Administer intravenous lidocaine.
4.‐ Immediately defibrillate the client.

2033 The nurse places highest priority on implementing Correct answer: 4 Pulses are assessed frequently to ensure that adequate circulation is present and an occlusion Recall that assessment is the first step in the nursing process. Option 4 reflects an
which of the following measures on the surgical unit or leakage of the graft has not occurred. Pulses should be marked preoperatively so the nurse assessment.
on the first postoperative day following surgical repair has a comparison point postoperatively. Pulses could be absent for the short‐term
of an aneurysm? postoperatively due to vasospasm or hypothermia.
1.‐ Administer anticoagulant therapy.
2.‐ Position the legs in Trendelenburg position.
3.‐ Apply elastic stockings to both legs.
4.‐ Palpate peripheral pulses q2–4h.
2034 A client with venous stasis ulcers is being treated with Correct answer: 1 Elevation of the extremities promotes venous return. Pulses are assessed to ensure adequate Eliminate options 2 and 4, as dependency in the position of the leg is contraindicated.
an Unna boot. Other interventions should include: circulation. Option 3 is unnecessary because the Unna boot is treating the ulcer and is changed
every 1–2 weeks.
1.‐ Elevating legs and assessment of peripheral pulses.
2.‐ Keeping legs dependent for pain relief and improved circulation.
3.‐ Wet to dry dressings to ulcer b.i.d.
4.‐ Elevating legs and standing as much as possible.

2035 In planning care for a client receiving warfarin sodium Correct answer: 4 Aspirin is an antiplatelet agent, and its properties will increase the risk of bleeding while the Recall the association of Coumadin with a contraindication of aspirin products.
(Coumadin), the nurse instructs the client to: client is taking anticoagulant therapy. The medication does not place the client at risk for
infection (option 1) or directly affect blood pressure (options 1 and 4).

1.‐ Monitor blood pressure weekly.


2.‐ Observe for signs and symptoms of infection.
3.‐ Get up slowly from the chair or bed.
4.‐ Avoid aspirin and aspirin‐containing products.

2036 For a client with deep vein thrombosis receiving Correct answer: 3 During heparin therapy, the therapeutic PTT equals 1.5–2.0 times the control value. The other Omit option 1, as the PTT should increase with therapy. Omit option 4, as this is a large
heparin sodium, the nurse evaluates the partial options do not reflect this ratio. variance.
thromboplastin time (PTT) as therapeutic when the
results are noted as:
1.‐ Control = 25 seconds, PTT = 20 seconds.
2.‐ Control = 20 seconds, PTT = 25 seconds.
3.‐ Control = 25 seconds, PTT = 50 seconds.
4.‐ Control = 30 seconds, PTT = 90 seconds.

2037 A client reports burning and numbness of the hands. Correct answer: 3 In Raynaud's disease, vasospasm causes the digits to turn blue, then white. As the vasospasm Recall that changes to hand color are often associated with Reynaud’s disease.
After noting them to be bluish‐white, changing to very ends and circulation returns, the digits become very red and warm. The other disorders listed
red, the nurse concludes that the findings likely do not have these manifestations.
represent:
1.‐ Intermittent claudication.
2.‐ Carpal tunnel syndrome.
3.‐ Raynaud's disease.
4.‐ Arterial occlusion.

2038 Evidence that the outcome of "restore tissue Correct answer: 4 A goal of venous ulcer care is for the client to experience no signs of inflammation or The question is referring to a stasis ulcer. Focus on no infection as a result of restoration
integrity" has been met in a client with a venous stasis infection. This is the goal that is directly related to tissue integrity. The other options are good of tissue integrity or healing.
ulcer includes: outcomes but do not relate directly to the question as stated.
1.‐ Absence of bleeding.
2.‐ No reports of pain.
3.‐ Increased activity tolerance.
4.‐ No signs of inflammation or infection.

2039 Which of these statements by a client receiving Correct answer: 1 Both butter and margarine have 4 grams of fat, making the client's statement incorrect and in Omit options 2, 3, and 4 as being true and generally recommended. Recognize option 1 as
dietary instruction for atherosclerosis would indicate a need of clarification. The responses in the other options are correct. false.
need for further discussion?
1.‐ "Margarine has less fat than butter, so I will no longer use butter."
2.‐ "I will steam, bake, or broil my foods."
3.‐ "American cheese has 76 percent fat calories."
4.‐ "I will increase my consumption of fruits and vegetables."
2040 The nurse would plan to instruct the client that which Correct answer: 2 Dry mouth is a common side effect of this centrally acting adrenergic blocker, which acts to Omit option 3, as all other options have descriptors. Recall the association of dry mouth
of the following medications might require the use of reduce the flow of blood through the sympathetic nerves to the blood vessels and heart. Use with the sympathetic nervous system.
sugarless chewing gum or hard candy to relieve the of sugarless chewing gum, hard candy, and frequent sips of water will help relieve this side
side effect of dry mouth? effect.
1.‐ Nadolol (Corgard), a beta‐blocking agent
2.‐ Clonidine HCl (Catapres), a centrally acting sympatholytic agent
3.‐ Warfarin (Coumadin)
4.‐ Hydralazine HCl (Apresoline), a vasodilator

2041 Which of the following items is most important to Correct answer: 4 Smoking is a major etiological factor for development of Buerger's disease. Nicotine is a Look for the most common risk factor for complications of this disease, smoking.
include in instructions given to a client with potent vasoconstrictor, and will exacerbate peripheral vascular disorders. The other options do
thromboangiitis obliterans (Buerger's disease) to not address the issue of the question.
prevent advancement of the disease?
1.‐ Checking blood pressure weekly
2.‐ Weighing self every morning and reporting gains of more than 3 pounds
3.‐ Not driving a vehicle at night
4.‐ Smoking cessation

2042 The nurse would plan to do which of the following as Correct answer: 3 Bedrest and immobility are risk factors for the development of deep vein thrombosis. Early DVT is a common postoperative complication. Identify option 3 as the most common
the most effective measure to reduce the risk of ambulation assists venous return to the heart because of muscle movement against veins, and method of prevention.
developing deep vein thrombosis? should be implemented as soon as possible in hospitalized clients.

1.‐ Active ROM


2.‐ Passive ROM
3.‐ Ambulation as soon as possible
4.‐ Repositioning every two hours

2043 A client is prescribed sublingual nitroglycerine for the Correct answer: 4 Nitroglycerine loses potency over time when exposed to light and heat. The tablets should be Select the option that indicates that the client has learned that nitroglycerine should be
treatment of angina pectoris. What response from the kept cool, dry place, and in a dark container. Clients should get a new bottle every six months. carried with her at all times and properly stored.
client indicates that the client understands this Tablets should be taken five minutes apart; taking more that one tablet at a time can actually
medication? decrease the effectiveness of the drug, and can cause severe hypotension.

1.‐ "Will the physician give me a year's supply of nitroglycerine tablets?"


2.‐ "I will carry my nitroglycerine tablets in the inside pocket of my jacket, so they are always close."
3.‐ "I usually take three of my nitroglycerine tablets at the same time. I find that they work better that way."
4.‐ "I have a small, labeled case for a few nitroglycerine tablets that I carry with me when I go out."

2044 A client is being evaluated for a possible myocardial Correct answer: 2 ST elevations indicate immediate myocardial injury; ST depressions indicate myocardial Omit options 3 and 4 as not reflective of the acute phase of myocardial injury. Recall that
infarction. The nurse performs a 12‐lead ECG for an ischemia; a Q wave forms several days after a myocardial infarction; a U wave is a sign of ST elevations reflect injury.
episode of new chest pain. The nurse will monitor for hypokalemia.
which sign of acute myocardial injury?

1.‐ ST depressions
2.‐ ST elevations
3.‐ New Q wave
4.‐ New U wave
2045 The nurse is caring for a client who underwent a Correct answer: 1 ST depressions are a sign of ischemia. The physician should be notified immediately of any The scenario describes a change that requires an action. Recognize that option 1 is the
percutaneous transluminal coronary angioplasty signs of ischemia after PTCA. This is the best first action, after which the nurse should continue only appropriate action.
(PTCA) four hours previously. The client has no change to assess the client for chest pain. Administration of nitroglycerine without an order is not an
in the catheter site or vital signs since returning from appropriate nursing action. There is no sign of bleeding at the site; therefore, there is no
the procedure. The nurse obtains a 12‐lead ECG, and indication to apply pressure.
notes that the client has ST depressions. The client
denies any chest pain. Which of the following is the
next action that the nurse should take?

1.‐ Notify the physician.


2.‐ Continue to assess the client for chest pain.
3.‐ Administer nitroglycerine.
4.‐ Apply pressure to the catheter site.

2046 The nurse is caring for a client who is being Correct answer: 3 St. Jude Medical is a mechanical valve. Lifelong anticoagulation therapy is required with this The question asks for an evaluation that the client has understood teaching. Select option
discharged after valve replacement surgery. The client mechanical valve because there is a risk of thrombus formation. If a valve is replaced with a 3 as the only correct statement.
has a new St. Jude Medical valve, and the nurse is tissue valve, anticoagulation might be required during the immediate postoperative period,
reviewing the instructions for the client's follow‐up but not necessarily lifelong anticoagulation. It is recommended to take antibiotics prior to
care. The nurse determines that the client understands dental care.
an important aspect of responsibility in the care of this
valve when the client makes which of the following
statements?
1.‐ "I will take Coumadin for two months, and get my blood drawn every week until I stop taking it."
2.‐ "I will remind the doctor to give me a prescription for anticoagulant medication every time I go to the dentist."
3.‐ "I will need to take anticoagulant medication for the rest of my life."
4.‐ "I won't take any anticoagulant medication or blood thinners because they could cause a problem with my new valve."

2047 The nurse is caring for a client on the third Correct answer: 2 Vigorous coughing is discouraged for post‐CABG clients because it can increase intrathoracic Omit options 3 and 4, as they are not applicable to the nursing diagnosis in the scenario.
postoperative day after coronary artery bypass (CABG) pressure and cause instability in the sternal area. Incentive spirometry and deep breathing are Select option 2 as addressing the nursing diagnosis and being safe for the client situation.
surgery. Because an important nursing diagnosis for the preferred techniques for lung expansion with these clients. Premedication before
post‐CABG clients is Ineffective Breathing Pattern, ambulation will facilitate activity tolerance; auscultating the lungs will detect adventitious lung
what is the best plan by the nurse? sounds resulting from the ineffective breathing pattern, but it is not an action to encourage
effective breathing patterns.

1.‐ Ensure that the client performs deep breathing and vigorous coughing every hour.
2.‐ Ensure that the client uses the incentive spirometer every hour.
3.‐ Premedicate the client before ambulation.
4.‐ Auscultate the lungs once a shift.

2048 The nurse is caring for a client with angina pectoris Correct answer: 4 A cholesterol level higher than 200 indicates elevated cholesterol; a ratio of HDL to total Identify the only option with abnormal total cholesterol, option 4.
who was ruled out for a myocardial infarction. The cholesterol of less than 1:5 indicates increased cardiovascular risk; triglycerides higher than
nurse reviews the client's laboratory results, and plans 190 indicate increased risk. (Exception: triglycerides higher than 190 without elevated
to include dietary teaching after noting that the client's cholesterol do not indicate increased cardiac risk until they reach 250.)
lipid profile shows which of the following sets of
values?
1.‐ Cholesterol: 180; HDL: 40; triglycerides: 220
2.‐ Cholesterol: 190; HDL: 40; triglycerides: 160
3.‐ Cholesterol: 120; HDL: 25; triglycerides: 220
4.‐ Cholesterol: 220; HDL: 40; triglycerides: 190
2049 The nurse is caring for a client with new‐onset atrial Correct answer: 3 Synchronized cardioversion is most effective with new‐onset atrial fibrillation. Pacemakers Use process of elimination to omit options 2 and 4, as both refer to defibrillation, which is
fibrillation. The nurse anticipates that which of the are indicated for heart block, AICDs are used for ventricular dysrhythmias, and defibrillation is not appropriate.
following is a possible treatment for this dysrhythmia indicated for ventricular fibrillation and pulseless ventricular tachycardia.
when it first develops?
1.‐ External pacemaker application
2.‐ Insertion of automatic internal cardiac defibrillator (AICD)
3.‐ Synchronized cardioversion
4.‐ Defibrillation

2050 The nurse is assessing a client the morning of a Correct answer: 4 The client should have a light meal with no caffeine before a cardiac stress test. Options 1, 2, Select the response that reflects the needs of a client having a cardiac stress test that day.
scheduled cardiac stress test. The client reports that and 3 are incorrect because they do not follow this guideline. Recall that means a light meal with no caffeine.
no breakfast was delivered this morning, and the client
is hungry. Which of the following is the nurse's best
action?
1.‐ Bring the client coffee and toast.
2.‐ Explain that a client should have no food the morning of a cardiac stress test.
3.‐ Call the nutrition department and get the client's regular full breakfast.
4.‐ Have the nursing assistant get the client cereal with milk and orange juice.

2051 A hospitalized client has continuous ECG monitoring, Correct answer: 3 The best first action is to assess the client's level of consciousness and assess if the ventricular Recall that assessment is the first step of the nursing process. Option 3 is an assessment.
and the monitor shows that the rhythm has changed tachycardia is perfusing the body (BP, pulse). With pulseless ventricular tachycardia,
to ventricular tachycardia. Which of the following is immediate defibrillation is performed by an ACLS‐certified nurse. If the client has a good BP
the first action that the nurse should take? and pulse, and is awake and alert, the nurse may administer lidocaine as prescribed or, in
some cases, a precordial thump.
1.‐ Administer intravenous lidocaine according to emergency protocol.
2.‐ Obtain the defibrillator, and defibrillate the client.
3.‐ Quickly assess the client's level of consciousness, blood pressure, and pulse.
4.‐ Administer a precordial thump.

2052 The physician has diagnosed a myocardial infarction Correct answer: 2 Anxiety and fear are common responses to a diagnosis of myocardial infarction, because of Select an answer for which there are adequate data provided. Omit option 3 as not being
on the basis of ECG changes for a client in the the possibility of death. This prevents the client and family from absorbing the detailed client‐centered. There is no evidence that Alzheimer s disease is present.
Emergency Department. The nurse is assessing the explanations about the care being provided. Memory lapses are not a common symptom of
client frequently, and notes that the client seems myocardial infarction, and there is not adequate information to determine that this memory
forgetful, making the nurse repeat the explanations lapse is associated with Alzheimer's disease. Nurses in the Emergency Department are capable
about the ECG and non‐invasive blood pressure of explaining procedures well to their clients.
monitors. The nurse concludes that the client's
response is most likely due to which of the following
reasons?
1.‐ The client is showing signs of Alzheimer's disease.
2.‐ The client is showing signs of fear and anxiety.
3.‐ Nurses in the Emergency Department are too busy to properly explain the purpose of equipment.
4.‐ Memory lapses are common with clients experiencing myocardial infarctions.

2053 The nurse is performing an assessment on a 50‐year‐ Correct answer: 3 Fifty percent of people over the age of 50 develop varicose veins, and a major risk factor is Make the association between advancing age and standing for long periods to identify
old male who is a cashier at a local store, and who standing for long periods of time at work. The other responses do not address this concern. option 3 as correct.
often stands 6–8 hours at a time. The nurse should
inspect the client for:
1.‐ Capillary dysfunction.
2.‐ Buerger's disease.
3.‐ Varicosities.
4.‐ Aneurysms.
2054 The nurse is conducting a screening clinic for Correct answer: 4 Primary hypertension is more common in African‐Americans than in people of other ethnic Remember the association between hypertension and those of African‐American decent.
hypertension in the community. For which of the backgrounds. For this reason, this client should be evaluated carefully.
following clients should the nurse pay particular
attention to blood pressure?
1.‐ Caucasian adult female
2.‐ Latino/Hispanic adult male
3.‐ Asian adult male
4.‐ African‐American adult male

2055 When assessing a client, the nurse determines the Correct answer: 3 Blanching of the nailbed for more than 3 seconds after release of pressure could indicate Omit options 2 and 4, as neither can be diagnosed by measurement of capillary refill.
capillary refill time to be 7 seconds. The nurse reduced arterial capillary perfusion, which can be an indication of decreased cardiac output.
determines the client might be experiencing: The other options are incorrect for the time frame indicated or do not apply.

1.‐ Normal signs of aging.


2.‐ Impending stroke.
3.‐ Decreased cardiac output.
4.‐ Hypokalemia.

2056 After the first dose of an antihypertensive agent, your Correct answer: 4 Elevating the legs increases venous return to the heart, and will assist in raising the blood Eliminate option 1, as it maintains the head above the heart. Option 2 would also be
client suddenly becomes hypotensive. The nurse pressure. A semi‐Fowler's position could lower the blood pressure even further. A side‐lying eliminated as not contributing to increasing perfusion of vital organs.
should position the client: position will have no beneficial effect, and the Trendelenburg position could impair
respirations by causing upward pressure on the diaphragm by gravity.

1.‐ In a semi‐Fowler's position.


2.‐ In a side‐lying position.
3.‐ In Trendelenburg position.
4.‐ With her legs elevated 30 degrees.

2057 The nurse is planning to instruct a client on the side Correct answer: 2 Calcium channel blockers relax arterial smooth muscle, which lowers peripheral resistance The drug is being used to treat hypertension. Select the option that is symptomatic of
effects of nifedipine (Procardia) for hypertension. through vasodilation. Dizziness is a common side effect due to orthostatic hypotension. Clients hypotension.
Which side effect should the nurse include? need to be taught to change position slowly to prevent falls.

1.‐ Hypokalemia
2.‐ Dizziness
3.‐ Bleeding
4.‐ Tachycardia

2058 A client taking spironolactone (Aldactone) complains Correct answer: 3 Spironolactone is a potassium‐sparing diuretic. Hyperkalemia (potassium level higher than 5.5 Recognize that the hyperkalemic value is presented. Omit the options for which there are
of irregular heart rate, diarrhea, and stomach mEq/L) is a possible side effect. The other responses are incorrect. no data.
cramping. The client's potassium level is 7.1 mEq/L.
The nurse concludes that the client is experiencing:

1.‐ Hyponatremia.
2.‐ Hypercalcemia.
3.‐ Hyperkalemia.
4.‐ Hypernatremia.

2059 The nurse explains to a client that the goal of Correct answer: 1 Anticoagulant therapy is used for deep vein thrombosis to prevent propagation of the clot, Eliminate options 3 and 4, as they do not directly address the clot. Omit option 2, as it is
anticoagulant therapy in a client with a deep vein development of a new thrombus, and embolization. It does not dissolve the clot. It has no false.
thrombosis is to: effect on infection, and does not allow for immediate ambulation.
1.‐ Prevent embolization.
2.‐ Dissolve the clot.
3.‐ Allow immediate ambulation.
4.‐ Prevent infection.

2060 The nurse needs to explore with a client her Correct answer: 3 Sclerotherapy, the injection of a sclerosing agent into a varicose vein followed by Use the suffixes in options 2 and 4 to eliminate these options. “‐graphy” is generally
understanding of treatment options for varicose veins compression with a compression bandage for a period of time, is a common procedure for indicative of a study or diagnostic measure. “‐therapy” in option 3 helps to identify this as a
that were just described by the physician. Which varicose veins. treatment.
treatment would the nurse plan to include in this
discussion?
1.‐ Endarterectomy
2.‐ Venography
3.‐ Sclerotherapy
4.‐ Plethysmography

2061 Which of the following medications is likely to be Correct answer: 3 Because of the risk for inflammation or a blood clot, low doses of aspirin are recommended Look for a common medication used for its ability to limit clotting, like aspirin.
administered on a daily basis to a client newly for all clients with peripheral vascular disease. Aspirin has antiplatelet activity; without platelet
admitted to the clinical nursing unit who has a history aggregation, a clot cannot form.
of peripheral arterial disease?
1.‐ Acetaminophen (Tylenol)
2.‐ Ibuprofen (Motrin)
3.‐ Aspirin
4.‐ Heparin

2062 Which of the following statements would indicate a Correct answer: 4 Sensation in the feet can be diminished in clients with arterial occlusive disease. Teach the Select the option that indicates that the client understood the teaching.
positive outcome for a client with chronic arterial client to check the bathwater with her hands to prevent the risk of a burn injury. The client
occlusive disease? should stop and rest when pain is experienced (option 3). Options 1 and 2 are useful
treatments for venous disease.
1.‐ "I will keep my feet elevated above the level of my heart when I sleep."
2.‐ "I will wear my compression stockings when awake."
3.‐ "I will keep walking even when I feel pain in my legs, to increase circulation."
4.‐ "I will check the temperature of my bathwater with my hands before getting into the water."

2063 The nurse teaching health maintenance strategies for Correct answer: 1 Clients with COPD are highly susceptible to respiratory infections such as influenza, so they The critical words in the stem of the question are health maintenance. This phrase
the client with COPD should include which of the should be immunized yearly. Clients with COPD should undergo a progressive rehabilitation indicates that you should focus on the option that prevents a health problem rather than
following? program to increase their activity tolerance. Fluid restriction is not needed with COPD unless diagnoses or treats it.
there is fluid retention from another etiology.

1.‐ Yearly influenza immunizations


2.‐ Annual tuberculin skin test
3.‐ Limitation of physical activity
4.‐ Fluid restriction

2064 The nurse who is explaining the pathophysiology of Correct answer: 1 The impaired gas exchange occurring with COPD is caused by the loss of alveolar surface area The core issue of the question is the nature of the pathophysiology of COPD. Use general
COPD to a client includes the fact that alveolar available for gas exchange. Destruction of alveoli is not related to increased dead space air, nursing knowledge and the process of elimination to make a selection.
destruction results in which of the following? pulmonary emboli, or chronic dilation of bronchioles. With COPD, there is progressive
narrowing of bronchioles.
1.‐ Decreased surface area for gas exchange
2.‐ Increased dead space air
3.‐ Pulmonary emboli
4.‐ Chronic dilation of bronchioles
2065 The nurse explains to a client and family that the Correct answer: 1 Symptoms of COPD typically appear in the fifth and sixth decades of life following chronic The core issue of the question is sharing with a client and family the correct basis of the
development of COPD in a young adult is likely caused abuse to pulmonary tissues from smoking or environmental pollutants. Onset of the current health problem. Use nursing knowledge and the process of elimination to make a
by which of the following? physiological changes compatible with COPD is most often associated with a hereditary selection.
deficiency of alpha‐1‐antitrypsin, an enzyme that protects lung tissue against loss of elasticity.
Onset of heavy smoking during childhood and heavy secondary smoke exposure during
childhood are not typically associated with early onset of the physiological alterations of COPD.
Use of smokeless tobacco during childhood is associated with development of oral cancer.

1.‐ Hereditary deficiency of alpha‐1‐antitrypsin


2.‐ Onset of smoking during childhood
3.‐ Heavy secondary smoke exposure during childhood
4.‐ Use of smokeless tobacco during childhood

2066 A client who develops acute respiratory distress Correct answer: 1 One of the primary alterations occurring with ARDS is the collapse of alveoli and therefore The core issues of the question are an understanding of disease process and how to select
syndrome (ARDS) is exhibiting hypoxemia unresponsive loss of ventilation in those areas. Perfusion might be normal, but gas exchange is impaired due appropriate concepts for family teaching. Use nursing knowledge and the process of
to oxygen therapy. In explaining the client’s condition to inadequate ventilation. Surfactant production decreases with ARDS, a factor that impairs elimination to make a selection.
to the family, the nurse would incorporate which of adequate gas exchange. Air does not become trapped in hyperinflated alveoli in ARDS; instead,
the following concepts? alveoli collapse.

1.‐ Blood is shunted past alveoli with no ventilation.


2.‐ The individual has difficulty expelling air trapped in the alveoli.
3.‐ There is excess surfactant production by the alveoli.
4.‐ Thick secretions block the airways.

2067 What intervention does the nurse identify as the Correct answer: 3 Coughing, deep breathing, and adequate hydration are essential for achieving effective airway The critical word in the stem of the question is priority, which tells you that more than one
priority for the client with a nursing diagnosis of clearance. Insertion of a tracheostomy is not a primary treatment to maintain airway option could be correct, and you must choose the most important one. Use nursing
ineffective airway clearance related to tumor mass? clearance. Elevating the head of the bed might help the client to cough more forcefully, but knowledge and the process of elimination to make a selection.
head elevation alone is not an effective maneuver.
1.‐ Provide supplemental oxygen.
2.‐ Keep the head of the bed elevated.
3.‐ Teach the client coughing and deep breathing, and maintain hydration.
4.‐ Prepare the client for the possible insertion of a tracheostomy tube.

2068 When assisting with psychological issues for the client Correct answer: 1 The nurse should help the client and family to approach the diagnosis of lung cancer from a The core issue of the question is the knowledge of the interrelationship between prognosis
with lung cancer, which epidemiological factor should realistic perspective. Symptoms of lung cancer usually appear late in the course of the disease. and the communication approaches used by the nurse. Use nursing knowledge and the
the nurse keep in mind? Tumor growth does typically begin in a bronchus and progress upward, but this information process of elimination to make a selection.
has no relation to the client’s psychological adaptation to the disease.

1.‐ The five‐year survival rate for lung cancer is less than 15 percent.
2.‐ Symptoms usually occur early during lung cancer progression.
3.‐ Tumor growth usually begins in a bronchus, then migrates upward in the tissue.
4.‐ Risk of lung cancer is associated with length of exposure to cigarette smoking.

2069 The nurse administers which of the following Correct answer: 3 Administration of anticoagulants (option 3) is an effective intervention to prevent pulmonary The core issue of the question is knowledge of medication to reduce risk of pulmonary
medications as a part of pharmacological treatment embolism. Thrombolytic drugs (option 1) may be used to dissolve a clot that is already formed. embolus. Use nursing knowledge and the process of elimination to make a selection.
aimed at prevention of pulmonary embolism? Vitamin K (option 2) and protamine sulfate (option 4) facilitate clotting and counteract the
effect of anticoagulants.
1.‐ Streptokinase
2.‐ AquaMEPHYTON (vitamin K)
3.‐ Enoxaparin (Lovenox)
4.‐ Protamine sulfate
2070 In the client with respiratory distress, which finding is Correct answer: 4 Increased respiratory rate, tachycardia, and agitation are all early signs of respiratory distress. The critical word in the stem of the question is worsening, which tells you that the
most compatible with a worsening clinical state? Cyanosis develops later in the progression of respiratory distress. symptom will be a late sign, not an early one. Cyanosis is always a late sign. Use nursing
knowledge and the process of elimination to make a selection.
1.‐ Increased respiratory rate
2.‐ Tachycardia
3.‐ Agitation
4.‐ Cyanosis

2071 For the hospitalized client, which factor would the Correct answer: 3 Symptoms associated with pulmonary embolism typically have a sudden onset. The client The critical words in the stem of the question are symptom and pulmonary embolism.
nurse assess to be a symptom of pulmonary often feels panic because of the sudden dyspnea. Increase in heart rate and respiratory rate is They tell you that the question is seeking an answer that is a correct assessment. Use
embolism? abrupt, not slow. Cyanosis of the upper torso is associated with embolism of a central vein nursing knowledge and the process of elimination to make a selection.
other than the pulmonary vasculature. Bilateral wheezing is more often associated with
asthma than with pulmonary embolism.
1.‐ A slow increase in heart rate and respiratory rate
2.‐ Cyanosis of the upper torso
3.‐ Abrupt onset of dyspnea and apprehension
4.‐ Significant bilateral wheezing

2072 A client underwent a thoracentesis a few hours Correct answer: 2 The finding of crepitus at any time is associated with pneumothorax, and should be reported The core issue of the question is the ability to recognize and prioritize complications that
earlier. Which finding should the nurse report immediately to the physician. Oozing of blood from the thoracentesis puncture site is not need to be reported to the health care provider. Use nursing knowledge and the process of
immediately to the physician? uncommon, and does not require emergency intervention, as would crepitus. Diminished elimination to make a selection.
breath sounds on the affected side and fever might or might not be related to the
thoracentesis. All of these findings should be noted and reported to the physician, but the
finding of crepitus is clearly related to development of pneumothorax, and signals immediate
need for intervention by the physician.

1.‐ Oozing of blood from the puncture site


2.‐ Crepitus
3.‐ Diminished breath sounds on the affected side
4.‐ Fever

2073 The nurse who is assisting the client with obstructive Correct answer: 3 The primary physiological alterations occurring with COPD are alveolar air trapping and The core issues of the question are knowledge of the underlying changes associated with
pulmonary disease to learn effective breathing alveolar hyperinflation, which lead to alveolar rupture and loss of area available for gas COPD and how to communicate that information effectively to a client or family. Use
techniques would use which of the following exchange. Decreased surfactant production is associated with ARDS, and is not a primary nursing knowledge and the process of elimination to make a selection.
statements to explain why dyspnea occurs? alteration of COPD. Lung compliance is decreased, but this is due to the alveolar air trapping
and hyperinflation.
1.‐ “Decreased surfactant causes many of your alveoli to collapse.”
2.‐ “You have difficulty breathing in enough air.”
3.‐ “Your airways open wider on inspiration, and trap air on expiration.”
4.‐ “Your lung compliance is decreased.”

2074 A client is admitted to the hospital with a medical Correct answer: 2 Viral pneumonia is considered less serious for the client because symptoms are not as The critical words in the stem are most frequent. With this in mind, you must compare
diagnosis of viral pneumonia. The nurse assesses for apparent compared with bacterial pneumonia. Viral pneumonia is associated with options in terms of their frequency. Use nursing knowledge and the process of elimination
which of the following most frequent manifestations? nonproductive cough, low‐grade fever, normal white blood cell count, and normal or minimal to make a selection.
chest x‐ray findings. Ghon's tubercles are seen on x‐ray in clients with tuberculosis.

1.‐ Presence of Ghon's tubercle on chest x‐ray


2.‐ Nonproductive cough
3.‐ Elevated white blood cell count
4.‐ High fever
2075 The nurse would question an order for ipratropium Correct answer: 1 Anticholinergics such as ipratropium are contraindicated in clients with angle‐closure The core issue of the question is knowledge of contraindications to medication therapy.
bromide (Atrovent) ordered for a client with asthma if glaucoma because they can inhibit flow of aqueous humor and raise intraocular pressure. The Use nursing knowledge and the process of elimination to make a selection.
the client had a concurrent history of which of the other options do not address this concern.
following?
1.‐ Glaucoma
2.‐ Cushing’s syndrome
3.‐ Warfarin therapy
4.‐ Fluid retention

2076 The nurse teaches the client newly diagnosed with Correct answer: 2 Clients with mild and infrequent asthma symptoms are treated with regular daily The core issue of the question is knowledge of the rapid management of symptoms in a
asthma who has infrequent acute episodes that which administration of an anti‐inflammatory inhaler and a short‐acting beta‐agonist inhaler for quick client with asthma. Use nursing knowledge and the process of elimination to make a
medication is most effective for providing quick relief relief in acute episodes. Bronchodilators and corticosteroids as oral or inhaled medication are selection.
in acute episodes? used for clients with more severe and frequent episodes of asthma.

1.‐ Corticosteroid via metered‐dose inhaler as needed


2.‐ Beta‐agonist via metered‐dose inhaler
3.‐ Anti‐inflammatory via metered‐dose inhaler
4.‐ Daily use of a bronchodilator inhaler

2077 The nurse caring for a client diagnosed with ARDS Correct answer: 3 A primary physiological alteration occurring with ARDS is shunting of blood around The core issue of the question is knowledge of pathophysiology in the development of
considers that, in this client, impaired gas exchange is nonventilated alveoli. Alveoli collapse in ARDS, and ventilation decreases. Blood perfusing to ARDS. Use nursing knowledge and the process of elimination to make a selection.
mostly likely related to which of the following? these areas cannot undergo adequate gas exchange.

1.‐ Air trapping in the alveoli


2.‐ Accumulation of exudative fluid into the alveoli
3.‐ Shunting of blood around nonventilated alveoli
4.‐ Excessive alpha‐1‐antitrypsin

2078 A child with laryngotracheobronchitis (LTB) is being Correct answer: 2 Epinephrine is a bronchodilator used to increase the diameter of the airways. The best The core issue of the question is the expected plan of care for a client with
treated in the Emergency Department. The nurse position is semi‐ to high‐Fowler s. Corticosteroids and antibiotics may be used, but will not laryngotracheobronchitis. Use nursing knowledge and the process of elimination to make a
would plan to do which of the following to ease ease respiratory distress immediately. selection.
respiratory distress?
1.‐ Turn the child onto his side.
2.‐ Administer racemic epinephrine.
3.‐ Administer corticosteroids.
4.‐ Administer intravenous antibiotics.

2079 The parents of an infant with bronchiolitis ask the Correct answer: 1 RSV is the cause of bronchiolitis in most cases; RSV can live for several hours on nonporous The core issue of the question is the ability of the nurse to explain the rationale and use of
nurse why their baby s room has a sign on the door surfaces, and can be transferred by the hands. isolation techniques. Use nursing knowledge and the process of elimination to make a
that says “Contact Precautions,” and why the nurses all selection.
wear gowns and gloves when they hold him. What is
the nurse s best response?
1.‐ “The virus that usually causes bronchiolitis can spread to other babies if extra precautions are not taken.”
2.‐ “Your baby is very ill, and we don’t want to have another baby catch what he has.”
3.‐ “It’s because your baby is in isolation.”
4.‐ “We always wear gowns when babies are coughing.”

2080 Which of the following would be a priority nursing Correct answer: 2 Maintaining strict I &amp; O will provide immediate notification of signs of dehydration; The core issue of the question is knowledge that a client with bronchiolitis has a priority
intervention for a child with bronchiolitis? children with bronchiolitis could already have a history of poor fluid intake when initially seen need for hydration. Use nursing knowledge and the process of elimination to make a
by medical personnel. Because of respiratory difficulty, the child should be kept quiet, with selection.
limited stimulation and visitors. If the child is tachypneic, oral fluids present a risk of aspiration.
1.‐ Keep the child well stimulated.
2.‐ Maintain strict intake and output.
3.‐ Encourage visitors.
4.‐ Encourage oral fluids, if tachypneic.

2081 When taking the nursing history of a child with cystic Correct answer: 3 Meconium ileus in the newborn period is often the first indication of cystic fibrosis. The other The core issue of the question is knowledge of the association between meconium ileus
fibrosis, what piece of information about the child’s options are unrelated to this question. and cystic fibrosis in the neonate. Use nursing knowledge and the process of elimination to
newborn period would the nurse expect the mother to make a selection.
report?
1.‐ That the child required resuscitation in the delivery room
2.‐ That labor was longer than 24 hours
3.‐ That the child had a meconium ileus
4.‐ That labor was less than four hours

2082 The nurse should counsel the parents of a child with Correct answer: 1 Bronchodilators open the airways and afford easier removal of secretions. Options 2 and 3 The core issue of the question is knowledge of the proper sequence of actions when a
asthma that before performing postural drainage are unnecessary. Option 4 could be done after the procedure, if necessary. client undergoes chest physiotherapy. Use nursing knowledge and the process of
exercises, they should do which of the following? elimination to make a selection.

1.‐ Administer her bronchodilator.


2.‐ Change her clothes.
3.‐ Administer her antibiotic.
4.‐ Suction her throat.

2083 If treatment for acute epiglottis is effective, the nurse Correct answer: 4 Clear breath sounds indicate effective airway clearance and decreased mucosal swelling and The core issue of this question is correctly identifying when an appropriate outcome
would expect to record that the child: obstruction. Tripod position is a clinical manifestation of a child in distress caused by measure has been achieved. Use nursing knowledge and the process of elimination to make
epiglottitis. Pale lips and mucous membranes could indicate hypoxia. Tachypneic and a selection.
dysphonic are symptoms of the disease.
1.‐ Has pale lips and mucous membranes.
2.‐ Maintains tripod position.
3.‐ Is tachypneic and dysphonic.
4.‐ Has clear, equal breath sounds.

2084 The parents of a child with bronchopulmonary Correct answer: 5 Tracheostomy suctioning can be stressful to the child, and increases risk for hypoxia, The critical issue of this question is the appropriate length of time for suctioning without
dysplasia (BPD) are receiving home instructions on infection, and mucosal damage. Each pass of the suction catheter should be limited to no more impairing the respiratory status of a child. Use nursing knowledge and the process of
tracheostomy care. With regard to suctioning, the than 5 seconds, and the child should be allowed to rest between passes with supplemental elimination to make a selection.
nurse should advise the parents that each suction pass oxygen, if needed.
should take no longer than _____ seconds. (Write in a
numerical answer.)

2085 A toddler is being discharged after an emergency Correct answer: 3 Toddlers are naturally inquisitive, and explore things with their hands and mouths. It is The core issue of the question is the best method to provide a safe environment for a
admission for foreign body aspiration. The parents ask developmentally inappropriate to attempt to teach a toddler to stop normal hand‐to‐mouth toddler. Use nursing knowledge and the process of elimination to make a selection.
the nurse what they can do to prevent another activity. Small objects and foods should be kept out of reach.
accident. The nurse should advise the parents to:

1.‐ Watch her very carefully.


2.‐ Teach her not to eat nonfood items.
3.‐ Keep small objects and toys out of her reach.
4.‐ Avoid leaving her with teen babysitters.
2086 The nurse wears gloves when assessing a child with Correct answer: 4 Handwashing is the most important infection‐control practice, and decreases the spread of The core issue of the question is basic principles of infection control using medical asepsis.
respiratory syncytial virus (RSV). After removing the RSV and other organisms. Option 1 is unnecessary because gloves are discarded in the trash Use nursing knowledge and the process of elimination to make a selection.
gloves, what should the nurse do next? basket. Options 2 and 3 are not timely.

1.‐ Discard the gloves in the laundry basket.


2.‐ Inspect the gloves for holes or fraying.
3.‐ Remind the parents to wear gloves.
4.‐ Wash hands.

2087 A 6‐year‐old child is hospitalized following an acute Correct answer: 2 Swimming is recommended for children with asthma because prolonged expiration under The core issue of the question is an understanding of the relationship of exercise to
asthmatic episode. Which statement by the parents water is beneficial. Cromolyn sodium is used prophylactically to prevent exercise‐induced episodes of asthma. Use nursing knowledge and the process of elimination to make a
indicates that further teaching is needed? asthma, and immediate access to a rescue inhaler is also recommended. selection.

1.‐ “Next time, we ll be sure he takes his cromolyn before soccer.”


2.‐ “After this episode, he will need to quit the swim team.”
3.‐ “We think this was an exercise‐induced asthma episode.”
4.‐ “We need to make sure he has his inhaler at all times.”

2088 The nurse would anticipate administering respiratory Correct answer: 1 Aerosol therapy such as a nebulizer is frequently used during hospitalization to administer The core issue of the question is an understanding of medication routes used in children,
medications to a child hospitalized with asthma by medications. An advantage is that this route delivers medication directly to the airways. specifically those with respiratory problems. Use nursing knowledge and the process of
which of the following most frequently used routes? elimination to make a selection.

1.‐ Aerosol
2.‐ Intravenous
3.‐ Subcutaneous
4.‐ Oral

2089 The nurse anticipates using postural drainage as a Correct answer: 3 Chest physiotherapy and postural drainage for children with cystic fibrosis help loosen The core issue of the question is the purpose of doing chest physiotherapy in a child with
treatment modality for which of the following pulmonary secretions and facilitate removal from airways. They are not used for epiglottitis cystic fibrosis. Use nursing knowledge and the process of elimination to make a selection.
conditions? and bronchopulmonary dysplasia because they can increase respiratory distress in those
conditions. They will not remove the foreign body.
1.‐ Epiglottitis
2.‐ Foreign body aspiration
3.‐ Cystic fibrosis
4.‐ Bronchopulmonary dysplasia

2090 The nurse teaches a mother how to attach a spacer to Correct answer: 4 Steroids given via metered‐dose inhaler on oral mucosa increase the risk for yeast infection. A The core issue of the question is the rationale for using a spacer. Use nursing knowledge
the metered‐dose inhaler for a young child, explaining spacer avoids the mucus membranes and works directly on the airways. and the process of elimination to make a selection.
that a spacer:
1.‐ Makes the device look less intimidating to a small child.
2.‐ Makes it unnecessary to shake the inhaler before administering the drug.
3.‐ Decreases the chances for undesired side effects of medication.
4.‐ Reduces the risk for oral yeast by depositing medication more deeply into the airways.

2091 The nurse documents which of the following expected Correct answer: 2 Excess fluid in the alveoli is a manifestation of bacterial pneumonia. The sound produced by The core issue of the question is the type of adventitious breath sound that is expected in
findings after auscultating the lungs of a child with fluid in the airways is crackles. Retractions are asymmetrical chest wall movements that are pneumonia. Eliminate apnea because the focus is respiratory, not the central nervous
bacterial pneumonia? seen in any client having respiratory difficulty. Wheezes are often typical of pneumonia caused system. Eliminate retractions next because they are seen rather than heard. Choose
by RSV, or conditions where the air passages are narrowed, such as asthma. Apnea is a pause crackles over wheezes, recalling that the infection process leads to fluid accumulation, not
in respirations, which is under the control of the central nervous system. to bronchoconstriction.

1.‐ Wheezes
2.‐ Crackles
3.‐ Apnea
4.‐ Retractions

2092 An infant with respiratory syncytial virus (RSV) is Correct answer: 3 Ribavirin is an antiviral drug that causes crystallization of soft contact lenses, and is associated The core issue of the question is safe administration of ribavirin to a client. Use nursing
receiving ribavirin. While caring for this infant, the with conjunctivitis. The other options are satisfactory items in the care of this client. knowledge and the process of elimination to make a selection.
nurse should not:
1.‐ Plan to become pregnant for at least one year.
2.‐ Care for any other children.
3.‐ Wear contact lenses.
4.‐ Stay in the room with the door closed.

2093 An adolescent with asthma says she heard her doctor Correct answer: 1 A trigger for an asthma attack can be exercise, smoke, allergic irritants, or emotions. Each Knowledge of the definition of trigger and its use in asthma management will aid in
say smoking was her trigger. The adolescent asks the child's trigger is individualized, and identification can afford some degree of protection in choosing the correct answer.
nurse what that means. The nurse explains to the avoiding asthmatic episodes. The other options indicate incorrect statements.
adolescent that a trigger is:
1.‐ A substance or condition that initiates an asthmatic episode.
2.‐ The term for narrowing of the airways during an asthmatic episode.
3.‐ Another way to describe asthma.
4.‐ The rapid breathing associated with an asthma attack.

2094 The pediatric nurse is observing a new nurse perform Correct answer: 2 CPT should be performed prior to meals. Waiting until after the child has eaten can lead to Critical words are “indicates the need for additional teaching,” which signal the need for
chest physiotherapy (CPT) on a child. Which vomiting. The child should wear a layer of clothing between the chest and the hands. A an incorrect strategy. Knowledge of the correct way to administer CPT will aid in choosing
observation by the new nurse indicates the need for clapping or popping sound is expected. A variety of positions may be used during CPT. the correct answer.
the first nurse to intervene?
1.‐ The child has on only a T‐shirt.
2.‐ The nurse delayed the treatment until the child had finished breakfast.
3.‐ The nurse s hand makes a popping sound when doing percussion.
4.‐ The child is positioned in various head‐down positions.

2095 A child in respiratory distress requires intubation. The Correct answer: 2, 4, 5 The endotracheal tube is inserted through the nose or mouth to maintain a patent airway by Consider all the information that parents would need to know about endotracheal
nurse would explain the intubation to the parents by bypassing an upper airway obstruction, or to reduce respiratory distress. While the tube is in intubation.
stating that the endotracheal tube: (Select all that place, the child will be unable to talk. The child might or might not require a tracheostomy,
apply.) depending on how long the child is intubated. Intubation does not allow for more accurate
blood gas sampling.
1.‐ Is a temporary measure until the physician can perform a tracheostomy.
2.‐ Helps keep the airway open so the child can breath easier.
3.‐ Allows the doctor to assess arterial blood gases more accurately.
4.‐ Will prevent the child from speaking while the tube is in place.
5.‐ May be placed through the child’s nose or mouth.

2096 The father of a premature infant asks why oxygen Correct answer: 4 Bronchopulmonary dysplasia is an obstructive lung disease that occurs primarily when Differentiate the causative factor in each of the diseases listed in the options. Knowledge
concentrations are not higher to help his son breathe premature infants are subject to prolonged mechanical ventilation and high levels of oxygen that the prolonged use of high levels of oxygen will result in BPD will aid in choosing the
better. The nurse's best response explains the risk for therapy. Pneumonia results from bacterial or viral infections. correct answer.
development of which of the following?

1.‐ Cystic fibrosis


2.‐ Pneumonia
3.‐ Bronchiolitis
4.‐ Bronchopulmonary dysplasia
2097 A child presents to the Emergency Department in Correct answer: 2 Epinephrine by the intravenous route is a rapid‐acting beta‐adrenergic agonist that relaxes Prioritizing the administration of drugs is a key concept. Consider which drugs would have
acute respiratory distress caused by an asthmatic smooth muscles by opening airways for immediate relief of bronchospasm. Terbutaline is an an immediate effect, and which effects would be delayed.
episode. Which of the following drugs would the nurse adrenergic bronchodilator, but would take longer to act due to oral or subcutaneous
plan to administer first according to a protocol order administration. Prednisone is a corticosteroid given to decrease inflammation and bronchial
set? hyper‐reactivity. Cromolyn sodium may be used prophylactically to avoid exercise‐induced
asthma, and is of little use once an episode occurs.

1.‐ Prednisone (Deltasone)


2.‐ Epinephrine (Adrenalin)
3.‐ Terbutaline (Brethaire)
4.‐ Cromolyn sodium (Intal)

2098 A child recently was diagnosed with asthma. The Correct answer: 3 Sudden temperature change is a common asthma trigger, and snow skiing would expose the Consider all the ramifications of each activity to determine the correct activity to avoid.
nurse and mother are discussing physical activities for child to cold air. Exercise is a trigger in some children, but activities with short bursts like
the child. The nurse will recommend that the child gymnastics are not usually a problem. Swimming is actually beneficial because of the breathing
avoid: of moistened air and prolonged expiration required under water. Restriction from playgrounds
would interfere with normal socialization.

1.‐ Swimming.
2.‐ Gymnastics.
3.‐ Snow skiing.
4.‐ Playgrounds.

2099 The highest‐priority nursing intervention for a child Correct answer: 2 Pulmonary infections must be treated aggressively with CF patients to minimize the chance All of the options except the fourth are appropriate. The stem asks for the priority
hospitalized with a bacterial respiratory infection and for colonization of resistant pathogens. Once colonized, children have a poorer survival rate. intervention, which would be the option that addresses the root cause of the client s
cystic fibrosis would be: The sweat chloride test is for diagnostic purposes only. Maintaining adequate hydration helps current problem, the bacterial respiratory infection.
liquefy secretions. Pancreatic enzymes help the child absorb food substances, which is not the
priority intervention.
1.‐ Administering pancreatic enzymes.
2.‐ Administering intravenous antibiotics.
3.‐ Recording vital signs every four hours.
4.‐ Arranging for sweat chloride testing.

2100 A 6‐year‐old client with cystic fibrosis (CF) is preparing Correct answer: 4 Children with CF require pancreatic enzymes before each meal and snack to manage Consider the nutritional problems of cystic fibrosis to determine the important assessment
to eat breakfast. What is the most important data the malabsorption and steatorrhea. Enzyme dosage is individualized based on nutritional status data.
nurse would want to obtain before the child eats? and stool consistency. It is also important that the child eat his prescribed high‐calorie, high‐
protein diet to support weight gain. The information in the other options are not necessary to
know before the child eats.
1.‐ Whether the meal is exactly what he ordered
2.‐ If he plans to eat all of it
3.‐ When he ate last
4.‐ If he has taken his enzymes

2101 A child was diagnosed with cystic fibrosis (CF) as an Correct answer: 4 Clubbing of the fingers is seen in pulmonary disease, and is associated with hypoxia and Options 2 and 4 address cystic fibrosis. Option 4 provides more complete information.
infant. The child shows you her hands, and asks why ischemia. In this instance, the clubbing occurs as a result of pulmonary changes due to cystic
her fingers look like that. The nurse's best response fibrosis.
would be:
1.‐ "You inherited them from your parents. Someone in your family probably has the same style of fingers."
2.‐ "It's a part of the cystic fibrosis syndrome, and is a recognizable symptom."
3.‐ "That's the way your fingers looked at birth."
4.‐ "The shape of your fingers is due to the hypoxia caused by your cystic fibrosis."
2102 A child in the clinic has been seen with recurrent Correct answer: 3 Handwashing is the most important infection‐control practice. Isolation is unrealistic, and Options 1 and 4 are unrealistic, and would be impossible to maintain with a child. Seeing a
respiratory infections. The nurse providing anticipatory limits the child's socialization opportunities. Respiratory infections are generally spread by pediatrician weekly will not prevent infection.
guidance in preventing future respiratory infections nasal pharyngeal secretions from infected persons.
would recommend which of the following?

1.‐ Keeping the child away from other children


2.‐ Seeing the pediatrician weekly
3.‐ Maintaining strict handwashing
4.‐ Avoiding all animals

2103 The mother of an infant diagnosed with bronchiolitis Correct answer: 3 At least one‐half of all cases of bronchiolitis are attributed to respiratory syncytial virus. The This question asks for basic information related to causation of bronchiolitis. Key words
asks the nurse what causes this disease. The nurse’s majority of cases of bronchiolitis are not attributable to Klebsiella, mycoplasma pneumoniae, are “majority” and “bronchiolitis.” Use knowledge of the etiology of the disease to choose
response would be based on the knowledge that the or haemophilus influenzae. the correct answer.
majority of infections that cause bronchiolitis are a
result of:
1.‐ Klebsiella infection.
2.‐ Mycoplasma pneumoniae.
3.‐ Respiratory syncytial virus (RSV).
4.‐ Haemophilus influenzae.

2104 A child is brought to the Emergency Department with Correct answer: 4 Any manipulation of the tongue or throat could stimulate the gag reflex and cause complete The key concept is differentiating between epiglottitis and laryngotracheobronchitis.
suspected epiglottitis. The nurse would avoid doing obstruction, so this is the action that should be avoided. The other actions are appropriate. Because of the critical word “avoid,” choose the option that would represent unsafe care to
which of the following for the child with epiglottitis, The child should be allowed to remain in a position of choice for ease of respiration. the client, based on knowledge of pathophysiology of the infection.
which distinguishes the care needed by this child from Emergency intubation equipment should be readily available before any examination of the
the care needed by a child in the adjacent treatment throat is attempted. Parents should be encouraged to comfort the child.
area who has laryngotracheobronchitis?

1.‐ Allowing the child to remain in the position of choice


2.‐ Placing intubation equipment at the bedside
3.‐ Encouraging the parents to comfort the child
4.‐ Examining the throat

2105 An 18‐month‐old boy who was seen in the Emergency Correct answer: 3 The child s respiratory distress makes it difficult for him to lie down. The child will breathe The core concept is positioning for respiratory distress. Determine the correct answer by
Department with respiratory distress is admitted to more easily in a semi‐ to high‐Fowler s position. Rocking the child and holding the child in the analyzing which choice best promotes oxygenation.
the nursing unit with a diagnosis of pneumonia. arms does not specify an upright position, and therefore these are too vague to be useful
Following the initial workup, the child is still short of suggestions. A sleeping pill is not indicated.
breath, but is rubbing his eyes as if he is sleepy. The
mother wants to lay the child down for his nap, but he
refuses to lie down. The nurse would suggest which of
the following as the most effective strategy to help the
child rest?

1.‐ Rock the baby until he is asleep, and then lay him down.
2.‐ Hold him in her arms while he sleeps.
3.‐ Allow him to sleep in an upright position.
4.‐ Give him an over‐the‐counter sleeping pill.

2106 Which statement by an 8‐year‐old girl who has Correct answer: 1 Peak expiratory flow readings over time indicate the child s respiratory ability when she is Critical words are “understands” and “peak expiratory flow meter.” Knowledge of this
asthma indicates that she understands the use of a well. Readings of 50 percent below “personal best” indicate an asthma episode is imminent. device and how it helps to manage asthma helps to answer the question.
peak expiratory flow meter? The meter does not prevent an attack.
1.‐ “My peak flow meter can tell me if an asthma episode might be coming, even though I might still be feeling okay.”
2.‐ “When I do my peak flow, it works best if I do three breaths without pausing in between breaths.”
3.‐ “I always start with the meter reading about halfway up. That way, I don t waste any breath.”
4.‐ “If I use my peak flow meter every day, I will not have an asthma attack.”

2107 A child with cystic fibrosis is hospitalized for a Correct answer: 1 Frothy, foul‐smelling stools reflect malabsorption and indicate that pancreatic enzymes are Eliminate normal findings to determine which documentation indicates the need for
respiratory infection. Which documentation in the not being consumed, or that dosages might need adjustment. Maintenance of weight and nutrition intervention.
chart would indicate the need for counseling regarding consuming meals and snacks are positive nutrition goals for children with cystic fibrosis.
nutrition and gastrointestinal complications?

1.‐ Frothy, foul‐smelling stools


2.‐ Weight unchanged from yesterday.
3.‐ Consumed 80% of breakfast.
4.‐ Eats three snacks every day.

2108 An adolescent was diagnosed with cystic fibrosis as an Correct answer: 4 The developmental task of adolescence is to set future goals, including marriage and family. The key word is “adolescent.” Consider the changes that occur with adolescence to
infant. The nurse anticipates that the adolescent will Men with cystic fibrosis are usually sterile, and women can have decreased fertility as thick determine the needs at this time.
need additional teaching related to: cervical mucus interferes with mobility of sperm. The difference between sterility and
impotence should also be addressed. The client does not need information about a sweat
chloride test (diagnostic test for the disease) or weight reduction. There is no adverse effect of
pancreatic enzymes on sex hormones.

1.‐ Obtaining a sweat chloride test.


2.‐ The effect of pancreatic enzymes on the sex hormones.
3.‐ Increased need for a weight‐reduction diet.
4.‐ Reproductive ability.

2109 A 10‐month‐old child is being admitted with Correct answer: 1, 2, 3, Both the child and the parents will be anxious about the child s condition and the need for Appropriate nursing diagnoses are based on the clinical picture of the disease. With this in
laryngotracheobronchitis in the middle of the night. 4 emergency admission to the hospital. Respiratory distress is an obvious sign of ineffective mind, select nursing diagnoses that relate to breathing, effects of dyspnea on fluid and food
The child has a loud stridor and moderate respiratory breathing. The child will need extra fluids due to increased insensible fluid loss and inability to intake, and the need for information about the disease.
distress. In planning care for this child, the nurse will take fluids due to respiratory distress. The parents will want to know the cause of the disease,
identify which of the following as appropriate nursing and interventions for future episodes. The outcome for this disease is usually positive, so
diagnoses? Select all that apply. Anticipatory Grieving would not be appropriate.

1.‐ Fear/Anxiety
2.‐ Ineffective Breathing Pattern
3.‐ Risk for Deficient Fluid Volume
4.‐ Deficient Knowledge
5.‐ Anticipatory Grieving

2110 A 9‐month‐old infant has been admitted to the Correct answer: 2, 4 RSV is very contagious, especially with infants and clients who are immunocompromised. The core concept is risk of transmission of infection from an infant with RSV. The correct
pediatric unit with respiratory syncytial virus infection. Therefore, in clients with immunodeficiency, chemotherapy and aplastic anemia should be choices would avoid infants and clients who are immunocompromised.
The nurse assigned to provide care to this infant will avoided.
need to be assigned to care for other clients as well.
The charge nurse should assign the nurse to which of
the following children as an appropriate assignment?
Select all that apply.

1.‐ A toddler with neuroblastoma undergoing chemotherapy


2.‐ A 10‐year‐old with a fractured femur in traction
3.‐ An infant with immunodeficiency
4.‐ A preschooler with impetigo
5.‐ A 2‐year‐old with aplastic anemia
2111 The nurse is teaching home tracheostomy care to the Correct answer: 2 Accumulating mucopurulent secretions might provide a medium for bacterial growth, and can The key concept is the most essential information for home tracheostomy care. Recall that
parents of a toddler. What information would be obstruct the lumen of the tube. Suctioning is another risk for introduction of bacteria. Early infection and obstruction are key concerns related to artificial airways.
essential for the nurse to include? recognition of signs of infection is important. The tube does not need to be changed every day
(option 1), and cannot be removed (option 3). It is small objects, not large objects (option 4),
that pose a risk to aspiration and would need to be avoided.

1.‐ The importance of changing the tracheostomy every day


2.‐ How to recognize signs of infection and obstruction
3.‐ How to remove the tracheostomy so the child can talk
4.‐ Teaching the child to keep large objects away from the tube

2112 A child with a respiratory infection is scheduled to Correct answer: 4 Children with cystic fibrosis have elevated chloride concentrations of sweat because of the The key word of this test item is “purpose.”
have a sweat test. After the physician discusses the dysfunction of the exocrine glands.
test with the child's mother, the mother approaches
the nurse and asks the purpose of this diagnostic test.
Which of the following responses by the nurse would
best reinforce the physician s explanation?

1.‐ “This will determine if your child is dehydrated.”


2.‐ “This will assess whether your child’s sweat glands are functioning.”
3.‐ “This will help us to identify the infectious organism.”
4.‐ “This will diagnose whether the child has cystic fibrosis.”

2113 The mother of an infant who has had recurrent Correct answer: 4 Infants and young children have narrower airways, and shorter distance between structures; Critical words are “why infants are at increased risk for complications from respiratory
respiratory infections asks the nurse why infants are at accessory muscles generally used for breathing are immature. The respiratory rate of infants is infections.” The core knowledge is the physiological differences between infants and older
increased risk for complications from respiratory faster than that of adults, and parents can be taught to assess the child for respiratory children. After eliminating options 1 and 2 as inaccurate, knowledge of infant anatomy will
infections. The best response by the nurse explains problems. lead to the selection of option 4.
that with infants:
1.‐ The airway structures are larger, allowing for entry of a greater number of organisms.
2.‐ The respiratory rate is slower than in adults.
3.‐ Parents are unable to assess respiratory problems accurately.
4.‐ The airways are narrower and more easily obstructed.

2114 The mother of a neonate hospitalized with an upper Correct answer: 3 Newborns are unable to coordinate breathing and sucking simultaneously. They are nose‐ Critical words are “neonate hospitalized with an upper respiratory tract infection” and
respiratory tract infection asks why her baby won’t breathers, and anything that interferes with nasal patency impairs feeding as well. The “baby won t take her bottle.” The core concept is a neonate with an upper respiratory
take her bottle. The nurse’s best answer would be: difficulty with sucking does not relate to hunger, or to selection of formula. infection (URI) and refusal to suck. The knowledge that normal coordination of breathing is
hampered by nasal congestion will help guide to the correct answer.

1.‐ “She’s probably not hungry.”


2.‐ “It s okay, because we re giving her intravenous fluids; therefore, she is not hungry.”
3.‐ “Newborns breathe through their noses. Congestion might be interfering with her ability to breathe and eat at the same time.”
4.‐ “She might need a different type of formula. We’ll call the physician to get a new order.”

2115 A 4‐year‐old female child presents to the Emergency Correct answer: 2, 3, 4, In epiglottitis, any manipulation of the throat can cause stimulation of the gag reflex. The Critical words are “epiglottitis” and “initial assessment should include.” Knowledge of
Department with a sore throat, difficulty swallowing, 5 inflamed, edematous epiglottis could then completely obstruct the airway. All other epiglottitis and its care and management is needed to determine which assessment should
and a suspected diagnosis of acute epiglottitis. Initial assessments should be made. not be done.
assessment of the child should include: (Select all that
apply.)
1.‐ Throat culture.
2.‐ Vital signs.
3.‐ Past medical history.
4.‐ Auscultation of chest.
5.‐ Observation of swallowing ability.
2116 The nurse is providing homecare instructions to the Correct answer: 3, 5 Children with cystic fibrosis require pancreatic enzymes with every meal and snack to counter Critical words are “child with cystic fibrosis” and “indicates the parents do not
parents of a child with cystic fibrosis. Which statement malabsorption and nutritional problems. They require well‐balanced diets with 120–150% of understand.” Eliminate all choices that would be appropriate for the parents to perform at
by the parents indicates that they do not understand RDA calories and 200% protein. Normal bowel movements indicate that enzyme dosage is home. This leaves the choices that illustrate a lack of understanding of the needed home
the treatment regimen? Select all that apply. appropriate. It is important to avoid other children with infections, but physical activity is care.
encouraged within the child’s capability. Chest percussion is a normal part of health
maintenance for this child.
1.‐ “We will perform chest physiotherapy and postural drainage four times a day.”
2.‐ “We will keep her away from the church nursery if any of the children are coughing and have fever or runny noses.”
3.‐ “If her bowel movements are normal and her appetite is good, she does not need her pancreatic enzymes.”
4.‐ “The relay races and swimming at our Sunday school picnic next week will be good exercise for her.”
5.‐ “My child will not need any special dietary intake.”

2117 A 2‐year‐old child is being discharged after Correct answer: 3 Developmentally, small children practice increased hand‐to‐mouth activity and explore Critical words are “removal of a coin” and “most important topic.” Knowledge of foreign
bronchoscopy for removal of a coin from his objects with their mouths. Any small toy or food can be ingested, and can potentially obstruct body aspiration and teaching about prevention of future aspirations and removal of
esophagus. The most important topic of discharge the airway. All of the other choices are correct, but 3 is most important. potential hazards are essential to prevent future problems.
teaching would be the importance of:
1.‐ Reassuring the child that he is fine.
2.‐ Proper nutrition for the next few days.
3.‐ Restricting his access to small toys or objects.
4.‐ Administering acetaminophen for his sore throat.

2118 A 15‐year‐old child with a history of cystic fibrosis is Correct answer: 4 Pulmonary pathogens are particularly detrimental to children with cystic fibrosis. Colonization Knowledge of cystic fibrosis and the illness management of the disease to prevent
admitted to the pediatric unit with assessment findings of the lungs with resistant organisms often leads to poor survival rates. Aggressive intravenous complications is essential to answer the question. The highest priority for the client will be
of crackles, increased cough, and greenish sputum. A administration of high‐dose antibiotics is always a priority. related to respiratory function. Therefore, planning for IV access allows for rapid
two‐week hospitalization is anticipated. Which nursing implementation of the medical plan of care.
intervention holds the highest priority?

1.‐ Referral to Child Life Services for school lesson plans


2.‐ Arranging for liberal visitation from peers
3.‐ Taking a diet history
4.‐ Gaining intravenous access

2119 A 7‐year‐old child is brought to the Emergency Correct answer: 2 A beta&lt;sub&gt;2&lt;/sub&gt; agonist (short‐acting) is the drug of choice for acute therapy The critical concept here is “acute” asthma attack. Knowledge of the medications used to
Department for an acute asthma attack. He is given via inhalation for emergency relief of acute bronchospasm; action is immediate, within treat asthma emergencies is necessary to answer the question correctly. Select the
wheezing, tachypneic, and diaphoretic, and looks 5–10 minutes. Use before an inhaled steroid. Methylprednisolone and prednisone are both medication that would be utilized during an acute attack.
frightened. The nurse should prepare to administer: corticosteroids to reduce the inflammatory process, but would not give immediate relief.
Cromolyn sodium is a preventive medication.
1.‐ IV methylprednisolone.
2.‐ Albuterol.
3.‐ Oral prednisone.
4.‐ Cromolyn sodium.

2120 The nurse would select which of the following as an Correct answer: 4 The sudden onset of severe respiratory distress is frightening and very stressful for the family Critical words are “toddler” “laryngotracheobronchitis.” Consider the symptoms of acute
appropriate nursing diagnosis for the family of a and child. There is no prolonged hospital confinement. There is no permanent loss for which to laryngotracheobronchitis. Since the diagnosis is frightening but not fatal, and has no long‐
toddler being treated for acute grieve, and growth and development are not likely to be affected. term sequelae, options 1 and 2 can be eliminated.
laryngotracheobronchitis?
1.‐ Anticipatory Grieving
2.‐ Impaired Growth and Development related to acute onset of illness
3.‐ Impaired Social Interaction related to confinement in hospital
4.‐ Fear/Anxiety related to dyspnea and noisy breathing
2121 A child with bacterial pneumonia is crying, and says it Correct answer: 1 Splinting the affected side with a pillow or stuffed animal lessens the discomfort experienced The child is complaining of pain during coughing, so look for a choice that would reduce
hurts when he coughs. The nurse would teach the child with bacterial pneumonia. pain without suppressing the cough reflex.
to:
1.‐ Hug his teddy bear when he coughs.
2.‐ Ask for pain medicine before he coughs.
3.‐ Take a sip of water before coughing.
4.‐ Try very hard not to cough.

2122 An infant with chronic bronchopulmonary dysplasia Correct answer: 3 Home oxygen therapy and tracheostomy care require access to emergency equipment Critical words are “home oxygen therapy” and “statement indicates that further teaching
(BPD) and a tracheostomy is being discharged to home typically not available on a camping trip. Additionally, campfires are hazardous. All other is needed.” Because wording of the question guides you to select an incorrect statement,
oxygen therapy. Which statement by the mother choices indicate correct information. eliminate all responses that are appropriate for home care of a child with a tracheostomy
indicates that further teaching is needed before who is receiving oxygen.
discharge?
1.‐ “I will call my pediatrician if she gets a fever or has more secretions than usual from her tracheostomy.”
2.‐ “I have a cute bib to loosely cover her tracheostomy when she eats and when we go outside in the wind.”
3.‐ “We are so glad the baby will get to go with us on our camping trip to Yellowstone National Park. We have been waiting for her to get well so we can go.”
4.‐ “We have already notified Alabama Power Company that our baby is coming home today.”

2123 A client has chronic respiratory acidosis caused by Correct answer: 1 COPD clients have low oxygen and high carbon dioxide levels. Therefore, hypoxia is the main Select option 1 as the best explanation of the physiological phenomenon of hypoxic drive.
end‐stage chronic obstructive pulmonary disease stimulus for ventilation in persons with chronic hypercapnia. Increasing the level of oxygen Note that options 1 and 2 are essentially opposite. Generally, one item in a pair of
(COPD). Oxygen is delivered at 1 L/min per nasal would decrease the stimulus to breathe. opposites will be correct.
cannula. The nurse teaches the family that the reason
for this precaution is to avoid respiratory depression,
based on which of the following as the best
explanation?
1.‐ COPD clients depend on a low oxygen level.
2.‐ COPD clients depend on a low carbon dioxide level.
3.‐ COPD clients tend to retain hydrogen ions if they are given high doses of oxygen.
4.‐ COPD clients thrive on a high oxygen level.

2124 A client presents to the Emergency Department with Correct answer: 4 A combined low PO&lt;sub&gt;2&lt;/sub&gt; and low Cal SO&lt;sub&gt;2&lt;/sub&gt; Blood gases are not diagnostic of disease states, but simply provide information, such as
acute respiratory distress and the following arterial represents hypoxia. The pH, PCO&lt;sub&gt;2&lt;/sub&gt;, and whether hypoxia is present.
blood gases (ABGs): pH 7.35; HCO&lt;sub&gt;3&lt;/sub&gt;&lt;sup&gt;‐&lt;/sup&gt; are normal. ABGs will not necessarily be
PCO&lt;sub&gt;2&lt;/sub&gt; 40 mmHg; altered in TB or pleural effusion. Initially, in pneumonia, both the PO&lt;sub&gt;2&lt;/sub&gt;
PO&lt;sub&gt;2&lt;/sub&gt; 63 mmHg; and PCO&lt;sub&gt;2&lt;/sub&gt; are usually low because the hypoxia leads to
HCO&lt;sub&gt;3&lt;/sub&gt;&lt;sup&gt;‐&lt;/sup&gt; hyperventilation.
23; oxygen saturation (Cal
SO&lt;sub&gt;2&lt;/sub&gt;) 93%. Which of the
following represents the best analysis of the etiology
of these ABGs?
1.‐ Tuberculosis (TB)
2.‐ Pneumonia
3.‐ Pleural effusion
4.‐ Hypoxia

2125 When assessing a client with early impairment of Correct answer: 3 The cardinal signs of respiratory problems and hypoxia are restlessness, diaphoresis, Note that options 2 and 3 reflect opposite symptoms in terms of heart rate. One of the
oxygen perfusion, such as in pulmonary embolus, the tachycardia, and cool skin. Bradycardia might occur much later in the process, when the opposites is likely correct. Tachycardia is the early symptom.
nurse would expect to find restlessness and which of condition is severe. Eupnea is normal respirations in rate and depth.
the following symptoms?

1.‐ Warm, dry skin


2.‐ Bradycardia
3.‐ Tachycardia
4.‐ Eupnea

2126 One day postoperative, the client complains of Correct answer: 2 The first three symptoms could be indicative of any of the conditions. The distinguishing Select the option that is common in the postoperative period, atelectasis.
dyspnea, respiratory rate (RR) is 35, slightly labored, symptom is the lack of breath sounds in the lower‐right base when a portion of the lung has
and there are no breath sounds in the lower‐right collapsed.
base. The nurse would suspect:
1.‐ Cor pulmonale.
2.‐ Atelectasis.
3.‐ Pulmonary embolus.
4.‐ Cardiac tamponade.

2127 A client with an acute case of pneumonia has a dry, Correct answer: 1 The effects of the respiratory treatment should break up the congestion and cause The question is asking about positive results of a respiratory treatment, which would
hacking cough, elevated temperature, elevated white bronchodilation; thus the change in lung sounds and more productive cough effort. As the include coughing up sputum.
blood cell (WBC) count, decreased breath sounds, and pneumonia resolves, the lungs should begin to clear and the cough diminish. Notice that the
pain upon deep inhalation or coughing. Which of the question asks about an acute case; be careful to note the situation in the stem.
following would indicate positive results after a
respiratory treatment of normal saline, acetylcysteine
(Mucomyst), and metaproterenol (Alupent)?

1.‐ Crackles and cough productive of a moderate amount of sputum


2.‐ Absent breath sounds in bases and normal breath sounds in upper lobes
3.‐ Wheezing, nonproductive cough
4.‐ Diminished breath sounds with a small amount of productive sputum

2128 Which of the following needs immediate medical Correct answer: 3 Option 3 is indicative of a tension pneumothorax, which is considered a medical emergency. No breath sounds is the key to identification of this as the correct answer.
attention and emergency intervention? The client who: The respiration system is severely compromised, and venous return to the heart is also
affected. The mediastinal shift is to the unaffected side. Option 1 contains symptoms of
pleurisy, and option 2 lists symptoms of bronchitis; neither is an emergency. The client in
option 4 should expect difficulty breathing after exercise when asthma is an existing condition,
and could need immediate attention if the rescue inhaler is ineffective.

1.‐ Complains of sharp pain upon taking a deep breath, and excessive coughing.
2.‐ Exhibits yellow, productive sputum, low‐grade fever, and crackles.
3.‐ Has a shift of the trachea to the left, with no breath sounds on the right.
4.‐ Has asthma, and complains of inability to "catch her breath" after exercise.

2129 A teenage client newly diagnosed with asthma is Correct answer: 2 A young person needs to know the triggers of asthma. Physical exercise in school and as a Prioritize methods of avoiding future attacks by identifying triggers.
being discharged from the hospital after an episode of part of life will be everpresent, and prevention of an attack before exercise is essential at this
status asthmaticus. Discharge teaching should include time in the client's life. Sports do not have to be limited in all asthmatic people. Living a
which of the following? productive, normal life should be stressed. The client might have to use preventative
medications before a sport of her choice. The fear associated with asthma is common, and
might take a while to overcome. Instructions on identifying triggers and using the rescue
inhalers need to be taught, and the fear will eventually subside.

1.‐ Incidence of status asthmaticus in children and teens


2.‐ The relationship of symptoms to a specific trigger, such as physical exercise
3.‐ Limitations in sports that will be imposed by the illness
4.‐ Specific instructions on staying calm during an attack
2130 A known cardiac client is experiencing angina at night Correct answer: 4 As the upper airflow obstruction occurs in sleep apnea, the CO&lt;sub&gt;2&lt;/sub&gt; rises, Recognize that options 4 is the only option that addresses the relationship to angina.
only and excessive fatigue, and the spouse states that and cardiac arrhythmias and angina can occur because of the lack of oxygenated blood supply
the client snores excessively. The physician orders a to the heart. Clients with sleep apnea do not get adequate amounts of REM sleep, and are
sleep apnea study with and without oxygen. The often awakened frequently during the night in order to make breathing possible.
rationale for considering a pulmonary source rather
than a cardiac source as the cause of the angina at this
time is:
1.‐ Sleep apnea is an obstruction of the lower airway, which impedes airflow.
2.‐ Clients with sleep apnea have adequate amounts of REM sleep, but snoring contributes to the decrease in oxygen levels.
3.‐ Sleep apnea causes an increase in muscle tone during REM sleep in order to make breathing possible.
4.‐ Excess periods of apnea during sleep and severe drops in oxygen levels can contribute to the angina, which is occurring only at night.

2131 Which of the following symptoms is most Correct answer: 2 The most common sign of cancer of the lung is a persistent cough that changes. Other signs Recall that cough is considered a cardinal symptom of cancer.
characteristic of a client with cancer of the lung? are dyspnea, bloody sputum, and long‐term pulmonary infection. Option 1 is common with
chronic obstructive pulmonary disease (COPD); option 3 is common with asthma; and option 4
is common with tuberculosis.
1.‐ Exertional dyspnea
2.‐ Persistent, changing cough
3.‐ Air hunger; dyspnea
4.‐ Cough with night sweats

2132 A client is admitted to the unit after a traumatic Correct answer: 3 Adult respiratory distress syndrome is common after a trauma or shock situation. Clients will The key is the reference to sudden changes, as ARDS often has a rapid onset.
encounter with a bull in a pasture that speared his often become hypoxic and alkalotic with pulmonary edema.
right chest wall. The client is admitted with a flail
chest, and is treated accordingly. The nurse should be
particularly observant to the sudden changes that
might be evidence of which of the following?

1.‐ Hypercapnia
2.‐ Sepsis
3.‐ Adult respiratory distress syndrome
4.‐ Metabolic acidosis

2133 A client was involved in a motor vehicle accident Correct answer: 3 A mediastinal shift, along with the other symptoms in the question, is indicative of a tension Tracheal deviation is always emergent.
(MVA) in which a seat belt was not worn. The client is pneumothorax. Since the individual was involved in an MVA, assessment would be targeted at
exhibiting crepitus and decreased breath sounds on acute traumatic injuries to the lungs, heart, or chest wall rather than the conditions indicated
the left, complains of shortness of breath (SOB), and in the other options. Option 1 is common with pneumonia; values in option 2 are not alarming;
has a respiratory rate of 34/min. Which of the and option 4 is typical of someone with chronic obstructive pulmonary disease (COPD).
following assessment findings should concern the
nurse the most?

1.‐ Temperature of 102°F and a productive cough


2.‐ Arterial blood gases (ABGs) with a PaO2 of 92 and PaCO2 of 40 mmHg
3.‐ Trachea deviating to the right
4.‐ Barrel‐chested appearance

2134 A nurse is teaching a client newly diagnosed with Correct answer: 2 Option 2 is the pathophysiology behind emphysema. Option 1 explains asthma, option 3 Use of the word “trapped” in option 2 is the key to identification of this option as correct
emphysema about the disease process. Which of the explains bronchitis, and option 4 explains cystic fibrosis.
following statements best explains the problems
associated with emphysema, and could be adapted for
use in the nurse's discussion with the client?

1.‐ Hyperactivity of the medium‐sized bronchi caused by an inflammatory response leads to wheezing and tightness in the chest.
2.‐ Larger‐than‐normal air spaces and loss of elastic recoil cause air to be trapped in the lung and collapse airways.
3.‐ Vasodilation, congestion, and mucosal edema cause a chronic cough and sputum production.
4.‐ Chloride is not being transported properly, producing excess absorption of water and sodium, and thick, viscous mucus.

2135 A sweat test, arterial blood gases (ABGs), and chest x‐ Correct answer: 4 Cystic fibrosis is diagnosed with a high chloride level (normal: less than 40 mmol/L) on the The sweat test would be significant if it showed high chloride levels. Select the highest
ray (CXR) are ordered for a child with symptoms sweat test, hypoxemia on the ABGs, and atelectasis or hyperinflation on the CXR. level.
suggestive of cystic fibrosis (CF). Which of the
following results would be consistent with this
diagnosis?
1.‐ Chloride 32 mmol/L; PaO2 82 mmHg; and atelectasis in the lower lobe
2.‐ Chloride 37 mmol/L; PaO2 95 mmHg; and hypoinflation of the lungs
3.‐ Chloride 29 mmol/L; PaO2 90 mmHg; and white‐out of the lungs
4.‐ Chloride 64 mmol/L; PaO2 70 mmHg; and hyperinflation of the lungs

2136 A middle‐aged man who suffered a large myocardial Correct answer: 3 Barotrauma (decreased cardiac output and damage to lung tissue) is a common complication Omit options 1, 2, and 4 as common findings in the intubated client.
infarction develops adult respiratory distress of PEEP. A drop in BP is associated with a decreased cardiac output. The sinus tachycardia
syndrome (ARDS) as a complication. The client is could be a compensatory mechanism to raise the BP, or a response to the ARDS. Anxiety is to
intubated and placed on positive end‐expiratory be expected with intubation, and a small rise in temperature might or might not indicate an
pressure (PEEP). Which of the following is a finding of infectious process.
concern while on the PEEP?

1.‐ Sinus tachycardia of 125 beats/min


2.‐ Anxiety
3.‐ Blood pressure (BP) of 88/52
4.‐ Temperature 100.5°F

2137 A client admitted to the medical nursing unit has Correct answer: 2. Exposure with a positive TB skin test usually requires six months of prophylactic treatment Conversion of a skin test following a potential exposure requires treatment.
classic symptoms of tuberculosis (TB), and tests unless contraindicated. The TB skin test should not be repeated; the results will always be
positive on the purified protein derivative (PPD) skin positive. A CXR is usually not required annually in the event of a positive skin test.
test. Several months later, the nurse who cared for the
client also tests positive on an annual TB skin test. The
most likely course of treatment if the nurse's chest x‐
ray (CXR) is negative is to:

1.‐ Repeat a TB skin test in six months.


2.‐ Treat the nurse with an anti‐infective agent for six months.
3.‐ Monitor for signs and symptoms within the next year.
4.‐ Follow up in one year at the next annual physical with a CXR only.

2138 Clients with severe sleep apnea who are members of Correct answer: 2 The deprivation of oxygen during the night often leaves individuals tired during the day. Any The question refers to rest periods, which should lead to option 2.
a support group should be educated to plan frequent activity increases the need for oxygen, which is already limited in a client with this disorder.
rest periods and activities around how well they feel in
order to maximize energy because they might
complain of which of the following during the day?

1.‐ Cardiac arrhythmias


2.‐ Fatigue
3.‐ Jaw pain
4.‐ Productive cough
2139 A client who has a known history of cardiac problems Correct answer: 1 Pleuritic pain is typically sharp and stabbing. Pleural effusion (option 2) and atelectasis (option Omit options 2, 3, and 4 as having other associated symptoms in addition to pain.
and is still smoking enters the clinic complaining of 3) can cause pain, but usually have other symptoms, like dyspnea and diminished or absent
sudden onset of sharp, stabbing pain that intensifies breath sounds. Pleurisy is common in smokers. Tuberculosis (option 4) causes chest pain along
with a deep breath. The pain is occurring on only one with other symptoms.
side, and can be isolated upon general assessment.
The nurse concludes that this description is most likely
caused by:
1.‐ Pleurisy.
2.‐ Pleural effusion.
3.‐ Atelectasis.
4.‐ Tuberculosis.

2140 The nurse is delivering postmortem care to a client Correct answer: 4 If the blockage is large enough and blood flow is hindered to the lung, the tissue will die. This Associate the term embolus with a subsequent infarct.
who died from a large pulmonary embolus (PE). The usually occurs when a large clot blocks the entire main pulmonary artery. Option 1 is rather
nurse concludes that the client's death was most likely vague because blood flow is decreased to the heart, lung, brain, and other vital organs because
caused by which of the following? of the blockage, but the amount of decrease can be variable. Option 2 is incorrect; dead space
is increased with PE. Option 3 is correct in pulmonary embolism, but is not usually the cause of
death.
1.‐ Decreased blood flow
2.‐ Decreased alveolar dead space
3.‐ Inefficiency of the heart insufficient to pump adequately
4.‐ Infarction of the lung tissue

2141 An elderly client recuperating from hip surgery will Correct answer: 3 Without any evidence of a blood clot or PE, LMWH is usually used for prevention purposes, Select the option that would be used most often for clot prophylaxis.
most likely be placed on which of the following in especially since the client is elderly, and will be on bedrest for a period of time. Heparin and
order to prevent a pulmonary embolism (PE)? Coumadin are used when a confirmed clot exists. Thrombolytics are used when a clot needs to
be dissolved immediately.
1.‐ Tissue plasminogen activator (TPA)
2.‐ Warfarin (Coumadin)
3.‐ Low–molecular weight heparin (LMWH)
4.‐ Heparin

2142 A 50‐year‐old client with chronic obstructive Correct answer: 1 A sustained elevation in the resting mean pressure above 20 mmHg from a pulmonary artery Option 1 gives information about pulmonary pressure.
pulmonary disease (COPD) who has smoked two packs is defined as pulmonary hypertension, which could be caused by the COPD. Rust‐colored
of cigarettes a day is being cared for in the intensive sputum (option 2) is usually indicative of lung cancer; thick, viscous mucus (option 3) can be
care unit for an acute exacerbation of the disease. significant in a number of disorders; and absent breath sounds (option 4) are indicative of
Which of the following should alert the nurse to the many pulmonary disorders, but not of pulmonary hypertension.
possibility of pulmonary hypertension?

1.‐ Pulmonary artery pressure of 30 mmHg


2.‐ Rust‐colored sputum
3.‐ Thick, viscous mucus
4.‐ Absent breath sounds

2143 A result of many chronic lung diseases is cor Correct answer: 2 Chronic lung disease causes hypertrophy of the right ventricle; eventually, the right ventricle Cor pulmonale is associated with right heart failure.
pulmonale. The nurse would explain to a client that the fails, mainly because of the increased pressure within the pulmonary artery that results from
pathophysiology behind this complication is that the the lung disease. Signs and symptoms of right‐sided heart failure will occur.
effect of the lung disease:

1.‐ Increases the workload of the left ventricle.


2.‐ Increases the workload of the right ventricle.
3.‐ Dilates the pulmonary artery.
4.‐ Constricts the pulmonary artery.
2144 Which of the following clients should be watched Correct answer: 4 ARDS is a problem with impaired diffusion, whereas upper airway obstruction (option 1), rib Recall that diffusion is specific to ARDS.
carefully for respiratory failure caused by impaired gas fractures (option 2), and drug overdose (option 3) are problems with impaired ventilation.
diffusion?
1.‐ A client who has meat lodged in the back of his throat, causing an upper airway obstruction
2.‐ A client who fell from a tall ladder and suffered several rib fractures
3.‐ A client who overdosed on morphine sulfate
4.‐ A client who is suddenly developing adult respiratory distress syndrome (ARDS).

2145 The nurse is explaining to a nursing assistant why it is Correct answer: 2 With chronic hypoxia, the kidneys increase the production of red blood cells, leading to The term “chronic hypoxia” in option 2 should lead to the selection of this option, as
important to report any client complaints of calf pain. polycythemia and increased viscosity of the blood. The increased viscosity can lead to emphysema is associated with chronic hypoxia.
The explanation for this is that in the client with formation of blood clots.
emphysema, the possibility of developing blood clots
exists because:

1.‐ The client is often on bedrest, and clots are more likely to develop.
2.‐ Polycythemia results from chronic hypoxia.
3.‐ Pulmonary emboli are more likely when there is hypercapnia.
4.‐ The trapping of air associated with emphysema can cause an air embolus.

2146 Which of the following sets of symptoms best Correct answer: 2 The respiratory rate and heart rate are increased when a portion of the lung has collapsed; Increased respiratory rate combined with reduced PaO&lt;sub&gt;2&lt;/sub&gt; and
describes a client experiencing atelectasis? hypoxia occurs, and there usually are diminished or absent breath sounds over the affected breath sounds are the combination to look for.
area.
1.‐ Dyspnea; respiratory rate (RR) 12/min; heart rate (HR) 125; PaO2 of 70 mmHg; rales
2.‐ Dyspnea; RR 32/min; HR 125; PaO2 70 mmHg; diminished breath sounds over one lobe
3.‐ Shallow, labored respirations; RR 22/min; HR 78; PaO2 90 mmHg; diminished breath sounds
4.‐ Deep respirations; RR 32/min; HR 115; PaO2 98 mmHg; crackles scattered bilaterally

2147 A client who is a long‐term smoker develops a cough Correct answer: 3 An antibiotic may be ordered if another respiratory infection, such as bronchitis, is present. A Pain is the most prominent symptom that would require treatment.
and severe pain with any chest movement, coughing, cough suppressant may be prescribed for nighttime only, but should not be ordered if a
or deep breathing. Upon assessment, the pain is productive cough occurs. The relief of pain is of top priority in order to allow the client to rest
localized on one side, and a diagnosis of pleurisy is and cough effectively, if necessary.
made. The highest priority for treatment is which of
the following?
1.‐ Antibiotics
2.‐ Cough suppressant
3.‐ Analgesics for pain
4.‐ Bedrest

2148 A client has a chest tube inserted for a Correct answer: 4 A properly working system should have fluctuation in the water‐seal compartment that Fluctuation with respiration is the key to functional chest drainage.
pneumothorax. The nurse assesses the drainage increases with inspiration and falls with expiration, and intermittent bubbling should be noted.
system for adequacy. Which of the following signifies
that the system is working correctly?
1.‐ There is no fluctuation in the water‐seal compartment.
2.‐ Constant bubbles are noted in the water‐seal without fluctuation.
3.‐ There are no bubbles noted in the water‐seal compartment.
4.‐ There is fluctuation in the water‐seal compartment coinciding with respirations.
2149 Theophylline (Theo‐dur) is ordered for a client with Correct answer: 3 The action of theophylline is bronchodilation, which should relax respiratory efforts. Eupnea Recall that eupnea means normal breathing, which is the goal of therapy.
emphysema for the main purpose of relaxing bronchial is normal respirations, which should be a direct result of bronchodilation.
smooth muscle. Which of the following indicate(s)
therapeutic effects of the drug?

1.‐ Decreased heart rate


2.‐ Thinner, clear‐to‐white secretions
3.‐ Eupnea respirations
4.‐ Increasing respiratory rate

2150 A client visits the clinic with a diagnosis of Correct answer: 2 The trapping of air causes the typical barrel chest appearance and the pink color of the skin Option 2 describes the typical clinical picture of this disease.
emphysema. Then nurse expects to note which of the (unless in the later stage of emphysema). The clubbing of the nails is related to the chronic
following typical characteristics for a client with this hypoxia.
diagnosis?
1.‐ Cyanosis, clubbing of the nails, and pigeon chest
2.‐ Barrel chest, pink color, and clubbing of the nails
3.‐ Barrel chest, cyanosis, and peripheral edema
4.‐ Thin body appearance and pigeon chest

2151 A client with chronic obstructive pulmonary disease Correct answer: 3 Side effects of the methylxanthines (related to caffeine) are headache, seizures, diarrhea, Shaking is associated with caffeine effects, and is similar to increased theophylline level.
(COPD) is prescribed a methylxanthine muscle twitching, and anorexia. A normal theophylline level is between 10 and 20 g/mL; the
(bronchodilator) to relieve the symptoms of dyspnea client should be closely monitored for toxicity, in which levels exceed the high end of the
and reduce the respiratory effort. Which of the normal range.
following should alert the nurse to notify the physician
to have a theophylline level drawn?

1.‐ Increased appetite


2.‐ Constipation
3.‐ Hand tremors
4.‐ Bradycardia

2152 A female client with asthma is instructed to keep a Correct answer: 2 Although 210 (option 4) and 250 (option 1) are better than the reading during an asthmatic The question is asking about effective treatment, so select the best reading.
daily log of her peak flow meter readings during a attack, effective medication should return the client to a fairly normal value, if not close to
normal day as well as prior to and after an asthma what the average reading is in a normal day. Ideally, 350 would be effective, but 300 is
attack. If the peak flow meter reading averages 350 certainly an improvement, and is at least 80% of the client's potential. (350 x 0.80 = 280.)
but drops to 200 during an asthma attack, the client
should expect to see which of the following readings if
the rescue inhaler is effective?

1.‐ 250
2.‐ 300
3.‐ 150
4.‐ 210

2153 In educating a group of clients with pulmonary Correct answer: 3 In conditions such as emphysema or asthma, where airways are constricted or airflow is The technique is used with clients with emphysema for whom the main problem is keeping
disorders about the use of pursed‐lip breathing, the limited, pursed‐lip breathing keeps the airways open by maintaining positive pressure. airways open.
nurse is asked how this technique helps them to
breathe. The best response would be that pursed‐lip
breathing:
1.‐ Improves lung expansion.
2.‐ Reduces anxiety that increases oxygen demand.
3.‐ Keeps airways open.
4.‐ Helps the client regain control of the attack.
2154 The nurse explains that during an acute attack of an Correct answer: 2 Decreasing anxiety which lessens oxygen demand as well as abdominal breathing to improve Think of the abdomen as a place for lung expansion.
obstructive pulmonary disorder, such as chronic lung expansion assists the client to breathe more easily and to relieve some of the air hunger
obstructive pulmonary disease (COPD) or asthma, that accompanies obstructive breathing disorders.
abdominal breathing might be most effective because
it:
1.‐ Keeps airways open.
2.‐ Improves lung expansion.
3.‐ Decreases the workload of the lungs.
4.‐ Decreases the pulmonary pressure.

2155 For a client with thick, tenacious secretions, which of Correct answer: 1 All of these answers are appropriate and correct for an individual who has respiratory Liquefy thick secretions with increased liquids.
the following actions by the nurse would be the most problems with productive sputum. The stem asks, however, which option is most effective
effective? with thick secretions. Increasing fluid intake can be as effective as an expectorant or mucolytic.

1.‐ Encouraging fluid intake of 2,000 cc per day unless contraindicated


2.‐ Assisting the client to sit upright to promote chest expansion
3.‐ Monitoring pulse oximetry, which indicates airway obstruction
4.‐ Providing humidified oxygen to decrease its drying effects

2156 A primary consideration by the nurse in preoperative Correct answer: 4 This measure assists the client in eliminating the anesthetic gases that can eventually lead to Recall this as a basic postoperative care strategy.
teaching for prevention of atelectasis would be to pneumonia. It also promotes lung expansion and rids the lungs of secretions that could be a
educate the client about: medium for growth of microorganisms.
1.‐ Lung complications postsurgery.
2.‐ How difficult coughing might be.
3.‐ The use of analgesics before the pain becomes too severe.
4.‐ Turning, coughing, and deep breathing every two hours.

2157 The nurse assesses for which of the following as a Correct answer: 2 The inflammatory process in the pleural cavity (which usually has a thin layer of serous fluid) Note the phrase “unique manifestation” in the question stem, and see that it would
unique manifestation in a client with pleurisy? can cause a friction rub when auscultating lung sounds. This symptom is unique to this health describe option 2.
problem.
1.‐ Dull, pressure type of pain
2.‐ Pleural friction rub
3.‐ Generalized chest discomfort
4.‐ Productive cough

2158 A client in the Emergency Department who suffered Correct answer: 3 A pneumothorax is the presence of air in the pleural cavity; a hemothorax is the presence of Use the word prefixes 'hemo‐', meaning blood, and 'pnumo‐', meaning air, to select the
chest injury in a motor vehicle accident has overheard blood in the pleural cavity. The cause (e.g., knife wound, crushing injury, or other thoracic correct answer.
conversation during the course of rapid treatment. He trauma), signs and symptoms, and treatment are basically the same.
asks the nurse about hemothorax and pneumothorax,
which were discussed in his presence. The nurse
explains that the difference between these two is the:

1.‐ Cause.
2.‐ Amount of collapsed lung.
3.‐ Presence of air versus blood in the pleural cavity.
4.‐ Difference in treatment options.
2159 The nurse expects to administer which of the Correct answer: 1 Heparin (an anticoagulant) is the initial treatment for a confirmed PE to prevent the extension Recall that heparin is used to treat PE. LMWH is used as prophylaxis.
following as the most important initial or propagation of thrombi, and to inhibit the formation of new clots. Coumadin is usually
pharmacological treatment for a cardiac client on started after the heparin has been infused for several days. LMWH is not the drug of choice for
bedrest just diagnosed with a pulmonary embolism this client, and TPA is a thrombolytic drug that dissolves clots that are already formed.
(PE)?
1.‐ Heparin
2.‐ Low–molecular weight heparin (LMWH)
3.‐ Warfarin (Coumadin)
4.‐ Tissue plasminogen activator (TPA)

2160 For a client on a heparin drip for a diagnosis of Correct answer: 2 For a therapeutic effect of heparin, the PTT value should be 1.5–2 times the normal control Remember 1.5–2 times control for PT and PTT values.
pulmonary embolus, which of the following partial level. Options 1 and 3 show subtherapeutic levels, and option 4 shows an excessively high
thromboplastin times (PTT) shows a therapeutic level.
effect?
1.‐ PTT value of 20 sec, control of 14
2.‐ PTT value of 29 sec, control of 14
3.‐ PTT value of 13 sec, control of 14
4.‐ PTT value of 42 sec, control of 14

2161 In planning care for a client, the nurse understands Correct answer: 3 The pulmonary vascular system has low pressure, high blood flow, and low resistance; Select the option that addresses both pulmonary and systemic pressure.
that pulmonary hypertension is a more severe therefore, it takes more to raise the pressure within the pulmonary artery. This is not reflected
condition than hypertension seen in a cardiac client in the systolic pressure as readily.
because:
1.‐ An underlying condition could be masking the symptoms of pulmonary hypertension.
2.‐ The low resistance in the pulmonary vascular system makes the diagnosis more difficult to detect.
3.‐ The pulmonary artery pressure rises significantly before a systolic increase occurs.
4.‐ The pulmonary vasculature thins, and can rupture, in pulmonary hypertension.

2162 A client receiving mechanical ventilation after acute Correct answer: 1 Because of the action of a neuromuscular blocking agent, movement and communication are Select the response that gives a truthful response in understandable terms.
respiratory failure has been given a neuromuscular blocked, and this can be frightening to a family member. Option 3 is true, but is an
blocking agent to suppress the ability to breathe and inappropriate answer for a concerned family member who probably will not understand what
lessen the agitation caused by the ventilator. A family was said. Options 2 and 4 are unacceptable.
member asks the nurse why the client does not move.
The nurse's best response is:

1.‐ "The lack of movement and communication is temporary, and will return when the drug wears off."
2.‐ "The depression of the drug shouldn't be taken seriously, although the client appears to be comatose."
3.‐ "A neuromuscular blocking agent was given, and this drug blocks the action of acetylcholine at the nicotinic receptors of skeletal muscles."
4.‐ "Perhaps you should speak to the physician."

2163 If a client demonstrates chest wall movement on Correct answer: 2 With flail chest, the classic chest wall movement is for the wall to collapse upon inspiration Flail chest is the only disorder listed that has a peculiar chest movement as a symptom.
inspiration and expiration, the nurse should be because of the negative pressure exerted within the lung cavity. The positive pressure causes
suspicious of: the chest wall to expand upon expiration. This does not happen with the other disorders listed.

1.‐ Pneumothorax.
2.‐ Flail chest.
3.‐ Hemothorax.
4.‐ Adult respiratory distress syndrome (ARDS).
2164 A client with extremely foul‐smelling, purulent Correct answer: 3 The smaller bronchi and bronchioles become dilated as a result of the infection in the Recall that purulent sputum is characteristic of bronchiectasis.
sputum; wheezing; dyspnea; and weight loss is respiratory tract. As the dilation occurs, pockets form where infectious material is trapped and
probably experiencing: allows abscesses to develop. The walls of the bronchi are weakened, and become necrotic,
resulting in foul‐smelling sputum.
1.‐ Lobar pneumonia.
2.‐ Emphysema.
3.‐ Bronchiectasis.
4.‐ Asthma.

2165 A male client who has chronic obstructive pulmonary Correct answer: 3 In emphysema, air is trapped, and causes hyperinflation of the lung, thus causing the barrel Recall that barrel chest is associated with emphysema.
disease (COPD) breathes better while sitting up on the chest. Although asthma and chronic bronchitis also can be a part of COPD, these diagnoses do
side of the bed. His appearance is cachectic and barrel‐ not account for the appearance of the chest.
chested. The nurse realizes that the barrel chest
appearance is a result of:

1.‐ Chronic bronchitis.


2.‐ Asthma.
3.‐ Emphysema.
4.‐ Empyema.

2166 A client newly diagnosed with pneumonia has fine Correct answer: 4 In a newly diagnosed client, the consolidation can be so dense that the client is ineffective in Omit options 1 and 2, as they would not address the issue. Option 3 would increase
rales, a nonproductive cough, dyspnea, fever, and pain removing the sputum, and lung sounds are quite diminished. Options 1 and 2 are correct, but congestion and ability to cough.
upon inspiration. Regardless of the effort to cough, the are not necessarily going to help with coughing, which can be very painful. Movement will help
client fatigues easily, and is not effective. A nursing the client, although planned rest periods are needed (option 3).
intervention that will help the ineffective cough is to:

1.‐ Use a nonrebreather mask.


2.‐ Offer endotracheal suctioning.
3.‐ Encourage bedrest.
4.‐ Teach the client to splint the chest with a pillow.

2167 A client with an exacerbation of chronic obstructive Correct answer: 1 Respiratory failure in a COPD client is manifested by a drop in oxygen of 10–15 mmHg from The reduction in PaO&lt;sub&gt;2&lt;/sub&gt; combined with the acidosis is the hint to
pulmonary disease (COPD) is admitted to the unit for the previous level. Although the other ABGs are not adequate, the values must be compared the correct response.
treatment. The arterial blood gases (ABGs) upon with previous values for the COPD client who is already hypoxic and hypercapnic.
admission are as follows: PaO&lt;sub&gt;2&lt;/sub&gt;
49 mmHg; PaCO&lt;sub&gt;2&lt;/sub&gt; 55 mmHg;
and pH 7.35. Within hours, the client becomes restless,
cyanotic, diaphoretic, tachypneic, and tachycardic. The
nurse calls the physician, and repeat ABGs are ordered.
Which set of ABGs would indicate respiratory failure?

1.‐ PaO2 39 mmHg; PaCO2 40 mmHg; pH 7.30


2.‐ PaO2 47 mmHg; PaCO2 62 mmHg; pH 7.37
3.‐ PaO2 50 mmHg; PaCO2 40 mmHg; pH 7.32
4.‐ PaO2 39 mmHg; PaCO2 60 mmHg; pH 7.35

2168 Which of the following would be the priority Correct answer: 2 Pain management is a primary consideration in any end‐stage cancer. Although all of the Recall that with end‐stage disease, comfort is the priority.
assessment for a client with end‐stage lung cancer? other answers are important when caring for a client with lung cancer, palliative support is
essential, and should include around‐the‐clock pain relief. This is not the appropriate time to
be concerned about opioid addiction.
1.‐ Lung sounds
2.‐ Pain
3.‐ Sputum color and consistency
4.‐ Any changes in respiratory function

2169 A client with chronic obstructive pulmonary disease Correct answer: 4 Oxygen is drying to the nasal passageway, and under the positive pressure of CPAP, a Recognize that the word drying in option 4 is the key to the correct response.
(COPD), sleep apnea, and cardiac problems has been nosebleed is likely to occur in individuals who are already prone to this. Applying a small
placed on continuous positive airway pressure (CPAP) amount of antibiotic ointment in the nostril prone to bleeding before bedtime might help.
at night at home to assist with the sleep apnea. He Cardiac conditions such as hypertension or arteriosclerosis, as well as medications such as
complains of having a nosebleed after about a week of anticoagulant or antiplatelet drugs, can also cause nosebleeds. The stem, however, does not
the home oxygen therapy. What is the nurse's best give enough information as to the type of cardiac problems that the client has; avoid reading
explanation for this? into the question.

1.‐ The oxygen therapy is probably not the cause; other reasons should be investigated.
2.‐ The cardiac problems are more likely the cause of the nosebleed.
3.‐ The oxygen causes a vasodilating effect, which could enhance a nosebleed.
4.‐ The oxygen is drying to the nasal passage, and could be irritating the membranes.

2170 Which of the following clients is at highest risk of Correct answer: 3 The etiology is not fully understood, but tall, young, thin‐chested men who smoke are more Recall that spontaneous pneumothorax occurs in young, tall, thin males.
experiencing a spontaneous pneumothorax? prone to developing a spontaneous pneumothorax. Option 1 is more likely to be positive for
tuberculosis (TB); option 2 might experience sleep apnea; and option 4 is more likely to
experience pulmonary hypertension.
1.‐ A client who is homeless, HIV‐positive, and lives in an overcrowded alley
2.‐ A client who is obese, elderly, and a chronic smoker
3.‐ A tall, thin young man who smokes
4.‐ A 35‐year‐old woman who smokes and is overweight

2171 Asthma, COPD, cystic fibrosis, and bronchiectasis all Correct answer: 1 Because of the obstruction to airflow, bronchoconstriction is associated with all of these Recall that obstruction is associated with wheezing.
cause problems with obstruction to airflow out of the diagnoses, mostly because of damage to the lung or inflammation. The other three symptoms
lung. The nurse expects which of the following are specific to one particular disorder.
assessment findings to be present in these diagnoses?

1.‐ Wheezing
2.‐ Angina
3.‐ Orthopnea
4.‐ Anxiety

2172 The nurse is teaching nutritional information to the Correct answer: 3 The excessive mucus production from cystic fibrosis blocks the ducts of the pancreas, which Look for the answer choice (option 3) that mentions pancreatic failure.
mother of a child with cystic fibrosis. As a basis for the prevents the digestive enzymes from being released. In the absence of these enzymes,
discussion, the nurse would include that malnutrition digestion is impaired.
and weight loss are caused by:
1.‐ Decreased appetite because of chloride loss.
2.‐ Suppressed appetite because of the increased work of breathing.
3.‐ Pancreatic failure and insufficient pancreatic enzymes to digest food.
4.‐ A downward push of the diaphragm, which makes eating more difficult.
2173 A client underwent bronchoscopy using conscious Correct answer: 2 An intact gag reflex indicates that topical sedation has lost its effect, and that the client is able Recall that presence of a gag reflex protects the airway, which is always a priority.
sedation. Which of the following outcomes is most to swallow, a major safety consideration prior to discharging the client from the health care
important to meet prior to discharging the client? facility. The ability to swallow would precede consumption of oral intake. Knowing symptoms
to report to the physician following discharge is important, but the physiological condition
takes priority in this case. The client's ability to verbalize discharge instructions prior to
discharge is not a good predictor of postdischarge memory; therefore, it is essential that
written instructions be sent home with the client. Fever, if present, can take hours or days to
resolve; the client might have been febrile at the onset of the procedure.

1.‐ The client verbalizes symptoms to report to the physician following discharge.
2.‐ The client has an intact gag reflex.
3.‐ The client is afebrile.
4.‐ The client is taking oral fluids.

2174 A client is admitted to the hospital with the medical Correct answer: 3 The medulla and pons are the areas of brain tissue that control breathing. Injury to these Recall that the medulla controls vital functions. Omit options 1 and 2 as relating to
diagnosis of traumatic brain injury. From the tissues would produce alterations in the client's breathing rate and pattern. The other options endocrine functions.
assessment finding of slow, shallow respirations, the are incorrect.
nurse concludes that which area of the brain is
affected by the injury?
1.‐ Anterior pituitary
2.‐ Hypothalamus
3.‐ Medulla
4.‐ Cerebral cortex

2175 In the client with right lung pneumonia, the nurse Correct answer: 4 With unilateral lung disease, the example to remember is "good lung down." Since ventilation Recall that keeping the infected lung up promotes expansion.
should encourage which position to facilitate optimal and perfusion are gravity‐dependent, enhancing ventilation and perfusion to healthy lung
oxygenation? tissue and alveoli will enhance oxygenation. Perfusion refers to the circulation of blood into
the tissues and cells. Supine positioning would provide near‐equal ventilation and perfusion to
both lungs. In the diseased lung, excess fluid and fibrosis inhibit gas exchange at the pulmonary
capillary membrane, thereby diminishing oxygenation.

1.‐ Prone position


2.‐ Supine position, with head elevated 30 degrees
3.‐ Positioned with the right side dependent
4.‐ Positioned with the left side dependent

2176 The nurse is making a home visit to a 70‐year‐old Correct answer: 2 Increased anterior–posterior diameter of the chest, pursed‐lip breathing, and circumoral Recognize that option 2 is the only finding that is considered out of the ordinary in light of
client with emphysema. Which assessment finding has cyanosis are chronic findings in clients with emphysema. They do not indicate acute changes in the diagnosis.
the most serious implication for this client's nursing the client's condition. Bilateral crackles throughout the lung fields indicate excessive
care? pulmonary fluid, requiring acute intervention. The etiology of the fluid excess in the lungs
needs to be explored in depth.
1.‐ Increased anterior–posterior diameter of the chest
2.‐ Bilateral crackles throughout the lung fields
3.‐ Pursed‐lip breathing
4.‐ Circumoral cyanosis
2177 A postoperative client with emphysema is receiving Correct answer: 2 Carbon dioxide level is one of the primary stimuli for breathing in clients with chronic Remember to always associate COPD with hypoxic drive.
oxygen at 2 L/min via nasal cannula when he complains obstructive lung disease, who adjust to higher‐than‐normal carbon dioxide levels. Abrupt
of feeling dyspneic. The client's wife asks the nurse to elevation of the oxygen level will depress the stimulus for breathing, and can even produce
increase the oxygen intake to help him breathe more respiratory arrest. Administration of 100% oxygen to the client with COPD who is not receiving
easily. Which response by the nurse is appropriate? mechanical ventilation is highly likely to lead to depressed breathing and respiratory arrest.
The wife's presence might be providing comfort and support for the client. Psychological
distress caused by her absence could worsen the dyspnea. Pain medication can depress
breathing.
1.‐ Switch the oxygen to a 100% non‐rebreathing mask.
2.‐ Explain to the wife that high concentration of oxygen could depress breathing.
3.‐ Ask the wife to leave the room to let the client get some sleep.
4.‐ Administer pain medication.

2178 For the client with bacterial pneumonia, the nurse Correct answer: 3 Productive cough is compatible with bacterial pneumonia, and differentiates it from viral Recognize that option 3 describes an expected symptom of pneumonia.
expects the finding of: pneumonia. Excessive sputum is produced as pulmonary bacteria die. The white blood cell
count is elevated in bacterial pneumonia compared with viral pneumonia. Chest x‐ray findings
with bacterial pneumonia usually show consolidation, whereas the chest x‐ray is often normal
with viral pneumonia.
1.‐ Normal white blood cell count.
2.‐ Atelectasis.
3.‐ Productive cough.
4.‐ Unremarkable chest x‐ray.

2179 The occupational health nurse teaches a group of Correct answer: 1 Many foreign particles inhaled from the environment are non‐biodegradable, and cause Recognize that the substances described are irritants, and cause inflammation.
employees to follow all safety policies because chronic inflammation of lung tissue. The chronic inflammation leads to progressive scarring
irreversible lung damage can result from occupational and fibrosis of lung tissue, thereby impairing the gas diffusion capabilities of the lungs.
exposure to substances such as coal, asbestos, or glass, Antigen–antibody reactions are related to exposure to protein substances.
due to:
1.‐ Chronic inflammation of lung tissue.
2.‐ Frequent antigen–antibody reaction to foreign substances.
3.‐ Chronic air trapping.
4.‐ Surfactant deficiency.

2180 In developing the care plan for a client with Correct answer: 2 Mycobacterium tuberculosis is transmitted via airborne droplets, so use of a properly fitted Recall that TB is spread by droplet. A special mask as described in option 2 is the key to the
pulmonary mycobacterium tuberculosis, what primary particulate filter mask is indicated to prevent its spread. The other options do not represent correct response.
precaution should be included? methods of preventing airborne droplet transmission.
1.‐ Contact skin precautions
2.‐ Use of special mask to avoid inhaling infected airborne droplets
3.‐ Avoidance of blood contamination
4.‐ Containment of draining wounds

2181 The family of a client with emphysema asks the nurse Correct answer: 4 Emphysema is a chronic disease with progressive destruction of alveoli and loss of alveolar The reference to surface area for gas exchange describes the pathology of emphysema.
about the disease process. The nurse explains that the area available for gas exchange. Paralysis of respiratory muscles, airway obstructions, and
disorder results from a decreased oxygen supply due pleural effusion would diminish ventilatory capacity, which ultimately could lead to decreased
to: oxygen supply.
1.‐ Paralysis of respiratory muscles.
2.‐ Infectious obstructions.
3.‐ Pleural effusion.
4.‐ Loss of surface area for gas exchange.
2182 In the client with new rib fractures, which assessment Correct answer: 3 Clients with rib fractures should be assessed periodically for the possible complication of Recognize that options 3 and 4 describe one‐sided changes. Option 3 would be an
finding would best alert the nurse to the possible pneumothorax. Decreased or absent breath sounds are related to pneumothorax, which expected finding.
development of a pneumothorax? compresses functional lung tissue. Pink, frothy sputum is a possible (but unlikely) finding in
clients with pneumothorax. Hoarseness is indicative of an airway obstruction or laryngeal
nerve paralysis. Percussion sounds are hyperresonant in the area of a pneumothorax due to
collection of air in the pleural space.

1.‐ Pink, frothy sputum


2.‐ Hoarseness
3.‐ Decreased breath sounds on the affected side
4.‐ Dullness to percussion on the unaffected side

2183 A client comes to the clinic with an acute asthma Correct answer: 2 Expiratory wheezing is a characteristic finding in acute asthma, due to airway constriction. Recall that wheezing is most commonly associated with asthma.
episode. Which breath sound characteristic does the Crackles are indicative of excess pulmonary fluid, which is not a typical finding with acute
nurse expect to find upon auscultation? asthma. Rhonchi are related to mucus obstruction of large airways, and are a common finding
in chronic obstructive pulmonary disease processes.
1.‐ Bilateral crackles
2.‐ Wheezing
3.‐ Diminished breath sounds in the upper lobes
4.‐ Rhonchi

2184 Which of the following blood gas reports would the Correct answer: 4 During the later stages of COPD, arterial blood gas findings indicate low pH, elevated Eliminate options 2 and 3, as they are relatively normal. Select option 4 based on the
nurse expect in a client with progressive chronic pCO&lt;sub&gt;2&lt;/sub&gt;, low pO&lt;sub&gt;2&lt;/sub&gt;, and elevated acidotic pH.
obstructive pulmonary disease (COPD)? HCO&lt;sub&gt;3&lt;/sub&gt;&lt;sup&gt;‐&lt;/sup&gt;, which indicate the body's attempt to
compensate for chronically low pH. Option 1 is indicative of respiratory alkalosis; options 2 and
3 are variations of normal ABG results.
1.‐ pH 7.55; PaCO2 30 mmHg; PaO2 80 mmHg; HCO3‐ 24 mEq/L
2.‐ pH 7.40; PaCO2 40 mmHg; PaO2 94 mmHg; HCO3‐ 22 mEq/L
3.‐ pH 7. 38; PaCO2 45 mmHg; PaO2 88 mmHg; HCO3‐ 20 mEq/L
4.‐ pH 7.30; PaCO2 60 mmHg; PaO2 70 mmHg; HCO3‐ 30 mEq/L

2185 What is the priority item in discharge teaching for the Correct answer: 2 Cigarette smoking is the primary etiology of chronic bronchitis, so cessation is the priority for Recall that smoking cessation is encouraged in anyone, but especially respiratory clients.
client with chronic bronchitis? the client. Teaching the client about potential side effects of any prescribed medications
should be included in all discharge teaching. Avoidance of crowds to lower the risk of
pulmonary infections is a recommendation that is more individualized and less common than
the need for smoking cessation. Fluids are often increased.

1.‐ Fluid restriction


2.‐ Smoking cessation
3.‐ Avoidance of crowds
4.‐ Side effects of drug therapy

2186 A client has returned to the clinic 72 hours after a Correct answer: 3 An induration of 5–9 mm resulting from a tuberculin skin test is indicative of close contact Recognize that option 3 is the only option in which all of the information presented is true.
tuberculin skin test with an induration of about 5–6 with an individual infected with mycobacterium tuberculosis. The client with this finding will be
mm at the administration site. The client is visibly prescribed isoniazid for 6–12 months as prophylaxis against development of active TB. History
upset, and states: "I can't believe I have TB!" Which of diabetes is not related to false positive tuberculin skin test. The nurse should demonstrate a
statement by the nurse is appropriate? calm, supportive, and informing manner with this client.

1.‐ "You'll need to put on a mask and wear it whenever you are around other people."
2.‐ "The doctor will prescribe Isoniazid for you to take for the next 3 months."
3.‐ "This finding does not confirm TB; it might indicate a recent exposure to tuberculosis."
4.‐ "We'll need to do a chest x‐ray. This could be falsely positive because of your history of diabetes."
2187 What instruction is most important for the nurse to Correct answer: 3 For any client with a tracheostomy, maintenance of the airway is clearly the priority. Clients Use the ABCs (airway, breathing, circulation) to identify option 3 as the best response.
provide during discharge teaching of a client who are taught to perform routine tracheostomy care to prevent airway obstruction. Only those
underwent a laryngectomy? clients discharged with a feeding tube will need instruction about operation of a feeding pump.
Wound care and use of a Passy‐Muir valve for communication are important factors to include
in discharge teaching, but the airway is the clear priority.

1.‐ Operation of feeding pump


2.‐ Use of a Passy‐Muir (speaking tracheostomy) valve
3.‐ Tracheostomy care
4.‐ Wound care

2188 A postoperative client has a sudden onset of Correct answer: 3 With sudden onset of shortness of breath, the priority is for the nurse to maintain airway Recognize that all of the actions are correct but need to be performed in a logical order.
shortness of breath. What initial action by the nurse is patency and gas exchange. Positioning the client supine with a high degree of head elevation Positioning to attempt to improve oxygenation is first priority.
indicated? will assist with airway maintenance and ventilation. The nurse should then rapidly assess the
client's heart and lung status before notifying the physician.
1.‐ Notify the physician.
2.‐ Assess oxygen saturation using pulse oximetry.
3.‐ Assist the client to a high Fowler's position.
4.‐ Auscultate the heart and lungs.

2189 A client has a right chest tube following a Correct answer: 1 Gravity helps maintain the water seal, thereby preventing backflow of air and fluid into the Recognize that options 2, 3, and 4 are interventions that are not recommended.
thoracotomy. When assisting the client to ambulate, chest. The chest tube should never be clamped, as this can cause pneumothorax. The chest
what measure is appropriate to maintain the water tube should not be milked unless ordered by the physician for clients with visible clots in the
seal? chest drainage tubing. Milking the chest tube creates suction within the tubing, and can cause
pleural tissue damage.
1.‐ Keep the collection device below the level of the chest.
2.‐ Clamp the chest tube before assisting the client out of bed.
3.‐ Milk the chest tube when the client returns to bed, to assess patency.
4.‐ Connect the collection device to a portable suction machine.

2190 A client is brought to the Emergency Department Correct answer: 3 Tension pneumothorax is a life‐threatening condition, so the nurse must recognize potential Recall that tracheal deviation is associated with tension pneumothorax.
following a motor vehicle crash with a tree. Which indicators. Deviation of the trachea toward the unaffected side occurs due to increased
finding is suggestive of a tension pneumothorax? pressure within the pleural cavity. Increasing pressure on the great vessels in the chest causes
decreased cardiac output, which can be fatal. Hypotension and tachypnea occur with
pneumothorax, but are also related to numerous other conditions. Unilateral wheezing is
indicative of narrowing of the airways.

1.‐ Tachypnea
2.‐ Hypotension
3.‐ Tracheal deviation
4.‐ Unilateral wheezing

2191 What is the priority nursing diagnosis for the finding Correct answer: 3 Secondary polycythemia, or increased red blood cell count, develops as chronic obstructive Recall that polycythemia influences perfusion because of chronic hypoxia.
of secondary polycythemia in a client with chronic pulmonary disease occurs, in response to chronic hypoxemia. Of the options above, impaired
obstructive pulmonary disease? tissue perfusion related to chronic hypoxemia is the only factor related to development of
secondary polycythemia. Risk for injury in these clients related to venous thrombi or use of
oxygen might or might not be present. Impaired gas exchange also could be a factor; however,
it is related to hypoxia.
1.‐ Risk for Injury related to venous thrombi
2.‐ Risk for Injury related to use of oxygen
3.‐ Impaired Tissue Perfusion related to chronic hypoxemia
4.‐ Impaired Gas Exchange related to factors other than hypoxia
2192 When auscultating breath sounds in the client with an Correct answer: 2 Wheezing is a common finding during an acute asthma episode; however, the wheezing is not Recall that in severe obstruction, there is no air passing, and therefore no wheezing.
acute asthma episode, the nurse uses which of the a consistent predictor of the severity of the attack. Airway obstruction might be so severe that
following to guide interpretation of severity of the client is moving little or no air, and is experiencing severe respiratory distress. Breath
findings? sounds are prolonged in expiration with asthma, but this factor does not alter the plan of care
in any way.
1.‐ Severity of airway obstruction is associated with intensity of wheezing.
2.‐ Wheezing might be absent with severe airway obstruction.
3.‐ Unilateral wheezing indicates an origin for respiratory distress other than asthma.
4.‐ Breath sounds are prolonged on expiration.

2193 What nursing intervention is most effective to Correct answer: 1 Frequent coughing and deep breathing is an easy maneuver for the postoperative client that Remember nursing basics of coughing and deep breathing to keep airways clear.
prevent atelectasis in the postoperative client? has great benefit to optimize ventilation. Good pain management facilitates effective coughing
and deep breathing. Getting the client out of bed, or administering oxygen or bronchodilators,
are all appropriate interventions for preventing or treating atelectasis, but clearly the best
option is to prevent its occurrence by simple maneuvers such as coughing and deep breathing.

1.‐ Frequent coughing and deep breathing


2.‐ Assisting the client out of bed
3.‐ Administration of bronchodilators
4.‐ Supplemental oxygen

2194 What blood gas alteration would the nurse expect to Correct answer: 2 Carbon dioxide is eliminated from the body as exhaled gas. The greater the rate of breathing, Recall that rapid breathing would reduce CO&lt;sub&gt;2&lt;/sub&gt;.
occur with persistent tachypnea? the greater the quantity of carbon dioxide eliminated. Normal CO&lt;sub&gt;2&lt;/sub&gt;
levels are 35–45 mmHg. Normal HCO&lt;sub&gt;3&lt;/sub&gt;&lt;sup&gt;‐&lt;/sup&gt; levels
are 22–26 mEq/L. With decreasing CO&lt;sub&gt;2&lt;/sub&gt; levels,
HCO&lt;sub&gt;3&lt;/sub&gt;&lt;sup&gt;‐&lt;/sup&gt; should also fall to compensate.

1.‐ pCO2 48 mmHg


2.‐ pCO2 30 mmHg
3.‐ pO2 82 mmHg
4.‐ HCO3‐ 29 mEq/L

2195 What nursing intervention might improve Correct answer: 4 For the client with ARDS, placing the client in a prone position allows for expansion of the Recall that prone positioning is rarely performed, but is associated with ARDS treatment.
ventilation–perfusion matching in clients with acute posterior chest wall, which might be effective in enhancing oxygenation. Transfusing red blood
respiratory distress syndrome (ARDS) who show no cells or albumin does not increase oxygenation in ARDS. Option 3 should have been performed
improvement with increases in oxygen concentration as an initial measure.
administered?
1.‐ Transfusion of packed red blood cells
2.‐ Infusion of albumin
3.‐ Positioning supine with head elevated 30–45 degrees
4.‐ Prone positioning

2196 The nurse would expect to find a diminished Correct answer: 1 Carbon dioxide is eliminated from the body as exhaled gas. The faster the rate of breathing, Recall that rapid breathing results in low pCO&lt;sub&gt;2&lt;/sub&gt;, from “blowing off”
pCO&lt;sub&gt;2&lt;/sub&gt; level in a client who has the greater the quantity of carbon dioxide eliminated. CO&lt;sub&gt;2&lt;/sub&gt;.
physical assessment finding of:
1.‐ Hyperventilation.
2.‐ Hypoventilation.
3.‐ Prolonged expiration.
4.‐ Stridor.
2197 While making rounds, the nurse observes a client Correct answer: 2 Oxygen administered by a Venturi mask can be regulated to deliver between 24 and 50 Eliminate option 4, as this is contraindicated in most circumstances. Eliminate option 1, as
receiving oxygen through a Venturi mask. The nurse percent, which is a benefit for clients who require higher oxygen supplement without the device does allow rebreathing.
expects that the primary benefit of this method of mechanical ventilation. The Venturi mask does not prevent rebreathing of carbon dioxide, as
oxygen supplementation is: does a non‐rebreather mask. Oxygen concentration of 100 percent would be administered to
COPD clients only in rare circumstances via mechanical ventilation.

1.‐ The ability to prevent rebreathing of exhaled carbon dioxide.


2.‐ Oxygen concentration can be regulated.
3.‐ Constant humidity can be administered to liquefy pulmonary secretions.
4.‐ The ability to deliver up to 100 percent oxygen concentration for clients with COPD.

2198 From the client's history, the nurse recognizes that Correct answer: 3 Cigarette smoking is the leading cause of lung cancer. Smokeless tobacco is more often Recall that smoking is the leading cause of lung cancer.
which of the following is the most significant factor associated with oral cancer. Air pollution could also be a contributing factor to development of
related to the development of bronchogenic lung cancer. History of asthma is not associated with greater risk of lung cancer.
carcinoma?
1.‐ Asthma
2.‐ Smokeless tobacco
3.‐ Cigarette smoking
4.‐ Air pollution

2199 Following placement of a central venous line, which Correct answer: 3 Increased heart rate and/or respiratory rate within minutes to several hours following central Use the ABCs (airway, breathing, and circulation) to identify changes in respiratory or
of the following data should the nurse report venous line insertion are symptoms of a pneumothorax caused by puncture of the pleura. The cardiac function that should be reported.
immediately to the physician? client will require a chest x‐ray to determine if a pneumothorax is present. If the client does
have a pneumothorax, placement of a chest tube is likely. Pain at central line insertion site,
fever, and diminished breath sounds in lung bases will require intervention, but the etiology of
these symptoms is not likely to be potentially life‐threatening, as is the development of a
pneumothorax.

1.‐ Pain at the insertion site


2.‐ Fever
3.‐ Increased heart rate and/or respiratory rate
4.‐ Diminished breath sounds in lung bases

2200 The nurse recognizes that which of the following Correct answer: 1 Pulmonary embolism is the most common complication of hospitalized clients, and is due The qualifier postoperative should lead to the selection of immobility, which is frequent in
factors in the postoperative client poses the greatest primarily to immobility. Postoperative clients are at risk for immobility associated with the postoperative period.
risk for development of pulmonary embolism, and surgery. Infrequent coughing and deep breathing is associated with postoperative atelectasis.
must be addressed in the client's care plan? Pulmonary embolism originates from deep veins in the extremities, not in superficial varicose
veins.
1.‐ Immobility
2.‐ Infrequent coughing and deep breathing
3.‐ Varicose veins
4.‐ Lower‐extremity edema

2201 When planning care for the client admitted with Correct answer: 1 All of these nursing diagnoses are appropriate for the client with COPD; however, the primary Look for the diagnosis that is most descriptive of the pathology.
exacerbation of chronic obstructive pulmonary disease alteration is related to impaired gas exchange. The alteration in gas exchange is the etiology of
(COPD), the nurse identifies which nursing diagnosis as the other problems.
the priority?
1.‐ Impaired Gas Exchange
2.‐ Activity Intolerance
3.‐ Risk for Infection related to impaired respiratory defenses
4.‐ Ineffective Breathing Pattern
2202 Nutritional recommendations for the client with Correct answer: 2 Clients with COPD are often underweight due to the fatigue associated with eating. Associate the increased respiratory effort of COPD with increased calorie need.
chronic obstructive pulmonary disease (COPD) should Therefore, calorie‐dense foods will help them to ingest more calories with less effort for
include which of the following? eating. Carbohydrate intake should be about 50 percent of total calories. Since carbohydrates
metabolize to carbon dioxide and water, excessive carbohydrate intake could contribute to
carbon dioxide retention. Small, frequent intakes of food should be encouraged.

1.‐ Weight‐reduction guidelines


2.‐ Identification of foods that are calorie‐dense
3.‐ Avoidance of between‐meal snacks
4.‐ More than 50 percent of intake as carbohydrates

2203 A client has been admitted for dehydration after Correct answer: 1 A prolonged fasting state can lead to dehydration. During fasting, the body reverts to cellular Note the critical word fasting that indicates this is a metabolic rather than respiratory
fasting for five days. For which of the following breakdown to maintain energy, and lactic and pyruvic acids build up in the body. This problem, which eliminates options 3 and 4. Choose option 1 over 2 because metabolic by‐
acid–base imbalances would the nurse assess this accumulation of acids leads to the development of metabolic acidosis. Options 2 and 4 are products are acidic in nature, not alkaline.
client? incorrect because alkalosis would not occur. Option 3 is incorrect because the primary
disturbance is not respiratory.
1.‐ Metabolic acidosis
2.‐ Metabolic alkalosis
3.‐ Respiratory acidosis
4.‐ Respiratory alkalosis

2204 A client is admitted to the hospital after vomiting for Correct answer: 2 Vomiting leads to the loss of hydrochloric acid from gastric acids. Hydrogen ions must leave Note the critical word vomiting, and recall that stomach contents are rich in acid. Loss of
three days. Which of the following ABG results would the blood to replace this acidity in the stomach. Option 2 reflects metabolic alkalosis elevated acid would raise the pH (eliminating option 1 and 3) and lead to increased free circulating
the nurse expect? pH and HCO&lt;sub&gt;3&lt;/sub&gt;&lt;sup&gt;‐&lt;/sup&gt;, and normal HCO&lt;sub&gt;3&lt;/sub&gt;&lt;sup&gt;‐&lt;/sup&gt;, eliminating option 4.
PaCO&lt;sub&gt;2&lt;/sub&gt;. Option 1 is incorrect because it reflects respiratory acidosis
with partial compensation decreased pH, and elevated PaCO&lt;sub&gt;2&lt;/sub&gt; and
HCO&lt;sub&gt;3&lt;/sub&gt;&lt;sup&gt;‐&lt;/sup&gt;. Option 3 is incorrect because it reflects
a mixed acid–base imbalance metabolic alkalosis with respiratory acidosis: normal pH, and
elevated PaCO&lt;sub&gt;2&lt;/sub&gt; and HCO&lt;sub&gt;3&lt;/sub&gt;&lt;sup&gt;‐
&lt;/sup&gt;. Option 4 is incorrect because it reflects respiratory alkalosis increased pH,
decreased PaCO&lt;sub&gt;2&lt;/sub&gt;, and normal
HCO&lt;sub&gt;3&lt;/sub&gt;&lt;sup&gt;‐&lt;/sup&gt;.

1.‐ pH 7.30; PaCO2 50; HCO3‐ 27


2.‐ pH 7.47; PaCO2 43; HCO3‐ 28
3.‐ pH 7.34; PaCO2 50; HCO3‐ 28
4.‐ pH 7.47; PaCO2 30; HCO3‐ 23

2205 A client is admitted to the hospital with a diagnosis of Correct answer: 1 Clients with respiratory acidosis from ingestion of barbiturates would have slow and shallow The wording of the question guides you to look for objective data, which eliminates option
respiratory acidosis secondary to overdose of respirations, leading to hypoventilation. Palpitations are a subjective complaint reported by 4. Also recall that barbiturates are CNS depressants, while options 2 and 3 indicate CNS
barbiturates. Which of the following assessments the client; the nurse cannot directly assess this symptom. In addition, palpitations are excitation. For this reason, eliminate options 2 and 3, which leaves option 1 as the correct
would the nurse encounter? associated with respiratory alkalosis. Tetany symptoms and increased deep tendon reflexes are option.
also associated with respiratory alkalosis.
1.‐ Slow, shallow respirations
2.‐ Tetany symptoms
3.‐ Increased deep tendon reflexes
4.‐ Palpitations
2206 A client is admitted with a diagnosis of renal failure. Correct answer: 2 Clients with renal failure have difficulty synthesizing HCO&lt;sub&gt;3&lt;/sub&gt;&lt;sup&gt;‐ First, recognize renal failure as a metabolic condition in which there is an impaired ability
Which of the following ABG results would the nurse &lt;/sup&gt; in the renal tubules secondary to the renal failure. These clients also retain to eliminate metabolic acids and wastes. With this in mind, eliminate options 1 and 4
expect to see with this client? K&lt;sup&gt; &lt;/sup&gt;, and subsequently develop metabolic acidosis. Option 2 reflects because of an elevated pH and a normal pH, respectively. Then choose option 2 over 3
uncompensated metabolic acidosis. Option 1 is incorrect because it reflects metabolic alkalosis because the bicarbonate (indicating metabolic status) is lower in option 2.
increased pH and HCO&lt;sub&gt;3&lt;/sub&gt;&lt;sup&gt;‐&lt;/sup&gt;, and normal
PaCO&lt;sub&gt;2&lt;/sub&gt;. Option 3 is incorrect because it reflects respiratory acidosis
decreased pH, increased PaCO&lt;sub&gt;2&lt;/sub&gt;, and normal
HCO&lt;sub&gt;3&lt;/sub&gt;&lt;sup&gt;‐&lt;/sup&gt;. Option 4 is incorrect because it reflects
a mixed acid–base imbalance metabolic alkalosis with a respiratory acidosis: normal pH, and
increased PaCO&lt;sub&gt;2&lt;/sub&gt; and HCO&lt;sub&gt;3&lt;/sub&gt;&lt;sup&gt;‐
&lt;/sup&gt;.

1.‐ pH 7.49; PaCO2 36; HCO3‐ 30


2.‐ pH 7.30; PaCO2 35; HCO3‐ 18
3.‐ pH 7.31; PaCO2 50; HCO3‐ 23
4.‐ pH 7.43; PaCO2 48; HCO3‐ 30

2207 A client is admitted to the hospital with atelectasis Correct answer: 4 A client with atelectasis has collapsed alveoli that retain CO&lt;sub&gt;2&lt;/sub&gt;, which The critical word in the stem of the question is atelectasis. Recall that this term is
and complaints of chest pain. For which of the can lead to respiratory acidosis. The client most likely would have hypoventilation as a associated with respiratory problems to eliminate options 2 and 3. Choose option 4 over
following acid–base imbalances would the nurse assess respiratory pattern, which would further contribute to the development of respiratory option 1, recalling that CO&lt;sub&gt;2&lt;/sub&gt; retention characterizes many
this client? acidosis. Options 1 and 3 are incorrect because the client would not be in an alkalotic state. respiratory conditions, leading to acidosis (since CO&lt;sub&gt;2&lt;/sub&gt; acts as an acid
Option 2 is incorrect because the primary disturbance is respiratory; clients with respiratory in the body).
problems can report “chest pain.” Further information would be needed to rule out cardiac
problems.
1.‐ Respiratory alkalosis
2.‐ Metabolic acidosis
3.‐ Metabolic alkalosis
4.‐ Respiratory acidosis

2208 A client is admitted to the hospital with respiratory Correct answer: 3 Obesity can lead to chest wall abnormalities and hypoventilation. Respiratory acidosis results Note the term acidosis in both the stem of the question and in the correct option to help
acidosis. Which of the following conditions most likely from hypoventilation. Option 1 is incorrect because prolonged diarrhea likely leads to the you choose option 2 correctly. Alternatively, eliminate options 1 and 4 because they are
led to the development of this state? development of metabolic acidosis. Option 2 is incorrect because DKA leads to the similar in that they both lead to fluid loss, and then eliminate option 3 as irrelevant.
development of metabolic acidosis. Option 4 is incorrect because diuretic administration leads
to the development of metabolic alkalosis.
1.‐ Severe diarrhea for several days
2.‐ Diabetic ketoacidosis
3.‐ Obesity
4.‐ Diuretics

2209 Which of the following signs and symptoms indicates Correct answer: 2 Clients who have metabolic acidosis develop Kussmaul's breathing (rapid and deep The critical words in the stem of the question are metabolic acidosis. Recall that in
that a client has metabolic acidosis? respirations). Weight gain and melena are not associated with the development of metabolic metabolic abnormalities, the respiratory system helps to compensate; this will help to
acidosis. Option 4 is incorrect because shallow breathing is associated with the development of eliminate options 1 and 3. Choose option 2 over option 4 because it is the option that
metabolic alkalosis. assists the body to “blow off” acid in the form of CO&lt;sub&gt;2&lt;/sub&gt;.

1.‐ Weight gain


2.‐ Rapid, deep respirations
3.‐ Melena
4.‐ Decreased respiratory rate and depth
2210 Which of the following medications should the nurse Correct answer: 3 Alkalosis, especially respiratory alkalosis, makes the client more sensitive to the effects of Specific knowledge of medications that are affected by alkalosis is needed to answer this
review first for its potential interaction in a client digoxin; toxicity can develop even at therapeutic levels. A serum digoxin level should be question. Use nursing knowledge and the process of elimination to make your selection.
admitted to the hospital with alkalosis? obtained, and the client evaluated for potential digoxin toxicity. Warfarin affects clotting
factors. Metformin can cause the development of lactic acidosis. Ibuprofen can cause gastric
irritation.
1.‐ Warfarin (Coumadin)
2.‐ Metformin (Glucophage)
3.‐ Digoxin (Lanoxin)
4.‐ Ibuprofen (Motrin)

2211 A client is admitted to the hospital with complaints of Correct answer: 2 Acute pain usually leads to hyperventilation, which causes CO&lt;sub&gt;2&lt;/sub&gt; to be Visualize a picture of the client in pain. This person is most likely to have an increased
sudden onset of severe abdominal pain. Which of the blown off, leading to an increased pH and decreased CO&lt;sub&gt;2&lt;/sub&gt; level. If the respiratory rate, which blows off carbon dioxide (eliminating option 1) and decreases the
following values in the arterial blood gas would the client has not compensated, the bicarbonate level will be normal. If the client is compensating, bicarbonate level as a compensatory mechanism. This will help you to easily choose option
nurse expect to see? then the bicarbonate level will decrease in an attempt to restore the pH. Option 1 is incorrect 2 as correct.
because it reflects only a slight elevation of PaCO&lt;sub&gt;2&lt;/sub&gt;; if the client were in
severe pain, the level would likely be higher. Option 3 is incorrect because the pH is only
slightly acidotic. Option 4 is incorrect because the oxygen saturation is within normal limits.

1.‐ PaCO2 48
2.‐ HCO3‐ 18
3.‐ pH 7.32
4.‐ SaO2 90

2212 A client is admitted to the hospital with an acid–base Correct answer: 3 The pH is low, indicating acidosis; the PaCO&lt;sub&gt;2&lt;/sub&gt; is elevated, indicating a First, eliminate option 2 because it is an alkalosis, and the client s pH of 7.33 indicates
imbalance. ABG results are pH 7.33; respiratory basis; and the HCO&lt;sub&gt;3&lt;/sub&gt;&lt;sup&gt;‐&lt;/sup&gt; is elevated, acidosis. Next, note that both the CO&lt;sub&gt;2&lt;/sub&gt; and
PaCO&lt;sub&gt;2&lt;/sub&gt; 49; indicating that compensatory mechanisms are partially working. Option 1 is incorrect because HCO&lt;sub&gt;3&lt;/sub&gt;&lt;sup&gt;‐&lt;/sup&gt; levels are abnormal, indicating that
HCO&lt;sub&gt;3&lt;/sub&gt;&lt;sup&gt;‐&lt;/sup&gt; compensation is taking place due to increased HCO&lt;sub&gt;3&lt;/sub&gt;&lt;sup&gt;‐ the body is attempting to compensate (eliminating option 1). Choose option 3 over 4
28. The nurse interprets these results as which of the &lt;/sup&gt; level. Option 2 is incorrect because the client is not alkalotic. Option 4 is incorrect because the elevated CO&lt;sub&gt;2&lt;/sub&gt; “matches” a respiratory acidosis, and the
following? because the primary disturbance is respiratory. The change in the HCO&lt;sub&gt;3&lt;/sub&gt;&lt;sup&gt;‐&lt;/sup&gt; (an alkaline substance) is rising to try
PaCO&lt;sub&gt;2&lt;/sub&gt; level is greater than the change in the to compensate.
HCO&lt;sub&gt;3&lt;/sub&gt;&lt;sup&gt;‐&lt;/sup&gt; level, which indicates a respiratory
disturbance.
1.‐ Respiratory acidosis, uncompensated
2.‐ Metabolic alkalosis, uncompensated
3.‐ Respiratory acidosis, partially compensated
4.‐ Metabolic acidosis, partially compensated

2213 A client is admitted to the hospital with numerous Correct answer: 4 The pH is just below the high limit, and the HCO&lt;sub&gt;3&lt;/sub&gt;&lt;sup&gt;‐ Note that the pH is within normal range, which indicates that the condition is
complaints of muscle weakness and twitching. ABG &lt;/sup&gt; is elevated, indicating a metabolic problem. The PaCO&lt;sub&gt;2&lt;/sub&gt; is compensated, thus eliminating options 1 and 3. Note that the high
results are pH 7.44; PaCO&lt;sub&gt;2&lt;/sub&gt; 49; elevated, indicating that compensation is taking place. Option 1 is incorrect because the client HCO&lt;sub&gt;3&lt;/sub&gt;&lt;sup&gt;‐&lt;/sup&gt; is a metabolic indicator (not a
HCO&lt;sub&gt;3&lt;/sub&gt;&lt;sup&gt;‐&lt;/sup&gt; is not acidotic. Option 2 is incorrect because the CO&lt;sub&gt;2&lt;/sub&gt; would be respiratory one), and is consistent with a pH near the high end of normal, to help you
30. The nurse interprets these as which of the decreased rather than elevated. Option 3 is incorrect because the primary disturbance is choose option 4 over 2.
following? metabolic, and the CO&lt;sub&gt;2&lt;/sub&gt; is elevated rather than decreased.

1.‐ Metabolic acidosis, uncompensated


2.‐ Respiratory alkalosis, compensated
3.‐ Respiratory alkalosis, uncompensated
4.‐ Metabolic alkalosis, compensated
2214 The nurse would suspect that a client who frequently Correct answer: 1 Excessive use of oral antacids can lead to metabolic alkalosis. Use of ibuprofen and Tylenol is Knowledge of medication side effects is needed to answer this question. First, eliminate
uses which of the following is at risk for the not associated with the development of metabolic alkalosis. Overdoses of aspirin can be options 2 and 4 because they are similar (non‐opioid analgesics). Then, eliminate option 3
development of metabolic alkalosis? associated with the development of respiratory alkalosis, and eventually can lead to metabolic because acid would not lead to alkalosis. Alternatively, recall that calcium carbonate is an
acidosis. antacid, which in excess could lead to metabolic alkalosis.

1.‐ Calcium carbonate (Tums)


2.‐ Ibuprofen (Motrin)
3.‐ Acetylsalicylic acid (aspirin)
4.‐ Acetaminophen (Tylenol)

2215 The nurse is admitting a client who has metabolic Correct answer: 4 Clinical manifestations of metabolic alkalosis are associated with the presence of tetany‐like Specific knowledge of the association between metabolic alkalosis and various electrolytes
alkalosis. The nurse plans to assess for signs and symptoms. Clients should be monitored for the presence of these symptoms because they is needed to answer this question. Use nursing knowledge and the process of elimination to
symptoms of which of the following electrolyte usually correlate with low levels of calcium. Although it is important to assess all serum make your selection.
imbalances? electrolyte values to obtain a comprehensive picture, the presence of hypocalcemia can cause
the client to have significant clinical symptoms. Early monitoring and prompt intervention can
result in restoration of balance.

1.‐ Hypernatremia
2.‐ Hypochloremia
3.‐ Hypermagnesemia
4.‐ Hypocalcemia

2216 A client s blood gas results are pH 7.48; Correct answer: 3 The client’s pH is high, indicating alkalosis. The PaCO&lt;sub&gt;2&lt;/sub&gt; is abnormal, Note that the pH is high, so the condition is not compensated, eliminating options 1 and 4.
PaCO&lt;sub&gt;2&lt;/sub&gt; 30; indicating a respiratory basis. The HCO&lt;sub&gt;3&lt;/sub&gt;&lt;sup&gt;‐&lt;/sup&gt; is Choose option 3 over 2 because a low CO&lt;sub&gt;2&lt;/sub&gt; correlates with a high
HCO&lt;sub&gt;3&lt;/sub&gt;&lt;sup&gt;‐&lt;/sup&gt; normal, indicating that compensation has not started. Option 1 is incorrect because the pH, whereas an HCO&lt;sub&gt;3&lt;/sub&gt;&lt;sup&gt;‐&lt;/sup&gt; at the lower end of
23. How will the nurse interpret these results? HCO&lt;sub&gt;3&lt;/sub&gt;&lt;sup&gt;‐&lt;/sup&gt; level would decrease with the normal range does not correlate with a high pH.
compensation. Options 2 and 4 are incorrect because the primary disturbance is respiratory, as
indicated by the decrease in the CO&lt;sub&gt;2&lt;/sub&gt; parameter.

1.‐ Respiratory alkalosis, compensated


2.‐ Metabolic alkalosis, uncompensated
3.‐ Respiratory alkalosis, uncompensated
4.‐ Metabolic alkalosis, compensated

2217 The nurse determines that a client with a nasogastric Correct answer: 2 A client who has prolonged nasogastric suction is apt to have higher levels of bicarbonate Eliminate options 1 and 4 first because impaction is a GI (metabolic) problem rather than a
tube on low suction for five days is at risk for because of hydrogen ion loss. Bicarbonate excess leads to a metabolic disturbance and the respiratory one. Choose option 2 over 3, recalling that pancreatic juices are rich in
developing which acid–base imbalance? development of metabolic alkalosis. Options 1 and 3 are incorrect because the client will not bicarbonate. A client who is impacted cannot eliminate bicarbonate in the stool, and thus it
experience acidosis. Option 4 is incorrect because the primary disturbance is caused by may be reabsorbed.
retained levels of bicarbonate in the body.

1.‐ Respiratory acidosis


2.‐ Metabolic alkalosis
3.‐ Metabolic acidosis
4.‐ Respiratory alkalosis

2218 The following ABG results are on the client s chart: pH Correct answer: 3 The pH indicates alkalosis; HCO&lt;sub&gt;3&lt;/sub&gt;&lt;sup&gt;‐&lt;/sup&gt; is high, First, note that the pH is high, and so the imbalance cannot be compensated (eliminating
7.50; PaCO&lt;sub&gt;2&lt;/sub&gt; 36; indicating a metabolic basis; and the PaCO&lt;sub&gt;2&lt;/sub&gt; is normal, which indicates option 2). Then note that HCO&lt;sub&gt;3&lt;/sub&gt;&lt;sup&gt;‐&lt;/sup&gt; is the
HCO&lt;sub&gt;3&lt;/sub&gt;&lt;sup&gt;‐&lt;/sup&gt; that compensation has not taken place. Option 1 is incorrect because with compensation, the abnormally high value (not CO&lt;sub&gt;2&lt;/sub&gt;), so the imbalance must be
30. How will the nurse interpret these blood gas PaCO&lt;sub&gt;2&lt;/sub&gt; level would be increased. Options 2 and 4 are incorrect because metabolic rather than respiratory (eliminating option 4). Choose option 3 over option 1
reports? the primary disturbance is metabolic, as reflected by the increased bicarbonate level. because the CO&lt;sub&gt;2&lt;/sub&gt; (normally 35–45) has made no attempt to rise to
compensate for the high HCO&lt;sub&gt;3&lt;/sub&gt;&lt;sup&gt;‐&lt;/sup&gt;.
1.‐ Metabolic alkalosis, partially compensated
2.‐ Respiratory alkalosis, compensated
3.‐ Metabolic alkalosis, uncompensated
4.‐ Respiratory alkalosis, uncompensated

2219 A client is admitted to the hospital. ABG results are Correct answer: 3 ABG results reflect elevated pH, indicating alkalosis, and normal An ability to interpret ABGs and specific knowledge of manifestations of metabolic
pH 7.50; PaCO&lt;sub&gt;2&lt;/sub&gt; 40; PaCO&lt;sub&gt;2&lt;/sub&gt; and an increased HCO&lt;sub&gt;3&lt;/sub&gt;&lt;sup&gt;‐ alkalosis are needed to answer this question. Use nursing knowledge and the process of
HCO&lt;sub&gt;3&lt;/sub&gt;&lt;sup&gt;‐&lt;/sup&gt; &lt;/sup&gt;, indicating metabolic alkalosis. Vomiting is a common cause of this condition. The elimination to make your selection.
29. Which of the following questions should the nurse presence of diarrhea is associated with metabolic acidosis. COPD and smoking are associated
ask the client in order to help determine an etiology with respiratory acidosis.
for this ABG result?

1.‐ “Have you had diarrhea lately?”


2.‐ “Do you have a history of COPD?”
3.‐ “How long have you had nausea and vomiting?”
4.‐ “Do you smoke?”

2220 A client s blood gas results are pH 7.36; Correct answer: 1 The pH is just within normal range, so the blood gas results are either normal or Because the pH is within normal range, eliminate options 3 and 4. Choose option 1 over 2
PaCO&lt;sub&gt;2&lt;/sub&gt; 50; compensated. However, the PaCO&lt;sub&gt;2&lt;/sub&gt; is high, indicating a respiratory because the pH is near the acidic end of the range and the high CO&lt;sub&gt;2&lt;/sub&gt;
HCO&lt;sub&gt;3&lt;/sub&gt;&lt;sup&gt;‐&lt;/sup&gt; problem, and thus the ABGs cannot be normal. The HCO&lt;sub&gt;3&lt;/sub&gt;&lt;sup&gt;‐ correlates with acidosis, whereas the high HCO&lt;sub&gt;3&lt;/sub&gt;&lt;sup&gt;‐
28. What do these results indicate to the nurse? &lt;/sup&gt; is also high, which along with a normal pH indicates complete compensation. &lt;/sup&gt; would correlate with an alkalosis.
Options 2 and 3 are incorrect because the primary disturbance is respiratory, as reflected by
the correlation between an elevated PaCO&lt;sub&gt;2&lt;/sub&gt; and a pH toward the low
end of normal. Option 4 is incorrect because the HCO&lt;sub&gt;3&lt;/sub&gt;&lt;sup&gt;‐
&lt;/sup&gt; level would be normal if there were no compensation taking place.

1.‐ Respiratory acidosis, compensated


2.‐ Metabolic acidosis, compensated
3.‐ Metabolic acidosis, uncompensated
4.‐ Respiratory acidosis, uncompensated

2221 Which of the following statements by the client Correct answer: 2 Respiratory alkalosis is caused by hyperventilation. Stress and anxiety are two things that can The critical word in the question is respiratory. Eliminate each of the incorrect options that
indicates that discharge teaching for respiratory cause hyperventilation. It is important that clients who are prone to develop respiratory would correlate better with a metabolic condition than with a respiratory one.
alkalosis is understood? alkalosis be aware of how to manage causative factors. Options 1 and 3 are incorrect because Alternatively, consider that a common cause of respiratory alkalosis is hyperventilation,
antacids and diuretics are associated with the development of metabolic alkalosis. Option 4 is which is often caused by anxiety, and managed with stress management.
incorrect because diarrhea is associated with the development of metabolic acidosis.

1.‐ “I will not take so many antacids anymore.”


2.‐ “I will take a stress management class.”
3.‐ “I will not take my Lasix without taking my potassium supplement.”
4.‐ “I will tell the doctor the next time I have diarrhea for so long.”

2222 A client is admitted with severe diarrhea. ABGs are pH Correct answer: 1 The pH and HCO&lt;sub&gt;3&lt;/sub&gt;&lt;sup&gt;‐&lt;/sup&gt; are decreased, indicating First, correlate diarrhea with a metabolic problem to eliminate options 2 and 4. Then, note
7.33; PaCO&lt;sub&gt;2&lt;/sub&gt; 42; metabolic acidosis. The PaCO&lt;sub&gt;2&lt;/sub&gt; is normal, indicating that compensatory that the pH is not within normal limits to choose option 1 over option 3.
HCO&lt;sub&gt;3&lt;/sub&gt;&lt;sup&gt;‐&lt;/sup&gt; mechanisms have not started working. Options 2 and 4 are incorrect because the primary
20. The nurse concludes this client has which of the disturbance is metabolic, as indicated by the low bicarbonate level. Option 3 is incorrect
following? because with compensation, a decrease in PaCO&lt;sub&gt;2&lt;/sub&gt; to restore balance
would be expected.
1.‐ Metabolic acidosis, uncompensated
2.‐ Respiratory acidosis, compensated
3.‐ Metabolic acidosis, compensated
4.‐ Respiratory acidosis, uncompensated
2223 An elderly client is admitted with pneumonia and Correct answer: 2 As people age, respiratory function decreases, and the alveolar exchange surfaces become Critical words are pneumonia and respiratory acidosis. Recall the physiology of changes in
respiratory acidosis. The nurse is aware that less effective in controlling respiratory acidosis. Options 1, 3, and 4 are incorrect because the elderly to be directed to option 2.
respiratory compensatory mechanisms are not as likely changes consistent with aging reflect a decrease in respiratory function and gas exchange.
to correct the acid–base balance due to:
1.‐ Increased pulmonary excretion of CO2 and increased alveolar exchange areas in the lungs.
2.‐ Decreased respiratory function and decreased alveolar exchange areas in the lungs.
3.‐ Increased pulmonary excretion of HCO3‐ and reduced alveolar exchange areas in the lungs.
4.‐ Increased pulmonary exchange of HCO3‐ and increased alveolar exchange areas in the lungs.

2224 A nurse is orienting a newly employed RN. Which of Correct answer: 1 The PaO&lt;sub&gt;2&lt;/sub&gt; is not the only determinant used to assess oxygen The critical words are lack of understanding. This is a negative‐response question,
the following statements by the new nurse indicates a saturation. Oxygen saturation reflects the total oxygen concentration that is carried on the indicating that three of the options are correct statements. Recognize that options 2, 3, and
lack of understanding about oxygen saturation? hemoglobin molecule. There is a relationship observed between the 4 are true statement about oxygenation and eliminate them.
PaO&lt;sub&gt;2&lt;/sub&gt; and SaO&lt;sub&gt;2&lt;/sub&gt; indicating safe and dangerous
levels as the PaO&lt;sub&gt;2&lt;/sub&gt; level drops. Options 2, 3, and 4 are consistent with
the concept of SaO&lt;sub&gt;2&lt;/sub&gt;.
1.‐ “Oxygen saturation is only assessed by analyzing the PaO2.”
2.‐ “The SaO2 indicates how well the blood is oxygenated.”
3.‐ “The SaO2 reflects the amount of oxygen that is bound to a hemoglobin molecule.”
4.‐ “Oxygen saturation can be affected by acid–base balance.”

2225 An elderly client admitted with a diagnosis of heart Correct answer: 3 Clients who take potassium‐wasting diuretics, such as Lasix, are at risk for developing Critical words are furosemide and potassium level of 3.2. Recognize the drug is a
failure has been taking furosemide (Lasix) for hypokalemia and metabolic alkalosis. Options 1 and 2 are incorrect because they reflect an potassium‐wasting diuretic that contributes to metabolic alkalosis. Determine that options
treatment of heart failure. Serum potassium level is acidotic state. Option 1 is a respiratory acidosis and option 2 is a respiratory acidosis with 1 and 2 reflect acidosis, and eliminate them. Option 4 is respiratory in nature, so eliminate
3.2 mEq/L. Which of the following ABG results would partial compensation. Option 4 is incorrect because the primary disturbance is a respiratory it.
the nurse expect this client to manifest? alkalosis due to the increased PaCO&lt;sub&gt;2&lt;/sub&gt; level.

1.‐ pH 7.20; PaCO2 50; HCO3‐ 22; PaO2 93


2.‐ pH 7.30; PaCO2 48; HCO3‐ 36; PaO2 90
3.‐ pH 7.49; PaCO2 36; HCO3‐ 30; PaO2 90
4.‐ pH 7.50; PaCO2 49; HCO3‐ 18; PaO2 90

2226 A nurse is caring for a client with pneumonia. Arterial Correct answer: 3 The pH is elevated, HCO&lt;sub&gt;3&lt;/sub&gt;&lt;sup&gt;‐&lt;/sup&gt; is elevated, and First, look at the pH to determine it indicates alkalosis. Then, note that carbon dioxide is
blood gas (ABG) results are pH 7.49; PaCO&lt;sub&gt;2&lt;/sub&gt; is low. This indicates that there is a mixed respiratory and low and bicarbonate is elevated, and recognize this reflects a mixed alkalosis.
PaCO&lt;sub&gt;2&lt;/sub&gt; 32 mmHg; metabolic alkalosis. Clients with pneumonia are prone to develop respiratory alkalosis. Option
HCO&lt;sub&gt;3&lt;/sub&gt;&lt;sup&gt;‐&lt;/sup&gt; 1 is incorrect because the HCO&lt;sub&gt;3&lt;/sub&gt;&lt;sup&gt;‐&lt;/sup&gt; level alone
28 mEq/L; PaO&lt;sub&gt;2&lt;/sub&gt; 89 mmHg. would be decreased. Options 2 and 4 are incorrect because the ABG values do not reflect these
This nurse analyzes these results as: conditions.

1.‐ Metabolic acidosis, uncompensated.


2.‐ Metabolic alkalosis, uncompensated.
3.‐ Mixed respiratory and metabolic alkalosis, compensated.
4.‐ Respiratory acidosis, uncompensated.
2227 The nurse expects which of the following arterial Correct answer: 2 Antacids contain a high proportion of HCO&lt;sub&gt;3&lt;/sub&gt;&lt;sup&gt;‐&lt;/sup&gt;. The critical word is antacids. Recognize that antacids contribute to metabolic alkalosis.
blood gases (ABGs) in a client who has had excessive Overuse of these agents places clients at risk for developing metabolic alkalosis due to high Eliminate option 1, since the pH indicates acidosis, and option 3, since the pH is normal.
use of antacids? concentrations of bicarbonate. Option 2 reflects metabolic alkalosis (pH alkalotic, Choose option 2, as the elevated bicarbonate level reflects a metabolic cause.
HCO&lt;sub&gt;3&lt;/sub&gt;&lt;sup&gt;‐&lt;/sup&gt; elevated, normal
PaCO&lt;sub&gt;2&lt;/sub&gt;). Option 1 reflects respiratory acidosis (pH acidotic,
HCO&lt;sub&gt;3&lt;/sub&gt;&lt;sup&gt;‐&lt;/sup&gt; normal, elevated
PaCO&lt;sub&gt;2&lt;/sub&gt;). Option 3 is incorrect because it reflects normal ABG values.
Option 4 is incorrect because it reflects respiratory alkalosis (pH alkalotic,
HCO&lt;sub&gt;3&lt;/sub&gt;&lt;sup&gt;‐&lt;/sup&gt; normal, decreased
PaCO&lt;sub&gt;2&lt;/sub&gt;).
1.‐ pH 7.30; PaCO2 48 mmHg; HCO3‐ 23 mEq/L
2.‐ pH 7.48; PaCO2 41 mmHg; HCO3‐ 30 mEq/L
3.‐ pH 7.42; PaCO2 40 mmHg; HCO3‐ 23 mEq/L
4.‐ pH 7.48; PaCO2 30 mmHg; HCO3‐ 24 mEq/L

2228 An elderly client with a history of not taking Correct answer: 1 A client with type 1 diabetes who doesn't take prescribed insulin is at risk for developing First, determine that the pH reflects acidosis, and since CO&lt;sub&gt;2&lt;/sub&gt; is
prescribed insulin is admitted to the hospital in an diabetic ketoacidosis, which leads to the formation of ketone bodies and the development of within normal limits, recognize the bicarbonate is low, and supports the cause's being
unresponsive state. Arterial blood gas (ABG) results are metabolic acidosis. Blood gas results indicate that no compensation is occurring, since the metabolic in nature.
pH 7.30; PaCO&lt;sub&gt;2&lt;/sub&gt; 40 mmHg; PaCO&lt;sub&gt;2&lt;/sub&gt; is normal, pH is acidotic, and bicarbonate level is decreased.
HCO&lt;sub&gt;3&lt;/sub&gt;&lt;sup&gt;‐&lt;/sup&gt; Option 3 is incorrect because with compensation, a rise in PaCO&lt;sub&gt;2&lt;/sub&gt;
15 mEq/L. The nurse analyzes that this client has: would be expected. Options 2 and 4 are incorrect because the ABG values are not consistent
with a respiratory disturbance, as the PaCO&lt;sub&gt;2&lt;/sub&gt; level is normal. The major
disturbance rests with the HCO&lt;sub&gt;3&lt;/sub&gt;&lt;sup&gt;‐&lt;/sup&gt; level, which
indicates a metabolic disturbance.

1.‐ Metabolic acidosis, uncompensated.


2.‐ Respiratory acidosis, uncompensated.
3.‐ Metabolic acidosis, compensated.
4.‐ Respiratory acidosis, compensated.

2229 A client is admitted with complaints of muscle Correct answer: 2 Clients with metabolic alkalosis are at risk for developing hypocalcemia. The client's Critical words aremuscle twitching and tingling around the mouth. Recognize that these
twitching, and numbness and tingling around the symptoms are consistent with hypocalcemia. Options 1, 3, and 4 are not consistent with the are classic signs of low calcium accompanying alkalosis.
mouth. Arterial blood gases (ABGs) are pH 7.49; client's presenting symptoms.
PaCO&lt;sub&gt;2&lt;/sub&gt; 38 mmHg; and
HCO&lt;sub&gt;3&lt;/sub&gt;&lt;sup&gt;‐&lt;/sup&gt;
29 mEq/L. The nurse concludes that this client should
also be assessed for which of the following?

1.‐ Hypokalemia
2.‐ Hypocalcemia
3.‐ Hyponatremia
4.‐ Hypochloremia

2230 A client had gastrointestinal (GI) surgery, and has a Correct answer: 1 NG suction removes H&lt;sup&gt; &lt;/sup&gt; ions from the stomach, and can lead to a Critical words are gastrointestinal (GI) surgery and nasogastric (NG) tube. Recall that these
nasogastric (NG) tube on low, intermittent suction. The client's developing metabolic alkalosis. Options 2 and 3 are incorrect because removal of GI both contribute to loss of body acids, predisposing the client to alkalosis.
nurse plans to assess the client for which of the drainage leads to acid loss and bicarbonate excess. Option 4 is incorrect because respiratory
following acid–base imbalances? alkalosis is more likely to occur with hypoxic states, central nervous system disorders, drug
use, or hyperdynamic states.
1.‐ Metabolic alkalosis
2.‐ Metabolic acidosis
3.‐ Respiratory acidosis
4.‐ Respiratory alkalosis
2231 The nurse carefully assesses a client with a new Correct answer: 2 Lower gastrointestinal (GI) tract fluid loss leads to a loss of The critical word is ileostomy. Recognize that intestinal fluids are alkaline, and that these
ileostomy for development of which of the following HCO&lt;sub&gt;3&lt;/sub&gt;&lt;sup&gt;‐&lt;/sup&gt;, resulting in the development of are lost via the ileostomy, leading to metabolic acidosis.
acid–base imbalances? metabolic acidosis. Options 3 and 4 are incorrect because a client with an ileostomy does not
retain bicarbonate. Option 1 is incorrect because respiratory acidosis involves a primary
disturbance in CO&lt;sub&gt;2&lt;/sub&gt; levels.
1.‐ Respiratory acidosis
2.‐ Metabolic acidosis
3.‐ Metabolic alkalosis
4.‐ Respiratory alkalosis

2232 When caring for a client with chronic obstructive Correct answer: 2 Due to long‐term lung disease, clients with COPD tend to develop respiratory acidosis The critical term is COPD. Recall that CO&lt;sub&gt;2&lt;/sub&gt; is retained with this
pulmonary disease (COPD), the nurse anticipates which because they compensate and adjust to a higher level of CO&lt;sub&gt;2&lt;/sub&gt;. The pH condition, contributing to acidosis. Look for the pH to reflect acidosis and
of the following ABG results? reflects acidemia,and there is an elevated PaCO&lt;sub&gt;2&lt;/sub&gt; and normal CO&lt;sub&gt;2&lt;/sub&gt; to be elevated.
HCO&lt;sub&gt;3&lt;/sub&gt;&lt;sup&gt;‐&lt;/sup&gt;. Option 1 is incorrect because the pH is
acidotic and there is a normal CO&lt;sub&gt;2&lt;/sub&gt; and decreased
HCO&lt;sub&gt;3&lt;/sub&gt;&lt;sup&gt;‐&lt;/sup&gt;. Option 3 is incorrect because the pH is
normal, but CO&lt;sub&gt;2&lt;/sub&gt; is increased and there is a normal
HCO&lt;sub&gt;3&lt;/sub&gt;&lt;sup&gt;‐&lt;/sup&gt;. Option 4 is incorrect because the pH is
alkalotic, and there are decreased CO&lt;sub&gt;2&lt;/sub&gt; and normal
HCO&lt;sub&gt;3&lt;/sub&gt;&lt;sup&gt;‐&lt;/sup&gt;.

1.‐ pH 7.30; PaCO2 38; HCO3‐ 18


2.‐ pH 7.32; PaCO2 48; HCO3‐ 23
3.‐ pH 7.42; PaCO2 47; HCO3‐ 25
4.‐ pH 7.55; PaCO2 30; HCO3‐ 22

2233 A client with COPD is admitted to the hospital with an Correct answer: 1 A pH of 7.30 indicates acidosis. A PaCO&lt;sub&gt;2&lt;/sub&gt; of 51 indicates a respiratory First, determine that the pH indicates acidosis, and then examine
exacerbation of the disease. Arterial blood gas (ABG) acidosis is occurring. Since the PaCO&lt;sub&gt;2&lt;/sub&gt; is elevated with a normal CO&lt;sub&gt;2&lt;/sub&gt; to determine that the cause is respiratory. Since bicarbonate is
results are pH 7.30; PaCO&lt;sub&gt;2&lt;/sub&gt; 51; HCO&lt;sub&gt;3&lt;/sub&gt;&lt;sup&gt;‐&lt;/sup&gt;, an uncompensated respiratory acidosis normal, choose option 1.
HCO&lt;sub&gt;3&lt;/sub&gt;&lt;sup&gt;‐&lt;/sup&gt; is occurring. Options 2 and 4 are incorrect because the pH value does not indicate that
25. How would the nurse interpret these? alkalosis could be present. Option 3 is incorrect because the bicarbonate level is normal,
indicating that compensation has not taken place. With compensation, you would expect an
increase in the bicarbonate level.

1.‐ Respiratory acidosis, uncompensated


2.‐ Respiratory alkalosis, partially compensated
3.‐ Respiratory acidosis, compensated
4.‐ Metabolic acidosis, compensated

2234 A client admitted to the Emergency Department with Correct answer: 2 A client with a chest injury is likely to hypoventilate (have a shallow respiratory pattern) as a Before looking at the options, recognize that the client is at greatest risk for respiratory
chest injuries following a motor vehicle accident result of pain due to associated trauma. It is unknown at this time whether there are any acidosis. Eliminate options 1 and 4, since pH is normal, and eliminate option 3, since pH
complains that it hurts to breathe. The client s internal injuries that could affect the client s oxygen saturation. This type of respiratory reflects alkalosis.
respiratory rate is 12 and very shallow. The nurse pattern is associated with respiratory acidosis. Options 1 and 4 reflect normal lab values.
would anticipate which of the following results on Option 3 reflects metabolic alkalosis (increased pH and
arterial blood gases (ABGs)? HCO&lt;sub&gt;3&lt;/sub&gt;&lt;sup&gt;‐&lt;/sup&gt;, decreased
pCO&lt;sub&gt;2&lt;/sub&gt; and SaO&lt;sub&gt;2&lt;/sub&gt;).
1.‐ pH 7.42; PaCO2 41 mmHg; HCO3‐ 23 mEq/L; SaO2 96%
2.‐ pH 7.31; PaCO2 49 mmHg; HCO3‐ 24 mEq/L; SaO2 87%
3.‐ pH 7.49; PaCO2 34 mmHg; HCO3‐ 30 mEq/L; SaO2 89%
4.‐ pH 7.38; PaCO2 38 mmHg; HCO3‐ 22 mEq/L; SaO2 90%
2235 What action should the nurse take initially to avoid Correct answer: 3 One of the first things a nurse should do when a client is hyperventilating is to give the client Critical words are “initially” and “hyperventilate.” Recognize that some of the options are
acid–base imbalance when a client becomes anxious a paper bag into which to breathe. This prevents the CO&lt;sub&gt;2&lt;/sub&gt; level from partially correct, but choose option 3, since this offers an option readily available that can
and starts to hyperventilate? decreasing, and rebreathing the gas in the bag will also help to decrease the respiratory rate. be tried before the other options might be needed.
Hyperventilation is associated with respiratory alkalosis. Although sedatives might be indicated
to decrease anxiety and hyperventilation, the use of a paper bag might stop the breathing
pattern response by redirecting the client s focus. Telling the client that he might pass out
could make him change the breathing pattern, but it could also make it worse by increasing
anxiety. Notifying the physician might be indicated to further assist in client treatment, but the
initial response should be to interrupt the present breathing pattern of the client.

1.‐ Tell the client to stop breathing so fast because he could pass out.
2.‐ Give the client a sedative to decrease anxiety and stop hyperventilation.
3.‐ Give the client a paper bag into which to breathe.
4.‐ Notify the physician.

2236 The nurse would closely monitor a client with diabetic Correct answer: 1 DKA is associated with an increase in acid production. Diabetic clients with DKA are unable to Note that ‐acidosis is part of the word ketoacidosis. Recall that diabetes is a metabolic
ketoacidosis (DKA) for which of following primary metabolize glucose, and the liver responds with an increase in fatty acid metabolism. These disorder to choose option 1.
acid–base imbalances? fatty acids are oxidized, leading to ketone body formation and increased acidity. Option 2 is
incorrect because metabolic alkalosis is the opposite imbalance. Option 3 is unrelated, and
option 4 would occur as a secondary compensatory mechanism for the client in DKA.

1.‐ Metabolic acidosis


2.‐ Metabolic alkalosis
3.‐ Respiratory acidosis
4.‐ Respiratory alkalosis

2237 A 36‐year‐old female is admitted with vomiting and Correct answer: 3 The pH indicates alkalosis, and the HCO&lt;sub&gt;3&lt;/sub&gt;&lt;sup&gt;‐&lt;/sup&gt; is First, determine that the pH reflects slight alkalosis. Note the increased bicarbonate level
dehydration after having the flu for three days. Arterial elevated, indicating a metabolic basis. The PaCO&lt;sub&gt;2&lt;/sub&gt; is slightly elevated, reflects that the cause is metabolic, and since CO&lt;sub&gt;2&lt;/sub&gt; is elevated, the
blood gas (ABG) results are pH 7.46; indicating that compensation is occurring. Options 1 and 2 are incorrect because the client s pH is partially compensated.
PaCO&lt;sub&gt;2&lt;/sub&gt; 50; pH reflects alkalosis. Option 4 is incorrect because compensation is occurring due to the
HCO&lt;sub&gt;3&lt;/sub&gt;&lt;sup&gt;‐&lt;/sup&gt; increased CO&lt;sub&gt;2&lt;/sub&gt; level.
33; SaO&lt;sub&gt;2&lt;/sub&gt; 95%. What do these
values indicate to the nurse?
1.‐ Metabolic acidosis, uncompensated
2.‐ Respiratory acidosis, compensated
3.‐ Metabolic alkalosis, partially compensated
4.‐ Metabolic alkalosis, uncompensated

2238 A client in a full cardiac arrest is admitted to the Correct answer: 3 During a cardiac arrest, the client develops profound respiratory acidosis, and needs to be Note that the client is in full cardiac arrest, necessitating initiation of ABCs airway,
Emergency Department. Arterial blood gases (ABGs) ventilated, first with a bag‐valve‐mask device and then by mechanical means once intubation is breathing, and circulation to direct you to option 3.
indicate a respiratory acidosis. How does the nurse accomplished. Other interventions will be instituted during the course of the code, but the
respond to correct this condition? nurse should always respond to any emergency situation with the ABCs (airway, breathing, and
circulation).
1.‐ Administer NaHCO3 to correct the acidosis.
2.‐ Administer epinephrine to get a heart rate so the acidosis can be corrected.
3.‐ Ventilate client to “blow off” excess CO2.
4.‐ Start cardiac compressions.

2239 The nurse identifies which of the following clients to Correct answer: 1, 4 Loss of acidic contents via NG drainage can lead to alkalosis and intake of antacids, which are Recall conditions that contribute to a loss of acidic body fluids and gain of alkaline
be at risk for developing metabolic alkalosis? Select all frequently alkaline substances. Diarrhea leads to a loss of alkalotic fluids, predisposing the substances to direct you to options 1 and 4.
that apply. client to acidosis. Salicylate toxicity results in acidosis. The client with asthmatic bronchitis
retains carbon dioxide, leading to respiratory acidosis.
1.‐ Has a nasogastric tube (NG) on continuous suction.
2.‐ Has had diarrhea for two days.
3.‐ Admitted with a salicylate toxicity.
4.‐ Takes antacids frequently for heartburn.
5.‐ Admitted with asthmatic bronchitis.

2240 The arterial blood gas (ABG) results of a 68‐year‐old Correct answer: 2 The slightly elevated pH (alkalosis), the low PaCO&lt;sub&gt;2&lt;/sub&gt; (respiratory First, determine that the pH is alkalotic. Then, recognize that the
client admitted with pneumonia are pH 7.46; origin), and the low HCO&lt;sub&gt;3&lt;/sub&gt;&lt;sup&gt;‐&lt;/sup&gt; indicate CO&lt;sub&gt;2&lt;/sub&gt; is low to determine that the cause is respiratory. Choose
PaCO&lt;sub&gt;2&lt;/sub&gt; 30; compensation is starting but is not yet fully complete, since the pH is still abnormal. In option, 2 since the bicarbonate level reflects some compensation.
HCO&lt;sub&gt;3&lt;/sub&gt;&lt;sup&gt;‐&lt;/sup&gt; addition, the SaO&lt;sub&gt;2&lt;/sub&gt; level is decreased significantly, which is not
19; SaO&lt;sub&gt;2&lt;/sub&gt; 72. The nurse consistent with aging alone. Option 1 is incorrect because the pH is alkalotic. Option 3 is
interprets this as: incorrect because in an uncompensated respiratory alkalosis, the bicarbonate level would be
normal. Option 4 is incorrect because the bicarbonate level is not elevated.

1.‐ Respiratory acidosis, uncompensated.


2.‐ Respiratory alkalosis, partially compensated.
3.‐ Respiratory alkalosis, uncompensated.
4.‐ Metabolic alkalosis, partially compensated.

2241 A 71‐year‐old client develops hypertension, Correct answer: 1 The pH is low (acidosis) and PaCO&lt;sub&gt;2&lt;/sub&gt; high (respiratory origin). The First, determine that the pH level indicates acidosis. Next, analyze the
tachycardia, and increased respirations two days after HCO&lt;sub&gt;3&lt;/sub&gt;&lt;sup&gt;‐&lt;/sup&gt; is normal, indicating that compensation CO&lt;sub&gt;2&lt;/sub&gt; to determine that the cause is respiratory. Choose option 1,
surgery. Arterial blood gas (ABG) results are pH 7.29; has not occurred. The client is experiencing hyperventilation, but blood gases reveal a since the pH is not normalized by the bicarbonate.
PaCO&lt;sub&gt;2&lt;/sub&gt; 52; respiratory acidosis, probably due to prior hypoventilation. Option 2 is incorrect because the
HCO&lt;sub&gt;3&lt;/sub&gt;&lt;sup&gt;‐&lt;/sup&gt; bicarbonate level has not increased in an attempt to restore balance. Options 3 and 4 are
24; SaO&lt;sub&gt;2&lt;/sub&gt; 95%. The nurse incorrect because the bicarbonate levels are within normal limits and yet the
interprets that these results indicate which of the PaCO&lt;sub&gt;2&lt;/sub&gt; level is still elevated.
following?
1.‐ Respiratory acidosis, uncompensated
2.‐ Respiratory acidosis, partially compensated
3.‐ Metabolic acidosis, uncompensated
4.‐ Metabolic acidosis, partially compensated

2242 A 57‐year‐old client is admitted with a diagnosis of Correct answer: 3 The pH is normal (but is nearer to the acidotic end), while the PaCO&lt;sub&gt;2&lt;/sub&gt; This question requires you to identify ABG results and quality of oxygenation. Since the
acute myocardial infarction. Arterial blood gas (ABG) is low (compensation has occurred) and the HCO&lt;sub&gt;3&lt;/sub&gt;&lt;sup&gt;‐ oxygen level is 100%, eliminate options 2 and 4. Since pH is within normal limits, choose
results are pH 7.36; PaCO&lt;sub&gt;2&lt;/sub&gt; 29; &lt;/sup&gt; is low (indicating metabolic origin). The oxygen saturation of 100% indicates the option 3.
HCO&lt;sub&gt;3&lt;/sub&gt;&lt;sup&gt;‐&lt;/sup&gt; blood is well oxygenated, making options 2 and 4 incorrect, because the client is not
20; SaO&lt;sub&gt;2&lt;/sub&gt; 100%. The nurse hypoxemic. Since the pH is within normal limits, it is more likely that there are mixed
interprets that this client is: acid–base disorders occurring that are compensating each other. Since the
PaCO&lt;sub&gt;2&lt;/sub&gt; and HCO&lt;sub&gt;3&lt;/sub&gt;&lt;sup&gt;‐&lt;/sup&gt; are
low, metabolic acidosis is occurring with a respiratory alkalosis.

1.‐ Well‐oxygenated, with uncompensated respiratory alkalosis.


2.‐ Hypoxemic, with compensated respiratory acidosis.
3.‐ Well‐oxygenated, with compensated metabolic acidosis.
4.‐ Hypoxemic, with compensated metabolic acidosis.
2243 Arterial blood gases on a client with pneumonia Correct answer: 3, 4 The respiratory acidosis in this client is secondary to retention of carbon dioxide. Coughing Recall that causes of respiratory acidosis are related to retention of carbon dioxide.
indicate the client is in respiratory acidosis. In order to and deep breathing will stimulate expectoration of secretions, allowing for improved gas Determine that option 1 provides measures to best promote improved gas exchange in the
best improve this acid–base imbalance, the nurse exchange. Option 1 is incorrect; fluids will help to liquefy secretions, and do not need to be lungs.
implements which of the following nursing restricted to water. It would be helpful to ambulate the client (option 2), which will promote
interventions? Select all that apply. lung expansion, but not as much as will option 3. Option 4 is correct; medicating the client
frequently with narcotics might decrease respiratory drive, but a nonnarcotic medication
might enable the client to breathe deeply. Option 5, magnesium, has no effect on acid–base.

1.‐ Restrict oral fluid intake to water only.


2.‐ Ambulate the client in hallways twice a shift.
3.‐ Encourage frequent cough and deep‐breathing exercises.
4.‐ Medicate with a nonopiate pain medication frequently for intercostal muscle pain.
5.‐ Give magnesium.

2244 A client receiving intravenous (IV) sodium bicarbonate Correct answer: 1 Symptoms of alkalosis include irritability, confusion, cyanosis, irregular pulse, slow The critical word is initially, indicating all or some of the options are correct, but one takes
for treatment of metabolic acidosis develops muscle respirations, and muscle twitching. These symptoms warrant discontinuing the medications first priority. Recognize that the client is experiencing metabolic alkalosis, and choose
twitching and an irregular pulse. Which actions should and notifying the primary health care provider, since the client might have received excessive option 1 as having highest priority.
be taken by the nurse initially? sodium bicarbonate. Option 2 is incorrect. Options 3 and 4 would be carried out after the
physician has been notified.
1.‐ Stop the infusion and notify the physician.
2.‐ Reduce the infusion by one‐half of the ordered rate and observe client closely.
3.‐ Monitor heart rate, blood pressure, and mentation every 15 minutes.
4.‐ Check pulse oximetry and place the client on bedrest.

2245 Which of the following arterial blood gases (ABGs) Correct answer: 2 Apnea and hypoventilation result in rising carbon dioxide levels, which lead to acidosis. The Critical words are apnea and acidosis, indicating the cause will be respiratory in nature.
would the nurse expect to see when a client has apnea ABG would likely reflect respiratory acidosis without compensation. Option 1 is incorrect Look for the ABG with a pH indicating acidosis to direct you to option 2.
and develops acidosis? because it reflects a normal pH, a slight increase in PaCO&lt;sub&gt;2&lt;/sub&gt;, and a
normal HCO&lt;sub&gt;3&lt;/sub&gt;&lt;sup&gt;‐&lt;/sup&gt; level. Option 3 is incorrect
because it reflects normal values for all three parameters. Option 4 reflects an alkalotic state
because the pH and HCO&lt;sub&gt;3&lt;/sub&gt;&lt;sup&gt;‐&lt;/sup&gt; are elevated, and
the PaCO&lt;sub&gt;2&lt;/sub&gt; is decreased.

1.‐ pH 7.42; PaCO2 48 mmHg; HCO3‐ 25 mEq/L


2.‐ pH 7.29; PaCO2 62 mmHg, HCO3‐ 23 mEq/L
3.‐ pH 7.36 PaCO2 42 mmHg HCO3‐ 26 mEq/L
4.‐ pH 7.49; PaCO2 30 mmHg; HCO3‐ 35 mEq/L

2246 The nurse concludes that which of the following Correct answer: 2 The kidneys respond more slowly to acid–base imbalances than do the lungs, but are more Note that some of the options are only partially correct to eliminate options 1 and 4.
statements by a student nurse reflects correct effective in restoring acid–base balance to the extracellular fluid. The primary response to Recall the role of the kidney in maintaining acid–base balance to choose option 2.
understanding about the body’s attempt to restore acidosis is with lung compensation. Option 1 is incorrect because the kidneys do not respond
homeostasis during periods of acidosis? immediately to correct acid–base imbalances. Option 3 is incorrect because the kidneys utilize
several mechanisms to restore acid–base balance, involving phosphate buffer salts,
reabsorption of bicarbonate, and excretion of ammonia. The bicarbonate buffer system is a
very strong buffer system in the body, and also helps to regulate the respiratory response to
acid–base balance. Option 4 is incorrect because the kidneys help to restore acid–base balance
by reabsorbing bicarbonate by ionizing carbonic acid. Ion exchange occurs between
Na&lt;sup&gt; &lt;/sup&gt; and H&lt;sup&gt; &lt;/sup&gt;, which leads to formation of
bicarbonate, which is then absorbed into the blood.

1.‐ “The kidneys start to work within seconds after an imbalance occurs, and are very effective in restoring the body to a correct acid–base balance.”
2.‐ “The kidneys might not start to function immediately, but are very effective as a buffer system to restore the acid–base balance.”
3.‐ “The kidneys are not as effective as the lungs in restoring the acid–base balance because the bicarbonate ion is not a good buffer.”
4.‐ “The kidneys are very slow to respond to any acid–base imbalance, but are very effective in ridding the body of carbonic acid.”
2247 Which of the following ABG results would the nurse Correct answer: 2 Diarrhea leads to loss of bicarbonate from the intestinal tract. This can cause metabolic The critical word is diarrhea. Recall that this leads to a loss of alkaline fluids, which will
expect to see when a client is admitted with diarrhea acidosis. With metabolic acidosis, the pH is low and the cause acidosis, to direct you to option 2.
that has lasted for four days? HCO&lt;sub&gt;3&lt;/sub&gt;&lt;sup&gt;‐&lt;/sup&gt; is also decreased. Option 1 is incorrect
because the pH is alkalotic, the PaCO&lt;sub&gt;2&lt;/sub&gt; is elevated, and the
HCO&lt;sub&gt;3&lt;/sub&gt;&lt;sup&gt;‐&lt;/sup&gt; is slightly elevated. These values reflect
metabolic alkalosis. Option 3 is incorrect because the pH and PaCO&lt;sub&gt;2&lt;/sub&gt;
are within normal limits, and the HCO&lt;sub&gt;3&lt;/sub&gt;&lt;sup&gt;‐&lt;/sup&gt; is
slightly elevated. These values do not reflect a cause for concern at this point in time. Option 4
is incorrect because the pH is alkalotic, the PaCO&lt;sub&gt;2&lt;/sub&gt; is normal, and the
HCO&lt;sub&gt;3&lt;/sub&gt;&lt;sup&gt;‐&lt;/sup&gt; is slightly elevated. These results reflect
metabolic alkalosis.

1.‐ pH 7.50; PaCO2 60 mmHg; HCO3‐ 28 mEq/L


2.‐ pH 7.30; PaCO2 40 mmHg; HCO3‐ 18 mEq/L
3.‐ pH 7.40; PaCO2 &lt; 38 mmHg; HCO3‐ 28 mEq/L
4.‐ pH 7.50; PaCO2 38 mmHg; HCO3‐ 32 mEq/L

2248 The nurse has been caring for a client who has Correct answer: 2 Clients who are extremely anxious tend to hyperventilate and have a rapid, shallow Critical words are anxious and agitated. Visualize this client in considering options.
become extremely anxious and agitated. When respiratory pattern. Cardiac rhythm and regulation are independent of respiratory function, Eliminate options 3 and 4, since they are related to pulses and not breathing. Recall
assessing the client, the nurse would expect to find and the rate can vary depending on the client s medical condition and/or treatment. A rapid breathing patterns seen in anxious clients to choose option 2.
which of the following that could ultimately lead to an deep respiratory pattern is associated with further respiratory compromise.
acid–base imbalance?
1.‐ Rapid, deep respiratory pattern
2.‐ Rapid, shallow respiratory pattern
3.‐ Rapid, irregular heart rate
4.‐ Slow, irregular heart rate

2249 Which of the following pH values would the nurse Correct answer: 3 Anxious clients hyperventilate, which leads to alkalosis because of a depletion of carbon The critical word is anxious. Recall that anxiety will cause an increase in respiratory rate
expect to see in an anxious client? dioxide. Options 1 and 2 reflect normal pH values. Option 4 reflects acidemia, and is associated with loss of carbon dioxide, resulting in alkalosis.
with clients who hypoventilate.
1.‐ 7.45
2.‐ 7.38
3.‐ 7.50
4.‐ 7.20

2250 The nurse anticipates which of the following Correct answer: 3 A client with metabolic acidosis will have an increase in respiratory rate and depth, in an Recall that the respiratory system will try to compensate acidosis by blowing off carbon
responses in a client who develops metabolic acidosis? attempt to compensate for the acidosis. Increases in heart rate, temperature, and urinary dioxide and water to direct you to option 3.
output are all metabolic responses that are not directly associated with maintaining acid–base
balance. Initial compensation with metabolic acidosis will be via the lungs.

1.‐ Heart rate will increase.


2.‐ Urinary output will increase.
3.‐ Respiratory rate will increase.
4.‐ Temperature will increase.

2251 The nurse assesses a client with uncontrolled type 1 Correct answer: 4 Due to a lack of insulin, diabetic clients are more likely to use fats as an energy source. During Recognize that this condition leads to ketoacidosis with retention of carbonic acid to direct
diabetes mellitus for which of the following acid–base the metabolism of fats, free fatty acids are released, leading to accumulation of fatty acid you to option 4.
imbalances? fragments and the development of diabetic ketoacidosis. Diabetic clients are likely to develop
metabolic acidosis characterized by decreased pH and
HCO&lt;sub&gt;3&lt;/sub&gt;&lt;sup&gt;‐&lt;/sup&gt; levels.
1.‐ Metabolic alkalosis
2.‐ Respiratory alkalosis
3.‐ Respiratory acidosis
4.‐ Metabolic acidosis

2252 The client with respiratory acidosis from COPD asks Correct answer: 2 The pulse oximeter measures the amount of oxygen in the blood, and is a good indication of Critical words are COPD and pulse oximeter. Recall the function and purpose of the latter
the nurse why a continuous pulse oximeter is ordered. oxygenation status. It is not meant to replace needed ABG monitoring, but rather is used in to choose option 2.
Which of the following responses by the nurse conjunction with appropriate respiratory assessment to provide important information on a
provides the best response? continuous basis. Pulse oximetry does not determine or reflect ventilatory effort, regardless of
client positioning.
1.‐ “The pulse oximeter measures your CO2 level so ABGs only need to be drawn once a day.”
2.‐ “The pulse oximeter measures the oxygen saturation in your blood at any given time.”
3.‐ “The pulse oximeter is being used so we don’t ever have to draw ABGs on you while you are in the hospital.”
4.‐ “The machine is used to adequately assess your ventilatory effort while you are in bed.”

2253 A 10‐month‐old infant is admitted to the emergency Correct answer: 2 The client’s history suggests fluid volume deficit and dehydration. Sunken eyes, altered The core issue of the question is the ability to correlate a clinical picture with risk for
department with a 102°F rectal temperature and a mental status and behavior, and dry, furrowed tongue are reliable signs of fluid volume deficit hypovolemia. Use nursing knowledge of signs of dehydration and the process of elimination
history of vomiting and diarrhea for the past 48 hours. in infants. Bulging fontanels, peripheral edema, and neck vein distention are seen with fluid to make a selection.
For what signs and symptoms should the nurse look volume excess.
related to this client s likely fluid imbalance?

1.‐ Bulging fontanels, tearless cry, and low urine output


2.‐ Sunken eyes, lethargy, and dry, furrowed tongue
3.‐ Weight loss, dilute urine, and peripheral edema
4.‐ Dry skin, thready pulse, and neck vein distention

2254 Which observation by the nurse is a reliable indicator Correct answer: 2 Venous congestion results from fluid volume excess, and causes full, bounding pulses, delayed The core issue of the question is knowledge of signs of fluid overload and normal findings.
that therapy for fluid volume excess is achieving the hand vein emptying, and S&lt;sub&gt;3&lt;/sub&gt; heart sounds. Flat neck veins with the Use nursing knowledge and the process of elimination to make a selection.
desired outcome? head of the bed elevated are an indicator of the absence of venous congestion.

1.‐ Full, bounding peripheral pulses


2.‐ Flat neck veins with the head of the bed elevated
3.‐ Hand vein emptying longer than 20 seconds
4.‐ S3 heart sound clearly audible on auscultation

2255 The nurse concludes that which of the following is a Correct answer: 2 Ascites is a form of third space fluid. Therapy is aimed at moving third space fluid back into The core issues of the question are recognition of ascites as a third space fluid and
reliable sign that ascites are being effectively mobilized the circulation, where it can be eliminated by the kidneys. When this fluid is drawn back into knowledge of effective mobilization of that fluid. Recall that mobilized fluid must be
in response to therapy? the vascular space (leading to a rise in BP and venous pressure), the kidneys increase the urine eliminated via the kidneys to assist in making a selection.
output to eliminate the excess fluid. Loss of fluid results in loss of weight.

1.‐ Weight gain of 1 pound in 24 hours


2.‐ Increase in urine output
3.‐ Drop in blood pressure
4.‐ Hand veins fill slowly

2256 Which of the following should be included in an Correct answer: 1 Those who exercise in hot climates need to continuously replace both fluid and electrolyte The core issue of the question is knowledge of measures to prevent fluid and electrolyte
education program to prevent dehydration for a hiking losses. Sports drinks provide carbohydrates, water, and electrolytes. Drinking large amounts of imbalance during exercise. Use nursing knowledge and the process of elimination to make a
club that is planning a 12‐mile hike in the summer? only water fails to replace electrolytes, which can lead to water intoxication. Salt tablets are no selection.
longer recommended, because too much salt has a hypertonic effect, causes diuresis, and can
actually worsen fluid loss.
1.‐ Take water and commercial sports drinks to sip often along the way.
2.‐ Drink large amounts of water, at least 16 ounces every hour, while hiking.
3.‐ Take salt tablets every 3–4 hours, and drink plenty of water while in the heat.
4.‐ Stop every 4 hours along the way, and drink a few ounces of water while resting.
2257 Which one of the following postoperative clients Correct answer: 3 A TURP procedure can place a client at risk for developing hyponatremia in the postoperative The core issue of the question is knowledge that procedures and surgeries requiring the
would be at risk for developing a sodium imbalance? period due to increased fluid irrigation used during and after surgery. Clients with a TURP use of water for irrigation can lead to dilutional hyponatremia. Use nursing knowledge and
procedure have a CBI (continuous bladder irrigation) as a routine part of their postoperative the process of elimination to make a selection.
care. The other options do not place a client at risk for development of sodium imbalances,
because they do not require lengthy fluid and dietary restrictions, or excessive fluid irrigation.

1.‐ A client who has just had a tonsillectomy


2.‐ A client who has a primary cesarean section for failure to progress in labor
3.‐ A client who has a transurethral resection of the prostate (TURP)
4.‐ A client who has a right knee arthroscopy

2258 The nurse is caring for a client experiencing Correct answer: 4 Hyponatremia can also be referred to as dilutional hyponatremia or water intoxication. Water The core issue of the question is effective treatment measures for hyponatremia. Use
hyponatremia. As part of the care, the nurse will restriction would be an important part of the treatment plan when caring for a client who has nursing knowledge and the process of elimination to make a selection.
restrict which of the following items for this client? hyponatremia. The restriction of Gatorade (electrolyte‐rich solution), eggs, cheese products,
and salt on the diet tray are not indicated, because the client is experiencing a sodium deficit.

1.‐ Sports drinks, such as Gatorade


2.‐ Eggs and cheese products
3.‐ Salt on the diet tray
4.‐ Water

2259 The nurse is caring for a client experiencing Correct answer: 3 Clients with hypernatremia are thirsty, and need water replacement to balance their The core issue of the question is knowledge of measures that effectively treat
hypernatremia. The nurse concludes that it is increased sodium levels. Cough medication and lactulose can further increase sodium levels, hypernatremia. Use nursing knowledge and the process of elimination to make a selection.
important to administer which of the following to this and should not be administered unless there is sufficient clinical information to warrant their
client? use. Three‐percent saline is a hypertonic solution that would also increase serum sodium
levels, and should not be given to this client.
1.‐ Cough suppressant to treat symptomatic cough
2.‐ Three‐percent saline solution
3.‐ Water
4.‐ Lactulose (Chronulac)

2260 The community health nurse is assigned to a client Correct answer: 4 The frequent use of Alka‐Seltzer can cause an increase in serum sodium levels. It is important The core issue of the question is knowledge of factors that can lead to elevated serum
who recently was discharged from the hospital with during an initial assessment to obtain information about all medications (prescription and OTC) sodium levels. Use nursing knowledge and the process of elimination to make a selection.
resolving hypernatremia. During the initial assessment that a client is taking. Options 1 and 2 are incorrect because they do not relate to potential
interview, what information would be of critical sodium imbalance. They are helpful in determining the client s support system and mobility
importance in determining a plan of care for this status. Option 3 suggests that the client might have diabetes, but this does not relate to
client? increases in serum sodium levels.
1.‐ The client lives on the second floor of an apartment building that has an elevator.
2.‐ The client has a neighbor who picks up the mail each day and brings it to the apartment.
3.‐ The client performs self‐monitoring of blood glucose once a day.
4.‐ The client uses Alka‐Seltzer on a frequent basis for gastrointestinal complaints.

2261 The nurse is caring for a client who has sustained Correct answer: 1 During periods of major trauma, potassium shifts from the ICF to the ECF because of cell The core issue of the question is knowledge that burn injury increases the risk of
second‐ and third‐degree burns over 30% of his body. death, leading to high serum levels of potassium. Hypokalemia is not seen in burn clients hyperkalemia. Use nursing knowledge and the process of elimination to make a selection.
The nurse assesses for which of the following during the time of fluid shifting secondary to trauma. The client with burns is more likely to be
electrolyte imbalances, which occurs as electrolyte hypovolemic and hypocalcemic at this point in time because there are fluid and electrolyte loss
shifts from the intracellular fluid (ICF) to the caused by altered capillary integrity.
extracellular fluid (ECF)?
1.‐ Hyperkalemia
2.‐ Hypokalemia
3.‐ Hypervolemia
4.‐ Hypercalcemia

2262 The nurse concludes that a history of which of the Correct answer: 2 In clients with cirrhosis, increased amounts of aldosterone are secreted, which leads to The core issue of the question is the ability to discriminate predisposing factors for
following conditions places a client at risk for possible sodium retention and potassium excretion from the kidneys; these clients are likely to become hypokalemia from factors for hyperkalemia. Use nursing knowledge and the process of
hypokalemia? hypokalemic. Clients with COPD, malignant melanoma, and CRF are likely to develop elimination to make a selection.
hyperkalemia due to retention of acids.
1.‐ Chronic obstructive pulmonary disease (COPD)
2.‐ Cirrhosis
3.‐ Malignant melanoma
4.‐ Chronic renal failure (CRF)

2263 Which of the following orders should the nurse Correct answer: 4 Potassium is never given as a bolus when it is administered intravenously. All of the other The core issue of the question is knowledge of safe and unsafe methods of administering
question regarding a client with severe hypokalemia? orders are within a safe and therapeutic range. KCl should never be given rapidly or by IV push, potassium as replacement therapy. Use nursing knowledge and the process of elimination
because serious arrhythmias or cardiac arrest can occur. to make a selection.
1.‐ Infuse 1,000 mL normal saline with 20 mEq potassium chloride IV over 8 hours.
2.‐ Give KCl 20 mEq PO daily after meals.
3.‐ Infuse 1,000 mL normal saline with 40 mEq KCl at 200 mL/hour.
4.‐ Give 20 mEq KCl/IV over 10 minutes.

2264 Which of the following treatment options does the Correct answer: 4 A serum potassium level of 3.5 mEq/L is at the low end of the normal range. With a low The core issue of the question is knowledge of treatment measures depending on the
nurse anticipate will be appropriate for a client with a normal level, it is better to continue to monitor the client and offer foods that are good severity of hypokalemia. First, recognize that this is a value at the low end of normal, and
potassium level of 3.5 mEq/L? sources of potassium. In the absence of additional medical history, it is not advisable to use then select the mildest intervention of the choices provided.
additional treatment options at this point in time. Therefore, options 1 and 3 would not be
indicated: they would be included for a client who is hypokalemic. The use of salt substitutes
would require more background information, because the client might have other conditions
for which their use is not advisable.

1.‐ Administration of Kayexalate per rectum


2.‐ Use of salt substitutes in the diet
3.‐ Administration of oral KCl
4.‐ Continue to monitor and offer foods high in potassium.

2265 Which one of the following assessments should be Correct answer: 3 A client who is at risk for developing hypocalcemia requires monitoring of serum albumin The core issue of the question is the ability to choose assessments to detect hypocalcemia.
included in a plan of care for a client who is at risk for (provides information relative to physiologically available calcium) and magnesium levels. Use nursing knowledge and the process of elimination to make a selection.
developing hypocalcemia? (Decreased magnesium levels are usually seen concurrently with low serum calcium levels.)
The other options reflect assessments that would be included for a client who would be at risk
to develop hypercalcemia.
1.‐ Monitor BUN and creatinine levels to determine renal dysfunction.
2.‐ Monitor the client for constipation.
3.‐ Monitor serum albumin and magnesium levels.
4.‐ Monitor for fluid volume excess related to intravenous saline therapy.

2266 A client with hypocalcemia is taking supplemental Correct answer: 3 Calcium is absorbed in the intestines only under the influence of vitamin D, which is activated The core issue of the question is knowledge of the purpose and effects of vitamin D in a
vitamin D. The nurse explains the rationale for vitamin in the kidneys. Option 1 is incorrect because parathyroid hormone directly opposes calcitonin. client with hypocalcemia. Use nursing knowledge and the process of elimination to make a
D is that: Option 2 is incorrect because renal disease prevents activation of vitamin D, thereby reducing selection.
the body’s ability to absorb calcium. Option 4 is incorrect: there are other ways to obtain
vitamin D in the body (such exposure to sunlight).

1.‐ It directly opposes calcitonin.


2.‐ It prevents renal disease in clients with hypocalcemia.
3.‐ Calcium is absorbed in the intestines only under the influence of activated vitamin D.
4.‐ The only way to obtain vitamin D is with oral supplementation.
2267 Which of the following medications reported by a Correct answer: 1 Anticonvulsants such as phenytoin (Dilantin) alter vitamin D metabolism and lead to The core issue of the question is knowledge of medications that increase the risk of
client during a nursing history could be associated with hypocalcemia. Options 2 and 3 represent calcium sources, and the inclusion of these in a hypocalcemia. Use nursing knowledge and the process of elimination to make a selection.
the development of hypocalcemia? treatment plan would lead to increased serum calcium levels. Option 4 is incorrect because
thiazide diuretics can lead to calcium retention.
1.‐ Phenytoin (Dilantin)
2.‐ Calcium carbonate (TUMS)
3.‐ Calcitriol
4.‐ Hydrochlorothiazide (HydroDIURIL)

2268 The family of a client with hypercalcemia states that Correct answer: 1 Clinical manifestations of hypercalcemia include personality changes. All other options are The core issue of the question is knowledge of manifestations of hypercalcemia. Use
the client is “not acting like himself.” The nurse focuses signs and symptoms of hypocalcemia. nursing knowledge and the process of elimination to make a selection.
assessment on which of the following symptoms?

1.‐ Personality change


2.‐ Anxiety
3.‐ Convulsions
4.‐ Carpal spasms

2269 The nurse who is assessing the client for signs of Correct answer: 2 Clinical manifestations of hypocalcemia include a positive Trousseau’s sign, which is an The core issue of the question is knowledge of manifestations of hypocalcemia. Use
hypocalcemia would conclude that this electrolyte ischemia‐induced carpopedal spasm. A positive Chvostek’s sign is associated with nursing knowledge and the process of elimination to make a selection.
imbalance exists after noting which of the following? hypocalcemia, while hypoactive bowel sounds are a sign of hypercalcemia. Kernig’s sign is an
indication of meningeal irritation.
1.‐ Negative Chvostek’s sign
2.‐ Positive Trousseau’s sign
3.‐ Positive Kernig’s sign
4.‐ Hypoactive bowel sounds

2270 The nurse would review a client’s electrolyte levels to Correct answer: 3 Addison’s disease, known also as adrenal insufficiency, can cause increased magnesium levels The core issue of the question is knowledge of risk factors for hypermagnesemia. Use
detect a possible increase in magnesium if the client resulting from volume depletion. Cushing’s syndrome is hyperfunction of the adrenal gland. nursing knowledge and the process of elimination to make a selection.
had which of the following conditions? Diabetes could lead to low magnesium levels if osmotic diuresis is present from hyperglycemia.
Splenomegaly is an unrelated finding.

1.‐ Cushing’s syndrome


2.‐ Diabetes
3.‐ Addison’s disease
4.‐ Splenomegaly

2271 The nurse concludes that a client does not have an Correct answer: 3 SVT is seen with decreased magnesium levels, as are premature ventricular contractions and The core issue of the question is the ability to discriminate signs of hyper‐ and
increased magnesium level based on which of the ventricular fibrillation. The other three options are findings with hypermagnesemia. hypomagnesemia. Use nursing knowledge and the process of elimination to make a
following findings? selection.
1.‐ Hypotension
2.‐ Bradycardia
3.‐ Supraventricular tachycardia (SVT)
4.‐ Flushing and sweating

2272 A client with renal failure is experiencing Correct answer: 1 Either hemodialysis or peritoneal dialysis is used to remove excess magnesium in the client The core issue of the question is knowledge of effective therapies for increased
hypermagnesemia. The nurse explains that which of with renal failure. Diuretics will not be effective if the kidneys are not functional. Fluid magnesium levels. Note the critical words renal failure, which lead you to look for a
the following treatments will decrease the magnesium restriction would be ineffective, and high‐volume IV fluid replacement would be treatment that does not involve functional kidneys. Use nursing knowledge and the process
level most effectively? contraindicated in renal failure. of elimination to make a selection.
1.‐ Dialysis
2.‐ Diuretics
3.‐ Fluid restriction
4.‐ High‐volume IV fluids

2273 The nurse reviews the laboratory test results for a Correct answer: 4 Decreased magnesium level (option 4) can occur in toxemia of pregnancy, pre‐eclampsia, and The core issue of the question is knowledge of conditions that are consistent with
client with pre‐eclampsia, expecting to find which of eclampsia, causing convulsions (seizures). The other responses are incorrect because they are decreased magnesium levels, and the ability to determine a reduced level. Use nursing
the following values? directed at sodium (options 1 and 2) or increased magnesium level (option 3). knowledge and the process of elimination to make a selection.

1.‐ Sodium 148 mEq/L


2.‐ Sodium 125 mEq/L
3.‐ Magnesium 3.1 mEq/L
4.‐ Magnesium 1.2 mEq/L

2274 A client was admitted to the hospital with a weight Correct answer: 2 Cushing s syndrome causes low potassium and magnesium levels and an increase in sodium The core issue of the question is the ability to synthesize electrolyte results with a clinical
gain of 30 pounds over the past month. Upon and chloride levels. The moon face and buffalo hump are also symptoms of excess picture in a client with Cushing’s syndrome. Use nursing knowledge and the process of
assessment, the client was noted to have a moon face corticosteroids. Option 1 is incorrect because Addison s disease causes low sodium, and elimination to make a selection.
and a “buffalo hump.” On admission, lab results increased magnesium and potassium. Option 3 is incorrect because burn states cause
indicated decreased serum potassium and magnesium, significant fluid and electrolyte disturbances (loss of sodium, chloride, and magnesium, with
and elevated serum chloride and sodium levels. The alterations in potassium depending on the stage of burn), but the presence of a moon face and
nurse interprets that which of the following disorders buffalo hump is characteristic of Cushing s syndrome. Option 4 is incorrect because SIADH is
is consistent with these manifestations? associated with hyponatremia.

1.‐ Addison’s disease


2.‐ Cushing’s syndrome
3.‐ Burns
4.‐ Syndrome of inappropriate ADH (SIADH)

2275 A home health nurse is making a visit to an elderly Correct answer: 2 The home health nurse should be most concerned with the decreased chloride level because The core issue of the question is the ability to determine abnormal electrolyte levels. Use
client who has a history of heart failure (CHF). The it can lead to complications such as dilutional hypochloremia. The client s history of CHF places nursing knowledge and the process of elimination to make a selection.
client was prescribed diuretics twice a day and a low‐ the client in a higher risk category for fluid retention, electrolyte disturbances, and acid–base
sodium diet. The nurse should be most concerned disorders. All of the other options reflect laboratory values that are within normal range, and
about which of the following laboratory results? are reassuring.

1.‐ Na+ 145 mEq/L


2.‐ Cl‐ 90 mEq/L
3.‐ K+ 4.2 mEq/L
4.‐ HCO3‐ 27 mEq/L

2276 Which of the following findings in a client’s history Correct answer: 1 Poor nutritional intake, vomiting, diarrhea, and the overuse of antacid are related to The core issue of the question is knowledge of risk factors for hypophosphatemia. Use
would alert the nurse to assess for signs and symptoms alcoholism and alcohol abuse. These can lead to hypophosphatemia. During oliguria, the nursing knowledge and the process of elimination to make a selection.
of hypophosphatemia? kidneys are unable to excrete phosphorus (option 2). Clients with prolonged (not short‐term)
gastric suction are more likely to experience hypophosphatemia (option 3). Prolonged or
continuous use of aluminum‐containing antacids (not occasional use) leads to
hypophosphatemia (option 4).
1.‐ Withdrawal from alcohol
2.‐ The oliguric phase of acute tubular necrosis
3.‐ Short‐term gastric suction
4.‐ Occasional use of aluminum‐containing antacids
2277 Which of the following concurrent electrolyte Correct answer: 3 Calcium and phosphorus have an inverse relationship in the body. For this reason, when The core issue of the question is knowledge that hypocalcemia accompanies
imbalances should the nurse anticipate while assigned phosphorus levels are high, calcium levels are low (option 3). The other responses do not hypermagnesemia. To answer the question correctly, you must also be able to recognize
to the care of a client with hyperphosphatemia? address this relationship. abnormal laboratory values. Use nursing knowledge and the process of elimination to make
a selection.
1.‐ Potassium 2.8 mEq/L
2.‐ Sodium 131 mEq/L
3.‐ Calcium 6.8 mEq/L
4.‐ Magnesium 3.4 mEq/L

2278 The nurse would report to the charge nurse that an Correct answer: 5.1 Hyperkalemia exists when the serum potassium level rises above the upper limit of normal, The core issue of the question is knowledge that hypocalcemia accompanies
assigned client has hyperkalemia after noting that the which is 5.1 mEq/L. hypermagnesemia. To answer the question correctly, you must also be able to recognize
serum potassium level drawn that morning was abnormal laboratory values. Use nursing knowledge and the process of elimination to make
greater than ____ mEq/L. a selection.

2279 The nurse notices that an elderly nursing home Correct answer: 3 Mental status changes and concentrated urine are common signs of dehydration or FVD in Critical words are best and fluid volume deficit. Note that options 1 and 4 are normal
resident has not been eating or drinking as much as the elderly. Tenting and dry, flaky skin are consistent changes seen with normal aging. Hand findings, and eliminate them. Recognize that option 3 is the best indicator of fluid volume
usual. Which assessment finding would best indicate veins that fill within 3–5 seconds and clear lungs sounds with unlabored breathing are normal deficit to select this option.
the presence of fluid volume deficit (FVD)? findings.

1.‐ Clear lung fields with unlabored respirations


2.‐ Tenting and dry, flaky skin
3.‐ Increased drowsiness, mild confusion, and concentrated urine
4.‐ Hand veins that fill within 3–5 seconds of being lowered below the heart

2280 A client who is three hours post‐tonsillectomy has Correct answer: 4 D&lt;sub&gt;5&lt;/sub&gt;W has a hypotonic effect when infused intravenously, providing Critical words are ice chips and D&lt;sub&gt;5&lt;/sub&gt;W; recall that these are
had minimal blood loss. The client has consumed 12 free water to cells after glucose is metabolized. Ice chips contain no solute, and also provide hypotonic fluids. Recognize that the client has consumed an excessive amount to choose
cups of ice chips to soothe throat discomfort, and has free water through the GI tract. This client has received an excess of free water without option 4.
received 3 liters of D&lt;sub&gt;5&lt;/sub&gt;W during electrolytes, and thus is at risk for hypotonic overhydration (water intoxication). The client is
this time frame. The nurse recognizes the client is at not experiencing deficit or dehydration.
risk for developing which fluid imbalance?

1.‐ Isotonic fluid volume deficit


2.‐ Hypertonic dehydration
3.‐ Isotonic fluid volume excess
4.‐ Hypotonic overhydration

2281 A 56‐year‐old client with a history of heart failure and Correct answer: 4 A history of heart failure increases the risk for fluid volume excess and pulmonary edema. The Critical words are history of heart failure and trouble breathing. Recall the priority of ABCs,
hypertension is hospitalized following abdominal priority is to detect signs of pulmonary edema by assessing for adventitious lung sounds airway, breathing and circulation, to be directed to option 4.
surgery. Vital signs have been stable, and (auscultation for moist crackles). The other observations provide useful information, but are
D&lt;sub&gt;5&lt;/sub&gt;1/2NS is infusing at 80 not the priority.
mL/hour. The client awakens in the middle of the night
complaining of trouble breathing. Which assessment
by the nurse should take priority at this time?

1.‐ Review intake and output record.


2.‐ Assess extremities for edema.
3.‐ Count apical heart rate.
4.‐ Auscultate lung fields.
2282 The nurse is assessing the hydration status of a 50‐ Correct answer: 2 Postural hypotension, a drop in systolic BP greater than 10 mmHg after rising from lying to The critical term is fluid volume deficit. Associate the drop in blood pressure to fluid loss in
year‐old client with gastroenteritis. Which finding standing, is a sign of decreased circulating blood volume. Elevated temperature is associated the blood vessels to be directed to option 2.
would be most indicative of a fluid volume deficit with fluid volume deficit. Falling pulse oximetry readings indicate a problem with oxygen
(FVD)? delivery from the lungs to the blood, which occurs with pulmonary edema, not fluid volume
deficit. The presence of S3 heart sounds in an adult is indicative of venous congestion and fluid
overload.
1.‐ Oral temperature 97.8°F
2.‐ 30 mmHg drop in systolic blood pressure (BP) when client stands
3.‐ Falling pulse oximetry values ranging from 90– 92%
4.‐ Presence of S3 heart sounds

2283 Which of the following interventions should the nurse Correct answer: 1, 2, 3, Ice chips are a source of fluid intake, and if taken in unlimited amounts, can easily contribute The critical term is fluid restriction. Note the word unlimited is opposite of restriction and
include when caring for a client on a fluid restriction? 5 to excess fluid intake. Fluids taken with medications must be included with measurements of recognize that all fluid intake must be measured to eliminate option 4.
Select all that apply. intake and output. Use of the other measures listed is indicated to maximize compliance with
fluid restriction.
1.‐ Involve the client in dividing fluid allowances over the 24‐hour period.
2.‐ Use an infusion pump to control the infusion of any required intravenous fluids.
3.‐ Include ice chips in the client's oral intake measurement.
4.‐ Provide unlimited ice chips to help keep the client's mouth and lips moist.
5.‐ Do not include fluid taken with medications.

2284 A 45‐year‐old client is admitted with extreme thirst, Correct answer: 2 High serum glucose increases blood osmolality, causing water to be pulled from the cells into Recognize that the symptoms reflect a fluid deficit, and that the elevated glucose indicates
frequent urination, tenting skin, sunken eyes, and a the vessels. The excess fluid volume in the vessels is then excreted by the kidneys, resulting in a hypertonic condition, and choose option 2.
dry, furrowed tongue. Because serum sodium is hypertonic dehydration of the cells and its associated symptoms.
normal and serum glucose is elevated, the nurse
determines the client is experiencing which fluid
imbalance?
1.‐ Isotonic fluid volume deficit
2.‐ Hypertonic dehydration
3.‐ Isotonic fluid volume excess
4.‐ Hypotonic overhydration

2285 A mother brings her 14‐month‐old to the Emergency Correct answer: 3 Assessing for low urine output (number of wet diapers) helps determine how severe the Recognize the age of the client and associate urine production with hydration status to
Department, concerned that the infant has been child's fluid losses are. Although an elevated temperature can affect fluid balance by increasing choose option 3.
vomiting for two days and "isn't keeping much down." basal metabolic rate, it is not the most accurate determinant of fluid status at the present
Which question should the nurse ask next to elicit the time. Dietary intake (including solids and fluid) are important in maintaining fluid balance, but
most relevant information needed at this point? the greatest concern is to focus on urinary output as a measure of volume status.

1.‐ "What does your baby usually eat?"


2.‐ "How much does your baby normally drink?"
3.‐ "How many diaper changes have you done in the last 24 hours?"
4.‐ "Did you take the baby's temperature?"

2286 The nurse identifies which of the following clients as Correct answer: 1 Infants and the elderly are the age groups at greatest risk for FVD, in part because they Associate infancy and fluid losses from vomiting with a fluid volume risk, and choose
being at the greatest risk for developing fluid volume cannot compensate for fluid losses as easily as older children and younger adults do. The 2‐ option 1.
deficit (FVD) and dehydration? month‐old is losing fluid from vomiting, and cannot compensate. Arthritis does not directly
increase the risk for FVD. Adults usually handle fluid changes efficiently unless there are
prolonged symptoms and/or underlying disease processes.

1.‐ A 2‐month‐old with vomiting


2.‐ A 30‐year‐old with pneumonia
3.‐ A 45‐year‐old with diarrhea
4.‐ A 70‐year‐old with arthritis
2287 A client with congestive heart failure (CHF) has come Correct answer: 3 Fluid gains cause acute weight gain, which should be explored with clients taking diuretics. Critical words are CHF, diuretic, and 5‐pound weight gain. Recognize that this weight gain
to the clinic for a prescription refill for a diuretic. The Missing doses can lead to fluid gain, or the dosage might need to be adjusted if the heart reflects fluid retention to be directed to option 3.
client reports a 5‐pound weight gain in the past two failure is worsening.
days, and that his shoes are tight. What response by
the nurse is most appropriate?
1.‐ "Have you changed your eating habits lately?"
2.‐ "Five pounds isn't much. You can skip desserts and lose that."
3.‐ "Such a rapid gain is probably water weight. Have you missed any days taking your pill?"
4.‐ "Your clothes still look good on you. Everyone's weight fluctuates during a week."

2288 The nurse determines that which of the following Correct answer: 1 Edema that is still present after lying down all night is more likely to be of cardiac origin. Local Critical words are edema and inadequate heart pumping action. Recall physiology of the
findings in a client most likely reflects edema related edema is caused by local obstruction or poor lymph flow (standing; sitting; lymph node heart and gravity to associate backup of fluid to swollen feet, and note that the fluid is still
to inadequate heart pumping action? removal). Dependent edema is more reflective of localized obstruction or valvular present in the morning, to choose option 1.
incompetence.
1.‐ Swollen feet and sacral area of an elderly client upon awakening in the morning
2.‐ Swelling of legs and feet in a 45‐year‐old client after a ten‐hour airplane flight
3.‐ Ankle swelling in 40‐year‐old waitress that resolves after sleeping all night
4.‐ Swelling in the arm of a client who has had axillary lymph nodes removed

2289 A 78‐year‐old client is admitted with dehydration and Correct answer: 2 This client presented with deficient fluid volume because of dehydration. Older adults have Note that the client is elderly, and has received too much fluid, indicating a fluid overload.
urinary tract infection. After IV infusion of 750 mL NS, less cardiac and renal reserve to compensate for acute fluid imbalances, and thus are more Recall knowledge of fluid volume excess to eliminate option 1. Recognizing that the fluid is
the client begins to cough, and asks for the head of the susceptible to overcorrection when being treated for them. JVD, tachypnea, cough, and isotonic eliminates option 3. Option 4 can be eliminated, since no signs of ascites are
bed to be raised to ease breathing. The nurse assesses dyspnea indicate that this client has received too much IV fluid at too rapid a rate. Older adult mentioned.
jugular vein distention (JVD) and increased respiratory clients cannot tolerate rapid rehydration, due to decreased cardiac and renal function.
rate. The nurse interprets that:

1.‐ The fluid volume deficit is worsening.


2.‐ Hypervolemia is developing.
3.‐ Hypotonic water intoxication is beginning.
4.‐ Ascites is causing respiratory compromise.

2290 The nurse is helping a client who recently was placed Correct answer: 1 Processed and canned foods (bologna, soup, tomato juice), sodas, and pickled foods are high Note the critical word best is used, indicating that one of the choices is better than the
on a low‐sodium diet to choose foods for lunch. The in sodium. Fresh foods (grilled chicken, fruit, vegetables) are lower in sodium. others. Recall knowledge of the sodium content of various foods to eliminate options 2 and
nurse recommends that which of the following lunch 3. Option 4 contains tomato juice, high in sodium, making option 1 best choice.
menus would be best for this client?

1.‐ Grilled chicken sandwich on white bread, apple, salad, and iced tea
2.‐ Bologna sandwich on wheat bread, canned fruit cocktail, salad, and a soda
3.‐ Ham and bean soup, fresh fruit salad, pickles, and a diet soda
4.‐ Cheeseburger, grapes, fresh pineapple, and tomato juice

2291 A 28‐year‐old client is admitted with severe bleeding Correct answer: 1 Acute bleeding results in isotonic fluid loss, and can quickly lead to shock and vascular The critical words are most appropriate and fluid losses. Recognize that the client needs
from a fractured femur. Which intravenous (IV) fluid collapse. The priority is to expand vascular volume and restore circulation using isotonic IV replacement of isotonic fluids to eliminate options 2, 3, and 4, since these are either
does the nurse anticipate as the most appropriate to fluid. Hypertonic (3% saline) and hypotonic (D&lt;sub&gt;5&lt;/sub&gt;W, hypertonic or hypotonic.
replace potential fluid losses? D&lt;sub&gt;5&lt;/sub&gt;¼NS) solutions are not indicated.

1.‐ 0.9% sodium chloride (0.9% NaCl)


2.‐ 3% sodium chloride (3%NaCl)
3.‐ 5% dextrose in water (D5W)
4.‐ 5% dextrose in 0.225 sodium chloride (D5¼NS)
2292 A 54‐year‐old client with liver failure due to cirrhosis Correct answer: 2 The failing liver does not make enough albumin to keep capillary oncotic pressure at normal Note that the client has cirrhosis, and that the symptoms reflect a deficit intravascularly
comes to the clinic complaining of a swollen abdomen levels; thus excess fluid is lost from vessels into the peritoneum, causing ascites and vascular and an excess extravascularly (the abdomen). Eliminate options 3 and 4, since the cause is
and dizziness upon standing. The client is pale, with fluid volume deficit. Orthostatic hypotension, weak peripheral pulses, and delayed hand vein not hormonal. Choose option 2, since symptoms support a deficient fluid volume:
weak radial pulses, delayed hand vein filling, and filling are all signs of low circulating fluid volume. intravascular.
distended abdomen. The nurse develops a care plan
identifying which of the following nursing diagnoses?

1.‐ Excess Fluid Volume: Intravascular related to third space fluid shifts
2.‐ Deficient Fluid Volume: Intravascular related to third space fluid shifts
3.‐ Excess Fluid Volume: Extravascular related to hormonal disturbances
4.‐ Deficient Fluid Volume: Extravascular related to hormonal disturbances

2293 The nurse caring for the following group of clients Correct answer: 3 The 76‐year‐old client has more risk factors than do the other clients. This client is elderly, Critical words are at greatest risk and deficient fluid volume. Recall knowledge of risk
considers which client to be at greatest risk for losing fluids via the NG tube as well as being NPO, and is postoperative of major surgery. factors contributing to fluid deficits, and determine that option 3 has the greater number of
developing deficient fluid volume? Clients taking steroids usually retain sodium and water. Repair of an inguinal hernia is not a them.
major surgery, and the client is likely to recover quickly and be able to resume fluid intake.
Following a sigmoidoscopy, fluids will be resumed.
1.‐ A thin, 52‐year‐old female receiving steroid therapy for bronchitis
2.‐ A 60‐year‐old male who had a left inguinal herniorrhaphy 12 hours ago
3.‐ A 76‐year‐old male who has a nasogastric (NG) tube on intermittent suction following colon resection
4.‐ A 68‐year‐old female who is NPO for a flexible sigmoidoscopy procedure

2294 A 17‐year‐old client who sustained a head injury in a Correct answer: 2 5% dextrose in water (D&lt;sub&gt;5&lt;/sub&gt;W) has a hypotonic effect when infused, Note that the client has a head injury, and recognize the danger of hypotonic fluids that
motorcycle collision two days ago is responsive only to providing free water to cells, which would worsen this client s cerebral edema. The other fluids could contribute to cerebral edema. Also note that the question requires the nurse to
pain. Which intravenous (IV) fluid order would the listed are isotonic, and would remain primarily in the extracellular spaces. question an order, indicating one option will be inappropriate and 3 are appropriate.
nurse question because it could increase the risk of Choose option 2 because it is a hypotonic fluid, and is therefore an incorrect solution to
complications? use.
1.‐ Ringer’s solution
2.‐ 5% dextrose in water (D5W)
3.‐ 0.9% sodium chloride (0.9% NaCl)
4.‐ Lactated Ringer’s solution

2295 The nurse is caring for a client admitted with Correct answer: 1, 2, 3 Hemoglobin and hematocrit can decrease or increase secondary to hemoconcentration or Note that the client has CHF. Recall knowledge of fluid imbalances, and correlate lab
congestive heart failure (CHF). When assessing the hemodilution. ANP is a cardiac hormone released when atria are stretched by increased blood studies associated with them. Eliminate options 4 and 5, since they are not affected by fluid
client’s risk for fluid imbalances, the nurse should volume, which would occur in CHF. Glucose and liver enzymes would not be affected by fluid volume.
check which of the following laboratory values? Select volume.
all that apply.
1.‐ Hemoglobin (Hgb)
2.‐ Hematocrit (Hct)
3.‐ Atrial natriuretic peptide (ANP)
4.‐ Blood glucose
5.‐ Liver enzymes

2296 A client with a nursing diagnosis of Excess Fluid Correct answer: 1 S&lt;sub&gt;3&lt;/sub&gt; heart sounds and moist lung crackles are signs associated with Note the critical phrases Excess Fluid Volume and has not yet been achieved. Recall signs
Volume has been treated with diuretics and fluid fluid overload, not deficit. Since they are resolving, the client is returning to normal status, but and symptoms of fluid excess, and eliminate option 2 because it indicates full resolution of
restriction. Which of the following findings would has not yet reached complete balance. Option 2 indicates full resolution of fluid balance, and is these signs. Eliminate options 3 and 4 because they indicate resolution of deficient fluid
indicate to the nurse that fluid volume balance has not therefore incorrect. Options 3 and 4 show resolving signs of fluid volume deficit and volume, which is not the focus of the question.
yet been achieved? dehydration, but the question is addressing excess fluid volume.

1.‐ S3 heart sound and moist lung crackles resolving


2.‐ Return to coherent conversation and appropriate behavior
3.‐ Urine output increasing and specific gravity decreasing
4.‐ Skin tenting decreasing and conjunctiva of eyes moist

2297 During intershift report, the nurse is told that a client Correct answer: 1 Diabetes insipidus (DI) is a condition caused by insufficient production and/or release of ADH. The critical words are diabetes insipidus and at risk for. Eliminate options 3 and 4 because
who has suffered a stroke has also developed diabetes Inadequate ADH leads to increased excretion of dilute urine. Excessive production of ADH, they relate to diabetes mellitus. Discriminate appropriately between DI and SIADH to
insipidus. The nurse concludes this client is now at risk known as SIADH, leads to fluid retention and excess fluid volume. Changes in glucose levels and choose correctly between options 1 and 2.
for which of the following? insulin production are associated with diabetes mellitus.

1.‐ Severe deficient fluid volume because of excess urine output


2.‐ Severe excess fluid volume because of inadequate urine output
3.‐ Hyperglycemia because of poor insulin production
4.‐ Hypoglycemia because of excess insulin production

2298 A father telephones the clinic nurse asking what he Correct answer: 2 Commercial oral rehydration fluids, such as Pedialyte or Rehydralyte, are balanced water, Recall knowledge of age‐appropriate fluids. Determine that the symptoms reflect risk for
should do for his 3‐year‐old son, who developed fever, carbohydrate, and electrolyte solutions that replace both fluids and electrolytes lost in fluid volume deficit to choose option 4.
vomiting, and diarrhea today. Which of the following diarrhea. They also do not have a high osmolality, caffeine, or excess sodium, all of which can
would be an appropriate recommendation by the worsen diarrhea and fluid loss. Replacing diarrhea losses with only water could lead to
nurse? electrolyte imbalances. Fruit juice and sports drinks are too high in sugar, which can worsen
diarrhea and fluid loss. Solid foods, including the BRAT diet, are not appropriate while the
client is vomiting.
1.‐ “Have him drink as much water as you can get him to swallow.”
2.‐ “Give small sips of commercial oral rehydration fluids frequently.”
3.‐ “Provide frequent sips of fruit juice and commercial sports drinks.”
4.‐ “Have him eat only bananas, rice, applesauce, and toast (BRAT diet).”

2299 The nurse determines that which of the following Correct answer: 1 Infants and the elderly don't compensate well for fluid losses. Clients with NG suction (loss of Recall concepts of fluid balance, and factors contributing to losses. Eliminate option 2,
clients is at greatest risk for developing a fluid volume fluids and electrolyes in fairly proportional amounts) are at greater risk for fluid volume deficit. since fluid is gained with steroids, and options 3 and 4 because they have fewer risk factors
deficit? The elderly client with NG suction has both risk factors, while the child’s age is the only risk than the correct option.
factor. The client taking glucocorticoids is predisposed to sodium and fluid retention rather
than fluid loss. The 30‐year‐old jogger is a young adult in a moderate climate, which lowers the
risk from exertion alone.

1.‐ A 76‐year‐old client who has a nasogastric (NG) tube on low suction following surgery for colon cancer
2.‐ A thin, 55‐year‐old client who smokes and takes glucocorticoids for chronic lung disease
3.‐ A 1‐year‐old child being treated in the clinic for a runny nose and ear infection
4.‐ A 30‐year‐old client jogging in 50‐degree weather

2300 The nurse is assisting in a health fair at a senior Correct answer: 4 Items that are liquid at body temperature are also considered part of fluid intake, so ice pops, Note the critical words elderly and remain hydrated. Recall knowledge of risk factors for
citizen center. Which of the following instructions gelatin, and ice cream can be considered part of overall fluid intake. The color of urine is only dehydration in the elderly and of fluid intake to make a selection.
should the nurse include when giving the elderly client one indicator of hydration, and many elderly people take a diuretic, which would produce
guidelines to remain hydrated in hot weather? more dilute urine, falsely reassuring the client. With aging, the thirst mechanism becomes less
effective. Significant fluid can be lost before thirst is triggered, so the elderly should not rely
solely on thirst to indicate when they need to drink fluids.

1.‐ “If your urine is clear yellow, you are drinking adequate fluids.”
2.‐ “Drink only water to keep yourself properly hydrated.”
3.‐ “Popsicles, gelatin, and ice cream provide fluid intake as well as liquids you drink.”
4.‐ “Use your thirst as a guide to the amount of fluid you should be drinking.”
2301 An adult client in the clinic complains of a cough, Correct answer: 3 The client has symptoms of fluid volume deficit (FVD) and hypovolemia. The presence of Recognize that the client has signs and symptoms of an FVD: intravascular, and recall
fever, nausea, and vomiting for three days. postural hypotension when rising from a lying position indicates the presence of significant clinical manifestations of such. Eliminate options 1 and 4 because they don t reflect
Examination reveals dry tongue and oral mucosa, and hypovolemia. The other vital signs are important for general reasons, but do not directly intravascular fluid losses. Choose option 3 over 2, since it best supports the client
concentrated urine. The client also reports feeling reflect circulating fluid volume. complaints of weakness and dizziness.
weak and dizzy. To best assess the client’s fluid status,
the nurse checks which of the following assessment
parameters?
1.‐ Temperature
2.‐ Respiratory rate and depth
3.‐ BP and pulse in lying and standing positions
4.‐ Pulse oximetry reading at rest

2302 The nurse evaluates the hydration status of a client Correct answer: 1, 4, 5 The neck veins are normally flat when the head of bed is elevated, due to gravity. Dyspnea Note the critical words hydration status and excess fluid volume. Choose those options
who has been receiving intravenous (IV) fluids at 150 with exertion is often a sign of fluid in the alveoli. Pitting edema reflects fluid in the interstitial that reflect a full vascular bed, which in this case are peripheral edema, venous
mL/hour. The nurse identifies that the client has excess spaces. Hand veins would remain full or empty slowly if excess fluid volume (EFV) is present. engorgement, and excessive pulmonary fluid.
fluid volume after assessing which of the following? Peripheral pulses would be bounding with an EFV.
Select all that apply.
1.‐ Neck veins are distended when the head of the bed is elevated 45 degrees.
2.‐ Hand veins are empty when the hand is raised above the heart.
3.‐ Peripheral pulses are rapid and weak.
4.‐ Client becomes short of breath when ambulating.
5.‐ Pitting edema is present over tibia.

2303 A client hospitalized for gastrointestinal (GI) bleeding Correct answer: 3 Normal saline (0.9% NaCl) is an isotonic fluid that prevents fluid shifts into or out of the GI Recall the principles of osmosis and diffusion, and the differences among hypotonic,
has orders for nasogastric tube (NGT) placement, with tract. Option 1 (3% saline) is hypertonic, and could pull water from the GI tract, resulting in isotonic, and hypertonic fluids. Eliminate options 1, 2, and 4, since fluid shifts will occur with
irrigations until the returns are clear. Which of the water loss. D&lt;sub&gt;5&lt;/sub&gt;W and sterile water are hypotonic, and could be pulled hypotonic and hypertonic solutions.
following solutions should the nurse plan on using? into GI tissue, as well as wash electrolytes from the GI tract, resulting in water intoxication.

1.‐ 3% Sodium chloride (NaCl)


2.‐ 5% Dextrose in water (D5W)
3.‐ 0.9% Sodium chloride (NaCl)
4.‐ Sterile water

2304 A 70‐year‐old hospitalized client with a past medical Correct answer: 4 A moist cough, dyspnea, and a falling pulse oximetry reading in a client with a history of heart Note the critical word first, indicating one option has a priority action. Use knowledge of
history of hypertension and myocardial infarction is disease are signs of developing pulmonary edema secondary to excess fluid volume (EFV). The cardiovascular disease and EFV to choose option 4.
postoperative following stomach surgery. Vital signs first action should be to reduce IV fluid intake, to prevent more fluid from accumulating in the
have been stable, and an IV of lungs; then further assessment can be done, emergency actions taken, and the physician
D&lt;sub&gt;5&lt;/sub&gt;HNS is infusing at 100 contacted.
mL/hour. The client now complains of trouble
breathing, and has a moist cough, and the pulse
oximetry reading has fallen to 92%. What action
should the nurse take first?
1.‐ Measure blood pressure and heart rate.
2.‐ Assess legs and arms for pitting edema.
3.‐ Telephone and notify the physician.
4.‐ Slow the intravenous rate to 10–20 mL/hour.
2305 A 45‐year‐old client with excess fluid volume (EFV) Correct answer: 2 Option 2 provides accurate information in simple terms without unduly alarming the client. Note the critical words best and response. Use knowledge of communication skills and
because of acute kidney dysfunction is placed on a Option 4 is technically correct, but is stated in an abrupt and alarming manner. Option 1 offers regulation of fluid imbalances to choose option 2.
1,000 mL fluid restriction per 24‐hour period. The no explanation to facilitate understanding. Option 3 assigns the client blame for the current
client asks the nurse, “Why there is such a severe fluid condition without providing a clear explanation for the fluid restriction.
restriction when I already have dry lips and mouth?”
Which response by the nurse is best?

1.‐ “The doctor ordered the fluid restriction, so it is important to comply with those orders.”
2.‐ “Your kidneys cannot eliminate extra fluid right now, so intake must be limited to protect your heart and lungs from being overloaded with fluid.”
3.‐ “You probably drank too much fluid before you got sick, so you can’t compare your usual intake to your limitations now that your kidneys are not working.”
4.‐ “Too much fluid will cause your heart to fail and your lungs to fill with water, which could be fatal.”

2306 A 45‐year‐old female client is receiving a loop diuretic Correct answer: 1 An excess response to diuretic therapy results in an excess loss of water and electrolytes in Note the critical words excessive response, indicating a greater‐than‐desired action is
for treatment of edema. The nurse determines the the urine, leaving the blood hemoconcentrated, and causes a high BUN (normally 8–22 mg/dL) achieved. Use knowledge of diuretic action to eliminate options 2, 3, and 4.
client is experiencing an excessive response to the and HCT (normally approximately 38–45%). The water loss results in an acute weight loss.
treatment when the client demonstrates: Weight gain indicates ineffective response to diuretic therapy.

1.‐ Blood urea nitrogen (BUN) 28 mg/dL; hematocrit (HCT) 45%; and an 8‐pound weight loss in 24 hours.
2.‐ BUN 21 mg/dL; HCT 29%; and an 8‐pound weight gain in 24 hours.
3.‐ BUN 16 mg/dL; HCT 31%; and an 8‐pound weight loss in 24 hours.
4.‐ BUN 25 mg/dL; HCT 33%; and an 8‐pound weight gain in 24 hours.

2307 A client is receiving an intravenous (IV) infusion of Correct answer: 4 G NS (0.225% sodium chloride) is a hypotonic solution that provides free water to the cells. Note critical words complications of therapy and most important, indicating that one of
0.0225% sodium chloride at 50 mL/hour. In order to Cerebral cells are especially sensitive to fluid gains from hypotonic fluids. If infused too rapidly, the options is of highest priority. Recall knowledge of fluid shifts with hypotonic fluids to
detect complications of therapy, it is most important the cerebral cells will be the first to gain fluid too quickly, resulting in neurological changes. choose option 4.
for the nurse to monitor: Monitoring the client for urine output, edema, and oral cavity dryness is important, but this
reflects a response to IV therapy rather than detection of a complication.

1.‐ Urine output and concentration.


2.‐ Legs and arms for edema.
3.‐ Tongue and mouth for dryness.
4.‐ Mental status and orientation.

2308 When caring for an adult receiving an intravenous (IV) Correct answer: 1 Three percent (3%) saline is very hypertonic, and, if infused too rapidly, will increase serum Note critical words priority and detect. Recall physiology of fluids shifts from hypertonic
infusion of 3% sodium chloride (NaCl), the nurse places sodium and osmolality, causing high volumes of water to be pulled into vessels from cells. This solutions to choose option 1.
priority on monitoring which of the following to detect results in cellular dehydration and vascular volume overload. Serum sodium levels,
complications of therapy? neurological status, and lung function should be closely monitored. Although daily weights are
important, they do not provide information leading to early detection of complications of
therapy. Vital signs, serum glucose levels, urine‐specific gravity, oxygen saturation, and
peripheral edema provide later indications of complications of therapy.

1.‐ Neurological status, lung sounds, and serum sodium levels


2.‐ Heart rate, blood pressure, and daily weights
3.‐ Serum glucose levels and urine‐specific gravity
4.‐ Pulse oximetry and peripheral edema in the legs

2309 The nurse identifies which of the following Correct answer: 3 A TURP procedure can place a client at risk for developing hyponatremia in the postoperative The critical term is sodium imbalance. Note that the question does not specify hypo‐ or
postoperative clients to be at risk for developing a period due to increased fluid irrigation used during treatment. Clients with a TURP procedure hypernatremia. Associate irrigation fluids post‐TURP with a sodium imbalance to choose
sodium imbalance? have a continuous bladder irrigation (CBI) as a routine part of their postoperative care. The option 3.
other options do not place a client at risk for the development of sodium imbalances, as they
do not require lengthy fluid and dietary restrictions or excessive fluid irrigation.

1.‐ A client who has just had a tonsillectomy


2.‐ A client who has a primary cesarean section for failure to progress in labor
3.‐ A client who has a transurethral resection of the prostate (TURP)
4.‐ A client who has a right knee arthroscopy

2310 The nurse is caring for a client experiencing Correct answer: 4 Hyponatremia can also be referred to as dilutional hyponatremia or water intoxication. Water The critical term is hyponatremia. Recall that water will further dilute the sodium, and
hyponatremia. As part of the care, the nurse will restriction would be an important part of the treatment plan when caring for a client who has choose option 4.
restrict: hyponatremia. The restriction of Gatorade (electrolyte‐rich solution), eggs, cheese products,
and salt on the diet tray are not indicated, because the client is experiencing a sodium deficit.

1.‐ Gatorade.
2.‐ Eggs and cheese products.
3.‐ Salt on the diet tray.
4.‐ Water.

2311 The nurse is caring for a client experiencing Correct answer: 3 Clients with hypernatremia are thirsty, and need water replacement to balance the increased The critical term is hypernatremia. Associate the role of water in dilution of sodium to
hypernatremia. The nurse concludes that it is sodium levels. Cough medication and lactulose can further increase sodium levels, and should choose option 3.
important to administer: not be administered unless there is sufficient clinical information to warrant their use. Three‐
percent saline is a hypertonic solution that would also increase serum sodium levels, and
should not be given to this client.
1.‐ Cough suppressant to treat symptomatic cough.
2.‐ 3% saline solution.
3.‐ Water.
4.‐ Lactulose.

2312 The nurse provided teaching about dietary sodium Correct answer: 3 Pork contains high amounts of sodium. Clients with CHF are prone to hypervolemic The question is a negative‐response item, indicating there are three correct responses
modifications for a client with congestive heart failure hypernatremia, and should decrease their salt intake. The other options list foods containing reflecting the client's correct choice of foods. Recall foods high in sodium to choose option
(CHF). Which of the following menu selections relatively small amounts of sodium that could easily be incorporated into the client's diet. 3.
indicates that the client needs further education?

1.‐ Broiled chicken breast


2.‐ Salad
3.‐ Baked pork chop
4.‐ Toast with margarine

2313 A client with a history of sodium imbalances presents Correct answer: 1, 2, 4 Osmotic pressure causes fluids to move in both directions simultaneously; it is not a Recall measurements that are reflective of fluid balance to choose options 1, 2, and 4.
to the clinic for a monthly checkup. Which of the parameter that is assessed. Skin turgor, intake and output, and blood pressure are physical
following parameters should the nurse assess related assessment parameters the nurse would consider when assessing fluid‐and‐electrolyte balance
to fluid and electrolyte balance? Select all that apply. in a client who has experienced prior sodium imbalances. Options 3 and 5 would not be
necessary for a routine fluid‐and‐electrolyte assessment. The key concept is fluid and
electrolyte balance.
1.‐ Blood pressure
2.‐ Intake and output
3.‐ Content of last meal
4.‐ Skin turgor
5.‐ Exercise tolerance
2314 A client has severe hyponatremia secondary to Correct answer: 2 When hyponatremia is severe, hypertonic IV solutions may be used as part of the treatment The key term is SIADH. Recall that this condition is characterized by hyponatremia and
syndrome of inappropriate antidiuretic hormone plan. Three‐percent saline is an example of a hypertonic solution. In SIADH, hypotonic and fluid volume excess to choose option 2.
(SIADH). The nurse concludes that the appropriate IV isotonic solutions are not indicated, because urine output is minimal and water is retained
solution for this client would be a(n): (options 1 and 3). Option 4 is also incorrect, because D&lt;sub&gt;5&lt;/sub&gt;W is not an
isotonic solution, but rather a hypotonic solution. It is important to know not only the correct
type of solution indicated but also the types of solutions that fall under each category.

1.‐ Isotonic solution, such as normal saline (NS).


2.‐ Hypertonic solution, such as 3% saline.
3.‐ Hypotonic solution, such as D5W.
4.‐ Isotonic solution, such as D5W.

2315 The nurse is monitoring the daily weight of a client Correct answer: 2 Weight loss of more than 0.5 pounds in 24 hours is considered to be due to fluid loss. A The question addresses the relationship of weight loss to fluid loss. Recall normal
with a sodium imbalance. The nurse interprets that weight loss of 0.25 pounds is not significant enough to be considered fluid loss. Although 1 equivalency to be directed to option 2.
fluid loss might be occurring when daily weight loss pound or 1 kilogram might indeed reflect a significant fluid loss, these are greater than the
begins to exceed: minimum level that denotes fluid loss.
1.‐ 0.25 pounds.
2.‐ 0.5 pounds.
3.‐ 1 pound.
4.‐ 1 kilogram.

2316 The nurse is providing care to a client experiencing Correct answer: 2 Hypotonic fluid volume excess (FVE) involves an increase in water volume without an increase Critical terms are hypotonic fluid and FVE. Recall that this condition reflects water excess
hypotonic fluid volume excess. The nurse incorporates in sodium concentration; thus an increase in sodium intake is part of the treatment. Decreased and sodium deficiency to be directed to option 2.
which of the following dietary considerations in the sodium intake will result in an even lower sodium level, since hypotonic FVE is associated with
menu for this client? low sodium. Fluids are restricted, and an intake of potassium‐rich foods is not related to the
treatment of hypotonic FVE.
1.‐ Decreased sodium intake
2.‐ Increased sodium intake
3.‐ Increased fluid intake
4.‐ Intake of potassium‐rich foods

2317 A client presents to the walk‐in clinic complaining of Correct answer: 2 The client's clinical presentation suggests that hyponatremia is occurring (tachycardia, Analyze the options for all relevant parameters related to fluid and sodium imbalances.
nausea and vomiting for two days, and development of hypotension, and dry skin/membranes). Obtaining information relative to intake and output Recall the importance of intake and output to fluid imbalance to choose option 2.
a mild headache this morning. Physical examination can help to identify potential/actual fluid losses and possibly identify the etiology of the
reveals BP of 100/70; pulse rate of 92; respiratory rate sodium deficit. It will also give vital information relative to the hydration status of the
of 20; and pale, dry skin. What other priority individual. While the other options should be addressed, they are not the priority at the
information would help the nurse to evaluate this present time, unless there is an acute presentation.
client relative to sodium balance?

1.‐ Pain characteristics related to the headache


2.‐ Intake and output over the last two days
3.‐ Whether the client is experiencing any other symptoms
4.‐ Whether the nausea continues throughout the day or is relieved after the client vomits
2318 A client's serum sodium level is 175 mEq/L. Which Correct answer: 4 It is vital to notify the physician of a laboratory value that is critical. A sodium level of 175 Recognize that the abnormal level reflects hypernatremia. Eliminate options 1 and 2, since
one of the following priority treatment measures mEq/L requires immediate medical attention and intervention. Option 1 is incorrect, as a they provide additional sodium. Because the level is critically high, choose option 4.
should the nurse use to restore normal sodium hypertonic solution is not indicated in the treatment of hypernatremia. Option 2 is incorrect
balance? because the TPN might be causing the client to experience hypernatremia as a source of salt
excess/concentrated hypertonic solution. Option 3 might be indicated at a later date, but it is
not a priority at the present time. Prompt recognition and treatment with appropriate fluids
(free water, NS, or D&lt;sub&gt;5&lt;/sub&gt;W depending on the client's volume status)
should be instituted per protocol. Remember that cerebral cells are adaptive to different
sodium levels, and that rapid correction can lead to further complications.

1.‐ Correct rapidly using a 3% saline solution to restore intravascular deficits.


2.‐ Continue to hang ordered TPN solution and wait for further physician orders based on the current serum value.
3.‐ Consult with a dietician to eliminate additional sodium in the diet.
4.‐ Notify the physician of the client's current sodium level and other serum and urine electrolyte levels that will help to identify the clinical picture.

2319 When assessing a client with diabetes insipidus (DI), Correct answer: 2 DI is characterized by a decrease in ADH secretion, resulting in loss of fluids through polyuria; Recall physiology of DI. If you have trouble recalling information, a clue might be the
the nurse expects to find which of the following? polyuria in turn leads to increased thirst. Options 1, 3, and 4 are not characteristic of DI. common word diabetes—although diabetes mellitus is different from diabetes insipidus,
they share the common symptoms of polyuria and polydipsia.
1.‐ Nausea and vomiting
2.‐ Polyuria and polydipsia
3.‐ Dysuria
4.‐ Confusion

2320 A client receiving treatment for hypernatremia is Correct answer: 2 Too rapid a correction of hypernatremia can lead to changes in vascular tone, which can Critical words are hypernatremia and complications. Note that the question addresses the
being monitored for signs and symptoms of affect vessels and cause increased fluid entry into the brain, thereby causing cerebral edema. complications of treatment. Recall that treatment involves fluid replacement, increasing
complications of therapy. The nurse would assess this Option 1 reflects cellular dehydration, which is caused by hypernatremia. Options 3 and 4 are risks of fluid shifts, to choose option 2.
client for which of the following? not viewed as risks when treating hypernatremia.

1.‐ Cellular dehydration


2.‐ Cerebral edema
3.‐ Red blood cell (RBC) destruction
4.‐ Renal shutdown

2321 A client is semiconscious and restless, and exhibits Correct answer: 4 This client has signs and symptoms of hypernatremia, and the serum sodium level would be Recognize that the symptoms in the question reflect hypernatremia. Systematically
tremors and muscle weakness. Physical examination greater than 145 mEq/L. Options 1 and 2 reflect decreased serum sodium levels, and are eliminate options lower than 145 mEq/L.
reveals a dry, swollen tongue, and body temperature considered to be hyponatremic. Option 3 reflects a normal serum sodium level.
of 99.8°F. The nurse anticipates that the serum sodium
value for this client is most likely to be which of the
following?
1.‐ 120 mEq/L
2.‐ 132 mEq/L
3.‐ 142 mEq/L
4.‐ 155 mEq/L

2322 A client was brought to the hospital following a near‐ Correct answer: 1 Near drowning in salt water often results in hypernatremia due to the high sodium level in Critical words are near‐drowning and ocean, indicating ingestion of salt water. Recognize
drowning experience in the Atlantic Ocean. In sea/salt water. Hyponatremia and disturbances in calcium levels are not seen in this clinical that this constitutes a sodium load, and choose option 1.
providing care to this client, the nurse plans to situation.
carefully monitor for which of the following?
1.‐ Hypernatremia
2.‐ Hyponatremia
3.‐ Hypocalcemia
4.‐ Hypercalcemia
2323 When caring for an adult client diagnosed with Correct answer: 1 In hyponatremia, water is already present in an excessive amount compared with the amount The critical word is restrict. Recall dangers related to further dilution of sodium to choose
hyponatremia, the nurse plans to restrict which of the of sodium present. This can result in water intoxication or dilutional hyponatremia; therefore, option 1.
following? water restriction is a cornerstone of therapy. The other electrolytes (sodium, potassium, and
chloride) should not be restricted, but rather should be included in the treatment plan so as to
prevent further electrolyte imbalances from occurring.

1.‐ Water
2.‐ Sodium

3.‐ Potassium
4.‐ Chloride

2324 For which of the following manifestations should the Correct answer: 4 Thirst is a primary indicator of sodium excess (hypernatremia), and should be assessed in a Critical words are manifestations and hypernatremia. Recall the primary compensatory
nurse assess when developing a plan of care for a plan of care for a client with hypernatremia. Muscle weakness is not reflective of mechanism for fluid balance to choose option 4.
client with hypernatremia? hypernatremia, but is more likely to be found with sodium deficit. Moist mucous membranes
are not associated with sodium imbalances, and reflect a normal parameter. An elevated
temperature would be expected with hypernatremia.
1.‐ Muscle weakness
2.‐ Moist mucous membranes
3.‐ Subnormal temperature
4.‐ Complaints of thirst

2325 The nurse is providing care to a client with syndrome Correct answer: 2 In SIADH, the antidiuretic hormone is present in excess amounts. This causes excessive water The question addresses the relationship between water and SIADH. Note that options 1, 3,
of inappropriate antidiuretic hormone (SIADH). The reabsorption. Water must be restricted, to avoid water intoxication. Giving additional fluids and 4 are similar in that all provide fluids in some form; therefore, these should be
nurse explains to the unlicensed assistant that water would only serve to further increase fluid levels and increase sodium deficit. While it is eliminated.
intake should be: important to consider a client s preference in fluid selection, fluid restriction is the major
priority. Administering fluids only via the intravenous route is not the preferred method. While
fluid therapy can be given IV, it is important to allow the client to take PO fluids, even if only
on a restricted basis.
1.‐ Encouraged.
2.‐ Restricted.
3.‐ Given according to the client’s preference.
4.‐ Given via intravenous fluids only.

2326 A client with a diagnosis of bipolar disorder has been Correct answer: 2 The client has exhibited behavior that could indicate a sodium‐and‐water imbalance, and is Recognize that the symptoms are reflective of a fluid volume excess and hyponatremia to
drinking copious amounts of water, and voiding actually exhibiting signs of hyponatremia. The nurse would check the electrolyte levels, choose option 2.
frequently. The client is experiencing bounding pulse expecting to find a low sodium level. Monitoring the CBC for a platelet level is not indicated, as
and confusion, and is reporting a headache. The nurse there is no correlation between sodium levels and platelet activity. The client s serum
checks laboratory test results for which of the osmolality and urine‐specific gravity are expected to be low due to water intoxication.
following?
1.‐ Low platelet count
2.‐ Low sodium level
3.‐ High serum osmolality
4.‐ High urine‐specific gravity

2327 A client with a feeding tube has been experiencing Correct answer: 1 The client is exhibiting signs of hypernatremia and dehydration. The most appropriate nursing Recognize that the symptoms reflect dehydration and hypernatremia. Eliminate options 3
severe watery diarrhea. The client is lethargic, with intervention is to measure and record intake and output and daily weight. Administering salt and 4 because fluids need to be replaced. Eliminate option 2 because additional salt is not
decreased skin turgor, pulse rate of 110, and tablets would further contribute to the client s hypernatremic state. Restricting fluid intake needed.
hyperactive reflexes. The nurse would include which of and holding feedings could further contribute to the client s state of hypernatremia with fluid
the following interventions on the client s plan of care? volume deficit (hypertonic dehydration), as the client already has extensive fluid loss due to
diarrhea, elevated pulse rate, and decreased skin turgor.
1.‐ Monitor and record intake, output, and daily weights.
2.‐ Administer salt tablets, and monitor hypertonic parenteral solutions.
3.‐ Withhold tube feedings until diarrhea subsides.
4.‐ Avoid adding additional water before and after tube feedings.

2328 The nurse assigned to a client with hyponatremia Correct answer: 3 Fluid retention can result in hyponatremia through dilutional effect. Options 1 and 2 could Analyze the options to identify the potential for sodium losses or dilution. Eliminate
would conclude that which of the following client lead to hypernatremia. Option 4 would lead to hypernatremia. options 1 and 2, since more water than sodium is lost. Eliminate option 4, since sodium
factors probably contributed to this electrolyte would be gained.
imbalance?
1.‐ Osmotic diuretic therapy
2.‐ Fever
3.‐ Fluid retention
4.‐ Excessive hypertonic intravenous infusion

2329 For which of the following serum electrolyte Correct answer: 3 The combination of high fever and severe dehydration leads to insensible water loss. This Critical words are imbalances, high fever, and severe dehydration. Systematically eliminate
imbalances would the nurse assess in a client admitted indicates a loss of pure water, which does not contain electrolytes. Therefore, excessive options containing an imbalance not associated with water losses and dehydration. Recall
with a high fever and severe dehydration? amounts of insensible water loss result in a hypertonic dehydration that leads to a state of that only water is lost with fever, combined with the dehydration, to choose option 3.
hypernatremia and hyperchloremia. Calcium levels usually decrease in the presence of
dehydration and fever. Phosphate levels usually increase in the presence of dehydration and
fever. Potassium levels can usually remain normal in the serum, and are increased in the urine.

1.‐ Hypercalcemia and hypophosphatemia


2.‐ Hypokalemia and hyponatremia
3.‐ Hypernatremia and hyperchloremia
4.‐ Hypophosphatemia and hypocalcemia

2330 Which of the following clients would the nurse Correct answer: 1 The use of corticosteroids can lead to the development of hypernatremia because they cause The critical word greatest indicates that all or some of the options are correct, but one will
identify as being at greatest risk to develop a sodium sodium to be retained and potassium to be excreted. The elderly client drinking eight glasses have the greatest influence on creating the imbalance. Eliminate option 2, since this is not
imbalance? of water each day is within a normal range of fluid intake, and is not at risk for developing an abnormal amount of water to consume. Eliminate option 3, as it is not associated with
sodium imbalances. The diabetic client whose blood glucose is within normal range is not at sodium, and option 4, since the client is young and better able to accommodate an
risk for developing sodium imbalances. The teenager who is using Gatorade as an oral imbalance.
replacement therapy to compensate for fluid and electrolyte loss during exercise is not at risk
for developing sodium imbalances.

1.‐ An adult client taking corticosteroid therapy


2.‐ An elderly client who drinks eight 8‐ounce glasses of water each day
3.‐ A diabetic client who is under glycemic control
4.‐ A teenager who is drinking Gatorade during exercise workouts

2331 When caring for a 79‐year‐old client who has a Correct answer: 1 The thirst mechanism is decreased in the elderly, and would normally serve as a Critical items to note include an elderly client, hypernatremia, and risk for dehydration.
sodium level of 149 mEq/L, the nurse identifies that compensatory mechanism to provide water intake. Aldosterone production would be Knowledge of compensatory mechanisms of fluid balance is required to answer this
the client will be at increased risk to develop decreased in the presence of hypernatremia. Muscle mass can be reduced in the elderly, but question. Eliminate options 2 and 4, since they are incorrect. Choose option 1 over 3, since
dehydration because of which of the following factors? the decreased thirst poses a greater risk. ADH is still produced. this most affects fluid balance.

1.‐ A diminished thirst drive


2.‐ An increased level of aldosterone
3.‐ A decrease in muscle mass
4.‐ ADH, antidiuretic hormone, is no longer produced.
2332 Which of the following interventions does the nurse Correct answer: 3 Clients with hypernatremia (normal sodium level is 135–145 mEq/L) should be assessed for The core concept of the question is knowledge of interventions that are necessary when a
complete when caring for a client admitted with a potential development of neurological complications, such as seizures. Malaise and nausea are client has hypernatremia. Recall that high sodium levels cause temperature elevations to
sodium level of 152 mEq/L? symptoms of hyponatremia. Blankets are not needed, since temperature is often elevated with eliminate option 1, and eliminate option 4, since fluids need to be encouraged. Remember
hypernatremia. Clients with hypernatremia have an increased need for fluids, not a decreased that seizures are a risk with high sodium levels to choose correctly between options 2 and
need. 3.
1.‐ Provide extra blankets for warmth.
2.‐ Observe the client for nausea and malaise.
3.‐ Observe and prepare for possible seizures.
4.‐ Restrict fluids to 1,200 mL per day.

2333 The nurse is teaching a client on a low‐sodium diet Correct answer: 1, 2, 3 Processed foods, seasonings, some baking products, and many over‐the‐counter cough, cold, Recall knowledge of foods or products containing sodium. Eliminate option 4 because it
how to read food labels and check for hidden sodium and flu remedies contain sodium. Clients need to be taught to look for products that list would be high in sugar, and option 5 because it is oil, which typically does not have sodium
content. The nurse informs the client that sodium is sodium as an ingredient. added.
contained in higher amounts in which of the following
products? Select all that apply.

1.‐ Baking goods containing baking powder


2.‐ Seasonings using monosodium glutamate (MSG)
3.‐ Over‐the‐counter cold and cough preparations
4.‐ Canned fruits
5.‐ Salad oil

2334 The nurse is caring for a client who is experiencing a Correct answer: 1 As sodium levels decrease, fluid shifts in the brain can lead to cerebral edema and seizures. Critical words are highest priority, indicating that all options will be appropriate, but one is
steady decline in sodium level. The nurse places the Clients should be assessed for headaches, lethargy, decreased responsiveness, and seizures. most important. Note the similarity between options 2 and 3 to eliminate them. Choose
highest priority on: Hyponatremia will also cause weakness and fatigue, and the client needs to conserve energy, option 1, since neurological status is of higher priority than skin care.
but neurological status is of the highest priority. Oral and skin care would also not be of
highest priority.
1.‐ Close monitoring of neurological status.
2.‐ Preventing weakness and fatigue.
3.‐ Spacing activities to conserve energy.
4.‐ Providing oral hygiene and skin care.

2335 When caring for a client with syndrome of Correct answer: 4 SIADH is caused by excessive production of ADH or an ADH‐like substance, resulting in The concept is testing application of SIADH knowledge. Recall that the client experiences
inappropriate antidiuretic hormone (SIADH), the nurse decreased serum sodium and hypervolemia. Loop diuretics are given to promote diuresis. Oral fluid retention and sodium losses to eliminate options 1, 2, and 3.
plans to do which of the following to correct the fluid‐ fluids are restricted due to the hypervolemia. Dietary sodium is encouraged. Hypertonic or
and‐electrolyte imbalance associated with this isotonic intravenous solutions are administered to provide needed sodium.
disorder?
1.‐ Encourage the client to drink plenty of water.
2.‐ Restrict dietary salt intake.
3.‐ Monitor infusion of hypotonic saline infusions.
4.‐ Administrate ordered loop diuretics.

2336 Lab chemistry results reveal that a client s serum Correct answer: 270; An estimate of serum osmolality is obtained by multiplying the sodium level by 2. The normal Recall normal serum sodium level, and multiply by 2.
sodium is within normal range. Based on this finding, 290 range of sodium is 135–145.
the nurse estimates the client s serum (plasma)
osmolality to be between _____ and _____ mOsm/kg.

2337 A client with abnormal sodium loss is receiving a Correct answer: 1 Processed foods such as cheese are high in sodium content. Ham is high in sodium because it The question requires recall of the sodium content of foods. Note that a food high in
regular diet. To encourage foods high in sodium, the is cured as a preservative process. The addition of these types of foods will supply extra sodium is the correct answer. Choose option 1, since processed and preserved foods
nurse would recommend which of the following foods sodium in the diet. The other options are lower in sodium content. contain a lot of sodium.
for lunch?
1.‐ An American cheese and ham sandwich
2.‐ Chicken salad on lettuce
3.‐ Tossed salad with vinegar dressing
4.‐ White fish and plain baked potato

2338 Which of the following interventions should the nurse Correct answer: 2 Hyponatremia is caused by an excess of water, which dilutes the amount of sodium present in The critical word is dilutional. Eliminate options 1 and 4, since additional water is not
anticipate implementing in a client who is experiencing the plasma. Clients who are experiencing dilutional hyponatremia are in fluid volume excess needed. Eliminate option 3, since additional sodium is not needed.
dilutional hyponatremia? (FVE). It is important to restrict additional fluids, as this can further increase the sodium deficit.
In addition, the client already is in an FVE state, which can lead to development of further
disturbances of fluid balance. Options 1and 4 are incorrect, as hypotonic fluids would further
complicate the hyponatremia. Sodium usually does not need to be replaced in dilutional
states.
1.‐ Administration of hypotonic intravenous solutions
2.‐ Restriction of additional oral fluids
3.‐ Increasing sodium intake in the diet
4.‐ Encouraging intake of tap water

2339 A client is given furosemide (Lasix) 80 mg IV push for Correct answer: 4 Furosemide (Lasix) is a loop diuretic that promotes potassium excretion. A weak pulse is seen Recall that Lasix is a potassium‐wasting diuretic, and that a weak pulse is a sign of
treatment of heart failure. The nurse would be in clients with hypokalemia, and could be attributed to the effects of Lasix administration. It hypovolemia and hypokalemia, to choose option 4.
concerned if the client manifested which of the also could be due to excessive diuresis of sodium and water. Decreased neck vein distention
following signs? and decreased adventitious breath sounds are suggestive of fluid volume reduction, and would
be beneficial for a client with heart failure. A BP of 120/78 is within the normal range.

1.‐ Decreased neck vein distention


2.‐ Decreased adventitious breath sounds
3.‐ BP 120/78
4.‐ Weak pulse

2340 A client has been taking a corticosteroid for six Correct answer: 4 Hypokalemia can result from corticosteroid use. Hypokalemia causes cardiac problems that First, recall that corticosteroids contribute to sodium retention and potassium loss. Then,
months. Which of the following electrocardiogram are manifested on an ECG as flattened T waves, depressed ST segments, and prominent U analyze each option for the EKG change associated with hypokalemia, and choose option 4.
(ECG) findings would indicate to the nurse that the waves. Peaked T waves and prolonged, depressed ST segments are seen in clients with
client is experiencing a serious side effect of this hyperkalemia.
medication?
1.‐ Tall, peaked T waves and depressed ST segments
2.‐ Flattened T waves and flattened U waves
3.‐ Prolonged ST segment and U waves
4.‐ Flattened T waves and depressed ST segments

2341 A client is being treated for diabetic ketoacidosis Correct answer: 4 DKA leads to an increase in the loss of potassium due to diuresis, and insulin leads to Recall the physiology of DKA, recognizing that potassium is lost in the urine. Also recall
(DKA) with large doses of regular insulin. The nurse potassium being shifted into intracellular fluid. Administration of insulin to a client with DKA that potassium is needed to transport insulin into the cell, resulting in hypokalemia.
should observe for which of the following electrolyte causes transcellular shifting that promotes potassium uptake back into the cell. This action can
imbalances? result in hypokalemia. Low calcium is seen with alkalosis, not acidosis. Hyponatremia is usually
seen in clients with DKA.
1.‐ Hyperkalemia
2.‐ Hypocalcemia
3.‐ Hypernatremia
4.‐ Hypokalemia
2342 For which of the following complications related to Correct answer: 2 A client who is on long‐term diuretic therapy is more likely to suffer effects of potassium The core concept is the long‐term use of a potassium‐wasting diuretic. Recognize that long‐
electrolyte imbalance should the nurse assess in a depletion. Tetany is associated with low levels of calcium (hypocalcemia), which can term depletion of potassium can also result in loss of calcium to be directed to option 1.
client who has been taking furosemide (Lasix) for two accompany hypokalemia. Nausea and vomiting are nonspecific complaints not directly
years? associated with loop diuretic therapy. Diabetes is not related to Lasix administration, although
the use of this loop diuretic can cause the client to become hyperglycemic. Respiratory
depression is not associated with use of this medication.
1.‐ Tetany
2.‐ Respiratory depression
3.‐ Nausea and vomiting
4.‐ Diabetes

2343 A client with hypokalemia reports being constipated. Correct answer: 2 Hypokalemia causes decreased gastrointestinal motility that can lead to constipation. Critical words are hypokalemia and constipated. Recall the effect of this imbalance on GI
The nurse provides which of the following explanations Hypokalemia does not promote fluid loss in the gastrointestinal (GI) tract. The effect of motility to be directed to option 2.
to the client about this problem before implementing potassium in the GI tract is on the smooth muscle, with imbalances causing hyper‐ or
treatment measures? hypomotility.

1.‐ "The level of potassium makes the stools very hard and difficult to expel."
2.‐ "Your potassium level makes your bowel less active, and might make you constipated."
3.‐ "The potassium pulls the fluid from the bowel, leading to constipation."
4.‐ "The low potassium level causes the bowel to stop acting, resulting in constipation."

2344 A client's potassium level is 3.0 mEq/L. The nurse Correct answer: 3 Large doses of penicillin, aminoglycosides, and glucocorticoids can lead to hypokalemia. ASA, Note that options 1 and 2 are both analgesics, and that options 3 and 4 are anti‐infectives.
suspects that which of the following medications might Tylenol, and antibiotics are not associated with hypokalemia. It is important for the nurse to be Recall that drugs in the latter category are often associated with electrolyte imbalances to
be contributing to this serum potassium level? aware of potential drug effects on serum electrolyte levels in order to effectively participate in narrow your options. Recognize that gentamicin is an aminoglycoside, a category associated
the client's plan of care. with potassium loss.
1.‐ Acetylsalicylic acid (aspirin, ASA)
2.‐ Acetaminophen (Tylenol)
3.‐ Gentamicin (Garamycin)
4.‐ Azithromycin (Zithromax)

2345 A client in the coronary care unit (CCU) is receiving Correct answer: 4 A client receiving continuous IV therapy containing KCl could become hyperkalemic and The core concept is to recognize that hypokalemia is being treated with replacement of
1,000 mL normal saline with 40 mEq potassium require immediate action. All of the incorrect choices reflect a hypokalemic state. While it is intravenous KCl. Recognize that this electrolyte imbalance can be overcorrected with IV
chloride (KCl) every eight hours for treatment of important to monitor response to treatment and to notify the physician, the priority infusion of KCl. Recognize the EKG changes seen with hyperkalemia to choose option 4.
hypokalemia. Which of the following assessment data information would be that the client has dyspnea and specific ECG changes that are suggestive
would the nurse report to the physician? of hyperkalemia.

1.‐ Complaints of muscle weakness, and an ECG showing ST segment depression


2.‐ Nausea and vomiting, with U waves evident on the monitor
3.‐ Muscle twitching, and ECG showing a flattened T wave
4.‐ Dyspnea and tall, peaked T waves on the monitor

2346 Which of the following should be included in the Correct answer: 3 KCl is never administered as an IM injection or as an IV push medication; that could cause Recall the importance of diluting IV KCl, and eliminate option 2. Recall that potassium is
nurse's plan when preparing to give a client potassium development of potentially lethal cardiac arrhythmias. KCl should be diluted in the correct never given IM to eliminate options 1 and 4.
chloride (KCl)? amount of IV solution, and administered via an infusion pump. Monitoring output is an
expectation of any IV therapy.
1.‐ Use an appropriate‐size needle for IM injection.
2.‐ Administer the KCL undiluted through a continuous running IV.
3.‐ Use an infusion pump when administering KCl.
4.‐ Administer only into the dorsogluteal muscle.
2347 Which of the following interventions would be Correct answer: 1 The client with hypokalemia could develop muscle weakness and therefore require assistance The question asks you to choose the interventions associated with a risk for injury related
appropriate for the client with hypokalemia who has a with ambulation. Monitoring the ECG for changes does not directly relate to injury potential to muscle weakness. Associate safety with assistance, and choose option 1.
nursing diagnosis of Risk for Injury related to skeletal related to general weakness. Medication and dietary interventions might be necessary to
muscle weakness? restore normal serum levels, but do not directly relate to injury potential.

1.‐ Assist the client with ambulation.


2.‐ Monitor the client's ECG for conduction changes.
3.‐ Administer KCl 20 mEq by mouth daily as ordered.
4.‐ Encourage a diet intake high in fiber and fruit.

2348 In explaining to a client the importance of potassium Correct answer: 2 Potassium (K&lt;sup&gt; &lt;/sup&gt;) in the extracellular fluid (ECF) is responsible for Read each option carefully, noting that not all options provide a thorough explanation.
to the body’s functioning, which statement should the conducting nerve impulses, which contribute to cardiac rate and rhythm. The amount of Recall that generally, potassium has neuromuscular, cardiac, and cellular functions.
nurse make? potassium in the ECF is actually a very small amount, explaining why even minor changes can Recognize that option 2 discusses muscle and heart function.
have a major impact on the body. While K&lt;sup&gt; &lt;/sup&gt; is an important electrolyte
in the body, it does not increase the metabolic rate of tissues in the body. K&lt;sup&gt;
&lt;/sup&gt; levels are not the regulators of salt and fluid retention. The body relies on a
balanced level of electrolytes in the body, and K&lt;sup&gt; &lt;/sup&gt; is just one element in
the body's ability to maintain homeostasis.

1.‐ "Potassium is important because it makes the muscle tissue work faster so the heart can pump effectively."
2.‐ "Potassium is vital to support the muscles by making them contract more effectively, including the muscles of the heart."
3.‐ "It is vital that the body have enough potassium so that the kidneys can function effectively to prevent salt and fluid retention."
4.‐ "Muscles are dependent on large amounts of potassium in order to work effectively."

2349 Which of the following serum potassium levels would Correct answer: 1 A client who has diarrhea or nasogastric suctioning will be more likely to develop Critical words are three‐day history of diarrhea. Review normal potassium levels, and
the nurse expect to see in a client in the Emergency hypokalemia. A serum potassium of 3.0 mEq/L is considered to be hypokalemic. A level of 3.6 recall that potassium is lost with diarrhea, to choose the lowest level.
Department with a three‐day history of diarrhea? mEq/L is just within the normal range, but one would expect a greater K&lt;sup&gt;
&lt;/sup&gt; loss, given the client s history of three days of diarrhea. A level of 4.1 mEq/L is
within the normal range, and does not reflect K&lt;sup&gt; &lt;/sup&gt; loss. A level of 5.8
mEq/L is suggestive of hyperkalemia.
1.‐ 3.0 mEq/L
2.‐ 3.6 mEq/L
3.‐ 4.1 mEq/L
4.‐ 5.8 mEq/L

2350 The nurse is instructing a client diagnosed with Correct answer: 3 Salt substitutes have potassium chloride as their main compound, and individuals with high Review foods high in potassium. Note that the question refers to the need for further
hyperkalemia about foods to avoid. Which of the potassium levels should not use salt substitutes. Bananas, tomatoes, and avocados are all instruction, and look for the one incorrect response. Recall content of salt substitutes to
following statements by the client indicates to the foods that are high in potassium, and should be limited in a client with hyperkalemia. Clients choose option 3.
nurse a need for further instruction? should be aware of foods to avoid that are high in potassium if teaching has been successful.

1.‐ “I should avoid eating a lot of bananas.”


2.‐ “I guess I can’t eat all the tomatoes I want this summer.”
3.‐ “I can still use my salt substitute instead of real salt.”
4.‐ “No more avocado salads for me.”

2351 A client is admitted to the hospital with a serum Correct answer: 4 A serum level of 2.8 mEq/L reflects hypokalemia, which often manifests as cardiac and Recognize that the value is indicative of severe hypokalemia. Eliminate option 2, since
potassium level of 2.8mEq/L. The nurse anticipates respiratory problems related to the ineffective smooth muscle contractions. Option 2 reflects findings are normal, and options 1 and 3, since they are not as severe as those in option 4.
assessment findings will include which of the normal findings. The symptoms listed in options 1 and 3 do not indicate severe hypokalemia. A
following? serum potassium of 2.8 mEq/L in conjunction with irregular pulse and shallow respirations is a
symptomatic presentation in this client, and suggests severe hypokalemia. It is important to
look at the whole clinical picture and not just the serum level to determine the severity of an
electrolyte imbalance.
1.‐ Elastic skin turgor, and vomiting a small amount of bile‐stained emesis
2.‐ Pink nail beds, and ECG showing a normal sinus rhythm with a rate of 76
3.‐ Respiratory rate 16 with equal bilateral breath sounds, and two loose stools this morning
4.‐ Irregular pulse rate, and shallow respirations

2352 The nurse instructs a client receiving Correct answer: 1 HCTZ is a potassium‐wasting diuretic, and its use can lead to hypokalemia. Leg cramps and Recall that this diuretic is potassium‐wasting to direct you to option 4.
hydrochlorothiazide (HCTZ) to report which of the muscle weakness are two of the symptoms seen in a client with hypokalemia. Diarrhea,
following symptoms to the health care provider? fatigue, nausea, and irritability are not usually seen with the use of this class of diuretics.

1.‐ Leg cramps and muscle weakness


2.‐ Muscle weakness and diarrhea
3.‐ Fatigue and irritability
4.‐ Nausea and irritability

2353 Which of the following foods should the nurse Correct answer: 2 Spironolactone is a potassium‐sparing diuretic, and clients need to be aware of their intake of This a reverse‐response question, where three options are permissible. Recall that
instruct the client who is taking spironolactone foods high in potassium. Cantaloupes are very high in potassium, and should be avoided. aldactone is potassium‐sparing to choose the food item highest in potassium.
(Aldactone) to avoid? Bread, green beans, and squash are not considered to be good sources of potassium. These
foods do not need to be restricted in the diet.
1.‐ Bread
2.‐ Cantaloupe
3.‐ Green beans
4.‐ Squash

2354 A postoperative client with a serum potassium level Correct answer: 10 The maximum routine rate of infusion for KCl is 5–10 mEq/hour. Clients who are moderately The critical words are exceed and hourly replacement rate. Note that the question
of 3.6 mEq/L is ordered to receive an IV with a hypokalemic may have potassium administered at a rate of 10–20 mEq/hour, but this client is indicates a peripheral line to help you identify an amount that is not high.
potassium supplement (KCl) via a peripheral line. The not moderately hypokalemic. Concentrations of potassium in solution can range from 10 to 40
nurse checks to determine that the amount of KCl mEq/L, and are administered via a peripheral vein with an infusion pump. Higher
ordered does not exceed the standard hourly concentrations of potassium can be administered via a central line in critically ill clients who
replacement rate of _____mEq. are hemodynamically monitored.

2355 The nurse concludes that a client has an Correct answer: 3 Lasix is a potassium‐wasting diuretic that can cause the client to become hypokalemic. This The core concept of the question is the relationship of low potassium to the diuretic.
understanding of the side effects of furosemide (Lasix) can manifest as a weak, thready pulse and onset of orthostatic hypotension. Diarrhea is not Eliminate options 1 and 4, as these are still important actions to take. Eliminate option 1,
and its relationship to potassium levels when the client usually seen as a side effect of this medication. Monitoring of one s pulse is not required for since this is not a usual effect of Lasix.
states: clients taking diuretic therapy, but is necessary for clients taking digoxin, or clients who have a
pacemaker. Bananas are a good source of dietary potassium, and might be warranted for this
client to maintain normal serum potassium levels.

1.‐ “I don’t need to take my pulse anymore when I take my Digoxin.”


2.‐ “I should call the doctor if I develop diarrhea.”
3.‐ “I should call my doctor if I feel myself becoming dizzy when I stand up.”
4.‐ “I don t need to eat bananas for breakfast any more, since I am taking this medication.”

2356 The nurse provides which of the following Correct answer: 3 Aldactone is a potassium‐sparing diuretic, and the intake of potassium‐rich foods should be Recall knowledge of diuretics to eliminate options 1, 2, and 4.
instructions to a client going home with a prescription discouraged. It is important that the client be aware of potassium‐retaining diuretics, since
for spironolactone (Aldactone)? most clients associate diuretics with potassium loss. Diuretics should be taken with food to
decrease GI upset. Diuretics should not be taken before going to bed, because their primary
effect is diuresis. This time frame could cause the client to experience altered sleep patterns
due to nocturia. Clients taking diuretics should be aware of their fluid intake, and monitor
accordingly.
1.‐ “Be sure to take this medication on an empty stomach.”
2.‐ “Take this pill just before you go to bed.”
3.‐ “Cut back on your intake of those foods on your list that are high in potassium.”
4.‐ “You don’t have to watch your intake of fluid while you are taking this medicine.”

2357 The nurse identifies which of the following clients Correct answer: 2, 5 A client with metabolic alkalosis is at risk for developing hypokalemia due to the shift of Recall knowledge of conditions in which potassium is lost, or shifts into the cell, to choose
admitted to the hospital as being at risk for developing potassium to the ICF from the ECF. Clients with NG tubes lose potassium from the stomach, options 2 and 5.
hypokalemia? Select all that apply. and the NPO status limits their intake. Clients with acute renal failure are usually hyperkalemic
due to a decreased ability to excrete potassium. Clients with ARDS are usually hyperkalemic
due to compromised ventilation, resulting in metabolic acidosis. Metabolic acidosis is
associated with hyperkalemia because potassium shifts from the ECF to the ICF as a result of
increase in hydrogen ion concentration.

1.‐ A client whose blood gases indicate metabolic acidosis


2.‐ A client who had developed metabolic alkalosis
3.‐ A client with acute renal failure
4.‐ A client with adult respiratory distress syndrome (ARDS)
5.‐ The client with a nasogastric tube on low intermittent suction

2358 The nurse plans to administer which of the following Correct answer: 3 Sodium bicarbonate will temporarily alkalinize the plasma, causing the potassium to move A critical word is acidosis. Recall that potassium is increased in acidosis to direct you to
intravenous (IV) treatments to a client for treatment of into the cells. NS is an isotonic solution, and therefore will not cause fluid or electrolyte option 3, in which alkalosis is the goal of treatment.
hyperkalemia associated with severe acidosis? shifting. Calcium gluconate is given to blunt the effects on the myocardium; it does not
decrease the serum K&lt;sup&gt; &lt;/sup&gt; level. Insulin and dextrose are given to decrease
K&lt;sup&gt; &lt;/sup&gt; levels by increasing K&lt;sup&gt; &lt;/sup&gt; uptake at the cellular
level.
1.‐ Calcium gluconate, to make the potassium shift from the intracellular fluid (ICF) to the extracellular fluid (ECF)
2.‐ Insulin and dextrose, to make the client hypoglycemic
3.‐ Sodium bicarbonate, to make the client alkalotic so the potassium will shift into the ECF
4.‐ Normal saline (NS), to provide extra sodium so the potassium will move out of the ICF into the ECF

2359 Which of the following potassium levels would be of Correct answer: 4 Clients who take furosemide (Lasix) lose potassium, and are in danger of developing The critical word is greatest. Recall normal potassium levels and eliminate options 1 and 2,
greatest concern to the nurse when seen in a client hypokalemia. The other choices reflect either a normal potassium level (options 2 or 3) or since one is elevated and one is normal. Choose option 4 over 3, since it is more abnormal.
who is taking furosemide (Lasix)? elevated levels (option 1), which would not be consistent with the action of this loop diuretic.

1.‐ 5.4 mEq/L


2.‐ 4.3mEq/L
3.‐ 3.4 mEq/L
4.‐ 3.1 mEq/L

2360 Which of the following statements should the nurse Correct answer: 3 To prevent gastric irritation, oral potassium supplements should be taken with at least 4 Recall that potassium should not be crushed and is irritating to the stomach to eliminate
include when teaching a client about oral potassium ounces of fluid, or with food. Oral potassium medication should not be crushed. The use of a options 1 and 3. Recall that salt substitutes contain potassium to eliminate option 4. Note
supplementation? salt substitute is not recommended when taking potassium as a medication, because it might that options 2 and 3 are opposites, indicating one could be true.
also contain potassium, leading to hyperkalemia. It is important for the client to have an
understanding of potassium medications, potential side effects, and food–drug interactions.

1.‐ “When you take your potassium pill, if you can’t swallow it, you can crush it up and put it in orange juice.”
2.‐ “Potassium should only be taken in the morning on an empty stomach.”
3.‐ “Take your potassium tablet after you have eaten breakfast.”
4.‐ “You can continue to use salt substitute while you are taking your potassium supplement.”

2361 The nurse anticipates that which of the following Correct answer: 1 Clients in chronic renal failure have diminished or no excretion of potassium from the The critical word is hyperkalemia. Review ways in which potassium is lost from the body to
clients would be at greatest risk to develop kidneys, causing hyperkalemia. Clients with intestinal or nasogastric suction, diarrhea, and/or eliminate options 3 and 4 and choose option 1.
hyperkalemia? cirrhosis are more likely to be hypokalemic, due to potassium losses.

1.‐ The client with chronic renal failure


2.‐ The client just diagnosed with cirrhosis
3.‐ The client with intestinal and nasogastric suctioning
4.‐ The client who has had diarrhea for the last four days

2362 The nurse should place highest priority on which of Correct answer: 1 Clients with renal failure have impaired excretion of potassium, resulting in hyperkalemia. The critical words are highest priority. Eliminate option 3, since the client is in renal failure.
the following nursing interventions for a client with Hyperkalemia leads to cardiac conduction problems and possible fatal dysrhythmias. ECG Remember ABCs airway, breathing, and circulation to choose option 1.
renal failure who has a potassium level of 6.8 mEq/L? monitoring is indicated for this type of client. LOC, urinary output, and ABGs are important
monitoring aspects for a client in renal failure, but hyperkalemia is potentially life‐threatening,
and should be addressed first as the primary intervention.

1.‐ Obtain an electrocardiogram (ECG).


2.‐ Evaluate level of consciousness.
3.‐ Measure urinary output.
4.‐ Draw arterial blood gases.

2363 The nurse should include diet teaching regarding Correct answer: 1, 5 HCTZ and Lasix are diuretics that increase the excretion of potassium, so clients should be The question is testing knowledge of potassium‐wasting diuretics. Recall that thiazides
adding potassium‐rich foods if which of the following taught to increase the intake of potassium in their diet. All of the other medications are decrease potassium to choose option 1.
diuretics are ordered? Select all that apply. considered K‐sparing or combination diuretics, and, as such, dietary supplementation would
not be indicated.
1.‐ Hydrochlorothiazide (HCTZ)
2.‐ Spironolactone (Aldactone)
3.‐ Maxizide (Triamterene with hydrochlorothiazide)
4.‐ Midamor (Amiloride)
5.‐ Furosemide (Lasix)

2364 The nurse anticipates using which of the following as Correct answer: 2 Kayexalate (cation exchange resin) is usually administered rectally, and binds potassium in The critical words are most effective, indicating one answer is most correct or the best
the most effective route to administer sodium exchange for sodium in the gastrointestinal tract. It is then excreted through the stool. choice. Eliminate options 1 and 4, since the drug is not given by these routes. Recall that the
polystyrene sulfonate (Kayexalate) ordered for a client Although Kayexalate can be administered orally, it requires administration with an osmotic drug works in the bowel to choose option 2.
who has a serum potassium level of 6.0 mEq/L? agent to prevent constipation, and might not be tolerated as well. This drug is not given by the
IV or subcutaneous routes.
1.‐ Intravenous
2.‐ Rectal
3.‐ Oral
4.‐ Subcutaneous

2365 When caring for a client who has a potassium level of Correct answer: 2 Hypokalemia can lead to alterations in smooth muscle functioning. Smooth muscle Recall the action of potassium on body systems and its action on the neuromuscular
2.8 mEq/L, the nurse should assess for which of the alterations in the gastrointestinal tract can lead to development of a paralytic ileus. system to choose option 2.
following? Complications of hypokalemia are usually not associated with renal failure, diabetes, or a
perforated bowel, as these conditions are more likely to lead to increased potassium levels.

1.‐ Perforated bowel


2.‐ Paralytic ileus
3.‐ Renal failure
4.‐ Diabetes mellitus

2366 The nurse determines that the intravenous (IV) Correct answer: 3 Calcium gluconate is given to antagonize the effects of the potassium on the conduction The question requires recall of the use of calcium in treating hyperkalemia. Eliminate
administration of calcium gluconate to a client with system of the heart. It is not given to promote potassium excretion (either in urine or stool). options 1, 2, and 4, since calcium is not given to eliminate potassium from the body.
hyperkalemia has been effective when which of the The medication acts to blunt the effects of elevated potassium on the myocardium.
following is seen on assessment?
1.‐ Urine output increases.
2.‐ Bowel movements are loose.
3.‐ Cardiac dysrhythmia is corrected.
4.‐ Bowel sounds become less hyperactive.

2367 Which of the following statements by a client Correct answer: 2 Bran flakes are not a source of potassium in the diet. It is important for the client to The question requires reverse thinking; three options are accurate statements and one is
indicates a need for further instruction regarding communicate to the physician if symptoms develop during the course of therapy. Bananas and wrong. Note similarities in options 1 and 2, and note that bran is not a good source of
treatment for hypokalemia? cantaloupe are excellent sources of dietary potassium. Taking potassium supplements on a full potassium to choose that as the incorrect response.
stomach will help to minimize gastric irritation, which is commonly associated with this
medication.
1.‐ “I will eat more bananas and cantaloupes for breakfast.”
2.‐ “I will eat more bran flakes to increase my potassium level.”
3.‐ “I will take my potassium in the morning after breakfast so it doesn’t upset my stomach.”
4.‐ “I will tell my doctor if I start having any of the symptoms on the list you gave me.”

2368 Which of the following is the best response by the Correct answer: 3 Potassium works to maintain cardiac contractility and normal heart rate. Hypokalemia leads The question asks for best answer, indicating that all options might be completely or
nurse to the 22‐year‐old daughter of a 46‐year‐old to the development of potential arrhythmias that can result in ischemia and death. While the partially correct, but one is better. Eliminate option 1, since it does not relate to potassium.
client who was admitted with hypokalemia and is length of bedrest and actual potassium level could be associated with a complaint of dizziness, Eliminate option 3, since the client is 46 years old. Choose option 4 over 2, since it provides
complaining of being dizzy upon standing? it is more likely that the dizziness is associated with orthostatic hypotension and inefficient a more thorough explanation.
heart‐pumping action due to hypokalemia. It is important for the client (and family) to
understand that electrolyte imbalances can have significant complications that can affect the
entire body.
1.‐ “Your mother has just stayed in bed too long, and when she stands up, she will get dizzy.”
2.‐ “The level of your mother’s potassium is making her dizzy.”
3.‐ “Your mother is probably dizzy because her heart is not pumping as effectively, making her blood pressure low.”
4.‐ “Your mother is dizzy because her nervous system isn t functioning correctly; once her potassium level goes up, she will improve.”

2369 When assessing a client with hypocalcemia, the nurse Correct answer: 1 Due to a lack of adequate calcium for neuromuscular transmission, hypocalcemia produces The critical word is hypocalcemia. Recall that there is insufficient calcium in this disorder,
should ask which of the following questions? neuromuscular irritability, which is first manifested by paresthesias, or a sensation of which allows for excessive nerve transmission, resulting in irritability.
numbness and tingling circumorally (around the lips) and in the fingertips and toes. The other
questions pertain to symptoms of hypercalcemia.
1.‐ “Do you have any tingling sensation in your fingers or toes?”
2.‐ “Have you been constipated lately?”
3.‐ “Do you have to urinate frequently?”
4.‐ “Are you having a lot of headaches?”

2370 After receiving an intershift report for the day shift at Correct answer: 4 Fosamax must be administered on an empty stomach with a full glass of water to ensure Knowledge of bisphosphonate is required to answer this question. Recall that this class of
07:00, the nurse is checking medications due to be proper absorption. The presence of food, juices, or other medications will interfere with medications must be administered before the client eats or drinks anything else to be
administered during the shift. The nurse notes that adequate absorption. All of the other options would be inappropriate. directed to option 4.
alendronate (Fosamax) 70mg weekly is scheduled to
be administered at 09:00 with the other daily
medications. The nurse should take which of the
following actions?
1.‐ Administer the Fosamax as ordered at 09:00, but be sure the client drinks at least 16 ounces of water.
2.‐ Adjust administration time of Fosamax to be given with breakfast.
3.‐ Withhold the dose of Fosamax, and reschedule it to be given at bedtime.
4.‐ Administer the Fosamax at 07:30, separating it by at least one hour from breakfast or the other medications.

2371 A client presents with an elevated parathyroid Correct answer: 2 Primary hyperparathyroidism is the most common cause of hypercalcemia in the general Critical words are PTH and etiology. Recall that PTH is secreted by the parathyroid glands
hormone (PTH) level. The nurse suspects that the population. There is excessive secretion of PTH by one or more of the parathyroid glands. and associate the word elevated with hyper‐ in option 2 to choose correctly.
etiology of this PTH elevation is: Excess thyroid‐stimulating hormone (TSH), not PTH, is associated with a hypoactive thyroid
gland. Metastasis rarely produces excessive PTH. Sarcoidosis is associated with unregulated
conversion of activated vitamin D.
1.‐ A hypoactive thyroid gland.
2.‐ Primary hyperparathyroidism, most likely from an adenoma.
3.‐ Pulmonary cancer with metastasis.
4.‐ Sarcoidosis.

2372 A client who is hemorrhaging is receiving massive Correct answer: 3 Citrate binds with calcium, decreasing ionized calcium levels. Acidosis leads to decreased Critical words are massive blood transfusions and low serum calcium. Recognize that
blood transfusions. The nurse notes a low serum binding of calcium, thereby increasing the calcium level. Options 1 and 4 are incorrect because massive transfusions will most likely necessitate use of stored blood, which contains citrate,
ionized calcium level, and concludes that a probable blood transfusion therapy would not warrant the clinical usage of rapid infusion of crystalloids, and choose option 3.
cause for this calcium imbalance is: and hypomagnesemia is not a related consequence of blood therapy.

1.‐ Rapid infusion of crystalloids.


2.‐ Development of metabolic acidosis.
3.‐ The citrate used as an anticoagulant in preserved blood.
4.‐ That hypomagnesemia has occurred.

2373 When a client with pancreatitis reports numbness Correct answer: 4 Pancreatitis causes hypocalcemia because byproducts released from the inflamed pancreas Recognize that the symptoms in the question are not typical signs and symptoms of
around the mouth and tingling in the hands and feet, chelate calcium. Numbness and tingling around the mouth and extremities are characteristics pancreatitis. Recall the physiology of this disease, and recognize that the symptoms reflect
the nurse should make it a priority to assess the client of tetany from hypocalcemia. All of the other options do not relate to this finding. low serum calcium.
for which of the following?

1.‐ Hypercalcemia
2.‐ Hypokalemia
3.‐ Hypophosphatemia
4.‐ Hypocalcemia

2374 A client has a serum calcium level of 11.5 mg/dL, an Correct answer: 1 Due to the inverse relationship of calcium with phosphorus, one would expect low Recognize that calcium levels are elevated, and recall that the relationship to phosphorus
ionized calcium of 5.6 mg/dL, and a 24‐hour urine phosphorus when the calcium is elevated. In most cases, when there are elevated total serum is inverse to calcium. Then determine which options identify abnormal levels to be directed
calcium level greater than 400 mg/dL. The nurse calcium and ionized calcium levels, then hyperparathyroidism needs to be considered. In this to the low phosphorus.
expects that additional review of laboratory test case, the alkaline phosphatase and PTH would also be elevated. Potassium is often low in cases
results will show which of the following associated of hypocalcemia.
electrolyte levels?
1.‐ Phosphorus lower than 2.5 mg/dL
2.‐ Alkaline phosphatase of 40 units/L
3.‐ Parathyroid hormone (PTH) level of lower than 11 picograms/dL
4.‐ Potassium level of 3.2 mg/dL

2375 Which of the following findings relative to calcium Correct answer: 2 A client in renal failure exhibits hypocalcemia in the presence of increased phosphate, The critical term is renal failure. Recall that the kidneys are unable to excrete most of the
would the nurse expect to see in a client in renal potassium, and creatinine levels. Option 1 is incorrect because due to the inverse relationship electrolytes in this condition. Recall that vitamin D synthesis is altered, leading to low
failure? between calcium and phosphorus, one would expect to see decreased calcium levels and calcium.
increased phosphate levels in renal failure. Option 3 is incorrect, as potassium levels would be
increased in renal failure. Option 4 is incorrect, as creatinine levels would be increased as renal
function deteriorates.
1.‐ Increased calcium levels and decreased phosphate levels
2.‐ Increased calcium excretion and phosphate retention
3.‐ Decreased calcium and potassium
4.‐ Increased calcium and decreased creatinine levels
2376 The client has a dietary deficiency of calcium. The Correct answer: 3 In chronic hypocalcemia from dietary deficiencies, oral calcium and vitamin D supplements The critical term is dietary deficiency. Recall that vitamin D is needed for adequate
nurse prepares to administer a dose of which of the might be all that is needed. Calcitriol has a more rapid onset and more rapid clearance from absorption of calcium to be directed to option 3.
following types of medications as ordered? the body, and is especially useful in renal failure. Option 1 is incorrect, as glucocorticoids
increase absorption of calcium in the intestines as well as increase calcium excretion,
worsening the hypocalcemia. Option 2 is incorrect because aluminum preparations should not
be used, due to aluminum accumulation in the bone, resulting in osteomalacia. Option 4 is
incorrect, since bisphosphonate therapy is used to treat hypercalcemia and osteoporosis.

1.‐ A glucocorticoid
2.‐ An aluminum‐containing antacid
3.‐ A calcitriol supplement
4.‐ A bisphosphonate

2377 A client is recovering from a parathyroidectomy to Correct answer: 4 A common cause of hypocalcemia after parathyroidectomy is lack of blood supply to the Recall that the parathyroid glands secrete PTH, which regulates serum calcium and
treat primary hyperparathyroidism. To ensure that remaining parathyroid gland, with a resultant decreased PTH level. To prevent complications phosphorus levels, to be directed to option 4.
nursing diagnosis goals have been met, the nurse and to indicate that goals for this client have been met, the calcium and PTH levels must be
monitors the level of serum ionized calcium and what closely monitored. The serum creatinine and magnesium are important laboratory levels to
other serum laboratory test? monitor, but are not the priority. Option 3 is incorrect, as calcitriol is the activated form of
vitamin D.
1.‐ Serum creatinine
2.‐ Serum magnesium
3.‐ Calcitriol
4.‐ The parathyroid hormone (PTH) level

2378 A client is admitted with an ionized calcium level of Correct answer: 1 Hypocalcemia becomes symptomatic when ionized calcium levels fall below normal, which First, recognize that the level is indicative of hypocalcemia. Then, recall that this condition
3.5 mg/dL. In anticipation of complications, the nurse this laboratory value indicates. The astute nurse is aware that seizures are signs of places the client at risk for seizures to be directed to option 1.
should do which of the following? hypocalcemia, and anticipates complications. All of the other options are pertinent to
hypercalcemia.
1.‐ Institute seizure precautions.
2.‐ Prepare the client for dialysis.
3.‐ Initiate an intravenous infusion of normal saline.
4.‐ Perform an electrocardiogram to assess for shortening QT interval.

2379 A client is admitted with chronic renal failure. The Correct answer: 4 Renal failure can result in hypocalcemia from diminished formation of calcitriol from renal The critical term is chronic renal failure. Recall physiology of renal failure to direct you to
nurse would use which of the following statements to cell damage, as well as from hyperphosphatemia. In renal disease, severe hypocalcemia can option 4.
explain the need to monitor for hypocalcemia? occur from abnormal renal losses of calcium. Serum creatinine would be high due to nephron
destruction in renal failure. Renal colic occurs from hypercalcemia.

1.‐ “Your kidneys do not eliminate as much calcium, so we need to check for signs of hypocalcemia.”
2.‐ “Your calcium level can decrease because it goes down when the creatinine in the bloodstream is high.”
3.‐ “Signs of hypocalcemia will appear before you experience pain from renal colic.”
4.‐ “Your kidneys are unable to produce calcitriol, which is needed to regulate calcium levels in the bloodstream.”

2380 A client presents with a mildly elevated calcium level. Correct answer: 1 Thiazide diuretics can cause mild hypercalcemia because they have calcium‐retaining effects Systematically evaluate each option for factors that would promote intake or retention of
After completing a nursing history, the nurse identifies on the kidney. Increasing urinary output and eating a high‐protein diet would lead to calcium. Eliminate options 2 and 3, since they contribute to loss of calcium. Eliminate
which of the following as a contributing factor to the hypocalcemia. Bisphosphonates lower serum calcium levels by preventing bone resorption of option 4, since this is given for treatment of hypercalcemia or osteoporosis.
abnormal calcium level? calcium.

1.‐ Use of a thiazide diuretic


2.‐ Recent reports of polyuria
3.‐ A high‐protein diet
4.‐ Ingesting a biphosphonate weekly
2381 A client with hypercalcemia is receiving digoxin Correct answer: 2 The client with hypercalcemia is more sensitive to the toxic effects of digoxin (Lanoxin). The core concept is that the client is on digoxin; recall the role of calcium in cardiac
(Lanoxin). The nurse plans to incorporate which of the Frequent apical and radial pulse checks by the nurse will aid in detecting potential contractility and the action of digoxin to direct you to option 2.
following in client assessments? complications. Options 1 and 4 are assessment priorities in hypocalcemia. Auscultation of
bowel sounds is appropriate, but not of high priority with digitalis therapy.

1.‐ Checking for Trousseau’s sign


2.‐ Frequent pulse checks
3.‐ Auscultation of bowel sounds
4.‐ Inspection of skin for signs of bleeding

2382 A client returns to the unit following a thyroidectomy. Correct answer: 1 Hypocalcemia frequently results from accidental removal or destruction of parathyroid tissue Recall that the parathyroid glands can be accidentally removed during surgery, resulting in
The nurse plans to frequently assess for which of the or its blood supply during surgery. Clinical manifestations of tetany include laryngospasm hypocalcemia. Eliminate options 2, 3, and 4, since they are related to hypercalcemia.
following? postoperatively. The other options are assessment criteria representative of hypercalcemia.

1.‐ Signs of laryngospasm


2.‐ Polyuria
3.‐ Hypertension
4.‐ Hypoactive deep tendon reflexes

2383 When developing a plan of care for a client with Correct answer: 3 Tetany and seizures are clinical manifestations of hypocalcemia. The nurse must be aware of Critical words are hypocalcemia and high‐priority, so eliminate options 1 and 4, since they
hypocalcemia, the nurse chooses which of the all potential risks to the client based on physiological factors of the presenting illness, and are associated with hypercalcemia. Choose option 3, since it is of highest priority.
following as a high‐priority nursing diagnosis? must plan for the client s safety. The other nursing diagnoses are appropriate for
hypercalcemia.
1.‐ Potential Complication: Electrolyte Excess
2.‐ Risk for Injury related to sensorium changes
3.‐ Risk for Injury related to tetany and seizures

4.‐ Deficient Fluid Volume

2384 The nurse notes that a client s serum calcium level is Correct answer: 4 Ionized calcium is approximately 40–50% of the total serum calcium. Ionized calcium is the The question requires correlation of a hypocalcemic lab value to its cause. Recall factors
7.9 mg/dL. Since the client has no symptoms of portion of the serum calcium that is not bound to protein and is physiologically active and that influence serum calcium levels to direct you to option 4.
imbalance at this time, the nurse interprets that: clinically important. Hypocalcemia (normal is 9–11 mg/dL) that is due to reduced protein
binding is asymptomatic. The question states that the total serum calcium is low; therefore,
option 1 is incorrect. Ionized calcium levels can remain normal even when total calcium levels
are low. Option 2 is incorrect, as hypocalcemia could be a result of hypomagnesemia, not
hypermagnesemia. Option 3 is incorrect, as phosphorus and calcium are inversely
proportional, so phosphorus would be high.

1.‐ This level reflects only the ionized calcium.


2.‐ The client’s magnesium is high, resulting in false levels of calcium.
3.‐ Phosphorus is low, resulting in low serum calcium levels.
4.‐ This does not reflect the ionized calcium that results in symptomatology.

2385 When caring for the client with signs of severe Correct answer: 2 Ten‐percent calcium gluconate is the treatment option for symptomatic severe hypocalcemia. Critical words are severe hypocalcemia. Eliminate options 1 and 3, since they would be
hypocalcemia, the nurse anticipates administration of: All IV calcium preparations are administered slowly to prevent dysrhythmias and damage to given to increase calcium. Eliminate option 4, recognizing that calcium should never be
veins. Normal saline and IV phosphorus are used to treat hypercalcemia. given rapidly.

1.‐ Isotonic normal saline as a rapid infusion.


2.‐ 10% calcium gluconate by slow IV push.
3.‐ Intravenous phosphorus over 6–8 hours.
4.‐ 10% calcium chloride by rapid IV push.
2386 A client who has a serum calcium level of 11.8 mg/dL Correct answer: 2 Calcium levels of 11–12 mg/dL indicate hypercalcemia. The mainstay of treatment of This question requires you to look for the absence of signs indicative of hypercalcemia.
is receiving a 0.9% sodium chloride infusion. The nurse hypercalcemia (normal is 9–11 mg/dL) is to increase renal calcium excretion with extracellular Recognize that option 1 could indicate that the calcium level has been brought down to
determines that hydration has been effective when: volume expansion. Chvostek s sign indicates hypocalcemia, which would not be an normal level, or is reduced, to choose it.
overcorrection of treatment. Serum creatinine elevation indicates that renal function is
diminished, and is therefore not an effective response to therapy.

1.‐ Chvostek’s sign is positive.


2.‐ Volume status has been restored.
3.‐ Calcium level is 11–12 mg/dL.
4.‐ Serum creatinine is elevated.

2387 The nurse caring for a client with a calcium imbalance Correct answer: 4 Although all systems are impacted by calcium imbalance, the major clinical manifestations of Note that the question does not specify if calcium is decreased or elevated. Recall the
places highest priority on nursing interventions that calcium imbalance are due to either increased or decreased neuromuscular irritability. major system affected by calcium to choose option 4.
help to manage:
1.‐ Renal signs and symptoms.
2.‐ Cardiac changes.
3.‐ Hematologic disorders.
4.‐ Neuromuscular clinical manifestations.

2388 The nurse determines that a client with a serum Correct answer: 4 The client with hypercalcemia (normal is 9–11 mg/dL) should increase fluid intake to 2–3 Recall the treatment modalities for hypercalcemia to answer this question. Recognize that
calcium level of 12 mg/dL understands client teaching liters a day. Hydration leads to increased calcium excretion, and prevents the development of TUMS are a source of calcium, and that bedrest would increase bone reabsorption of
when the client states: kidney stones. TUMS is a calcium carbonate supplement that can be used to increase calcium; calcium, to eliminate options 1 and 3. Choose option 4 as providing the most complete
the client in the question already has hypercalcemia, so this is not indicated. Phosphorus answer.
supplements can decrease calcium, but need to be taken more than once a day. Strict bedrest
leads to increased calcium from osteoclastic activity.

1.‐ “If my stomach becomes upset, I can just take more TUMS.”
2.‐ “I’ll need to take my phosphorus supplements once a day.”
3.‐ “I’ll need to be on strict bedrest to help with this problem.”
4.‐ “I ll need to drink many more fluids than I have been, even up to 2–3 liters each day.”

2389 A client with hypocalcemia asks the nurse why Correct answer: 2 Calcium plays a unique role in the regulation of many enzymes and intracellular signaling. The critical word is unique. Note that all the options are functions of electrolytes, but only
calcium is so important. The nurse makes which of the Although calcium does play an important role in acid–base balance, other electrolytes do this one is unique to calcium.
following statements to describe the function that is as well. Sodium and potassium are also needed for heart muscle contraction. Sodium is
unique to calcium? primarily responsible for shifts in body water.
1.‐ “It helps to regulate acid–base balance.”
2.‐ “It activates enzymes that stimulate many chemical reactions.”
3.‐ “It stimulates the heart muscle so it will produce a contraction.”
4.‐ “It helps the body to maintain overall water balance.”

2390 The results of a client s laboratory tests show Correct answer: 3 In hyperparathyroidism, the ionized calcium is almost always elevated. In Critical words are elevated and parathyroid. Eliminate options 1 and 4, since these would
elevated ionized calcium and parathyroid hormone hyperparathyroidism, the level of intact PTH is elevated, and is best interpreted in conjunction be seen with hypocalcemia, and eliminate option 2, since it would be related to
(PTH) levels. The nurse suspects that the cause is: with ionized calcium. PTH is suppressed in clients with most other causes of hypercalcemia, hypercalcemia only.
which makes the other options incorrect.
1.‐ Hypoparathyroidism.
2.‐ A malignancy.
3.‐ Hyperparathyroidism.
4.‐ Vitamin D deficiency.
2391 A client develops hypocalcemia as a result of Correct answer: 1 Prolonged NGT suctioning leads to metabolic alkalosis. Changes in pH will alter the level of Recognize that NG suctioning removes acidic fluids, resulting in an alkalotic state. Recall
prolonged nasogastric (NG) tube suctioning. The nurse ionized calcium. Alkalosis increases calcium binding to albumin, leading to a decrease in that calcium salts are bound in alkalosis and that serum levels decrease to choose option 1.
concludes that the primary cause for hypocalcemia at ionized calcium. There might be fluid shifts from hypoalbuminemia, but this would not be due
this time is: to NG tube suctioning. Hypomagnesemia can be a cause of hypocalcemia. Metabolic acidosis
decreases calcium binding to albumin, leading to more ionized calcium.

1.‐ Metabolic alkalosis.


2.‐ Fluid shifts from hypoalbuminemia.
3.‐ Hypermagnesemia.
4.‐ Metabolic acidosis.

2392 A client has a diagnosis of ovarian cancer, and is Correct answer: 2 Many malignant tumors produce chemicals that are carried in the blood and cause release of Recall the physiology of malignancy to stimulate release of calcium from the bones to
undergoing chemotherapy. Because the client s calcium from the bones, most commonly in association with ovarian cancer, renal cell direct you to option 2.
calcium level is now elevated, the nurse suspects carcinoma, and breast cancer, among others. Several antineoplastic medications cause
which of the following? hypocalcemia; lack of dairy products and pancreatitis cause hypocalcemia.

1.‐ Antineoplastic medications are the cause for this elevation in calcium.
2.‐ The ovarian cancer has metastasized, causing the increase in calcium.
3.‐ The client is not eating enough dairy products, as a result of decreased appetite.
4.‐ The client is developing pancreatitis.

2393 A client with hypocalcemia has been started on Correct answer: 2 Large doses of glucocorticoids decrease calcium absorption in the intestines, leading to a Critical words are hypocalcemia and corticosteroids. Recall the signs and symptoms of
intravenous (IV) corticosteroids. Which of the further decrease in serum calcium levels. A positive Chvostek’s sign indicates hypocalcemia hypocalcemia to direct you to option 2.
following findings would indicate to the nurse a further and hypomagnesemia. A positive Trousseau’s sign would be seen with hypocalcemia. Polyuria
decrease in calcium level in the client? and muscle weakness are seen with hypercalcemia.

1.‐ Absence of Trousseau's sign


2.‐ Positive Chvostek’s sign
3.‐ Muscle weakness
4.‐ Frequent urination

2394 When assessing a client with hypercalcemia, the Correct answer: 3 Hypercalcemia causes decreased neuromuscular irritability, while hypocalcemia has clinical Recall that excessive calcium results in decreased transmission at the neuromuscular
nurse concludes that which of the following findings in manifestations that indicate increased neuromuscular irritability. Options 1, 2, and 4 are all junction to direct you to option 3.
the neuromuscular examination is consistent with that signs of increased neuromuscular irritability, signs of hypocalcemia.
electrolyte imbalance?
1.‐ Tetany
2.‐ A positive Trousseau’s sign
3.‐ Muscle weakness
4.‐ Hyperactive deep tendon reflexes

2395 When caring for a client with hypercalcemia who is Correct answer: 2 The cardiac effects of hypercalcemia include shortened plateau phase of the action potential, This question requires knowledge of ECG interpretation and changes associated with
on a cardiac monitor, the nurse checks the cardiac which causes shortening of the QT interval. Atrial fibrillation can develop, but heart block (with electrolyte abnormalities. Recall ECG changes associated with high calcium levels to direct
rhythm strip for which of the following typical prolonged PR interval) is more of a concern with hypercalcemia, due to delayed you to option 2.
changes? atrioventricular conduction. Peaked T waves are associated with hyperkalemia.

1.‐ Development of atrial fibrillation


2.‐ Shortening of the QT interval
3.‐ Shortening of the PR interval
4.‐ Peaked T wave
2396 A nurse prepares to administer calcium gluconate to Correct answer: 1 Hypoparathyroidism is characterized by hypocalcemia and hyperphosphatemia, and is often The critical words are post‐thyroidectomy. Recall anatomy and function of the parathyroid
a client post‐thyroidectomy. The nurse explains to the associated with tetany. Hypoparathyroidism usually results from accidental removal of or gland to choose option 1.
licensed practical nurse (LPN) that this replacement damage to parathyroid glands during thyroidectomy. Because hypocalcemia can be severe,
therapy is needed: prolonged parenteral administration of calcium might be necessary to avoid serious
postoperative complications. Hypoparathyroidism results from a deficiency or absence of PTH;
therefore, option 2 is incorrect. Immobility results in hypercalcemia, making option 3 incorrect.

1.‐ Because of accidental removal of the parathyroid gland.


2.‐ Because it is related to increased parathyroid hormone (PTH) release during surgery.
3.‐ To prevent complications from immobility postoperatively.
4.‐ Due to hypophosphatemia after this type of surgery.

2397 A client presents with complaints of fatigue, Correct answer: 4 Symptoms of fatigue, headache, and increasing muscle weakness are clinical manifestations First determine that the symptoms reflect hypercalcemia. Eliminate options 1, 2, and 3,
headache, and increasing muscle weakness, and has of hypercalcemia. Increased hydration is needed to reduce the serum concentration and aid in since these would all increase calcium levels even further. Alternatively, note that options 3
blood work drawn to evaluate the serum calcium level. elimination. All of the other options will worsen the client s symptoms and increase and 4 are opposites, which is a clue that one of them might be the correct choice.
The nurse anticipates medical management for an hypercalcemia. Thiazide diuretics inhibit calcium excretion; vitamin D supplements will
abnormal value to include which of the following? increase absorption of vitamin D in the intestine; and fluid restriction will cause
hemoconcentration, leading to increased serum calcium.
1.‐ Thiazide diuretics
2.‐ Vitamin D supplements
3.‐ Fluid restriction
4.‐ Increased hydration

2398 The nurse evaluates that discharge teaching has been Correct answer: 2 Tingling or numbness around the mouth is called circumoral paresthesia, and is a sign of Review nursing and medical interventions to increase calcium levels, and eliminate option
effective when the client with hypocalcemia states: impending tetany. A health care provider should be notified immediately. TUMS is a brand 1, since TUMS are a source of calcium. Recall early warning signs of tetany to direct you to
name for calcium carbonate, which can be used as a calcium supplement when calcium intake option 2.
is inadequate. To prevent hypocalcemia, the client should increase the protein in the diet.
Kidney stones are a sign of hypercalcemia.
1.‐ “I shouldn t take antacids, such as TUMS.”
2.‐ “I should notify my health care provider if I start to feel tingling or numbness around my mouth.”
3.‐ “I will need to cut down on the amount of protein I include in my diet each day.”
4.‐ “I will watch my urine for signs of kidney stones.”

2399 The nurse correlates which of the following Correct answer: 3 Parathyroid hormone (PTH) is important in renal magnesium regulation, and This question requires specific knowledge of each diagnosis listed. Recall the role of PTH to
conditions with increased renal wasting of magnesium, hypoparathyroidism is associated with renal wasting of magnesium. The other conditions listed magnesium to be directed to option 3.
and so plans to monitor the client for signs of do not have this effect.
hypomagnesemia?
1.‐ Hypothyroidism
2.‐ Hypertension
3.‐ Hypoparathyroidism
4.‐ Hyperparathyroidism

2400 When caring for a client with hypercalcemia, the Correct answer: 1 Calcium competes with magnesium in the loop of Henle; therefore, an elevated calcium level The question requires knowledge of the effect of calcium on magnesium. Recall the
nurse would expect to see which of the following could decrease renal absorption of magnesium, causing hypomagnesemia. antagonistic relationship to be directed to option 1.
effects on the magnesium level?
1.‐ A decreased magnesium level
2.‐ A rise in magnesium level
3.‐ No change from baseline magnesium level
4.‐ A normal magnesium level
2401 The nurse anticipates that a client receiving which of Correct answer: 2 Furosemide, a loop diuretic, increases urinary output and excretion, decreasing magnesium Analyze each option for the drug's effect on magnesium. Recall that loop diuretics can
the following drugs is at risk for magnesium reabsorption. The following drugs promote magnesium loss: aminoglycosides, potassium‐ cause magnesium loss to choose option 2.
imbalance? wasting diuretics (such as furosemide), cortisone, amphoteracin B, and digoxin. Cimetidine (a
histamine 2 receptor blocker), erythromycin (an antibiotic), and aspirin (an antipyretic,
analgesic, and antiplatelet agent) do not have this effect.

1.‐ Cimetidine (Tagamet)


2.‐ Furosemide (Lasix)
3.‐ Erythromycin (E‐Mycin)
4.‐ Aspirin (generic)

2402 A prenatal client is admitted for pre‐eclampsia. The Correct answer: 1 Low magnesium produces clinical manifestations such as seizures, tremors, spasticity, and The critical word is pre‐eclampsia. Recall that this disorder is associated with
nurse would include in client teaching that magnesium increased reflexes. Magnesium sulfate is the preferred agent for prevention and treatment of hypomagnesemia, placing the client at risk for seizures, to be directed to option 1.
is a standard form of therapy to accomplish which of seizures in pre‐eclampsia and eclampsia due to its efficacy and low neonatal morbidity. The
the following? other options are incorrect reasons for administering this drug.

1.‐ Control seizures.


2.‐ Maintain intrauterine homeostasis.
3.‐ Achieve optimal strength of contractions.
4.‐ Increase renal function.

2403 The nurse assesses for which of the following Correct answer: 2, 4, 5 The ECG and actual magnesium level can provide information about the severity of the Specific knowledge of EKG and magnesium imbalances is needed to answer this question.
electrocardiogram (ECG) changes that can occur in a electrolyte imbalance. A prolonged QT interval might be evident on an ECG due to lengthening Review this material if this question was difficult to answer.
client whose laboratory test results reveal of the ST segment. The T wave might be flattened, while the QRS might have diminished
hypomagnesemia? Select all that apply. voltage. The ST segment is not elevated. Changes in the ST segment and T wave can ultimately
precipitate ventricular tachycardia, while a prolonged QT interval can precipitate heart block.

1.‐ ST segment elevation


2.‐ Prolonged QT interval
3.‐ Absent Q wave
4.‐ Flattened T wave
5.‐ Depressed ST segment

2404 The nurse would expect a client diagnosed with Correct answer: 4 An elevated serum magnesium level is usually due to renal insufficiency and the decreased Recall conditions contributing to magnesium retention. Associate decreased renal
hypermagnesemia to have which of the following ability of the kidneys to excrete magnesium. Diabetes and chronic alcoholism can lead to excretion of electrolytes to hypermagnesemia, and choose option 4.
disorders listed in the medical record? hypomagnesemia. Hypertension is not part of the clinical picture.

1.‐ Chronic alcoholism


2.‐ Diabetes mellitus
3.‐ Hypertension
4.‐ Renal insufficiency

2405 The nurse would assess for hypermagnesemia when Correct answer: 1 Magnesium sulfate is a cathartic used to stimulate peristalsis and increase elimination of The critical words are magnesium sulfate and repeatedly used. Recall that magnesium has
magnesium sulfate is repeatedly used to control which stool. It is not used to treat vomiting, anorexia, or fever. laxative properties to be directed to option 1.
of the following symptoms?
1.‐ Constipation
2.‐ Vomiting
3.‐ Anorexia
4.‐ Fever
2406 When caring for a client with hypomagnesemia, the Correct answer: 3, 5 Magnesium deficiency can contribute to decreased levels of calcium, potassium, and Recall the electrolytes associated with magnesium imbalances. Recognize that potassium
nurse should closely monitor levels of which other phosphates. For this reason, each of these electrolyte levels should be monitored. and calcium are frequently decreased along with magnesium to choose option 3.
electrolyte? Select all that apply.
1.‐ Sodium
2.‐ Chloride
3.‐ Phosphate
4.‐ Bicarbonate
5.‐ Calcium

2407 The nurse determines that a client with Correct answer: 1 Tapping on the facial nerve below the temple is the method of testing for Chvostek's sign. A Recall how Chvostek's sign is checked. Associate the “ch” in Chvostek with the tapping of
hypomagnesemia has a positive Chvostek's sign after positive sign would be a twitch of the nose or lip. the “ch” in cheek. Recognize that options 2 and 3 relate to the eye (too high on the face),
noting which of the following responses to and eliminate them. Recall that this is a physical sign that is observed to choose option 1.
stimulation?
1.‐ Twitch of the nose or lip
2.‐ Blink
3.‐ Lacrimation
4.‐ Pain

2408 The nurse determines that treatment for Correct answer: 4 Effects of hypomagnesemia are mainly due to increased neuromuscular responses, and are The critical word is effective. Recognize that three of the options are similar, and indicate
hypomagnesemia has been most effective when the manifested by neuromuscular irritability, increased deep tendon reflexes, and signs of tetany. neuromuscular irritability still exists, and eliminate them. Associate the word normal in
client demonstrates which of the following? Decreased or normal deep tendon reflex indicates that the treatment was effective. option 4 with correction of the magnesium deficiency.

1.‐ A slight improvement in muscle paralysis


2.‐ Paresthesias of the hands
3.‐ Numbness in the extremities
4.‐ Normal deep tendon reflexes

2409 When caring for a client receiving intravenous (IV) Correct answer: 1 Excessive or too‐rapid infusion of magnesium sulfate can result in a rapid rise of serum The critical words in the question are IV, magnesium sulfate, and complications. Recall the
replacement of magnesium sulfate, the nurse should magnesium, which can manifest itself as respiratory depression or decreased deep tendon need to infuse magnesium slowly to prevent rapid increases in plasma levels to direct you
plan to monitor the client for which of the following reflexes. Abdominal cramping can occur secondary to diarrhea with oral magnesium sulfate to option 1.
potential complications? supplementation. Respirations would decrease, not increase. Headaches would not be caused
by high magnesium levels.
1.‐ Rebound hypermagnesemia
2.‐ Abdominal cramping
3.‐ Tachypnea
4.‐ Headaches

2410 The nurse doing health promotion with a group of Correct answer: 1 Magnesium binds calcium to tooth enamel, and thus helps to maintain the health of teeth. Critical words are magnesium, functions, and teeth. Recall functions of magnesium to
clients should explain that magnesium has which of the The other responses are incorrect statements. choose option 1.
following important functions to maintain the health
of teeth?
1.‐ Binds calcium to tooth enamel.
2.‐ Decreases risk of gum disease.
3.‐ Protects teeth from bacteria.
4.‐ Raises serum calcium levels.

2411 The nurse would recommend to a client who has Correct answer: 2, 3, 5 Legumes, seafood, and whole grains are high in magnesium. The other options listed contain This question requires specific knowledge to make the correct choices. Recall food sources
hypomagnesemia that the client increase intake of either low or trace amounts of magnesium. high in magnesium to choose options 2, 3, and 5.
which of the following foods? Select all that apply.

1.‐ Rice
2.‐ Seafood
3.‐ Legumes
4.‐ Fresh fruit
5.‐ Whole grains

2412 The nurse anticipates that which of the following Correct answer: 2 Treatment for hypermagnesemia is to promote urinary excretion of magnesium to decrease First, determine that the magnesium level is elevated. Eliminate option 1, since it is a
treatments would be used for a client who has a serum levels, so a diuretic might be indicated. Laxatives and antacids often contain source of magnesium. Eliminate option 4, since it would inhibit magnesium excretion, and
magnesium level of 2.5 mEq/L? magnesium, which could worsen the imbalance. Fluid restriction would be contraindicated, option 3, because it is not useful.
because it would prevent flushing of excess magnesium from the body.

1.‐ Magnesium oxide (MagOx)


2.‐ Furosemide (Lasix)
3.‐ Calcium carbonate (TUMS)
4.‐ Fluid restriction

2413 The nurse anticipates that which of the following Correct answer: 4 Renal failure interferes with excretion of electrolytes, including magnesium. All of the Critical words are hypermagnesemia and risk. Eliminate options 1 and 2, since intake of
clients is at risk for hypermagnesemia? conditions listed in the incorrect options increase the risk of hypomagnesemia by interfering magnesium is reduced in these conditions. Recall that magnesium is absorbed in the small
with magnesium absorption in the small intestine. intestine, and eliminate option 3.
1.‐ An anorexic 16‐year‐old female
2.‐ A 57‐year‐old alcoholic male
3.‐ A 47‐year‐old female with a history of partial gastrectomy
4.‐ A 62‐year‐old male with chronic renal failure

2414 The nurse explains to a new nurse orientee that a Correct answer: 2 The hyperglycemic diabetic client experiences osmotic diuresis and polyuria, which increase Recognize that hyperglycemia produces a hyperosmotic state in the blood, leading to
hyperglycemic diabetic client might experience low the risk of excess urinary excretion of magnesium, leading to lowered magnesium levels. The diuresis, to direct you to option 2.
magnesium levels due to which of the following? other options are incorrect.

1.‐ Liver toxicity


2.‐ Osmotic diuresis
3.‐ Kidney failure
4.‐ Low serum osmolarity

2415 The nurse would assess for which of the following Correct answer: 4 Oral administration of magnesium can cause diarrhea, which would further decrease The critical word is oral. Recall that magnesium is a common ingredient in laxatives to
common side effects when administering oral magnesium absorption. The other options are unrelated items. direct you to option 4.
magnesium to a client?
1.‐ Decreased appetite
2.‐ Decreased urine output
3.‐ Increased thirst
4.‐ Diarrhea

2416 The nurse would assess for signs of hypomagnesemia Correct answer: 4 Gentamicin is one medication whose use can lead to hypomagnesemia. All the conditions or Eliminate options 1 and 4, since these conditions would contribute to an increase of
in which of the following clients? circumstances listed in the other options increase serum magnesium levels by enhancing the magnesium. Eliminate option 2, since this would reduce the amount of magnesium lost in
absorption of magnesium by the small intestine or interfering with its excretion by the kidney. the urine.

1.‐ A client with a history of laxative abuse


2.‐ A client who is noncompliant with diuretic therapy
3.‐ A client who takes magnesium‐containing antacids
4.‐ A client who is taking gentamicin (Garamycin)
2417 A client is admitted with new‐onset renal failure. The Correct answer: 2 Neuromuscular symptoms such as depressed deep tendon reflexes are among the most Recall magnesium’s role in the regulation of acetylcholine at the neuromuscular junction
nurse would observe for which most common clinical common clinical manifestations of hypermagnesemia. Decreased respirations, hypotension, to direct you to option 2.
manifestation of hypermagnesemia? and ventricular arrhythmias also can occur in some clients, but are not the most common
signs.
1.‐ Palpitations
2.‐ Decreased deep tendon reflexes
3.‐ Decreased respirations
4.‐ Hypertension

2418 After treating hypomagnesemia with IV fluids, a Correct answer: 2 The repeat serum magnesium level is very high. Calcium gluconate is an antagonist of Recognize that the magnesium level is dangerously elevated. Recall the need to antagonize
client's repeat serum magnesium level is 4.0 mEq/L. magnesium, and is used intravenously to counteract toxicity. the cardiac and muscular effects of excessive magnesium to choose option 2.
The nurse anticipates receiving an order for which of
the following medications?
1.‐ Dextrose
2.‐ Calcium gluconate
3.‐ Potassium chloride
4.‐ Sodium chloride

2419 The nurse would expect a client to have a high serum Correct answer: 3 Many laxatives are magnesium‐based compounds. Overuse could result in increased The question requires you to correlate a high magnesium level to a cause. Eliminate
level of magnesium after seeing which of the following absorption of magnesium and decreased kidney excretion. The other problems listed do not options 1 and 3, since they would contribute to lowering magnesium. Recall the content of
health problems listed in the medical history? elevate magnesium levels. many laxatives to choose option 3.

1.‐ Malabsorption
2.‐ Anemia
3.‐ Overuse of laxatives
4.‐ Alcoholism

2420 The nurse should assess for which of the following Correct answer: 4 Deep tendon reflexes (DTRs) can be diminished or absent when magnesium levels are high The critical word is manifestations, and the concept is hypermagnesemia. Recall the role
classic manifestations in a client with a magnesium (normal is 1.4–2.1 mEq/L). This is because magnesium diminishes acetylcholine activity at the of magnesium in regulating neuromuscular conduction to direct you to option 4.
level of 2.5 mEq/L? myoneural junction, impairing impulse transmission.
1.‐ Diarrhea
2.‐ Hyperreflexia
3.‐ Hypertension
4.‐ Diminished deep tendon reflexes

2421 The nurse is educating the client who has a Correct answer: 3 Clients should be instructed to eat foods that are high in magnesium in order to raise blood Simply remember that food supplies the electrolytes needed by the body. Use this concept
magnesium level of 1.2 mEq/L. What information is levels to within the normal range. Read each option carefully, and analyze the need for to focus on diet counseling as the correct option.
most important for the nurse to include in discussions teaching to correct the magnesium imbalance. Recognize that option 3 will help to provide
with the client? correction of the condition, and choose it.
1.‐ Avoid hazardous activities.
2.‐ Weekly laboratory evaluation
3.‐ Diet counseling
4.‐ Moderate alcohol consumption

2422 A client who takes a mild diuretic complains of leg Correct answer: 4 Magnesium deficiency often coexists with other electrolyte imbalances, especially decreased Determine if the electrolyte levels are elevated, normal, or decreased. Then recall that leg
cramps. After determining that the client s potassium calcium and potassium. Leg cramps are a manifestation of hypomagnesemia. The calcium and cramps can also be precipitated by low magnesium to direct you to option 4.
level is normal, the nurse suspects that which of the phosphorus levels are elevated, and chloride would not be a causing leg cramps.
following abnormal laboratory studies could be the
underlying cause?
1.‐ Calcium level of 11.6 mg/dL
2.‐ Chloride level of 90 mg/dL
3.‐ Phosphorus level of 4.8 mg/dL
4.‐ Magnesium level of 1.2 mEq/L

2423 A client with chronic renal failure has a magnesium Correct answer: 1, 4 A magnesium level of 2.8 is elevated (normal is 1.4–2.1 mEq/L), most likely as a result of First, recognize the magnesium level is elevated. Recall foods high in magnesium to choose
level of 2.8 mEq/L. When reviewing the client’s dietary inadequate renal secretion secondary to the chronic renal failure. Foods high in magnesium options 1 and 4.
history, the nurse identifies which of the following include whole grains; legumes; oranges; bananas; green, leafy vegetables; and chocolate.
frequently eaten foods as a possible cause of this
laboratory value? Select all that apply.

1.‐ Hot chocolate


2.‐ Apples
3.‐ Pork sausage
4.‐ Spinach salad
5.‐ Swiss cheese

2424 A client admitted with a history of alcoholism has a Correct answer: 4 Decreased magnesium levels also contribute to reductions in potassium, calcium, and First, recognize that the magnesium level is decreased. Then, recall which electrolyte
magnesium level of 1.2 mEq/L. The nurse should also phosphate, since all are cations involved in cellular metabolisms. might be abnormal in the presence of low magnesium to direct you to option 4.
plan to check the results of serum laboratory studies
for which of the following?
1.‐ Elevated potassium
2.‐ Elevated phosphorus
3.‐ Decreased sodium
4.‐ Decreased calcium

2425 The nurse who is teaching a review of basic nutrition Correct answer: 2 Magnesium decreases the amount of acetylcholine activity, thereby causing muscle This question requires you to translate the chemical action of magnesium to its
is discussing the effects of various electrolytes and relaxation. The other responses are incorrect. physiological effect in the body. Recall the action of acetycholine on neuromuscular
minerals in the body. In describing the action of function to direct you to option 2.
magnesium, the nurse would explain that it diminishes
acetylcholine and thus acts as a:

1.‐ Nerve stimulant.


2.‐ Muscle relaxant.
3.‐ Vitamin metabolizer.
4.‐ Stimulant for blood sugar.

2426 Following bowel resection surgery, a client’s Correct answer: 1 Hyperactive reflexes are early signs of tetany; the low magnesium level (normal is 1.4–2.1 Recognize the critical level of the magnesium level. Recall that this imbalance can lead to
magnesium level is 1.0 mEq/L. Which of the following mEq/L) could lead to tetany and seizures, and should be reported to the physician. The other seizures to direct you to option 1.
assessment findings should be reported to the symptoms can be related to electrolyte imbalances, but might also be secondary to the
physician immediately? surgery, effects of anesthesia, and narcotic analgesics given for pain.

1.‐ Hyperactive reflexes


2.‐ Nausea
3.‐ Anorexia
4.‐ Abdominal pain

2427 The nurse doing health promotion instructs the 28‐ Correct answer: 3 The RDA for magnesium is 310–320 mg for young adult women, and 400–420 mg for young Recall RDA requirements to answer this question. Beyond that, questions that require
year‐old male client to take in how many milligrams of adult men. Extra requirements beyond this amount are needed during pregnancy and numbers for answers can tend to “bury” the number in the middle of the set, rather than as
magnesium to be within the recommended dietary lactation. the first or last option, where it could be more obvious.
allowance (RDA) for magnesium?

1.‐ 75 mg
2.‐ 200 mg
3.‐ 400 mg
4.‐ 650 mg

2428 When caring for a client with a magnesium level of Correct answer: 4 Sources of magnesium in the diet include green, leafy vegetables; nuts; legumes; whole Specific knowledge of the mineral content of various types of foods is needed to answer
1.1 mEq/L secondary to malabsorption, the nurse grains; seafood; bananas; oranges; and chocolate. the question. Recall foods high in magnesium to choose option 4.
encourages the client to increase the intake of which
of the following foods?
1.‐ Poultry
2.‐ Tomatoes
3.‐ Dairy products
4.‐ Nuts

2429 The nurse evaluates a client admitted to the hospital Correct answer: 3 Use of drugs such as chlorothiazide diuretics can lead to hypochloremia, a decrease in serum The critical words are diuretic and six months. Recognize that the diuretic is a thiazide
with a history of taking chlorothiazide (Diuril) as a chloride, because the client is losing chloride in the urine. Options 1 and 2 indicate increased type. Recall that it causes sodium and chloride losses and calcium retention to be directed
diuretic for the past six months in an attempt to lose serum electrolyte levels that would not occur with the use of this class of diuretic drug. Option to option 4.
weight. Which of the following disturbances in 4 is incorrect because thiazide diuretics cause retention of calcium by increasing the action of
laboratory values should the nurse assess as part of parathyroid hormone on the kidneys, and therefore hypercalcemia would be expected.
the general nursing care plan?

1.‐ Hyperchloremia
2.‐ Hypermagnesemia
3.‐ Hypochloremia
4.‐ Hypocalcemia

2430 Preadmission laboratory work was drawn for a client Correct answer: 2 Bromides can cause a false elevation of chloride levels. The client's serum sodium level is at The critical word is bromide. Recognize that the chloride level is excessively high in
admitted to the hospital for surgery. The results the high end of normal range, and therefore, given the client's history, a repeat lab draw would relation to sodium to be directed to option 2.
indicated a chloride level of 120 mEq/L and sodium be indicated. All of the other options are not warranted, given the client's history of bromide
level of 145 mEq/L. The client reported a history of ingestion.
taking bromides for several days. The nurse should
expect the client to receive which of the following
laboratory assessment orders next?

1.‐ Reassess the sodium level.


2.‐ Reassess the chloride level.
3.‐ Draw a potassium level.
4.‐ Draw a magnesium level.

2431 A client treated for hypochloremia is being prepared Correct answer: 1 It is important for the client to know foods and other dietary items that are high in chloride, The critical word is hypochloremia. Recall that sources of chloride include salt, dairy
for discharge. The nurse determines that the discharge such as table salt. All of the other options are inappropriate actions because they will decrease products, and processed foods to choose option 1.
instructions have been understood when the client serum chloride levels.
makes which of the following statements?

1.‐ "I will increase salt in my diet."


2.‐ "I will increase my dose of diuretics."
3.‐ "I will reduce processed foods in my diet."
4.‐ "I will reduce foods such as lettuce, tomatoes, and celery in my diet."

2432 A client was admitted to the hospital with a diagnosis Correct answer: 3 Hemodilution of body fluids can decrease the serum chloride level. Options 1 and 4 are The question asks for identification of risk factors contributing to losses of chloride.
of hypochloremia. The nurse interprets that which of incorrect because both increased adrenocortical hormones and seawater will increase serum Analyze each option to determine that excess water will contribute to dilution of the
the following would predispose the client to this chloride levels. Option 2 is incorrect because increased environmental temperature causes chloride, and choose option 3.
condition? perspiration and loss of fluids.
1.‐ Increased adrenocortical hormone
2.‐ Increased environmental temperature
3.‐ Drinking excessive amounts of plain water
4.‐ Drinking excessive amounts of bottled sports drinks

2433 When caring for a client who has second‐ and third‐ Correct answer: 4 Chloride loss occurs with oozing at the burn surface. Option 1 is incorrect because sodium is Recall the fluids shifts and losses that occur with this level of burn. Recognize that chloride
degree burns, the nurse should monitor for fluid lost with body fluids, causing hyponatremia. Option 2 is incorrect because calcium is lost in the accompanies sodium losses to be directed to option 4.
imbalances and which electrolyte imbalance? edematous fluid. Option 3 is incorrect because potassium leaves the cells as sodium shifts into
the cells, causing hyperkalemia.
1.‐ Hypernatremia
2.‐ Hypocalcemia
3.‐ Hypokalemia
4.‐ Hypochloremia

2434 For which of the following should the nurse monitor Correct answer: 1 Large doses or prolonged use of oral cortisone therapy increases serum chloride, and The critical word is corticosteroid. Recall that this drug promotes sodium and potassium
in a client with a long history of corticosteroid decreases potassium and magnesium levels. losses, and eliminate options 3 and 4. Recognize that chloride accompanies the sodium
treatment? retention to choose option 1.
1.‐ Increased serum chloride levels
2.‐ Increased serum magnesium levels
3.‐ Increased serum potassium levels
4.‐ Decreased serum sodium levels

2435 When examining a client admitted with complaints of Correct answer: 4 The nurse should be aware that the client is exhibiting symptoms of hyperchloremia due to a Read the question carefully, and recognize that the symptoms reflect metabolic acidosis
weakness and lethargy, the nurse notices that the fluid deficit. A hypotonic solution should be administered to increase the extracellular fluid and accompanying chloride retention. Note the similarity of sodium content in options 1
client's respirations are deep and rapid. Upon further (ECF) and decrease the serum osmolality. All of the other options would not be indicated, and 2, and eliminate them. Recognize the need for hydration, and choose option 4.
assessment, it is documented that the client is because they are all high in saline and chloride, and could cause the client to develop further
dehydrated from inadequate fluid intake for more than fluid and electrolyte complications.
four days. The nurse should expect to administer which
of the following as ordered?

1.‐ 3% saline solution


2.‐ Salt tablets
3.‐ Isotonic saline with 10% dextrose
4.‐ Hypotonic IV fluids

2436 A client with a history of bulimia is admitted for Correct answer: 4 The client with bulimia often purges, or uses excessive laxatives, resulting in the loss of The critical words are sodium, chloride, and limit. Note that options 1, 2, and 3 are all high
treatment of electrolyte imbalances. Recognizing the sodium, chloride, and potassium. Excessive water intake can lead to dilutional hyponatremia in sodium, and would be encouraged to restore electrolyte balance, and eliminate them.
need to prevent further losses of sodium and chloride, and hypochloremia. All of the other foods and fluids would provide needed salt and chloride.
the nurse instructs the client to limit the intake of:

1.‐ Milk and dairy products.


2.‐ Broths to two cups a day.
3.‐ Processed foods and snacks.
4.‐ Bottled or tap water.

2437 Intake of which of the following foods would indicate Correct answer: 2 Fruit has the lowest amount of chloride, and is an appropriate item for intake as part of a low‐ The critical term is low‐chloride diet. Recognize that foods high in sodium also contain
to the nurse that dietary instruction has been chloride diet. The other options are incorrect because they are all high in chloride. chloride, and eliminate options 1, 3, and 4.
adequate for a client placed on a low‐chloride diet?

1.‐ Rye
2.‐ Fruit
3.‐ Seaweed
4.‐ Canned vegetables
2438 A client returns to the clinic after being prescribed a Correct answer: 2 Compliance with the medication administration schedule will influence effectiveness of the First, recognize that the chloride level is on the high end of normal. Then, recognize that
10‐day course of diuretics for fluid retention. After medication. Diuretics should reduce chloride levels if taken appropriately. Further data chloride is lost with diuretics, but the level does not correlate with the loss that would be
reviewing the lab data, the nurse notices that the collection is needed prior to intervention. Option 1 is incorrect, since merely drawing another expected, and choose option 2.
chloride level of 106 mEq/L was higher than on the blood sample would provide no additional information at this point in time. There is no reason
first visit. The nurse's initial action should be to: to suspect that there has been a laboratory error or that the initial result is inconclusive.
Options 3 and 4 are also incorrect, because there is not enough information to warrant the
change of diuretic therapy or referral to a specialist at this point in time.

1.‐ Draw another blood sample.


2.‐ Determine if the client took the medication as prescribed.
3.‐ Request that the physician order a different diuretic.
4.‐ Make an appointment for the client to be seen by a specialist.

2439 Laboratory test results indicate that the client has a Correct answer: 4 Weakness and lethargy occur with hyperchloremia (normal is 95–108 mEq/L). All of the other The core issue of the question is that the client has hyperchloremia. Recognize the
serum chloride level of 114 mEq/L. The nurse would options reflect manifestations that are associated with hypochloremia. similarities in options 1, 2, and 3 to eliminate them.
assess the client for which of the following anticipated
manifestations?
1.‐ Tremors and flaccid muscles
2.‐ Twitching and spastic reflexes
3.‐ Hyperreflexia and tremors
4.‐ Weakness and lethargy

2440 Which of the following should be included as a Correct answer: 1 In metabolic alkalosis, bicarbonate ions are retained, and the kidneys respond by excreting Recall first that metabolic alkalosis leads to hypochloremia. Eliminate options 3 and 4,
priority intervention for a client diagnosed with chloride ions, which in turn causes reciprocal hypochloremia. Option 2 is incorrect because since they would cause further hyperchloremia. Recall signs of hypochloremia to choose
metabolic alkalosis associated with hypochloremia? deep, rapid respirations and stupor are symptoms of hyperchloremia. Options 3 and 4 are option 1.
incorrect because serum chloride levels are decreased, and the restriction of salt and
administration of diuretics will normally cause further chloride losses to occur, which could
further compromise the client s status.

1.‐ Assess for muscle tremors and slow, deep respirations.


2.‐ Assess for rapid, deep respirations and stupor.
3.‐ Restrict salt in the diet.
4.‐ Administer diuretics.

2441 When a client is admitted with a chloride level of 80 Correct answer: 3 The client presents with hypochloremia, and most likely is experiencing other electrolyte First, recognize that the level is dangerously low, and recall that sodium and potassium
mEq/L, the nurse anticipates administration of which deficiencies as well, most notably sodium and potassium. A solution with 0.45% saline with losses occur with low chloride, and will need to be replaced. Option 3 provides both of
of the following intravenous solutions? added potassium would be an appropriate option, because this would correct all fluid and these electrolytes.
electrolyte imbalances. Option 2 would not be appropriate, because it does not address the
issue of additional electrolyte deficiencies. Option 1 is incorrect because these fluids can
further dilute the plasma and the serum chloride level. Option 4 is incorrect. Hypertonic saline
is usually administered in cases of severe hyponatremia.

1.‐ 5% dextrose and water


2.‐ 0.9% sodium chloride
3.‐ 0.45% sodium chloride with 20 mEq of potassium
4.‐ 3% sodium chloride
2442 Which of the following statements would the nurse Correct answer: 4 Dates and bananas are high in chloride, and therefore can be included in a dietary pattern to The core issue of the question is knowledge of foods that are naturally high in chloride. A
make during dietary teaching for a client who has a increase chloride levels. Option 1 is incorrect; foods containing rye should be included in the critical word in the question is dietary, which helps to eliminate option 3. Recall foods high
serum chloride level of 86 mEq/L? diet, because they are high in chloride. Option 2 is incorrect because diuretics can increase the in chloride to choose option 4.
excretion of chloride and thereby reduce serum chloride levels. In addition, the nurse cannot
tell a client to alter prescribed medication therapy. Option 3, while increasing the amount of
citrus fruit in the diet provides nutritional benefit, does not increase chloride levels. Citrus is
high in potassium.

1.‐ “Avoid eating foods containing rye.”


2.‐ “Take your prescribed diuretic daily.”
3.‐ “Increase the amount of citrus fruit in your diet.”
4.‐ “Increase intake of dates and bananas in your diet.”

2443 After checking the medical record of a client admitted Correct answer: 2 A serum value of 110 mEq/L reflects an elevated serum chloride level. Cushing s syndrome First, recognize that the level is elevated. Recall that hyperchloremia is seen with elevated
with shortness of breath and lethargy, the nurse noted causes retention of excess sodium and chloride, and potassium deficit. Option 1 is incorrect sodium to direct you to option 2.
a chloride level of 110 mEq/L. Which of the following because Addison s disease is associated with decreased levels of sodium and chloride, and
coexisting health problems would the nurse suspect? potassium excess. Option 3 is incorrect because elevated chloride levels are usually associated
with metabolic acidosis. Option 4 is incorrect because SIADH is associated with chloride deficit.

1.‐ Addison’s disease


2.‐ Cushing’s syndrome
3.‐ Metabolic alkalosis
4.‐ Syndrome of inappropriate antidiuretic hormone (SIADH)

2444 Which of the following nursing diagnoses would the Correct answer: 3 The stated value represents an elevated chloride level. Increased use of table salt will cause First, determine that the level is elevated. Eliminate options 1, 2, and 4, as they are
nurse most likely identify for a client with a serum an increase in both sodium and chloride levels. Option 1 is incorrect because the use of NG associated with low chloride levels.
chloride level of 112 mEq/L? suctioning causes HCl acid to be lost, thereby decreasing the chloride level. Option 2 is
incorrect because it is not a nursing diagnosis, and the client has hyperchloremia, not
hypochloremia. Option 4 is incorrect because it is not a nursing diagnosis, and chloride excess
is associated with metabolic acidosis, and is seen in clients who have acute renal failure.

1.‐ Ineffective Health Maintenance related to nasogastric suctioning


2.‐ Altered Electrolytes related to hypochloremia
3.‐ Imbalanced Nutrition: More than Body Requirements related to excess intake of foods rich in salt
4.‐ Impaired Renal Function (Chronic) related to acid–base imbalance of metabolic alkalosis

2445 A client who is experiencing hypochloremia is Correct answer: 1 Chloride levels are typically drawn as part of general serum electrolytes, and do not require Recognize that caffeine and hormones do not affect chloride levels. Recall that chloride is
scheduled to have blood drawn for a chloride level. In the client to be NPO prior to the test; in addition, the client does not need to alter typical salt drawn with other electrolytes to choose option 1.
preparation for the blood work, the nurse instructs the intake. Caffeine and hormones will not interfere with test results.
client to:
1.‐ Eat a typical diet with ordinary or typical salt intake.
2.‐ Restrict intake of caffeine‐containing beverages the day before blood is drawn.
3.‐ Fast for 8 hours before the blood is to be drawn.
4.‐ Stop taking any hormone medications for 24 hours.

2446 A client is admitted to the intensive care unit (ICU) Correct answer: 4 An admitting clinical diagnosis of metabolic alkalosis, hypokalemia, and hyponatremia is Recall that chloride goes in the same direction as sodium to help make the correct
with metabolic alkalosis, hypokalemia, and usually associated with chloride deficit. It would be prudent to check serum chloride levels in selection of which test on which to focus.
hyponatremia. The nurse should look at the results of order to ascertain the client s baseline in the presence of multiple electrolyte deficiencies. The
serum laboratory studies to detect which additional other test results do not reflect serum electrolytes, and therefore will not change in
test that likely has an abnormal result? proportion to the electrolyte changes.

1.‐ Ammonia
2.‐ Uric acid
3.‐ Creatinine
4.‐ Chloride

2447 Which of the following items in a client’s recent Correct answer: 1 The use of acetazolamide (Diamox) can lead to the development of hyperchloremic acidosis, Recognize that options 2, 3, and 4 would contribute to loss of chloride, and eliminate
history does the nurse identify as contributing to a because it increases chloride levels. All of the other options are incorrect, because they would them.
state of hyperchloremic acidosis? lead to chloride deficiencies that would result in an alkalotic state.

1.‐ Administration of acetazolamide (Diamox)


2.‐ Administration of antacids
3.‐ Administration of thiazide diuretics
4.‐ Chronic laxative use

2448 Which of the following changes in urine levels would Correct answer: 1, 2 Options 1 and 2 are correct. Low urine sodium and chloride indicates chloride retention in the Critical words are cardiac and pitting edema. Recognize that sodium and chloride would be
be expected in a client with cardiac disease who is body, especially with overhydration or fluid excess. Option 3 is incorrect because it indicates retained with edema and as a result renal losses of chloride and sodium would decrease to
experiencing pitting edema? Select all that apply. fluid deficit. Option 4 is incorrect because calcium is not altered by fluid retention due to choose options 1 and 2.
cardiac disorders. Option 5 is incorrect, as phosphorus would not be affected.

1.‐ Decreased sodium


2.‐ Decreased chloride
3.‐ Increased phosphorus
4.‐ Decreased calcium
5.‐ Increased phosphorus

2449 A client is being discharged with a prescription for Correct answer: 1 Glucocorticoids cause retention of chloride and sodium, leading to fluid retention. All of the The critical words are prednisone and side effects. Recall that this drug is a corticosteroid,
prednisone. Which statement would indicate the client other options, such as decreasing diuretic intake, eating more vegetables, and eating foods and that this type of drug causes retention of sodium and chloride, to direct you to option
understands the side effects that affect the client’s such as spinach and celery (high in chloride content), will increase the serum chloride level. 1.
serum sodium and chloride levels?
1.‐ “I should limit my salt intake.”
2.‐ “I will not need to take my diuretic now.”
3.‐ “It will be important to eat more vegetables.”
4.‐ “I should increase my intake of spinach and celery, which I enjoy.”

2450 Which of the following should be included in the plan Correct answer: 2 Increased use of sodium bicarbonate causes excretion of chloride or hypochloremia; Critical words are multiple ampules and sodium bicarbonate. Recall that chloride losses
of care for a client who received multiple ampules of therefore, it would be appropriate to have serum chloride levels monitored for potential occur with bicarbonate use or retention to choose option 2.
sodium bicarbonate over several days? deficits. Option 1 is incorrect because D&lt;sub&gt;5&lt;/sub&gt;W infusion is hypotonic, and
will cause further fluid shifting and more potential electrolyte imbalances, given the high rate.
This will decrease chloride levels if administered over a prolonged time. Option 4 is incorrect,
because diuretics will decrease chloride levels. Option 3 is incorrect because a magnesium
value will not give any additional information, and is therefore unnecessary.

1.‐ Administer dextrose 5% in water infusion at 125 mL/hour for five days.
2.‐ Closely monitor serum chloride level.
3.‐ Check serum magnesium level daily.
4.‐ Administer diuretics to prevent metabolic acidosis.

2451 Which of the following laboratory test results would Correct answer: 1 When serum osmolality is higher than 295 mOsm/kg, there are more sodium and chloride Critical words are Cushing's syndrome and osmolality of 298. Recognize that the osmolality
the nurse expect to see in a client admitted with ions in proportion to water. Therefore, you would expect to see a higher serum chloride level. is slightly high, indicating an increase of solutes in the serum, which are usually sodium and
Cushing’s syndrome who has a serum osmolality of 298 In a client who has Cushing s syndrome, one would expect to see elevated serum chloride and chloride. Recall normal lab values for electrolytes, and choose option 1, since this value is
mOsm/kg? sodium levels, elevated urinary chloride, and a decreased potassium level. All of the other elevated.
options are inconsistent with the clinical presentation of Cushing s syndrome.

1.‐ Serum chloride level of 111 mEq/L


2.‐ Serum sodium level of 130 mEq/L
3.‐ Serum potassium level of 5 mEq/L
4.‐ Urine chloride level of 90 mEq/L

2452 Which of the following interventions should the nurse Correct answer: 3, 4 It is important for accurate results that the blood sample not be hemolyzed. A tourniquet can Recall knowledge of the effect of hemolysis on blood samples to aid in making the correct
complete when preparing to draw a serum chloride cause turbulence in blood flow, and alter results by hemolyzing erythrocytes. If possible, blood selection(s). Recall that many routine labs, such as electrolytes, do not require NPO status.
level on a client? Select all that apply. should be drawn without the use of a tourniquet. Option 1 is incorrect because drawing blood
from an implanted port used for chemotherapy is not recommended procedure. Option 2 is
incorrect because the action of clenching and unclenching the fist can lead to hemolysis of
RBCs and cause altered test results. The client does not need to be NPO prior to drawing any
electrolyte sample.

1.‐ Draw the blood from an implanted port used for chemotherapy, if necessary.
2.‐ Do not ask the client to clench and unclench his hand prior to drawing the blood.
3.‐ Draw a 3–5 mL blood sample without a tourniquet, if possible.
4.‐ Ensure that the specimen is not hemolyzed.
5.‐ Ensure that the client has been NPO for at least six hours prior to drawing the blood sample.

2453 Which one of the following clients does the nurse Correct answer: 3 In CHF, chloride is increased, and the administration of hypertonic saline could cause a lethal This question requires you analyze each option for the effect of additional sodium and
identify as being the least likely to benefit from the hypervolemia. In addition, mechanisms for excreting sodium, chloride, and water are ability to improve chloride level. Eliminate options 1, 2, and 4, since sodium could improve
administration of 3% saline solution for compromised in CHF, causing significant fluid and electrolyte alterations if such a therapy were these conditions.
hypochloremia? utilized. The clients in options 1 and 2 would benefit from administration of a hypertonic
solution in a closely monitored situation. Option 4 is incorrect because a client diagnosed with
alkalosis would benefit from administration of a hypertonic solution, since the client would
most likely be experiencing chloride and sodium deficits.

1.‐ A client diagnosed with Addison s disease.


2.‐ A client who has been NPO for several days.
3.‐ A client diagnosed with congestive heart failure (CHF).
4.‐ A client experiencing metabolic alkalosis.

2454 A client has developed weakness, lethargy, and Correct answer: 4 The clinical symptoms noted above (weakness, lethargy, and fatigue) are associated with Note that this question does not identify the type of chloride imbalance. Recognize that
fatigue because of a chloride imbalance. Which of the hyperchloremia. Option 1 is incorrect because the symptoms related to chloride deficiency can the symptoms reflect hyperchoremia to direct you to option 4.
following statements to the client by the nurse is most be reversed with clinical treatment that restores serum chloride levels to normal. Option 2 is
accurate? incorrect because there is no correlation between exercise and increase in serum chloride
level. It is likely that increased exercise would lead to a chloride deficiency through sweat and
perspiration losses. Option 3 is incorrect, because merely increasing salt intake will not
automatically increase serum chloride levels.

1.‐ “Your symptoms are permanent, but you can learn to live with them.”
2.‐ “If you increase your exercise routine, your chloride level will return to normal.”
3.‐ “You will have to increase your salt intake in order to eliminate your symptoms.”
4.‐ “Your symptoms will disappear after your chloride level decreases to normal.”

2455 The nurse should place highest priority on which of Correct answer: 1 With severe chloride and ECF losses, the blood pressure drops, potentially leading to shock, if The critical words are highest priority. Recognize that the chloride level is critically low and
the following interventions when caring for a client not corrected. The nurse should place the highest priority on monitoring the client to prevent that fluid loss is also present to direct you to option 1.
admitted with symptoms related to a chloride level of development of potential complications and to maintain client safety. Although it might be
70 mEq/L and extracellular fluid (ECF) loss? necessary to assist the client to the bathroom, this is not the priority intervention. If there is
sufficient ECF loss, the client would most likely be too weak to ambulate, and bedrest would be
indicated. Option 3 is incorrect because starting IV therapy with a hypotonic solution could
further exacerbate the client s clinical condition. Although it would be important to monitor
the client s pulse, this again is not the priority intervention at this point in time.
1.‐ Monitoring blood pressure for decrease in value
2.‐ Assisting client to the restroom to prevent injury
3.‐ Starting an IV with dextrose in water
4.‐ Monitoring pulse for pounding, slow rate

2456 Which of the following nursing diagnoses would be a Correct answer: 3 Neurological alteration related to chloride imbalance includes tremors and twitching of the The critical word in the question is neuromuscular. Eliminate options 1 and 2, since these
priority for a client experiencing neuromuscular muscles with hypochloremia, or weakness and lethargy with hyperchloremia. These are related to fluid imbalances, and eliminate option 4, since this might or might not be a
abnormalities related to a chloride imbalance? manifestations place the client at risk for injury. Options 1, 2, and 4 might be potential related diagnosis.
diagnoses, but more data would be needed to determine the priorities.

1.‐ Excess Fluid Volume


2.‐ Deficient Fluid Volume
3.‐ Risk for Injury
4.‐ Interrupted Thought Processes

2457 An alert client is admitted with a diagnosis of Correct answer: 2 Documenting the history can assist in determining the cause of the elevated chloride level. The critical words in the question are alert, hyperchloremia, and priority. Eliminate option
hyperchloremia. The nurse should implement which of This should occur, if possible, prior to intervention. Option 1 is incorrect because isolation is 1, since it is not warranted. Eliminate option 3, since this would be unsafe until a full
the following as a priority intervention? not indicated for hyperchloremia. Option 3 is incorrect because ambulating independently assessment was performed. And eliminate option 4, since the client does not need more
without assessment is unsafe, since the client will probably have weakness and lethargy. sodium chloride.
Option 4 is incorrect because infusing saline is an unsafe intervention that would lead to
increased chloride levels.
1.‐ Place the client on isolation precautions.
2.‐ Document the client’s history.
3.‐ Allow the client to ambulate ad lib unassisted.
4.‐ Start an IV of 0.9% sodium chloride infusion.

2458 A client with anorexia nervosa has been taught to Correct answer: 1, 2 Foods high in chloride include bananas and dates; green, leafy vegetables; seafood; poultry; Clients with anorexia nervosa usually have electrolyte deficiencies. Foods high in chloride
increase intake of foods high in chloride. The nurse and dairy products. Canned soups tend to be higher in sodium, and chloride is combined with are also generally foods high in sodium.
determines that teaching has been effective when the sodium as salt.
client identifies which of the following foods as high in
chloride? Select all that apply.

1.‐ Bananas
2.‐ Canned soups
3.‐ Apples
4.‐ Beef
5.‐ Pasta

2459 The nurse is teaching a class on osteoporosis to a Correct answer: 3, 5 Phosphorus is a critical mineral in the development of teeth and bones. It also plays a role in Critical words are osteoporosis and phosphorus. Recall the functions and physiology of
community group, and explains the role of calcium and acid–base balance (option 5), and it assists in regulating calcium levels (option 1). Phosphorus phosphate as they are involved in bone health to be directed to option 3.
phosphorus. The nurse should include which of the is found in the cell membranes as phospholipids (option 2), and is essential in the metabolism
following information? Select all that apply. of carbohydrates, fats, and proteins (option 4).

1.‐ Phosphate levels regulate potassium levels.


2.‐ Phosphate is found in the cell membrane as triglycerides.
3.‐ Phosphorus participates in the formation of teeth and bones.
4.‐ Phosphorus plays a minor role in the body's metabolism.
5.‐ Phosphorous also acts as an acid–base buffer.

2460 The nurse understands that phosphorus levels might Correct answer: 1 The kidneys are responsible for 90% of phosphate excretion to maintain normal phosphate Note that the question only indicates that level might be abnormal. Read each option, and
be abnormal in the client with which of the following balance. Impaired renal function usually results in decreased phosphorus excretion. The other analyze any correlation to phosphorus. Recall that phosphorus is reabsorbed and excreted
conditions? options are incorrect. via the kidneys to be directed to option 1.
1.‐ Impaired renal function
2.‐ Hepatobiliary disease
3.‐ Asthma
4.‐ Abdominal ascites

2461 When reviewing lab results, the nurse notes a Correct answer: 4 Hypophosphatemia causes platelet destruction and dysfunction from lack of adenosine Recognize that the phosphorus level indicates hypophosphatemia and that platelets are
phosphorus level of 1.7 mg/dL, indicating triphosphate (ATP). Myocardial contractility is decreased, leading to shock (option 1); muscles decreased to be directed to option 4.
hypophosphatemia. The nurse plans to assess the are weakened, leading to respiratory failure (option 2); and granulomatous activity is
client for: depressed, leading to signs of infection (option 3).
1.‐ Tachycardia.
2.‐ Tachypnea.
3.‐ Elevated white blood cell count.
4.‐ Bruising.

2462 A client has a phosphorus level of 5.3 mg/dL. The Correct answer: 1 The level reflects hyperphosphatemia, which has a reciprocal relationship with calcium. Since First, recognize that the phosphorus level reflects hyperphosphatemia. Recall that
nurse plans to closely monitor the client for: the symptoms of hypocalcemia are often most prominent, the client should be assessed for hypocalcemia occurs reciprocal to the elevated phosphorus to be directed to option 1.
signs of tetany. The other choices are associated with hypophosphatemia.

1.‐ Signs of tetany.


2.‐ Cardiac dysrhythmias.
3.‐ Elevated blood glucose.
4.‐ Gastrointestinal bleeding.

2463 The nurse notes that a client's serum phosphorus Correct answer: 3 Acute renal failure impairs the ability to excrete phosphorus normally, since the kidneys are First, determine that this is an elevated phosphorus level. Recall conditions associated
level is 4.9 mEq/L. The nurse concludes that this is the major organs for excretion of phosphorus. The other options are incorrect. with phosphorus excretion to be directed to option 3.
compatible with which of the following disorders in the
client's recent history?

1.‐ Asthma
2.‐ Pituitary tumor
3.‐ Acute renal failure
4.‐ Peripheral vascular disease

2464 The nurse explains to a client with Correct answer: 2 Eggs are high in phosphorus content, along with dairy products, poultry, organ meats, red Recall that foods high in phosphorus are proteins and dairy products. Recognize that
hyperphosphatemia that which of the following foods meat, legumes, and whole grains. For this reason, they should be limited in the diet when option 2 is high in protein to choose it.
should be limited or avoided as a breakfast item? hyperphosphatemia is present. The other foods listed have less phosphorus content.

1.‐ Orange juice


2.‐ Eggs
3.‐ White bread toast
4.‐ Pancakes

2465 The nurse working on a surgical unit plans to monitor Correct answer: 2 A client who has had thyroidectomy might accidentally have had all or part of the parathyroid Analyze each option to determine if a risk for impaired phosphorus regulation exists.
for hyperphosphatemia in a client who just underwent gland, which regulates calcium level, removed. If the client becomes hypocalcemic, then serum Recognize the risk for hypocalcemia with thyroidectomy, and recall this imbalance has a
which procedure? phosphorus levels will rise. reciprocal or inverse relationship with phosphorus.
1.‐ Tonsillectomy
2.‐ Thyroidectomy
3.‐ Total knee replacement
4.‐ Open reduction of a fracture
2466 A client with anorexia, nausea, vomiting, and Correct answer: 1 A client who has had hypercalcemia is at risk for hypophosphatemia, since calcium and Recognize the reciprocal relationship of phosphorus and calcium. Recall normal values of
evidence of a pathological fracture has hypercalcemia. phosphorus have an inverse relationship in the body. The normal value is 2.5–4.5 mEq/L, phosphorus, and choose option 1 because it is below normal.
Which of the following serum phosphorus levels would making option 1 the value that is below the normal range.
the nurse anticipate finding on the serum laboratory
report for this client?

1.‐ 1.9 mEq/L


2.‐ 2.8 mEq/L
3.‐ 3.9 mEq/L
4.‐ 5.0 mEq/L

2467 For which of the following manifestations should the Correct answer: 2 Muscle spasms and tetany accompany hyperphosphatemia because of the corresponding Recognize that chronic use of laxatives will lead to hyperphosphatemia and hypocalcemia.
nurse assess in a client with a history of chronic use of drop in serum calcium level. The other options listed are signs of hypophosphatemia. Recognize that muscle spasms correlate to these imbalances to choose option 2.
phosphorus‐containing laxatives?

1.‐ Profound muscle weakness


2.‐ Muscle spasms
3.‐ Malaise
4.‐ Muscle pain and tenderness

2468 The nurse determines that which of the following Correct answer: 2 Rhabdomyolysis is the breakdown of striated muscle, which causes large amounts of Analyze each option to determine risk for phosphorus retention or impaired regulation.
clients is at greatest risk for developing phosphorus to enter the bloodstream. The other conditions listed place the client at risk for Recall that phosphorus is intracellular, and released with muscle damage, to be directed to
hyperphosphatemia? hypophosphatemia. option 2.
1.‐ A severely malnourished client undergoing refeeding
2.‐ A client with rhabdomyolysis
3.‐ A client with chronic antacid use
4.‐ A client with severe alcohol abuse

2469 The nurse utilizes which of the following concepts Correct answer: 2 Newborn levels of phosphorus are nearly twice those of an adult. Normal adult serum The critical word is newborn. Recognize that options 1, 3, and 4 are incorrect statements
about phosphate levels in the newborn when phosphate levels in the adult range from 2.5 to 4.5 mg/dL, and newborn levels range from to eliminate them.
implementing client care? about 4.0 to 7.0 mg/dL (option 3). Phosphate levels vary throughout the day (option 1).
Phosphorus is the second most abundant mineral in the body (option 4).

1.‐ Phosphate levels are consistent throughout the day.


2.‐ Phosphate levels in newborns are nearly twice the adult level.
3.‐ Normal serum phosphate levels range from 1.5 to 2.0 mg/dL.
4.‐ Phosphorus is the most abundant mineral in the body.

2470 When caring for a client with a phosphorus level of Correct answer: 1 A decrease in phosphorus depletes available adenosine triphosphate (ATP) needed for cellular Recall that the normal phosphorus level is 2.5–4.5 mg/dL.
1.8mg/dL, the nurse plans interventions to promote: energy production, which will be manifested in the client as fatigue and muscle weakness.
Option 2 is incorrect; the kidneys try to reabsorb phosphorus in order to conserve it when
levels are low. Option 3 is incorrect; bowel sounds and motility are already decreased with low
phosphorus levels. Option 4 is incorrect; the rate and depth of breathing are already increased
in response to hypoxemia experienced with hypophosphatemia.

1.‐ Conservation of energy.


2.‐ Increased renal perfusion.
3.‐ A decrease in peristalsis.
4.‐ Deep and rapid breathing.
2471 A client experiencing hypophosphatemia has been Correct answer: 1, 5 Oxalate (in spinach and rhubarb) and phytates (found in bran and whole grains) can interfere Calcium and phosphorus have an inverse relationship, and there is no harm from too much
started on sodium phosphate (Neutra‐Phos). The nurse with absorption of phosphate by binding with them in the intestines. Milk, orange juice, and phosphorus in the diet.
instructs the client to avoid taking the medication with chicken do not pose this problem for absorption of phosphate.
which of the following foods? Select all that apply.

1.‐ Whole grains


2.‐ Milk
3.‐ Orange juice
4.‐ Chicken
5.‐ Spinach

2472 The nurse would assess for signs and symptoms of Correct answer: 1 Severe vomiting and diarrhea deplete the body s many stores of electrolytes, including Critical words are predisposing clinical conditions. Note the word severe in option 1, and
hypophosphatemia in the client who has which of the phosphorus. Prolonged use of aluminum‐ and magnesium‐containing antacids that bind to recognize that phosphorus will be lost with vomiting and diarrhea.
following predisposing clinical conditions? phosphorus (option 2) is a condition leading to hypophosphatemia. Balanced TPN solutions
contain adequate levels of phosphorus (option 3). Vitamin D deficiencies lead to decreased
intestinal absorption of phosphorus (option 4).

1.‐ Severe vomiting and diarrhea


2.‐ Occasional use of magnesium‐containing antacids
3.‐ Infusion of balanced total parenteral nutrition (TPN) solutions
4.‐ Vitamin D excess

2473 When caring for a client with a serum phosphorus Correct answer: 2 Hypophosphatemia results in decreased ATP production, decreasing enzyme levels of 2,3‐ Recall that phosphorus is needed for ATP production and oxygenation. Recognize that
level of 1.9 mg/dL, the nurse should be alert to which DPG, which in turn keeps oxygen bound to hemoglobin and less available to the tissues. Clients these deficiencies can contribute to irregular heart rhythms to direct you to option 2.
of the following signs? with hypophosphatemia will experience hypoactive bowel sounds (option 1), muscle weakness
(option 3), and paresthesias and anemia due to RBC fragility from low ATP levels (option 4).

1.‐ Hyperactive bowel sounds


2.‐ Dysrhythmias related to decreased oxygenation
3.‐ Increased muscle tone
4.‐ Polycythemia

2474 When caring for the client with hypophosphatemia, Correct answer: 3 Hypophosphatemia leads to a decline in 2,3‐DPG levels, reducing the release of oxygen to the Recognize the need for adequate phosphorus for oxygenation needs to direct you to
the nurse should anticipate complaints of which of the tissues. Clients are more likely to have complaints of apprehension than euphoria (option 1). option 3.
following? Decreased gastric motility leads to anorexia in the hypophosphatemic client (options 2 and 4).

1.‐ Euphoria
2.‐ Hunger
3.‐ Chest pain
4.‐ Thirst

2475 A client with chronic renal failure (CRF) has been Correct answer: 3 In order to maximize binding of the phosphate, phosphate binders should be given with the Note similarities in options 1 and 4 to eliminate them. Recognize the purpose and action
started on a phosphate binder for treatment of meal, or shortly after, for the medication to have contact with the phosphate in the food. of the medication to direct you to option 3.
hyperphosphatemia. To enhance effectiveness of the
medication, the nurse plans to administer it:

1.‐ On an empty stomach.


2.‐ Thirty minutes before the meal.
3.‐ With meals.
4.‐ Two hours after a meal.
2476 A history of which of the following disorders in a Correct answer: 2 Chronic renal failure impairs the ability to excrete phosphorus normally, since the kidneys are Recall that the kidneys are the major organ for excretion of electrolytes to direct you to
client would lead the nurse to suspect a problem with the major organs for excretion of phosphorus. The other options are incorrect. option 2.
phosphorus excretion?
1.‐ Bowel obstruction
2.‐ Chronic renal failure
3.‐ Chronic obstructive pulmonary disease
4.‐ Coronary artery disease

2477 The nurse determines a client with a phosphorus level Correct answer: 3 Red meat is high in phosphorus content, as are dairy products, eggs, poultry, organ meats, Recall foods high in phosphorus to direct you to option 3.
of 4.9 mg/dL indicates an understanding of dietary legumes, and whole grains. For this reason, it should be limited in the diet when
instructions when the client limits intake of which of hyperphosphatemia is present. The other foods listed have less phosphorus content.
the following food items?

1.‐ Pork chops


2.‐ White rice
3.‐ Sirloin steak
4.‐ Green peas

2478 The nurse determines that a client who chronically Correct answer: 1 Enemas can be high in phosphorus, putting the client at risk for hyperphosphatemia if they Analyze each option for phosphorus content, eliminating options 2, 3, and 4, since they are
uses which of the following types of over‐the‐counter are frequently used. The other products listed do not have large amounts of phosphorus in not high in phosphorus.
products is at greatest risk for developing them.
hyperphosphatemia?
1.‐ Enemas
2.‐ Cough preparations
3.‐ Cold preparations
4.‐ Bedtime sleeping aids

2479 The physician orders a high‐phosphorus diet for the Correct answer: 2 Dairy products are naturally high in phosphorus. Although many food sources contain The critical words are high‐phosphorus diet. Recall foods high in phosphorus to direct you
client with hypophosphatemia. You would expect to phosphorus, the greatest amounts are found in red and organ meats, fish, poultry, eggs, dairy to option 1.
see which of the following dietary items on the meal products, nuts, whole grains, and legumes. Carbonated soft drinks are also high in phosphorus,
tray? although they are low in nutrient value.
1.‐ An apple
2.‐ Milk
3.‐ Lemonade
4.‐ White bread

2480 When checking serum phosphorus levels on a Correct answer: 2 Children have higher phosphate levels than do adults because of their more rapid bone First, recognize that this is a pediatric client, and that pediatric phosphorus levels would
pediatric client with hypophosphatemia, the nurse development rate. Options 1, 3, and 4 are incorrect. Replacement therapy would result in an normally be higher. Eliminate arterial sampling, as ABGs are the only common lab drawn
anticipates that: increase in phosphorus level. A venous, not arterial, sample is taken. Serum phosphate levels arterially. After starting phosphorous replacement, the levels should rise.
vary throughout the day.
1.‐ Levels will be highest in the early morning.
2.‐ Normal levels are slightly higher secondary to rapid skeletal growth.
3.‐ A decrease will be seen initially after starting replacement therapy.
4.‐ An arterial sample must be used to provide the most accurate level.

2481 When checking laboratory values on a client, the Correct answer: 2 A phosphate level of 1.7 mg/dL reflects hypophosphatemia. Since phosphorus is needed for First, recognize that the phosphate level reflects hypophosphatemia. Recall the role of
nurse notes the phosphate level is 1.7 mg/dL. The formation of the red blood cell enzyme 2, 3‐DPG, deficiency states can lead to anemia, which phosphorus in red blood cell production to direct you to option 2.
nurse should also check laboratory values for evidence would be reflected by the low hemoglobin level. The other options are incorrect. Platelets also
of which of the following? can be decreased. Calcium and phosphorus have a reciprocal relationship, and therefore
calcium would be elevated. Magnesium would be decreased.

1.‐ An elevated platelet count


2.‐ A decrease in hemoglobin level
3.‐ A decrease in calcium level
4.‐ An elevated magnesium level

2482 The nurse would assess for signs and symptoms of Correct answer: 2 Clients in diabetic ketoacidosis lose excessive amounts of phosphate in the urine. Clients with Critical words are hypophosphatemia and predisposing clinical conditions. Analyze each
hypophosphatemia in the client who had which of the first‐degree burns do not experience severe fluid shifts that affect phosphorus levels (option option, recalling that phosphorus is lost in the urine. Recognize that polyuria seen with
following predisposing clinical conditions? 1). Hypomagnesemia can result in renal excretion of phosphorus (option 3). Decreased urine acidosis will contribute to phosphorus loss to choose option 2.
output results in less renal filtration of phosphorus (option 4).

1.‐ First‐degree burns


2.‐ Diabetic ketoacidosis
3.‐ Hypermagnesemia
4.‐ Oliguria

2483 The nurse would plan to include which of the Correct answer: 4, 5 Although phosphorus is found in a large number of food items, it is found in greatest Recall foods high in phosphorus to choose options 4 and 5.
following foods in the diet when providing discharge quantities in red and organ meats; fish; poultry; eggs; milk and milk products; legumes; whole
teaching for a client with hypophosphatemia? Select grains; and nuts. The other options identify foods that have lesser amounts of phosphorus.
all that apply.
1.‐ Green, leafy vegetables
2.‐ White bread
3.‐ Citrus fruits
4.‐ Eggs
5.‐ Liver

2484 A client has just been started on total parenteral Correct answer: 3 TPN is a concentrated glucose‐and‐protein solution that utilizes phosphorus in metabolism of First, recall the purpose and content of TPN. Recall that electrolytes are utilized for
nutrition (TPN) for severe malnutrition. The nurse the nutrients and produces a shift of phosphorus into the cells, thus causing a serum enzyme reactions and metabolism, and recognize that the phosphorus level is low, to
determines that refeeding syndrome has occurred phosphorus deficit. Options 1, 2, and 4 are incorrect. These electrolytes also can be decreased choose option 3.
after noting which of the following laboratory test in the refeeding syndrome.
results?
1.‐ Magnesium 2 mg/dL
2.‐ Calcium 9.8 mg/dL
3.‐ Phosphorus 1.2 mg/dL
4.‐ Potassium 4.2 mEq/L

2485 Which of the following findings in a client’s history Correct answer: 1 Poor nutritional intake, vomiting, diarrhea, and overuse of antacids are related to alcoholism Recognize that some of the options have conditions that would contribute to a low
would alert the nurse to assess for signs and symptoms and alcohol abuse. These can lead to hypophosphatemia. During oliguria, the kidneys are phosphorus level, but identify option 1 as having the greatest risk.
of hypophosphatemia? unable to excrete phosphorus (option 2). Clients with prolonged (not short‐term) gastric
suction are more likely to experience hypophosphatemia (option 3). Prolonged or continuous
use of aluminum‐containing antacids (not occasional use) leads to hypophosphatemia (option
4).
1.‐ Withdrawal from alcohol
2.‐ The oliguric phase of acute tubular necrosis
3.‐ Short‐term gastric suction
4.‐ Occasional use of aluminum‐containing antacids

2486 Which of the following concurrent electrolyte Correct answer: 3 The client has hyperphosphatemia, since the normal phosphate level is 2.5–4.5 mg/dL. Recall the relationship between calcium and phosphorus to direct you to option 3.
imbalances should the nurse anticipate while caring for Calcium and phosphorus have an inverse relationship in the body. For this reason, when
a client with a phosphate level of 4.9 mg/dL? phosphorus levels are high, calcium levels are low. The other responses do not address this
relationship.
1.‐ Hyperkalemia
2.‐ Hyponatremia
3.‐ Hypocalcemia
4.‐ Hypermagesemia

2487 A client has developed a serum phosphorus level of Correct answer: 2 In clients with hyperphosphatemia (normal is 2.5–4.5 mg/dL) from use of cytotoxic drugs, Note that there are two conditions in the question hyperphosphatemia and hyperuricemia
5.0 mg/dL secondary to cytotoxic drug therapy. allopurinol (Zyloprim) may be ordered to decrease uric acid production, which prevents the but the question addresses only treatment of the latter. Eliminate option 1, since this is
Because of the hyperuricemia that also occurs with formation of uric acid calculi in the kidney and uric acid nephropathy. Aluminum hydroxide used to treat hyperphosphatemia. Recall that allopurinol is used to reduce uric acid levels
this therapy, the nurse anticipates administration of (Amphogel) is an antacid that would be useful in binding phosphates (option 1); acetazolamide to choose option 2.
which of the following medications? is a diuretic (option 3); and hydralazine (option 4) is an antihypertensive.

1.‐ Aluminum hydroxide (Amphogel)


2.‐ Allopurinol (Zyloprim)
3.‐ Acetazolamide (Diamox)
4.‐ Hydralazine (Apresoline)

2488 When providing discharge teaching for a client who Correct answer: 1 Food additives tend to be high in phosphates. For this reason, clients should be taught to read Key words are discharge teaching and diet high in phosphates. Recall knowledge of
requires a diet high in phosphates, the nurse would food labels carefully. Vitamin D, not vitamin A, will enhance phosphorus absorption (option 2). phosphorus sources to direct you to option 1.
include which of the following statements? Aluminum‐containing antacids decrease phosphorus absorption by binding to it (option 3). Soy‐
based foods are low in phosphorus (option 4).

1.‐ “High levels of phosphates are found in food additives.”


2.‐ “Increase your vitamin A intake to enhance phosphorus absorption.”
3.‐ “Aluminum‐based antacids increase phosphorus absorption.”
4.‐ “Soy and soy products are excellent sources of phosphorus.”

2489 The following neonates are admitted to the nursery. Correct answer: 3 Feeding a baby with a respiratory rate greater than 60 breaths/min orally increases the risk of Simply recall that breathing and swallowing cannot be done at the same time. This will
The nurse should withhold the scheduled initial feeding aspiration. A heart rate of 118 is slightly below the normal range of 120–160 beats/min, but it help you to select the infant with an elevated respiratory rate as the one who is at risk if
on which newborn? is not a contraindication to feeding the infant. A hypothermic or SGA infant is at risk for given feedings orally.
hypoglycemia, and requires a consistent source of glucose.

1.‐ A neonate with a sustained heart rate of 118 beats/min


2.‐ A neonate with an axillary temperature of 97.5°F
3.‐ A neonate with a sustained respiratory rate of 68 breaths/min
4.‐ A neonate who is small for gestational age (SGA)

2490 The nurse hears the parents of a 26‐weeks'‐gestation Correct answer: 2 Families are often in a state of denial with the birth of a sick newborn. It is important for Use knowledge of therapeutic communication techniques to answer the question. The
newborn tell family members, “We ll be ready to bring nurses to gently encourage the parents to be realistic. By agreeing with the parent s statement correct response is one that provides factual information about the infant’s status while
the baby home in a few weeks.” The most therapeutic (option 1), the nurse is prolonging the state of denial and making it more difficult for the respecting the parent’s potentially vulnerable status.
response by the nurse is which of the following? parents to see the situation realistically. Some parents do benefit from professional
counseling, but nurses still need to provide support when working with families. It is not
important if the nursery is ready yet (option 4), and this distracts from the real issues this
family is facing at this time.
1.‐ “I’m glad he’s doing so well.”
2.‐ “He probably won’t be ready to come home for a few months.”
3.‐ “A therapist could help you resolve your feelings of denial.”
4.‐ “Do you have the nursery ready yet?”

2491 While observing parents interact with their high‐risk Correct answer: 2 The act of taping family pictures to the sides of the isolette promotes bonding and infant The wording of the question tells you that the correct answer is an option that contains an
newborn, the nurse recognizes that teaching has been stimulation. Parents should wash their hands when they enter the unit, but do not need to appropriate action on the part of the parents. Use nursing knowledge and the process of
effective if the parents do which of the following? wear gloves when in contact with their infant. Young children often harbor organisms that elimination to make a selection.
could be transmitted to vulnerable newborns, and should not have contact until the infant is
moved out of the neonatal intensive care unit.
1.‐ Wear gloves every time they touch their baby.
2.‐ Put family pictures in the isolette.
3.‐ Bring a 2‐year‐old sibling to visit.
4.‐ Turn off the cardiac monitor when at the newborn’s bedside.

2492 The nurse is developing a plan of care for an infant Correct answer: 3 A healthy respiratory rate for all newborns is 30–60 breaths/min. The other interventions are Specific knowledge of expected fetal development by gestational age is needed to answer
born at 28 weeks' gestation. A realistic goal for this not timely for a 28‐weeks'‐gestation infant at 1 week of age. this question. Use nursing knowledge and the process of elimination to make your
infant is that within one week, the infant will: selection.

1.‐ Drink from a bottle.


2.‐ Recognize the parents.
3.‐ Maintain her respiratory rate between 30 and 60 breaths/minute.
4.‐ Maintain her body temperature in a bassinet.

2493 The nurse is making patient assignments. Which baby Correct answer: 3 An LPN/LVN is qualified to perform certain procedures and care for stable patients (option 3). Specific knowledge of scope of practice by RNs and LPNs/LVNs is needed to answer this
could be appropriately assigned to an LPN/LVN? An LPN/LVN is not qualified to admit a patient, administer blood, or make nursing decisions question. Use this knowledge and the process of elimination to make your selection.
based on changes in a patient’s assessment. The infants identified in the other options require
assessment and care by a registered nurse.
1.‐ An infant being admitted with hypoglycemia
2.‐ An infant scheduled to receive blood this shift
3.‐ A stable premature infant being fed every two hours
4.‐ An infant with rising bilirubin levels

2494 A newborn is receiving phototherapy for the Correct answer: 4 It is important to protect the infant s eyes from the bililight to prevent permanent damage. The core issue of the question is knowledge that the ultraviolet light used to treat jaundice
treatment of hyperbilirubinemia. The nurse evaluates The infant should be unclothed, to allow as much skin exposure to the bililight as possible. can cause damage to the infant’s retinas. Specific knowledge of treatment of jaundice is
that teaching has been effective when the parents do Breastfeeding is not contraindicated with hyperbilirubinemia. Loose green stools are a side needed to answer this question. Use nursing knowledge and the process of elimination to
which of the following? effect of bilirubin excretion through the intestines. make your selection.
1.‐ Cover the infant with a blanket while under the bililights.
2.‐ Stop breastfeeding because of the jaundice.
3.‐ Limit the infant’s formula intake due to loose green stools.
4.‐ Cover the infant s eyes before placing him under the bililight.

2495 Which of the following would be most important to Correct answer: 1 Narcotics cross the placenta, and, if given close to delivery, can cause respiratory depression Note that critical words in the stem of the question are most important. This tells you that
note as part of the initial assessment of a newborn’s in the newborn. The other three answers might warrant further investigation, but the priority some or all of the options are correct, but you must select the priority option. Use nursing
history? at delivery is to establish and maintain an airway. knowledge and the process of elimination to make your selection.

1.‐ Mother received meperidine (Demerol) 50 mg IV 20 minutes before delivery.


2.‐ Mother reports drinking a glass of wine with dinner each night.
3.‐ Mother’s age is 14.
4.‐ Mother’s blood type is O negative.

2496 The parents of a preterm neonate ask why their baby Correct answer: 2 Preterm infants have minimal adipose tissue, so they lose heat more quickly through their The wording of the question tells you that the correct option must be a true statement.
gets cold so easily. The nurse explains that preterm skin. The skin is thin, with blood vessels near the surface, which increases the amount of heat Use knowledge about the physical characteristics of premature infants and the process of
neonates: lost through their skin. Because they are weak and neurologically immature, they aren t able to elimination to make your selection.
lie in a tight fetal position, allowing exposure of a greater percentage of the body to the air,
which causes heat loss. In general, infants are not able to shiver to produce body heat when
they are cold.
1.‐ Are able to shiver to produce body heat.
2.‐ Have minimal body fat to retain body heat.
3.‐ Have blood vessels that are deep under the skin surface.
4.‐ Lose heat faster because they lie in a fetal position.
2497 While feeding an infant, the nurse notices white, Correct answer: 2 The primary sign of an oral yeast infection, or thrush, is the presence of white patches in the The core issue of the question is the significance of white patches in the infant’s mouth.
adherent patches on the infant s gums and buccal mouth that tend to bleed if they are touched. This is not a normal finding, and is unrelated to Eliminate option 1 because this is not a normal finding. Eliminate option 3 because vitamin
cavity. The nurse should take which of the following whether vitamin K was given at delivery, or maternal history of herpes simplex. K aids in blood clotting. Herpes simplex (cold sores) would present as vesicles, not white
actions? patches.
1.‐ Document this normal finding.
2.‐ Further evaluate for yeast infection.
3.‐ Verify that vitamin K (AquaMEPHYTON) was given at delivery.
4.‐ Assess for maternal history of herpes simplex.

2498 Which of the following data would alert the nurse Correct answer: 4 Signs of dehydration in an infant include dry mucous membranes, sunken fontanel, and dry Specific knowledge of manifestations of dehydration is needed to answer this question.
that the infant is experiencing dehydration? skin turgor. The other assessment data are expected findings in an infant. Use nursing knowledge and the process of elimination to make your selection.

1.‐ Urine‐specific gravity 1.006


2.‐ Urine volume 2 mL/kg/hr
3.‐ Low serum sodium
4.‐ Sunken anterior fontanel

2499 A newborn male is admitted to the nursery 15 Correct answer: 4 The highest priority after delivery is to maintain and support respiratory function. This infant Follow the ABCs of resuscitation airway, breathing, and circulation to select the correct
minutes after delivery. His skin is mottled, and mucous is demonstrating initial signs of respiratory deficiency. Once this is done, the nurse may check answer to this question. Airway and breathing are assessed before circulation (bleeding).
membranes are blue; he is active, and is wrapped in a the umbilical cord for bleeding, measure temperature, and, finally, check for visible
blanket. The primary nursing assessment should be to deformities.
assess which of the following?

1.‐ Umbilical cord for bleeding


2.‐ Infant’s temperature
3.‐ Visible deformities
4.‐ Patent airway

2500 Which nursing intervention is appropriate in the care Correct answer: 1 Infants use additional oxygen and glucose when faced with cold stress. Infants with RDS are Note that the core issue of the question is care of an infant with respiratory distress. First,
of an infant with respiratory distress syndrome (RDS)? already compromised, so it is important to keep environmental temperatures stable to eliminate option 2 because of the word complete. Choose option 1 over options 3 and 4
minimize their oxygen and glucose requirements. A complete assessment could increase because there is no evidence in the question that these are needed.
oxygenation requirements even further (option 2). Chest physiotherapy (option 3) might or
might not be needed. There is no specific evidence in the question that meconium is present
(option 4).
1.‐ Maintain a neutral thermal environment.
2.‐ Perform a complete gestational age assessment.
3.‐ Perform chest physiotherapy twice a day.
4.‐ Suction meconium from airway as needed.

2501 A 26‐weeks'‐gestation neonate has received 80–100% Correct answer: 2 This infant has been receiving high levels of oxygen for several weeks, and is at risk for The core issue of the question is knowledge of the effects of long‐term oxygen therapy for
oxygen via mechanical ventilation for two weeks, and retinopathy of prematurity (ROP). All preterm infants who receive oxygen should have a a neonate. Use nursing knowledge and the process of elimination to make your selection.
has received several blood transfusions for anemia. thorough eye exam done by an ophthamologist prior to discharge. It is important to administer
The nurse should plan for which of the following the minimum amount of oxygen to infants, to decrease the risk that this condition will
interventions? develop. Oxygen should be weaned, and not withdrawn suddenly. Artificial surfactant may be
administered within the first several days of life to decrease the risk of respiratory distress
syndrome (RDS).
1.‐ Begin phototherapy.
2.‐ Schedule an eye exam by an ophthalmologist prior to discharge.
3.‐ Discontinue oxygen immediately.
4.‐ Administer surfactant via the endotracheal tube.
2502 The nurse is caring for an infant born to a mother Correct answer: 3 Hepatosplenomegaly (enlarged liver and spleen) can be an early sign of HIV infection in an The core issue of the question is discriminating normal findings from abnormal findings in
who is HIV‐positive. Which sign in the newborn should infant. All other assessment data are within normal limits. a newborn born to a mother who is HIV‐positive. Use nursing knowledge and the process of
be evaluated further? elimination to make your selection.
1.‐ Absence of tears
2.‐ White bumps on nose
3.‐ Enlarged liver
4.‐ Fine, red rash over trunk

2503 An infant of a diabetic mother (IDM) is admitted to Correct answer: 4 An infant of a diabetic mother is at risk for hypoglycemia, and should be monitored closely Note the critical word priority in the stem of the question. This tells you that multiple
the nursery. Which of the following is the priority after delivery. All other interventions are important, but are not the highest priority. options are technically correct, but you must decide which has the greatest importance at
nursing intervention at this time? Therefore, these can be completed once the blood glucose level has been measured and this time. Note the connection between the word diabetic in the stem and the word
treated, if necessary. glucose in the correct option to help you make a selection.

1.‐ Clean the cord with alcohol.


2.‐ Administer vitamin K (AquaMEPHYTON) intramuscularly.
3.‐ Complete a gestational age assessment.
4.‐ Assess the infant’s blood glucose level.

2504 A father asks how the bilirubin lights make the Correct answer: 3 Phototherapy assists the body in converting unconjugated bilirubin to conjugated bilirubin, The core issue of the question is knowledge of how phototherapy assists in lowering the
bilirubin level go down. The nurse’s best reply is which which is water‐soluble and easier for the body to eliminate. The other statements are not bilirubin levels of a jaundiced newborn. Use nursing knowledge and the process of
of the following? accurate explanations. elimination to make your selection.
1.‐ “The lights prevent more bilirubin from being released into your baby’s body.”
2.‐ “Exposing the skin to the air helps get rid of the jaundice. The bililights really just keep the baby warm while this occurs.”
3.‐ “The bililights help convert the bilirubin to a form the baby can get rid of.”
4.‐ “The bililights release a substance in the body that attacks the bilirubin and destroys it.”

2505 The nurse assesses a newborn and obtains the Correct answer: 3 This infant has signs of Erb's paralysis. It is important to provide passive range of motion on The core issue of this question is recognition of and appropriate intervention for an infant
following information: Left arm limp and extended; left the affected side to prevent muscle wasting. The infant should not be positioned on the with Erb's paralysis. Use nursing knowledge and the process of elimination to make your
hand internally rotated; positive grasp reflex affected side. Occasionally, a splint may be applied, but a cast is not indicated. selection.
bilaterally; no response on left side to Moro reflex.
What is the most appropriate nursing intervention for
this infant?
1.‐ Assess for congenital hip dysplasia.
2.‐ Turn infant to left side.
3.‐ Passive range of motion
4.‐ Prepare supplies for a cast application.

2506 A neonatal nurse is attending a high‐risk delivery, and Correct answer: 1 Narcotics cross the placenta, and can cause respiratory depression in a neonate when given The core issue of the question is knowledge of adverse effects of morphine sulfate. Use
is told that the mother received morphine sulfate IV 30 shortly before delivery. Naloxone (Narcan) is the drug of choice to reverse respiratory nursing knowledge and the process of elimination to make your selection. The wording of
minutes ago. The nurse should be prepared to give depression in the neonate caused by narcotics. Insulin would be given for hyperglycemia. the question tells you that there is only one correct option.
which of the following medications to the infant Double doses of vitamin K are not given. Magnesium sulfate is given to the mother to prevent
immediately after delivery? eclampsia.

1.‐ Naloxone (Narcan)


2.‐ Regular insulin
3.‐ Double dose of vitamin K (AquaMEPHYTON)
4.‐ Magnesium sulfate

2507 Which of the following infants is at greatest risk for Correct answer: 1 SGA infants often experience intrauterine growth restriction related to decreased blood flow The wording of the question tells you the correct answer is the infant who is at greatest
the nursing diagnosis of High Risk for Infection? to the placenta, which increases their risk for infection. In comparison, the infants in the other risk for infection. Knowledge of the relative risk for infection in each of the neonates listed
options are at less risk for infection. is needed to answer this question. Use nursing knowledge and the process of elimination to
make your selection.
1.‐ 38 weeks' gestation, small for gestational age (SGA)
2.‐ 39 weeks' gestation, diagnosed with caput succedaneum
3.‐ 38 weeks' gestation, cesarean delivery for breech presentation
4.‐ 41 weeks' gestation, infant of a diabetic mother (IDM)

2508 An infant with fetal alcohol syndrome is about to be Correct answer: 3, 2, 4, Infants with fetal alcohol syndrome have an increased risk of feeding difficulties related to Use Maslow’s hierarchy of needs to guide priority setting. Physiological needs come first,
discharged home with foster parents. Place in order 1 hyperactivity. Nutrition is a key concern for this infant for proper growth and development. followed by safety needs, then psychosocial needs.
the priority of the nurse in teaching the following Infection prevention is the next concern, since this will help to maintain healthy physiological
topics to the foster parents. Click and drag the options condition. The immunization schedule has third priority because it is also related to prevention
below to move them up or down. of communicable diseases and infection. Finally, although toy safety is important, it is the
fourth priority because newborns are not developed sufficiently to play with toys.

1.‐ Toy safety


2.‐ Infection prevention
3.‐ Feeding methods
4.‐ Immunizations

2509 A nurse is admitting a baby to the nursery 30 minutes Correct answer: 4 A maternal history of diabetes increases the risk of hypoglycemia in the newborn, and this Knowledge that the infant born to the diabetic mother is at risk for hypoglycemia will aid
after delivery. Which information from the mother's infant should be monitored closely. If the woman received meperidine (Demerol), most of the in determining the correct answer. Remember that the information of greatest concern is
history should be of greatest concern? drug would be metabolized within three hours, and should not cause respiratory depression in related to the greatest risk for the infant's safety.
the infant at delivery. A marginal placenta previa increases the mother's risk of bleeding during
pregnancy, but should not cause significant complications in the newborn after delivery.
Membranes ruptured greater than 24 hours prior to delivery increase the risk of infection for
mother and infant.
1.‐ Received meperidine (Demerol) IV three hours prior to delivery.
2.‐ Marginal placenta previa
3.‐ Membranes ruptured 10 hours prior to delivery.
4.‐ Preexisting insulin‐dependent diabetes mellitus

2510 A baby's mother is hepatitis B–positive. Which of the Correct answer: 1 Infants born to mothers who are hepatitis B–positive should receive a hepatitis B vaccine The key focus of the question is the risk of transmission of hepatitis B from mother to
following interventions is most important when within 12 hours of birth to decrease their risk of acquiring the infection from maternal infant. The correct answer would be the option that contains a nursing action to reduce the
planning care for this newborn? exposure. It is appropriate to assess for HIV risk factors in all infants, not just those at risk for risk of disease transmission for this infant.
hepatitis B. An exchange transfusion and isolating the infant are not appropriate in this
situation.
1.‐ Administer hepatitis B vaccine within 12 hours after delivery.
2.‐ Assess for HIV risk factors.
3.‐ Prepare for exchange transfusion.
4.‐ Isolate the newborn.

2511 The nurse is observing a graduate nurse administering Correct answer: 4 Gavage feedings should be administered over 5–10 minutes to decrease the risk of GI Critical words are "[t]he nurse must intervene," which means that the graduate nurse is
a gavage feeding to a newborn. The nurse must distress. All of the other options are correct when administering a gavage feeding. performing an incorrect step in the process. Therefore, you are looking for an incorrect
intervene if which of the following is observed? method. Knowledge of how to administer a gavage feeding is necessary to choose the
correct answer.
1.‐ The gavage tube is measured from the tip of the ear to the sternum to the xiphoid process.
2.‐ The stomach contents are aspirated prior to administering the feeding.
3.‐ The infant is offered a pacifier during the feeding.
4.‐ The feeding is administered within 15 seconds.
2512 The nurse realizes that a neonate born at 34 weeks' Correct answer: 4, 5 Preterm infants lack adequate surfactant to keep their alveoli open during expiration. This can The focus of the question is assessment findings in a premature infant with the potential
gestation might not have enough surfactant, so the lead to the development of respiratory distress syndrome (RDS), which would be evidenced by for developing respiratory distress. Eliminate options 1, 2, and 3 because abdominal
nurse should observe closely for: (Select all that apply.) signs of respiratory distress, including sternal retractions and tachypnea. Abdominal distention, distention, jaundice, and jitteriness are not directly related to RDS.
jaundice, and jitteriness are not directly related to RDS.

1.‐ Abdominal distention.


2.‐ Jaundice.
3.‐ Jitteriness.
4.‐ Sternal retractions.
5.‐ Tachypnea.

2513 A nurse observes that a preterm infant's urine output Correct answer: 2 Adequate hydration is evidenced by urine output of 1–3 mL/kg/hr and specific gravity Knowledge of specific gravity and adequate urinary output will aid in determining the
is less than 1 mL/kg/hr with a specific gravity greater &amp;lt;1.013. This newborn shows signs of dehydration. Metabolic acidosis and electrolyte correct answer.
than 1.020. The nurse determines that this indicates: imbalance would be determined by serum, not by urine analysis.

1.‐ Adequate hydration.


2.‐ Dehydration.
3.‐ Metabolic acidosis.
4.‐ Electrolyte imbalance.

2514 Which neonate requires the closest observation by Correct answer: 2 Central cyanosis is always considered abnormal, and warrants further evaluation. All of the The key focus of the question is abnormal assessment findings in need of further
the nurse? other assessments are normal for an infant. investigation. Eliminate options 1, 3, and 4 because these assessment findings are normal
for an infant. Knowledge of the ominous sign of central cyanosis and the need for close
observation will also help to determine the correct answer.
1.‐ The neonate with irregular respirations at 30–40 breaths/minute
2.‐ The neonate whose color became cyanotic during the first feeding
3.‐ The baby who startles at loud sounds
4.‐ The neonate with enlarged breast tissue

2515 The nurse can best promote parental bonding with a Correct answer: 1 Parents should be given a Polaroid picture of the infant before the baby is transported. Calling Knowledge of care of the family and of promoting attachment with the high‐risk infant will
high‐risk newborn by doing which of the following? the unit to check on their baby might help bonding, but seeing the baby is more effective. aid in choosing the correct answer.
Parents are typically allowed to visit as often and for as long as they want. It is important to be
honest with parents, even if the prognosis is poor.

1.‐ Giving the parents a Polaroid picture of the baby prior to transport to the NICU
2.‐ Encouraging the parents to call the NICU daily
3.‐ Allowing parents to see the newborn for 15 minutes three times each day
4.‐ Not discussing how sick the infant is

2516 The nurse is observing a student practicing Correct answer: 4 Infants are obligate nose breathers. A gastric tube may be inserted to keep the stomach Critical words are "[t]he nurse must intervene if which of the following is observed." This
maintaining a patent airway on a newborn doll. The decompressed and allow for easier lung expansion. But if it is inserted nasally, it occludes one means that you are looking for an incorrect action on the part of the student. Knowledge of
nurse must intervene if which of the following is nare, and might make respiratory effort more difficult. All other options are correct the correct procedure to maintain a patent airway will be necessary to determine which of
observed? interventions for maintaining a patent airway. the answers gives an incorrect action.
1.‐ Suctioning the mouth, then the nose
2.‐ Squeezing the bulb syringe before insertion into mouth
3.‐ Positioning the infant in a "sniffing" position
4.‐ Inserting a nasogastic tube
2517 A priority nursing intervention for a newborn Correct answer: 1 The newborn reacts to hypothermia by burning brown fat to produce body heat. This process The key focus of the question is nursing care to reduce the risk to the newborn from
experiencing hypothermia is: requires oxygen and glucose. When an infant experiences hypothermia, glucose and oxygen hypothermia; a risk related to hypothermia is hypoglycemia. Eliminate option 2 because
needs increase, and hypoglycemia can result. The infant might require oxygen administration, infants should be rwarmed slowly to prevent hypotension. Eliminate option 3 because no
but the need should always be assessed first. Infants should be rewarmed slowly to prevent data have been given to support this action at this time. Eliminate option 4 because
hypotension. Phototherapy is not indicated. phototherapy is not indicated in this situation.
1.‐ Monitoring for hypoglycemia.
2.‐ Rapidly rewarming the newborn.
3.‐ Administering oxygen.
4.‐ Starting phototherapy.

2518 The nurse is caring for a 30‐weeks'‐gestation infant at Correct answer: 2 Changes in the gastrointestinal assessment, including abdominal distention, occur with NEC. The key focus of the question is necrotizing enterocolitis, a digestive disorder. Eliminate
risk for necrotizing enterocolitis (NEC). The nurse The other choices should be reported to the health care provider, but are not related to NEC. options 1, 3, and 4 because these assessment findings should be reported to the health care
should observe for which of the following? provider, but are not related to NEC.

1.‐ A decrease in respiratory rate


2.‐ Abdominal distention
3.‐ Discolored feet
4.‐ A bulging fontanelle

2519 The parents of a 28‐weeks'‐gestation neonate ask the Correct answer: 2 Neonates generally aren't able to effectively coordinate sucking, swallowing, and breathing The wording of the question indicates that the correct option is also a true statement.
nurse, "Why does he have to be fed through a tube in until 34–36 weeks' gestation. If fed orally before that time, they are at greater risk of Knowledge of the preterm neonate's capabilities regarding nutritional intake will help to
his mouth?" The nurse's best response is that: aspiration. Typically, they will be fed through a gavage tube until they are able to drink from a choose the correct answer.
bottle or breastfeed. Intake can be accurately assessed with oral and gavage feedings. The
stomach of a preterm infant can digest small amounts of formula or breast milk. Thrush is an
oral yeast infection commonly caused during passage through the birth canal, and gavage
feedings will not prevent it from occurring.

1.‐ "It allows us to accurately determine the baby's intake."


2.‐ "The baby's sucking, swallowing, and breathing are not coordinated yet."
3.‐ "The baby's stomach cannot digest formula at this time."
4.‐ "It helps to prevent thrush, an infection that could affect the baby's mouth."

2520 Which nursing diagnosis should be the highest priority Correct answer: 1 Newborns compensate for hypothermia by metabolizing brown fat. This process requires Remember ABCs. The correct answer would be the option that contains a true statement
of the nurse who is caring for a preterm newborn? glucose and oxygen. Preterm newborns are at risk for hypoglycemia and respiratory distress, that could negatively impact breathing and circulation. Cold stress can contribute to
so hypoglycemia can further increase their needs for oxygen and glucose, and can cause respiratory distress.
serious complications. The other diagnoses are appropriate, but not the highest priority.

1.‐ Ineffective Thermoregulation related to lack of subcutaneous fat


2.‐ Anticipatory Grieving related to loss of "perfect delivery"
3.‐ Imbalanced Nutrition: Less than Body Requirements related to immature digestive system
4.‐ Risk for Injury related to thin epidermis

2521 A nurse is caring for a 12‐hour‐old newborn. The Correct answer: 2 Jaundice in an infant less than 24 hours old is often caused by Rh or ABO incompatibility. A The focus of the question is jaundice. Eliminate options 1, 3, and 4 because they do not
nurse notes a yellow tint to the baby's skin and sclera. direct Coombs' test determines the presence of maternal antibodies in the baby's blood. The provide data related to this abnormal condition.
What lab test should the nurse anticipate being other lab tests are not related to hyperbilirubinemia.
ordered?
1.‐ Blood glucose
2.‐ Direct Coombs'
3.‐ Blood culture
4.‐ Arterial blood gas (ABG)
2522 A newborn is admitted with a diagnosis of transient Correct answer: 2 Transient tachypnea of the newborn (TTN) is caused by delayed absorption of fetal lung fluid. Recall that transient tachypnea is associated with amniotic fluid in the newborn lungs.
tachypnea of the newborn (TTN). When planning Nursing care is focused on supporting oxygenation needs to allow the newborn's body to Eliminate options 1, 3, and 4 because they are not related to this problem.
nursing care for this baby, the nurse's goal should be reabsorb the fluid. TTN causes tachypnea, so stimulating respirations is not appropriate.
to: Inadequate surfactant is related to prematurity and respiratory distress syndrome. Meconium
in the airway results in meconium aspiration syndrome, and is usually associated with fetal
asphyxia.
1.‐ Promote adequate quantity of surfactant.
2.‐ Promote absorption of fetal lung fluid.
3.‐ Assist in the removal of meconium from the airway.
4.‐ Stimulate respirations.

2523 The nurse is assigned to a baby receiving Correct answer: 3 Infants should be unclothed while receiving phototherapy, to increase the circulating blood The focus of the question is an abnormal finding in the present treatment of jaundice.
phototherapy. Which assessment warrants further volume exposed to the phototherapy light. However, this increases the risk of temperature Eliminate options 1, 2, and 4 because they are normal findings in a newborn with jaundice
investigation by the nurse? instability, and infant temperatures should be monitored carefully. Any temperature below being treated with phototherapy.
97.6°F is considered hypothermia, and requires immediate attention. Loose green stools and a
yellow tint to the skin are expected findings with hyperbilirubinemia. A fine, raised red rash
might appear on the infant's skin as a side effect of the phototherapy, and does not require
intervention.
1.‐ Loose green stools
2.‐ Yellow tint to the skin
3.‐ Temperature 97.2°F
4.‐ Fine, red rash on trunk

2524 A mother is crying while sitting by the isolette of her Correct answer: 4 Reflection allows the client to verbalize her feelings. The nurse should not give the client false The key focus of the question is therapeutic communication. The correct answer would be
premature newborn, who was born at 25 weeks' hope. Clients often do not know why they feel the way they do, and it is not helpful to ask the option that validates the client's feelings and invites further communication by the
gestation. The most therapeutic communication by the them. Some clients might find comfort in a religious leader, but care should be taken not to client.
nurse is: stereotype the client's religious beliefs.
1.‐ "It's important to try not to worry. Let's hope that everything will work out."
2.‐ "Can you tell me some specific things that have gotten you upset?"
3.‐ "Would you like me to call the hospital chaplain? This has helped many others."
4.‐ "This must be hard for you. Can you share with me what has you most concerned at this time?"

2525 A baby's mother is HIV‐positive. Which of the Correct answer: 2 Administering zidovudine (ZDV, formerly AZT) to the mother prenatally and intrapartally, and The core focus of the question is reduction of HIV transmission from mother to infant.
following interventions is most important for the nurse to the infant immediately after delivery, decreases the prenatal risk of transmission of HIV by Eliminate option 1, as it would increase risk of transmission. Eliminate options 3 and 4, as
to include when planning care for this newborn? 60–70%. Breastfeeding is contraindicated in an HIV‐positive mother because the virus can be they are not related to HIV transmission.
passed through breast milk. Cuddling the infant is important, but not the highest priority in this
situation. Decreasing environmental stimulation is not indicated.

1.‐ Encourage the mother to breastfeed.


2.‐ Administer zidovudine (ZDV) after delivery.
3.‐ Cuddle the baby as much as possible.
4.‐ Place the baby's crib in a quiet corner of the nursery.

2526 The nurse is preparing to initiate bottle feeding in a Correct answer: 4 Any sustained respiratory rate higher than 60 breaths/minute increases the risk of aspiration The focus of the question is identification of abnormal findings that would contraindicate
preterm infant. In which of the following situations in the infant. Oral feedings should be withheld on infants experiencing tachypnea to decrease feeding. The correct answer would be the option that contains an abnormal finding related
would the nurse withhold the feeding and notify the the risk of aspiration. An apical heart rate of 120 is a normal finding. Although an infant to a condition that could be exacerbated by feeding.
health care provider? temperature of 97.2°F is considered hypothermia, it would not be a contraindication to oral
feedings. Jaundice can be considered abnormal, but it alone would not be an indication to
withhold an oral feeding.
1.‐ Apical heart rate 120
2.‐ Axillary temperature 97.2°F
3.‐ Yellow tint to skin and sclera
4.‐ Respiratory rate 72
2527 A newborn's mother has a history of prenatal narcotic Correct answer: 2, 5 Infants experiencing neonatal abstinence syndrome (NAS) often have an increased need for The focus of the question is a newborn experiencing abstinence syndrome, a condition
abuse. Which of the following nursing interventions non‐nutritive sucking, and offering a pacifier would help meet this need. The other three associated with hyperstimulation. The correct answer(s) would be the option(s) that reduce
would be most appropriate for this infant? Select all answers are incorrect because they all involve increasing environmental stimulation. This is stimulation and quiet the infant.
that apply. contraindicated in these infants because they are already hyperstimulated from the drug
withdrawal process. Place the infant on the right side or in semi‐Fowler's position to avoid
possible aspiration of vomitus or secretions.

1.‐ Hold and rock the infant as much as possible.


2.‐ Offer the infant a pacifier.
3.‐ Place a mobile on crib.
4.‐ Encourage family members to stroke and talk to the infant.
5.‐ Position the infant on the right side or in semi‐Fowler's position.

2528 The nurse is caring for a preterm infant who is at risk Correct answer: 2 Increasing occipital frontal circumference (OFC) is an indication of increasing intracranial The key focus of the question is intraventricular hemorrhage, frequently manifested by
for an intraventricular hemorrhage (IVH). Which daily pressure, which could result from an intraventricular hemorrhage (IVH). It should be assessed increasing fetal head circumference. Associate the word "intraventricular" in the stem with
assessment is most critical for this infant? in infants at risk for an IVH every 8–12 hours. Changes in blood pressure also might occur, but the assessment of the head in the correct option, recalling that soft fontanelles will lead to
the changes might not be as noticeable, and can be caused by many other problems. Changes increasing head circumference in the presence of bleeding in the cranium.
in Moro reflex are not an indication of an IVH. Intake and output are routine measurements
that are not directly helpful in this situation.

1.‐ Blood pressure


2.‐ Occipital frontal circumference (OFC)
3.‐ Intake and output
4.‐ Moro reflex

2529 The nurse is admitting a neonate two hours after Correct answer: 2, 5 Nasal flaring and retractions could be signs of respiratory distress, and require immediate Critical words are "neonate two hours after delivery" and "About which assessment data
delivery. About which assessment data should the intervention. The other assessment data are normal findings for a neonate at 2 hours of age. should the nurse be concerned". This indicates the need to look for an abnormal sign that
nurse be concerned? Select all that apply. indicates a problem. Only options 2 and 5 indicate abnormal findings.

1.‐ Hands and feet blue.


2.‐ Nasal flaring
3.‐ Minimal response to verbal stimulation
4.‐ Apical heart rate 156
5.‐ Retractions

2530 Of the following nursing diagnoses for a high‐risk Correct answer: 3 Maintaining a patent airway is the highest priority when providing care for a newborn. A Remember ABCs. Maintaining an open airway would be the priority.
newborn, which requires the most immediate newborn's condition will deteriorate rapidly without a patent airway.
intervention by the nurse?
1.‐ Acute Pain related to frequent heelsticks
2.‐ Imbalanced Nutrition: Less than Body Requirements related to limited oral intake
3.‐ Ineffective Airway Clearance related to pulmonary secretions
4.‐ Deficient Knowledge related to infant care needs

2531 On admission to the nursery, it is noted that the Correct answer: 1 This newborn is at risk for sepsis caused by prolonged rupture of membranes and maternal The focus of the question is the risk for neonatal sepsis. The correct answer would be the
mother's membranes were ruptured for 48 hours fever. A primary sign of sepsis in the newborn is temperature instability, particularly option that contains abnormal assessment data related to infection in the newborn.
before delivery, and her temperature is 102°F. What hypothermia. An irregular respiratory pattern is normal. Jitteriness could be a sign of Eliminate options 1, 3, and 4 because they are not related to sepsis.
information from this newborn's assessment should hypoglycemia. Excessive bruising is often related to a difficult delivery with an increased risk of
the nurse evaluate further? hyperbilirubinemia.
1.‐ Axillary temperature 97.2°F
2.‐ Irregular respiratory rate
3.‐ Jitteriness
4.‐ Excessive bruising of presenting part

2532 The nurse would take which of the following actions Correct answer: 2 Neonatal abstinence syndrome, or drug withdrawal, causes hyperstimulation of the neonate's Recall that neonatal abstinence syndrome is accompanied by hyperstimulation of the
as part of nursing care of the baby experiencing nervous system. Nursing interventions should focus on decreasing environmental and sensory central nervous system. The correct answer would be the option that contains a strategy to
neonatal abstinence syndrome? stimulation during the withdrawal period. reduce stimulation. Eliminate options 1, 3, and 4, as they increase stimulation.

1.‐ Place stuffed animals and mobiles in the crib to provide visual stimulation.
2.‐ Position the baby's crib in a quiet corner of the nursery.
3.‐ Avoid the use of pacifiers.
4.‐ Spend extra time holding and rocking the baby.

2533 A mother was diagnosed with gonorrhea immediately Correct answer: 1 A newborn can become infected with gonorrhea as it passes through the birth canal. The focus of the question is providing safety for the newborn of a mother with a
after delivery. When providing nursing care for her Gonorrhea can cause permanent blindness in the newborn, called ophthalmia neonatorum. All gonococcal infection. The correct answer would be the option that addresses preventing
baby, an important goal of the nurse is to: babies' eyes are treated with an antibiotic prophylactically after birth. the spread of the infection from the mother to the infant.

1.‐ Prevent the development of ophthalmia neonatorum.


2.‐ Lubricate the eyes.
3.‐ Prevent the development of thrush.
4.‐ Teach the danger of breastfeeding with gonorrhea.

2534 A full‐term newborn weighed 10 pounds, 5 ounces at Correct answer: 2 Newborns experiencing macrosomia are more likely to experience birth injuries during The core focus of the question is an abnormally large infant. The correct answer would be
birth. A priority nursing diagnosis for this baby is: delivery. Nursing care after delivery should focus on assessing for signs of birth injuries and the option that addresses a risk for this newborn.
intervening, if appropriate.
1.‐ Ineffective Thermoregulation related to lack of subcutaneous fat.
2.‐ Risk for Injury related to macrosomia.
3.‐ Impaired Gas Exchange related to lack of surfactant.
4.‐ Deficient Knowledge related to newborn care.

2535 The nurse finds the mother of a 28‐weeks'‐gestation Correct answer: 3 Reflecting on what the client said offers her an opportunity to share her feelings. Avoid giving The key focus of the question is therapeutic communication. The correct answer would be
infant crying in her room. The mother states, "I just false reassurance or asking a client "why" they feel the way they do. the option that validates the client's feelings.
know my baby is going to die." What is the most
therapeutic response by the nurse?

1.‐ "I know this seems overly optimistic, but it is likely that everything will be fine."
2.‐ "Why do you think that?"
3.‐ "You seem very worried about what will happen to your baby."
4.‐ "My baby was born at 27 weeks, and he is fine now."

2536 A nurse is admitting the infant of a diabetic mother Correct answer: 2 Infants of diabetic mothers are at risk for hypoglycemia after delivery. A primary sign of Knowledge of the care of the newborn of a diabetic mother will aid in answering the
(IDM). At 1 hour of age, the nurse notices that the hypoglycemia is jitteriness. The newborn is not showing any signs of hypoxia, so oxygen would question correctly. Recall that blood glucose is the primary test to assess diabetic control.
newborn is very jittery. Which action by the nurse is not be appropriate. Putting the newborn under a warmer or on a monitor would not harm the
most appropriate? infant, but these are not the priority interventions at this time.

1.‐ Begin oxygen by nasal cannula.


2.‐ Assess the newborn's blood sugar.
3.‐ Place the newborn under a radiant warmer.
4.‐ Initiate use of a cardiac/apnea monitor.
2537 A newborn's temperature is 97.4°F. The priority Correct answer: 4 This newborn has a low temperature, and the nurse must intervene quickly to prevent The focus of the question is an abnormal finding indicating cold stress. The correct answer
nursing intervention is to: complications related to hypothermia. Wrapping the baby in warm blankets and covering the would be an intervention to counteract this problem and safely warm the newborn.
head will help prevent heat loss through conduction, convection, and radiation, and are the
most important initial interventions. A baby can lose a large amount of heat from his head, so
keeping it covered will help stabilize the temperature.

1.‐ Notify the physician or nurse practitioner immediately.


2.‐ Take the newborn to the nursery, and observe for two hours.
3.‐ Reassess the temperature in four hours.
4.‐ Wrap the newborn in two warm blankets, and place a cap on the head.

2538 A nurse is assessing a neonate born 12 hours ago, and Correct answer: 2 This newborn has signs of jaundice, which include a yellow tint to the sclera and skin. Jaundice This question requires additional assessment of jaundice, an abnormal finding in a
notes a yellow tint to the sclera. The nurse should read is considered pathologic if it occurs in the first 24 hours of life, when it is most often caused by newborn at this age. The correct answer would be the option that contains information
the medical record for what other assessment that is Rh or ABO incompatibility. It would be important to assess both the mother's and newborn's related to pathologic jaundice.
important to note at this time? blood type and Rh factor to determine if this could be causing the jaundice. A bilirubin level
should also be obtained.
1.‐ Blood glucose
2.‐ Blood type and Rh factor of mother and newborn
3.‐ Most recent blood pressure
4.‐ Length of time prior to delivery that membranes ruptured

2539 The nurse determines that the client who is at Correct answer: 2 A rapid labor and delivery can cause exhaustion of the uterine muscle and prevent First, eliminate options that are similar (options 3 and 4), because they identify infants of
greatest risk for postpartum hemorrhage is the one contraction of the uterus after delivery, which controls the amount of bleeding. The infants in similar size who were delivered within reasonably similar time frames. Choose option 2
who delivered which of the following infants? the other options either were not identified for length of labor (option 1) or were delivered over 1 because of the very short duration of labor.
after 9‐hour and 12‐hour labors (options 3 and 4).
1.‐ 5‐pound, 12‐ounce infant
2.‐ 6‐pound infant after a 2‐hour labor
3.‐ 7‐pound, 6‐ounce infant after a 9‐hour labor
4.‐ 8‐pound infant after a 12‐hour labor

2540 The nurse is preparing for beginning‐of‐shift rounds Correct answer: 2 A hematoma is a collection of blood in the pelvic tissue caused by damage to a vessel wall The core issue of the question is knowledge of fetal size at the time of delivery as a risk
on assigned postpartum clients. After reviewing the without laceration of the tissue. A gestational diabetic client is more prone to have a large factor for hematoma formation. Eliminate option 1 first because of the infant s size; next,
assignment, the nurse plans to assess for hematoma baby that could cause tissue trauma during delivery. She had to be delivered with forceps, eliminate option 3 because twins are more likely to be small than large. Choose option 2
formation in which of the following clients, who is at which is another high‐risk factor for developing a postpartum hematoma. Maternal age does over 4 because option 4 addresses age rather than size.
greatest risk for this postpartum complication? not affect the development of a hematoma (options 1 and 4). The size of the newborn, rather
than the number, determines risk for hematoma formation (option 3).

1.‐ A 17‐year‐old client who gave birth to a small‐for‐gestational‐age infant


2.‐ A 26‐year‐old client with gestational diabetes and forceps delivery of a large‐for‐gestational‐age infant
3.‐ A 35‐year‐old client having twins
4.‐ A 40‐year‐old client having her first infant

2541 The clinic nurse receives a telephone call from a Correct answer: 3 Late‐postpartum hemorrhage occurs anytime after the first 24 hours postdelivery. The causes Specific knowledge of how to discriminate etiology of early‐ and late‐postpartum
seven‐day postpartum client who states she is having of early hemorrhage are uterine atony, DIC, hematomas, and lacerations. This leaves retained hemorrhage is needed to answer this question. Use nursing knowledge and the process of
increased vaginal bleeding and asks if it is serious, and placental fragments as the cause for late‐postpartum hemorrhage. The retained fragments elimination to make your selection.
what could be causing it. The nurse suspects which of undergo necrosis, forming fibrin deposits. These deposits form polyps, which eventually detach
the following, the most common cause of such late‐ from the myometrium, causing hemorrhage.
postpartum hemorrhage?

1.‐ Uterine atony


2.‐ Disseminated intravascular coagulation (DIC)
3.‐ Retained placental fragments
4.‐ Laceration
2542 The postpartum nurse would utilize which of the Correct answer: 3 Adequate fluid intake (up to 3,000 mL/day) prevents urinary stasis, dilutes urine, and flushes The core issue of the question is knowing how to decrease risk for UTIs. Recall that
following therapeutic measures to help prevent a out waste products, all of which help to prevent UTI. Bedrest is of no value (option 1). The increased fluid intake and intake of foods and beverages that yield acidic urine (cranberry
urinary tract infection (UTI) in an assigned client who client should attempt to void every few hours, rather than waiting to regain a sense of a full juice, ascorbic acid in high doses) can decrease the risk. With this in mind, eliminate each of
has just delivered an infant? bladder (option 2). While intake of juices is healthy (option 4), it is the large volume of fluid the incorrect options easily.
consumed that aids in flushing out wastes.
1.‐ Promote bedrest for 12 hours postdelivery.
2.‐ Discourage voiding until the bladder regains the sensation of being full.
3.‐ Force fluids to at least 3,000 mL per day.
4.‐ Encourage the intake of orange, grapefruit, or apple juice.

2543 A newly postpartum client is going into hypovolemic Correct answer: 4 Increasing the rate of IV fluids is an effective initial measure necessary to replace lost fluid The core issue of the question is fluid volume replacement. Eliminate each of the incorrect
shock as a result of uterine inversion. Which initial volume that occurs in uterine inversion caused by hemorrhage. Blood products might also be options because they do not replace fluids, although they might be helpful in a specific way.
order should the nurse expect to implement to restore necessary, but generally take some time to obtain from the blood bank. Oxygen also would be
fluid volume? given (option 1) to increase perfusion to tissues, but does not restore circulating volume. An
oxytocic drug (option 2) will help to limit further bleeding, but will not restore circulating
volume. Monitoring pulse is an assessment, and will not limit the condition (option 3); an
intervention is needed in this situation.

1.‐ Administer oxygen at 3–4 L/min via nasal cannula.


2.‐ Administer an oxytocic drug via IV.
3.‐ Monitor heart rate every five minutes.
4.‐ Increase the IV infusion rate.

2544 A client delivered by vaginal birth after cesarean Correct answer: 4 A common risk associated with VBAC is uterine rupture. Pain in the abdomen and between Note the core issue of the question is recognition of internal hemorrhage due to uterine
(VBAC). During postpartum recovery, she suddenly the scapulae can occur when the uterus ruptures, the hemorrhage is concealed, and blood rupture. Eliminate each of the incorrect options because they fail to provide effective
complains of severe pain in the abdomen and between accumulates under the diaphragm. This is an emergency, and requires immediate medical treatment for this medical emergency.
her scapulae. She has a minimal amount of vaginal intervention, which is initiated by calling the physician. The client may be put in modified
bleeding. The nurse s priority action should be to: Trendelenburg position to manage shock, not Trendelenburg (option 1); uterine atony is not
the problem (option 2); and IV fluids would be increased, rather than maintained (option 3).

1.‐ Put the client in Trendelenburg position.


2.‐ Continue to assess for uterine atony.
3.‐ Maintain the rate of IV fluids.
4.‐ Notify the physician promptly.

2545 The nurse interprets that which client would be Correct answer: 3 The traditional definition of early postpartum hemorrhage after a vaginal birth is more than The core issue of the question is an understanding of criteria for postpartum hemorrhage.
classified as having early postpartum hemorrhage? 500 mL in the first 24 hours. Given this, each of the other options is incorrect. Use nursing knowledge and the process of elimination to make your selection.

1.‐ A client who had a blood loss of 350 mL in the first 24 hours after delivery
2.‐ A client who had a blood loss of 1,000 mL in the first 48 hours after a cesarean delivery
3.‐ A client with a more than 500 mL blood loss in the first 24 hours postdelivery
4.‐ A client with a blood loss of 500 mL in the first 48 hours postdelivery

2546 To prevent early‐postpartum hemorrhage in the Correct answer: 2 Maintaining contraction of the uterus is important in controlling bleeding from the placental Note that the critical word in the question is prevent. Use the process of elimination to
woman who just had a cesarean birth, the recovery site. Assessing the fundus every 15 minutes helps assure that this is taking place. Early make a selection. Note that none of the incorrect options address the root cause of
nurse should implement which of the following detection of a boggy uterus can lead to actions that will prevent hemorrhage. While the other postpartum hemorrhage.
measures? assessments might be appropriate for the client, they will not help to detect early‐postpartum
hemorrhage.
1.‐ Maintain an IV rate of 125 mL/hr.
2.‐ Assess the uterus for firmness every 15 minutes.
3.‐ Assess abdominal dressing for drainage.
4.‐ Monitor urinary output.

2547 A client has been taking methylergonovine maleate Correct answer: 1 Late‐postpartum hemorrhage most frequently occurs due to retained placental tissue. Consider first that the client is bleeding, and determine the most likely cause, retained
(Methergine) for uterine subinvolution. It has not been Dilatation and curettage is the vaginal procedure of choice to remove retained tissue from the placental fragments. Then visualize each of the surgeries described, and use the process of
effective in controlling late‐postpartum hemorrhage. uterus. The other procedures are abdominal surgeries, and are not used to treat this condition. elimination and nursing knowledge to choose the one that will effectively treat the
Which procedure does the clinic nurse anticipate will condition.
be ordered to correct the cause of this condition?

1.‐ Dilatation and curettage


2.‐ Laparotomy
3.‐ Hysterotomy
4.‐ Hysterectomy

2548 The husband of a client who delivered four days ago Correct answer: 2 Before providing further instructions, explain that these are signs of postpartum blues, which The core issue of the question is recognition and appropriate instruction regarding mood
calls the nurses station stating that his wife is happy is a normal process related to hormonal changes. Option 1 does not address the client s changes in the postpartum period. Use nursing knowledge and the process of elimination to
one minute and crying the next. He states, “She never concern. (In this case, the husband is the client.) Option 3 is excessive, and option 4 is make your selection.
was like this before the baby was born.” The nurse s unnecessary and excessive.
initial response should be to:

1.‐ Tell him to ignore the mood swings, as they will go away.
2.‐ Reassure him that this is normal in the postpartum period because of hormonal changes.
3.‐ Advise him to contact a psychiatrist immediately; this is the first step in postpartum psychosis.
4.‐ Instruct the husband in signs and symptoms of postpartum psychiatric disorders.

2549 The postpartum nurse reviews the clinical Correct answer: 4 Uterine atony accounts for 80–90% of all early (within first 24 hours) hemorrhage. Infants First, recall the causes of early postpartum hemorrhage, which might help you to easily
assignment, and determines that the client at greatest weighing between 5 and 7 pounds would not overdistend the uterus (option 1). The client s select the correct option. Alternatively, eliminate option 3 first because it is a different
risk for early‐postpartum hemorrhage is which of the age (option 2) also does not increase the incidence of postpartum hemorrhage. Endometritis postpartum complication, then option 1 because the baby is small, and finally option 2
following? (option 3) could cause late‐postpartum hemorrhage, not early hemorrhage. because it is irrelevant.

1.‐ A client with an infant weighing 5 pounds, 7 ounces


2.‐ A client who is 17 years old
3.‐ A client with endometritis
4.‐ A client with uterine atony

2550 While performing a postpartum assessment, the Correct answer: 1 Excessive bleeding must be evaluated and managed immediately to prevent excessive loss of The core issue of the question is knowledge of measures to reduce postpartum bleeding.
nurse notices the client’s lochia is very heavy. Which of blood and shock. Repositioning the client will do nothing. Waiting will only hurt the client. With this in mind, eliminate option 3 first because it delays action. Choose option 1 over
the following should be the nurse’s first response? Bleeding should be assessed immediately. options 2 and 4 because positioning will not correct a boggy uterus.

1.‐ Palpate and massage the uterus.


2.‐ Elevate the head of the bed to Fowler’s position.
3.‐ Reevaluate in 10 minutes to see if the problem has corrected itself.
4.‐ Place the client in modified Trendelenburg position.

2551 The postpartum nurse would use which of the Correct answer: 2 Calf pain upon dorsiflexion of the foot indicates a positive Homans' sign, a sign of Specific knowledge of assessment of thrombophlebitis is needed to answer this question.
following interventions as most effective in detecting thrombophlebitis. If there is any question of thrombus formation, the legs, especially the You can easily eliminate option 3, because legs should not be massaged, and option 4,
the development of thrombophlebitis? calves, should not be massaged, because doing so could dislodge a potential clot. Petechiae because the problem is not evidenced by bleeding into skin tissue. Choose option 2 over
are not a clinical sign of thrombophlebitis. option 1 because it is a more specific sign than elevated temperature.

1.‐ Monitoring the client’s temperature


2.‐ Assessing for Homans' sign
3.‐ Asking the client if she has pain when the nurse massages her leg
4.‐ Assessing for petechiae on the lower extremities
2552 The nurse expects that the client’s lochia and location Correct answer: 2 The fundus should be midline, two fingerbreadths below the umbilicus, with dark red lochia, Specific knowledge of changes in lochia during the postpartum period is needed to answer
of the uterine fundus on the second day after delivery which might contain small clots. This explains lochia rubra. Option 1 explains lochia alba, which this question. Use nursing knowledge and the process of elimination to make your
would most likely be documented as which of the does not occur until about 10 days postpartum. Option 3 describes lochia serosa, which usually selection.
following? occurs between days 4 and 9 postpartum. Option 4 describes a complication of subinvolution.

1.‐ Yellowish‐white lochia with no clots, fundus three fingerbreadths below the umbilicus
2.‐ Dark red lochia with small clots, fundus midline and two fingerbreadths below the umbilicus
3.‐ Pinkish‐brown lochia with no clots, fundus midline and four fingerbreadths below the umbilicus
4.‐ A large amount of bright red lochia with large clots, fundus midline and at the umbilicus

2553 The clinic nurse working with women during the Correct answer: 3 Option 3 indicates that the mother is interacting with her infant and accepting responsibility The core issue of the question is healthy adaptation to life with a new infant. Choose the
postpartum period would interpret that which of the for self‐care and care of her infant. Options 1 and 2 indicate potential psychiatric problems option that indicates the greatest resemblance to healthy behavior and adaptive coping.
following behaviors exhibited by a client is typical that require additional investigation. Option 4 requires investigation of why the mother is not
during this time? eating well, which is important during the postpartum period.

1.‐ The mother experiences feelings of depression as she assumes responsibility for her new baby.
2.‐ The mother does not take care of herself, but attends well to her infant.
3.‐ The mother is receptive to learning about her baby, and demonstrates bonding.
4.‐ The mother does not sleep or eat well, but tries to take care of herself.

2554 The nurse has been caring for a postpartum client Correct answer: 4 Applying a cold pack will minimize swelling, bleeding, and discomfort. Labial hematomas do The core issue of the question is knowledge of heat and cold applications to aid in
diagnosed with a right labial hematoma. When not necessarily need to be drained (option 3); they will usually resolve on their own. A hot pack reabsorption of hematoma. Use basic nursing knowledge and the process of elimination to
planning care, the nurse should explain to the client is incorrect because it will increase engorgement at the site via vasodilation (option 1). Witch make your selection.
that: hazel will not decrease the swelling in the area (option 2).

1.‐ A hot pack will be used to increase comfort and to decrease blood loss.
2.‐ Witch hazel pads will be applied to reduce discomfort.
3.‐ She needs to give informed consent for surgery to incise and drain the hematoma.
4.‐ A cold pack will help to decrease bleeding and reduce the swelling.

2555 A postpartum patient develops thrombophlebitis in Correct answer: 3 Bedrest is recommended following a diagnosis of thrombophlebitis, to help prevent an The core issue of the question is knowledge of measures to prevent complications of
her right calf, and is started on heparin therapy. Which embolus. Clients receiving heparin therapy should avoid aspirin and nonsteroidal anti‐ thrombophlebitis. Note that options 1 and 3 are opposites, which means that one of them
of the following interventions would be most inflammatory drugs because they will potentiate the action of heparin. Once the likely is correct. Use knowledge that activity can cause thrombi to yield emboli to choose
appropriate? thrombophlebitis resolves, the client should not experience any residual effects. option 3 over 1.
1.‐ Encourage the client to ambulate to reduce lower‐extremity swelling.
2.‐ Encourage the client to take salicylic acid (aspirin) for leg pain.
3.‐ Instruct the client to remain on bedrest to reduce the possibility of embolism.
4.‐ Inform the client that she will experience numbness in her leg for several months.

2556 A postpartum client receiving heparin asks whether Correct answer: 3 A woman can continue to breastfeed while on heparin. Heparin will not affect the Specific knowledge of acceptable medication to use while breastfeeding is needed to
she can continue to breastfeed. Which of the following breastfeeding, and requires no special precautions. Heparin does not pass to the breast milk. answer this question. Use nursing knowledge and the process of elimination to make your
is the best advice for the nurse to give? selection.

1.‐ She should stop breastfeeding immediately.


2.‐ She can continue to breastfeed, but must assess the baby daily for ecchymotic spots.
3.‐ Heparin will not affect the breastfeeding, and requires no special precautions.
4.‐ She should alternate breast‐ and bottle feeding.
2557 During a home visit, a postpartum client complains of Correct answer: 1 These symptoms are suggestive of mastitis, and require prompt attention by the client s The core issues of the question are recognition of mastitis and applying knowledge of
a reddened, swollen, and tender breast 10 days after physician. It is not therapeutic to wait for the symptoms to resolve on their own. Breastfeeding appropriate intervention. Use nursing knowledge and the process of elimination to make
delivery. Based on this finding, the nurse would advise does not have to be stopped if mastitis is present. your selection.
the client that:
1.‐ These symptoms suggest an inflammatory or infectious process, and require immediate physician notification.
2.‐ She should mention it to her physician at her two‐week checkup, because it will be abnormal if it continues after two weeks.
3.‐ This is normal breast engorgement, and should subside within another week.
4.‐ She has to stop breastfeeding immediately until the swelling and redness resolve on their own.

2558 The postpartum nurse is caring for a client who Correct answer: 2 DIC is a disorder of widespread microvascular clotting that can result in bleeding once clotting The core issue of the question is knowledge that DIC can be evidenced by bleeding once
developed disseminated intravascular coagulopathy factors are consumed. Vaginal bleeding can be excessive if a coagulation disorder is present. clotting factors have been consumed. When answering questions about DIC, look first for
(DIC) following a placenta previa. The nursing priority Antibiotics will not affect a clotting disorder. DIC does not affect a client s reflexes. Homans' an option that addresses bleeding.
would be to: sign is associated with thrombophlebitis, not with DIC.

1.‐ Assess Homans' sign hourly.


2.‐ Frequently monitor her vaginal bleeding.
3.‐ Administer antibiotics.
4.‐ Monitor reflexes hourly.

2559 The postpartum nurse is caring for a client with Correct answer: 1, 3, 4 Symptoms of pulmonary embolus include sudden onset of dyspnea, chest pain, anxiety, Specific knowledge of manifestations of pulmonary embolism is needed to answer this
thrombophlebitis. The nurse monitors the client for diaphoresis, elevated pulse, and hypotension. Confusion can also occur because of decreased question. Use nursing knowledge and the process of elimination to make your selection.
which of the following symptoms of complications? oxygenation to the brain resulting from loss of adequate gas exchange in the affected area of
Select all that apply. the lung. The client would not experience chills or fever; these are more indicative of infection.

1.‐ Confusion
2.‐ Sudden high fever
3.‐ Dyspnea
4.‐ Diaphoresis
5.‐ Sudden onset of chills

2560 The nurse observes that a postpartal client who Correct answer: 3 Heavy bleeding is an abnormal postpartal finding. Early hemorrhage can be caused by uterine Recognize the assessment findings in the question as abnormal; the time period indicates
delivered three hours ago has saturated four peripads atony or by a lacerated cervix. Palpation of the uterine fundus can determine uterine atony. early‐postpartal hemorrhage. Eliminate option 1 because subinvolution causes the majority
with bright red blood during the past hour. Her vital The client did not report excessive perineal pain or pressure, which would be caused by a of late‐postpartal hemorrhages occurring after the first 24 hours following delivery.
signs are stable. The nurse assesses her bleeding to be: hematoma. Blood is retained in the tissue with a hematoma, and is not usually visible on the Eliminate option 2 because blood is retained in the tissue with a hematoma, and is not
perineal pad. Subinvolution causes the majority of late‐postpartal hemorrhages occurring after usually visible on the perineal pad. Eliminate option 4 because this is not a normal finding.
the first 24 hours following delivery.

1.‐ A normal indication of subinvolution.


2.‐ Abnormal, requiring inspection for a hematoma.
3.‐ Abnormal, indicating the need to palpate the uterine fundus.
4.‐ Normal, requiring no further action by the nurse.

2561 A client has just been diagnosed with mastitis. The Correct answer: 1 The infant will not be affected by the infection in the mother's breast; it does not get into the The focus of the question is client education to facilitate resolution of mastitis. The correct
nurse should place highest priority on teaching the breast milk. The client needs to empty her breasts frequently to prevent stasis of milk, which answer would be the action that facilitates breastfeeding and milk flow.
client which of the following? can cause further problems with the mastitis. Stopping the breastfeeding or binding the
breasts would do nothing to help with the mastitis.
1.‐ Breastfeed frequently to prevent any stasis of milk.
2.‐ Nurse the infant only from the uninvolved breast.
3.‐ Stop breastfeeding totally to allow the infection to heal.
4.‐ Tightly bind the breasts with elastic bandages.
2562 The nurse notices that a client who has just had her Correct answer: 2 The client is demonstrating symptoms of depression. Primiparas without support are at Recall that adequate support systems are a key factor to psychological adaptation. The
first baby is staying in her room, has had no visitors, higher risk for postpartum depression. Determining her lack of support systems will help to correct answer would be the option that contains assessment of client support.
has not taken a shower since delivery, and keeps her assess her risk for depression and the need to develop an appropriate plan to deal with this
back turned to the baby's bassinet when the baby is in concern. The client's vital signs and symptoms of pain will not assist the nurse in helping the
the room. What further evaluation should the nurse client with symptoms of depression.
make?
1.‐ Assess the client's vital signs.
2.‐ Look at the chart to see if the client had anyone with her during labor and what kind of support system she has.
3.‐ Check to see if the client listed a contact person in case of emergency.
4.‐ Assess the client for pain.

2563 The nurse would place high priority on assessing Correct answer: 2 A client with gestational diabetes is more likely to have a large baby that could cause tissue The key focus of the question is hematoma, a complication associated with trauma to
which of the following postpartum clients who would trauma during delivery. The use of forceps during delivery is another risk factor for developing maternal tissues at the time of delivery. The correct answer would be the option that
be most likely to develop a hematoma? a postpartal hematoma. The age factor does not affect the development of a hematoma nor presents the greatest potential for maternal tissue trauma at birth.
does a small for gestational age infant.
1.‐ A 17‐year‐old who had a small‐for‐gestational‐age infant.
2.‐ A 26‐year‐old with gestational diabetes who required forceps to deliver her large‐for‐gestational‐age infant.
3.‐ A 35‐year‐old having twins.
4.‐ A 40‐year‐old having her first infant.

2564 The nurse determines that the client with which of Correct answer: 3 Late‐postpartal hemorrhage occurs anytime after the first 24 hours postdelivery. The causes Knowledge of the causes of late‐postpartal hemorrhage will be needed to answer this
the following is at greatest risk for late‐postpartal of early postpartal hemorrhage include uterine atony, DIC, hematoma, and lacerations. question. Focus on the critical word "late" in making a selection.
hemorrhage? Retained placental fragments are the primary cause of late‐postpartal hemorrhage. The
retained fragments undergo necrosis, forming fibrin deposits. These deposits form polyps,
which eventually detach from the myometrium, causing hemorrhage.

1.‐ Uterine atony


2.‐ Disseminated intravascular coagulation (DIC)
3.‐ Retained placental fragments
4.‐ Hematomas and lacerations

2565 Twenty‐four hours after delivery, a postpartal client Correct answer: 4 The client is complaining of symptoms that might indicate a urinary tract infection. Cystitis is Recognize these assessment findings as abnormal and indicative of urinary tract infection.
develops a temperature of 99.8°F, has been voiding not an uncommon infection after delivery. The low‐grade temperature, dysuria, and frequent The correct answer would be the option that promotes client safety. Reporting these
small amounts frequently, and complains of pain upon voidings of small amounts are symptoms of cystitis. Several factors place women at risk for findings so treatment can begin protects the client
urination. The nurse would take which of the following postpartal urinary tract infections. These include bladder trauma, stasis of urine caused by
actions? hypotonicity of the bladder, and catheterization during labor. Fifty percent of women who are
catheterized during labor develop a urinary tract infection. The other answers do not address
urinary tract infection.

1.‐ Notify the nursery that the client might have an infection, and separate the baby from the mother.
2.‐ Explain that some women have these symptoms.
3.‐ Suspect that the client needs to complain because of stress.
4.‐ Notify the physician of the symptoms.

2566 A postpartal client complains of sweating, has a Correct answer: 4, 5 All of these symptoms are normal findings in the first 24 hours postpartum. The body is Recognize these assessment findings as normal in the postpartal woman. Eliminate
temperature of 99.6°F, and has voided over 2,000 mL beginning to return to the prepregnancy state. The nurse can explain to the client that these options 1, 2, and 3 because they are actions based on abnormal findings not observed in
since delivery. Nursing actions would include which of are normal symptoms, and the reasons for the diaphoresis, frequent urination, and low‐grade this client.
the following? Select all that apply. temperature. Offering comfort measures such as a dry gown and linens will enhance the
client's feeling of well‐being.
1.‐ Notify the physician; the client is showing symptoms of puerperal infection.
2.‐ Draw a blood culture and sensitivity.
3.‐ Isolate the client in a private room.
4.‐ Explain to the client that these are normal postpartal changes.
5.‐ Offer a fresh, dry gown and bed linens to enhance comfort.

2567 The nurse explains to a breastfeeding client that Correct answer: 4, 5 Cracked and bleeding nipples provide an entrance for bacteria. Therefore, frequently checking The wording of the question tells you that the correct answer would be the option that
which of the following is generally helpful in the nipples for cracking will help to prevent mastitis, as will adequate instruction in proper contains a nursing action to prevent mastitis. Eliminate options 1, 2, and 3 because they do
preventing mastitis? Select all that apply. breastfeeding techniques prenatally. Routine antibiotics are not given to prevent mastitis. not help prevent mastitis.
Clients can develop bacteria that are antibiotic‐resistant. Waiting too long between feedings
and decreasing nursing time can lead to stasis of milk and clogged ducts, which contribute to
the development of mastitis.

1.‐ Decreased nursing time, to prevent sore nipples


2.‐ Decreased frequency in nursing, so the infant will be hungrier at each feeding
3.‐ Prophylactic antibiotics for all nursing mothers
4.‐ Frequent assessment of the condition of the nipples, to prevent cracking and bleeding
5.‐ Prenatal instruction on proper breastfeeding techniques

2568 Which of the following statements would the nurse Correct answer: 4 The client should take all the prescribed medication, and ask her physician about Knowledge of the nursing care of the client with mastitis will aid in choosing the correct
include in educating a client with mastitis? recommending an analgesic. The client should empty her breasts frequently, either through answer. The wording of the question tells you the correct answer is a true statement.
nursing the baby or through pumping her breasts. This will prevent stasis of the milk and
further clogged ducts, which could cause further complications and development of an
abscess.
1.‐ "You can stop the antibiotics when you feel better."
2.‐ "You should give the baby formula until you are better."
3.‐ "You can take any kind of over‐the‐counter analgesics, since medications do not go into breast milk."
4.‐ "Nurse your baby frequently to prevent stasis of milk and potential further complications."

2569 Which nursing measure would be appropriate for the Correct answer: 3 Leg exercises promote venous blood flow and prevent venous stasis while the client is still on The key focus of the question is prevention of thrombophlebitis; improving circulation
nurse to implement to prevent thrombophlebitis in the bedrest. Options 1 and 2 increase the risk for thrombophlebitis, while option 4 might not be prevents this complication. Eliminate options 1 and 2 because they increase the risk for
immediate recovery period following a cesarean birth? realistic. thrombophlebitis, while option 4 might not be realistic.

1.‐ Place a rolled blanket underneath the client's knees.


2.‐ Limit fluid intake for the first 24 hours.
3.‐ Assist the client in performing leg exercises every two hours.
4.‐ Ambulate the client as soon as the anesthesia wears off.

2570 Which of the following items of clothing worn by a Correct answer: 3 The postpartal woman is prone to develop superficial thrombophlebitis due to increased Recognize that all options include statements about hosiery; the probable complication to
postpartal client could possibly promote a problem for amounts of clotting factors in the blood during the postpartal period, as well as an increased be considered is thrombophlebitis. Eliminate option 1 because it prevents circulatory stasis.
the woman? amount of platelets and increased adhesiveness. Any restrictive clothing on the legs should be Eliminate options 2 and 4 because these items do not impede circulation.
avoided.
1.‐ Pantyhose
2.‐ Short athletic socks
3.‐ Knee‐highs
4.‐ Colored tights

2571 Which of the following actions by a lactating client Correct answer: 4, 5 Preventing stasis of the milk and emptying the breast frequently will help prevent mastitis. The question is worded in a positive manner, indicating that the correct options are items
would the nurse support to help the client prevent The other options will not. that will prevent mastitis. Use knowledge about the prevention of mastisis to choose the
mastitis? Select all that apply. correct answer.
1.‐ Apply vitamin E cream to soften the nipples.
2.‐ Wear a tight, supportive bra.
3.‐ When the client's nipples are sore, offer the infant a bottle.
4.‐ Encourage the client to breastfeed her infant frequently.
5.‐ Instruct the mother in breastfeeding shortly after birth, and review correct techniques often.
2572 The home health nurse is making a home visit to a Correct answer: 3 The signs of a postpartal infection would include a temperature of greater than 100.4°F on The focus of the question is an infection, an abnormal finding in postpartum. Postpartal
postpartal client. The nurse would document and two successive days after the first 24 postpartal hours; tachycardia; foul‐smelling lochia; and infections are usually located in the uterus. The correct answer would be the option that
report which of the following as a symptom of pain and tenderness of the abdomen. The pinkish lochia is normal, and the temperature might contains abnormal assessment data associated with uterine infection.
infection? indicate a cold or breast milk coming in. Bradycardia would be an unrelated finding.

1.‐ Pinkish lochia


2.‐ Bradycardia
3.‐ Abdominal tenderness
4.‐ Oral temperature of 99.2°F

2573 Which of the following nursing interventions would Correct answer: 3 A full bladder can cause uterine atony, and contribute to bleeding. If a client has Eliminate option 1 because overstimulation can cause uterine relaxation, and option 4
have the greatest direct effect on reducing postpartum hemorrhaged, a Foley catheter might also be needed to allow accurate measurement of urine because a tocolytic drug causes uterine relaxation; uterine relaxation would increase
hemorrhage? output, which is an indicator for kidney function. Overly aggressive stimulation of the fundus bleeding. Option 3 is incorrect because maternal position does not impact hemorrhage.
could cause decreased uterine tone; this is detrimental, because overstimulation of the uterine
muscle fibers can contribute to uterine atony. Avoid the Trendelenburg position because it has
been reported to interfere with cardiac and respiratory function by increasing pressure on
chemoreceptors and decreasing the area for lung expansion. A tocolytic agent relaxes the
uterus; in this case, an oxytocic drug to contract the uterus would be indicated.

1.‐ Continuous fundal massage to decrease bleeding and facilitate uterine contraction
2.‐ Trendelenburg position to facilitate cardiac function
3.‐ Bladder catheterization to maintain uterine contraction
4.‐ Administration of a tocolytic drug

2574 The nurse interprets that which factor in a client's Correct answer: 1 Factors contributing to postpartum endometritis include the introduction of pathogens with The focus of the question is risk for uterine infection. Eliminate options 2, 3, and 4 because
history places the woman at greatest risk for invasive procedures, prolonged labor, and prolonged rupture of membranes. The risk of they include common, noninvasive actions or data that represent normal birth experiences.
postpartal endometritis? endometritis is greatest after a cesarean delivery, especially after a long labor and prolonged
rupture of membranes. Options 2, 3, and 4 are not invasive, and do not increase the client's
risk for infection.
1.‐ Cesarean delivery after 24 hours of labor and failure to progress
2.‐ Use of external fetal monitoring during labor
3.‐ Ruptured membranes for four hours prior to delivery
4.‐ Spontaneous vaginal delivery after eight hours of labor

2575 After the delivery of a large‐for‐gestational‐age Correct answer: 1 Suspect lacerations if the client is bleeding and the fundus is firm. If the cause were uterine The critical concept in the question is continuous bleeding with a firm uterine fundus, a
infant, the nurse notes a client has bright red blood atony, the fundus would not be firm. When there are fragments of the placenta or the classic symptom of a laceration. Eliminate option 2 because bleeding would be concealed,
continuously trickling from the vagina. Her fundus is membranes, the uterus will not contract effectively. option 3 because the uterine is not firm with atony, and option 4 because this finding is
firm and located in the midline. The nurse concludes commonly associated with late‐postpartum hemorrhage.
that which of the following is the most likely cause of
this bleeding?
1.‐ Lacerations
2.‐ Hematoma
3.‐ Uterine atony
4.‐ Retained fragments of conception
2576 A client is in the immediate postpartal period after Correct answer: 4 Of the options given, the only one that immediately affects the bleeding is uterine massage. It The focus of the question is hemorrhage in the presence of uterine relaxation. The correct
delivery of a 9‐pound, 14‐ounce baby. The client is a might be important to start an IV with oxytocin at a rapid rate, and to allow the client to empty answer would be the option that contains a nursing action to contract the uterus and
gravida 6, para 5. The nurse has been checking the her bladder; however, the first action is to massage the uterus to stop or slow down the blood prevent further hemorrhage.
client every 15 minutes for the last 45 minutes. The flow.
client has been stable with a firm fundus and moderate
amount of lochia. As the nurse begins her final 15‐
minute assessment, she notices some new blood stains
on the top sheet, and discovers the client lying in a
pool of blood that covers the protective bedpad. The
fundus is located above the umbilicus and is boggy.
What would the nurse's first action be?

1.‐ Take the client's blood pressure.


2.‐ Put the client on a bedpan in case she needs to empty her bladder.
3.‐ Start an IV.
4.‐ Massage the uterus.

2577 A woman who delivered three weeks ago calls the Correct answer: 3 Mastitis most frequently occurs 2–4 weeks after delivery with initial flulike symptoms plus The focus of the question is breastfeeding and the potential complication of mastitis. The
postpartum unit with breastfeeding questions. She breast tenderness and redness. The client might be describing symptoms of a breast infection. correct answer would be the option that obtains further assessment data related to the
wants to know if it is all right to continue to breastfeed Sleep, lochia, and edema with visual disturbances are not associated with breast problems. breasts.
while she has the flu. She states that she feels achy all
over, has been having chills, and her temperature is
103°F. What question is important for the nurse to
ask?
1.‐ "Have you been sleeping well?"
2.‐ "What does your lochia look like now?"
3.‐ "Do you have any reddened areas or tenderness on your breasts, or unusual breast discharge?"
4.‐ "Do you have any swelling in your legs or visual disturbances?"

2578 It is most important for the nurse to have which drug Correct answer: 3 Protamine sulfate is the drug used to combat bleeding problems related to heparin overdose. The key focus of the question is anticoagulation related to heparin use. The correct answer
readily available when the client is being treated with Option 1 raises serum calcium levels. Option 2 is the antidote for warfarin. Option 4 is an iron would be the option that contains a drug with an action to combat bleeding.
heparin therapy for thrombophlebitis? supplement.

1.‐ Calcium gluconate


2.‐ AquaMEPHYTON
3.‐ Protamine sulfate
4.‐ Ferrous sulfate

2579 The homecare nurse is caring for a postpartal client, Correct answer: 1, 2, 3 Postpartum psychosis usually becomes evident within three months of delivery. Delusions The question is worded as a positive statement. The correct answers would be the options
and suspects the development of a postpartum and hallucinations are common. The risk for suicide or infanticide is increased by the psychotic that contain assessment findings for postpartal psychosis.
psychosis. Which of the following findings support the woman's distorted thoughts about herself and/or the baby. The psychotic woman would
nurse's judgment? Select all that apply. typically display agitation, hyperactivity, and confusion. Adjustment reaction with depressed
mood, commonly known as maternal or baby blues, occurs in 50–70% of women, and is
characterized by feelings of fatigue, anxiety, or being overwhelmed by the new maternal role.
A key feature is episodic tearfulness without reason that typically occurs within a few days of
birth and resolves spontaneously on about the 10th postpartal day.

1.‐ The client has a history of a bipolar (manic‐depressive) disorder.


2.‐ The client reports voices telling her the baby is evil, and must die.
3.‐ The client can't remember details of the delivery or when the infant last ate.
4.‐ The woman is tearful without an identifiable reason.
5.‐ The woman is calm and seated during the home visit.
2580 The client is a 36‐year‐old woman, gravida 6, para 6, Correct answer: 1 Women that are parity of 6 or higher (grandmultiparity) are at the greatest risk of uterine The focus of the question is identification of risk for uterine atony. Eliminate options 2
who delivered a baby girl at 38 weeks' gestation after atony due to repeated distention of uterine musculature during pregnancy. Labor leads to and 4, as this client's infant is of normal size, and labor was of average duration. Option 3
eight hours of labor. The baby weighed 7 pounds, 14 muscle stretching, diminished tone, and muscle relaxation. The client's age is not a factor in should be eliminated because maternal age is not a risk factor for hemorrhage.
ounces. The client's vital signs are stable, and her uterine atony; the length of labor is not considered to be prolonged or precipitous; and the
lochia is bright red and heavy, and contains various size of the baby is considered appropriate for gestational age, and is not considered to be
clots. The largest clot is about half‐dollar size. The macrosomic.
nurse would consider the client to be at high risk for
uterine atony because of which of the following?

1.‐ Grandmultiparity
2.‐ Size of the baby
3.‐ Length of labor
4.‐ Client's age

2581 A client continues to pass large amounts of clots and Correct answer: 4 Cervidil is used to ripen the cervix before labor; terbutaline sulfate is a tocolytic, and could The focus of the question is a second‐line agent to stimulate uterine contraction. Eliminate
bright red lochia despite the nurse's attempt to cause further muscle relaxation; magnesium sulfate is used to decrease contractions or options 1, 2, and 3 because none of the drugs identified possess this action.
massage the fundus. Upon reexamination, the nurse prevent seizures; and Hemabate is a prostaglandin, used to manage uterine atony. Oxytocin
finds that the client's uterine fundus remains boggy. remains the first‐line drug, the prostaglandins now are more commonly used as the second‐
The nursing actions and oxytocin (Pitocin) do not seem line drug, and carboprost (Prostin 15‐M or Hemabate) is the most commonly used uterotonin.
to be helping to keep the fundus firm. What second As many as 68% of clients respond to a single carboprost injection, with 86% responding by the
medication does the nurse anticipate the physician will second dose.
order to manage uterine atony?

1.‐ Dinoprostone (Cervidil)


2.‐ Terbutaline sulfate (Brethine)
3.‐ Magnesium sulfate
4.‐ Carboprost (Prostin 15‐M or Hemabate)

2582 A new mother with mastitis is concerned about Correct answer: 3 The organisms are localized in breast tissue, and are not excreted in the breast milk. The The wording of the question indicates that the correct option is a true statement.
breastfeeding while she has an active infection. The other answers are not correct. Knowledge of the care and pathophysiology of the woman with mastitis will aid in choosing
nurse should explain that: correctly.
1.‐ The infant is protected from infection by immunoglobulins in the breast milk.
2.‐ The infant is not susceptible to the organisms that cause mastitis.
3.‐ The organisms that cause mastitis are not passed in the milk.
4.‐ The organisms will be inactivated by gastric acid.

2583 If the nurse suspects a uterine infection in the Correct answer: 2 An abnormal odor of the lochia indicates infection in the uterus. The vital signs might be The critical words in the question are uterine infection. The correct answer would be the
postpartum client, the nurse should make which affected by an infection, but that is not definitive enough to suspect a uterine infection. A option that includes an assessment specific to uterine infection. Options 1, 3, and 4 should
priority assessment? distended abdomen usually indicates a problem with gas, perhaps a paralytic ileus. Inspection be eliminated because they are not specific assessments for uterine infection.
of the episiotomy site would not provide information regarding a uterine infection.

1.‐ Pulse and blood pressure


2.‐ Odor of the lochia
3.‐ Episiotomy site
4.‐ The abdomen for distention

2584 A postpartum client develops a temperature during Correct answer: 4 A temperature elevation greater than 100.4°F on two postpartum days not including the first The focus of the question is assessment data that define puerperal morbidity. This
her postpartum course. Which of the following 24 hours meets the criteria for infection. This criterion is the most common standard in the definition includes a time element (after 24 hours) and a threshold for elevated
temperatures indicates the presence of postpartum United States. It is not abnormal for a postpartum client to run a low‐grade fever in the first 24 temperature (greater than 100.4°F. Options 1, 2, and 3 should be eliminated because they
infection? hours. This can be caused by the body's reaction to labor, dehydration, or a reaction to do not meet the definition for postpartal infection.
epidural anesthesia. Postpartum nurses should assess other signs and symptoms of infection in
addition to fever and WBCs when evaluating the possibility of infection in mothers who had
epidural analgesia.
1.‐ 99.0°F 12 hours after delivery, decreasing after 18 hours
2.‐ 100.2°F 24 hours after delivery, decreasing on the second postpartum day
3.‐ 100.4°F 24 hours after delivery, remaining until the second postpartum day
4.‐ 100.6°F 48 hours after delivery, continuing into the third postpartum day

2585 For which of the following signs of thrombophlebitis Correct answer: 3 These are classic signs of thrombophlebitis that appear at the site of inflammation; the other The wording of the question indicates the correct option is a true statement. Knowledge
should the nurse instruct the postpartal client to look signs listed are not. of the signs and symptoms of thrombophlebitis will help to choose the correct answer.
when at home after discharge from the hospital?

1.‐ Muscle soreness in her legs after exercise


2.‐ Varicose veins in her legs
3.‐ Local tenderness, heat, and swelling
4.‐ Bruising

2586 Which instruction should the nurse include in the Correct answer: 4 An increase in lochia or a return to bright red bleeding after the lochia has changed to pink The wording of the question indicates the correct option is a true statement. Knowledge
discharge teaching plan to assist the postpartal client indicates a complication. The other statements are not early signs of complications. of complications for the postpartal client will aid in choosing the correct answer.
to recognize early signs of complications?

1.‐ Expect to pass clots, which occasionally can be the size of a small orange.
2.‐ Report any decrease in the amount of brownish‐red lochia.
3.‐ Palpate the fundus daily to make sure it is soft.
4.‐ Notify your health care provider of any increase in the amount of lochia or a return to bright red bleeding.

2587 A client delivered a 9‐pound, 10‐ounce infant assisted Correct answer: 4 Bleeding into the connective tissue beneath the vulvar skin can cause the formation of vulvar The question presents abnormal assessment data that warrant further assessment. The
by forceps. When the nurse performs the second 15‐ hematomas, which develop as a result of injury to tissues with spontaneous as well as correct answer would be the option that includes an action for the nurse to take to obtain
minute assessment, the client complains of increasing operative deliveries (use of forceps). One of the first signs of a hematoma might be complaint additional assessment findings related to perineal pain and pressure.
perineal pain and a lot of pressure. What action should of pressure, pain, or an inability to void. An ice pack to the perineum can be used to reduce
the nurse take? swelling, but a hematoma is abnormal, and should be reported to the physician. The fundus
should be assessed, but the client's complaints warrant perineal or vaginal assessment.

1.‐ Put an ice pack on the client's perineum, reassuring the client that this is normal.
2.‐ Call for assistance.
3.‐ Assess the fundus for firmness.
4.‐ Check the perineum for a hematoma.

2588 On the client's third postpartum day, the nurse enters Correct answer: 3 Creating an environment where a client and her family can discuss emotional concerns is The focus of the question is therapeutic communication. The correct answer would be the
the room and finds the client crying. The client states essential. Sharing time with the new mother to discuss thoughts and feelings is important to option that validates and explores the client's feelings.
that she doesn't know why she is crying, and that she clients. Responding with patronizing answers (options 1 and #4) does nothing to assist the
can't stop. Which of the following is the most mother to talk about her thoughts and feelings, and could increase her sense of isolation and
appropriate statement for the nurse to make? feelings of inadequacy and despair.

1.‐ "There is no need to cry, you have a healthy baby."


2.‐ "Are you dissatisfied with your care?"
3.‐ "Many new mothers have shared with us their same confusion of feelings. Would you like to talk about them?"
4.‐ "This happens to lots of mothers. You'll get over it."
2589 Because postpartum depression occurs in 8–26% of Correct answer: 4, 5 Risk factors for postpartum depression include primiparity; ambivalence about maintaining The focus of the question is risk factors for postpartum depression. Eliminate options 2
postpartal women, the prenatal nurse assesses clients the pregnancy throughout the pregnancy; history of previous depression or bipolar illness; lack and 3, as they contain findings common in a normal pregnancy. Option 1 can be eliminated
for risk factors for postpartum depression during the of a stable support system; lack of a stable relationship with parents or partner; poor body because the presence of support can reduce the risk of psychological complications.
prenatal period. Which of the following clients would image; and lack of a supportive relationship with parents, especially her father as a child.
the nurse consider to be at risk for postpartum Ambivalence regarding pregnancy is a normal response in the first and into the second
depression? Select all that apply. trimester, but should be resolved by the third trimester. Postpartum blues occur in
approximately 50–80% of postpartum women; the blues do not particularly indicate that a
woman will develop postpartum depression.

1.‐ A client who is an unmarried primipara with family support


2.‐ A client who has previously had postpartum blues
3.‐ A client who is a primipara with documented ambivalence about her pregnancy in the first trimester
4.‐ A client who is a primipara with a history of depression and lack of a supportive relationship
5.‐ A client who is an unmarried primipara living on her own who was consistently ambivalent about maintaining the pregnancy

2590 The nurse caring for a high‐risk client in labor Correct answer: 2 Variability of fetal heart rate indicates fetal well‐being. Loss of variability or decreased The core issue of the question is knowledge of the significance of variability in fetal heart
observes the presence of variability of the fetal heart variability (less than 2–5 beats per minute) is associated with depression of the autonomic rate. Recall that less or loss of variability might be a cause for concern, depending on the
rate of 10–12 beats per minute as recorded by the nervous system that regulates heart rate. Hypoxia can cause loss of variability of the FHR, as circumstances leading to it. Use nursing knowledge and the process of elimination to make
internal fetal monitor. Which of the following can maternal sedation and fetal sleep, though the last two are less serious signs. The presence your selection.
conditions does the nurse suspect? of variability is assessed by internal fetal monitoring, since there is less artifact that could be
mistaken for variability of heart rate.

1.‐ Fetal hypoxia


2.‐ Fetal well‐being
3.‐ Umbilical cord compression
4.‐ Uteroplacental insufficiency

2591 The nurse locates fetal heart tones in the upper‐right Correct answer: 3 Fetal heart tones are heard loudest over the fetal back. In breech presentation, this tends to Specific knowledge related to fetal position and associated location of fetal heart sounds is
quadrant. This finding should cause suspicion that the be above the umbilicus. Fetal heart tones are heard just below the midline of the umbilicus in needed to answer the question. Use nursing knowledge and the process of elimination to
fetus is in a(n): shoulder presentation or transverse lie. make your selection.
1.‐ Occiput posterior position.
2.‐ Occiput transverse position.
3.‐ Breech presentation.
4.‐ Shoulder presentation.

2592 A client s contractions have become less frequent and Correct answer: 1 The nurse should suspect CPD because of the lack of progress since the last exam. The The core issues of the question are recognition of lack of progress in labor and the nurse s
less intense in the past hour. Vaginal examination physician might assess the maternal pelvis by CT, MRI, or other means, or could stimulate decision‐making ability once this is detected. Eliminate options 2 and 3 first, because they
reveals 6‐cm dilatation and 0 station, indicating there contractions with oxytocin (Pitocin), opting for a trial of labor (TOL). Lack of progress could be do nothing to help labor progress again, and choose option 1 over 4 because there is not
has been no change since the last examination over caused by inadequate contractions, and a vaginal delivery could be possible, so it is too early to enough information yet to indicate that cesarean delivery is needed.
two hours ago. The nurse should take which of the anticipate cesarean delivery. Encouraging rest and continued observation will do nothing to
following actions at this time? resolve the problem.

1.‐ Notify the physician of the last exam.


2.‐ Continue to observe over the next hour for further progress.
3.‐ Encourage the client to turn on her side and relax.
4.‐ Anticipate the need for cesarean delivery.

2593 A delivery of a client who has shoulder dystocia is Correct answer: 1 Flexing the thighs against the abdomen (McRoberts maneuver) increases the pelvic angle Specific knowledge of the McRoberts maneuver is needed to answer this question. Use
best accomplished using the McRoberts maneuver. from symphysis pubis to sacrum, and facilitates delivery by making the bony pelvis less nursing knowledge and the process of elimination to make your selection.
The nurse should have the client perform this restrictive.
maneuver by doing which of the following?

1.‐ Flexing the thighs against the abdomen


2.‐ Placing her legs in stirrups
3.‐ Assuming a side‐lying position for delivery
4.‐ Sitting upright for delivery

2594 The nurse explains to a client with premature labor Correct answer: 3 Corticosteroids such as betamethasone have been shown to enhance fetal lung maturity and Specific knowledge of the purpose of betamethasone late in pregnancy is needed to
that betamethasone (Celestone) is administered for prevent respiratory distress. Betamethasone does not stop labor or cervical changes. A side answer this question. Use nursing knowledge and the process of elimination to make your
which of the following purposes? effect is increased risk of infection. selection.

1.‐ Stop uterine contractions.


2.‐ Prevent infection.
3.‐ Hasten fetal lung maturity.
4.‐ Prevent cervical dilatation.

2595 After teaching the client and her husband about Correct answer: 4 Signs of premature labor can include abdominal cramping and pressure, or persistent back The wording of the question tells you that the correct answer is an option that contains a
premature labor, the nurse recognizes that the pain. The client should be instructed to empty her bladder, lie down on her side, and drink 3–4 true statement. Eliminate option 3 because this symptom should not be ignored, and would
intervention was effective when the client states: cups of water. If symptoms do not disappear within an hour, the health care provider should not be relieved by rest. Eliminate option 1 because it does not have to do with labor, and
be notified. Unusual vaginal discharge should be reported sooner. Excessive fetal movement eliminate option 2 because back pain might or might not indicate labor. Also, option 4 is
can sometimes indicate fetal distress, but is not a sign of premature labor. very detailed and comprehensive, which is a clue that this is the correct choice.

1.‐ “I will call the office if I notice excessive fetal movement.”


2.‐ “I will come to the hospital if I have back pain.”
3.‐ “I will lie down and rest awhile if I notice watery vaginal discharge.”
4.‐ “I will call the office if I have abdominal cramps or pressure that does not go away after I drink three or four cups of liquid and rest for an hour.”

2596 The client is admitted in active labor with a breech Correct answer: 2 Fetal heart rate greater than 160 beats per minutes is considered fetal tachycardia, an early The core issue of this question is the ability to correlate knowledge of breech presentation
presentation. Which of the following signs would sign of distress. Meconium passage often occurs in breech presentation because of pressure with knowledge of fetal distress. Choose option 2 over the other options by recalling that
indicate fetal distress? on the presenting part, and is not an indication of fetal distress in this situation. Mild variable the normal fetal heart rate (FHR) is 120–160 beats per minute. An FHR outside this range is
decelerations and increased variability are not indications of fetal distress, and occur more generally a cause for concern regardless of the specific situation.
frequently in breech presentations.
1.‐ Meconium‐stained amniotic fluid
2.‐ Fetal heart rate of 180 beats per minute
3.‐ Mild, variable decelerations
4.‐ Increased fetal heart rate variability

2597 A nulliparous client has not made any progress in Correct answer: 4 Dilatation has stopped (arrested) after considerable progress. The cause could be hypotonic Note the critical phrases not made any progress and over two hours ago. Correlate these
cervical dilatation or station since she was 7 cm and 0 uterine contractions, malposition, or cephalopelvic disproportion. The terms prolonged phrases with the word arrest to eliminate options 1 and 2. Choose option 4 over 3 because
station over two hours ago. According to the labor (option 1) and protracted (option 2) mean that progress occurs at a very slow rate. Arrest of dilatation has not changed and because the word secondary implies that labor was active at
graph, or Friedman curve, this represents: descent (option 3) occurs when the station, rather than cervical dilatation, does not change. one time, which is true in this case.

1.‐ Prolonged deceleration phase.


2.‐ Protracted active phase.
3.‐ Arrest of descent.
4.‐ Secondary arrest of dilatation.

2598 The nurse explains to a client that the presence of a Correct answer: 1 Bandl’s ring forms when labor is obstructed. The upper uterine segment continues to thicken Specific knowledge of Bandl’s ring is needed to answer this question. Use nursing
Bandl s ring, which the client overheard from the while the lower segment thins and retracts. If left untreated, uterine rupture can occur. Bandl’s knowledge and the process of elimination to make your selection.
physician, will alter the client s plan of care because it ring necessitates cesarean section.
is:
1.‐ A serious complication requiring cesarean section.
2.‐ A constriction ring that could prolong labor.
3.‐ The normal physiologic division between the upper and lower uterine segments.
4.‐ An abnormal depression in the lower uterine segment.
2599 The client in labor tells the nurse that she overheard Correct answer: 3 Frequent contractions and increased uterine muscle tone impede the blood flow through The core issue of the question is knowledge of how hypertonic uterine contractions affect
the physician say she is having hypertonic uterine uterine arteries to the placenta. The incidence of umbilical cord compression is not increased, the well‐being of the fetus. Eliminate options 1 and 4 as least likely to happen, then
contractions. She is worried that this will harm the and hypertonic contractions are not necessarily associated with placental separation. While eliminate option 2 because the cord might or might not be compressed, depending on the
baby. The nurse explains that the contractions could maternal exhaustion and lactic acid accumulation can occur over time, they do not position of the fetus.
increase the risk of fetal distress because: immediately threaten fetal well‐being.

1.‐ Maternal exhaustion occurs, producing a buildup of lactic acids.


2.‐ Umbilical cord compression occurs, decreasing oxygen supply.
3.‐ Increased uterine tone and frequent contractions interfere with blood flow from mother to fetus through the uterine arteries.
4.‐ Placental separation can occur.

2600 After the initial care following amniotomy, the nurse Correct answer: 4 The risk of infection is increased after rupture of membranes. Therefore, the nurse should Recognize that the term amniotomy (‐otomy means “cutting into”) refers to artificial
should include which of the following assessments assess for signs of infection, including fever, foul‐smelling amniotic fluid, and tenderness. Blood rupture of the membranes. Correlate this with increased risk for infection as a complication
every two hours? pressure, pulse, and fetal movement are checked more often during active labor. Color and to choose option 4 as an indicator of infection.
consistency of amniotic fluid are assessed immediately after rupture, and each time, the
underpad is changed.
1.‐ Maternal blood pressure and pulse
2.‐ Fetal movement
3.‐ Color and consistency of amniotic fluid
4.‐ Oral temperature

2601 Which of the following conditions, if identified in the Correct answer: 4 Although not always preventable, uterine inversion can occur because of excessive traction Specific knowledge of the etiology and risks of uterine inversion is needed to answer this
pregnant client’s history, places her at increased risk on the umbilical cord during the third stage of labor with or without vigorous fundal massage question. Use nursing knowledge and the process of elimination to make your selection.
for uterine inversion during the current labor and to remove the placenta, especially if the placenta is implanted in the fundus. The other options
delivery? are not associated with inversion.
1.‐ Forceps delivery of the infant
2.‐ Fundal pressure during delivery of the head and body
3.‐ Precipitous birth of less than three hours' duration
4.‐ Traction on the umbilical cord and vigorous fundal massage in the third stage

2602 Which of the following is the priority nursing goal in Correct answer: 1 While all of the answers are appropriate goals, establishing a trusting relationship with the Note the critical word priority in the question, which tells you all options might be partially
helping a client during a complicated labor? client and her family is a priority. In an emergency situation, the nurse might have little time to or totally correct, and you must choose the most important one. A client experiencing a
ensure that the client knows what to expect, or to protect her privacy. It is not always possible complicated labor is likely to experience both fear and lack of knowledge. Choose option 1
to prevent fear and anxiety. A trusting relationship increases the likelihood of cooperation and over the others because a trusting relationship is the foundation for assisting the client
compliance during a crisis. through the labor process and reducing fear and lack of knowledge.

1.‐ Establish a trusting relationship.


2.‐ Ensure that the client knows what to expect.
3.‐ Prevent invasion of privacy.
4.‐ Prevent fear and anxiety.

2603 Late decelerations on the fetal monitor should cause Correct answer: 3 Uteroplacental insufficiency (UPI) is believed to be the cause of late decelerations. The Specific knowledge of the significance of late decelerations is needed to answer this
the nurse to suspect which of the following insufficiency or decreased uteroplacental blood flow leads to fetal hypoxia. Several factors question. Use nursing knowledge and the process of elimination to make your selection.
conditions? including maternal hypotension, anemia, vasoconstriction, uterine tetany, and dehydration can
be primary causes of UPI. Head compression (option 1) causes early deceleration, and cord
compression (option 2) causes variable deceleration. Option 4 is incorrect because it might not
lead to UPI and eventual late deceleration.

1.‐ Head compression


2.‐ Cord compression
3.‐ Decreased uteroplacental blood flow
4.‐ Close uterine contractions

2604 A client asks what trial of labor means. Which of the Correct answer: 1 A trial of labor means that the client will be followed closely and given more time to show Eliminate option 4 because it provides knowledge to the client in retrospect, while nursing
following is the best response by the nurse? progress before considering a cesarean. Options 2 and 3 make cesarean delivery seem information is given in a current and timely manner whenever possible. Eliminate option 3
inevitable, and can increase the client s anxiety. Option 4 is incorrect because the client will be because it does not directly answer the client s question. Choose option 1 over option 2
allowed to continue laboring as long as some progress is made. because of the time frame and because there is an undercurrent of threat in the wording of
option 2.
1.‐ “The doctor has decided to give you more time to make progress in labor before considering a cesarean delivery.”
2.‐ “You will be expected to make progress in the next hour, or a cesarean will be done.”
3.‐ “Even though your pelvis is small, sometimes it is possible to deliver your baby vaginally.”
4.‐ “A cesarean delivery will be done because you have had a trial of labor, and have not made much progress.”

2605 The nurse should suspect cephalopelvic disproportion Correct answer: 4 Cephalopelvic disproportion (CPD) means that the fetal head is too large to pass through the Specific knowledge of cephalopelvic disproportion is needed to answer this question. Use
after noting documentation of which of the following bony pelvis. Options 1 and 2 refer to a smaller‐than‐normal pelvis, but do not take into account nursing knowledge and the process of elimination to make your selection.
for a laboring client? the fetal head size. Option 3 refers to shoulder dystocia.
1.‐ Pelvic outlet is less than 9 cm.
2.‐ Midpelvis is contracted.
3.‐ Fetal shoulders are too large to pass through the bony pelvis.
4.‐ Fetal head is too large to pass through the bony pelvis.

2606 The nurse explains to a pregnant client at 37 weeks' Correct answer: 1 The Bishop score, an assessment of the mother’s physical readiness for labor, takes into Recall that a Bishop score focuses primarily on the mother rather, not on the fetus, which
gestation that a Bishop score is being completed to account cervical dilatation, effacement, consistency, cervical position, and station before eliminates option 2. Choose option 1 over options 3 and 4 because the stem indicates that
determine which of the following? contractions begin. The higher the score, the more likely a client can be successfully induced. the client is not yet in labor.

1.‐ The client’s readiness for labor


2.‐ The fetus’s readiness for labor
3.‐ Progress during induction
4.‐ Cervical changes in labor

2607 Which of the following priority items should the nurse Correct answer: 3 Self‐image refers to how a client feels about herself. A positive self‐image enables a client to Note that the focus of the question is on the client in active labor. With this in mind,
assess because of the potential impact on the laboring deal with labor and delivery realistically, even in the event of complications. Research has choose option 3 because it is the only option that specifically relates to the client's current
client’s psychological status? shown that self‐image impacts the laboring patient’s psyche. Other options have not been status.
identified as having a significant impact during labor.

1.‐ Attitude about parenting


2.‐ Relationship with the client’s own mother
3.‐ Self‐image
4.‐ Beliefs about health

2608 A client is experiencing contractions that occur every Correct answer: 2 Hypotonic uterine dysfunction occurs most often during the active phase. It is characterized Note that the question contains the critical words mild intensity and active phase.
3–4 minutes, of 35‐second duration and mild intensity, by contractions that have become further apart, less intense, and of shorter duration. Reasoning that active labor should be characterized by strong contractions, you would
during the active phase of labor. After making this Contractions are typically 2–3 minutes apart, strong, and last 45–60 seconds in the active select option 2 because it contains the word hypotonic. Alternatively, eliminate options 3
assessment, the nurse suspects which of the phase of labor. and 4 because they are similar, and eliminate option 1 because the word hypertonic
following? conveys the opposite of what the client in the question is experiencing.

1.‐ Hypertonic uterine dysfunction


2.‐ Hypotonic uterine dysfunction
3.‐ Normal uterine activity
4.‐ Progressive labor pattern
2609 In preparing the client in labor for vacuum extraction, Correct answer: 1 Suction applied over the occiput commonly causes edema and bruising of the scalp. Although Note the critical word vacuum in the stem of the question, and eliminate each of the
it is important to teach her that the infant might it might appear to be a deformity of the fetal head, the edema disappears in 2–3 days. Suction incorrect options because they refer to a part of the face, rather than to the occiput of the
initially have which of the following after delivery? is not applied to the face (options 2, 3, and 4). head.

1.‐ A large caput and bruising of the scalp


2.‐ Red marks on the face
3.‐ Edema of the face
4.‐ Swelling of the eyes

2610 While performing a vaginal examination, the nurse Correct answer: 1 A compound presentation involves two fetal parts. The fetal head and a hand are the most The core issue of the question is a compound presentation, two different fetal parts.
should suspect a compound presentation if what is common combination, although hand‐and‐foot presentation is also possible. Option 2 is Eliminate option 2 unless a head is also presenting, since this could occur with shoulder
palpated? incorrect unless a head is also presenting, since this could occur with shoulder presentation. presentation. Eliminate option 3 because only one fetal part is presenting, the shoulder.
Both feet presenting is called "double footling breech presentation." Eliminate option 4 because both feet presenting is called "double footling breech
presentation."
1.‐ Presentation of two fetal parts
2.‐ Protrusion of the hand and arm through the cervix during labor
3.‐ A shoulder presentation
4.‐ Both feet protruding through the introitus

2611 The nurse should encourage the pregnant client not Correct answer: 3 Molding of the fetal head does not occur during labor in the breech presentation. The fetal The core issue of the question is the risk from pushing during vaginal breech delivery.
to push during vaginal delivery of a breech infant to body can pass through an incompletely dilated cervix, leaving the larger, firmer fetal head Eliminate options 1 and 2 because prolapsed cord and fetal distress are risks inherent with
avoid which of the following: entrapped. The woman might feel a strong urge to push before complete dilatation due to breech delivery whether the woman pushes or not. Eliminate Option 4 because cervical
pressure from the fetal body, increasing the risk of head entrapment. Options 1 and 2 are lacerations most often occur at the time of delivery from the application of forceps or
incorrect because prolapsed cord and fetal distress are risks inherent with breech delivery manipulation of the fetus to deliver the after‐coming head, rather than from pushing.
whether the woman pushes or not. Option 4 is incorrect, since cervical lacerations most often
occur at the time of delivery from the application of forceps or manipulation of the fetus to
deliver the after‐coming head.

1.‐ Prolapsed cord


2.‐ Fetal distress
3.‐ Fetal head entrapment
4.‐ Cervical lacerations

2612 The nurse encourages limiting the number of vaginal Correct answer: 3 Even with strict adherence to aseptic technique, the risk of infection increases with frequent The focus of the question is identifying a risk from excessive vaginal examinations in labor;
examinations to minimize the client's risk of: vaginal examinations, especially if membranes are ruptured, or if labor is prolonged. Most the greatest risk is infection. Options 1, 2, and 4 can be eliminated because they do not
authorities recommend keeping the number of vaginal examinations to a minimum for this contribute to an increased risk of infection.
reason, as well as for client comfort. Cervical lacerations, bleeding, and loss of control are
incorrect.
1.‐ Cervical lacerations.
2.‐ Bleeding.
3.‐ Infection.
4.‐ Loss of control.

2613 A term fetus with a face presentation is in a left Correct answer: 1 If the chin (mentum) of the fetus presents and remains in a posterior position, vaginal Recall the memory aid 'Every darn fool in Rotterdam eats rotten eggrolls everyday.'
mentum posterior (LMP) position. The nurse delivery cannot occur. The inability of the fetus to flex and extend the head as it moves Eliminate options 2, 3, and 4 because normal flexion and then extension necessary for
anticipates which of the following? through the mid‐pelvis causes an arrest of descent. A cesarean delivery is necessary. The other delivery are not possible in the LMP position.
answers are incorrect.
1.‐ Cesarean delivery
2.‐ Vaginal delivery
3.‐ Prolonged labor
4.‐ Forceps‐assisted vaginal delivery
2614 The nurse is assigned to a pregnant client who is Correct answer: 2, 5 The uterus should be assessed for overdistention and an elevated resting tone due to The key focus of the question is amnioinfusion, the infusion of fluid into the uterus.
having an amnioinfusion. In addition to fetal heart rate overinfusion of solution into the uterine cavity or trapping of infused solution behind a Eliminate options 1, 3, and 4, as they are not specifically related to assessment during
and contractions, the nurse monitors which of the presenting part. The elevation in uterine tone can lead to fetal distress. Maternal vital signs are amnioinfusion.
following during the procedure? Select all that apply. also assessed, to determine physiological status. The other answers do not represent items
that require assessment during this procedure.
1.‐ Fetal movement
2.‐ Uterine tone
3.‐ Maternal vital signs
4.‐ Labor progress
5.‐ Uterine overdistention

2615 The nurse interprets that a pregnant client Correct answer: 4 External cephalic version involves abdominal manipulation to rotate the fetus from either a Knowledge of external cephalic version will aid in answering the question correctly. The
understands the purpose and the procedure of breech or shoulder presentation to vertex. Clients need to know that the attempt is not always question is worded as a positive statement. The correct answer would be the option that
external cephalic version when the client states: successful. Option 1 is not correct. Version involves turning the fetal body, not just the head. contains a true statement about a point of client education.
Staying in bed after a version is not necessary. The procedure will be stopped immediately if
there is maternal or fetal distress.
1.‐ "My doctor will place a hand inside my uterus and pull the baby into a head down position."
2.‐ "My baby's head will be turned slightly to make the delivery easier."
3.‐ "After the baby is turned, I must remain in bed."
4.‐ "The procedure will be stopped if my baby shows signs of distress."

2616 Which of the following interventions carried out by Correct answer: 2 Counterpressure over the sacral area during contractions is helpful in alleviating the lower Eliminate incorrect choices. Eliminate option 1, as IV analgesics are not always necessary.
the nurse would be most helpful for the client with back pain without the use of narcotics that can be transferred across the placenta to the fetus. Eliminate option 3 because ice packs have not been shown to be helpful. Eliminate option 4
lower back pain caused by occiput posterior position? Ice packs have not been shown to be helpful. Pelvic rocking has been helpful for back pain in because pelvic rocking has been helpful for back pain in the antepartum period, but
the antepartum period, but patients in labor seem to benefit more from conscious relaxation patients in labor seem to benefit more from conscious relaxation during contractions
during contractions (avoiding movement and tension). (avoiding movement and tension).

1.‐ Administer small amounts of the ordered analgesic IV.


2.‐ Provide counterpressure over the sacral area with the palm of the hand.
3.‐ Provide an ice pack to place over the lower back.
4.‐ Teach the client how to do pelvic rocking.

2617 For the pregnant client whose fetus has a face Correct answer: 4 The spiral (fetal) electrode used for internal monitoring penetrates 2 mm into the presenting The focus of the question is client safety with this abnormal fetal presentation. The correct
presentation, which of the following actions should the part. While this does not create a problem on the fetal scalp, the face should be avoided for answer would be the option that creates the greatest risk, and that therefore should be
nurse avoid? cosmetic reasons, and to avoid eye injury. None of the other actions would be harmful for a avoided.
patient with face presentation.
1.‐ Performing Leopold's maneuver
2.‐ Ambulating the client to the bathroom
3.‐ Placing the client in a high semi‐Fowler's position
4.‐ Monitoring of internal fetal heart rate

2618 A long loop of umbilical cord has prolapsed through Correct answer: 1 Covering the cord with warmed, saline‐soaked, sterile towels will prevent it from drying out The key focus of the question is promoting safety during this obstetric emergency.
the introitus. The nurse notes that it is still pulsating, and occluding blood flow until the delivery can be accomplished. The other answers do not Eliminate options 2 and 3 because they can impede uteroplacental blood flow. Eliminate
and the fetal monitor indicates that the fetal heart protect the umbilical cord. option 4 because this is an emergency situation that requires prompt and safe action.
rate (FHR) is between 110 and 120. In addition to
contacting the physician immediately, what action
should the nurse take?
1.‐ Cover the cord with a sterile, warm, saline‐soaked towel.
2.‐ Cover the cord with a dry, sterile towel.
3.‐ Continue to palpate for pulsations.
4.‐ Do nothing until the physician arrives.
2619 A nurse is monitoring a client receiving intravenous Correct answer: 5, 4, 1, Uterine hyperactivity decreases the oxygen supply to the fetus. The priority of the nurse is to Recognizing that the client is experiencing hypertonic uterine contractions will assist in
oxytocin (Pitocin) to induce labor, and identifies 3, 2 reduce uterine activity by stopping the oxytocin (Pitocin). The nurse then acts to increase fetal determining what the priority action would be to stop the oxytocin (Pitocin) that stimulates
hypertonic uterine contractions. List in order of oxygenation by administering oxygen by face mask to the mother and repositioning the client uterine contractions. Actions to increase fetal oxygenation would be next, followed by
priority the actions the nurse should take. Click and to the side, left side‐lying preferred, to improve uteroplacental blood flow. The nurse would assessment of the client's labor status and blood pressure.
drag the options below to move them up or down. then perform a vaginal examination to determine labor progress, and check for a prolapsed
cord. The nurse would assess the blood pressure for hypotension or hypertension.

1.‐ Administer oxygen by face mask at 8–10 L/min.


2.‐ Check the client's blood pressure.
3.‐ Perform a vaginal examination.
4.‐ Reposition the client.
5.‐ Stop the oxytocin (Pitocin).

2620 A multiparous client who has been in labor for almost Correct answer: 1 Nursing action should be directed toward preventing a rapid and uncontrolled delivery of the This question focuses on the imminent delivery of the infant in a safe manner. Eliminate
three hours suddenly announces that the baby is infant's head. Directing the client to pant prevents pushing. If time allows, the nurse may don options 3 and 4 because they decrease safety by leaving the client's bedside. Eliminate
coming. The nurse sees the infant crowning. Which of gloves, or obtain a towel or blanket, to support the fetal head. Delivery is imminent, so there option 2 because the action is desirable but not required in an emergency.
the following interventions should the nurse do first? might not be time to obtain sterile gloves or to contact the physician. The client should not be
left alone, so going to the nursing station to get the precipitous delivery tray is not an option.

1.‐ Ask the woman to pant while preparing to place gentle counterpressure on the infant's head as it is delivered.
2.‐ Quickly obtain sterile gloves and a towel.
3.‐ Retrieve the precipitous delivery tray from the nursing station.
4.‐ Telephone the physician using the bedside phone.

2621 The nurse determines that a client does not Correct answer: 4 Blood transfusions are not routinely given during cesarean sections. Although blood typing The wording of the question indicates that the correct options are incorrect statements.
understand what to expect during cesarean delivery and screening are often ordered prior to surgery, it is seldom necessary for a client to receive a Use knowledge of the care of the client undergoing a cesarean section to aid in determining
when the client states: (Select all that apply.) blood transfusion. IV lines are necessary for instillation of fluid, medications, and potential the appropriate options.
blood products during surgery. A Foley catheter is inserted to prevent bladder damage during
surgery, and an antacid is administered to prevent aspiration of acidic gastric contents, thus
reducing the risk of lung damage. The client's husband or primary support person is usually
present at the birth except in extreme emergencies.

1.‐ "An indwelling (Foley) catheter will be inserted before surgery."


2.‐ "My husband can be present during birth."
3.‐ "I may be given an antacid before surgery."
4.‐ "I will receive a blood transfusion during surgery."
5.‐ "I will not need an IV, since I will have an epidural anesthesia."

2622 The nurse concludes that deceleration of the fetal Correct answer: 1 The pattern described is a variable deceleration, which is associated with umbilical cord The key words in this question are deceleration and rapid return to baseline. Eliminate
heart rate from 130 to 70 beats per minute with compression. During variable decelerations, the FHR drops below 90 beats a minute very options 2, 3, and 4, as they are associated with decelerations that develop and resolve
contractions, followed by a rapid return to a normal quickly as fetal blood flow through the umbilical cord is interrupted. FHR returns rapidly to more slowly, mirroring the shape of contractions.
baseline rate, is most likely a client's response to: baseline as soon as the cord compression is relieved. FHR patterns associated with fetal head
compression (early deceleration) and utero‐placental insufficiency (late deceleration) have a
shallower appearance, since they do not drop as precipitously. Variable deceleration, unless
severe (lasting longer than 60 seconds), does not indicate severe hypoxia.

1.‐ Umbilical cord compression.


2.‐ Fetal head compression.
3.‐ Severe fetal hypoxia.
4.‐ Utero‐placental insufficiency.
2623 The nurse determines that fetal distress is occurring Correct answer: 3 Meconium passage prior to birth occurs in response to a stressful event for the fetus. Key words are "fetal distress," indicating that the focus of this question is abnormal
after noting which of the following signs? Moderate bloody show often occurs late in labor. Pink‐tinged amniotic fluid occurs because of assessment data. Eliminate options 1, 2, and 4, as they are common and normal findings
a small amount of blood, usually from the cervix. Accelerations of FHR are considered a normal during labor.
response, and do not indicate fetal distress.
1.‐ Moderate amount of bloody show
2.‐ Pink‐tinged amniotic fluid
3.‐ Meconium‐stained amniotic fluid
4.‐ Acceleration of fetal heart rate with each contraction

2624 On performing Leopold's maneuver on a multiparous Correct answer: 3 Findings on palpation are consistent with shoulder presentation or transverse lie. Vaginal The focus of this question is a client in labor with a transverse lie, a presentation that is
client in early labor, the nurse finds no fetal parts in delivery is not possible, so the nurse should anticipate cesarean section. Since the client is in incompatible with vaginal delivery. The correct answer would be the option that provides a
the fundus or above the symphysis. The fetal head is labor, version is contraindicated. safe alternative to vaginal delivery.
palpated in the right mid quadrant. The nurse notifies
the admitting physician and anticipates:

1.‐ An external version.


2.‐ An internal version.
3.‐ A cesarean delivery.
4.‐ Prolonged labor.

2625 The nurse discovers a loop of the umbilical cord Correct answer: 4 Pressure on the cord must be relieved to save the life of the fetus. Applying upward manual The focus of this question is maintaining the safety of the fetus. The correct option would
protruding through the vagina when preparing to pressure to the presenting part and having the mother assume a knee–chest position are contain an action to prevent cord compression and promote adequate oxygenation to the
perform a vaginal examination. The most appropriate appropriate emergency actions, followed by starting oxygen and calling the physician. Options fetus.
intervention is to: 2 and 3 do nothing to relieve cord occlusion.
1.‐ Call the physician immediately.
2.‐ Place a moist, clean towel over the cord to prevent drying.
3.‐ Immediately turn the client on her side, and listen to the fetal heart rate.
4.‐ Perform a vaginal examination, and apply upward digital pressure to the presenting part while having the mother assume a knee–chest position.

2626 The client has refused sedation ordered by the Correct answer: 1 Prolonged latent‐phase labor is associated with uncoordinated, hypertonic, and painful The wording of the question indicates that the correct option is a true statement.
physician for hypertonic contractions and prolonged contractions that do little to dilate or efface the cervix. Maternal exhaustion and dehydration Knowledge of the care of the labor client with hypertonic labor will aid in answering the
latent‐phase labor for fear that her labor will stop. The are concerns. Medical management is directed toward providing rest and hydration, and question.
nurse can help by explaining: allowing time for contractions to become coordinated. Often, clients awaken from sedation in
progressive labor. While option 2 is correct, this does little to explain the rationale for
sedation. Option 3 is incorrect. There is very little risk to the fetus unless contractions are
intense and &amp;lt; 2 minutes apart. Option 4 is not correct, because it is too soon to
anticipate the need for cesarean delivery.

1.‐ Sedation helps to provide needed rest, and allows time for the uterine contractions to become coordinated, so that labor is progressive.
2.‐ If the woman is experiencing true labor, contractions will not stop even with sedation.
3.‐ If contractions continue without cervical effacement and dilatation, the fetus is at risk for hypoxia.
4.‐ Sedation will stop contractions that are uncoordinated, and will provide more time to determine if a cesarean delivery is needed.

2627 The client is receiving intravenous magnesium sulfate Correct answer: 3 Early signs of magnesium toxicity that could lead to respiratory arrest are loss of patellar The focus of the question is the action of magnesium sulfate. Recall that the drug is a
at 2 grams/hr to stop premature labor. The nurse reflexes and decreased respiratory rate (&amp;lt; 12/min). Since magnesium is excreted from central nervous system depressant, and remember the ABCs; the correct answer would be
determines that the most important nursing the body through the renal system, hourly urine output should be assessed. Although blood the option that includes assessment of airway, breathing, and circulation.
assessments of this client include: pressure is a standard assessment for most antepartum clients, there is minimal blood
pressure change, if any, associated with administration of magnesium sulfate.

1.‐ Intake and output, level of consciousness, and blood pressure.


2.‐ Blood pressure, pulse, and uterine activity.
3.‐ Deep tendon reflexes, hourly urine output, and respiratory rate.
4.‐ Intake and output, blood pressure, and reflexes.
2628 During augmentation of labor with intravenous Correct answer: 3 Although rupture of the uterus is rare, there is an increased risk for multiparas and clients Critical words in this question are tearing sensation and oxytocin. A tearing sensation is
oxytocin (Pitocin), a multiparous client becomes pale undergoing induction or augmentation of labor. Early signs include pain and a tearing the classic symptom of uterine rupture, a risk with oxytocin administration. Eliminate
and diaphoretic, and complains of severe lower sensation, signs of shock, and fetal distress. Blood loss is usually severe, but might not be options 1, 2, and 4, as the symptom presented in the question would not be present with
abdominal pain with a tearing sensation. Fetal distress visible. Amniotic fluid embolus is frequently associated with cardiac and respiratory distress. these conditions.
is noted on the monitor. The nurse should suspect: Symptoms of precipitate labor and uterine prolapse do not include pallor, diaphoresis, or fetal
distress.
1.‐ Precipitate labor.
2.‐ Amniotic fluid embolus.
3.‐ Rupture of the uterus.
4.‐ Uterine prolapse.

2629 During vaginal examination, the nurse palpates the Correct answer: 4 In a brow presentation, the fetal forehead and the large, diamond‐shaped anterior fontanelle The critical words in this question are fetal head and diamond‐shaped fontanelle.
fetal head and a large, diamond‐shaped fontanelle. The are palpated during vaginal exam. In vertex presentation, the back of the fetal head (occiput) Eliminate option 2, as the fetal head would not be palpable in a transverse lie. Eliminate
nurse documents that the fetal presentation is: and small, triangular fontanelle are palpated. In breech and shoulder presentations, fetal parts options 1 and 4, as the anterior fontanelle would not be palpable with these presentations.
would feel soft and irregular. The correct answer would be the presentation that permits palpation of the anterior
fontanelle.
1.‐ Face.
2.‐ Transverse.
3.‐ Vertex.
4.‐ Brow.

2630 Following amniotomy, the nurse would implement Correct answer: 2, 3, 4 The risk of umbilical cord compression or prolapse increases when amniotic fluid is released. Recall knowledge of the care of the client undergoing amniotomy, and the resulting
which of the following as important nursing actions? Listening to fetal heart tones after amniotomy will quickly detect the presence of cord complications, such as umbilical cord compression and infection. The correct answer will be
Select all that apply. compression. Observing color and consistency of the fluid should be done next. Placing a clean the option that best promotes the safety of mother and fetus. Eliminate options that are
underpad on the bed and repositioning the mother are important in providing comfort, but are important but not critical to safety.
not the first priority. Temperature should be monitored every 1–2 hours for signs of infection.

1.‐ Position the mother in lithotomy position for delivery.


2.‐ Place a clean underpad on the bed.
3.‐ Listen to fetal heart tones.
4.‐ Observe the color and consistency of the amniotic fluid.
5.‐ Take vital signs every four hours to monitor for infection.

2631 The nurse can help a client with a fetus in the right Correct answer: 2 Gravity can help the fetus rotate to an anterior position for vaginal delivery. The positions in This is a negative statement. The correct answer would be the option that includes an
occiput posterior (ROP) position by avoiding which of options 1, 3, and 4 enlist the aid of gravity. Option 2 should be avoided because it will not help undesirable nursing action in this situation. This situation could be improved by the use of
the following actions? the fetus to rotate. gravity to rotate the fetus. Look for the option that does not use gravity; option 2 fits this
criterion.
1.‐ Positioning her on her left side
2.‐ Positioning her on her right side
3.‐ Helping her walk around the room
4.‐ Assisting her to a knee–chest position

2632 A nulliparous client has not made any progress in Correct answer: 4 Dilatation has stopped (arrested) after considerable progress. Causes can include hypotonic The core focus in this question is progress to 7 cm dilatation followed by no change in two
cervical dilatation or station since she was 7 cm and 0 uterine contractions, malposition, or cephalopelvic disproportion. Options 1 and 2 are not hours. Eliminate options 1 and 2 because prolonged and protracted mean that progress
station over two hours ago. The nurse interprets that correct, because prolonged and protracted mean that progress occurs at a very slow rate. occurs at a very slow rate. Eliminate option 3 because it focuses on the station rather than
according to the Friedman curve, this client is Arrest of descent (option 3) occurs when the station, rather than cervical dilatation, does not on cervical dilatation.
experiencing: change.
1.‐ Prolonged deceleration phase.
2.‐ Protracted active phase.
3.‐ Arrest of descent.
4.‐ Secondary arrest of dilatation.
2633 A client's amniotic fluid is greenish‐tinged. The fetal Correct answer: 1 Meconium released by the fetus causes amniotic fluid to be greenish‐tinged. Although the Recall the ABCs; meconium in the amniotic fluid can interfere with a clear airway and
presentation is vertex. Fetal heart rate (FHR) and presence of meconium is associated with fetal distress, there is no evidence of immediate effective breathing. The correct answer will include an action to clear the airway prior to
uterine activity have remained within normal limits. At danger to the fetus during labor in this case. However, the infant is at risk for aspirating the newborn's first breath.
the time of delivery, the nurse should anticipate the meconium at the time of delivery. Steps to prevent aspiration include thorough suctioning of
need for: the nasopharynx, including visualization of the vocal cords to remove meconium particles
before the first breath.
1.‐ An infant laryngoscope and suction catheters.
2.‐ Forceps.
3.‐ A transport isolette.
4.‐ Emergency cesarean setup.

2634 A client who is at 34 weeks' gestation has been having Correct answer: 4 Hydration has been shown to decrease premature labor contractions. Therefore, drinking Knowledge of the nursing care of the premature‐labor client will help to answer the
contractions every 10 minutes. In addition to water or other noncaffeinated beverages is recommended. If contractions continue at 10 question correctly. The wording of the question is positive, indicating that the correct
instructing her to lie down and rest while continuing to minutes apart or less for an hour with rest, the client should call her health care provider. option is a true statement of fact.
time contractions, the nurse should also tell her to:

1.‐ Refrain from eating or drinking anything.


2.‐ Take slow, deep breaths with each contraction.
3.‐ Go to the hospital if contractions continue for more than an hour.
4.‐ Drink 3–4 cups of water.

2635 The client who has had a previous cesarean birth asks Correct answer: 2 A classical incision involves the upper uterine segment, and is more likely to separate or Focus on safety of the mother and infant with this question. The greatest risk during a
about vaginal birth after cesarean (VBAC). Which of rupture with subsequent uterine contractions. Induction is not a contraindication if managed VBAC is uterine rupture; potential for rupture is influenced by location of the previous
the following factors from her history is a judiciously. The type of abdominal incision is not a concern, since it is not affected by uterine uterine scar. Eliminate options 1, 2, and 4 because they do not address the prior uterine
contraindication for VBAC? contractions. incision.
1.‐ PThe pevious cesarean was for breech presentation.
2.‐ The client had a classic uterine incision.
3.‐ The abdominal incision was vertical rather than transverse.
4.‐ An induction of labor is planned for this delivery.

2636 Which of the following statements by the nurse is Correct answer: 3 Promoting a positive feeling about how well she was able to cope with an emergency This question focuses on therapeutic communication. The correct answer would be the
most therapeutic in talking with a client and her family cesarean delivery will have an influence on self‐image and the client's feelings about her ability option that best supports the client's feelings and behaviors. Eliminate options 1, 2, and 4,
following emergency cesarean birth? to handle future pregnancies and births. In addition, providing an opportunity for the client which are nontherapeutic responses. Only option 3 is therapeutic and promotes a positive
and her family to ask questions and to express feelings helps in dealing with any feeling about her coping through the delivery.
disappointment, anger, or guilt they might feel. Other options indicate that the birth was not
normal, and can promote negative feelings about the infant or the experience.

1.‐ "I'm sorry that you couldn't have a normal delivery."


2.‐ "Your baby was really in danger. I think he is doing better now."
3.‐ "You did so well throughout the delivery. I'm sorry I didn't have more time to explain things."
4.‐ "I know you never expected this to happen. Maybe things will work out better next time."

2637 The pregnant client is receiving oxytocin (Pitocin) to Correct answer: 2, 5 Oxytocin (Pitocin) stimulates uterine contractility; exceeding maximum doses or increasing The focus of this question is adverse maternal effects of oxytocin (Pitocin). Recall that the
induce labor. The nurse should monitor the client for doses too rapidly can result in uterine hyperstimulation. Blood pressure might initially action of the drug is to stimulate the smooth muscle of the uterus and blood vessels, and
which of the following adverse maternal effects? decrease, but after prolonged drug administration, it could rise to 30% above baseline. The that the drug has an antidiuretic effect. An adverse effect can be an excess of a known drug
Select all that apply. antidiuretic effect of oxytocin decreases water exchange in the kidney and reduces urinary action; therefore, options 2 and 5 would be correct actions.
output, leading to fluid overload rather than dehydration. Bradycardia and jaundice are
possible adverse effects for the fetus rather, not for the mother.

1.‐ Bradycardia
2.‐ Decreased urine output
3.‐ Dehydration
4.‐ Jaundice
5.‐ Uterine hyperstimulation

2638 In addition to routine assessment and care, nursing Correct answer: 3 Terbutaline, a beta‐adrenergic agent, has many maternal and fetal side effects, including The focus of the question is the action of terbutaline (Brethine), a beta‐adrenergic agent.
care of the client who is receiving terbutaline tachycardia, cardiac arrhythmias, and pulmonary edema. In addition to taking vital signs, the Recall that beta‐adrenergic drugs cause side effects related to cardiopulmonary function.
(Brethine) to prevent premature labor should include nurse should assess for pulmonary edema. The frequency of assessment of fetal heart tones Remember the ABCs; the correct answer would be the option that includes assessing
assessing: and oral temperature depends on the intensity and length of the drug therapy, as well as maternal heart and lung function.
surrounding circumstances. Deep tendon reflex assessment is not indicated.

1.‐ Oral temperature every two hours.


2.‐ Fetal heart tones every 30 minutes.
3.‐ Breath sounds every four hours.
4.‐ Deep tendon reflexes every four hours.

2639 The nurse monitors a client during a vaginal delivery Correct answer: 1 With breech presentation, fetal parts do not completely fill the lower uterine segment, Remember the ABCs related to safety of the infant at birth. The correct option would
of a breech infant for which of the following as the allowing more opportunity for the umbilical cord to proceed through the cervix or become jeopardize the safety of the infant by interfering with normal oxygenation and circulation
greatest risk? compressed by the fetus, especially following rupture of membranes. The incidence of the through the umbilical cord.
other options is no higher in breech than it is with vertex presentation.

1.‐ Umbilical cord prolapse


2.‐ Intracranial hemorrhage
3.‐ Meconium aspiration
4.‐ Fracture of the clavicle

2640 The client is a 37‐year‐old gravida 1 at 38 weeks' Correct answer: 4 Amniocentesis for genetic testing is usually done early in the second trimester. This test, on a The critical words in the question are 38 weeks' gestation. Use knowledge of the timing of
gestation. She was diagnosed with diabetes at age 17, client who has diabetes, and is at 38 weeks' gestation, is probably being done to assess lung tests used to diagnose genetic defects to systematically eliminate the incorrect options. As
and is scheduled for an amniocentesis. The nurse maturity in anticipation of delivery. an alternative, consider that lung maturity is a key concern as a pregnant client approaches
concludes that the procedure is probably being done the due date.
to assess for the presence of which of the following?

1.‐ Neural tube defects


2.‐ Down syndrome
3.‐ Effects of TORCH syndrome
4.‐ Lung maturity

2641 A client has been scheduled for an amniocentesis. Correct answer: 2 The test, completed on an outpatient basis, is done under guidance of ultrasound Use knowledge of the procedure to eliminate options 1 and 4. Choose option 2 over 3
Which of the following actions should the nurse plan visualization. The test is done without anesthetic, or with a local anesthetic. The client is because the procedure is guided by ultrasound, and option 3 does not address risk of
to take in the care of this client? positioned on her back with a wedge under her left hip to avoid hypotension from pressure of pressure on the vena cava in the supine position.
the uterus on the vena cava.
1.‐ Arrange for the client’s admission to the hospital.
2.‐ Arrange for access to an ultrasound machine for use during the procedure.
3.‐ Assist the woman in assuming a supine position.
4.‐ Arrange for administration of general anesthetic.

2642 The client is scheduled to have an amniocentesis for Correct answer: 3 The amount of lecithin increases as the fetal lungs mature. The ratio of lecithin to Recall that amniotic fluid is clear to eliminate options 2 and 4. Choose option 3 over 1
assessment of lung maturity. She seems upset, and sphingomyelin is used to assess lung maturity; changes in color (options 2 and 4) are not. because option 1 is not a therapeutic response.
says that she doesn t understand how this test could Option 1 is not a therapeutic response.
tell if a baby s lungs are mature. What is the best
response by the nurse?

1.‐ “There is no need for you to worry about that. Your doctor knows the procedure well.”
2.‐ “The fluid changes color as the fetal lungs mature. We assess the color to determine the lung maturity.”
3.‐ “A chemical called lecithin is made by the fetal
lungs. The amount of it increases as gestation
continues. It flows out into the amniotic fluid, where
we can measure it.”
4.‐ “The amount of bilirubin in the fluid increases as lung maturity increases. We measure the yellow color in the fluid to assess lung maturity.”

2643 The nurse would formulate which of the following as Correct answer: 3 Most women view invasive antenatal testing with anxiety because of the reason for the test, Use knowledge of the procedure to assist in eliminating incorrect options. Eliminate
the highest‐priority nursing diagnosis for a client about the impending results, and concern about maternal and fetus complications. Because of the options 1 and 2 first, because there is no need for NPO status and there is no anesthesia.
to undergo an amniocentesis? small amount of fluid removed, option 4 is unnecessary. Options 1 and 2 are completely Choose option 3 over 2 (a valid concern, but a psychosocial issue rather than a physiological
incorrect. one) because the amount of fluid removed is very small.

1.‐ Imbalanced Nutrition: Less than Body Requirements related to NPO status
2.‐ Risk for Aspiration related to anesthesia
3.‐ Anxiety related to concern for fetal well‐being
4.‐ Risk for Deficient Fluid Volume related to removal of amniotic fluid

2644 The nurse assesses that which of the following Correct answer: 4 Contractions elicited during the test could cause increased bleeding if an abruption is present. Note the critical word contraindication in the stem of the question. This tells you that the
findings would be a contraindication for conducting a Intrauterine growth restriction, diabetes mellitus, and post‐term pregnancy are all indications correct answer is likely an item that could pose risk of harm to the fetus. From there, recall
contraction stress test? for completing a contraction stress test. that abruptio placentae can lead to bleeding to help you choose correctly.

1.‐ Intrauterine growth restriction


2.‐ Diabetes mellitus
3.‐ Pregnancy at 42 weeks' gestation
4.‐ Marginal abruptio placentae

2645 A primigravida is hospitalized at 32 weeks' gestation Correct answer: 1 The client has stated that she is worried, which creates anxiety. The information presented Note that the client exhibits appropriate nonverbal behavior (subdued and sad), and is
after a second hemorrhage from a complete placenta does not represent denial or immaturity. There are insufficient data to determine whether the able to articulate a concern (worried about husband). Consider that all of these are
previa. The client appears subdued and sad after the client s coping is effective at this time. expected reactions to choose anxiety over the other options.
physician informs her that she will remain in the
hospital until delivery. She says that she is worried
about her husband, who will be at home alone much
of the time. The nurse interprets the client s response
as which of the following?

1.‐ Anxiety
2.‐ Denial
3.‐ Immaturity
4.‐ Ineffective coping

2646 The nurse reviews the client s chart for results of Correct answer: 4 Percutaneous umbilical blood sampling (PUBS) obtains an actual sample of fetal blood for Note the word erythroblastosis in the question, and correlate that with erythrocytes or
which of the following diagnostic tests, which will best analysis. The other options provide information about fetal well‐being, but do not directly red blood cells. Eliminate options 1 and 2 first because they are not related to red blood
indicate a diagnosis of erythroblastosis fetalis? sample the fetal erythrocytes. cells. Then choose option 4 over 3 because it allows access to fetal cells, not to maternal
cells.
1.‐ Amniocentesis
2.‐ Biophysical profile
3.‐ Indirect Coombs’ test
4.‐ Percutaneous umbilical blood sampling
2647 The nurse caring for a client with a concealed Correct answer: 3 A concealed abruption could result in a Couvelaire uterus, which doesn t contract effectively Specific knowledge of the risks of concealed abruptio placentae is needed to answer the
abruptio placentae prepares to assess the client for after delivery, leading to uterine atony. The other complications could occur in any client. question. Use nursing knowledge and the process of elimination to make your selection.
which complication as a priority after delivery?

1.‐ Retained placental fragments


2.‐ Urinary tract infection
3.‐ Uterine atony
4.‐ Vaginal hematoma

2648 A pregnant client who has class II heart disease has Correct answer: 1 Prophylactic antibiotics are given during labor to prevent bacterial endocarditis. The other The core issue of the question is knowledge of the significance of class II heart disease
progressed throughout her pregnancy without medications might be needed based on additional assessment findings, or might not be during labor. Use nursing knowledge and the process of elimination to make your selection.
complication, and is admitted to the labor and delivery needed at all for a client with class II heart disease. Consider that antihypertensives, cardiac glycosides, and diuretics are used to manage
unit in active labor. The nurse anticipates symptoms that are not present in the stem of the question. An antibiotic is the only drug
administering which medication based on the client s listed that could prevent a new problem (infection).
history?
1.‐ An antibiotic
2.‐ An antihypertensive
3.‐ A cardiac glycoside
4.‐ A loop diuretic

2649 A client with placenta previa reports that she has Correct answer: 3 The client is likely to lose some blood with a placenta previa. Increasing iron in her diet is a The core issue of this question is culturally competent care to reduce risk of complications.
religious beliefs that prohibit receiving blood or blood positive response that does not interfere with her religious beliefs. Option 1 is not a true Eliminate option 1 because of the word force, and eliminate option 2 because of against
products. The nurse provides client teaching, and statement. Option 2 does not address the client’s need or right to care. medical advice. Choose option 3 over 4 because the client cannot predict the risk of
evaluates that the teaching has been effective if the bleeding during pregnancy.
client states:
1.‐ “A judge will force me to accept a transfusion if I really need it.”
2.‐ “I might have to sign out of the hospital against medical advice (AMA).”
3.‐ “I will meet with the dietician to increase the amount of iron in my diet.”
4.‐ “There is little chance that I will bleed heavily during this pregnancy.”

2650 The nurse would assess the pregnant client with a Correct answer: 1 The client with multiple partners is at high risk for sexually transmitted diseases and Note the critical words multiple sexual partners and greatest concern in the question. This
history of multiple sexual partners for which ascending infection that can lead to blockage in the fallopian tubes. Ultimately, this process tells you that the correct answer has a connection to risks associated with multiple sex
complication of pregnancy that is of greatest concern could lead to ectopic pregnancy. The other options do not address this particular partners. Use knowledge of complications of sexually transmitted infections to choose
in this situation? pathophysiological concern. option 1 over the others.
1.‐ Ectopic pregnancy
2.‐ Premature rupture of membranes
3.‐ Pregnancy‐induced hypertension
4.‐ Rh‐incompatibility

2651 The nurse anticipates that a pregnant client with a Correct answer: 3 The chance of transmission of HIV is less than 1% if the infant is delivered by cesarean prior to The core issue of the question is knowledge of methods of transmitting infection from
history of which of the following might benefit from a membrane rupture. Only the client with active herpes lesions should be delivered by cesarean mother to newborn during the delivery process. Eliminate options 1 and 4 first because
scheduled cesarean birth to achieve an improved to prevent transmission of the virus during vaginal birth. they do not address infection. Choose option 3 over 2 because there is greater risk of
outcome for the infant? transmitting HIV during delivery, while only active herpes lesions transmit that virus.

1.‐ Diabetes mellitus


2.‐ Herpes simplex type II
3.‐ Human immunodeficiency virus
4.‐ Systemic lupus erythematosus
2652 Which of the following short‐term client outcomes Correct answer: 2 A short‐term outcome of maintained weight is appropriate while the client is being stabilized The critical words in the question are client outcomes. With this in mind, eliminate options
would be most appropriate for a client admitted to the in the hospital. An outcome is the result of nursing care. Options 1 and 3 are nursing 1 and 3 because they are interventions. Choose option 1 over 4 because it directly
hospital with hyperemesis gravidarum and the nursing interventions. Option 4 does not address the nursing diagnosis. correlates with nutrition, which is the focus of the nursing diagnosis.
diagnosis of Imbalanced Nutrition: Less than Body
Requirements?

1.‐ Assess hourly intake and output.


2.‐ Maintain present weight.
3.‐ Provide favorite foods.
4.‐ Verbalize risks to the fetus.

2653 A type 1 diabetic prenatal client asks the clinic nurse Correct answer: 2 Breastfeeding should be encouraged, because it benefits both the mother and her infant. It is Note the key words most accurate, which tell you that the correct answer is the one that
whether she will be able to breastfeed her baby. not contraindicated for diabetic mothers (option 1), might or might not help prevent future is a true statement, while the others are false to a greater or lesser degree. Eliminate
Which response by the nurse is most accurate? pregnancy during lactation (option 3), and does not necessarily lead to loss of blood glucose options 1 and 4 first because of the words contraindicated and a lot of difficulty,
control with careful management (option 4). respectively. Then choose option 2 over 3 because option 3 might not be true, depending
on individual circumstances.
1.‐ “Breastfeeding is contraindicated for insulin‐dependent moms.”
2.‐ “Certainly, breastfeeding will be beneficial for both of you.”
3.‐ “I think this is a good idea because it also prevents pregnancy.”
4.‐ “You will have a lot of difficulty maintaining a stable blood sugar.”

2654 A client is admitted with membranes that ruptured Correct answer: 2 The client with premature ruptured membranes is at risk for developing an infection, and The core issue of the question is knowledge of infection as the key risk following rupture
four hours ago, and occasional mild contractions. The should have her vital signs, specifically temperature, monitored every two hours. The client of the membranes. Eliminate options that do not address this risk. Only option 2 addresses
term fetus looks healthy on external monitoring. may be on bedrest, not ambulating, following rupture of the membranes (option 1). Promoting vital signs, which includes monitoring temperature as one way of detecting infection.
Which of the following is the priority in the nursing rest (option 3) and providing clear liquids (option 4) are slightly lower priorities for this client.
plan of care for this client?

1.‐ Encourage ambulation.


2.‐ Monitor vital signs.
3.‐ Promote rest.
4.‐ Provide clear liquids.

2655 A prenatal client at 14 weeks' gestation complains of Correct answer: 4 Ultrasound confirms the diagnosis of molar pregnancy that is indicated by the client’s The core issue of the question is the best method to determine hydatidiform mole. Choose
continuous nausea and vomiting, and a severe symptoms. The client will have high hCG levels and low maternal serum alpha‐fetoprotein option 4 over all the others because it is the only one that provides for visualization of the
headache. The client has elevated blood pressure, and levels, but these are not conclusive for hydatidiform mole. Option 1 is inappropriate before the reproductive structures, and that allows discrimination of true pregnancy from
the fundal height is 21 centimeters. Which diagnostic third trimester because it evaluates the fetus. hydatidiform mole.
test does the nurse anticipate will be ordered to
confirm a hydatidiform mole?

1.‐ Biophysical profile


2.‐ Human chorionic gonadotropin
3.‐ Maternal serum alpha‐fetoprotein
4.‐ Sonography

2656 An HIV‐positive client in active labor with ruptured Correct answer: 4 The rate of transmission of HIV to the newborn is decreased from 17% to less than 7% if the The core issue of the question is management of the HIV client in active labor with respect
membranes is being transported to the hospital via mother is given prophylactic zidovudine (Retrovir) orally during pregnancy and by IV during to preventing transmission of HIV to the newborn. With this in mind, eliminate options 2
ambulance. The labor and delivery nurse anticipates labor. There are no indications presented in the question for any of the other medications and 3 first, as they do not prevent or treat infection. Choose option 4 over 1 because it is an
priority administration of which medication to this listed, although an antibiotic could be administered if the mother acquired an infection antiviral rather than antibacterial. (Note also the ‐vir that indicates virus in the drug name
client? secondary to ruptured membranes. Retrovir.)
1.‐ Antibiotics
2.‐ Immune globulin
3.‐ Oxytocin (Pitocin)
4.‐ Zidovudine (Retrovir)
2657 A client who admits to crack cocaine use during her Correct answer: 1 Option 1 is a therapeutic response to the client’s concerns. The nurse should remain The core issue of the question is a therapeutic response to a concern shared by the client.
pregnancy asks the nurse not to inform the baby’s nonjudgmental when clients reveal information about substance abuse. Option 2 is Eliminate each of the incorrect options systematically because they do not invite further
father about the substance abuse. Which response by nontherapeutic because it does not explore the client’s concern. Option 3 is inaccurate, and sharing of information between client and nurse.
the nurse is most appropriate? option 4 is judgmental.
1.‐ “You must be very worried about how he will react to that information.”
2.‐ “This is your pregnancy and your body, so I’ll keep your information private.”
3.‐ “Your baby will probably not survive, so there is no need for him to know.”
4.‐ “Have you considered that he deserves to know what you may have done to his baby?”

2658 A client experiencing profuse hemorrhage from Correct answer: 2 The left lateral position facilitates uteroplacental perfusion. Semi‐Fowler s position would The core issue of the question is how to maintain uteroplacental perfusion for the client in
placenta previa is being prepared for an emergency decrease maternal cerebral perfusion; Trendelenburg puts the weight of the gravid uterus shock. With this in mind, choose the position that turns the client to the left side and takes
cesarean birth. The client exhibits signs of against the maternal lungs; and knee‐chest is unlikely to be maintained by a client in shock. pressure of the gravid uterus off the great vessels in the abdomen.
hypovolemia. The nurse makes it a priority to place the
client into which of the following positions?

1.‐ Knee‐chest
2.‐ Left lateral
3.‐ Semi‐Fowler’s
4.‐ Trendelenburg

2659 Which of the following nursing diagnoses has the Correct answer: 2 The client with DIC is at risk for hemorrhage, which takes priority over the non–life‐ The issue of the question is knowledge of complications of DIC, specifically hemorrhage
highest priority for a client with a missed abortion who threatening options 1 and 4. The client could experience bruising or other areas of local and loss of circulating volume. With this in mind, focus on physiologically based nursing
has developed disseminated intravascular bleeding from the disorder, but hypovolemia from hemorrhage takes priority over risk for diagnoses, and eliminate options 1 and 4. Choose option 2 because it addresses a greater
coagulopathy (DIC)? injury (option 3). and more specific physiological risk.
1.‐ Anticipatory Grieving
2.‐ High Risk for Deficient Fluid Volume
3.‐ High Risk for Injury
4.‐ Spiritual Distress

2660 A client with premature spontaneous rupture of Correct answer: 1 Glucocorticoids such as betamethasone are contraindicated for use in diabetic clients The core issue of the question is knowledge of key side effects of betamethasone, which
membranes (SROM) at 33 weeks' gestation is to be because they raise the blood glucose level even further. The other disorders are not helps to select the client for whom it should not be used. Recall that the glucocorticoids
given betamethasone (Celestone) to increase fetal lung contraindications for giving betamethasone. often end in ‐sone to help you recognize the drug as a glucocorticoid. Recall next the risk of
maturity. The nurse checks the client s record to elevating blood glucose levels to choose option 1 over the others.
ensure that the client does not have what disorder
that would be a contraindication for this drug?

1.‐ Diabetes mellitus


2.‐ History of alcohol abuse
3.‐ Incompetent cervix
4.‐ Intrauterine growth restriction (IUGR)

2661 The nurse concludes that a client is at risk for Correct answer: 2 An increase of 30 mmHg systolic and 15 mmHg diastolic on two occasions is diagnostic for Specific knowledge of the criteria for PIH is needed to answer this question. Use nursing
pregnancy‐induced hypertension (PIH) when the vital PIH. The blood pressures in each of the other options do not meet the criteria for increase in knowledge and the process of elimination to make your selection. As an alternative, choose
signs taken today show that the blood pressure has either the systolic or the diastolic blood pressure reading. option 2 because it has the greatest degree of change in both systolic and diastolic
increased during pregnancy from: measurements.
1.‐ 90/56 to 110/70.
2.‐ 100/60 to 130/76.
3.‐ 122/80 to 138/86.
4.‐ 134/80 to 140/88.
2662 A client who has experienced a spontaneous abortion Correct answer: 1 The majority of early abortions are related to abnormal chromosomes. The client might fear Specific knowledge of the etiologies of spontaneous abortion is needed to answer the
at 8 weeks asks the nurse why this happened. The that she has caused the loss, and should be provided with accurate information. The other question. Use nursing knowledge and the process of elimination to make your selection.
nurse provides accurate information by stating that responses are not accurate.
the most common cause of “miscarriage” is:

1.‐ Chromosome abnormalities.


2.‐ Environmental teratogens.
3.‐ Excessive activity.
4.‐ Substance abuse.

2663 A client who received no prenatal care delivers a 9‐ Correct answer: 3 An LGA infant who demonstrates respiratory immaturity could have a diabetic mother. The The core issues of the question are prenatal risks for LGA infants and the consequences
pound, 4‐ounce baby boy who exhibits signs of infant produces his own insulin during pregnancy, and stores the excess glucose as fat to after delivery. Use nursing knowledge and the process of elimination to make your
respiratory distress. The nurse obtains a blood sample compensate for high maternal glucose loads. However, after delivery, the infant is at high risk selection.
from the infant to assess for which of the following? for hypoglycemia because excess maternal glucose is now absent from the infant s circulation.

1.‐ Hemolysis
2.‐ Hyperbilirubinemia
3.‐ Hypoglycemia
4.‐ Sepsis

2664 The nurse explains to a client who had a cervical cone Correct answer: 2 Cervical trauma and scarring can result in cervical incompetence during pregnancy. The other Note the critical word cervical in the stem of the question, and choose option 2 over the
biopsy several years ago that she is now at increased options are unrelated to cone biopsy. others because it also refers to the cervix.
risk for which complication of pregnancy?

1.‐ Abdominal pregnancy


2.‐ Incompetent cervix
3.‐ Gestational trophoblastic disease
4.‐ Placenta previa

2665 A client with a complete placenta previa is Correct answer: 3 The priority diagnosis is related to maintaining circulation and oxygenation. The other options Remember the ABCs. The correct answer would be the option that contains a nursing
hospitalized on bedrest at 24 weeks' gestation. In are of lesser importance. diagnosis related to airway, breathing, or circulation.
preparing the plan of care, what nursing diagnosis
takes priority for this client?
1.‐ Activity Intolerance related to enforced bedrest
2.‐ Imbalanced Nutrition: Greater than Body Requirements related to sedentary lifestyle
3.‐ Ineffective Tissue Perfusion related to placental location
4.‐ Anticipatory Grieving related to potential fetal loss

2666 A 20‐year‐old gravida 2, para 0 at 37 weeks' gestation Correct answer: 4 The MacDonald cerclage is a purse‐string suture that ties the cervix closed. The suture needs Knowledge of the need to remove the cerclage prior to delivery and labor for the 37‐
calls the nurse because she is experiencing to be removed before vaginal delivery is possible. Options 1 and 2 place the client at risk of weeks client will help to identify the correct response and the correct answer.
contractions every 7–8 minutes. Her first pregnancy cervical injury. The cerclage is usually removed at 37 weeks to allow natural labor to begin.
ended with a spontaneous abortion at 18 weeks, and
the client had a MacDonald cerclage placed early in
the current pregnancy. Which of the following
instructions by the nurse are the most appropriate?

1.‐ "Try a warm bath and relaxation techniques to see if the contractions will go away."
2.‐ "You must wait until your contractions are every 5 minutes before going to the hospital."
3.‐ "You need to go to the hospital, so we can stop your premature labor this time."
4.‐ "You should go to the hospital to be evaluated and have the cerclage removed."
2667 A pregnant client comes to the hospital at 36 weeks Correct answer: 2 During pregnancy, only amniotic fluid will dry to a ferning pattern. Urine occasionally might The core issue of the question is the most objective data to document rupture of
reporting that her "water broke," but denies any be alkaline, and turn nitrazine paper blue, or old nitrazine paper might be unreliable. membranes. Eliminate option 2 because other fluids might result in this finding. Eliminate
contractions. Which of the following assessment data Performing a vaginal exam places the client at unnecessary risk for an ascending infection, and option 3 because it jeopardizes the safety of the client, and can result in inaccurate data.
provides the nurse with the most reliable indication of feeling for membranes is unreliable. A watery vaginal discharge is not necessarily amniotic Eliminate option 4 because this finding is not conclusive for amniotic fluid.
premature rupture of membranes? fluid.

1.‐ A dried specimen shows a microscopic fern pattern.


2.‐ Fluid from the perineum turns nitrazine paper dark blue.
3.‐ No membranes are felt on a sterile vaginal exam.
4.‐ The client has a visible watery vaginal discharge.

2668 During which of the following procedures should the Correct answer: 1 According to universal precautions, the caregiver should wear goggles when contamination Recall that goggles are indicated to prevent contamination of the eyes. Eliminate options
nurse wear protective goggles in addition to gloves? from splashing is possible, as when the membranes are artificially ruptured. The other options 1, 3, and 4 because they place the nurse at risk for contamination from skin contact,
place the nurse at risk for contamination from skin contact, necessitating the use of gloves. necessitating the use of gloves, not goggles.

1.‐ Changing a soaked disposable bed pad


2.‐ Performing an amniotomy
3.‐ Starting an intravenous line
4.‐ Washing dirty instruments

2669 When caring for a client with pre‐eclampsia, which of Correct answer: 4 The normal platelet value is 150,000–450,000/mm&lt;sup&gt;3&lt;/sup&gt;. The pre‐ The focus of the question is abnormal laboratory results in pre‐eclampsia. Eliminate
the following laboratory results should the nurse eclamptic client is at risk to develop the potentially fatal HELLP syndrome, with low platelets as options 1, 2, and 3 because they contain normal values for pregnancy.
report to the physician immediately? one of the defining factors. The other options are all within normal values for pregnancy.

1.‐ Creatinine 0.3 mg/dL


2.‐ Fasting blood glucose 65 mg/dL
3.‐ Hemoglobin 11 grams/dL
4.‐ Platelets 50,000/mm3

2670 When providing discharge information to an Rh‐ Correct answer: 2 Rh‐immune globulin, RhoGAM, must be administered within 72 hours of any event that Knowledge of the risks for the Rh‐sensitized pregnant woman will help to choose the
negative mother, the nurse evaluates that the client presents a possibility for the mother to become sensitized to the Rh antigen. This includes correct answer. The question is worded as a positive statement. The correct answer would
has understood the teaching when she states: pregnancy with an Rh‐positive infant, which could be the case in a first‐trimester abortion be the option that contains a true statement about a point of client education.
(miscarriage), though the blood type is not usually obtained. If the father is also Rh‐negative,
the fetus will be also, and no problems will occur with this pregnancy. The indirect Coombs'
test is used to determine maternal Rh sensitization. While option 4 might be correct during
this one reproductive event, precautions must be taken during future pregnancies.

1.‐ "I don't have to be concerned unless my husband is also Rh‐negative."


2.‐ "I must have Rh‐immune globulin if I ever have a miscarriage."
3.‐ "The direct Coombs' test will determine if I am sensitized to Rh‐positive blood."
4.‐ "Since my baby is also Rh‐negative, I don't need to worry about it."

2671 A client with severe pre‐eclampsia has a physician's Correct answer: 600 The mixed concentration of magnesium sulfate is 2 grams in 100 mL of fluid. The client should Knowledge of the calculation needed to administer the required dose of medication will
order that reads, "magnesium sulfate 4 grams loading receive 200 mL (4 grams) over 20 minutes, so the pump must be set to deliver 600 mL over 60 be essential to arrive at the correct answer.
dose, then 2 grams/hour." The nurse mixes 20 grams minutes.
of 10% magnesium sulfate in 1,000 mL of Ringer's
lactate per agency protocol. Using a mini‐drip IV set,
the nurse sets the IV pump to deliver the loading dose
at _____ mL/hour over 20 minutes.
2672 A client who has experienced a complete Correct answer: 1 Feelings of anger are commonly experienced during the grieving process. The client who has The focus of the question is a childbearing loss. The correct answer would be a common
spontaneous abortion expresses her anger at the had a spontaneous abortion is grieving the loss of her imagined child, and should receive psychological experience after loss, grief.
physician and the nurses for not doing enough to save supportive care.
her pregnancy. The nurse evaluates this behavior as:

1.‐ A common grief response.


2.‐ Displaced marital strife.
3.‐ Ineffective social skills.
4.‐ Maladaptive coping.

2673 Which of the following factors found in a prenatal Correct answer: 2 Previous endometriosis and pelvic inflammatory disease can cause scar tissue formation that The focus of the question is a causative factor for ectopic pregnancy. Eliminate options 1,
client's history would place her at increased risk for could block the normal passage of a fertilized ovum through the fallopian tube. The other 3, and 4 because they would not interfere with movement of the ovum in the fallopian
ectopic pregnancy? Select all that apply. options would not interfere with movement of the ovum. tube, the common location for an ectopic pregnancy.

1.‐ Android pelvis


2.‐ Endometriosis
3.‐ Late menarche
4.‐ Previous cesarean
5.‐ Pelvic inflammatory disease

2674 The nurse caring for a pre‐eclamptic client discovers Correct answer: 3 The nurse remains with the client to prevent injury during the seizure. Insertion of a tongue Knowledge of the nursing interventions for the client who is seizing is necessary to answer
her on the bathroom floor having an eclamptic seizure. blade is not recommended, because of the risk of injury to both nurse and client. The client the question correctly. This question focuses on the safety of the client. The correct answer
What are the nurse's priority actions? should be placed on her side, to avoid aspiration. would be the option that presents the first action to prevent injury.

1.‐ Call the physician, and prepare for cesarean birth.


2.‐ Insert a tongue blade, to prevent biting the tongue.
3.‐ Remain with the client, and call for help.
4.‐ Turn the client to her back, and observe her movements.

2675 A client at 10 weeks' gestation who has recently Correct answer: 1 The client has three risk factors of molar pregnancy: Japanese background, brownish, "prune The core issue of this question is a cluster of symptoms. Eliminate options 2 and 3 because
emigrated from Japan comes to the prenatal clinic juice" vaginal bleeding, and the severe nausea and vomiting associated with excessive hCG they are characterized by one main symptom. Eliminate option 4 because no psychological
because she is having some dark brown vaginal found in trophoblastic disease. The client has only one symptom of hyperemesis; placenta symptoms are presented.
spotting, and is experiencing severe nausea and previa presents with bright red bleeding; and there is no information suggestive of psychosis.
vomiting. The nurse interprets that these symptoms
are compatible with which of the following conditions?

1.‐ Gestational trophoblastic disease


2.‐ Hyperemesis gravidarum
3.‐ Placenta previa
4.‐ Pregnancy‐induced psychosis

2676 A client who experienced an incompetent cervix with Correct answer: 1 The Shirodkar operation is closure of the cervix with suture material to prevent preterm Recall that the placement of a suture in cerclage places a barrier to safe delivery of an
a previous pregnancy has had a Shirodkar operation dilatation. When labor ensues, the suture must be cut so the fetus can pass through the birth infant as the due date approaches. With this in mind, select the option that allows for labor
done at 18 weeks in the current pregnancy. The client canal. Waiting for harder contractions will increase the likelihood of cervical damage from the to progress safely, or to begin safely, if the labor is false at this time.
calls the clinic at 37 weeks' gestation because she is suture. Option 3 does not address the client's risk, which is the priority. Clients who expect to
having irregular contractions every 5–7 minutes. have several future pregnancies may be delivered by cesarean to avoid repeated cerclage, but
Which response by the nurse is most appropriate? there is no necessity to this option.

1.‐ "You need to go to the hospital to have the cerclage removed before your baby is born."
2.‐ "You should wait, and come in when the contractions are closer and harder."
3.‐ "You sound like you are worried about this baby. It must be frightening for you."
4.‐ "You will need to have a cesarean birth with the Shirodkar cerclage in place."
2677 A client who had no prenatal care presents to the Correct answer: 1 The risk for placental abruption is increased with cocaine abuse. The other factors make the Knowledge of the risks from cocaine abuse will aid in answering this question correctly.
labor and delivery unit with a moderate amount of client high risk for complications of pregnancy, but not particularly for abruption. Eliminate the other answers, which are not factors that increase the risk for abruptio
vaginal bleeding and complaints of severe abdominal placentae.
pain. Fundal height is 34 centimeters. Contractions are
every 1.5 minutes, lasting 60 seconds, and strong, with
increasing resting tone. The monitor shows consistent
late decelerations. What information from the nursing
assessment is most consistent with a risk for placental
abruption?

1.‐ The client admits to using cocaine.


2.‐ The client has had no prenatal care.
3.‐ The client is HIV‐positive.
4.‐ The client is poor and uneducated.

2678 A client is hospitalized on the antepartum unit with Correct answer: 4 The client with ruptured membranes prior to the beginning of labor is at increased risk for The core focus of this question is planning care for a client with a complication. The
premature rupture of membranes at 37 weeks' ascending infection (chorioamnionitis). The client's temperature should be taken every 2–4 correct answer would be the option that includes additional or more frequent assessment
gestation. Which of the following routine physician hours, to identify early signs of sepsis. of the client's status.
orders would the nurse question for this client?

1.‐ Bedrest with bathroom privileges


2.‐ Diet as tolerated
3.‐ External fetal monitor prn
4.‐ Vital signs every shift

2679 The nurse would question an order for which of the Correct answer: 3 The infant of an HIV‐positive mother will test positive on an ELISA test for the human The focus of this question is an immune disorder. The correct answer would be the option
following laboratory tests, which is inappropriate to immunodeficieny virus because the maternal antibodies cross the placenta during pregnancy. that includes a test designed to assess antigen–antibody responses. Knowledge of the care
test the current condition of a newborn of an HIV‐ This does not indicate that the newborn has HIV. The diagnosis using the ELISA test for the of the newborn exposed to HIV/AIDS and of the transmission of maternal antibodies to the
positive mother? baby is not made until around 15 months, when maternal antibodies are degraded and the newborn will aid in answering correctly.
infant forms antibodies to HIV if infected. The other tests give information about the infant's
current condition.
1.‐ Bilirubin level
2.‐ Blood glucose level
3.‐ ELISA testing
4.‐ Hematocrit

2680 A client with heart disease has been prescribed Correct answer: 2 Digoxin is a cardiac glycoside that increases cardiac output by increasing the strength of A critical word in the question is "effective." This indicates the correct option is a true
digoxin (Lanoxin) during her pregnancy. The nurse contraction of the myocardium and slowing the heart rate. A pulse rate lower than 60 is a statement. Knowledge of the action of digoxin and its side effects will help to choose the
evaluates that client teaching has been effective when serious adverse effect of the medication, and the dose should be held. The client needs correct answer.
the client states: adequate potassium for myocardial function. Antibiotics are not contraindicated with digoxin.
The drug may be given with or without food.
1.‐ "I will avoid eating foods high in potassium while taking this medication."
2.‐ "I will check my pulse, and not take the medication if it is less than 60."
3.‐ "I will not take antibiotics at the same time as this medication."
4.‐ "I will take this medication with a full glass of water before breakfast."

2681 A 34‐year‐old client comes to the Emergency Correct answer: 1, 5 The nurse should provide emotional support to all clients experiencing perinatal loss. Offering Choose options that contain therapeutic communications. Only options 1 and 5 are
Department with cramping and vaginal bleeding. She the client an opportunity to talk with another health care professional or clergy for additional therapeutic and offer support; the other answers can be eliminated, since they are not
has missed two menstrual periods. Which of the help is also supportive. The other answers are insensitive, and option #4 might not be true. supportive measures.
following statements by the nurse is most appropriate
when the client is diagnosed with an incomplete
abortion? Select all that apply.
1.‐ "I am so sorry. This must be difficult for you."
2.‐ "The doctor will clean out your womb with a D and C."
3.‐ "Did you really want to be pregnant now?"
4.‐ "You'll still be able to have children after this is over."
5.‐ "Would you like to speak with a hospital chaplain or counselor?"

2682 A client with pre‐eclampsia is receiving magnesium Correct answer: 2 Magnesium sulfate is a CNS depressant used to prevent seizure activity in the pre‐eclamptic Recall that the classification of magnesium sulfate is a CNS depressant, which will help you
sulfate and oxytocin (Pitocin) IV to induce labor at 38 client. The other options might occur, but are not the intended effect of the drug. to recall that it will prevent seizures, the major risk of pre‐eclampsia. The other options are
weeks. The nurse determines the magnesium sulfate all similar and incorrect; they might occur when the drug is used, but are not the primary
has been effective after noting which of the following action of the drug.
effects on the client?

1.‐ Lowered blood pressure


2.‐ Absence of seizures
3.‐ Onset of sedation
4.‐ Stools that are soft

2683 The nurse anticipates that which of the following Correct answer: 1 A sinusoidal fetal heart rhythm is associated with fetal anemia, which could be associated Identify the option that is different from the rest. Blood loss from abruptio placentae can
complications of pregnancy would be most consistent with an abruption. The other complications would result in other signs of fetal distress, such as cause fetal anemia, which is associated with a sinusoidal fetal heart rhythm. The other
with development of a sinusoidal fetal heart rate tachycardia, loss of variability, and late decelerations. options are similar because they can cause fetal distress but not fetal anemia, and are likely
pattern during labor? to be incorrect.
1.‐ Abruptio placentae
2.‐ Chorioamnionitis
3.‐ Pregnancy‐induced hypertension
4.‐ Prolapsed cord

2684 A client who admits to substance abuse during Correct answer: 4 Option 4 is the only answer that acknowledges the client's intent to cut down on substance Eliminate options 2 and 3, which are negative and nontherapeutic. Option 1 can be
pregnancy tells the nurse, "I know I am just a really abuse while seeking additional information about the client's self‐concept. Option 1 places the eliminated because it doesn't focus on the client and her needs to explain herself and her
weak person, but I will try to cut down while I'm emphasis on the nurse, while options 2 and 3 are demeaning and negative. feelings.
pregnant." Which response by the nurse would be
most therapeutic?
1.‐ "I am concerned about you and your baby. What can I do to help you?"
2.‐ "I don't believe that you are weak at all. You just need to say no to drugs."
3.‐ "I have heard that before. You need to get serious now, or your baby will suffer."
4.‐ "That is a very positive plan. Could you tell me more about feeling like a weak person?"

2685 A client with class II heart disease is being seen for Correct answer: 3, 5 Anemia increases the cardiac workload, and should be avoided by clients with heart disease. Knowledge of the complications from heart disease during pregnancy will help to answer
her first prenatal visit. Which of the following teaching The client should discuss medications with her caregiver, but she may be allowed to take the question correctly.
points would the nurse stress for this client? Select all acetaminophen or a few other OTC medications. The client with class II cardiac disease is
that apply. slightly compromised with ordinary activity levels, and would not tolerate exercise. There is a
2–4% chance that the baby will inherit a congenital defect.

1.‐ Avoid all over‐the‐counter (OTC) medications during pregnancy.


2.‐ Regular exercise will help increase cardiac capacity during pregnancy.
3.‐ It’s important to take prenatal vitamins and iron as prescribed.
4.‐ The client’s fetus will probably have a similar congenital heart defect.
5.‐ Adequate nutrition to prevent anemia and avoid excessive weight gain
2686 A client with type 1 diabetes mellitus gives birth. The Correct answer: 1 The placenta produces human placental lactogen (hPL) and increased amounts of estrogen This question is time‐dependent, as noted by the words “insulin requirements in the first
postpartum nurse monitors the blood glucose level and progesterone. These hormones interfere with maternal glucose metabolism, and require 24 hours after delivery.” Recall that this is a time of rapid and dramatic changes for the
carefully, expecting that the client’s insulin increased insulin production or supplementation. As soon as the placenta is expelled, these woman. Eliminate options that do not meet this criteria.
requirements in the first 24 hours after delivery will: hormone levels fall dramatically, and the mother might require no insulin at all or a very
reduced dose in the first 24 hours.
1.‐ Drop significantly.
2.‐ Gradually return to normal.
3.‐ Increase slightly.
4.‐ Stay the same as before delivery.

2687 The nurse is especially interested in which of the Correct answer: 2 The glycosylated hemoglobin (H. A&lt;sub&gt;1c&lt;/sub&gt;) test provides an indication of The critical words in the question are “ongoing control.” This tells you that the correct
following lab tests, which provides the nurse with the what glucose levels have been over the last 4–8 weeks, because glucose attaches to the red option identifies a test measurement that does not reflect only current time. Knowledge of
best information about ongoing control of type 1 blood cells (RBC) and remains there for the residual life of the RBC. Increased blood glucose the laboratory assessment of long‐term glucose control will aid in answering the question
diabetes mellitus in a pregnant adolescent? levels will be reflected in an increased percentage of H. A&lt;sub&gt;1c&lt;/sub&gt;. The other correctly.
tests indicate current blood glucose levels only.

1.‐ Fasting blood glucose


2.‐ Glycosylated hemoglobin (H. A1c)
3.‐ Oral glucose tolerance test (OGTT)
4.‐ Post‐prandial test

2688 The nurse should be aware that pregnant women Correct answer: 3 Pregnancy presents an ideal time for nurses to reach out to substance‐abusing clients in a Options 1, 2, and 4 are punitive, and should be eliminated. Option 3 is the only one that
who practice substance abuse and present themselves caring way, since the client herself recognizes that she and her baby will benefit from prenatal indicates a therapeutic response to someone who recognizes the need for prenatal care
for prenatal care: care. Option 1 is unrealistic, option 2 is punitive, and option 4 is judgmental. even though she is abusing drugs.

1.‐ Are ready to kick their habit.


2.‐ Must be reported to the authorities.
3.‐ Recognize the need for caring interventions.
4.‐ Will lack appropriate parenting skills.

2689 Which of the following nursing actions would take Correct answer: 2 The client with a suspected ectopic pregnancy might be at risk for the development of The critical word in this question is “suspected.” The correct answer would be the option
priority when caring for the woman with a suspected hypovolemic shock. Assessment is the first step of the nursing process, and airway, breathing, that provides further assessment of the client’s condition.
ectopic pregnancy? and circulation are the priorities. Options 1 and 4 are possible later interventions, and option 3
is the surgeon s responsibility.
1.‐ Administering oxygen
2.‐ Monitoring vital signs
3.‐ Obtaining surgical consent
4.‐ Providing emotional support

2690 A client is being discharged from the hospital after Correct answer: 3 The client requires frequent monitoring to rule out development of malignancy after The wording of the question reflects a negative stem, and so the correct option will be an
evacuation of a molar pregnancy. The nurse recognizes experiencing trophoblastic gestational disease. Weekly hCG measurements are done until option that is false. Knowledge of the complications from hydatidiform mole and the
that additional discharge teaching is required when the normal levels are recorded for three weeks. Option 2 is a possibility for this client. The client required medical regime will aid in choosing the correct answer.
client states: should use contraception for at least one year during the follow‐up care (option 4), and
expressions of sadness are appropriate for any pregnancy loss, even if no fetus developed
(option 1).
1.‐ “I am so sad that I lost this baby.”
2.‐ “I may need to have chemotherapy after this.”
3.‐ “I will need to see the doctor yearly for follow‐up.”
4.‐ “I will use contraception for the next year.”
2691 The charge nurse in the labor and delivery unit has Correct answer: 3 The registered nurse is responsible for client assessments (options 2 and 4), and for client Recall that assessment and teaching should not be delegated to unlicensed personnel to
become overwhelmed with admissions and births. For teaching (option 1). The intervention of helping the client to the bathroom is within the eliminate incorrect options.
which client can the charge nurse best delegate the practice abilities of a CNA if the RN has determined that it is safe for this client to get out of
needed care to a trusted certified nursing assistant bed.
(CNA) who is currently going to school to become a
nurse?
1.‐ A client in false labor who needs teaching about true versus false labor signs
2.‐ A client with PIH who needs to be evaluated for reflexes and clonus
3.‐ A primigravida in early labor who needs to be helped to the bathroom
4.‐ An obese laboring client who needs to have her fetal monitor adjusted

2692 A client with a known placenta previa is admitted at Correct answer: 1 One mL of blood weighs approximately 1 gram. The other responses are incorrect. The critical information needed to answer the question is that 1 mL is approximately equal
30 weeks gestation with painless vaginal bleeding. The to 1 gram. Learn this information now if this question was difficult.
nurse weighs the client s peripads to monitor blood
loss. After noting an increased weight of 50 grams, the
nurse would document that this equals approximately
_____ mL blood loss.

2693 A client with hyperemesis gravidarum would most Correct answer: 2 The client with hyperemesis gravidarum is anxious or even fearful about the effects of her The core focus in this question is the effects of hyperemesis, excessive vomiting, and
likely benefit from nursing care designed to address condition on the fetus. The etiology of hyperemesis is unknown, but the incidence is increased deficient nutrition. Eliminate option 1, as it deals with overnutrition. Eliminate options 3
which of the following nursing diagnoses? in conditions with increased hCG. There might be an emotional component, but there is no and 4, as there are no data to support them.
indication that this is an unwanted pregnancy. With appropriate treatment, the prognosis is
favorable for the fetus. The client experiences excessive vomiting, and would have the
diagnosis of Imbalanced Nutrition: Less than Body Requirements.

1.‐ Imbalanced Nutrition: More than Body Requirements related to pregnancy


2.‐ Anxiety related to effects of hyperemesis on fetal well‐being
3.‐ Anticipatory Grieving related to inevitable pregnancy loss
4.‐ Ineffective Coping related to unwanted pregnancy

2694 The initial laboratory results for a primigravida Correct answer: 1 The Rh‐negative client whose partner is Rh‐positive could carry an Rh‐positive fetus, and Only option 1 indicates a need that is identified by the scenario. The other answers are not
indicate a hemoglobin of 12 grams/dL, hematocrit of would be at risk for Rh‐sensitization, which could create risks for future pregnancies. This correct, given the normal hemoglobin and hematocrit. In addition, three options are similar
36%, and a blood group and type of A, Rh‐negative. father of the baby needs to have his blood type assessed. The client is not anemic based on (focusing on nutrition), and are therefore likely to be incorrect. The correct answer, option
What would be the priority nursing action to promote these hemoglobin and hematocrit values, so options 2 and 4 are incorrect. There is no 1, is different.
a healthy pregnancy for this client and her fetus? relationship between the lab values and the client s weight in this scenario (option 3).

1.‐ Determine the blood type of the father.


2.‐ Encourage the client to eat more dark green, leafy vegetables.
3.‐ Provide information on weight gain during pregnancy.
4.‐ Suggest an iron supplement in addition to prenatal vitamins.

2695 The nurse has taught the client to perform deep‐ Correct answer: 3 Placing the hands directly on the incision during coughing will diminish the discomfort The words needs more teaching in the stem of the question tells you that the incorrect
breathing and coughing exercises. The nurse associated with coughing. Each of the other options indicates correct procedure on the part of client statement is the correct option. Use knowledge of nursing fundamentals to make a
determines that the client needs more teaching when the client. selection.
the client is observed doing which of the following
activities?
1.‐ Sitting upright before performing deep‐breathing and coughing exercises
2.‐ Taking deep breaths before attempting to cough
3.‐ Placing both hands vertically and lightly on either side of the incision
4.‐ Using a pillow for splinting during coughing
2696 A toddler is being prepared for a surgical procedure. Correct answer: 4 The child fears separation from her parents. The child has no previous experiences to The critical word greatest in the stem of the question provides a clue that more than one
This is the child’s first experience with surgery. The compare to this experience, so she will not anticipate pain. The child cannot anticipate any option could be partially true. Use knowledge of growth and development to make a
child’s mother expresses concern about the child’s changes in her body, and does not worry about communication. selection, recalling that toddlers fear separation from their parents.
psychological adaptation to the surgery. While
planning for postoperative care, the nurse recognizes
that which of the following is likely to be the child’s
greatest concern?

1.‐ Anticipated pain


2.‐ Body image changes
3.‐ Communication difficulties
4.‐ Separation from parents

2697 A female client is being prepared for surgery. When Correct answer: 4 Taping a wedding band in place is acceptable for the client who does not wish to remove it, Identify the core issue of the question, which is the method of safeguarding client property
the nurse asks the client to remove her wedding ring, unless there is danger the finger might swell during or after surgery. Option 1 assumes the ring during surgery. Choose the option that meets the needs of the client and protects both the
the client refuses. Which of the following would be the is tight, and that the client wishes to remove it. Option 3 is a false statement, while option 2 hospital and the client’s property.
most appropriate response by the nurse? creates unnecessary fear at a time when anxiety already is likely to be increased.

1.‐ Encourage the client to use soapy water to remove the ring, if it is tight.
2.‐ Explain that the hospital cannot be responsible for jewelry worn during surgery.
3.‐ Notify the surgeon’s office that the surgeon must see the client in the preoperative holding area.
4.‐ Tape the ring in place before the client is transported to the preoperative holding area.

2698 The nurse is caring for clients in the preanesthesia Correct answer: 1 Option 1 is correct because with increased age, there is a greater likelihood that the kidneys For questions that ask you to choose one client over others, determine which client
room. The nurse notes that one client, who is an older start to degenerate. All the other options are incorrect: hunger does not necessarily cause description indicates the worst client status or greatest risk for complications. In this case,
adult, has an increased surgical risk based on which of hyperacidity, comprehension is not altered in all older adults, and cardiovascular problems do note that fluid and electrolyte balance poses the greatest risk in the intraoperative period,
the following factors? not necessarily diminish pain sensations. which is the core issue of the question.

1.‐ Decreased kidney function leading to potential fluid and electrolyte imbalances
2.‐ Increased hunger sensations leading to postoperative complications from hyperacidity
3.‐ Inability to comprehend the seriousness of surgical interventions, leading to noncompliance
4.‐ Poor cardiovascular status leading to decreased pain sensation

2699 A client who takes numerous medications is being Correct answer: 3 Corticosteroids can lead to weight gain because of salt and water retention, and also can To answer this question, recall the actions and adverse effects of each drug class listed.
prepared for surgery. The nurse reviewing the client delay wound healing. An antidysrhythmic helps to regulate the cardiac rhythm (option 1). A Use the process of elimination, focusing on the risk to the client during an actual surgical
medication list is most concerned about which sedative‐hypnotic can interfere with uptake of the anesthetics, but does not affect healing procedure, to make your selection.
medication that increases surgical risk? (option 2). An oral hypoglycemic agent is used for diabetes, but the medication itself does not
pose added risk to the client during surgery (option 4).

1.‐ An antidysrhythmic
2.‐ A sedative‐hypnotic
3.‐ A corticosteroid
4.‐ An oral hypoglycemic

2700 The following clients are in the preanesthesia holding Correct answer: 4 Nephrectomy is a major type of surgery because the kidney is a major vital organ, loss of The core issue of the question is the degree of surgical risk associated with each
room. The nurse determines that the client undergoing blood is likely to be greater than with the other mentioned surgeries, and there is greater procedure. Use the process of elimination, focusing on the nature of each surgical
which procedure is having the most serious or major likelihood of complications. Options 1, 2, and 3 are all examples of minor surgery because they procedure and the seriousness of each one.
surgery? do not involve a high degree of risk.
1.‐ Tonsillectomy
2.‐ Biopsy of the breast
3.‐ Arthroscopy
4.‐ Nephrectomy
2701 All of the following clients will be having surgery this Correct answer: 1 Dementia affects the person s understanding of the proposed surgery and ability to The core issue of the question is the degree of surgical risk associated with each client
morning. The nurse concludes that which client is most cooperate with the perioperative care; it also affects the medications given. Cultural circumstance. Use the process of elimination, recalling that physiological issues take
likely to be at higher surgical risk? differences should not pose a risk unless the client s beliefs are contrary to the proposed precedence over psychosocial ones, and that previous surgery might or might not be
measures. Mild anxiety will not create a risk, and previous surgeries could be helpful for the relevant to the current surgery.
client to draw on previous experiences.
1.‐ A client who has dementia
2.‐ A client who is culturally different than the medical personnel
3.‐ A client who has mild anxiety
4.‐ A client who has had previous surgeries

2702 The nurse is preparing a client for surgery. Prior to Correct answer: 1 Abrasions, pustules, or other skin conditions have to be assessed and documented because The core issue of the question is knowledge of integumentary risks to a surgical procedure.
completing the skin preparation, the nurse assesses they can interfere with wound healing, or increase the risk of infection. Hair growth lack of it Use the process of elimination, focusing on skin breaks or alterations as the option that
the surgical site for which of the following? or presence of lanugo or fine hair will not interfere with the skin preparation. Pulsation is not interferes with the protective function of the skin.
always visible or available to assess, depending upon the part of the body being operated on.

1.‐ Presence of pustules or abrasions


2.‐ Absence of hair growth
3.‐ Presence of lanugo
4.‐ Absence of pulsation

2703 The nurse asks the client about previous surgeries. Correct answer: 4 Previous surgeries can reveal possible difficulties or problems with certain anesthetic agents, Focus on the issue of the question, which is the need to gather assessment data that could
The client asks the nurse why this information is but do not necessarily interfere with absorption of anesthetics (option 1), hinder put the client at risk during the surgical procedure. With this concept in mind, eliminate
important. The nurse would explain that previous comprehension of instructions (option 2), or affect the central nervous system (option 3). each of the other options that are false statements.
surgeries can do which of the following? However, they can affect the physiological or psychological responses of the client to the
planned surgery.
1.‐ Interfere with the absorption of anesthetic agents.
2.‐ Affect the ability of the client to comprehend the instructions prior to surgery.
3.‐ Affect the central nervous system.
4.‐ Alter the client s responses to surgery.

2704 A client who is scheduled for an outpatient surgical Correct answer: 1 Alcohol affects the central nervous system, and therefore the client s response to surgery and The core issue of the question is knowledge that alcohol has an interactive effect with
procedure arrives at the hospital. During the the anesthetic itself. Smoking, not alcohol (in small amounts), poses respiratory risks. Alcohol anesthesia and possibly other medications used during surgery. Focus on the option that
preoperative assessment, the nurse smells alcohol on effects cannot be reduced by the use of sedatives or hypnotics. Past and recent intake of safeguards the client’s physical status as the reason for notifying the surgeon.
the client’s breath. The nurse reports this finding to alcohol can impact responses, which can be either slowed down or escalated.
the surgeon, prior to completing the preoperative
assessment, after drawing which conclusion about the
significance of this finding?

1.‐ Alcohol can affect the client’s response to anesthesia and surgery.
2.‐ Alcohol can increase the risk for respiratory complications.
3.‐ Use of sedatives and hypnotics prior to surgery can decrease the alcohol effects.
4.‐ Physiological and psychological responses are slowed down by recent alcohol intake.

2705 The staff nurse is asking questions about the Correct answer: 4 The ability of the client to see and hear could affect the preoperative and postoperative Eliminate options 1 and 2 because the postdischarge time frame makes these less relevant
preoperative client s vision and hearing. A family teaching methods used. Social resources and accident prevention rely not only on the client s to the current situation. Choose option 4 over option 3 because it is more specific, and
member asks the nurse why these questions are vision and hearing (options 1 and 2), but also on family supports and the client s physical and applies to the client's situation.
important prior to surgery. What information should mental status. Unexpected needs is a very general term that can be applied not just to vision
the nurse provide as the primary reason for seeking and hearing but also to any area of client functioning (option 3).
this information?
1.‐ “This will help us determine the need for additional resources after discharge.”
2.‐ “This will help assess the risk of accidents in the home after surgery, which could affect the surgical outcome.”
3.‐ “This helps identify any unanticipated needs prior to beginning the surgery.”
4.‐ “This will help us to individualize how we provide preoperative and postoperative teaching.”

2706 A client is admitted for surgery. During the Correct answer: 3 Anticoagulants inhibit clotting of the blood, putting the client at increased risk for bleeding The core issue of the question is knowledge that warfarin sodium is an anticoagulant, and
preoperative assessment, the nurse learns the client is postoperatively. Delirium tremens needs to be monitored for clients who had problems with that this medication increases risk of bleeding unless stopped for a sufficient amount of
taking warfarin sodium (Coumadin). When planning alcohol use (option 1). Respiratory compromise might occur if clients take sedatives or time before surgery (approximately 7 days, depending on client and surgery). With this in
postoperative care for this client, the nurse would hypnotics (option 2). If clients are taking diuretics or cardiovascular agents, fluid volume could mind, eliminate options 1 and 2 first. Choose option 3 over 4 because there are other
include monitoring for which of the following? be a problem (option 4). causes of hypovolemia besides bleeding, such as inadequate fluid replacement during
surgery or the postoperative period.
1.‐ Delirium tremens
2.‐ Respiratory depression
3.‐ Bleeding or oozing at the surgical wound
4.‐ Hypovolemia

2707 While planning postoperative care for an obese client Correct answer: 1 Wound and cardiovascular complications are more common among clients who are obese. Recall that obesity leads to increased cardiovascular risks in general, and that it can also
prior to surgery, the nurse would develop which The heart is stressed from its workload, and the added stress of surgery could place the client be a risk factor for poor wound healing after surgery. Eliminate options 2 and 4 first as
nursing diagnosis specific to the effect obesity has on at risk. The client has no risk for excess fluid volume (option 2), and decreasing fluid intake being least related to the core issue of the question, and choose option 1 over 3 as the
postsurgical recovery? could complicate wound healing. Pressure ulcers occur more frequently in emaciated clients priority risk.
than in obese ones (option 3). The obese client has no problem with thermoregulation (option
4).
1.‐ Risk for Ineffective Tissue Perfusion (Cardiopulmonary)
2.‐ Excess Fluid Volume
3.‐ Risk for Impaired Skin Integrity (Pressure Ulcers)
4.‐ Ineffective Thermoregulation

2708 The postsurgical unit nurse is implementing measures Correct answer: 1 Antiembolic stockings facilitate venous return from the lower extremities. Smoking can Focus on the critical word in the stem, prevent. Discriminate between those options that
to prevent thrombophlebitis. Which of the following is contribute to cardiovascular events, but cessation will not necessarily lessen the chance of address assessment and those that address prevention.
the priority action by the nurse? thrombophlebitis in the immediate postsurgical period. Assessment of the leg will help with
detection, but it will not prevent thrombophlebitis. Homans' sign is pain on dorsiflexion of the
leg, and this is also a means of detection, not prevention.
1.‐ Apply ordered antiembolic stockings.
2.‐ Reinforce importance of smoking cessation.
3.‐ Assess the legs with each set of vital signs.
4.‐ Teach the client to report Homans' sign.

2709 A client has been admitted for surgery for resection Correct answer: 4 A surgical procedure that relieves symptoms of disease or pain but does not cure is described Use the process of elimination, selecting the answer that is an accurate description of the
of nerve roots. The client, observing the written as palliative. The other options are incorrect explanations. meaning of the term palliative.
comment that the surgery is palliative, asks what this
means. The nurse would offer which of the following
explanations?
1.‐ The surgery schedule is overbooked, so the client s surgery could be delayed.
2.‐ The surgeon is against performing the surgery.
3.‐ The exact surgical procedure has not been decided.
4.‐ The procedure will be done to relieve pain, but will not cure the problem.

2710 The physician progress note indicated a plan to let a Correct answer: 4 A wide scar occurs in tertiary intention because the edges are not approximated, and they First, recall the definition of tertiary intention. Then, visualize the appearance of the
client’s wound heal by tertiary intention. The nurse regenerate via granulation. Options 1 and 3 refer to secondary healing, while option 2 is wound to make your selection.
determines that healing has occurred when which of characteristic of primary healing.
the following is observed?
1.‐ The wound is smaller but irregular.
2.‐ Very little scarring has occurred.
3.‐ Tissue loss prevents the edges from approximating.
4.‐ A wide scar is present over the area of wound closure.
2711 A postoperative client is observed to have moderate Correct answer: 4 Purulent drainage is made up of tissue debris, WBCs, and bacteria; it could be comprised of The critical word in the stem of the question is next. This means that the correct option is
wound drainage that has a greenish tinge. The nurse different colors, depending upon the type of bacteria, and it is thick in consistency. It often the one that contains a critical‐thinking sequence based on the information presented.
should take which of the following actions next? indicates wound infection. The next action by the nurse would be to gather additional data Correlate the word purulent with infection, and then choose the option that assesses for
that could indicate infection, such as elevated temperature and WBC count. The nurse would signs of infection.
document the findings at some point, but this is not the priority action. There is no reason to
assess for bleeding within the wound, or to measure pulse and BP.

1.‐ Document the expected findings.


2.‐ Check for bleeding at the base of the wound.
3.‐ Take the pulse and blood pressure, and compare with previous readings.
4.‐ Note the latest temperature and white blood cell (WBC) count.

2712 A client experiences wound dehiscence when Correct answer: 3 Covering the wound with sterile, saline‐moistened gauze keeps the wound moist, and The core issue of the question is nursing management of wound dehiscence. Recall that
coughing. After assisting the client to a low Fowler’s protects it from infection. Option 1 is incorrect; in wound dehiscence, the layers of the wound the priority sequence of actions is to remove the effects of gravity on the wound, and then
position with legs slightly elevated, what is the next are disrupted, but there is no protrusion of vital organs. In addition, pushing back organs such to protect the wound. Eliminate options 1 and 4 as ineffective, and then choose correctly
best action? as the intestines is extremely dangerous because it could cause strangulation. A hydrocolloid based on knowledge of various types of wound dressings.
dressing is not indicated (option 2), because its absorptive properties are not needed. Option 4
would be ineffective, and does not protect underlying tissue.

1.‐ Push the internal organs back into the abdominal opening.
2.‐ Cover the wound with a moist hydrocolloid dressing.
3.‐ Cover the wound with a sterile, saline‐moistened dressing.
4.‐ Use Steri‐Strips to hold the wound together.

2713 A client is scheduled for surgery, and has been placed Correct answer: 2 By keeping the stomach empty during surgery, the risk of vomiting and aspiration is Use knowledge of basic principles of preoperative care to make a selection. The wording
on NPO status. The client complains of thirst and decreased. The other options are unrelated to NPO status. of the question tells you that there is only one correct choice.
hunger, and asks for breakfast. The nurse would
explain that which of the following is the purpose of
NPO status?
1.‐ To make anesthesia induction easier
2.‐ To avoid the risk of aspiration
3.‐ To prevent excessive bleeding
4.‐ To allow the wound to heal faster

2714 The nurse is teaching a client about wound care in Correct answer: 2, 3 Return demonstration is the best way to evaluate teaching of a procedure. Ideally, the Use fundamental principles of teaching and learning to answer the question, recalling that
preparation for discharge. How should the nurse teaching is done over a few days, and is then evaluated. Having the client explain the the best methods of evaluation involve knowledge and action on the part of the client,
evaluate the effectiveness of homecare teaching on procedure is also appropriate, because it indicates that the client has the necessary knowledge which can be determined by verbal explanation and return demonstration.
wound care? Select all that apply. to perform the procedure. Giving a paper‐and‐pencil quiz and having the client critique a video
would measure cognitive aspects of learning, but are not realistic. Asking the client detailed
questions during the procedure is not helpful, because it detracts from learning.

1.‐ Give a paper‐and‐pencil quiz.


2.‐ Have the caregiver or client demonstrate the procedure.
3.‐ Have the client or caregiver explain the procedure.
4.‐ Have the client or caregiver critique a video on the procedure.
5.‐ Ask the client detailed questions while demonstrating the procedure.
2715 Which of the following activities would the nurse Correct answer: 1 Assessment in the preoperative phase includes anticipating any health problems that might Recognize that the word “identify” in option 1 is a clue to the correct response. Nursing
carry out in the preoperative period for a client occur during and after surgery. Option 2 is applicable during the intraoperative phase, when care during the preoperative period is aimed at identification of issues that might present
scheduled for surgery? specific, specialized activities are carried out in the operating room. Option 3 is a very general problems that would result in complications for the operative client.
activity that should occur continually. Prevention of complications (option 4) occurs in the
postoperative stage.
1.‐ Identify potential or actual health problems.
2.‐ Perform specialized procedures to maintain safety.
3.‐ Assess the client's response to interventions.
4.‐ Intervene to prevent complications.

2716 The nurse is caring for several clients in the pre‐ Correct answer: 3 Ablative surgery involves removal of diseased body parts. Option 1 is an example of Use the meaning of the word “ablate” to identify the correct response as one that would
anesthesia room. Of the following client situations, transplant surgery, option 2 is for a diagnostic purpose, and option 3 is for a palliative purpose. indicate removal of a part.
which merits ablative surgery?
1.‐ Replacing a hip that has degenerative disease
2.‐ Identifying if a tumor is malignant
3.‐ Removing a diseased part of the kidney
4.‐ Resecting a nerve root

2717 A client having surgery has a degree of risk associated Correct answer: 2 Risk is not associated with the place where surgery is performed; also, this is not a client‐ Focus on a client‐related factor that would raise surgical risk.
with the surgery. What client‐related factor is related factor. Risk is associated with poor nutritional status, so option 3 is incorrect. The
responsible for a high degree of risk associated with higher the likelihood of complications, the greater the risk, making option 4 incorrect. When
surgery? surgery is performed on vital organs, there is a greater likelihood for complications, and
therefore the risk is higher.
1.‐ Type of institution where surgery is performed
2.‐ Involvement of vital organs
3.‐ Average nutritional status
4.‐ Little likelihood of complications

2718 An infant who is having surgery has a higher risk than Correct answer: 2 The infant has immature vital organs that affect his ability to metabolize medications, such as Recognize that the word immaturity in option 2 is key.
does an adult. The nurse would conclude that which of the anesthetic, and the ability to resist infection. Infants do not suffer from declines in
the following is a reason for the increased risk? functioning (option 1). Hypothermia is more likely to occur than hyperthermia, since the infant
has an immature temperature regulation and large body surface area (option 3). The volume
of blood in an infant is limited, and does not fluctuate (option 4).

1.‐ Decline in functioning


2.‐ Immaturity of vital organs
3.‐ Increased possibility of hyperthermia
4.‐ Volume of blood fluctuation

2719 A preschool‐age child is facing surgery, and might Correct answer: 1 Option 1 is correct, since toddlers and preschool‐age children are fearful of painful events. Look for the response that best corresponds to the developmental issues experienced by a
have fears related to the surgery. The type of fear the The other options are incorrect; appearance is not a primary concern at this age, anticipating preschool‐age child.
nurse would anticipate in this child would be which of an inability to do certain things is not a concern at this developmental level, and children at
the following? this age are unaware of competency issues of medical personnel.

1.‐ Being awake during surgery, and experiencing pain


2.‐ Looking drastically different after surgery
3.‐ Not being able to do the things she used to do
4.‐ Medical personnel not knowing what they are doing
2720 A client has just entered the postanesthesia care unit Correct answer: 1 Although all the options contain aspects that need assessment, the parameters in option 1 Select the option that addresses the most important thing to monitor, vital signs.
(PACU) from surgery. The postoperative client's are the most important to assess initially because they relate to physiological needs and are
immediate needs include initial monitoring of which of more global indicators of overall functioning than are the other options.
the following items?
1.‐ Vital signs, level of consciousness, and presence of pain
2.‐ Skin coloring, surgical incision, and limb movements
3.‐ Skin temperature, blood pressure, and mental status
4.‐ Temperature, emotional status, and social support

2721 The nurse in the PACU is assessing a postoperative Correct answer: 1 The color of the skin, nails, and lips are indicators of tissue perfusion, and pallor and cyanosis Recall that tissue perfusion would be noted only by change in color.
client. Which of the following indicators suggest indicate alteration. Mobility, pain, and fluid loss are incorrect, as they are not signs of tissue
alteration in tissue perfusion? perfusion.
1.‐ Pallor or cyanosis
2.‐ Difficulty with mobility
3.‐ Pain in the incision area
4.‐ Fluid loss

2722 After surgery, the nurse encourages the client to Correct answer: 4 Turning side‐to‐side allows the lungs alternately to expand properly. Peristalsis increases with Select the option that is of greatest concern in the postoperative client, which is
move from side to side at least once every two hours. movement even without the turning, and muscle weakness can be lessened with movement. respiratory function.
The client questions this activity. The nurse explains Turning does not necessarily induce sleep.
this intervention is to achieve which of the following?

1.‐ Lets peristalsis return at a faster rate


2.‐ Lessens muscle weakness
3.‐ Increases the client's ability to sleep
4.‐ Lets the lungs alternately achieve maximum expansion

2723 The nurse is assessing the client's surgical wound in Correct answer: 3 The first signs of healing are absence of bleeding and wound edges bound by fibrin in the clot. Eliminate option 2 as not being indicative of healing, and option 4 as being a later stage
the postoperative period. Which finding indicates the Inflammation at the wound edges follows the first sign, and then, when the clot diminishes, than is described in the question stem.
first stage of healing? inflammation decreases, and collagen forms a scar.
1.‐ Inflammation in the wound edges
2.‐ Bleeding around the incision
3.‐ Clot binding the wound edges
4.‐ Collagen synthesis

2724 The nursing team is creating a care plan for a Correct answer: 3 Absence of pain indicates that the client is comfortable. The other options indicate goals of Recognize that option 3 is the only outcome that directly addresses the diagnosis.
postoperative client. The diagnosis is Pain. An either risk reduction or restoration of well‐being.
appropriate client outcome for this client would be
which of the following?
1.‐ Balanced fluid intake and output
2.‐ Seeks help as needed.
3.‐ Absence of nonverbal signs of pain
4.‐ Performs leg exercises as instructed.

2725 A client is being admitted to the hospital on the day Correct answer: 3 Option 1 is not the best question initially, as it focuses not on the client but on the doctor. Select the option that is open‐ended, and gives the client the opportunity to use his own
before a scheduled surgery. Which of the following is Option 2 is not an appropriate initial question. Option 4 is challenging, and not appropriate as words in explaining the procedure.
the most appropriate initial question to ask this an initial question. Option 3 is correct, as it will lead to the nurse's further exploration.
preoperative client?
1.‐ "Has your doctor talked to you about the type of surgery you are having? What did the doctor say?"
2.‐ "What questions do you have about your surgery?"
3.‐ "What type of surgery are you having, and why are you having it done?"
4.‐ "What do you know about what will be done to you?"
2726 A presurgical client asks the nurse for more Correct answer: 2 Respiratory and circulatory depression is a disadvantage of general anesthetics because there The question asks for the advantage of general anesthesia. Option 2 is the only response
information about the advantages of a general is a greater risk for complications, especially for clients with chronic illnesses. General that can be considered an advantage.
anesthetic. The nurse's answer would correctly reflect anesthetic agents are rapidly excreted, and produce amnesia. General anesthetics produce
what information? central nervous system depression, so clients do not feel the pain of surgery.

1.‐ The respiratory and circulatory functions are depressed.


2.‐ The client loses consciousness, and does not perceive pain.
3.‐ The anesthetic agent is not rapidly excreted, so the timing of the surgery can be adjusted.
4.‐ General anesthesia reduces the chance that the client suffers from amnesia.

2727 A benzodiazepine has been administered to a client Correct answer: 2 Benzodiazepines such as lorazepam and diazepam decrease anxiety and produce side effects Preoperative medications are generally given to reduce anxiety, and, as a side effect,
preoperatively. After the drug has been administered, such as hypotension and sedation. Major tranquilizers such as chlorpromazine produce produce hypotension.
1.‐ Anxiety
2.‐ Hypotension
3.‐ Hypocalcemia
4.‐ Extrapyramidal reactions

2728 A preoperative client has elevated hemoglobin and Correct answer: 4 Increased hemoglobin and hematocrit might be a result of dehydration. Immune deficiency is Increased hemoglobin and hematocrit are often the result of hemoconcentration caused
hematocrit levels. What would the nurse suspect an indication of decreased white blood cell count (option 1), while an increase in electrolytes, by dehydration.
regarding the significance of these increased values? such as potassium, sodium, or chloride, indicates kidney dysfunction (option 2). Malignancy
may be suspected with increased platelet count.
1.‐ Immune deficiency
2.‐ Kidney dysfunction
3.‐ Malignancy
4.‐ Dehydration

2729 The nurse has completed preoperative teaching to a Correct answer: 1 The anesthetic agent is injected into the subarachnoid space for spinal anesthesia, and into Eliminate options 3 and 4 as being descriptive of other forms of anesthesia.
pregnant woman. During the discussion, the nurse the epidural space (which is outside the dura mater) in epidural anesthesia. Regional
describes the different types of anesthesia available. anesthesia can include local or topical anesthesia or nerve blocks, and does not require clients
Which statement by the client indicates understanding to have sedation, or produce amnesia.
of regional anesthesia?
1.‐ In spinal anesthesia, the anesthetic agent is injected into the subarachnoid space.
2.‐ The anesthetic agent is injected into the dura mater of the spinal cord for epidural anesthesia.
3.‐ The client is sedated, and has some awareness of the event.
4.‐ Regional anesthesia produces analgesia and amnesia.

2730 The client arrives in the PACU in an unconscious state. Correct answer: 1 Option 1 is the correct answer, as in this position, gravity keeps the tongue forward, which Select the option that provides the highest level of airway protection.
In what position is the unconscious client placed in the prevents aspiration. A pillow elevates the head, semi‐prone position is unsafe in most cases, as
immediate postanesthetic stage? it can interfere with breathing.
1.‐ Side‐lying, with the face slightly down
2.‐ Side‐lying, with a pillow under the client's head
3.‐ Semi‐prone, with the head tilted to the side
4.‐ Dorsal recumbent, with the head turned to the side

2731 The client has been in the PACU unit for an hour. The Correct answer: 3 Excessive bloody drainage on dressings or the bedclothes, often underneath the client The bedclothes underneath the client are often the site of blood collection from
client is now groggy, but able to respond to voice (because of gravity), indicates hemorrhage. hemorrhage.
commands. While assessing the client, the nurse
checks the bedclothes underneath the client. The
nurse is assessing:
1.‐ Drainage from the tubes or drains.
2.‐ Fluid balance.
3.‐ Hemorrhage.
4.‐ Perspiration.

2732 A client is in the postoperative stage, and the Correct answer: 2 Option 2 is the correct answer, as the client has an inability to retain the information, and Focus on the key phrase in the question, “[t]he client has shown difficulty understanding
physician has ordered ambulation. The client has therefore has a deficiency in knowledge base. Self‐care Deficit is incorrect, as there is no ...”, to select the correct answer.
shown difficulty understanding the necessity for early indication of inability to perform self‐care activities such as bathing and eating. Options 3 and 4
ambulation. An appropriate nursing diagnosis for this are definitely incorrect.
client would be:
1.‐ Self‐care Deficit.
2.‐ Knowledge Deficit.
3.‐ Ineffective Coping.
4.‐ Risk for Injury.

2733 The nurse is assessing the client on return to the Correct answer: 1 The drain allows for drainage of excessive fluid and purulent material that might have Select the option that best addresses the purpose of a surgical drain, which is to remove
hospital unit from the PACU, and notes the presence of accumulated during the surgery. Healing is promoted, but not necessarily at a rapid rate, and fluids.
a drain in the surgical wound. A family member not all drains have to be shortened or connected to suction.
observes the drain, and asks why the tube was left in
the wound. The nurse explains that drains:

1.‐ Allow drainage of excessive fluids such as blood, edema, or pus from the surgical site.
2.‐ Allow healing to occur at a very rapid rate.
3.‐ Have to be shortened to allow healing to occur from the inside out.
4.‐ Have to be connected to suction tubes.

2734 A client is being discharged following outpatient Correct answer: 2 Option 2 could indicate wound infection. All options except 2 indicate normal wound healing Omit options 1, 3, and 4 as being expected, and not necessary to report.
surgery. The nurse is providing the caregiver with or characteristics.
instructions for wound care. The nurse would instruct
the caregiver to report which of the following findings
to the surgeon?
1.‐ Scar formation
2.‐ Increased redness or drainage
3.‐ No odor of the wound drainage
4.‐ No unusual color of the drainage

2735 The nurse admitting a female client from the Correct answer: 2 With decreased oxygen saturation, more oxygen is needed, but the best choice is to stimulate The nurse may presume that pain is the cause for decreased oxygen levels and increased
postanesthesia care unit (PACU) gives the following the client who still has some anesthetic effects after surgery. The other options will not have pulse. But without the blood pressure and respiratory rate also increased, pain usually is
report: BP 100/64; temperature 97; pulse 90; this beneficial effect. not the problem. Increasing the IV fluids will not improve the oxygen problem. Increasing
respirations 12; and O&lt;sub&gt;2&lt;/sub&gt; the oxygen flow rate might help, but if the client is not breathing deeply enough, or often
saturation 88%. The skin is pale but warm to the touch; enough, increasing the flow rate will not solve the problem. Stimulation of the client to
pulse is irregular but strong and equal bilaterally; breathe more and deeper will oxygenate the client better.
respirations shallow without extra effort; drowsy, but
responds to verbal command; IV is intact; Foley
catheter has 250 mL new urine since the last hour;
chest tube connected to three‐chamber collection
device in place to low wall suction; nasal cannula in
place with oxygen at 2 liters/min. The nurse should
take which priority action based upon the assessment
data?

1.‐ Ask the client if she is in pain


2.‐ Stimulate the client to breathe deeply and cough
3.‐ Increase the IV flow rate, to balance losses in urine and chest drainage
4.‐ Increase the oxygen flow rate, to raise the O2 saturation level

2736 The nurse concludes that which postoperative activity Correct answer: 3 Preoperative teaching should be reinforced postoperatively. By asking to take the TED hose The directions for the incentive spirometer, turning, and fluid status are correct. No
by the client would demonstrate a need for additional and SCDs off, the client indicates that he does not comprehend that the purpose is to additional teaching is needed for those topics. Only the desire for removal of the TED hose
teaching? maximize blood flow to the legs at all times, especially when lying in bed. and SCDs needs additional instructions.

1.‐ Demonstrates how and when to use the incentive spirometer for ten repetitions several times per hour.
2.‐ Tells the staff to help turning every two hours when log rolling is ordered.
3.‐ Requests that the elastic hose and sequential compression devices (SCDs) be removed while sleeping at night.
4.‐ Tries to drink at least eight ounces of fluids every hour while awake.

2737 The client had abdominal surgery five weeks ago, and Correct answer: 1, 3, 4 The complication that could have been avoided was deep vein thrombosis (DVT). Fluid intake can help with circulation to some extent, but if blood is allowed to become
now is having leg pains. What activities could the nurse Repositioning every two hours, avoiding crossing one s legs, and wearing TED hose and SCDs all stagnant, or is restricted by positioning, fluids do not decrease the risk of clot formation. All
have performed when the client was hospitalized that will increase the blood flow to the legs and minimize venous status, which causes the clots to other activities will minimize venous status and decrease the risk of DVT.
would have minimized this complication’s risk? Select form.
all that apply.
1.‐ Turn the client every two hours.
2.‐ Increase the fluid intake to at least eight ounces per hour.
3.‐ Tell the client to avoid crossing the legs.
4.‐ Instruct the client to wear TED hose and SCDs while in bed.
5.‐ Massage the client’s legs.

2738 A 78‐year‐old client with chronic obstructive Correct answer: 2, 4 The symptoms are of possible dehiscence and evisceration. The nurse cannot report to the If you try to call the doctor before assessing the wound, you have no data to report.
pulmonary disease (COPD) has had abdominal surgery, physician what has occurred without first assessing the problem. A sterile towel and sterile Applying pressure over the wound will not return the contents into the abdominal cavity if
and suddenly feels something “let go” in the incision normal saline are used to maintain a moist environment until the client goes back to surgery. they have eviscerated. You can do harm by decreasing the blood flow from the pressure,
underneath the dressing when coughing. What are the Staying with the client is essential to calm him down and explain the circumstance. You should causing tissue necrosis or tissue hypoxia if continuous pressure is applied.
nurse’s immediate actions? Select all that apply. have a second nurse call the doctor to report the problem.

1.‐ Call the physician before checking the wound.


2.‐ Open the dressing, and view the problem.
3.‐ Apply pressure over the site, and have someone else call the physician.
4.‐ Use a sterile towel and sterile saline to keep the open incision moist.
5.‐ Sit the client upright in bed.

2739 What findings might indicate to the nurse that a client Correct answer: 1 The calf is the most common place for a deep vein thrombosis to develop. Although it is a Recall first the common location of DVTs, and recall that DVTs lead to a disturbance in
has developed a deep vein thrombosis (DVT)? nonspecific finding, the classic symptom is pain on dorsiflexion. As the calf muscle compresses venous blood flow. Use this knowledge to eliminate options 2 and 3, and eliminate option 4
the thrombus, pain is triggered. Although the affected leg could be warm to the touch (option because these are systemic symptoms rather than local ones.
3), the warmth would be in the affected calf area, not the thigh. Skin coolness (option 2) and
decreased pulses below groin level would indicate a decreased arterial blood flow, not venous,
and this rare occurrence would likely be associated with arterial thrombosis (a possible
complication of catheter insertion at the femoral artery). A generalized fever, chills, and
dyspnea (option 4) would indicate a systemic infection rather than a DVT.

1.‐ Pain in the calf on dorsiflexion


2.‐ Skin coolness of the entire limb, with decreased pulses starting at the groin
3.‐ Skin hot to the touch with a tenderness felt over the thigh
4.‐ Generalized fever of 101°F, chills, and dyspnea
2740 The nurse plans to do which of the following as part Correct answer: 1, 2 A three‐way urinary catheter is used to irrigate the bladder after surgery most of the time. Limiting salt is not a good idea, since the sodium level can drop with frequent irrigations of
of the postoperative management of a client who will The common assessment factors will indicate the rate of flow, bleeding, and other possible the bladder, depending upon the solution that is chosen. Restricting oral fluids is not
be admitted from the postanesthesia care unit with a problems. A common problem with surgery on the bladder is bladder spasms that create pain feasible, and should be discouraged; additional fluids will help to keep the urinary system
three‐way urinary catheter? Select all that apply. with the cramping. The B &amp; O suppository is the most common treatment to relax these flowing without complications. When the irrigating solution is adjusted, the following rules
spasms, since it is a belladonna‐and‐opioid combination drug. apply: a) Increase the rate when the urine is darker, to try to wash out any excess tissue or
bleeding; b) Decrease the rate when the urine is becoming clearer, to further assess the
status of the bladder. Increasing the flow when the urine is clear will only serve to wash out
electrolytes and waste solution.

1.‐ Assessment of color, amount, consistency, and content of urine


2.‐ Administration of belladonna‐and‐opium (B & O) suppository for bladder spasms
3.‐ Limit salt and oral fluids to prevent fluid overload.
4.‐ Increase the irrigation solution when the urine becomes clearer.
5.‐ Clamp the tube for 15 minutes out of each hour.

2741 A client is one day postop for an abdominal hernia Correct answer: 2 Atelectasis is collapsed lung tissue from decreased depth of breathing. The sounds that would A friction rub, as described in option 1, is present when two adjacent tissues are inflamed
repair. Which of the following symptoms alerts the be heard over the collapsed tissue would be diminished or absent, depending upon the and rubbing against each other, such as when an infarcted myocardium rubs against the
nurse that the client might be developing atelectasis? amount of tissue involved. Since most clients lie in bed on their backs, the collapsed tissue is pericardial sac. Because of the location described, this must be a pericardial friction rub
usually bilaterally in the posterior lower lobes. rather than a pleural friction rub. Crackles (option 3) are heard when fluid is present in the
alveoli, but this would be expected in the lower lobes if they occurred because of
atelectasis. Wheezing (option 4) is present when spasms reduce the air flow through the
bronchi and bronchioles of the lung.

1.‐ Friction rub at fifth anterior intercostal space at the midclavicular line
2.‐ Diminished posterior breath sounds bilaterally
3.‐ Crackles in the upper lobes the of anterior chest
4.‐ Wheezing in the upper bronchi on expiration

2742 An elderly client has intravenous (IV) fluids infusing at Correct answer: 2 The elderly are more prone to congestive heart failure when their systems are overloaded Raising the head of the bed will help the dyspnea, but will not improve the cardiac
125 mL/hour. The client reports the following with fluids that cannot be adequately pumped. The symptoms in the question stem are functioning enough as a first step. But raising the HOB could be the second step. Decreasing
symptoms: bubbling sputum, difficulty breathing classical symptoms of congestive heart failure. Reducing the IV flow rate will not do harm, and the IV flow rate will decrease the workload of the heart. Intubation and ventilation might
except in an upright position, and progressive will allow further assessment without increasing the strain on the heart. be possible, but not at this time, since the client is still breathing on her own. Increasing the
weakness. On inspection, the nurse finds increased flow rate of oxygen might help, but 4 liters per minute could cause decreased respiratory
jugular vein distention and blood‐tinged coloring in the rate, since it is the CO&lt;sub&gt;2&lt;/sub&gt; in the blood that stimulates respirations,
sputum. What is the nurse’s first action? and a high oxygen flow rate might reduce the CO&lt;sub&gt;2&lt;/sub&gt; rate below the
respiratory stimulation levels.

1.‐ Raise the head of bed.


2.‐ Decrease the rate of IV fluids.
3.‐ Prepare for intubation and ventilation.
4.‐ Apply oxygen at 4 liters/minute.

2743 When teaching a client about home management Correct answer: 1, 2 Postoperative management of any eye surgery client includes avoiding activities that might Shampooing should be delayed, to avoid contamination of the surgical site with dirt or
following cataract removal with lens implant surgery, increase intraocular pressure. Straining at stool and lifting heavy objects both raise the shampoo solutions. Strict bedrest is not recommended, and most clients are asked to sit up
the nurse should include which of the following in the intraocular pressure, and should be avoided. Stool softeners are used to present this straining. in recliners rather than lie flat in bed. Getting up and performing daily activities are not
teaching plan? Select all that apply. contraindicated, since they do not increase pressure in the eye. Safety issues with partial
loss of vision from dressings, eye patches, or blurring should be included in the educational
process to prevent injury from falls and other problems.

1.‐ Take a stool softener and/or laxative daily to prevent constipation.


2.‐ No lifting or bending over for at least six weeks.
3.‐ Shampoo the hair daily with mild soap.
4.‐ Stay on strict bedrest until you return to the doctor in three weeks.
5.‐ Wash the face and around the eyes three times daily.
2744 In a client who is one day postoperative following Correct answer: 3 Urinary output should be at least 30 milliliters per hour, and 150 milliliters per shift does not Blood loss is expected in laminectomy surgery, and the hemoglobin of 10.5 is low, but
lumbar laminectomy, which finding would be of show adequate renal perfusion or functions. within acceptable ranges. BUN also is at the upper edge of normal levels, and should be
greatest concern to the nurse? watched, but does not require urgent management. The first 24 hours postanesthesia, it is
not unusual not to have bowel sounds in all four quadrants. Continued assessment would
be necessary, but not urgent. Without adequate renal functions, the client could go into
complete renal shutdown, and further assessment should be made immediately.

1.‐ Hemoglobin 10.5 grams/dL


2.‐ Blood urea nitrogen (BUN) 8 mg/dL; creatinine 1.2 mg/dL
3.‐ Urinary output of 150 mL for last shift
4.‐ Decreased bowel sounds in all four quadrants

2745 The nurse working in a busy surgical unit has four Correct answer: 4 The client who already has impaired respirations due to increased weight would be at Respirations of 14 are within normal range, and as long as the client can be awakened by
postsurgical clients who are receiving patient‐ greatest risk for respiratory complications when receiving a narcotic that suppresses verbal stimuli, the client is not at risk for respiratory problems. The COPD client has good
controlled analgesia (PCA) with fentanyl (Sublimaze). respirations. Snoring loudly reflects poor air exchange. Narcolepsy suggests that he is sleep‐ oxygen saturation, and is not in any immediate danger. The small, frail, elderly client could
Which of the following clients does the nurse plan to deprived from respiratory obstruction at night. Close assessment should be given to this client. develop problems, but has been healthy all her life, and does not have any symptoms of
assess most frequently? problems at present. With her low weight and loss of subcutaneous tissue, she would need
to be encouraged to breathe deeply to prevent problems, but she currently is not in any
difficulty.
1.‐ A client with respirations of 14 per minute who is easily awakened by spoken command
2.‐ A client with COPD who has an oxygen saturation of 92%
3.‐ A 94‐year‐old female who has never been sick, but weighs only 45 Kg
4.‐ A male client weighing 150 kg who has noisy breathing and a history of narcolepsy

2746 Which client with a nasogastric (NG) suctioning tube Correct answer: 4 Whenever the client states she is choking or having difficulty breathing, further assessment The color and amounts of solutions are correct for each situation based upon the surgical
would be the nurse’s priority for client assessment? should be done immediately. The tubing could be coiled at the back of her throat, or swelling problem involved. Each is an expected assessment that is normal. Choking and dyspnea are
might have occurred to limit the air flow. never normal.
1.‐ A two‐day‐postoperative client who had an open cholecystectomy with 150 mL of greenish drainage in the collection bag
2.‐ A one‐day‐postoperative client who had a partial gastrectomy with 200 mL of sanguineous fluid in the collection bag
3.‐ A five‐day‐postoperative client with a repair of a
gunshot wound that nicked the liver and spleen, who
had 40 mL of thick yellowish green secretions in the
surgical drain
4.‐ A client with a newly inserted NG tube who complains of “trouble breathing” from the tube

2747 When staffing the surgical nursing unit, which of the Correct answer: 1 Delegation to UAP can include any activity that is within their role. Ambulating the client and Activities that require assessment, sterile procedures, and clients in unstable conditions
following clients could the nurse delegate to an administering an enema are within this role. are not assigned to UAP staff. These are roles of the professional nurse, and cannot be
unlicensed assistive person (UAP)? delegated.
1.‐ A third‐day‐postoperative client who needs to be walked and have an enema this bedtime
2.‐ A new surgical client who has just complained of oozing from the mastectomy site
3.‐ A new amputee who is stable, but needs dressings reinforced
4.‐ A fourth‐day coronary artery bypass client who still has stable angina after surgery

2748 Which PRN medication would the nurse anticipate Correct answer: 1 Drug management includes knowing what the counteragent is for common dangerous drugs. Protamine sulfate is used to counteract heparin. Vitamin K is used to treat overdoses of
using when a client is suspected to have taken too Narcan is the acceptable antidote for narcotic overdose. Coumadin. Romazicon is used to treat overdoses of benzodiazepines.
many narcotics, and has a respiratory rate of
approximately 8 breaths per minute?

1.‐ Naloxone (Narcan)


2.‐ Protamine sulfate (generic)
3.‐ Phytonadione (vitamin K)
4.‐ Flumazenil (Romazicon)

2749 A client has a triple‐lumen catheter in place. There Correct answer: 1 Triple‐lumen catheters have three ports for openings at the tip, which is located just outside The other two ports (medial, proximal) are smaller openings, and could damage the blood
are orders for a unit of packed red blood cells (PRBCs) the right atrium of the heart. The distal port is located at the tip of the catheter, and is the cells when forced through them. A separate IV site is not necessary, since the medication
and normal saline at 75 mL/hour IV, and the client largest gauge of the three. It is 16‐gauge in size, and is suitable for infusing colloids and blood can be given through the saline IV solution with an intravenous push of the medication. If
needs an IV push pain medication at the same time. To products. The other two ports are 18‐gauge openings, and are used for blood draws or routine the unused port is used for pain med administration, then the port must be re‐heparinized
which port would the nurse connect the PRBCs for the IV crystalloid fluids. after the drug administration, to maintain patency between uses. Starting another site is
safest administration? also probably difficult, since the client would not have the central line unless other accesses
were difficult or inadequate.

1.‐ Distal port


2.‐ Medial port
3.‐ Proximal port
4.‐ None; start a second IV site for the blood.

2750 A client is about to undergo skin biopsy to determine Correct answer: 3 The area is anesthetized using a local anesthetic before skin biopsy, so the client should only The core issue of the question is pain during a skin biopsy procedure. Eliminate option 4
if a skin lesion is malignant. The client asks how much feel discomfort while the anesthetic is administered. Analgesics are not given before the because it does not address the client’s concern. Use knowledge that local anesthesia is
the biopsy will hurt. Which response by the nurse is procedure (option 1), and the procedure is not pain‐free (option 2). The client may take used during the procedure to make your selection.
best? medication such as acetaminophen following the procedure (option 4), but this does not
address the client’s question about pain during the procedure.

1.‐ “We will give you a pain pill in just a moment that will minimize any pain during the biopsy.”
2.‐ “Luckily, this type of procedure does not cause any pain for most people.”
3.‐ “You may feel some discomfort while the local anesthetic is injected, but this will numb the area for the actual biopsy.”
4.‐ “This procedure does cause some pain, but you can manage any soreness afterward with over‐the‐counter medications such as acetaminophen (Tylenol).”

2751 The client is about to undergo a computerized Correct answer: 2 Because a contrast agent will be used for the test, it is most important for the nurse to ask The core issue of the question is knowledge that a client who is allergic to iodine or
tomography (CT) scan of the head with contrast. about an allergy to iodine or shellfish. While it is good to know if the client has had a similar shellfish is likely to have an allergic reaction to iodinated contrast media. Eliminate options
Which of the following questions by the nurse is most test for anxiety reduction, it is not the priority. The client should not have anything to eat or 1 and 4 as least important, and then eliminate option 3 because fluids are not allowed prior
important to ask while preparing the client for the drink for four hours prior to the test. It is generally helpful for the client to void before leaving to CT scans with contrast.
test? the unit to avoid having to do so during the test, but this is also a lower‐priority item than
assessing for allergy.
1.‐ “Have you ever had a procedure like this before?”
2.‐ “Do you have an allergy to iodine or shellfish?”
3.‐ “Would you like something to drink before you go to the Radiology Department?”
4.‐ “Have you voided in the bathroom in the last few hours?”

2752 The client is scheduled for a magnetic resonance Correct answer: 4 Because the MRI scanner uses magnets, the client cannot wear any metal, and clients who The core issue of the question is knowledge that metal cannot be worn in the vicinity of an
imaging (MRI) study of the spine. The outpatient nurse have implanted metal might be ineligible for this study. The client does not need to withhold MRI scanner because of the magnetic field. Use nursing knowledge about this test and the
instructs the client that which of the following is an food or fluids before the test. The client does not need to remain in the department for process of elimination to make a selection.
important part of preprocedure care? additional observation after the test, and can drive herself home.

1.‐ “Do not eat or drink after midnight the day before the test.”
2.‐ “Plan to have someone drive you home after the test.”
3.‐ “Expect to stay in the MRI Department for an hour afterward for observation.”
4.‐ “Do not wear any metal, such as jewelry or hairclips.”
2753 A client will undergo a radionuclide scan of the Correct answer: 2 The amount of residual radioactivity following radionuclide scanning is very small, and poses The core issue of the question is knowledge that the amount of radioactivity following
thyroid. A nursing assistant asks what needs to be no risk to visitors or staff. Using standard precautions in handling blood or body fluids is radionuclide imaging is very small. With this in mind, eliminate each of the incorrect
done to protect staff from any residual radiation sufficient for protection. It is unnecessary to stand six feet away from the client, use a private options, which contain excessive and unnecessary steps for protection of staff.
following the scan. Which of the following is an room, or place the client on contact precautions.
appropriate response by the nurse?

1.‐ “All caregivers and visitors need to stand six feet away from the client for 24 hours. I need to put a sign above the bed.”
2.‐ “Using standard precautions for handling body fluids will be sufficient to protect staff.”
3.‐ “I have arranged for the client to be moved to a private room for 48 hours after the test.”
4.‐ “The client will need to be on contact precautions. I will have the unit secretary call the central processing department for a cart with gowns and gloves.”

2754 A 17‐year‐old girl is brought to the Emergency Correct answer: 3 The most important question is to determine whether the client could be pregnant, since x‐ The core issue of the question is knowledge that x‐rays are contraindicated during
Department for x‐rays after twisting her ankle while rays are contraindicated during pregnancy, especially during the first trimester. The question pregnancy. Note the critical words most important in the question, which tells you that
playing basketball. Which of the following questions is second in importance would be whether the client is wearing any metal, but possible more than one option could be correct, and you must prioritize your answer. Use general
most important for the nurse to ask the client before pregnancy is a priority. It is helpful, but not of highest priority, to know if the client has had an knowledge of x‐rays and the process of elimination to make a selection.
sending her to the Radiology Department? x‐ray before, to alleviate concerns. Asking about fear of small or closed spaces would be
important for MRI machines and possibly for CT scanning machines.

1.‐ “Do you experience claustrophobia when in small spaces?”


2.‐ “Are you wearing any necklaces or other metal objects?”
3.‐ “When was your last monthly period?”
4.‐ “Have you ever had an x‐ray before?”

2755 A female client is returning to the nursing unit Correct answer: 1 There is no special aftercare following pelvic ultrasound. For this reason, the nurse should The core issue of the question is knowledge that there is no special aftercare following
following a pelvic ultrasound. The nurse plans to do make the client comfortable and ask if she needs anything before leaving the room. The client ultrasound as a diagnostic test. Use nursing knowledge and the process of elimination to
which of the following at this time for the client? does not need to drink fluids, should not have cramping pains, and does not need to wait an make a selection.
hour before eating.
1.‐ Make the client comfortable, and ask if she needs anything.
2.‐ Instruct the client to drink at least one quart of water over the next hour.
3.‐ Explain that analgesic medication is available to relieve the expected cramping pain.
4.‐ Tell the client that she will be able to eat in one hour.

2756 A client who underwent bronchoscopy four hours ago Correct answer: 4 Before offering food or fluids to a client following bronchoscopy, it is essential to ensure that The core issue of the question is knowledge that gag and swallow reflexes need to be
is asking for something to drink to ease his sore throat. gag and swallow reflexes have returned. A local anesthetic is used to numb the throat to ease present before offering clients food or beverages to prevent aspiration. Think of gag and
The nurse obtains some juice for the client after noting passage of the bronchoscope, and if protective reflexes have not returned, the client could swallow reflexes as a possible priority concern whenever a client has had a procedure
which of the following assessment data? aspirate. The other client data are also normal, but would not indicate whether the client can ending in ‐oscopy. Otherwise, use nursing knowledge of this key principle and the process
safely swallow. of elimination to make a selection.
1.‐ Respiratory rate has ranged from 16 to 18.
2.‐ Breath sounds are clear bilaterally.
3.‐ The client has had no hemoptysis.
4.‐ Gag and swallow reflexes have returned.

2757 A client underwent angiography of the left leg. Which Correct answer: 3 The assessment finding that should be of greatest concern to the nurse is the adverse change The core issue of the question is the most serious adverse change in the neurovascular
of the following data obtained during the current in the distal pulse on the leg that underwent angiography. Skin that is paler and cooler is also status of a client who underwent angiography. The critical words in the stem of the
assessment is of greatest concern to the nurse? of concern, but the reason for these adverse changes is the reduced circulation to the leg, question are greatest concern, which indicates that more than one piece of data might be
which is in turn caused by the decreased pulse. A bandage that has a small amount of old abnormal. Use nursing knowledge about adverse circulatory changes and the process of
blood is expected, and is not of concern at this time. elimination to make a selection.

1.‐ Skin paler on left foot than right.


2.‐ Skin temperature cooler on left foot than right.
3.‐ Left dorsalis pedis pulse audible by Doppler, previously 2 .
4.‐ BAND‐AID at femoral access site has trace amount of dark red blood.
2758 The client who will undergo a cardiac catheterization Correct answer: 4 The client is asked to lie still except for specific requests, such as to cough or deep‐breathe to The core issue of the question is knowledge of typical events during a cardiac
says to the nurse: “I am nervous about having a cardiac aid in catheter movement, or to terminate cardiac dysrhythmias caused by irritation of the catheterization. Knowledge of these factors helps alleviate client fears. Use nursing
catheterization. Can you tell me what to expect during catheter. The procedure is done in a special cardiac catheterization room in the radiology knowledge and the process of elimination to make a selection.
this test?” Which of the following replies by the nurse department, not in the operating room. The lights in the room may be dimmed at times so
is appropriate? catheter movement can be visualized on a fluoroscopy screen. The catheter insertion site is
anesthetized with a local anesthetic, so the client should feel pressure but not pain.

1.‐ “The procedure will be done in the operating room to help ensure sterile conditions.”
2.‐ “The room will be brightly lit at all times.”
3.‐ “The insertion of the catheter in the femoral area will be one of the few painful moments of the procedure.”
4.‐ “The physician will ask you to lie still except to do specific things, such as cough or take a deep breath.”

2759 A client who is scheduled for a cardiac Correct answer: 1 There is no restriction of food or fluids prior to a cardiac (or any) echocardiogram. This test The core issue of the question is knowledge of client preparation for echocardiography.
echocardiogram at 9 a.m. the following morning asks uses sound waves emitted from and reflected back to a transducer, and it is noninvasive. Recall that this is a noninvasive test to eliminate option 4, and then use nursing knowledge
the nurse if he will be able to eat breakfast before the Options 2 through 4 are all variations of an incorrect response. and the process of elimination to make a selection.
test. Which of the following responses by the nurse is
appropriate?
1.‐ “Yes, we can arrange for your breakfast tray to arrive a half hour early so that you have time to eat before the test.”
2.‐ “Yes, but you will need to get up at 5 o clock so that you will be without food or fluids for four hours before the test.”
3.‐ “Yes, but you can only drink clear liquids, such as ginger ale, black tea, or apple juice, and you cannot eat solid food until after the test.”
4.‐ “No, you cannot eat or drink before the test, but you can have a full breakfast after the test.”

2760 A client has just received a Holter cardiac monitor to Correct answer: 3 The client should go about his usual daily activities and exercise pattern while wearing the The core issue of the question is knowledge of proper use of a Holter monitor. The
wear for the next 24 hours. The nurse determines that monitor, and should record activities and any symptoms experienced in the diary. The client wording of the question tells you that only one option is a correct statement. Use nursing
the client understands its use when the client makes does not need to make diary entries every 30 minutes, but as needed to provide an overview knowledge and the process of elimination to make a selection.
which of the following statements? of activity so that it can be correlated with any cardiac abnormalities on the time‐stamped
electrocardiogram being recorded. The client should not take a bath or a shower while wearing
the device, which has electrical circuitry. The client does not need to walk a total of three miles
during the 24‐hour period.

1.‐ “I should write in the diary what I am doing every half‐hour.”


2.‐ “I should only take a bath, not a shower, for the next 24 hours.”
3.‐ “I can continue with my usual activity and exercise pattern while wearing the monitor.”
4.‐ “I need to try to walk a total of three miles over the next 24 hours while wearing the monitor.”

2761 A client who underwent cystography 16 hours ago has Correct answer: 3 The client has 15 mL less than the expected minimum urine output of 240 mL in 8 hours. The The critical words in the question are at this time, which tell you that more than one
a urinary output of 225 mL in the previous 8 hours. first steps by the nurse would be to assess the client s fluid intake and encourage the client to option might be plausible, but there is a sequence of nursing actions appropriate to the
Which of the following actions should the nurse take drink increased fluids. Although it is not incorrect to measure specific gravity, the nurse could situation. Recall that when urine output is low, an early measure is to offer increased fluids.
as a priority at this time? expect the value to be high if the urine output volume were low because of poor intake. Use this nursing knowledge and the process of elimination to make a selection.
Documenting the value is insufficient because further nursing action is warranted. The nurse
should call the physician if the reduced output continues after increasing the client s fluid
intake.
1.‐ Measure the specific gravity of the urine.
2.‐ Document the volume on the client’s flowsheet.
3.‐ Encourage the client to drink more fluids.
4.‐ Notify the physician.

2762 The nurse has assigned a nursing assistant to a client Correct answer: 2 The nursing assistant should wash the client s hair to remove the paste or colloidon that was Use nursing knowledge and the process of elimination to make a selection.
who just returned to the nursing unit at 0945 after used to secure the electrodes to the head for the diagnostic test. The client should be able to
electroencephalography. Which of the following eat and drink, and can resume usual activity unless otherwise ordered. There is no dye used in
directions should the nurse give the nursing assistant this diagnostic test.
regarding care for the client?
1.‐ “Do not give any food or fluids until lunchtime.”
2.‐ “Wash the client’s hair at your earliest opportunity.”
3.‐ “Keep the client on bedrest for the remainder of the shift.”
4.‐ “Encourage the client to drink fluids to flush dye through the kidneys.”

2763 A client has just returned to the nursing unit following Correct answer: 1 Following myelogram with water‐based contrast, the head of the bed needs to be elevated to The core issue of the question is knowledge of correct head position following myelogram
a myelogram, which was done to diagnose a herniated 60 degrees to reduce the risk of meningeal irritation from any residual contrast in the spinal using water‐based contrast. Use nursing knowledge and the process of elimination to make
intervertebral disk. The medical record indicates that a fluid. If an oil‐based contrast was used, the head of the bed would need to remain flat. The a selection.
water‐based contrast medium was used during the other options indicate incorrect responses, because the head of the bed is too low to prevent
test. The nurse should assist the client to which of the headache from meningeal irritation as a complication of the procedure.
following positions in bed after transferring from the
stretcher?

1.‐ Supine, with the head of the bed elevated 60 degrees


2.‐ Supine, with the head of the bed elevated 15 degrees
3.‐ Left side‐lying, with the head of the bed flat
4.‐ Any position of the client's choice, with the head of the bed elevated 30 degrees

2764 A client has received discharge instructions after Correct answer: 3 The client should limit joint movement, including walking, for 2–3 days after arthroscopy. The core issue of the question is knowledge of measures to prevent complications and aid
undergoing arthroscopy of the knee earlier in the day. Analgesics are often needed to manage pain, and the client should be instructed about what to healing after arthroscopy. Use nursing knowledge about care following arthroscopy and the
The nurse concludes that the client understands self‐ use and how often to take it. The physician may order ice to control swelling, but not heat, process of elimination to make a selection.
care after discharge when the client makes which of which would aggravate swelling. Increased swelling and bleeding after discharge should be
the following statements? reported, because these are abnormal findings, and could indicate a complication of the
procedure.
1.‐ “I should not expect to need pain medication following this procedure.”
2.‐ “I should apply warm, moist heat to my knee to maintain comfort.”
3.‐ “I should limit my activities, including walking, for 2–3 days.”
4.‐ “I should expect increased swelling and perhaps some bleeding in the knee area after going home.”

2765 A client with gastroesophageal reflux disease has just Correct answer: 1 Because the throat is anesthetized so the client can tolerate the endoscope, the client’s gag Use the ABCs airway, breathing, and circulation to answer the question. Options that
undergone esophagogastroscopy. Which of the and swallow reflexes are temporarily lost during any upper endoscopy procedure, such as involve the airway are frequently the highest‐priority items. Use nursing knowledge and the
following client data is the nurse’s highest priority for esophagogastroscopy. The nurse’s priority is to monitor for return of these protective airway process of elimination to make a selection.
continued monitoring? reflexes. While mildly elevated temperature and reports of heartburn and sore throat also
bear continued monitoring, they are of lesser priority than concerns related to the client’s
airway.
1.‐ Inability to swallow saliva
2.‐ Temperature of 99.4°F oral
3.‐ Client report of heartburn
4.‐ Client report of sore throat

2766 The nurse has given instructions to a client who will Correct answer: 3 The client should not eat or drink anything for 8–12 hours before the test, so the client should The core issue of the question is knowledge of dietary preparation before a barium
have a barium swallow in three days. The nurse not eat or drink anything after midnight. Oral medications are usually withheld before the swallow or upper GI series. Use ordinary logic to determine that the GI organs would be
determines that the client understands how to procedure as well. difficult to visualize if they contained food or fluid. Use nursing knowledge and the process
properly prepare for the test after the client makes of elimination to make a selection.
which of the following statements?
1.‐ “I should eat a low‐fat meal for the next two days, and then have clear liquids the day before the test.”
2.‐ “I should stop taking all medication except antacids the day before the test.”
3.‐ “I should not eat or drink anything after midnight on the day of the test.”
4.‐ “I should eat a high‐carbohydrate diet for the three days before the test.”
2767 The nurse is providing instructions to a client who is Correct answer: 2 The diet may be resumed after colonoscopy, but the client usually tolerates it better if it is The core issue of the question is knowledge of self‐care following colonoscopy. Use
returning to home following colonoscopy. Which of resumed gradually. The client should not drive for about 24 hours, until all medications have nursing knowledge and the process of elimination to make a selection.
the following statements would be appropriate for the fully worn off. It is normal to pass gas and feel bloated because of the carbon dioxide used to
nurse to include? insufflate the colon to visualize the area. It is abnormal for bleeding to be present, and the
client should notify the physician if it occurs.
1.‐ “You may drive in about six hours, after all the medication given during the procedure has fully worn off.”
2.‐ “It is alright to eat and drink, but it is helpful to resume the diet gradually.”
3.‐ “You should call the doctor if you feel distended, or begin passing gas.”
4.‐ “Bleeding from the rectum is expected after this procedure, but call the physician if it gets severe.”

2768 The nurse would give which instruction regarding Correct answer: 2 The client should avoid using any skin products, such as lotions or deodorant, on the skin of The core issue of the question is knowledge that skin products should be avoided to
preprocedure care to a woman who is scheduled for a the breast or underarm prior to mammogram. The client may eat and drink as usual. Although prevent possible skin damage prior to a radiographic procedure such as a mammogram.
mammogram? the procedure might cause some women discomfort with compression of the breast, it is not The wording of the question tells you that only one option is a correct statement. Use
necessary to premedicate with analgesics. There is no activity restriction following the test. nursing knowledge and the process of elimination to make a selection.

1.‐ “Drink liquids, but don’t eat breakfast on the morning of the mammogram.”
2.‐ “Do not use any deodorant or lotions on the chest or underarms before the mammogram.”
3.‐ “Take a mild analgesic such as acetaminophen (Tylenol) before coming to the clinic for the mammogram.”
4.‐ “Plan a light schedule for the day of the mammogram, so you can plan on getting some rest after the procedure.”

2769 The occupational health nurse has given an Correct answer: 2 A Mantoux test (or PPD test) to screen for tuberculosis should be read in 48–72 hours. If the The core issue of the question is knowledge of specific time frames for reading a PPD test.
intradermal injection of purified protein derivative test was planted on Monday, the result must be read in 2–3 days, which is Wednesday or Remember that when there is more than one part to an option, the entire option must be
(PPD) to a client to screen for tuberculosis. After Thursday. The other options are either partially (options 1 and 3) or completely incorrect correct for that option to be the correct answer. Use nursing knowledge and the process of
noting that the current day is Monday, the nurse (option 4). elimination to make a selection.
instructs the client to return to have the result read on
which of the following days?

1.‐ Tuesday or Wednesday


2.‐ Wednesday or Thursday
3.‐ Thursday or Friday
4.‐ Friday or the following Monday

2770 A client will undergo basal gastric acid secretion Correct answer: 1, 3, 5 Selected drugs (antacids, steroids, cholinergics, and anticholinergics) and coffee and alcohol The core issue of the question is knowledge of drugs that could interfere with basal gastric
analysis. The client is taking the following types of should be restricted for at least 24 hours prior to test; note on the test request form if the acid testing and analysis. The wording of the question tells you that more than one option
medications. Which of the following types of drugs client has not complied with the restrictions. There is no reason to withhold a cardiac glycoside might be correct. Use nursing knowledge and the process of elimination.
should the nurse withhold prior to the test? Select all or a diuretic, because these medications would not affect the test results.
that apply.
1.‐ Anticholinergic
2.‐ Cardiac glycoside
3.‐ Antacid
4.‐ Diuretic
5.‐ Steroid

2771 A client has a blood urea nitrogen (BUN) of 68 mg/dL Correct answer: 4 Potassium is contraindicated in clients with renal dysfunctions. It cannot be filtered out if Protein creates more potassium in the body, and the lab shows that the kidneys are not
and a creatinine level of 6.0 mg/dL. The IV fluid is 5% there is decreased renal filtration. With increased damage in tissues, additional potassium is filtering as they should. Additional potassium from protein metabolism could cause death.
dextrose in 0.9% sodium chloride, with 40 mEq released, causing an even higher level of potassium that can be life‐threatening. Activities such as ambulation will not change the BUN or creatinine, since they reflect
potassium chloride (KCl) at 100 mL/hour. Which action filtration of the renal system, and not the rate of circulation of the blood. Taking the vital
would be most appropriate for the nurse to take? signs every hour only gives you information about the circulatory status, and does not
explain or improve the renal functions. Action needs to be taken immediately to
discontinue the IV with the potassium to minimize the buildup of potassium to toxic levels
that could be life‐threatening.

1.‐ Encourage more protein in the diet.


2.‐ Ambulate the client more, to increase circulation.
3.‐ Take vital signs every hour.
4.‐ Question the use of potassium in the IV fluids.

2772 A client is suspected of having hyperkalemia. Which Correct answer: 1 Since potassium is an intracellular ion, higher levels will alter the electrical pattern of the EKG. Muscle weakness, flaccidity of muscles, diarrhea, abdominal cramping, and cerebral
symptom or sign would the nurse expect to find when The peaking of a T wave is an indication that potassium is too high. irritability/restlessness are present with hyperkalemia (higher‐than‐normal potassium
assessing the client? levels). Therefore, bowel sounds would be hyperactive and not silent, such as with an ileus.
Muscles are weak and flaccid, not in a cramping state. Cerebral functions are stimulated,
and somnolence (sleeping, sluggishness) is not present.

1.‐ Peaking of T wave on the telemetry monitor


2.‐ The absence of bowel sounds, such as in an ileus
3.‐ Muscle cramping of the lower extremities
4.‐ Somnolence with early changes

2773 When a client has arterial blood gases (ABGs) drawn Correct answer: 3 Packing the sample in ice will minimize the changes in gas levels during the transportation of The site, since it is an artery, should be held for at least five minutes or more if the client is
from the radial artery, the nurse should be prepared to the specimen to the lab. anticoagulated. The syringe is heparinized, and the blood is not transferred to a test tube.
perform which follow‐up action? The needle on the syringe needs to be sealed to minimize changes in gas levels during
transport. A second specimen is not necessary for blood gases. If the client is having blood
cultures, many times the doctor will ask for a “blood culture times 2,” which implies a
second specimen is requested. Since these are blood gases and not blood cultures, a second
draw is not needed. (Note: Blood gases are drawn from arteries, and blood cultures are
done from veins.)

1.‐ Apply pressure to the site for up to one minute.


2.‐ Transfer the blood sample to a heparinized test tube.
3.‐ Pack the sample in ice for transporting to the laboratory.
4.‐ Obtain a second specimen after ten minutes, to compare results.

2774 Which laboratory test results would indicate that the Correct answer: 3 Prothrombin time is the test that measures the coagulation times related to Coumadin PTTs are done for heparin drug management, not for Coumadin. INR converts to a
client is adequately anticoagulated on sodium warfarin management. The time that is preferred is 1½–2 times the time of normal clotting without standardized formula the ratio between the client s PT value and the laboratory control
(Coumadin) during the postoperative period, when the drug usage. Therefore, for a therapeutic level, the prothrombin time is expected to be value. INR levels are expected to be between 2 and 3 after 3–5 days of oral anticoagulation;
physician is trying to prevent deep vein thrombosis between 20–30 seconds while on Coumadin. therefore, 1 is too low. Clot retraction time has nothing to do with Coumadin therapy, and
(DVT)? is used to evaluate platelet and fibrinolysis, such as in clients with DIC, snakebite, malaria,
and adult respiratory distress syndrome.
1.‐ Partial prothrombin time (PTT) 25 seconds
2.‐ International normalized ratio (INR) 1
3.‐ Prothrombin time (PT) 19 seconds
4.‐ Clot retraction test 75% in two hours

2775 Which diagnostic results would you expect to find in Correct answer: 1 A low T cell count (normal is 1,500–4,000/mm&lt;sup&gt;3&lt;/sup&gt;) indicates that the T Negative ELISA means that there currently are no antibodies present in the blood, not that
the client who has been newly diagnosed with lymphocytes have been damaged or destroyed by the AIDS virus, causing immune suppression. the virus is not present in the tissue. A negative ELISA test score can occur even if HIV is
acquired immunodeficiency syndrome (AIDS)? CD4/CD8 ratios are altered with AIDS virus damage, causing the ratios to decrease significantly present, since some of the phases eliminate the antibodies temporarily after exposure. A
(normal is 1.0–3.5). This result is within normal range, and does not reflect a decreased T cell high platelet count does not reflect AIDS; this occurs with a splenectomy or polycythemia
count. vera, where excessive cells are present.

1.‐ T cell count of 400


2.‐ Negative enzyme‐linked immunosorbent assay (ELISA)
3.‐ Platelet count of 500,000/mm3
4.‐ CD 4/CD 8 test of 1.5
2776 When a client’s guaiac test for occult blood in feces is Correct answer: 2 The blood of red meat can be identified with the guaiac test; therefore, it often is Iron products will not contribute to a false chemical response that might be interpreted as
positive, the nurse should consider which of the recommended to avoid meat for several days prior to the exam, to minimize the risk that a blood being present. Occult blood by guaiac testing is an excellent indicator for screening
following? false positive might occur. for colon cancer. Few other reasons cause a bleed, and therefore this gives an early
warning for further assessment to possibly catch the cancer in an earlier stage. A blue
coloring result on the test is a positive confirmation that blood is present.

1.‐ Iron preparations can give a false positive.


2.‐ Red meat can give a false positive.
3.‐ Occult blood is not a good screening test for cancer of the colon.
4.‐ A blue color on the results of the test indicates that no blood is present.

2777 When planning to administer blood products by Correct answer: 4 Type O has antibodies against both the A and B antigens (but not antigens on the RBCs). Type Type A has A antigens on the RBC, and B antibodies in serum. Type B has B antigens on the
typing and crossmatching of blood, what should the AB has no antibodies against A or B, but does have A and B antigens on the RBC. Therefore, RBC, and A antibodies. Type AB has A and B antigens on the RBC, and no antibodies. Type O
nurse recall in order to prevent ABO incompatibilities? type O blood antibodies would agglutinate the RBCs of type AB blood. has no antigens on the RBC, and both antibodies in serum. Type O, as the universal donor,
can be given to all other blood types, because none of the principle antigens occur on the
RBCs. Those with type AB are universal recipients, since AB contains no antibodies in the
serum.
1.‐ Type A blood products can be given only to type B clients.
2.‐ Type B can be given to type O clients.
3.‐ Type O can only receive type AB blood products.
4.‐ Type AB cannot receive type O blood products.

2778 The nurse would anticipate that a client with a Correct answer: 2 Coombs’ test tells us that the maternal blood has exposed the fetus’s red blood cells to Coombs testing only evaluates the antibodies attached to the RBCs. The thyroid does not
positive Coombs’ test would be likely to have which antibodies or globulin complexes that can cause agglutination of the RBCs. Agglutination of affect the RBC or immunity by antibodies. SLE is an autoimmune disease process that is
clinical condition? RBCs means that the red blood cell can be broken apart or hemolyzed by these foreign tested by the sedimentation rate or C‐reactive protein analysis that reflects inflammation.
antibodies from the mother as they cross the placental barrier. Coombs test does not have anything to do with SLE. Thrush and fungal infections are not
tested with Coombs either, and are best identified by Wet Mount using KOH preparations
to show the budding yeast.
1.‐ Autoimmune thyroid condition
2.‐ Hemolytic diseases of the newborn
3.‐ Systemic lupus erythematosus (SLE)
4.‐ Thrush or another fungal infection

2779 A client is admitted with Graves’ disease. Which Correct answer: 4 Graves’ disease is an overproduction of thyroid hormone, usually from an autoimmune attack Parathormone is secreted by the parathyroid, and does regulate calcium levels, but it does
testing results would the nurse expect to find when on the thyroid gland. The lab results would be an increase in the total thyroxine (T4) and a not relate to thyroid functioning or to Graves’ disease. Somatotropin is growth hormone
1.‐ Increased parathormone levels, with calcium levels above normal
2.‐ Elevated somatotropin hormones, with decreased basal metabolic rate (BMR)
3.‐ Elevated adrenocorticotropic hormone (ACTH) level
4.‐ Elevated total thyroxine (T4), and decreased thyroid‐stimulating hormone (TSH) levels

2780 Which intervention would the nurse use to be most Correct answer: 4 Hospital‐acquired infections are spread by cross‐contamination between clients. Simple Prophylactic antibiotics can help minimize the risk of infection prior to surgery, but they do
effective in preventing and managing hospital‐acquired handwashing and/or antiseptic foam use between clients will prevent the spread of both types not prevent hospital‐acquired infections that occur from cross‐contamination after surgery.
infections often caused by staphylococci and gram‐ of bacteria. Isolation protocols are not necessary, increase expense for all clients who have fevers, and
negative rods? still would not be effective if prevention of crosscontamination were not maintained.
Culturing all open wounds could identify possible sources of contaminants, but again would
not prevent the spread if techniques were not performed to prevent the spread by
crosscontamination.

1.‐ Administer prophylactic antibiotics to surgical clients.


2.‐ Implement isolation techniques for all clients who have high fevers.
3.‐ Culture all open wounds.
4.‐ Wash hands or use antiseptic foam between all clients.
2781 When trying to obtain the best sputum specimen, Correct answer: 3 The first morning specimen is more concentrated, and is easier to analyze. Mouthwash Use knowledge of basic procedures for specimen gathering and handling, and the process
which directions should the nurse include when contains antiseptic solution that would change the flora present in the specimen. Rinsing with of elimination, to make a selection.
teaching the client how to collect the specimen? water will remove secretions of the mouth without contamination by the mouthwash (option
1). Spitting only removes saliva, and does not require the depth of respiration and coughing
that is needed to produce a specimen of lung secretions. Letting the secretions set for several
hours could cross‐contaminate the specimen with bacteria in the air. Specimens should be
taken to lab for analysis immediately.

1.‐ Gargle with mouthwash before getting the specimen, to remove oral bacteria.
2.‐ Spitting is just as effective as coughing, if it hurts to breathe deeply.
3.‐ Save your first morning specimen, since it is the most concentrated.
4.‐ Save all you can for several hours to get the correct amount that is needed.

2782 If a wound is inflamed, and secretions are noted Correct answer: 3 Aspiration of a wound by sterile needle and syringe is preferred due to the lower likelihood of Swabbing a wound or the Penrose tip would risk the cross‐contamination of skin flora on
around a Penrose drain, what would be the best contamination from skin flora. Both the needle and syringe are sterile, and the surface area the sterile swab. Cutting the dressing will not yield the secretions needed, since they
method for the nurse to obtain a specimen for that the needle touches is the least likely source for contamination from skin flora. already are absorbed into the material. Swabbing is not recommended; however, a sterile
culture? swab can be inserted into the draining wound without touching the skin, to obtain a deep
culture from a wound.
1.‐ Swab the tip of the Penrose drain.
2.‐ Swab around the skin around the Penrose, but not the Penrose tip.
3.‐ Aspirate some secretions into a syringe with a small needle.
4.‐ Cut part of the dressing that has secretions on it, and send it to the lab.

2783 A urinary catheter is suspected as the source of a Correct answer: 2 To prevent cross‐contamination from any other source, a sterile container is used to collect The entire catheter is not needed, and additional flora might result from cross‐
possible infection in a client. How would the nurse plan the tip of the catheter that has been cut with sterile scissors. Sterile‐to‐sterile is always the contamination from the tubing or the bag, from poor technique. Only the tip is required for
to get a specimen from the catheter tip that was just rule to prevent contamination from other sources. analysis. Clean equipment or containers do still contain bacteria, and false readings might
removed from the client? be obtained from the scissors or the clean cup. Swabbing the tip might not get the bacteria,
and the actual tip is needed to see what bacteria or contaminant is present.

1.‐ Place the entire catheter in a sterile container.


2.‐ Cut the tip off with sterile scissors, dropping it into a sterile container.
3.‐ Cut the tip with your bandage scissors, and place in a clean container.
4.‐ Swab the tip, and place the swab in a sterile container.

2784 The nurse would complete which of the following as a Correct answer: 4 Anxiety reduction is needed when the client is waiting for the outcome of tests, to assist her The purpose of the test is to identify a possible problem, and the client's greatest fear is
priority action for a client following a full‐body scan? in processing her feelings and exploring her options based upon the results of the test. that the test will validate that something is wrong. Therefore, communication is the first
priority in care of a post‐test client. Pain is a possibility, since the test does require the
client to lie still for a while. But fear will intensify pain even more if not addressed first. Only
minimal radiation exposure occurs during a scan, and not enough to cause any radiation
sickness (nausea). Bleeding is not a possible outcome from the scan, since the procedure is
not invasive.
1.‐ Pain assessment, due to discomfort of the actual procedure
2.‐ Vital signs, to assess for possible bleeding
3.‐ Prophylactic anti‐emetic, due to radiation exposure causing nausea
4.‐ Therapeutic communication, to reduce possible anxiety caused by outcomes
2785 After which exam would the nurse plan to follow a Correct answer: 1 When testing with a PET scan, the client is injected with a radionuclide that emits positrons, Use knowledge of the various types of diagnostic test and the process of elimination to
protocol to prevent possible exposure to radioisotopes which are special isotopes. The emissions are translated into color‐coded images. It is effective make a selection.
used during the diagnostic procedure? to evaluate stroke, epilepsy, migraine headaches, Parkinson’s disease, dementia, and
schizophrenia. The isotopes are removed through renal excretions, and precautions are
needed to prevent exposure risks. The other tests listed do not require isotopes for analysis of
body tissue. Therefore, none of them require precautions after the testing related to isotope
excretion from the body.
1.‐ Positron emission tomography (PET) scan
2.‐ Magnetic resonance imaging (MRI)
3.‐ Computed tomography (CT) scan
4.‐ Angiography

2786 When giving directions for a client who is about to Correct answer: 3 A full bladder is necessary to bounce the sound waves off to compare other tissues or Fluids are needed to fill the bladder, and are not withheld prior to testing. Bowel
undergo a pelvic sonogram, which statement should structures being assessed. If done during pregnancy, the fetus must be older than 26 weeks in structures do not interfere with the assessment of structures, and an enema is not
the nurse include? order to not have the restriction for the full bladder, since the amniotic fluid would be used at required. Medications do not impact on sound waves, and holding medications is not
that point. necessary for any reason.
1.‐ “Drink nothing for several hours prior to the exam.”
2.‐ “You will be given an enema to cleanse the bowel.”
3.‐ “Have a full bladder.”
4.‐ “Do not have any medications prior to the exam.”

2787 What would the nurse calculate a client s pulse rate Correct answer: 3 One large square is 0.20 seconds in time. The formula is to divide 60 seconds by the number Knowledge of timing on the ECG paper and recognizing the components of the individual
to be if there were four larger squares on the ECG of squares between beats multiplied by 0.2 seconds to equal the beats per minute.<BR /> heartbeat are necessary to calculate heart rates and the timing of various parts of the
graph paper between each of the QRS complexes? individual heartbeat. Recognizing the differences between the larger and smaller blocks is
necessary to calculate rates per minute. One smaller block is 0.04 seconds; the larger block
is 0.2 seconds of time. Formulas are necessary to show rates. Another formula would be to
count the number of beats per 30 larger squares and multiply it by 10 to get a full minute s
count. (Thirty larger squares are equal to six seconds of time.)

1.‐ 55 beats per minute


2.‐ 65 beats per minute
3.‐ 75 beats per minute
4.‐ 85 beats per minute

2788 Which of the following nursing actions would be Correct answer: 3 The EEG measures electrical brain wave activities, and allowing the client to take a nap prior The hair needs to be clean, to allow the gel to give the best electrical conduction possible.
inappropriate for a client who is about to undergo an to the test will alter the outcome of the brain activities. Food is necessary to prevent changes from hypoglycemia, but stimulants are avoided.
electroencephalogram (EEG)? Sleeping will alter the EEG reading, and often the client is sleep‐deprived prior to the exam,
to give a better reading or testing outcome. Medications that alter brain functions are also
withheld for up to 48 hours prior to the exam, to prevent inaccurate readings due to drug
changes.
1.‐ Make sure the hair is cleaned by a shampoo.
2.‐ Avoid caffeine products prior to the exam.
3.‐ Allow a nap prior to the exam.
4.‐ Withhold any tranquilizers or sedatives prior to the exam.
2789 Which of the following pulmonary function Correct answer: 4 With obstructive disorders, there is retention of CO&lt;sub&gt;2&lt;/sub&gt; by constriction An incentive spirometer measures air flow, and increases total lung capacities by allowing
techniques would the nurse consider to be most at the alveolar levels. With pursed breathing techniques, the air is pressurized between the inhalation of more air. Clients with COPD or pulmonary obstructive disorders have
helpful for clients with obstructive pulmonary atmospheric pressure and the alveolar pressure by reducing the air flow on exhalation, thus vasoconstriction of the alveoli, and air will not pass through the constriction. Turning,
disorders? allowing more CO&lt;sub&gt;2&lt;/sub&gt; to be released into the atmospheric air. coughing, and deep breathing will help normal tissue to prevent atelectasis, but diseased
lung tissue will not respond to this simple technique. By raising the flow of oxygen to 4 liters
per minute, you increase the availability of O&lt;sub&gt;2&lt;/sub&gt;, but you decrease
the CO&lt;sub&gt;2&lt;/sub&gt; levels. The obstructive pulmonary clients need to have to
have high levels of CO&lt;sub&gt;2&lt;/sub&gt; to stimulate the medulla s respiratory
center to initiate breathing. Without adequate blood levels of CO&lt;sub&gt;2&lt;/sub&gt;,
breathing will cease.

1.‐ Incentive spirometry


2.‐ Turning, coughing, and deep breathing
3.‐ Oxygen flow rates above 4 liters per minute
4.‐ Pursed‐lip breathing exercises

2790 After a lung biopsy and thoracentesis, in which 500 Correct answer: 3 When removing the fluid or tissue from the lungs, the diaphragm might have been irritated, Pallor and delayed capillary refills are common, due to poor oxygenation from a pleural
mL of purulent fluid was removed, what nursing causing the spasms of coughing episodes. If this cannot be minimized, more respiratory efforts effusion and/or thoracentesis. Improving oxygen saturations is a good outcome, and no
assessment would indicate a need for emergency will deprive the client of the needed oxygen–carbon dioxide exchange that is necessary for life. emergency is anticipated, as they improve in the perfusion process. Equal chest movement
intervention? means that both sides of the thoracic cavity are moving equally well, and without collapse
or shifting of lung tissue. Answer options 2 and 4 are desired outcomes following a
thoracentesis.
1.‐ Lip coloring that is pale, and capillary refills greater than three seconds
2.‐ Oxygen saturation that was moving from 87% to 92%
3.‐ Coughing spasms that cannot be slowed or controlled by the client
4.‐ Equal chest movement with less muscle effort noted.

2791 The nurse is providing care to a client who underwent Correct answer: 1 Diabetes insipidus is a complication for any client who has undergone removal of a pituitary This question calls for specific knowledge of ADH and the alteration in ADH secretion that
removal of a pituitary tumor. Eighteen hours after tumor. Edema of the remaining pituitary gland can cause reduction in the release of commonly occurs following surgery for removal of a pituitary tumor. Remember that ADH
surgery, the urine output is markedly increased, and antidiuretic hormone (ADH), resulting in the movement of water from the glomerulus into the results in the movement of free water (that is, water without sodium) into the collecting
the urine‐specific gravity is 1.002. The nurse expects to collecting tubules of the nephron. The client excretes large volumes of urine with a low specific tubules of the nephron, which results in large volumes of water being removed from the
note which of the following corresponding findings gravity. As water is removed from the vascular compartment, the serum sodium becomes blood. The specific gravity (concentration) of the urine decreases. In addition, removal of
when reviewing results of laboratory tests? concentrated. Thus, the lab result indicates hypernatremia. The serum potassium, osmolality, water from the serum concentrates (and thereby elevates) the serum sodium. Recall also
and hematocrit would not be low (options 2, 3, and 4). that hemoconcentration could also raise, not lower, other lab values.

1.‐ Serum sodium 148 mEq/L


2.‐ Serum potassium 3.4 mEq/L
3.‐ Serum osmolality 263 mOsm/L
4.‐ Hematocrit 29%

2792 The nurse would be most concerned about which of Correct answer: 4 Furosemide inhibits reabsorption of sodium, water, and potassium from the distal renal This question calls for specific knowledge of the action of furosemide, and knowledge that
the following laboratory values obtained for a client tubules and the loop of Henle, leading to a diuresis. The most common electrolyte disturbance hypokalemia is a common side effect of furosemide therapy. Use nursing knowledge and
receiving furosemide (Lasix) therapy? associated with furosemide administration is hypokalemia. BUN and creatinine can be either the process of elimination to make a selection.
elevated or lowered, depending on a client s individualized response to therapy. Similarly, the
hematocrit could rise or fall, depending on the amount of fluid retained in the vascular
compartment.
1.‐ Blood urea nitrogen 20 mg/dL
2.‐ Hematocrit 46%

3.‐ Creatinine 1.1 mg/dL

4.‐ Potassium 3.2 mEq/L


2793 The nurse inserts a nasogastric tube, and it Correct answer: 2 Hypokalemia is an almost universal complication of loss of gastric hydrochloric acid. In this This question calls for specific knowledge that loss of hydrochloric acid triggers the
immediately drains 1,000 mL of fluid. Which of the scenario, loss of the hydrogen ions results in a metabolic alkalosis. In turn, compensation for mechanism whereby the kidneys lose potassium. Use nursing knowledge and the process of
following electrolyte assessments is of greatest this loss takes place in the nephron, where hydrogen ions are retained. The nephron is elimination to make a selection.
concern to the nurse at this time? obligated to excrete potassium, which could result in profound hypokalemia and require
vigilant IV replacement. Other electrolytes might be affected, but not to the degree that
potassium homeostasis is altered.
1.‐ Sodium
2.‐ Potassium
3.‐ Chloride
4.‐ CO2 content

2794 A client who was just admitted to the nursing unit has Correct answer: 3 Elevated uric acid levels are commonly seen with gout, and this is the initial question to ask Note that the stem of the question has the critical word initially, which tells you that more
a uric acid level of 9.5 mg/dL. Which question would the client. Uric acid does not rise with gallbladder disease, and is not affected by green tea. than one option might be technically correct but one is best. Use nursing knowledge related
the nurse ask initially? Although the client could experience renal stones from precipitation of uric acid crystals to uric acid level and gout, and the process of elimination, to make a selection.
(causing flank pain), this is not the initial question to ask, since renal stones are a complication
of gout.
1.‐ “Do you have a history of gallbladder disease?”
2.‐ “Do you drink large amounts of green tea?”
3.‐ “Do you have a history of gout?”
4.‐ “Have you been having any pains in the flank area?”

2795 The nurse is caring for a client who received a renal Correct answer: 2 Serum creatinine is the best indicator of renal function. Decreases in serum creatinine often This question calls for the specific knowledge that creatinine is the best indicator of renal
transplant 24 hours previously. Which of the following are dramatic following renal transplantation. Regular monitoring of serum creatinine levels is function. Use nursing knowledge and the process of elimination to make a selection.
trends in lab studies indicates to the nurse that the imperative in assessing the function of the transplanted kidney. Hemoglobin levels can
new kidney is functioning? increase postoperatively due to blood transfusions. Serum phosphate might decrease long‐
term as the kidney increases excretion of phosphates. However, this is not a reliable indicator
of renal function. Serum sodium levels might fluctuate according to individual clients
sodium–water balance.
1.‐ Hemoglobin 12%, increased from 11.8%
2.‐ Serum creatinine 1.6 mg/dL, decreased from 1.9 mg/dL
3.‐ Serum sodium 140 mEq/L, increased from 136 mEq/L
4.‐ Serum phosphate 4.4 mg/dL, decreased from 4.8 mg/dL

2796 The nurse is caring for a client who has just returned Correct answer: 3 The hematocrit is an indicator of the proportion occupied by the cells in a given volume of This question calls for the specific knowledge that hematocrit can be reduced even if there
from the operating room. Blood loss was minimal, but blood. The hematocrit might decrease when cell volume of the blood is decreased because of is no blood loss. In this case, it is indicative of increased volume, most likely due to
the client was given large volumes of crystalloid fluid blood loss. In addition, the hematocrit might decrease when the liquid portion of the blood administration of intravenous fluids. Use nursing knowledge and the process of elimination
during the procedure. Which of the following volume increases, as would be the case when large volumes of intravenous fluid are to make a selection.
laboratory test results suggests overhydration? administered. Hemoglobin would not be increased in this client situation, nor would sodium
and calcium balance be altered.
1.‐ Sodium 147 mEq/L
2.‐ Hemoglobin 14%
3.‐ Hematocrit 33%
4.‐ Calcium level 8.8 mg/dL

2797 A client is being evaluated for possible appendicitis. Correct answer: 1 Neutrophils are responsible for destruction of bacterial invaders. In acute bacterial infections, This question calls for the specific knowledge that neutrophils are responsible for
Which of the following results of laboratory tests such as appendicitis, the percentage of neutrophils (especially immature bands) in the destruction of bacterial invaders, and will be elevated in acute bacterial infections. Use
suggests most strongly to the nurse the presence an complete blood count will increase. This presence of an increased number of bands in the CBC nursing knowledge and the process of elimination to make a selection.
acute bacterial infection? is known as a “shift to the left.” Lymphocytes are responsible for destruction of viruses.
Erythrocytes and platelets are not affected by infections.

1.‐ Elevated neutrophils


2.‐ Elevated erythrocytes
3.‐ Elevated lymphocytes
4.‐ Elevated platelets

2798 The nurse is assigned to the care of a client who has Correct answer: 4 Lymphocytes are responsible for the destruction of viruses. Thus, the presence of This question calls for the specific knowledge that lymphocytes are responsible for
been admitted with meningitis. A spinal tap has been lymphocytes in the cerebrospinal fluid (CSF) suggests that the meningitis is viral in etiology, destruction of viruses. Use nursing knowledge and the process of elimination to make a

1.‐ Platelets

2.‐ Neutrophils

3.‐ Red blood cells


4.‐ Lymphocytes

2799 In caring for a female client who has a urinary tract Correct answer: 2 A positive leukocyte esterase suggests a urinary tract infection. Leukocytes (white blood cells) This question calls for the specific knowledge that leukocyte esterase will be positive in a
infection with more than 100,000 colonies of contain esterases that react with substances contained in urine. More than 100,000 colonies of sample of urine infected with bacteria. Use nursing knowledge and the process of
Escherichia coli bacteria, which of the following would bacteria (per high‐powered field) are needed before the patient can be diagnosed with a UTI. elimination to make a selection.
the nurse expect to see on the client’s urinalysis Testing for the presence of leukocyte esterase may be done using a voided urine sample, which
report? is simple compared with the need for a catheterized urine sample. Nitrites might be positive in
this situation. White blood cells could be elevated. Potassium in the urine is unaffected, and
rarely measured.

1.‐ Negative nitrites


2.‐ Positive leukocyte esterase
3.‐ Positive potassium
4.‐ Negative WBCs

2800 The nurse is reviewing the results of follow‐up Correct answer: 2 The goal for total serum cholesterol is to keep the value below 200 mg/dL. While options 1 The core issue of the question is knowledge of normal serum cholesterol levels. Use
laboratory studies on a client diagnosed with and 2 are within the normal range, the best outcome of therapy is the value that is the lower nursing knowledge and the process of elimination to make a selection.
hyperlipidemia. Which of the following total of the two.
cholesterol levels indicates to the nurse that the client
has been compliant with diet and medication therapy?

1.‐ 198 mg/dL


2.‐ 174 mg/dL
3.‐ 269 mg/dL
4.‐ 214 mg/dL

2801 A nurse notes the client’s albumin level is 2.4 Correct answer: 1 Albumin is a protein responsible for increasing osmotic pressure and maintaining Determine that this test result is an abnormally low albumin level. Recall that albumin is
grams/dL. The nurse should plan to assess the client intravascular fluid volume. Low albumin levels result in a decrease in intravascular colloid necessary for maintenance of fluid balance between body compartments. Systematically
for which of the following at this time? osmotic pressure, which in turn allows fluid to move out of the blood vessel and into eliminate incorrect options.
interstitial tissues. This condition will be assessed as fluid retention in the form of edema,
crackles, and so on. Skin turgor will be elastic when fluid shifts into the interstitial spaces.
Bowel sounds and mucous membranes would not be affected.
1.‐ Fluid retention
2.‐ Inelastic skin turgor
3.‐ Hypoactive bowel sounds
4.‐ Dry mucous membranes
2802 A client is admitted with complaints of severe nausea Correct answer: 2 The loss of stomach acids creates an imbalance, leading to an excess of alkaline fluids in the First, determine if the imbalance is metabolic or respiratory. Loss of GI fluids is a metabolic
and vomiting for several days. The nurse expects the body. The source of the loss is metabolic, not respiratory. function, so options 3 and 4 can be eliminated. Next, determine if the imbalance is acid or
client is at risk for experiencing which acid–base base. Loss of body acids will lead to an excess of bicarbonate in the body.
imbalance?
1.‐ Metabolic acidosis
2.‐ Metabolic alkalosis
3.‐ Respiratory acidosis
4.‐ Respiratory alkalosis

2803 Troponin levels are ordered on a client to confirm a Correct answer: 3 Troponin is a specific marker for cardiac injury. Elevations in serum levels usually begin 4–6 The question calls for specific knowledge of troponin physiology. Recall the times for
myocardial infarction. When should the nurse plan to hours after onset of symptoms, and peak in 12–24 hours. Drawing the blood in the first 2 hours elevations following cardiac injury and choose the option closest. Choose option 3 over 2
have blood drawn for this test? would be too soon, and waiting longer than 24 hours would miss the times for peak levels. because it is more specific.

1.‐ Within 1–2 hours of onset of chest pain


2.‐ Within the first 24 hours of onset of chest pain
3.‐ Between 6 and 24 hours of onset of chest pain
4.‐ Between 24 and 48 hours of onset of chest pain

2804 The nurse would anticipate that a client with cirrhosis Correct answer: 1 In cirrhosis, the damaged liver is unable to properly metabolize amino acids and synthesize This question tests knowledge of liver functions and cirrhosis. Recall the liver s function as
of the liver would have decreased levels of which of albumin, resulting in decreased serum concentrations. The damaged liver is unable to related to each of the lab values, and systematically eliminate incorrect options.
the following? completely break down bilirubin, and serum levels are elevated. Ammonia is normally
converted to urea; serum levels are increased with liver damage. Prothrombin times are
increased when the liver is unable to synthesize clotting factors.

1.‐ Albumin
2.‐ Bilirubin
3.‐ Ammonia
4.‐ Prothrombin time

2805 The nurse checks the prothrombin time on a client Correct answer: 2 Prothrombin times should be 1.5–2 times the control when anticoagulation therapy is being First, determine if the level is abnormal. Recall that in order to be therapeutic,
with aortic valve replacement who is receiving sodium given, indicating the Coumadin is effective. The level is in the therapeutic range, and the next prothrombin times must be 1.5–2 times the control. Eliminate options 1 and 4 because they
warfarin (Coumadin). The client’s level is 20 seconds; dose should be given as scheduled. Encouraging the client to eat foods high in vitamin K would would be counterproductive to the purpose of the Coumadin therapy. Eliminate option 3
control is 13 seconds. The nurse should take which of reduce effectiveness of the Coumadin. The client would be at risk for bleeding, not for deep because the client is not at risk for this problem.
the following actions? vein thrombosis, when prothrombin times are prolonged.

1.‐ Encourage the client to eat foods high in vitamin K.


2.‐ Administer the daily dose of Coumadin as ordered.
3.‐ Monitor the client closely for signs of a deep vein thrombosis.
4.‐ Withhold the next scheduled dose of Coumadin, and notify the prescriber.

2806 A client is being evaluated for hypothyroidism, and Correct answer: 2 In hypothyroidism, the thyroid gland does not produce thyroxine (T&lt;sub&gt;4&lt;/sub&gt;), The question requires knowledge of pituitary and thyroid hormone functions. Recall the
has had blood drawn to determine TSH and despite being stimulated by the pituitary gland (TSH, thyroid‐stimulating hormone) to do so. negative feedback loop of the endocrine system. Eliminate options 1 and 4 because there is
T&lt;sub&gt;4&lt;/sub&gt; levels. Which of the Elevated TSH and T&lt;sub&gt;4&lt;/sub&gt; levels are seen with secondary hyperthyroidism an increased T&lt;sub&gt;4&lt;/sub&gt; level, which would not be seen with primary
following would support the diagnosis? caused by excessive TSH production by the pituitary. A decreased TSH and elevated hypothyroidism.
T&lt;sub&gt;4&lt;/sub&gt; are seen with primary hyperthyroidism. Decreased TSH and
T&lt;sub&gt;4&lt;/sub&gt; levels are seen in hypothyroidism secondary to insufficient pituitary
secretions.
1.‐ An elevated TSH and elevated T4 levels
2.‐ An elevated TSH and decreased T4 levels
3.‐ A decreased TSH and elevated T4 levels
4.‐ A decreased TSH and decreased T4 levels
2807 A client is admitted with dehydration secondary to Correct answer: 3 Dehydration results in loss of fluids, causing a hemoconcentration of BUN, which is elevated. The question requires analysis of fluid losses on common lab values. Recall that vomiting
prolonged nausea and vomiting. Which of the Sodium would be elevated, not normal, with dehydration. The potassium level is normal, and will result in loss of sodium, potassium, and water. Eliminate options 1 and 2 because they
following lab values would the nurse expect to see? would most likely be lower because of losses from the vomiting. The hematocrit would also be are normal, and you are looking for abnormal values. Eliminate option 4 because it is also
elevated secondary to hemoconcentration. normal, and would be elevated with fluid losses.

1.‐ Sodium 138 mEq/dL


2.‐ Potassium 4.2 mEq/dL
3.‐ BUN 30mg/dL
4.‐ Hematocrit 40%

2808 The nurse is establishing a plan of care for a client Correct answer: 1 Hemoglobin is the oxygen‐carrying component of red blood cells. When levels are decreased, The question asks to choose a priority, indicating that all options might be partially or
who has a hemoglobin level of 7.6 grams/dL. Which of the client will be fatigued, and tire easily. Altered nutrition might be the cause of the low totally correct. Eliminate options 2 and 3 because they are not as high a priority. Choose
the following does the nurse identify as a priority hemoglobin, but it is not of highest priority, since intolerance to activity involves safety option 1 over 4 because it addresses a problem related to the function of hemoglobin.
nursing diagnosis? concerns. Constipation and risk for deficient fluid volume would not be as high in priority.

1.‐ Activity Intolerance


2.‐ Constipation
3.‐ Risk for Deficient Fluid Volume
4.‐ Imbalanced Nutrition: Less than Body Requirements

2809 The nurse should assess for Trousseau's sign in the Correct answer: 4 Hypocalcemia causes excitability of skeletal, cardiac, and smooth muscle tissues. Evidence of Specific knowledge of Trousseau's sign is needed to answer this question. Recall that this is
client with which of the following electrolyte this is seen in Trousseau's sign, a carpopedal spasm. Hypokalemia, hyponatremia, and a carpopedal spasm seen with low calcium and magnesium levels. Eliminate options 1, 2,
abnormalities? hypochloremia would not cause this sign. and 4 because they would lead to muscle weakness, not neuromuscular excitability.

1.‐ Hypokalemia
2.‐ Hyponatremia
3.‐ Hypochloremia
4.‐ Hypocalcemia

2810 The white blood cell (WBC) count of a client is 18,000 Correct answer: 4 Tissue injury can cause an increase in WBCs. The WBC count could decrease with rheumatoid First, determine that the value is abnormally high. Recall conditions that elevate the white
cells/microliter. The nurse attributes this value to arthritis, alcoholism, and viral infections (options 1, 2, and 3). count, such as bacterial infections, stress, and tissue injury. Evaluate each option to
which of the following health problems of this client? eliminate conditions in which the white count is decreased. Choose option 4 because
wound dehiscence is a type of tissue injury.
1.‐ Rheumatoid arthritis
2.‐ History of alcoholism
3.‐ Viral infection
4.‐ Wound dehiscence

2811 The nurse would assess the client for fever and other Correct answer: The normal range for the white blood cell count is The core issue of the question is knowledge of normal laboratory values. Use this
signs of infection if the client s white blood cell count 10,000|10000 5,000–10,000/mm&lt;sup&gt;3&lt;/sup&gt;. For this reason, the nurse would be concerned knowledge to choose an answer. Since specific knowledge is needed to answer correctly,
was noted to be greater than _____ about the risk of infection if the count exceeded 10,000. memorize this value if you found the question difficult.
cells/mm&lt;sup&gt;3&lt;/sup&gt; on the laboratory
report.

2812 A client has an albumin level of 1.8 grams/dL. The Correct answer: 1, 3, 4 Low albumin levels are commonly seen in diseases of the liver, since proper liver function is First, identify that the albumin level is low. Consider conditions in which protein is not
nurse recognizes that probable causes of this include: needed to synthesize albumin. Low levels also are seen in conditions in which adequate synthesized or absorbed adequately. Systematically rule out the options that are not
(Select all that apply.) protein is not ingested or absorbed. Albumin levels often appear elevated in dehydration. connected to albumin.
Albumin levels would not be affected by ketoacidosis or pericarditis.

1.‐ Malabsorption syndrome.


2.‐ Dehydration.
3.‐ Cirrhosis.
4.‐ Liver failure.
5.‐ Diabetic ketoacidosis.

2813 A client has an admitting diagnosis of Rule out Correct answer: 4 Troponin levels greater than 2.2 ng/mL are diagnostic for an MI. Abnormal potassium levels The question asks for the best answer, indicating that choice is most specific to the
myocardial infarction (MI). The nurse interprets that can contribute to cardiac arrhythmias, but are not diagnostic for an MI. An elevated content. In case you are unable to remember the normal values for all tests listed, the
which of the following abnormal findings would best triglyceride contributes to coronary artery disease, but also is not diagnostic for MI. Although question also indicates that all options are abnormal values. Recall knowledge of tests used
support the diagnosis of MI? elevated creatinine kinase levels are used to support a diagnosis of MI, the fractionated to confirm a diagnosis of MI. Eliminate option 1, since it is associated with cardiac
portion of the enzyme, CK‐MB, is preferred, since it is specific to the heart muscle. dysrhythmias. Eliminate option 2, since this is a risk factor. Choose option 4 over 3, since
this is more specific to cardiac injury.
1.‐ Potassium 3.4 mEq/L
2.‐ Triglycerides 200mg/dL
3.‐ Creatinine kinase 45 mcg/dL
4.‐ Troponin 2.5 ng/mL

2814 A client is admitted to the Emergency Department Correct answer: 1 Administration of an intramuscular injection could cause a false elevation of the isoenzyme The item is testing knowledge of factors that would influence or alter the level of CK
with complaints of severe chest pain, and a myocardial levels secondary to muscle trauma. Catheterizing the client, administering oxygen, and isoenzymes. Recall that creatinine kinase is found in cardiac, skeletal, and smooth muscles.
infarction is suspected. Blood work for creatine inserting an intravenous catheter would not affect levels of the isoenzymes. Eliminate options 2 and 3, since they would not affect muscle tissue. Choose option 1 over
phosphokinase (CK) isoenzymes is ordered. Prior to the 4, since the injection goes directly into the muscle; insertion of an intravenous cannula has
blood being obtained, the nurse should avoid: less effect on the smooth muscle of the vein.

1.‐ Administering an intramuscular injection.


2.‐ Catheterizing the client.
3.‐ Giving oxygen to the client by nasal cannula.
4.‐ Inserting an intravenous catheter into the antecubital fossa.

2815 The nurse checks the laboratory test results on a Correct answer: 1 Therapeutic levels of heparin therapy are measured by checking the activated partial The item is testing knowledge of lab studies that measure effectiveness and safety of
client receiving heparin therapy. Which of the thromboplastin time (APTT), which should be 1.5–2.5 times the normal of 20–35 seconds. heparin therapy. Eliminate options 3 and 4, since this test does not measure heparin
following values indicates the therapy needs to be Thirty‐four seconds would be within the normal value, but lower than the therapeutic level to therapy. Choose option 1 over 2, since this level is too low to provide a therapeutic effect.
adjusted? provide anticoagulation. Sixty seconds is within the therapeutic range. Prothrombin times are
used to measure therapeutic levels of Coumadin therapy.

1.‐ APTT 34 seconds


2.‐ APTT 60 seconds
3.‐ Prothrombin time 15 seconds
4.‐ Prothrombin time 30 seconds

2816 In assessing the laboratory findings for a client with Correct answer: 1 Calcium levels are low in clients with renal failure secondary to the kidneys inability to This question tests knowledge of the complications of chronic renal failure. Recall the
chronic renal failure (CRF), the nurse should be aware activate vitamin D, which is needed to absorb calcium. Levels of potassium, phosphorus, and normal functions of the kidney. Systematically eliminate options that are incorrect.
that a decreased serum level of which electrolyte magnesium are elevated secondary to the kidneys inability to excrete excess amounts.
should be expected?
1.‐ Calcium
2.‐ Potassium
3.‐ Phosphorus
4.‐ Magnesium

2817 A client is being evaluated to confirm a diagnosis of Correct answer: 1 Antinuclear antibody is a screening test for collagen disorders, and is specific to SLE. Amylase This question tests specific knowledge of SLE and related immunology lab tests. Recall that
systemic lupus erythematosus (SLE). The nurse would levels are elevated in pancreatitis. IgE is released in response to allergic and anaphylactic SLE is an autoimmune disease in which tissues and cells are destroyed. ANA detects
expect which of the following laboratory tests to be reactions. C‐reactive protein is present with acute bacterial inflammatory conditions. antibodies that destroy cells and tissues. Eliminate option 2, since it is specific to the
performed? pancreas. Even though items 3 and 4 are also immunology tests, they can be eliminated,
since they are not specific to antibodies.
1.‐ Antinuclear antibody
2.‐ Serum amylase level
3.‐ Immunoglobulin IgE
4.‐ C‐reactive protein

2818 A client s laboratory report indicates that his Correct answer: 2 A potassium level of 2.8 is indicative of hypokalemia. Muscle weakness and fatigue frequently First, determine that the level of 2.8 is abnormally low. Recall signs and symptoms of
potassium level is 2.8 mEq/L. The nurse should assess are seen in hypokalemia. Hyperactive bowel sounds and diarrhea are seen in hyperkalemia. A hypokalemia. Systematically eliminate the options that are not related to functions of
the client for which of the following manifestations? positive Chvostek’s sign is seen with hypocalcemia and hypomagnesemia. Blurred vision is not potassium, options 3 and 4.
associated with hypokalemia.
1.‐ Hyperactive bowel sounds
2.‐ Muscle weakness
3.‐ Presence of Chvostek’s sign
4.‐ Blurred vision

2819 A laboratory report on an assigned client indicates Correct answer: 1 Bilirubin levels above 2.0 mg frequently are evidenced by a yellow discoloring of the sclera First, determine that this is an elevated bilirubin level. Recall the signs and symptoms seen
the total bilirubin level is 2.2 mg/dL. When assessing and skin. Dark‐green stools are not associated with elevated bilirubin; clay‐colored stools are. with elevated levels. Systematically eliminate the incorrect options.
the client, the nurse anticipates which of the following Peripheral edema and dry mucous membranes can be associated with causes of elevated
physical assessment findings? bilirubin, but information about the cause of the abnormal level is not provided in the
question.
1.‐ Jaundiced sclera
2.‐ Peripheral edema
3.‐ Dark‐green stools
4.‐ Dry mucous membranes

2820 A client with type 2 diabetes mellitus has blood drawn Correct answer: 4 HGB measures the amount of glucose attached to a hemoglobin molecule. Measurements up The question tests specific knowledge of glycosylated hemoglobin. A critical word is poor,
for glycosylated hemoglobin (HGB). Which of the to 7.5% indicate good diabetic control. Measurements greater than 9% indicate poor control. indicating the most abnormal level is the correct answer. Identify options 1, 2, and 3 as
following levels would indicate that the client has poor acceptable percentages.
control of the diabetes?
1.‐ 4%
2.‐ 5%
3.‐ 7%
4.‐ 9%

2821 The nurse is checking the laboratory tests on a client Correct answer: 1, 5 Pancreatitis produces an inflammation of the organ, which causes an elevation in the white The question tests knowledge of the imbalances in body chemistry seen with pancreatitis.
with chronic pancreatitis. Which of following would blood cell count and the pancreatic enzymes (amylase, lipase, and trypsin). The blood glucose Recall the normal functions of the pancreas. Eliminate option 1, since it is not associated
support this diagnosis? Select all that apply. would be elevated secondary to impaired insulin production from the pancreas. Triglycerides with pancreatic function. Eliminate the options that do not correlate with impaired
are not reflective of pancreatic function. pancreatic function.
1.‐ Elevated triglycerides
2.‐ Elevated lipase
3.‐ Decreased blood sugar
4.‐ Decreased amylase
5.‐ Elevated white blood cell count

2822 A client is receiving allopurinol (Zyloprim) for Correct answer: 1 Allopurinol is a uricosuric drug that inhibits reabsorption of uric acid in the kidney, thereby The question tests specific knowledge of gout and the drug allopurinol. A key word is
treatment of gouty arthritis. Which of the following decreasing serum levels. The white blood cell count also might decrease as the inflammation is effective. Look for options containing the desired action of allopurinol. Eliminate options 3
changes in laboratory values would indicate to the reduced, but this is not due to the action of allopurinol. (Colchicine, also given for gout, and 4, since they do not pertain to gout. Choose option 1 over 2, since it is specific to the
nurse that the medication is effective? reduces migration of leukocytes.) Elevated alkaline phosphatase levels are seen with bone action of allopurinol.
metastasis and bone disease. An increase in neutrophil count might be seen with improvement
of bacterial and viral infections, and leukemia.

1.‐ A decrease in uric acid


2.‐ A decrease in the white blood cell count
3.‐ An increase in alkaline phosphatase
4.‐ An increase in neutrophil count
2823 A client has had blood drawn for carcinoembryonic Correct answer: 2 Chronic cigarette smoking can produce elevated CEA levels. Alcohol, a high‐fat diet, and The question requires specific knowledge of CEA levels. Systematically eliminate options
antigen (CEA). The nurse recognizes that which of the aerobic exercise do not have an impact on the level. you know will not affect the test.
client’s daily habits could produce a falsely elevated
level?
1.‐ Alcohol use
2.‐ Cigarette smoking
3.‐ High‐fat diet
4.‐ Aerobic exercise

2824 The urinalysis report on a client indicates that the Correct answer: 1 Urine‐specific gravity is a measurement of the solute concentration and diluteness of the Recognize that this is an abnormally high level. Analyze each option for a correlation with
specific gravity is 1.035. The nurse can anticipate the urine. 1.035 is greater than the norm of 1.005–1.030, indicating the urine is concentrated with a high urine‐specific gravity. Options 2 and 3 can be eliminated, since they can occur with a
client will: a low water content and/or high solute content. It will be darker than normal, pale yellow high specific gravity but are not as specific as option 1.
urine is, and more concentrated. The urine might have a high specific gravity secondary to a
urinary tract infection, but that cannot be determined by specific gravity. It would be
malodorous secondary to certain medications or infections. It does correspond to frequency of
urination.
1.‐ Have dark, concentrated urine.
2.‐ Have a urinary tract infection.
3.‐ Have malodorous urine.
4.‐ Need to urinate frequently.

2825 A client is being evaluated for possible Correct answer: 1 Hyperparathyroidism is associated with hypercalcemia and hypophosphatemia. Normal Note that all the options are electrolytes. Recall what imbalances are created with
hyperparathyroidism. The nurse interprets that which serum calcium levels are 8.5–10.5, and normal serum phosphorus levels are 1.7–2.6. Sodium is hyperparathyroidism. Options 2 and 4 are within normal limits, and can be eliminated
of the following laboratory values would support this not affected. easily. Choose option 1 over 4, since phosphorus levels are decreased with
diagnosis? hyperparathyroidism.
1.‐ Calcium 12.5 mEq/L
2.‐ Calcium 9 mEq/L
3.‐ Phosphorus 4.0 mEq/L
4.‐ Sodium 142 mEq/L

2826 A client is admitted with angina, and has a history of Correct answer: 3 The range of magnesium is narrow (1.7–2.6). Low levels of magnesium predispose the client Critical aspects of the question are the client s report of angina and history of coronary
coronary heart disease. Which of the following to premature ventricular contractions and dysrhythmias. The sodium is slightly low, but does artery disease, and the word immediately, indicating there could be serious consequences if
laboratory values should the nurse report to the not present as great a danger, and also could be dilutional. The sodium and carbon dioxide the values are ignored. Eliminate options 1 and 4, since the levels are within normal limits.
primary care provider immediately? levels are within normal limits. Choose option 3 over 2, since the level is several points below normal, and magnesium has
a very low therapeutic level.
1.‐ Potassium 3.8 mEq/L
2.‐ Sodium 133 mEq/L
3.‐ Magnesium 1.2 mEq/L
4.‐ Carbon dioxide 26 mEq/L

2827 A client has developed syndrome of inappropriate Correct answer: 2 Serum osmolarity is decreased in SIADH secondary to fluid retention. The serum sodium level The question tests specific knowledge of SIADH. Recall that fluid is retained and sodium is
antidiuretic hormone (SIADH).Which of the following will be decreased due to fluid retention and excess losses through kidney excretion. BUN lost via the kidneys, and the blood becomes hemodiluted. Option 4 can be eliminated, since
laboratory findings would the nurse expect to find? would be low due to fluid retention. Potassium is not affected. it is not affected by SIADH. Eliminate option 3, since it is the high end of normal. Choose
option 2 over 1, since it reflects the fluid retention and low sodium level.

1.‐ Blood urea nitrogen (BUN) 35 mg/dL


2.‐ Serum osmolality 250 mOsm/kg
3.‐ Sodium level 148 mEq/L
4.‐ Potassium level 5.0 mEq/L
2828 The nurse is checking laboratory results on a client Correct answer: 1 Serum carbon dioxide reflects the precursor to bicarbonate levels in the body. It is combined The question requires specific knowledge of acidosis and alkalosis. The question is direct;
who is receiving a carbonic anhydrase inhibitor for with another hydrogen and oxygen molecule to make bicarbonate. Elevated levels indicate there is only one right answer. Eliminate options 2 and 4, since they are not connected to
treatment of uric acid stones. The nurse should check alkalosis, while decreased levels indicate acidosis. Osmolarity measures the percentage of acid–base balance. Choose option 1 over 3, since the urine pH would not always correlate
which laboratory value to detect an adverse effect of solutes in the blood. Carbonic anhydrase inhibitors help to eradicate uric acid stones by with serum pH.
this type of drug? producing alkaline urine, but the urine pH would not be the best indicator of serum acidity or
alkalinity. Serum ammonia levels reflect liver functioning.

1.‐ Serum carbon dioxide


2.‐ Serum osmolarity
3.‐ Urine pH
4.‐ Serum ammonia

2829 A client is receiving a thiazide diuretic. Which of the Correct answer: 3 Thiazide diuretics can cause reabsorption of calcium, leading to hypercalcemia. The value of Specific knowledge of thiazide diuretics is needed. The question directs you to look for a
following laboratory findings would indicate that the 11 mEq/L is above the normal value of 8.5–10.5. Thiazides would cause loss of potassium and side effect, so an abnormal level would be expected. Eliminate options 1, 2, and 4, since
client is experiencing a side effect of the drug? sodium. The sodium level is low, but still within normal limits. The potassium is at a high they are within normal limits.
normal level. Magnesium is within normal limits.
1.‐ Potassium 5.0 mEq/L
2.‐ Sodium 137 mEq/L
3.‐ Calcium 11.5 mEq/L
4.‐ Magnesium 3.0 mEq/L

2830 When checking a client’s prostate‐specific antigen Correct answer: 1, 3, 4, PSA levels are elevated with enlargement, inflammation, and cancer of the prostate, or when The question calls for specific knowledge of the prostate gland and related disorders.
(PSA) level, the nurse considers that the serum level 5 the gland has been manipulated, as in a rectal examination. Levels are not elevated with other Eliminate option 2, since it is not connected to prostate function.
can be elevated in which of the following conditions? cancers or urinary tract infections.
Select all that apply.
1.‐ Prostate cancer
2.‐ Sigmoid carcinoma
3.‐ Benign prostatic hypertrophy
4.‐ Prostatitis
5.‐ Recent rectal examination

2831 The nurse is caring for a client receiving a heparin Correct answer: 1 Correct dosage calculation must be performed. First, calculate the units/mL contained in the Use knowledge of basic pharmacological math calculations to solve the problem.
infusion containing 25,000 units heparin in 250 mL 5% solution; then, calculate the desired amount using the standard formula:<BR /><BR />
dextrose in water. The activated partial
thromboplastin time (APTT) is 48 seconds. Standing
orders for the infusion indicate the infusion should be
increased by 100 units per hour if the APTT is less than
50 seconds. The nurse should increase the infusion by
_______ mL/hour.

2832 A client has a new order to receive vancomycin Correct answer: 2 Because vancomycin (Vancocin) can cause thrombophlebitis, this adverse effect is less likely Focus on the critical word systemic in the stem of the question. Use this word and
(Vancocin) to treat a systemic infection. The nurse to occur with central IV administration than with a peripheral IV administration (option 1). knowledge of safe administration of this drug to eliminate the incorrect options regarding
anticipates the order would indicate which of the Dilution of the drug lessens irritation to the vein. Vancomycin is not absorbed in the GI tract, best route.
following routes for optimal administration? so the oral route is used only to treat Clostridium difficile associated with antibiotic‐induced
pseudomembranous colitis (option 4). Intramuscular administration is contraindicated (option
3).
1.‐ Peripheral venous access
2.‐ Central venous access
3.‐ Intramuscular
4.‐ Oral
2833 Gentamicin (Garamycin) therapy is to be initiated. Correct answer: 3 Increased serum creatinine indicates renal dysfunction. Nephrotoxicity is a common adverse Note the critical words common adverse effect in the stem. Recall that aminoglycosides
Which of the following laboratory test results would effect of aminoglycosides such as gentamicin. The urine creatinine clearance would be often adversely affect the kidneys to narrow the options to 1 and 3. Note that option 3
indicate to the nurse that the client is manifesting a decreased in renal impairment (option 1). Coagulation disturbances and hypokalemia are not indicates renal impairment to select it over option 1.
common adverse effect? attributed to this class of antibiotic as a direct adverse reaction (options 2 and 4).

1.‐ Elevated urine creatinine clearance


2.‐ Increased prothrombin time (PT)
3.‐ Increased serum creatinine
4.‐ Hypokalemia

2834 The nurse teaches the client who is taking isoniazid Correct answer: 1 Peripheral neuritis is the most common side effect of isoniazid (INH). Adding vitamin The core issue of the question is knowledge of what nutrient will reduce adverse drug
(INH) and experiencing paresthesia as a common side B&lt;sub&gt;6&lt;/sub&gt; (pyridoxine) to the client s intake is the therapy to correct this side effects of INH. Recall that vitamin B&lt;sub&gt;6&lt;/sub&gt; will assist in this action, and
effect to include what food in the diet? effect. The diet may be supplemented with vitamin B&lt;sub&gt;6&lt;/sub&gt;. Foods highest then choose the food that is highest in this vitamin.
in vitamin B&lt;sub&gt;6&lt;/sub&gt; include beef, chicken, and pork, including beef liver and
chicken liver. Other foods listed are not high in pyridoxine. Foods other than meats that could
be included are raw avocados, baked potato with skin, raw banana, figs, and soybeans.

1.‐ Liver
2.‐ Peanuts
3.‐ Whole milk
4.‐ Raw apples

2835 A client taking digoxin (Lanoxin) along with Correct answer: 1 Amphotericin‐induced hypokalemia might potentiate toxicity of digoxin because hypokalemia The core issue of the question is knowledge of drug interaction of digoxin and
amphotericin B (Fungizone) could experience digitalis is a primary cause for digitalis toxicity. Amphotericin B is available for only intravenous and amphoteracin B. Use specific nursing knowledge and the process of elimination to make a
toxicity due to which of the following? topical routes. Antifungal agents and cardioglycosides do not compete for the same receptor selection.
sites.
1.‐ Hypokalemia
2.‐ Antifungal attaching to receptors before digoxin
3.‐ Increased plasma concentration of amphotericin B
4.‐ Increased gastrointestinal absorption of digoxin and amphotericin B

2836 A client is receiving levofloxacin (Levaquin) in addition Correct answer: 4 Eradication of the intestinal flora can occur during antibiotic therapy. Absorption of vitamin K The core issue of the question is knowledge of drug interaction between levofloxacin and
to an oral anticoagulant. Which of the following from the intestines can be interrupted, and prolonged bleeding can result due to inadequate oral anticoagulants. Recall that vitamin K reverses bleeding to help identify the drug that
treatments would the nurse anticipate administering if serum level of prothrombin (hypothrombinemia). Vitamin K is essential to the synthesis of reverses the effect of oral anticoagulant drugs.
the client experiences an adverse drug effect as a prothrombin (factor II) by the liver. Appropriate therapy in this case is to administer
result of this combination? phytonadione or menadiol sodium diphosphate (Synkayvite). Since intestinal absorption might
not be optimal, the parenteral route is preferred. In addition, the nurse must assess for and
protect against increased bleeding.

1.‐ Albumin
2.‐ Platelets
3.‐ Protamine sulfate
4.‐ Phytonadione (vitamin K)

2837 The nurse assesses a client taking doxycycline Correct answer: 3 ALT is specific for diagnosing and monitoring liver disease or impairment. Differential The core issue of the question is the laboratory test that will help evaluate the presence of
(Vibramycin) as being jaundiced and lethargic. What diagnosis of etiology of jaundice between hepatic dysfunction and hemolysis of red blood cells jaundice as an adverse effect of doxycycline. Use specific nursing knowledge and the
laboratory study would be most specific for the nurse is indicated by the bilirubin. AST can help to diagnose or monitor heart disease or disease of process of elimination to make a selection.
to assess? the liver.
1.‐ Bilirubin
2.‐ Alkaline phosphatase (ALP)
3.‐ Alanine aminotransferase (ALT or SGPT)
4.‐ Aspartate aminotransferase (AST or SGOT)
2838 A client taking ampicillin (Omnipen) develops a Correct answer: 3 A minor rash is the most common side effect of the penicillins, and might be relatively The core issue of the question is the significance of a rash that develops in a client taking
macular rash on the chest. The nurse should draw insignificant. Its presence does not signify an allergic reaction, and does not prohibit future ampicillin. Use specific nursing knowledge and the process of elimination to make a
which of the following conclusions about this administration of penicillin. However, the nurse closely monitors for further hypersensitivity selection. Recall that not all drug rashes indicate hypersensitivity to aid in choosing the
assessment finding? reaction because other clinical manifestations could develop, such as fever, urticaria, chills, correct option.
erythema, Stevens‐Johnson syndrome, respiratory distress, and anaphylaxis. If itching occurs,
an antihistamine such as diphenhydramine (Benadryl) may be prescribed. All available
antimicrobials are capable of stimulating an exaggerated immune response, but not all clients
experience allergy with antibiotic therapy. Stevens‐Johnson syndrome is a more serious
aberration of the skin associated with antimicrobial adverse reactions; it resembles a second‐
degree burn in that necrolysis separates the epidermis from the dermis, causing blisters.

1.‐ This reaction is Stevens‐Johnson syndrome.


2.‐ A minor rash usually precipitates the development of more severe reactions.
3.‐ A minor rash requires notification of the prescriber, but might be well tolerated, and might fade with continued treatment.
4.‐ Hypersensitivity reactions requiring discontinuation of the antibiotic occur to some extent with all clients taking a penicillin agent.

2839 The nurse teaches the client who is started on Correct answer: 1 Improvement in clinical manifestations of the infection should be noted within 48–72 hours. The core issues of the question are knowledge of unsatisfactory progress after beginning
erythromycin (Erythrocin) as treatment for pneumonia Otherwise, compliance with prescribed drug therapy should be assessed, and adjustment of erythromycin and indicators that need to be reported to the prescriber. Use specific nursing
to contact the health care provider for which of the drug, dose, and/or administration frequency might be needed. Anorexia, nausea, and knowledge and the process of elimination to make a selection.
following reasons? fluctuating febrile state can be common sequelae in systemic infections (options 3 and 4). The
client s ability to take in fluids can temporarily sustain his nutritional status for a few days,
particularly if dietary supplements are also included, which is appropriate for client education
(option 2).
1.‐ Improvement of fever, cough, or respiratory effort is not observed in 48–72 hours.
2.‐ Client is taking fluids orally, but continues to refuse to eat after 24 hours.
3.‐ Client develops anorexia and nausea within 24 hours.
4.‐ Fever fluctuates.

2840 A child with otitis media is taking trimethoprim‐ Correct answer: 4 The volume of a household teaspoon can vary by 2–10 mL, so a calibrated device is necessary The core issue of the question is the proper method of administration of trimethoprim‐
sulfamethoxazole (TMP‐SMZ) as a suspension. The for accurate dosing. The suspension is to be shaken to disperse the particles just prior to sulfamethoxazole to a child. Use specific nursing knowledge and the process of elimination
nurse provides what instruction to the mother? measurement. It is recommended that a glass of water be given with the medication, and that to make a selection.
adequate urinary output be maintained (option 1). The medication is stable at room
temperature, but the taste might be more palatable if cold (option 3). Food does not interfere
with absorption of the medication, and could help to minimize gastrointestinal side effects
(option 2).
1.‐ Do not allow the child to drink water immediately after taking the medication.
2.‐ The medication is to be taken on an empty stomach.
3.‐ The medication must be kept refrigerated.
4.‐ Use a calibrated measuring device.

2841 A client receiving an anti‐infective drug begins to Correct answer: 1 Epinephrine is the primary drug used when bronchoconstriction causes inadequate The core issue of the question is knowledge of drugs that are used to treat hypersensitivity
wheeze. The nurse anticipates initial administration of respiratory exchange, as in anaphylactic shock. Marked improvement in respiration occurs or anaphylactic reactions. Use specific nursing knowledge and the process of elimination to
what drug? within a few minutes after subcutaneous administration of 0.1–0.5 mL of 1:1000 strength make a selection.
epinephrine. Corticosteroids may be given to minimize the inflammation and edema, but are
not the initial agent given (option 2). Atropine might minimize secretions, but would not be
given unless vagal‐induced bradycardia or asystole occurred; then atropine could be given as
an IV bolus rapidly before, during, or after cardiopulmonary arrest (option 3). Dopamine HCL, a
catecholamine (as is epinephrine), may be given to increase blood pressure if shock develops
(option 4).

1.‐ Epinephrine HCL (Adrenalin Chloride)


2.‐ Methylprednisolone (Solu‐Medrol)
3.‐ Atropine sulfate (Atropine)
4.‐ Dopamine HCL (Intropin)

2842 The nurse would teach a client that alcohol taken in Correct answer: 1 A disulfiram‐like effect is associated with certain drugs, including metronidazole (Flagyl). The core issue of the question is knowledge of which drugs can cause a disulfiram‐like
conjunction with which of the following ordered Onset is usually within 15–30 minutes of ingestion of alcohol, but can occur up to 72 hours reaction when used with alcohol. Use specific nursing knowledge and the process of
medications can cause a reaction causing flushing, after Flagyl has been discontinued. The reaction lasts approximately 20–30 minutes but can elimination to make a selection.
dizziness, pounding headache, sweating, abdominal remain up to 24 hours.
cramps, nausea, irritability, and low blood pressure?

1.‐ Cefoxitan (Mefoxin)


2.‐ Metronidazole (Flagyl)
3.‐ Clindamycin (Cleocin)
4.‐ Ampicillin (Omnipen)

2843 The nurse assesses the client receiving cefotazime Correct answer: 2 Opportunistic infections or superinfections are manifested by these signs and symptoms. The core issue of the question is knowledge of adverse drug effects of cefotazime and
(Claforan), and notes three diarrhea stools in the past Common ones are vaginal and GI tract infections, including candidiasis and diarrhea. They their significance. Use specific nursing knowledge and the process of elimination to make a
24 hours, rectal itching, glossitis, and fever. The nurse often result from broad‐spectrum antibiotic use that destroys bacteria in the normal flora, selection.
concludes these adverse effects probably indicate allowing the resistant pathogens to proliferate. Early recognition and intervention with
which of the following? administration of sensitive anti‐infectives is important in controlling discomfort and the
severity of the reaction. The other options represent incorrect conclusions about the data
presented.
1.‐ Leukocytosis
2.‐ Opportunistic infection
3.‐ Bone marrow depression
4.‐ Drug failure against original infective organism

2844 What teaching or intervention is appropriate for a Correct answer: 2 Yogurt and buttermilk products can decrease the diarrhea as well as add protein to the diet The core issue of the question is knowledge of client teaching points related to antibiotic
client taking an antibiotic that causes diarrhea to provide albumin for drug binding. Blood or mucus in the stool with increased number of therapy that has diarrhea as a side effect. Use specific nursing knowledge and the process
secondary to elimination of normal intestinal flora? stools indicates the possibility of pseudomembranous colitis that should be reported to the of elimination to make a selection.
health care provider. Antacids would interfere with the effectiveness of the antibiotic (option
4). The route of administration of antibiotics is not the cause of destruction of normal flora
(option 3). Clients are not usually taught to test their stool for occult blood (option 1).

1.‐ Test stool for occult blood.


2.‐ Include yogurt or buttermilk products in the diet.
3.‐ Arrange for IV administration instead of oral route for the antibiotic.
4.‐ Take antacids with oral antibiotic to slow absorption of antibiotic, and to reduce the severity of the diarrhea.

2845 A disulfiram‐like reaction occurs in a client taking Correct answer: 3 Disulfiram‐ or antabuse‐like reactions can occur when cephalosporins are taken with The core issue of the question is knowledge of the causes of disulfiram‐like drug reactions.
cefoperazone sodium (Cefobid). The nurse suspects ingestion of alcohol during and up to 72 hours after discontinuation of the cephalosporin. Use specific nursing knowledge and the process of elimination to make a selection.
this reaction is a drug interaction resulting from the Caffeine and the other medications listed would not cause this reaction.
client’s ingestion of which of the following substances
within the last few hours?

1.‐ Caffeine in tea or coffee


2.‐ Sulfamethoxazole (Gantonol) for a chronic urinary tract infection
3.‐ Alcohol‐containing cough syrup
4.‐ Ampicillin (Omnipen), which has a cross‐sensitivity to cephalosporins
2846 The nurse evaluates for an adverse reaction to Correct answer: 2 Vestibular ototoxicity as well as cochlear otoxicity can occur with administration of an The core issue of the question is knowledge of assessment techniques that will help
tobramycin sulfate (Tobrex) by conducting what aminoglycoside such as tobramycin. A positive Romberg s test indicates vertigo or loss of determine whether ototoxicity is occurring in a client taking tobramycin. Use specific
assessment? balance, and can suggest a vestibular problem. Babinski s reflex present in the adult reflects a nursing knowledge and the process of elimination to make a selection.
possible lesion in the corticospinal tract (option 4). Chvostek s sign is seen in tetany and
hypocalcemia (option 3). One method of assessing peripheral circulation is to check the
capillary refill (option 1).
1.‐ Capillary refill
2.‐ Romberg’s test
3.‐ Chvostek’s sign
4.‐ Babinski s reflex

2847 What does the nurse teach an adult premenopausal Correct answer: 1 Griseofulvin does not cause increased bleeding unless the client is also on anticoagulant The core issue of the question is knowledge of key teaching points regarding systemic
client receiving griseofulvin microsize (Grisfulvin V) for therapy (option 2). The agent also has no known effect on blood pressure, and known griseofulvin. Use specific nursing knowledge and the process of elimination to make a
a systemic antifungal condition? interaction with calcium intake (options 3 and 4). However, griseofulvin can interfere with the selection.
effectiveness of estrogen‐containing oral contraceptives (option 1).

1.‐ If taking oral contraceptives, use an alternative form of contraception during and for one month after use of griseofulvin.
2.‐ Keep a record of the number of absorbent products used daily, to monitor for increased menstrual flow while taking griseofulvin.
3.‐ Check blood pressure daily during treatment if taking oral contraceptive and griseofulvin, as both can increase blood pressure.
4.‐ Avoid taking calcium supplements concurrently with griseofulvin.

2848 Because of the mechanisms of action of tetracycline, Correct answer: 1 Bacteriostatic agents inhibit or retard bacterial growth and replication, but they do not kill The core issue of the question is the mechanism of action of tetracycline, and how it leads
the client needs to have which of the following in the entire bacteria. These agents depend on the host s defense mechanisms to complete to eradication of infection. Use specific nursing knowledge and the process of elimination
order for the drug to be effective? elimination of the bacteria. Bactericidal agents actually kill and lyse the bacteria. Tetracyclines to make a selection.
are bacteriostatic. Supplemental vitamin B&lt;sub&gt;6&lt;/sub&gt; is indicated with isoniazid
(INH) administration (option 3). Iron, as well as antacids, laxatives, food, and dairy products,
should be separated one hour before or two hours after administration of a tetracycline
(option 2). These substances interfere with the absorption of tetracyclines. Option 4 is
unnecessary.
1.‐ A competent client immune system
2.‐ Concurrent administration of iron
3.‐ Supplemental pyridoxine HCL (vitamin B6)
4.‐ Weekly evaluation of the complete blood count

2849 The nurse assesses for which lab result when a client Correct answer: 3 The prothrombin time (PT) and the international normalization ratio (INR) values are standard The core issue of the question is the expected change in laboratory results for a client
is receiving long‐term oral anticoagulation therapy and tests to monitor warfarin (Coumadin) levels. The beta‐lactam antibiotics can cause increased taking a beta‐lactam penicillin and an oral anticoagulant. Use specific nursing knowledge
is also taking a beta‐lactam penicillin? PT and INR. The bleeding time (option 1) evaluates the integrity of the vascular and platelet and the process of elimination to make a selection.
factors associated with stagnated blood. Thrombin time (option 2) evaluates the fibrinogen‐to‐
fibrin conversion factor that can be used to gauge heparin effectiveness. However, the APPT
(option 4) is currently used most often in regulating heparin therapy.

1.‐ Decreased bleeding time


2.‐ Increased thrombin time (TT)
3.‐ Increased prothrombin time (PT)
4.‐ Increased activated partial thromboplastin time (APPT)

2850 During a routine screening, a client has a positive Correct answer: 4 The PPD injection stimulates a local inflammatory response at the injection site in the client The core issue of the question is the significance of PPD test results. Use specific nursing
response to intradermal injection of purified protein who has been exposed to the tubercle bacillus in the past. The client develops a cellular knowledge and the process of elimination to make a selection.
derivative (PPD or Mantoux test). The nurse draws response to tubercle bacillus at 3–10 weeks after infection. A positive PPD result does not
which of the following conclusions about this result? indicate that the client currently has active tuberculosis, or is in an infectious state (options 1
and 2). Follow‐up sputum tests for tubercle bacillus and/or chest films are done to clarify
current status.
1.‐ The client is currently infectious.
2.‐ The client has active tuberculosis.
3.‐ The client has been exposed to the tubercle bacillus within the past 2weeks.
4.‐ The client has been infected with tuberculosis, and has developed a cellular (T cell) response to the tubercle bacillus.

2851 Appropriate teaching for a young adult female related Correct answer: 4 Antibiotics, especially aminopenicillins such as ampicillin, can decrease the effectiveness of The core issue of the question is client teaching that is needed for a client beginning drug
to a new prescription for ampicillin (Omnipen) orally oral contraceptives. The other clinical manifestations are not related to penicillin therapy. therapy with ampicillin. Use specific nursing knowledge and the process of elimination to
would include which of the following? make a selection.

1.‐ Observe for easy bruising.


2.‐ Observe for clinical extrapyramidal tract manifestations.
3.‐ Change positions slowly to avoid orthostatic hypotension.
4.‐ If applicable, use an alternative to oral contraceptives during and for one month after therapy.

2852 A female client is taking sargramostim (Leukine) Correct answer: 3 It is the responsibility of the nurse to address the client’s body image related to alopecia. The core issue of the question is knowledge of how adverse drug effects of sargramostim
following a bone marrow transplant. During an Option 1 does not apply. Option 2 is not a nursing diagnosis. Option 4 is a secondary diagnosis can affect a client, necessitating formulation of a nursing diagnosis. Use specific nursing
assessment, the client voices concern about her hair that could apply, but option 3 is more relevant to the side effect of the medication. knowledge and the process of elimination to make a selection.
falling out. Based on this assessment, which of the
following would be the priority nursing diagnosis?

1.‐ Impaired Skin Integrity


2.‐ Alopecia
3.‐ Disturbed Body Image
4.‐ Anxiety

2853 A client receiving aldesleukin (Proleukin) begins to Correct answer: 1 Potential flulike symptoms can occur with aldesleukin (Proleukin). For this reason, the nurse The core issue of the question is knowledge of how adverse drug effects of aldesleukin can
complain of a fever and pain. Which of the following is must provide for adequate fluid‐and‐electrolyte balance to prevent deficient fluid volume. affect a client, necessitating formulation of a nursing diagnosis. Use specific nursing
the priority nursing diagnostic statement for this Options 2 and 4 have lesser priority because they are not side effects (although they are knowledge and the process of elimination to make a selection.
client? usually general priority items), and option 3 is the opposite of the actual problem.

1.‐ Risk for Deficient Fluid Volume related to flulike symptoms


2.‐ Ineffective Airway Clearance related to fever
3.‐ Excess Fluid Volume related to flulike symptoms
4.‐ Ineffective Airway Clearance related to increased pulmonary secretions

2854 Clients receiving oprelvekin (Neumega) should be Correct answer: 2 Oprelvekin (Neumega) can cause cardiopulmonary insufficiency with irregular heart rate and The core issue of the question is knowledge of adverse drug effects of oprelvekin. Use
assessed frequently for signs and symptoms of which fluid retention. Thus, it is a nursing priority to assess the client frequently for signs and specific nursing knowledge and the process of elimination to make a selection.
of the following? symptoms of congestive heart failure. The other options do not address this priority concern.

1.‐ Dehydration
2.‐ Congestive heart failure (CHF)
3.‐ Anxiety
4.‐ Hyperuricemia

2855 Based on the prescribed therapy of mycophenolate Correct answer: 3 Mycophenalate (CellCept) is administered orally 72 hours after transplant. The time frames The core issue of the question is knowledge of time frames to begin drug therapy with
(CellCept), the nurse can expect the renal transplant listed in each of the other options are incorrect. mycophenolate following transplant. Use specific nursing knowledge and the process of
client to receive the first dose at which of the following elimination to make a selection.
times?
1.‐ In the postanesthesia recovery area
2.‐ Within one hour of admission to intensive care
3.‐ Seventy‐two hours following the transplant
4.‐ One week following the transplant
2856 A 28‐year‐old male client is admitted to the Correct answer: 2 It is recommended that every client have a tetanus vaccine every 10 years to prevent The critical words in the question are laceration and Emergency Department. Recall that
Emergency Department with a three‐inch laceration infection caused by tetanus. The primary opportunity for this assessment is following a the skin is the first line of defense against infection to choose option 2.
over his left eye. The nurse should assess which of the laceration. Delayed wound healing is a possibility with corticosteroid therapy, but assessment
following priority items related to the risk of infection of tetanus immunization status takes priority. Temperature and blood pressure measurement
before beginning drug therapy to prevent infection? (options 1 and 3) do not address the risk of infection caused by trauma while the client is in the
Emergency Department.
1.‐ The client’s temperature
2.‐ The date of the client’s last tetanus vaccine
3.‐ If the client’s blood pressure is decreased
4.‐ Whether the client is taking corticosteroid medication

2857 A pit bull has bitten a 28‐year‐old woman. Upon Correct answer: 3 Antirabies serum equine 55 U/Kg IM can be applied to the animal bite wound. The medication The core issue of the question is knowledge of how to administer antirabies serum. Use
admission to the Emergency Department, the nurse does not need to be administered parenterally (options 1, 2, and 4). specific nursing knowledge and the process of elimination to make a selection.
will administer the prescribed antirabies serum
(equine) into which of the following locations?

1.‐ Intravenously
2.‐ Intramuscularly into the right gluteal muscle
3.‐ Into the animal bite
4.‐ Z‐track into the anterolateral thigh

2858 In conducting client teaching about Beta 1b Correct answer: 4 Beta 1b (Betaseron) reduces the severity of acute exacerbations of multiple sclerosis. The The core issue of the question is knowledge of goals of drug therapy with interferon
(Betaseron), the nurse would explain that the goal of drug decreases the demyelination in brain tissue. The other responses do not accurately reflect (Betaseron). Use specific nursing knowledge and the process of elimination to make a
the administration is which of the following? the action of this medication. selection.

1.‐ Cure the client of multiple sclerosis.


2.‐ Prevent signs and symptoms of anaphylaxis.
3.‐ Destroy nerve tissue that is laden with plaque.
4.‐ Decrease the demyelination in the brain tissue.

2859 To decrease renal insufficiency side effects in clients Correct answer: 3 Adequate fluid intake greater than 2,000–3,000 mL per day allows for the kidneys to flush The core issue of the question is knowledge of measures to prevent the development of
receiving cyclophosphamide (Sandimmune), the nurse renal toxins, and prevents renal insufficiency. Option 1 is a general measure to prevent renal side effects with use of cyclophosphamide. Use specific nursing knowledge and the
should instruct the client to do which of the following? constipation. Option 2 would be a monitoring function, but would not prevent renal process of elimination to make a selection.
insufficiency. The nurse would not instruct the client to take additional medication that is not
specifically part of the plan of care (option 4).
1.‐ Consume a diet high in fiber.
2.‐ Have his creatinine level assessed weekly.
3.‐ Drink 3,000 mL of fluid per day.
4.‐ Administer hydrochlorothiazide (HCTZ) with cyclophosphamide.

2860 To assess a client s baseline prior to the Correct answer: 1 The most significant laboratory test to utilize prior to medication therapy with azathioprine is The core issue of the question is knowledge of adverse drug effects of azathioprine and of
administration of azathioprine (Imuran), the nurse creatinine level, because renal and hepatic function should be assessed. Option 2 is irrelevant, which laboratory test to use as a baseline measure. Use specific nursing knowledge and the
should put highest priority on evaluating which of the while options 3 and 4 evaluate blood clotting and components of the blood, respectively. process of elimination to make a selection.
following laboratory test results? Other risks of azathioprine would be increased white cell count (infection) and decreased
platelet count.
1.‐ Creatinine
2.‐ Uric acid
3.‐ PT and PTT
4.‐ Red blood cell count
2861 Azathioprine (Imuran) and allopurinol (Zyloprim) are Correct answer: 2 Azathioprine (Imuran) is administered to treat multiple sclerosis, and allopurinol (Zyloprim) is The core issue of the question is knowledge of drug interactive effects of azathioprine and
administered to a client diagnosed with multiple administered to treat symptoms of gout. When these two medications are administered allopurinol. Use specific nursing knowledge and the process of elimination to make a
sclerosis and gout. It is important for the nurse to together, the dose of azathioprine should be reduced. The uric acid level and client symptoms selection.
assess the results of which of the following laboratory should be assessed to determine the control of gout.
tests in this client?
1.‐ Creatinine
2.‐ Uric acid
3.‐ Blood glucose
4.‐ Blood urea nitrogen (BUN)

2862 A client is scheduled for diagnostic testing for Correct answer: 2 Edrophonium (Tensilon) is used for diagnostic purposes. Clients who receive an injection of The core issue of the question is knowledge of medications used for diagnosis of
myasthenia gravis. The nurse would prepare which of edrophonium and exhibit a temporary relief of symptoms are diagnosed with myasthenia myasthenia gravis. Use specific nursing knowledge and the process of elimination to make a
the following medications necessary for this testing? gravis, which is characterized by a decrease in the concentration of acetylcholine in the selection.
neuromuscular junction. The medications listed in the other options are not used to diagnose
myasthenia gravis.
1.‐ Ambenonium (Mytelase)
2.‐ Edrophonium (Tensilon)
3.‐ Neostigmine (Prostigmine)
4.‐ Physostigmine (Eserine)

2863 Which of the following points should be included in a Correct answer: 1 Medications used to treat symptoms of multiple sclerosis have been noted to increase The core issue of the question is knowledge of essential teaching points for a client being
teaching plan for a client receiving medications to pulmonary edema, leading to chest pain and shortness of breath. Option 2 could increase risk treated with drug therapy for multiple sclerosis. Use specific nursing knowledge and the
treat multiple sclerosis? of urinary tract infection. Option 3 is useful to avoid infection, but does not specifically relate process of elimination to make a selection.
to medication teaching. Option 4 could increase fatigue, if done to excess, and lead to
exacerbation of symptoms.
1.‐ The signs and symptoms of pulmonary edema
2.‐ The restriction of fluids
3.‐ The requirement to remain restricted from crowds
4.‐ The requirement to exercise to enhance muscle strength

2864 A client has been exposed to hepatitis A. Which of the Correct answer: 3 Immune serum globulin should not be administered to clients with a history of coagulation The core issue of the question is knowledge of safe administration of serum immune
following client factors would be an indication for disorders. The other options do not represent contraindications to administration of this globulin. Use specific nursing knowledge and the process of elimination to make a selection.
withholding administration of immune serum globulin medication.
to the client?
1.‐ The client has received a hepatitis B vaccine.
2.‐ The client has recently fallen and suffered a hip fracture.
3.‐ The client has a history of a coagulation disorder.
4.‐ The client is scheduled for foreign travel.

2865 A client with Parkinson s disease is admitted to the Correct answer: 4 Physostigmine (Eserine) is an anticholinesterase agent that crosses the blood–brain barrier. It The core issue of the question is knowledge of drug therapy to reverse excessive effects of
Emergency Department with lethargy, hypotension, is used as an agent to correct anticholinergic poisoning. The other medications listed do not medications used to treat Parkinson’s disease. Use specific nursing knowledge and the
and weakened gait. The nurse should be prepared to have this effect. process of elimination to make a selection.
administer which of the following medications?

1.‐ Carbidopa (Lodosyn)


2.‐ Levodopa (Dopar)
3.‐ Atropine (generic)
4.‐ Physostigmine (Eserine)

2866 To prevent toxicity during IV administration of Correct answer: 4 The intravenous administration of physostigmine (Eserine) should be no faster than 1 mg per The core issue of the question is knowledge of safe administration technique for
physostigmine (Eserine), the nurse should administer minute, to prevent toxic adverse reactions. The other options do not relate to toxicity of physostigmine. Use specific nursing knowledge and the process of elimination to make a
the medication: physostigmine. selection.
1.‐ Retrograde.
2.‐ Through a central line.
3.‐ Concurrently with Parlodel.
4.‐ No faster than 1 mg per minute.

2867 The client taking isoniazid (INH) reports paresthesia of Correct answer: 4 Administration of vitamin B&lt;sub&gt;6&lt;/sub&gt; is recommended during therapy with In order to answer this question, recall nursing interventions associated with the
the extremities. The nurse initially assesses the client isoniazid (INH), to reduce the incidence of peripheral neuritis, which could be associated with administration of this medication. If this was difficult, review the nursing interventions in
for which of the following? isoniazid. Monitoring motor reflexes would not be indicated (option 1). Paresthesia is not the administration of the medication.
usually a clinical manifestation of hypercalcemia (option 2). Antacids interfere with absorption
of INH when taken within 1–2 hours of the INH, but would not cause the symptoms reported
by the client (option 3).

1.‐ Hyperactive motor reflex responses


2.‐ Other clinical manifestations of hypercalcemia
3.‐ Concurrent self‐administration of aluminum antacids
4.‐ Compliance with taking pyridoxine (vitamin B6) supplement

2868 Rifampin (Rifadin) is being initiated prophylactically Correct answer: 3 Rifampin causes an orange‐red discoloration of body fluids, including urine. The client needs In order to answer this question correctly, recall the side effects and client responses to
for a client who lives with a family member who has to be aware of this. The drug is being ordered prophylactically to prevent the development of listed medications. If this was difficult, review side effects and client responses to the
Haemophilus influenzae meningitis. What client meningitis, not to treat it (option 1). Adverse effects are generally minor with rifampin (option medications.
teaching would be most appropriate? 2). Because the drug is metabolized by the liver, regular liver function tests should be
monitored (option 4). Rifampin should be used with caution in the presence of elevated liver
enzymes or hepatic dysfunction.
1.‐ Explain that rifampin is being prescribed to treat meningitis.
2.‐ Adverse effects might be severe, such as convulsions and coma.
3.‐ Protect undergarments, because with rifampin (Rifadin), urine will become orange‐red, and will stain.
4.‐ The client will need to keep follow‐up visits with her health care provider, but it will not be necessary to continue blood test monitoring.

2869 A client with benign prostatic hyperplasia (BPH) is Correct answer: 1 Amantadine (Symmetrel) can cause anticholinergic effects, two of which are bladder In order to answer this question correctly, recall the side effects and client responses to
receiving amantadine (Symmetrel) for influenza A. The relaxation and detrusor muscle contraction. Urinary retention could become more of a listed medications. If this was difficult, review side effects and client responses to the
nurse includes in the care plan to monitor the client problem for a client with BPH on this medication. Hypermotility of the bowel and increased medications.
for which of the following side effects? lacrimation are cholinergic effects, not anticholinergic effects (options 2 and 3). Amantadine is
not particularly nephrotoxic (option 4).
1.‐ Increased risk for urinary retention
2.‐ Hypermotility of bowel
3.‐ Increased lacrimation
4.‐ Nephrotoxicity

2870 A client with herpes zoster infection (shingles) has Correct answer: 4 Agents for herpes virus as herpes zoster can be nephritic. Important interventions include In order to select the correct answer to this question, recall that the medication is toxic to
started therapy with acyclovir (Zovirax). The nurse monitoring renal function and ensuring good hydration to decrease toxic effects. This drug is the kidneys. Knowing this, be directed to option 4. The other answers do not apply to
performs what important intervention during the not reported to be particularly hepatotoxic (option 1). Sexual intercourse is to be avoided if nephrotoxicity.
course of this treatment? the client is being treated with the acylovir for genital herpes (option 2). Insomnia is not a side
effect of acyclovir (option 3).
1.‐ Monitors for jaundice and elevated liver enzymes.
2.‐ Teaches the client to avoid sexual intercourse during therapy.
3.‐ Administers the dose early in the day, as it could cause insomnia.
4.‐ Encourages fluid intake of 2,500–3,000 mL daily, since it is not contraindicated by other client conditions.
2871 The nurse determines that the client understands an Correct answer: 1 A full course of antibiotic therapy must be taken in order to decrease the risk of resistance to Note that the question stem asks for selection of the answer that indicates that the client
important principle of self‐administration of an oral the antibiotic, or reoccurrence of the infection. Missed doses should be taken as soon as they understands correct administration of medication. Reviewing basic information about
antibiotic when the client makes which of the are remembered, but the dose should not be doubled by taking two doses at the same time medications will lead to option 1 as the correct answer.
following statements? (option 2). Antibiotic doses are to be taken at regular intervals spaced throughout the 24
hours, without interrupting sleep when possible, in order to maintain effective therapeutic
blood level of the antibiotic (option 3). Chewable tablets must be crushed or chewed, or the
drug might not absorb adequately (option 4).

1.‐ "I will continue to take the antibiotic as it is ordered, even though I no longer have a cough with yellow sputum."
2.‐ "When I missed a dose of my antibiotic this morning, I made up for it by taking two doses when it was time to take the next dose."
3.‐ "I am careful to take the antibiotic every day at breakfast, lunch, and dinner."
4.‐ "Even though the doctor prescribed amoxicillin (Amoxil) chewable tablet, I have no problem swallowing it whole."

2872 A client who has been on anti‐infective therapy for 10 Correct answer: 4 More than 4–6 watery stools per day and stools with blood are clinical manifestations of The key information in the question stem is that the client has been on anti‐infective
days has developed diarrhea, with 10 watery stools a pseudomembranous colitis. C. difficile is the causative microorganism for this superinfection. therapy for 10 days, and has now developed diarrhea with watery stools. The length of time
day. The nurse should anticipate an order for which of The client is at risk for developing metabolic acidosis due to increased loss of bowel contents should direct selection of the first intervention before any treatment can begin. This would
the following? with loss of base (option 1). Antiperistaltic agents can promote retention of toxins, and should lead to first collecting the stool specimen, as is indicated in option 4.
not be given (option 2). Antidiarrheal agents may be given for mild diarrhea, but not when
toxins need to be eliminated (option 3).

1.‐ Monitor for clinical manifestations of metabolic alkalosis.


2.‐ Administer an anti‐peristaltic agent, such as dicyclomine HCl (Bentyl).
3.‐ Administer an antidiarrheal agent, such as kaolin and pectin (Kaopectolin).
4.‐ Collect a stool specimen for cytotoxin assay to detect <i>Clostridium difficile</i>.

2873 The client with pneumonia is being treated with Correct answer: 4 Specific indicators of improvement, such as the resolution of pulmonary infiltrates, improved Picking up on the key system of respiratory in the question stem should assist in selecting
amoxicillin (Amoxil). The nurse monitors for breath sounds, and normalization of pulse oximetry, are important outcomes to monitor in the correct answer. Looking for an answer that addresses respiration will point to option 4.
therapeutic effectiveness by noting which of the pneumonia. Systemic signs including fever, malaise, and leukocytosis are expected to Eliminate option 3 since there is no indication of hypotension in the question stem.
following? demonstrate improvement within 48–72 hours of antibiotic therapy (option 1). Option 2 does
not indicate therapeutic effectiveness, and option 3 is unrelated to the question.

1.‐ Normalization of fever beginning 96 hours after therapy starts


2.‐ No clinical manifestations of hypersensitivity
3.‐ Resolution of orthostatic hypotension
4.‐ Pulse oximetry of 98%

2874 A client receiving penicillin (PCN) for several days Correct answer: 1 The penicillins are structured with a sodium or potassium salt. When a high‐sodium‐content In reviewing the question stem, the client is experiencing cardiovascular changes. Knowing
complains of weakness, numbness, tingling in the penicillin is administered, serum sodium can be elevated, which often results in hypokalemia. that low potassium will lead to cardiovascular changes will lead to the answer of
extremities, and nausea. The nurse palpates a weak This client is demonstrating clinical manifestations of hypokalemia. With the elevated sodium, hypokalemia.
pulse, and auscultates an irregular heart rate and the accompanying anion would most likely be chloride, resulting in hyperchloremia, and not
decreased bowel sounds. The nurse further monitors hypochloremia.
the client for specific signs of which of the following
imbalances?
1.‐ Hypokalemia
2.‐ Hypochloremia
3.‐ Hypercalcemia
4.‐ Hypophosphatemia
2875 The prescriber has ordered cefdinir (Omnicef), a third‐ Correct answer: 4 A cross‐allergenicity with penicillin might exist. Cephalosporins cannot be assumed to be an Using the process of elimination should eliminate option 1, as this is a normal result.
generation cephalosporin, for a client with a absolutely safe alternative in penicillin‐allergic clients. If the cephalosporin is administered to Option 2 is an expected result with the disease process. Option 3 can be eliminated, since it
staphylococcal infection. The nurse collaborates with this client, the nurse needs to administer it cautiously, observing for manifestations of does not apply to the question. This should lead to the only correct option.
the prescriber about which of the following data hypersensitivity, especially respiratory difficulty. Emergency equipment should be readily
related to the client? accessible. BUN is within normal limits. It is expected that the granulocytosis would occur in
response to a bacterial infection. To wait several hours for the results of the C &amp; S could
compromise the client's response to the treatment. If the C &amp; S findings reveal that the
bacteria are resistant to the prescribed antibiotic, the drug can be changed to one that will be
effective against the organism.

1.‐ BUN 14 mg/dL


2.‐ Elevated granulocyte count
3.‐ Culture and sensitivity (C & S) results not yet available.
4.‐ History of type I hypersensitivity to penicillin

2876 A client's white blood cell count differential shows a Correct answer: 1 A "shift to the left" refers to an increase in neutrophils, and immature neutrophils called Having knowledge of the term “shift to the left” will lead to the only correct answer. The
"shift to the left." The nurse recognizes that the client bands or stab cells. Production of these white blood cells is stimulated by an acute bacterial shift refers to an increase in neutrophils, with an increase indicating a bacterial infection.
needs to be assessed for what type of infection? infection. Lymphocytes, T cells, and B cells are increased primarily in viral infections. This answer is listed in option 1.
Monocytes also fight bacterial infection by phagocytic action. Eosinophils and basophils are
elevated in allergic reaction.
1.‐ Bacterial
2.‐ Acute viral
3.‐ Parasitic
4.‐ Retroviral

2877 Prior to the administration of filgrastim (Neupogen), Correct answer: 4 Filgrastim (Neupogen) is contraindicated with a hypersensitivity to E. coli products. It is Take into account the word hypersensitivity, and be alerted that the question is a safety
the client reports a history of hypersensitivity to E. coli important for the nurse to assess for any contraindications prior to the administration of this question. Options 1, 2, and 3 do not consider safety for the client, and should be eliminated.
products. The nurse should take which of the following medication due to the risk of allergic reaction. Option 1 constitutes changing a medication The only correct answer that addresses safety is option 4.
actions? dosage, and is unsafe. Options 2 and 3 do not provide for client safety.

1.‐ Reduce the dosage of filgrastim by 5 micrograms.


2.‐ Assess the client for leukopenia and fever.
3.‐ Administer diphenhydramine (Benadryl) prior to administering filgrastim.
4.‐ Withhold the administration of filgrastim, and notify the physician.

2878 A client is taking sargramostim (Leukine). The nurse is Correct answer: 3 Adult respiratory distress syndrome can develop due to the toxicity of colony‐stimulating Knowing the side effects of the medication will lead to the correct answer choice. If this
especially careful to conduct a pulmonary assessment factors. The other options do not reflect concerns specific to this medication. was difficult, review the side effects of the medication.
to detect which of the following risks associated with
this drug?
1.‐ Congestive heart failure (CHF)
2.‐ Respiratory alkalosis
3.‐ Adult respiratory distress syndrome (ARDS)
4.‐ Pulmonary embolism

2879 A client with rheumatoid arthritis has developed a Correct answer: 1 Medications used to treat the symptoms of rheumatoid arthritis increase the client's Note in the question stem that the client has rheumatoid arthritis. Medications that are
fever. The nurse suspects that this might be related to susceptibility to infection. Fever accompanies infection, and requires further assessment. given for rheumatoid arthritis cause an increased susceptibility to infection. Knowing this
which of the following? Options 2 and 4 are incorrect because they do not reflect a concern related to this type of information and using the process of elimination should lead to selection of option 1 as the
medication. Option 3 is incorrect because infection is harmful to the client, and needs to be correct answer.
treated.
1.‐ The effect of prescribed medications on immune response
2.‐ A stable effect unrelated to medication
3.‐ A known, expected reaction to medication that is not harmful
4.‐ The client's ability to adapt to the medication regime
2880 Following the administration of immune serum Correct answer: 3 Immune serum globulin will irritate tissues, and the application of heat will reduce pain and Recognize that the question is referring to the injection site. The only answer that
globulin, the nurse should instruct the client to do discomfort. The other responses are unrelated to the issue of the question, which is local addresses the injection site is option 3. The other options do not address the administration
which of the following? discomfort at the injection site. of a medication.
1.‐ Avoid exposure to children.
2.‐ Call the physician with signs of bleeding.
3.‐ Apply heat to the injection site.
4.‐ Repeat the dose within one week.

2881 A client with a known seizure disorder is taking Correct answer: 1 Anticonvulsant medications administered concurrently with cyclosporine will cause decreased In order to answer this question, recall that anticonvulsants are maintained at a
cyclosporine (Sandimmune) following a kidney therapeutic levels of the cyclosporine medication. For this reason, the cyclosporine dose will therapeutic level to maintain blood levels. They should not be changed unless there is a
transplant. Based on this information, the nurse is need to be increased. The anticonvulsant dose needs to be given at standard dosage to therapeutic need. This would eliminate options 2 and 4. If this was difficult, review the
aware that which of the following medication dosages maintain therapeutic blood levels. effects of anticonvulsants on immunologic medications.
will need to be altered?
1.‐ The standard cyclosporine dose will need to be increased.
2.‐ The client's anticonvulsant dose will need to be increased.
3.‐ The standard cyclosporine dose will need to be decreased.
4.‐ The client's anticonvulsant dose will need to be decreased.

2882 A male client calls the physician's office complaining Correct answer: 3 The client and family should be instructed to assess for fluid retention and an irregular heart This question is asking for selection of the option that would address the safety of the
of shortness of breath and edema. He is currently rate. However, with the client complaining of shortness of breath and edema two days after client. Option 3 is the only safe answer in consideration of the other options. The other
taking hydrochlorothiazide (HCTZ) and digoxin chemotherapy and administration of oprelvekin (Neumega), the nurse should suspect that the options delay safe care, and should be eliminated.
(Lanoxin). Two days ago, he received chemotherapy, client is in cardiopulmonary insufficiency requiring medical attention. Options 1, 2, and 4 delay
followed by oprelvekin (Neumega). Based on this necessary medical attention and place the client at further risk for complications.
information, the nurse should take which of the
following actions?
1.‐ Schedule him an appointment to be seen by the nurse practitioner.
2.‐ Instruct him to lie down with his head elevated 45 degrees.
3.‐ Instruct him to call 911 and be transported to the Emergency Department.
4.‐ Call the client's family to assess his heart rate.

2883 A renal transplant recipient enjoys a grapefruit for Correct answer: 2 The only food interaction of significance with cyclosporine (Sandimmune) is grapefruit. This The only correct statement is answer 2. In selecting the correct answer, recall the
breakfast. He is taking cyclosporine (Sandimmune). The combination will result in an increased serum cyclosporine level. The other options are contraindications when taking the medication.
nurse should instruct him that ingestion of grapefruit completely false.
would have which of the following effects?

1.‐ Cause an increase in edema and renal insufficiency.


2.‐ Increase the serum level of cyclosporine.
3.‐ Assist in relieving his constipation.
4.‐ Increase renal metabolism of cyclosporine.

2884 It is reported in the news that a restaurant worker at Correct answer: 4 Exposure to hepatitis A from a restaurant worker necessitates the administration of immune The question is asking for selection of the priority option that protects the patron.
a local burger establishment has been diagnosed with serum globulin to prevent hepatitis A. The other options fail to address the client's need for Recognize that options 1, 2, and 3 do not address protection of the client. The only option
hepatitis A. A patron of this establishment calls the protection against possible exposure. that addresses protection is option 4.
health department for advice. The nurse should
provide which of the following instructions to the
patron?
1.‐ Call the physician for guidance.
2.‐ Assess self for future development of jaundice.
3.‐ Come to the health department and have a hepatitis blood test.
4.‐ Come to the health department to receive immune serum globulin.
2885 A young mother asks the obstetrics nurse why her Correct answer: 2 Childhood illnesses possess a high rate of mortality and morbidity. The use of immunizations In selecting the best answer, eliminate options 1, 3, and 4, as these options do not address
baby needs to receive immunizations. Which of the assists in the promotion of health and prevention of illness. Immunizations are begun shortly the client’s question. The best response is to explain to the client why the immunizations
following would be the best response by the nurse? after birth, and are required before a child can attend school. Option 3 is false. Options 1 and 4 are given, as presented in option 2.
are true, but do not address the reason immunizations need to be administered.

1.‐ "Immunizations are required by law."


2.‐ "Immunizations prevent illnesses that are associated with a high death rate."
3.‐ "Immunizations are safe, without side effects."
4.‐ "Immunizations are inexpensive, and can be provided free by the health department."

2886 A client with myasthenia gravis presents in the Correct answer: 4 Atropine (an anticholinergic medication) should be administered to counteract the cholinergic In selecting the best answer, identify which of the medications is classified as an
Emergency Department with severe ataxia and reaction of the medications used to treat myasthenia gravis. The other options are incorrect anticholinergic. The only medication that has this classification is option 4.
tremors. The nurse should be ready to administer because they do not have anticholinergic activity.
which of the following medications?
1.‐ Edrophonium (Tensilon)
2.‐ Ambenonium (Mytelase)
3.‐ Neostigmine (Prostigmine)
4.‐ Atropine (generic)

2887 While teaching the client about taking a new Correct answer: 1 Metronidazole (Flagyl) has several gastrointestinal side effects, including metallic or bitter Knowing the nursing interventions regarding client teaching will lead to the correct
prescription of oral metronidazole (Flagyl), the nurse taste. It can be taken with food, unless in the form of Flagyl ER, which should be taken one answer. If this was difficult, review nursing interventions regarding teaching to clients.
advises the client which of the following? hour before or two hours after meals. There is no evidence that Flagyl causes visual
disturbances or urinary retention (options 2 and 3). Alcohol taken during drug therapy or
within 48 hours after the drug is discontinued can induce a disulfiram‐like effect (option 4).

1.‐ To take with food to minimize the metallic or bitter taste


2.‐ That visual disturbances are a common side effect
3.‐ To monitor for urinary retention
4.‐ That alcohol should be avoided until the last dose is taken

2888 The nurse evaluates that the immunocompromised Correct answer: 3 A different gloved finger or a different finger cot should be used to apply acyclovir to each The question stem is asking for selection of an answer that indicates the client
client best understands use of topical acyclovir lesion, not only to prevent spread on the client's own body, but also to prevent transmission to understands the treatment procedure. Options 2 and 4 can be eliminated because they do
(Zovirax) for treating genital herpes when the client others. Caution needs to be taken as well not to contaminate the ointment in the container by not address treatment. Option 3 is the only correct answer because it addresses the caution
makes which of the following statements? obtaining ointment with a contaminated finger cot/glove. Handwashing is important, but is not needed in the application to a client in an immunocompromised state.
a barrier protection (option 1). Acyclovir is the drug of choice for primary herpes lesions in the
immunosuppressed client, but it has not been proven that acyclovir benefits the
immunocompetent client, although it could reduce viral shedding (option 2). Option 4 is a false
statement.

1.‐ "I need to wash my hands for at least 10 seconds with a teaspoon of antibacterial soap before and after application of the drug."
2.‐ "My sister isn't immunocompromised, but she has genital herpes, and should use acyclovir too."
3.‐ "I need to use a different finger cot when getting ointment and applying the ointment to each sore, to prevent self‐inoculation in other areas."
4.‐ "Acyclovir has been around a long time. I know there have to be newer drugs that are better to treat my genital herpes."

2889 The nurse prepares to initiate therapy with acyclovir Correct answer: 3 The nurse assesses hydration status; intake and output; creatinine; BUN; creatinine clearance; In order to select the correct answer, recall that the medication is toxic to the kidneys, and
(Zovirax) for a client with AIDS. What intervention is and other laboratory tests for renal dysfunction. The nurse also assesses for concurrent be directed to answer 3. The other answers do not apply to nephrotoxicity.
most appropriate for the nurse to perform? nephrotoxic agents being taken, since these drugs could increase the risk for nephrotoxicity
developing with administration of acyclovir (Zovirax). The bilirubin differentiates jaundice
caused by liver impairment from that caused by hemolysis (option 1). Bowel pattern and
hypokalemia are not particular to acyclovir therapy (options 2 and 4).

1.‐ Assess bilirubin.


2.‐ Assess for hypokalemia.
3.‐ Assess for other nephrotoxic drugs being taken.
4.‐ Assess the number of stools in the past 24 hours, and their consistency.

2890 The nurse anticipates that the client will be Correct answer: 1 Almost all clients receiving IV amphotericin B experience adverse reactions involving fever, Knowing the side effects and client responses will lead to the correct answer. The only
premedicated with an antipyretic, an antihistamine, chills, piloerection, hypotension, tachycardia, malaise, myalgia, arthralgia, anorexia, nausea, option that has the side effects that require premedication is option 1.
and an antiemetic to decrease infusion‐related vomiting, and headache. The other agents listed do not cause this cluster of severe side
reaction. Which of the following anti‐infectives is most effects. Meperidine (Demerol) may also be given to help manage the side effects.
likely prescribed?
1.‐ Amphotericin B (Fungizone)
2.‐ Acyclovir (Zovirax)
3.‐ Cefazolin sodium (Ancef)
4.‐ Methicillin (Staphcillin)

2891 Knowing that serum protein is needed to bind with Correct answer: 4 Complete proteins are higher‐quality proteins, and contain all nine essential amino acids in Having knowledge of foods made with complete protein will assist in selecting the correct
anti‐infectives in order to maintain therapeutic sufficient amounts to meet the body's needs. Sources of these proteins are of animal origin, answer. If this was difficult, review foods that are composed of complete proteins.
response rate, the nurse recommends which of the such as eggs, milk, cheese, and meat. Gelatin, an animal product, is an exception, since it is an
following desserts? incomplete protein. The pudding contains milk, but the egg custard contains the largest
quantity of animal proteins such as eggs and milk.
1.‐ Sherbet
2.‐ Pudding
3.‐ Fruit gelatin
4.‐ Egg custard

2892 The nurse assesses the client receiving gentamicin Correct answer: 2 The most significant adverse effects related to the aminoglycosides, of which gentamycin Knowing the side effects of the medication will lead to the correct answer choice. Realizing
(Garamycin) for what two most specific toxicities? (Garamycin) is a member, are nephrotoxicity and ototoxicity. Risk for ototoxicity is increased in that nephrotoxicity is present with gentamycin will lead to the one answer with
the presence of nephrotoxicity. nephrotoxicity in the answer. If this was difficult, review the side effects of the medication.

1.‐ Hepatotoxicity and neurotoxicity


2.‐ Nephrotoxicity and ototoxicity
3.‐ Leukocytosis and thrombocytopenia
4.‐ Pseudomembranous colitis and crystalluria

2893 The nurse assesses for what most significant side Correct answer: 1 The most common adverse effect of isoniazid (INH) is peripheral neuritis manifested as Knowing the side effects of the medication will lead to the correct answer choice. This is
effect in a client with tuberculosis taking isoniazid paresthesia of the extremities. The items in the other options are not side effects of this the only correct answer, and can be selected with understanding of the side effects of the
(INH)? therapy. medication. If this was difficult, review the side effects of the medication.

1.‐ Paresthesia in limbs


2.‐ Hearing loss
3.‐ Visual acuity
4.‐ Crystalluria

2894 During administration of vancomycin (Vancocin), the Correct answer: 3 "Red man syndrome" or "red neck syndrome" is flushing of the face, neck, and upper chest Knowing the side effects and client responses of the medication will lead to the correct
nurse recognizes which of the following clinical associated with too‐rapid IV administration of vancomycin (Vancocin). Hypotension with shock answer choice. This is the only correct answer, and can be selected if the client responses of
manifestations as a specific response to the drug being (not hypertension) also can result from the histamines released with too‐rapid infusion (option the medication are understood. If this was difficult, review the side effects of the
infused too rapidly? 1). Option 2 does not occur. Pseudomembranous colitis (option 4) is the result of a medication.
superinfection.
1.‐ Hypertension
2.‐ Projectile vomiting
3.‐ "Red neck syndrome"
4.‐ Pseudomembranous colitis
2895 Tetracycline has been ordered for a 2‐year‐old child. Correct answer: 2 Tetracycline should not be given to children under 8 years of age. The drug forms deposits in In order to select the correct answer, recall the contraindicated population for
What intervention by the nurse is most appropriate? the bone and primary dentition in growing children that can cause underdevelopment of the administration of the medication. Review the contraindications for the medication if this
child's bones and teeth, temporary stunting of the child's growth, and discoloration of the was difficult.
child's teeth. Discoloration of the teeth is not caused by direct contact of teeth with the
medication, as can happen with iron preparations (option 1). Since the drug should not be
administered to a child under 8 years of age, options 3 and 4 are less important, and therefore
incorrect.
1.‐ Teach the child to drink liquid tetracycline, to avoid discoloration of the teeth.
2.‐ Collaborate with the prescriber about appropriateness of the order.
3.‐ Evaluate renal function prior to initiation of therapy.
4.‐ Administer with 6–8 ounces of water.

2896 The nurse teaches a client taking a tetracycline or a Correct answer: 2 Photosensitivity is a side effect of these two classes of antibiotics. The client avoids sun Knowing the side effects of the medication will lead to the correct answer choice. This is
sulfonamide to do which of the following? exposure and tanning beds. Milk and food interferes with effectiveness of the tetracyclines, so the only correct answer, and can be selected if the side effects of the medication are
tetracyclines are taken on an empty stomach (option 3). These drugs are not associated with understood. If this was difficult, review the side effects of the medication.
increased bleeding or orthostatic hypotension (options 1 and 4).

1.‐ Have regular evaluation of the prothrombin time (PT) and international rationalized ratio (INR).
2.‐ Wear long sleeves and long‐legged pants, hat, and sunglasses when in the sunlight.
3.‐ Take with milk or food, to minimize gastrointestinal disturbances.
4.‐ Change position slowly, to avoid orthostatic hypotension.

2897 The preceptor assigned to a new graduate nurse has Correct answer: 2 Shaking the vial of filgrastim (Neupogen) will result in destruction of the medication's protein. Note that the question is asking which action to take related to administrating the
delegated him to administer filgrastim (Neupogen) to a Option 1 is unnecessary, and options 3 and 4 are not actions related to the administration of medication. Options 3 and 4 can be eliminated, since they do not address administration of
client with cancer. The preceptor directs the graduate this medication. the medication. The correct answer is option 2, since it considers safety as well as
nurse to take which of the following actions related to administration.
administration of this medication?

1.‐ Administer the medication with IV normal saline.


2.‐ Never shake the bottle, due to destruction of the protein.
3.‐ Assess the mobilization of stem cells.
4.‐ Administer before bone marrow transplant is done.

2898 A client diagnosed with cancer who is receiving Correct answer: 2 The provision of good oral hygiene will reduce the chance of stomatitis, which is common in Note that the client has cancer, and is receiving chemotherapy. Since these clients are high‐
sargramostim (Leukine) should be instructed to do immunosuppressed clients with cancer, who are in need of this medication. There is no need risk for stomatitis, try to prevent this complication of chemotherapy. This should lead to the
which of the following? to receive vitamin B&lt;sub&gt;6&lt;/sub&gt; concurrently (option 1). Alopecia and elevated option 2, which addresses prevention of stomatitis.
blood glucose levels (options 3 and 4) are not related to sargramostim.

1.‐ Receive vitamin B6 concurrently.


2.‐ Use good oral hygiene.
3.‐ Expect alopecia.
4.‐ Assess blood glucose levels daily.

2899 A client with rheumatoid arthritis develops type 1 Correct answer: 2 Monitoring strategies of care, particularly the ability to fill syringes and administer insulin, is a The question is asking for the priority nursing intervention. There are two options listed
insulin‐dependent diabetes mellitus (IDDM). The nurse nursing priority. Rheumatoid arthritis might limit fine motor movements of the hands that are that would be considered interventions: options 2 and 3. Using the process of elimination,
should make it a priority to make which of the needed for self‐administration of insulin, and this client might need further assistance from option 2 is the priority, since it also covers safety concerns for the client.
following client assessments? family or other caregivers. The assessments in the other options are also important, but the
ability to manage medication therapy takes priority.

1.‐ Response to corticosteroids


2.‐ Ability to fill syringes and give injections
3.‐ Ability to exercise
4.‐ Response to home environment

2900 A client who underwent a liver transplant asks the Correct answer: 3 Immunosuppressant agents reduce the client's ability to fight all infections, including In reviewing the question, note that the client has had a liver transplant, and will need
nurse why it is necessary to do such frequent mouth inflammation and infection in the mouth (stomatitis). The client is at risk of developing medication to prevent rejection. Recall that one of the side effects from the medications is
care. The best response by the nurse would be to state infection from organisms normally found in controlled numbers in the oral cavity. The other stomatitis. Having stomatitis requires good oral hygiene. After thinking the question
that it helps prevent the development of which of the options do not relate to this particular effect of immunosuppressants. through in this manner, select option 3.
following client problems?

1.‐ Dysphagia
2.‐ Halitosis
3.‐ Stomatitis
4.‐ Dental caries

2901 A client diagnosed with cancer is receiving Correct answer: 2 Sargramostim (Leukine) increases the production of granulocytes and macrophages. It also In order to select the correct answer, identify the purpose of the medication. If this was
sargramostim (Leukine) prior to stem cell collection. mobilizes stem cells to allow for stem cell collection. The information contained in the other difficult, review the purpose of the medication.
The client asks the reason for the medication. Which of options does not correctly describe the action of sargramostim.
the following would be the best response to the client?

1.‐ The medication diminishes the number of immature stem cells.


2.‐ The medication assists in the mobilization of stem cells, allowing for collection.
3.‐ The medication blocks the body's inflammatory response to an antigen.
4.‐ The medication suppresses T cell production.

2902 Tacrolimus (Prograf) is being administered to a client Correct answer: 4 AST and ALT, which are liver enzymes, are important to monitor, since tacrolimus (Prograf) Recognize that the medication is metabolized, and therefore toxic to the liver, to be
to prevent transplant rejection following organ and other immunosuppressants are hepatotoxic. The other responses do not relate as directed to option 4, which features the only liver studies listed in the options.
transplant. Which of the following laboratory studies is specifically to this adverse effect.
it of great importance for the nurse to monitor?

1.‐ LDH and CPK


2.‐ Uric acid and bilirubin
3.‐ Alkaline phosphatase and albumin
4.‐ AST and ALT

2903 During a well‐baby visit, the nurse notes that a child is Correct answer: 2 Hepatitis B is administered in three doses. The second dose follows one month after the first Use knowledge of correct administration of immunizations to select the correct answer.
one year late for his third hepatitis B vaccine. What dose, and the third dose is given six months after the original dose. If too much time elapses Eliminate options 1, 3, and 4, since these are incorrect actions by the nurse.
should the nurse do to correct this? between doses, as in this case, the series might need to be restarted. Options 1 and 4 could
result in incomplete or insufficient vaccination. The nurse does not set up immunization
schedules independently (option 3).
1.‐ Give the vaccine now.
2.‐ Inform the parent that the series will need to be restarted, and consult with the physician.
3.‐ Inform the parent that the vaccine will need to be given now and in one month.
4.‐ Inform the parent that two vaccines are sufficient.

2904 An infant girl is brought to the clinic for a well‐baby Correct answer: 1 For all clients, immunizations are contraindicated during a moderate‐to‐severe febrile illness. Use knowledge of correct administration of immunizations to answer this question
visit. She is scheduled to receive her second The nurse should withhold the vaccines, and have the mother bring the infant in to receive correctly. Review contraindications to immunization administration if this question was
diphtheria, pertussis, and tetanus (DPT) and H‐flu them after the illness has subsided. The actions in the other responses are incorrect. difficult.
vaccines, along with her oral poliovirus vaccine. While
measuring the infant's weight, the nurse decides that
the infant feels warm, and assesses her temperature,
which is 101ºF. The nurse should take which of the
following actions?
1.‐ Withhold the vaccines, and reschedule her visit for when she is not febrile.
2.‐ Instruct the mother on vaccine administration, and have her give them tomorrow.
3.‐ Administer acetaminophen orally, and give the immunizations.
4.‐ Obtain titers on the needed immunizations, and withhold them until the results are obtained.

2905 A client who has multiple sclerosis and receives Correct answer: 4 Cyclophosphamide (Cytoxan) combined with digoxin (Lanoxin) can result in digoxin toxicity, so Note in the question stem that the client is complaining of nausea. This should lead to the
cyclophosphamide (Cytoxan) and digoxin (Lanoxin) the health care team must continually assess for signs and symptoms of toxicity. Nausea is an answer related to digoxin, since the initial signs of digoxin toxicity are nausea and vomiting.
complains of nausea. The nurse would place highest early sign of digoxin toxicity, making option 4 the best action. Option 1 does not help the The only correct answer to this question that relates to nausea is answer 4.
priority on which of the following actions? situation; options 2 and 3 treat the symptom, rather than the problem.

1.‐ Evaluate the cyclophosphamide level.


2.‐ Administer an oral antiemetic daily.
3.‐ Provide six small, frequent meals.
4.‐ Evaluate the digoxin level.

2906 A client is receiving cyclosporine (Sandimmune). The Correct answer: 1 A side effect of cyclosporine (Sandimmune) is hirsutism, so the nurse should assess for signs Knowing the side effects of the medication will lead to the correct answer choice.
nurse will need to assess the client for evidence of and symptoms that relate to the nursing diagnosis of Disturbed Body Image. Options 2 and 3 Recognize that the question is asking for selection of a nursing diagnosis. If this was difficult,
which of the following nursing diagnoses? do not apply to the client with the information given in the question. Option 4 is not a nursing review the side effects of the medication.
diagnosis.
1.‐ Disturbed Body Image
2.‐ Pain
3.‐ Deficient Fluid Volume
4.‐ Altered neurological status

2907 A client exposed to Mycobacterium tuberculosis Correct answer: 1 To prevent active tuberculosis after exposure, the client is initiated on a single‐agent regimen, In order to select the correct option, recall the correct administration for the medication.
starts on chemoprophylaxis. The nurse provides what usually isoniazid (INH). For newly diagnosed active disease (option 2), a combination of If this was difficult, review administration of chemoprophylaxis to a client exposed to
instruction to the client? antitubercular agents is used for at least the first several weeks: isoniazid (INH), rifampin tuberculosis.
(Rifadin), and pyrazinamide (Tebrazid). The combination therapy lessens the risk of drug
resistance (option 3). Except for streptomycin, which is for IM use, the antitubercular agents
are administered orally (option 4).
1.‐ "You will take a single drug as isoniazid (INH) by mouth every day for 6–12 months."
2.‐ "You will be on at least two drugs effective against the tubercle bacillus for 3 months."
3.‐ "You will be on combination therapy in order to prevent development of drug resistance."
4.‐ "You will need to learn to give yourself subcutaneous injections."

2908 To minimize the pain related to intramuscular (IM) Correct answer: 3 Administration of very thick preparations of drugs, such as penicillin G with benzathine This question addresses the principles of safety in the administration of intramuscular (IM)
injection of 2 mL of penicillin G benzathine (Bicillin LA) (Bicillin LA), can be painful. To lessen the pain, intramuscular injection into a larger gluteal injections. In order to answer this question, recall the principles of IM injections, as well as
in the adult, the nurse will take which of the following muscle should be administered over 12–15 seconds, to separate the muscle fibers more safety related to the injections. If this was difficult, review the basic principles of
actions? gradually. Cold compresses to the injection site would delay absorption of the drug (option 1). intramuscular administration.
Aspiration for blood return with all IM injections is necessary for safety, since muscles contain
larger blood vessels (option 4). Injection into the deltoid could also result in prolonged
discomfort, resulting in limited motion of the upper extremities (option 2). Rotating sites, light
massage, and warm compress to the site may also be employed to limit discomfort.

1.‐ Apply a cold compress to the site after injection.


2.‐ Divide the dose, and inject half into each deltoid.
3.‐ Administer the drug deep IM slowly into a large muscle such as the gluteus.
4.‐ Limit prolonging the time taken to administer the drug by not aspirating.
2909 The nurse suspects hepatotoxicity evolving in a dark‐ Correct answer: 4 Jaundice in the dark‐skinned client can best be observed by assessing the hard palate. Note that the question is referring to liver toxicity, which would appear as jaundice. Recall
skinned client who is on an antibiotic. In what area of Normally, fat may be deposited in the layer beneath the conjunctivae that can reflect as a that assessment involves looking in the client s mouth for skin discoloration. If this was
the body should the nurse assess for jaundice? yellowish hue of the conjunctivae and the adjacent sclera in contrast to the dark periorbital difficult, review alterations to physical assessment.
skin. In these clients, palms and soles might appear jaundiced, but instead, calluses on the
surface of their skin can make the skin appear yellow.
1.‐ Palms
2.‐ Sclera
3.‐ Conjunctivae
4.‐ Hard palate of the oral cavity

2910 The nurse assesses the results of a vancomycin Correct answer: 4 A serum specimen for peak level is drawn 15–30 minutes after IV administration, to test for In order to answer this question correctly, recall the definition and purposes of peak and
(Vancocin) blood level drawn just prior to the next toxicity. Trough drug levels are drawn just prior to administration of the next IV dose, to trough levels of medications. If this was difficult, review the purposes of peak and trough
scheduled intravenous (IV) dose. The nurse would measure whether satisfactory therapeutic levels are being maintained. If the peak is too high, levels.
collaborate with the prescriber if which of the toxicity can occur, and the dose needs to be reduced, and/or the frequency of administration
following occurs? extended. If the trough is too low, then the dosage and/or frequency of administration need to
be increased.
1.‐ There is a high serum level, indicating the peak level is too high.
2.‐ This test measures the highest therapeutic concentration, and it is low.
3.‐ Toxicity is evident, suggesting the drug's half‐life is too short with the frequency prescribed.
4.‐ The drug level is low, indicating the drug dosage and/or frequency should be increased to maintain therapeutic blood levels between doses.

2911 The nurse observes cream‐colored or bluish‐white Correct answer: 1 Candidiasis may be treated with oral antifungal agents. The vaginal tablet used as a lozenge Recognize that the description in the question stem is that of a fungus. After scraping the
patches of exudates on the client's tongue and delivers a slow rate of dissolution over 15–30 minutes, which extends the length of contact tongue fails to remove the plaques, the nurse should be alerted to the need to provide an
pharyngeal mucosa. The nurse gently scrapes these with infected areas; the client is to swallow the saliva, but not to chew or swallow the troche antifungal medication. Use the process of elimination to be directed to option 1, which lists
plaques, and erythema appears. What intervention whole. The troche can be offered one‐half at a time, if needed. If the client wears dentures, an antifungal as a source of treatment.
does the nurse anticipate as being the most they are to be soaked overnight in oral suspension of the antifungal agent to destroy the
therapeutic? fungus on the dentures and prevent reinfection (option 2). Rinsing the mouth frequently with
warm sodium chloride solution may be palliative, not therapeutic or curative, but should not
be performed when the antifungal agent is still in contact with the oral mucosa (option 3). The
oral antifungal therapy must continue for 48 hours after signs and symptoms have been
resolved in order to prevent relapse (option 4).

1.‐ Provide prescribed nystatin (Mycostatin) vaginal troche as a lozenge.


2.‐ If the client wears dentures, provide nystatin (Mycostatin) swish and swallow.
3.‐ Rinse the client's mouth every four hours with warm sodium chloride solution.
4.‐ Collaborate with the prescriber for oral antiprotozoan agent just until clinical manifestations are resolved.

2912 The nurse administering chloramphenicol Correct answer: 1 Both of these drugs can cause bone marrow depression, adversely affecting the immune Knowing the side effects of the medication will lead to the correct answer choice. If this
(Chloromycetin) and methylprednisolone (Solu‐ system. The glucocorticosteroids also can mask clinical manifestations of infection. Observe for was difficult, review the side effects of the medication.
Medrol) concurrently for meningitis would assess for paleness; capillary refill greater than 3 seconds; sore throat; suprapubic pain or pressure; low
which of the following? back pain; low‐grade fever; bruising; bleeding; petichiae; and fatigue. Serum levels of
chloramphenicol (Chloromycetin) should be maintained in the range of 10–25 µg/mL. (More
than 30 µg/mL increases risk for bone marrow depression.) Hypercalcemia is more likely than
hypocalcemia to occur as an adverse effect of steroid therapy (option 3). Taste alterations and
hallucinations are not particular adverse effects of these agents (options 2 and 4).

1.‐ Occult signs of infection


2.‐ Taste alteration affecting nutrition
3.‐ Chvostek's sign, indicating hypocalcemia
4.‐ Psychotropic effect, including hallucinations
2913 The nurse who administers ketoconazole (Nizoral) for Correct answer: 1 Food will stimulate secretion of gastric enzymes, increasing the gastric acid environment, In the question stem, the priority words are gastrointestinal absorption. Since food
optimal gastrointestinal absorption will administer the which is needed for absorption of ketoconazole. Water will not stimulate gastric secretions stimulates gastric secretions, absorption will be increased. This is the only correct answer.
drug with which of the following? (option 2). Antacids and H&lt;sub&gt;2&lt;/sub&gt; histamine blockers should be avoided, or Recognize that the other options will not assist with gastrointestinal absorption.
ketoconazole (Nizoral) should be given one hour before or two hours after these agents
(options 3 and 4).
1.‐ Food
2.‐ Water
3.‐ Antacids
4.‐ An H2 histamine blocker

2914 A client is to start on sulfamethoxazole (Gantanol) for Correct answer: 3 The nurse ensures the urine specimen for C &amp; S is collected prior to the initial The question is asking for selection of the priority intervention. To make certain that a
a urinary tract infection (UTI). What priority nursing administration of the urinary tract anti‐infective agent, so that the causative microorganism client is being given the correct medication for the infection, a C &amp; S must be
intervention precedes administration of the first dose? and the anti‐infectives to which the organism is sensitive can be identified. The results of the performed before beginning the medication. The other options are incorrect when
urine C &amp; S will become available in several hours, but delaying anti‐infective therapy considering the priority intervention.
could cause the UTI to worsen (option 4). If the results of the test do not affirm the drug's
efficacy, the nurse will collaborate with the prescriber. Checking the bilirubin would be
appropriate only if attempting to differentiate the cause of existing jaundice as being hepatic
impairment or hemolysis (option 1). The urine specimen should be a clean‐catch or
catheterized specimen for more accurate results (option 4). Option 2 is not essential prior to
initiating drug therapy.

1.‐ Check the bilirubin level.


2.‐ Have client empty her bladder.
3.‐ Ensure that a urine specimen for culture and sensitivity (C & S) has been obtained.
4.‐ Analyze results of a random voided urine specimen for culture and sensitivity.

2915 A client taking amoxicillin (Amoxil) experiences eight Correct answer: 3 The client is demonstrating classic clinical manifestations of pseudomembranous colitis. It is Knowing the classical symptoms of Clostridium difficile will lead to the only correct
watery stools in the past eight hours, with abdominal important to identify whether this is a superinfection caused by C. difficile. Cisapride answer. The specific intervention relates to the symptoms of watery stools. Option 3 is the
tenderness and cramping and fever. The nurse (Propulsid) is a GI stimulant, and would not be indicated as therapy for this condition (option correct answer listing the collecting of stools to determine if the client has Clostridium
anticipates performing what specific intervention 1). Hepatotoxicity and diet are not directly related to pseudomembranous colitis (options 2 difficile.
related to these clinical manifestations? and 4).

1.‐ Administering cisapride (Propulsid) to stimulate GI motility to effectively remove toxins


2.‐ Analyzing liver enzymes for evidence of hepatotoxicity
3.‐ Collecting stool specimen for <i>Clostridium difficile</i>
4.‐ Teaching the client to reduce the fat in his diet

2916 A client is receiving ciprofloxacin (Cipro). To intervene Correct answer: 3 Because ciprofloxacin can cause photosensitivity, clients are advised to use sunscreen, wear Knowing the side effects of the medication will lead to the correct answer choice. If this
appropriately, the nurse utilizes which of the following protective clothing, and limit exposure to sunlight, especially since sunscreens might not was difficult, review the side effects of the medication
information about this medication? protect the skin from this type of reaction. When Cipro first became available, it was believed
it would be effective against MRSA, but resistance has developed (option 2). Cipro is processed
through the liver and kidneys for elimination (option 1). Renal failure does extend the half‐life
of each of the fluoroquinolones. Ciprofloxacin achieves serum levels that are effective against
several systemic infections caused by gram‐negative bacilli (option 4).

1.‐ Ciprofloxacin is eliminated in the bile, so dosages do not need to be adjusted for clients with renal impairment.
2.‐ Ciprofloxacin is one of the few antibiotics demonstrating efficacy against methicillin‐resistant staphylococcus aureus (MRSA).
3.‐ Sunscreen or sunblock applications might not prevent sunburn in clients receiving ciprofloxacin.
4.‐ Ciprofloxacin is effective in treating urinary tract infections, but not systemic infections.
2917 Following the administration of an MMR vaccine, the Correct answer: 2 The nurse should assess for signs and symptoms of hypersensitivity reaction following the Note that the question stem asks for selection of a priority assessment. Immediately after
nurse should make a priority assessment for which of administration of all vaccines. Wheezing is a sign of hypersensitivity reaction, and warrants administration of a medication, the priority assessment must relate to hypersensitivity.
the following client manifestations? immediate further assessment and emergency action to prevent possible death. Local Hypersensitivity in a client is usually manifested with respiratory symptoms. This symptom
discomfort (option 1) may be expected, and is treated, if necessary, with acetaminophen. is present in option 2, which is the correct answer.
Anxiety and vomiting (options 3 and 4) are not associated with administration.

1.‐ Pain
2.‐ Wheezing
3.‐ Anxiety
4.‐ Vomiting

2918 Following a liver transplant, the nurse should instruct Correct answer: 4 Liver function includes the regulation of blood clotting. Thus, the client should be instructed The question is asking for the priority physiological state that a client is experiencing. Also
the client to make it a priority to report which of the to report signs and symptoms of increased bleeding. Option 1 is a side effect of note that the client had a liver transplant. Since liver function involves blood clotting,
following signs and symptoms to the health care corticosteroids, but is not the priority from a physiological basis. Options 2 and 3 do not reflect bleeding as stated in option 4 would be the correct priority that the client should report.
provider? the vascularity of the liver and the associated risk of bleeding.
1.‐ Moon face
2.‐ Diminished pigmentation
3.‐ Dysphagia
4.‐ Bleeding

2919 A 3‐month‐old infant is diagnosed with leukemia. Correct answer: 3 Immunizations should be withheld during leukemia exacerbations, because the immune In planning the care and reviewing the possible answers for an infant diagnosed with
Which of the following does the nurse anticipate will system is compromised, and the client cannot manage an appropriate response to the leukemia, eliminate options 2 and 4, as they do not relate to what is stated in the question
be part of the plan of care for this infant? immunization. There is no need to place the client in isolation without added evidence of stem. Option 1 can also be eliminated, because there is no evidence in the question stem
immunosuppression (option 1). Options 2 and 4 are irrelevant to the issue of the question. that the infant is immunosuppressed. This would allow selection of the correct option, 3.

1.‐ The baby will be placed in isolation.


2.‐ Leukemia is familial, and other children should be assessed.
3.‐ All immunizations will be withheld during exacerbations.
4.‐ The baby will be NPO during chemotherapy.

2920 Following the administration of a Correct answer: 1 An inspiratory stridor is indicative of a hypersensitivity reaction to the DPT immunization, and Note that the question stem asks for selection of a priority action. Immediately after
diphtheria/pertussis/tetanus (DPT) immunization, the epinephrine should be administered to counteract the symptoms of the allergic response. administration of a medication, the priority action must relate to hypersensitivity.
nurse notes that the infant has an inspiratory stridor. Options 2 and 3 are irrelevant, and option 4 places the infant at risk for injury or death. Hypersensitivity in a client is usually manifested with respiratory symptoms, and the action
The nurse should take which of the following actions? is relief of the symptoms, which would lead to option 1.

1.‐ Administer epinephrine as per protocol orders.


2.‐ Evaluate for pulmonary edema.
3.‐ Inspect for periorbital edema.
4.‐ Assess the baby again in 15 minutes.

2921 A client who is taking cyclophosphamide is Correct answer: 1 Dysuria and bleeding are consistent with hemorrhagic cystitis, an adverse effect of Knowing the side effects of the medication will lead to the correct answer choice. If this
complaining of dysuria and bleeding. The nurse draws cyclophosphamide. These data should be reported to the physician or nurse practitioner. The was difficult, review the side effects of the medication.
which of the following conclusions? other options represent incorrect conclusions from the manifestations listed.

1.‐ This is a side effect of cyclophosphamide.


2.‐ This is due to a medication interaction with ibuprofen.
3.‐ The client has an increased risk for renal calculi, which is causing the symptoms.
4.‐ The client is exhibiting signs and symptoms of renal failure.
2922 A nursing student requires a rubella immunization Correct answer: 3 Women of childbearing age should not become pregnant for three months after receiving a After administration of the immunization, teaching points should be given to the client.
prior to beginning a clinical rotation in a local health rubella immunization. The other options are incorrect statements regarding aftercare Knowing that rubella has the potential to be teratogenic to a fetus will lead to the correct
care facility. The health center nurse will need to following rubella immunization. answer, option 3.
instruct the student to do which of the following?

1.‐ Apply ice to the injection site.


2.‐ Have a titer drawn in three days.
3.‐ Avoid getting pregnant for at least three months.
4.‐ Delay beginning clinical for three weeks.

2923 A client with myasthenia gravis contracts poison ivy, Correct answer: 2 Prednisone is a corticosteroid medication. When corticosteroids are given with medications Knowing the pathophysiological disease process of myasthenia gravis and complications
and is started on prednisone by mouth to decrease to treat myasthenia gravis, they decrease the effect of anticholinesterase medications. associated with it will assist in selecting the correct answer. If this was difficult, review the
inflammation. The nurse will need to instruct the client Because of this, symptoms of the disease could reappear, including respiratory difficulty. The complications associated with myasthenia gravis.
to do which of the following? other options do not address the interactive effects of these medications. In addition, the
nurse does not instruct a client to change a dose of a medication (option 4).

1.‐ Eat small, frequent meals.


2.‐ Be aware of any respiratory difficulty, and notify the health care provider.
3.‐ Expect increased libido.
4.‐ Decrease her usual dose of neostigmine until after the prednisone is finished.

2924 When teaching a client about medication therapy for Correct answer: 3 Gold salts suppress the activity produced by prostaglandins that contributes to the In order to select the correct answer, recall the action and client responses to the
rheumatoid arthritis, the nurse would incorporate destruction of joints. The statements in the other options do not reflect the action of this type medication, gold salts. If this was difficult, review the action of the medication.
which of the following points about gold salts in the of medication.
discussion?
1.‐ They increase glycoproteins to stimulate neutrophil production.
2.‐ They increase sedimentation rate and reduce stiffness.
3.‐ They decrease prostaglandin activity that contributes to joint destruction.
4.‐ They decrease immunoglobulin to suppress arthritic symptoms.

2925 To decrease the possibility of hepatotoxicity in a Correct answer: 4 Because of hepatotoxic effects of azathioprine and alcohol, the two substances should not be The question stem mentions decreasing the possibility of hepatotoxicity. Considering that
woman taking azathioprine (Imuran) to treat systemic administered together. Thus, the client should be instructed to avoid products containing alcohol has a direct effect on the liver, this would be the correct answer choice. The other 3
lupus erythematosus, the nurse should instruct the alcohol. The client should not take acetaminophen, which is also toxic to the liver (option 1). options are incorrect, and should not be taught to the client.
client to do which of the following? Azathioprine should not be taken with grapefruit juice (option 2), which reduces the drug's
effectiveness. Option 3 is a good general measure, but does not address the issue of
hepatotoxicity.
1.‐ Take acetaminophen (Tylenol) for pain.
2.‐ Take azathioprine with grapefruit juice.
3.‐ Prevent exposure to infection.
4.‐ Avoid the use of alcohol.

2926 When teaching the client measures to prevent a Correct answer: 4 The implementation of a high‐fiber diet will reduce the risk of constipation, a common side Knowing the side effects of the medication will lead to the correct answer choice. If this
common side effect of filgrastim (Neupogen), the effect in clients taking filgrastim (Neupogen). Option 1 is hazardous because it could cause was difficult, review the side effects of the medication.
nurse should instruct the client to do which of the dependency in the client. Option 2 indicates an insufficient amount of fluids to maintain
following? regular bowel function. Option 3 is irrelevant to the issue of the question.

1.‐ Take a laxative daily.


2.‐ Drink 1,000 mL per day.
3.‐ Eat a high‐protein diet.
4.‐ Eat a diet high in fiber.
2927 The nurse instructs the client receiving chemotherapy Correct answer: 4 A client with thrombocytopenia should avoid activities that could result in injury and bleeding. The core issue of the question is safety measures for chemotherapy‐induced
to avoid what risk associated with thrombocytopenia? For this reason, the client should avoid trimming the nails with a nail clipper, and should use a thrombocytopenia. Use nursing knowledge and the process of elimination to make a
nail file instead. Option 3 indicates the safe method for shaving; straight razors should be selection.
avoided, but electric razors are acceptable. Not all clients with thrombocytopenia also
experience concurrent leukopenia. Options 1 and 2 should be avoided to minimize risk of
infection, or when the client s white blood cell count is low.

1.‐ Being near individuals with upper respiratory infection


2.‐ Keeping fresh flowers and plants in the home
3.‐ Shaving with an electric razor
4.‐ Trimming nails with a nail clipper

2928 A newly registered nurse asks the nurse preceptor Correct answer: 3 Upon graduation, the registered nurse has attained adequate knowledge to manage standard The core issue of the question is prerequisite qualifications to administering
what qualifications are needed in order to administer clinical problems. Before accepting an assignment to administer chemotherapy, the nurse chemotherapy. Make your selection based on knowledge that this is a specialized skill
chemotherapy agents. The nurse preceptor should should receive additional education on the management of treatment side effects, requiring additional certification.
reply that a requirement is to: pharmacology, administration principles, and safe handling. Although a bachelor s degree and
one year of clinical experience might be helpful, they are not required to safely administer
chemotherapy. Certification in oncology nursing does not imply skill or knowledge of
chemotherapy administration procedures.

1.‐ Hold a bachelor s degree in nursing.


2.‐ Be certified by an approved chemotherapy administration program.
3.‐ Be certified as an oncology nurse.
4.‐ Have at least one year of clinical experience after graduation.

2929 During an intravenous (IV) push administration of Correct answer: 2 It is not uncommon to lose a blood return during IV administration of a vesicant. The core issue of the question is safe nursing practice relative to administration of IV
doxorubicin peripherally into client’s left forearm, the Repositioning the IV will only ensure infiltration due to manipulation. Restarting the IV gives no medications that are chemotherapeutic agents. Choose the option that is safest in avoiding
nurse becomes unable to obtain a blood return. The assurance that the blood return will not be lost again. Clients can experience extravasation harm to the client if the line is infiltrated after losing a blood return. Eliminate options 3
client has no complaints of discomfort at the site, and without pain, but not without swelling. With no evidence of swelling or pain, it is safe to flush and 4 first because they are potentially dangerous. Choose option 2 over 1 because option
no swelling is noted. The nurse should take which of with 20 30 c mL of saline to ensure an extravasation has not occurred. 1 might be unnecessary and excessive at this point.
the following actions?

1.‐ Remove the IV catheter, and restart it at another site.


2.‐ Flush the IV with saline, to ensure no extravasation has occurred.
3.‐ Reposition the needle, in hopes of obtaining a blood return.
4.‐ Since the client has no complaint of pain, continue the administration.

2930 The nurse should become concerned about which of Correct answer: 1 Prolonged diarrhea without adequate management will cause dehydration, nutritional The core issue of the question is knowledge of complications of side effects from
the following risks to a client receiving chemotherapy malabsorption, and circulatory collapse. Option 2 can accompany diarrhea, but is not a chemotherapeutic agents. Recall that nausea and vomiting deplete both fluid volume and
who had prolonged diarrhea at home without resulting clinical problem. Options 3 and 4 do not result from untreated diarrhea. nutrients. Make the selection that is consistent with one of these, which is option 1.
adequate management?
1.‐ Malnutrition
2.‐ Increased gastric motility
3.‐ Insidious weight gain and jaundice
4.‐ Renal failure

2931 The nurse would apply which of the following clinical Correct answer: 1 Delayed nausea may occur 24–48 hours after chemotherapy administration, primarily due to The critical words in the stem of the question are 24 hours after. From there, evaluate
labels to nausea and vomiting experienced by a client the ongoing effect that the metabolites exert on the CNS or GI tract. Despite effective each option in terms of time frame. Eliminate options 2 and 3 because they are focused on
24 hours after chemotherapeutic drug administration antiemetic regimens, 93% of clients receiving cisplatin experience delayed nausea. Anticipatory the present, and eliminate option 4 because it is future‐oriented related to the
with cisplatin (Platinol)? nausea occurs in approximately 25% of clients due to the classic conditioning response from chemotherapy administration.
prior therapy. Acute nausea occurs 1–2 hours after chemotherapy administration.

1.‐ Delayed nausea and vomiting


2.‐ Retching
3.‐ Acute nausea and vomiting
4.‐ Anticipatory nausea and vomiting

2932 The client is receiving chemotherapy with fluorouracil Correct answer: 4 Both 5‐FU and radiation therapy to the abdomen can cause diarrhea. Alopecia is uncommon Note that the stem of the question contains the critical words greatest risk. This tells you
(5‐FU) and concurrent radiation therapy to the with 5‐FU, and only occurs with irradiation to the skull. Some myelosuppression could result, that multiple options are correct, and that you must prioritize your answer. Choose the
abdomen for colon cancer. The client is at greatest risk but is not the greatest risk. Peripheral neuropathy is not related to either therapy. option that is a lower GI symptom, which is the area that both chemotherapy and radiation
for developing which of the following adverse effects? therapy will target.

1.‐ Peripheral neuropathy


2.‐ Alopecia
3.‐ Thrombocytopenia
4.‐ Diarrhea

2933 The nurse would assess for pulmonary toxicity in a Correct answer: 4 Pulmonary toxicity is a dose‐ and age‐related toxic effect of bleomycin. Cardiotoxicity is a The core issue of the question is knowledge of toxicity caused by bleomycin. If you
client receiving which of the following toxic effect of doxorubicin. Vincristine causes neurotoxicity. Cytoxan can cause hemorrhagic associate the bl in bleomycin with the color blue for cyanosis, this might help you to recall
chemotherapeutic agents? cystitis. that this drug causes pulmonary toxicity.
1.‐ Doxorubicin (Adriamycin)
2.‐ Vincristine (Oncovin)
3.‐ Cyclophosphamide (Cytoxan)
4.‐ Bleomycin (Blenoxane)

2934 A client receiving cyclophosphamide (Cytoxan) as Correct answer: 2 Although painful urination, dysuria, suprapubic pain, and blood‐tinged urine can occur when a The core issue of the question is knowledge of toxicity caused by cyclophosphamide. If you
treatment for cancer experiences painful urination, client experiences a urinary tract infection, in the oncology client receiving Cytoxan or Ifex, the associate the cy in cyclophosphamide or Cytoxan with the beginning of the word cystitis,
dysuria, suprapubic pain, and blood in the urine. The cause is usually chemically induced. A decreased platelet count might cause blood in the urine, this might help you to recall that this drug can lead to hemorrhagic cystitis.
nurse determines that these classic signs are but rarely causes the other symptoms. Renal dysfunction usually does not cause anuria.
compatible with which of the following problems?

1.‐ Thrombocytopenia
2.‐ Hemorrhagic cystitis
3.‐ Renal dysfunction
4.‐ Urinary tract infection

2935 For what major risk factors influencing gonad toxicity Correct answer: 2 The likelihood that chemotherapy will affect a client s fertility depends in part on the client s The critical word in the stem of the question is gonad. From there, eliminate the options
would a nurse assess in a client who is receiving gender and age, and on the specific agent. Since chemotherapy affects rapidly dividing cells, that refer to renal function (options 1 and 3). Choose option 2 over 4 because not all agents
chemotherapy drugs? men are more affected than women. Women over 30 are less likely to regain ovarian function, have the same degree of gonadal toxicity.
because they have fewer oocytes.
1.‐ Renal function, specific agents, and blood levels of chemotherapeutic drugs
2.‐ Gender, age, and specific agents
3.‐ Renal function, blood levels of drugs, and age
4.‐ Age, blood levels of drugs, and gender

2936 The nurse suspects that a client receiving Correct answer: 3 One of the most serious consequences of cancer is that the treatment intended to cure the The core issue of the question is knowledge of chemotherapeutic agents that cause added
chemotherapy with which of the following classes of client can contribute to the occurrence of a second malignancy. The alkylating agents, risk for future malignancy. Specific knowledge of the drug classes is needed to answer the
drugs is most likely to be at risk for developing a nitrosoureas, and procarbazine are the agents most likely to cause chemotherapy‐related question. Use this knowledge and the process of elimination to make a selection.
second malignancy? malignancies.
1.‐ Antimetabolite
2.‐ Taxane
3.‐ Alkylating agent
4.‐ Vinca alkaloid
2937 The nurse caring for a client receiving chemotherapy Correct answer: 4 Educating clients to manage treatment side effects so that their treatment programs can The core issue of the question is knowledge of the nurse’s role related to chemotherapy
incorporates which of the following practices into the remain on course is an essential function of nursing. Although the nurse should review that is prescribed for a client. Use general nursing knowledge and the process of elimination
routine for the day’s work shift? laboratory values before administering chemotherapy, it is the physician’s responsibility to to make a selection.
make dose reductions. Only a pharmacist can dispense medications. It is inappropriate to
influence the client’s decision making related to alternative treatment options.

1.‐ Determine if dosage reductions should be made based on the client’s laboratory values.
2.‐ Dispense the drugs that are part of the treatment regimen for the client.
3.‐ Encourage the client to consider alternative nonpharmaceutical treatment options.
4.‐ Educate the client about how to manage treatment side effects to maintain dose intensity.

2938 Which of the following is the most appropriate Correct answer: 2 Since many chemotherapeutic agents can cause ototoxicity, continued administration could The core issue of the question is the ability to translate the label ototoxicity into a working
nursing diagnosis for a client who has experienced cause irreparable hearing loss. Clients can become anxious when their hearing is nursing diagnosis that can be used to guide nursing care. Use general nursing knowledge
ototoxicity as a result of chemotherapy compromised. The hearing loss is not related to cerebral blood flow, and there is no pain and the process of elimination to make a selection.
administration? related with the condition. Hearing deficits are unrelated to skin integrity.

1.‐ Chronic Pain related to side effects of chemotherapy


2.‐ Anxiety related to potential or actual sensory loss secondary to chemotherapy
3.‐ Risk for Ineffective Tissue Perfusion related to decrease in cerebral blood flow
4.‐ Risk for Impaired Skin Integrity related to decreased cerebral blood flow

2939 A client who is 90 years old is exhibiting altered Correct answer: 4 The nurse should be specific when addressing a confused/disoriented client. In options 1 and The core issue of the question is communication techniques that will be most effective in a
mental status after chemotherapy with a neurotoxic 2, questions about the environment and the need to make choices might be too challenging, client who has experienced confusion secondary to neurotoxicity. Use knowledge of general
agent. Which of the following statements provides the and could decrease the client s self‐esteem. Option 3 provides irrelevant information. communication techniques and the process of elimination to make a selection.
most appropriate reality orientation when the client
first awakens in the morning?

1.‐ “Do you remember who I am, or what day it is? Would you like to write it down, so you can refer to it later?”
2.‐ “Did you sleep well? Which gown would you prefer to wear today, the pink or the blue?”
3.‐ “Today is Tuesday, and we will be having pancakes and sausage for breakfast.”
4.‐ “This is your second day in St. Elizabeth’s hospital. My name is Susan, and I will be your nurse for today.”

2940 In teaching the client receiving a continuous Correct answer: 4 The client should assess the oral cavity daily (not weekly) when receiving chemotherapy. A The core issue of the question is knowledge of oral assessment protocols for a client
fluorouracil (5‐FU) infusion, the nurse should instruct low‐residue, semi‐soft diet does not prevent stomatitis, but might decrease diarrhea. White receiving chemotherapy. Use knowledge of the signs and symptoms of stomatitis and fungal
the client about the importance of an oral care patches on the mouth or tongue should be reported to the physician so that an antifungal infection to choose option 4 over the others, which do not allow for appropriate
protocol, including which of the following statements? agent (Nystatin) can be prescribed. The patches should not be manually removed, because recognition (option 1) or treatment (options 2 and 3).
doing so will cause bleeding of the mucous membranes.

1.‐ “Assess the inside of your mouth weekly.”


2.‐ “Remove white patches with your toothbrush, and apply a lanolin‐based jelly.”
3.‐ “Eat a low‐residue, semi‐soft diet to prevent stomatitis.”
4.‐ “Call your doctor if white patches appear on your tongue or mouth.”

2941 The nurse is developing a plan of care for a client who Correct answer: 2 Client education is a nursing intervention, and does not require a physician s order. Although The core issue of the question is appropriate timing of client and family teaching related to
will receive chemotherapy with a neurotoxic agent. the nurse should assess client readiness, education related to chemotherapeutic agents should adverse effects of chemotherapy. Recall that a principle of client education is to conduct
The nurse plans to implement client and family occur before the treatment is administered, rather than when the symptoms develop. If the teaching about a problem beforehand whenever possible. This will easily help you to
education regarding these side effects at what client does not want to receive education, then the nurse should educate the caregiver. choose option 2 over the others.
appropriate time?
1.‐ When the client requests it
2.‐ Before the treatment begins
3.‐ When the symptoms occur
4.‐ When the physician requests client education
2942 Which of the following is an important intervention in Correct answer: 2 Broad‐spectrum antibiotics may be ordered for a client according to individual client The core issues of the question are recognition that the client is neutropenic and selection
a client receiving chemotherapy who has an absolute circumstances when the client is notably neutropenic, and at great risk of infection. A rectal of an appropriate action. Eliminate options 3 and 4 first because they increase the risk of
granulocyte count (AGC) of less than temperature should not be performed on a neutropenic client. The neutropenic client should infection. Then choose option 2 over 1 because it focuses on infection instead of possible
500/mm&lt;sup&gt;3&lt;/sup&gt;? avoid fresh fruits and vegetables. A daily bath will remove pathogens from the skin and bleeding as the risk.
decrease their potential for causing infection.
1.‐ Assess the client s rectal temperature every four hours.
2.‐ Administer broad‐spectrum antibiotics according to physician orders.
3.‐ Encourage intake of fresh fruits, to increase potassium level.
4.‐ Avoid bathing for two days, to reduce risk of skin irritation.

2943 Which of the following nursing interventions should Correct answer: 3 Avoiding fatty, spicy foods will often decrease nausea related to chemotherapy. Avoiding The core issue of the question is knowledge of methods to reduce nausea and vomiting in
be the highest priority of the oncology nurse to nutrition before treatments should be discouraged, and has little to no benefit. Salty foods a client receiving chemotherapy. Recall that fatty and spicy foods can aggravate nausea to
decrease nausea in a client receiving chemotherapy? might help, but potato chips are also high in fat. Antiemetics should be administered on a help you make the appropriate selection. Note also that options 2 and 3 are opposite in
scheduled basis, not after vomiting has already occurred. terms of dietary fat, which is a clue that one of them might be the correct answer.

1.‐ Avoid oral nutrition 24 hours before chemotherapy administration.


2.‐ Encourage the client to eat salty snacks, such as potato chips.
3.‐ Encourage the client to avoid fatty or spicy foods.
4.‐ Administer an antiemetic each time vomiting is experienced.

2944 The nurse would explain to a client receiving which of Correct answer: 2 A common complaint of clients receiving nitrogen mustard, cisplatin, and cyclophosphamide The core issue of this question is knowledge of chemotherapeutic agents that cause
the following chemotherapeutic agents that the drug is a metallic taste. Some clients become so sensitized to this taste that they become nauseated metallic taste as a side effect of therapy. Specific medication knowledge is needed to
could cause a metallic taste during administration, and in anticipation of their administration. Sucking on peppermints during administration might answer this question. Use the process of elimination to make a selection. You might be able
lead to taste changes? help. None of the other drugs listed are known to cause this problem. to partially remember this by realizing that Cytoxan and tongue both have the letters to.

1.‐ Etoposide (VP‐16)


2.‐ Cyclophosphamide (Cytoxan)
3.‐ Doxorubicin (Adriamycin)
4.‐ Prednisone (Deltasone)

2945 The nurse would assess the client receiving bleomycin Correct answer: 3 The cardinal sign of pulmonary toxicity is dyspnea, which can be present both on exertion and The core issue of the question is recognition of manifestations of pulmonary toxicity in a
(Blenoxane) as treatment for cancer for which of the at rest. The client also might have generalized fatigue and respiratory acidosis, but these are client receiving chemotherapy for cancer. The critical word cardinal tells you that multiple
following cardinal symptoms of pulmonary toxicity? not cardinal signs. A client does not have absence of breath sounds in upper lobes with options might be partially or totally correct, and you must prioritize your answer. Recall
pulmonary toxicity. that a classic sign of respiratory distress or pulmonary toxicity is dyspnea to help you to
prioritize this answer over the others.

1.‐ Absent breath sounds in upper lobes


2.‐ Generalized fatigue
3.‐ Dyspnea on exertion and at rest
4.‐ Respiratory acidosis on arterial blood gases

2946 The oncology nurse assesses a client receiving Correct answer: 1 Cardiomyopathy occurs in 40% of clients receiving anthracycline chemotherapy agents. While The critical words in the stem of the question are most ... common. This tells you that
chemotherapy for what most well‐known and common other cardiac problems can occur, cardiomyopathy is by far the most common. more than one option might be partially or totally correct, and that you must prioritize your
chronic cardiac toxicity? answer. Use medication knowledge and the process of elimination to make a selection.

1.‐ Cardiomyopathy
2.‐ Asymptomatic bradycardia
3.‐ Hemorrhagic myocardial necrosis
4.‐ Coronary artery spasm
2947 Neurological assessment is an important parameter Correct answer: 3 A decreased Achilles reflex is the earliest sign of neuropathy. Other assessment findings listed Recall knowledge of neuropathy signs and symptoms, and use the process of elimination
when administering plant (vinca) alkaloids. Which of are adverse medication effects that occur later in the trajectory of toxicity. to be led to the correct answer.
the following assessed by the nurse indicates an early
warning sign of pending impairment?

1.‐ Confusion
2.‐ Short‐term memory loss
3.‐ Depression of the Achilles reflex
4.‐ Decreased hand‐grasp strength

2948 Which of the following items should be included in Correct answer: 3 Exercise strengthens cardiac muscle, increasing function. Dexrazoxane protects the cardiac In the question stem, there is no mention that the client needs smoking cessation;
the therapeutic management to prevent doxorubicin‐ muscle against the toxic effects of doxorubicin. The other options do not apply to this therefore, options 1 and 2 can be eliminated. Oxygen administration is not mentioned in
related congestive heart failure from cardiotoxicity in a situation. the question either, and so option 4 can also be eliminated.
client who has cancer?

1.‐ Exercise and smoking cessation


2.‐ Smoking cessation and oxygen administration
3.‐ Exercise and administration of dexrazoxane (Zinecard), a cardioprotective antidote
4.‐ Oxygen administration and a low fat‐diet

2949 A client is receiving chemotherapy that has Correct answer: 2 With pursed‐lip breathing, back pressure is created to keep the airways open. This promotes The only correct answer to this question is option 2. Recall knowledge that pursed‐lip
pulmonary toxicity as an adverse effect. Which of the a more complete exhalation, and facilitates removal of secretions from the bronchial tree. The breathing is the most effective in keeping the airways open.
following breathing techniques is important to teach methods outlined in the other options do not achieve this effect.
the client to improve the efficiency of breathing?

1.‐ Exhaling through the nose


2.‐ Exhaling through pursed lips
3.‐ Exhaling one half, and then inhaling
4.‐ Exhaling using accessory muscles

2950 The nurse assesses for chemotherapy‐induced Correct answer: 2 Pulmonary toxicity is a known adverse effect of bleomycin. The other responses are incorrect. Recall the side effects of the antineoplastic agents listed. If this was difficult, review side
pulmonary toxicity in a client receiving which of the It is helpful to think of "blue for bleomycin" to recall that it has adverse respiratory effects. effects of the medications.
following antineoplastic agents?
1.‐ Fluorouracil (5‐FU)
2.‐ Bleomycin (Blenoxane)
3.‐ Etoposide (Vepesid)
4.‐ 6‐mercaptopurine (Purinethol)

2951 A nurse is administering chemotherapy to a group of Correct answer: 2 Intact skin is the body's first defense against infection. Although altered mucous membranes Note the word mucositis in the question. The suffix “‐itis” would indicate an inflammatory
clients. The nurse implements measures to reduce the are uncomfortable, and can affect the client's self‐image, they are not the most important process. Recognize this to be directed to an answer that lists infection, which is option 2.
risk of mucositis for which most important of the reasons. Occurrence of mucositis has no relationship to cancer recurrence.
following reasons?
1.‐ To increase comfort
2.‐ To decrease infection
3.‐ To improve self‐image
4.‐ To reduce cancer recurrence

2952 Which of the following clients is most likely to Correct answer: 1 A client's prior nausea history is indicative of her individual nausea threshold, and is predictive In answering this question, be aware that a person s susceptibility to nausea is dependent
experience chemotherapy‐induced nausea? of how chemotherapy‐induced nausea will be handled. Women are at higher risk for on the past history. In looking at the answers, the one client who had the most significant
developing chemotherapy‐induced nausea than are men. nausea is option 1, which is the correct selection. The others should be eliminated, since
they have less history of nausea than the priority option 1.
1.‐ A mother of two who experienced severe morning sickness during both of her pregnancies
2.‐ A navy pilot who experienced motion sickness as a child
3.‐ A 38‐year‐old male who has alcoholism
4.‐ A young woman with no significant history of emesis

2953 Which of the following interventions should the nurse Correct answer: 3 Since most clients become septic from organisms on their skin and in their environment, Remembering that interventions should be based on least invasive to most invasive will
plan to use for a client experiencing neutropenia? meticulous hygiene is the best way to prevent infection. An indwelling catheter is a source of allow elimination of options 1 and 4. There is no mention of any wounds in the question
infection, and should be used only if the client is immobile, and has a high risk for skin stem, so option 2 can be eliminated as well. This would lead to the correct option, 3.
breakdown. Dressings should cover all open wounds to prevent contamination. Long‐term
catheters should be flushed after use or Q 8 hours, to prevent clotting; however, this will not
decrease the client's risk for infection.

1.‐ Insert an indwelling urinary catheter to maintain accurate intake and output.
2.‐ Leave all wounds open to the air, for better healing.
3.‐ Encourage daily hygiene, regular oral care, and perineal care after each stool.
4.‐ Flush all lumens of a long‐term catheter with heparinized saline every eight hours.

2954 Which of the following interventions would be Correct answer: 3 Once stomatitis develops, meticulous oral hygiene must continue with a soft toothbrush or Since the client has stomatitis, it is important to use a soft toothbrush. Options 2 and 4 do
recommended for the client with the nursing diagnosis toothettes, fluoride‐containing toothpaste, and rinsing with a dilute baking soda solution. not mention this, and should be eliminated. In considering options 1 and 3, option 1 lists
of Risk for Altered Oral Mucous Membranes related to Dilute hydrogen peroxide mouthwashes are not recommended, since they can dry the mouth nonfluoride toothpaste, and should be eliminated, since fluoride is important in dental
side effects of fluorouracil (5‐FU)? and inhibit granulation tissue formation. Commercial mouthwashes are never recommended, health. This leads to the only correct answer, 3.
since they usually contain alcohol, which is drying, and can burn irritated tissues.

1.‐ Schedule oral hygiene Q 2 hours with a soft toothbrush and non‐fluoride toothpaste.
2.‐ Rinse mouth Q 2 hours during the day and Q 4 hours at night with a baking‐soda‐and‐peroxide rinse.
3.‐ Schedule oral care Q 2–4 hours with a soft toothbrush, fluorinated toothpaste, and a baking soda with isotonic saline rinse.
4.‐ Continue oral hygiene BID with toothettes and baking soda, and rinse with commercial mouthwash to decrease mouth odor.

2955 Which lifestyle changes should the nurse teach the Correct answer: 2 Increasing fiber in the diet increases bulk and gastric motility, thereby decreasing Adding fiber increases bulk, which decreases constipation. It is important to eliminate
client in order to decrease the risk of developing constipation. Increasing activity and fluids is also helpful. The use of stool softeners should be options 1 and 4, as these are not appropriate actions. Option 3 is discouraged as the first
constipation after receiving vincristine (Oncovin)? encouraged, but laxative usage should be kept to a minimum to prevent becoming laxative‐ option in preventing constipation.
dependent.
1.‐ Decrease physical activity.
2.‐ Add fiber to the diet.
3.‐ Increase laxative usage.
4.‐ Decrease fluid intake.

2956 The nurse explains to a client that an important Correct answer: 2 In most cancers, single‐drug therapy has proven unsuccessful, and leads to major tumor drug The only correct answer to this question is option 2. If this was difficult, review the
advantage of combination chemotherapy over single‐ resistance. Combination chemotherapy has demonstrated long‐term remission, more effective purpose and goals of combination chemotherapy.
drug regimens is that this will do which of the prevention of drug resistance, and tolerable treatment side effects.
following?
1.‐ Reduce the potential for nausea and vomiting
2.‐ Reduce the potential for tumor resistance
3.‐ Spare the normal cells from severe toxicity
4.‐ Decrease the likelihood of drug‐induced gonadal sterility
2957 Which of the following interventions represents the Correct answer: 2 Vesicant therapy will cause tissue irritation with eventual sloughing without the appropriate Note that the question asks for selection of the best nursing action. With knowledge of
best nursing action should an extravasation of a antidote. Protocols should be in place to administer the antidote immediately after an potential tissue irritation, you should select an answer that addresses care of the skin. This
vesicant occur? extravasation is observed, to neutralize the vesicant and minimize tissue trauma. The site would lead to option 2, which lists care of skin irritation. The other options do not initially
should be observed for 3–4 weeks, but a plastic surgeon need be consulted only if tissue address treatment for skin irritation.
damage occurs (option 3). While the infusion must be stopped, the priority is not choosing a
new site for administration (option 1). Emergency care is needed for anaphylaxis, not
extravasation (option 4).
1.‐ The infusion of the drug should be stopped, and a new site chosen for administration.
2.‐ The site should be treated with the appropriate antidote, and observed for 3–4 weeks.
3.‐ A plastic surgeon should be consulted immediately.
4.‐ Emergency medical care, including corticosteroids and epinephrine, should be administered immediately.

2958 The nurse is especially careful to assess for hematuria Correct answer: 1 Cystitis can occur in the bladder as a result of chemotherapy with cyclophosphamide or In order to answer this question correctly, recall the side effects and client responses to
and dysuria when a client is receiving which of the ifosfamide. Nephrotoxicity can occur higher in the renal tubules as a result of therapy with listed medications. If this was difficult, review side effects and client responses to the
following chemotherapeutic agents? cisplatin, methotrexate, streptozocin, and, more rarely, with mitomycin. medications.

1.‐ Cyclophosphamide (Cytoxan)


2.‐ Doxorubicin (Adriamycin)
3.‐ Fluorouracil (5‐FU)
4.‐ Cytarabine (Ara‐C)

2959 After noting altered gait, altered reflexes, and ileus in Correct answer: 2 The symptoms exhibited are most descriptive of neurotoxicity. Symptoms can arise as a result In selecting the correct answer note in the question stem that there are two physiologic
a client receiving chemotherapy, the nurse would of direct or indirect damage to the central nervous system, peripheral nervous system, cranial changes that are occurring that relate to neurology. This indicates the higher‐level system
report to the oncologist that which of the following nerves, or any combination of the three. stated in option 2.
types of organ damage is occurring?

1.‐ Hepatic
2.‐ Neurologic
3.‐ Renal
4.‐ Gastrointestinal

2960 The nurse places highest priority on assessing for Correct answer: 3 Myelosuppression is the most common and lethal side effect of chemotherapy. Since Knowing that a side effect of chemotherapy is the vulnerability of the cells to
which of the following most common and most lethal hematopoietic cells divide rapidly, they are most vulnerable to chemotherapy. Although chemotherapy will lead to the correct option, 3. The other options are serious, but can be
side effects of chemotherapy? respiratory failure can be lethal, an increased respiratory rate does not indicate respiratory treated, and are not considered lethal.
failure (option 1). Electrolyte imbalances and liver function study elevations can occur with
greater or lesser severity, and are therefore not the most lethal (options 2 and 4).

1.‐ Increased respiratory rate


2.‐ Electrolyte imbalance
3.‐ Myelosuppression
4.‐ Elevated liver function studies

2961 Which of the following is the initial step that the Correct answer: 1 Clarifying information and dispelling the myths that surround cancer and cancer treatment Note that this question asks for selection of an initial step. Recognize that options 2, 3, and
nurse takes in client and family education regarding are the initial step in client and family education. Most institutions do not require a signed 4 are all interventions. Before implementing an intervention it is important to make certain
chemotherapy administration? informed consent to administer chemotherapy. Choosing the venous access device and the that the client understands the process, and to teach any additional information that is
necessary safety equipment occurs after the initial education has been completed. required. This should lead to option 1.

1.‐ Clarify information and dispel misconceptions.


2.‐ Obtain informed consent.
3.‐ Choose an appropriate venous access device.
4.‐ Demonstrate safe gloving and gowning.
2962 A client asks the nurse to explain the term "cell Correct answer: 3 Drugs that are cell cycle–specific act preferentially on cells that are proliferating (dividing). Recall the definition of the term listed to be able to select the correct option.
cycle–specific," which was overheard when the health Cells in the G&lt;sub&gt;0&lt;/sub&gt; phase or resting phase are dormant, and are out of the
care team made rounds. The nurse replies that cell cycle. Cells that are differentiated are also out of the cell cycle.
chemotherapy drugs that are cell cycle–specific act
preferentially on cells that:
1.‐ Are well developed.
2.‐ Have entered a resting phase, and are not growing.
3.‐ Are actively getting ready to divide or are dividing.
4.‐ Are no longer alive.

2963 When administering vesicant chemotherapy to a Correct answer: 1 The client should be instructed to notify the nurse of any burning or pain during Note that the question asks for selection of the best nursing action. With knowledge of
client, the nurse should do which of the following? administration, so that the treatment can be stopped. Vesicant therapy should be potential tissue irritation, select an answer that addresses any irritation to the skin during
administered in the large veins midway between the wrist and elbow. A blood return should be infusion.
checked with every 1–2 mL of drug administered. Warm compresses should not be applied
during administration, since a potential extravasation could be missed.

1.‐ Instruct the client to report any pain or burning experienced during the infusion.
2.‐ Infuse this type of medication in the large veins of the hands or wrist.
3.‐ Assess for a blood return after each 5–10 mL of chemotherapeutic drug administered.
4.‐ Apply a warm compress during the infusion to dilate the vein.

2964 The nurse assesses for cardiotoxicity most carefully in Correct answer: 2 Vincristine, doxorubicin, and nitrogen mustard are all vesicants. Doxorubicin has a dose‐ In order to answer this question correctly, recall the side effects and client responses to
a client receiving which of the following limiting cardiotoxic effect. The major side effect of vincristine is peripheral neuropathy. The listed medications. The only medication that has major side effects related to the cardiac
chemotherapeutic agents? major side effects of nitrogen mustard are severe nausea and vomiting, and system is option 2. If this was difficult, review side effects and client responses to the
thrombocytopenia. The major side effects of cisplatin are severe nausea and vomiting, and medications.
nephrotoxicity.
1.‐ Vincristine (Oncovin)
2.‐ Doxorubicin (Adriamycin)
3.‐ Nitrogen mustard (Mustargen)
4.‐ Cisplatin (Platinol)

2965 When administering chemotherapy, the nurse should Correct answer: 2 The incidence of drug interactions increases with the number of medications the client takes. In order to answer this question correctly, recall the principles of safe administration of
be aware of which of the following drug interactions? Pretreatment and ongoing assessment are essential to detect potential interactions and avert the medications. Note the similarities between options 1 3, and 4 . Knowing that some
or minimize an adverse outcome. Contrary to options 1 and 3, it is often necessary to chemotherapeutic medications must be given concurrently will allow elimination of these
administer chemotherapeutic agents concurrently or sequentially with other drugs. options.

1.‐ Cytotoxic chemotherapy drugs should never be given concurrently with another drug.
2.‐ Drug interactions can result in additive toxicity, decreased effectiveness, or altered activity of non‐chemotherapeutic medications.
3.‐ Chemotherapy should never be administered in sequence with another drug.
4.‐ The number of drugs a client is taking does not influence the incidence of drug interactions.

2966 The nurse reinforcing health teaching with a client Correct answer: 4 Although a client's physical well‐being and response to previous treatments are important to Note the word “limitations” in the question stem. In thinking about the word "limitation,"
explains that dose limitations of chemotherapy are know, toxicities of the drug commonly determine the maximum amount of the drug that can consider toxicity when giving chemotherapeutic agents. Since chemotherapeutic agents are
determined by which of the following client‐ or drug‐ be administered safely. The number of cancer cells in the body has little to do with the dose toxic, the only correct answer is option 4.
related factors? limitations of the medications.
1.‐ Physical status and medical history
2.‐ History of previous treatments
3.‐ Number of cancer cells in the body
4.‐ The toxicities of a particular drug
2967 The nurse is administering nitrogen mustard Correct answer: 2 The question indicates that extravasation might be occurring. Prompt nursing action in Note that the question asks for selection of the first nursing action. With knowledge of
(Mustargen), and notes swelling at the intravenous (IV) general will minimize tissue damage; therefore, nursing actions should be initially directed potential tissue irritation from chemotherapeutic agents, select an answer that addresses
site. The nurse should first: toward the suspicious site. The drug administration should be stopped, since failure to do so extravasation. This would lead to option 2. The other options are not the initial treatment.
will further disperse drug into the tissue. Clients can experience extravasation without pain, They are interventions that could be implemented after stopping the IV to assess the site.
but not without swelling. Flushing the line with saline or dextrose is not advised, since there
still might be vesicant drug remaining in the tubing.

1.‐ Continuing trying to aspirate for a blood return.


2.‐ Stop administration, and attempt to aspirate.
3.‐ Flush the line with saline.
4.‐ Obtain a new site for drug administration.

2968 During the nadir period, which of the following Correct answer: 2 Medications that inhibit platelet aggregation should be avoided. Aspirin, ibuprofen, and In order to answer this question, recall the definition of nadir. This can be termed the
medications should be avoided? indomethacin are examples of some of these agents. Tylenol is the drug of choice for mild pain period of reconstruction. Any medications that inhibit platelet reconstruction should be
and fever. Benadryl is often used for sinus drainage or as an antihistamine, and Robitussin is avoided. The medication to be avoided is the one listed in option 2. The other options do
used to manage cough. not have an effect on platelet reconstruction.
1.‐ Acetaminophen (Tylenol)
2.‐ Ibuprofen (Motrin)
3.‐ Diphenhydramine (Benadryl)
4.‐ Guanefesin (Robitussin)

2969 The nurse would be most careful to assess for Correct answer: 1 Although many chemotherapy agents can cause stomatitis, the antimetabolites are Knowing the classification of the medications should assist in selecting the correct answer.
stomatitis in a client receiving which of the following commonly known for causing this side effect. Fluorouracil is the only drug listed in this class. Antimetabolites are known to have the side effect of stomatitis. The other medications are
chemotherapeutic agents? Cisplatin is an alkylating agent; bleomycin is an antitumor antibiotic; and vincristine is a plant not known for causing stomatitis, and can be eliminated.
(vinca) alkaloid.
1.‐ Fluorouracil (5‐FU)
2.‐ Cisplatin (Platinol)
3.‐ Bleomycin (Blenoxane)
4.‐ Vincristine (Oncovin)

2970 A client experiences severe nausea for two weeks Correct answer: 3 A client at risk for nausea should not lie down for at least 30 minutes after meals, to avoid This question is asking for an answer that indicates that the client does not understand the
following her chemotherapy treatment. Which aspiration. The physician should be notified of excessive weight loss (option 1). Foods and teaching for management of nausea. Option 3 is the only correct option, because it shows a
statement indicates a need for further instruction on beverages are better tolerated when they are neither hot nor cold (option 2). Option 4 is a lack of understanding, as lying down can lead to reflux, causing nausea. The other
management of nausea? good client action if other measures fail. statement options indicate understanding.
1.‐ "I need to call my doctor if I lose more than 10 percent of my body weight."
2.‐ "I should try to eat bland, chilled foods, and drink liquids separate from my meals."
3.‐ "I need to lie down for an hour after each meal."
4.‐ "I should call the doctor if my nausea doesn't go away, to see if a different anti‐emetic could provide better relief."

2971 The nurse would assess for functional constipation as Correct answer: 1 Neurotoxicity is a side effect of the vinca alkaloid classification, to which vincristine and Knowing that the side effects of a vinca alkaloid classification cause neurotoxic effects of
a result of neurotoxicity when the treatment plan vinblastine belong. Fluorouracil, methotrexate, and doxorubicin often cause diarrhea. constipation will lead to option 1. The other medications do cause this effect, and can be
involves use of which of the following chemotherapy Gemcitabine and mitoxantrone commonly cause myelosuppression. eliminated. If this was difficult, review the classifications of the medications.
agents?
1.‐ Vincristine (Oncovin) and vinblastine (Velban)
2.‐ Fluorouracil (5‐FU) and bleomycin (Blenoxane)
3.‐ Methotrexate (Folex) and doxorubicin (Adriamycin)
4.‐ Gemcitabine (Gemzar) and mitoxantrone (Novantrone)
2972 The nurse is caring for a client receiving Correct answer: 3 Anthracyclines are known to cause cardiotoxicity by directly damaging the cardiac myocyte In order to answer this question correctly, recall the side effects and client responses to
chemotherapy as part of cancer treatment. The nurse cells. Not all anti‐tumor antibiotics cause cardiomyopathy. Cardiotoxicity is not a common side listed medications. The only medication that has major side effects related to the cardiac
looks for signs of cardiotoxicity due to direct damage effect of vinca alkaloids or alkylating agents. system is the one in option 3. If this was difficult, review side effects and client responses to
to cardiac myocyte cells in a client receiving drugs in the medications.
which of the following classifications?

1.‐ Vinca (plant) alkaloids


2.‐ Alkylating agents
3.‐ Anthracyclines
4.‐ Antitumor antibiotics

2973 The nurse provides anticipatory guidance about Correct answer: 2 The classification of chemotherapeutic agents that have the greatest effect on the gastric Knowing the side effect of constipation with the antimetabolites classification of
managing diarrhea for a client receiving chemotherapy mucosa is the antimetabolites. This can lead to diarrhea, which in turn can lead to fluid and chemotherapeutic agents will allow selection of option 2. The other medications do cause
drugs in which of the following classifications? electrolyte imbalances if not managed well. this effect, and can be eliminated. If this was difficult, review the classifications of the
medications.
1.‐ Vinca alkaloids
2.‐ Antimetabolites
3.‐ Hormonal agents
4.‐ Nitrosoureas

2974 Which of the following is the most appropriate Correct answer: 1 An appropriate outcome is one that supports the client's functional ability in daily life. Clients Remember that in nursing, the goal is to assist clients to perform as many normal daily
outcome for a client with cardiotoxicity related to with cardiac toxicities might experience discomfort with cardiac ischemia (option 3), but can activities as possible. The only answer that addresses this goal is option 1. The other
cancer treatment? expect that it will be controlled. Dyspnea and shortness of breath will occur (option 4), but options should be eliminated.
should be controlled to allow the client to maintain normal daily activities. Option 3 is
unrelated to the question, and might be unrealistic.
1.‐ Client will maintain activity levels that allow normal daily activities as identified by the client.
2.‐ Client will experience no pain.
3.‐ Client and caregiver will express relief about possible cancer cure.
4.‐ Client will experience no dyspnea during any activities.

2975 The nurse would avoid using intravenous (IV) solution Correct answer: 3 When administering Paclitaxel, PVC bags and infusion sets should be avoided because of In order to answer this question correctly, recall the methods of administration for the
bags and infusion sets made from polyvinyl chloride leaching of DEPH (plasticizer). All other chemotherapy agents are safe to administer with chemotherapeutic agents listed. If this was difficult, review the safe methods of
(PVC) when administering which chemotherapy agent? conventional IV sets. administration of the medications.

1.‐ Doxorubicin (Adriamycin)


2.‐ Vincristine (Oncovin)
3.‐ Paclitaxel (Taxol)
4.‐ Methotrexate (Folex)

2976 The nurse instructs the client receiving chemotherapy Correct answer: 4 A client with thrombocytopenia should avoid activities that could result in injury and bleeding. The word “thrombocytopenia” in the question stem should alert to the condition of low
that which of the following should be avoided to For this reason, the client should avoid trimming the nails with a nail clipper, and should use a platelets. A client with low platelets would be high‐risk for bleeding. The only option that
reduce the risks associated with thrombocytopenia? nail file instead. Option 3 indicates the safe method for shaving; straight razors should be addresses a high risk for bleeding is option 4.
avoided, but electric razors are acceptable. Not all clients with thrombocytopenia also
experience concurrent leukopenia. Options 1 and 2 should be avoided to minimize risk of
infection, or when the client's white blood cell count is low.

1.‐ Being near individuals with upper respiratory infection


2.‐ Keeping fresh flowers and plants in the home
3.‐ Shaving with an electric razor
4.‐ Trimming nails with a nail clipper
2977 A client being treated for glaucoma complains of Correct answer: 3 Clients experiencing photophobia are instructed to wear dark sunglasses and to avoid bright Focus on the word photophobia and use the process of elimination to choose the answer
photophobia. The nurse’s teaching instructions include lights. Not enough information is provided to warrant discontinuing the medication. Eyes that shields the eyes from light.
which of the following? should not be wiped with tissue immediately after instillation of drops, and no special glasses
are required.
1.‐ Discontinue use of the medication.
2.‐ Wipe the eyes with tissue immediately after instillation of eye drops.
3.‐ Wear dark glasses when outside, or when around bright lights.
4.‐ Special glasses are necessary while being treated for glaucoma.

2978 Which of the following statements by the client Correct answer: 2 Difficulty in adjusting quickly to changes in illumination occurs as a result of miosis, an effect Specific knowledge of the important teaching points related to pilocarpine is needed to
demonstrates an understanding of client education of pilocarpine. The client will experience more difficulty seeing at night (option 1). Driving is answer the question. Use this knowledge and the process of elimination to make a
regarding pilocarpine (Isopto Carpine)? not contraindicated (option 4); however, nighttime driving might not be possible, because of selection.
the miosis.
1.‐ “I will see better at night.”
2.‐ “I may have trouble adjusting to darkness.”
3.‐ “I should not have any trouble adjusting to changes from light to dark.”
4.‐ “I will not use the medication if I plan to drive.”

2979 The nurse is providing care to a client taking Correct answer: 4 The diuretic effects of methazolamide could lead to electrolyte disturbances of hypokalemia The core issue of the question is knowledge of electrolyte disturbances for which the client
methazolamide (Neptazane), a carbonic anhydrase and hypernatremia. Options 1, 2, and 3 are either partially or totally incorrect. is at risk during therapy with methozolamide. Recall that the medication has a diuretic
inhibitor for glaucoma. The plan of care includes effect, and reason that potassium might be lost while sodium is retained. Use this
monitoring for which of the following electrolyte knowledge and the process of elimination to make a selection.
imbalances?
1.‐ Hyperkalemia and hypernatremia
2.‐ Hypokalemia and hyponatremia
3.‐ Hyperkalemia and hyponatremia
4.‐ Hypokalemia and hypernatremia

2980 Which of the following statements by a client who Correct answer: 2 Improvement of the infection should be noticed within a few days of beginning the antibiotic The wording of the question tells you that the correct answer is an inaccurate statement.
has a prescription for otic chloramphenicol therapy. Superinfections are known to occur with this medication; therefore, seven days is too Use the process of elimination, and select the option that represents incorrect information.
(Chloromycetin) indicates a need for further long to seek further evaluation and treatment. Options 1, 3, and 4 are correct actions by the
instructions? client experiencing any ear infection or disorder.
1.‐ “I will inform my doctor of increased ear pain.”
2.‐ “I will inform my doctor if my ear infection has not improved within seven days.”
3.‐ “I will inform my doctor if I have an increase in drainage from my ear.”
4.‐ “I will inform my doctor if I experience any hearing disturbances.”

2981 A client is receiving pilocarpine (Isopto Carpine) for Correct answer: 1 Symptoms of systemic absorption of pilocarpine include diaphoresis, diarrhea, bradycardia, The core issue of the question is recognition of signs of systemic absorption of pilocarpine.
the treatment of glaucoma. Which of the following and hypotension. Options 2, 3, and 4 are incorrect because they are opposites of actual signs Specific knowledge of systemic effects of this medication is needed to answer the question.
symptoms experienced by the client does the nurse of systemic absorption. Use this knowledge and the process of elimination to make a selection.
attribute to systemic absorption?
1.‐ Diaphoresis
2.‐ Constipation
3.‐ Tachycardia
4.‐ Hypertension

2982 A client is receiving cyclopentolate and phenylphrin Correct answer: 2 Cyclomydril and other mydriatics are applied topically to produce mydriasis (dilated pupil) to The core issue of the question is knowledge of the intended effects of cyclomydril. Note
(Cyclomydril) before an ocular examination. The nurse facilitate ocular examination. Options 1, 3, and 4 are incorrect because they drugs do not that the name of the drug contains the letters myd, which is also the beginning of the word
would explain the purpose of the medication as which constrict the pupil, provide anesthesia, or prevent infection, respectively. mydriasis (meaning to dilate the pupils). Using simple word association will sometimes
of the following? assist in making the correct selection.

1.‐ To constrict the pupil


2.‐ To dilate the pupil
3.‐ To provide anesthesia
4.‐ To provide a prophylactic antibiotic

2983 Which of the following symptoms described by a Correct answer: 1 Systemic side effects of ophthalmic atropine include tachycardia, confusion, dry mouth, The core issue of the question is knowledge of side/adverse effects of atropine. Recall that
client would lead the nurse to suspect a systemic side drowsiness, and slurred speech. Options 2 and 3 are opposites of known systemic side effects, when used for cardiac reasons, the medication speeds up heart rate. With this in mind,
effect of atropine ophthalmic solution? while option 4 (diaphoresis) is unrelated. eliminate each of the incorrect responses, and choose tachycardia as the correct answer.

1.‐ Tachycardia
2.‐ Bradycardia
3.‐ Salivation
4.‐ Diaphoresis

2984 Which of the following statements demonstrates the Correct answer: 1 Ophthalmic solution that has darkened or become cloudy should be discarded. Most solutions The core issue of the question is safe self‐administration of ophthalmic medications. Use
client’s understanding of proper administration of are clear (option 2). Swabs should not be used to apply medication (option 3), and the nursing knowledge and the process of elimination to answer the question.
ophthalmic solutions? medications generally have a shelf life of three months (option 4).
1.‐ “I will not use any medication if it has turned brown.”
2.‐ “I will not use my medication if it is clear in color.”
3.‐ “I will use a cotton swab to apply my medication.”
4.‐ “I will not use any medication that is more than one month old.”

2985 In teaching a client about side effects of medications, Correct answer: 1 Salicylates can cause tinnitus, vertigo, and hearing loss, if ingested in high doses. Vitamin C The core issue of the question is an understanding of the types of drugs that can cause
for which of the following over‐the‐counter (option 2), diphenhydramine (option 3), and vitamin A (option 4) do not present this concern. ototoxicity. Use nursing knowledge and the process of elimination to make a selection.
medications would the nurse discuss ototoxicity?

1.‐ Salicylates (aspirin)


2.‐ Vitamin C
3.‐ Diphenhydramine (Benadryl)
4.‐ Vitamin A

2986 During a follow‐up visit at the clinician’s office, a Correct answer: 4 Inserting objects, including medication droppers, into the ear canal can perforate the The core issue of the question is an understanding of the procedure for safe self‐
client states: “I insert the ear dropper deep into my ear tympanic membrane. Though the ear canal might be obstructed with cerumen (option 1), administration of an otic medication. Use nursing knowledge and the process of elimination
so the medication doesn’t run back out.” The nurse’s there are other reasons, such as inappropriate instillation technique, for the medication to not to make a selection.
response and priority teaching to the client is based on flow into the ear canal (option 2). The client is instructed to lie on the unaffected side, not the
which of the following? affected side (option 3), to allow flow of medication into the ear.

1.‐ The client’s ear canal is likely obstructed with cerumen.


2.‐ The client is using the appropriate technique for administering an otic solution.
3.‐ The client should lie on the same side as the affected ear for five minutes to allow medication to flow into the ear.
4.‐ The medication dropper or any other item should not be inserted into the ear canal.

2987 Which of the following actions observed by the nurse Correct answer: 1 The recommended wait time between administrations of two ophthalmic solutions is five The core issue of the question is an understanding of the procedure for safe self‐
demonstrates appropriate technique by a client self‐ minutes. If an ophthalmic ointment is instilled, the waiting time is ten minutes between the administration of an ophthalmic medication. Use nursing knowledge and the process of
administering an ophthalmic medication? ointment and the next medication. elimination to make a selection.

1.‐ The client waits five minutes between instillation of two different ophthalmic solutions.
2.‐ The client administers ophthalmic ointment immediately after administering ophthalmic solution.
3.‐ The client administers the second ophthalmic solution immediately after administering the first ophthalmic solution.
4.‐ The client administers ophthalmic solution immediately after administering ophthalmic ointment.
2988 The nurse is observing a client give a return Correct answer: 3 The eye is cleansed with sterile irrigating solution or sterile normal saline, to decrease risk of The wording of the question tells you that the correct option is an inappropriate action.
demonstration of the administration of eye drops. contamination. Options 1, 2, and 4 represent appropriate techniques demonstrated by the Analyze each option to decide if it is a true or false statement, and make the selection that
Which of the following actions taken by the client client. represents false information.
indicates a need for further teaching?
1.‐ The client pulls the lower lid of the eye down, forming a sac.
2.‐ The client instills the medication into the conjunctival sac.
3.‐ The client cleanses the eyelid with cotton balls moistened with warm tap water.
4.‐ The client cleanses the eye from inner canthus to outer canthus.

2989 A client with open‐angle glaucoma is receiving timolol Correct answer: 3 Timolol is a beta‐adrenergic blocker that decreases the production of aqueous humor, Note that the drug name ends in ‐olol, and reason that the medication is a beta‐blocking
(Timoptic) for treatment. When assessing the client’s thereby decreasing intraocular pressure. Sympathomimetics also decrease aqueous humor agent. With this in mind, recall the actions of beta‐blocker medications in the eye. Use
response to the medication, the nurse expects production. Prostaglandins increase the outflow of aqueous humor to decrease intraocular nursing knowledge and the process of elimination to make a selection.
therapeutic effects to be the result of which of the pressure.
following?
1.‐ A decrease in the outflow of aqueous humor
2.‐ An increase in the outflow of aqueous humor
3.‐ A decrease in aqueous humor production
4.‐ An increase in aqueous humor production

2990 The nurse is providing information on safety Correct answer: 1 Carbachol causes miosis (pupil constriction), making quick changes in illumination difficult. Note that options 1 and 2 are opposites. When two options are opposite, consider the
measures to the family of an elderly client being Nighttime is particularly hazardous for the elderly client. The client and family are instructed possibility that one of them is the correct answer. In this case, note that the client is elderly,
treated with carbachol (Carboptic), an ophthalmic on methods such as lighting hallways and bathrooms at night, to reduce the potential for and is not in a situation (such as an eye exam) when the pupils would be dilated. With this
cholinesterase inhibitor. The safety measures injury. Mydriasis (option 2) is not a concern. Systemic side effects of carbachol include diarrhea in mind, choose option 1 over 2.
implemented are related to which of the following? (option 3) and hypotension (option 4).

1.‐ The client will experience difficulty in making quick changes in illumination due to miosis.
2.‐ The client will experience difficulty in making quick changes in illumination due to mydriasis.
3.‐ The client will experience a side effect of constipation.
4.‐ The client will experience a side effect of hypertension.

2991 The nurse is developing a plan of care for a client Correct answer: 2 Carbonic anhydrase inhibitors produce increased urinary elimination and subsequent The core issue of the question is knowledge of what to monitor regarding side effects of
receiving a carbonic anhydrase inhibitor for treatment increased excretion of potassium. Clients are monitored for fluid volume deficit (not excess, as carbonic anhydrase inhibitors. Use medication knowledge and the process of elimination to
of glaucoma. The nurse identifies that the client is at in option 1) and hypokalemia (options 3 and 4). Assess electrolytes; intake and output; daily make a selection. Note also that options 1 and 2 are opposites, which suggests that one of
risk for which of the following nursing diagnoses? weights; mucous membranes; and skin turgor. them might be correct.

1.‐ Excess Fluid Volume


2.‐ Deficient Fluid Volume
3.‐ Electrolyte Imbalance: Hyperkalemia
4.‐ Electrolyte Imbalance: Hypocalcemia

2992 A client who just self‐administered the first dose of Correct answer: 2 Redness and swelling are signs of hypersensitivity to vidarabine. The medication should be Note that the question contains key information about adverse effects that began after
vidarabine (Vira‐A) calls the clinic and reports eye discontinued, and the client should return to the clinic immediately for evaluation. Options 1, the first dose of the medication. When symptoms suddenly appear, as in this question,
redness and swelling not present before treatment 2, and 4 are incorrect because they place the client at risk. consider the possibility of a hypersensitivity reaction, and choose an option accordingly.
began. The nurse instructs the client to take which of
the following actions?
1.‐ No action is necessary, because these are normal signs and symptoms of the medication.
2.‐ Discontinue use of the medication, and return to clinic immediately for evaluation.
3.‐ If redness continues after three days, return to the clinic for evaluation.
4.‐ Discontinue use of the medication, and return to clinic at the next scheduled appointment.
2993 Which of the following statements made by a client Correct answer: 4 Viroptic, used in the treatment of viral infections such as herpes, is administered for an The core issue of the question is knowledge of appropriate information about Viroptic as
being treated with ophthalmic trifluridine (Viroptic) additional 5–7 days after healing has occurred. Ophthalmic medications are stored in a cool, an ophthalmic medication. Use nursing knowledge and the process of elimination to make a
indicates an understanding of the medication dry place (option 3), and some are recommended for refrigeration (check label for selection.
instructions? instructions). Options 1 and 2 are incorrect because their time frames are too limited.

1.‐ “I will stop the medication once healing has occurred.”


2.‐ “I will administer the treatment for 7 days.”
3.‐ “I will store the medication in a warm place.”
4.‐ “I will continue the medication for 5–7 days after healing has occurred.”

2994 The parent of a 2‐year‐old child exhibits correct Correct answer: 1 The child’s pinna is pulled down and back for administration of otic solutions. The pinna in the One quick way to remember the direction for pulling the pinna is to associate the direction
administration technique for otic solutions when doing adult is pulled up and back. Droppers should never be inserted into the ear canal, and the head with the height of the person. Since an adult is taller, pull the pinna up and back, while for a
which of the following in a return demonstration? is tilted toward the unaffected side. child, who is shorter, pull the pinna down and back.

1.‐ The parent pulls the child’s pinna down and back before administering the medication.
2.‐ The parent pulls the child’s pinna up and back before administering the medication.
3.‐ The parent places the dropper into the child’s ear canal before administering the medication.
4.‐ The parent tilts the head of the child towards the affected side before administering the medication.

2995 A client in the rural health clinic complains of Correct answer: 3 Acetazolamide, a carbonic anhydrase inhibitor, causes diuresis. The nurse should instruct the The core issue of the question is recognition of and client teaching to prevent nocturia, a
frequent urination during the night. Upon evaluation, client to take the medication early in the day, to avoid nocturia. Clients receiving side effect of carbonic anhydrase inhibitors. Use medication knowledge and general
the nurse suspects that which of the following actions acetazolamide are encouraged to consume at least 2,000 mL of fluid per day to avoid fluid principles for timing the administration of diuretics to answer the question.
by the client taking acetazolamide (Diamox) is likely volume depletion, and acetazolamide may be taken with juice or food to minimize
the cause of the nocturia? gastrointestinal irritation.

1.‐ The client takes oral Diamox every morning.


2.‐ The client consumes 2,000 cc of fluid per day.
3.‐ The client takes oral Diamox before supper.
4.‐ The client takes oral Diamox with juice.

2996 The nurse is orienting a newly hired nurse to the Correct answer: 2 Proparacaine is administered to prevent pain during procedures such as tonometry and Note that the name of the drug ends in ‐caine to help remember that this drug has an
outpatient ophthalmic clinic. The nurse concludes that removal of foreign bodies. The medication has a rapid onset, within 20 seconds, and duration anesthetic action. With this in mind, choose the option that best protects an eye that has
the orientee understands instructions for of 15–20 minutes. Options 1, 3, and 4 are incorrect actions taken by the nurse because of the no sensation, which in this case is the option that utilizes the medication just prior to the
administering ophthalmic anesthetics for tonometry time frames identified. exam.
after observing which of the following actions?

1.‐ The orientee administers proparacaine hydrochloride (Ophthaine) 15 minutes before the scheduled tonometry.
2.‐ The orientee administers Ophthaine immediately before the scheduled tonometry.
3.‐ The orientee administers Ophthaine 5 minutes before the scheduled tonometry.
4.‐ The orientee administers Ophthaine after the tonometry is completed.

2997 Which of the following is the priority in nursing Correct answer: 3 Assessment of allergies and reactions to medications is essential when administering a new Knowing that one of the safety factors prior to administering any medication is assessing
management of the client prior to administering the medication. Hypersensitivity responses can occur with ophthalmic medications, and severe for any allergies or hypersensitivities will lead to the correct option. Recognize that while
first dose of an ophthalmic medication? adverse reactions can occur with hypersensitivity to the medication, because it is systemically the other options are important, option 3 is the priority action.
absorbed. Options 1, 2, and 4 are important to the nursing management of the client;
however, avoiding reactions to the medication is the priority.

1.‐ Assessing the client's understanding of the purpose of the medication


2.‐ Assessing the client's eye and vision status
3.‐ Assessing the client's history of hypersensitivity to medications
4.‐ Assessing the client's understanding of the action of the medication
2998 Which of the following techniques performed by the Correct answer: 2 Correct technique for administration of ophthalmic medications includes pulling the lower Use the process of elimination and knowledge of basic care in the instillation of eye
client demonstrates an understanding of appropriate eyelid down and instilling the medication into the conjunctival sac. Options 1, 3, and 4 each medications to be led to option 2. Review the procedure for instillation of eye medication if
administration of ophthalmic medications? contain information that is either partially or totally incorrect. this was difficult.

1.‐ Pulls the lower lid down, and instills the medication directly onto the eye.
2.‐ Pulls the lower lid down, and instills the medication into the conjunctival sac.
3.‐ Pulls the lower lid up, and instills the medication directly onto the eye.
4.‐ Pulls the lower lid up, and instills the medication into the conjunctival sac.

2999 The external ear canal of a client with an ear infection Correct answer: 1 For an external ear canal obstructed with edema, a gauze wick is inserted past the edematous Note that the question stem asks for selection of a technique. The only techniques listed
is obstructed with edema. Which of the following segment. The medication is then applied to the outside wick, allowing the medication to be are option 1 and 4. Using the process of elimination, and considering safety, option 1 is the
techniques does the nurse instruct the client to use absorbed along the path of the wick. Option 2 delays treatment. Option 3 is unnecessary, and only correct answer.
regarding medication administration? option 4 is a hazardous activity that could cause damage to the client's ear.

1.‐ Insert a gauze ear wick, and apply medication to wick.


2.‐ Wait until swelling subsides before instilling medication.
3.‐ Request a change in the route of medication.
4.‐ Insert the dropper past the edematous canal.

3000 A client with open‐angle glaucoma is being treated Correct answer: 3 Acetazolamide should not be mixed with alcohol or glycerin. To minimize gastrointestinal In order to answer this question correctly, recall the nursing interventions related to client
with oral acetazolamide (Diamox). Which of the distress, the client may take the medication with milk, or may crush it and mix it with juice. safety. If this was difficult, review the nursing interventions related to the medication
following statements made by the client indicates a Acetazolamide is taken in the morning to avoid nocturnal diuresis. acetazolamide (Diamox).
need for further teaching?
1.‐ "I can take the medication with milk."
2.‐ "I should take the medication in the morning."
3.‐ "I can mix the medication with alcohol."
4.‐ "I can crush the tablet, and mix it in juice."

3001 A client being treated with dorzolamide (TRUSOPT) as Correct answer: 1 Carbonic anhydrase inhibitor agents such as dorzolamide decrease aqueous production by Answer this question by recalling the pathophysiology of glaucoma. The question is not
treatment for glaucoma asks for an explanation of how approximately one‐half of baseline, thereby lowering intraocular pressure. Dorzolamide does specific about the type of glaucoma, so remember a general understanding of the disease
the medication will affect the disease. The nurse's not cause pupil constriction (option 2), increase aqueous humor production (option 3), or process. With this knowledge eliminate all the options except 1, which is the goal behind
response includes which of the following? increase outflow of aqueous humor (option 4). the treatment for clients with glaucoma.

1.‐ The medication decreases production of aqueous humor.


2.‐ The medication causes pupil constriction.
3.‐ The medication increases the production of aqueous humor.
4.‐ The medication increases the outflow of aqueous humor.

3002 A client is describing symptoms experienced since Correct answer: 3 Precipitation of an asthmatic attack is a systemic side effect of pilocarpine. Other side effects With knowledge of the side effects of the medication pilocarpine (Isopto Carpine), select
beginning pilocarpine (Isopto Carpine) for treatment of include salivation, hypotension, diarrhea, nausea, and vomiting. Dry mouth (option 1), option 3. This is the only correct answer, and can be selected with knowledge of the side
glaucoma. The nurse concludes that which of the hypertension (option 2), and constipation (option 4) are opposites of known side effects. effects of the medication.
following symptoms indicates a side effect from
systemic absorption?
1.‐ Dry mouth
2.‐ Hypertension
3.‐ Exacerbation of asthma
4.‐ Constipation
3003 The nurse concludes that an adult client understands Correct answer: 2 The adult client pulls the pinna up and back for administration of otic solutions. The pinna is Note that the question stem asks for validation of a client s technique for instilling ear
proper otic medication administration after observing pulled down in the child (option 1). Droppers should never be inserted into the ear canal drops. Using the process of elimination and basic care knowledge of the instillation of ear
the client use which of the following techniques for (option 3), and the head should be tilted toward the unaffected side (option 4). drops will assist in selecting the correct answer.
administering an otic solution?

1.‐ The client pulls the pinna down and back before administering the medication.
2.‐ The client pulls the pinna up and back before administering the medication.
3.‐ The client places the dropper into the ear canal before administering the medication.
4.‐ The client tilts the head towards the affected side before administering the medication.

3004 The nurse determines that a client with newly Correct answer: 1 Ophthalmic beta‐blockers are administered to reduce intraocular pressure by decreasing Answer this question with knowledge of the pathophysiology of glaucoma. The question is
diagnosed glaucoma understands the purpose for the production of aqueous humor. The medication must be continued as lifelong therapy to not specific about the type of glaucoma, so recall a general understanding of the disease
prescribed ophthalmic beta‐blocker when which of the maintain a stable intraocular pressure (option 2). Some glaucoma may be surgically treated process. Use this knowledge to eliminate all the options except that 1, which is the goal
following statements is made? (option 4). behind the treatment for clients with glaucoma. Options 2 and 4 can be eliminated because
they do not address what the question is asking.

1.‐ "The medication is given to reduce my intraocular pressure."


2.‐ "I can stop the medication once my intraocular pressure is normal."
3.‐ "The medication is given to increase my intraocular pressure."
4.‐ "This medication is the only treatment available for glaucoma."

3005 As a nurse working in an outpatient surgical clinic, Correct answer: 1 Atropine, an anticholinergic agent, can precipitate acute glaucoma as a result of pupillary Understanding the pathophysiology with glaucoma will assist in selecting the correct
which of the following preoperative medications dilation; therefore, clients with pre‐existing glaucoma or a predisposition to acute glaucoma answer. Also recall the medications that are contraindicated for a client who has been
should be questioned for a client with a history of should not receive atropine. There are no contraindications for diphenhydramine, hydroxyzine, diagnosed with acute glaucoma.
glaucoma? or promethazine in the client with glaucoma.
1.‐ Atropine (generic)
2.‐ Diphenhydramine (Benadryl)
3.‐ Hydroxyzine (Vistaril)
4.‐ Promethazine (Phenergan)

3006 A client who has begun taking brinzolamide (Azopt) Correct answer: 2 Carbonic anhydrase inhibitors such as brinzolamide can exacerbate the potential for renal In order to select the correct answer to this question, recall the mechanism of excretion
indicates understanding of medication instructions calculi. Increasing fluid intake to two liters per day could reduce this risk. Diet with the medication. Since it is excreted via the kidney, there is a higher incidence of side
when making which of the following statements? recommendations include increasing potassium and reducing sodium. effects with the renal system. Teaching the client this side effect will encourage the client
to consume more fluids, leading to the correct option, 2.
1.‐ "I will reduce my daily fluid intake."
2.‐ "I will consume two liters of fluid daily."
3.‐ "I will consume a diet high in sodium."
4.‐ "I will consume a diet low in potassium."

3007 A client with type 1 diabetes mellitus is ingesting Correct answer: 2 If the client is monitored properly, the beta‐blocking agents will not mask symptoms of Determine the relationship between Ocupress and the needs of a client with type 1
carteolol (Ocupress). Which of the following outcomes hypoglycemia, but might reduce the effect of some oral hypoglycemics (option 2). A serum diabetes to select the correct answer.
is considered to have the most immediate significance, glucose level significantly lower than normal can place the client at great risk more rapidly
and should be given first priority? than does elevated blood pressure. Many clients live for long periods with hypertension
without immediate risks to their health (option 1). NSAIDs can reduce the antihypertensive
effect of the drug (option 3). The diarrhea needs to be controlled because of the potential fluid
and electrolyte loss, but is not the first priority (option 4).

1.‐ Blood pressure is within normal limits (WNL).


2.‐ Serum glucose is within normal limits (WNL).
3.‐ Headache is controlled with non‐steroidal anti‐inflammatory drugs (NSAIDs).
4.‐ Diarrhea is controlled with loperamide (Imodium).
3008 A client indicates an understanding of instructions Correct answer: 2 Warming eardrops (if not contraindicated) makes administration of the medication more Using the process of elimination, and knowledge of basic care in the instillation of eye
about self‐administration of the prescribed otic comfortable. Warming can be achieved by running the bottle under warm water, placing the medications, will lead you to option 2. Review the procedure for instillation of eye
solution when she makes which of the following bottle of medication in a cup of warm water (not cool, as in option 1), or by carrying in the medication if this was difficult.
statements? hand or pocket for 30 minutes (option 4). The medication should never be warmed in the
microwave (option 3); serious injury to ear canal and tympanic membrane could occur.

1.‐ "I run the bottle of medication under cool running water before administering the medication."
2.‐ "I run the bottle of medication under warm running water before administering the medication."
3.‐ "I warm the bottle of medication in the microwave before administering the medication."
4.‐ "I warm the bottle of medication in my hand for five minutes before administering the medication."

3009 A client with a history of pulmonary disease is being Correct answer: 4 Clients with pulmonary disease are generally prescribed Betoptic for glaucoma because it is Note that option 4 is the only one that addresses the client s question. The other choices
treated with betaxolol (Betoptic). After the nurse has Beta&lt;sub&gt;1&lt;/sub&gt; selective (cardioselective). However, the client must still be do not address the client s question, and should be eliminated.
provided instructions and information about the monitored for pulmonary side effects and respiratory difficulties that can occur with systemic
medication, the client asks, "How can eye drops affect absorption. The explanation in options 1, 2, and 3 do not address this effect.
my lungs?" The nurse's explanation includes which of
the following?
1.‐ The medication does not have any effects on the pulmonary system.
2.‐ The client is only at risk if the prescribed ophthalmic medication is cardioselective (Beta1).
3.‐ The client is only at risk if the prescribed ophthalmic medication is given at the same time as the oral medications taken for pulmonary disease.
4.‐ If the ophthalmic medication is systemically absorbed, it can have the same systemic effects as other beta‐blocking agents.

3010 A client telephones the outpatient clinic and Correct answer: 1 The symptoms the client reports might indicate a ruptured tympanic membrane. The ear Using the process of elimination, and knowledge of basic care in the instillation of ear
complains of severe ear pain that ceased suddenly. canal and tympanic membrane should always be evaluated before instilling otic medications, medications, will lead you to option 1. Knowledge that medication previously prescribed for
Now, the ear is draining. The client has otic antibiotics making options 2, 3, and 4 incorrect. a condition should not be used for another condition will lead to option 1.
remaining from a previous ear infection two months
ago, and wants to know if it is safe to use the
medication. The nurse's response is based on which of
the following?
1.‐ The client should be referred to a health care provider, and should not use any medications until the ear is evaluated.
2.‐ Since the client was recently treated with otic antibiotics for an ear infection, the medication would be safe to use.
3.‐ The client should begin using the antibiotic, and seek evaluation if no improvement is seen within two days.
4.‐ The shelf life of otic medications is three months; therefore, the medication would be safe to use.

3011 The nurse evaluates that a client is demonstrating Correct answer: 2 Crust from eyes is cleansed using cotton balls, wiping from the inner canthus to the outer Using the process of elimination, and knowledge of basic care in the instillation of eye
appropriate technique for using ophthalmic canthus. Swabs (options 3 and 4) should not be used, as damage to the eye could occur. Option medications, will lead to option 2. Review the procedure for instillation of eye medication if
medication when the client does which of the 1 represents incorrect technique. this was difficult.
following?
1.‐ Cleanses crust from the eye by wiping from the outer canthus inward with a cotton ball.
2.‐ Cleanses crust from the eye by wiping from the inner canthus outward with a cotton ball.
3.‐ Cleanses crust from the eye by wiping from the outer canthus inward with a cotton swab.
4.‐ Cleanses crust from the eye by wiping from the inner canthus outward with a cotton swab.

3012 A client prescribed gentamicin sulfate (Garamycin) for Correct answer: 2 Otic Garamycin is not approved for use in the United States. It is a safe and accepted practice Note that option 2 is the only one that addresses the client s question. Options 1 and 3 are
an ear infection telephones the clinic, states that the for clinicians to prescribed ophthalmic Garamycin for otic use. The client should be informed of erroneous statements, and option 4 does not address the client s question, and should be
medication bottle indicates "for ophthalmic use," and this practice. Options 1 and 3 are incorrect because no error was made. Option 4 is incorrect eliminated.
refuses to use the medication. The nurse s response to because the client has not indicated inadequate knowledge of medication administration.
the client is based on which of the following?

1.‐ An error is likely in the dispensing of the medication, since the clinician is treating an otic infection.
2.‐ It is an accepted and safe practice in the United States for clinicians to prescribe ophthalmic Garamycin for otic use.
3.‐ An error was likely committed by the clinician in prescribing the medication.
4.‐ The client requires further teaching on proper medication administration.
3013 A client with a history of cardiovascular disease is Correct answer: 1 Carteolol is a beta‐blocking agent with side effects of hypotension and bradycardia if Knowing that carteolol (Ocupress) is classified as a beta‐blocker will assist in selecting the
admitted to the nursing unit with bradycardia and systemically absorbed. The other medications acetazolamide and dorzolamide, carbonic correct answer. This medication will affect the heart rate. The other medications do not
hypotension. The nurse suspects that these symptoms anhydrase inhibitors, and latanoprost, a prostaglandin do not affect heart rate and blood affect the heart rate.
might be adverse effects of which medication recently pressure.
started as therapy for glaucoma?

1.‐ Carteolol (Ocupress)


2.‐ Acetazolamide (Diamox)
3.‐ Dorzolamide (TRUSOPT)
4.‐ Latanoprost (Xalatan)

3014 A client is receiving an ophthalmic anesthetic agent Correct answer: 4 The blink reflex is lost when ophthalmic anesthetic agents are used; therefore, the eye is at Note that the question stem includes the word “ophthalmic.” Recognize that the eye is
preoperatively for removal of sutures. Priority nursing risk for injury. Priority is given to protecting the cornea from irritants, debris, and rubbing. being referred to in order to select the correct answer, 4, which lists eye in the answer
care includes which of the following? Generally, an eye patch is applied for protection. Since the medication is local, and the client is statement.
not anesthetized, the airway is not compromised (option 1), and the body temperature should
remain at preprocedure reading (option 3). Clients are assessed for allergies or past
hypersensitivity reactions (option 2) before the medication is administered.

1.‐ Measures to protect the airway


2.‐ Measures to reduce hypersensitivity
3.‐ Measures to control body temperature
4.‐ Measures to protect the eye

3015 A client with narrow‐angle glaucoma informs the Correct answer: 1 Atropine sulfate is commonly used preoperatively in outpatient procedures such as a Recall knowledge of medications used during colonoscopies to assist in selecting the
nurse of an outpatient colonoscopy scheduled for later colonoscopy. The client needs to alert the staff about the diagnosis of glaucoma, since the use correct answer. Option 1 would be the best choice, since it is most important to inform the
in the week. The client demonstrates understanding of of atropine is contraindicated in narrow‐angle glaucoma because it could precipitate acute practitioner of any medical problems being treated, as well as the medications prescribed.
subsequent teaching when which of the following glaucoma. This would be the priority response in this question.
statements are made?

1.‐ "I will inform my doctor and the nursing staff of my glaucoma and the medication I am taking."
2.‐ "I will stop taking my medication two days before the colonoscopy."
3.‐ "I will stop taking my medication one day before the colonoscopy."
4.‐ "My glaucoma is not a factor when having outpatient procedures done."

3016 Which action by the client indicates an understanding Correct answer: 1 Maintaining pressure on the lacrimal sac for 1–2 minutes is recommended for dipivefrin, to Use the process of elimination and knowledge of basic care in the instillation of eye
of instructions for administration of dipivefrin minimize systemic absorption of the medication. Eye drops are instilled into the conjunctival medications to select option 1. Review the procedure for instillation of eye medication if
(Propine)? sac, never directly onto the eye. this was difficult.
1.‐ The client maintains pressure on the lacrimal sac for 1–2 minutes after instillation of medication.
2.‐ The client avoids lacrimal pressure after instillation of medication.
3.‐ The client instills medication directly onto the eye.
4.‐ The client maintains pressure on lacrimal sac for 30 seconds after instillation of medication.

3017 While teaching a client about the proper Correct answer: 3 To promote absorption, the client should not blink for 30 seconds after the administration of Use the process of elimination and knowledge of basic care in the instillation of eye
administration of dipivefrin (Propine), the nurse would dipivefrin. Options 1, 2, and 4 are incorrect for the administration of dipivefrin. medications to select option 3. Review the procedure for instillation of eye medication if
provide which of the following instructions? this was difficult.

1.‐ Gently squeeze eyes closed for 30 seconds immediately after instillation of medication.
2.‐ Close, but do not squeeze, the eyes immediately after instillation of medication.
3.‐ Do not blink for 30 seconds after instillation of medication.
4.‐ Close the eyes for one full minute after instillation of medication.
3018 A client receiving hydroxyamphetamine (Paredrine) Correct answer: 4 Confusion and increased heart rate are signs of toxicity or adverse side effects of In order to answer this question correctly, recall the toxic side effects of the medication.
for open‐angle glaucoma demonstrates an hydroxyamphetamine. Stinging, headache, and brow ache are usual side effects of Option 2 and 3 can be eliminated, since there is only an “occasional” problem. Option 4
understanding of the medication's serious side effects hydroxyamphetamine. could indicate a central nervous system problem that could indicate toxicity. Review the
when informing the health care provider of which of high‐risk complications of the medication if this was difficult.
the following symptoms?

1.‐ Stinging on instillation


2.‐ Occasional headache
3.‐ Occasional brow ache
4.‐ Confusion

3019 A client is scheduled for an ophthalmic examination. Correct answer: 3 Ophthalmic epinephrine is used to produce mydriasis for ocular examination. Dilation of pupil Recall the pathophysiology of glaucoma to answer this question. The question is not
Before administering the prescribed epinephrine further constricts ocular fluid outflow, possibly causing an acute attack of glaucoma in a client specific about the type of eye condition, but knowing that an epinephrine solution results in
solution, the nurse would assess for which of the with narrow‐angle glaucoma. Systemic absorption also causes hypertension and tachycardia. dilation will enable consideration of glaucoma and the complication associated with it. If
following conditions? Brow ache is a typical side effect of adrenergic agonists such as epinephrine (option 4). this was difficult, review the general principles associated with an ophthalmic examination.

1.‐ Hypotension
2.‐ Wide‐angle glaucoma
3.‐ Narrow‐angle glaucoma
4.‐ Brow ache

3020 Which of the following statements by a client Correct answer: 1 Clients receiving ophthalmic corticosteroids have an increased risk of infection. Contact Note two items to consider when selecting the correct option to this question: safety, and
receiving ophthalmic corticosteroids indicates a need lenses should not be used during ophthalmic corticosteroid therapy. Options 2, 3, and 4 which of the statements indicates the client does not understand the teaching. Options 2, 3,
for further teaching? indicate an appropriate understanding of ophthalmic corticosteroid therapy. and 4 indicate that the client understands safe care during eyedrop instillation. Option 1
should be the selection, since this indicates a lack of safe care and the client not
understanding eye drop instillation.
1.‐ "I remove my contact lenses before instilling the medication, then put them back in after 30 minutes."
2.‐ "I am not wearing my contact lenses for the duration of the corticosteroid treatment."
3.‐ "I will take my medication for the length of time prescribed by my physician."
4.‐ "I will return to my physician to have my eyes examined after my treatment is completed."

3021 Which of the following actions by the client indicates Correct answer: 1 Eyedrops are considered sterile. To reduce the chance of introducing organisms into the eye, Use the process of elimination and knowledge of basic care in the instillation of eye
an understanding of the proper administration the dropper should not touch the eye, eyelashes, or any other object. The dropper should not medications to select option 3. Review the procedure for instillation of eye medication if
techniques for ophthalmic solutions? be rinsed with water (option 2). The eyes should not be squeezed tightly shut after instilling this was difficult.
the medication (option 3). The medication is instilled into the conjunctival sac, not directly
onto the eye (option 4).
1.‐ The client does not allow the tip of the dropper to touch the eye or any other object.
2.‐ The client rinses the eyedropper with water after every application.
3.‐ The client squeezes her eyes tightly shut after administration of solution.
4.‐ The client pulls the lower lid down, and drops the solution onto the eye.

3022 A client complains of dizziness and nausea upon self‐ Correct answer: 1 The internal ear is sensitive to temperature extremes. Administration of cold medication into Use the process of elimination and knowledge of basic care in the instillation of ear
administration of ear drops. After discussing the the ear can cause dizziness and nausea. To avoid these conditions, the client should warm the medications to select option 1. Review the procedure for instillation of ear medication if
client's usual medication administration procedure, medication to body temperature before administering the eardrops (option 2). The dizziness is this was difficult.
the nurse determines that which of the following is unrelated to hypersensitivity (option 3) or speed of administration (option 4).
likely the cause of these symptoms?

1.‐ The client does not warm the medication bottle before administering the medication.
2.‐ The client warms the medication bottle before administering the medication.
3.‐ The client is experiencing a hypersensitivity reaction to the medication.
4.‐ The client is administering the medication too quickly.
3023 A client with open‐angle glaucoma is receiving Correct answer: 2 Latanoprost is a prostaglandin agonist that increases the outflow of aqueous humor to Recall the pathophysiology and usual treatment for glaucoma to correctly answer this
latanoprost (Xalatan) for treatment. When assessing decrease intraocular pressure (not decreasing outflow, as in option 1). Beta‐blockers, question. Use this knowledge about glaucoma to be able to eliminate all the options except
the client's response to the medication, the nurse adrenergics, and sympathomimetics decrease intraocular pressure by decreasing aqueous 2, which is the goal behind the treatment for clients with glaucoma.
expects therapeutic effects to be the result of which of humor production (option 3 and 4).
the following?

1.‐ A decrease in the outflow of aqueous humor


2.‐ An increase in the outflow of aqueous humor
3.‐ A decrease in aqueous humor production
4.‐ An increase in aqueous humor production

3024 A client receiving ophthalmic scopolamine Correct answer: 2 Dry mouth could be a sign of toxicity in the client receiving scopolamine hydrobromide. Recall the side effects and client responses to the medication. Recognize that that the
hydrobromide telephones the ophthalmic clinic to Tachycardia is also a sign of toxicity. For these symptoms, the client is instructed to withhold other three options do not address the complication of toxicity.
report a dry mouth. The nurse should instruct the the medication and seek immediate evaluation for toxicity. Options 1, 3, and 4 are incorrect
client regarding which of the following? instructions that place the client at risk.
1.‐ Dry mouth is a normal side effect of scopolamine hydrobromide.
2.‐ Withhold the medication, and return to the clinic immediately.
3.‐ Increase oral fluid intake to 2,000 mL per day.
4.‐ Dry mouth is not a side effect or a sign of toxicity for scopolamine hydrobromide.

3025 Which of the following statements made by the client Correct answer: 2 Systemic absorption of beta‐blocking agents such as carteolol can lead to serious Use the process of elimination and knowledge of basic care in the instillation of eye
receiving ophthalmic carteolol (Ocupress) indicates the cardiovascular and pulmonary side effects. Nasolacrimal pressure is applied to prevent medications to select option 2. Review the procedure for instillation of eye medication if
client understands the nurse's teaching about systemic absorption of ophthalmic medications. Options 1, 3, and 4 are incorrect techniques. this was difficult.
medication self‐administration?
1.‐ "I will put pressure on the lacrimal duct to prevent the medication from running out of my eye."
2.‐ "I will put pressure on the lacrimal duct to prevent systemic absorption of the medication."
3.‐ "I will not put pressure on the lacrimal duct."
4.‐ "I will put pressure on the lacrimal duct only if the medication runs out of my eye."

3026 A client being treated with dipivefrin (Propine) for Correct answer: 2 Dipivefrin lowers intraocular pressure by decreasing aqueous humor production and To correctly answer this question, recall the pathophysiology and usual treatment for
glaucoma asks for an explanation regarding how the increasing aqueous humor outflow. Options 1, 3, and 4 are either partially or totally incorrect. glaucoma. Use this knowledge about glaucoma to eliminate all the options except 2, which
medication will affect the disease. The nurse's is the goal behind the treatment for clients with glaucoma.
explanation includes that dipivefrin lowers intraocular
pressure by which of the following actions?

1.‐ A decrease in aqueous humor production and a decrease in the outflow of aqueous humor
2.‐ A decrease in aqueous humor production and an increase in the outflow of aqueous humor
3.‐ A decrease in aqueous humor production
4.‐ An increase in aqueous humor product

3027 The nurse explains to a client that a product Correct answer: 3 Alpha‐hydroxy acids are useful keratinolytics that help reduce the effects of photoaging. The core issue of the question is knowledge of products that assist the skin to appear
containing which of the following ingredients would be Propylene glycol is used to treat ichthyosis. Salicylic acid and resorcinol are keratinolytics that younger and resist the aging effects of light. Use knowledge of these ordinary products and
the most useful agent to treat photoaging of the skin? are used to treat a variety of other skin disorders. the process of elimination to make a selection.

1.‐ Propylene glycol


2.‐ Salicylic acid
3.‐ Alpha‐hydroxy acids
4.‐ Resorcinol
3028 The nurse would include which of the following pieces Correct answer: 4 Emollients contain petrolatum, oils, propylene glycol, or other substances, and make the skin The core issue of the question is general knowledge of integumentary products that are
of information when explaining the skin emollient soft and pliable by increasing hydration of the stratum corneum. They do not dry the skin emollients. Use knowledge of these ordinary products and the process of elimination to
Dermasil to a client? (option 2) or contain corticosteroids (option 3). Option 1 is not always necessary. make a selection.

1.‐ It requires shaking before each use.


2.‐ It has a drying effect on the skin when the water evaporates.
3.‐ It includes a corticosteroid component.
4.‐ It is of use when skin is dry.

3029 What instructions should the nurse give the client Correct answer: 4 Tretinoin is a retinoic acid derivative that needs to be applied once daily in a thin layer before The core issue of the question is knowledge of proper use of tretinoin. Use medication
who is receiving tretinoin (Retin‐A)? retiring. The area to be treated should be washed at least 30 minutes before applying. knowledge and the process of elimination to make a selection.
Increased intake of vitamin A, not vitamin C, needs to be avoided.
1.‐ Apply the preparation in the morning.
2.‐ Use gloves, and apply a thick layer four times a day.
3.‐ Avoid products containing vitamin C.
4.‐ Apply to dry skin 30 minutes after washing.

3030 The nurse anticipates that mafenide (Sulfamylon) Correct answer: 1 Mafenide is useful in treatment of partial‐ and full‐thickness burns to prevent septicemia The core issue of the question is knowledge of the uses of mafenide in a client with burn
would be ordered for use if it is known that a client’s caused by organisms such as Pseudomonas aeruginosa. Mafenide does not have a defined use injury. Use medication knowledge and the process of elimination to make a selection.
burn is infected with which of the following with the other infectious organisms mentioned.
organisms?
1.‐ Pseudomonas aeruginosa
2.‐ Tubercle bacillus
3.‐ Methicillin‐resistant Staphylococcus aureus
4.‐ Candida albicans

3031 A female client who is using salicylic acid to treat Correct answer: 2 There is no cure for psoriasis. Psoriasis is notoriously chronic and recurrent. The cause is The core issue of the question is general knowledge about medications used to treat
psoriasis asks how long she will have to use this drug. unknown. Each situation is individual, and the dermatologist who knows the client s situation psoriasis. Use medication knowledge and the process of elimination to make a selection.
Which of the following would be the best response by the longest is a good resource, but nevertheless, the best answer for most clients is that the
the nurse? disease is recurrent, and therapy will need to be continued.

1.‐ “Response is rapid, and the drug will not be needed after three months of therapy.”
2.‐ “Drugs often do not produce prolonged remission, and maintenance therapy often is needed.”
3.‐ “Each situation is so individual that it is not possible to answer the question accurately.”
4.‐ “The dermatologist caring for you is the best resource for such a question.”

3032 The nurse would recommend that a client with Correct answer: 2 A 1% lotion of selenium sulfide is used to relieve the itching and flaking of the scalp The core issue of the question is general knowledge about medications used to treat
excessive dandruff use a medicated shampoo that associated with dandruff. A shampoo with lindane 1% (Kwell) would be used for pediculosis dandruff. Use medication knowledge and the process of elimination to make a selection.
contains which of the following active ingredients? capitis. Corticosteroids can be used for many things, but dandruff is not one of them. Silver
sulfadiazine is a cream used in the prevention and treatment of infection in partial‐ and full‐
thickness burns.
1.‐ Silver sulfadiazine
2.‐ Selenium sulfide
3.‐ Corticosteroid
4.‐ Lindane
3033 It is winter, and the client has extremely dry skin. Correct answer: 3 Dry skin can occur in otherwise healthy skin, and is usually worse in winter, when forced‐air The core issue of the question is general knowledge about products used to treat dry skin.
Which type of preparation should the nurse heating reduces humidity inside many dwellings. Excessive washing with harsh soaps (such as Use product knowledge and the process of elimination to make a selection.
recommend first? Dial) strips stratum corneum of its natural lipids, and exacerbates dry skin. No shake lotion is
made specifically for management of dry skin. Itching can occur with dry skin, but before an
antipruritic lotion is used, an emollient lotion or emollient should be tried. Emollient lotions
are dilute dispersions of emulsified lipids in water. These provide smooth application and the
most rapid hydration if applied to dry skin, but they do not provide a protective effect on the
lipid barrier. Emollients (e.g., petrolatum) are occlusive agents that make the skin soft and
pliable by increasing hydration of the stratum corneum.

1.‐ Regular use of soap, such as Dial


2.‐ Shake lotion
3.‐ Emollient or emollient‐containing lotion
4.‐ Antipruritic lotion

3034 Which of the following client disorders might require Correct answer: 1 Acyclovir is an antiviral agent that is useful in the treatment of herpes simplex viruses. The The core issue of the question is knowledge of the uses of acyclovir in a client with herpes
the use of acyclovir (Zovirax)? other conditions would require therapy with an anti‐infective, but not of the antiviral type. infection. Use medication knowledge and the process of elimination to make a selection.
Remember that an antiviral medication often contains vir somewhere in its name.

1.‐ Herpes simplex viruses


2.‐ Chronic dermatitis
3.‐ Pseudofolliculitis
4.‐ Candidiasis

3035 A child has scraped his finger on a sharp spot on a Correct answer: 1 Bacitracin is a topical antibiotic that is bactericidal against Gram‐positive cocci and bacilli, The core issue of the question is knowledge of the types of medications used for various
shower door edge. The mother would like to use a including staphylococci and streptococci. These organisms might cause infection in a skin skin conditions. Use medication knowledge and the process of elimination to make a
topical antibiotic to prevent infection. Which agent wound. Malathion (option 2) is an antiparasitic agent for pediculosis. Ketoconazole (option 3) selection. Remember that cuts or open wounds often heal effectively when topical
would the pediatric telephone consultation nurse is an antifungal agent. Mafenide (option 4) is an agent used for burns. antibiotics are used to prevent infection at the site.
recommend?
1.‐ Bacitracin (Baciguent Topical)
2.‐ Malathion (Ovide Lotion)
3.‐ Ketoconazole (Nizoral)
4.‐ Mafenide (Sulfamylon)

3036 The elderly client is being treated for a pressure ulcer. Correct answer: 4 Ulcers with necrotic material should be debrided, either by sharp debridement (e.g., using a The core issue of the question is the type of topical agent to use when debridement is
The nurse would anticipate use of which of the scalpel) or chemical debridement (e.g., wound cleanser, such as an enzyme). An example of needed. Use medication knowledge and the process of elimination to make a selection.
following types of agents to topically debride this such a preparation is collagenase (Santyl), which is inactivated by metal salts, Enzymes often end in ase, which makes them easy to recognize on sight.
ulcer? hexachlorophene, or acidic solutions. Hydrocolloid dressings can be helpful with uninfected
wounds with fibrinous bases. Topical antibiotics will not help remove necrotic material.
Allylamines are selected for fungal infections.

1.‐ Hydrocolloid dressing


2.‐ Antibiotic‐impregnated gauze packing
3.‐ Allylamine
4.‐ Enzyme

3037 The nurse is preparing to do tracheostomy care, and Correct answer: 2 Hydrogen peroxide is an oxidizing antiseptic that can be used to clean wounds or The core issue of the question is knowledge of the type of skin cleansing agent used for
notes that the client s tracheostomy has encrusted tracheostomy tubes. Options 1 and 3 as cleaning agents do not have the bubbling action of tracheostomy. Use knowledge of these agents and the process of elimination to make a
debris around the tube. The nurse should dilute which hydrogen peroxide. Mafenide is an antimicrobial used to treat burn injury. selection.
of the following antiseptic solutions to half strength to
most effectively clean the skin around the
tracheostomy?
1.‐ Iodine
2.‐ Hydrogen peroxide
3.‐ Chlorhexidine
4.‐ Isopropyl alcohol

3038 The physician’s order sheet calls for topical Correct answer: 4 Proteolytic enzymes such as Elase ointment can be used to chemically debride tissue. These The core issue of the question is knowledge of the debriding agents appropriate for use at
application of the proteolytic enzyme Elase. The nurse areas commonly include venous stasis ulcers, burn wounds, and pressure ulcers. The areas various skin sites. Use knowledge of these agents and the process of elimination to make a
carries out this order by applying this product to which listed in the other options are not appropriate for treatment with proteolytic enzymes. selection.
of the following areas on the client?

1.‐ External ear canal


2.‐ Rectal area
3.‐ Dry skin on feet
4.‐ Sacral pressure ulcer

3039 The nurse explains to a client who seeks treatment Correct answer: 3 A keratinolytic agent such as salicyclic acid is used to treat warts. Keratinolytics are also used The core issue of the question is what type of medication is effective in treating warts.
for a wart that which of the following types of to treat corns, calluses, and other keratin‐containing skin lesions. Astringents cause topical Begin to answer by reasoning that treatment of a wart includes breaking it down for
products will be effective in removing this growth? vasoconstriction. Antiseptics inhibit bacterial growth. Proteolytic enzymes are used to debride removal. Next note the suffix ‐lytic in the correct option, which means “to break down.”
tissue.
1.‐ Astringent
2.‐ Antiseptic
3.‐ Keratinolytic
4.‐ Proteolytic

3040 The nurse would be most careful when using a topical Correct answer: 3 Children have an increased risk of systemic toxicity from topically applied drugs because of The core issue of this question is the age group that is at greatest risk because of its skin
drug for a client in which of the following age groups, the greater ratio of surface area to weight. The other responses are incorrect because they characteristics when topical drugs are used. Recall that the greater the area involved, the
because of increased risk of toxicity? have similar body surface area–to‐weight ratios. greater the risk of absorption and toxic effects. Finally, recall that infants and children have
a greater skin surface–to‐weight ratio than an adult of any age.

1.‐ Middle‐aged adult


2.‐ Older adult
3.‐ Child
4.‐ Adolescent

3041 Which of the following types of dermatological Correct answer: 1 Acne can be successfully treated with drying agents. Steroids would be of no benefit for this The core issue of the question is what type of medication is effective in treating acne.
products would the nurse recommend as having the problem. Emollients and lubricants are moisturizers that might worsen the condition. Since this condition is characterized by inflammation and drainage, consider that an agent
most benefit for a client with acne? that has a drying effect would be opposite to the characteristics of the condition, and would
help reduce symptoms.
1.‐ Drying agent
2.‐ Steroid
3.‐ Emollient
4.‐ Lubricant

3042 The nurse would evaluate for which of the following Correct answer: 4 Corticosteroids such as hydrocortisone are anti‐inflammatory drugs. They do not exert The core issue of the question is knowledge of the intended effects of corticosteroids as
effects in a client who has been using hydrocortisone antimicrobial action, and, in fact, they can increase risk of infection by suppressing the anti‐inflammatory agents. Use this information and the process of elimination to make a
1% cream (Ala‐Cort) as a topical agent? inflammatory response. Corticosteroids are not moisturizing or drying agents. selection.

1.‐ Moisturizing
2.‐ Drying
3.‐ Antimicrobial
4.‐ Anti‐inflammatory
3043 A client with psoriasis needs to apply a lubricating Correct answer: 1 Psoriatic plaques need to be lubricated so that they are easier to loosen and remove. Note the word lubricating in the stem of the question. This tells you that regardless of the
lotion to a psoriatic placque. The nurse plans to teach Emollients and lubricants are fatty or oily substances that can be used for this purpose because client s health problem, the agent is one that has a moisturizing effect on the skin. Use the
the client to use which of the following types of they keep skin soft, and prevent water evaporation. The other products listed are harsher, and process of elimination and knowledge of the categories of skin products to make a
substances? some might have a drying effect. selection.
1.‐ Emollient
2.‐ Antiseptic
3.‐ Alcohol
4.‐ Astringent

3044 The nurse prepares to apply which ordered type of Correct answer: 2 Silver sulfadiazine is a metallic type of antiseptic that is widely used on burns. The silver in the The core issue of the question is the type of agent that would be effective in preventing
antiseptic to a client with a burn wound once the area solution is toxic to bacteria, and prevents them from reproducing. The agents in options 3 and microbial growth in a client with a burn injury. Use knowledge of topical antimicrobial
has been cleansed with sterile saline? 4 would not be beneficial. Option 1 is a fictitious solution. agents and the process of elimination to make a selection.
1.‐ Copper‐containing
2.‐ Silver‐containing
3.‐ Biguanide
4.‐ Acetic acid

3045 The nurse anticipates that a wound infection that is Correct answer: 4 A chlorine preparation such as Dakin s solution is used for infected wounds when other The core issue of the question is knowledge of antiseptics that are useful in treating
resistant to treatment with several antiseptics will treatments are ineffective. They are useful because they also dissolve necrotic materials and problems involving the skin. Use this knowledge and the process of elimination to make a
most likely respond to treatment with which of the blood clots; however, a disadvantage is that they delay blood clotting, which later could selection.
following antiseptics? interfere with wound healing. Options 1, 2, and 3 are not helpful in treating infections resistant
to several antiseptics.
1.‐ Hydrogen peroxide
2.‐ Phenol derivative
3.‐ Isopropyl alcohol
4.‐ Dakin’s solution

3046 The nurse is choosing a protective wound dressing for Correct answer: 1 Tegaderm is a protective dressing that is permeable to oxygen. The other products listed do The core issue of the question is the type of dressing that is permeable to oxygen. Use the
a client. Which of the following products should be not have this advantage. DuoDERM and Tegasorb are absorbent products that exclude bacteria process of elimination and knowledge of wound care products to make a selection.
used when selecting a dressing that is permeable to and adhere to the site. Replicare excludes bacteria.
oxygen?
1.‐ Tegaderm
2.‐ DuoDERM
3.‐ Replicare
4.‐ Tegasorb

3047 The nurse is working with a client newly diagnosed Correct answer: 1 Psoriasis is a common, chronically recurring skin disease with scaly patches of varying size In selecting the best option for this question, it is important to be as complete as possible
with psoriasis. The client asks the nurse about this most commonly seen on elbows, knees, and scalp. Pityriasis rosea is a common skin disorder with responses to the client. Option 1 is the only correct answer; the other options could be
disorder. Which of the following responses by the generally localized to the chest and trunk of young adults, and characterized by erythematous eliminated because they are not complete. For example, option 3 lists the word disease,
nurse is best? discrete lesions (option 2). Option 3 is a vague description unrelated to psoriasis. Contact which is not the case with a skin disorder.
dermatitis (option 4) develops after exposure to an irritant or allergen.

1.‐ "It is a chronic skin disorder characterized by whitish, scaling patches."


2.‐ "It is a disorder with reddish‐colored lesions of the chest and trunk that usually do not recur."
3.‐ "It is a skin disease characterized by redness, tenderness, and edema."
4.‐ "It is a contact dermatitis that can involve any part of body."

3048 A child has been diagnosed as having impetigo. The Correct answer: 2 Mupirocin is a topical antimicrobial agent effective against impetigo caused by Knowing the causative agent for the diagnosis will assist in selecting the correct answer.
nurse anticipates that which topical agent will be Staphylococcus aureus, beta‐hemolytic streptococci, and Streptococcus pyogenes. Recall the classification of the medications in determining which is the best option. If this
prescribed? Ketoconazole is an antifungal agent; capsaicin is a topical agent that has been useful in certain was difficult, review the diagnosis and usual method of treatment.
painful syndromes; and acyclovir is an antiviral agent.
1.‐ Ketoconazole (Nizoral)
2.‐ Mupirocin (Bactroban)
3.‐ Capsaicin (Capsin)
4.‐ Acyclovir (Zovirax) ointment

3049 The nurse should teach a client to use which one of Correct answer: 3 Synthetic detergent bars are milder on the skin. Dove is classified as synthetic detergent bar. Using the process of elimination will lead to the correct answer. The question lists the
the following for a skin disorder in which the use of Dial and Safeguard are deodorant soaps of a harsher nature. Ivory is classified as a true soap. word “mild,” so the choice would be the soap that would be least abrasive to the skin.
mild soap is needed?
1.‐ Dial
2.‐ Safeguard
3.‐ Dove
4.‐ Ivory

3050 The nurse conducting a community health teaching Correct answer: 4 The effectiveness of a sunscreen when compared to no use of sunscreen is usually indicated Recall knowledge that SPF is an abbreviation for "sun protection factor" to allow selection
session explains that sunscreen with an SPF of 6 means by its sun protection factor (SPF) (e.g., 6, 15, 30). Option 1 is a false interpretation. Sunscreens of the correct option, 4.
that the product has which of the following also can be classified as water‐resistant (option 2) or waterproof (option 3), but the SPF
characteristics? number does not indicate this information.
1.‐ Provides protection from sun's rays for six hours.
2.‐ Is water‐resistant, but not waterproof.
3.‐ Is waterproof for six immersions in the water.
4.‐ Provides six times the sun exposure protection as use of no sunscreen.

3051 The nurse anticipates that which of the following Correct answer: 2 Lindane has long been considered an appropriate treatment for pediculosis (lice). Terbinafine In order to select the correct answer, recall that the usual treatment for pediculosis is with
agents would be used to treat pediculosis infestation is an antifungal agent for tinea infections. Collagenase is an enzyme used as a debriding lindane. Use this knowledge to select the correct option.
in an adult client? preparation. Chlorhexidine is a skin and wound cleanser.
1.‐ Chlorhexidine (Hibiclens)
2.‐ Lindane (Kwell)
3.‐ Collagenase (Santyl)
4.‐ Terbinafine (Lamisal)

3052 An adult client has scabies on the trunk. Permethrin Correct answer: 1 Permethrin is the preferred treatment for scabies at the present time. A variety of treatment Use the process of elimination and knowledge of the usual treatment for scabies to lead
(Elimite Cream) is prescribed. What instructions should protocols are suggested, but the greatest success is reported when product is left on for at you to option 1. It is important not to confuse scabies with pediculosis, as several of the
be given to the client? least 8 hours. If the treatment is repeated, it is repeated at 7 days, not 6 hours (option 3). options relate to treatment of pediculosis. Review treatment of scabies if this was difficult.
Household articles in direct contact with the client need to be thoroughly washed or
disinfected, or both, but the human is the host for this parasite (option 4). Kwell can be used
for pediculosis or scabies, but two similar agents would not be used together (option 2).

1.‐ "Apply the agent from the neck down. Leave on for 8–14 hours before washing it off."
2.‐ "Use this product with an agent such as lindane (Kwell)."
3.‐ "Apply the agent to the affected area; repeat application in 6 hours."
4.‐ "Treat the clothing rather than the body."

3053 The nurse explains to a client with acne that which of Correct answer: 2 Benzoyl peroxide has bactericidal activity against Propionibacterium acnes. It is available in Recall knowledge of the correct treatment for acne to lead to the correct option. The
the following products is one of the most effective over‐the‐counter and prescription formulations, including bar soaps, washes, gels, and lotions, question is not asking about anything other than the best choice of medication. This can be
topical agents for use in treating this condition? in a variety of concentrations. Mafenide (option 1) is a preparation used in burn therapy. determined with knowledge of the usual treatment for acne.
Chlorhexidine (option 3) is a skin and wound cleanser; it may be used for preoperative
preparation of the skin. Cryotherapy (option 4) is a treatment used for some warts.

1.‐ Mafenide (Sulfamylon)


2.‐ Benzoyl peroxide
3.‐ Chlorhexidine (Hibiclens)
4.‐ Cryotherapy
3054 The nurse would provide instructions about how to Correct answer: 4 Isotretinoin is available in capsule form. The other products are also used for acne, but are Recall knowledge of the usual method of administration to assist in selecting the correct
take which oral agent that can be used in treatment of topical preparations. option. Since option 4 is the only oral medication, and the question states the word "oral"
acne? in the stem, this would be the correct answer. The other options are all incorrect. If this
was difficult, review methods of administration of the medications.

1.‐ Adapalene (Differin)


2.‐ Tretinoin (Retin‐A)
3.‐ Clindamycin (Cleocin T)
4.‐ Isotretinoin (Accutane)

3055 Silver sulfadiazine (Silvadene) is used to prevent and Correct answer: 2 Silver sulfadiazine reacts with DNA and releases sulfadiazine. The silver replaces the hydrogen In order to correctly answer this question, recall the mechanism of action of the
treat sepsis in burns. The Burn Unit nurse explains to bonding between strands of DNA, and prevents replication of the bacteria. It does not facilitate medication. If this was difficult, review the medication s action, along with the
an orientee that an important point about its skin cell replication, and it is not of small molecular size. It is not classified as an antifibrinolytic classification.
mechanism of action is its: agent.
1.‐ Facilitation of skin cell replication.
2.‐ Replacement of hydrogen bonding between DNA strands.
3.‐ Antifibrinolytic property.
4.‐ Small molecular size.

3056 A client is to receive medication to treat external Correct answer: 1 Salicylic acid preparations are useful for the removal of common warts. Povidone‐iodine Having knowledge of the correct treatment for external warts will lead to the correct
warts. The nurse would prepare instructions related to (option 2) is used to prepare or cleanse skin preoperatively; masoprocol (option 3) is indicated option. The question is not asking about anything other than the best choice of medication.
which of the following medications? for actinic keratosis; and crotamiton (option 4) is an antiparasitic drug. This can be determined with knowledge of the usual treatment for external warts.

1.‐ Salicylic acid (Duofilm)


2.‐ Povidone‐iodine (Betadine)
3.‐ Masoprocol (Actinex)
4.‐ Crotamiton (Eurax)

3057 The nurse would apply a closed soak to a client's skin Correct answer: 4 Closed soaks lead to heat retention, and are not used for a cooling effect. They are typically In determining the best answer for this question, think of the vasoconstriction. In a closed
to achieve which of the following benefits? applied for 1–2 hours at a time, 2–3 times a day. They are not impermeable to air. area there is heat conservation, which would lead to the only correct option, 3.

1.‐ Cooling effect


2.‐ Impermeability to air
3.‐ Ability to change it once every three days
4.‐ Heat retention

3058 The nurse recommends the use of a topical cream to Correct answer: 1 A cream is an emulsion of oil in water; an ointment is considered to be a water‐in‐oil product. The question is asking for selection of a skin product. Option 3 can be eliminated because
a client who needs which of the following ingredients Lotions are suspensions of powder in water or a liquid emulsion of thin consistency. A cream the question is not referring to the need to dry the skin. Options 2 and 4 are too broad for
in a skin product? might or might not contain an antimicrobial agent, and it is not a drying agent. the client. This would leave correct option 1.

1.‐ Emulsifying agent


2.‐ Ointment
3.‐ A drying agent
4.‐ Lotion

3059 The nurse teaches the client that which of the Correct answer: 3 Iodine preparations stain skin and clothing. Benzalkonium, hexachlorophene, and hydrogen Note the word iodine in option 3. Knowing that iodine stains the skin and clothing would
following skin cleansers is likely to stain the skin peroxide do not cause staining. allow selection of this option.
and/or clothing?
1.‐ Benzalkonium (Zephiran)
2.‐ Hexachlorophene (Phisohex)
3.‐ Povidone‐iodine (Betadine)
4.‐ Hydrogen peroxide

3060 The nurse would plan to use DuoDERM for a client Correct answer: 4 DuoDERM is indicated for ulcers with moderate drainage, but which are uninfected. Agents Knowledge that DuoDerm can only be used for wounds that are not infected will allow
with which of the following skin conditions? like Iodosorb might be used for ulcers with necrotic material (option 3), and Hydrasorb with selection of option 4. All the other options allude to infections, and therefore can be
ulcers with heavy drainage (option 2). Antimicrobials may be used to treat infected wounds. eliminated.

1.‐ An infected wound


2.‐ An ulcer with heavy drainage
3.‐ An ulcer with moderate amounts of necrotic material
4.‐ An uninfected venous stasis ulcer

3061 A client with acne is prescribed oral minocycline. The Correct answer: 2 Minocycline can lead to development of a lupus‐like syndrome, and also can cause In order to select the correct answer, recall the side effects of the medication minocycline.
nurse would explain to the client that a disadvantage pigmentation changes. Minocycline does not suppress sebum production (option 1), lead to If this was difficult, review the side effects of the medication.
(or disadvantages) of this preparation is/are which of spontaneous abortions (option 4) or have low/lack of treatment efficacy (option 3).
the following?
1.‐ Suppression of sebum production
2.‐ Lupus‐like syndrome, and pigmentation changes
3.‐ Lack of treatment efficacy
4.‐ Occurrence of spontaneous abortions

3062 The nurse who is using a very high–potency topical Correct answer: 2 Potent topical steroid therapy should be tapered within 2 weeks. Very high–potency topical Note the words “very high–potency” in the question stem. Using the process of
corticosteroid would generally plan to use it: corticosteroids can induce atrophy, telangiectasis, and striae as early as 2–3 weeks following elimination should lead to an option that indicates short‐term use, as in answer 2.
daily application. High‐potency topical corticosteroids and use of occlusion could induce
hypothalamic‐pituitary‐adrenal (HPA) axis suppression and adverse reactions typically
associated with chronic oral therapy. Dosing of topical corticosteroids more frequently than
2–3 times daily neither is indicated nor has proven benefit.

1.‐ For any type of skin disorder.


2.‐ For no longer than 2 weeks.
3.‐ Under an appropriate dressing.
4.‐ At least four times a day for best effect.

3063 The client has moderate acne. After assessment, a Correct answer: 1 Adverse reactions of acne preparations can include erythema; burning or stinging; excessive Note the word “peroxide” in the stem of the question. Realizing that peroxide is drying will
topical preparation of benzoyl peroxide is dryness; and increased susceptibility to sunburn. Directions include using a thin application lead, by the process of elimination, you to option 1. None of the other options mention
recommended. The nurse will advise the client not to twice a day. Excessive use would be more likely to result in evidence of adverse reaction than drying, and they can therefore be eliminated.
overuse the product because it could result in which of would use as per product directions. Adverse reactions do not include extreme pruritus (option
the following problems? 2), non‐healing lesions (option 3), or localized infection (option 4).

1.‐ Excessive dryness


2.‐ Extreme pruritus
3.‐ Lack of healing of lesions
4.‐ Localized infection

3064 An 18‐year‐old female client has severe acne. There Correct answer: 3 Accutane is an oral preparation that is a known teratogen; strict adherence to avoidance of Knowing that isotretinoin (Accutane) that has terotogenic effects, as well as that the client
has been no improvement from the use of various pregnancy is mandatory. Accutane is contraindicated with pregnancy because of the is an 18‐year‐old female, should assist in selecting the correct option, 3. The other options
preparations, and isotretinoin (Accutane) is being occurrence of spontaneous abortions, as well as major abnormalities in the fetus at birth, such are incorrect, since they do not address the teratogenic effects of the medication.
prescribed. The nurse evaluates that the client as hydrocephalus. Elevated triglyceride levels might occur, but changes in hormone functioning
understood medication instructions if the client stated (option 4) are not anticipated. The medication should be applied thinly (option 1), and sun
to do which of the following? exposure provides no added benefit (option 2).

1.‐ Apply a thick layer of isotretinoin twice a day.


2.‐ Increase exposure to the sun for added benefit.
3.‐ Have a pregnancy test prior to beginning therapy, and use contraception.
4.‐ Have blood drawn for hormonal studies monthly for the first six months.

3065 A client who experienced a burn injury is being Correct answer: 3 Mafenide acetate is a water‐soluble cream that is used to treat burn injuries. It can cause a Being aware of the clinical manifestations of the medication will assist in selecting the
treated with topical mafenide acetate (Sulfamylon). stinging or burning sensation after it is applied, and this is considered to be normal. Options 1 correct answer. If this was difficult, review the clinical manifestations, as well as nursing
The client reports a stinging and burning sensation and 2 are inappropriate actions, while option 4 will not prevent stinging of the medication. interventions, of the medication.
when the medication is applied. The nurse determines
that it would be appropriate to do which of the
following?
1.‐ Withhold the medication, and notify the physician.
2.‐ Wash off that dose of the medication.
3.‐ Explain that this is a normal sensation.
4.‐ Chill the preparation before using it next.

3066 A client has developed a bacterial infection in skin Correct answer: 4 The two most common organisms causing infection are Staphylococcus aureus and Using the process of elimination, option 4 can be the only correct answer. Options 2 and 3
damaged by thermal trauma. The nurse anticipates Streptococcus pyogenes. Herpes simplex and herpes zoster are members of the herpes virus would not be a causative agent in a thermal trauma, so eliminate them. Option 1 relates to
that wound culture will reveal the presence of one or family. Dermatophytes cause fungal infections of the skin, such as tinea capitis and tinea pedis. a respiratory diagnosis, and can therefore be eliminated.
both of which two most common causative agents? Haemophilus aegyptius causes pinkeye. Haemophilus influenzae can cause meningitis,
pneumonia, and serious throat and ear infections.
1.‐ <i>Haemophilus influenzae</i> and <i>Haemophilus aegyptius</i>
2.‐ Herpes simplex and herpes zoster
3.‐ Tinea capitis and tinea pedis
4.‐ <i>Staphylococcus aureus</i> and <i>Streptococcus pyogenes</i>

3067 The nurse would include which of the following pieces Correct answer: 4 Emollients contain petrolatum, oils, propylene glycol, or other substances, and make the skin Note that the question stem asks for an explanation of a skin emollient. The word
of information when explaining a skin emollient to a soft and pliable by increasing hydration of the stratum corneum. It does not dry the skin emollient indicates a measure that would make the skin soft. Since emollients contain an oil
client? (option 2) or contain corticosteroids (option 3), and does not always require shaking before base, the skin should be dry to begin with, unlike with a water base product, so that it can
use (option 1). be absorbed better into the skin. If this was difficult, review the definition of an emollient.

1.‐ It requires shaking before each use.


2.‐ It has a drying effect on the skin when the water evaporates.
3.‐ It includes a corticosteroid component.
4.‐ It is of use when skin is dry.

3068 The nurse explains to a client that the aluminum Correct answer: 1 Burow's solution is an astringent that coagulates bacterial and serum protein. It is classified as In order to answer this question, recall the classification of the medication. If this was
acetate solution (Burow's solution) ordered for a skin a soothing solution for the relief of inflammation associated with some skin problems. It is difficult, review the classification and purpose of aluminum acetate solution (Burow’s
condition is which type of preparation? useful for conditions with vesicles where there is oozing, as might occur with poison ivy or solution).
allergic reactions. It is not an emollient (option 2), a detergent (option 3), or a balm (option 4).

1.‐ Soothing astringent


2.‐ Emollient
3.‐ Detergent
4.‐ Balm

3069 The nurse would assess a client for which of the Correct answer: 3 Side effects associated with the use of topical corticosteroids include acneiform eruptions, In order to answer this question correctly, recall the common side effects of a topical
following side effects associated with topical allergic contact dermatitis, skin atrophy, burning sensation, dryness (option 2), itching, corticosteroid. Options 1 and 4 are similar in that there is a skin color change, and
corticosteroid use? irritation, hypopigmentation (option 1), alopecia, and hypothalamic‐pituitary‐adrenocortical therefore, they can be eliminated. Use knowledge of the side effects to be led to the
(HPA) axis suppression. Topical corticosteroids do not stain the skin (option 4). correct option.

1.‐ Hyperpigmentation
2.‐ Oiliness
3.‐ Burning sensation
4.‐ Staining of skin

3070 The client is experiencing severe itching with a skin Correct answer: 3 Hydroxyzine hydrochloride is an antihistamine that is a competitive inhibitor of the Knowing the action of the medications will lead to the correct answer, option 3. The other
disorder. Which of the following drugs, if ordered, H&lt;sub&gt;1&lt;/sub&gt; receptor. It is used to treat various reactions that are mediated by options are not used for complaints of itching. If this was difficult, review the purposes of
would the nurse administer as an appropriate oral histamine. It will decrease the pruritus produced by the release of histamine. Cimetidine is an the medication to assist in selecting the correct option.
preparation to decrease the itching? H&lt;sub&gt;2&lt;/sub&gt; histamine antagonist, and these agents are not effective against
hypersensitivity reactions. Lorazepam is a short‐acting benzodiazepine that is indicated for
anxiety. Bupivacaine is a local anesthetic for nerve blocks.

1.‐ Cimetidine (Tagamet)


2.‐ Lorazepam (Ativan)
3.‐ Hydroxyzine (Atarax)
4.‐ Bupivacaine (Sensorcaine)

3071 The client has sustained both partial‐ and full‐ Correct answer: 3 Mafenide is a carbonic anhydrase inhibitor that rarely causes metabolic acidosis. In that In order to answer this question correctly, recall the side effects of the medication.
thickness burns. Mafenide (Sulfamylon) is being used situation, the body would try to compensate to maintain a balanced pH. Hyperventilation or Knowledge that the medication does not cause gastrointestinal disturbances will allow
as part of treatment. While evaluating for potential labored respirations, and difficulty in breathing, would be evidence of the body's attempt to elimination of options 1 and 4. Use knowledge of the side effects to select the correct
adverse reactions of this drug, the nurse will observe compensate. Mafenide would not cause diarrhea (option 1), hypertension (option 2), or option.
the client for which of the following? abdominal pain (option 4).

1.‐ Diarrhea
2.‐ Hypertension
3.‐ Rapid breathing
4.‐ Abdominal pain

3072 The client with partial‐thickness burns is being Correct answer: 4 Wound infection is a potential complication with burns. Microorganisms proliferate rapidly in Note that the question stem asks for assessment of the burn. The only option that directly
managed as an outpatient. As part of client burn wounds. Monitoring arterial blood gases, and intake and output, is essential for the addresses assessment is option 4. Be certain to read carefully the question, and answer
assessment, the nurse will do which of the following? hospitalized client with more severe burns, but these are not key assessments for clients being according to the nursing process requested.
managed on an outpatient basis. The nurse removes any topical agents at the beginning of the
dressing change, to allow for adequate wound assessment.

1.‐ Remove any topical treatment after making an assessment.


2.‐ Monitor arterial blood gases at periodic intervals.
3.‐ Evaluate fluid intake and output as a measure of hydration.
4.‐ Assess burn areas for evidence of infection and superinfection.

3073 The nurse is aware that in addition to anti‐ Correct answer: 2 Topical corticosteroids have vasoconstrictive action as well as anti‐inflammatory effects. They Recall knowledge of the mechanism of action of corticosteroids to be led to the only
inflammatory effects, a mechanism of action of topical do not have fibrinolytic, emollient, or proteolytic effects. correct option, 2. If this was difficult, review the mechanism of action.
corticosteroids would be which of the following?

1.‐ Fibrinolytic
2.‐ Vasoconstriction
3.‐ Emollient
4.‐ Proteolytic

3074 In planning care for a client receiving a topical Correct answer: 2 Medications that have ointment bases have enhanced penetration of skin lesions when used Use the process of elimination and consider the words “enhancing penetration” to select
corticosteroid, which of the following vehicles would as topical preparations. The other options have less penetrating action. option 2. Ointments have the highest level of absorption in the skin.
be most effective in enhancing the penetration of this
medication?
1.‐ Gel
2.‐ Ointment
3.‐ Cream
4.‐ Lotion

3075 When evaluating the effectiveness of topical Correct answer: 2 The treatment of many skin disorders could take several weeks. A realistic timetable should Note that the question stem asks for evaluation of the effectiveness of the treatment. The
preparations in treating a skin disorder, it is important be presented to clients. Drying soaps should not be used. It is unnecessary to use gloves for all only answer that addresses evaluation of the nursing process is option 2. Options 1 and 3
for the nurse to utilize an understanding of which of preparations. Nonadherance to the prescribed regimen is sometimes a problem in managing are interventions in the nursing process. Option 4 does not address the question being
the following? skin conditions. asked.
1.‐ Washing the skin with drying soaps before application should be part of regimen.
2.‐ Skin lesions are often very slow to improve.
3.‐ Gloves should be used for application of all preparations.
4.‐ Nonadherance to the prescribed regimen is rarely a problem.

3076 A client has a diagnosis of psoriasis. The nurse Correct answer: 3 Anthralin is an antipsoriatic agent available as either a cream or ointment for the treatment Recall that option 3 is the medication of choice for psoriasis to allow for elimination of the
anticipates that the client's medication history will of psoriasis. Hexachlorophene is a topical antiseptic that is incorporated into soaps, other options. If this was difficult, review the usual treatment for psoriasis.
most likely reveal use of which of the following topical detergents, and other vehicles for topical use. Chlorhexidine is a skin and wound cleanser.
agents as treatment for this condition? Capsaicin is a topical agent that has been useful in certain painful disorders.

1.‐ Hexachlorophene (Phisohex)


2.‐ Chlorhexidine (Hibiclens)
3.‐ Anthralin (Antra‐Derm; Drithocreme)
4.‐ Capsaicin (Capsin)

3077 A young child who has been taking growth hormone Correct answer: 2 Adverse/side effects during the first 2–3 months of treatment with growth hormone include Note the key words in the question are first few months. This tells you that the adverse
(Somatropin) for one month is complaining of flank hypercalciuria, with resultant renal calculi. The client is not at increased risk for acute effect occurs soon after medication therapy begins. Use medication knowledge and the
pain, colic, and GI symptoms. The nurse concludes that glomerulonephritis, bowel obstruction, or duodenal ulcer (options 1, 3, and 4). process of elimination to make a selection.
this client is at increased risk for which adverse effects
that is more likely to occur during the first few months
of treatment?

1.‐ Acute glomerulonephritis


2.‐ Renal calculi
3.‐ Bowel obstruction
4.‐ Duodenal ulcer

3078 A client with coronary artery disease and Correct answer: 1 Clients with coronary artery insufficiency and hypertensive cardiovascular disease who take Note that options 1 and 2 are opposite, which could be a clue that one of them is correct.
hypertension was recently diagnosed with diabetes ADH are at increased risk for developing fluid overload and edema. Option 2 is the opposite of Recall that ADH causes fluid retention to link this information with the associated risks for a
insipidus. The nurse concludes that treatment with what would occur. Options 3 and 4 are not related to this client’s situation. client with cardiac disease.
antidiuretic hormone (ADH) is contraindicated for this
client for which of the following reasons?

1.‐ Fluid overload and elevated BP could occur.


2.‐ Volume depletion and decreased blood pressure could occur.
3.‐ Overstimulation and agitation could occur.
4.‐ Hypercalciuria and renal calculi could occur.

3079 A client with cardiovascular disease has been recently Correct answer: 2 Clients with known cardiovascular disease who are prescribed thyroid hormone replacement The critical words in the stem of the question are most important, which tell you that
diagnosed with hypothyroidism, and levothyroxine therapy can develop chest pain that could lead to myocardial infarction. For this reason, it is more than one answer might be correct, and that you must prioritize your answer. Use the
(Levoxyl) has been prescribed. Which of the following the most important manifestation for the client to report. The other manifestations, if they ABCs (airway, breathing, and circulation) to select the answer most important to a client
signs and symptoms related to this medication is most occurred in this client, would have a lesser priority. with cardiac disease (which affects circulation).
important for the client to report to the physician?

1.‐ Increased urine output


2.‐ Chest pain
3.‐ Increase in appetite
4.‐ Loose stools

3080 A client with a history of cardiac disease is exhibiting Correct answer: 3 Clients with severe symptoms of hypothyroidism and a history of cardiac disease must be The core issue of the question is knowledge of the principles of beginning a medication
severe symptoms of hypothyroidism, and is started on started on the lowest dose possible of hormone therapy and have the dose gradually that stimulates metabolism in a client with heart disease. Use knowledge of
medication therapy with levothyroxine (Synthroid). increased, in order to prevent onset of severe hypertension, heart failure, and myocardial pathophysiology to select the option that causes the least stress on the heart during
The nurse anticipates that which of the following infarction (MI). The other options could put the client at risk for chest pain and subsequent MI. initiation of therapy.
principles will be followed for initiation of drug
therapy?
1.‐ The client will be started with the highest dose possible, and the dose will then be titrated according to the client’s response.
2.‐ The client will be started with the highest dose possible, and will be given a beta blocker to prevent any incidences of tachycardia.
3.‐ The client will be started with as low a dose as possible, and the dose will gradually be increased over a period of weeks.
4.‐ The client will receive a fixed dose calculated according to the client’s weight; this dose will not be adjusted unless necessary.

3081 A client with acute adrenal insufficiency (adrenal Correct answer: 3 Clients with acute adrenal insufficiency will complain of musculoskeletal symptoms of The wording of the question tells you that the core issue is knowledge of signs and
crisis) is admitted to the hospital. The nurse monitors weakness and fatigue; GI complaints of anorexia, nausea and vomiting, and weight loss; symptoms of adrenal insufficiency that should respond to drug therapy. Use nursing
for resolution of which of the following manifestations integumentary symptoms of vitiligo and hyperpigmentation; and cardiovascular symptoms knowledge and the process of elimination to make a selection.
to determine that drug therapy with cortisone related to anemia, hypotension, hyponatremia, hyperkalemia, and hypercalcemia. The
(Cortone) has been effective? manifestations in options 1, 2, and 4 are opposite those seen with adrenal insufficiency, and
are therefore incorrect.
1.‐ Hyperexcitability and restlessness
2.‐ Increased appetite and weight gain
3.‐ Vitiligo and hyperpigmentation
4.‐ Hypertension and hypernatremia

3082 The nurse is assessing the laboratory data of a client Correct answer: 1 Clients with Cushing’s syndrome or hypercortisolism have elevated levels of cortisol, low The core issue of the question is knowledge of laboratory values that are expected to
diagnosed with Cushing’s syndrome. The nurse would ACTH levels, increased blood glucose levels, elevated white blood cell counts, elevated change once drug therapy is initiated for Cushing’s syndrome. Use nursing knowledge and
expect to note which of the following laboratory lymphocyte counts, increased sodium levels, decreased serum calcium levels, and decreased the process of elimination to make a selection.
values prior to initiation of drug therapy? serum potassium levels. Drug therapy will reduce serum cortisol levels when given as directed.
The laboratory trends noted in options 2, 3, and 4 are opposite what would be expected in
Cushing’s syndrome.
1.‐ Elevated plasma cortisol level
2.‐ Decreased blood glucose level
3.‐ Decreased white blood cell count
4.‐ Decreased sodium level

3083 The nurse is caring for a client who has just been Correct answer: 4 Graves’ disease is caused by elevated levels of thyroid hormone. Clients experience The core issue of the question is the type of medication therapy that will be effective in
diagnosed with Graves’ disease. Client education needs tachycardia, nervousness, insomnia, increased heat production, and weight loss. Medication treating hyperthyroidism or Graves’ disease. Use nursing knowledge and the process of
to include which of the following? therapy with an agent such as propylthiouracil will help control the disorder. Option 1 is elimination to make a selection.
irrelevant, while option 2 is indicated for hypothyroidism. A client with this disorder needs a
high‐calorie diet, not a low‐calorie one (option 3).
1.‐ Atropine‐like medication
2.‐ Thyroid hormone replacement therapy
3.‐ A low‐calorie diet
4.‐ Propylthiouracil (PTU)

3084 A client with Graves’ disease related to Correct answer: 4 Cardiac problems related to Graves’ disease and hyperthyroidism include increased systolic The core issue of the question is knowledge of which symptoms should resolve once
hyperthyroidism has been taking medication therapy blood pressure, a widened pulse pressure, tachycardia, and other dysrhythmias. Appropriate medication therapy for Graves disease has begun. Recall that Graves disease represents a
as prescribed. Which of the following findings noted on control of the disorder with medication therapy would prevent these manifestations from hypermetabolic state, and then eliminate each option that does not correlate with
cardiac assessment indicates to the nurse that the occurring. excessive metabolic activity.
client has not had a sufficient response to medication
therapy?
1.‐ Decreased systolic blood pressure
2.‐ Narrow pulse pressure
3.‐ Bradycardia
4.‐ Tachycardia

3085 The nurse is caring for a client who recently was Correct answer: 2 Management of hypoparathyroidism is aimed at correcting hypocalcemia, vitamin D The core issue of the question is knowledge of abnormal laboratory values that should
diagnosed with hypoparathyroidism. To determine the deficiency, and hypomagnesemia. Options 1 and 4 are incorrect because they are the opposite resolve with effective medication therapy for hypoparathyroidism. Use knowledge of
effectiveness of medication therapy with calcitrol of effective treatment of hypoparathyroidism, while option 3 is an unrelated finding. medication actions to eliminate each of the incorrect options. Options 1 and 2 are opposite,
(Rocaltrol), the nurse assesses laboratory findings to which is a clue that one of them is likely to be the correct answer.
see whether which of the following has occurred?

1.‐ Hypercalcemia is resolving.


2.‐ Hypocalcemia is resolving.
3.‐ Hypermagnesemia is resolving.
4.‐ Vitamin D levels are decreasing.

3086 A client with type 2 diabetes mellitus has been Correct answer: 1 Actos is a thiazolidinedione type of oral antidiabetic agent; its action enhances insulin action The core issue of this question is knowledge of the adverse effects of thiazolidinedione
prescribed pioglitazone (Actos). Which one of the and promotes glucose utilization in peripheral tissues. This drug improves sensitivity to insulin antidiabetic agents. Recall as a general strategy that many drug classes have adverse effects
following tests does the nurse anticipate will be done in muscle and fat tissue, and inhibits glucogenesis. Because of the potential for liver damage, on the liver. Use medication knowledge and the process of elimination to make a selection.
before drug therapy is initiated with this medication? clients taking drugs in this class must have liver function studies done before therapy is begun,
and periodically thereafter.
1.‐ Liver function tests
2.‐ Thyroid function tests
3.‐ Respiratory function tests
4.‐ Pituitary function tests

3087 A client newly diagnosed with adrenal insufficiency is Correct answer: 2 Adrenocortical replacement therapy medications are divided into mineralocorticoids and Specific medication knowledge about the effects of mineralocorticoid drug therapy is
to begin therapy with fludrocortisone (Florinef). The glucocorticoids. Mineralcorticoids such as fludrocortisone increase resorption of sodium by needed to answer this question. Use medication knowledge and the process of elimination
nurse explains to the client in simple terms that this increasing hydrogen and potassium excretion in the distal tubule. Glucocorticoids decrease to make a selection.
medication will do which of the following? inflammation by suppressing leukocyte migration and modifying the body’s immune response.
The statements in the remaining options do not reflect these actions.

1.‐ Decrease resorption of sodium by decreasing hydrogen and potassium excretion in the distal tubule
2.‐ Increase resorption of sodium by increasing hydrogen and potassium excretion in the distal tubule
3.‐ Decrease inflammation by suppressing migration of leukocytes and modifying the body’s immune response to many stimuli
4.‐ Increase inflammation by stimulating the production of leukocytes and enhancing the body’s immune response to many stimuli

3088 A homebound client with type 2 diabetes mellitus Correct answer: 4 If anorexia, nausea, or vomiting is present, sick‐day diabetic management care requires The core issue of the question is knowledge of how to manage medication therapy during
calls the nurse to report nausea and flulike symptoms clients to check their blood glucose level every 4–6 hours. Clients should not eliminate or illness. With this in mind, choose the option that does not decrease the drug dose (since
for two days. What advice should the nurse give the adjust their doses of insulin or oral hypoglycemics. They also should drink 8–12 ounces of sugar‐ blood glucose rises during illness), and that provides for safe and effective treatment
client? free liquids as tolerated every hour, to prevent dehydration. Meals should be eaten at regular (calling the physician when the level is excessive).
times, and clients should consume foods and liquids that are more easily tolerated. To prevent
diabetic ketoacidosis, this client must regularly check urine ketone levels if the blood glucose
level is higher than 240 mg/dL.

1.‐ “Be sure to check your blood glucose level in the morning on a daily basis.”
2.‐ “Only take half of your regular dose of insulin and oral hypoglycemic agent.”
3.‐ “Limit fluid intake, and eat only when you feel hungry.”
4.‐ “Test your urine for ketones if your blood glucose is higher than 240 mg/dL.”
3089 A client was just diagnosed with hypothyroidism. The Correct answer: 2 Thyroid hormones increase the effects of anticoagulants. Assessment of PT or INR will The core issue of the question is understanding of interactive effects of thyroid hormones
client also takes sodium warfarin (Coumadin). Before determine if the anticoagulant dosage must be decreased. The nurse also assesses the client and anticoagulants such as warfarin. Use knowledge of both medications and associated
giving any thyroid replacement hormone, the nurse for evidence of bruising or bleeding. A CBC could detect anemia caused by bleeding as a laboratory values to make a selection.
checks the results of what laboratory value? complication of excessive warfarin therapy. APTT measures the effectiveness of heparin.
Coumadin levels are not drawn.
1.‐ Complete blood count (CBC)
2.‐ Prothrombin time (PT) or international normalized ratio (INR)
3.‐ Activated partial thromboplastin time (APTT)
4.‐ Warfarin (Coumadin) level

3090 A client who has osteoporosis has been receiving Correct answer: 3 Hypercalcemia, hypomagnesemia, and digitalis toxicity could result when calcium The core issue of the question is knowledge of the effects of calcium supplements in a
calcium supplements on a long‐term basis. The supplements interact with digoxin. Clients must be instructed to take these two drugs at client who also takes digoxin. Use knowledge of medication interactions and the process of
physician has ordered the client to start taking digoxin separate times of the day. Also, antacids must not be taken with digoxin. Hyperkalemia is not a elimination to make a selection.
(Lanoxin). For what type of drug interaction would the concern with calcium supplementation.
homecare nurse assess during future visits?

1.‐ Hypocalcemia
2.‐ Hyperkalemia
3.‐ Digoxin toxicity
4.‐ Hypermagnesemia

3091 The nurse who is working in a women’s health clinic Correct answer: 4 Primary osteoporosis most often occurs in postmenopausal women who are thin and lean‐ The core issue of the question is knowledge of the clients at risk for osteoporosis and
has several clients to see during the day. Which of built. It is more prevalent in Caucasian and Asian women. amenable to therapy with calcium supplementation. Use general nursing knowledge and
these clients does the nurse anticipate will need the process of elimination to make a selection.
medication teaching for calcium supplementation to
treat primary osteoporosis?
1.‐ A premenopausal client
2.‐ An overweight client
3.‐ An African‐American client
4.‐ A Caucasian client

3092 A client with osteoporosis has been prescribed Correct answer: 2 Progesterone frequently is given along with estrogen as part of HRT to minimize the The core issue of the question is knowledge of the adverse effects of HRT given in addition
hormone replacement therapy (HRT) in addition to occurrence of endometrial or breast cancer. The other options do not relate to this to calcium supplementation for prevention or treatment of osteoporosis. Use knowledge of
calcium supplements. The nurse teaches the client that medication. adverse drug effects and the process of elimination to make a selection.
progesterone is given along with estrogen in this
therapy to decrease occurrence of what condition?

1.‐ Vaginitis
2.‐ Endometrial or breast cancer
3.‐ Benign breast tumors
4.‐ Ovarian cysts

3093 A client newly diagnosed with diabetes mellitus has Correct answer: 1 If diabetes is well controlled, blood glucose levels are not affected by mild consumption of The core issue of the question is knowledge of how alcohol can affect blood glucose levels
begun taking insulin. The client asks the nurse about alcohol. Male clients taking insulin may ingest two alcoholic beverages daily, and female clients in a client taking insulin. Use knowledge of interactive effects of medications and alcohol to
alcohol consumption. What should the nurse tell the may ingest one alcoholic beverage with, or in addition to, the regular meal plan. Because of make a selection.
client? the risk of alcohol‐induced hypoglycemia, diabetic clients must ingest alcohol only with or
shortly after meals. In all cases, the client should confer with the prescribing physician and
dietitian to determine whether alcohol may be utilized as part of the overall caloric intake.

1.‐ “Moderate‐to‐high alcohol consumption without food can cause your blood glucose level to go down too low.”
2.‐ “Moderate‐to‐high alcohol consumption without food can cause your blood glucose level to rise too high.”
3.‐ “Consumption of alcohol has no effect on your blood glucose, as long as you don t eat while drinking.”
4.‐ “As long as you only drink beer and wine, and not hard liquor, there should be no effect from the alcohol.”
3094 The nurse is instructing the newly diagnosed diabetic Correct answer: 3 The following statements are commonly used to teach clients how to combine insulins in one The core issue of the question is proper technique for drawing up mixed insulins. Choose
client how to mix regular insulin and NPH insulin. What syringe: Gently roll the bottle of intermediate insulin to mix, because vigorous shaking creates the option that does not contaminate the shorter‐acting insulin with the longer‐acting one,
should the nurse tell the client? bubbles, leading to an inaccurate dose. Air must be injected into each bottle before which would necessitate discarding the contaminated vial.
withdrawing. Withdrawing the shorter‐acting insulin first prevents the longer‐acting insulin
from mixing in the bottle with the shorter‐acting insulin.

1.‐ Shake the bottle of intermediate insulin before withdrawing the amount.
2.‐ Withdraw the longer‐acting insulin first.
3.‐ Withdraw the shorter‐acting insulin first.
4.‐ Never inject air into the bottles before withdrawing.

3095 Metformin (Glucophage) has been prescribed for a Correct answer: 4 Metformin is given to clients with stable type 2 diabetes mellitus to inhibit glucose production The core issue of the question is knowledge of the actions of metformin on reducing blood
client newly diagnosed with type 2 diabetes mellitus. by the liver and increase sensitivity of peripheral tissue to insulin. The other three options glucose. Use medication knowledge and the process of elimination to make a selection.
The nurse evaluates that the client understood contain factually incorrect statements.
medication teaching when the client states that this
medication does which of the following?

1.‐ Decreases sensitivity of peripheral tissue to insulin.


2.‐ Stimulates glucose production in the liver.
3.‐ Treats unstable type 2 diabetes mellitus.
4.‐ Decreases production of glucose by the liver.

3096 The physician has prescribed vitamin D for a client. Correct answer: 2 Vitamin D regulates calcium and phosphorus levels by increasing blood levels, increasing The core issue of the question is knowledge of the effects of vitamin D. Use medication
The client asks the nurse what the medication is for. intestinal absorption and mobilization from bone, and reducing renal excretion of both knowledge and the process of elimination to make a selection.
Which of the following is the best response by the elements. The statements in the other options are the opposites of the actions of vitamin D.
nurse?
1.‐ “Vitamin D decreases intestinal absorption of calcium and phosphorus, and decreases their mobilization from bone.”
2.‐ “Vitamin D helps regulate calcium and phosphorus balance.”
3.‐ “Vitamin D helps the kidneys rid the body of excess calcium and phosphorus.”
4.‐ “Vitamin D decreases blood levels of calcium and phosphorus.”

3097 A mother comes to the clinic with her 5‐year‐old son. Correct answer: 3 Growth hormone is approved for use in children only to treat a documented lack of growth Options 2 and 4 can be eliminated, since these do not answer the mother s question.
She is concerned that he is not growing fast enough, hormone. It is available as a parenteral medication only, to be given IM or SC (option 2). Only Option 1 is a false statement, so it can also be eliminated. This process leaves option 3 as
and asks the nurse if he can receive growth hormone long bones are affected (option 3). Option 4 is incorrect because this response implies that this the correct answer.
(GH). Which of the following would be the best treatment is appropriate despite the lack of additional diagnostic evidence needed for this
response by the nurse? therapy.
1.‐ "Growth hormone will only affect your child's short bones."
2.‐ "Can your son swallow pills easily?"
3.‐ "Growth hormone is only given to children if there is a documented lack of growth hormone."
4.‐ "How tall do you want him to be?"

3098 A client has been receiving high doses of Correct answer: 2 Abrupt cessation of long‐term steroid therapy can cause acute adrenal insufficiency, which Recall knowledge of the pathophysiological process of glucocorticoids. Option 2 is the only
glucocorticoids for several weeks. The client asks the could lead to death. Options 1 and 4 are incorrect statements. Central nervous system correct pathophysiological process of the classification. If this was difficult, review the
nurse when he can stop taking the medication. The symptoms such as confusion and psychosis are adverse effects of steroids such as prednisone action of glucocorticoids.
nurse's response incorporates which of the following (option 3).
information?
1.‐ Even at high doses, adverse reactions are unlikely if the medication is abruptly withdrawn.
2.‐ If steroid medication is withdrawn suddenly, a client could die of acute adrenal insufficiency.
3.‐ The client could experience severe psychological symptoms when the medication is withdrawn.
4.‐ Tapering of the medication requires daily assessment of serum chemistries.
3099 A client recently diagnosed with diabetes insipidus is Correct answer: 1 Desmopressin is not given by the intramuscular route. This medication may be given by the This question requires knowledge of the correct administration methods of the
to receive desmopressin (DDAVP). The client expresses intravenous, subcutaneous, or intranasal routes (options 2, 3, and 4) in the treatment of medication. If unsure of the correct option for this medication, review administration of the
concern about possible inability to properly self‐ diabetes insipidus. medication in a medication book.
administer the medication. In responding to the client,
the nurse conveys that this medication is not given by
which of the following routes?

1.‐ Intramuscular
2.‐ Intravenous
3.‐ Intranasal
4.‐ Subcutaneous

3100 A client with Addison's disease is taking Correct answer: 1 Fludrocortisone is a mineralocorticoid used to treat Addison's disease. High doses of In order to select the correct answer to this question, recall the effects of the medication.
fludrocortisone (Florinef) for replacement therapy. In fludrocortisone can result in excess retention of salt and water, and depletion of potassium. If this was difficult to answer, review the effects of the medication on the disease process.
evaluating the effects of drug therapy, the nurse Options 2 and 3 contain incorrect statements. In the treatment of Addison's disease,
anticipates that which of the following could occur fludrocortisone is commonly used in combination with a glucocorticoid (option 4).
when high doses of this drug are given?

1.‐ Excess sodium and water are retained, and potassium is depleted.
2.‐ Sodium and water are depleted, and potassium is retained.
3.‐ Hypotension and hypokalemia could develop.
4.‐ This could lead to toxic effects when given with any glucocorticoid.

3101 A client newly diagnosed with hypothyroidism is Correct answer: 4 After the start of therapy, peak levels of the drug should not be expected for many weeks to Note in the question stem that the nurse is being asked by the client when the energy level
placed on levothyroxine sodium (Synthroid). The client months. Thus, increased energy levels cannot be expected within a few days (option 2). The will increase. Options 1 and 3 should be eliminated, since they do not address the question
asks when his lack of energy will improve. The nurse's drug works best when taken before breakfast on an empty stomach (option 3). Lack of energy the client is asking. This leaves options 2 and 4 as choices. Since the medication is long‐term
response includes which of the following? is a common symptom with hypothyroidism (option 1). therapy, immediate results cannot be expected, which would eliminate option 2.

1.‐ Lack of energy is probably caused by depression, not hypothyroidism.


2.‐ Dramatic improvement in energy levels can be experienced, usually in 1–2 days.
3.‐ The drug works best when taken after a full meal.
4.‐ Optimum effectiveness of the drug might not occur for several weeks.

3102 A client with hypothyroidism is prescribed liotrix Correct answer: 2 Symptoms of adverse effects and thyrotoxicosis of liotrix (Thyrolar) include tachycardia, In order to select the correct answer, recall the definition of thyrotoxicosis. The only sign
(Thyrolar). The nurse teaches the client which of the angina, tremor, nervousness, insomnia, hyperthermia, heat tolerance, and sweating. Options or symptom of hyperthyroidism is option 2. The other options are of hypothyroidism, and
following signs and symptoms of thyrotoxicosis prior 1, 3, and 4 represent manifestations that are opposite those of thyrotoxicosis, which are also can be eliminated.
to discharge? manifestations of hypothyroidism.
1.‐ Bradycardia and hypothermia
2.‐ Tachycardia and hyperthermia
3.‐ Unusual lethargy, and inability to keep awake
4.‐ Complaints of chills and dry skin

3103 A client with Graves' disease who has been taking Correct answer: 3 Agranulocytosis is the most serious toxic effect of this drug, and it can make the client Using the process of elimination, omit option 4 because Haemophilus influenzae is not a
propylthiouracil (PTU) for six weeks complains of a predisposed to a variety of infections. Although rare, this adverse effect can occur within the complaint, but a diagnosis. Option 1 can be eliminated because hyperthyroidism is another
sore throat and fever. The nurse concludes that this first few months of treatment. Options 1 and 2 are incorrect conclusions; although option 4 is name for Graves disease. Option 2 does not relate to the thyroid, so it can also be
complaint could be an early sign of which of the possible, the manifestations reported are general signs of infection that might or might not be eliminated. This would lead to the correct choice, 3.
following? consistent with influenza.
1.‐ Hyperthyroidism
2.‐ Hyperpituitarism
3.‐ Agranulocytosis
4.‐ Haemophilus influenzae
3104 A client with Paget's disease is prescribed calcitonin Correct answer: 1 Calcitonin rapidly lowers blood calcium levels by reducing mobilization of calcium from bone, Knowing the pathophysiology of Paget s disease will lead to the only correct answer, 1. If
(Calcimar). The nurse tells the client that the purpose decreasing intestinal resorption, and promoting urinary excretion of calcium. Options 2 and 3 this was difficult, review the pathophysiology of Paget s disease.
of this drug is to do which of the following? are effects that are opposite to the ones caused by calcitonin, while option 4 is incorrect
because of the word "gradual."
1.‐ Decrease mobilization of calcium from the bone
2.‐ Increase intestinal absorption of calcium
3.‐ Promote urinary retention of calcium
4.‐ Provide gradual lowering of blood calcium levels

3105 A nurse is caring for a client who is receiving insulin. Correct answer: 3 Hunger; nausea; pale, cool skin; and sweating are signs of a hypoglycemic reaction. Fruity Knowing the signs and symptoms of hypoglycemia will lead to the correct answer, which is
For which of the following signs of hypoglycemic breath (option 1) can accompany ketoacidosis. Flushing of the face (option 2) can accompany option 3. If this was difficulty, be sure to be able to distinguish between hypoglycemia and
reaction should the nurse observe the client? hyperglycemia. Dry, flaky skin (option 4) is unrelated to hypoglycemia. hyperglycemia signs and symptoms.

1.‐ Fruity breath


2.‐ Flushing of the face
3.‐ Hunger
4.‐ Dry, flaky skin

3106 The nurse is evaluating the client's knowledge of Correct answer: 4 The initial action by the client is to take some form of oral glucose in order to raise the blood Note in the question stem that the nurse is to select an action for the client to be able to
treatment if an insulin reaction occurs. Which of the glucose level. Option 1 would delay appropriate self‐treatment. Options 2 and 3 would cause understand. While the client could notify the physician, this would delay recovery. The first
following actions is most appropriate for the client to further harm to the client. choice for the client to implement is option 4.
understand?
1.‐ Notify the doctor.
2.‐ Inject a dose of regular insulin.
3.‐ Lie down and wait for the reaction to disappear.
4.‐ Take an oral form of glucose.

3107 A child is about to begin taking growth hormone. Correct answer: 1 The expected growth rate with growth hormone therapy is 3–5 inches in the first year. Height The question stem stresses points to explain to the parents. Use the process of
Which of the following teaching points should the and weight are measured monthly (option 2). Growth hormone is discontinued when optimum elimination. If this was difficult, review the expected results of growth hormone
nurse stress to the parent? adult height is attained, when fusion of epiphyseal plates has occurred, or when there is no administration to a child.
response to growth hormone (option 3). Growth hormone is related to growth of long bones,
not fat deposition (option 4).
1.‐ "Your child's expected growth rate is 3–5 inches during the first year of treatment."
2.‐ "You need to measure your child's height and weight daily."
3.‐ "Growth hormone therapy, once started, must be taken until the child reaches the age of 21."
4.‐ "The amount of subcutaneous fat your child has will increase during the treatment period."

3108 The nurse is instructing the client about insulin Correct answer: 3 Jogging increases insulin requirements, and absorption can be increased if the drug is injected Using the process of elimination, rule out options 1 and 4 because they do not relate to
administration. Which of the following pieces of client into the thigh. This lifestyle factor of the client requires special instruction. Options 1 and 4 are teaching that is necessary. Option 2 does not require any special instruction but more
information alerts the nurse that special instruction unrelated to teaching about insulin administration. Option 2 guides the nurse to include the involvement of the family. This leaves the correct option, 3.
regarding insulin is necessary? spouse in teaching, but it does not indicate the need for special instruction regarding insulin.

1.‐ Client lives in an apartment with spouse.


2.‐ Client wishes to teach her spouse how to administer insulin.
3.‐ Client jogs 3–4 miles every other day.
4.‐ Client takes a nap in the afternoon.
3109 A client with a history of alcoholism was just Correct answer: 2 Tolbutamide interacting with alcohol can lead to a disulfiram‐like reaction, causing complaints In order to answer this question, recall the contraindications with the use of the
diagnosed with type 2 diabetes mellitus and placed on of headache and flushing of the skin. This is an important teaching point for the client who has medication. If this was difficult, review the contraindications with the use of tolbutamide
tolbutamide (Orinase). The nurse explains that which a history of alcoholism, even if he currently is not drinking. The reactions listed in the (Orinase).
of the following reactions could occur if the client remaining options do not occur as a result of co‐ingestion with alcohol.
drinks alcohol while taking this medication?

1.‐ Decreased diuresis


2.‐ Disulfiram‐like reaction
3.‐ Anaphylaxis
4.‐ Increased tolerance to the medication

3110 A child is placed on somatrem (Protropin). The Correct answer: 4 Resistance to growth hormone eventually develops, and the rate of growth begins to slow Knowing that the efficacy of the medication declines with age will lead to the only correct
parents ask the nurse if the child will keep growing down with increasing age. Efficacy of the drug is usually lost by the age of 20–24 years (options option.
after the drug has been discontinued. The nurse 2 and 3). The medication is quite effective in children (option 1), as long as there is a
utilizes which of the following points about this demonstrated deficiency in growth hormone.
medication in a response?
1.‐ The drug is not effective until the client has reached teenage years.
2.‐ The client must take the drug for a lifetime.
3.‐ The client can expect to grow well into his fourth or fifth decade.
4.‐ Efficacy of therapy declines as the client grows older.

3111 A client is taking metyrapone (Metopirone), and Correct answer: 2 In the presence of adrenal insufficiency, metyrapone can cause an adrenal crisis by stopping Note in the question stem that the client is experiencing an adrenal crisis. Options 1, 2,
experiences an adrenal crisis. Which of the following the synthesis of cortisol. Options 1 and 4 are the opposite of what is occurring with the client. and 4 can be eliminated because they are the direct opposite of the scenario in the
most likely predisposed the client to this occurrence? Option 3 is an unrelated finding. question. Option 2 is the only option that addresses adrenal insufficiency.

1.‐ Cortisol synthesis has increased.


2.‐ The client has adrenal insufficiency.
3.‐ The client has type 1 diabetes mellitus.
4.‐ The client has no adrenal insufficiency.

3112 A client is scheduled to have a bilateral Correct answer: 1 Hydrocortisone succinate may be given IV or IM, and is the preferred drug for replacement Knowing the usual mode of treatment after an adrenalectomy will lead to the only correct
adrenalectomy. Which of the following drugs does the therapy in all forms of adrenocortical insufficiency. ACTH is mostly used for diagnostic testing answer to this question. If this was difficult, review the usual treatment after an
nurse expect to administer in the postoperative (option 3). Dexamethasone is used for nonendocrine disorders (option 2), and ketoconazole adrenalectomy.
period? (option 4) is used to suppress the synthesis of adrenal steroids.

1.‐ Hydrocortisone succinate


2.‐ Dexamethasone
3.‐ Adrenocorticotropic hormone (ACTH)
4.‐ Ketoconazole

3113 A client who was recently started on drug therapy Correct answer: 4 Desmopressin is a drug used to treat diabetes insipidus. The manifestations listed are all signs Using the process of elimination will lead to the correct answer. Options 1 and 2 do not
with desmopressin (DDAVP) complains of a headache, of water intoxication, which could occur as an excessive effect of the medication. Options 1 relate to the disease process. Knowing that the clinical manifestations are signs of water
lethargy, and drowsiness. The nurse concludes that and 2 are unrelated to this medication, while option 3 is associated with diabetes insipidus, the intoxication will lead you to select option 4.
which of the following might be responsible for this underlying condition for which this drug would be ordered.
reaction?
1.‐ Streptococcal infection
2.‐ Excessive ingestion of calcium
3.‐ Dehydration
4.‐ Fluid overload
3114 A client with hyperthyroidism is being prepared for Correct answer: 1 Propranolol is a beta‐adrenergic blocker, and is used to treat sympathetic nervous system Knowing the classification of the medication will lead to the only correct answer, option 1.
surgery, and propranolol (Inderal) is prescribed. The symptoms related to hyperthyroidism, such as tachycardia, cardiac dysrhythmias, and mental If this was difficult, review the purposes of the medication.
nurse explains to the client that this drug is being given agitation. The manifestations identified in the other options would not be adequately treated
to control which symptom? with this medication.

1.‐ Tachycardia
2.‐ Hypotension
3.‐ Dyspnea
4.‐ Drowsiness

3115 A client who is taking digoxin (Lanoxin) is to receive a Correct answer: 2 Parenteral calcium can cause severe bradycardia in clients taking digoxin. Option 1 is the Knowing that calcium can result in changes in heart rate will assist to eliminate options 3
dose of intravenous calcium. For which of the opposite effect of what could occur. Hypertension is not an expected effect (option 4); and 4. When determining whether the answer is option 1 or 2, review the effects of digoxin
following drug interactions must the nurse be hypotension could occur as a result of severe bradycardia, but this is a secondary effect (Lanoxin).
prepared? (option 3).
1.‐ Severe tachycardia
2.‐ Severe bradycardia
3.‐ Severe hypotension
4.‐ Severe hypertension

3116 A client with hypocalcemia needs to increase his Correct answer: 4 Vitamin D regulates calcium and phosphorus metabolism, and increases blood levels of both In order to select the correct answer, recall that calcium is best absorbed with vitamin D. If
calcium absorption. The nurse explains to the client elements. The vitamins in the other options do not have this beneficial effect. this was difficult, review vitamins and their absorption.
that which of the following vitamins will be most
beneficial to the client?
1.‐ Vitamin A
2.‐ Vitamin C
3.‐ Vitamin B12
4.‐ Vitamin D

3117 A child diagnosed with deficiency of growth hormone Correct answer: 2 Children with growth hormone deficiency are smaller than peers, and frequently experience Note in the question stem that the client is a child receiving growth hormone. Options 1, 3,
who needs replacement drug therapy comes to the problems with self‐esteem and body image. Option 1 would be the opposite problem of what and 4 do not address the concerns with a child who needs growth hormone.
clinic for treatment. Which of the following nursing the client is experiencing. The nursing diagnoses in options 3 and 4 are unrelated to the client
diagnoses would be most appropriate for this client? in this question.

1.‐ Imbalanced Nutrition: More than Body Requirements


2.‐ Disturbed Body Image
3.‐ Diversional Activity Deficit
4.‐ Decreased Cardiac Output

3118 A client with diabetes insipidus who has been taking Correct answer: 1 Signs of overdosage of desmopressin, an antidiuretic hormone, include blood pressure and Note that the question asks for selection of the next action or intervention. Safety should
desmopressin (DDAVP) intranasally comes to the clinic pulse elevation; mental status changes; and water and sodium retention. Because the also be taken into account in selecting the next intervention. Option 2 would not be safe.
for a regularly scheduled appointment. The nurse medication therapy needs to be interrupted, the nurse should notify the physician. Option 2 Options 3 and 4 do not address what the question is asking for next. Recognize that option
assesses the client's mental status and notes some would place the client at risk because of lack of timely treatment. Options 3 and 4 would not 1 would be the next action that is appropriate as well as safe.
disorientation and behavioral changes. Significant address the current complication.
pedal edema is also present. What should be the
nurse's next action(s)?

1.‐ Check vital signs, and notify the physician.


2.‐ Have the client return in the morning for reevaluation.
3.‐ Instruct the client to limit salt intake for a few days.
4.‐ Suggest that the client change the route of administration to subcutaneous injections.
3119 The nurse is caring for a client who has just been Correct answer: 4 Graves' disease is caused by elevated levels of thyroid hormone. Clients experience Knowing the usual treatment modality will lead you to option 4. The other options are not
diagnosed with Graves' disease. Client education needs tachycardia, nervousness, insomnia, increased heat production, and weight loss. Medication correct education statements for the disease process. If this was difficult, review the
to include which of the following? therapy with an agent such as propylthiouracil will help control the disorder. Option 1 is treatment modalities for Graves disease.
irrelevant, while option 2 is indicated for hypothyroidism. A client with this disorder needs a
high‐calorie diet, not a low‐calorie one (option 3).
1.‐ Atropine‐like medication
2.‐ Thyroid hormone replacement therapy
3.‐ A low‐calorie diet
4.‐ Propylthiouracil (PTU)

3120 The physician has prescribed vitamin D for a client. Correct answer: 2 Vitamin D regulates calcium and phosphorus levels by increasing blood levels, increasing In order to correctly answer this question, recall the actions of calcium. The only correct
The client asks the nurse what the medication is for. intestinal absorption and mobilization from bone, and reducing renal excretion of both option is option 2. If this was difficult, review vitamin D action.
Which of the following is the best response by the elements. The statements in the other options are the opposites of the actions of vitamin D.
nurse?
1.‐ "Vitamin D decreases intestinal absorption of calcium and phosphorus, and decreases their mobilization from bone."
2.‐ "Vitamin D helps regulate calcium and phosphorus balance."
3.‐ "Vitamin D helps the kidneys rid the body of excess calcium and phosphorus."
4.‐ "Vitamin D decreases blood levels of calcium and phosphorus."

3121 The nurse is caring for a client with Paget's disease. Correct answer: 1 Drug therapy for Paget's disease focuses on decreasing calcium release by decreasing activity In order to correctly answer this question, recall the goals of pharmacological therapy for
The nurse should include in the teaching plan for the of osteoclasts, thereby decreasing bone resorption. The other options indicate effects that are a client with Paget s disease.
client that pharmacological therapy for treating this opposite those that are intended with drug therapy.
disease is aimed at which of the following results?

1.‐ Decreasing bone resorption


2.‐ Increasing calcium release
3.‐ Increasing bone resorption
4.‐ Increasing activity of osteoclasts

3122 Glipizide (Glucotrol XL) has been prescribed for a Correct answer: 3 Glipizide is given to type 2 diabetic clients; it is used as an adjunct to diet therapy and In order to answer this question, recall the purpose of glipizide (Glucotrol XL). If this was
client with diabetes mellitus. The nurse plans to teach exercise, but does not replace insulin for clients who need it (option 1). It increases the release difficult, review the purpose of the medication in a medication handbook.
the client that this medication is being used for which of insulin from pancreatic islet cells (option 4). Its use is contraindicated during pregnancy and
of the following purposes? lactation (option 2).
1.‐ To take the place of insulin injections
2.‐ To treat clients with diabetes who are pregnant
3.‐ To treat type 2 diabetes mellitus
4.‐ To decrease release of insulin from pancreatic islet cells

3123 Metformin (Glucophage) has been prescribed for a Correct answer: 4 Metformin is given to clients with stable, type 2 diabetes mellitus to inhibit glucose In order to answer this question, recall the purpose of metformin (Glucophage). If this was
client newly diagnosed with type 2 diabetes mellitus. production by the liver and increase sensitivity of peripheral tissue to insulin. The other three difficult, review the purpose of the medication in a medication handbook.
The nurse evaluates that the client understood options are not actions of metformin.
medication teaching when the client states that this
medication does which of the following?

1.‐ Decreases sensitivity of peripheral tissue to insulin.


2.‐ Stimulates glucose production in the liver.
3.‐ Treats unstable type 2 diabetes mellitus.
4.‐ Decreases production of glucose by the liver.
3124 The nurse is providing discharge education to a client Correct answer: 2 Early signs of hypoglycemia and lactic acidosis include hyperventilation, myalgia, malaise, and In order to answer this question, recall knowledge of hypoglycemic reactions that a client
newly diagnosed with type 2 diabetes mellitus. The unusual somnolence. The client should learn to recognize these signs in order to check blood will experience. Having knowledge of hyperglycemic and hypoglycemic reactions will allow
nurse tells the client that early signs of hypoglycemia glucose levels and take corrective action. selection of the correct answer.
and lactic acidosis can include which of the following?

1.‐ Bradycardia
2.‐ Hyperventilation
3.‐ Agitation
4.‐ Hypertension

3125 A client is being evaluated for control of type 2 Correct answer: 4 Glycosylated hemoglobin concentrations are representative of a client's average blood In order to answer this question, recall the definition and purpose of this lab test.
diabetes mellitus with exercise, diet, and oral glucose levels over the previuos 2–3 months, not in recent hours or days (option 1). It does not
medication therapy. A glycosylated hemoglobin test is diagnose anemia or kidney damage (options 2 and 3).
ordered. The client asks the nurse what that test is for.
Which of the following is the best response by the
nurse?
1.‐ "Results of this test reveal your blood glucose over the last 24 hours."
2.‐ "Results of this test can indicate whether you are anemic."
3.‐ "Results of this test will show if you have any kidney damage."
4.‐ "Results of this test show your average blood glucose over the last several weeks."

3126 A 60‐year‐old client has been prescribed rabeprazole Correct answer: 3 Omeprazole, pantoprazole, and rabeprazole must be swallowed whole. Lansoprazole and The core issue of the question is knowledge of the formulations of various proton pump
(Aciphex) for symptoms of gastroesophageal reflux esomeprazole capsules may be opened and sprinkled on applesauce or dissolved in 40 mL of inhibitors. Use medication knowledge and the process of elimination to make a selection.
disease (GERD). He has trouble swallowing pills. What juice.
alternate medication could be used for this client?

1.‐ Omeprazole (Prilosec)


2.‐ Pantoprazole (Protonix)
3.‐ Lansoprazole (Prevacid)
4.‐ There is no substitute for Aciphex.

3127 A nurse is teaching a female client newly diagnosed Correct answer: 4 Ciprofloxacin is not recommended for Helicobacter pylori infection during pregnancy. The The core issue of the question is knowledge of what medications are safe for use in
with Helicobacter pylori infection. The nurse expects other medications can be used after consulting with the physician. pregnancy for a client with Helicobacter pylori infection. Use medication knowledge and the
that which of the following medications will not be process of elimination to make a selection.
used after learning the client is pregnant?

1.‐ Metronidazole (Flagyl)


2.‐ Amoxicillin (Amoxil)
3.‐ Clarithromycin (Biaxin)
4.‐ Ciprofloxacin (Cipro)

3128 A client is taking bismuth for diarrhea. For which of Correct answer: 1 Bismuth‐containing preparations, such as Pepto‐Bismol, can cause all the listed side effects, The critical word in the stem of this question is unique. This means that the answer is a
the following side effects unique to this medication but transient darkening of the tongue and stool is a side effect specific to bismuth. side effect that does not occur with other drugs that the client is receiving. Use specific
would a nurse monitor? medication knowledge and the process of elimination to make a selection.

1.‐ Darkening of the tongue


2.‐ Dyspepsia
3.‐ Abdominal pain
4.‐ Diarrhea
3129 A client’s breath urease test is positive for Correct answer: 3 The highest rate of eradication of Helicobacter pylori infection is achieved by using a proton The core issue of the question is knowledge of what medications are commonly used in
Helicobacter pylori organisms. The nurse anticipates pump inhibitor and two antibiotics (usually clarithromycin and amoxicillin or metronidazole). treating Helicobacter pylori infection. Use medication knowledge and the process of
that which of the following medications will be The combinations of medications in options 1, 2, and 4 do not provide for this level of elimination to make a selection.
ordered to eradicate this infection most effectively? effectiveness.

1.‐ Antacids and amoxicillin


2.‐ Omeprazole, ranitidine, and amoxicillin
3.‐ Combination of proton pump inhibitor, amoxicillin, and clarithromycin
4.‐ Clarithromycin and bismuth salicylate

3130 The nurse would interpret that which of the following Correct answer: 1 Promethazine is usually given 25 mg every 4–6 hours prn. Dosing may start at 12.5 mg every The core issue of the question is knowledge of the dosing schedule for various antiemetics.
medications is ordered at a safe and effective dosage 4–6 hours prn depending on client status; however, 25 mg is the usual dose. Normal doses of Use medication knowledge and the process of elimination to make a selection.
range for an adult client who is experiencing nausea the other medications are as follows: prochlorperazine 5–10 mg t.i.d.–q.i.d.; metoclopramide
and vomiting? 10 mg 30 minutes AC and HS; and trimethobenzamide hydrochloride 250 mg t.i.d.–q.i.d. prn.

1.‐ Promethazine (Phenergan) 25 mg every 4–6 hours prn


2.‐ Prochlorperazine (Compazine) 200 mg every 6 hours
3.‐ Metoclopramide (Reglan) 30 mg ac and HS
4.‐ Trimethobenzamide hydrochloride (Tigan) 20 mg t.i.d. prn

3131 A client is receiving omeprazole (Prilosec) for Correct answer: 3 Omeprazole can cause an increase in liver enzyme levels (AST, ALT, alkaline phosphatase, and The core issue of the question is knowledge of the laboratory values that could be affected
esophageal reflux. The nurse makes it a priority to bilirubin), leading to adverse reactions of liver necrosis and hepatic failure. For this reason, the by administration of omeprazole. With this group of medications, it is necessary to
monitor the results of which of the following nurse should monitor these lab values as they become available. Monitoring of BUN, uric acid, remember the liver. Use medication knowledge and the process of elimination to make a
laboratory studies? and WBC count have a lesser priority; they are monitored only as indicated based on an selection.
individual client s identified health need.

1.‐ Blood urea nitrogen (BUN)


2.‐ Uric acid
3.‐ Liver enzymes
4.‐ White blood cell (WBC) count

3132 When caring for a client with onset of severe nausea, Correct answer: 2 All the medications listed are antiemetic agents, but transdermal scopolamine has the fastest The core issue of the question is knowledge of onset of action of various antiemetics.
the nurse telephones the health care provider for an onset of action. For this reason, it is most effective in providing relief from nausea for a Recall that transdermal systems begin to absorb into the skin immediately after application,
order for which emetic that has the fastest onset of prolonged period of time. while oral doses take various amounts of time to absorb through the GI tract. Use
action? medication knowledge and the process of elimination to make a selection.

1.‐ Oral promethazine (Phenergan)


2.‐ Scopolamine transdermal (Transderm‐Scop)
3.‐ Oral metoclopramide (Reglan)
4.‐ Haloperidol (Haldol)

3133 A client has been diagnosed with severe erosive Correct answer: 2 Because of their antisecretory effect, proton pump inhibitors such as omeprazole are the The core issue of the question is knowledge of the various GI medications that are useful
esophagitis. The nurse developing a medication drugs of choice for moderate‐to‐severe erosive esophagitis. The course of therapy is usually 4‐ in treating digestive system health problems. Recall that disorders that end in ‐itis involve
teaching plan would be sure to include information 8 weeks. The other medications might be helpful for certain clients; however, proton pump inflammation, so the correct answer is one that reduces inflammation either by its own
about which of the following most appropriate inhibitors are the medication classification of choice. action or by inhibiting other irritants, such as gastric acid. Use medication knowledge of
medications to treat the disorder? omeprazole as a proton pump inhibitor to make a selection.

1.‐ Sucralfate (Carafate)


2.‐ Omeprazole (Prilosec)
3.‐ Nizatidine (Axid)
4.‐ Amoxicillin (Amoxil)
3134 A client has been advised to take an antacid to Correct answer: 1 Antacids should be chewed well and followed with four ounces of water for optimal effect. The core issue of the question is knowledge of medication administration procedures for
neutralize gastric acid, and thereby decrease the pain They should be taken regularly after meals, but the client should allow at least two hours antacids. Recall that antacids are more effective when given with fluid to help disperse
of gastric irritation. What antacid‐related between taking the antacid and any other oral medication. Antacids should not be taken for medication in the stomach. Use medication knowledge and the process of elimination to
administration issues should be discussed with the longer than two weeks without further evaluation. They can be taken before gastric upset make a selection.
client? occurs for better symptom control.
1.‐ To chew tablets and follow with four ounces of water
2.‐ To take the dose of antacid within two hours of any other prescribed medication
3.‐ To take the antacids on a regular basis for up to six weeks
4.‐ To take antacids after the onset of episodes of gastric irritation

3135 A client who has a history of glaucoma is diagnosed Correct answer: 1 Clients with glaucoma should not take anticholinergic agents such as dicyclomine, because The core issue of the question is knowledge of medications that are contraindicated with
with a gastrointestinal disorder. The nurse would the medication affects pupillary dilatation, and therefore indirectly affects the outflow of glaucoma. Specific medication knowledge is needed to answer the question. Use
consult with the physician after noting a medication aqueous humor. The other medications listed do not pose this problem to the client. medication knowledge and the process of elimination to make a selection.
order for which of the following medications that
should be used very cautiously, if at all, for this client?

1.‐ Dicyclomine (Bentyl)


2.‐ Omeprazole (Prilosec)
3.‐ Metoclopramide (Reglan)
4.‐ Magnesium hydroxide (Milk of Magnesia)

3136 A client who needs to take a histamine Correct answer: 4 Cimetidine decreases metabolism of beta blockers, phenytoin, procainamide, quinidine, The core issue of the question is knowledge of histamine antagonists that are highest in
H&lt;sub&gt;2&lt;/sub&gt; antagonist also has a benzodiazepines, metronidazole, tricyclic antidepressants, and warfarin, leading to increased side or adverse effects. Specific medication knowledge is needed to answer the question.
history of several health problems. The nurse explains risk of drug toxicity. Ranitidine, famotidine, and nizatidine are histamine blockers that are Use medication knowledge and the process of elimination to make a selection.
to this client that which histamine antagonist should newer than cimetidine, and have fewer side effects.
be avoided because it has the greatest number of drug
interactions?
1.‐ Famotidine (Pepcid)
2.‐ Ranitidine (Zantac)
3.‐ Nizatidine (Axid)
4.‐ Cimetidine (Tagamet)

3137 A client newly diagnosed with a gastric ulcer has been Correct answer: 2 Sucralfate forms an adhesive barrier on the surface of the gastric mucosa, protecting it from The core issue of the question is knowledge of the mechanism of action of sucralfate
prescribed sucralfate (Carafate). The nurse explains gastric acid. It does not reduce spasms, relieve nausea or vomiting, or act as an anticholinergic. (Carafate). Specific medication knowledge is needed to answer the question. Use this
that this medication will have which of the following knowledge and the process of elimination to make a selection.
beneficial effects for the client?
1.‐ It will reduce GI spasms.
2.‐ It will protect the eroded ulcer surface from stomach acid.
3.‐ It will help relieve nausea and vomiting.
4.‐ It will act as an anticholinergic.

3138 A client with chronic pancreatitis has been prescribed Correct answer: 3 Pancrease, a pancreatic enzyme replacement, increases digestion of starches and fats, and The core issue of the question is knowledge of the mechanism of action of pancrease.
pancrelipase (Pancrease). The nurse who is teaching thereby decreases the incidence of steatorrhea (fatty, frothy, foul‐smelling stools). Each of the Specific medication knowledge is needed to answer the question. Use this knowledge and
the client about this medication would include that other options is only partially correct. the process of elimination to make a selection.
pancrelipase increases digestion of what foods?

1.‐ Proteins and starches


2.‐ Proteins and fats
3.‐ Starches and fats
4.‐ Vitamins and starches
3139 A female client reports having diarrhea for the past Correct answer: 4 Associated symptoms of fever, abdominal pain, and dehydration might suggest pathological The stem of the question contains the critical words highest‐priority. This tells you that
24 hours. She took loperamide (Imodium) all day diarrhea. The health care provider should be contacted for further evaluation. The other more than one of the options could be partially or totally correct. Use general nursing
yesterday per dosage instructions without relief. actions listed do not provide for the current physiological needs of the client. knowledge about diarrhea associated with fever, and the process of elimination, to make a
Today, she has a temperature of 102ºF, excessive selection.
thirst, and severe abdominal cramping. What is the
highest‐priority action for the nurse at this time?

1.‐ Obtain a further history of digestive disorders.


2.‐ Discuss dietary factors that might be causing the diarrhea.
3.‐ Suggest acetaminophen (Tylenol) for fever and pain.
4.‐ Notify the health care provider.

3140 A parent of a 2‐year‐old child asks the nurse why Correct answer: 2 Bismuth subsalicylate contains small amounts of naturally occurring lead, but Reye's Note the critical word salicylate in the stem of the question. Immediately associate this
bismuth subsalicylate (Pepto‐Bismol) should not be syndrome is a theorized complication of salicylate use in young children. Taste and darkening word with aspirin, which is contraindicated for use in children because of the risk of
used for diarrhea in children under 3 years old. The of the tongue are not the issue being addressed in this question. developing Reye's syndrome.
nurse should include which of the following rationales
in a response?
1.‐ It has an offensive taste that children do not like.
2.‐ It could lead to development of Reye's syndrome because of its salicylate content.
3.‐ It has a higher‐than‐recommended lead content.
4.‐ It can cause darkening of the tongue, which is frightening for children of that age.

3141 The nurse is giving follow‐up instructions to a client Correct answer: 1 Methylcellulose is a bulk‐forming cellulose that absorbs intestinal fluids. This action helps The core issue of the question is knowledge of a medication that relieves both
with irritable bowel syndrome (IBS). The nurse prevent constipation and reduce or eliminate diarrhea. Bisacodyl and docusate sodium are constipation and diarrhea. With this in mind, you need to select a medication that regulates
provides information about which medication that laxatives, while dicyclomine is an antispasmodic. the bowel. Use medication knowledge and the process of elimination to make a selection.
would be beneficial to treat both the diarrhea and
constipation associated with IBS?

1.‐ Methylcellulose (Citrucel)


2.‐ Docusate sodium (Colace)
3.‐ Dicyclomine (Bentyl)
4.‐ Bisacodyl (Dulcolax)

3142 A 26‐year‐old female client comes to the clinic for an Correct answer: 4 A serious adverse effect of misoprostol (Cytotec) is that a pregnant woman who takes the The core issue of the question is associated risks of taking misoprostol during pregnancy.
annual health examination. She has been taking medication could experience a miscarriage. Misoprostol should be discontinued at least one Eliminate options 1 and 3 first because they do not relate to misoprostol. Then choose
misoprostol (Cytotec) for several years following a month before pregnancy occurs. Options 1 and 2 have lower priority, and option 3 might not option 4 over option 2 because it is more specific to the medication.
gastric ulcer. She is getting married next month. What be necessary for this client.
is the priority nursing intervention for this client?

1.‐ Discuss whether to continue taking her oral contraceptive.


2.‐ Discuss family planning.
3.‐ Ask the client about the need for sexually transmitted disease (STD) counseling.
4.‐ Explain the risks of using misoprostol during pregnancy.
3143 An 82‐year‐old female who has had compensated Correct answer: 1 Bisacodyl is a stimulant laxative that can cause fluid and electrolyte imbalance. This can have Note that the question contains the critical word hyponatremia. With this in mind,
congestive heart failure for years comes to the additive effects, because the diuretic use would also contribute to this finding. For this reason, evaluate each option in terms of its relevance to the low sodium value. Eliminate each of
ambulatory care center for a quarterly routine health the nurse should assess the use of the laxative. The other options suggest items that would not the incorrect options, because they would not cause the electrolyte imbalance stated in the
examination. At this time, she reports increased help determine the cause of the client s current symptoms. question.
fatigue, weakness, and dizziness, although physical
examination findings are normal. Laboratory results
include a normal CBC; sodium 123 mEq/L; and
potassium 3.5 mEq/L. Her medications include Lasix 20
mg daily; K‐Dur 10 mEq daily; Lanoxin 0.125 mg daily;
and prn bisacodyl (Dulcolax) suppository. What
information would be helpful to evaluate the cause of
the client s hyponatremia?

1.‐ How frequently the client uses the bisacodyl suppository


2.‐ Whether the client has skipped digoxin doses in the last week
3.‐ Whether the client has taken excess doses of K‐Dur
4.‐ How frequently the client eats salty foods in restaurants

3144 The nurse would recommend which type of laxative Correct answer: 1 Bulk‐forming laxatives, such as methylcellulose, absorb intestinal fluid, increasing stool The critical words in the stem of the question are best choice. This tells you that more
as the best choice for a client with constipation and a volume, stimulating peristalsis, and decreasing straining on defecation. This type of laxative is than one option might be partially or totally correct, but one option is best. Keeping in mind
history of coronary artery disease and congestive heart the best choice for a client with a history of heart disease complicated by heart failure. The that the client has heart disease, use the process of elimination to choose the bulk‐forming
failure? laxatives in the other options are more likely to cause straining at stool for the client, and laxative as least likely to cause strain on the heart.
therefore are less helpful for the client s overall status.

1.‐ A bulk‐forming laxative


2.‐ A saline laxative
3.‐ A stimulant laxative
4.‐ PRN enemas

3145 The client comes to the office to get a refill of a Correct answer: 2 Dicyclomine HCl is an antispasmodic drug. Peripheral side effects include hot, flushed, dry The core issue of the question is adverse effects of dicyclomine. A clue in the question is
prescription for dicyclomine hydrochloride (Bentyl). skin; hyperthermia; and intolerance to high temperatures manifested by dizziness. The client the reference to Florida, which suggests that option 2 (high temperature) is the correct
She tells the nurse she is leaving the next day for a should be advised to drink extra fluids (nonalcoholic) if exposed to high temperatures to option.
vacation to Florida. What education should the nurse reduce this occurrence.
provide this client?
1.‐ “You do not need to be so concerned about taking the medication on vacation, because you probably won t be able to watch your diet very well anyway.”
2.‐ “This medication may make you more sensitive to high temperatures, which could lead to dizziness.”
3.‐ “You probably should discontinue the medication until you return home, because alcohol can increase the drug s effects on the central nervous system.”
4.‐ “While you are away, you should take antacids with this medication to decrease any symptoms of GERD.”

3146 A mother rushes her 2‐year‐old child to the urgent Correct answer: 2 Activated charcoal absorbs ipecac syrup, thus decreasing its effect by inhibiting absorption When selecting the correct answer to the question, note that the question stem asks for
care clinic. She has given the child two doses of ipecac from the GI tract into the general circulation. While calling the poison control center is selection of the priority option. Calling the poison control center first would delay
syrup following ingestion of a bottle of children's important, it is not the highest‐priority action to ensure the safety of the client. Option 3 is treatment, and would not be the first choice. Giving the child activated charcoal is the first
aspirin. It has been over 40 minutes, and the child has incorrect, and option 4 could result in harm to the client. choice to slow absorption of the ipecac, and then calling the poison control center would be
still not vomited. What should the nurse do first? the next.

1.‐ Call the poison control center.


2.‐ Give activated charcoal.
3.‐ Offer milk or carbonated soda.
4.‐ Wait for physician to assess the client.
3147 The nurse is caring for a client with gastroesophageal Correct answer: 1 Metoclopramide is a GI stimulant, increasing motility of the GI tract and shortening gastric In order to correctly select the answer to this question, recall knowledge of the purpose of
reflux disease (GERD) who is taking metoclopramide emptying time. The other options do not represent actions of this medication. the medication. If this was difficult, review the purpose of the medication.
(Reglan). The nurse determines that the client
understands the purpose of the medication when the
client verbalizes that the medication has which of the
following actions?

1.‐ Increases GI motility.


2.‐ Decreases GI motility.
3.‐ Combats diarrhea.
4.‐ Kills <i>H. pylori</i> organisms.

3148 A client is taking dicyclomine (Antispas) for irritable Correct answer: 4 Dicyclomine is a cholinergic‐blocking agent that decreases hypermotility and spasms of the GI In order to answer the question correctly, review the method of administration. If this was
bowel disorder. The nurse explains to the client that tract. The dose should be taken before a meal to be effective when needed. difficult, review the methods of administration in a medication handbook.
which of the following represents the optimal dosing
for dicyclomine?
1.‐ Take after meals.
2.‐ Take with meals.
3.‐ Take only as needed.
4.‐ Take 30–60 minutes before meals and bedtime.

3149 A male client has been diagnosed with acute diarrhea. Correct answer: 2 Pepto‐Bismol (bismuth salicylate), is contraindicated in clients who are allergic to aspirin or Knowing the compositions of the medications will lead to the correct answer. Realizing
He is allergic to aspirin (ASA), and takes no salicylates. The other medications can be given to the client safely. that salicylate refers to aspirin, as is listed in the question stem, will lead to the only correct
medications. Which antidiarrheal medication should answer that contains aspirin, option 2.
not be given to this client?
1.‐ Kaopectate
2.‐ Pepto‐Bismol
3.‐ Lomotil
4.‐ Imodium

3150 An ambulatory care nurse is working with a client Correct answer: 4 Metamucil is a bulk‐forming laxative that could aggravate diarrhea. Kaopectate (option 2) is Note that the question stem asks for selection of the option that would establish that the
who has come to the clinic because of diarrhea. The an antidiarrheal agent that is commonly used to manage this health problem, which is usually client needs further teaching. Note that the words “bulk‐forming” are present in option 4.
nurse determines that the client needs instruction self‐limiting. The client should contact the health care provider again if diarrhea persists This would lead you to suspect that diarrhea would be aggravated, with more stool
about management of this health problem when the (option 3), because diarrhea lasting more than two days requires attention. Dairy products are produced, if a “bulk‐forming” agent were consumed. The other answers all indicate that the
client states to do which of the following? a food source that can aggravate diarrhea (option 1). client understands management of the illness.

1.‐ Avoid dairy products.


2.‐ Take Kaopectate as directed.
3.‐ Contact health care provider if symptoms not resolved in two days.
4.‐ Use a bulk‐forming agent, such as Metamucil.

3151 Because discoloration of client's stool to dark green Correct answer: 4 To prevent unnecessary anxiety, the nurse should tell the client to expect the stool change Recall that ferrous sulfate therapy causes black, tarry stools; a harmless side effect.
or black is a side effect of oral ferrous sulfate, the (option 4). Since this is a harmless side effect, no change in initial process in needed (option 1).
nurse performs which of the following: Taking the iron with food can reduce GI side effects such as nausea/vomiting, but will not
prevent the dark stools. Ingestion of food with iron will decrease absorption (option 2).
Administering the drug on an empty stomach can increase absorption, but will not prevent the
change in stool color (option 3).
1.‐ Consult with the prescriber about ordering the drug IM rather than oral.
2.‐ Administer the drug with food to avoid this side effect.
3.‐ Administer the drug on an empty stomach to avoid this side effect.
4.‐ Inform the client to expect a change in stool color.
3152 A 3‐year‐old client weighing 33 pounds is to receive Correct answer: 7.5 Use the following formula to calculate: <BR /> Use the X equation to determine the correct amount.
liquid Advil (ibuprofen) 150 mg PO q6 hours prn for
temperature above 101 degrees F. The nurse should
administer _____ mL to the client from a bottle
labeled 100 mg/5 mL. Fill in the numeric answer
below.

3153 An older adult client ingesting glyburide (Micronase) Correct answer: 2 Because of the drug’s action (hypoglycemic), duration (24 hours) and the half‐life (10 hours), Determine the client’s health maintenance ability based on the content of the statements
1.25 mg PO daily makes which of the following the client could be at significant risk if the dose is doubled (option 2). The common dosage is to select the correct answer.
statements that would cause the home health nurse to once daily before or with breakfast (option 1). Ingestion of alcohol with the drug can result in a
schedule another home appointment for the following disulfiram reaction or excessive sympathomimetic reaction (option 3). HBA1c provides
day? information regarding the glucose level over a 4 month period (option 4).

1.‐ "I take the tablet with breakfast every day."


2.‐ "If I forget the dose on one day, I will take two tablets the next day."
3.‐ "I will avoid drinking alcohol while taking this drug."
4.‐ "I will keep appointments to have my HBA1c checked every 4 months."

3154 After receiving digoxin (Lanoxin) 0.125 po daily for 5 Correct answer: 4 The base for the antidote is made from sheep's blood so the nurse should monitor for an Recall that the antidote (Digibind) is made from sheep's blood and may cause allergic
days, the client exhibited signs and symptoms of major allergic reaction (option 4). Response in adults occurs within 15‐60 minutes (option 1). reaction.
side effects associated with the drug. The nurse Hypokalemia is more likely to occur (option 2). The client should be monitored for at least 2‐3
administers the antidote and does which of the weeks (option 3).
following?
1.‐ Monitor serum digoxin at least 12 hrs after administration of antidote
2.‐ Monitor for hyperkalemia
3.‐ Inform prescriber improved cardiac rate occurred 2 hours after administration
4.‐ Monitor for allergic reaction

3155 When the nurse is administering medication to a Correct answer: 2 Because the identification bracelet is tangible and attached to the client, it is the most Select the option that offers the best method for communication of the information.
hospitalized client, which of the following is the most appropriate method for determining client identity prior to administering medication. The
accurate way to assure that the right client gets the bracelet contains the client’s name as well as the medical record number. Within the last few
medication? years, it has become acceptable to ask an oriented client to state his or her name and date of
birth. The nurse must have tangible information to match the client’s response (option 2).
Hospitalized or ill clients are often anxious, medicated, or confused and could respond
incorrectly to this question (option 1). A nurse administering medications may be unaware that
the UAPs/NAs may have changed room and bed assignments. Therefore, there is a very high
risk of incorrect identification (option 3). Because the same diagnosis with the same
medication may be appropriate for more than one client, matching the medication to the
diagnosis or need will place the client at risk (option 4).

1.‐ Ask the client, "Are you (<i>name</i>)?"


2.‐ Check the client's identification band
3.‐ Check the client's room number and bed assignment
4.‐ Match the medication with the client’s diagnosis

3156 The client received isophane (NPH) insulin 30 units SC Correct answer: 3 4 to 12 hours is the peak time period for NPH (option 3). 2 to 3 hours is the peak time period Associate this drug with an intermediate range.
at 7:30 A.M. What time is the nurse most likely to for Regular insulin (option 1). 4 to 7 hours is the peak time period for Semilente (option 2). 8 to
observe signs or symptoms of hypoglycemia? 12 hours is the peak time period for Lente (option 4).

1.‐ 9:30 am to 10:30 am


2.‐ 11:30 am to 2:30 pm
3.‐ 11:30 am to 7:30 pm
4.‐ 3:30 pm to 7:30 pm
3157 The client asks the nurse, "Can you give me the Correct answer: 3 Heparin will be destroyed by the gastric juices if taken orally (option 3). Because it will be not Recall the correct administration of Heparin and why it is given in this way.
heparin dose in a pill or liquid form instead of these be absorbed intact, it is not produced in an oral form (option 1). Option 2 is not the reason
injections?" The most appropriate nursing response Heparin is not administered orally. Option 4 is an incomplete response. While the drug
includes: company does not produce in tablet form, this is not the reason it is not administered orally
(option 4).
1.‐ "The oral route could be used if I call the physician first."
2.‐ "When taken orally Heparin may cause ulcerations.”
3.‐ "Gastric juices will destroy Heparin if taken orally."
4.‐ "The drug company does not produce Heparin in an oral form."

3158 The client taking diltiazem hydrochloride (Cardizem) Correct answer: 4 Grapefruit juice changes the metabolism of the calcium channel blocker drugs and leads to an Apply knowledge of the primary cause of toxicity in calcium channel blockers.
30 mg PO qid is experiencing symptoms of toxicity. increase in blood levels of the drug resulting serious signs and symptoms such as dysrhythmia,
Which of the following assessments should be the angina, heart block, bradycardia, and hypotension (option 4). Assessment of respiratory
nurse's highest priority? pattern may be important because of the toxicity, but does not address the primary problem
directly (option 2). Assessment should focus on the cause. There is no direct relationship
between altered body temperature and toxic levels of this drug (option 1). Weight gain is an
adverse effect of this drug. The highest priority is insisting the client ingest this drug safely.
Identification of the cause of the toxicity takes the highest priority (option 3).

1.‐ Client's body temperature, looking for elevation


2.‐ Rate, depth and regularity of the client's respirations
3.‐ Client's daily weight, looking for weight loss
4.‐ If client has ingested grapefruit juice with meals

3159 A client who is taking several prescribed oral Correct answer: 1, 3 Drug interactions, drug side effects, drug actions, and drug absorption should be examined Select options providing the most effective approach to the drug therapy.
medications for the treatment of congestive heart before ingesting multiple prescriptions (option 1). Reaching the goals of the drug therapy
failure tells the nurse, "I usually take all of my would indicate that this approach is appropriate (option 3). Option 2 is a correct statement,
medication with my breakfast." The nurse's best but is not comprehensive. The time is only one of the elements of concern (option 4). The
response includes which of the following? Select all nurse needs to be sure there are no significant risks (option 5).
that apply.
1.‐ "It might be good idea to examine each drug to determine the best times and conditions to ingest.”
2.‐ "That depends on what you eat at breakfast."
3.‐ “Are you reaching the goals of the drug therapy?”
4.‐ "What time do you usually have breakfast?"
5.‐ "That's a great idea."

3160 While talking with a client, the nurse finds that the Correct answer: 3 If the client were unaware of the risks of bleeding and ease of tissue damage, appropriate Lack of knowledge places client at risk for injury.
client is unaware of the side effects of warfarin protective mechanisms would not be in place so injury is most likely to occur (option 3). There
(Coumadin). Which of the following nursing diagnoses is no evidence of an oxygen deficit (option 1) nor evidence of reduced self‐management
best describes the client’s current status? (option 2). There is no evidence that the client is at risk for infection (option 4).

1.‐ Ineffective gas exchange


2.‐ Reduced self‐management
3.‐ Risk for injury
4.‐ Risk for infection
3161 The medication administration record shows that the Correct answer: 4 This is an example of two different drugs in the same sub‐category with very similar names. Apply principles of drug administration to select the correct answer.
client is to receive lisinopril (Zestril) 10 mg PO at 9:00 The question of which drug should be administered to the client is answered by checking the
A.M. On hand are tablets labeled fosinopril (Monopril) original physician order and giving that drug (option 4). These actions violate matching the
20 mg. Which action by the nurse ensures that the drug with the MAR (options 1 &amp; option 2). This action violates using the appropriate
right drug and the right dose are administered? sources to validate the accuracy of a transposed order (option 3).

1.‐ Give one tablet of Monopril from the drug supply


2.‐ Given one‐half tablet of Monopril from the drug supply
3.‐ Ask the client if the 20 mg tablet looks familiar
4.‐ Read the original physician order to verify the drug order

3162 A nurse educator is conducting a seminar for clients Correct answer: 3 Vitamin K is the antidote for the actions of Coumadin; any food (such as broccoli or cabbage) Recall that some vegetables have a polar relationship with the drug.
ingesting warfarin (Coumadin). The nurse instructs the or drug that contains vitamin K reduces the effects of the drug (option 3). Yellow vegetables
clients to make which of the following dietary have no overlapping characteristics with Coumadin (option 1). Coffee and tea have no known
adjustments? overlapping characteristics with the drug (option 2). Vitamin C may increase the risk of
bleeding. The nurse would not want to enhance the effects of Coumadin (option 4).

1.‐ Reduce intake of yellow vegetables


2.‐ Reduce intake of coffee and tea
3.‐ Reduce intake of foods like broccoli and cabbage
4.‐ Increase intake of citrus fruits to enhance effect of the drug

3163 Which of the following data indicates that the client Correct answer: 1, 2, 3, When microbes become established in body tissues, an inflammatory process develops which Apply knowledge of the inflammatory process to select the correct answer.
with cellulitis who is receiving an antiobiotic has been 4 required a significant increase in the metabolism. An increased metabolism results in an
achieved satisfactory response to the medication? elevated body temperature. A continued elevation in body temperature indicates the microbes
Select all that apply. continue to overcome the body defenses. The prescribed antibiotic needs to be re‐evaluated
(option 2). When the offending agent is established and growing fluid drains into the area to
reduce the irritation caused by the toxins. Decrease swelling would indicate the antibiotic has
successfully reduced the number of microbes (option 1). Pain at the site is related to increased
pressure on nerve ending. Reduction in the level of pain is indicative of decreased
inflammation (option 3). When microorganisms are multiplying the WBC increase in number
and migrate to the site. A reduced serum WBC would indicate the antibiotic is effective
resulting in a reduced number of microorganism (option 4). Enlarged regional lymph nodes are
common signs/symptoms of the disease (option 5).

1.‐ Swelling of the right arm is reduced


2.‐ Body temperature is 101 degrees F
3.‐ Client reports less pain in right arm
4.‐ White blood cell count has decreased
5.‐ Regional lymph nodes are enlarged

3164 The nurse is planning to instruct a Hispanic American Correct answer: 1 Because a client of Hispanic heritage often lives in the community with an extended family, Apply knowledge of this culture with this scenario to select the correct answer.
client about the drug regimen prescribed for newly the nurse should include the family members in the educational process (i. e., privacy rights
diagnosed hypertension. When developing the plan, have been addressed). Since men are typically decision makers, the nurse should attempt to
the nursing actions should be based on which of the include the male in this client’s life in the teaching sessions. The nurse must be sensitive to the
following most relevant cultural information? client's wishes regarding privacy when doing health teaching. Because this cultural group
cherishes its language, teaching materials in the native language will be appreciated (option 1).
Whether or not this culture values education in all forms does not have a direct relationship to
teaching the client about the medication (option 2). This culture prefers more personalized
approach to health care (option 3). Disliking the use of medication to treat disease is aligned
more with the practice of Christian Scientist (option 4).

1.‐ The client lives in an extended family setting in which men are the decision makers
2.‐ The client values education in all forms
3.‐ The client prefers using written materials as a way of learning
4.‐ The client dislikes using medication to treat health problems

3165 The nurse is administering tetracycline hydrochloride Correct answer: 3 The drug tetracycline is absorbed best when given to the client on an empty stomach with a Remember that oral administration of this drug increases the risk of decreased absorption.
(Tetracyn) 500 mg PO QID to a client with gonorrhea. glass of water (option 3). Food (option 1), dairy products (option 2), iron, and antacids
Which of the following nursing actions best matches decrease the absorption of this drug (option 4).
the client’s needs?
1.‐ Administer drug with meals
2.‐ Give drug with a full glass of milk to prevent gastric irritation
3.‐ Give drug on an empty stomach with a full glass of water
4.‐ Give drug with a glass of orange juice or other source of vitamin C

3166 While still in the recovery room, a client with a large Correct answer: 2 The normal dosage range for morphine administered IV is 2.5‐15 mg q4 hours. Since the nurse MS is a potent narcotic. If dosage and pain intensity are not matched, then adjustment is
abdominal wound requested a second injection of cannot alter the order, the health care provider needs to be contacted (option 2). Abdominal needed. Note that current order is less than maximum allowed.
morphine sulfate (MS) 3 mg IV 2 hours after the first pain on the same day of surgery is usually very severe. Since the level of pain appears to be
injection. If the order reads “MS 4‐10 mg IV q 3‐4 hours appropriate for the surgical procedure, assessing the wound to aid in determining the
prn pain,” the nurse should perform which of the appropriateness of the pain response is illogical (option 1). Assessing the respiratory status
following? needs to occur after it is determined an injection will be administered (option 3). Since the
client has a large abdominal wound, the general anesthesia along with administration of
analgesics will result in drowsiness for the next 24 hours. It will not aid in resolving the
problem of the imbalance between the dosage level and the pain response (option 4).

1.‐ Assess the surgical wound


2.‐ Contact the prescriber
3.‐ Assess the respiratory status
4.‐ Assess the level of consciousness (LOC)

3167 The mother of the pediatric client asks the nurse, Correct answer: 2 This response answers the questions and provides safe instructions. Changing from brand Select the option that is most accurate and places the client at the least amount of risk.
"What is the difference between Advil and ibuprofen? I name drugs to generic drugs should be done by the health care provider (option 1). This is an
can buy ibuprofen at a cheaper price, but the inaccurate statement. Advil, and enteric coated tablet, is a brand name of ibuprofen. Not all
instructions from the clinic say to use Children's Advil ibuprofen is enteric coated (option 2). The drugs are the same. Ibuprofen is the generic name.
Liquid." The nurse’s best response includes: Option 3 does not answer the question. It is inappropriate to make a referral when the nurse is
capable of meeting the need (option 4).

1.‐ "Similarities do exist, but follow the instructions for now.”


2.‐ "There is no difference between Advil and ibuprofen."
3.‐ "Advil and ibuprofen are two different drugs with similar effects."
4.‐ "You need to talk to the person that prescribed the drug.”

3168 A client is taking bismuth for diarrhea. For which side Correct answer: 1 Bismuth‐containing preparations, such as Pepto‐Bismol, can cause transient darkening of the In order to answer this question correctly, recall the side effects of bismuth. The only
effect unique to this medication would a nurse tongue and stool as a side effect. correct answer for this question is option 1. The other options are all incorrect. If this was
monitor? difficult, review the side effects to bismuth.
1.‐ Darkening of the tongue
2.‐ Dyspepsia
3.‐ Abdominal pain
4.‐ Diarrhea
3169 After observing the client taking phenelzine sulfate Correct answer: 4 The foods are high in tyramine, a chemical that can result in a hypertensive crisis if ingested Associate negative reactions between MAOIs and foods containing tyramine.
(Nardil), eating a lunch of yogurt, sliced bananas, and while taking MAOIs such as phenelzine (option 4). The central nervous system side effects
chocolate milk, the nurse should perform which of the involve mental alterations such confusion and anxiety rather than physical changes such as
following? temperature elevation (option 1). Respiratory depression is more likely to occur (option 2).
Diabetes mellitus is not associated with the drug (option 3).

1.‐ Monitor client's body temperature for elevation above normal


2.‐ Observe client for dyspnea
3.‐ Test a urine specimen for glucose and ketones
4.‐ Monitor client for elevated blood pressure

3170 After a prescriber writes a medication order for a Correct answer: 2 The nurse has a legal and ethical responsibility to assure that the client receives the correct Seek the resource that is able to resolve the problem.
client, the nurse determines that the dose is above the dosage. Since dosages may sometimes be outside recommended ranges, contacting the
usual dosage range. The nurse should perform which prescriber for clarification is appropriate. A nurse initiate a medication order after assuring the
of the following “rights” associated with medication administration have been met (option 2). When the
problem is resolved, both the prescriber and the nurse administering the drug will be in
agreement. This will not occur if the nurse talks with a second nurse (option 1). Appropriate
initiation of the drug administration process includes considering if the “right” dosage is
prescribed. The prescriber’s qualifications are not the central focus (option 3). Documentation
of concerns should be entered in organizational records rather than the client’s record (option
4).

1.‐ Ask another nurse about the dosage order


2.‐ Contact the prescriber about the dosage
3.‐ Recognize that the prescriber is qualified to write drug orders
4.‐ Document the concern in the client's record

3171 Because an over‐the‐counter (OTC) iron preparation Correct answer: 3 Pregnancy Category A is assigned to drugs that have not shown to have adverse effects on Apply knowledge of Pregnancy categories associated with drug therapy to select the
is categorized as Pregnancy Category A, the nurse fetal development. It is safe to ingest the drug during pregnancy (option 3). Option 1 is correct answer.
teaches a pregnant client which of the following most incorrect information. Clients should report to physician all OTC drugs ingested, but this
important information? response does not correlate with the pregnancy categories (option 2). option 4 is a side effect
of the tetracyclines.
1.‐ Should ingest the medication during pregnancy
2.‐ Immediately report to the physician that she has taken the drug while pregnant
3.‐ Safe to ingest this medication during pregnancy
4.‐ May be staining of the baby's first teeth from this medication

3172 The client taking theophylline (TheoDur) 16 mg/kg PO Correct answer: 2 Since the drug has a narrow therapeutic range, toxicity can develop quickly. The normal Recall that this drug has a profound effect on the GI tract and cardiac function. Toxicity is
for asthma has a serum theophylline level of 22 therapeutic dose level is 10‐20 mcg/mL. Hence, the signs and symptoms are indicative of likely to have an effect on the 2 systems.
mcg/mL. Because the client is experiencing nausea, toxicity. The nurse should suspend the order until the prescriber is contacted (option 2).
vomiting, and headache the nurse should perform Procuring and administering antiemetic would reduce the signs/symptoms but would not
which of the following actions? resolve the problem (option 1). The lab test validates that toxicity exists (option 3). The nurse
should suspend the order until the dosage can be changed. The drug’s duration is 4‐8 hours.
Holding it for 24 hours would create the opposite problem (option 4).

1.‐ Seek an order for an antiemetic


2.‐ Temporarily suspend the order and notify the prescriber
3.‐ Ask if the client if mucous obtained during respiratory therapy was expectorated
4.‐ Stop ordered doses of TheoDur for 24 hours
3173 A client with advanced liver disease is being treated Correct answer: 3 The liver is the organ chiefly responsible for detoxification of drugs. Active drug may Recall that most drugs are transformed by the liver then excreted by the kidneys.
with antibiotics for a respiratory infection. The nurse accumulate to toxic levels in clients with impaired liver function (option 3). Every client
should assess the client for which of the following? receiving drugs, especially antibiotics, should be assessed for drug allergy (option 1). The
question does not state that the client is pregnant so teratogenic effects are not of concern in
this situation. Signs/symptoms of tetragenic effects are too vague and too complex to be
assessed during a general assessment (option 2). Antibiotics do not cause drug dependence to
develop (option 4).
1.‐ High risk for allergy to drugs
2.‐ Teratogenic effects of antibiotics
3.‐ Symptoms of drug toxicity
4.‐ Drug dependence

3174 Which of the following physical changes commonly Correct answer: 1, 2 Since elderly clients experience a decreased rate of drug excretion, reduction of dosage Select the options that would place the client at greatest risk and that correlate closest to
associated with aging are most likely to require a would be appropriate (option 1). The decreased total body fluid proportion that accompanies the information in the question stem.
reduction in medication dosage in an elderly client? physical aging increases the concentration of water‐soluble drugs and requires lower dosing in
Select all that apply. older adults (option 2). Decreased efficiency in drug distribution would not correlate with a
need to lower the dosage (option 3). Elderly clients experience a decreased rate of drug
metabolism (option 4). Most older adults tend to lose weight as they age (option 5).

1.‐ Increased rate of drug retention


2.‐ Decreased total body fluid proportionate to body mass
3.‐ Decreased efficiency in drug distribution
4.‐ Decreased rate of drug metabolism by the liver
5.‐ Significant weight gain

3175 A health care provider prescribed a prochlorperazine Correct answer: 3 The nurse should consult with the pharmacist before contacting the prescriber (option 3). Remember to presume the drug is not evenly distributed throughout the medium.
(Compazine) 12.5 mg suppository for a client with Drug companies do not guarantee that the medication is equally distributed throughout the
severe nausea. The nurse has on hand a 25 mg medium (option 1). The dosage is not stable across routes. The nurse may not change
suppository. The next step for the nurse includes which prescriptions (option 2). After the dosage problem is resolved, the rectal vault should be
of the following? cleared adequately to guarantee maximum absorption (option 4).

1.‐ Cut the suppository in half and insert the rounded edge
2.‐ Administer the drug orally
3.‐ Contact the prescriber, if unable to locate the dosage

4.‐ Clear the rectal vault of the contents before administering half of the suppository

3176 A client is taking nalidixic acid (NegGram) for Correct answer: 3 Nalidixic acid is bactericidal, and inhibits microbial synthesis of DNA. The spectrum includes The core issue of the question is knowledge of basic information about nalidixic acid and
treatment of a urinary problem. The nurse explains to most Gram‐negative organisms except Pseudomonas. This medication does not belong to the its uses. Use the process of elimination and nursing knowledge to make a selection.
the client that the medication is best described as antispasmodic, antigout, or analgesic families.
which of the following?
1.‐ An antispasmodic
2.‐ A uricosuric
3.‐ An anti‐infective

4.‐ An analgesic
3177 The client with congestive heart failure (CHF) is eating Correct answer: 2 Spironolactone is a potassium‐sparing diuretic that promotes sodium excretion while The core issue of the question is knowledge of drugs that are potassium‐sparing diuretics.
a 1‐gram‐sodium diet, and will be having a potassium‐ conserving potassium. Options 1 and 3 are diuretics, but not potassium‐sparing diuretics. Use the process of elimination and nursing knowledge to make a selection.
sparing diuretic added to the medication regimen. The Option 4 is not a diuretic; it is an antihypertensive of the beta‐blocker type.
nurse prepares to conduct teaching about which
medication that is likely to be prescribed?

1.‐ Hydrochlorothiazide (HCTZ)

2.‐ Spironolactone (Aldactone)


3.‐ Furosemide (Lasix)
4.‐ Atenolol (Tenormin)

3178 The home health care nurse is visiting an elderly Correct answer: 4 Tea and coffee are poor choices for hydration. They are mild diuretics, and can cause severe The critical words in the stem of the question are need for further teaching. This tells you
client who is taking furosemide (Lasix) twice daily. dehydration if used concurrently with diuretics. Taking medication at the same time each day that the correct answer is an incorrect statement on the part of the client. Use the process
Which of the following statements made by the client improves compliance. In addition, morning and early evening are the best times to take Lasix, of elimination and knowledge of diuretic therapy to narrow the selection to the one that
indicates the need for further teaching? so as not to interrupt sleep. Notifying the physician when edema is noticed is important, and utilizes additional diuretic substances.
should be emphasized by the nurse.
1.‐ “I will take my medication in the morning and early evening.”
2.‐ “I will change my position slowly, so that I don t fall.”
3.‐ “I will notify my physician if my ankles swell.”
4.‐ “I will drink coffee and tea whenever I wish to get enough fluid.”

3179 A client is receiving dopamine (Intropin) therapy at 10 Correct answer: 3 The client receiving dopamine therapy should be assessed for impaired tissue perfusion The core issue of the question is knowledge that higher doses of dopamine can lead to
mcg/kg/minute. The nurse assesses the client for related to peripheral vasoconstriction. There is not enough information in the question to selective vasoconstriction, which in turn can reduce perfusion to peripheral tissues. Use the
evidence of which nursing diagnosis, for which the determine if option 1 is appropriate. Option 2 is not a nursing diagnosis, although decreased process of elimination and knowledge of dose‐related drug side effects to make a selection.
client is at risk? cardiac output is one. Option 4 does not relate to the question as stated.

1.‐ Excess Fluid Volume


2.‐ Increased Cardiac Output
3.‐ Impaired Tissue Perfusion
4.‐ Disturbed Body Image

3180 The nurse is reviewing the medication administration Correct answer: 1 Thiazide diuretics are sulfa‐based medications; therefore, a client with a sulfa allergy is at risk Remember that in order for an option to be correct, all of the parts of the option must be
record for a client newly admitted to the nursing unit for an allergic reaction. The side effects of hydrochlorothiazide are hypokalemia, correct. Options to which this strategy applies will include the word and. Recall that
for congestive heart failure. The client is receiving hyperglycemia, hyperuricemia, and hypercalcemia. Options 2, 3, and 4 are either partially or diuretics generally can cause hypokalemia or hyperkalemia as adverse effects. Since only
hydrochlorothiazide (HydroDiuril). Which of the totally incorrect. options 1 and 2 address these conditions, eliminate options 3 and 4. Recall that sulfa rather
following would concern the nurse in relation to than penicillin is of concern to choose option 1 over option 2.
administration of this medication?

1.‐ Hypokalemia, hyperglycemia, and sulfa allergy


2.‐ Hyperkalemia, hypoglycemia, and penicillin allergy
3.‐ Hypouricemia and hyperglycemia
4.‐ Hyponatremia and hypocalcemia

3181 The nurse is planning to administer furosemide (Lasix) Correct answer: 3 Furosemide should be given at a rate of 20 mg/minute or less. Rapid injection of furosemide Eliminate option 4 because it is not typical to dilute IV push medications in sterile water.
40 mg by the IV push route. The nurse uses which of can cause hearing loss as a result of ototoxicity. It does not need to be further diluted before Eliminate option 2 because it is an IV push medication, and a volume of 50 mL is usually
the following techniques in administering this injection (options 2 and 4). given as an IV piggyback. Choose option 3 over option 1 because of the size of the dose and
medication? knowledge of adverse effects.
1.‐ Pushes the medication steadily over 1 minute.
2.‐ Gives the medication slowly diluted in 50 mL of NS.
3.‐ Injects the medication over 2–3 minutes.
4.‐ Dilutes the medication with sterile water, and injects over 5 minutes.
3182 A client is being prescribed oxybutynin (Ditropan) for Correct answer: 1 Excessive dosing of oxybutynin produces nervousness, hallucinations, restlessness, Note that when two options are opposite, one of them likely is correct. Use this strategy
a neurogenic bladder. The nurse determines that the tachycardia, confusion, flushed or red face, and signs of respiratory depression. Options 2, 3, to eliminate options 3 and 4 first. Use the process of elimination and medication knowledge
client is possibly experiencing toxic effects of this and 4 are opposite effects of what would be expected in this case. to choose option 1 over 2.
medication after noting which of the following?

1.‐ Restlessness
2.‐ Drowsiness
3.‐ Pallor
4.‐ Bradycardia

3183 The nurse is admitting a client with a hypertensive Correct answer: 1 Furosemide is a loop diuretic. The antihypertensive action involves renal and peripheral Specific knowledge of the benefits of furosemide as a loop diuretic and as the diuretic of
emergency and a history of renal insufficiency. The vasodilation, a temporary increase in glomerular filtration rate (GFR), and decreased choice with renal insufficiency is needed to answer this question. Take time to learn about
nurse ensures that which of the following diuretics is peripheral vascular resistance. For this reason, it is the drug of choice for clients with low GFR this common medication if you had difficulty with this question.
readily available for use if ordered? as a result of renal insufficiency. Hydrochlorothiazide, chlorthalidone, and spironolactone are
not associated with use in clients with low GFR.

1.‐ Furosemide (Lasix)


2.‐ Hydrochlorothiazide (HCTZ)
3.‐ Chlorthalidone (Hygroton)
4.‐ Spironolactone (Aldactone)

3184 A client being discharged from the hospital is Correct answer: 2 Anuria is the absence of urine formation, and is a contraindication for using this medication. The core issue of the question is knowledge of expected effects and adverse effects of
beginning medication therapy with bumetanide Diuretics such as bumetanide are used to increase the amount of urine excreted in clients with loop diuretics such as bumetanide. Knowing that diuretics help to relieve the symptoms in
(Bumex). The nurse instructs the client to contact the CHF (option 2), pulmonary edema (option 3), and hypertension (option 4). options 1, 3, and 4 will help you to eliminate each of them.
prescriber if which of the following contraindications
for use develops while using this medication?

1.‐ Increase in peripheral edema


2.‐ Absence of urine output
3.‐ Shortness of breath
4.‐ Increase in blood pressure

3185 The nurse notes while taking an admission history Correct answer: 3 Acetazolamide is a carbonic anhydrase inhibitor. Inhibition of carbonic anhydrase decreases The critical word in the stem of the question is next, which tells you that more than one
that a client is taking acetazolamide (Diamox). The the rate of formation of aqueous humor, and thereby reduces intraocular pressure. follow‐up question might be appropriate, but you must select the most important one.
nurse next questions the client about a history of Acetazolamide may be used for treatment of edema caused by CHF, but it is not a first‐line Eliminate options 1 and 4 first because they are not targeted by this type of therapy.
which of the following medical conditions? therapy. This medication does not have a therapeutic effect on hypertensive crisis or Choose option 3 over 2 because of the frequency of its use for open‐angle glaucoma.
peripheral vascular disease.
1.‐ Hypertensive crisis
2.‐ Congestive heart failure (CHF)
3.‐ Open‐angle glaucoma
4.‐ Peripheral vascular disease

3186 A client is taking cyclosporine (Neoral) for a kidney Correct answer: 2 Some medications, such as phenytoin, rifampin, and phenobarbital, are known to reduce the The core issue of the question is knowledge of drug interactions. Specific knowledge is
transplant. The nurse is reviewing the list of current level of cyclosporine in the body. Therefore, the cyclosporine level should be monitored needed to answer this question. Use the process of elimination, and take time to review
medications. The nurse would be concerned after regularly while the client is taking these medications. Occasionally, dosage adjustment is this important drug if this question was difficult.
noting that the client is presently taking which of the required.
following medications?

1.‐ Prednisone (Deltasone)


2.‐ Phenytoin (Dilantin)
3.‐ Diltiazem (Cardizem)
4.‐ Acetylsalicylic acid (Aspirin)

3187 A client who has been taking bethanechol chloride Correct answer: 2 The client is exhibiting signs of cholinergic toxicity, and atropine is the antidote. Phytonadione Use the process of elimination, focusing on the critical words abdominal pain and difficulty
(Urecholine) for three days begins to complain of or vitamin K (option 1) is the antidote to warfarin (Coumadin). Oxybutinin is indicated for use breathing. After determining that the client is experiencing adverse or toxic effects of the
abdominal pain and difficulty breathing. After assigning as a urinary antispasmodic. Epinephrine is used to treat severe hypersensitivity reactions medication, choose the option that is an anticholinergic drug, which will treat the
another staff member to remain with the client, the (anaphylaxis). cholinergic symptoms.
nurse checks to see that which medication is available
on the nursing unit?

1.‐ Phytonadione (AquaMEPHYTON)


2.‐ Atropine sulfate (generic)
3.‐ Oxybutynin (Ditropan)
4.‐ Epinephrine (Adrenalin)

3188 A client who requires diuretic therapy has a Correct answer: 4 Loop diuretics have the disadvantage of requiring more frequent dosing, but are The core issue of the question is knowledge that loop diuretics are the most beneficial
creatinine clearance less than 30mL/min. The nurse advantageous in clients with creatinine clearance less than 30mL/min. The other types of type of diuretic for clients with low creatinine clearance levels. Specific knowledge of this
checks the physician order sheet, expecting to find an diuretics (osmotic in option 1, potassium‐sparing in option 2, and thiazide in option 3) are not drug category and the ability to recognize drugs from each diuretic class are needed to
order for which of the following types of medication? as useful when the client has a decreased creatinine clearance level. answer this question.

1.‐ Mannitol (Osmitrol)


2.‐ Spironolactone (Aldactone)
3.‐ Chlorothiazide (Diuril)
4.‐ Furosemide (Lasix)

3189 The nurse practitioner has prescribed oxybutynin Correct answer: 4 Oxybutynin (Ditropan) is an antispasmodic medication used to restore normal voiding Use the process of elimination. The core issue of this question is knowledge of side and
(Ditropan) for a 65‐year‐old female with urinary patterns in clients with spasms of smooth muscle of the urinary bladder. It produces adverse effects of this medication. Choose the option that indicates an anticholinergic
frequency and urgency. The nurse teaching the client anticholinergic side effects such as dry mouth, constipation, urinary hesitancy, and decreased effect, such as dry mouth.
about the side effects of this medication should gastroenteritis motility. Periodic interruptions in therapy are recommended to assess
explain that which of the following manifestations is continued need for this medication.
associated with this medication?

1.‐ Dizziness
2.‐ Increased bruising
3.‐ Diarrhea
4.‐ Dry mouth

3190 Phenazopyridine (Pyridium) is prescribed to a client Correct answer: 4 Phenazopyridine is a urinary analgesic with a local anesthetic effect on the urinary tract The core issue of the question is knowledge of the effects of phenazopyridine on the urine.
with dysuria. The nurse explains that the client should mucosa. This medication relieves pain during urinary tract infection. It causes the urine to have A critical word in the stem of the question is dysuria, which reminds you that the
expect which of the following urine characteristics an orange‐to‐red color. It has no effect on volume of urine. Foul odor to the urine can be medication is a urinary analgesic, and as the only drug of its type, it causes orange‐to‐red
while taking phenazopyridine? caused by urinary tract infection. discoloration in body fluids, including urine.
1.‐ Decrease in volume
2.‐ Odor that is foul
3.‐ Increase in volume
4.‐ Color that is orange or red

3191 A client in shock has a blood pressure (BP) that Correct answer: 2 Dopamine acts on the alpha‐ and beta‐ adrenergic receptors, resulting in vasoconstriction; The core issue of the question is knowledge of the use of dopamine for the client in shock.
continues to drop despite IV fluids. Intravenous increased systemic BP; and increased force and rate of myocardial contraction. Option 1 is a Recall that it can be used to achieve increased urine output in early shock, and to cause
dopamine is ordered at 8 mcg/kg/minute. The anxious false statement. Option 3 is the opposite of dopamine s effect. Option 4 is true in low doses vasoconstriction and increased BP in later shock, to make the correct selection.
client asks the nurse how the medication will help him (2–5 mcg/kg/minute), but the focus of dopamine for the client in shock who is not responding
get better. Which is the best response by the nurse? to fluids is to achieve vasoconstriction.

1.‐ “Dopamine will interfere with poor electrical conduction in your heart.”
2.‐ “It will cause blood vessel walls to tighten, to help increase your overall blood pressure.”
3.‐ “It will make your heart contract less forcefully and thus decrease the amount of work it has to do.”
4.‐ “It will increase your urine output through its effect on the kidneys.”

3192 A client arrives in the Emergency Department after Correct answer: 4 Aldactone is a potassium‐sparing diuretic that increases sodium excretion and decreases The core issue of the question is the need to monitor potassium levels in a client taking
complaining of unrelieved edema in her legs. The client potassium secretion in the distal convoluted tubule. Potassium levels higher than 5.5 mEq/L potassium‐sparing diuretics. Use knowledge of expected effects of diuretics to eliminate
has been taking spironolactone (Aldactone) at home. are contraindicated with spironolactone, due to increased risk of hyperkalemia. The elevated options 1 and 2, and choose option 4 over 3 because the medication is a potassium‐sparing
The nurse shares which of the following data with the blood glucose in option 1 is not a priority issue related to this medication. The elevated sodium diuretic, and the risk of hyperkalemia is great.
physician that indicates a need to withhold the level could be alleviated by the medication, and thus it is not a reason to withhold the dose.
medication? The blood pressure is normal, and does not warrant withholding a dose.

1.‐ Blood glucose level of 170


2.‐ Blood pressure of 110/70
3.‐ Sodium level of 146 mEq/L
4.‐ Potassium of 5.9 mEq/L

3193 The nurse is preparing to administer a first dose of Correct answer: 1 Epinephrine and oxygen should be available at the bedside because of the risk of anaphylaxis The core issue of the question is knowledge that anaphylaxis is possible with
cyclosporine (Sandimmune) by the IV route. Which of during administration. An oral airway and suction machine are not the priority items, although administration of cyclosporine. Note that the question contains the critical words priority
the following priority items should the nurse make maintaining an airway would be necessary if the client actually did go into anaphylaxis. Since items. Realize that more than one option might be partially or totally correct, and that you
available at the bedside during administration of this an oral airway and portable suction machine are usually part of the contents of a code cart, need to prioritize the first and therefore most important item you would need. Recall that
medication? the full cart would not also be necessary. epinephrine is used to treat anaphylaxis to make this selection.

1.‐ Epinephrine
2.‐ Oral airway
3.‐ A code cart
4.‐ A suction catheter

3194 The client beginning medication therapy with Correct answer: 2 Each dose of this medication should be administered with a full glass of water, and the client Specific drug knowledge is needed to answer the question; however, recalling that fluid
sulfisoxazole (Gantrisin) needs instructions for its use. should be encouraged to maintain a high fluid intake. The client should not discontinue or intake should be increased with urinary infections should help to eliminate the other
Which of the following items would the nurse include decrease the dosage without consulting with physician. Sulfisoxazole does not discolor urine incorrect options.
in client teaching about the medication? brown (although nitrofurantoin does), and it is not harmful (option 1). It is not necessary to
restrict salt intake (option 3), and the dose should not be decreased even if symptoms improve
(option 4).
1.‐ Call the prescriber if the urine turns dark brown.
2.‐ Maintain a high fluid intake.
3.‐ Restrict salt intake.
4.‐ Decrease the dosage when symptoms are improving.

3195 The nurse has an order to administer a first dose of Correct answer: 1, 2 Epoetin is given to stimulate red blood cell production in the client with chronic renal failure. Use the process of elimination and knowledge of drug therapy. Recall that dialysis is
epoetin (Epogen) to a client with chronic renal failure. For this reason, the nurse should look at the hemoglobin and hematocrit as baseline needed to treat renal failure, and thus options 4 and 5 can be eliminated first. Recall that
The nurse would make note of which of the following measurements. A white blood cell stimulant such as filgrastim (Neupogen) would be given to clients in renal failure are anemic because of impaired ability to produce erythropoietin to
laboratory test results to establish a baseline? Select raise white blood cell counts (option 3). Epoetin alfa will not treat creatinine or BUN levels eliminate the white blood cell count.
all that apply. (options 4 and 5); the client would be receiving dialysis to control these values.

1.‐ Hemoglobin of 9%
2.‐ Hematocrit of 26%
3.‐ White blood cell count 3,000/mm3
4.‐ Creatinine 3.2 mEq/L
5.‐ Blood urea nitrogen 56 mg/dL
3196 The following medication orders were discovered in a Correct answer: 4 The Tylenol order contains all of the pertinent information except the dosage. The prescriber Recall that the sequence of noting medication orders includes a complete order along with
client’s chart. The nurse should note and implement all needs to be contacted (option 4). The Digoxin order contains the drug, dosage, route, time appropriate components for the specific client.
of the orders except which of the following? interval (option 1). Because all of the components are present, the Ambien order can be
implemented (option 2). The Lasix order includes all of the specific information: drug, form,
dosage, route, and time period (option 3).
1.‐ Digoxin (Lanoxin) 0.25 mg PO daily
2.‐ Zolpidem (Ambien) 10 mg PO at bedtime PRN
3.‐ Furosemide (Lasix) oral solution 40 mg PO daily
4.‐ Acetaminophen (Tylenol) 2 tablets PO q4 hours PRN for headache

3197 A client receiving nadolol (Corgard) for hypertension Correct answer: 1 Feeling dizzy when moving from lying or sitting to standing position is referred to as Recall that the primary concept is safety. Select the option providing the greatest safety.
tells the nurse, "I get dizzy when I stand up." Which of orthostatic hypotension and is a common side effect of the beta blocker drugs such as nadolol.
the following is the nurse's most appropriate The client should be instructed to change positions slowly. Signs and symptoms should
response? diminish after a few weeks of treatment (option 1). Dizziness is not a sign of toxicity, but is
related to the body’s adjustment to reduced decreased cardiac output and reduced central
circulating volume (option 2). Dizziness can be a sign or symptom of hypertension. Since the
client is ingesting an antihypertensive drug, postural hypotension related to decreased central
circulating volume should be suspected (option 4).

1.‐ "This is an expected side effect of the drug, and you should use caution and move slowly when standing up."
2.‐ "You may be experiencing a toxic effect of the drug, and I will notify the physician."
3.‐ "Dizziness is not related to the drug, but I will need to ask you a few more questions."
4.‐ "Episodes of dizziness when moving are common symptoms of elevated blood pressure."

3198 A health care provider prescribes 1000 mL D5W Correct answer: 166 The rate for administration is calculated by: <BR /> Perform the appropriate conversions, use the X equation, and calculate the number of mL
0.09% NSS/ 6 hours. The flow rate for this infusion per hour
is___ mL per hour.

3199 Meperidine HCL (Demerol) 100 mg tablet PO is Correct answer: 1 Because IM or IV is a more efficient route the drug is likely be more effective more rapidly Associate the route with the magnitude of the client’s needs.
prescribed for a first‐day postoperative client with an (option 1). Food will delay absorption (option 2). The capsule may prevent the delay caused by
abdominal wound. The best nursing action includes food, but the oral route is not the most appropriate route for this client (option 3). Peak time
which of the following? period is one hour (option 4).
1.‐ Procure an order for a change to the parenteral route
2.‐ Administer the drug with food
3.‐ Place the tablet in a capsule
4.‐ Encourage coughing and deep breathing two hours after ingestion

3200 While administering Insulin (NPH, Isophane) Correct answer: 3 Since insulin can lower blood glucose to a life‐threatening level, it is imperative that client be Apply knowledge of the pharmacokinetics associated with NPH insulin to select the correct
subcutaneously, the client asks the nurse, "When aware of the pharmacokinetics of insulin. The client should expect the glucose level to be at answer.
should I expect my blood sugar to be at the lowest the lowest level in 4 to 12 hours – NPH insulin (option 3). A 2 to 3 hour peak is more
level?” The nurse's best response includes: appropriate for regular insulin (option 1). A 4 to 12 hour peak is more appropriate for insulin
aspart (Novolog) (option 2). An 8 to 12 hour peak is more appropriate for Lente insulin (option
4).
1.‐ 2 to 3 hours.
2.‐ Appropriately 60 minutes.
3.‐ 4 to 12 hours.
4.‐ 8 to 12 hours.
3201 The nurse is giving general information about Correct answer: 3 Because ACE inhibitors can cause fetal harm or death, they should be discontinued as soon as There is only one correct answer to this question. In order to select the correct answer,
antihypertensive medications to a young female client pregnancy is detected. The effect on breastfeeding infants is unknown. The effect of other review the complications of each medication.
with a history of hypertension. The nurse includes that medications is unknown during pregnancy.
pregnant or lactating women should avoid using which
of the following types of antihypertensives?

1.‐ Vasodilators
2.‐ Diuretics
3.‐ Angiotensin‐converting enzyme (ACE) inhibitors
4.‐ Calcium channel blockers

3202 The home health care nurse is visiting an elderly Correct answer: 2 Calcium channel blockers should be administered with a high‐fat meal; grapefruit should be There is only one correct answer to this question. In order to select the correct option,
client who is taking a prescribed calcium channel avoided before and after dosing, due to altered effects. The foods listed in the other options review the nursing interventions related to teaching aspects for a client when taking
blocker. In conducting dietary teaching, the nurse will not have a dose‐altering effect. calcium channel blockers.
instructs the client that what food is contraindicated to
take with a calcium channel blocker?

1.‐ Oranges
2.‐ Grapefruit
3.‐ Bananas
4.‐ Grapes

3203 The nurse provides discharge instructions to the Correct answer: 3 In hypertensive urgencies, clients present with a systolic BP higher than 240 mmHg and Having knowledge of the normal standards of care will lead to the correct option. If this
client taking an antihypertensive medication. The diastolic BP higher than 120 mmHg. In hypertensive emergencies, the client's diastolic BP is was difficult, review the parameters for notification to the practitioner when taking
nurse should include in the teaching plan to notify the higher than 130mmHg. antihypertensives.
health care provider that a hypertensive crisis exists if
the diastolic blood pressure (BP) is higher than which
of the following?

1.‐ 100 mmHg


2.‐ 120 mmHg
3.‐ 130 mmHg
4.‐ 140 mmHg

3204 The nurse provides instructions to the client Correct answer: 3 Spironolactone is a potassium‐sparing diuretic used to treat hypertension. Gynecomastia is There is only one correct answer to this question. Having knowledge of the adverse
prescribed a diuretic. The nurse informs the client that one of its adverse reactions. Adverse reactions usually disappear after the drug is reactions to the medication will lead to the correct answer. If this was difficult, review the
which diuretic causes a persistent gynecomastia? discontinued; however, gynecomastia can persist after discontinuation of spironolactone. side effects of the medications.

1.‐ Hydrochlorothiazide (HCTZ)


2.‐ Furosemide (Lasix)
3.‐ Spironolactone (Aldactone)
4.‐ Indapamide (Lozol)

3205 A client was just prescribed hydrochlorothiazide Correct answer: 2 Thiazide diuretics increase urinary excretion of sodium and water by inhibiting sodium Having knowledge of pathophysiological processes of diuretics will assist in selecting the
(HCTZ) for hypertension. The client asks the nurse how reabsorption in the cortical diluting tubule of the nephron, thus relieving edema. The loop correct answer. If this was difficult, review the glomerular filtration system, and its
this medication works in the body. The nurse explains diuretics inhibit electrolyte reabsorption in the thick, ascending loop of Henle, thereby functioning with diuretics.
that thiazide diuretics increase urinary excretion of promoting the excretion of sodium, water, and potassium. Potassium‐sparing diuretics directly
sodium and water by affecting which part of the increase sodium excretion and decrease potassium secretion in the distal convoluted tubule.
kidney?
1.‐ Loop of Henle
2.‐ Cortical diluting tubule
3.‐ Collecting ducts
4.‐ Glomerulus
3206 The nurse taking care of a client with benign prostatic Correct answer: 3 Alpha‐adrenergic blockers are used for peripheral vascular disorders, hypertension, and BPH. There is only one correct answer to this question. While all the listed options are
hyperplasia (BPH) explains to the client that what Options 1, 2, and 4 are incorrect. classifications of antihypertensive options, review the purposes of each classification in
category of antihypertensive drug can also be order to select the correct option.
prescribed for BPH?
1.‐ Beta blocker
2.‐ Calcium channel blocker
3.‐ Alpha‐adrenergic blocker
4.‐ Vasodilator

3207 The nurse would monitor the client receiving which Correct answer: 4 Verapamil and diltiazem, of all the calcium channel blockers, have the greatest effect on the Knowing that one of the actions of calcium channel blockers slows the heart rate will assist
type of antihypertensive drug because it slows the AV AV node to slow the heart rate. Additional drug effects are slowing of the ventricular rate in in selecting option 4. If this was difficult, review the effects of each listed classification.
node, and thus decreases the heart rate? atrial fibrillation or flutter, and conversion of supraventricular tachycardia (SVT) to a normal
sinus rhythm (NSR).
1.‐ Angiotensin‐converting enzyme (ACE) inhibitor
2.‐ Beta blocker
3.‐ Alpha blocker
4.‐ Calcium channel blocker

3208 An elderly client is given a prescription for celecoxib Correct answer: 1 Rationale: Older clients often take other prescribed drugs, herbs, or other alternative Elderly is the key term. Correlate knowledge associated with elderly clients and drug
(Celebrex) for pain and stiffness of osteoarthritis of the remedies, and client maybe ingesting an over‐the‐counter (OTC) remedy for arthritis. Because therapy and recall that assessment is the first step in managing client needs.
hips and back. The nurse should perform which of the there is a potentially high risk for drug‐drug or drug‐herb interaction, getting a thorough
following first? picture of the client's current drug regimen is the first step in planning for client education
when a new drug is ordered (option 1). Providing written materials is appropriate as a
supplement after the nurse has taught the client about the medication (option 2). Because of
cognitive changes, the client may need a strategy to help maintaining the drug schedule, but
should not be the first action. First action is based on the potential risk to the client (option 3).
Teaching about the drug is very relevant to the management of the client’s health needs, but
assessing the risk to the client should occur before implementation (option 4).

1.‐ Do a thorough medication assessment to see what other drugs the client is taking
2.‐ Provide the client with a printed pamphlet describing the new drug
3.‐ Inform the client where a medication organizer can be purchased
4.‐ Give the client a short, simple verbal explanation about the drug and its side effects

3209 The nurse would assess a client receiving a Correct answer: 1 Nonselective beta blockers are associated with adverse events of hyperglycemia and Having knowledge of the adverse reactions of a nonselective beta blocker will lead to the
nonselective beta blocker for development of which of hyperlipidemia. These changes might be temporary, but the client should be monitored for correct answer. If this was difficult, review the complications of the medication type.
the following complications related to drug therapy? occurrence. Decreased liver enzymes and increased BUN are not directly related to non‐
selective beta blockers.
1.‐ Hyperglycemia and hyperlipidemia
2.‐ Decreased liver enzymes
3.‐ Hypoglycemia
4.‐ Increased BUN

3210 The nurse monitors a client receiving a Correct answer: 1 Sympathomimetics act predominantly by direct stimulation of alpha‐adrenergic receptors, Having an understanding of the purpose of a sympathomimetic agent will lead to the only
sympathomimetic agent for which of the following which constrict blood vessels and increase their resistance. This in turn results in increased correct answer. If this was difficult, review the purpose of a sympathomimetic agent.
expected effects? total peripheral resistance, and increased systolic and diastolic BP.

1.‐ Increased blood pressure (BP)


2.‐ Decreased BP
3.‐ Increased heart rate
4.‐ Decreased heart rate

3211 The client has been taking norethindrone (Micronor) Correct answer: 2 Norethindrone (Micronor) contains only progestin, and no estrogen. Because estrogen can Use the process of elimination, and know that contraceptives containing estrogen are
oral contraceptive pills. Which of the following items is decrease lactation, progestin‐only pills are commonly used by lactating women. The other contraindicated in. If this was difficult, review medication classifications contraindicated
most likely to be found in her health history? options do not address the issue of contraception during lactation. with women who are breastfeeding.

1.‐ Superficial phlebitis


2.‐ Current breastfeeding
3.‐ Dysmenorrhea
4.‐ Menarche at age 18

3212 The client is taking Ortho‐Prefest Correct answer: 2 Breast tenderness, abdominal bloating, and monthly bleeding are common side effects of Using the process of elimination, select option 2, as this is the only option that would alert
(estradiol/norgestimate) for hormone replacement hormone replacement therapy. Severe leg pain in either the calf or the thigh could indicate to a high risk complication. The other options are expected side effects of the medication.
therapy. Which statement indicates that the client deep vein thrombophlebitis, and requires physician assessment.
needs further education?
1.‐ "My breasts may be tender at times."
2.‐ "I can expect to get severe pain in my calf."
3.‐ "I might have abdominal bloating."
4.‐ “Menstrual‐like bleeding might start.”

3213 Which of the following medication orders should the Correct answer: 1 Methylergonovine (Methergine) is only administered postpartum to control or prevent Using the process of elimination and knowing that methergine is given only postpartum
nurse question? excessive uterine bleeding. It is not used during pregnancy. The other options represent will lead to option 1. If this was difficult, review the purpose of methergine.
appropriate orders that the nurse does not need to question.
1.‐ Methylergonovine (Methergine) 0.2 mg PO for a pregnant client
2.‐ Oxytocin (Pitocin) 20 units in 1 liter D5W for postpartum hemorrhage
3.‐ Methylergonovine (Methergine) 2.5 mg PO for postpartum hemorrhage
4.‐ Oxytocin (Pitocin) 10 units IM after placental delivery

3214 The client is a vegan, and has been prescribed Correct answer: 4 Premarin (conjugated estrogens, equine) is derived from the urine of female horses, and Using the process of elimination to rule out all the answers other than option 4. This is the
Premarin. She asks if the medication has any animal therefore is animal‐based. Premarin might be rejected as a hormone replacement therapy only option that addresses the client s concerns.
products in it. The nurse's best answer would be: product by women who follow strict vegetarian guidelines. The statements in options 1 and 3
are factually incorrect, while option 2 does not address the client's concern.

1.‐ "Premarin is synthetic, and not derived from any animal products."
2.‐ "Your physician knows your concerns, and has written an appropriate prescription."
3.‐ "Premarin is a combination of synthetic and animal‐derived estrogens."
4.‐ "Premarin is made from a substance that is collected from live horses."

3215 The client with infertility is on a medication regimen Correct answer: 4 Ovarian hyperstimulation can result after fertility drugs are utilized. Women are instructed to Recognize that the question is referring to hyperstimulation. With this in mind, options 1
of menotropin (Pergonal) and human chorionic rest, and to avoid heavy lifting or activities that could cause their abdomens to be bumped, and 2 can be eliminated, since any activity can cause rupture of the stimulated ovary.
gonadotropin (Chorex). Her ultrasound examination because of the risk of rupture of the ovary. Intercourse is prohibited to prevent a multifetal Option 3 does not address hyperstimulation of the ovary, and can be eliminated.
detects ovarian hyperstimulation. What information pregnancy and ovarian rupture.
does the client need?
1.‐ "Have intercourse today and again tomorrow."
2.‐ "You may continue to play soccer every day."
3.‐ "Take your temperature each morning at the same time."
4.‐ "Limit your activity to resting, and avoid heavy lifting."
3216 The client has been using an illegally obtained Correct answer: 1 Anabolic steroids promote protein buildup. They are similar to testosterone, but with less Having an understanding of the purpose of anabolic steroids will lead to the only correct
anabolic steroid, nandrolone. He asks, "What's the big androgenic activity. The other statements are incorrect. answer. If this was difficult, review the purpose of anabolic steroids.
deal? My body makes steroids naturally." Which of the
following is the nurse’s best response?

1.‐ "The steroids your body makes are glucocorticoids. The anabolic steroids you've been using are synthetic, and are similar to testosterone."
2.‐ "The drugs you have been using will counteract the effect of the naturally occurring steroids your body makes, possibly making you ill."
3.‐ "The steroids from your adrenal glands make your body create more proteins. The drugs you've been using cause the proteins in your cells to break down."
4.‐ "You are correct. The anabolic steroids you have been using are virtually indistinguishable from those that your body creates."

3217 The client with primary amenorrhea has been started Correct answer: 4 Different forms of estrogen are administered IM, transdermal, or PO. Ethinyl estradiol is Note that the question is asking for selection of the answer that indicates effective
on ethinyl estradiol (Estinyl) 0.05 mg PO t.i.d. for 14 administered orally in many combination oral contraceptive pills, and IM in the product named teaching. Options 1, 2, and 3 are all incorrect statements. Option 4 encompasses all aspects
days, followed by 14 days of the same plus progestin. Lunelle. of administration of estrogen, while the other options are not complete. This should lead
The nurse concludes that the teaching plan has been you to the correct answer.
effective when the client states:

1.‐ "This medication is taken by mouth because estrogen can't be given any other way."
2.‐ "Estrogens are more effective when taken by mouth than when given by injection."
3.‐ "I have to take this in pill form because there is no skin patch–type estrogen preparation."
4.‐ "Different estrogens are administered in different ways. Mine is taken as a pill by mouth."

3218 The client with amenorrhea has been started on 10 Correct answer: 1 Medroxyprogesterone acetate (Provera) taken for 10 days facilitates thickened endometrial Having an understanding of the purpose of a medication will assist in selecting the correct
days of oral medroxyprogesterone acetate (Provera). growth, and when the medication is ended, a menstrual cycle–like bleeding episode will occur. answer. Note in the question stem that the client has amenorrhea. Use this knowledge to
Medication instruction should include which of the It does not immediately induce menstrual bleeding (option 2), rule out pregnancy (option 3), or be directed toward an answer that discusses producing a menstrual‐like bleeding. This is
following points? maintain a state of amenorrhea (option 4). present in option 1.

1.‐ The client should have menstrual‐like bleeding after the medication has been completed.
2.‐ Progestins prevent endometrial growth, and will quickly induce menstrual bleeding.
3.‐ Administration of this medication will rule out a possible pregnancy.
4.‐ Amenorrhea will continue if this medication is taken as directed.

3219 A female client has come to the clinic to be started on Correct answer: 4 Medroxyprogesterone acetate (Depo‐Provera) is given IM every 85–90 days. Amenorrhea In selecting the correct answer to this question, it is important to understand the side
medroxyprogesterone acetate (Depo‐Provera) during usually develops after the second or third injection (option 4), while breakthrough bleeding is effects and client responses to the medication. It is also important to know who is
her mense. Which of the following statements would common during the first 3–6 months (option 2). Contraindications include inability to receive contraindicated for taking this medication. With this knowledge select the only correct
the nurse include in medication teaching? the injections on time and desire to become pregnant within a year (option 3). answer, option 4.

1.‐ "The IM injections of medroxyprogesterone acetate (Depo‐Provera) must be received every 90–100 days."
2.‐ "Your menstrual periods will not change on medroxyprogesterone acetate (Depo‐Provera)."
3.‐ "Women with migraine headaches should not take medroxyprogesterone acetate (Depo‐Provera)."
4.‐ "After taking medroxyprogesterone acetate (Depo‐Provera) for several months, amenorrhea may develop."

3220 The female client with infertility will be starting Correct answer: 1 Menotropin (Pergonal) is given IM (not IV, as in option 2) for 9–12 days to mature ovarian Note that the question stem is referring to infertility. The question is also asking for the
menotropin (Pergonal) this cycle. Which statement will follicles. The rate of multifetal pregnancy is about 20 percent of pregnancies (option 3). Several use of the medication. The only question that refers to the teaching plan and use is option
the nurse include in a teaching plan for menotropin cycles of fertility medications are often needed to achieve pregnancy (option 4). 1. If this was difficult, review menotropin (Pergonal).
(Pergonal) use?
1.‐ Ovarian hyperstimulation could result.
2.‐ The medication is given IV.
3.‐ Multifetal pregnancies very rarely occur.
4.‐ The first cycle will result in pregnancy.
3221 The physician has written an order for sildenafil Correct answer: 1 Dosing of all medication is started small and increased if needed, to minimize the risks of side While the question is referring to dosage, and all the answer choices refer to dosage, the
(Viagra). The client asks about the dosage of this effects. Viagra should be taken approximately one hour prior to sexual activity. Option 3 is method to choosing the correct answer relies on knowledge of the medication. The only
medication. Which of the following items is included in incorrect because Viagra comes at three different doses: 25, 50, and 100 mg. Option 2 is correct option is 1.
the nurse's response? incorrect because the smallest dose is utilized, while option 4 is incorrect because the client
does not titrate the dose at will.
1.‐ The smallest possible dose to achieve erection is utilized.
2.‐ The dose is started high and worked down until erection is not achieved.
3.‐ Almost every man uses 50 mg, so that is what the doctor prescribes.
4.‐ Higher doses create longer erections, so it is up to the client which dose is ordered.

3222 The nurse is preparing the pregnant client to have her Correct answer: 3 Oxytocin (Pitocin) used for labor induction augments the endogenous oxytocin. It is The question is asking for identification of understanding of the medication by the client.
labor augmented with oxytocin (Pitocin). Further administered as a dilute solution of 10 or 20 units in 1 liter IV fluid via infusion pump, with a Having knowledge of the purpose of contractions should lead to the correct answer.
teaching is needed when the client states, "This goal of increasing the frequency and intensity of contractions. Options 1, 2, and 4 are correct Knowing that there should be a rest between contractions will enable selection of option 3,
medication will:” statements, indicating that the client understands these aspects of medication administration. which can have negative effects for the fetus. The other options are all correct statements
about oxytocin (Pitocin).
1.‐ “Work with the hormones that my body is producing that make contractions.”
2.‐ “Cause my uterus to contract harder and more often.”
3.‐ “Create one long continuous contraction until the baby is born.”
4.‐ “Be given through my IV a little bit at the time.”

3223 The client with preterm labor is being administered Correct answer: 1 Magnesium sulfate, when given parenterally, acts as a central nervous system (CNS) Knowing that magnesium sulfate is a CNS depressant will lead to the correct answer. The
magnesium sulfate (generic) intravenously. Which of depressant and a depressant of smooth, skeletal, and cardiac muscle function. The side effects only option that is a physiologic response to a CNS depressant is option 1.
the following client manifestations is an expected of this medication when taken IV are drowsiness, flushing, heaviness in the limbs, and
effect of this medication? decreased deep tendon reflexes. Option 2 is incorrect because a decreased respiratory rate is a
sign of magnesium toxicity. Option 3 is incorrect because these are signs of CNS excitability.
Option 4 is incorrect as well because this is the side effect of magnesium sulfate when taken
orally.
1.‐ Decreased deep tendon reflexes
2.‐ Decreased respiratory rate
3.‐ Nervousness and tremors
4.‐ Nausea and diarrhea

3224 The nurse is teaching a client about clomiphene Correct answer: 1 Clomiphene (Clomid) stimulates the production of lutein hormone (LH) and follicle‐ In order to select the correct answer, recall knowledge of the purpose of the medication
(Clomid). The nurse explains to the client that this stimulating hormone (FSH), and therefore increases ovulation in women with anovulatory clomiphene (Clomid). If this was difficult, review the purpose of the medication.
medication is indicated for treatment of which of the infertility. Clomiphene is not used to treat hypogonadism (option 2) or postpartum
following health problems? hemorrhage (option 3), or as hormone replacement therapy (option 4).
1.‐ Infertility
2.‐ Hypogonadism
3.‐ Postpartum hemorrhage
4.‐ Hormone replacement therapy

3225 The client taking Ortho Tri‐Cyclen combination oral Correct answer: 1 When two pills are missed, the client should "catch‐up" by taking two pills per day for two Knowing the “catch‐up” protocol will lead to the correct answer. Options 1 and 2 should
contraceptives calls the clinic reporting that she has days and then one pill until finishing the pill pack. This will keep her cycle controlled, and will be reviewed as correct answers, but option 1 is the most correct, since it states that the
forgotten her pills for the last two days. Which of the minimize the chance of mid‐cycle bleeding. However, the client could ovulate when missing client should use a backup method.
following suggestions would the nurse provide to the two or more pills. Thus a backup method such as condoms should be utilized for the rest of the
client? cycle.
1.‐ "Take two pills today and tomorrow, and use a backup method for the rest of the cycle."
2.‐ "Take two pills today, and then one each day until the pill pack is finished."
3.‐ "Take one pill each day until all the pills are gone, and use a backup method for the rest of the cycle."
4.‐ "Stop taking the pills, and use condoms until the next mense, then restart a new pill pack."
3226 For which of the following clients would the nurse Correct answer: 1 Ritodrine (Yutopar) is a beta‐adrenergic medication utilized for tocolysis in the treatment of In order to select the correct answer, recall knowledge of the purpose of the medication.
expect the physician to prescribe ritodrine (Yutopar)? preterm labor. It stimulates beta&lt;sub&gt;2&lt;/sub&gt; receptors in uterine smooth muscle, The sole purpose of the medication is in the treatment of preterm labor. Since preterm
reducing intensity and frequency of uterine contractions, and lengthening gestation period. labor is defined as before 38 weeks’ gestation, the only correct answer is option 1.
Options 2 and 4 are incorrect because the clients do not require medication with a tocolytic
effect. Option 3 is incorrect because ritodrine is not used for postpartum hemorrhage.

1.‐ A client at 27 weeks' gestation with regular uterine contractions


2.‐ A client at 41 weeks' gestation with irregular uterine contractions
3.‐ A client who delivered at 40 weeks' gestation who is having postpartum hemorrhage
4.‐ A client at 38 weeks' gestation with hypertension and seizures

3227 The hypertensive client is experiencing a postpartum Correct answer: 3 Oxytocin (Pitocin) is used to control postpartum hemorrhage and promotion of postpartum Note in the question stem that the client is hypertensive, as well as suffering from
hemorrhage. Which medication would the nurse uterine involution. It causes the least increase in blood pressure of all of these oxytocic postpartum hemorrhage. All the options are given in the treatment of postpartum
expect the certified nurse‐midwife to prescribe? medications, and therefore, considering the history of the client, would be used first in this hemorrhage, but it is imperative that the correct answer choice consider any side effect of
case. increased blood pressure. The medication that has the least increase on blood pressure is
option 3.
1.‐ Ergonovine (Ergotrate)
2.‐ Methylergonovine (Methergine)
3.‐ Oxytocin (Pitocin)
4.‐ Carboprost tromethamine (Hemabate)

3228 The client with infertility will be starting menotropin Correct answer: 1 Human chorionic gonadotropin (Chorex) serves to release the matured ovum from the In order to select the correct answer, recall knowledge of the purpose of the medication.
(Humegon) and human chorionic gonadotropin follicle, which has matured by the action of the menotropin or Humegon (option 2). Chorex The sole purpose of the medication is to release the matured ovum from the follicle. This is
(Chorex) injections this cycle. During instruction does not limit the number of ova released (option 3) or prepare the uterine lining for the the only correct answer. If this was difficult, review the purpose of the medication.
regarding medication administration, the client asks fertilized egg (option 4).
why she needs to have the Chorex, and not just the
menotropin (Humegon). The nurse's response would
include that Chorex:

1.‐ Facilitates release of the mature follicle from the ovary.


2.‐ Matures the follicle, so an ovum is ready to be released.
3.‐ Prevents more than one or two ova from being released.
4.‐ Prepares the lining of the uterus for the fertilized egg.

3229 A 16‐year‐old male client has been prescribed Correct answer: 4 Testosterone is responsible for development of male sex organs and the secondary sex In order to select the correct answer, recall knowledge of the purpose of the medication.
testosterone cypionate (Andonate) to treat characteristics, and facilitates growth of bone and muscle. In cases of hypogonadism, too little Options 1 and 3 can be eliminated, as they do not address the question the client is asking.
hypogonadism. He asks, "Why do I need these testosterone is naturally produced, and supplementation might be required. The responses in Option 2 can be eliminated, since it does not address any learning for the client. This would
hormones anyway?" Which of the following would be options 1 and 3 do not identify the purpose of this medication for a 16‐year‐old. The response leave correct answer 4.
the best response by the nurse? in option 2 is incomplete, and does not address the client's learning need.

1.‐ "Testosterone is necessary to prevent your muscles from atrophying."


2.‐ "This medication is a form of testosterone. Your doctor can best explain it to you."
3.‐ "Testosterone prevents your body from becoming feminine‐looking."
4.‐ "This medication will replace the testosterone that your body is not producing."

3230 The client has been prescribed danocrine (Danazol) Correct answer: 1 Danocrine (Danazol) is an androgen used in the treatment of endometriosis. It is taken orally In order to answer this question, you will need to know the nursing interventions required
for treatment of endometriosis. The nurse determines b.i.d. for several months. Option 2 is incorrect because the medication is given for 3–6 months; when teaching the client about the medication. The only correct option with this question is
that the instruction on taking this medication has been therapy may be extended to 9 months, if necessary. It is important to know that the regimen option 1. If this was difficult, review nursing interventions.
successful when the client states: cannot be repeated. Danazol is only given orally (options 3 and 4).

1.‐ "This medication is taken by mouth twice daily."


2.‐ "I'll need to take this medication for the rest of my life."
3.‐ "I'll be giving myself injections weekly with this medication."
4.‐ "This medication is worn as a patch on my abdomen."

3231 The client asks why her oral contraceptive, Desogen, Correct answer: 2 Progestins thicken cervical mucus to prevent sperm penetration, while estrogen Note in the question stem that the medication is described as an oral contraceptive.
contains both desogestrel and ethinyl estradiol. Which administration prevents the luteinizing hormone (LH) surge that stimulates ova maturation. Knowing the purpose of an oral contraceptive will lead you to the correct answer, 2. If this
of the following would be the nurse s best response? Option 1 is incorrect because additional estrogen is not needed. Option 3 is incorrect because was difficult, review the action of oral contraceptives.
taking estrogens alone would not prevent pregnancy. Option 4 is incorrect because the
contraceptive is made as a combination product, without the option of taking them separately.

1.‐ "Desogestrel is a weak estrogen‐and‐progestin combination that requires additional estrogen to work."
2.‐ "Together, the hormones prevent the ovum from maturing, and the sperm from penetrating through the cervical mucus."
3.‐ "Estrogens alone would prevent pregnancy, but progestins are added so that spotting doesn't occur."
4.‐ "Taking each hormone separately would prevent most pregnancies, but the combination is more effective."

3232 The nurse preparing to conduct medication teaching Correct answer: 3 Sildenafil is a medication used for erectile dysfunction among the male population. It is In order to select the correct answer, recall knowledge of the contraindications for the
would question an order for sildenafil (Viagra) when contraindicated if the client has had MI, cerebrovascular accident (CVA), or life‐threatening medication sildenafil (Viagra). If this was difficult, review the contraindications for the
the client has which of the following conditions? dysrhythmia in the past six months, or if the client has hypotension, hypertension, unstable medication.
angina, or CHF.
1.‐ History of type II diabetes and peripheral vascular disease
2.‐ Myocardial infarction (MI) four years ago
3.‐ Congestive heart failure (CHF)
4.‐ Benign prostatic hyperplasia

3233 The client is to begin clomiphene (Clomid) for Correct answer: 3 Clomiphene (Clomid) induces ovulation through stimulation of luteinizing hormone (LH) and Note that the question stem asks for selection of the answer that indicates that the client
treatment of infertility. The nurse concludes that the follicular‐stimulating hormone (FSH). It is taken orally in 50 mg dosage for five days each does not understand the purpose of the medication. In order to select the answer, recall
client needs additional teaching on this medication month, beginning on day 5 of the menstrual cycle. Options 1, 2, and 4 are incorrect because the purpose and method of administration of the medication clomiphene (Clomid). The
when she states: they are factually correct, which means the client understands medication teaching. only answer that indicates the client does not understand administration is option 3. If this
was difficult, review the purpose and dosing of the medication.

1.‐ "This medication will help make me ovulate."


2.‐ "I'll take this medicine orally for five days this month."
3.‐ "I'll be giving myself shots five days out of the month."
4.‐ "If this medication doesn't work, there are others to try."

3234 Which client is most likely to have testosterone Correct answer: 4 Androderm will replace the testosterone that should be produced by the testes. This therapy Note in the question stem that the medication being addressed is a testosterone. The only
transdermal (Androderm) prescribed for his condition? is utilized when the testes have been removed, to maintain libido, sexual functioning, and disease process listed that could require testosterone is option 4. The post‐surgical
secondary male characteristics. The clients in options 1, 2, and 3 do not require additional procedure involves removal of the testes, which produce testosterone. The other options
testosterone. do not require testosterone replacement.

1.‐ A 45‐year‐old with status post–bilateral vasectomy


2.‐ A 17‐year‐old with status post–cryptorchidism repair
3.‐ A 23‐year‐old with status post–varicocele repair
4.‐ A 32‐year‐old with status post–bilateral orchiectomy

3235 The client is being prepared to have labor induced Correct answer: 3 Oxytocin (Pitocin) is administered by diluting 10 or 20 units in 1 L of IV fluid, and The question is asking for selection of the answer that would indicate that teaching is
with oxytocin (Pitocin). Medication teaching has been administering small amounts (not large, as in option 4) via infusion pump. The synthetic successful. Knowing that the medication is only given IV will assist to eliminate options 1
effective when the client states: oxytocin supplements the endogenous oxytocin, and uterine contractions result. The and 2. Option 4 can also be eliminated, since it is not given as a large dose. Using the
medication is not given by mouth (option 1) or IM (option 2). process of elimination will lead you to the correct option, 3.

1.‐ "I'll take this medication by mouth to make my uterus contract."


2.‐ "This medication will be given to me as an IM injection."
3.‐ "Pitocin is a synthetic version of a hormone my body produces naturally."
4.‐ "A large dose of this medication will be given to me in my IV line."
3236 For which of the following clients would the nurse Correct answer: 4 Ergonovine (Ergotrate) causes uterine contractions, and is indicated for use only in In order to answer this question, recall the classification of the medication. Knowing the
anticipate the physician ordering ergonovine normotensive postpartum women. This medication causes an increase in blood pressure, so classification will lead to the only correct answer, 4. If this was difficult, review the purpose
(Ergotrate)? option 1 is incorrect. The clients described in options 2 and 3 are not presenting any signs and of the medication.
symptoms of bleeding, so using this medication would be futile.

1.‐ A client with untreated chronic hypertension


2.‐ A breastfeeding mother whose baby was delivered by forceps
3.‐ A bottle‐feeding mother who is a carrier for hepatitis C
4.‐ A normotensive mother with postpartum hemorrhage

3237 A female client taking Demulen, a combination oral Correct answer: 1 The symptoms of possible complications of combination oral contraceptives form the In order to answer this question, recall knowledge of the complications associated with
contraceptive pill, calls with sudden onset of blurred acronym ACHES: Abdominal pain; Chest pain; Headache; Eye problems; and Severe leg pain taking oral contraceptives. Since a sudden onset of blurred vision is a complication, it
vision. The nurse's best response would be to ask her (calf or thigh). The complications indicated by these symptoms are: Abdominal pain–liver requires immediate attention, which is stated in option 1. The other options do not address
to: tumor formation; Chest pain, Headache, and Eye problems‐embolus; Severe leg pain‐ this as a complication, and should be eliminated. If this was difficult, review complications
thrombophlebitis. Sudden onset of blurred vision could indicate blood clot formation and of oral contraceptive pills.
subsequent pressure on the optic nerves.
1.‐ Come in to see the physician now.
2.‐ Make an appointment to see the physician tomorrow.
3.‐ See an ophthalmologist as soon as possible.
4.‐ Have her call back after taking a nap.

3238 The client in preterm labor has just received 2.5 mg Correct answer: 4 Terbutaline sulfate (Brethine) is a smooth muscle relaxant, and therefore used to treat both In order to select the correct answer, recall knowledge of the side effects of terbutaline
terbutaline sulfate (Brethine) SC. Which statement bronchospasm and premature labor. A beta‐adrenergic, the medication causes side effects of sulfate (Brethine). The only correct answer is option 4. The other options are the opposites
would the client be most likely to make? increased heart rate with a sensation of the heart beating harder, palpitations, muscle of side effects of the medication. If this was difficult, review the side effects of the
tremors, and nervousness. The symptoms in options 1, 2, and 3 are opposites of those caused medication.
by terbutaline.
1.‐ "My heart seems to be beating really slowly."
2.‐ "My arms and legs feel so heavy and thick."
3.‐ "This medication makes me feel so sleepy."
4.‐ "My hands are so shaky, I can't write."

3239 For which of the following clients would the nurse Correct answer: 2 Testosterone preparations are contraindicated with pre‐existing liver disease. Acne (option Knowing system metabolism for the medication will lead to the correct answer. It is also
question an order for testosterone enanthate 1), melanoma (option 3), and testicular cancer (option 4) are not contraindications for use of important to know the contraindications for the medication. The only correct answer is
(Delatestryl)? this medication. option 2. If this was difficult, review the contraindications as well as system metabolism for
the medication.
1.‐ A 16‐year‐old with moderate acne
2.‐ A 32‐year‐old with liver failure
3.‐ A 66‐year‐old with melanoma
4.‐ A 25‐year‐old with testicular cancer

3240 The client is to receive dinoprostone (Prepidil) for Correct answer: 3 Prepidil is a form of prostaglandin E2, and is used for cervical ripening. The gel is inserted Note in the question stem that the medication is for cervical ripening. Option 3 is the
cervical ripening prior to induction of labor. For the around the cervix, through either a speculum or sterile vaginal exam. Positioning the client on correct answer, since the medication is inserted at the cervix via speculum.
administration of this medication, the client should be her back, with knees up and apart, will facilitate administration of the gel.
placed in which of the following positions?

1.‐ On her left side


2.‐ On her right side
3.‐ On her back, with knees bent and apart
4.‐ On her hands and knees
3241 A client who is receiving phenytoin (Dilantin) to Correct answer: 3 Clients with seizure disorders rarely are able to stop taking the anticonvulsants. The last The wording of the question tells you that the correct answer is also a true statement. Use
control seizures indicates an understanding of option is incorrect because of the word never. The first two options are incorrect statements. the process of elimination and medication knowledge to make a selection.
medication by making which of the following Extra doses are not taken related to stress, and there is no way to know at this time whether
statements? medication therapy could be terminated near the one‐year mark.

1.‐ “I need to take more of my Dilantin when I am having a stressful day.”


2.‐ “I will be able to stop taking this medicine in about a year.”
3.‐ “I will probably need to take this medicine all my life.”
4.‐ “I will never have another seizure if I take this medicine.”

3242 A client with a history of seizures is admitted with a Correct answer: 1 Phenytoin (Dilantin) is an anticonvulsant most effective in controlling tonic‐clonic seizures. The words most important in the stem of the question tell you that more than one or
partial occlusion of the left common carotid artery. Data collection before planning nursing care for a client with a seizure disorder should always perhaps all options might be correct and that you must choose the best option. Use
The client has taken phenytoin (Dilantin) for 10 years. include a history of seizure incidence. Option 4 might be a prodromal phase in some clients, medication knowledge as well as knowledge of how to manage a client during a seizure to
When planning care for this client, it is most important but a history of incidence is more important data. Removal of dentures might be indicated eliminate any of the incorrect options.
that the nurse does which of the following? during a seizure, but not at this time. Placing an airway or restraining a patient during a seizure
could cause harm.
1.‐ Obtains a history of seizure incidence.
2.‐ Places an airway, suction, and restraints at the bedside.
3.‐ Asks the client to remove any dentures.
4.‐ Observes the client for increased restlessness and agitation.

3243 A client with a history of seizures is scheduled for an Correct answer: 3 The therapeutic blood levels of the anticonvulsant need to be maintained. The nurse should The core issue of the question is how to maintain the client in a seizure‐free state while
arteriogram at 10:00 a.m., and is to have nothing by question the physician about alternate routes of administration. Omission of a dose is not NPO. Analyze each of the options to determine the method that will best protect the client
mouth before the test. The client is scheduled to prudent, nor is changing the route without a physician order. from seizure activity.
receive phenytoin (Dilantin) at 9:00 a.m. The nurse
should take which of the following actions?

1.‐ Omit the 9:00 a.m. dose.


2.‐ Give the same dosage of the drug rectally.
3.‐ Ask the physician if the drug can be given IV.
4.‐ Administer the drug with 30 mL of water at 9:00 a.m.

3244 The nurse is assessing a client with Parkinson’s Correct answer: 1 Destruction of the neurons of the basal ganglia in Parkinson s disease results in decreased The core issue of the question is the symptom that should be abolished by medication
disease to determine effectiveness of medication muscle tone. This gives the face a masklike appearance, and causes a monotone speech therapy. Use medication knowledge and the process of elimination to make a selection.
therapy. The nurse would determine that the pattern that can be interpreted as flat. If medication therapy was ineffective, the client would
medication is not working optimally if the client is still exhibit symptoms of the disorder, such as flattened affect. The other options do not apply
demonstrating which of the following characteristics? to this disease.

1.‐ A flattened affect


2.‐ Tonic‐clonic seizures
3.‐ Decreased intelligence
4.‐ Changes in pain tolerance

3245 The client with Parkinson’s disease asks the nurse, Correct answer: 3 Levodopa is the precursor of dopamine. It is converted to dopamine in the brain cells until The wording of the question tells you that the correct option is also a true statement. Use
“How will levodopa treat this disease?” The nurse needed as a neurotransmitter. Improved neural myelination, acetylcholine production, and medication knowledge and the process of elimination to make a selection.
would incorporate into a response that levodopa does regeneration of injured cells cannot be attributed to levodopa.
which of the following?
1.‐ Improves myelination of neurons.
2.‐ Increases acetylcholine production.
3.‐ Replaces dopamine in the brain cells.
4.‐ Causes regeneration of injured thalamic cells.
3246 A female client taking medications for seizures has Correct answer: 3 Barbiturates decrease the body's response to warfarin (Coumadin). As a result, there is less The words most important in the stem of the question tell you that more than one or
been placed on warfarin (Coumadin) for suppression of prothrombin; when inhibition caused by barbiturates disappears, hemorrhage perhaps all options might be partially or totally correct, and that you must choose the most
thrombophlebitis. After the weekly prothrombin time, could result. Withdrawal symptoms are not a priority concern if the client just takes the important option. The core issue of the question is knowledge of the interactive effects of
the client contacts the office to see if there will be a barbiturate for sleep (option 1). Absence of sleep is not likely to result in seizure activity barbiturates and warfarin.
dosage change. The client also mentions that she is out (option 2). Control of seizure activity is not dependent on combined use of phenytoin and the
of her barbiturate sleeping pill. She states that she will barbiturate sleep aid (option 4).
wait until her next appointment to get a refill. The
nurse instructs her to come for the refill immediately
for which of the following most important reason?

1.‐ She otherwise could develop withdrawal symptoms.


2.‐ Absence of sleep could precipitate seizures.
3.‐ Discontinuance of the drug can affect the prothrombin level.
4.‐ Control of seizures is dependent on the combined action of phenytoin (Dilantin) and the sleeping medication.

3247 A client is brought to the Emergency Department in Correct answer: 2 Diazepam is a benzodiazepene tranquilizer and an anticonvulsant used to relax smooth The wording of the question tells you that the correct option is also a true effect of the
the midst of a persistent tonic‐clonic seizure. muscles during seizures. Diazepam does not slow cardiac contractions (option 1), dilate medication. Use medication knowledge and the process of elimination to make a selection.
Diazepam (Valium) is administered intravenously. The tracheobronchial structures (option 3), or provide amnesia of seizure activity (option 4).
nurse anticipates that the effects of diazepam will be
to decrease central neuronal activity and:

1.‐ Sslow cardiac contractions.


2.‐ Relax peripheral muscles.
3.‐ Dilate the tracheobronchial structures.
4.‐ Provide amnesia of the seizure episode.

3248 The nurse would assess for which of the following as Correct answer: 4 Morphine is a CNS depressant. Its major adverse effect is respiratory depression. It can also The wording of the question tells you that the correct option is also a true effect of the
symptoms of morphine overdose in a client receiving lead to lethargy, pupillary constriction, and depressed reflexes. Morphine does not slow the medication. Use medication knowledge and the process of elimination to make a selection.
patient‐controlled analgesia? pulse rate (option 1), although it could lower blood pressure. It does not cause restlessness
(option 2) or profuse sweating (option 3).
1.‐ Slow pulse; slow respirations; sedation
2.‐ Slow respirations; dilated pupils; restlessness
3.‐ Profuse sweating; pinpoint pupils; deep sleep
4.‐ Slow respirations; constricted pupils; sedation

3249 Levodopa is prescribed for a client with Parkinson’s Correct answer: 4 Levodopa is the precursor of dopamine. It reduces sympathetic outflow by limiting The wording of the question tells you that the correct option is also a true statement that
disease. Which of the following would the nurse vasoconstriction, which can result in orthostatic hypotension. The medication should be would be included in client teaching. Use medication knowledge and the process of
include in the teaching plan for the client about administered with food to minimize gastric irritation (option 1). It is not monitored by weekly elimination to make a selection.
levodopa? laboratory tests (option 2), nor does it cause initial euphoria followed by depression (option 3).

1.‐ It is poorly absorbed if given with meals.


2.‐ It must be monitored by weekly laboratory tests.
3.‐ It causes an initial euphoria, followed by depression.
4.‐ It can cause a side effect of orthostatic hypotension.

3250 When caring for the client who is receiving phenytoin Correct answer: 1 Gingival hyperplasia is an adverse effect of long‐term phenytoin (Dilantin) therapy. The wording of the question tells you that the correct option is also a true statement that
(Dilantin), the nurse emphasizes meticulous oral Maintaining therapeutic blood levels and meticulous oral hygiene, including regular check‐ups would be included in client teaching. Use medication knowledge and the process of
hygiene to the client, stating that phenytoin has which with a dentist, can decrease the incidence of hyperplasia. It does not alkalinize oral secretions elimination to make a selection.
of the following effects on oral tissue? (option 2), destroy tooth enamel (option 3), or increase plaque and bacterial growth at gum
lines (option 4).
1.‐ It causes hyperplasia of the gums.
2.‐ It increases alkalinity of the oral secretions.
3.‐ It irritates gingival tissue, and destroys tooth enamel.
4.‐ It increases plaque and bacterial growth at the gum lines.

3251 The physician prescribes phenobarbital sodium Correct answer: 1 Phenobarbital depresses the CNS, particularly the motor cortex, producing side effects such Use medication knowledge and the process of elimination to make a selection.
(Luminal) for a client who has had a tonic‐clonic as lethargy, loss of appetite, depression, and vertigo. The other side effects listed for
seizure. The nurse concludes that the client phenobarbital do not include anal itching or dizziness upon standing (option 2), diarrhea or
understands the side effects of phenobarbital when upper body rash (option 3), or decreased tolerance to common foods or constipation (option
the client states, “I should call the doctor if I develop:" 4).

1.‐ "Loss of appetite, or persistent fatigue.”


2.‐ "Dizziness when I stand up, or anal itching.”
3.‐ "Diarrhea, or a rash on the upper part of my body.”
4.‐ "Decreased tolerance to common foods, or constipation.”

3252 The nurse administering methylphenidate (Ritalin) is Correct answer: 2 Fever is not a side effect of methylphenidate. Insomnia (option 1), rash (option 3), and Note that the stem of the question contains the word not, which indicates that the correct
monitoring the client for symptoms associated with palpitations (option 4) are possible side effects of methylphenidate. option is not an actual manifestation of the drug. Use knowledge of side/adverse
this medication. Which of the following is not a medication effects to make a selection.
manifestation for which the nurse would assess?

1.‐ Insomnia
2.‐ Fever
3.‐ Rash
4.‐ Palpitations

3253 The physician prescribes phenytoin (Dilantin) for a Correct answer: 3 The primary action is to reduce voltage, frequency, and spread of electrical discharges within Use medication knowledge and the process of elimination to make a selection.
client to control tonic‐clonic seizures. The nurse the motor cortex, resulting in inhibition of seizure activity. The drug does not act directly on
explains in simple terms to the client that the expected muscles (option 1), prevent CNS depression (option 2), or change permeability of cell
effect of the drug is to do which of the following? membranes (option 4).

1.‐ Produce an antispasmodic action on the muscles.


2.‐ Prevent depression of the central nervous system.
3.‐ Control nerve impulses originating in the motor cortex.
4.‐ Alter the permeability of the cell membrane to potassium.

3254 The nurse is providing information to a client taking Correct answer: 3 OTC medications with alcohol (another CNS depressant) should be avoided unless specifically Note that the question contains the words more instruction, which indicates that the
benztropine (Cogentin), an anticholinergic given for directed by the provider. The other statements would indicate understanding of the correct answer option contains incorrect information. Use medication knowledge and the
Parkinson’s disease. Which of the following statements medication teaching. process of elimination to make a selection.
made by the client would indicate that more
instruction is needed regarding this medication?

1.‐ “I may crush the tablets to make them easier to take.”


2.‐ “I should avoid driving until I know how this medication will affect me.”
3.‐ “I can take OTC cough or cold pills that have alcohol in them.”
4.‐ “I should never discontinue the medication abruptly.”

3255 The nurse is providing discharge instructions to a Correct answer: 2 The Alzheimer s client and family will need much support. Medication therapy will delay The wording of the question tells you that the correct answer is an option that is a priority
client with Alzheimer’s disease and his family after progression of symptoms, but will not effect a cure. The primary concern is for the safety of teaching measure. To make your selection, recall the nature of the disorder, and that
completing medication teaching. Because medication the client, so constant supervision is necessary. The other options are incorrect approaches. medication therapy will not reverse symptoms.
therapy will not reverse symptoms that have
developed, which of the following should be included
in this discussion?
1.‐ Keep the client in her own home, regardless of circumstances.
2.‐ Provide supervision to protect the client from becoming injured, humiliated, or lost.
3.‐ Attend a 12‐step program with a homecare agency.
4.‐ Resist using adaptive‐assistive equipment.

3256 Which of the following would be the most important Correct answer: 4 The disturbance in thought processes is the primary nursing diagnosis. Effective medication Note the key words most important in the stem of the question, which tells you that you
nursing diagnosis for the client newly diagnosed with therapy will reduce the progression of symptoms of the dementia. A disturbance in sleep need to prioritize your answer. Use knowledge of the key effects of medication therapy and
Alzheimer’s disease who is just beginning medication pattern (option 3) could become important later, but initially, the thought process is most the process of elimination to make a selection.
therapy? significant. Altered fluid and electrolytes (option 1) is not a nursing diagnosis, while pain
(option 2) is not applicable, given the information in the question.

1.‐ Altered Fluid and Electrolytes


2.‐ Pain
3.‐ Disturbed Sleep Pattern
4.‐ Disturbed Thought Processes

3257 The nurse would include which of the following items Correct answer: 1 Tacrine (Cognex) increases the available acetylcholine in the brain; therefore, the The wording of the question tells you that the correct option is also a true statement that
in an assessment of the client with Alzheimer’s disease parasympathetic system is stimulated. Blood pressure, mental status, and GI status would be would be included in client teaching. Use medication knowledge and the process of
who is receiving tacrine (Cognex)? affected. Hemoglobin, red and white blood cell count, liver function, electrocyte balance, and elimination to make a selection.
edema in legs do not relate to this medication.
1.‐ Blood pressure (BP), mental status, and gastrointestinal (GI) status
2.‐ Hemoglobin (Hgb), white blood cells (WBCs), and liver function tests
3.‐ Hgb, red blood cells (RBCs), and mental status
4.‐ BP, electrolytes, and edema in legs

3258 The client has migraine headaches. The provider has Correct answer: 4 Tagamet can increase the levels of Elavil in the blood, causing seizures, tachycardia, Options that are similar are not likely to be correct. All of the incorrect options relate to
prescribed amitriptyline hydrochloride (Elavil) as hypertension, or toxicity. Acetaminophen (option 1), aspirin (option 2), and NSAIDs (option 3) analgesia, and so must be eliminated.
prophylaxis for the headaches. What over‐the‐counter do not have that effect.
(OTC) medication might intensify the actions of Elavil,
meriting a warning from the nurse?

1.‐ Acetaminophen (Tylenol)


2.‐ Aspirin (ASA)
3.‐ Nonsteroidal anti‐inflammatory drugs (NSAIDs)
4.‐ Cimetidine (Tagamet HB)

3259 When administering anticholinergic medications for Correct answer: 3 Dry mouth, constipation, and urinary retention or hesitancy are all possible side effects of The key words in the stem of the question are least concerned. This tells you that the
Parkinson’s disease, the nurse would be least anticholinergic medications. Fever is not a side effect of anticholinergic medications. correct answer is an option that is not characteristic of this medication. Use medication
concerned with assessing the client for which of the knowledge and the process of elimination to make a selection.
following?
1.‐ Dry mouth
2.‐ Constipation
3.‐ Fever
4.‐ Urinary retention or hesitancy

3260 The client receiving phenytoin (Dilantin) asks the Correct answer: 4 Phenytoin inhibits folic acid absorption, and potentiates effects of folic acid antagonists. Folic Use knowledge of the key side effects of medication therapy and the process of
nurse why the doctor has prescribed folic acid with this acid is helpful in correcting some anemias that can result from phenytoin administration. The elimination to make a selection.
medication. The nurse s response would be based on other options are incorrect statements.
which of the following?
1.‐ It improves absorption of iron from foods.
2.‐ Its content in common foods is inadequate.
3.‐ It prevents the neuropathy caused by phenytoin.
4.‐ Its absorption from foods is inhibited by phenytoin.
3261 The client has been diagnosed with narcolepsy. The Correct answer: 3 Methylphenidate is a central nervous system stimulant. It increases the release of In order to select the correct answer to this question, recall knowledge of the
provider is considering prescribing methylphenidate norepinephrine and dopamine in cerebral cortex to the reticular activating system. Ritalin is contraindications for the medication. If this was difficult, review Ritalin and the
(Ritalin). The nurse notes that the client has a history contraindicated in clients with glaucoma. Congestive heart failure, diabetes mellitus, and contraindications for the medication.
of which of the following prior medical conditions that hyperthyroidism do not represent contraindications to the use of methylphenidate.
would disqualify the client from using this medication?

1.‐ Congestive heart failure (CHF)


2.‐ Diabetes mellitus
3.‐ Glaucoma
4.‐ Hyperthyroidism

3262 The client has just been diagnosed with a seizure Correct answer: 2 Medication must be taken to maintain therapeutic blood levels, even if there is no seizure Note that the question asks for selection of the highest‐priority teaching option. The
disorder. The medication regimen has controlled the activity. The urine might turn pink or brown, but that is not the most important item to teach. highest priority in teaching about medication is to adhere to the medication protocol,
seizures for several days. Prior to discharge, the nurse Options 1 and 4 are incorrect. After six months with no seizures, a client can often drive again. which will lead you to option 2.
should place highest priority on sharing which of the
following information with the client?

1.‐ Seizure disorders will often eventually stop on their own.


2.‐ Adherence to medication therapy is essential to avoid recurrence of seizures.
3.‐ Urine will turn pink or brown from the medication.
4.‐ The client can never drive a vehicle again.

3263 The client with a history of cluster headaches should Correct answer: 4 The client needs to give the medication the opportunity to work without aggravating the In looking at the stem, note that the client has cluster headaches. Knowing that a quiet,
be taught which of the following information regarding headache. Ergotamine should be given orally 1–2 mg, followed by 1–2 mg every 30 minutes darkened room will help clients with pain relief from cluster headaches will lead to the
use of ergotamine tartrate (Gynergen)? until the headache abates or until the maximum dose of 6 mg/24 hours (option 1). It is correct answer, 4. Understanding the disease process and the usual medical regimen will
unnecessary to drink large amounts of fluids (option 2), and increased warmth and energy are assist in selecting the correct answer.
not associated with this medication (option 3).
1.‐ "Take the medication every 4 hours."
2.‐ "Take the medication with plenty of water."
3.‐ "You will feel energetic and warm after taking the medication."
4.‐ "Lie down in a darkened room after taking the medication."

3264 The client is experiencing spasticity related to a spinal Correct answer: 2 Dantrolene is a central‐acting skeletal muscle relaxant. This medication may be used to In order to select the correct option for this question, first know the classification of each
cord injury. The nurse anticipates that which of the control spasticity after spinal cord injury. Dexamethasone is a corticosteroid used to decrease of the medications. Option 2 is the only medication that will work as a muscle relaxant. If
following medications is most likely going to be added swelling, especially cerebral edema. Dichlorphenamide is a carbonic anhydrase inhibitor used this was difficult, review the classification of the medications.
to the client's medication list? to treat glaucoma by decreasing production of aqueous humor, thereby lowering intraocular
pressure. Dobutamine is a medication used to treat hypotension by increasing cardiac output.

1.‐ Dexamethasone (Decadron)


2.‐ Dantrolene (Dantrium)
3.‐ Dichlorphenamide (Daranide)
4.‐ Dobutamine (Dobutrex)

3265 Which of the following are priority assessments by Correct answer: 1 Because opioid analgesics relieve pain, the nurse needs to assess the client's pain intensity Note that the question asks for selection of a priority assessment. In considering the
the nurse when administering opioid analgesics to a before and 30 minutes after administering a dose. The respiratory rate and level of nursing process, consider the fifth vital sign (pain) as part of the priority assessment. Also
client? consciousness need to be assessed because respiratory depression and sedation are two see in the stem that the question is referring to an analgesic, which would again refer to
adverse effects of this drug class. The items in the each of the other options are only partially pain. Knowing that opioids reduce respiratory rate will lead to option 1 as your choice.
correct. Urine output, liver function studies, seizure activity, electrolytes, and blood glucose While pain is listed in two other options, they do not identify respiration as a priority.
are not ongoing assessments directly related to opioid administration.

1.‐ Pain intensity, respiratory rate, and level of consciousness


2.‐ Liver function studies, urine output, and pain intensity
3.‐ Seizure activity, mental status, and respiratory status
4.‐ Electrolytes, blood glucose, and pain intensity

3266 The nurse is teaching a client about anti‐inflammatory Correct answer: 2 Clients might take more aspirin, acetaminophen, and NSAIDs than prescribed by their Part of any teaching plan is to caution people to consult with their practitioner prior to
medications. Client education regarding taking aspirin, providers if they are not aware that many OTC medications are combined with these taking over‐the‐counter medications when they take prescribed medications. This is stated
acetaminophen, or non‐steroidal anti‐inflammatory medications. The other answers are cautions for a variety of other types of medications. in option 2, and would be the priority choice. The other options are incorrect, as these
drugs (NSAIDs) should include which of the following instructions are usually given with a variety of prescribed medications.
cautions?
1.‐ Radial pulse and temperature should be taken prior to medication administration.
2.‐ Consult a health care provider before taking over‐the‐counter (OTC) medications, since many are combinations that can include more of the prescribed medication than is
safe.
3.‐ Cholesterol levels must be measured prior to treatment with medication.
4.‐ Do not discontinue use of medication abruptly; discontinuation must be tapered over a week.

3267 The nurse is caring for a hospitalized client with Correct answer: 3 The client should be assessed for seizure activity, changes in mental status, and respiratory Note that the question asks for selection of a priority assessment. Knowing that the
diagnosis of seizure disorder. The client has an order status as highest priority. Assessing kidney function (BUN and creatinine) and urine output is medication is prescribed for seizure activity will lead to option 3, which is the priority
for phenytoin (Dilantin). The nurse makes it a priority not the priority nursing consideration when the client is taking anticonvulsants. The assessment. The other options do not list seizure activity in the answers, so they can be
to assess which of the following parameters? assessments in the other options are pertinent for a variety of other types of medications. eliminated as your priority assessment.

1.‐ Respiratory rate, and level of consciousness


2.‐ BUN, creatinine, and urine output
3.‐ Seizure activity, mental status, and respiratory status
4.‐ Electrolytes, serum osmolality, and leg edema

3268 The client is prescribed carbamazepine (Tegretol) for Correct answer: 4 Tegretol is contraindicated within 14 days of taking MAOIs, to help prevent a fatal reaction. In order to select the correct answer to this question, recall knowledge of the
a seizure disorder. The nurse cautions the client to The other drug classes listed do not have this interactive effect with carbamazepine. NSAIDs contraindications for the medication. There is only one correct option for this question. If
avoid taking which of the following types of are used to treat inflammation and pain, while opioid analgesics and skeletal muscle relaxants this was difficult, review Tegretol and the contraindications for the medication.
medications that could cause a fatal reaction with this are drug classes that exert an effect on the central nervous system.
medication?
1.‐ Nonsteroidal anti‐inflammatory drugs (NSAIDs)
2.‐ Opioid analgesics
3.‐ Skeletal muscle relaxants
4.‐ Monoamine oxidase inhibitors (MAOIs)

3269 A client with a seizure disorder has been started on Correct answer: 2 The seizure threshold is decreased when anorexiants or amphetamines are used concurrently In order to select the correct answer to this question, recall the contraindications for the
medication therapy. The nurse should emphasize that with anticonvulsants, due to changes in the brain chemicals caused by the anorexiants and medication. There is only one correct option for this question. If this was difficult, review
which of the following types of medications should not amphetamines. The medications listed in options 1 and 3 do not change the seizure threshold. anticonvulsants, as well as which combinations of medications should not be taken with
be taken concurrently with anticonvulsants, as they The medications listed in option 4 may be used to treat seizures, which could raise the seizure anticonvulsants.
might lower the seizure threshold? threshold.

1.‐ Aspirin, acetaminophen, and nonsteroidal anti‐inflammatory drugs (NSAIDs)


2.‐ Anorexiants and amphetamines
3.‐ Anticholinergics and dopamine agonists
4.‐ Hydantoins and benzodiazepines

3270 The client with a spinal cord injury is taking Correct answer: 3 Dantrolene is a skeletal muscle relaxant. Hepatotoxicity is an adverse reaction for dantrolene, Having knowledge of the side effects of dantrolene (Dantrium) will assist in selecting the
dantrolene (Dantrium) for spasticity. The nurse should which can be manifested by abdominal pain, jaundiced sclera, or clay‐colored stools. The items correct answer. Knowing that hepatotoxicity is a side effect of the medication should lead
instruct the client to notify the physician immediately in the other options do not address this adverse effect. to an answer that considers the gastrointestinal (GI) system. The only answer that considers
if which of the following adverse effects of the the GI system is option 3.
medication occurs?
1.‐ Twitching, diarrhea, or rash
2.‐ Change in blood or urine glucose levels
3.‐ Abdominal pain, jaundiced sclera, or clay‐colored stools
4.‐ Urine changing to pink or brown, or gingival hyperplasia

3271 The client is newly diagnosed with Parkinson's Correct answer: 4 The client needs to understand that high‐protein foods must be avoided so that the The question is asking for selection of the priority intervention in a teaching plan. A
disease. The medication that has been prescribed is medication can be absorbed properly. Side effects do not include cushingoid symptoms (option priority intervention with teaching about medication administration is to teach what might
levodopa (Dopar). The nurse should place a high 1) or oral ulcerations (option 3). There is no need to avoid vaccinations (option 2). interfere with the absorption of the medication. Effectiveness of the medication is
priority on teaching which of the following information important. Option 4 lists the priority need of absorption and effectiveness.
prior to discharge?
1.‐ Side effects include cushiongoid symptoms such as moon face and weight gain.
2.‐ There is a need to avoid vaccinations.
3.‐ Report any ulcerations or sores in the mouth to a health care provider immediately.
4.‐ Avoid high‐protein foods, since they interfere with absorption of the medication.

3272 Which of the following statements made by the client Correct answer: 3 Medication must be taken to maintain therapeutic blood levels, even with no seizure activity. Understanding that anticonvulsants must be taken regularly will lead to the only correct
indicates to the nurse an understanding of client If the client understands that adherence is important, he is more likely to be compliant with answer, option 3. If this was difficult, review the teaching points for anticonvulsants.
teaching regarding anticonvulsant therapy? the medication regimen. The client does not need to lie down after a dose (option 1), or have
cholesterol levels checked (option 2). Anticonvulsant therapy is prescribed for long‐term or
lifelong use (option 4).
1.‐ "After taking the medication, I should lie down."
2.‐ "I must be sure to have my cholesterol levels checked regularly."
3.‐ "It is essential for me to continue to take this medication to avoid recurrence of seizures."
4.‐ "Seizures will often stop without intervention, so I may not need this medication for long."

3273 Which of the following statements by the client with Correct answer: 1 If the client understands the importance of the finding the triggering factors, she will be more Understanding the need to find the triggering mechanisms for migraines headaches will
migraine headaches would indicate to the nurse that willing to be involved in decreasing the triggers, including lifestyle changes that might be assist in selecting the only correct option, 1. If this was difficult, review the teaching points
she has understood client teaching regarding migraine necessary. The client should continue to exercise for general health and stress management for a client who is taking migraine medication.
medication? (option 2). Medication might not be needed every four hours (option 3), and driving is
permitted (option 4).
1.‐ "I will be keeping a diary of my headaches, so that I can see if there is a pattern."
2.‐ "I will be able to stop my exercise program, since it has not helped my headaches."
3.‐ "I will take my headache medication every four hours."
4.‐ "I will never be able to drive again, due to my headaches."

3274 The client has been diagnosed with attention deficit Correct answer: 4 A history of Tourette's syndrome is a contraindication for Ritalin. There are other medications In order to select the correct answer to this question, recall the contraindications for the
hyperactivity disorder (ADHD), and the provider is that could be used for treatment of ADHD, such as pemoline (Cylert) or dextroamphetamine medication. If this was difficult, review Ritalin and the contraindications for the medication.
considering prescribing methylphenidate (Ritalin). (Dexedrine). The medications listed in the other options are not contraindications for
Which of the following past medical histories would be methylphenidate.
a contraindication for Ritalin?

1.‐ A history of chickenpox


2.‐ A history of diabetes
3.‐ A history of seizures
4.‐ A history of Tourette's syndrome

3275 A female client is being medicated with a central Correct answer: 3 The dose of an antihypertensive medication usually needs to be adjusted when a CNS To select the correct answer to this question, recall knowledge of which medication will
nervous system (CNS) stimulant. The nurse should stimulant is added to a client's medication regimen. The other options are incorrect because have adverse drug interactions when taken with a CNS stimulant. If this was difficult, review
place highest priority on educating the client regarding NSAIDs, skeletal muscle relaxants, and opioids are less affected by CNS stimulants than are general considerations for clients taking a CNS stimulant.
adverse drug interactions with which type of antihypertensives.
medication she is presently taking?

1.‐ Nonsteroidal anti‐inflammatory drug (NSAID)


2.‐ Skeletal muscle relaxant
3.‐ Antihypertensive
4.‐ Opioid analgesic

3276 A client has begun taking an anticholinergic Correct answer: 2 Anticholinergic medications cause decreased stimulation in the GI and urinary tract systems, Knowing the systems involved when taking an anticholinergic will lead to the correct
medication. The nurse plans to carefully assess which leading to urinary and bowel problems such as urinary retention, hesitancy, and constipation. answer. The systems involved are the GI and urinary. The only answer that lists side effects
of the following client parameters? Other side effects of the anticholinergic medications are dry mouth and constipation. The related to these systems is option 2. The other options do not state answers related to
items in options 1, 3, and 4 do not represent particular concerns when administering an these two systems, and can be eliminated.
anticholinergic medication.
1.‐ Pain intensity, respiratory rate, and level of consciousness
2.‐ Urinary retention, hesitancy, and constipation
3.‐ Seizure activity, mental status, and respiratory status
4.‐ Electrolytes, blood glucose levels, and leg edema

3277 The nurse is transcribing medication orders for a Correct answer: 2 Potentially fatal interactions occur between selegiline and opioids, especially meperidine In order to select the correct answer to this question, recall knowledge of the
client taking selegiline (Eldepryl). The nurse makes it a (Demerol). Therefore, nurses should be aware of all medications that a client routinely takes contraindications for the medication. There is only one correct option for this question. If
priority to telephone the prescriber after noting an when selegiline is ordered concurrently. The other classifications might have interactions, but this was difficult, review selegiline (Eldepryl) and the contraindications for the medication.
order for which of the following types of medications? none are potentially fatal.

1.‐ Monoamine oxidase inhibitor (MAOI)


2.‐ Opioid analgesic
3.‐ Skeletal muscle relaxant
4.‐ Anticholinergic

3278 A client has been given a prescription for a Correct answer: 4 The medications used to treat disorders of either the neurological or musculoskeletal system The question is asking for selection of significant data. In teaching clients about their
medication affecting the musculoskeletal system. are complex, and require the client to understand how they work. When the client is informed, medication, the most significant information that should be taught to clients is that
Which of the following points would the nurse include he is more likely to take the medication correctly. Options 1 and 2 might not apply, and option identified in option 4.
as significant when teaching the client about this 3 is of lesser importance than is option 4.
medication?
1.‐ To decrease risk factors by eating a low‐fat diet, increasing exercise, and ceasing smoking
2.‐ The use of sunscreen, vitamins, and hats
3.‐ The history of the disease for which he is being medicated
4.‐ Name, dose, schedule, side effects, and possible adverse effects of the medication

3279 A client with a seizure disorder is being medicated Correct answer: 1 Phenytoin binds with the protein in the tube feedings, which decreases the medication Knowing the rate of absorption will lead to the correct answer. If this was difficult, review
with IV phenytoin (Dilantin). The physician changes the absorption into the blood. The tube feedings might need to be shut off for 30 minutes to an nursing interventions related to administration of medications by feeding tube.
mode of administration to feeding tube. Of which of hour before and after the dose. The other answers are incorrect statements.
the following points does the nurse need to be aware
so that the client's blood level of phenytoin does not
decrease?

1.‐ The tube feeding should be stopped 30–60 minutes before and after administration of the medication, to allow for proper absorption.
2.‐ The suspension of phenytoin is specially designed to be absorbed with tube feedings.
3.‐ The dose of IV phenytoin must be increased when given by the feeding tube.
4.‐ No specific requirements are needed to ensure the drug level stays steady.

3280 The client has a back injury. The provider orders Correct answer: 1 The effects of muscle relaxants are intensified when taken in combination with other central In order to select the correct answer to this question, recall knowledge of the
cyclobenzaprine (Flexeril) for the muscle spasms. What nervous system (CNS) depressants, such as alcohol or cough preparations. The client should contraindications of other medication. If this was difficult, review the use of muscle
significant educational point does the nurse need to consult with the provider before taking other medications. Cholesterol levels do not need to relaxants with CNS depressants.
share with the client? be checked (option 2). Apical pulse measurement is unnecessary (option 3), and it is
antibiotics, not cyclobenzaprine, that must be finished even if symptoms improve (option 4).
1.‐ "Do not take this medication with alcohol or cough preparations, and do not drive during its use."
2.‐ "It is important for you to have your cholesterol levels checked regularly."
3.‐ "An apical pulse must be taken prior to taking this medication."
4.‐ "The medication must be finished, even if symptoms improve or cease."

3281 For which of the following symptoms would the nurse Correct answer: 4 Tonic‐clonic seizures are the most common generalized seizures. Periods of inattention and Knowing that generalized seizures are termed tonic‐clonic seizures will lead to the only
assess in a client with the most common generalized daydreaming characterize an absence seizure. Sudden loss of muscle tone and falling correct option, 4. The other answers are not components of generalized seizures, and can
seizure disorder? characterize an atomic seizure. Repetitive small muscle group activity characterizes a partial be eliminated. If this was difficult, review seizure types and their classifications.
seizure.
1.‐ Periods of inattention and daydreaming
2.‐ Sudden loss of muscle tone, and falling
3.‐ Repetitive small muscle group activity
4.‐ Tonic and clonic activity of the extremities

3282 After a client has experienced a seizure, what is the Correct answer: 2 After the seizure, the client will be postictal, which is a deep sleeping state. He could aspirate Using the process of elimination and considering safety for clients who are high‐risk for
most appropriate position in which the nurse should secretions unless side‐lying to promote drainage from the upper airway. Positioning the client aspiration after a seizure will lead to the correct answer. The only correct answer that
place the client? on his back (option 1) increases the risk of aspiration. Positioning the client on his abdomen would address a safe position is option 2. The other options would not be safe interventions
(option 3) or upright in a chair (option 4) is unrealistic, given the client's postictal state. for the client.

1.‐ On his back, with his head raised 15 degrees


2.‐ On his side
3.‐ On his abdomen
4.‐ Upright in a chair

3283 A client is experiencing seizure activity. The nurse Correct answer: 3 Phenytoin is a first‐line anticonvulsant medication that is used to control seizure activity. Knowing the classification of the medications will lead to the correct answer. In the
should prepare to administer which of the following Selegilene (option 1) is used to treat Parkinson's disease. Diclofenac (option 2) is an NSAID, treatment of seizures, it is important to administer an anticonvulsant. The only
medications according to protocol? while sumatriptan (option 4) is used to treat headaches. anticonvulsant listed in the choices is option 3. If this was difficult, review medications that
might be prescribed for seizure disorders.
1.‐ Selegilene (Eldepryl)
2.‐ Diclofenac sodium (Voltaren)
3.‐ Phenytoin (Dilantin)
4.‐ Sumatriptan (Imitrex)

3284 Which of the following should be the highest priority Correct answer: 1 The client must understand that medication information is a priority item. Option 2 is a false The question is asking for selection of significant data. In teaching clients about their
of the education plan for a client being treated for a statement. Effective medication dosing should control seizure activity (option 4). Teaching that medication, the most significant information that should be taught to clients is option 1.
generalized seizure disorder? urine might turn pink or brown may be included if appropriate, but is not the highest priority
and global response.
1.‐ Medication information, and to take medication even if there is no seizure activity
2.‐ Physical dependency might result from extended use of medications.
3.‐ Urine might turn pink or brown, but this is not harmful.
4.‐ Therapeutic effects of medications might not be seen for 2–3 weeks.

3285 Education considerations for clients with migraines Correct answer: 3 The ability to avoid the headache triggers is important for the client, and therefore must be Understanding the need to find the triggering mechanisms for migraines headaches will
must include which of the following? included in teaching. The urine does not change color (option 1). Effective therapy should not assist in selecting the correct answer. Option 1 is the only correct answer to the question. If
take 1–2 years (option 2). Gingival hyperplasia is a concern with phenytoin (option 4), which is this was difficult, review the teaching points to a client who is taking migraine medication.
used to control seizure activity.
1.‐ Urine might turn pink or brown.
2.‐ Therapeutic effects might not be seen for 1–2 years.
3.‐ Identification of triggers for headaches, and how to decrease them
4.‐ Proper brushing of teeth with soft toothbrush, and flossing, to prevent gingival hyperplasia
3286 The nurse explains to a client recently diagnosed with Correct answer: 3 Dopamine is the neurotransmitter that is lacking in Parkinson's disease (PD). The other Knowing the pathophysiology with Parkinson’s disease will lead to the correct answer. If
Parkinson's disease that most of the medications used neurotransmitters are not as integral in PD. this was difficult, review Parkinson’s disease.
to treat the disorder improve the availability of what
substance in the brain?

1.‐ Acetylcholine
2.‐ Serotonin
3.‐ Dopamine
4.‐ Glutamic acid

3287 The nurse observing a client with Parkinson's disease Correct answer: 2 Clients with PD have difficulty initiating movement, so arising from a chair is difficult without Knowing the clinical manifestations of Parkinson’s disease will assist in selecting the
(PD) would conclude that which of the following assistance. The tremors of PD are resting tremors, not intentional. Medication therapy is correct answer. The only correct manifestation of Parkinson’s disease is option 2. If this was
manifestations is or are most likely to be seen? targeted at controlling tremors. The other options do not reflect manifestations of this difficult, review clinical manifestations of Parkinson’s disease.
disorder.
1.‐ Regular fine motor tremors of both hands that persist with intentional activity
2.‐ Difficulty in arising from a chair and beginning to walk without assistance
3.‐ Follows objects around the room with her eyes, and often smiles to herself when left alone in room.
4.‐ Weakness of right leg, becoming more noticeable after a warm bath or shower

3288 A client had surgery for a herniated disk two days Correct answer: 2 The client must have pain control with oral medications and have begun an exercise program Note that the question states that the client has an IV/IM medication ordered, but also an
ago. The medications ordered include IV/IM morphine prior to discharge. Option 1 is completely incorrect, while options 3 and 4 are partially oral medication. This should allow for consideration that the client’s pain control should
sulfate (Morphine), an oral pain pill, and oral muscle incorrect. eventually be with an oral medication. Knowing this, select option 2.
relaxant, cyclobenzaprine (Flexeril). What are the
nursing goals of care so the client can be discharged
tomorrow?
1.‐ Pain control achieved with IV medication only, and client maintains bedrest.
2.‐ Pain control achieved with oral medications, and an exercise program is begun.
3.‐ Pain control achieved with combination of IV/IM and oral medications, and an exercise program is begun.
4.‐ Pain control achieved with oral medications, and client maintains bedrest.

3289 Which of the following nursing diagnoses has highest Correct answer: 2 The most important nursing diagnosis for the client at this time is related to inadequate The question is asking for selection of the highest‐priority answer. When a client is
priority for a client newly diagnosed with tonic‐clonic knowledge of medication management. Ineffective sexuality patterns and impaired thought beginning a new medication, the highest priority is usually knowledge deficiency. The client
seizure disorder beginning medication therapy? processes may be addressed later as needed, and pain related to headache is not applicable. will need to be taught the significant information about the medication and its side effects.

1.‐ Ineffective Sexuality Patterns


2.‐ Deficient Knowledge
3.‐ Pain related to headache
4.‐ Impaired Thought Processes

3290 Which of the following nursing diagnoses has highest Correct answer: 3 The primary issue for migraine headache sufferers is pain relief, which is amenable to The question is asking for selection of the highest‐priority answer. When a client has a
priority for a client newly diagnosed with migraine treatment with medication therapy. Disturbed sleep pattern may be addressed later, while diagnosis of migraine headaches, pain is usually the priority. Option 3 is the correct priority
headaches? ineffective sexuality patterns and impaired thought processes are not applicable. diagnosis with this question.

1.‐ Ineffective Sexuality Patterns


2.‐ Disturbed Sleep Pattern
3.‐ Pain related to headache
4.‐ Impaired Thought Processes

3291 A client has an order to begin an IV nitroglycerin Correct answer: 2 Intravenous nitroglycerin (NTG) must be prepared only in glass bottles, and infused via the The core issue of the question is knowledge that nitroglycerin adsorbs into plastic, making
(Nitrostat) drip. The nurse prepares this medication by manufacturer‐provided tubing. The polyvinyl chloride in regular tubing will adsorb (leech out) it necessary to use a glass bottle and special IV tubing from the manufacturer. Use nursing
mixing the medication: the nitroglycerin. NTG is stable in a glass bottle for 24 hours, and does not require laminar flow knowledge related to pharmacology and the process of elimination to make a selection.
ventilation.
1.‐ In a solution that is covered by a plastic bag.
2.‐ In a solution that is in a glass bottle.
3.‐ Every 2 hours, because it is unstable.
4.‐ Under a laminar flow hood.

3292 A client with angina pectoris received nitroglycerin Correct answer: 2 Headache is a common side effect (not adverse reaction) related to the vasodilation The core issue of the question is knowledge of common adverse effects of nitroglycerin
tablets sublingually for chest pain. The client dislikes properties of nitroglycerin. The incidence of headache decreases over time as the client therapy. Use nursing knowledge related to pharmacology and the process of elimination to
the medication because it causes headache. The nurse develops tolerance to the medication. The client should be encouraged to continue its use as make a selection.
makes which of the following interpretations about the needed, and to take acetaminophen or aspirin for headache, according to the preference of
client’s statement? the physician.

1.‐ This is a common but unhealthy response to the medication.


2.‐ This is a common response that will diminish as tolerance to the medication develops.
3.‐ This is a response caused by cerebral hypoxia induced by the medication.
4.‐ This is an adverse reaction that must be reported to the physician immediately.

3293 A client who has just been diagnosed with Correct answer: 2 Adverse effects of beta‐adrenergic blockers such as propranolol include their potential to The core issue of the question is knowledge of contraindications for beta‐adrenergic
hypertension also smokes, and has diabetes mellitus. cause bronchospasm and to mask hypoglycemia attacks. Therefore, the clients who are at risk blockers, such as propranolol. Use nursing knowledge related to pharmacology and the
The nurse would question an order for which of the for these conditions should not utilize beta blockers as antihypertensive medications. Calcium process of elimination to make a selection.
following antihypertensive medications? channel blockers, alpha blockers, and diuretics do not directly affect these conditions.

1.‐ Diltiazem (Cardizem)


2.‐ Propranolol (Inderal)
3.‐ Prazosin (Minipress)
4.‐ Furosemide (Lasix)

3294 Diltiazem (Cardizem) is prescribed for a client with Correct answer: 3 Diltiazem (Cardizem) is a calcium channel blocker. It is usually administered before meals and The wording of the question tells you that the correct answer is an incorrect statement.
chronic, stable angina. The clinic nurse determines that at bedtime to increase the absorption of medication. Postural hypotension can occur, so the Recall information about calcium channel blockers and use the process of elimination to
the client needs additional medication information if client must be instructed to rise slowly to avoid dizziness and falling. The medication can cause make a selection.
the client makes which of the following statements? a decrease in mental alertness until the body adjusts and the proper dosage is established. The
client should notify the physician if she develops shortness of breath, irregular heartbeat,
pronounced dizziness, nausea, or constipation.

1.‐ “I will call the physician if shortness of breath occurs.”


2.‐ “I will rise slowly when getting out of bed in the morning.”
3.‐ “I will take medications after meals.”
4.‐ “I will avoid activities requiring mental alertness until my body adjusts to the medication.”

3295 The nurse has given medication instructions to the Correct answer: 2 Nicardipine (Cardene) is a calcium channel blocker. Weight gain and edema are potential The core issue of the question is knowledge of teaching points regarding calcium channel
client receiving nicardipine (Cardene) for angina. The signs of heart failure, and must be reported to the physician. The client taking this medication blockers, such as nicardipine. The wording of the question tells you that the correct answer
nurse would reinforce the teaching if the client makes should keep track of angina episodes, and report any increase in the episodes or change in the is an incorrect statement. Use nursing knowledge related to pharmacology and the process
which of the following statements? pattern. The client may take a missed dose of medication if not too close to the next dose; of elimination to make a selection.
otherwise, the dose should be omitted. The client should be taught to check his pulse, note the
rate, and report if the heart rate is lower than 50 beats per minute.

1.‐ “I will keep track of angina episodes, and report them if they increase.”
2.‐ “I will ignore edema or weight gain as an expected side effect of the medication.”
3.‐ “I will report a pulse rate of fewer than 50 beats per minute.”
4.‐ “I will take any missed dose as soon as remembered, unless it is almost time for the next dose.”
3296 A client with hypertension has been given a Correct answer: 1 Cough and loss of taste are common side effects of angiotensin‐converting enzyme (ACE) The core issue of the question is knowledge that ACE inhibitors lead to cough and loss of
prescription to treat the disorder. The nurse would inhibitors such as lisinopril. They disappear with discontinuation of the medication. The taste perception. From there, you must be able to identify which drug is an ACE inhibitor.
explain that cough and loss of taste are side effects if medications listed in the other options do not produce cough or change taste perception. Recall that these drugs end in ‐pril to help make a selection.
which of the following antihypertensive agents is
prescribed?
1.‐ Lisinopril (Prinivil)
2.‐ Propranolol (Inderal)
3.‐ Diltiazem (Cardizem)
4.‐ Furosemide (Lasix)

3297 The physician prescribes losartan (Cozaar) for a client Correct answer: 4 Losartan is an angiotensin II antagonist that inhibits the conversion of angiotensin I to The core issue of the question is knowledge of the mechanism of action of angiotensin‐
with hypertension. The nurse carrying out the order angiotensin II. Because angiotensin II is a powerful vasoconstrictor, this inhibition results in receptor blockers. To reach the correct answer, it is necessary to recognize that the drug is
explains to the client that this medication promotes vasodilation and normalizing blood pressure. The client should be assessed for dizziness, in this class. Use nursing knowledge related to pharmacology and the process of elimination
vasodilation by: cough, and diarrhea while taking this medication. to make a selection.
1.‐ Preventing calcium from going into the cells.
2.‐ Promoting epinephrine and norepinephrine.
3.‐ Promoting release of aldosterone.
4.‐ Inhibiting conversion of a substance that would cause vasoconstriction.

3298 The client with hypertension states that he monitors Correct answer: 2 Lack of adherence to pharmacologic treatment strategies prevents the client from The core issue of the question is knowledge that antihypertensive medications need to be
his blood pressure daily. His medications include establishing good control of the disease, and ultimately places him at risk for developing long‐ taken as scheduled without missing or skipping doses. Use nursing knowledge related to
verapamil (Calan SR) 240 mg daily and term complications of hypertension. Noncompliance with the therapeutic program is a pharmacology and the process of elimination to make a selection.
hydrochlorothiazide (HCTZ) 12.5 mg daily. He states significant problem in people with hypertension. The client should not skip doses of
that if his BP reading is lower than 140 systolic, he medications without consulting the physician.
skips his dose for the day. What would be the most
appropriate response by the nurse?

1.‐ “As long as the systolic is lower than 140, it is OK to skip the dose.”
2.‐ “You should not skip doses unless instructed by the ordering physician.”
3.‐ “Maybe you won’t even need your BP medications in a few more months.”
4.‐ “Your doctor may want to stop the HCTZ and have you take only the Calan.”

3299 Adenosine (Adenocard) is to be administered to a Correct answer: 3 Adenosine (Adenocard) is an antidysrhythmic used in the treatment of paroxysmal The core issue of the question is the most important piece of equipment needed to
client in the Emergency Department. Before preparing supraventricular tachycardia (SVT). Cardiac performance must be assessed before and monitor a client receiving adenosine. Recall that this drug is an antidysrhythmic to help
the medication, the nurse ensures that which of the throughout treatment by cardiac monitoring. An endotracheal tube may be used if an choose the cardiac monitor as the appropriate answer.
following priority pieces of equipment is operational? emergency necessitates mechanical ventilation, but the tube itself is a rather isolated item. An
IV pump might be needed, but is not a priority, because this medication is administered rapidly
by IV push. A pulse oximetry machine might be helpful in assessing oxygenation, but is not a
priority item.
1.‐ A pulse oximetry machine
2.‐ An IV infusion pump
3.‐ A cardiac monitor
4.‐ An endotracheal tube

3300 The home health nurse would be most concerned Correct answer: 2 Anorexia, nausea, and yellow vision are signs of digoxin toxicity. Other signs include other The core issue of the question is knowledge of early signs of digoxin toxicity. Recall that
that a client is developing digoxin toxicity after noting visual disturbances, vomiting, and diarrhea. The clusters of other symptoms listed does not fit early signs are usually more subtle than are later signs. Use nursing knowledge related to
which of the following during a routine visit? the profile of digoxin toxicity. pharmacology and the process of elimination to make a selection.

1.‐ Palpitations, elevated blood pressure, and shortness of breath


2.‐ Anorexia, nausea, and reports of yellow vision
3.‐ Chest pain, fatigue, and decreased blood pressure
4.‐ Taste alterations, dry mouth, and constipation
3301 A client with myocardial infarction is experiencing Correct answer: 4 Lidocaine is a class I antidysrhythmic used to treat ventricular dysrhythmias. Other The core issue of the question is knowledge of first‐line treatment for PVCs. Use nursing
new, multiform, premature ventricular contractions medications that might be ordered include procainamide, amiodarone, or magnesium sulfate. knowledge related to pharmacology and the process of elimination to make a selection.
(PVCs). The nurse checks the medication cart to ensure The other medications would not be used as a primary treatment of ventricular dysrhythmias
that which of the following medications is available for because they are a cardiac glycoside (option 1), a beta blocker (option 2), and a calcium
immediate use? channel blocker (option 3), respectively.

1.‐ Digoxin (Lanoxin)


2.‐ Metoprolol (Lopressor)
3.‐ Verapamil (Isoptin)
4.‐ Lidocaine (Xylocaine)

3302 The nurse is preparing to administer amiodarone Correct answer: 3 Amiodarone is a class III antiarrhythmic used to treat life‐threatening ventricular The core issue of the question is knowledge of what parameter needs to be monitored
(Cordarone) IV. The nurse should assure that which of dysrhythmias that do not respond to first‐line drugs (like lidocaine). The client should have carefully during amiodarone therapy. Eliminate options 1 and 4 (oxygen and oxygen
the following is in use at this time? continuous EKG monitoring, and the medication should be infused through an IV pump. saturation monitor) because they are similar. Recall that the drug is an antidysrhythmic
Oxygen therapy might be needed, but is unrelated to this medication. Options 2 and 4 are not agent to make the final selection.
critical during administration of this medication, although they are generally useful adjuncts.

1.‐ Oxygen therapy


2.‐ Noninvasive blood pressure monitoring
3.‐ Continuous cardiac monitoring
4.‐ Oxygen saturation monitoring

3303 Which medication does the nurse anticipate will be Correct answer: 3 Heparin is the drug of choice in pregnancy. Low‐molecular‐weight heparins, of which The core issue of the question is knowledge of the anticoagulant that is safe to use during
used for a pregnant client who requires enoxaparin is an example, are not recommended for use during pregnancy (options 1 and 4). pregnancy. Use nursing knowledge related to pharmacology and the process of elimination
anticoagulation therapy? Epoetin alfa (Procrit) is a colony‐stimulating growth factor, and is not used for anticoagulation to make a selection.
(option 2).
1.‐ Low‐molecular‐weight heparin (LMWH)
2.‐ Epoetin (Procrit)
3.‐ Heparin (Liquaemin)
4.‐ Enoxaparin (Lovenox)

3304 The nurse would include in a teaching plan that a Correct answer: 3 Clients who have atrial fibrillation are at risk to develop emboli. Therapy with Coumadin is The core issue of the question is knowledge that treatment for prevention of blood clot
client taking warfarin (Coumadin) for atrial fibrillation considered to be ongoing in nature, in order to prevent such an occurrence. The other time formation from atrial fibrillation is indefinite. Use nursing knowledge related to
will need to remain on drug therapy for what period of frames are too short to achieve a preventative goal. In addition, the likelihood of emboli pharmacology and the process of elimination to make a selection.
time? formation does not significantly diminish unless the client is anticoagulated on a long‐term
basis.
1.‐ 6 months
2.‐ 2–3 months
3.‐ Indefinite, or long‐term
4.‐ 1 year

3305 A client is placed on ticlopidine (Ticlid) following a Correct answer: 1 A client taking ticlopidine should be monitored for potential blood dyscrasias that can occur The core issue of the question is knowledge of adverse effects of ticlopidine that can be
stroke. What follow‐up blood work is indicated in with this drug. Monthly PT and INR levels are not indicated as follow‐up for this medication, detected using laboratory monitoring. Use nursing knowledge related to pharmacology and
managing the client? but are used in conjunction with Coumadin therapy (option 2). ABGs are not indicated in the the process of elimination to make a selection.
management of clients who are taking ticlopidine (option 3) There are no reported electrolyte
imbalances with the use of this medication (option 4).

1.‐ Frequent CBC monitoring to evaluate for blood dyscrasias


2.‐ Monthly PT and INR levels to evaluate for clotting problems
3.‐ ABGs to evaluate respiratory status
4.‐ Serum chemistries to monitor for potential electrolyte imbalances
3306 A client is undergoing percutaneous transluminal Correct answer: 2 ReoPro is often given IV following this type of procedure, to help prevent possible reocclusion The core issue of the question is knowledge of drugs that have antiplatelet properties and
coronary angioplasty (PTCA), and requires an of the coronary artery that has been treated. It can be administered in conjunction with weight‐ that can be used IV following interventional cardiology procedures such as PTCA. Use
antiplatelet agent. The nurse prepares to administer based heparin therapy, but heparin is an anticoagulant agent (option 1). Plavix and ASA are nursing knowledge related to pharmacology and the process of elimination to make a
which drug for this type of client immediately examples of antiplatelet agents that are given orally, and are not utilized in this particular selection.
following the procedure? acute‐care setting (options 3 and 4). However, ASA can be given later as follow‐up to the
procedure, to prevent possible complications related to vessel occlusion.

1.‐ Heparin (Liquaemin)


2.‐ Abciximab (ReoPro)
3.‐ Clopidrogrel bisulfate (Plavix)
4.‐ Aspirin (ASA)

3307 A client is receiving thrombolytic therapy. The nurse Correct answer: 3 Clients taking thrombolytic therapy should be monitored closely for the possibility of The core issue of the question is knowledge that thrombolytics can lead to bleeding. With
monitors the client for what potential problem? hemorrhage, as therapy increases the risk of bleeding. Monitoring includes evaluating the skin this in mind, recall the various ways that bleeding can manifest in a client taking drugs that
for bruising and gums, and venipuncture sites for bleeding, and testing stool and urine for interfere with clotting. Use nursing knowledge related to pharmacology and the process of
occult or obvious blood. Headache, fever, and bone pain are not directly related to elimination to make a selection.
thrombolytic therapy (options 1, 2, and 4).
1.‐ Headache
2.‐ Fever
3.‐ Hematuria
4.‐ Bone pain

3308 The nurse would assess a client who is receiving Correct answer: 4 The use of thrombolytic agents can cause cardiac irritation and lead to development of The core issue of the question is knowledge that thrombolytic drugs can cause reperfusion
thrombolytic therapy for which of the following? dysrhythmias that can be life‐threatening. The nurse must be aware of the serious likelihood dysrhythmias as a result of clot lysis. Use nursing knowledge related to pharmacology and
that treatment can cause further cardiac compromise. Dry mouth, decreased urine output, and the process of elimination to make a selection.
decreased clotting times (options 1, 2, and 3) are not seen with thrombolytic therapy.

1.‐ Dry mouth


2.‐ Decreased urine output
3.‐ Decreased clotting times
4.‐ Cardiac dysrhythmias

3309 For which of the following would a nurse monitor in a Correct answer: 1 Folic acid (in large doses) can cause the urine to become discolored and turn to a darker‐ The core issue of the question is knowledge of expected side effects of folic acid. Recall
client taking folic acid (Folvite)? yellow color. Dark‐green or black stools are more commonly associated with iron therapy that B complex vitamins can turn the urine a darker yellow as an aid to answering the
(option 2). Temperature elevations and changes in pulse rate are not associated with folic acid question. Use nursing knowledge related to pharmacology and the process of elimination to
(options 3 and 4). make a selection.
1.‐ Dark‐yellow urine
2.‐ Dark‐green or black stools
3.‐ Temperature elevations
4.‐ Increased pulse rate

3310 Which of the following measures should the nurse Correct answer: 3 Liquid iron preparations can cause staining of teeth. It is important for the nurse to be aware The core issue of the question is knowing that liquid iron stains the teeth. With this in
utilize when administering ferrous sulfate (Feosol) of proper administration methods, which include drinking the mixture through a straw. Mixing mind, select the option that prevents the liquid from being absorbed through tooth enamel.
elixir? medication with milk and carbonated beverages will decrease its absorption (options 1 and 4). Keeping other administration principles in mind helps you to eliminate the incorrect
The medication is usually taken with food to minimize GI upset (option 2). options.

1.‐ Mix the medication with milk to decrease GI effects.


2.‐ Administer oral medication without food.
3.‐ Administer the medication through a straw.
4.‐ Mix the medication with carbonated beverages to minimize gastric upset.
3311 A client has an order to receive 5,000 units of heparin Correct answer: 0.5 To calculate the dose, divide the desired dose (5,000) by the dose on hand (10,000 units) and The core issue of the question is the ability to calculate a drug dose. If necessary,
subcutaneously. Available is a vial labeled "Heparin multiply that by the quantity (1 mL). The result is 0.5 mL. memorize this basic formula for use in solving many medication questions.
10,000 units per mL." The nurse should administer
_____ mL of heparin solution.

3312 A client has been diagnosed with iron deficiency Correct answer: 4 Vitamin C helps to enhance the absorption of iron in the diet, and is an easy step in diet Note that the question is referring to maintaining iron levels, which should lead to thinking
anemia (IDA), and wants to know what foods would management towards improving iron levels in the body. A strict vegetarian diet focuses on non‐ about how iron can best be absorbed. Knowledge of the need to consume vitamin C with
help to maintain adequate iron levels in the body. heme sources of iron that are not as readily absorbable as heme sources (option 1). Eating ice iron will lead you to the only correct option, 4.
Which of the following items regarding diet cubes is an example of pica, which is ingestion of a non‐food substance (option 2). Nonfood
information would be beneficial for the nurse to items will not help to maintain iron, or prevent iron deficiency, and in certain cases can
include in a teaching plan for achieving the goal of actually lead to deficiency states. Tea contains tannic acid, and cereals contain phytates and
increased iron levels? fibers, all of which lead to decreased iron absorption in the diet (option 3).

1.‐ Maintain a strict vegetarian diet.


2.‐ Eat ice cubes that are present in beverages.
3.‐ Increase tea and cereal in the diet.
4.‐ Use adequate sources of vitamin C in the diet.

3313 A client is taking anti‐platelet medication for several Correct answer: 4 Inspection of the client's skin is necessary to verify if there are additional areas of bruising or The question is asking about assessment, the first part of the nursing process. Inspection
weeks, and presents with a noticeable bruise on the discoloration of which the client might not be aware. It is important to review current findings of the client s body for other bruising and discolorations should be the first part of care. The
arm. What information should the nurse assess first in and compare them with baseline findings, as this might provide data to support a potential other options are all subjective data, and can be gathered as a unit after inspecting the
order to determine if this skin manifestation is related response to drug therapy. Asking the client if the bruising is related to a particular incident is client s body.
to drug therapy? important (option 1); however, it does not rule out the possibility that drug therapy has made
the individual more susceptible to bruising or bleeding tendencies. If the client has not taken
the medication as ordered, it would be unlikely that the bruise would be a consequence of
drug therapy (option 2). It is important for the client to continue to self‐monitor during drug
therapy, but that choice by itself does not answer the question (option 3).

1.‐ Whether the bruising is a result of a specific injury, and therefore not caused by drug therapy
2.‐ Whether the client has taken the medication for the last several days
3.‐ Whether the client self‐monitors for skin manifestations
4.‐ Whether the client has bruising and discoloration on other areas of the body

3314 In order to verify iron stores in the body, for which Correct answer: 1 Ferritin levels reflect the visceral stores of iron in the body. Transferrin levels reflect how iron Having knowledge of specific laboratory tests will assist in selecting the correct answer.
priority laboratory test result would the nurse look in a is transported in the body (option 2). Hemoglobin and hematocrit refer to concentration and The only correct answer for this question is option 1, since it directly will reflect iron stores
client's medical record? proportion measures of red blood cells (RBCs). While they provide information relative to in the body. If this was difficult, specifically review laboratory tests and their purposes.
blood count, they are not specific to body iron store values (option 3). A CBC will provide
information relative to blood concentration of all three cell lines (red, white, and platelets) but
again, it is not specific to body iron store values (option 4).

1.‐ Ferritin level


2.‐ Transferrin level
3.‐ Hemoglobin and hematocrit
4.‐ Complete blood count (CBC)

3315 A client is being treated with alteplase (Activase) in Correct answer: 2 Activase is used in the emergency setting post‐stroke and –myocardial infarction (MI) in order Knowledge of the medication and side effects will lead to the correct answer. Since this
the Emergency Department following a to dissolve clots and increase perfusion. A client receiving this medication is at risk to develop medication has the primary side effect of dysrhythmias, the nurse would need to assess the
cerebrovascular accident (CVA or stroke). What is the significant cardiac dysrhythmias, and therefore should be placed on a cardiac monitor during client closely through cardiac monitoring. If this was difficult, review the side effects of the
priority nursing intervention related to the care of a treatment. While vital signs (including blood pressure and temperature) are important, they medication.
client receiving this drug therapy? are not the priority intervention (options 1 and 4). The monitoring of urinary output is not a
priority unless there are underlying conditions regarding volume management (option 3).
1.‐ Perform vital signs every 15 minutes until the client's blood pressure is stabilized.
2.‐ Place the client on a cardiac monitor, and observe for potential dysrhythmias, as this medication can have cardiac effects.
3.‐ Insert a urinary catheter, and maintain accurate hourly output measures.
4.‐ Monitor the client for hypothermia, as this is a common side effect of this medication.

3316 A client is taking epoetin alfa (Epogen) for treatment Correct answer: 3 The target range for hematocrit with epoetin alfa therapy is 30–36%. A client who is taking Recall knowledge of client responses to the medication to be led to option 3. If this was
of anemia related to chronic renal disease. What Epogen must be monitored closely, to prevent adverse side effects that can occur because of difficult, review the acceptable client responses to the medication.
clinical finding reveals to the nurse that this either a rapid increase or high‐level hematocrit. Rapid or increased hematocrit levels can cause
medication is working effectively? the client to develop seizures and hypertension (option 2). Bone pain and fever are seen in
response to administration, and are not indicators of effective drug management (options 1
and 4).
1.‐ The client is not experiencing any related bone pain when the medication is being administered.
2.‐ The client's hemoglobin and hematocrit levels are rising rapidly based on the latest two daily blood draws.
3.‐ The client's hematocrit is in the established target range at 33%.
4.‐ The client is afebrile.

3317 A client who has elevated cholesterol levels has been Correct answer: 2 Facial flushing is an expected side effect of niacin, caused by its vasodilator properties. While The question is asking for selection of side effects of the medication. The only option that
prescribed nicotinic acid (Niacin). What information dosages are often adjusted, it is usually for the purpose of managing side effects related to refers to side effects is option 2. Knowledge of the side effects of the medication will lead
would the nurse provide to this client? gastrointestinal complaints, and not based on lab values (option 1). Additional dietary sources to the correct answer. The other options are all interventions, and can be eliminated, since
of niacin are not required to enhance the effect of niacin supplements (option 3). Clients the question is referring to side effects.
should not take niacin and lovastatin concurrently, as this can lead to the development of
myopathy (option 4).
1.‐ Niacin treatment is highly individualized, and there might be dose adjustments based on lab values.
2.‐ Expect facial flushing, as this is a common expected effect of this medication.
3.‐ Dietary sources of niacin are necessary to ensure that the medication works effectively.
4.‐ Niacin can be taken concurrently with lovastatin (Mevacor) in order to maximize the therapeutic effect.

3318 A client with von Willebrand's disease is taking Correct answer: 1 DDAVP and cryoprecipitate are considered effective treatments for clients with von Having knowledge of the disease process and usual mode of treatment will lead to the
desmopressin (DDAVP) and cryoprecipitate as part of Willebrand's disease because they contain specific clotting factors (vW and VIII). They are correct answer. If this was difficult, review the disease process and medication regimen.
the treatment regimen. The nurse would explain that considered a form of replacement therapy, but do not stimulate production of blood factors
medication therapy will provide what pharmacologic (option 4). While DDAVP is an antidiuretic, it also has other pharmacological actions in the
benefit to this client? body; specifically, it stimulates the body to release vW factor (option 2). These medications do
not have to be given concurrently in order to potentiate their effects (option 3). Clients can
receive them independently, based on physician preference.

1.‐ These medications will help to restore and release deficient clotting factors (vW and VIII) that occur in this disease process.
2.‐ DDAVP is given for its antidiuretic effect to promote fluid management.
3.‐ Cryoprecipitate and DDAVP must be given concurrently in order to potentiate their therapeutic effect.
4.‐ These medications will stimulate the client's own production of specific blood factors.

3319 The nurse reviewing a client's laboratory test results Correct answer: 4 Hemostasis is the ability of the body to prevent bleeding and hemorrhage using platelets in Understanding the definition of hemostasis will lead to the correct answer. The only
would conclude that which of the following would the coagulation process. Thrombocytopenia (a reduced level of platelets in the body) can correct option is 4, since thrombocytopenia refers to platelets, which in turn relate
affect the client's ability to maintain hemostasis? profoundly affect the body's ability to react to a vascular insult. Neutropenia (a reduced specifically to the ability to prevent bleeding. The other options do not refer to hemostasis,
neutrophil count affecting WBCs) can profoundly affect the body's ability to react to an and can therefore be eliminated.
immune response (option 3). Low ferritin levels and elevated triglyceride levels do not directly
affect a body's ability to maintain hemostasis (options 1 and 2).

1.‐ Low ferritin level


2.‐ Elevated triglyceride level
3.‐ Neutropenia
4.‐ Thrombocytopenia
3320 A client is being discharged on warfarin (Coumadin) Correct answer: 2 Clients who have valve replacement surgery require lifelong anticoagulation therapy. The question refers to the teaching required for a client who is taking anticoagulant
following heart valve replacement surgery. What Therefore, they must be instructed as to the possible risks for bleeding, and must modify their therapy. Since all the answer choices are examples of teaching, the method of selecting the
information would the nurse give to the client to environment and activities of daily living accordingly. Follow‐up lab testing is required, but is correct option will depend on knowledge of the medication. Recall understanding of the
provide safe and effective care during the course of not usually limited to a weekly basis (option 1). Coumadin therapy is usually taken as an teaching required for clients taking the medication to select the correct option.
therapy? evening dose (option 3). Clients are instructed not to double up doses, and to take the
medication specifically as ordered, to assure safe and therapeutic effects (option 4).

1.‐ Follow‐up lab testing is required on a weekly basis in order to monitor client response.
2.‐ Be alert to the possibility of bleeding tendencies caused by this drug therapy, and use electric razors and soft toothbrushes.
3.‐ Take medication upon arising in the morning on an empty stomach to maximize absorption.
4.‐ Since the therapy is based on achieving a therapeutic blood level, if a dose is missed, double up the next dose.

3321 The nurse has just received an order to start Correct answer: 3 Heparin administration requires the use of an infusion pump in order to maintain an accurate Note that this question refers to safety as the main issue. Select the most important
intravenous heparin (Liquaemin) therapy on a client level of medication. While vitamin K is important in the coagulation cascade, it is not required intervention, option 3, as there must be accurate administration of the medication, and
admitted for deep vein thrombosis (DVT). What to be readily available when heparin is being infused. Protamine sulfate is a heparin antagonist, thus safety.
nursing intervention would the nurse employ in order and should be used when reversal is indicated (option 1). The client does not have to be NPO
to implement this order? during this type of therapy (option 2). If using weight‐based therapy, it is important to weigh
the client at the same time with the same scale each day to verify accuracy (option 4).

1.‐ Vitamin K should be readily available as long as the client remains on heparin therapy.
2.‐ The client should remain NPO as long as the medication is infusing.
3.‐ An infusion pump should be utilized for the administration of heparin.
4.‐ The client should be weighed twice a day in order to evaluate for potential fluid overload.

3322 A client is being discharged with a diagnosis of angina. Correct answer: 2 Teach the client that the activity in which he is engaged might be causing the chest pain. The question is referring to safe and effective administration of the medication. The
The nurse is teaching the client about the use of Instruct the client in the exact method of taking NTG to avoid dizziness. The teaching about the options are all interventions, so in order to select the correct answer, recall knowledge of
nitroglycerine (NTG) tablets at home. What client frequency of the medication must be accurate and specific to prevent overdose, as could the correct administration of the medication.
teaching is needed related to this medication? happen in option 4. Option 1 is incorrect because NTG becomes unstable when exposed to
heat, light, and moisture. Option 3 is incorrect because the client shouldn't drive, for safety
reasons.
1.‐ "Keep NTG tablets in your pants pocket next to your body to keep them handy at all times. Take two NTG tablets with a glass of water, and then go back to your activities."

2.‐ "Stop your activities and sit down near a telephone if possible, and place one NTG under your tongue. Take no more than three tablets, one every five minutes. If the pain is
unrelieved after a total of three NTG tablets, seek help."
3.‐ "Stop your activities, take two NTG tablets, and drive immediately to your doctor's office."
4.‐ "Continue your activities slowly. Take three NTG tablets every five minutes until your chest pain is gone."

3323 The client states, "I always put my nitroglycerin (NTG) Correct answer: 3 Nitroglycerine patches and ointments must be rotated daily to a hairless area to reduce skin The question is referring to safe and effective administration of the medication. The
patch in the same place, so I do not forget to take it irritation. Options 2 and 4 are incorrect statements, while option 1 is only partially correct. options are all interventions, so in order to select the correct answer to the question, recall
off." The nurse s best response would be: knowledge of the correct administration of the medication.

1.‐ "This is good, but it is very important to take the patch off each night."
2.‐ "Sometimes the patch should be placed over hairy areas to vary the absorption."
3.‐ "You should rotate the NTG patch to a different hairless area each day."
4.‐ "It does not matter if you leave your patch on when you put the next one on. The medication is all gone."

3324 A client is hospitalized for heart failure, and is Correct answer: 2 It is important to monitor the apical‐radial pulse for a full minute before the administration of The question is referring to client responses. Option 1 can be eliminated, since it refers to
receiving digoxin (Lanoxin) IV push. The nurse should digoxin. Record and report significant changes from the client's own baseline data. Without history, and not responses. The priority intervention with the medication is to take an apical
withhold the drug and notify physician if the client's: solid data regarding the client's baseline, it is prudent to report a heart rate lower than 60, pulse before administration, so option 2 is correct in referring to the client response.
since bradycardia could indicate drug toxicity. Depression, respiratory rate, and blood pressure Having knowledge of when to hold the medication will assist in selecting this answer.
are unrelated to this medication.
1.‐ History reveals depression.
2.‐ Pulse is 53 beats per minute.
3.‐ Respiratory rate falls below 18.
4.‐ Blood pressure is 148/90.

3325 A physician orders a nitroglycerine (NTG) drip to be Correct answer: 3 The blood pressure and heart rate must be monitored closely during titration to prevent Side effects of the medication are hypotension and tachycardia. Having this knowledge
titrated. The nurse carrying out the order would hypotension and tachycardia. Shortness of breath is important, but not directly related to NTG base will lead to option 3 as the correct answer choice. If this was difficult, review the side
monitor which of the following parameters? infusion. Respirations, urine output, and headache do not determine the titration rate. effects for the medication.
Headache frequently occurs, and is treated commonly with acetaminophen or another mild
analgesic.
1.‐ Shortness of breath and level of consciousness
2.‐ Respirations and urine output
3.‐ Blood pressure and pulse rate
4.‐ Headache and blood pressure

3326 A client is receiving metoprolol (Lopressor) for Correct answer: 1 Metoprolol is a beta blocking agent that blocks the effects of both β&lt;sub&gt;1&lt;/sub&gt; Knowledge of any respiratory complications would be required prior to administering this
hypertension. The nurse should ask the client about a and β&lt;sub&gt;2&lt;/sub&gt; receptors, leading to a reduction in systemic vascular medication. This is the only correct answer, since the other options are not significant in
history of which of the following conditions? resistance. This effect also can lead to bronchospasm (from bronchoconstriction secondary to the administration of the medication. If this was difficult, review the medication s purpose
β&lt;sub&gt;2&lt;/sub&gt; blockade), and therefore metoprolol would be contraindicated in and outcome effects.
clients with bronchospastic illness. The drug has no effect on seizure activity or on myasthenia
gravis, a neuromuscular disorder.

1.‐ Bronchospasm
2.‐ Seizure
3.‐ Hypertension
4.‐ Myasthenia gravis

3327 A nurse assesses a client for side effects of verapamil Correct answer: 4 Verapamil is a calcium channel blocker used to treat angina. Constipation is a frequent Recall uses and side effects of this medication. If this was difficult, review the side effects
(Calan SR). Which client manifestation would the nurse complaint of clients taking the sustained‐release form of verapamil. Many elderly clients have of the medication.
anticipate as a side effect of this medication? difficulty with this, and the nurse must anticipate the need for teaching about increasing fiber
and fluid intake. Hypotension is an adverse reaction to verapamil. Skin rash is unrelated to the
medication.
1.‐ Hypertension
2.‐ Angina
3.‐ Skin rash
4.‐ Constipation

3328 The client is receiving a lidocaine infusion at a rate of Correct answer: 1 Early indications of toxicity to lidocaine include various central nervous system (CNS) Having knowledge of toxicity of this medication on the central nervous system (CNS) will
2 mg per minute via infusion pump. The nurse should complications. These can include slurred speech, dizziness, confusion, and paresthesias. If they lead to the correct answer. Since the CNS is the system that is mostly affected with this
discontinue the infusion and notify the physician are ignored, the client can develop seizures that are often difficult to stop, and death could medication, the only correct option is 1. The other options can be eliminated with
immediately when the client does which of the ensue. Sinus tachycardia is unrelated to lidocaine. Concerns about the myocardial infarction knowledge of the primary side effects with the CNS.
following? are normal, and should be addressed. Leg cramps are unrelated to lidocaine use.

1.‐ Demonstrates slurred speech.


2.‐ Demonstrates a sinus tachycardia of 102.
3.‐ States he is concerned about his "heart attack."
4.‐ Reports leg cramps.

3329 The nurse anticipates a positive inotropic effect after Correct answer: 2 Digoxin is classified as a positive inotropic drug. It increases contractility (inotropy) of heart, Knowledge that an inotropic heart drug affects the force with which the heart muscle
administering which of the following medications? whereas propranolol (a beta blocker) and verapamil (a calcium channel blocker) are negative contracts will lead to the correct answer. The medication that fits this definition is option 2.
inotropic medications. Atropine has a neutral effect on contractility. The other medications do not fit with the definition of an inotropic heart drug, and can
therefore be eliminated.
1.‐ Atropine sulfate
2.‐ Digoxin (Lanoxin)
3.‐ Propranolol (Inderal)
4.‐ Verapamil (Calan)

3330 A client asks the nurse to explain why she is receiving Correct answer: 3 Lidocaine IV is used to treat ventricular dysrhythmias (premature ventricular contractions, The question is asking for the nurse to tell the client why she is receiving the medication.
lidocaine (Xylocaine) in her IV when her dentist injects ventricular tachycardia, and ventricular fibrillation), particularly in clients with a myocardial Option 3 is the only answer that would directly answer the client s question. Knowledge of
it into her gums to numb the teeth before a filling. The infarction. It is a class 1‐A antiarrhythmic. The drug causes no anesthetic effect unless the the classification and purpose of the medication will also assist in selecting the correct
best response by the nurse is: client receives an overdose that is evidenced by a central nervous system deficit (paresthesias, answer.
confusion, and slurred speech). Lidocaine can cause drowsiness, but is not used for relaxation
(option 2). It is not a diuretic (option 4).

1.‐ "The medication is more effective intravenously, and you will feel no pain anywhere."
2.‐ "As an IV medication, it will help you relax and rest, so your heart can heal."
3.‐ "When given IV, this medication reduces the irritability of the cells in your heart, and helps reduce dysrhythmias."
4.‐ "The medication will increase your urine output, so that you will be less likely to suffer from heart failure."

3331 The nurse provides discharge instructions to a client Correct answer: 2 The side effects of amiodarone take several weeks or longer to manifest themselves. It is important to understand the onset and duration of the medication in order to teach
about the use of amiodarone (Cordarone). Which of Sometimes they persist for up to four months, and because photosensitivity is a continuing the client side effects and responses to the medication. Option 2 is the only correct option,
the following statements by the client indicates an concern, the client should avoid tanning. The pulse should be monitored, and if it remains and can be selected when the duration of the medication is understood. This knowledge
understanding of discharge teaching about this above 100, the physician should be notified. If a dose is missed, the client should call the will allow elimination of option 1. Recall client responses relative to tachycardia with the
medication? physician before taking any more medication. medication. If a client misses a dose, it is important that he follow up up with his primary
practitioner, and not just take a second dose, thereby eliminating option 4.

1.‐ "As soon as the physician says I can stop taking this medication, I will be able to enjoy the sun again."
2.‐ "The side effects of this medication may not begin to show up for several weeks or even months after I start taking it."
3.‐ "If my pulse drops below 100 beats per minute, I should call the physician right away."
4.‐ "If I miss a dose of medication, I should take it as soon as I remember it."

3332 A client is taking lovastatin (Mevacor) for treatment Correct answer: 4 Mevacor belongs to a group of drugs classified as statins. They work by inhibiting cholesterol Knowledge of the medication will assist in answering this question, as will looking at the
of high cholesterol levels. In conducting medication synthesis in the liver. Bile‐acid resins and fibric acid derivatives also work to decrease ending of the medication. Knowing that a statin works by inhibiting cholesterol synthesis
teaching, the nurse explains that this medication works cholesterol levels, but they work at different sites (options 1 and 3). Bile‐acid resins work in the will lead you to the correct answer. Use this knowledge to eliminate the other options.
because it: gastrointestinal tract, and bind bile salts in the intestine. Fibric acid derivatives work on
lipoproteins and triglycerides to reduce cholesterol. Mevacor is not a hormone (option 2).

1.‐ Is a bile‐acid resin.


2.‐ Is a hormone.
3.‐ Consists of a fibric acid derivative.
4.‐ Inhibits cholesterol synthesis.

3333 A client diagnosed with pernicious anemia is told that Correct answer: 3 It is important for the nurse to understand that both intrinsic and extrinsic factors are Knowledge of medication administration facts will lead to the correct answer. Since taking
vitamin B&lt;sub&gt;12&lt;/sub&gt; injections are necessary for the absorption of vitamin B&lt;sub&gt;12&lt;/sub&gt;. If the client lacks intrinsic the medication is a lifelong process, option 2 can be eliminated. Option 4 can be eliminated,
required. The client doesn't like "shots," and wants to factor (has pernicious anemia), the medication must be administered via the IM or deep SC as it does not answer the client s question. Option 1 is incorrect, which will lead to the only
know why the medication can't be taken orally. How route. The PO route of administration will not solve the problem if intrinsic factor is absent correct answer, option 3, which does address the client s question.
would the nurse respond? (option 1). Lifelong administration of vitamin B&lt;sub&gt;12&lt;/sub&gt; is required for clients
with this type of therapy (option 2). Injection therapy starts out on a more frequent dosing
schedule. Therefore, it is incorrect to tell the client that injections will be given only once a
month (option 4). In addition, this response minimizes the client's concerns, and is therefore
not therapeutic.

1.‐ Initially, the medication must be given by injection, but it then can be switched over to the oral route once serum levels are maintained.
2.‐ The medication is only available as an injection, but the length of therapy is only a few months in duration.
3.‐ The medication cannot be given orally because this type of anemia causes a lack of intrinsic factor that is necessary for the absorption of vitamin B12.
4.‐ The medication can be given once a month by injection, and therefore shouldn't be too uncomfortable.
3334 A client is receiving intravenous (IV) thrombolytic Correct answer: 2 When administering IV thrombolytic medication, the IV site should be changed to the Options 1, 3, and 4 do not address the safety issue to which the question refers. The only
therapy, and complains of pain and redness at the opposite extremity if the client complains of pain or redness at the infusion site. These safe option is to restart the medication, since the IV site is red, which can indicate
insertion site. Which nursing actions are most symptoms can indicate thrombophlebitis, and should be taken seriously and acted on thrombophlebitis. Reassessing and repositioning the site will not safely care for the pain the
appropriate? immediately. Continuing to monitor the IV site without appropriate intervention is not client is experiencing.
considered safe practice (options 1 and 4). Decreasing the rate of solution might be an option
based on documented protocol, but the fact that the client complains of pain and redness
takes immediate priority (option 3).
1.‐ Continue to monitor the IV site q 15 minutes, and check for infiltration or phlebitis.
2.‐ Restart infusion in the opposite extremity. Further dilute medication per protocol to prevent thrombophlebitis.
3.‐ Decrease the rate of infusion, monitor IV site q 15 minutes, and continue to assess the client for complaints of pain.
4.‐ Reposition the client's extremity, and continue to monitor the IV site.

3335 The nurse would monitor the results of which of the Correct answer: 1 The reticulocyte count is an indication of the number of immature RBCs found circulating in Having knowledge of the purpose of the lab studies will assist in selecting the correct
following laboratory tests to determine whether a the body. An increased reticulocyte count indicates that the bone marrow is functioning, and answer. If this was difficult, review the purposes of the listed lab studies.
client is responding to ferrous sulfate therapy? that RBC production has been stimulated. INR, PT, and APTT (options 2, 3, and 4) all refer to
coagulation studies that are useful in managing anticoagulation therapy or clients who have
coagulation disorders.
1.‐ Reticulocyte count
2.‐ International normalized ratio (INR)
3.‐ Prothrombin time (PT)
4.‐ Activated partial thromboplastin time (APTT)

3336 A client is taking folic acid (Folvite) for treatment of Correct answer: 4 Oral contraceptives taken concurrently with folic acid will diminish the effectiveness of the Knowledge of the contraindications of folic acid with other medications will assist in
folic acid anemia. Which one of the following would folic acid. The nurse should be alert to potential drug interactions, educate the client, and selecting the correct answer. If this was difficult, review the contraindications of other
the nurse recognize as causing a potential drug notify the physician of potential interactions. Vitamin E, tetracycline, and allopurinol (options medications with folic acid.
interaction for this type of therapy? 1, 2, and 3) all affect the administration of iron.

1.‐ Vitamin E
2.‐ Tetracycline
3.‐ Allopurinol
4.‐ Oral contraceptives

3337 A client being treated with heparin therapy is deemed Correct answer: 3 Because heparin is metabolized quickly in the body, it is important to know both the amount The question is a safe, effective care, and the nursing process must be used in answering
to have an overdose. The nurse takes which of the of drug that was given and the elapsed time frame. The dose of protamine sulfate will be this question. Since the first step in the nursing process is assessment, select the option
following priority actions? calculated based on individual need. While it is true that the maximum dose of protamine is 50 that is an assessment answer. Recognize that options 1, 2, and 4 are all interventions, and
mg over a 10‐minute period, that might not be the dosage required, since the pertinent could be considered after the initial assessment.
information relative to heparin is not stated (option 1). APTT levels would be monitored, but
are not the priority action at this time (option 2). Vitamin K should not be given with
protamine sulfate, as it is the antagonist to Coumadin (option 4).

1.‐ Administer 50 mg of protamine sulfate over a 10‐minute period.


2.‐ Monitor the client's APTT levels prior to and following administration of protamine.
3.‐ Determine the amount of heparin that was given and the time frame that has elapsed since its administration.
4.‐ Administer protamine sulfate concurrently with vitamin K to increase its therapeutic effect.
3338 A surgical client has suddenly begun to bleed Correct answer: 2 Hemostatics such as aminocaproic acid are used to control excessive bleeding. They can be Understanding of the classification of the listed medications is needed to answer this
excessively while in the operating room. The nurse applied topically to stop a local hemorrhage, or they can be administered parenterally to stop question. If this was difficult, review the medications, and be familiar with medication
anticipates that an order will be given for which of the a systemic hemorrhage. Thrombolytics are used to dissolve existing clots (option 1). classifications.
following medications? Antiplatelet agents are used to prevent platelet aggregation, and anticoagulants act on the
coagulation cascade to prevent clot formation (options 3 and 4). It is important for the nurse
to have a basic understanding of each of the drug groupings so that appropriate therapy can be
properly monitored.

1.‐ Alteplase (Activase)


2.‐ Aminocaproic acid (Amicar)
3.‐ Dipyridamole (Persantine)
4.‐ Heparin (Liquaemin)

3339 A client who has been diagnosed with anemia reports Correct answer: 4 If the client still complains of symptoms of anemia, it is possible that treatment with folic acid Note that the question stem asks for further assessment. Options 1, 2, and 3 are all
that symptoms have not improved since treatment might not be the primary problem. Folic acid and vitamin B&lt;sub&gt;12&lt;/sub&gt; work considered interventions, and can therefore be eliminated. The only assessment answer is
with folic acid was started. What additional together to aid the growth of RBCs. Obtaining and comparing APTT results will not option 4, which would be to refer to a physician for further assessment to determine if
assessments would the nurse want to make to demonstrate whether the drug treatment is effective, as this test looks at the intrinsic there are other conditions causing the problem.
determine the effectiveness of drug therapy? coagulation pathway. While it is true that alcohol has a major effect on folic acid levels and
intake, there are many other medications that can affect folic acid levels. This response, while
of concern, does not indicate that the drug therapy is effective. A review of dietary sources
might be indicated, but if the client were taking drug therapy, then the amount of folic acid in
the diet would not indicate whether the drug therapy was effective.

1.‐ Obtain current APTT, and compare with the client's previous APTT to see if the treatment is effective.
2.‐ Review dietary sources of folic acid with the client to verify if the treatment is effective.
3.‐ Verify that the client is not taking alcohol to see if the treatment is effective.
4.‐ Refer the client to the physician for possible deficiencies of other B vitamins, as folic acid and B12 deficiencies often occur together.

3340 A client taking long‐term warfarin (Coumadin) Correct answer: 3 It is very important for the client to be aware of foods that are high in vitamin K while on It is important to know that eating foods high in vitamin K is contraindicated with clients
therapy following cardiac valve replacement surgery Coumadin therapy. Green, leafy vegetables are very high in vitamin K, and if the client is eating who are taking coumadin. Recall knowledge of foods high in vitamin K. If this was difficult,
comes in to the office for a follow‐up visit. Upon a large amount of these during the week, this might affect the action of Coumadin. While milk review the client teaching needs for a client taking coumadin.
conducting a diet survey, which one of the following is a good source of vitamin K, the amount that the client is taking is not enough to cause
client's dietary patterns would be of concern to the concern with regard to Coumadin interaction (option 1). The amount of wine and wax beans is
nurse? not clinically significant to affect Coumadin interaction (options 2 and 4).

1.‐ The client is drinking a glass of milk at bedtime once a week.


2.‐ The client is eating yellow wax beans as a vegetable twice a week.
3.‐ The client has a salad for lunch on weekdays.
4.‐ The client has a drink of wine once a week with an evening meal.

3341 A client is receiving heparin therapy, and the Correct answer: 4 The APTT level should be 1.5–2.5 times the control value to reach a therapeutic range. A Knowledge of the normal lab results will assist in selecting the correct answer. If this was
activated partial thromboplastin time (APTT) is being control value is always run with the test to make sure that the results are referenced. Higher difficult, review normal lab results of the APTT test.
monitored. For what level does the nurse look as a APTT levels are not considered to be therapeutic, and might require that the medication be
therapeutic level? stopped until APTT levels fall back into a safe and therapeutic range.

1.‐ APTT level of 3 times the control value


2.‐ APTT level consistent with the control value
3.‐ APTT level of 3.5 times the control value
4.‐ APTT level of 1.5–2.5 times the control value
3342 The nurse is caring for a client with chronic angina Correct answer: 4 Sotalol is a beta‐adrenergic blocking agent. Side effects include bradycardia, difficulty Note that the question stem asks for selection of a side effect of the medication. While all
pectoris. The client is receiving sotalol (Betapace) 80 breathing, wheezing, bronchospasm, GI disturbances, anxiety, nervousness, weakness, mood of the listed options would be considered side effects, the only side effect listed for this
mg PO daily. Which client manifestation would the changes, depression, and loss of libido. The symptoms listed in options 1, 2, and 3 do not medication is option 4. If this was difficult, review the side effects of the medication.
nurse conclude is a side effect of this medication? occur.

1.‐ Difficulty swallowing


2.‐ Diaphoresis
3.‐ Dry mouth
4.‐ Bradycardia

3343 The nurse is caring for a client with a history of mild Correct answer: 3 Calcium channel blocker agents such as diltiazem (Cardizem) are used cautiously in clients Knowing that the medication is a calcium channel blocker should assist in selecting the
congestive heart failure (CHF) who is receiving with aortic stenosis, bradycardia, CHF, acute myocardial infarction, and hypotension. The nurse correct answer. Knowing that the client is being treated for CHF also should also assist in
diltiazem (Cardizem) for hypertension. The nurse would assess for signs that indicate worsening of these underlying conditions. Bradycardia and selecting the answer. Realize that tachycardia is not a side effect to assist in eliminating
would assess the client for which of the following? peripheral edema signal adverse effects of this class of medication, and require follow‐up if options 1 and 4. Two of the side effects of the medication are edema and bradycardia.
they occur.
1.‐ Tachycardia and rebound hypertension
2.‐ Wheezing and shortness of breath
3.‐ Bradycardia and peripheral edema
4.‐ Chest pain and tachycardia

3344 The nurse has just administered a dose of hydralazine Correct answer: 3 Hydralazine (Apresoline) is a powerful vasodilator that exerts its action on the smooth muscle Knowing the classification of the medication as a drug to reduce blood pressure will assist
(Apresoline) intravenously to a client. After the initial walls of arterioles. After a parenteral dose, blood pressure is checked every 15 minutes until in selecting the correct answer. Since the medication has an effect on the blood pressure,
dose, which of the following measurements is the stable, and then every hour. Although options 1, 2, and 4 are components of assessment, they the correct answer is option 3.
priority assessment? are not directly related to the action of the medication.

1.‐ Cardiac rhythm


2.‐ Oxygen saturation
3.‐ Blood pressure
4.‐ Respiratory rate

3345 The nurse has begun a continuous infusion of Correct answer: 4 Nitroglycerin is an antianginal of the nitrate type that causes vasodilation of coronary and Use the process of elimination, and realize that options 1 and 2 are expected outcomes of
nitroglycerin (Nitrostat) intravenously. Which of the other arteries. It would be expected to cause a decrease in PCWP and CVP. The heart rate the medication, to assist in selecting the correct answer. A decrease in heart rate (option 3)
following indicates to the nurse that the client is could also decrease with overall improvement in cardiac output. A decrease in BP from 130/80 is expected, but the drop in BP is excessive. This would lead to the correct option, 4.
experiencing an adverse reaction? to 90/64 is excessive, and warrants further assessment by the nurse to determine whether
perfusion to major organs is adequate.
1.‐ Pulmonary capillary wedge pressure (PCWP) falling from 13 to 11 mm Hg
2.‐ Central venous pressure (CVP) falling from 10 to 7 mm Hg
3.‐ Heart rate (HR) falling from 96 to 78
4.‐ Blood pressure (BP) falling from 130/80 to 90/64

3346 The nurse is caring for a client with chronic stable Correct answer: 2 Amlodipine (Norvasc) is a calcium channel blocker. Adverse or toxic reactions from over Use the process of elimination and recall understanding of the purpose of the medication
angina receiving amlodipine (Norvasc). In developing a dosage could produce excessive peripheral vasodilation, and marked hypotension with reflex to be assisted in selecting the correct answer. Since hypotension (option 2) is a side effect
medication teaching plan, the nurse should include tachycardia. Frequent side effects include peripheral edema, headache, and flushing. Some of the calcium channel blocker, option 1 can be eliminated. The other options are not side
which of the following findings as an adverse reaction sustained‐release forms of calcium channel blockers (such as Calan SR) could lead to effects of the medication. If this was difficult, review general characteristics of calcium
to this medication? constipation, a milder side effect than the others. channel blockers.

1.‐ Hypertension
2.‐ Hypotension
3.‐ Constipation
4.‐ Diarrhea
3347 A female client tells the nurse, "Since I have been Correct answer: 2 Atenolol is a beta‐adrenergic blocker that causes a decreased heart rate, blood pressure, and The question is asking for selection of a physiologic nursing diagnosis. Knowing this can
taking that medicine, I feel so tired." She is being cardiac output. Fatigue is the most common side effect. If fatigue becomes severe enough, it eliminate options 3 and 4. Option 1 can also be eliminated, since there is nothing in the
treated with atenolol (Tenormin) for hypertension. The could interfere with the client's activity level. Activity intolerance is the state in which an question referring to a risk for insufficient energy to carry out activities, which is the
client's statement is reflective of which nursing individual has insufficient energy to complete activities of daily living. There is no evidence that definition of activity intolerance. The only correct option is 2.
diagnosis? the client has ineffective cerebral perfusion, ineffective health maintenance, or self‐care
deficit.
1.‐ Risk for Activity Intolerance
2.‐ Cardiac Output, Decreased
3.‐ Health Maintenance, Altered
4.‐ Self‐care Deficit

3348 The nurse is scheduled to administer a dose of digoxin Correct answer: 4 The normal reference range for potassium for an adult is 3.5–5.1 mEq/L. Hypokalemia can Knowing the therapeutic level for digoxin will lead to the correct answer. Use this
(Lanoxin) to an adult client with atrial fibrillation. The make the client more susceptible to digitalis toxicity. The nurse monitors the results of knowledge and the process of elimination to lead you to the only correct answer, 4.
client has a potassium level of 4.6 mEq/L. The nurse electrolytes for the potassium level. If the potassium level is low, the dose is withheld, and the
interprets that the: physician is notified. This client's result is in the normal range, so the dose should be
administered.
1.‐ Dose should be omitted only for that day.
2.‐ Client needs a dose of potassium before receiving the digoxin.
3.‐ Dose should be withheld, and the physician notified.
4.‐ Dose should be administered as ordered.

3349 The nurse is planning to care for a client with Correct answer: 3 Lasix depletes potassium stores, and a client taking digoxin and furosemide needs to maintain Recall knowledge that potassium is depleted with clients who are taking Lasix to assist in
congestive heart failure being treated with digoxin normal potassium levels and moderate salt intake. Hypokalemia makes the client more selecting the correct option. Option 3 food choices are the highest in potassium, and are
(Lanoxin) and furosemide (Lasix). Which is the susceptible to digitalis toxicity. Option 3 is the best choice because all three foods are high in therefore the best option. If this was difficult, review foods high in potassium.
following dinners would be the best choice from the potassium and low in sodium.
daily menu?
1.‐ Beef vegetable soup, macaroni and cheese, and a dinner roll
2.‐ Beef ravioli, spinach soufflé, and Italian bread
3.‐ Baked whitefish, mashed potatoes, and carrot‐raisin salad
4.‐ Roasted chicken breast, brown rice, and stewed tomatoes

3350 The physician has prescribed propranolol (Inderal) for Correct answer: 2 The medication has side effects that could be disturbing to the client. These include When a nurse is teaching a client about side effects of medications, it is important that the
a client with frequent premature ventricular hypotension, insomnia, lethargy, bronchospasm, mood changes, and decreased libido. The teaching is at a level that the client will understand. The most important part of the
contractions (PVCs). The nurse collects material to client should be alert to these so that he can notify the physician or another health care teaching process with medications is to make sure the client understands the side effects
conduct an education session with the client. Which of provider. It is not the nurse's role to describe alternatives to the currently ordered medication and adverse reactions of the medication.
the following should the nurse plan to include in the (option 1). It is unnecessary to teach about effects at the cellular level (option 3) unless the
teaching session? client has interest in this. It is also unnecessary to teach the client about various dysrhythmias
(option 4), because this is not pertinent.

1.‐ A description of other effective medications


2.‐ Information about side effects and adverse reactions
3.‐ Material about the cellular effect of the medication
4.‐ Data regarding various dysrhythmias

3351 Chemical cardioversion is prescribed for a client in Correct answer: 1 Quinidine (Quinidex) is a Class I‐A antiarrhythmic that is very effective as a chemical Use the process of elimination to select the correct answer. Options 2, 3, and 4 can all be
atrial fibrillation. The nurse prepares which of the cardioversion agent. Verapamil is a calcium channel blocker generally used to control heart eliminated, as they are not medications used in chemical cardioversion. The medication
following medications specifically for chemical rate. Nifedipine is a calcium channel blocker used as a vasodilator. Bretylium is generally used used in chemical cardioversion is an antiarrhythmic, and this will lead to option 1.
cardioversion? for control of ventricular arrhythmias.
1.‐ Quinidine (Quinidex)
2.‐ Verapamil (Calan)
3.‐ Nifedipine (Procardia)
4.‐ Bretylium (Bretylol)
3352 A client taking cholestyramine (Questran) to lower Correct answer: 3 Clients who are taking cholestyramine (which is a bile resin) should be monitored for fat‐ Using the process of elimination and knowledge that a deficiency of fat‐soluble vitamins
lipid levels should be monitored for possible deficiency soluble vitamin deficiencies (vitamins A, D, E, and K), as the gastrointestinal side effects of the occurs with medications that lower lipid levels to select the correct answer. Options 1, 2,
of which vitamin(s)? medication can lead to reduced absorption. Niacin, folic acid, and vitamin and 4 are all water‐soluble, and can therefore be eliminated.
B&lt;sub&gt;12&lt;/sub&gt; (options 1, 2, and 4) are all examples of B complex vitamins that
are water‐soluble.
1.‐ Niacin
2.‐ Folic acid
3.‐ Vitamins A and D
4.‐ Vitamin B12

3353 A client who is taking warfarin (Coumadin) therapy Correct answer: 3 Clients who are taking Coumadin should be alerted to the potential for drug interactions Part of the nursing assessment is determining what medication the client is current taking.
comes to the office for a follow‐up visit, and states when they are on long‐term anticoagulation therapy. Aspirin can potentiate the effect of Once determined that the client is taking Coumadin and Darvon together, the nurse should
that he has taken propoxyphene with aspirin (Darvon Coumadin, and interfere with the ability to maintain a therapeutic level. The use of Darvon, be alerted to the fact that the client needs further teaching regarding the risk of decreasing
Compound 65) for aches and pains related to an old although previously prescribed, is not in the best interest of the client at this time due to the effectiveness of coumadin when taking aspirin. The best answer is option 3, since once
back injury. How should the ambulatory care nurse Coumadin therapy. Telling the client to keep taking Darvon would lead to drug interactions the nurse assesses for medication usage, he can continue with the nursing process and
respond to this information? (option 2). While a further assessment of the client's back pain might be necessary (option 1), develop a plan of care.
it is not the primary action that the nurse should be addressing at this time. Option 4 is a false
statement, because the two drugs together could enhance bleeding.

1.‐ Ask the client how long his back been hurting him, and assess the need for a referral for pain management.
2.‐ Tell the client that it is important to prevent the pain cycle from starting, and that the client can continue to take Darvon as ordered by the physician.
3.‐ Advise the client that Darvon contains aspirin, which can interfere with Coumadin therapy. Consult the physician for an alternate form of pain‐relief therapy.
4.‐ Instruct the client that continued use of Darvon on a daily basis would help to relieve back pain, but would require an increase in the dose of Coumadin.

3354 A postoperative client who has an order for 5,000 Correct answer: 4 Low‐dose heparin therapy is indicated in many postoperative clients to prevent the Using the process of elimination will lead to the correct answer. Option 1 should be
units of heparin SC for three doses wants to know why development of thromboembolic episodes. It is not used in every postoperative situation eliminated because of the word “all.” This is a false statement. Option 2 should also be
this drug is being ordered. What information would (option 1), but it is usually used for clients who have orthopedic surgery, or are anticipated to eliminated because of the word “essential,” which is also a false statement. Option 3 should
the nurse provide to the client to answer the question? be immobilized for a time following surgery. Short‐term therapy is not given to maintain be eliminated because it does not address what the client is asking.
adequate blood clotting levels (option 2), but merely to intervene as a preventative measure.
The statement that heparin is given SC into the abdomen and is not usually painful is factual,
but is not the reason the medication is given to the client (option 3).

1.‐ Heparin is used as a common medication in all postoperative situations.


2.‐ It is essential to have heparin during the postoperative period in order to maintain adequate blood clotting levels.
3.‐ The injections will be given in the abdomen, and are not usually associated with discomfort.
4.‐ Heparin is being used postoperatively to prevent blood clots from forming as a result of surgery or decreased mobility.

3355 A client who is receiving heparin protocol has an Correct answer: 3 The effectiveness of a heparin protocol is monitored by trending APTT results to achieve a Having knowledge of the normal lab results will assist in selecting the correct answer. If
activated partial thromboplastin time (APTT) level of therapeutic level. An APTT of 140 is above the therapeutic level of anticoagulation, and this was difficult, review normal lab results of the APTT test.
140 seconds (control time is 36 seconds). What is the therefore the infusion should be stopped per protocol, and resumed at a decreased dose in 1
priority action that the nurse should institute? hour's time, with a repeat APTT ordered in 2‐3 hours per protocol. The dose should not be
increased, as this would cause serious consequences to the client. Stopping the medication for
a total 6 hours would undermine the anticoagulation control that the physician is trying to
achieve. Ordering another APTT and continuing to run the infusion could also cause serious
consequences to the client.

1.‐ Increase the heparin dose, as the APTT level is not therapeutic. Obtain a repeat APTT in 6 hours.
2.‐ Stop the heparin therapy for 6 hours. Then restart the therapy at the same unit dose, and obtain a repeat APTT in 6 hours.
3.‐ Stop the heparin therapy for 1 hour. Decrease the
rate of infusion per protocol, and restart the
medication in one hour. Obtain a repeat APTT 2–3
hours from the restart of the infusion.
4.‐ Obtain an additional APTT in 1 hour, and continue to monitor the client.
3356 A client receiving warfarin (Coumadin) therapy has a Correct answer: 2 With an INR level that high, and no incidence of bleeding, administration of vitamin K and Having knowledge of the normal lab results will assist in selecting the correct answer. If
very high INR (above 6), and shows no signs of withholding of Coumadin doses are usually indicated. Protamine sulfate is the antagonist to this was difficult, review normal lab results of the INR test.
bleeding. What actions would the nurse employ to heparin administration, and is not indicated in this situation. Merely withholding the Coumadin
protect the client from having a bleeding episode? dose is not appropriate, since the INR level is too high. The amount of vitamin K in the diet is
not an immediate concern at this point, as the INR level is too high.

1.‐ Give the client protamine sulfate as ordered to normalize INR levels.
2.‐ Give the client vitamin K as ordered to normalize INR levels, and withhold the next Coumadin dose.
3.‐ Hold the next two Coumadin doses, and obtain an INR level the next day.
4.‐ Increase the amount of vitamin K in the diet, and continue to assess the client for potential bleeding.

3357 A client has been diagnosed with folic acid anemia. Correct answer: 2 Folic acid helps to reduce hemocysteine levels, and is therefore important for cardiovascular Knowledge of the aspects of folic acid deficiency will assist with selecting the correct
Which of the following additional assessments should health. Decreased levels of folic acid and other B vitamins (B&lt;sub&gt;12&lt;/sub&gt; and answer. If this was difficult, review the effects of folic acid deficiency on the cardiovascular
the nurse make after learning of this diagnosis? B&lt;sub&gt;6&lt;/sub&gt;) can lead to increased levels of homocysteine, which are associated system.
with increased risk of cardiovascular disease (option 1). Method of cooking/preparation can
affect the bioavailability of vitamins, but is not usually the main cause of folic acid anemia
(option 3). Folic acid anemia is not related to the development of diabetes (option 4).

1.‐ Other B‐vitamin deficiencies, and therefore increased homocysteine levels that are cardioprotective
2.‐ Cardiovascular health, because folic acid helps to decrease homocysteine levels
3.‐ Adequacy of folic acid sources in the diet, and method of cooking/preparation, as this might have caused the clinical deficiency
4.‐ The concurrent diagnosis of diabetes mellitus

3358 A client has been prescribed lovastatin (Mevacor) to Correct answer: 4 Often, clients who have multiple risk factors for developing hyperlipidemia must use a Understanding the treatment protocol in reducing cholesterol levels will lead to the
treat hyperlipidemia, since diet therapy and weight combination therapy of drugs and diet to achieve results. Diet management, weight control, correct answer. Option 1, 2 and 3 can be eliminated because they are false statements. The
reduction have not been successful in lowering and the use of drug therapy work together in supporting and maintaining lipid levels. There is only option that is addressing the client’s question is option 4.
cholesterol levels. The client does not understand why also a genetic component to hyperlipidemia that needs to be addressed by the client. Clients
medication is necessary, because the client has only who are compliant with diet therapy often have to use drug therapy because of this genetic
been watching what he eats for three months. How predisposition to produce more cholesterol. Drug therapy will not by itself eliminate the need
would the nurse respond to questions about the use of for watching fat intake. Effective drug therapy is not only seen with documented weight loss.
drug therapy as it relates to dietary management? The chemical actions of the drug depend on compliance to the medication schedule. Lovastatin
(Mevacor) does not reduce triglyceride levels. Different antilipidemia agents work on various
lipoproteins and triglycerides.

1.‐ Drug therapy should eliminate the need for watching the diet, and therefore the addition of the medication would be beneficial.
2.‐ Drug therapy is only effective when there is documented weight loss.
3.‐ Drug therapy will reduce both cholesterol and triglyceride levels, if taken properly.
4.‐ Combination therapy (drugs and diet) often helps clients meet their treatment goals to reduce cholesterol levels.

3359 The nurse should review the results of which of the Correct answer: 2 When using a lipid‐lowering agent, the standard of care is that liver enzymes be routinely Knowing the purpose of the laboratory tests will lead to the correct answer. Lipid‐lowering
following to evaluate a client's response to a lipid‐ evaluated both before and during the course of therapy. These medications can cause agents can have an effect on liver function tests, which would lead you to option 2.
lowering agent? abnormal results in liver function tests that can lead to serious consequences if the client is not
properly monitored. CBC, clotting studies, and Doppler studies are not indicated for the
management of this drug therapy.
1.‐ CBC with differential
2.‐ Liver function tests
3.‐ Clotting studies
4.‐ Doppler studies
3360 A client is admitted for deep vein thrombosis (DVT) of Correct answer: 3 Heparin is the drug of choice for clients who have presenting symptoms of DVT, due to its Note that the client is being seen for the first time for a DVT. This should lead to selection
the lower left extremity. Which one of the following rapid onset of action. Lovenox is an example of a low‐molecular‐weight heparin (LMWH), low‐ of the first medication of choice in resolving a DVT, option 3: heparin. The other options are
medications should the nurse expect to be used to dose therapy that is used as a prophylactic measure to prevent thromboembolism (option 2). not used as the initial medication, and should be eliminated.
treat this condition? Coumadin is an oral anticoagulant that has a slower onset of action, and is therefore not
appropriate as first‐line therapy for a client with a DVT (option 1). Persantine is an antiplatelet
medication that works on decreasing platelet aggregation. It works to prevent arterial
thrombosis (option 4).
1.‐ Warfarin (Coumadin)
2.‐ Enoxaparin (Lovenox)
3.‐ Heparin (Liquaemin)
4.‐ Dipyridamole (Persantine)

3361 Which of the following statements indicates to the Correct answer: 3 With initial therapy, it is recommended that iron preparations be taken with meals to prevent The question is asking for selection of the answer that indicates the client understands
nurse that a client understands the administration of GI upset. As therapy is tolerated, the medication can be given between meals to maximize how to take the medication initially. In reviewing the choices, the one answer that
iron therapy? absorption. Antacids and antibiotics when taken together, with iron, decrease its absorption addresses initial medication consumption is option 3.
(options 1 and 4). Oral iron salts differ in their amount of elemental iron; therefore, iron
preparations should not be used interchangeably (option 2).

1.‐ "I will take an antacid when I take my iron pill, to decrease the GI effects of the drug."
2.‐ "It makes no difference which iron pill I take because all iron is alike."
3.‐ "I will take my iron medication initially with meals, to prevent GI upset."
4.‐ "I will take my iron medication when I take my antibiotic."

3362 In assessing a hospitalized client 1 hour after Correct answer: 2 Apresoline is a vasodilator, and if the client becomes dehydrated, hypotension will result. In Knowledge of side effects and nursing interventions when taking Apresoline will lead to
receiving hydralazine (Apresoline) 20 mg PO, the nurse other words, during dehydration, both preload and afterload are reduced, causing the "tank" the correct answer. Option 2 is a client response that relates to the side effects. If this was
notes that the BP is 68/42. The client has been taking to get larger with less volume. The normal dose of hydralazine is 5–25 mg PO. Serum difficult, review the medication Apresoline, the side effects of the medication, and client
this medication for several years at home without potassium is high, but unrelated to Apresoline. The increased heart rate is a reflexive response responses related to interventions.
difficulty. Which of the following factors most likely to the low cardiac output, to compensate with decreased preload and afterload.
contributed to this episode of hypotension?

1.‐ The dose is excessive for this medication.


2.‐ Total intake for the previous 24 hours is 1,000 mL.
3.‐ Serum potassium is 5.8 mEq/L.
4.‐ Heart rate is 145 beats per minute.

3363 The client is receiving a loading dose of lidocaine Correct answer: 1 Lidocaine is given via IV push in doses of 1–1.5 mg/kg. The initial loading dose (bolus) is Note that the question stem is asking for selection of the next nursing intervention after
(Xylocaine) 100 mg IV for treatment of ventricular intended to achieve adequate blood levels to suppress ventricular dysrhythmias, and is the initial bolus. The next intervention is to begin an IV drip, so the correct option is 1. If
tachycardia. The nurse prepares to take which action followed by an infusion of 1–4 mg/min via infusion pump. The initial bolus lasts approximately this was difficult, review the nursing interventions for the medication.
next? 10 minutes, so the infusion must not be delayed. The dose may be repeated one time under
certain conditions, but the total dose should not exceed 3mg/kg. Oral therapy and pacemaker
insertion are not indicated at this time.

1.‐ Start a continuous IV infusion at 1–4 mg/minute.


2.‐ Repeat the dose every 10 minutes for one hour or PRN.
3.‐ Begin oral procainamide (Pronestyl) therapy.
4.‐ Prepare for pacemaker insertion to override the dysrhythmia.
3364 A client has been admitted to the hospital with chest Correct answer: 3 The standard protocol is to administer up to three doses of NTG five minutes apart as long as Note that the question asks for selection of the next action. Knowing that the dose of NTG
pain. The client's symptom has not been relieved after the vital signs remain stable. After three doses, the physician should be called if pain is can be repeated will lead to the correct option, 3, as this is the appropriate action.
one dose of nitroglycerine (NTG) sublingually. Upon unrelieved. An electrocardiogram (ECG) may be ordered, but not an EEG (to measure brain
monitoring the vital signs (VS), the nurse notices that waves). Using NTG paste, a longer‐acting form of the medication, is not appropriate at this
they are stable. Which of the following indicates the time.
appropriate action for nurse to take next?

1.‐ Notify the physician.


2.‐ Obtain an electroencephalogram (EEG).
3.‐ Give another dose of nitroglycerine.
4.‐ Add a dose of nitroglycerine paste.

3365 After three defibrillation attempts, the client Correct answer: 3 Lidocaine is the primary medication used to treat ventricular dysrhythmias. Lidocaine Knowledge of the purpose of lidocaine will assist in selecting the correct answer. Pay
continues to be in a pulseless ventricular tachycardia. suppresses automaticity in the HIS‐Purkinje system by elevating electrical stimulation particular attention to the question that is being asked. Option 3 is the only correct
A lidocaine bolus of 100 mg IV is administered. The threshold of the ventricle during diastole, thus decreasing ventricular irritability. Ventricular response to the administration of lidocaine during ventricular tachycardia.
nurse would expect to see which of the following as a fibrillation (option 1) is a worsening dysrhythmia. Slowing the heart rate (option 2) without
therapeutic response to lidocaine? converting the rhythm to an atrial or sinus rhythm is not therapeutic. An increase in level of
consciousness (option 4) would only occur once the ventricular rhythm is terminated.

1.‐ Conversion from a ventricular tachycardia to a ventricular fibrillation


2.‐ Slowing of heart rate to 80 beats per minute
3.‐ A reduction in ventricular irritability
4.‐ An increase in the level of consciousness

3366 A client is being discharged after treatment for left‐ Correct answer: 1 The client's diet should be high in potassium to avoid hypokalemia. Hydrochlorothiazide is a The question is asking for selection of a statement by the client that indicates that she
sided heart failure. The nurse is teaching the client potassium‐depleting diuretic. If the client develops hypokalemia, she would be at a greater risk does not understand interventions taught during a teaching session. Since one of the side
about the purpose, adverse effects, and dose of for developing digitalis toxicity. The client should measure the pulse before taking the cardiac effects of the medication is hypokalemia, the goal with the medications would be to
digoxin (Lanoxin) and hydrochlorothiazide glycoside (digoxin). For the best therapeutic effect, the medications should be taken at the prevent this. Clients are encouraged to eat foods high in potassium, leading to option 1 as
(HydroDIURIL). Which statement by the client indicates same time each day, preferably in the morning (so client will not have nocturia from the the answer that would indicate more teaching is required. If this was difficult, review the
to the nurse that further discharge teaching is needed? diuretic). A combined therapeutic effect increases the urinary output via the positive inotropic medication and teaching needs for clients.
effect of digoxin and the diuretic effect of hydrochlorothiazide.

1.‐ "I should decrease my intake of foods high in potassium, such as bananas."
2.‐ "I should take my radial pulse before taking this medication."
3.‐ "This medication can cause an increase in my urinary output."
4.‐ "This medication should be taken in the morning rather that in the evening."

3367 A client is being discharged after recovery from an Correct answer: 1 Because of the vasodilating effects of nitrates, headache is a common side effect. Medical The question is asking for selection of the option indicating that the client needs further
acute anterior myocardial infarction (MI) with attention is not necessary unless the headaches increase in frequency or severity. All three teaching. Knowing the classification of all the medications will allow elimination of options
recurrent angina. The nurse is teaching the client the medications are nitrates, and will increase coronary artery blood flow by dilating the coronary 2 and 3, as these are accurate. Option 4 is correct information as stated by the client.
following medications: diltiazem (Cardizem SR) 90 mg arteries and collateral blood vessels, which results in increasing blood flow to the heart. The Headaches are common, but do not require the client to contact the physician. Since this
PO twice daily; isosorbide dinitate (Isordil) 10 mg PO medications are used to prevent the frequency, intensity, and duration of anginal attacks. All statement indicates the client is unclear when to call the physician, further clarification
three times daily; and nitroglycerine (Nitrostat) 0.4 mg should be stored in a cool, dry place. would be required. If this was difficult, review the medications.
sublingually as needed. Which statement by the client
would indicate that further discharge teaching is
needed?

1.‐ "I should notify my health care provider if I experience headaches with any of these medications."
2.‐ "All three of these medications will increase blood flow to my heart."
3.‐ "All three of these medications will help to decrease the intensity of my chest pain.'"
4.‐ "I will store these medications in a cool, dry place."
3368 Adenosine (Adenocard) is to be administered to a Correct answer: 3 Adenosine (Adenocard) is an antidysrhythmic used in the treatment of paroxysmal Having knowledge of the purpose of the medication will aid in selecting the correct
client in the Emergency Department. Before the supraventricular tachycardia (SVT). Cardiac performance must be assessed before and answer. The question is asking for selection of priority equipment. Since the medication is
preparation of the medication, the nurse's priority throughout treatment by cardiac monitoring. An endotracheal tube may be used if an used for SVT, the correct priority choice should relate to cardiac, which will lead you to
should be to ensure that which of the following emergency arises necessitating mechanical ventilation, but the tube itself is a rather isolated option 3.
equipment is operational? item. An IV pump might be needed, but is not a priority because this medication is
administered rapidly by IV push. A pulse oximetry machine might be helpful in assessing
oxygenation, but is not a priority item.
1.‐ A pulse oximetry machine
2.‐ An IV infusion pump
3.‐ A cardiac monitor
4.‐ An endotracheal tube

3369 The nurse is conducting teaching with a client who Correct answer: 4 Prinzmetal's angina results from spasm of the coronary vessels. Calcium channel blockers are Knowing the pathophysiology and treatment protocol of variant angina will lead to the
recently was diagnosed with Prinzmetal's (variant) the medication prescribed for this condition. The risk factors are unknown, and Prinzmetal's correct answer. The only correct answer is option 4. If this was difficult, review the
angina. The nurse would evaluate the session as being angina is relatively unresponsive to nitrates. Beta blockers can worsen the spasm. Diet therapy pathophysiology and usual treatment protocol for the medication.
successful if the client stated that this form of angina: is not indicated.

1.‐ Has the same risk factors as stable and unstable angina.
2.‐ Responds readily to a low‐sodium diet.
3.‐ Is most effectively managed by beta blocker medications.
4.‐ Is treated with calcium channel blocking medications.

3370 The community health nurse visits a client at home. Correct answer: 1 Amiodarone (Cordarone) is a class III antiarrhythmic medication, and will probably not In order to select the correct answer, recall knowledge of the client responses to the
Amiodarone (Cordarone) has been prescribed for the demonstrate therapeutic effects for 1–3 weeks. This medication can cause fatigue, cough, and medication. It is also important to note in the question the need to pick the answer in
client. The nurse teaches the client about the pleuritic pain. The client must wear dark glasses and avoid exposure to the sunlight. The which the client needs further teaching. If this was difficult, review the medication and the
medication. Which of the following statements by the medication is given with food to avoid gastroenteritis distress. therapeutic effects of the medication.
client indicates that further teaching is necessary?

1.‐ "I will notify my doctor if I don't feel better in a couple of days."
2.‐ "I'll report any tiredness, coughing, or chest pain to my doctor."
3.‐ "I'll be careful to use dark glasses, and to avoid skin exposure to the sun."
4.‐ "I'll take this medication with food."

3371 The client is in ventricular tachycardia. The nurse is to Correct answer: 2 D&lt;sub&gt;5&lt;/sub&gt;W is compatible with lidocaine, and should remain available to The question is asking for selection of an action in administering the medication. The
give an IV bolus of lidocaine (Xylocaine) immediately. flush the medication into the client as soon as the IV push bolus is completed. The nurse might correct option is number 2, as this answer addresses the direct administration of the
Currently, an IV solution of 5% dextrose not have enough time to start another IV line. In addition, this is an emergency situation, and medication.
(D&lt;sub&gt;5&lt;/sub&gt;W) is infusing. What is the another IV site is not necessary for the IV lidocaine.
most appropriate action to take during administering
of the lidocaine?

1.‐ Stop the IV, flush the IV line, and then give the lidocaine IV push.
2.‐ Pinch the IV tubing above the injection port, and give the lidocaine directly into the IV line.
3.‐ Start another IV site.
4.‐ Check for incompatibility of lidocaine with other IV medications.

3372 A client is beginning medication therapy with Correct answer: 1 A client taking a diuretic such as furosemide should self‐administer the medication in the By understanding that Lasix is a diuretic, and that diuresis will occur, the earlier time is the
furosemide (Lasix) once daily. The nurse should morning to allow for diuresis throughout the day. This will help to prevent nocturia, which best option. The goal with Lasix is to prevent nocturia. By selecting an early time option,
instruct the client to take the medication at which of could cause disruption to the client's nightly sleep pattern. The time frame in option 2 is not as this can be prevented.
the following optimal times? early as in option 1, while options 3 and 4 clearly increase the risk of nocturia.

1.‐ 8:00 A.M.


2.‐ 12 noon
3.‐ 6:00 P.M.
4.‐ At bedtime

3373 The nurse is assessing the blood pressure (BP) of a Correct answer: 2 The National Institutes of Health (NIH) Committee has defined hypertension as a systolic The process of elimination and knowledge of the NIH guidelines will enable selection of
client diagnosed with primary hypertension. The nurse pressure of 140 or higher and diastolic of 90 or higher when two or more blood pressure the correct answer. Option 2 is the only correct option that correlates with the guidelines.
explains to the client that the basis for the diagnosis of measurements are averaged on 2 or more subsequent visits. Options 1, 3, and 4 are incorrect.
hypertension should be established by:

1.‐ Five readings one month apart.


2.‐ At least three readings with average blood pressure of 140/90.
3.‐ One reading of blood pressure greater than 140/90.
4.‐ Three blood pressure readings taken on the same day in different positions.

3374 The physician prescribes losartan (Cozaar) for a client Correct answer: 4 Losartan is an angiotensin II antagonist that inhibits the conversion of angiotensin I to Understanding of the purpose of the medication will enable selection the correct answer.
with hypertension. The nurse carrying out the order angiotensin II, resulting in vasodilation and normalizing blood pressure. The client should be If this was difficult, review the classification of the medication.
teaches the client that this medication promotes assessed for dizziness, cough, and diarrhea while taking this medication.
vasodilation by:
1.‐ Inhibiting calcium influx.
2.‐ Promoting catecholamines.
3.‐ Promoting release of aldosterone.
4.‐ Inhibiting conversion of angiotensin I to angiotensin II.

3375 After beginning an antihypertensive medication, the Correct answer: 2 Dry, persistent, tickling, nonproductive cough is a common side effect of angiotensin‐ Knowing the side effects of the medications will lead to the correct answer. If this was
client returns for a follow‐up visit and complains of a converting enzyme (ACE) inhibitors. difficult, review side effects of the medication.
dry, nonproductive cough. The nurse knows that this
side effect is mostly caused by which type of
antihypertensive medication?

1.‐ Beta blocker


2.‐ Angiotensin‐converting enzyme (ACE) inhibitor
3.‐ Calcium channel blocker
4.‐ Diuretic

3376 The nurse is teaching a newly diagnosed client with Correct answer: 4 Beta blockers inhibit cardiac beta 1 receptors, but also can affect beta 2 receptors in General knowledge of the pathophysiology of COPD and usual treatment should assist in
hypertension about her medications. The client has a bronchial and vascular smooth muscle, causing bronchoconstriction. Therefore, the client with selecting the correct option. If this was difficult, review COPD and the usual treatment
history of chronic obstructive pulmonary disease COPD should avoid taking beta blockers. Options 1, 2, and 3 are incorrect. regimen.
(COPD). The nurse informs the client that she should
avoid which antihypertensive medication?

1.‐ Angiotensin‐converting enzyme (ACE) inhibitors


2.‐ Calcium channel blockers
3.‐ Diuretics
4.‐ Beta blockers

3377 A 45‐year‐old female comes to the clinic with Correct answer: 2 Indipamide (Lozol) is a thiazide diuretic. Its hypertensive effect might be due to direct Note that the question states that the client is complaining of leg cramps. Notice that the
complaints of leg cramps. She has hypertension, and arteriolar vasodilation and decreased total peripheral resistance. Lozol can cause hypokalemia. client is taking a diuretic. After looking at these two characteristics, select option 2, as
has been taking indipamide (Lozol) 2.5 mg daily. Her Signs and symptoms of hypokalemia include muscle weakness and leg cramps. Electrolytes, diuretics can cause a decrease in potassium, leading to leading to leg cramps. The other
blood pressure is 126/70 upon arrival. After particularly potassium, need to be evaluated. The other options are irrelevant. options do not apply, and can be eliminated.
completion of assessment, the nurse plans to:

1.‐ Stop the indipamide (Lozol).


2.‐ Evaluate the electrolytes.
3.‐ Switch to furosemide (Lasix).
4.‐ Give her a nonsteroidal anti‐inflammatory drug (NSAID).

3378 A hypertensive male client presents to the clinic with Correct answer: 1 Thiazide diuretics decrease the effect of antigout medication by increasing hypersensitivity to Consider both the gout and the hypertension. If this was difficult, review the medications
complaints of a red, painful toe. In addition to treating allopurinol. Hyperuricemia is a side effect of thiazide diuretics. Option 3 is not the best answer, for contraindications.
the client for gout, the nurse concludes that this could because while clients might experience pain and a burning sensation in the lower extremities,
be due to: as well as signs of infection, with a diabetic neuropathy, there is no mention of diabetes
history in the above question.
1.‐ A thiazide diuretic.
2.‐ Obesity.
3.‐ Diabetes.
4.‐ Alcohol intake.

3379 The nurse practitioner has prescribed oxybutynin Correct answer: 4 Oxybutynin (Ditropan) is an antispasmodic medication used for urinary tract problems. It Recall the side effects of the medication. If this was difficult, review the side effects of the
(Ditropan) for a 65‐year‐old female with urinary produces anticholinergic side effects such as dry mouth, constipation, urinary hesitancy, and medication. Understand that the medication produces anticholinergic side effects, and
frequency and urgency. The nurse teaching the client decreased gastroenteritis motility. Periodic interruptions in therapy are recommended to select option 4.
about the side effects of this medication should assess continued need for this medication.
explain that which of the following manifestations
is/are associated with this medication?

1.‐ Dizziness
2.‐ Increased bruising
3.‐ Diarrhea
4.‐ Dry mouth and increased thirst

3380 A 25‐year‐old female presents to the office with Correct answer: 2 Urinalysis and urine dipstick should be performed to assess for the presence of blood cells Selection in this question needs to be based on the nursing process, of which assessment
complaints of burning, frequency, and urgency. The and bacteria in the urine. Infection should be established before instituting pharmacologic is the first step. The other three choices are interventions, and cannot be considered until
most appropriate intervention taken by the nurse is: therapy. Clients with urinary problems should be encouraged to increase fluid intake, but it is the assessment is completed.
not the most important intervention at this time.

1.‐ To increase fluid.


2.‐ Dipstick urine for leukocytes.
3.‐ To order phenazopyridine (Pyridium).
4.‐ To start antibiotic therapy.

3381 A 45‐year‐old man has been prescribed finasteride Correct answer: 2 Proscar is an androgen inhibitor used to treat benign prostatic hyperplasia (BPH). Pregnant Knowing the side effects of the medication will lead to the correct answer, option 2.
(Proscar) for his enlarged prostate. The nurse plans to women and women of childbearing age should not be exposed to semen fluid of a male taking
include which of the following points in the teaching finasteride (Proscar). Proscar is teratogenic, and can produce fetal abnormalities. Options 1, 3,
session? and 4 are incorrect.
1.‐ Abstain from sex.
2.‐ Use contraceptives.
3.‐ Increase fluid intake.
4.‐ Take for only three days.

3382 A client is prescribed verapamil (Isoptin) to manage Correct answer: 2 Verapamil is a calcium channel blocker that decreases blood pressure and heart rate. Option Recalling the side effects of the medication will lead to the correct answer. If this was
his hypertension. The nurse should instruct the client 1 is incomplete. Calcium channel blockers have no effect on urinary output (option 3). Option 4 difficult, review the medication and side effects.
that verapamil will have which of the following effects is an opposite effect to this medication.
on the body?
1.‐ Lower blood pressure
2.‐ Lower blood pressure and heart rate
3.‐ Increase urinary output
4.‐ Increase heart rate
3383 Phenazopyridine (Pyridium) is prescribed to a client Correct answer: 4 Phenazopyridine is a urinary analgesic with a local anesthetic effect on the urinary tract Knowing the side effects of the medication will lead to the only correct answer, option 4. If
with dysuria. The client asks the nurse about the side mucosa. This medication relieves pain during urinary tract infection. It causes the urine to have this was difficult, review the side effects of the medication.
effects of this medication. The nurse should inform the an orange/red color. It has no effect on volume of urine. Foul odor to the urine might be
client to expect which of the following urine caused by urinary tract infection.
characteristics?
1.‐ Decreased volume
2.‐ Foul in odor
3.‐ Increased volume
4.‐ Orange/red in color

3384 A client's blood pressure (BP) continues to drop Correct answer: 2 Dopamine acts on the alpha/beta‐adrenergic receptors, resulting in vasoconstriction, Knowing the purposes of the medication will lead to the only correct answer, option 2. The
despite IV fluids. Intravenous dopamine is ordered. The increasing systemic BP, and increasing force and rate of myocardial contraction. Options 1, 3, other options are not purposes of the medication. If this was difficult, review the purpose
client asks the nurse about the possible benefits of this and 4 are incorrect. of the medication.
medication. The nurse responds to the client based on
the understanding that dopamine treats shock by:

1.‐ Blocking AV node conduction.


2.‐ Causing vasoconstriction, and increasing systemic BP.
3.‐ Decreasing the rate of myocardial contraction.
4.‐ Promoting diuresis.

3385 The client being seen in an ambulatory clinic has Correct answer: 1 Emphasis should be placed on the client's adherence to the plan of treatment to avoid serious Knowing that antihypertensives should be taken consistently will lead to the correct
hypertension. During assessment, the client mentions consequences of noncompliance. The complications of high blood pressure include stroke, answer. Note that the question is asking for development of a teaching plan, and only
that she will stop taking her antihypertensive cardiac failure, and chronic renal failure. option 1 incorporates teaching.
medications as soon as her blood pressure is under
control. In developing a medication teaching plan, the
nurse includes which of the following instructions?

1.‐ "In order to maintain control of your blood pressure, the medication must be continued indefinitely."
2.‐ "Only the physician can answer this question."
3.‐ “The medication will probably be stopped after your blood pressure is normal.”
4.‐ "The medication will be decreased in time."

3386 The home health nurse instructs the client about use Correct answer: 3 Some clients will experience an increased blood pressure with OTC cold preparations such as Knowing the classification of an antihypertensive and the effects of the options will lead to
of an antihypertensive medication. The client plans to pseudoephedrine, due to vasoconstriction. Therefore, they should avoid taking these the correct answer. If this was difficult, review the general considerations of an
take an over‐the‐counter (OTC) medication for his medications with an antihypertensive. Options 1, 2, and 4 are incorrect. antihypertensive medication.
cold. The nurse should instruct the client to avoid
which of the following OTC products while taking an
antihypertensive?
1.‐ Acetaminophen
2.‐ Aspirin
3.‐ Pseudoephedrine
4.‐ Steroid cream

3387 A client with congestive heart failure is taking digoxin Correct answer: 4 Risk of hypokalemia is worsened by the concurrent use of a potassium‐wasting diuretic (Lasix) The core issue of this question is the effects of taking a potassium‐losing diuretic with
(Lanoxin) and furosemide (Lasix). A new diagnosis of and a beta‐agonist (albuterol) medication. Furthermore, the risk of cardiac glycoside toxicity is other cardiac or respiratory medications, which helps you to focus on hypokalemia. Note
acute bronchitis is made, and albuterol (Proventil) via worse in the presence of hypokalemia. Hyperkalemia (option 1), hypernatremia (option 2), and also that options 1 and 4 are opposite, so there is a greater chance that one of these is the
inhalation is started. The nurse concludes that this hypocalcemia (option 3) are not concerns with this drug regimen. correct answer.
client is at risk for which of the following
complications?
1.‐ Hyperkalemia
2.‐ Hypernatremia
3.‐ Hypocalcemia
4.‐ Hypokalemia

3388 A client with asthma has started to take a beta‐ Correct answer: 2 Concurrent use of an MAOI and a beta‐agonist can lead to hypertensive crisis. The beta‐ The core issue of the question is interactive effects of beta‐agonists and MAOIs. Use the
adrenergic agent. The client also takes a monoamine agonist could lead to tachycardia (option 3), but since no specific agent is listed, the nurse process of elimination, and recall that an agonist type of drug enhances the action of a
oxidase inhibitor (MAOI). The nurse assesses the client should consider the potential interaction of the MAOI and the beta‐agonist first. Hypotension system, in this case the beta‐adrenergic system. Then recall that these effects include
for which of the following complications? (option 1) and bradycardia (option 4) are not of concern with this combination of medications. increased pulse and blood pressure. Note also that two of these options are opposites,
making it more likely that one of them is the correct answer.

1.‐ Hypotension
2.‐ Hypertension
3.‐ Tachycardia
4.‐ Bradycardia

3389 A diabetic client admitted to the Emergency Correct answer: 1 Epinephrine is a beta‐adrenergic agent used to dilate bronchial airways. It can cause an The core issue of the question is knowledge that beta‐adrenergic medications stimulate
Department with acute bronchospasm is given increased blood glucose level, which is especially an issue for a client with diabetes mellitus. the sympathetic nervous system, and that one of the end effects of the stimulation is
epinephrine (Bronkaid). The nurse should assess the Diabetic clients should be instructed to monitor blood glucose levels because an adjustment in increased blood glucose. Correlate this knowledge with the client in the question, who has
client for which side effect of this medication? maintenance doses of hypoglycemic agents could be indicated. Option 2, 3, and 4 are diabetes, to make a correct selection relative to hyperglycemia.
unrelated to effects of the medication on diabetic clients.

1.‐ Blood glucose level 156 mg/dL


2.‐ Blood glucose level 77 mg/dL
3.‐ Potassium level 5.4 mEq/L
4.‐ Potassium level 3.1 mEq/L

3390 The nurse is teaching a client with chronic obstructive Correct answer: 2 The client should wait at least one minute between inhalations. Dosages should be taken The core issue of the question is correct administration procedure for inhaled
pulmonary disease (COPD) how to administer multiple exactly as prescribed (option 1). The OTC products should not be added without consulting the medications. The wording of the question tells you the correct answer is an option phrased
medications by inhalation. Which statement by the physician (option 3). Inhaler equipment should be rinsed and dried daily to keep it clean as a true statement.
client indicates an understanding of the instruction? (option 4).

1.‐ “If my symptoms get worse, I can double my dosage.”


2.‐ “I will wait at least one minute between use of my different inhalers.”
3.‐ “I can take any of the over‐the‐counter medications I need for my symptoms.”
4.‐ “I cannot rinse my inhaler equipment, because it is not supposed to get wet.”

3391 A client complains that a newly prescribed beta‐ Correct answer: 1 Nervousness and tremors might be experienced when medication is newly administered, but Eliminate option 2 because this is not advice a nurse would give, and choose option 1 over
adrenergic agent is causing nervousness and tremors. they frequently decrease over time. Clients should not terminate medication use without 3 and 4 because it is the true statement.
Which statement by the nurse is effective? consulting the prescriber (option 2). Caffeine would exacerbate the problem (option 3). The
symptoms are likely related to the medication, and not to the disease process (option 4).

1.‐ “Sometimes, those symptoms occur when first taking the medication but decrease over time.”
2.‐ “Stop taking the medicine, because those symptoms will only get worse.”
3.‐ “Drinking coffee or tea will help decrease the symptoms.”
4.‐ “Those symptoms indicate a worsening disease process, and should be reported to the physician.”

3392 The nurse is teaching the client about home Correct answer: 4 Taking the medication with food can decrease GI symptoms. Sustained‐release forms should The core issues of the question are general medication knowledge and instructions for use
administration of theophylline (Theo‐Dur). Which not be crushed or chewed, because doing so irritates the gastric mucosa and changes the of theophylline. Eliminate options 1 and 2 first because they are not consistent with general
statement by the nurse is reflective of appropriate absorption of the medication (option 1). Medications should be taken as prescribed, without principles of self‐administration. Choose option 4 over 3 because the medication is not for
teaching? omissions or doubled doses (option 2). Medications should be taken at all times, not just when p.r.n. use.
symptomatic (option 3). Prophylaxis, not acute treatment, is the goal.

1.‐ “Sustained‐release forms can be crushed, so they are easier to swallow.”


2.‐ “If a dose is omitted, take a double dose the next time.”
3.‐ “Only take the medication when acute symptoms occur.”
4.‐ “Take the medication with food if gastrointestinal symptoms occur.”

3393 A client who takes theophylline (Theo‐Dur) complains Correct answer: 1 Restlessness is a sign of theophylline toxicity, but often is a first indicator of hypoxia. The first The question contains the word first, which indicates that more than one option might be
of restlessness. Which is the most appropriate action and best action is to assess for hypoxia. After ruling it out, the other actions should be taken: technically correct, but one is best. Choose option 1 over options 2, 3, and 4 because all the
for the nurse to take first? assessing for other signs and symptoms of toxicity, obtaining an order for the blood level, and incorrect options refer to the theme of toxicity. If options are very similar, none of them
explaining toxicity to the client. can be correct.
1.‐ Assess the client for hypoxia.
2.‐ Explain that this is a toxic reaction, and call the physician.
3.‐ Assess the client for other signs and symptoms of theophylline toxicity.
4.‐ Call the physician to obtain an order for a theophylline level.

3394 The client asks the nurse why the physician ordered Correct answer: 2 Beclovent is an inhaled corticosteroid that is thought to decrease inflammation and dilate the Use medication knowledge and the process of elimination to make a selection.
beclomethasone (Beclovent) for his chronic airway. The exact mechanism of action is unknown. Beclovent, like any other corticosteroid,
obstructive pulmonary disease (COPD). Which does suppress the immune response, but this is not the rationale for administration of the
statement by the nurse is most appropriate? medication (option 3). Inhaled corticosteroids are thought to increase responsiveness of
bronchial smooth muscle to beta‐agonist drugs (option 4).

1.‐ “Beclovent prevents airway dilation.”


2.‐ “Beclovent decreases inflammation, and makes it easier to breathe.”
3.‐ “Beclovent suppresses the immune response.”
4.‐ “Beclovent decreases responsiveness to medications that dilate the airway.”

3395 The nurse is teaching a client about cromolyn (Intal). Correct answer: 2 Cromolyn is a nonsteroidal agent that stabilizes mast cells so bronchoconstrictive and Use medication knowledge and the process of elimination to make a selection.
Which of the following statements should the nurse inflammatory substances are not released when stimulated with an allergen. It is used to treat
make about the mechanism of action of this inflammation of the airway. It does not cause bronchoconstriction (option 4), and is not a
medication? bronchodilator (option 1) or expectorant (option 3).
1.‐ “Cromolyn relaxes bronchial smooth muscle, to assist with bronchodilation.”
2.‐ “Cromolyn limits inflammation, and therefore bronchoconstriction, with exposure to an allergen.”
3.‐ “Cromolyn helps to liquefy secretions to promote expectoration.”
4.‐ “Cromolyn promotes bronchoconstriction of overly dilated airways.”

3396 The nurse is assessing the client who takes cromolyn Correct answer: 4 Side effects of cromolyn include dry mouth, irritated throat, cough, unpleasant taste, and Specific medication knowledge is needed to answer this question. Use the process of
(Intal). Which symptom indicates to the nurse that the headaches. Side effects do not include vomiting (option 1) or tachycardia (option 3). Moist elimination to make a selection.
client is experiencing a potential side effect? mucous membranes (option 2) are a normal finding.

1.‐ Vomiting
2.‐ Moist mucous membranes
3.‐ Tachycardia
4.‐ Headache

3397 A client beginning medication therapy with Correct answer: 1 Leukotrienes are released when a client is exposed to an allergen. Leukotrienes cause Specific medication knowledge is needed to answer this question. Use the process of
montelukast (Singulair) asks the nurse how the inflammation, bronchoconstriction, and mucus production. Leukotriene modifiers such as elimination to make a selection.
medication is helping his symptoms. Which is the best montelukast block the action of leukotrienes, and therefore decrease mucous secretion and
response? reduce inflammation, preventing bronchoconstriction.

1.‐ “Singulair decreases inflammation and mucus secretion.”


2.‐ “Singulair increases mucus secretion and bronchodilation.”
3.‐ “Singulair prevents smooth muscle contraction by nervous system stimulation.”
4.‐ “Singulair increases the inflammatory response and mucus secretion.”
3398 The client asks the nurse about self‐care related to Correct answer: 2 Liver function tests should be monitored with leukotriene modifiers because of the potential Recall that metabolism and excretion of many drugs occur in either the hepatic or renal
newly ordered zafirlukast (Accolate). The nurse for liver dysfunction with this type of medication. Renal studies are unnecessary (option 1) in systems. This provides a clue that either option 1 or 2 is correct. Choose option 2 over
responds by telling the client that: relation to this medication. Fluid intake should be increased (option 3), unless contraindicated option 1 by associating the letter l in leukotrienes with the letter l for liver.
by another condition, in order to thin secretions and assist in their mobilization. The
medication should be taken one hour before or two hours after meals (option 4).

1.‐ Renal function tests should be monitored.


2.‐ Liver function tests should be monitored.
3.‐ Fluid intake should be decreased.
4.‐ The medication should be taken with meals.

3399 A client asks the nurse if there is a benefit to taking Correct answer: 3 Second‐generation antihistamines cause less sedation than do first‐generation medications, Recall that second‐generation drugs generally have some type of improvement over first‐
second‐generation antihistamines instead of first‐ so the client experiences less drowsiness. They are selective for peripheral H1 histamine generation drugs. In this case, since antihistamines often cause drowsiness, it is easy to
generation antihistamines. The nurse replies that receptors, and do not cross the blood–brain barrier. Nausea (option 1), anxiety (option 2), and reason that this side effect would be decreased in second‐generation medications in this
second‐generation drugs cause less: euphoria (option 4) are unrelated as comparison points between first‐ and second‐generation category.
antihistamines.
1.‐ Nausea.
2.‐ Anxiety.
3.‐ Drowsiness.
4.‐ Euphoria.

3400 A client asks the nurse about drug interactions with Correct answer: 1 The effects of first‐generation antihistamines are increased with alcohol, tricyclic Recall first that first‐generation antihistamines cause drowsiness, and look for an option
diphenhydramine (Benadryl). The nurse informs the antidepressants, antianxiety agents, antipsychotic agents, opioid analgesics, sedative that would have an additive effect. Since alcohol is a CNS depressant, it is the correct choice
client that which of the following will increase the hypnotics, and monoamine oxidase inhibitors. Nicotine (option 2), caffeine (option 3), and CNS for this question; the others represent agents that stimulate the CNS.
effects of Benadryl? stimulants (option 4) would not have an additive effect.
1.‐ Alcohol
2.‐ Nicotine
3.‐ Caffeine
4.‐ Central nervous system stimulants

3401 The nurse is teaching the client proper technique for Correct answer: 2 The proper application of nasal spray decongestants is with the client sitting and squeezing A key word in the question is accurate. The core issue of the question is the instruction
administration of nasal sprays. Which explanation the bottle once, holding a finger over the other nostril, and inhaling. Administering more than that is a true statement. Use the process of elimination and knowledge of basic medication
should the nurse use in order to provide accurate one squeeze application (options 1 and 4) would increase the dose. The applicator should be administration procedures to make a selection.
information? rinsed after each use to prevent contamination (option 3).

1.‐ “Lie down and instill the nasal spray, squeezing the bottle twice for each application.”
2.‐ “Sit and squeeze the nasal spray once as you inhale while holding your finger over the other nostril.”
3.‐ “Be careful not to rinse the tip of the spray bottle after use, or you will contaminate the medication.”
4.‐ “Lean your head back and administer two applications to each nostril for each dose, to be sure some of the medication is instilled.”

3402 The nurse is developing a teaching plan for a client Correct answer: 3 Avoidance of eating or drinking for 30 minutes after medication administration allows the The wording of the question tells you the correct answer is also a true statement.
using nasal decongestant. The nurse should include medication time to work. Nasal spray decongestants should not be taken for more than three Eliminate options 2 and 4 first as least likely to be true. Choose option 3 over option 1,
which of the following instructions in the teaching days, because they can cause rebound congestion (option 1). Fluid intake should be increased knowing the side effects of decongestants.
plan? to 2–3 L/day, not decreased, to liquefy secretions (option 2). Smoking should be avoided
because it increases secretions, and decreases ciliary action (option 4).

1.‐ “Be sure to stay on the medication for at least seven days.”
2.‐ “Decrease fluid intake to 16 ounces/day to decrease nasal secretions.”
3.‐ “Avoid eating or drinking for 30 minutes after medication administration.”
4.‐ “Smoking decreases secretions, and increases ciliary action.”
3403 The nurse is preparing to teach the client important Correct answer: 1 It is important to teach clients side effects of medications. The side effects of expectorants Use the process of elimination and general medication knowledge to answer the question.
information related to self‐administration of include nausea, vomiting, gastric irritation, and rash. If a cough lasts longer than a week Eliminate option 2 because the time frame is excessive, and option 4 because it is not
guaifenesin (Robitussin). Which of the following should (option 2), it should be reported to the physician. The client should avoid eating or drinking for standard medication teaching. Choose option 1 over option 3 because it is a true
be included in the teaching plan? 30 minutes after medication administration to allow the medication to work (option 3). The statement, and because option 3 could hinder medication absorption.
medication should be taken as directed, and doses should not be doubled (option 4).

1.‐ Side effects include nausea, vomiting, and rash.


2.‐ Report a cough to the physician if it lasts longer than two weeks.
3.‐ Take the medication with meals.
4.‐ If the medication is not effective, double the dose.

3404 The nurse is assessing a client for side effects of an Correct answer: 4 All of the symptoms listed are potential side effects of opioid antitussives. The most The key words most significant tell you that more than one option might be technically
opioid antitussive. Of the following, which is the most significant side effect is respiratory depression, evidenced by a respiratory rate of 10, when correct, and that you must select the most important option. Use the ABCs (airway,
significant side effect assessed? normal is 12–20 breaths/minute. breathing, and circulation) to make your selection.
1.‐ Dry, cracked lips
2.‐ Complaints of blurred vision
3.‐ Inability to stay awake
4.‐ Respirations of 10/min

3405 Which of the following statements by a client taking a Correct answer: 4 Unused medication should be discarded after four days, not seven. Avoidance of smoking is The phrase further teaching tells you the correct answer is an incorrect or false statement.
mucolytic agent indicates a need for further teaching? necessary because smoking increases secretions and decreases ciliary action (option 2). Eliminate option 1 first because this is an important principle to follow, and then option 3
Increasing fluids assists with thinning secretions (option 1). Rinsing the mouth after because this is a general medication administration principle. Choose option 4 over 2
administration decreases oropharyngeal irritation (option 3). because smoking cessation is always recommended.

1.‐ “I will drink at least 2–3 liters of fluid each day.”


2.‐ “I will avoid smoking.”
3.‐ “I will rinse my mouth after I take my medicine.”
4.‐ “I can keep the medicine for a week before discarding what I do not use.”

3406 The nurse has an order to administer 50% oxygen to a Correct answer: 4 The Venturi mask has a dial to set the percentage of oxygen, and can administer 50% oxygen. Specific knowledge of the various concepts related to oxygen therapy is needed to answer
client with pulmonary edema. The nurse would select The nonrebreather mask (option 2) administers 60–100% oxygen. The partial rebreather mask this question. Use the process of elimination to make a selection, and remember that
which of the following oxygen administration systems (option 3) administers 70–90% oxygen. The nasal cannula (option 1) can administer up to 6 Venturi masks deliver precise oxygen concentrations.
that allows that percentage of oxygen to be delivered? L/min, which is approximately 44% oxygen.

1.‐ Nasal cannula


2.‐ Nonrebreather mask
3.‐ Partial rebreather mask
4.‐ Venturi mask

3407 The nurse who administers albuterol (Proventil) to a Correct answer: 1 The symptoms of an acute asthma attack are related to constriction of the airway. The Note that this question asks for identification of the intended effects. This refers to client
client with symptoms of an acute asthma attack medication is a beta‐adrenergic agent administered to dilate the airway. Option 2 is a side response. Option 1 is the correct answer, as this is the intended response. Understanding
anticipates which of the following intended effects? effect of the medication, but is not the intended effect. Options 3 and 4 are incorrect because the differences between side effects and client responses will lead to choosing the correct
bradycardia and bronchoconstriction are the opposites of the expected side effect and answer.
intended effect, respectively.
1.‐ Dilation of the airway
2.‐ Elevation of the heart rate
3.‐ Constriction of the airway
4.‐ Slowing of the heart rate
3408 The nurse should question an order for epinephrine Correct answer: 2 Adrenergic agents are contraindicated for clients with cardiovascular disease because of the Recall knowledge of the disease process coronary artery disease to choose the correct
(Primatene) in the treatment of acute bronchitis when potential to increase myocardial oxygen demand. Epinephrine would raise the heart rate and answer. Using the process of elimination and knowledge of the pathophysiology of the
the client has which of the following diseases? blood pressure, but could decrease oxygenation of the myocardium for the client with disease process, the only correct option is number 2.
cardiovascular disease. Asthma (option 1), hypotension (option 3), and bradycardia (option 4)
are not contraindications for use of epinephrine (Primatene).

1.‐ Asthma
2.‐ Coronary artery disease
3.‐ Hypotension
4.‐ Bradycardia

3409 Which item should the nurse plan to omit from the Correct answer: 4 Caffeine in coffee or tea can have an additive effect with theophylline, and therefore coffee Recall knowledge of the side effect of theophylline, vasodilation, to be led to the correct
meal tray of the client being treated with theophylline should be eliminated from the meal tray. Peas (option 1), beans (option 2), and milk (option 3) answer. By eliminating caffeine from the client s food tray, the additional additive effect of
(Theo‐Dur)? are not problematic because they do not contain caffeine. caffeine can be avoided.

1.‐ Peas
2.‐ Beans
3.‐ Milk
4.‐ Coffee

3410 The nurse is assessing a client with chronic Correct answer: 3 A potential side effect of an inhaled corticosteroid is oral fungal infection. It would be Note that the question is asking for identification of the side effects of the medication. Use
obstructive pulmonary disease (COPD) who is being therapeutic to have a decrease in audible wheezes (option 1). Inhaled corticosteroids can the process of elimination to rule out all choices except option 3. If this was difficult, review
treated with beclomethasone dipropionate (Beclovent) cause dry mouth (option 2), and with less respiratory effort from effective therapy, the nurse side effects of inhaled corticosteroids.
via oral inhaler. Which client manifestation would the should anticipate a decreased respiratory rate (option 4).
nurse conclude is a side effect of this medication?

1.‐ Moist mucous membranes


2.‐ Decrease in the audible wheezes
3.‐ Oral fungal infection
4.‐ Decreased respiratory rate

3411 A client on the inpatient unit is having an acute Correct answer: 2 Bitolterol is an adrenergic bronchodilator that is effective to provide bronchodilation in an The question is asking for selection of the correct answer for a client who is having an
asthma attack. Which of the medications from the acute asthma attack. Aminophylline (option 1) is a xanthine, triamcinolone (option 3) is an acute attack. Considering this, realize that in selecting the medication, the response should
medication sheet would the nurse administer? inhaled corticosteroid, and cromolyn (option 4) is an inhaled nonsteroidal. All three of these be immediate bronchodilation. While all medications are effective in the treatment of
agents can be used with asthma; however, they are not effective during an acute attack. asthma, the key concept is acute treatment.

1.‐ Aminophylline (Truphylline)


2.‐ Bitolterol (Tornalate)
3.‐ Triamcinolone acetonide (Azmacort)
4.‐ Cromolyn (Intal)

3412 The nurse is evaluating the effectiveness of client Correct answer: 2 Zafirlukast is a leukotriene modifier. This is a newer class of medication for the prophylaxis Note that the question is asking for selection of the response indicating that the client
teaching about home medication administration. and chronic treatment of asthma. Because they are not to be used during an acute attack, this needs more teaching. Use knowledge about the medication and the process of elimination
Which statement by the client indicates that more response indicates that the client needs more teaching. Fluid intake should increase to liquefy to select the correct option. Zafirlukast is contraindicated in an acute asthma attack. If the
teaching is needed related to zafirlukast (Accolate)? secretions and assist the client with expectoration. This medication should be taken one hour client is stating she will use the medication during an acute attack, this would indicate that
before meals or two hours after meals. It does take a few weeks of medication administration further teaching is required.
for the client to begin to see positive results.

1.‐ "I will take the drug a few weeks before I expect to notice an improvement in my symptoms."
2.‐ "I will use this medication when I have the symptoms of an acute asthma attack."
3.‐ "I will increase my fluid intake while I am taking this medication."
4.‐ "I will plan to take my medicine one hour before meals."
3413 Which statement would the nurse include when Correct answer: 4 Claritin should be taken on an empty stomach, to increase absorption. It is a second‐ Recall knowledge of the medication loratadine (Claritin) to select the correct answer.
giving a client instructions about loratadine (Claritin)? generation antihistamine, and does not cause drowsiness as the first‐generation medications Option 4 is the only correct answer, since it should be taken on an empty stomach to
do (option 1). It has a rapid onset of action (option 2), and is not effective in an acute asthma increase the absorption rate and thus effectiveness. If this was difficult, review nursing
attack (option 3). interventions for the medication of loratadine (Claritin).
1.‐ "This medication will make you drowsy."
2.‐ "It will take this medication a few hours to take effect."
3.‐ "This medication will help in an acute asthma attack."
4.‐ "Be sure to take this medication on an empty stomach."

3414 In which circumstance should the nurse inform the Correct answer: 2 Cardiovascular side effects are possible with the administration of decongestants. If the client Note that the question is asking for selection of a circumstance requiring a client response,
client to stop taking oral phenylephrine (Neo‐ develops these symptoms, the medication should be discontinued, and the physician notified. indicating that the client should stop taking the medication. Use the process of elimination
Synephrine)? Oral agents should be used for long‐term therapy (option 1). Rebound congestion (option 4) is and knowledge of the side effects of oral decongestants to select option 2. This medication
more likely with nasal spray decongestants. Often, decongestants cause a dry mouth (option can increase blood pressure and heart rate. If this was difficult, review side effects of oral
3), but the client should use hard, sugarless candy rather than discontinue the medication. decongestants.

1.‐ After three days of continuous use


2.‐ If the client develops hypertension or tachycardia
3.‐ If the client develops a dry, irritated oral cavity
4.‐ If the client develops rebound congestion

3415 The nurse correctly teaches the client about which Correct answer: 1 Guaifenesin is an expectorant. Potential side effects are nausea, vomiting, gastric irritation, Use the process of elimination and knowledge of the side effects for guaifenesin
potential side effect of guaifenesin (Robitussin)? rash, dizziness, and headache. It does not cause hypertension (option 2), hypotension (option (Robitussin) to select option 1. The other choices are not side effects, and can be
3), or urinary retention (option 4). eliminated. If this was difficult, review the side effects of guaifenesin (Robitussin).

1.‐ Gastric irritation


2.‐ Hypertension
3.‐ Hypotension
4.‐ Urinary retention

3416 The nurse is caring for a client with acute bronchitis Correct answer: 3 A non‐rebreather mask should have flaps on the sides that are open during expiration and In selecting the correct response to this question, recall knowledge of the function of a
who is using a non‐rebreather mask. The nurse notices closed on inspiration. The idea is for the client to breathe in oxygen, and not the expired non‐rebreather mask. If this was difficult, review the correct functioning and purpose of a
that the flaps on the sides of the mask are off. Which carbon dioxide. If the flaps are missing, the client needs a new mask. The nurse should not non‐rebreather mask.
action is appropriate? change the oxygen order (option 2), and it is unnecessary to call the physician (option 4).

1.‐ Allow the client to continue to use this mask, as it appears to be functional.
2.‐ Change the client to a nasal cannula.
3.‐ Replace the non‐rebreather mask with a new one.
4.‐ Call the physician.

3417 A 6‐year‐old child with asthma is being treated with Correct answer: 1 Potential side effects of this medication are stimulation of the central nervous system (CNS) Recall knowledge of the medication metaproterenol (Alupent) to select the correct answer
metaproterenol (Alupent). The mother informs the and cardiovascular (CV) system. Metaproterenol is a beta 2 stimulant, and these effects are in option 1, as administering increased doses to a child can cause stimulation of the CNS
nurse that she has been using the medication more not as likely, but with increased doses, they could occur, especially in a 6‐year‐old child. and CV systems. If this was difficult, review the medication side effects.
frequently lately because the child's symptoms have Lethargy and bradycardia (option 2), decreased blood pressure (option 3), and fatigue (option
worsened. For what potential side effects should the 4) are not consistent with either CNS or CV stimulation.
nurse monitor the client?

1.‐ Nervousness and tachycardia


2.‐ Lethargy and bradycardia
3.‐ Decreased blood pressure and dizziness
4.‐ Increased blood pressure and fatigue
3418 The nurse is teaching a client about salmeterol Correct answer: 4 Use of salmeterol is prophylactic, not for an acute attack. Salmeterol is predominately a beta Knowledge of the purpose of the medication salmeterol (Serevent) will lead you to the
(Serevent), which is to be used at home. Which 2 stimulant, and therefore does not frequently cause tachycardia (option 3). It takes 20 correct answer. This medication is for prophylactic use, and the only answer choice that
statement indicates that the client has understood the minutes for onset of action, and is used for prophylaxis, not treatment of acute attack (option refers to prophylaxis is option 4. If this was difficult, review the administration interventions
teaching? 2). It is dosed every 12 hours because of a 12‐hour duration of action (option 1). for the medication.

1.‐ "I will use this medication every 6 hours."


2.‐ "I will take a dose of this medicine when I notice I am wheezing."
3.‐ "I know this medicine commonly causes increased heart rate."
4.‐ "This medicine is to keep me from having an attack, not to stop one that has started."

3419 A client takes oxtriphylline (Choledyl) for chronic Correct answer: 3 Oxtriphylline is a xanthine brochodilator, and the mechanism of action is to increase the Note that the question asks for an explanation of the rationale for taking the medication
obstructive pulmonary disease (COPD). Which amount of cyclic adenosine monophosphate (cAMP), which leads to bronchial dilation due to relative to chronic obstructive pulmonary disease (COPD). Understand the disease process
explanation by the nurse correctly teaches the client relaxation of smooth muscle. Xanthines can increase heart rate and force of myocardial and the purpose of the medication to be directed to the correct answer. The only correct
the rationale for taking this medication? contraction, but that is not the rationale for the administration of the medication. choice is option 3, as this is the desired effect for a client with COPD.

1.‐ "The medicine increases your heart rate to help with blood flow."
2.‐ "The medicine helps your heart beat stronger and get more blood to the lungs."
3.‐ "The medicine is used to dilate your airways, and make it easier for you to breathe."
4.‐ "The medicine thins the secretions in your lungs, and makes it easier for you to cough."

3420 A client is to be started on theophylline (Theo‐Dur). Correct answer: 3 Theophylline is contraindicated in clients with hyperthyroidism, as the disease can be Use the process of elimination and knowledge of the medication to select the correct
The nurse should plan to consult the physician about exacerbated. It is also contraindicated in clients with tachydysrhythmias. Options 2 and 4 result answer. The only correct answer is option 3, as this medication is contraindicated in clients
changing the order if the client has which of the in low heart rates, and are therefore incorrect. with hyperthyroidism.
following conditions?
1.‐ Hypothyroidism
2.‐ Bradycardia
3.‐ Hyperthyroidism
4.‐ Sick sinus syndrome

3421 A client is admitted to the hospital with an Correct answer: 4 When both an inhaled and a systemic corticosteroid are used, a decrease in the dose of one When administering two corticosteroids to a client, smaller doses of the two medications
exacerbation of chronic obstructive pulmonary disease or the other medication might be appropriate, due to the additive effect of local and systemic can be anticipated.
(COPD). Triamcinolone acetomide (Azmacort) is one of corticosteroids. Options 1 and 2 are incorrect because they indicate increased doses, while
the home medications. Dexamethasone (Decadron) is option 3 is incorrect because the symptoms should decrease, rather than increase.
added intravenously in the hospital. The nurse should
anticipate which of the following because of
interaction of these two medications?

1.‐ An increase in the Azmacort


2.‐ A higher dose of Decadron
3.‐ An increase in the symptoms with administration of the two drugs
4.‐ A smaller dose of Azmacort and/or Decadron

3422 The nurse should question an order for fluticasone Correct answer: 1 Administration of corticosteroids such as fluticasone suppresses the immune system, and the Knowledge of the side effects of immunosuppression with corticosteroids, as well as
aerosol (Flovent) when a client has which of the administration of these drugs is contraindicated in clients with suppressed immune systems (as knowledge of the pathophysiologic process of AIDS, will lead to the correct answer. Since
following conditions? in AIDS). Fluticasone might be helpful with asthma (option 2) and COPD (option 4). It is not AIDS clients have a suppressed immune response, further suppression would result when a
contraindicated with CAD (option 3), although it should be used cautiously because of possible corticosteroid is administered. This is the only correct answer.
fluid retention.
1.‐ Acquired immunodeficiency syndrome (AIDS)
2.‐ Asthma
3.‐ Coronary artery disease (CAD)
4.‐ Chronic obstructive pulmonary disease (COPD)
3423 Which statement by the client indicates that teaching Correct answer: 3 Nedocromil should be used as ordered, even if no symptoms are noted. This medication is Having knowledge of the purpose of the medication nedocromil (Tilade) will lead to the
has been effective about the administration of used for the prophylaxis of asthma, not during an acute attack (option 1). It is possible that a correct answer. This medication is for prophylactic use. The only option that refers to
nedocromil (Tilade)? decreased (not increased, as in option 2) amount of bronchodilator and/or inhaled prophylaxis is option 3. If this was difficult, review the administration interventions for the
corticosteroid might be needed after starting this medication, but this is not certain. It can medication.
take three weeks of daily dosing prior to seeing therapeutic effects (option 4).

1.‐ "I will take this medication only during an acute attack."
2.‐ "I may have to increase my dose of beclomethasone (Beclovent) after I start taking this medication."
3.‐ "I will take this medication daily, regardless of whether I experience symptoms."
4.‐ "I should see therapeutic effects of this medication as soon as I begin taking it."

3424 A nurse is doing an admission history on a client who Correct answer: 3 Zileuton is a leukotriene modifier that blocks production of leukotriene, and thereby reduces Use the process of elimination and knowledge of zileuton (Zyflo) to select the only correct
takes zileuton (Zyflo). Which manifestation noted inflammation. The side effects of zileuton include headaches, dyspepsia, nausea, dizziness, and answer in this question, option 3. If this was difficult, review side effects of the medication
during the initial assessment would the nurse conclude insomnia. They do not include lethargy (option 1), constipation (option 2), or diarrhea (option zileuton (Zyflo).
is most likely a side effect of the medication? 4), although zafirlukast, another leukotriene modifier, can cause nausea and diarrhea.

1.‐ Lethargy
2.‐ Constipation
3.‐ Headaches
4.‐ Diarrhea

3425 An 80‐year‐old client who complaines of nausea is Correct answer: 4 Promethazine is a traditional antihistamine that causes drowsiness because it works centrally Note that the question asks for identification of safety. Options 2, 3, and 4 provide a
being given promethazine (Phenergan) 12.5 mg as well as peripherally. It can cause central nervous system depression or stimulation. The limited amount of safety, while option 1 provides the highest level. Eliminate the lower
intravenously. Which safety measure should the nurse client should be kept in bed, with the side rails up, until the effects of the drug wear off, to safety level options to be led to the higher safety level stated in option 1.
institute? promote client safety. The effects are heightened by the client's age. The actions in options 1,
2, and 3 provide a lower margin of safety for the client.

1.‐ Provide the client with no‐skid slippers for ambulation.


2.‐ Show the client the emergency cord in the bathroom, and give instructions for use.
3.‐ Put a chair next to the client's bed to make it easier for her to get herself out of bed.
4.‐ Keep the client in bed, with the side rails up.

3426 The nurse informs the client that oxymetazoline Correct answer: 2 Afrin is a topical decongestant and an adrenergic agent that promotes nasal decongestion by Having knowledge of the side effects of oxymetazoline (Afrin) and the modality of the
(Afrin) should not be utilized if the client has which of vasoconstriction. Adrenergic decongestants are contraindicated for the client with treatment will lead to selection of the correct answer. Vasoconstriction leads to
the following conditions? hypertension and coronary artery disease. Contraindications do not include hypotension hypertension, which is listed in option 2. If this was difficult, review the side effects of the
(option 1), hypothyroidism (option 3), or emphysema (option 4). medication oxymetazoline (Afrin).

1.‐ Hypotension
2.‐ Hypertension
3.‐ Hypothyroidism
4.‐ Emphysema

3427 A client presents to the Emergency Department with Correct answer: 2 Epinephrine is a beta‐adrenergic agent that that has beta 1 adrenergic action, causing Recall knowledge of the onset of the medication Epinephrine (Bronkaid). The only correct
inspiratory and expiratory wheezes, and intercostal increased heart rate and increased force of myocardial contraction. The results of answer is option 2. If this was difficult, review the medication and its onset.
retractions. A diagnosis of acute bronchospasm subcutaneous epinephrine should be seen in five minutes. The effects can last up to four hours.
secondary to acute bronchitis is made. Epinephrine The other options are incorrect.
(Bronkaid) is ordered to be given subcutaneously. The
nurse would anticipate seeing the intended effect of
the medication in:

1.‐ One minute.


2.‐ Five minutes.
3.‐ Ten minutes.
4.‐ Fifteen minutes.

3428 An order for which beta agonist should be questioned Correct answer: 3 Terbutaline, pirbuterol, and metaproterenol are all beta 2 stimulants. Isoproterenol In order to answer this question, recall knowledge of beta 1 and beta 2 stimulants and
because it is contraindicated for a client with a history stimulates beta 1 and beta 2 receptors, and therefore should not be used with clients with their effects on the heart. Use the process of elimination, and exclude those medications of
of atrial fibrillation? tachydysrhythmias. the same category, to be led to the correct choice.
1.‐ Terbutaline (Brethine)
2.‐ Pirbuterol (Maxair)
3.‐ Isoproterenol (Isuprel)
4.‐ Metaproterenol (Alupent)

3429 A 70‐year‐old client with chronic obstructive Correct answer: 3 With increased age, there is an increased sensitivity to xanthines. Also, there are other Knowledge of a normal therapeutic level of theophylline will lead to the correct answer.
pulmonary disease (COPD) is taking theophylline (Theo‐ disease processes that could lead to this elevated value. The dose of theophylline should be Since the question refers to a result of 25 mg/dl, which is high therapeutically, the only
Dur). A blood level is drawn, and the result is 25 decreased to get the blood level to the 10–20 mg/dL range. Theophylline doses should be correct option is 3. If this was difficult, review the therapeutic lab values for theophylline.
mg/dL. What explanation by the nurse helps the client based on lean body weight, to prevent entering the medication into the adipose tissue.
understand this lab result?
1.‐ "Your dose of theophylline needs to be increased."
2.‐ "Your blood level is low because the dose was based on your total body weight instead of on your lean body weight."
3.‐ "The lab value could be high because of your age. We may have to decrease the dosage of your medication."
4.‐ "I am sure that lab value is incorrect. Theophylline levels are never that high."

3430 The nurse is assessing a 30‐year‐old client with a prior Correct answer: 2 Theophylline is a xanthine that causes bronchial dilation due to smooth muscle relaxation. Pay close attention to the assessment data in the stem to eliminate wrong answers and be
history of smoking who takes theophylline (Theo‐Dur) Increased levels of theophylline occur with liver disease and congestive heart failure. Option 3 led to option 3.
for chronic obstructive pulmonary disease. Additional is incorrect because the client is young, and therefore the age is insignificant. The smoking
diagnoses include liver disease and congestive heart history (option 1) is not an issue; in fact, smokers metabolize theophylline more quickly, and
failure. The client is experiencing tremors, dizziness, might need increased doses. There are no data about the client's weight (option 4) in the stem.
tachycardia, and nausea. The nurse explains to the
client that these symptoms could be the result of:

1.‐ A history of smoking cigarettes.


2.‐ Liver disease.
3.‐ The client's age.
4.‐ The client's weight.

3431 For which complication of administration of inhaled Correct answer: 2 The child receiving inhaled corticosteroids should be assessed for impaired bone growth. Note that the question asks for selection of the complication; with the knowledge of the
corticosteroids should the nurse assess a 4‐year‐old Bone growth should be monitored closely, especially in children between 4 and 10 years of side effects of delayed growth and development with children with this medication use, the
child with asthma? age. Improved respiratory function is an expected outcome of treatment (option 1). Decreased correct choice 2 can be selected. If this was difficult, review corticosteroid responses in the
urinary output (option 3) and increased immune response (option 4) do not apply. pediatric population.

1.‐ Improved respiratory function


2.‐ Impaired bone growth
3.‐ Decreased urinary output
4.‐ Increased immune response

3432 The nurse is preparing a teaching plan for the client Correct answer: 3 Inhaled corticosteroids do predispose clients to osteoporosis. The mouth should be rinsed With a strong knowledge base of nursing interventions related to the intake of
taking flunisolide (Aero‐Bid‐M). Which of the following after the medication is administered to decrease the likelihood of oropharyngeal candidiasis corticosteroids, the correct answer can be selected. Use the process of elimination to be
items should be included? (option 1). Clients should avoid allergens (option 2). The medication should be taken directed to option 3. If this was difficult, review the general nursing interventions in the
prophylactically, not with an acute attack (option 4). long‐term use of corticosteroids.
1.‐ Avoid rinsing mouth after use of the inhaler.
2.‐ Exposure to allergens will assist with the healing process.
3.‐ Inhaled corticosteroids predispose the client to osteoporosis.
4.‐ Only take the medication when experiencing activity intolerance and/or awakening with symptoms of asthma.
3433 The nurse is teaching a client the proper technique Correct answer: 4 The nurse should teach the client proper administration technique for inhaled medications. Recall general knowledge of the use of oral inhalers to be led to the correct answer,
for administration of oral inhalers. Which statement by The canister should be shaken (option 1), and the cap removed. The client should sit or stand option 4. If this was difficult, review the teaching protocol for the client who has been
the client indicates the teaching has been effective? for maximal lung inflation (option 2). The client should coordinate pressing the canister to prescribed an oral inhalator.
release the medication and inhalation to get the medication into the lungs. The client should
hold her breath for 10 seconds. One to three minutes should elapse between inhalations or
prior to administration of another medication (option 3).

1.‐ "I will be sure I do not shake the canister prior to administration of the medication."
2.‐ "I will lie on my bed when I take my inhaler."
3.‐ "I will wait 10 minutes between inhalations of medication."
4.‐ "I will sit upright and practice coordinating pressing the canister as I inhale."

3434 The nurse is teaching the client about a newly Correct answer: 3 It is not advised to take two antihistamines concurrently, as they can have additive effects. General knowledge of the use of antihistamines will allow selection of the correct option,
prescribed antihistamine, loratadine (Claritin). The Loratadine is a second‐generation antihistamine, and is not as likely to cause drowsiness 3. If this was difficult, review the teaching protocol for the client who has been prescribed
nurse would include which of the following points? (option 1). Loratadine, unlike other antihistamines, should be taken on an empty stomach to an antihistamine.
increase absorption (option 2). Prolonged exposure to sunlight (option 4) can cause sunburn,
especially while on antihistamines.
1.‐ "This medication causes profound drowsiness."
2.‐ "This medication should be taken with meals."
3.‐ "Do not take another antihistamine at the same time as Claritin."
4.‐ "Prolonged exposure to sunlight is not a problem while on this medication."

3435 The client asks the nurse about the difference Correct answer: 4 Dimetane‐DC is an opioid antitussive that contains codeine, affecting the cough center Recall the difference between the letters DM and DC after a medication. DC would
between dextromethorphan (Robitussin DM) and directly and suppressing the central nervous system. Dextromethorphan is a nonopioid indicate that codeine is in the medication. Use of this knowledge will lead to the only
Dimetane‐DC. The nurse's response should be: antitussive that suppresses the cough reflex directly by affecting the cough center. correct option, 4. DM after a medication name does not refer to a narcotic in the
medication. This will allow you to eliminate the other three options.

1.‐ "Dextromethorphan contains a narcotic."


2.‐ "Dimetane‐DC is non‐narcotic."
3.‐ "Dextromethorphan is an opioid antitussive that contains codeine."
4.‐ "Dimetane‐DC is an opioid antitussive that contains codeine."

3436 A client is admitted through the Emergency Correct answer: 4 Clients with COPD usually have a lower oxygen tension in the blood than do clients who do Knowledge of the usual treatment modalities for a client with COPD will lead to the
Department with an acute exacerbation of chronic not have chronic lung disease. In fact, because COPD clients usually have high carbon dioxide correct answer. With this knowledge and the process of elimination, select the only correct
obstructive pulmonary disease (COPD). The nurse levels, their drive to breathe is from the low oxygen tension instead of the high carbon dioxide option, 4. If this was difficult, review the pathophysiology of COPD and treatment
starts oxygen at a rate of 2 liters/minute with a pulse level. Starting oxygen at higher than 2 liters/min could cause hypoventilation, because it could modalities.
oximeter (SO&lt;sub&gt;2&lt;/sub&gt;) reading of 89% diminish the drive to breathe. SO&lt;sub&gt;2&lt;/sub&gt; values of 87–90% are usually
15 minutes after the oxygen is started. The correct satisfactory for clients with COPD.
interpretation of this action is:

1.‐ The nurse acted inappropriately; the flow rate should be increased now to bring the SO2 up to 92%.
2.‐ The nurse acted inappropriately; the flow rate of oxygen should be decreased, as a SO2 of 84% would be satisfactory for this client.
3.‐ The nurse acted appropriately; a client with COPD can start with a flow rate of 2 L/min, but the nurse should continue to titrate the oxygen to achieve a SO2 level of 90%.

4.‐ The nurse acted appropriately; the flow rates should be kept as low as possible for a client with COPD, to prevent hypoventilation. A SO2 of 89% is acceptable for a client
with COPD.

3437 The client is reporting vague dread; she is pacing and Correct answer: 2 This client is suffering from anxiety. The correct answer is an anxiolytic. Option 1 is a This question requires you to interpret the client s symptoms as being representative of
hyperventilating. Her jaw is clenched, and she is sedative–hypnotic, which would not be prescribed. The client is not suffering from psychosis or anxiety, and to choose the drug category that is effective against it. Focus on the client
wringing her hands. The nurse concludes that this hallucinations, so option 3 is inappropriate. Option 4 is inappropriate for the signs and manifestations to eliminate each of the incorrect options.
client is in need of which of the following types of symptoms described.
medications?
1.‐ A barbiturate
2.‐ An anxiolytic
3.‐ An antipsychotic
4.‐ A CNS stimulant

3438 The client is taking zolpidem (Ambien). What would Correct answer: 3 Zolpidem (Ambien) is a sedative–hypnotic medication used to treat insomnia. Therefore, First, recall that zolpidem is a sedative–hypnotic medication. Next, consider its use as a
be a priority nursing diagnosis for this client? Disturbed Sleep Pattern is the appropriate priority nursing diagnosis. There is not enough bedtime sleep aid to make the appropriate selection.
information in the question to determine whether Self‐care Deficit, Risk for Violence, or
Deficient Fluid Volume would be pertinent for the client.

1.‐ Self‐care Deficit


2.‐ Risk for Violence
3.‐ Disturbed Sleep Pattern
4.‐ Deficient Fluid Volume

3439 The nurse determines that the client understands the Correct answer: 2 The medication normally works within 30 minutes to one hour after administration, making The word understands in the stem of the question tells you the correct answer is also a
effects of flurazepam (Dalmane) ordered at a dose of option 2 correct. Option 1 is incorrect because the client should not be watching stimulating true statement. Recall that the medication is used to enhance sleep, and utilize principles of
30 mg by which of the following client statements? shows on television before trying to fall asleep. Option 3 is incorrect because the medication good sleep hygiene to eliminate each of the incorrect options.
will not work instantly. Option 4 is incorrect because the client should not take a sedative and
then stay active for 30 minutes to one hour after taking medication.

1.‐ “After I take my medication at bedtime, I should be able to watch the boxing match or late night TV show, then go to bed and sleep.”
2.‐ “Once I take my medicine, I should be able to go to bed and read, and I will fall asleep within one hour.”
3.‐ “I will take my medicine, go to bed, and go to sleep.”
4.‐ “I will take my medicine, make my lunch for tomorrow, take my shower, get my clothes ready for work tomorrow, and then go to bed.”

3440 The nurse conducting medication teaching with a Correct answer: 2 In order to safely monitor clozapine, a weekly blood test is mandatory. If the client does not The core issue of the question is knowledge of possible adverse effects of clozapine. Recall
client explains that a safe, effective dose of clozapine have the hematologic exam, the medication is not given for the following week. This is to that the drug can cause bone marrow depression and agranulocytosis to choose the correct
(Clozaril) is established by weekly: monitor for agranulocytosis, the drug s major adverse effect. A full physical exam (option 1) monitoring technique.
and urinalysis (option 3) are unnecessary. Follow‐up visits (option 3) are done periodically, but
might not be needed weekly with the physician.
1.‐ Physical exam by a psychiatrist.
2.‐ Hematological monitoring.
3.‐ Follow‐up visits with a physician.
4.‐ Urinalysis.

3441 A client is taking sertraline (Zoloft). The nurse explains Correct answer: 2 Sertraline is an antidepressant of the SSRI type. These agents work within 1–4 weeks. Option Specific knowledge of the time frame in which SSRIs exert an effect is needed to answer
to the client that how much time will pass before the 1 is an insufficient amount of time, while options 3 and 4 are excessive, as well as similar. this question. Use medication knowledge and the process of elimination to make a
onset of the medication occurs? selection.
1.‐ 5–7 days
2.‐ 1–4 weeks
3.‐ 4–6 weeks
4.‐ 4–8 weeks

3442 The visiting nurse is evaluating for client safety. The Correct answer: 4 With an MAOI, such as phenelzine, the client must eliminate foods that contain tyramine. The core issue of the question is knowledge that phenelzine is an MAOI. From there, recall
client is taking phenelzine (Nardil). A priority of the Intake of tyramine‐containing foods could lead to severe hypertension and other that foods high in tyramine need to be avoided to make the correct selection.
nurse’s teaching includes which of the following? complications. All of the other considerations are not major teaching considerations for
MAOIs.
1.‐ Limiting daily intake of salt
2.‐ Encouraging a fluid intake of at least 2,000 mL
3.‐ Encouraging the client to have scheduled blood tests on time
4.‐ Eliminating foods containing tyramine
3443 The client is diaphoretic, is disoriented, has a Correct answer: 1 These symptoms are the commonly seen symptoms of withdrawal from alcohol or other CNS Clients withdrawing from a substance are likely to experience the opposite effects of the
temperature of 100°F, has insomnia, and is depressants. Option 2 is incorrect because there would usually not be complaints of original drug. With this in mind, review the client s symptoms, and note that they represent
complaining of feeling anxious and unable to sit still. disorientation or insomnia with flulike symptoms. Individuals do not usually have withdrawal excitation of the CNS. With this in mind, you can deduce that the original substance was
The nurse suspects this client might: symptoms from antipsychotic medications (option 3), nor are these the signs of lithium some type of CNS depressant, leading you to option 1.
carbonate discontinuation (option 4).
1.‐ Be withdrawing from alcohol or CNS depressants.
2.‐ Be demonstrating flu symptoms.
3.‐ Be withdrawing from an antipsychotic medication.
4.‐ Have abruptly discontinued lithium carbonate.

3444 Immediately after taking alprazolam (Xanax) the Correct answer: 3 In the correct response, the nurse acknowledges the client s feelings, and asks the client to The correct answer to the question is one that is the most therapeutic response by the
client says, “I know I shouldn’t feel this guilty, but I discuss his feelings and thoughts. In options 1, 2, and 4, the nurse is not acknowledging the nurse. Use knowledge of communication techniques and the process of elimination to make
don’t want to take medicine that makes me feel this client s feelings or thoughts. An open and trusting nurse–client relationship helps support the a selection.
way.” What would be the most appropriate response client in decisions related to medication therapy.
by the nurse?
1.‐ “Are you worried about what people will say because you are taking this medicine?”
2.‐ “Once the medication begins to work, you’ll feel differently about taking it.”
3.‐ “Can we talk about how you’re feeling about taking Xanax?”
4.‐ “It will be better for your long‐term mental health if you don’t worry about what other people think.”

3445 The client is taking carbamazepine (Tegretol) for Correct answer: 2 The most serious side effect of carbamazepine is agranulocytosis (low WBC count). The core issue of the question is that an adverse effect of carbamazepine is
treatment of mania. The client is instructed to come to Neuroleptic malignant syndrome is not common with carbamazepine, nor is there a need for agranulocytosis. With this in mind, eliminate each of the incorrect responses that do not
the laboratory weekly for monitoring of which of the weekly monitoring for low platelet count (option 3) or anemia (option 4) while taking address this concern.
following? carbamazepine.
1.‐ Neuroleptic malignant syndrome
2.‐ Agranulocytosis
3.‐ Thrombocytopenia
4.‐ Anemia

3446 A client with schizophrenia has been taking Correct answer: 3 Akathisia is an inability to sit. This is the most common extrapyramidal side effect of The core issue of the question is correct identification of side effects of haloperidol. First,
haloperidol (Haldol) for three weeks with good effect. haloperidol. The side effects in option 1 would include dry mouth, urinary hesitance, eliminate options 2 and 4, because gustatory refers to taste and oculo‐ refers to eyes.
Today, he comes to group, but is complaining that he constipation, mydriasis, tachycardia, and diminished lacrimation. Option 2, gustatory Choose option 3 over option 1 because the word akathisia is consistent with the client s
feels like his legs are on fire. He is moving hallucination, is tasting something that is not present, while option 4 is a painful twisting and presentation, while the word anticholinergic is not.
continuously, and states, “I can t sit here anymore.” turning of the head and neck.
The nurse documents and reports that the client is
experiencing which of the following medication side
effects?
1.‐ Anticholinergic effects
2.‐ Gustatory hallucinations
3.‐ Akathisia
4.‐ Oculogyric crisis

3447 The nurse concludes the client understands the Correct answer: 3 Trazadone is an atypical antidepressant that is used more for insomnia than for depression. First, recall that trazodone is often used as a sleep aid. With this in mind, eliminate each of
desired effects and major side effects of trazodone Abuse potential is minimal, so option 4 is incorrect. Option 1, for safety reasons, is not a good the incorrect responses because they are not consistent with knowledge of its use for this
(Desyrel) when he makes which of the following practice when taking trazodone as a sleep aid, and option 2 is incorrect because taking more purpose.
statements? fluids will not increase the effectiveness of the medication.

1.‐ “I know I will be able to get up and go downstairs to the bathroom during the night as long as I leave a nightlight on.”
2.‐ “I am drinking more fluids now that I am taking this medication, so it will work the way it is supposed to.”
3.‐ “This medicine should help me sleep without my
having to worry about becoming addicted to it, and if I
have a problem with priapism, I will notify my doctor
immediately.”
4.‐ “I will feel more energetic after three or four weeks
of taking this medication, and I understand I must take
it only as prescribed so that I will not become addicted
to it.”

3448 The client is hospitalized because of a suicide attempt Correct answer: 3 The client needs to make an agreement with the nurse to remain safe, or to report to the Note that the question contains a key phrase: most important safety measure. This tells
while in a manic phase. The client has now been taking nurse if not feeling safe. Option 4 does not keep the client safe for 24 hours, only during meals. you that more than one option might be partially or totally correct, and that you must
chlordiazepoxide (Librium) for two days. Which of the The agreement in option 1 is too vague, and does not give specific responsibility to the nurse prioritize. Keep in mind that medication therapy alone is not sufficient in treating this client.
following is the most important safety measure for the or the client. The promise in option 2 is also vague, and does not make the client accountable Choose option 3 over the others because it is the most inclusive of the client s safety needs.
nurse to implement with this client? to the health care professionals.

1.‐ Frequently remind the client to remain visible to the nurses at all times.
2.‐ Elicit the client s promise to tell someone if she is feeling suicidal.
3.‐ Make a contract for safety.
4.‐ Enforce the client’s promise to eat all meals in the dining room.

3449 If an overdose of benzodiazepines is suspected, the Correct answer: 4 Flumazenil is the only drug available that acts as an antagonist to the benzodiazepines. Specific knowledge of the antidote to benzodiazepines is needed to answer this question.
nurse obtains which of the following medications to Options 1 and 2 are benzodiazepines themselves, while option 3 is a selective serotonin It might help to remember that the antidote to benzodiazepines is a generic drug name that
reverse that drug’s effects as ordered? reuptake inhibitor (SSRI) type of antidepressant. contains the letters ze, and is not a benzodiazepene itself.

1.‐ Diazepam (Valium)


2.‐ Triazolam (Halcion)
3.‐ Fluvoxamine (Luvox)
4.‐ Flumazenil (Romazicon)

3450 The nurse is making a plan of care for a client who is Correct answer: 3 Dry mouth occurs from the anticholinergic effects seen with fluphenazine. Options 1 and 2 First, eliminate options 1 and 2, because they are similar in referring to orthostatic
prescribed fluphenazine (Prolixin) 1 mg daily at are incorrect because orthostatic hypotension is not a major side effect of fluphenazine. hypotension. Choose option 3 over option 4 because option 3 addresses anticholinergic
bedtime. The nurse will include which of the following Confusion (option 4) is not a side effect of this agent. effects of the drug, which are of concern.
to monitor for side effects of the medication?

1.‐ Frequently remind the client to move slowly when getting out of bed or rising from a chair.
2.‐ Assess for dizziness or lightheadedness frequently during the day.
3.‐ Make sugarless hard candy, gum, and water available during the day.
4.‐ Monitor frequently for confusion.

3451 The client expresses an increase in appetite, a desire Correct answer: 1 Option 1 is the correct response to the situation. Option 2 is incorrect because imipramine is To answer this question correctly, recall that antidepressants generally take more than 5
to participate in group today, and a request to have very slow to become effective 2–6 weeks, not 5 days and as the client begins to “feel better,” it days for full effect. With this in mind, realize that the client is still a safety risk, and choose
her mother come to visit later in the week. The client is not appropriate to make major changes, especially after a suicide attempt. Options 3 and 4 the option that keeps the client safest while medication therapy is progressing.
has been in the inpatient locked unit on suicide are not true, but gradual change and monitoring for changes in mood and behavior are still
precautions for 5 days, and is taking imipramine very critical for this client.
(Tofranil). The nurse does not make major changes to
the nursing care plan at this time for which of the
following reasons?
1.‐ It is not safe after only 5 days to address too many changes at once. It is better to make gradual changes once the medication is more effective.
2.‐ Imipramine (Tofranil) is beginning to make the client feel less suicidal.
3.‐ After a suicide attempt, the likelihood of another attempt is rare, so it is better to let the client make adjustments, and then the nurse can change the care plan.
4.‐ The client is less likely to attempt suicide again if there is no change in the care plan or expectations of her.
3452 The client is receiving thioridazine (Mellaril) 100 mg Correct answer: 1 With thioridazine, the anticholinergic side effects of dry mouth, constipation, urinary Specific knowledge of the intended effects and side effects of this medication is needed to
t.i.d. Today, he comes to the clinic with the chief retention, and blurred vision are usually severe. Dry mouth is not associated with answer the question. Use the process of elimination to make your selection.
complaint of feeling like his mouth is “always dry.” The extrapyramidal side effects (option 2) or neuroleptic malignant syndrome (option 4). There is
nurse concludes this side effect is related to which of usually a weight gain, not a weight loss, as a side effect of thioridazine (option 3).
the following?
1.‐ High anticholinergic effects of thioridazine
2.‐ Extrapyramidal side effects (EPSE)
3.‐ Weight loss effect of the medication
4.‐ Neuroleptic malignant syndrome (NMS) side effect

3453 A client asks the nurse if it is true that marijuana is Correct answer: 1 Marijuana has been used for individuals with AIDS to increase their appetite. Marijuana does The core issue of the question is legitimate uses for marijuana. To aid in making the
not just a street drug but has legitimate uses, as he not increase organization and motivation (option 2), reduce stress (option 3), or have correct selection, keep in mind that an effect of the drug is to stimulate appetite.
read in a magazine article. The nurse replies that antibacterial properties (option 4).
marijuana does have therapeutic uses, such as the
ability to:
1.‐ Stimulate appetite.
2.‐ Stimulate clients to become more organized and motivated.
3.‐ Help clients to reduce stress.
4.‐ Act as an antibacterial agent.

3454 The nursing diagnosis is “Anxiety related to recent Correct answer: 3 Although options 1, 2, and 4 are all appropriate nursing interventions, the need for physical Note that the drug prescribed is an anxiolytic. Next, relate the cause of the anxiety to the
attack and robbery in apartment, evidenced by safety is the primary nursing priority for the client at this time. The others can be addressed need for the medication. From there, choose option 3 over the others because it targets
episodes of immobilizing apprehension.” A short‐term once physical safety is established. the best concern of the client and is congruent with the need for the medication.
anxiolytic has been prescribed. What is the primary
nursing priority for this client?

1.‐ Help client learn alternative responses to the anxiety.


2.‐ Promote involvement of family/client in group or community support activities.
3.‐ Provide for physical safety.
4.‐ Assist with desensitization to phobic place (apartment).

3455 The client is admitted to the inpatient unit with a Correct answer: 4 Risperidone has very few side effects; they include orthostatic hypotension (option 4) and The core issue of this question is recognition of dizziness as a sign of orthostatic
diagnosis of paranoid schizophrenia. He is prescribed insomnia, agitation, headache, anxiety, and rhinitis. Options 1, 2, and 3 are incorrect hypotension. With this concept in mind, eliminate each of the other options, because they
risperidone (Risperdal). After five days of treatment, conclusions about the causes of the client’s dizziness. do not relate to this concern.
the client reports feeling dizzy. The nurse explains to
the client that this is associated with:

1.‐ The desired effect of sedation.


2.‐ The side effect of loss of appetite.
3.‐ The anticholinergic side effects prominent with this agent.
4.‐ The side effect of orthostatic hypotension.

3456 Which of the following is the highest priority for the Correct answer: 3 The principle of remaining abstinent is one of the three most important goals of treatment The core issue of the question is the interactive effect of disulfiram and alcohol. With this
client after withdrawing from alcohol and beginning for alcoholism. It is also critical when taking disulfiram, in order to avoid adverse effects from in mind, eliminate each of the incorrect options because they do not address this critical
use of disulfiram (Antabuse)? the interaction of the medication and alcohol. The other two goals of treatment are concern.
amelioration of concurrent psychiatric conditions and long‐term prevention of relapse.
Options 1, 2, and 4 are important components, but without option 3, the others could not
occur, and they do not directly correlate with disulfiram therapy.

1.‐ Social reintegration


2.‐ Learning about the disease process
3.‐ Remaining abstinent
4.‐ Remaining in the rehabilitation unit
3457 The psychiatrist is prescribing chlorpromazine Correct answer: 2 Because the client is hospitalized and is receiving an IM dose of Thorazine, the primary Note the key word “initial,” and focus on the diagnosis of psychosis. Recall strong
(Thorazine) 50 mg IM as an initial dose for a client concern should be to monitor for a decrease in the psychosis. Blood pressure and pulse should knowledge base of disease process of psychosis and usual mode of treatment to select the
hospitalized with psychosis. Your initial concern is to be monitored as a general measure for initial treatment with Thorazine, whether IM or PO. correct answer. Using the process of elimination, omit choices 1, 3, and 4, as they are not
monitor: Ability to walk and to eat lunch are not significant to the issue of initial concern. the most important initial concerns on which to focus. If this question was difficult, review
the usual treatment medication protocol for psychosis.

1.‐ Blood pressure and pulse.


2.‐ A decrease in psychotic symptoms.
3.‐ The client's ability to walk.
4.‐ The client's ability to eat lunch.

3458 The home care nurse is visiting a client discharged Correct answer: 3 The only correct option is slurred speech and drowsiness. Olanzapine is a relatively new drug Recall strong knowledge of schizophrenia and usual treatment to select the correct
yesterday from an inpatient unit. The client is taking approved for schizophrenia and other psychotic disorders. This agent is generally well answer. If this was difficult, review the usual treatment medication protocol for
olanzapine (Zyprexa) 10 mg daily. The client states that tolerated, and appears devoid of serious adverse effects. schizophrenia.
he needs more medication, because what he was given
yesterday is all gone. He was given a 10‐day supply
upon discharge. The nurse would then assess for which
of the following signs?

1.‐ Headache and psychosis


2.‐ Lightheadedness and diarrhea
3.‐ Slurred speech and drowsiness
4.‐ Diarrhea and vomiting

3459 A female client reports during an initial interview that Correct answer: 1 The only correct answer is option 1. Trazodone is an atypical antidepressant used more often Use the process of elimination and knowledge of the treatment of insomnia to assist in
she has been prescribed to take trazodone (Desyrel). for insomnia than for depression. It is not used for panic attacks or anxiety. elimination of choices 2, 3, and 4, and to be led to the correct answer. If this was difficult,
The nurse questions her about a history of which of review insomnia medication treatment modalities.
the following problems?

1.‐ Insomnia
2.‐ Panic attacks
3.‐ Mania
4.‐ Anxiety

3460 A client is discharged taking a monoamine oxidase Correct answer: 3 The most important person to instruct is the client (not a family member, as indicated in Note the key words “most important.” Options 2 and 4 can be eliminated, as only the
inhibitor (MAOI). Which of the following are most option 4). With MAOIs, it is important to give the client not only oral but also complete written family is included in the answer, and not the client. Option 3 is the most complete answer
important client teaching objectives? instructions concerning medication administration, food interactions, etc. It is good to instruct because it includes all the components of teaching medications, while option 1 only
the client how to notify the appropriate health care professional, but it is not a major objective includes administration instructions. If this was difficult, review teaching components with
to teach the family how to contact an appropriate health care professional. medication administration.

1.‐ Give the client written and oral instructions on how to take daily doses of the medication.
2.‐ Instruct family members how to notify the appropriate health care professional after discharge.
3.‐ Give client written and oral instructions about medication administration, side effects, adverse effects, and food interactions.
4.‐ Instruct the client's family about the administration of medication.

3461 Keeping in mind that few benzodiazepines are safe Correct answer: 4 Diazepam and chlordiazepoxide are contraindicated for use with elderly clients (options 1 and Note that the question is asking about medication safety in the elderly. Understanding the
for use by the elderly, the nurse would anticipate that 2), while trazadone is an atypical antidepressant, not a benzodiazepine. Lorazepam is the only individualized physiologic adaptations of the elderly and medication prescription safety will
which of the following would be the most effective for appropriate benzodiazepine listed that is good for use by elderly clients. lead to the correct answer. If this was difficult, review safe medication use and the elderly.
use by a 74‐year‐old client?

1.‐ Diazepam (Valium)


2.‐ Chlordiazepoxide (Librium)
3.‐ Trazadone (Desyrel)
4.‐ Lorazepam (Ativan)

3462 The client with a bipolar disorder is being discharged Correct answer: 3 The primary reason for re‐hospitalization is that a client with bipolar disorder stops taking Note that the reason the client is placed on lithium is to control the bipolar disorder. By
today on lithium. Which of the following would be the medication (option 3). There will always be other problems in families and in life (options 1 using the process of elimination, omit answers that relate to problems in a client s life
most likely reason the client will be readmitted to the and 4), but these do not necessarily bring the client back to the hospital. If the client decides to requiring adaptation (choices 1 and 4) without hospitalization. A person losing weight can
inpatient unit? lose weight, this in itself does not indicate that the client will need to be hospitalized. be eliminated, since it is not life‐threatening. If this was difficult, review bipolar disorders
and medication treatment modalities.

1.‐ There will be a crisis in the client's family.


2.‐ The client will begin a diet regime to lose weight.
3.‐ The client will stop taking lithium as prescribed.
4.‐ The client's spouse will become seriously ill.

3463 The nurse is working in the Emergency Department Correct answer: 4 The nurse should ask if the client is taking disulfiram (option 4) because this medication Note the word “first” in the question stem. Recall knowledge of the side effect vomiting of
when a client is brought in vomiting profusely and causes the adverse reactions described in the question when alcohol is also ingested. Option 1 the medication disulfiram (Antabuse) to eliminate the other choices. The first part of the
smelling of alcohol. The client keeps repeating, "It's the is not highest priority, because the smell of alcohol indicates that the time of the last intake assessment is to determine if the client is taking medication that results in the symptoms. If
medicine that's doing it; if I live through this, I'll never was relatively recent. Options 2 and 3 are not the first questions that the client should be this was difficult, remember assessment is always the first step in the nursing process, and
drink again." Based on what the client is saying, you asked in this situation because they do not relate to the issue of vomiting and drug care begins with assessing the client first, whether subjectively or objectively.
begin your assessment by first asking the client: interactions.

1.‐ "When was the last time you drank alcohol?"


2.‐ "Are you taking antihypertensive medications?"
3.‐ "Have you eaten today?"
4.‐ "Are you taking Antabuse?"

3464 The client states, "Before I came to the hospital this Correct answer: 4 The nurse should refer the client back to the physician for clarification rather than try to Note that the question is asking for the “best” response. The question is asking why the
time, I used to take Tofranil for my depression. The second‐guess the physician's thoughts or speak for the physician. Option 1 is not appropriate physician changed the medication, and the only answer that would address this is option 4.
doctor said he thought it was time for me to change to because there is no basis for the statement. Option 2 is only one aspect of the difference Since the client would like to know why the medication was changed by the physician, the
a newer medicine, but I can't remember what he said between a tricyclic antidepressant and a selective serotonin reuptake inhibitor, and it does not best answer is for the client to speak to the physician. If this was difficult, review depression
about why this Zoloft would be better for me." The address the client's concern. Option 3 is accusatory, and therefore inappropriate. and medication treatment modalities.
nurse's best response to this client would be which of
the following?

1.‐ "Maybe the physician didn't think the Tofranil was working anymore."
2.‐ "I know there are fewer side effects with Zoloft than with Tofranil."
3.‐ "Did you always take the Tofranil as prescribed?"
4.‐ "Would you like me to get the physician so you can talk with him about this?"

3465 The client is visiting the clinic today after taking Correct answer: 3 Option 3 is correct. Clients taking fluoxetine usually demonstrate weight loss. The client needs Recall knowledge of the side effect weight loss with the medication fluoxetine (Prozac) to
fluoxetine (Prozac) 20 mg PO daily in the morning for to weigh himself daily and adjust nutritional intake as necessary. The client will need to select the correct answer, option 3. The other options are not appropriate, as they do not
three weeks. He is complaining of weight loss of 10 increase his caloric intake (option 2), not his fluid intake. The client does not necessarily need a address the client's concern of weight loss. If this was difficult, review the side effects of the
pounds in the last two weeks, but states he has not change in medication (option 1). Option 4 is incorrect; it will take 3–4 weeks for the effect of medication.
changed his diet at all. The nurse would draw which of the medication to be seen.
the following conclusions from this assessment data?

1.‐ The client will need to have a change in medication.


2.‐ The nurse will need to instruct the client on how to increase his daily fluid intake.
3.‐ This is a normal effect for clients taking fluoxetine.
4.‐ The physician needs to decrease the dosage of the medication.
3466 A client is experiencing recurrent hiccups. The nurse Correct answer: 3 Chlorpromazine (Thorazine) is not only the oldest of the antipsychotic medications; it can also Recall knowledge of treatment of hiccups and the use of the medication chlorpromazine
telephones the physician, anticipating an order for be used for relief of intractable hiccups. Risperidone (option 1), molindone (option 2), and (Thorazine) to be led to the correct answer. None of the other medications are used for the
which of the following phenothiazines that is also used thioridazine (option 4) do not have this effect. treatment of hiccups.
as an antipsychotic agent?
1.‐ Risperidone (Risperdal)
2.‐ Molindone (Moban)
3.‐ Chlorpromazine (Thorazine)
4.‐ Thioridazine (Mellaril)

3467 The client is taking haloperidol (Haldol) 2 mg t.i.d. The Correct answer: 2 Haloperidol is a high‐potency antipsychotic. Option 1 is not a classification of antipsychotics. Use the process of elimination and understanding of the classification of haloperidol
nurse plans to assess the client frequently for dystonia, Options 3 and 4 are valid classifications of antipsychotics, but they do not describe haloperidol. (Haldol) to select the correct answer. If this was difficult, review the classification of the
parkinsonism, and akathisia as adverse effects of this: medication listed.

1.‐ Relative‐potency antipsychotic.


2.‐ High‐potency antipsychotic.
3.‐ Medium‐potency antipsychotic.
4.‐ Low‐potency antipsychotic.

3468 A client taking bupropion (Wellbutrin) 100 mg twice Correct answer: 3 Dizziness, drowsiness, headache, and insomnia are some of the common CNS adverse effects Recall understanding of which medical diagnosis bupropion (Wellbutrin) is used for
daily for two weeks is returning to the clinic. As part of of bupropion. Decreased appetite (option 1) is not a concern. Option 2 is incorrect because it treatment to select the correct answer. The other choices do not address the side effects of
the nursing evaluation, you ask the client which of the asks about depression, while bupropion is used to treat anxiety. Option 4 indicates that the this medication. If this was difficult, review the treatment modalities for anxiety as well as
following questions regarding adverse effects of the client has had hallucinations, which are not associated with bupropion. the side effects of bupropion (Wellbutrin).
medication?
1.‐ "Have you experienced a decrease in appetite?"
2.‐ "Have you experienced a change from your depressed mood?"
3.‐ "Have you had any episodes of dizziness, drowsiness, headache, or insomnia?"
4.‐ "Are you still hearing voices?"

3469 A client with major depression is on suicide Correct answer: 3 The nurse should observe the same safety standards of medication administration as with all Recall understanding of the principles of safe medication administration to select the
precautions since being admitted two days ago. While clients. Crushing a medication and placing it in applesauce is not necessary for a client on correct answer. The question is asking for a general answer pertaining to safe medication
formulating the care plan, the nurse includes which of suicide precautions unless there is a problem with swallowing or taking tablets or capsules administration. Option 3 is the only choice that is a generalization related to safe
the following interventions pertaining to safe (option 1). Option 2 is incorrect because it is not the responsibility of the nursing assistant to medication administration. The other three options are specific to certain client types.
medication administration? remain with a client taking medications. As with all clients, staying with the client for five
minutes (option 4) is not necessary for safe medication administration.

1.‐ Always crush the medication and put it in applesauce.


2.‐ Ask the nursing assistant to watch the client drink all of the water given with the medication.
3.‐ Observe the same safety precautions of medication administration as for all clients.
4.‐ Remain with the client at least five minutes after medication administration.

3470 The client taking lithium carbonate (Eskalith) is having Correct answer: 4 The symptoms listed are those of lithium toxicity, and are seen when the serum level is 2–3 Recall knowledge of normal and toxic lithium levels to be led to the correct choice. There
a difficult time walking, and is confused, agitated, and mEq/L. The other options indicate lesser serum concentrations that would not produce these is only one choice that is correct and with knowledge of the laboratory value the correct
complaining of blurred vision. The nurse checks the manifestations. answer can be selected.
lithium level drawn earlier in the day, expecting the
level to be within which of the following ranges?

1.‐ 0.5–0.8 mEq/L


2.‐ 1.2–1.5 mEq/L
3.‐ 1.5–1.8 mEq/L
4.‐ 2.0–3.0 mEq/L
3471 The client is taking an antidepressant medication. She Correct answer: 2 In option 2, the client is demonstrating progress in returning to usual living. She is working a Use the process of elimination to select the correct answer. The only correct choice is
arrives at the clinic for a 6‐week follow‐up visit. The reasonable amount of the day, sleeping regularly, and making plans with others. In option 1, option 2, since this choice demonstrates progress toward positive activities of daily living.
nurse evaluates the medication as being effective if the the client is sleeping too many hours each day. She is reporting very little activity outside of The other options can be eliminated, since they do not reflect effective adaptation.
client: sleeping. She remains withdrawn, and demonstrates no change in mood or activities. Option 3
indicates that the client is overworking, sleeping only 6 hours each night, and still reporting
feelings of depression. Option 4 alone shows no signs of change.

1.‐ Reports going back to work 2 hours a day and sleeping 12 hours each night and 3 hours after coming home from her office job.
2.‐ Is talking about vacation plans for the following
month, reports sleeping 9 hours each night, working a
full‐time job, and feeling less tired and anxious than
she was two weeks ago.
3.‐ Reports working a full‐time job and being able to
work at least 3 or 4 hours a week overtime, sleeping 6
hours each night, and only feeling depressed after
working 10 hoursa day.
4.‐ Is complaining of weight gain, but obtains 3 extra hours of sleep per night.

3472 A nurse has admitted a client who is undergoing Correct answer: 2 Clients undergoing withdrawal from heroin exhibit craving, lacrimation, rhinorrhea, yawning, Use the process of elimination to omit incorrect choices. By having knowledge of the side
withdrawal from heroin. The nurse expects the client and diaphoresis. Option 1 is incorrect because irritability and insomnia are seen with effects of heroin withdrawal, option 2 can be selected as the correct answer. Options 1, 3,
to exhibit which of the following during the initial withdrawal from marijuana. The manifestations listed in options 3 and 4 pertain to withdrawal and 4 can be eliminated, since they are not withdrawal symptoms of heroin.
phase of withdrawal? from alcohol.
1.‐ Irritability and insomnia
2.‐ Drug craving, lacrimation, rhinorrhea, yawning, and diaphoresis
3.‐ Tremors, agitation, anxiety, and diaphoresis
4.‐ Hallucinations, delusions, and increases in blood pressure and pulse

3473 An adult client has been taking alprazolam (Xanax) 2 Correct answer: 3 Option 1 is true for the symptoms of dry mouth, but is not the priority response because it Note that the question is asking for the priority response. Remember Maslow s hierarchy
mg for generalized anxiety for the last two weeks. does not fully address the information provided by the client. Option 2 is incorrect. The client of needs, and answer this in consideration of safety and security, which are of high priority
Today, he is in the clinic, and states, "I can't believe is describing expected side effects of the medication ordered to decrease anxiety, so the in the hierarchy. While option 1 does address a nursing intervention related to relief of dry
how dry my mouth is since I have been taking this dosage should not be changed. Option 3 is the priority because of the need for safety when mouth it is not a priority. Option 3 is the priority response, since the client is asking the
stuff. And sometimes I get so dizzy and lightheaded, using a benzodiazepine, given the common side effects the client described. Option 4 will serve nurse to validate if dizziness and lightheadedness are acceptable. This answer will respond
but otherwise it works great. I don't feel all anxious no benefit to treat or help with the described side effects. to his question, which is the priority.
anymore." The priority response by the nurse will be
which of the following?

1.‐ "You can use gum or hard candy or sugarless gum to relieve some of those symptoms."
2.‐ "You will need to have the dosage of your medication lowered today so that you will not experience the side effects you are describing."

3.‐ "You feel dizzy and lightheaded, which is a side


effect of the medication. Don't participate in activities
that require you to be alert or operate heavy
equipment, which could be a safety hazard, until you
no longer have these side effects or are not taking the
medication anymore."
4.‐ "You will need to take this medication with food from now on."

3474 A client is beginning treatment with an Correct answer: 3 TCAs account for 70 percent of all deaths from intentional drug overdose. SSRIs are not Use the process of elimination and knowledge of classifications of antidepressant
antidepressant medication. The nurse will include usually fatal if overdose is taken. MAOIs can be fatal if the client experiences a hypertensive medications to select the correct answer. In choosing this answer, it is important to know
appropriate teaching strategies and precautions in the crisis, but MAOIs are not usually that widely prescribed because of their numerous side effects, general categorizations of antidepressants.
care plan, knowing that there is a high risk for especially with tyramine‐rich foods and drug–drug interactions. Anxiolytics are not a class of
successful suicide with medications in which of the antidepressants.
following antidepressant classes?
1.‐ Selective serotonin reuptake inhibitors (SSRIs)
2.‐ Monoamine oxidase inhibitors (MAOIs)
3.‐ Tricyclic antidepressants (TCAs)
4.‐ Anxiolytics

3475 A 46‐year‐old client newly diagnosed with Correct answer: 1 It is essential to teach both the client and his mother about the medication and why This question asks for means for the client to be compliant with taking medication. While
schizophrenia is being discharged to home in five days. compliance is very important. Option 2 is incorrect because it places responsibility for eating it is important that the client understand the way to take the medication, it is also
The client lives in a two‐bedroom apartment with his and taking medications on the mother, not on the client, which is inappropriate. Option 3 also important to involve a significant other for support. The only option that addresses the
elderly mother, who is frail but self‐sufficient. The is incorrect because it is the client's responsibility to take prescribed medications. Family client and a significant other is option 1. Options 2 and 3 place the responsibility on the
nursing care plan must include which of the following members should know about the medications and be able to support the client and remind the mother and not the client, while option 4 gives the client false assurance that he will be
to promote compliance with medication client of the benefits of taking the prescribed medication, but it is ultimately the client's symptom‐free.
administration? responsibility. Option 4 is inappropriate because there is no guarantee for the client that he
will remain symptom‐free indefinitely.

1.‐ Teaching the client and his mother about the prescribed medication and why compliance is so important to the client's ongoing recovery process
2.‐ Teaching the client's mother the importance of regular meals so the client can take his medication after breakfast
3.‐ Instructing the mother to be sure that the client is taking his medication daily as prescribed
4.‐ Teaching the client about his medication regime and how taking his medication as prescribed will help him remain symptom‐free indefinitely

3476 An adult client is in the clinic today for a follow‐up Correct answer: 2 Option 1 is a piece of subjective data only, and should not be the sole information for the When evaluating the effects of medication, it is important to assess using subjective and
two‐week visit. Diazepam (Valium) 10 mg was nurses' evaluation of the effects of the anxiolytic medication. Option 2 is more objective, and objective data, but to place a higher emphasis on the objective data.
prescribed during the client's last visit. In order to uses previous data and present facts to evaluate the client's condition. Option 3 is false; there
evaluate that there are positive effects from the should be a change observed with an anxiolytic after two weeks; and option 4 does not
medication, the nurse would: address the primary effect desired for use of anxiolytics (decrease of the anxiety).

1.‐ Hear the client saying that she is feeling much calmer now.
2.‐ See a change from the very anxious person documented at the last visit to a calmer and more focused person at present.
3.‐ Notice nothing; there are probably no observable changes in the client's behavior after only two weeks.
4.‐ Hear the client describe her increased appetite and how much more she is able to do since taking the medication.

3477 The client will be discharged to home tomorrow on Correct answer: 3 Option 3 is correct because it acknowledges the client's feelings and addresses his concerns The question is asking for the best response. Option 3 is the only choice that acknowledges
an antidepressant medication that will be taken once while still allowing him to make decisions for his present and future. Options 1 and 2 disregard that the client has specific concerns related to becoming more independent and about
daily in the morning. He asks: "Do I have to take and negate the client's feelings. Option 4 acknowledges his concern but takes away his decision‐ reentry into society. Options 1 and 2 do not acknowledge that the client has concerns.
medicine every day? How will I be able to sleep when I making options by having someone else (the nurse) make a plan for his daily activities, rather Option 4 does not allow the client to move into society, but rather makes the nurse the
go home? Do you think I'll be able to work, too, even than have him participate and make decisions for himself with help. person on whom he must depend.
though I have been in the hospital this long?" The
nurse's best response is:

1.‐ "The best approach is to take it one step at a time, so that everything will work out."
2.‐ "I understand you're worried, but you and your wife will decide tomorrow when you get home."
3.‐ "You seem to be worried about when you get home and how you will function. Would you like to sit and discuss a plan for your daily activities?"
4.‐ "I'll do my best to set up a plan for discharge that you can take home with you and refer to later."

3478 The nurse is making a plan of care for a client who is Correct answer: 3 Dry mouth occurs from the anticholinergic effects seen with fluphenazine. Options 1 and 2 Recall knowledge of the side effects and client responses of fluphenazine (Prolixin) to
prescribed fluphenazine (Prolixin) 1 mg daily at are incorrect because orthostatic hypotension is not a major side effect of fluphenazine. select the correct answer. The only correct answer is option 3.
bedtime. The nurse will do which of the following to Confusion (option 4) is not a side effect of this agent.
monitor for side effects of the medication?

1.‐ Remind him frequently to rise slowly when getting out of bed or from a chair.
2.‐ Assess for dizziness or lightheadedness frequently during the day.
3.‐ Make sugarless hard candy, gum, and water available during the day.
4.‐ Monitor for confusion frequently.
3479 The school nurse is assessing a muscular 17‐year‐old Correct answer: 2 The symptoms of hair loss, the student's age, and edema indicate that this is not a stage of Note that the question is asking for selection of the primary analysis of data. Recall
female who is coming to the high school health service puberty. The symptoms are not indicated in abuse of barbiturates or marijuana use. By the knowledge of times of pubescence onset for females to eliminate option 1. Options 3 and 4
for complaints of edema, voice changes, and hair loss. process of elimination, the correct answer is option 2. In order to answer this correctly, you would not produce the physiologic changes mentioned in the question. The only correct
Your primary analysis based on the subjective and need to have noted the muscular build of the student and to know the signs and symptoms of answer is option 2, as the question stem lists symptoms of illegal steroid use.
objective data is that the student: illegal steroid use.

1.‐ Is going through a stage of puberty.


2.‐ Might be using steroids.
3.‐ Might be abusing barbiturates.
4.‐ Is using marijuana regularly.

3480 The nurse is preparing a client for discharge who will Correct answer: 4 Option 4 is correct because the client is honest, has an understanding of how to take the The question is asking for the option that demonstrates that the client is comfortable with
be taking lithium carbonate. Which of the following medication and what the side effects are, and knows that the side effect will subside the plan of care of the medication prescription. The only correct option is 4, as this
statements indicates that the client is feeling eventually. Options 1 and 2 indicate that the client is feeling forced to take the medication but addresses the client s understanding of the medication and the need to be compliant with
comfortable with being discharged on an antimanic has no desire or understanding of the benefits of the daily routine and dosages. Option 3 the interventions needed for the plan of care.
medication? indicates that the client has memorized the actions but does not understand the benefits or
side effects of the medications.
1.‐ "I don't want to take the medicine you will give me, but you said I have to."
2.‐ "I know that if I take my lithium every day, I won't have to come to the hospital again."

3.‐ "I have a hard time taking this medicine, and I don't
like the shaking, but I will take it every day with meals,
and have my blood tests done, and come back to the
clinic next month for my checkup like you said."

4.‐ "Even though I don't like taking medicine, I will take


the lithium daily with my meals and have my blood
tests on the dates I marked on my calendar. I should
be able to do my normal things every day, and in a
couple of weeks I won't feel shaky anymore."

3481 After taking chlordiazepoxide (Librium) 50 mg in the Correct answer: 3 Option 3 is correct because it addresses the client's concern and addresses his issue. Option 1 When answering this question, it is imperative that the client s concerns are
detoxification unit, the client states: "I'm not sure I denies the client's feelings and does not address his concern. Option 2 is disrespectful, and acknowledged. Option 3 is the only answer that acknowledges the feelings of the client.
should be taking this stuff. I'm already addicted to again does not acknowledge the client's feelings or concerns. Option 4 is incorrect because it is When answering questions in which a client has concerns, select the answer in which the
alcohol, and I don't want to be addicted to this stuff, disrespectful, denies the client's feelings, and does not address the client's issue. feelings of the client are acknowledged.
too." Which of the following would be the nurse's best
response?

1.‐ "You will not become addicted to chlordiazepoxide,


because you are only using it to help you avoid the side
effects of withdrawing from alcohol and any other
drugs you have taken."
2.‐ "You do not have to worry about that. We won't let
that happen to you while you are here in the
detoxification unit. The dosages are prescribed as part
of protocol orders, and therefore they are safe for just
about everyone."
3.‐ "You seem to be worried about the addiction
potential of chlordiazepoxide. This medication helps
you avoid the effects of alcohol withdrawal. It is given
in predetermined doses so you won't become more ill
but will be safe and eventually drug‐free. That is why
you are being monitored medically until you are safely
withdrawn from the alcohol and the
chlordiazepoxide."
4.‐ "The physician has ordered the right doses of the
medication so that you won't become addicted. This
should enable you to relax and not be concerned
about this any longer."

3482 A client is admitted to the locked unit because of Correct answer: 3 There is a high potential risk for NMS with the use of haloperidol. Monitoring for tardive Use the process of elimination and knowledge of side effects of haloperidol to select the
frequent auditory hallucinations. Voices are telling the dyskinesia is not indicated this early in treatment. There is no reason to monitor for intake and correct answer. Option 3 is the only correct answer. If this was difficult, review the side
client to harm himself. The client is being given output (option 2). Option 4 is unnecessary, although the nurse monitors mood, behavior, and effects of haloperidol.
haloperidol IM. The nurse will include in the plan of orientation during therapy.
care which of the following primary interventions to
provide a safe environment for the client?

1.‐ Monitor for tardive dyskinesia (TD).


2.‐ Monitor intake and output every two hours.
3.‐ Monitor for neuroleptic malignant syndrome (NMS).
4.‐ Assess alertness every 15 minutes.

3483 The nurse explains to the family of a client Correct answer: 4 It is part of standard nursing practice to evaluate the effectiveness of medications that are The question is asking for utilization of the last step of the nursing process, which is
hospitalized for schizophrenia that the effects of administered. Option 1 is inaccurate because antipsychotic medications are not addictive. evaluation. Use the process of elimination to select the only correct answer, option 4, as it
antipsychotic medication will need to be evaluated Options 2 and 3 have nothing to do with the effects of an antipsychotic medication. evaluates the effectiveness of the medication, which is what the question is asking. If this
primarily to: was difficult, be sure to note what part of the nursing process the question is asking about
and to select an appropriate answer that is congruent with the step of the nursing process.

1.‐ Prevent the client from becoming addicted to the medication.


2.‐ Continually encourage the client to increase his self‐care capability.
3.‐ Keep the client from isolating.
4.‐ Assess for effectiveness of the medication.

3484 The nurse would assess which of the following as the Correct answer: 3 Measuring the blood alcohol level (option 3) is the most accurate test to indicate intoxication Note that the question is asking for selection of the most accurate level of intoxication.
most accurate measure of the client's level of level. Testing urine for alcohol level (option 1) is not an accurate measure for alcohol. Testing Option 3 would be the correct choice, as this is the most accurate level of evaluation of
intoxication? MCV and GGT (options 2 and 4) will indicate if the individual has been using alcohol chronically. current intoxication. If this was difficult, be certain to pay close attention to the words
listed as most, first, or highest priority, and select the option that is most conclusive.

1.‐ A urine test for alcohol


2.‐ Mean cell volume (MCV)
3.‐ A blood alcohol level
4.‐ Gamma‐glutamyl transpeptidase (GGT)

3485 The nurse determines that the client understands the Correct answer: 2 The medication normally works within ½–1 hour after administration, making option 2 Use the process of elimination and knowledge of client responses to flurazepam
effects of flurazepam (Dalmane) ordered at a dose of correct. Option 1 is incorrect because the client should not be watching stimulating shows on (Dalmane) to select the correct answer. Option 2 is the only correct answer, as it addresses
30 mg by which of the following client statements? TV before trying to fall asleep. Option 3 is incorrect because the medication will not work client responses. The other options do not address client responses but more actions after
instantly. Option 4 is incorrect because the client should not take a sedative and then stay the medication is taken. If this was difficult, review the medication and responses to the
active for ½–1 hour after taking medication. medication.
1.‐ "After I take my medication at bedtime, I should be able to watch the boxing match or late‐night TV show, then go to bed and sleep."
2.‐ "Once I take my medicine, I should be able to go to bed and read, and I will fall asleep within one hour."
3.‐ "I will take my medicine, go to bed, and go to sleep."
4.‐ "I will take my medicine, make my lunch for tomorrow, take my shower, and get my clothes ready for work tomorrow, and then go to bed."

3486 The visiting nurse is evaluating for client safety. The Correct answer: 3 With an MAO inhibitor such as phenelzine, the client needs to eliminate foods that contain All the options should be included in a teaching plan, but the only option that addresses
client is taking phenelzine (Nardil). A priority part of tyramine. Intake of tyramine‐containing foods could lead to severe hypertension and other safety with this medication is option 3. If this was difficult, review the side effects of the
the nurse's teaching component includes which of the complications. All of the other considerations are not major teaching considerations for MAO medication.
following? inhibitors.
1.‐ Limiting daily intake of salt
2.‐ Encouraging a fluid intake of at least 2,000 mL
3.‐ Eliminating foods containing tyramine
4.‐ Encouraging the client to have scheduled blood tests on time

3487 The nurse should anticipate that which of the Correct answer: 1 Methergine provides long‐sustained contraction of the uterus. It is commonly used to treat Specific knowledge of ergonovine maleate (Methergine) is needed to answer this question.
following would be included in the therapeutic plan of late postpartum hemorrhage (subinvolution). Oxytocin (option 2) and prostaglandin are more Use medication knowledge and the process of elimination to make your selection.
care for a postpartum client with subinvolution? frequently used to treat early postpartum hemorrhage caused by uterine atony. When blood
products are used (option 4), they are generally ordered for early postpartum hemorrhage.
Increased fluid intake (option 3) is a general, helpful measure for any client who has lost body
fluid volume, but it is not a specific therapy.

1.‐ Oral methylergonovine maleate (Methergine)


2.‐ Oxytocin (Pitocin) IV infusion for eight hours
3.‐ Oral fluids to 3,000 mL per day
4.‐ Blood replacement

3488 In which postpartum client would the nurse conclude Correct answer: 3 Methergine has a side effect of raising the blood pressure. A woman with hypertension or Specific knowledge of ergonovine maleate (Methergine) is needed to answer this question.
that methylergonovine maleate (Methergine) be pregnancy‐induced hypertension would not be a good candidate for use of Methergine. An Use nursing knowledge and the process of elimination to make your selection.
contraindicated? alternative would be necessary. The client in option 1 has a normal blood pressure, which is
not a contraindication. A pulse of 60 (option 2) or respiratory rate of 12 (option 4) are not
contraindications to use of Methergine.
1.‐ A client with a blood pressure of 120/60
2.‐ A client with a heart rate of 60
3.‐ A client with a blood pressure of 140/100
4.‐ A client with a respiratory rate of 12

3489 A postpartum client has an epidural catheter in place Correct answer: 4 Naloxone is the antidote to the opioid analgesics that are used with epidural analgesia. If The core issue of the question is a priority medication to have on hand during epidural
following delivery of an infant via cesarean section. respiratory depression occurs, this medication needs to be readily available for use. analgesia. Use the process of elimination to select the antidote needed for respiratory
The nurse determines that which of the following Meperidine is an opioid analgesic, but is not used for epidural analgesia. Betamethasone is a depression, a priority adverse effect of epidural analgesia.
medications is a priority to have on hand for use if glucocorticoid used to enhance fetal lung maturity before premature delivery. Carboprost is an
needed? abortifacient.
1.‐ Meperidine hydrochloride (Demerol)
2.‐ Betamethasone (Celestone)
3.‐ Carboprost (Hemabate)
4.‐ Naloxone (Narcan)

3490 The nurse is monitoring a client in labor who is Correct answer: 1 Contractions lasting longer than 90 seconds indicate uterine hyperstimulation, which is a The core issues of the question are knowledge of adverse effects of oxytocin and how to
receiving oxytocin (Pitocin) as a continuous infusion to reason to stop the oxytocin infusion. The increase in blood pressure is not of concern. Early recognize them in the woman in labor. Use the process of elimination and knowledge of
augment labor. Which of the following observations of decelerations of fetal heart rate do not indicate fetal distress; rather, they are a reassuring adverse drug effects and uterine hyperstimulation to make a selection.
the client would indicate to the nurse that the infusion sign. Squeezing the eyes shut during contractions could have variable meanings, including
needs to be stopped? coping with the contraction, and needs to be correlated with other client data for proper
interpretation.
1.‐ Contractions lasting 120 seconds
2.‐ Maternal blood pressure increase from 124/82 to 130/86
3.‐ Early fetal heart rate decelerations on the fetal monitor
4.‐ The mother squeezing her eyes shut during each contraction

3491 The nurse is evaluating the status of a pregnant client Correct answer: 4 The danger of pre‐eclampsia is that it can progress to eclampsia, characterized by seizure The core issue of the question is the action of magnesium sulfate in a client with pre‐
receiving magnesium sulfate. The nurse concludes that activity. Magnesium sulfate is given to prevent seizures. It is not given to stabilize BP, although eclampsia. Use drug knowledge and the process of elimination to make a selection.
the medication is having the intended effect if which of it can cause a transient decline in BP. It is not given to regulate the magnesium level or uterine
the following is noted? contractions.

1.‐ BP has stabilized at 128/76.


2.‐ Serum magnesium level reaches 2.2 mEq/L.
3.‐ Contractions are steady at a frequency of every four minutes.
4.‐ There is an absence of seizure activity.

3492 The nurse notes that the client is Rh‐negative and her Correct answer: 3 An indirect Coombs' test assesses for the presence of Rh antibodies in the maternal blood. The core issue of the question is the laboratory indicator that signals the need for
baby is Rh‐positive. Which maternal laboratory result Direct Coombs' test and bilirubin tests are conducted on the newborn. Hemoglobin is not a administration of RhoGAM. Specific knowledge of this drug is needed to answer this
would be important to interpret next in determining if determinant for the administration of RhoGAM. question. Use the process of elimination.
the client is a candidate for RhoGAM?

1.‐ Hemoglobin level


2.‐ Direct Coombs' test
3.‐ Indirect Coombs' test
4.‐ Bilirubin level

3493 In addition to routine assessment and care, nursing Correct answer: 3 Terbutaline, a beta‐adrenergic agent, has many maternal and fetal side effects, including Use the process of elimination. The core issue of the question is knowledge that
care of the client who is receiving terbutaline tachycardia, cardiac arryhthmias, and pulmonary edema. In addition to taking routine vital terbutaline is a beta‐adrenergic drug that can lead to adverse effects, including pulmonary
(Brethine) to prevent premature labor should include signs, the nurse should assess for pulmonary edema. The frequency of assessment of fetal edema. Use the ABCs to help focus on breathing and respiratory assessment.
assessing which of the following as an indicator of heart tones and oral temperature depends on the intensity and length of the drug therapy, as
adverse drug effects? well as surrounding circumstances. Deep‐tendon reflex assessment is not indicated.

1.‐ Oral temperature every two hours


2.‐ Fetal heart tones every 30 minutes
3.‐ Breath sounds every four hours
4.‐ Deep tendon reflexes every four hours

3494 A client in premature labor is scheduled to receive a Correct answer: 3 Corticosteroids such as betamethasone have been shown to enhance fetal lung maturity and Specific medication knowledge is needed to answer the question. Recall that a drug ending
dose of betamethasone (Celestone). In teaching the prevent respiratory distress. Betamethasone does not stop labor or cervical changes. A side in ‐sone is likely to be a steroid, and this hastens lung maturity in the fetus at risk for
client about this medication, the nurse would explain effect is increased risk of infection. premature delivery.
that the purpose of the medication is to do which of
the following?
1.‐ Stop uterine contractions.
2.‐ Prevent infection.
3.‐ Hasten fetal lung maturity.
4.‐ Prevent cervical dilatation.

3495 The nurse is preparing to administer an intramuscular Correct answer: 2 The best explanation is the one that explains the use of phytonadione. The medication is Note the critical word best in the stem of the question, which tells you that more than one
injection of phytonadione (AquaMEPHYTON) to a given to supply vitamin K, which the newborn cannot produce in the early days of life because or all answers might be factually correct. Note also the critical word healthy, which
healthy newborn. Which of the following is the best of lack of the intestinal flora needed to synthesize it. Although phytonadione does treat eliminates option 1. Eliminate options 3 and 4 because they contain inaccurate information.
explanation for the nurse to give the neonate’s hemorrhagic disease of the newborn, its use in the healthy infant is prophylactic. The
mother? medication is not water‐soluble, nor is it a multivitamin.

1.‐ “This medication will treat hemorrhagic disease of the newborn.”


2.‐ “This medication supplies vitamin K, which the newborn cannot produce in the first 5–8 days of life.”

3.‐ “This medication is a multivitamin that has many effects, including helping to produce prothrombin in the blood.”

4.‐ “This medication is also known as vitamin K, and it is a water‐soluble vitamin that is deficient in newborns.”

3496 The nurse caring for a newborn 30 minutes after birth Correct answer: 2 The nurse would give the ophthalmic dose by applying a 0.5–1 cm ribbon of ointment into Use the process of elimination, keeping in mind principles of aseptic technique and
would do which of the following when preparing to each lower conjunctival sac. The dose can be delayed up to an hour after birth, but not two standard procedure for administration of eye medications.
give a prescribed dose of ophthalmic erythromycin? hours. The eyes are not cleansed or irrigated after the dose, and a new tube is used for each
newborn.
1.‐ Withhold the dose for two hours to allow for parent–infant bonding.
2.‐ Administer the dose into each lower conjunctival sac.
3.‐ Irrigate the eyes after the dose to flush out microorganisms.
4.‐ Use a tube of ointment from a previous birth that has only been open two hours.

3497 A female client comes for her 24‐week prenatal visit. Correct answer: 4 German measles is also termed rubella. Pregnant women are tested at their first prenatal Look for key differences in the answers. Here we have rubella and Rh isoimmunization,
The nurse midwife tells her, “Your blood tests reveal visit for immunity to rubella. If the client is found to be nonimmune, immunization will be both of which are tested during pregnancy. Rh isoimmunization is prevented by giving
that you do not show immunity to the German given after delivery, before discharge. RhoGAM at 28 weeks and again after delivery if the baby is found to be Rh‐positive,
measles.” Which notation will the nurse include in the Coombs'‐negative. Rubella vaccine is not given during pregnancy.
plan of care for the client? “Client will need:

1.‐ "Rh‐immune globulin at the next visit.”


2.‐ "Rh‐immune globulin within two days of delivery.”
3.‐ "Rubella vaccine at the next visit.”
4.‐ "Rubella vaccine after delivery on the day of discharge.”

3498 A client who is breastfeeding her newborn is to be Correct answer: 1 The rubella vaccine is prepared with a live virus; therefore, it is not appropriate to administer Knowledge of immunizations is critical to planning care for clients. Rubella vaccine is a live‐
discharged from the postpartum unit. She has been during pregnancy. Clients are counseled to avoid pregnancy for three months after virus vaccine, and therefore pregnancy should be avoided while immunity is formed.
found to have no immunity to rubella, and has orders immunization.
to receive rubella vaccine on the day of discharge.
What is the most important instruction for the nurse
to include in the discharge plan?

1.‐ Practice contraception and avoid conception for at least 2–3 months.
2.‐ Discontinue breastfeeding to prevent the infant from becoming infected with the rubella virus.
3.‐ Avoid contact with women who are pregnant or who suspect they might be pregnant.
4.‐ Have the infant screened for active rubella virus at the 2‐month checkup.

3499 A primigravida with blood type A‐negative is at 28 Correct answer: 1 This client statement indicates that she does not understand the fundamental indications for Mapping out the case management of a client with Rh‐negative blood is helpful in
weeks gestation. Today, her physician has ordered a treatment of this potential blood incompatibility. If an Rh‐negative client is carrying an Rh‐ choosing potential interventions throughout pregnancy.
RhoGAM injection. Which statement by the client positive infant, the potential for mixing of fetal blood into the maternal system could occur at
demonstrates that more teaching is needed related to midpregnancy and again at delivery of the placenta. If the infant is found to be Rh‐positive, the
this therapy? client will be given RhoGAM within 72 hours of delivery to block any antigen–antibody
formation.
1.‐ “I’m getting this shot so that my baby won’t develop antibodies against my blood, right?”
2.‐ “I understand that if my baby is Rh‐positive, I ll be getting another one of these injections.”
3.‐ “This shot will prevent me from becoming sensitized to Rh‐positive blood.”
4.‐ “This shot should help to protect me in future pregnancies if this baby is Rh‐positive, like my husband.”
3500 Aerosol therapy is an important part of the therapy Correct answer: 3 Aerosol medications are delivered via a liquid mist, which delivers medication to the lower Remember the underlying principle of respiratory care entailing postural drainage, mist
for a pregnant woman with an exacerbation of asthma. respiratory tract. Postural drainage would be done if indicated. The droplets need to be small, oxygen that delivers medication to the lower respiratory tract, and oral medications to
Which of the following factors is most important for not large. Drugs are always administered during pregnancy after evaluating both the benefit to decrease viscous secretions. The pregnant client actually breathes in more volume than the
the nurse to consider in delivering aerosol medication? the mother and the risks to the fetus. nonpregnant client. The nurse would be wise to monitor the fetal effects of the medications
given to the mother; in this case, beta‐adrenergic agents cause fetal tachycardia.

1.‐ It is used instead of postural drainage.


2.‐ The particles of moisture produced must be large enough to dilate the bronchioles.
3.‐ The aerosol delivers medication to the lower respiratory tract.
4.‐ Unlike with many pulmonary diseases, medications administered through aerosol therapy are contraindicated in pregnancy.

3501 The nurse explains to a new nurse orientee that Correct answer: 3 The neonate intestinal tract is sterile at birth. Colonization of bacteria in the gut necessary for Vitamin K is critical to normal clotting; careful attention to the subtle differences in the
phytonadione (AquaMEPHYTON) needs to be vitamin K synthesis takes approximately a week to occur. The other options listed contain available answers will support better scores.
administered to the neonate for which of the following incorrect rationales.
reasons?
1.‐ It prevents gonorrhea, and it is a state law.
2.‐ It inhibits the production of prothrombin by the liver.
3.‐ The neonate lacks the intestinal flora for vitamin K production.
4.‐ The neonate cannot synthesize phytonadione.

3502 A pregnant client is receiving magnesium sulfate. The Correct answer: 2 Magnesium sulfate is a CNS depressant; therefore, disappearance of the patellar or knee‐jerk Remember your CNS assessments and that the patellar reflex is a specific indicator of CNS
nurse evaluates which of the following as a sign of reflex would indicate serious CNS depression. The other options do not indicate adverse integrity. In a client receiving magnesium sulfate, you would expect the patellar reflex to be
excessive blood levels of the drug? effects of the medication. diminished but not absent.

1.‐ Development of seizures


2.‐ Disappearance of the knee‐jerk reflex
3.‐ Increase in respiratory rate
4.‐ Increase in blood pressure

3503 After receiving magnesium sulfate, a client develops Correct answer: 4 The antidote for magnesium sulfate is calcium gluconate. The other drugs listed are not. Calcium gluconate is an antidote for excessive magnesium sulfate, and safe practice
signs of toxicity. The nurse should be prepared to indicates that this drug should be available at the bedside.
administer which of the following?
1.‐ Oxygen
2.‐ Epinephrine
3.‐ Potassium chloride
4.‐ Calcium gluconate

3504 Before administering IV magnesium sulfate therapy to Correct answer: 3 Excretion of magnesium sulfate is primarily accomplished through the renal system. Critical The key to correctly answering this question is to focus on indications for stopping the
a client with pregnancy‐induced hypertension, the assessments prior to administration of the drug would be focused on the body's ability to drug; if these signs are present prior to administration, they must be reported to the
nurse would assess which of the following parameters excrete the medication and the status of the CNS. Both assessments should be within normal prescriber.
that have highest priority? limits, or the prescribing health care provider should be notified.

1.‐ Urinary glucose, acetone, and specific gravity


2.‐ Temperature, blood pressure, and respirations
3.‐ Urinary output, respirations, and patellar reflexes
4.‐ Level of consciousness, funduscopic appearance, and knee reflex

3505 During the administration of magnesium sulfate to Correct answer: 2 Magnesium sulfate is an anticonvulsant medication given to pregnant women with pre‐ Remember that CNS depressants should diminish reflex activity, not stop it altogether, or
the client with pre‐eclampsia, the nurse would observe eclampsia to diminish the risk of convulsions. The drug is a CNS depressant and therefore acts the client will cease respiratory and cardiac function.
for which of the following toxic effects of the drug? to reduce central nervous system activity. CNS activity should not be absent.

1.‐ Dry, pale skin


2.‐ Hyporeflexia
3.‐ Agitation
4.‐ Increased respirations

3506 A client in active labor is to have an epidural block. Correct answer: 4 Epidural medications cause vasodilatation, which can lead to hypotension. This is the primary Remember that many local anesthetics cause vasodilatation.
While this is being administered, which of the risk factor the nurse needs to monitor after placement. Other considerations can be
following the nursing actions takes priority? considered once the client's ABCs are stable.
1.‐ Checking the uterine contractions for an increase in strength
2.‐ Positioning the mother flat in bed to avoid postspinal headache
3.‐ Telling the mother she will feel the need to void more frequently
4.‐ Monitoring the maternal blood pressure for possible hypotension

3507 While a client is receiving magnesium sulfate for Correct answer: 3, 4, 5 The most critical incident that could occur in a client receiving magnesium sulfate is toxic CNS The critical word in the stem of the question is appropriate, which tells you that the
severe pre‐eclampsia, the nurse would carry out which depression, which could affect respiratory and cardiac function. Therefore, the antidote should correct options are also correct interventions. Use knowledge of magnesium sulfate and
of the following appropriate nursing interventions? be available at the bedside. The nurse should assess for patellar reflexes to detect excessive the process of elimination to make a selection.
Select all that apply. dosing. It is also important to keep the room quiet. It is not necessary to prepare for
precipitous birth (option 2) or severely limit fluid intake (option 1).

1.‐ Limit fluid intake to 1000 mL/24 hours.


2.‐ Prepare for the possibility of a precipitate delivery.
3.‐ Restrict visitors and keep the room darkened and quiet.
4.‐ Obtain calcium gluconate for use as an antagonist if necessary.
5.‐ Assess for patellar reflexes.

3508 After consulting with the health care provider, the Correct answer: 1 Very common substances to be avoided during pregnancy include aspirin, caffeine, cough and Specific knowledge of the adverse effects of various drugs on the developing fetus is
nurse explains to a woman who has just found out she cold products, stimulants (such as diet pills), and nicotine, but acetaminophen generally is safe needed to answer the question. Take time to review this material if the question was
is pregnant that which of the following common drugs and effective for occasional use. difficult.
may be taken if needed during pregnancy?

1.‐ Cough and cold products


2.‐ Diet pills
3.‐ Acetaminophen (Tylenol)
4.‐ Aspirin

3509 Which statement by the nurse to a group of pregnant Correct answer: 4 Almost all forms of drugs in the maternal circulation can be readily transferred to the Use basic principles of drug distribution to make your selection.
women who plan to breastfeed best reflects the need colostrum and breast milk. For this reason, women should use drugs only if necessary during
for cautious medication use during lactation? lactation. The statements contained in the other options are not true.

1.‐ "Almost half of mothers probably should not breastfeed, because there are many interactions between drugs and foods."
2.‐ "Because a single dose of almost any drug is not of much concern, breastfeeding is a preferred method of feeding the newborn."
3.‐ "As long as the pregnant woman delivers at term, the infant's liver is well developed, and so the mother can take several commonly used medications."
4.‐ "Most drugs that enter the mother's bloodstream can be readily transferred to the breast milk."

3510 The neonatal nurse assigned to work with a newborn Correct answer: 3 The neonate has as immature renal system, and cannot metabolize medications effectively. Use knowledge of fetal and newborn growth and development to make a selection. Recall
interprets that which infant‐related factor adversely This places the neonate at risk from the viewpoint of pharmacokinetics (absorption, that most drugs are affected by the liver and kidneys to help guide your selection.
affects the neonate with respect to pharmacokinetics? distribution, biotransformation, and excretion). The neonate might not have hyperactive bowel
sounds; the musculoskeletal system is not necessarily weak; and thermoregulatory issues can
affect temperature regulation, but would not directly affect pharmacokinetics, making option
3 the best answer.
1.‐ Hyperactive bowel sounds
2.‐ Weak musculoskeletal system
3.‐ Immature kidney function
4.‐ Insufficiently developed thermoregulation

3511 A woman who is lactating needs to take a medication. Correct answer: 1 If a breastfeeding mother must take medication, it is suggested that the dose be timed Use basic principles of pharmacokinetics and knowledge of growth and development to
The nurse explains that the drug effects on the infant immediately after breastfeeding to maximize the time span before the next feeding. This will help guide your selection.
can be minimized by taking the drug at which of the allow for the greatest amount of the drug to be metabolized and excreted. The other time
following times? frames listed provide less time for clearance from the maternal bloodstream, and therefore
can cross into the breast milk more readily.
1.‐ Immediately after breastfeeding
2.‐ Immediately before breastfeeding
3.‐ One hour before breastfeeding
4.‐ Two hours after breastfeeding

3512 While caring for a client receiving total parenteral Correct answer: 1 TPN contains a 20–60% glucose solution, which often causes hyperglycemia and is routinely The question asks for nursing responsibilities and the wording of the question indicates the
nutrition (TPN), nursing responsibilities will include regulated by giving regular insulin. The solution should be clear and homogenous without correct option is also a true statement. Recall TPN solutions contain high concentrations of
which of the following? layering or cracking. TPN solutions are infused at a steady rate. An alternative solution would glucose. Systematically eliminate incorrect options, choosing option 1, which addresses the
be used to adjust fluids for urine output, NG losses, etc. Injection caps are changed per high glucose content.
institution protocol, usually every 72 hours. More frequent changes increase risk for bacterial
contamination.
1.‐ Covering elevated blood glucose levels with a sliding scale of regular insulin
2.‐ Inspecting solution to ensure "layering" of contents is present
3.‐ Adjusting rate of solution to client's output every shift
4.‐ Changing injection caps on the intravenous tubing every shift

3513 Clients suffering from profound malnutrition may Correct answer: 1 Magnesium, potassium, and phosphate levels may drop because magnesium (needed for ATP The question requires specific knowledge of the refeeding syndrome. Eliminate option 4
experience the refeeding syndrome when first synthesis) and phosphorus (a component of ATP) are utilized rapidly as the TPN solution is because it is unrelated to the other 3 options. Eliminate option 3 because TPN is often given
initiating total parenteral nutrition (TPN). To metabolized, and potassium is taken up intracellularly in energy metabolism. Blood sugars are to rest the bowel. Choose option 1 over 2 because it is more specific and blood sugars are
determine if this occurs, the nurse must do which of checked frequently but are not related to the refeeding syndrome. Bowel sounds and level of specifically connected to hyperglycemia.
the following? consciousness would be part of a routine assessment but do not reflect refeeding syndrome.

1.‐ Monitor potassium, phosphorus, and magnesium levels closely.


2.‐ Check blood glucose levels every 6 hours.
3.‐ Assess client for hyperactive bowel sounds.
4.‐ Assess client's level of consciousness every shift.

3514 Which of the following conditions leads the nurse to Correct answer: 3 TPN solutions contain hypertonic glucose (20–70%), which would cause severe irritation and Recall the effect of osmolarity on blood vessels. Eliminate option 1 because it is an
conclude that a client's total parenteral nutrition (TPN) phlebitis to a peripheral vein. Central veins are much larger, and the solution becomes diluted excessively high flow rate. Options 2 and 4 are correct statements but can be eliminated
solution needs to be administered through a central quickly. A rate of 150–200 would provide excessive fluid volume and calories. The infusion is of because they are not as specific. Option 3 gives a justification.
venous catheter? a high caloric content, but this fact does not explain why it must be given centrally. Peripheral
sites do require more frequent changes, but frequency of changes does not influence the
decision to use a central vein.

1.‐ The client will be receiving fluids at a rate of 150–200 mL/hr.


2.‐ The client will be receiving an infusion with a high caloric content.
3.‐ The end concentration of dextrose in the solution will be 25%.
4.‐ The use of a peripheral vein would require more frequent site changes.

3515 A client recovering from multiple trauma is started on Correct answer: 1 TPN provides a readily available source of carbohydrates, fats, and proteins in order to A key word in the question is major, indicating some or all of the options may be correct,
total parenteral nutrition (TPN) therapy. The nurse restore or maintain positive nitrogen balance. Recovery from multiple trauma utilizes protein but one is considered more important. Recognize the major purpose of TPN is to provide
determines that which of the following is a major goal and fat stores, leading to a negative nitrogen balance. TPN provides some hydration and helps calories and nutrition. Choose option 1 because it is more global and pertains to the
of this therapy? to maintain urine output, but the primary purpose is to spare the body's own energy stores. nutritional needs.
Trace minerals are added to TPN solutions but are not the primary reason TPN solutions are
used.
1.‐ Prevent a negative nitrogen balance in the client.
2.‐ Maintain a high urine output.
3.‐ Provide adequate hydration.
4.‐ Ensure client receives needed trace minerals.

3516 A 46‐year‐old male client has had a central venous Correct answer: 1 Inspiration into a deflated lung produces sharp chest pain as resistance to airflow is met and Note the question asks for the best answer, indicating one option is a better choice. Omit
catheter inserted in the subclavian vein recently so is a sign of a pneumothorax. Oxygenation would decrease as ventilation capacity is decreased, options 2 and 4 because they could be symptomatic of many respiratory and cardiovascular
that he can be started on total parenteral nutrition but this effect could be attributed to many factors and is not the best indicator of a problems. Omit option 3 because it is the least related to a respiratory problem.
(TPN) therapy. Which of the following assessment pneumothorax. A rapid pulse is also not a specific indicator of pneumothorax. A red and
findings best indicates the client may have a swollen insertion site is a sign of irritation or infection.
pneumothorax?
1.‐ Client complains of sharp chest pain.
2.‐ Pulse oximetry is 90% on room air.
3.‐ Catheter insertion site is red and swollen.
4.‐ Radial pulse is rapid.

3517 A 34‐year‐old female client recovering from severe Correct answer: 3 A 2‐pound weekly gain is the ideal weight gain when TPN is given to restore nutritional Key words in the question are weekly and desired goal. Also note the client is receiving the
weight loss secondary to Crohn's disease is being balance and improve weight. It demonstrates the treatment is effective. Maintenance of TPN to improve weight. Eliminate option 1 because a weight gain is desired. Eliminate
discharged and has received instructions on home weight would be a goal if weight gain were not desired. Weight gains of 1 pound or less per option 4 because it is monthly; a weekly goal is desired. Choose option 3 over 2 because it
parenteral nutrition. The nurse instructs the client to week are less than desirable. gives an easily measurable weight (as some scales do measure half pounds).
weigh herself weekly, suggesting the desired goal of
therapy is:
1.‐ To maintain current weight.
2.‐ A weight gain of 0.5 pound per week.
3.‐ A weight gain of 2 pounds per week.
4.‐ A monthly weight gain of 4 pounds.

3518 Total parenteral nutrition (TPN) is being started on a Correct answer: 2 A baseline weight is needed to provide a foundation for clinical therapy and to assess Recall the nurse's role in hanging TPN and the content of the solution. Eliminate option 1
client with malabsorption syndrome. Prior to starting response to treatment. The calculation of nutrients is done by a dietitian. It is not necessary to because it is not the nurse's responsibility. Eliminate option 3 as unnecessary and option 4
the infusion, nursing responsibilities will include which have a baseline EKG. Client should be assessed for allergies to eggs. as incorrect.
of the following?
1.‐ Calculating the nutrients needed for an individualized formula
2.‐ Obtaining a baseline weight
3.‐ Ensuring an EKG is performed on client prior to starting infusion
4.‐ Checking for allergies to wheat

3519 Prior to hanging a total parenteral nutrition (TPN) Correct answer: 1, 3, 5 In addition to the base solution, TPN contains electrolytes, minerals, and multivitamins. Specific knowledge of the content of TPN is needed. Systematically eliminate ingredients
solution, the nurse checks the content of the solution. Regular insulin may be added, but NPH cannot. If diuretics are needed because of underlying that you recognize would not be appropriate.
Which of the following ingredients would the nurse client condition, it would not be added to the TPN solution.
expect to be included? Select all that apply.

1.‐ Trace minerals


2.‐ NPH insulin
3.‐ Electrolytes
4.‐ Diuretic
5.‐ Multivitamin
3520 A client is receiving an infusion of TPN at 83 mL/hr. Correct answer: 91 If TPN infusion is interrupted, the nurse should not play "catch up," but TPN can be safely Correctly calculate by multiplying the baseline rate of 83 by 10%. The exact calculation of
The infusion is stopped for 4 hours while client is off restarted at up to 10% of the baseline rate with an appropriate order to help replace nutrients 91.3 is rounded to the nearest tenth, making the correct answer 91 mL.<BR />
the nursing unit. When the client returns, the standing missed.
order indicates the infusion should be restarted at a
rate of 10% greater than the baseline rate. The
infusion should be run at _____ mL/hour. Write in a
numerical answer.

3521 The nurse is careful to ensure that when a client's Correct answer: 4 TPN should be gradually discontinued over a 24–48 hour period to allow for adjustment in The question indicates the action should be done gradually, implying that abrupt
total parenteral nutrition (TPN) infusion is metabolic function and prevent a sudden drop in blood glucose. Refeeding syndrome occurs withdrawal would have consequences. Consider each option in terms of what would occur
discontinued, it is done gradually. The client questions when TPN is first initiated. Hypovolemia and hyponatremia are not as significant a risk as if the solution were stopped suddenly. Eliminate option 1 because refeeding syndrome
why gradual tapering is necessary. The nurse responds hypoglycemia. occurs at the beginning of TPN therapy.
that this measure will prevent which of the following?

1.‐ Refeeding syndrome


2.‐ Hypovolemia
3.‐ Hyponatremia
4.‐ Rebound hypoglycemia

3522 The nurse is preparing to hang the next scheduled bag Correct answer: 2 It is recommended that TPN be brought to room temperature before infusing to prevent Knowledge of nursing responsibilities related to TPN is necessary. Although protocols
of total parenteral nutrition (TPN) solution. Prior to client discomfort and lowering of body temperature. It is not necessary to irrigate the IV line differ among institutions, the options are general and apply to basic principles. Eliminate
hanging the bag, the nurse does which of the with heparin or saline. Bags should be changed using aseptic technique, but sterile gloves are option 4 because sterile technique is not used when changing IV bags.
following? not needed.
1.‐ Irrigates the intravenous port with Heparin
2.‐ Removes solution from refrigerator 1 hour prior to hanging it
3.‐ Infuses 100 mL of normal saline to clear the intravenous line
4.‐ Has sterile gloves available to use when changing bags of solution

3523 A subclavian catheter has been inserted in a client Correct answer: 2 Confirmation of correct catheter placement is essential before TPN is started to assure The core concept in the question is identifying the action that must be taken before
who will be receiving total parenteral nutrition (TPN). infusion will enter the superior vena cava. Confirming client is afebrile and obtaining a baseline therapy can be started, and the key word is most, indicating all or some of the options are
Before beginning the infusion, it is most important for weight are also important, but not of highest priorty. Intake and output should be monitored important, but one has a higher priority. Analyze each option and choose option 2 because
the nurse to do which of the following? once TPN is started. it is most critical.

1.‐ Obtain a baseline weight.


2.‐ Confirm x‐ray report of correct catheter placement.
3.‐ Determine client is afebrile.
4.‐ Check intake and output for the past 24 hours.

3524 A client is receiving a continuous total parenteral Correct answer: 1 The client is at greater risk for hyperglycemia because he or she is both receiving TPN and Recall TPN solutions contain a high caloric content and, combined with another source of
nutrition (TPN) solution and is also on a regular diet taking in additional calories on a regular diet. Assessing urine output, offering a nutritional calories, will increase risk of hyperglycemia for client. The key word in the stem is most,
while recovering from a multiple trauma. Which of the supplement, and encouraging high‐protein foods are all appropriate, but not of highest since all of the options are correct.
following nursing actions is of most importance when priority.
caring for this client?
1.‐ Monitor blood glucose levels closely.
2.‐ Assess urine output.
3.‐ Encourage intake of high‐protein foods.
4.‐ Offer nutritional supplements at bedtime.
3525 A client admitted with malnutrition has received total Correct answer: 3 Prealbumin levels are the best indicators of protein stores and nitrogen balance, reflecting The question asks that you evaluate effectiveness of treatment. A key word is best,
parenteral nutrition (TPN) for 2 weeks. To best the client's nutritional status. Checking blood glucose levels assesses for hypoglycemia and indicating that some or all options are partially correct. Eliminate options 1 and 4 because
evaluate the effectiveness of the treatment, the nurse hyperglycemia. Assessment of weight gain is indicated but is not the best indicator of they don't measure an outcome of treatment. Choose option 3 over 2 because it is a more
should do which of the following? nutritional status because fluid retention may influence the weight gain. Skin turgor is a specific measurement.
measure of hydration status.
1.‐ Monitor recent blood glucose levels.
2.‐ Check for recent weight gain.
3.‐ Check prealbumin levels.
4.‐ Evaluate skin turgor.

3526 The nurse is assisting a student nurse in the Correct answer: 1 Aseptic technique is used when changing solution bags. Sterile gloves are not needed and The word unnecessary is significant and indicates that three of the options are important
preparation of a continuous infusion of total would be contaminated as soon as the outside of the bag is touched. TPN requires use of a or required for the task. Analyze each option for appropriateness to TPN therapy.
parenteral nutrition (TPN) solution. The nurse special micron filter. Solutions that are layered or cracked should not be used. The primary
recognizes that which action by the student is provider must order the content of TPN.
unnecessary?
1.‐ Dons sterile gloves when connecting tubing to the solution bag
2.‐ Attaches tubing that contains a micron filter to the solution bag
3.‐ Checks solution for evidence of layering and cracking
4.‐ Verifies orders are current for TPN

3527 The client has been receiving total parenteral Correct answer: 2 The client's body has adjusted to higher blood glucose levels as a result of receiving TPN with Select the response that would best correspond to abrupt withdrawal of the major
nutrition (TPN) for several days. The central venous high dextrose concentrations. Abruptly stopping TPN can result in hypoglycemia. The other component of TPN, which is glucose.
access device became dislodged and the nurse notes answers are incorrect.
that the client's IV has not been running for several
hours. The nurse would monitor the client for which of
the following complications related to the stopped
infusion?
1.‐ Hypocalcemia
2.‐ Hypoglycemia
3.‐ Sepsis
4.‐ Hyperkalemia

3528 A nurse has obtained a unit of packed red blood cells Correct answer: 1 Vital signs are taken immediately prior to beginning the transfusion. Because most blood The question focuses on the action that should be taken immediately prior to beginning
(PRBCs) from the blood bank. She and another nurse transfusion reactions occur within 15 minutes of starting infusion, it is of great importance to the transfusion.
have confirmed that it is the correct blood for the establish the pre‐infusion baseline immediately prior to beginning the transfusion. Skin color,
patient. Immediately prior to starting the blood hemoglobin levels, and renal function would be important considerations; however, it would
transfusion, the nurse should assess which of the not be essential that such assessments take place immediately prior to the start of the
following? transfusion.
1.‐ Vital signs
2.‐ Skin color
3.‐ Hemoglobin level
4.‐ Creatinine clearance

3529 A nurse is preparing to administer a unit of packed Correct answer: 4 Normal saline is the solution of choice when used as an adjunct to a transfusion. Ringer's The focus of the question is compatibility of blood with intravenous solutions. Use nursing
red blood cells (PRBCs). When obtaining the necessary lactate and dextrose solutions are contraindicated due to the potential for clotting and knowledge and the process of elimination to make a selection.
supplies, the nurse would obtain which of the hemolysis.
following IV solutions to hang with the unit of blood?

1.‐ Ringer's lactate


2.‐ 5% dextrose in 0.9% sodium chloride
3.‐ 5% dextrose in 0.45% sodium chloride
4.‐ 0.9% sodium chloride
3530 A nurse returns to evaluate a client who has been Correct answer: 3 Circulatory overload is a complication associated with rapid transfusion administration. The core issue of the question is the ability to recognize signs of circulatory overload. Use
receiving a blood transfusion for the past 30 minutes. Symptoms include bounding pulse, dyspnea, and crackles in the lungs. Crackles in the lungs nursing knowledge and the process of elimination to make a selection.
The client is observed to be dyspneic. Upon would not be associated with an immune response, hypovolemia, or polycythemia vera.
assessment, the nurse auscultates the presence of
crackles in the lung bases and an apical heart rate of
110 beats per minute. The nurse suspects that the
client is experiencing which of the following
complications associated with blood transfusions?

1.‐ Immune response to transfusion


2.‐ Hypovolemia
3.‐ Fluid overload
4.‐ Polycythemia vera

3531 A nurse determines that a client receiving a unit of Correct answer: 3 The nurse is to stop the transfusion immediately and keep the IV line open with normal The core issue of the question is the ability to take proper action when a transfusion
packed red blood cells (PRBCs) is experiencing a saline. The nurse would then notify the physician. A white blood cell count or a saline bolus reaction is suspected. Use nursing knowledge and the process of elimination to make a
transfusion reaction. The nurse promptly stops the would be ordered by the physician; however, these actions would not be independently selection.
blood transfusion and next does which of the initiated by the nurse without contacting the physician first.
following?
1.‐ Contact the physician.
2.‐ Obtain a white blood cell count.
3.‐ Run normal saline at keep vein open (KVO) rate.
4.‐ Infuse a normal saline bolus.

3532 A client arrives at the emergency department Correct answer: 1 A transfusion of FFP is indicated for clients who are actively bleeding with a prothrombin time The core issue of the question is the ability to anticipate the need for fresh frozen plasma.
following a gunshot wound. The client is actively greater than 1½<BR /> Use nursing knowledge and the process of elimination to make a selection.
bleeding and has been taking warfarin (Coumadin)
therapy. His prothrombin time is twice the desired
amount. The nurse expects the physician will order a
transfusion with which of the following blood
products?
1.‐ Fresh frozen plasma
2.‐ Random donor platelets
3.‐ Red blood cells
4.‐ Crystalloids

3533 An adult female client has a hemoglobin level of 9.2 Correct answer: 4 Iron deficiency anemia can result from blood loss and is common in menstruating women. The core issue of the question is the ability to anticipate the needs of a client with iron
grams/dL. A nurse interprets that this is most likely Leukemia is reflected in the white blood cell count. Amenorrhea, the absence of menstruation, deficiency anemia. Use nursing knowledge and the process of elimination to make a
related to which of the following conditions? is unlikely to cause of iron deficiency anemia. Vitamin B&lt;sub&gt;12&lt;/sub&gt; deficiency selection.
anemia is often associated with dietary deficiency, such as experienced by vegetarians or those
who avoid dairy products.
1.‐ Leukemia
2.‐ Amenorrhea
3.‐ Vitamin B12 deficiency anemia
4.‐ Iron deficiency anemia

3534 A nurse has received an order to transfuse a client Correct answer: 3 An in‐line filter is required for the administration of blood. Tubing with a microdrip, an air The core issue of the question is knowledge that an in‐line filter is required for blood
with one unit of PRBCs. In preparation for the infusion, vent, and tinting that protects the blood from light would not be indicated for blood transfusion. Use nursing knowledge and the process of elimination to make a selection.
the nurse selects the appropriate tubing for blood administration.
administration. The nurse is aware that the tubing is
manufactured with which of the following?
1.‐ A macrodrip chamber
2.‐ An air vent
3.‐ An in‐line filter
4.‐ Tinting that protects blood from exposure to light

3535 A postoperative client is to receive a transfusion of Correct answer: 3 A transfusion of platelets is indicated for the client with active bleeding. A transfusion with The core issue is knowledge of platelet transfusion therapy. Recall that platelets are
platelets because of a critically low platelet count. The PRBCs would result in increased hemoglobin and hematocrit levels. Platelet administration is critical for proper blood clotting to make a selection.
client requests information on the benefits of the not associated with the prevention of deep vein thrombosis. The administration of platelets is
transfusion. In response to the request, the nurse's not associated with the return of the prothrombin time to normal.
answer should include which of the following?

1.‐ Improvement of hemoglobin and hematocrit


2.‐ Prevention of deep vein thrombosis
3.‐ Decrease in bleeding from surgical site
4.‐ Return of prothrombin time to expected range

3536 The nurse has received an order to transfuse two Correct answer: 175 If a 350 mL unit of packed red blood cells is to infuse over 2 hours, the rate will be 175 mL per Use knowledge of pharmacological math to calculate the drip rate.
units of PRBCs to a client for more than 2 hours apiece. hour.
The nurse notes after obtaining the first unit that it
contains 350 mL. The nurse would administer the
infusion at an hourly rate of _____ mL/hour. Write in
a numerical answer.

3537 A nurse has received a report on a client being Correct answer: 1, 3 Key features of anemia include coolness to touch, intolerance to cold, tachycardia, The core issue of the question is the ability to anticipate the needs of a client with
admitted with anemia who requires a blood orthostatic hypotension, and headaches. Hypertension, bounding pulses, and diaphoresis are moderately severe anemia. Use nursing knowledge and the process of elimination to make
transfusion. The nurse will anticipate which of the not associated with anemia. a selection.
following assessment findings? Select all that apply.

1.‐ Tachycardia
2.‐ Hypertension
3.‐ Headache
4.‐ Diaphoresis
5.‐ Bounding peripheral pulses

3538 A client is scheduled for elective surgery in 4 weeks. Correct answer: 4 An autologous transfusion involves the collection of the client's blood prior to the anticipated The core issue of the question is knowledge of various types of transfusions. Use nursing
When the nurse in the surgeon's office initiates need, thus compatibility is not problematic and the potential for contamination is eliminated. knowledge and the process of elimination to make a selection.
preoperative education, the client expresses concern It would not be appropriate for the nurse to predict blood loss as a result of a surgical
regarding the potential need for a blood transfusion. procedure. Further, stressing the safety of blood transfusions may elicit a false sense of
The nurse's best response is: comfort for the client. While the family may be able to donate blood, this would not be as
potentially beneficial to the client as an autologous blood transfusion. Not all relatives share
the same blood type.
1.‐ "It is unlikely that you will lose that much blood during the surgery."
2.‐ "Blood transfusions are safer now than in the past."
3.‐ "Your family may be able to donate blood for you."
4.‐ "You may want to consider an autologous blood transfusion."

3539 A nurse is caring for an immunocompromised client Correct answer: 4 The normal white blood cell count is between 5,000 and The core issue of the question is knowledge of white blood cell counts. Use nursing
with cancer. A nurse would consider implementing 10,000/mm&lt;sup&gt;3&lt;/sup&gt;. The nurse should consider implementing neutropenic knowledge and the process of elimination to make a selection.
neutropenic precautions when the client's white blood precautions when the white blood cell count is at or below
cell count is: 2,000/mm&lt;sup&gt;3&lt;/sup&gt;.
1.‐ 10,500/mm3
2.‐ 7,650/mm3
3.‐ 6,000/mm3
4.‐ 2,000/mm3

3540 A client has experienced an adverse reaction shortly Correct answer: 2 When a transfusion reaction has occurred, the nurse must return any remaining blood to the The core issue of the question is knowledge of critical actions to take when a transfusion
after a blood transfusion is initiated. The nurse blood bank. If a reaction had not occurred, the nurse would dispose of the blood in an reaction occurs. Use nursing knowledge and the process of elimination to make a selection.
documents the event according to hospital policy and appropriate biohazard bag. It would not be appropriate for the nurse to send the blood to the
does which of the following with the remainder of the laboratory or infection control department.
blood that has not been transfused?

1.‐ Discards the blood in the appropriate biohazard bag


2.‐ Returns the blood to the blood bank
3.‐ Sends the blood to the chemistry laboratory for analysis
4.‐ Sends the blood to the infection control department

3541 A client with a low hemoglobin and hematocrit is to Correct answer: 1 Because the client is febrile, the nurse must notify the health care provider. The health care The core issue of the question is knowledge of critical actions to take when a client
receive a unit of packed red blood cells (RBCs). Prior to provider will determine if the client can tolerate the transfusion or if additional therapeutic requiring a blood transfusion has an elevated temperature. Use nursing knowledge and the
initiating the transfusion, the nurse determines that intervention is warranted, which may include the administration of acetaminophen (Tylenol) or process of elimination to make a selection.
the client’s temperature is 100.8 degrees F orally. The an antihistamine.
most appropriate action for the nurse to take is which
of the following?

1.‐ Delay hanging the blood and notify the physician.


2.‐ Begin the transfusion as prescribed.
3.‐ Administer 650mg of acetaminophen (Tylenol) and begin the transfusion.
4.‐ Administer an antihistamine and begin the transfusion.

3542 A client presents to the emergency department Correct answer: 4 Albumin is used as a plasma expander and is used in the treatment of hypovolemic shock. The core issue of the question is knowledge of the uses of various blood products. Use
following a motorcycle accident. The client is in Packed RBCs are indicated in the treatment of anemia. Platelets are indicated in the treatment nursing knowledge and the process of elimination to make a selection.
hypovolemic shock. The health care provider has of thrombocytopenia. Cryoprecipitate is administered to treat von Willebrand’s disease and
ordered plasma expansion. The nurse anticipates that fibrinogen levels below 100 mg/dL.
the client will be transfused with which of the
following blood products?
1.‐ Packed red blood cells
2.‐ Cryoprecipitate
3.‐ Platelets
4.‐ Albumin

3543 A client has received a granulocyte transfusion. The Correct answer: 3 Granulocyte transfusions are administered to neutropenic clients with infections for white The core issue of the question is knowledge of appropriate outcomes of blood component
nurse will assess which of the following labs to blood cell replacement. Therefore, the white blood cell count would indicate the success of the therapy. Use nursing knowledge and the process of elimination to make a selection.
determine if the client has benefited from the treatment. The hemoglobin, hematocrit, erythrocytes, and platelet counts would not be
transfusion? appropriate for evaluating this therapy.
1.‐ Hemoglobin and hematocrit
2.‐ Erythrocytes
3.‐ White blood cells
4.‐ Platelets

3544 A client has experienced an adverse reaction to a Correct answer: 1 Diphenhydramine (Benadryl) is administered for the treatment of anaphylaxis. Benadryl The core issue of the question is knowledge of appropriate treatments for transfusion
blood transfusion. The client is found to have a pruritic competes with the H&lt;sub&gt;1&lt;/sub&gt; receptors on effector cells, thus blocking the reactions. Use nursing knowledge and the process of elimination to make a selection.
rash and urticaria. The nurse anticipates that which of effects of histamine. Tylenol and hydrocortisone would provide symptomatic relief from signs
the following medications will be ordered for the and symptoms of a transfusion reaction. Aspirin would not be indicated for treatment of a
client? transfusion reaction.
1.‐ Diphenhydramine (Benadryl)
2.‐ Acetaminophen (Tylenol)
3.‐ Hydrocortisone cream
4.‐ Acetylsalicylic acid (Aspirin)

3545 The nurse recognizes albumin therapy would be Correct answer: 2 A client who has heart failure should not receive albumin therapy due to potential side effects Recall that albumin is a colloid and will increase blood volume. Recognize it is useful in the
contraindicated for which of the following clients? of circulatory failure and fluid overload that may further exacerbate the client’s cardiac status. treatment of conditions in options 1, 3, and 4 and eliminate them.
All of the other options reflect clinical conditions whereby albumin therapy might be indicated
and of benefit in the clinical setting.
1.‐ A client in hypovolemic shock
2.‐ A client with a history of congestive heart failure (CHF)
3.‐ A client who has hypoalbuminemia
4.‐ An infant with hemolytic disease of the newborn

3546 A client has received 10 units of stored blood Correct answer: 2 Citric acid in the transfused blood binds with calcium; the multiple transfusions would put the The critical words are 10 units of stored blood and electrolyte imbalance. Recognize there
following massive blood loss. The nurse plans to check client at risk for hypocalcemia. Hemolysis of cells occurs with stored blood causing the release is an association between the transfusions and the imbalance. Recall the action of citrate
for which of the following electrolyte imbalances? of intracellular potassium, leading to hyperkalemia. Options 3 and 4 would not be expected. used in stored blood to be directed to option 2.

1.‐ Hypokalemia
2.‐ Hypocalcemia
3.‐ Hyperphosphatemia
4.‐ Hypernatremia

3547 A client being treated for fluid volume deficit (FVD) is Correct answer: 3 The client’s weight gain indicates retention of volume but the symptoms of extracellular fluid Recognize the symptoms of hypotension and tachycardia indicate vascular volume is still
gaining weight as expected but remains orthostatic volume deficit (tachycardia, orthostatic, and dry mucous membranes) indicate that the fluid is depleted. Options 1 and 2 reflect vascular fluid excess and eliminate them. Associate the
and tachycardic with dry mucous membranes. The moving into a third space. Options 1 and 2 are incorrect because the presence of JVD and an weight gain with third space fluids and choose option 3.
nurse should assess the client for: S&lt;sub&gt;3&lt;/sub&gt; heart sound would indicate fluid volume overload, and the client
clearly has an extracellular fluid volume deficit. Option 4 is incorrect because the presence of
elastic skin turgor indicates adequate hydration.

1.‐ Jugular venous distention (JVD).


2.‐ A third heart sound.
3.‐ Third spacing of body fluids.
4.‐ Elastic skin turgor.

3548 The nurse is reviewing the intravenous (IV) therapy Correct answer: 3, 5 A client with SIADH needs fluids with solutes. Individuals with liver disease are unable to Read each option and analyze appropriateness of the solution to the condition. Recall the
orders for assigned clients. Which of the following IV metabolize the lactate in lactated Ringer's solution. All of the other options are incorrect liver metabolism needed for lactated ringers to choose option 3. Recall that
orders should the nurse question? Select all that apply. because they are appropriate solutions for the indicated clients. D&lt;sub&gt;5&lt;/sub&gt;W D&lt;sub&gt;5&lt;/sub&gt;W does not replace electrolytes to choose option 5.
is hypotonic and provides free water for the cells of the dehydrated client. NS is isotonic and is
used to replace ECF volume loss from any cause, in this case NG suction. NS is also indicated
for post‐op fluid management in order to avoid significant hyponatremia from the
administration of hypotonic fluid.

1.‐ D5W for the dehydrated client


2.‐ 0.9 % NaCl (NS) replacement of losses from nasogastric (NG) suction
3.‐ Lactated Ringer’s for the client with liver disease
4.‐ 0.9 % NaCl for the new postoperative client
5.‐ D5W for a client with syndrome of inappropriate antidiuretic hormone (SIADH)
3549 A client presents with a blood glucose level of 620 Correct answer: 1 Significantly increased blood glucose causes osmotic diuresis with resultant fluid volume The critical term is glucose of 620mg/dL. Recognize that the level is extremely elevated
mg/dL. The nurse would expect which other abnormal deficit (FVD). Option 2 is incorrect as a decrease in CO&lt;sub&gt;2&lt;/sub&gt; content and would increase serum osmolarity and promote diuresis. Recognize the FVD that exists
laboratory values in this client? consistent with metabolic acidosis would be expected; the client is clearly experiencing to choose option 1.
diabetic acidosis with a blood glucose of 620 mg/dL. Option 3 is incorrect because FVD means
an increased solute concentration in the blood or increased serum osmolality. Option 4 is
incorrect because the serum potassium is generally increased in clients with FVD, even though
total body potassium may be low.

1.‐ Increased blood urea nitrogen (BUN)


2.‐ Increased CO2 content
3.‐ Decreased serum osmolality
4.‐ Decreased serum potassium

3550 A client has received 4 units of packed red blood cells Correct answer: 4 The client receiving four units of packed RBCs for severe anemia will have an increased blood Recall that the heart rate and respirations will increase to compensate for the anemia.
(RBCs) for severe anemia. Which of the following volume, which in turn increases blood pressure and decreases the heart rate. The heart will Recognize 4 units of packed RBCs should help to restore blood volume and compensatory
assessment findings would the nurse expect to see? not have to work as hard to try and oxygenate body tissues and tissue perfusion will increase. mechanisms will resolve and be directed to option 4.
Likewise the lungs will not have to work as hard to oxygenate the blood, as greater oxygen
exchange will occur at the pulmonary capillary level with more RBCs available.

1.‐ Increased heart rate


2.‐ Decreased blood pressure
3.‐ Decreased tissue perfusion
4.‐ Decreased respiratory rate

3551 A client with an isotonic fluid volume deficit is being Correct answer: 4 Treatment with normal saline should expand the extracellular fluid (ECF) volume, because it is The critical term is isotonic normal saline. Recognize this fluid is used to restore ECF fluid
appropriately treated with normal saline solution. an isotonic fluid like the ECF. Expansion of this volume in turn will increase blood pressure, and will not cause fluid shifts. Recognize correction of the vascular volume will eliminate
Which of the following changes would the nurse decrease heart rate, increase body weight (not decrease it), and increase urine output, the need for a compensatory increase in heart rate.
expect to see in this client? because there is improved renal perfusion. Urine specific gravity should decrease, not
increase, because urine will be less concentrated.
1.‐ A decrease in urine output
2.‐ An increase in urine specific gravity
3.‐ A decrease in body weight
4.‐ A decrease in heart rate

3552 A client with congestive heart failure (CHF) who is Correct answer: 3 It is imperative that the nurse report an increase in adventitious lung sounds, because the Recall the priority of ABCs (airway, breathing, and circulation) to be directed to option 3.
being treated with diuretics for a fluid volume excess client's CHF could be worsening and pulmonary edema may develop. Airway, ventilation, and
does not seem to be responding adequately to oxygenation are always the first priority. All of the other options should also be reported to
treatment. Which of the following clinical findings is the physician because they all support inadequate treatment for a fluid volume overload, but
most important for the nurse to report to the they are not the priority assessment for reporting.
physician?
1.‐ Increase in blood pressure
2.‐ Increase in peripheral edema
3.‐ Increase in adventitious breath sounds
4.‐ Increase in body weight

3553 After receiving 100mL of RBCs, a client develops Correct answer: 3 The symptoms indicate a hemolytic reaction, usually as a result of ABO incompatibility. Recognize the client has received approximately half of the infusion when symptoms
lumbar pain, nausea and complains of burning at the Clumping of RBCs can block capillaries and reduce blood flow to vital organs, necessitating started. Associate the symptoms of pain with blockage to organs. Recall the effect of
insertion site. After quickly assessing the client, the immediate discontinuation of the infusion. The IV line should be kept patent with normal saline hemolytic reactions to be directed to option 3.
nurse's next action should be which of the following? to allow for emergency access as needed. The other options would not be appropriate.

1.‐ Slow the infusion to half the rate at which it was infusing.
2.‐ Notify the physician and have another nurse monitor the client.
3.‐ Discontinue the infusion of RBCs and maintain the IV with normal saline solution.
4.‐ Decrease the infusion rate and reassess client in 10 minutes.

3554 A client receiving a transfusion of PRBCs suddenly Correct answer: 1 The client is experiencing a severe allergic/anaphylactic reaction. The nurse should first stop The critical word is priority, indicating all or some of the options are correct and may be
sounds hoarse, begins wheezing, is diaphoretic and the infusion of any more blood. Next, the nurse should maintain the client’s airway, administer done almost simultaneously, but one takes highest precedence. Recognize the client's
short of breath, and reports palpitations. Blood oxygen, and infuse normal saline solution through a clean IV tubing (i.e., one not contaminated symptoms represent an anaphylactic response to direct you to option 1.
pressure is 76/52. What is the priority nursing action? with blood) to maintain the intravascular volume and prevent vascular collapse. Hospital
protocol may include the administration of epinephrine and steroids. It is always critical to
follow current standards of care and hospital protocols during transfusion therapy.

1.‐ Stop the transfusion.


2.‐ Rapidly infuse the normal saline solution to maintain intravascular volume.
3.‐ Administer epinephrine and steroids.
4.‐ Maintain the client's airway and notify the physician.

3555 Which of the following actions should the nurse take Correct answer: 2, 5 A client receiving granulocytes is expected to experience fever and chills due to high potential The critical word is granulocyte. Recall the increased risk for reactions with this type of
when a client is receiving a granulocyte transfusion? for development of allergic reactions. Option 1 is incorrect because granulocytes should be transfusion to direct you to options 2 and 5.
Select all that apply. administered slowly as in option 5, and the client should be premedicated with antihistamines,
steroids, and an antipyretic. Option 3 is incorrect because a microaggregate filter would trap
the granulocytes and nullify the transfusion. Option 4 is incorrect because, although
lymphocytes are present in the transfusion, they are granulocytes and the neutrophil count is
the appropriate laboratory value to trend.

1.‐ Administer the granulocytes rapidly.


2.‐ Premedicate with an antihistamine, a steroid, and an antipyretic.
3.‐ Attach a microaggregate filter to the IV tubing.
4.‐ Check lymphocyte count following the transfusion.
5.‐ Administer the granulocytes slowly.

3556 Which of the following laboratory tests should the Correct answer: 2 A client with CHF has a compromised cardiac pump and therefore already has an increased Critical words are elderly, CHF, and albumin. Recall the effect of albumin on fluid balance
nurse monitor closely in an elderly client with risk for fluid volume excess or overload. Albumin is a hypertonic colloid solution that can cause and underlying pathophysiology of CHF to direct you to option 2.
congestive heart failure (CHF) who is receiving IV circulatory overload. This represents a double risk, then, for a client with CHF. The hematocrit
albumin? would decrease as the plasma volume increases (hemodilution). All of the other options are
incorrect as the platelet count, PT, and serum bilirubin will be unaffected by albumin
administration.
1.‐ Platelet count
2.‐ Hematocrit
3.‐ Serum bilirubin
4.‐ Prothrombin time (PT)

3557 The nurse is conducting a class for oncology clients Correct answer: 2 Clients may develop HLA antibodies in response to previous transfusions of blood that are not The critical word is alloimmunized. Recall the physiology of this process to direct you to
who frequently receive blood products. The nurse leukodepleted. Since platelets carry class 1 HLA antigens, the HLA antibodies will quickly option 2.
explains that if a client becomes alloimmunized, the destroy them. Therefore, attempting to match the donor’s platelet antigens with the
most effective way to increase the platelet count recipient’s and then transfusing these platelets should increase the platelet count. All of the
would be to: other options are incorrect because they will not reverse alloimmunization.

1.‐ Transfuse single‐donor platelet units.


2.‐ Transfuse HLA‐matched donor platelets.
3.‐ Use a WBC filter to minimize infusion of white blood cells.
4.‐ Premedicate the client with diphenhydramine (Benadryl) and acetaminophen (Tylenol).
3558 A physician has ordered dextran for a client while Correct answer: 1 Dextran is a glucose solution that is not extracted from human plasma and therefore presents Note that dextran is being given in lieu of blood products to provide some clues to its
waiting for the results of the type and cross‐match. no risk of transmission of viruses. Option 2 is incorrect because dextran has a higher risk for advantages. Recall the properties of dextran to eliminate options 2, 3, and 4.
The nurse anticipates that an advantage of using anaphylaxis as compared to hetastarch and albumin. Options 3 and 4 are incorrect because
dextran for this client is that it: dextran promotes dehydration of tissues due to its hyperosmolar effect and does affect
clotting factors.
1.‐ Contains no risk of transfusion‐related illnesses.
2.‐ Has a decreased risk for anaphylaxis as compared to hetastarch or albumin.
3.‐ Promotes hydration of tissues.
4.‐ Has no effect on clotting factors.

3559 Which of the following interventions should the nurse Correct answer: 4 Hespan can dilute clotting factors and therefore create transient changes in PT, PTT, and Review purpose of using Hespan to expand intravascular fluid volume. Recall Hespan's
include in developing a plan of care for a client clotting times. Option 1 is incorrect, because Hespan will not interfere with blood typing and effect on clotting factors to choose option 4.
receiving hetastarch (Hespan)? cross‐matching. Option 2 is incorrect because Hespan expands the plasma volume and thus
can cause hypervolemia. Option 3 is incorrect; urine output will be increased because Hespan
causes osmotic diuresis.
1.‐ Draw specimen for type and cross‐match prior to beginning Hespan infusion.
2.‐ Monitor client for signs of hypovolemia.
3.‐ Expect decreased urine output as body begins conserving plasma volume.
4.‐ Monitor for transient changes in PT, PTT, and clotting times.

3560 A client will receive 2 units of packed RBCs. The nurse Correct answer: 3 The nurse's highest priority is client safety; therefore, it is imperative that the client know The critical word is highest priority, indicating all or some of the options are correct, but
places highest priority on teaching the client which of what to report to the nurse should a reaction occur, i.e., chilling, fever, itching, shortness of one takes more precedence. Choose option 3 since this reflects safety.
the following? breath, back pain. All of the other options should be explained to the client to promote
understanding and comfort, but are not the highest priority.
1.‐ The rationale for the transfusion
2.‐ Overview of the procedure so the client will know what to expect
3.‐ Signs and symptoms to report to the nurse if they should occur
4.‐ Frequency with which vital signs will be taken so as not to alarm the client

3561 A client scheduled to receive 4 units of packed RBCs Correct answer: 3 Option 3 describes a mild nonhemolytic febrile reaction, which is the most common reaction Recall the purpose for giving premedication before packed RBCs to direct you to option 3.
with premedication therapy wants to know what effect and one that may be minimized through premedication. Option 1 is incorrect because hives
premedication will have on reducing chances of a are a common occurrence, but airway swelling is not and will probably not be managed by
transfusion reaction. The best response by the nurse to premedication alone. Option 2 is incorrect because it gives false reassurance and is
the client's concern would be: nontherapeutic. Option 4 is incorrect because it suggests a possible hemolytic reaction that is
uncommon and will not be minimized by premedication.

1.‐ "Hives and airway swelling may occur due to an allergic reaction but will be prevented by the medications."
2.‐ "You have no need to worry because your doctor has ordered medications to prevent a reaction."
3.‐ "Chilling and fever caused by previous exposure to blood or blood products may occur but these are minimized by the medications."
4.‐ "The premedication will prevent incompatible blood reactions that could result in shortness of breath and kidney problems."

3562 A trauma victim admitted to the emergency Correct answer: 3, 5 A client who is hemorrhaging and in shock requires immediate restoration of oxygen‐carrying Recognize the need for the client to receive a blood replacement secondary to the type of
department is hemorrhaging, in shock, and has lost a capacity. With no time available for cross‐matching, universal donor blood (type O, Rh‐ fluid losses to choose the option that establishes IV access. Recall Type O is a universal
significant percentage of blood volume. Since there is negative) is administered. Option 1 is incorrect because type AB, Rh‐positive blood can only be donor to choose option 3.
no time to perform a cross‐match, which of the given to type AB, Rh‐positive recipients. Option 2 is incorrect because albumin has no oxygen‐
following actions should the nurse take immediately? carrying capacity, which is essential for a trauma client. In addition, it would remain in the
Select all that apply. intravascular space and would not assist in restoring blood volume and maintaining adequate
circulation. Option 4 is incorrect because platelets may be administered if needed, but they are
not oxygen‐carrying cells, which is the first priority. Option 5 is correct. Establishing an
intravenous site should be done prior to transfusing blood products.

1.‐ Transfuse type AB, Rh‐positive blood.


2.‐ Transfuse albumin to expand the remaining plasma volume.
3.‐ Transfuse type O, Rh‐negative blood.
4.‐ Transfuse platelets to restore adequate clotting ability.
5.‐ Establish an intravenous line.

3563 A client with pretransfusion hemoglobin and Correct answer: 3 Each unit of PRBCs should raise the hemoglobin by l gram and hematocrit by 3%. Option 3 is Recall the expected changes that would be effected by a transfusion of PRBCs and multiply
hematocrit values of 9 grams and 27%, respectively, the only option illustrating the expected increase. The nurse should be aware of expected that by 2.
received two units of packed red blood cells (PRBCs) responses to therapy in order to validate that treatment has been effective.
on the evening shift. The nurse determines the
transfusions were effective when repeat laboratory
tests indicate which of the following results?

1.‐ 11 grams, 33%


2.‐ 12 grams, 36%
3.‐ 13 grams, 30%
4.‐ 15 grams, 39%

3564 In addition to monitoring for bleeding, which of the Correct answer: 1, 5 Coumadin depresses the synthesis of vitamin K dependent clotting factors in the liver, Recall the role of Coumadin in the clotting cascade to direct you to option 1 and 5.
following interventions should the nurse include in a resulting in a prolonged PT/INR (extrinsic coagulation pathway). FFP contains the needed
plan of care for a client who has accidentally taken too clotting factors and will reverse the PT/INR. The PT/INR needs to be adjusted to the
much warfarin (Coumadin)? (Select all that apply.) therapeutic range for client’s underlying condition. Option 2 is incorrect because the aPTT/PTT
measures the intrinsic coagulation pathway. Options 3 and 4 are incorrect because neither
albumin nor platelets will restore clotting factors.

1.‐ Check prothrombin time (PT) or International Normalized Ratio (INR).


2.‐ Check partial thromboplastin time (PTT) or activated PTT (aPTT).
3.‐ Prepare to administer albumin.
4.‐ Prepare to administer platelets.
5.‐ Prepare to administer fresh frozen plasma.

3565 A female client with type B, Rh‐negative blood has Correct answer: 2 A client with type B blood can only receive type B (client/recipient has no anti‐B antibodies) Recognize the risk for Rh incompatibility existing in the client to direct you to option 2.
been exposed to Rh‐positive blood in the past. The and type O (contains no antigens for recipient to react to). Since the client is Rh‐negative and
nurse will evaluate that instruction regarding blood has been previously exposed to Rh‐positive blood, the client may have antibodies to Rh‐
compatibility has been effective when the client positive blood. Therefore only Rh‐negative blood should be administered. Option 1 is incorrect
verbalizes it is safe to receive which of the following because the client cannot receive type O positive blood due to the identified negative Rh
types of blood? factor. Options 3 and 4 are incorrect because they each contain at least one incompatible
blood type component (type A antigens and type O positive blood).

1.‐ Type B positive and type O positive blood.


2.‐ Type B negative and type O negative blood
3.‐ Type AB negative and type O negative blood
4.‐ Type A positive and type O positive blood

3566 A client in need of a blood transfusion is concerned Correct answer: 3 Option 3 is accurate and realistic and may help the client with decision making. Option 1 is Critical words are alleviate anxiety and disease transmission. Eliminate option 1 since it is
about the possibility of disease transmission. Which of not accurate and offers false reassurance. Option 2 is incorrect because it involves giving inaccurate and provides false reassurance. Eliminate option 2 since it does not answer the
the following statements by the nurse may help to "medical" advice, is nontherapeutic, and may increase client anxiety due to questioning of client's question. And eliminate option 4 since it is not totally accurate.
alleviate some anxiety for the client? treatment plan. Option 4 is incorrect because it may delay treatment and there is no evidence
that designated donor blood is safer.
1.‐ "All blood products are absolutely safe after testing, and there is no need to worry."
2.‐ "If you do not want the transfusion, do not sign the consent. Perhaps the physician can give you some iron pills."
3.‐ "More sophisticated screening tests have made the blood supply safer, and the risk of infection, while it exists, is very low."
4.‐ "Have a family member or friend donate blood for you because this will guarantee its safety."
3567 Which of the following blood products does the nurse Correct answer: 3 FFP is derived from one unit of whole blood and contains the clotting factors that the client The critical word is hemophilia. Recall the deficiency of clotting factors associated with this
anticipate the physician will order for a client needs plus fibrinogen. Option 1 is incorrect because even though whole blood contains some illness to choose option 3.
diagnosed with hemophilia? clotting factors, it is deficient in others and is indicated for significant acute blood loss (which is
not the client's problem). Option 2 is incorrect because improved oxygen‐carrying capacity
(rendered by the infusion of PRBCs) is something the client does not need. Hemophilia is a
clotting disorder that requires clotting factor replacement. Option 4 is incorrect, because
albumin contains no clotting factors.

1.‐ Whole blood


2.‐ Packed red blood cells (PRBCs)
3.‐ Fresh frozen plasma (FFP)
4.‐ Albumin

3568 A client received a severe burn in a house fire. On the Correct answer: 4 Protein is responsible for a significant portion of the osmotic pressure found in the blood In severe burns, third spacing of fluids is the issue that replacement of plasma proteins
second day of hospitalization, the physician orders the vessels and maintains fluid within the vessels. In burn injuries, protein is lost allowing fluid to such as albumin would be used to address.
client to receive albumin. Which of the following does escape into the tissues. Albumin is used to replace the lost proteins and pull fluids from the
the nurse cite while explaining the rationale for interstitial space back into the vascular system. It does not contain clotting factors (option 1),
albumin administration? red blood cells (option 3), nor is there enough fluid volume to consider it as part of primary
fluid resuscitation (option 2).
1.‐ Improves the level of clotting factors and prevent bleeding
2.‐ Provides fluid resuscitation to prevent dehydration
3.‐ Replaces the lost red blood cells and reduce the anemia
4.‐ Provides proteins to increase the osmotic pressure in the blood

3569 A client is to receive his first dose of epoetin Correct answer: 3 Uncontrolled hypertension is a contraindication for the administration of epoetin because the To accurately answer the question, the learner must understand the indication for using
(Epogen). Which of the following client assessment increase in hematocrit increases blood viscosity and peripheral vascular resistance. This will epoetin. Based upon this knowledge, select responses can be eliminated. An understanding
findings would be a contraindication to the nurse lead to a further increase in blood pressure. A hemoglobin level lower than 10 mg/dL would of the physiological response of the medication also is necessary, as is evident in this
administering the medication? not be a contraindication, as the use of epoetin is associated with an increase in the scenario.
hemoglobin and hematocrit levels. While an elevated blood glucose level is significant, it is not
a contraindication for the use of epoetin. Similarly, a heart rate of 58 beats per minute would
not impact the client’s ability to tolerate epoetin therapy.

1.‐ A hemoglobin level of 10 grams/dL


2.‐ A blood glucose level of 280 mg/dL
3.‐ A blood pressure of 180/100
4.‐ A heart rate of 58 beats per minute

3570 A client is to begin weight‐based intravenous heparin Correct answer: 6 The key to answering the question on weight‐based heparin is to identify the infusion rate, The client's weight in kilograms equals 100. The client is to receive 6 units per kilogram,
therapy for treatment of a deep vein thrombosis. The which is 6 units per kilogram per hour. Thus, to accurately obtain the information, the weight which equals 600 units per hour. The amount of heparin on hand is 100 units of heparin per
client's weight is 220 pounds. The client is to receive an must be converted from pounds to kilograms. There are 2.2 pounds per kilogram. To convert 1 mL. Thus, the client should receive 6 mL per hour.
infusion of 6 units per kilogram per hour. The this amount into milliliters, it is necessary to identify how many units are in each milliliter of
intravenous heparin on hand contains 100 units per available solution.
milliliter. The client will receive _______ mL of heparin
per hour. Write in a numerical answer.

3571 The nurse has completed a type and crossmatch on Correct answer: 1, 2, 3, A client who has type AB blood is known as a universal recipient, and therefore can be To answer the question accurately, the student must understand the principles of blood
the client she will be transfusing. The client is 4 transfused with any of the identified blood types (A, B, AB, or O). There is no type ABO. typing. Select blood types are able to receive all blood types, while others are able to
identified as having type AB blood. The nurse would be donate to all blood types.
able to safely transfuse with which of the following
blood types? Select all that apply.
1.‐ AB
2.‐ A
3.‐ B
4.‐ O
5.‐ ABO

3572 A client is admitted to the medical/surgical unit with Correct answer: 2 Alcoholics are at risk for folic acid deficiency because alcohol suppresses the metabolism of Knowledge of the anemia associated with chronic alcohol use is needed to correctly
liver cirrhosis related to chronic alcohol use. The client folate, from which folic acid is formed. Iron deficiency anemia often is associated with blood identify the answer. Further, by using a process of elimination, incorrect answers can be
also is found to have a low hemoglobin level, as well as loss, increased energy demands, gastrointestinal malabsorption, and dietary deficiencies. eliminated.
a diminished number of circulating red blood cells. The Hemolytic anemia is associated with increased red blood cell destruction as the result of
nurse suspects that the client's anemia is cause by trauma, infection, and autoimmune reactions. Vitamin B&lt;sub&gt;12&lt;/sub&gt; deficiency
which of the following? is associated with dietary deficiencies, malabsorption from intestinal tract, or a deficiency of
intrinsic factor (Lemone and Burke, 2000).

1.‐ Iron deficiency anemia


2.‐ Folic acid deficiency anemia
3.‐ Hemolytic anemia
4.‐ Vitamin B12–deficiency anemia

3573 A client with von Willebrand's disease presents to the Correct answer: 1 Cryoprecipitate, a product derived from plasma, contains von Willebrand's clotting factor. Knowledge of von Willebrand's disease is needed to answer this question, as well as
hospital with prolonged bleeding following a dental Packed red blood cells, granulocytes, and platelets do not contain von Willebrand's clotting knowledge of the contents of each of the identified blood products. By identifying the
procedure. The nurse is aware that the client might factor (Ignatavicius and Workman, 2002). connection between the deficit in the disease process and the supplement in the blood
need to be transfused with which of the following product, the learner can establish the relationship.
products?
1.‐ Cryoprecipitate
2.‐ PRBCs
3.‐ Granulocytes
4.‐ Platelets

3574 A client has been receiving intravenous heparin for Correct answer: 4 The onset of action for warfarin is slow; therefore, heparin usually is given during the first few The question asks for the most appropriate response. Thus, more than one response might
treatment of a deep vein thrombosis. The client is to days of treatment. Therefore, the diagnosis of a deep vein thrombosis does not necessarily be correct; however, one is most appropriate. To answer the question accurately, the
begin warfarin (Coumadin) therapy today. When the indicate that the client will require dual therapy. The heparin and warfarin will be administered learner must understand the pharmacotherapeutics for both of the medications.
nurse attempts to administer the warfarin, the client concurrently until the warfarin becomes effective, not necessarily until the client is discharged
states "I am not taking two blood thinners. I could from the hospital. The nurse would not need to clarify the order unless a contraindication,
bleed to death." What would be the most appropriate such as a previous reaction to the therapy, were identified.
response from the nurse?

1.‐ "Both warfarin and heparin are necessary when you have a deep vein thrombosis."
2.‐ "I will call the physician to clarify this order."
3.‐ "You will only take both while you are in the hospital."
4.‐ "The warfarin will not become effective for several days."

3575 A client with leukemia presents with dyspnea and Correct answer: 4 The purpose of the d‐dimer is to detect the presence of DIC in a client. The prothrombin time, The key to answering the question is the client’s suspected diagnosis of DIC. The question
prolonged bleeding. The client is suspected to have activated partial thromboplastin time, and white blood cell count are not specific for the asks for the test that would confirm the diagnosis. The process of elimination can be
disseminated intravascular coagulation (DIC). Which of diagnosis of disseminated intravascular coagulation. utilized to reach the answer.
the following lab tests does the nurse plan to obtain to
confirm the suspected diagnosis?

1.‐ Prothrombin time


2.‐ Activated partial thromboplastin time
3.‐ White blood cell count
4.‐ D‐dimer test
3576 A nurse is caring for a client with disseminated Correct answer: 3 DIC is characterized by abnormal clot formation. The widespread clotting consumes all of the The question asks which of the actions would be most appropriate. Utilizing understanding
intravascular coagulation (DIC). Which of the following circulating clotting factors and platelets. This is followed by excessive bleeding while blood the pathophysiology of DIC, identify the risk factors associated with the disease process. By
actions by the nurse would be most appropriate? vessels are blocked by clots, leading to decreased blood flow to major organs. The nurse would the process of elimination, the correct answer can be determined.
not assess the client's blood pressure with an automatic cuff, obtain labs via peripheral
venipuncture, or encourage the client to ambulate, due to the risk for hemorrhage.

1.‐ Assess blood pressure using an automatic cuff.


2.‐ Obtain all lab specimens via peripheral venipuncture.
3.‐ Assess the skin for hematomas and signs of tissue ischemia.
4.‐ Encourage client to ambulate at least 50 feet with assistance.

3577 A client recently was diagnosed with pernicious Correct answer: 2 Pernicious anemia is caused by the body’s failure to absorb vitamin To answer the question accurately, it is necessary to understand the pathophysiology of
anemia. The nurse anticipates that the client will be B&lt;sub&gt;12&lt;/sub&gt;. Therefore, folic acid, iron, and albumin would not be indicated in pernicious anemia. By identifying the deficiency in this type of anemia, one can anticipate
treated with which of the following? the treatment of pernicious anemia. the treatment.
1.‐ Folic acid
2.‐ Vitamin B12
3.‐ Iron
4.‐ Albumin

3578 A nurse is preparing to initiate a blood transfusion. Correct answer: 3 A 19‐gauge needle is the smallest size intravenous catheter that should be used for a blood The knowledge of intravenous catheter gauges is essential to correctly answer the
The client does not have venous access. When transfusion. Large‐bore catheters allow cells to flow more easily, without occluding the lumen question. Recall that the larger the number, the smaller the diameter of the catheter. Note
obtaining venous access, what is the smallest size of the catheter. A 20‐gauge or 22‐gauge catheter, both smaller than 19 gauges, is not the critical words smallest size to focus your attention on the correct option. If you did not
intravenous catheter the nurse should use? preferable, due to the potential for occlusion. A 16‐gauge catheter is larger than a 19‐gauge know the answer to the question, realize that in this type of question it may be possible
but the question asks for the smallest size that can be used. that the correct answer is a number in the middle of the set provided.

1.‐ 20‐gauge needle


2.‐ 22‐gauge needle
3.‐ 19‐gauge needle
4.‐ 16‐gauge needle

3579 A nurse is about to begin a blood transfusion of Correct answer: 3 The expiration date would be checked prior to administering the transfusion. The ABO type, To answer the question accurately, it is necessary for the learner to understand the
packed red blood cells (PRBCs). The nurse inspects the Rh type, and requisition slip would not indicate the freshness of the product. significance of the blood type, Rh type, expiration date, and requisition slip. This would
quality of the transfusion to ensure that it does not allow the elimination of select answers. The key is which of the identifiers would support
show signs of deterioration. Which of the following the freshness of the product.
would the nurse assess prior to initiating the
transfusion to support the freshness of the product?

1.‐ ABO type


2.‐ Rh type
3.‐ Expiration date
4.‐ Requisition slip

3580 A client needs to have a type and crossmatch Correct answer: 1 Blood group antigens are present on the membrane of the red blood cell, and form the basis Based upon the potential responses, it is evident that the blood type is either an antibody
completed prior to administering a transfusion. The for the ABO blood categorization. The type and crossmatch do not evaluate the presence of or antigen, and is located either on the red blood cell membrane or in the blood plasma.
nurse is aware that the blood type of the client antibodies on the red blood cell membrane or in the blood plasma. Antigens found in the Based upon these options, the process of elimination will be useful.
represents which of the following? blood plasma are also not evaluated in the type and crossmatch.

1.‐ An antigen found on the red blood cell membrane


2.‐ An antibody found on the red blood cell membrane
3.‐ An antigen found circulating in the blood plasma
4.‐ An antibody found circulating in the blood plasma
3581 A client with a critically low hemoglobin and Correct answer: 4 The nurse must respect the client's beliefs, and avoid cultural imposition. The physician The question asks which of the responses is most appropriate. Therefore, one or more of
hematocrit will require a blood transfusion to prevent should be made aware so that the client’s treatment plan can be reevaluated. It would not be the responses might be correct. It is necessary to have an understanding of the ethical,
shock. However, when the nurse approaches the client appropriate for the nurse to inform the client of the likelihood of death associated with legal, and cultural considerations for the client receiving a transfusion. However, given the
with these findings, the client responds, "My religion declining a treatment. While the health care institution might require the client to sign an available information, the most appropriate response would be selected.
does not permit me to have a blood transfusion." against medical advise form, it would not be the most therapeutic response for the nurse. By
Which of the following responses from the nurse suggesting that the client consider having a family member donate blood, the nurse is
would be most appropriate? imposing personal beliefs on the client.

1.‐ "You will die without it, so please reconsider."


2.‐ "You will need to sign an AMA (against medical advice) form."
3.‐ "Perhaps your family could donate the blood."
4.‐ "I respect your wishes and will notify the physician of your beliefs."

3582 A client with leukemia is receiving chemotherapy. The Correct answer: 2 Recall that neutrophils are granulocytes. The medication that would increase the neutrophil To answer the question correctly, it is necessary to have an understanding of the
client is immunosuppressed, with a low absolute count is G‐CSF (granulocyte colony‐stimulating factor). Zidovudine is an antiviral medication classification of each of the medications. This knowledge will allow the learner to eliminate
neutrophil count. The nurse anticipates that which of and would be useful if the client had a viral infection. Raloxifene is a selective estrogen the incorrect answers.
the following medications would be used in the receptor modulator that is used to reduce bone resorption in post‐menopausal women with or
treatment of the client? at risk for osteoporosis. Toremifene is an antiestrogen similar to tamoxifen (Nolvadex) that is
used to treat metastatic breast cancer in post‐menopausal women.

1.‐ Zidovudine (AZT)


2.‐ G‐CSF (Neupogen)
3.‐ Raloxifene (Evista)
4.‐ Toremifene (Fareston)

3583 A client presents to the emergency department (ED) Correct answer: 1 Thrombocytopenia is characterized by a platelet count lower than The key information presented in the scenario includes ecchymotic areas and the low
with numerous ecchymotic areas on his arms, legs, and 50,000/mm&lt;sup&gt;3&lt;/sup&gt;, and often presents as purple spots on the client’s skin. platelet count. By understanding the clinical presentation of all possible answers, the
chest. The client's platelet count level is Polycythemia vera is characterized by an increase in hemoglobin and hematocrit levels. learner can eliminate those that do not fit with the findings presented in the question.
45,000/mm&lt;sup&gt;3&lt;/sup&gt;. The nurse Leukopenia is characterized by a low white blood cell count. Leukemia is associated with an
suspects the client has which of the following overproduction of white blood cells.
conditions?
1.‐ Thrombocytopenia
2.‐ Polycythemia vera
3.‐ Leukopenia
4.‐ Leukemia

3584 A client presents to the ED with chest pain and Correct answer: 2 Thrombolytics are used to treat acute thromboembolic disorders in an acute care setting. The question asks which of the agents will be administered immediately. While one or
shortness of breath. The physician believes that the These agents dissolve clots via the endogenous fibrinolytic system. While anticoagulants and more of the medications might be appropriate therapy for the client with a myocardial
client is experiencing a myocardial infarction due to antiplatelets might be appropriate for the client at risk for myocardial infarction, the client infarction, the correct answer will be administered immediately.
thrombosis. The nurse anticipates administering which experiencing an acute myocardial infarction would be a candidate for thrombolytic therapy.
of the following immediately? Beta blockers are used to prevent myocardial infarction, and are not associated with
treatment of acute thrombolytic events.

1.‐ Anticoagulant
2.‐ Thrombolytic
3.‐ Beta blocker
4.‐ Antiplatelet
3585 A client scheduled to receive a transfusion of red Correct answer: 3 Washed RBCs are transfused when hyperactivity reactions are likely, or for those with a The key to this question is to determine which of the answers is most accurate. This
blood cells has a history of numerous transfusion history of reactions. Clients with a previous history of transfusion reaction are not necessarily suggests that one or more responses may be applicable. Therefore, the learner must select
reactions. Which of the following statements is most excluded from receiving a transfusion. While type O blood is considered the universal donor, the most accurate of all responses.
accurate? the client still can experience an adverse reaction. While artificial blood might be available, it is
not the first line of transfusion therapy for the client with a known history of transfusion
reactions.
1.‐ "You will not be able to have a transfusion."
2.‐ "You will be able to receive type O blood."
3.‐ "You will receive washed red cells."
4.‐ "You will receive artificial blood."

3586 A client has a continuously running peripheral Correct answer: 4 Before making a decision about how to infuse the antibiotic, the nurse should check The core issue of the question is the knowledge that it is critical to check for
infusion. The physician orders an antibiotic as a compatibility of the antibiotic with the continuous IV solution. If the drug and the infusion compatibilities when infusing IV solutions through the same line. Use this knowledge and
piggyback infusion four times per day. In order to were compatible, they would be run at the same time. If the drug and infusion were the process of elimination to make a selection.
administer the antibiotic, the nurse should do which of incompatible, the nurse would stop the infusion during the period of antibiotic administration
the following? and flush the line carefully before and after the antibiotic. It is often inadvisable to start a
second IV site unless absolutely necessary. The other answers are incorrect.

1.‐ Start a new IV access to administer the antibiotic so that there will not be compatibility issues.
2.‐ Start a new IV access to eliminate the problem of too much volume for one site.
3.‐ Increase the flow rate of the continuous infusion to facilitate the administration of the antibiotic.
4.‐ Check to see if the antibiotic is compatible with the continuous infusion.

3587 The family of a home infusion client calls the home Correct answer: 3 Alarms sound on electronic infusion devices when the infusion is complete, there is an The core issue of this question is the ability to interpret the significance of an alarm on an
health nurse one night to report that the electronic occlusion, air is in the line, the battery is low, or the cassette is improperly loaded. The other infusion pump. Use knowledge of pump function in general and the process of elimination
infusion pump is alarming. The nurse anticipates that answers are incorrect reasons for an alarm. to make a selection.
the infusion pump alarm could be caused by which of
the following?
1.‐ The client's pulse and blood pressure are falling.
2.‐ The client is experiencing a reaction to the medication.
3.‐ The infusion is complete or there is an occlusion.
4.‐ There is an incompatibility with the medications.

3588 The home health nurse is monitoring a client who Correct answer: 1 All catheters should be flushed with syringes with barrels of 10 mL or larger. The smaller the The wording of the question tells you the correct option is an incorrect statement. Use
performs self‐care of a central line. The nurse observes barrel size, the greater the pressure that comes from the tip. Smaller syringes could damage knowledge of IV catheter flush protocols and the process of elimination to make a
the client doing all of the following activities. Which the catheter. All other activities are done correctly. selection.
activity indicates the need for further education?

1.‐ Flushing the central line with a 3 mL syringe


2.‐ Cleaning the needleless injection cap with alcohol before accessing
3.‐ Using sterile gloves to change the central line dressing
4.‐ Wearing a mask while changing the central line dressing

3589 The client has a tunneled Groshong catheter for Correct answer: 3 Groshong catheters have a three‐way pressure‐sensitive valve that restricts air from entering The core issue of the question is knowledge of proper management and care of Groshong
intermittent medication administration. After the venous system and prevents backflow of blood; therefore, the catheter should not be catheters. Use this knowledge and the process of elimination to make a selection.
administering the medication, the nurse prepares to clamped and the client does not need to perform the Valsalva maneuver. The catheter is
do which of the following? designed so that only saline is used to flush. The other answers are incorrect actions for
catheter maintenance.
1.‐ Clamp the catheter.
2.‐ Flush the catheter with saline, then heparin.
3.‐ Flush the catheter with saline.
4.‐ Ask the client to perform Valsalva's maneuver when the medication IV tubing is disconnected.
3590 The client has a percutaneous jugular central venous Correct answer: 1 Although it is not necessary to flush peripheral capped access devices with heparinized The core issue of the question is knowledge of proper management and care of central
line that is capped and used for intermittent infusions. normal saline (100 or 10 units per 1 mL of normal saline), central venous access devices that venous catheters. Use this knowledge and the process of elimination to make a selection.
After administering the medication, the best method are not Groshong catheters are flushed per agency protocols with heparinized normal saline.
to maintain patency is to do which of the following? When medications are administered, the access device is first flushed with normal saline, then
with heparinized normal saline. Heparin is incompatible with many medications, and for this
reason, normal saline is used prior to the administration of heparinized saline that maintains
patency of the catheter. The other answers are incorrect procedure.

1.‐ Flush the line first with 3–5 mL of normal saline, then with 1–3 mL of heparinized normal saline.
2.‐ Flush the line with 3–5 mL of normal saline.
3.‐ Flush the line with 3–5 mL of heparinized normal saline.
4.‐ Flush the line first with 3–5 mL of heparin, then with 1–3 mL of normal saline.

3591 The nurse is caring for a client with a Hickman central Correct answer: 4 One of the complications of IV therapy is air embolism, which is the introduction of air into The core issue of the question is the ability of the nurse to detect situations that could
line. While changing the central line dressing, the the vein. Air embolism can be prevented by using luer lock devices on all attachments. The lead to complications of IV therapy. Use knowledge of these risks to aid in making a
nurse notes that the injection cap (e.g., heplock other responses are unrelated to this connection. selection.
adapter) is of the slip lock variety instead of a luer lock
device. The nurse recognizes that this adapter puts the
client at risk for which of the following complications?

1.‐ Sepsis
2.‐ Occlusion
3.‐ Phlebitis
4.‐ Air embolism

3592 The client is to receive vancomycin (Vancocin), an Correct answer: 3 The device that provides the most accurate infusion rate is the electronic infusion pump. The The core issue of the question is the best method to prevent speed‐related adverse
intravenous medication. To prevent adverse reactions other devices are less accurate and less controllable. reactions from a drug infused intravenously. Use knowledge of IV infusion devices and the
from rapid infusion, the nurse would plan to process of elimination to make a selection.
administer this drug using which of the following
methods?
1.‐ Gravity
2.‐ With a regulator
3.‐ Electronic infusion pump
4.‐ Elastomeric pump

3593 The physician is going to order a hypotonic Correct answer: 4 0.45% sodium chloride (one‐half normal saline) is a hypotonic solution that draws fluid from The core issue of the question is knowledge of the tonicity of various intravenous
intravenous solution for a client with cellular the vascular compartment into the cells. Normal saline and lactated Ringer’s are isotonic solutions. Use this knowledge and the process of elimination to make a selection.
dehydration. The nurse would expect which of the solutions, while 5% dextrose in normal saline is a hypertonic solution until the glucose is
following fluids to be administered? metabolized, then it is isotonic.
1.‐ 0.9% normal saline
2.‐ 5% dextrose in normal saline
3.‐ Lactated Ringer's solution
4.‐ 0.45% sodium chloride

3594 The nurse is caring for several clients with central Correct answer: 1 The Groshong catheter is designed with a three‐way pressure‐sensitive valve that restricts air The core issue of the question is knowledge of various central venous catheters and which
venous catheters. While changing the tubing on the from entering the venous system and prevents backflow of blood. The other options do not one poses the least risk of air embolism requiring Valsalva's maneuver. Use this knowledge
central lines, the nurse would not need to instruct the have this protection. and the process of elimination to make a selection.
client to perform Valsalva's maneuver when the client
has which of the following catheters?

1.‐ Groshong
2.‐ Single‐lumen
3.‐ Percutaneous
4.‐ Accessed subcutaneous venous port

3595 The client is receiving 5% dextrose in 0.45% normal Correct answer: 3 0.9% sodium chloride (normal saline) is the only solution that can be administered with blood The core issue of the question is the knowledge that only normal saline is compatible with
saline. The physician has ordered the client receive one or blood products. Other solutions may cause the blood cells to clump or cause clotting. The any blood product. Use this knowledge and the process of elimination to make a selection.
unit of packed cells. Prior to hanging the blood, the other options are incorrect.
nurse will prime the blood tubing with which of the
following solutions?

1.‐ 5% dextrose
2.‐ Lactated Ringer's
3.‐ 0.9% sodium chloride
4.‐ 5% dextrose in 0.45% sodium chloride

3596 While assessing a client’s intravenous (IV) line, the Correct answer: 1 Infiltration is leakage of fluids into the surrounding tissues, resulting in edema around the The core issue of the question is the ability to accurately interpret an IV complication. Use
nurse notes that the area is swollen, cool, pale, and insertion site, blanching, and coolness of skin around the site. The other options would not knowledge of various IV complications and the process of elimination to make a selection.
causes the client discomfort. The nurse documents have these manifestations.
which of the following complications of IV therapy?

1.‐ Infiltration
2.‐ Phlebitis
3.‐ Infection
4.‐ Air embolism

3597 The client is receiving 5% dextrose and 0.45% sodium Correct answer: 2 Continuing the infusion at that site would only increase the phlebitis. The IV is discontinued The core issue of the question is the ability to accurately interpret an IV complication. Use
chloride intravenously and is complaining of pain at and restarted at a new site. Applying a warm compress to an area of phlebitis dilates the knowledge of various IV complications and the process of elimination to make a selection.
the IV site. The nurse assesses the site and notes vessel, improving circulation, and reduces the resistance to blood flow from within the vein
erythema and edema. Which of the following would be reducing the pain. The other options are incorrect.
the appropriate nursing action?

1.‐ Slow the infusion rate.


2.‐ Discontinue the IV and apply a warm compress to the IV site.
3.‐ Apply antibiotic ointment to the IV site.
4.‐ Gently pull back the IV access device to reposition within the vein.

3598 The nurse is starting a new peripheral intravenous Correct answer: 2 Tourniquets are made of latex. A blood pressure cuff can be used as an alternative method of The core issue of the question is providing for safety of the client who has a latex allergy.
(IV) line in a client. The client reports a latex allergy. vein distention. A new tourniquet may not resolve the latex issue. The other responses do not Use this knowledge and the process of elimination to make a selection.
The nurse has a typical IV start kit. Because of the latex address the latex issue.
allergy, the nurse should take which of the following
actions?
1.‐ Utilize a new tourniquet for this client.
2.‐ Utilize a blood pressure cuff to distend the vein.
3.‐ Avoid putting povidone iodine on the skin.
4.‐ Suggest an alternative therapy to a peripheral intravenous line.

3599 The nurse is inserting an intravenous (IV) line into a Correct answer: 2 The nurse would refrain from advancing the catheter if mild resistance is noted. The other The core issue of the question is knowledge of normal IV insertion procedure. Use this
client. After piercing the skin and entering the vein, the data are normal. The IV should be inserted bevel side up. The client should not experience knowledge and the process of elimination to make a selection.
nurse would refrain from advancing the catheter if pain, and a backflow is normal on insertion, indicating that the vein has been pierced.
which of the following were noted?

1.‐ Blood backflow into the IV catheter


2.‐ Mild resistance with advancement
3.‐ No reports of client discomfort
4.‐ The IV catheter was inserted bevel side up.

3600 The nurse is inserting a peripheral intravenous (IV) Correct answer: 5, 3, 1, The first step is to gather equipment. The nurse then selects a vein and cleanses the site. The Use this knowledge and the process of elimination to make a selection.
line. Place the following steps in order to perform this 2, 4 nurse applies a tourniquet and inserts the catheter. Finally, the nurse attaches the primed
procedure correctly. Click and drag the options below tubing and regulates the drip rate. Additional steps are to release the tourniquet, continue to
to move them up or down. assess the site, apply a dressing, and document the procedure.

1.‐ Apply tourniquet.


2.‐ Insert catheter at 5–15 degree angle through skin.
3.‐ Select vein.
4.‐ Attach tubing primed with IV solution.
5.‐ Gather equipment.

3601 A client with dry skin and mucous membranes is Correct answer: 4 The client is manifesting signs and symptoms of dehydration. Since the serum remains Critical words are normal osmolality, indicating the fluid replacement will need to
weak, has orthostatic blood pressure changes, and has isotonic, this is isotonic dehydration or hypovolemia. Appropriate treatment is with an isotonic maintain the normal osmolality. Recognize the need to restore fluid balance with use of an
decreased urine output. The client's serum osmolality, fluid to replace fluid volume. Options 1 and 2 are incorrect because they are hypotonic isotonic solution to direct you to option 4.
however, is normal. Which of the following IV fluids solutions and would cause fluid shifting leading to cellular edema (i.e., client's cells are normal
would the nurse anticipate being prescribed for this size and free water is not needed for cells). Option 3 is incorrect because the solution is
client? hypertonic and will cause further fluid shifting leading to cellular dehydration.

1.‐ 5% dextrose in water


2.‐ one‐half normal saline
3.‐ 10% dextrose in water
4.‐ Normal saline

3602 Which of the following outcomes would the nurse Correct answer: 4 25% albumin is a hypertonic colloid solution that will expand the plasma volume. This Recognize albumin is a plasma expander to direct you to option 4.
anticipate after infusion of 25% albumin to a client in increase in plasma volume should increase blood pressure, which in turn will decrease the
hypovolemic shock? strain on the heart and thereby decrease heart rate. The increase in volume will not lower
temperature or decrease peripheral perfusion; rather, it will have the opposite effect.

1.‐ Increase in heart rate


2.‐ Decrease in temperature
3.‐ Decrease in peripheral perfusion
4.‐ Increase in blood pressure

3603 A client with gastrointestinal (GI) bleeding suddenly Correct answer: 2 Normal saline solution is an isotonic solution that will replace lost vascular volume and Recognize the need for replacement with a isotonic fluid to direct you to option 2.
develops diaphoresis with a rapid and thready pulse, promote perfusion. All of the other options are incorrect because they are either hypotonic or
and the nurse finds it difficult to hear a blood pressure. act as hypotonic solutions in the bloodstream, providing free water that moves into the
Which of the following IV fluids does the nurse interstitial space and cells. Administration of these fluids can cause further fluid shifting, which
anticipate the physician will order stat? will not help to replace lost volume or promote perfusion. In addition, when blood is available,
it can be hung with the normal saline. Dextrose will cause lysis of red blood cells.

1.‐ Dextrose in water (D5W)


2.‐ 0.9% sodium chloride (normal saline)
3.‐ 0.45% sodium chloride (normal saline)
4.‐ Dextrose 5% in 0.45% sodium chloride (D5 HNS)
3604 The nurse is caring for a client who is experiencing Correct answer: 1 Albumin is given to facilitate remobilization of third space fluids. In the case of ascites it Critical words are ascites and albumin. Recall the physiology of ascites and recognize the
severe abdominal ascites secondary to cirrhosis. The would pull fluid from the abdomen into the intravascular space, resulting in a decrease in purpose of the albumin in treatment of ascites to direct you to option 1.
nurse determines an infusion of albumin has been abdominal girth. Option 2 is incorrect; the increase in intravascular fluid would lead to an
effective when assessment findings indicate which of increase in blood pressure. The pulse may increase in compensation to the increased blood
the following? volume, but this does not reflect effectiveness of the albumin treatment. A decrease in weight
would most likely be seen as the reabsorbed abdominal fluid is excreted by the kidneys.

1.‐ Decrease in abdominal girth


2.‐ Decrease in blood pressure
3.‐ Increase in pulse
4.‐ Increase in weight

3605 Which of the following changes in laboratory values Correct answer: 2 The client's plasma is hypertonic (i.e., very concentrated) to begin with and thus serum Critical words are isotonic fluids and hypertonic dehydration. Recall the effect of isotonic
would the nurse anticipate after administering isotonic osmolality, BUN, and hematocrit would be elevated from hemoconcentration. Once isotonic fluids on serum osmolarity to choose option 2.
intravenous fluids to a client experiencing hypertonic fluids are administered, the plasma concentration should decrease and all three laboratory
dehydration? test results should show a corresponding decrease. Option 1 is incorrect because one would
expect to see an improvement upon administration of isotonic fluids and BUN and serum
osmolality remain increased. Option 3 is incorrect because these findings would be consistent
in a client who has not been treated for hypertonic dehydration. Option 4 is incorrect because
one would expect to see a decrease in hematocrit with the administration of isotonic fluid
therapy.
1.‐ Increased serum osmolality, increased blood urea nitrogen (BUN), and decreased hematocrit (HCT)
2.‐ Decreased serum osmolality, decreased BUN, and decreased HCT
3.‐ Increased serum osmolality, increased BUN, and increased HCT
4.‐ Decreased serum osmolality, decreased BUN, and increased HCT

3606 A client with a history of congestive heart failure Correct answer: 3 Abrupt changes in weight are an important clue to changes in fluid status. Unusual losses, Critical words are CHF and isotonic dehydration. Eliminate options 1, 2, and 4 since they
(CHF) has been carefully rehydrated with normal e.g., fever or diarrhea, are significant; they need to be reported and may help the client reflect unsafe behaviors.
(0.9%) saline solution for isotonic dehydration related prevent a fluid volume deficit (FVD) in the future, especially since the client is taking a diuretic.
to overzealous diuresis. Which of the following Option 1 is incorrect because increasing salt and fluids may put the client at significant risk for
statements by the client indicates that the nurse's fluid volume excess (FVE) considering the history of CHF. Options 2 and 4 may put the client at
discharge teaching has been effective? risk for FVE or FVD, respectively.

1.‐ "I will increase my salt intake and double up on my fluid intake."
2.‐ "I will take my diuretic pill every other day."
3.‐ "I will weigh myself daily and notify the physician if I develop a fever or diarrhea."
4.‐ "I will drink only one glass of water a day so I can eventually stop taking my pill."

3607 The physician is going to order a hypotonic Correct answer: 4 0.45% NS is a hypotonic solution that draws fluid from the vascular compartment into the 0.45% sodium chloride is referred to as half normal. Normal saline is normal or isotonic.
intravenous solution for a client with cellular cells. The other answers are incorrect. Half normal would be hypotonic.
dehydration. The nurse would expect which of the
following fluids to be administered?
1.‐ 0.9% normal saline
2.‐ 5% dextrose in normal saline
3.‐ Lactated Ringer's
4.‐ 0.45% sodium chloride

3608 Several clients are being admitted to the hospital unit Correct answer: 3 The client with the airborne infection can spread this infection simply by breathing and The critical term is highest priority. Recall CDC precaution guidelines to enable you to
at one time. There is only one private room available. requires isolation in a private room. The client with the abdominal wound (option 2) would not make safe room assignments.
Which client has the highest priority for being be as likely to spread this organism when the wound is dressed. The clients in options 1 and 4
admitted to this private room? have no medical need for a private room.

1.‐ Cient admitted for elective surgery that requested a private room prior to admission
2.‐ Client with a large infected abdominal wound
3.‐ Cient who has a communicable respiratory infection
4.‐ Client under the age of 12

3609 The client is receiving 5% dextrose in 0.45 normal Correct answer: 3 0.9 sodium chloride (normal saline) is the only solution that can be administered with blood Blood is only hung and flushed with normal saline.
saline. The physician has ordered the client receive one or blood products. Other solutions may cause the blood cells to clump or cause clotting. The
unit of packed cells. Prior to hanging the blood, the other options are incorrect.
nurse will prime the blood tubing with which of the
following solutions?

1.‐ 5% dextrose
2.‐ Lactated Ringer's
3.‐ 0.9% sodium chloride
4.‐ 5% dextrose in 0.45% sodium chloride

3610 A client is placed on a patient‐controlled analgesia Correct answer: 2 Respiratory compromise is rare with opioid administration yet feared by many healthcare The critical words are highest priority. Recall information about common side effects of
(PCA) pump with an opioid medication following total workers (option 4). Sedation precedes a fall in respiratory rate and/or depth and, therefore, patient‐controlled analgesia to assist in assuring client safety during its use.
hip replacement surgery. After the client has should be noted and recorded. It may not be feasible to allow the surgical client to rest
administered a bolus of the prescribed medication, the uninterrupted for a several‐hour period (option 1). The infusion pump continuously records the
nurse should do which of the following as highest amount of medication infused (option 3).
priority?
1.‐ Allow client to rest uninterrupted for several hours
2.‐ Assess the client's level of sedation
3.‐ Record amount of medication the client received
4.‐ Monitor the client for respiratory arrest

3611 The client is receiving 5% dextrose and 0.45% sodium Correct answer: 2 Continuing the infusion at that site would only increase the phlebitis. The IV is discontinued Discontinuing the IV is usually the best option when site problems are described.
chloride intravenously and is complaining of pain at and restarted at a new site. Applying a warm compress to an area of phlebitis dilates the
the IV site. The nurse assesses the site and notes vessel, improving circulation, and reduces the resistance to blood flow from within the vein
erythema and edema. Recognizing these as signs of reducing the pain. The other options are incorrect.
phlebitis, which of the following would be the
appropriate nursing actions?

1.‐ Slow the infusion rate.


2.‐ Discontinue the IV and apply a warm compress to the IV site.
3.‐ Apply antibiotic ointment to the IV site.
4.‐ Gently pull back the IV access device to reposition within the vein.

3612 The nurse is assessing several clients with different Correct answer: 1 A contusion is a crushing of the tissues; there is no break in the skin. Therefore this wound is The critical words in the question are least likely. This tells you that the correct option is
types of injuries. The nurse would conclude that the less likely to become infected. A septic wound is one that has been invaded by pathogenic one that has the data that is the nearest to normal of the options presented. Use nursing
client who is least likely to develop a wound infection microorganisms (option 3). Purulent exudate also is an indicator of infection (option 4). A knowledge and the process of elimination to make a selection.
would be the client with which of the following? wound healing by second intention is a wound in which there is extensive injury and the edges
of the wound are not well approximated. Because of this factor, this type of wound has a risk
of infection.
1.‐ A contusion
2.‐ A wound healing by second intention
3.‐ A septic wound
4.‐ A wound with purulent exudate
3613 A client has a continuously running peripheral Correct answer: 4 Before making a decision about how to infuse the antibiotic, the nurse should check Omit options 1 and 2 as they call for an unnecessary stick. Omit option 3 as the flow rate is
infusion. The physician orders the addition of a compatibility of the antibiotic with the continuous IV solution. If the drug and the infusion not addressed in the item and this does not answer the question.
antibiotic as a piggyback infusion four times per day. In were compatible, they would be run at the same time. If the drug and infusion were
order to administer the antibiotic, the nurse should do incompatible, the nurse would stop the infusion during the period of antibiotic administration
which of the following? and flush the line carefully before and after the antibiotic. It is always inadvisable to start a
second IV site unless absolutely necessary. The other answers are incorrect.

1.‐ Start a new IV access to administer the antibiotic so that there will not be compatibility issues.
2.‐ Start a new IV access to eliminate the problem of too much volume for one site.
3.‐ Increase the flow rate of the continuous infusion to facilitate the administration of the antibiotic.
4.‐ Check to see if the antibiotic is compatible with the continuous infusion.

3614 When learning of the diagnosis of deep vein Correct answer: 2 To gain a client's trust, respect must be conveyed even if there is disagreement with the belief Awareness of nursing interventions to promote spiritual health will assist in selecting the
thrombosis, a client states that "if it is God's will, I will expressed. Introductions and further assessment are important but ineffective if respect is not option that aids in establishing a positive relationship with the client. Recall that religion
get better." Which of the following would be the conveyed. Notifying the physician does not have priority at this time. may provide a framework for a client’s health beliefs.
highest priority intervention in order to provide
culturally competent care?
1.‐ Notify the physician immediately
2.‐ Convey respect for the client's belief
3.‐ Further assess the client's knowledge of the disease
4.‐ Introduce yourself with your title

3615 A 4‐month‐old client has an order for Correct answer: 40 Use the following formula to calculate the rate of IV solutions:<BR /> Use knowledge of basic IV calculation to set up the question. Calculate the problem
D&lt;sub&gt;5&lt;/sub&gt; half NS IV to run at a rate of carefully and double check your answer for accuracy.
40 mL/hr. While the unlicensed assistive person (UAP)
is obtaining an IV infusion pump, the nurse sets the
drip rate at ____ drops/minute, using a Soluset with
microdrip tubing that has a drop factor of 60
gtts/mL.<BR />

3616 A 3½‐month‐old infant has an order for Correct answer: 0.45 The problem can be set up using the following formula:<BR /> Use knowledge of basic dosage calculation to set up the question. Calculate the problem
acetaminophen (Tylenol) suspension 45 mg po q4h carefully and double check your answer for accuracy.
prn. The product label lists a concentration of
500mg/5mL. After determining that the dosage is safe,
the nurse should administer _____ mL. Write the
numerical answer.

3617 A 4‐year‐old client's medication order reads, Correct answer: 2 First calculate the daily dose of the medication by dividing the number of mg (1380) by the Use knowledge of basic pediatric dosage calculation to set up the question. Calculate the
cefotaxime (Claforan) 1380 mg IV every 8 hours. The client's weight in kg (13.8) to yield a single dose of 100 mg/kg. Because there are three doses problem carefully and double check your answer for accuracy. Remember that any dose
client weighs 13.8 kg. Which of the following nursing ordered during a 24‐hour period, multiply 100 by 3 to yield 300 mg/kg/day. Since the dose that falls outside the safe dosage range needs to be questioned.
actions is appropriate if the safe dosage range for a range is 100 to 200 mg/kg/day, the nurse should question the order for the excessively high
child from 1 month to 12 years of age is listed as dose as a safe nursing action.
100–200 mg/kg/day given in divided doses?

1.‐ Give the dose as scheduled and document it appropriately.


2.‐ Question the order for the excessively high dose.
3.‐ Administer the slightly high dose but give it at half the recommended rate.
4.‐ Withhold the dose and question the prescriber, since it is below the recommended range.
3618 A 6‐year‐old client who weighs 18 kg is scheduled to Correct answer: 1 First calculate the total daily dose by dividing 240 mg by 18 kg to yield a single dose of 14.44 Use knowledge of basic pediatric dosage calculation to set up the question. Calculate the
receive a dose of vancomycin (Vancocin) 240 mg IV mg/kg. Multiply the single dose by 4 doses (every 6 hours) to yield a daily total dose of 57.76 problem carefully and double check your answer for accuracy. Remember that any dose
ordered every 6 hours. The safe dose range for a child mg/kg/day. Since this exceeds the safe total daily dose of 40 mg/kg/day, the nurse should that falls outside the safe dosage range needs to be questioned.
is listed as 40mg/kg/day divided every 6 hours. What is question the order.
the best nursing action?

1.‐ Question the dosage of the order.


2.‐ Question the frequency of the order.
3.‐ Administer the dose, being sure to use an infusion pump.
4.‐ Give the dose over at least 60–90 minutes to avoid adverse effects.

3619 A 5‐year‐old client has an order for baclofen (Lioresal) Correct answer: 4 The dose can be administered as ordered. The order for half of a 10 mg tab means that the Use knowledge of basic pediatric dosage calculation to set up the question. Calculate the
half of 10 mg tab po three times per day. The safe dose child is receiving 5 mg per dose. If there are three doses per day, then the total daily dose is 15 problem carefully and double check your answer for accuracy. Recall that any dose that falls
range for a 2–7‐year‐old child is 10–15 mg/day in mg, which is within the safe dosage range. within the safe dosage range may be administered.
divided doses. Which of the following nursing actions is
most appropriate?
1.‐ Question the total daily dose ordered.
2.‐ Question the single dose ordered.
3.‐ Refuse to give the dose because the child’s weight is not factored into the dose.
4.‐ Administer the dose as ordered.

3620 A 3‐year‐old client has an order for 120 mg Correct answer: 6 Convert 2.6 grams to 2600 mg. Since a single dose is only 120 mg, the client could Use knowledge of basic math calculations to determine your answer. If the drug can be
acetaminophen (Tylenol) every 4–6 hours prn for pain. theoretically receive this dose 21 times (2600 divided by 120 = 21.66) within a 24‐hour period given no more frequently than every 4 hours, it cannot exceed six doses, since 24 divided by
The maximum total dose is 2.6 grams/day for a child of without exceeding the top of the dosage range. However, since the medication is ordered only 4 is 6.
2–3 years. The nurse could legally administer the every 4–6 hours, the nurse can legally administer the medication only six times maximum
medication to this child ____ times during a 24‐hour (every 4 hours).
period. Write the numerical answer.

3621 The client has an order for cefotaxime (Claforan) 1180 Correct answer: 12.4 The following formula illustrates one way to set up the problem:<BR /> Use knowledge of basic dosage calculation procedures to set up the question. Calculate
mg IV q6h. The reconstituted medication vial is labeled the problem carefully and double check your answer for accuracy.
as having a concentration of 95 mg/mL. The nurse
should draw up _____ mL of solution to add to the bag
of IV solution that will be used for the intermittent
infusion. Write the numerical answer, rounding to the
nearest tenth.

3622 The 6‐year‐old client has an order for fexofenadine Correct answer: 60 One half of a 60 mg tablet is 30 mg. Because the dose is ordered twice a day, the total daily Use knowledge of basic math calculation to set up the problem. Calculate carefully and
(Allegra) half of a 60 mg tab po twice daily. The nurse dose is 30 multiplied by 2, or 60 mg. double check your answer for accuracy.
calculates the child’s total daily dose as ____ mg.
Write a numerical answer, rounding to the nearest
tenth.

3623 The nurse is reviewing insulin administration Correct answer: 4 Correct administration technique is to use a 90‐degree angle (insulin syringes have a short, The core issue of the question is knowledge of subcutaneous injection techniques. Use the
techniques with a 13‐year‐old client with uncontrolled half‐inch needle), avoiding aspiration before injection (e.g., to avoid tissue complications over process of elimination and nursing knowledge to answer the question. To help eliminate
diabetes. The nurse evaluates that the client is using time), and avoiding massaging the area after injection, which would enhance quicker incorrect answers, recall that insulin and heparin should not be massaged and that a 90‐
proper procedure after noting that the client does absorption of the dose. degree angle is used for injection.
which of the following during self‐injection?

1.‐ Aspirates before injection but does not massage the site following injection
2.‐ Uses a 45‐degree injection angle and aspirates before injection
3.‐ Uses a 90‐degree angle and massages the site following injection
4.‐ Uses a 90‐degree injection angle and does not massage the site following injection

3624 A 15‐year‐old client admitted with dehydration has an Correct answer: 83 Since the infusion device delivers fluid in drops/min, the nurse must calculate the number at Use knowledge of basic IV calculation procedures to set up the question. Calculate the
order for a bolus infusion of normal saline (NS) 500 mL which to set the machine. The formula to use is:<BR /><BR /> problem carefully and double check your answer for accuracy.
IV for 1 hour. An infusion device is available that
counts the number of drops per minute delivered. The
IV tubing has a drop factor of 10 drops/mL. If the bolus
is to infuse on time, the nurse should set the drip rate
to ____ drops per minute. Write the numerical
answer, rounding to the nearest whole number.

3625 A 6‐year‐old postoperative client has a medication Correct answer: 100 First convert the client's weight to kg by dividing 44 by 2.2 to yield a weight of 20 kg. Then Use knowledge of basic dosage calculation to set up the question. Calculate the problem
order for cefazolin (Ancef) 500 mg IV every 6 hours. calculate the daily dose of the medication by dividing the number of mg (500) by the client's carefully and double‐check your answer for accuracy.
The client weighs 44 pounds. The safe dose range of weight in kg (20) to yield a single dose of 25 mg/kg. Because there are four doses (every 6
cefazolin for a child is 25–100 mg/kg/day in three to hours) ordered during a 24‐hour period, multiply 25 by 4 to yield a total daily dose of 100
four divided doses. The total daily dose that this client mg/kg/day.
will receive is _____ mg/kg/day. Write the numerical
answer as a whole number.

3626 A child who sustained a head injury has an order for Correct answer: 66.7 Since the infusion pump delivers fluid in mL/hour, the nurse needs to calculate the equivalent Use knowledge of basic math calculation to set up the question. Read the question
mannitol (Osmitrol) 20 grams. Available is a bag of 20% hourly rate when infusing the 100 mL over 90 minutes. The problem can be set up to cancel carefully, noting that the time needs to convert from minutes to hours in order to have the
solution that contains 20 grams mannitol in 100 mL out the labels and end up with mL/hour:<BR /> correct labeling. Calculate the problem carefully and double check your answer for
volume. The medication may be administered over accuracy.
30–90 minutes. If the nurse wishes to infuse the
medication over 90 minutes, the infusion pump should
be set at _____ mL/hour if the pediatric infusion pump
can be set to include tenths of a milliliter. Write in a
numerical answer, rounding to the nearest tenth.

3627 A client has a medication order for ceftazidime Correct answer: 30 First convert the child’s weight to kg by dividing 55 by 2.2 to yield a weight of 25 kg. Then Use knowledge of basic pediatric dosage calculation to set up the question. Calculate the
(Fortaz) 250 mg IV every 8 hours. The client weighs 55 calculate the daily dose of the medication by dividing the mg (250) by the client's weight in kg problem carefully and double check your answer for accuracy.
pounds. The safe dose range of ceftazidime for a child (25) to yield a single dose of 10 mg/kg. Because there are three doses (every 8 hours) ordered
is 30–50 mg/kg/day in three divided doses. The total during a 24‐hour period, multiply 10 by 3 to yield a total daily dose of 30 mg/kg/day.
daily dose ordered for this client is _____ mg/kg/day.
Write in a numerical answer, rounding to the nearest
whole number.

3628 A child has an order to receive an NS bolus 400 mL IV. Correct answer: 267 Since the infusion pump delivers fluid in mL/hour, the nurse must calculate the equivalent Use knowledge of basic math calculation to set up the question. Read the question
If the nurse wishes to infuse this volume for 90 hourly rate when infusing the 400 mL over 90 minutes. The problem can be set up as follows to carefully, noting that the time needs to convert from minutes to hours in order to have the
minutes, the infusion pump should be set at ____ cancel out the labels and end up with mL/hour:<BR /> correct labeling. Calculate the problem carefully and double check your answer for
mL/hour. Write a numerical answer, rounding to the accuracy.
nearest whole number.
3629 The nurse is about to administer a dose of Correct answer: 12.5 The problem can be set up using ratio and proportion as follows:<BR /> Use knowledge of basic pediatric dosage calculation to set up the question. Calculate the
acetaminophen (Tylenol) 200 mg via the NG tube to a problem carefully and double check your answer for accuracy.
child. Available is a suspension with a concentration of
80 mg per 5 mL. The nurse should administer _____
mL to give the dose. Write a numerical answer,
rounding to the nearest tenth.

3630 A pediatric client has been diagnosed with Correct answer: 4 Eye drops are placed in the lower conjunctival sac to prevent damage to the cornea and to Option 4 describes the area of the eye that will allow the medication to remain in contact
conjunctivitis. The nurse is to administer eye drops facilitate coating the eye with the medication. The other answers are incorrect. with the infected area.
four times a day. The nurse should administer the
medication by gently dropping the medication onto
which of the following areas?

1.‐ Center of the cornea


2.‐ Sclera by the inner canthus
3.‐ Sclera by the outer canthus
4.‐ Lower conjunctival sac

3631 A mother is to be taught to administer ear drops to Correct answer: 4 Pulling the ear pinna down and back straightens the ear canal allowing the drops to enter the Option 4 has the most direct relationship to the actual teaching needed to accomplish the
her 4‐month‐old infant. The important concept in ear. The other choices are not the most important concepts in the teaching. task.
teaching would be to do which of the following?

1.‐ Wear gloves when administering the ear drops.


2.‐ Not contaminate the bottle by touching the nozzle to the ear.
3.‐ Turn the baby on its back after administration to avoid the risk of SIDS.
4.‐ Pull the pinna gently downward and backward.

3632 The nurse is administering an intradermal tuberculin Correct answer: 1 For an intradermal injection, the needle enters the skin at a 10‐ to 15‐degree angle and the The core issue is depth of penetration of medication utilizing correct needle angle. Choose
skin test to a client. The client comments that this medication forms a bleb under the epidermis. The other angles would permit the medication the option that has the smallest angle, which will keep the injection from going too deeply.
"shot" is different than other shots in the past. The to be deposited too deeply into either subcutaneous or muscle tissue, depending on needle
nurse explains that because the medication goes into length and size of client.
the dermal tissue, the angle for intradermal injections
is:
1.‐ 10–15 degrees.
2.‐ 30–40 degrees.
3.‐ 45 degrees.
4.‐ 90 degrees.

3633 The nurse is preparing to administer a less viscous Correct answer: 4 Several factors indicate the size and length of the needle to be used: the muscle, the type of The critical concepts in the question are that the deltoid muscle allows the use of a shorter
(i.e., watery) intramuscular injection into the deltoid solution, the amount of adipose tissue covering the muscle, and the age of the client. A smaller needle and a watery solution allows the use of a smaller gauge needle. Use the process of
muscle of a 160‐pound male. What is the preferred needle such as a 23‐ to 25‐gauge needle that is 1 inch long is commonly used for the deltoid elimination to choose the option that combines these concepts.
needle size for the medication, muscle, and weight of muscle. More viscous solutions require a larger gauge (e.g., 20 gauge). The other answers are
the client? less appropriate because of incorrect needle length or gauge.

1.‐ 1.5 inch, 20 gauge


2.‐ 1 inch, 20 gauge
3.‐ 1.5 inch, 25 gauge
4.‐ 1 inch, 25 gauge
3634 A nurse giving an intramuscular injection places the Correct answer: 2 The ventrogluteal site is in the gluteus medius muscle with the greater trochanter, the Use the process of elimination. Basic knowledge of injection sites and landmarks is
heel of the hand on the client's greater trochanter, anterior superior iliac spine, and the iliac crest as the landmarks. The vastus lateralis and the necessary to answer this question. Choose the option that matches the description given in
with the fingers pointing toward the client's head. The rectus femoris are located on the thigh, and the dorsogluteal is located on the buttocks, the question.
nurse places the index finger on the client's anterior making the other options incorrect.
superior iliac spine, while the middle finger is stretched
dorsally, palpating the iliac crest. After giving the
injection in the triangle formed, the nurse documents
the injection as being given in which intramuscular
injection site?

1.‐ Vastus lateralis


2.‐ Ventrogluteal
3.‐ Dorsogluteal
4.‐ Rectus femoris

3635 The nurse feels a client is at risk for skin breakdown Correct answer: 3 This is a risk diagnosis, and diagnostic statement has two parts: the human response The critical phrase is at risk for skin breakdown. Use knowledge that nursing diagnoses are
because the client has only had clear liquids for the (impaired skin integrity) and the related/risk factor (malnutrition). Options 1 and 2 do not have developed to address client responses that are amenable to nursing intervention to
last 10 days (and essentially no protein intake). The related factors that are under the control of the nurse (i.e., type of diet ordered). The diagnosis eliminate options 1 and 2. Note that the diagnosis in option 4 is incomplete to choose
nurse would formulate which diagnostic statement to in option 4 does not specify the type of impairment (greater than or less than body option 3.
best reflect this problem? requirements) and is therefore incomplete. It also does not provide direction for development
of goals and interventions.
1.‐ Risk for malnutrition related to clear liquid diet
2.‐ Impaired skin integrity related to no protein intake
3.‐ Risk for impaired skin integrity related to malnutrition
4.‐ Impaired nutrition related to current illness

3636 A client is postoperative with an IV in place. The client Correct answer: 3 The essential parts of a drug that must be present in order to implement the order are name The core issue of the question is knowledge that medication cannot be given with an
is taking a soft diet and, when asked, rates the pain as of the drug, date and time the order was written, dosage, route, frequency, and signature of incomplete order. Eliminate options 1 and 2 because they do not specify a route in the
6 on a 1–10 scale. The following order is noted in the the person writing the order. Nurses may not independently administer a medication without original order, and eliminate option 4 because it gives a medication without an order.
client’s chart: morphine sulfate 6–8 mg q 4 hr prn for all of the essential parts or determine a route based upon the client’s condition. Administering
pain. Considering the client's pain level and noting that Tylenol without a medical order constitutes practicing medicine without a license. In
no route was ordered, the nurse should do which of hospitalized clients, an order must be present for any medication to be given.
the following?

1.‐ Administer the dose intramuscularly (IM) because the client is on a soft diet and this is a safe IM dose.
2.‐ Recognizing this to be a safe IV dose, administer the dose IV until clarification is received from the physician.
3.‐ Withhold the dose and contact the physician for clarification of the order.
4.‐ Withhold the dose until a route is ordered by the physician, but administer Tylenol from stock supplies since these are available over the counter.

3637 The physician asks the nurse to take a telephone Correct answer: 2 A nurse can take a telephone order from a physician. When the nurse documents the order, The core issue of the question is knowledge of how to properly record a telephone order.
order for acetaminophen (Tylenol) 500 mg by mouth q "telephone order" and the physician's name must be written on the order and the physician The wording of the question tells you that only one option is correct. Use the process of
4 hr prn for a temperature elevation higher than 100° must cosign it, usually within 24 hours. The other answers are incorrect. Option 1 is a false elimination and basic nursing knowledge to make a selection.
F. How should the nurse respond? statement, option 3 fails to note that it is a telephone order, and option 4 is unnecessary.

1.‐ Explain to the physician that nurses are not permitted to write orders.
2.‐ Record the order with "telephone order" from physician and the nurse's signature, and remind the physician to cosign it within the next 24 hours.
3.‐ Record the order and sign the physician's name first, followed by the nurse's signature.
4.‐ Ask the physician to restate the telephone order with another nurse witnessing and record the order with both the witness and the nurse's name.
3638 The nurse is administering a medication to a client Correct answer: 1 Blood levels of two metabolically produced substances, urea and creatinine, are routinely The core issue of the question is knowledge of what to assess to determine kidney
with a history of renal impairment. The medication is used to evaluate renal function. Both are normally eliminated by the kidneys and are measured function as an indicator of clearance of medications. Use the process of elimination and
known to be excreted through the kidneys. To monitor as serum BUN and creatinine. The color and odor of the urine are general observations (option basic nursing knowledge to make a selection.
the client for adverse reactions, the nurse would 2). Sugar and acetone in urine are found in diabetes mellitus with ketoacidosis (option 3).
monitor which of the following? Serum hemoglobin (option 4) is a measure of the RBC count but does not reflect kidney
function.
1.‐ Serum blood urea nitrogen (BUN) and creatinine
2.‐ Color and odor of the urine
3.‐ Urine sugar and acetone levels
4.‐ Serum hemoglobin

3639 A client has a continuously running peripheral Correct answer: 4 Before making a decision about how to infuse the antibiotic, the nurse should check The core issue of the question is the need to check compatibility of medications and IV
intravenous (IV) infusion. The physician orders the compatibility of the antibiotic with the continuous IV solution. If the drug and the infusion solutions as a beginning point to decision‐making. Eliminate each of the incorrect options
addition of an antibiotic as a piggyback infusion 4 times were compatible, they would run together through the same line. If the drug and infusion were because they do not begin with compatibility checks (options 1 and 2) or are incorrect
a day. In order to administer the antibiotic safely, the incompatible, the nurse would stop the infusion during the period of antibiotic administration nursing actions (option 3).
nurse should do which of the following? and flush the line carefully before and after the antibiotic. It is always inadvisable to start a
second IV site unless absolutely necessary. Increasing the IV flow rate constitutes changing a
medical order, and does not address the issue of compatibility.

1.‐ Start a new IV access to administer the antibiotic so that there will not be compatibility issues.
2.‐ Start a new IV access to eliminate the problem of too much volume for one site.
3.‐ Increase the flow rate of the continuous infusion to facilitate the administration of the antibiotic.
4.‐ Check to see if the antibiotic is compatible with the continuous infusion.

3640 The nurse is preparing to administer an oral Correct answer: 3 The correct action should be to withhold the medication and call the physician. Nurses cannot Use the process of elimination and general measures for administering medications safely
medication to a client. Upon entering the client's independently change the route of a medication. Oral medications should not be administered to make a selection. The wording of the question tells you that there is only one correct
room, the nurse finds that the client's condition has to clients who are vomiting, which could interfere with the ability to absorb the medication answer.
changed and the client is now vomiting, has diarrhea, and possibly initiate further vomiting. The nurse should not just omit the dose without
is confused, and has a fever. What should the nurse do notifying the physician of the client's change in condition.
next?
1.‐ Administer the medication IM.
2.‐ Wait 2 hours and give the medication if vomiting subsides.
3.‐ Withhold the medication and call the physician.
4.‐ Omit this dose of medication.

3641 A nurse has prepared an IM injection for a Correct answer: 1 The nurse who prepares the medication must be the nurse to give the medication. It would Use the process of elimination. The core issue of the question is the principle that nurses
preoperative client. Suddenly, another client becomes be prudent for the second nurse to assist the second client so that the first nurse may continue may not administer medications prepared by another nurse. The wording of the questions
entangled in an IV tubing and yells for help. The nurse medication administration. Option 2 is incorrect. Option 3 is acceptable but requires tells you there is only one correct answer.
rushes to assist. The surgery orderly is waiting for the destruction of the original medication, which is an added expense. Option 4 is appropriate but
preoperative client, so the nurse asks a second nurse does not resolve the issue of the preoperative client.
to give the injection to the preoperative client. Which
of the following is the best response by the second
nurse?

1.‐ Help the second client so the nurse can give the preoperative client the injection.
2.‐ Give the client the preoperative medication.
3.‐ Prepare a new syringe for the preoperative client.
4.‐ Explain to the nurse that no other nurse can administer the already prepared medication.
3642 The client is in the bathroom. When the nurse enters Correct answer: 4 Medications should not be left at the bedside, with certain exceptions that are ordered in Use the process of elimination. There is one correct answer utilizing basic principles of
the room to give medications, the client asks the nurse advance (e.g., nitroglycerin and cough syrup). The other answers are not prudent nursing medication administration.
to leave the pills on the bedside table. What is the best actions because they either fail to ensure that the medication is taken (option 1), waste the
nursing action? nurse's time (option 2), or invade the client's privacy unnecessarily (option 3).

1.‐ Leave the medication on the bedside table.


2.‐ Wait in the room until the client comes out of the bathroom.
3.‐ Go into the bathroom and give the client the pills.
4.‐ Tell the client you will return in a little while with the medication.

3643 The client has an order for dexamethasone Correct answer: 1.5 Use the following formula to solve the problem:<BR /> Use knowledge of basic pharmacological math to set up the question. Carefully review
(Decadron) 6 mg IV push stat. Available is a vial of your work and double‐check placement of decimals for accuracy.
dexamethasone with a concentration of 4 mg/mL. The
nurse draws _____ mL into the syringe to administer
the dose. Write a numerical answer.

3644 The client has an order for cefotaxime (Claforan) 1 Correct answer: 10.5 One gram is equal to 1000 mg. Use the following formula as one way to set up the Use knowledge of basic pharmacological math to set up the question. Carefully review
gram IV q6h. The reconstituted vial in the client's problem:<BR /> your work and double‐check placement of decimals for accuracy.
medication drawer is labeled with a concentration of
95 mg/mL. For the intermittent infusion _____
milliliters of solution should be added to the IV bag.
Write a numerical answer.

3645 The client has an order for glyburide (DiaBeta) 1.25 Correct answer: 0.5 The following is one way to set up the calculation:<BR /> Use knowledge of basic pharmacological math procedures to set up the question. Check
mg before breakfast and dinner. Available are 2.5 mg your work carefully and double check placement of decimals for accuracy.
tablets. The nurse should plan to administer ____
tablets. Write in a numerical answer.

3646 The client has an order for lorazepam (Ativan) 0.5 mg Correct answer: 0.25 The following is one way to set up the calculation:<BR /> Use knowledge of basic pharmacological math procedures to set up the question. Carefully
IV q6h PRN for agitation. Available is a vial containing review your work and double‐check placement of decimals for accuracy.
2mg/mL. The nurse should draw up ____ mL of
solution for injection. Write in a numerical answer.

3647 A client has a temperature of 101.2° F. There is an Correct answer: 2 The following is one way to set up the calculation:<BR /> Use knowledge of basic pharmacological math procedures to set up the question. Check
order for acetaminophen (Tylenol) 650 mg PO for your work carefully.
fever, and 325 mg tablets are available. The nurse
should give ____ tablets. Write a numerical answer.

3648 A client has an order for cefazolin (Ancef) 2 grams Correct answer: 20 Since the dose is 2 grams and each vial contains 1 gram, the nurse needs to use two vials. The Use knowledge of basic pharmacological math procedures to set up the question. Check
IVPB. Available are vials filled with powder containing 1 nurse then adds 10 mL of sterile water to each vial of powder based on the direction to "add your work carefully.
gram of cefazolin. The instructions state to "dilute 10 mL of sterile water per 1 gram of medication." Once both vials are reconstituted, the
each 1 gram with 10 mL of sterile water." After concentration of each solution is 1 gram/10 mL. The nurse then must draw up the contents of
reconstituting the medication, ____ total milliliters of both vials, making the total volume 20 mL.
solution should be drawn up to prepare the dose.
Write in a numerical answer.
3649 A client has an order for a dose of digoxin (Lanoxin) Correct answer: 2 The following is one way to set up the calculation:<BR /> Use knowledge of basic pharmacological math to set up the question. Carefully review
0.25 mg IV push. Available is a vial containing 0.125 your work and double‐check placement of decimals for accuracy.
mg/mL. The nurse should draw up ____ mL to
administer the dose. Write a numerical answer.

3650 The client has an order to receive methylprednisolone Correct answer: 3 The following is one way to set up the calculation:<BR /> Use knowledge of basic pharmacological math procedures to set up the question. Check
(SoluMedrol) 120 mg IVPB q6h. Available is a solution your work carefully for accuracy.
containing 40 mg/mL. _____ mL of medication should
be added to the IV piggyback solution. Write a
numerical answer.

3651 The client has an order to receive 40 mg prednisone Correct answer: 4 The following is one way to set up the calculation:<BR /> Use knowledge of basic pharmacological math to set up the question. Check your work
(Deltasone) by mouth daily. Available are 10 mg carefully for accuracy.
tablets. The nurse should prepare to give ____ tablets.
Write a numerical answer.

3652 The client is a known diabetic. The nurse administers Correct answer: 3 Under the law, if a medication order is written incorrectly, the nurse who administers the Only option 3 acknowledges that the nurse is responsible for his or her own actions
20 units NPH insulin IV stat per the physician's order. incorrect order is responsible for the error. This includes both the right medication and the regardless of the incorrect route in the physician’s order.
Subsequent to the client receiving the insulin dose, the right dose (two of the six "rights" of medication administration). The other responses are
client had an anaphylactoid reaction and died as a incorrect.
result of receiving the NPH insulin via IV rather than
subcutaneous, the only appropriate route. What
liability is involved in this case?

1.‐ The nurse is not legally liable because the medication was administered as ordered by the physician.
2.‐ Only the physician is liable because the physician wrote the order.
3.‐ The nurse is legally liable for the medications administered even though the order was written incorrectly.
4.‐ The nurse is not legally liable because the nurse gave the correct medication, regardless of the route.

3653 While the nurse is administering a client's dose of Correct answer: 1 The thin layer of epithelium and the vast network of capillaries under the tongue enhance Note that options 1 and 2 are opposites. When opposites are offered, one of the opposites
nitroglycerin sublingual, the client asks why it is sublingual absorption. This medication dissolves rapidly and is absorbed immediately. The is usually correct. Option 1 gives the correct information about the drug.
administered sublingually rather than orally. Which of other responses are incorrect.
the following is the best response by the nurse?

1.‐ "It is absorbed more rapidly sublingually than when swallowed."


2.‐ "It is absorbed more rapidly when swallowed than sublingually."
3.‐ "The absorptions are the same so it really doesn't matter."
4.‐ "Sublingual provides a sustained release of the medication."

3654 The nurse is to administer 25 mg of promethazine Correct answer: 4 For an adult with well‐developed muscle mass, the preferred intramuscular (IM) injection site Use the process of elimination to narrow options. Dorsogluteal is not recommended for
(Phenergan) IM to a 150‐pound client. The nurse for medications requiring a large muscle mass is the ventrogluteal. The vastus lateralis is the anyone. Vastus lateralis is recommended for injection in children who are very young and
knows that this medication should be given into a deep preferred IM injection site for children under 7 months of age. The other responses are cannot yet walk. Ventrogluteal is larger and better developed in most adults than the
large muscle mass. The preferred site of injection for incorrect. deltoid.
this client would be which of the following?

1.‐ Deltoid
2.‐ Dorsogluteal
3.‐ Vastus lateralis
4.‐ Ventrolgluteal

3655 To administer 1 mL of a flu vaccine intramuscularly Correct answer: 2 For a well‐developed adult, a 5/8–1‐inch needle is the appropriate size for an IM deltoid The range for an average size adult is 5/8–1 inch. For deltoid injection in an obese client,
(IM) to an obese adult in the deltoid area the nurse injection. Because this is an obese client, the longer needle is appropriate to ensure it reaches select the larger.
would use what size needle? the muscle. The other responses are incorrect.

1.‐ 5/8 inch


2.‐ 1 inch
3.‐ 1½ inch
4.‐ 2 inch

3656 The nurse is preparing an IM injection of hydroxyzine Correct answer: 4 Z‐track technique prevents "tracking" and is used for administering medications that are Note the use and emphasis on “tracking.” Prevention of tracking is accomplished by z‐track
(Vistaril) that is especially irritating to subcutaneous especially irritating to subcutaneous tissue. With z‐track, the skin is pulled approximately 1 method.
tissue. To prevent "tracking" of the medication and inch laterally away from the injection site, the medication is injected, the needle withdrawn
irritation to the tissues, it is best to take which of the and the tissue released. The other answers are incorrect.
following actions?
1.‐ Use a small‐gauge needle
2.‐ Administer at a 45‐degree angle
3.‐ Apply ice to the injection site
4.‐ Use the z‐track technique

3657 The client is to receive vancomycin (Vancocin), an Correct answer: 3 The device that provides the most accurate infusion rate is the electronic infusion pump. The The scenario stresses that rapid infusion can be life‐threatening, so select the option that
intravenous medication. Too rapid administration of other devices are less accurate. provides the most control over the rate of infusion.
vancomycin is associated with life‐threatening adverse
reactions. The nurse would plan to administer this drug
using which of the following methods?

1.‐ Gravity
2.‐ Regulator
3.‐ Electronic infusion pump
4.‐ Elastomeric pump

3658 The nurse is to administer 10 grains of aspirin, which Correct answer: 2 In the apothecary system, 1 grain equals 60–65 mg. 10 grains equal 600–650 mg. Therefore, 2 Use knowledge of apothecary system and simple calculation.
comes 325 mg per tablet. The nurse would give _____ tablets would be needed at 325 mg each to reach the needed dose.
tablets to administer 10 grains? Write a numerical
answer.

3659 While administering an intramuscular (IM) injection Correct answer: 4 If blood returns while aspirating during an IM injection, the nurse should discard and prepare Note that only one of the options directs the nurse to discard the medication. This is the
of meperidine (Demerol), the nurse aspirates and finds a new injection. Blood indicates that the needle has entered a blood vessel, and medication correct option.
blood in the syringe prior to injecting the medication. injected directly into the bloodstream may be dangerous.
Which of the following actions by the nurse would be
appropriate?
1.‐ Continue to administer the medication because it is compatible with blood and would not present a harmful effect.
2.‐ Continue to administer the medication because the needle has hit a capillary and would not be an intravenous administration.
3.‐ Withdraw the needle, cleanse the needle and the new injection site with alcohol, and administer the medication.
4.‐ Withdraw the needle, discard the medication, and begin again with the medication administration.
3660 The nurse has instructed the client in using a metered‐ Correct answer: 2 Clients should be instructed to hold inhaler 2 inches away from mouth, hold the breath for 10 Use process of elimination to omit options 1and 4 because medication will not remain in
dose inhaler. The nurse determines that the client seconds, slowly exhale through pursed lips, and wait 2 minutes between puffs. The other contact with the respiratory tree in the desired dose with either option. Omit option 3 as
understands the instructions when the client is options are incorrect. shaking is recommended.
observed doing which of the following?

1.‐ Administering the two puffs in rapid order between breaths


2.‐ Holding the inhaler 2 inches away from the mouth
3.‐ Not shaking the canister before puffs
4.‐ Exhaling immediately after administering the puff

3661 A client is receiving a continuous PEG tube feeding. Correct answer: 3 When medications are administered enterally and cannot be administered with tube Note that options 1 and 2 pass responsibility to the physician and do not make good sense
The physician has ordered phenytoin (Dilantin) to be feedings, it is best to stop the tube feedings for at least 30 minutes prior to and after the in regard to this drug.
administered through the PEG tube. The nurse notes administration of the medication. A time period of thirty minutes allows for the tube feeding
that the medication cannot be administered with tube to clear the GI tract and therefore not mix with the medication. The other answers are
feedings. Which of the following would be the best incorrect.
intervention?
1.‐ Contact the physician for an order to administer the Dilantin by another route
2.‐ Contact the physician to change the type of tube feeding to one that is compatible.
3.‐ Stop the tube feeding for at least 30 minutes before and after the administration of the Dilantin.
4.‐ Stop the tube feeding, flush the feeding tube with water, administer the Dilantin, flush the tube feeding again with water and continue the tube feeding.

3662 A client is refusing to take her daily anti‐hypertensive Correct answer: 3 A client has the right to refuse a medication regardless how important it may be to their Select the option which best allows for client autonomy.
medication. The nurse has explained to the client why health. Withholding the medication because of client refusal does not require an incident
the medication is important and the client verbalizes report, but it should be documented and reported to the physician. The other answers are
understanding but still refuses. Which of the following incorrect.
is the best nursing action?

1.‐ Administer the medication because it is important for the client.


2.‐ Inform the client that the medication needs to be taken until the nurse gets an order to discontinue it.
3.‐ Withhold the medication and report it to the physician.
4.‐ Withhold the medication and complete an incident report.

3663 Which of the following findings would be of greatest Correct answer: 4 Options 1, 2, &amp; 3 are expected findings and should not be considered cause for concern. The critical words in the question are greatest concern. Recall the significance of common
concern to the nurse taking a client’s pulse? A pulse deficit, with an apical rate greater than the peripheral pulse rate may be a sign of variations in pulse rates and volumes to enable you to recognize significant changes and
significant vascular disease or cardiac dysfunction and should be reported immediately. take appropriate action.

1.‐ Mild tachycardia in a febrile client


2.‐ Mild bradycardia in a young, otherwise healthy sleeping male
3.‐ 18‐month‐old with a heart rate of 120/minute
4.‐ Pulse deficit with an apical rate of 88 and a peripheral pulse of 72

3664 The nurse is preparing to administer a less viscous Correct answer: 4 Several factors indicate the size and length of the needle to be used: the muscle, the type of Select the combination of shortest length and smallest gauge.
(i.e., watery) IM injection into the deltoid muscle of a solution, the amount of adipose tissue covering the muscle, and the age of the client. A smaller
160‐pound male. What would be the preferred size needle such as an inch‐long, #23–25 gauge needle is commonly used for the deltoid muscle.
needle for the medication, muscle, and size of the More viscous solutions require a larger gauge (e.g., #20 gauge). The other answers are
client? incorrect.
1.‐ 1½‐inch, #20 gauge
2.‐ 1‐inch, #20 gauge
3.‐ 1½‐inch, #25 gauge
4.‐ 1‐inch, #25 gauge
3665 A nurse is observed giving an intramuscular injection. Correct answer: 2 The vastus lateralis and the rectus femoris are located on the thigh. The dorsogluteal is Eliminate options 1 and 3 based on location in the thigh. Dorsal (dorsogluteal) means back.
The nurse places the heel of the hand on the client's located on the buttocks. The ventrogluteal site is in the gluteus medius muscle with the greater The area described is ventral.
greater trochanter, with the fingers pointing toward trochanter, the anterior superior iliac spine, and the iliac crest as the landmarks. The other
the client's head. With the nurse's index finger on the answers are incorrect.
client's anterior superior iliac spine, the middle finger
is stretched dorsally, palpating the iliac crest. The index
finger, the third finger, and the iliac crest form a
triangle land marking which intramuscular injection
site?

1.‐ Vastus lateralis


2.‐ Ventrogluteal
3.‐ Dorsogluteal
4.‐ Rectus femoris

3666 A client is postoperative with an IV in place. The client Correct answer: 3 The essential parts of a drug that must be present in order to implement the order are: name Option 3 is the only option which recognizes that the order described is missing a
is taking a soft diet and, when asked, rates the pain as of the drug, date and time the order was written, dosage, route, frequency, and signature of component. That medication cannot be administered until the order is complete. Over‐the‐
a 9 on a 1–10 scale. The following order is noted in the the person writing the order. Nurses may not administer a medication without all of the counter medication (option 4) requires an order in this situation.
client's chart: Demerol 50 to 75 mg q 4 hr prn for pain. essential parts, or determine a route based upon the client's condition. Administering Tylenol
Considering the client's pain level and noting that no without a medical order is practicing medicine without an order.
route was ordered for the Demerol, the nurse should
do which of the following?

1.‐ Administer the Demerol IM since the client is on a soft diet and the nurse recognizes this to be a safe IM dose.
2.‐ Recognizing this to be a safe IV dose, administer the Demerol IV until clarification is received from the physician.
3.‐ Withhold the Demerol until a route is ordered from the physician.
4.‐ Withhold the Demerol until you get a route ordered from the physician; administer Tylenol from stock supplies since these are available over the counter.

3667 While making rounds with the nurse, the physician Correct answer: 2 A nurse can take a verbal order from a physician. When the nurse transcribes the order, Select the option that returns the responsibility to the physician.
asks the nurse to write an order for Tylenol 500 mg q 4 "verbal order" is recorded with the order and the physician must co‐sign the order usually
hr prn for a temperature elevation &gt;100. What within 24 hours. The other answers are incorrect.
should be the nurse's response?

1.‐ Explain to the physician that nurses are not permitted to write orders.
2.‐ Record the order with "verbal order" written with the nurse's signature and remind the physician to co sign it within the next 24 hours.
3.‐ Record the order and sign the physician's name first, followed by the nurse's signature.
4.‐ Ask the physician to restate the verbal with another nurse witnessing and record the order with both the witness and the nurse's name.

3668 The nurse is administering a medication to a client Correct answer: 1 Blood levels of two metabolically produced substances, urea and creatinine, are routinely Select the option which addresses renal function, rather than appearance of urine or
with a history of renal impairment. The medication is used to evaluate renal function. Both are normally eliminated by the kidneys and are measured evaluation of urine characteristics.
known to be excreted through the kidneys. To monitor as serum BUN and creatinine. The other answers are incorrect.
the client for adverse reactions, the nurse would
monitor which of the following?

1.‐ Serum BUN and creatinine


2.‐ Color and odor of the urine
3.‐ Urine sugar and acetone levels
4.‐ Serum hemoglobin
3669 Which of the following assessment data would Correct answer: 4 The client doesn't need prior experience to gain the benefits of imagery. The client reporting To answer this question, the nurse needs to know what clients will benefit from imagery
prohibit the use of imagery with a client? a 6 on a 0–10 scale will benefit from the imagery session. Closing the eyes aids in establishing a and what the contraindication is for this technique.
state of internal awareness but is not necessary for an imagery session. The client could gaze
at a fixed point 1–2 feet away instead of closing his or her eyes. When the client begins to trust
the process, his or her eyes will get heavy and close. In clients with a history of organic brain
syndrome or psychosis, deep relaxation may exacerbate symptoms of psychosis. Other
relaxation methods should be instituted.

1.‐ No previous history of using imagery techniques


2.‐ States anxiety level of 6 on a 0–10 scale
3.‐ Client feels reluctant to close eyes for the imagery session
4.‐ Client has a history of psychosis

3670 The nurse has determined that music therapy may be Correct answer: 3, 4, 5 Music without words is recommended so that the client does not concentrate on the words. The answer to this question reflects the critical elements when using music therapy with a
appropriate for use with a client. Which of the Music therapy needs to be at least 20 minutes in length to be effective. Music selections client.
following should the nurse consider when choosing the should be based on the type of music the client perceives as relaxing. The nurse should
music? Select all that apply. encourage the client to let the body respond to the music in any way it wishes, such as
humming, relaxing muscles, or clapping. Analyzing the music will take away the focus from
relaxing to thinking about the music. Any distracting thoughts should simply be let go, and the
client should be instructed to concentrate on the music.

1.‐ Choose only music with words.


2.‐ Choose music that is 5–7 minutes in duration.
3.‐ Allow the client to choose music of his or her choice.
4.‐ Instruct the client to listen to the music and let the music take him or her wherever the music wants to go.
5.‐ Ask the client not to analyze the music.

3671 The nurse is using meditation with a client to help Correct answer: 2 The client needs to be involved in deciding which type of meditation to learn. The client’s This question requires that you know when it is appropriate for the nurse to utilize
him decrease his pain. Which of the following factors is condition or schedule may prohibit the use of this holistic therapy. Meditation can be used in meditation.
important to consider when using this type of therapy? any facility and is not restricted to outpatient settings. Anyone can learn how to meditate.

1.‐ The type of meditation is best determined by the nurse or other health care provider.
2.‐ Consideration of the client's condition, schedule, and personal preference is necessary when choosing a type of meditation.

3.‐ The type of meditation used is based on whether it is taught in an inpatient or outpatient setting.
4.‐ A certified meditation professional should be the one teaching the client how to perform meditation.

3672 The client asks the nurse how humor therapy affects Correct answer: 2, 3 Sympathetic nervous system is stimulated with humor therapy, leading to an increase in heart An understanding of the pathophysiological effects of humor on the body is needed in
the client physiologically. The nurse responds that rate, respiratory rate, blood pressure, and oxygen saturation. The arousal state is followed by a order to answer this question correctly.
laughter has which effect(s)? Select all that apply. relaxation state in which vital signs return to or below pre‐laughter baseline. Research suggests
that humor therapy increases IgA levels in the saliva, which helps prevent upper respiratory
infections. Laughter decreasesstress hormones such as cortisol. Laughter increases T
lymphocyte cells, thereby increasing the immune response. Temperature is not affected.

1.‐ Decreases heart rate and oxygen saturation


2.‐ Increases salivary immunoglobulin A (S‐IgA)
3.‐ Produces an antagonist response to stress hormones
4.‐ Decreases the immune response by decreasing T‐lymphocytes
5.‐ Increases temperature set poin in the brain
3673 The nurse decides to teach a client with hypertension Correct answer: 3 The client needs to be in a comfortable position for relaxation to occur. Electromyogram Knowledge of the process involved in performing progressive relaxation is needed to
the progressive relaxation response. Which sensors are applied to the forehead when using biofeedback to assess the physiological answer this question.
instructions should the nurse give to the client when response to relaxation technique. The progressive relaxation response requires the client to
using this relaxation method? contract and relax muscles to gain a deeper state of relaxation. Repeating a phrase or word
silently helps the client turn off other thoughts and focus on a neutral, monotonous stimulus.

1.‐ Sit in an upright position with legs crossed.


2.‐ Place sensors on the forehead to monitor physiological activity.
3.‐ Contract and then relax all your muscles sequentially from head to feet.
4.‐ Repeat a word or phrase forcefully with your breathing.

3674 The nurse teaches the client about massage therapy. Correct answer: 4 Massage stimulates circulation, thereby improving blood flow and preventing the formation An understanding of the pathophysiological effects of massage on the body is needed to
Which statement by the client demonstrates a correct of blood clots as well as decreasing muscle tension. Massage also stimulates the lymphatic answer this question.
understanding of the benefits of massage? system, enhancing lymphatic drainage. Massage causes the release of lactic acid that has
accumulated during exercise.
1.‐ "Massage increases blood clot formation."
2.‐ "Massage inhibits lymphatic drainage."
3.‐ "Massage causes an accumulation of lactic acid as a result of the exercise."
4.‐ "Massage stimulates circulation while causing relaxation."

3675 You have been invited to talk to the Woman's Guild in Correct answer: 3 Essential oils are distilled from flowers, roots, bark, leaves, wood, resins, citrus rinds, and The core issue of the question is that essential oils can cause an allergic reaction in the
your community about the cautions of aromatherapy. more; the quality of essential oils varies. Many oils should be kept out of the sunlight and heat. client.
Which of the following would be correctly presented Essential oils can be toxic and produce an allergic reaction in some individuals. Some essential
to the audience? oils are unsafe during pregnancy, so it is critical to contact the primary care provider prior to
trying aromatherapy to determine which oils are safe and which are toxic.

1.‐ Aromatic oils are produced by a standard‐quality formula.


2.‐ The oils can be stored in any type of container.
3.‐ The skin should always be tested for allergies by applying a small amount of oil to the area before treatment.
4.‐ Oils should not be used during pregnancy.

3676 Your class project is to prepare a speech on the Correct answer: 2 Humor decreases the social distance between people, putting them at ease. Humor reduces An understanding of the benefits of humor therapy is needed to answer the question
functions of humor in nursing situations for your peers. the presenter's anxiety and gains the audience's attention, which facilitates learning. Humor correctly.
Following the presentation, you know the participants helps individuals act out impulses in a safe, nonthreatening environment, thus releasing anger
understood your message when they respond that and aggression. Humor diminishes anxiety and fear, reducing tension and enabling the client to
humor: confront and deal with the situation.

1.‐ increases the social distance between people and assists in putting them at ease.
2.‐ facilitates learning.
3.‐ holds in anger and aggression.
4.‐ aggravates coping mechanisms.

3677 In evaluating the effectiveness of guided imagery for Correct answer: 4 The first statement indicates that the imagery session increases the client's anxiety level. The Understanding that the purpose of imagery is to help the client relax will help you choose
a client with preoperative anxiety, which data indicates second statement indicates that the client did not focus well during the session, which the correct answer.
that this therapy is successful? prevents total relaxation. The third statement indicates the therapy was unsuccessful because
guided imagery needs to be practiced daily to obtain the desired effect. The fourth statement
indicates that the client's anxiety is decreased, which is the overall outcome of guided imagery
therapy.
1.‐ "I hope that I don't have dreams like that tonight."
2.‐ "I couldn’t concentrate very much with all I have to do today."
3.‐ "I'll practice what I learned next week."
4.‐ "I feel less anxious about the upcoming surgery."
3678 The nurse is using progressive relaxation in a client Correct answer: 3 It is not necessary to assess muscle strength prior to the relaxation therapy. It is not The core issue of the question is the knowledge that a client can become so relaxed during
who is under a lot of stress. What nursing actions must necessary to assess for the use of sedatives because this therapy will cause relaxation. It is the progressive relaxation session that falling is a risk if his or her body is not supported.
be instituted prior to the start of the session to protect essential to have the body supported as the muscles begin to relax. The client could obtain
the client's safety? injury if he or she falls as muscle tension dissolves. It is not pertinent to know the client's
allergies before performing progressive relaxation.
1.‐ Assess the client's muscle strength.
2.‐ Check to make sure the client is not on any sedatives.
3.‐ Remember to position the body so it is totally supported to prevent falls.
4.‐ Assess for any allergies.

3679 The nurse taught the client's wife how to perform a Correct answer: 4 The skin being massaged needs to be exposed. Massaging the back requires long strokes Knowing the steps in giving a back massage will help you answer this question.
back massage on the client. Which observation by the along the spine with small, circular strokes peripherally. Warmed lotion helps enhance
nurse indicates that the spouse understands how to relaxation. Massages should last 3–5 minutes and be given in an unhurried manner.
give a back massage?
1.‐ Client is in his pajamas lying in bed.
2.‐ The spouse is rubbing his back with large, circular motions.
3.‐ The spouse is using cold lotion when performing the massage.
4.‐ The spouse massages the client's back for 3–5 minutes.

3680 The nurse taught the client about meditation. Which Correct answer: 4 Meditation involves focusing on the present moment and not on the future. Although In order to answer this question correctly, you need to know the definition of meditation,
statement by the client demonstrates a correct meditation was viewed as a religious practice, religious conviction is not required for the types of meditation, and how it is utilized.
understanding? meditation. Many types of meditation exist, and the techniques differ. All types of meditation
involve relaxation and focused attention.
1.‐ "Meditation is a technique used to quiet the mind and focus on the future."
2.‐ "Meditation involves a religious conviction."
3.‐ "There is only one way to meditate."
4.‐ "Meditation involves both relaxation and focus of attention."

3681 The nurse is performing an imagery session with a Correct answer: 2 You would give the instructions in option 1 to a client who wants to meditate. Imagery To answer this question, the nurse must know the difference between imagery and other
group of clients. What instruction should the nurse involves visualization to assist in healing. An image often used in imagery is the healing of an ill therapies like meditation, progressive relaxation, and music therapy.
give the clients? area of the body. Other images that can be used include destroying certain foreign substances
(e.g., cancer cells) or connecting with a higher level of consciousness. The instructions in
option 3 are appropriate for progressive relaxation exercises. Listening to music is not
essential for the use of imagery.
1.‐ "Try to concentrate on your breathing, letting go of all your stress."
2.‐ "Imagine that your body is healing itself, and it is using all its energy to attain this goal."
3.‐ "Contract the muscles of your arm and then relax."
4.‐ "Listen to some music of your choice and let the music take you away."

3682 A client is using aromatherapy to treat stress. Which Correct answer: 1 Dermatitis or eczema is a common allergic reaction to topical aromatherapy. Aromatherapy The question requires that the nurse know what to look for when a client has an allergic
assessment data indicates an allergic reaction in a will not affect skin turgor. Aromatherapy will not change the pigmentation of the skin involved. reaction to essential oils.
client who has received an aromatherapy session? Edema is not a side effect of aromatherapy.

1.‐ Presence of a new rash


2.‐ Increased skin turgor
3.‐ Decreased pigmenation
4.‐ Edema peripherally
3683 The preoperative waiting area has soft instrumental Correct answer: 2 Music therapy causes a hypometabolic state that stimulates the parasympathetic system and Knowledge of the physiological effects of music is needed to answer this question
music playing in the background. The client asks the causes a relaxation response. Music therapy also helps reduce the stress and anxiety of the correctly.
nurse, "Why are they playing this type of music instead preoperative client. Music therapy stimulates the right hemisphere of the brain where
of tuning into a radio station?" The nurse explains that creativity resides. A radio station or soft intrumental music can be used to cover up the normal
the purpose of using music therapy in the preoperative noises found in the hospital setting.
client is that it:

1.‐ produces a hypermetabolic state.


2.‐ helps reduce physiological stress, pain, anxiety, and isolation.
3.‐ enhances the functions of the left hemisphere of the brain.
4.‐ covers up the normal noises found in the hospital setting.

3684 The client tells the nurse that when he walks, he Correct answer: 3 All of the symptoms described suggest intermittent claudication. Ginkgo is the only herb The core issue of the question is the effect of gingko in reducing blood coagulation. Use
develops a pain in his right leg. He describes the pain effective for this condition because its anticoagulant properties will enhance blood flow to the this knowledge and the process of elimination to determine the correct option.
as cramping or burning in his muscles that subsides extremities. All the other options are herbs that will not be effective with this painful condition
with rest. Based on these symptoms, the nurse of the calf caused by reduced peripheral circulation.
supports the use of which of the following herbs?

1.‐ Feverfew
2.‐ Garlic
3.‐ Ginkgo
4.‐ Ginseng

3685 A client is taking chlorpromazine (Thorazine). Based Correct answer: 3 Chlorpromazine (Thorazine), a phenothiazine, is metabolized in the liver. Milk thistle, the liver The core issue of this question is recognition that milk thistle has a beneficial effect on the
on metabolism of this prescribed medication, the herb, is known to reduce risk of hepatotoxicity caused by phenothiazines. The other options do liver. Use this knowledge and the process of elimination to determine the correct option.
nurse supports the concomitant use of which of the not have this beneficial effect.
following herbs?
1.‐ Valerian root
2.‐ Ginger
3.‐ Milk thistle
4.‐ Hawthorn

3686 Which of the following therapeutic changes in Correct answer: 3 Garlic is used most widely to reduce total serum cholesterol and triglyceride levels. Option 1 The core issue of the question is the use of garlic as an aid in reducing cholesterol and
laboratory values would the nurse anticipate in the is incorrect because garlic would reduce, not increase, platelet aggregation, thus leading to triglycerides levels. Use this knowledge and the process of elimination to determine the
client taking garlic? bleeding tendencies. Although garlic has been shown to boost immunity, it has not correct option.
demonstrated effects on the white blood cell count, making option 2 incorrect. Option 4 is
incorrect because there is no known relationship between garlic and serum glucose levels.

1.‐ Increased platelet aggregation


2.‐ Increased white blood cell count
3.‐ Decreased serum cholesterol levels
4.‐ Decreased serum glucose levels

3687 The female client tells the nurse that she is planning a Correct answer: 4 The use of any herb should be discussed with the health care provider, particularly in The core issue of the question is that many herbs have unknown or adverse effects on the
pregnancy soon. In providing client education related pregnancy and lactation. Most herbs are contraindicated at this time, regardless of the form developing fetus and are therefore used cautiously or avoided during pregnancy. Use this
to the use of herbs during pregnancy, which of the and even when taken as directed or in lower doses. (Although some sources recommend knowledge and the process of elimination to determine the correct option.
following statements by the nurse is most ginger for morning sickness, other sources claim safety during pregnancy is unknown. Black
appropriate? cohosh has been known to promote labor and should be avoided until birth is imminent.)

1.‐ "Most herbs are safe when taken as directed."


2.‐ "Only herbs in the topical form are safe."
3.‐ "Certain herbs are safe and effective when taken in lower doses."
4.‐ "You should discuss the use of any herbs with your health care provider."
3688 The client presents to the health care clinic with an Correct answer: 1 Echinacea is effective when used topically to promote wound healing. It is also used internally The core issue of the question is the use of echinacea in wound healing and immune
abrasion to the left knee from a fall. After cleaning the to boost the immune system, particularly in the prevention and adjunct treatment of colds and system enhancement. Note the correlation between these properties and the skin injury of
abrasion, the nurse might support the use of which influenza. The other options do not have this beneficial effect. the client. Use this knowledge and the process of elimination to determine the correct
client‐chosen herb as adjunct therapy to treat the option.
abrasion?
1.‐ Echinacea
2.‐ Ginger
3.‐ Valerian root
4.‐ Feverfew

3689 The client scheduled for arthroscopic knee surgery Correct answer: 1 Ginger is known to inhibit thromboxane production. The inhibition of this prostaglandin The core issue of the question is the risk of bleeding associated with the use of ginger.
has been taking ginger for relief of arthritic pain reduces platelet aggregation, increasing the risk of bleeding in the postoperative client. Option Note the association of that property of this herb and the word postoperative in the stem
preoperatively at home. The client asks the nurse 2 is incorrect because ginger would be effective in this situation but is unsafe. Option 3 is of the question. Use this knowledge and the process of elimination to determine the
about the use of ginger after surgery for continued incorrect; although it is a true statement, it addresses dosing but not safety when used correct option.
relief of pain and also as a relief of postoperative postoperatively. There is no data to support ginger as potentiating the effects of opioid
nausea. In providing client education, the nurse medications (option 4).
explains that ginger:
1.‐ cannot be used safely postoperatively.
2.‐ would not be effective in this situation.
3.‐ may be repeated every 4 hours as needed.
4.‐ may potentiate the effects of opioid medications.

3690 In planning care for the client who is taking hawthorn, Correct answer: 2 The nurse should monitor blood pressure in the client taking hawthorn, which is known to The core issue of the question is the effect of hawthorn on hemodynamics, such as
the nurse includes which of the following decrease peripheral vascular resistance, thus decreasing blood pressure. There is no evidence lowering blood pressure and decreasing vascular resistance. Use this knowledge and the
interventions? that hawthorn has an affect on any of the other options. process of elimination to determine the correct option.
1.‐ Monitor blood glucose levels
2.‐ Monitor blood pressure
3.‐ Monitor white blood cell count
4.‐ Monitor temperature

3691 The client tells the nurse that a neighbor Correct answer: 2 One of the principle active ingredients of bilberry is pectin, a soluble fiber that decreases The core issue of this question is recognition that bilberry contains pectin and that pectin
recommends the use of bilberry in treating simple diarrhea. The other options do not contain soluble pectin, and therefore they would be of no is used to control diarrhea. Use this knowledge and the process of elimination to determine
diarrhea. The nurse supports the client's use of bilberry use in controlling simple diarrhea. the correct option.
based on which of the following?
1.‐ Anthocynanosides in the berry decrease peristalsis.
2.‐ The berry contains pectin, which acts as a soluble fiber.
3.‐ Bilberry acts to counteract antimicrobial suppression of normal intestinal flora.
4.‐ Action of the berry works to decrease bacterial or viral causes of diarrhea.

3692 The nurse instructs the client taking Saint John's wort Correct answer: 4 The psychotherapeutic effects of Saint John's wort are not well understood. The herb is The core issue of the question is the risk of interactive effects of foods containing tyramine
that which of the following foods preferred by the thought to work by inhibition of serotonin reuptake, but it may have a slight inhibition of with the MAO inhibitor effect of Saint John's wort. Use this knowledge and the process of
client may be safely consumed while taking this herb? monoamine oxidase (MAO). It is therefore important for the nurse to instruct the client that it elimination to determine the correct option.
is safe to eat ice cream while taking Saint John's wort. The other options are incorrect because
they contain tyramine, which, when consumed with MAO inhibition, may lead to severe
hypertension.
1.‐ Chocolate
2.‐ Aged cheeses
3.‐ Beer
4.‐ Vanilla ice cream
3693 Which of the following clients would benefit from the Correct answer: 3, 5 The client with coronary artery disease or peripheral vascular disease would benefit most The core issue of the question is the use of garlic as an aid in reducing cholesterol and
therapeutic effects of garlic? Select all that apply. from use of garlic because of its ability to lower cholesterol and triglyceride levels. Garlic is not triglycerides as serum lipids. Use this knowledge and the process of elimination to
helpful in treating low blood pressure or liver disease (options 1 and 2). Because it inhibits determine the correct option.
platelet aggregation, the use of garlic by clients with a bleeding disorder could be hazardous
(option 4).
1.‐ The client with decreased blood pressure
2.‐ The client with liver disease
3.‐ The client with coronary artery disease
4.‐ The client with a bleeding disorder
5.‐ The client with peripheral vascular disease

3694 The client returns from a vacation after a 5‐hour car Correct answer: 1 Ginger is most effective when started several days before traveling. A dose should be Select the response that reflects the optimal method of treating motion sickness which is
ride stating that the ginger taken to treat motion administered 30 minutes before the trip and may be repeated every 4 hours as needed. to medicate before the travel begins, rather than focusing on the herb.
sickness was ineffective. The ginger was taken in Options 2 and 3 are incorrect as motion sickness from any type of travel, including seasickness,
capsule form after riding inside the car for an hour. may be treated effectively with ginger. Option 4 is incorrect as the form of ginger is not
The nurse explains that the ginger was most likely relevant to its effectiveness.
ineffective for which of the following reasons?

1.‐ The herb is most effective to prevent motion sickness when started several days before traveling.
2.‐ Ginger is not recommended to prevent car sickness.
3.‐ Ginger is only effective to prevent motion sickness from air travel.
4.‐ The candied ginger form of the herb is recommended in this situation.

3695 The client taking black cohosh should be cautioned by Correct answer: 4 Black cohosh should be avoided with concomitant use of antihypertensive medications as an Eliminate medications that would likely not have a life threatening interaction, such as
the nurse to avoid which of the following medications? adverse effect of the herb is hypotension. The other options are incorrect although the client Tylenol and Prozac.
should discuss the use of any herbal medications with the health care provider.

1.‐ Warfarin (Coumadin)


2.‐ Acetaminophen (Tylenol)
3.‐ Fluoxetine (Prozac)
4.‐ Metoprolol (Lopressor)

3696 The client reports use of echinacea for the past 2 Correct answer: 1 The client should be instructed about long‐term use of the herb. Echinacea, as with any other Note that the scenario stresses the client has been taking the herb for 2 years. Select the
years to ward off colds and influenza. Based on herb, is not meant for long‐term use. Although most commonly used to boost the immune response that focuses on the length of use.
knowledge related to safe and effective use of system and ward off colds and influenza, echinacea may lead to hepatotoxicity when taken for
echinacea, the nurse plans to instruct the client about more than 8 weeks; prolonged use may cause suppression of the immune system. Therefore,
which of the following? option 4 is incorrect as the client does not demonstrate accurate knowledge related to the use
of echinacea. Although correct dosage is important, it is not relevant in this client since the
problem lies in the length of therapy, making option 2 incorrect. Option 3 is incorrect as there
is no evidence that use of antihypertensive medications are contraindicated while using the
herb, although all medications should be discussed with the health care provider.

1.‐ The effects of long‐term use of the herb


2.‐ Echinacea should be taken in the morning in tea form.
3.‐ The client should avoid concomitant use of acetaminophen (Tylenol).
4.‐ The client is already demonstrating accurate knowledge about use of the herb.

3697 The client taking ginkgo should be monitored by the Correct answer: 4 Ginkgo inhibits platelet aggregation and adhesion thus the client should be monitored Select the response that corresponds to the most frequent issue with this herb which is
nurse for which of the following adverse effects? bruising and bleeding. The other options are incorrect. decreased platelet count.

1.‐ Decreased blood glucose levels


2.‐ Infection
3.‐ Hepatotoxicity
4.‐ Bruising and bleeding

3698 The nurse includes which of the following Correct answer: 3 The nurse should monitor the client for headache and mild, temporary stomach upset, Note that only option 3 gives a combination of two assessments which is often a clue to
assessments in planning care for the client taking common adverse effects of valerian root. With large doses, the client may experience severe the correct response.
valerian root? headache, nausea, morning headache and blurred vision. Option 1 is incorrect as the herb is
not known to have any effect on the immune system. Option 2 is incorrect as valerian root is
known to have similar action to benzodiazepines, without addiction. Option 4 is incorrect as
the use of the herb has no know effect on blood pressures.

1.‐ Immune system suppression


2.‐ Addiction
3.‐ Headache and mild, temporary upset stomach
4.‐ Hypertension

3699 The client taking garlic demonstrates the need for Correct answer: 3 Garlic decreases cholesterol by decreasing, not increasing, low‐density lipoproteins. It also Increasing low‐density lipoproteins is not a desired effect, therefore would require
further teaching when he states that garlic: decreases triglycerides and increases high‐density lipoproteins. The other options are correct additional teaching.
and do not require further teaching: garlic should be chewed or crushed when taken in the raw
form (clove) to effectively convert to the active ingredient allicin (option 1); garlic, as with all
other phytomedicines, should be used cautiously with other phytomedicines (option 2); garlic
may reduce the risk of stroke by its actions in reducing cholesterol and inhibiting platelet
aggregation.
1.‐ Should be chewed if the fresh bulb is used.
2.‐ Should be used cautiously with other phytomedicines.
3.‐ Is known to increase low‐density lipoproteins.
4.‐ May reduce the risk of stroke.

3700 The nurse evaluates the effectiveness of therapy with Correct answer: 2 The most common use of bilberry is the relief of simple diarrhea. Other known uses are the In this question note that all of the options, except the correct response, have multiple
bilberry by monitoring for which of the following prevention and treatment of eye disorders, such as diabetic retinopathy, night blindness, conditions in the string.
outcomes? macular degeneration, glaucoma, and cataracts. It is also used in the treatment of diabetes
mellitus, as an antioxidant and the possible treatment of varicose veins and hemorrhoids. The
other options are incorrect.
1.‐ Decreased urinary frequency, nocturia and dysuria
2.‐ Relief of diarrhea
3.‐ Relief of night sweats and hot flashes
4.‐ Reduction of symptoms of cold/influenza

3701 The client with hepatitis is taking milk thistle. The Correct answer: 2 Milk thistle, commonly referred to as the liver herb, is used as adjunct therapy in the Note the word water in the correct response. This is a key to the correct option as other
nurse instructs the client to do which of the following treatment of liver inflammation related to cirrhosis, hepatitis and fatty infiltrates related to bases used for the herb, such as alcohol, would be contraindicated.
regarding its use? the use of alcohol and other toxins. The client should use water‐based extract and avoid the
alcohol‐based extract. Option 1 is incorrect as there is no reason to discontinue the herb at
this time. Option 3 is incorrect as milk thistle is not soluble in water and should not be used in
tea form. Although rare, loose stools and diarrhea are the common adverse effects of the
herb, not nausea and vomiting, making option 4 incorrect.

1.‐ Discontinue the milk thistle immediately.


2.‐ Use the water‐based extract of the herb.
3.‐ Administer the milk thistle in tea form.
4.‐ Report common adverse effects of nausea and vomiting.
3702 The nurse cautions the client taking black cohosh to Correct answer: 4 The client taking black cohosh should avoid concomitant use of antihypertensives, such as Select the response that would enhance the expected effect of the herb, in this case
avoid which of the following medications? captopril (Capoten), an ACE inhibitor. The other options are incorrect, although use of all hypotension.
medications should be discussed with the health care provider.
1.‐ Acetaminophen (Tylenol)
2.‐ Lorazepam (Ativan)
3.‐ Over‐the‐counter (OTC) cold and flu preparations
4.‐ Captopril (Capoten)

3703 Which of the following assessments would the nurse Correct answer: 3 American ginseng is not known to have any effect on liver function. The other options are Knowledge that ginseng is often used as a preventative treatment in cardiac health can be
be least likely to monitor in the client taking American assessments that the nurse would be expected to monitor: Korean (American) ginseng helpful in eliminating options 2 and 4.
ginseng? improves glycosylated hemoglobin (HbA1c) (option 1); it improves serum cholesterol and
triglyceride levels (option 2); and it increases blood pressure with low doses and decreases
blood pressure with higher doses (option 4).
1.‐ Glycosylated hemoglobin
2.‐ Serum cholesterol and triglyceride levels
3.‐ Liver function studies
4.‐ Blood pressure

3704 The client asks the nurse about a magazine Correct answer: 1 The focus of the education should be on the client's ability to assess the reliability of the Reliability is the key word and is helpful in selecting the correct response.
advertisement related to the use of ginger for the information source. Although all of the other options are also correct, option 1 is a global
treatment of arthritis. The nurse focuses client response and takes into consideration all of the other options.
education on which of the following?
1.‐ The client's ability to accurately assess the reliability of information sources
2.‐ The author of the article
3.‐ The type of magazine in which the advertisement was found
4.‐ The client's level of education

3705 A female client taking feverfew for the prevention of Correct answer: 1 Feverfew has not been proven safe for use in pregnancy and lactation. It is important for the Select the option which describes the safest option for the fetus, which is to discontinue
migraine headaches reports to the nurse that she client to discontinue use of the feverfew immediately until the pregnancy has been verified. the herb.
thinks she may be pregnant. Based on knowledge of Client education should also include that those who may possibly become pregnant should
feverfew, what would be the nurse's priority action? avoid the use of any herbs. Therefore, first the client should be instructed to discontinue use of
the feverfew and then have a pregnancy test performed. Option 3 is incorrect because the
client would need to se the health care provider only if she is pregnant. Option 4 is incorrect
because the dosage of the feverfew is not relevant.

1.‐ Instruct the client to discontinue use of the feverfew herb.


2.‐ Instruct the client to have a pregnancy test performed.
3.‐ Arrange for the client to see her health care provider.
4.‐ Instruct the client to reduce the dosage of the feverfew.

3706 A male client taking valerian root as a sleep aid Correct answer: 2 The active ingredient, valepotriate, might be carcinogenic and should be removed from the Omit option 1 as the herb and this drug work in the same manner. Omit option 3 as this
demonstrates safe administration of the herb when he form of the herb. Option 1 is incorrect as lorazepam (Ativan) and valerian root exhibit similar herbal remedy which acts like a benzodiazepine would not be recommended for use in
states that the herb: actions and may not be taken together safely. Option 3 is incorrect as the herb has not been children.
proven safe in children, regardless of age. Option 4 is incorrect as the form of the herb is
irrelevant to its effectiveness.
1.‐ May be taken safely with lorazepam (Ativan) in small doses.
2.‐ Should have the active ingredient valepotriate removed from the extract.
3.‐ May be used safely in children over the age of 5.
4.‐ Is not effective in the capsule form.
3707 A client taking saw palmetto to treat symptoms of Correct answer: 4 Saw palmetto taken in doses greater than the recommended 160 mg standardized liposterolic Select the option that would most likely be related to any adverse effect, which is dosage.
benign prostatic hyperplasia (BPH) complains of acid BID can produce diarrhea. This side effect is not related to the form of the extract or the
diarrhea. Based on knowledge of this herb, the nurse client's age (options 3 and 4). Allergy to ragweed is not related to the use of saw palmetto
concludes that the diarrhea is most likely related to (option 2).
which of the following?
1.‐ The form of the herb extract
2.‐ Client history of allergy
3.‐ The age of the client
4.‐ The dose of the herb

3708 A 50‐year‐old female client presents to the health Correct answer: 2 One of the major uses of black cohosh is in the treatment of postmenopausal symptoms, Use elimination to omit options that name herbs more commonly used for other
clinic with complaints of hot flashes and night sweats. which include hot flashes and night sweats. Black cohosh is also used to promote labor in conditions. For example, echinacea is used to treat colds; bilberry is used to prevent eye
After determining that the symptoms are not related pregnancy, to reduce blood pressure and cholesterol levels and is used in the treatment of disorders; and valerian root is used like benzodiazepines.
to any underlying disease process, the nurse supports poisonous snake bites. The other options identify incorrect herbs for treatment of the
client use of which of the following phytomedicines? symptoms listed.

1.‐ Echinacea
2.‐ Black cohosh
3.‐ Bilberry
4.‐ Valerian root

3709 The client is taking garlic on a daily basis. Which of Correct answer: 1 The effectiveness of garlic is based on scientific evidence and clinical trials. Garlic has the Omit options 3 and 4 as being either too general (4) or stated as an absolute (3).
the following statements demonstrates the client's ability to affect bleeding times and should not be taken with other herbs that have the same
understanding of its use? action, making option 2 incorrect. Option 3 is incorrect because garlic may be taken in enteric‐
coated tablets of garlic powder, which is effective in reducing bad breath although this form
may not be as potent as raw garlic. Option 4 is incorrect because garlic should not be used
with OTC medications such as ASA (Aspirin) or ibuprofen (Motrin) that may also affect bleeding
times.
1.‐ "The effectiveness of garlic is based on scientific research."
2.‐ "Garlic may be used safely with ginger."
3.‐ "There are no remedies for the bad breath caused by the garlic."
4.‐ "I can take garlic safely with over‐the‐counter medications."

3710 Which of the following statements made by the nurse Correct answer: 3 Hawthorn should be avoided with concomitant use of prescription antihypertensive Omit option 1 as herbal remedies would not generally be recommended for acute cardiac
is most appropriate when providing client education medications, such as captopril (Capoten), which is an ACE inhibitor. Hawthorn is similar in episodes. Select the option that best corresponds with the action of the drug which is the
regarding the use of hawthorn? action to the ACE inhibitors and works to reduce blood pressure by blocking the conversion of blocking of vasoconstriction.
Angiotensin I to Angiotensin II, which is a potent vasoconstrictor. Option 1 is incorrect because
hawthorn is only effective in the treatment of chronic stable angina. As with any phytotherapy,
use should be restricted to chronic, self‐limiting conditions and not for acute episodes. Option
2 is incorrect as hawthorn has a negative chronotropic effect to decrease heart rate. Option 4
is incorrect as verapamil (Calan) is a calcium channel blocker and works to decrease blood
pressure by blocking the influx of calcium ions across the cardiac and arterial muscle cell
membrane.

1.‐ "You may use hawthorn for acute episodes of chest pain or angina."
2.‐ "Hawthorn will not affect your heart rate."
3.‐ "You should not take hawthorn while taking captopril (Capoten)."
4.‐ "Hawthorn is known to produce the same effects as verapamil (Calan).”
3711 A client has been taking nutrition bars containing Correct answer: 3 Korean ginseng should not be used in combination with coffee, tea or colas. Option 1 is Focus on the material in the question that describes the reason for the use of the herb.
Korean ginseng for the past 2 weeks to increase incorrect as there is no evidence that the herb affects the ability to drive a car or operate Substances containing caffeine are identified in option 3, which would not combine well
concentration and stamina. Based on knowledge of the machinery (i.e., there are no known CNS effects). Option 2 is incorrect as the form of the herb with an herb being used to increase stamina.
herb, the nurse instructs the client to do which of the is not related the adverse effects. Option 4 is incorrect as there are specific precautions with
following? the use of Korean ginseng; for example, a ginseng‐free period is recommended, usually 2–3
weeks on and 1–2 weeks off.
1.‐ Avoid operating machinery or driving a car while taking the nutrition bar.
2.‐ Avoid this form of the herb, which is known to increase adverse effects.
3.‐ Avoid use of the nutrition bar with coffee, tea, or cola.
4.‐ Continue use of the nutrition bar daily as desired.

3712 A client taking Saint John's wort in capsule form to Correct answer: 3 Saint John's wort may take several weeks before the effects are evident. (All phytomedicines The question mentions a specific time frame. Select the answer that addresses the time
treat mild depression for the past week complains of it are slower to work than prescribed medications.) The form of the herb is not related to its frame.
ineffectiveness. Which of the following statements effectiveness, making option 1 incorrect. Saint John's wort in capsule form should be taken
made by the nurse is most appropriate? three times per day, making option 2 incorrect. Option 4 is incorrect as Saint John's wort is
appropriate for the treatment of mild to moderate depression.

1.‐ "You may need to switch to another form of the herb."


2.‐ "You should take the herb at night."
3.‐ "It may take several weeks for the therapy to be effective."
4.‐ "Saint John's wort may not be effective for your type of depression."

3713 In evaluating the effectiveness of saw palmetto for Correct answer: 2 There is conflicting evidence that saw palmetto will actually reduce the size of the prostate. Select the response that most directly relates to the use of the herb. The other options
the treatment of benign prostatic hyperplasia (BPH), Clinical evidence supports the use of the herb to reduce symptoms of residual volume (option could have multiple causes.
the nurse would be least likely to focus on which of the 1), urinary frequency (option 3) and dysuria (option 4), common symptoms of the disorder.
following assessments?
1.‐ Residual volume
2.‐ Size of the prostate
3.‐ Urinary frequency
4.‐ Dysuria

3714 Which of the following statements made by the client Correct answer: 1 Saw palmetto should be avoided in tea form as it is insoluble in water. The other options are Focus on the response that relates to self‐administration of the herb as this is frequently
who is taking saw palmetto indicates a need for true statements and therefore do not require further teaching. an area that requires reinforcement of teaching.
further teaching?
1.‐ "I will take saw palmetto in the morning as a tea."
2.‐ "I will take the herb with meals or food."
3.‐ "Large amounts of the herb may cause diarrhea."
4.‐ "It may take as long as 6–8 weeks before effects of the therapy may be noticed."

3715 Use of hawthorn to treat hypertension is similar in Correct answer: 2 Hawthorn is similar in action to ACE inhibitors. They prevent the conversion of angiotensin I Recall that hawthorn acts by preventing vasoconstriction. Eliminate the options that do
action to which of the following medications? to angiotensin II, a potent vasoconstrictor. The other answers are incorrect. Furosemide (Lasix) not act in this manner.
is a loop diuretic (option 1); hydralazine (Apresoline) is a direct‐acting peripheral vasodilator
(option 3); amlodipine (Norvasc) is a calcium channel blocker (option 4).

1.‐ Furosemide (Lasix)


2.‐ Capoten (Captopril)
3.‐ Hydralazine (Apresoline)
4.‐ Amlodipine (Norvasc)
3716 A client using phytomedicines is able to demonstrate Correct answer: 3 Many herbs may be used for different reasons. The dosage of some herbs depends on its Eliminate option 2 as an untrue statement. Options 1 and 4 are stated as absolutes, and
safe and effective use when the client makes which of specific use. Option 1 is incorrect as acute and sudden illness should not be treated with are not true.
the following statements? phytotherapy. Option 2 is incorrect as the FDA does not evaluate phytomedicines for the
treatment, cure or prevention of disease, but how they affect the structure and function of the
human body. Although not regulated in the United States by the FDA, most are regulated by
the Dietary Supplement and Health Education Act of 1994 as dietary supplements. Option 4 is
incorrect as many side effects and adverse reactions may occur with improper dosing and the
words natural or all‐natural can be misleading.

1.‐ "Phytomedicines may be used effectively for sudden and acute illness."
2.‐ "The FDA regulates the medicinal use of herbs only when taken as directed."
3.‐ "It is important to know the use of each particular herb in order to determine the appropriate dose."
4.‐ "Large doses of phytomedicines are safe since they are natural substances."

3717 A client taking ginkgo biloba complains of itchy skin. Correct answer: 1 Unprocessed ginkgo leaves should be avoided as they contain ginkgolic acids, which are Select the response which asks for more information from the client regarding the
Which of the following statements made by the nurse potent allergens related to the substance uroshiol, which is the chemical responsible for the therapy.
is most appropriate? itch in poison ivy. Option 2 is incorrect because the symptoms will continue with further use of
the unprocessed leaves. Option 3 is incorrect as photosensitivity is not related to the use of
ginkgo in any form. Option 4 is incorrect as relief of symptoms can be obtained from
discontinuing use but will only reoccur if use of the unprocessed leaves is resumed.

1.‐ "Tell me what form of ginkgo you have been using."


2.‐ "This reaction is common and should disappear in a few days."
3.‐ "It is important to avoid the sun while taking gingko."
4.‐ "Discontinue use of the ginkgo until the symptoms disappear."

3718 The client taking Saint John's wort should be Correct answer: 2 One of the side effects of Saint John's wort is photosensitivity requiring the client to avoid Note that sun exposure is often problematic for clients when medications are in use.
instructed to avoid which of the following activities? direct exposure to sunlight, especially if fair‐skinned. The other options are incorrect and not
relevant.
1.‐ Bowling
2.‐ Sunbathing
3.‐ Yoga
4.‐ Weightlifting

3719 The client asks the nurse about the purpose of taking Correct answer: 4 Valerian root is used as a sedative, to reduce anxiety, treat insomnia, and relieve muscle Note that options 1, 2, and 4 are all common uses of benzodiazepines. Valerian root and
valerian root. Which of the following uses would be spasms. It is also used as adjunct therapy for the withdrawal of benzodiazepines, as it has benzodiazepines are similar in their actions.
least appropriate? similar action to this class of medications without the addiction or dependence. Valerian root
has not been known to affect blood pressure, making option 4 incorrect.

1.‐ Relief of insomnia


2.‐ Relief of muscle spasms
3.‐ Reduction of anxiety
4.‐ Treatment of hypertension

3720 The nurse supports use of bilberry for which of the Correct answer: 4 There are no known contraindications for the use of bilberry in conjunction with estrogen, Omit options 1, 2, and 3 since these drugs are known to have interactions with this herb.
following clients? although clients should always inform the health care provider when planning to utilize
phytomedications. Bilberry may interfere with iron absorption when taken internally, making
option 1 incorrect. Option 2 is incorrect as bilberry may increase coagulation time and should
be avoided with the use of anticoagulants such as warfarin (Coumadin). Since vitamin E can
antagonize vitamin K, it plays a peripheral role in blood clotting and should be avoided with
concomitant use of bilberry, making option 3 incorrect.

1.‐ The client taking iron pills for the treatment of anemia
2.‐ The client taking warfarin (Coumadin) for the treatment of thrombophlebitis
3.‐ The client taking vitamin E to improve the immune system
4.‐ The client taking estrogen for the treatment of premenstrual syndrome

3721 The nurse instructs the client taking valerian root to Correct answer: 4 Alprazolam (Xanax) is a short‐ to intermediate‐acting benzodiazepine. Concomitant use of this Valerian root is known for acting similarly to benzodiazepines. Xanax is the only
avoid the use of which of the following medications? drug with valerian root should be avoided since their actions are similar and the herb may benzodiazepine option.
potentiate the action of the alprazolam. The other options are incorrect.

1.‐ ASA (aspirin)


2.‐ Codeine
3.‐ Clonidine (Catapres)
4.‐ Alprazolam (Xanax)

3722 The nurse suggests that a client diagnosed with Correct answer: 4 Increased intraocular pressure is symptomatic of glaucoma. Bilberry is useful in the treatment Omit the herbs known for other common uses. Select Bilberry as the herb often used to
increased intraocular pressure might benefit from of this and other eye disorders, such as diabetic retinopathy, night blindness, macular prevent eye disorders.
which of the following phytomedicines? degeneration, and cataracts. The other options are incorrect.
1.‐ Black cohosh
2.‐ Valerian root
3.‐ Saw palmetto
4.‐ Bilberry

3723 The nurse would recommend ginger to treat which of Correct answer: 1 The most common use of ginger is the relief of nausea and vomiting. It is also useful in the Omit option 2 as being a condition that is generally not self‐treated. Select the option
the following complaints by the client? treatment of motion sickness and as an anti‐inflammatory in osteoarthritis and rheumatoid most closely related with the use of this herb.
arthritis. The other options are incorrect.
1.‐ Nausea and vomiting
2.‐ Varicose veins
3.‐ Cold and flu symptoms
4.‐ Fatigue

3724 When assessing the appropriateness of humor Correct answer: 1 It is most important to determine what the client sees as humorous. Humor is individualistic; To answer this question correctly, the nurse needs to know when it is appropriate to use
therapy for a client, the nurse must first: what is funny for one client might not be funny to another client. Laughing when you might not humor therapy.
think something is humorous does not help you be genuine with the client. When using humor
therapy, jokes related to the client's religion, culture, or gender should be avoided. These jokes
can make the client feel inferior, which will not enhance the client's healing.

1.‐ Observe the type of humor that the client creates.


2.‐ See the client laugh in response to a cartoon most clients find funny.
3.‐ Laugh at the same things the client does.
4.‐ Use jokes based on the client's culture as an icebreaker.

3725 The nurse has taught the client with nausea how to Correct answer: 1 If essential oil gets into the eyes, it must be flushed out immediately with milk or a carrier oil, To answer this question correctly, the nurse needs to know how to administer and store
use aromatherapy to alleviate the symptoms. Which rather than water, since the oils are not dissolvable in water. There are some contraindications oil, and what to do when it accidentally gets in one's eyes.
statement by the client indicates that he has for some types of essential oils, so the nurse needs to consult the list of contraindications in
demonstrated correct understanding of the training manual. The choice to use aromatherapy is best decided by a professional in the
aromatherapy? field. Nurses should not administer essential oils orally, since this is not covered by the nurse's
scope of practice. Also, some essential oils can be poisonous if given incorrectly, and should be
administered orally by a trained professional in this field. Oils need to be stored in a dark,
covered bottle and kept away from light and heat.

1.‐ "If I get the essential oil in my eyes, I need to flush them out with either milk or a carrier oil."
2.‐ "I can share my treatment with anyone else, since I now know how to use aromatherapy."
3.‐ "I can take it orally, if needed."
4.‐ "I need to store my oil in any container as long as it is away from the light."

3726 A client is using music therapy to treat pain. To Correct answer: 4 It is unrealistic to expect the client to practice this technique every hour, and it takes 20 The nurse needs to know how long and often the client should practice music therapy to
achieve the therapeutic outcome of being pain‐free, minutes to have an effective music therapy session. Music therapy needs to be practiced more answer this question correctly.
the client should practice music therapy: often than once a day. Every other day does not provide frequent‐enough practice to elicit the
relaxation response. A minimum of 20‐minute sessions, twice a day is needed to obtain the
therapeutic response of music therapy.
1.‐ Five minutes every hour.
2.‐ Once a day for 45 minutes.
3.‐ Every other day for 10 minutes.
4.‐ Twenty minutes, twice a day.

3727 A tense and anxious client is a good candidate for the Correct answer: 1, 2, 3 The setting for the imagery needs to be quiet, and the client needs to be in a comfortable The correct answers reflect what the nurse can do to facilitate the imagery session. For
use of imagery. After discussing this therapy with her, position for the relaxation and imagery session to take place. Relaxation exercises are used in example, a quiet setting, using relaxation exercises, and allowing the client to create her
the nurse would do which of the following to support the beginning of the imagery session to assist the client to deep relaxation. Allowing the client own images are essential for a successful therapy.
the client's use of imagery? Select all that apply. to create her own images allows the client's subconscious to select images with which the
client is ready to deal, and to block those with which the client is unwilling or unready to deal
at this time. The nurse does not need to know the specifics of the client’s images, or interpret
the meaning, but should encourage the client to explore what these images mean to her.

1.‐ Select a quiet, comfortable setting for the session.


2.‐ Use relaxation exercises to assist the client to relax prior to the imagery session.
3.‐ Allow the client to create her own images.
4.‐ Obtain specific information about the client's images during a session.
5.‐ Help the client interpret the meaning of her symbols and images.

3728 While performing a massage on a client, the nurse Correct answer: 4 The nurse cannot assess the gastrointestinal system during the massage, since that would To answer this question correctly, the nurse needs to know what assessments can be done
can concurrently assess which body system? involve listening to bowel sounds and percussing the abdomen. The nurse cannot assess the while giving a massage.
musculoskeletal system during the massage, since it would involve the palpation, range of joint
motion, and strength of all the joints. During a massage, the muscles are compressed to
enhance relaxation, and the joints are not mobilized or tested. The nurse cannot assess the
peripheral vascular system during the massage, because this would involve assessing pulses of
the extremity. The nurse can assess the skin color, temperature, and hair distribution, and
observe for any lesions, while performing the massage.

1.‐ Gastrointestinal
2.‐ Musculoskeletal
3.‐ Peripheral vascular
4.‐ Integumentary

3729 The nurse is using progressive relaxation with a client Correct answer: 2 Blood pressure will decrease in the relaxation state with because the body is in a state of Knowing that progressive relaxation will stimulate the parasympathetic system and how
who has chronic back pain. The nurse evaluates the lowered metabolic need. The respiratory rate will decrease, which is one of the first signs that this will affect the vital signs is necessary to answer this question correctly.
therapy as being effective when observing a(n): the client is entering the relaxation state. The heart rate also will decrease when the client is in
the relaxed state. Oxygen saturation will increase in a relaxed state, due to decreased oxygen
consumption by the tissues.
1.‐ Increase in blood pressure.
2.‐ Decreased respiratory rate.
3.‐ Increase in heart rate.
4.‐ Decrease in oxygen saturation.
3730 The nurse teaches the client that there are three Correct answer: 1 The three requisites to relaxation are correct posture, a quiet environment, and a mind at The nurse needs to know what is required in perform relaxation exercises in order to
requisites for using relaxation. The client would be rest. A lying position, music, or an hour of time is not necessary for meditation. Moving the answer this question correctly.
using these requisites when the client is: body through postures while focusing on a thought is yoga. Assessing energy changes when the
hand is passed over a portion of the body is an energetic therapy, such as healing touch or
therapeutic touch.
1.‐ Sitting in a chair with good posture, with the room quiet and the mind at rest.
2.‐ Lying on a couch, listening to soft music for 1 hour.
3.‐ Concentrating on a thought and moving the body in various postures.
4.‐ Assessing the energy with the palms of the hand over an area of the body.

3731 The primary purpose of humor therapy for a client in Correct answer: 1 Humor therapy stimulates the release of endorphins, which will reduce the perception of An understanding of the pathophysiological effects of humor and laughter is needed to
pain is to: painful impulses. Laughter does cause an increase in sympathetic response initially, which is answer this question.
then reversed in the relaxation stage. Both of these responses affect the perception of the
pain. The client's level of conscious will increase with the sympathetic response, but this is not
the primary purpose of using humor therapy. Increasing the blood supply to the brain with
sympathetic stimulation does not decrease the perception of the pain.

1.‐ Stimulate the production of endorphins.


2.‐ Elicit the response of the sympathetic nervous system.
3.‐ Increase the client's level of consciousness.
4.‐ Improve blood supply to the brain.

3732 Which of the following findings would the nurse Correct answer: 1 During music therapy, the parasympathetic system will be stimulated, causing vasodilation to An understanding of the pathophysiological effects of music therapy on the body is
observe in a client who is successfully using music the skin, thereby increasing skin temperature. An increased respiratory rate would indicate a essential to answer this question correctly.
therapy? sympathetic response. An increased heart rate also would indicate a sympathetic response. An
increased blood pressure also would indicate a sympathetic response.

1.‐ Increased peripheral temperature


2.‐ Increased respiratory rate
3.‐ Increased heart rate
4.‐ Increased blood pressure

3733 The nurse discusses ways for the preoperative client Correct answer: 4 Allowing the client to create his own images allows the subconscious to select images with Knowing how to perform an imagery session and the effects of imagery on the body is
to enhance healing, and the client decides to learn which the client is ready to deal, and block those with which the client is unwilling or unready necessary to answer this question.
imagery. When conducting an imagery session, the to deal at this time. Imagery will cause a decrease in respiratory rate, due to parasympathetic
nurse needs to keep in mind that: stimulation. The nurse does not need to know the specifics of the client’s images, or interpret
the meaning, but should encourage the client to explore what these images mean to him.
There is no way to predict what will surface in the client's imagination, as every experience is
different.
1.‐ It can be dangerous to allow a client to create his images.
2.‐ Imagery generally increases the client's respiratory rate.
3.‐ It is necessary to obtain specifics about the client's images during a session.
4.‐ No one can predict what images will occur in the client's imagination.

3734 Nurses assist in performing relaxation, biofeedback, Correct answer: 1 All three of these therapies involve deep breathing while learning how to relax the body. An understanding of relaxation, imagery, and biofeedback is needed to determine what
and imagery techniques with a client. These techniques Meditation also involves deep breathing and relaxation, but therapeutic touch involves elements are in common.
share which of the following processes? assessing and correcting energy field imbalances in a manner that is different from relaxation,
biofeedback, and imagery techniques. The body is not engaged in physical activity or touched
with biofeedback, imagery, or relaxation therapies. Physical resting is common to these
therapies, but reflexology involves applying pressure to the foot in certain areas to obtain a
desired response.
1.‐ Physical resting and rhythmic breathing
2.‐ Therapeutic touch and meditation
3.‐ Physical activity and therapeutic touch
4.‐ Physical resting and reflexology

3735 The client is using meditation to relieve stress. Which Correct answer: 1 The client needs a distraction‐free environment, to facilitate relaxation. The client should be Knowledge of the steps to perform meditation is needed to determine if the client has
statement by the client indicates an accurate sitting cross‐legged on the floor or be upright in a straight‐backed chair with the spine upright demonstrated correct knowledge on the topic.
understanding of meditation? for meditation. Lying down will increase the chance that the client will fall asleep. Meditations
need to be 10–20 minutes in length to obtain the desired effect. The client should be
instructed to focus on her breathing or a mental image. When distracting thoughts enter the
mind, she should let them drift back out by not focusing on them.

1.‐ "I try to meditate when I wake up, since everyone at home is still asleep."
2.‐ "I lie down in bed when I meditate."
3.‐ "I perform the meditation for at least five minutes at a time."
4.‐ "I find that it gives me time to think about my life and what I want in life."

3736 The nurse is instructing the client how to use the Correct answer: 1, 2 Essential oils can be inhaled by placing them in a humidifier or vaporizer. Adding essential oils Knowledge of how essential oils are administered is necessary to answer this question.
essential oils in aromatherapy. Which statements by to the bath will cause inhalation as the oil rises with the steam of the water. Essential oils
the client indicate an understanding of how essential should not be placed directly on the skin without the use of carrier oil (e.g., almond oil).
oils can be administered? Select all that apply. Placing the essential oil directly on the skin can cause irritation. Nurses should not administer
essential oils orally, since this is not covered by the nurse's scope of practice. Also, some
essential oils can be poisonous if given incorrectly, and should be administered orally by a
trained professional in this field.
1.‐ "I can inhale the oils by putting a few drops in my humidifier."
2.‐ "I can add a few drops in my bath to help me relax."
3.‐ "I can rub the essential oils over achy areas of my body."
4.‐ "I can add the oils to a cup of hot water, and drink it with lemon."
5.‐ "I can heat the drops of oil, and sprinkle them on my skin."

3737 A client is using imagery to treat cancer. An outcome Correct answer: 1, 2, 4 The outcome for imagery is to give the client a sense of peace with a sense of purpose and An understanding that the outcome of imagery is relaxation, peace, and relief of pain, and
for this healing intervention would include which of meaning of one's life. Remission or cure is not the outcome of imagery. Imagery is a useful not necessarily the cure of the illness, is needed to answer this question.
the following? Select all that apply. technique to decrease the client’s perception of pain.

1.‐ A sense of peace and integration


2.‐ A deeper understanding of the meaning of life's events
3.‐ A total remission of the disease process
4.‐ Relief of pain
5.‐ Lengthened survival rate

3738 When using humor therapy with a client, the nurse Correct answer: 3 The nurse needs to establish a professional relationship with the client initially; so humorous Knowledge of how to implement humor therapy is needed to answer this question
should do which of the following? interventions are inappropriate at this time. Humor therapy should come naturally, and not be correctly.
forced, as would be the case if the nurse were using many jokes and props trying to make the
client laugh. The nurse needs to meet the physical and emotional needs of the client before
initiating humor therapy. If humor is used when the client's needs are not met, the nurse will
be insensitive. What one person thinks is humorous might not be something the other person
feels is humorous.
1.‐ Use a humorous intervention within the first 20 minutes of meeting a client to establish the connection of shared laughter.
2.‐ Keep trying a different joke or prop until the nurse gets a laugh.
3.‐ Use humorous interventions after the nurse attended to the physical or emotional pain that the client might be experiencing.
4.‐ Try different jokes. If the client does not respond with observable laughter, he is unable to appreciate the humor the nurse shared.
3739 The nurse is participating in a therapeutic touch Correct answer: 4 When assessing the client's energy fields, the nurse is noting changes in temperature and Knowledge of how to administer a therapeutic touch session and when to stop the session
session with a client. The nurse evaluates that it would density; when there are none, it indicates a normal response. The end of the therapeutic touch is needed to answer this question.
be appropriate to stop the session when which of the session occurs when the practitioner assesses the whole body and tries to balance the areas
following occurs? where the imbalances have been felt. This indicates that the client is relaxed, and the nurse
still can provide therapy with the client sleeping. There is no minimal or maximal time
requirement for a session. It is recommended to stop a therapeutic touch session when the
imbalance in the energy field is resolved.

1.‐ There is no noticeable difference in temperature or density of energy fields.


2.‐ The client's muscles relax and the client falls asleep.
3.‐ The session has lasted the maximum time of one hour.
4.‐ The practitioner perceives that the imbalance have been resolved.

3740 The nurse is caring for a client who is using Correct answer: 4 Massage is a CAM therapy. Use of relaxation tapes is also a CAM technique. Seeking the An understanding of what therapies are involved in CAM is needed to answer this question
complementary and alternative medicine (CAM) assistance of an acupuncturist is also a CAM therapy. Having surgical repair of the foot is using correctly.
therapies for treatment of foot pain. Which statement medical therapy and not CAM therapy. CAM therapies are holistic therapies that the client
by the client does not reflect accurate use of CAM uses alongside the conventional medical management of the illness.
therapies?
1.‐ "I go see my massage therapist to enhance my range of motion."
2.‐ "I try to listen to my relaxation tapes to decrease the pain intensity."
3.‐ "I go see the acupuncturist to decrease the pain in my foot."
4.‐ "I plan to have surgery on my foot next month."

3741 The relative health and well‐being of the nurse in Correct answer: 4 Nurses need to work as a team to receive the support they need to meet the needs of all the The nurse would need to know that an essential component to care for others holistically
relation to helping clients constitutes a vital force in clients. Working through breaks will lead to exhaustion, and is not making the needs of the is for the nurse to care for herself first.
the healing process. A method that would be helpful to nurse a priority. By caring for the nurse's needs, she will be better equipped to care for the
nurses in fostering their own health is: needs of the clients. Setting unrealistic goals will lead to frustration when they cannot be
accomplished. It is best to start out with small, achievable goals to build confidence, and then
increase the difficulty of the goals to obtain the ultimate outcome. Self‐care of the nurse is
essential, and needs to be the top priority for the nurse to be able to care for her client’s
holistic needs.
1.‐ Trying to do things independently, so as to not bother the other nurses.
2.‐ Working through breaks to meet the needs of the clients.
3.‐ Setting unrealistic goals.
4.‐ Reflecting on their own beliefs and values, and making self‐care a priority.

3742 A client has returned to the nursing unit following a Correct answer: 1 The status of the client's airway and breathing is of highest concern. Once the nurse has Remember the ABC's: airway, breathing, and circulation. In questions asking for a priority
tracheostomy. The nurse would place highest priority assessed the airway and breathing, then the amount of oxygen and dressing status can be action, all options may be correct, and you need to select the option that is most important
on assessing which of the following? assessed. Finally, the time lapse since any analgesic medication can be determined. or timely. Remembering the sequence of airway, breathing, and circulation may help with
questions that relate to respiratory or circulatory disorders or procedures.

1.‐ Respiratory rate and breath sounds


2.‐ Amount of oxygen ordered to be delivered
3.‐ How long ago client received any pain medication
4.‐ Status of tracheostomy dressing

3743 The nurse is providing care to a client who had a Correct answer: 3 The priority action of the nurse restores a patent airway. With this in mind, the nurse spreads Remember the ABC's: airway, breathing, and circulation. The correct answer is one that
tracheostomy performed 2 weeks ago. The client the retention sutures to reopen the stomal area. The nurse then quickly calls aloud for help so directly affects the client's airway, which is opening the stoma. The other options are
coughs the tube out of the trachea. Which of the assistance will arrive to aid in tube reinsertion. The nurse is not likely to suction the area at this incorrect because they either are not the first action (options 1 and 4) or may not be done
following actions should the nurse take first? time, and the nurse would reinsert a new tracheostomy tube if allowed by agency policy, since at all (option 2).
the tube has been in place for more than 72 hours.

1.‐ Call aloud for help.


2.‐ Suction the stoma to remove residual secretions.
3.‐ Grasp and spread the retention sutures to open the stoma.
4.‐ Attempt to reinsert a new tracheostomy tube.

3744 The client has just had emergency intubation for Correct answer: 2 The first action by the nurse is to assess for bilateral breath sounds as an initial indication of Remember the ABC's: airway, breathing, and circulation. The correct answer is one that
respiratory distress. Immediately after endotracheal correct tube placement. The nurse would next secure the tube and then call for chest x‐ray to directly affects the client's airway, which is assessing for bilateral breath sounds. Because
tube insertion, which of the following actions by the confirm tube placement. Once the client's airway and breathing have been attended to, then the question asks for the priority action of the nurse, the other actions must be
nurse is most appropriate? the nurse can assure the client about alternative communication means. systematically eliminated.

1.‐ Tape the tube securely in place.


2.‐ Assess for bilateral breath sounds.
3.‐ Call for a chest x‐ray to determine placement.
4.‐ Assure the client that alternative communication means will be provided.

3745 Which of the following respiratory assessment Correct answer: 4 A harsh or crowing sound with inspiration indicates stridor, which is consistent with airway Note the critical words of greatest concern. This tells you that the correct option is the
findings is of greatest concern to the nurse following narrowing and edema following endotracheal tube removal. This is of greatest concern finding that is most abnormal. Use the process of elimination and knowledge of normal and
endotracheal tube extubation? because it could lead to upper respiratory obstruction. The nurse needs to notify the physician. abnormal physical assessment data to make a selection.
The other options are of less concern, since clients may be expected to have secretions or
some rhonchi immediately after tube removal. An increase in respiratory rate from 16–20
bears watching for trends but is still with normal limits.

1.‐ Increased respiratory rate from 16–20


2.‐ Scattered bilateral rhonchi
3.‐ Expectoration of whitish yellow secretions
4.‐ A harsh or crowing sound with inspiration

3746 A client with a closed chest drainage system tries to Correct answer: 1 The priority action of the nurse is to submerge the tube in sterile water or saline to Note the critical word first in the question. This tells you that more than one option may
get out of bed alone and disconnects the chest tube reestablish the underwater seal. This will prevent the client from sucking air through the chest be technically correct but one is better than the others based on the client’s status or the
from the drainage system, which falls on the floor. tube into the pleural space during inspiration, thereby causing pneumothorax. After this initial needs of the situation. Recall that an underwater seal is critical to chest tube functioning to
Which of the following actions should the nurse take action, the nurse would assess the client's respiratory status, set up a new system, and then select the correct option.
first upon entering the client's room? check the client's full vital signs before reporting incident to the physician.

1.‐ Submerge the tube in sterile water or saline.


2.‐ Set up and attach a new closed chest drainage system.
3.‐ Assess the client's respiratory status.
4.‐ Check the client's pulse and blood pressure.

3747 Following chest tube insertion, the nurse notes Correct answer: 1 The nurse should document this normal finding and continue to monitor. Fluid addition or The wording of the question tells you that there is a single correct answer. Recall that
gentle, continuous bubbling in the suction control removal is based on fluid level, not on bubbling action of suction. The nurse should not turn up gentle bubbling is normal to guide your answer. Use nursing knowledge about closed chest
chamber of the closed chest drainage system. Which suction because the gentle bubbling indicates proper function, and increased suction could drainage systems and the process of elimination to make a selection.
of the following actions should the nurse plan to take cause more rapid evaporation of water from chamber.
at this time?
1.‐ Document and continue to monitor the bubbling.
2.‐ Add water to the suction control chamber.
3.‐ Remove water from the suction control chamber.
4.‐ Turn up the suction on the wall suction unit.
3748 During routine chest tube assessment, the nurse Correct answer: 2 Continuous bubbling in the water seal chamber most often indicates a leak or loose The core issue of the question is the significance of finding continuous bubbling in the
notes the presence of continuous bubbling in the connection in the system, and air is being sucked continuously into the closed chest drainage water seal chamber. Use the process of elimination and knowledge of closed chest drainage
water seal chamber of the closed chest drainage system. If the client experienced a new large pneumothorax, there could be rapid bubbling, but systems to make a selection.
system. The nurse suspects that which of the following this is not the most likely explanation. Turning up the suction on the wall unit would increase
has most likely occurred? the bubbling in the suction control chamber, not the water seal chamber. Taping the
connections too tightly is not a concern.
1.‐ The client has developed a sudden new pneumothorax.
2.‐ There is an air leak in the system.
3.‐ The wall suction unit has been set to intermediate or high level instead of low suction.
4.‐ The connections have been taped too tightly.

3749 The client is scheduled for removal of a chest tube at Correct answer: 2 The client should be premedicated approximately 30 minutes prior to chest tube removal if The core issue of the question is what action is timely 30 minutes before chest tube
09:00. At approximately 08:30, the nurse should take the client has an analgesic order and the medication can be given at this time. It is the removal. Analyze each option to determine whether each needs to occur at that time.
which of the following actions? physician’s responsibility to determine the results of the daily chest x‐ray. Obtaining Recall that analgesics are usually given about 30 minutes prior to a painful procedure to
equipment for removal and explaining the procedure to the client can be done earlier or later select the correct option.
than the timeframe indicated.
1.‐ Call the radiology department for a telephone report of the morning chest x‐ray findings.
2.‐ Premedicate the client with an analgesic if it is ordered and can be given at this time.
3.‐ Ensure that a suture‐removal set and dressing materials are available.
4.‐ Explain to the client the upcoming removal procedure.

3750 The nurse has received a telephone order to irrigate a Correct answer: 2 The maximum amount of fluid that should be used to irrigate a nephrostomy tube is 5 mL. The core issue of the question is knowledge of appropriate volumes of fluid that should be
nephrostomy tube after notifying the physician that it The nurse should also use strict aseptic technique to prevent infection of the renal pelvis as a used to irrigate tubes such as a nephrostomy tube. Eliminate the smallest and largest
has stopped draining. The nurse plans to use no more result of the procedure. numbers as being least plausible. Choose 5 mL over 10 mL after visualizing the amount in
than how many milliliters to carry out this procedure the syringe and estimating the size of the renal pelvis. Memorize this number if the
safely? question was difficult.
1.‐ 2 mL
2.‐ 5 mL
3.‐ 10 mL
4.‐ 20 mL

3751 The nurse should take which of the following actions Correct answer: 3 The nurse should ensure that the tubing is free of kinks or other obstructions to urine flow. Use general principles of tube management to answer the question. Eliminate option 4
when caring for a client with a nephrostomy tube? The tube is irrigated according to physician order only. The tube should never be clamped. first as being hazardous. Eliminate option 1 next as being excessive. Choose option 3 over 2
Taping the drainage bag to the bedrail is dangerous because it could cause traction when the knowing that the tubing should be kept free of kinks and that these tubes should not be
client moves in bed and become dislodged. clamped.
1.‐ Irrigate the tube every hour regardless of drainage.
2.‐ Keep a clamp at the bedside.
3.‐ Ensure the tubing is free of kinks.
4.‐ Tape the drainage bag to the bedrail.

3752 A nurse is assigned to a client with a nasogastric tube Correct answer: 1 Gastric pH is acidic and readings should be 4 or less if the tube is placed properly in the The core issue of the question is knowledge of pH readings that are consistent with
and is checking gastric pH to verify correct tube stomach. The other options indicate placement in the intestine or higher up in the esophagus, placement of a nasogastric tube in the stomach. Use specific nursing knowledge and the
placement. The nurse determines that the tube is since normal body pH is 7.35 to 7.45. process of elimination to make a selection.
properly positioned after obtaining which of the
following pH readings?
1.‐ 4
2.‐ 6
3.‐ 7
4.‐ 8
3753 The nurse would use which of the following Correct answer: 4 The nurse correctly measures the distance from tip of nose to earlobe and then to the xiphoid The core issue of the question is knowledge of how to properly measure a client for
landmarks to correctly measure a client prior to process and marks the tube at this length prior to insertion. The other options identify one or nasogastric tube insertion. Use specific nursing knowledge and the process of elimination to
nasogastric tube insertion? more incorrect landmarks. make a selection.
1.‐ Tip of nose, mandible, and sternal notch
2.‐ Tip of nose, mandible, and xiphoid process
3.‐ Tip of nose, earlobe, and sternal notch
4.‐ Tip of nose, earlobe, and xiphoid process

3754 A client with a partial bowel obstruction will have a Correct answer: 4 In order for the tube to migrate to the area of intestinal blockage, the tube must pass through The core issue of the question is knowledge of proper client position following nasoenteric
nasoenteric tube placed by the physician later in the the pyloric sphincter of the stomach. Recall that this tube has a weighted tip and thus gravity tube placement. Visualize tube movement using laws of gravity to aid in making a selection.
day. The nurse explains to the client that which of the will affect its movement, as will peristalsis. Positioning the client with head elevated and on Gravity should be a prime consideration as a possible influence whenever answering
following positions will be utilized following tube the right side will utilize gravity to help the tube migrate into the intestines. The other questions related to tube placement.
placement so it will migrate to the intended area? responses will lead to less effective tube movement.

1.‐ Flat and on the left side


2.‐ Flat and on the right side
3.‐ Head of bed elevated and on the left side
4.‐ Head of bed elevated and on the right side

3755 The nurse enters the room of a client who underwent Correct answer: 3 Nasoenteric tubes are not taped in place until they have migrated to proper position and The core issue of the question is knowledge that nasoenteric tubes are not taped until
insertion of a nasoenteric tube for partial bowel been confirmed by x‐ray. The nurse should note the assessment finding on the medical record. they have reached final position. Use specific nursing knowledge and the process of
obstruction the previous evening. The nurse notes that The nurse does not need to call the physician, and it is unnecessary to immediately determine elimination to answer this question.
the tube is not taped at the nose. Which of the the x‐rays that are scheduled. It is completely unnecessary to notify the physician at this time.
following actions is most appropriate at this time?

1.‐ Call the physician immediately.


2.‐ Tape the tube in place.
3.‐ Note the finding on the client's flowsheet.
4.‐ Call the radiology department to see if an abdominal x‐ray has been done.

3756 The nurse is about to receive an intershift report on a Correct answer: 2 A Sengstaken‐Blakemore tube is inserted to control bleeding from esophageal varices, which The core issue of the question is knowledge of the rationale for placement of a Sengstaken‐
client who has a Sengstaken‐Blakemore tube in place. is the primary health problem of concern with use of this tube. The underlying health problem Blakemore tube. Use knowledge of pathophysiology and the process of elimination to make
The nurse expects that the client has which of the that causes the bleeding is portal hypertension, which is a complication of cirrhosis of the liver. a selection.
following health problems as the primary reason for Abdominal ascites may also accompany cirrhosis.
tube placement?
1.‐ Cirrhosis of the liver
2.‐ Esophageal varices
3.‐ Portal hypertension
4.‐ Abdominal ascites

3757 The nurse has just assisted with insertion of a Correct answer: 3 Scissors needs to be kept at the bedside of a client who has a Sengstaken‐Blakemore tube. If The core issue of the question is knowledge that tube dislodgement can block the airway
Sengstaken‐Blakemore tube. Before leaving the client's the tube becomes dislodged and the client cannot breathe, the nurse cuts the tube to allow and that scissors are needed for rapid balloon deflation if this occurs. Use nursing
room, the nurse ensures that which of the following the balloons to deflate and restore a patent airway. A suction machine and oxygen are knowledge and the process of elimination to make a selection.
equipment is at the bedside in case of tube generally helpful airway adjuncts, but they do not apply to this situation. A laryngoscope is
dislodgement? used to insert an endotracheal tube but it does not apply to this question.

1.‐ Suction machine


2.‐ Oxygen mask
3.‐ Scissors
4.‐ Laryngoscope
3758 The nurse working in the emergency department Correct answer: 3 An Ewald tube is a large‐bore tube used to evacuate stomach contents rapidly following The core issue of the question is knowledge of the use of various types of drainage tubes.
learns that a client will be arriving who took an poisoning or overdose. Minnesota and Sengstaken‐Blakemore tubes are used for clients with Use nursing knowledge and the process of elimination to make a selection.
overdose of acetaminophen (Tylenol) a short while bleeding esophageal varices. A Miller‐Abbot tube is a nasoenteric tube used to decompress the
ago. The nurse should anticipate that which of the bowel with small bowel obstruction.
following tubes will be used to evacuate the client's
stomach upon arrival?

1.‐ Minnesota
2.‐ Sengstaken‐Blakemore
3.‐ Ewald
4.‐ Miller‐Abbott

3759 A client who took an overdose of a prescribed Correct answer: 1 The risk of aspiration with gastric lavage is of concern to the nurse. For this reason, The core issue of the question is knowledge that gastric lavage can lead to aspiration as a
medication was treated with gastric lavage. The nurse assessment of respiratory status, including respiratory rate and breath sounds, is of great complication. Use nursing knowledge and the process of elimination to make a selection.
assesses the client carefully for which of the following concern. Other vital signs are also important as a measure of general condition but are not
as a priority to detect possible complications of focused on detection of complications of this procedure. Urine output is of general concern,
treatment? but peripheral edema is not a priority.
1.‐ Respiratory status and breath sounds
2.‐ Heart rate and blood pressure
3.‐ Skin color and body temperature
4.‐ Urine output and peripheral edema

3760 The nurse has emptied a Jackson Pratt wound‐ Correct answer: 2 The nurse should squeeze the collecting chamber to reestablish negative pressure and suction The core issue of the question is which action by the nurse will reestablish suction to a
drainage device and needs to reestablish suction to the to the device. The nurse then wipes the port with alcohol before closing to reduce the risk of Jackson Pratt wound‐drainage device. Use nursing knowledge and the process of
tube. Which of the following actions should the nurse infection. The tubing should always be free of kinks to prevent obstruction. elimination to make a selection.
take to accomplish this objective?

1.‐ Ensure the tubing has no kinks.


2.‐ Squeeze the collection chamber.
3.‐ Wipe the port with alcohol.
4.‐ Close the cap on the device.

3761 The nurse has received report on a client who Correct answer: 3 A Penrose drain allows free flow of abdominal drainage out of the abdominal cavity and onto The core issue of the question is knowledge that this type of drain can cause skin irritation
underwent pelvic exenteration the previous day and thick layers of gauze dressings that are placed around the drain. It is used when moderate to by nature of the volume of drainage. Use nursing knowledge and the process of elimination
has a Penrose drain placed in the lower abdomen. The large amounts of drainage are expected, as with extensive abdominal surgeries. The nurse to make a selection.
nurse should take which of the following actions in the should assess the skin for irritation and breakdown from contact with abdominal skin if
care of this wound drain? dressing changes are not done on time or if an insufficient number of gauze dressings are used
around drain. The drain may be advanced over days for gradual removal. The surgeon does not
need to be notified of moderate amounts of drainage because it is expected.

1.‐ Ensure that the drain always stays in the original position placed by surgeon.
2.‐ Place only a few gauze dressings around the tube to allow for easier assessment.
3.‐ Assess the abdominal skin for irritation or breakdown with dressing changes.
4.‐ Notify the physician for moderate amount of drainage.

3762 The nurse has an order to insert a nasogastric tube Correct answer: 2, 3, 1, The client's head of bed is raised first because airway and breathing are the priority. Next the The core issue of the question is knowledge of the insertion procedure for a nasogastric
into the stomach of an assigned client. Place in order 4, 5 tube is measured for accurate length of insertion. The tube is then advanced past the tube. Use nursing knowledge to sequence the steps that the nurse needs to take. Visualize
the steps that the nurse would follow to complete the nasopharynx. The client is then asked to take sips to help with tube advancement into the the procedure to aid in answering the question.
procedure. Click and drag the options below to move stomach. Finally, the tube is taped once placement is assured.
them up or down.
1.‐ Place distal end of tube at tip of nose and measure to earlobe and then to xiphoid process to determine distance for tube insertion
2.‐ Insert tube into naris and advance upward and backward until resistance met; rotate catheter gently and advance into nasopharynx
3.‐ Sit client upright (high Fowler's position)
4.‐ Ask client to take sips of water if able while tube advanced gently into stomach
5.‐ Tape tube in place

3763 A client has had a tracheostomy tube in place for Correct answer: 3 Catheters should be inserted only during one suctioning period to minimize the risk of The so‐called 10‐second rule for suctioning allows for maximum suctioning without a loss
three days. The client has learned how to assist in total contamination from allowing the catheter to lie out of the sterile packet, which would allow of oxygen to the client. When moving from a sterile tracheostomy opening to the oral
tracheostomy care as needed. Which statement would bacteria to grow between suctioning periods. cavity, it is acceptable to move from sterile to clean surfaces. It is not acceptable to move
indicate a need for additional teaching? from oral to tracheostomy opening, as the mouth contains bacteria that could cross‐
contaminate the sterile tracheostomy environment. Allowing the significant other to
participate in the process in this non‐threatening manner encourages participation in the
task despite minimal knowledge. In addition, the significant other can support the client's
self‐esteem and independence in the process by the accomplishment of the task as needed.

1.‐ The client says, "I can suction no more than 10 seconds each time before removing the catheter."
2.‐ The client says, "I can reuse the catheter for oral secretions after the tracheostomy suctioning, if needed."
3.‐ The client says, "I will reuse the catheters several times before discarding them, to save money."
4.‐ The client says, "I let my significant other assist by holding the mirror, while I clean underneath the flat part (flange) of the tracheostomy."

3764 A client is to receive a soap suds enema. Which Correct answer: 4 Deep breaths will relax the abdominal muscles and allow additional fluids to enter the colon Three hours is too long to hold a soap suds enema. Up to 1 hour is more than sufficient for
directions would be included to assist in a successful while under less abdominal pressure. the soap suds and water to stimulate peristalsis of the bowel. Most clients cannot hold it
outcome? The client should: that long. Instantaneous results from an enema are more likely and realistic. Bearing down
with the abdominal muscles will increase abdominal pressure, not decreasing the cramping
but increasing peristalsis. An upright position is not the preferred position for the fluid to be
instilled for an enema, since the fluid would have to go against gravity as it travels up the
colon. The left side‐lying (Sims') position is preferred for the solution to flow downward into
the rectum and descending colon with gravity. Sitting on the toilet will not give the
maximum benefit of the enema itself. For safety purposes, a bedpan or bedside commode
would be safer than trying to run across the room to the toilet. Instilling the maximum
amount of solution will allow the most productive evacuation. The urge to void can be
delayed by clamping the solution temporarily or slowing down the solution flow rate until
the feeling passes temporarily to be able to complete the enema instillation.

1.‐ Hold the enema for at least 3 hours to get the best results.
2.‐ Bear down with the abdominal muscles when cramping, to decrease abdominal pressure.
3.‐ Sit on the toilet to decrease the risk of falls when hurrying to the toilet.
4.‐ Take deep breaths when feeling the urge to go until all of the solution is given.

3765 When a rectal tube is used to collect continuous Correct answer: 2 Increasing girth shows a backup or buildup of contents or air in the abdominal area without Bowel sounds will indicate the speed of peristalsis and rate of elimination.<BR />
watery diarrhea, which assessment would indicate an adequate movement within the gastrointestinal system to remove stool. If the source of the
urgent need that should be addressed by the nurse? problem is not identified, additional complications will develop as stool stagnates or goes
backwards in the GI tract. The most common problem of loss of peristalsis is an ileus.

1.‐ Increasing bowel sounds


2.‐ Increasing girth of the abdomen
3.‐ Decreasing bowel sounds
4.‐ Increased volume of stool contents
3766 The nurse assesses the functioning of a client's Salem Correct answer: 3 The air vent opening should remain open, to allow air to decompress the stomach, and Normal pH of stomach contents is strongly acid (0–4). If the tube passed into the
sump nasogastric suction following abdominal surgery. suction is connected to the larger, primary opening of the double‐lumen tube. An anti‐reflex intestines, the secretions will yield a pH of 6–8. Accidental placement of the tube in the
Which indicates a problem exists with the suctioning? valve might be covering the air vent, but it does allow air to enter. Decompressing the stomach pulmonary tree would yield a 6–7 pH range. Suction for nasogastric tubing is to be in the
is achieved when some air enters the stomach through the air vent to prevent the pull of low range of 30–40 mm Hg for continuous suction or high, intermittent suction up to 120
stomach mucosa into the hole of the larger lumen to which the suction is connected. mm Hg. Normal gastric secretions are yellow–greenish to tan or off‐white. Duodenal
samples can be deep yellow. Pulmonary secretions are clear to light yellow. The color of
secretions is not a reliable indicator of location of the tube. Connecting the suction tubing
to the air vent tubing will pull secretions from the stomach through the smaller tubing
canal, and probably clog the tubing. If the air vent is clogged, the risk of damage to the
mucosa is increased, and fewer secretions will be removed from the stomach in the long
run. Only air should be pushed through the air vent tubing, to maintain the integrity of the
system.
1.‐ A pH of 3.0 on the aspirated gastric contents when tested
2.‐ Suction regulator set at 100 mmHg intermittent suction
3.‐ Suction tubing connected to the air vent tube
4.‐ Gastric secretions that are yellow/green in color

3767 Which of the following would indicate a serious Correct answer: 2 Bradycardia occurs when the vagus nerve has been stimulated during the suctioning process. Note that all of the incorrect responses indicate a temporary decrease in oxygen when
complication from excessive suctioning of an Vagal stimulation will trigger a decreased heart rate, and can lead to cardiac arrest if the heart suctioning. Recall that bradycardia indicates a vagal stimulation response that could cause
endotracheal tube that requires immediate nursing rate drops too low. The nurse should stop the suctioning until vagal stimulation is stopped and cardiac arrest to choose this as the correct answer.
intervention? the client is stable. Tachycardia is expected in response to suctioning due to stimulation, as the
heart rate increases to compensate for the decreased oxygen levels caused by suctioning.
Pallor and slight cyanosis could be expected to occur as oxygen is removed along with the
respiratory secretions. However, if severe cyanosis occurred, it would mean that too much
oxygen has been removed and, if present, would be of great concern.

1.‐ Tachycardia
2.‐ Bradycardia
3.‐ Pallor
4.‐ Slight cyanosis

3768 When evaluating a new graduate nurse, which Correct answer: 2 Hyperventilation is giving too much volume of air into the lungs, and is not recommended, Maintaining maximum oxygenation should always be the focus when suctioning the client
statement indicates a need for additional training due to the risk of rupture of lung tissue. Other responses are appropriate actions that are who has a compromised respiratory function. Hyperventilation means to overextend the
when performing nasotracheal suctioning? The new recommended to minimize the loss of oxygen when suctioning the client. lung tissue. This is not recommended in an already‐compromised client, since this can
graduate that says she will: rupture alveoli and decrease the surface areas available for oxygen exchange. Waiting will
allow the client to regain any lost oxygen between suctioning episodes. Hyperoxygenating is
beneficial to minimize oxygen loss during suctioning, since extra oxygen will be available by
flushing the client with 100% oxygen for 1–2 minutes prior to the suctioning process.
Rotating the catheter only on the way out when suctioning will remove only secretions and
prevent pulling tissue into the catheter tip. Rotating the catheter when entering the
endotracheal tube is of no benefit, since the suction is only on during the withdrawal of the
catheter.

1.‐ Wait 2–3 minutes before suctioning again.


2.‐ Hyperventilate the lungs prior to suctioning.
3.‐ Hyperoxygenate before and after suctioning.
4.‐ Rotate the suction catheter only on the way out when suctioning.
3769 When a client is dealing with the stress of managing a Correct answer: 4 Stress of a life change such as having a permanent suprapubic catheter must be addressed Answer option 1 dealt only with giving facts or knowledge related to the catheter, and
permanent suprapubic catheter, the nurse should: like any other loss with which a client must deal while hospitalized. By exploring feelings and does not explore how the client feels about the tube. Option 2 dismisses the client's
coping strategies that are present, the nurse will be able to work on strategies using familiar feelings by giving medication to cover up or prevent the exploration of the feelings with
coping strategies that have been successful for the client. Also, if the strategies have not which the client should be dealing during the stressful process. Option 3 is generalizing the
worked in the past to reduce stress, then new strategies can be introduced to the client as issue and then giving false reassurance that everything will be OK. The nurse can not give
options to deal with this new stressor once the feelings have been clarified with the client. that reassurance, because anything could happen. Only answer 4 addresses the actual
feelings and strategies to cope with the stress itself.

1.‐ Tell the client all of the facts, to make sure home care will be done correctly.
2.‐ Give an anti‐anxiety med that is ordered.
3.‐ Reassure the client that many people live with these and have no problems.
4.‐ Identify coping skills, and explore the client’s feelings.

3770 Following a chest trauma three days ago, a chest tube Correct answer: 4 The chest tube in the second intercostals space is used to remove free air in the chest cavity. The chest tube is inserted to keep the lung inflated by removing free air from the chest
has been in place at the right second intercostals space Therefore, the chest wall should move equally bilaterally by the third day, when the air is cavity. Lung sounds should be present. There should be no secretions from the tubing, since
of the anterior chest wall. The nurse would expect removed. it only removes air. Bubbling on a continuous basis indicates an air leak somewhere in the
which assessment? system, and requires immediate intervention. Bilateral chest movement should be equal, to
show that both sides are fully expanding, since the lungs would be inflated by the third day.

1.‐ Absence of breath sounds in upper‐right chest area


2.‐ Secretions in the tubing to be serous
3.‐ Continuous bubbling when the collection device is connected to low wall suction
4.‐ Bilateral equal movement of the anterior chest wall

3771 A client has a new chest tube located at the base of Correct answer: 2 Crepitus would indicate an air leak into the subcutaneous tissues. This is a common side Clamping the tube is not recommended unless there is an air leak or break in the closed
thoracic cavity. Which activity is expected during effect that requires further assessment to make sure no further leaks are occurring. None of system. If the tube is clamped and the client inhales, the lung tissue might collapse again
nursing care? The nurse should: the other actions are normally done for a chest tube drain located at the base of the thoracic from the lack of negative pressure that is to be maintained by the water seal system.
cavity, which is designed to drain secretions from the chest cavity. Dressing changes are not done daily, since the sealed system needs to be maintained, and
risk of tube displacement would increase with manipulation of the dressing. Dressings are
reinforced or retaped but not changed without medical supervision. Drainage chambers are
meant to be measured and not emptied. To empty the chamber, a break in the closed
system would be necessary, and air could possibly enter the system as the client inhales
(while it is open), causing further tissue to collapse. Crepitus is painless air bubbles under
the skin that can occur within 1½ inches around the chest tube, and can easily be palpated
with the fingers. If it extends further than this distance, additional leaks should be located.
Options 1, 2, and 4 are never appropriate.

1.‐ Teach the client to clamp the tube when getting up to walk.
2.‐ Palpate the chest wall for crepitus near the insertion site.
3.‐ Change the dressing daily, to decrease infection.
4.‐ Empty the drainage chamber at the end of each shift.

3772 When a drainage tube is located at the site of a Correct answer: 1, 2 Edema develops, pushing both internally and externally when it forms. Edema pushes on Swelling often indicates that the drain is occluded or the suction has been lost. A leak in
radical neck dissection, and edema is noted, what whatever structures are near it (airway, gag point, vocal cords). Pressure can occlude any of the tubing or displacement of the tube should also be assessed. Waiting for the edema to
should the nurse do? Select all that apply. the areas that are vital to air flow, speech, or gagging. Rechecking the equipment will eliminate change can be life‐threatening if it cuts off the airway or ruptures the incision, causing
the possible cause of the loss of suction from no negative pressure. With no negative pressure bleeding. Opening up a suture for release of pressure on an incision is a medical
to remove secretions, edema will stay in the tissues, create an obstruction, or damage vital management, and should not be a decision of the professional nurse. Opening the incision
tissue. still does not guarantee an open airway. An airway must be maintained at all times for
maximum O&lt;sub&gt;2&lt;/sub&gt;/CO&lt;sub&gt;2&lt;/sub&gt; exchange for life to
continue.

1.‐ Assess the airway, gag reflex, and the ability to talk.
2.‐ Check for negative pressure, to make sure suction is present in the drainage tube.
3.‐ Wait 2 hours to see if the edema increases in size or location.
4.‐ Release the suture before it ruptures the incision.
5.‐ Check quickly for edema of the feet and sacrum.

3773 The doctor removed 500 mL of solution by a Correct answer: 2 Coughing would indicate irritation or increased stimulation of the diaphragm. When the fluids Once fluids have been drained from the thoracic cavity, the lungs are free to expand to
thoracentesis. Following the procedure, which are removed, all of the other symptoms are expected as signs of improved ventilation and the allow air flow for normal lung sounds, which in turn increases oxygen intake and gas
assessment would require immediate notification of removal of the pleural effusion. exchange. Skin coloring should be increasingly less cyanotic or pale. Crackles indicate fluid
the doctor? present in the lung bases, and once the fluid is removed, a decrease of crackles should be
noted upon auscultation. An increased frequency in coughing indicates increased irritability
of the diaphragm.
1.‐ Increased lung sounds in the area where the pleural effusion was located
2.‐ Increased coughing after the procedure
3.‐ Decreased crackles noted at the base of the lungs
4.‐ Decreased cyanosis and perioral pallor

3774 A new postoperative client has hourly hemodynamic Correct answer: 2 A gradual decline in urinary output shows lessening pressures through the kidney and Central venous pressure (CVP) reflects hydration status of the body, and can vary slightly
assessments. Which would indicate that an imbalance potential cardiovascular collapse. When urinary output is below 30 mL/hour on a consistent based upon position of the tubing and several other factors. The CVP readings are fairly
is present, and needs urgent care? basis, either the circulating volume is decreasing or the kidneys are not perfusing. Either of stable or consistent, and do not represent a trend upward or downward. Chest tube
these requires immediate medical attention. drainage is not excessive, and can vary with the position of the client. Unless the chest tube
drainage is excessive, or there is a major change in coloring to frank bleeding, the drainage
is not in need of urgent care. The listed pulses, although slightly fast, do not represent a
trend or change that is serious at present. The declining urinary output, however, reflects
an immediate need for medical management or further assessments by professionals.

1.‐ CVP pressures: 8, 11, 10, 9 mm of Hg


2.‐ Urinary output: 40 mL, 30 mL, 20 mL, 20 mL
3.‐ Chest tube drainage: 60 mL, 45 mL, 50 mL, 20 mL
4.‐ Pulses: 94, 100, 98, 92 beats per minute

3775 Which symptom would indicate excessive electrolytes Correct answer: 1 Option 1's symptoms are those of potassium lost during excessive washing out of gastric This question requires specific knowledge of electrolyte imbalances, their cause, and their
have been removed from a client with a nasogastric secretions, causing hypokalemia. Option 2's symptoms are those of hyperkalemia. Option 3's manifestations. Take time to review this information if the question was difficult.
tube who has been eating large amounts of ice? symptoms are those of hypocalcemia. Option 4's symptoms are those of hypercalcemia. Note
that pathological fractures occur as the body pulls calcium from the bones to maintain the high
blood levels.
1.‐ EKG changes: U waves, flat T; anorexia, muscle weakness, ileus
2.‐ EKG changes: peaked T wave, prolonged QRS; muscle cramps, diarrhea
3.‐ Tetany, numbness of fingers/toes, twitching of nose or lips
4.‐ Pathological fractures, bradycardia, slowed reflexes

3776 A client had a total hip replacement with the insertion Correct answer: 2 By the second day, the drainage fluid should have slowed down considerably. A volume of 500 Options 1 and 3 are normal, expected progression of drainage from a hip replacement.
of a Hemovac suction drain for drainage during the mL is almost two units of blood, and if the drainage is still sanguineous on the second day, the Option 4 reflects increased muscle action that is also common during increased muscle
postoperative period. When assessing the drainage, nurse should suspect that a problem is present. Frank bleeding should have stopped soon after activity, creating an increased drainage at the site where the tube has been removed. Only
which statement would require immediate surgery, and secretions gradually should become more serosanguineous in nature within a few the amount and color of drainage in option 2 reflect a cause for continued action by the
intervention by the nurse? hours post‐op. nurse.
1.‐ In the first hour postoperative, 250 mL of sanguineous fluid was drained.
2.‐ During a four‐hour period on the second day, 500 mL of sanguineous fluid was noted.
3.‐ On the third day post‐op, less than 30 mL/hour of serous fluid was noted.
4.‐ A 3‐inch circle of sanguineous fluid was noted on the 4‐by‐4 dressing after the client was up walking after the drain was removed on the third day.

3777 A client has a left arteriovenous (AV) graft in the Correct answer: 4 Normal capillary refill is less than 3 seconds; 5 seconds represents a delay in circulation. The Recall that the thrill and bruit indicate that there is adequate circulation through the graft,
forearm. Which nursing assessment would indicate a graft should have no negative impact on the circulation to the hand. Additional assessment and these should be present at all times. The visibility of the graft on the forearm depends
need for additional interventions by the nurse? needs to be performed by the nurse. upon the amount of musculature and subcutaneous fat that are present in the client.
1.‐ The thrill is present upon palpation of the graft.
2.‐ A bruit is noted upon auscultation.
3.‐ The graft is visible on the forearm.
4.‐ The capillary refill of the left hand is 5 seconds.

3778 When using vacuum‐assisted closure (VAC) for a Correct answer: 3 The wound must be open enough to place the foam dressing onto the site before the sealed The dressing is to be changed every two hours to maximize the effectiveness of secretion
wound, which finding would require additional dressing can be applied to minimize the damage to the site with the VAC suction. removal, and to minimize possible damage to the wound site. Only 2 cm of intact skin is
modification by the nurse to prevent complications? required, and one inch exceeds that guideline. Irrigating the wound with normal saline is
necessary to remove excessive exudates and to maintain moisture of wound for the
dressing to be effective. Within a 24‐hour period, the suction can be held for only 2 hours
per day to be effective.
1.‐ VAC suction dressing is changed every two hours.
2.‐ More than one inch of intact periwound tissue is available to get a tight seal.
3.‐ The wound is closed enough to seal the dressing without foam placement.
4.‐ Irrigation with normal saline is done during each dressing change.

3779 When inserting an 18‐gauge French urinary catheter Correct answer: 2 If the catheter is well lubricated, the obstruction is based upon the collapsing of tissue near Recall that by lifting and stretching out the shaft of the penis, the meatus is opened more
into a male client, the nurse met resistance when the the base of the penis. By lifting and using a 90‐degree angle for the shaft, the pathway is directly, and curling up of the urinary catheter is less likely to occur.
catheter was inserted less than one‐half of the opened. A smaller size is not the issue if the tissue is collapsed at the base; no matter what size
distance required. What actions would be most the catheter is, it will not pass (option 1). A stylet does increase the rigidity of the catheter
beneficial to complete the insertion of the catheter (option 3), but is not a nursing procedure. Only a trained doctor should use the stylet, since
into the bladder? damage can occur with its use. Placing the shaft of the penis parallel to the body (option 4) will
not open up the passageway for the catheter to pass.

1.‐ Removing that catheter and getting a smaller size


2.‐ Lifting the shaft of the penis to a 90‐degree angle to the abdomen
3.‐ Getting a stylette and using it to insert the urinary catheter
4.‐ Pulling the penile shaft to a parallel position with the body

3780 A urinary catheter has been in place for several days, Correct answer: 2 The urinary system should remain a closed‐seal system to minimize the risk of cross‐ Prior to collection of urine, the tubing often must be clamped temporarily to maintain
and the doctor has ordered a urine culture and contamination. The port hole is designed for puncturing for specimens. Using only the area some urine in the bladder, rather than draining it all into the bag as soon as it is produced.
sensitivity to be done. Which action by the nurse designed to be punctured (which will reseal when the needle is removed) is the only safe way After clamping, the only way to collect by a sterile procedure is through the port hole.
would be appropriate? to collect the specimen without cross‐contamination to the specimen or the catheter's Through breaking the seal between the tubing and the catheter itself, or using the drain tip
drainage system. in the bag, cross‐contamination can occur to the specimen and/or to the system. By
puncturing the catheter, you create a hole in the catheter that will not reseal, and the urine
can leak out and bacteria can enter.

1.‐ Get urine from the bag after wiping the drain tip with alcohol.
2.‐ Wipe the port hole with alcohol, and aspirate the urine with a sterile needle and syringe.
3.‐ Wipe the connection between the catheter and drainage tubing with alcohol, and open the connection at the catheter to collect the urine.
4.‐ Puncture the catheter near the meatus after wiping with alcohol, to aspirate the urine.

3781 When assessing for placement of a nasogastric tube, Correct answer: 2 Gastric pH should be between 0–4 pH. The color of the gastric content should be All nasogastric tubes are to be checked by radiological assessment, to be absolutely sure
which approach by the nurse would be most reliable yellow/green or tan, but this is a less reliable indicator of location. they are in the correct place. However, other assessments are helpful prior to the final
for confirmation of correct placement? assessment by x‐ray. Abdominal assessment by air is fairly reliable, but is not as reliable as
pH confirmation. The absence of gagging does not mean that the placement is correct, only
that the gag reflex is suppressed. In an unconscious or sedated client, the gag or cough
reflex might be damaged or non‐functioning; therefore, it should not be the basis for your
decision that the tube is in the correct location. Sticking the tube into a glass of water while
looking for bubbles can create a risk for aspiration of fluids from the cup, if the tube is in
the lung and the client inhales.

1.‐ Abdominal assessment with air pushed through the tubing


2.‐ Aspiration of secretions for confirmation of color and pH
3.‐ Placing the tip of the tubing into a glass of water to observe for bubbling
4.‐ An absence of gagging or coughing by the client

3782 When a small‐bore feeding tube is in place for Correct answer: 3 The feeding should not hang for more than 4 hours, if continuous. However, for intermittent Placement should be checked before each feeding. Retention checks are to be done prior
intermittent feedings every 6 hours, which nursing feedings, if a bag is used, the bag should be changed every day to minimize the risk of bacterial to each feeding, to assess peristalsis. For water, 30–60 mL is the usual amount used before
action is most appropriate? growth in the tubing. Most intermittent feedings are hand‐poured into a syringe. and after feedings to flush the tubing and maintain hydration.

1.‐ Placement should be checked each day.


2.‐ Retention checks should be done at least once a day.
3.‐ Bag tubing should be changed out each day.
4.‐ A minimum of 100 cc of water should be used before and after each feeding.

3783 While completing a nursing assessment, the client Correct answer: 3 Age above 65 is a risk factor for cataracts. Double vision, increased intraocular pressure, and The core issue of the question is knowledge of factors that increase client risk for
states he is 70 years old, has a history of blurry vision are signs of glaucoma. conditions affecting sensory perception. Use the process of elimination and nursing
staphylococcus infections, increased intraocular knowledge to make a selection.
pressure, and blurry vision. The nurse concludes that
which item reported by the client is a risk factor for
the development of cataracts?
1.‐ History of staphylococcus infections
2.‐ Increased intraocular pressure
3.‐ 70 years old
4.‐ Long complaint of blurry vision

3784 A 92‐year‐old client is in the hospital. The client is Correct answer: 3 Hearing loss, especially of upper‐range tones, is common in the elderly. Speaking to the client The core issue of this question is the optimal choice for communicating with a client who
very hard of hearing, and the nurse needs to do the slowly and in a lower‐pitched voice while facing the client is the best means of communication. is hearing impaired. Eliminate options 1 and 4 first as least helpful or unnecessary
admission interview. Which of the following should the Options 1 and 4 are not helpful, and option 2 is unnecessary. measures. Eliminate option 2 because it may not allow the client to see the nurse’s face,
nurse do when assessing the client? which assists in determining words when a client is hard of hearing.

1.‐ Obtain a cotton swab to clean cerumen in the client's ear before beginning the interview.
2.‐ Speak louder in the client's better ear after determining which has better hearing.
3.‐ Lower the pitch of the voice and face the client during the interview.
4.‐ Put new batteries in the hearing aid to ensure proper functioning.

3785 A 72‐year‐old client has been in the ICU for the past 2 Correct answer: 3 Providing the client with a clock and calendar helps the client to be oriented to time and date. The core issue of the question is nursing actions that can prevent the client from
days. Which intervention would be the most These would be meaningful stimuli for the client and decrease the chance for sensory experiencing sensory deprivation. Use knowledge of basic nursing measures to help a client
appropriate in decreasing the risk for sensory deprivation. It may not be realistic in an ICU to remove equipment from the room. Explaining stay oriented to time, place, and person make a selection.
deprivation? all procedures and routines would increase the risk of overload. Giving the client prolonged
rest periods would only increase the risk for deprivation.
1.‐ Remove equipment from the room.
2.‐ Explain all procedures and routines to the client upon admission.
3.‐ Provide a clock and calendar in the client's room.
4.‐ Provide the client with prolonged rest periods.

3786 A client has a sprained ankle, and the nurse must Correct answer: 1 To prevent vascular impairment, proper application of elastic bandages is required. Wrapping The core issue of the question is knowledge of basic wound‐care procedures. Use nursing
apply an elastic bandage to provide support to the distal to proximal is compatible with the flow of venous return. Wrapping the bandage evenly knowledge of these procedures and concepts related to blood flow to make your selection.
ankle. Which of the following actions should the nurse while stretching it ensures that there will be even tension applied to the extremity. Wrapping it
take during this procedure? loosely will not secure the bandage in place. Excessive pressure would cause circulation to be
compromised.
1.‐ Wrap the bandage while stretching it from distal extremity to proximal.
2.‐ Apply the bandage loosely around the extremity.
3.‐ Apply heavy pressure with each turn of the bandage.
4.‐ Start at the upper end of the extremity and work toward the distal end.

3787 All of the following clients appear in the emergency Correct answer: 4 A closed fracture has no break in the skin. A cat bite, a laceration, and a stab wound all impair The core issue of the question is appropriate use of antibiotic therapy. Restate this
room during one shift. The nurse would clarify with the skin integrity, which could lead to infection. question in the following way: "Which client is not at risk for infection?" Use the process of
physician the reason for an antibiotic order for a client elimination and nursing knowledge to make the selection that poses little risk of infection.
with which of the following injuries?

1.‐ Cat bite to the hand of an elderly client


2.‐ Laceration from broken glass in a 6‐year‐old client
3.‐ Stab wound in a 37‐year‐old client
4.‐ Closed‐fractured ankle in a 40‐year‐old soccer player

3788 A client is on complete bed rest and is at risk for Correct answer: 2 Disuse syndrome is a result of prolonged immobility. Stating "the client remains free of Recall that goal statements are indicators of what the nurse wants to happen as a result of
disuse syndrome. Which of the following client goals is contractures" describes in active terms the desired outcome for the client. The last two care. With this general principle in mind, eliminate each of the incorrect responses because
appropriate? options describe nursing activities to meet the stated client goal. The nurse has no control over they are focused on the nurse or are beyond the nurse’s control.
option 1.
1.‐ The client has shorter periods of immobility.
2.‐ The client remains free of contractures in lower extremities.
3.‐ The nurse turns the client every 2 hours.
4.‐ The nurse performs passive range of motion to lower extremities every 4 hours.

3789 This is the first hospitalization for an adult client who, Correct answer: 4 The client is in a new environment. Changes in environment bring about uncertainty, and the The core issue of the question is the ability to draw correct conclusions about client
after being admitted 3 days ago, is now having trouble client may be unable to sleep or may sleep less well than at home. Although the client is assessment data and translate it to a nursing diagnosis. In this case, the original problem is
sleeping. The nurse also notes some confusion during confused, there is no other data presented that could be the cause, making disturbed sleep sleeping, and the correct answer is one that focuses on this problem.
waking hours. Which of the following would be the pattern a more appropriate selection than disturbed sensory perception. In addition, disturbed
most appropriate nursing diagnosis for this client? sensory perception relates to one of the five senses. Ineffective health maintenance and
ineffective individual coping are more global and not applicable to this client's situation.

1.‐ Ineffective health maintenance


2.‐ Ineffective individual coping
3.‐ Disturbed sensory perception
4.‐ Disturbed sleep pattern

3790 An 80‐year‐old client has been admitted to the Correct answer: 2 Weight‐bearing exercise is the best approach to preventing disuse syndrome. Disuse The core issues of the question are the cause of disuse syndrome and nursing approaches
nursing unit with Parkinson’s disease. Which of the syndrome occurs because the stresses of weight bearing are absent and the bone releases that will minimize it. Use concepts of basic nursing care to answer the question.
following activities would be most appropriate in calcium. The other options list general nursing interventions that are not specific to weight‐
preventing disuse syndrome? bearing.
1.‐ Providing for the nutritional needs of the client
2.‐ Promoting weight‐bearing exercises
3.‐ Encouraging 8 glasses of fluid in 24 hours
4.‐ Turning and positioning every 2 hours

3791 A client has a pressure ulcer on the left hip. The Correct answer: 4 Options 1 and 2 indicate nursing activities aimed at promoting healing. Option 3 refers to The core issue of the question is the type of documentation that best described progress
nursing staff has written a nursing diagnosis of other areas of the body. Option 4 refers to the wound itself and is the best indication of the toward wound healing. Questions such as these seek an answer that reflects an outcome
Impaired skin integrity with a client goal of "skin heals wound’s current status. rather than activities.
by 6/12." Prior to June 12, the nurse evaluates
progress on reaching this goal. Which statement is the
best notation of progress toward the goal?

1.‐ Turning every 2 hours and avoiding lying on left side


2.‐ Wet to moist dressing changed every 4 hours
3.‐ No additional areas of breakdown noted
4.‐ Ulcer less reddened and granulation tissue noted on edges

3792 In assessing a client who has been immobilized Correct answer: 2 After immobilization, unexercised muscles will atrophy. The muscles would not be flexible or This question requires application of basic nursing knowledge to a client situation. Select
because of illness, the nurse would most likely hardened. Hypertrophy is the opposite of atrophy. the term that is a common finding in clients who are immobilized.
document the client’s muscles as which of the
following?
1.‐ Hypertrophied
2.‐ Atrophied
3.‐ Flexible
4.‐ Hardened

3793 A female client can move her right arm and leg but Correct answer: 2 Passive range of motion is most appropriate because the client is unable to move that side of The critical words in the question are unable to move. This indicates that the client has
has hemiplegia on the left. The nurse instructs the the body on her own. The other exercises require resistance on the part of the muscles on the hemiplegia and is unable to actively participate in exercising the joints. The wording of the
nursing assistant to do which of the following exercises left side, and the client is unable to do that. question tells you that there is only one correct answer.
on the client’s left side during care?

1.‐ Active range of motion


2.‐ Passive range of motion
3.‐ Isotonic
4.‐ Isometric

3794 A client has weakness of the lower extremities and Correct answer: 3 The weight of the body should be borne on the arms, not the axillae. When clients allow the The core issue of the question is proper use of crutches. Keep in mind that improper use
uses crutches for mobility. The nurse concludes the axillae to bear the weight of the body, they are at risk of developing crutch palsy, a nerve can lead to nerve injury, and select the option that puts the client at risk.
client needs further information about using crutches damage. The other options represent correct information about use of crutches, and therefore
when the client does which of the following? no further information is needed on those points.

1.‐ Uses the swing‐to gait


2.‐ Uses axillary crutches
3.‐ Bears weight on the armpits
4.‐ Has new rubber tips on the crutches

3795 A male client suffered numerous types of wounds Correct answer: 1 Primary intention healing occurs when the wound edges are well approximated; wounds that The core issue of this question is knowledge of physiological wound healing. Use nursing
when he lost control of his motorcycle and was thrown heal by secondary intention have edges that cannot be approximated. Scarring is greater for knowledge and the process of elimination to make a selection.
onto the pavement. The client asks the nurse which wounds that heal by secondary intention and those that become infected. The location of a
wounds will scar more. The nurse's reply will be based wound has little to do with scarring.
on analysis that which of the following wounds would
generally be least likely to scar?

1.‐ A wound that heals by primary intention


2.‐ A wound that heals by secondary intention
3.‐ A wound that becomes infected
4.‐ A wound to an extremity

3796 A postoperative client tells the nurse that he Correct answer: 2 Activities that are likely to lead to dehiscence include vomiting and coughing because they The core issue of the question is knowledge of risk factors for dehiscence. Recall that
developed dehiscence after his last surgery and wants increase intraabdominal pressure. In addition, clients who are obese and those with poor dehiscence is most likely to occur when there is some type of stress on the incision line.
to make sure it doesn't happen this time. In attempting nutrition are candidates for dehiscence. Since the client is already postoperative, encouraging Consider that vomiting puts sudden tension on the suture line to select it as the option that
to prevent dehiscence from occurring, the nurse's weight loss at this time would not affect risk for dehiscence. is most likely to be harmful to the client.
interventions would be aimed at doing which of the
following?
1.‐ Helping the client lose weight
2.‐ Preventing vomiting
3.‐ Administering antibiotics
4.‐ Keeping the wound dry

3797 An elderly postoperative client has an abdominal Correct answer: 1 Vascular changes, such as atherosclerosis and atrophy of capillaries, impair blood flow to the The core issue of the question is age‐related changes that have a negative impact on
wound. Weeks after the surgery, the wound is still wound. The other statements are false. Older adults are not necessarily overweight, although wound healing. Use nursing knowledge and the process of elimination to make a selection.
healing. The client asks the clinic nurse why the wound weight gain does tend to occur with increasing age. Decreased activity levels with aging does
is healing so slowly. The nurse would explain that not diminish local blood supply to a healing wound. Keloid formation is an abnormal type of
which of the following is one factor that negatively healing of a wound.
affects healing in the elderly?

1.‐ Vascular changes decrease the blood flow to the wound.


2.‐ Most elderly clients are overweight.
3.‐ Decreased activity levels prevent blood from reaching the area.
4.‐ Keloid formation prevents healing.

3798 A young adult is admitted to the hospital for Correct answer: 3 Protein and vitamin C are necessary for building and maintaining tissues. A deficiency of The core issue of this question is the role of vitamin C in wound healing. Use nursing
gallbladder surgery. The client is also diagnosed as vitamin C would prolong wound healing. The other options have nothing to do with vitamin C. knowledge and the process of elimination to make a selection.
having a vitamin C deficiency. The nurse places high
priority on assessing this client for which development
postoperatively?
1.‐ Unusual muscle weakness
2.‐ Mental confusion
3.‐ Delayed wound healing
4.‐ Ataxia upon ambulating

3799 The nurse is assisting a client with a mobility problem Correct answer: 2 The client has bilateral weakness of the lower extremities, and the proper assistive device is The core issue of the question is the assistive device that will provide the safest support to
in determining the appropriate assistance device. The one that will provide bilateral support. In this case, a walker provides the most support. the client. The critical word legs in the stem of the question guides you to look for an option
client has no paralysis of the lower extremities, but the Additionally, a four‐wheeled walker does not require the client to lift the walker as steps are that provides bilateral support.
legs are very weak. Upper‐body strength is also taken.
reduced. The nurse would suggest which device for this
client?
1.‐ Cane
2.‐ Four‐wheeled walker
3.‐ Canadian or elbow extension crutch
4.‐ Lofstrand crutch

3800 The nurse is evaluating a client using a cane. Which Correct answer: 1 To provide maximum support and appropriate body alignment while walking, the cane is held The core issue of the question is the proper use of a cane as an assistive aid. Use the
assessment made by the nurse would indicate that the in the hand on the stronger side. The tip of the cane should have rubber to prevent slipping. process of elimination and basic nursing knowledge to make a selection.
client is using the cane appropriately?

1.‐ Client holds the cane with the hand on the stronger side.
2.‐ Client holds the cane with the hand on the affected side.
3.‐ Client moves the cane and the affected leg together.
4.‐ The cane tip is made of aluminum to prevent slippage.

3801 A client is at risk for the developing a pressure ulcer Correct answer: 4 Unless the skin loss is extensive, the skin will continue to absorb vitamin D and prevent the The core issue of the question is knowledge of the functions of the skin and how pressure
and is placed on a repositioning regimen. The client loss of heat from the body. Tactile stimulation can still occur with a wound. However, a loss of ulcers interfere with normal skin function. Use the process of elimination and basic nursing
does not like to lie on his side and complains about the skin integrity places the client at risk for bacterial invasion and subsequent infection. knowledge to make a selection.
need to turn. To enhance compliance, the nurse uses
which of the following most effective explanations?

1.‐ "Turning will help you to maintain normal sensation in all skin areas under pressure."
2.‐ "Excess pressure interferes with skin absorption of vitamin D."
3.‐ "Staying in one position prevents proper heat loss from that area of the body."
4.‐ "Changing position prevents tissue breakdown that could ultimately become infected."

3802 The nurse is changing the dressing of a client who is 4 Correct answer: 4 Evisceration occurs when internal viscera protrude from an incision that is dehiscing. In this The core issue of this question is the ability to draw accurate conclusions about the status
days postoperative with an abdominal wound. The situation, the nurse notes changes in wound appearance such as increased serosanguineous of a surgical wound. Use the process of elimination and basic nursing knowledge to make a
nurse has changed this dressing daily since surgery. drainage, edges lacking approximation, and the protruding viscera. selection.
Today, the nurse notes increased serosanguineous
drainage, wound edges not approximated, and puffy
tissue protruding through the wound. The nurse
concludes which of the following conditions exists?

1.‐ Hemorrhage
2.‐ Normal healing by primary intention
3.‐ Normal healing by secondary intention
4.‐ Evisceration

3803 The nurse needs to conduct an admission interview Correct answer: 1, 4, 5 The nurse should position herself or himself within the client's line of vision to enable the The core issue of the question is effective communication strategies with a client whose
with a 74‐year‐old client who is hearing impaired. client to read lips during the conversation. It is good to decrease background noises that hearing is impaired. Remember that correct answers to questions such as these focus on
Which of the following should the nurse do to enhance interfere with the client’s ability to hear the nurse. It is also helpful to speak at a moderate enhancing the client's vision during communication and are moderate in overall approach
the client's ability to hear? Select all that apply. rate and use the same voice tone throughout each sentence, not dropping the tone at the end (i.e., not excessive or insufficient).
of a sentence. The lighting should not be dimmed because doing so would interfere with the
client’s ability to read lips. Words should not be overarticulated; exaggerated, unnatural
movement of the lips can distort words for the client who relies on lip reading to compensate
for hearing loss.

1.‐ Position self to be within the client's line of vision.


2.‐ Dim the lights in the room.
3.‐ Overarticulate words.
4.‐ Turn down the television in the room.
5.‐ Talk at a moderate rate and with the same tone for all words.

3804 The school health nurse is interested in promoting Correct answer: 2 Option 1 is a factor related to the adult; option 3 is related to school‐age children, and option Select the option that identifies a characteristic common in adolescents, in this case,
safety in the school population. In planning safety 4 is related to the elderly. feelings of immortality.
education for this age group and parents, the nurse
would recognize that which of the following is a
developmental risk factor for adolescents?

1.‐ Substance abuse as a means of dealing with stress.


2.‐ Feelings of immortality related to the perception of being invulnerable to risks that affect others.
3.‐ Sports‐related injuries that are usually related to not obeying rules and/or intense competition.
4.‐ Polypharmacy, which results in mixing of multiple medications.

3805 The nurse is caring for a young child who has mitt Correct answer: 1 It is important that circulation is checked regularly. Typically the restraints are removed, one Look for a response that addresses a basic rule of restraint use, in this case, checking skin
restraints. Which of the following priority actions at a time, every 2 hours to evaluate skin condition and circulation. Although options 3 and 4 and circulation.
needs to be done regularly to ensure that the child's are correct, they are not the best response as they do not have to be checked as regularly as
needs are met? the circulation and skin condition. Option 2 applies to an elbow restraint.

1.‐ Check adequacy of circulation and skin condition


2.‐ Check that the tongue blades in pockets are intact and ends covered or padded
3.‐ Ensure that the straps are tied to non‐movable parts of the crib
4.‐ Check that the call bell is pinned to the child's gown
3806 The nurse assesses the respiratory functioning of an Correct answer: 2 Options 1, 3, and 4 are all age‐related changes and do not suggest alterations. Orthopnea or The word difficulty in option 2 should lead you to the determination that this is not an age‐
elderly client. Which of the following symptoms would difficulty breathing when not in a sitting or upright position is suggestive of airway obstruction related change, but an abnormal symptom.
be considered unrelated to the client's age? or respiratory or cardiac disease.

1.‐ Increased rate and depth of respirations while walking quickly


2.‐ Difficulty breathing when lying in bed
3.‐ Coughing that is not as effective in clearing passages
4.‐ Frequent upper respiratory infections

3807 A middle‐aged client complains to the nurse that Correct answer: 2 Middle‐aged adults have a decrease in deep sleep, stage IV NREM. Option 1 is an expected Select the option that describes the sleep change most closely matched with the middle‐
sleep patterns are different than when the client was pattern in older adults; option 3 is expected in young adults, and option 4 is expected in aged group, gradual reduction in quality sleep.
younger. Which of the following suggests a normal neonates.
developmental pattern?
1.‐ Client sleeps about 6 hours per night with about 20–25 percent of REM sleep and a marked decrease in Stage IV NREM sleep.
2.‐ Client sleeps 6–8 hours per night with about 20 percent REM sleep and the amount of Stage IV NREM sleep. decreases.
3.‐ Client sleeps erratically because of work schedules with about 30 percent of REM sleep and no marked decrease in Stage IV NREM sleep.
4.‐ Client has light sleep with equal amounts of REM sleep and NREM sleep.

3808 A client's hemoglobin level is 14 g/dL. Which of the Correct answer: 2 The laboratory value given is within normal limits (12–16.5 g/dL). All the other statements are Select the only option that lists the value as normal. The others indicate some level of
following interpretations of the laboratory value is inaccurate. abnormality.
most accurate?
1.‐ Client has a low value and is malnourished
2.‐ Client has a normal laboratory value and has no nutritional risk
3.‐ Client has a low to normal value indicative of a nutritional risk
4.‐ Client has an elevated value indicative of polycythemia

3809 The client is undergoing surgery for the formation of Correct answer: 3 The ileum, which is a part of the small bowel, is used to form a pouch where the ureters are Select the option that uses the words small bowel and urine as the correct option.
an ileal conduit. Which of the following statements by implanted. Option 1 is incorrect because stool does not drain. Option 2 refers to a cutaneous
the client best indicates an understanding of the ureterostomy. Option 4 is incorrect because the ileum is not part of the big intestines.
surgery?
1.‐ "The hole in my abdomen will drain stool and urine."
2.‐ "There will be two small openings that drain urine out."
3.‐ "Part of my small bowel will be used as a pouch where the urine can drain out."
4.‐ "The urine will drain into a pouch made up by attaching the ureters to a part of my big bowel."

3810 The nurse is working with a client suffering from Correct answer: 3 Anxiety or anger increases peristalsis leading to subsequent diarrhea. All the other options Eliminate the options that are known causes of constipation: 1, 2, and 4.
diarrhea. In teaching ways to reduce diarrhea, the lead to the development of constipation.
nurse would encourage the client to avoid which of the
following that contribute most to the development of
diarrhea?
1.‐ Excessive intake of cheese and eggs
2.‐ Habitually ignoring the urge to defecate
3.‐ Anxiety or anger
4.‐ Lack of exercise

3811 A client is scheduled to have a divided colostomy. Correct answer: 1 In a divided colostomy, the opening from the digestive or proximal end produces fecal wastes Eliminate options 3 and 4 as being incorrect. Select the option from the remaining pair
While doing client teaching, the nurse will explain that while the other stoma, often called the mucous fistula, produces only mucus. that makes sense from a physiological stand point.
after healing, the client can expect to have stool drain
from which of the following?

1.‐ The proximal stoma


2.‐ The distal stoma
3.‐ Both stomas
4.‐ Neither stoma, as this is a urinary diversion

3812 Parents of a group of toddlers are participating in a Correct answer: 1 A toddler is mobile and naturally curious and experiments with things in the environment; Select the option that would relate to accidents most often encountered with toddlers,
safety education class to prevent accidents. Which of therefore, the parents need to know that supervision will be necessary. Toddlers' reflexes are which are usually due to their ability to move about and curiosity.
the following aspects need to be included in the health not necessarily slow, and reading is not a concern. Social and personality development is a
teaching session? good topic for health teaching but is not the main concern in regards to safety.

1.‐ The child's physical capacities and curiosity


2.‐ The child's slow reflexes
3.‐ The child's difficulty in reading
4.‐ The child's social and personality development

3813 A nurse on the unit observes that the night shift nurse Correct answer: 2 One of the purposes of restraints should be to prevent interruption of therapy such as the use Look for the circumstance in which lack of restraints would affect recovery or care.
has placed restraints on all of the following clients. In of dressings. Restraints should not be used for the convenience of the staff (option 1), nor
which of the following situations would the use of should they be used because a client is weak or distraught (option 3). The client in option 4 has
restraints be appropriate? no need for restraints.
1.‐ A hyperactive child
2.‐ A child scratching the incision site postoperatively
3.‐ A postoperative client who is alert but still weak
4.‐ A client who is severely anxious about test results

3814 An elderly client on bed rest for a few days has been Correct answer: 2 Because the client has been incontinent, the possibility of skin bacteria reacting with the urea The use of the word ammonia in option 1 is a clue to the correct answer as urine has a
incontinent. The client is now complaining of pruritus in the urine can lead to ammonia dermatitis. Erythema (option 1) is reddening of the skin; high ammonia content.
and excessively dry skin, particularly in the area of the contact dermatitis (option 3) is a possibility if a client is allergic to soaps or other substances;
lower back. To which of the following skin problems is and petechiae (option 4) are tiny pinpoints of bleeding in the skin.
this client most susceptible?

1.‐ Erythema
2.‐ Ammonia dermatitis
3.‐ Contact dermatitis
4.‐ Petechiae

3815 A client has been instructed on the use of an Correct answer: 3 The proper sequence for using a spirometer is to exhale completely, place the mouthpiece, Because the incentive spirometer is used to increase respiratory capacity select the option
incentive spirometer. The nurse evaluates that the inhale, remove the mouthpiece, hold breath, and exhale. A Fowler's or sitting position best that in which inhalation and the measurement indicator are paired.
client understood the instructions if the client allows full chest expansion. Slower breaths are better and deeper than fast ones. Client should
performs which of the following actions? remove the mouthpiece, exhale through pursed lips, and not exhale into the spirometer.

1.‐ Maintains a supine position while using the spirometer


2.‐ Inhales rapidly, exhales into the spirometer to reach the indicator, and waits 10 seconds before repeating the process
3.‐ Exhales completely, places mouth around mouthpiece before inhaling slowly to reach the indicator, removes mouthpiece, holds breath, and exhales slowly
4.‐ Purses lips tightly around mouthpiece, inhales slowly and deeply, and exhales slowly into the spirometer until spirometer reaches indicator mark

3816 A client complains of tickling in the throat and a Correct answer: 4 The client did not vomit blood (i.e., hematemesis), has no manifestations of being able to Narrow down the options by considering the two with heme or blood as part of the
bubbling sensation in the chest, and then coughs up breathe only when in a sitting position (i.e., orthopnea), and has no indrawing chest movement response. Further eliminate option 1 because hematemesis means vomiting blood.
bright red, frothy blood mixed with sputum. The nurse between the ribs (i.e., intercostal retraction). The client has the symptoms associated with
documents that the client is experiencing which of the hemoptysis: bubbling sensation in the chest, tickling in the throat, and coughing up blood with
following? the sputum.
1.‐ Hematemesis
2.‐ Orthopnea
3.‐ Intercostal retraction
4.‐ Hemoptysis
3817 A client is being discharged with oxygen therapy via a Correct answer: 3 Electrical equipment in good condition (with no frayed wires) is acceptable for use in the Eliminate options referring to the use of petroleum or oil productions which are
cannula. Which of the following instructions should the vicinity of oxygen. Petroleum products and most oils have the potential for being flammable contraindicated in use with oxygen.
nurse give to the client and family? when used on the body, which is a contraindication for their use. Cotton clothing limits static.

1.‐ Use battery‐operated equipment instead of electrical equipment.


2.‐ Use petroleum jelly for the nares to prevent chafing.
3.‐ Use cotton clothing to avoid static electricity.
4.‐ Use oil to protect the facial skin.

3818 A client is complaining of inability to sleep through Correct answer: 1 Pain can often interfere with sleep. Options 2, 3, and 4 do not negatively affect or interfere Eliminate choices that are known to enhance sleep like options 2, 3, and 4. Pain can
the night since admission three days ago. Which of the with sleep. Absence of unfamiliar stimuli (option 2) can assist with sleep; dealing with stress by interfere with ability to sleep.
following factors is most likely to negatively affect the talking about the day's events (option 3) promotes relaxation and eventually sleep; moderate
client's sleep patterns? fatigue (option 4) may lead to a restful sleep.
1.‐ Presence of pain
2.‐ Absence of unfamiliar stimuli
3.‐ Ability to talk about day's events
4.‐ Moderate fatigue

3819 An elderly bedridden client complains of being Correct answer: 4 To promote bowel function, instruct clients to drink plenty of liquids, including fruit juices Eliminate options 1, 2, and 3 as being contrary to promotion of normal bowel activity.
constipated but can not understand why. What such as apple and prune. In addition, foods that are high in fiber and roughage should be
instruction should be given by the nurse? encouraged to avoid constipation secondary to immobility.
1.‐ Decrease fluid intake
2.‐ Encourage bland and low‐residue foods
3.‐ Avoid beverages with caffeine
4.‐ Drink hot liquids and fruit juices

3820 A client complains of occasional urinary incontinence. Correct answer: 2 When a premature urge to void occurs, focused breathing exercises may assist the client to Notice that option 2 addresses the issue of training the bladder, whereas the others
The nurse has worked with the client with regard to overcome the sense of urgency. The intervals between voiding should eventually lengthen, address incontinence.
bladder training. The nurse would evaluate the client rather than voiding every hour or more often when an urge is felt. Protector pads should be
has achieved the expected outcome when the client is worn continuously for leakage. Adult diapers are used only as a last resort.
able to which of the following?

1.‐ Void every time there is an urge


2.‐ Practice deep, slow breathing until the urge to void diminishes
3.‐ Use adult diapers continuously
4.‐ Use protector pads only when going out

3821 A client is taking a full fluid diet following gastric Correct answer: 2 A full liquid diet allows such items as puddings, creamed soups, sherbet, strained cereals, and Omit options 3 and 4 as incorrect as they are not liquid. Omit option 1 because fruit is a
surgery. The nurse evaluates the health teaching to be all items that are liquid at room temperature. Options 1, 3, and 4 would not be appropriate. solid at room temperature and eating the pureed form is not allowed on a full fluid diet.
successful when the family brings in which of the
following for the client to eat?
1.‐ Pureed fruits
2.‐ Custard
3.‐ Soft cake
4.‐ Chopped vegetables
3822 The family member of an elderly client objects that Correct answer: 3 The client needs to be supervised and monitored and placed in a room that is more Choose the option that provides for client safety while best protecting the rights and
restraints are being used to prevent the client from accessible. Assessment is needed to determine causes of wandering. Stimulation is not safety of others.
wandering to other clients' rooms, especially in the necessary for a client who is a wanderer. Anti‐anxiety medications may cause more agitation,
evening. Which of the following alternatives to the use and locking other clients' rooms will not prevent the client from wandering.
of restraints should the nurse consider?

1.‐ Provide visual and auditory stimuli.


2.‐ Use anti‐anxiety medications as prescribed.
3.‐ Assign client to a room near the nurse's station.
4.‐ Lock the other clients' rooms.

3823 A client is hospitalized for the first time. Which of the Correct answer: 1 The environment must be clutter‐free; therefore, unnecessary pieces of equipment or Omit options 2 and 4 as unrealistic. Option 3 is a threat to safety.
following actions ensures the safety of the client? furniture should be out of the way. Lights on and side rails up are not mandatory at all times. It
is unnecessary to keep equipment out of view.
1.‐ Keep unnecessary furniture out of the way.
2.‐ Keep lights on all the time.
3.‐ Keep side rails up at all times.
4.‐ Keep all equipment out of view.

3824 A client who is unconscious needs frequent mouth Correct answer: 2 In the side‐lying position, fluid is more likely to flow readily out of the mouth or pool in the Omit the options which would put the client at risk for aspiration by having the client
care. While performing mouth care, in what position side of the mouth where it can easily be suctioned. Fowler's position and Trendelenburg positioned on the back or sitting.
should the nurse place the client? positions are not appropriate since the unconscious client does not have the control to stay up
in those positions. The supine position is unsafe as the client may aspirate the fluids.

1.‐ Fowler's position


2.‐ Side‐lying position
3.‐ Supine position
4.‐ Trendelenburg position

3825 The nurse is providing health teaching to the client Correct answer: 3 Stress and long‐term alcohol use increase the blood pressure. Physical exercise increases Options 3 and 4 speak specifically about tissue oxygenation, but only option 3 gives correct
about lifestyle factors that affect oxygenation. Which respirations and cardiac rate, increasing the supply of oxygen to the body. Nicotine increases information.
of the following explanations is most accurate? blood pressure and vasoconstriction which prevents oxygen from reaching the different parts
of the body.
1.‐ Epinephrine and norepinephrine released under stress decrease blood pressure and cardiac rate.
2.‐ Long‐term use of alcohol stabilizes blood pressure and cardiac functioning.
3.‐ Nicotine increases heart rate, blood pressure, and peripheral resistance, and produces vasoconstriction that decreases oxygenation to tissues.
4.‐ Physical exercise decreases the depth of respirations and cardiac rate and eventually lowers the need for oxygen by the tissues.

3826 A nurse is performing oropharyngeal suctioning on an Correct answer: 3 Gentle rotation ensures that all surfaces are reached and prevents trauma to any one area Look for time periods and measurements that do not make sense, as in options 1, 2, and 4.
unconscious client. Which of the following actions is caused by prolonged suctioning. In oropharyngeal suctioning, the catheter should be advanced
safe? to 10–15 cm; 20 cm is the distance for tracheal suctioning (option 1). Fifteen minutes of
suctioning (option 2) and applying suction while inserting the catheter (options 1 and 4) can
cause trauma to the mucous membranes.
1.‐ Insert the catheter approximately 20 cm while applying suction.
2.‐ Allow 20–30‐second intervals between each suction, and limit suctioning to a total of 15 minutes.
3.‐ Gently rotate the catheter while applying suction.
4.‐ Apply suction for 5 seconds while inserting the catheter and continue for another 5 seconds before withdrawing.

3827 A client with chest tubes is admitted to the nursing Correct answer: 4 Although all of the actions are appropriate, the highest priority on admission is to anticipate The question asks for prioritization. Select the option that provides for emergency
unit. The nurse should place the highest priority during any emergency that may occur if problems with the chest tubes occur, such as disconnection equipment to be made available.
admission on doing which of the following? or accidental removal.
1.‐ Plan to measure client's vital signs, respiratory and cardiovascular status regularly
2.‐ Explain the importance of deep‐breathing and coughing regularly
3.‐ Report if drainage exceeds 100 mL/h
4.‐ Place rubber‐tipped clamps, sterile water, and a sterile occlusive dressing materials near the client

3828 While doing a physical assessment on a client, the Correct answer: 1 Soft, flaccid muscles are signs of inadequate nutritional status. Muscles should be firm and Eliminate options 1, 2, and 3 as these findings are associated with overall good health and
nurse suspects that the client has poor nutritional well developed. All of the other options are signs of adequate nutritional status. nutrition.
status. Which of the following would confirm the
nurse's observations?
1.‐ Flaccid, soft muscles
2.‐ Firm, smooth pink nails
3.‐ Moist buccal cavity mucous membranes
4.‐ Erect posture

3829 The nurse evaluates the results of laboratory tests Correct answer: 4 Options 1 to 3 are all normal levels; option 4 is indicative of potassium depletion that occurs Identify the only abnormal laboratory value.
completed on a client. Which of the following values in severe cases of malnutrition.
indicate an abnormality related to nutritional status?

1.‐ Blood urea nitrogen (BUN) 15 mg/dL


2.‐ Urinary creatinine 800 mg/24 h in an adult female
3.‐ Albumin 5 g/dL
4.‐ Serum potassium 2.0 mEq/L

3830 To avoid complications associated with urinary Correct answer: 1 Symptoms and ways of preventing an infection are crucial for a client to understand. The question asks for identification of an outcome and option 1 reflects an outcome which
elimination, the nurse teaches the client to perform Performance of perineal care independently and disposal of urinary output are not appropriate addresses prevention of complication.
certain actions. Which of the following indicates that outcomes. Tub baths are to be avoided, especially in females, as they may increase the
the expected outcome is achieved? possibility of developing lower tract infections.

1.‐ Identifies symptoms of and measures to prevent urinary tract infection


2.‐ Able to perform perineal care by self
3.‐ Maintains proper disposal of urinary output
4.‐ Takes regular tub baths and appropriate personal hygiene measures

3831 A client with a colostomy is asking the nurse about Correct answer: 2 The foods that thicken stools are in option 2. Option 1 foods increase stool odor; option 3 and Select the option that contains food that is used to increase the thickness of stool such as
the types of foods that may loosen stool and cause 4 foods loosen stools. pectin in bananas and apples.
leakage into the pouch. Which of the following foods
should the client be told to include in the diet to
prevent this problem?
1.‐ Asparagus, beans, eggs, fish, onions
2.‐ Applesauce, bananas, rice, tapioca, yogurt
3.‐ Fried foods, spicy foods, raw fruits and vegetables
4.‐ Carbonated drinks, fruit juices, greasy and pureed foods

3832 An adolescent client is competing in a long‐distance Correct answer: 2 The diet before competition should be high in complex carbohydrates and low in fat and The core issue of the question is knowledge that complex carbohydrates are beneficial
running event. The nurse is teaching the client about protein. Option 2 reflects the best selection to meet this dietary balance. All of the other before lengthy exercise. Use nutrition knowledge and the process of elimination to make a
eating for competition. The nurse explains that which options are high in fat and/or protein and would not be beneficial in terms of supporting selection.
of the following is an appropriate meal before the athletic performance.
competition?
1.‐ Sausages, eggs, biscuits, and gravy
2.‐ Pancakes with fresh strawberries, orange juice, wheat toast, and fresh melon slices
3.‐ Yogurt, milk fortified with dry skim milk powder, and a protein bar
4.‐ Cheese omelet, hash brown potatoes, bacon, and coffee
3833 The nurse has completed a comprehensive health Correct answer: 2 One characteristic of Hispanic diets is the high‐fat preparation method used in cooking. The core issue of the question is knowledge of methods of reducing fat in the diet. Use
assessment of a Hispanic client. Some cultural food Suggesting a new preparation method for a familiar food item would best help the client to nutrition knowledge and the process of elimination to make a selection.
practices place the client at risk for cardiovascular begin changing cooking habits. Option 1 is incorrect: Complex carbohydrates should not be
disease. Which suggestion by the nurse is appropriate eliminated because they are components of a healthy diet. Option 3 is incorrect: The Hispanic
for this client? client would probably be unwilling to relinquish beans and nuts in the diet because these are
considered staple food products. Option 4 is incorrect because stewing is considered a high‐fat
method of cooking because the fat from the meat does not drain off.

1.‐ "Try to stop eating so many complex carbohydrates."


2.‐ "Try to bake some foods instead of frying them."
3.‐ "Lean meats should replace the beans and nuts in your diet."
4.‐ "Do not stop stewing meat and vegetables together; it is a healthy cooking method."

3834 Which of the following dietary recommendations Correct answer: 4 Egg yolks are high in cholesterol and should be limited to two or three per week. Dietary The core issue of the question is knowledge of the components of a low‐fat diet and that
should the nurse include in the discharge instructions fiber, fish, and soybean products have been shown to lower blood lipids. Dietary fiber is this is the diet required by clients with heart disease. Use nutrition knowledge and the
of a client diagnosed with coronary artery disease? necessary in the body to promote regulation of elimination patterns and to help lower blood process of elimination to make a selection.
lipids. Soybean products are a source of phytoestrogens and have been shown to be
cardioprotective. Tuna is an excellent source of omega‐3 fatty acids, which are helpful in
protecting cardiac function and decreasing clot formation.

1.‐ Limit intake of whole grains.


2.‐ Limit intake of tuna.
3.‐ Limit intake of soybean products.
4.‐ Limit intake of egg yolks.

3835 The nurse is planning discharge teaching for the client Correct answer: 2 Foods that decrease lower esophageal sphincter (LES) pressure should be avoided to reduce The core issue of the question is knowledge of foods that lower LES pressure and increase
with gastroesophageal reflux disease (GERD). What reflux symptoms; these include caffeine, alcohol, and chocolate. Clients should also avoid risk of reflux in GERD. Use nutrition knowledge and the process of elimination to make a
dietary modification should be included? eating large meals, drinking fluids with meals, and eating at bedtime; they should remain selection.
upright for 1–2 hours after eating.
1.‐ Eat three meals and a bedtime snack.
2.‐ Avoid intake of caffeine and alcoholic beverages.
3.‐ Drink 12–16 ounces of water with each meal.
4.‐ Lie down for 15–20 minutes after eating.

3836 The nurse interprets that which of the following client Correct answer: 3 A 2‐gram sodium‐restricted diet requires use of no salt in cooking, no salt added at the table, The core issue of the question is knowledge of the various degrees of sodium restriction in
behaviors reflects compliance with a 2‐gram sodium‐ avoiding high‐sodium foods, and limiting milk to two cups per day. Option 1 is incorrect—no the diet. Use nutrition knowledge and the process of elimination to make a selection.
restricted diet? salt can be added in this restricted diet. Options 2 and 4 are incorrect. One cup of milk per day
and the use of salt‐free butter are requirements of a 1‐gram sodium restricted diet.

1.‐ Using only the two packets of salt found on the meal tray
2.‐ Limiting milk to one cup per day
3.‐ Avoiding use of salt in cooking
4.‐ Using salt‐free butter with meals

3837 The nurse determines that a hypertensive client Correct answer: 2 The DASH diet increases daily servings of vegetables and fruits, and recommends low‐fat dairy The core issue of the question is knowledge that hypertensive clients should lose weight if
understands the DASH diet (Dietary Approaches to foods and reduced intake of saturated fats and cholesterol. Option 1 reflects increased fats; necessary and restrict dietary intake of sodium. Use nutrition knowledge and the process of
Stop Hypertension) when the client chooses which option 3 represents increased fat, cholesterol, and sugar; option 4 reflects increased sodium elimination to make a selection.
items from a sample menu used in dietary teaching? content.

1.‐ Caesar salad, bread sticks, frozen yogurt


2.‐ Grilled chicken sandwich, strawberries, and lettuce salad
3.‐ Grilled cheese sandwich, canned pineapple, brownie
4.‐ Chicken and vegetable stir‐fry, rice, egg roll

3838 A client who is recovering from partial‐ and full‐ Correct answer: 1 The client with burns needs increased amounts of protein and vitamins C and D until the The core issue of the question is knowledge that increased protein and vitamins are
thickness burns has been advanced to a general diet. wounds are completely healed. Option 1 reflects high‐protein sources, an antioxidant source, needed for wound healing. Use nutrition knowledge about food sources of protein and
Which foods should the nurse encourage the client to and fortified milk that includes vitamin D and calcium. The other options do not reflect the vitamins and the process of elimination to make a selection.
eat most often? necessary protein, vitamins, and mineral sources needed for the care of clients recovering
from burns.
1.‐ Meats, citrus fruits, milk
2.‐ Vegetables, cheese, pasta
3.‐ Milkshakes, salads, soups
4.‐ Breads, cereals, yogurt

3839 Which of the following client comments indicates to Correct answer: 3 Simple sugars and carbohydrates, including honey and jelly, increase the osmolality of the The stem of the question has a negative wording, which tells you that the correct answer
the nurse that more teaching is needed for the client gastric contents and enhance movement of food out of the stomach. Therefore, these should is an incorrect statement. Recall that sugars and concentrated carbohydrates should be
experiencing dumping syndrome after gastric surgery? be avoided by the client at risk for dumping syndrome. Six small meals per day, not taking avoided to help choose option 3 as the statement that is incorrect.
fluids with meals, and lying down for 30–60 minutes after meals will help reduce the risk of
dumping syndrome.
1.‐ "I should eat six small meals per day."
2.‐ "I should not drink fluids with my meals."
3.‐ "I should use honey or jelly instead of butter."
4.‐ "I should lie down for 30–60 minutes after eating."

3840 Which of the following dietary teaching statements Correct answer: 3 Increased dietary protein can lead to increased uric acid formation, which in turn lowers The core issue of the question is knowledge that protein sources can lead to uric acid
would the nurse make to a client who has renal urinary pH and causes precipitation of uric acid stones. Clients should not exceed protein formation and subsequent stone formation. Use nutrition knowledge and the process of
calculi? intake of 100 grams per day and should monitor purine content of foods. Option 1 is incorrect elimination to make a selection.
because factors other than dietary intake can cause stone formation, specifically alterations in
urinary pH and the presence of metabolic disease. Option 2 is incorrect because there is no
clinical evidence to suggest that decreasing calcium intake will prevent the formation of renal
calculi; rather, research is showing that a high‐calcium diet offers protection against stone
formation. Even though most renal calculi are composed of calcium oxalate, it is the oxalate
component that appears to cause the formation of stones. Option 4 is incorrect because
increasing intake of complex carbohydrates is recommended to prevent renal calculi
formation.

1.‐ "The presence of renal calculi is directly correlated to dietary intake."


2.‐ "Decreasing calcium intake will prevent the formation of renal calculi."
3.‐ "An increase in dietary protein can increase the likelihood of renal calculi."
4.‐ "Reducing dietary intake of complex carbohydrates decreases formation of renal calculi."

3841 A client with chronic obstructive pulmonary disease Correct answer: 3 Clients with COPD who overeat, in addition to consuming excess carbohydrates, have The core issue of the question is knowledge that excessive carbohydrate intake results in
(COPD), who has been consuming more than 3,000 increasing difficulty with breathing because of excessive CO&lt;sub&gt;2&lt;/sub&gt; levels excess carbon dioxide production, which can be harmful to the client with COPD. Use
calories/day to gain weight, now reports increased that place additional stress on the lungs. The client should eat a proper diet and correct nutrition knowledge and the process of elimination to make a selection.
breathing difficulty. The client states, "I thought that if percentages of macronutrients to maintain adequate weight. In addition, chronic COPD is
I gained weight by eating more, I would feel better." associated with PEM (protein‐energy malnutrition), infection, and unintentional weight loss.
How would the nurse respond to the client's concern? Option 1 is incorrect since increased calories alone can lead to increased work of breathing.
The percentage of fat in the diet may also pose a problem if the client is experiencing
contributory disease or malabsorption. Option 2 is incorrect because merely providing
medication therapy to stimulate weight gain does not address the problem of the obvious
excess of calories that the client is consuming or that the client is experiencing difficulty
breathing. Option 4 is incorrect because an increase in high‐quality proteins will not help to
correct the clinical symptoms.
1.‐ "The increase in calories is not as important as an increase in the fat percentage in the diet."
2.‐ "It is not necessary to increase caloric intake because medication therapy can be given to help with desired weight gain."
3.‐ "An increase in both calories and carbohydrates can lead to increased respiratory effort and clinical symptoms that you are having."
4.‐ "An increase in high‐quality proteins will help correct the respiratory symptoms."

3842 Which snack selection would be most appropriate for Correct answer: 4 A client who has stomatitis will have pain upon ingestion of food caused by the inflammatory The core issue of the question is knowledge that clients with stomatitis need foods that
the nurse to make for a client with cancer who has process. Cool foods are often tolerated better than hot foods, as are soft, creamy products. are soft, nonirritating, and not hot. Use nutrition knowledge and the process of elimination
stomatitis? Option 1 is incorrect because peanut butter is a thick, dense food that may irritate the mouth to make a selection.
by sticking on mucous membranes and requiring more effort to swallow. Option 2 is incorrect
because pretzels are high in salt, which may cause further irritation to the oral cavity. Option 3
is incorrect because tomatoes are high in ascorbic acid; even though they are in the form of a
soup, they may cause irritation.

1.‐ Peanut butter sandwich


2.‐ Soft pretzels with salt
3.‐ Tomato soup
4.‐ Yogurt

3843 A client who is lactose intolerant is recovering from a Correct answer: 4 A client who is lactose intolerant has a difficulty handling milk and dairy products because of The core issue of the question is knowledge of concepts related to lactose intolerance. Use
surgical procedure. What impact does the nurse deficient lactase enzyme. Full liquid diets are based on milk and dairy products. If a client is nutrition knowledge and the process of elimination to make a selection.
expect this to have on progression of diet as tolerated? known to be lactose intolerant, the diet will have to be adjusted to reflect lactose‐reduced or
lactose‐free products in order to prevent GI irritation. Option 1 is incorrect because it does not
reflect the added clinical condition of lactose intolerance but merely refers to progression of
diet. Option 2 is incorrect because lactose intolerance does have an impact on diet patterns.
Option 3 is incorrect because diet progression does not rely merely on the return of a bowel
movement pattern.

1.‐ The client will be able to progress from a clear to full liquid diet easily once bowel sounds and gag reflex return.
2.‐ There is no impact with regard to diet progression because of lactose intolerance.
3.‐ The client's diet can be progressed following a bowel movement indicating return of bowel activity.
4.‐ The client's full liquid diet may have to be altered because this diet contains milk products.

3844 A client is placed on enteral feedings via nasogastric Correct answer: 4 A client who is receiving enteral feedings via nasogastric tube can be at risk for dehydration The core issue of the question is knowledge that a client receiving enteral feedings is at
tube to meet nutritional goals. Which of the following caused by inadequate fluid intake. It is therefore important to irrigate the tube with water as risk for dehydration if there are no sources of free water provided. Use nutrition knowledge
assessments should be included in a plan of care in ordered (before and after feedings or medication administration) and include these irrigations and the process of elimination to make a selection.
order to maintain fluid balance? in the client’s total I&amp;O measurements. Option 1 is incorrect because although inspection
of the skin surrounding the tube is necessary, it does not relate specifically to fluid balance.
Option 2 is incorrect because clients are often weighed daily. Option 3 is incorrect because
feeding tubes are not flushed only once a day.

1.‐ Assess the skin area around the tube site.


2.‐ Weigh the client every other day.
3.‐ Maintain strict I&O and flush the tube once a day to ensure patency.
4.‐ Irrigate the tube with water as ordered and include this fluid in total I&O.
3845 Which of the following points would the nurse make Correct answer: 3 A client with a significant cardiac history on a fat‐restricted diet should not use additional fat Use nutrition knowledge and the process of elimination to make a selection.
when doing dietary teaching with a client placed on a during the cooking or preparation process. Option 1 is incorrect because a client with a
fat‐restricted diet because of significant cardiac significant cardiac history will require some form of fat control or restriction as part of a
history? dietary pattern for the rest of his or her life. Option 2 is incorrect because fat is a necessary
nutrient for the body. To deprive a client of all fat sources can lead to a clinical deficiency of
essential fatty acids that can cause further problems for the client. Option 4 is incorrect
because ice cream is considered a high‐fat product. The client could have low‐fat ice cream,
yogurt, or sherbet to satisfy dietary needs.

1.‐ "This diet will be used temporarily to reduce saturated fat intake and cholesterol levels."
2.‐ "All forms of fat should be restricted because of your significant cardiac history."
3.‐ "No additional fat should be utilized when cooking or preparing foods."
4.‐ "Ice cream can be included in the diet, although fat from butter and meat is excluded."

3846 A client's mother wants to know why she should have Correct answer: 3 Allergy I and II diets are used in sequence to identify and eliminate potential food allergens. The core issue of the question is knowledge of the purposes of allergy I and II diets. Use
her daughter follow an allergy I and II diet. How would Option 1 is incorrect. Even though this diet pattern is used over a short time period, this nutrition knowledge and the process of elimination to make a selection.
the nurse reply to this mother's concern? response does not address why it is necessary to follow the diet pattern. Option 2 is incorrect
because it provides false and inaccurate information. Option 4 is incorrect because referral to
an immunologist for allergy testing is not a required accompaniment to this dietary pattern.
Referral to an immunologist for allergy testing may eventually be indicated if the client is found
to have a multiple allergy profile.

1.‐ "This is a short‐term therapy diet, and it will be over before you know it."
2.‐ "This diet needs to be followed until your daughter grows out of her allergy phase."
3.‐ "This diet sequence helps to both identify and eliminate potential allergens, making future diet selection choices easier."
4.‐ "Allergy testing usually accompanies this type of diet pattern, and you should see an immunologist."

3847 Which of the following items should the nurse Correct answer: 4 Foods that contain gluten (wheat, oats, rye, and barley) are restricted for a client with celiac The core issue of the question is knowledge that a foods containing wheat, rye, oats, and
encourage in the diet of a client diagnosed with celiac disease because of an inability to handle gluten protein. All of the other choices reflect items barley are restricted in celiac disease. Use nutrition knowledge about foods containing
disease? that cannot be used in a gluten‐restricted diet. these grains and the process of elimination to make a selection.
1.‐ Oatmeal
2.‐ Whole wheat toast
3.‐ Beef barley soup
4.‐ Cornflakes

3848 Which of the following foods would the nurse suggest Correct answer: 3 A client taking MAO inhibitors has to avoid foods that are high in tyramine because it can lead The core issue of the question is knowledge that clients taking MAOIs require a low‐
as an item from the lunch menu for a client taking a to significant complications resulting in hypertensive crisis. Cottage cheese is an unfermented tyramine diet. Use nutrition knowledge of low‐tyramine foods and the process of
monoamine oxidase (MAO) inhibitor? cheese that can be used in the diet. All of the other options reflect foods that are high in elimination to make a selection.
tyramine. Aged cheeses are not allowed on the diet.

1.‐ Smoked fish


2.‐ Bologna sandwich
3.‐ Cottage cheese
4.‐ Salad with bleu cheese dressing

3849 How would the nurse best respond to a client's Correct answer: 3 The perception of being in a state of "always dieting" can be problematic in terms of The core issue of the question is identification of a client’s need for assistance with weight
statement: "I am always dieting, but I never seem to compliance and goal attainment because it can be viewed either as a restriction or as a form of control. Use nutrition knowledge and the process of elimination to make a selection.
be losing weight?" punishment. Option 1 is not true: wanting to lose weight is not the only factor to consider;
many other variables affect weight loss. While it is important to find out the type of the diet
the client is on (or has been on), this knowledge doesn’t address the main concern of the client
regarding “always dieting” and yo‐yo effect (weight cycling).

1.‐ "Weight loss is only maintained if you really want to lose the weight."
2.‐ "Dieting is a way of life, and compliance is required to maintain weight loss."
3.‐ "By saying you are always dieting, it sounds like you need some assistance in attaining your weight‐loss goals."
4.‐ "I need to know which type of the diet you are on because it may not be effective."

3850 The nurse would encourage the client wishing to Correct answer: 2 Diets that are rich in fruits and vegetables have been proven to be effective in decreasing the The core issue of the question is knowledge of foods that contain protective chemicals
reduce risk of cancer to maintain adequate intake of risk of developing cancer because these foods contain phytochemicals. None of the other food against cancer development. Use nutrition knowledge and the process of elimination to
which foods? groupings have been shown to decrease the risk of cancer. make a selection.

1.‐ Meat and dairy products


2.‐ Fruits and vegetables
3.‐ Rice and beans
4.‐ Milk and cheese

3851 An athletic client states that he is thinking of using Correct answer: 4 When an athletic client is considering utilizing any ergonomic aid or supplement, trainers The core issue of the question is appropriate anticipatory guidance to a client seeking to
carbohydrate (CHO) loading to increase his and/or nutritional specialists can monitor the client closely to establish a client baseline, use nontraditional methods of nutrition for supplemental use. Use knowledge that this
performance. How should the nurse respond to this provide education, and prevent potential complications related to therapy. Option 1 is client requires special monitoring and the process of elimination to make a selection.
client? incorrect because a nurse should not suggest an alternative ergonomic aid. The client needs a
proper referral to an expert in the field. Option 2 is incorrect because it does not address the
priority need—to make the referral. Although it is important to note what type of exercise the
client practices, it is still more important to refer the client to the proper specialist who can
assist in supervising an athletic treatment regimen.

1.‐ Suggest the use of alternative ergonomic aids such as creatine or creatinine because they provide better results.
2.‐ Discuss foods that are high in CHOs to assist the client in meeting his desired goal.
3.‐ Ask the client what type of sport activity he is doing to see if this method would help.
4.‐ Refer the client to a sports/nutritional specialist or trainer so that he can be properly supervised.

3852 The nurse would consult with the physician about Correct answer: 2 A clear liquid diet is recommended for short‐term use (1–2 days). Therefore, the maximum is The core issue of the question is the length of time a clear liquid diet is appropriate. Use
either advancing the diet or instituting parenteral 2 days. It can be used both before and after surgery or diagnostic procedures, during acute knowledge of clear liquid diet as a therapeutic diet to make a selection.
nutrition if the client has been on a clear liquid diet for stages of illness, or as an initial diet after a significant period of GI inactivity or bowel rest.
the maximum _____ days? Write a numerical answer.

3853 The nurse concludes that the client education about Correct answer: 2 A client is considered to be obese when his or her weight is greater than or equal to 20 Note that the question identifies the client is over ideal weight to eliminate option 4.
nutritional status has been effective when the client percent over ideal body weight and the calculated BMI is greater than or equal to 30. The Eliminate option 3 since this is not a term used in nutrition. Recall the parameters of BMI to
states: "Because I weigh 20 percent more than is ideal above statement by the client implies that there is understanding of this classification. Option direct you to option 1.
and my body mass index (BMI) is greater than 30, I am 1 is incorrect because a BMI between 25 and 30 is categorized as "overweight." Option 3 is
considered: incorrect as the findings relate to clinical obesity and not merely overnutrition. Option 4 is
incorrect because the client is not underweight as defined by the information presented.

1.‐ Overweight."
2.‐ Obese."
3.‐ To have overnutrition."
4.‐ Underweight."
3854 What method would be most appropriate for the Correct answer: 4 Visualization of serving sizes is an easy tool that can be used to teach clients how to The critical words are most appropriate and adolescent, recognizing that teaching needs
nurse to use when teaching an adolescent client who is determine accurate serving sizes without the use of formal measurement tools. It allows for a to be geared to this age group. Eliminate option 1 since it is not practical. Eliminate option 2
interested in learning how to measure serving‐size common reference between known objects and serving size and offers visual reinforcement. since it does not answer the question and eliminate option 3 since this would not provide a
portions? Option 1 is incorrect as it would not be appropriate to ask the client to take measuring cups measuring tool.
with her to school. Option 2 is incorrect because it has nothing to do with determining serving
size. Option 3 is incorrect because reading the food label will provide information relative to a
serving size for the item, but may not help to identify the concept of what a serving size is to
the adolescent client.

1.‐ Have the client bring a set of measuring cups with her to school
2.‐ Have the client eat only half of what is on her plate during all meals
3.‐ Have the client read the food labels on packages
4.‐ Use visualization of serving sizes to illustrate the concept

3855 When participating in a local health fair the nurse Correct answer: 2 Lycopene is a phytochemical with powerful antioxidant activity found in tomato products and The critical words are risk and prostate cancer. Recall the role of lycopene in risk reduction
explains the risk of prostate cancer can be reduced by has been associated with a decrease in risk of prostate cancer. Options 2, 3, and 4 are not to direct you to option 1.
adopting which of the following nutritional habits? associated with reduction of prostate cancer.

1.‐ Increasing intake of tomato products


2.‐ Limiting intake of concentrated sweets
3.‐ Eating large amounts whole grains
4.‐ Decreasing consumption of carbonated beverages

3856 The client has a low total lymphocyte count (TLC). Correct answer: 2 The total number of lymphocytes will decrease when protein deficiency occurs. Options 1 and The critical words are low lymphocyte count and cause. Recall what factors are needed for
Which of the following does the nurse conclude to be 3 are incorrect because they will not directly affect the TLC. A bacterial infection will usually lymphocyte production. Eliminate option 1 as it would not contribute to lymphocyte
the most likely cause? stimulate white blood cell production. production and eliminate option 3 since this would result in an increased count. Recall that
iron plays a role in hemoglobin to eliminate option 4.
1.‐ Rapid weight gain
2.‐ Protein deficiency
3.‐ Bacterial infection
4.‐ Inadequate iron intake

3857 Which statement made by a client indicates the Correct answer: 3 All meat products come from one particular group whose name contains the word meat. All Note the question seeks to determine if teaching has been effective, so only one choice
nurse's teaching about MyPyramid has been effective? of the other choices are incorrect because hot dogs (beef, turkey, pork, or soy) belong to the will be correct. Recall knowledge of food groups and content of hot dogs to direct you to
"Hot dogs belong to the: meat group. option 3.
1.‐ Fruit group."
2.‐ Bread, cereal, rice, and pasta group."
3.‐ Meat, poultry, fish, dry beans, eggs, and nuts group."
4.‐ Fats, oils, and sweets group."

3858 The nurse is assisting a client who is a Correct answer: 3 Pescovegetarians eat fish to supplement a vegetarian diet. The other options would not be The critical words are pescovegetarian and protein. Recognize pesco means fish to direct
pescovegetarian in menu selection. An appropriate used by type of vegetarian. you to option 3.
protein choice for the nurse to suggest the would be:

1.‐ Turkey.
2.‐ Eggs.
3.‐ Fish.
4.‐ Pork.
3859 The nurse is preparing teaching materials for a local Correct answer: 1 Soluble fiber from psyllium seed husk or whole oats is connected to decreasing the risk of The critical words are psyllium, oats, and reduced risk. Make the connection between the
health fair. The nurse includes the health claim that coronary heart disease and is one of the health claims allowed on food products by the Food fiber of psyllium and oats to lower cholesterol levels to direct you to option 1.
consumption of psyllium seed husk or whole oats may and Drug Administration (FDA). Option 2 is incorrect as low folic acid levels are associated with
reduce the risk of which of the following? neural tube defects. Option 3 is incorrect because decreased intake of fats (less than or equal
to 30 percent) and increased consumption of fruits and vegetables correlate with reduced
cancer risk. Option 4 is incorrect since calcium intake correlates with a decreased risk of
osteoporosis.
1.‐ Coronary heart disease
2.‐ Neural tube defects
3.‐ Cancer
4.‐ Osteoporosis

3860 When teaching a client about the concept of nutrient Correct answer: 4 Nutrient density refers to the amount of calories (in carbohydrates, proteins, and/or fats), Note the word nutrient in the question to eliminate option 1, since it only includes
density, the nurse explains it is the: vitamins, and minerals in a given quantity of food. Option 1 is incorrect as it relates to caloric calories. Also note the term includes density and eliminate option 2 since it only includes
value. Options 2 and 3 are incorrect because the quality or quantity of vitamins and minerals in the quality of food. Choose option 4 because this provides a more inclusive definition.
a food is only one part of the calculated nutrient density.
1.‐ Amount of total calories in a specific food.
2.‐ Quality of the vitamins and minerals in a food.
3.‐ Quantity of the vitamins and minerals in a food.
4.‐ Calories, minerals, and vitamins in a quantity of food.

3861 A client has been involved in a nutritional research Correct answer: 1 Reliability is the accuracy or consistency of a study, often measured by the ability to replicate Note that the study is repeated three times obtaining the same results and recall
study that has been performed three times and the the study and obtain the same results. All of the other options are incorrect because they knowledge of research to direct you to option 1.
same findings have been obtained. The nurse relate to validity issues, which determine whether the study accurately reflects what it
concludes this study has a good measure of: purports.

1.‐ Reliability.
2.‐ Statistical value.
3.‐ Validity.
4.‐ Worth.

3862 The nurse has instructed a client on the ingredients of Correct answer: 2 Ingredients are listed in order by descending weight. The information identified in each of the The critical terms are food labels, ingredients, and effective teaching. Since food labels
food labels. Which client statement indicates the other options is incorrect. contain a lot of information, look for the one correct answer specific to ingredients.
nutritional teaching has been effective? "The
ingredients of a food:
1.‐ Are listed in alphabetical order."
2.‐ Get listed in descending order by weight."
3.‐ Have proteins listed first, then carbohydrates, then fats."
4.‐ Are presented in the order of their importance for human nutrition."

3863 What consistent statement in the Dietary Guidelines Correct answer: 2 The 2005 Dietary Guidelines apply the principles of scientific evidence in promoting health Note the critical word in the stem of the question consistent option 2 is the only choice
for Americans 2005 should the nurse include in and reducing risk of chronic disease through diet and physical activity. Option 2 is correct mentioned more than once.
teaching the client to stay healthy? because it includes the two key components found in more than one guideline topics. Option 1
is incorrect because guidelines recommend a total fat intake of 20–35 percent and less than
300 mg of cholesterol per day. Options 3 and 4 are incorrect because under the topic "Food
Groups to Encourage" milk is mentioned but only sufficient amounts while staying within
energy needs.
1.‐ Restrict almost all fat and cholesterol.
2.‐ Balance the intake of food with physical activity.
3.‐ Include higher amounts of milk and meats.
4.‐ Increase portions to ensure adequate amounts of nutrients.
3864 A client has received teaching of basic nutritional Correct answer: 1 Alcohol provides calories but is not an essential nutrient. All of the other choices reflect Note the question asks to identify the need for further teaching, indicating a false
principles. The nurse determines the client needs statements that are consistent with basic nutritional knowledge. response question. Thus, you need to look for an option that indicates an incorrect
additional teaching when he states: statement. Identify options 2, 3, and 4 as being correct statements, leading to option 1 as
the answer to the question.
1.‐ "Alcohol is the chemical basis of one of the essential nutrients."
2.‐ "Protein and carbohydrates are examples of macronutrients."
3.‐ "Water is necessary for human life."
4.‐ "Calcium is a micronutrient."

3865 The nutrition consult done on a client indicates the Correct answer: 3 A BMI of less than 18.5 is classified as underweight. The most important nursing diagnosis is Critical phrase is most appropriate, indicating some or all of the options may be indicated,
body mass index (BMI) is 17. Based on this information Imbalanced nutrition: less than body requirements because the client’s intake is not sufficient but one is more correct. Eliminate options 1 and 2 since insufficient information is given in
the nurse identifies which of the following nursing to maintain a normal BMI. A low BMI places client as risk for respiratory diseases, TB, digestive question to support their choice. Recall knowledge of BMI norms to choose option 3.
diagnoses as being most appropriate? diseases, and some cancers, but does not create risk for dehydration (option 4). The client
could also be at risk for impaired growth, depending on age and specific nutritional deficits
existing, but this information is not provided. The client could also have a knowledge deficit,
but this is also not evident from the question, so would not be the most appropriate diagnosis.

1.‐ Impaired growth


2.‐ Deficient knowledge: nutrition
3.‐ Imbalanced nutrition: less than body requirements.
4.‐ Risk for injury: dehydration

3866 Which of the following data choices would the nurse Correct answer: 1 Height and weight are physical parameters that are considered to be anthropometric The critical words are anthropometric measurement. Recall knowledge of them to
utilize as anthropometric measurements in measurements. Options 2 and 3 are incorrect because they represent laboratory/diagnostic eliminate options 2 and 3, since they are laboratory measurements. Eliminate option 4
determining a client's nutritional status? test results. Option 4 is incorrect because a 24‐hour dietary recall is part of a documented because it is not a measurement.
history.
1.‐ Height and weight
2.‐ Hemoglobin and albumin
3.‐ Total lymphocyte count (TLC)
4.‐ 24‐hour dietary recall

3867 The nurse teaching a client about food product Correct answer: 1 The terms "light" or "lite" can only be used if the product has greater than or equal to one‐ Note that the question describes a food as "light," which refers to the fat content.
labeling explains that a label that reads "Light Salad third fewer calories or greater than or equal to 50 percent less fat than a comparable regular Eliminate options 2 and 3 since they refer to sodium and protein content. Choose option 1
Dressing" contains: product. Option 2 is incorrect because it refers to sodium levels, and the term "light" is related over 4 as it specifically addresses fat content. Recall knowledge of food labeling
to specific caloric levels and/or fat. Option 3 is incorrect because it refers to an increased requirements.
protein level. Option 4 is incorrect because the caloric decrease stated is much higher than the
identified level.
1.‐ 50 percent less fat than comparable regular product.
2.‐ 35 percent less sodium than comparable regular product.
3.‐ 20 percent more protein than comparable regular product.
4.‐ At least 50 percent fewer calories than a comparable regular product.

3868 The nurse caring for a client with a low basal Correct answer: 2 A low BMR occurs when calories are being burned at a slower than normal rate; therefore, Critical words are most likely, indicating there is a tendency for it to occur. Eliminate
metabolic rate (BMR) recognizes the client would be weight gain may occur. Option 1 is incorrect because a low BMR would correlate with options 3 and 4 because they do not reflect metabolism. Choose option 2 since weight gain
most likely to experience which of the following increased weight gain due to possible endocrine disturbances. Options 3 and 4 are incorrect correlates with a slowed metabolism.
nutritional problems? because they reflect poor nutritional status, fatigue, and decreased oxygen‐carrying capacity,
which can occur because a variety of other metabolic factors, not only nutrition.

1.‐ Undernutrition
2.‐ Obesity
3.‐ Low serum albumin
4.‐ Low hemoglobin
3869 The nurse is instructing a client on how to maintain a Correct answer: 3 A food diary involves recording intake of foods and beverages over a specific period of time Note the word diary is part of the plan, indicating more than just listing of food items will
food diary. The nurse explains the diary should include: and also includes emotions and rationales for eating. It can be used to help clients identify be involved. Eliminate options 1 and 2 since they just include food quantities. Eliminate
unhealthy patterns, and it can also be used for weight loss and bulimia/anorexia. Option option 4 because it is very general and less specific than option 3.
describes a dietary intake record, which does not include emotions and reasons. Option 2
describes a 24‐hour recall. Option 4 might or might not be part of a food diary.

1.‐ Every type of food and beverage consumed in the past week.
2.‐ Exact amounts of foods and beverages consumed in the past 24 hours.
3.‐ Exact amounts of food consumed along with feelings and reasons for eating.
4.‐ When and where food is consumed during the last week.

3870 The nurse is instructing a client who wants to begin Correct answer: 1, 2, 3 Soy, beans, and nuts are part of the protein group and would be appropriate for a vegetarian. Notice that the client is vegetarian; eliminate option 4 since it is of animal origin. Recall
following a vegetarian diet about foods that provide Vegetarians do not eat animals that must be killed prior to consumption; most vegetarians knowledge of protein‐containing foods. Eliminate option 5 since it is a grain.
protein. The nurse explains that which of the following (other than vegans) will consume eggs, legumes, and dairy products.
choices would be appropriate for this client? Select all
that apply.
1.‐ Soy
2.‐ Beans
3.‐ Nuts
4.‐ Turkey
5.‐ Wheat germ

3871 The nurse reviewing the laboratory test results on an Correct answer: 3 Transferrin levels provide information relative to iron stores and visceral protein. Option 1 is The question requires knowledge of transferring. A key word is decreased level. Recall
assigned client that indicate a decreased transferrin incorrect because transferrin does not specifically relate to a low dietary intake level. Option 2 transferrin reflects stored iron to eliminate option 1. Eliminate options 2 and 4 since
level. The nurse interprets that this finding would be is incorrect because transferrin levels do not relate to blood loss through hemorrhage, they transferring is not related to blood loss or infection.
associated with which of the following in the client? relate to body stores. Option 4 is incorrect because transferrin levels are not affected by
bacterial infections.
1.‐ Low dietary iron intake
2.‐ Recent episode of blood loss
3.‐ Malnutrition, especially lack of protein
4.‐ Recent bacterial infection

3872 The nurse is writing a care plan for a client at risk for Correct answer: 2 Promoting healthy nutritional practices incorporates both the categories of undernutrition Question requires identifying a goal that is realistic and relevant. Eliminate options 1 and 4
a nutritional deficit. An appropriate goal would be: and overnutrition. The correct option provides a goal that is global and relevant to nutrition. since they are nursing intervention, not goals. Eliminate option 3 since the question did not
Option 1 and 4 are incorrect because they reflect an intervention, not a goal. Option 3 is identify a weight problem.
incorrect because weight maintenance would only be an appropriate goal if the client is
underweight and there is insufficient information to determine this.

1.‐ Client will wear prescribed oxygen at 2 liters/minute.


2.‐ Client will verbalize two healthy nutritional practices.
3.‐ Client will maintain current weight.
4.‐ Client will monitor weight twice daily at the same times each day.

3873 An elderly client asks the nurse, "What’s all this stuff Correct answer: 1 A Daily Recommended Intake (DRI) is a nutritional guideline that goes beyond the RDAs. It Note the critical word best, indicating some options may be partially correct. Eliminate
about DRIs that I keep hearing?" Which of the includes Tolerable Upper Limits and Adequate Intake amounts and is established by the option 4 because it does not offer an explanation to the client. Eliminate options 2 and 3
following statements by the nurse provides the best government to provide a more comprehensive approach to nutrition. Option 2 is incorrect since they do not completely address DRIs.
explanation? because DRIs do not merely relate to caloric intake. Option 3 is incorrect because DRIs are not
based solely on weight variables but reflect a recommended intake of nutrient factors. Option
4 is incorrect because even though the physician is knowledgeable, any one healthcare
provider does not have complete knowledge of nutritional therapies. A collaborative approach
is necessary to provide information to the client.
1.‐ "They are updated nutritional guidelines that the government developed."
2.‐ "The DRIs are guidelines for how many calories you need in a day."
3.‐ "They refer to guidelines for weight loss for persons who are 25% overweight."
4.‐ "Don’t worry, your doctor will tell you what you need to know about nutrition."

3874 The nurse is assisting the client in menu planning Correct answer: 2 MyPyramid recommends three servings a day from milk, cheese, and dairy products. One Note that question asks for adequate daily intake, not just a single serving size. Options 1
using MyPyramid as a standard. The nurse encourages serving is equal to 8 ounces of milk or 2 ounces of cheese. Option 1 does is equal to one and 3 are similar and can be eliminated. Eliminate option 4 since eggs are not part of the
the client to choose which of the following items on serving, but not a day’s supply. Option 3 is an insufficient amount. Option 4 is also insufficient, milk and dairy group.
the menu to obtain an adequate daily intake of milk and eggs are not part of the protein category.
and dairy products?
1.‐ 8 ounces of skim or whole milk
2.‐ 8 ounces of milk and 2 ounces of cheese
3.‐ One‐half cup of yogurt
4.‐ One‐half cup of yogurt and one egg

3875 A 34‐year‐old diabetic client who is receiving insulin Correct answer: 2 The use of an exchange list is recommended by the American Dietetic Association for use by Note the question asks for a method or guide to diet planning. Eliminate options 3 and 4
therapy is not making healthy dietary choices to diabetic clients for meal planning. Option 1 is incorrect: Even though the MyPyramid is since they involve record keeping of a diet, but do not involve planning. Recall knowledge of
control blood glucose. The nurse would use which of developed by the USDA to illustrate healthy diet choices, the use of an exchange system is diabetic diets to choose exchange lists.
the following as the most appropriate aid when geared specifically to equivalent carbohydrate contents that are critical for a diabetic client.
reviewing diet planning with the client? Options 3 and 4 are incorrect because they are not diet planning guides but rather examples of
tools that can be used to evaluate nutritional status.

1.‐ MyPyramid
2.‐ Exchange list
3.‐ Food diary
4.‐ 24‐hour recall

3876 The nurse is demonstrating to a client how to read Correct answer: 4 Sugars are a carbohydrate (CHO), and each gram of CHO or sugar contains 4 kcal. Multiply 3 Identify how many grams of sugar are listed on the label. Be sure to select just sugar, not
food labels. Using the label below, the nurse calculates grams 3 4 = 12 calories. total CHOs. Use key at bottom of label to multiply and arrive at calories/Gm.
the food item derives ____ calories from sugars
contained in the product? Write a numerical answer.

3877 During a client teaching session, which of the Correct answer: 4 A hot dog bun is found in the bread, cereal, rice, and pasta group. Option 1 is incorrect. Recall knowledge of the food groups to eliminate options 1, 2, and 3, which are protein
following foods would the nurse identify as belonging Peanuts are found in the meat, poultry, and fish group because they are considered to be and vegetable groups.
to the bread, cereal, rice, and pasta group of legumes. Options 2 and 3 are incorrect because coconut is in the vegetable group; navy beans
MyPyramid? are in the meat, poultry, and dried beans group.
1.‐ Peanuts
2.‐ Coconut
3.‐ Navy beans
4.‐ Hot dog bun
3878 The nurse is involved with a nutritional research Correct answer: 1 Epidemiologic nutritional research studies nutritional factors that influence the cause, Note the critical word quantifying and that a specific population, the elderly, is being
project that involves quantifying calcium intake and frequency, and distribution of disease, injury, or other health‐related events in a defined studied. Recall knowledge of types of research to choose option 1.
osteoporosis rates of occurrence in elderly adults. The human population. Option 2 is incorrect because laboratory research is used to test scientific
nurse concludes that this is which of the following hypothesis in a controlled setting. Option 3 is incorrect because the use of human studies
types of studies? research involves manipulation of variables affecting nutrient intake. The stated example
above relates to an intake study that is attempting to correlate rates of disease occurrence.
Option 4 is incorrect because qualitative research looks at the quality of a given variable. The
nutritional research study proposed does not provide enough information relative to type and
amount of calcium intake, so one cannot categorize this research as either qualitative or
quantitative.

1.‐ Epidemiologic research


2.‐ Laboratory research
3.‐ Human studies research
4.‐ Qualitative research

3879 The nurse has taught a client to read nutrition labels. Correct answer: 2 Health claims on food products are regulated by the FDA, are trustworthy, and are based on The question is asking to assess effectiveness of teaching. Recall specific knowledge of
Which of the following statements by the client scientific evidence. The food products making the claims must meet specific rigid criteria. labeling standards and laws to choose option 2.
indicates to the nurse that the teaching has been Options 1 and 3 are incorrect because FDA regulations are strictly enforced, and companies
effective? must comply with federal guidelines or risk penalty. Option 4 is incorrect because food product
labels are regulated by the FDA and not by individual states.

1.‐ "Those nutrition labels are misleading. Companies can put whatever they want on the labels."
2.‐ "When a label says a food product helps reduce the risk of cancer, you can believe it."
3.‐ "Food label health claims are exaggerated and based on eating abnormally large amounts."
4.‐ "The health claims on food product labels are individually determined by each state."

3880 A nurse conducting a nutrition class tells the class Correct answer: 2 Calories or kilocalories are the most commonly used measurement of a food product’s Critical words are energy and measurement. Recognize that options 1, 3, and 4 are
that the energy contained in a specific amount of a energy. Grams and pounds reflect weight measurements. Joules represent electrical energy. measures of weight and quantity and thus they can be eliminated.
food product is measured in:
1.‐ Joules.
2.‐ Calories.
3.‐ Grams.
4.‐ Pounds.

3881 The school nurse is assigned to collect Correct answer: 3, 4 Height, weight, and skin fold thickness are common anthropometric data (i.e., measurement Critical words are anthropometric data and measurement. Eliminate option 2 since it does
anthropometric data from preschool children of body parts) obtained by the nurse, as well as the body mass index. Options 1 and 2 are not involve an actual measurement. Note also that Options 1, 2, and 3 are distracters
regarding nutrition. From which of the following incorrect: CBC (and hemoglobin, which is a test included in the CBC) reflect laboratory relating to diagnostic studies and not anthropometric data.
methods can the nurse choose to collect this type of parameters. Option 5 is incorrect because albumin is a type of serum protein and is another
data? Select all that apply. laboratory measurement.
1.‐ Complete blood count (CBC)
2.‐ Hemoglobin
3.‐ Body mass index (BMI)
4.‐ Triceps skin folds
5.‐ Albumin
3882 When assessing a client's nutritional status, the nurse Correct answer: 4 The Body Mass Index (BMI) is calculated by taking the weight in kilograms and dividing by the The focus of the question is the purpose or applicability of BMI. Recall BMI involves
uses the client's Body Mass Index (BMI) to determine: height in meters squared. It assesses weight compared to height, or fatness. Option 1 is measurement of height and weight. Eliminate options 2 and 3 since they are refer to
incorrect because these are merely descriptive terms that relate to body frame variables and metabolism and blood chemistries. Eliminate option 1 since it is a shape, not related to
overall distribution of weight. Option 2 is incorrect because the metabolic rate of an individual height and weight.
relates to basal metabolic rate (BMR). Option 3 is incorrect, because BMI does not detect
hypercholesterolemia. Elevated cholesterol levels can be found in individuals with differing
BMI values because both genetic and metabolic factors influence its development.

1.‐ Apple‐ versus pear‐shaped body type.


2.‐ Metabolic rate of the individual.
3.‐ Hypercholesterolemia.
4.‐ Relative fatness or weight to height.

3883 When planning nutritional requirements to promote Correct answer: 3 A positive nitrogen balance implies that a client is in a growth state whereby extra high Recall that a positive nitrogen balance is achieved through protein to direct you to option
a positive nitrogen balance in a client the nurse should biologic value protein sources are needed for growth of tissue. Option 1 is incorrect because 3.
include which of the following? additional protein, not CHO, is needed to restore nitrogen balance. While CHO sources are the
primary source of energy, periods of positive nitrogen balance require additional protein as
the necessary nutrient. Even though a decrease in the amount of saturated fat in the diet is
advisable, the issue of nitrogen balance addresses protein. Option 4 is incorrect because
although milk is a good source of protein, three servings is the usual recommendation in the
diet and this will not meet the needs of a client who is in positive nitrogen balance.

1.‐ Increased carbohydrates (CHOs) in the diet to provide energy


2.‐ Decreased intake of saturated fats in the diet
3.‐ Increased intake of high biologic value protein foods
4.‐ Three servings of milk products in the daily diet

3884 The nurse identifies which of the following clients Correct answer: 4 A client who is in the hospital due to a GI infection is at risk to develop protein deficiencies if Analyze each option for indication of protein deficiency or potential to contribute to same
may be at risk for protein deficiency? GI symptoms such as diarrhea and intestinal inflammation exist. Option 1 is incorrect because to choose option 4.
the combination of milk and cereal represents a complementary protein and will support
needed protein requirements. Option 2 is incorrect because soy burgers are an example of a
complete protein source. Option 3 is incorrect as a client who is in renal failure is more likely
to have problems related to excess protein due to an inability of the kidneys to handle an
increased solute load.

1.‐ A young child who eats milk and cereal each day for breakfast
2.‐ A college student who eats soy burgers for lunch
3.‐ A client who is in renal failure
4.‐ A client who has been hospitalized for a gastrointestinal (GI) infection

3885 The nurse is teaching a group of clients with cardiac Correct answer: 3 The highest concentration of omega‐3 fatty acids is found in cooking oils, margarine, fatty fish Critical words are omega‐3 fatty acids. Recall that many fish contain this type of fat to
disease about the health benefits of omega‐3 fatty (e.g., cod, tuna, salmon, shrimp, and mackerel), and flaxseed oil. The other options are not choose option 1.
acids. The nurse suggests increasing consumption of good sources of omega‐3 fatty acids.
which of the following?

1.‐ Chicken
2.‐ Butter
3.‐ Tuna
4.‐ Peanuts
3886 Which of the following findings does the nurse Correct answer: 1 Increased amounts of fiber in the diet (in excess of dietary recommendations or increased too The critical word is excessive. Recall the action of fiber in the GI tract to direct you to
attribute to the client's excess fiber intake in the diet? rapidly) can lead to GI presentations that include diarrhea, flatulence, and generalized option 1.
discomfort. Option 1 is incorrect as increased use of insoluble fiber can decrease the
absorption of certain minerals. Option 3 is incorrect because excess fiber will increase GI
motility leading to diarrhea. Option 4 is incorrect because fiber helps to bind cholesterol in the
body for elimination.
1.‐ Diarrhea and abdominal bloating
2.‐ Elevated calcium and magnesium levels
3.‐ Decreased motility of the gastrointestinal (GI) tract
4.‐ Increased serum cholesterol levels

3887 What dietary instruction should the nurse give a Correct answer: 2 The client needs to increase water intake while increasing fiber intake to prevent Note that the client is increasing fiber to eliminate options 2 and 4, since they limit intake
client who is seeking to add more fiber to the daily constipation. It is recommended to have 3–5 servings of vegetables and 2–4 servings of fruit of foods with fiber. Choose option 1 as appropriate advice to give regarding fiber.
diet? daily, making option 1 incorrect. Option 3 is incorrect as many foods are high in fiber. Option 4
is incorrect because vegetables provide a good source of insoluble fiber while grains and
broccoli tend to provide soluble fiber.
1.‐ Limit fruits and vegetables to twice a day.
2.‐ Increase water intake to accompany fiber intake.
3.‐ Use fiber supplements to increase dietary fiber because few foods are high in fiber.
4.‐ Limit the number of salads in the diet.

3888 What physiological response should the nurse expect Correct answer: 3 A high‐CHO meal will lead to an increase in circulating insulin levels as part of the glycemic Critical terms are high CHO diet and prolonged time. Recall the effect of CHO on insulin,
in a client who consumes a high‐carbohydrate (CHO) response. Option 1 is incorrect because insulin production will increase in response to a high glucose, and fats to eliminate options 1, 2, and 4.
diet over a prolonged period of time? CHO meal. Option 2 is incorrect as a decrease in insulin sensitivity and an increase in insulin
resistance would be expected in a client who has been consuming a high CHO diet for a
prolonged period of time. Option 4 is incorrect because some form of dyslipidemia would be
expected in a client who has been consuming a high CHO diet for a prolonged period of time.

1.‐ A decrease in the amount of circulating insulin


2.‐ An increase in insulin sensitivity
3.‐ An increased glycemic response
4.‐ A stable lipid profile

3889 The nurse has instructed a client on food choices to Correct answer: 2 Option 2 represents the best source of complex CHOs because it includes whole grains and Analyze each option for complex CHO content. Eliminate options 3 and 4 since they
increase the amount of complex carbohydrates (CHOs) fruit. Option 1 is incorrect as instant rice and canned green peas are examples of food items contain more fats and proteins than CHO. Choose option 2 over 1 since it is less processed
in the diet Which menu selection made by the client that have been processed or refined. Option 3 is incorrect because scrambled eggs with ham and refined.
indicates the teaching was effective? and sausage represent a meal that is high in protein and saturated fats. Option 4 is incorrect
because fried potatoes are high in fat content and fruit juice is high in simple sugars.

1.‐ Grilled chicken, instant rice, and canned green peas


2.‐ Grilled chicken sandwich on whole wheat bread with an apple
3.‐ Scrambled eggs with ham and sausage
4.‐ Fried potatoes and fruit beverage

3890 Which of the following statements, if made to the Correct answer: 2 The incorporation of tuna in the diet reflects a food that is high in omega‐3 fatty acids, which Critical words are CAD and fat intake. Eliminate options 3 and 4 since they contain high fat
nurse, reflects that a client with coronary artery is beneficial in reducing cardiovascular risks. Option 1 is incorrect because the amount of fiber food items. Eliminate option 1 since fiber should be increased with CAD.
disease (CAD) has been compliant with dietary specified is far below the recommended daily dietary intake. Option 3 is incorrect because the
teaching to decrease fat intake? use of butter and partially hydrogenated cooking oils increases both the amounts of saturated
fat and trans‐fatty acids in the diet. Option 4 is incorrect because the use of fried food
products contributes to an increase in fat intake.

1.‐ "I have been limiting my fiber intake to 10 grams per day and drinking more fluids."
2.‐ "I eat tuna once a week."
3.‐ "I use butter and partially hydrogenated cooking oils."
4.‐ "I eat chicken nuggets a few times a week."

3891 What information should the nurse give to a client Correct answer: 3 Food and Drug Administration (FDA) regulations require that foods containing olestra have a The question requires a comparison of artificial fats with natural fats. Recall that artificial
who is using artificial fats in the diet to replace natural warning label stating that abdominal cramping and loose stools can occur from the use of this fats are not well absorbed in the GI tract to choose option 3.
sources of fat? artificial fat product. Option 1 is incorrect because artificial fat products cannot be substituted
for low fat products. Low fat describes the content of a food item as having 3 grams or less of
fat per serving. Option 2 is incorrect because there are different types of artificial fats derived
from different chemical sources (carbohydrates and proteins). Option 4 is incorrect because
the use of artificial fats in the diet can lead to decreased absorption of nutrients because most
artificial fats are not absorbed well from the GI tract.

1.‐ "Artificial fats can be substituted for foods that are low in fat in the diet because they are chemically similar."
2.‐ "All artificial fat products are chemically similar."
3.‐ "Products that contain olestra can cause a client to experience gastrointestinal (GI) symptoms."
4.‐ "The intake of artificial fats leads to increased absorption of nutrients."

3892 Which of the following food items should the nurse Correct answer: 3 Natural grain products usually contain little fat, and a half a bagel has a very low fat content. Note the question does not address a certain type of fat and analyze each option for
suggest to a client who is seeking to lower his fat The other choices reflect foods that have a high fat content and represent saturated (cheese overall fat content, choosing option 3 as lowest.
intake? and coconut) and monounsaturated (almonds) types of fat.
1.‐ Natural cheddar cheese
2.‐ Coconut
3.‐ Half a bagel
4.‐ Almonds

3893 The nurse is providing dietary counseling to a client Correct answer: 1 Omega‐3 fatty acids have been shown to reduce risk factors associated with heart disease. The question requires recall of knowledge of foods high in omega‐3 fatty acids. Since fatty
with coronary artery disease (CAD). The nurse They are found in fatty fish, fish and flaxseed oils, and cooking oils. acids are the product of fat breakdown, eliminate options 2 and 3 since they do not contain
encourages intake of foods high in omega‐3 fatty acids, or have very little fat. Eliminate poultry (option 4) since it contains saturated fat in the skin.
such as those found in which of the following?

1.‐ Fish oils


2.‐ Whole grains
3.‐ Grapefruit
4.‐ Poultry

3894 Laboratory results indicate a client has elevated Correct answer: 3 Clinical evidence has supported that elevated homocysteine levels correlate with increased A critical word is increased. Recall homocysteine is associated with cardiovascular disease
homocysteine levels. The nurse explains to the client risk for the development of atherosclerotic heart disease and deficiencies of certain B complex to choose option 2.
that this can contribute to which of the following? vitamins. Option 1 is incorrect—homocysteine is an amino acid that is produced by the
breakdown of the amino acid methionine. Option 2 is incorrect because elevated
homocysteine levels are associated with vitamin B deficiencies. Option 4 is incorrect because
homocysteine metabolism is not related to lipid absorption.

1.‐ Decreased utilization of essential amino acids


2.‐ Increased metabolic production of B vitamins
3.‐ Increased risk of atherosclerotic disease
4.‐ Decreased absorption of lipids

3895 The nurse explains to a client who is following a low‐ Correct answer: 4 Clients with deficient protein intake are at risk for immune dysfunction and fatigue. High Critical words are low protein diet. Eliminate option 1 because it is related to high protein
protein diet that the following consequences could protein intake may put clients at risk of coronary heart disease, not low protein. Steatorrhea is intake. Option 2 is associated with fat metabolism. Recall protein is needed for adequate
possibly occur? usually associated with altered fat metabolism. Pyelonephritis is an infection and is not caused cellular metabolism to choose option 4.
by low protein diets.
1.‐ Increased risk of coronary artery disease
2.‐ Frequent episodes of steatorrhea
3.‐ Development of pyelonephritis
4.‐ Impaired immune function

3896 The nurse has conducted dietary teaching with a Correct answer: 4 Clients with heart disease benefit from foods that are low in fat and, if hypertensive, low in Note animal foods provide richest sources of fat, so the beef can be eliminated first
client who has heart disease. In evaluating the salt. Baked fish without added sources of fat represent the best choice on the sample menu. followed by the poultry. This leaves fish as the best choice, especially since there are no
effectiveness of the session, the nurse concludes that The turkey would be a good choice if it did not have a cream sauce, which is high in fat. Beef is added sauces or coatings that contain fat.
the best outcome is obtained when the client selects high in fat because it is an animal product and contains more fat than poultry. The baked
which of the following food items from a sample chicken would be a better choice if the skin were removed, because skin is high in fat also.
restaurant menu?
1.‐ Turkey casserole in a cream sauce
2.‐ Barbecued beef ribs
3.‐ Baked chicken breast with skin
4.‐ Baked fish coated in bread crumbs

3897 The nurse would include which strategy as most Correct answer: 1 Trans‐fatty acids are found in processed hydrogenated foods. The other options address A critical concept in the question is trans fatty acids. Options 2 and 4 can be eliminated
appropriate in the care plan of a client who needs to reduction of cholesterol and saturated fats in the diet, but do not address trans‐fatty acids since they address cholesterol and saturated fats. Recall hydrogenated fats are a common
limit intake of trans‐fatty acids? specifically. Some foods high in cholesterol and fat, although they should be reduced, may not source of trans fatty acids to choose option 1.
be high in trans‐fatty acids.
1.‐ Teach client to read nutrition labels checking for hydrogenated fats.
2.‐ Have client select low‐cholesterol foods from a dietary plan.
3.‐ Assist client to choose non‐dairy foods when filling out the menu.
4.‐ Ask client to name three foods high in saturated fats.

3898 Which of the following consequences would the Correct answer: 1 A state of prolonged CHO deficiency can lead to protein breakdown that results in formation Critical words are prolonged deficiency and CHOs. Eliminate option 4 as it is associated
nurse anticipate in a client who has had prolonged of ketone bodies and altered acid‐base balance (metabolic acidosis). Options 2 and 3 are with excessive CHO intake. Recognize options 2 and 3 are associated with protein and fats
carbohydrate (CHO) deficiency? incorrect because they are associated with protein deficiency and lipid deficiency, respectively. to eliminate them.
Option 4 is incorrect because these are associated with an increased amount of CHOs in the
diet.
1.‐ Ketosis and metabolic acidosis
2.‐ Anemia and edema
3.‐ Skin lesions and weight loss
4.‐ Dental caries and obesity

3899 The nurse has instructed a client on complementary Correct answer: 2 Pinto beans and rice are examples of the use of complementary protein because this Recall knowledge of combining plant proteins to attain a complete protein. Eliminate
proteins. Which diet selection by the client illustrates combines two different food items to yield a complete protein source. All of the other options options 1, 3, and 4 since they include meat proteins.
that the client has understood the material presented? do not combine to make a complete protein source. A complete protein source provides all of
the essential amino acids and is of high biologic quality and value.

1.‐ Eggs and bacon


2.‐ Pinto beans and rice
3.‐ Fish and chips
4.‐ Hamburger and fries
3900 Which of the following dietary methods could the Correct answer: 3 High intake of sugars (CHOs) is associated with dental caries and progression of dental Note the critical word best, indicating one option is a better choice. Consider the residual
nurse best utilize in a plan of care for a client to disease. By limiting the intake of soft drinks in the diet, one is reducing the daily CHO content. effect of candy, soda and popcorn residue on teeth to eliminate options 1, 2, and 4.
minimize the risk of dental caries? Hard candies are mainly composed of sugar and will contribute to dental disease, because they
increase the sugar medium in the oral cavity (option 1). Even air‐popped popcorn can easily
become lodged between the teeth, leading to food's remaining in the oral cavity and bacterial
progression (option 2). Option 4 is incorrect because rinsing of the mouth after eating candy
will not effectively remove the extra sugar that the candy provided. Brushing and flossing the
teeth would prove to be a better option because this would remove food remnants.

1.‐ Use hard candies to prevent dryness of the oral cavity.


2.‐ Eat air‐popped popcorn when eating a snack food.
3.‐ Limit intake of soft drinks to one or two a day.
4.‐ Rinse mouth after eating candy or anything sweet.

3901 Which of the following signs should the nurse check Correct answer: 3 Olestra blocks absorption of fat in the diet, sometimes causing abdominal cramping and loose Recall knowledge of the function of Olestra to block fat. Note options 1, 2, and 4 all are
for in a client who admits to eating a lot of snack foods stools. It is not associated with the symptoms in options 1, 2, and 4. similar in relating to the neurovascular status. Blockage of fat would occur in the intestines,
containing Olestra? providing a clue to the abdominal cramping.
1.‐ Numbness in fingers
2.‐ Headaches
3.‐ Abdominal cramping
4.‐ Dizziness

3902 A client needs to follow a low‐fat diet. Which of the Correct answer: 1, 2, 3 The degree of unsaturation is in the firmness of fats at room temperature. The The critical word avoid in the question stem tells you that the correct answer(s) will be
following products would the nurse instruct the client polyunsaturated vegetable oils are liquid; therefore the more saturated animal fats are solid. items that are incorrect to use when trying to lower fat content in the diet.
to avoid using when cooking? Select all that apply. Options 1 and 2 are solid and should be avoided. Option 3 should also be avoided because
even though coconut oil is not polyunsaturated.
1.‐ Solid margarine
2.‐ Shortening
3.‐ Coconut oil
4.‐ Corn oil
5.‐ Canola oil

3903 Which information should the nurse provide to a Correct answer: 1 CHOs are the primary energy source used to maintain the body and a minimum level of CHOs Critical words are low CHO and lose weight. Recognize there are many ways to approach
client who is planning to begin a low‐carbohydrate (50–100 grams/day) is needed in order to avoid protein breakdown. Option 2 is incorrect weight reduction and eliminate option 2. Eliminate options 3 and 4 since they are incorrect
(CHO) diet to lose weight? because there are other methods (besides lowering CHO intake) to establish sustained weight statements.
loss. Option 3 is incorrect. A balanced intake of all three macronutrients is needed in order to
maximize function, prevent breakdown of constituent products, and maintain energy. Option 4
is incorrect because an increase in dietary fiber will not compensate for a decrease in total
CHOs. Although dietary fiber provides health benefits related to elimination and cholesterol
levels, an increase in fiber above current recommendations (20–35 grams/day) may cause an
increase in GI symptoms and lead to constipation.

1.‐ A minimum level (50–100 grams/day) of CHOs is needed in the body in order to prevent protein breakdown.
2.‐ Lowering of CHOs is the only way to effect sustained weight loss.
3.‐ A decrease of CHOs in the diet will not affect other nutrient proportions in the body.
4.‐ The client should increase dietary fiber level to compensate for decreased CHOs.
3904 The nurse has instructed a vegetarian client regarding Correct answer: 4 The use of complementary proteins in a diet pattern refers to the combining of different plant Note that the critical word in the question is complementary and this word is repeated in
the use of complementary proteins in the diet. Which proteins in a day to form a complete protein that is of high biologic value. Vegetarians should option 4. Recall knowledge of complete versus complementary proteins to eliminate
of the following statements by the client indicates that receive instruction on this method in order to maintain required essential amino acid options 1 and 2. Eliminate option 3 because fruits are not adequate protein sources.
client teaching has been adequate? requirements in the body and prevent clinical deficiencies that could arise due to their choice
of vegetarianism. Option 1 is incorrect because gelatin is an animal protein of low biologic
value and would not be included in a vegetarian diet. Option 2 is incorrect because the idea of
complementary proteins is to combine food choices and not merely to increase the amount of
complete protein sources. Option 3 is incorrect because an increase in fruits will not provide
complete protein.

1.‐ "I will eat gelatin for dessert."


2.‐ "I will eat more complete protein food sources."
3.‐ "I will increase my intake of fruits and eat a wide variety of them."
4.‐ "I will eat different plant food products together to provide complementary proteins."

3905 A client asks the nurse, "How can I best reduce Correct answer: 3 It is recommended that cholesterol intake be limited to 300 mg/day. No more than 10 The question addresses reduction of cholesterol. Eliminate option 2 since it does not
cholesterol in my diet?" Which of the following is the percent of fats should be saturated. Checking labels is advisable but does not provide the client answer the client's question. Eliminate options 1 and 4 because they are incorrect.
nurse's best response? with specific guidelines. Eggs can be eaten but should be limited to 2–3/week.

1.‐ "No more than 30 percent of your diet should come from saturated fats."
2.‐ "Check the labels on foods to identify cholesterol content."
3.‐ "Limit intake of cholesterol to 300 mg/day."
4.‐ "Do not eat any eggs or products containing eggs."

3906 When assessing a client's nutritional status, the nurse Correct answer: 4 Nitrogen balance refers to the concept of a balanced protein state in the body to support Note that options 1 and 2 are normal lab values and can be eliminated. Recall nitrogen is
concludes that which of the following conditions is metabolism. A client who is in positive nitrogen balance is taking more nitrogen in and the core molecule of protein to choose option 4.
most likely to affect the action of protein in the body? excreting less nitrogen in order to meet metabolic needs (growth state with increased
demands). Options 1 and 2 are incorrect because they represent normal range findings for
sodium and potassium levels in the body. It is important to note that protein function does
depend on the interaction of acid/base and serum electrolytes in order to function effectively,
and the nurse must be alert to look at pertinent laboratory findings. A slight state of fluid
overload (option 3) is less significant than a state of positive protein balance, which correlates
directly with the question.

1.‐ Serum potassium level of 4.0 mEq/L


2.‐ Serum sodium level of 145 mEq/L
3.‐ A state of slight fluid overload
4.‐ A state of positive nitrogen balance

3907 The nurse is teaching a client ways to reduce total fat Correct answer: 4 Vegetables, fruits, and grains in the diet are low in fat and are rich in nutrients and Note question addresses reduction of fats. Eliminate options 1 and 3 because they suggest
intake in the diet. The nurse instructs the client to phytochemicals. Option 1 is incorrect because trans‐fatty acids are associated with increased increasing fats. Eliminate option 2 since it does not address fat intake.
increase intake of which of the following to achieve cardiovascular risks. Option 2 is incorrect because the use of fish oil supplements can interfere
this result? with bleeding times, diabetic state, immune status, and wound healing. A balanced level of
omega‐3 and omega‐6 fatty acids is recommended in the diet using natural sources. Option 3 is
incorrect because the process of hydrogenation increases the amount of trans‐fatty acid and
would not be a prudent choice.

1.‐ Trans‐fatty acids


2.‐ Omega‐3 fatty acids via fish oil supplements
3.‐ Hydrogenated food products
4.‐ Vegetables, fruits, and grains
3908 When caring for a client with a diet history of high Correct answer: 1 Increased intake of animal protein may be associated with cardiac disease because animal Critical words are history of high animal protein intake and increased risk. Eliminate
animal protein intake, the nurse interprets that the food protein sources are also high in saturated fat content. Options 2 and 3 reflect clinical options 1 and 2 since they are associated with protein losses or deficits. Recall correlation
client is at risk for developing which of the following? conditions in which there are protein losses due to either intake or cell breakdown. Diabetes of animal protein sources with fat to choose option 3.
(option 4) is associated with excessive caloric intake or an autoimmune origin.

1.‐ Marasmus
2.‐ Diarrhea
3.‐ Cardiac disease
4.‐ Diabetes

3909 A client has received teaching about the function of Correct answer: 2 Lipids are not a primary energy source but rather serve as an energy reserve in the body. This question requires identifying one incorrect answer, as 3 options are correct. Recall
lipids in the body. The nurse determines further CHOs are the primary energy source of the body. Options 1, 3, and 4 are incorrect as they all options 1, 3, and 4 are normal functions of fat and eliminate them.
information is required when the client states that represent functions of lipids in the body (e.g., part of cell membranes, support of internal
fats: organs, and insulation).
1.‐ Are an integral part of cell membranes.
2.‐ Act as the primary source of energy in the body.
3.‐ Support and cushion internal organs.
4.‐ Maintain temperature by providing insulation.

3910 Which of the following artificial sweeteners would Correct answer: 2 A client with PKU has a genetic condition that prevents the utilization and conversion of the The question requires specific recall of PKU and the content of artificial sweeteners. If you
the nurse recommend for a client who has amino acid phenylalanine, leading to increased levels with toxic clinical manifestations. The had difficulty answering the question, review this content.
phenylketonuria (PKU)? product Sweet ’n’ Low contains saccharin as the sweetening agent, which will not cause
problems for the PKU client. All of the other options are incorrect, because both Equal and
Nutrasweet (options 1 and 3) contain aspartame (option 4) as their active ingredient.
Aspartame contains aspartic acid, a methyl group, and phenylalanine. Intake of these products
can be dangerous for a client who has a clinical diagnosis of PKU; warning labels are found on
packages of these food products denoting this fact.

1.‐ Equal™
2.‐ Sweet ‘n’ Low™
3.‐ Nutrasweet™
4.‐ Aspartame

3911 A food diary indicates that a client is taking in an Correct answer: 315 Each gram of fat supplies 9 kcal. Multiply the 9 kcal by 35 (the number of daily grams of fat in The core issue of the question is knowledge that each gram of fat contains 9 kilocalories.
average of 35 grams of fat per day. The nurse the diet) to obtain a result of 315 kcal. (9 3 35 = 315) Take time to become familiar with these basic nutritional facts if this question was difficult.
calculates that the client is obtaining ____ kilocalories
from fat in the daily diet. Write in a numerical answer.

3912 The nurse is preparing to teach a group of 16‐year‐ Correct answer: 2 Essential nutrients are needed by the body in their original form, as the body cannot Note the word best, indicating all or some options may be correct, but one provides a
olds in a health class about essential nutrients. Which synthesize them from other materials in the body. Option 1 is incorrect because an essential more thorough answer. Options 2 and contain similar concepts, but recall essential
of the following statements would the nurse use to nutrient does not provide all the necessary energy requirements for the body. Option 3 is nutrients cannot be synthesized to choose option 2.
best describe the concept of an essential nutrient? incorrect since essential nutrients differ in their amounts of fat, protein, and carbohydrates.
Option 4 is incorrect because this statement describes a nonessential nutrient.

1.‐ "An essential nutrient provides all the necessary energy requirements for the body."
2.‐ "Nutrients that are essential are supplied to the body in their active form, as the body cannot synthesize them from other sources."
3.‐ "An essential nutrient provides the same amount of fat, protein, and carbohydrates."
4.‐ "An essential nutrient can be synthesized in the body from precursor forms."
3913 Which of the following statements would indicate to Correct answer: 3 Most healthy clients are able to meet their vitamin requirements through dietary intake. The critical word is vitamin. Eliminate option 1 since it is incorrect and eliminate option 2
the nurse that client teaching regarding vitamins has Option 1 is incorrect as vegetables are usually very good sources of vitamins. Option 2 is since it relates primarily to potassium replacement. Recall that food intake should be the
been successful? incorrect because even though a banana has vitamin activity (A, C, K, and folate), the client is first choice for intake of nutrients to direct you to option 3.
taking this to replace potassium losses incurred with the use of a diuretic. Option 4 is incorrect
since vitamin needs are usually able to be met from food substances along with proper diet.

1.‐ "I need to eat small amounts of vegetables because they are not good vitamin sources."
2.‐ "I will eat a banana each day after taking my water pill."
3.‐ "As long as I eat a well‐balanced diet, I should be able to meet my vitamin requirements."
4.‐ "I will have to take vitamin supplements to meet my vitamin requirements."

3914 A client complains of oral discomfort. When the Correct answer: 2 B complex vitamin deficiencies can often present with clinical presentations affecting the Critical words are cheilosis and glossitis. Note that the question asks for an increased
physical examination reveals cheilosis and glossitis, the mouth and tongue resulting in cheilosis (inflammation of mucous membranes in the mouth intake of something, indicating a deficiency exists. Recall knowledge of these conditions to
nurse should instruct the client to increase intake of and lip) and glossitis (inflammation of the tongue). The other options are not associated with direct you to option 2.
which of the following? this presentation.
1.‐ Fat‐soluble vitamins
2.‐ B complex vitamins
3.‐ Sodium and potassium
4.‐ Vitamin A

3915 A client has just been diagnosed with night blindness. Correct answer: 1 Night blindness is often the first indicator that a clinical deficiency of vitamin A exists in the Critical words are night blindness and highest priority. Recall that this condition is related
The nurse should place the highest priority on body. Correction with vitamin A at the time of early clinical diagnosis will correct the clinical to vitamin A deficiency to direct you to option 1.
reinforcing with the client with which of the following condition. Option 2 is incorrect because this condition is not permanent if nutrient deficiency is
pieces of information? detected and treatment is started. Option 3 is incorrect as this does not require long‐term
hospitalization. Option 4 is incorrect because this condition is not caused by an allergic
reaction and therapeutic treatment is needed to correct this problem.

1.‐ "This is only a temporary condition that can be reversed with effective medical treatment of vitamin A."
2.‐ "This condition will be permanent so a referral is needed for occupational therapy to assist you with living with this type of deficit."
3.‐ "Long‐term hospitalization may be needed for this condition."
4.‐ "This condition usually resolves spontaneously and is often a result of an allergic reaction."

3916 When caring for homebound clients the home health Correct answer: 1 A homebound client may be more at risk to develop a vitamin D deficiency because sunlight Recognize the clients are homebound and therefore unable to go outside. Recall the need
nurse recognizes the need to assess for deficiency of plays a part in the activation of vitamin D in the body. The other options are not related to of sunshine for Vitamin D synthesis to choose option 1.
which of the following vitamins? being indoors on a consistent basis.
1.‐ D
2.‐ A
3.‐ C
4.‐ E

3917 Because a client being treated for potassium Correct answer: 1 A client who does not respond appropriately to potassium therapy often has coexisting Critical words are potassium deficiency and additional nutrient deficiencies. Recall the role
deficiency is not responding to treatment, the nurse calcium and magnesium deficiencies as well. It is important to assess for each of these three of other electrolytes in cellular metabolism and neuromuscular transmission to be directed
should assess for which of the following additional electrolytes in order to correct the disturbance. Option 2 is incorrect because the client may to option 1.
nutrient deficiencies? be experiencing a higher level of phosphorus due to the inverse relationship that exists
between it and calcium. Options 3 and 4 are incorrect as neither is related to potassium
deficiency.
1.‐ Calcium and magnesium
2.‐ Phosphorus and vitamin D
3.‐ Chromium and selenium
4.‐ Vitamins C and E
3918 In order to promote the absorption of calcium, the Correct answer: 3 Absorption of calcium in the diet can be promoted by acidic foods. All of the other options The critical words are promotes absorption and calcium. Recognize that the foods that
nurse encourages a client taking calcium supplements would lead to a state of decreased absorption of calcium. The fiber in beans, oxalates in block or inhibit calcium are the first 3 options and eliminate them.
to take the pills with: spinach, and protein in beef would reduce the absorption of the calcium.
1.‐ Beans.
2.‐ Spinach.
3.‐ Beef.
4.‐ Orange juice.

3919 A client has been instructed on foods that interfere Correct answer: 4 Several dietary factors can reduce the absorption of iron in the body, including tannins found This is a negative response question indicating there are three correct responses and one
with the absorption of iron in the diet. The nurse in coffee or tea. All of the other options will lead to increased absorption of iron in the diet. incorrect. Recall factors which interfere with absorption of iron to be directed to option 4.
determines the teaching was effective when the client
states he will avoid taking the pills with:

1.‐ Grape juice.


2.‐ Poultry.
3.‐ Cheese.
4.‐ Coffee or tea.

3920 A premenopausal client is concerned about Correct answer: 4 A pre‐menopausal client should take 1,500 milligrams of calcium per day as a standard Critical words are premenopausal, osteoporosis, and dietary suggestions. Recognize the
developing osteoporosis in the future. What dietary recommendation. Dietary sources are recommended rather than calcium supplements factors affecting calcium absorption to eliminate option 1. Eliminate option 2 since this will
suggestions could the nurse give to help prevent this because they differ in their absorption due to bioavailability. Option 1 is incorrect because an not affect calcium and option 3 since the question specifically asks for dietary suggestions.
occurrence? increase in protein levels can lead to a decrease in absorption of calcium. Option 2 is incorrect
as additional fluids will not help to increase calcium levels. Option 3 is incorrect because
supplementation is not advised as a primary dietary treatment and can lead to the
development of other potential imbalances.
1.‐ "Increase the amount of dietary protein to strengthen existing bones."
2.‐ "Increase fluid intake to maintain hydration."
3.‐ "Take Tums™ as an additional source of calcium in the diet."
4.‐ "Increase the amount of high calcium foods to obtain a daily intake of 1,500 milligrams per day."

3921 Because a client has recognized that large doses of Correct answer: 1 High zinc levels in the body can cause development of a copper deficiency. The other choices Critical words are zinc and signs of deficiency. Note that all options contain other minerals
zinc aid in wound healing, the nurse should plan to are not affected by high levels of zinc. and recall knowledge of these. Recognize factors affecting absorption of these minerals to
assess for signs of which of the following deficiencies? be directed to option 1.

1.‐ Copper
2.‐ Fluoride
3.‐ Chromium
4.‐ Sodium

3922 A client who works in a mine presents to the Correct answer: 2 Occupational exposure to the manganese can result from dust inhalation and can have Critical words are mine and neurological problems. Note that options 2 and 3 contain the
Emergency Department with neurological problems. profound neurological effects. The other options would not be applicable to this client. word inhalation, which correlates to confinement in a mind. Recall knowledge of
The nurse suspects which mineral imbalance could be manganese sources to be directed to option 2.
a likely cause for this presentation?

1.‐ Fluoride ingestion


2.‐ Inhalation of manganese dust
3.‐ Excess copper inhalation
4.‐ Sodium deficiency due to dehydration
3923 A client informs the nurse he takes 3 grams of vitamin Correct answer: 2 Increased intake of vitamin C can increase risk for stone formation. The daily recommended Knowledge of normal requirements is necessary. Recognize 3 grams is excessive and, being
C a day to prevent "catching a cold." Which of the dose is 90 mg per day. The increased vitamin C intake would not impact the other options. water soluble, the vitamin will be excreted via the kidneys to direct you to option 2.
following client conditions would be of concern to the
nurse?
1.‐ An iron deficiency anemia
2.‐ A history of kidney stones
3.‐ Occasional anginal episodes
4.‐ A history of cholecystitis

3924 The nurse would encourage which of the following Correct answer: 1, 3, 5 Good food sources of thiamine include wheat germ, lean pork, beef, liver, and whole and Refer to the similar options of 4 and 2, including fruits. In this case the similar options are
foods to the client with a deficiency in thiamine? enriched grains, seeds, nuts, and a few vegetables. incorrect and should be eliminated.
Select all that apply.
1.‐ Sunflower and sesame seeds
2.‐ Citrus fruits
3.‐ Lean pork, beef, and liver
4.‐ Strawberries
5.‐ Peanuts

3925 A client who abuses alcohol presents with significant Correct answer: 2 Clients who abuse alcohol are prone to develop thiamin deficiency because ethanol affects Critical items in the question are alcohol and neuronal symptoms. Recall knowledge of
mental status changes and loss of balance. Which the intestinal absorption of thiamin. Wernicke‐Korsakoff syndrome is associated with a state of vitamin absorption and alcohol intake to choose option 2.
vitamin deficiency does the nurse suspect as the encephalopathy that is seen in clients with alcoholism and presents with mental status
possible cause of this presentation? changes, psychosis, and coma. Option 1 is incorrect because vitamin C deficiency is associated
with scurvy. Option 3 is incorrect because riboflavin deficiency is associated with
ariboflavinosis. Option 4 is incorrect because niacin deficiency is associated with pellagra.

1.‐ Vitamin C
2.‐ Thiamin
3.‐ Riboflavin
4.‐ Niacin

3926 The nurse determines that dietary teaching has been Correct answer: 4 Processed foods have the highest sodium content. Chocolate pudding is the only option that Recall sodium is found in processed foods and many snack foods to direct you to option 4.
effective when a client states that which of the reflects a processed food item. Meat and milk are animal products and as such have
following food items has the highest sodium content? physiological saline. Fresh fruit is lowest in sodium.

1.‐ Milk
2.‐ Fresh fruit
3.‐ Meats
4.‐ Chocolate pudding

3927 The nurse is teaching a client with type 2 diabetes Correct answer: 1 Chromium is helpful in maintaining glucose homeostasis by enhancing the activity of the Identify the unique option 1 and associate it with the core issue (mineral) in the stem of
mellitus about blood glucose control. The nurse would hormone insulin. The other options are sources of vitamins and not minerals. question.
include in the discussion that it is important to take
adequate amounts of which of the following in the diet
or in supplements?
1.‐ Chromium
2.‐ Biotin
3.‐ Niacin
4.‐ Sodium
3928 The clinic nurse is completing a history and physical Correct answer: 2 Iodine is predominately found in the thyroid gland, which secretes thyroid hormones that Recall normal thyroid function requires iodine to produce the hormone thyroxine to
examination on a client who has a history of thyroid affect the body’s metabolic rate. The other options are not associated with thyroid problems. choose option 2.
problems. The nurse assesses this client for signs of Other minerals that might be affected include sodium, potassium, iron, and calcium,
which potential mineral imbalance? depending on the underlying presentation.

1.‐ Magnesium
2.‐ Iodine
3.‐ Zinc
4.‐ Selenium

3929 During a routine history and physical examination, the Correct answer: 2 Clients who exhibit behaviors of eating nonfood items such as ice, clay, and dirt are likely to Critical phrases desire to eat, indicating a craving and also possible, indicating there is not
client reports the desire to eat large amounts of ice have iron deficiency. This odd symptom presentation is often the first indicator that there may an absolute cause and effect. Eliminate option 4 since the sodium deficiency would be
(two 8‐ounce cups) during the day. This nurse assesses be a potential problem. Option 1 is incorrect because a client who is dehydrated would be caused by the excessive water intake and not produce a craving for it. Eliminate option 3 as
the client for other manifestations of which of the more apt to drink fluids than ice chips. Option 3 is incorrect because there is nothing to ice is not a source of protein. Recall pica is food craving for nonfood items to choose option
following possible underlying clinical conditions? indicate that the client is suffering from a protein deficiency. Option 4 is incorrect because the 2.
client is not ingesting sufficient fluid to dilute the serum sodium level.

1.‐ Water deficiency due to dehydration


2.‐ Iron deficiency associated with pica
3.‐ Malnutrition due to protein deficiency
4.‐ Sodium deficiency due to ingestion of ice

3930 The nurse who is teaching a prenatal class about the Correct answer: 4 Folic acid deficiencies have been proven to cause neural tube defects in the developing fetus. Recall functions of folic acid in the body to choose option 4.
need for vitamin supplements includes that deficiency Option 1 is incorrect because high levels of folic acid can prevent identification of vitamin
of folic acid can lead to which of the following? B&lt;sub&gt;12&lt;/sub&gt; deficiency. Option 2 is incorrect because high homocysteine levels
are associated with folic acid and other B vitamin deficiencies. Option 3 is incorrect because
folic acid deficiency results in a macrocytic anemia.

1.‐ Masking of vitamin B12 deficiency


2.‐ Decreased homocysteine levels
3.‐ Microcyctic anemia
4.‐ Neural tube defects

3931 When conducting discharge teaching for a client who Correct answer: 2 Pernicious anemia is due to a clinical deficiency of intrinsic factor that prevents the Refer to the core issue in the question of the client with pernicious anemia. Pernicious
is at risk for pernicious anemia because of absorption of vitamin B&lt;sub&gt;12&lt;/sub&gt; in the body. This commonly occurs following anemia is associated with the intrinsic factor as well as vitamin
gastrectomy, the nurse reinforces that the client will gastrectomy. Thus, for treatment to be effective, vitamin B&lt;sub&gt;12&lt;/sub&gt; must be B&lt;sub&gt;12&lt;/sub&gt;itself.
require which of the following for preventive administered via injection for the rest of the client’s life. Options 1 and 3 are incorrect because
treatment? niacin is unrelated to pernicious anemia, and iron will not correct pernicious anemia. Option 4
is incorrect because riboflavin is not related to the issue of pernicious anemia and riboflavin is
administered orally.

1.‐ Niacin supplementation for lifetime


2.‐ Vitamin B12 injections for lifetime
3.‐ A single dose injection of ferrous sulfate to correct the condition
4.‐ Riboflavin injections monthly for at least a year

3932 A client taking folate supplements reports to the Correct answer: 3 This is a normal finding with folate. Increasing water intake might dilute the urine and make it Eliminate option 1 since the question relates to the intake of folate. Eliminate option 4
nurse her urine is very dark yellow and she is quite less yellow, but this response does not provide reassurance this is a normal reaction. Reducing since it is not a serious threat. Knowledge of normal folate action directs you to option 3.
concerned that. Which response by the nurse is amount of folate may not reduce the color. It does not need to be reported.
appropriate?
1.‐ "You probably need to drink more water."
2.‐ "You probably need to reduce the amount of folate you are taking."
3.‐ "Folate can cause your urine to be a deep yellow color."
4.‐ "You should report this finding to your physician."

3933 The nurse encourages a client with macrocytic Correct answer: 2 Vitamin B&lt;sub&gt;12&lt;/sub&gt; deficiency can result in the development of a macrocytic Recall knowledge of macrocytic anemia to answer to direct you to option 2.
nutritional anemia to increase intake of which of the nutritional anemia because it is necessary for red blood cell production in the body. Option 1 is
following micronutrients? incorrect because: A deficiency of calcium leads to bone demineralization. Option 3 is incorrect
because iron deficiency results in a nutritional anemia that is microcytic. Option 4 is incorrect
since vitamin B&lt;sub&gt;1&lt;/sub&gt; (thiamin) clinical deficiency results in beriberi.

1.‐ Calcium
2.‐ Vitamin B12
3.‐ Iron
4.‐ Vitamin B1

3934 The nurse tells the mother of a newborn infant who Correct answer: 1 Infants receive an injection of vitamin K to protect them against the development of Recognize that AquaMEPHYTON is vitamin K, which plays a role in the clotting process to
received an injection of phytonadione hemorrhagic disease of the newborn. Infants are born with a sterile gut and are therefore direct you to option 1.
(AquaMEPHYTON) in the delivery room that this unable to synthesize vitamin K in the small intestines. A single injection of vitamin K helps to
injection is the same as vitamin K, which will prevent introduce enough of the vitamin so that the infant is afforded protection. All of the other
development of which clinical condition? options are incorrect because this drug has no clinical impact on these clinical conditions.

1.‐ Hemorrhagic disease of the newborn


2.‐ Hepatitis B
3.‐ Skin infections in the newborn
4.‐ Dehydration

3935 While obtaining a stool specimen for a guaiac test, Correct answer: 3 Clients who take more than 1 gram of vitamin C daily may have a false negative result on The question asks what effect the vitamin C will have, implying an effect will occur, so
the nurse discovers that the client has been taking stool guaiac testing. It is important for the nurse to understand that the test results will be eliminate option 1. Eliminate option 4 as being too vague. Recall knowledge of vitamin C to
large doses of vitamin C for the past several days. The inconclusive and should be repeated in a few days once the client has stopped taking the choose option 2.
nurse considers which of the following information additional vitamin C. Option 1 is incorrect because there is a relationship between the testing
when deciding what action to take? chemical reaction. Option 4 is incorrect because the results are not valid and not open to
interpretation due to drug interactions.
1.‐ There will be no effect on the test results.
2.‐ The test may show in a false positive result.
3.‐ The test may show in a false negative result.
4.‐ The test results will be open to interpretation.

3936 A child presents with bowed legs and a pigeon breast Correct answer: 2 A clinical deficiency of vitamin D during childhood can result in structural deformities that Critical words are bowed legs and pigeon chest, indicating a bone malformation. Recall
on physical examination. The nurse suspects these result in a clinical diagnosis of rickets. The other options will not cause this type of structural knowledge of bone development to choose option 2.
deformities to be caused by a deficiency of which of deformity.
the following nutrients?
1.‐ Vitamin A
2.‐ Vitamin D
3.‐ Folic acid
4.‐ Phosphorus

3937 The nurse concludes that a client has an adequate Correct answer: 1 Baked potato and broccoli are high in potassium. All of the other options reflect food sources Option 1 includes two sources of potassium and therefore a more comprehensive answer.
understanding of potassium‐rich foods when the client that are lower in potassium.
makes which of the following selections from a
luncheon menu?
1.‐ Baked potato topped with broccoli and cheese
2.‐ Grilled cheese sandwich and pretzels
3.‐ Pasta salad and roll
4.‐ A bagel with cream cheese and dill pickle
3938 Which nutrient deficiencies might the nurse expect to Correct answer: 1 Whole grain products can contain large amounts of phytic acid (phytates), which can limit the Critical words are nutrient deficiencies and grain products. Recall specific information on
see in the client who reports eating a large number of absorption of several nutrients: calcium, zinc, iron, and magnesium. Options 2 and 3 are phytates to choose option 1.
whole grain products in the diet? incorrect because phytic acid is composed of inorganic phosphate compounds. Option 4 is
incorrect because phytic acid does not affect either sodium or chloride levels.

1.‐ Calcium, zinc, iron, and magnesium


2.‐ Calcium and phosphorus
3.‐ Calcium, vitamin D, and phosphorus
4.‐ Sodium and chloride

3939 A client diagnosed with anemia who is being treated Correct answer: 3 Anemic clients who do not respond to iron replacement therapy and present with symptoms Recognize the symptoms are indicative of peripheral neuropathy to direct you option 3.
with iron replacement therapy is not responding to associated with neuropathy are likely to be suffering from an underlying vitamin
clinical treatment and reports tingling and paresthesias B&lt;sub&gt;12&lt;/sub&gt; deficiency. Option 1 is incorrect because, even if the client were
of the extremities. The nurse questions the client to not being compliant, these types of symptoms would be due to an associated clinical
determine if the client: deficiency. Option 2 is unrelated to any information presented in the question. Option 4 is
incorrect because the use of vitamin C supplements would cause symptoms of iron overload
since it enhances the absorption of iron.
1.‐ Is compliant with iron replacement therapy.
2.‐ Has an underlying medical condition of diabetes that is complicating the course of treatment.
3.‐ May also have a vitamin B12 deficiency that may account for presentation of neuropathy symptoms.
4.‐ May be taking vitamin C supplements that may account for presentation of neuropathy symptoms.

3940 Which of the following symptoms, if seen in a client Correct answer: 2 Peptic ulcer disease can be aggravated by niacin, and the clients' complaints of heartburn are The critical words in the stem of the question are of most concern. This tells you that more
who takes large doses of over‐the‐counter niacin to indicative of GI irritation. Flushing is a side effect and although uncomfortable, it would not be than one option may be correct and that you must choose the most important option.
treat high cholesterol, would be of most concern to as serious as gastric irritation. Dryness of the mouth is not life threatening. Diarrhea would be Recall niacin causes GI irritation to direct you to option 2.
the nurse? a symptom of niacin deficiency.
1.‐ Warmth and flushing of the skin
2.‐ Heartburn and abdominal fullness
3.‐ Dryness of the mouth
4.‐ Occasional diarrhea

3941 The nurse is assessing the dietary intake of a client Correct answer: 1 Avidin is a protein found in raw eggs that binds with biotin and decreases absorption of this Critical words are frequent intake and biotin deficiency. If specific recall of biotin is poor,
with a biotin deficiency. The nurse identifies that the vitamin. Options 2 and 3 represent foods that are high in biotin; option 4 is high in vitamin C. recognize raw eggs are not advised to choose option 1.
frequent intake of which of the following foods may
have contributed to the deficiency?

1.‐ Raw eggs


2.‐ Liver
3.‐ Dark green vegetables
4.‐ Citrus fruits

3942 The nurse is instructing a client who has been started Correct answer: 2 Heme iron is considered to be the most absorbable form of iron in the body. In order to Critical words in the question are increased absorption. Recall iron absorption is increased
on iron supplements. In order to increase absorption maximize absorption of iron, meat, fish, poultry, and ascorbic acid (vitamin C) can be used. by ascorbic acid to choose option 2.
of the iron, the nurse suggests the client take them Milk would interfere with absorption of the iron, some green leafy vegetables contain oxalate,
with which of the following? which also interferes with absorption. Water would not increase absorption.

1.‐ Milk
2.‐ Orange juice
3.‐ Green leafy vegetables
4.‐ A full glass of water
3943 When caring for a client experiencing catabolism, the Correct answer: 3 The catabolism of fats provides the most energy (460 molecules of Adenosine triphosphate) Critical words are catabolism, nutrients, and energy. Recall the kilocalorie per gram yield
nurse encourages intake of which nutrient to generate because they are the most concentrated energy source (9 kilocalories per gram). Option 1 is to be directed to option 3.
the most energy? incorrect because vitamins do not yield energy although they function as coenzymes in
metabolic processes. Options 2 and 4 are incorrect as both proteins and carbohydrates provide
4 kilocalories per gram.
1.‐ Vitamins
2.‐ Proteins
3.‐ Fats
4.‐ Carbohydrates

3944 The nurse suspects a client with a history of alcohol Correct answer: 3 Vitamin B deficiency can be manifested as peripheral neuropathy, usually experienced as Critical concepts are alcohol abuse and vitamin B deficiency. Recall the symptoms of
abuse has a vitamin B deficiency when the client numbness, tingling, and pain in the extremities. Options 1 and 4 can be related to several Vitamin B deficiency to eliminate options 1 and 4. Recognize option 2 may be secondary to
reports which of the following? factors. Option 2 is frequently seen in clients with gastritis or gastroesophageal reflux disease, the alcohol use, but not to a vitamin deficiency and eliminate it also.
secondary to excessive alcohol intake.
1.‐ Frequent headaches
2.‐ Epigastric burning
3.‐ Numbness in the fingers
4.‐ Constipation

3945 A client has received dietary teaching on the nutrient Correct answer: 2 Alcohol contains 7 kilocalories/gram and provides energy from these calories, but is Critical words are nutrient content and wine. Recall the make‐up of wine to eliminate
content of various foods and beverages. The nurse considered a non‐nutrient because it is not needed by the body. Option 1 is incorrect because options 1, 3, and 4, since they are only partially correct.
determines the teaching has been effective when the alcohol provides calories but is not high in protein. Option 3 is incorrect as it is not a
client states that a glass of wine is: concentrated fat source. And option 4 is incorrect because alcohol provides energy but not
protein.
1.‐ High in protein and calories.
2.‐ Considered a non‐nutrient food.
3.‐ A source of concentrated fat calories.
4.‐ A good source of energy and protein.

3946 The nurse provides which of the following suggestions Correct answer: 1 Foods known to decrease the lower esophageal sphincter (LES) allowing reflux of gastric acid Critical terms are dietary modifications and GERD. Recall the physiology of GERD to be
for dietary modifications to reduce symptoms in a include chocolate, alcohol, coffee, tea, spearmint, and peppermint. Options 1 and 4 are directed to option 1.
client with gastroesophageal reflux disorder (GERD): incorrect because fruit juices and high fiber foods do not decrease the LES. Option 3 is
incorrect as dairy products would not need to be increased.

1.‐ "Avoid eating chocolate and peppermint."


2.‐ "Restrict your intake of fruit juices."
3.‐ "Increase your intake of dairy products."
4.‐ "Eat high‐fiber foods in moderation."

3947 A client receiving large amounts of antibiotics has Correct answer: 3 Prebiotics are supplements that stimulate bacterial growth, often in the colon, which has Critical words are antibiotics and prebiotics. Note the similarity in endings of both words,
been started on prebiotics. When the client asks what been destroyed by antibiotic therapy. Probiotics are given to boost the immune system. but differences in the prefix and eliminate option 4. Recall the function of prebiotics to be
this is for the nurse explains, “It will: Because they help to restore normal bacterial flora, they may prevent diarrhea, but this is not directed to option 3.
how they work. They do not destroy bacteria.
1.‐ Help to boost your immune system."
2.‐ Prevent the antibiotic from causing diarrhea."
3.‐ Help to restore normal bacterial in your body."
4.‐ Destroy bacteria that are not killed by the antibiotic."
3948 When caring for a client with hyperthyroidism, the Correct answer: 2 Due to the increased rate of metabolism associated with hyperthyroidism, additional calories Recall that the thyroid gland regulates cellular metabolism to be directed to option 2.
nurse will consult with a dietitian to consider the are needed to meet increased energy needs. Protein may be increased, but overall calories are
client's need for: needed for energy. Vitamin supplements and iron are not necessary provided dietary intake is
adequate.
1.‐ Increased protein.
2.‐ Increased energy.
3.‐ Vitamin supplements.
4.‐ Iron supplements.

3949 The nurse identifies which of the following clients to Correct answer: 2 Fever increases basal metabolic rate (BMR) approximately 7 percent for each one degree rise Critical words are increased metabolism and additional caloric intake. Read each option
have an increased metabolism and plans for additional in temperature. Option 1 is incorrect as metabolism is reduced in hypothyroidism. Option 3 is and systematically eliminate options 1, 3, and 4 since metabolism is reduced.
caloric intake? The client: incorrect because although the energy needs of the post‐op client are needed for wound
repair, activity is reduced and client is NPO the first day. Option 4 is incorrect since metabolism
decreases with age and fat cells require less energy than lean tissue.

1.‐ Just diagnosed with hypothyroidism.


2.‐ With a fever of 102° F.
3.‐ Who is one day post‐op colon resection.
4.‐ Who is elderly and obese.

3950 The nurse anticipates alterations in weight secondary Correct answer: 1 Hyperthyroidism causes an increase in the production of thyroxine, which in turn increases Critical words are weight and altered hormones. Read each option and recognize that the
to altered hormone levels in the client with which of metabolism and utilization of energy, often leading to weight loss. Osteoporosis does not thyroid gland secretes hormones related to metabolism to choose option 1.
the following conditions? produce a change in hormone levels and cause weight changes. Weight loss may be associated
with options 3 and 4 secondary to anorexia and vomiting, but not due to hormone imbalances.
The gall bladder stores and secretes bile for digestion and the pancreas secretes digestive
hormones. In severe cases of pancreatitis, the production of the hormone insulin may be
affected as well.
1.‐ Hyperthyroidism
2.‐ Osteoporosis
3.‐ Cholecystitis
4.‐ Pancreatitis

3951 The nurse is discussing dietary modifications with a Correct answer: 2 Foods known to produce gas, such as carbonated beverages, vegetables, milk products, Critical words are dietary modifications and IBS. Recall foods that increase gas production
client experiencing irritable bowel syndrome (IBS) and caffeine, and high fat and fat substitutes, can cause symptoms of IBS. A high protein diet is not and are irritating to the gastrointestinal tract should be restricted or avoided to be directed
offers which of the following suggestions? known to contribute to IBS. Fiber in the diet would not be contraindicated. Cholesterol is to option 2.
avoided with coronary heart disease.
1.‐ "Eat a diet high in protein."
2.‐ "Limit your intake of products containing caffeine."
3.‐ "Consume a low‐fiber diet."
4.‐ "Avoid eating foods high in cholesterol."

3952 The nurse concludes that which of the following Correct answer: 3 Ketone production in the body can be seen in response to dehydration, starvation, low Recall conditions that lead to the break down of fatty acids for energy production to be
conditions most likely contributed to the formation of carbohydrate states, and metabolic conditions such as diabetes. Option 1 is incorrect because directed to option 3.
ketones in an assigned client? the presence of ketones reflects an acidic medium because ketones are composed of keto
acids. Option 2 is incorrect as adequate carbohydrates prevent the formation of ketone bodies.
Option 4 is incorrect because increased fluid intake decreases the likelihood of ketone
production in the body.
1.‐ Metabolic alkalosis
2.‐ Adequate carbohydrates in the diet
3.‐ Dehydration
4.‐ Increased fluid intake
3953 The nurse would explain to a client who underwent Correct answer: 4 Meals that are high in carbohydrates, such as the meal in option 4, promote rapid gastric Recall gastric resection reduces size of stomach and review physiology of food
gastric resection that which of the following meals is emptying. The other options are associated with increased transit time because they contain metabolism. Note CHO will increase osmotic load, which will promote peristalsis, to choose
most likely to cause rapid emptying of the stomach? sources of protein or fat, and meals of these types remain in the stomach for a longer time. option 4.

1.‐ Broiled steak and green beans


2.‐ Fried chicken and creamed potatoes
3.‐ Baked fish and fresh carrots
4.‐ Pasta with broccoli and garlic bread sticks

3954 The nurse is caring for a client at risk for short bowel Correct answer: 3 The increased surface area of the microvilli on the brush border of the small intestine favors Note the nurse is explaining the function of the small intestine in relation to its absorption
syndrome. The nurse would choose which of the the process of absorption by increasing the surface contact area. Option 1 is incorrect since function and needs to explain it in lay terms. Eliminate options 2 and 4 as incorrect; option
following statements to explain the role of the small there is specialization in the GI tract that allows for specific nutrient release in order to 1 does not provide a clear explanation.
intestine in absorption of nutrients? maximize absorption. Option 2 is incorrect because the small intestine has an alkaline
environment. Option 4 is incorrect because dietary fiber consists of undigested material that
usually enters the large intestine for bacterial degradation and elimination as feces.

1.‐ "Nutrients are delivered to the small intestine in a rapid manner to facilitate absorption."
2.‐ "The acidic environment of the small intestine enhances digestive enzyme function."
3.‐ "Increased surface area of the microvilli on the lining of the intestine favors absorption of nutrients."
4.‐ "The small intestine is able to facilitate the absorption of dietary fibers."

3955 The nurse identifies the client who has which of the Correct answer: 4 Gastric surgical resection can cause an alteration in the absorption of nutrients due to altered Eliminate options 1 and 3 since these conditions would not affect nutrient absorption.
following to be most at risk for malabsorption of surface area, thereby delaying entry of food from the stomach to the intestines (i.e., Note option 4 involves the gastric area to direct you to it.
nutrients? decreasing absorption and digestion). Option 1 is incorrect because hypoactive bowel sounds
might or might not affect absorption. Option 2 would affect fluid reabsorption in the large
intestine, but most nutrients would have been absorbed before entering the large intestine.
Option 3 may affect elimination patterns but does not affect the absorption of nutrients.

1.‐ Hypoactive bowel sounds


2.‐ Colon resection
3.‐ Rectal polyps
4.‐ Gastric surgical resection

3956 The nurse instructs the client who develops heartburn Correct answer: 3 Ingestion of chocolate can reduce lower esophageal sphincter (LES) pressure, leading to reflux The concept of the question is identification of foods that will complicate GERD. Recall the
as a result of gastroesophageal reflux disease (GERD) and clinical symptoms of GERD. All of the other foods do not affect LES pressure. physiology of this to choose option 3.
to avoid which of the following foods?

1.‐ Lettuce
2.‐ Eggs
3.‐ Chocolate
4.‐ Butterscotch

3957 Which of the following food items should the nurse Correct answer: 3 Antibiotic therapy can lead to destruction of normal intestinal flora that is used to synthesize Critical words are intestinal synthesis of vitamin K and antibiotic therapy. Remember that
recommend to facilitate intestinal synthesis of vitamin vitamin K. Yogurt contains bacteria that help to promote intestinal synthesis. The other options yogurt is a probiotic.
K for a client receiving antibiotic therapy? do not contain necessary bacteria.

1.‐ Eggs
2.‐ Wheat germ
3.‐ Yogurt
4.‐ Fish
3958 Which of the following foods should the nurse Correct answer: 2 Celiac disease is a malabsorption disorder affecting the small intestine in which there is a Recall knowledge of celiac disease and foods containing gluten to direct you to option 2.
remove from the lunch tray of a client diagnosed with problem with the ingestion of gluten, a protein normally found in grain products such as
celiac disease? wheat, rye, oats, or barley. The other options reflect substances that do not contain gluten and
should not pose problems for a client with this disorder.
1.‐ Butter
2.‐ Beef barley soup
3.‐ Fresh yellow squash
4.‐ Coffee

3959 When caring for a client with a history of alcoholism, Correct answer: 1 Because of deficient nutritional intake, the client with alcoholism frequently has deficiencies Key words are alcoholism, lab values, and deficiencies. Note all options have two answers
the nurse checks laboratory values, anticipating of many nutrients. Electrolytes that are particularly affected are magnesium and phosphorus, and both must be correct for the option to be correct. Recall nutritional intake deficiencies
deficiencies of which of the following micronutrients? since they are utilized in maintaining energy production and numerous enzyme reactions. associated with alcohol intake to direct you to option 1.

1.‐ Magnesium and phosphorus


2.‐ Sodium and potassium
3.‐ Magnesium and chloride
4.‐ Calcium and potassium

3960 The nurse anticipates that which of the following Correct answer: 2 Clients who are in shock are likely to form lactic acid as the body reverts to anaerobic Critical words are metabolic processes and hypovolemic shock. Recall concepts of shock
metabolic processes will most likely to occur in a client metabolism in order to maintain homeostasis. Options 1 and 2 are incorrect because these and energy requirements to direct you to option 2.
recently admitted in hypovolemic shock? represent metabolic processes that require oxygen (aerobic metabolism). Option 4 is incorrect
because transamination involves the exchange of amine groups in amino acids and reflects an
anabolic process.
1.‐ Gluconeogenesis
2.‐ Anaerobic metabolism
3.‐ Glycogenesis
4.‐ Transamination

3961 A nurse teaching a group of 13‐year‐old students Correct answer: 2 Vitamins function as coenzymes in many of the metabolic processes in the body to facilitate Eliminate options 1 and 3 since both are similar concepts. Eliminate option 4 since this
about the role of vitamins in the metabolism of energy release. Options 1 and 3 are incorrect since by themselves vitamins do not provide practice would be dangerous with certain vitamins.
nutrients would make which of the following energy or supply additional calories. Option 4 is incorrect—vitamins should not be consumed
statements? in large quantities because they can reach toxic levels (fat‐soluble vitamins).

1.‐ "Vitamins are needed because they provide additional energy sources."
2.‐ "Vitamins are needed because they participate as enzymes in metabolic processes."
3.‐ "Vitamins are needed to supply additional calories."
4.‐ "Vitamins are needed in large quantities in order to prevent oxidation of vital nutrients."

3962 A client who has a history of alcoholism is receiving Correct answer: 1 Clients with alcoholism are usually deficient in thiamine, which is needed for carbohydrate Critical words are alcoholism and thiamine. Recall the role of thiamine in CHO metabolism,
intravenous therapy with 5% dextrose in 0.45% sodium metabolism. Administration of glucose solutions can precipitate symptoms of Wernicke's and recognize the client is receiving a dextrose solution to associate the symptoms of
chloride and a regular diet. When the client refuses the encephalopathy, characterized by disorientation, memory difficulties, diplopia, ataxia, and option 1 to Wernicke's encephalopathy.
ordered thiamine supplement, the nurse should nystagmus. The other symptoms may be experienced for other reasons in the alcoholic client,
monitor for which potential complication? but option 1 is most indicative of the thiamine deficiency.

1.‐ Confusion and ataxia


2.‐ Abdominal cramps and diarrhea
3.‐ Headaches and nausea
4.‐ Numbness and tingling in the extremitites.
3963 The client with Crohn's disease is not following Correct answer: 1, 2, 3 Crohn's disease can occur at any location in the GI tract, and the most common clinical Refer to similarities of indicators associated with the GI tract. Uremia is associated with
dietary recommendations. The nurse would expect to manifestations are diarrhea, abdominal pain, weight loss, and fatigue. The disease is not renal disease and so cannot be a correct option. Constipation is opposite of the clinical
assess which of the following clinical manifestations in characterized by constipation or uremia. The amount of bulk or fiber in the diet is reduced findings in Crohn's disease and can thus be eliminated also.
this client? Select all that apply. during acute episodes to control the diarrhea.

1.‐ Diarrhea
2.‐ Abdominal pain
3.‐ Weight loss
4.‐ Uremia
5.‐ Constipation

3964 The nurse would make which of the following Correct answer: 2 The inclusion of dietary fibers helps to add fecal weight (soluble) and bulking (insoluble), Note that each option contains the word fiber, so read each option carefully and
statements in explaining how dietary fiber aids in the which assists with elimination patterns. Option 1 is incorrect because fiber does not provide a systematically eliminate options 1, 3, and 4 as incorrect. Choose option 2 because it is
processes of digestion, transport, and absorption? coating effect to the gastric lining. Option 3 is incorrect because dietary fiber can affect comprehensive for processes of digestion.
digestion of nutrients, especially when consumed in large amounts. Option 4 is incorrect
because there is no evidence to support the practice of taking mineral oil with dietary fiber to
facilitate digestion, transport, and absorption.

1.‐ "Fiber helps to coat the gastric lining of the stomach in order to decrease the acidic environment."
2.‐ "Soluble fiber adds weight to feces, and insoluble fiber acts as a bulking agent to assist in the elimination process."
3.‐ "A large amount of fiber in the diet has no effect on the digestion of nutrients."
4.‐ "Dietary fiber must be taken with mineral oil to be effective in facilitating nutrient digestion, transport, and absorption."

3965 Which of the following clients is at greatest risk for Correct answer: 4 A client with TMJ disorder is at risk for developing swallowing problems due to pain Refer to options 3 and 4, which have similarities and association to the oral cavity. Choose
developing a digestive problem related to the oral experienced from incorrect jaw alignment. Option 1 is incorrect because this client would have option 4 because it is more comprehensive.
cavity? problems related to the esophagus. Option 2 is incorrect because "fast food" ingestion does
not cause digestive problems. Option 3 is incorrect because a client with wisdom teeth
removal may have initial discomfort and swallowing problems, but they usually resolve as
healing occurs. Clients with TMJ disorder are likely to have acute exacerbations that can
become problematic, affecting dietary intake and leading to a significant complication such as
weight loss.
1.‐ A 48‐year‐old male client with a history of gastric reflux
2.‐ An 18‐year‐old college student eating "fast food" for lunch
3.‐ A 20‐year‐old client who has past medical history of wisdom teeth removal
4.‐ A 50‐year‐old client with a history of temporomandibular joint (TMJ) disorder

3966 The nurse is performing a gastrointestinal assessment Correct answer: 1 Steatorrhea (bulky, foul‐smelling stool) is a common finding related to malabsorption of fats. The core concept is malabsorption of fat. Recall physiology of bile action on stool to direct
on a client. Which of the following clinical findings Option 2 is incorrect because gastric emptying time would be decreased as nutrients pass you to option 1. If you had difficulty with the question, note similarity in ending of word
would the nurse associate with malabsorption of fat? more quickly because they cannot be absorbed. Option 3 is incorrect because diarrhea and steatorrhea and diarrhea.
steatorrhea are more common findings. Option 4 is a nonspecific finding that is independent of
fat malabsorption and may be related to other factors such as anemia and decreased
oxygenation states.
1.‐ Steatorrhea
2.‐ Increased gastric emptying
3.‐ Constipation
4.‐ Pallor
3967 What client statement would indicate that dietary Correct answer: 2 Eating late at night can lead to development of GI symptoms as increased presence of food in Associate the client action (lying down) with definition of GERD (backflow of acidic gastric
teaching has been effective in a client with the stomach leads to an increase in acid secretion. If the client lies down with a full stomach, it juices).
gastroesophageal reflux disease (GERD) regarding further causes gastric distention and aggravation of clinical symptoms. Option 1 is incorrect
dietary practices to facilitate digestion? because use of a daily antacid can cause alterations in digestion, transport, and absorption of
nutrients that can further increase GI discomfort. Option 3 is incorrect because there is
nothing to indicate that this client has a problem with swallowing or dentition. Option 4 is
incorrect because water and fiber are necessary in the diet to facilitate adequate elimination
patterns.
1.‐ "I will continue to use antacids on a daily basis."
2.‐ "I will limit late‐night snacking to prevent GI symptoms."
3.‐ "I will cut up my food into very small pieces to make it easier to chew."
4.‐ "I will limit the amount of fluid and fiber in my diet."

3968 The nurse anticipates that which of the following Correct answer: 4 Catabolism refers to processes involving the release of energy in order to restore body Option 4 is more comprehensive than the other options offered.
clients admitted to the nursing unit is in the most dynamics and is seen in clients undergoing acute periods of starvation and/or traumatic injury.
severe catabolic pattern of metabolism? The client in option 4 has the most severe triggering condition for catabolism. The client in
option 1 would undergo catabolism, but this is a short‐term event compared to option 4. The
clients in options 2 and 3 are in an anabolic pattern of metabolism representing growth states
and new tissue development.
1.‐ A 12‐year‐old with acute appendicitis
2.‐ A 22‐year‐old pregnant client with slight painless vaginal bleeding
3.‐ A client who is 5 days postoperative for femoropopliteal bypass
4.‐ A client who was admitted 3 days ago with multiple trauma

3969 The nurse expects that a client experiencing which Correct answer: 2 Lactic acidosis is associated with anaerobic metabolism All of the other options represent The core issue of the question is knowledge of the nature of lactic acidosis as a bodily
metabolic process will develop lactic acidasis? metabolic processes during which oxygen is required. process. Analyze each option in terms of its use of oxygen.

1.‐ Glycolysis
2.‐ Anaerobic metabolism
3.‐ Gluconeogenesis
4.‐ Oxidative phosphorylation

3970 A client with a history of lactose intolerance confides Correct answer: 1 Lactose intolerant individuals usually lack the enzyme lactase, necessary to break down milk Recall function of lactase to break down milk sugars and inability of this will result in
to the nurse she consumed a large amount of ice sugars. Lack of the enzyme causes abdominal cramps, diarrhea, and gas formation. The other formation of gas. Systematically eliminate options 1 and 3 as not due to abdominal gas
cream at a social event. The nurse can expect the symptoms would not be caused by lack of lactase. formation. Eliminate option 2 since it is associated with lack of bile and pancreatic enzymes.
client to experience:
1.‐ Bloating and diarrhea
2.‐ Constipation for 2–3 days
3.‐ Foul smelling, bulky stools
4.‐ A headache

3971 A client's urine tests positive for presence of kettle Correct answer: 4 Ketone bodies are formed in response to the incomplete breakdown of fatty acids, when Recall ketone bodies are formed from fatty acid breakdown and recall conditions when
bodies. The nurse concludes that this test result lipids are used as an alternate energy source in response to low ingestion of CHOs, high body does this to direct you to option 4.
indicates which of the following? protein or high fat intake, or fasting states. The presence of ketone bodies is associated with
acidotic states.
1.‐ Excessive intake of simple sugars in diet
2.‐ Decreased lipids in the diet
3.‐ An alkaloid condition in the body
4.‐ Inadequate carbohydrate intake in the diet
3972 The nurse would explain to a client with celiac disease Correct answer: 3 Celiac disease is due to inability to digest gluten sources, such as found in wheat, rye, oats, Refer to the sources of gluten in option 3.
that which foods are best to avoid to reduce flares of and barley products. All other options have an unrelated source of gluten.
the disease?
1.‐ Wheat, corn, and rice
2.‐ Barley, soybeans, and corn
3.‐ Wheat, barley, and rye
4.‐ Rice, oats, wheat

3973 A client continues to be an avid runner during her Correct answer: 3 Dizziness is a sign of overexertion and as such should alert the client to alter her present Key concepts are pregnancy and exercise intolerance. Recall the normal response to
pregnancy. The nurse realizes that which of the exercise regimen. Option 1 is incorrect because decreased, not increased, fetal movement is exercise to be directed to option 3 as being a danger sign.
following symptoms indicates the client needs to alter sign of overexertion due to decreased oxygen supply. Option 2 is incorrect as headache may
the intensity of her exercise routine? occur for many other reasons and is not necessarily correlated with overexertion. Option 4 is
incorrect because diaphoresis is a normal response to exercise.

1.‐ Increased fetal movement


2.‐ Headache
3.‐ Dizziness
4.‐ Increased diaphoresis

3974 A 23‐year‐old postpartum client is breastfeeding. The Correct answer: 2 Protein needs increase by 20 grams during lactation, and therefore protein‐rich foods should Key words are breastfeeding and dietary teaching. Eliminate options 3 and 4 as incorrect
nurse has completed teaching about dietary changes be consumed. Option 1 is incorrect because caloric needs only increase by 500 calories during information. Eliminate option 1 since this is too high.
and is evaluating teaching effectiveness. Which of the lactation, and a 750 calorie increase would result in weight gain. Option 3 is incorrect as it is
following statements by the client indicates the unwise to decrease calories during the pregnancy period because the mother's nutritional
teaching was effective? stores will be depleted. Option 4 is incorrect because if a diet is adequate in calcium, then
supplementation is not needed; the requirement is the same for similar‐aged lactating and non‐
lactating women.
1.‐ "I need to increase my caloric intake by about 750 calories per day."
2.‐ "I need to increase my consumption of protein‐rich foods such as legumes."
3.‐ "Now is the time to decrease my calories so I can lose my pregnancy weight."
4.‐ "My calcium consumption must increase to be able to produce the milk."

3975 A client has chosen to formula feed her infant. The Correct answer: 1 Formula must be prepared according to manufacturer's directions to ensure that adequate The key concept is formula feeding. This question is a negative response type indicating
nurse realizes that which of the following statements nutrients are received. All of the other options reflect adequate client teaching. that three options are correct and the incorrect response is the correct answer. Recall
by the client indicates further teaching is needed? knowledge of formula feeding to choose option 1.

1.‐ "I can make the dry powder formula last longer by adding more water."
2.‐ "I can expect my baby to eat about six to eight times per day."
3.‐ "My baby will still be getting adequate nutrition with the formula."
4.‐ "I need know hunger cues to prevent overfeeding. Formula‐fed infants tend to gain weight faster."

3976 A full‐term infant is being formula fed. At the one‐ Correct answer: 1 Most infants feed 6–8 times per day and consume 2–4 ounces per feeding. This infant does Note that the question requires analysis of the amount of formula necessary for a week‐
week checkup, the infant's mother reports to the not seem to be feeding often enough. A likely explanation is a sleepy infant who is not being old infant. Recall the amount recommended at this age to be directed to option 1.
nurse that the infant consumes 10 ounces per day in roused frequently enough for eating. Options 2 and 3 are incorrect because the intake pattern
about four feedings. Which of the following is not acceptable. Option 4 is incorrect as it presumes that there is a more serious problem
statements by the nurse is most appropriate? than is reflected by the information provided.

1.‐ "Your infant is consuming less than I would expect at this age. Do you wake the baby for feedings?"
2.‐ "Your infant seems to be feeding well. Do you have any concerns?"
3.‐ "Your infant is consuming more than I would expect at this age. Let's weigh your infant to determine weight gain."
4.‐ "I think your infant may have a metabolism problem. We need to run some tests."
3977 The nurse has completed health histories on several Correct answer: 1 A young pregnant adolescent is at highest risk because this age is associated with rapid Read each option carefully and analyze the risk factors for adequate nutritional intake.
pregnant clients. Which of the following clients is at growth and increased nutritional demands that are further exacerbated by pregnancy. The Recognize the high risk of an adolescent to be directed to option 1.
highest risk for nutritional alterations and requires clients in the other options have concerns but they are not as immediate. The 27‐year‐old will
immediate nutritional teaching? have to be assessed for signs of continued altered eating habits, but the choice reflects a past
history. The 37‐year‐old is at risk because of age versus nutrition and the fact that this is her
first pregnancy. The 23‐year‐old will have to be monitored because she is already overweight
prior to the demands of pregnancy.

1.‐ A 13‐year‐old living with parents


2.‐ A 27‐year‐old with a past history of bulimia nervosa in college
3.‐ A 37‐year‐old in her first pregnancy
4.‐ A 23‐year‐old who was 10 pounds overweight before pregnancy

3978 A pregnant client has undergone a glucose tolerance Correct answer: 3 Values that meet or exceed two or more of the designated glucose parameters are Recall the norms of glucose tolerance testing in pregnancy to be directed to option 3.
test. The results are 105 milligrams per deciliter ‐ considered diagnostic for GDM. This client exceeds parameters in all four levels (i.e., greater
fasting, 200‐1 hour, 175‐2 hours, and 160‐3 hours. How than or equal to 95, greater than or equal to 80, greater than or equal to 155, greater than or
would the nurse interpret the meaning of these equal to 140). The other options are incorrect because they do not indicate the client has GDM
results? and there is nothing to indicate that the test results are not valid.

1.‐ Glucose levels are within normal limits; therefore, the client does not have gestational diabetes mellitus (GDM).
2.‐ Glucose levels indicate that the client is hypoglycemic and dietary alterations should be instituted.
3.‐ Glucose levels indicate GDM and treatment should be initiated.
4.‐ Glucose levels are inconclusive and the test needs to be repeated.

3979 A client has pregnancy‐induced hypertension and is Correct answer: 3 Protein increases are recommended due to losses in urine, and sodium intake is reduced Critical words are pre‐eclamptic and dietary modifications. Recall the need to alter protein
considered pre‐eclamptic. The nurse has discussed slightly due to potential edema formation. Options 2 and 4 are incorrect because fluids should and sodium both with this condition to be directed to option 3.
dietary modifications and wishes to evaluate teaching. not be restricted in pregnancy due to a risk of dehydration. Option 1 is incorrect as proteins
Which of the statements by the client indicates should never be restricted because of losses in urine, and a proportional increase in
teaching was effective? carbohydrates may result in increased weight.
1.‐ "I need to decrease protein to 10 percent of my diet and increase carbohydrates by 20 percent."
2.‐ "I must restrict fluids to one liter per day and limit sodium to three grams."
3.‐ "I should increase protein to 1.5 grams per kilogram per day and limit sodium intake to 6 grams per day or less."
4.‐ "I need to restrict fluids to one liter per day and increase protein to 1.5 grams per kilogram per day."

3980 A pregnant client has a seizure disorder. The nurse Correct answer: 4 Some anticonvulsants interfere with vitamin D production, increasing the risk of deficiency. Critical terms are seizure disorder, pregnancy, and medication interactions. Eliminate
realizes which of the following changes will most likely Therefore, supplementation may be indicated. Option 1 is incorrect as changing drugs is not options 1 and 3 since it would be unwise to alter the seizure medication at this time. Recall
need to be made in the client's medication and dietary usually wise if the disorder is well‐controlled. Option 2 is incorrect because even though folate the role of vitamin K in anticonvulsant therapy to choose option 4.
supplement regime as a result of drug interactions? has been known to alter anticonvulsant uptake, the risk of neural tube defects outweighs this
difficulty so supplementation is still indicated. Option 3 is incorrect since women may
experience increases in seizure activity during the pregnancy so decreasing anticonvulsant
doses would not be appropriate.

1.‐ Change current anticonvulsant drug(s)


2.‐ Eliminate folate supplementation
3.‐ Decrease anticonvulsant doses
4.‐ Add vitamin D supplementation

3981 When assessing an infant diagnosed with failure to Correct answer: 5 Weight below the 5th percentile for height is diagnostic for FTT. Recall the parameters for FTT.
thrive (FTT), the nurse would expect to find the infant's
weight is below the ___________th percentile for age
and gender. Write a numerical answer.
3982 The nurse is teaching a mother about feeding her Correct answer: 1 Young children should not be given hard, round foods that do not dissolve easily, such as hot The critical word is toddler. Note that the question is a negative response item indicating
toddler. Which of the following statements by the dogs, because the risk of choking is too high. All of the other options contain foods that are three options are correct information, and one is the incorrect information nurse is
mother indicates further teaching is required? appropriate or prepared in such a way to make them appropriate and do not pose safety risks. evaluating.

1.‐ "She just loves hot dogs. She can hold them in her fingers to eat them."
2.‐ "I need to cook her vegetables and cut them into small pieces."
3.‐ "Graham crackers and milk would be a nice snack."
4.‐ "Cheese puffs would be a better choice than popcorn at this stage."

3983 A moderately overweight client who is 4 weeks Correct answer: 4 Weight reduction is never indicated during pregnancy. Moderate weight gains of 15–25 Critical words are overweight, 4 weeks pregnant, and highest priority. Recall concepts of
pregnant is at the first prenatal visit. The nurse should pounds are recommended for overweight clients. Option 1 is incorrect since an in increase in nutritional needs in the first trimester of pregnancy to choose option 4.
give highest priority to which of the following calories in the first trimester is not indicated, whereas in the second trimester and third,
nutritional information during teaching? caloric needs are increased by 300. Option 2 is incorrect because weight reduction is not
indicated during pregnancy. Option 3 is incorrect because all pregnant women are expected to
receive supplements (prenatal vitamins) to ensure adequate folic acid is obtained, regardless
of dietary intake.
1.‐ "Add an additional 300 calories to your daily diet now to ensure adequate nutrients are available to support your developing fetus."
2.‐ "You will need to follow a weight reduction diet, no lower than 1200 calories and adequate in essential nutrients."
3.‐ "As long as your diet is well balanced in all nutrients, a vitamin/mineral supplement will not be required."
4.‐ "You must eat a nutritionally sound diet. Pregnancy is not the time to lose weight."

3984 A 2‐day postpartum client informs the nurse, Correct answer: 3 Colostrum is produced before milk until about 4 days postpartum. It is yellow, rich in Note that the client is only 2 days postpartum. Recall colostrum is secreted in the first few
“Something is wrong with my milk; it is yellow.” Which nutrients, and should be consumed by the baby. Option 1 is incorrect because there is nothing days to direct you to option 3.
of the following statements by the nurse is most to suggest that the client has a breast infection (temperature or breast soreness). Option 2 is
appropriate? incorrect because dehydration may be associated with letdown of milk, not color. Option 4 is
incorrect. Foods can alter a milk's taste, but not color.

1.‐ "You probably have a breast infection and will need to stop breastfeeding."
2.‐ "You are most likely dehydrated. Increase your consumption of caffeine‐free fluids so the milk will not be so thick."
3.‐ "The yellow fluid is normal and is called colostrum, a precursor to milk. It is full of nutrients and wonderful for the baby to drink."
4.‐ "What have you been eating? Some foods can discolor the milk."

3985 The nurse has completed teaching the client about Correct answer: 4 Single‐grain infant cereals are recommended first because they are easily digestible and have Recall need to begin solid food introduction with a cereal to choose option 4.
introducing solid foods to an infant. The nurse added iron content. Option 3 is incorrect because yogurt is a milk product, and introduction
determines teaching has been effective when the should be delayed until 12 months because of the risk of milk allergy. Options 1 and 2 are
mother identifies the first solid food she will introduce incorrect because fruits and vegetables are usually given following the introduction of cereals.
is:
1.‐ Pureed canned squash
2.‐ Pureed apples
3.‐ Yogurt
4.‐ Infant rice cereal

3986 When caring for a pregnant client with congenital Correct answer: 2 Caffeine may increase heart rate that is already stressed due to pregnancy. Sodium may Theoretical words heart disease. Recall the significance of caffeine and sodium on cardiac
heart disease, the nurse plans for which alterations in cause fluid retention. Both may need to be restricted. The other answers are incorrect because function to direct you to option 2.
the client's diet during pregnancy? calories, fat, and protein are not usually decreased due to the risk of nutrient deficiencies.

1.‐ Reduced calories and reduced fat


2.‐ Caffeine and sodium restrictions
3.‐ Decreased protein and increased complex carbohydrates
4.‐ Fluid restriction and reduced calories
3987 An adult female client has been treated for iron Correct answer: 2 A sign of anemia is pale conjunctiva. If resolved, the conjunctiva should be pinker. The other Critical words are iron deficiency and effectiveness. Recall symptoms of anemia to direct
deficiency anemia. To evaluate the effectiveness of the symptoms are not specific to iron deficiency and are usually associated with other vitamin or you to option 2.
treatment, the nurse would assess for resolution of mineral deficiencies.
which of the following symptoms?
1.‐ Dermatitis
2.‐ Pale conjunctiva
3.‐ Bleeding gums
4.‐ Hair pigment changes

3988 An infant with congenital hypothyroidism is placed on Correct answer: 3 At the beginning of therapy, it is essential that parents understand its importance. Other Critical words are most essential, indicating some or all of the options are correct, but one
thyroid replacement therapy. Which of the following information is less of a priority. The other options contain information about administration, is much more important. Note the condition is congenital, indicating it will be life long to
information is the most essential information for the assessing effectiveness, and side effects, which can be explained later. direct you to option 3.
nurse to include in discharge teaching with the
parents?
1.‐ The supplement may be mixed with formula.
2.‐ Notify the physician if the infant continues to be excessively sleepy.
3.‐ This replacement must be taken for the child's entire life to ensure normal growth and development.
4.‐ Notify the physician if the child becomes excessively irritable and diaphoretic.

3989 The nurse interprets that a premature infant is at Correct answer: 3 Vitamin E is a fat‐soluble vitamin, and the infant is at greatest risk of deficiencies due to The core issue of the question is the ability to discriminate between fat‐soluble and water‐
greatest risk for inadequate intake of which of the impaired fat absorption. The other nutrients are also at risk for deficiency but usually because soluble vitamins. Note correlation of altered fat absorption in question to the only option
following vitamins because of alterations in fat of inadequate stores. All of the other options represent water‐soluble vitamins. that is a fat‐soluble vitamin.
absorption?
1.‐ B complex
2.‐ Vitamin C
3.‐ Vitamin E
4.‐ Folic acid

3990 An adult client being started on sodium warfarin Correct answer: 1 A client receiving anticoagulant therapy should not take additional supplementation of The core issue of the question is an interaction between oral anticoagulant therapy and
(Coumadin) had previously been taking over‐the‐ vitamin K, either through dietary intake or supplemental therapies, because vitamin K vitamin K. Recall a correlation of vitamin K to clotting to direct you to option 1.
counter vitamin K supplements. The nurse teaches the antagonizes sodium warfarin (Coumadin). Option 2 is not advised because increased dietary
client to do which of the following at this time? intake can also influence this drug, resulting in altered clotting times. Option 3 is incorrect for
the reasons stated above. While it is important to discuss any supplemental therapy with the
healthcare provider (option 4), the time delay would place the client at risk for complications
related to anticoagulant therapy.

1.‐ Discontinue taking the supplements.


2.‐ Increase the intake of dietary sources of vitamin K so no additional supplementation will be needed.
3.‐ Take vitamin K supplements every other day while on anticoagulant therapy.
4.‐ Discuss this issue with the healthcare provider at the next scheduled appointment.

3991 A regular diet has been resumed for a client following Correct answer: 2 Nutrient stores exhausted during major trauma include protein, B complex vitamins, zinc, and Critical words are major traumatic injury and replenish nutrients. Recall significance of
a major traumatic injury. The nurse selects from the vitamins A and C. Option 2 is rich in all these nutrients. All of the other options reflect a lack of protein and vitamins for wound healing to direct you to option 2.
diet menu which of the following meals that would be specific nutrients needed to replenish stores.
most appropriate for the client?

1.‐ Vegetable lasagna, bibb lettuce with dressing, white roll, and a slice of pound cake
2.‐ Chicken breast, brown rice, broccoli, and fresh orange slices
3.‐ Fried codfish fillet, macaroni and cheese, peas, and flavored gelatin
4.‐ Roast beef, mashed potatoes, corn, and ice cream
3992 The nurse has completed a comprehensive health Correct answer: 2 The most likely explanation is the death of the spouse. All of the other choices reflect factors Critical words are recently widowed. Recognize all options contain factors contributing to
history on an 80‐year‐old recently widowed client who that would have been present previously; the client had no problems until the spouse died. poor dietary habits of the elderly, but option 2 has a connection to the loss of spouse.
lost 15 pounds in 2 months. The nurse anticipates Depression and loneliness have been documented as major causes of nutrient alterations in
which of the following is the most likely explanation? the elderly.

1.‐ Reliance on fixed income


2.‐ Depression and sense of loss over spouse's death
3.‐ Use of limited funds for medications
4.‐ Limited opportunities for social eating

3993 As prophylaxis against neural tube defects (NTD), the Correct answer: 0.4 Due to strong correlation between NTD and folic acid deficiency, a 0.4 mg/day supplement is Note the question addresses the recommended dose for someone trying to conceive,
nurse should recommend that a client planning to recommended for client considering pregnancy. Note that a dose of 0.6 mg/day is which may be different than for someone already pregnant. Recall specific knowledge of
conceive for the first time should take in ____ recommended for pregnant women; 4 mg/day is recommended for short‐term dosing for folate requirements to type in the correct response.
milligrams of folic acid daily. Write a numerical women with past history of pregnancy with NTD.
answer.

3994 A new breastfeeding client is having difficulty with Correct answer: 2 A poorly positioned infant can cause trauma to the nipple. Although nipples can become A critical word in the question is most, indicating all or some of the options may be
sore nipples. The nurse assesses the client for which of infected, this is not the most common cause; breasts should be cleansed after feeding; letting correct, but one choice is the best. Eliminate option 1 since this is not frequently seen.
the following since it is the most probable cause of this breast milk dry on nipples has been an effective treatment for sore nipples due to high fat Eliminate options 3 and 4 since they are also not a likely cause.
problem? content and anti‐infective substances in breast milk.
1.‐ Infection of the nipples
2.‐ Infant is poorly positioned
3.‐ Not cleansing breasts after feeding
4.‐ Allowing breast milk to dry on nipples

3995 A pregnant client is at the first prenatal visit. The Correct answer: 2 According to BMI criteria, the client would be considered overweight and advised to maintain Critical words are prenatal and BMI of 27.5. Recognize this BMI is high to direct you to
nurse determines the client’s body mass index (BMI) is weight gain between 15–25 pounds. A less than 15‐pound gain is the restriction for obese option 2.
27.5. The nurse identifies the recommended weight clients; weight gains of 25 pounds or greater are for clients of normal weight or who are
gain for this client is: underweight.
1.‐ Less than or equal to 15 pounds.
2.‐ 15–25 pounds.
3.‐ 25–35 pounds.
4.‐ 28–40 pounds.

3996 An elderly client states that fruits in any form cause Correct answer: 1 Dark, leafy vegetables such as spinach are an alternative source of vitamin C. Corn is a good Key words are most appropriate, alternative, and vitamin C. Note all options are
diarrhea. Which of the following foods would the source of fiber but not vitamin C; sweet potatoes are rich in vitamin A; celery provides water vegetables and recall those with a high vitamin C content to choose option 1.
nurse suggest to be the most appropriate alternative and fiber.
to ensure the client receives adequate vitamin C?

1.‐ Spinach
2.‐ Corn
3.‐ Sweet potatoes
4.‐ Celery

3997 A new postpartum client has been told that her infant Correct answer: 3 PKU is a genetic disorder that reflects a problem with the metabolism of phenyalanine (amino A critical phrase is adequate nutritional management. Recall knowledge of PKU and need
has phenylketonuria (PKU). Which of the following acid). A special diet should be followed that restricts/limits the intake of this amino acid in for lifelong therapy to direct you to option 3.
instructions should the nurse provide to the mother to order to avoid potential metabolic complications. The other options do not acknowledge that
ensure adequate nutritional management for this this inborn error of metabolism has lifelong consequences.
newborn?
1.‐ "Feed the baby a wide variety of foods when she starts eating solid foods."
2.‐ "A special PKU diet will be necessary during the first year."
3.‐ "Your baby must follow a diet that restricts the amount of phenylalanine."
4.‐ "Your baby will grow out of this and can be managed using standard nutritional support."

3998 The nurse is teaching a client how to introduce solids Correct answer: 4 Foods should be introduced singly to identify possible allergies. Combination foods, such as Read each option carefully and systematically eliminate options 1, 2, and 3 since they do
to an infant. Which of the following statements by the those that would be served to the family, are not advised due to difficulty identifying allergies, not answer the question or give incorrect advice.
client indicates that the client has correct and some table foods may have high‐sodium substances not tolerated well by an infant. Cereal
understanding? should not be placed in bottle because it deprives the infant of an opportunity to develop
chewing muscles. Formula should not be stopped all at once; it should be gradually weaned as
the amount of solids increases to prevent weight and nutrient loss.

1.‐ "I can puree whatever the family is eating each night and offer it to my baby."
2.‐ "I can stop the formula now that solids are being given."
3.‐ "I can add cereal to my infant's bottle several times each day until he is starts to eat finger foods."
4.‐ "I’ll introduce one pureed food at a time and keep feeding it for several days before trying a new one."

3999 The nurse is caring for a client who suffered major Correct answer: 1 Vitamin C promotes collagen formation and hence wound healing. Vitamins The core concept is difficulty with wound healing. Recall that ascorbic acid aids in tissue
trauma in an auto accident and has been having B&lt;sub&gt;1&lt;/sub&gt; and B&lt;sub&gt;12&lt;/sub&gt; are involved primarily with the repair to choose option 1.
difficulty healing wounds. The nurse encourages the neurological system; vitamin K is involved with the blood coagulation cascade.
client to increase foods high in which of the following
vitamins?
1.‐ Vitamin C
2.‐ Vitamin B1
3.‐ Vitamin B12
4.‐ Vitamin K

4000 A pregnant client with a glycosylated hemoglobin Correct answer: 4 Glycosylated hemoglobin (HbA&lt;sub&gt;1c&lt;/sub&gt;) levels are indicators of longer term The core concept of the question addresses interpretation of glycosylated hemoglobin
(HbA&lt;sub&gt;1c&lt;/sub&gt;) level of 12% asks the glucose control (past 4–8 weeks); 12% indicates poor control, not good control. To determine levels, with this being a high level. Systematically eliminate incorrect options 1 and 2. Note
nurse what this level indicates. The nurse explains: cause and best treatment, a comprehensive evaluation of diet and so on would be indicated. option 3 may be correct but cannot be told to client with absolute certainty and choose
One value of 12% would not necessitate insulin administration. option 4.

1.‐ "You have been consuming inadequate calories and carbohydrates."


2.‐ "You have been maintaining your glucose levels well during past 24 hours."
3.‐ "You must be started on insulin; your glucose levels are out of control."
4.‐ "You have had poor glucose control during the last 4–8 weeks.”

4001 An elderly client complains of difficulty swallowing Correct answer: 4 Decreased saliva makes it difficult to moisten the food bolus so that it can be swallowed. Critical words are elderly, and difficulty swallowing. Eliminate option 2 since it does not
foods such as bread. The nurse plans to assess for Periodontal disease and jaw deterioration affect chewing. Decreased peristalsis affects passage involve chewing and swallowing. Recall physiological changes of aging to choose option 4.
which of the following normal age‐related of bolus once in esophagus.
physiological changes as the most likely explanation
for the problem?
1.‐ Jaw bone deterioration
2.‐ Decreased peristalsis
3.‐ Periodontal disease
4.‐ Decreased saliva

4002 The nurse has instructed a client with constipation Correct answer: 3 Kidney beans are legumes and contain 5.6 g of insoluble fiber per half cup. The other foods Recall knowledge of fiber content of foods. Eliminate options 1 and 4 as incorrect. Note
ways in which to increase fiber in the diet. Which of are not fiber rich. spinach is creamed to eliminate it as well.
the following food selections by the client indicates
teaching has been successful?

1.‐ White bread


2.‐ Pureed spinach
3.‐ Kidney beans
4.‐ Spaghetti

4003 A client recently diagnosed with type 1 diabetes Correct answer: 4 The American Diabetes Association Exchange Lists divide food into similar content (milk, The core concept of the question is identification of diabetic exchange equivalents.
mellitus is learning to use the American Diabetes vegetables, fruit, starch/bread, meat, and fat). Each food within a list is similar in calories, Recognize white rice is a starch to be directed to option 4.
Association Exchange Lists. The nurse concludes that protein, fat, and carbohydrates if eaten in a certain size portion. Foods may be exchanged
the teaching has been effective if the client chooses within the same list. Rice and bread are starches, egg is meat, tomato is vegetable, and orange
which of the following as an appropriate exchange for is fruit.
white rice?
1.‐ Egg
2.‐ Tomato
3.‐ Orange
4.‐ Bread

4004 The nurse is setting up the breakfast tray for a client Correct answer: 3 Foods that reduce lower esophageal sphincter (LES) pressure will increase reflux symptoms. Recall dietary recommendations for GERD to reduce reflux of gastric acid. Recognize foods
with gastroesophageal reflux disease (GERD) and These include coffee, fatty foods, alcohol, and chocolate. All the other items can be given to which decrease the lower esophageal sphincter tone should be avoided to choose option 3.
notices one food item the client should not eat. The the client.
nurse should remove the:
1.‐ Poached egg.
2.‐ Dry toast.
3.‐ Coffee with cream.
4.‐ Skim milk.

4005 The nurse is admitting a client with thermal burns to Correct answer: 3 Clients should remain NPO upon admission to the clinical setting with a major burn. Initial Note that the client has burns to the anterior trunk, necessitating the need to determine
both arms and the anterior trunk. When the client asks fluid replacement is started via the parenteral route. NPO status is maintained because burn swallowing ability prior to allowing client to drink.
for a drink of water the most appropriate response for injuries may cause internal damage to body structures and aspiration can occur. Options 1, 2,
the nurse is? and 4 are incorrect because fluids and food via the mouth would be restricted at this time.

1.‐ "I'll get you a drink as soon as I'm finished establishing an intravenous line."
2.‐ "I can only give you juice to drink, not water."
3.‐ "I'm sorry, you cannot have anything to drink right now. Let me moisten your mouth instead."
4.‐ "Would you also like me to order you a meal tray?"

4006 Which of the following would be an appropriate Correct answer: 1 Abuse of laxatives and diuretics is a frequent "purging" behavior for bulimic clients. Options 2 Recall behaviors associated with bulimia to be directed to option 1.
intervention for the nurse to include in the plan of care and 3 pertain to anorexia nervosa clients. In regard to option 4, food should never be used as a
for a client with a clinical diagnosis of bulimia? reward.

1.‐ Assess for laxative and diuretic possession


2.‐ Supervise mealtimes to ensure eating
3.‐ Observe for ritualistic eating patterns
4.‐ Reward non‐purging behavior with a favorite snack

4007 A client with congestive heart failure (CHF) has been Correct answer: 2 In a 2‐gram sodium diet, foods high in sodium content should be eliminated. It is not enough Critical words are CHF and low sodium. Eliminate options 1, 3, and 4 since they indicate
advised to follow a low‐sodium diet. Which statement to stop adding salt or to go only by taste; clients should also be taught to read food labels for false assumptions and are too general.
by the client indicates to the nurse that diet teaching hidden sodium content. Added salt while cooking is allowed in a 4‐gram sodium diet, not a 2‐
has been effective? gram sodium diet.
1.‐ "If I stop adding table salt, I shouldn't have any problems."
2.‐ "I need to avoid eating processed foods and canned meats and vegetables."
3.‐ "I can still use a small amount of table salt in cooking."
4.‐ "I only have to worry about salty‐tasting foods like potato chips."
4008 Which breakfast choice indicates to the nurse that Correct answer: 3 The American Heart Association recommends a diet with reduced saturated fats and The core concept of the question is CAD. Note this is a negative response question
the client with coronary artery disease (CAD) requires cholesterol for clients with coronary artery disease. Canned peaches are high in concentrated indicating that three of the choices are correct. Recall the diet should be restricted in fats
further diet instruction? sugars, which increase triglyceride levels. Egg yolks are high in cholesterol and whole milk is and cholesterol to choose option 3.
high in saturated fats. The other options reflect appropriate food selections that are low in
saturated fat and cholesterol content.
1.‐ Orange juice, shredded wheat, skim milk, and toast with jelly
2.‐ Grapefruit juice, oatmeal, 1% milk, and bagel with jelly
3.‐ Canned peaches, egg omelet, whole milk, and fruit yogurt
4.‐ Applesauce, bagel with margarine, egg‐white omelet, and skim milk

4009 Which nutritional measures should the nurse suggest Correct answer: 3 A client with GERD should limit (or possibly eliminate) the intake of coffee because this can Recall factors which will reduce the lower esophageal sphincter tone and eliminate
to a client with gastroesophageal reflux disease (GERD) relax lower esophageal sphincter (LES) pressure and lead to symptoms. The other options options 3 and 4. Eliminate option 1 since large meals will increase upward pressure and
to minimize the risk of symptoms? would not be warranted because all would contribute to the development of symptoms: large contribute to reflux.
meals, spicy foods (e.g., extra garlic), and peppermint (which would relax LES pressure).

1.‐ Eat three large meals a day with no snacks


2.‐ Use a lot of garlic to season food rather than salt
3.‐ Limit intake of coffee drinks to two or fewer cups a day
4.‐ Use peppermint candies to take away the bitter taste in the mouth

4010 The nurse is caring for the client who is recovering Correct answer: 2 The eggs provide 24 grams of protein and the whole milk adds calories. The other options are Recall that clients with burns need additional protein and calories to be directed to option
from partial thickness burns. Which of the following lower in protein and calories. A client recovering from burns requires a high‐protein, high‐ 2.
breakfast choices indicates the client's understanding calorie diet. Option 1 does not reflect an adequate protein source. Option 3 reflects an
of the recommended diet? increased carbohydrate source and bacon is considered a fat, not protein. Option 4 does not
reflect a high‐protein, high‐calorie meal but rather a low‐calorie meal selection with greater
carbohydrate content.
1.‐ Two slices of toast with butter, orange juice, and skim milk
2.‐ Two poached eggs, hash brown potatoes, and whole milk
3.‐ Three pancakes with syrup, two slices of bacon, and apple juice
4.‐ One cup of oatmeal with skim milk, half of one grapefruit, and coffee

4011 Which approach should the nurse use to develop a Correct answer: 4 Consistency and gentle firmness allow the client to learn that the nurse will follow through Recall that psychosocial and communication skills are needed to establish relationships
trusting relationship with the client who has an eating and do what is promised. Option 1 is not therapeutic; it may actually push the client away by and recall the need for limits and routine for clients with eating disorder to choose option
disorder? making too many demands at too rapid a pace. Trust must be established slowly and 4.
respectfully. Option 2 allows the client to use manipulative behavior, and option 3 does not
develop trust; it is directing/ordering behavior.
1.‐ Set strict limits that are detailed and numerous
2.‐ Encourage use of "testing" behaviors
3.‐ Tell the client how to behave
4.‐ Utilize consistency and gentle firmness

4012 A nurse is discussing the home maintenance regimen Correct answer: 1 Regular exercise can help to normalize bowel function. Cigarette smoking and gum chewing Recall factors which contribute to increased symptoms in IBS and eliminate options 2, 3,
with a client who has irritable bowel syndrome (IBS). increase swallowed air; fresh vegetables are gas‐producing. and 4.
Which of the following statements indicates client
understanding?
1.‐ "I'll take a walk after dinner each evening."
2.‐ "I'll have a cigarette after meals to relax."
3.‐ "I'll chew gum between meals to curb my appetite."
4.‐ "I'll eat more fresh vegetables and fruits."
4013 A client with anorexia nervosa has a nursing diagnosis Correct answer: 3 One issue for clients with anorexia nervosa is an altered view of their body appearance (i.e., Note similarity in option 3 of body size to body image in question to choose this option.
of disturbed body image. The nurse identifies which of visualizing themselves as being fat even when they are emaciated). Option 1 involves a
the following as an appropriate outcome? knowledge deficit; option 2 involves possible resolution of family dynamic issues; option 4
involves psychological adaptation.
1.‐ Verbalizes knowledge of maintenance diet.
2.‐ Demonstrates assertiveness with family.
3.‐ Verbalizes body size accurately.
4.‐ Demonstrates control of obsessive behaviors.

4014 The nurse encourages increased intake of which of Correct answer: 1 Clients with burns are hypermetabolic and require increased protein levels in order to Eliminate option 2 first as the lowest protein source. Discriminate among the other three
the following foods to best assist a client who has maintain a positive nitrogen balance. Vegetables (option 2 and fruits (option 3) are low in options by selecting the option that has two protein sources listed rather than one.
major burns to maintain a positive nitrogen balance? protein, although the nuts in option 3 are reasonable sources of protein. Dairy products and
shellfish contain protein but are not as good sources as the foods in option 1.

1.‐ Meats and legumes


2.‐ Vegetables and clear liquids
3.‐ Fruits and nuts
4.‐ Dairy products and shellfish

4015 A client has begun therapy with captopril (Capoten) Correct answer: 1, 4, 5 Captopril is an ACE inhibitor that leads to an elevation of serum potassium levels. Foods high Refer to knowledge of mineral potassium vital to cardiac function to associate with
for hypertension. Which of the following foods should in potassium such as oranges and bananas should be avoided. Other foods to avoid are options offered.
the nurse caution the client to avoid? Select all that potatoes and beans, and vegetables such as broccoll and carrots.
apply.
1.‐ Oranges and bananas
2.‐ Cheese and yogurt
3.‐ Milk and milk products
4.‐ Potato and beans
5.‐ Broccoli and carrots

4016 As part of the teaching plan for a client with type 1 Correct answer: 4 Active exercise increases insulin sensitivity, thus lowering blood glucose levels. Additional Critical words are Type 1 diabetic, indicating client is insulin dependent. Systematically
diabetes mellitus, the nurse should include that carbohydrates may be needed to balance the usual insulin dose. All of the other options will analyze each option for the effect on blood sugar and choose option since exercise will
carbohydrate needs can increase under which of the increase blood glucose levels. lower blood sugar.
following circumstances?
1.‐ The client has an infection.
2.‐ The client has an emotional upset.
3.‐ The client eats a large meal.
4.‐ The client engages vigorous exercise.

4017 Which of the following dietary measures should the Correct answer: 3 Small meals prevent overdistention and rapid emptying of stomach, thus helping to prevent Critical words in the question are nutritional intake and subtotal gastrectomy. Note
nurse include when planning for the nutritional needs dumping syndrome. A low‐residue diet is not necessary for this client because this diet plan is correlation of small meals to reduced size of stomach to direct you to option 3.
of a client who has had a subtotal gastrectomy? usually used as a transition diet from liquids to solid foods to allow the colon to rest. A fluid
intake below 1000 mL/day is too low and could cause the client to become dehydrated.
Instead, the client should drink liquids between meals. A high‐carbohydrate diet is not
recommended because concentrated sweets pass rapidly out of stomach and will intensify
symptoms of dumping syndrome. A high‐protein diet is needed for tissue repair.

1.‐ Low‐residue, bland diet


2.‐ Fluid intake below 1000 mL/day
3.‐ Six small meals per day
4.‐ Low‐protein, high‐carbohydrate diet
4018 The mother of a 16‐year‐old client calls to express Correct answer: 3 One form of bulimia is the "nonpurging" type. Clients with this type of bulimia use fasting and Note the connection between obsession with dieting but lack of weight loss to direct you
concern about the teen's obsession with dieting and excessive exercise to compensate for food binges. Many clients with bulimia will appear in a to option 3.
exercising. The teen appears healthy and has not lost normal weight range and perform their eating behaviors in secret. Option 1 is incorrect since
any weight. What might the nurse suspect? anorexia presents with documented weight loss. Options 2 and 4 could be possibilities, but
there is no evidence to support depression or use of drugs at the present time given the
information provided.
1.‐ Anorexia
2.‐ Depression
3.‐ Bulimia
4.‐ Drug abuse

4019 What would the nurse prepare to do for the client Correct answer: 2 The client with burns often develops paralytic ileus within a few hours, thus a nasogastric Recognize the symptoms are indicative of a paralytic ileus. Eliminate options 1, 3, and 4
who is 6 hours post‐burn and has absent bowel sounds tube should be used for stomach decompression. When bowel sounds return, feeding can since oral intake could cause complications.
and abdominal distention? begin, either via feeding tube or orally.
1.‐ Insert a feeding tube for nutrition.
2.‐ Insert a nasogastric tube to low intermittent suction.
3.‐ Withhold oral intake, except for water.
4.‐ Start a diet of clear liquids only.

4020 The nurse is reviewing the results of a lipid profile, Correct answer: 1 High HDL levels are associated with reduced risk for coronary artery disease (CAD) and are Systematically review each option and eliminate all incorrect ones. If you had difficulty
including high‐density lipoproteins (HDLs), low‐density thought to be cardioprotective. Decreased LDL and VLDL levels are associated with reduced with this question, review lipid metabolism and coronary heart disease.
lipoproteins (LDLs), and very low‐density lipoproteins risk for CAD. Increased levels of LDL and VLDL are associated with increased risk for CAD as are
(VLDLs), for a client who is following a low‐fat diet. The low HDL levels.
nurse concludes that the client has the desired pattern
of results if the laboratory values show:

1.‐ High HDL, low LDL, low VLDL.


2.‐ Low HDL, high LDL, high VLDL.
3.‐ Low HDL, low LDL, low VLDL.
4.‐ High HDL, high LDL, high VLDL.

4021 A nurse is caring for a client who has burns over 50 Correct answer: 108 A nutritional goal for a client with burns is to maintain weight within 10 percent of the pre‐ Calculate 10 percent of 120 pounds by multiplying 120 by 0.10 to yield 12 pounds; subtract
percent of the body. The client's pre‐burn weight is burn weight. 12 pounds from 120 to yield a weight of 108 pounds.
120 pounds. When developing the plan of care, the
nurse sets a goal that client's weight will not drop
below _____ pounds. Write a numerical answer.

4022 Which of the following foods will the nurse include in Correct answer: 4 Clients with hypoparathyroidism require calcium replacement. These foods are high in Refer to the physiological function of the parathyroid gland and the regulation of blood
the diet plan for a client with hypoparathyroidism? calcium. Option 1 reflects foods that are high in potassium; option 2 reflects foods that are calcium.
high in sodium; option 3 reflects foods that are high in starches.
1.‐ Bananas, spinach, sweet potatoes
2.‐ Bacon, rice, canned tuna
3.‐ Bran cereal, lima beans, corn
4.‐ Cheese, yogurt, legumes
4023 A client newly diagnosed with anorexia nervosa Correct answer: 2 It is important in the early stages of treatment that a staff member sits with the client during It is significant to note the client is newly diagnosed. Eliminate option 4 since this would be
refuses to eat. Which of the following interventions by mealtimes to offer encouragement and help calm fears of eating. The other options are not a last‐resort measure. Recognize the emotional and psychological nature of the disorder to
the nurse would be most appropriate? appropriate at this time. Having the client’s mother come during meals may affect the client’s direct you to option 3.
coping status, while leaving the client alone may cause the client to refuse to eat. Obtaining an
order for a feeding tube is not warranted at this time, since there is no clinical information to
support an alternate feeding approach.

1.‐ Have the client's mother come to the nursing unit during meals.
2.‐ Sit with the client and offer gentle encouragement.
3.‐ Leave the client alone to eat in privacy.
4.‐ Obtain an order to place a feeding tube.

4024 When planning for the nutritional needs of a client Correct answer: 2 Total kilocalories are based on the hypermetabolism response, which is proportional to the The core concept of the question is nutritional needs and total kilocalories in a client with
with partial‐ and full‐thickness burns, the nurse size of the wound or total body surface area burned. Weight does figure into the formula, but major burn injury. Note the connection between the types of burns, partial and full
calculates the total kilocalories needs in relation to not height; cause and location do not affect total kilocalorie needs. thickness, to direct you to extent in option 2.
which of the following?
1.‐ Preburn height and weight
2.‐ Extent of the burn
3.‐ Cause of the burn
4.‐ Location of the burn

4025 A client with a colostomy has been experiencing Correct answer: 3 Increased intake of salad and fresh fruits and vegetables can lead to increased flatus The question requires you to identify which food pattern would most contribute to gas
increased flatus for the past 3 days. Which client formation in a client with a colostomy. Eating pasta, cereal, and milk and increasing fluids are formation. Many vegetables are in this category, allowing you choose option 3.
information provided during assessment would lead not associated with increased gas formation. It is important for both the nurse and client to
the nurse to suspect an etiology for this occurrence? recognize foods that can be gas‐forming and limit their inclusion in the diet.

1.‐ The client has been eating pasta for the past 3 days.
2.‐ The client has been eating cereal and milk for breakfast each morning.
3.‐ The client has been eating at a salad bar for lunch for the past 3 days.
4.‐ The client has been drinking more fluids for the past 3 days.

4026 The nurse is assisting a client who has a newly Correct answer: 2, 3, 5 Sodium and potassium are lost via an ileostomy, and these foods are high in potassium (e.g., The question requires you to determine what electrolytes are lost. Recall drainage from an
created ileostomy with menu selection. In order to oranges and potatoes) and high in sodium (e.g., tomato juice). Asparagus is not high in either ileostomy is in the ileum and the body has not had time to reabsorb all the water and
offset the potential electrolyte losses from the and can cause an odor. Chicken breast is a healthy choice, but not to offset the electrolyte electrolytes.
ileostomy, the nurse suggests the following foods? losses.
Select all that apply.
1.‐ Asparagus
2.‐ Potatoes
3.‐ An orange
4.‐ Chicken breast
5.‐ Tomato juice

4027 The nurse is counseling a 14‐year‐old diabetic client Correct answer: 1 Type 1 diabetics should monitor blood glucose levels before, during, and after routine Recall knowledge of normal blood glucose levels and correlate effect of exercise on Insulin
about diet and insulin. The client has soccer practice exercise. If levels before exercise are above 100 mg/dL, no additional food is needed. Exercise utilization. Eliminate option 3 since levels are already elevated. Recall exercise will help to
every day after school. Because the blood glucose will lower blood glucose, so additional insulin is not needed. Adjustment of CHO intake prior to reduce blood sugar to choose option 1.
records indicate daily levels are between 120 and 140 practice is not indicated as client's blood glucose level is above 100 mg/dL.
mg/dL before practice, the nurse provides which
instruction to the client?

1.‐ No additional food is needed before practice.


2.‐ Decrease carbohydrate (CHO) intake by 15 grams before practice.
3.‐ Increase CHO intake by 15 grams before practice.
4.‐ Increase regular insulin injection by 2 units.
4028 The nurse has reviewed the American Diabetes Correct answer: 2 The American Diabetes Association Exchange Lists group foods according to composition Systematically review each option and eliminate options 3 and 4 since they contain foods
Association Exchange Lists with a diabetic client. The (similar calories, fat, protein, carbohydrate). One serving can be exchanged for another within from different groups. Eliminate option 1 since peanut butter is a fat and ground beef is a
nurse concludes that instruction effective when the the same list. Milk and yogurt are on the milk list. Peanut butter is on the fat list, while ground protein.
client chooses which of the following food pairs as an beef is on the meat list. Carrots and eggplant are on the vegetable list, while grapefruit is on
equivalent allowable exchange? the fruit list. Bagels are on the starch/bread list.

1.‐ 1 tablespoon peanut butter = 1 ounce ground beef


2.‐ half of a cup of milk = 1 cup yogurt
3.‐ half of a cup of carrots = half of a grapefruit
4.‐ half of a bagel = half of a cup eggplant

4029 A client newly diagnosed with irritable bowel Correct answer: 4 Emotional stress, psychological factors, and food intolerances have been identified as factors Critical words are irritable bowel and prevented. Eliminate option 2 since daily laxative use
syndrome asks the nurse how future attacks can be that can precipitate irritable bowel syndrome. Carbonated beverages (including seltzer) promotes dependency and would not be recommended. Recall the emotional aspect of the
prevented. What is the nurse's best response? increase intestinal gas; laxatives can perpetuate constipation and should be avoided; fiber and disorder to direct you to option 4.
bulk help to regulate bowel movements and should be increased.

1.‐ Include seltzer water with each meal.


2.‐ Use a stimulant laxative once a day.
3.‐ Reduce the amount of fiber in the diet.
4.‐ Identify and reduce emotional stressors.

4030 In order to reduce the development of dumping Correct answer: 3 To minimize the risk of a client's developing dumping syndrome, the client should take several Note all options are related to dietary intake. Recall the stomach size has been greatly
syndrome in a post‐gastric resection client, the nurse small meals throughout the day rather than large meals, which would cause increased stomach reduced to direct you to option 3.
encourages the client to: distention. Fluids should be taken either before or after meals to minimize the possibility of
developing nausea. The diet should be low in simple sugars, moderate in fat, and higher in
complex CHOs and protein. The addition of milk with every meal can cause possible abdominal
bloating.
1.‐ Increase fluid intake with meals to decrease nausea.
2.‐ Provide a diet that is low in complex carbohydrates and high in fat and protein.
3.‐ Eat several small meals throughout the day.
4.‐ Have milk with every meal to coat the stomach lining.

4031 A client is diagnosed with hypercholesterolemia. The Correct answer: 2, 4 Liver is an organ meat and is therefore high in cholesterol. Egg yolks are also high in Critical words are hypercholesteremia and limiting of food. Recall cholesterol and
nurse would instruct the client to limit intake of which cholesterol. Chicken and yogurt are low in cholesterol, while carrots are a plant product and saturated fats contribute to the disease to choose option 2.
of the following favorite foods in the client's diet? do not contain cholesterol.
Select all that apply.

1.‐ Yogurt
2.‐ Liver
3.‐ Chicken
4.‐ Eggs
5.‐ Carrots

4032 The nurse has just admitted a client who has bulimia Correct answer: 1 The most important objective is to normalize food intake with close supervision to control Critical words are highest priority and goal, indicating one goal is more important overall.
and has been abusing laxatives and diet pills. The nurse purging (e.g., vomiting, laxatives, diuretics). The other options are secondary to stabilizing Eliminate option 4, since the focus is not on dieting at this time. Note similarities in the
places highest priority on which of the following goals nutritional status. psychological nature of options 2 and 3 and choose option 1, since it identifies a
of care? physiological need.
1.‐ Promote adequate nutrition and retention of food.
2.‐ Promote the acceptance of self and body.
3.‐ Promote the development of insight into the behaviors.
4.‐ Promote the development of realistic dieting expectations.
4033 The nurse reviewing the dietary assessment of an HIV‐ Correct answer: 4 A client who is HIV‐positive (regardless of sex) is likely to lose weight due to repeated cycles Key concepts are weight loss and HIV. Recognize the malnutrition that occurs with HIV to
positive female notes the client has been skipping of wasting and malnutrition. The client, who might be unable to merely increase caloric intake, be directed to option 4.
meals and progressively losing weight. What dietary should be instructed in dietary techniques that maximize quality of intake. Option 1 is incorrect
interventions would be best for the nurse to suggest to because even though a food diary would provide pertinent information, the response allows
promote weight gain? for a delay in treatment that could result in further weight loss for the client. The priority is to
intervene early on to prevent the onset of wasting. Option 2 is incorrect as it provides the
client with a false belief that fluid retention changes associated with the menstrual cycle may
have an impact on nutritional status. Option 3 is incorrect because even though increased salt
in the diet can lead to fluid retention and weight, it does not address the underlying issue of
nutritional balance.

1.‐ Have the client keep a food diary recording food preferences and usual dietary pattern.
2.‐ Tell the client that her weight may fluctuate in response to her menstrual cycle so there is no need to worry for now.
3.‐ Tell the client that additional salt in the diet will help to increase weight.
4.‐ Tell the client that the use of nutrient‐dense food and fortified protein shakes will help promote weight gain.

4034 A client is admitted to the hospital with a primary Correct answer: 4 Sjögren's syndrome is an autoimmune disease that destroys exocrine glands in the body, and Of importance is the diagnosis of Sjögrens syndrome. Recall the tendency for dryness and
diagnosis of hip fracture and a secondary diagnosis of leads to a generalized "dryness" of body systems. The restriction of fluids is a concern because xerostomia to be directed to option 4, since this would be contraindicated.
Sjögren's syndrome. Which one of the following orders the use of fluids helps to keep the oral cavity moist. There is no information to suggest that the
would be of most concern to the nurse with regard to client has a need for fluid restriction due to other disease processes so this order should be
the nutritional status of the client? clarified. All of the other options are reasonable for this client.

1.‐ NPO after midnight for surgery with a 7:30 a.m. case
2.‐ Intravenous of Lactated Ringer's at 125 milliliters per hour
3.‐ Maintain diet as tolerated
4.‐ Restrict oral fluids to 1000 milliliters per day

4035 The nurse should provide which of the following Correct answer: 1 Due to the anorexia and fatigue frequently experienced by clients with SLE and the need to Recall the chronic nature of the illness. Identify that option 1 is sound advice to give any
nutritional suggestions for a client with systemic lupus maintain adequate nutrient intake, small frequent meals are usually tolerated better than client to increase caloric intake and reduce nausea.
erythematosus (SLE)? large meals. Option 2 is incorrect because the diet should include all nutrients, not just fats and
protein. Options 3 and 4 are incorrect as citrus and spicy foods do not necessarily need to be
avoided, unless they induce nausea in the client.

1.‐ Eat small, but frequent meals throughout the day.


2.‐ Eat foods high in fat and protein.
3.‐ Avoid intake of spicy and highly seasoned foods.
4.‐ Limit intake of citric fruits and juices.

4036 A client recovering from Guillain‐Barré syndrome is Correct answer: 2 A client who is recovering from Guillain‐Barré syndrome will need a diet that promotes Note that the client is being admitted to a rehabilitation unit and recovering from an
admitted to the rehabilitation unit. Which of the positive nitrogen balance in order to counteract the effects of long periods of immobility on illness, indicating a need to restore mobility and health.
following methods does the nurse anticipate using to the body. Option 1 is incorrect as there is no evidence to support that the client is experiencing
provide nutritional support for the client during this malabsorption at this time. Option 3 is incorrect because there is no clinical reason to limit
time? fresh fruit. Even though the client may experience difficulty in chewing and swallowing, this is
usually in the acute phase of the disease process. Option 4 is incorrect as there is nothing to
suggest that the client is experiencing problems in this area or is at risk for aspiration.

1.‐ Use of a gastrostomy tube for feedings due to high incidence of malabsorption
2.‐ Maintenance of oral intake with adequate calories to maintain positive nitrogen balance
3.‐ Limit of fresh fruit in the diet
4.‐ Use of thickened liquids to prevent aspiration
4037 Which of the following client statements regarding Correct answer: 1 Although some raw foods could be a source of contamination to the client with HIV who is Recall the problems clients with HIV might have with nausea, anemia, and constipation to
HIV/AIDS would require further clarification from the immunocompromised, it would not be necessary to avoid all uncooked foods, such as fruits recognize that options 2, 3, and 4 are correct.
nurse? and vegetables. The nurse should clarify this statement by the client in order to provide
accurate information. All of the other client statements reflect information that is appropriate
for the management of client with HIV/AIDS.
1.‐ "I should avoid eating any raw or uncooked foods."
2.‐ "Blood tests will tell me if I have a nutritional anemia."
3.‐ "Maintaining adequate fluid and fiber intake will help me."
4.‐ "If I feel sick to my stomach, I should not drink liquids."

4038 When developing a plan of care for a client with Correct answer: 3 Malnutrition is seen as a consequence of the HIV/AIDS virus because the disease process has Recall the course of the illness to eliminate options 1, 2, and 4 as incorrect.
HIV/AIDS, the nurse recognizes the following a progressive effect on client's nutritional status. Option 1 is incorrect because even clients
statement is true concerning the nutritional status of who are asymptomatic may already have nutrient deficiencies and could be experiencing
this disorder? "subclinical" signs of malnutrition. Option 2 is incorrect as wasting syndrome occurs early in
the disease process; current clinical research states that the maintenance and preservation of
nutritional status is a priority in the clinical management of this condition. Option 4 is incorrect
because clients can experience vitamin and mineral deficiencies early on during the disease
process.
1.‐ Clients who are asymptomatic have adequate nutritional stores of nutrients.
2.‐ The HIV wasting syndrome is seen in the latter stages of the disease process.
3.‐ Malnutrition is seen as a consequence of the immune disease.
4.‐ Vitamin and mineral deficiencies occur in the latter stages of the disease process.

4039 A client is receiving radiation to the head and neck Correct answer: 4 Dry mouth can be a common complaint of clients undergoing radiation therapy. Using sugar‐ Note that the client is complaining of a dry mouth and eliminate options 1 and 2 since they
area for treatment of cancer. What interventions free candies or gum will help to stimulate the flow of saliva and ease the discomfort that the would not help with dryness. Recall mouthwashes often contain alcohol which is drying and
would the nurse use to help the client's complaint of a client is experiencing. Eating meals prior to radiation therapy may lead to increased nausea eliminate option 3.
dry mouth? because the client would be lying down following eating the meal. It has no effect on
complaints of a dry mouth. Eating larger portions of food will not help to ease complaints of a
dry mouth. Furthermore, the client may not be able to increase the size of meals due to side
effects experienced as a result of radiation therapy. The use of mouthwash can further cause
the mouth to be dry and intensify the client's symptoms.

1.‐ Have the client eat prior to radiation therapy.


2.‐ Encourage the client to eat larger portions of food.
3.‐ Advise the client to use mouthwash.
4.‐ Suggest use of sugar‐free candies.

4040 A client has been referred for dietary teaching Correct answer: 4 Nutritional goals for a client with hepatitis are aimed at providing a diet that is high in calories Recall the need for liver regeneration and increased nutritional support to choose option
regarding the management of hepatitis. The nurse (3,000–4,000 kilocalories) and high in quality protein (1.5–2.0 grams per kilogram). The diet 4.
should base development of nutritional goals on which should also be adequate in carbohydrates to spare protein and fat, provide concentrated
of the following information? calories, and improve the taste of food. Option 1 is incorrect because the nutritional
management of hepatitis is the same for all types. Option 2 is incorrect as there is no clinical
indication to place the client on tube feedings given the information provided. If the gut works,
then the usual clinical model is to use it. Option 3 is incorrect because dietary fat should not be
limited unless the client is experiencing problems with malabsorption (steatorrhea) and there
is no evidence to support this.

1.‐ Type of hepatitis the client has, as this will affect treatment
2.‐ Need for tube feedings to allow liver to rest and regenerate
3.‐ Dietary fats should be limited
4.‐ Diet should be high in calories and protein
4041 Because the albumin level of a client with cirrhosis is Correct answer: 1 An albumin level of 2.5 milligrams per deciliter indicates decreased protein stores and Recognize that the albumin level is decreased and recall the need for increased protein
2.5 milligrams per deciliter, the nurse encourages the decreased albumin synthesis by the diseased liver. In cases of hepatic encephalopathy protein with cirrhosis to be directed to option 1.
client to increase the intake of: will be limited, but there is not evidence of this. Although dairy products do provide some
protein, beef and chicken would provide the best source of protein. Fruits and grains would
not provide protein.
1.‐ Beef and chicken.
2.‐ Oranges and strawberries.
3.‐ Cheese and yogurt.
4.‐ Cereals and whole grain breads.

4042 When caring for a client with chronic obstructive Correct answer: 2 COPD places a client at risk to develop malnutrition due to reduction in muscle mass and fat Recall that clients with COPD have increased needs for calories and protein to be directed
pulmonary disease (COPD), the nurse develops a reserves. Option 1 is incorrect because COPD clients are more likely to suffer from respiratory to option 2.
nutritional plan of care to include which of the infections due to altered immune response (e.g., decreased cell‐mediated immunity, altered
following concepts? immunoglobulin production, and impaired cellular resistance). Options 3 and 4 are incorrect
because COPD clients usually present with weight loss and are hypermetabolic (i.e., require
additional calories due to increased energy requirements as a result of increased work of
breathing).
1.‐ The client has an adequate immune response.
2.‐ Clients with COPD are at increased risk of suffering from malnutrition.
3.‐ The client is likely to experience weight gain due to fluid retention.
4.‐ Decreased energy requirements often lead to weight gain.

4043 A 28‐year‐old client who is admitted to the unit with a Correct answer: 2 A client with MS is prone to developing both bowel and bladder dysfunction as a result of this Note that the question addresses the need to treat constipation without laxatives to
relapse of multiple sclerosis (MS) is experiencing progressive degenerative neurological disease. Increasing fluids and roughage in the diet will eliminate option 4. Option 3 will not have a direct effect on bowel elimination and can be
constipation. The client asks what other methods help to facilitate evacuation by improving stool consistency. Option 1 is incorrect because the eliminated. Option 1 will aggravate constipation and needs to be eliminated as well.
besides using laxatives can be used to prevent or treat client needs increased fluids. Option 3 is incorrect because increasing ROM exercises provides
this condition. How should the nurse respond to the for joint motion but does not necessarily exercise the abdominal muscles, which could
client's concern? influence peristalsis. Option 4 is incorrect because there is not enough clinical information
provided to make this assessment. The nurse would have to assess further for elimination
pattern and the date of the client's most recent bowel movement.

1.‐ Tell the client to reduce fluid intake.


2.‐ Have the client increase roughage in the diet.
3.‐ Have the client increase range of motion (ROM) exercises to stimulate peristalsis.
4.‐ Call the physician regarding an order for an enema.

4044 A client covering from a spinal cord injury (SCI) has Correct answer: 1 Even though a client has had an SCI, the use of a diet high in protein, carbohydrates, and fiber Eliminate options that do not address the client's concern (2 and 3). Choose option 1 over
been referred for nutritional counseling due to weight is necessary to prevent both the catabolic process that occurs following SCI and potential 4 since it provides a broader, more inclusive answer.
loss. The client states, "If I eat too much, the weight problems with bowel function. Option 2 is incorrect because it reflects the belief that weight
will just stay on and I will become fat." How would the loss is an easy goal to achieve. Option 3 is incorrect because it assumes that merely getting
nurse best respond to this statement? foods that the client likes will correct the problem. Option 4 is incorrect because excess
nutrient stores will not merely help to preserve skin integrity but are needed for overall
support of the client's metabolism and immune response.

1.‐ "It is important to continue to eat a diet high in protein, carbohydrates, and fiber to maintain optimal body function."
2.‐ "I know that you are concerned about weight gain, but you can always diet later on."
3.‐ "Let me know what your food preferences are and I will get you additional portions of whatever you like."
4.‐ "It is important to have extra nutrient stores in order to preserve skin integrity."
4045 A client being treated for gout is being evaluated for Correct answer: 1 Scrambled eggs, white toast, and coffee are all foods that are low in purine content. A client Recognize that gout is affected by purine metabolism and systematically eliminate foods
compliance with diet therapy. Which of the following who is being treated for gout should restrict dietary purine sources because they can lead to containing purines.
meal selections would indicate that the client has an exacerbation of the disease process. All of the other choices reflect dietary selections that
adhered to the diet plan? range from moderate to high purine content. If dietary education is successful, then the client
would avoid/limit these food selections.
1.‐ Scrambled eggs, white toast, and coffee
2.‐ Seafood casserole, wheat roll, and soda
3.‐ Pizza with anchovies and soda
4.‐ Braised liver, lentils, green peas, and tea

4046 A client is to undergo bone marrow transplantation Correct answer: 3 A client undergoing a BMT will probably be fed by TPN in the post‐transplant period due to Recognize the need for prolonged nutritional therapy and to bypass the enteral system
(BMT) for treatment of leukemia and is receiving pre‐ potential complications affecting the mouth, esophagus, and intestines, leading to diarrhea following a transplant to direct you to option 3.
procedure teaching about nutrition. Which of the and malabsorption. Option 1 is incorrect. Supplemental enteral feedings would not help
following postoperative nutritional support options because the client's GI tract has been affected by chemotherapy and other medical
does the nurse anticipate will be utilized? treatments. In addition, merely supplementing the client will not provide sufficient calories
and nutrients. Option 2 is incorrect because oral intake is usually not available due to side
effects from high dose chemotherapy regimens that lead to anorexia, taste perception,
nausea, vomiting, and inflammation of mucous membranes. Initiation of oral feedings will not
prevent gastroparesis. Option 4 is incorrect because there is nothing to suggest that a PEG
tube would be indicated. The goal with BMT clients is to return to a "normal" route‐feeding
regimen as soon as possible once clinical effects of immunosuppression have been resolved.

1.‐ Supplementation with enteral feedings to prevent catabolism


2.‐ Oral feedings as soon as possible following BMT to prevent gastroparesis
3.‐ Total parenteral nutrition (TPN) for a period of months to maintain nutritional balance
4.‐ Insertion of a PEG tube following the BMT to maintain nutritional balance

4047 A 42‐year‐old male client with AIDS is admitted with Correct answer: 3 A client complaining that he has "difficulty eating and swallowing just about anything" may Critical words are difficulty eating and swallowing. Eliminate options 1 and 4 since they do
dehydration. The client states he has "difficulty eating have a fungal infection of the mouth and/or esophagus. A clinical diagnosis of AIDS suggests not address the problem directly. Eliminate option 2 because anorexia is not the problem.
and swallowing just about anything." The client has that the client is at high risk for developing an opportunistic infection. Option 1 is incorrect.
lost 10 pounds over a 3‐week period. What does the Even if the client may not be able to shop because of fatigue or other factors, it doesn’t
nurse identify as the most likely etiology for the directly explain the client’s statement. Option 2 is incorrect because the client's complaint
client’s chief complaint? addresses the issue of swallowing, not anorexia. Option 4 is incorrect—there is nothing to
suggest that the client has not been compliant with the medication regimen. The presence of
opportunistic disease can occur even in the presence of medication therapy due to underlying
immunosuppression.

1.‐ The client has been too weak to shop for food.
2.‐ The client's medication profile is causing him to develop anorexia.
3.‐ The client could be developing an opportunistic infection.
4.‐ The client has not been compliant with medication regimen.

4048 A 60‐year‐old male client who has had chronic Correct answer: 4 A client with COPD is often hypermetabolic from the disease process and requires increased Recall the need for a high‐protein, high‐caloric diet with COPD. Eliminate options 1 and 2
obstructive pulmonary disease (COPD) for 15 years is calories, proteins, vitamins, and minerals in order to maintain desired weight and meet since fats and CHO should not be increased. Eliminate option 3 since this could cause
experiencing weight loss despite insisting that he has additional energy demands. Option 1 is incorrect: Caloric intake is not adequate, and further weight loss.
been "eating a well‐balanced diet." The diet history increasing fat percentage above 30% is not prudent. Option 2 is incorrect: Increasing
indicates the client has been consuming an adequate carbohydrates in the diet can lead to increased respiratory workload due to excess acid
caloric intake of approximately 2,500 kilocalories per production. Option 3 is incorrect because increasing activity level will not help to prevent
day composed of 15% protein, 70% carbohydrates weight loss. In addition, the client may not be able to increase activity level due to effects of
(CHOs), and 15% fat. What recommendations would COPD.
the nurse make regarding the client's weight status?
1.‐ Maintain calories and increase percentage of fat to 35% in the diet to promote weight gain.
2.‐ Decrease the amount of fat in the diet and increase complex CHOs.
3.‐ Increase activity level as caloric intake and percentages of nutrients are adequate to sustain weight status.
4.‐ Increase calories, protein, fat, vitamins, and minerals in order to prevent further weight loss.

4049 When assessing a client with scleroderma, the nurse Correct answer: 3 A client with scleroderma often suffers from increased acid secretion and esophageal reflux. Recall that clients with scleroderma experience esophageal reflux to direct you to option
identifies that which of the following conditions This could pose a significant nutritional problem. Option 1 is incorrect. Anorexia is not 3, since clients with reflux often report heartburn.
associated with this disease could present a nutritional commonly associated with this disease process. Option 2 is incorrect because alternating
problem? periods of constipation and diarrhea are usually seen in a client who is experiencing irritable
bowel syndrome (IBS). Option 4 is incorrect. Skin becomes hardened during this disease
process and skin turgor is not increased.
1.‐ Diarrhea and anorexia
2.‐ Alternating constipation and diarrhea
3.‐ Reports of frequent heartburn
4.‐ Increased skin turgor

4050 Which of the following statements about cancer Correct answer: 3 Cancer cachexia is a syndrome that occurs in clients with cancer (malignancy) that leads to a First eliminate option as too general. Recall cancer cachexia is associated with tumors to
cachexia would the nurse use in response to a cancer loss of muscle, fat, and body weight. It is thought to occur due to tumor‐induced changes that direct you to option 3.
client questioning weight loss and wasting? cause profound effects on metabolism, nutrient losses, and anorexia. A cycle of wasting is
established because alterations in nutrient requirements and intake lead to high cell turnover
in body organs, affecting the GI tract and bone marrow. Alterations in digestion occur along
with decreased immune response. Option 1 is incorrect because in simple starvation the body
adapts to a lower metabolic rate. A client with cancer cachexia does not have an adaptive
metabolic rate. The metabolic rate can be normal, decreased, or increased. Option 2 is
incorrect because cancer cachexia occurs in the presence of both chemotherapy and radiation.
Option 4 is incorrect—cancer cachexia can be seen in clients who have adequate caloric intake
because it is not calorie dependent.

1.‐ "It is no different than simple starvation because the metabolic rate declines in response to tumor growth."
2.‐ "Cancer cachexia occurs as a result of chemotherapy but not radiation therapy."
3.‐ "Cancer cachexia occurs as a result of tumor‐induced changes."
4.‐ "It is usually seen in clients who have limited caloric intake."

4051 A 48‐year‐old male client who is HIV‐positive is being Correct answer: 2 A client who is receiving isoniazid (INH) as a prophylaxis for tuberculosis is at highest risk for Recall significance of INH to intake of vitamin B&lt;sub&gt;6&lt;/sub&gt; to prevent
treated prophylactically with isoniazid (INH). The nurse deficiencies of vitamins, specifically vitamin B&lt;sub&gt;6&lt;/sub&gt; (because the drug acts peripheral neuropathies. This will direct you to option 2.
concludes that this client is at highest risk for which of as a vitamin antagonist) and vitamin B&lt;sub&gt;12&lt;/sub&gt; (interferes with absorption).
the following nutritional problems? All of the other choices do not occur as a result of the action of this medication.

1.‐ Frequent bouts of diarrhea


2.‐ Deficiency of B vitamins
3.‐ Development of dental caries
4.‐ Excessive flatus formation

4052 A 52‐year‐old male client being treated for cancer is Correct answer: 2 Megesterol acetate (Megace) is oral progesterone that is used for both male and female Recall the use of Megace in cancer and malnourished clients to direct you to option 2.
use of a "female hormone," megesterol acetate clients to boost appetite and promote weight gain. It is important that all clients receive
(Megace), as part of the treatment regimen. He is accurate information about prescribed medications and are aware of the indication for the
afraid that it will alter his appearance. The nurse drug, potential side effects, and expected response to treatment. The nurse should respond to
should respond to the client's concern of altered body the client's concern initially with factual information because the client does not seem to
image by first explaining: understand the effect of the medication. Options 1 and 4 are incorrect because they do not
address the client's concern and might further increase his anxiety about body image changes.
Option 3 is incorrect because even though the client has the right to refuse any treatment, the
response does not attempt to communicate pertinent factual information.
1.‐ "The physical changes are only temporary."
2.‐ "This medication is used for its ability to stimulate appetite."
3.‐ "Your concern is realistic and you should not take the medication if you feel this way."
4.‐ "The medication will be used for a short time and any effects will be self‐limiting."

4053 Which one of the following actions by the nurse Correct answer: 1 Food should be cooked to reduce bacteria, which the immunosuppressed client cannot fight The critical word is immunosuppressed. Recall the client is a risk for infection to direct you
would be most appropriate related to diet selection effectively. Option 3 is incorrect, while options 2 and 4 are not relevant to the question as to option 1.
for an immunosuppressed client? stated.
1.‐ Provide any food enjoyed by the client as long as it is thoroughly cooked (i.e., well done).
2.‐ Limit fluids to prevent edema due to decreased protein stores.
3.‐ Encourage fresh foods and vegetable produce, which are essential to maintain adequate nutrition.
4.‐ Cut foods into small pieces to facilitate chewing.

4054 A 47‐year‐old male client with renal disease does not Correct answer: 3 Even though protein restriction is the mainstay of therapy for clients with impaired renal The critical words are high biologic value. Eliminate options 1 and 2 as incorrect
understand why the nurse is instructing him to include function, high biologic value proteins are favored due to their high content of essential amino information. Note the restriction of only in option 4 and eliminate it.
high biologic value protein in the diet, since he has acids. Option 1 is incorrect because high biologic value proteins help to minimize urea
always been told to restrict protein. What explanation production by allowing synthesis of nonessential amino acids from essential amino acids.
should the nurse give to the client? Option 2 is incorrect: Protein restriction is needed because the kidneys’ ability to excrete
nitrogenous end products is impaired in clients with renal disease. Option 4 is incorrect: Even
though it is true the high biologic value proteins are necessary, they are not reserved only for
clients on dialysis.
1.‐ High biologic value proteins help to increase urea excretion.
2.‐ Increased protein is needed to prevent catabolism, regardless of the stage of renal disease.
3.‐ High biologic value proteins contain essential amino acids that are necessary to maintain nutritional balance.
4.‐ High biologic value proteins are needed during times of stress to maximize metabolic efforts only in clients on dialysis therapy.

4055 When assessing a client with a history of kidney Correct answer: 3 Vitamin C in megadoses can increase the risk for oxalate stone formation. It would be Identify the relationship between kidney stones and vitamin C to direct you to option 3.
stones, the nurse notes the client is taking daily important to determine the amount of vitamin C that the client is taking in relation to the
vitamin C supplements. What dietary counseling potential effects of stone formation. Option 1 is incorrect: Even though vitamin C has
should the nurse provide to this client? antioxidant effects, the potential for stone formation outweighs the benefit of taking large
doses of vitamin C. While it is important to increase fluids to prevent urinary stasis, option 2 is
incorrect because the statement does not specifically address the issue of vitamin C
supplements. Option 4 is incorrect since animal protein should be decreased in order to
minimize potential stone formation.

1.‐ The client should increase daily intake of vitamin C for its antioxidant effects.
2.‐ Fluid intake should be monitored to prevent stone formation.
3.‐ Limit intake of supplemental vitamin C, which can exacerbate stone formation if taken in high doses.
4.‐ Stop taking vitamin C because it is only beneficial for common cold symptoms.

4056 What interventions should the nurse plan for in a Correct answer: 4 Elemental zinc taken with food or milk will help correct alterations in taste (i.e., dysgeusia). Recall meaning of the word dysgeusia. If you have difficulty with the question, recognize
client admitted to the oncology unit for chemotherapy Option 1 is incorrect because this intervention is used to treat anticipatory nausea. While it is options 1 and 2 are applicable for many conditions and option 3 would not be
who is experiencing dysgeusia? important to assess a client for signs of dehydration (option 2), it is more important to correct recommended for a client receiving chemotherapy.
altered taste sensation to enable the client to increase intake. Option 3 is incorrect because
highly seasoned foods can cause nausea and irritation.

1.‐ Premedicate the client with an antiemetic.


2.‐ Observe the client for signs of dehydration.
3.‐ Use highly seasoned foods to stimulate taste buds.
4.‐ Obtain an order for zinc and give with food or milk to treat the symptom.
4057 Which dietary instruction is most appropriate for the Correct answer: 3 Small frequent meals provide for adequate intake with reduced fatigue and SOB. Simple Refer to most appropriate in the stem of question and option 3 that makes reference to
client with chronic obstructive pulmonary disease carbohydrates do provide quick energy, but a mixture of nutrients reduces carbon dioxide mealtime and decrease of energy needs.
(COPD) experiencing fatigue and shortness of breath production and maintains respiratory function. Fat consumption can lead to hyperlipidemia
during mealtime? and should only provide approximately 30 percent of total calories. Most individuals have
more energy in the morning than evening.
1.‐ Include simple carbohydrates (CHO) for quick energy.
2.‐ Eat fatty foods to increase calorie intake.
3.‐ Eat frequent small meals to decrease energy use.
4.‐ Eat the largest meal before bedtime.

4058 A client presenting with ascites secondary to liver Correct answer: 4 A client experiencing ascites due to liver failure has decreased protein levels (albumin) that Core concept is liver failure and ascites. Recall the fluid shifts that occur to direct you to
failure is being evaluated for fluid balance. The nurse lead to third spacing of fluids. The calculation of dry weight (i.e., total weight minus the weight option 4.
would best assess fluid status using which of the of ascites) is critical to determining fluid status and medical management of the client. Option
following? 1 is incorrect because it does not address the issue of ascites specifically but rather looks at a
strict volume measurement. Option 2 is incorrect because one would expect abnormal liver
function tests but this information is again not specific to fluid status but rather to the status
of liver function. Option 3 is incorrect: Even though serum protein levels would be expected to
be low, the caloric intake level would not help to define fluid status.

1.‐ Intake and output measurement


2.‐ Liver function test results
3.‐ Caloric intake and serum protein levels
4.‐ Dry weight calculation

4059 A client who has cirrhosis of the liver is now Correct answer: 2 A client being treated for hepatic encephalopathy has increased ammonia levels and is likely Core concepts are encephalopathy and nutritional support. Eliminate options 1, 3, and 4
diagnosed with hepatic encephalopathy. The dietitian to be experiencing mental status changes and fluid retention (ascites). It is important for the since they do not address current eating pattern or intake.
has been consulted to evaluate this client for dietitian to note that the client's mental status precludes normal intake and nutrition support
appropriate nutritional therapy. What priority may be indicated. Option 1 is incorrect. Although a weight and caloric baseline would be
information should the nurse provide to the dietician important for the dietician to review, the current nutritional goal would be to decrease factors
to help formulate nutritional goals for the client? that could lead to fluid retention and increased ammonia levels. Option 3 is less important
than understanding the mental status as a basis for formulating nutritional goals. Although it is
nice to know that the client has been compliant with medical treatment thus far, option 4 is
incorrect because it does not specifically address the establishment of nutritional goals.

1.‐ Client's usual weight and caloric intake pattern prior to admission
2.‐ Client's reduced intake secondary to decreased mental status
3.‐ Client has a preference for snack foods and sodas
4.‐ Client has been compliant with medical treatment during this hospitalization

4060 The nurse is teaching the wife of a client who has a Correct answer: 4 The neutropenic client is immunocompromised and susceptible to bacterial contamination Critical words are neutropenic precautions. Review new guidelines and recognize client is
neutrophil count of 500/mm&lt;sup&gt;3&lt;/sup&gt; from food. Cross contamination is avoided by using separate cutting boards. Vegetables may immunosuppressed to direct you to option 4.
about dietary precautions that should be instituted. be eaten raw as long as they are thoroughly washed. It is not necessary to boil liquids, and
Which of the following instructions should the nurse seeds and nuts may be eaten.
include?

1.‐ Avoid eating any raw vegetables.


2.‐ Boil all liquids before serving them.
3.‐ Do not let him eat any seeds or nuts.
4.‐ Use a separate cutting board for beef and poultry.
4061 The spouse of a client with Parkinson’s wants to know Correct answer: 1 A client with Parkinson's is at risk for aspiration. The statement by the client's wife indicates The problem with swallowing indicates a safety concern. Eliminate options 2, 3, and 4
how to best assist her husband during feeding as he is that the client is experiencing an increase in clinical symptoms such as drooling and impaired since they do not offer appropriate suggestions to reduce risk of aspiration.
having "increasing problems with drooling and swallowing. The use of thickened liquids and proper positioning can minimize the risk of
swallowing." What instruction should the nurse aspiration and help the client's wife to feel comfortable and knowledgeable regarding feeding
provide to the family member? concerns. The spouse should also notify the physician because an adjustment in medications
may be needed. Option 2 is incorrect. There is not enough information to state that the client
should be switched to enteral feedings at this time. Option 3 is incorrect because merely
increasing fluids in a client experiencing increased drooling and difficulty swallowing could
further increase the risk of aspiration. Option 4 is incorrect. Merely using a straw will not help
to correct the underlying problems and could possibly increase the risk of aspiration due to
inability to manage fluids.

1.‐ "Use thickened liquids along with upright positioning during feeding."
2.‐ "It might be time to switch to enteral feedings if you are afraid that your husband may choke."
3.‐ "Increase the amount of fluids he receives to decrease saliva formation and improve swallowing."
4.‐ "Use a straw during feedings to facilitate swallowing."

4062 Which of the following foods enjoyed by a client with Correct answer: 1, 2, 3 Organ meats, such as liver, kidney, brain, and sweet breads are high in purines. Considered as The core issue of the question is knowledge of foods that need to be avoided with gout
gout would the nurse encourage the client to continue moderately high would be meats, seafood, and dried beans. The other choices are not high in because they are high in purines. With this in mind, choose the foods that are not high in
to include in the diet? Select all that apply. purines. purines.

1.‐ Beets
2.‐ Milk
3.‐ Eggs
4.‐ Sweetbreads
5.‐ Sardines

4063 Which of the following items should the nurse include Correct answer: 4 Foods containing gluten (e.g., wheat, oats, rye, and barley) are restricted for a client with Recall that clients with celiac disease cannot metabolize gluten to be directed to option 4.
in the diet of a client diagnosed with celiac disease? celiac disease, due to the client's inability to handle gluten protein. All of the other choices
reflect items that cannot be used in a gluten‐restricted diet.
1.‐ Oatmeal
2.‐ Whole wheat toast
3.‐ Beef barley soup
4.‐ Cornflakes

4064 Which of the following food selections would the Correct answer: 3 A client taking MAO inhibitors has to avoid foods that are high in tyramine because this can Recall the need to eliminate foods high in tyramine when taking MAO inhibitors. Recall
nurse include in the diet of a client taking monoamine lead to significant complications, resulting in hypertensive crisis. Cottage cheese represents an foods in this category are often smoked, aged, or fermented to help you eliminate options
oxidase (MAO) inhibitors? unfermented cheese that can be used in the diet. All of the other options reflect foods that are 1, 2, and 4.
high in tyramine. Aged cheeses are not allowed on the diet.

1.‐ Smoked fish


2.‐ Bologna sandwich
3.‐ Cottage cheese
4.‐ Salad with bleu cheese dressing

4065 The nurse places highest priority on which Correct answer: 2 It is critical to verify tube placement prior to administration of any enteral feeding regimen to Key word is enteral. Recall the importance of preventing instillation of the feeding into the
intervention when preparing to administer an enteral prevent the risk of aspiration. All of the other options are important but they are not the lungs to be directed to option 2.
feeding? highest priority at the present time.
1.‐ Checking gastric residual
2.‐ Verifyimng tube placement
3.‐ Monitoring glucose levels
4.‐ Verifying intake and output
4066 A client who is severely malnourished because of Correct answer: 4 A severely malnourished client with a nonfunctional gut is unable to meet nutritional goals Key concept is severe malnutrition. Note the gut is not functioning and eliminate option 2.
nonfunctional gut requires nutritional support. The through enteral feeding. This client needs to meet nutritional goals through total parenteral Recall that TPN provides the most concentrated source of nutrition to be directed to option
nurse recognizes which of the following methods nutrition (TPN) via a central line placement. Option 1 is incorrect because this client will not be 4.
would assist the client in meeting nutritional goals? able to tolerate cyclic feedings due to existing clinical state. Options 2 and 3 are incorrect
because both enteral tube feedings and PPN will not be able to supply enough calories and
nutritional support for this type of client.

1.‐ Cyclic feeding of total parenteral nutrition (TPN) via central line placement
2.‐ Continuous enteral feeding via tube placement
3.‐ Nutritional support through partial parenteral nutrition (PPN)
4.‐ TPN via central line placement

4067 What factors should the nurse include in the dietary Correct answer: 3 A client with gestational diabetes encounters increased metabolic needs from the pregnancy Key term is gestational diabetes. Recognize the need for glucose control while providing
plan for a client with gestational diabetes? that result in an increased hormonal response and insulin sensitivity. Incorporating snacks in for increased metabolic demands on the mother to be directed to option 3.
the diet plan will help to maintain a constant glucose supply and prevent potential imbalances.
Option 1 is incorrect because a decrease in CHO sources is recommended for a client who has
gestational diabetes in order to prevent excessive glycemic response. Option 2 is incorrect as
an increase in calories is warranted, but this amount is too excessive and may further
contribute to health problems and weight gain. Option 4 is incorrect because increasing CHO
intake at one meal will lead to an overactive glycemic response.

1.‐ Increase the amount of carbohydrate (CHO) sources to maintain insulin response.
2.‐ Increase caloric intake to the normal value to meet growing metabolic needs of the maternal fetal unit.
3.‐ Incorporate snacks to maintain glucose balance because of increased metabolic needs.
4.‐ Have the majority of CHO intake during the breakfast meal to act as a significant fuel source for the rest of the day.

4068 What measures can be taken by the nurse to prevent Correct answer: 2 TPN solutions should be administered via an infusion pump so that fluid rate and volume can Key terms are FVE and TPN. Note that all the options are related to responsibilities of TPN
fluid volume excess (FVE) from developing in a client be controlled. This will prevent the risk of developing FVE as a result of inadequate rate infusion, but only option 2 involves regulation of fluid volume.
who is receiving total parenteral nutrition (TPN)? regulation, leading to increased potential volume. All of the other options reflect acceptable
nursing actions in regards to TPN therapy, but these will not affect the nursing diagnosis of
FVE.
1.‐ Monitor client's blood glucose level during infusion of TPN
2.‐ Use infusion pump to administer TPN fluids
3.‐ Change TPN solution every 24 hours
4.‐ Monitor catheter insertion site during therapy

4069 Which of the following diagnostic tests should the Correct answer: 4 Serum transferrin levels indicate visceral protein stores in the body. Albumin and prealbumin Key term is protein status. Review the purpose of each laboratory test provided in the
nurse check to monitor the protein status of a client levels also serve to indicate protein status. Option 1 is incorrect because BUN levels can be choices, choosing option 4 since it is most specific to protein stores.
receiving nutritional support therapy? affected by a multitude of factors ranging from dehydration to renal status. Option 2 is
incorrect as CBC with differential indicates hematology status. While it may reflect anemia, it is
not considered a protein status indicator. Option 3 is incorrect because it would give
information about to elimination and renal status. While it may reflect protein spilled in the
urine, it is not considered a protein status indicator.

1.‐ BUN
2.‐ CBC with differential
3.‐ Urinalysis
4.‐ Serum transferrin
4070 The nurse teaches a client with lactose intolerance to Correct answer: 1, 4 Casein and whey are often used as additives and stabilizers in processed foods. The client Key term is lactose intolerance, indicating the client has difficulty digesting milk products.
check food labels for which additives, since they may with lactose intolerance may have difficulty digesting these additives since they are milk Recognize casein and whey as by products of milk.
cause gas and bloating? Select all that apply. products. Sodium phosphate is a preservative. Lecithin is an emulsifier. Maltodextrin is a sugar.

1.‐ Casein
2.‐ Sodium phosphate
3.‐ Lecithin
4.‐ Whey
5.‐ Maltodextrin

4071 A client with a history of gout is concerned about Correct answer: 4 It is important to acknowledge the client's concerns about lifestyle changes. The offer of Key words are purine‐restricted diet and best. Recall foods high in purines and note the
maintaining a purine‐restricted diet for the rest of his assisting the client in designing a diet plan with the required restriction but yet focusing on psychological aspect of the question.
life. How should the nurse best respond to the client's palatability will increase client compliance. Option 1 is incorrect because this statement does
concern? not take into account client preferences or stated client concerns. Option 2 is incorrect as this
statement does not acknowledge the fact that this is a necessary diet for the client to prevent
clinical symptoms. Option 3 is incorrect because this statement is not therapeutic and is
viewed as being a punitive choice.

1.‐ Suggest that the client learn to eat pizza without anchovies.
2.‐ Have the client provide a listing of food preferences and suggest that he continue to eat what he likes, but in smaller quantities.
3.‐ Suggest to the client that this is for his own good to prevent further gouty attacks.
4.‐ Assist the client in designing a diet plan that minimizes purine ingestion but yet is palatable.

4072 An elderly client is admitted to the hospital with Correct answer: 1 This client presents in a depressed state with mild dehydration and recent weight loss. Enteral Note that the client is dehydrated, depressed, and with significant weight loss. Recognize
dehydration, weight loss of 15 pounds within the last feedings via an NG tube would help support the client in meeting nutritional goals. Option 2 is this indicates a need to restore fluids and nutritional balance to be directed to option 1.
month, and in a depressive state. Which of the not correct because the client's clinical status does not indicate the need for TPN
following nutritional support methods will the nurse administration at this time. Option 3 is not correct as surgical placement of a feeding tube is an
anticipate being initiated for this client? invasive procedure that is not clinically indicated given the client information. Option 4 is
incorrect because merely encouraging fluids, although high in protein, will not provide
sufficient calories to maintain nutritional goals. Also, the client is admitted in a depressed state
and, as shown by the recent history of weight loss, has not maintained adequate oral intake.

1.‐ Nasogastric (NG) enteral feeding


2.‐ Total parenteral nutrition (TPN)
3.‐ Percutaneous endoscopic gastronomy (PEG) tube insertion with enteral feeding
4.‐ Encouraged intake of high protein liquid supplements

4073 A client with an acute exacerbation of Crohn’s disease Correct answer: 2 Clients should have a transition period from TPN to oral feedings. The GI tract will need time The core concept of the question is transitioning from TPN to oral intake. Recall the bowel
is having oral intake resumed after having been NPO to adjust, and the client may experience some GI upset, so TPN is not stopped abruptly. has not been metabolizing food for 3 weeks while client was NPO. Recognize the need to
and receiving total parenteral nutrition (TPN) for 3 Restriction to ice chips would not be necessary; diet can be resumed starting with fluids or as ease the transition and associate the word gradually in option 2 to this process.
weeks. The nurse plans to do which of the following in tolerated. The client may not eat sufficient calories during the day, so rate of TPN is usually
order to ease transition of feeding methods? tapered, rather than only infusing it at nighttime.

1.‐ Allow client to have only ice chips for the first 2 days.
2.‐ Gradually begin oral feedings as parenteral solution is decreased.
3.‐ Infuse the TPN solution during the nighttime only when client is sleeping.
4.‐ Begin oral feedings of soft foods and stop TPN infusion.
4074 Which of the following should the nurse include in a Correct answer: 2 TPN solutions should be changed every 24 hours in order to prevent bacterial overgrowth due Review principles related to administration of TPN and recognize the actions in options 1,
plan of care for a client receiving total parenteral to hypertonicity of the solution. Option 1 is incorrect because medication therapy can continue 3, and 4 are not necessary. Recall any intravenous solution should be changed after 24
nutrition (TPN)? during TPN therapy. Option 3 is incorrect since flushing is not required for TPN administration. hours to choose option 2.
Option 4 is incorrect because the initiation of TPN does not require a client to remain on bed
rest during therapy. However, other clinical conditions of the client may affect mobility issues
and warrant the client's being on bed rest.

1.‐ Withhold oral medications while the TPN is infusing.


2.‐ Change TPN solution every 24 hours.
3.‐ Flush the TPN line with water prior to initiating nutritional support.
4.‐ Keep client on complete bedrest during TPN therapy.

4075 Which of the following should the nurse plan to do in Correct answer: 1 TPN solutions are hypertonic, hyperosmolar solutions that lead to an increased glycemic load. Note options 2 and 3 are opposite and require a physician’s order to eliminate them.
order to maintain normal glucose levels in a client In response to this hyperglycemia, often a sliding scale insulin (with regular insulin) is used to Recognize regular insulin is the only insulin that can be given intravenously and eliminate
receiving total parenteral nutrition (TPN) therapy? restore, prevent, or control the effects of the hyperglycemia. Options 2 and 3 are incorrect option 4. Recall the high glucose content of TPN to direct you to option 1.
because the TPN rate is individualized to the client and should not be adjusted unless directed
by the physician because of changes in the client’s clinical conditions. Option 4 is incorrect
because only regular insulin is used as an additive in TPN solutions.

1.‐ Monitor blood glucose every 4–6 hours depending on the client's acuity and follow regular insulin sliding scale.
2.‐ Decrease rate of TPN solution to maintain euglycemic levels.
3.‐ Increase the rate of TPN solution to maintain euglycemic levels.
4.‐ Use NPH insulin as an additive in TPN solutions to help maintain euglycemic levels.

4076 The nurse anticipates which type of enteral feeding Correct answer: 3 Elemental formulas represent predigested formulations of macronutrients that are beneficial Critical words are severe digestive problems and absorption. Recall content of the various
formula will be selected for a client who has problems to clients with severe digestive or absorption problems. Option 1 is incorrect: Modular formulas in the options, noting the client has severe problems and option 1 contains only 1
with severe digestion or absorption of nutrients? formulas are not nutritionally complete because they provide only one nutrient source. nutrient source. If you had difficulty with this question, review content of the various
Options 2 and 4 represent intact protein sources that are not suitable for a client with nutritional formulas.
digestive or absorptive problems.
1.‐ Modular
2.‐ Polymeric
3.‐ Elemental
4.‐ Ensure

4077 A client with a history of food allergies asks how to Correct answer: 3 Avoidance and restriction of food items known to cause allergies is the most effective way to The core concept is avoidance of food allergens. Eliminate options 1 and 4 since they are
decrease the likelihood of allergic potential. What prevent the development of potential food allergies. Option 1 is incorrect—even small unsafe advice. Recognize option 2 is not true to eliminate it.
response by the nurse would be best? amounts of “allergic” food items can trigger a response (sensitizing—challenging dose). Option
2 is incorrect—increasing fluids does not affect allergy development. Option 4 is incorrect
because the use of antihistamine medication may alleviate symptoms of allergic responses but
should not be used as a prophylactic measure in assisting dietary selection.

1.‐ "Eat only small amounts of a food if it has allergic potential."


2.‐ "Increase the fluid content of the diet to minimize risk of food allergies."
3.‐ "Avoid food items identified as potential allergens."
4.‐ "Take antihistamines to prevent allergic reactions and eat what you like."

4078 A client is placed on a low‐residue diet. The nurse Correct answer: 4 Milk and milk products are limited in low‐residue diets. All of the other diet selections can be A critical term is low‐residue. Recall knowledge of residue and systematically eliminate
evaluates that the client understood dietary used for this type of diet and indicate client understanding. options 1, 2, and 3.
instructions given if the client states that she will
refrain from eating which of the following favorite
food items?
1.‐ Orange juice
2.‐ Baked potatoes
3.‐ Toasted white bread
4.‐ Milk

4079 The nurse considers which of the following factors to Correct answer: 3 Presence of bowel sounds accompanied by passage of flatus indicates gastric motility and Critical words are progression of diet and postoperative. Recall peristalsis is temporarily
be necessary in order to allow progression of the diet return of "normal" GI functioning. Option 1 is incorrect because postoperative clients are stopped secondary to anesthesia. Determine safety to begin feeding is present with return
in a postoperative client? progressed in diet to assist in the restoration of ”normal” bowel activity. A bowel movement is of peristalsis, indicated by passage of flatus and bowel sounds to direct you to option 3.
not the initiating factor for diet progression. Option 2 is incorrect because abdominal
distention might indicate a potential problem affecting GI motility. Option 4 is incorrect since a
client's hunger is not the deciding factor in diet progression.

1.‐ Passage of a bowel movement


2.‐ Increased flatus production and slight abdominal distention
3.‐ Presence of bowel sounds and passage of flatus.
4.‐ Client reports feelings of hunger

4080 The nurse suggests which of the following diet Correct answer: 4 A full liquid diet contains all food items found on a clear liquid diet plus dairy products and The core concept in the question is a full liquid diet. Eliminate options 1 and 2 since they
selections for a client placed on a full liquid diet? prepared liquid formulas. Options 1 and 2 represent selections that are only found on a clear reflect clear liquids. Eliminate option 3 since it contains a solid food.
liquid diet. Option 3 represents a selection found on a low‐residue diet.

1.‐ Beef bouillon, cranberry juice, and tea


2.‐ Decaffeinated tea, gelatin, and ice pops
3.‐ Poached egg, coffee, and orange juice
4.‐ Plain yogurt and apple juice

4081 Which of the following should the nurse include as a Correct answer: 4 Transition (or progressive) diets are used on a short‐term basis to help the client move The critical words are primary focus and transitional diet. Note the similarity in the word
primary focus in the teaching plan for a client placed toward resumption of a regular diet pattern. A transition diet can progress rapidly from one transition in the question with the word temporary in option 4.
on a transition diet? meal to the next if the client tolerates the feedings. Option 1 is incorrect because a transition
diet is not given on a long‐term basis. Even though option 2 includes meal planning and diet
selection techniques, this is not the primary focus in establishing a plan of care for this client in
this short‐term therapy. Option 3 is important, but again, it is not the primary focus for this
short‐term therapy.
1.‐ The diet will be used on a long‐term basis.
2.‐ The focus of the diet is on meal planning and diet selection technique.
3.‐ The client should understand the types of food items that are restricted on this diet.
4.‐ This diet plan will be a temporary dietary measure.

4082 The nurse should include which of the following in a Correct answer: 2 Most clients with lactose intolerance can tolerate H cup milk at one time, and it provides a The critical words we lactose intolerant. Recognize the information in options 3 and 4
plan of care for a client who is mildly lactose calcium source. Option 1 is incorrect because the elimination of all dairy products can lead to would not be recommended to eliminate them. Recall small amounts may be tolerated to
intolerant? significant clinical deficiencies of other nutrients and may not be necessary. Option 3 is choose option 2.
incorrect because drinking milk on an empty stomach can exacerbate clinical symptoms.
Drinking milk with a meal may benefit the client because other foods (especially fat) may
decrease transit time and allow for increased lactase activity. Option 4 is incorrect because
although individual tolerance should be acknowledged, spreading out the use of known dairy
products will usually exacerbate clinical symptoms.

1.‐ Remove all dairy products from the diet.


2.‐ Consume only an H cup of dairy products at one time.
3.‐ Drink small amounts of milk on an empty stomach.
4.‐ Spread out selection of dairy products throughout the day.
4083 When caring for an elderly client who has difficulty Correct answer: 4 A mechanical soft diet can be used as part of a long‐term treatment plan because it includes Critical words in the question are long‐term and difficulty chewing. Recognize option 2
chewing, the nurse identifies which of the following most foods found on a regular diet, except the texture is modified to assist clients who have would not provide sufficient nutrients for long term and eliminate it. A low‐residue diet
diets to be most appropriate to use as a long‐term chewing problems. All of the other options reflect diets that should not be used on a long‐term (option 1) helps reduce diarrhea but is not appropriate for the client who has difficulty with
treatment measure? basis. Clear and full liquid diets are not nutritionally complete and are missing calories, protein, chewing. Long‐term high protein intake could also be harmful so eliminate option 3 as well.
vitamins, and electrolytes. In order to meet nutritional goals, a full liquid diet would require
additional source supplementation. Option 3 is incorrect because a long‐term high‐protein diet
can place additional renal demands on the individual client because of imposed solute loads.

1.‐ A low‐residue diet


2.‐ A full liquid diet
3.‐ A high‐protein diet
4.‐ A mechanical soft diet

4084 A client who receives intermittent enteral feedings Correct answer: 2 It is important to check residuals prior to intermittent feedings (and every 4 hours for A critical word is prior. Eliminate option 1 since it does not pertain to the feeding.
has a feeding ordered during the shift. Which of the continuous feedings) in order to evaluate if the client is able to process the feeding. Option 1 is Recognize option 3 would be dangerous to eliminate it. Note the similarity in the word prior
following interventions would the nurse perform prior incorrect: Going to the bathroom prior to a feeding will not affect the feeding status, and the in the question with the word before in option 2 to direct you to the correct answer.
to starting the next feeding? client may also not be able to physically comply with this request. Option 3 is incorrect: Placing
the client in this position is not warranted and can cause potential problems relative to
impaired feeding or potential aspiration. Although monitoring of the client's intake and output
for the last 24 hours is important, it is not as critical as checking for a residual at this point in
time.
1.‐ Have the client go to the bathroom in preparation for the enteral feeding.
2.‐ Check the residual before beginning the feeding and note the amount.
3.‐ Place the client in a lateral recumbent position to facilitate the feeding.
4.‐ Tally the client's intake and output for the past 24 hours.

4085 Which of the following nutritional interventions Correct answer: 3 The use of thickening agents is recommended for clients who have had CVA and have residual Focus on the critical words assistance, suggesting client should not be left alone, and CVA
would be of most assistance to a client who has deficits that affect swallowing. The thickening agents are added to maximize texture, facilitate to consider a risk for aspiration.
recently suffered a cerebrovascular accident (CVA)? the swallowing process, and minimize potential aspiration risks. Option 1 is incorrect because
bite‐sized portions of foods may increase the risk of aspiration if they are swallowed and
occlude the airway. The diet should be soft. Option 2 is incorrect. Merely placing the client on
a full liquid diet gives no indication that the client is being assessed for potential aspiration or
neurological deficits. Option 4 is incorrect because clients who are post‐CVA often encounter
this type of problem; therefore, they should be properly monitored and assessed.

1.‐ Have the client eat bite‐sized portions of foods to facilitate digestion.
2.‐ Place the client on a full liquid diet.
3.‐ Use thickening agents to minimize the risk of aspiration and monitor the client closely during all feedings.
4.‐ Allow the client to eat alone as post‐CVA clients are often self‐conscious about their residual deficits.

4086 The nurse would expect which of the following Correct answer: 1, 3 TPN is indicated for disease states such as trauma/stress, surgical interventions, and/or Focus on knowledge of indications for TPN, especially the long‐term indicator.
assigned clients to be candidates for total parenteral related pathology of GI tract or oncological conditions. TPN is more appropriate for long‐term
nutrition (TPN)? Select all that apply. nutritional support. All other options are clinical indications for enteral nutrition.

1.‐ A client suffering from severe trauma who is in a hypermetabolic state


2.‐ A client undergoing a cholecystectomy
3.‐ A client with short bowel syndrome
4.‐ A client undergoing radiation treatment for lung cancer
5.‐ A client recovering from a cerebrovascular accident (CVA)
4087 A client given a clear‐liquid diet tray prior to Correct answer: 4 It is important to follow test diet instructions prior to diagnostic testing to ensure reliability The critical word is best, indicating some options are correct, but one answers the
diagnostic testing states, "I'm not eating this and want and consistency of test results. Conveying the indication for the use of a clear liquid tray will question more thoroughly. Eliminate option 1 since it may offer false reassurance. Eliminate
some real food." Which of the following would be the help the client to understand the treatment plan and foster compliance. Although option 1 options 2 and 3 since they do offer the best explanation and do not adequately answer the
best response by the nurse? might represent an accurate statement, it will not help the client with to deal with the present client's question.
situation. Option 2 does not provide an adequate explanation to the client. Although option 3
is technically true, this response may serve to alienate the client as the nurse is not being
sensitive to the client's needs.

1.‐ "I am sure that you will have a regular diet after the test, so please just try some of this for now."
2.‐ "Would like me to get you some other type of broth or juice?"
3.‐ "This is the diet that has been ordered for you. It is the only diet you can have right now."
4.‐ "I understand this is not your usual diet, but it is needed to help the test establish a diagnosis."

4088 The nurse determines that which of the following Correct answer: 1 Increased fluid needs are indicated for a client who has burn injuries due to release of plasma Systematically evaluate each condition, recognizing fluid is retained in the conditions in
assigned clients has an increased need for fluid intake? fluids through tissue destruction. All of the other options reflect clinical conditions that require options 2 and 4 and eliminate them. Recall the extensive fluid losses and shifts that occur in
a decrease in fluid needs. burn injuries to direct you to option 1.
1.‐ A client who has burn injuries
2.‐ A client in cardiac failure
3.‐ A client receiving blood transfusions
4.‐ A client in the oliguric phase of renal failure

4089 The nurse has conducted discharge teaching for a Correct answer: 4 A clinical diagnosis of gout is associated with high uric acid levels in the body. Uric acid The critical word is gout. Recall physiology of this metabolic disorder and need to restrict
client diagnosed with gout. The nurse evaluates the represents the end product of purine catabolism in the body; therefore, foods that are high in foods high in purines to choose option 4.
client understood the instructions if the client states purines should be avoided. Anchovies are high in purine. All of the other options represent diet
he will refrain from eating which of the following selections that are low in purine.
favorite foods?
1.‐ Steak
2.‐ Poultry
3.‐ Dairy products
4.‐ Anchovies

4090 A client receiving total parenteral nutrition (TPN) via Correct answer: 24 The bag of TPN solution is changed every 24 hours. By reducing the number of times the TPN This question requires knowledge of factual information. Consider the common procedure
central venous catheter asks the nurse why the bag is tubing needs to be connected and disconnected from the central venous catheter, the risk of for TPN bag changes to formulate a response.
not changed as often as the IV bag of another client in infection is reduced, especially since high glucose levels in TPN can lead to bacterial growth.
the room, who has IV solution ordered at a rate of 150
mL/hr. The nurse responds that the TPN bag is
routinely changed every _____ hours. Write in a
numerical answer.

4091 Which one of the following diets would the nurse Correct answer: 3 A client being treated for dyslipidemia has an abnormal lipid profile that is high in cholesterol A critical word is dyslipidemia. Recall saturated fats contribute to lipid formation to
recommend for a client being treated for and triglycerides. The client needs a diet low in saturated fats with an increase in eliminate options 1, 2, and 4.
dyslipidemia? monounsaturated fats, small amounts of PUFA, and restricted sodium and hydrogenated food
products. Option 1 is incorrect because high intake of PUFAs will cause a further increase in
lipid levels. Option 2 is incorrect because protein‐controlled diets are usually indicated for
clients who have renal disease. Option 4 is incorrect because monounsaturated fats should be
increased, not decreased.
1.‐ High polyunsaturated fat diet
2.‐ Protein‐controlled diet with mild sodium restriction
3.‐ Low saturated fat diet
4.‐ Decreased monounsaturated fat diet
4092 The nurse uses which of the following rationales in Correct answer: 2 MAO inhibitor drug therapy can be complicated by excess intake of foods are high in tyramine Critical words are rationales and MAO inhibitors. Note option 1 does not address the
deciding to perform a dietary assessment on a client (i.e., an intermediate product of amino acid metabolism), such as chocolate and cheese. These medication to eliminate it. Eliminate option 3 since allergies are not the problem, and
who is taking monoamine oxidase (MAO) inhibitors? can alter drug action, resulting in hypertensive crisis. Although all of the other options are also eliminate option 4 because it is too general.
important, they are not the priority consideration when assessing a client on MAO therapy.

1.‐ It is important to determine food preferences since this may interfere with intake of nutrients.
2.‐ Certain foods, such as cheese or chocolate, can affect drug action, leading to serious complications.
3.‐ It is important to recognize potential food‐drug allergies because this may cause serious health problems.
4.‐ It is an expected assessment that is needed for any client.

4093 Which of the following strategies should the nurse Correct answer: 2 Visualization of portion sizes is an extremely important strategy whereby the client is shown Note that options 3 and 4 increase intake of an item, which is counterproductive to the
share with a client who wishes to avoid becoming graphic representations of what constitutes a serving size. Most often clients do not realize goal and eliminate them. Eliminate option 1 since it would also not contribute to weight
overweight? they are eating too large a portion, since traditional and "fast food" restaurants provide large loss.
servings. Option 1 is incorrect because following a low fiber diet may contribute to the
development of weight gain because there is little satiety value in the diet. Options 3 and 4 are
incorrect because an increase in other nutrients can lead to fat conversion during the
metabolic process if intake exceeds the body's individual needs.

1.‐ Maintain a low‐fiber diet.


2.‐ Use visualization of portion sizes to control intake.
3.‐ Limit fat and increase the other nutrients such as protein and carbohydrates.
4.‐ Increase the amount of carbohydrates in the overall diet.

4094 The nurse plans to monitor a client with malnutrition Correct answer: 2 Malnutrition leads to severe metabolic and physiological consequences, resulting in the Key words are malnutrition and consequences. Note that most of the options involve an
for which of the following consequences? inability to maintain adequate temperature regulation. A client with malnutrition is most likely increased or elevated condition which would be the opposite of what would occur with lack
to be found hypothermic upon physical examination. Option 1 is incorrect because of nutrient intake.
hypoglycemia is more likely to occur with malnutrition. Option 3 is incorrect as a decrease in
metabolic rate is usually seen with malnutrition. Option 4 is incorrect because immune
function is depressed due to loss of protein stores in malnutrition.

1.‐ Hyperglycemia
2.‐ Hypothermia
3.‐ Increased metabolic rate
4.‐ Increased immune function

4095 What advice can the nurse give to a client concerned Correct answer: 4 A comprehensive effort leads to the most effective long‐term management in regard to Key phrase is weight cycling. Note that option 4 is the only one to include exercise and
with weight control to help avoid the occurrence of weight control and will help to prevent the occurrence of weight cycling. Diet alone is not the recall the necessity of incorporating this into any weight loss maintenance to choose it.
weight cycling? answer to maintaining weight loss and avoiding the pitfall of weight cycling. If there are no
other lifestyle modifications, then it is more likely that the client will regain or even surpass the
initial starting weight. Although an increase in fluids is usually a beneficial choice in most diet
plans (unless the client has fluid restriction issues related to disease processes), the addition of
fluid will not help to prevent the occurrence of weight cycling. An altered meal pattern intake
consisting of shakes/supplements for one meal may not translate to a balanced eating pattern.
Again, if the client is not utilizing realistic goals and interventions, then resuming a "regular"
diet pattern may lead to weight cycling.

1.‐ "Follow your dietary plan exactly as ordered and do not make any adjustments."
2.‐ "Increase the amount of fluids in your diet to maintain hydration."
3.‐ "Use nutritional shakes/supplements in the place of one meal each day."
4.‐ "Incorporate dietary reduction measures and physical activity into your weight loss program."
4096 When a client asks why central obesity is considered Correct answer: 1 The presence of central obesity (intra‐abdominal fat/truncal obesity) is associated with an Key words are central obesity and risk factor. Recall that central obesity is also referred to
to be a health risk factor; which of the following replies increased cardiac risk (hypertension and stroke) and diabetes. The accumulation of intra‐ as an apple shape and eliminate option 3. Recall correlation to heart disease to choose
by the nurse would be best? abdominal fat leads to increased cholesterol levels because the liver converts them directly option 1.
into low density lipoproteins (LDL), a known risk factor for several disease processes. Option 2
is not clinically correlated as clients with cancer may present with cachexia or could be
underweight. Option 3 is incorrect because an increase in central obesity is associated with an
apple profile and it does not affect bone demineralization. Option 4 is incorrect since central
obesity is usually seen in clients who smoke or drink alcohol, and it does not relate to vitamin
deficiency.

1.‐ "Intra‐abdominal fat is associated with dyslipidemias, which increase risk of cardiovascular disease and diabetes."
2.‐ "Central obesity is associated with the development of cancer at several sites in the body."
3.‐ "Central obesity is associated with a pear‐like profile and this leads to bone demineralization over time."
4.‐ "Central obesity is seen in people who do not smoke or drink alcohol, but who tend to have vitamin deficiencies."

4097 A client informs the nurse he is trying to incorporate Correct answer: 4 Indoles are found in vegetables such as broccoli, cauliflower, and cabbage and offer Key word is indoles. Note that all the options are phytochemicals. Specific knowledge of
more phytochemicals in his diet. Which of the protection against carcinogen development. All the other choices are phytochemicals; soymilk indoles is necessary.
following menu choices made by the client indicate a and green tea provide isoflavones; carrots are high in carotenoids.
choice of indoles?
1.‐ One cup of soy milk
2.‐ One‐half cup of carrots
3.‐ One cup of green tea
4.‐ One‐half cup of broccoli

4098 A mother is concerned that her daughter is going to Correct answer: 4 It is essential to address the parent's concern as being important and to include the daughter Recognize that height and weight may indicate an overweight condition, but more
become obese as she is 5 feet tall and weighs 120 (client) in the plan of care. A comprehensive nutritional assessment is needed to support both information is needed to direct you to option 4.
pounds. The nurse should respond to the mother by of the family members. It is important to intervene, not merely react. Options 1 and 2 are
saying: incorrect because they are not based on nutritional evidence. Option 3 reflects an alarmist
attitude that could worsen the situation.
1.‐ "Children often grow out of their 'fat' stage, so there is little need to worry at this time."
2.‐ "It is too early to tell what your daughter's weight will be as an adult, so increase physical activity to prevent further weight gain."
3.‐ "This is indeed a serious concern and your daughter should probably be tested for diabetes."
4.‐ "A nutritional history should be done to assess typical intake pattern, physical activity level, and discuss body image concerns."

4099 When taking a diet history the client admits to eating Correct answer: 2 Garlic can inhibit platelet function and ultimately affect coagulation ability during and after Key word is garlic. Analyze each option for impact or interaction of garlic intake, recalling
several cloves of garlic daily for its cardioprotective surgery. Nonsteroidal anti‐inflammatory drugs and aspirin, but not acetaminophen, can the action this herb/food has on clotting ability and choose option 2.
effect. Which of the following client situations would potentiate the action of garlic. The other conditions would not be affected by garlic
be of most concern to the nurse? The client who: consumption.

1.‐ Is on strict bedrest.


2.‐ Is scheduled to have surgery.
3.‐ Has a history of migraine headaches.
4.‐ Takes acetaminophen for arthritic pain.

4100 Physical screening reveals a male client who is 6 feet Correct answer: 3 A client who is 6 feet tall with an average size frame should weigh roughly 178 pounds (106 Determine if the height and weight represent a normal, underweight, or overweight
tall and weighs 199 pounds. What information does for the first five feet with 6 pounds for every inch above). Taking into account body frame, condition.
this provide to the nurse in relation to a normal weight there could be a weight range from 168–188 pounds (small to large) that would be considered
pattern? normal. Option 1 is incorrect because clearly the client is overweight with a calculated BMI of
27. Option 2 is incorrect as the client would not be considered to be obese but rather
overweight. Option 4 is incorrect because the client clearly does not need additional calories in
the diet.
1.‐ The client's weight is appropriate for his height and therefore no further action is needed.
2.‐ Given the client's height, his weight is not appropriate and the client should be treated for obesity.
3.‐ It is important to also note the client's body frame, as this will impact evaluation of normal weight pattern.
4.‐ The client is underweight and needs additional calories in the diet.

4101 A client is planning to use a diuretic to lose extra body Correct answer: 1 It is important that a client receives accurate information about reported weight loss Key words are diuretics and weight loss. Note some of the options are partially correct,
weight. The nurse explains that diuretics: medication. Although diuretics promote fluid loss and are used in the clinical management of but do not address the concern of using diuretics for weight loss and be directed to option
disease states (such as congestive heart failure and pulmonary edema), they do not promote 1.
fat loss and are therefore not an effective weight loss measure. Option 2 is incorrect because
even though a prescription is required for the medication, it is not an effective weight loss
treatment. Option 3 is incorrect as diuretics do have a therapeutic effect on fluid loss in the
clinical setting in the management of disease states. Option 4 is incorrect because diuretics are
not prescribed as an adjunct therapy for weight loss but may be used as an adjunct in the
treatment of hypertension to promote fluid loss.

1.‐ Work to promote fluid loss, which is not the same as promoting fat loss.
2.‐ Require a prescription and long‐term therapy can be effective as a weight loss measure.
3.‐ Are used in the treatment of disease and have no therapeutic benefit on fluid loss.
4.‐ Should only be used as an adjunct to other weight loss programs.

4102 The nurse has defined a goal of a weight gain of 2 Correct answer: 2 In order to affect a two‐pound weight gain, a client would need an extra 7,000 kilocalories per Key words are weight gain of 2 pounds per week. Note options 1 and 4 do not provide
pounds in 1 week for an underweight client and week, which would correlate to 1,000 kilocalories per day. Option 1 is incorrect because it specific amounts. Recall the number of calories needed to gain one pound is 3,500 and
identifies an appropriate nursing intervention to be: does not give a specific amount of foods or calories. Option 3 is incorrect because 3,500 choose option 2.
kilocalories per week would also lead to an increase of one pound. Option 4 is incorrect since a
nutritional supplement may not provide the additional 1,000 calories per day needed.

1.‐ Incorporate more high fat and high carbohydrate foods into the diet.
2.‐ Increase caloric intake by 1,000 kilocalories per day for the week.
3.‐ Increase the caloric intake by 3,500 kilocalories per week.
4.‐ Have the client consume one high calorie nutritional supplement daily.

4103 The nurse evaluates that a client understands the Correct answer: 3 A recommended weight loss pattern for the obese client is 0.5 to 1 pound per week. Option 3 Critical words are healthy weight loss. Recognize that only 0.5 to 1 pound per week is the
need for a healthy weight loss pattern by indicating an offers the best possibility of maintaining weight loss. All of the other options are incorrect recommended healthy guideline to direct you to option 3.
intention to lose: because too great a loss may predispose the client to weight cycling or loss of lean body mass.

1.‐ 10 pounds in one month.


2.‐ 5 pounds in one week.
3.‐ 1 pound in one week.
4.‐ 3 pounds in one week.

4104 Which of the following statements would the nurse Correct answer: 1 VLCD diets are used in the clinical treatment of obesity under close medical supervision. The The critical word is best, indicating all or some options may be correct, but one answers
use to best describe a very low kilocalorie diet (VLCD) diet is low in calories, high in quality protein, and has a minimum of carbohydrates in order to the question more thoroughly, as is the case in option 1.
to a client? spare protein and prevent ketosis.
1.‐ "This diet is low in calories and high in protein and must used under close medical supervision."
2.‐ "This is a long‐term treatment measure that will assist obese people who can't lose weight."
3.‐ "The VLCD consists of solid food items that are pureed to facilitate digestion and absorption."
4.‐ "A VLCD contains very little protein."
4105 A 40‐year‐old female client with a family history of Correct answer: 2, 5 Carotenoids are phytochemicals that have been found to decrease risk of CAD and are found This question requires identification of foods high in carotenoids. If you have difficulty
coronary heart disease (CAD) expresses interest in in green, orange, red, and yellow fruits and vegetables. Option 1 is high in indoles. Options 3 remembering this specific information, the word carotenoid is similar to the word carrots,
modifying her diet to include foods to reduce and 4 are high in isoflavones. which are orange in color and would help direct you to options 2 and 5.
associated risk factors. The nurse suggests she increase
intake of foods high in carotenoids by including which
of the following? Select all that apply.

1.‐ Cauliflower and cabbage


2.‐ Strawberries and oranges
3.‐ Black and green tea
4.‐ Soy products
5.‐ Green peppers and squash

4106 When explaining to a group of adolescents why Correct answer: 4 Creatine has been demonstrated to improve the body's response in an exercise pattern The critical phrase is beneficial for athletic performance. Eliminate option 1 since a
creatine is being suggested as beneficial for athletic consisting of repetitive short‐term activities. Option 1 is incorrect: Creatine is supposed to catabolic effect would be a negative consequence. Recall use of creatine to muscle
performance, the nurse would explain that creatine: promote anabolism, not catabolism. Option 2 is incorrect because creatine has not been physiology to choose option 4.
proven effective for long‐term exercise patterns. Option 3 is incorrect because creatine does
not decrease lean body mass.
1.‐ Is an efficient fuel source in the body that promotes catabolic effects.
2.‐ Is effective for long‐term endurance performance.
3.‐ Decreases lean body mass and increases muscle strength.
4.‐ Assists the body in repetitive short‐term activity that requires energy bursts.

4107 A client states that he is considering using herbal Correct answer: 3 It is important to inform the healthcare provider at the start of herbal therapy, because this Read each option, systematically eliminating those with incorrect information regarding
therapy as a natural source to aid in dietary health. can prevent problems from potential drug interactions, verify indication for therapy, and herbal supplements. Recall importance of potential for interactions with herbal and OTC
What suggestion should the nurse give to the client to acknowledge client’s concerns over common complaints. Option 1 is incorrect since it is critical products to direct you to option 3.
assist with this decision? for the client to read all labels in order to be an informed consumer. Even though there are
standard products, herbal therapy ingredients can vary in different types of formulations. With
option 2, no prescription is required, but herbal therapy can cause a financial burden to the
client. Option 4 is incorrect since herbal therapy can cause side effects.

1.‐ "Herbal therapy treatments reflect standard doses so all similar products will provide the same biologic effect."
2.‐ "Herbal therapy requires a prescription and may be an expensive treatment modality."
3.‐ "It is important to inform your healthcare practitioner about your choice to start herbal therapy."
4.‐ "Herbal therapy is a natural form of treatment with very few side effects."

4108 The diet history of client receiving anticoagulants Correct answer: 4 Garlic is a food/herbal product that has long been recognized for its health benefits. It lowers Critical words are garlic and anticoagulants. Recall garlic's effect on clotting function to
indicates the client eats a lot of garlic. The nurse plans cholesterol/triglycerides, improves immune function, and decreases blood pressure. But garlic direct you to option 4.
to include which of the following information when can inhibit platelet aggregation and therefore prevents blood clot formation. A client who is
teaching the client about possible drug/food taking anticoagulation therapy should be advised of potential interactions with excessive
interactions? amounts of garlic in the diet. Option 1 is incorrect since there is an increased risk of bleeding.
Option 2 is incorrect because garlic does indeed affect blood coagulation. Even though garlic
does help to support immune function (option 3), this fact does not directly relate to
anticoagulation therapy.

1.‐ Garlic can enhance the coagulation process and accelerate clot formation.
2.‐ Garlic has no effect on blood coagulation.
3.‐ Garlic helps to support immune function but does not affect coagulation.
4.‐ Bleeding can occur because garlic inhibits platelet aggregation.
4109 The nurse encourages eating foods high in which of Correct answer: 2, 5 Isoflavones and phytoestrogens are deemed beneficial in protecting female clients from Critical words are phytochemicals and osteoporosis. Recall isoflavones and estrogen have
the following phytochemicals as a possible preventive developing osteoporosis due to their estrogen‐like enhanced effects. Option 1 is incorrect: a beneficial effect on bones to direct you to options 2 and 5.
measure for a female client concerned about Phenolic acids are effective against cancers because they act as pro‐oxidants. Option 3 is
developing osteoporosis? Select all that apply. incorrect as indoles make estrogen less effective. Option 4 is incorrect because carotenoids are
considered to be in the classification of phenolic acids and help to decrease cancer risks.

1.‐ Phenolic acids


2.‐ Isoflavones
3.‐ Indoles
4.‐ Carotenoids
5.‐ Phytoestrogens

4110 Which of the following does the nurse discuss with Correct answer: 3 It is important the client demonstrate an understanding of the basics of the treatment Recognize the need to reduce caloric intake as well as increase activity level to eliminate
the client — who is 5 feet 6 inches tall and weighs 250 program, focusing on a multifaceted approach of intake, physical activity, and weight control. option 1. Eliminate option 2 since it does not provide specific suggestions, and eliminate
pounds — as a realistic goal to start a weight loss Each one of these is an interrelated variable that affects the client's ability to achieve and option 4 since this caloric restriction is unsafe.
program? maintain weight control. Option 1 is incorrect because this may not be prudent; physical
activity is usually increased. Option 2 is incorrect since it is not wise to utilize this feeding
pattern because it may contribute to weight gain. Option 4 is incorrect: The total calories may
be somewhat low, but the percentage of calories from fat is too high to effect substantial
weight loss.
1.‐ Maintain present physical activity level and decrease caloric intake.
2.‐ Eat only when hungry.
3.‐ Demonstrate understanding of caloric intake, weight control, and physical activity.
4.‐ Maintain an intake of 1,000 calories or less with 30 percent of calories from fat.

4111 Which of the following instructions should the nurse Correct answer: 2 Anesthesiologists recommend that all herbal and botanical preparations be stopped 2–3 The core concept is preparation for outpatient surgery. Recall many drugs and herbs may
give to a client who takes numerous herbal weeks prior to scheduled surgery in order to minimize possible interactions between take several days to be eliminated from the body to direct you to option 2.
supplements and is being scheduled for an outpatient anesthesia induction and blood pressure response. Options 1 and 3 are incorrect for the
surgical procedure? reason just stated. Even though it might be good to discuss herbal therapies with a dietitian
(option 4), it is more important to acknowledge that herbal therapies should be stopped prior
to scheduled surgeries to minimize anesthesia risks.

1.‐ "Take your supplements as usual until the morning of the surgical procedure."
2.‐ "Stop herbal and botanical products at least 2–3 weeks prior to the day of surgery."
3.‐ "Maintain usual doses of herbal products and stop a day before surgery."
4.‐ "Consult with the dietician prior to surgery about effects of herbal therapies."

4112 The nurse instructs the underweight client to Correct answer: 500 An extra 3,500 kilocalories per week is needed for a 1‐pound weight gain (500 kilocalories per Focus on critical information of 3,500 kilocalorie equaling 1 pound of fat.
consume an extra _____ kilocalories per day for a 1‐ day). Weight gain strategies revolve around consuming foods that provide many kilocalories in
pound weight gain per week. Write a numerical small volume along with building muscle.
answer.

4113 A 53‐year‐old male comes to the clinic for a routine Correct answer: 3 Obesity is defined by BMI of 30 or above with no co‐morbid conditions. It is calculated by Note the question indicates that client has a high weight and is not very tall. This is a clue
physical examination. During the intake screening, the utilizing a chart‐nomogram that plots height and weight. This client’s BMI is 35, indicating that options 1 and 3 are incorrect. Recall the categories for BMI to choose between options
nurse notes the client's weight is 216 pounds and obesity. The other responses represent inaccurate interpretations of the client's BMI. 3 and 4.
height is 66 inches. The body mass index (BMI) is
calculated at 35. The nurse interprets that the client's
weight:
1.‐ Is within normal limits, so a weight reduction diet is unnecessary.
2.‐ Is lower than normal, so education about nutrient dense foods is needed.
3.‐ Indicates obesity, so weight reduction and exercise are needed.
4.‐ Indicates an overweight status, so dietary modification and exercise is needed.
4114 When evaluating a client for malnutrition, the nurse Correct answer: 1 Objective anthropometric measurements such as triceps skin fold and mid‐arm circumference Critical words are malnutrition, evaluation, and nutritional status. Note options 2 and 3
utilizes which of the following to provide information (MAC), along with weight, are usually used to diagnose malnutrition. While all of the other measure glucose levels only to eliminate them. Recognize option 4 is used to measure
about the client's nutritional status? choices represent tests that may provide useful information, they also may be affected by coronary artery disease risk and eliminate it.
variables other than malnutrition.
1.‐ Triceps skin fold measurement
2.‐ Fasting blood glucose level
3.‐ Hemoglobin A1c level
4.‐ Serum lipid profile results

4115 A client states that he has been told ketchup is a Correct answer: 2 Ketchup contains lycopene, which is a phytochemical that has health benefits. Option 1 does The critical word is beneficial. Recognize what health and nutritional benefits are provided
beneficial food and asks how can this be true. What not address the concern about health benefits. Option 3 is incorrect because even though by lycopene in ketchup to direct you to option 2.
would be the nurse's best response to answer the ketchup contains a large amount of sodium, this is not a health benefit and even can be viewed
client’s concern? as a "hidden" source of sodium in the diet if used in excess. Option 4 is incorrect since ketchup
is not a source of fiber in the diet.
1.‐ "Ketchup is considered to be a food enhancer and intensifies taste perception."
2.‐ "Ketchup contains lycopene, which has been shown to be effective against heart disease and prostate cancer."
3.‐ "Ketchup contains sodium, which provides health benefits."
4.‐ "Ketchup provides a fiber source in the diet that protects against cancer."

4116 The nurse interprets that which of the following Correct answer: 2 A waist‐to‐hip ratio of greater than 1.0 in a male client indicates an increased risk to develop The critical words are increased risk and obesity. Recall norms of BMI and waist‐to‐hip
clients is at increased risk to develop obesity? obesity. This indicates a larger amount of abdominal fat and correlates with an apple body ratio to choose option 2.
shape. Option 1 is incorrect because a decrease in visceral fat stores in the abdomen would
improve a client’s health status. Option 3 is incorrect since a 19 BMI is associated with being
underweight. Option 4 is incorrect because a male client who is 6 feet tall and 162 pounds has
a BMI of 22, which is considered within normal range.

1.‐ A client with decreased visceral fat stores in the abdomen


2.‐ A waist‐to‐hip ratio of greater than 1.0 in a male client
3.‐ A BMI of 19 in a female client
4.‐ A male client who is 6 feet tall and weighs 162 pounds

4117 A client has heard information about functional foods Correct answer: 3 Using vegetables as a main ingredient will help to increase the amount of functional foods Specific knowledge of functional foods is required to answer this question. If you cannot
and asks how he can include them in his diet. Which of that have phytochemical activity. Option 1 is incorrect: Even though milk is a good source of recall this material, note the question addresses what food should be included. Eliminate
the following suggestions should the nurse provide? vitamin D and calcium, it will not by itself increase the amount of functional foods in the diet. option 2, since the food is eliminated and option 4, since salt is not a food, but a seasoning
While limiting the amount of refined food products is beneficial (option 2), this does not and electrolyte.
increase the amount of functional foods in the diet. Option 3 is incorrect: Seasoning to taste
may be important with regard to sodium level; however, it does not specifically relate to
functional foods.
1.‐ Increase milk in the diet.
2.‐ Limit refined food products in the diet.
3.‐ Use vegetables as main‐dish ingredients.
4.‐ Season food to taste with salt.

4118 The nurse would make which of the following Correct answer: 3 Leptin is a protein hormone that is secreted by adipose tissue; it is called the obesity gene. Specific knowledge of leptin is necessary to answer this question correctly.
responses when questioned by a client about the Leptin increases the total fat mass in obese clients. Option 1 is incorrect because the presence
leptin's role in the body? of leptin usually decreases food intake in individuals of normal weight. Option 2 is incorrect
because it does not affect the regulation of steroid hormones but does have some effect on
insulin release.
1.‐ "It increases food intake in clients of normal weight, thereby promoting obesity."
2.‐ "It assists in the regulation of steroid hormones."
3.‐ "It increases the total fat mass of people who are obese."
4.‐ "It decreases the total fat mass in the body of those who are obese."
4119 The nurse is assessing a client who is taking orlistat Correct answer: 2 A side effect of orlistat, a lipase inhibitor that aids in weight loss, is rectal incontinence, Note the question asks for an indication that the drug is taken incorrectly, therefore
(Xenical) for weight reduction. The nurse suspects and/or oily stools that are associated with urgency. The fact that the client is presenting with indicating three of the options will have correct information regarding use of orlistat.
incorrect use of the drug when the client reports which complaints of this symptom suggests he or she is not following the treatment regimen and is
of the following? eating high‐fat meals. Options 3 and 4 reflect compliance with the treatment regimen.
Increasing fluid intake does not affect compliance. Systematically eliminate options 1, 3, and 4
since the behaviors are correct.

1.‐ Increasing fluid intake to 8–10 glasses/day


2.‐ Urgent bowel movements since starting the medication
3.‐ Taking fat‐soluble vitamin supplements 2 hours after the medication
4.‐ Taking the medication with meals and following a low‐fat diet

4120 When performing a nutritional assessment on a Correct answer: 4 A BMI of less than 18.5 is considered to represent a client who is underweight and possibly at The critical word is underweight. Eliminate option 2 since it reflects an overweight status.
female client, which of the following data would risk for malnutrition. Option 1 is incorrect because a BMI of 22 is considered within normal Recall knowledge of BMI to choose option 4.
indicate to the nurse the client is underweight? range. Option 2 is incorrect because being above IBW is not consistent with being
underweight. Option 3 is incorrect because a waist/hip ratio of less than 0.8 in a female client
represents a normal finding. It is important for the nurse to be aware of objective
anthropometric measurements (both normal and abnormal values) so that the data can be
interpreted adequately.
1.‐ A body mass index (BMI) of 22
2.‐ Being 0.5% above ideal body weight (IBW)
3.‐ A waist/hip ratio of less than 0.8
4.‐ A BMI of 18.

4121 The nurse determines which one of the following Correct answer: 3 Pharmacotherapy for obesity is indicated when a client has been unable to achieve weight Critical words are pharmacotherapy and obesity. Read each stem and determine history of
clients should consider pharmacotherapy for the loss of 1 pound/week after 6 months of therapy while following suggested lifestyle changes. weight loss attempts and risk factors. Eliminate option 1 since this reflects an unhealthy
treatment of obesity? Option 1 is incorrect: This is not a quick process decision. Option 2 is incorrect because the choice. Eliminate option 2 since the client has already lost weight. Choose option 3 since
client has already demonstrated a reasonable weight loss pattern. Option 4 is incorrect the client has had unsuccessful attempts with other modalities.
because pharmacotherapy is not indicated for an adolescent client.

1.‐ A 20‐year‐old college student who wants to lose weight quickly


2.‐ A 32‐year‐old male who has lost 20 pounds during the past 6 months.
3.‐ A 28‐year‐old female who has been trying unsuccessfully to lose weight for 6 months following changes in lifestyle and exercise.
4.‐ A 12‐year‐old adolescent female who is tired of being overweight and does not have a social network of supportive friends.

4122 A client with renal insufficiency expresses interest in Correct answer: 2 A client with renal insufficiency should not start a low CHO diet—this implies that protein and The critical words are renal insufficiency and low CHO diet. Recall nutritional needs related
starting a low‐carbohydrate (CHO) diet. The nurse fat levels will be increased, resulting in an increased renal solute load. Option 1 is to decreased renal function in regards to restriction of protein and need for CHO as a
discourages the client from doing so, recognizing it incorrect—30 grams of CHO is not enough to spare protein, thereby making fat the primary caloric source.
could have which of the following implications? energy source and leading to the development of ketosis, which will further compromise the
client's clinical status. Option 3 is incorrect—osteoporosis is not associated with being on a low
CHO diet but rather is due to multifactorial losses involved with calcium. Option 4 is incorrect
because a client with renal insufficiency is unable to handle protein. While high biological value
sources are warranted, the protein intake must be monitored cautiously to prevent further
metabolic imbalances.

1.‐ As long as the client eats a minimum of 30 grams of CHO/day, there should be no problem.
2.‐ The client's clinical condition is a contraindication to starting a low CHO diet.
3.‐ Calcium supplements should be utilized to prevent the development of osteoporosis while on a low CHO diet.
4.‐ As long as the client eats foods that are high biologic protein sources, there will be no problems with following a low CHO diet.
4123 The nurse develops which goal as most realistic for a Correct answer: 3 A client diagnosed with Pickwickian syndrome is typically clinically obese and has Critical words are Pickwickian syndrome and most realistic. Recall aspects of this
client who is being referred for dietary counseling and hypoventilation symptoms. A realistic goal is to establish a weight loss plan because it will help syndrome to be directed to option 3.
has been diagnosed with Pickwickian syndrome? to improve breathing, relieve respiratory symptoms, and decrease the workload placed on
both the heart and lungs. Option 1 is incorrect because maintaining the client's current weight
will not help to improve clinical symptoms. Option 2 is incorrect as increasing caloric intake
will further contribute to weight gain and affect respiratory status. Although the use of
increased fluids may be helpful to thin secretions, the client with Pickwickian syndrome does
not present with thick secretions, but rather has disturbances with sleep apnea.

1.‐ Maintain current body weight.


2.‐ Increase caloric intake by 2,000 calories.
3.‐ Follow prescribed weight loss plan.
4.‐ Increase fluids to thin secretions and maintain respiratory effort.

4124 A client who has received a kidney transplant is being Correct answer: 2 Azathioprine (Imuran) can cause a client to develop esophageal lesions; therefore, a soft diet Critical words are Imuran and side effects. Recall the side effect on the esophagus to be
treated with azathioprine (Imuran) post‐transplant as would be an appropriate choice if the client had noted side effects from the medication. directed to option 2.
part of the immunosuppressive regimen. When the Option 1 is incorrect because vomiting and diarrhea are also commonly seen side effects from
client experiences drug side effects the nurse this medication and would only serve to increase symptoms. Option 3 is incorrect as fresh fruit
encourages which dietary change? would not be an appropriate choice due to high bacteria content. Option 4 is incorrect because
orange juice might be irritating to a client who had esophageal erosions and/or not be
tolerated if a client was experiencing vomiting.

1.‐ Increased fluids


2.‐ Soft diet
3.‐ Fresh fruits
4.‐ Orange juice

4125 When caring for a client who has experienced major Correct answer: 2 A client who has experienced major trauma is under severe stress and experiences a The critical terms are major trauma and decrease. Recall that the physiology of trauma will
trauma, the nurse develops a plan of care to decreased release of insulin, leading to a hyperglycemic response. The other options are necessitate an increase of calories and compensatory mechanisms will kick in to
incorporate the client's anticipated decrease in: incorrect because caloric needs, aldosterone, and ADH would all be increased during periods of compensate for fluid losses to eliminate options 1, 3, and 4.
severe stress.
1.‐ Kilocalorie needs.
2.‐ Release of insulin.
3.‐ Aldosterone secretion.
4.‐ Antidiuretic hormone (ADH) secretion.

4126 Which of the following laboratory results should the Correct answer: 2 A client at risk for re‐feeding syndrome is likely to experience decreases in serum potassium, Note that all options have two parts and both parts must be correct. Recall that re‐feeding
nurse monitor for in a client at risk for re‐feeding magnesium, and phosphorus. All of the other choices reflect options that are not seen with re‐ syndrome involves a drop in electrolyte levels, allowing elimination of options 1, 3, and 4.
syndrome? feeding syndrome, such as hyperkalemia, hypernatremia, and hypercalcemia.

1.‐ Hyperkalemia and hyponatremia


2.‐ Hypokalemia and hypophosphatemia
3.‐ Hypernatremia and hyperkalemia
4.‐ Hypokalemia and hypercalcemia

4127 An elderly client comes in for a routine visit to the Correct answer: 3 The elderly client who has been given a prescription for antibiotics and plans to take them Recall that the bioavailability of medications is affected by the presence of food and
clinic for renewal of prescriptions. The nurse identifies following meals is more likely to be at risk for diet/medication interactions because absorption gastric pH to be directed to option 3.
the client might be at risk for diet/medication of the antibiotic may be reduced due to food intake. All of the other options do not pose
interactions because the client: significant risk for diet/medication interactions.
1.‐ Takes a daily multivitamin supplement each in addition to prescribed medications.
2.‐ Drinks two cups of coffee each day.
3.‐ Has been given a prescription for antibiotics and plans to take them following meals.
4.‐ Is taking three different types of blood pressure medication.
4128 A 45‐year‐old female who is 5 feet 4 inches tall and Correct answer: 2 A client who has undergone severe stress will have increased energy needs. Using the Harris‐ Critical words are severe trauma and energy needs. Recall the need for additional calories
weighs 180 pounds has undergone severe trauma. In Benedict equation to estimate basal energy expenditure (BEE) requires the client's actual for tissue repair and the need to be calculated on weight.
determining the client's energy needs the nurse places weight and height. Depending on the nature of the stress, additional calories will be estimated
highest priority on: ranging from 20–100 percent. Option 1 is incorrect because the client's IBW is not sufficient to
base calculations on and will lead to a false starting point. Option 3 is incorrect as a 20 percent
increase is not appropriate for a client who has undergone severe trauma; a much higher
percentage would be required to meet the client's energy needs. Option 4 is incorrect because
even though albumin levels are needed, they are not part of the client's energy calculation.

1.‐ Estimating caloric intake based on ideal body weight (IBW).


2.‐ Recording actual body weight.
3.‐ Increasing caloric intake by 20 percent above calculated rate.
4.‐ Checking serum albumin levels.

4129 When caring for a client with acute respiratory failure Correct answer: 4 A client on mechanical ventilation as a result of acute respiratory failure is already Critical words are acute respiratory failure and mechanical ventilation. Note options 1, 2,
who is placed on mechanical ventilation, the nurse experiencing fatigue and exhaustion from the work of breathing. The use of nutrient‐dense and 3 are less specific to the needs of the client with a respiratory condition and eliminate
anticipates the use of which nutritional measure to pulmonary formulas to support the client is advised at this time because they provide fewer them.
provide the client with the most effective nutritional carbohydrates and a greater fat content to minimize the production of carbon dioxide
support? (CO&lt;sub&gt;2&lt;/sub&gt;). Option 1 is incorrect because high carbohydrates lead to an
increase in CO&lt;sub&gt;2&lt;/sub&gt; production correlating with acidosis. Option 2 is
incorrect as parenteral fluids are not being given to loosen secretions; parenteral fluids are
used to correct fluid and acid‐base imbalances. Option 3 is incorrect because increased
kilocalories above client needs could lead to overfeeding that could further increase
CO&lt;sub&gt;2&lt;/sub&gt; levels.

1.‐ Use of high carbohydrate solutions to meet nutrient needs


2.‐ Increased fluids through the parenteral route to loosen secretions
3.‐ Increased kilocalories to meet energy and protein needs
4.‐ Use of nutrient‐dense pulmonary formulas to support client

4130 A client recently started on sodium warfarin Correct answer: 4 The dosage of Coumadin is regulated by the client's prothrombin time (PT). Excessive intake Critical words are Coumadin and spinach salads. Recall that spinach is high in vitamin K and
(Coumadin) has received teaching on drug‐food of foods rich in vitamin K, such as spinach, could prolong the PT. A consistent daily intake recognize the effect it has on blood clotting to be directed to option 4.
interactions. When the client informs the nurse he should not affect the PT, whereas a fluctuating amount eaten each day would impact the
loves to eat spinach salads, the nurse should make clotting time. It is not necessary to totally omit foods with vitamin K and the other factors
which response? would not affect the clotting.
1.‐ "Unfortunately you will not be able to eat them while taking Coumadin."
2.‐ "Be sure you eat the salads at least 12 hours before or after taking the Coumadin."
3.‐ "It is okay to eat them as long as you use an oil‐based dressing."
4.‐ "As long as you eat a consistent amount of foods containing vitamin K each day it is OK."

4131 When caring for a client with sickle cell crisis, which Correct answer: 4 The sickled shape of the red blood cell (RBC) can lead to occluded circulation and impaired The critical concept is sickle cell crisis. Recall the shape of the RBC in this condition and the
dietary measures should the nurse emphasize? RBC production. Adequate hydration is essential to improve blood flow, reduce pain, and impact it has on the circulatory system to be directed to option 4.
prevent renal damage. There are no specific restrictions or recommendations in regards to
protein, sodium, or frequency of meals.
1.‐ High‐protein supplements
2.‐ Limited intake of sodium‐rich foods
3.‐ Six small meals eaten throughout the day
4.‐ High fluid intake
4132 A client taking sodium warfarin (Coumadin) for the Correct answer: 3 A client who is taking an anticoagulant should not be drinking excessive amounts of alcohol. The critical word is Coumadin. Recall the purpose and food and drug interactions of this
past several months following valve replacement The fact that the client has been consistently drinking two glasses of wine is a concern since drug to be directed to option 3.
comes into the clinic for a check‐up. Which of the alcohol can enhance the effects of the medication. This information bears further review by
following findings by the nurse would trigger concern the nurse with communication to the healthcare provider. All of the other options do not
regarding compliance with medication therapy? indicate any concern regarding compliance with medication therapy.

1.‐ The client has been following a well‐balanced diet for the past two months.
2.‐ The client has been going out to eat at a restaurant once a week.
3.‐ The client has been drinking two glasses of wine each night.
4.‐ The client has a salad with his evening meal twice a week.

4133 A client taking warfarin (Coumadin) has had difficulty Correct answer: 1 Mayonnaise and salad oils are high in vitamin K, which will prolong the anticoagulation effects Critical words are warfarin and dietary factors. Recall type of drug warfarin is and
regulating prothrombin times. Which of the following of warfarin. CHOs and fruit juice are not high in vitamin K. Option 2 does not directly affect recognize dietary factors that would impact this drug. Eliminate option 2 since it would
questions should the nurse ask to identify dietary coagulation. affect coagulation. Recall foods with vitamin K content to direct you to option 1.
factors that could influence the drug levels?

1.‐ "Do you use a lot of mayonnaise and salad oils?"


2.‐ "How many times a day do you eat?"
3.‐ "Do you eat a diet high in carbohydrates (CHOs)?"
4.‐ "Are you drinking a lot of fruit juices?"

4134 A 68‐year‐old male client who has longstanding Correct answer: 4 A client who has frequent hospitalizations due to chronic disease is likely to exhibit signs of Note the nature of the question is psychosocial. Review options for the one which best
diabetes with resultant complications has been sadness, depression, and loss of control regarding the disease process. The nurse should allow addresses clients refusal to eat to choose option 4.
hospitalized with recurring frequency over the years the client to vent his feelings in the hopes of sharing concerns and offering emotional support.
and is extremely upset over this most current Option 1 is incorrect, although it is important to ascertain a client's food preference, the
admission. The client does not want to eat and information provided states that the client is repeatedly pushing away the food tray at several
repeatedly pushes away the food tray at each meal. meals. Option 2 is incorrect because this client behavior presents as a continued pattern and
How should the nurse best respond to the client's not an isolated incident. Therefore, the nurse should do more than try again with the next
actions? meal to get the client to eat. Option 3 is incorrect because although it might be important to
consult with the dietitian regarding food selections, this option does not address the
immediate problem of the client pushing away the food trays.

1.‐ Tell the client that you can get him something else that would be more to his liking.
2.‐ Take the tray away and try again with the next meal to get the client to eat.
3.‐ Consult with the dietitian to improve client compliance.
4.‐ Ask the client to share with you why he doesn’t feel like eating.

4135 The nurse is caring for a 32‐year‐old female client Correct answer: 4 The first priority with any trauma client is to establish baseline information by assessing skin Critical words are multiple trauma and priority. Recognize need to obtain baseline data to
with multiple traumas. The nurse identifies which of turgor, vital signs, and review of pertinent diagnostic tests in order to stabilize the client and determine specific needs of clients to direct you to option 4.
the following as a priority intervention for nutritional determine the extent of injuries. Looking at the client's physical and diagnostic presentations
support? will help to determine the client’s fluid balance and pertinent stressors. Option 1 is
incorrect—even if the client is hungry, the existing trauma condition may preclude any feeding
attempts at this time. Although it is important to establish IV access for a trauma client, the
nurse cannot increase the rate without a physician's order, therefore option 2 is incorrect.
More importantly, increasing the IV rate will not help to maintain nutritional status but rather
will help to restore fluid balance. Option 3 is incorrect because the client's underlying trauma
may require surgical intervention and therefore the client should be kept NPO until the exact
extent of injuries is known.

1.‐ Determine if the client is hungry and what her favorite foods are.
2.‐ Provide IV access and increase fluid rate to maintain nutritional status.
3.‐ Offer sips of water and ice chips as tolerated.
4.‐ Assess skin turgor, vital signs, and diagnostic test results before seeking a diet order.
4136 When caring for a client diagnosed with multiple Correct answer: 2 A client who is diagnosed with MODS in an ICU setting is critically ill. Determination of overall The core concept is MODS. Eliminate options 1, 3, and 4 since they focus on one type of
organ dysfunction syndrome (MODS), the nurse would health status is usually reviewed using the APACHE scoring, which provides relative parameter, which would be insufficient to identify nutritional needs.
utilize which of the following parameters to determine information regarding risk of mortality. APACHE scoring looks at acute physiological indicators,
overall nutritional status? age, and presence of chronic health conditions to evaluate a client’s response and prognosis.
Option 1 is incorrect—plasma osmolality does not serve as an indicator for acute physiology
scoring. Option 3 is incorrect—these choices only reveal information about the client’s age and
possibly chronic respiratory health problems. Option 4 is incorrect because a CBC with
differential does not provide comprehensive information about the client’s overall health
status.
1.‐ Plasma osmolality and presence of chronic disease process
2.‐ Age of client, presence of chronic health conditions, and physiologic parameters
3.‐ Rectal temperature and respiratory rate
4.‐ CBC with differential, electrolyte panel, and liver function tests

4137 A 42‐year‐old male client says, "I am an alcoholic. I Correct answer: 2 It is important to provide a client with the most comprehensive information available to Note the psychological nature of the question as well as need to clarify misinformation.
heard that drinking is good for you in terms of answer questions and clarify concerns. The consumption pattern (i.e., type and amount) of Eliminate the assumption made by Option 1. And eliminate options 3 and 4 as they do not
preventing heart disease, so why should I stop?" What alcohol has been shown to be a risk factor in many disease processes—this should be clearly address the client's question.
is the best response of the nurse to the client's stated to the client. Option 1 is incorrect because even though moderate alcohol consumption
statement? has been documented to provide some cardiovascular benefits, the last portion of the
statement indicates bias relative to the client’s alcoholism. Option 3 is incorrect because it
does not address the client’s concern at this time. Option 4 is incorrect because it implies that
the nurse does not believe the client's statement that he is an alcoholic. It is important for the
nurse to respond to the question asked before delving further into confirming a diagnosis of
alcoholism.

1.‐ "Alcohol is only beneficial if used in moderate amounts; so this fact does not apply to your situation."
2.‐ "Alcohol is a risk factor in many disease processes and the type and amount of alcohol consumed do have significant health consequences."
3.‐ "Are you interested in seeing a counselor for help with alcohol dependency?"
4.‐ "Will you complete a CAGE questionnaire so we can accurately determine if you really are an alcoholic?"

4138 The nurse determines which of the following clients Correct answer: 4 A 48‐year‐old female client with the contributory diseases of hypertension (HTN) and Read each option, identifying risk factors that would contribute to nutritional problems.
would be at the greatest risk for nutritional problems? diabetes is at a greater risk for nutritional problems than any of the other clients. HTN and Eliminate options 1 and 3 since each contains only one risk factor. Choose option 4 since
diabetes both have dramatic effects on vascular status and lipid physiology. Option 1 is this client has two chronic conditions.
incorrect because there is no information to suggest that the client has any underlying health
problems. In option 2, even though the client is obese, this represents only a single‐risk factor
for nutritional status. Similarly, the client in option 3 has only a single‐risk factor, that of
asthma.
1.‐ A 42‐year‐old male admitted with a fracture of the femur as a result of a fall
2.‐ A 36‐year‐old obese female client admitted for pyelonephritis
3.‐ A 20‐year‐old female client admitted with exacerbation of asthma
4.‐ A 48‐year‐old female client who has a history of hypertension (HTN) and diabetes admitted for abdominal pain
4139 The nurse determines that a client taking sodium Correct answer: 4 A client taking warfarin should be aware of pertinent medication facts prior to initiation of Critical words are complications and warfarin. Recall this drug is an anticoagulant that
warfarin (Coumadin) is being compliant with treatment therapy to avoid possible interactions and to maintain an adequate anticoagulation response. requires maintenance of consistent blood levels and eliminate options 1 and 2 since they
if the client: The client should eat a well‐balanced diet and consume a constant amount of vitamin K that would alter steady blood levels. Choose option 4 since it addresses the most significant
can interfere with the action of the medication. Green, leafy vegetables are high in vitamin K. dietary information regarding warfarin.
Option 1 is incorrect—the client should not double the dose because this can lead to severe
consequences and altered coagulation. Option 2 is incorrect because a client taking warfarin
should not take other medications unless the physician prescribes them. Option 3 is
incorrect—even though it is important for the client to take the medication at the same time
every day, it is also critical that the client be aware of the dosage. This information should be
related as part of the client's pertinent medical history and can influence medical treatment by
other healthcare providers.

1.‐ Doubles the dose when he misses a pill.


2.‐ Is taking OTC cold medication in addition to warfarin therapy.
3.‐ Takes the pill at the same time every day but is not aware of the dosage.
4.‐ Maintains a balanced diet and eats a constant amount of green, leafy vegetables.

4140 When caring for a client who has had a liver Correct answer: 3 Early tube feeding leads to fewer complications than parenteral feedings in the acute post‐ The critical words are post‐transplant needs and liver. Recall protein needs in catabolic
transplant, the nurse would consider which of the transplant period and is the preferred method if the client has a functioning GI tract. In options and acute stress conditions to eliminate options 1 and 2. Eliminate option 4 since these
following when planning for the clients' post‐ 1 and 2, there is increased protein catabolism in the acute post‐transplant period as well as an acids are decreased in the post‐operative state. If you had difficulty with this question,
transplant nutritional needs? increase in the amount of urinary nitrogen. Option 4 is incorrect because the presence of end‐ review content on the post‐transplant client.
stage liver disease is associated with a decrease in the amounts of branched chain amino acids,
leading to an alteration in aromatic amino acids. This can further contribute to the
presentation of hepatic encephalopathy.

1.‐ There is a decrease in protein catabolism.


2.‐ Urinary nitrogen losses stabilize in the acute post‐transplant period.
3.‐ Early tube feeding is favored over parenteral feeding.
4.‐ Branched chain amino acids are found in adequate amounts in the post‐transplant period.

4141 The nurse will include dietary teaching on what Correct answer: 1 Adequate amounts of micronutrients (e.g., vitamins A and C, calcium, and zinc) will help The core issue of the question is knowledge of which micronutrients play a key role in
micronutrient to help support the client's immune support immune function and restore healing in the trauma client. immune function and healing. Use this information and the process of elimination to make
function and restore healing? a selection.
1.‐ Vitamin C
2.‐ Vitamin D
3.‐ Niacin
4.‐ Magnesium

4142 The nurse plans to monitor for which of the following Correct answer: 4 Refeeding syndrome can occur in the critically ill client in response to feeding attempts Recall that refeeding syndrome occurs secondary to utilization of electrolytes involved in
consequences of refeeding syndrome in a critically ill whereby glucose and electrolytes (phosphorus, potassium, and magnesium) rapidly enter into energy metabolic pathways to direct you to option 4.
client receiving total parenteral nutrition (TPN)? body cells. These electrolytes are involved in enzyme reactions and ATP physiology that is part
of metabolizing the TPN. In option 1, serum phosphorus levels are decreased dramatically in
response to increased glucose needs. In options 2 and 3, hyperglycemia is present along with
increased insulin resistance.

1.‐ Increase in serum phosphorus and sodium levels


2.‐ Hypoglycemia
3.‐ Decreased fasting blood sugar levels
4.‐ Decreased serum potassium, magnesium, and phosphorus levels
4143 A client is admitted to the hospital with a painful Correct answer: 2 The client is usually kept NPO in order to minimize secretion of digestive enzymes that can Critical words are acute, pancreatitis, and pain. Recall impact of severe pain on appetite
attack of pancreatitis. The nurse anticipates that which contribute to the condition, and the client in acute pain is unlikely to want to eat until the pain and need to be NPO until acute inflammatory stage is over to direct you to option 2.
of the following changes will impact nutritional status is adequately controlled. IV therapy may be instituted to maintain hydration levels, and the
during treatment of this acute episode? client may also require administration of total parenteral nutrition (TPN) if the case is severe.
The client with a mild case may receive enteral nutrition for support. The client will not have
altered taste perception, and gastroparesis is not part of the clinical picture. The client should
not lose more than 10 percent of body weight if enteral or parenteral nutritional support is
adequate.
1.‐ Altered taste perception due to increased pain
2.‐ Inability to eat due to increased pain
3.‐ Potential for gastroparesis to develop due to disease process
4.‐ Weight loss of greater than 10 percent during management of the acute phase

4144 A client with anemia of chronic disease (ACD) is Correct answer: 3 To make valid recommendations to meet nutritional goals with regard to ACD, it is vital to The critical words are most helpful. Recognize importance of assessing baseline data in a
referred for nutritional counseling regarding dietary evaluate the client’s present dietary pattern for nutritional adequacy. This helps to establish a client. Eliminate option 2 since it does not address the anemia or nutritional status.
intake. Which of the following methods would be the nutritional baseline and determine food preferences and other related factors that influence Eliminate options 1 and 4 since they are specific to only one aspect of the problem.
most helpful for the nurse to implement in order to the client’s intake pattern. Option 1 is incorrect—even though medication therapy is aimed at
meet nutritional goals? increasing RBC production, it does not specifically address nutritional adequacy in terms of
maintaining nutritional stores. Option 2 is incorrect—merely increasing caloric intake without
regard to adequacy or balance may place the client at an increased risk of nutritional
imbalance. Consuming an adequate diet with all essential nutrients is as important as
increasing caloric intake in clients with chronic disease processes. Although it is important to
instruct the client about good sources of iron, this option is limited, since it will not provide the
most comprehensive approach in dietary evaluation.

1.‐ Have the client continue to take Epogen/Procrit as prescribed because that will help to maintain adequate nutritional stores.
2.‐ Have the client consume more calories in order to meet increased nutritional demands.
3.‐ Evaluate client's dietary pattern for nutritional adequacy.
4.‐ Instruct the client on adequate sources of iron in the diet.

4145 An unlicensed caregiver asks the critical care nurse Correct answer: 2 Critically ill clients on ventilator support must be provided nutritional support in order to Option 1 is a factually incorrect statement. Options 3 and 4 are incorrect because enteral
why it is so important to assess the nutritional status maintain nutritional adequacy, prevent depletion of nutrient stores, and respond to increasing feeding is associated with a stimulation of blood flow in the gut and improved mucosal
of a critically ill client on ventilator support, who has hypermetabolic demands of illness and therapies. Clients receiving medications such as integrity and liver function.
just been started on enteral nutritional support. Which dopamine and narcotics (opiates) are at risk to develop delayed gastric emptying, which can
of the following items would the nurse consider when lead to further problems. In addition, changes in acid‐base and fluid/electrolyte balance can
formulating a response? lead to decreased gastric emptying.

1.‐ Parenteral feeding is associated with a decreased rate of sepsis and mortality.
2.‐ Gastric emptying can be decreased in response to medication and acid‐base and fluid/electrolyte imbalances.
3.‐ The use of enteral feeding methods is associated with a decrease in liver function.
4.‐ Enteral feedings lead to a decrease in mucosal blood flow.

4146 The nurse would place highest priority on assessing Correct answer: 3 The use of hypertonic enteral nutrition can lead to bowel necrosis and therefore should not Read each option and determine if a risk factor is present. Note the word stable in option
which of the following critically ill clients for be used for a critically ill client. All of the other choices would not lead to feeding 4 and eliminate it. Recall knowledge of osmolarity and physiology to choose option 3.
complications associated with enteral feedings? complications in the critically ill client. Elemental formulas require minimal digestion and are
readily absorbed. The use of full‐strength formula feedings in small volumes with appropriate
monitoring according to individual tolerance is an accepted practice. A client should be
hemodynamically stable prior to the initiation of enteral feeding.

1.‐ A client who is receiving an elemental formula


2.‐ A client receiving full‐strength formula feeding in small volumes
3.‐ A client receiving hypertonic enteral nutrition solutions
4.‐ A client who is hemodynamically stable
4147 The nurse should plan to monitor which of the Correct answer: 3 A client who is critically ill on a ventilator and receiving parenteral nutrition should have daily Focus on the critical words most reliable. Eliminate vague options first, such as option 4.
following indicators as the most reliable method of weight measured as a reliable indicator of nutritional status. Serum albumin levels are not Eliminate next the two options that reflect fluid volume status only.
assessing the nutritional status of a critically client on a reliable indicators of effectiveness of nutritional therapy, although prealbumin levels reflect
mechanical ventilator who is receiving nutritional nutritional status over the last few days. Intake and output are excellent measures of fluid
support via parenteral therapy? volume status, but not overall nutritional status. Skin turgor is a measure of fluid volume, but
can be affected by other factors, such as age, and is not a reliable indicator of overall
nutritional status.
1.‐ Serum albumin levels
2.‐ Intake and output
3.‐ Daily weight
4.‐ Skin turgor

4148 A client with a longstanding history of diabetes, Correct answer: 2 A client with a multiple disease profile is at great risk both medically and nutritionally because Note that the client has history of three different chronic conditions. Recall that multiple
hypertension, and heart failure has been referred for of multiple organ system problems that could alter metabolism and absorption of nutrients. food and drug interactions that are possible. Eliminate option 3 since the BMI is not
dietary counseling. Which of the following should the Since the client is likely to have a multiple medication profile, it would be prudent to obtain a calculated with those parameters. Options 1 and 4 can be eliminated as they may provide
nurse do initially? listing of all medication (both prescription and OTC) in order to evaluate potential drug–drug some valuable information after baseline information is gathered.
and drug–food interactions. Option 1 is incorrect because a 3‐day food diary will only provide
information relative to intake. Option 3 is incorrect—vital signs will not enable the nurse to
calculate a BMI, since height and weight are needed. Even though it is important to ask if the
client is satisfied with current management of disease processes, option 4 is stated as a closed‐
ended question, which will provide no further information, and the client may not be able to
provide an accurate assessment of his or her own treatment.

1.‐ Have the client provide a 3‐day food diary at the next scheduled appointment.
2.‐ Obtain a list of all medications currently taken, both prescription and OTC.
3.‐ Obtain vital signs (e.g., blood pressure, pulse, and respirations) in order to calculate BMI.
4.‐ Ask if the client is satisfied with ease of maintaining current therapies.

4149 The nurse would instruct the client taking sodium Correct answer: 3 A client on Coumadin (sodium warfarin) needs to avoid foods that are high in vitamin K, which Recall that green leafy vegetables are high in vitamin K to eliminate the first two options. It
warfarin (Coumadin) for a deep vein thrombosis (DVT) acts as an antidote to the drug. Foods high in vitamin K include green leafy vegetables (options is then necessary to understand that tomatoes are a rich source of vitamin K to choose
of the left leg that it is acceptable to continue eating 1 and 2), and tomatoes (option 4), as well as wheat grains and liver. Corn is not high in vitamin correctly between this option and the one containing corn.
which of the following foods that the client prefers? K.

1.‐ Broccoli
2.‐ Spinach
3.‐ Corn
4.‐ Tomatoes

4150 When caring for a client who has undergone a liver Correct answer: 3 Increased weight gain can be attributed to fluid retention due to medical treatment therapies Read each option, checking for accuracy of content and relevance to the question.
transplant, the nurse should consider which factor post‐transplant and as such does not reflect accurate information about nutritional status. In
when assessing the nutritional status of the client? option 1, the use of immunosuppressant drugs can lead to increased nutrient needs due to
side effects (e.g., nausea, vomiting, mouth lesions, and diarrhea). Option 2 is incorrect because
the client's pre‐transplant nutritional baseline status has a very profound impact on post‐
transplant nutritional status. The organ is being replaced, not the vascular system, and clients
with liver failure often have longstanding nutritional deficits as a result of altered liver
metabolism. In option 4, a post‐transplant client still has to follow dietary restrictions due to
existing medical treatment regimens and is followed closely by a dietitian.

1.‐ Use of immunosuppressant drugs, which decreases nutrient needs


2.‐ Nutritional status pre‐transplant, which has little influence on post‐transplant nutrition.
3.‐ Increased weight, which directly correlates with adequate nutrition.
4.‐ Awareness that no dietary restrictions are needed post‐transplant.
4151 Which of the following nutritional complications Correct answer: 2 A client with a longstanding history of congestive heart failure is likely to develop chronic Critical words are nutritional complications. Eliminate options 3 and 4 since they are
would the nurse assess for in a client with longstanding protein energy malnutrition resulting in cardiac cachexia. Option 1 is incorrect because clients complications related to airway and oxygenation. Note the word cachexia to direct you to
congestive heart failure (CHF)? with chronic CHF often have weight loss with superimposed edema that goes unnoticed, option 2.
thereby masking poor nutritional status. Options 3 and 4 are incorrect because clients with
chronic CHF have decreased activity tolerance and airway clearance due to disease pathology.

1.‐ Weight gain due to fluid retention


2.‐ Cardiac cachexia
3.‐ An increase in activity tolerance.
4.‐ Increased airway clearance

4152 Which of the following would be most helpful for the Correct answer: 4 Discussing dietary meal planning activities with an obese client who has multisystem Note that the question addresses the nutritional needs of an obese client. First, eliminate
nurse to do when planning for the nutritional disorders would be the most helpful in terms of prospective therapeutic management. This option 3 since neither of the topics is addressed. Eliminate option 1 since there is no
management of an obese client with multisystem option would allow the client to provide information relative to meal planning and mention of a mental health problem. Eliminate option 2 once this defers the problem and
disorders? demonstrate both application and compliance with medical/nursing treatments. Option 1 is does not involve the nurse.
incorrect because there is no indication that the client requires intervention by a psychiatrist.
This option reflects a judgment by the nurse with no other defining information to suggest that
the client is having a psychological problem. Option 2 is incorrect because there is no
information to suggest that the client needs or is ready to accept medical treatment for
obesity. Although it is important to ascertain that the client is being compliant with medication
therapy, option 3 does not answer the question with regard to the nutritional management of
the client.

1.‐ Refer to a psychiatrist for counseling.


2.‐ Refer to a physician who specializes in weight disorders.
3.‐ Make sure that the client continues to take all medications ordered to treat disease processes.
4.‐ Discuss dietary meal planning activities with client.

4153 While the nurse is discussing a client's likely death Correct answer: 3 Peripheral circulation decreases and shifts to the vital organs. The vascular system collapses, Note the issue of the question, which is knowledge of impending signs of death. The words
with family members, one of the adult children causing decreasing pulse and blood pressure. The gag reflex is lost, and mucus accumulates in best response in the question tell you it is a true statement and will be a priority in client
inquires, "We plan on taking turns being here for now, the back of the throat. Respirations decrease in rate, and the rhythm is irregular. Muscle care. Options 1 and 4 are inaccurate. Choose option 3 because option 2 does not provide
but we all want to be here at the time of death. Is rigidity typically occurs after death. Vision is blurred. A lucid moment is not a pattern in death. information about the characteristics of impending death.
there any way we can tell when that time is close?" It is difficult to pinpoint the exact time when death will occur, but the imminence of clinical
Which of the following is the nurse’s best response? death can be detected.

1.‐ "Often, people become lucid for about 15 minutes during the last hour before death. Watch for your [family member] to become more alert, with clearer eyes, and to look
around, focus on faces, and clear his [or her] throat. Call the others in at that time."
2.‐ "I wish I could tell you that there was a way to know. It could be minutes from now or another three days. One just never knows."
3.‐ "The arms and legs become more bluish in color and are cool to touch. Breathing will become irregular and shallow and will change speed. Call me if you hear mucus in the
throat. The pulse and blood pressure will decrease."
4.‐ "You can expect the muscles to become rigid, with staring eyes and mouth closed. The head is pulled back with neck rigidity. Don't be alarmed when you hear a death rattle
in the throat."

4154 A 90‐year‐old client expresses a wish to die at home Correct answer: 1 Hospice specializes in end‐of‐life care. A rabbi is an important person during the end of life, The key issue is that the client wishes to die at home. Option 1, hospice care, can be
after being told that an esophageal stricture prevents but there is not an immediate need to make this call. An attorney or medical examiner is not provided in the home at all times. The other options (the attorney or medical examiner) do
swallowing. The client refuses a feeding tube. The necessary at this time. not address the client's issue, which is 24‐hour care in the home at the end of life.
family fully supports this decision. Which of the
following would be most appropriate for the nurse to
call?
1.‐ Hospice care
2.‐ The rabbi
3.‐ An attorney
4.‐ The medical examiner's office

4155 The nurse is providing postmortem care for a client. Correct answer: 1, 4 The body is to be handled with dignity at all times. Even though humor can alleviate stress, it Use the process of elimination to identify the options that contain inaccurate information.
Which of the following interventions would be is not appropriate at this time. Once the body is cleaned, all external tubes and drains are While some of the information is correct in options 3 and 5, there are elements that make
appropriate prior to allowing the family to visit? Select removed, the linen is freshened, the sheet is pulled to cover the client’s shoulders. While those responses incorrect. For example sterile gloves are not necessary, although clean
all that apply. gloves should be worn during postmortem care, sterility is not an issue. State laws and policies gloves are worn. The family can visit before the physician is called.
differ regarding the nurse's ability to declare death. Even if a physician is required to declare
death, the time of death cannot be verified exactly and is not required prior to the family being
allowed to view the client after death.

1.‐ Prepare the body to look as clean and natural as possible.


2.‐ Keep the sheet over the client's face until the family is comfortably seated in the room.
3.‐ Wear sterile gloves to pack the anal canal with gauze.
4.‐ Remove the external tubes and drains.
5.‐ Call the physician to verify the time of death before taking the body to the morgue.

4156 A dying client's spouse is afraid to leave the client's Correct answer: 2 The signs and symptoms listed indicate death will occur soon and the spouse is fearful to The key issue is the signs and symptoms of imminent death of the client and the spouse’s
room to get a meal in the cafeteria for fear the client leave the room at this time. Obtaining a meal for the client's spouse while she remains at the desire to be with him when he dies. Options 1 and 3 would increase the spouse’s anxiety if
will die while she is gone. There are no other family bedside and supporting her during the client's imminent death demonstrate knowledge of the she were notified while she was absent. Option 4 is correct but does not include the
members or visitors present. The client is dying process in addition to compassion and concern for the client and spouse. support from the nurse exemplified in option 2.
nonresponsive, his pulse is irregular and bradycardic,
and he has Cheyne‐Stokes respirations. Which of the
following represents the best course of action for the
nurse?
1.‐ Encourage the client's spouse to take a break and go to the cafeteria and eat. The client is nonresponsive and won't know she is gone.
2.‐ Make arrangements for the client's spouse to receive a meal in the client's room.
3.‐ Tell the client's spouse she will be called if there are any changes and ask a nurse aide to sit with the client while the wife is gone to the cafeteria.
4.‐ Do not interfere with the spouse's decision.

4157 The family of a client diagnosed with terminal cancer Correct answer: 4 Hospice care is provided to those clients who have 6 months or less to live. Hospice nurses The core issue of the question is knowledge of the purpose and goals of hospice care. The
has been informed that he is not expected to live more are skilled in pain and symptom management as well as in emotional support to the dying other options indicate that the family expects improvement in the client’s condition, which
than 2 months. Which of the following statements clients and their families. Hospice care does not terminate once families learn to provide care is not realistic.
made by the family indicates to the nurse that the (option 1), and a client in need of hospice services cannot be expected to resume self‐care
family understands the client's prognosis? (option 3).

1.‐ "Hospice nurses are going to help care for him at home until he gets better."
2.‐ "Hospice nurses are going to help care for him until we learn how to provide the care."
3.‐ "Hospice nurses are going to help care for him until he can take care of himself."
4.‐ "Hospice nurses are going to help care for him to make him more comfortable."

4158 A client's spouse is upset and crying because her Correct answer: 3 This client situation acknowledges that while the lack of nutrition and fluids will produce The focus of this question is the ability to apply the ethical principles to end‐of‐life care.
husband is no longer taking liquids. She understands ketones and cause somnolence to decrease the client's anxiety and promote overall comfort, Eliminate options 1, 2, and 4 because they do not represent the principle of doing good.
fluids can delay his death but is most concerned that the wife is more concerned about how dehydration might feel to her husband. Beneficence
he will become dehydrated and feel uncomfortable. promotes doing good for the client, the focus of a quality death. Veracity and fairness are not
The nurse will formulate an answer addressing which considerations in this situation.
ethical principle?
1.‐ Justice
2.‐ Beneficence
3.‐ Nonmaleficence
4.‐ Veracity
4159 The nurse anticipates that which of the following Correct answer: 3 People cope better when they accept what their life had to offer, have learned to cope with The focus of this question is knowledge of how older adults cope with a diagnosis of a life‐
clients newly diagnosed with a terminal illness is least personal losses, have the time and ability to recover emotionally between multiple deaths, threatening illness. Using the criteria outlined in the rationale, use the process of
likely to have difficulty facing his or her mortality? believe that death is a part of living, and have religious beliefs. Option 3 indicates the individual elimination to make the correct choice.
has planned for the future and believes that death is part of living. The client in option 1 is
incorrect because this client has lost three people over a brief period of time (6 months). The
client in option 2 is incorrect because he shows dependence, having never moved out of the
home of his parents, who are both healthy. The client in option 4 is incorrect because
individuals with religious beliefs are found to cope better.

1.‐ A 71‐year‐old female whose grandson, sister, and best friend died over the past 6 months
2.‐ A 59‐year‐old male who never married, is an only child, and whose parents are both healthy
3.‐ A 70‐year‐old male who has planned his funeral and enjoys riding his motorcycle at high speeds in rural areas
4.‐ A 68‐year‐old female who is an atheist

4160 A client with terminal lung cancer is receiving total Correct answer: 2 Clients go through multiple stages and tasks when they are dying. During bargaining, they This question focuses on Kubler‐Ross's five stages of death and dying. The client
brain radiation therapy to control the tremors in hands "negotiate" to meet a life goal. The other stages are not consistent with the client's statement. statements on options 3 and 4 do not include reflections of anger or depression. Denial
due to multiple metastatic lesions. As the nurse assists Denial would be refusal to accept the diagnosis of terminal cancer. Anger and depression are statements would show refusal to accept the diagnosis or pushing away the reality from
him back to his wheelchair, he comments "I'm hoping natural reactions to anticipated loss. Acceptance is shown when the client has come to terms consciousness. Option 5 indicates the client has accepted the need for treatment but has
this treatment will let me see my first tomato on the with the diagnosis and anticipated death. not accepted the reality of the extent of his disease.
Fourth of July." According to Kubler‐Ross, this
statement is an example of which stage of death and
dying?

1.‐ Denial
2.‐ Bargaining
3.‐ Anger
4.‐ Depression
5.‐ Acceptance

4161 The Registered Nurse would intervene after hearing a Correct answer: 4 Option 4 does not treat the client with respect and sensitivity and therefore is an example of The critical word in the stem of the question is intervene, making the correct option the
Licensed Practical Nurse (LPN) make which of the maleficence. Option 1 provides a rationale for a therapy that may be uncomfortable. Options 2 one that is physically or emotionally harmful. Option 4 can cause anxiety and distress to the
following statements regarding a client with severe and 3 advocate for the client. client and is therefore the statement that potentially causes harm. Eliminate option 1
arthritis who is also newly diagnosed with a rapidly because it merely questions the rationale behind an intervention and option 2 because it
growing colorectal cancer? tries to safeguard the client. Eliminate option 3 because it is a communication about client
desires.
1.‐ "Even though it hurts a bit, your arthritic joints will become less stiff with gentle exercise."
2.‐ "If we give more pain medication, will it stop his breathing?"
3.‐ "He has a living will that says he does not want to be resuscitated."
4.‐ "You have on a diaper so it's OK if you do not make it to the bathroom."

4162 Based on Rando's process of bereavement, place in Correct answer: 1, 5, 3, Rando’s process of bereavement is to (1) recognize the loss and death, (2) react to experience Remember the Six R's of Rando: recognize, react, reminisce, relinquish, readjust, and
order the following statements indicating the steps of 4, 2 and express the separation and pain, (3) reminisce, (4) relinquish old attachments, (5) readjust reinvest. They all begin with re and then the letters CAMLAI. A helpful memory aid could be
the process as experienced by a client regarding his and adapt to the new role while maintaining memories and form a new identity, and (6) Chocolate Always Makes Lads Act Icky.
father's death. Click and drag the options below to reinvest.
move them up or down.

1.‐ "This is the second anniversary of my father's death."


2.‐ "My father was an alcoholic, so every Christmas I make certain that local AA groups have coffee and chocolates to help the members through the holidays."
3.‐ "It was so much fun to rummage through the antique stores together."
4.‐ "It's too difficult to be around his stepfamily, so I visit friends on vacation."
5.‐ "The homestead has run down since his death. It was hard to drive past and see the lack of care in his vegetable garden."
4163 A 46‐year‐old female client with a history of head and Correct answer: 1 The definition for the ethical principle of justice is "fairness." Option 2 reflects anger on the Option 1 is the only statement indicating fairness in client care. Option 2 is coercion, and 4
neck cancer was recently told she has multiple part of the family, while option 3 reflects anger on the part of the client. Option 4 is an ethical dilemma. Option 3 is an example of the stages of the grief process.
metastatic sites in her lung. The nurse is discussing the demonstrates a concern that the client will suffer an injustice by enduring unnecessary tests.
situation with the client and her sister. Which
statement during the conversation reflects the ethical
principle of justice?
1.‐ "The staff will do everything possible to make your sister comfortable while she is in hospice."
2.‐ "She told the doctor she did not want to lose her hair. It is not right that he coerced her into taking that experimental chemotherapy. Now she is bald and dying."
3.‐ "Why did I have to get this terrible disease? I just want my life back."
4.‐ "We have made special arrangements for her care
at home. Now the doctor says she has to have more
tests to see if there is cancer in her liver. Why? We
know she's dying."

4164 A 22‐year‐old hospitalized client with a recent Correct answer: 4 Anger is a common element to all the theories of grief and stages of dying. It is important to Eliminate option 1 because it indicates the nurse is taking the statement personally and
diagnosis of acquired immunodeficiency syndrome acknowledge the client's anger, help him or her identify the source of the anger, and offer assuming what the client needs. Option 2 also makes an assumption but does address the
(AIDS) says to the nurse, "The food on this breakfast choices or control when possible. It is important to be nonconfrontational (option 2), not to anger. The anger is also addressed in the third option but does not encourage the client to
tray is terrible. Why can't you people do even simple take the anger personally (option 1), and not to ignore the client's issue (option 3). share the source of the anger. For these reasons, options 2 and 3 should be eliminated also.
things well?" What is the nurse's best response? The last option addresses the normalcy of the anger in this situation and encourages the
client to engage in further conversation with the nurse.

1.‐ "I know you are angry, but I cannot let you make me the object of your anger. I will send up the dietitian."
2.‐ "This is not about breakfast. Tell me what you are really angry about."
3.‐ "I understand you are angry. I'll shut the door and let you cool off."
4.‐ "I hear a lot of anger in your voice that is quite normal and healthy. Do you want a new breakfast or do you want something else?"

4165 While talking to adult children of a dying male client, Correct answer: 4 Open communication with concrete evidence of emotional attachments assists in coping at Use the process of elimination and address the client in the question. In this case, both the
the nurse finds them tearful, with ambivalent feelings the end of life. Option 4 provides concrete assurance in the presence of the loved ones. client and the adult children are the affected clients, so the correct option is one that
toward the client. The client often expresses beliefs of Relaxation tapes help with stress reduction but do not help with resolution of problems benefits all of them.
a wasted life. The children say that their father often experienced by the family. Staffing needs do not permit a nurse to be with one client
showed love but followed it with criticism, anger, continually, and families require privacy as well. Assurance that the past no longer matters is
damaging accusations, and emotional abuse. The nurse an assurance lacking concrete properties.
would suggest which of the following interventions
that is most likely to be helpful at this time?

1.‐ Listen to relaxation tapes before visiting each other. If negative feelings arise, listen to the tapes together.
2.‐ Have a nurse present in the room at all times when a family member visits the client so that the nurse can intervene with conflict resolution if problems arise.
3.‐ Assure the client and children that the past no
longer matters; the only time that matters is the
present and the future. Encourage the children to
spend more time with their father.
4.‐ Make a videotape of each adult child telling a story
of a time when their father showed love, while the
client tells of a special love for each child. Plan a time
for them to watch it together.

4166 A client questions the nurse about the difference Correct answer: 1 A living will is written by the client and includes desires for use of different types of treatment The core issue of the question is knowledge of a living will. The wording of the question
between a living will and power of attorney. The in case of a life‐threatening illness. A durable power of attorney is a legal document indicates that the correct answer is a true statement. Systematically eliminate options 2, 3,
nurse's best response is which of the following? designating an individual to make legal decisions if the client is unable to make choices and 4 because they are incorrect statements.
independently.
1.‐ "The living will allows the client to indicate specific
medical treatments to be omitted in the event of
terminal illness, while durable power of attorney
legally appoints another to make those decisions on
the behalf of the client."
2.‐ "A lawyer carries out a living will, while a designated family member or friend carries out advanced directives."
3.‐ "In a living will, the client specifies medical
treatments to be carried out should he or she be
incapable of making decisions. Durable power of
attorney allows the client to include both treatments
to be carried out and treatments to be omitted in the
event of terminal illness."
4.‐ "The living will indicates when a client wishes life
support to be discontinued, while durable power of
attorney gives that power to another in the event of
terminal illness."

4167 The nurse working with a terminally ill client wishes Correct answer: 3 The nurse needs to consider the client's wishes while also acting within the law. Euthanasia Eliminate option 1 because it is illegal. Nurses need to provide unbiased care to all clients,
to support the client's decisions concerning end‐of‐life constitutes illegal nursing practice in the United States at this time. To act ethically, the nurse regardless of conflicts with belief systems, so option 2 is also incorrect. Option 4 is incorrect
care. To do this appropriately, the nurse should do should provide care to clients according to need, regardless of belief systems. Clients who are because it may or may not reflect the client's preference at this time.
plan to which of the following? diagnosed with terminal illness may or may not be ready for do‐not‐resuscitate orders,
depending on anticipated life expectancy, quality of current life, and psychosocial variables.

1.‐ Be comfortable in assisting the client with euthanasia when requested to do so.
2.‐ Ask another nurse to provide care if the client has a belief system that differs from the nurse's belief system.
3.‐ Respect the client's wishes about death to the extent possible by law.
4.‐ Encourage the client to request a do‐not‐resuscitate order because the client has been diagnosed with a terminal illness.

4168 The terminally ill client asks the nurse for information Correct answer: 4 The focus of hospice is improving the quality of life and preserving dignity for the client in Eliminate options 1, 2, and 3 because they represent incorrect definitions of hospice and
about hospice care. The nurse would best respond by death. Hospice care may be provided by nurses, volunteers, or other members of the health do not accurately reflect the care provided. In addition, option 1 is insensitive.
stating: care team in a variety of settings. It is available to any client who has a terminal illness with a
life expectancy of 6 months or less.
1.‐ "Hospice care is home nursing care provided to terminal cancer clients."
2.‐ "The client qualifies for hospice care at the time of diagnosis with a terminal illness."
3.‐ "The main focus of hospice is to educate the client concerning treatment options and alternatives."
4.‐ "Hospice regards dying as a normal part of life and provides support for a dignified and peaceful death."

4169 The nurse concludes that which of the following Correct answer: 2 Grief resolution requires letting go of the past and looking forward to the future. The client The critical word in the question is not, which tells you the correct option is a statement
would not be considered a sign of grief resolution in needs to be able to put the loss in perspective and engage fully and effectively in daily life as that indicates inadequate coping with the loss or unresolved grief.
the bereaved client whose husband died a year ago? an independent person. In option 2, the client has not let go of the past because decisions are
made in the present only through memories of preferences of the deceased. Options 1, 3, and
4 all indicate healthy grief resolution.
1.‐ Becoming future‐oriented when discussing details of everyday life
2.‐ Considering the opinions of the deceased prior to making decisions about everyday life
3.‐ Experiencing occasional waves of grief triggered by pictures or events
4.‐ Sharing stories of good times that the client and her husband shared over the years

4170 Which of the following nursing interventions would Correct answer: 1 Hope instillation is often an effective intervention in dealing with anticipatory grieving. Option The key words in the stem of the question are anticipatory grieving. Use the process of
be most appropriate for the nurse to include in the 3 deals with the symptom and not the actual problem. Options 2 and 4 are not appropriate elimination to select the option that focuses on anticipated loss. Choose option 1 because
care plan of a client with a nursing diagnosis of because there is no evidence in the stem of the question to support their need. of the word hope in the option, which relates to the word grieving in the question.
anticipatory grieving?
1.‐ Hope instillation
2.‐ Forgiveness facilitation
3.‐ Medication management
4.‐ Hypnotherapy

4171 A client is dying, is in excruciating pain, and refuses Correct answer: 3 The Hindu and Buddhist religions require that believers are alert and mindful as they leave Use the process of elimination. Only the religion identified in option 3 requires a state of
anything for relief that alters his sensorium. The nurse the life on earth and transcend to their next life. This requirement is not found in Islam or being mindful and alert as their faithful transcend from life into death.
checks the psychosocial data part of the client's Catholicism.
medical record, expecting that which of the following
religions is most likely practiced by the client?

1.‐ Islam
2.‐ Judaism
3.‐ Buddhism
4.‐ Catholicism

4172 Which of the following indicates to the nurse that a Correct answer: 4 Knowledge of response to pain offers accurate and careful assessment of pain with earlier Options 1 and 2 are due to inactivity from uncontrolled pain. Requesting assistance to eat
noncommunicative client's pain is not well‐managed? and more complete pain relief. It should include physical and emotional behaviors. is not an indication of pain, so option 3 is not a correct response.

1.‐ Crackles in the lung


2.‐ Hyperactive bowel sounds
3.‐ Unwillingness to eat without assistance
4.‐ Constant restlessness and leg movement

4173 A nurse who seeks to uphold the Patient Self‐ Correct answer: 1 The Patient Self‐Determination Act became federal law in 1990. This law states that clients Look for a specific answer to a specific question about this law. Recall situations seen in
Determination Act would do which of the following? have a right to participate in their own care. In addition, healthcare facilities are required to clinical that are examples of actions that are consistent with the provisions of this law.
inform clients of the right to accept or refuse medical care. The other options are not
provisions within the law.
1.‐ Inform clients of their right to be part of the healthcare decision process.
2.‐ Provide clients with a community case manager to manage their living wills.
3.‐ Allow clients the opportunity to choose between voluntary and involuntary treatment.
4.‐ Inform clients of their right to sue and to be compensated if a facility does not provide a mechanism for self‐determination.

4174 In order to most effectively support a client through Correct answer: 3 Option 3 is correct. For nurses to be able to talk openly about death and dying, they must This is not a difficult question. Don't make it unnecessarily hard. In order to best support
the death and dying process, a nurse should do which have some level of comfort with death, dying, and the experience of loss. It is important that the client through anything difficult, the nurse has to be comfortable with the process. This
of the following? they have addressed their own personal losses and done the necessary grieving. If this is not is no different. If the nurse is comfortable with loss and dying. Then he/she will be able to
satisfactorily done, losses at work will trigger grief reactions. This may render the nurse less make the client feel more comfortable also.
able to be available to assist the client, it may emotionally. Not being able to develop a healthy
attitude about death and dying may also prevent the nurse from being able to communicate
effectively with clients. Clients should be encouraged to talk about their feelings, both positive
and negative. Although the process of life is important, clients should be given permission to
talk about their own deaths and deaths of their loved ones. Options 1 and 2 are incorrect as
they would actively prevent the client from talking about death. Option 4 would place an
unreasonable responsibility on the nurse. Not only is the nurse not responsible for preventing
or facilitating the clients from these feelings, the nurse will recognize that experiencing
feelings of helplessness and depression are normal during the process of normal grieving.

1.‐ Encourage clients to focus on being positive


2.‐ Place a greater importance on the process of life, rather than a focus on death
3.‐ Have a healthy attitude about death and dying
4.‐ Have the ability to keep clients from becoming depressed and helpless
4175 A nurse working in a hospice program understands Correct answer: 1 Option 1 is correct. Cultures have rules for grieving which delineate the appropriate Think about cultural rituals engaged in during a time of grieving. What psychologic
that client cultural rituals and practices provide expression of feelings and determines patterns of appropriate behavior. Option 2 is incorrect purposes were served by the rituals?
individuals experiencing grief with: as it may be a consequence of cultural influence on the individual. Option 3 is incorrect
because it will not assist a client in dealing with feelings of anger. Option 4 is incorrect as it
may or may not be a tenet dictated by cultural beliefs.
1.‐ Assistance in stabilizing and coping when they are feeling confused and chaotic.
2.‐ Opportunity to identify more closely with their cultural heritage.
3.‐ Opportunity to oppose ethnic structures in order to displace feelings of anger.
4.‐ Assistance in establishing boundaries between themselves and the dying person.

4176 A client whose newborn baby died at birth six weeks Correct answer: 3 Option 3 is correct. Normal grief is the total response to the emotional experience related to Recall basic definitions of different types of grief. Look carefully at the descriptors of the
ago tells the nurse that she has been having trouble loss or death. Clients experience grief at different levels of intensity and for different periods client’s behaviors.
sleeping and eating. She also reports having difficulty of time. Option 1 is incorrect as inhibited grief is a suppressed response to loss, which may be
concentrating at work and does not experience any expressed by somatic complaints, such as physical symptoms around the anniversary of a loss
pleasure from painting canvases like she once did. The or during holidays. Option 2 is incorrect because disenfranchised grief is a response to a loss in
nurse concludes these statements as indicative of: which the bereaved is not regarded as having the right to grieve or is unable to acknowledge
the loss to other persons. The loss is one which is not publicly sanctioned or acknowledged. As
a result, the griever does not receive the needed support and validation for their pain.
Examples might be abortion, miscarriage, or death of loved one from AIDS. Option 4 is
incorrect as delayed grief is a postponed response in which the bereaved person may have a
reaction at the time of the loss, but it is not sufficient in proportion to the loss. However, a loss
at a later time may trigger a reaction that is out of proportion to the significance of the current
loss.

1.‐ Inhibited grief.


2.‐ Disenfranchised grief.
3.‐ Normal grief.
4.‐ Delayed grief.

4177 The nurse working with a bereaved family utilizes Correct answer: 4 Option 4 is correct. Grief is a pervasive, individualized, and dynamic process that may result in Notice that three of the options give partially correct information that is included in the
which of the following concepts about grief work? physical, emotional, or spiritual distress because of loss or death of a loved one or cherished more comprehensive option. Choose the most comprehensive answer.
Grief is primarily a: object. Options 1, 2, and 3 are incorrect. Although each of these options are partially correct,
option 4 provides the most comprehensive information about the conceptual framework the
nurse utilizes with a bereaved family doing grief work.

1.‐ Physiological response following a loss or death.


2.‐ Psychological response following a loss or death.
3.‐ Spiritual response following a loss or death.
4.‐ Dynamic process of physiological, psychological, and spiritual responses to a loss or death.

4178 A 52‐year‐old client suffers a massive myocardial Correct answer: 1 Option 1 is correct. In adults between ages 45–65, the aging process can create an emotional Look for the combination of factors: age of the client and recent changes in physical
infarction and cannot return to usual activities because reaction known as a grieving response. The loss of peak physical functioning, a change in health health.
of the large amount of damage to the heart. The nurse status, attempts to change to "healthier" habits, and adjustment to a new body image are all
actively assists this client who is most likely losses and may engender a grief reaction. Options 2, 3, and 4 are incorrect as they are not part
experiencing: of this developmental stage.
1.‐ The grieving response.
2.‐ Life goal disappointment.
3.‐ Acceptance of failures.
4.‐ Mistrust of care givers.
4179 A terminally ill client with a grave prognosis will soon Correct answer: 3 Option 3 is correct. Anticipatory grief is anxiety and sorrow experienced prior to an expected Recognize that individuals often believe that anticipatory grieving will ease the pain of
be removed from life support. The nurse caring for the loss or death. The family members of a chronically or terminally ill client may anticipate the their loss once the death occurs. In actuality, the knowing, preparing for and, anticipating
client notes that the family is beginning to distance loss of a loved one before her or his death because of the prognosis or severity of the person’s the dying of a loved one does not necessarily make the actual death any easier for the
themselves from the client. The nurse concludes the illness. The family is beginning to "pull away" or distance themselves from the grief and sorrow bereaved.
family is experiencing: of the expected death of the client. Option 1 is incorrect as dysfunctional grief is unresolved
that does not lead to a successful conclusion. Option 2 is incorrect because disenfranchised
grief is a response to a loss or death in which the individual is not regarded as having the right
to grieve or is unable to acknowledge the loss to other persons. Option 3 is incorrect as
inhibited grief is a suppressed response, which may be expressed in other ways, such as
somatic complaints (e.g., having physically symptoms on the anniversary of a loss or during
holidays).

1.‐ Dysfunctional grief.


2.‐ Disenfranchised grief.
3.‐ Anticipatory grief.
4.‐ Inhibited grief.

4180 A client states, "I don’t know what I'm going to do to Correct answer: 4 Option 4 is correct. Reflecting the communication of the client will enable them to have a Avoid selecting an option that contains platitudes. Recognize that reflecting the client’s
manage my family now that my husband has died." feeling of being heard. The goal in assisting a client in grieving can be achieved by the nurse feelings back to the client will facilitate further expression of feeling, which is very
What would be the nurse's best response to this who actively listens and encourages the client to discuss personal feelings. Options 1 and 2 are important when the individual is grieving. Remember listening is what is most important at
statement? incorrect as these options provide false reassurance and do not allow the client to discuss this time. Not making assumptions or giving advice.
feelings of loss. Option 3 is incorrect because while it may be appropriate at some time, at this
juncture the client needs someone to listen and provide support.

1.‐ "Everything will be all right. You will be able to survive without him. You'll see."
2.‐ "I know in time you will be able to get over your husband's death. Time heals all wounds."
3.‐ "I think you should contact your minister for advice. You need spiritual guidance now."
4.‐ "What I hear you saying is that you are feeling overwhelmed now."

4181 A nurse conducting health promotion classes in the Correct answer: 4 Option 4 is correct. Adolescents fantasize that death can be defied and may act out defiance Recall important concepts of developmental theorists, as well as behaviors of adolescents
community would teach parents that which age group through reckless behaviors. Option 1 is incorrect as toddlers do not have an understanding of you known or have known.
is most likely to take unnecessary risks because they death. Option 2 is incorrect because preschool children believe death is reversible. Option 3 is
believe they are invincible to death? incorrect as school‐aged children express a fear of death.

1.‐ Toddler
2.‐ Preschool
3.‐ School‐age
4.‐ Adolescent

4182 A client has a terminal illness and is unable to eat. The Correct answer: 3 Option 3 is correct. Voluntary passive euthanasia occurs when treatment is intentionally Eliminate the two options containing involuntary. If the client has made advanced directive
client's advance directive clearly states a wish not to withheld by voluntary consent of the individual who is dying. Option 1 is incorrect as voluntary statements, from a legal perspective, these were made voluntarily. Then note that the
be fed by external feedings or with IV fluid active euthanasia occurs when the person being euthanized has agreed and volunteered for client is asking for withholding of treatment, not administering of a treatment to hasten
replacement. The nurse interprets this wish to be: death. Option 2 is incorrect because involuntary active euthanasia occurs when the person death
being euthanized has not agreed or volunteered for death. Option 4 is incorrect as involuntary
passive euthanasia occurs when treatment is intentionally withheld without voluntary consent
from the person who is dying.

1.‐ Voluntary active euthanasia.


2.‐ Involuntary active euthanasia.
3.‐ Voluntary passive euthanasia.
4.‐ Involuntary passive euthanasia.
4183 With a child newly diagnosed with leukemia, which Correct answer: 3 The child and family will be overwhelmed with such a life‐threatening illness; anticipating the Imagine yourself in the situation of this family. Would not your emotional level be very
nursing care measure would the nurse identify as a loss of a child would be a priority for the family. Options 1, 2, and 4 suggest pertinent high?
teaching priority for the child and family? interventions, but during the initial period following learning of the diagnosis, the first need of
the family is to react emotionally and begin to adjust to the losses implied by the diagnosis.

1.‐ Comfort measures


2.‐ Distraction activities
3.‐ Anticipatory grieving
4.‐ Bereavement counseling

4184 The nurse is to counsel a mother who recently placed Correct answer: 4 Self‐awareness is a key component of any nurse/client experience. The nurse must be able to Look at what the question is asking, which is, In what way can self‐awareness benefit the
her newborn baby up for adoption. Before beginning examine personal feelings, actions, and reactions in order to better assist the client in fully nurse?
the counseling, it is important for the nurse to deal expressing his or her own feelings and thoughts. A firm understanding and acceptance of self
with personal feelings about adoption, grief, and loss. allows the nurse to acknowledge a client’s differences and uniqueness. In order to be
This self‐awareness would: empathetic to the client, the nurse must be aware of his or her own feelings. Options 1, 2, and
3 do not focus on self‐awareness.

1.‐ Prevent the nurse from being personally affected by the client's choice in adoption.
2.‐ Prevent the nurse from sharing any personal feelings with the client.
3.‐ Assist the nurse to avoid discussing unpleasant feelings with the client.
4.‐ Assist the nurse to help the client express grief fully.

4185 During a counseling session, a 21‐year‐old client with Correct answer: 1 Schizophrenia most often occurs in young adults who are in the prime of life and attempting Consider this client's reaction to be normal.
schizophrenia verbalizes feelings of sadness and anger to achieve a normal adulthood. The individual experiences many losses, and the nurse should
about being unable to keep a job or continue attending assist the client through the grieving process. The situation does not indicate that the client is
college. The nurse should formulate which of the experiencing significant anxiety (option 2). This situation does not indicate that that the client
following as the most applicable nursing diagnosis: is experiencing impaired coping or that hopelessness is present (option 3). Sadness and
hopelessness are different emotions. The client’s grieving is not dysfunctional (option 4), as the
client is actively grieving a recent loss.

1.‐ Grief related to perceived inability to achieve developmental milestones


2.‐ Anxiety related to fear of unknown and fear of failure
3.‐ Ineffective coping related to feelings of hopelessness and anger
4.‐ Dysfunctional grief related to unrealistic expectations of abilities and lack of achievement

4186 The community health nurse arrives at a home for a Correct answer: 2 Even though great emotional pain is felt after a loss, it is necessary for the grieving person to Notice that the loss has occurred very recently.
routine monthly visit. The client, who is crying, invites talk about memories of the lost person. This process begins early in the grief experience.
the nurse in and says, "My mother’s funeral was Options 1 and 3 would actively prevent the reminiscing that is necessary in early. Additionally,
yesterday. I'm so sad." What is the most appropriate option 3 would also change the focus from the grieving person's feelings to a more impersonal
action by the nurse at this time? and clinical topic. In option 4, while it is true that people with a high level of ambivalence
about the lost person may have difficulty resolving grief, this sort of exploration is
inappropriate at this time. The client needs to express initial feelings of loss before moving on
to other grieving tasks, which include reviewing the relationship.

1.‐ Encourage the client to think about something other than the mother's death.
2.‐ Allow the client to talk about personal memories of the mother.
3.‐ Ask the client to describe what led to the mother’s death.
4.‐ Explore the nature of the client's relationship with the mother.
4187 A client reports that since her husband's death 4 Correct answer: 2 Somatic complaints may be experienced around the date of a loss. This is called an Notice the time period since the spouse's death.
years ago, she has experienced migraine headaches anniversary grief response and is not a dysfunctional grief experience if the physical symptoms
and severe nausea each year around the date of the occur only around the specific date of the loss and the individual has otherwise returned to a
husband's death. The nurse suspects the client is full life. Individuals who have been together for many years often have these experiences for
experiencing which of the following? many years. Delayed grief (option 1) is a postponed response in which the bereaved person
may not grieve sufficiently at the time of the loss but instead has a disproportionate reaction
to a later loss, which can be much more minor than the original loss. Disenfranchised grief
(option 3) is a response to a loss, which the individual is not regarded as having the right to
grieve, or is unable to acknowledge the loss to other persons. Unresolved grief (option 4) is a
response that is prolonged or extended in length and severity of response.

1.‐ Delayed grief.


2.‐ Anniversary grief experience.
3.‐ Disenfranchised grief.
4.‐ Unresolved grief.

4188 The nurse works in a crisis clinic. A client arrives in a Correct answer: 3 Disenfranchised grief is a response to a loss or death in which the individual is not regarded as Recognize the extent of this client's aloneness and the absence of a support system.
very agitated state saying, "My life partner of 15 years having the right to grieve or is unable to acknowledge the loss to other persons. Delayed grief
has died of cancer and the family will not allow me to (option 1) is a postponed response in which the bereaved person may have a reaction at the
attend the funeral. They never accepted our time of the loss, but it is not sufficient to the loss. However, a later loss may trigger a reaction
relationship." The nurse plans to facilitate the grieving that is out of proportion to the meaning of the current loss. Inhibited grief (option 2) is a
process in this client because circumstances place the suppressed response to loss that may be expressed by somatic complaints, such as physical
client at risk for which type of grief? symptoms around the anniversary of a loss or during holidays. Unresolved grief (option 4) is a
response that is prolonged or extended in length and severity of response.

1.‐ Delayed
2.‐ Inhibited
3.‐ Disenfranchised
4.‐ Unresolved

4189 The nurse informs a 20‐year‐old client that both Correct answer: 4 Although the stages of grief should be used with caution in labeling expected behaviors and Recall commonly identified steps in the grief response. Think of grieving persons you have
parents and two older siblings were killed in an feelings, many clients will experience the five stages of grief as denial or shock, anger, known.
automobile accident. The client screams "No! No!" bargaining, depression, and acceptance. Although the stages of grief should be used with
while covering the ears and crying. Which of the caution in labeling expected behaviors and feelings, many clients will experience the five stages
following types of behavior is the nurse likely to of grief as denial or shock, anger, bargaining, depression, and acceptance.
observe next?
1.‐ Denial
2.‐ Depression
3.‐ Bargaining
4.‐ Anger

4190 When questioned by a client about what an advance Correct answer: 1 Advance directive is a general term that refers to a client's written instructions about future Recall situations in the clinical area in which advance directives have been utilized.
directive or living will is, the nurse should respond that medical care in the event that the client becomes unable to speak or is incapacitated. Specific
this type of document indicates: instructions about what medical treatment the client chooses to omit or refuse (e.g., ventilator
support) in the event that the client is unable to make those decisions is also included.
Advance directives do not specify particular practitioners or family members preferred for
providing end of life care (options 2 and 3). When an advance directive document is created,
the individual makes the decisions about future treatment to be administered or withheld
(option 4). Advance directives are not the same as designating another person to make
medical decisions for the individual; this is a specific legal process known as healthcare proxy
or medical power of attorney.

1.‐ What treatment should be provided or omitted if the client becomes incapacitated.
2.‐ Which practitioners should be allowed to provide end‐of‐life care.
3.‐ Details about preferred caregivers for end‐of‐life care.
4.‐ Which family members are to be responsible for making end‐of‐life decision.

4191 The grandfather of a 3‐year‐old client died 2 days ago. Correct answer: 3 Preschool children believe death is reversible. They do not have a developed sense of death, Review concepts of major theorists, such as Piaget.
Based on an understanding of normal growth and and they are unable to understand the permanent impact of death and dying. Children
development, the nurse anticipates hearing the young between 5–9 years of age believe wishes or unrelated actions can be responsible for death
client make a comment such as: (option 1). Preschool children do not have a developed sense of death, and are unable to
understand the permanent impact of death and dying. The responses in options 2 and 4
indicate the child is aware the death is permanent.
1.‐ "Grandfather would not have died if I had wished a little harder."
2.‐ “Grandfather will be waiting for me when I die."
3.‐ "Grandfather will be back to take me to the ballgame next week."
4.‐ "Grandfather is gone, and now I have to be strong and not cry."

4192 The client with lung cancer is expected to die within 3 Correct answer: 2 This client needs to be assisted to move through the final developmental landmarks and tasks Recognize that providing appropriate end‐of‐life care is an important nursing role.
months. The nurse determines that a priority in at the end of life, which include closure and completion in relations with family and friends.
working with this client is to assist the client to do This will also assist the family to engage in anticipatory grieving. Option 1 would actively
which of the following? prevent the client from moving through the final developmental landmarks and end‐of‐life
tasks. Relief of pain is of highest priority when the client is terminal (option 3). The nurse
should be aware of the fact that there are means of using narcotic analgesics without causing
clouding of consciousness. At end of life, the dying person should be allowed to be as
independent as possible in order to preserve self‐esteem (option 4).

1.‐ Avoid discussing the future or making future plans, as the future is uncertain.
2.‐ Discuss feelings of impending death and acknowledge the inevitable outcome.
3.‐ Verbalize need to avoid taking narcotic analgesics since they can cause clouded thinking.
4.‐ Allow caregivers to provide as much care as possible to reduce stress and to preserve energy.

4193 A client underwent a gastric banding surgical Correct answer: 1, 2 A perceived loss is experienced by one person but cannot be verified by others (e.g., loss of Recognize that this client will experience both positive and negative psychological effects
procedure as treatment for morbid obesity. Following self‐esteem or body image). An actual loss can be identified by others and can arise in from having had the surgery. Look beyond the obvious and evaluate each option in terms of
a 140‐pound weight loss, the client's body mass index response to a significant change in a person’s appearance, body, or life circumstances, such as its relevance to the client in the situation.
(BMI) is now 24.8. The client states, "I'm too fat. I weight loss after surgery. An anticipatory loss (option 3) is experienced before the loss actually
always have been." The nurse concludes that this client occurs (e.g., terminal illness). A permanent loss (option 4) is an irreversible deprivation (e.g.,
has a disturbed body image related to which type(s) of paralysis). A painful loss (option 5) is a generalized term that does not have universal meaning.
psychological loss? Select all that apply.

1.‐ Actual
2.‐ Perceived
3.‐ Anticipatory
4.‐ Permanent
5.‐ Painful

4194 Before counseling parents who have recently lost a Correct answer: 1 The capacity for self‐awareness allows the nurse to reflect and make choices. Nurses who Review concepts of grieving. Recognize that the nurse, too, can grieve.
child to death, it is important for the nurse to have understand their own feelings and beliefs will be able to be therapeutic when clients need to
already dealt with personal feelings about death, grief, address issues which are disturbing and difficult. The death of a child will personally affect the
and loss in children. Having this self‐awareness is nurse, and it is critical for the nurse to share these feelings with others, including the parents
important because it: (option 2). The nurse must be available both physically and emotionally for the parents in
discussing unpleasant and difficult feelings (options 3 and 4).

1.‐ Assists the nurse to help the parents express their grief fully.
2.‐ Prevents the nurse from being personally affected by the loss.
3.‐ Prevents the nurse from sharing any personal feelings with the parents.
4.‐ Assists the nurse to avoid discussing unpleasant feelings with the parents.
4195 A young woman arrives at a routine medical visit, Correct answer: 1 Although the loss of a child can be devastating, the ability of a parent to reintegrate Review concepts or grief and mourning. Note that the child has been dead for 2½ years.
appears depressed, and tells the nurse she is having involvement in usual activities is important to successfully resolving grief and loss. The client's
difficulty dealing with the death of her infant son. The behavior indicates that she has not moved past the initial stage of grief in which preoccupation
nurse learns the infant died 30 months ago in an with feelings of loss and intense emotional pain are prevalent. Options 1, 3, and 4 are more
automobile accident. The initial nursing diagnosis is average responses to the death of a child.
dysfunctional grieving. Which statement by the client
would support this diagnosis?

1.‐ "When my children play in playgrounds, it makes me angry that my son will never be able to play like other children."
2.‐ "I spend most of every day crying in my son's room because he's not there any more."
3.‐ "I watch other toddlers in the neighborhood play, and I wish my son was still alive."
4.‐ "I think of my son and I am sad that my new baby will never be able to know his brother."

4196 The client whose spouse died 4 months ago is Correct answer: 1 The client is showing denial, which is the earliest reaction to loss. This first stage of grieving is Notice the time frame given in the question. Compare this with theories of grief and
admitted for inpatient psychiatric care. The client has normally short‐lived, and the grieving person moves on to other grief responses, like anger. mourning.
been unable to work since the spouse's death and has The total time for acute grieving is very individualized, but feelings of numbness, emptiness,
lost 20 pounds. The client cries frequently and says, and active denial of the death 4 months later indicates that this client's mourning is not
"No, I won’t believe it. It's not true." The client further progressing normally. Disenfranchised grief (option 2) is not present because there is nothing
describes feeling "numb" and "empty." The admitting in the situation (e.g., a clandestine relationship) that would prevent the client from expressing
medical diagnosis is major depression. The nurse grief. The client is grieving the actual loss of the spouse, not another situation, as would occur
identifies which of the following as the priority in distorted grief (option 3). This client’s reaction is not consistent with the normal pattern of
problem? grieving (option 4).

1.‐ Dysfunctional grief


2.‐ Disenfranchised grief
3.‐ Distorted grief
4.‐ Normal grief

4197 A family who has gathered around their dying Correct answer: 3 Spiritual and cultural beliefs and practices greatly influence both a person's and family's When one or more options appear to be partially or completely correct, look for an
grandmother's bed refuses to allow a feeding tube to reaction to death and subsequent behavior. Options 1, 2, and 3 may also be correct, but the answer that is more comprehensive and can include the other answers.
be removed and to stop feeding her, even after the common organizing underpinnings to each of these options are cultural and spiritual beliefs.
healthcare team has stated that nothing else could be
done to help her recover. The nurse concludes that the
family's resistance to removing the feeding tube is
most likely based on which factor?

1.‐ Their refusal to accept the finality of death


2.‐ Their need to try every possible solution
3.‐ Their spiritual and cultural beliefs
4.‐ Their distrust of the healthcare system

4198 A father who recently lost his eldest son to cancer Correct answer: 1 Male or masculine expression of loss or death is commonly limited and less overt. Intense Recall information about grief styles of males versus females in American society since
refuses to share his feelings in a support group and has feelings are usually experienced privately with a general reluctance to discuss these with there is a lack of other cultural reference in the question.
not shown any tears related to the loss. What is the others. Option 2 would not be indicated at this time because the father is probably not
nurse's most appropriate interpretation of this experiencing dysfunctional grief. There is not enough data in options 3 or 4 to support these
behavior? options.
1.‐ A common expression of how men grieve loss and death.
2.‐ Dysfunctional expression of grief and the client should be referred to counseling.
3.‐ A common expression of denial and refusal to accept death or loss.
4.‐ The father's attempt to be strong for the rest of the family.
4199 A 4‐year‐old says, “If I can make a big enough wish, Correct answer: 2 Preschool children (ages 3–5) do not understand the finality of death, but instead may see it Pay attention to the age of the child. Review theories about cognitive development in
my daddy will not be dead any more.” The nurse as separation. They engage in magical thinking and truly believe in the power of wishes. children.
concludes that the child is: Magical thinking is most common in preschool aged children (option 1). The child is not
experiencing delusions or making up a story to avoid feelings (options 3 and 4).

1.‐ Expressing magical thinking common to much older children.


2.‐ Voicing thoughts that are normal for children this age.
3.‐ Delusional and should be evaluated by a psychiatrist.
4.‐ Making up the story in order to avoid feeling sad and scared.

4200 A terminally ill 78‐year‐old female client tells the Correct answer: 2 With mutual pretense, the client, family, and/or healthcare providers know that the prognosis Notice that both the client and the family are making a decision independently of each
nurse she does not want her adult children to know is terminal but agree not to talk about it and make an effort not to raise the subject. In closed other that they think is in the best interests of all. Select the option that is comprehensive
she is dying. Later that day when the adult children awareness (option 1), the client and family are unaware of impending loss or death. There is of all people in the question.
visit, they tell the nurse that they know their mother is no mutual concern (option 3). In open awareness (option 4), the client and involved individuals
dying but will not talk about this front of her. The know about the impending loss or death and feel comfortable discussing it, even though it may
nurse mentions in intershift report that this family's be difficult.
situation is one of:
1.‐ Closed awareness.
2.‐ Mutual pretense.
3.‐ Mutual concern.
4.‐ Open awareness.

4201 A frail 79‐year‐old female calls the home health nurse Correct answer: 4 As the care receiver becomes more chronically ill and the caregiving burden becomes more Imagine yourself in this spouse's situation. Can you feel the frustration, fatigue and strain?
and says, "I am a failure, and I can no longer care demanding, a great strain can be placed on the caregiver's emotional and physical health.
adequately for my husband who has Alzheimer's There is not enough data to suggest ineffective coping, dysfunctional grief, or social isolation
disease." She has been his primary caregiver for more (options 1, 2, and 3).
than 5 years. Now he has become despondent, is
unable to ambulate, and is difficult to manage. The
nurse determines that which nursing diagnosis is most
appropriate?
1.‐ Ineffective coping related to chronic illness
2.‐ Social isolation of family unit related to altered state of health
3.‐ Dysfunctional grief related to not accepting personal limitations
4.‐ Caregiver role strain related to overwhelming caregiving tasks and expectations

4202 The nurse is counseling a client who has lost the Correct answer: 3 A major outcome of grief counseling is to assist the client in sharing his or her loss and to Recall basic grief processes. Consider the normal need of the grieving person to talk about
spouse through death. Which of the following is an accept support from others. It is critical for the spouse to share the feelings of loss and grief personal loss. Recognize the mutuality of loss in supportive personal relationships.
appropriate outcome criterion? The client will: with others. A vital part of normal grieving is expressing feelings of loss and grief in a
supportive interpersonal environment, particularly with those who are most significant in the
grieving person’s life (options 1 and 4). It is too early to memorialize the spouse; the client
must grieve the loss of client first (option 2).
1.‐ Stop expressing feelings about the spouse's death.
2.‐ Plan a memorial tribute for the spouse.
3.‐ Attend grief support groups.
4.‐ Avoid sharing loss with significant others.

4203 A 34‐year‐old unconscious client with severe brain Correct answer: 2 The purpose of having a health care proxy is to allow that person the right to make health The core issue is the right of the authorized surrogate. Option 1 is an inappropriate
damage due to a ruptured aneurysm has a surrogate care decisions on the client's behalf when needed. To uphold the rights of the client at this response. Option 3 is offering a rationale for what could occur, but is not a supportive
caregiver who has decided to stop all treatment and time, when the client can no longer convey his or her own wishes, the health care providers answer for the surrogate. If the nurse knows the decision maker is an authorized surrogate,
allow the individual to die. The surrogate caregiver is need to respect the authorized surrogate's right to make the decision to refuse or stop as noted in the stem, it is an inappropriate response.
also the client's health care proxy. The nurse's best treatment.
response would be:
1.‐ "Are you certain you want to give up hope?"
2.‐ "I know this is a difficult decision to make about your loved one, but you have the right to make that decision."
3.‐ "Stopping all treatment could make the client very dehydrated and uncomfortable."
4.‐ "How do I know you have the right to make that treatment‐related decision?"

4204 A 28‐year‐old woman is dying with breast cancer. She Correct answer: 3 The goal of palliative care is to assess and treat the symptoms the dying client is experiencing. The focus is on the definition of palliative care and application to client care. Only Option 3
has metastatic disease to her bones and liver. She is to If treating the cause can alleviate the symptom, it is appropriate. Treating the disease is not is an intervention to decrease the pain.
receive palliative care. Which of the following would appropriate.
the nurse expect to be included in the plan of care?

1.‐ Chemotherapy to treat the breast cancer


2.‐ No treatments
3.‐ Radiation therapy to her right arm to treat the bone pain from metastasis
4.‐ Hydration and tube feeding only

4205 A client with a diagnosis of leukemia has excruciating Correct answer: 1 It is "ethically justified" to offer pain medication without fear of causing further respiratory Only option 1 provides immediate pain relief for the terminally ill client.
pain. It has been two hours since the previous IV depression to a terminally ill client.
medication for pain, and so it is ordered prn. The
client's vital signs are temperature 98.8° F, pulse 68,
and respirations 6. The nurse's best response would be
to:
1.‐ Medicate the client for pain.
2.‐ Wait to medicate until the respirations are greater than 8.
3.‐ Call the physician for another stat order for pain medication.
4.‐ Offer a back rub to provide an alternative intervention to medication.

4206 A non‐responsive client with signs of imminent death Correct answer: 4 Relaxation techniques and minimizing stimulation will decrease the level of delirium and the Knowledge of the signs and symptoms of imminent death are key to this question. Options
has facial grimacing, frequent moaning, and restless agitation associated with it. 1, 2, and 3 are incorrect. Option 4 identifies pain as a rationale for these unusual signs as
legs. The nurse concludes that these are probably: death approaches.

1.‐ Attempts to communicate to individuals in the room.


2.‐ Reflex responses to environmental stimuli.
3.‐ Additional signs of impending death.
4.‐ Signs of intractable pain.

4207 The family of a dying client is distressed due to the Correct answer: 3 Minimizing environmental stimulation will help to reduce the sensory input that can Identify the interventions that are factually correct and focus on options to decrease or
delirium without drowsiness that the client continues aggravate delirium. Delirium without drowsiness is an acute state of disorientation, and eliminate the delirium. Use this knowledge and the process of elimination to make a
to experience while going through the dying process. frequently is associated with agitation. It has an underlying metabolic cause, and should not be selection.
What information would be helpful for the nurse to confused with the chronic condition of dementia (option 4). Option 2 has faulty logic. Giving
provide to this family? fluids could help to reduce the dehydration that can aggravate delirium, and aspiration might
be not as critical a concern for the client who is in the process of dying (option 2). Talking to
the client should not be done for the purpose of raising the level of awareness, but rather to
comfort the client and to say goodbye (option 1).

1.‐ The family should talk to the client frequently to raise the level of awareness.
2.‐ Minimizing food and fluid given to the client will lessen the likelihood of aspiration.
3.‐ General environmental stimulation of the client should be reduced .
4.‐ This would be expected behavior if the client also has a history of dementia.
4208 A client with a terminal illness is experiencing Correct answer: 1 General muscle wasting and lack of nutrition are seen with most terminal illnesses. They are The focus of this question is knowledge of cachexia and support to the spouse in the form
cachexia. The spouse is very upset, and notes that caused by a combination of multiple factors, including metabolic changes, depression, of education. Options 2 and 4 are incorrect because more than decreased food intake is
"there has never been a problem with my husband's treatments for disease or symptoms, or even the symptoms associated with the disease, such responsible for cachexia. Option 3 minimizes the concerns of the spouse.
appetite." Which response from the nurse would be as pain.
helpful?
1.‐ "Cachexia can be caused by multiple factors, such as depression and GI disruptions."
2.‐ "The most likely reason for this loss of weight is decreased food intake."
3.‐ "I can understand you are upset, but that will not solve the problem. We need to focus on your husband."
4.‐ "Would you like for tube feeding to be started, to assure adequate nutritional intake?"

4209 A home health nurse is making a visit to a client who Correct answer: 3 Fecal impaction causes a diarrhea that has rapid onset. Diarrhea caused by anal incontinence Review the assessment information for diarrhea that occurs in clients who are near the
is terminally ill and whose condition is deteriorating. A (option 2) often occurs twice a day, and the stool caused by malabsorption (option 1) would be end of life. The incorrect options do not provide sufficient or correct explanations. Use
family member reports to the nurse that his terminally foul‐smelling, fatty and pale. Diarrhea caused by food poisoning (option 4) would occur hours nursing knowledge and the process of elimination to make a selection.
ill father experienced rapid‐onset diarrhea after eating after ingestion of food, typically accompanied by severe nausea and vomiting.
breakfast. The nurse explains this was most likely due
to:
1.‐ Malabsorption.
2.‐ Anal incontinence.
3.‐ Fecal impaction.
4.‐ Food poisoning.

4210 Assessment of the right hip of a terminally ill client Correct answer: 1 An emollient lotion will lubricate and moisturize the skin in order to maintain tissue integrity. The focus is maintaining tissue integrity. Only option 1 can accomplish this. Option 2 is
shows it has started to break down due to decreasing Alcohol is a drying agent, while hydrogen peroxide and warm towels are not indicated, and drying, and option 3 can cause cellular damage as a result of its turbulent effect. Option 4 is
circulation. The nurse determines that massage with could cause harm. rough and dry, which could cause further injury to sensitive tissue.
which of the following would be helpful to prevent
further injury to the tissue?
1.‐ Emollient lotion
2.‐ Lubricant with alcohol
3.‐ Hydrogen peroxide
4.‐ Warm towel

4211 The family of a dying client is hesitant to stop IV fluids Correct answer: 2 Dehydration causes a variety of physiological changes, inducing somnolence in the dying Only option 2 provides comfort to a dying client. The other options are incorrect.
because they are concerned it will cause suffering for client. The statements in the other options are not stated correctly, and do not help the
their dying father. It would be helpful for the nurse to client’s family to understand the role of fluid therapy in the terminally ill.
explain which of the following to the family?

1.‐ Their father cannot live without water, so he will not suffer for long.
2.‐ Dehydration increases ketone production, causing sleepiness, euphoria, and a decrease in pain.
3.‐ A lack of water will prevent the client from having loose stools immediately after death.
4.‐ Stopping fluids will alleviate edema and hearth failure.

4212 The family of a client dying from congestive heart Correct answer: 1, 2, 5 Families need to be taught that lack of fluids can promote comfort in the dying client with Options 1, 2, and 5 promote comfort in the dying client as a result of fluid limitation.
failure is concerned about making certain their mother decreased secretions, minimize fluid accumulation in the peripheral tissues, and stimulate Options 3 and 4 are incorrect.
is comfortable as she goes through the process of endorphin production. Lack of fluids, however, would contribute to the client's sense of dry
dying. Which of the following statements indicate to mouth.
the nurse that the family understands that lack of fluid
and nutrition can promote comfort as a client dies?
Select all that apply.

1.‐ "There will be less nausea and vomiting, since the GI secretions will be decreased."
2.‐ "Natural analgesia from the body's endorphin production will help with pain control."
3.‐ "A dry mouth will not be a problem, even without fluid."
4.‐ "Coughing and mucus production will continue, even after the fluids are discontinued."
5.‐ "Swelling and edema might decrease once we stop giving fluids."

4213 Which statement would the nurse use when Correct answer: 1 Knowledge that energy surges are indicative of the nearness of death can allow the nurse to Identify the correct physical symptoms in the dying client. Options 2, 3, and 4 are
explaining to a family the physical symptoms typically communicate with family members who wish to be with the client when they die. The incorrect.
seen in clients approaching death? explanations in the other options are incorrect.
1.‐ Energy surges indicate altered metabolism at the end of life.
2.‐ Restlessness at the end of life is due to lack of control of intractable pain.
3.‐ Clients frequently become irritable and agitated at the end of life.
4.‐ Activity and vigor change at least two weeks prior to death.

4214 The teenage daughter of a woman with a terminal Correct answer: 1 Role changes can cause altered family dynamics and social change. They do not necessarily Option 2 would result in more time with friends. Option 3 would result in acting out,
illness begins to have difficulty with grades and no relate to lack of discipline, sibling rivalry, or resolution of disputes with friends. negative behaviors, or "perfect child" behavior. While Option 4 is a possibility, with a dying
longer spends time with her friends. She is the eldest parent, Option 1 is the more likely answer.
of six children. The father spends a lot of time between
work and caring for his wife. The nurse concludes that
the teenage daughter could be experiencing difficulty
with which of the following?

1.‐ Role change


2.‐ Lack of discipline
3.‐ Sibling rivalry
4.‐ Resolving disputes with friends

4215 When speaking with a child who has a terminal Correct answer: 1 Being able to determine behaviors that indicate the terminally ill child no is longer interested Option 2 is a strategy to identify depression. Option 3 and 4 indicate the child is open and
illness, which behavior indicates to the nurse that the in talking is important when developing trust between client and nurse. interested in talking. Only option 1 shows a cue signifying the end of discussion.
child no longer is interested in talking about end‐of‐life
issues?
1.‐ Changing the subject
2.‐ Drawing pictures with lots of black figures
3.‐ Asking questions about heaven
4.‐ Putting a puzzle together while chatting

4216 The nurse is speaking with the parents of a dying child Correct answer: 2 Siblings often feel left out as the parents focus on the dying child. The correct statement is Option 1 and 4 indicate the child still is actively interested in seeing his dying sibling.
about how the two siblings are coping as the parents one that illustrates emotional distance between the siblings and either the dying child or the Option 3 suggests the child would like a return of normalcy to life. Only option 2 shows the
focus on the child that is ill. The parents share some parents. perfect child behavior indicative of stress in the well child.
conversations they have had with the children that
afternoon. Which statement indicates there is a
potential problem with the children feeling left out of
their parents' lives?

1.‐ "When will we get to go to the hospital?"


2.‐ "I made your bed and walked the dog while you were gone."
3.‐ "I'm tired of going to the hospital instead of playing with my friends."
4.‐ "Could we take a friend when we go to the hospital?"

4217 The nurse teaches a family that memory framing can Correct answer: 3 Life review assists the living to understand and remember the meaningful events of the dying Note that the critical word in the question is key. This tells you that more than one option
be an important activity as a loved one nears the end family member. Option 1 only includes remembering, while the option 3 (the correct option) may be partially or totally correct, and that you must choose between competing options.
of life. The nurse explains to the family that a key incorporates the importance of the memory to the dying individual, and is therefore of greater Eliminate option 4 first as inappropriate. Eliminate options 1 and 2 because they are similar
benefit of this intervention is that it will assist them to: overall benefit. Option 2 might be interesting, but is not important in this setting. Option 4 is in essence, and must therefore be incorrect.
inappropriate.
1.‐ Reminisce as they wait for the death of their family member.
2.‐ Recall the capers performed by the grown children when they were teenagers.
3.‐ Identify the important memories held by the dying family member.
4.‐ Discuss memory loss of the member who is dying.

4218 Nurses who continuously care for clients at the end of Correct answer: 4 Expression of grief in "safe" and supportive situations helps the nurse acknowledge and Only option 4 indicates a situation where nurses discuss the death of clients. Option 3 is
life need to have formal support mechanisms to express the sadness and mourning experienced while caring for multiple and continuing client important, but allows the nurse to "get away" from the clinical setting. Option 2 identifies
express their grief. Which statement indicates a nurse deaths. how family expressions of gratitude make the nurse feel appreciated, but does not address
has been able to effectively deal with grief? debriefing of feelings. Option 1 only acknowledges a death.

1.‐ "I read the obituary of our client who died yesterday."
2.‐ "The family brought a gift basket to the office to express their thankfulness for our care. That makes it all worth it."
3.‐ "My vacation is only one week away."
4.‐ "The weekly clinical debriefings are tough when we have a lot of deaths."

4219 Which statement indicates the nurse who works with Correct answer: 2 Bereavement overload and dysfunctional grieving can cause a nurse to experience death Option 2 indicates a nurse who is focusing only on the physical needs of the client and
terminally ill clients might be experiencing death anxiety. avoiding emotion‐laden issues. Options 1, 3, and 4 are statements of a nurse who is still
anxiety? focusing on the emotional needs of the dying.
1.‐ "I have three clients who are close to death and require lots of emotional support."
2.‐ "It is all I can do to deal with their physical needs."
3.‐ "It was tough, but I felt complimented to be at the bedside at the time of death."
4.‐ "We were afraid the eldest son was not going to get there in time to make the decision before the client died."

4220 Nurses working with dying clients might need to Correct answer: 4, 2, 3, Recognizing the five stages of adaptation is important for nurses working with dying clients Think “ISDAD” adapting? It is an acronym for Intellectualization, Emotional Survival,
move through the five stages of adaptation. Place 5, 1 and their families. Depression, Emotional Arrival, and Deep Compassion.
these stages in the order of their occurrence by
clicking and dragging the options below to move them
up or down.
1.‐ Deep compassion
2.‐ Emotional survival
3.‐ Depression
4.‐ Intellectualization
5.‐ Emotional arrival

4221 An organ transplant nurse is meeting with various Correct answer: 3 Only option 3 includes a religion or culture that is restricted in the donation of an individual's To answer this question correctly, it is necessary to know specific cultural information as it
cultural and religious groups to discuss organ donation. organs. Individuals with the religious backgrounds in Options 1, 2, and 4 are able to donate if relates to organ donation. Use this knowledge and the process of elimination to make a
The nurse is aware that members of which group are they desire. Nurses need to be aware of the diverse cultural religious practices and beliefs. selection.
restricted from donating their organs?

1.‐ Hindu
2.‐ Islam
3.‐ Native American
4.‐ Christianity

4222 An Islamic client has died, and the nurse has entered Correct answer: 4 Nurses need to be aware of the diverse religious practices and beliefs and the implications for Only option 4 is characteristic of Islam. The other options are incorrect.
the room where the family is present. The nurse needs the end of life.
to be aware of which of the following?

1.‐ Public grieving is accepted.


2.‐ The client and family believe in reincarnation.
3.‐ They will actively mourn, and would appreciate nursing intervention.
4.‐ Grief counseling might be intrusive.
4223 When conducting an admission evaluation or an Correct answer: 2 Clients need to have communicated to them that they are in control of their own behaviors The core issue of the question is effective communication with a client at risk for acting
assessment of the client within the unit for the and that "acting out" will result in consequences. Reassuring the client that the staff will make out. Use the process of elimination and choose the option that provides accurate
potential for violent or aggressive behavior, it is sure nothing happens (option 1) takes away responsibility from the client. Just explaining that information to the client and holds the client accountable for his or her actions.
important for the nurse to: violence is unacceptable and not explaining to the client that he or she is in control (option 3)
is nontherapeutic. Acting out is usually not allowed (option 4) because of safety of client and
others.
1.‐ Reassure the client that everything will be all right, and the staff will make sure nothing untoward happens.
2.‐ Reinforce that the client is solely responsible for his or her own actions and will experience the consequences of acting out.
3.‐ Explain that violence is not acceptable, and the staff will not allow the client to act out.
4.‐ Reassure the client that limited acting out will be allowed but only in a controlled setting.

4224 When responding to clients who display the potential Correct answer: 4 Preventing a client from free mobility is the most restrictive technique. Meeting in a quiet Note the critical word most in the stem of the question. This tells you that you must order
for violence, the nurse would use which of the room (option 1) is the least restrictive and most therapeutic. Chemical restraint (option 2) and the interventions presented from least restrictive to most restrictive to enable you to
following as the most restrictive intervention? escorting a client (option 3) are restrictive but less so than full four‐point restraint. choose correctly.

1.‐ Meeting in a quiet room to reduce stimulation


2.‐ Administering a PRN medication to reduce anxiety
3.‐ Providing physical interventions, such as two‐person escort out of a program area
4.‐ Using restraints, such as a four‐point restraint

4225 Which of the following is the most important Correct answer: 1 Once a client has escalated beyond least restrictive interventions, the nurse should plan for The wording of the question tells you that more than one option may be partially or totally
intervention by the nurse when a client does not the next step. Bargaining (option 2) with a client is counterproductive and positively reinforces correct and that you must prioritize your answer. Choose the option that best protects the
respond to less restrictive interventions and is rapidly behavior. Offering a PRN medication (option 3) to reduce anxiety would occur after safety of all people in the environment, including other clients and staff.
escalating toward violence? negotiation for least restrictive interventions is complete. Asking a client to take a time out
(option 4) is a least restrictive intervention to which the client is not responding.

1.‐ Cease negotiation with client and implement plan of intervention to control client and provide safety.
2.‐ Bargain with client to determine what can be done to prevent assaultive behavior.
3.‐ Offer a PRN medication to reduce anxiety.
4.‐ Ask client to move to a less stimulating, private area and spend some time alone.

4226 After a staff member has been involved in a Correct answer: 2 Debriefing allows the staff an opportunity to ventilate feelings and to calm down (option 1). It The core issue of the question is the need for staff to process personal feelings after an
particularly violent episode with a client, debriefing should always occur, and all staff should be encouraged to participate (option 3). Debriefing episode of violence occurs with a client. Use the process of elimination and knowledge that
should: following a violent episode should occur as soon as possible after the client and others are safe staff can be traumatized by these events to choose the correct option.
(option 4).
1.‐ Occur after the staff has had the opportunity to calm.
2.‐ Take place immediately to facilitate processing of feelings.
3.‐ Not occur until the staff requests such intervention.
4.‐ Be done after a 3‐day time‐off period.

4227 Which one of the following situations experienced by Correct answer: 2 A situational crisis is one that occurs from external life events. An event involving normal Use the process of elimination. The core issue of the question is the ability to differentiate
a client would the nurse document as a situational stages of development (option 1) is a maturation crisis. A natural disaster (option 3) and an among various types of crises and to document them appropriately.
crisis? armed conflict (option 4) are examples of community crises.
1.‐ Being in the middle of menopause
2.‐ Recently being involved in an automobile accident
3.‐ Being a survivor of a flood following a hurricane
4.‐ Recently returning home from military duty after an armed conflict
4228 Which of the following coping behaviors does the Correct answer: 3 When a person is threatened and perceives himself or herself to be vulnerable to a situation, The core issue of the question is the expected response of a client to a threatening
nurse expect to note in a client who encounters a coping behaviors are self‐protective. Coping behaviors may be ineffective to provide strength situation. Use knowledge of coping skills and the process of elimination to find the correct
situation in which there is a significant psychological (option 1). Coping during a crisis is oriented toward the immediate here‐and‐now, not mastery answer.
threat and great personal vulnerability? (option 2). Coping behaviors may or may not be immobilized (option 4).

1.‐ Finding inner strength to get through the crisis


2.‐ Being more oriented toward mastery
3.‐ Acting in a more self‐protective manner
4.‐ Being totally immobilized

4229 A nurse is planning an intervention for a client in crisis Correct answer: 2 Assisting the client in identifying coping patterns and then supporting them is essential to The critical words in the question are at this time. This tells you that more than one option
who witnessed a violent crime. Which of the following managing a crisis. Identifying the client's maladaptive coping mechanisms (option 1) may be may be correct, but one of them is more timely than the others. Use nursing knowledge
is a key component of crisis intervention that the nurse beneficial after identifying the client’s strengths. Assisting the client to forget (option 3) is not and the process of elimination to make a selection.
should plan to utilize at this time? a therapeutic intervention for crisis management. Teaching a client to handle future crises
(option 4) is more appropriate once the current crisis has abated.

1.‐ Identify the client's maladaptive coping mechanisms.


2.‐ Identify and support the client's coping patterns.
3.‐ Assist the client in forgetting the crisis situation.
4.‐ Teach the client to handle future crises.

4230 The nurse developing a care plan for a client using Correct answer: 1 Providing support and guidance are the primary objectives of crisis management. The client's The critical word in the question is primary. This tells you that more than one option may
crisis management principles would base interventions anxiety (option 2) may be needed in order for him or her to be energized to cope with the be correct, but one of them is more important than the others. Use nursing knowledge and
on which of the following primary tasks of crisis crisis; the goal is to achieve a manageable level of anxiety. Providing encouragement (option 3) the process of elimination to make a selection.
management? and fostering independence (option 4) are important and may occur during crisis intervention,
but they are not the primary task of crisis management.

1.‐ Provide support.


2.‐ Relieve anxiety.
3.‐ Provide encouragement.
4.‐ Foster independence.

4231 An adult client is having difficulty coping with a new Correct answer: 3 Short‐acting antianxiety agents are most useful in helping a client to achieve an effective The critical words in the question are crisis situation, which tell you that the correct
diagnosis of colon cancer. The nurse telephones the reduction in level of anxiety. Antipsychotics (option 1) should be avoided. Antidepressants answer is a medication that will have a rapid onset of action and be effective in treating the
physician for an order for medication therapy to assist (option 3) require some time to achieve therapeutic levels and are not useful in a crisis client’s reaction to the diagnosis of colon cancer. Use nursing knowledge and the process of
the client in coping with this crisis situation. The nurse situation. Mood stabilizers (option 4) are not indicated. elimination to make a selection.
anticipates an order for which of the following types of
medication?

1.‐ Haloperidol (Haldol)


2.‐ Amitriptyline (Elavi)
3.‐ Lorazepam (Ativan)
4.‐ Valproic acid (Depakote)

4232 The nurse working with the family of a client with Correct answer: 3 Medications will help decrease the frequency and intensity of suicidal thoughts. Medication The critical word in the question is best. This tells you that more than one option may be
suicidal ideations is asked if the medication the client may treat the underlying cause of the suicidal ideation but does not necessarily reduce the risk partially correct, but one of them is better than the others. Use nursing knowledge and the
is taking will prevent suicide. Which of the following for completing suicide. Medication does not prevent suicide; in fact, many times when clients process of elimination to make a selection.
would be the best response by the nurse? regain their energy from medications, they are at an increased risk for completing suicide
(options 1 and 2). A client may not be currently suicidal, but medications do not assure that
they will not be suicidal in the future (option 4).

1.‐ "Clients who take their medication as prescribed are at decreased risk for suicide."
2.‐ "Medication helps to treat an underlying mood disorder associated with suicidal thinking and therefore prevents suicide."
3.‐ "Medication helps decrease the frequency and intensity of suicidal thoughts."
4.‐ "The client has said that she would never try to hurt herself again. There is no need to worry."

4233 A suicidal client with low self‐esteem seems less Correct answer: 4 A client who is just regaining his or her energy should be encouraged to do simple tasks, The core issue of the question is a safe activity for a client who is suicidal. The correct
lethargic today and agrees to participate in an which will also promote the client's self‐esteem. Suicidal clients are most at danger when they answer is the option that does not pose risk to the client or provide the client with the
occupational therapy program. To help make the are feeling better and regaining their energy. Introducing the client to wood carving (option 1) means to engage in self‐harm.
session successful, the nurse should do which of the and making a belt from rope (option 3) place the client at risk for self‐harm. The nurse should
following? encourage the client participate in the occupational therapy for self‐expression (option 2).

1.‐ Introduce the client to wood carving; show him how to safely use the carving and burning tools.
2.‐ Stay away from the client in occupational therapy so that he is free to express himself.
3.‐ Teach the client to macrame a plant hanger from jute rope and encourage him to work on it later in his room.
4.‐ Structure his activity to help him complete one simple task, such as painting a picture.

4234 A client has recently been admitted for depression Correct answer: 4 Suicidal clients are at most risk when they begin to demonstrate improvement and have the The core issue of the question is recognition that the risk of suicide increases when a client
and suicidal ideations with a plan to hang himself. The energy to carry out suicide. A mute client who is not willing to share with others (option 1) is at begins to feel better, since the client now may have the energy to carry out a suicide
nurse assesses the client most carefully for risk for risk for suicide but may be placed on constant observation. Being afraid to go home (option 2) attempt. Use the process of elimination and this knowledge to make a selection. The
attempting suicide when: may be a positive sign that the client is aware of the danger he may pose to himself. Vacation wording of the question tells you that only one answer is correct.
is a stressful time, and being left alone (option 3) would place the client at risk; however, it is
well documented that clients are at greatest risk when showing signs of improvement.

1.‐ He is mute and unlikely to tell anyone.


2.‐ He is ready to go home and afraid of leaving the hospital.
3.‐ His family goes on vacation.
4.‐ He begins to demonstrate clinical improvement.

4235 Which of the following individuals is at greatest risk Correct answer: 2 The group at highest risk for successfully completing suicide attempts are European‐American The core issue of the question is knowledge of high‐risk groups for suicide. The wording of
for suicide? males over the age of 50 (white, male, older adult). The clients in options 1, 3, and 4 are not in the question tells you only one answer is correct. Use nursing knowledge and the process of
high‐risk groups. elimination to make a selection.
1.‐ A 65‐year‐old African‐American male
2.‐ A 70‐year‐old European‐American male
3.‐ A 30‐year‐old Hispanic‐American female
4.‐ A 16‐year‐old African‐American female

4236 A client states that voices are telling him to hang Correct answer: 4 Voices telling a client to hurt himself or others are called command hallucinations. There is The core issue of the question is correct interpretation of a client's symptoms. The
himself. The nurse documents that the client is at risk not enough data to support hopelessness (option 1), emotional pain (option 2), or delusions of wording of the question tells you only one answer is correct. Use nursing knowledge and
for suicide on the basis of which of the following? grandeur (option 3). the process of elimination to make a selection.

1.‐ An intractable sense of hopelessness


2.‐ Intolerable emotional pain
3.‐ Delusions of grandeur
4.‐ Command hallucinations

4237 Which statement made by a client would indicate the Correct answer: 1 The client is communicating that he or she may not be around for the nurse to worry about. The critical words in the stem of the question is highest risk. This tells you that more than
highest risk for suicide? Creating a solution (option 2), expressing hope for the future and making plans (options 3), and one option may indicate risk, but one is stronger than the others. Use nursing knowledge
decreasing frequency of voices (option 4) indicate that the client is experiencing a reduction in and the process of elimination to make a selection.
the risk for suicide.
1.‐ "I know you've been worried about me. You won't have to worry too much longer."
2.‐ "I think I've found a solution to my problem. I'm going to check it out with my doctor."
3.‐ "I'm looking forward to the holiday season and the kids coming home from school. They will be a good distraction."
4.‐ "Over the past week I have been hearing the voices that tell me to hurt myself less often."
4238 A client who became violent on the psychiatric unit Correct answer: 3 Releasing restraints at least every 2 hours is a standard of care to prevent physical harm. The core issue of the question is knowledge of safe care to a client who is in restraints. Use
had restraints applied at 10:00. The nurse makes a Every 15 minutes (option 1) or hour (option 2) may be too often, and every 4 hours (option 4) the process of elimination and nursing knowledge to make a selection, recalling that 30‐
note to release the restraint at no later than which of is too long and may cause the client injury. In addition to the intervention described, the minute circulation checks and 2‐hour release times are standards of care.
the following times? client's circulation should be checked every 30 minutes. Ensuring the client's safety and well‐
being are a priority.
1.‐ 10:15
2.‐ 11:00
3.‐ 12:00
4.‐ 14:00

4239 A female client has been admitted to the psychiatric Correct answer: 2 Safety of the client is always a priority for clients who have recently attempted suicide. The critical word in the stem of the question is priority. This tells you that more than one
unit after spending 24 hours in the intensive care unit. Options 1, 3, and 4 are all appropriate goals after safety has been assured. or all options may be correct actions for the client, but one is more important than the
Before the client's admission, she overdosed on 12 others. To aid in making a selection, recall that safety needs are high priority for clients
sertraline (Zoloft) tablets. Of the following nursing following a suicide attempt.
goals, which would be a priority on admission?

1.‐ Assuring the client that someone is concerned about her


2.‐ Protecting the client until she can protect herself
3.‐ Teaching the client how to solve problems
4.‐ Discussing the meaning of death

4240 When interviewing a potentially violent or aggressive Correct answer: 1 The nurse should ensure that the interview be conducted in a quiet environment. Interruption The critical words in the stem of the question are most important. This is a clue that more
client, which of the following environmental factors is should be kept to a minimum (option 2), but may not be possible to prevent. Intimidation of than one option may be partially or totally correct, but one is more important than the
most important for the nurse to consider? the client (options 3 and 4) is inappropriate. others. Use nursing knowledge and the process of elimination to make a selection.

1.‐ The interview should take place in a calm and quiet area to reduce stimuli.
2.‐ Care should be taken to make sure that other staff do not interrupt.
3.‐ Restraint devices should be in full view of the client to reinforce consequences for violent behavior.
4.‐ The client should be told that violent behavior will not be tolerated.

4241 A new nurse orientee asks why a client admitted to Correct answer: 1 Decreasing sensory input may decrease the anxiety or anger and help the client regain The core issue of the question is the benefit of seclusion as a therapy for a potentially
the psychiatric unit has been placed in seclusion. The control. Seclusion should never be used for staffing ratios (option 2). Communication with violent client. Use nursing knowledge and the process of elimination to make a selection.
nurse who is precepting the orientee explains that others (option 3) is part of milieu therapy. Seclusion takes away the client's responsibility
which of the following is a benefit of seclusion? temporarily (option 4).

1.‐ The reduced sensory input allows the client to regain control.
2.‐ The unit can be managed with fewer staff.
3.‐ Clients are encouraged to communicate with others.
4.‐ Clients are forced to be responsible for themselves.

4242 A 19‐year‐old female client recently admitted after Correct answer: 4 Option 4 provides the client with information that the nurse is concerned about her, which The core issue of the question is a therapeutic communication technique to use with a
attempting suicide becomes very dejected and states may ease her emotional pain. Telling the client, "Of course people care" (option 1) is false client who attempted suicide and is experiencing emotional pain. Use knowledge of
that life is not meaningful and no one really cares what reassurance. Telling the client not to talk about sad things (option 2) invalidates and ignores therapeutic communication techniques and the process of elimination to make a selection.
happens to her. The nurse's best response would be the client’s feelings. Option 3 may be seeking clarification but may also cause the client to feel
which of the following? she has to defend her position.
1.‐ "Of course people care. Your parents stayed with you in the ICU."
2.‐ "Let's not talk about sad things. Why don't we go for a walk?"
3.‐ "Can you write down a list of who does not care for you?"
4.‐ "I care about you, and I am concerned that you feel so down."
4243 The nurse has been working with a teenage female Correct answer: 1, 3, 4 The client demonstrates effective coping by being able to discuss the incident without The core issue of the question is knowledge of adaptive responses to crisis or near‐crisis
client who was in crisis after she was assaulted and excessive distress and formulating a realistic plan to prevent recurrence, which will reduce situations. Choose the options that demonstrate adequate coping, which are ones that are
robbed late one night when leaving a local mall at anxiety (parking closer to buildings and enrolling in self‐defense classes). If the client states she neither insufficient nor extreme in tone.
closing time. Which of the following outcomes will never shop at the mall again, or will go shopping only when accompanied, this shows
indicates to the nurse that the client has achieved the unresolved anxiety and a nonadaptive approach that is likely to interfere with her lifestyle.
expected outcomes of treatment? Select all that apply.

1.‐ The client can talk about the incident without excessive distress.
2.‐ The client states she will never shop at the mall again.
3.‐ The client states a plan to choose parking spaces that are close to building entrances when possible.
4.‐ The client is enrolling in a local self‐defense class for women.
5.‐ The client states she will go shopping only when she has someone available to accompany her.

4244 A client recently admitted to an in‐patient psychiatric Correct answer: 1, 2, 3, Options 1, 2, 3, and 4 are correct. Psychomotor retardation is a pervasive phenomenon Develop a mental picture of this client sitting before you. Notice that four of the options
unit demonstrates extreme psychomotor retardation. 4 involving slowing of all bodily and psychologic processes. Clients with psychomotor retardation are physiologic.
Unless prompted, the client sits and stares at the floor not only move, think and act slowly; they also experience slowing of involuntary internal
for prolonged periods of time. The nurse considers processes and are therefore at risk for a wide variety of physical and emotional complications.
that this client is at risk for which problems? Select all Option 1 is correct as visceral functioning is highly impacted by the state of depression. This
that apply. leads to hypomobility and hyposecretion of the gastrointestinal tract. Constipation, fecal
impactions and even bowel obstruction are possible. Other factors contributing to constipation
are decreased physical activity, medication side effects and food and or fluid deficiency.
Additionally, the client may not be responsive to normal sensations to evacuate the bowel.
Option 2 is correct because the client with severe psychomotor retardation will have
insufficient energy to initiate drinking and ask for fluids. Additionally, the client will tire easily
and may not be able to drink even one full container of liquid. Option 3 is correct as the client’s
psychomotor retardation will be accompanied by fatigue and anergy, which will contribute to
activity intolerance. Option 4 is correct because this client is remaining immobile and
maintaining the same position for prolonged periods of time. This increases the risk of
pressure related skin problems, which can be intensified by fluid and or nutritional deficits
accompanying extreme psychomotor retardation. Additionally, the client’s responsiveness to
painful stimuli can be diminished. Option 5 is incorrect as the client is not at risk for impaired
individual coping. Instead, this is an actual problem of the client.

1.‐ Constipation
2.‐ Deficient fluid volume
3.‐ Activity intolerance
4.‐ Impaired skin integrity
5.‐ Ineffective coping
4245 A client has recently been admitted for severe major Correct answer: 4 Option 4 is correct. Suicidal clients who are depressed are at highest risk for suicide when Remember that the nurse should be particularly vigilant when a depressed client's mood
depression and a plan to cause self‐harm using a gun. they begin to demonstrate improvement and have sufficient energy to carry out a suicidal act. and energy level begin to improve.
The nurse considers that while in the hospital, this Thus for a time, treatment and improvement, instead of decreasing the suicidal risk, actually
client will be at highest risk for attempting suicide increases it. Option 1 is incorrect because in the severely depressed client, mutism is
when: considered to be a manifestation of psychomotor retardation. In addition to slowing visceral
and motor responses, psychomotor retardation also affects cognitive processes that would be
needed for formulating and acting on a suicidal plan. Option 2 is incorrect as interestingly, non‐
responsiveness to medication may not increase the client's risk for suicide while in the
hospital. If the client is showing severe psychomotor retardation, medication non‐
responsiveness would lead to continuance of severe depressive symptoms that would make
the client unable to form or act on a plan for suicide. Option 3 is incorrect because before the
medication is begun, or before it becomes effective, the client’s risk for suicide remains stable.
As stated in Options 1 and 2, the severely depressed client typically does not have the
cognitive or the physical capacity to formulate or carry out a suicidal plan.

1.‐ Mute and uncommunicative.


2.‐ Unresponsive to medication.
3.‐ Medication is first being initiated.
4.‐ The level of depression decreases.

4246 When the nurse is interviewing a potentially violent Correct answer: 3 Option 3 is correct. The nurse should ensure that the interview be conducted in a non‐public, Remember that agitated and potentially violent persons are hyperresponsive to the
or aggressive client, which of the following quiet area in order to reduce stimuli. A potentially aggressive client is hypervigilant, environment and will respond aggressively to events that others would not notice or would
environmental factors is most important? The distractible, and over reactive. At the same time, the nurse needs to be aware of own safety as not consider bothersome.
interview should be: well. Option 1 is incorrect because the potentially violent person is likely to perceive an
extended hand, or the touch associated with it, as a threatening gesture and/or act of
aggression. Touch of any type should be avoided or used very cautiously with aggression prone
clients. Option 2 is incorrect as large public areas of a unit are designed for use by groups of
clients and staff. In such a setting, people come and go freely, and these kinds of interruptions
and distractions are likely to further agitate the client. Option 4 is incorrect because this client,
who is hyperresponsive and suspicious of the motives of others, is likely to interpret this as a
statement of threat.

1.‐ Initiated with a firm handshake.


2.‐ Conducted in a large public area of the unit.
3.‐ Conducted in a calm quiet area.
4.‐ Preceded by informing the client that violent behavior will not be tolerated.

4247 The client is suicidal. A nursing diagnosis is Correct answer: 4 Option 4 is correct. Having clients verbalize feelings of anger and hopelessness that have led Recall that nursing outcomes are ultimately directed toward modification or elimination of
"Ineffective coping related to feelings of anger and to their being suicidal is an initial step in helping clients learn how to cope more effectively. the identified problem.
hopelessness." Which expected short‐term outcome is One cannot cope if one does not know exactly what the adaptive challenges are. Option 1 is
the most appropriate? The client will cope more incorrect as denying feelings of hopelessness and anger is not therapeutic. Indeed, the nursing
effectively as evidenced by: diagnosis indicates that this client’s impaired coping was attributable to underlying feelings of
anger and hopelessness. These feelings must be acknowledged and dealt with directly if this
client is to experience improved coping and reduced potential for suicide. Option 2 is incorrect
because it is stated in an immeasurable way. How does one measure happy behavior? How can
it be recognized? Also, the nurse should keep in mind that absence of anger does require that
one be happy. Option 3 is incorrect as voicing no complaints does not necessarily mean that
anger and hopelessness are absent. Various meanings for this behavior are possible. For
instance, the individual could be suppressing verbal expression of strong negative feelings. The
client could be displaying anger passive‐aggressively, or be knowledgeable about the
hospitalization process and be aware that voicing complaints will delay the date of discharge.

1.‐ Denying feelings of hopelessness and anger.


2.‐ Demonstrating happy behavior.
3.‐ Voicing no complaints.
4.‐ Verbalizing feelings of anger and hopelessness.

4248 When the client is at high risk for suicide, which Correct answer: 2 Option 2 is correct. When the client is at risk for suicide, the highest priority of the nurse is to Recall that safety and security, along with basic physiologic needs, are always of highest
action is of highest priority to the nurse? provide security and safety measures for the client. This will involve different levels of special priority.
interventions and precautions, beginning with constant visual observation of the client and
confinement in a safe physical environment. As the client improves, less intense and or less
frequent observations of the client are necessary. Option 1 is incorrect because while
medications may be expected to be effective in reducing the client’s level of suicidality, the
most important priority of the nurse is to provide a safe physical and psychological
environment. Option 3 is incorrect as the nurse should encourage direct and open discussion
of the client's suicidal thoughts and feelings. Contrary to popular opinion, talking about suicide
does not increase the risk of suicide. Instead, if discussed in a caring and nonjudgmental
manner, talking about the suicidal urges may diminish the likelihood of the person's acting on
them. Additionally, in order for the nurse to know the level of potential lethality in the client’s
situation, direct and frank discussion between the nurse and the client must occur. Option 4 is
incorrect because social isolation is a risk factor for suicide. The more socially isolated a person
is, the more likely it is that they will act on suicidal urges.

1.‐ Administer medications to make the client less suicidal.


2.‐ Monitor the client's location and behavior constantly.
3.‐ Change the subject whenever the client mentions suicide.
4.‐ Allow client time alone to reflect on feelings.

4249 The client is being discharged after a suicidal crisis. Correct answer: 2 Option 2 is correct. Alcohol use is highly correlated with suicide attempts, both in dependent Recall the prevalence of alcohol being used as a self medication when people are stressed
When giving the client discharge instructions, what and non‐dependent drinkers. The alcohol does not cause the suicidal act, but since alcohol is a or distressed.
should the nurse emphasize? CNS depressant, use of it impairs thinking and judgment. Option 1 is incorrect as the nurse
should anticipate that the client will function best if able to re‐connect to a former social
support system. This is one of the principles of crisis intervention. Option 3 is incorrect
because eating a nutritious diet is important for all persons, not just for those who have been
suicidal. Option 4 is incorrect as unless the client has agoraphobia or for some other reason is
uncomfortable around crowds, this is not a relevant intervention for the post‐suicidal client.

1.‐ Developing a new set of friends


2.‐ Avoiding alcohol use
3.‐ Eating a nutritious diet
4.‐ Avoiding crowds

4250 While taking a comprehensive history, the nurse Correct answer: 3 Option 3 correct. It assumes that the client was abused. However, not all clients who are Notice that the stem of this question is asking not for the best answer, but for an answer
wants to assess the client's potential for violence or violent or aggressive have been sexually or physically abused. Asking the question as if the that identifies the least effective approach. Thus the correct option is the most incorrect
aggression. Which question would be least client were sexually or physically abused may cause them to become aggressive. If the nurse question by the nurse.
appropriate? wants to know whether the client was a childhood victim of abuse, a better way of asking the
question would be, "Have you ever been sexually or physically abused?" Options 1, 2, and 4 are
incorrect as all of these options are appropriate questions to ask a potentially violent or
aggressive client. They should be asked calmly and matter‐of‐factly.

1.‐ "What is the closest you have come to being violent?"


2.‐ "Have you ever been arrested for violent behavior?"
3.‐ "Were you violent when you were abused as a child?"
4.‐ "Do you worry about being violent?"
4251 Referring to a client who made an unsuccessful Correct answer: 4, 5 Options 4 and 5 are correct. While each of the statements carry some weight as a risk factor Look for current factors that are "red flags" indicating a serious situation.
attempt at suicide, a staff member says, "It wasn't a for suicide, two are particularly significant: overdose of tricyclic antidepressant and plan to
real attempt. She just wanted attention." To assist the drive car into a tree. Tricyclic antidepressants, more than many other antidepressants and
staff member understand the potential lethality in this antipsychotics, are very dangerous when taken in overdose, as they can cause significant
client's situation, on which client‐related factor should cardiac dysrhythmias. Planning to drive a car into a tree (assuming that trees exist in the
the nurse's response focus? Select all that apply. immediate environment) constitutes having a potentially lethal plan with available and
proximate means for suicide, a potentially lethal combination of factors.

1.‐ Last child left for college one month ago


2.‐ Divorced for 10 years
3.‐ Increased stress at work
4.‐ Overdose of tricyclic antidepressant yesterday
5.‐ Suicidal plan for driving car into a tree

4252 The day after a suicide attempt, a client is in tears and Correct answer: 3 Option 3 is correct. The client is still verbalizing hopelessness and worthlessness. The client is Consider carefully what the client has said. Notice its negative character. Eliminate any
tells the nurse, "I'm a failure at everything, and now, still at risk for suicide. Option 1 is incorrect as the client is showing no signs of remorse. The option that has a positive character.
I've even failed at killing myself." What interpretation client is at risk to attempt again because of feelings of worthlessness and personal failure.
of this statement by the nurse would be best? Option 2 is incorrect because the client is not verbalizing guilt or thankfulness to be alive. The
client is verbalizing feelings of worthlessness and personal failure and remains at risk for
suicide. Option 4 is incorrect as the client is not verbalizing ambivalence. The client’s statement
should be interpreted as an indication that the client is likely to attempt suicide again.

1.‐ The client is remorseful over the failed suicide attempt and unlikely to try again.
2.‐ The client feels guilty over the suicide attempt and is happy to be alive.
3.‐ The client verbalizes hopelessness and the potential for another attempt is great.
4.‐ The client is ambivalent about whether to live or die.

4253 When teaching staff members how to deal with Correct answer: 4 Option 4 is correct. Violent and aggressive behavior is a learned response that can be Recognize that staff members need practical, helpful information, rather than lengthy
potentially violent and aggressive client behaviors, changed. Option 1 is incorrect because many times, strict, rigid rules cause clients to "act out" more scientific explanations.
what will be important for the nurse to emphasize? in anger and aggression. Option 2 is incorrect as it is appropriate and expected for nursing staff
to share their fears, anxieties, and concerns with their peers. Option 3 is incorrect because
when demands for performance and/or participation are made, the likelihood of violence
increases.
1.‐ Violent/aggressive behavior is best managed by strict behavioral controls, rigid unit rules, and medication.
2.‐ Nursing staff should not discuss with peers their feelings of fear and anxiety.
3.‐ Violent/aggressive behavior will decline when performance and participation are allowed.
4.‐ Clients can understand the reasons for their behavior and change their response patterns.

4254 The client presents in a crisis center saying, "They Correct answer: 3 This question is asking for the nurse to recognize that availability of social support, one of the Recall theory about balancing factors in crisis situations. Determine which of the balancing
didn’t warn me. After 20 years, and they just walk in balancing factors that determine whether one will enter into a crisis state, is absent. Balancing factors is/are absent.
and say I no longer have a job." The client's personal factors include how the person perceives the event, past experience in coping, available coping
counselor is ill and unavailable, and the immediate mechanisms, and availability of people who can be supportive. There is no indication that the
family is away and unreachable by telephone. The client is misperceiving the event (option 1). Option 2 indicates a normal or expected response
nurse interprets that the most significant reason this to an unexpected loss. While the nurse will observe the present state of confusion and shock in
client is in crisis is that the client: assessing the client, this factor does not address why the client is in a crisis state at this time.
Going to the crisis clinic for assistance (option 4) indicates that the client is making a serious
attempt at coping. The client is feeling overwhelmed and does not have access to the normal
support system.

1.‐ Is misperceiving the event.


2.‐ Feels confusion and shock about the event.
3.‐ Is unable to process the event with the usual support network.
4.‐ Is not making sufficient attempts to cope with the event.
4255 An unlicensed mental health worker asks the nurse to Correct answer: 2 Crisis intervention assists a client in resolving an immediate problem that the client perceives To select the correct response, translate more complex theoretical concepts into easily
explain how crisis intervention works. Which of the as overwhelming. Issues from early life experiences and the client’s personality (option 1) are understood language.
following is the most appropriate response of the not dealt with during crisis intervention. Crisis intervention focuses on the immediate situation
nurse? "Crisis intervention helps the client to: and presenting problem. Other issues, such as personality and early life experiences are dealt
with in other therapy modalities and would not be addressed until the presenting crisis is
resolved. In early crisis intervention, rather than teaching the client to develop new coping
techniques (option 3), the client is encouraged to use previously successful coping skills. The
emphasis of crisis intervention is on strengths and coping skills of the individual, not on
personal limitations (option 4).

1.‐ Uncover unconscious processes and early life experiences."


2.‐ Find a solution to an immediate and overwhelming problem."
3.‐ Use new ways of coping with an unexpected major problem."
4.‐ Become aware of personal limitations that led to the crisis state."

4256 The client in a crisis state is having difficulty asking for Correct answer: 1 It is natural for clients in crisis to feel isolated and withdrawn. Clients frequently need help Realize the client has experienced a crisis. Analyze each option and put yourself in the
help from significant others. The nurse explains to communicating with others directly, especially if they place a high value on independence. Role client’s position. Select the option that characterizes how you would expect to ask in a crisis
caregivers that it is important to role model asking for modeling by the nurse helps the client to learn this skill. Option 2 (e.g., being resistant to situation.
help because clients in crisis are overwhelmed and: verbal suggestions) is in opposition to a common characteristic of the client in crisis, which is
openness to suggestions. Option 3 (e.g., being hesitant) is also in opposition to a common
characteristic of the client in crisis: readiness to depend on others for assistance with decision
making. Most clients in crisis are not guarded (option 4); instead, they give free and direct
expression to their feelings of anxiety. Severe and panic levels of anxiety are common to crisis
states.

1.‐ Uncertain about how to communicate personal needs.


2.‐ Resistant to verbal suggestions about how problems can be approached.
3.‐ Hesitant to depend on others for assistance with problem resolution.
4.‐ Guarded and protective about talking about anxiety and other feelings.

4257 A client seeks help in a crisis clinic after several family Correct answer: 3 The focus of crisis intervention is on the present, not the past. Particular attention is given to Apply the principles of presence and support. Also reflect back on the behaviors you
members were involved in a serious automobile allowing ventilation of current feelings, helping the client with coping mechanisms, and observed in a client who was in a crisis state. Consider commonly used crisis intervention
accident. The client speaks in a loud, disorganized identifying social supports. The focus of the nurse should be on the client, not the family techniques and eliminate options that are not in included among such techniques.
manner with frequent changes of subject. Which (option 1). The nurse should keep the client focused and provide direction to avoid
nursing approach is most likely to be effective? fragmentation of the client’s efforts. Sending the client to a chapel (option 2) is not
appropriate. The client's behavior and the circumstances require that the nurse respond
directly to the client. The client needs to ventilate feelings in order to begin to feel less
anxious. Nonpharmacological strategies should be attempted prior to pharmacological
strategies (option 4).
1.‐ Encourage the client to identify family members involved in the accident.
2.‐ Assist the client locate the chapel or another quiet area.
3.‐ Help the client to identify the problem and possible ways to manage it.
4.‐ Arrange for one‐time anxiolytic medication for the client.

4258 A client whose life partner recently died from Correct answer: 1 The nurse should recognize that ideas of self‐harm are very common in situations of this type. Recognize that this client has just heard news that many people would consider a death
complications of AIDS has received lab results While some clients will not introduce thoughts of self‐harm, they will usually talk about suicidal sentence. In such situations, suicide as a means of remaining in control is not uncommon.
indicating conversion to HIV‐positive status. The thoughts when asked. The nurse should ask if the client has a plan and the means for suicide.
attending physician's office referred the client to the Safety is the priority, and suicidal clients should not be left alone. Altered thought process
crisis unit because the client "shut down" emotionally (option 2), availability of social support (option 3), and ability to afford medication (option 4)
after receiving the lab results. In the initial assessment are important assessment areas after the client’s safety has been ensured.
interview, the nurse's priority is to determine if the
client has:
1.‐ Ideas of self‐harm.
2.‐ Altered thought processes.
3.‐ An available social support network.
4.‐ Financial means to obtain anti‐AIDS medications.

4259 A young adult client frequently engages in high‐risk Correct answer: 2 The nurse should recognize that the all behaviors cited in the stem of the question could Look for an option that recognizes the intensity and dangerousness of the client's
behaviors, including driving at high speed, drinking result in loss of life. They are therefore considered indicators of indirect self destructive behaviors.
excessively, and engaging in high‐risk sexual behaviors. behavior. There is no indication of a loss that would have precipitated grieving (option 1).
It is most important for the nurse assessing this client While it is true that the client is young and is making unwise choices, there is no indication that
to recognize that there is a high probability that: the client's development has been arrested (option 3). Disregard for the life and needs of
others is seen in persons with antisocial personality disorders, but there is no indication in the
stem that the client is disregarding or abusing the life of others (option 4).

1.‐ Unhealthy grieving is occurring.


2.‐ Unconscious thoughts of suicide are present.
3.‐ Arrested maturation is impairing judgment.
4.‐ Antisocial personality traits are causing disregard for life.

4260 The client is hospitalized following an unsuccessful Correct answer: 2 Providing safety and preventing violence on an inpatient unit involves one‐to‐one supervision Notice the client's hesitance and indefiniteness in not contracting. Recognize that as an
suicidal attempt by drug overdose. When offered a no‐ for the client as warranted, based on an assessment of current lethality level. This client did indicator of continuing suicidal risk.
harm contract by the nurse, the client says, "I don't not make a commitment to the no‐harm contract, so the nurse should consider that the risk
think I can agree to that." Which nursing intervention for self‐harm is still present. (The nurse should be aware of the suicide protocol in the agency
is most appropriate? of employment. In some situations, this client might be placed on a different level of suicidal
precautions.) Checks every 15 minutes (option 1), whether during both day and night or only
during waking hours may not be adequate to ensure client safety. The situation does not
suggest an urgent, high‐lethality situation that would call for the nurse to remain at arm's
length from the client at all times (option 3). Constant visual observation only during waking
hours (option 4) may not be adequate to ensure client safety.

1.‐ Visual observation of the client every 15 minutes, during both day and night
2.‐ Constant visual observation of the client, including when in the bathroom
3.‐ Constant visual observation of the client, remaining at arm’s length at all times
4.‐ Constant visual observation at all times during waking hours

4261 Five days ago, a client was admitted to the hospital Correct answer: 3 When a client has been suicidal, it is essential that discharge preparations include a plan for Remember that a danger period for suicide among depressed clients is the point at which
with major depression and suicidal ideations. The safety including social support contacts that can be used after discharge. This is particularly so they begin to improve and have more energy.
client is now preparing for discharge. Which client if the client has acted on suicidal urges, rather than just having had suicidal impulses. Option 1
statement made to the nurse demonstrates that an does not clarify what the client's sleep pattern has been. Additionally, the nurse should
important outcome/evaluation measure has been remember that disturbed sleep could indicate continuing depressed mood. Options 2 and 4
met? "When I go home: suggest that the client is feeling more energetic and optimistic, which are of course signs of
progress. They are, however, not as urgently important as having a post‐discharge safety plan.

1.‐ I'll finally be able to get some sleep."


2.‐ I’ll be able to take care of my plants again."
3.‐ I have a list of people that I can call if I need to."
4.‐ I’ll cook for myself."

4262 When working with a depressed client who has Correct answer: 1 Option 1 is correct. Nurses help reduce the client's feelings of being overwhelmed by helping Note that the client is feeling overwhelmed by personal problems. Isn't it reasonable to
suicidal ideation, the nurse anticipates that the client the client to prioritize concerns and problems. Supporting the client to put off problem‐solving think that if these problems can be managed, the client will no longer feel suicidal?
may be overwhelmed by personal problems. With this is not advisable (option 2). Working on problem‐solving within a group setting is one of many
in mind, the nurse should take which action to best ways to solve problems (option 3). Being directive and setting the priorities for the client
assist the client to cope more effectively? should be avoided (option 4).

1.‐ Encourage the client to make a list of problems from most urgent to least urgent.
2.‐ Support the client's decision to put off problem solving until outpatient therapy has begun.
3.‐ Encourage the client to work on problems only in group therapy.
4.‐ Take a directive approach and advise the client how to prioritize personal problems.

4263 The client has suicidal ideations with a vague plan for Correct answer: 2, 4 Warning signs of suicide generally exist but they may not be recognized until after a suicidal Look for myths and facts about suicide. Recognize the importance of teaching facts before
suicide. When teaching the family how to care for the attempt or suicidal death (option 2). Since almost all suicidal persons are ambivalent about dispelling myths.
person at home, what should the nurse emphasize? dying, they either consciously or unconsciously communicate their intent to others hoping
Select all that apply. (consciously or unconsciously) to be rescued from their own impulses. The nurse should
recognize that the risk for future attempts always increases once a person has made an
unsuccessful attempt (option 4). This means that the nurse should always inquire about past
suicidal behaviors and attempts, including those that occurred in the distant past. Suicide is a
very individual act that does not necessarily reflect negative relationships in the family (option
1). Option 3 reflects a commonly held myth about suicide. Sometimes the person does not talk
about suicide because he or she has made a specific plan and has the means to carry it out.
Not talking about suicide can be a warning sign, and the nurse and family members need to
know this. Family members are not responsible for preventing future suicidal attempts (option
5). They should be encouraged to create safe interpersonal and physical environments, but in
spite of their best efforts, they may not be able to prevent their family member from ultimate
self‐destruction. Telling the family members this will lead to an increased sense of guilt if their
family member successfully suicides at a later time.

1.‐ Suicide occurring within the family environment indicates family dysfunction.
2.‐ Warning signs, even if indirect, generally are present before a suicidal attempt.
3.‐ When the client no longer talks about suicide, the risk of suicide has decreased.
4.‐ Following a failed suicidal attempt, the risk for future attempts is increased.
5.‐ Family members are responsible for preventing future suicidal attempts.

4264 A client presents in the mental health clinic saying, "I Correct answer: 3 Option 3 is correct. A situational crisis is one that is often unexpected and unavoidable and Recall types of crisis responses. Notice that the precipitant event was unexpected and the
didn’t expect it. They just told me this morning that I causes an acute state of emotional disequilibrium. The stressful event threatens a person's event applies only to this client.
don't have a job any more. I can't think straight. I feel physical, emotional, and/or social integrity. The person feels anxious, overwhelmed, and
like I’m going crazy." The nurse documents that the confused. This experience is accompanied by a sense of disorganization and an inability to
client is experiencing which type of crisis? make effective decisions. An adventitious crisis (option 1) occurs following a major
catastrophic event, such as earthquake, hurricane, or war. This type of crisis represents one in
which others experiencing the same or comparable event would react similarly. This question
describes a unique, personal situation and a response that cannot be generalized to a larger
population of individuals. A maturational crisis (option 2) occurs as part of the person’s normal
development and maturation. Such crises are predictable and can be expected to occur as
individuals age and progress through life events, changes, and stages, such as adolescence and
older adulthood. A cultural crisis (option 4) is a response that occurs while a person is adapting
to a new culture or returning to a previous culture after having assimilated into another. There
is no indication that this client is in a culturally challenging situation.

1.‐ Adventitious
2.‐ Maturational
3.‐ Situational
4.‐ Cultural
4265 The client is in a crisis state. At the beginning of the Correct answer: 1, 2 Options 1 and 2 are correct. It is helpful for the client to identify and ventilate personal Remember that crisis intervention focuses on a current problem and the "here and now."
initial assessment interview, what should the nurse feelings being experienced. This relieves anxiety, allows the client to feel validated, and
assist the client to identify? Select all that apply. prepares the nurse and client to progress to other steps in crisis resolution. The client's
perception of the situation should occur very early in crisis intervention. The nurse must have a
clear idea of what the problem represents to the client and also be able to identify the current
reality the crisis presents for the client. Then action plans can be developed. The focus of crisis
intervention is on the individual who is experiencing the crisis response, not on others (option
3). The goal is to assist the person in crisis to reestablish equilibrium by using previously
effective coping techniques. It is premature to develop an action plan at this time (option 4).
Complete assessment and analysis of the problem must occur before proceeding to develop an
action plan. Past emotional traumas (option 5) are not explored in crisis intervention.
Intervention should focus on the current problem and facilitating the client’s coping in order
that a return to pre‐crisis baseline may be accomplished. If past emotional traumas become
apparent during a crisis, referral for counseling at a later time would be appropriate.

1.‐ Current feelings


2.‐ The realistic nature of the event
3.‐ Others who might be affected by the event
4.‐ An immediate action plan
5.‐ Past emotional traumas

4266 A client with suicidal ideation and a specific lethal Correct answer: 3, 4 Options 3 and 4 are correct. The client's statement directly indicates feelings of hopelessness, Look carefully at what the client is saying. The client directly expresses feelings of being
plan for self‐harm was admitted to the hospital. The as well as more indirect expressions of risk for suicide. The spouse’s death has left the client alone and isolated. Additionally, there is an indirect message that could indicate suicidal
client's spouse died recently after a very brief illness. without adequate interpersonal support to cope with and adjust to a significant loss. The client intent. Make sure that the etiologic factor in the nursing diagnosis is shown in the stem of
The client states, "There's no reason to go on living. is experiencing a situational crisis. In order to cope effectively in a crisis situation, individuals the question.
My best friend is gone, and I’m all alone now. We did must identify and be able to rely on others in their world to support them emotionally both
everything together. Now I have no one to turn to or during and after the crisis. While the client is lonely, there is no indication that the client is
do things with." Which nursing diagnoses are helpless (option 1). Additionally, the question does not indicate there was an actual suicide
appropriate? Select all that apply. attempt. Note also that as the diagnosis is written it says that the suicidal attempt caused
helplessness, which is illogical. The client's statement in option 2 does not suggest difficulty
with decision making, although this can be one of the manifestations of a crisis state. The
nursing diagnosis in option 5 does not follow the "problem related to etiologic factor" format.
Notice also that the two parts of the nursing diagnosis are reversed.

1.‐ Helplessness related to suicidal attempt


2.‐ Decisional conflict related to loneliness
3.‐ Risk for suicide related to hopelessness
4.‐ Social isolation related to the loss of support system
5.‐ Acute grief caused from risk for suicide
4267 A client seeks assistance at a crisis center. The client Correct answer: 4 Option 4 is correct. The client is feeling overwhelmed by feelings associated with the crisis Note that this client was exposed to an overwhelming stressor and that symptoms began
describes being intensely anxious and sleepless since precipitant. Before advancing to other interventions, including exploring habitual coping styles after exposure to the stressor. Consider how you would feel if you had been involved in this
assisting with cleanup activities at a school where a and assisting the client with problem solving, the nurse needs to allow the client to freely situation. Like most other crisis responses, the client's response to this intense situation is
student fatally shot a classmate. To assist the client to express emotions being experienced. There is no evidence to suggest that the client would understandable.
cope more effectively, what should be the first benefit from a visit from clergy (option 1). This may be an effective intervention later, but at
intervention of the nurse? this point it is not appropriate. The nurse should not make assumptions about this client’s
religious or spiritual needs. More complete assessment data is needed. Option 2 is incorrect
because the nurse's first intervention should be to help the client cope with the precipitant
and actual event. Emphasis is on perception of presenting event, past experience in coping,
available coping mechanisms, and availability of social supports. The client is in a state of
emotional crisis that is considered a normal response to the event. Unless the crisis response
intensifies and the client develops severe psychologic or physiologic symptoms, the crisis
situation should be treatable in a nonhospital setting (option 3).

1.‐ Arrange for a member of the clergy to visit the client.


2.‐ Advise the client to avoid going near the school for at least 6 weeks.
3.‐ Send the client to the emergency department for further evaluation.
4.‐ Allow ventilation of feelings.

4268 When the nurse is working with a client in crisis, Correct answer: 2 Option 2 is correct. The nurse must remain focused on the immediate problem. Crisis Review steps in crisis intervention. Think also about what makes sense. If the client is
which nursing action is most important? intervention is viewed as a "here‐and‐now" type of therapy. The only history that is relevant at currently feeling overwhelmed, doesn’t it seem reasonable for the nurse to focus on now,
this juncture is the recent history of events that led up to the crisis (option 1). Obtaining a not then?
complete past history at this time would impede the nurse's efforts toward assisting in the
effective resolution of client's crisis state and would not be appropriate. Early life experiences
are not examined in crisis intervention (option 3). The goal is to reestablish equilibrium and
return the individual to the pre‐crisis level of functioning. Examination of early life experiences
occurs in more traditional insight‐oriented psychotherapy. In crisis intervention, the nurse and
the client enter into a relationship where action plans are developed jointly (option 4).
However, it is appropriate for the nurse to be more directive than in other types of
interventions.

1.‐ Obtaining complete assessment of the client's past history


2.‐ Remaining focused on the client's immediate problem
3.‐ Determining the relationship of early life experiences and the crisis state
4.‐ Developing an action plan for the client.

4269 For the third time within a month, a client with Correct answer: 2 Option 2 is correct. The risk of suicide is not reduced because a person makes frequent Notice that the client has made three suicidal attempts within one month's time. Even if
borderline personality disorder took a handful of pills, attempts or threats of suicide. Instead, the risk for successful suicide is greater once a single you didn't know that the client had borderline personality disorder, focus on the probability
called 911, and was admitted to the emergency attempt has been carried out. Persons who make verbal threats or attempts are conveying that that the client must have a certain sense of desperation to aid in making a selection.
department. The nurse overhears an unlicensed staff their desperateness and need for assistance in controlling their own impulses for self‐harm.
member say, "Here she comes again. If she was serious Clients with certain personality disorders, including borderline personality disorder, are
about committing suicide, she’d have done it by now." actually at higher risk for suicide (option 1). These individuals are easily overwhelmed and tend
The nurse determines there is a need to teach the staff to react dramatically to events that others would find more tolerable. The nurse is obligated to
member which of the following? provide protection to the client in a suicidal crisis (option 3). In every suicidal situation, the
nurse must objectively explore the nature, frequency, and specificity of the suicidal thoughts.
Talking about this, and determining whether a plan and a means for suicide are present, is a
standard part of nursing intervention with the suicidal client. Talking about suicide does not
increase the risk for suicide, which is a commonly held myth among nonprofessionals. There is
not enough information to know whether the client will be admitted to an inpatient unit
(option 4). If a sufficient action plan is developed that will provide for the client’s safety
outside the hospital, it is possible that the client can be discharged from the emergency
department.

1.‐ Clients with personality disorders rarely kill themselves.


2.‐ Each suicidal attempt should be taken seriously.
3.‐ Exploration of suicidal ideas and intent should be avoided.
4.‐ The nurse should prepare the client for direct inpatient admission.

4270 A client has been treated in the surgical intensive care Correct answer: 1, 3, 4 Options 1, 3, and 4 are correct. A priority goal for the client, once safety has been assured, is Recall principles of lethality assessment. The SAD Persons scale would be useful.
unit after sustaining a self‐inflicted gunshot wound. to explore life events leading to the decision to die (option 1). This can be followed by
The client is now admitted to a psychiatric unit. The reviewing current feelings and determining whether the client still has active suicidal urges. If
nurse schedules time to meet with the client on a one‐ so, the client must be adequately protected while in the inpatient psychiatric setting. When
to‐one basis with the goals that the client will: (select determining potential lethality in a situation with a suicidal client, it is useful to know whether
all that apply) the person has made previous attempts (option 3). If so, this increases the potential lethality
of the situation. One of the most effective ways of providing for safety of the suicidal client is
to have the client agree (or contract) with the nurse to notify a staff member if the urge to act
on suicidal ideas occurs (option 4). If the client cannot agree to this, or if the client is very
ambivalent or hesitant about agreeing, the nurse should recognize that the risk for self‐injury
remains very high. It is the nurse's responsibility to initiate contact with the client and to
determine whether the risk for suicide is still present (option 2). Do not assume that the client
will be able to initiate interaction with the nurse. In fact, the client may resist such contact,
and this behavior could suggest continuing suicidal potential. The priority should be on
continually assessing client's suicidality and keeping the client safe in the present environment
(option 5). While discharge planning is important, it is premature at this time.

1.‐ Explore current life events that led to the suicide attempt.
2.‐ Initiate contact with the nurse spontaneously.
3.‐ Discuss past suicidal ideations and behavior.
4.‐ Enter into a contract for safety with the nurse.
5.‐ Identify post‐discharge living arrangements.

4271 A client who admits to having frequent suicidal Correct answer: 3 The nurse should look beyond the words that the client uses and determine what the Notice the words at peace. Does it seem likely that someone so recently admitted to a
ideations is admitted to the psychiatric inpatient unit. underlying meaning is (option 3). Improvement in mood and energy often occurs just before psychiatric‐mental health unit would feel peaceful? Recognize this as a "red flag"
During the assessment interview, the client says, "I the suicidal person carries out a suicidal act. Once a suicidal plan has been made, it is as if the statement.
really don't need to be here. I'm very much at peace individual feels relieved of a great burden. While the client's words may suggest that the risk of
with myself now." The nurse should interpret that the suicide is lessened, the hidden or indirectly expressed message is different (option 1). The
client probably: nurse should recognize that this client may be describing a feeling of relief often experienced
by a suicidal client after making a plan to end his or her life. The client may need to be put on
heightened suicide precautions (option 2). The nurse should realize the client’s verbalizations
indicate an actual plan to end life may have been made and could be carried out after
discharge. Improvement in affect and energy is often observed in suicidal clients right after a
suicide plan is made and right before the suicidal client carries it out (option 4).

1.‐ Has resolved suicidal feelings and is no longer at risk for self‐harm.
2.‐ Is ready to be discharged from the inpatient setting.
3.‐ Continues to be at significant risk for suicide.
4.‐ Has concluded that the risk for self‐harm is no longer present.

4272 The nurse determines that which nursing diagnosis Correct answer: 1 Option 1 is correct. The first priority in caring for the client with suicidal ideation and intent is Don't try to make this question harder than it is. You already know that preservation of life
would be the priority for a client with suicidal ideations maintaining safety. Options 2 (ineffective individual coping), 3 (hopelessness), and 4 (defensive is always a priority for the nurse. The fact that this is a psychiatric‐mental health client
and intent? coping) are incorrect. All other issues, including major psychological ones, are secondary to doesn't change that.
safety. Ineffective individual coping, hopelessness, and defensive coping would be appropriate
nursing diagnoses only after safety has been assured.

1.‐ Risk for violence, self‐directed


2.‐ Ineffective coping
3.‐ Hopelessness
4.‐ Defensive coping

4273 A suicidal client is placed on one‐to‐one observation. Correct answer: 1 Acknowledging the client's feelings of frustration and reaffirming the need for safety is the Keep in mind the need to be compassionate and calm while providing for the client's
When the nurse accompanies the client to the priority. The nurse should remain calm and matter‐of‐fact in this situation while tolerating the safety. Recognize that what the client is feeling and saying is common and very
bathroom, the client loudly shouts, "I'm sick of being client's verbal outburst and allowing for expression of feelings. The nurse's response should understandable in situations involving one‐to‐one care.
followed around and treated like a child who can’t be include validation of client's feelings and information about the intent of the one‐to‐one
trusted." What would be the best response by the observation, which is to keep the client safe (option 2). During one‐to‐one observation, the
nurse? nurse must remain at arm's length from the client at all times, including during toileting
activities (option 3). The response in option 4 lacks compassion and is demeaning. While it
suggests some awareness of safety considerations and procedure, it fails to express concern
for this client's safety.

1.‐ "I understand that you do not like this, but I must be able to see you at all times to make sure you are safe."
2.‐ "You don’t have to be so loud. I do trust you, but I can't change the rules for you."
3.‐ "Since this is upsetting to you, leave the door open and I’ll wait outside it for you."
4.‐ "Being angry and uncooperative won't change anything. I can’t leave a suicidal client alone."

4274 A client is transitioning to a less intensive level of Correct answer: 4 The problem with thoughts of using is keeping them a secret. When keeping things secret, the Use the process of elimination and nursing knowledge to answer the question. The
outpatient treatment for addiction. The client client is not telling the whole truth and is manipulating something. Engaging in secrets is wording of the question tells you that one answer is better than the others because it
statement that most reflects risk for relapse includes: reminiscent of using behaviors and can trigger using behaviors. It is natural to feel sad (option contains the key word best.
2), hungry, or tired (option 3), and to have thoughts of using (option 1).

1.‐ Dreaming about gambling or engaging in compulsive sex.


2.‐ Not feeling happy.
3.‐ Feeling hungry or tired.
4.‐ Keeping thoughts of using a secret.

4275 What statement made by the mother of a recovering Correct answer: 1 Checking on the compliance of a family member is an example of codependent behavior. The The wording of the question tells you that only one answer is correct. Use knowledge of
compulsive Internet user would indicate the need for nurse would focus the teaching on helping the mother detach from her son and his recovery codependency to differentiate the problematic behavior from the other expected
more teaching? program and focus on her own well‐being. Options 3 and 4 would indicate that she is trying to behaviors.
identify and deal with her feelings. Option 2 is an obvious healthy behavior.

1.‐ "My son is not going to enough 12‐step meetings, he doesn't do his daily readings, and I don't think he is taking this seriously enough."
2.‐ "My daughter and I are going to go to Al‐Anon for the first time because we realize we have been affected by my son's addiction."
3.‐ "I need to sign up for a meditation class for me because I get too preoccupied with what my son is or is not doing."
4.‐ "I still have a lot of anger about the relationship problems that occurred between my son and me as a result of his addiction."

4276 The nurse observes a family visit on the unit and Correct answer: 1 There are three communication rules learned in families in which addiction is present: don’t The core issue of the question is underlying consequences to families when addiction is
recognizes that the family is suffering with effects of talk, don’t trust, don't feel. While these experiences cause anger, anxiety, or maladaptive present. Use nursing knowledge and the process of elimination to make a selection.
addiction and codependence. What long‐lasting coping, they can also contribute to the development of shame, depression, and low self‐
interpersonal problems might the nurse expect family esteem. Without family healing, these problems can create much pain and suffering for all
members to manifest? involved.
1.‐ Lowered self‐esteem
2.‐ Impatience
3.‐ Frustration tolerance
4.‐ Being argumentative
4277 A mother brings her daughter into the Emergency Correct answer: 3 The client most likely has used one of the "club drugs" or "rave drugs," these substances most Note the stem of the question contains the critical words most concerned. This tells you
Department. She was at a party and danced for the last often are a cross between a stimulant and a hallucinogen. Such drugs are used at dance parties that more than one option may be partially correct and that you must prioritize an answer.
few hours. Now she is sweating and does not look well. and along with black lighting or strobe lights create a surreal experience. The stimulant effect Correlate elevated body temperature and weight loss with fluid balance to choose option 3
The nursing assessment reveals temperature of 103° F, of the drug causes users to grind their teeth. To avoid this, teens often use pacifiers to suck on. as correct.
grinding the teeth, rapid weight loss. What should the The combination of drug, dancing, and dehydration lead to dangerous body temperature
nurse be most concerned about? increases, which must be addressed immediately. This client may also have an eating disorder,
but that would not be the nurse's primary concern (option 2). Options 1 and 4 are incorrect.

1.‐ Poor nutrition from excessive alcohol consumption


2.‐ Possible eating disorder
3.‐ Dehydration and electrolyte imbalance
4.‐ Flu with accompanying high fever

4278 The nurse is completing an admission for a client with Correct answer: 4 While most individuals believe that drugs of abuse enhance their sexual experience, the The core issue of the question is the relationship between chronic substance abuse and
alcohol dependence. During the admission process, the opposite is mostly true. The four types of sexual problems that commonly occur as the result sexual performance. Use the process of elimination and nursing knowledge to answer the
client acknowledges occasional sexual performance of chemical use are: anxiety about one's sexual performance; decrease or absence of sexual question. The wording of the question tells you that only one option is correct.
problems. Then he says, "It’s nothing a little alcohol arousal; difficulties in reaching orgasm; and decrease or absence of pleasure in and/or
can't fix." The nurse provides education about the intensity of orgasm.
effect of alcohol on sexual functioning by sharing that
regular alcohol use causes which of the following?

1.‐ Increased desire and performance ability


2.‐ Headaches and the "too tired syndrome"
3.‐ Hyperarousal and premature ejaculation for men and anorgasmia for women
4.‐ Decreased desire and ability to perform

4279 A physician just wrote an order for a client to take Correct answer: 3 Naltrexone is an excellent medication to treat alcohol or opiate dependence. It helps to Note the key words greatest concern in the question. This tells you that more than one
naltrexone (ReVia). What would be the greatest prevent cravings and triggers to use, and it blocks the euphoric response if alcohol or opioids option could be partially correct and that you must prioritize an answer. Use nursing
concern of the nurse while getting ready to administer are ingested (option 1). The nurse should always evaluate the client’s current knowledge level knowledge related to this medication to choose the option in which the client is at greatest
this medication? and provide education as needed (option 2). However, if the client is not completely detoxified risk.
from opiates, the use of naltrexone can precipitate withdrawal (option 3). Persons should be
opiate‐free for 7–10 days before starting this medication.

1.‐ The medication blocks the euphoric feeling from narcotics and alcohol.
2.‐ Whether the physician provided good medication teaching.
3.‐ The medication can precipitate withdrawal if the client is not completely detoxified.
4.‐ The client will not be able to experience pleasurable sensations.

4280 You are conducting a daily nursing assessment on a Correct answer: 1 Recovering clients may tend to underestimate how difficult it will be to stay sober if they visit The wording of the question tells you that the correct option is a statement that contains
client with gambling and alcohol addictions who is in with friends who are still using or frequent old "hangout" places where they used to engage in either a false statement or one that indicates the client is at risk. Choose option 1 over the
the outpatient addiction program. As she checks in addictive behaviors. In early recovery, clients are encouraged to detach from people, places, others because it puts the client in an area where temptation is likely.
with you, she makes which of the following statements and things associated with their addiction. As the person gains sobriety and recovery, he or she
that reflects a need for more teaching? may be able to re‐engage, on a limited basis, with certain activities, such as being with friends
who drink or celebrating an occasion at a bar. Options 2, 3, and 4 demonstrate positive coping
measures and good management of potential triggers.

1.‐ "I am going to have a night out with some friends at an area night club."
2.‐ "I felt like drinking, so I cleaned the house instead."
3.‐ "It is hard for me to make phone calls if I feel like using, but I did it last night."
4.‐ "I told my brother that I couldn't help him as much as I have in the past."
4281 After completing a family session about addiction, a Correct answer: 2 Addiction affects the entire family system: communication roles and boundaries. Some The core issue of the question is the family dynamics and impact on the family of a
woman approaches the nurse and shares that as a problems that individual family members experience are low self‐esteem, guilt, shame, substance‐using family member. Use nursing knowledge and the process of elimination to
mother, she will always have to bear the suffering of insecurity, and preoccupation with the chemically dependent family member. Families need make a selection. The wording of the question indicates that only one answer is correct.
having a chemically dependent daughter who could treatment to facilitate their own healing. If they get involved in a treatment facility–operated
relapse at any time. What would be important family program, a spiritually centered family recovery program, or any of the family 12‐step
information to share about family recovery from programs, active healing can take place whether the addict is using or not. Options 1 and 3 are
addiction? incorrect. If the family is not engaged in its own treatment, it may not make any difference
how many meetings the addicted member attends (option 4).

1.‐ Family recovery can begin when the addictive behavior ceases.
2.‐ Family recovery can begin even if active use continues.
3.‐ Family recovery will fail if the recovering addict relapses.
4.‐ Family recovery will be enhanced if the recovering addict attends several Alcoholics Anonymous meetings.

4282 A married client with marijuana dependence has Correct answer: 3 The client is spending a great deal of time on the Internet, which seems to be interfering with Use the process of elimination and critical thinking skills to answer the question. The
difficulty keeping her house clean because she spends not only her parental relationships but also her relationship with her husband as well. If the correct answer is one that is most comprehensive of all aspects of the problem described
a lot of time playing an entertaining game on the client does not stop using marijuana and start practicing recovery, and if her Internet problems and does not place judgment on the client.
Internet. She also says that she waits until everyone are not addressed, they are unlikely to "go away on their own" and her family problems may
goes to bed to start writing messages with sexual get worse (options 1, 2, and 4).
content online with another man. What are the nurse's
concerns?
1.‐ The Internet will cause her to break her marriage vows.
2.‐ Her children won't bring any friends home because the house is messy.
3.‐ She seems preoccupied with the Internet and is using poor judgment.
4.‐ She is depressed and finds her marriage unfulfilling.

4283 The nurse is educating parents about the purpose of Correct answer: 2 Environment and peer pressure play very strong roles in the development of addiction. Most Use the process of elimination to answer the question. The correct answer is the one that
laws that prohibit nicotine advertising on billboards smokers (90 percent) are addicted to nicotine by age 20. Although only 28 percent of the U.S. reduces the exposure of children and teens to advertising about tobacco in locations that
within 1,000 feet of children in academic and social population smokes, the vast majority of new smokers are under age 18. The Federal Drug they tend to frequent, thereby diminishing the pressure to use.
areas. In response to a question from the group, she Administration (FDA) is trying to reduce smoking among children and teens by regulating
shares that the law was designed to do which of the tobacco advertisements near schools and youth centers. The rationale is that by restricting
following? tobacco advertising to youths, the desire to smoke will be reduced. It is good for the public to
be educated about the hazards of smoking, including second‐hand smoke. This law does not
specifically address option 1 and 3. Option 4 is part of the tobacco industry response to the
proposed FDA regulation.

1.‐ Educate the general public about the hazards of smoking cigarettes.
2.‐ Diminish the environmental risk to teens.
3.‐ Diminish the effects of second‐hand smoke.
4.‐ Limit the free speech of children.

4284 A new mother who bottle‐feeds her infant comes in Correct answer: 3 Antidepressants regulate dysfunction in the neurotransmitter system, which results in mood The core issue of the question is the interaction of a prescribed antidepressant with
for her 6‐week postpartum visit and talks about how equilibrium. Alcohol is a depressant that causes dysfunction in the neurotransmitter system, alcohol use. Recall that alcohol is a CNS depressant, which has an opposite effect of
depressed she is feeling. The health care provider which can cause depression and/or anxiety. Use of alcohol or other mood‐altering drugs while antidepressants. Use the process of elimination and general knowledge of drug interactions
prescribes an antidepressant for her. As the nurse taking antidepressants is contraindicated. to make a selection.
delivering medication education, you assess the
client's alcohol‐use patterns, and she shares with you
that she has 1–2 drinks once or twice a week. You
inform the client that she should not drink alcohol or
use drugs in this situation because:

1.‐ It will cause nausea and vomiting.


2.‐ It will increase the effectiveness of the antidepressant.
3.‐ It will decrease the effectiveness of the antidepressant.
4.‐ It will cause increased blood pressure.

4285 A cocaine‐dependent client in recovery shares with Correct answer: 1 Persons can experience tolerance or tolerance‐like symptoms in response to taking certain Use the process of elimination and nursing knowledge to answer the question. Eliminate
the nurse that she has been using an OTC medication OTC medications. OTC sleep medications or psychoactive sleep medications are meant for incorrect options because of the presence of "red flag" words cannot in options 2 and 4 and
to help her get to sleep each night for the past 3 short‐term use, no longer than 1 week consecutively (options 2 and 3). The FDA does not firmly and persistently in option 3.
weeks. She gets defensive when the nurse raises regulate herbal products, and it is difficult to know what dose to recommend or how the
concerns, stating emphatically that it is not addictive. product might interact with the client. Sleep difficulties are often a problem for people in early
How does the nurse respond to the client? recovery. Providing education on sleep hygiene and validating experiences proves helpful in
addressing this problem (option 4).
1.‐ Validate how difficult it is to have a tough time sleeping and explain that nonaddictive medication can be abused if taken in larger doses or more frequency than
recommended.
2.‐ Acknowledge to the client that because the medication cannot be abused, it is not addictive. State there is a general concern that it could become a problem.
3.‐ Confront her firmly and persistently because she is increasing the risk for developing addiction if she uses this medication too frequently.
4.‐ Suggest she take some natural herbal sleep medication such as valerian or melatonin because they cannot cause any activation of the brain reward system.

4286 As the nurse asks about sexuality during a nursing Correct answer: 4 The client acknowledges her problem and has tried to stop on her own; this puts her in the The critical words in the question are stage of change. This tells you that the correct
assessment, the client acknowledges that she has action stage. She is actively trying to change. The correct action of the nurse, then, is to assist. answer is the one in which the action of the nurse matches the stage of change represented
sexual problems. She shares that she has a Option 1 demonstrates precontemplation; option 2, contemplation; and option 3, by the client statements. Use the process of elimination and nursing knowledge to make a
masturbation compulsion and is trying to stop on her determination and preparation. selection.
own but can't. She says she needs to know what she
can do to stop. Based on her stage of change, the
nurse should use which approach to care?

1.‐ Help her to see that she has a serious problem.


2.‐ Encourage her that she will feel better if she stops the compulsive sexual behavior.
3.‐ Identify a date for her to stop her compulsive sexual behavior.
4.‐ Review strategies to assist her to stop the compulsive sexual behavior.

4287 The mental health nurse reminds clients who are Correct answer: 3 Alcohol and benzodiazepines are both depressants. Persons often use two drugs within the The core issue of the question is knowledge that cross‐addiction occurs between drugs in
learning about cross‐addiction that there is a same class to enhance their effects. The capacity of other psychoactive substances within the the same class. Use the process of elimination and knowledge of the categories of the
synergistic or addictive effect from using various kinds same class of drugs to enhance the effect of the primary drug is called cross‐tolerance. Options chemicals in the options to make a selection.
of chemicals together. The nurse uses which of the 1 and 4 are examples of combining a stimulant with a depressant, while option 2 has only a
following as examples of combinations of chemicals stimulant.
that create this additive effect?

1.‐ Drinking beer and smoking cigarettes


2.‐ Drinking coffee and eating donuts
3.‐ Drinking wine and taking a benzodiazepine
4.‐ Drinking wine and coffee

4288 A male client is saying he is "wired," feels like he is on Correct answer: 3 Tactile disturbances are a symptom of alcohol dependence, and if the client reports stopping The wording indicates that the core issue of the question is possible withdrawal. Use the
"pins and needles," and is irritable. He says he stopped alcohol use abruptly, he or she may be starting to experience withdrawal symptoms. However, process of elimination for options 1 and 4. Choose option 3 over option 2 because it is more
using alcohol abruptly. What is the nurse's next the client may also have used and stopped other substances abruptly as well. The nurse must comprehensive and includes option 2 within it.
intervention in caring for this client? assess for other substances used. Multiple drug use is the rule more than the exception.
Waiting (option 1) places the client at risk, and stimulants are not indicated (option 4).

1.‐ Wait to see if any other symptoms occur in the next few hours and then report them to the physician.
2.‐ Assess the time of his last drink and begin assessing signs and symptoms of alcohol withdrawal.
3.‐ Assess the client for all current substance‐use patterns, including time of last usage, and begin to assess for withdrawal.
4.‐ Ask the physician to write an order for a stimulant medication to help prevent delirium tremens.
4289 The nurse has coordinated a health fair for the church Correct answer: 3 It takes the average person 1 hour to metabolize 1 ounce of alcohol or a 4‐ounce glass of Use the process of elimination to answer the question. Eliminate options 1 and 2 first
parish with four other nurses. The first nurse speaks wine. If three to five glasses of wine are consumed within an hour, the average person because they contain small amounts of alcohol and because they tend to be more rigid
about safe driving and includes that coordination and becomes intoxicated. Options 1 and 2 are insufficient, while option 4 is greatly excessive. amounts. Recognizing that the question has the key words average person, choose option 3
mental alertness are affected at a blood alcohol level over option 4 because it is more moderate and because it allows for individual variation,
of 0.04, even though many states have a legal limit of which options 1 and 2 do not.
intoxication of 0.08 (formerly 0.10). When asked how
many drinks per hour the average person needs to
reach 0.08 intoxication level, the nurse would make
which of the following replies?

1.‐ A 4‐ounce glass of wine if the individual has eaten recently


2.‐ A 1–4‐ounce glass of wine on an empty stomach
3.‐ Three to five 4‐ounce glasses of wine, depending on how recently food was consumed
4.‐ Seven to eight 4‐ounce glasses of wine, depending on how recently food was consumed

4290 A nurse working in the addictions unit is stopped in Correct answer: 2 Impaired control is the defining symptom that moves someone’s use or abuse category to the The core issue of the question is knowledge of hallmarks of alcohol abuse. Use the process
the cafeteria by a coworker who states she is upset dependence category. The symptom of "use despite negative consequences" fits in both the of elimination and nursing knowledge to make a selection.
about something the nurse told to a client. The client abuse and dependence category (option 1). Withdrawal (option 3) and/or tolerance (option 4)
understands that the nurse said, "If your drinking has may or may not be present for someone who has dependence.
created any problems for you, then you have
addiction." The nurse clarifies that the statement was,
"If you have the hallmark symptom of drinking, you
have addiction." The nurse goes on to share which of
the following as the hallmark?

1.‐ Use despite negative consequences


2.‐ Impaired control of use
3.‐ Withdrawal
4.‐ Tolerance

4291 An orthopedic client who broke his ankle while Correct answer: 4 The behavior of drinking and driving fits in the abuse category as "recurrent substance use in Use the process of elimination to make a selection, matching the client statements in the
drinking at a party is wondering if his drinking is hazardous situations." Option 1 is incorrect. Option 2 demonstrates the category of question with the conclusions in the correct option. The wording of the question tells you
"okay." He says he has never been arrested for driving dependence. Option 3, "black out," is a symptom of intoxication. that only one option contains a correct statement.
intoxicated, nor has he experienced any health or
relationship problems. He called in sick to work one
time and drove intoxicated several times. Family
information validates his self‐report. The nurse
concludes the client has an alcohol abuse problem on
the basis of which of the following characteristics of
this syndrome?
1.‐ Drinking more than two drinks per occasion
2.‐ The inability to stop drinking despite negative consequences
3.‐ Drinking that causes an individual to pass out or experience a blackout
4.‐ Drinking too much and too often with using poor judgment, and having negative consequences

4292 The school nurse at the local high school is teaching a Correct answer: 2 Any substance, legal or illegal, that activates the pleasure center in the brain has the potential Use the process of elimination and basic knowledge of addiction to make a selection. The
drug prevention class to teens, who don't believe to cause dependence. Nicotine takes only 10 seconds to reach the brain. Nicotine causes both wording of the question tells you that only one option contains a correct statement.
cigarettes should be labeled a drug. The nurse explains physical and psychological dependence.
that when dealing with addiction, the word "drug"
means which of the following?

1.‐ An illegal substance that activates the pleasure center


2.‐ A substance that activates the pleasure center in the brain
3.‐ A chemical that produces a pharmacological action when ingested
4.‐ Any kind of pill that is broken down in the stomach by digestive action

4293 A female student nurse visiting an outpatient Correct answer: 3 Methadone maintenance therapy seems to be an effective treatment regimen for a select Use the process of elimination and basic knowledge of addiction to make a selection. The
addiction program is reviewing nursing care plans. She population. Clients with heroin and/or other opiate addictions receiving oral methadone do wording of the question tells you that only one option contains a correct statement.
notices that there is one client with opiate addiction not receive the euphoria associated with their drug of choice (option 1). The person on
who takes methadone and will not be tapering off the methadone maintenance who works a recovery program and is abstinent of all other mood‐
medication. The student nurse asks the preceptor‐for‐ altering substances is in good recovery (option 2 and 4).
the‐day about this. The preceptor responds that clients
with an opioid addiction who are on methadone
maintenance:

1.‐ Are at high risk for using opiates.


2.‐ Are not really in recovery because they are still using a drug.
3.‐ Do fairly well in recovery as long as they are not using other drugs.
4.‐ Are exempt from having to participate in a 12‐step program.

4294 The nurse is conducting an education session about Correct answer: 1, 2, 4 Options 1, 2, and 4 are correct. DSM IV‐TR specifies that substance dependency can be Review DSM IV‐TR criteria for substance dependency.
alcoholism. Which of the following statements should diagnosed if the behaviors of the client over the past 12‐month period are consistent with
the nurse include when explaining the concept of three or more of seven specific criteria. This option describes one of the criteria. Option 3 is
alcohol dependences? "Alcohol dependence involves incorrect as this option is not included among the DSM IV‐TR criteria. These criteria recognize
(select all that apply): that alcohol dependency can follow many different patterns, including episodic drinking to
excess. The only time frame mentioned in the criteria is one year, and this is because
persistent patterns of use are necessary to establish the diagnosis of alcohol dependency.
Option 5 is incorrect because this option is not included among the DSM IV‐TR criteria. Also,
confabulation is not considered one of the characteristic defense mechanisms of the alcohol
dependent individual. These are denial, rationalization and projection.

1.‐ Continuing to drink despite critical alcohol related problems."


2.‐ Drinking larger amounts or over a longer time than was intended."
3.‐ Drinking to the point of drunkenness at least once per week."
4.‐ Experiencing a diminished effect with continued use of the same amount of alcohol."
5.‐ Using confabulation as a defense mechanism."

4295 A client is dually diagnosed with alcohol dependency Correct answer: 3 Option 3 is correct. For the client who is dually diagnosed with alcohol dependency and Remember that persons with psychiatric disorders are at risk for substance dependency if
and depression. The client says, "I think that once my another psychiatric‐mental health problem, he or she must receive treatment for both both co‐morbidities are not treated. Individuals often will use alcohol and drugs for
depression is managed, I won’t drink anymore." What illnesses in order to have the optimal chance of recovering. Treatment of only one of the purposes of self medicating their untreated psychiatric illnesses.
is the most appropriate response by the nurse? disorder is incomplete and can lead to relapse. Options 1, 2, and 4 are incorrect because
"Experts in the field generally agree that: treatment of only one of the disorders is incomplete and can lead to relapse.

1.‐ Decisions regarding sequencing of treatments should be made on an individual basis."


2.‐ Treatment for depression should occur before treatment for alcohol dependency."
3.‐ Treatment for both alcoholism and depression should occur at the same time."
4.‐ Alcohol related treatment should occur before treatment for depression."
4296 The substance dependent client is in the rehabilitative Correct answer: 1, 2, 4 Options 1, 2, and 4 are correct. The substance dependent client must recognize that relapse is Recall information about usual behaviors and defense mechanisms associated with
stage of treatment. When teaching the client relapse common among substance abusers. Clients need to be taught how to prevent relapse so that substance use. Recognize the importance of giving the client specific guidance that will
prevention skills, what should the nurse emphasize? they can gain confidence and the expectation of being able to cope without using a substance. assist with learning more effective coping skills.
Select all that apply. It can be useful for the client to learn to apply the HALT mnemonic, which teaches avoidance
of situations that are known to promote relapse. These include being: Hungry, Angry, Lonely,
and Tired. Other relapse prevention skills include involvement in an active recovery program,
often a 12 step program that considers recovery to be a life‐long process that is best
accomplished with the support of peers with the same addiction. Option 3 is incorrect as
treatment programs emphasize the need to be open and honest about having urges to return
to use of the substance. Suppressing feelings is considered to be an addictive behavior. Option
5 is incorrect because the recent social network of the client is most likely to be that of a group
of substance users. Relapse prevention skills include developing a new social network of
persons who will support efforts toward sobriety.

1.‐ Preventing fatigue


2.‐ Maintaining physical health
3.‐ Suppressing thoughts of returning to substance use
4.‐ Reducing amount of solitary unstructured time
5.‐ Reconnected with the recent social network

4297 Which of the following comments by a substance Correct answer: 1, 3, 4, Options 1, 3, 4, and 5 are correct. Substance dependence clients should be taught to have Look for options that clearly indicate that the client is in a situation of risk. Notice that
dependent client should cause the nurse to conclude 5 structure and routine in their lives, as well as to avoid boredom and loneliness. Option 3 only one option shows insight. However, that is not what is being asked for in this question.
that the client is vulnerable for relapse? Select all that suggests that the client feels overly confident, which can be lead to unwise behaviors that test
apply. the recovery. Option 4 indicates dissatisfaction and impatience with others and can lead the
client to feel justified in returning to the "solace" of using a substance. Option 5 indicates that
the client is putting self into a situation of high risk to return to alcohol use. Also, the client is
showing complacency rather than cautiousness. Option 2 is incorrect as this statement
indicates that the client has an awareness of a high‐risk situation that could lead to relapse.
After developing the awareness, the client can then make a conscious decision as to how to
cope with the situation without using a substance.

1.‐ "I like being able to have a lot of free time."


2.‐ "Going to a football game makes me want to use again."
3.‐ "I've been sober for 2 years. I've got this problem under control."
4.‐ "No one else seems to work as hard as I do."
5.‐ "It's easy. I can go to a club and just drink soft drinks."

4298 Clients newly diagnosed with an addictive disorder Correct answer: 4 Option 4 is correct. Most substance use treatment programs encourage the client to Notice that this question is specifically dealing with issues surrounding independence.
often have difficulty dealing with the threat of loss of acknowledge that they are unable to maintain sobriety of their own accord. Accepting the Disregard all other issues when looking at the options.
independence. Which type of statement made to the need for help and asking for it are fundamental to the recovery process. Most 12‐step
nurse would indicate that the client is successfully programs teach the concepts of powerlessness, surrender acceptance, and asking for help as
working on these issues? The client is: primary to recovery from the disease. Options 1, 2, and 3 are incorrect as dealing with
existential issues such as "who am I," feelings of loss, and anger are important parts of the
recovery process, but this question is specifically asking for a response regarding independence
issues.
1.‐ Asking, "What is the meaning of life?"
2.‐ Dealing with feelings of loss.
3.‐ Expressing anger.
4.‐ Asking for help.
4299 The client has a compulsive gambling addiction. The Correct answer: 2 Option 2 is correct. In the contemplation stage of change the client is becoming aware of a Review definitions and basic information about stages of change.
nurse has assessed the client as being in the problem but has not yet become motivated to change. Option 1 is incorrect as this reflects the
contemplation stage of change. Based on this preparation‐determination stage of change in which the client is getting ready for a change
assessment, the nurse anticipates that the client is and developing a plan to seek help. Option 3 is incorrect because stopping the gambling
dealing with which issue? behavior is the action stage of change. Option 4 is incorrect as continuing the behavior of the
action stage of change occurs in the maintenance stage of change, which, optimally, is a
lifelong pattern.
1.‐ Completing the evaluation of the gambling problem and making plans to go to treatment
2.‐ Discussing the good and bad aspects regarding gambling
3.‐ Stopping gambling and beginning to take a longer route home to avoid the casino
4.‐ Taking action to assure staying away from gambling permanently.

4300 A 30‐year‐old female client is admitted to a program Correct answer: 4 Option 4 is correct. When clients are of child‐bearing age, it is essential that evaluation for Notice the age and sex of the client. Most questions do not provide this information, so
for clients diagnosed with alcohol dependency and pregnancy occurs before active psychopharmacologic interventions are begun. This is so that the fact that this appears in this question makes it very important.
depression. Before administering any ordered substances known to be dangerous to unborn children (which includes many antidepressants)
medications to this client, it is most important for the will not be given to the client. If the client is pregnant, prenatal exposure to alcohol has already
nurse to verify that the client's record includes which occurred and the client is already at risk for delivering a child with one or more alcohol‐related
of the following? birth defects, including fetal alcohol syndrome. Of all substances commonly abused or abused,
alcohol carries the greatest risk for unborn babies. Option 1 is incorrect because while it is true
that vitamin administration is an important part of treating both pregnant women and alcohol
dependent persons, this is not as important a priority as is protecting a developing fetus from
further exposure to dangerous chemicals. Option 2 is incorrect as this information is useful to
assess the client's overall health, but this test only assumes urgent importance if the client has
significant health problems or is to begin a drug that is known to cause hematologic changes.
Option 3 is incorrect because recording of height and weight does provide baseline data for
further treatment of the pregnant and/or alcohol dependent client. However, many
medications can be safely begun without this data.

1.‐ Medical orders for vitamins


2.‐ Complete blood count (CBC) findings
3.‐ Recording of height and weight
4.‐ Pregnancy test results

4301 A substance dependent client is attempting to Correct answer: 4 Option 4 is correct When the client is addicted, common elements in cognitive restructuring Look for an option that indicates a change in the client's usual defensive and cognitive
maintain sobriety. The nurse is teaching the client the include making a commitment to choosing sobriety; not engaging in distorted thinking like pattern.
technique of cognitive restructuring. What statement blaming others and increasing the sense of perceived control. Option 1 is incorrect as this
by the client indicates to the nurse that the client is response indicates that the client is projecting blame and rationalizing the drinking behavior by
making progress? blaming outside circumstances. When the client is addicted, common elements in cognitive
restructuring include making a commitment to choosing sobriety; not engaging in distorted
thinking like blaming others and increasing the sense of perceived control. Option 2 is incorrect
because this statement indicates that the client is continuing to blame others. Additionally, it
exhibits self pity and high expectations of the behavior of others, both of which indicate
relapse vulnerability. The client who makes this statement is maintaining, not changing,
personal cognition. Option 3 is incorrect as this statement indicates that the client is feeling
overly‐confident and grandiose, both of which indicate relapse vulnerability. The client who
makes this statement is maintaining, not changing, personal cognition.

1.‐ "If I didn't have so many problems in my life, I wouldn't drink."


2.‐ "My family hasn't yet realized that I'm doing this for them."
3.‐ "I know how to work this program. I'm too smart to fail again."
4.‐ "Addiction has brought a lot of hard lessons and
many insights into my house. While it's not easy to
abstain from drinking, the 12 steps have made me
wiser and they help me make better choices. If I'm
going to depend on anything again, it's going to a
higher power."

4302 The nurse determines that which nursing diagnosis Correct answer: 4 Option 4 is correct. The definition for spiritual distress is disruption in the life principle that Recall that spirituality and religion are separate phenomena. Apply the broader concept of
would be appropriate for a client who is in the anger pervades a person’s entire being and that integrates and transcends one’s biological and spirituality to this question.
stage of grieving the loss of his or her addiction? psychosocial nature. The lifestyle change that is necessary to recover pervades a person's
entire being and it helps explains why someone would be angry about having to make such a
great change. Option 1 is incorrect as Ineffective denial is a disavowing of the meaning of
something. This diagnosis is not reflected in the client’s current status. Option 2 is incorrect
because Ineffective management of therapeutic regime suggests that one is not managing the
recovery process well. This diagnosis is not reflected in the client's current status. Option 3 is
incorrect as Knowledge deficit suggests a lack of knowledge about the disease. This diagnosis is
not reflected in the client's current status.

1.‐ Ineffective denial


2.‐ Deficient knowledge
3.‐ Ineffective therapeutic regime management
4.‐ Spiritual distress

4303 Recovering substance‐dependent clients are Correct answer: 3 Option 3 is correct. If clients call someone when feeling depressed they are demonstrating Notice that the question is asking for a specific indicator regarding use of a sober social
completing a short question‐answer test after a coping they have learned how to use support. Reaching out to others for sobriety support and network.
skills lecture the nurse delivered. What statement interpersonal support is key to maintaining recovery from depression and addiction. Option 1
indicates that a client is learning how to use a sober is incorrect as in recovery programs that utilize a sponsor, the sponsor's role is to provide
support network? social and personal support, not financial support. Option 2 is incorrect because this statement
indicates that the client has not accepted the importance of social support in a recovery
program. Individual who try to "go it alone" are at high risk for relapse. Option 4 is incorrect as
this statement indicates an awareness of areas of commonality between the client and others,
but it does not indicate that the client knows how to utilize the social network.

1.‐ "My sponsor will loan me money if I need it."


2.‐ "I feel grateful that I have been able to stop using on my own."
3.‐ "If I start feeling depressed, I should let someone know."
4.‐ "I’ve learned that I'm not different from all the others in treatment."

4304 The nurse instructs the client about addiction. The Correct answer: 2, 3, 4 Alcoholism was officially listed as a disease in 1956, and Jellinek’s identification of the four Remember that treatment approaches for alcoholism include both biomedical and
nurse determines that the client understands the phases of disease progression in 1960 reinforced the disease concept (option 2). Addiction biosocial models.
information given when the client makes which includes behavioral habits and emotional attachment, but it is seen first as a medical disease
statement(s)? Select all that apply. (options 3 and 4). Although alcoholism has been recognized as a disease for approximately 50
years, many members of the general public continue to view addiction as a moral weakness
(option 1). Addiction experts do not consider that addiction can be cured (option 5). Instead,
they consider it a chronic medical disease that can be managed.

1.‐ "Addiction is a moral problem."


2.‐ "Addiction is a medical illness."
3.‐ "Addiction is a behavioral habit."
4.‐ "Addiction is an emotional attachment."
5.‐ "Addiction is difficult to cure."
4305 A client says, "I have a very small drink every morning Correct answer: 3 Taking a drink in the morning to steady one's nerves is a sign of physical dependence and is Think about people you know and consider to be normal drinkers. Have you ever seen
to calm my nerves and stop my hands from trembling." done to avoid withdrawal symptoms. Tremors are one of the the 10 symptoms of alcohol them engage in an early morning drink of alcohol? If your answer is affirmative, then
The nurse concludes that this client is describing which withdrawal listed in the Clinical Institute Withdrawal Assessment of alcohol symptoms. People recognize that this person is probably not a normal drinker.
of the following? with anxiety may have tremors, but the tremors would occur throughout the day (option 1).
Tolerance is not indicated because the client does not describe needing to have a larger drink
in order to prevent symptoms (option 2). This client has clearly progressed from alcohol abuse
to alcohol dependency (option 4).

1.‐ An anxiety disorder


2.‐ Tolerance
3.‐ Withdrawal
4.‐ Alcohol abuse

4306 The emergency department client is admitted Correct answer: 4 At a blood level of 0.35%, the non‐physically dependent, nontolerant drinker would be Review statistics about blood alcohol levels (BAL) and effects on behavior. Take note of the
following a blow to the head sustained in a motor confused, ataxic, and either semi‐comatose or comatose. Death is expected when the BAL client's absence of usual behaviors associated with a BAL of 0.35%.
vehicle accident. The blood alcohol level (BAL) is 0.35% reaches approximately 0.50%. The situation suggests that this client has been drinking regularly
and the client is walking without stumbling and talking over a long period of time and is now experiencing tolerance to alcohol (needing an increasing
rationally about the accident. What alcoholic amount of alcohol to bring about the desired effect). Tolerance can only develop once the
phenomenon should the nurse recognize? person is physically dependent on alcohol (option 2). This client is not acutely intoxicated, even
though the BAL exceeds the normal level for intoxication (0.08–0.10%). This client's body now
accepts unusually high concentrations of alcohol (tolerance) and has adapted to the presence
of the alcohol (physical dependence) (option 4). There is no evidence of withdrawal symptoms
(option 1), such as anxiety, tremulousness, and marked elevations in vital signs. No information
is given that would allow recognition of psychological dependence (option 3), which can come
very early in the drinking history and precede physical dependence and tolerance.

1.‐ Alcohol withdrawal syndrome


2.‐ Intolerance
3.‐ Psychological dependence
4.‐ Alcohol dependency

4307 A client with a long history of relapsing from cocaine Correct answer: 4 Cravings appear to be the result of pleasurable memories engendered from the psycho‐ Think of the BRS as a hungry animal that is very difficult to satiate.
dependence states that in spite of having a desire to activating effect of engaging in addictive behaviors. Substances of abuse alter the brain's
be sober, thoughts of reusing cocaine continue to reward system by artificially boosting dopamine effects, which keeps the pleasure circuit firing.
occur. The nurse decides to educate the client about In option 1, it is true that environment and role models influence use patterns but this is not
the role the brain reward system (BRS) plays in part of the BRS phenomenon. In option 2, it is true that people employ addictive behaviors to
addiction. The nurse considers that the teaching has self‐medicate stress and pressure experienced, but this is not part of the BRS phenomenon.
been effective if the client says, "The BRS: Option 3 indicates that the BRS is a positive phenomenon that assists with drug abstinence.
Instead, the BRS is a negative phenomenon that assists with maintaining or returning to the
substance use pattern.

1.‐ Reinforces the value of having positive role models.


2.‐ Offers a means of mediating job stress and pressure."
3.‐ Reduces physiologic and psychological cravings."
4.‐ Facilitates cravings and triggers for reusing."

4308 A client asks the nurse to provide information about Correct answer: 3 Medically supervised withdrawal from benzodiazepines generally involves gradual downward Compare protocols for alcohol withdrawal and benzodiazepine withdrawal. Identify areas
the detoxification process and withdrawal from a titration of doses of the drug commonly used (option 3). Rapid or abrupt discontinuation of a of commonality and reasons for same.
benzodiazepine. The nurse should inform the client benzodiazepine is physiologically dangerous and can lead to death (options 1 and 2) Option 4 is
that the process will involve which of the following? incorrect because most antipsychotics lower the seizure threshold and are therefore not
appropriate for clients in active benzodiazepine withdrawal because they would increase the
risk of seizure activity.
1.‐ Rapid reduction in amount and frequency of the drug normally used.
2.‐ Abrupt discontinuation of the drug commonly used.
3.‐ Gradual downward reduction in dosage of the drug commonly used.
4.‐ Planned, progressive addition of an anti‐psychotic drug.

4309 When the nurse is caring for clients experiencing Correct answer: 4 Alcohol withdrawal delirium (delirium tremens, or DTs) is a physiologically dangerous process Remember that alcohol withdrawal delirium is considered a medical emergency and can
delirium tremens, what is the most important nursing with potentially fatal consequences. Various medical approaches are used to treat it, and the lead to death if not properly treated.
intervention? nurse's care must fit into the protocol of the particular agency. Priority is assigned to the
client's physical needs during this major withdrawal phenomenon. Beginning education about
the disease (option 1) and encouraging development of a relapse prevention plans (option 2)
are not appropriate at this time because the client is in physiologic peril. These options can be
appropriate after the withdrawal period has ended. Administering anticraving medications
(option 3) is not the highest current priority, as the client is actively withdrawing from alcohol
and can be at risk physiologically.

1.‐ Present psycho‐education on the dangers of drug and alcohol use.


2.‐ Encourage the client to develop a relapse prevention plan.
3.‐ Administer anti‐craving medications.
4.‐ Provide withdrawal care based on unit protocol.

4310 A client detoxifying from alcohol requires medications Correct answer: 4 The nurse should recognize possible signs of autonomic hyperactivity that is a part of alcohol Review the concepts of physical dependency, autonomic hyperactivity, and cross‐
to treat the withdrawal. The nurse observes coarse withdrawal delirium. If the vital signs (also a part of autonomic hyperactivity) are elevated, the tolerance
hand tremors and diaphoresis. How should the nurse client will require a prn dose of the cross‐tolerant drug that is being used as part of the
first react to this observation? The nurse should: withdrawal protocol. Because the client is in active withdrawal, this is not the time to teach
the client (option 1). The priority is on maintaining physiologic functioning and environmental
safety. Thiamine and folic acid may be ordered for the client who is withdrawing from alcohol,
but they are used to treat complications of alcoholism, not to manage the acute symptoms of
withdrawal (option 2). This can only be done with a drug that is cross‐tolerant with alcohol.
Option 3 is inappropriate, as it would not provide current data. The nurse should be able to
recognize and respond to the clinical signs of increasing intensity of withdrawal symptoms.

1.‐ Explain the concepts of withdrawal to the client.


2.‐ Administer ordered thiamine and folic acid.
3.‐ Determine the most recent blood alcohol level.
4.‐ Assess vital signs.

4311 A nurse is teaching a group of community health Correct answer: 3 Naltrexone (ReVia) is a narcotic antagonist that is useful for treating alcohol‐dependent Review the concept of cross‐tolerance. Determine how it might relate to naltrexone being
colleagues about the use of naltrexone (ReVia) in persons with high levels of craving and somatic symptoms. It works by blocking opiate used to treat alcoholism.
treating alcoholism. The nurse interprets that the receptors and reducing or eliminating the alcohol craving. Naltrexone does not prevent
teaching was effective if the colleagues state that withdrawal symptoms (option 1). Since it is a narcotic antagonist, and narcotics and alcohol
naltrexone (ReVia) is used to do which of the are both CNS depressants, it is possible that naltrexone (ReVia) could precipitate withdrawal
following? symptoms in an individual who has had recent intake of alcohol. Naltrexone (ReVia) is not
expected to prevent or reduce alcoholic blackouts (option 2) or to directly manage anxiety
(option 4).
1.‐ Prevent withdrawal symptoms.
2.‐ Reduce number of blackouts.
3.‐ Reduce craving for alcohol.
4.‐ Manage heightened anxiety.
4312 Adolescent alcoholic clients often relapse into Correct answer: 2 The quality of an adolescent's recovery environment can be helpful or hurtful to someone Consider the tremendous impact that peer pressure has on adolescents.
drinking because they feel pressured by their peers. attempting to maintain sobriety. Friends or acquaintances may encourage a recovering person
Which skill training should the nurse plan for these to use. The recovering adolescent may want to refuse, but may not know how. Behavioral
clients in order to assist them in relapse prevention? rehearsal, saying "no thanks" to an offer to engage in addictive behavior, can increase a
recovering person's confidence. Vocational skills (option 1) will not help the adolescent to
refuse a drink. Problem‐solving skills (option 3) and communication skills (option 4) may be
useful but not as helpful as skills directly related to refusing to drink.

1.‐ Critical thinking skills


2.‐ Drinking refusal skills
3.‐ Problem‐solving skills
4.‐ Communication skills

4313 A client is admitted to a medical unit for treatment of Correct answer: 4 The definition of decisional conflict is uncertainty about a course of action to be taken when Look carefully at the information that is given in the stem. Do not assume that risk for
chest pain. A family member reports a client history of choice among competing actions involve risk, loss, or challenge to personal life values. In injury, which is often a correct response, is correct in this instance.
chemical abuse. The client is ambivalent about the option 1, dysfunctional family processes: alcoholism may apply, but it is more appropriate for
recommendation for treatment made by an addiction the family than the individual. Ineffective management of therapeutic regimen (option 2)
consulting team. An appropriate nursing diagnosis for implies that the client has already made a commitment to recovery. Since the client probably
the client would be: abuses or is dependent on alcohol, risk for injury (option 3) may be present. However, what is
shown in the stem of the question is behavior that indicates decisional conflict.

1.‐ Dysfunctional family processes: alcoholism.


2.‐ Ineffective therapeutic regimen management.
3.‐ Risk for injury.
4.‐ Decisional conflict.

4314 A nurse is teaching a group of clients about addiction. Correct answer: 1 The key symptom of addiction is impaired control, or the inability to control, or regulate, You might find it easier to answer this question if you reword it for yourself and then look
One client says he can stop drinking whenever he one's addictive behavior. In addition to loss of control, the addicted person is not able to view for the things that are true about addiction.
wants. The nurse concludes that this client does not the addictive behaviors realistically (option 4) and frequently uses the defense mechanisms of
yet understand that addiction is a disease in which denial, rationalization, and projection. While persons with addiction do not change their
individuals lose ability to: behavior because of negative consequences suffered (option 2), it is not that they do not
recognize the consequences. Rather, they continue the addictive behavior in spite of
consequences experienced. Acting sober when intoxicated (option 3) is an addictive behavior.

1.‐ Control addictive and impulsive behaviors.


2.‐ Recognize that addictive behavior is harmful to themselves and others.
3.‐ Act sober even if they are not.
4.‐ Think logically about their addictive behaviors.

4315 As part of assessment activities to determine if the Correct answer: 3 The G in the CAGE mnemonic represents &lt;u&gt;guilt&lt;/u&gt;. Not gulping drinks, as in Notice the words is not consistent in the stem of this question. This tells you that the
client is alcohol dependent, the nurse needs to Option 1, indicates the C: &lt;u&gt;cutting&lt;/u&gt; down or reducing alcohol. Option 2 correct answer is an incorrect item in the options. Analyze each option and select the one
conduct a CAGE assessment with the client. Which represents the A, being &lt;u&gt;annoyed&lt;/u&gt; at what others say about the drinking. that is not part of that set of questions.
question asked by the nurse would not be consistent Option 4 represents the E, having an early morning drink to open the &lt;u&gt;eyes&lt;/u&gt;
with the structure of CAGE? "Have you ever: and calm the nerves.

1.‐ Felt that you needed to cut down on your drinking?"


2.‐ Been annoyed by comments made about your drinking?"
3.‐ Found yourself gulping drinks before going out?"
4.‐ Had a morning 'eye‐opener' to calm your nerves?"
4316 After orienting the client to the addiction treatment Correct answer: 4 Option 2 presents reality to the client in a matter‐of‐fact, informative way and creates an Choose the answer that is most accurate, informative, and respectful of the client.
unit, the nurse suggests that the client invite his 13‐ opportunity for the nurse to help the client see that the parent–child relationship has no doubt
year‐old son to the family sessions. The client been impacted by the addiction. Option 1 uses a judgmental and demeaning term a drunk,
questions why the son needs to participate, because although the information it is conveying is accurate. Option 3 offers approval or praise and
he has not seen his father drunk. What is the best allows the client to feel like a protective and good parent, instead of a parent whose behavior
response by the nurse? has impacted negatively on the son. Option 4 removes the personal focus that is necessary to
help the addicted parent recognize the impact of the addiction on the son.

1.‐ "Your son probably knows that you are a drunk."


2.‐ "Your son has probably seen changes in you when you were drinking."
3.‐ "It's good that you have concern for your underage son."
4.‐ "Thirteen‐year‐olds are old enough to start learning about the effects of alcohol."

4317 The nurse conducts an inservice session about Correct answer: 2 The most significant risk factors that lead nurses to abuse drugs and become drug dependent Identify what is different about the environment and knowledge levels of nurses and non‐
impaired nursing practice. The nurse evaluates that the are: (1) exposure to substances, (2) knowledge about specific effects of certain drugs, and (3) nurses. This will help guide you to the correct option.
teaching was effective when one of the nurses says belief that knowledge about drugs will allow them to use drugs and alcohol safely. Some
that the most influential risk for impaired nursing nurses have grown up in a dysfunctional family (option 1), but this does not put them at more
practice is which of the following? risk than those in the general public who have similar backgrounds. Most nurses know that
healthcare providers and professionals are at a high risk for drug dependency, but they deny
that this could happen to them, as they feel protected by their knowledge about drugs (options
2 and 3). Some nurses may have problems with codependence (option 4), but this does not put
them at more risk than those in the general public who have similar problems.

1.‐ Having grown up in a dysfunctional family


2.‐ Feeling that their knowledge about drugs protects them from drug dependency
3.‐ Thinking that professionals are not at high risk for substance dependency
4.‐ Having a tendency to involve self in co‐dependent professional and personal relationship

4318 A male client comes to day treatment surrounded by Correct answer: 1 It would not be unusual for a client who has severe addiction to come to day treatment Recall the common trio of defense mechanisms used by substance dependent individuals.
an intense odor of alcohol. The client staggers when intoxicated and deny it. Denial would cause a client to insist he or she is not intoxicated or Look for a behavioral example of one of them.
walking but insists that that he has not consumed any doesn't have a problem with alcoholism despite concrete evidence of the problem.
alcohol. The nurse concludes that this behavior Rationalization (option 2) is a frequently used defense mechanism of the alcoholic individual,
constitutes which of the following? but if it were being used, the client would offer an explanation for the odor of alcohol (such as
"I spilled a bottle of cologne as I was getting dressed." Transference is the unconscious process
of displacing feelings for significant people in the past onto the nurse in the present
relationship (option 3). Countertransference (option 4) is the nurse’s emotional reaction to
clients based on feelings for significant people in the nurse's past.

1.‐ Denial
2.‐ Rationalization
3.‐ Transference
4.‐ Countertransference

4319 A female alcohol‐dependent client who has Correct answer: 4 Option 4 indicates one of the areas of the CAGE questionnaire that deals with expressed Look for areas of commonalty between options. Look for the one that is different. In this
cardiomyopathy tells the nurse that she is certain that concern from others about client’s drinking. Options 1, 2, and 3 would support the client's case, it is the most matter‐of‐fact and least opinionated statement.
her family and friends are against her. The client goes belief that others are against her or have no right to be concerned about her. Specifically, each
on to say, "They stay on my back about my drinking option would support either the client's denial, projection, or rationalization.
and say I could die from it." What would be the best
response by the nurse?
1.‐ "Anyone saying this to you must have a problem with his or her own drinking."
2.‐ "Although their intentions are good, they have no right to judge another person’s drinking."
3.‐ "Do you think they may be jealous that you can drink more than they can?"
4.‐ "Perhaps they have noticed that your drinking creates consequences for you."
4320 A client who is recovering from alcoholism presents in Correct answer: 3 This client will likely be dually diagnosed with alcoholism and depression. The nurse should Notice that this client has two problems, one of which (alcoholism) is considered a long‐
the psychiatric unit and tells the admitting nurse she is recognize that current standards of addiction practice call for the substance use disorder and term problem that can be managed but never fully recovered from.
very depressed and has a hard time staying sober. The the psychiatric disorder to be treated simultaneously. Options 1, 2, and 4 do not recognize that
nurse concludes that the most likely treatment in instances of dual diagnosis; current standards of addiction practice call for the substance use
approach for this client will be one that involves disorder and the psychiatric disorder to be treated simultaneously.
treating the client's:

1.‐ Depression before the sobriety issue.


2.‐ Sobriety issue before the depression.
3.‐ Sobriety issue and depression at the same time.
4.‐ Depression after the sobriety issue has been resolved.

4321 As part of the clinical experience, a student nurse is Correct answer: 2 AA teaches that a client with alcoholism can never safely return to social drinking and that Recall that alcoholism is considered a chronic disease that is characterized by remissions
required to attend an Alcoholics Anonymous (AA) total abstinence is the only course in treating the addiction. When sobriety has been achieved, and exacerbations.
meeting and write a report about what was learned. people don't "graduate" (option 1); they stay and help others achieve sobriety. Acceptance and
What information would the student include in the Higher Power (options 3 and 4) are active concepts in AA, but practicing these principles does
report about the 12‐step program? not remove urges to drink and does not guarantee sobriety.

1.‐ Once an individual learns how to be sober, he or she can graduate from attending meetings.
2.‐ Once an individual has achieved sobriety, he or she continues to be at risk for relapse into drinking.
3.‐ Acceptance of being an alcoholic will prevent urges to drink, since it represents giving up one's denial.
4.‐ A "Higher Power" will protect individuals if they feel like using.

4322 The nurse working in the maternal care area is Correct answer: 3 Alcohol use during pregnancy causes dysmorphic prenatal and postnatal difficulties and Think about the nature of the drugs listed and their potential effects to make a selection.
reinforcing physician health teaching about the risks of central nervous system dysfunction. These problems range from subtle cognitive‐behavioral Do not be misled by the fact that alcohol is a drug that can be obtained legally without a
substance use during pregnancy. When questioned by impairments to fetal alcohol syndrome, both of which predispose the infant to later academic prescription
the client, the nurse should reply, "The drugs that are and behavioral problems, as well as mental illness. Since alcohol is so widely used, many
most likely to lead to significant physical, cognitive, people do not recognize its dangers, as they either do not consider it a drug or think that it is a
and developmental problems for any infant would be: safe drug. Options 1, 2, and 4 indicate substances that can cause significant health problems
for the infant, but these problems are not as pervasive as those associated with the mother's
using alcohol during pregnancy.

1.‐ Benzodiazepines."
2.‐ Hallucinogens."
3.‐ Alcohol."
4.‐ Cocaine."

4323 Having requested it as part of a comprehensive Correct answer: 1 The adverse reaction of disulfiram (Antabuse) will occur if the person taking this drug ingests, Recall the principles of operant conditioning and integrate that with your knowledge of
treatment program, the client is to receive disulfiram inhales, or absorbs alcohol, even in very small doses (such as inhaling vapors from paints or the expected drug effect.
(Antabuse). Which statement should the nurse include woodstains, or oral ingestion in products such as mouthwash). These reactions include
when teaching the client about this drug? throbbing headache, tachycardia, diaphoresis, and respiratory distress. Death can occur. This
drug is not used often, but the nurse should know about its uses and dangers. While eating
improperly cooked seafood (option 2) might lead to gastric distress and/or liver problems,
uncooked seafood does not precipitate a disulfiram reaction. Disulfiram does not reduce the
craving for alcohol (option 3), but opioid antagonists, such as naltrexone (ReVia) do. Disulfiram
works on the classical principle of conditioned avoidance. If the individual drinks alcohol while
taking disulfiram, intensely unpleasant and dangerous physical reactions can occur. The effect
of disulfiram (Antabuse) when combined with alcohol is not intoxication (option 4). Instead,
the individual experiences intensely unpleasant and dangerous physical reactions.

1.‐ "Inhaling fumes from paints and wood stains may cause a disulfiram reaction."
2.‐ "Eating inadequately cooked seafood may lead to disulfiram (Antabuse) resistance."
3.‐ "Taking disulfiram will reduce your physical craving for alcohol."
4.‐ "If you consume alcohol while taking disulfiram, rapid intoxication will occur."

4324 A nurse has been told that a client's anxiety is at the Correct answer: 1 Subjective complaints of panic level of anxiety include choking or smothering sensation, The core issue of the question is an ability to identify signs of panic in a client with anxiety.
panic level. The nurse would assess the client for which dizziness, chest pain or pressure, and fear of loss of control and death. Feelings of stomach Use nursing knowledge and the process of elimination to make a selection.
of the following? "butterflies" are seen in the fight‐or‐flight response. Feelings of fatigue and inability to remain
awake may be seen in the exhaustion stage of the general adaptation syndrome. Obsessive
thoughts and compulsive behaviors are common in obsessive‐compulsive disorder.

1.‐ Dizziness, palpitations, and nausea


2.‐ Feelings of "butterflies" in the stomach
3.‐ Feelings of fatigue and inability to remain awake
4.‐ Obsessive thoughts and compulsive behavior

4325 The nurse concludes that a client has agoraphobia Correct answer: 3 Agoraphobia involves fear of being away from home and being alone in public places. Specific The core issue of the question is an ability to identify signs of agoraphobia in a client. Use
after the client states a fear of which of the following? phobia involves unrealistic fear of a particular object or situation. Social phobia is excessive nursing knowledge and the process of elimination to make a selection.
fear of embarrassment and humiliation in public settings. Fear of loss of control is common in
most phobias.
1.‐ Spiders
2.‐ Being embarrassed in public
3.‐ Leaving the home
4.‐ Losing control

4326 A nurse asks a client, "Have you ever felt a sudden, Correct answer: 3 The onset of a panic attack is sudden, and the client may not be aware of the source of the The core issue of the question is an ability to identify signs of panic disorder. Use nursing
intense fear for no apparent reason?" When the client anxiety. Agoraphobia is fear of being incapacitated by being trapped in an unbearable situation knowledge and the process of elimination to make a selection.
responds affirmatively, the nurse would assess the from which there is no escape. Obsessive‐compulsive disorder is characterized by obsessive
client for other symptoms compatible with which of thoughts and compulsive behaviors. Posttraumatic stress disorder is associated with exposure
the following? to an extremely traumatic, menacing event.

1.‐ Agoraphobia
2.‐ Obsessive‐compulsive disorder
3.‐ Panic disorder
4.‐ Posttraumatic stress disorder

4327 A client has just been told that another operation Correct answer: 2 Because coping resources are depleted, the client can no longer deal with stressors. Stage of The core issue of the question is an ability to differentiate stages of anxiety based on client
needs to be performed to correct a physical health alarm is characterized by the fight‐or‐flight response, and increased alertness is focused on the presentation. Use nursing knowledge and the process of elimination to make a selection.
problem. The client begins to cry and says, "I just can't immediate task or threat. Stage of resistance occurs when the body mobilizes resources to
take it anymore. Everything has gone wrong. I can't combat stress.
even think straight anymore." The nurse interprets
that the client is in which stage of anxiety?

1.‐ Alarm
2.‐ Exhaustion
3.‐ Fight‐or‐flight
4.‐ Resistance

4328 A nurse working with an extremely anxious client Correct answer: 1 Anxiety in a client may be empathetically experienced by the nurse. It is imperative that the The core issue of the question is the ability to determine the effect that an anxious client
reports feeling short of breath, tense, restless, nurse recognize these symptoms. There is not enough data to support the client being angry. can have on the nurse. Use nursing knowledge and the process of elimination to make a
apprehensive, and nervous. The nurse would most Even a nurse with high self‐esteem is receptive to experiencing anxiety empathetically. selection.
appropriately draw which conclusion?

1.‐ The client's anxious feelings have been transmitted to the nurse.
2.‐ The client is probably becoming angry.
3.‐ The client should be reassigned to a different nurse.
4.‐ The nurse probably has a self‐esteem disorder.

4329 A client is going to begin electroconvulsive therapy Correct answer: 1 The client always needs to be treated with dignity and respect. There is no reason to bring the The core issue of the question is the right of every client to be treated in a respectful
(ECT). The nurse knows that ECT is usually prescribed client to the treatment suite on a stretcher (option 2), nor does the client usually need to have manner. Use nursing knowledge and the process of elimination to make a selection.
for individuals who have major depression. The nurse a procedure explained many times by the physician or the nurse (option 3). The client with
prepares a teaching plan keeping in mind that clients major depression can receive ECT (option 4) if medication therapy is not effective.
with major depression:

1.‐ Need to be treated with respect and dignity.


2.‐ Need to be brought to the treatment suite on a stretcher.
3.‐ Should have the procedure explained to them many times because they cannot understand or retain the information.
4.‐ Should not receive ECT.

4330 The visiting nurse is at the home of a 52‐year‐old Correct answer: 2 The only possible correct answer is option 2. Hypomania is a mood of elation (option 1), while The core issue of the question is an ability to identify clients at risk for various forms of
postoperative client. During his presurgical physical cyclothymia (option 3) is a disorder of at least 2 years’ duration with episodes of hypomania, depression. Use nursing knowledge and the process of elimination to make a selection.
assessment, the client was diagnosed with type 2 and dysthymia (option 4) is a depressive disorder of at least 2 years’ duration.
diabetes mellitus. At that time, he also related that he
was not sleeping well and had a decreased appetite.
He reported that he lost his job of 34 years 3 weeks
before the preoperative physical exam and was very
angry. The nurse then asks the client additional
questions to elicit data about possible:

1.‐ Hypomania.
2.‐ Unipolar depression.
3.‐ Cyclothymia.
4.‐ Dysthymia.

4331 Which would be the safest living environment for a Correct answer: 4 The client would be safest in a closed‐door seclusion room (option 4) or in a locked unit (not The core issue of the question is placement of a client who has harmed another. Use
client who inflicted harm on a family member earlier in an option here). The client would not have the continuous monitored care if he were in a nursing knowledge and the process of elimination to make a selection. Keep in mind that
the day? respite home (option 1). He would have less safety or care in the home of a relative in another the safety of the client and others around him or her is the first priority.
state (option 2). In an open‐door seclusion room (option 3), the client could leave the area and
harm others if there were distractions to the staff on the unit.

1.‐ In a local respite home


2.‐ With a family member in another state
3.‐ In an open‐door seclusion room
4.‐ In a closed‐door seclusion room

4332 Part of a discharge plan for a client on a psychiatric Correct answer: 3 Options 1, 2, and 4 may be correct intervention terms, but option 3 is the most correct The core issue of the question is the ability to determine various types of nursing
inpatient unit includes walking for half an hour 3 days terminology for the nursing intervention described to prevent increased anxiety and stress. interventions needed by a hospitalized client with a mental health problem. Use nursing
per week to maintain cardiovascular health and knowledge and the process of elimination to make a selection.
decrease stress levels. The nurse includes this in the
care plan as what type of nursing intervention?

1.‐ Active
2.‐ Performance
3.‐ Preventive
4.‐ Physical
4333 A client has a diagnosis of bipolar disorder. The nurse Correct answer: 4 The client who demonstrates a calm, focused exchange of information and self‐care The core issue of the question is appropriate behavior exhibited by a client with bipolar
is evaluating the client in a home environment after information would demonstrate control of the disorder, which is expected following discharge disorder after treatment. Use nursing knowledge and the process of elimination to make a
discharge from an inpatient unit 2 weeks ago. The from an inpatient setting. The client in a manic state would present with the option 1 or 2 selection.
nurse assesses the client for which of the following behaviors, while the client with depression would present as option 3 indicates.
expected behaviors?
1.‐ Euphoric and talkative presentation with nurse
2.‐ Gregarious interactions with significant others
3.‐ Quiet and evasive presentation
4.‐ Calm, focused exchange of self‐care information with nurse

4334 A 21‐year‐old male college student has become Correct answer: 2 Voicing doubt about the delusions is the most therapeutic intervention. The client will The core issue of the question is appropriate therapeutic communication techniques to
increasingly suspicious of his professor and fellow continue to voice a delusion even though the evidence would suggest otherwise (option 1). A use with a client who is paranoid. Use nursing knowledge and the process of elimination to
classmates. He has accused the professor of conspiring paranoid client cannot use logic to dispel delusions (option 3). Option 4 challenges the client's make a selection.
with two other classmates to get him expelled from belief instead of voicing doubt. Providing evidence will not usually sway a paranoid client.
school. The client is admitted to a psychiatric unit after
telephoning and threatening to kill the professor and
his classmates. The client tells the nurse, "They are all
out to get me expelled. I think they are even trying to
kill me. I have to stop them." What would be the most
appropriate response by the nurse?

1.‐ "What makes you think they are out to get you expelled or to kill you?"
2.‐ "I find it hard to believe that your professor and classmates are out to get you expelled or to kill you."
3.‐ "It's not right to kill others even if they are out to get you expelled or want to kill you."
4.‐ "Your professor and classmates are not out to get you expelled or to kill you. Let's look at the facts."

4335 A client with schizophrenia is exhibiting delusions, Correct answer: 1 Minimal self‐care is a behavioral negative symptom of schizophrenia. A delusion is a cognitive The core issue of the question is an ability to discriminate between positive and negative
hallucinations, minimal self‐care, and hyperactive positive symptom (option 2); hallucination is a perceptual positive symptom (option 3); and signs of schizophrenia. Use nursing knowledge and the process of elimination to make a
behavior. Which of the following would the nurse inappropriate affect (option 4) is an affective positive symptom. selection.
document as a negative symptom of schizophrenia?

1.‐ Minimal self‐care


2.‐ Delusions
3.‐ Hallucinations
4.‐ Inappropriate affect

4336 A client hears voices telling him that he is a terrible Correct answer: 2 Client safety is a priority. Hearing voices (hallucination) is a sensory‐perceptual alteration, but The core issue of the question is an ability to set priorities for a client who experiences
person who would be better off dead. Which of the safety is a priority (option 3). There is not enough data to support the nursing diagnoses of auditory hallucinations. Look for the option that puts client safety first. Use nursing
following would be a priority nursing diagnosis? impaired verbal communication (option 1) or impaired social interaction (option 4). knowledge and the process of elimination to make a selection.

1.‐ Impaired verbal communication


2.‐ Risk for violence, self‐directed
3.‐ Impaired sensory‐perception
4.‐ Impaired social interaction

4337 A client living in an assisted living facility is taking Correct answer: 3 Neuroleptic malignant syndrome (NMS) is a potentially fatal extrapyramidal symptom. The core issue of the question is an ability to identify signs of adverse medication effects in
conventional antipsychotic medications. One evening Symptoms of NMS develop suddenly and include muscle rigidity, respiratory problems, and a client being treated for psychosis. Use nursing knowledge and the process of elimination
the nurse notices that the client is experiencing muscle hyperpyrexia. Dystonia (option 1) and akathisia (option 2) are both extrapyramidal symptoms to make a selection.
rigidity, confusion, delirium, and has a temperature of that are usually not fatal (option 1). Tardive dyskinesia symptoms include frowning, blinking,
104° F. The nurse interprets these as symptoms of: grimacing, puckering, blowing, smacking, licking, chewing, tongue protrusion, and spastic facial
distortions, which can be socially disfiguring (option 4).
1.‐ Dystonia.
2.‐ Akathisia.
3.‐ Neuroleptic malignant syndrome.
4.‐ Tardive dyskinesia.

4338 A client states that he is able to receive radio waves Correct answer: 4 A delusion is a false belief that cannot be changed by logical reasoning or evidence. A The core issue of the question is an ability to correctly identify the types of thought
from aliens because they placed a computer chip in his hallucination is the occurrence of a sight, sound, touch, smell, or taste without any external patterns expressed in a client's communications. Use nursing knowledge and the process of
brain. The nurse would document this behavior as stimulus to the corresponding sensory organ; it is real to the client (option 1). The client is not elimination to make a selection.
which of the following in the medical record? exhibiting reality orientation (option 2). An illusion is a sensory misperception of
environmental stimuli (option 3).
1.‐ A hallucination
2.‐ Reality‐oriented
3.‐ An illusion
4.‐ A delusion

4339 The most appropriate outcome of care for a male Correct answer: 2 The client who gains coping skills reduces anxiety to a level at which dissociation is unlikely to The core issue of the question is an appropriate outcome of care for a client who was in a
client who has experienced a dissociative fugue is that occur. The client does not remember what occurred during the fugue state, nor does he dissociative fugue state. Use nursing knowledge and the process of elimination to make a
the client will do which of the following? experience depersonalization. selection, recalling that anxiety is usually the cause of the state, and therefore the answer
points to an item that reduces anxiety.
1.‐ Remember what occurred during his fugue state
2.‐ Gain additional coping skills to deal with his current problems
3.‐ Report no feelings of being detached from his body
4.‐ State three positive aspects about himself

4340 A client with dissociate identity disorder (DID) who Correct answer: 3 Self‐mutilation, not uncommon with DID clients, is identified in the assessment. The client The core issue of the question is a priority nursing diagnosis. Questions such as these are
has just been admitted with several fresh burns on her remains at risk for injuring herself, producing tissue damage that provides tension relief. There frequently focused on safety. Use nursing knowledge and the process of elimination to
ankles and wrists is refusing to attend group therapy. is no intent to kill; however, the client will need to learn less damaging ways to obtain relief. make a selection.
What is the priority nursing diagnosis?

1.‐ Self‐care deficit


2.‐ Impaired sensory perception
3.‐ Risk for self‐mutilation
4.‐ Noncompliance

4341 A client reports depersonalization experiences that Correct answer: 1 This response demonstrates empathy and encourages the client to elaborate further about The core issue of the question is a therapeutic response to a client who is experiencing
have been frightening to him. Which of the following is his experience. Options 2 and 3 dismiss the affective component or miss the point of the depersonalizing events. Use nursing knowledge of therapeutic communication skills and use
the most therapeutic response by the nurse? client's statement. Option 4 is helpful in making connections between events but is not the the process of elimination to make a selection.
best response to the client’s original comment.
1.‐ "It must be very scary for you. Tell me more about how they occur."
2.‐ "Don't worry, you will always come back together."
3.‐ "Being in the hospital must be very frightening."
4.‐ "Let's focus on the stresses in your life."

4342 Which of the following behaviors would indicate that Correct answer: 3 The goal of care is to eliminate or reduce dissociative experiences, which can be The core issue of the question is the ability to determine an appropriate outcome of care
care for a client who dissociates has been effective? accomplished in part by anxiety‐produced stress management techniques. for a client who dissociates. Recall that anxiety plays a role in dissociation and choose the
option that reduces anxiety.
1.‐ Client reports dissociative episodes to the nurse
2.‐ Client seeks out social relationships
3.‐ Client demonstrates three stress management techniques
4.‐ Client is free from injury
4343 A client with amnesia is hospitalized. What might the Correct answer: 4 Amnesia is precipitated by stress related to trauma or conflict. The amnesia occurs abruptly The core issue of the question is knowledge of expected assessment findings in a client
nurse expect to find during the initial assessment? and there is no attempt to cover the memory loss. Confabulation, gradual loss of memory, and with amnesia. Use nursing knowledge and the process of elimination to make a selection.
disheveled appearance are common in clients experiencing dementia.

1.‐ Confabulation of historical information


2.‐ Gradual loss of memory over months
3.‐ Disheveled appearance
4.‐ History of severe stress

4344 The nurse would look for which of the following Correct answer: 3 A client with OCD, a Cluster C disorder, appears anxious or fearful. Individuals with a Cluster A The core issue of the question is knowledge of expected behaviors that would be exhibited
characteristics in the behavior of a client diagnosed disorder appear odd or eccentric; those with a Cluster B disorder appear dramatic or erratic. by a client with OCD. Recall that a client with this type of diagnosis is very ritualistic and
with obsessive‐compulsive disorder (OCD)? The category of rigid/critical does not reflect a diagnostic cluster. therefore rigid. Use nursing knowledge and the process of elimination to make a selection.

1.‐ Dramatic/erratic
2.‐ Odd/eccentric
3.‐ Anxious/fearful
4.‐ Rigid/critical

4345 The nurse places highest priority on which of the Correct answer: 3 In caring for clients diagnosed with antisocial personality disorder, it is important to maintain The core issue of the question is an ability to set priorities for a client who has an
following nursing interventions when caring for a client a structured and consistent environment to decrease their attempts to control the situation antisocial personality disorder. Use nursing knowledge and the process of elimination to
diagnosed with antisocial personality disorder? through manipulation. It is unlikely that they will develop insight as the causes of problems in make a selection.
living are externalized. They are frequently quite sociable and take advantage of others for
personal profit. Suicidal ideation is not associated with this disorder.

1.‐ Supporting the development of insight


2.‐ Encouraging socialization
3.‐ Maintaining consistent limits
4.‐ Monitoring for suicidal ideation

4346 The nurse looks for which of the following Correct answer: 2 Individuals diagnosed with personality disorders display either functional impairment or The core issue of the question is knowledge of expected assessment findings regarding
characteristics in a client diagnosed with a personality subjective distress. Frequently these problems in living are reflected in impaired interpersonal behavior style in a client with a personality disorder. Note that the type of disorder is not
disorder? relationships. Flexibility and adaptability to stress (option 1) are incongruent with a diagnosis specified, so the answer is a global or general pattern. Use nursing knowledge and the
of a personality disorder. The presence of a physical disorder (option 3) has no relation to the process of elimination to make a selection.
diagnosis of a personality disorder. These individuals may or may not desire interpersonal
relationships (option 4).
1.‐ Flexibility and adaptability to stress
2.‐ A tendency to evoke some form of interpersonal conflict
3.‐ A concomitant physical disorder
4.‐ A desire for interpersonal relationships

4347 Which of the following beliefs by the nurse as a Correct answer: 1 Individuals diagnosed with antisocial personality disorder frequently try to play one staff The core issue of the question is core beliefs about the value and roles of the
member of the interdisciplinary team is most member against the other in order to control their environment. It is imperative that staff interdisciplinary team. Use nursing knowledge and the process of elimination to make a
important to remember when developing a care plan present a unified, consistent, and structured approach to care to prevent this. Options 2, 3, selection.
for a client diagnosed with antisocial personality and 4 are incorrect because they would result in lack of team unity and an unstructured
disorder? approach to care.
1.‐ Everyone involved in the client's care must agree with the diagnosis, goals, and plan.
2.‐ The team leader must determine the diagnosis and treatment plan to insure accuracy.
3.‐ The involvement of all team members in developing a nursing care plan is not necessary.
4.‐ An unstructured treatment approach is usually beneficial.
4348 The nurse is assessing a client with obsessive‐ Correct answer: 1 Individuals diagnosed with obsessive‐compulsive personality disorder become overly involved The core issue of the question is knowledge of expected behavior styles in a client with
compulsive personality disorder. Most of the client's in details such as rules and regulations related to a need to be perfect. As a result, they fail to OCD. Use nursing knowledge and the process of elimination to make a selection.
cognitive content will be centered around which of the see the big picture. Their relationships with others and participation in leisure activities are
following? less important to them than is their devotion to work and productivity.

1.‐ The importance of rules and regulations


2.‐ Global approaches to problem solving
3.‐ Relationships with others
4.‐ Preferred leisure activities

4349 A client continues to have pain despite negative Correct answer: 1 The continuance of pain is related to reinforcement of the symptoms, such as the caring The core issue of the question is knowledge of possible explanations of causation of pain
neurological findings. The nurse concludes that such responses of others, which give the client benefits that otherwise might not occur. There is no in a client with no physical basis for pain. Use nursing knowledge and the process of
pain is likely to continue because of which of the organic basis for the pain. High endorphin levels are associated with feelings of euphoria. elimination to make a selection.
following?
1.‐ Secondary gain
2.‐ High endorphin levels
3.‐ Structural changes of tissue
4.‐ Derealization

4350 A client with a conversion disorder has been eagerly Correct answer: 4 The client with a conversion disorder is characteristically indifferent to the symptoms (la belle The core issue of the question is knowledge of expected assessment findings related to
preparing for his marriage. On the morning of the indifference) rather than being depressed, anxious, or blunted in affect. mood in a client with a conversion disorder. Use nursing knowledge and the process of
wedding, he is unable to move his legs. What would elimination to make a selection.
the nurse expect to find in the mental status
examination?
1.‐ Mood: depressed
2.‐ Mood: anxious
3.‐ Mood: blunted
4.‐ Mood: la belle indifference

4351 A female client with hypochondriasis discloses that Correct answer: 3 Exploration of the client's decision is nonjudgmental and affirms the client's personal power. The core issue of the question is a therapeutic communication to a client with
she may decide to leave the psychiatric facility without This reponse also helps the client understand connections in her own decision‐making process. hypochondriasis. Use nursing knowledge of therapeutic communication skills and the
completing her course of treatment and seek process of elimination to make a selection.
exploratory surgery. The nurse's best response is which
of the following?
1.‐ "If you decide to leave now, you will be committed against your will."
2.‐ "You should not go until your doctor releases you. She knows what you need."
3.‐ "Tell me more about your decision."
4.‐ "Your surgery will just prove useless. Please stay."

4352 A client who has had many different physical illnesses Correct answer: 3 Data indicates that the client perceives a lack of control over her situation. There is The core issue of the question is the ability to form an appropriate nursing diagnosis based
in the past few years is no longer employed, rarely insufficient data to select any of the other nursing diagnoses. on client assessment data. Use nursing knowledge and the process of elimination to make a
does housework or shopping, and states that she "just selection.
can’t seem to do anything." Which of the following is a
priority nursing diagnosis?

1.‐ Impaired home maintenance management


2.‐ Fatigue
3.‐ Powerlessness
4.‐ Body image disturbance
4353 The nurse evaluates that the plan of care for a client Correct answer: 1 When stressors and anxiety are decreased, there remains no need for conversion symptoms, The core issue of the question is knowledge of appropriate outcomes of care for a client
who suddenly lost her hearing (diagnosed as a and normal function resumes. with a conversion disorder. Use nursing knowledge and the process of elimination to make
conversion disorder) was effective if the client: a selection.

1.‐ Resumed normal hearing.


2.‐ Began learning sign language.
3.‐ Was fitted for a hearing aid.
4.‐ Agreed to have a stapedectomy.

4354 Which of the following approaches would be best for Correct answer: 2 Verbal communication should be clear, concise, and unhurried. Shouting may be interpreted The core issue of the question is a therapeutic communication to a client with a cognitive
the nurse who is communicating with the cognitively as anger; therefore, a pleasant, calm, supportive tone of voice should be used. The use of sign impairment. Use nursing knowledge of therapeutic communication skills and the process of
impaired client? language or mostly nonverbal gestures would be frustrating to the client who may not elimination to make a selection.
understand what is being said.
1.‐ Loud and precise
2.‐ Simple and direct
3.‐ As nonverbal as possible
4.‐ Sign language

4355 Which of the following evaluation criteria should the Correct answer: 4 The most important area of concern identified by both family and staff is the safety of clients The core issue of the question is the ability to determine the appropriate priority of care
nurse give first priority to when planning the care of a with dementia. The risk for injury is always present in clients with dementia, and as the disease for a client who has dementia. Focus on safety as an early priority whenever a client has a
client with dementia? progresses, the need for a safe and secure environment increases. The other options are neurological impairment. Use nursing knowledge and the process of elimination to make a
appropriate for dementia but are not the first priority. selection.

1.‐ Preventing further deterioration


2.‐ Finding suitable nursing home placement
3.‐ Supporting family caregivers
4.‐ Preventing injury

4356 A client with suspected Alzheimer's disease is Correct answer: 4 Alzheimer's disease is diagnosed by ruling out causes for the client's symptoms. The only The core issue of the question is methods of diagnosis for Alzheimer's disease. Use nursing
undergoing diagnostic workup. When the family asks definitive method of diagnosis is postmortem examination of brain tissue. knowledge and the process of elimination to make a selection.
the nurse the reasons for the "tests," the nurse
responds that the diagnosis of Alzheimer's disease is
usually based on which of the following?

1.‐ Abnormal laboratory findings


2.‐ A definitive CT scan
3.‐ Physiological findings
4.‐ Ruling out other causes for symptoms

4357 The nurse writing a care plan for a client with Correct answer: 2 Dementia is a progressive disease that causes the individual to lose the ability to perform The core issue of the question is the ability to set goals of care for a client with Alzheimer’s
dementia would include that the overall goal for tasks that were once familiar and routine. Self‐care deficits involving many functional abilities disease. Use nursing knowledge and the process of elimination to make a selection.
nursing care is which of the following? occur to varying degrees. The most effective and respectful goals are those that allow the
client to carry out as much self‐care as possible. Option 2 is the most global goal that
encompasses meeting needs for food, water, dressing, bathing, and so on.

1.‐ Reorient the client to reality.


2.‐ Keep the loss of capacity for self‐care to a minimum.
3.‐ Assist the client with tasks of daily living.
4.‐ Maintain adequate hydration and nutrition.
4358 Which of the following nursing interventions would Correct answer: 1 Cognitive function will be supported by participation in meaningful activities that the client The core issue of the question is planning for a client with dementia that supports
support optimal memory function for a client with enjoys. Stimulating activities will also promote self‐esteem and encourage the client to attain remaining memory function. Use nursing knowledge and the process of elimination to make
dementia? the highest level of cognitive function possible. Options 2, 3, and 4 could lead to frustration a selection.
and more confusion.
1.‐ Develop stimulating and meaningful therapeutic activities.
2.‐ Remind the client of forgotten events.
3.‐ Orient the client to reality.
4.‐ Restrain the client when agitated.

4359 The nurse would conclude that a client with Correct answer: 2, 3 Positive symptoms of schizophrenia are those behaviors that a client would not usually The core issue of the question is the ability to discriminate between positive and negative
schizophrenia is exhibiting positive symptoms of the exhibit in everyday life, including delusion of being a king (option 2) or echolalia (option 3). symptoms of schizophrenia. Use nursing knowledge of these manifestations and the
disorder after noting that the client does which of the Negative symptoms of schizophrenia are those that reflect the absence of what is normally process of elimination to make a selection.
following? Select all that apply. seen in a person's behavior. These would include anergy (option 1), flat affect (option 4), and
social withdrawal (option 5).
1.‐ Exhibits lack of energy
2.‐ States he is a king
3.‐ Repeats words the nurse says
4.‐ Has a flat affect
5.‐ Withdraws from other people

4360 The nurse is establishing outcomes for a client who is Correct answer: 2 Option 2 is correct. This option is measurable, and the others are not. Statements of client Recall basic elements of a well‐written outcome statement, including its measurability.
depressed. The outcomes will be entered into the outcomes should be written in specific measurable terms so that any nurse could determine
nursing care plan and used by all members of the outcome achievement or lack of achievement. This option indicates that a specific numerical
treatment team. The best stated outcome is the client comparison can be made. Options 1, 3, and 4 are incorrect. Each of them indicates the nurse's
will: intention to bring about a change in the client's status, but not one of them is measurable.
How does one measure "more" or "increase" without a standard of comparison?

1.‐ Feel less depressed.


2.‐ Decrease score on depression scale by one half.
3.‐ Develop more insight into his problems.
4.‐ Increase amount of time spent with other clients.

4361 The client is a 5‐year‐old whose pet cat has died Correct answer: 4 Option 4 is correct. At age 5, children do not understand the finality of death, but rather Recall basic facts about children's concepts of death at different ages.
within the past two days. Which of the following consider it a temporary state. Options 1 and 2 are incorrect as these responses would be those
statements about this is the nurse most likely to hear? of older children. Option 3 is incorrect because 5‐year‐olds do not understand the
irrevocability of death.
1.‐ "I don't want to die. My cat did."
2.‐ "The boogie man took my cat away."
3.‐ "I won't ever see my cat again. She's dead."
4.‐ "She'll be back tomorrow. She's dead today."

4362 A parent of a 14‐year‐old client has asked the nurse Correct answer: 3 Option 3 is correct. Erickson's theory places emphasis on peers and the culture and Notice the age of the client and relate that to Erickson's theory.
to explain the client's current difficulty mastering usual environment. Emphasis is given to attempts of the adolescent to incorporate beliefs and values
psychosocial tasks. If the nurse bases a response on of the culture. Option 1 is incorrect as adolescents struggle and strive to become independent
the theories of Erickson, the nurse should say, of the family. Option 2 is incorrect because Erickson's theory places emphasis on peers and the
"Erickson believed that: culture and environment, with the family being a sociocultural unit in the environment. Option
4 is incorrect as the adolescent is in Erickson's stage of identity versus role confusion.

1.‐ Adolescents are overwhelmed at the prospect of becoming independent of the family."
2.‐ The family is the most important influence on the adolescent's development."
3.‐ Modern culture makes identity crisis the most challenging developmental task to resolve."
4.‐ Mastery of doubts preoccupies the adolescent and leads to resistant behavior within the family."
4363 When the nurse informs an adolescent client that a Correct answer: 2 Option 2 is correct. Displacement is the transfer of emotional reactions from one object or Recall definitions of defense mechanisms of the ego and reasons why they are employed.
scheduled parental visit will not occur, the client person to another object or person. Like other defense mechanisms of the ego, displacement
throws a cup at the nurse. In order to best respond to is an unconsciously determined behavior that attempts to reduce anxiety. Reaction formation
this behavior, the nurse should conclude that the client is a mechanism that causes a person to act exactly opposite to the way they feel. Projection is
is displaying which of the following? a process in which blame is attached to others or the environment. Denial is an attempt to
screen or ignore unacceptable realities by refusing to acknowledge them

1.‐ Reaction formation


2.‐ Displacement
3.‐ Projection
4.‐ Denial

4364 A 7‐year‐old recently admitted to a hospital states, Correct answer: 3 Option 3 is correct. It is average and expected for a 7‐year‐old to be anxious and fearful when Recall definitions of defense mechanisms of the ego and reasons why they are employed.
"I’m not afraid of this place; I'm not afraid of admitted to a hospital. One of the ways that the client can reduce the anxiety is to use
anything." Which defense mechanism of the ego does unconsciously motivated defense mechanisms of the ego. Reaction formation is a mechanism
the nurse recognize? that causes a person to act exactly opposite to the way they feel. Regression is resorting to an
earlier, more comfortable level of function. Repression is the unconscious mechanism by
which threatening thoughts, feelings, and desires are kept from becoming conscious.
Rationalization is the justification of certain behaviors by faulty logic and ascription of motives
that are socially acceptable.

1.‐ Regression
2.‐ Repression
3.‐ Reaction formation
4.‐ Rationalization

4365 The client is a child with attention‐deficit Correct answer: 3 Option 3 is correct. A client who is receiving Ritalin is at risk for delayed growth and Review important side effects of stimulants. Consider which of these are most relevant
hyperactivity disorder (ADHD) who is receiving development. This is one of the most adverse effects that may occur if Ritalin (or most other when the client is a child.
methylphenidate (Ritalin). The nurse teaches the stimulants) is administered to children over a long period of time. The child should be weighed
parents to carefully monitor which of the following in 2–3 times per week and weight loss should be reported promptly to the doctor or other
the child? prescribing health care team member. Dental care, although important for child health, is
nonspecific to therapy. Drying of membranes is a bothersome, but manageable side effect.
Like other children, children taking Ritalin have specific physiologic needs for milk. However,
there is no reason to decrease or increase milk intake in client's taking Ritalin. Instead, intake
of caffeine should be restricted.

1.‐ Dental health


2.‐ Oral mucous membranes
3.‐ Height and weight
4.‐ Milk consumption

4366 Because of academic failure, a 10‐year‐old is to Correct answer: 3 Option 3 is correct. According to Erikson's stages of development, a 10‐year‐old is Review Erickson's developmental tasks by age group and apply the concepts to the age of
repeat a grade in school. When counseling the parents experiencing industry versus inferiority. Shame (option 1), guilt (option 2), and role confusion this child.
of this child, the school nurse who ascribes to (option 4) occur at other developmental levels.
Erickson's theories will advise the parents to be alert
for indications of which of the following?

1.‐ Shame
2.‐ Guilt
3.‐ Inferiority
4.‐ Role confusion
4367 A 14‐year‐old client is to be admitted to a psychiatric Correct answer: 4 Option 4 is correct. Children and adolescents with bipolar disorders are often misdiagnosed Recognize the similarities and differences between behaviors associated with the various
in‐patient unit for treatment of bipolar disorder. When as having conduct disorder or ADHD, which are part of the spectrum disorders of childhood. spectrum disorders of children and adolescents. Compare these to the clinical
assisting staff to understand behaviors that are likely The adolescent client with bipolar disorder is likely to experience multiple and extreme mood presentations of mania in adults.
to be seen in this client, what should the nurse swings in the course of a day. The adolescent client with bipolar disorder does have inflated
emphasize? self esteem, but this is not as evident as it in the adult with Bipolar disorder. Instead, the
adolescent client is much more likely to be irritable and socially aggressive and to experience
multiple mood swings in the course of the day. The adolescent client with bipolar disorder
does tend to manage money poorly, but this is not as evident as in the adult with Bipolar
disorder. Instead, the adolescent client is much more likely to be irritable and socially
aggressive and to experience multiple mood swings in the course of the day.

1.‐ Prolonged periods of extreme mood


2.‐ Inflated self‐esteem
3.‐ Poor management of spending money
4.‐ Inattention and distractibility

4368 While conducting a mental status examination with Correct answer: 2 Option 2 is correct. This child is demonstrating inability to think abstractly. Rather than being Notice the age of the child. Apply that to the basic concepts of Piaget's theory.
an 8‐year‐old girl, the nurse asks the client to explain alarmed by this, the nurse should recognize it as normal growth and development. Eight‐year‐
the meaning of the expression "Don't cry over spilled olds cannot be expected to think abstractly. In Piaget's theory, abstract thinking develops
milk." The child looks puzzled and shrugs her during the formal operational phase (12 years to adult). Children between 7–12 are in Piaget's
shoulders. The nurse using Piaget's theory will view the concrete operational phase and cannot think abstractly, although they can pretend. Option 1 is
child's response as suggestive of which of the incorrect because while Piaget’s theory does focus on cognitive styles, and this option contains
following? the term "cognition," Piaget's theory does not address impaired cognition. Impaired cognition
is abnormal. Piaget's theory deals with normal growth and development. Option 3 is incorrect
as this client is not able to think abstractly. In Piaget's theory, ability to reason abstractly is a
function of formal operational thinking that begins at approximately 12 years of age and
extends into adulthood. Option 4 is incorrect because Piaget's theory does not use the term
"illogical thought processes." To describe any period of normal growth and development.

1.‐ Impaired cognition


2.‐ Concrete operational thinking
3.‐ Formal operational thinking
4.‐ Immature thought processes

4369 The parents of a 10‐year‐old take the child to the Correct answer: 2 Option 2 is correct. Children at 10 years of age are egocentric and concerned with Think of known 10‐year‐olds. How did they respond to adults?
mental health clinic. The nurse establishes rapport and themselves. Focusing on behavioral symptoms could lead to an adversarial relationship.
credibility with the child by asking the child about Children often are uncomfortable talking about friends and family until they get to know a
which of the following? person better. Most children are unconcerned about past medical problems; they are focused
on the "here and now."
1.‐ Behavioral symptoms
2.‐ Hobbies and interests
3.‐ Relationships with friends and family members.
4.‐ Medical problems in the past.

4370 The nurse is caring for a 4‐year‐old. To elicit Correct answer: 2 In Piaget's concept of preoperational thinking the child can use symbols to represent objects Note the age of the child. Recall the main age related concepts of the different theorists.
information about the child's feelings, the nurse offers and has the ability to pretend. The child does not think logically. Sullivan describes dynamisms
the child a series of pictures showing facial expressions (option 1) as habits that highlight personality traits. Freud’s theory regarding defense
and asks the child to point to the picture that shows mechanisms of the ego (option 3) is that they are unconsciously motivated ways of dealing
the child's own feelings. The nurse bases these actions with anxiety. According to Erickson's theory (option 4), these developmental tasks are
on which developmental concept? appropriate to middle childhood (i.e., 6–11 years).

1.‐ Sullivan's concept of dynamism


2.‐ Piaget's concept of preoperational thinking
3.‐ Freud's concept of mechanisms of the ego
4.‐ Erickson's concept of industry versus inferiority

4371 The nurse is providing community education about Correct answer: 4 Children with autistic disorders are highly indifferent to shows of affection by anyone and do Recall the basic characteristics of autism, particularly the ones that relate to their social
autism to a group of parents. The nurse concludes that not relate well with others. Autistic children's play is generally ritualistic and repetitive, rather skills, interest in others, and ability to connect with others.
teaching has been effective if the parents describe than creative and imaginative (option 1) and involves inanimate objects rather than people.
which of the following as common behavioral signs of Rather than having early speech development many autistic children have delayed
autism? development of language and mental retardation (option 2). Children with autistic disorders
are not overly affectionate (option 4). They instead are remote and uninvolved emotionally,
usually resisting shows of affection by the parents.

1.‐ Highly creative, imaginative play


2.‐ Early development of language
3.‐ Overly affectionate behavior toward parents
4.‐ Indifference to being held or hugged.

4372 Which of the following is the highest priority Correct answer: 2 The primary focus of treating anxiety disorders in children is to decrease fear and anxiety. The Consider the primary affect associated with phobias. Look for an option that would
intervention for the nurse who is working with a child most useful techniques for reducing phobia and the anxiety and fear associated with them are minimize that affect.
with a phobia? desensitization, reciprocal inhibition, and cognitive restructuring. These techniques can be
used with any age group. Having the child face his or her fear (option 1) is often unrealistic
because of the developmental level of the child. Children are easily overwhelmed when forced
to face fears directly. The primary focus of treating anxiety disorders in children is to decrease
fear and anxiety. Decreasing the fear, not protecting from fear, is the aim of treatment (option
3). Allowing the child to express his or her fears may be useful (option 4) but does not
necessarily lead to decreased anxiety or fears.

1.‐ Have the child face his or her fear


2.‐ Decrease fear and anxiety
3.‐ Protect the child from fears
4.‐ Allow the child to express fears

4373 The nurse is conducting a community education Correct answer: 1, 3, 4, The correct ranking is: 1. Accidents; 2. AIDS; 3. Homicides; 4. Suicide. For people 15–24 years Rank in the order of frequency, beginning with 1 as the most frequent. Specific knowledge
session about preventing deaths in adolescents. Place 2 of age, suicide is the third leading cause of deaths, behind accidents and homicide. More is needed to answer the question.
in order from most frequent to least frequent the American teenagers and young adults die from suicide each year than from heart disease,
causes of preventable adolescent deaths that the cancer, AIDS, birth defects, stroke, pneumonia, influenza, and chronic lung disease combined.
nurse needs to include in the presentation. Click and Because of their lack of maturation and the developmental issues they face, many adolescents
drag the options below to move them up or down. have a fatalistic perspective of the future and view suicide as the only option to manage their
pain or problems.
1.‐ Accidents
2.‐ AIDS
3.‐ Homicide
4.‐ Suicide
4374 The nurse is teaching a group of young adolescents Correct answer: 2 Anorexia nervosa occurs more often in young girls who perceive themselves to be grossly Disregard any options that are exclusive and include or infer the word only.
about eating disorders. The nurse would consider the overweight. Anorexia nervosa and other eating disorders are considered to be manifestations
sessions effective if the participants state that anorexia of underlying psychological issues, such as control, power, and denied sexuality. If untreated or
nervosa is best defined as an eating disorder that inadequately treated, complications of anorexia nervosa can lead to death. Anorexia occurs
occurs: most often in young girls, but increasing numbers of boys and adult women and men are
affected as well (option 1). Depression often coexists with anorexia, and antidepressants are
often given to the client who is anorexic. Anorexia nervosa occurs more often in girls whose
families are perfectionistic and rigid (option 3). If they develop an eating disorder, girls whose
families are more chaotic and impulsive tend to have bulimia nervosa. Anorexia nervosa is not
seen exclusively in young boys and girls (option 4). Increasing numbers of adult males and
females are developing this problem.

1.‐ Only in young girls who are depressed.


2.‐ Mainly in young girls who perceive themselves to be grossly overweight.
3.‐ Primarily in young girls who live in chaotic families.
4.‐ In young boys and girls alike.

4375 The nurse is conducting an in‐service education Correct answer: 2 Binging (excessive overeating) and purging (intentionally ridding one's body of food ingested) Mentally compare and contrast usual signs and symptoms of the eating disorders. Select
session about the relationship between anxiety and are characteristics of bulimia nervosa. These behaviors represent unhealthy attempts to cope the answer that is most specific to bulimia nervosa.
bulimia nervosa. The nurse best describes the and increase in frequency as anxiety increases. At the beginning of a bingeing episode, the
relationship by saying, "When the client has bulimia client loses all self‐control and ingests enormous quantities of food in a short period of time.
nervosa, an increase in the anxiety level will generally The purging activity temporarily—and falsely—restores the sense of control. Rigid control of
result in: what one eats is characteristic of anorexia nervosa (option 1). The person with bulimia actually
loses all self‐control when beginning a bingeing episode and subsequently ingests enormous
quantities of food. When the client has bulimia nervosa, overeating does occur, but it is
followed by purging, which is used as a relief behavior (option 3). Overeating in the absence of
purging is not considered to be a typical behavior associated with bulimia nervosa. Since the
client with an eating disorder is using food‐related behaviors as an unhealthy means of coping
with stress, the client is also likely to use other unhealthy coping methods, such as excessive
alcohol intake (option 4). However, bulimia nervosa is characterized by both bingeing and
purging involving food, not alcoholic beverages.

1.‐ Rigidly controlling what he or she eats."


2.‐ Binging and purging."
3.‐ Overeating."
4.‐ Consuming alcohol."

4376 When assessing an adolescent client for depression, it Correct answer: 2 Depression in adolescents is often masked by aggressive and/or behavioral problems. Consider normal or expected adolescent behavior. Ask yourself what relationship exists
is most important for the nurse to recognize that Symptoms are usually different from adults in that adolescents often exhibit intense mood between this developmental level and the way in which the adolescent experiences
depression in adolescents is often: swings, academic difficulties, antisocial behavior, and hypersomnia. While the DSM‐IV‐TR depression.
criteria for depression are the same for adults, children, and adolescents, the clinical
presentation may be different in the different age groups (option 1). Depression in adolescents
is often masked by aggressive and/or behavioral problems, including intense mood swings,
academic difficulties, antisocial behavior, and hypersomnia. Depression in adolescents can
have the same consequences as in adults and should be treated seriously (option 3). Family
dysfunction may or may not be present when the adolescent client is depressed (option 4). As
with adults who are depressed, there is evidence that depression in adolescence is highly
associated with psychobiologic changes, especially in neuroendocrine functioning.

1.‐ Similar in presentation to depression in adult clients.


2.‐ Masked by aggressive behaviors.
3.‐ Situational and not as serious as depression in adults.
4.‐ An indication of family dysfunction
4377 The school nurse is conducting an assessment to Correct answer: 1, 2, 3, Option 1 indicates that the client is experiencing amenorrhea. DSM‐IV‐TR criteria for anorexia Consider DSM‐IV‐TR criteria for anorexia nervosa. Apply them to the client's statements.
determine if a client has anorexia nervosa. Which 4 nervosa include the absence of at least three menstrual cycles when there is not another
statement(s) by the client will most suggest that the medical reason for this, including pregnancy. Option 2 indicates that the client has disturbed
client may indeed have anorexia nervosa? Select all body image and an unrealistic perception of own body appearance. A client suffering from
that apply. anorexia nervosa has a weight loss of 15 percent or greater of normal body weight because of
self‐imposed dietary restrictions and/or excessive exercise regimes. Even when dangerously
underweight and in physiologic peril, the anorexic client will continue to believe that more
weight should be lost. Option 3 indicates a serious interest in food, but not in eating it. Persons
with anorexia are indeed often preoccupied with food, but they refuse to allow it into their
bodies. It is not uncommon for them to prepare food for others but not to partake of it. Option
4 indicates a severe revulsion to food, which is common to persons who have anorexia
nervosa. It does not suggest that the client will self‐stimulate nausea, as would the bulimic
individual. Option 5 is not a correct statement. In addition to denying the reality of their
extreme thinness, clients with anorexia deny that they have an eating problem.

1.‐ "I don't have periods any more. I'm glad."


2.‐ “People say I'm skinny, but I'm fat and repulsive."
3.‐ "I want to be a chef and cook for other people."
4.‐ "The idea of eating makes me nauseated."
5.‐ "I know that I have a problem with eating."

4378 In order to be admitted to an inpatient treatment Correct answer: 120 Set this up as a ratio and proportion problem. <BR /> Review basic information about ratio and proportion. No equivalencies are necessary for
program, clients with anorexia nervosa must meet the this calculation, which is based on basic mathematical and algebraic principles.
admission criterion of having experienced at least a 30
percent weight loss over the past 6 months. The client
currently weighs 84 pounds. The nurse calculates that
6 months ago, this client weighed at least _____
pounds. Write in a numerical answer.

4379 The adolescent client is depressed. The client's Correct answer: 2 It is important for the nurse to answer this question in an accurate and factual way that the Review basic information about neurotransmitters and antidepressants.
ordered medication is fluoxetine (Prozac). Which of adolescent can understand. The nurse should know that fluoxetine (Prozac) is classified as a
the following is the best response by the nurse when selective serotonin reuptake inhibitor (SSRI) and that it is one of the antidepressants fully
the client says, "What will this medicine do inside my approved for treating children and adolescents. In option 1, the nurse is not responding to
brain?" what the client has asked. This is a non‐answer answer, and the adolescent is likely to feel
demeaned and belittled by it, feeling that the nurse is treating the client like a child. Unlike
tricyclic antidepressants, it is not associated with changes in norepinephrine levels (option 3)
and is not known to have a direct effect on blood glucose and dopamine levels (option 4).

1.‐ "It will help you feel less depressed."


2.‐ "It will regulate a neurotransmitter called serotonin."
3.‐ "It will raise your level of the brain hormone norepinephrine."
4.‐ "It will balance blood glucose and dopamine levels."
4380 The nurse is evaluating the progress of an adolescent Correct answer: 2 It is important for the client with an eating disorder to be able to connect emotion to the Recall that underlying psychological problems and emotions exist in clients with eating
bulimic client who is being treated as an outpatient. relief‐seeking behavior: bulimia, anorexia, or binging without purging. Completing such a diary disorders.
Which behavior would indicate that the client is in an honest manner will assist the client to recognize that the eating behavior is an unhealthy
making positive progress? The client: attempt to deal with uncomfortable feelings. Individuals with eating disorders are often
preoccupied with food and its nutritional content, although they do not eat normally. The
client's remaining focus of attention in this area does not suggest that therapeutic progress is
being made (option 1). Persons with bulimia generally isolate themselves from others before,
during, and after eating. They eat in secret, ingest unusually large quantities of food while
alone, and purge themselves as soon as possible after the meal (options 3 and 4). One of the
means of discouraging purging is to have the individual remain in the presence of others for at
least an hour after eating. Also, the client is an adolescent and should continue to function as a
member of the family, even if family discord is present (which is often the case in families with
bulimic members).

1.‐ Asks the nurse many details about the nutritional content of foods.
2.‐ Shows the nurse a completed food and emotion diary.
3.‐ Reports enjoying spending time alone after meals.
4.‐ Describes eating at times other than when the family members eat.

4381 A 13‐year‐old child is brought to the clinic with a Correct answer: 2 Option 2 is correct. Conduct problems are considered manifestations of acting‐out behaviors. Look carefully at the options. Notice that limit setting is the only one that deals with
history of conduct disorder. The nursing history reveals Inconsistent limit setting with very harsh discipline is often characteristic of families with behaviors, or conduct.
several facts about the family. Which one is most likely children suffering from conduct disorders. Imposing high expectations on the child may cause
to have contributed to the child's conduct problems? the child to be anxious, but this parental behavior is not generally thought to be directly
The parents: related to conduct disorders (option 1). Being excessively involved in the child's life (option 3)
may indeed make the child anxious, but it is not directly related to conduct disorders. Conduct
disorders occur in one‐child and multichild families (option 4).

1.‐ Have very high expectations of the child.


2.‐ Employ harsh discipline and inconsistent limit setting.
3.‐ Are excessively involved in the everyday life of the child.
4.‐ Have no other children.

4382 The parent of a child with attention‐deficit Correct answer: 4 The child with ADHD has difficulty concentrating and maintaining a focus. Behavior that is Look exactly at the parent's statement. The child doesn’t follow instructions. Ask yourself
hyperactivity disorder (ADHD) tells the nurse that the seen as resistant may actually result from the child's not having properly understood what has if any of the other options show a relationship to that behavior.
child does not follow instructions well. Which strategy been said. If the child repeats what was heard, the parent will be able to know if the intended
should the nurse recommend to the parent? message was actually received. If not, the message can be sent again in simple, concrete
language. It is also helpful for the parent to know that giving only one instruction at a time is
likely to be more effective than giving a complex set of instructions at the same time.<BR />

1.‐ "Teach your child to be less aggressive and more assertive."


2.‐ "Consider developing a predictable daily routine."
3.‐ "It could be helpful to assign time out if instructions aren't followed."
4.‐ "Try having your child repeat what was said before starting the task."

4383 Which primary interventions should the nurse plan Correct answer: 1 Option 1 is correct. Behavior modification is an effective strategy with children and Notice that the client has conduct disorder and that specific problematic behaviors are
for when a child has conduct disorder and is impulsive adolescents. The child is told exactly what behaviors are expected, what is not acceptable, and cited. Then select the option that is most directly related to changing behavior.
and aggressive? the consequences for specific undesirable behaviors These strategies use limit setting and
require consistency for correct implementation. Open communication is effective, but a
flexible approach to acceptable behavior may be confusing to the child (option 2). Open
expression of feelings and assertiveness training (options 3 and 4) are useful techniques;
however, they are more effective within a controlled environment and will not necessarily
address impulsive and aggressive behavior.

1.‐ Limit setting and consistency


2.‐ Open communication and a flexible approach
3.‐ Open expression of feelings
4.‐ Assertiveness training

4384 The school nurse is teaching parents of 6‐ and 7‐year‐ Correct answer: 3 Separation anxiety disorder may develop at any age, although it is most common in children, Recall statistics about normal growth and development and mental health disorders in
old students about anxiety disorders in early school‐ with the peak onset between 7 and 9 years old. When it does occur, the child generally has children.
aged children. Which disorder should the nurse performance and attendance difficulties in school. Obsessive‐compulsive disorder (option 1) is
emphasize in the discussion? not common in children, although obsessive compulsive personality traits may be present in
children this age. While the incidence of depression in children is increasing (option 2), it is not
as common a problem in this age group as is separation anxiety. Posttraumatic stress disorder
(option 4) can occur in persons of any age, but it is not as common as separation anxiety. As
with adults, PTSD in children occurs following some intensely emotionally painful event, which
is not an average occurrence for children. However, separation anxiety is considered more
common.

1.‐ Obsessive‐compulsive disorder (OCD)


2.‐ Depression
3.‐ Separation anxiety disorder
4.‐ Posttraumatic stress disorder (PTSD)

4385 When planning the care of a 6‐year‐child with Correct answer: 3 Behavior modification is quite effective with young children and adolescents. The child is told Notice that this child's problem is related to conduct, or inappropriate behaviors. Choose
oppositional defiant disorder, the psychiatric nurse what is expected, what is not acceptable, and what the consequences will be for undesirable the option that is expected to have the most immediate effect on modifying behavior.
should include which of the following? behavior. The child is rewarded for changes in behavior. Reminiscence therapy (option 1) is
indicated for clients who have memory deficits. It is often used with the geriatric population,
particularly individuals with dementia. Emotive therapy (option 2) is more effective in older
children. The young child with oppositional defiant behavior is often very emotive, but in an
inappropriate way. While cognitive reframing (option 4) can be used with young children, the
first approach should be to modify the behavior. Once the child is behaving more acceptably,
other interventions such as cognitive reframing can be used.

1.‐ Reminiscence therapy


2.‐ Emotive therapy
3.‐ Behavior modification
4.‐ Cognitive reframing

4386 A 3‐year‐old client has been diagnosed with attention‐ Correct answer: 3 Option 3 is correct. Central nervous system stimulants such as methylphenidate (Ritalin) and Consider the classifications of each drug. Recall that children often have paradoxical
deficit/hyperactivity disorder (ADHD). The child's amphetamine and dextroamphetamine (Adderal) are the most frequently used medications for responses to drugs. Think about recent controversies in the public media about giving
parents report that a friend told them that the child ADHD. These medications increase the ability to focus attention by blocking out irrelevant stimulant drugs to children.
will likely receive "lots of drugs." The nurse should thoughts and impulses. Antidepressants (options 1 and 2) are not typically prescribed for the
reply that the child will most likely be given a drug such treatment of ADHD. When they are used for depression, venlafaxine (Effexor) and fluoxetine
as: (Prozac) seem to be the most effective. Haloperidol (Haldol) in option 4 is an antipsychotic
drug that is useful in treating children and adolescents with Tourette's syndrome. It is not
commonly prescribed for clients with ADHD. When it is, the nurse must be particularly
observant for the occurrence of extrapyramidal side effects (EPS).

1.‐ Amitriptyline (Elavil).


2.‐ Paroxetine (Paxil).
3.‐ Amphetamine and dextroamphetamine (Adderal).
4.‐ Haloperidol (Haldol).
4387 The nurse employs play therapy with a small group of Correct answer: 1 Play therapy is especially useful for children under 12 because their developmental level Recall developmentally appropriate behaviors of 6‐year‐old children. Think perhaps about
6‐year‐old clients. The primary expected outcome is makes them less able to verbalize thoughts and feelings. Learning to talk openly about 6‐year‐old children you have known or worked with in clinical settings.
for the clients to do which of the following? themselves (option 2), learning how to give and receive feedback (option 3), and learning
problem‐solving skills (option 4) are not the intended goals of play therapy. Options 2, 3, and 4
require more structured group and individual activities than 6‐year‐olds are able to master.
Play therapy provides an opportunity for children to express their feelings through play,
without the need for advanced verbal or social skill sets.

1.‐ Act out feelings in a constructive manner.


2.‐ Learn to talk openly about themselves.
3.‐ Learn how to give and receive feedback.
4.‐ Learn problem‐solving skills.

4388 The parent of a child recently diagnosed with Correct answer: 3, 4 Options 3 and 4 are correct. Recall that children with ODD are disruptive, argumentative, Recall behaviors associated with ODD. Look for behaviors that are oppositional, but not
oppositional defiant disorder (ODD) asks the nurse to hostile, and irritable. These children also have social problems with peers and adults and have dangerous or antisocial.
explain what behaviors are associated with the impaired academic functioning. Stealing (option 2), along with cruelty (option 1) and arson
condition. Which information should the nurse include (option 5), are included in the antisocial behaviors seen in children with CD. On the other hand,
in an answer. Select all that apply. children with ODD are primarily disruptive, argumentative, hostile, and irritable.

1.‐ Cruelty
2.‐ Stealing
3.‐ Argumentativeness
4.‐ Irritability
5.‐ Arson

4389 The school nurse is planning a community education Correct answer: 3 Childhood disorders that appear to be genetically transmitted include enuresis, autism, Notice that autism is a more pervasive problem than the other three options. Recall
program about childhood mental health problems that mental retardation, some language disorders, Tourette's syndrome, and attention‐deficit statistics about childhood mental health disorders to make a selection.
appear to be genetically transmitted. While conducting hyperactivity disorder (ADHD). Of these, autism is the more pervasive disorder. Anxiety (option
the program, the nurse will emphasize information 1), sleepwalking (option 2), and oppositional defiance disorder (option 4) do not appear to be
about which problem? genetically transmitted.

1.‐ Anxiety states


2.‐ Sleepwalking
3.‐ Enuresis
4.‐ Oppositional defiance disorder (ODD)

4390 The client being seen in the out‐patient clinic has Correct answer: 3 Option 3 is correct. This will allow for collection of base‐line data that can be used for Choose an option that allows for more data collection before a specific plan of action is
been taking olanzapine (Zyprexa) for one month and developing a weight control/loss program. Option 1 is incorrect as weighing daily can be developed.
has experienced a 12‐pound weight gain during that discouraging, as within the course of a week, several normal fluctuations in weight might
time. When the client expresses an interest in occur. Additionally, reporting to the clinic daily (if the client has not already been doing so) is
preventing further weight gain, the first action of the unnecessary and would add to both expense and stress levels of the client. Options 2 and 4 are
nurse should make which recommendation to the also incorrect. It is premature to enroll in a formal weight management program or see a
client? nutritionist at this time. The nurse should assist the client to establish baseline data before
seeing other professionals.
1.‐ Report to the clinic for daily weights
2.‐ Enroll in a weight management program
3.‐ Keep a careful record of all food intake for one week
4.‐ Make an appointment with a nutritionist
4391 The client diagnosed with bipolar I disorder is in an Correct answer: 2 Option 2 is correct. Redirecting the client in a calm, firm, non‐defensive manner is the most Recall that persons with elevated mood are distractible, as well as irritable and hostile.
inpatient locked unit. The client begins to yell loudly at appropriate action to begin de‐escalation. This client will be distractible but irritable, so it is
another client who is also sitting in the dayroom. In important that the nurse’s approach is one of quiet, matter‐of‐fact calmness. Options 1, 2, and
order to provide a safe environment for both clients, 3 are incorrect. Turning on the television is not an appropriate approach because it is not likely
the nurse should take which action? to distract the client. The nurse should personally gain the client's attention and attempt to
verbally redirect the client. If the client with bipolar disorder is in a locked unit, prn orders
should already be in place. At this time, the client's behavior requires the nurse's personal
attention and intervention, not medication. In fact, if medication is given before other less
restrictive interventions are used, it is illegal. Medication is considered a form of chemical
restraint.<BR />

1.‐ Turn on the television in the dayroom to distract the client.


2.‐ Redirect the client in a calm, firm, non‐defensive manner.
3.‐ Call the physician for a prn medication order for the client who is escalating.
4.‐ Escort the client to the seclusion room.

4392 The community mental health client says, "I'm afraid Correct answer: 2 Option 2 is correct. The nurse should recognize that this client is describing feelings which Choose an option that would be most protective for the client. Remember that discussing
something is wrong with me. I don't have any appetite. could result in suicide. Even though the client has not directly described feeling self suicide will not increase the likelihood of a person acting on suicidal impulses, nor will it
I don't get much sleep and some days I don't want to destructive, the nurse should recognize that there is a strong likelihood that these feelings are create suicidal impulses in the non‐suicidal person.
be alive." What is the most important first assessment present but unacknowledged. The nurse should directly and kindly inquire whether such
by the nurse? thoughts are present and provide safety measures as appropriate to the client’s response.
Having a suicidal plan and the means to carry out the plan increase the potential lethality of
any situation with a client in suicidal crisis. Option 1 is incorrect because while the client is
describing disturbance in physiologic functioning that is consistent with the mood state of
depression, the nurse should know that assessing for self destructive intent is of higher priority
than assessing basic physical needs. Option 3 is incorrect as completing the Beck's Depression
Scale is not an urgent priority. Information gained from the test will contribute to planning
appropriate care, but the test can be delayed. The priority action of the nurse should be to
determine if the client is suicidal. Option 4 is incorrect because putting the client in a quiet
room is contra indicated because the client needs to be in the staff's eyesight until suicidal
ideation is confirmed or eliminated. Additionally, the client has begun to express feelings of
depression and appears ready to discuss it. The nurse must be responsive to the client's
emotional state.

1.‐ Ask the client specific questions about appetite and sleep patterns
2.‐ Establish the client's level of suicidality
3.‐ Complete a Beck's Depression Scale with the client
4.‐ Place the client in a quiet room

4393 The client who takes fluoxetine (Prozac) 20 mg each Correct answer: 2 Option 2 is correct. Helping the client to avoid large amounts of caffeine could help improve Notice that the evening drinks of the client contain caffeine and correlate that with the
morning complains of daytime fatigue and being able his sleep. Side effects of caffeine, particularly large doses or doses taken after noon, include limitation of caffeine in option 2.
to sleep only five hours per night. Describing feeling insomnia, irritability and anxiety. Option 1 is incorrect as Fluoxetine (Prozac) should not be
less depressed than previously, the client reports given at bedtime because it impairs sleep. Option 3 is incorrect because decreasing the
feeling bored and anxious after work, even after amount of time with friends (unless the time spent is unreasonably long) would not be
drinking coffee or colas during in the evening while therapeutic for the client, since the client is being treated for depression. Withdrawal from
socializing with friends. What recommendation should social contacts is one of the symptoms of depression, so if the client is experiencing
the nurse make to the client? satisfaction from being with friends, this indicates improvement. Option 4 is incorrect as for
most people, napping late in the day interferes with successful nighttime sleeping. Avoiding
this is a part of good sleep hygiene for all persons.

1.‐ Begin taking fluoxetine at bedtime


2.‐ Limit caffeine amount and restrict to morning hours
3.‐ Return home earlier from social activities with friends
4.‐ Begin taking a short nap before meeting friends
4394 The client has been taking lithium carbonate (Lithium) Correct answer: 4 Option 4 is correct. This client is within the therapeutic blood level for clients newly begin on Recall therapeutic blood levels of Lithium, both during the initiation and maintenance
300 mg by mouth for six days. The serum lithium level Lithium (0.8 mg/% to 1.4 mg/%) and is displaying two of the commonly expected side effects of phases of therapy.
is 1.10 mg/%. In addition to noticing minor Lithium.<BR />
tremulousness in the client's hands, the nurse also
observes the client drinking water frequently. What
conclusion should the nurse reach? The client is:

1.‐ Showing probable signs of lithium toxicity while at non‐therapeutic lithium level.
2.‐ Experiencing early renal impairment associated with side effects of lithium.
3.‐ Manifesting behaviors that suggest early return of manic symptoms.
4.‐ Experiencing normal side effects while at therapeutic blood level.

4395 The client is being discharged after a suicide attempt. Correct answer: 3 Option 3 is correct. The family needs the nurse to listen and support them. Reinforcing signs Recognize that both survivors of suicidal attempts and their family members should be
Discharge plans include medications and participation and symptoms of relapse will help the family members decrease their anxiety, knowing they provided assistance and support. Remember also that once a person has made a suicide
in a day treatment program, as well as returning to live have a clear role in the recovery and ongoing care of the client without being totally attempt, they are statistically at increased risk for suicide at all points in the future.
with family members. When the family members tell responsible for the client. In option 1, the family needs to express their concerns, not be asked
the nurse that they are concerned about the client's to identify what will make them feel better. Option 2 does not address the family concerns and
safety, what is the most appropriate response by the attempts to give them false reassurance. Option 4 is an unreasonable recommendation that
nurse? would place undue pressure on the family. The family and the client should be assisted to
formulate safety plans that will allow all to carry out usual responsibilities.

1.‐ Listen to the family and ask them how the nurse can make them feel less concerned.
2.‐ Reassure the family that the day hospital program will provide the client with emergency telephone numbers.
3.‐ Allow the family to voice concerns and teach them the indications of possible relapse.
4.‐ Recommend developing a planned schedule so that a family member is always with the client.

4396 The nurse is preparing to take a client to the Correct answer: 3 Option 3 is correct. The client should be NPO before the procedure in order to be given Recall the similarity between the pre‐surgical and pre‐ECT procedures.
electroconvulsive therapy (ECT) treatment suite. The anesthesia for the procedure. In option 1, the client, not the spouse, should sign the consent
nurse must ensure that which pre‐treatment process is form. The client should be wearing loose‐fitting clothing that would not restrict movements of
completed before the treatment can be administered? breathing (option 2). In option 4, the client should be NPO before the procedure in order to be
given anesthesia.
1.‐ The client's spouse has signed the consent form.
2.‐ The client is wearing snug‐fitting clothing.
3.‐ The client is NPO.
4.‐ The client has been given a light meal before the procedure.

4397 The nurse is assigned to the care of a client who has Correct answer: 2 Option 2 is correct. For clients with dysthymia, major concerns are chronic dissatisfaction and Recall usual behaviors, particularly psychosocial ones, associated with dysthymic disorder.
dysthymic disorder. What is the most important social isolation. Additionally, they may have changes in appetite, although this is not as
nursing intervention that the nurse should include in common as it is in persons with major depression. Option 1 is incorrect because it would
the client's plan of care? encourage or perpetuate continuance of social withdrawal. The client will feel tired and may
resist being more active, but increased physical activity can help to combat feelings of
depression and any accompanying feelings of anxiety.<BR />

1.‐ Provide at least 2 hours of quiet time every morning for the client.
2.‐ Encourage the client to eat in the main dining room with other clients.
3.‐ Teach the client about the expected effects of second generation antispychotics.
4.‐ Observe the client carefully for indications of "spacing out" into another personality.
4398 The in‐patient client has bipolar I mood disorder and Correct answer: 3 Option 3 is correct. The client's level of risk for self‐harm is a major concern. While the mood Remember that bipolar disorders are characterized by unpredictable and cyclic changes in
is in a manic state. Which of the following is most is elevated, the client may injure self from restless hyperactivity or poor judgment. mood, with resultant changes in behavior.
important for the nurse to consider when planning Additionally, the client may experience unpredictable mood swings and act on accompanying
care to address the client's safety needs? The client suicidal urges. Options 1, 2, and 4 each describe a common client presentation in states of
may: elevated affect. However, none of them are as important in regard to safety as is the likelihood
of rapid mood swings.
1.‐ Experience increased stimulation while in the presence of others.
2.‐ Express frequent verbal hostility and harsh sarcasm while mood is elevated.
3.‐ React impulsively to self‐destructive feelings during a mood shift.
4.‐ Exhibit hypersexuality and socially inappropriate behavior.

4399 The nurse is conducting an educational group on an Correct answer: 1 Option 1 is correct. This is a respectful response of the nurse. It shows that the nurse Note that this client is a new member of the group. Imagine how you would feel in this
inpatient psychiatric unit. One client has not spoken recognizes that during this initial phase of the group process, the client may be (1) being situation.
during the group. What would be the most effective observant in order to decide whether he/she feels safe in the group or (2) remaining silent
therapeutic response of the nurse? because of feelings of inadequacy.<BR />

1.‐ Allow the client to remain present but non‐participative.


2.‐ Explain to the client that everyone in the group needs to participate.
3.‐ Ask the rest of the group members how they feel about this member not sharing.
4.‐ Stop the group and ask the client to leave.

4400 The inpatient mental health client is being treated for Correct answer: 1, 3 The nurse should recognize that this client is experiencing psychomotor retardation, which, in Recall physiologic manifestations of depression and anticholinergic side effects.
major depression. The client moves very slowly, speaks addition to slowing voluntary motor activities, will also slow other bodily processes. The
very little, and is extremely inactive physically, walking anticholinergic effects of the client’s medication will decrease motility of the GI tract,
only to the bathroom and back. The client takes an specifically the stomach and bowel. The nurse should remember that the client's low level of
antidepressant that causes anticholinergic side effects. physical activity increases the risk for constipation. If the client's constipation is not managed
The nurse should conclude that this client is at adequately, other serious problems of fecal impaction and bowel obstruction may occur. There
particular risk for developing which of the following? is nothing to indicate that the client is at high risk for vomiting (option 2), although the other
Select all that apply. GI symptoms noted in option 1 and 3 are likely. Option 4. The nurse should recognize that this
client not at risk for diarrhea (option 4), but is actually at high risk for constipation. Most
experts consider weight gain (a common side effect of antipsychotics) to be an anticholinergic
side effect (option 5).

1.‐ Digestive difficulties


2.‐ Vomiting
3.‐ Constipation
4.‐ Diarrhea
5.‐ Weight loss

4401 The 26‐year‐old female hospitalized client is being Correct answer: 1 The client is demonstrating a pattern of behavior that should be investigated. The nurse Notice that the client has not expressed any reasons for wanting to return to own room.
treated for major depression. The client participated should take the time to assess the client’s feelings, thoughts, and actions. The client may just This indicates a need for more information.
actively in group therapy during the hour before be tired and have a need to rest, but the nurse needs to be sure that the client is safe and not
lunchtime. When it is time for lunch, the client tells the upset. The nurse should not make assumptions about what the client is feeling (option 2).
nurse, “I’m not going. I’m going to my room.” The Before ascribing a meaning to the client’s behavior, the nurse should first talk to the client to
nurse’s best response would be to: determine what feelings she is experiencing. Additionally, the nurse is using a closed‐ended
question that can be answered with a “yes” or “no.” This is not the correct therapeutic
technique to use when interviewing a client. The response in option 3 does acknowledge the
client’s change in attitude but does not indicate that the nurse has any concern for the client.
The response in option 4 changes the focus from the client to food in addition to prematurely
and inappropriately presuming to understand the meaning of the client’s behavior.

1.‐ Ask the client to sit for a few minutes to discuss this.
2.‐ Ask the client if she is angry.
3.‐ Tell the client that there is a unit schedule that must be followed by everyone.
4.‐ Ask the client if there is a problem with the food.

4402 The nurse is conducting discharge teaching for a client Correct answer: 2 Salami is a cured meat and must be avoided by clients taking tranylcypromine, a monoamine Notice that salami is the only food listed that is prepared for a long‐term shelf‐life.
taking tranylcypromine (Parnate). The nurse oxidase inhibitor (MAOI). Foods rich in tyramine or tryptophan, such as cured foods, may
determines that the client understands the induce a hypertensive episode in clients taking MAOI medication. Other foods to be avoided
instructions given if the client says, "While I take this include those that have been aged, pickled, fermented, or smoked. Clients taking monoamine
medicine, I should not eat: oxidase inhibitors (MAOIs) can eat potatoes (option 1), baked chicken option 3), and cottage
cheese in reasonable amounts (option 4).

1.‐ Potatoes."
2.‐ Salami."
3.‐ Baked chicken."
4.‐ Cottage cheese."

4403 A client is admitted to a secure psychiatric inpatient Correct answer: 3 Providing safety for the nurse and the client is the primary concern immediately after Notice the aggression of the client and recognize its possible danger to client and others.
unit for the treatment of bipolar I disorder. The nurse admission when the client is in a manic state. This is because the client is likely to be labile,
begins the intake assessment but the client stands up hostile, and uncooperative. The information given in this question suggests that the client's
and begins to walk around the room and shouts, "You elevated and angry mood poses potential safety hazards for the client and the nurse. While
can’t do this to me! Do you know who I am? I want out obtaining the intake assessment data is important (option 1), this activity can be delayed until
of here!!" The best action of the nurse at this time safety issues have been addressed. When the client demonstrates an elevated angry mood, it
focuses on: poses a potential safety risk for the client and the nurse. It is this risk that requires the nurse’s
immediate focus, not nutritional imbalance (option 2), even though this is often a nursing
diagnosis for a client in a manic episode. There is nothing in the question that indicates that
the client’s behavior is so extreme that medications must be given (option 4). The nurse should
remember that prn neuroleptic medication is considered to be a chemical form of restraint.
Other less restrictive measures, such as environmental manipulation, must always be used
before the nurse administers prn medication.

1.‐ Obtaining the assessment information from the client.


2.‐ Providing the client with adequate food and fluids to maintain homeostasis.
3.‐ Providing client and self with a safe environment.
4.‐ Administering the prescribed prn neuroleptic medication to prevent escalation of behavior.

4404 The nurse observed that earlier in the afternoon a Correct answer: 2 It acknowledges independent and positive actions by the client. Information is reported in a Notice that the question is asking about feedback and that the client is depressed.
depressed client visited the coffee shop and sat at a matter‐of‐fact manner to the client, and no evaluative words are used. While option 1 is Integrate your knowledge of each as you make your choice.
table with two other clients. What is the best feedback probably intended to convey encouragement, it is very much overstated. Depressed clients do
the nurse can give to the client? not accept praise easily. The appropriate feedback for the nurse to provide is a matter‐of‐fact
observation that does not include evaluative terms like "wonderful." Options 3 and 4 use of
the technique of exploring. The question that is being asked concerns feedback. Feedback
involves the giving of information to the client, rather than getting information from the client.
Questions are not asked when feedback is given.

1.‐ "You are doing such a wonderful job interacting with the other clients."
2.‐ "I saw that you sat with others in the coffee shop this afternoon."
3.‐ "How are you feeling after your visit to the coffee shop with other clients today?"
4.‐ "Do you plan to go to the coffee shop again tomorrow?"
4405 The hospitalized client is in a manic phase of bipolar I Correct answer: 1, 3, 4 Providing nursing care to clients with elevated mood (mania) can be particularly challenging Develop a mental image of the client with elevated mood. As you read an option, "see"
disorder. When developing the nursing care plan for for the nurse. The client will generally be excited, physically hyperactive, labile, and the client. Will he or she be likely to carry out this behavior?
this client, the nurse should anticipate that in social unpredictable. Clients in manic states tend to be exhibit behaviors that are controlling,
interactions, this client's behavior is very likely to be: competitive, irritable, aggressive, and domineering in social situations. They are often socially
(select all that apply) intrusive and inappropriate. When their demands are not met, they can easily become
aggressive in ways that are dangerous to self or others. Because of this, the nurse should
always consider the person in a manic state to be at risk for injury to self or others especially.
Persons experiencing manic states will resist being alone and act as if they feel compelled to
interact with others at all times (option 2). Persons in a manic state have unrealistically
elevated self‐esteem, often feeling that they are the brightest, the wisest, or the most
knowledgeable person in the world (option 5). They will therefore not hesitate to make
decisions, but the decisions are made impulsively and without regard for consequences.

1.‐ Unpredictable.
2.‐ Isolative.
3.‐ Demanding.
4.‐ Competitive.
5.‐ Indecisive.

4406 The nurse observes that a client is pacing in the Correct answer: 3 Psychomotor agitation is recognized when a person’s behavior involves increased physical Look for a behavior, not an emotion.
hallway, talking rapidly, and gesturing dramatically. activity, restless, and/or aggression. The behavioral manifestations will be accompanied by
The nurse concludes that the client is beginning to strong affects, such as anxiety, and speeding of physiologic processes. When psychomotor
demonstrate what kind of behavior? agitation is present, the client and others are at risk for injury. Psychomotor retardation
(option 1) is a term that refers to slowing of physical activities and bodily processes. It is the
opposite of what is described in this situation. Anxiety is a feeling, not a behavior (option 2).
This question calls for the nurse to identify a behavior. The behavior of the client may be
indeed be related to anxiety, but there are other possible reasons for the behavior. Depression
is a mood state, not a behavior (option 4). The behavior of the client may be indeed be related
to depressed mood, but there are other possible reasons for the behavior.

1.‐ Psychomotor retardation


2.‐ Anxiety
3.‐ Psychomotor agitation
4.‐ Depression

4407 A client states, "I just want to sleep all the time. I am Correct answer: 1 The client is describing symptoms that are consistent with those of a mood disorder, probably Ask yourself if this client is managing life very well.
overweight again. I will go to work and do my grocery dysthymic disorder. The client's life is not well‐managed, nor does the client experience
shopping, but that's all. My life's a mess." The nurse pleasure. There is nothing in the client's statement to indicate an immediate risk for violence
should conclude that which nursing diagnosis is most (option 2). The client is dispirited and dissatisfied, but does not indicate that this is
relevant? overwhelming enough to cause self‐destructive ideas or urges. The client's statement does not
suggest inability to tolerate physical activity (option 3), but rather fatigue and disinterest.
Activity intolerance is much more intense and specific than fatigue. Nothing in the client's
statement suggests that acute anxiety is present (option 4). Anxiety is a common human
emotion, but in order for it to be a nursing diagnosis, there should be clear evidence that the
anxiety level is elevated beyond normal.

1.‐ Ineffective coping


2.‐ Risk for violence: self‐directed
3.‐ Activity intolerance
4.‐ Anxiety
4408 The client who has a diagnosis of bipolar I disorder Correct answer: 2 The nurse should know that carbamazepine (Tegretol) frequently causes changes in liver Read the question carefully and draw on knowledge of pharmacology and common
has a new order for carbamazepine (Tegretol). Before enzymes. These changes can result in dangerous or fatal problems for the client. Therefore, adverse drug effects.
beginning to administer the medication, the nurse baseline laboratory results must be available before the first dose of the drug is administered.
checks to see that which laboratory results are in the Subsequent results are then compared to this set of baseline data as the prescriber makes
client’s record? decisions about future doses. Changes in blood glucose (option 1) are not commonly
associated with carbamazepine. Instead, there is a strong relationship to blood glucose
changes and certain second generation antipsychotics, such as olanzepine (Zyprexa). While it is
true that bleeding problems can be associated with carbamazepine (option 3), baseline
bleed/clotting times are not required before the drug is administered. If the client were being
given lithium (option 4), baseline information about thyroid functioning would be required.

1.‐ Blood glucose


2.‐ Liver function studies
3.‐ Bleeding and clotting time
4.‐ Thyroid profile

4409 A client in an inpatient unit is awake at 1 a.m. and Correct answer: 3 The client has indicated that environmental noise and activity are preventing sleep. The nurse Pay attention to the client's words. They define the exact nature of the problem.
tells the nurse, "I can't sleep because of the light in the should first attempt to minimize environmental stimuli. Simply closing the client's door is a
hall and the noise from the kitchen. I need to have noninvasive, nonstimulating strategy that may work, assuming that it will not pose a safety
another sleeping pill." The most appropriate nursing hazard for the client. Before administering a prn sedative (option 1), the nurse should attempt
intervention is to: other nonpharmacological options. It is not necessary to move the client (option 2) when
closing the door can produce a noninvasive strategy. However, the door should not be closed if
this would pose a risk for the client's safety. Turning on the television (option 4) will increase
the amount of noise in the environment and could further stimulate the client and/or others.

1.‐ Administer a prn sedative.


2.‐ Move the client to a quieter room.
3.‐ Close the door to the client's room.
4.‐ Allow the client to watch television for 1 hour.

4410 The client is scheduled for electroconvulsive therapy Correct answer: 2, 5 The client receiving a series of ECTs can be expected to have "patchy" memories of events Recall information about expected side effects of ECT. Present this information to the
(ECT). When teaching the client about what to expect occurring during the days or weeks of the treatment period (option 2). This may or may not client in a simple and nonalarming style.
in the post‐ECT period, which statement(s) should the resolve as time passes. While modifications in the ECT procedure have increased the level of
nurse make? Select all that apply. safety for the client, amnesia and possibly temporary confusion still remain as side effects of
the procedure (option 5). The client is likely to have amnesia for recent events, especially for
events that occurred just before the treatment was administered (option 1), so the client will
not remember that the treatment has been administered. The memory loss that may occur as
a side effect to ECT is not selective. Both positive and negative life events may be forgotten
(option 3). Memory deficits are common in the client receiving ECT. When they do occur, the
client and significant others generally feel alarmed. However, the nurse should recognize this
as an expected and nonurgent side effect (option 4).

1.‐ "You should expect that you will be able to remember recent events more clearly than you did before you started receiving ECT."
2.‐ "It may hard for you to remember everything that happens during the days and weeks you receive ECT."
3.‐ "It is common for persons who receive ECT to lose all painful memories of early life."
4.‐ "If you notice that you are having changes in your memory, let a staff member know immediately."
5.‐ "Even though modifications have been made in ECT over the years, you may have some disorientation briefly upon awakening from the procedure."
4411 The client has bipolar I disorder. Lithium carbonate Correct answer: 1 The nurse should recognize that the client is experiencing side effects that are normal at this Recall normal and expected side effects of Lithium. Choose a response that gives that
(Lithium) 300 mg 4 times daily has been prescribed. time. That information should be conveyed to the client, as well as information about what to information to the client in simple terms.
After 3 days of lithium therapy, the client says, "What's expect in the future. It is important that the client continue to take the medication as
wrong? My hands are shaking a little." The best prescribed. Serum lithium levels should be monitored frequently in order to determine
response of the nurse is: therapeutic blood levels and prevent lithium toxicity. Minor hand tremors do not indicate
lithium toxicity, but they can interfere with writing and other motor skills. Helping the client
understand that the tremors can subside or disappear after 1 or 2 weeks is reassuring. Option
2 fails to give the client important information about side effects of Lithium. It also fails to
respond to the client’s concern that something is wrong. Fine hand tremors are expected side
effects at this time in the client's treatment (option 3). However, the presence of coarse
tremors, coupled with such symptoms are gait changes, would suggest possible toxicity, which
would require a totally different response from the nurse. In option 4, the nurse is probably
trying to be reassuring but the client will likely feel demeaned. This response also deprives the
client of the opportunity to receive important teaching.

1.‐ "Minor hand trembling often happens for a few days after lithium is started. It usually stops in 1–2 weeks."
2.‐ "There's no reason to worry about that. We won't, unless it lasts longer than a couple of weeks."
3.‐ "Just in case your blood level is too high, I am not going to give you your next dose of Lithium."
4.‐ "I wouldn't worry about it if I were you. It's a small tremor that doesn’t interfere with your functioning."

4412 The client is being admitted to the inpatient Correct answer: 1, 4 The nurse should understand that in order for a client to be diagnosed with major depression, Compare and contrast usual presenting symptoms and DSM‐IV‐TR criteria for dysthymia
psychiatric unit with a diagnosis of major depression. DSM‐IV‐TR specifies that symptoms consistent with at least 5 of 9 criteria must have been and major depression.
During the initial nursing assessment, the nurse present for at least 2 weeks. Suicidal ideations and plans are included in one criterion (option
anticipates that the client will acknowledge which of 1). The nurse should keep in mind that the presence of suicidal ideations alone would not
the following? Select all that apply. support the diagnosis of major depression. Another criterion for major depression involves
markedly diminished interest or pleasure in all, or almost all, activities (option 4). Option 2 is
incorrect because major depressive symptoms represent a more recent change in functioning,
not a single episode within 2 years. While persons with major depression often have changes
in weight and appetite (option 3), the relevant DSM‐ IV‐TR criterion for major depression does
not involve a 3‐day period. It also allows for either increases or decreases in appetite to have
occurred. Hallucinations (option 5) may occur in psychotic levels of major depression, but they
are not part of the diagnostic criteria in DSM‐IV‐TR.

1.‐ Suicidal thoughts or plans of suicide over at least the past 2 weeks
2.‐ History of one depressive episode within the past 2 years
3.‐ Loss of appetite for approximately 3 days
4.‐ Loss of interest in previously enjoyed activities
5.‐ Presence of hallucinations for at least 3 days

4413 An client hospitalized with bipolar disorder is in a Correct answer: 3 The manic client has poor judgment and is impulsive and at risk for injury. The nurse should Ask yourself if it would be a good idea for this client to be able to leave the hospital at will.
state of mania. The client, who was admitted on a attend to the safety needs of this client before taking other actions. It is possible that the client Consider the obligation of the nurse to provide safety for all clients. Recall relevant facts
formal voluntary status, demands immediate discharge will not be able to contract for safety. (At this point the first part of the contract would be for about types of psychiatric admissions.
from the facility. The nurse will first: the client to remain in the hospital.) If the client could not do this, the nurse's next action is to
explain the terms of the client’s admission status. Because the client is being treated on a
formal voluntary basis, the nurse cannot comply with the client’s demand to be discharged. It
is not appropriate to involve the police (option 1). While informing the spouse might be an
appropriate later action, at this time the nurse should focus attention on the client. It is
appropriate for the nurse to report the client's request to the nursing supervisor, but the first
response to the client's request should be made to the client (option 4). Meeting the client's
safety needs is of higher priority than informing the supervisor.

1.‐ Notify the police of the client's intention.


2.‐ Inform the client's spouse of the request of the client.
3.‐ Offer the client a contract for safety.
4.‐ Notify the supervisor on the nursing unit.

4414 The nurse needs to teach a client about newly Correct answer: 1, 5 The nurse should know that one of the common side effects of sertraline (Zoloft) is insomnia. Translate the statements of the nurse into medical terminology and then determine if
prescribed sertraline (Zoloft). Which information is Therefore, most clients are given sertraline (Zoloft) early in the day. Sexual side effects to each option is related to a known side effect of sertraline (Zoloft). .
essential to include in the teaching? Select all that sertraline (Zoloft) and other SSRIs are common. One way to decrease the likelihood of
apply. noncompliance is to inform the client that prompt reporting of such side effects can lead to
corrective treatment measures. It is important for the nurse to know that diarrhea is a much
more common side effect to sertraline (Zoloft) than is constipation (option 2). It is vital for the
nurse to know that all persons taking sertraline (Zoloft) or other SSRIs should be taught to
recognize early symptoms of possible central serotonin syndrome (option 3). Such symptoms
include sudden onset fever, sweating, and extrapyramidal side effects (EPS). The development
of central serotonin syndrome is a rare medical emergency, but the client will need intensive
medical treatment because the mortality rate is very high. The nurse should teach the client
that while some reduction in symptoms may occur in a relatively short period of time, it may
be several weeks before full therapeutic effects are realized (option 4). Without this
knowledge, clients often become discouraged and think that the medication is ineffective.

1.‐ Sertraline is most often taken as a morning dose.


2.‐ Constipation is a common side effect of sertraline.
3.‐ Fever and flulike symptoms are bothersome but not dangerous side effects of sertraline.
4.‐ Clients taking sertraline will usually recognize improvement within one week.
5.‐ It is possible that sexual side effects will occur.

4415 A client who had coronary bypass surgery 6 days ago Correct answer: 3 The clinical presentation of depressed mood is similar to that of medically diagnosed Look carefully at the client's words. Think about clinical signs of depression. Remember
reports having no appetite and feeling very sad. The depression. There is a high incidence of depressed mood and depression among hospitalized that depression is a medical diagnosis, not a nursing diagnosis.
client further complains of having difficulty falling clients. Usually the more severe the illness, the more pronounced the symptoms. Clinical
asleep. The nurse concludes that it is likely that this depression in the recovery period is relatively common among cardiovascular surgical clients.
client is experiencing: There is nothing in the client’s complaint that suggests body image concerns (option 1),
although most postsurgical clients have this concern to some degree. The statements of this
client instead suggest that the client is experiencing depressed mood, which is similar to the
medical diagnosis of depression. Nothing the client has said indicates that the client is
intolerant of activity (option 2). Feeling tired is a much more moderate problem than being
intolerant of activity. There is no data in the stem of this question to suggest that the client’s
recovery is delayed (option 4).

1.‐ Disturbed body image.


2.‐ Activity intolerance.
3.‐ Depressed mood.
4.‐ Delayed surgical recovery.

4416 The client diagnosed with dysthymia asks the nurse to Correct answer: 4 According to DSM‐IV‐TR, dysthymic disorder cannot be fully diagnosed until the depressed Review DSM‐IV‐TR criteria for dysthymia and major depression.
explain what the diagnosis means. When responding to mood has been present for at least 2 years. Additional criteria are specified to diagnose
the client, the nurse should state that before dysthymic disorder, and once the depressed mood criterion has been met, only some of the
dysthymia can be diagnosed, depressed mood needs other criteria must be met. This is in contrast to the DSM‐IV‐TR criteria for diagnosing major
to be present for at least: depression. In this case, at least 5 of 9 criteria must be met.

1.‐ 2 weeks.
2.‐ 4 weeks.
3.‐ 1 year.
4.‐ 2 years.
4417 The hospitalized client is in the acute stage of mania. Correct answer: 4 The client in the manic state is generally intrusive and insensitive to the needs of others and Develop a mental picture of how the person with elevated affect is likely to act. "See" this
Which of the following is an appropriate goal for the does not recognize boundaries, whether psychologic or physical. The client also tends to have client before you, and the correct answer will be obvious.
nurse and the client to work toward? The client will: an intense preoccupation with sexual urges and frequently touches others or positions self in
socially inappropriate ways. The nurse must encourage the client to set and maintain
boundaries while interacting with others. The person in a manic state is unlikely to be able to
conform to the schedule in option 1 or to sustain attention for this period of time. Additionally,
the client is likely to find the activity room more stimulating than a quieter area of the unit.
Individuals experiencing manic affect tend toward overreaction and overdramatization in any
situation, so participating in a such a group would likely increase the client's manic
hyperactivity and dramatization (option 2). While physical exercising will allow the client to
sublimate some of the excessive energy that is felt (option 3), the client is likely to be
domineering, overbearing, and highly competitive in groups. Further, it is likely that the client's
level of mania will increase because of the extra stimulation.

1.‐ Spend at least 30 minutes per hour watching TV in the activity room.
2.‐ Participate actively in the psychodrama group each day.
3.‐ Lead other clients in group physical exercises each morning.
4.‐ Maintain distance of 2–3 feet at all times when interacting with others.

4418 The family members of a client in an acute state of Correct answer: 1 The nurse must first attend to safety and physiologic needs necessary to sustain life. Develop a mental picture of this client. Notice the perspiration and connect it with the
mania relate that the client has not slept for 4 nights. Considering the prior level of physical activity of the client and that the client is currently family’s report of extreme hyperactivity. Recall basic facts about fluid and electrolyte
They further report that the client climbed up and perspiring profusely, the nurse should recognize that this client could easily enter into a state balance to choose correctly.
down the stairs of a nearby sports stadium for at least of fluid and/or electrolyte imbalance. This at‐risk problem will take priority over actual
6 hours without stopping. The client now has blisters problems that do not involve basic physical needs or safety. Impaired skin integrity (option 3) is
on the feet and is perspiring profusely. When planning incorrect. The nurse should recognize that while this is an actual physical problem, it does not
care for this client, the nurse should give priority to pose the same level of risk that the risk for injury and fluid deficit pose for this client. Like the
which of the following problems? psychological needs, this problem can be addressed after basic physiologic and safety needs
are addressed. Ineffective coping and impaired adjustment (options 2 and 4) are not priority
problems at this time. They are psychologic problems that can be addressed after basic
physiologic and safety needs are addressed.

1.‐ Risk for deficient fluid volume


2.‐ Ineffective coping
3.‐ Impaired skin integrity
4.‐ Impaired adjustment

4419 The nurse assesses that a client in a state of elevated Correct answer: 2 Clients in a manic state are vulnerable to injure self or others, either through restless Recall behaviors common to the client with elevated mood. Recognize that the nurse’s
affect is at risk for self harm. The nurse then places hyperactivity and poor judgment or through actions during unpredictable mood swings. priority is always to provide for the safety of individual clients, as well as groups of clients.
high priority on including which of the following in the Continuous and close nursing supervision of these clients to ensure safety is imperative,
plan of care? regardless of the type of room to which they are assigned. A quiet room near the nurse's
station might be advisable for the client with elevated affect (option 1), but this will not
substitute for constant immediate supervision of the client. Medications (option 3) should be
given orally if possible, since the manic client is likely to overreact to any physical contact,
either interpreting it as a physical assault or ascribing a sexual intent to it. Being in a quiet
nonstimulating private room (option 4) is advisable for this client, but it is not as urgent as
close personal supervision by the nurse.

1.‐ A room that is observable from the nurses' station


2.‐ Constant supervision of the client
3.‐ Administration of all medications intramuscularly rather than orally
4.‐ A quiet, nonstimulating private room for the client
4420 Two years after getting mugged in a parking lot, the Correct answer: 2 Option 2 is correct. Post‐traumatic stress disorder is associated with a traumatic event. Notice the specific nature of the client's fears and sleep experiences. Recognize that post‐
client verbalizes a fear of parking lots and complains of Symptoms such as nightmares and flashbacks are commonly associated with this disorder. traumatic stress disorder (PTSD) and the conditions mentioned in the other options are
recurring "bad dreams" about them. The nurse should Option 1 is incorrect as generalized anxiety disorder is associated with having a great deal of medical, not nursing diagnoses.
assess this behavior as a possible indicator of which difficulty controlling unrealistic, excessive anxiety associated with common daily experiences.
problem? Option 3 is incorrect because social phobias present as fears of social occasions, and the
phobic individual avoids many or most social situations because of dread of being embarrassed
or humiliated. Phobic persons recognize that their behavior is irrational. Option 4 is incorrect
because dysthymic disorder is a long lasting problem of the individual that involves lowered
mood and a variety of other experiences that have lasted over at least a two year period. Sleep
patte

You might also like